You are on page 1of 1577

"SUGGESTED ANSWERS/HINTS"

Chartered Accountancy Examination


(December 2014)

CAP II Examination

Education Department
The Institute of Chartered Accountants of Nepal
Satdobato, Lalitpur, P.O.Box: 5289, Kathmandu, Nepal
Tel: +977-1-5530832, 5530730, Fax: +977-1-5550774
E-mail: ican@ntc.net.np, Website: www.ican.org.np
The suggested answers/hints published herein do not constitute the basis for evaluation of the
students’ answers in the examination. The answerds are prepared by concerned Resource Persons
and complied by the Education Department of the Institute with a view to assist the students in
their education. While due care is taken in preparation of answers, if any errors or omissions are
noted, the same may be brought to attention of the Education Department. The council of the
Institute is not in any way responsible for the correctness or otherwise of the answers published
herewith.

Publisher : The Institute of Chartered Accountants of Nepal


Satdobato, Lalitpur, P.O.Box: 5289, Kathmandu
Tel: 977-1-5530832, 5530730, Fax: 977-1-5550774
E-mail: ican@ntc.net.np, Website: www.ican.org.np

© The Institute of Chartered Accountants of Nepal

Price : NRs. 40/- (Rupees Fourty only)

Published in April 2015

CONTENT Page No.


Paper 1 Advanced Accounting 1-11
Paper 2 Audit & Assurance 12-18
Paper 3 Corporate & Other Law 19-25
Paper 4 Financial Management 26-35
Paper 5 Cost & Management Accounting 36-46
Paper 6 (A) Business Communication 47-50
Paper 6 (B) Marketing 51-54
Paper 7 Income Tax & VAT 55-64

Printed at:
Print and Art Service
Bagbazar, Kathmandu
Tel: 4244419, 4239154
Suggested Answers - Advanced Accounting CAP-II Examination - December 2014

Advanced Accounting

1. X Ltd. and Y Ltd. amalgamate to form a new company XY Ltd. The Financial Position of these two companies on
the date of amalgamation was as under:
Capital and Liabilities X Ltd.(Rs.) Y Ltd.(Rs.) Assets X Ltd.(Rs.) Y Ltd. (Rs.)
Share Capital Goodwill 80,000
Equity Share of Rs. 100 Land and
each 800,000 300,000 Building 450,000 300,000
7% Preference Share of Plant
Rs. 100 each 400,000 300,000 &Machinery 620,000 500,000
Furniture and
5 % Debenture 200,000 - Fittings 60,000 20,000
General Reserve - 100,000 Sundry Debtors 275,000 175,000
Profit and Loss account 431,375 97,175 Stores and Stock 225,000 140,000
Sundry Creditors 100,000 210,000 Cash at Bank 120,000 55,000
Secured Loan - 200,000 Cash in Hand 41,375 17,175
Preliminary
Expense 60,000
Total 1,931,375 1,207,175 Total 1,931,375 1,207,175

The terms of amalgamation are as under:


a) The assumption of liabilities of both companies.
Issue of 5 preference share of Rs.20 each in XY Ltd. @ Rs. 18 paid up at premium of Rs. 4 per share for each
preference share held.Issue of 6 Equity shares of Rs. 20 each in XY Ltd. @ Rs. 18 paid up at a premium of
Rs. 4 per share for each equity share held in both companies .In addition, necessary cash should be paid to
equity shareholders of both companies as is required to adjust the rights of shareholder of both the
Companies in accordance with intrinsic value of the share of both the companies.
Issue of such amount of fully paid 6 % debenture in XY Ltd. as is sufficient to discharge the 5 % debenture in
X Ltd. at a discount of 5% after takeover.

b) The assets and liabilities are to be taken at book values stock and debtors for which provision at 2 % and
2.5% respectively to be made.
The XY Ltd. to issue 15,000 new equity shares of Rs. 20 each Rs. 18 paid up at a premium of Rs. 4 per share
so as to have sufficient working capital.

Required: 20
Prepare Ledger accounts in the books of X Ltd. and Y Ltd. to close their books.
Answer
Books of X Ltd.
Realization Account
Dr Cr.
Particulars Rs. Particulars Rs.
To Goodwill 80,000.00 By 5% debenture 200,000.00
To land and Building 450,000.00 By Sundry Creditors 100,000.00
To Plant and machinery 620,000.00 By XY Ltd (purchase Consideration) 1,560,000.00
To Furniture and Fitting 60,000.00 By Equity Shareholders A/c( Loss) 51,375.00
To sundry debtors 275,000.00
To stores and stock 225,000.00
To cash at bank 120,000.00
To Cash in Hand 41,375.00
To preference shareholder(excess payment) 40,000.00
1,911,375.00 1,911,375.00

Equity Shareholder account


Dr Cr
Particulars Rs. Particulars Rs.
To preliminary Expenses 60,000.00 By share Capital 800,000.00
To Realization Account 51,375.00 By Profit and Loss A/c 431,375.00
To Equity Share in XY Ltd 1,056,000.00
To Cash 64,000.00
1,231,375.00 1,231,375.00

The Institute of Chartered Accountants of Nepal 1


Suggested Answers - Advanced Accounting CAP-II Examination - December 2014

XY Ltd
Dr. Cr.
Particulars Rs. Particulars Rs.
To Realisation A/C 1,560,000.00 By Equity Shares in xy Ltd
For Equity 10,56,000
For Preference 440000 1,496,000.00
By cash 64,000.00
1,560,000.00 1,560,000.00

Books of Y Ltd
Realization Account
Particulars Rs. Particulars Rs.
To land and Building 300,000.00 By Sundry Creditors 210,000.00
To Plant and machinery 500,000.00 By Secured Loan 200,000.00
To Furniture and Fitting 20,000.00 By XY Ltd ( purchase Consideration) 790,000.00
To sundry debtors 175,000.00 By Equity shareholder A/c (loss) 37,175.00
To stores and stock 140,000.00
To cash at bank 55,000.00
To Cash in Hand 17,175.00
To preference shareholder 30,000.00
1,237,175.00 1,237,175.00

XY Ltd Account
Particulars Rs. Particulars Rs.
To Realisation A/C 790,000.00 By Equity Shares in XY Ltd
For Equity 396000
For Preference 330000 726,000.00
By cash 64,000.00
790,000.00 790,000.00

Equity Shareholder Account


Particulars Rs. Particulars Rs.
To Equity Share in XY Ltd 396,000.00 By Share Capital 300,000.00
To Realisaation 37,175.00 By profit and Loss A/C 97,175.00
To cash 64,000.00 By Reserve 100,000.00
497,175.00 497,175.00

Working Note 1
Purchase consideration X Ltd (Rs.) Y Ltd (Rs.)
Payable to preference shareholder
Preference share @ 22 per share 440,000.00 330,000.00
Equity Shares at Rs 22 per share 1,056,000.00 396,000.00
cash ( WN 2 ) 64,000.00 64,000.00
1,560,000.00 790,000.00

The Institute of Chartered Accountants of Nepal 2


Suggested Answers - Advanced Accounting CAP-II Examination - December 2014

Working Note 2
Value of Net Assets X Ltd Y Ltd
Goodwill 80,000.00
land and Building 450,000.00 300,000.00
Plant and machinery 620,000.00 500,000.00
Furniture and Fittings 60,000.00 20,000.00
Debtor Less 2.5% 268,125.00 170,625.00
Stock less 2 % 220,500.00 137,200.00
Cash at Bank 120,000.00 55,000.00
Cash in Hand 41,375.00 17,175.00
1,860,000.00 1,200,000.00
Less
Debenture 200,000.00 -
Creditors 100,000.00 210,000.00
Secured Loan - 200,000.00
1,560,000.00 790,000.00
Payable in Shares 1,496,000.00 726,000.00
Payable in cash 64,000.00 64,000.00

2.
a) Gold, Silver and Diamond are partners in a firm sharing profits and losses in the ratio of 5:3:2. Gold retires on
32.03.2071 on which date the balance sheet of the firm stood as under:

Liabilities Amount (Rs.) Assets Amount (Rs.)


Capital Accounts: Goodwill 18,000
Gold 190,000 Furniture 45,000
Silver 120,000 Office Equipments 138,000

Diamond 100,000 410,000 Building 202,000


General Reserve 68,000 Stock 176,000
Sundry Creditors 182,000 Sundry Debtors 78,000
Bills Payable 14,000 Cash 17,000

674,000 674,000

It is provided in the deed of the partnership that in the event of the death or retirement of a partner, goodwill is to
be valued at 2 years’ purchase of the Average Profit of the last 4 years. The profits for the last 4 years are:

(Rs.)
• 2070-71: 46,000
• 2069-70: 40,000
• 2068-69: 10,000
• 2067-68: (28,000)

Furniture and Building are revalued at Rs. 40,000 and Rs. 225,000 respectively, stock is overvalued by 10%. It
was decided that Gold should be paid Rs. 140,000 immediately on retirement and balance on his Capital account
is to be treated as a loan to the firm. Silver and Diamond contributed necessary sums in equal proportions for
payment to Gold and to leave Rs. 33,000 cash as working capital.

Assuming that above mentioned arrangements is given effect to show the Revaluation Account, Cash Account
and Capital Account of the partners. 10

The Institute of Chartered Accountants of Nepal 3


Suggested Answers - Advanced Accounting CAP-II Examination - December 2014

b) A Kathmandu based Shop opens a new branch in Biratnagar which trades independently of the head office. The
transactions at the branch for the year ended 31st March 2013 are as under:

(Rs.)
Goods supplied by head office 200,000
Purchases from outsiders:-
Credit 155,500
Cash 30,000 185,500
Sales:
Credit 250,500
Cash 46,000 296,500
Cash received from customers 304,500
Cash paid to creditors 142,500
Expenses paid to creditors 89,500
Furniture purchased by the branch on credit 35,000
Cash received from head office initially 40,000
Remittances to head office 110,000

Prepare the branch final accounts after taking the following into consideration: 10
i) Write off depreciation on furniture at 5% per annum for full year.
ii) A remittance of Rs. 20,000 from the branch to the head office is in transit.
iii) The values its closing stock at Rs. 120,000

Answer a)
In the Books of the Firm
Revaluation A/c
Dr. Cr.
Particulars Amount (Rs.) Particulars Amount (Rs.)
To Furniture A/c 5,000.00 By Building A/c 23,000.00
To Stock A/c 16,000.00
To Partners Capital A/c:
Gold 1,000.00
Silver 600.00
Diamond 400.00 2,000.00
23,000.00 23,000.00

Cash A/c
Cr.
Particulars Amount (Rs.) Particulars Amount (Rs.)
To Balance b/d 17,000.00 By Gold Capital A/c 140,000.00
To Partners Capital A/c:
(Bal Figure)
Silver 78,000.00
Diamond 78,000.00 156,000.00 By Balance c/d 33,000.00
173,000.00 173,000.00

Partners Capital A/c


Dr. Cr.
Diamond Diamond
Particulars Gold (Rs) Silver (Rs.) (Rs.) Particulars Gold (Rs.) Silver (Rs.) (Rs.)
To Bank A/c 140,000.00 - - By Balance b/d 190,000.00 120,000.00 100,000.00
To Gold Loan By General Reserve
A/c 93,000.00 - - A/c 34,000.00 20,400.00 13,600.00
To Goodwill
A/c - 20,400.00 13,600.00 By Goodwill A/c 8,000.00 4,800.00 3,200.00
By Revaluation A/c 1,000.00 600.00 400.00
To Balance c/d - 203,400.00 181,600.00 By Cash A/c 78,000.00 78,000.00

233,000.00 223,800.00 195,200.00 233,000.00 223,800.00 195,200.00

The Institute of Chartered Accountants of Nepal 4


Suggested Answers - Advanced Accounting CAP-II Examination - December 2014

Working Notes:
1) Calculation of Goodwill and its Treatment
2067-68 (28,000.00)
2068-69 10,000.00
2069-70 40,000.00
2070-71 46,000.00
Total 68,000.00
Average Profit of the last 4 years 17,000.00
Goodwill 2 years Average Profit 34,000.00

Goodwill already in the books of accounts 18,000.00


Additional Goodwill 16,000.00

Goodwill Account has already been raised by Rs.18,000 therefore before Gold’s retirement it should be raised by Rs.
16,000(34,000-18,000) in the current profit sharing ratio 5:3:2.
Gold 5/10 X 16,000=8,000
Silver 3/10 X 16,000=4,800
Diamond 2/10 X 16,000=3,200
Total 16,000

Therefore Goodwill is to be written off in the new profit sharing Ratio 3:2
Silver 3/5 X 34,000=20,400
Diamond 2/5 X 16,000 =13,600
Total 34,000

Profit Sharing Ratio


Old New
Gold 5 -
Silver 3 3
Diamond 2 2
Total 10 5

2) Actual Value of Stock


Value of Stock =Rs. 176,000 x 100 = Rs. 160000
110
Therefore it is Overvalued By (176,000-160,000) = Rs. 16,000

b) In Branch Book
Trading and Profit & Loss Account for the year ended on 31st March 2013
Dr. Cr.
Particulars Amount Rs. Particulars Amount Rs.
To Goods supplied by head office 200,000 By Sales:
To Purchases: Credit 250,500
Credit 155,000 Cash 46,000 296,500
Cash 30,000 185,500 By Closing Stock 120,000
To Gross Profit c/d 31,000
Total 416,500 Total 416,500
To Expenses 89,500 BY Gross Profit b/d 31,000
To Depreciation on Furniture 1,750 BY Head Office- transfer of net loss 60,250
Total 91,250 Total 91,250

The Institute of Chartered Accountants of Nepal 5


Suggested Answers - Advanced Accounting CAP-II Examination - December 2014

Balance Sheet as on 31st March 2013


Liabilities Amount Assets Amount Rs
Rs
Sundry Trade Creditors Closing Stock 120,000
(155,500-142,500) 13,000 Cash in Transit 20,000
Creditor for Furniture 35,000 Cash in hand 18,500
Advances from Trade Debtors
(304,500-250,500) 54,000
Head Office Account 56,500
Total 158,500 Total 158,500

Cash Book
Liabilities Amount Assets Amount Rs
Rs
To Head Office (Initial Receipt) 40,000 By Cash Purchases 30,000
To Cash Sales 46,000 By Trade Creditors 142,500
To Trade Debtors 304,500 By Expenses 89,500
By Head Office Remittances 110,000
By Balance c/d 18,500

Total 390,500 Total 390,500

3.
a) An electricity company decided to replace some parts of its plant by an improved plant. The plant to be replaced
was built in 2043 for Rs. 4,200,000. It is estimated that it would cost Rs. 7,800,000 to build a new plant of the
same size and capacity.The cost of the new plant as per the improved design was Rs. 12,600,000 and in addition,
material belonging to the old plant valued at Rs. 456,000 was used in the construction of the new plant. The
balance of the plant was sold for Rs. 360,000.
Compute the amount to be written off to revenue and the amount to be capitalized. Also prepare Plant Account
and Replacement Account. 7

b) Explain ‘Non Performing Assets’. 3


th st
c) A Ltd. prepares accounts on 30 September each year, but on 31 December 2013 fire destroyed the greater part
of its stock .Following information was collected from books of account:

Stock as 1.10 .2013 Rs. 29,700,000


Purchase from 1.10.2013 to 31.12.2013 Rs.75,000,000
Wages from 1.10.2013 to 31.12.2013 Rs.33,000,000
Sales from 1.10.2013 to 31.12.2013 Rs.140,000,000

The rate of gross profit is 33.33% on cost .Stock to the value of Rs.3,000,000 was salvaged .Insurance policy was
for Rs.25,000,000.
Additional Information:
i) Stock in the beginning was calculated at 10% less than cost.
ii) A plant was installed by firm’s own workers.They were paid Rs. 500,000 which was included in the wages.
iii) Purchase includes the purchase of plant of Rs. 5,000,000.
You are required to calculate the claim for the loss of stock. 7
d) Write short note about unexpired risk reserve in Insurance Company 3

The Institute of Chartered Accountants of Nepal 6


Suggested Answers - Advanced Accounting CAP-II Examination - December 2014

Answer
a)
(i) Calculation of Amount Chargeable to Revenue
Rs. Rs.
Estimated Current cost of Replacing old plant 7,800,000.00
Less: Value of replacing old Plant 360,000.00

Value of material belonging to the old plant


used in the construction of new plant 456,000.00 816,000.00
Total 6,984,000.00

(ii) Calculation of Amount to be Capitalized


Rs. Rs.
Cost of Building new plant 12,600,000.00

Add: Value of Material belonging to the old


plant used in the Construction of the new Plant 456,000.00 13,056,000.00
Less: Estimate Current Cost of Replacing old Plant 7,800,000.00
Total 5,256,000.00

(iii)
Plant Account
Rs. Rs.
To Balance b/d 4,200,000.00 By Balance c/d 9,456,000.00

To Cost of Construction 4,800,000.00


To Cost of Old Material 456,000.00
9,456,000.00
9,456,000.00

(iv)Replacement Account

To Bank A/c By Bank A/c 360,000.00


(portion to be written out off By Plant A/c 456,000.00
replacement cost) 7,800,000.00 By Revenue A/c 6,984,000.00
(Bal Fig)
7,800,000.00 7,800,000.00

b) NPA is a classification used by financial institutions that refer to loans that are in jeopardy of default. Once the borrower has
failed to make interest or principle payments for 90 days the loan is considered to be a non-performing asset. Non-performing
assets are problematic for financial institutions since they depend on interest payments for income. Troublesome pressure from
the economy can lead to a sharp increase in non-performing loans and often results in massive write-downs.

The nonperforming asset is not yielding any income to the lender in the form of principal and interest payments.
c)
Computation of claim for loss of stock
Stock on the date of fire (31.12.2013) Rs. 30,500,000
Less salvage stock Rs. 3,000,000
Loss of Stock Rs.27,500,000
Amount of claim = insured value/cost of stock on date of fire *loss of stock
= 25,000,000/30,500,000*27,500,000
= 22,541,000

The Institute of Chartered Accountants of Nepal 7


Suggested Answers - Advanced Accounting CAP-II Examination - December 2014

Working Note
Memorandum Trading Account for period from 1.10.2013 to 31.12.2013
Particulars Rs. Particulars Rs.
To opening stock(29700000*100/90) 33,000,000 By sales 140,000,000
To Purchase 75,000,000 By Closing Stock(balancing figure) 30,500,000
Less cost of plant 5,000,000 70,000,000
To wages 33,000,000 32,500,000
Less wages for plant 500,000
To gross profit (33.33% of cost or 25% of sale) 35,000,000
Total 170,500,000 Total 170,500,000

d) As per Rule 15 of the Insurance Regulation 2049, every insurance Business operating,Non Life Insurance Business shall transfer an
amount not less than fifty percent of the Net Insurance Premium shown in Revenue Account to the ‘Unexpired Risk Reserve
account.Such amount shall be allocated for every category of Insurance the Insures operating eg. an insures operating non life
business and accepting risk for Fire Insurance,Marine Insurance Motor insurance and aviation insurance, then the insurer shall
maintain the Unexpired Risk Reserve for each of the fire,marine, motor and aviation insurance.

Such Unexpired Risk reserve shall be recognized as income in the next year except the Unexpired Risk reserve maintained for
Marine Insurance. In case of marine insurance, unexpired risk reserve maintained for it shall not be recognized as income for at
least three year.
4.
a) The Management of KAS Ltd. appointed a new young sales manger towards the end of Ashadh 2070. The new
manager devised a plan to increase sales and profit by reducing selling price and extending credit period to
customers.

The extracts of the operational result are enlisted here below:


2069-70 2070-71
Particulars (Amount in (Rs.) Lakhs) (Amount in (Rs.) Lakhs)
Sales 1,200 1,800
Gross Profit 150 162
Net Profit 60 63
Equity Share Capital 600 800

Explain whether you believe that the profitability for the year ended 2071 has improved as a result of the new
policies adopted by the new manager. You should support your answer with appropriate ratios. 5

b) Differentiate between Government Accounting and Business Accounting. 5

c) From the following information, calculate the value of an equity share: 5


i) The subscribed share capital of a company consists of 10 lakhs 13% preference shares of Rs. 10 each and
Rs. 20 lakhs equity shares of Rs. 10 each. All the shares are fully paid up.
ii) The average annual profits of the company after providing depreciation but before taxation are Rs.
18,000,000. It is considered necessary to transfer Rs. 3,450,000 to general reserve before declaring any
dividend. Rate of taxation is 30%.
iii) The normal return expected by investors on equity shares from the type of business carried on by the
company is 20%.
iv) Ignore corporate dividend Tax.

Answer
a) Calculation of Ratios:

Ratios Formula 2069-70 2070-71


Gross Profit Margin
=Gross Profit / Sales X 100% 12.50% 9.00%
Net Profit Margin
=Net Profit / Sales X 100% 5.00% 3.50%
Return on Equity
=Net Profit / Equity X 100% 10.00% 7.88%

• With the extracts it could be seen that there has been a rocket increase in the sales which has marked a growth of 50%
• However, the Net Profit has marginally increased from Rs. 60 less to Rs. 63 Lacs in absolute terms.
• The company’s Gross Profit Ratio has been reduced from 12.50% to 9.00%.

The Institute of Chartered Accountants of Nepal 8


Suggested Answers - Advanced Accounting CAP-II Examination - December 2014

• The company’s Net Profit Ratio has been reduced from 5.00% to 3.50%; indicating hard time for share holders in future.
• The company’s Return on Equity has reduced from 10% to 7.88%.
• The new credit policy of the Management may force the company to hold huge stocks as well as the figures of the debtors
could be increased, likely resulting to a higher risk of Bad Debts.

Conclusion
The company should review the new Sales Policy so that profitability in relative terms can also be increased.

b)
1. Meaning
The accounting system maintained by the government offices is known as government accounting.
The accounting system maintained by business organizations is known as business accounting.

2. Objective
Government accounting is maintained by the government offices to know the position of public fund.
Business accounting is maintained by business organizations to know the profit or loss and the financial position of the business.

3. Budget
Government accounting strictly follows the government budgeting system.
Business accounting does not follow the government budgeting system.

4. Basis
Government accounting is prepared on cash basis.
Business accounting is prepared on cash as well as accrual basis.

5. Level of Accounting
Government accounting has the system of central level and operating level accounting.
Business accounting has no provision of central level and operating level accounting.

6. Rules and Provisions:


Government accounting is strictly maintained by following the financial rules and provisions of government.
Business accounting is maintained by following the rules and principles of 'Generally Accepted Accounting Principles'.

7. Information:
Government accounting provides information to the government about the receipts, transfer and deposition of public funds.
Business accounting provides information to the concerned parties about the operating result and financial position of the
business.

c)
Rs.
Average Annual profits before tax 18,000,000
Less: Income Tax @ 30% 5,400,000
12,600,000
Less: Transfer to general reserve 3,450,000
Amount available for dividend 9,150,000
Less: Preference dividend @13% on Rs. 10,000,000 1,300,000
Amount available for equity dividend 7,850,000

Rate of Dividend = 7,850,000/20,000,000 x100 = 39.25%


Normal Rate of Dividend = 20%
Value of an equity share = 39.25 / 20 x 10 = 19.63

5.
a) Prudent prepares financial statement to 31st December each Year, on 1st January 20x0 the entity purchased a non
current asset for Rs 1.6 million that had an anticipated useful life of four Years .This asset qualified for
immediate tax relief of 100% of the cost of that asset.

For the year ended 31st December 20x0; the draft account showed a profit before tax of Rs. 2 million. The
directors anticipate that this level of profit will be maintained for the foreseeable future.

The Institute of Chartered Accountants of Nepal 9


Suggested Answers - Advanced Accounting CAP-II Examination - December 2014

Prudent pays tax at a rate of 30%.Apart from the difference caused by the purchase of no current asset in 20x0,
there are no other difference between accounting profit and taxable profit or the tax base and carrying amount of
net asset. Current year tax for the year is Rs 240,000 and for the years 20x1 to 20x3 are Rs. 720,000 for each
year.

You are required to compute the pre, and post tax profits for the Prudent for each of the four Years ending 31st
December 20x0to 20x3 assuming deferred tax is recognized. 5
b) Discuss the treatment of capitalization of borrowing cost as per NAS 08 5
th
c) R has developed a software program during the year to 30 November 2014.The cost of developing the software
was Rs. 50 million.The software is used by the rest of the group and sold to third parties.Net revenue of Rs. 40
million is expected from sale of the software .Which has quickly become market leader in this field. The software
is expected to generate revenue for four years, after which an upgraded version will be developed

Should R recognize the internally developed software as an Intangible asset? 5

Answer a)
Particulars 20x0 (Rs.) 20x1 (Rs.) 20x2 (Rs.) 20x3 (Rs.)
Carrying Amount 1,200.00 800.00 400.00 -
Tax Base - - - -
Temporary difference 1,200.00 800.00 400.00 -

Closing deferred tax


Liability@30% 360.00 240.00 120.00 -

Opening deferred tax Liability - (360.00) (240.00) (120.00)


Charge/(credit) to income 360.00 (120.00) (120.00) (120.00)

Income statement 20x0 (Rs.) 20x1 (Rs.) 20x2 (Rs.) 20x3 (Rs.)
Profit Before tax 2,000.00 2,000.00 2,000.00 2,000.00
Current Tax (240.00) (720.00) (720.00) (720.00)
Deferred Tax (360.00) 120.00 120.00 120.00
Profit after Tax 1,400.00 1,400.00 1,400.00 1,400.00

b)

Borrowing costs that are directly attributable to the acquisition, construction or production of a qualifying asset shall be capitalized
as part of the cost of asset. Borrowing cost are interest and other cost incurred by an entity in connection with the borrowing of
funds.
A qualifying asset is an asset that necessarily takes a substantial period of time to get ready for its intended use or sale.
Commencement of capitalization
The capitalization of borrowing cost as part of qualifying asset shall commence when:
Expenditures for the asset are being incurred;Borrowing costs are being incurred; and
Activities that are necessary to prepare the asset for its intended use or sale are in progress.
Suspension of capitalization
Capitalization of borrowing cost shall be suspended during extended periods in which active development is interrupted.
Cessation of capitalization
Capitalization of borrowing cost shall cease when substantially all the activities necessary to prepare qualifying asset for its
intended use or sale are complete.

c)

NAS 27 prohibits recognizing Internally Generated Goodwill.NAS 27 also requires internally generated intangibles to be
recognized, provided they meet certain criteria. An entity must demonstrate the technical feasibility of the asset, the ability to
complete and use or sell of the asset, the probable future economic benefits of the asset and the availability of adequate technical,

The Institute of Chartered Accountants of Nepal 10


Suggested Answers - Advanced Accounting CAP-II Examination - December 2014

financial and other resource before it can be recognized. It must also be possible to measure the expenditure attributable the asset
reliably and it should be capable of generating cash inflow in excess of cash outflows.
Because the software is generating external revenue, these criteria appear to be met. But only the cost attributable to the
development of software after the development shall qualify for recognition and the amount so recognized shall be amortized over
useful life of 4 years.

6. Write short notes on: (4×2.5=10)

a) Sweat share
b) Firm underwriting
c) Related party transaction
d) Off balance sheet item
Answer
a) Sweat shares means the equity shares issued by company to its employees or directors at a discount or for a consideration other than
cash for providing technical know-how or making available rights in the nature of property rights or value additions.

Such issue must be authorized by a special resolution passed by the company in a general meeting.
The resolution must specify the category of employees etc, to whom shares are to be issued,number of shares, price, etc.

b) ‘Firm underwriting’ signifies a definite commitment to take up a specified number of shares irrespective of the number of shares
subscribed by the public. In such a case, unless it has been otherwise agreed, the underwriter’s liability is determined without
taking into account the number of shares taken up ‘firm’ by him, i.e. the underwriter is obliged to take up:
1. the number of shares he has applied for ‘firm’; and
2. the number of shares he is obliged to take up on the basis of the underwriting agreement.

c) A business deal or arrangement between two parties who are joined by a special relationship prior to the deal. For example, a
business transaction between a major shareholder and the corporation would be deemed a related-party transaction. As per the
Provision of Nepal Accounting Standards, any such transaction should be properly disclosed in the Financial Statement of the
Organization.

d) An asset or debt that does not appear on a company’s balance sheet. Items that are considered off balance sheet are generally ones in
which the company does not have legal claim or responsibility.

For example, loans issued by a bank are typically kept on the bank's books. If those loans are securitized and sold off as
investments, however, the securitized debt is not kept on the bank's books. One of the most common off-balance sheet items is an
operating lease.

The Institute of Chartered Accountants of Nepal 11


Suggested Answers - Audit & Assurance CAP-II Examination - December 2015

Audit & Assurance

1. As an auditor, give your opinions with reasons on the following cases: (4×5=20)

a) Aashriya& Associates has two Partners holding Certificate of practice. It has 14 Public company clients and other small
clients. The firm is in dilemma as to accept the forthcoming audit request of two Public companies.
b) Smart Pvt. Limited sold a Television worth of Rs. 500,000 (exclusive of all taxes) on 32 Ashadh 2071 which includes Rs.
100,000 for servicing fees of the Television for 5 years from the date of sales. The servicing fee is estimated equal
amount for each guaranteed service years. Smart Pvt. Limited booked entire Rs. 500,000 as revenue for financial year
2070/71.
c) A company purchased a plant for Rs. 20 crores on 1 Shrawan 2070. The company has the policy to charge depreciation
at the rate of 10% on such plants on straight line basis. Due to long dispute between the management and the labour of
the company, the factory was closed from Kartik 1, 2070 to Chaitra end 2070. Management has charged depreciation of
Rs. 1 crore on the said plant to the income statement for the year because the newly purchased plant was not used for 6
months in the year.

d) XYZ is a manufacturing company. There was huge fire in the factory of XYZ on 1 Ashoj 2071 and fixed assets of
written down value Rs. 10 crores was lost out of total fixed assets of Rs. 20 crores of the company. The financial
statements of the company for the year 2070/71 was approved by the Board on 30 Ashoj 2071 in which fixed assets have
been presented at WDV of Rs. 20 crores despite the severe loss due to fire and the information about the loss due to fire
is properly explained in the Notes to the financial statements.

Answer

a) The Directive issued by ICAN on Ceiling over the number of audit requires that a member holding COP can audit the books of
accounts of a maximum100 clients only, in a financial year. Out of these 100 clients, number ofPublic Companies shall not exceed
15.The above limit is applicable for each member of a partnership firm.Provided, organizations whose annual turnover is less than
NRs. 2lakhs, such as small Cooperatives, Religious organizations, SocialOrganizations, Consumer Group, Different Committees,
Trade Unions, Professional Associations and other entities of similar nature are notincluded while calculating the above limit.Thus,
in view of above requirement, new audit can be accommodated subject to total limit for two partners.

b) As per NAS 7 "Revenue" when the selling price of a product includes an identifiable amount for subsequent servicing (for
example, after sales support and product enhancement on the sale of software), that amount is deferred and recognized as revenue
over the period during which the service is performed. The amount deferred is that which will cover the expected costs of the
services under the agreement, together with a reasonable profit on those services.Accordingly in the light of aforesaid provision of
NAS 7; the accounting treatment made by Smart Pvt. Limited is not correct. Smart should book Rs. 400,000 as revenue for FY
2070/71 and Rs. 100,000 should be deferred. For each coming year Rs. 20,000 should be recognized as revenue to match its
services cost.

c) Depreciation of an asset begins when it is available for use, i.e. when it is in the location and condition necessary for it to be
capable of operating in the manner intended by management. Depreciation of an asset ceases at the earlier of the date that the asset
is classified as held for sale (or included in a disposal group that is classified as held for sale) in accordance with NAS 06 and the
date that the asset is derecognized. Therefore, depreciation does not cease when the asset becomes idle or is retired from active use
unless the asset is fully depreciated. So, in the given case, depreciation expenses to be charged to the income statement for the year
2070/71 should be Rs 2 crores (i.e. for full year) instead of Rs 1 crore.

d) As per NAS-05, Event After the Balance Sheet Date, events after the reporting period are those events, favorable and unfavorable,
thatoccur between the end of the reporting period and the date when the financial statements are authorized for issue. Two types of
events can be identified:

i. those that provide evidence of conditions that existed at the end of the reporting period (adjusting events after the
reporting period); and
ii. those that are indicative of conditions that arose after the reporting period (non-adjusting events after the reporting period)

So, the event in the given case (fire after reporting period) is a non-adjusting event.An entity shall not adjust the amounts
recognized in its financial statements to reflect non-adjusting events after the reporting period.

If events after the balance sheet date impacts going concern status of the entity, the entity is required to prepare its financial
statement on break-up value basis. This does not seem to be the case here.

The Institute of Chartered Accountants of Nepal 12


Suggested Answers - Audit & Assurance CAP-II Examination - December 2015

If non-adjusting events after the reporting period are material, non-disclosure could influence the economic decisions that users
make on the basis of the financial statements. Accordingly, an entity shall disclose the following for each material category of non-
adjusting event after the reporting period:

i. the nature of the event; and


ii. an estimate of its financial effect, or a statement that such an estimate cannot be made.

So, presenting fixed assets at Rs 20crores in the balance sheet with appropriate disclosure in the Notes to Accounts seems to be
appropriate.

2. Answer the following: (3×5=15)

a) Explain ‘An appropriate subject matter’ as an element of assurance.


b) Explain ‘Audit Sampling’ and ‘Sampling Risk’ in the light of NSA - 530 Audit Sampling and Other Selective Testing
Procedures.
c) Based on study and evaluation of internal control system, auditor concludes that internal controls are well designed
and functioning effectively. Still auditor performs appropriate substantive audit procedures to form an opinion on the
financial statements before issuance of audit report. Explain why auditor has to perform appropriate substantive audit
procedures in such an ideal situation also.

Answer

a) Assurance engagement is an engagement in which a practitioner expresses a conclusion designed to enhance the degree of
confidence of the intended users other than the responsible party about the outcome of the evaluation or measurement of a subject
matter against criteria.

The subject matter, and subject matter information, of an assurance engagement can take many forms, such as:

• Financial performance or conditions (for example, historical or prospective financial position, financial performance and
cash flows) for which the subject matter information may be the recognition, measurement, presentation and disclosure
represented in financial statements.
• Non-financial performance or conditions (for example, performance of an entity) for which the subject matter information
may be key indicators of efficiency and effectiveness.
• Physical characteristics (for example, capacity of a facility) for which the subject matter information may be a
specifications document.
• Systems and processes (for example, an entity’s internal control or IT system) for which the subject matter information may
be an assertion about effectiveness.
• Behavior (for example, corporate governance, compliance with regulation, human resource practices) for which the subject
matter information may be a statement of compliance or a statement of effectiveness.

Subject matters have different characteristics, including the degree to which information about them is qualitative versus
quantitative, subjective versus objective, historical versus prospective, and relates to a point in time or covers a period. Such
characteristics affect the:

(a) Precision with which the subject matter can be evaluated or measured against criteria; and
(b) The persuasiveness of available evidence.
(c)
The assurance report notes characteristics of particular relevance to the intended users.
An appropriate subject matter is:

(a) Identifiable, and capable of consistent evaluation or measurement against the identified criteria; and
(b) Such that the information about it can be subjected to procedures for gathering sufficient appropriate evidence to
support a reasonable assurance or limited assurance conclusion, as appropriate.

b) Audit sampling involves the application of audit procedures to less than 100% of items within an account balance or class of
transactions such that all sampling units have a chance of selection. This will enable the auditor to obtain and evaluate audit
evidence about some characteristic of the items selected in order to form or assist in forming a conclusion concerning the
population from which the sample is drawn. Audit sampling can use either a statistical or a non-statistical approach.

The Institute of Chartered Accountants of Nepal 13


Suggested Answers - Audit & Assurance CAP-II Examination - December 2015

Sampling risk arises from the possibility that the auditor's conclusion, based on a sample may be different from the conclusion
reached if the entire population were subjected to the same audit procedure. There are two types of sampling risk:

i. the risk the auditor will conclude, in the case of a test of control, that control risk is lower than it actually is, or in the
case of a substantive test, that a material error does not exist when in fact it does. This type of risk affects audit
effectiveness and is more likely to lead to an inappropriate audit opinion; and
ii. the risk the auditor will conclude, in the case of a test of control, that control risk is higher than it actually is, or in the
case of a substantive test, that a material error exists when in fact it does not. This type of risk affects audit efficiency
as it would usually lead to additional work to establish that initial conclusions were incorrect.

c) An internal control system can provide only reasonable assurance that the management’s objectives in establishing the system are
achieved. This is due to the fact that internal control system has the following inherent limitations:

i) Controls have to be cost effective. So, some controls may not be instituted because they are not cost effective.
ii) Most controls are directed at transactions of usual or routine nature. Therefore transactions of unusual nature may
escape from ambit of internal control.
iii) The potential of human error remains in any system of control.
iv) Controls may not prevent frauds through collusion between two or more persons.
v) Management may override controls.
vi) Controls may not keep pace with changing circumstances.
vii) Management itself may manipulate transactions or accounting estimates.

Above inherent limitations of internal control system makes it necessary for the auditor to perform substantive procedures to
express an appropriate opinion on the financial statements. But if the "Test of Control" conducted by the auditor concludes that
internal control system is in existence, operated throughout the period and functioning effectively, the auditor may not consider
necessary to perform substantive audit procedure for that particular area. Substantive procedures though cannot be fully eliminated
and the auditor determines the area where substantive procedures should be used.

3. Give your comments on the following: (3×5=15)

a) Chitale Limited has declared dividend of 12% for FY 2069/70 in the month of Ashoj 2070. The company has booked
this as Interim Dividend in Financial Statement of 2069/70.
b) “Responsibility for properly determining the quantity and value of inventories rests with the management of the
entity”.
c) Ramu& Co. had conducted the audit of XYZ Limited for the financial years 2067/68, 2068/69 and 2069/70. CA. Raman
Saha, partner of Ramu& Co, left the firm on 1 Baishakh 2069. CA. Raman Saha has been appointed as the auditor of
XYZ limited for the year 2070/71.

Answer

a) Dividend has been declared at year end so this should be final dividend.

Further, Nepal Accounting Standards 05 on Events after the Balance Sheet Date provides that if an entity declares dividends to
holders of equity instruments after the balance sheet date, the entity shall not recognise those dividends as a liability at the balance
sheet date.

If dividends are declared after the balance sheet date but before the financial statements are authorised for issue, the dividends are
not recognised as a liability at the balance sheet date because they do not meet the criteria of a present obligation. Such dividends
are disclosed in the notes in accordance with NAS 01 Presentation of Financial Statements.

Thus, the treatment followed by Chitale limited is not in line with NAS.

b) As per NSA 200; "Objectives and General Principles Governing an Audit of Financial Statements" the responsibility for preparing
and presenting the financial statements is that of the management of the entity. Accordingly the responsibility for properly
determining the quantity and value of inventories rests with the management of theentity. Therefore, it is the responsibility of the
management of the entity to ensure that inventories included in the financial information are physically in existence and
representall owned by the entity.

The management can satisfy this responsibility by carrying out appropriate procedures such as verification of all items of inventory
at least once in every financial year. The auditor is expected to examine the adequacy of the methods and procedures of physical

The Institute of Chartered Accountants of Nepal 14


Suggested Answers - Audit & Assurance CAP-II Examination - December 2015

verification followed by the entity. He is also required to determine whether the procedures for identifying defective, damaged,
obsolete, excess and slow-moving items are well-designed and operate properly.

This responsibility of the management is not reduced even where the auditor attends any physical count of inventories in order to
obtain audit evidence. The entities usually maintain detailed stock records in the form of Stores/Stock ledgers showing in respect
of each major item the receipts, issues and balances. The extent of examination of these records by an auditor with reference to the
relevant basic documents (e.g., goods received notes, inspection reports, material issue notes, bin cards, etc.) depends upon the
facts and circumstances of each case. In valuation aspects, compliance with NAS 4 should also be ensured.

c) As per section 111(3) of the Companies Act, no auditor or his/her partner or ex-partner or employee or ex-employee shall be
appointed as auditor for more than three consecutive terms to perform the audit of a public company. Provided, however, that this
restriction shall not apply to any partner who ended partnership or any employee who left the service of such auditor three years
before.

In the present case, Ramu& Co has been auditor of XYZ Ltd for three consecutive years till 2069/70. Since Raman Saha is the ex-
partner of Ramu& Co and 3 years has not elapsed from the date of ending his partnership with Ramu& Co (as date of separation
being 1 Baisakh 2069), his appointment as auditor of the company for the year 2070/71 cannot be considered as lawful.

4. Answer the following: (3×5=15)


a) What are the principal contents of Terms of Audit Engagement as per NSA 210?
b) ABC Hydropower Pvt. Ltd. has purchased equipment worth of Rs. 4 million which is kept stand by for urgent usage on
need basis for repairing the heavy equipment as and when default is reported in functioning of heavy equipment. The
accountant has treated it as recurring inventory item and charged to profit and loss account at the end of each financial
year based on consumption pattern calculated on reasonable basis. Is the accounting treatment made by accountant is
correct? Comment.
c) Write down any five audit steps for the audit of Debtors.

Answer

a) The form and content of audit engagement letters may vary for each client, but they would generally include reference to:

• The objective of the audit of financial statements.


• Management's responsibility for the financial statements.
• The scope of the audit, including reference to applicable legislation, regulations or pronouncements of the Institute of
Chartered Accountants of Nepal (ICAN).
• The form of any reports or other communication of results of the engagement
• The fact that because of the test nature and other inherent limitations of an audit, together with the inherent limitations of any
accounting and internal control system, there is an unavoidable risk that even some material misstatement may remain
undiscovered.
• Unrestricted access to whatever records, documentation and other information requested in connection with the audit.
• Management's responsibility for establishing and maintaining effective internal control.

Other matters that may be included are:


• Description of the basis for fees.
• Ownership and accessibility of the auditor’s files to external parties.
• Interactions with specialists, internal auditors, and the predecessor auditor needed to conduct the audit
• Restrictions to the auditor's liability.

b) As per NAS 6- Property, Plant and Equipment, spare parts and servicing equipment are usually carried as inventory and recognized
in profit or loss as consumed. However, major spare parts and stand-by equipment qualify as property, plant and equipment when
an entity expects to use them during more than one period. Similarly, if the spare parts and servicing equipment can be used only in
connection with an item of property, plant and equipment, they are accounted for as property, plant and equipment.

In view of the above provision made in NAS 6, the equipment purchased by ABC Hydropower Pvt. Ltd. should be treated as
property, plant and equipment. Though it is servicing equipment used on repairing heavy equipment; it has to be kept stand-by and
can be used only in connection with heavy equipment and usable for more than one accounting period, it should be treated as
property, plant and equipment instead of treating it as inventory.

The Institute of Chartered Accountants of Nepal 15


Suggested Answers - Audit & Assurance CAP-II Examination - December 2015

c) Five audit steps for the audit of Debtors out of various possible steps are presented below:

i. Verify the opening balances with last year closing audited balances
ii. Obtain a schedule of receivables and determine whether the total agrees withthe trial balance.
iii. Obtain and consider explanations of significant variations in Debtor’s turnover ratio of current period and prior period and
anticipated debtor turnover ratio.
iv. Select a sample of debtors and obtain direct confirmation from the parties concerned.
v. Obtain an aged analysis of the debtors. Inquire about the reason for unusually large accounts, credit balances on accounts
or any other unusual balances and inquire about the collectibility of receivables.

5. Comment on the following situations/statements: (3×5=15)


a) Explain the types of threat as explained in ICAN Code of ethics.
b) "The rendering of two or more types of professional services concurrently does not by itself impair integrity, objectivity
or independence". Comment.

c) Lalu and Associates has been appointed as auditor for the year 2070/71 of ABC Bank Limited, a commercial bank in
the general meeting dated 25 Paush 2070 for remuneration of Rs. 500,000. In the course of the audit, the auditor finds
that the audit fee is too low and negotiates with the management for increase of the fees to the tune of Rs. 700,000. The
board approves the increased fee.

Answer

a) The various types of threats explained in ICAN code of ethics are as follows:

ƒ Self-interest threat – the threat that a financial or other interest willinappropriately influence the professional accountant’s
judgment orbehavior; For e.g. lowballing, hospitality or other benefits, contingent fees, loans to clients etc.

ƒ Self-review threat – the threat that a professional accountant will notappropriately evaluate the results of a previous
judgment made or serviceperformed by the professional accountant, or by another individualwithin the professional
accountant’s firm or employing organization,on which the accountant will rely when forming a judgment as part ofproviding
a current service; For e.g. valuation services along with audit service, accounting services.

ƒ Advocacy threat – the threat that a professional accountant will promotea client’s or employer’s position to the point that the
professionalaccountant’s objectivity is compromised; For e.g. the auditor Should not offer legal services to a client and
defendthem in dispute or litigation which is material to the financial statement.

ƒ Familiarity threat - the threat that due to a long or close relationshipwith a client or employer, a professional accountant will
be toosympathetic to their interests or too accepting of their work. For e.g. participation in client affairs, family and personal
relationship, audit partners leaving to join clients etc.

ƒ Intimidation threat – the threat that a professional accountant willbe deterred from acting objectively because of actual or
perceivedpressures, including attempts to exercise undue influence over theprofessional accountant. For e.g. if there is actual
or threatened litigationbetween client and assurance firm, the firmshould not continue to act

b) Section 11 of Code of Ethics of the Institute of the Chartered Accountants of Nepaldeals with "Activities Incompatible with the
Practice of Public Accountancy". Section 11 (2) stated that "the rendering of two or more types of professional services
concurrently does not by itself impair integrity, objectivity or independence".

As per Section 11 (1) a professional accountant in public practice should not concurrently engage in any business, occupation or
activity which impairs or might impair integrity, objectivity or independence, or the good reputation of the profession and therefore
would be incompatible with the rendering of professional services.

Further as per Section 11 (3) the simultaneous engagement in another business, occupation or activity unrelated to professional
services which have the effect of not allowing the professional accountant in public practice properly to conduct a professional
practice in accordance with the fundamental ethical principles of the accountancy profession should be regarded as inconsistent
with the practice of public accountancy.

c) Asper section 67 of Bank andFinancial Institutions Act, 2063, the remuneration of the auditor shall be as prescribed by the general
meeting if he or she has been appointed by the general meeting and by the board if he or she has been appointed by the board. In

The Institute of Chartered Accountants of Nepal 16


Suggested Answers - Audit & Assurance CAP-II Examination - December 2015

the given case Lalu and Associates has been appointed by the general meeting of ABC Bank Limited and hence his remuneration
shall be as prescribed by general meeting. So, the revision in the audit fees by the board in this case is not valid.

6. Write short notes on the following: (4×2.5=10)


a) Revaluation of Fixed Assets
b) Objectives of Audit of the Financial Statement
c) Ownership and Custody of Working Papers
d) Qualities of an Auditor

Answer

a) Revaluation of fixed assets is the process of increasing or decreasing their carrying value in case of major changes in fair market
value of the fixed asset. NAS-06, Property, Plant and Equipment requires fixed assets to be initially recorded at cost but they allow
two models for subsequent accounting for fixed assets, namely the cost model and the revaluation model. Under the Revaluation
model, the asset is carried at a revalued amount, being its fair value at the date of revaluation less subsequent depreciation and
impairment, provided that fair value can be measured reliably. Revaluations should be carried out regularly, so that the carrying
amount of an asset does not differ materially from its fair value at the balance sheet date. If an item is revalued, the entire class of
assets to which that asset belongs should be revalued. Revalued assets are depreciated in the same way as under the cost model. If a
revaluation results in an increase in value, it should be credited to other comprehensive income and accumulated in equity under
the heading "revaluation surplus" unless it represents the reversal of a revaluation decrease of the same asset previously recognised
as an expense, in which case it should be recognised in profit or loss. A decrease arising as a result of a revaluation should be
recognised as an expense to the extent that it exceeds any amount previously credited to the revaluation surplus relating to the same
asset. When a revalued asset is disposed of, any revaluation surplus may be transferred directly to retained earnings, or it may be
left in equity under the heading revaluation surplus. The transfer to retained earnings should not be made through profit or loss.

b) In conducting an audit of financial statements, the overall objectives of the auditor are:
• To obtain reasonable assurance about whether the financial statements as a whole are free from material misstatement,
whether due to fraud or error, thereby enabling the auditor to express an opinion on whether the financial statements are
prepared, in all material respects, in accordance with an applicable financial reporting framework;and
• To reporton the financial statements,and communicate as required by the NSAs,in accordance with the auditor’s findings.

Audits are not conducted with the objective of identifying fraud and errors prevalent in the company, but the audit process
normally result in identification of fraud or errors while conducting internal control evaluation and using substantive procedures.
This is because the auditing standards requires an auditor to bear in mind the possibility of the existence of frauds or errors in the
accounts under audit since they may cause the financial position to be mis-stated. So, detection of material frauds and errors as an
incidental objective of independent financial auditing flows from the main objective of determining whether or not the financial
statements give a true and fair view.

c) NSA 230, Audit Documentation contains the set of standards that deal with working papers. The standard provides that the auditor
should prepare, on a timely basis, audit documentation that provides a sufficient appropriate record of the basis for the auditor’s
report, andevidence that the audit was performed in accordance with NSAs and applicable legal and regulatory requirements.
Working papers provide evidence that an effective, efficient, and economic audit has been carried out. As such, working papers are
the property of the auditor. The auditor may, at his discretion, make portions of or extracts from his working papers available to his
client. The auditor should retain the working papers for the period specific or the period necessary to fulfill professional
responsibility relating to the assignment.

d) An auditor should adhere to the fundamental principles applicable for the auditor. The fundamental principles are integrity,
objectivity, professional competence and due care, confidentiality and professional behavior. The qualities required in and auditor
are tact, caution, firmness, good temper, integrity, discretion, industry judgment, patience, clear headedness and reliability. In
addition, he must have the shine of culture for attaining a great height. He must have the highest degree of integrity backed by
adequate independence. He must have a thorough knowledge of general principles of law which govern matters with which he is
likely to be intimate contact. He must pursue an intensive program of theoretical education in subjects like financial and
management accounting, general management, business and corporate laws, computer and information systems, taxation,
economics etc. An auditor must be honest; i.e; he must not certify what he does not believe to be true and must take reasonable
care and skill before he believes that what he certifies is true.

The Institute of Chartered Accountants of Nepal 17


Suggested Answers - Audit & Assurance CAP-II Examination - December 2015

7. Distinguish between: (2×5=10)


a) Auditing and Investigation
b) Batch Processing and On-Line Real Time System

Answer

a) Objects: The object of auditing is to find out whether Financial Statements givea true and fair view of business. Investigation is
undertaken to know the essential facts about a matter under inquiry. It is done with some special purpose of view.
Period: Auditing usually covers one accounting year . Investigation may cover more than one accounting year.
Legally Binding :Auditing is conducted for proprietors or is normally legally binding. Investigation is carried out on behalf of any
party interested in the matter.
Scope :Auditing is legally compulsory for companies & restricted to the financial statement. Investigation is voluntaryand carried
out on certain circumstances & may be carried out beyond the financial statement.
Time :Auditing may be conducted at the year end. Investigationmay be conducted atany time in case ofsuspicion about
anytransaction.
Report :In auditing form of report is prescribed. Ininvestigationit is not prescribed.
Appointment : In auditing appointment is made by shareholders in AGM. Ininvestigationappointment can be done by other
delegated authorities.
Qualification :Auditors qualification is prescribed by Laws. Investigators qualification has not prescribed by Laws.
Re-work :Re-audit is not generally carried out . Re-investigation may be carried out.
Perception :Audit is not carried out with doubtful mind . Investigation may be carried out with doubtful mind.

b) Data processing system in computerized environment is generally Batch Processing or On-Line Real Time (OLRT) system which
can be distinguished as under:

Batch Processing On-Line Real Time (OLRT) system


♦ Transactions are accumulated and processed in ♦ Transactions are processed as and when they occur.
group.
♦ Two types of files are maintained master file is ♦ Only master file is maintained. It keeps updating.
updated when batch processing is run.
♦ Updating does not take place as quickly as in On- ♦ Though updating takes place immediately the
Line Real time system. processing becomes complex.
♦ Not useful when instant and updated results are ♦ Useful for immediate reporting system.
required.
♦ Generally provides Audit trail. ♦ Generally, does not provide audit trail and hence
requires more attention of auditor.

The Institute of Chartered Accountants of Nepal 18


Suggested Answers - Corporate & Other Laws CAP-II Examination - December 2014

Corporate & Other Laws

1. Answer the following questions:

a) Miss Sita Sharma, a chartered accountant licensed by ICAN has been appointed as the auditor of XYZ Trading Ltd. for the
fiscal year 2071/72. It has been revealed that she was appointed by the general meeting of thecompany of the fiscal year
2070/71 on the basis of recommendation of her maternal uncle's son who has been working in the company as a general
manager since the last several years. Later, the appointment was questioned as the close relatives of an employee cannot be
appointed as an auditor in the company pursuant to the Companies Act, 2006. Discuss referring to the relevant provisions of the
Companies Act,2063 the validity of her appointment with disqualification of the auditor alongwith its consequence. 10

b) Auditor of Company ABC found out some irregularities during the course of audit and would like the shareholders to know
about these irregularities, therefore, he wants to have the company’s Extra Ordinary General Meeting (EGM) convened. But he
doesn't know how it would be convened,so he approaches you. Could you explain how theEGM of Company ABC can be
convened pursuant tothe Companies Act, 2063? 5

c) PQR Company is making profit for the last two years, the Board of Directors (BOD) of PQR Company wants to issue premium
shares. The BOD consults you regarding the pre-conditions of and the procedures for issuing premium shares. Suggest the
BOD of PQR Company about the pre-conditions to and the procedures for the issue of premium shares under the Companies
Act, 2063, and also subject to it whether PQR Company can premium shares or not. 5

d) A Proposed Sagarmatha silk public limited company applied to the Company Registrar’sOffice (CRO) for the registration of
apublic company with authorized capital Rs. 50 lacks only. After scrutinizing all the relevant papers Company Registrar refused
to register the company. Answer, what was the reason to refuse registration of that proposed company. 5
Answer:
a) Under Section 111(1) of the Companies Act, 2063, an auditor of a company shall be appointed from amongst the auditors
licensed to carry out audit under the prevailing law by the general meeting, in the case of a public company. The auditor shall
hold office until the next annual general meeting. For such appointment the provisions of Memorandum of Association and
Articles of Association of the Company shall also have to be confirmed. 1

Section 112 of the Companies Act, 2063 specifies the disqualifications of auditor as follows: 1) none of the following persons
or the firms or companies in which such persons are partners shall be qualified for appointment as auditor and shall, despite
appointment as auditor, continue to hold office: (1+3=4)
i) A director, advisor appointed with entitlement to regular remuneration or cash benefit, a person or employee or worker
involved in the management of the company or a partner of any of them or employee of any of such partners or a close
relatives of a director or partner, out of them, or an employee of such relative;
ii) A debtor who has borrowed moneys from the company in any manner, or a person who has failed to pay any dues
payable to the company within the time limit and is in such arrears or a close relative of such person;
iii) A person who has been sentenced to punishment for an offense pertaining to audit and a period of five years has not
elapsed thereafter;
iv) A person who has been declared insolvent;
v) A substantial shareholder of the company or a shareholder holding one percent or more of the paid up capital of the
company or his/her close relative;
vi) A person who has been sentenced to punishment for an offense of corruption , fraud or a criminal offense involving
moral turpitude and a period of five years has not elapsed thereafter;
vii) An auditor or his partner or ex- partner or employee or ex- employee who has already served as auditor for more than
three consecutive terms;
viii) In the case of a public company, any person who works, whether full time or part time, for any Governmental body or
anybody owned fully or partly by the Government of Nepal or any other company or a partner of such person or a
person who is working as an employee of such partner or a person who is authorized to sign any documents or reports to
be prepared by the management of the company;
ix) A company or corporate body with limited liability;
x) A person having interest in any transaction with the company or his/her close relative or a director, officer or substantial
shareholder of another company having any interest in any transaction with the company.

Section 2(z9) of Companies Act, 2063 defines the term “close relative” which means a partition shareholder in a joint family
or husband, wife, father, mother, mother-in-law, father-in-law, elder brother, younger brother, elder sister, younger sister,

The Institute of Chartered Accountants of Nepal 19


Suggested Answers - Corporate & Other Laws CAP-II Examination - December 2014

sister-in -law , (elder or younger brother's wife), brother in –law, sister in –law, brother in law(husband of elder sister), uncle,
aunt, maternal uncle, maternal aunt, son, daughter, daughter-in –law, grand-son, grand-daughter, grand-daughter-in law or
son-in-law.

On the basis of the above legal provisions the of the Companies Act, 2063 maternal uncle's son does not include under the
definition of close relative, therefore, Miss Sita Sharma is not disqualified to be appointed as director of XYZ Trading Ltd.,
though she was recommended for appointment by her maternal uncle’s son who was working as General Manager in that
Company.

Section 112(2) of the Companies Act, 2063, states that the auditor shall, prior to her appointment give information in writing
to the company that pursuant to Section 112(3) of this Act she is not disqualified.So Miss Sita Sharma’s prior information to
the company is also okay.
Section 82 of the Companies Act, 2063 has included provisions relating to a Company’s EGM,which are as follows:
Pursuant to section 82(1) of the Companies Act, 2063, the board of directors of a company may convene an extra-ordinary
general meeting (EGM) if it deems necessary.

b) Pursuant to section 82(2) of this Act, if in the course of examining the account of the Company ABC, it is deemed necessary
to call an extra ordinary general meeting for discussion of the irregularities at the EGM, the auditor may request to the board
of directors to call such meeting; and if the board of directors fails to call the meeting accordingly, the auditor may make an
application, setting out the matter, to the Office of Company Registrar (Office) and if an application is so made, the Office
may call the extra-ordinary general meeting of the company.

c) Section 2(z2) of the Companies Act, 2063 defines the term “premium share”as a share issued by a company as to sell it for a
value in excess of its face value.

d) Section 29 of the Companies Act, 2063 providesthe pre conditionsof and procedures to issue premium shares, which are as
follows:
(1) any company fulfilling the following conditions may, with the prior approval of the Company Registrar’s Office
(Office), issue shares at a premium:
a) the company has been making profits and distributing dividends for three consecutive years,
b) the company's net worth exceeds its total liabilities,
c) the company's general meeting has decided to issue shares at a premium

As PQR Company has been making profit for the last two years only, it doesn't meet the pre-condition mentioned above under
section 29(1)(a) of the Companies Act, 2063.Therefore, PQR Company cannot issue premium shares.

Refusal of registrar to register the public limited company is valid in pursuance to Section 11(1) of the Companies Act, 2063.
Pursuant to this section,the paid up capital of a public company shall be a minimum of ten million rupees, except as otherwise
provided in the prevailing law or in a notification by the Government of Nepal in the Nepal Gazette that the paid up capital of
any particular company shall be in excess of the said required minimum.

The Company Registrar’s Office(CRO), pursuant to section 6(1)(d) of the Companies Act, 2063 may refuse to register a
company if the requirements for the incorporation of a company under this Act are not fulfilled. So the CRO refused to register
this proposed public Company for not fulfilling the requirement of section 11(1) of the above Act.

2. Answer the following questions:


A) Nepal Rastra Bank (NRB) takes XYZ Bank under its control after suspending its board of directors as it failed to honour
liability and also it has not been properly operated. It was later on found out that XYZ Bank was able to honour its commitment
and was also in a position to be operated properly. Answer, what action may be taken by NRB under Nepal Rastra Bank Act,
2058 in this situation. 5
B) Mr. A, ex- governor of Nepal Rastra Bank, has been appointed as Director of XYZ Bank Ltd. immediately after his retirement
from the post of governor. His appointment was questioned by one of the shareholders. In this circumstance, whether his
appointment was against the legal provision or not? Answer it on the basis of the Banks and Financial Institutions Act (BAFIA),
2063. 5
C) Dwell upon the criminal liability of a drawer of a cheque in case it committed a crime under the provisions of Nepalese
Negotiable Instruments Act, 2034. 5
Answer:
a) Pursuant to section 86(1) of Nepal Rastra Bank Act,2058, NRB had taken such commercial bank under its control after
suspending its Board of Directors for having that commercial bank failed to honour its liability and also that has not been
properly operated.

The Institute of Chartered Accountants of Nepal 20


Suggested Answers - Corporate & Other Laws CAP-II Examination - December 2014

b) In pursuance to section 86(2) of this Act, NRB may after taking the management of XYZCommercial Bank under its control,
operate its business by itself or appoint a person, firm, company or institution to operate or manage the business of such
commercial bank. NRB shall, within one year of operating the business on its own or through any other person, firm, company
or institution audit or cause to audit the accounts of the concerned commercial bank and publish the report thereof pursuant to
section 86(3)) of this Act.
c) Pursuant to section 86(5) of this Act, NRB may take the following actions if it is convinced from the auditing report that the
concerned commercial bank is able to honour its commitment or such commercial bank is in a position to be operated properly:
i) To release the suspension of the BOD of the commercial bank and to order the same BOD to operate the business; or
ii) To dismiss the BOD of the commercial bank suspended and to have the business conducted by a new BOD formed from
amongst the shareholders of the commercial bank.
iii) To get the new BOD elected after convening the general meeting of the shareholders of that commercial bank and to have
the business conducted by it; or
iv) To take any other appropriate action as NRB may deem fit.

Pursuant to sub-sections (6) and (7) of Section 86 of Nepal Rastra Bank Act, 2058, NRB shall, before taking the commercial
bank under its control, provide an opportunity to defend within fifteen days to such commercial bank as per the situation. The
commercial bank in that situation shall bear all the expenses incurred while operating the commercial bank after NRB has taken
over pursuant to this Section.

d) Yes, Mr. A's appointment as a director of XYZ Bank Ltd. is against the existing legal provision of BAFIA, 2063. Section 18(j)
of the BAFIA, 2063 mentions that governor, deputy governor or special class officer of Nepal Rastra Bank are disqualified to
become Director unless one year of time from the date of such relief or retirement from the service has not elapsed. As Mr. A
has been appointed right after his retirement, one year time span has not been elapsed from his retirement from the post of
governor, he is automatically disqualified to become director of any other commercial bank. Therefore, his appointment as a
director of commercial bank is against section 18(j) of BAFIA2063.

e) Section 2(h) of Negotiable Instruments Act, 2034 defines a “Cheque” as a Bill of Exchange drawn on a certain Bank payable on
demand. Section 107A of the Negotiable instruments Act,2034 imposes the following criminal liability or punishment against a
drawer who commits the following corresponding offence against the Act:
In case any person who deliberately transfers a Cheque by drawing it to somebody that he/she does not bear deposit in the Bank
or even if there is a deposit which is not sufficient, and if the Cheque thus transferred is dishonoured due to lack of sufficient
deposit when the Cheque is presented to the concerned Bank for the payment, the amount mentioned in the Cheque as well as
interest on it shall be caused to be recovered to the Holder from the Drawer and he/she shall be punished with an imprisonment
up to Three months or a fine up toThree Thousand Rupees or both.

3. Answer the following questions:


a) Mr. Z, one of the directors of PQR ltd., passes some secret information regarding the securities of this company to his relatives.
On the basis of that secret information his relatives bought huge number of shares of that company. Few months later, share
price went up and Mr. Z's relatives earned huge amount of money from the transaction of those shares. Did Mr. Z commit any
crime under the Securities Act, 2063? If yes, what crime did he commit? and what punishment could be imposed on him?
Answer it on the basis of Securities Act,2063. 5
b) What are the liabilities of the concerned persons for the matter referred to in the prospectus as per the Securities Act, 2063?
5
Answer:
a) Yes, Mr. Z one of the directors of PQR Ltd. committed a crime against the Securities Act,2063. His activities are known as
insider trading and it is a punishable crime under Nepalese law. Section 91 of the Securities Act,2063 states that 'in case any
person conducts or causes others to conduct transactions in securities on the basis of any inside information or report which
has not been made public and which can have an impact on the value of the securities, or supplies any such information or
report possessed by him to any other person, except in the course of discharging his duties, he shall be deemed to have carried
out an insider trading of securities’.
b) Regarding the punishment, pursuant to section 101 of this Act,Mr. Z a director of PQR Ltd. Company is an insider of this
Companywho deals in securities in contravention of section 91 of this Act, is proved to have done so, he shall be punished
with a fine equal to the amount involved in the offence or with imprisonment for a term not exceeding one year or with both.
1. The liabilities of the concerned personsfor the matters referred to in the prospectus are laid down under Section 33 of the
Securities Act,2063, which are as follows:
2. The concerned body corporate and the director signing a prospectus and the expert preparing such a prospectus shall be
personally and collectively liable for the truth of the details and documents underpinning the information set down in the
prospectus submitted to Nepal Securities Board (Board) for the purpose of registering securities with the Board and
obtaining permission to issue such securities.

The Institute of Chartered Accountants of Nepal 21


Suggested Answers - Corporate & Other Laws CAP-II Examination - December 2014

3. Where any person who subscribes for any securities on the faith of the matters set down in the prospectus subsequently
sustains any loss or damage by the reason that the matters set down in the prospectus have been set down with mala fide
intention or untrue or false statements have been included therein knowingly, the body, director or experts preparing the
prospectus shall be liable to pay compensation for such loss or damage. Provided that no director shall be liable to pay such
compensation if he or she proves that he or she has resigned prior to making a decision on the matters set down in the
prospectus with ulterior motive or knowingly or that he or she did not know that the prospectus was untrue.

4. Where any investor sustains any loss or damage by the reason that the prospectus, information, statements or returns submitted by a
body corporate to the Board, such an investor may make a petition to the concerned District Court for compensation within thirty
five days from the date of knowledge within one year after the making of investment.

5. Answer the following questions:


a) Mr. JamesSaw, a foreign investor invested in Nepal in a joint venture with a Nepali investor and operated an industry since
2012. Later, a dispute arose regarding the sharing of profit and loss between him, Nepali investor and the industry. He wants to
file a case against the Nepali investor and the industry. So he consults you regarding the way of resolving the dispute between
him, the Nepali investor and the industry. Advise him the way of resolving the dispute between him and the industry under the
Foreign Investment and Technology Transfer Act, 2049. 5
b) Enumerate the functions, duties and powers of the Industrial Promotion Board as provided by the Industrial Enterprises Act,
2049. 5
Answer:
a) Section 7 of Foreign Investment and Technology Transfer Act,1992 has provided the way out for solving the dispute between
the foreign investor and the concerned industry. According to it, if any dispute arises between a foreign investor, national
investoror the concerned industry, the concerned parties shall be required to settle the dispute by mutual consultations in the
presence of Department of Industry. If the dispute could not be settled through mutual consultation, it shall be settled by
arbitration in accordance with the prevailing Arbitration Rules of the UNCITRAL. Moreover, the place of arbitration shall be
held in Kathmandu and the Nepalese laws shall be applicable in the arbitration. Therefore, Mr. James Saw should try to settle
the dispute by following the provisions mentioned above, but pursuant to the foreign investment agreement concluded between
him and the national (Nepali) investor.

Notwithstanding anything contained in sub-sections(1),(2) and (3) above, disputes arising in regards to foreign investment
made in the industries with investments as prescribed may be settled as mentioned in the foreign investment agreement.

b) Section 13 of the Industrial Enterprises Act,2049 provides the following power, duties and functions to the Industrial
Promotion Board (Board)
a. To render necessary cooperation in formulating and implementing policies, laws and regulations pertain to the
industrialization of the country.
b. To give guidelines in attaining the objectives of liberal, open and competitive economic policies pursued by the country
so as to make the industrial sector competitive.
c. To maintain coordination between the policy level and the implementation level of the industrial policy.
d. To cause to follow the ways and means for the prevention of the environmental pollution by putting more emphasis on the
avoidance of effects on the environment and the public health.
e. To make recommendation to Government of Nepal for the inclusion of any industry in the classification of industries.
f. To make recommendation to Government of Nepal to introduce changes in the Areas mentioned in Annex-3 by making
evaluation thereof from time to time.
g. To give directives to the concerned body after making enquiries into the application submitted by any industry
complaining that the industry has not received the facilities and concessions to be made available by the committee.
h. Other functions, duties and power of the Board shall be as prescribed.

6. Answer the following questions:


a) Explain the provisions relating to ceiling on bonus payment to be made to the employees as per the Bonus Act,2030. 5
b) Mention the objectives of the Institute of Chartered Accountants of Nepal (ICAN) under Nepal Chartered Accountants Act,
2053. 5
c) What do you understand by the term Corporate Body Substantially owned by the Government of Nepal? Who will audit the
account of such Corporate Body Substantially owned by the Government of Nepal? and how it will be audited? Answer it in the
context of the Audit Act, 2048. (2+3=5)

The Institute of Chartered Accountants of Nepal 22


Suggested Answers - Corporate & Other Laws CAP-II Examination - December 2014

Answer:
a) Ceiling on the distribution of bonus has been mentioned in Section 7(3) of Bonus Act 2030, which are as follows:
b) The management shall assess the percentage of bonus amount to be obtained by on Employee in a Fiscal year.While
computing bonus amountto be received by an employee, the amount so separated for the distribution of bonus by the
enterprise for the particular fiscal year shall be multiplied by a sum of one hundred and the amount so deducted shall be
divided by the pay or wage amount of the employee so entitled to receive the bonus and the amount so deducted shall be the
percentage of bonus amount.
Whatever mentioned in this Act, the bonus to be obtained by an employee shall not exceed the following amounts:
a. An amount equivalent to the salary or wage of Six months, to an employee, who obtains upto Five Thousand Rupees as
salary or wage.
b. An amount equivalent to the salary or wage of Four months to an employee, who obtains Five thousand One Rupees to
Fifteen Thousand Rupees as salary or wage.
c. An amount equivalent to the wage or salary of Three months to an employee who obtains more than Fifteen Thousand
rupees as salary or wage.
d. The minimum bonus amount to be obtained under Clauses (b) and (c) of Sub-section (3)of this section above shall not be
less than the maximum bonus amount to be obtained under Clauses (a) and (b) respectively as above.
i) Section 5 of Nepal Chartered Accountants Act,2053 states the objectives of the ICAN, which are as follows:
ii) To play the role of regulatory body to encourage the members to carry on accounting profession being within the
extent of the code of conduct in order to consolidate and develop accounting profession as a cause for economic
development of the nation.
iii) To enhance social integration and faith in accounting profession by raising awareness of the general public towards
the importance of accounting profession and the economic and social responsibility of professional accountants.
iv) To develop, protect and promote the accounting profession by enabling professional accountants understand their
responsibility towards the importance of accounting profession and accountancy.
v) To develop mechanism of registration, evaluation and examination of accounting professionals in consonance with
international norms and practices so as to make the accounting profession respectable and reliable.

c) Section 2(e) of the Audit Act,2048 defines the term 'corporate body substantially owned by theGovernment of Nepal.
According to this provision corporate body substantially owned by Nepal government means a corporate body whose more
than 50 percent shares or assets are owned by the Government ofNepal. So far as the audit of the account of such body is
concerned, it is mentioned under section 7 of this Act. According to which, the audit of such body shall be done in
accordance with the existing laws relating to such body. For that purpose,the Auditor General shall be consulted while
appointing an auditor for auditing of such body. A copy of the report of audit submitted by such Auditor shall be delivered to
the office of theAuditor Generalby such body. The procedures to be followed while consulting the Auditor General for
appointing auditors and on matters to principles of audit to be followed by the auditors doing their audit shall be as prescribed
by the Auditor General. Pursuant to section 7(5) of this Act, the Auditor General may issue directives to the concerned
organization is respect of the irregularities observed in the report received and it shall be the duty of the concerned
organization to abide by such directives.

7. Answer the following questions:


a) What are the circumstances that makes contract voidable? Answer it in the light of the Contract Act, 2056. 5
b) Explain about the payment of insurance claims after the cancellation of the insurer in the light of Insurance Act, 2049. 5
c) Explain the provisions relating to 'specific performance' of a contract under the Contract Act,2056. 5
Answer:
a) A contract means an agreement enforceable by law concluded between two or more parties for performing or not performing
any work. Voidable contracts are such which are defective in the process of making it, but not really void due to defects in the
essence of it. Voidable contracts can be made enforceable by the aggrieved party, following the prescribed procedures, and
within the prescribed period.

Section 14 of the Contract Act, 2056 mentions few circumstances that makes an agreement voidable, which are as follows:
1. Contracts concluded through coercion: Explanation to section 14(1)(a) of this Act explains it as follows: A person shall be
deemed to have indulged in coercion if he/she, withthe objective of compelling any person, to accept any contract against
his/her will, withholds or threatens to withhold property belonging to him/her, or threatens to defame him/her, or takesor
threatens to take any other action in contravention of prevailing law.

2. Contract concluded through Undue influence: Explanation to section 14(1)(b) of this Act explains it as follows: (1) Undue
influence means influence exercised by a person upon another person who is under his/her influence and is amenable to
his/her personal benefit or interest. (2) without prejudice to the generality of clause (1), the following persons shall be
deemed to be under the influence of any person and amenable to his/her wishes-(i) A person living under his/her

The Institute of Chartered Accountants of Nepal 23


Suggested Answers - Corporate & Other Laws CAP-II Examination - December 2014

guardianship, protection or custody; (ii) Persons who cannot take care of their interest temporarily or permanently by
reason of old age, sickness or physical or mental weakness; (iii) A person who can subjected to under one’s economic or
ranking influences.

3. Contract concluded through fraud: Explanation to section 14(1)(c) of this Act explains it as follows : A party to a contract
or his/her agent shall be deemed to have committed fraudif he/she, leads the other party or his/her agent to believe or takes
any action to believe the particular matter is true, although he/she knows that it is false, orsuppressesany information in
his/her possession, or indulges in any other fraudulentactpunishable under prevailing law, with the intention of deceiving
the opposite party or his/her agent.

4. Contract concluded through deceit: Explanation to section 14(1)(d) of this Act explains it as follows: Any of the following
act shall be taken as deceit:
i. Submission of false particulars on any matter without reasonable basis for doing so;
ii. Misleading any party so as to aggrieve him/her:
iii. Causing any wrong deliberately on the matter of contract.

b) Section 16 of the Insurance Act, 2052provides the following about the payment of insurance claims after the cancellation of the
insurer: After the cancellation of the insurer, the insurer, dissolved by the cause of the cancellation of its registration pursuant to
section 13, shall refund the amount received by it for insurance to the person, organization or theInsuranceBoard (Board), within the
period and method specified by the Board. It shall refund the principal amount alongwith bonus as specified by the Board in the
case of life insurance, and it shall refund the principal amount as specified by the Board on a proportional basisin the case of non-
life insurance.

c) Section 86 of the Contract Act, 2056 provides for ‘Specific Performance’. Accordingly in case the cash compensation paid in
consideration of the actual loss or damage suffered by the aggrieved party as a result of breach of contract is not reasonable or
adequate, the aggrieved party may demand the execution of the contract as stipulated specific performance instead of making a
claim for compensation (section 86(1))
Notwithstanding anything contained in Sub-section (1) above, no claims for execution of the contract as stipulated specific
performance shall be heard in any of the following circumstances;
i. In case the amount paid in cash as compensation for breach of contract is adequate;
ii. In case the court cannot supervise whether or not the work to be performed under the contract has been actually performed;
iii. In case the contract has been signed for providing services relating to personal expertise, skill or knowledge;
iv. In case the situation is -such that the contract cannot be executed as stipulated;
v. In case the party violating the contract him/herself demands that the contract be executed as stipulated.

8. Write short notes on the following: (2x5=10)


a) Powers of Labour Officer
b) Exchange Equalization Fund

Answer:
a) Pursuant to section 64 of the Labour Act, 2048, the Government of Nepal by publishing a notice in the Nepal Gazette may
appoint one or more Labour Officers, as per necessity or designate any other officer to perform the functions of a Labour
Officer for one region/sector.
b) Pursuant to Section 65 of theLabour Act, 2048, the Labour Officer shall have the following powers –
1. To enter into the premises of the Enterprise as per necessity;
2. To examine the documents and registers of the Enterprise relating to workers and employees;
3. To function or advise as per necessity for improving labour relations;
4. To attempt for solving disputes arising between workers or employees and the Proprietor;
5. To implement welfare provisions, if it does not exit and where if exist, to supervise whether or not it is operated properly;
6. To supervise the implementation of minimum remuneration prescribed by Government of Nepal;
7. To record statement, of anybody to fulfill the objectives of this Act, as per necessity;
8. To perform tasks of the Factory Inspector in his/her absence, except technical tasks; and
9. To perform other tasks as per the directives of Government of Nepal and Department of Labour.
10. Other powers, functions and duties shall be as prescribed.

c) Section 45 of the Banks and Financial Institutions Act, 2063 has provision for the Exchange Equalization Fund. Accordingly:
(1) a licensed institution which has obtained the license to carry on foreign exchange transactions shall make necessary
accounts adjustments in the profit and loss account of the revaluation profits earned as a result of fluctuations in the exchange
rates of foreign currencies, other than the Indian currency, every year at the end of the same fiscal year. While making such

The Institute of Chartered Accountants of Nepal 24


Suggested Answers - Corporate & Other Laws CAP-II Examination - December 2014

accounts adjustment in the profit and loss account, if revaluation earning has been made in any fiscal year, at least twenty five
per cent of such profits shall be credited to the exchange equalization fund.

d) Provided that in the case of revaluation profit-loss resulting from fluctuation in the exchange rate of the Indian currency, it
shall be as prescribed by the Rastra Bank.

e) (2) No amount credited to the exchange equalization fund pursuant to Sub-section (1) above shall, without the approval of the
Rastra Bank, be spent or transferred for any purpose other than the adjustment of loss resulting from the devaluation of foreign
currencies.

The Institute of Chartered Accountants of Nepal 25


Suggested Answers - Financial Management CAP-II Examination - December 2014

Financial Management
1. Beta Company Limited is considering replacement of its existing machine by a new machine which is expected to cost Rs. 264,000.
The new machine will have a life of five years and will yield annual cash revenues of Rs. 568,750 and incur annual cash expenses of
Rs. 295,750. The estimated salvage value of the new machine is Rs. 18,200. The existing machine has a book value of Rs. 91,000
and can be sold for Rs. 45,500 today.

The existing machine has a remaining useful life of five years. The annual cash revenues from this machine will be Rs. 455,000 and
associated annual cash expenses will be Rs. 318,500. The existing machine will have a salvage value of Rs. 4,550 at the end of its
useful life.

The company is in 25% tax bracket, and writes off depreciation at 25% on written-down value method.

The company has a target debt to total capital ratio of 15%. It has raised debt at 11% in the past and it can raise fresh debt at 10.5%.
The company plans to follow dividend discount model to estimate the cost of equity capital. It further plans to pay a dividend of Rs.
2 per share in the next year. The dividend per equity share of the company is expected to grow at 8% p.a. The current market price
of the company's equity share is Rs. 20 per equity share.

Required: (4+8+3+4+1=20)
a) Compute the weighted average cost of the capital of the company.
b) Compute the incremental cash flows for replacement decision.
c) Find out the net present value of the replacement decision.
d) Estimate the discounted payback period of the replacement decision.
e) Should the company replace the existing machine? Advise.

Answer
a) Computation of Weightedaverage cost of capital of the company (WACC)
Ke = D1/P0 + g
=2/20 + 0.08
= 0.18 = 18%
Kdt = 10.5%×(1-0.25)
= 7.875%

WACC= Kd×D/(D+E) + Ke×E/(D+E)


=(7.875%×15%) + (18%× 85%)
=1.18% + 15.3%
=16.48%

(b& c) Incremental cash flow and NPV of replacement decision


Particulars Year 0 Year 1 Year 2 Year 3 Year 4 Year 5
Incremental Initial Cash Outlay (WN 1) (207,125)
Incremental Revenue (WN 2) 113,750 113,750 113,750 113,750 113,750
Saving in Expenses (W.N 2) 22,750 22,750 22,750 22,750 22,750
Less: Incremental Depreciation (W.N 3) 43,250 32,437 24,328 18,246 13,685
Earnings before Tax 93,250 104,063 112,172 118,254 122,815
Less: Tax @ 25% 23,313 26,016 28,043 29,564 30,704
Earnings after Tax 69,937 78,074 84,129 88,690 92,111
Add: Incremental Depreciation 43,250 32,437 24,328 18,246 13,685
Add: Incremental salvage value (18200-
4550) 13,650
Add: Incremental Tax Saving on Loss
on Sale(W.N 4) 6,851
Incremental cash flows (207,125) 113,187 110,511 108,457 106,936 126,297
PV Factor@16.48% 1.00 0.860 0.740 0.636 0.547 0.470
Present Value (207,125) 97,341 81,778 68,979 58,494 59,360
NPV 158,827

The Institute of Chartered Accountants of Nepal 26


Suggested Answers - Financial Management CAP-II Examination - December 2014

d) Calculation of Discounted PBP of replacement decision:

Year Cumulative PV (Rs.)


0 (158,827)
1 (109,784)
2 (28,006)
3 40,973
4 99,467
5 158,827

Discounted Payback Period


= 2 Years + Cumulative PV of Cash flows in 2nd Year
PV of Incremental Cash flow in 3rd Year
= 2 years + 28,006/68,979
= 2.406 years or 2 Years 5 months (approx)

e) The company should replace the machine since the incremental NPV of the decision is positive and discounted PBP is much lower
than the life of the machine.

Working Notes:
1. Incremental initial cash outlay

Purchase price of new machine = Rs. 264,000


Less: Current sales price of old machine = Rs. 45,500
BSV- 91,000
CSV- 45,500
Loss 45,500
Tax Saving on due to loss (45,500*25%) 11,375
Incremental initial cash outlay = Rs. 207,125

2. Calculation of incremental revenue and saving in expenses

Annual Incremental revenue = Rs. 568,750 – Rs. 455,000


= Rs. 113,750
Annual Saving in expenses = Rs. 318,500 - Rs. 295,750
=Rs.22,750

3. Calculation of incremental depreciation


Year Depreciation of new machine (Rs.) Depreciation of Old machine (Rs.) Difference
(Rs.)
1 264,000 × 25% = 66,000 91,000×25% = 22,750 43,250
2 198,000× 25%= 49,500 68,250 ×25%=17063 32,437
3 148,500 × 25%= 37,125 51,187 × 25%=127,97 24,328
4 111,375 × 25%= 27,844 38,390 × 25%=9,598 18,246
5 83,531 × 25%= 20,883 28,792 × 25%= 7,198 13,685

4. Incremental loss on sale at the end of 5th year


BV of new machine = Rs. 83,531 –Rs. 20,883 = Rs. 62,648
Less : BV of old machine = Rs. 28,792 –Rs.7,198 = Rs. 21,594
Incremental book value = Rs. 41,054
Less: Incremental sales value = Rs. 18,200 –Rs. 4550 = Rs. 13,650
Incremental loss = Rs. 27,404
Tax savings @ 25% = Rs. 6,851

2.
a) Explain the important ratios that would be used in each of the following situations: 6
i) A bank is approached by a company for a loan of Rs. 50 lakhs for working capital purpose.
ii) A long term creditor interested in determining whether his claim is adequately secured.
iii) A shareholder who is examining his portfolio to decide whether he should hold or sell his holdings in a company.
b) A company has 8 lakhs equity shares outstanding at the beginning of the year. The current market price per share is Rs. 120.
The board of directors of the company is contemplating to declare a dividend of Rs. 6.40 per share. The rate of capitalization,
appropriate to the risk-class to which the company belongs, is 9.6 percent.

The Institute of Chartered Accountants of Nepal 27


Suggested Answers - Financial Management CAP-II Examination - December 2014

Required: (4+5=9)
i) Based on M-M approach, calculate the market price of the share of the company, when dividend is (i) declared and (ii) not
declared.
ii) How many new shares are to be issued by the company, if the company desires to fund an investment budget of Rs. 320
lakhs by the end of the year, assuming net income for the year will be Rs. 160 lakhs?

Answer
a) Important Ratios used in different situations
(i) Liquidity Ratios –
Liquidity or short term solvency ratios would be used by the bank to check the ability of the company to repay its short-
term liabilities. A Bank may use current ratio or Quick ratio to judge short term solvency of the company. Further interest
coverage ratio shall also be analysed to ensure the interest repayment security.
(ii) Capital Structure or Leverage Ratios –
The long-term creditor would use the capital structure or leverage ratios to ensure the long term stability and structure of
the firm. A long term creditor interested in determining whether his claim is adequately secured may use Debt-servicing
coverage and interest coverage ratio.
(iii) Profitability Ratios –
The shareholder would use the profitability ratios tomeasure the operational efficiency of the company to see the final
results of business operations. A shareholder may use return on equity, earning per share and dividend per share ratios.
Price earning ratio and book value per share are also analysed to decide wheather a particular share is to sell or hold.
b)
i) M-M Approach of calculating share price:
P0= (P1+D1) / (1+Ke)
Where,
P0= Existing market price per share i.e. Rs. 120
P1= Market price of the share at the year end (to be determined)
D1= Contemplated dividend per share i.e. Rs. 6.40
Ke= Capitalization rate i.e. 9.6 %

Calculation of Share price when:


(i)Dividend is declared
P0= (P1+D1) / (1+Ke)
120 = (P1 + 6.4)/ (1+0.096)
P1= Rs. 125.12
(ii)Dividend is not declared
P0= (P1+D1) / (1+Ke)
120 = (P1 + 0)/ (1+0.096)
P1= Rs. 131.52

ii. Calculation of No. of shares to be issued:


Particulars If dividend is If dividend is not
declared declared
Nets Income (Rs. In Lakhs) 160 160
Less: Dividend (Rs. In Lakhs) 51.20 -
Retained Earnings (Rs. In Lakhs) 108.80 160
Investment budget (Rs. In Lakhs) 320 320
Amount to be raised by issue of shares (Rs. In Lakhs) 211.20 160
Market price per share (in Rs.) 125.12 131.52
No. of new shares to be issued ( in numbers) 1,68,798 1,21,655

3.
a) You are provided with the following extract of cost sheet of ABC Ltd:
Per unit (Rs.)
Raw material 50
Direct Labour 20
Overhead (including depreciation of Rs. 10) 40
Total Cost 110
Profit 20
Selling price 130

The Institute of Chartered Accountants of Nepal 28


Suggested Answers - Financial Management CAP-II Examination - December 2014

Average raw material in stock is for one month. Average material in work-in-progress is for half month. The suppliers allow
credit for one month to the company; and it also allows one month credit to its customers. Average time lag in payment of
wages and overheadsare 10 days and 30 days respectively. 25% of the sales are on cash basis. Cash balance is expected to be
Rs. 100,000. Finished goods are expected to lie in the warehouse for one month.Production is carried on evenly throughout the
year and wages and overheads accrue accordingly.

Required: 7

Prepare a statement of the working capital needed to finance the level of activity of 54,000 units of output. State your
assumptions, if any.

b) JK Ltd. has appointed you as its Finance Manager. The company wants to implement a project for which Rs. 60 lakh is
required to be raised from the market as a means of financing the project. The following financing plans and their options are at
hand:
(Rs. in lakh)
Particular Plan X Plan Y Plan Z
Option 1: Equity Shares 60 60 60
Option 2: Equity Shares 30 40 20
13% Preference Shares Nil 20 20
10% Non Convertible Debentures 30 Nil 20

Assume corporate tax rate to be 25 per cent, and the face value of all the shares and debentures to be Rs. 100 each.
Required: (7+1=8)
i) Calculate the indifference points and earnings per share (EPS) for each of the financing plan.
ii) Which plan should be accepted by the company? Why?

Answer
a) As the annual level of activity is given at 54,000 units, it means that the monthly would be 54,000/12=4,500 units. The monthly
working capital requirement for this monthly turnover can now be estimated as follows:
Estimation of Working Capital Requirement

I. Current Assets: Amount(Rs.)

Minimum Cash Balance 100,000


Inventories:
Raw Materials (4,500×Rs.50) 225,000
Work in Progress:
Materials (4,500× Rs. 50)/2 112,500
Wages 50% of (4,500× Rs. 20)/2 22,500
Overheads 50% of (4,500× Rs. 30)/2 33,750
Finished Goods (4,500× Rs.100) 450,000
Debtors(4,500× Rs.100×75%) (at cost) 337,500
Gross Working Capital 1,281,250

II. Current Liabilities:

Creditors for Materials(4500× Rs.50) 225,000


Creditors for Wages(4,500× Rs.20)/3 30,000
Creditors for Overheads(4,500× Rs. 30) 135,000
Total Current Liabilities 390,000
Net Working Capital (I-II) 891,250

Working Note/ Assumptions:

Note: Alternatively, student may assume debtors on selling price i.e. Rs. 130 in such a case , overhead needs to be calculated including
depreciation i.e. at Rs. 40 while calculating WIP & finished goods.
1. The Overheads of Rs. 40 per unit include depreciation Rs. 10 per unit, which is non-cash item. This depreciation cost has been
ignored for valuation of work-in-progress, finished goods and debtor. The overhead cost, therefore, has been taken only at Rs.
30 per unit.
2. In the valuation of work-in-progress, the raw materials have been taken at full requirements for 15 days; but the wages and
overheads have been taken only at 50% on the assumption that on an average all units in work in progress are 50% complete.

The Institute of Chartered Accountants of Nepal 29


Suggested Answers - Financial Management CAP-II Examination - December 2014

3. Since, the wages are paid with a time lag of 10 days, the working capital provided by wages have been taken by dividing the
monthly wages by 3 (assuming a month to consist of 30 days)

b)
i) Determination of indifference point under plans X, Y, Z
Let X be the EBIT in all cases.
Plan X : X(1-t)/N1 = (X-Interest)(1-t)/N2
Or,X (1-0.25)/60,000 = (X-300,000)(1-0.25)/30,000
Or, X-0.25X=2(0.75X-225,000)
Or, X-0.25X=1.5X-450,000
Or, 0.75X=Rs. 450,000
Or, X = Rs. 600,000
Plan Y : X(1-t)/N1 = [(X-Interest)(1-t)-Dp]/N2
Or, X (1-0.25)/60,000 = [X(1-0.25)-260,000]/40,000
Or, 0.75X/60,000=(0.75X-260,000)/40,000
Or, 2(0.75X)=3(0.75X-260,000)
Or, 1.50X=2.25X-780,000
Or, X = 780,000/0.75
Or, X = Rs. 1,040,000
Plan Z : X(1-t)/N1 = [(X-Interest)(1-t)-Dp]/N2
Or, X (1-0.25)/60,000 = [(X-200,000)×(1-0.25)-260,000]/20,000
Or, 0.75X/60,000=(0.75X-410,000)/20,000
Or, 0.75X=3(0.75X-410,000)
Or, 0.75X=2.25X-1,230,000
Or, X = Rs. 820,000
Determination of EPS under Plans X , Y and Z for options 1 and 2

Plan X Plan Y Plan Z


Particulars
1 2 1 2 1 2
EBIT 600,000 600,000 1,040,000 1,040,000 820,000 820,000
Less : Interest - 300,000 - - - 200,000
EBT 600,000 300,000 1,040,000 1,040,000 820,000 620,000
Less : Taxes @ 25% 150,000 75,000 260,000 260,000 205,000 155,000
EAT 450,000 225,000 780,000 780,000 615,000 465,000
Less: Preference Dividend - - - 260,000 - 260,000
Earnings available to Equity holders 450,000 225,000 780,000 520,000 615,000 205,000
No. of Equity Shares 60,000 30,000 60,000 40,000 60,000 20,000
EPS 7.50 7.50 13 13 10.25 10.25

ii) The company should adopt Plan Y since the EPS is maximum under this plan.

4.
a) Consider from the following information:

Equity share capital (Rs.100 each) Rs. 5,000,000


Reserves and surplus Rs.500,000
15% secured loans Rs. 2,500,000
12.5% unsecured loans Rs. 1,000,000
Fixed assets Rs. 3,000,000
Investments Rs.500,000
Operating profit Rs. 2,500,000
Tax rate 25%
PE ratio 12.5
Required: 5
Calculate the value of each equity share.

The Institute of Chartered Accountants of Nepal 30


Suggested Answers - Financial Management CAP-II Examination - December 2014

b)
i) Acompany has to make the payment of Rs. 2,000,000 on 5th of March 2015. It has some surplus moneytoday i.e.,4th
December 2014 and it has decided to invest in a deposit of bank at 8% per annum to meet the amount for payment. What
money is required to be invested now? Take year as 365 days. 2
ii) Calculate the market sensitivity index and the expected return on investment from the following data: 3

Standard deviation of an asset 2.5%


Market standard deviation 2%
Risk free rate of return 13%
Expected return on market portfolio 15%
Correlation coefficient of portfolio with market 0.8

c) ABC limited expects, with some degree of certainty, to generate the following profits and to have the following capital
investment during the next five years.
(Rs. in thousand)
Year 1 2 3 4 5
Net Income 5,000 4,000 2,500 2,000 1,500
Investment 2,500 2,500 3,200 4,000 5,000

The investments are financed first from the same year profit and the shortfall,if any, shall be externally financed.
The company currently has 1,000,000 equity shares and pays dividend of Rs. 5 per share.
Required:
i) Determine dividend per share, if dividend policy is treated as a residual decision. 2
ii) Determine dividend per share and the amount of the external financing that will be necessary, if a dividend payout ratio of
50% is maintained. 3
Answer
a) We have,
Value= EPS × PE Ratio
EPS calculation Rs.
Operating profit i.e. EBIT 2,500,000
Less: Interest on 15% secured loans 375,000
Interest on 12.5% unsecured loans 125,000
Profit before tax 2,000,000
Tax @ 25% 500,000
PAT 1,500,000
Number of equity shares = (Rs 5,000,000/100) = 50,000
Therefore EPS = (Rs 1,500,000/50,000)= Rs. 30
P/E Ratio is given as 12.5
Therefore,
Value of equity share =EPS×PE Ratio= 30×12.5 =Rs.375

b)
i) Target money: Rs. 2,000,000
The amount to be invested now is in fact the present money of this targeted money. The FVF may be ascertained as follows:
FVF = 1+ [(annual rate of interest × (Deposit period/365)]
=1+[(0.08x(90/365)]
=1.01972
Now the present value of the target amount can be ascertained as follows:
Present value = Target amount/1.01972
= Rs. 1,961,323

Note: Student may use PVF and in such case present value = Target money×PVF
the deposit of Rs. 1,961,323 at the rate of 8% for a period of 90 days will accumulate to Rs. 2,000,000. Therefore, amount to be
invested is Rs. 1,961,323

ii) Market sensitivity index i.e. Beta factor can be calculated as follows:
Βeta = (Standard Deviation of asset/Standard Deviation of Market) ×Correlation Coefficient of Market
= (2.5/2)×0.80
=1

The Institute of Chartered Accountants of Nepal 31


Suggested Answers - Financial Management CAP-II Examination - December 2014

Now, the expected return on the investment canbe ascertained with the help of CAPM equation as follows:
R =rf+(Rm-rf)β
=13%+(15-13%)1
=15%

c)
i) Calculation of the Dividend per share if dividend policy is treated as a residual decision

Year Profit(Rs.) Investment(Rs.) Balance(Rs.) DPS(Rs.)


1 5,000,000 2,500,000 2,500,000 2.50
2 4,000,000 2,500,000 1,500,000 1.50
3 2,500,000 3,200,000 - 0
4 2,000,000 4,000,000 - 0
5 1,500,000 5,000,000 - 0

ii) Calculation of the Dividend per share and external financing required at 50% payout
External
Year Profit(Rs.) Dividends(Rs.) DPS(Rs.) Investment(Rs.) Financing(Rs.)

1 5,000,000 2,500,000 2.50 2,500,000 -

2 4,000,000 2,000,000 2.00 2,500,000 500,000

3 2,500,000 1,250,000 1.25 3,200,000 1,950,000

4 2,000,000 1,000,000 1.00 4,000,000 3,000,000

5 1,500,000 750,000 0.75 5,000,000 4,250,000

5.
a) Following is the data regarding six securities:

Securities A B C D E F
Return % 10 10 15 5 11 10
Risk (SD) % 5 6 13 5 6 7

Required: (2+4=6)
i) Which of three securities will be selected by an investor and why?
ii) Assuming perfect negative correlation, analyse whether it is preferable to invest 80% of money in Security A and 20% in
Security C or to invest 100% of money in Security E.
b) A firm is contemplating to increase its credit period from 30 days to 60 days. The average collection period, which is at present
45 days, is expected to increase to 75 days. Due to this change, the bad debt expenses is expected to increase from the current
level of 1 percent to 3 percent of sales. Total credit sales are expected to increase from the level of 30,000 units to 34,500 units.
The present average cost per unit is Rs. 8. The variable cost and sales are Rs. 6 and Rs. 10 per unit respectively. The firm
expects a rate of return of 15 percent.

Required: 5
Analyse the firm's proposal to change the credit period and advise.
c) Discuss about the fundamental principles governing capital structure. 4

The Institute of Chartered Accountants of Nepal 32


Suggested Answers - Financial Management CAP-II Examination - December 2014

Answer
a)
i) Arranging the securities in the increasing order of risk:
Risk % Return % Security Invest
5 10 A Yes
5 5 D No
6 10 B No
6 11 E Yes
7 10 F No
13 15 C Yes

Securities A, E and C should be selected. The investor will try to minimize risk and maximize return. Therefore, security A is
better than D, security E is better than B and C is better with highest return.

(ii) Calculation of overall return and overall standard deviation (Risk) when 80% in Security A and 20% in Security C is invested.
Overall return = ReturnA×WeightA+ ReturnC×WeightC
= 10%×0.8 + 15%×0.2
=11%
Overall Variance (SD2) = (SDA× WA)2 + (SDC× WC)2 + 2× (SDA× WA) × (SDC× WC) ×rAC
SD2 = (5×0.80)2 + (13×0.20)2 + 2 ×(5×0.8) × (13×0.2) × (-1)
2
SD = 16 + 6.76 + 2 X 4 X 2.6 X (-1)
= 22.76 - 20.8
= 1.96
SD = 1.4
Summary
Alternative-1 Alternative-2 (100%
(80% in A and 20% in C) in E)
SD (%) 1.4 6
Overall return (%) 11 11

Conclusion:
Since same return i.e. 11% is available from lower risk (SD) of 1.4 %, Alternative-1 is preferable over alternative-2.

b) Profit on additional sales =Rs.(10 –6) × (34,500 -30,000)units


= 4 ×Rs. 4,500
= Rs. 18,000

Average Investment in Accounts Receivable:


Present = (Total Sales Qty ×Average Cost Per Unit)/Receivable Turnover ratio
= (30,000×8) / (360/45)
= 240,000/8
=Rs.30,000
Proposed = (Existing Cost of sales + additional cost of sales)/Receivable Turnover ratio
= (2,40,000+4,500×6) / (360/75)
= 2,67,000 / 4.8
= Rs. 55,625
Additional Investment in Accounts Receivable = Rs. 55,625 – Rs. 30,000
= Rs. 25,625
Cost of additional investment in Accounts Receivable at 15% = Rs. 25,625× 15%
= Rs. 3,843.75
Additional Bad debt expenses
= (34,500×10×0.03) – (30,000×10×0.01)
= 10,350 – 3,000
= Rs. 7,350
The Net Effect of Proposed increase in credit period
= Additional Profit – additional expenses
= Rs. 18,000 – Rs.3,843.75 – Rs. 7,350
= Rs. 18,000 – Rs. 11,193.75
= Rs. 6,806.25

Conclusion:
The extension of credit period would result in net gain of Rs. 6,806.25, so the firm is advised to extend credit period from
30 days to 60 days.

The Institute of Chartered Accountants of Nepal 33


Suggested Answers - Financial Management CAP-II Examination - December 2014

c) The fundamental principles governing capital structure are:


(i) Cost Principle –
This principle suggests that an ideal capital structure is one that minimizes cost of capital structure and maximizes earning
per share.
(ii) Risk Principle –
This principle suggests using more proportion of common equity for financing requirement. Use of more and more debts
means higher commitment in form of interest payout. This would lead to erosion of shareholders value in unfavorable
business situation.
(iii) Control Principle –
This principle suggests to consider interest of maintain existing management and operational control over the company.
Management may wish to have control undisturbed.
(iv) Flexibility Principle –
This principle gives flexibility to the management who decides such a combination of sources of financing which it finds
easier to adjust according to changes in need of funds in future too.
(v) Other considerations –
Besides above principles, other factors such as regulatory requirements, nature of industry, timing of issue and
competition in the industry should also be considered.

6. Write short notes on: (4×2.5=10)


a) Profitability Index
b) Treasury Bills
c) Limitations of profit maximization objective of Financial Management
d) Capital Rationing
Answer
a) Profitability index is an investment appraisal technique calculated by dividing the present value of future cash flows of a
project by the initial investment required for the project.
Profitability Index is calculated as follows:
Present Value of Future Cash Flows
PI =
Initial Investment Required
Profitability index is actually a modification of the net present value method. While present value is an absolute measure (i.e. it
gives as the total figure for a project), the profitability index is a relative measure (i.e. it gives as the figure as a ratio).

The decision rule is to accept a project if the profitability index is greater than 1, stay indifferent if the profitability index is
zero and don't accept a project if the profitability index is below 1.

Profitability index is sometimes called benefit-cost ratio too and is useful in capital rationing since it helps in ranking projects
based on their per dollar return.

b) Treasury bills are obligation of the government. They are sold on discounted basis. The investor does not receive an actual
interest payment. The return is the difference between the purchase price and the face (par) value of the bill.

The treasury bills are issued only in bearer form. They are purchased, therefore, without investors' name upon them. This
makes them easily transferable from one investor to another. A very active secondary market exists for these bills. The
secondary market for bills not only makes them highly liquid but also allows purchase of bills with very short maturities. As
the bills have the full financial backing of the government, they are, for all practical purposes, risk-free. The negligible
financial risk and the high degree of liquidity make their yield lower than those on the other marketable securities. Due to their
virtually risk free nature and because of active secondary market for them, treasury bills are one of the most popular marketable
securities even though the yield on them is lower.

c) Profit maximization objective of financial management has following limitations:


¾ It ignores time factor.
¾ It ignores the risk factors.
¾ It focuses on short term profits and overlooks long term vision.
¾ This is vague because it is not clear whether the term relates to economic profit, accounting profit, profit after tax or
before tax.
¾ The term profit is also ambiguous.

d) Capital Rationing is the process hereby the limited funds available are allocated amongst the financially viable projects which
are not mutually exclusive under consideration so as to maximize the wealth of the shareholders. Thus, capital rationing
situation is said to exist if:
i. Limited funds are available for investment.
ii. More than one financially viable projected which are not mutually exclusive are under consideration.

The Institute of Chartered Accountants of Nepal 34


Suggested Answers - Financial Management CAP-II Examination - December 2014

7. Distinguish between: (4×2.5=10)


a) Capital structure and Financial structure b) Clean packing credit and Packing credit against hypothecation of goods
c) Capital market and Money market d) Systematic risk and Unsystematic risk
Answer
a) Capital Structure is the permanent long term financing of the company including Long term debt, equity capital, preferential
shares and retained earnings is called capital structure. It can be also termed as a mix of long term finances used by the
company.It is the financing plan of the company. It differs from Financial Structure which includes short term debt and
accounts payable also.
Whereas, financial Structure entails the ways the assets of the companies are financed such as trade accounts payable, short
terms borrowings as well as long term borrowings and ownership equity. Financial structure is distinguished from capital
structure where only long term debt and equity are included. A company’s financial structure is influenced by many factors
such as growth rate, stability of sales. It is the basic frame of references for analysis concerned with financial leveraging
decisions.
b) Packing Credit is an advance extended by banks to an exporter for the purpose of buying, manufacturing, processing, packing,
shipping goods to overseas. If an exporter has a firm export order placed with him by his foreign customer (buyer) or all
irrevocable Letter of Credit in his favour, he can approach a Bank for Packing Credit Facility.
Clean Packing Credit
• This facility is extended only on production of a firm export order or a letter of credit.
• There is no charge or control over raw material or finishes goods that constitute the supply.
• The bank takes into consideration trade requirements, credit worthiness of exporter and its margin.
• Export Credit Guarantee Corporation (ECGC) insurance cover should be obtained by the bank.
Packing credit against hypothecation of goods
• This facility is extended only on production of a firm export order or a letter of credit.
• The goods which constitute the supply are hypothecated to the Bank as security with stipulated margin.
• The goods shall be exported by the borrower. The Bank does not have any effective possession of the same.
• The exporter has to submit stock statements at the time of sanction and also periodically and for whenever there is any
movement in stock.

c) Capital market and money market are two basic components of financial system. Capital market deals with long and medium
term instruments of financing while money market deals with short term instruments. Capital market instruments are shares,
debentures, mutual funds etc. while money market instruments are interbank placement, call money, commercial papers,
treasury bills etc.
Capital market is usually classified as primary market and secondary market while there is no such classification of money
market.
Capital market participants include retail investors, institutional investors, financial institutions, corporate houses and banks
while money market participants include banks, financial institutions, central bank and government.
d)
Systematic Risk Unsystematic Risk
A part of the risk that arises on account of the A part of the risk that arises from uncertainties which are
economy-wide uncertainties and the tendency of unique to individual securities, and which are diversifiable if
individual securities to move together with changes large number of securities is combined to form well-
in the market. This part of risk cannot be reduced diversified portfolios. The unique risks of individual securities
through diversification, and it is called systematic or in a portfolio cancel out each other. This part of the risk can
market risk. be totally reduced through diversification and is called
unsystematic or unique risk.
Investors are exposed to market risk even when they
hold well-diversified portfolios of the securities.
Examples of systematic risk are: Examples of unsystematic risk are:
• The Government changes the interest rate • workers declare strike in a company
policy • the R&D expert of the company leaves
• The corporate tax rate is increased • a formidable competitor enters the market
• The Government resorts to massive deficit • the company loses a big contract in a bid
financing • the company makes a breakthrough in process innovation
• The inflation rate increases • the Government increases custom duty on the material used
• The Nepal Rastra bank promulgates a by the company
restrictive credit policy • the company is not able to obtain adequate quantity of raw
materials from the suppliers.

The Institute of Chartered Accountants of Nepal 35


Suggested Answers - Cost & Management Accounting CAP-II Examination - December 2014

Cost & Management Accounting

1. A company markets products X and Y which it makes by using its capacity to the extent of 50% on X and 30% on Y. Budget for
2014 is as given below:
X Y
Production units 5,000 4,500
Direct materials/unit Rs. 300 Rs. 200
Conversion cost/unit
Variable Rs. 100 Rs. 80
Fixed Rs. 50 Rs. 40

Selling price per unit Rs. 500 Rs. 350


Profit per unit Rs. 50 Rs. 30

For the next year's budget, the following factors are relevant:
ƒ Direct Material cost will go up by 6%
ƒ Variable conversion cost will increase by 10%
ƒ Selling price of: X will be increased by 4% and Y will be increased by 6%

To utilize the idle capacity of 20%, three proposals as under were put forth:

a) Produce X and sell the output at the revised price. Production of X from the idle however will be less by 10%
b) Produce Y but the increased production will be sold at the existing price
c) Utilize the idle capacity to produce a new product Z whose details are as under:

ƒ Production form idle capacity: 2,000 units


ƒ Direct Materials: Rs.400 per unit
ƒ Variable conversion cost: Rs.200 per unit
ƒ Selling price: Rs.700 per unit
ƒ Special publicity Expense: Rs.20,000
ƒ The present allocation of 50% and 30% respectively on products X and Y cannot be changed.
You are required to prepare Statements of Profitability for 2014 and 2015. 20

Answer:

Statement showing the profitability of the products for 2014 as per budget production (units)
X= 5,000 and Y=4,500
Details Per unit Total Total
X Y X Y
1.Selling Price 500 350 2,500,000 1,575,000 4,075,000
2.Costs:
Direct Material 300 200 1,500,000 900,000 2,400,000
Conversion Cost:
Variable 100 80 500,000 360,000 860,000
Fixed 50 40 250,000 180,000 430,000
3.Total of 2 450 320 2,250,000 1,440,000 3,690,000
4.Profit (1-3) 50 30 250,000 135,000 385,000

a) Total contribution if Product X is produced


Production of X at 50% Capacity 5,000 units
Production of X at 20% Capacity 2,000 units
Less: 10% of this production 2,00 units
Net production of X if idle capacity is utilized 1800 units

The Institute of Chartered Accountants of Nepal 36


Suggested Answers - Cost & Management Accounting CAP-II Examination - December 2014

Existing Increase Revised


(%) (Per unit)
Selling price 500 4% 520
Direct Materials 300 6% 318
Conversion Costs –variable 100 10% 110
Total Variable Cost 428
Contribution per unit 92
Total Contribution (1,800* Rs.92) Rs.165,600

b) Total contribution if Product Y is produced


Production of Y at 30% Capacity 4,500 units
Production of Y at 20% Capacity 3,000 units

Existing Increase Revised


(%) (Per unit)
Selling price 350 350
Direct Materials 200 6% 212
Conversion Costs –variable 80 10% 88
Total Variable Cost 300
Contribution per unit 50
Total Contribution (3,000* Rs.50) Rs.150,000

c) Total contribution if a new product Z is produced


(Per unit)
Selling price Rs.700
Direct Materials Rs.400
Variable conversion cost Rs.200
Total Variable Cost Rs.600

Contribution per unit Rs.100

Total contribution ( 2,000 units * Rs.100) Rs.200,000


Less: Special publicity expense Rs.20,000

Net Total Contribution Rs.180,000

This proposal should be accepted as the total contribution is highest when product Z is produced.

It is given in the proposal ( b) above that if product Y is produced in the idle time, then additional production will be sold at the
existing prices. Since proposal (c) is recommended, the revised selling price of product Y will be:

Existing selling price per unit Rs.350


Add: 6% increase in price Rs.21
Revised price of product Y Rs.371

Statement showing the profitability of three products for 2015 as per revised budget
production (units) X= 5,000, Y=4,500 and Z=2,000
Details Per unit Total Total
X Y Z X Y Z
1.Selling Price 520 371 700 2,600,000 1,669,500 1,400,000 5,669,500
Direct Material 318 212 400 1,590,000 954,000 800,000 3,344,000
Conversion Cost- 110 88 200 550,000 396,000 400,000 1,346,000
variable
Total V. costs 428 300 600 2,140,000 1,350,000 1,200,000 4,690,000
Contribution 92 71 100 460,000 319,500 200,000
Total Contribution 979,500
Less: Fixed Cost 430,000
Publicity for Z 20,000
Net Profit 529,500

The Institute of Chartered Accountants of Nepal 37


Suggested Answers - Cost & Management Accounting CAP-II Examination - December 2014

2.
a) The following information is available from the financial books of a company having a normal production capacity of 60,000
units for the year ended 31st March, 2014:

• Sales Rs. 10,00,000 ( 50,000 units)


• There was no opening and closing stock of finished units.
• Direct Material and direct wages cost were Rs. 5,00,000 and Rs. 2,50,000 respectively.
• Actual factory expenses were Rs. 1,50,000 of which 60% are fixed.
• Actual administrative expenses were Rs. 45,000 which are completely fixed.
• Actual selling and distribution expenses were Rs. 30,000 of which 40% are fixed.
• Interest and dividends received Rs. 15,000.

You are required to: (3+4+3=10)


st
i) Find out profit as per financial books for the year ended 31 March, 2014.
ii) Prepare the cost sheet and ascertain the profit as per cost accounts for the year ended 31st March 2014 assuming that the
indirect expenses are absorbed on the basis of normal production capacity; and
iii) Prepare a statement reconciling profit shown by financial and cost books.

b) Shakti Engineers are engaged in construction and erection of a bridge under along-term contract. The cost incurred up to
31.03.2014 was as under:

Fabrication Rs.InLakhs
Direct Material 280
Direct Labour 100
Overheads 60
440
Erection cost to date 110
550
The contract price is Rs.11 crores and the cash received on account till 31.03.2014 was Rs.6 crores.
The technical estimate of the contract indicates the following degree of completion of work. Fabrication: Direct
Material 70%, Direct Labour and Overheads 60% Erection 40%.
You are required to estimate the profit that could be taken to Profit and Loss Account against this partly completed contract
as at 31.03.2014. 10

Answer 2 (a) (i)


As per Financial Books
Profit and Loss Account
(for the year ended 31st March 2014)

To Direct Material Rs. 5,00,000 By Sales ( 50,000 units) Rs. 10,00,000


To Direct Wages 2,50,000 By Interest and dividend 15,000
To Factory Expenses ( Actual) 1,50,000
To Admn. Exenses 45,000
To Selling and Distribution 30,000
To Profit 40,000 _____________
10,15,000 10,15,000
As per above account, profit is Rs. 40,000 for the year ended 31st March, 2014.

Answer 2 (a) (ii)


Cost Sheet
( for the year ended 31st March, 2014)
_________________________________________________________________________________
Normal Production capacity (units) 60,000
Sales/ Production (units) 50,000

Direct material Rs. 5,00,000


Direct wages 2,50,000
Prime cost 7,50,000
Factory overhead - Variable Rs. 60,000
- Fixed Rs. 90,000 X 5/6 75,000 1,35,000

The Institute of Chartered Accountants of Nepal 38


Suggested Answers - Cost & Management Accounting CAP-II Examination - December 2014

Works cost 8,85,000


Administrative expenses Rs. 45,000 X 5/6 37,500
Total cost of production 9,22,500
Selling and distribution expenses
- Variable Rs. 18,000
- Fixed Rs. 12,000 X 5/6 10000 28,000
Cost of Sales 9,50,500
Profit (balance) 49,500
Sales 10,00,000

Answer 2 (a) (iii)


Reconciliation Statement
Profit as per Cost Accounts Rs. 49,500
Add: Income from dividend ( not considered in Cost Accounts) 15,000 64,500

Less: Expenses undercharged in Cost Accounts:


i) Factory expenses ( 1,50,000-1,35,000) 15,000
ii) Adm. Expenses ( 45,000-37,500) 7,500
iii) Selling & Distribution ( 30,000-28,000) 2,000 24500
Profit as per financial accounts 40,000

Answer 2 (b)
EstimationofProfittobetakentoProfitandLossAccountagainstpartlycompletedcontractasat31.03.2014.
Profit to be taken to P/L Account= 2/3 X Notional profit X Cash received
Work certified
(Refer to working notes 1,2,3,&4)
= 2/3 X Rs. 92.48 lakhs X Rs. 600 lakhs
Rs. 642.48 lakhs
= Rs. 57.576 lakhs
Working Notes:
1. Statement showing estimated profit to date and future profit on the completion of contract

Particulars Cost to date Further Cost Total Cost

% Amount % Amount
Completion Rs. Completion Rs. Rs.
To date (a) to be done (b) (a)+(b)
Direct Material 70 280.00 30 120.00 400.00
Directlabour 60 100.00 40 66.67 166.67
Overheads 60 60.00 40 40.00 100.00
TotalFabricationcost(A) 440.00 226.67 666.67
Erectioncost:(B) 40 110.00 60 165.00 275.00
Totalestimatedcosts:(A+B) 550.00 391.67 941.67
Profit(Refertoworkingnote2) 92.48 65.85 158.33
642.48 457.52 1,100.00

2. Profit to date (notional profit) and future are calculated as below:


Profit to date (notional profit) = Estimated profit on whole contract X cost to date
Total Cost
= Rs. 158.33 XRs. 550
Rs. 941.67
= Rs. 92.48 ( Lakhs)

Future Profit = Rs. 158- Rs. 92.48


= Rs. 65.85

The Institute of Chartered Accountants of Nepal 39


Suggested Answers - Cost & Management Accounting CAP-II Examination - December 2014

3. Work certified
= cost of the contract to date + profit to date
= Rs. 550 + Rs. 92.49 = Rs. 642.48 lakhs

4. Degree of completion of contract to date


= Cost of the contract to date X 100
Contract Price
= Rs. 642.48 lakhs X 100
Rs. 1,100 lakhs
= 58.40%

3.
a) MTC Limited uses chemicalX in one of its finished products. The chemical-X is purchased from a vendor outside Nepal.
MTC Limited purchases 36,000 Ltr. of chemicalX per year at the rate of Rs. 900 per Ltr plus import duty @10% on such
purchases.

The chemicalX is used evenly throughout the year in the production process on a 360dayperyear basis. The Company incurs
Rs. 1,75,000 on one year agreement for material supply with the vendor and it estimates that Rs. 35,000 will be incurred to
place a single purchase order. The chemicalX is needed to be kept in a very carefully controlled temperature and humidity
conditions. MTC Ltd. Incurs 1.5% and 0.2676% of the value of inventory as storage cost and as insurance cost respectively.

Delivery from the vendor generally takes 12 days, but it can take as much as 16 days. The days of delivery time and
percentage of their occurrence are shown in the following tabulation:

Delivery time (days) : 12 13 14 15 16


Percentage of occurrence : 70 10 10 5 5
Required: (3+3+4=10)
i) Compute the economic order quantity (EOQ).
ii) Assume the company is willing to assume a 10% risk of being out of stock. What would be the safety stock? The re-
order point?
iii) Assume 5% stock-out risk. What would be the total cost of ordering and carrying inventory for one year?

b) Aryan Ltd. has three production department M,N and O and the two service department P and Q.
The following particulars are available for the month of September 2013:

(Rs.)
Lease rental 35,000
Power and Fuel 4,20,000
Wages to factory supervisor 6,400
Electricity 5,600
Depreciation on machinery 16,100
Depreciation on building 18,000
Payroll expenses 21,000
Canteen expenses 28,000
Provident fund contribution 58,000

Following are the further details available:

Particulars M N O P Q
Floor space (Sq.M) 1,200 1,000 1,600 400 800
Light Points(Nos) 42 52 32 18 16
Cost Of machine(Rs.) 12,00,000 10,00,000 14,00,000 4,00,000 6,00,000
No of Employees (Nos) 48 52 45 15 25
Direct wages (Rs.) 1,72,800 1,66,400 1,53,000 36,000 53,000
HP of Machines 150 180 120 - -
Working Hours(hours) 1,240 1,600 1,200 1,440 1,440

The Institute of Chartered Accountants of Nepal 40


Suggested Answers - Cost & Management Accounting CAP-II Examination - December 2014

The Expenses of service department are to be allocated in the following manner:

M N O P Q
P 30% 35% 25% - 10%
Q 40% 25% 20% 15% -

You are required to calculate the overhead absorption rate per hour in respect of the three production departments. 10
Answer
3 a)
(i) Economic Order Quantity (E.O.Q)
= √2AO = √2x36,000 Ltrs. xRs 35000 = 12,000 litres
C Rs 17.50
(ii) Safety Stock at 10% risk of being out of stock
Safety Stock required for two days i.e. for 13th and 14th day

Safety stock = 36,000ltr x 2days = 200 litres


360 days
Re-order Point = Minimum Stock level + Average lead time x Average consumption
= 200+12 x 100
= 1400 litres
(iii) At 5% risk of being out stock, safety stock will be safety stock for three days
100 ltr x 3 days =300 ltr.

Total Ordering Cost =36,000ltr x Rs 35,000 = Rs 1,05,000


12,000ltr

Total Carrying cost of inventory = (Safety Stock + Average inventory) Carrying Cost per litre per annum
= (300 + 1/2 x 12,000 ltr.) Rs 17.50
= Rs 1,10,250
Total cost of ordering & carrying inventory = 1,05,000 + 1,10,250 = Rs 2,15,250

Working Notes

1. Risk of being out of stock

Percentage of Occurrence Cumulative percentage of Risk of non Occurrence


Delivery Time
(%) Occurrence (%) (%)

12 days 70 70 30

13 days 10 80 20

14 days 10 90 10

15 days 5 95 5

16 days 5 100 0

2 (a) Ordering Cost per order (O)- Rs 35,000


(b) Cost per litre of chemical-X
Rate per litre Rs 900
Add: Import duty @10% Rs 90
Rs 990
Carrying cost per litre per annum of chemical-X (C)
1.7676% (1.5%+0.2676%) of Rs 990= Rs17.50
(Note: Amount of Rs. 175,000 incurred on making agreement for material supply will be apportioned over the entire quantity
of 36,000 ltr and included with cost of chemical X. However for the purpose of calculating carrying cost i.e storage cost and
insurance cost only invoice cost of material is taken. Invoice cost consist of cost per litre of chemical X plus import duty.)

The Institute of Chartered Accountants of Nepal 41


Suggested Answers - Cost & Management Accounting CAP-II Examination - December 2014

3(b)

Item of cost Basis of apportion ment Total Production Department Service Department
(Rs)
M N O P Q
(Rs) (Rs) (Rs) (Rs) (Rs)
Lease Rental Floor space 35,000 8,400 7,000 11,200 2,800 5,600
(6:5:8:2:4)
Power & Fuel HP of machines x 4,20,000 1,26,408 1,95,728 97,864 - -
Working hours
(93:144:72)
Supervisor’s Working hours 6,400 1,964 2,535 1,901 - -
wages* (31:40:30)
Electricity Light points 5,600 1,470 1,820 1,120 630 560
(21:26:16:9:8)
Depreciation Value of machinery 16,100 4,200 3,500 4,900 1,400 2,100
On Machinery (6:5:7:2:3)
Depreciation Floor space 18,000 4,320 3,600 5,760 1,440 2,880
on building (6:5:8:2:4)
Payroll No Of Employes 21,000 5,448 5,903 5,108 1,703 2,838
Expenses (48:52:45:15:25)
Canteen No Of Employes 28,000 7,625 7,870 6,811 2,270 3,784
expenses (48:52:45:15:25)
PF Direct wages 58,000 17,244 16,606 15,268 3,593 5,289
contribution (864:832:765:180:265)
Total 608,100 176,719 244,562 149,932 13,836 23,051

*wages to supervisor is to be distributed to production departments only.


Since the service department has incurred direct wages, it has also to be allocated to the production department. Therefore , the process of
allocation is as under.

P Q
Apportioned Overhead 13,836 23,051
Add: Allocated direct wages 36,000 53,000
49,836 76,051

P = 49,836+ 0.15 Q …………..(I)


Q = 76,051+ 0.10 P……………(II)
Substituting the value of Q in (I) we get
P = 49,836 + 0.15 (76,051+ 0.10 P)
P = 49,836 + 1140Q +0.015 P
P = 61244/0985
P = Rs 62177
And Q = 76,051+0.10 x 62,177
=Rs 82,269
Secondary Distribution summary
Particulars Total M N O
Allocated and apportioned overheads as per 5,71,213 1,76,719 2,44,562 1,49,932
primary distribution
Add: 90% of P(30%,35% & 25%) 55,959 18,653 21,762 15,544
Add: 85% of Q(40%,25% & 20%) 69,928 32,907 20,567 16,454
2,28,279 2,86,891 1,81,930

Overhead rate per hour


M N O
Total Overhead cost (Rs.) 2,28,279 2,86,891 1,81,930
Workings hours 1,240 1,600 1,200
Rate per hour (Rs.) 184.09 179.31 151.61

The Institute of Chartered Accountants of Nepal 42


Suggested Answers - Cost & Management Accounting CAP-II Examination - December 2014

4.
a) Pawanputra Air owns single jet aircraft and operates between Kathmandu and Delhi only. Flights leave Kathmandu on
Mondays and Thursdays and depart from Delhi on Wednesdays and Saturdays. Pawanputra Air cannot afford any more flights
between Kathmandu and Delhi. Only tourist class seats are available on its flights and all tickets are booked by travel agents.
The following informations are collected.

Seating capacity per plane 360


Average passengers per flight 200
Flights per week 4
Flights per year 208
Average one way fare Rs. 5,000
Variable fuel cost Rs. 1,40,000 per flight
Food service to passengers (not charged to Passengers) Rs. 200 per passenger
Commission to travel agents 8% of fare
Fixed annual lease cost allocated to each flight Rs. 5,30,000 per flight
Fixed ground services (maintenance, check in,
Baggage handling cost) allocated to each flight Rs. 70,000 per flight
Fixed salaries of flight crew allocated to each flight Rs. 40,000 per flight
For the sake of simplicity assume that fuel cost are unaffected by the actual number of passengers on a flight.

Required: (5+3+2=10)
i) What is the operating income that Pawanputra Air makes on each way flight between Kathmandu and Delhi?
ii) The market research department of Pawanputra Air indicates that lowering the average one way fare to Rs. 4,800 will
increase the average number of passenger per flight to 212. Should Pawanputra Air lower its fare?
iii) Gem Travels, a tour operator approaches Pawanputra Air to charter its jet aircrafttwice each month, first to take Gem’s
international tourists from Kathmandu to Delhi and then bring the tourists back from Delhi to Kathmandu. If Pawanputra
Air accepts the offer, it will be able to offer only 184 (208 less 24) of its own flights each year. The terms of the charter
are:

(a) For each one-way flight Gem will pay Pawanputra Rs. 7,50,000 to charter the plane and to use its flight crew and
ground service staff.
(b) Gem will pay for fuel costs.
(c) Gem will pay for all food costs.

On purely financial considerations, should Pawanputra Air accept the offer from Gem Travels?

b) Two fitters, a labourer and a boy undertake a job on piece rate for Rs.1,290. The time spent by each of them is 220 ordinary
working hours. The rates of pay on time-rate basis are Rs.1.50 per hour for each of the two fitters, Re. 1 per hour for the
labourer and Re. 0.50 per hour for the boy.

Required: (6+2=8)
i) The amount of piece-work premium and the share of each worker, when the piece-work premium is divided
proportionately to the time wages paid.

ii) The selling price of the above job on the basis of the following additional data:
Cost of direct material Rs.2,010
Works Overhead at 20% of Prime Cost
Selling Overhead at 10% of Work Cost and
Profit at 25% on Cost of Sales.
c) Write short notes on the advantages of time rate remuneration plans. 2

The Institute of Chartered Accountants of Nepal 43


Suggested Answers - Cost & Management Accounting CAP-II Examination - December 2014

Answer 4(a)

i) Statement of operating income of Pawanputra Air for Kathmandu-Delhi flight (one way)

Fare received (per flight): 200 passengers x Rs. 5,000 Rs.10,00,000


Variable costs (per flight)
Fuel cost Rs. 1,40,000
Food (200 x Rs. 200) 40,000
Commission to Travel Agents (8% of Rs. 10,00,000) 80,000 Rs. 2,60,000
Contribution per flight Rs.7,40,000

Fixed cost (per flight)


Annual lease cost Rs. 5,30,000
Fixed ground service costs 70,000
Salaries of flight crew 40,000 Rs. 6,40,000
Operating income per flight 1,00,000

ii) Fare received (per flight): 212 passengers x Rs. 4,800 Rs.10,17,600
Variable costs (per flight)
Fuel cost Rs. 1,40,000
Food (212 x Rs. 200) 42,400
Commission to Travel Agents (8% of Rs. 10,17,600) 81,408 Rs. 2,63,808
Contribution per flight Rs.7,53,792

Excess contribution due to lowering of fare (Rs. 7,53,792 – Rs. 7,40,000) = Rs. 13,792
Pawanputra Air should lower its fare as it would increase its contribution by Rs. 13,792

iii) For financial considerations to decide whether Pawanputra Air should charter its plane to Gem Travels most profitable
option i.e. option (b) should be used.

Contribution of Pawanputra Air under option (b) Rs. 7,53,792


Pawanputra Air would get (per flight) for charter of plane Rs. 7,50,000

As there is a loss of Rs. 3,792 per flight Pawanputra Air should not accept the offer.
4 (b)

(i) Calculation of wages:


2 Fitters @ Rs. 1.50 per hour for 220hours each Rs. 660
1 Labourers @ Re. 1 per hour for 220 hours Rs. 220
1 Boy @ Re. 0.50 per hour for 220 hours Rs. 110
Total Rs. 990

Piece Work Premium


Total Wages agreed on piece-rate basis Rs. 1,290
Less: Wages calculated on time basis Rs. 990
Piece Work Premium Rs. 300

Amount of premium will be paid to workers in the ratio of 660:220:110 (or 6:2:1) as follows:
2 Fitters Rs.200.00
1 Labourer Rs. 66.67
1 Boy Rs. 33.33
Total Rs. 300.00

(ii) Computation of Selling Price: Rs.


Direct Material 2,010
Direct Wages 1,290
Prime Cost 3,300
Work Overheads at 20% in Prime Cost 660
Work Cost 3,960
Selling Expenses at 10% in Work Cost 396
Cost of Sales 4,356
Add: Profit at 25% on Cost of Sales 1,089
Selling Price 5,445

The Institute of Chartered Accountants of Nepal 44


Suggested Answers - Cost & Management Accounting CAP-II Examination - December 2014

4(c) The advantages of time rate remuneration plans are as follows:


(i) It is commonly recognized by all trade unions as well as worker .
(ii) It is a guaranteed income assured to the worker.
(iii) It is very easy to understand and simple to calculate the earnings of worker.
(iv) It involves less clerical work and detailed records are not necessary.
(v) Since the production is not the criteria for calculation of wages, tools and materials are handled carefully. Wastage is also
minimized.

5. Distinguish between: (4×2.5=10)


a) Controllable costs and Uncontrollable costs
b) Indifference point and Break-even Point
c) Production account and Cost Sheet
d) Bill of material and Material requisition note

Answer5(a)

Controllable costs and Uncontrollable costs

Controllable costs are the costs which can be influenced by the action of a specified member of an undertaking. Controllable costs
incurred in a particular responsibility centre can be influenced by the action of the executive heading that responsibility centre.

Uncontrollable costs are those costs which cannot be influenced by the action of a specified member of an undertaking. The
distinction between controllable and uncontrollable costs is not very sharp and is sometimes left to individual judgement.In Fact, no
cost is controllable; it is only in relation to a particular individual that we may specify a particular cost to be either controllable or
non-controllable.

Answer 5 (b)

Particulars Indifference Point Break-Even Point


Definition Indifference Point is the level of Sales at which BEP is the level of sales at which there isneither
Total costs and Profits of two options are equal. a Profit nor a Loss to thefirm. At BEP,
the total Contribution equals Fixed Cost.
Formula Indifference Point (in Rs.) =Difference in Fixed Break Even Point (in Rs.) =
CostDifference in Var. Cost ratio or PV ratio Fixed Cost
PV ratio

Significance It is the activity level at which Total Cost under It is the activity level at which the Total Revenue
two alternatives are equal. from a product mix is equal to its Total cost.
Purpose Used to choose between two alternative Used for profit planning.
options for achieving the same objective.

Answer 5(c)
The following are the points of difference between a Production Account and a Cost Sheet.
i) Production Account is based on double entry system whereas cost sheet is not based on double entry system.
ii) Production Account consists of two parts. The first part shows cost of the components and total productioncost. The second
part shows the cost of sales and profit for theperiod. Cost sheet presents the elements ofcosts in a classified manner and the cost
is ascertained at different stages such as prime cost; works cost ofproduction; cost of goods sold; cost of sales and total cost.
i) Production account shows the cost in aggregate and thus facilitates comparison with other financial accounts. Cost sheet shows
the cost in detail and analytical manner which facilitates comparison of costfor the purpose of cost control.
ii) Production account is not useful for preparing tenders or quotations. Estimated cost sheets can beprepared on the basis of actual
costs sheets and these are useful for preparing tenders or quotations.

The Institute of Chartered Accountants of Nepal 45


Suggested Answers - Cost & Management Accounting CAP-II Examination - December 2014

Answer 5(d)
Bill of material Material requisition note
It is a comprehensive list of materials with It is a formal written demand or request, usuallyfrom
exactdescription and specifications,required for a jobor the production department to store for thesupply of
other production units. This also providesinformation specified materials, stores etc. Itauthorises the
about required quantities so that ifthere is any deviation storekeeper to issue therequisitioned materials and
from the standards, it caneasily be detected. It is record the same onbin card.
prepared by theEngineeringor Planning Department in a
standard form.
The purpose of bill of material is to act as a The purpose of material requisition note is to
singleauthorisation for the issue of all materials drawmaterial from the store byconcerned departments.
andstores items mentioned in it. It provides anadvance
intimation to store department about therequirements of
materials. It reduces paper work.It serves as a work order
to the productiondepartment and a document for
computing thecost of material for a particular job or
work orderto the cost department.

6. Answer the following questions: (4×2.5=10)


a) Discuss the treatment of research and development expenditures in cost accounting.
b) The more kilometers you travel with your own vehicle, the cheaper it becomes. “Comment briefly on this statement”.
c) Discuss the accounting treatment for spoilage and defectives in cost-accounts
d) Enumerate the remedial steps to be taken to minimize the labour turnover.
Answer 6(a)
If research is conducted in the methods of production, the expenses should be charged to production overhead. If the research
relates to administration, the expenses arecharged to administration overheads. If it is related to market research, the expenses are
charged to S & D overheads. Development costs incurred in connection with a particular product should be charged directly to that
product. Such expenses are usually treated as deferred revenue expenditure and recovered as cost per unit of the product when
production is fully established. Routine nature research expenses are charged to general overheads.

Answer 6(b)
The cost per kilometre, (if one travels in his own vehicle) will decline when he travels more kilometers. This is because the
majority of costs for running and maintaining vehicles are of fixed and the component of fixed cost per kilometre goes on
decreasing with an increase in kilometre travel. Hence, the given statement is true.

Answer 6(c)
Normal spoilage (i.e. which is inherent in the operation) costs are included in cost either by charging the loss due to spoilage to the
production order or by charging it to production overhead so that it is spread over all the products. Any value realized from the sale
of spoilage is credited to production order or production overhead account, as the case may be. The cost of abnormal spoilage are
charged to Costing Profit & Loss Account.
Defectives that are considered inherent in the process and are identified as normal can be recovered by using any one of the
following method.
• Charged to good products
• Charged to general overheads
• Charged to departmental overheads.
If defectives are abnormal, they are to be debited to Costing Profit & Loss Account.

Answer 6(d)
The following steps are useful for minimizing labour turnover:
¾ Exit interview: An interview be arranged with each outgoing employee to ascertain the reasons of his leaving the organization.
¾ Job analysis and evaluation: to ascertain the requirement of each job.
¾ Organisation should make use of a scientific system of recruitment, placement and promotion for employees.
¾ Organisation should create healthy atmosphere, providing education, medical and housing facilities for workers.
¾ Committee for settling workers grievances.

The Institute of Chartered Accountants of Nepal 46


Suggested Answers - Business Communication CAP-II Examination - December 2014

Business Communication
Section -'A'

1. Read the following case carefully and answer the questions given below: (3.5+3.5+3=10)
Anand Roy, an officer at Research Division in Real Drinks Co. Ltd. is asked to prepare a report after conducting a nation-wide
survey about the sales condition of the products of the company. The Sales Manager has instructed Mr Roy to conduct the research
with a key intention of increasing the sales of ‘Real Drink’ in the coming summer. Other general aspects of the survey will be some
of those including the ways of increasing the demand of the product, improvements to be brought, desires of costumers, cost,
quantity, and marketing strategies, and so on. The survey should include at least ten thousand respondents across the country with
one thousand in each Development Region. Mr. Roy has to present some clear findings and recommendations in the report, and he
has now kept himself busy in developing research tools, sampling procedures, research design, analyzing measures, rating scales,
action plans, time table, etc.

Questions:
a) Choose a particular area of research, for example, marketing strategy, costumer desires, costumer perceptions, etc., and prepare
a set of questionnaire based on the chosen area in order to elicit information for the survey. The questionnaire should include at
least seven questions.
b) What could be three major objectives of the survey/report?
c) Prepare a timetable for writing the report that is required by the sales manager of the company.
Answer
a) Questionnaire on costumer perception on Real Drink(RD)
(To be asked to the costumers)
Questions Yes! Yes No No! Remarks
Do you love the flavor of Real Drink?
Do your friends love RD?
Do you prefer RD to other drinks?
Do you think quality of RD pays its cost?
Is chilled RD better than normal one?
Do you find RD in the parties too?
Can you imagine your lunch without RD?

b) The objectives of the study are:

i) To find out the ways of increasing the sales throughout the country,
ii) To find out the perception of consumers about the quality of the product,
iii) To find out the efficiency of marketing strategies of the company.

c) Time Schedule for the Survey


Actions/Activities Time allocated
Research tool design and planning 1 week
Survey procedure(sampling and data collection ) 4 weeks
Analysis and interpretation of data 2 weeks
Writing report 1 week
Submission after 8 weeks

2. As a manager of Express Welfare Organization, you are going to organize a blood donation program which is the part of company’s
annual activity. Since the people might become reluctant to participate in the program and donate the blood, you need to convince
them highlighting its importance and overcoming their possible objections. Now, write a persuasive letter to distribute the possible
donors in and outside the organization. Follow the techniques of writing persuasive message. 10

The Institute of Chartered Accountants of Nepal 47


Suggested Answers - Business Communication CAP-II Examination - December 2014

Answer
Express Welfare Organization
Baneshwor, Kathmandu

November 23, 2014

Dear Mr.BinayaJha

We are pleased to let you know that we are going to organize a blood donation program. It’s one of the annual activities of our
company. Many people die each year due to the lack of blood in their treatment. In this regard, I would like to present a case in
which we contributed to saving life through blood donation. A few years ago, an employee of Express Welfare was driving to a
friend’s wedding when an oncoming car, operated by a drunk driver, swerved across the center line. Brad doesn’t remember the
crash. But he does remember two months spent in the hospital, two months of surgery and therapy.

Without the help of people like us, Saurav would not have lived. Some Express Welfare employees save lives regularly. We’re blood
donors. Please be a lifesaver and join us on Friday, March 19th, for Express Welfare’s annual blood drive.

Your help is needed for a successful drive.

Giving blood is simple. The entire process will take less than 45 minutes.

Giving blood is safe. Experienced health professionals from the Red Cross Society will be on-site to conduct the procedure exactly
as they would in a clinic setting.

Giving blood is convenient. The Red Cross staff will be in Room 401, Building B, between 9:00 A.M. and 3:00 P.M. To save time,
make an appointment to donate. Call the Red Cross Blood Center at 569-1170.

Giving blood is important. Nobody knows who will need blood next, but one thing is certain—it will be available only if healthy,
caring people take time to give it. Saurav’s accident required 110 units—more than 12 gallons—of blood. Because 110 people set
aside 45 minutes, Saurav Nepal has a lifetime of minutes to be grateful.

Take a few moments now to make your pledge on the reverse side of this letter. Then return it to the Community Relations
Department, Mail Location 12, by March 15th. For more information about the drive, call the Red Cross Society at 552-7116.

From Saurav and from other families—like yours and mine—who might need it in the days to come,

Sincerely,

HemantaShrestha
Manager

3. Why are meeting minutes important? What should be done while minuting? (4+6=10)
Answer:
It is important to capture the essence of the meeting, including details such as:
- decisions made (motions made, votes, etc.)
- next steps planned
- identification and tracking of action items

Minutes are a tangible record of the meeting for its participants and a source of information for members who were unable to attend.
In some cases, meeting minutes can act as a reference point, for example:
- when a meeting’s outcomes impact other collaborative activities or projects within the organization
- minutes can serve to notify (or remind) individuals of tasks assigned to them and/or timelines

The Institute of Chartered Accountants of Nepal 48


Suggested Answers - Business Communication CAP-II Examination - December 2014

Tips for record taking


Generally, meeting minutes usually include the following:
- Date and time of the meeting
- Names of the meeting participants and those unable to attend (e.g., “regrets”)
- Acceptance or corrections/amendments to previous meeting minutes
- Decisions made about each agenda item, for example:
o Actions taken or agreed to be taken
o Next steps
o Voting outcomes
o Motions taken or rejected
o Items to be held over
o New business
o Next meeting date and time

Tips for minuting:


Create an outline – having an outline (or template) based on the agenda makes it easy for you to simply jot down notes, decisions,
etc. under each item as you go along. If you are taking notes by hand, consider including space below each item on your outline for
your hand-written notes, then print these out and use this to capture minutes.
Check-off attendees as they enter the room - if you know the meeting attendees, you can check them off as they arrive, if not have
folks introduce themselves at the start of the meeting or circulate an attendance list they can check-off themselves.
Record decisions or notes on action items in your outline as soon as they occur to be sure they are recorded accurately
Ask for clarification if necessary – e.g., if the group moves on without making a decision or an obvious conclusion, ask for
clarification of the decision and/or next steps involved.
Don’t try to capture it all – you can’t keep up if you try to write down the whole conversation, so be sure to simply and clearly
write or type just the decisions, assignments, action steps, etc.
Record it – literally, if you are concerned about being able to keep up with note taking, consider recording the meeting (e.g., on your
smart phone, iPad, recording device, etc.) but be sure to let participants know they are being recorded and it can come in handy if
you need clarification.

4. What would you include while writing a persuasive resume or CV? 10


Answer:
Writing a persuasive resume is well worth the effort. Regardless of the job market, it is important to have a persuasive and well
written curriculum vitae, or CV, also known as a resume. The focus of a CV is experience, skills and achievements. Employers hire
people who are qualified and demonstrate an understanding of their business.

The following points should be considered focusing on the important components to be included in a persuasive CV.
- Remove redundant information from your resume. Do not mention past job functions that do not highlight skills or point out
accomplishments relevant to the job you are applying to.
- Tailor the CV to the advertised position. Carefully review the advertisement and show how your skills and experiences fit the
job requirements.
- Use the same keywords and terminologies that appear in the job description. The resume should demonstrate that the applicant
understands the employer's business and industry. Some resumes are scanned by computers: having matching keywords may
stop your resume from getting filtered out before an actual person has had a chance to see it.
- Select a simple design for your CV. The length will depend on your education and work experience, but keep it brief. Hiring
managers may be reviewing dozens of resumes, so they will not have time to read through a CV that reads like an essay.
- Set yourself apart. Mention unique experiences or unusual skills that may not be related to the position. Mention that you speak
a foreign language and that you have traveled extensively. Talk about your community activities, such as coaching the school
football team or volunteering at the local food bank.
- Summarize your qualifications for the position advertised. Use a brief statement of your experience and skills as the headline for
your resume.
- Avoid applying for positions that require skills that you don't possess. Your resume will come across as unpersuasive. Even if
you get an interview, it may turn out to be a waste of valuable time.
- Keep in mind the main purpose of the resume. The resume is not intended to be a complete biography. Its main purpose is to get
you an interview. Remember that persuasion requires clarity and precision.

The Institute of Chartered Accountants of Nepal 49


Suggested Answers - Business Communication CAP-II Examination - December 2014

5. Briefly explain any FOUR of the following: (4×2.5=10)


a) Kinesics
b) Overcoming communication barriers
c) Interpersonal communication
d) Pros and cons of workforce diversity
e) Techniques to close the job interview
Answer:
a) Kinesics
Kinesics is one of the important types of nonverbal communication. When communication takes place with the help of the
movement of the body parts such as head, hands, face, etc., it is known as kinesics. The eyes, fingers, legs, body postures, etc.
also have very important role to convey very specific meaning in a very specific context. Kinesics can be more effective than
the linguistic tool such as words and utterances in some specific context of communication. For example we can communicate
by nodding our head, blinking eyes, waving hands, and so on. Such movements can express more effectively our strong
feelings, intentions, and affective conditions.

b) Overcoming communication barriers


Communication systems may face a number of challenges such as interruption, interference, lack of knowledge in the
participants, etc. These obstacles and challenges are called communication barriers. These are mainly of two types: physical and
mental. Physical barriers are related to noises and physical disturbances. Mental barriers are related to the lack of linguistic as
well as cultural knowledge, emotions, etc. communication barriers need to be overcome because they may result in
organizational failure. In order to overcome such communication barriers the following strategies may be useful.
• Using familiar words and terms: Start with the familiar words to introduce a new idea. As inception of a new idea causes
resistance, the safest way to discuss and accept any new information is to begin with what is known. Start from what it is to
what it could be.
• Understanding traits of cross-cultural communication traits
• Understanding perceptions and conceptions of others
• Using listener-oriented viewpoints
• Avoidance of ambiguous words, jargons, slang and semantic words

c) Interpersonal communication
Interpersonal communication is the sending and receiving of messages between two or more people. Everyone communicates in
life and business, however, various barriers can exist that distort the message. Individuals must be able to identify the barriers
and find ways to eliminate them in a way that improves the communication flow. Learning a few techniques or using proper
preparation can help individuals tailor their communication in a way that provides focus and improves the message.

Effective interpersonal communication is an ongoing process. As new messages come up or audiences change, senders must
tailor their messages to ensure effective communication. Failing to observe the barriers in communication can result in more
time clarifying the message than creating it. Senders will often spend more time correcting the message, which can result in
more time spent on a basic message.

d) Pros and cons of workforce diversity


Having a diverse workforce can benefit consumers, work teams, and business organizations. However, diversity can also cause
divisiveness among identity groups. Business communicators should be aware of and sensitive to differences in the
communication techniques of men and women. To promote harmony and communication in diverse workplaces, many
organizations develop diversity training programs. You must understand and accept the value of differences. Don’t expect
conformity, and create zero tolerance for bias and prejudice. Learn about your cultural self, make fewer assumptions, and seek
common ground when disagreements arise.

e) Techniques to close the job interview


Once the interview nears conclusion, start thinking about how to end on a positive note. It is easy to become flustered after a
challenging interview, so as an interviewee one need to be sure to practice questions that you plan to ask. Also, focus on how to
leave a lasting positive impression.
• Asking your own questions: Be prepared with meaningful, thoughtful questions to help you determine whether this job is
right for you.
• Ending the interview positively: Summarize your strongest qualifications, show your enthusiasm for the job, and thank the
interviewer. Ask for the interviewer’s business card. Shake hands, and acknowledge anyone else on the way out.

The Institute of Chartered Accountants of Nepal 50


Suggested Answers - Marketing CAP-II Examination - December 2014

Marketing

Section -'B'

1. Read the following case and answer the questions given below:
According to a report published in one of the popular daily newspapers of Nepal on September 13, 2011, Government lab test
confirmed contamination in Gudpak, a popular sweet produced by Corner TajaGudpakBhandar and Shree Krishna Gudpak at New
Road along with Kanhaiya Brand Ghee.
The Department of Commerce had sent samples of the Gudpak and Kanhaiya Ghee to the Department of Food Technology and
Quality for tests. The report found thatKanhaiya Ghee was substandard and contained 16 percent fat against the needed 28 percent. It
was not safe to consume. It posed a risk to people’s health,The food technology department sent the report to the Kathmandu District
Administration Office (DAO).

The DAO assured strict action as per the Black Marketing and Some Other Social Offences Punishment Act against those found
guilty of producing low quality food.As per the Consumer protection Act, the government can send the offenders to 14 years in jail
and fine them up to Rs. 500,000.

Meanwhile, a monitoring team from the Kathmandu DAO and the food technology department inspected Pushpa Dairy. The
inspection team found the dairy using a broken filter system.Contaminated water was being mixed in the milk. The inspection team
destroyed some 1,000 litres of contaminated milk and ordered the firm to change the filter system, label the products properly and
not to use powdered milk until they come up with a lab report.
Questions:

a) Which marketing concept was applied by the firm and why? 5


b) What should be the responsibility of business community in society in the context of the above case? 5

Answer According to the situation given in the case, the marketing firms should adopt the societal marketing concept because the
conduct of marketing firms affect the whole society and its welfare.
The punishment decision taken by the government was appropriate, but it should create awareness among the business
community regarding their role in social welfare and well-being.It should not be a window dressing; it should be a continuous
process.
Since the final users of any product are consumers, they must be aware of the consequences of the use of product. Therefore,
before making buying decision, they must have adequate information about the products and their impacts.
The business community must understand the business ethics and should be responsive to the society. Only those communities
can sustain their business in this competitive world which are aware of social obligation and work for social welfare and well-
being.
2.
a) What do you know about selling concept of marketing? Explain its features. 5
b) What is marketing environment? Identify the components and features of macro environment. (2+3=5)
Answer a)
Selling concept holds that consumers and businesses, if left alone, will ordinarily not buy enough of the products without selling and
promotion effort. Therefore organizations must undertake aggressive selling and promotion efforts. The selling concept is practiced most
aggressively with unsought products, products that buyers normally do not think buying, such as insurance policy and encyclopedias.
Most firms practice the selling concept when they have overcapacity. Their aim is to sell what they make rather than make what the
market wants. Such marketing carries high risks. It focuses on creating sales transactions rather than on building profitable relationships
with customers. It assumes that consumers who are coaxed into buying the product will like it. These are usually poor assumptions to
make about buyers. Most studies show that dissatisfied customers do not buy it again. Whereas the satisfied customer tells three others
about good experiences, the average dissatisfied customer tells ten others about bad experiences.

The features of selling concept are


1. It satisfies seller's needs.
2. It aims to sell what it made.
3. Aggressive selling and promotion
4. Profit through high sales volume.

The Institute of Chartered Accountants of Nepal 51


Suggested Answers - Marketing CAP-II Examination - December 2014

b) Marketing environment encompasses all those surroundings or forces which are constantly changing and carry with them both
opportunities and threats. Macro environmental forces include the following components –
i. Economic Environment
ii. Demographic Environment
iii. Socio-Cultural Environment
iv. Technological Environment
v. Political and Legal Environment
vi. Natural Environment
vii. Competitive Environment
These forces have distinct features, which are as follow:
• These forces are external in nature and cannot be controlled or manipulated by the marketer when required;
• Forces in marketing environment are dynamic in nature and go on changing over time, such as habit of people, their needs
and wants, technology, demographic character of the people, politics, rules and regulations, competition, etc.
• These forces create opportunities as well as threats to the management.Without their consideration smooth functioning of
management is not possible.
• Marketing environmental forces are potentially relevant to the marketing decision-making.without the consideration of
marketing environment marketing program can neither be formulated nor implemented successfully.
• Marketing environment is the source of not only threats to the management but also the source of opportunities. A
marketer identifies business opportunities from among the marketing environmental forces.
3.
a) Give the meaning of marketing research and also point out the marketing research process. (2+3=5)
b) Describe the meaning and importance of market segmentation. (2+3=5)
Answer a)
Marketing managers often commission formal marketing studies of specific problems and opportunities, such as a market survey, a
product-preference test, a sales forecast by region or an advertising evaluation. We define marketing research as the systematic design,
collection, analysis and reporting of data and findings that are relevant to a specific marketing situation facing the company.
A company can obtain marketing research in a number of ways. Most large companies have their own marketing research departments.
Some small companies can hire the services of a marketing research firm or conduct research in creative and affordable ways. They can
engage students or professors to design and carry out projects, they can use the Internet, and they can visit competitors.
Companies normally budget marketing research at 1 to 2 percent of company sales. Much of this budget is spent with outside research
firms, which fall into three categories. Syndicated-service research firms gather consumer and trade information, which they sell for a fee.
Custom marketing research firms design studies, carry them out, and report the findings. Specialty-line marketing research firms provide
specialized services such as field interview.
Effective marketing research process can be mentioned as follows:
1. Define the problem and research objectives
2. Develop the research plan
3. Collect the information
4. Analyze the information
5. Present the findings
b) Market segmentation is a process of dividing the total market into several smaller, internally homogeneous groups. It is based on
customer oriented philosophy. No marketer can satisfy the needs of all customers by developing single marketing mix. The essence
of segmentation is that the members of each group are similar with respect to the factors that influence demand. In segmentation, we
first identify the wants of customers within a submarket and then decide if it is practical to develop a marketing mix to satisfy those
wants.
According to Philip Kotler, “Market segmentation is the subdividing of a market into homogeneous subsets of customers, where any
subset may conceivable be selected as a market target to be reached with a distinct marketing mix.”
There are several important reasons why businesses should attempt to segment their markets carefully. They are summarized below:
i. Better matching of customer needs: Customer needs differ. Creating separate offers for each segment makes sense and
provides customers with a better solution.
ii. Enhanced profits for business: Customers have different disposable income. They are, therefore, different in how sensitive
they are to price. By segmenting markets, businesses can raise average prices and subsequently enhance profit.
iii. Better opportunities for growth: Market segmentation can build sales. For example, customers can be encouraged to "trade-
up" after being introduced to a particular product with an introductory, lower-priced product.

The Institute of Chartered Accountants of Nepal 52


Suggested Answers - Marketing CAP-II Examination - December 2014

iv. Retain more customers: Customer circumstances change, for example they grow older, form families, change jobs or get
promoted, change their buying patterns. By marketing products that appeal to customers at different stages of their life ("life-
cycle"), a business can retain customers who might otherwise switch to competing products and brands.
v. Target marketing communicationbusinesses need to deliver their marketing message to a relevant customer audience. If the
target market is too broad, there is a strong risk that the key customers are missed and the cost of communicating to customers
becomes too high. By segmenting markets, the target customer can be reached more often and at lower cost.
vi. Gain share of the market segment: Minor brands suffer from lack of scale economies in production and marketing,
pressures from distributors and limited space on the shelves. Through careful segmentation and targeting, businesses can often
achieve competitive production and marketing costs and become the preferred choice of customers and distributors. In other
words, segmentation offers the opportunity for smaller firms to compete with bigger ones.

4.
a) What is service product? Explain its distinct features. (2+3=5)
b) What is pricing? Explain the importance of pricing to economy. (2+3=5)
Answer a)
A service product may be defined as any task performed by the company as 'provision' of facility or product. Services may or may not
be attached with the physical product. If the main goal of the company is to provide services rather than a physical good, then services
become the main 'product' to the company. For example, repair and maintenance, medical and health care, banking, insurance,
research and consultancy, transportation, communication etc.
Characteristics of Services
Service product has distinct features as follows:
• Services are intangible (Intangibility): Services are abstract in nature, so they cannot see, feel, taste, hear, or smell. Before
they are purchased, customer cannot handle, examine or tried out.
• Services are perishable and cannot be stored (Perishability): Services disappear quickly; they are perishable in nature and
have no stored value. For example, if a dentist's customer fails to make an appointment, a half-hour or a certain time of the
dentist is gone forever.
• Services vary widely in quality (Variability in quality): Services are heterogeneous in nature. It is impossible for a service
industry, or even and individual seller of services, to standardize service output, this is because the skills of several providers of
services may vary widely, and even a single service provider may provide different quality services to the customers. For
example, an airline does not provide the same quality of service on each trip.
• Services are not separable from their provider (Non-seperability): Services are inseparable from the supplier or service
provider. In services marketing, the service delivery agents or service sellers become the surrogates or symbol of the quality of
service. For example, if the customers are crazy of services from a particular dentist because of his quality services. But in the
mean time, instead of inspection by himself, a substitute dentist is supplied, the customers will be dissatisfied as such customers
may divert to another dentist they feel superior or better. Hence, services are nontransferable, i.e., they cannot be transferred to
another party through the channels of distribution like tangible goods.
• Most of the services are purchased and consumed at production site. For example, to get dental services, customers have to
visit to the dentist's clinic: for the maintenance of bicycles, the bicyclist has to visit to the repair and maintenance workshop, etc.
• Non-measurability: Like physical items, services do not have physical features like shape, weight, colour, design, length,
breadth, etc. Therefore, service items cannot be measured like physical items.
b) Price is the exchange value of the product or service expressed in terms of money. It is the amount of money needed to buy the
product. In another words, price is the amount of money that customers pay for the product or service. It is what customers pay for
what they get.
Pricing refers to an act of determining the exchange value between what buyers get and what the sellers receives. It is a process of
setting prices for various products and services. In short, pricing is the act of determining exchange value of a product or service.
Pricing involves formulating pricing objectives, policies and strategies, setting base price, determining discount and commission etc.
Pricing is the most visible but secretly performed activity in a business organization.
We can explain the importance of pricing to economy as follows:
1. Price influences factors of production
Rent, wages, interest and profit are paid as price for land, labor, capital and entrepreneurship. Changes in rent, wages, interest
and profit influences the demand and supply of the factors of production.
2. Price determines demand and supply
From the economic point of view, high prices increase supply and low prices increase demand. The level of demand and
supply are determined by price. The price of the products determines the numbers of buyers whose purchasing power allows
them to buy it.

The Institute of Chartered Accountants of Nepal 53


Suggested Answers - Marketing CAP-II Examination - December 2014

3. Price affects saving and investment


High prices discourage saving and investment. Government controls prices to encourage saving and investment. They provide
subsidies to stabilize and lower prices. Inflation also affects price.
4. Price is a tool for economic management
Government use price in the form of taxes to manage the economy. They administer prices for public utilities like electricity,
water, telephone, postal services. They regulate prices of public enterprises.

5. Briefly explain the following: (5×2=10)


a) Benefits of niche marketing
b) Importance of warehouse in distribution system
c) Specialty product
d) Promotion
e) Reference groups
Answer a)
A niche is more narrowly defined group seeking a distinctive mix of benefits. Marketers usually identify niches by dividing a segment
into sub-segments. For example, the segment of heavy smokers includes two niches: those who are trying to stop smoking and those who
don’t care.
In an attractive niche, customers have a distinct set of needs; they will pay a premium price to the firm that best satisfies their needs: Its
benefits are that niche is not likely to attract other competitors. The nicher gains certain economies through specialization. The niche has
size, profit and growth potential.

b) Warehouse may be a place (open or closed) where goods are stored for certain period for the distribution of goods to the target
market whenever and wherever required. While warehousing is a specialized system and process of storing surplus goods (keeping
buffer stock) for use or consumption in future because goods may not have demand immediately after the goods are manufactured.
There are three types of warehouse –Private warehouse, Public warehouse and Bonded warehouse
Warehouses play an important role in the following ways –
Importance of Warehousing
Warehouses play an important role in the following ways –
i. Lengthens the life of products and maintains the product quality.
ii. Reduces the level of product damage and losses.
iii. They keep buffer stock of goods and facilitate supplying whenever demand creates in the market.
iv. Provides the facilities of marketing activities like; bulk-breaking, packaging, labeling, selling goods in small lots, etc.
v. Makes available the seasonal products in off-seasons also.
vi. Helps meeting required demand of the customers living in diverse geographical areas.
c) Specialty products are consumer products and services with unique characteristics or brand identification for which a significant
group of buyers is willing to make a special purchase effort. Buyers normally do not compare specialty products. They only invest
the time needed to reach dealers carrying the wanted products. Customers are usually willing to travel great distance to buy
specialty product. Dealers do not need convenient locations, although they must let prospective buyers know their locations. Bata
shoes, Mercedes car are examples of specialty product.
d) Promotion is one of the major elements of marketing mix which provides valuable information to consumers about product, price,
availability, utilities and benefits. Promotion consists of various activities that facilitate exchanges with target customers through
persuasive communication which stimulates the demand of the product.
According to Philip Kotler, “Promotion includes all the activities the company undertakes to communicate and promote its products
to the target market.”
Promotion includes various types of activities such as advertising, sales promotion, personal selling, publicity and public relation to
inform, persuade, remind and reinforce the target market about the company’s offerings. These activities influence the customer’s
feelings, belief or behavior for buying.
e) Reference groups have a direct/ face to face or indirect influence on a person’s attitudes or behavior. Groups having a direct
influence on a person are called membership groups. Some primary groups are family, friends, neighbors, and co-workers, with
whom individuals interact fairly continuously and informally. Secondary groups, such as professional and trade union groups tend
to be more formal and require less continuous interaction. Reference groups expose people to new behaviors and lifestyles,
influence attitudes and self-concept and create pressures for conformity that affect product and brand choices.

The Institute of Chartered Accountants of Nepal 54


Suggested Answers - Income Tax & VAT CAP-II Examination - December 2014

Income Tax & VAT


1.
a) Saru Pvt. Ltd., Kathmandu declared a Voluntary Retirement Scheme (VRS) for its staffs during FY 2071/72. Provident funds are
deposited in CIT, an approved retirement fund. Man Bahadur decided to take VRS with effect from 1st Baishakh 2072. The
following details are available for income of Man Bahadur for the year.
i) Basic Salary till Chaitra 2071 Rs. 50,000 per month.
ii) Monthly Allowance Rs. 30,000 per month.
iii) Provident Fund contributed by employer 10% of basic salary
An equal amount was contributed by employee.
iv) Life Insurance premium paid by Man Bahadur for himself Rs. 25,000
v) Remote area allowance for working in.
Category C district for two months Rs. 5,000 per month.
vi) Provident Fund paid by CIT on 15th Baishakh 2072 Rs. 12,00,000.
Retirement payment paid by Saru Pvt. Ltd. as per VRS was Rs. 20, 00,000, paid to Man Bahadur directly by the company.

Required:
(a) Calculate taxable income and tax liability of Man Bahadur for F/Y 2071/72 as per Income from Employment. Assume
individual. 7
(b) Identify any payments subject to final withholding Tax (Final TDS) and applicable TDS rate and Tax amount. 3

b) JKL Limited purchased 100 shares of MNO Bank Limited for each Rs. 200 each including brokerage and other expenses, as on
Chaitra 19, 2058, the company was holding 250 shares of MNO including the shares purchased and bonus shares paid by MNO.
The quoted closing price on Chaitra 18th, 2058 on MNO shares was Rs. 800 each.
Calculate the gain from the disposal of the shares under the circumstances given below: 10
i) On Jestha 15, 2071 the company had disposed of the total shares at the rate of Rs. 1,000 each. The brokerage paid was
1.2% of the sales amount.
ii) On Jestha 15, 2071 the company had disposed the total shares at the rate of Rs. 750 each. The brokerage paid was 1.2% of
the sales amount.
iii) On Falgun 10, 2070 the company had received 125 bonus shares for the 250 shares held by it. On Jestha 15, 2071 the
company had disposed 250 shares at the rate of Rs. 750 each. The brokerage paid was 1.2% of the sales amount.

Answers:
1a) Answer to part a) of question 1(a)
Calculation of taxable income and tax liability of Man Bahadur for FY 2071/72
Details Amount Notes
Salary 450,000.00
Monthly Allowance 270,000.00
Provident Fund by Employer 45,000.00
Remote Area Allowance 10,000.00
Total Assessable Income 775,000.00
Less: Contribution to Approved Retirement Fund (90,000.00) 1
1/3rd of Assessable Income 258,333.33
Contribution 90,000.00
Maximum 300,000.00
Taxable Income 685,000.00
Less: Life Insurance Premium (20,000.00) 2
Less: Remote Area Benefit (5,000.00) 3
Net Taxable Income 660,000.00
Calculation of tax liability for FY 2071/72
Tax Slab Tax Liability
1st Rs. 250000 @1% 2500
Next Rs. 100,000 @15% 15000
Balance Rs. 310,000 @25% 77500
Total Tax Liability 95,000

The Institute of Chartered Accountants of Nepal 55


Suggested Answers - Income Tax & VAT CAP-II Examination - December 2014

• Note 1- Contribution to approved retirement fund is deductible upto 1/3rd of assessable income or actual contribution or Rs. 3
lakh whichever is lower. Here, the amount deductible is Rs. 90,000.
• Note 2- Life Insurance premium paid is deductible actual premium paid not exceeding Rs. 20,000.
• Note 3- Remote area benefit available for Category C district is Rs. 30,000, proportionate for period of stay. Here, two
months.

Answer to part (b) of question 1(a)

The retirement payments made to Man Bahadur are subject to TDS as per Sec. 88 of the Act.
a. Provident Fund paid by CIT
Total Payment (Note 1) Rs. 12,00,000
Less 50% of the payment or Rs. 5,00,000 whichever is higher (Rs. 6,00,000)
Net Payment subject to tax Rs. 6,00,000
TDS @ 5% Rs. 30,000
The payment is final TDS as per Sec. 92.
Note 1: The payment for provident fund is assumed to have related only with the period after the Income Tax Act, 2058.

b. Retirement Payment paid by employer is subject to 15% tax as per Sec. 88. The tax amount for Man Bahadur is Rs. 3,00,000.
The payment is final as per Sec. 92 of the Act.
1b)
Solution under the first circumstance:
Cost of shares (100 shares @ Rs. 200 each Rs. 20,000
Cost of bonus shares (150 Bonus Share) Nil
Cost of 250 shares Rs. 20,000
But the index value of the shares as on Chaitra 18, 2058 was Rs. 800 per share and so the deemed cost of the 250 shares shall be
Rs. 800×250=Rs. 200,000 + Rs. 3,000 (1.2% of the sale proceeds for brokerage on sales.)
The amount received from the disposal is 250×Rs. 1000=Rs. 250,000.
Thus the gain is of Rs. 250,000-Rs.203,000=Rs. 47,000.

ii. Solution under the second circumstance:


The index value of the shares as on Chaitra 18, 2058 was Rs. 800 per share and so the deemed cost of the 250 shares shall be Rs.
800×250=Rs. 200,000 + Rs. 2,250 (1.2% of the sale proceeds for brokerage on sales.)
The amount received from the disposal is 250×Rs. 750=Rs. 187,500.
Thus the loss is of Rs. 187,500-Rs.202,250=Rs. 14,750.

iii. Solution under the third circumstance:


The index value of the shares as on Chaitra 18, 2058 was Rs. 800 per share and so the deemed cost of the 250 shares shall be Rs.
(800×250) = Rs. 200,000
Plus the cost of 125 bonus shares received after Chaitra 18, 2058 = Rs. Nil
The total cost of 375 shares is Rs. 200,000 and so the cost of one share comes to Rs. = 533.33.
The cost of 250 shares sold comes to Rs. 133,332.50+2,250 (1.2% of the sale proceeds for brokerage on sales).
The amount received from the disposal is 250×Rs. 750=Rs. 187,500.
Thus the gain is of Rs. 187,500-Rs. 135,582.50 =Rs. 51,917.50.

2.
a) Mr. Rich has purchased a private building amounting to Rs. 2 crores on Kartik 23, 2060 at Kathmandu. He sold such building
on Rs. 3 crores on Poush 26, 2070. During such period, Mr. Rich was gone abroad, intermittently for a period of 120 days. Will
such building be considered as "Non-Business Chargeable Assets"? What will be your answer if such building was sold after
two months (i.e. on Falgun 26, 2070)? 5

b) Munal Trade Link is a various goods supplier. The trade link is using accrual basis of accounting for the income generated
from the business. He gave an application to the department to change his basis of accounting from accrual to cash in Income
Year 2070/071. In accordance with the generally accepted accounting principles, Income Tax Department has given the
approval to account for his income on cash basis in Income Year 2070/71. Some accounting information of the trade link at the
end of 2069/070 before changing the basis of accounting, was as below;
i) Goods worth of Rs. 75,000 has already been supplied to a retailer in Income Year 2069/070 but amount has not been
received. Advance Rs. 45,000 has been received from another retailer.
ii) Goods worth of Rs. 60,000 has been purchased from a vendor in Income Year 2069/070 but amount has not been paid.
Advance Rs. 35,000 has been paid to another vendor.

The Institute of Chartered Accountants of Nepal 56


Suggested Answers - Income Tax & VAT CAP-II Examination - December 2014

iii) Communications and electricity expenses of Rs. 3,500 were paid on Bhadra, 2071. These expenses were incurred in Jestha
and Ashadh, 2070.
State how you make adjustments of income and expense in FY 2070/071 in above conditions? 5

c) Mr. Komal purchased a piece of land at Rs. 30 lakhs. He sold the land at Rs. 45 lakhs. He paid registration expenses Rs. 2
lakhs for this land. In this case, what would be the tax implications on the following situations?
i) The land was purchased on Chitra 2064 and sold it on Magh 2070.
ii) The land was purchased on Magh 2067 and sold it on Magh 2070.
iii) The land was purchased on Chitra 2059 and sold it on Baishak 2071.
iv) If selling and buying of the land were completed through a sole shareholder of a Pvt. Ltd. The shareholder is Mr. Komal.
5
d) Standard Chartered Bank has appointed AX Consultancy Pvt. Ltd. as Tax Consultant on Magh 20, 2070 and the effective date
of contract is Shrawan 1, 2071. Under the contract, the bank is liable to pay an annual fee of Rs. 500,000. As per the contract,
an advance amount of Rs. 100,000 is to be paid on appointment date (i. e. on Magh 20, 2070). AX Consultancy Pvt. Ltd. is in
confusion whether the amount of Rs. 100,000 to be included in its income of Income Year 207/071 as the appointment letter is
already received during the Income Year. Advise. 5

Answers:
2a)
Section 2 (da) (2) of Income Tax Act, 2058 has excluded the following assets from the definition of "Non-Business Chargeable
Assets" in case of an individual;
A private building of an individual that has been;
o owned continuously for ten years or more; and
o lived in continuously or intermittently for a total period of ten years or more.

In the given case, Mr. Rich has owned his private building for a period of ten years in which first condition is satisfied. But he lived
in for a period of less than ten years in which second condition is not satisfied.
Hence, such building is to be considered as "Non-Business Chargeable Asset" as per Income Tax Act, 2058.
But, in case where Mr. Rich sold such building after two months (on Falgun, 26, 2070) second condition of 'lived in continuously
or intermittently for a total period of ten years or more' will also be fulfilled and hence such building shall not be considered as
"Non-Business Chargeable Asset".

2b)
As mentioned in Chapter 6 of Income Tax Act, 2058, if any person takes the approval from Inland Revenue Department for the
change in basis of accounting for the purpose of taxation, or his basis of accounting is changed because of various provisions
mentioned in the same Chapter, inclusion of his income, claim of expenditure shall be computed in such a manner that there is no
repetition or short of income or claim of expenditure in that Fiscal Year.

Accordingly, the following adjustments shall be made in Income Year 2070/71;


i) When goods are supplied, the income shall be recognized in accrual basis of accounting. So, no need to adjustment for the
goods of Rs. 75,000.00 that was included in Income Year 2069/70. Advance amounting Rs. 45,000.00 was not assessable
income for the Income year 2069/70 as per accrual basis of accounting, so, it shall be included in Income Year 2070/71 on
cash basis
ii) When goods are purchased, the expenses shall be recognized in accrual basis of accounting. So, no need to adjustment for the
goods of Rs. 60,000.00 that was deducted in Income Year 2069/70. Advance payment Rs. 35,000.00 was not deductible
expenses for the Income year 2069/70 as per accrual basis of accounting, so, it shall be deducted in Income Year 2070/71 on
cash basis.
iii) Communications and electricity expenses of Rs. 3,500.00 were also eligible expenses for the Income Year 2069/70 on
accrual basis of accounting, so no need to adjustment in Income Year 2070/71. After making adjustments of inclusion of
income and claim of expenditure in Income Year 2070/71, the income or expenditure shall be accounted on cash basis for
subsequent years.
2c)

Land and building disposed for a value less than 30 lakhs by any natural person is not treated non-business taxable assets as per
Section 2(Da). The disposal value more than the limit shall be taken into account in computing the gain and loss on income from
investment. Tax liability is calculated on the basis of the net gain from the disposal of those assets. Withholding tax is managed for
such transactions under section 95 Ka (3) of the Act for natural person. If the land is sold after holding it for more than 5 years the
tax is 2.5 % and if it is sold after holding it for less than 5 years the tax rate is 5%. The Land Revenue Office withholds the tax on
such net gain.

The Institute of Chartered Accountants of Nepal 57


Suggested Answers - Income Tax & VAT CAP-II Examination - December 2014

Calculation of net gain on this land:


Particular Amount Amount
Incoming Rs. 45 Lakhs
Outgoings Rs. 32 Lakhs
Purchase cost Rs. 30 Lakhs
Transfer expenses Rs. 2 Lakhs
Net gain Rs. 13 Lakhs

So, TDS shall be as follows in this case. Any exemption limit available reduces the tax liability of the natural person. This depends
on other taxable income of the tax payer and a tax return should be submitted for this.

i) The land was purchased on Chitra 2064 and sold it on Magh 2070. Mr. Komal sold the land after holding 5 years. The TDS
is Rs. 32,500.00 (2.5% of 13 Lakhs)
ii) The land was purchased on Magh 2067 and sold it on Magh 2070. He sold the land within 5 years. The TDS is Rs.
65,000.00 (5% of 13 Lakhs
iii) The land was purchased on Chitra 2059 and sold it on Baishak 2071. Mr. Komal sold the land after holding more than 10
years. However, the TDS is Rs. 32,500.00
iv) If selling and buying of the land were completed through a sole shareholder of a Pvt Ltd, it will be regarded as disposal of
business assets. There will be no TDS, but the net gain is calculated under Sec. 36 of the Act and net gain after set off with
any unrelieved lossis included as income from business under Section 7.

2d)
As per section 22(3) of Income Tax Act, 2058 a company has to account for its income on accrual basis for tax purpose. Further,
as per section 27(2), the time the payment is derived, incurred, made, received or otherwise taken into account for tax purpose
shall be treated as the time of quantification of amount. In the given case, AX Consultancy Pvt Ltd is a company and has to
account for its income on accrual basis for tax purpose. Performance of service is the time to recognize the income. The time the
payment is derived shall be treated as the time of quantification of the amount. Hence, advance payment is irrelevant to recognize
the income and shall be simply recorded as advance payment received in Income Year 2070.71. Since the effective date of
contract is Shrawan 01, 2071 the service will accrue only on Income Year 2071.72 and shall be recognized the income
accordingly.

3.
a) Super company is a proprietorship industry producing tobaccos. The Industry has total sales of Rs. 2,00,00,000 and taxable
income of Rs. 35,00,000 in Income Year 2070/071. The company has not submitted estimated tax return during the year. In
addition, it has not paid any taxes. You are requested to assess the total Tax liability for this industry upto Mansir 20, 2071.
5
b) Mr. Ramesh has been retired from Government of Nepal on 15th Jestha, 2071. He has received the following retirement
payments in Ashadh, 2071. He has not any other sources of income except salary. Remuneration tax already has been deducted
and deposited. Assume, no retirement payment was accrued at the commencement of this act in connection with this
employment.

Particulars Amount (Rs.)


Payment from GON against accumulated leave and medical allowances Rs. 6,00,000
Payment from Employee Provident Fund against contribution Rs. 7,00,000
Payment from Citizen Investment Trust against contribution Rs. 5,00,000
5

The Institute of Chartered Accountants of Nepal 58


Suggested Answers - Income Tax & VAT CAP-II Examination - December 2014

Answers:
3a)
The company is a proprietorship firm, so the tax rate is applicable as per schedule 1(1). Thirty percent tax rate for tobacco business
is applicable only for entity.

Assumption:
The taxpayer has selected for single.
Calculation of total tax liability for F/Y 2070/71
Total taxable income Rs. 35,00,000.00
Particulars Tax rate Tax amount Rs.
Upto Rs. 200,000.00 0% -
Next Rs. 100,000.00 15% 15,000.00
Balance Rs. 32,00,000.00 25% 8,00,000.00
Additional Tax on the tax amount levied @ 25% for taxable
income above 25,00,000.00 (10,00,000.00*25%) i.e on Rs.
2,50,000.00 40% 100,000.00
Total tax amount 9,15,000.00
Fees under section 117 (1)(ka) 2,000.00
Fees under section 117 (1)(ga) (WN 2) 3,333.33
Interest under section 118 (1) (WN 2) 64835.62
Interest under section 119 (from Kartik to Mansir 2071) @ 15 % of
Rs.9,15,000.00 22,875.00
Total tax liability till Mangsir end 2071 1008043.95

Working Note 2: Calculation of Fees and interest


• Under Section 117 (1)(ga): Total assessable income as per the section is Rs. 2,00,00,000.00, fees 0.1 % annually of Rs.
2,00,00,000.00 for 2 months (from Kartik to Mansir, 2071) is Rs. 3,333.33 or Rs. 100 per month whichever is higher.
• Under Section 118: total tax Rs. 9,15,000.00

Cumulative 90% of Actual Interest


Interest @
Tax Cumulative Paid Chargeable Period
Installments 15 % amount
Amount Tax Instalme Amount (d) (Months)
Rs.
Rs. (a) Amount (b) nt (c) = (b)-(c)
Poush end 3,66,000.00 3,29,400.00 0 3,29,400.00 3 12,337.5
Chaitra end 6,40,500.00 5,76,450.00 0 5,76,450.00 3 21,616.87
Ashad end 9,15,000.00 8,23,500.00 0 8,23,500.00 3 30,881.25
Total 64,835.62

If students assume a Couple u/s 50 for calculation of tax liability, and calculate the interest u/s 118 taking the difference
from 100%, full marks will be awarded.

3b)
i) Fifty percent of the paid amount Or Rs. 5 Lakhs, whichever is higher, shall be deducted on the lump sum retirement payment
from any Approved Retirement Fund or Nepal government in computing tax liability of any natural person as per section 65(1).

Further, section 88 (1) (1) states that in the case of the retirement payment made by the Government of Nepal or by the
approved retirement fund, at the Rate of Five percent shall be deducted from the benefits calculated pursuant to section 65(1).
Such retirement payment is final withholding as per section 92 (1) (Chha).

Retirement payment to Mr. Ramesh from retirement fund and Nepal government and 50 percent thereof are as follows:
Particulars Amount
Payment from GoN against accumulated leave and medical allowances Rs. 6,00,000.00
Payment from Employee Provident Fund against contribution Rs. 7,00,000.00
Payment from Citizen Investment Trust against contribution Rs. 5,00,000.00
Total amount (WN 1) Rs. 18,00,000.00
50% of Total Payment (a) Rs. 9,00,000.00
Fixed amount u/s 65 (b) Rs. 5,00,000.00
Deduction allowed for the higher amount (a) or (b) above Rs. 9,00,000.00
Taxable Amount RS. 9,00,000.00

The Institute of Chartered Accountants of Nepal 59


Suggested Answers - Income Tax & VAT CAP-II Examination - December 2014

The EPF will withhold Rs.10,000.00 @ 5% on Rs. 2,00,000.00 (Rs.7,00,000.00-Rs.5,00,000.00) and similarly, the CIT will not
withhold any amount being only Rs.5,00,000.00. But, the GON will withhold Rs. 35,000.00 (i.e Rs. 45,000.00 @ 5% on
9,00,0000.00 minus the amount Rs. 10,000.00 already withheld by the EPF). It will be a final withholding u/s 92.

4. Write short notes of the following with reference to Income Tax Act, 2058. (5×2=10)
a) Advance ruling and public circulars are different in nature.
b) Not all persons are required to submit Income Tax returns.
c) Entity
d) Tax exempted incomes
e) Right of Tax payer.

Answers:
4a)
a. Advance Ruling and Public Circulars are different in nature.
Public Circulars are issued to general public to achieve consistency in the implementation of Act and to assist the tax officers
and tax payers in application of the provisions of the law. The public circular is mandatory for the department, but not for the
public. If the tax payer is not satisfied with the circular issued by IRD, he may choose for legal recourse. Detailed provisions
are stated in Section 75 of Income Tax Act.

Advance Ruling is issued to the applicant tax payer for clarifying application of tax law on proposed arrangement of the
applicant. The advance ruling is only applicable for the concerned applicant and is not available for general public. Detailed
provisions are stated in Section 76 of Income Tax Act.
4b)
b. Not all persons are required to submit income tax returns.
There are certain exceptions to provisions requiring submission of tax returns by the tax payer. As per Sec. 97, the following
persons are not required to submit tax returns:
• Person who do not involve in any economic activities leading to taxable income
• A natural person with only income from an employment based on a source in Nepal, with all employers during the
income year as resident persons and only one employer at a time and who only claims medical costs and retirement
contribution paid by the employer and do not claim donations or gifts paid to tax exempt organization.
• Person earning final withholding paymentsNatural persons owning public vehicles who pay presumptive tax as per
Schedule 1 of the Act.

4c)
Entity
As per section 2 (Bha) of the Act, entity is defined the following organization or body:
(1) A partnership, trust or company,
(2) Village Development Committee, Municipality or District Development Committee,
(3) Government of Nepal,
(4) Any foreign government or provincial or local government under that government or a public international organization
established by any treaty, or
(5) A permanent establishment of the organization or body referred to in clauses (1), (2), (3) and (4), which is not situated in a
country of which it is a resident.

4d)
As per Section 10 of Income Tax Act, 2058 the following amounts are tax exempted incomes:
i) Amounts derived by a person entitled to privileges under a bilateral or a multilateral treaty concluded between Nepal
Government and a foreign country or an international organization.
ii) Amounts derived by an individual from employment in the public service of the government of a foreign country.
Provided that-
a) The individual is a resident person solely by reason of performing the employment or is a non-resident person; and
b) The amounts are payable from the public funds of the country.
iii) Amounts derived from public fund of the foreign country by an individual who is not a citizen of Nepal as referred to in
paragraph (ii) or by a member of the immediate family of the individual.
iv) Amounts derived by an individual who is not a citizen of Nepal from employment by Nepal Government on terms of tax
exemption.
v) Allowances paid by Nepal Government to widows, elder citizens, or physically disabled individuals.

The Institute of Chartered Accountants of Nepal 60


Suggested Answers - Income Tax & VAT CAP-II Examination - December 2014

vi) Amounts derived by an exempt organization by way of-


a) Gift; or
b) Other contributions that are directly related to the organization’s function referred to in paragraph(s) (1) of the
definition of the exempt organization in section 2, whether or not the contribution is made in return for
consideration provided by the organization, or
c) Income earned by Nepal Rastra Bank as per its objective, or
d) Income earned by Securities Board of Nepal as per its objective.
vii) Pension received by a Nepali citizen retired from the army or police service of a foreign country provided the amounts
are payable from the public fund of that country.
viii) Any income of Nepal Government.

4 e)
Taxpayer’s Rights as per section 74 of Income Tax Act, 2058;
i. Tax Payer shall abide by the duties in accordance with this Act.
ii. A taxpayer with respect of paying tax under this Act shall have the following rights:-
a. Right to get respectful behavior;
b. Right to receive tax related information as per the prevailing laws;
c. Right to get opportunity of submitting proof in own favor in respect of tax matters;
d. Right to appoint lawyers or auditors for defense; and
e. Right to secrecy in respect of tax matters and keep it inviolable
[Clarification:- For the purpose of this section, taxpayer means a person whom the tax is imposed on and realized from as
referred to in section 3.]

5.
a) Explain whether the following statements are true or false with reasons: (5×1=5)
i) A person shall apply for the close of taxable transaction in the format prescribed in Schedule 10 of VAT Regulation, 2053.
ii) A person shall be imposed fine of Rs. 10,000 at each time in case of refusal for the inspection of books of account by the
Tax officer.
iii) The Tax paid by a diplomatic body or diplomat on the purchase of taxable goods or services shall not be refunded if the
amount purchase is less than Rs. 5,000 at one time.
iv) Tax Officer may assess Tax as per section 20 if Kathmandu District Development Committee collects the tax on taxable
transactions and deposit in its internal revenue account.
v) A foreign tourist can take refund of VAT on the purchase of goods amounting to Rs. 15,000 or more.

b) Apar International Pvt. Ltd. is a Recondition House involved in Purchase/Sale of Used Vehicle, Motorcycle, TV and
Refrigerator. The company purchases the used/old Vehicle, Motorcycle, TV and Refrigerator and Sale those goods to the
consumer by repairing and improving the same. Company purchased from Mr. Ram residing in Satungal at Rs. 20 lakhs and
after repairing the same sold to Ms. Sunita at Rs. 25 lakhs Apar Ltd. seeks your advise as how much VAT shall he collect on
such sales amount.
5
c) Saurav Publication prints the various books and sells to the wholesaler. The Company itself has constructed a building for its
Publication House. The cost of construction was as follows:
Labour charges Rs. 30,00,000
All Material (Cement, rod etc) purchase cost Rs. 5,50,000 inclusive VAT.

State the VAT Implications on this case, if applicable with references to VAT Act, 2052 and rules, 2053. 5

d) Ilam Tea Pvt. Ltd. is a tea processing and producing industry, located in Ilam. It sells the tea within Nepal. It has following
transaction in the month of Bhadra, 2071.
Particulars Transactions Amount (Rs.)
Sales:-
VAT registered party 20,00,000
VAT Non-registered party 5,00,000
Purchase:-
Wages and others that are VAT exempt 12,00,000

Compute the VAT amount payable/receivable by the Tea Industry with reference to the VAT Act, 2052 for the Month. Assume
no VAT on opening. 5

The Institute of Chartered Accountants of Nepal 61


Suggested Answers - Income Tax & VAT CAP-II Examination - December 2014

Answers:
5a)
i) False. Application for the close of VAT shall be filed in the format prescribed in Schedule – 11 of VAT Regulation, 2053.
ii) False. A person shall be imposed fine of Rs. 20,000.00 at each time in case of refusal for the inspection of books of account by
the tax officer.
iii) True. The tax paid by a diplomatic body or diplomat on the purchase of taxable goods or services shall not be refunded if the
amount purchase is less than Rs. 5,000.00 at one time.
iv) True. Tax officer may assess tax as per Section 20 if Kathmandu District Development Committee collects the tax on taxable
transactions and deposit in its internal revenue account.
v) False. A foreign tourist can take refund of VAT on the purchase of goods amounting to Rs. 25,000 or more.

5b)
a) Section 17(5) of the VAT Act, 2052 read with Rule 33 of VAT Rules 2053 provides that, in case of person dealing with used
assets, VAT shall be determined on the difference amount between sale value and purchase value only.
In the given case, on the basis of above provision, VAT shall be levied on Rs. 5 lacs only and accordingly VAT amount will be
Rs. 65,000.(being 13% of Rs. 500,000).

5c)

Saurav Publication is engaged in VAT exempt transaction as per Group 7 of Schedule 1 of the VAT Act, 2052.
If it has constructed a building for less than Rs. 50 lakh, it is not attracted u/s 8(3) which states that even though the construction of
a building or apartment or shopping complex and similar other structure as specified by the Department, of which value is more
than Rs. 50 lakhs, and which is built for business purpose is procured from a person who is not registered, tax shall be assessed and
collected from a person who has ownership in that structure as if such construction were procured from a registered person.

Rule 6 (Kha) is also not attracted as it mentions that any person who is constructing building, apartment, shopping complex or
similar structure as specified by the Department for business purpose amounting more than Rs. 50 lakhs, he has to get it
constructed only from VAT registered person.

Thus, VAT paid on purchase of the materials are not eligible for credit u/s 17 and will be capitalized in the cost of the building.

5d)
Schedule 1 of VAT Act, 2052 has provided refund facility for Tea Industry. As per the provisions, 50 % of collected VAT amount
on the sales to VAT registrants shall be refunded as specified by the Inland Revenue Department to the tea producing and
processing industry of Nepal.

The Ilam Tea Industry has collected VAT Amount in the Month of Bhadra, 2071 as follows:

Particulars Transactions Amount Rs. VAT collected Rs.


Sales
VAT registered party 20,00,000.00 2,60,000.00
VAT Non-registered party 5,00,000.00 65,000.00
VAT Payable 3,25,000.00

The industry should pay VAT amount of Rs. 3,25,000.00 on Aswin, 2071 for the month of Bhadra, 2071. It can apply for refund
amounting of Rs. 1,30,000.00 (50 % of Rs. 2,60,000.00). If the Inland Revenue Department permits the Tea Industry for the
adjustment of refundable amount on the payable amount, it shall pay the VAT amount accordingly. So, at the end of Bhadra, 2071,
VAT payable amount is Rs. 3,25,000.00 and VAT receivable amount is Rs. 1,30,000.00

6.
a) Finance Manager of ABC Hardware is worrying about the price adjustment with it's supplier. He had heard that he can do the
price adjustment through the use of Debit/Credit Note. You are requested to advise him regarding the content of Debit/Credit
Note as per VAT Rules. 5
b) Input Tax Credit (ITC) under sec. 17 of Value Added Tax (VAT) Act, 2052. 5
c) M/s RK Trading Private Limited is a Value Added Tax Registered company. It is engaged in the business of importing wines
& liquor and selling in local market. The company is in confusion to take tax credit on procurement of wines and liquor as per
Value Added Tax Act/Rules, Advise. What will be your answer if the company does not have such primary business? 5
d) Discuss the amount to be included in Taxable Value as per section 12 of Value Added Tax Act, 2052. 5

The Institute of Chartered Accountants of Nepal 62


Suggested Answers - Income Tax & VAT CAP-II Examination - December 2014

Answers:
6a)
As per Rule 20(1) of the Vat Rules, 2053 a person registered under VAT can adjust the price difference on Sale of goods or services
by issuing. Debit Note or Credit Note. As per the said Rule, the following are the content of Debit/Credit Note:
a. Serial Number
b. Date of Issue
c. Name, Address and PAN Number of Supplier
d. Name, Address and PAN Number (if he is registered) of Receiver
e. Invoice No. and Date of Concerned Transaction
f. Detail of Product or Service and Reason of Debit or Credit
g. Amount of Debit or Credit
h. Vat Debit or Credit amount.

6b)
The following provisions are mentioned for tax offset in section 17 of the VAT Act, 2052:

(1) A registered person can offset the amount of tax he has collected against the tax he had paid or due in importing or receiving
goods or services related to his own taxable transactions.
(2) Notwithstanding anything contained in Subsection (1), it may be provided that no deduction or only a partial deduction may be
granted in the case of the prescribed goods that can be used for personal purpose or for business purpose or for both purposes.
(3) If the entire portions of goods or services transacted in a month were not used for taxable transactions the tax previously paid on
the goods or services shall be offset as prescribed for the portion that was solely used for taxable transaction of the goods or
services.
(4) If goods or services for which offset privileges pursuant to this section have been allowed cease to be used for taxable taxation
before the end of its useful life, as may be prescribed, such goods or services shall be treated as sold at the immediate market
value and tax shall be collected.
(5) Notwithstanding anything contained in sub-section (1), offset privileges to be provided for a registered person who deals with
the used goods shall be as prescribed.
(5a) The concerned taxpayer shall be allowed to deduct the tax paid on the capital goods that were imported or purchased by entering
into a loan agreement under financial lease subject to Sub-section (2).
(5b) The concerned taxpayer shall be allowed to deduct tax paid pursuant to Sub-section (2) of Section 8, Section 12A and Sub-
section (3) of Section 15.
(6) The offset privileges under this Act shall be provided only when a claim is substantiated by documents as prescribed.
(7) The provision of offset on tax paid or payable on goods, which has remained unused at the time of the registration and is for use
in making taxable transactions, shall be as prescribed.
(8) Notwithstanding anything contained elsewhere in this Section, the name of a taxpayer who does not submit the tax returns for
Six months consecutively shall be made public, and if there is any amount due for tax deduction by such a taxpayer, such
amount may be suspended and his or her registration may also be suspended…

6c)
Rule 41(1) of Value Added Tax Rules, 2053 prescribes the following provision.

For the purpose of section 17 of the act, tax credit shall not be allowed in respect of the following goods or services;
- Alcohol or alcohol mixed beverages such as liquor, beer.
Rule 41(3) prescribes the following provision:
For a registered person, who carries on a business of those goods mentioned in sub- rule (1) and (2) of Rule 41 as the principal
business, there shall be no restriction for the tax credit in accordance with the procedures mentioned in these rules.

The principal business of M/s RK Trading Private Limited is of importing wines & liquor and selling in local market and tax credit is
allowed. If the company's primary business is not of importing wines and liquor and selling in local market, input tax credit shall not
be available as prescribed under rule 41(1).

6d)
Section 12 of Value Added Tax Act, 2052 of the act lists the amount to be included in Taxable value.

Section 12(1)
Except otherwise provided in this act, where only cash is the consideration, the taxable value shall be the price the supplier receives
from the recipient of supply.

The Institute of Chartered Accountants of Nepal 63


Suggested Answers - Income Tax & VAT CAP-II Examination - December 2014

Section 12(2)
The following amounts shall be included in the taxable value:
(a) The amount of all expenditures related to transportation and distribution, which was borne by a supplier in connection with the
transaction, and the amount of profit.
(b) Amount of excise duties, ownership fee and all other tax amount other than a tax imposed under is act.

Clarification – For the purpose of this section, “other tax amount” means duties, charges and fees prescribed by the annual Finance
Act.

Section 12(3)
Taxable value shall not include the amount of discount, commission or other similar commercial rebates granted in the value of
supplying goods or service.

Section 12(4)
The taxable value of any goods or service exchanged or bartered shall be equal to the market value of the goods or services so
exchanged or bartered.

Section 12(5)
Except otherwise provided in this act, while determining the taxable value of any imported goods all amounts of transportation,
insurance, freight, commission of agents and other persons, customs duties, countervailing duties plus any taxes if levied on imports
other than value added taxes, shall be added.

Section 12(6)
Where the value of any goods or services is found to be lower than the prevailing market value, the taxable value of such goods or
services shall be equal to the market value.

Section 12(7)
The taxable value of goods or services supplied for partial consideration shall be equal to the market value.

Section 12(8)
A deposit, given in respect of goods or services, shall not be treated as taxable value unless the supplier adjusts the deposit as
consideration for the supply as prescribed.

The Institute of Chartered Accountants of Nepal 64


(1)

CAP-II, Advanced Accounting, Dec 2011


Suggested Answer

Roll No……………. Maximum Marks - 100


Total No. of Questions - 6 Total No. of Printed Pages - 4
Time Allowed - 3 Hours
Marks
Attempt all questions. Working notes should form part of the answer.

1. Laxman and Hari are partners of the firm LH & Co., from 1.4.2007. Initially both of
them contributed Rs. 1,00,000 each as capital. They did not contribute any capital
thereafter. They maintain accounts of the firm on mercantile basis. They were sharing
profits and losses in the ratio of 5:4. After the accounts for the year ended 31.3.2011
were finalized, the partners decided to share profits and losses equally with effect
from 1.4.2007. It was also discovered that in ascertaining the results in the earlier
years certain adjustments, details of which are given below, were not taken into
account.
Year ended 31st March 2008 2009 2010 2011
Rs. Rs. Rs. Rs.
Profit as per accounts
prepared and finalized 1,40,000 2,60,000 3,20,000 3,60,000
Expenses not provided
for (as at 31st March) 30,000 20,000 36,000 24,000
Incomes not taken into
account (as at 31st March) 18,000 15,000 12,000 21,000
The partners decided to admit Chakra as a partner with effect from 1.4.2011. It was
decided that Chakra would be allotted 20% share in the firm and he must bring 20%
of the combined capital of Laxman and Hari.
Following is the Balance sheet of the firm as on 31.3.2011 before admission of
Chakra and before adjustment of revised profits between Laxman and Hari.
Balance Sheet of LH & Co. as at 31.3.2011
Liabilities Rs. Assets Rs.
Capital Accounts: Plant and machinery 60,000
Laxman 2,11,500 Cash on hand 10,000
Hari 1,51,500 Cash at bank 5,000
Sundry creditors 2,27,000 Stock in trade 3,10,000
Sundry debtors 2,05,000
5,90,000 5,90,000
You are required to prepare: 20

a) Profit and Loss Adjustment account;


b) Capital accounts of the partners; and
c) Balance Sheet of the firm after the admission of Chakra.

YSZ P.T.O.
(2)

Answer No. 1
(i) Profit and Loss Adjustment Account
Particulars Rs. Particulars Rs.
To Expenses not provided for: By Income not considered for:
Year ended 31.03.2008 30,000 Year ended 31.03.2008 18,000
Year ended 31.03.2009 20,000 Year ended 31.03.2009 15,000
Year ended 31.03.2010 36,000 Year ended 31.03.2010 12,000
Year ended 31.03.2011 24,000 110,000 Year ended 31.03.2011 21,000 66,000
By Partners capital account:
Laxman 22,000
Hari 22,000 44,000
110,000 110,000

(ii) Partners’ Capital Accounts


Laxman Hari Chakra Laxman Hari Chakra
Particulars Particulars
Rs. Rs. Rs. Rs. Rs. Rs.
To P / L Adj. A/c - Loss 22,000 22,000 - By Balance b/d 211,500 151,500 -
To Hari 60,000 - - By Laxman - 60,000 -
To Balance c/d 129,500 189,500 63,800 By Cash/Bank - - 63,800
211,500 211,500 63,800 211,500 211,500 63,800

(iii) Balance Sheet of LH & Co. as on 1.4.2011


(After admission of Chakra)
Liabilities Rs. Assets Rs.
Capital Accounts: Plant & Machinery 60,000
Laxman 129,500 Stock in Trade 310,000
Hari 189,500 Sundry Debtors 205,000
Chakra 63,800 382,800 Accrued Income 66,000
Sundry Creditors 227,000 Cash at Bank 5,000
Outstanding Expenses 110,000 Cash in Hand 73,800
719,800 719,800

Working Notes
1. Computation of Profit and Loss distributed among partners:
Rs.
Profit for the year ended 31.3.2008 140,000
31.3.2009 260,000
31.3.2010 320,000
31.3.2011 360,000
Total Profit 1,080,000

Laxman Hari Total


Rs. Rs. Rs.
Profit shared in old ratio i.e 5 : 4 600,000 480,000 1,080,000
Profit to be shared as per new ratio i.e. 1 : 1 540,000 540,000 1,080,000
Excess / (Deficit) Profit 60,000 (60,000) -

YSZ P.T.O.
(3)

Laxman to be debited by Rs. 60,000 and Hari to be credited by Rs. 60,000

2. Capital brought in by Chakra – 20% of the combined capital of Laxman & Hari:
Particulars Rs.
Capital of Laxman (As per Capital Account above) 129,500
Capital of Hari (As per Capital Account above) 189,500
Combined Capital 319,000
20% of Rs. 319,000 63,800

3. Cash in hand:
Particulars Rs.
Cash balance as per Balance Sheet 10,000
Add: Cash brought in by Chakra 63,800
Total 73,800

2.
a) The hire purchase department of Whirlpool Finance Ltd. sells television sets and
room coolers. This department was newly started in 2068. The relevant
information is as follows:
Television Room
Set Coolers
Cost (Rs.) 5,400 2,000
Cash Price (Rs.) 6,300 2,400
Cash down payment (Rs.) 900 400
Monthly installment (Rs.) 600 200
Number of installments 10 12
During the year, 100 television sets and 120 room coolers were sold on hire
purchase basis. Two television sets on which 3 installments only could be
collected and 4 room coolers on which 5 installments had been collected were
repossessed. These were valued at Rs. 10,000 and after reconditioning at a cost
of Rs. 1,000 were sold outright for Rs. 14,000. Other installments collected and
those due (customer still paying) were respectively as follows:
Television sets 270 and 20
Room coolers 400 and 30
Prepare Accounts on Stocks and Debtors System to reveal the profit of the
Department. 10

b) White Ltd. agreed to acquire the business of Green Ltd. as on 32nd Ashad, 2068
The summarized Balance Sheet of Green Ltd. at that date was as follows:
Balance Sheet as on 32nd Ashad, 2068

Liabilities Rs. Assets Rs.


Share Capital in fully paid
equity shares of Rs. 10
each 6,00,000 Goodwill 1,00,000
General Reserve 1,70,000 Land and Buildings 2,30,000
Profit and Loss Account 1,10,000 Plant and Machinery 4,10,000
12% Debentures 1,00,000 Stock in Trade 1,68,000
Creditors 20,000 Debtors 36,000
YSZ P.T.O.
(4)

Cash at Bank 56,000


10,00,000 10,00,000
The consideration payable by White Ltd. was agreed as follows:
i) A cash payment equivalent to Rs. 2.50 for every Rs. 10 share in Green Ltd.

ii) The issue of 90,000 shares of Rs.10 each fully paid up in White Ltd. having an
agreed value of Rs. 15 per share.

White Ltd. also agreed to discharge the 12% Debentures of Green Ltd. at a
premium of 20% by allotment of its 14% Debentures at 96 percent.
When computing the agreed consideration, the directors of White Ltd. valued the
following assets at values noted against them:
Rs.
Land and Buildings 7,50,000
Plant and Machinery 4,50,000
Stock in Trade 1,42,000
Debtors Subject to an allowance of 5% to cover
doubtful debts.
The cost of liquidation of Green Ltd. came to Rs. 5,000 which was borne by
White Ltd.
Give ledger accounts to close the books of the Green Ltd. 10

YSZ P.T.O.
(5)

Answer No. 2
a)
Whirlpool Limited
Hire Purchase Stock A/c
Particulars Rs. Particulars Rs.
To Goods sold on H.P. A/c 1,026,000 By H.P. Debtors A/c 405,600
By Goods Repossessed A/c –
Installments not due on
repossessed goods. 14,000
By Balance c/d – Installments
not yet due 606,400
1,026,000 1,026,000

Hire Purchase Debtors A/c


Particulars Rs. Particulars Rs.
To Hire Purchase Stock A/c 405,600 By Bank A/c 387,600
By Balance c/d 18,000
405,600 405,600

Goods Repossessed A/c


Particulars Rs. Particulars Rs.
To H.P. Stock A/c 14,000 By H.P. Adjustment A/c – Bal. fig. 4,000
By Balance c/d 10,000
14,000 14,000
To Balance b/d 10,000 By Bank – Sales 14,000
To Bank – Expenses 1,000
To H.P. Adjustment A/c – Profit 3,000
14,000 14,000

Goods sold on Hire Purchase A/c


Particulars Rs. Particulars Rs.
To Hire Purchase Adj. A/c – Loading 246,000 By Hire Purchase Stock A/c 1,026,000
To Trading A/c 780,000
1,026,000 1,026,000

Hire Purchase Adjustment A/c


Particulars Rs. Particulars Rs.
To Goods Repossessed A/c - Loss 4,000 By Goods Sold on H.P. – Loading 246,000
To Stock Reserve 144,971 By Goods Repossessed A/c - Profit 3,000
To Profit 100,029
249,000 249,000

YSZ P.T.O.
(6)

Working Notes:
(i) Hire Purchase Price is Rs. 6,900 for each Television Set and Rs. 2,800 for each Room
Cooler. Total cost and sales on this basis are as follows:
H.P. Price Cost Price
Rs. Rs.
Television Sets – 100 690,000 540,000
Room Coolers – 120 336,000 240,000
Total 1,026,000 780,000

(ii) Cash Collected:


T.V. Set Cooler
Rs. Rs.
Down payment (Rs. 900 × 100) & (Rs. 400 × 120) 90,000 48,000
Installments collected (Rs. 600 × 270) & (Rs. 400 × 200) 162,000 80,000
Amount collected on repossessed goods (Rs. 600 × 2 × 3)
& (Rs. 200 × 5 × 4) 3,600 4,000
255,600 132,000
(iii) Installment not yet due:
Television Set: Total installments on 98 sets 980
Installments collected & due 290
Balance 690
Amount of 690 installments @ Rs. 600 each [A] Rs. 414,000
Room Cooler: Total installments on 116 sets 1,392
Installments collected & due 430
Balance 962
Amount of 962 installments @ Rs. 200 each [B] Rs. 192,400
Total Amount [A] + [B] Rs. 606,400

(iv) Stock Reserve:


Television Set: [1,500 / 6,900 × 414,000] Rs. 90,000
Room Cooler: [800 / 2,800 ×192,400] Rs. 54,971
Total Rs. 144,971

(v) Installment not due on repossessed goods:


2 Television Sets 7 installments on each @ Rs. 600 Rs. 8,400
4 Room Coolers 7 installments on each @ Rs. 200 Rs. 5,600
Total Rs. 14,000

(vi) Installment due but not collected:


Television Sets: (20 × 600) Rs. 12,000
Room Cooler: (30 × 200) Rs. 6,000
Total Rs. 18,000

YSZ P.T.O.
(7)

b)
In the books of Green Ltd.
Realization Account
Particulars Rs. Particulars Rs.
To Goodwill 100,000 By 12% Debenture 100,000
To Land & Buildings 230,000 By Sundry Creditors 20,000
To Plant & Machinery 410,000 By White Ltd. – Consideration 1,500,000
To Stock in Trade 168,000
To Sundry Debtors 36,000
To Bank 56,000
To Shareholders A/c – Profit 620,000
1,620,000 1,620,000
White Ltd. Account
Particulars Rs. Particulars Rs.
To Realization Account 1,500,000 By Bank 150,000
By Shares in White Ltd. 1,350,000
1,500,000 1,500,000
Equity Shareholders Account
Particulars Rs. Particulars Rs.
To Bank 150,000 By Equity Share Capital Account 600,000
To Shares in White Ltd. 1,350,000 By General Reserve 170,000
By Profit & Loss Account 110,000
By Realization Account – Profit 620,000
1,500,000 1,500,000
Cash Book (Bank Column)
Particulars Rs. Particulars Rs.
To Balance b/d 56,000 By Realization Account 56,000
To White Ltd. 150,000 By Equity Shareholders Account 150,000
206,000 206,000
Working Note:
Calculation of purchase consideration:
Rs.
Cash [Rs. 2.5 × 60,000] 150,000
Shares [Rs. 15 × 90,000] 1,350,000
Total 1,500,000

YSZ P.T.O.
(8)

3.
a)
i) What is Profit or Loss prior to Incorporation? 2

ii) M. Ltd. resolved on 32nd Ashad, 2067 that the company be wound up
voluntarily. The following was the trial balance extracted from its
books as on that date:

Dr. Cr.
Rs. Rs.
Equity shares of Rs. 10 2,00,000
9% Preference shares of Rs. 10 each 1,00,000
Plant (Less depreciation w/off Rs. 85,000) 2,15,000
Stock in trade 2,50,000
Sundry Debtors 55,000
Sundry Creditors 75,000
Bank Balance 74,000
Preliminary Expenses 6,000
Profit & Loss A/c (Balance as on 1-4-2066) 30,000
Trading loss for the year 2066/67 24,000
Preference dividend for the year 2066/67 6,000
Outstanding expenses (Including mortgage interest) 25,000
4% Mortgage loan 2,00,000
Total 6,30,000 6,30,000
On 1st Shrawan, 2067 the liquidator sold to N. Ltd. Plant for Rs. 2,05,000
and stock in trade for Rs. 2,00,000. The sale was completed in Shrawan,
2067 and the consideration satisfied as to Rs. 2,62,200 in cash and as to
the balance in 6% Debentures of the purchasing company issued to the
liquidator at a premium of 2%.
The remaining steps in the liquidation were as follows:
(a) The liquidator realized Rs. 52,000 out of the book debts and the cost
of collection amounted to Rs. 2,000.

(b) The loan mortgage was discharged on 31st Shrawan, 2067 along with
interest from 31st Magh 2066. Creditors were discharged subject to
discount of 2% and outstanding expenses excluding mortgage interest
were settled for Rs. 2,000;

(c) On 30th Poush 2067 six month‟s interest on debentures was received
from N. Ltd.

(d) Liquidation expenses amounting to Rs. 3,000 and liquidator‟s


remuneration of 3% on disbursements to members were paid on 30th
Poush, 2067 when:

 The preference shareholders were paid out in cash; and

 The debentures of N. Ltd. and the balance of cash were distributed


ratably among the equity shareholders.
YSZ P.T.O.
(9)

Prepare the Liquidator‟s Statement of Account showing the distribution. 8


b) The following is the Balance Sheet of a concern on 32.3.2067:
Liabilities Rs. Assets Rs.
Capital 10,00,000 Fixed Assets 4,00,000
Creditors (Trade) 1,40,000 Stock 3,00,000
Profit & Loss A/c 60,000 Debtors 1,50,000
________ Cash & Bank 3,50,000
12,00,000 12,00,000
The management estimates the purchases and sales for the year ended
32.3.2068 as under:
Up to Jestha 2068 Ashad 2068
Rs. Rs.
Purchases 14,10,000 1,10,000
Sales 19,20,000 2,00,000
It was decided to invest Rs. 1,00,000 in purchases of fixed assets, which
are depreciated @ 10% on cost.
The time lag for payment to Trade Creditors for purchase and receipt
from Sales (Debtor's) is one month. The business earns a gross profit of
30% on turnover. The expenses against gross profit amount to 10% of the
turnover. The amount of depreciation is not included in these expenses.
Draft a Balance Sheet as at 32.3.2068 assuming that creditors are all
Trade Creditors for purchases and Debtors for sales and there is no other
item of current assets and liabilities apart from stock and cash and bank
balances. 10
Answer No. 3
a)
i) When a running business is taken over by the promoters of a company, from a date
before the company which is to manage and own the business is registered, the amount
of profit or loss of such a business for the period prior to the date the company came
into existence is referred to as pre incorporation profits or losses. Such profits or
losses, though belonging to the company or payable by it, are of capital nature, it is
necessary to disclose them separately.

ii)
M. Ltd. (in liquidation)
Liquidator’s Statement of Account from 1st Shrawan, 2067 to 30th Poush, 2067
Particulars Rs. Rs. Particulars Rs. Rs.
Balance at Bank 74,000 Liquidators Remuneration 7,302
Realization from: Liquidation Expenses 3,000
Sundry Debtors 52,000 Mortgage loan with Interest 204,000
Less: Cost of Collection 2,000 50,000 Creditors including Outstanding Exp. 75,500
N Ltd.: 6% Pref. Shareholders @ Rs. 10 each 100,000
6% Debenture @ Rs. 102 142,800 Equity Shareholders:
Cash 262,200 405,000 6% Debentures 142,800
Interest on Debentures – 6 months 4,200 Cash – Approximately
03 paisa per share 598 143,398
533,200 533,200

YSZ P.T.O.
(10)

Working note:
(i) Calculation of Liquidators Remuneration:
Particulars Rs. Rs.
Total realization including debentures in N Ltd. 533,200
Distribution to outsiders:
Liquidation expenses 3,000
Mortgage loan with interest 204,000
Creditors including outstanding expenses 75,500 282,500
Amount available for liquidators remuneration & members settlement 250,700
Liquidators remuneration [Rs. 250,700 × 3 / 103] 7,302

(ii) It is assumed that loan is secured by a floating charge.

(iii) Calculation of debentures to be issued: value of Assets sold 205,000

Add: value of stock sold 200,000

Total 405,000

Less: cash received 262,200

6% debenture @ Rs. 102 = 142,800

(Assumption: face value of debenture to be issued at Rs. 100 each)

b)
Projected Balance Sheet of......
As on 32.3.2068
Liabilities Rs. Assets Rs.
Capital 1,000,000 Fixed Assets 450,000
Profit & Loss Account: Stock in Trade 336,000
01.04.2067 60,000 Sundry Debtors 200,000
Add: Profit for the year 374,000 434,000 Cash & Bank Balances 558,000
Trade Creditors 110,000
1,544,000 1,544,000

Working Notes:
1. Projected Trading and Profit and Loss Account
For the year ended 32.3.2068
Particulars Rs. Particulars Rs.
To Opening Stock 300,000 By Sales 2,120,000
To Purchases 1,520,000 By Closing Stock – Bal. fig. 336,000
To Gross Profit c/d – 30% on Sales 636,000
2,456,000 2,456,000
To Sundry Expenses – 10% on Sales 212,000 By Gross Profit b/d 636,000

YSZ P.T.O.
(11)

To Depreciation 50,000
To Net Profit 374,000
636,000 636,000

2. Cash and Bank Account


Particulars Rs. Particulars Rs.
To Balance b/d 350,000 By Sundry Creditors - Payment 1,550,000
To Sundry Debtors - Collection 2,070,000 By Expenses 212,000
By Fixed Assets 100,000
By Balance c/d 558,000
2,420,000 2,420,000
3. Fixed Assets and Depreciation:
Rs.
Opening Balance 400,000
Add: Purchases during the year 100,000
500,000
Less: Depreciation @ 10% 50,000
Closing Balance 450,000

4. Sundry Debtors A/c

Particulars Rs. Particulars Rs.


To Balance b/d 150,000 By Cash & Bank 2,070,000
To Sales 2,120,000 By Balance c/d 200,000
2,270,000 2,270,000

5. Sundry creditors A/c

Particulars Rs. Particulars Rs.


To Cash & Bank 1,550,000 By Balance b/d 140,000
To Balance c/d 110,000 By Purchases 1,520,000
1,660,000 1,660,000

YSZ P.T.O.
(12)

4.
a) On 30/06/2068, an accidental fire destroyed a major part of the stocks in the shop
of Davidson Enterprises. Stocks costing Rs. 60,000 could be salvaged but not their
stores ledgers. A fire insurance policy was in force under which the sum insured
was Rs. 700,000. From available records, the following information was retrieved:
i) Total of sales invoices during the period 01/04/2068-30/06/2068 amounted to
Rs. 6,040,000. An analysis showed that goods of the value of Rs. 600,000 had
been returned by the customers before the date of the fire.
ii) Opening stock as on 01/04/2068 was Rs. 440,000 including stocks of the value
of Rs. 40,000 being lower of cost and net value subsequently realized.
iii) Purchases during the period 01/04/2068-30/06/2068 amounted to Rs. 4,200,000.
iv) Normal gross profit rate was 33.33% on sales.
v) A sum of Rs. 60,000 was incurred by way of fire fighting expenses on the day
of the fire.
Prepare a statement showing the Insurance Claim Recoverable. 5
b) A company sold 25% of the goods on cash basis and the balance on credit basis.
Debtors are allowed 2 months credit and their balance as on 31.3.2011 is Rs.
140,000. Assume that the sale is uniform throughout the year. Calculate the total
sales of the company for the year ended 31.3.2011. 5
c) What are the classifications of Loans/Advances as per Nepal Rastra Bank
Directives for Banking & Financial Institution? 5

Answer No. 4
a) Calculation of Insurance Claim Recoverable
Stock Destroyed: Rs.
Stock on the date of fire – Working note 1,000,000
Less: Salvaged Stock 60,000
Stock Destroyed by fire 940,000
As the policy amount is less than the value of Closing Stock, average clause will apply.
Therefore, Amount of Claim = [Policy Amount × Loss / Stock on the date of fire]
= [Rs. 700,000 × 940,000 / 1,000,000]
= Rs. 658,000
As fire fighting expenses have been incurred, these expenses may be admitted as part of
the loss. In that case loss will be Rs. 940,000 + Rs. 60,000 = Rs. 1,000,000.
Applying average clause, claim = [Rs. 700,000 × 1,000,000 / 1,000,000] = Rs. 700,000
Working note:
Memorandum Trading Account
for the three months period ended on 30th Aswin, 2068
Particulars Rs. Rs. Particulars Rs. Rs.
To Opening Stock 440,000 By Sales 6,040,000
Less: Abnormal Item 40,000 400,000 Less: Abnormal Item 40,000
To Purchases 4,200,000 6,000,000
To Gross Profit – 1/3 of Sales
rd
1,800,000 Less: Return 600,000 5,400,000
By Closing Stock – Bal. figure 1,000,000
6,400,000 6,400,000
YSZ P.T.O.
(13)

b) Calculation of Total Sales for the year ended 31st March, 2011
Since debtors are allowed two months credit term, debtors as on 31st March (Rs. 140,000)
represents two months (February & March) credit sales. Therefore, monthly credit sales
are Rs. 70,000. As per question, cash sales is 25% of total sales and sales being uniform
throughout the year, total sales will be calculated as follows:
Rs.
Total Credit Sales for the year [Rs. 70,000 × 12] 840,000
Add: Cash Sales – Being 25% of total sales [Rs. 840,000 × 25 / 75] 280,000
Total Sales 1,120,000

c) Entire loans and advances extended by a licensed institution have to be classified as


follows based on expiry of the deadline of repayment of the principal and interest of such
loans/advances:
1. Pass: Loans/advances which have not overdue and which are overdue by a period up to
three months.
2. Sub-Standard: Loans/advances which are overdue by a period from three months to a
maximum period of six months.
3. Doubtful: Loans/advances which are overdue by a period from six-months to a
maximum period of one year.
4. Loss: Loans/advances which are overdue by a period of more than one year.

The loans which are in pass class and which have been rescheduled/restructured are called
as "the performing loan, and the sub-standard, doubtful and loss categories are called non-
performing loans.
Note: Loans/advances also include bills purchased and discounted.

5.
a) Explain „Recognition of Assets‟ as per Framework for the Preparation
and Presentation of Financial Statements under NAS. 5
b) What are the items that are to be excluded in determination of the cost of
inventories as per NAS-04? 5
c) X Limited has recognized Rs. 10 lakhs on accrual basis income from
dividend on shares of the face value of Rs. 50 lakhs held by it as at the
end of the financial year 32.3.2068. The dividends on shares were
declared at the rate of 20% on 15.6.2068. The dividend was proposed on
10.4.2068 by the declaring company. Whether the treatment is as per the
relevant Accounting Standard? Answer with reference to provisions of
NAS-07. 5

Answer No. 5
a) An asset is recognized in the balance sheet when it is probable that the future economic
benefits will flow to the enterprise and the asset has a cost or value that can be measured
reliably.
An asset is not recognized in the balance sheet when expenditure has been incurred for
which it is considered improbable that economic benefits will flow to the enterprise
beyond the current accounting period. Instead such a transaction results in the recognition
of an expense in the income statement. This treatment does not imply either that the
intention of management in incurring expenditure was other than to generate future
economic benefits for the enterprise or that management was misguided. The only

YSZ P.T.O.
(14)

implication is that the degree of certainty that economic benefits will flow to the enterprise
beyond the current accounting period is insufficient to warrant the recognition of an asset.

b) Items that are to be excluded in determination of the cost of inventories as per NAS 4 on
„Valuation of Inventories‟ are:
1. Abnormal amounts of wasted materials, labor or other production costs.
2. Storage costs unless those costs are necessary in the production process prior to a
further production stage.
3. Administrative overheads that do not contribute to bringing the inventories to their
present location and condition; and
4. Selling costs.

c) NAS 07 states that dividends from investments in shares are not recognized in the
statement of profit and loss until a right to receive payment is established. In the given
case, the dividend is proposed on 10.4.2068, while it is declared on 15.6.2068. Hence, the
right to receive payment is established only on 15.6.2068. As per NAS 07, income from
dividend on shares should be recognized by X Ltd. in the financial year ended 31.3.2069.
The recognition of Rs. 10 lakhs on accrual basis in the financial year 2066.68 is not as per
NAS 07 'Revenue.'

6. Write short notes on following: (4×2.5=10)


a) Going Concern
b) Classification of Reserves
c) Re-Insurance
d) Sinking Fund

Answer No. 6
a) The financial statements are normally prepared on the assumption that an enterprise is a
going concern and will continue in operation for the foreseeable future. Hence, it is
assumed that the enterprise has neither the intention nor the need to liquidate or curtail
materially the scale of its operations; if such an intention or need exists, the financial
statements may have to be prepared on a different basis and, if so, the basis used is
disclosed.
b) Reserves are generally classified into: (a) Capital Reserves and (b) Revenue Reserves.
1. Capital Reserves are those which are not distributed (expected for certain
circumstances) as profits as a matter of law, prudence or business policy. They arise
mainly out of capital profits, for example, profits on sale of fixed asset or upward
revaluation of fixed assets or from capital receipts, e.g. issue of shares or debentures at
a premium. They also arise from non–trading incomes during the period prior to
incorporation. These reserves may or may not involve any receipts of cash.
2. Revenue Reserves are any reserves which are not capital reserves and are available for
distribution as profits. These are created by retaining profits. Examples of revenue
reserves are credit balance of the Profit and Loss Account, general reserve, dividend
equalization reserves, investment fluctuation reserves etc. Revenue reserves can
further be classified into: (a) General Reserves and (b) Specific Reserves.
(i) General Reserves: These reserves are not created for any particular purpose.
These are created for safeguarding the business against unforeseen losses in the
future or with a view to planning for further development of the business. General
reserves are not earmarked against any particular asset.

YSZ P.T.O.
(15)

(ii) Specific Reserves: These reserves are created for some specific purpose and are
utilized for these purposes only. These are generally earmarked against some
particular asset and expressed as „reserve fund‟. An amount of the reserve created
is invested outside the business in securities for a specified period. At the end of
that specified period, all investments are sold away. The proceeds are utilized for
meeting that particular purpose for which the reserve was created.

c) Re – Insurance:
If an insurer does not wish to bear the whole risk of policy written by him, he may reinsure
a part of the risk with some other insurer. In such a case the insurer is said to have ceded a
part of his business to other insurer. The reinsurance transaction may thus be defined as an
agreement between a „ceding company‟ and „reinsurer‟ whereby the former agreed to
„cede‟ and the latter agrees to accept a certain specified share of risk or liability upon terms
as set out in the agreement. A „ceding company‟ is the original insurance company which
has accepted the risk and has agreed to „cede‟ or pass on that risk to another insurance
company or a reinsurance company. It may however be emphasized that the original
insured does not acquire any right under a reinsurance contract against the reinsurer. In the
event of loss, therefore, the insured‟s claim for full amount is against the original insurer.
The original insurer has to claim the proportionate amount from the reinsurer. There are
two types of reinsurance contracts, namely, facultative reinsurance and treaty reinsurance.
Under facultative reinsurance each transaction has to be negotiated individually and each
party to the transaction has a free choice, i.e., for the ceding company to offer and the
reinsurer to accept. Under treaty reinsurance a treaty agreement is entered into between
ceding company and the reinsurer whereby the volume of the reinsurance transactions
remain within the limits of the treaty.

d) Sinking Fund:
The most common method of supplementing capital available to a company is to issue
debentures, which are usually redeemable. Redeemable debentures may be redeemed after
a fixed number of years or any time after a certain number of years has elapsed since their
issue, on giving a specified notice or by annual drawing.
Usually, according to the conditions of the issue, the company is required to create a fund
by appropriating annually a certain percentage of, or a fixed amount out of, its profit. The
fund so created is normally known as Sinking Fund or Debenture Redemption Reserve
Fund. The company invests the amount of the fund either in the purchase of securities
which are readily saleable or takes a policy that shall mature at the time the debentures will
fall due for payment. Such an arrangement would ensure that the company will have
sufficient liquid funds for the redemption of debentures at the time they shall fall due for
payment. In the case of debentures that are redeemable at premium, the appropriation to
the Sinking Fund should be sufficient to pay both the amount of debenture and the
premium on redemption. On redemption of debentures, out of the balance lying in the
Sinking Fund an amount equal to the debentures so redeemed is transferred to General
Reserve.

YSZ P.T.O.
(16)

CAP-II, Audit & Assurance, Dec 2011


Suggested Answer
Roll No……………. Maximum Marks - 100

Total No. of Questions- 7 Total No. of Printed Pages- 2


Time Allowed - 3 Hours
Marks
Attempt all the questions.
1. G
ive your opinions with reasons on the following cases: (45=20)
a) Audit of the Sagarmatha Byapar Co. Pvt. Ltd. is in final stage and the audit
report is being drafted. Management of the company, including Chief
Executive Officer, is of the view that auditors are primarily responsible for
preparation and fair presentation of financial statements. Management of the
company is of the view that they are free to prepare accounts in their own way
and it is the duty of the auditor to satisfy about financial statements to
government and other regulatory agencies.
b) Grand Industries Ltd. is engaged in manufacturing and supply of gear boxes to
Suzaki Vehicles Ltd. As per terms of supply, full price of the goods are not
released by Suzaki Vehicles Ltd. but 15% thereof is retained and paid after
one year, if there is satisfactory performance of the parts supplied. Grand
Industries Ltd. accounts for only 85% of the invoice value as sale at the time
of supply and balance 15% is accounted as sale in the year of receipt of
payment.
c) During the audit period, the liaison officer deputed by the client Lumbini
Insurance Company constantly asks you to participate in lunch arranged
outside office premises. He has also informed you that there is a program of
welcome and farewell dinner too. Liaison officer has also informed you that
you need not worry about time schedule, as there would be someone available
among client's staff to bailout if needed. Daily newspapers and magazines are
made available in plenty in the audit room. Other staff of the client are also
having regular visit in the working area.
d) The Company`s plant & machinery was Rs. 200 million as on 1st Shrawan
2067. It provided depreciation at 15% per annum under WDV method.
However it noticed that about Rs. 20 million worth of imported asset, which is
component of above plant & machinery acquired on 1st Shrawan 2067, would
be obsolete in 2 years. Company wants to write off this asset over 2 years. Can
Company do so? Give Comments.
Answer No. 1
a) Paragraph 15 of Nepal Standard on Auditing (NSA 200) mentions that while the
auditor is responsible for forming and expressing an opinion on the financial statements,
the responsibility for preparing and presenting the financial statements is that of the
management of the entity. The audit of the financial statements does not relieve
management of its responsibility. Management rather than the auditor is responsible for
the preparation and fair presentation of financial statements. This responsibility
includes designing, implementing and maintaining internal control relevant to the
preparation and fair presentation of financial statements that are free from material
misstatements, whether due to fraud or error, selecting and applying appropriate
YSZ P.T.O.
(17)

accounting policies, and making accounting estimates that are reasonable in the
circumstances. Thus, the contention of the management is not correct.
b) According to NAS-7 on Revenue, revenue recognition from sale of goods should be
recognized when the seller has transferred to the buyer, the property or the goods for a
price or when the seller has transferred all significant risk and rewards and the seller
has no effective control over goods and no significant uncertainty exists regarding the
amount of consideration and its collectability. In the given case the goods as well as
the risk and ownership has been transferred by Grand Industries Ltd., to Suzaki
Vehicles Ltd., on the basis of invoice and delivery of material. In the instant case,
therefore, Grand Industries Ltd. should recognize sale at full 100% of the invoice
value in spite of the fact that 15% payment will be released after one year. However,
depending upon the past experience regarding collectability of 15% amount, they can
make a provision for the amount that is not likely to be realized. Hence, the treatment
given by the company is not correct and if they do not correct it, the auditor should
qualify his report.

c) Section 260 of Code of Ethics for the members of the Institute of Chartered
Accountants of Nepal -2060 prescribed the ICAN states about the gift and hospitality
to the auditors by the client.

The existence and significance of any threat will depend on the nature, value, and
intent of the offer. Where gifts or hospitality are offered that a reasonable and
informed third party, weighing all the specific facts and circumstances, would
consider trivial and inconsequential, a professional accountant in public practice may
conclude that the offer is made in the normal course of business without the specific
intent to influence decision making or to obtain information. In such cases, the
professional accountant in public practice may generally conclude that any threat to
compliance with the fundamental principles is at an acceptable level.

The objectives of the accountancy profession are to work to the highest standards of
professionalism, to attain the highest levels of performance and generally to meet the
public interest requirement set out above. These objectives require four basic needs to
be met. They are credibility, professionalism, quality of services and confidence. An
auditor is expected to maintain the highest degree of integrity, professionalism and
independence. He represents the trust and faith of the whole professional body. There
could be various obstacle and impediments while conducting his professional duty.
However, he should remain intact and vigilant towards achieving his professional
goal.

Provision of lunch is governed by conditions laid in engagement letter and


appointment letter. If the contract states provision of lunch, it should not be construed
as accepting any undue advantage whether arranged inside or outside. However,
attendance in farewell party is not concerned with the auditor. Thus, the auditor needs
to avoid such attendance. Receiving Newspapers as courtesy from the client and the
YSZ P.T.O.
(18)

regular visits by the staff of the client in the working place is not the problem but it
must be seen that there would be no compromise in the professional ethics and
standards.

d) As per Nepal Accounting Standard 6 (Property, Plant and Equipment), where the
addition or extension retains a separate identity and is capable being used after the
existing asset is disposed off, depreciation should be provided independently on the
basis of an estimate of its own useful life. As it appears that imported assets of Rs. 20
million, which is component of plant and machinery, is having independent useful
life. Therefore, the company`s policy to write off over two years is correct.

2. A
nswer the following:
a) You are appointed as manager of quality control section in one of the leading
audit firm of Nepal, the senior partner of the firm instructs you to draft
objectives statement for quality control policies & procedures. 8
b) List out the analytical procedures that you would adopt in audit of Revenue of
an entity. What are the factors that determine the extent of reliance on such
analytical procedures? 7

Answer No. 2
a) As per Nepal Standards on Quality Control(NSQC) 1, it is necessary for quality control
for firm`s that perform audits and review of historical financial information and other,
accordingly, the relevance of quality control The relevance of quality control issue in
auditing services is equal importance like in other services. If quality control could not
be done in auditing work, quality services could not be provided to client which may
result negative consequences both for auditor and client including in overall all
economy. Hence, every auditor should carry out the audit work based on formal quality
control policy & procedures. The objective statement of quality control policies &
procedures would be:
i. Professional Requirements
personnel in the firm should adhere to the principles of independence,
integrity, objectivity, confidentiality and professional behavioral number
of procedures may be framed to achieve this objective .for example, a
firm may require all its personnel to make a written statement every year
as to whether they hold any shares or any other interest in the enterprises
being audited by the firm.
ii. Skills and Competence
the audit skills firm should be staffed by personnel who have attained
and maintain the technical standards and professional required to enable
them to fulfill their responsibilities with due care .for example, a firm
can achieve this objective through proper recruitment procedures,
periodic staff and a system whereby latest information relating to current
development in professional standards, law, etc is regularly
communicated to audit staff.
iii. Assignment
Audit work should be assigned to such personnel as have the degree of
technical training and proficiency required in the circumstances.
iv. Delegation

YSZ P.T.O.
(19)

There should be sufficient direction, supervision and review of work at


all levels to provide reasonable assurance that the work performed meets
appropriate quality standards. For example, a firm may establish
guidelines relating to the form and content of working papers; use of
standardized forms, etc .similarly, the audit plans may identify the
staffing requirements and timing of various phases of audit to facilitate
delegation of audit now.
v. Consultation
Where ever necessary, persons having appropriate expertise, within or
outside the firm, should be consulted.
vi. Acceptance and Retention of Clients
Before accepting an audit, the firm should evaluate its independence and
ability to serve the prospective client properly. A similar review should
be made, on-going basis, of association with the existing clients.
vii. Monitoring
The continued adequacy and operational effectiveness of the quality
control policies and procedures should be maintained.

b) Analytical procedures are one of many audit processes which help an auditor
understand the client's business and changes in the business, and to identify potential
risk areas to plan other audit procedures. It includes comparison of financial
information, relating financial and non financial information and consideration of
practicable relationship of data.

The analytical procedures that will be adopted in obtaining audit evidence regarding the
various assertions relating to revenue are as follows:
i. Comparison of Gross-profit ratio to sales for the current year with the
corresponding figures of the previous years.
ii. Comparison of ratio of sales returns to sales for the current year with the
corresponding figures for previous years.
iii. Comparison of trade discount to sales for the current year with previous year.
iv. Review of Reconciliation of Excise/VAT booked during the year with
Excise/VAT returns submitted with the total sales booked.
v. Comparison of dividend/interest/royalty for the current year with the
corresponding figures for previous years.
vi. Comparison of ratio of income on investments to average investment for the
current year with corresponding figures for the previous year.

The factors that affect the extent of reliance on analytical procedures are as follows:
i. Materiality: When items are material, the auditor doesn‟t solely rely on the
analytical procedures in forming conclusions but will carry other substantive
procedures also.
ii. Other procedures: When other procedures are also directed towards the same
objective, it might confirm or dispel the questions raised from the application of
analytical procedures.
iii.Weak controls: When internal controls are weak, greater reliance is placed on tests
of balances and tests of details of transactions rather than on analytical procedures.
iv. Accuracy: The accuracy with which expected results of analytical procedures can be
predicted. For example greater reliance is place on gross profit ratio compared to
previous year than in comparing discretionary expenses such as donation.

YSZ P.T.O.
(20)

3. G
ive your comments on the following: (35=15)
a) Mr. Alex Pitt, partner of M/s Pitt Marwick & Associates, a Chartered
Accountant firm was appointed as an auditor of Cosmopolitan Commercial
Bank Ltd. One of the shareholders of the bank lodged a complain against the
auditor for not assuming the responsibilities to consider laws and regulations
with regards to lending to a business house in an audit of financial statements
of the bank. You are asked to identify the auditors' responsibilities to consider
laws and regulations in an audit of financial statements of bank with regards to
complaint lodged against the auditor?

b) On being appointed, the auditor of a company for the first time you find that
the cashier also handles the books of account and the cash receipt are not
being banked intact but parts of these are being utilized for cash payments.
What are the risks involved? What could be your recommendations to mitigate
the risks envisaged?
c) Xylocain Communications Company is one of the leading companies dealing
with various telecommunication services, wireless networking and internet
related services. It receives huge collection of cash daily from its customer on
various accounts. Everything looked well, until one day it was noticed that
large amount was missing in the bank account. In fact that said amount was
not credited in the bank account at all and it was missing since last few years.
When the case was revealed, probably it was too late to recover the money.
Considering the above case, what are the likely documents you would like to
examine as an auditor of the company?
Answer No. 3
a) In accordance with NSA 200: “Objective and General Principles Governing an Audit of
Financial Statements,” the auditor should plan and perform the audit with an attitude of
professional skepticism recognizing that the audit may reveal conditions or events that
would lead to questioning whether an entity is complying with laws and regulations. In
accordance with specific statutory requirements, the auditor may be specifically
required to report as part of the audit of the financial statements whether the entity
complies with certain provisions of laws or regulations. In these circumstances, the
auditor would plan to test for compliance with these provisions of the laws and
regulations.
Further, the auditor should obtain sufficient appropriate audit evidence about
compliance with those laws and regulations generally recognized by the auditor to have
an effect on the determination of material amounts and disclosures in financial
statements. The auditor should have a sufficient understanding of these laws and
regulations in order to consider them when auditing the assertions related to the
determination of the amounts to be recorded and the disclosures to be made. Section
115(3) (e) of the Nepal Company Act, 2063 specially states that the auditor should
report any non compliance of prevailing laws. Further, Banks and financial Institutions
Act 2063 also requires report of auditors on certain matters. Thus, Here, in given
situation, the auditor should plan his audit and report for lending to a business house in
accordance with the prevailing banking rules and regulations. He should have sufficient
knowledge of statutory provisions of lending and its reporting in prescribed format.

b) Since the cashier handles books of accounts and cash is not deposited intact in the bank
account the internal control system of the company is found very weak. There are the
risks of cash being misappropriated. Examples could be:
 failure to record purchases properly in order to misappropriate cash.
 misappropriation cash from a machine or whilst cash is in transit.
 acceptance or solicitation of money or a benefit to provide cash to a third party.
YSZ P.T.O.
(21)

 acceptance or solicitation of money or a benefit to provide goods or services to a


third party without taking a cash payment from that party.

This may create significant risk to the company that may erupt from the weak internal
control system. Recommendations to company for the enhancement of the internal
control system of the company are as follows:
a. The cashier should not have access to the books of accounts. The books of account
should be handled by the accountants who should not be directly involved with the
cash transaction.
b. All the cash collections should be deposited to the bank.
c. Cash payments should be made through the Bank.
d. Petty expenses should be managed through the Petty cash book, for which purpose
the petty cash, imprest should be provided to the petty cashier through the bank.

c) Considering the case presented the auditor of Xylocain Communications Company


should check all the books of accounts and documents related with the cash and bank
transactions. Hence, the likely documents the auditor should examine in the
circumstances presented are mentioned hereunder:

i) Separ
ate Cash receipt register for different services like tele-communication, wireless
networking and internet related services
ii) Prenumbered Receipt pad for collecting cash.
iii) Security arrangement in the counter area.
iv) Procedures followed to deposit the amount in the bank
v) Special arrangements with bank for transferring funds, if any
vi) Bank reconciliation statement
vii) Duty rotation of the personnel, directly handling cash collection
viii) Reporting arrangement to central office
ix) Fidelity Insurance etc
x) Bank deposit slips

4. A
nswer the following: (35=15)
a) What is the relationship between materiality and audit risk and how audit risk
can be reduced to an acceptable level?
b) What are the points to be considered while using the work of an expert?
c) What are the steps followed while conducting audit of incomplete records?
Answer No. 4

a)
The auditor is concerned with expressing opinion on true and fairness of the the
financial statements. Similarly, as per NSA-240 Nepal Standards on Auditing the
auditors' responsibility to consider fraud and error in an audit of financial statements. Audit
Risk is the risk that the auditor gives an inappropriate audit opinion when the financial
statements are materially misstated or the frauds and errors are not reported. Due to nature
of audit evidence and the characteristics of fraud, the auditor is not able to obtain absolute
assurance of detecting misstatements and frauds. Thus, the auditor gives only reasonable
assurance that material misstatements are detected.

YSZ P.T.O.
(22)

The concept of materiality recognizes that some matters, either individually or in the
aggregate, are important for fair presentation of financial statements in conformity with
generally accepted accounting principles, while other matters are not important. As per
NSA-320 on Audit Materiality the auditor should consider materiality and its relationship
with audit risk when conducting an audit. "Materiality" is defined in the Nepal Accounting
Standards Board's "Framework for the Preparation and Presentation of Financial
Statements" in the following terms: "Information is material if its omission or misstatement
could influence the economic decisions of users taken on the basis of the financial
statements. Materiality depends on the size of the item or error judged in the particular
circumstances of its omission or misstatement. Thus, materiality provides a threshold or
cut-off point rather than being a primary qualitative characteristic which information must
have if it is to be useful."

According to NSA-320 on Audit materiality, there is an inverse relationship between


materiality and degree of audit risk. Higher the materiality level, lower the audit risk
and vice-versa. The risk that a particular account balance or class of transaction would
be mis-stated by an extremely large amount might be very low. But the risk that it could
be mis-stated by an extremely small amount might be very high. The auditor considers
this inverse relationship when he determines the nature, timing and extent of his audit
procedures. If after planning for specific audit procedures, he concludes that acceptable
materiality level is lower, audit risk is increased. The auditor should try to reduce the
audit risk to an acceptable level by:-
i. Reducing the assessed degree of control risk by carrying out extended or
additional test of control, or
ii. Reducing detection risk by modifying the nature, timing and extent of his
substantive procedures.

b) NSA 620 Nepal Standards on Auditing using the work of an expert prescribes that when
using the work performed by an expert, the auditor should obtain sufficient appropriate audit
evidence that such work is adequate for the purposes of the audit. During the audit, the auditor
may seek to obtain, in conjunction with the client or independently, audit evidence in the
form of reports, opinions, valuations, and statements of an expert. Examples are:

a. Valuations of certain types of assets, for example, land and buildings, plant and
machinery, works of art, and precious stones.
b. Determination of quantities or physical condition of assets, for example,
minerals stored in stockpiles, mineral and petroleum reserves, and remaining
useful life of plant and machinery.
c. Determination of amounts using specialized techniques or methods, for
example, an actuarial valuation.
d. The measurement of work completed and to be completed on contracts in
progress for the purpose of revenue recognition.
e. Legal opinions concerning interpretations of agreements, statutes, regulations,
notifications circulars, etc.

When determining whether to use the work of an expert or not, the auditor should
consider:
a) The materiality of the item being examined in relation to the financial
information as a whole.
b) the nature and complexity of the item including the risk of error therein,
and
YSZ P.T.O.
(23)

c) the other audit evidence available with respect to the item.


When the auditor plans to use the expert‟s work as audit evidence, he should satisfy
himself as to the expert‟s skills and competence by considering the expert‟s
professional certification, license or membership in an appropriate professional body
and experience and reputation in the field in which the auditor is seeking evidence.
However, when the auditor uses the work of an expert employed by him, he will not
need to inquire into his skills and competence.
The auditor should also consider the objectivity of the expert. The risk that an expert‟s
objectivity will be impaired increases when the expert is employed by the client, or
related in some other manner to the client.
Accordingly, in these circumstances, the auditor should (after taking into account the
factors stated above) consider performing more extensive procedures than would
otherwise have been planned, or lie might consider engaging another expert.

c) Incomplete records emanate risk of misstatement of financial statement due to omission


and wrong recording of transactions. Therefore, the auditor should be careful in this
regard. Steps that are followed in audit of incomplete records are as below:
a) Ascertain the exact status of accounting records available including memoranda
records.
b) Ensure that the management compiles / reconstructs accounting records to the
extent practicable.
c) Perform compliance procedure to assess whether any control system is in
operation.
d) Vouch transactions recorded in books of account with reference to appropriate
audit evidence. Check posting, casting etc.
e) Examine the system in operation in respect of custody managed cash memos,
receipts, check books etc.
f) Verify fixed assets by observing physical verification.
g) Conduct surprise checks to verify cash in hand, inventory etc.
h) Apply analytical review procedures in depth and notice deviations to investigate
in detail.
i) Formulate an appropriate audit opinion based on above findings.

5. A
nswer the following: (35=15)
a) Mr. Saroj Khandelwal, a professional accountant and member of ICAN
performs his auditing services in a country other than Nepal also. He
published his advertisement in one of the country other than Nepal explaining
about his competency and skill to perform auditing, management and
consultancy services at very reasonable fee. He argued that it was permitted
by the local ethical requirement of that country. How would you analyze this
issue if you were a member of disciplinary committee of ICAN and the issue
had been lodged before the committee?
b) Mr. Shushil Maharjan, a Chartered Accountant published a book and gave his
personal details as the author. These details also mentioned his professional
experience and his present association as partner with M/s Daniel Brisk, a
Swedish firm of Chartered Accountant Firm. What will be your opinion in this
respect?
YSZ P.T.O.
(24)

c) Mr. Shanti Das a Chartered Accountant audited a religious institution


established for purely promoting social values without charging any audit fee.
Give your comment.
Answer No. 5
a) The issue mentioned in the question above should be analyzed as per section 6 of the
Code of Ethics issued by the ICAN to its members which states that a professional
accountant qualifying in Nepal may reside in another country or may be temporarily
visiting that country to perform professional services. In all circumstances, the
professional accountant should carry out professional services in accordance with the
relevant technical standards and ethical requirements. When a professional accountant
performs services in a country other than Nepal and differences on specific matters
exist between ethical requirements of the two countries the following provisions should
be applied:
i. When the ethical requirements of the country in which the services are
being performed are less strict than the ICAN Code of Ethics, then the
ICAN Code of Ethics should be applied.
ii. When the ethical requirements of the country in which services are
being performed are stricter than the ICAN Code of Ethics, then the
ethical requirements in the country where services are being performed
should be applied.
iii. When the ethical requirements of Nepal are mandatory for services
performed outside that country and are stricter than set out in (i) and (ii)
above, then the ethical requirements of Nepal should be applied.
Hence, in the given context, Mr Saroj Khandelwal will be held liable for disciplinary
action under the Codes of Ethics of ICAN.

b) The supplementary Directive to code of ethics issued by the Institute of Chartered


Accountants of Nepal on “Publicity and Advertisement” prescribes the provisions for
publishing book or article. It allows the members to state their name, professional
qualification and the name of their firms but restricts the members from disclosing the
services provided by them. In the given case, Mr. Shushil Maharjan. a chartered
accountant, published the book and mentioned his professional experience and his
association as a partner with M/s Daniel Brisk, a Swedish firm of chartered accountants.
Mr. Shushil Maharjan being a chartered accountant in practice has committed the
professional misconduct by mentioning that at present he is a partner in M/s Daniel
Brisk, a Swedish firm of chartered accountants.

c) The Code of Ethics issued by the Institute of Chartered Accountants of Nepal on


“Soliciting Business, Accepting New Engagement, Fees and Commission” prescribes
the provisions for providing service free of cost. Subject to non compromise in the
quality of professional services being offered, it allows the members to provide services
without charging any fee to the welfare or spiritual institutions established with an
objective of providing social services. Further, the council of The Institute of Chartered
Accountants of Nepal has waved (vide decision dated 2067/6/17) requirement of
minimum fee for the religious organizations which have annual transactions of less than
2 lacs.

YSZ P.T.O.
(25)

Accordingly in the given case, if the transactions of the institution does not exceed 2
lacs, it will not be termed as guilty of professional misconduct as the service was
provided to a religious institution established for purely promoting social values.
However, if the transactions of the institution exceeds 2 lacs, the auditor shall be guilty
of violating code of conduct.

6. W
rite short notes on the following (Any Two): (25=10)
a) Inherent Limitations of an Audit
b) Audit trail in a computerized accounting environment
c) Management representation
d) Control Risk

Answer No. 6
a) Inherent Limitations of an Audit
An auditor cannot obtain absolute assurance that material misstatements in the financial
statements will be detected. Owing to the inherent limitations of an audit, there is an
unavoidable risk that some material misstatements of the financial statements will not
be detected, even though the audit is properly planned and performed in accordance
with NSAs. An audit does not guarantee all material misstatements will be detected
because of such factors as the use of judgment, the use of testing, the inherent
limitations of internal control and the fact that much of the evidence available to the
auditor is persuasive rather than conclusive in nature. For these reasons, the auditor is
able to obtain only reasonable assurance that material misstatements in the financial
statements will be detected.

b) Audit Trail in a Computerized Accounting Environment:


An audit trail refers to a situation where it is possible to relate „one-to-one‟ basis, the
original input along with the final output. The work of an auditor would be hardly
affected if “Audit Trail” is maintained i.e. if it were still possible to relate, on a „one-to-
one‟ basis, the original input with the final output. It refers to a simplified
representation of the documentation in a manually created audit trail. The particular
credit notes may be located by the auditor at any time he may wish to examine them,
even months after the balance sheet date. He also has the means, should he so wish, of
directly verifying the accuracy of the totals and sub-totals that feature in the control
listing, by reference to individual credit notes. He can, of course, check all detailed
calculations, casts and postings in the accounting records, at any time. In first and early
second-generation computer systems, such a complete and trail was generally available,
no doubt, to management‟s own healthy skepticism of what the new machine could be
relied upon to achieve – an attitude obviously shared by the auditor. In such a system (i)
the output itself is as complete and as detailed as in any manual system. (ii) The trail,
from beginning to end, is complete, so that all documents may be identified by located
for purposes of vouching, totaling and cross-referencing.

Any form of audit checking is possible, including depth testing in either direction. In
case audit trail is missing, the auditor employs Computer Assisted Techniques
(CAATs) to ensure the validity of accounting data.

c) Management representation

It is a representation made by management to the auditor during the course of an audit,


either unsolicited or in response to specific enquiries. It acknowledges its responsibility
for the implementation and operation of accounting and internal control system that are
YSZ P.T.O.
(26)

designed to prevent and detect fraud and error. Nepal Standard on Auditing – 580,
Management Representation states that the auditor should obtain appropriate
representation from management evidencing that the management acknowledges its
responsibility for the fair presentation of the financial statements in accordance with the
relevant financial reporting framework and has approved the financial statements. The
auditor can obtain evidence of management‟s acknowledgment of such responsibility and
approval from relevant minutes of meetings of the board of directors or similar body or by
obtaining a written representation from management or a signed copy of the financial
statements. The possibility of misunderstandings between the auditor and management is
reduced when oral representations are confirmed by management in writing. However,
representations by management cannot be a substitute for other audit evidence that the
auditor could reasonably expect to be available. If the auditor is unable to obtain sufficient
appropriate audit evidence regarding a matter which has, or may have, a material effect on
the financial statements and such evidence is expected to be available; this will constitute a
limitation in the scope of the audit, even if a representation from management has been
received on the matter.

d) Control Risk
Audit risk is the risk that an auditor may give an inappropriate opinion on financial
information that is materially misstated. Audit risk is composed of three components
viz. inherent risk, control risk and detection risk. Control risk is the risk that
misstatement that could occur in an account balance or class of transactions and that
could be material, individually or when aggregated with misstatements in other
balances or classes, will not be prevented or detected on a timely basis by the system of
internal control. There will always be some control risk because of the intrinsic
limitations of any system of internal control.
For assessing control risk, the auditor should consider the adequacy of control design,
as well as test adherence to control procedures. In the absence of such assessment, the
auditor should assume that control risk is high. The auditor ordinarily assesses control
risk at a high level for some or all assertions where:
- The entity`s policies and procedures relating to an assertion are not effective or
- Evaluating the effectiveness of the entity`s policies and procedures would be
inefficient
The auditor may make a preliminary assessment of control risk at less than a high level
only when the auditor:
- Is able to identify policies and procedures of the accounting and internal control
systems relevant to specific assertions which are likely to prevent or detect material
misstatements in the financial statements ; and

- Plans to perform tests of control or support the assessment.


It may be noted that nature, timing and extent of substantive audit procedures to be
performed would depend upon the auditor`s assessment of the inter-relationship
between inherent risk, control risk and detection risk.

7. D
istinguish between the following (Any Two): (25=10)
a) Audit Planning and Audit Program
b) Management Audit and Operational Audit
c) Compliance Procedures and Substantive Procedure
d) Tagging and Tracing

YSZ P.T.O.
(27)

Answer No. 7
a) Audit Planning and Audit Program

As per NSA-300 on Audit Planning, the auditor should plan the audit work so that the audit
will be performed in an effective manner. The "Audit Planning" means developing a general
strategy and a detailed approach for the expected nature, timing and extent of the audit. The
auditor plans to perform the audit in an efficient and timely manner. Audit planning is one of
the basic principles of auditing. Plan should be based on knowledge of the clients business.
Audit planning is a continuous process throughout the audit engagement and covers
developing an overall plan for the expected scope and conduct of the audit and developing
an audit program showing the nature, timing and extent of audit procedures. Adequate
planning of the audit work helps to ensure that appropriate attention is devoted to important
areas of the audit, those potential problems are identified and that the work is completed
expeditiously. Planning also assists in proper assignment of work to assistants and in
coordination of work done by other auditors and experts. The extent of planning will vary
according to the size of the entity, the complexity of the audit and the auditor's experience with
the entity and knowledge of the business. Matters to be considered by the auditor in developing
the overall audit plan include knowledge of business, understanding the accounting and internal
control systems, risk and materiality, nature, timing and extent of procedures, coordination,
direction supervision and review process etc.
The same NSA-300 mentions that the auditor should develop and document an audit program
setting out the nature, timing and extent of planned audit procedures required to implement the
overall audit plan. Hence, audit program is nothing but a list of examination and
verification steps to be applied set out in such a way that the inter-relationship of one step
to another in clearly shown and designed, keeping in view the assertions discernible in the
statements of account produced for audit or on the basis of an appraisal of the accounting
records of the client. In other words, an audit program is a detail plan of applying the audit
procedures in the given circumstances with instructions for the appropriate techniques to be
adopted for accomplishing the audit objectives.
The difference between audit plan and audit program is that audit program is a part of audit
plan. While audit plans involves each step of audit from engagement to issuance of final
report audit program deals with only nature, timing and extent of audit procedures.
b) Management Audit and Operational Audit.
Management audit is an audit of the management. The management audit is, therefore,
concerned itself with the whole field of activities of the concern, from top to bottom
starting from the top, because we are primarily concerned with whether the general
management is functioning smoothly and satisfactorily. Management audit is concerned
with appraising management‟s accomplishment of organizational objectives, the
management functions of planning, organizing, directing and controlling, and the adequacy
of management‟s decisions and actions in moving towards its stated objectives.
Management audit is a complex task closely linked with the process of management. It
usually involves the following steps:
a) Identification of the objectives of the organization.
b) The overall objectives are to be split down into detailed targets and plans for
various segments.
c) The organizational structure is to be reviewed to assess whether or not it can
effectively achieve the overall objectives and detailed targets.
d) The performance of each functional area or responsibility center is to be examined
and compared with the targets and objectives.
e) On
the basis of above examination, a realistic course of action may be recommended.

YSZ P.T.O.
(28)

On the other hand, operational audit is an audit for the management. It is undertaken at
the instance of the management for providing it with information and appraisal of
operations and activities. Operational auditing is essentially a review and appraisal of
operations of an organization carried on by a competent independent person. Hence,
operational audit refers to a systematic independent appraisal activity within an
organization for a review of the entire departmental operations as a service to
management. Operations audit is a technique for regularly and systematically,
appraising unit or function effectiveness against corporate and industry standards by
utilizing personnel who are not specialists in the area of study with the objective of
assuring a given management that its aims are being carried out in identifying
conditions capable of being improved.
Management audit deals with various aspects of the management process whereas
operational audit is confined to various activities and operations in the functional areas.
Management audit attempts to evaluate the performance of various management
process and functions.

c) Compliance procedures and substantive procedure


Compliance procedures are those tests designed to obtain reasonable assurance that
those internal controls on which audit reliance is to be placed are in effect. In obtaining
audit evidence from compliance procedures, the auditor is concerned with assertions that
the control exists, the control is operating effectively and the control has so operated
throughout the period of intended reliance.
Whereas substantive procedures are tests designed to obtain evidences as to the
completeness, accuracy and validity of the data produced by accounting system. They
are of two types: tests of details of transactions and balances and analysis of significant
ratios and trends. As mentioned in NSA-500 Audit Evidence, when obtaining audit
evidence from substantive procedures, the auditor should consider the sufficiency and
appropriateness of audit evidence from such procedures together with any evidence from
tests of control to support financial statement assertions.

d) It is a technique better than Integrated Test Data Facility. It involves tagging the client`s
input data in such a way that relevant information is displayed at key points. It uses the
actual data, and so the question of elimination of special entries test data designed under
Integrated Test Data Facility does not arise. The hard copy, so produced is available
only to the auditor and may describe such inputs as hours worked in a pay period in
excess of 60; or sales orders processed in excess of Rs. 200,000 etc. This enables the
auditor to examine transactions at the intermediate steps in processing.
The advantage of tagging and tracing approach lies in the use of actual data and
elimination of the need for reversing journal entries. The disadvantage is that the
erroneous data will not necessarily be tagged. An effective combination approach may
be to use the ITF approach (Integrated Test Facility) for a few hypothetical transactions
and the tagging and tracing approach to follow line data through a complex system.

YSZ P.T.O.
(29)

CAP-II, Corporate & other Laws, Dec 2011


Suggested Answer
Maximum Marks - 100

Total No. of Questions - 7

Time Allowed - 3 Hours


Marks
All questions are compulsory.

Part : "A"
1.
a) Mrs. Sindhu had applied for the allotment of 1,000 shares in a company. No
allotment of shares was made to her by the company. Later on, without any
further application from Mrs. Sindhu, the company transferred 1,000 partly-
paid shares to her and placed her name in the Register of Members. Mrs.
Sindhu, knowing that her name was placed in the Register of Members, took
no steps to get her name removed from the Register of members. The
company later on made final call. Mrs. Sindhu refuses to pay for this call.
Referring to the provisions of the Companies Act, 1956, examine whether
Mrs. Sindhu‟s refusal to pay for the call is tenable and whether she can
escape herself from the liability as a member of the company. 8
b) Who is an 'Expert'? When is an expert not liable for the mis-statement in the
prospectus of a public company as per the provisions of the Companies Act,
1956? 7
c) B issued a cheque for Rs. 1, 25,000 in favour of S. B had sufficient amount
in his account with the Bank. The cheque was not presented within
reasonable time to the Bank for payment and the Bank in the meantime,
became insolvent.

Decide, under the provisions of the Negotiable Instrument Act, 1881


whether S can recover the money from B. 5
d) The governor of a state is a member of Nava Bharat Fertilizer Ltd. and has
appointed X, a member of the company as his representative for the purpose
of attending meetings of the company. Both the Governor and his
representative are unable to attend the forthcoming general meeting though
they are anxious to do so. Answer following question referring the
Companies Act, 1956.

i) Do you think X can appoint another proxy instead of himself? 3


ii) Would your answer be different in case X was the non member of the
company? 2
Answer No. 1
a) According to Section 164 of the Companies Act, 1956, the register of member is a
prima facie evidence of the truth of its contents. The contents of the register of
members are of decisive importance in determining as to who were the
shareholders of the company at a crucial time. Accordingly, if a person‟s name, to
his knowledge, is there in the register, she/he shall be deemed to be a member. In
the given case Mrs. Sindhu knows that her name is included in the register of
shareholders and stands by and allows her name to remain, she is holding out to the
public that she is shareholder and thereby she will be liable as shareholder.

YSZ P.T.O.
(30)

b) According to Section 59(2) of the Companies Act, 1956, the expression "expert"
includes an engineer, a valuer, an accountant and any other person whose profession
gives authority to a statement made by him in the prospectus.

Restriction as per Section 57:


The report of an expert cannot be included in a prospectus if he is in any way
connected with the formation or promotion or management of the company.

When an expert is not liable [Section 62(3)]: An expert who would be liable by reason
of having given his consent under Section 58 to the issue of the prospectus containing
a statement made by him shall not be so liable if he can prove:
(a) that, having given his consent under section 58 to the issue of the prospectus, he
withdrew it in writing before delivery of a copy of the prospectus for registration;
(b) that, after delivery of a copy of the prospectus for registration and before allotment
thereunder, he, on becoming aware of the untrue statement, withdrew his consent in
writing and gave reasonable public notice of the withdrawal and of the reason
therefore; or
(c) that he was competent to make the statement and that he had reasonable ground to
believe, and did up to the time of the allotment of the shares or debentures, believe,
that the statement was true.

c) The problem as asked in the question is based on Sections 72 and 74 of the Negotiable
Instrument Act, 1881; subject to the Provision of Section 84.
As per Section 72, it is the duty of the holder of a cheque to present the same to the
bank for payment before the relationship between the bank and drawer is spoilt. It is
only if he does so that he can hold the drawer liable in case of non payment and not
otherwise.
As per Section 74, if the cheque is not presented by the holder for payment within
reasonable time and meanwhile the relation between drawer/customer of the bank is
spoilt and if the bank refuses to make payment or is incapable of making such
payment, then the drawer cannot be held liable for such dishonour.
Section 31 prescribes the liability of drawee of cheque. The drawee of a cheque having
sufficient funds of the drawer in his hands properly applicable to the payment of such
cheque must pay the cheque when duly required so to do, and in default of such
payment, must compensate the drawer of any loss or damage caused by such default.
Accordingly Section 84 prescribes the situation when cheque not duly presented and
drawer damaged there by under Section 84(1) where a cheque is not presented for
payment within a reasonable time of its issue, and the drawer or person on whose
account it is drawn had the right, at the time when presentment ought to have been
made, as between himself and the banker, to have the cheque paid and suffers actual
damage through the delay, he is discharged to the extent of such damage, that is to say,
to the extent to which such drawer a person is a creditor of the banker to a large
amount than he would have been if such cheque had been paid.
Hence applying the above provisions, S cannot recover the money from B.

d)
i) Under Section 187A(1) The president of India or the Governor of a State, if he is a
member of a company, may appoint such person as he thinks fit to act as his
representative at any meeting of the company or at any meeting of any class of
members of the company.
(2) A person appointed to act as aforesaid shall for the purposes of this Act be deemed
to be a member of such a company and shall be entitled to exercise the same rights
YSZ P.T.O.
(31)

and powers (including the right to vote by proxy) as the President or as the case
may be, the Governor could exercise as a member of the company.
On the basis of above provision X can shall be entitled to exercise the same rights
and power of governor including the right to vote by proxy. Hence governor is
entitled to appoint proxy; X is also entitled to appoint another proxy instead of
himself. So X can appoint another proxy.
ii) Under section 176 any person whether a member or not may be appointed as
proxy. So the answer would not be different. A non member can also be appointed
as proxy. In this context, even if X were non member of the company, he would be
eligible to be appointed as proxy and for this purpose he shall be deemed to be a
member of such company.
Part: "B"
2.
a) Mr. Sharma was convicted by the Supreme Court of an offense of
corruption and was sent to imprisonment for the period of 5 years in the
year 2050. In the year 2060, he was appointed as a director of Jagadamba
Traders Ltd. Do you think his appointment was valid under Companies Act,
2063? Would your answer be different in case it was the case of Jagadamba
Hydropower Private Limited? 7

b) What is the procedure on deciding a resolution at a general meeting and


mention the matters requiring Special Resolution at the general meeting of
a company as per the Companies Act, 2063. 8
Answer No. 2
a) Under Section 89(1) (d) of the Companies Act, 2063: Person convicted of an offense
of corruption or of an offense involving moral turpitude shall not be eligible to be
appointed to the office of director.
Provided, that in the case of a private company, a period of three years has not lapsed
from the date of such sentence.
Since Mr. Sharma was appointed in the post of director of public limited company. He
was disqualified to be appointed in the post. He has disqualified to be appointed in the
post of director of public company forever once he was convicted of an offense of
corruption. The completion of five years imprisonment does not matter. If he were
appointed in the private limited company, it would have been valid because in case of
private limited company the limitation even the case is an offense of corruption shall
be extended only upto three years from the date of offense. On the basis of above
observation, Mr. Sharma could not be appointed in the post of director of Jadamba
Traders Ltd. However, he could be appointed in the post of director of Jagadamba
Hydropower Pvt. Ltd. as the period of three years has been completed.

b)
Procedure for taking decision at a general meeting:

Section 74(3) of the Companies Act, 2063: The opinion of majority of the shareholders
present in the meeting shall be deemed to be the decision of that meeting on every
matter put to vote. Such voting may be taken in such manner including a show of
hands, voice voting, division of shareholders in groups or poll as well as other
appropriate method as prescribed by the chairman.

Provided, however, that in the case of a special resolution, the resolution shall be
deemed to have been adopted by the meeting only if the shareholders representing
seventy five per cent shares out of the shareholders present in the meeting vote in favor
of the resolution.
YSZ P.T.O.
(32)

The matters requiring Special Resolution at the general meeting of a company is


specified in Section 83 of the Companies Act 2063 as follows:

Section 83: Special resolutions to be presented:


Special resolutions shall be presented in the general meeting of a company for decision
on the following matters:
(i) Increasing the authorized capital of the company,
(ii) Decreasing or altering the share capital of the company,
(iii) Altering the name or main objects of the company,
(iv) Amalgamating one company into another company,
(v) Issuing bonus share,
(vi) Buying back of own shares by the company,
(vii) Selling shares at a discount,
(viii) Converting a private company into a public company or vice versa,
(ix) Such other matter in respect of which the company is required by this Act or
the articles of association to adopt a special resolution.

3.
a) A Public Company proposes to purchase its own shares. State the sources
of funds that can be utilized by the Company for purchasing its own shares
and the requirements to be complied with by the Company under
Companies Act, 2063. 5

b) Enumerate the provisions regarding appointment of Company Secretary as


provided in the Companies Act, 2063. 5

Answer No. 3
a) Section 61(2) of the Companies Act, 2063 allows a company to buy its own shares
out of its free reserves available for being distributed as dividends. The conditions
for buy back under this section are as follows:
(i) Where the shares issued by the company are fully paid up;
(ii) Where the shares issued by a public company have been listed in the Securities
Board;
(iii) Where the buy-back of shares is authorized by the articles of association of the
concerned company;
(iv) Where a special resolution has been adopted at the general meeting of the
concerned company authorizing the buy-back;
(v) Where the ratio of the debt owed by the company is not more than twice the
capital and general reserve fund after buy-back of shares;
Explanation: For the purpose of this Clause, "debt" means all amounts of secured
or unsecured debts borrowed by a company.
(vi) Where the value of shares to be bought back by a company is not more than
twenty percent of the total paid up capital and general reserve fund of that
company;
(vii) Where the buy-back of shares is not in contravention of the directives issued
by the Office in this respect.
A resolution to be presented at the general meeting shall state the
following matters;
(a) The reason and necessity for the buy-back of shares.

YSZ P.T.O.
(33)

(b) A statement of the evaluation of possible impacts of the financial situation of


the concerned as a result of the buy-back of shares,
(c) The class and number of shares intended to be bought back;
(d) The maximum or minimum amount required to buy-back shares as referred to
in Clause (c) and financial sources of such amount;
(e) The time limit for the buy-back of shares;
(f) The mode of the buy-back shares;
(g) Such other necessary matters as specified by the office and as required to be
disclosed under the prevailing law, in respect of the buy-back of shares;

b) The provisions of Companies Act, 2063 regarding the appointment of company


secretary are as follows (Section 185 )
(1) A company with the paid-up capital of ten million rupees or more shall appoint to
the post of company secretary a Nepalese citizen who has the qualification
mentioned in sub-section (2).
(2) A Nepalese citizen who has worked in the related field for at least two years after
obtaining the professional certificate of company secretary issued by a native or
foreign body authorized to issue the professional certificate of company secretary
pursuant to the prevailing law or who has worked in the related field or in the field
of company management for at least three years after doing at least bachelor
degree in law, management, commerce or economics may be appointed to the post
of company secretary.
Provided, however, that this provision shall not apply to the company secretary
who is incumbent at the time of commencement of this Act for three years after the
date of commencement of this Act.
(3) No director of the concerned company shall be eligible to be appointed as the
company secretary of such company.
(4) A person shall not be appointed to the post of company secretary of more than one
company at the same time.
Provided, however, that this provision shall not bar the appointing of the company
secretary of any principal company to the post of company secretary of the
subsidiary company of such company.
(5) Where it is provided by this Act, the prevailing law or articles of association that
any act has to be done by or through the company secretary, and the post of
company secretary remains vacant in the company for any reason or the incumbent
company secretary fails to do such act or shows his inability to do such act, then
any such employee of the company, who has the qualification referred to in this
Act, as designated by the board of directors to do such act may perform such act in
the capacity of company secretary.

4.
a) What acts are deemed as offences under Banks & Financial Institutions
Act, 2063? 5

b) Enumerate the circumstances under which the Nepal Securities Board may
revoke a license issued under the Securities Act, 2063. 5

Answer No. 4
YSZ P.T.O.
(34)

a) Commission of any of the following acts shall be deemed to be a commission of an


offense under Banks and Financial Institutions Act, 2063( Section 70)

(i) Carrying on the financial transactions without obtaining a license under this Act
or in contravention of the conditions of the license or accepting deposits or
supplying credit or issuing debentures or other financial instruments in
violation of this Act or the orders or
directives issued under this Act or paying or obtaining interest in contravention
of the term prescribed by the Rastra Bank or dealing in foreign exchange in
contravention of this Act or the laws in force or the orders or directives issued
under this Act;
(ii) Failure of any officer who is responsible for maintaining the secrecy of the
financial transactions, accounts, records, ledgers, books and other accounts
related documents of the licensed institution pursuant to this Act or the orders
or directives issued under this Act to maintain such secrecy;
(iii) Violation by any one of this Act or any order or directive issued under this
Act, other than that mentioned in Clause (i) or (ii).

Above acts have been deemed to be a commission of an offense under the Act.
In nutshell, any acts in contravention of BAFIA, rules framed there under or
directives shall be deemed an offense under the Act.

b) Section 89. Revocation of License by the Nepal Securities Board:


(1) The Board may, on any of the following circumstances, revoke a license obtained
by any stock exchange or securities business (the licensee) to operate the stock
exchange or securities business under this Act:-
(a) If the licensee stops operating the stock exchange or securities business,
(b) If the licensee operates the stock exchange or securities business in contrary to
the interest of investors,
(c) If the licensee violates the terms set forth in the license,
(d) If the licensee violates any provision of this Act or the rules and bye-laws
framed under this Act,
(e) If the licensee fails to comply with any order or direction issued by the Board,
(f) If the licensee becomes insolvent being unable to repay its dues to its creditors,
(g) If the company or body having obtained a license to operate the stock exchange
or securities business is wound up,
(h) If the licensee having obtained a license to operate the stock exchange or
securities business makes an application for the revocation of the license,
(i) If the securities business having removed the agent appointed by it does not
appoint another agent in lieu of such an agent,
(j) If the licensee fails to submit such financial and transaction related statements
as required to be submitted to the Board,
(k) If the licensee fails to pay such fees as required to be paid pursuant to Section
50 to the Board within the specified time limit.
(2) No revocation of a license made pursuant to subsection (1) shall have any
effect on any liabilities arising from any activities done by such a stock
exchange or securities business person prior to the cancellation of license.

YSZ P.T.O.
(35)

5.
a) AED Company Ltd. could not avail skilled manpower for its specialized
operation in Nepal and hence intends to hire manpower from the
international market. What are the provisions in this regard as provided by
the Labor Act, 2048? 5

b) What are the disqualifications for becoming an Insurance Agent, Surveyor


and Broker as per the Insurance Act, 2049? 5

c) Explain the provisions of Nepalese Negotiable Instruments Act, 2034 with


regard to notice of dishonor and mode in which notice may be given. 5
Answer No. 5
a) The employer is prohibited from employing foreigners in Nepalese organizations
except satisfying the grounds of employment. Section 4A provides of the Labor Act,
2048 provides for employment of foreigners in certain cases as follows.

Section 4A: Prohibition on Engaging Non-Nepalese Citizens at Work:


(1) Non-Nepalese citizens shall not be permitted to be engaged at work in any of the
posts classified pursuant to Section 3.
(2) Notwithstanding anything contained in Sub-section (1), if a Nepalese citizen
could not be available for any skilled technical post even after publishing an
advertisement in national level public newspapers and journals, the Manager may
submit an application to the Department of Labour along with the evidence of
such fact for the approval to appoint a non-Nepalese citizen.
(3) If it is found, in conduction of an inquiry upon the submission of any application
pursuant to Sub-section (2), that a Nepalese citizen would not be available for the
skilled technical post mentioned in the application, the Department of Labour
may, on the recommendation of the Labour Office, grant approval to engage a
non-Nepalese citizen at work years for a maximum period of upto five years not
exceeding two years at a time and, in the specialized kind of skilled technical
post, for a period upto seven years.
(4) The Manager, who engages non-Nepalese citizens at work pursuant to Sub-
section (3), shall have to make arrangements for making the Nepalese citizens
skilled and for replacing the non-Nepalese citizens gradually by them. "

AED Company Ltd can hire foreign manpower seeking approval of the
Department of Labor as mentioned above.

b) Disqualifications of Insurance Agents, Surveyors and Brokers (Section 32 of the


Insurance Act , 2049):
No person can become an Insurance Agent, Surveyor or Broker in the following
circumstances:
i. In case he/she has not attained the age of 16 years,
ii. In case he/she is of unsound mind,
iii. In case he/she has been declared insolvent and bankrupt,
iv. In case he she has been convicted and sentenced to punishment by a court
in the offence involving any type of theft , fraud and misappropriation or
embezzlement of the property entrusted to him, or

YSZ P.T.O.
(36)

v. In case he/she has done anything in the course of work regarding to


Insurance Business causing loss or damage to the insurer or to any
Insurance Policy Holder.

c) Notice of Dishonor
When a Bill of Exchange is dishonored by non acceptance or non- payment ,the
holder thereof , or some party thereto who remains liable thereon , must give notice to
all other parties related to such instrument , and if any party among them has not been
noticed , such party shall not be liable to that instrument .
Provided that, nothing in this section renders it necessary to give notice of
dishonor to the maker of the Promissory Note, acceptor of the Bill of Exchange or the
Drawee.
Mode in which notice may be given:
Notice of dishonor may be given to a person to whom it is required to be given or to
his /her duly authorized agent or, where he /she has died ,to his/ her heir , may be oral
or written . While giving written notice, it can be sent by post and may be in any form.
In such notice it must be informed to the party to whom it is given, either in express
terms or reasonable intendment that the Negotiable Instrument has been dishonored
and he / she will be held liable thereon and it must be given within reasonable time, at
his place of business or residence. If the notice is duly directed and sent by post and
miscarries, such miscarries does not tender the notice invalid.

6.
a) What are the provisions of Nepal Chartered Accountants Act, 2053 with
regard to removal and re-instatement of the name of any member? 5
b) The General Manager of PQR Industry, a corporate body wholly owned by
Government of Nepal, has appointed Mr. Aryal, a Chartered Accountant, to
audit the PQR Industry. Is the appointment legally valid? Answer referring
the provisions of Audit Act, 2048. 5

Answer No. 6
a) Provision of Removal of Names and Re-instatement (Section 22 of Nepal Chartered
Accountants Act, 2053)
(1) The council may issue an order to remove the name of any member from the
Membership Register on any of the following circumstances:-
(a) If he is convicted by an court in a criminal offense involving moral turpitude
and punished there for,
(b)If he fails to pay any fees required to be paid to the institute,
(c) If he fails to abide by the professional conduct referred to in this Act and the
Rules framed under this Act,
(d)If he becomes unsound-minded, or
(e)If he is dead.
(2) In case, a member whose name has been removed from the Membership
Register pursuant to sub-section (1) makes an application, along with the fees as

YSZ P.T.O.
(37)

prescribed, stating reasonable ground to restore his membership, the Council


may consider restoring his membership by reinstating his name in the
Membership Register

b) As per section 6 of Audit Act, 2048, the audit of the corporate bodies wholly owned by
Government of Nepal shall be audited by the Auditor General of Nepal. If the Auditor
General is constrained by time and resources to audit such corporate bodies, he may
appoint professional auditors according to the existing laws as his assistants. In the
given situation, auditor was appointed by the General Manager. As per Audit Act,
audit of such corporate bodies wholly owned by the Government of Nepal shall be
done by Auditor General and in certain condition Auditor General may appoint other
professional auditors to audit such corporate bodies.
Viewing the above provisions, the appointment of Mr. Aryal as an auditor by the
General Manager is illegal. General Manager has acted beyond his legal authority.

7. Write short notes on the following: (3×5=15)

a) Sale through Samples

b) Constitution of Industrial Promotion Board

c) Welfare Fund under Bonus Act, 2030

Answer No. 7
a) Sale through Samples
In case provisions have been made in a contract to sell goods after inspecting their
samples directly or otherwise, it shall be deemed to have been signed to sell goods
after inspecting their samples. In case a contract has been signed to sell goods after
inspecting their samples, it shall be deemed to include the following conditions, except
when otherwise provided for in the contract:
(i) The bulk of goods shall correspond to the samples in quality.
(ii) They buyer shall have, a reasonable opportunity to compare the quality of the
bulk of the goods with the sample.
(iii) The goods sold or to be sold shall be free from any defect, and that their
merchantable quality shall be apparent while inspecting them at the time of
comparing them with the sample.
b) Constitution of Industrial Promotion Board:

Government of Nepal shall constitute an Industrial Promotion Board consisting of the


following members:
(a) The Minister or State Minister for Industries -Chairman
(b) The Assistant Minister for Industries -Member
(c) Member (looking after industries), National Planning commission -Member
(d) The Governor, Nepal Rastra Bank -Member
(e) The Secretary, Ministry of Industry -Member
(f) The Secretary, Ministry of Finance -Member
(g) The Secretary, Ministry of commerce -Member
(h) The Secretary, Ministry of Tourism -Member
YSZ P.T.O.
(38)

(i) The Director General, Department of Cottage and Small Industries - Member
(j) Representative, Federation of Nepal Chamber of Commerce and Industry -
Member
(k) Two persons nominated by Government of Nepal, either from among the
industry, commerce and tourism sector organizations or from among the persons
of high distinction in the same field -Member
(l) The Director General, Department of Industries -Member Secretary

c) Welfare Fund under Bonus Act


Seventy percent of the residuary amount after distribution of bonus from the allocated
amount for bonus pursuant to section 5 of Bonus Act, 2030 shall be deposited with the
welfare Fund established in accordance with section 37 of the Labour Act 2048 and
remaining thirty percent shall be deposited with the National Level Welfare Fund,
established by Government of Nepal for the interest of the employees of the
enterprises. The operation of the welfare fund mentioned above shall be in
participation of the employees as prescribed.

YSZ P.T.O.
(39)

CAP-II, Cost & Management Accounting, Dec 2011


Suggested Answer
Maximum Marks - 100
Total No. of Questions: 6
Time Allowed - 3 Hours

Marks
All questions are compulsory. Working notes should form part of the
answer.
Make assumptions wherever necessary.
1. Z Ltd. makes a range of five products to which the following standard
apply:
Per unit
Particulars A B C D E
Rs. Rs. Rs. Rs. Rs.
Selling Price 500 600 700 800 900
Direct Materials 90 100 170 120 210
Direct Wages 160 200 240 280 320
Variable Production Overheads 80 100 120 140 160
Variable Selling & Distribution Overheads 50 60 70 80 90
Fixed Overheads 40 50 60 70 80
Total Cost 420 510 660 690 860
The direct labour wage rate is Rs. 40 per hour. Fixed overheads have
been allocated on the basis of direct labour hours. The company has
commitments to produce a minimum of 400 units of each product
per month. Direct labour hours cannot exceed 13,000 hours per
month due to restriction of space.
The Board is now considering an offer of a new three year contract
to produce an additional 400 units of product B per month at a
selling price of Rs. 580 per unit. The contract would involve an
outlay of Rs. 1,000,000 on the lease of additional factory premises
and purchase of new plant and equipment. There would be no
residual value at the end of the contract. Variable production costs
would be in accordance with existing standards and variable selling
& distribution costs would be one-half of the existing rate and cash
outflows on fixed costs would be Rs. 200,000 per annum. There
would be no changes to existing production arrangements.
An outside supplier has offered to supply 400 units of product B per
month at a price of Rs. 480 per unit. If purchased externally cash
outflows on additional fixed costs will be Rs. 250,000 per annum.
Required: (7+5+8=20)
a) Give recommendations, supported by calculations, to show
how direct labour hours in the existing factory should be
utilised in order to maximise profits.
b) Calculate budgeted trading results on the basis of your
recommendation in (a).

YSZ P.T.O.
(40)

c) Determine whether or not the proposed contract for product B


should be accepted and, if so, whether it should be purchased
externally or manufactured in the new premises. The
company's cost of capital is 10%. You may take the present
value of an annuity of Rs. 1 for three years at 10% as Rs. 2.49.
Ignore taxation and inflation.
Answer
a) Since availability of direct labour hours is a constraint, 13,000
direct labour hours per month should be utilised as shown by the
following statement to maximise contribution per labour hour:

Statement showing contribution per labour hour


Rs. per unit
Particulars
A B C D E
Selling Price 500 600 700 800 900
Variable Costs:
Direct Materials 90 100 170 120 210
Direct Wages 160 200 240 280 320
Production
80 100 120 140 160
Overheads
Selling &
50 60 70 80 90
Distribution OHs.
Total Variable Costs 380 460 600 620 780
Contribution per unit 120 140 100 180 120
Labour hours per unit (160/40)=4 (200/40)=5 (240/40)=6 (280/40)=7 (320/40)=8
Contribution per labour
30.00 28.00 16.67 25. 71 15.00
hour
Ranking I II IV III V

Since the company has commitments to produce minimum of 400 units of each product,
production should be planned in such a way that product B, C, D & E each are produced to
the extent of 400 units only and balance labour hours should be utilised to produce product
A. Therefore, production should be in the following pattern:
Labour hour Total labour Balance labour
Product Units produced
per unit hours used hours
B 400 5 2,000 11,000
C 400 6 2,400 8,600
D 400 7 2,800 5,800
E 400 8 3,200 2,600
A 650* 4 2,600 -
(b) *2600/4=650
Budgeted trading result as per recommendation in (a) above
Products
Particulars
A B C D E Total

YSZ P.T.O.
(41)

Units 650 400 400 400 400 2,250


Rs. Rs. Rs. Rs. Rs. Rs.
Selling Price/unit 500 600 700 800 900 -
Sales 325,000 240,000 280,000 320,000 360,000 1,525,000
Direct Materials 58,500 40,000 68,000 48,000 84,000 298,500
Direct Wages 104,000 80,000 96,000 112,000 128,000 520,000
Variable Production Ohs. 52,000 40,000 48,000 56,000 64,000 260,000
Variable Selling Ohs. 32,500 24,000 28,000 32,000 36,000 152,500
Variable Costs 247,000 184,000 240,000 248,000 312,000 1,231,000
Contribution 78,000 56,000 40,000 72,000 48,000 294,000
Fixed Overheads* 26,000 20,000 24,000 28,000 32,000 130,000
Net Profit 52,000 36,000 16,000 44,000 16,000 164,000
* Allocated on the basis of direct labour hour – given in the question.

[c] (i)
Statement showing additional contribution if contract is accepted and product B is
manufactured in new premises
Per unit 4,800 units**
Particulars
(Rs.) (Rs.)
Selling Price 580 2,784,000
Direct Materials 100 480,000
Direct Wages 200 960,000
Variable Production Overheads 100 480,000
Variable Selling Overheads – 50% of Rs. 60 30 144,000
Variable Cost 430 2,064,000
Contribution 150 720,000
Additional Fixed Overheads 200,000
Increase in Annual Cash Flow 520,000
** Unit for one year.
The present value of cash inflows for 3 years at 10% = Rs. 520,000 × 2.49 = Rs. 1,294,800
Initial capital outlay being Rs. 1,000,000, NPV of the proposal = Rs. 294,800. Hence
the proposal should be accepted.
[c] (ii)
Statement showing additional contribution if contract is accepted and product
B is purchased externally
Per unit 4,800 units**
Particulars
(Rs.) (Rs.)
Selling Price 580 2,784,000
Purchase Price 480 2,304,000
Contribution 100 480,000
Additional Fixed Overheads 250,000
Increase in Annual Cash Flow 230,000
YSZ P.T.O.
(42)

** Units for one year.


The present value of cash inflows for 3 years at 10% = Rs. 230,000 × 2.49 = Rs. 572,700
Initial capital outlay being zero, NPV of the proposal = Rs. 572,700
Conclusion / Recommendation:
Both 'Make Internally' and 'Purchase Externally' options give a positive
NPV. Hence, the contract should be accepted. However, the 'Purchase'
option gives a greater NPV hence product B should therefore be
purchased externally. This option has added advantages of minimising
risk, since in the circumstances, it appears unnecessary to make any
capital investment to cope with the contract.
2.
a) The direct labor hour requirements of three of the product
manufactured in a factory, each involving more than one labor
operation is estimated as follows:

Direct labor hours per unit (in minutes)


Operation Product
1 2 3
A 18 42 30
B - 12 24
C 9 6 -

The factory works 8 hours per day, 6 days in a week. The budget
quarter is taken as 13 weeks and during a quarter, lost hours due to
leave and holidays and other causes is estimated to be 124 hours.
The hourly budgeted rates for the workers manning the operation A,
B and C are Rs. 2.00; Rs. 2.50 and Rs. 3.00 respectively.
The sales of the products during the quarter are:

Product 1 9,000 units


2 15,000 units
3 12,000 units
There is a carryover of 5,000 units of product 2 and 4,000 units of
product 3 and it is proposed to build up a stock at the end of the
budget quarter as follows:
Product 1 1,000 units
3 2,000 units
Prepare a man-power budget for the quarter showing for each
operation (i) direct labor hours (ii) direct labor cost, and (iii) the
number of workers. 10

b) In a chemical manufacturing company, three products A, B and


C emerges at a single split off stage in department P. Product A
is further processed in department Q, Product B in department R
and Product C in department S. There is no loss in further
processing of any of the three products. The cost data for a
particular month are as under:
Rs.

YSZ P.T.O.
(43)

Cost of raw materials introduced in department P 1,268,800


Direct Wages: Department P 384,000
Department Q 96,000
Department R 64,000
Department S 36,000
Factory overheads of Rs. 464,000 are to be apportioned to the
departments on direct wages basis. During the month under
reference, the company sold all three products after processing
them further as under:
A B C
Output sold in kg. 44,000 40,000 20,000
Selling Price per kg. (Rs.) 32 24 16
There is no Opening or Closing Stocks. If these products were
sold at the split off stage i.e. without further processing, the
selling prices would have been Rs. 20, Rs. 22 and Rs. 10 per kg.
for A, B and C respectively.
Required: (3+3+2+2=10)
i) Prepare a statement showing the apportionment of joint costs to
three products.
ii) Prepare a statement showing product-wise and total profit
for the month under reference as per the company's current
processing policy.
iii) What processing decision should have been taken to
improve the profitability?
iv) Calculate the product-wise and total profit arising from your
recommendation in (iii).

Answer (a)
Quarterly man-power
Operation Hourly Product 1 Product 2 Product 3 Total No. of
rate workers
Rs. DL Cost DL Cost DL Cost DL Cost
Hrs Rs. Hrs Rs. Hrs Rs. Hrs Rs.
A 2.00 3,000 6,000 7,000 14,000 5,000 10,000 15,000 30,000 30
B 2.50 - - 2,000 5,000 4,000 10,000 6,000 15,000 12
C 3.00 1,500 4,500 1,000 3,000 - - 2,500 7,500 5
Total 4,500 10,500 10,000 22,000 9,000 20,000 23,500 52,500 47

Working Notes
1. Production budget
Product 1 2 3
Units Units Units
Sales 9,000 15,000 12,000
Add: Closing Stock 1,000 - 2,000
Less: Opening - 5,000 4,000
Stock
Production Budget 10,000 10,000 10,000

YSZ P.T.O.
(44)

2. Total available hours in a quarter per worker


Total hours = 8 * 6 * 13 = 624
Less: hours lost due to leave etc. = 124
Total available hours per man = 500

3. The calculation of direct labor hours, direct labor cost and number of men has been
made as follows (illustrated for product 1):

Direct labor hours = (18*10,000)/60 = 3,000 Hours


Direct labor cost = 3,000 hours * Rs. 2 = Rs. 6,000
Number of men required = (Direct labor hours required/Total available hours per
man)
= 15,000/500
= 30 men.
Similarly, calculations have been made for the other products.

Answer (b)
i) Statement showing the apportionment of joint costs to joint products:
Particulars A B C Total
Output sold in kg. 44,000 40,000 20,000
Selling price per kg. at split off (Rs.) 20 22 10
Sales value at split off (Rs.) 880,000 880,000 200,000 1,960,000
Ratio 22 22 5
Joint costs apportioned(W.N. a) 880,000 880,000 200,000 1,960,000

Working Note (a)

Computation of joint cost:


Cost of raw materials 1,268,800
Direct wages 384,000
Factory overhead (464000/580000×384000) 307,200
(Apportioned on direct wages basis.) 1,960,000

(ii) Statement showing product-wise and total profit for the month under reference as
per company's current processing policy:
Particulars A B C Total
Output sold in kg. 44,000 40,000 20,000
Selling price per kg. (Rs.) 32 24 16
Sales value (Rs.) 1,408,000 960,000 320,000 2,688,000
Joint costs (Rs.) 880,000 880,000 200,000 1,960,000
Further processing costs (Rs.)
172,800 115,200 64,800 352,800
W.N. (b)
Total costs (Rs.) 1,052,800 995,200 264,800 2,312,800
Profit / (Loss) (Rs.) 355,200 (35,200) 55,200 375,200

Working Note (b) Q R


S

YSZ P.T.O.
(45)

Direct wages 96,000 64,000


36,000
Factory Overhead (464,000/580,000×96,000) (464,000/580,000×64,000)
(464,000/580,000×36,000)
(Apportioned on direct wages basis) =76,800 =51,200
=28,800
172,800 115,200
64,800

(iii) Statement of incremental profit on further processing:


Particulars A B C
Output sold in kg. 44,000 40,000 20,000
Incremental selling price per kg (Rs.) 12 2 6
Incremental sales value (Rs.) 528,000 80,000 120,000
Further processing costs (Rs.) 172,800 115,200 64,800
Profit / (Loss) (Rs.) 355,200 (35,200) 55,200

Processing decision:
44,000 kg of product A and 20,000 kg of product C should be further processed
because the incremental sales revenue generated after further processing is more than
the further processing costs incurred. 40,000 kg of product B should be sold at the
point of split off because the incremental revenue generated after further processing is
less than the further processing costs incurred.

(iv) Product-wise and total profit arising from the recommendation in (iii) is as follows:

Profit for A 355,200


Profit for C 55,200
Profit for B (At split -off)
Sales- (as per i above) 880,000
Less: Joint cost (W.N.a) 880,000
Profit /Loss Nil
Total Profit 410,400
W.N (C): Costs incurred in the department P are joint costs of products A, B and C.
Similarly costs incurred in the departments Q, R and S are further
processing costs of products A, B and C respectively.
3.
a) Siddhartha Transport Pvt. Ltd. has been given a route of 300
km. long to run bus. The bus costs Rs. 4,000,000. It has been
insured at 3% p.a. and the annual tax will amount to Rs. 2,000.
Garage rent is Rs. 800 per month. Annual repairs will be Rs.
30,000 and the bus is likely to last for 8 years. The driver and
the conductor‟s salary will be Rs. 15,000 and Rs. 10,000 per
month respectively in addition to 10% of takings as
commission, to be shared equally. Office expenses including
stationery will be Rs. 1,600 per month. A manager cum
accountant is employed for a monthly salary for Rs. 20,000. The
bus gives mileage of 3 km. per liter of fuel, which costs Rs. 85
YSZ P.T.O.
(46)

per liter. The bus will make one round trip in 2 days carrying on
an average 40 passengers on each single way trip. The bus will
run on an average 28 days in a month.
Required:
Assuming 20% profit on takings, calculate the bus fare to be
charged from each passenger for a single way trip, and bus fare
per passenger km. 8

b) Rolland Limited operates a group incentive scheme in one of its


department. A minimum hourly rate is guaranteed to each of the
six employees in the group if actual output for the week is less
than the standard output. If actual output is greater than the
standard output, the hourly rate of each employee is increased
by 4% for each additional 600 units of output produced. The
standard output for the group is 12,000 units for a 40 hour week.
During the week ended 31st December, 2010, each employee in
the group worked 40 hours; actual output and minimum hourly
rates were as follows:
Employee Actual output (units) Minimum hourly rate (Rs.)
Lal 2,500 60
Hari 2,700 100
Mohan 2,400 60
Shyam 2,500 80
Hanuman 2,460 60
Krishna 2,440 40
You are required to: (5+2=7)
i) Calculate the earnings of each employee;
ii) Appraise the effectiveness to the company of this group incentive
scheme.

c) During Poush 2067, a company purchased 1,200 kg. of raw


materials. The following particulars relate to the purchase:
i) Lot prices quoted by supplier and accepted by the company for
placing the purchase order:
Lot up to 1000 kg. @ Rs.22/ kg.
Between 1000 – 1500 kg. @ Rs.20/ kg.
Between 1500 – 2000 kg. @ Rs.18/ kg.
ii) Trade discount 20%.
iii) VAT at 13% extra; credit is to be taken.
iv) Freight paid Rs. 240.
v) Insurance paid at 2.5% on Invoice Value.
vi) Stores overhead applied at 5% on total purchase cost of material.
The entire quantity was received and issued to production.
Required:
Prepare a statement showing total cost of material purchased
and unit cost of material issued to production. 5
YSZ P.T.O.
(47)

Answer a)
Statement showing charge per passenger for single way trip
Particulars Annual Amount Monthly Amoun
(Rs.) (Rs.)
Standing charges:
Insurance premium @3% on Rs.4,000,000 120,000.00 10,000.
Tax 2,000.00 166.
Garage rent 800.
Driver's salary 15,000.
Conductor's salary 10,000.
Office expenses 1,600.
Manager cum accountant's salary 20,000.
Total standing charges 57,566.
Running expenses:
Depreciation 500,000.00 41,666.
Repairs 30,000.00 2,500.
Fuel 238,000.
Total running charges excluding commission 282,166.
Total cost without commission 339,733.
Commission (WN1) 48,533.
Total Cost 388,266.
Profit 97066.
Total Takings 485,333.3
Fare per passenger per single way trip (485,333.33/1120) 433.3
Fare per passenger km (485,333.33/336000) 1.4

Working Note:1
Let total taking be x
Commission = 10% of x = x/10
Profit = 20% of x = x/5
Now,
x = 339733.33+x/10+x/5
x-x/10-x/5 = 339733.33
(10x-x-2x)/10 =339733.33
7x = 3397333.30
x = 3397333.30/7
x = 485333.33
Now,
Commission = 10% of 485,333.33 = Rs.48,533.33
Profit = 20% of 48533.33 = Rs.97,066.67
Working Note:2
Average no.of passengers carried during the month (28/2×2×40) 1120
Average passenger km(28/2×2×40×300) 336000
Cost of fuel
Total distance (km) covered 300× 28 = 8,400
Fuel required = 8,400/3 = 2800 Litre.
Cost per liter = Rs. 85
Total cost = Rs. 85 × 2,800 = Rs. 228,000

YSZ P.T.O.
(48)

Answer (b)
ii) Statement showing the earning of each employee in the group during the
week ended 31st December 2010.
Employee Minimum Hourly premium Total rate (Rs.) Total wages for
hourly rate (Rs.) rate (Rs.)(20%of the week 40
minimum Wages Hours (Rs.)
W.N (a)
Lal 60 12 72 2,880
Hari 100 20 120 4,800
Mohan 60 12 72 2,880
Shyam 80 16 96 3,840
Hanuman 60 12 72 2,880
Krishna 40 8 48 1,920
Total 19,200

Working notes:
a) Calculation of the percentage increase in the hourly rate due to higher
efficiency:
(Output for 40 hour week)
Standard 12,000 units
Actual 15000 units
Additional production 3,000 units
Increase in wage rate is 4% for each additional 600 units. Hence total increase;
= (3000/600) * 4% = 20%

ii) Hourly rate for all employee (60+100+60+80+60+40) =Rs.400.


Std production per hour 12000/40×6= 50units.
Hence Std. wages cost of actual production =Rs. 400×50units =Rs. 20000.
Actual wages paid Rs. 19,200.
Therefore, there is a saving of Rs. 800. The gain on account of efficiency of the
worker has thus been shared by the employer and the employee.

Answer (c) Answer:


Statement of total and per unit cost of materials
Particulars Total Amount Per unit Amoun
(Rs.) (Rs.)
Raw material 1200 kg @ Rs.20/ kg 24,000.00 20.
Less: Trade discount @ 20% 4,800.00 4.
19,200.00 16.
Add: VAT @13% 2,496.00 2.
Invoice Value 21,696.00 18.
Freight paid 240.00 0.
Insurance @2.5% on Rs.21,696.00 542.40 0.
22,478.40 18.
Add: Stores overhead @5% 1,123.92 0.
Less: VAT credit 2,496.00 2.
` 21,106.32 17.5

4.
a) In a factory, the expenses of factory are charged on a fixed
percentage basis on wages and office overhead expenses are
calculated on the basis of percentage of works cost.
YSZ P.T.O.
(49)

Following information is supplied to you:


Order I (Rs.) Order II (Rs.)
Materials 12,500 18,000
Wages 10,000 14,000
Selling Price 44,850 61,880
Percentage of profit on cost 15% 12%
Find out percentage for factory overhead and office overhead. Justify
with verification. 7

b) Alcon Construction Company Ltd. commenced its business of


construction on 1st April, 2010. The Trial Balance as on 31st
March, 2011 showed the following balances:
Debit Credit
Particulars
Rs. Rs.
Share capital 1,000,000
Cash received on account of contract - 80% of work certified 1,200,000
Land and Buildings 300,000
Machinery at cost (75% at site) 400,000
Bank balances 40,000
Material at site 400,000
Direct Wages 550,000
Expenses at site 20,000
Lorries and Vehicles 300,000
Furniture 10,000
Office Equipment 100,000
Office Expenses 5,000
Postage and Telegrams 20,000
Rate and Taxes 30,000
Fuel and Power 25,000
2,200,000 2,200,000
The contract price is Rs. 3,000,000 and work certified is Rs.
1,500,000. The work completed since certification is estimated at
Rs. 10,000 (at cost). Machinery costing Rs. 20,000 was returned
to stores at the end of the year. Stock of material at site on 31 st
March, 2011 was of the value of Rs. 50,000. Wages outstanding
were Rs. 2,000. Depreciation on machinery is at 10% per annum.
Prepare Contract Account showing the profit from the contract
and also show how the work-in-progress will appear in the
Balance Sheet as at 31st March, 2011. 9

c) Explain briefly the factors to be considered before establishing


an integrated cost accounting system. 4

Answer (a) Order I


Selling price = Rs. 44,850 including 15% profit on cost.

Cost of production is therefore, 44,850* (100/115) = Rs. 39,000


YSZ P.T.O.
(50)

Order II
Selling price = Rs. 61,880 including 12% profit on cost.

Cost of production is therefore, 61,880 *(100/112) = Rs. 55,250

Cost of production = Direct material (DM) + Direct Wages (DW) + Factory Overhead
(FO) + Office
Overhead (OH)

According to the problem factory expenses is charged at a fixed percentage on


wages while OH is charged at a percentage of Works cost (WC). Both these
figures are unknown. Hence we have to solve it algebraically.

Let FO be x% of wages and OH be y% of works cost.

Then in the Order I the equation will be:


39,000 = 12,500 + 10,000 + (x% * 10,000) + y% (22500 + 100x)
39,000 = 22,500 +100x + 225y + y% (22500 + 100x)
16,500 = 100x + 225y + xy ……………………………………..
……………………………….(i)

And in the Order II the equation will be :


55,250 = 18,000+14,000 +(x% *14,000) + y% (32,000+140x)
Or, 23,250 = 140x + 320y +1.4xy
…………………………………………………………………(ii)

On multiplying equation (i) by 1.4 and equation (ii) by 1.0 we get


23.100 = 140x + 315y +1.4xy
…………………………………………………………………...(iii)
23,250 = 140x +320y + 1.4xy
…………………………………………………………………… (iv)

Bu subtracting (iii) from (iv) we get


150 = 5y or y = 30
On putting the value of y in (i) we get
16,500 = 100x + 6750 + 30x
Or, 9750 = 130x
Or, x = 75
Factory overhead is therefore 75% of wages and office overhead is 30% on works cost.
Verification
Order I (Rs.) Order II (Rs.)
Material 12,500 18,000
Wages 10,000 14,000
Factory Overhead (75% on wages) 7,500 10,500
Works cost 30,000 42,500
Office overhead (30% on works cost) 9,000 12,750
Cost of production 39,000 55,250
Add: Profit 15% and 12% respectively for I & 2 order 5,850 6,630
Selling price 44,850 61,880

b)
Contract Account
YSZ P.T.O.
(51)

For the year ending 31st March, 2011


Particulars Rs. Rs. Particulars Rs. Rs.
By Work-in-
To Materials 400,000
progress:
To Machinery (400,000 × 75%) 300,000 Work Certified 1,500,000
Work
To Wages 550,000 10,000 1,510,000
Uncertified
Add: By Machinery
2,000 552,000 20,000
Outstanding returned
To Expenses at Less:
20,000 2,000 18,000
site Depreciation
To Notional Profit c/d 558,000 By Machinery at site 280,000
Less:
28,000 252,000
Depreciation
By Material at site 50,000
1,830,000 1,830,000
To Profit & Loss By Notional Profit
297,600 558,000
A/c (W.N. i) b/d
To WIP – Reserve (balancing
260,400
figure)
558,000 558,000

Extracts from Balance Sheet as at 31st March, 2011


Liabilities Rs. Rs. Assets Rs. Rs.
Work-in-progress:
Work Certified 1,500,000
Work Uncertified 10,000
1,510,000
Less: Reserve 260,400
1,249,600
Less: Cash received 1,200,000 49,600

Working Notes:
(i) Profit taken to Profit & Loss Account:
(Rs. 558,000 × 2 / 3 × 80 / 100) = Rs. 297,600
(ii)Postage and Telegrams, Office Expenses, Rates and Taxes, Fuel
and Power, Depreciation on Machinery Costing Rs. 100,000 (25%)
will be charged to Profit and Loss Account of the company
c) The following factors should be considered before establishing an integrated cost
accounting system:
i) Degree of integration: The degree of integration should be determined. Some
business firms may integrate up to the stage of prime cost or factory cost. On the
other hand, many undertakings integrate the whole of the records.
ii) Control accounts: In place of classifying expenditure according to its nature,
control accounts may be prepared for each of the elements of cost, such as:

YSZ P.T.O.
(52)

Material Control Account


Direct Labor Control Account
Factory Overhead Control Account
Administrative Overhead Control Account
Selling and Distribution Overhead Control Account
Some of the above control accounts should be separated into fixed and variable
depending on the circumstances.
iii) Cost accumulation purposes: Full details about the cost data are provided in the
cost accounting department so as achieve the following objectives;
a. To provide the necessary costing data
b. To form the basis of journal entries so that the control accounts can
be cleared to suitable revenue accounts resulting into a cost of sales
accounts
iv) Provision for accrued expenses, prepayments and stocks should be dealt with by
transfers to suitable suspense accounts, so that the balance remaining in each
control account represents the charges for the period
5. Distinguish between: (4×2.5=10)
a) Protective purpose and constructive purpose of cost audit
b) Cost allocation and cost apportionment
c) Cost control and cost reduction
d) Methods of costing and techniques of costing

Answer
a) Protective purpose of Cost Audit: Under this, cost audit aims at
examining that there is no undue wastage or losses and the costing
system brings out the correct and realistic cost of production or
processing. The benefit of this protective function is derived by the
organization, its owners and consumers.
Constructive purpose of Cost Audit: Cost audit has a constructive
purpose as well. Cost auditor plays a constructive role by providing
management of the company with information useful in regulating
production; choosing economical methods of operation, reducing
operations costs and re-formulating plans etc. on the basis of his
findings during the course of cost audit

b) Cost allocation is the process of charging the full amount of an


individual item of cost directly to a cost centre for which this item of
cost was incurred. For example, where separate electric meters are
installed in each of the department, the electricity charges on the basis
of electricity bill can be allocated to the respective departments.
Similarly, salary and wages paid to indirect workers of each
department can be allocated to the respective departments.
Cost apportionment on the other hand, is the process of charging the
proportion of common items of cost to two or more cost centres on
some reasonable and equitable basis. For example, where only one
electric meter is installed in a factory, the common electricity
charges should be apportioned to all the departments on the basis of
number of light points or floor area. Similarly, if factory rent is
incurred for the factory as a whole and benefits all the departments in
the factory, it should be apportioned to all the departments on the
basis of floor area occupied by each department.
YSZ P.T.O.
(53)

c)
 Cost control process involves setting standards, ascertaining actual
performance, comparing actual with standards, investigating
variances, and taking corrective actions where as cost reduction is
the final result in the cost control process.
 Cost control aims at achieving standards where as cost reduction
aims at improving the standards. Cost reduction assumes existence
of concealed potential savings in the standards.
 Cost control lacks dynamic approach where as cost reduction is
dynamic and always looks for measures of reducing cost.
 Cost control is preventive. It attempts to optimize costs before they
are incurred. Cost reduction is corrective and attempts to find scope
of reducing incurred costs under controlled conditions.
 Cost control is applicable to items which have standards where as
cost reduction is applicable to every activity of the business.
 Cost control contains the management guidelines where as cost
reduction provides guidelines to management
d)
Method of costing refers to the techniques and processes of determining
costs of a product manufactured or a service rendered. Broadly there are
two methods of costing, i.e. Job costing and Process costing. All other
methods of costing are only variants of the two.
Techniques of costing refer to the manner of ascertaining costs of a
product, job or activity. It indicates what types of costs are being
ascertained such as historical cost, standard cost, full cost, marginal cost
etc. The general techniques of costing are historical costing, standard
costing, full or absorption costing, marginal or variable costing etc
6. Write short notes on: (4×2.5=10)
a) Efficiency audit
b) Reciprocal method of cost allocation
c) Multiple costing
d) Synergetic effect in cost reduction
Answer
a) Efficiency audit is directed towards the measurement of whether corporate
plans have been effectively executed. It is concerned with the utilisation of
the resources in economic and most remunerative manner to achieve the
objective of the concern. It comprises of studying the plans of organisation,
comparing actual performance with plans and investigating the reasons for
variances to take remedial action. For example, the effective utilisation of
capital in an organisation can be gauged by determining the return on
capital employed.
b) The reciprocal method allocates support department costs to operating
departments by fully recognizing the mutual services provided among all
support departments. For example, the Plant Maintenance department
maintains all the computer equipments in Information System department.
Similarly, Information System department provides database support for
Plant Maintenance. The reciprocal method fully incorporates
interdepartmental relationships into the support department cost allocations.
YSZ P.T.O.
(54)

c) Multiple costing involves the application of two or more methods of


costing in respect of same product. This costing method is used in
industries where a number of components are separately produced and then
assembled into a final product. Suppose a firm manufactures bicycles
including its components, the component parts will be costed by the system
of job or batch costing but the cost of assembling the bicycle will be
computed by the single or output costing method. The whole system of
costing is known as multiple or composite costing.
d) Synergy is the interaction of two or more agents or forces so that their
combined effect is greater than the sum of their individual effects. Business
interaction among groups, especially among the acquired subsidiaries or
merged parts of a corporation that creates an enhanced combined effect is
known as synergy effect. When the presence of one unit enhances the
effects of the second it is call synergistic effect. The word synergy means
that the sum of parts is greater than the total value of each part. The result
of two or more people, groups, processes or organizations working together
enhanced the combined effect of individual people, process or organization.
In other words, one and one equals three! The word is used quite often to
mean that combining forces produces a better product.

In the field of costing, synergy is the result of combined process or unit or


organization that result in cost reduction due to minimization of waste,
shortening processes, division of labour and enhancing productivity,
sharing common cost etc. Synergy affects cost control and ultimately
reduction. Synergy is usually accomplished by combining functions (such
as production or customer service) or facilities (such as warehousing)
eliminating some duplication. Synergetic effect normally helps cost
reduction as follows:
 It increases business volume, so the fixed cost per unit gets reduced.
 It eliminates duplication process or function.
 Labour productivity increases and the cost of labour per unit get reduced.
 Minimizes material waste due to process combination.
 Indirect cost e.g. administration, selling and distribution expenses
minimizes due combination of function

YSZ P.T.O.
(55)

CAP-II, Financial Management, Dec 2011


Suggested Answer

Roll No……………. Maximum Marks - 100


Total No. of Questions – 7 Total No. of Printed Pages – 4
Time Allowed – 3 Hours
Marks
Attempt all questions.

Working notes should form part of the answer. Make assumptions wherever
necessary.
1. SC Co. is evaluating the purchase of a new machine to produce product P,
which has a short product life-cycle due to rapidly changing technology. The
machine is expected to cost Rs. 1 million. Production and sales of product P
are forecasted to be as follows:
Year 1 2 3 4
Production and sales (units) 35,000 53,000 75,000 36,000
The selling price of product P (in current price terms) will be Rs. 20 per unit,
while the variable cost of the product (in current price terms) will be Rs. 12
per unit. Selling price inflation is expected to be 4% per year and variable
cost inflation is expected to be 5% per year. No increase in existing fixed
costs is expected since SC Co. has spare capacity in both space and labour
terms.
Producing and selling product P will call for increased investment in working
capital. Analysis of historical levels of working capital within SC Co.
indicates that at the start of each year, investment in working capital for
product P will need to be 7% of sales revenue for that year.
SC Co. pays tax of 30% per year in the year in which the taxable profit
occurs. Liability to tax is reduced by capital allowances on machinery (tax-
allowable depreciation), which SC Co. can claim on a straight-line basis over
the four-year life of the proposed investment. The new machine is expected
to have no scrap value at the end of the four-year period.
SC Co. uses a nominal (money terms) after-tax cost of capital of 12% for
investment appraisal purposes.
Required: (12+3+5=20)
a) Calculate the net present value of the proposed investment in new
machine for production of product P.
b) Calculate the internal rate of return of the proposed investment in new
machine for production of product P.
c) Advise on the acceptability of the proposed investment in new machine
for production of product P and discuss the limitations of the evaluations
that have been carried out.
Answer:
a) Calculation of net present value

Year 0 1 2 3 4
Rs. Rs. Rs. Rs. Rs.

YSZ P.T.O.
(56)

Sales revenue (WN 1) 728,000 1,146,390 1,687,500 842,400


Variable costs (WN 2) (441,000) (701,190) (1,041,750) (524,880)
––––––––– –––––––––– ––––––––––– –––––––––
Contribution 287,000 445,200 645,750 317,520
Capital allowances (depreciation)
(Cost ÷ 4) (250,000) (250,000) (250,000) (250,000)
––––––––– –––––––––– ––––––––––– –––––––––
Taxable profit 37,000 195,200 395,750 67,520
Taxation (11,100) (58,560) (118,725) (20,256)
––––––––– –––––––––– ––––––––––– –––––––––
After-tax profit 25,900 136,640 277,025 47,264
Capital allowances 250,000 250,000 250,000 250,000
––––––––– –––––––––– ––––––––––– –––––––––
After-tax cash flow 275,900 386,640 527,025 297,264

Initial investment (1,000,000)


Working capital (WN 4) (50,960) (29,287) (37,878) 59,157 58,968
––––––––––– ––––––––– –––––––––– ––––––––––– ––––––
Net cash flows (1,050,960) 246,613 348,762 586,182 356,232
PVIF at 12% 1·000 0·893 0·797 0·712 0·636
––––––––––– ––––––––– –––––––––– ––––––––––– ––––––
Present values (1,050,960) 220,225 277,963 417,362 226,564
––––––––––– ––––––––– –––––––––– ––––––––––– –––––––––
NPV = Rs. 91,154
Working Notes:
1) Sales revenue

Year 0 1 2 3 4
Selling price (Rs./unit) 20.00 20·80 21·63 22·50
23·40
(expected to increase @ 4% per year)
Sales volume (units) - 35,000 53,000 75,000
36,000
Sales revenue (Rs.) - 728,000 1,146,390 1,687,500
842,400

2) Variable costs

Year 0 1 2 3 4
Variable cost (Rs./unit) 12.00 12·60 13·23 13·89
14·58
(expected to increase @ 5% per year)
Sales volume (units) - 35,000 53,000 75,000
36,000
Variable costs (Rs.) - 441,000 701,190 1,041,750
524,880

3) Total investment in working capital

Year 0 investment = 728,000 x 0·07 = Rs. 50,960


Year 1 investment = 1,146,390 x 0·07 = Rs. 80,247
Year 2 investment = 1,687,500 x 0·07 = Rs. 118,125
Year 3 investment = 842,400 x 0·07 = Rs. 58,968

YSZ P.T.O.
(57)

4) Incremental investment in working capital

Year 0 investment = 728,000 x 0·07 = (Rs.50,960))


Year 1 investment = 80,247 – 50,960 = (Rs. 29,287)
Year 2 investment = 118,125 – 80,247 = (Rs. 37,878)
Year 3 recovery = 58,968 – 118,125 = Rs. 59,157
Year 4 recovery = Rs. 58,968

(b) Calculation of internal rate of return


Year 0 1 2 3 4
Rs. Rs. Rs. Rs. Rs.
Net cash flows (1,050,960) 246,613 348,762 586,182 356,232
PVIF at 20% 1·000 0·833 0·694 0·579 0·482
––––––––––– –––––––– –––––––– –––––––– ––––––––
Present values (1,050,960) 205,429 242,041 339,399 171,704
––––––––––– –––––––– –––––––– –––––––– ––––––––
NPV at 20% = (Rs.92,387)
NPV at 12% = Rs.91,154

IRR = 12 + [(20 – 12) x 91,154/(91,154 + 92,387)] = 12 + 4 = 16%(Approx.)

(c) Acceptability of the proposed investment in new machine production of Product P

 The NPV is positive. Hence, the proposed investment can be recommended on financial
grounds.
 The IRR is greater than the discount rate used by SC Co for investment appraisal
purposes. Hence, the proposed investment is financially acceptable. The cash flows of
the proposed investment are conventional and so there is only one internal rate of
return. Furthermore, only one proposed investment is being considered. Therefore,
there is no conflict between the advice offered by the IRR and NPV investment
appraisal methods.
Limitations of the investment evaluations
 Both the NPV and IRR evaluations are heavily dependent on the production and sales
volumes that have been forecast and so SC Co should investigate the key assumptions
underlying these forecast volumes. It is difficult to forecast the length and features of a
product‟s life cycle so there is likely to be a degree of uncertainty associated with the
forecast sales volumes. Scenario analysis may be of assistance here in providing
information on other possible outcomes to the proposed investment.
 The inflation rates for selling price per unit and variable cost per unit have been
assumed to be constant in future periods. In reality, interaction between a range of
economic and other forces influencing selling price per unit and variable cost per unit
will lead to unanticipated changes in both of these project variables. The assumption of
constant inflation rates limits the accuracy of the investment evaluations and could be
an important consideration if the investment were only marginally acceptable.
 Since no increase in fixed costs is expected because SC Co has spare capacity in both
space and labour terms, fixed costs are not relevant to the evaluation and have been
omitted. No information has been offered on whether the spare capacity exists in future
periods as well as in the current period. Since production of Product P is expected to be
more than double over three years, future capacity needs should be assessed before a
decision is made to proceed, in order to determine whether any future incremental fixed
costs may arise.
2.
YSZ P.T.O.
(58)

a) The market portfolio has an expected return of 15 percent and a


standard deviation of 20 percent. The standard deviation of ABC
Company's stock is 25 percent and its correlation coefficient with the
market portfolio is 0.50.
Required: (2+4+1+1=8)
i) What is the beta of ABC Company's stock?
ii) What would happen to the beta if ABC Company's standard
deviation were 40 percent? And, if the correlation coefficient were
0.60?
iii) Of the total variance of the stock in the original case, what
proportion is accounted for by Systematic risk? Unsystematic
risk? (The correlation coefficient squared is R-squared).
iv) What, in general, would happen to the total variance of a portfolio
if a portion of the ABC Company stock were sold and the
proceeds were invested in unrelated stocks having the same beta?
b) Three companies A, B and C are in the same type of business and
hence have similar operating risks. However, the capital structure of
each of them is different and the following are the details:
A B
C
Equity Share Capital (Rs.) 400,000 250,000
500,000
[Face value Rs. 10 per share]
Debentures (Rs) - 100,000
250,000
[Face value per debenture Rs. 100]
Market value per share (Rs.) 15 20
12
Market value per debenture (Rs.) - 125
80
Dividend per share (Rs.) 2.70 4
2.88
Interest Rate - 10%
8%
Assume that the current levels of dividends are generally expected to
continue indefinitely and the income-tax rate at 50%.
Required:
Compute the weighted average cost of capital of each company. 7

Answer:
a) Given,
Expected Return on market portfolio, E (Rm) =15%
Standard Deviation on market portfolio,σm = 20%
Standard Deviation on ABC Company, σB = 25%
Correlation coefficient between the company & market,pBm =0.50

i) ρBm * σB * σm
The beta of ABC company (βB) =
σm2
= 0.50 * 0.25 * 0.2/ (0.2)2 = 0.625

YSZ P.T.O.
(59)

ii) In both the instances, Beta increases which is evident from the following equation:
First, if the standard deviation of the company is 40 percent.
ρBm * σB * σm
The beta of ABC company (βB) =
σ m2
= 0.50 * 0.40 * 0.2/ (0.2)2 = 1
Second, if the correlation coefficient were 0.60.
ρBm * σB * σm
The beta of ABC company (βB) =
σ m2
= 0.60 * 0.25 * 0.2/ (0.2)2 = 0.75

iii) R- squared measures the proportion of the total variance of the dependent variable
explained by the independent variable. In our case, R squared = 0.5 * 0.5 = 0.25.
Therefore, 25 percent of the ABC Company's total variance is explained by market
movements (Systematic Risk) while unsystematic risk accounts for 75 percent of the
total variance.

iv) The more stocks with the same beta are added to a portfolio and at the same time the
ABC Company's stock is subtracted, the total variance of the portfolio declines.
Systematic risk stays the same, but unsystematic risk declines with diversification.

b) Calculation of Weighted Average Cost of Capital(WACC)

Amount Weights After Tax Weighted

(Rs.) Cost Cost

i) Cost of Capital of Shares at Market Value

A (400000×15/10) 6,00,000 1.00 (2.70/15) = 18% 18%


B (250000×20/10) 5,00,000 0.80 (4/20) = 20% 16%
C (500000×12/10) 6,00,000 0.75 (2.88/12) = 24% 18%

ii) Cost of capital of Debenture at Market Value


A - - - - -
B (100000×125/100) 1,25,000 0.20 (10/125) ×(1 - 0.5) = 4%
0.80%
C (250000×80/100) 2,00,000 0.25 (8/80) - 0.5) = 5%
1.25%

Weighted average cost of capital


A= + 00% = 18%
B= + 0.8% = 16.8%
C= + 1.25% = 19.25%
Working notes:
Weight of Share capital = MV of Share
MV of Share + MV Debenture.

YSZ P.T.O.
(60)

Weight of Debenture = MV of Debenture


MV of Share +MV
of Debenture
3.
a) Exorbitant Ltd. has investigated the profitability of its assets and the
cost of its funds. The result of such an investigation has indicated
that:
Current assets earn 1%, Fixed assets return 13%, Current liabilities cost
3%, and Average long-term funds cost 10%.
The current condensed balance sheet of the company is as follows:
Liabilities Amount (Rs.) Assets Amount (Rs.)
Long-term funds 140,000 Fixed assets 120,000
Current liabilities 20,000 Current assets 40,000
160,000 160,000
Required: (3+4.5+1.5=9)
i) Ascertain the net profitability of the firm.
ii) The company is contemplating lowering its net working capital to
Rs. 14,000 by (A) either shifting Rs. 6,000 of its current assets
into fixed assets, or (B) shifting Rs. 6,000 of its long-term funds
into current liabilities. Work out the profitability for each of these
alternatives. Which do you prefer? Why?
iii) Ascertain the effects on the net profitability, if both these
alternatives are implemented simultaneously.
b) A bank offers a fixed deposit scheme whereby Rs. 100,000 matures to
Rs. 126,250 after 2 years, on half-yearly compounding basis. If the
bank wishes to amend the scheme by compounding interest every
quarter, what will be the revised maturity value? 6

Answer
a)
(i) Computation of Net profit of the Firm:
Net profit = Total profit – Total cost of financing
Total profit = Return on fixed assets + Return on current assets
Return on fixed assets = 13% of Rs.120,000 = Rs. 15,600
Return on current assets = 1% of Rs. 40,000 = Rs. 400
Total Rs. 16,000

Total cost of financing = Cost of long-term financing + Cost of current liabilities (short-
term funds)
Cost of long-term funds = 10% of Rs. 140,000 = Rs. 14,000
Cost of current liabilities = 3% of Rs. 20,000 = Rs. 600
= Rs. 14,600
Net profit = Rs. 16,000 – Rs. 14,600 = Rs. 1,400

(ii) (A) Net profit on Shift of Rs. 6,000 of current assets into fixed assets:
Total Profit
Return on fixed assets = 13% of 126,000 = Rs. 16,380
Return on current assets = 1% of Rs. 34,000 = Rs. 340
YSZ P.T.O.
(61)

Rs. 16,720
Cost of financing:
As computed in (i) above Rs. 14,600
Net Profit = Rs. 16,720 – 14,600= Rs. 2,120

(B) Net profit on Shift of Rs. 6,000 of its long-term funds into current liabilities:
Total Profit = As computed in (i) above Rs. 16,000
Cost of financing:
Cost of long-term funds = 10% of Rs. 134,000 = Rs. 13,400
Cost of current liabilities = 3% of Rs. 26,000 = Rs. 780
= Rs. 14,180
Net profit = Rs. 16,000 – Rs. 14,180 = Rs. 1,820

The profitability of alternative (A) is more, i.e. when Rs. 6000 of current assets are
shifted to fixed assets. It is, therefore, preferable.

(iii)If both alternatives (ii) (A) and (ii) (B) are implemented simultaneously:
Total return = Rs.16,720 [as computed in (ii) (A)]
Total cost = Rs. 14,180 [as computed in (ii (B)]
Net profit = Rs. 16,720 – Rs. 14,180 = Rs. 2,540
Net Profit will increase.

b) Computation of Rate of Interest:


Principal = Rs. 100,000
Amount = Rs. 126,250
100,000 = 126,250
(1 + i)4
Pn = A X (PVF n, i)
100,000 = 126,250 (PV 4, i)
(PV 4, i) = 0.7921
According to the Table on Present Value Factor (PVF4, i), a PVF of 0.7921 for
half-yearly interest of 6 per cent becomes a lump sum of Re. 1. Therefore, the
annual interest rate is 2 X 0.06 = 12 per cent.
Revised maturity value, if interest is compounded quarterly:
Revised maturity value = 100,000 (1 + 12 X 1) 2 X 4 = 100,000 ( 1 + 3 )8
100 4 100
= 100,000 × (1.03) 8
= 100,000 × 1.267 [since (CVF 8, 3) = 1.267]
= Rs. 126,700
Therefore, the revised maturity value will be Rs. 126,700.

YSZ P.T.O.
(62)

4.
a) A 10-year, 12% semi-annual coupon bond, with a par value of Rs.
1,000 may be called in 4 years at a call price of Rs. 1,060. The bond
sells for Rs. 1,100. Assume that the bond has just been issued.
Required: (4+1+1+4=10)
i) What is the bond‟s effective annual yield to maturity?
ii) What is the bond‟s annual current yield?
iii) What is the bond‟s capital gain or loss?
iv) What is the bond‟s effective annual yield to call?
b) For each of the companies described below, would you expect it to
have a low, medium, or high dividend payout ratio? Explain why? 5
i) A company with a large proportion of inside ownership, all of
whom are high income individuals.
ii) A growth company with an abundance of good investment
opportunities.
iii) A company that has high liquidity and much unused borrowing
capacity and experiencing ordinary growth.
iv) A dividend paying company that experiences an unexpected drop
in earnings from an upward sloping trend line.
v) A company with volatile earnings and high business risk.
Answer
a) Given,

Par value (M) = Rs. 1,000


Coupon rate = 12%
Annual coupon (I) = 12% of Rs 1,000 = Rs 120
Maturity period (n) = 10 years
Call period = 4 years
Call price = Rs. 1,060
Selling price = Rs 1,100
Semi-annual compounding.

i. Calculation of bond‟s effective annual yield to maturity (YTM)

Vd = I(PVIFAkd% , n) + M(PVIFkd% , n)

So, for semiannual bond,

Rs 1,100 = I/2(PVIFAkd%/2 , n x 2) + M(PVIFkd%/2 , n x 2) ………………..(1)

Approximate semiannual YTM

YSZ P.T.O.
(63)

= 5.16%
Now, trying at 5%,

PV = Rs 60(PVIFA5% , 20 ) + 1,000(PVIF5% , 20)


= 60 x 12.4622 + 1,000 x 0.3769 = Rs 1,124.63 > Rs 11,00
Trying at 6%,

PV = Rs 60(PVIFA6% , 20 ) + 1,000(PVIF6% , 20)


= 60 x 11.4699 + 1,000 x 0.3118 = Rs 999.99 < Rs 1,100

By interpolating,

Semiannual YTM = 5.2%

Therefore, nominal YTM = 5.2% x 2 = 10.4%

The effective annual YTM = (1 + 0.052)2 – 1 = 1.1067 – 1 = 0.1067 = 10.67%

ii. Calculation of bond‟s current yield


= Coupon payments / Price of the Bond
= 120/1,100 = 0.1091
= 10.91%

iii. Calculation of bond‟s Capital gain or loss


We have,

YTM = Current Yield + Capital Gain or Loss Yield


10.67% = 10.91% + Capital Gain or Loss Yield
Capital Gain or Loss Yield = 10.67% - 10.91% = (0.24%)
So, Capital loss is 0.24%
iv. Calculation Bond‟s Yield to Call (YTC)
We have,

Rs 1,100 = Rs 60(PVIFAkd%/2 , 2 x 4) + Rs 1,060(PVIFkd%/2 , 2 x 4)

Approximate semiannual YTC

= 5.061%

Now, trying at 5%,

PV = Rs 60(PVIFA5% , 8 ) + 1,060(PVIF5% , 8)
= 60 x 6.4632 + 1,060 x 0.6768
YSZ P.T.O.
(64)

= Rs 1,105.20 > Rs 1,100

Trying at 6%,

PV = Rs 60(PVIFA6% , 8 ) + 1,060(PVIF6% , 8)
= 60 x 16.2098 + 1,060 x 0.6274
= Rs 1037.63 < Rs 1,100

By interpolating,

Semiannual YTC = 5.08%

Therefore, nominal YTC = 5.08% x 2 = 10.16%

The effective annual YTC = (1 + 0.0508)2 – 1 = 1.1042 – 1 = 0.1042 =


10.42%

b)
i. Low payout ratio. Highly taxed owners probably will want to realize their
returns through capital gains.
ii. Low payout ratio. There will be no or low residual funds.
iii. Medium or high payout ratio. There are likely to be funds left over after
funding capital expenditures. Moreover, the liquidity and access to borrowing
give the company considerable flexibility.
iv. Medium or high payout ratio. Unless the company cuts its dividend, which
probably is unlikely in the short run, its payout ratio will rise with the drop in
earnings.
v. Low payout ratio. The company will probably wish to retain earnings to build
its financial strength in order to offset the business risk

5.
a) Maya Limited is planning to change its credit policy which is
expected to increase the average collection period from one month to
two months. The relaxation of credit terms is expected to produce an
increase in sales volume by 25%. Following are other relevant data:
Sales price per unit Rs.10
Profit per unit Rs.1.5
Current Sales Revenue per annum Rs. 2,400,000
Required rate of return on investment 20%
Assume that the 25% increase in sales would result additional stock
of Rs. 100,000 and additional creditors of Rs. 20,000.
Required:
Advise the company whether the credit terms should be revised in
following circumstances: (4+3=7)
i) If all the customers take longer credit terms of 2 months.
ii) If current customers do not opt for revised credit terms and only
new customers opt the revised credit terms.

YSZ P.T.O.
(65)

b) Stock X has an expected rate of return of 10 percent, a beta


coefficient of 0.8, and a standard deviation of expected return of 15
percent. Stock Y has an expected rate of return of 15 percent, a beta
coefficient of 1.5, and a standard deviation of expected returns of 20
percent. The risk-free rate is 4 percent, and the market risk premium
is 6 percent.
Required: (2+2+2+2=8)
i) Which stock is riskier in terms of total risk?
ii) Which stock is riskier for diversified investor?
iii) Calculate each stock‟s required rate of return. What is its
significance?
iv) Calculate expected return of a portfolio that has Rs. 60,000
invested in Stock X and Rs. 40,000 invested in Stock Y?
Answer
a) The revision of credit terms is justifiable if the rate of return on the additional
investment in working capital exceeds 20%.

Extra profit from the revision


Profit Margin (1.5/10) = 15%
Increase in sales revenue = 2,400,000x25% = Rs. 600,000
Increase in profit = 600,000x15% = Rs. 90,000
Total sales revenue after revision = (2,400,000+600,000) = Rs. 3,000,000
Now, we need to calculate return on extra investment in working capital so as to assess whether
the revision of credit terms is justifiable. This is generally done by taking the figure of debtors
on the basis of cost of sales and sometimes on the basis of sales. Computations have been done
below by following both the basis.
(i) If all Debtors take two months’ Credit:
Cost of Sales
Sales Basis Basis
Rs. Rs.
Average Debtors after the Sales Increase 425,000 500,000
(2/12 x Rs. 3,000,000 x 85%)
(2/12 x Rs. 3,000,000)
Current Average Debtors 170,000 200,000
(1/12 x Rs 2,400,000 x 85%)
(1/12 x Rs 2,400,000)
Increase in Debtors 255,000 300,000
Increase in Stocks 100,000 100,000
355,000 400,000
Increase in Creditors (20,000) (20,000)
Net Increase in Working Capital Investment 335,000 380,000
Return on Extra Investment (Cost of Sales Basis) = 90,000/335,000 = 26.87%
Return on Extra Investment (Sales Basis) = 90,000/380,000 = 23.7%

(ii) If only the New Debtors take Two Months’ Credit:


Cost of Sales
Sales Basis Basis
Rs. Rs.
Increase in Debtors 85,000 100,000
(2/12 x Rs. 600,000 x 85%)
(2/12 x Rs. 600,000)
YSZ P.T.O.
(66)

Increase in Stocks 100,000 100,000


Increase in Creditors (20,000) (20,000)
Net Increase in Working Capital Investment 165,000 180,000

Return on Extra Investment (Cost of Sales Basis) = 90,000/165,000 = 54.55%


Return on Extra Investment (Sales Basis) = 90,000/180,000 = 50%

Recommendation:
In both the cases (i) and (ii), the new credit policy appears to be worthwhile
under both the basis. Furthermore, the most of the product can also support extra
sales. If the firm has high fixed costs but low variable costs, the extra production
and sales could provide a substantial contribution at little extra cost.

Alternative Solution:
Statement of evaluation of credit policy- if all debtors take 2 months' credit
Increase/
Particulars Current New (Decrease)
Average collection period (ACP) month 1 2
ACP in year 0.0833 0.1667
Required Rate of return on investment 20%
Sales Revenue Rs. 2,400,000 3,000,000
Sales Volume (SR/ SP) unit 240,000 300,000
Selling Price/ unit Rs. 10 10
Cost of sales/ unit (SP-P) Rs. 8.50 8.50
Profit/ unit Rs. 1.50 1.50

Profit before cost of investment Rs. 360,000 450,000 90,000


(Profit x Sales unit)
Less: Cost of investment (WN 1) 34,000 101,000 67,000
Net profit 326,000 349,000 23,000
Since net profit increases by Rs.23,000 the revision in credit policy is justifiable.

YSZ P.T.O.
(67)

Statement of evaluation of credit policy- if only the new debtors take 2 months' credit
Increase/
Particulars Current New (Decrease)
Average collection period (ACP) month 1 2
ACP in year 0.0833 0.1667
Required Rate of return on investment 20%
Sales Revenue Rs. 2,400,000 3,000,000
Sales Volume (SR/ SP) unit 240,000 300,000
Selling Price/ unit Rs. 10 10
Cost of sales/ unit (SP-P) Rs. 8.50 8.50
Profit/ unit Rs. 1.50 1.50

Profit before cost of investment Rs. 360,000 450,000 90,000


(Profit x Sales unit)
Less: Cost of investment (WN 2) 34,000 67,000 33,000
Net profit 326,000 383,000 57,000
Since net profit increases by Rs.57,000 the revision in credit policy is justifiable.

Working Notes:
1. Calculation of cost of investment: Rs. Rs. Rs.
Cost of Sales 2,040,000 2,550,000
(Sales unit * CoS per unit)
Investment in debtor (CoS x ACP in yr) 170,000 425,000
Increase in Stock 100,000
Increase in Creditors (20,000)
Additional working capital investment - 80,000
(a) Cost of investment in debtors 34,000 85,000 51,000
(Investment in Debtor x required rate of return)
(b) Cost of investment in additional WC - 16,000 16,000
(Additional WC x required rate of return)
Total cost of investment (a + b) 34,000 101,000 67,000

YSZ P.T.O.
(68)

2. Calculation of cost of investment: Rs. Rs. Rs.


Cost of Sales 2,040,000 2,550,000
(Sales unit * CoS per unit)
Investment in debtor (CoS x ACP in yr) 170,000 255,000
Increase in Stock 100,000
Increase in Creditors (20,000)
Additional working capital investment - 80,000
(a) Cost of investment in debtors 34,000 51,000 17,000
(Investment in Debtor x required rate of return)
(b) Cost of investment in additional WC - 16,000 16,000
(Additional WC x required rate of return)
Total cost of investment (a + b) 34,000 67,000 33,000

Note:
If only the new debtors take 2 months credit, New total investment in debtors is calculated as below:
Investment in debtors=
Current investment in debtor + (New CoS- Old CoS) x required rate of investment

b) Given,

Stock X Stock Y

Expected rate of return 10% 15%

Beta coefficient 0.8 1.5

Standard deviation of expected return 15% 20%

Rf = 4%
Market risk Premium (MRP) = 6%
i. Total risk is measured by coefficient of variance (CV).
CV of stock X= standard deviation/Expected rate of return
CV of stock X= 0.15/0.1= 1.5
CV of stock Y= 0.2/0.15=1.3333
Since stock X has higher CV, stock X is riskier than stock Y.

ii. Which stock is riskier for diversified investor?


For diversified investor, the stock having higher Beta is more riskier. Beta measures
the risk of individual assets relative to the market portfolio. The beta of the market
portfolio is 1. Assets with beta less than 1 are called defensive assets and assets with
beta greater than 1 are called aggressive assets. Risk free assets have a beta equal to
zero. Therefore, Stock Y is riskier as it has higher beta.

iii. Calculate each stock‟s required rate of return. What is its significance?
Required rate of return= Rf +Beta × MRP

Required rate of return of X = 4%+6% × 0.8=8.8%


Required rate of return of Y= 4%+6%×1.5=13%
iv) Calculate expected return of a portfolio that has Rs 60,000 invested in
Stock X and Rs 40,000 invested in Stock Y

YSZ P.T.O.
(69)

Expected Rate of Stock X = 10%

Expected Rate of Stock Y = 15%

Expected Rate of Portfolio = 10% 0.6 + 15% 0.4


= 6% + 6%
= 12%
6. Write short notes on: (4×2.5=10)
a) Debt Securitization
b) Significance of Debt-equity Ratio as a Measure of Long-term Solvency
c) Assumptions of Capital Asset Pricing Model (CAPM)
d) Sensitivity Analysis
Answer :
a) It is a method of recycling of funds. It is especially beneficial to
financial intermediaries to support the lending volumes. Assets
generating steady cash flows are packaged together and against this
asset pool, market securities can be issued, e.g. housing finance, auto
loans, and credit card receivables.
Process of Debt Securitisation:
(i) The origination function – A borrower seeks a loan from a bank
and financial institution. The credit worthiness of borrower is
evaluated and contract is entered into with repayment schedule
structured over the life of the loan.
(ii) The pooling function – Similar loans on receivables are clubbed
together to create an underlying pool of assets. The pool is
transferred in favour of Special purpose Vehicle (SPV), which acts
as a trustee for investors.
(iii) The securitisation function – SPV will structure and issue
securities on the basis of asset pool. The securities carry a coupon
and expected maturity which can be asset-based/mortgage based.
These are generally sold to investors through merchant bankers.
Investors are – pension funds, mutual funds, insurance funds.
The process of securitization is generally without recourse i.e.
investors bear the credit risk and issuer is under an obligation to pay to
investors only if the cash flows are received by him from the
collateral. The benefits to the originator are that assets are shifted off
the balance sheet, thus giving the originator recourse to off-balance
sheet funding
b) The Debt-equity (D/E) ratio is calculated by comparing the long term
debts with the total shareholders funds. The D/E ratio throws light on the
margin of safety available to the debt providers of the firm.
If a firm with a high D/E ratio fails, then a part of the financial loss may
have to be borne by the debt providers. Thus, the greater the D/E ratio,
higher would be the risk of the lenders. From the view point of
shareholders, a high D/E ratio implies that the firm is having a high
degree of financial leverage which offers the opportunity and benefit of
trading on equity. In such a case, if the rate of return of the firm is more
than the cost of debt, then higher degree of financial leverage means
relatively higher return to the shareholders.

YSZ P.T.O.
(70)

The higher D/E ratio may also have an impact on the ability of a firm to
service the debt. In addition to the payment of principal and interest on
debt, such a ratio might have an adverse impact on a firm‟s ability to pay
other fixed and contractual payments in addition to the principal and
interest. On the contrary, a low D/E ratio implies a low risk to the lenders
and creditors for the firm but it will not offer the benefit of trading on
equity. Therefore, a proper balance between the proportion of debt and
equity is very much essential in order to take care of the interests of both
the lenders and the shareholders and for the long term sustainability and
solvency of the firm.
c) The capital asset pricing model (CAPM) is based upon the following
assumptions:
i. The investors are basically risk averse and diversification is needed to
reduce the risk.
ii. All investors want to maximize the wealth and therefore choose a
portfolio solely on the basis of assessment of risk and return.
iii. All investors can borrow or lend an unlimited amount of funds at risk-
free rate of interest.
iv. All investors have identical estimates of risk and return of all securities.
v. All securities are perfectly divisible and liquid and there is no
transaction cost or tax.
vi. The security market is efficient and purchases and sales by a single
investor can not affect the prices which also mean that there is perfect
competition in the market.
vii. All investors are efficiently diversified and have eliminated the
unsystematic risk. Thus, only the systematic risk is relevant in the
determination of estimated return.

d) The net present value or Internal Rate of Return of a project is determined by analyzing the
after tax cash flows arrived at by combining forecasts of various variables like Sales
volume, unit selling price, unit variable cost, fixed cost etc. It is difficult to arrive at an
accurate and unbiased forecast of each variable. It can't be certain about the outcome of
any of these variables. The reliability of the NPV or IRR of the project will depend on the
reliability of the forecasts of variables underlying the estimates of net cash flows. To
determine the reliability of the project's NPV or IRR, we can work out how much difference it
makes if any of these forecasts go wrong.. We can change each of the forecasts, one at a
time, to at least three values: Pessimistic, expected and optimistic. The NPV of a project is
recalculated under these different assumptions. The method of recalculating NPV or IRR by
changing each forecast is called Sensitivity Analysis.

Sensitivity Analysis is a way of analyzing change in the project's NPV or


IRR for a given change in one of the variables. It indicates how sensitive a
project's NPV or IRR is to changes in particular variables. It basically
examines the sensitivity of the variables underlying the computation of NPV
or IRR rather than attempting to quantify risk. It can be applied to any
variable which is an input for the after tax cash flows. It can be conducted
with regard to volume, price, costs etc.
7. Distinguish between: (4×2.5=10)
a) External Capital Rationing and Internal Capital Rationing
b) Yield to Call and Yield to Maturity
c) Recourse and Non-recourse Factoring
d) Profit Maximization and Wealth Maximization Objective
Answer
a) External Capital Rationing mainly occurs on account of the imperfections in
capital markets. Imperfections may be caused be deficiencies in market
YSZ P.T.O.
(71)

information or by rigidities of attitude that may hamper the free flow of


capital. For example, A Ltd. is a closely held company. It borrows from the
financial institutions as much as it can. It still has investment opportunities,
which can be financed by issuing equity capital. But it doesn't issue shares.
The owners- managers do not approve the idea of the public issue of shares
because of the fear of losing control of the business.
Internal capital Rationing is caused by self- imposed restrictions by the
management. Various types of constraints may be imposed. e.g, it may be
decided not to obtain additional funds by incurring debt. This may be part of
management's conservative financial policy. Management may fix an
arbitrary limit to the amount of funds to be invested by the divisional
managers. Sometimes, management may resort to capital rationing by
requiring a minimum rate of return higher than the cost of capital.

b) The yield to Maturity (YTM) is the measure of a bond's rate of return that
considers both the interest income and any capital gain or loss. YTM is the
bond's internal rate of return. To calculate the Actual yield to maturity, first
the approximate yield to maturity is calculated as follws:
I + M – Vd
n ( 1- T)
AYTM=
M + 2 * Vd
3
Where, T = Tax Rate, M = Maturity Value, n = Maturity period, Vd = Market
Price of bond, I = Interest payment on bond.
After calculating the approximate yield to maturity, interpolation shall be
done by using the two rates which are below and above the approximate rates
to get the actual yield to maturity.
A number of companies issue bonds with buyback or call provision. Thus, a
bond can be redeemed or called before maturity. YTC is the yield or the rate
of return of a bond that may be redeemed before maturity. The procedure for
calculating the yield to call is the same as yield to maturity. The call period
would be different from the maturity period and the call or redemption value
could be different from the maturity value.

c) In a recourse or pure factoring, the factor firm is only involved in the work
of collection of the receivables. It does not bear any risk of default by the
debtors. Such a risk will have to be invariably borne by the selling firm.
Thus, in case of default by a customer, the selling firm will have to refund the
amount of advance together with charges as per the agreement which was
given by the factor to the selling firm against the receivables.
In a non-recourse factoring, the factor firm purchases the receivable of the
selling firm by paying the agreed amount (sales value less commission) to
the latter. The payment may be made immediately or after receiving from
the customer buying.
The main feature of non-recourse factoring is that the risk of default by the
buyer is borne by the factor firm and the selling firm receives the sales
amount. Thus, this type of factoring will result in the purchase of receivable
by the factor firm.

YSZ P.T.O.
(72)

In non-recourse factoring, the factor also undertakes the receivables


management including evaluation of creditworthiness, thereby also
assessing the risk of bad debts. In this type of factoring, the factor firm will
normally insist on discounting the seller‟s entire book debts subject to
careful examination of the debtors and their creditworthiness. In such cases,
the seller is entitled to sell to other customers not evaluated as credit-worthy
but these sales would obviously be excluded from the services of the non-
recourse factor.

d) The company may pursue profit maximization goal but that may not result
into creation of shareholder value. The profits will be maximized if company
grows through diversification and expansion. But all growth may not be
profitable. Only that growth is profitable where
ROA > WACC or ROE > KE or Firms invest in project with positive NPV,
However, profit maximization cannot be the sole objective of a company. It is at best a
limited objective. If profit is given undue importance, a number of problems can arise like the
term profit is vague, profit maximization has to be attempted with a realization of risks
involved, it does not take into account the time pattern of returns and as an objective it is too
narrow.
Whereas, on the other hand, wealth maximization, as an objective, means
that the company is using its resources in a good manner. If the share value
is to stay high, the company has to reduce its costs and use the resources
properly. If the company follows the goal of wealth maximization, it means
that the company will promote only those policies that will lead to an
efficient allocation of resources.

YSZ P.T.O.
(73)

CAP-II, Businesss Communication, Dec 2011


Suggested Answer

Maximum Marks –
100

Total No. of Questions - 10

Time Allowed – 3 Hours


Marks
All questions are compulsory.
Section -'A'
1. Read the following case carefully and answer the questions given below: 10
The CEO of the Siddhu Bank Ltd., with its head office based in Kathmandu,
has asked you, Senior Manager of the bank to write a reply to a group of
hard-working and devoted managers, who have frequently expressed their
dissatisfaction before the CEO that employees of some branches have to
work devotedly, whereas some in some other branches of the same office
spend their duty hours leisurely, gossiping and chit-chatting most of the
time. The complaint made argued that such a situation might have a negative
impact on the sincere and hard-working employees, ultimately resulting in
organizational ineffectiveness. Also they drew attention of the concerned
towards taking necessary measures.
Reply without concealing reality of the situation including the following:
a) That the top management of the bank is well aware of the situation.
b) That such a situation exists more or less in every organization, but it is
not good.
c) That decision has recently been taken to monitor and evaluate employee
performance of each and every section from the next month, and actions
would be taken against those who are negligent in their regular duties.

Answer No. 1

Jan 11, 2012

Siddhu Bank Limited


Babarmahal, Kathmandu
www.sidhu.org.np
014222133

Dear Managers,
We would, with serious concern, like to let you know that your complaint about the
lack of employee performance evaluation in the bank‟s operational system is justified to
some extent. The complaint clearly intends to hint upon negligence of top management
towards an absence of a fair and just employee evaluation system. Your suspicion
sounds reasonable to the extent that employees of some sections do not seem to be
working seriously and devotedly.
The reality of the situation is that disparity in staff performance exists in other
organizations also. This is a human factor and you would all realize that of all the
aspects of management, human resource management (HRM) is very crucial and
YSZ P.T.O.
(74)

challenging, because this is purely a subjective matter. But such an argument, however,
does not justify disparity in our organization. All the staff members of the organization
have to be equally performing better, ultimately contributing to organizational
effectiveness.
One thing, to the credit of the quality and excellence of your performance, your hard
work, devotion and dedication will pay off not only for your organization, but also for
honing up your own professional skills and performance effectiveness. Such a creative
activity should be taken as a stepping stone to career building.
For your information and satisfaction, the top management has recently taken decision
to regularly monitor and evaluate employee performance of all the sections to address
the problem with justice and impartiality. Departmental actions would be taken against
acts of negligence.
Rest assured that things will go well with the times.

On behalf of the top management,


Senior manager
Prakash Regmi

2. “Organizational communication cannot be complete and effective without


proper use of non-verbal communication”.
Justify this statement citing five different situations, in which organizational
people make the best use of non-verbal communication - cues, signs and
symbols, gestures, graphics, etc. to fulfill the intended communication need. 10

Answer No. 2

Non-verbal communication, entirely different from verbal communication, is employed in


particular circumstances in which the speaker remains incapable in communicating verbally,
or the speaker thinks that articulating something verbally might sound uneasy, impractical and
unethical. Non-verbal communication also applies to the style of directness and indirectness.
Indirect styles of non-verbal communication are used in circumstances in which the speaker is
poised to communicate matters (prohibited or inconvenient or unethical) indirectly.
There are lots of examples of situations in which verbal communication becomes inadequate,
inconvenient and non-verbal communication becomes indispensable.

Five different situations, in which organizational people make the best use of non-verbal
commutation – cues, signs and symbols, gestures, graphics, etc. to fulfill human
communication needs, are as follows:
Example No. 1.
During paper presentation, the director of an Insurance Company (ABC Insurance Company),
who intends to speak something in a Board meeting about the lingering corruption in the
organization he belongs to, cannot directly express the matter of corruption. At this, he is
obliged to use non-verbal communication. Wearing a reddish face, he rubs his palms together
to convey the message indirectly.
Example No 2.
Use of Signs: At offices and important public places “No smoking” message is transmitted
with a cross laid across a burning cigarette butt.
Example No. 3

YSZ P.T.O.
(75)

Someone wants to say good luck to a friend on his winning gold medal in a 200 meter race, but
he cannot do so because of the distance. Here, he holds his thumb upwards.
Example No 4.
Some villagers overtaken by a fear of vandalism created by a group of miscreants from a
neighboring village turn pale, wear frightened looks, and seem terrified, but afraid and unable
to speak anything. Their movements- the lowering of eyebrows, clenching of jaws and, labio-
dental contact (contact between lip and teeth) reflect their fear and anxiety.
Example No 5.
An angler in England will demonstrate the size of the fish he has caught by holding his two
hands, palms facing each other, but an angler from certain parts of Africa will measure off the
size along his left arm with his right hand.

3. Assume you didn‟t get the job. Although someone else was selected, you
hope that other jobs may become available. Write a 3-4 paragraph follow-up
letter that keeps the door open. Write in the „simplified letter style‟ format as
suggested in your textbook. 10

Answer No. 3

Prayag Marg
New Baneshwor, Kathmandu

July 23, 2011

Mr. Akhilesh Poudel


Human Resourse Manager
TransNepal Business House
Durbarmarg, Kathmandu

Dear Mr. Poudel

Although I am disappointed that someone else was selected for your accounting
position, I appreciate your promptness and courtesy in notifying me. I‟ve heard
much about the reputation and fame of your institution. I believe that the selection
process that you have adopted is highly reliable and justifying. The selected
candidate, I hope, is obviously more suitable than myself. I‟m well convinced that
my desire to serve in your institution will be fulfilled in the vacancy announcement
that will be made in the future.

Because I firmly believe that I have the technical and interpersonal skills needed to
work in your fast-paced environment, I hope you will keep my resume in your
active file. My desire to become a productive member of your TransNepal staff
remains strong.

I enjoyed our interview, and I especially appreciate the time you and Mr. Satish
Gyawali spent describing your company‟s expansion into international markets. To
enhance my qualifications, I‟ve enrolled in a course in International Accounting at
CSU. I have also submitted the certificate that I‟ve recently obtained after
completing the IT training launched by the Institution of Information Technology,
KU.

YSZ P.T.O.
(76)

Should you have an opening for which I am qualified, you may reach me at 9844-
××××××.

Sincerely
Hari Prasad Aryal

4. What do you understand by a meeting in organizational context? What are


the procedures to be followed for conducting an effective meeting? 10

Answer No. 4

Meeting is a formal gathering of people who are involved in an organization or office.


The members of the organization hold meetings in order to discuss and make decisions
about the issues related to their business. The participants of a meeting have to deserve
certain rules, regulations and maxims. Meeting is also defined as meeting is a series of
proceedings in which the chairperson and the secretary, apart from the members play
significant roles right from the first to the final stage/phase of the meeting.

Conducting an effective meeting in organizational context involves a series of


procedures.
Procedures followed during the preparation and meeting conducting phases are as
follows.
 Collecting facts, information available and relevant materials.
This is done by the secretary in consultation with the President
 Setting objectives with guidelines. This is done by the president with the support
and cooperation of the secretary.
 Preparing agendas and sub-agendas. The agendas are as per organizational
requirements. They are jointly produced by the president and the secretary.
 Considering procedures likely to be followed. They are decided as directed by the
president. In most cases, the members are informed about them before the meeting
is conducted.
 Pre-informing the participants about meeting details. This is the fundamental job of
the office- secretary. The information contains the agendas, venue, date and time of
the meeting. The secretary also makes available copies of decisions taken during the
preceding meeting.
 Getting and taking the attendance. The secretary should maintain in the attendance
register. The attendance should contain signatures put down against the names.
 Minuting down the discussions and decisions taken in the meeting. The secretary is
required to forward copies of the minutes after the meeting is over.

5. Write short notes on Any Four of the following: (4×2.5=10)


a) Minutes and follow-up
b) Checklist for improving listening
c) Ethical communication in business
d) Report and proposal
e) Executive summary

YSZ P.T.O.
(77)

Answer No. 5

a) Minutes and follow-up


The minutes of a meeting can be very different in style and detail: „minutes vary from the
cursory “bullet” variety to the long “court reporter” variety‟ (Tropman, 1996). At one end
of the scale, we have decisions or action points recorded as a list with no explanation or
elaboration of the discussion. This is appropriate for some meetings, say a small project
group. At the other end, we have a complete record of what everyone said. A verbatim
report of that kind is far too time-consuming, and is unnecessary for most if not all business
meetings. A useful compromise is to prepare „content minutes‟– for each item on the
agenda. A minute is written as two separate paragraphs which:
 summarize the main points in the discussion;
 summarize the decision taken or the action agreed, naming whoever has to carry it out,
and giving the timescale or deadline.

Follow-up is a process of confirming the implications of meeting decisions or minutes


when decisions are made in the meeting and they are not implemented well, then the
follow-up activities need to be carried out. The major goal of follow-up is to ensure the
implementation of the decisions of a meeting.

b) Checklist for improving listening: Listening is a receptive skill of communication. A


successful communication event requires very careful and decent subskills of listening. The
participants need to observe certain maxims for making listening more effective. The
following points can be involved in the checklist for improving listening.
 Stop talking
 Work hard at listening
 Block out competing thoughts
 Control the listening environment
 Maintain an open mind
 Paraphrase the speaker‟s ideas
 Listen between the lines
 Capitalize on lag time

c) Ethical communication in business


Every business is dependent on effective ethical communication. It's what makes
new policy in government, raises money for nonprofits and strengthens a business.
Business communication occurs any time a message is given or received, whether
it's verbal or nonverbal, between two businesses, a business and its employees or a
business and the public. The messages sent and received by a business need to
follow ethical norms that don't offend or make individuals feel uncomfortable.

d) Report and proposal


The general purpose of a report is to identify a specific problem, explain it and
recommend action that will lead to a solution. It is usually written to someone with
authority (e.g. manager, boss, director, public official, etc.) or peer (e.g. colleague,
associate, classmate, etc.). Generally, a report is written after a survey, research, or
action. It may include valuable recommendations related to the topic or isssue on
which it is written.

YSZ P.T.O.
(78)

The general purpose of a proposal is to identify a particular need, explain it and


recommend how this need can best be met. It is usually written to someone who
needs to make a decision usually which involves spending or investing money (e.g.
a client or customer, a committee, someone responsible for finances within
company or organization, etc.). A proposal is generally prepared before the action
or research. It may include details about plans, objectives, cost, time, methodology,
etc. needed for carrying out the action or research.

e) Executive summary
Executive summary denotes a short document that summarizes a longer report,
proposal or group of related reports in such a way that readers can rapidly become
acquainted with a large body of material without having to read it all. It will usually
contain a brief statement of the problem or proposal covered in the major
document(s), background information, concise analysis and main conclusions. It is
intended as an aid to decision making by business managers.

The executive summary will make a potential funder, partner or customer want to
go further. In many instances, it could be all that they look at to make a decision on
something important to you and your business success.

YSZ P.T.O.
(79)

CAP-II, Marketing, Dec 2011


Suggested Answer

Section -'B'

6. Read the following case carefully and answer the questions given below: (2×5=10)
Nabin Mahatto, a fresh chemical engineer, developed a new detergent
during his M. Tech. course. Upon passing out, he invested a small capital to
start manufacturing and selling the detergent under the brand name „Nippo‟.
Coming from a middle-class family himself, Mahatto decided to price his
detergent much lower than the other detergents available in the market at
that time. His pricing strategy paid rich dividends and Nippo became an
instant success. Using his newly acquired financial strength, Nabin
developed premium quality bathing soap with a unique colour and perfume
combination. Although, the cost price of this soap was high, Nabin priced it
lower to make it a comparable product in the market by maintaining a low
profit margin.
Questions:
a) Identify the factors Nabin considered in formulating pricing strategy of
the new bathing soap.
b) Comment on the suitability of pricing strategy in the case of the bathing
soap.
Answer No. 6
a) Determining the pricing strategy for a product depends on the product itself and
its distribution and promotion aspects. The pricing at the time of the launch of
the product is important from the point of growth or likely arrival of
competition. These factors were fully considered to formulate pricing strategy
for the new bathing soap.

b) I think, basically, keep an introductory price low and increase the same as the
sales build up. Stabilize the price as sales growth levels off. Reintroduce low
prices when the sales decline till the product has to be withdrawn.

Merits: Nabin's pricing strategy for the new bathing soap will help in easy and
quick growth of the sales during the introductory phase.

Demerits: This strategy doesn‟t take into account the fact that the product can
be easily copied. The strategy also doesn‟t cater to the effects of competition.
Even low price of a product like a new and unique bathing soap could create
doubts about the quality of the product among the customers.

7.
a) What is marketing? Explain the major features of marketing in Nepal. (2+3=5)
b) What are the requirements for effective segmentation of a consumer
market? 5

Answer No. 7

YSZ P.T.O.
(80)

a) Marketing is a total system of business activities designed to plan, price,


promote and distribute want satisfying product to target markets to achieve
organizational objectives. Marketing process involves planning and
implementation of all the marketing related activities to meet organizational
goals. It includes all the exchange related business activities which direct the
flow of goods and services from the producer to consumers. Today marketing is
fast growing, dynamic and highly competitive. The entire business activities
should be customer-oriented. Customer supremacy is seen in modern marketing,
besides realization of social responsibilities. Marketing provides values and
benefits to customers, firms and the society. Technology, globalization, growing
income and support services, have helped a lot in the fast growth of marketing
activities. A large number of people have got employment in this sector.

The following major features are found in Nepalese marketing:

 Low customer awareness


 Price sensitive customers
 Domination of middlemen
 Weak transportation system
 Lack of professionalism
 Small market size
 Uncertain environment
 Less customer orientation
 Weak information and communication
 Push selling; behavioral variables are neglected, etc.

b) Business organizations operate in different markets and the markets consist of


all types of existing and potential customers with their varying needs and wants.
Market Segmentation is the process of sub-dividing or breaking up the market
into different cohesive and homogeneous groups with a distinct marketing mix.
The Requirements for effective segmentation are:

Measurable: The size, purchasing power and profits of the segments should be
measured.

Accessible: The market segments should be effectively reached and served.

Substantial: The market segments should be large or profitable enough to serve.

Differential: The segments should be conceptually distinguishable and respond


differently to different marketing mix elements and programs.
Actionable: Effective programs should be designed for attracting and serving
the segments.
8.
a) "Buyer behavior is a decision making process". Comment. 5
b) Describe the role of product lifecycle in marketing decision making. 5
Answer No. 8

YSZ P.T.O.
(81)

a) While trying to understand the complex buyer decision making process,


marketers are making an attempt to understand the buyer behavior in terms of
who buys products or services, what they buy, how do they buy, when do they
buy, where do they buy, why do they buy, how often do they buy and so on
before developing marketing plans. It has been understood that factors
influencing the buyer behavior are social, cultural, personal and psychological.
Marketers are approaching this problem of trying to understand the complex
buyer behavior with the help of various models and theories. The buyer‟s choice
is the result of complex interplay of the influencing factors of buyer behavior.
Thus, buyer behavior is decision making process.

b) The product life cycle is a concept that attempts to describe a product's sales
profits, customers, competitors, and marketing emphasis from its beginning
until it is removed from the market. The product life cycle concept became
popular for the first time in 1965 through the efforts of Theodore Levitt.
Actually, the product life cycle is an attempt to recognize distinct stages in the
sales and profit history of the product. It explains the following facts to the
marketing decision makers:
 Products have a limited life after which the product may be dead if
appropriate strategies are not adopted.
 Product sales pass through distinct stages, each posing different challenges
to the seller.
 Product profits rise and fall at different stages of the product life cycle.
 Products require different marketing, financial, manufacturing, purchasing,
and personnel strategies at different stages of their life cycle.

The product life cycle helps marketers in the following ways:


i. It communicates the marketer that a product, like a living creature, has a
limited life.
ii. It helps the marketers in identifying the sales and profit situation of the
particular product in the market place.
iii. It helps to adopt right investment strategy at different stages of product
life cycle.
iv. It helps to adopt appropriate pricing, promotion, distribution, and
competitive policy in different stages of product life cycle.
v. It also warns the marketer and makes him alert about changes regarding
sales, profit, and competition.

More particularly, the product life cycle concept helps a marketer to formulate
and adopt suitable marketing, investment and competitive strategies at right
time.
9.
a) Discuss the effects of Socio-political environment on Nepalese
marketing system. 5
b) Give the meaning of advertising and explain the objectives of
advertising. 5

Answer No. 9
YSZ P.T.O.
(82)

a) The socio-political environment include two major marketing environmental


forces – social environmental and political environment. These are two major
environmental forces that affect freedom in work. If the social and political
environments are suitable, they will provide freedom in work and vice versa. In
Nepal both social and political environments are not supporting business
communities. Some of the major reasons are as follows:
i. Frequent change in government has annoyed business sector;
ii.Political unrest has created risk of security to the people;
iii.
Lawless situation has pushed the political cadres towards social barbarism;
iv.Labor unrest has damaged industrial sector and hence weakened production
and productivity in the country;
v. The present social and political environment has made the country
dependent on foreign products.

b) Advertising is any paid form of non-personal presentation and promotion of


ideas, goods, or service by an identified sponsor. It is a cost effective way to
disseminate message to scattered large masses. Advertising can be used to meet
the goals like giving information, persuading consumers, reminding and
reassuring customers.

Advertising today has got a prominent role in the promotion program of all
firms/companies. Many methods, media, agencies are playing active role to
meet advertising objectives of the companies. A lot of people are involved to
design attractive advertising focusing on mission, money, message, media, and
measurement elements known as the five Ms of advertising. Newspapers,
journals, posters, radio, TV, video are a few ad vehicles popularly used by the
organizations.

The following are some of the objectives of advertising:


 Communication of information/message
 Creation and extension of demand
 Introduce new product.
 Educate, persuade and remind.
 Influence at sight of buying decision.
 Save time and money.
 Selling on mass scale.
 Promotion of competition/ image building.
 Help middlemen, etc.

10. Briefly explain the following : (5×2=10)


a) Channel Dynamics
b) Decision support system as a component of MIS
c) Pull Vs Push strategy
d) Test marketing
e) E-commerce

YSZ P.T.O.
(83)

Answer No. 10
a) Channel Dynamics: The distribution channel system in marketing is dynamic.
Channel dynamics means the changes that evolve in the distribution channels
due to emergence of new marketing intermediaries (i.e. wholesalers and
retailers). Distribution channels do not stand still. They change in accordance
with the emergence of new channel participants. There are two types of channel
dynamics i.e. structural dynamics and behavioral dynamics.
b) Decision support system, also called as information analysis system or
marketing decision support system, is a set of statistical or mathematical tools
and decision models that help the marketing managers in analyzing data
gathered or supplied through MIS for making rational decision. Analytical
system helps marketers to make decisions on the problems by telling the truth
about the situations. In other words, analytical systems allow a firm to ask,
"Why did that happen?" and "What will happen if……..?" Mathematical and/or
statistical tools and decision models are used to solve several marketing
problems.
c) Pull Vs Push strategy: The promotion mix is affected by whether the
organization adopts a push strategy or a pull strategy. Pull strategy calls for
spending a lot on advertising and consumer promotion to build up consumer
demand. If the strategy is successful, consumers will ask their retailers for the
product, the retailers will ask the wholesalers and the wholesalers will ask the
producers. Push strategy calls for using the sales force and trade promotion to
push the product through channels. The producer promotes the product to the
wholesalers, the wholesalers to the retailers and the retailers to consumers.
d) It is related with testing commercial viability of new product and the marketing
mix designed for the product in the context of a market segment on a limited
scale. Marketing is done to make necessary changes in the marketing mix. The
product is sold in selected areas with the help of local level promotions in few
shops. Test marketing is periodically evaluated in terms of trial rate, advertising
effectiveness, and repeat purchase rate like variables.
e) E-commerce or electronic commerce refers to selling and buying something
electronically by using Internet facility. Therefore, e-commerce is also known as
„Internet shopping‟ or „Internet commerce‟. The buying or selling products
through Internet are software, documents, graphics, music, etc. These days,
physical products are also marketed through Internet.

Basically, there are three types of e-commerce; they are as follows:


i. Business-to-Consumer (B2C) Electronic Commerce: This type of
electronic commerce involves retailing products and services directly to
individual shoppers. For example, Barnes&Noble.com sells books, software,
and music to individual consumers.
ii. Business-to-Business (B2B) Electronic Commerce: This type of electronic
commerce involves in selling of goods and services among businesses. Web
auction site is used for selling cutting tools, grinding wheels, and metal
working fluids to more than 100,000 small machining businesses.
iii. Consumer-to-Consumer (C2C) Electronic Commerce: This type of
electronic commerce involves consumers selling directly to consumers. For
example, eBay, the giant Web auction site, allows people to sell their goods
to other consumers by auctioning the merchandise off to the highest bidder.

YSZ P.T.O.
(84)

CAP II - Income Tax & VAT – Dec 2011


Suggested Answers

Maximum Marks - 100

Total No. of Questions - 6

Time Allowed - 3 Hours


Marks
Attempt all questions. Working note should form part of the answer.

8. Answer the following with reference to the Indian Income Tax Act, 1961.
a) The term “Assessment Year” and “Previous Year” have the same
meaning. Do you agree? If not why? 2
b) List down the incomes which do not form part of Total Income? 2
c) During the previous year 2009-10 R and Sons HUF was partly
controlled from India by its Karta R who is citizen of India but stays
outside India. For the purpose of managing the affairs of the HUF, R has
been regularly visiting India. Determine the residential status of the HUF
for the assessment year 2010-11 if; 6
i) R has been visiting India for 100 days every year for the last 12 years
ii) R has been visiting India for 110 days every year for the last 12 years
iii) R has been visiting India for the last 12 years. During the
immediately preceding 4 previous years he was in India for 50 days
every year and prior to that for 200 days every year.
Answer No.1
a)
The terms “Assessment Year” and “Previous Year” have the different meaning as per
Income Tax Act, 1961 of India.
As per section 2(9) of Income Tax Act, 1961 of India, “Assessment Year” means the
period of twelve months commencing on the first day of April every year. It is
therefore, the period from 1st of April to 31st of March.
As per section 2(34) of Indian Income Tax Act, 1961 “Previous Year” means the
previous year as defined in section 3. According to section 3, “Previous Year” means
the financial year immediately preceding the assessment year.

b)
As per Income Tax Act, 1961 of India the following incomes do not form part of total
income subject to fulfilling the conditions mentioned in respective sections;
i. Income not included in total income of any person (section 10)
ii. Income of newly industrial undertaking in Free Trade Zone, etc (section
10A)
iii. Income of newly established units in Special Economic Zones (section
10AA)
iv. Income of newly established in 100% Export Oriented Undertaking
(section 10B)
v. Profit and gains derived by an undertaking from the export of eligible
article or things (section 10BA)
vi. Income from property held for charitable or religious purposes (section
11-13)
YSZ P.T.O.
(85)

vii. Income of political parties (section 13A)

c)
During the previous year 2009-10, the control and management of the affairs of the
HUF was partly situated in India. Hence the HUF is resident for the assessment year
2010-11.
It is further to be examined whether the HUF can be said to be “resident and ordinarily
resident in India”. For this purpose, the Karta should satisfy both the following
conditions;
i. He (Karta) has been resident in India in at least 2 out of 10 previous
years immediately preceding the relevant previous year; and
ii. He has been in India for a period of 730 days or more in 7 previous years
immediately preceding the relevant previous year

In case (A) R neither satisfies the first condition nor the second condition mentioned
above as Karta is citizen of India. Where an individual is citizen of India and who is
outside India makes a visit to India he shall be said to be resident of India only when he
was an in Indian in the relevant previous year for 182 days as the condition of 60 days
along with 365 days in the 4 preceding previous years shall not apply. In this case 60
days shall have to be substituted by 182 days. Therefore, the HUF is resident but not
ordinarily resident.

In case (B), R, although satisfies condition (ii) mentioned above as he was here for 770
days in the past 7 years but he does not satisfy the condition (i) above as he is not a
resident in India in at least two out of ten preceding previous years. Therefore, the HUF
is resident but not ordinarily resident in India.

In case (C) R was non-resident for 4 preceding previous years but prior to that he was
resident. Hence, the first condition of category B, being resident in at least 2 out of 10
preceding years is satisfied. Further, he has been in India for 800 days (50*4+200*3)
during 7 years preceding the relevant previous year. Therefore, the second condition of
being in India for 730 days or more is also satisfied. Hence, the HUF is resident and
ordinarily resident in India.

9.
a) Mr. Bhai Raja is a person working in a Financial Institution as Chief
Manager. Besides working in Financial Institution, he is a professor of
Account in Shankar Dev Campus. He generates the following income
from the employers in Fiscal Year 2067/68.
i) Monthly Remuneration NPR 40,000; Dashain Allowance NPR
40,000; Education Allowance NPR 2,000 per month; Expenditure
Allowance NPR 2,000 per month; Bonus NPR 60,000;
Remuneration from teaching in Shankar Dev Campus NPR 6,000
per month.
ii) He is provided a car along with driver for his official and personal
purpose. Monthly remuneration and allowance of driver is NPR
8,000.

YSZ P.T.O.
(86)

iii) Petrol Expenditure is provided for his car which is NPR 10,000 per
month and maintenance expenditure of such car is NPR 25,000 in
Fiscal Year 2067/68.
iv) He is provided a housing facility by the employer.
v) Monthly expenditure of Telephone connected in his resident is NPR
2,000 per month. Out of this 50% is his personal telephone
expenditure.
vi) A security guard is being provided in his resident and it is being
paid NPR 6,000 per month by the employer. Because of being used
the security guard in his resident, NPR 3,000 is being deducted from
his monthly salary.
vii) He has used the loan amounting to NPR 10, 00,000 under housing
loan facility at the rate of 5% interest. 8% interest rate is being
charged by the institution under similar loan to other borrowers.
viii) He has received one month salary against his house leave during
Mangsir, 2067.
ix) The employer contributes 10% of his drawn salary and the
employee same amount from his salary, to approved provident fund.
Also, the employer contributes NPR 20,000 per month to Citizen
Investment Fund up on request of employee.
x) He has covered the insurance of NPR 250,000 and NPR 150,000 of
his wife in Rastriya Beema Sansthan. He pays the premium of NPR
19,000 and NPR 14,000 respectively in that Fiscal Year. Also, he
has contributed to tax exempted entity approved from department as
donation amounting to NPR 60,000.
xi) He has expensed out NPR 20,000 for his treatment in Teaching
Hospital.
xii) He and his wife declared that they are couple in the Fiscal Year
2067/68 and his wife does not have any income in that Fiscal Year.
Compute Assessable Income, Taxable Income and Tax liability of Mr.
Bhai Raja for the Fiscal Year 2067/68. 12

b) Mr. Devan Mahara is lawer. He is using cash basis of accounting for the
income generated from his profession. He gave an application to the
department to change his basis of accounting from cash to accrual in
Fiscal Year (FY) 2067/68. In accordance with the generally accepted
accounting principles, Income Tax Department has given the approval to
account for his income on accrual basis in Fiscal Year 2067/68. The
accounts of Mr. Mahara immediately before changing the basis of
accounting, was as below;
i) Service has already been rendered in FY 2066/67 but amount has not
been received and hence no income is included in that FY NPR
80,000
ii) In FY 2066/67 advance has been taken (service has been rendered on
FY 2067/68) but amount is included in that FY on cash basis though
service is still to be rendered NPR 30,000
iii) In FY 2066/67 House Rent from Magh, 2066 to Poush 2067 has
been paid and expense has been claimed in FY 2066/67 on cash basis
of accounting NPR 48,000
State how you make adjustments of income and expense in FY 2067/68
in above conditions? 5
YSZ P.T.O.
(87)

c) Adishree & Co. is conducting business in Pokhara and has taken the
office premises on rent from Mr. Garibdash. According to agreement
between them, Adishree & Co. is required to pay the house owner NPR
50,000 per month. Further, Adishree & Co.has taken on rent a vehicle
for NPR 25,000 from Mr. Daman Thapa for its business purpose. Mr.
Daman Thapa has paid the annual tax as per Section 1 (13) of Schedule
1 of Income Tax Act.
State whether the TDS as per Income Tax Act, 2058 is to be deducted in
above circumstances. If yes, what is the amount to be deducted as TDS? 3

Answer No.2
a) Computation of Assessable Income, Taxable Income and Tax liability of Mr. Bhai Raja for
the Fiscal Year 2067/68
Amount
Particulars (NPR)

Salary 480,000

Dashain Allowance 40,000

Education Allowance 24,000

Expenditure Allowance 24,000

Bonus 60,000

Vehicle Facility (480,000*0.5%) 2,400


Driver's Salary Not taxable
Petrol Expenditure Not taxable
Maintenance Expenditure Not taxable

Housing Facility (480,000*2%) 9,600

Telephone Expenditure (2,000*12*50%) 12,000

Security Guard Facility [(6,000-3,000)*12] 36,000

Loan Facility (8%-5%)*10,00,000 30,000

Salary against House Leave 40,000

Contribution to Provident Fund (PF) 48,000

Income from employment from Financial Institution 806,000

Salary from teaching in Shankar Dev Campus 72,000

Assessable income from employment 878,000


Deductible Amount

Deductible amount against contribution to PF (292,667)


Actual Contribution
Employer's Contribution 48,000
YSZ P.T.O.
(88)

Employee's Contribution 48,000


Contribution to Citizen Investment Fund 240,000
Total Contribution Amount 336,000
1/3 rd of Assessable income 292,667
Maximum Limit 300,000

Adjusted Taxable Income 585,333

Donation Expenditure (29,267)


5% of Adjusted Taxable Income 29,267
Actual Donation Expenditure 60,000
Maximum Limit 100,000

Taxable Income 556,067

Deduction against Insurance (20,000)


Premium paid for himself and for his wife 33,000
Maximum Limit 20,000

Taxable Income (For Tax Calculation) 536,067


Computation of Tax Liability
Up to NPR 200,000 (For Couple) 2,000
NPR 100,000 @ 15% 15,000
Balance @ 25% 59,017
Total 760,17
Less: Medical Expenditure (750)

Total Tax Liability 75,267

Notes:
1. Mr. Bhai Raja has expensed out NPR 20,000 in medical treatment. 15% of NPR
20,000 comes to NPR 3,000. Out of this NPR 3,000, NPR 750 is deductible from his
tax liability as medical claim and balance NPR 2,250 can be claimed in subsequent
Fiscal Year.
2. Since Mr. Bhai Raja has claimed Donation expenditure, contributed to approved fund
and has two employers at the same time he has to file the return of income.
3. As per Income Tax Directive, 2066, In case the employer also provides driver and
fuel facility to the employee along with the vehicle facility, the salary of the driver and
the cost of fuel is also not required to add to the taxable salary of the employee.
4. As per sec. 1 (2) (ka) of Schedule 1 of the Act, income tax at the rate 1% is to be
deducted from the salary income up to Rs. 200,000. The 1% salary tax is also a part of
income tax, but, it is named as social security tax and as per a circular this amount is to
be deposited in a separate account.

b) As mentioned in Chapter 6 of Income Tax Act, 2058, if any person takes the approval
from Inland Revenue Department for the change in basis of accounting for the purpose
of taxation, or his basis of accounting is changed because of various provisions
mentioned in the same Chapter, inclusion of his income, claim of expenditure shall be
computed in such a manner that there is no repetition or short of income or claim of
expenditure in that Fiscal Year.
Accordingly, the following adjustments shall be made in Fiscal Year 2067/68;
i. For the service which has already been rendered in FY 2066/67 but no amount
is received, the amount of NPR 80,000 shall be included in income of FY

YSZ P.T.O.
(89)

2067/68 on accrual basis.

ii. The amount of NPR 30,000 taken as advance in FY 2066/67 and included in the
income of same FY is not required to be included in FY 2067/68 though
service has been rendered in this FY. The amount of NPR 30,000 already
included in income of FY 2066/67.

iii. House Rent of Six months amounting to NPR 24,000 though pertains to FY
2067/68 has already been claimed in FY 2066/67 on cash basis of
accounting, so no further claim is allowed in FY 2067/68.

After making adjustments of inclusion of income and claim of expenditure in FY


2067/68, the income or expenditure shall be accounted on accrual basis for subsequent
years.

c) As per section 88(1)(5) of Income Tax Act, 2058, resident person is required to deduct
withholding tax on payment of rent, having source in Nepal, on building or vehicle
@10% of the amount of the rent.

TDS deducted on house rent is final withholding income for Mr. Garibdash.

In case of rent for vehicle those taxed under sec. 1 (13) of schedule 1 of Income Tax
Act, 2058, even the tax paid is equal to the total of the tax liability of the person for the
year, but sec. 88 (4) does not relieve a person from deducting withholding tax under
such situation. Thus, TDS @ 10% should be deducted from payment of rent.

10.
a) Nirdhan Samaj Sewa is a not for profit organization registered in
Kathmandu. It has obtained tax exemption certificate from IRD. During
the year 2067/68 the organization received the following incomes:
Membership Fees Rs. 275,000
Interest Rs. 170,000
Donation Rs. 450,000
Dividend income Rs. 1,200,000
Net profit from runnig a school Rs. 120,000

Nirdhan Samaj Sewa has claimed that it is a tax exempted organization


and so it is not obliged to pay tax and has filed a refund application for
TDS on interest of Rs. 30,000. Justify its claim on the basis of the
provisions of Income Tax Act, 2058.
Also state the legal provisions regarding filing of income tax return and
renewal of tax exemption certificate by Nirdhan Samaj Sewa. 5
b) Torrent Co. Ltd. is a large publicly listed company whose main activity
involves construction contracts. Torrent won a global contract to build a
new football stadium owned by Star Youth Club at a contracted price of
YSZ P.T.O.
(90)

Rs. 40,00,000 thousand. This construction contract was to be completed


over the period of three years starting from financial year 2065/66.
Initially, the total cost of completing the said stadium was estimated at
Rs. 36,00,000 thousand.
Star Youth Club had placed additional variation order in 2nd and 3rd
financial years with the details as given below:
2066/67 2067/68
Variation order Rs. 70,000 thousand Rs. 80,000
thousand

As the consequence of variation orders as aforesaid, it had to incur the


additional cost which had been estimated as follows:
2066/67 2067/68
Additional estimated costs Rs. 50,000 thousand Rs. 4,20,000
thousand

Details of actual cumulative costs over the period of 3 years to build


above football stadium were as follows:

(Rs. In ‘000)
2065/66 2066/67 2067/68
Actual accumulated costs to date 18,00,000 26,28,000 39,86,000
Rectification costs - - 34,000

The rectification costs are the costs incurred in widening access roads to
the stadium. This was the result of an error by Torrent‟s architect when
he made his initial drawings.

Compute the extracts of taxable income over those three years ending
F/Y 2067/68 and amount of losses carried back. 15

Answer No.3
a) Applicability of tax in the income of Nirdhan Samaj Sewa is as follows:

a. As Nirdhan Samaj Sewa is “not for profit organization” and it has obtained tax
exemption certificate from IRD, its income from membership fees and donation
will be exempted from tax.

b. Interest Rs. 170,000 and dividend Rs. 1,200,000 are the final withholding
income. Therefore, claim of Nirdhan Samaj Sewa is not tenable and it could not
claim for the refund of TDS deducted in interest income as it is final
withholding tax.

c. Profit generated from the operation of school is exempt income of the


organization if it is operated as per its constitution/bidhan. If running of school
is not as per its constitution/bidhan, then it will be fully taxable treating it as
business income.

Filing of income tax return and renewal of tax exemption certificate:

According to Rule 5 of Income Tax Regulation, 2053 and Inland Revenue


Department Circular No 2065/66 dated 2065/05/02 prescribes the requirement to
YSZ P.T.O.
(91)

tax exempted organizations to submit the documents and renewal of tax exemption
certificate.

b) Statement of computation of Taxable Income for M/s Torrent Company Ltd.

(Rs. In „000)
Details F/Y 2065/66 F/Y 2066/67 F/Y 2067/68
Initial contract price 40,00,000 40,00,000 40,70,000
Variation Order - __ 70,000 __ 80,000
Sub-total 40,00,000 40,70,000 41,50,000
Total contract price (A) 40,00,000 40,70,000 41,50,000

Initial estimated total cost 36,00,000 36,00,000 -


Additional estimated cost - __ 50,000 -
Total estimated cost (B) 36,00,000 36,50,000 -

Actual yearly expenses (Cumulative) 18,00,000 26,28,000 39,86,000


Rectification costs - __ - 34,000
Total actual costs (C) 18,00,000 26,28,000 40,20,000

Contract completion percent (E)


C/B * 100 50% 72% 100%

Computation of annual income


Contract revenue (Total) 40,00,000 40,70,000 41,50,000
Contract cost actual (36,00,000) (36,50,000) 40,20,000
Estimated/income from the contract 4,00,000 4,20,000 1,30,000
Portion of this year‟s income 2,00,000 3,02,400 1,30,000
Previous year‟s income 0 2,00,000 3,02,400
Net income of the year 2,00,000 1,02,400 (172,400)

As per sec. 20(4) of the Act, the Company has to seek advice from IRD, and IRD may
allow it to carry back the loss up to profit assessed during the year. So the Company
shall get back the amount of tax paid as prescribed by IRD at the applicable rate on Rs.
172,400. The person can apply for refund of the amount.

11.
a) AMEXO Bank is registered in the USA and operating its liaison office
in Kathmandu. During the financial year 2067/68, it has following
summarized transactions.

(Rs. In ‘000)
Income recognized 50,00
Expenses 40,00 except income tax

The liaison office has policy to repatriate all the remaining profits to its
corporate office.

Compute maximum amount that can be repatriated from Kathmandu


liaison office. 5
YSZ P.T.O.
(92)

b) The paid up share capital of ABC Ltd. consists of 1,50,000 shares of Rs.
100 each. As of Ashadh end 2068, the P&L Appropriation Accounts of
the company is abstracted as below:
P&L Appropriation Account:

Profit for the year Rs. 10,00,000


Accumulated profit b/d Rs. 15,00,000
Total distributable profit Rs. 25,00,000

The Board of Directors (BOD) of the company proposed to issue bonus


share at 10%. Also, it is proposed that dividend tax on the capitalized
profit (sec 88(1)) is to be borne by company by distributing cash
dividend so that shareholders do not have to pay any tax at source as
required under sec. 88(1).
You are required to compute the amount of such cash dividend. 5
Answer No.4
a) As per the provision of Income Tax Act, AMEXO is taxed at two stages i.e.

 Resident bank having taxable income @ 30% of taxable amount.


 5% on repatriated amount.

Now,
Taxable amount = Rs. 5000 thousand – Rs. 4000 thousand
= Rs. 1000 thousand
Tax @ 30% = Rs. 1000 thousand * 30%
= Rs. 300 thousand
Remaining profit = Rs. 1000 thousand - Rs. 300 thousand
= Rs. 700 thousand
Tax of repatriated amount = Rs. 700 thousand * 5/105
= Rs. 33.33 thousand

Therefore, amount that can be repatriated = Rs. 700 thousand – Rs. 33.33 thousand
= Rs. 666.66 thousand

b) Total number of bonus share to be issued @ 10% = 15,000 shares


Total amount of bonus shares = 15,000 share @ Rs. 100 each
= Rs. 15,00,000
Dividend Tax @ 5% = 5% of Rs. 15,00,000
= Rs. 75,000

In case withholding tax on such bonus shares is to be paid by ABC Ltd. itself by
proposing to declare cash dividend, such cash dividend (i.e. tax on bonus shares)
is deemed as further distribution of dividend subject to dividend tax @ 5%.

Therefore, total cash dividend to be proposed = Rs. 75,000 + (5/95 *75,000)


= Rs. 75,000 + Rs. 3,947.37
= Rs. 78,947

YSZ P.T.O.
(93)

12. Explain the provisions of the Value Added Tax Act relating to the
following: (4×5=20)
a) Applicability of VAT on import of goods and services.
b) “Situation out of control” as prescribed in Section 19(4) of VAT Act,
2052.
c) Temporary registration of VAT.
d) Provisions relating to excess VAT credit under section 24 of VAT Act,
2052.
Answer No.5
a) Applicability of VAT on import of goods and services.

Section 28 of the VAT Act, 2052 prescribes provisions relating to import of goods. Customs
Officer shall collect VAT on the goods imported into Nepal as per VAT Act, 2052 if
Ministry of Finance, Government of Nepal has not prescribed otherwise.

The Customs Office can use all the authorities as per Value Added Tax Act, 2052 and
Customs Act for the purpose of collecting tax at the point of import.

As per section 8(2) of VAT Act, a person in Nepal receives services in Nepal from any person
not registered in Nepal and residing outside Nepal, in that case the person in Nepal is required
to assess tax on the value of the service and deposit the amount. The assessment of tax shall be
as per the Act.

b) “Situation out of control” as prescribed in Section 19(4) of VAT Act, 2052.

Section 19(4) of the Value Added Tax Act, 2052 prescribes the situation when the taxpayer
can apply to the Director General to waive the additional duty/tec which was applied due to
situation out of the control of the tax payer. In this situation the Director General can waive
the fine if he found the causes genuine.

The “situation out of control” as prescribed in this section is elaborated in the Rule 35, of
VAT Regulation, 2053. For the purpose of section 19(4) the following are the situation out of
control:
a. The tax payer recovered from the illness, then within 7 days from the date of recovery.
b. The tax payer has to do rituals due to the death of parents, then within 7 days from the
finishing date of ritual.
c. The female tax payer has given birth of a child, then within 35 days from the date of
maternity.
d. In case of death, mental disorder or missing, the legal heir has to apply within 35 days,
then within 7 days from the date of application.
e. In case of road blocked due to flood, landslide or snow and tax payer could not reach
to tax office, then within 7 days from the date of opening of the road.
f. The transportation has totally halted and the tax payer could not reach the tax office,
then in the very next day of opening.
The tax payer shall apply for the extension of date with the recommendation of
VDC/Municipality.

c) Temporary registration of VAT.

YSZ P.T.O.
(94)

Section 10Ka of the VAT Act, 2052 prescribes the special conditions of temporary
registration.
1. In case of exhibition or mela organized in temporary basis, the organizer of the mela
etc or any other person participating in the mela by hiring stall but who is not
registered with VAT dealing with taxable goods shall apply for the temporary
registration. The tax officer can ask for the deposit in this case.

2. The existing registered person can transfer its stock for the purpose of putting in the
exhibition.

3. The person registered temporarily, will have to file VAT return and deposit the tax
amount within 7 days of the closure of exhibition. He shall also get the temporary
registration cancelled.

d) Provisions relating to excess VAT credit under section 24 of VAT Act, 2052.

Section 24 of the VAT Act, 2052 prescribes the following provisions relating to excess VAT
credit:
1. A registered person can deduct the amount of VAT paid on input as per section 17 of
the Act, from the VAT collected on sales in that month, in case the VAT collected is
excess of the paid VAT.

2. VAT credit remaining in one month can be carried forward to next month and set off
with the amount of VAT payable in that month.

3. The tax payer can apply for the refund of VAT credit if there is excess amount of VAT
credit for continuous period of six months.

4. If the tax payer has exported 40% or more of his total sales of that month, then he can
apply for the refund of tax immediately in next month.

5. The tax officer shall refund the amount of tax as per the application of tax payer if he
found the amount is refundable to the tax payer. If the amount not refunded within the
prescribed time then the amount shall be refunded with interest.

6. After claiming the refund of tax, the tax payer cannot set off the amount with the
amount of VAT payable in the next month.

13.
a) You are a tax expert. Answer stating the provisions relating to “Time of
Supply” as per Value Added Tax (VAT) Act, 2052, what happens when: 10
i) A customer takes delivery of the goods from the business place
before the supplier issues invoice.

YSZ P.T.O.
(95)

ii) A customer paid NPR 10,000 to a supplier along with a list of certain
goods to be supplied to the customer. As per the list the total goods
are worth NPR 12,000
iii) A supplier at Kathmandu receives order for supply goods from a
customer from Pokhara. The supplier packs the goods as per the
order, asks a labor to deliver the goods to the transporter and the
transporter gives delivery of the goods to the party of Pokhara at
Pokhara. When the buyer receives the goods, the supplier issues the
invoices.

b) Sharma & co, a registered person dealing in second hand vehicles has
purchased a secondhand motorcycle for NPR 10,000 plus chargeable
VAT on it. The following expenses are incurred for repair of the
motorcycle to make it saleable;
Particulars Amount (NPR) VAT (NPR)
Spare Parts 4,000 520
Labor Charges 2,000 0

The person sells the motorcycle for NPR 20,000. Calculate the taxable
value of the motorcycle as per VAT Act, 2052. 5

c) Tax Accountant of Subhashree & Co. contends that the terms “Zero
VAT” and “No VAT” have the same meaning. As a Tax Expert, express
your opinion within the framework of VAT Act, 2052. 5
Answer No.6
a)
Section 6 of VAT Act, 2052 deals with time of supply.
For the purpose of assessment and collection of tax under this Act, the supply of any
goods or services shall be considered to have taken place at the earliest time of the
following times;
a. When an invoice is issued by a supplier
b. In the case of the supply of the goods, when the recipient removes or takes
possession of the goods from the supplier‟s transaction place
c. In the case of supply of services when the services are provided; and
d. When the supplier receives a consideration for the goods or services.

Accordingly;
i. As soon as the customer takes delivery of the goods from the business place of
the supplier, as per section 6 of the Act, the supply is completed. Thus, supply
without issuing invoice is infringement of provisions of VAT Act.

ii. The customer has given order for supply of goods with cash before delivery of
NPR 10,000. Such consideration shall be treated as payment of consideration
and thus, VAT to be collected. Had it be the advance only, VAT would not
require to collect.

iii. As per the usual practice the labor when picks up the goods from the business
place of the supplier, it is supposed that the buyer itself has taken the delivery
of the goods. The supply of the goods is completed as and when the labor has
removed the goods from the place of supplier.
YSZ P.T.O.
(96)

b)

Rule 33 of VAT, Rules, 2053 states that a person who is dealing in used or second
hand materials has to determine the taxable value as per this rule. The taxable value is
the selling value of the goods less purchase price including VAT.

Accordingly;
Selling price
NPR 20,000
Less: Purchase price and other expenses plus VAT
NPR 17,820
(NPR 10,000+1,300+4,520+2,000)
Taxable Value
NPR 2,180

So, in case of a registered person dealing in used or second hand materials is not
required to collect VAT on selling price but it has to collect VAT on the difference the
selling price and the expenses incurred for purchase and repair of it including VAT.

c)
All the transactions of goods and services are taxable, but transactions of those goods
and services which are included in Schedule 1 of the VAT Act, 2052 (called negative
list of goods and services) are exempted from tax. This is also called no VAT items.

Out of the goods or services, which are subject to VAT and are transacted as per
Schedule 2 of the Act, the rate of VAT shall be charged by zero percentage.

In both the conditions of no VAT and zero VAT, VAT is not collected. But, there are
fundamental differences in the concepts behind each of them. The concepts of no VAT
and Zero VAT are described hereunder:

No VAT
Those goods and services, which are necessary for the people and those that are
generally provided by the government, are included in the “No VAT Schedule”.
Exemptions primarily are proposed from three types of reasoning; they are: for basic
life necessities or services or on social welfare grounds and reducing complexities in
administration.

Where a supply of a commodity or services is exempt from VAT, the input tax is not
deductible or shall not be refunded.

Zero VAT
Schedule 2 of the Act has prescribed certain circumstances such as export etc, under
which the tax shall be charged at a zero rate. If a transaction of goods or service is
zero-rated the input tax or tax paid on purchases is creditable or refundable. This
means, the taxpayer not only does not pay tax on the value of his exports, it will be

YSZ P.T.O.
(97)

fully compensated for the tax it pays on inputs and, therefore, is truly exempt from
VAT.

Hence the contention of the Tax Accountant is incorrect.

YSZ P.T.O.
(98)

YSZ P.T.O.
CAP-II, Advanced Accounting, Dec 2013
Suggested Answer
Roll No……………. Maximum Marks - 100
Total No. of Questions - 6 Total No. of Printed Pages - 4
Time Allowed - 3 Hours
Marks
Attempt all questions. Working notes should form part of the answer.
1. Anil and Mohan who are equal partners carrying on manufacturing business as
manufacture. Their balance sheet as on 31st Ashadh 2069 was as follows:
Liabilities Rs. Assets Rs.
Sundry Creditors 55,500 Book Debts 60,000
Bank Overdraft 28,500 Stock in Trade 42,000
Bills Payable 10,500 Joint Life Policy- surrender value 7,500
Anil's Capital A/C 33,000 Office Furniture 1,500
Mohan's Capital 52,500 Machinery & Plant 18,000
Leasehold Property 22,500
Anil's Drawing 9,000
Mohan's Drawing 3,000
Profit & Loss A/C 16,500
Total 180,000 180,000
The business is carried on till 30th Poush 2069, by which time a net profit of
Rs. 12,300 has been made for the half year after depreciation @ 10% per annum has
been written off on Leasehold Property. Meanwhile, Sundry Creditors have been
reduced by Rs. 12,000, Bills Payable by Rs. 2,925 and Bank Overdraft by Rs. 3,000.
Partners' drawings for six months amounted to Rs. 3,000 each. Stock in trade stood
at Rs. 45,300 and book debts at Rs. 46,200 on 30th Poush 2069, other assets subject
to any necessary adjustment, stood at the figure as on 31st Ashadh 2069.
In Chaitra 2069 the firm agrees to sell the business to Govind Ltd. on the basis that
the stock shall be taken over at a discount of 5% and book debts at a discount of
2.5% as on 30th Poush 2069. The company pays Rs. 7,500 for profits in the interval
up to Chaitra 2069 subject to drawings of Rs. 2,000 by Anil and Rs. 1,000 by Mohan
during three months to Chaitra 2069; the company doesn't take over the Joint Life
Policy. The company takes over all other assets and liabilities paying Rs. 37,500 for
goodwill. The consideration in discharge by payment of Rs. 60,000 in cash and the
balance in preference shares of Rs. 10 each.
Prepare necessary accounts to close the books of the firm. 20
Answer
Realization Account
Dr. Cr.
Chaitra Rs. Chaitra Rs.
2069 To Sundry Assets 2069 By Sundry Creditors 43,500
Stock 45,300 By Bank Overdraft 25,500
Machinery 18,000 By Bills Payable 7,575
Furniture 1,500 By Govind Ltd.-
Book Debts 46,200 consideration 94,380
JLP 7,500 By Bank (Joint Life
Leasehold Pro 21,375 Policy realized) 7,500
Misc. Assets 4,500 144,375
To Profit transferred to-

ZYA P.T.O.
(2)

Anil's Cap A/C 17,040


Mohan's Cap A/C 17,040 34,080
178,455 178,455
Bank Account
Dr. Cr.
Chaitra Rs. Chaitra Rs.
2069 2069
To Realization Account- 25,500 By Balance b/f 25,500
Transfer By Anil's Capital A/c 24,970
To Realization Account- 7,500 By Mohan's Capital A/c 42,530
Joint Life Policy realized
To Govind Ltd- Cash 60,000
received 93,000 93,000

Capital Accounts
Dr. Cr.
Date Particulars Anil Mohan Date Particulars Anil Mohan
2069 2069
Srawan 1 To Drawings 9,000 3,000 sarawan 1 By Balance b/d 33,000 52,500
Srawan 1 To P/L Account 8,250 8,250 Poush 30 By Profit & loss App A/C 6,150 6,150
Poush 30 To Drawings 3,000 3,000 Chaitra 31 By Profit for three months
Chaitra 31 To Drawings 2,000 1,000 up Chaitra 31 3,750 3,750
Chaitra 31 To Balance c/d 37,690 64,190 By Realization A/C- Profit 17,040 17,040

Total 59,940 79,440 Total 59,940 79,440


Chaitra 31 To Preference Chaitra 31 By Balance b/d 37,690 64,190
Shares in Govind
Chaitra 31 Ltd. 12,720 21,660
To Bank 24,970 42,530

Total 37,690 64,190 Total 37,690 64,190

Working Note:-
Balance Sheet of the Firm as on 30th Poush 2069
Liabilities Rs. Assets Rs.
Sundry Creditors 43,500 Book Debts 46,200
Bank Overdraft 25,500 Stock in Trade 45,300
Bills Payable 7,575 JLP Surrender Value 7,500
Anil's Capital Office Furniture 1,500
Balance as on 1st srawan 2069 33,000 Machinery & Plant 18,000
Less: Drawings 12,000 Leasehold Property 22,500
Less: Loss up to 30th poush 2069 2,100 18,900 Less: 10% Dep for 6month 1,125
Mohan's Capital 21,375
Balance as on 1st srawan 2069 52,500
Less: Drawings 6,000
Less: Loss up to 30th poush 2069 2,100 44,400
Total 139,875 Total 139,875

Note: Loss up to 31st Ashadh 2069 was Rs. 16,500: after that there has been a net profit of Rs. 12,300 reducing the loss to
Rs. 4,200. This has been deducted from the partner's capital accounts in the profit sharing ratio.
Govind Ltd., will pay Rs. 94,380 calculated as under:
Assets taken over:
Book Debts 2.5% less 45,045
Stock 5% less 43,035
Office Furniture 1,500
Machinery & Plant 18,000

ZYA P.T.O.
(3)

Leasehold Property 21,375


Goodwill 37,500
Miscellaneous assets for profit up to 31st Chaitra 2069
Less: drawings (7,500- 2,000- 1,000) 4,500
Total 170,955
Less: Liabilities taken over
Sundry Creditors 43,500
Bank Overdraft 25,500
Bills Payable 7,575
Total 76,575
Net Assets to be paid for 94,380
Govind Ltd will pay Rs. 60,000 in cash and Rs. 34,380 in preference shares of Rs. 10 each, the number of shares being
3438.
The preference shares 3,438 in number have been divided between Anil and Mohan in the ratio of their final claims i.e.,
Anil Rs. 37,690 and Mohan Rs. 64,190.
Anil= (37,690 x 3,438)/101,880= 1,272= Rs. 12,720
Mohan= (64,190 x 3,438)/101,880= 2,166= Rs. 21,660
The balance due to them has been paid in cash
2.
a) Balance Sheet of Raj Motors is given below:
Liabilities Amount (in 000 Rs.) Assets Amount (in 000 Rs.)
31.03.2069 31.03.2070 31.03.2069 31.03.2070
Share Capital 500 500 Land & Building 300 300
9% Debentures 200 160 Machinery 164 180
Sundry Creditors 230 216 Stock in Trade 200 228
Profit & Loss A/c 40 54 Sundry Debtors 170 162
Depreciation Fund 80 88 Cash & Bank 120 110
Balances
Contingency Reserve 140 110 Current Investment 262 190
Outstanding Expenses 30 48 Prepaid Expenses 4 6
1,220 1,176 1,220 1,176

The following information is furnished:


i) An Old Machinery, which has original cost of Rs. 30,000, was sold for
Rs. 10,000. The accumulated depreciation in respect of the said machinery
amounts to Rs. 16,000.
ii) One New Machinery was acquired for Rs. 46,000.
iii) 9 % Debentures was redeemed at a discount of 4% of their face value (at
the beginning of the year).
iv) Dividend at 12% was declared and paid in cash.
v) Income Tax-Liability of Rs. 30,000 paid was debited to contingency
reserve.
You are required to prepare Cash Flow Statement on Indirect Method as per the
NAS. 10
b) On 1st Shrawan 2069; Mr. Akash purchased 5,000 equity shares of Rs. 100 each
in BB Bank Ltd. @ Rs. 120 each from a broker who charged 2% brokerage. On
31st Baisakh 2070; bonus shares were declared in the ratio 1:2. Before and after
the record of bonus shares, the shares were quoted Rs. 175 per share and Rs. 90
per share respectively. On 31st Ashadh 2070, Mr. Akash sold bonus shares to a
broker who charged 2% brokerage.
Show the investment account in the books of Mr. Akash, who held the shares as
current assets and closing value should be made as per NAS. 10
Answer
ZYA P.T.O.
(4)

a) Cash Flow Statement


A. Cash Flow from Operating Activities Rs Rs
Net Profit Before Tax (Rs 54,000 - Rs 40,000 + Rs 60,000 74,000
dividend paid)
Add: Adjustment for Depreciation (W.N.1) 24,000
Interest on debentures (9% X Rs 1,60,000) 14,400
Loss on Sale Machinery (W.N.2) 4,000
1,16,400
Less: Profit on redemption of Debentures -1,600
1,14,800
Less: Income Tax Paid -30,000
84,800
Add: Increase in Outstanding Expenses 18,000
Decrease in Sundry Debtors 8,000
Decrease in Current Investment 72,000 98,000
1,82,800
Less: Decrease in Sundry Creditors 14,000
Increase in Stock In Trade 28,000
Increase in Prepaid Expenses 2,000 -44,000
Net Cash from Operating Activities 1,38,800
B. Cash Flow from Investing Activities
Sale of old Machinery 10,000
Purchase of Machinery -46,000
Net Cash used in Investing Activities -36,000
C. Cash Flow from Financing Activities
Redemption of Debentures(Rs. 40000-Rs. 1,600) -38,400
Payment of Dividend -60,000
Payment of Interest on Debentures -14,400
Net Cash Used Financing Activities -1,12,800
Net Decrease in Cash & Cash Equivalent during the year -10,000
Cash & Cash Equivalent at the beginning of the year 1,20,000
Cash & Cash Equivalent at the end of the year 1,10,000
Depreciation Fund
Particulars Amount Particulars Amount
To Machinery A/c 16000 By Balance b/d 80000
To Balance c/d 88000 By Profit & Loss A/c 24000
104000 104000
Machinery A/c
Note:
Particulars Amount Particulars Amount
To balance B/d 1,64,000 By Depreciation Fund 16,000
To Bank 46,000 By Bank 10,000
By Profit & Loss ( Loss on Sales) 4,000
By Balance c/d 1,80,000
2,10,000 2,10,000

ZYA P.T.O.
(5)

It is assumed that current investments cannot be liquidated within short duration


of 3 months; therefore it has not been considered as part of cash & cash
Equivalents
b)
Investment in Equity shares of BB Bank Ltd
Account in the books of Mr Akash For the Year Ended 31st Ashadh 2070
Working Notes:
Date Particulars Nominal Cost Date Particulars Nominal Cost
1.4.2069 To Bank A/C 5,00,000 6,12,000 31.3.2070 By Bank 2,50,000 2,20,500
A/C
(W.N.1) (W.N.2)
31.3.2070 To Bonus Shares 2,50,000 - 31.3.2070 By Balance 5,00,000 4,08,000
C/d
31.3.2070 To Profit & Loss - 16,500
Account
(W.N.3)
7,50,000 6,28,500 7,50,000 6,28,500
1.Calculation of cost of equity shares purchased on 1.4.2069
5000 x Rs 120 + 2% of Rs 600,000 = 6120000

2. Calculation of profit proceeds of equity shares sold on 31.3.2070


2500 x Rs 90-2% of Rs 2,25,000 = Rs 2,20,500

3. Calculation of profit on sale of bonus shares on 31.3.2070


Sales Proceeds -Average Cost
220,500-204,000 i .e. (612000x250000/750000) = Rs 16,500

4. Valuation of equity shares on 31.3.2070


Cost = 612000 x 500000/750000 = Rs 408000
Market Value = Rs 5000 X Rs 90 = Rs 450000
Hence, Valuation is taken as Rs 408000

3.
a) On 1st Shrawan 2068, M/s Krishna Traders had a Bad Debts Provision of
Rs. 6,500. On 31st Ashadh 2069 the total debts amounted to Rs. 254,000 out of
which Rs. 4,000 were bad debts and had to be written off. The firm wants to
maintain the Bad Debts Provision at 5% of the debtors.
On 31st Ashadh 2070, total debts amounted to Rs. 103,200. Out of the debts, the
position of some of the parties was as follows:
Sundar Rs. 3,200 Bad to be written off.
Ramesh Rs. 8,000 Expected to realize only 70%
Basu Rs. 6,000 Expected to realize only 60%
Indra Rs. 4,000 Financial condition very poor and no recovery is likely.
A suitable provision is supposed to be maintained in respect of doubtful debts.
You are required to show the Bad debts Account and the Bad Debts Provision
Account for the years ended 31st Ashadh 2069 and 31st Ashadh 2070. (4+4=8)

ZYA P.T.O.
(6)

b) Great Bank Limited has the following status of Loans and Advances as on 31st
Ashadh 2070:
Category Amount (Rs.)
Good 3,309,000
Rescheduled /Restructured -
Substandard 292,000
Doubtful 665,000
Bad 99,000
You are required to calculate the loan loss provision as per the directive issued by
Nepal Rastra Bank and find the percentage of total Non-performing assets. 2
c) The final accounts of Jyoti Limited ended on 31st Ashadh 2069 showed sales at
Rs. 7,400,000 with a net profit of Rs. 612,000 after debiting standing charges of
Rs. 128,000. On 1st Shrawan 2069 the company took a loss of profit insurance
policy for Rs. 819,000 under which the period of indemnity was four months with
an average clause. On 1st Mangsir 2069 a fire broke out in the company.
Consequently, sales were affected during five months. The comparative sales
figures were as under:
Months Year 2068 (Rs.) Year 2069 (Rs.)
Mangsir 600,000 100,000
Poush 580,000 200,000
Magh 630,000 350,000
Falgun 610,000 470,000
Chaitra 650,000 590,000
The sales for twelve months ended on 30 Kartik 2069 were Rs. 7,800,000. On 1st
th

Shrawan 2069 the old plant was replaced and a modern plant was installed to
increase the profitability by 2% on the sales of the financial year 2069/70.
You are required to prepare a statement showing the calculation of insurance
claim. 7
d) From the Following Financial Statement, calculate the given ratios: 3
Summarised Profit & Loss Account
Particulars Amount Rs. (In '000)
Revenue 3,029
Gross Profit 179
Net Profit 45
Summarised Balance Sheets
Particulars Amount Rs. (In '000)
Fixed Assets at Book Value 257
Stock 236
Debtors 9
Other Assets 66
Total 568
Share Holders‟ Equity 320
Long Term Liabilities 64
Current Liabilities 184
Total 568
Ratios:

ZYA P.T.O.
(7)

i) Net Profit Percentage, ii) Total Asset Turnover, iii) Rate of Return on Gross
Assets, iv) Liquidity Ratio and v) Stock Turnover Ratio

Answer
a) Note: It can be seen that Rs. 3,200 is to be written off, the extent of the amount expected to be
unrealisable is Rs.8,800 as shown below:
Rs.
Ramesh 30% of Rs.8,000 2,400
Basu 40% of Rs.6,000 2,400
Indra 100% of Rs.4,000 4,000
Total 8,800
The provision for doubtful debts required to be maintained as on 31st Ashad 2070 is Rs. 8,800.
Bad Debts Account
Date Particulars Amount Date Particulars Amount
2069 2069
31/3 To Sundry Debtors (2) 4,000 31/3 By Provision for Bad debts (3) 4,000

2070 To Sundry Debtos (6) 3,200 2070 By Provision for Bad debts (7) 3,200
31/3 31/3

Provision for Bad Debts Account


Date Particulars Amount Date Particulars Amount
2069 2068
31/3 To Bad Debts A/c (3) 4,000 1/4 By Balance c/d (1) 6,500
To balance c/d 12,500 2069
31/3 By Profit and Loss A/c (4) 10,000
16,500 16,500
2070 2069
31/3 To Bad Debts A/c (7) 3,200 1/4 By balance c/d 12,500
To Profit and Loss A/c (8) 500
To balance c/d (9) 8,800
12,500 12,500
2070
1/4 By balance c/d 8,800

Note:
1. The numerical at the end of each entry denote successive steps.
2. The balance to be carried forward is 5% of debtors after writing off bad debts on 31 st Ashad
2069 i.e. 5% of Rs. 2,50,000. The amounts to be transferred to Profit and Loss Account is the
shortfall in Provision for Bad Debts Account after leaving the desired balance of Rs. 12,500. On
31st Ashad 2070 the required balance of Provision for Bad Debts Account is 8,800
b) Calculation of Loan Loss Provisioning and Non Performing Loans
Provision
Category Weight Amount Amount
1. Good 1% 3,309,000 33,090
2. Rescheduled /Restructured 12.50% - 0
3. Substandard 25% 292,000 73,000
4. Doubtful 50% 665,000 332,500
5. Bad 100% 99,000 99,000
TOTAL 4,365,000 537,590
ZYA P.T.O.
(8)

Non Performing Loan (NPL) 1,056,000


NPL/Total Loan 24.19
c) Statement Showing Calculation of Insurance Claim
Sales from 1 Mangsir 2068 to 30th falgun 2068
st
Rs.
(600,000+580,000+630,000+610,000) 2,420,000
st th
Less: Sales from 1 Mangsir 2069 to 30 falgun 2069
(100,000+200,000+350,000+470,000) 1,120,000
Short Sales 1,300,000
Gross Profit Ratio= Net profit+ Insured standing charges X 100
Annual sales
= 612,000+128,000 X 100
7,400,000
=10%
Add: Increase in profitability on sales = 2%
Effective Gross Profit Ratio = 12%
Loss of profit= Short sales x Effective Gross Profit Ratio
= 1,300,000 X 12/100= 156,000
Sales for twelve months preceding the date of fire = 7,800,000
Insurable amount of loss of profit= 7,800,000 X 12/100= 936,000
But the policy has been taken for NPR 819,000 only.
Hence, average clause is applicable:
Amount of claim =( Loss of profit X Sum insured) / Insurable value
= (156,000 X 819,000)/ 936,000 = 136,500.
d) Accounting Ratios:
i) Net Profit Percentage: Net Profit/Sales=45/3029=1.49%
ii) Total Asset Turnover: Sales/Total Assets=3029/568=5.33
iii) Rate of Return on Gross Assets: Net Profit/Total Assets=45/568=7.92%
iv) Liquidity Ratio: Current Assets /Current Liabilities=245/184=1.33:1
v) Stock Turnover Ratio: Cost of Sales/Average Inventory=2850/236=12.08
4.
a) From the following particulars, calculate: 7
i) Agreed value of two plants taken back by the hire vendor.
ii) Book value of plant left with the hire purchaser.
iii) Profit or loss to hire purchaser on two plants taken back by the hire vendor.
iv) Profit or loss on plants repossessed, when sold by the hire vendor.
Particulars:
 X purchased three plants from Y on hire purchase basis, the cash price of each
plant being Rs. 100,000.
 The hire purchaser charged depreciation @ 20% on diminishing balance
method.
 Two plants were seized by the hire vendor when second installment was not
paid at the end of the second year. The hire vendor valued the two plants at
cash price less 30% depreciation annually charged at diminishing balance
method.
 The hire vendor spent Rs. 40,000 on overhauling of the plants and then sold
them for a total sum of Rs. 160,000.
b) Swastik Nirman Ltd. started work on 1st Baishakh 2070 on a contract for
Rs. 500,000. On 31st Ashadh 2070, when he prepared his final accounts, the

ZYA P.T.O.
(9)

following information relating to the contract was extracted from his books of
account:
Particulars Rs.
Material issued from stores and sent to site 160,000
Wages paid 101,200
Wages outstanding on 31st Ashadh 2070 37,520
st
New machinery purchased and sent to site on 1 Baisakh 2070 148,000
Direct charges paid 7,500
st
Direct charges outstanding on 31 Ashadh 2070 600
Establishment charges apportioned to the contract 6,400
On 31st Ashadh 2070 materials lying unused at the site were valued at Rs. 21,620.
Machines were depreciated at 20% per annum. Value of work certified as on 31st
Ashadh 2070 was Rs. 350,000 while the cost of work done but not yet certified as
on that date was Rs. 18,000. On the basis of architect's certificate Swastik Nirman
Ltd. has received Rs. 280,000 from the contractee till 31st Ashadh 2070.
Prepare Contract Account in the ledger of Kali Prasad Singh. Also show the
relevant portion of the balance sheet in the books of Swastik Nirman Ltd. (4+4=8)
Answer
a)
i) Price of two plants = Rs. 100,000 x 2 200,000
Less: Depreciation for the first year @ 30% 60,000
140,000
Less: Depreciation for the second year Rs. 140,000 x 30/100 42,000
Agreed value of two plants taken by the hire vendor 98,000
ii) Cash purchase price of one plant 100,000
Less: Depreciation on Rs. 100,000 @ 20% for first year 20,000
Written down value of the plant at the end of first year 80,000
Less: Depreciation on Rs. 80,000 @ 20% for the second year 16,000
Book value of the plant left with the hire purchaser 64,000
iii) Book Value of one plant (ii) 64,000
Book Value of two plants = Rs. 64,000 x 2 128,000
Book Value at which two plants taken back (i) 98,000
Hence, loss on plants taken back = 128,000-98,000 = Rs. 30,000
iv) Sale proceeds of plants repossessed 160,000
Less: Value at which plants were taken back 98,000
Overhauling Charges 40,000 138,000
Profit on resale 22,000
b) In the books of Swastik Nirman Ltd.
Contract Account
Date Particulars Amount Date Particulars Amount
To materials sent to site 1,60,000 By Unused materials at site as on 21,620
To wages paid 1,01,200 31st Ashadhs 2070
Add: O/s 37,520 1,38,720 By Machines (WDV as on 31st 1,40,600
To machines (cost) 148,000 Ashad 2070)
To Direct Charges 7,500 By Work in Progress:
Add: O/s 600 8,100 Certified 3,50,000
To establishment charges 6,400 Uncertified 18,000 3,68,000
To Profit and Loss A/c
(transfer of profit) 36,800
To balance c/d 32,200

ZYA P.T.O.
(10)

5,30,220 5,30,220
To Unused materials at 21,620
site
To machines 1,40,600
To Work in Progress:
Certified 3,50,000
Uncertified 18,000
3,68,000
Less: balance b/d 32,200 3,35,800
Working Notes:
1. Calculation of WDV of the machine as pm 31st Ashad 2070:
Rs:
Cost of machines on 1st Baisakh 2070 1,48,000
Less: Depreciation on Rs. 148,000@20% for 3 months
=148,000X20%X3/12 7,400
1,40,600

2. Calculation of amount to be transferred from Contract Account to Profit and Loss A/c
Total surplus on contract as on 31st Ashad 2070
=21,620+1,40,600+3,68,000-1,60,000-1,38,720-148,000-8,100-6,400
=Rs.69,000
Thus Profit to be credited to Profit and Loss A/c
=69,000X2/3 X 280,000/3,50,000
=Rs.36,800
In the books of Swastik Nirman Ltd.
Balance Sheet (Extracts only)
Capital and Liabilities Amount Assets Amount
O/s Expenses Machines 1,48,000
Wages 37,520 Less: Depreciation 7,400 1,40,600
Direct Charges 600 38,120 Work in progress:
Capital Account 36,800 Certified 3,50,000
Profit on contract Uncertified 18,000
3,68,000
Less: Profit in reserve 32,200
3,35,800
Less: Amount received
From contractee 2,80,000 55,800
Unused Materials at Site 21,620

5.
a) Rajesh Suppliers purchased goods on credit from Prakash Hardwares for Rs. 1.5
crores for export. The export order was cancelled. Rajesh Suppliers decided to sell
the same goods in the local market with a price discount. Prakash Hardwares was
requested to offer a price discount of 15 %. The Chief Accountant of Prakash
Hardwares wants to adjust the sales figures to the extent of the discount requested
by Rajesh Suppliers. Discuss whether the treatment is justified. 5
b) A company with a turnover of Rs. 250 crores and an annual advertising budget of
Rs. 2 crore has taken up the marketing of a new product .It was estimated that the
company would have a turnover of Rs. 25 crores from the new product. The
company has debited to its Profit and Loss Account the total expenditure of Rs. 2
crore incurred on extensive special initial advertisement campaign for the new
product. Is the procedure adopted by the company correct? 5
ZYA P.T.O.
(11)

c) During the current year 2069/70, M/S Harish Power made the following
expenditure relating to its Plant & Machinery:
Particulars Amount (Rs.)
General Repairs 400,000
Repairing of Electric Motors 100,000
Partial Replacement of Parts of Machinery 50,000
Substantial improvements to the electrical wiring system
which will increase efficiency of the plant & machinery 1,000,000
Explain with reference to relevant NAS; how the above expenses should be
treated? 5
Answer
a) Prakash Hardwares has sold goods to Rajesh Suppliers on Credit worth Rs. 1.5 Crores and the
sale was completed in all respects. Rajesh Suppliers‟s decision to sell the same in the domestic
market at a discount does not affect the amount recorded as sales by the Prakash Hardwares. The
Price Discount of 15% offered by Prakash Hardwares after request of Rajesh Suppliers was not
in the nature of a discount given during the ordinary course of trade because otherwise the same
would have been given at the time of sale itself. Now, as far as Prakash Hardwares is concerned,
there appears to be an uncertainty relating to the collectability of the debt, which has arisen
subsequent to the time of sale therefore, it would be appropriate to make a separate provision to
reflect the uncertainty relating to collectability rather than to adjust the amount of revenue
originally recorded. Therefore, such discount should be written off to the Profit and Loss A/c and
not shown as deduction from the sales figure.
b) As per NAS 27, expenditure on an intangible item should be recognised as an expense
when it is in current unless it forms part of the cost of an intangible asset.
In the given case, advertisement expenditure of Rs 2 crores had been taken up for the marketing
of a new product which may provide future economic benefits to an enterprise by having a
turnover 25 crores. Here, no intangible asset or other asset is acquired or created that can be
recognised. Therefore, the accounting treatment by the company of debiting the entire
advertisement expenditure of Rs 2 crores to the profit and loss account of the year is correct.

c) As per NAS 6, Property, Plant and Equipment, expenditure that increases the future benefits
from the existing asset beyond its previously assessed standard of performance is included in
the gross book value e.g.an increase in capacity .Hence in the given case, repairs amounting Rs
5 Lakhs and partial replacement of parts machinery worth Rs 50,000 should be charged to Profit
& Loss Accounts. 10 Lakhs incurred for substantial improvement to the electrical wiring system
which will increase efficiency should be capitalized

6. Write short notes on: (4×2.5=10)


a) Money Measurement Concept
b) Accounting for Non Banking assets as per NRB Directives.
c) Deferred Tax Liability
d) Lease
Answer
a) Money Measurement Concept:The concept of Money measurement is an important aspect of
accounting. It is an important convention in accounting which explains that any transaction or event
that can be measured in terms of money can only be recorded in accounting. Accounting is disclosure
of all the business activities in an organized way that includes the figures. Anything that cannot be
measured in terms of money cannot be shown in accounts. E.g. even though human resources are an
important asset to any entity, these cannot be recorded in the books of accounts as they cannot be
reasonably measured in monetary terms.
ZYA P.T.O.
(12)

b) Accounting for Non Banking assets as per NRB Directives.


Banks and financial institutions are primarily engaged in lending activities. No bank or financial
institution lends without getting adequate level of collateral. All the debts of banks are securitized as
there is adequate level of collateral back up. In case where there are least possibilities of recovery of
loans and advances, recovery procedures are initiated and the assets taken as collaterals are taken over
by banks as Non Banking Assets. The Non banking assets are to be taken over lower of the following
values:
1. Fair Valuation of collateral at the time of taking over as non banking assets, or
2. Principal and interest outstanding on the day immediately proceeding the day of taking over as NBA.
In case if Fair Valuation of collateral is less than outstanding loans and advances on the day
immediately preceding the day of taking over as NBA, the balance amount should be written off to
profit and loss account.
c) Deferred Tax Liability:
According to NAS, A deferred tax liability shall be recognized for all taxable temporary differences,
unless the deferred tax liability arises from:
(a) the initial recognition of an asset or liability in a transaction which:
(i) is not a business combination; and
(ii) at the time of the transaction, affects neither accounting profit nor taxable profit
(tax loss).
It is inherent in the recognition of an asset that its carrying amount will be recovered in the form of
economic benefits that flow to the entity in future periods. When the carrying amount of the asset
exceeds its tax base, the amount of taxable economic benefits will exceed the amount that will be
allowed as a deduction for tax purposes. This difference is a taxable temporary difference and the
obligation to pay the resulting income taxes in future periods is a deferred tax liability. As the entity
recovers the carrying amount of the asset, the taxable temporary difference will reverse and the entity
will have taxable profit. This makes it probable that economic benefits will flow from the entity in the
form of tax payments.
d) Lease:
A lease is a contract calling for the lessee (user) to pay the lessor (owner) for use of an asset. A rental
agreement is a lease in which the asset is tangible property. Leases for intangible property could
include use of a computer program (similar to a license, but with different provisions), or use of a radio
frequency (such as a contract with a cell-phone provider). A written agreement under which a property
owner allows a tenant to use the property for a specified period of time and rent. The lease will either
provide specific provisions regarding the responsibilities and rights of the lessee and lessor, or there
will be automatic provisions as a result of local law. In general, by paying the negotiated fee to the
lessor, the lessee (also called a tenant) has possession and use (the rental) of the leased property to the
exclusion of the lessor and all others except with the invitation of the tenant

ZYA P.T.O.
CAP-II, Audit & Assurance, Dec 2013
Suggested Answer

Roll No……………. Maximum Marks - 100

Total No. of Questions- 7 Total No. of Printed Pages- 2

Time Allowed - 3 Hours


Marks

Attempt all the questions.

1. As an auditor, give your opinions with reasons on the following cases: (45=20)

a) M/s Merita Impex is engaged in the business of manufacturing and trading of


musical instruments. A sum of Rs. 5 lakhs, received from an insurance company
as an insurance claim for loss of goods in transit costing Rs. 4 lakhs, is credited to
the purchase account.
b) Alpha Limited purchased a high value plant from Beta Limited in exchange of
10,000 units of its finished products. The plant was in use in Beta Limited for last
3 years and expert expects that its useful life could be further 17 years (i.e. 20
years in total). Alpha limited sells its finished products in the market at Rs 1,000
per unit whereas there is no specific market for the used plant but the expert
valuation of the used plant indicates the value of the plant as Rs. 1.2 crores.
Hence, Alpha Limited has recorded the cost price of the plant at Rs. 1.2 crores.
c) Normal waste of material in production process is 2 % of input. 10,000 kg. of
input was made in process with resultant wastage of 500 kg. Cost per kg of input
is Rs. 100. The entire quantity of waste is in stock at the year end. As an auditor
how do you ensure the proper valuation of inventory?
d) Z Ltd. paid Rs. 100,000 income tax for disposal of capital assets during fiscal year
2069/070 and showed as deduction from cash flow from operating activities. Is
this treatment in compliance with NAS 03? Suggest correct treatment in cash flow
statement.

Answer:
a) All items of income and expense which are recognised in a period should be included in
the determination of net profit or loss for the period. The claim for loss of goods in transit
is arising out of ordinary activities of the impex as a part of its normal course of business.
However, the cost of goods lost in transit is only Rs.4,00,000 while the insurance money
received is Rs.5,00,000. Purchases Account need not be credited since it would distort the
purchases done during the year and as also the gross profit. Therefore, entire amount of 5
lakhs needs to be taken to profit and loss account under an appropriate head. This is an
income arising from an ordinary activity of the enterprise but having regard to amount
involved and exceptional nature, a separate disclosure be made in the profit and loss
account. Such disclosure would enable the users to understand the performance of an
enterprise for the period.

As per NAS 18 Revenue is the gross inflow of economic benefits during the period
arising in the course of the ordinary activities of an entity when those inflows result in
increases in equity, other than increases relating to contributions from equity participants.
In a given case it has been shown after deducting in the purchase which is not as per this
standard and it should be disclosed in gross as per this NAS.

IOJ P.T.O.
(2)
b) The fair value of the asset given up by Alpha Limited is Rs 10,000,000 (i.e. 10,000 units
* Rs 1,000). As per NAS 6 on property plant and equipment, if an entity is able to
determine reliably the fair value of either the asset received or the asset given up, then the
fair value of the asset given up is used to measure the cost of the asset received unless the
fair value of the asset received is more clearly evident. In the present case, the fair value
of the used plan is not clearly evident because there is no specific market for the used
plant.

Hence the plant should be recorded at the fair value of finished stock of Alpha Limited
given up to purchase the used plant; i.e. Rs 1 crore. So, the accounting treatment on
measurement of cost of used plant purchased from Beta Limited by Alpha Limited is not
in accordance with the requirement of NAS.

c) As per Para 16 of NAS-2, abnormal amounts of waste materials, labour or other


production costs, storage costs, unless those costs are necessary in the productin process
before a further production stage; administrative overheads that do not contribute to
bringing inventories to their present location and condition; and selling costs are excluded
from cost of inventories and such costs are recognized as expenses in the period in which
they are incurred.

In this case, normal waste is 200 kg and abnormal waste is 300 kg. The cost of 200 kg
will be included in determining the cost of inventories (Finished Products) at the year
end. The cost of abnormal waste amounting to Rs. 30,000 (300 kg *Rs. 100) will be
charged in the profit and loss statement.

d) As per Para 33 of NAS-3, cash flows arising from taxes on income shall be separately
disclosed and shall be classified as cash flows from operating activities unless they can be
specifically identified with financing and investing activities.

Further Para 34 added that taxes paid are usually classified as cash flows from operating
activities. However, when it is practicable to identify the tax cash flow with an individual
transaction that gives rise to cash flows that are classified as investing or financing
activities, the tax cash flow is classified as an investing or financing activity as
appropriate.

In view of the above provision in NAS -03, the treatment for income tax (capital gain tax)
on disposal of capital asset is not in compliance with NAS 03, accordingly it should be
shown under cash flow from investing activities.

2. Answer the following:


a) Explain different methods of obtaining audit evidence. 7
b) What is audit risk? Explain three components of audit risk and also define their
relationship. 8

Answer:
a) Different methods of obtaining audit evidence are described below:
i) Inspection of Records or Documents: Inspection consists of examining records or
documents, whether internal or external, in paper form, electronic form, or other media.
Inspection of records and documents provides audit evidence of varying degrees of
reliability, depending on their nature and source and, in the case of internal records and
documents, on the effectiveness of the controls over their production. An example of
inspection used as a test of controls is inspection of records or documents for evidence of
authorization.
ii) Inspection of Tangible Assets: Inspection of tangible assets consists of physical
examination of the assets. Inspection of tangible assets may provide reliable audit
evidence with respect to their existence, but not necessarily about the entity‟s rights and

IOJ
(3)
obligations or the valuation of the assets. Inspection of individual inventory items
ordinarily accompanies the observation of inventory counting.
iii) Observation: Observation consists of looking at a process or procedure being
performed by others. Examples include observation of the counting of inventories by the
entity‟s personnel and observation of the performance of control activities. Observation
provides audit evidence about the performance of a process or procedure, but is limited to
the point in time at which the observation takes place and by the fact that the act of being
observed may affect how the process or procedure is performed.
iv) Inquiry: Inquiry consists of seeking information of knowledgeable persons, both
financial and non-financial, throughout the entity or outside the entity. Inquiry is an audit
procedure that is used extensively throughout the audit and often is complementary to
performing other audit procedures. Inquiries may range from formal written inquiries to
informal oral inquiries. Evaluating responses to inquiries is an integral part of the inquiry
process.
v) Confirmation: Confirmation, which is a specific type of inquiry, is the process of
obtaining a representation of information or of an existing condition directly from a third
party. For example, the auditor may seek direct confirmation of receivables by
communication with debtors. Confirmations are frequently used in relation to account
balances and their components, but need not be restricted to these items. For example, the
auditor may request confirmation of the terms of agreements or transactions an entity has
with third parties; the confirmation request is designed to ask if any modifications have
been made to the agreement and, if so, what the relevant details are.
vi) Recalculation: Recalculation consists of checking the mathematical accuracy of
documents or records. Recalculation can be performed through the use of information
technology, for example, by obtaining an electronic file from the entity and using CAATs
to check the accuracy of the summarization of the file.
vii) Re-performance: Re-performance is the auditor‟s independent execution of
procedures or controls that were originally performed as part of the entity‟s internal
control, either manually or through the use of CAATs.
viii) Analytical Procedures: Analytical procedures consist of evaluations of financial
information made by a study of plausible relationships among both financial and non-
financial data. Analytical procedures also encompass the investigation of identified
fluctuations and relationships that are inconsistent with other relevant information or
deviate significantly from predicted amounts.
b) Audit Risk (AR): Audit risk is the risk that the auditor gives an inappropriate audit
opinion when the financial statements are materially misstated. In other words, it is a
function of the risk of material misstatement of the financial statements (or simply, the
“risk of material misstatement”) (i.e., the risk that the financial statements are materially
misstated prior to audit) and the risk that the auditor will not detect such misstatement
(“detection risk”).

Three components of audit Risk:


Inherent Risk (IR): “Inherent risk” is the susceptibility of an assertion to a misstatement
that could be material, either individually or when aggregated with other misstatements,
assuming that there are no related controls.
Control Risk (CR): “Control risk” is the risk that a misstatement that could occur in an
assertion and that could be material, either individually or when aggregated with other
misstatements, will not be prevented, or detected and corrected, on a timely basis by the
entity‟s internal control. That risk is a function of the effectiveness of the design and
operation of internal control in achieving the entity‟s objectives relevant to preparation of
the entity‟s financial statements. Some control risk will always exist because of the
inherent limitations of internal control.

IOJ
(4)
Detection Risk (DR): “Detection risk” is the risk that the auditor will not detect a
misstatement that exists in an assertion that could be material, either individually or when
aggregated with other misstatements. Detection risk is a function of the effectiveness of
an audit procedure and of its application by the auditor. Detection risk cannot be reduced
to zero because the auditor usually does not examine all of a class of transactions,
account balance, or disclosure and because of other factors. Such other factors include the
possibility that an auditor might select an inappropriate audit procedure, misapply an
appropriate audit procedure, or misinterpret the audit results. These other factors
ordinarily can be addressed through adequate planning, proper assignment of personnel to
the engagement team, the application of professional skepticism, and supervision and
review of the audit work performed.
Relationship of Audit Risk with its components:
The relationship is expressed by the following Equation:
AR=IR*CR*DR
From the definition of various components of audit risk stated above, it is clear that IR
and CR cannot be reduced by the auditor. In other words the auditor has control over DR
only. So, to keep the audit risk at acceptable level based on above equation, if the product
of IR and CR is high, the auditor should reduce DR.

3. Give your comments on the following: (35=15)


a) An audit firm was requested to perform the audit of a trading company for the
year 2068/69 where all records relating to sales transactions were seized by Inland
Revenue Department for investigation. The company requested the auditor to
accept the appointment saying that audit report with qualification on sales is
acceptable.
b) Cash book of an entity showed huge debit balances throughout the year.
c) The auditor of G-Mall had completed the audit of the financial statements for the
year 2069/70. The date of the audit report was 8 Kartik 2070 whereas financial
statements were approved by the management of the company on 10 Kartik 2070.
Answer:
a) In a given case the client is imposing the scope of audit. As per NSA 01, the term scope
of an audit refers to the audit procedures deemed necessary in the circumstances to achieve
the objective of the audit. The procedures required to conduct an audit in accordance with
NSA should be determined by the auditor having regard to the requirements of NSA,
relevant professional bodies, legislation, regulations and, where appropriate, the terms of
the audit engagement and reporting requirements.

An audit in accordance with NSA is designed to provide reasonable assurance that the
financial statements taken as a whole are free from material misstatement. Reasonable
assurance is a concept relating to the accumulation of the audit evidence necessary for the
auditor to conclude that there are no material misstatements in the financial statements
taken as a whole. Reasonable assurance relates to the whole audit process. 12. However,
there are inherent limitations in an audit that affect the auditor's ability to detect material
misstatements. These limitations result from factors such as:
a) The use of testing
b) The inherent limitations of any accounting and internal control system (e.g. the
possibility of collusion)
c) The fact that most audit evidence is persuasive rather than conclusive

Also the work undertaken by the auditor to form an opinion is permeated by judgment, in
particular regarding the gathering of audit evidence, e.g. in deciding the nature, timing and
extent of audit procedures; and the drawing of conclusion based on audit evidence
gathered, e.g. assessing the reasonableness of the estimates made by management in
preparing the financial statements.
IOJ
(5)

Other limitations may affect the persuasiveness of evidence available to draw conclusions on
particular financial statement assertions (e.g. transactions between related parties). In these
cases certain NSA(s) identify specified procedures which will, because of the nature of the
particular assertions provide sufficient appropriate audit evidence in the absence of unusual
circumstances which increase the risk of material misstatement beyond that which would
ordinarily be expected; or any indication that a material misstatement has occurred.

In the given situation the client is imposing a limitation on the scope of audit. Since all
the records relating to sales are not available for audit, the auditor is not in the position to
perform alternate audit procedures for audit of sales. Further, sales is the most important
item of profit or loss account (being a trading company) and hence without auditing sales
an opinion on profit or loss account cannot be provided. That is the auditor has no option
but to provide disclaimer of opinion. Since this situation is known to auditor before
acceptance of engagement, it is advisable not to accept such limited scope engagement
unless required by statute.

b) Cash balance is maintained to meet the day to day operational needs of the
organization. So, the auditor has to perform audit procedures particularly having
regard to the fact that maintaining such huge balance is highly prone to
misappropriation and other forms of fraud. Accordingly since the entity is
maintaining consistently huge cash balance which is not required for its
operational needs, the auditor should carry out the surprise verification of cash
more frequently to ascertain whether the actual cash in hand agrees with the
balance as per cash book. If the actual cash balance is not in agreement with the
book balance, he should seek explanations from senior official of the entity. In
case any material difference is not satisfactorily explained, the auditor should
state this fact appropriately in his audit report. In any case, he should satisfy
himself regarding the necessity for such huge cash balance having regard to the
normal working requirements of the entity. The entity may also be advised to
deposit the whole or major part of the cash balance in the bank at reasonable
interval.

Material cash on hand if found during the audit in relation to the financial
statements taken as a whole, or if there are significant negotiable securities in the
custody of the client, consider performing the following additional procedures:
- Count the cash fund or observe and list the securities in the presence of a client
representative.
- Tie amount counted to general ledger balances.
- In most of small businesses, cash on hand is immaterial and should not be
counted, unless it is at the clients' request.

c) NSA 8 Para 25 has defined the date of report. As per this para t he auditor should date the
report as of the completion date of the audit. This informs the reader that the auditor has
considered the effect on the financial statements and on the report of events and transactions
of which the auditor became aware and that occurred up to that date. Since the auditor's
responsibility is to report on the financial statements as prepared and presented by
management, the auditor should not date the report earlier than the date on which the
financial statements are signed or approved by management.

The auditor should date the report on the financial statements no earlier than the date on
which the auditor has obtained sufficient appropriate audit evidence on which to base the
opinion on the financial statements. Sufficient appropriate audit evidence should include
evidence that the entity‟s complete set of financial statements has been prepared and that
those with the recognized authority have asserted that they have taken responsibility for
them. Since the management has approved the complete set of the financial statements on
10 Kartick 2070, it cannot be considered that the management has taken the responsibility
IOJ
(6)
of the financial statements before 10 Kartick 2070. So, the auditor should not have dated
his report before 10 kartick 2070.

4. Answer the following: (35=15)


a) What are the elements that an auditor has to be considered while evaluating the
design of the entity‟s control environment?
b) State the reporting responsibilities of an auditor in the context of non-compliance
of laws and regulations in an audit of financial statements.
c) Mention briefly the conditions or events, which increase the risk of fraud or error
leading to material misstatement in financial statements.

Answer:
a) As per NSA 315 Section 67 the auditor should obtain an understanding of the control
environment. The control environment includes the governance and management
functions and the attitudes, awareness, and actions of those charged with governance and
management concerning the entity‟s internal control and its importance in the entity. The
control environment sets the tone of an organization, influencing the control
consciousness of its people. It is the foundation for effective internal control, providing
discipline and structure.

Section 68 has defined the primary responsibility for the prevention and detection of
fraud and error rests with both those charged with governance and the management of an
entity. In evaluating the design of the control environment and determining whether it has
been implemented, the auditor understands how management, with the oversight of those
charged with governance, has created and maintained a culture of honesty and ethical
behavior, and established appropriate controls to prevent and detect fraud and error
within the entity.

In evaluating the design of the entity‟s control environment, the auditor considers the
following elements and how they have been incorporated into the entity‟s processes:

i. Communication and enforcement of integrity and ethical values – essential elements


which influence the effectiveness of the design, administration and monitoring of
controls.
ii. Commitment to competence – management‟s consideration of the competence levels
for particular jobs and how those levels translate into requisite skills and knowledge.
iii. Participation by those charged with governance – independence from management,
their experience and stature, the extent of their involvement and scrutiny of
activities, the information they receive, the degree to which difficult questions are
raised and pursued with management and their interaction with internal and external
auditors.
iv. Management‟s philosophy and operating style – management‟s approach to taking
and managing business risks, and management‟s attitudes and actions toward
financial reporting, information processing and accounting functions and personnel.
v. Organizational structure – the framework within which an entity‟s activities for
achieving its objectives are planned, executed, controlled and reviewed.
vi. Assignment of authority and responsibility – how authority and responsibility
for operating activities are assigned and how reporting relationships and
authorization hierarchies are established.
vii. Human resource policies and practices – recruitment, orientation, training,
evaluating, counseling, promoting, compensating and remedial actions.

b) NSA 15 Para 14 has defined the reporting responsibilities of an auditor in the context of
non-compliance of laws and regulations in an audit of financial statements.
IOJ
(7)
The auditor should as soon as practicable, either communicate with the audit committee,
the Board of Directors and senior management or obtain evidence that they are
appropriately informed regarding non-compliance that comes to the auditors attention.

If in the auditor‟s Judgment, the non compliance is believed to be intentional and/ or


material, the auditor should communicate the findings without delay.

If the auditor suspects that members of senior management, including members of the
Board of Directors, are involved in non-compliance, the auditor should communicate the
matter to the next higher level of authority at the entity, such as, the audit committee or
Board of Directors, to the users of the auditors‟ report or financial statements.

If the auditor concludes that the non-compliance has a material effect on the financial
statements and has not been properly reflected in the financial statements the auditor
should express a qualified or an adverse opinion.

If the auditor is precluded by the entity from obtaining sufficient and appropriate audit
evidence to evaluate whether non-compliance is, or is likely to have occurred that have or
may have material impact on the financial statements, the auditor should express a
qualified opinion or a disclaimer of opinion on the financial statements on the basis of a
limitation on the scope of the audit.

If the auditor is unable to determine whether non compliance has occurred because of
limitations imposed by the circumstances rather then by the entity, the auditor should
consider the effect on the auditor‟s report.

The auditor‟s duty of confidentiality would ordinarily preclude reporting non compliance
to a third party. However, in certain circumstances, that duty of confidentiality is
overridden by statement, law or by courts of laws.

c) NSA 5 has defined the risk of fraud and error in the entity. In planning and performing
his examination, the auditor should take into consideration the risk of material
misstatements of the financial information caused by fraud or error. Weaknesses in the
design of the internal control system and non-compliance with identified control
procedures amongst other conditions or events which increase the risk of fraud or errors
are:
i. Weaknesses in the design of the internal control system and non-compliance with
the laid down control procedures.
ii. Doubts about the integrity or competence of the management.
iii. Unusual pressures within the entity
iv. Unusual transactions
v. Problem in obtaining sufficient and appropriate audit evidence.

5. Comment on the following situations/statements: (35=15)


a) Ram, a Chartered Accountant prepared a project report for one of his clients to
obtain bank finance (long-term) of Rs. 50 lakhs from a commercial bank.
Consequent to the sanction of the loan by the bank, Ram charged 2% fee on the
figures of loan sanctioned.
b) Goma Limited, a leading trading business in electronic goods, has appointed CA.
Ranaji Ojha a statutory auditor with Rs. 1.5 lakhs audit fees. Besides, the
company has offered 5 latest model iphones costing of Rs. 1.5 lakhs to his family
members as gift.
c) While submitting the financial proposal on consultancy works of Emiliya Gypsum
Limited, PZT Associates, a CA. firm has calculated the cost as follows:

IOJ
(8)
FCA Rs. 5,000 Per day/person
ACA Rs. 2,500 Per day/person
RA Rs. 2,000 Per day/person
Assistants Rs. 1,000 Per day/person

Answer:
a) Section 10 of Code of Conduct has defined the way how the practicing professional
accountants charge fee on their works.
10.2 Professional fees should be a fair reflection of the value of the professional services
performed for the client, taking into account:
(a) The skill and knowledge required for the type of professional services involved.
(b) The level of training and experience of the persons necessarily engaged in performing
the professional services.
(c) The time necessarily occupied by each person engaged in performing the professional
services.
(d) The degree of responsibility that performing those services entails.
10.3 Professional fees should normally be computed on the basis of appropriate rates per
hour or per day for the time of each person engaged in performing professional services.
These rates should be based on the fundamental premise that the organization and
conduct of the professional accountant in public practice and the services provided to
clients are well planned, controlled and managed. They should take into account the
factors set out in paragraph 10.2 and are influenced by the legal, social and economic
conditions of each country. It is for each professional accountant in public practice to
determine the appropriate rates.
10.4 A professional accountant in public practice should not make a representation that
specific professional services in current or future periods will be performed for either a
stated fee, estimated fee, or fee range if it is likely at the time of the representation that
such fees will be substantially increased and the prospective client is not advised of that
likelihood.
10.5 When performing professional services for a client it may be necessary or expedient
to charge a pre-arranged fee, in which event the professional accountant in public practice
should estimate a fee taking into account the matters referred to in paragraphs 10.2
through 10.4.
Section 34(10) of Nepal Chartered Accountants Act, 1997 states that “Members holding
Certificate of Practice shall not base their remuneration as a percentage on the profit or on
any other uncertain results”.

Entering into a contingent fee arrangement relating to an assurance engagement is an


example of self interest threat. In the present case, audit fee is contingent upon the profit
after tax of the client. So, this is the situation of self interest threat to the auditor and
hence it is advisable for the auditor not to accept the engagement under such fee
arrangement.

b) A professional accountant should not accept any gifts or goods from the client which
threat the independence. Code of Conduct Section 8.6 and 8.9 attracts to the professional
accountant to accept the gift and goods. Section 8.6 says personal and family
relationships can affect independence. There is a particular need to ensure that an

IOJ
(9)
independent approach to any assignment is not endangered as a consequence of any
personal or family relationship.
It is recognized that it would be impracticable to attempt to prescribe in detail in ethical
requirements the permissible extent of a personal relationship between a professional
accountant in public practice and a client, or those occupying responsible executive
positions (e.g., director, chief executive, financial officer or another employee in a similar
position) with a client.
Section 8.10 of the Code of Conduct has defined not to accept any goods and services
from the client. Acceptance of goods and services from a client may be a threat to
independence. Acceptance of undue hospitality poses a similar threat. Goods and services
should not be accepted by professional accountants in public practice, their spouses or
dependent children except on business terms no more favorable than those generally
available to others. Hospitality and gifts on a scale which is not commensurate with the
normal courtesies of social life should not be accepted.
In a given case offering of 5 mobile sets of significant value to the auditor by Goma
Limited seems to have the intent of influencing the decision making of the auditor. Hence
this is the situation of self interest threat, familiarity threat and intimidation threat. In this
situation the auditor should evaluate the significance of the threats and apply safeguards
to eliminate threats or reduce them to an acceptable level. If the threat cannot be
eliminated or reduced to an acceptable level, the gifts shall not be accepted by the
auditor.
c) Nepal Chartered Accountant Act, 2053, Nepal Chartered Accountants Rules, 2061 and
the Code of Conduct of the Institute have given authority to make certain standards on
the charging of fees by its members. Applying the power as provided in the above
statutes, the Council has decided and fixed certain fundamental principles on minimum
fees, consultancy services fee and fee on certification work. As per the clause 2 of the
decision, the Council has fixed the minimum consultancy charges as follows:
FCA member Rs. 6000/- per working days
CA member Rs. 4000/- per working days
RA member Rs. 2000/- per working days
Other/assistants Rs. 1500/- per working days

In a given case, the fee as quoted by the CA firm is not complying with the decision as
made by the Council except RA member charge which is as per the decision of the
Council.

6. Write short notes on the following: (42.5=10)


a) Guiding factors of materiality concept
b) Adverse opinion
c) Going concern
d) A qualifying insurance policy
Answer:
a) Guiding Factors of Materiality Concept
NSA 320 “Audit Materiality”, establishes standards on the concept of materiality and the
relationship with audit risk while conducting an audit. Guiding factors for determining
the materiality are:
i. Item of materiality may be determined individually or in aggregate.
ii. The materiality depends on the regulatory or legal considerations.

IOJ
(10)
iii. Materiality is not often reckoned with respect to quantitative details above. It has
qualitative dimensions as well.
b) Adverse Opinion
An adverse opinion should be expressed by the auditor when the effect of a disagreement
is so material and pervasive to the financial statements that the auditor concludes that a
qualification of the report is not adequate to disclose the misleading or incomplete nature
of the financial statements. Whenever the auditor expresses an opinion that is other than
unqualified, a clear description of all the substantive reasons should be included in the
report and, unless impracticable, a quantification of the possible effect(s) on the financial
statements. Ordinarily, this information would be set out in a separate paragraph
preceding the opinion or disclaimer of opinion on the financial statements and may
include a reference to a more extensive discussion, if any, in a note to the financial
statements.

c) Going Concern
The going concern assumption is a fundamental principle in the preparation of financial
statements. Under the going concern assumption, an entity is ordinarily viewed as
continuing in business for the foreseeable future with neither the intention nor the
necessity of liquidation, ceasing trading or seeking protection from creditors pursuant to
laws or regulations. Accordingly, assets and liabilities are recorded on the basis that the
entity will be able to realise its assets and discharge its liabilities in the normal course of
business.
d) A qualifying insurance policy is an insurance policy issued by an insurer that is not a
related party (as defined in NAS 16 Related Party Disclosures) of the reporting entity, if
the proceeds of the policy:
a. can be used only to pay or fund employee benefits under a defined benefit plan; and
b. are not available to the reporting entity‟s own creditors (even in bankruptcy) and
cannot be paid to the reporting entity, unless either:
i. the proceeds represent surplus assets that are not needed for the policy to meet
all the related employee benefit obligations; or
ii. the proceeds are returned to the reporting entity to reimburse it for employee
benefits already paid.

7. Distinguish between: (25=10)


a) Audit plan and audit program
b) Audit report and certificate
Answer:
a) Audit Plan and Audit Program
The Distinction between Audit Plan and Audit Program are outlined as follows:
Audit plan is the developing a general strategy and a detailed approach for the expected
nature, timing and extent of the audit. The auditor plans to perform the audit in efficient
and timely manner where as audit program is the step and guidance and works as a tool
for performing/implementing audit at the execution level. The distinctions of those two
are:
i. Audit plan is prepared before preparing audit program.
ii. Audit plan is border scope than audit program.
iii. Audit plan assists acquiring knowledge of clients business and concentrating on
risk areas which will help for preparing effective audit program.
iv. Audit plan generally prepares by senior auditors and program may be prepared by
juniors based on plan and duly approved by seniors.

IOJ
(11)
b) Audit Report and Certificate
i. The term „report‟ is used where an expression of opinion is involved. The term
„certificate‟ is preferable where the auditor comments on or verifies facts
ii. The Auditor Report is based on facts, estimates and assumptions whereas Auditor's
Certificate is based on actual facts
iii. Auditor Report is not a guarantee of the absolute correctness & accuracy of
the books of accounts. But the auditor certificate serves as a guarantee of the
absolute correctness & accuracy of the books of accounts
iv. If the Auditor Report is later on found to be wrong, he cannot be held responsible
since he has given merely his opinion on the state of affairs of the company. But if
the duly signed certificate is found as wrong, he will be held responsible

IOJ
CAP-II, Corporate & Other laws, Dec 2013
Suggested Answer

Roll No……………. Maximum Marks - 100

Total No. of Questions - 7

Time Allowed - 3 Hours


Marks
Attempt all questions.

Part: "A"
1.
a) Star Company Limited decides to buy-back its own shares. Advise the company's
Board of Directors about the sources out of which the company can buy-back its
own shares. What conditions are attached to the buy-back scheme of the company
in accordance with the provisions of the Companies Act, 1956? Explain. 10
b) There is a provision in the articles of a private company that the directors may at
any time in their absolute and uncontrolled discretion refuse to register any
transfer of shares. The issued capital of the company consisted of 100,000 shares
of which the two directors of the company Raghubeer and Manjit held 50,000
each. When Raghubeer died, his son applied to have the shares registered in his
name. But Manjit, in exercise of the power stated in the articles, refused to
transfer and offered to accept the applicant up to 30,000 shares, provided the
remaining were sold to him at a fixed price. An action has been brought against
the refusal to register the transfer. Explain whether the refusal to transfer of shares
is valid. 5
c) Explain the principle of "lifting the corporate veil". 5
d) What is the meaning of 'maturity of negotiable instrument'? When such
instruments become mature under the Negotiable Instrument Act, 1881? (2+3=5)
Answer
a)
Buy-back of shares, sources and conditions (Section. 77A of the Companies Act,
1956):
In accordance with the provisions of the Companies Act, 1956, Section 77A, a company
desirous of buy back of its own shares, can be advised to buy-back out of the following
sources:
1. Company's free reserves; or
2. Company's securities premium Account; or
3. Out of the proceeds of any shared or other specified securities. [Section.77A(1)].
However, no buy-back of any kind of shares or other specified securities shall be
made out of the proceeds of an earlier issue of the same kind of shares or same
kind of other specified securities.

Conditions
(1) Buy-back is authorised by the Company’s Articles.

KCP
P.T.O.
(13)

(2) A special resolution has been passed in general meeting of the


company authorising buy-back.
(3) The buy-back is less than 25% of the total paid-up capital and free
reserves of the company. The buy-back, however, cannot exceed
25% of company’s paid up equity share capital in a financial year.
Further the Companies (Amendment) Act 2001 (w.e.f. 23-10-2001)
has authorized the buy back by passing a resolution of a meeting
of the Board of Directors provided the buy back does not exceed
10% of the total paid up equity share capital and free reserves.
However, there cannot be more than one such buy-back in any
period of 365 days
(4) The ratio of debt owed by the company is not more than two times
the equity capital and free reserve after such buy back. However,
the Central Government may prescribe a higher ratio of the debt
for a class or classes of companies.
(5) All shares or other specified securities are fully-paid-up,
(6) The buy back with respect to listed securities is in accordance with
the regulations made by the SEBI in this behalf.

b)
In Smith and Fawcett Ltd, Re: [1942] Ch 304, it was held that the
court would not be justified in interfering in the discretion of the
directors. The directors must exercise their discretion bona fide in
what they consider- not what a court may consider – is in the
interest of the company. And if they have done that, the court
cannot substitute its judgment for theirs.

The principle laid down in this case were cited with approval by the Nagpur High
Court in Balwant Transport Co v Deshpande, AIR 1956 Nag 20 where the court
refused to interfere with the director’s discretion. It follows that the court will
interfere to protect the minority only where the majority shareholders of a
company propose to benefit themselves at the expense of the minority.

In this case, the refusal is treated as valid because the court, in general, will not
intervene in the matters of internal administration of a company. Further, by
such refusal, there is no adverse effect to the interest of Raghubir's son in the
company as he is getting reasonable price of his shares not registered in his
name. The court, in small number of cases, can intervene in the administration
of the affairs of a company to protect the minority shareholders from some
improper conducts in the company.

c)
A company is a legal person distinct from its members and as a
result members or shareholders cannot be liable for the acts of the
company. This principle has been accepted in famous case
Salomon vs Salomon and Co. (1897). However, when promoters in
their ingenuity, use this veil of corporate personality as a cloak for

KCP
(14)

fraud or improper conduct, it becomes necessary for courts to


break through or lift the corporate veil and look at the persons
behind the company who are the real beneficiaries of the corporate
function. In other words 'lifting the corporate veil' means
disregarding the corporate entity and paying regard, instead, to the
realities behind the legal façade. Where the courts ignore the
company and concern themselves directly with the members or
managers, the corporate veil may be said to have been lifted.

The court may lift the veil. The court has lifted veil in case of
determination of enemy character of the company, for protection of
revenue, determination of technical competition of the company
etc. There are various provision under Indian Companies Act to lift
the corporate under which directors can be held personally liable.

d)
Maturity means the date on which payment of negotiable
instrument falls due. A cheque becomes mature immediately after
its execution. However, section 22 of Negotiable Instrument Act,
1881 states that every promissory note or bill of exchange
expressed to be payable on a specified day, or at a certain period
after date or after sight or at a certain period after happening of an
event which is certain to happen is at maturity on the third day
after the day on which it is expressed to be payable.

Other Provisions under the Act


Sec 23. Calculating maturity of bill or notes payable so many months after date or
sight-
In calculating the date at which a promissory note or a bill of exchange, made payable at
stated number of months after date or after sight, or after a certain event ,is at maturity ,
the period stated shall be held to terminate on the day of the months which corresponds
with the day on which the instrument is dated, or protested for non-acceptance ,or the
event happens ,or, where the instrument is a bill of exchange made payable as stated
number of months after sight and has been accepted for honour, with the day on which it
was so accepted, if the month in which, the period would terminate has no corresponding
day, the period shall be held to terminate on the last day of such month.

Illustrations
(A) A negotiable instrument dated 29thJanuary, 1878, is made payable at one month after
date. The instrument is at maturity on the third after the 28th February, 1878.

(B) A negotiable instrument, dated 30th august, 1878, is made payable three months after
date. the instrument is maturity on the 3rd December, 1878
(C) A promissory note or bill of exchange of exchange, dated 31st august, 1878, is made
payable three months after the date. The instrument is at maturity on the 3rd December,
1878.

Sec 24.calcuiating maturity of bill or note payable so many day after date or sight-

KCP
(15)

In calculating the date at which a promissory note or bill of exchange made payable a
certain number of days after date or presentment for acceptance or sight , or of protest
for non –acceptance, or on which the event happens , shall be excluded.

Sec 25. When day of maturity is a holiday –


When the day on which a promissory note or bill of exchange is at maturity is a public
holiday, the instrument shall be denoted to be due on the next preceding business day.

Explanation- the expression "public holiday" includes Sunday and any


Other day declared by the central Government, by notification in the official Gazette, to
be a public holiday.

Part: "B"

2.
a) What is the procedure of making decision on the agendas at a general meeting and
mention the matters requiring special resolution at the general meeting of a
company as per the Companies Act, 2063. 5
b) Auditor of company ABC found out some irregularities during the course of audit
and would like that the shareholders know about these irregularities, therefore, he
wants to convene Extra Ordinary General Meeting (EGM). He doesn't know how
to convene EGM and approaches you. Explain how EGM can be convened on the
basis of Companies Act, 2063? 5
c) Mr. D started self service system in his shop. Mr. A entered the shop, took basket
and after taking articles of his choice into the basket reached to the cashier for the
payment. The cashier refused to take the money. Can Mr. D be compelled by Mr.
A to sell the said articles? Explain. 5
Answer
a)
As per the provision of the section 77 of the Companies Act 2063 the procedures on
deciding a resolution at a general meeting are as follows:
(1) The directors shall present the annual financial statements as audited,
auditor’sreport and director’s report at the annual general meeting of a public
company.

(2) If the shareholder or shareholders representing at least five per cent of the total
number of votes shall so desire, he/they may, by submitting anapplication to
the directors prior the issue of a notice under Sub-section (2) of Section 67,
cause any matter to be presented at the annual general meeting for discussion
and decision.

(3) At least twenty one days prior to the holding of the annual general meeting,
every public company shall make arrangement so that the shareholders can
inspect and obtain copies of the annual financial statement, directors’ report
and auditors’ report as referred to in Section 84 and publish a notice in a
national daily newspapers for information thereof.

KCP
(16)

(4) Information of the statements and reports as referred to in Subsection (3) may
also be disseminated through electronic communication media as per
necessity.
(5) If any shareholder makes a request for a copy of the annual financial statement,
directors’ report and auditor’s report as referred to in Sub-section (3),the
company shall provide a copy of such reports or statements to such
shareholder.
(6) Except as otherwise provided in this Act, matters of distribution of dividends to
shareholders, appointment of directors and their remuneration, appointment of
auditor and his/her remuneration of such other items as required by this Act or
the articles of association to be decided by the annual general meeting of the
company can be presented at the decided by the annual general meeting of the
company can be presented at and decided by that meeting.

Provided, however, that on the rate of dividends to be distributed to the shareholders


shall be made in a manner to exceed the rate of such dividendsfixed by the board of
directors.

As per the provision of Section 83 of the companies Act 2063 the Special resolutions
must be presented in the general meeting of a company for decision on the following
matters;
(a) Increasing the authorized capital of the company,
(b) Decreasing or altering the share capital of the company,
(c) Altering the name or main objectives of the company,
(d) Amalgamating one company into another company,
(e) Issuing bonus share,
(f) Buying back of own shares by the company,
(g) Selling shares at a discount,
(h) Converting a private company into a public company or vice versa,
(i) Such other matter in respect of which the company is required by this
Act or the articles of association to adopt a special resolution.

b)
Section 82 of the Companies Act, 2063 has included provisions related to Extra
Ordinary General Meeting (EGM), as follows:
Extra-ordinary general meeting:
(1) The board of directors of a company may convene an extra-ordinary general meeting
if it deems necessary.
(2) If, in the course of examining the account of a company, it is deemed necessary to
call an extra ordinary general meeting for any reason, the auditor may request the
board of directors to call such meeting; and if the board of directors fails to call the
meeting accordingly, the auditor may make an application, setting out the matter, to
the Office of the Company Registrar; and if an application is so made, the Office may
call the extra-ordinary general meeting of the company.
(3) If the shareholders holding at least ten percent shares of the paid–up capital of a
company or at least twenty five per cent shareholders of the total number of
shareholders make an application, setting out the reasons therefore, to the registered
office of the company for calling an extra-ordinary general meeting of the company.
(4) If the board of directors does not call the extra-ordinary general meeting within
thirty days from the date on which an application is made pursuant to Sub-section (3),
the concerned shareholders may make a petition to the Office setting out the matter;
and if such petition is made, the Office may cause to call such meeting.
(5) If the Office deems necessary to call an extra-ordinary general meeting in view of the
findings of any inspection or investigation or for any others reason, it itself call or cause
there board of directors to call such meeting.

KCP
(17)

On the basis of above provision , the auditor can not himself convene EGM. But the
auditor may request the board of directors to call such meeting; and if the board of
directors fails to call the meeting accordingly, the auditor may make an application, to
the Office of the Company Registrar; and if an application is so made, the Office may
call the extra-ordinary general meeting of the company.

c)
This problem is related to "Invitation to Offer". This is the issue related to differences
between the "Offer" and "Invitation to Offer".An offer is the final expression of
willingness by the offeror to be bound by his offer and the other party chooses to accept
it, but where a party, without expressing his final willingness, proposes certain terms
on which he is willing to negotiate, he does not make an offer, but invites only the other
party to make an offer on those terms. This is the basic distinction between offer and
invitation to offer.

Thus, on the basis of above explanation, the display of articles with a price in it in a self
serice shop is merely an invitation to offer. It is in no sense an offer for sale, the
acceptance of which constitutes a contract. In this case, Mr. A by selecting some
articles and approaching the cashier for payment simply made an offer to buy the
articles selected by him. If the cashier does not accept the price, the interested buyer
cannot compel him to sell. This issue is clearly stated in the case Fisher V. Bell (1961)
Q.B. 394.
Thus, accordingly, Mr. A cannot compel Mr. D to sell the articles.

3.
a) Gandaki General Insurance Company Ltd. has been given ordered to cancel the
registration by Insurance Board. The following are the liabilities of the company
however the company could only realize Rs. 600 million from its assets on
liquidation. You have been appointed as a liquidator of the company. How do you
settle the following liabilities in the priority basis? 5
i) Loan amount 150 million
ii) Amount due to the Board 120 million
iii) Amount due to GON 30 million
iv) Dues to the employees 35 million
v) Insurance claim 250 million
vi) Amount of liquidation 15 million
b) What do you understand by collective investment scheme as defined in Securities
Act, 2063? In which circumstances the Securities Board can cancel the approved
collective investment scheme? (2+3=5)
Answer
a)Section 41B of the Insurance Act 2049 has made provision regarding the cancellation
of the registration and settlement of the liabilities on priority basis of the Insurance
Company.
If any Insurer is dissolved due to the cancellation of its registration pursuant to Section
13, the liabilities shall be settled in the following order of priority :-

(a) The expenses incurred for the dissolution,

KCP
(18)

(b) The amount to be paid against the insurance claims to the Insured pursuant to
Section 16,
(c) The remuneration and other outstanding amounts to be obtained by the employees
of the Insurer,
(d) Loan amounts,
(e) The amount to be paid to the Board,
(f) The amount to be paid to the Government of Nepal .

In the given question Gandaki General insurance Company has been given ordered to
cancel the registration by Insurance Board. Being appointed as a liquidator of the
company I have to follow up the procedure and provision stated in section 41B for the
settlement of the liabilities. As given in the question the realization of the amount from
the liquidation of the assets Rs 600 million, the priority has to make as under:
a) The expenses incurred for the dissolution 15
b) Insurance claim 250
c) Due to employees for remuneration 35
d) Loan Amount 150
e) Amount to be paid the Board 120
f) Amount to be paid to the Government of Nepal 30

b)Collective Investment Scheme


As per section 2(w) of Securities Exchange Act 2063, Collective Investment Scheme
means an investment fund, unit fund, or any other similar participatory fund
management program prescribed by the Board from time to time, operated by a scheme
manager under this Act with the objective of taking under its responsibility the savings
and investment funds mobilized by ensuring the participation of different individuals or
institutions, and proportionately distributing the returns accruing from its skilled
investment services among the participants of the scheme.

As per Section 72, under following circumstances the Board can exercise its Power to
cancel permission granted by it to operate a collective investment scheme:
a. In case the necessary conditions relating to the operation (of the
scheme) are not fulfilled in a satisfactory manner.
b. In case it is not deemed appropriate to give continuity to the scheme
when judged from the view point of the welfare of the participants.
c. In case the scheme manager and the depository contravene this
Ordinance or the rules or bye-rules framed under this Act, or submit
any false statement of the scheme to the Board.
While cancelling permission, the Board may subject the manager, depositary, direction
or the concerned employee connected with the scheme to necessary examinations.The
working procedure to be adopted while closing the operation of a scheme and clearing
the accounts according to the order of the Board shall be prescribed
4.
a) Mr. Ram Singh Yadav is an employee in Hulas Industries Ltd., Simara. A dispute
has arisen between Mr. Ram Singh and company management on the issue
relating to payment of bonus. Outline the dispute settlement procedure under the
existing laws. 5
b) Enumerate the provisions regarding the appointment of company secretary as
provided in the Companies Act, 2063. 5
Answer

KCP
(19)

a)As per the provision of section 16 of Bonus Act 2030 if any dispute arises between the
employees and the management regarding bonus, the labor office shall have to resolve
such dispute by talks keeping both the side employees and management.
In case dispute could not be resolved by talks, the labor office may make decision by
requiring necessary documents and ledger accounts from the establishment and the
employees, then also if any of the side not satisfied with the decision of the labor office
made may appeal to the labor court within 35 days after receipt of the notice of such
decision and the decision given by the labor will be final decision.
In the given case the dispute has arisen between the employee Mr. Ram Singh and the
management of Hulas industries in regard of the bonus. Hence, as per the provision of
the Act both of them try to resolve such dispute with dialogue if not possible may go to
the labour office where the labour office may make decision by requiring necessary
document and ledger accounts from both the side if any of the side, not satisfied with
the decision of the labour may appeal to the labour court within 35 days after receipt
the decision from the labour office and the decision of given by the labour court shall be
final decision. This is the procedure to resolve the dispute arises between the
management of the establishment and the labour.

b)Appointment of company secretary:


As per the provision of Companies Act 2063 Section 185, a public company with the
paid –up capital of ten million rupees or more shall appoint to the post of company
secretary a Nepalese citizen who has worked in the related field for at least two years
after obtaining the professional certificate of company secretary issued by a native or
foreign body authorized to issue the professional certificate of company secretary
pursuant to the prevailing law or who has worked in the related field or in the field of
company management for at least three years after doing at least bachelor degree in
law, management ,commerce or economics may be appointed to the post of company
secretary.

Provided that,this provision shall not apply to the company secretary who is incumbent
at the time of commencement of this Act for three years after the date of
commencement of this Act.

No director of the concerned company shall be eligible to be appointed as the company


secretary of such company. A person shall not be appointed to the post of company
secretary of more than one company at the same time. Provided, however, that this
provision shall not bar the appointing of the company secretary of any principal
company to the post of company secretary of the subsidiary company of such company.

Where it is provided by this Act, the prevailing law or articles of association that any
act has to be done by or through the company secretary, and the post of company
secretary remains vacant in the company or any reason the incumbent company
secretary fails to do such act or shows has inability to do such act, then any such
employee of the company, who has the qualification referred to in this Act, as
designated by the board of directors to do such act may perform such act in the
capacity of company secretary.

5. Give the answer of the following questions:


a) Discuss the rights and powers of the Industrial Promotion Board. 5

KCP
(20)

b) What do you mean by insider trading of securities? What is the punishment to be


imposed under the Securities Act, 2063 to the person who engaged in insider
trading? (2+3=5)

c) Mr. Jack, a British National, wants to apply for visa for a period of 8 months to
undertake research on industrial environment of Nepal. He desires to apply for the
visa under the Foreign Investment and Technology Transfer Act, 1992. Will he be
eligible for visa under the FITTA 1992? What would be your answer if Mr. Jack
has already been made investment of USD 150,000 under the FITTA 1992? 5
Answer

a)
13. Functions, Duties and Power of the Board: The function, duties and powers of the
Board shall be as follows:-
(a) To render necessary cooperation in formulating and implementing policies, laws and
regulations pertaining to the industrialization of the country.
(b) To give guidelines in attaining the objectives of liberal, open and competitive
economic policies pursued by the country so as to make the industrial sector competitive.
(c) To maintain coordination between the policy level and the implementation level of the
industrial policy.
(d) To cause to follow the ways and means for the prevention of the
environmental pollution by putting more emphasis on the avoidance of effects on the
environment and the public health.
(e) To make recommendation to Government of Nepal for the inclusion of any industry in
the classification of industries.
(f) To make recommendation to Government of Nepal to introduce changes in the Areas
mentioned in Annex-3 by making evaluation thereof from time to time.
(g) To give directives to the concerned body after making enquires into the application
submitted by any industry complaining that the industry has not received the facilities
and concessions to be made available by the committee.
(h) Other functions, duties and power of the board shall be as prescribed.

b) Section 91 of Securities Act,2063 states that 'in case any person


conducts or causes others to conduct transactions in securities on
the basis of any inside information or report which has not been
made public and which can have an impact on the value of the
securities, or supplies any such information or report possessed by
him to any other person, except in the course of discharging his
duties, he shall be deemed to have carried out an insider trading of
securities.' Moreover, the punishment to be imposed on the person
who is engaged in such business is mentioned in section 101 of
Securities Act. According to that provision 'any insider who deals
in securities in contravention of section 91 is proved to have done
so, he shall be punished with a fine equal to the amount involved
in the offense, or with imprisonment for a term not exceeding one
year, or with both .

KCP
(21)

c) Provisions Relating to Visa:


A foreign national visiting Nepal in connection with undertaking any study or carrying
out any research with the objective of making investment in Nepal shall be provided a
non-tourist visa for upto six months.

A foreign investor or dependent family or authorized representative of such a foreign


investor and department family of such authorized representative shall for the purpose
of stay in Nepal be provided a business visa until the foreign investment is retained.
Provided that a foreign investor who, at a time, makes investment in an amount no less
than one hundred thousand United States dollar or in convertible foreign currency
equivalent thereto, and his/her dependent family shall be granted a residential visa
until such investment is retained.

In the given question Mr. Jack desires to apply visa for the period of 8 months for the
purpose to undertake research on industrial environment of Nepal i.e.more than the
period as provided in the act for the non tourist visa hence he will not be able to obtain
such visa under FITTA.
Mr. Jack is not eligible to get visa to undertake research. There must be intention to
make investment to get visa under section 6 of the Act

However, in the given question Mr. Jack who has already invested of USD 150000
which is more than the limit prescribed of USD 100000 hence not even for him but also
the dependent family shall be granted a residential visa until such investment is
retained.
6. Answer the following questions:
a) The auditor of RB Limited of which 100% shares have been taken by
Government of Nepal, believes that he should be doing his work independently as
per his own approach. Do you agree to his approach? Will your answer be the
same if he is the auditor of KB Limited where the Government of Nepal has taken
51% shares? 5
b) Mention the objectives of Institute of the Chartered Accountant of Nepal under
the Nepal Chartered Accountant Act, 2053. 5
Answer
a) As per section 6 of the Audit Act 2048, the audit of corporate Bodies wholly
owned by Government of Nepal is stated as under:

The audit of the corporate bodies wholly owned by Government of Nepal shall be
audited by the Auditor General pursuant to this Act. If the Auditor General is
constrained by time and resources to audit the corporate bodies wholly owned by
Government of Nepal he/she may appoint license holder auditors under the prevailing
laws an assistant. While appointing auditor as such, he/sheshall give priority to the
Nepali citizen.

The auditor appointed shall act under the direction, supervision and control of the
Auditor General. The powers, functions, duties and responsibilities of the auditors
appointed and the procedures to be followed by them in course audit and provisions
relating to their report shall be as prescribed by the Auditor General.

KCP
(22)

The remuneration to be paid by the concerned organization to the auditors shall be


fixed by the Auditor General keeping in view the volume of financial transactions, status
of accounts, number of branches and sub-branches, work load and workprogress of the
concerned organization.

While the audit of the corporate bodies substantially owned by government of Nepal has
made provision under section 7 of the Act as:

The Auditor General shall be consulted while appointing an auditor for auditing of the
corporate bodies substantially owned by Government of Nepal.The procedures to be
followed while consulting the Auditor General for appointing auditors on matters to
principles of audit to be followed by the auditors during their audit shall be as
prescribed by the Auditor General. The concerned organization shall deliver at the
Office of the Auditor General a copy of the report submitted by the auditor appointed in
consultation with the Auditor General.

The Auditor General may issue directives to the concerned organization in respect of the
irregularities observed in the report received and it shall be the duty of concerned
organization to abide by such directives.

So in case of RB Limited he cannot be appointed as an auditor but can be an assistant


of auditor general. In case of KB limited he can be appointed as the auditor with the
consultation made by the company with auditor general.

b) Section 5 of Nepal Chartered Accountant Act,2053 states the objectives of the


ICAN which are as follows:
i) To play the role of regulatory body to encourage the members to
carry on accounting profession being within the extend of the code
of conduct in order to consolidate and develop accounting
profession as a cause for economic development of the nations.
ii) To enhance social integration and faith in accounting profession by
raising awareness of the general public towards the importance of
accounting profession and the economic and social responsibility
of professional accountants.
iii) To develop, protect and promote the accounting profession by
enabling professional accountants understand their responsibility
towards the importance of accounting profession and accountancy.
iv) To develop mechanism of registration, evaluation and examination
of accounting professionals in consonance with international
norms and practices so as to make the accounting profession
respectable and reliable.

7. Write short notes on the following: (3×5=15)


a) Formation of the Nepal Securities Board
b) Functions not to be carried out by Nepal Rastra Bank under the Nepal Rastra
Bank Act, 2058
c) Factory Inspector
Answer

KCP
(23)

a) Formation of the Nepal Securities Board


Section 3 of Securities Act- There shall be established a Board by the name of Nepal
Securities Board in order to regulate and manage the activities of the securities markets
and persons involved in securities business by regulating the issue, purchase, sale and
exchange of securities in order to develop capital market and protect the interests of
investors in securities. The Board shall consist of the following members:-
a. A person appointed by the Government of Nepal -
Chairperson
b. Joint Secretary, Ministry of Finance -
Member
c. Joint Secretary, Ministry of Law, Justice and
Parliamentary Affairs -Member
d. Representative, Nepal Rastra Bank -
Member
e. Representative, Institute of Chartered
Accountants of Nepal -Member
f. Representative, Federation of Nepalese
Chambers of Commerce and Industries -
Member
g. A person nominated by the Government of Nepal
from amongst the experts who have obtained at least master degree in
economics, management, finance, commerce or law from recognized
university and gained at least seven years of experience in stock
exchange, management, capital market development, finance and
economic sector - Member.
b) Functions not to be carried out by the Nepal Rastra Bank:
As per Section 7 of Nepal Rastra Bank Act, 2058, except otherwise provided for in
thisAct, the Bank shall not carry out the following functions:-
a. Providing any loan, accepting any type of deposit or making any type
of financial gift;
b. Purchasing shares of any commercial bank, financial institution,
publiccorporation or a company or acquiring any type of proprietary
right in anyfinancial, commercial, agricultural, industrial or other
institution;
c. Carrying out any type of business; and
d. Acquiring right over movable and immovable property by way of
purchase,lease or in any manner whatsoever.

Provided that the Bank may acquire such property as required forcarrying out its
function or for achieving its objectives.
(2) Notwithstanding anything contained in sub-section (1), the Bank may carryout
the following functions: -
(a) To provide loan to and invest in the shares of the institutions which carry
out the functions helpful in carrying out the function of the Bank or in
attaining itsobjectives, not exceeding ten percent of the total capital of
such institutions.
Explanation:

KCP
(24)

For the purpose of this clause, "institutions which carry out the functions which
are helpful in carrying out the functions of the Bank or inattaining its objectives"
mean any institutions, companies which areexclusively engaged in evaluating,
managing, protecting the security,restructuring and transferring the securities
of commercial banks or offinancial institutions, carry out the function of credit
rating, exchange of creditinformation, process and transmit data, print financial
instruments, clearingpayments, liquidate property, produce bank notes and
coins and act as trusteeand any other institution or companies established for
carrying out similarfunction and prescribed by the Bank.
(b) To provide loan to its own employees.

c) Factory Inspector:
It is an important designation or authority designed under the Labour Act, 2048 to
protect and enforce the interests of the labour in the industrial relation within the
country. Pursuant to section 66 of the Labour Act, 2048, the Government of Nepal may,
by a notification published in the Nepal Gazette, appoint one or more Factory Inspector
for any region or may appoint one Chief Factory Inspector for whole of the state of
Nepal. The basic function of the factory inspector is to assist the government to make
sure that the provisions of the Labour Act have been followed by all the concerned
parties of the industrial relation. To make investigation and to take appropriate actions
for the good industrial relation, the Act has conferred the various rights and powers to
the factory inspector.
Powers of the Factory Inspector:
Pursuant to section 67 of the Labor Act, 2048, the Factory Inspector shall have the
following powers -
a. To enter into the premises of the factory as per necessity;
b. To examine building, land, plant, machine, health and safety features
of the factory, to collect and test the samples of finished or semi-
finished materials used in the factory or to cause them to be tested, to
inspect the registers and documents relating to the factory and to
record statements of any person, as per necessity;
c. To examine the boilers and pressure vessels and to permit the
operations thereof;
d. To provide necessary advice and assistance to the Establishment on
making arrangements of training for workers or employees;
e. To exercise the powers, functions and duties of the Labor Officer during
his absence; and
f. To perform other functions as per the directives of the Government and
Department of Labor.

Other powers, functions and duties of the Factory Inspector shall be as prescribed from
time to time by the Government of Nepal and in the labour rules.

KCP
CAP-II, Cost & Management Accounting, Dec 2013
(Suggested)

Roll No……………. Maximum Marks - 100


Total No. of Questions: 6 Total No. of Printed Pages - 4
Time Allowed - 3 Hours
Marks
All questions are compulsory. Working notes should form part of the answer.
Make assumptions wherever necessary.

1. A company having its head office in Centown has three factories situated at Uptown,
Middletown and Downtown. The operations of Middletown have been unprofitable
for a number of years. The leasehold of Middletown will also expire by the end of
current year. In view of continued losses the management has decided to close down
the said factory rather than renew the lease. The factory's plant and machinery can be
sold at a price higher than the written down value and the surplus funds will be
sufficient to cover all termination costs.
Projected profitability of the factories for the next year is as under:
(Rs. in lakhs)
Uptown Middletown Downtown Total
Particulars
Rs. Rs. Rs. Rs.
Sales 400 100 300 800
Variable costs 220 75 195 490
Fixed costs:
Factory 80 30 40 150
Selling & administration 30 5 15 50
Head office expenses apportioned 25 15 25 65
Profit 45 (25) 25 45
The company however would like to continue to serve the customers now being
served by the Middletown factory if it could do so economically. Accordingly
following proposals were put forward for consideration based on a selling price of
Rs. 250 per unit.
a) Close down Middletown factory and expand the operations of Downtown factory
for which surplus capacity exists. This proposal will involve the following
changes:
i) Sales revenue of Downtown factory will increase by 25%.
ii) Factory fixed costs of Downtown factory will increase by 10%.
iii) Fixed selling and administration cost of Downtown factory will increase by
5%.
iv) Variable distribution costs of additional output will increase by Rs. 4 per unit.
b) Close down Middletown factory and expand the operations of Uptown factory
subject to the following changes in case of later:
i) Sales revenue will increase by Rs. 80 lakhs.
ii) Factory fixed costs will increase by 20%.
iii) Fixed selling and administration cost will increase by 10%.
iv) Variable distribution costs in respect of the additional units will increase by
Rs. 5 per unit.
c) Close down Middletown factory and enter into a long term contract with an
independent manufacturer to serve the customers of Middletown factory. The
manufacturer will pay a royalty of Rs. 5 per unit to the company. In that event the
sales of the area served by the Middletown factory will fall by 25%.
d) Close down Middletown factory and discontinue serving the present customers of
that area.

JTN P.T.O.
(26)
You are required to evaluate each of these proposals and advise the management of
the action to be taken in the interest of improving profitability of the company. 20

Answer No. 1
Evaluation of Proposal (i)
Closing of Middletown factory and expanding Downtown factory
Statement of Profit
(Rs. in lakhs)
Uptown Downtown Total
Particulars
Rs. Rs. Rs.
Sales 400.00 375.00 775.00
Variable costs:
Factory [WN 1] 220.00 243.75 463.75
Additional distribution [WN 2] - 1.20 1.20
Total variable costs 220.00 244.95 464.95
Contribution 180.00 130.05 310.05
Fixed costs:
Factory 80.00 44.00 124.00
Selling & administration [WN 3] 30.00 15.75 45.75
Total fixed costs 110.00 59.75 169.75
Factory contribution 70.00 70.30 140.30
Head office expenses apportioned 65.00
Profit 75.30
Working Notes:
1. Variable factory cost of Downtown factory = (Rs. 195 lakhs × 1.25) = Rs. 243.75 lakhs.
2. Present sales at Downtown Rs. 300.00 lakhs.
Selling price per unit = Rs. 250.00
Number of units sold = Rs. 300.00 lakhs / Rs. 250.00 = 120,000 units.
Additional sales as per proposal (i) = 120,000 × 25% = 30,000 units.
Additional variable distribution costs = 30,000 × Rs. 4 = Rs. 120,000
3. Fixed selling and administration costs in Downtown = Rs. 15.00 lakhs × 1.05 = Rs.
15.75 lakhs.

Evaluation of Proposal (ii)


Closing of Middletown factory and expanding Uptown factory
Statement of Profit
(Rs. in lakhs)
Uptown Downtown Total
Particulars
Rs. Rs. Rs.
Sales 480.00 300.00 780.00
Variable costs:
Factory [WN 1] 264.00 195.00 459.00
Additional distribution [WN 2] 1.60 - 1.60
Total variable costs 265.60 195.00 460.60
Contribution 214.40 105.00 319.40
Fixed costs:
Factory 96.00 40.00 136.00
Selling & administration 33.00 15.00 48.00
Total fixed costs 129.00 55.00 184.00
Factory contribution 85.40 50.00 135.40
Head office expenses apportioned 65.00
Profit 70.40
Working Notes:
JTN P.T.O.
(27)
1. Variable costs at Uptown = Rs. (220 × 480 / 400) = Rs. 264.00 lakhs.
2. Additional sales in units at Uptown = Rs. 80.00 lakhs / Rs. 250.00 = 32,000 units.
Variable distribution costs = Rs. 5 × 32,000 = Rs. 160,000

Evaluation of Proposal (iii)


Closing of Middletown factory and subcontracting production
Statement of Profit
(Rs. in lakhs)
Uptown Downtown Total
Particulars
Rs. Rs. Rs.
Sales 400.00 300.00 700.00
Variable costs 220.00 195.00 415.00
Contribution 180.00 105.00 285.00
Fixed costs:
Factory 80.00 40.00 120.00
Selling & administration 30.00 15.00 45.00
Total fixed costs 110.00 55.00 165.00
Factory contribution 70.00 50.00 120.00
Add: Royalty for Middletown factory 1.50
121.50
Head office expenses apportioned 65.00
Profit 56.50
Working Notes:
Present sales of Middletown factory = Rs. 100.00 lakhs.
Sales under new arrangement = Rs. 100 lakhs × 75% = Rs. 75 lakhs.
Number of units = Rs. 75 lakhs / Rs. 250 = 30,000 units.
Royalty = Rs. 5.00 × 30,000 = Rs. 150,000

Evaluation of Proposal (iv)


Closing of Middletown factory
Statement of Profit
(Rs. in lakhs)
Uptown Downtown Total
Particulars
Rs. Rs. Rs.
Factory contribution [See (iii) above] 70.00 50.00 120.00
Head office expenses apportioned 65.00
Profit 55.00
Conclusion / Recommendation:
The above calculations show that the company will make the highest profit under
proposal (i) i.e. "Closing of Middletown factory and expanding the operations of
Downtown factory". Hence, this proposal should be preferred over other proposals.

2.
a) A company has three production departments (M1, M2 and A1) and three service
departments, one of which “Engineering service department” is servicing the M1
and M2 only. The relevant information are as follows:
Product X Product Y
M1 10 Machine hours 6 Machine hours
M2 4 Machine hours 14 Machine hours
A1 14 Direct Labour hours 18 Direct Labour hours

The Annual budgeted overhead costs for the year are:


Indirect Wages Consumable supplies
Rs. Rs.
JTN P.T.O.
(28)
M1 46,520 12,600
M2 41,340 18,200
A1 16,220 4,200
Stores 8,200 2,800
Engineering Services 5,340 4,200
General Service 7,520 3,200

Depreciation on Machinery 39,600


Insurance on Machinery 7,200
Insurance on Building 3,240
(Total building insurance cost for M1 is one third of annual premium)
Power 6,480
Light 5,400
Rent 12,675
(The general service dept. is located in a building owned by the company. It is
valued at Rs. 6,000 and is charged into cost at notional value of 8% per annum.
This cost is additional to the rent shown above.)
The value of materials issued to the production departments are in the same
proportion as shown above for the consumable supplies.
The following data are also available:
Department Book value Area Effective Production Capacity
Machinery (sq. ft.) H.P. Direct Labour Machine
Rs. Hours % Hours Hours
M1 120,000 5,000 50 200,000 40,000
M2 90,000 6,000 35 150,000 50,000
A1 30,000 8,000 05 300,000
Stores 12,000 2,000 -
Engg. Service 36,000 2,500 10
General Service 12,000 1,500 -

Required: (5+3+2+2=12)
i) Prepare an overhead analysis sheet, showing the bases of apportionment of
overhead to departments.
ii) Allocate service department overheads to production department ignoring the
apportionment of service department costs among service departments.
iii) Calculate suitable overhead absorption rate for the production departments.
iv) Calculate the overheads to be absorbed by two products, X and Y.

b) Raman Garment Company employs 70 direct workers. Each worker is paid wages
Rs. 400 per week of 40 hours. When necessary, overtime worked up to a
maximum of 16 hours per week per worker is paid at time rate plus one –half as
premium. The current output on an average is 6 units per person-hour, which may
be regarded as standard output. If bonus scheme is introduced, it is expected that
the output will increase to 8 units per person-hour. The worker will, if necessary,
continue to work overtime up to the specified limit although no premium no
incentives will be paid.
The company is considering introduction of either Halsey scheme or Rowan
scheme of wage incentive system. The budgeted weekly output is 20,000 units.
The selling price is Rs. 15 per unit and the direct material cost is Rs. 10 per unit.
The variable overhead amount to Rs. 0.5 per direct labor hour and the fixed
overhead is Rs. 10,000 per week.
Prepare a statement to show the effect on the company‟s weekly profit of the
proposal to introduced (i) Halsey scheme, and (ii) Rowan scheme. 8

JTN P.T.O.
(29)
Answer No. 2
a)
(i) Summary of Apportionment of Overheads

Items Basis of Total Production Deptt. Service deptt.


Apportionment amount M1 M2 A1Store Engineering General
Service Service Service
Indirect Given 1,25,140 46,520 41,340 16,220 8,200 5,340 7,520
Wages
Consumable
Stores Given 45,200 12,600 18,200 4,200 2,800 4,200 3,200

Depreciation Machine value 39,600 15,840 11,880 3,960 1,584 4,752 1,584
(20:15:5:2:6:2)
Insurance of Machine value 7,200 2,880 2,160 720 288 864 288
Machine (20:15:5:2:6:2)
Insurance on 1/3 to M1 & 3,240 1,080 648 864 216 270 162
On building balance area
basis 12:16:4:5:3)
Power HP Hr. % 6,480 3,240 2,268 324 -- 648 --
(50:35:5:0:10:0)
Light Area 5,400 1,080 1,296 1,728 432 540 324
(10:12:16:4:5:3)
Rent Area 12,675 2,535 3,042 4,056 1,014 1,268 760
(10:12:16:4:5:3)
Rent of Direct 480 -- -- -- -- -- 480
General (8% of 6,000)
Service _______ _______ ______ _______________ _________ _________
Total 2,45,415 85,775 80,834 32,072 14,534 17,882 14,318

(ii) Allocation of service department overheads


Service Basis of Production dept Service dept.
Dept. Apportionment M1 M2 A1 Store Engineering General

Store Consumable value


(126:182:42) 5,232 7,558 1,744 (14,534) -- --
Engineering Machine Hrs
(4:5) 7,948 9,934 -- -- (17,882) --

General LHR basis


(20:15:30) 4,406 3,304 6,608 -- -- (14,318)

Production Department Allocated in (i)


85,775 80,834 32,072
Total 2,45,415 1,03,361 1,01,630 40,424

(iii) Overhead Absorption rate


M1 M2 A1
Total overhead allocated 1,03,361 1,01,630 40,424
Machine hours 40,000 50,000 --
Labour hours -- -- 3,00,000
Rate per MHR 2.584 2.033 --
Rate per Direct labour -- -- 0.135
JTN P.T.O.
(30)

(iv) Statement showing overhead absorption for Product X and Y


Machine Absorption Product X Product Y
Deptt.Rate Hours Rs. Hours Rs.
M1 2.584 10 25.84 6 15.50
M2 2.033 4 8.13 14 28.46
A1 0.135 14 __1.89 18 __2.43
35.87 46.39

b) Statement of company‟s weekly profit


Present Scheme Halsey Scheme Rowan Scheme
Output (units) 20,000 20,000 20,000
Selling price Rs. Per unit 15.00 15.00 15.00
Direct Material Cost Rs. 10.00 10.00 10.00
Per unit
Direct Labour Rs. Per unit 36,000/20,000 = 1.80 29,167/20,000 = 1.46 31,250/20,000 = 1.56
Variable Overhead Rs. Per (3,333*0.5)/20,000 = 0.08 (2,500×0.5)/20000= (2,500×0.5)/20000=
unit (0.5 per direct labour 0.06 0.06
hour)
Total Variable costs Rs. 11.88 11.52 11.62
Per unit
Contribution Rs. Per unit 3.12 3.48 3.38
Total Contribution Rs. 62,400 69,600 67,600
Less: Fixed Overhead Rs. 10,000 10,000 10,000
Net Profit Rs. 52,400 59,600 57,600

Working Notes:
Wage rate per hour per workers = Rs400\Rs40 = Rs 10 per hour
Overtime rate per hours per workers = normal rate per hour + 50%
= Rs 15 per hour
Overtime premium = Rs. 15- Rs. 10 = Rs. 5
Average current output/hour = 6 units

Hours to be worked for budgeted weekly output of 20,000 units


=20,000\6 units =3333.33 hours

Total normal hours available per week = No. of workers ×hour per week
= 70 workers × 40 hours
= 2,800 hours
Maximum overtime hours per week for all workers = 70 x 16
= 1120
Overtime hour required
= Total hour to be worked - total normal hrs available
= 3333.33 hrs. – 2800 hrs.
= 533.33 hrs.
Hence, all overtime hours worked are eligible for bonus.

Total wages under the proposed scheme:


Normal wages for total hrs worked (3333.33 hrs ×10) 33,333
Add: overtime premium (5 × 533.33) 2,667
36,000
Time saved
JTN P.T.O.
(31)
Time allowed = 1 hour for 6 units
For 20,000 units = 20,000\6 units = 3,333.33 hrs

Time taken: = 1 hour for 8 units


20,000 units need = 20,000/8 = 2,500 hours

Time saved = Time allowed – Time taken


= 3,333.33 hrs – 2,500 hrs
= 833.33 hrs

Under Halsey Scheme:


Total wages = (Time taken × Standard wage rate) + (50% × Time saved × Standard wage rate)
= (2,500×10) + (50%×833.33×10)
= 25,000+4,167
=29,167

Under Rowan Scheme:


Total wages = Normal wages + Bonus
= (Time taken × Standard wage rate) + [Time saved × Time taken\ Time allowed ×
Standard wage rate]
= (2,500 x10) + (833.33 x 2,500/3,333.33 x 10)
= 25,000+6,250
= 31,250

3.
a) Chemicals Ltd. produces two joint products 'J' and 'K' in Department A from a
basic raw material. The input-output ratio of Department A is 100:90. Product 'J'
which becomes the input of Department B can be further processed in Department
B to make one of the most popular industrial product 'N'. The input-output ratio
of Department B is 100:95. Alternatively, product 'J' can also be sold at the split
off stage. The selling prices of the products are:
Product Selling price per Kg. (Rs.)
J 29.40
K 26.00
N 31.50
The raw material cost, departmental expenses, production data and selling
expenses envisaged in the budget are as under:
i. Raw material cost is Rs. 16 per Kg.
ii. Departmental expenses:
Particulars Department A (Rs. lakhs) Department B (Rs. lakhs)
Direct materials 10.00 3.00
Direct wages 15.00 5.00
Variable overheads 20.00 7.00
Fixed overheads 25.00 10.00
iii. Production data and selling expenses:
Product Production (Kg.) Selling expenses (Rs.)
J - 100,000
K 850,000 200,000
N 475,000 200,000
Required: (7+5+3=15)
i. Prepare a statement showing the apportionment of joint costs between
products 'J' and 'K'.

JTN P.T.O.
(32)
ii. Advise whether the company should process product 'J' further into product
'N' or not. Show your workings.
iii. Prepare a statement of profitability based on your decision in (ii) above.
b) A company is at present working at 90% of its capacity and producing 13,500
units per annum. It operates a Flexible Budgetary Control System. The following
figures are obtained from its budget:
Capacity utilization 90% 100%
Sales Rs. 1,500,000 Rs. 1,600,000
Fixed expenses 300,500 300,600
Semi fixed expenses 97,500 100,500
Variable expenses 145,000 149,500
Units manufactured 13,500 15,000
Material and labour costs per unit are constant under the present conditions.
Current profit margin is 10% on sales.
Required: (3+2=5)
i) Determine the differential cost of producing 1,500 units by increasing
capacity utilization to 100 percent.
ii) What would you recommend as an export price per unit for these 1,500 units
after considering that overseas prices are much lower than inland prices?
Answer No. 3
a)
i. Statement showing the apportionment of joint costs between Products 'J' and 'K'
Particulars Product 'J' Product 'K' Total
Output (kgs) (WN: 1) 500,000 850,000 -
Selling price (Rs.) 29.40 26.00 -
Sales value (Rs.) 14,700,000 22,100,000 36,800,000
Less: Selling expenses (Rs.) 100,000 200,000 300,000
Market value at spilt off stage (Rs.) 14,600,000 21,900,000 36,500,000
Ratio (146:219) or (40:60) 40 60 100
Joint cost apportioned (Rs.) 12,400,000 18,600,000 31,000,000

Working notes:
1) Output of product 'J':
Production of product 'N' in department B: 475,000 kgs.
Process loss in department B is 5% of input.
Hence, input into department B: 475,000 × 100/95 = 500,000 kgs.
Output of product 'J' is equal to input into department B. Hence, output of 'J' = 500,000 kgs.

2) Input of raw materials into department A:


Total output of department A: 500,000 kgs. + 850,000 kgs = 1,350,000 kgs.
Process loss in department A is 10% of input
Hence, input into department A: 1,350,000 × 100/90 = 1,500,000 kgs.

3) Joint costs in department A:


Rs.
Raw material costs [1,500,000 × Rs. 16] 24,000,000
Direct materials 1,000,000
Direct wages 1,500,000
Variable overheads 2,000,000
Fixed overheads 2,500,000
Total joint costs 31,000,000

ii. Statement showing Profit or (Loss) when product 'J' is processed further into 'N'
Particulars Rs. Rs.
JTN P.T.O.
(33)
Sales value [475,000 × Rs. 31.50] 14,962,500
Less: Joint cost of product 'J' 12,400,000
Department B costs: Direct materials 300,000
Direct wages 500,000
Variable overheads 700,000
Fixed overheads 1,000,000
Selling expenses 200,000 15,100,000
Loss from further processing 137,500

Statement showing Profit or (Loss) when product 'J' is not processed further into 'N'
Particulars Rs. Rs.
Sales value [500,000 × Rs. 29.40] 14,700,000
Less: Joint cost of product 'J' 12,400,000
Selling expenses 100,000 12,500,000
Profit 2,200,000
Less: Under recovery of fixed overheads 1,000,000
Net Profit 1,200,000
The above statements show that there is a profit of Rs. 1,200,000 in case 'J' is not further processed
as compared to loss of Rs. 137,500 if 'J' is processed into N. Hence, further processing of 'J' should
not be undertaken. It should rather be sold at the split-off points. Moreover, if it is assumed that
fixed overheads to department B amounting to Rs. 10 lakhs can be avoided, if 'J' is not further
processed, the amount of profit of would be Rs. 22 lakhs.

iii. Statement of profitability as per decision given in (b) above


Particulars Product 'J' Product 'K' Total
Output & Sales (kgs) 500,000 850,000 -
Selling price (Rs.) 29.40 26.00 -
Sales value (Rs.) 14,700,000 22,100,000 36,800,000
Joint cost (Rs.) 12,400,000 18,600,000 31,000,000
Selling expenses (Rs.) 100,000 200,000 300,000
Total costs (Rs.) 12,500,000 18,800,000 31,300,000
Profit (Rs.) 2,200,000 3,300,000 5,500,000
Less : Under recovery of fixed overheads (Rs.) 1,000,000
Net Profit (Rs.) 4,500,000

b.
(i) Differential cost of producing 1,500 units:
Existing Proposed Differences
Capacity levels 90% 100% 10%
Output (in units) 13,500 15,000 1,500
Costs: Rs. Rs. Rs.
Materials and Labour (WN:1) 807,000 896,667 89,667
Variable expenses 145,000 149,500 4,500
Semi-fixed expenses 97,500 100,500 3,000
Fixed expenses 300,500 300,600 100
1,350,000 1,447,267 97,267
Hence, differential cost for producing 1,500 units is Rs. 97,267

(ii) Minimum price for export = Rs. 97,267/1,500 = Rs. 64.84


At the minimum price of Rs. 64.84 per unit, there will be no additional profit to the company. A
price below this may be considered if there is a possibility of getting some benefits because of
export e.g. exports incentives or subsidy from the government.
It has been presumed in the above case that utilization of full capacity would not require any
additional capital investment.

JTN P.T.O.
(34)

Working notes:
1. Computation of cost of Material and Labour:
Rs. Rs.
Sales at 90% Capacity 1,500,000
Less: Profit @ 10% 150,000
Total cost 1,350,000
Less other expenses:
Fixed 300,500
Semi-fixed 97,500
Variable 145,000 543,000
Material & Labour cost: At 90% capacity 807,000
Material & Labour cost: At 100% capacity (Rs. 807,000 × 100/90) 896,667

4.
a) A contractor commenced a contract on 1-1-2013. The costing records concerning
the said contract reveal the following information as on 30-9-2013:
Rs.
Material sent to site 774,300
Labour paid 1,079,000
Labour outstanding as on 30-9-2013 102,500
Salary to engineer 20,500 per month
Cost of plant sent to site (1-1-2013) 771,000
th
Salary to supervisor – 3/4 time devoted to contract 9,000 per month
Administration and other expenses 460,000
Prepaid administration expenses 10,000
Material in hand at site as on 30-9-2013 75,800
Plant used for the contract has an estimated life of 7 years with residual value at
the end of life Rs. 50,000. Some of materials costing Rs. 13,500 was found
unsuitable and sold for Rs. 10,000. Contract price was Rs. 4,500,000. On 30-9-
2013 two third of the contract was completed. The architect issued certificate
covering 50% of the contract price and contractor has been paid Rs. 2,000,000 on
account. Depreciation on plant is charged on straight line basis.
Prepare Contract Account. 8
b) A company wants to buy a new machine to replace one, which is having frequent
breakdown. It received offers for two models, M1 and M2. Further details
regarding these two models are given below.
Particulars M1 M2
Installed Capacity [Units] 10,000 10,000
Fixed overheads per annum Rs. 240,000 Rs. 100,000
Estimated profit at the above Rs. 160,000 Rs. 100,000
capacity
The product manufactured using this type of machine, M1 or M2, is sold at
Rs. 100 per unit.
Required: (4+2=6)
i) Determine the break-even level of sales for each model.
ii) Calculate the level of sales at which both the models will earn the same profit.
c) Sambriddhi Coffee Ltd. is the supplier of high quality coffee with gift packaging
for export. Company is newly establishes and very conscious about the cost
control. To be competitive in the market; company maintain definite level of
stock as well. The following are the details of their operating during 2012:
Average monthly market demand for coffee 2,500 Packet
JTN P.T.O.
(35)
Ordering cost Rs. 100 per order
Inventory carrying cost 20% per annum
Cost of high quality material Rs. 500 per Kg.
Normal usage of material 100 Kg. per week
Minimum usage 50 Kg. per week
Maximum usage 300 Kg. per week
Lead time 4-6 weeks
Using the above data and information compute: (1+1+1+1+2=6)
i) Economic Order Quantity
ii) Reorder Level
iii) Maximum Level of Stock
iv) Minimum Level of Stock
v) If the supplier is willing to supply quarterly 1,500 Kg. in one lot at a discount
of 5%, is it worth accepting?

Answer No. 4
a)
Contract Account
For the period 1-1-2013 to 30-9-2013
Particulars Rs. Particulars Rs.
To Materials issued 774,300 By Bank – Materials sold 10,000
To Labour 1,079,000 By P/L A/c – Loss 3,500
Add: Outstanding 102,500 1,181,500 By Material in hand 75,800
To Salary to engineer (20,500 × 9) 184,500 By Cost of contract c/d 2,639,000
To Salary to supervisor (9,000 × 9 × ¾) 60,750 (balancing figure)
To Admn. expenses 460,000
Less: Prepaid 10,000 450,000
To Depreciation on plant (WN:1) 77,250
2,728,300 2,728,300
To Cost of contract b/d 2,639,000 By Work-in-Progress:
To Notional Profit c/d 270,750 Work certified 50%
(balancing figure) of Rs. 4,500,000 2,250,000
Work uncertified
659,750
(WN:2)
2,909,750 2,909,750
To P/L A/c (WN:3) 160,444 By Notional Profit b/d 270,750
To Reserve
110,306
(balancing figure)
270,750 270,750
Working Notes:
1. Calculation of depreciation on plant:
Rs.
Cost of plant 771,000
Less: Residual value 50,000
Depreciable amount 721,000
Estimated life 7 years
Depreciation per annum 103,000
Depreciation for 9 months [Rs. 103,000 × 9 / 12] 77,250

2. Cost of work uncertified


Cost of contract completed is Rs. 2,639,000 which represent 2/3rd of the contract. Hence, cost of
100% of the contract will be (Rs. 2,639,000 / 2 × 3) = Rs. 39,58,500.

JTN P.T.O.
(36)
Therefore cost of 50% of the contract certified is 19,79,250. Hence, cost of works uncertified =
2,639,000 - 19,79,250 = 6,59,750

3. Calculation of Profit to be transferred:


2 Cash Re ceived
Notional Pr ofit  
3 Work Certified
=Rs. 270,750× (2/3)×(2,000,000/2,250,000)
=Rs. 160,444

b)
i. Computation of break-even level for both the machines
 Machine M1: Fixed cost Rs.2, 40,000; The variable cost and contribution per unit will be:

Particulars Amount Rs.


Installed capacity – 10000 units
Fixed overheads 2,40,000
Estimated profits 1,60,000
Total contribution [Fixed overheads + Estimated profits] 4,00,000
Sales value: 10000 units X Rs.100 10,00,000
Variable cost [Sales – Contribution 6,00,000
Variable cost per unit 60
Contribution per unit 40
Profit/Volume ratio: Contribution/Sales X 100 40%

Break Even Sales = Fixed Cost / P/V Ratio = Rs.2, 40, 000 / 40%
= Rs.6, 00,000
Break Even Sales = 6,000 units

 Break Even Sales: M2


c)
Particulars Amount (Rs.)
Installed capacity – 10000 units
Fixed overheads 1,00,000
Estimated profits 1,00,000
Total contribution [Fixed overheads + Estimated profits] 2,00,000
Sales value: 10000 units X Rs.100 10,00,000
Variable cost [Sales – Contribution] 8,00,000
Variable cost per unit 80
Contribution per unit 20
Profit/Volume ratio: Contribution/Sales X 100 20%

Break Even Sales: Fixed Cost/Profit/Volume Ratio = Rs.1, 00,000 /20% = Rs.5, 00,000
Break Even Units = 5, 000

ii. The level of sales at which both the machines will earn the same profit
Level of sales at which both machines will earn the same profit will be the difference in fixed
cost/difference in variable cost
= (Rs.2, 40,000 – Rs.1, 00,000) / (Rs.80 – Rs.60)
= Rs.1, 40,000 /Rs.20
= 7000 units

Thus, at 7000 units, the total costs of both the machines will be same and hence they will
earn the same amount of profits.

JTN P.T.O.
(37)

c)
i. Computing EOQ
Let A= annual usage = Normal usage per week ×52 Weeks
= 100×52
=5,200 Kg
O = ordering cost per order =Rs. 100
Cost per unit =Rs. 500/Kg
C= stock holding rate p.a. = 20% p.a.
Q=Re-order quantity
# CS= Carrying cost per unit p.a. = 500×20% = Rs 100

EOQ=√2AO/C
=√2×5,200×100/100
=102 units

ii. Reorder Level = Max. Usage × Maximum Lead Time


=300 Kg/week × 6 weeks
= 1,800 Kg

iii. Maximum Level = Reorder Level + Reorder Qty - (Minimum Usage × Minimum Lead Time)
= 1,800 +102 ( W.N.1) - (50 Kg per week × 4 weeks)
=1,702 Kg

iv. Minimum level = Reorder Level – (Avg or Normal Usage × Average Lead Time)
=1,800 – ((100 × (4+6/2))
= 1,300 Kg

v. Evaluation of price discount offer:


The price discount offer can be decided upon only after comparing the total annual inventory
cost at EOQ & total annual inventory cost at 1500 units of order size.

Total annual inventory cost = Total Purchase Cost + Total Ordering Cost + Total Carrying
cost of Average Inventory.
= (A×C) + (A\Q×O) + 1\2×Q×CS
At Q= 102 units (i.e. EOQ) = (5,200 ×500) + (5200\102×100) + (1\2×102×100)
= 26, 00,000 + 5098+5100
= Rs 26,10,198 (approx.)

At Q= 1500 units = (5,200 × (500-5%)) + (5,200\1500×100) + (1\2×1500 × (500-5%×20%))


=24,70,000 + 347 + 71250
= Rs 25,41,597 (approx.)

If the re-order quantity is fixed at 1500 Kg, the total annual inventory cost will be lower. So the discount
offer should be accepted.

Assumption:
Since, there is no specification about re order quantity, it has been assumed to be equal to EOQ units i.e.
102 Kg/order.

5. Distinguish between: (4×2.5=10)


a) Efficiency Audit and Proprietary Audit
b) Contribution and Profit
c) Cost Allocation and Cost Apportionment

JTN P.T.O.
(38)
d) Job Order Costing and Process Costing
Answer No. 5
a) Efficiency audit and Proprietary audit
Efficiency audit is directed towards the measurement of whether corporate plans have been
effectively executed. It is concerned with the utilisation of resources in economic and most
remunerative manner to achieve the objectives of the concern. It comprises of studying the
plans of organisation, comparing actual performance with plans and investigating the reasons
for variances to take remedial action. For example, the effective utilization of capital in an
organization can be gauged by determining return on capital employed.

Propriety audit is concerned with executive actions and plans bearing on the finances &
expenditure of the company. The cost auditor has to judge:
 Whether the planned expenditure is designed to give optimum results.
 Whether the size and channels of expenditure were designed to produce the best results,
and
 Whether the return from expenditure on capital as well as current operations could be
bettered by some other alternative plan of action.

b) Contribution and Profit


Contribution Profit
It includes fixed cost and profit. It does not include Fixed cost.
Marginal Costing technique uses the concept of Profit is the accounting concept to
contribution. determine profit or loss of a business
concern.
At break-even point, contribution equals to Only the sales in excess of break-even
Fixed cost. points results in profit.
Contribution concept is used in managerial Profit is computed to determine the
decision making. profitability of product and the concern.

c) Cost Allocation and Cost Apportionment


Cost allocation is the process of charging the full amount of an individual item of cost
directly to a cost centre for which the cost was incurred. For example, where separate electric
meters are installed in each of the department, the electricity charges on the basis of
electricity bill can be allocated to the respective departments. Similarly, salary and wages
paid to indirect workers of each department can be allocated to the respective departments.

Cost apportionment on the other hand, is the process of charging the proportion of common
items of cost to two or more cost centers on some reasonable and equitable basis. For
example, where only one electric meter is installed in a factory, the common electricity
charges should be apportioned to all the departments on the basis of number of light points or
floor area. Similarly, if factory rent is incurred for the factory as a whole and benefits all the
departments in the factory, it should be apportioned to all the departments on the basis of
floor area occupied by each department.

d) Job order costing and process costing


Job order costing Process costing
 It is used in industries where production is  Process costing is used in continuous and
carried on according to specific job order. mass production industries producing like
units of standard specification.
 Cost is collected for each individual job  Cost is collected according to processes
worked and department
 Items of prime cost can be traced with job  Items of prime cost cannot be traced with
order a particular order due to continuous
production.

JTN P.T.O.
(39)
 Job cost is computed when job is  Process cost is computed at the end of
completed cost period.
 Costs of one process are transferred to
 The cost of each unit in production is costs of next process, until goods are
separately identified. completely manufactured.

6. Write short notes on: (4×2.5=10)


a) Value Analysis
b) Advantages of Bin Cards
c) Limitations of Uniform Costing
d) Scope of Cost Reduction

Answer No. 6
a) Value Analysis
Value analysis is a technique applied to analyze all aspect of an existing product or
component to determine in minimum cost necessary for specific function requirement. It is
also known as value engineering. The primary advantage of value analysis is reduction in
product cost. It improves sale and customer satisfaction as it determines the exact
requirements of customer and product designing is done accordingly. Steps involved in value
analysis are:
 Collecting relevant information
 Deciding alternatives
 Approval of the accepted alternative
 Execution
 Follow-up

b) Advantages of Bin Cards :


i. There would be fewer chances of mistakes being made as entries will be made at the
same time as goods are received or issued by the person actually handling the materials.
ii. Control over stock can be more effective, in as much as comparison of the actual quantity
in hand at any time with the book balance is possible.
iii. Identification of the different items of materials is facilitated by reference to the Bin Card
the bin or storage receptacle.

c) Limitations of Uniform Costing


(i) Sometimes it is not possible to adopt uniform standards, methods and procedures of
costing in different firms due to differing circumstances in which they operate. Hence,
the adoption of uniform costing becomes difficult in such firms.
(ii) Disclosure of cost information and other data is an essential requirement of a uniform
costing system. Many firms do not wish to share such information with their competitors
in the same industry.
(iii)Small firms in an industry believe that uniform costing system is only meant for big and
medium size firms, because they can't afford it.
(iv) It induces monopolistic trend in the business, due to which prices may be increased
artificially and supplies withheld.

d) Scope of Cost Reduction


Cost reduction is attainable in almost all areas of business activities. There is perhaps no
situation which cannot be improved. It covers a wide range like new layout, product design,
production methods, material substitution, improved tool design, better utilization of men,
materials and machines in factories as well as in offices, innovation in marketing etc. It also
extends to specified activities like purchasing, handling, packaging, shipping, warehousing,
marketing, use of administrative facilities and even utilization of financial resources.
Excessive cost may result in every organization from:
JTN P.T.O.
(40)
(i) Lack of information about raw materials, processes, products, components etc.
(ii) Lack of utilization of ideas generated from performance and economic analysis.
(iii)Honest but wrong beliefs that certain things are impossible of achievement.
(iv) Temporary circumstances like features developed under pressure or modifications made
to meet certain circumstances.
(v) Habits and attitudes confining one to conventional methods.

It is not necessary for management to proceed in any specific sequence in considering the
various aspects of cost reduction and it may be necessary to start the campaign in more than
one direction at the same time.

JTN P.T.O.
CAP-II, Financial Management, Dec 2013
Suggested Answer

Roll No……………. Maximum Marks - 100


Total No. of Questions – 7 Total No. of Printed Pages – 3
Time Allowed – 3 Hours
Marks
Attempt all questions.

Working notes should form part of the answer. Make assumptions wherever
necessary.

1. The Cash flows of two mutually exclusive projects are as under:

Project Year 0 Year 1 Year 2 Year 3 Year 4 Year 5 Year 6


P (40,000) 13,000 8,000 14,000 12,000 11,000 15,000
J (20,000) 7,000 13,000 12,000 - - -

Required: (5+5+5+5=20)
a) Estimate the net present value of the project "P" and "J" using 15% as
burled rate, and state which project should be chosen.
b) Estimate the internal rate of return of the project "P" and "J", and state
which project should be chosen.
c) Why is there a conflict in the project choice by using net present value and
internal rate of return criterion?
d) What method will you use in such a conflicting situation? Show your
calculation and also make the project choice.

Solution:
a) Estimation of NPV of the projects P and J

Year Discounting Project P Project J


factor @15% Cash flow(Rs.) P.V.(Rs.) Cash flow(Rs.) P.V.(Rs.)
0 1.0000 (40,000) (40,000) (20,000) (20,000)
1 0.8696 13,000 11,305 7,000 6,087
2 0.7561 8,000 6,049 13,000 9,829
3 0.6575 14,000 9,205 12,000 7,890
4 0.5718 12,000 6,862 - -
5 0.4972 11,000 5,469 - -
6 0.4324 15,000 6,486 - -
Net Present Value 5,376 3,806

On the basis of NPV, project P is desirable.

b) Estimation of Internal Rate of Return of projects P and J:


For Project P
Average Cash Flow of Project P is as follows:
(13,000+8,000+14,000+12,000 +11,000+15,000)/6= Rs. 73,000/6= Rs. 12,167

Factor to be located =40,000/12,167= 3.2876

OYX P.T.O.
(2)
The nearest rate of return for 3.2876 in compound value table is shown at 22%. We can use
the cash flow values for discounting at 20% by trial and error method as given in the problem.
While discounting, we get Rs.278 negative as NPV.

Thus the IRR would be as follows:


IRR for Project P = 15+[5376/ (5376+278)] x 5=19.75%

For Project J
Average Cash Flow of Project J is as follows:
(7,000+13,000+12,000)/3= Rs. 32,000/3= Rs. 10,667

Factor to be located =20,000/10,667= 1.875

The nearest rate of return for 1.875 in compound value table is shown at 24%. We can use the
cash flow values for discounting at 26% by trial and error method as given in the problem.
While discounting, we get Rs. 256 negative as NPV.

Thus the IRR would be as follows:


IRR for Project J = 15+ [3,806/(3,806+256)]×11=25.31%

On the basis of IRR, project J is desirable.

c) Reason for conflict in NPV and IRR criterion:


Conflict between the results of NPV and IRR arises:
a) If the project has multiple cash outflows;
b) When there are mutually exclusive projects under consideration;
c) When projects have unequal lives or scale of investment;
d) When projects are borrowing and not lending.

In the given case, above (b) and (c) situations apply.

The selection of project using NPV is more realistic than IRR. The wealth maximization
concept is taken into consideration in NPV method, and it is based on financial manager‟s
judgment.

d) Where there are unequal lives, unequal scale of investment, projects are mutually exclusive
and there arises the conflict in selection of projects using NPV and IRR, we can use
Equivalent Annual Value Method for project selection as follows:

Equivalent Annual Value of Project P


= NPV/Cumulative P.V. of Re.1 p.a. @15% for 6 years
= Rs. 5,376/3.7845=Rs. 1,420.

Equivalent Annual Value of Project J


= NPV/Cumulative P.V. of Re.1 p.a. @15% for 3 years
= Rs. 3,806/2.2832=Rs. 1,667

Selection of Project:
Since the Equivalent Annual Value of Project J is higher than Project P as calculated above,
we may select Project J. It will minimize risk of uncertain future.

2.
a) A company requires Rs. 1,500,000 for the installation of a new unit, which
would yield an annual EBIT of Rs. 250,000. The company‟s objective is to

OYX P.T.O.
(3)
maximize EPS. It is considering the possibility of raising a debt of either Rs.
300,000 or Rs. 600,000 or Rs. 900,000 plus issuing equity shares. The
current market price per share is Rs. 50 which is expected to drop to Rs. 40
per share, if the market borrowings were to exceed Rs. 700,000. The cost of
borrowings are indicated as follows:
Level of borrowings Cost of borrowings
Up to Rs. 200,000 12% p.a
More than Rs. 200,000 to Rs. 600,000 15% p.a
More than Rs. 600,000 to Rs. 900,000 17% p.a

Required: (5+3=8)
i) Assuming a tax rate of 50%, work out the EPS and the scheme, which
you would recommend to the company.
ii) Calculate return on capital employed under each scheme and explain the
leverage effect.

b) JL Ltd. is considering the revision of its credit policy with a view to increase
its sales and profit. Currently, all its sales are on credit and the customers are
given one month‟s time to settle the dues. It has recorded a contribution of
40% on sales. It can raise additional funds at a cost of 20% per annum. The
manager of the company has given the following options along with
estimates for consideration:

Particulars Current Option I Option II Option III


position
Sales (Rs. in millions) 20 21 22 25
Average collection period 1 1.50 2 3
(in months)
Bad debts (% of sales) 2 2 .50 3 5
Cost of credit administration 0.12 0.13 0.15 0.30
(Rs. in millions)

Required: 7
Consider debtors at cost, unless otherwise mentioned, and advise the
company for the best option to implement.
Solution:
a)
i) Statement showing EPS under the different schemes

Particulars Scheme I Scheme II Scheme III


Capital required (Rs.) 1,500,000 1,500,000 1,500,000
Less: Debt Content(Rs.) 300,000 600,000 900,000
Balance Equity Capital required (Rs.) 1,200,000 900,000 600,000
Market Price per Share(Rs.) 50 50 40
Number of Equity Shares to be issued 24,000 18,000 15,000
= shares shares shares
EBIT(given) (Rs.) 250,000 250,000 250,000
Less: Interest on Debt (Rs.)
First Rs. 2,00,000 at 12% 24,000 24,000 24,000
Next up to Rs. 4,00,000 at 15% 15,000 60,000 60,000

OYX P.T.O.
(4)
Balance at 17% - - 51,000
Total interest cost (Rs.) 39,000 84,000 135,000
Earnings Before Tax (EBT) (Rs.) 211,000 166,000 115,000
Less: Tax at 50% (Rs.) 105,000 83,000 57,500
Earnings After Tax (EAT) (Rs.) 105,000 83,000 57,500
EPS (Rs.) 4.375 4.61 3.83

Recommendation:
EPS is maximum under Schemes II and is hence preferable, i.e., raising a debt of Rs. 600,000
and remaining from issue of equity shares.

ii) Since EBIT and capital employed are same in every scheme; i.e. ROCE = 250,000/1,500,000
= 16.67%.

Use of Debt Funds and Financial Leverage will have a favorable effect only if ROCE > Interest
Rate. ROCE is 16.67% and hence up to 15% Interest Rate, i.e. Scheme II, use of debt will have
favorable impact on EPS and ROE. However, when interest rate is higher at 17% (i.e. in Scheme
III, for Debt above Rs. 6 lakhs), Financial Leverage have negative impact and hence EPS falls
from Rs. 4.61 to Rs. 3.83.

b) (Rs. in millions)
Particulars Present Option I Option II Option III
1. Sales 20 21 22 25
2. Variable Cost at 60% of sales 12 12.6 13.2 15
3. Contribution (1-2) 8 8.4 8.8 10
4. Cost of Debtors p.a. =Variable Cost of Sales 12 12.6 13.2 15
5. Average Collection Period (in months) 1 1.5 2 3
( ) ( )
6. Average Debtors outstanding = 1 1.575 2.20 3.75

7. Interest on Average Debtors [ ( )] 0.20 0.315 0.440 0.75


8. Bad Debt [ ] 0.40 0.525 0.660 1.250
9. Cost of Credit Administration 0.12 0.13 0.15 0.30
10. Net Benefit [( ) ( ) ( ) ( )] 7.28 7.43 7.55 7.70

Conclusion:
Option III may be chosen due to maximum Net benefit.

3.
a) You are provided with the following information of Zinc Ltd.:
Fixed assets (After writing off 30% value) Rs. 1,050,000
Fixed assets turnover ratio (on cost of sales) 2
Finished goods turnover ratio (on cost of sales) 6
Gross profit rate on sales 25%
Net profit (before interest) to sales 8%
Fixed charges cover (debenture interest 7%) 8
Debt collection period 1.5 months
Materials consumed to sales 30%
Stock of raw materials (in terms of month's consumption) 3 months
Current ratio 2.4
Quick ratio 1.0
Reserves to capital ratio 0.21
Required: 10
Use the above information and prepare the balance sheet of Zinc Ltd.

OYX P.T.O.
(5)
b) Madhu opened an account on Shrawan 1, 2069 with a deposit of Rs. 800.
The account paid 6% interest compounded quarterly. On Magh 1, 2069, she
closed the account and added enough additional money to invest in a 6-
month time deposit for Rs. 1,000 earning 6% interest compounded monthly.
Required: (2+2+1=5)
i) How much additional amount did Madhu invest on Magh 1?
ii) What was the maturity value of her time deposit on Shrawan 1, 2070?
iii) How much total interest was earned during the period?
(Given that (1+ i) n is 1.03022500 for, i= 1.5%, n=2, and is 1.03037751 for
i=0.5% and n =6)

Solution:
a)
Working Notes:
1. Calculation of cost of sales
Fixed assets turnover ratio = 2 (given)
Cost of sales/Fixed assets =2
Cost of Sales/10,50,000 =2
Cost of sales =2 X 10,50,000 = Rs. 21,00,000

2. Calculation of value of finished goods:


Inventory turnover ratio =6 (given)
Cost of Sales/Finished goods =6
Rs.21,00,000/Finished goods =6
Finished goods =21,00,000/6 = Rs. 3,50,000

3. Calculation of Sales and Gross Profit


Gross Profit ratio =25% (given)
Gross Profit/sales =25%
If cost of sales i.e. Rs.21,00,000 is 75%
Sales value would be 100%
Thus sales=21,00,000 X100/75 = Rs. 28,00,000
Gross Profit would be =28,00,000-21,00,000 =Rs. 7,00,000

4. Calculation of Net Profit:


Net Profit Ratio = 8% (given)
Net Profit/Sales = 8%
Net Profit/28,00,000 = 8%
Thus Net Profit =28,00,000 X 8% = Rs. 2,24,000

5. Calculation of interest charges


Interest service coverage ratio = 8 (given)
Net Profit before interest/ Interest =8
2,24,000/Interest =8
Interest =224,000/8=Rs. 28,000

6. Calculation of value of 7% Debentures:


Interest on debentures @ 7% =28,000
Thus value of debentures = 28,000X100/7 =Rs.4,00,000

7. Calculation of Debtors:
Debt collection period = 1.5 months (given)

OYX P.T.O.
(6)
Debtors/Sales X12 = 1.5
Debtors/28,00,000X12 = 1.5
Debtors =28,00,000X1.5/12 =Rs. 3,50,000

8. Calculation of Material consumption:


Material consumption =30% of sales (given)
=30% of Rs. 28,00,000 =Rs. 8,40,000

9. Calculation of Raw Materials stock:


Raw materials stock = 3 months of material consumption (given)
=8,40,000X3/12 =Rs. 2,10,000

10. Calculation of current assets and current liabilities:


Current ratio =2.4 (given)
Current assets/Current liabilities =2.4
Quick ratio =1 (given)
Liquid assets/Current liabilities =1
Thus value of stock =2.4-1 = 1.4
Value of stock =Finished goods + Raw materials
=3,50,000 + 2,10,000 =Rs. 5,60,000
Thus if 1.4 times is 5,60,000 then
1 times would be = 5,60,000/1.4
=4,00,000
Therefore, current liabilities =4,00,000
Current Assets =4,00,000X2.4 =Rs. 9,60,000

11. Calculation of Cash balance:


Cash balance = Current assets – stock of FG and RM and debtors
=960,000-(3,50,000+2,10,000+3,50,000) =Rs. 50,000

12. Calculation of capital and reserves:


Ratio of reserves to capital =0.21 (given)
If capital is 1
Reserves is 0.21
Net worth would be 1.21
Net worth =12,10,000
Capital is =12,10,000/1.21=Rs. 10,00,000
Reserves would be =12,10,000-10,00,000 =Rs. 2,10,000

After the above calculations, the balance sheet of Zinc Limited would be as under:

Capital and Liabilities Amount(Rs.) Assets Amount (Rs.)


Capital 10,00,000 Fixed Assets 10,50,000
Reserves 2,10,000 Current Assets
Debentures 4,00,000 Debtors 3,50,000
Current Liabilities 4,00,000 Stock (RM and FG) 5,60,000
Cash balance 50,000
20,10,000 20,10,000

b)
i) The initial investment earned interest from Shrawan to Poush, i.e. for two quarters

In this case, i=6/4=1.50%, n=2, P=Rs. 800; and


the compounded amount =800×1.03022500= Rs. 824.18
The additional amount invested on Magh 1= Rs. (1,000 - 824.18)=Rs. 175.82

OYX P.T.O.
(7)

ii) In this case, the time deposit earned interest compounded monthly for 6 months
Here, i =6/12=0.5%, n=6 , P=Rs. 1,000
Required maturity value = 1000×1.03037751= Rs. 1,030.38

iii) Total interest earned = Rs. 24.18 + Rs.30.38= Rs. 54.56

4.
a) Koshi Traders is a growing supplier of office materials. Analysts project the
following free cash flow during the next 3 years of operation of the firm,
after which free cash flow is expected to grow at a constant 7% rate.
Year 1 2 3
Free Cash Flow (Rs. in millions) -20 30 40

The firm's weighted average cost of capital is 13%.


Required: (3+2+2=7)
rd
i) What is the terminal or horizon value of free cash flows after 3 year?
ii) What is the value of the firm today?
iii) Suppose the company has Rs. 100 million in debt and 10 million shares
of stock. What is the price per share?
b) Himal Steels Ltd. requires Rs. 500,000 for construction of a new plant. It is
considering following three alternative financial plans:
i) The company may issue 50,000 ordinary shares at Rs. 10 per share;
ii) The company may issue 25,000 ordinary shares at Rs. 10 per share and
2,500 debentures of Rs. 100 denominations bearing a 8% rate of
interest; and
iii) The company may issue 25,000 ordinary shares at Rs. 10 per share and
2,500 preference shares at Rs. 100 per share bearing a 8 % rate of
dividend.
The different possible level of earnings before interest and taxes (EBIT) of
Himal Steels Ltd. are Rs. 10,000; Rs. 20,000; Rs. 40,000; Rs. 60,000 and
Rs. 100,000. The applicable tax rate is 50%.
Required: (6+2=8)
i. Compute the earnings per share under each of the three financial plans
and different levels of EBIT.
ii. Which alternative would you recommend for the company and why?

Solution:
a)
i) Terminal or Horizon Value of FCF after 3rd year:

= Free Cash flow of 3rd year(1+g)


(WACC-g)
= 40(1+0.07)
0.13- 0.07
= Rs. 713.33 Million

ii) Calculation of Value of the firm today


OYX P.T.O.
(8)
FCF/ Terminal value
Year (Rs. in millions) PVIF @ 13% PV (Rs. in millions)
1 -20 0.8850 (17.70)
2 30 0.7831 23.493
3 40 0.6931 27.724
3 713.33 0.6931 494.41
Value of the Firm today 527.927

iii) Calculation of price per share


Value of Common Equity = Value of Firm today – Value of debt
=527.927 Million-100 Million
=Rs. 427.927 Million

Price per share = Value of Equity


No of Equity Shares

= Rs. 427.927 Million


10 Million
Price per share = Rs. 42.7927
b)
i) Earnings per share under three financial plans for Himal Steel Limited

First Financial Plan- Issue of equity shares only


Particulars Rs. Rs. Rs. Rs. Rs.
EBIT 10,000 20,000 40,000 60,000 100,000
Interest 0 0 0 0 0
PBT 10,000 20,000 40,000 60,000 100,000
Taxes @50% 5,000 10,000 20,000 30,000 50,000
PAT 5,000 10,000 20,000 30,000 50,000
No of shares 50,000 50,000 50,000 50,000 50,000
EPS 0.10 0.20 0.40 0.60 1.00

Second Financial Plan- Issue of Equity Shares and Debentures


Particulars Rs. Rs. Rs. Rs. Rs.
EBIT 10,000 20,000 40,000 60,000 100,000
Interest 20,000 20,000 20,000 20,000 20,000
PBT (10,000) 0 20,000 40,000 80,000
Taxes @50% (5,000) 0 10,000 20,000 40,000
PAT (5,000) 0 10,000 20,000 40,000
No of shares 25,000 25,000 25,000 25,000 25,000
EPS (0.20) 0.00 0.40 0.80 1.60
It is assumed that the company will be able to set off losses against their profit. If the
company has no profits from other operations, losses will be carried forward.

Third financial plan- Issue of Equity shares and Preference shares


Particulars Rs. Rs. Rs. Rs. Rs.
EBIT 10,000 20,000 40,000 60,000 100,000
Interest 0 0 0 0 0
PBT 10,000 20,000 40,000 60,000 100,000
Taxes @50% 5,000 10,000 20,000 30,000 50,000

OYX P.T.O.
(9)
PAT 5,000 10,000 20,000 30,000 50,000
Preference 20,000 20,000 20,000 20,000 20,000
dividend
PAT for ordinary 0 0 0 10,000 30,000
shareholders
No of shares 25,000 25,000 25,000 25,000 25,000
EPS 0 0 0 0.40 1.20

ii) The choice of financial plan will depend upon the economic condition. If Himal Steel
Limited‟s sales are increasing, the earnings per share will be maximum under second
financial plan under favourable conditions, debt financing gives more benefit than equity or
preference capital because interest on debt is tax deductible while preference dividend is
not.

5.
a) A company belongs to a risk class for which the approximate capitalization
rate is 10%. It currently has 25,000 shares outstanding, selling at Rs. 100
each. The company is contemplating the declaration of a dividend of Rs. 5
per share at the end of the current financial year. It expects to have a net
income of Rs. 250,000 and has a proposal for making new investments of
Rs. 500,000.
Required: 7
Show that under MM assumptions, the payment of dividend does not affect
the value of the firm.
b) The present capital structure of the Shree Ram Mills Ltd. is as follows:
Rs. in millions
Equity shares (face value Rs. 10) 240
Reserves 360
11% Preference shares (face value Rs. 10) 120
12% Debentures (face value Rs. 100) 120
14% Term loans 360
Total 1,200
Following additional information is available:
The company's equity beta 1.06
Yield on long term treasury bonds 10%
Stock market risk premium 6%
Current ex-dividend equity share price Rs. 15
Current ex-dividend preference share price Rs. 12
Current ex-interest debenture market value Rs. 102.50
Corporate tax rate 40%
The debentures are redeemable after 3 years and interest is paid annually.
Required: 8
Ignore floatation costs and calculate the company's weighted average market
value cost of capital.

Solution:
a)
A. Value of the firm, when Dividends are paid
I. Price per share at the end of year 1

OYX P.T.O.
(10)
1
(D1 P1)
( )
1
(Rs 5 P1)
1 10
110 = Rs.5 +P1 or P1= Rs. 105

II. Amount required to be raised from the issues of new shares


=Total required investment – (Net income – Dividend paid)
=Rs,500,000- ( Rs.250,000- Rs.125,000)= Rs.375,000

III. Number of additional shares to be issued,


=Rs. 375,000/Rs. 105
=75,000/21 shares
Value of the firm
( ) ( )

Where, P1 = Market price


I=Investment
E=Earnings
Now,
Value of the firms
[ ] ( )

=2,750,000/1.10
=Rs. 2,500,000

B. Value of the firm when dividends are not paid


Price per share at the end of the year 1, Rs.100=P1/110, or P1=110
Amount required to be raised from the issue of new shares = Rs. 500,000 – Rs. 250,000
=Rs. 250,000
Number of additional share to be issued = Rs.250,000/Rs.110= 25,000/11 shares

Value of the firm


[ ] ( )

= Rs.2,750,000/1.1
= Rs. 2,500,000

Conclusion:
Thus, whether dividends are paid or not, value of the firm remains the same under MM
assumptions. Thus, shareholders are indifferent between the retention of profits and the payment
of dividend

b)
Cost of Equity share capital under capital assets pricing model
Ke = Rf + β (Rm-Rf)
=0.10+1.06(0.06)= 0.1636=16.36%

Cost of Preference Share capital


Kp =Dp/Po = 1.10/12= 0.0917= 9.17%

OYX P.T.O.
(11)
Cost of Redeemable Debentures (Kd)
The debentures are redeemable after 3 years and interest is paid annually. The current ex-interest
debentures market value is Rs. 102.50, which represents present value of stream of future cash
flows in the form of interest and maturity value.
Therefore, pretax cost of debenture is:
Approximate YTM ={ I +(M-V0)/n}/ (M+2V0)/3
= {12+(100-102.5)/3}/(100+2*102.5)/3
= {12-0.8333}/101.6667
= 11.1667/101.6667
= 0.1098 = 10.98% or 11%

Thus kd= 11%

Cost of Long Term Loan (Kt)


= I(1-t) = 14%(1-0.40)= 8.40%

Rs. in million
Equity Share Capital (Rs.240 million/Rs. 10) X15 360
11% Preference Share Capital (Rs.120 million/Rs. 10) X12 144
12% Debentures (Rs.120 million/Rs. 100) X 102.50 123
Term Loan 360
Market Value of total Capital 987

Source Market Value Weight Pre-tax cost Post-tax cost Weighted Cost
Rs. In million (%) (%) (%)
Equity Share 360 0.365 16.36 16.36 5.97
Capital
Preference 144 0.146 9.17 9.17 1.34
Share Capital
Debentures 123 0.124 11.00 6.60 0.82
Term Loan 360 0.365 14.00 8.40 3.07
WACC 11.20%

6. Write short notes on: (4×2.5=10)


a) Limitations of financial ratios
b) Two basic functions of financial management
c) Miller Orr Model of cash management
d) Purpose of working capital management

Answer:
a) Financial ratios have following limitations:

i) Many large firms operate different divisions in different industries. For these companies, it
is difficult to find a meaningful set of industry-average ratios.
ii) Inflation may badly distort a company's balance sheet. In this case, profits will also be
affected. Thus a ratio analysis of one company over time or a comparative analysis of
companies of different ages must be interpreted with judgment.
iii) Seasonal factors can also distort ratio analysis. Understanding seasonal factors that affect a
business can reduce the chance of misinterpretation. For example, a retailer's inventory
may be high in the summer in preparation for the back-to-school season. As a result, the
company's accounts payable will be high and its ROA low.
iv) Different accounting practices can distort comparisons even within the same company,
e.g. leasing versus buying equipment, LIFO versus FIFO, etc..
OYX P.T.O.
(12)
v) It is difficult to generalize about whether a ratio is good or not. A high cash ratio in a
historically classified growth company may be interpreted as a good sign, but could also
be seen as a sign that the company is no longer a growth company and should command
lower valuations.
vi) A company may have some good and some bad ratios, making it difficult to tell if it's a
good or weak company.

b) Financial management deals with the procurement of funds and their effective utilization in
the business. The first basic function of financial management is procurement of funds and the
other is their effective utilization.
i) Procurement of Funds: Funds can be procured from different sources. The procurement
is a complex problem for business concerns. Funds procured from different sources have
different characteristics in terms of risk, cost and control.
ii) Effective utilization of Funds: Since all the funds are procured at a certain cost, it is
necessary for the finance manager, to take appropriate and timely actions so that the
funds do not remain idle. If these funds are not utilized in manner so that they generate an
income higher than the cost of procuring, then there is no point in running the business.

c) The Miller and Orr model of cash management is one of the various cash management models in
operation. It is an important cash management model as well. It helps the present day companies to
manage their cash while taking into consideration the fluctuations in daily cash flow. As per the Miller
and Orr model of cash management, the companies let their cash balance move within two limits -
the upper limit and the lower limit. The companies buy or sell the marketable securities only if the
cash balance is equal to any one of these. When the cash balance of a company touches the upper
limit, it purchases a certain number of salable securities that helps them to come back to the desired
level. If the cash balance of the company reaches the lower level, then the company trades its
salable securities and gathers enough cash to fix the problem. It is normally assumed in such cases
that the average value of the distribution of net cash flows is zero. It is also understood that the
distribution of net cash flows has a standard deviation. The Miller and Orr model of cash
management also assumes that distribution of cash flows is normal.

d) There are two important aims of the working capital management which are profitability and
solvency. A liquid firm has less risk of solvency i.e. it will hardly experience a cash shortage
or stock out situation. However there is a cost associated with maintaining a sound liquidity
position. However, to have higher profitability the firm may have to sacrifice solvency and
maintain a relatively low level of current assets. This will improve firm‟s profitability as
fewer funds will be tied up in idle current assets but its solvency would be threatened and
exposed to greater risk of cash shortage and stock out. Therefore, optimal level of working
capital need to be maintained so that profit is high and risk of solvency is low. This is also
known as risk return trade off of liquidity management.

7. Distinguish between: (4×2.5=10)


a) Promised yield and Realized yield
b) Funds flow statement and Cash flow statement
c) NPV and IRR
d) Risk and Uncertainty

Answer
a) Promised Yield indicates the total rate of return earned on bond if it is held to maturity. It is
also known as Yield-to-Maturity. This is the rate of return anticipated on a bond if held until
the end of its lifetime. YTM is considered a long-term bond yield expressed as an annual rate.

OYX P.T.O.
(13)
The YTM calculation takes into account the bond‟s current market price, par value, coupon
interest rate and time to maturity. It is also assumed that all coupon payments are reinvested at
the same rate as the bond‟s current yield. YTM is a complex but accurate calculation of a
bond‟s return that helps investors to compare bonds with different maturities and coupons.
Realized Yield is the actual amount of return earned on a security investment over a period of
time. This period of time is typically the holding period which may differ from the expected
yield at maturity. The realized yield also includes the returns that have been earned from
reinvested interest, dividends and other cash distributions.
The realized yield tends to differ from the yield at maturity in scenarios where the holding
period is less than that of the maturity date. In other words, the security is settled or sold prior
to the maturity date given at the time of purchase. For example, suppose an investor purchases
a 10-year bond for Rs. 1,000 that issues a 5% annual coupon. Furthermore, if the investor sells
the bond for Rs.1,000 at the end of the first year (and after receiving the first coupon
payment), his realized yield would only include the Rs. 50 coupon payment.

b) Funds Flow statement ascertains the changes in the financial position between two accounting
periods. It analyses the reasons for change in financial positions between two balance sheets.
It reveals the sources and application of funds. It helps to test whether working capital has
been effectively used or not.
Cash Flow statement ascertains the changes in balance of cash in hand and cash at bank
between two dates. It analyses the reasons for changes in cash and bank balance on a
particular date. It shows the inflows and outflows of cash. It is an important tool for short term
analysis. The two significant areas of analysis are cash gernerating efficiency and free cash
flows.

c) Both NPV and IRR are techniques of capital budgeting decision. Under the NPV rule, we
discount the cash flows at a given rate which is normally the cost of capital and arrive at NPV.
Whereas, in IRR, we find a discount rate that makes NPV zero and compare this rate with the
cost of capital.
NPV is absolute measurement while IRR is a relative measurement of the project‟s worth.
NPV shows the project‟s worth in monetary term whereas IRR does in terms of rate of return
on investment. Theoretically, NPV shows how much the market value of the firm will rise if
projects are accepted and IRR shows what rate of return will the project yield if it is a
accepted. NPV assumes that cash flows are reinvested at required rate of return and IRR
assumes that they will be reinvested at project rate of return.

d) In common parlan, the terms „Risk‟ and „Uncertainty‟ have synonymous meaning. However,
they differ from each other.
Risk may be defined as “the chance of future loss that can be foreseen”. In other word, in
case of risk an estimate can be made about the degree of happening of the loss. This is usually
done by assigning probabilities to the risk on the basis of past data and the probable trends.
Uncertainty may be defined as” the unforeseen chance for future loss or damages.” In case of
uncertainty, since the firm cannot anticipate the future loss, and hence it cannot directly deal
with it in its planning process, as is possible in the case of risk. For example, a firm can not
foresee the loss which may be due to destruction of its plant in account of earthquake.

OYX P.T.O.
CAP-II, Business Communication, Dec 2013
Suggested Answer
Roll No……………. Maximum Marks – 100
Total No. of Questions - 10 Total No. of Printed Pages -2
Time Allowed – 3 Hours
Marks
All questions are compulsory.

Section -'A'

1. Read the following case carefully and answer the questions given below: (5+5=10)
Assume that you are the head of the department of loan in Prabhu Commercial Bank.
Barahi Consultancy is one of the regular debtors of your bank. The institution had
been paying the regular installments of the loan. However, for the last two months the
consultancy has not paid the two installments of the loan to your bank despite having
them learn about it through reminder letters twice earlier.
a) As an authority of the bank, write a reminder letter to the director of Barahi
Consultancy, reminding him/her about the payment based on the case.
b) As an assistant accountant of Barahi Consultancy, write a reply to the reminder
letter of Prabhu Bank.
Answer:
a)
Prabhu Commercial Bank
Central office
New Road, Kathmandu

Date: April 20, 2013


Ref no: 144/013

Mr SP Gupto
Managing Director
Barahi Consultancy
Putalisadak, Kathmandu.

Dear Mr Gupto,
This is to remind you that your payment for last two installments has already delayed.
Though we reminded you about this twice before, you have not taken any action and
response to this. We understand that you are one of the most sincere and ethical business
partners of ours. However, from this case we are obliged to think that you have
overlooked this being concerned much about your business. We have decided to remind
you about this issue once through this letter and take necessary actions if you deny to
respond.
We still hope that you will understand the situation and co-operate us with payment at
your earliest convenience. Please let us know about your intentions.

QCS P.T.O.
(15)
Yours sincerely
………………..
Romesh Karn
Head, Department of Loan

b)
Barahi Consultancy
Putalisadak, Kathmandu

April 22, 2013

Mr Romesh Karn
HoD
The Department of Loan
Prabhu Commercial Bank
Kathmandu

Dear Mr Karn
We regret to the latest transaction that we with some internal obstacles of our house are
lacking to manage well. We are very honestly accepting the facts of your reminder letter.
Since we are reaching the conclusion of the talk with our staff union, we are going to
settle our internal problem soon, and we will prioritize your payment.
We feel sorry about the inconvenience that we have caused, and we commit for earliest
settlement of the instalment.

Thanking you,

Sincerely,
…………….
Suban Adhikari
Assistant Account Officer

2. Why is it important for business persons to develop the intercultural communication


skills? 10
Answer:
The globalization process is forcing business people to rethink their strategies.
Intercultural communication skills assume an ever larger role in global marketing
and sale strategies. Consequently, language programs need to respond to these
changes. Future business managers must acquire effective intercultural competence.
The one world market has forced businesses to think global, act local, and integrate.
In this pretext, intercultural communication serves a vital role since it helps to avoid
miscommunication, prevent misunderstandings, and avert mistakes.
Three trends are working together to crystallize the growing need for developing
multicultural sensitivities and improved communication techniques: (1) globalization
of markets, (2) technological advancements, and (3) a multicultural workforce.
(Students may include the following details)
a) Globalization of Markets: --Doing business beyond our borders is now
common place. National boundaries mean less as business expands through
mergers, alliances, and acquisitions.
QCS
(16)
--Favorable trade agreements and the growth of the middle class fuel the
expansion of global markets

b) Technological advancements: -- Advancements in transportation and


information technologies contribute to global interconnectivity.
--Companies use the Web to sell products, provide support, offer service,
investigate the competition, and link to suppliers.

c) Multicultural workforce: -- Immigration makes cross-cultural communication


skills increasingly necessary. People are on the move. Lured by the prospects
of peace, prosperity, education, or a fresh start, persons from many cultures
are moving to countries promising to fulfill their dreams.

3. Why and how do businesses communicate with their customers? (5+5=10)


Answer
Businesses communicate with their customers for many reasons, including gaining
feedback on satisfaction, answering queries and providing updates on new products
or events. In order to do so, a number of interaction techniques can be employed,
ranging from the highly sophisticated to the fairly conventional.
For promotional purposes, the most common form of contact is a letter or leaflet sent
to the customer‟s address. This is straightforward way of informing the customer of
new developments.
Lately, there has been a growth in the popularity of e-mail messages to customers.
Furthermore, videos can be transmitted electronically, allowing viewers to watch
product demonstrations or interviews from the comfort of their own homes. It is now
possible to upload a video to YouTube.
Increasingly, firms have become more reliant upon websites as another way of
“talking” to customers. As well as being an excellent advertisement tool, a website
can be used to track customers‟ buying habits.
Organizations wishing to measure their customers‟ satisfaction can distribute
questionnaires, or provide them online. By asking a range of questions, they can use
feedback to improve future performance and deter from purchasing rival products.
To provide a more personal means of communication, most businesses operate
telephone services. The main benefit of the telephone is that it can assist both
parties; businesses can call clients to gain opinion or promote events, and customers
can call businesses to ask questions or express interests.
4. Team work is assumed to be effective in the success of a given task in an
organization. Drawbacks of team work are also pointed out by the practitioners of the
field. Explain in brief five advantages and five drawbacks of group or team work in
the business organizations. (5+5=10)
Answer
Groups are formed in the business sector with a motto that a defined goal should be
fulfilled by exploring the potentials of the members of the group. The professionals,
experts, technical minded people, etc may gather with a thought of initiating some
entrepreneurial work, and this is how groups or teams are formed in the business
organizations. However, a mere aggregation of people does not make a group in
business. They need to possess a common goal related to some business activities.
They need to be bond with some institutional rules and systems. A well formed
business group possesses a number of merits since collaboration of ideas, skills, and

QCS
(17)
talents always work better than the single one. Some of the advantages of team work
(or group work) include:
 Group for mutual feedback, collaboration and reinforcement
 Groups offer outlets for creative and innovative expressions of the members
 Group for increased mutual accountability and collective achievement
 Group enhances accessibility to increased sources of information
 Group for diversity of views and alternatives
Despite a number of advantages, a group is supposed to have some disadvantages or
drawbacks too. Some of the striking drawbacks of group in business are:
 Sacrifice of individual capacity, identity and distinctiveness
 Unwanted pressure and rigidity of principles
 Conflicts and misleading procedures
 Lack of accountability
 Consensus problem in heterogeneous groups

5. Briefly explain any FOUR of the following: (4×2.5=10)


a) Major parts or components of a report
b) Communicating nonverbally
c) Preparing for interview
d) Communicating in the workplace
e) Roles of audio-visual material in oral presentation
Answer:
a) Major parts or components of a report
A report is generally prepared after carrying out a research or study, being based
on its specific issues and essence including objectives, methodology and findings.
The basic components or parts of a complete report may include: title page, cover
letter, table of contents, list of illustrations, abstract, body of report, bibliography,
and appendices. The body includes introduction, objectives, problem statement,
methodology, discussion, findings, recommendations, and conclusion. The cover
page is meant for exhibiting the information on the title, researcher or report
writer, institution where the report is submitted, and date of submission. The table
of contents and the list of illustrations obviously present the contents, figures,
tables, and charts involved inside the report. Abstract is the summary of the entire
report. The body of report includes different information in detail but each dealing
needs to be specific, brief, and systematic. Finally, in the appendices, the data and
other relevant materials that cannot be kept in the report but are important to
understand the essence of the report are kept.
b) Communicating nonverbally
Nonverbal messages are sent by our eyes, face, and body. For example, sustained
eye contact indicates trust or admiration; brief eye contact may signify fear or
stress. Expressions on a communicator‟s face can supplement or entirely replace
verbal messages. Posture can indicate status, confidence, shyness, or
submissiveness. Gestures also send nonverbal messages, many of which are
culture dependent. Moreover, how a communicator uses time, space, and territory
sends messages that require no words. The amount of space we need for social
interaction can be another means of sending messages nonverbally.
c) Preparing for interview
An interviewee is required to have a good preparation for the interview. Particularly, in the job
interview he or she needs to persuade the interviewer(s) by exhibiting proper confidence, power

QCS
(18)
to deal with problems, ability of contents, and so on. Generally, the interviewee needs to focus
on the following things;
1. Do research about the company: The interviewee needs to be equipped with the relevant
information about the company. For this, he or she may conduct a research like task that
helps him or her to find out the appropriate information.
2. Plan your schedule: Planning about the time table is also an important preparation for an
interview. An interviewee needs to be punctual.
3. Dress up appropriately: Normally, formal dress up is proffered in an interview. The dress
up needs to be suitable to one‟s personality too.
4. Rehearse and prepare: Rehearsing is an important activity before interview. One can ask
and answer himself or herself just like in the interview. Or, one can ask his friend or any
person to act as interviewer. This promotes confidence in the candidate.

d) Communicating in the workplace


Communication in the workplace can be accomplished by many means, including:
 Signs and posters
 Letter and written notices from management
 Formal presentations to groups of employees
 group meetings and discussions
 training sessions
 observing experienced employees
 one-on-one conversations
 written memos and reports
e) Roles of audio-visual materials in oral presentation
Oral presentation needs to be highly systematic and comprehensible to the
audience. Unlike in the case of written presentation, the presenter gets very few
opportunities for thinking, acting, editing, and presenting in the oral presentation.
So, a comprehensive plan and equipment are required by the oral presentation.
The audio visual materials play important roles in making the presentation more
effective and meaningful to its audience. When the presenter is speaking
continuously, it may be monotonous, boring, and incomprehensible for the
listeners. They could be demotivated toward the essence of the presentation. But,
if the presentation is accompanied with some audio visual materials such as in
the computer, or in the video, then such materials may attract the attention of the
audience, and the presentation may be more meaningful to them. Additionally,
audio visual materials help people develop proper schemata about the given
concept.

QCS
Section -'B'
CAP-II-Marketing-Sugg.Dec.2013
6. Read the following case carefully and answer the questions given below:
Wai Wai brand of instant noodles is produced by Chaudhary Group in Nepal under
technical collaboration with Thailand. Out of many brand of noodles in Nepalese
market Wai Wai is commanding about 50% share of Nepal‟s market which is
increasing by about 10% per annum. Recently due to competition from Mayos, Rara,
2 PM, Maggi brands, Wai Wai has only about 40% of the domestic market share. In
India, it has been commanding over 50% market share. It has started to expand the
market to other South Asian countries as well.
Wai Wai has 38 distributors in 20 states of India and a 50 member sales force. In
addition, Nepali Youths studying in India act as "Wai Wai ambassadors" to back its
marketing drive. The brand is at present available in Gujarat, Karnataka and Tamil
Nadu. Now it is being expanded to the markets of six more Indian States. Wai Wai
noodles of Indian Rs. 400 million are being exported to India. Wai Wai is already a
market leader in Bhutan and also sells well in Tibet. The next expansion is scheduled
for Bangladesh and Pakistan.
To capitalize on this opportunity to expand trade, heavy marketing expenses and
innovative promotional ideas are essential. Looking at competitors in Indian market,
Chaudhary Group also plans to invest handsomely in advertising the product over
India`s electronic media besides using brand ambassadors.
Questions:
a) Assess the marketing strategy of Chaudhary Group. 5
b) Give your opinion about the prospects of noodles industry in Nepal. 5
Answer:
a) In the given case Chaudhary Group of Industry(CGI) has been found practicing market oriented
strategic planning. The following factors can be regarded as marketing strategy of wai wai noodles
in the national and international market.
- CGI has successfully penetrated its brand of noodles to Nepalese as well as Indian and South Asian
market as a step by step process.
- CGI has given proper attention to it‟s marketing mix in which it has well expansion program as
well as promotional program.
- Satisfactory marketing mix has made wai wai popular and its demand has increased fast.
- CGI has given due attention to it's strengths, weaknesses, opportunities & threats by scanning the
market environment.
- CGI has established its position in national as well as international market.
- Use of youth studying in India as brand ambassadors & use of electronic media are also parts of
marketing strategy.
At the end we can say CGI has prepared a good business plan to conduct successful business.

b) The prospects in favor of noodles industry in Nepal are:


- Numerous brands of noodles have shown their ability to have a profitable sale. Success of CGI can
be taken as illustration.
- Noodles industries are doing business analysis as well as SWOT analysis. Volume of production,
sales & profit of different brands & their market share/size is increasing.
- Comparatively CGI has been able to capture larger share in national market & is making profitable
sale in international market.

ISV P.T.O.
(20)
- If satisfactory marketing mix is developed, definitely noodles products would have bright future
prospects.
If good business plan is prepared with effective product position, differentiation strategy, proper
product life cycle strategy & strategic control which social responsibilities, noodles industry in
Nepal has bright prospects.
7.
a) What is marketing? Explain the selling concept of marketing. (2+3=5)
b) What is marketing research? Give your opinion about the essentials of marketing
research process. (2+3=5)
Answer:
a) Marketing activities are all activities associated with identifying the wants and needs of target
market of customers, and satisfying those customers better than their competitors. This involves
doing marketing research about customers, analyzing their needs and making strategic decisions
about product design, pricing, promotion, and distribution. Actually, marketing deals with
identifying and meeting human and social needs.
Thus, marketing is meeting customer needs profitably.
Selling concept holds that consumers and businesses, if left alone, will ordinarily not buy enough
of the organization‟s products unless it undertakes a large-scale selling and promotion effort.
Therefore the organization must undertake an aggressive selling and promotion effort. The selling
concept is practiced most aggressively with unsought products, products that buyers normally do not
think buying, such as insurance and encyclopedias. Most firms practice the selling concept when
they have overcapacity. Their aim is to sell what they make rather than make what the market wants.
Such marketing carries high risk. It focuses on creating sales transactions rather than on building
profitable relationships with customers. It assumes that consumers who are coaxed into buying the
product will like it. Most studies show that dissatisfied customers do not buy again. whereas the
satisfied customer tells others about good experiences. The dissatisfied customer tells others about
bad experiences.
b) Marketing research is an objective and systematic search for analysis of information relating to the
identification and solution of marketing problems. The main objective of marketing research is to
provide most relevant, internal as well as external, information or most pertinent information to the
marketing manager for making suitable marketing plans and decisions.
A good marketing research should have the following essential features:
 Objectivity is important requirement of marketing research. The main focus is that the marketer
should be free of bias to identify the real problem.
 It should be a systematic search i.e., it should follow step-by-step process like in scientific
research.
 It should be informative i.e., marketing research is a game of information that should collect
adequate information and disseminate adequate messages to the decision-maker to make right
decisions.
 Marketing research should start with the identification of real marketing problem so that it can
provide pertinent solution to the problem.
 Marketing research is actually a decisive search; it provides decision support information to the
marketing decision-makers.

8.
ISV
(21)
a) What is promotion? Explain its objectives. (2+3=5)
b) What is product life-cycle in marketing? Explain the major contributions of
product life-cycle that a marketer must understand to make business successful
and sustainable. (2+3=5)
Answer:
a) Promotion is one of the major elements of marketing mix which provides valuable information to
consumers about product, its price, availability, utilities and benefits. Promotion consists of various
activities that facilitate exchanges with target customers through persuasive communication to
stimulate the demand of the product.
According to Philip Kotler, “Promotion includes all the activities the company undertakes to
communicate and promote its products to the target market.”
Promotion includes various types of activities such as advertising, sales promotion, personal selling,
publicity and public relation to inform, persuade, remind and reinforce the target market about the
company‟s offerings. These activities influence the customer‟s feelings, belief and behavior for
buying.
The major objectives of promotion are informing, persuading, reminding and reinforcing the
customer about the product.
i. Informing:
The main objective of promotion is to inform the market about product, price, availability, utilities
and benefits. It helps to develop awareness about the product It also provides alternatives to the
customers for purchase decision.
ii. Persuading:
Promotion persuades customers to make the purchase decision in favor of the promoted brand.
Promotion is persuasive communication. It stimulates product demand through appealing ads,
incentives and benefits. Promotion influences, buyer behavior. Promotion also persuades middlemen
to carry such product.
iii. Reminding:
Customer normally have short-lived memory. It is impossible for customers to remember all
advertised brands. So, marketer reminds customers about the product by using various promotional
tools. The marketer assumes that customer may forget unless they are constantly reminded.
iv. Reinforcing:
Promotion is equally important in post purchase stage of the buying process. Consumer may feel
anxiety after their purchase decision. Marketer often provides reinforcing message to increase
customer‟s satisfaction level. Repeated reinforcement also leads to brand loyalty. By reassurance to
customer, marketer tries to reduce cognitive dissonance and build brand and corporate image.
b) The product life-cycle is a concept that attempts to describe a product‟s sales, profits, customers,
competitors, and marketing emphasis from its beginning until it is removed from the market. The
product life-cycle concept is used as the strategy to sustain the firm‟s product in the market by
adopting suitable strategies relating to marketing mix, competitive effort, investment, and other
essential tools whenever and wherever necessary.
Product life-cycle (PLC) concept can be beneficial to the marketers for making their business
success and sustainable in the market in the following ways;
 The PLC concept gives messages to the marketers that all products have limited life like a living
creature; they may be dead if appropriate strategies are not adopted at appropriate time period.
 A product passes through five stages – introduction stage, growth stage, maturity stage,
saturation stage and declining stages. These are the various stages of product life-cycle.
 It also communicates to the marketers that product sales pass through distinct stages, each posing
challenges to the seller. Even a popular brand may have to pass through several conditions and
threats.

ISV
(22)
 Products cannot guarantee good profits on regular basis; it may rise and fall at different stages of
the PLC depending upon the changed situation.
 Products require different marketing strategies at different stages of their life-cycle to make them
successful and sustainable in the market.
Marketers, who understand the product life-cycle concept clearly and before and after the launch of
product in the market, can enjoy a lot of opportunities, reduce the level of business risks, win the
competitive edge and succeed in their business efforts.
9.
a) What is advertising? Explain the importance of advertising to the marketers. (2+3=5)
b) How can economic factors affect the buying decision of an individual buyer?
Explain with the help of suitable examples. 5
Answer:
a) Advertising is any paid form of non-personal presentation and promotion of ideas, goods, or service
by an identified sponsor. It is a cost effective way to disseminate message to scattered large masses.
Advertising can be used to give information, persuading consumers, reminding and reassuring them
for creation of demand.
Advertising today has got a prominent role in the promotion program of all firms/companies. Many
methods, media, agencies are playing active role to meet advertising objectives of the companies. A
lot of people are involved to design attractive advertising messages. Newspaper, journals, posters,
radio, TV, video are a few ad vehicles popularly used by the organizations.
The following variables clarify importance of advertising to the marketers:
 Communication of information/message.
 Creation and extension of demand.
 Introduce new product.
 Educate, persuade and remind.
 Influence to make buying decision.
 Save time and money.
 Help to achieve organizational goals.
 Image building.
 Selling on mass scale.
 Help middle men
 Protection from competition etc.
b) An economic factor indicates the ability and willingness of an individual buyer to buy a product.
Ability and willingness to buy reflects the purchasing power of that buyer. There are many economic
forces that affect the buying decision of an individual buyer. They are:
 Level of regular personal income, including both discretionary income and disposable income.
Higher disposable income increases purchasing capacity of the individual consumer.
 Income of other members of the family i.e., when many of the family members earn income, the
purchasing power of an individual family member becomes better.
 Expectation of income in near future from other sources or income through lottery, or from any
other means. These evidences may improve the purchasing capacity of an individual consumer.
 Availability of liquid assets in the form of bank balance, share investment, government bond, etc.
If the consumer has adequate liquid assets, purchasing capacity can be high even though the
regular income is limited.
 Availability of credit facility in the country. The purchasing capacity may be higher if credit
facilities are available in the country by financial institutions for desired purposes.

ISV
(23)
 Past expenditure habit of the consumer. If the consumer was extravagant in the past, he will
spend more on desired product to maintain his/her standard of living even by selling his/her
property.

10. Briefly explain the following: (5×2=10)


a) Marketing environment
b) Market entry pricing strategy of a firm
c) Niche marketing
d) Market segmentation
e) Sales promotion and its features
Answer:
a) A company‟s marketing environment consists of all the factors and forces outside marketing that
affect marketing management‟s ability to build and maintain successful relationship with target
customers. Successful companies know the importance of constantly watching and adapting to the
changing environment.
The marketing environment surrounds and impacts upon the organization. There are two key
perspectives on the marketing environment, namely the macro environment and the micro
environment. Macro environment consists of political, economic, socio-cultural and technological
forces.
b) Market-entry pricing strategy indicates the pricing strategy to be adopted when a new firm enters the
target market for the first time. Marketers can use two alternative strategies in this situation –
penetration pricing strategy and market-skimming pricing strategy.
Market penetration pricing involves setting low price for the new product in the initial period with a
view to expand the market size and thus obtain a large sales volume and larger market share. When
the market is established in desired size, price will gradually be increased according to increase in
demand for product.
While market-skimming pricing strategy involves setting high price for a new product in the initial
stage assuming that the customers will pay high price for the new product. But when the level of
competition goes on increasing, effort will be made to reduce the price level to meet the competition
and survive in the marketplace.
c) A niche market is more narrowly defined group seeking a distinctive mix of benefits. Marketers
usually identify niches by dividing a segment into sub-segments. For example, the segment of heavy
smokers includes two niches: those who are trying to stop smoking and those who don‟t care.
In an attractive niche, customers have a distinct set of needs. They will pay a premium price to the
firm that best satisfies their needs. The niche is not likely to attract other competitors. The nicher
gains certain economies through specialization if the niche has size, profit and growth potential.
d) Market segmentation is a process of dividing the total market into several smaller groups, such that
the members of each group are similar with respect to the factors that influence demand. Dividing
the total market in distinct smaller groups helps to identify real needs and characteristics of buyers.
If market is heterogeneous, it is difficult to satisfy needs but if divided into homogenous units
market needs can be assessed and fulfilled in a proper manner. In Nepal its application is increasing.
Mass marketing, product variety marketing, individual marketing and target marketing are some of
the common approaches found in dealing with the market. There are two bases for segmenting
customer markets: consumer characteristics and consumer responses.
e) Sales promotion consists of a diverse collection of incentive tools, mostly short term, designed to
stimulate quicker and /or greater purchase of particular products/services by consumers.
Features of Sales Promotion
 Sales promotion includes incentive tools
 Sales promotion is a short term measure
ISV
(24)
 Sales promotion is targeted at quicker and /or greater sales
 Sales promotion is directed either at customers or at the channel members

ISV
CAP-II, Income Tax & VAT, Dec 2013
Suggested Answer
Roll No……………. Maximum Marks - 100
Total No. of Questions - 6 Total No. of Printed Pages -5
Time Allowed - 3 Hours
Marks
Attempt all questions. Working note should form part of the answer.
8. Answer the following with reference to the Indian Income Tax Act, 1961.
a) Define the terms "Person" and "Assessment Year". 2
b) Z furnishes the following particulars of his income earned during the previous
year relevant to the assessment year 2013/14:
i) Interest on German Development bonds (One-sixth is received in India)
Rs. 36,000.
ii) Income from agriculture in Pakistan, received there (later on Rs. 86,000 is
remitted to India) Rs. 3,41,000.
iii) Interest from property in USA received outside India (Rs. 92,000 is used in
Canada for meeting the education expenses of Z's son in Canada and
Rs. 2,48,000 is later on remitted to India) Rs. 3,40,000.
iv) Profits from business in New Delhi and managed from outside India (60
percent of the profit is received outside India) Rs. 92,000.
v) Profits on sale of a building in India but received in Nepal Rs. 18,74,000.
vi) Gift in foreign currency from a friend received in India on September 6, 2012
Rs. 80,000.
Find out the gross total income of Z: 6
(a) if he is resident and ordinarily resident in India
(b) if he is resident but not ordinarily resident in India
c) Explain the salient features of Section 176 of Indian Income Tax Act, 1961 on
'Discontinued Business' wherein income of previous year is not taxable in the
immediately following assessment year. 2
Answer a)
As per section 2(31) the term "Person" includes;
- an individual
- a Hindu undivided family
- a company
- a firm
- an association of persons or a body of individuals, whether incorporated or not
- a local authority; and
- every artificial judicial person, not falling within any of the preceding categories
As per section 2(9), the term "Assessment Year" means the period of twelve months starting from
April 1 of every year and ending on March 31 of the next year.

MHW P.T.O.
(26)
b) Computation of Gross Total Income of Z

Particulars (i) Resident (ii) Resident


and ordinarily but not
resident Rs. ordinarily
resident Rs.
1. Interest on German Development bonds:
 One-sixth is taxable on receipt basis 6,000 6,000
 Five-sixth is taxable in the case of resident and
ordinarily resident on accrual basis 30,000 -
2. Income from agriculture in Pakistan:
 Income accrued and received outside India
341,000 -
3. Income from property in USA received outside India:
 Income received outside India 340,000 -
4. Profits from a business in New Delhi and managed from
outside India:
 Income accrued in India 92,000 92,000
5. Profit on sale of a building in India but received in
Nepal:
1,874,000 1,874,000
 Income deemed to accrue or arise in India
6. Gift in foreign currency:
 Now taken as income 80,000 80,000
Gross Total Income 2,763,000 2,052,000

c)
The salient features of section 176 of Indian Income Tax Act, 1961 are as follows:
i. A business or profession is discontinued in any assessment year.
ii. Income of the business/profession from April 1 of the assessment year (in which the
business/profession is discontinued) to the date of discontinuation may be taxable in the
assessment year in which business/profession is discontinued.
iii. The above income is taxable at the discretion of the assessing officer in the assessment year in
which business is discontinued or it may be taxed in the normal assessment year (i. e. assessment
year immediately following the previous year).
iv. If it is taxable in the assessment year in which the business/profession is discontinued, then it is
chargeable to tax at the rate applicable to that assessment year.
9.
a) Mr. Sinha is an employee of Surya Moon Corporation Ltd. His financial activities
F/Y 2069/70 includes the following:
Basic Salary Rs. 40,000 P.M.
Traveling Allowance (From Home to Office and Back) Rs. 4,000 P.M.
Contribution to approved Provident fund Rs. 4,000 P.M.
Dividend income from Surya Moon Corporation Ltd. Rs. 10,000
Mr. Sinha also deposited equal amount to the same provident fund.
On the occasion of 25th Anniversary of the corporation, the corporation organized
a National 1 Km. race competition in association with the Nation Game
MHW P.T.O.
(27)
Association. Mr. Sinha participated in the race and won the first prize worth
Rs. 50,000. Mr. Sinha has a Life Insurance Policy worth Rs. 2,00,000 and annual
premium paid is Rs. 15,000.
Calculate the total income and tax liability from the employment for the fiscal
year 2069/70, if he selects to be assessed as couple. (2+5=7)
b) Simple Innovations Pvt. Ltd., a manufacturing company received a tax assessment
order for fiscal year 2066/67 on 2069.08.20 from Inland Revenue Office. Total
tax assessed by tax authority is Rs. 75,00,000 which includes disputed tax of
Rs. 54,43,500. Some of the issues stated in the assessment order are as follows:
 Company has capitalized interest cost worth Rs. 19,34,569 (interest till
67.01.10, on loan taken for machinery) to plant and machinery ready to /put to
use in 2067.01.10. As per Section 14(1), interest on loan which is utilized in
the same fiscal year or loan taken for purchase of assets which is utilized in
same fiscal year is allowed as deduction. Thus, interest on assets put to use in
fiscal year 2066/67 is not allowed to be capitalized and need to be removed
from depreciation base.
 Value of opening stock includes Repair & Maintenance Expense of
Rs. 7,56,760 and Closing Stock includes Repair Expense Rs. 6,54, 650. Same
is not allowed and need to be written back.
Management is not satisfied with the assessment order and seeks your
consultancy.
i) Comment on the above issues in light of provisions of Income Tax Act,
2058. (3+3=6)
ii) Company is planning to apply for administrative review. Answer the
following in this regards. (21=2)
(a) Time limit to file application for administrative review.
(b) Amount of deposit to be kept for administrative review.
c) Mohit Shrestha, citizen of Nepal provides rental services/hiring services of Mini
Bus, Power Tiller, Auto Rickshaw and other vehicles. Following is the detail of
his business. Vehicles are owned by him in his individual name for fiscal year
2069/70.
Rates per No. of services Provided Number of
Services (Rs.) during the year vehicle owned
Power Tiller 5,500 220 7
Tractor 5,000 180 11
Auto Rickshaw 5,000 10 1
Micro Bus 8,500 50 4
Car 8,500 310 8
Truck 11,000 200 4
He incurred following expenses during the year.
Particulars Amount (Rs.)
i. Petrol 1,01,000
ii. Diesel 1,20,000
iii. Oil & Lubricants 21,375
MHW P.T.O.
(28)
iv. Repair 68,245
v. Staff's Salary 39,00,000
Required:
i) Calculate the tax to be paid by Mr. Mohit for fiscal year 2069/70. 4
ii) Mr. Mohit did not submit Annual Income Tax Return under Section 96. What
are its implications? 1

a) Answer Calculation of total income from employment of Mr. Sinha F/Y 2069/70

Particulars Amount (Rs.) Amount (Rs.)


Basic Salary 40000 x 12 4,80,000.00
Traveling Allowances (Note-1) 4000 x 12 48,000.00
Dividend Income (Note-2) -
Contribution of Provident Fund 4000 x 12 48,000.00
Prize in Race (Note-3) -
Total income from employment 576,000.00
Less:
Contribution to approval Provident fund (Note-4) 96,000.00
Life Insurance Premium (Note-5) 15,000.00
Taxable income from employment 465,000.00

Calculation of Employment Tax:

Particulars Amount (Rs.)


Tax on first Rs. 200000 @ 1% 2,000.00
Tax on Next Rs. 100000 @ 15% 15,000.00
Tax on Balance Rs. 165000 @ 25% 41,250.00
Total tax on employment 58,250.00

Notes:
1. As per Sec. 21, payment of traveling allowances from home to office & back is defined as
payment of personal expense. Payment of personal expenses by employer are taxable for
employees.
2. Assuming that the payer company is a resident company, dividend is final withholding
payment, subject to 5% WHT and so is not included in the income from employment.
3. Prize won in the race is not related with employment & thus is not to be included in income
from employment but it is subject to casual income tax @ 25%.
4. Contribution to Approved Provident Fund.
As per Section 63 (2) & Rule 21 of Income Tax Act, lower of following is allowed as
deduction for contribution to Retirement fund.

Particulars Amount Amount


i. Actual Contribution
- Employer's Contribution 48,000.
- Employee's Contribution 48,000. 96,000.
(deducted by employer)
MHW P.T.O.
(29)
ii. 1/3rd of Assessable Income. 1/3 X 576,000 192,000.
iii. Maximum Unit. Rs. 300,000. Rs. 300,000.
Lower of above Rs. 96,000.

5. As per Section 1(12) of Schedule 1 of income tax act, A resident person is eligible to get
deduction of actual premium paid against Investment Insurance or Rs. 20,000 whichever is
lower. Here,

Particulars Amount
15000
a. Actual Premium
20000
b.Maximum limit
Allowable(Lower of above) 15000
b)
i.
 As per Sec. 14 (1) of Income Tax Act 2058, clarified by Income Tax directives, interest
paid for the purpose of purchasing/construction of assets that is put to use during the year,
can be claimed as interest expense for the whole year. This concept differs with normal
accounting concept where direct & indirect expenses (including interest cost) incurred till
the machine is ready to use need to be capitalized. The machine has been ready to use in
2067.01.10, which means all interest expense for 2066/67 is deductible for tax purpose.
So, the tax officer is right to note that the depreciation base is overstated due to interest
expenses deductible as expenses is capitalized in the concerned pool.

 Simple Innovation is a legal entity and so is required to follow absorption-cost method


for calculating cost of trading stock. Section 15(7) (read with clarification) provides that
as per absorption cost method, cost of trading stock is equal to the sum of direct material
costs, direct labour costs, and factory overhead costs, but factory overhead cost do not
include any amount of repair and improvement and depreciation allowances on
depreciable assets.

Assuming that the given repair and improvement are incurred for depreciable assets (and
not for stocks in which case the repair of stocks can be claimed u/s15), the Repair &
Maintenance expense is not allowed to be incorporated in factory overhead cost, forming
part of Stock Valuation.

Thus, the company is wrong and assessment officer is correct in this regard, and
Following amount need to be added back in its income:

Particulars Amount Effect


Repair expenses incorporated in Opening 756,760 Profit Got decreased due to
Stock same
Repair expenses incorporated in Closing 654,650 Profit Got increased due to
Stock same
Net Effect on Profitability 102,110 Profit got decreased due to
same
MHW P.T.O.
(30)

Thus, 102,110 need to be added back to cost of goods sold.


ii.
 The application should be submitted to IRD within 30 days of receipt of the notice of
decision by the tax payer ( section 115 (3 ka)). Here, Simple Innovation can apply till 30
days from 2069.08.20. In case a tax payer is not able to submit an application within the
specified days, it can apply to IRD for extension of the time, specifying the valid reasons
for the delay, within seven days from expiry of time. IRD may, where a reasonable cause is
shown extend the time for a period not exceeding 30 days.

 As per section 115(6), one need to deposit an amount equal to 100% of tax & fines in
which the tax payer has no dispute with the assessment order and 1/3rd of the disputed
amount. The case gives disputed amount of tax as Rs. 54,43,500 and so it needs to deposit
1/3rd of 54,43,500=18,14,500 and 100% of undisputed amount of tax, (i.e. 75,00,000-
54,43,500=20,56,500). Thus, total deposit required for administrative appeal
= 20,56,500+18,14,500 = 38,71,000.
c)
i. As per Schedule 1 of Income Tax Act, 2058. Person engaged in hiring vehicles has to pay a
fixed amount of taxes per vehicle, for different vehicle and if the owner of the vehicle is
natural person, then tax paid in such manner shall be final. The amount of taxes are as
follows:

Particulars Amount
i. Per minibus, mini-truck, truck & bus 1500
ii. Per Car, Jeep , Van, Micro-bus 1200
iii. Per Three wheeler, auto rickshaw & tempo 850
iv. per tractor and power tiller 750

The taxes to be paid by Mr. Mohit shall be as follows, since he is a natural person.
Type of Vehicle No. of vehicle owned Tax per Vehicle Total Tax
- Power tiller 7 750 5250
- Tractor 11 750 8250
- Auto Rickshaw 1 850 850
- Micro bus 4 1200 4800
- Car 8 1200 9600
- Truck 4 1500 6000
Total 34750

Thus, Mr. Mohit is liable to pay Rs. 34,750 tax for F/Y 2069/70

ii. As per section 97 (d), natural person having income from vehicles subject to payment of tax
at a fixed amount annually do not need to submit annual tax return.
Thus, Mr. Mohit need not submit annual tax return.

3.
a) Mr. X is engaged in legal consultancy services. Mr. X has maintained his books
of accounts on cash basis till income year 2068/69. From income year 2069/70 he
MHW P.T.O.
(31)
wants to changeover to accrual basis of accounting. Following information from
records of his books of accounts of income year 2068/69 is abstracted.
i) Service provided but fee not received Rs. 5,00,000 which was not included in
the income.
ii) Advance received from customers but service not rendered Rs. 3,00,000. The
amount was included in the income in the income year 2068/69.
iii) Rent for the period 2069 Shrawan to 2069 Paush paid in Ashadh 2069. The
amount is included in the expense in the income year 2068/69.
You are required to advise Mr. X 5
b) Explain the following with reasons; on tax/withholding taxes and implication
thereon, with reference to Income Tax Act, 2058. (5×2=10)
i) Z & Co. has taken loan from M/s Nabil Bank Ltd. amounting Rs. 10 crores for
conducting its business. The company has paid interest at the rate of 10%
amounting to Rs. 1 crore during income year 2069/70.
ii) Mr. Gokul has given a contract to Z & engineers, a construction company for
construction of his residential building for Rs. 1 crore. While making
payments to Z & engineers, Mr. Gokul does not deduct any withholding taxes.
iii) Ms. Rakhi has taken the insurance policy of Rs. 16,00,000 from Rastriya
Beema Sansthan. She pays annual premium of Rs. 80,000. After maturity of
the policy, she is receiving Rs. 24,00,000 from Rastriya Beema Sansthan.
iv) Mr. Smart has won the cash lucky draw of Rs. 1 million from a trading
company under its scheme. He claims that the amount is not subjected to any
tax deduction and wants to receive full payment.
v) X & Co. made the following payment to Z & Co., under a contract, for the
supply of materials required for its company. Both the company are registered
under Value Added Tax (VAT).
Date Amount (Rs.)
2069.04.08 25,000
2069.04.12 10,000
2069.04.13 5,000
2069.04.15 10,000
2069.04.17 5,000

Since the payments are less than Rs. 50,000, X & Co. claims no deduction of
withholding taxes on such payments.
c) X & Co. has received from Y & Co., a resident company, Rs. 10 lakhs as
dividend after deduction of taxes. The profit of X & Co. before inclusion of such
dividend income is Rs. 90 lakhs. X & Co. has decided to declare the dividend
distribution to its shareholders during current fiscal year amounting to Rs. 100
lakhs (including Rs. 10 lakhs of dividend received from Y & Co.). Mr. Bibhu is
one of the shareholders of X & Co. and supposed to receive from Rs. 5 lakhs as

MHW P.T.O.
(32)
dividend. Mention the relevant provision and calculate the amount of Tax to be
deducted while making payment to Mr. Bibhu. 5
Answer a) As per section 22 of Income Tax Act, 2058, Mr. X requires approval from Inland Revenue
Department (IRD) for change over to accrual basis of accounting. IRD while giving permission may
employ the following conditions:
i. To include Rs. 500,000 in taxable income of 2069.70 as fee accrued but not accounted for in books,
ii. To deduct Rs. 300,000 from taxable income as advance received from customer but treated as
income, and
iii. To allow the rent paid for the rest period as rent expenses.
b)
i.
As per section 88(4)(kha) of Income Tax Act, 2058, payment of interest to a resident bank or other
financial institution is not subject to withholding tax. Hence, payment of interest of Rs. 1 crore by Z &
Co. is not subject to withholding tax and the company will pay full amount without deduction of
withholding tax.
ii.
As per section 89(4)(ka) of Income Tax Act, 2058, payments made by Mr. Gokul, an individual natural
person to Z & engineers, a construction company, is not subjected to withholding taxes and hence, no
withholding tax is to be deducted. But Z & engineers has to include in its income such payments.
iii.
As per section 92(1)(ga) of Income Tax Act, 2058, payments on gain of investment insurance by a
resident person is subjected to final withholding taxes. Hence, Rastriya Beema Sansthan (RBS) is liable
to make the payment of Rs. 2,360,000 after deduction of withholding tax at the rate of 5% (amounting to
Rs. 40,000) on gain of Rs. 800,000 (Rs. 2,400,000-Rs. 1,600,000) and it is not required to be included in
the income of Ms. Rakhi.
iv.
As per section 92(1)(jha) of Income Tax Act, 2058, payments from win fall gain is subjected to final
withholding taxes. Amount received from lucky draw is win fall gain to Mr. Smart. Hence, the trading
company is liable to make the payment after deduction of withholding tax at the rate of 25% of Rs. 1
million i. e. Rs. 250,000 and the claim of Mr. Smart is not tenable. The amount received by Mr. smart is
not required to be included in his income.
v.
Section 89 of Income Tax Act, 2058 provides the following;
1) A resident person, who makes a payment under a contract or agreement exceeding NPR
50,000, shall withhold tax on the gross amount of payment at the rate of 1.5 percent.
2) The amount referred to in subsection (1) shall be determined by aggregating a payment made
by a person under contract with any other payment made by the person or an associate of the
person during the previous ten days under the same contract to the same payee or an
associate of the payee.

Accordingly, for the purpose of ensuring NPR 50,000, the payment made during last ten days is
counted.
Date Last Ten Days Amount (Rs)
2069.04.17 - 5,000
2069.04.15 3 10,000
2069.04.13 2 5,000
MHW P.T.O.
(33)
2069.04.12 1 10,000
2069.04.08 4 25,000
10 55,000

The payment made during last ten days is NPR 55,000. The amount of withholding tax is to be
deducted is NPR 825 (1.5% of NPR 55,000).

c) As per section 54(1) of Income Tax Act, 2058, dividends distributed by a resident company to its
shareholders shall be taxed in the form of final withholding tax.
Section 54(3) further envisages that notwithstanding anything contained in section 54(1), distribution
of dividend that is derived after withholding tax as per subsection (1) shall be exempt from tax.
Computation of Tax to be deducted while making payment of dividend to Mr. Bibhu
Particulars Amount (Rs)
Total amount of dividend distribution 100 Lakhs
Dividend income after deduction of tax 10 Lakhs
Proportion of dividend income (Rs. 10 lakhs/Rs. 100 lakhs) 10 %
Payment to Mr. Bibhu 500,000
Amount not subject to tax (10%) 50,000
Amount subject to tax (90%) 450,000
Tax amount at the rate of 5% 22,500

4. Write short notes of the following with reference to Income Tax Act, 2058. (4×2.5=10)
a) Trustee
b) Royalty
c) Permanent Establishment
d) Investment Insurance

Answer

The answer below defines the terms given in the question. Students are welcome to write any other
relevant provision in the exams, such as the residential status of trustee and how it is taxed, the treatment
of royalty for tax purpose, tax implications for PE and deductions allowed for investment insurance.

a) Trustee
As per sec 2(u) of Income tax Act, 2058, A "Trustee" means an individual or Goothi or corporate
body holding assets in a fiduciary capacity, whether held alone or jointly with other individuals
or corporate bodies, and includes the following persons-
(i) any executor or administrator of a deceased individual's estate;
(ii) any liquidator, receiver, or trustee;
(iii) any person having, either in a private or official capacity, the
possession, direction, control, or management of the assets of an
incapacitated person;
MHW P.T.O.
(34)
(iv) any person who manages assets under a private foundation or other
similar arrangement; and
(v) any person in a similar position to a person mentioned in subparagraphs
(i), (ii), (iii) and (iv).
(b) Royalty
As per sec 2(ak) of Income tax Act, 2058, "Royalty" means any payment made under a lease
of an intangible asset and includes any payment made for the following purpose:-
(i) the use of, or the right to use, a copyright, patent, design, model, plan, secret
formula or process, or trademark;
(ii) the supply of know-how;
(iii) the use of, or right to use, a cinematography film, video tape, sound recording,
or any other like medium and the supply of information concerning
industrial, commercial, or scientific experience;
(iv) the supply of assistance ancillary to a matter referred to in paragraphs (i), (ii)
or (iii); or
(v) a total or partial forbearance with respect to a matter referred to in paragraphs
(i), (ii), (iii) or (iv). provided that, the term

(c) Permanent Establishment


As per sec 2(bb) of Income tax Act, 2058, "Permanent establishment" means a place where a
person wholly or partly carries on a business, and includes the following places:-
(i) a place where a person wholly or partly carries on a business through an agent,
other than a general agent of independent status acting in the ordinary course of
business as such;
(ii) a place where a person has, is using, or is installing substantial equipment or
substantial machinery;
(iii) one or more places within a country where a person furnishes
(whether through employees or otherwise) related services (including technical,
professional, or consultancy services) for a period or periods aggregating more
than 90 days within any 12 month period; or
(iv) a place where a person is engaged in a construction, assembly, or installation
project for 90 days or more, including a place where a person is conducting
supervisory activities in relation to such a project.

(d) Investment Insurance


As per sec 2(am) of Income tax Act, 2058, "Investment insurance" means insurance of any of
the following classes:
(i) insurance where the event covered is the death of an individual who is the insured or an
associate of the insured;
(ii) insurance where the event covered is an individual who is the insured or an associate of the
insured sustaining personal injury or becoming incapacitated in a particular manner;
(iii) insurance where the insurance agreement is expressed to be in effect for at least five years or
without limit of time and is not terminable by the insurer before the expiry of five years
except in special circumstances specified in the contract;
(iv) insurance under which an amount or series of amounts is to become payable to the insured in
the future; and
(v) reinsurance of insurance referred to under subparagraphs (i), (ii),or (iv); and
(vi) reinsurance of reinsurance referred to under subparagraph (v).

5. Answer the following with reference to Value Added Tax (VAT) Act/Rules:
MHW P.T.O.
(35)
a) EFG Garment Pvt. Ltd. had the following transaction in the month of Shrawan
2070. Calculate the VAT payable/receivable from the information below. 10
Particulars Amount (Rs.)
Local sale 50,00,000
Export sale 1,00,00,000
Purchase of clothes, Stitching, Packing Materials,
loose tools for machineries 81,00,000
Special packing for export 4,00,000
Payment of consultancy charges abroad 5,00,000
Purchase of bus for staff transportation 20,00,000
Purchase of motorcycle of hire purchase 4,00,000
Telephone expenses 76,000
Purchase of diesel for generator 80,000
Purchase of diesel for bus 24,000
Purchase of petrol for motorcycle 30,000
Purchase of computers 90,000
Purchase of soft drinks 12,000
Additional information:
Opening VAT receivable for the month was Rs. 91,560. Diesel for bus for
Rs. 16,000 and soft drinks for Rs. 6,000 was purchased through abbreviated tax
invoice. Items above are exclusive of VAT.
b) State the conditions, procedures for and effect of cancellation of registration under
Value Added Tax Act/Rules. 10

Answer a) Calculation of VAT payable/receivable for the month of Shrawan 2070 for EFG Garment Pvt.
Ltd.

Particulars Amount (Rs) VAT (Rs) Remarks


Local Sale 5,000,000 650,000 Full
Export Sale 10,000,000 0 0% for export
Total output VAT 15,000,000 650,000
Purchase of clothes, Stitching, Packing
Materials, loose tools for machineries 8,100,000 1,053,000 Full
Special packing for export 400,000 52,000 Full
reverse charging,
assuming the VAT is
Payment of consultancy charges abroad 500,000 65,000 already paid.
Purchase of bus for staff transportation 2,000,000 104,000 only 40% allowed
Purchase of motorcycle of hire purchase 400,000 52,000 Full
Telephone expenses 76,000 9,880 Full
Purchase of diesel for generator 80,000 10,400 Full
Abbreviated Tax Invoice
Purchase of diesel for bus 24,000 1,040 not allowed
Purchase of petrol for motorcycle 30,000 0 Not allowed Rule 41
Purchase of computers 90,000 11,700 Full

MHW P.T.O.
(36)
Purchase of soft drinks 12,000 0 Not allowed Rule 41
Total Input tax credit 1,359,020
Calculation of VAT Payable/Receivable for the month of Shrawan 2070
Opening VAT Receivable 91,560
Add: input VAT 1,359,020
Less: output VAT 650,000
Net Receivable 800,580

b)
Following provisions are mentioned regarding the conditions, procedures for and effect of cancellation
under Value Added Act/Rules.

Conditions for cancellation

Section 11 (1) and (1ka) mentions the following conditions for cancellation of registration;
(a) In the case of body corporate, if the body corporate is closed down, sold or transferred or if the
body corporate otherwise ceases to exist
(b) In the case of an individual ownership, if the owner dies
(c) In the case of a partnership firm, if it is dissolved or any of the partners expires
(d) If a registered person ceases to be engaged in taxable transactions
(e) In case the registered person submits zero tax return continuously for one year or it has not
submitted tax return till the date
(f) If registered mistakenly

In case, the person is voluntarily registered, cancellation of registration can only be done after at least
one year period has elapsed after registration.

In case the condition for de-registration is triggered and the person is willing to deregister, an
application might be filed within 30 days of such de-registration event along with the VAT returns till
such date and any VAT due.

If the tax payer has any input tax credit claimed stocks or capital assets on date of such application, such
assets are considered to be as disposed for VAT purchase. VAT should be paid for such deemed
disposed items on basis of their market value.

In case a person has applied for cancellation of its registration number has to produce its records and
documents for audit within fifteen day of the application to the tax office. The tax officer shall audit the
records and documents and within three months of the application submitted may allow or reject the
cancellation. ,

Once the application for cancellation of registration is submitted as per sub-rule (1) of rule 12, tax return
shall have to be submitted until the notice of cancellation or within three months.

It shall be the responsibility of concerned tax officer to give notice to the person submitted the
application for cancellation of registration, about the cancellation, within three months of submission of
application.

MHW P.T.O.
(37)
6.
a) AMP Wire Industries Pvt. Ltd. purchased the following in the month of Kartik
2070.
Particulars VAT Paid (Rs.)
Used photocopy machine 1,30,000
Brand new Fax machine 2,600
Used generator set 3,90,000
Import of Brand new generator set 5,20,000

Required:
Calculate the VAT amount to be claimed for offset. 5
b) "Supertech Infosys Pvt. Ltd." deals in computer & related IT technology business.
In the course of its business, it received service from "Key Technology Corp.,
Japan. Key against which, Technology Corp. raised bill of Rs. 21.60 lakhs.
Management of Supertech is confused as to whether VAT shall be charged on
same.
Advise him as regards to VAT applicable on same and net payment to be send.
Specify the provision of VAT Act, 2052 in this regards. (Ignore other taxes, if
applicable). (2.5+2.5=5)

c) Answer the following in the light of VAT Act, 2052. (2×3=6)


i) Swasthya Flour Mill Pvt. Ltd. is a Maida producing industry. It is a complete
manufacturing unit, and does not involve in local/import purchase of Maida.
For fiscal year 2069/70, it had following VAT transactions:
Particulars VAT Amount (Rs.)
A. VAT collected on sales
i. From VAT registered party. 15,00,000
ii. From Non-VAT registered party. 2,00,000
B. VAT paid on purchase. 13,00,000
What amount of VAT refunds is Swasthya Flour Mill Pvt. Ltd. facilitated?
ii) Healthy Oil Industries is manufacturing Mustard Oil. During shortage, the
industry also, imports oil and sell it to its customers to maintain its market.
The VAT accounts of Healthy Oil Industries. Showed the following data for
fiscal year 2069/70.
Particulars Trading of Mustard Manufacturing
Oil (import of Mustard Oil
purchase and sales)
A. VAT collected on sales
i From VAT registered party 17,00,000 18,00,000

ii From Non-VAT 6,00,000 1,90,000


registered party
B. VAT paid on purchase 15,00,000 14,00,000
Calculate the VAT amount which Healthy Oil Industries is facilitated to get
refund from Inland Revenue Department.
MHW P.T.O.
(38)

d) What shall be the fine and penalty chargeable under the following situations as
per VAT Act, 2052? (2×2=4)
i) Late payment of VAT amount.
ii) Tax plate not kept/misplaced.
Answer
a) AMP Wire Industries (P) Ltd can offset the following VAT paid on purchase.

Particulars Amount (Rs.)


Used Photocopy Machines 130000
Brand new Fax Machines 2600
Used Generator set 390000
Import of Brand new Generator set 520000
1042600

As per section 17(1) of the Value Added Tax Act 2052, registered person may offset VAT
paid/payable on import or purchase of goods or services related to its business from the VAT
collected.
As per section 17 (2) & Rule 41 (1) & (2) VAT paid on certain goods & services can‟t be offset.
Here, the items purchased by AMP Wire Industries (P) Ltd are not in the list of section 17 (2) &
Rule 41 (1) & (2). Hence, the VAT paid on such items can be offset.

b) As per section 8 (2) of Vat Act, 2052, "Any registered / unregistered person receiving service
within Nepal from unregistered person outside Nepal, need to determine and collect Vat on
taxable Value as per provisions of this Act.
Bill raised by key consultant is for NRS 21,60,000.Thus, Taxable Value for the VAT is NRS.
21,60,000.

Thus, following VAT need to be charged:


Particulars Taxable value Rate Amount Remarks
VAT 21,60,000 13% 280,800 As per Section 8 (2) of Vat Act.

The amount to be paid to Key Technology Corporation shall be Rs. 21.60 Lakhs which remains
unchanged with the value of service.

Thus, VAT of Rs 280,800 need to be charged on the above service, and deposited to VAT office.
The paid VAT can be claimed for set-off under Reverse Charging System.
c)
i) Schedule 1 of VAT Act 2052, prescribes the following provision with respect to VAT
refund to Maida industries:
25% of VAT Amount after deducting VAT paid on Purchase/Import from VAT collected
on Sales of own produced maida shall be refunded as prescribed by IRD.
Thus, Swasthya Flour Mill Pvt. Ltd, producing Maida shall have following Vat refund:
Particulars VAT Amount
Vat collected on Sales
From VAT Registered Party 15,00,000
From VAT Unregistered Party 2,00,000
Total VAT Collected 17,00,000

MHW P.T.O.
(39)
Less:
VAT Paid on Purchase 13,00,000
Net VAT Amount 4,00,000
Net Amount for VAT Refund (25%) 1,00,000

ii) As per Schedule 1 of VAT Act 2052, 50% of VAT collected on sale of own produced
mustard oil by domestic mustard oil industry, if sold to VAT registered person shall be
refunded as prescribed by IRD. Healthy oil industries cannot enjoy VAT refund on
Trading of Mustard oil.On its Manufacturing sales, it can have following VAT refund:
VAT collected From Sales to VAT registered party(Of manufactured Mustard Oil)
= 18,00,000.
VAT Refund ( 50% of it) = 900,000.
Thus, Healthy Oil Industries can claim against 900,000 VAT Refund.
d)
i. Late payment of VAT amount.
Additional fee:
On late payment of VAT amount to Inland Revenue, additional fee of 10% annually shall
be imposed on payable VAT amount (Section 19 (2)).
Interest
On late payment of VAT amount to Inland Revenue, 15% annual interest shall be
imposed in such outstanding amount. (Section 26).

ii. Tax plate not kept/misplaced.


As per section 29(1) if tax plate is not kept, penalty @ 2000 per week shall be charged
and if it is not kept at prescribed place, penalty @ 1000 shall be imposed.

MHW P.T.O.
CHARTERED ACCOUNTANCY PROFESSIONAL CAP-II

SUGGESTED ANSWER
December 2017

Gf]kfn rf6{8{ PsfpG6]G6\; ;+:yf


The Institute of Chartered Accountants of Nepal
Advanced Accounting
Maximum Marks - 100
Total No. of Questions - 6 Total No. of Printed Pages - 5
Time Allowed - 3 Hours
Marks
Attempt all questions. Working notes should form part of the answer.
1. Batliboi & Co. Ltd. carried on manufacturing business. Its products were sold to
wholesalers and the company had its own retail shop. Adhikary & Co. (P) Ltd.
carried on similar manufacturing business, but all goods produced were sold
through the company's own retail shops.
The summarized balance sheets of the two companies as at 31st March, 2014
were as follows:
Batliboi & Adhikary & Batliboi & Adhikary &
Co. Ltd. Co. (P) Ltd. Co. Ltd. Co. (P) Ltd
(Rs.) (Rs.) (Rs.) (Rs.)
Share Capital Fixed Assets:
Authorized equity Freehold Pro-
shares of Rs. 10 40,00,000 6,00,000 perties at cost 10,00,000 2,50,000
Plant & Machinery
Issued & fully at cost less
paid up 25,00,000 6,00,000 depreciation 13,00,000 1,00,000
23,00,000 3,50,000
P & L A/c 3,40,000 90,000 Current Assets:
Creditors 4,20,000 70,000 Stock 4,80,000 1,20,000
Debtors 2,30,000 80,000
Bank 2,50,000 2,10,000
32,60,000 7,60,000 32,60,000 7,60,000
The Original cost of Plant and Machinery was:
Batliboi & Co. Ltd. Rs. 26,00,000
Adhikary & Co. (P) Ltd. Rs. 2,00,000
The following arrangements were made and carried out on April 1, 2014.
a) Batliboi & Co. Ltd. purchased from the shareholders of Adhikary & Co. (P) Ltd. all
the issued shares @ Rs. 14 per share.
b) The shareholders of Adhikary & Co. (P) Ltd. took over one of the freehold properties
of Adhikary & Co. (P) Ltd. for Rs. 60,000, at the book value of the same. It was
agreed that the amount should be set off against the amount due to them under (a)
above and the balance due to them to be satisfied by the issue of an appropriate
number of equity shares in Batliboi & Co. Ltd. at Rs. 19.50 per share.
The necessary transfer in regard to the setting off the price of the property taken over
by the shareholders against the amount due to them from Batliboi & Co. Ltd. were
made in the books of the two companies.
c) All manufacturing was to be carried on by Batliboi and Co. Ltd. and all retail business
is to be carried on by Adhikary & Co. (P) Ltd. in this connection.
i) Batliboi & Co. Ltd. purchased the whole of Adhikary & Co. (P) Ltd.'s plant and
machinery for Rs. 1,50,000 and certain of their free-hold property (cost Rs.
1,00,000) at Rs. 1,20,000.
ii) Adhikary & Co. (P) Ltd. purchased Batliboi & Co. Ltd.‘s freehold retail shop
Buildings (Cost to Batliboi & Co. Ltd., Rs. 75,000) at Rs. 60,000 and took over
the retail stock at Rs. 80,000 at the book value.
d) Batliboi & Co. Ltd. drew a cheque in favour of Adhikary & Co. (P) Ltd. for
the net amount due, taking into account all the matters mentioned above.
e) Immediately after the transfer of shares in (a) above. Adhikary & Co. (P)
Ltd. declared and paid dividend of Rs. 60,000 (ignore income-tax).
You are required to prepare the Balance Sheets of Batliboi & Co. Ltd. and
Adhikary & Co. (P) Ltd. immediately after the completion of the above
transaction. 20
ANSWERS:
Balance Sheet of Batliboi & Co. as on 1st April, 2014

Liabilities Rs. Assets Rs.


Share Capital Fixed Assets
Authorized Freehold properties
4, 00,000 shares of Rs. 10 as per last Balance
Each 40, 00,000 Sheet 10, 00,000
Issued, subscribed & paid-up Addition during the
2, 90,000 shares of Rs. 10 each Year 1, 20,000
Fully paid of which 40,000 1120000
Shares were issued pursuant Less: sold during the 75,000
To contract without pay- Year 1045000
Ment being received in cash 29, 00,000 Plant and Machinery:
As per 26, 00,000
Balance Sheet
Reserved & Surplus Addition during 1, 50,000
The year
Share premium A/c 3, 80,000 27, 50,000
Profit & Loss A/c 3, 25,000 Less: Depreciation 13, 00,000
Current Liabilities & 14, 50,000
Provisions: Investments
Sundry Creditors 4, 20,000 Shares in subsidiary 7, 80,000
Company
Current, Assets, Loans & Advances
Stock in trade 4, 00,000
Sundry debtors 2, 30,000
Cash at Bank 1, 20,000
40, 25,000 40, 25,000
Working Notes:
(1) Value of shares in Adhikary & Co. (P) Ltd. Rs.
60,000 Shares @ Rs. 14 per share 8, 40,000
Less: Value of freehold property 60,000
Net amount due 7, 80,000

No of shares issued 7, 80,000/ Rs. 19.5= 40,000


Amount credited to Share Capital 4, 00,000
Amount credited to Share premium 3, 80,000
7, 80,000
(2) Rs. 15,000 loss on the sale of Building to Adhikary & Co. (P) Ltd. has been debited to
the profit & Loss Amount.
(3) Investment A/c has been credited by dividend received Rs. 60,000 out of pre-
acquisition profit.(780,000+60,000-60,000)
(4) Amount payable Adhikary & Co. (P) Ltd. Rs.
Value of assets purchased from Adhikary & Co. (P) Ltd. 2, 70,000
Less: Value of assets sold 1, 40,000
1, 30,000
Value of freehold property taken over by shareholders
Of Adhikary & Co. (P) Ltd. 60,000
1, 90,000
(5) Cash Balance:
As given 2, 50,000
Add: Dividend received 60,000
3, 10,000
Less: paid to Adhikary & Co. Ltd. including
Rs. 60,000 for assets taken over by its erstwhile shareholders 1, 90,000
1, 20,000

Balance Sheet of Adhikary & Co. (P) Ltd. as on 1st April, 2014

Liabilities Rs. Assets Rs.


Share capital Fixed Assets
Authorized Freehold properties:
60,000 shares of Rs. 10 each 6, 00,000 as per last Balance
Issued, subscribed & paid up Sheet 2, 50,000
60,000 shares of Rs. 10 Addition during the year 60,000
Each full paid 6, 00,000 3, 10,000
Reserved and Surplus Less: Sold during the year 1, 60,000 1,50,000
Plant & Machinery
Profit & Loss Account 1, 00,000 Cost 2, 00,000
Current Liabilities & Provision Less: Sold (Cost) 2, 00,000 nil
Sundry Creditors 70,000 Current Assets, Loan & Advances
Stock in trade 2,00,000
Sundry debtors 80,000
Cash at Bank 3,40,000
7, 70,000 7, 70,000
Working Notes: Rs.
1) Profit & Loss Account (given) 90,000
Add: Profit on sale of machinery and freehold property 70,000
1, 60,000
Less: Dividend paid 60,000
1, 00,000

2) Freehold properties have been reduced by Rs. 1, 00,000 transferred to Batilboi &
Co. & Rs. 60,000 taken over by the shareholders of Adhikari & Co. (P) Ltd.
3) Cash at Bank: Balance as given Rs.
Add: Received from Batliboi & Co. 2, 10,000
1, 90,000
4, 00,000
Less: Dividend Paid 60,000
3, 40,000
2.
a) Eddi and Freddi carrying on business in partnership sharing Profit and
Losses equally, wished to dissolve the firm and sell the business to Sreddi
Ltd. on 31.03.2074, when the firm‘s position was as follows:
Balance Sheet as at 31.03.2074
Liabilities Amount (Rs.) Assets Amount (Rs.)
Eddi‘s Capital 175,000 Land and Building 125,000
Freddi‘s Capital 130,000 Furniture 52,500
Sundry Creditors 80,000 Stock 112,500
Debtors 91,000
Cash 4,000
Total 385,000 Total 385,000
The arrangement with Sreddi Limited Company was as follows:
i) Land and Building was purchased at 20% more than the book value.
ii) Furniture and stock were purchased at book values less 15%.
iii) The goodwill of the firm was valued at Rs. 40,000.
iv) The firm‘s debtors, cash and creditors were not to be taken over, but the
company agreed to collect the book debts of the firm and discharge the
creditors of the firm as an agent, for which services, the company was
to be paid 5% on all collections from the firm‘s debtors and 3% on cash
paid to firm‘s creditors.
v) The purchase price was to be discharged by the company in fully paid
equity shares of Rs. 10 each at a premium of Rs. 2 per share.
The company collected all the amounts from debtors. The creditors were
paid off less by Rs. 1,000 allowed by them as discount. The company paid
the balance due to the vendors in cash.
Prepare the Realization Account, the Capital Accounts of the partners
and the Cash Account in the books of partnership firm. 10
b) Following information as at second quarter ending 2073 were drawn from
the records of Minamma Bank Limited as under:
Loan outstanding for Amount (Rs.)
Up to 3 months 1,673,000
More than 3 months but not more than 6 months 13,612
More than 6 months but not more than 12 months 782
More than 12 months 2,198
Total 1,689,592
The bank has not restructured or rescheduled any of its credit.
Following additional information relating to previous quarter ending were
extracted from the records of the bank:
Particulars Amount (Rs.)
Paid up equity share capital 171,010
Statutory general reserve 155,432
Retained earnings 87,886
General loan loss provision 16,983
Exchange equalization reserve 22,313
Un-audited current year cumulative profit 31,991
Deferred revenue expenses 2,884
The bank is in the process of preparing the documents for quarterly
reporting. The bank has also provided a term loan of Rs. 125,000 to a single
party during the period under review. As a reporting and compliance officer
of the bank you are required to calculate movement in loan loss provision
amount. 10
ANSWERS:
a)
Amount in NPR
Dr Realization A/c Cr
Particulars Amount Particulars Amount
To Land & Building 125,000 By Sundry Creditors 80,000
To Furniture 52,500 By Sreddi Ltd Co- 330,250
Purchase consideration (W.N-1)
To Stock 112,500 By Sreddi Ltd Co- Sundry
Debtors 91,000
To Debtors 91,000 Less- Commission 86,450
(5% on 91,000) 4,550
To Sreddi Ltd Co- 79,000
Sundry Creditors
To Sreddi Ltd Co- 2,370
Commission
(3% on 79,000)
To Profit transferred to 34,330
Eddi‘s Capital 17,165
Freddi‘s Capital 17,165
Total 496,700 Total 496,700

Amount in NPR
Dr Capital Account Cr
Particulars Eddi Freddi Particulars Eddi Freddi
To Shares in 187,023 143,227 By Balance b/d 175,000 130,000
Sreddi Ltd (W.N-
2)
To Cash –Final 5,142 3,938 By Realization A/c 17,165 17,165
Payment
Total 192,165 147,165 Total 192,165 147,165

Amount in NPR
Dr Cash Account Cr
Particulars Amount Particulars Amount
To Balance b/d 4,000 By Eddi‘s Capital – Final 5,142
payment
To Sreddi Ltd Co- 5,080 By Freddi‘s Capital – Final 3,938
(Amount realized from Debtors payment
less amount paid to creditors)
( W.N-3)
Total 9,080 Total 9,080

Working Notes
1. Computation of Purchase Consideration

Particulars Amount
Land & Building (125,000+20% of 125,000) 150,000
Furniture (52,500-15% of 52,500) 44,625
Stock (112,500-15 % of 112,500) 95,625
Goodwill ( given) 40,000
Total 330,250
2. The shares received from the company have been distributed between the two partners
Eddi & Freddi in the ratio of their final claims i.e., 192,165: 147,165

No of shares received from the company=


Eddi gets ( Shares valued at 15,585*12 =NPR.187, 023.
Freddi gets the remaining 11,936 shares valued at 11,936*12 = NPR.143, 227
3. Calculation of net amount received from Sreddi Ltd on account of amount realized from
debtors less amount paid to creditors.
Particulars Amount
Amount realized from debtors 91,000
Less: Commission for realization from debtors (5% on 91,000) 4,550
86,450
Less: Amount paid to creditors 79,000
7,450
Less: Commission for cash paid to creditors (3% on 79,000) 2,370
Total 5,080
b) As per the provision of the Directive 3 of unified Directives, a bank can provide a credit
up to 25% of its core capital to a single party. This limit is called the single obligatory
limit (SOL). While calculating the SOL, core capital of previous quarter shall be taken as
base. In case any excess credit than SOL has been provided by the bank, additional 100%
provision shall be made for such excess credit amount.
Before calculating the provision amount, SOL of the bank shall be tested upon.
Computation of SOL and credit amount in excess of SOL
Particulars Amount
Core Capital
Paid up Equity Share Capital 171,010
Statutory General reserve 155,432
Retained earnings 87,886
Un-audited current year cumulative profit 31,991
Less: Deferred Revenue expenses (2,884)
Total Core capital 443,435
Single obligatory limit ( 25% of the core capital) 110,859
Loan to single party 125,000
Loan in excess of SOL 14,141
Computation of loan loss provision amount as per provisions of the Directive 2 of the NRB
Computation of Loan Loss Provision amount
Categories Loan Amt Provision Provision
Rate Amount
Not due or <=3 months Pass 1,673,000 1% 16,730
>3 months <= 6 mths Sub-standard 13,612 25% 3,403
>6 months <= 12 mths Doubtful 782 50% 391
>12 months Loss 2,198 100% 2,198
Total 1,689,592 22,722
Additional provision for loan in excess of SOL 14,141
Total Provision amount 36,863

Minamma Bank Ltd


Movement in Provision Amount
For Second Quarter of Fiscal Year 2073/74
Amount in NPR
Particulars Amount
Opening Provision amount 16,983
Closing Provision amount 36,863
Movement in provision amount (addition during the quarter) 19,880

3.
a) On 19th Mansir, 2073 premises of National Trading were destroyed by fire,
but sufficient records were saved, wherefrom the following particulars were
ascertained:
Rs.
Stock at cost on 1.4.2072 36,750
Stock at cost on 31.03.2073 39,800
Purchases less returns during 2072/73 1,99,000
Sales less return during 2072/73 2,43,500
Purchases less returns during 1.4.2073 to 19.8.2073 81,000
Sales less returns during 1.4.2073 to 19.8.2073 1,15,600
National Trading closes his books every 31st Ashadh. In valuing the stock
for the balance sheet as at 31st Ashadh 2073 Rs. 1,150 had been written off
on certain stock which was a poor selling line having the cost Rs. 3,450. A
portion of these goods were sold in Ashoj, 2073 at a loss of Rs. 125 on
original cost of Rs. 1,725. The remainder of this stock was now estimated to
be worth the original cost. Subject to the above exceptions, gross profit has
remained at uniform rate throughout. The stock salvaged was Rs. 2,900.
Show the amount of the claim of stock destroyed by fire. Memorandum
Trading Account to be prepared for the period from 1-4-2073 to 19-8-2073
for normal and abnormal items. 10
b) Mahakali Bank Ltd. issues shares of Rs. 10 each at a premium of 10%
payable as follows:
On application Rs. 2
On allotment Rs. 3 (including premium)
On first call Rs. 2
On final call Rs. 4
Mahesh who was holding 50 shares did not pay his allotment and first call
and his shares were forfeited. Suresh, who was holding 30 shares, did not
pay first call and his shares also were forfeited. Journalize transaction
relating to forfeiture of shares. 5
ANSWERS:
a)
National Trading
Trading Account for the year ended on 31st Ashadh, 2073
Rs Rs Rs
To Opening Stock 36,750 By Sales A/c 2,43,500
To Purchases 1,99,000 By Closing Stock
To Gross Profit 48,700 As valued 39,800
Add: Amount written off 1,150 40,950
to restore stock to full
cost
2,84,450 2,84,450

The normal rate of gross profit to sales is =48,700 x 100 =20%


2, 43,500
Memorandum Trading Account up to 19, Mansir, 2073
Normal Abnormal Total Normal Abnormal Total
Items Items Items Items
Rs Rs Rs Rs Rs Rs
To 37,500 3,450* 40,950 By Sales 1,14,000 1,600 1,15,600
Opening
Stock
To 81,000 - 81,000 By Loss - 125 125
Purchases
To Gross 22,800 - 22,800 By 27,300 1,725 29,025
Profit Closing
(20% on Stock
Rs (bal.fig)
1,14,000)
1,41,300 3,450 1,44,750 1,41,300 3,450 1,44,750
*at cost.
Calculation of Insurance Claim
Particulars Rs.
th
Value of Stock on 19 Mansir, 2073 29,025
Less :Salvage (2,900)
Loss of Stock 26,125
Therefore insurance claim will be for Rs. 26, 125 only.

b)
Share capital A/C –Dr 50*6 300
Securities Premium A/c –Dr (50*1) 50
To forfeited shares A/C (50*2) 100
To share allotment A/C (50*3) 150
To Share first call A/C( 50*2) 100
( Being forfeiture of Mahesh Shares)
Share Capital A/C 30*6 180
To forfeited shares A/C (30*4) 120
To share first call A/C(30*2) 60
(Being forfeiture of Suresh A/C)
Note: in case of Mahesh securities premium account has been cancelled by debiting it
because the premium has not been paid but in case of Suresh it has not been done
because the premium has been paid, premium once received cannot be cancelled.

4.
a) From the following information of Reliable Marine Insurance Ltd. for the
st
year ending 31 Ashadh, 2074 find out the 10
i) Net premiums earned
ii) Net claims incurred
Particulars Direct Business (Rs.) Re-insurance (Rs.)
Premium:
Received 88,00,000 7,52,000
Receivable– 01.04.2073 4,39,000 36,000
Receivable– 31.03.2074 3,77,000 32,000
Paid 6,09,000
Payable– 01.04.2073 27,000
Payable– 31.03.2074 18,000
Claims:
Paid 69,00,000 5,54,000
Payable– 01.04.2073 89,000 15,000
Payable– 31.03.2074 95,000 12,000
Received 2,01,000
Receivable– 01.04.2073 40,000
Receivable– 31.03.2074 38,000
b) M/s Baboon & Co. has four departments A, B, C, D. Each department was
being managed by a departmental manager whose commission was 10% of
the respective departmental profit, subject to a minimum of Rs. 6,000 in
each case. Inter departmental transfers took place at a loaded price as
follows.
From Department A to Department B: 10% above cost
From Department C to Department D: 20% above cost
From Department A to Department D: 20% above cost
From Department C to Department B: 20% above cost
For the year ended 31st Ashadh, 2074 the company had already prepared and
closed the Departmental Trading and Profit and Loss Account. Subsequently
it was discovered that the closing stock of departments had included inter
departmentally transferred good at loaded price instead of cost price.
From the following information, prepare a statement re-computing the
departmental profit or loss. 5

(Rs.)
Particulars Dept. A Dept. B Dept. C Dept. D
Final Profit/ Loss 38,000 50,400 72,000 1,08,000
(Loss) (Profit) (Profit) (Profit)
Inter departmental 70,000 4,800
transfers included at (22,000 from Dept. (3,600 from Dept.
loaded price in the A and 48,000 from C and 1,200 from
departmental stock Dept. C) Dept. A)
ANSWERS:
a)
Net Premium earned
Particulars Rs. Rs.
Premium from direct business received 88,00,000

Add: Premium on re-insurance accepted 7,52,000


Add :Receivable as on 31.03.2074 32,000
Less: Receivable as on 01.04.2073 (36,000) 7,48,000
95,48,000
Less: Premium on re-insurance ceded 6,09,000
Add: Payable ason31.03.2074 18,000
Less :Payable as on 01.04.2073 (27,000) (6,00,000)
Net Premium earned 89,48,000

(ii) Net Claims incurred


Rs.
Claims paid on direct business 69,00,000
Add: Re-insurance 5,54,000
Add: Outstanding as on 31.3.2073 12,000
Less: Outstanding as on 1.4.2074 (15,000) 5,51,000
74,51,000
Less: Claims received from re-insurance 2,01,000
Add: Outstanding as on 31.3.2074 38,000
Less: Outstanding as on 1.4.2073 (40,000) (1,99,000)
72,52,000
Add: Outstanding direct claims at the end of the year 95,000
73,47,000
Less :Outstanding claims at the beginning of the year (89,000)
Net claims incurred 72,58,000
b)
Statement of re-computation of department profit/Loss
Particulars Dept. A Dept. B Dept. C Dept. D
Final profit/Loss as computed earlier (38,000) 50,400 72,000 1,08,000
Add –Department manager‘s commission 6,000 6,000 8,000 12,000
already deducted from profit ( W.N-1)
Profit before charging manager‘s (32,000) 56,400 80,000 120,000
commission
Loss- Profit earned due to transfer of (2,200) - (8,600) -
goods at loaded price and included in
departmental unsold stock ( WN-2)
Correct departmental profit before (34,200) 56,400 71,400 120,000
charging manager‘s commission
Manager‘s Commission @10% of (6,000) (6,000) (7,140) (12,000)
departmental profit subject to a minimum
of Rs.6,000
Adjusted Profit/Loss (40,200) 50,400 64,260 108,000

Working Notes
Manager‘s commission is payable @10 % of departmental profit before charging such
commission (subject to a minimum of Rs.6, 000). Hence manager‘s commission already
deducted will be as follows.
Departments Profit /Loss after Manager’s Commission
charging manager’s
commission
A (38,000) 6,000
B 50,400 1/9 of Rs.50,400 or 6,000 whichever is higher
i.e. 6,000
C 72,000 1/9 of Rs.72,000=8,000
D 108,000 1/9 of Rs 1,08,000=12,000

2. Unrealized profit on unsold departmental stock:


Profit earned by Department A by transfer of stock Amount
Department B@110%= Rs.22,000*10/110=2,000
Department D @120%= Rs.1,200*20/120=200 2,200

Profit earned by the Department C by transferring stock to


Department D@120%= Rs.3,600*20/120=600
Department B@120%= Rs.48,000*20/120=8,000 8,600

5.
a) From the following information, find out the risk weighted exposure for
operational risk assuming 11% is the capital adequacy requirement. 5
(
Rs.)
Particulars Year I Year II Year III
Interest Income (Rs.)
1,466,454 1,626,474 1,775,583
Interest Expenses 561,964 648,842 767,411
Commission and Discount Income 132,816 165,448 193,224
Other Operating Income 137,301 198,130 151,637
Exchange Fluctuation Income 41,301 52,325 40,329
Addition in interest Suspense Account 25,693 34,376 36,711
Non-Operating and Extraordinary Income 2,795 1,887 100,257
b) Nepa Roadways has taken a transit insurance policy. Suddenly in the year
2073-2074 the percentage of accident has gone up to 12% and the company
wants to recognize insurance claim as revenue in 2073-2074 in accordance
with relevant Accounting Standard. Do you agree? 5
c) Kalika Constructions undertook the construction of a highway on
01.04.2073. The contract was to be completed in 2 years. The contract price
was estimated at Rs. 150 crores. Up to 31.03.2074 the company incurred Rs.
120 crores on the construction. The engineers involved in the project
estimated that a further Rs. 45 crores would be incurred for completing the
work.
What amount should be charged to revenue for the year 2073-2074 as per
the provision of Nepal Accounting Standard 11 ―Construction Contract"?
Show the extract of the Profit & Loss A/c in the books of Kalika
Constructions. 5

ANSWERS:
a)
Net Interest Income 904,490.00 977,632.00 1,008,172.00

Commission and Discount Income 132,816.00 165,448.00 193,224.00

Other Operating Income 137,301.00 198,130.00 151,637.00

Exchange Fluctuation Income 41,301.00 52,325.00 40,329.00


Addition/Deduction in Interest Suspense during the
period 25,693.00 34,376.00 36,711.00

Gross Income( a) 1,241,601.00 1,427,911.00 1,430,073.00


Alfa (b) 15% 15% 15%

Fixed percentage of Gross Income © 186,240.15 214,186.65 214,510.95


Capital Requirement for operational risk (d )
(average of C) 204,979.25

Risk weight in times(e) 11

Equivalent Risk Weight Exposure (f=d*e) 225,4771.75

Net Interest Income


Year I Year II Year III
Interest Income 1,466,454.00 1,626,474.00 1,775,583.00
Interest Expenses 561,964.00 648,842.00 767,411.00
Net Interest Income 904,490.00 977,632.00 1,008,172.00

b) NAS 18 on ―Revenue‖ defines revenue as ―gross inflow of economic benefits during the
period arising in the course of the ordinary activities of an entity when those inflows result in
increases in equity, other than increases relating to contributions from equity participants‖
To recognize revenue NAS 18 requires that revenue arises from ordinary activities and that it
can be measured reliably and it is probable that the economic benefits associated with the
transaction will flow to the entity. As per the Standard, where the ability to assess the
ultimate collection with reasonable certainty is lacking at the time of raising any claim,
revenue recognition is postponed to the extent of uncertainty involved. In such cases, it may
be appropriate to recognize revenue only when it is reasonably certain that the ultimate
collection will be made.
In the given case, Nepa Roadways wants to recognize Insurance claim because it has
increased over the previous year. But, there are uncertainties involved in the settlement of
the claim. Also, the claim does not seem to be in the course of ordinary activity of the
company.
Hence, Nepa Roadways is not advised to recognize the Insurance claim as revenue.
Marking Scheme
3 marks for provision
2 marks for decision
c)
Statement showing the amount to be charge to Revenue as per NAS 11
Particulars Rs. in Crores
Cost of construction incurred up to 31.03.2074 120
Add: Estimated future Cost 45
Total estimated cost of construction 165
Degree of completion (120/165×100) 72.73%
Revenue recognized (72.73% of 150) 109 ( approx.)
Total foreseeable loss (165-150) 15
Less: Loss for the current year (120-109) 11
Loss to be provided for 4

Profit and Loss Account (Extract)


Particulars Rs. in crores Particulars Rs. in crores
To Construction Costs 120 By Contract Price 109
To Provision for Loss 4 By Net Loss 15
124 124

6. Write short notes on: (5×3=15)


a) Fair Value Accounting
b) Exchange Equalization Fund
c) Underlying assumption on Preparation and presentation of Financial Statement
d) Sale and Lease Back Transactions as per NAS 17.
e) Write any five peculiar features of Farm Accounting.
ANSWERS:
a) Fair Value is the amount for which an asset could be exchanged or a liability settled between
knowledgeable willing parties in an arm's length transaction. Fair value is measured using the
price in the principal market for the asset or liability (i.e. the market with the greatest volume and
level of activity for the asset or liability) or, in the absence of a principal market, the most
advantageous market for the asset or liability. Detailed guidance shall be required for measuring
the fair value of liabilities, including a description of the compensation that market participants
would demand to take on an obligation.
b) Section 45 of the Bank and Financial Institution Act, 2073 requires that every Bank and Financial
Institution which has obtained a license to deal in foreign exchange must maintain a Exchange
Equalization Fund. The Bank and Financial Institution must transfer 25% of the revaluation
profit earned as a result of changes in the exchange rates of foreign currencies other than the
Indian currency every year at the end of the same fiscal year to the exchange equalization fund.
The amount credited to the Exchange Equalization Fund may not be spent or Transferred for any
purpose other than the adjustment of losses resulting from the devaluation of foreign currencies
without the approval of the Nepal Rastra Bank.

c)
Accrual Basis
In order to make their objectives financial statements are prepared on the accrual basis of
accounting. Under this basis, the effects of transaction and other events are recognized when they
occur (and not as cash or equivalent is received or paid) and they are recorded in the accounting
records and reported in financial statements of the periods to which they relate.
Going Concern
The financial statements are normally prepared on the assumption that an entity is a going
concern and will continue in operation for foreseeable future. Hence, it is assumed that the entity
has neither the intention nor the need to liquidate or curtail materially the scale of operations; if
such intention or need exists, the financial statements may have to be prepared on a different
basis.
d) As per NAS 17 on ‗Leases‘, a sale and leaseback transaction involves the sale of an asset by the
vendor and the leasing back of same asset to the vendor. The lease payments and the sale price
are usually interdependent, as they are negotiated as a package. The accounting treatment of a
sale and lease back transaction depends upon the type of lease involved.
If a sale and leaseback transaction results in a finance lease, any excess of sale proceeds over the
carrying amount shall not be immediately recognized as income by a seller-lessee. Instead it shall
be deferred and amortized over the lease term.
If sale and leaseback transaction results in a operating lease, and it is clear that the transaction is
established at fair value, any profit or loss should be recognized immediately. If the sale price is
below fair value any profit or loss should be recognized immediately except that, if the loss is
compensated by future lease payments at below market price, it should be deferred and amortized
in proportion to the lease payments over the period for which the asset is expected to be used. If
the sale price is above fair value, the excess over fair value should be deferred and amortized
over the period for which the asset is expected to be used.
Poultry, livestock and other agricultural produces.
e) While preparing the farm account one should be conversant with the peculiar features of farm
accounting which are as below:
1. The business is family type and there may be single bank account both for business and
family purpose.
2. A large chunk of the farm produce is appropriated towards consumption by the family
members.
3. The family provides labor for the farm in addition to the time devoted for management.
4. There are several divisions in the farm such as crops, dairy, poultry etc.
5. The output of certain division becomes the input of other divisions.
6. Farming operations are fraught with uncertainties as to the weather, pests, market price of the
input and outputs and government policies.
7. In some cases, agriculture is a part time occupation.
8. Due to the intrinsic nature, there are difficulties in ascertaining the value of standing crops
poultry, livestock and other agricultural produces.
Audit and Assurance
Maximum Marks - 100

Total No. of Questions- 7 Total No. of Printed Pages- 9

Time Allowed - 3 Hours


Marks
Attempt all questions.
1. As an auditor, give your opinion with explanations on the following cases: (45=20)
a) You are an engagement manager for an audit team for M/s Sun Life Ltd. for
financial statements as on Ashadh 31, 2073. Your team is about to drafting
management letter after all transaction testing and review; the board of director of
M/s Sun Life Ltd. has appointed new CEO. After assuming his office, he decided to
change few general managers and auditor for FY 2072-73 with immediate effect.
b) M/s Raddison Hotel on 15.4.2073 imported two Mercedes Benz from Germany at a
price Euro 200 thousand each upon terms of credit that price should be settled
within three months from the date of purchase. The company capitalized the asset
and created a liability for the capital goods converting the foreign currency liability
to Nepalese Rupees at a rate of exchange prevailing as on 15.4.2073. When the
company settled the liability on 30.8.2073, it had to incur an additional amount of
Rs.15,00,000 due to foreign exchange rate on the date of settlement. It added this
additional amount of exchange variation in the capital cost of the asset and charged
depreciation upon an enhanced amount of asset value from 30.8.2073.
c) Mr. A Kumar and Mr. B Kumar are the two chartered accountants just qualified
their exams and took membership from ICAN in Bhadra 2074. A Kumar, who
without holding the Certificate of Practice, signed a document in capacity of the
member holding Certificate of Practice and B Kumar, as being a member of ICAN,
committed an act contrary to the provisions of Section 41of ICAN Act.
d) Mr. Rajaram is a director in M/s P Ltd. and also in M/s Q Ltd. M/s Q Ltd. purchases
goods from M/s P Ltd. The two companies reported the transaction as transaction
with related party as on Ashadh 31, 2074 under NAS-24.

Answer:
a) As per provision contained in Section 119 of companies Act 2063 relating to removal of
appointed auditor, no auditor appointed pursuant to Companies Act shall be removed pending
the completion of audit of accounts of any financial year for which he/she was appointed as the
auditor.

However, if any auditor breaches the code of conduct of auditors or does any act against the
interest of the company which has appointed him as the auditor or commits any act contrary to
the prevailing law, such auditor may be removed through the same process whereby he/she was
appointed as auditor, by giving prior information to the Nepal Chartered Accountants Institute,
and with the approval of the regulatory authority, if any authorized by the prevailing law for the
regulation of business of the company concerned , and failing such authority, with the approval
of the Office of company registrar.
In case of M/s Sun Life Ltd, CEO is not the person to appoint and remove statutory auditor
and the actions taken by the CEO is null and void and additionally, an appropriate
communication to those charged with governance is required.

b) Nepal Accounting Standards (NAS) 21 states that the exchange differences on the settlement of
monetary items at a rate different from those at which they were translated on initial
recognition during the period or in previous financial statements shall be recognized in profit or
loss in the period in which they arise. Therefore, in view of the above, the M/s Raddison Hotel
should charge the additional amount of Rs.15,00,000 to profit and loss account in accordance
with the said standard and not to capitalize.

c) Section 41 of Nepal Chartered Accountants Act, 2053 has made different level of punishment
for different levels of culpability. Here in case of A Kumar, If a person, who has not obtained a
Certificate of Practice and is proved to have signed any document in capacity of the member
holding Certificate of Practice, shall be liable to punishment with a penalty up to two thousand
rupees or imprisonment for a period of up to three months or both.

In case of B Kumar, if a member, who commits any act contrary to the provisions of this Act or
Regulations framed under this Act other than the provisions of this section, shall be suspended
for a maximum period of five years and shall be liable of punishment with a maximum penalty
of two thousand rupees or imprisonment for a maximum period of three months or both.
So, A Kumar and B Kumar are to be punished accordingly.

d) According to the Provisions of NAS-24, the given transaction between P Ltd. and Q Ltd. is not
a related party transaction. According to the standard, related party relationship includes
enterprises owned by directors or major shareholders of the reporting enterprise and enterprises
that have a member of key management in common with the reporting enterprise. In the given
case, none of the enterprises is owned by Rajaram. He is only a director in both the enterprises
not a Key Management Personnel (Such as Managing Director, Whole time Director, etc).
Therefore, P Ltd should not report the transaction as related party transaction.

2. Give your comments on the following cases: (45=20)


a) A proprietary audit firm of Fellow Chartered Accountants has accepted the
engagement to audit the accounts of a company with annual turnover of Rs. 10
million at the audit fee of Rs. 12,000. The firm anticipates that the audit will
consume estimated time of 3 man days of the Chartered Accountant.
b) During the course of audit of a company for fiscal year 2073/74, the finance director
of the company offered your audit team a free weekend at Fulbari Resort Pokhara.
c) The auditor of H Ltd. wanted to obtain confirmation from its creditors. But the
management made a request to the auditor not to seek confirmation from certain
creditors citing disputes. Can the auditor of H Ltd. accept to this request?
d) M/s KBC Associates, a Chartered Accountant was engaged by Hanuman& Co.
Ltd. for auditing their accounts. He sent his letter of engagement to the Board of
Directors, which was accepted by the company. In the course of audit ofthe
company, the auditor was unable to obtain appropriate sufficient audit evidence
regarding receivables. The client requested for a change in the terms of engagement.
Offer your comments in this regard for acceptance of changes in terms of
engagement.

Answer:
a) As per the decision of the council, a FCA member holding COP shall charge the audit fee to his
clients and the fee shall not be less than Rs. 15,000. In the present case, the FCA member has
charged Rs. 12,000 to a private company is less than the minimum fee. The auditor should have
charged at least Rs. 15,000 as the audit fee. Hence the member seems to have not followed the
directives of the council and accordingly may be subject to disciplinary action.

b) As per the Section 260 of code of ethics issued by ICAN, accepting gifts or hospitality from
an audit client may create self-interest threat and familiarity threats. If a firm or member of
audit team accepts gifts or hospitality unless the value is nominal and inconsequential the threat
created would be so significant that no safeguard could reduce the threats to an acceptable low
level. Consequently a firm or member of the audit team shall not accept such gifts or
hospitality.

The given case represents a self-interest threat as the acceptance of goods and services unless
value of such goods or service offered is very insignificant .As it is unlikely that a weekend at a
luxury hotel for whole audit team has an insignificant value, then, this offer should politely be
declined.

c) NSA 505, ―External Confirmations‖, establishes standards on the auditor‘s use of external
confirmation as a means of obtaining audit evidence. It requires that the auditor should employ
external confirmation procedures in consultation with the management.

The auditor may come across certain situations in which the management may request him not
to seek external confirmation from certain parties because of some reasons, for example, due to
a dispute with the particular creditor or debtor.

If the management refuses to allow the auditor to a send a confirmation request, the auditor shall
 Inquire as to Management‘s reasons for the refusal, and seek audit evidence as to their
validity and reasonableness,
 Evaluate the implications of management‘s refusal on the auditor‘s assessment of the
relevant risks of material misstatement, including the risk of fraud, and on the nature,
timing and extent of other audit procedures, and
 Perform alternative audit procedures designed to obtain relevant and reliable
audit evidence.

If the auditor concludes that management‘s refusal to allow the auditor to send a confirmation
request is unreasonable or the auditor is unable to obtain relevant and reliable audit evidence
from alternative audit procedures, the auditor shall communicate with Those Charged With
Governance (TCWG) and also determine its implication for the audit and his opinion.
d) NSA 210 ―Agreeing the Terms of Audit Engagement‖ deals with the auditor‘s responsibilities
in agreeing the terms of the audit engagement with management. As per NSA 210, if prior to
completing the audit engagement, the auditor is requested to change the audit engagement to an
engagement that conveys a lower level of assurance, the auditor shall determine whether there
is reasonable justification for doing so.

The auditor shall not agree to a change in the terms of the audit engagement where there is no
reasonable justification for doing so. If the terms of the audit engagement are changed, the
auditor and management shall agree on and record the new terms of the engagement in an
engagement letter or other suitable form of written agreement.

If the auditor is unable to agree to a change of the terms of the audit engagement and is not
permitted by management to continue the original audit engagement, the auditor shall:

 Withdraw from the audit engagement where possible under applicable law or regulation;
and
 Determine whether there is any obligation, either contractual or otherwise, to report the
circumstances to other parties, such as Those Charged With Governance (TCWG), owners
or regulators.

3. Answer the following: (35=15)


a) What are the general limitations of audit?
b) Pandit & Associates is the firm of practicing chartered accountants. The firm accept
the audit of Siddhartha Ltd. where Government of Nepal (GoN) holds 60 %
ownership. What is the responsibility of Pandit & Associates before accepting such
audit.
c) Write down examples of situations where external confirmations may be used by the
auditor.
Answer:
a) An audit in accordance with NSA is designed to provide reasonable assurance, so a degree of
imprecision is inevitable due to some inherent limitations as mentioned below:

i. Test Checking/Sampling:- Auditor uses sampling during performance of audit. Audit based
on test nature does not detect all frauds and errors.

ii. Nature of evidence:- The evidence obtained by the auditor are persuasive rather than
conclusive. They only enable auditor to provide reasonable assurance (not absolute
conclusion).

iii. Judgement: - Auditor uses professional judgement while obtaining audit evidence and
evaluating the reasonableness of assertion and estimates made by the management. The
judgement made by the auditor may not always be correct.

iv. Inherent limitation of internal control: - Internal controls suffer from limitation such as
collusion among the employees or abuse of authority by management.
vi. Constraint of Time:- Since most of the audits are carried out only after the year end and
have similar deadlines. Provided limited time period auditor may not able to observe all the
transactions resulting possibilities of oversight.

b) As per the provision of section 7 (2) Audit Act 2048, the auditor of companies where GoN
holds majorities of share should be done in consultation with the Office of Auditor General of
Nepal. Further the practicing chartered accountant as per section 34 (13) of ICAN Act 2053
should not accept his appointment as an auditor of an organization without ascertaining that all
required procedures for appointment as the auditor under the prevailing law has been duly
fulfilled.

In the light of aforesaid provisions of Audit Act 2048, Pandit & Associates before accepting
appointment, should ensure that whether Siddhartha Ltd. has consulted with the Office of
Auditor General of Nepal for their appointment.

c) As per provision sated in NSA 505 (External Confirmations); external confirmations are
frequently used in relation to account balances and their components, but need not be restricted
to these items. Accordingly examples of situations where external confirmations may be used
by the auditor are:

i. bank balances and other information from bankers,


ii. accounts receivable balances,
iii. stocks held by third parties at bonded warehouses for processing or on consignment,
iv. property title deeds held by lawyers or financiers for safe custody or as security,
v. investments purchased from stockbrokers but not delivered at the balance sheet date,
vi. loans from lenders, and
vii. accounts payable balances.

4. Answer/Comment on the following: (35=15)


a) Explain the provision related to Second Opinions with reference to Code of Ethics.
b) Explain the various types of threat explained in ICAN Code of Ethics along with
examples.
c) Pradhan Ghimire & Associates, a firm of chartered accountant has two partners. The
firm is already holding audit of 100 companies including audit of 20 public limited
companies. The firm is further offered for the audit of Premier Investment Limited.
Answer:
a) Section 230 of Code of Ethics contains the provision relating to Second Opinions.
When asked to provide second opinion on the application of accounting, auditing, reporting or
other standards or principles to specific circumstances or transactions by a company or on
behalf of a company or any entity that is not an existing client, a professional accountant in
public practice shall evaluate the significance of any threats and apply safeguards where
necessary to eliminate them or reduce them to an acceptable level.

The existence and significance of the threat will depend on the circumstances of the request and
all the other available facts and assumptions relevant to the expression of professional
judgment. Examples of such safeguards include seeking client permission to contact the
existing accountant, describing the limitations surrounding any opinion on communications
with the client and providing the existing accountant with a copy of the opinion. If the company
or entity seeking the opinion will not permit communication with the existing accountant, a
professional accountant in public practice shall determine whether, taking all the circumstances
into account, it is appropriate to provide the opinion sought.

b) The various kinds of threats as explained in Section 200 of ICAN code of ethics are as follows:

Self Interest Threat: The threat which occur when an auditing firm, its partner or associate
could benefit from a financial interest in an audit client Examples include direct financial
interest or materially significant indirect financial interest in a client, loan or guarantee from
concerned client, close business relationship with an audit client, potential employment with
client.

Self-Review Threat: The threat that a professional accountant will inappropriately evaluate the
result of a previous judgment made or service performed by the professional accountant or
another individual within the professional accountant‘s firm or employing organization on
which the accountant will rely when forming a judgement as part of providing a current service.
For eg: valuation service along with audit service, accounting service.

Advocacy Threat: The threat that a professional accountant will promote client‘s opinion to the
point where people may believe that objectivity is getting compromised. For eg: Auditor acting
as an advocate on behalf of audit client in litigation or dispute with the third parties.

Familiarity Threat: The threat that a profession accountant due to a long or close relationship
with a client, will be too sympathetic to their interests or too accepting of their work. For eg:
participation in client‘s affair, family and personal relationship, audit partners leaving to join
the clients etc.

Intimidation Threat: The threat that a professional accountant will be deterred from acting
objectively because of actual or perceived pressure, including attempts to exercise undue
influence over professional accountant. For eg: A firm being treated with dismissal from a
client engagement.

c) The council vide its meeting no 194 dated Falgun 4, 2071 has amended provision relating to
ceiling over the number of audit. The amended provision is as below and is effective for
appointment from Shrawan 1, 2072.

A member holding Certificate of Practice of can perform the audit of not more than 100 entities
in a financial year, out of which audit of public companies shall not exceed 10. The above limit
is applicable for each member of a partnership firm. However, audit of certain entity having
turnover of less than 20 lacs is not included while calculating the above limit.

Thus, the firm can take maximum of 200 audits (100*2), but the number of public company audit
can‘t exceed 20. As, the firm is already holding audit of 20 public limited companies Pradhan,
Ghimire & Associates can‘t take the audit of the Premier Investment Limited.

5. Answer the following: (25=10)


a) What are qualities of statutory auditor.
b) Explain the use of written representation as audit evidence, including its limitations.

Answer:
a) An auditor should adhere to the fundamental principles mentioned in code of ethics. The
fundamental principles are integrity, objectivity, professional competence and due care,
confidentiality and professional behavior. Auditor should be honest, sincere and
straightforward while performing the professional duties. Further, the qualities that an auditor
must possessed are independence, knowledge, technical skills, tact, caution, firmness, good
temper, discretion, industry judgment, patience, clear headedness and good communication
skills. He/She must be able to analyse and interpret the problems so that the situations can be
handled accordingly.

He/she must have a thorough knowledge client‘s business and other disciplines like general
principles of law, economics, taxation, information technology, management, financial
management.

b) The auditor should seek written representation as evidence that management acknowledges its
responsibility for the fair presentation of financial statement. The auditor should obtain written
representations from management on matters material to the financial statement when other
sufficient appropriate audit evidence cannot reasonably expect to exist. Further, the possibility
of misunderstanding between the auditor and management is reduced when oral representations
are confirmed by management in writing. The written representation shall be in the form of
letter addressed to the auditor. If the auditor has concerns about the competence, integrity or
diligence of management, the auditor shall determine their effect on the reliability of written
representation.
However, although written representations are a form of audit evidence, they do have their
limitations. Although written representations provide necessary evidence, they do not provide
sufficient appropriate audit evidence on their own. Furthermore, the fact that management has
provided reliable written representation does not affect the nature or extent of other audit
evidence that the auditor obtains.

6. Write short notes on the following: (42.5=10)


a) Impairment of Assets
b) Audit Note-book
c) Subsequent Events
d) Vouching of Advances to the Suppliers
Answer:
a) Impairment of Assets: Besides charging annual depreciation on assets by the reason of normal
wear and tear, affluxion of time and obsolescence to re-instate the correct value of the assets
considering the future cash flows that the assets can generate, impairment loss needs to be
provided. The difference between the carrying amount of an asset and recoverable amount is
termed as impairment loss. The treatment of impairment loss is similar to depreciation except
the fact that it can be re-instated in future, if the recoverable amount of the asset exceeds
the carrying amount. The auditor must ensure that the provisions of NAS 36 ―Impairment of
Assets‖ are followed.

a) Audit Note-book: An audit note book is usually a bound book in which a large variety of
matters observed during the course of audit are recorded. Audit note books form part of audit
working papers and for each year a fresh audit note book is maintained. In case an auditor
classifies his working paper into permanent and current, then audit note book shall form part of
the current file. It is in any case a part of the permanent record of the auditor available for
reference later on, if required.
The audit note book also provides a valuable help to the auditor in picking up the links of work
when the concerned assistant is away or the work is stopped temporarily. It is also used for
recording the various queries raised in the course of the work and their state of disposal. In
respect of disposed queries, explanation obtained and evidence seen would be recorded in the
said book, while queries remaining un-disposed of would be noted for follow up.

b) Subsequent Events: NSA 560 on ―Subsequent Events‖, defines the term ―subsequent events‖ as
events occurring between the date of the financial statements and the date of the auditor‘s
report, and facts that become known to the auditor after the date of the auditor‘s report.,
―subsequent events‖ also refer to significant events which occurred upto the date of report of
the auditor of that component. Thus, subsequent events are those events which occur after the
date of the balance sheet till the audit report is signed by the auditor.

c) Vouching of Advances to suppliers can be done as follows:

 Obtain schedule of debit balances in trade payables‘ account and pay particular attention to
the age of the balances. Also scrutinize the bought ledger.
 Enquiry should be made for long unadjusted outstanding and check as to whether any of
them would require provisioning.
 Examine that the advances have not been shown as deposits in balance sheet.
 Confirmation of balances should be obtained and reconciliation be done in case of any
discrepancies.

7. Distinguish between: (25=10)


a) Internal Check and Internal Audit
b) Vouching and Verification
Answer:
Distinction between Internal Check & Internal Audit

Internal Check Internal Audit


SN SN
1 Internal check is not a specific check, 1 Internal audit is specifically done to
but the duties of different persons are check that the accounts are properly
so arranged that a person‘s work is maintained and the systems are in
automatically checked by another control.
person while carrying out the normal
duty.
2 Internal check does the preventive job 2 Internal audit does the detective job of
i.e. internal check is derived so that identifying frauds and errors and
frauds and errors are prevented. rectifying them.
3 It is more of process in a day to day 3 It is specific defined job.
functioning of the business.
4 All the persons in the organization are 4 Specific persons are appointed to the
involved to maintain the internal internal audit.
check system.
5 It is required in all organization in 5 Carrying out internal audit is not
formal or informal. compulsory. It is done based on
management decision.
6 It does not include internal audit. 6 It include internal check.

d) Distinction between Vouching & Verification:

Points of diff. Vouching Verification


Meaning The act of examining the vouchers is Verification van be explained as
known as vouching. A voucher is any establishing the truth or securing
documentary evidence in support of a some kind of confirmation with
transaction entered in the books of account.
respect to the assets and
liabilities appearing in the
Balance Sheet of a concern.
Nature Vouching involves establishing the Verification goes beyond
& arithmetical accuracy and the authenticity vouching. It seeks to establish
Purpose ofthe transactions of a concern. Vouching that assets as stated in the
proves that an asset ought to exist. Balance Sheet of a concern exist
in fact and that the liabilities are
properly disclosed. Verification
proves that an asset does exist.
Time It is done during the whole year. It is done during end of the year.
Utility Certifies correctness of records. Certifies correctness of assets
and liabilities.
Personnel It is done by the junior staff of the auditor It is done by the auditor himself
under the supervision of a senior person. assisted by senior.
Corporate & Other Law
Maximum Marks - 100

Total No. of Questions - 7 Total No. of Printed Pages -11


Time Allowed - 3 Hours
Marks
Attempt all questions.
1. Answer the following questions: (5×5=25)
a) Mount Everest Ltd. passed a special resolution at the meeting of the
shareholders to issue shares at a discount to a commercial bank. The objective
of the issue is to convert the loan borrowed by the company from the
commercial bank to equity share. Discuss the validity of resolution passed in
the context of the provisions regarding prohibition to issue shares at a discount
as specified in the Companies Act, 2063?
b) The meeting of the Board of Directors of Royale Tours Ltd. decided to issue
preference share as it was authorised by its memorandum and articles of
association. Advise them regarding the matters to be disclosed while issuing
the preferential shares.
c) Mr. Harry is in the process of incorporating public company in association
with his friends. But he is not aware about the validity of different expenses
done by him during the process of incorporating companies and wants your
expert advice on this. Suggest him the relevant provision of the Companies
Act, 2063 about the pre-incorporation contract.
d) 'Doctrine of indoor management can't be claimed by an outsider dealing with
the company in every circumstances'. Justify your answer.
e) CEO of the Reliance Hydropower Limited is in the process to sue against the
supplier of the Electro mechanical equipment to the company for breach of the
contract. But he is not sure about the legal provision of the Companies Act
whether the company can sue or not. Advise the CEO of the Hydropower
company regarding the relevant provision of the Companies Act whether the
company can sue or not?
Answer:
a) Section 64 of Company Act, 2063 makes provision regarding the prohibition on issue or
sale of share at a discount. Provision of Section 64 of Company Act, 2063 is as follows:
Section 64 - Prohibition on issue or sale of share at a discount
(1) A Company shall not issue or sale its share at a discount.
(2) Notwithstanding anything contained in sub-section (1), a company may, for the following
purpose, issue or sell shares at discount by adopting a special resolution at the General
Meeting to that effect, not being less than the percentage specified in that resolution. (3
for a, b, c, d)
(a) for issuing or selling shares pursuant to a capital restructuring scheme of the
company.
(b) for issuing or selling shares pursuant to a scheme of converting loans borrowed by
the company into shares with the consent of creditors.
(c) for issuing or selling shares pursuant to an employee share scheme.
(d) for issuing shares on such other conditions/purposes as approved by the office.

As the company has passed a special resolution to convert the loan borrowed by the company
from the commercial bank to equity share, resolution passed is valid as per the provision
above mentioned.

b) A company may issue preference shares as provided for in the Companies Act, 2063,
memorandum or articles of association. While issuing preference share, the issuing
company must disclose the various matters as prescribed by the Act. As per Section
65(3), the following maters shall be disclosed:
 Whether preference is given to receive dividends against ordinary shares;
 Percentage of dividends receivable by preference shareholders;
 Whether dividends get cumulated every year (cumulative) or profits are distributed
only in a year wherein profit is made (non-cumulative);
 Whether preference is given while paying amount of share in the event of liquidation
of company;
 Whether voting right is attached there to; and if voting right is attached, whether such
right is available only in the case of preference share or also in other matters;
 Whether voting right is available also in other matters pursuant to Clause (e), the
proportion to which such right is exercisable;
 Whether preference shares can be converted into ordinary shares;
 Whether the amount of preference shares can be redeemed (redeemable) or cannot be
redeemed (irredeemable) after a certain period;
 Whether in redeeming preference shares, premium is payable on redemption.

c) As per section 17 of Companies Act, 2063


(1) A contract made prior to the incorporation of a company shall be a proposed contract
only, and such contract shall not be binding on the company.
(2) If, prior to the incorporation of a company, any person carries on any transaction or
borrows money on behalf of the company, such person shall be personally liable for
any contract related with the transaction so carried on, subject to Sub-section (3).
(3) If, within the time mentioned in any transactions or within the reasonable time after
the incorporation of a company, the company, through its act, action or conduct,
accepts any act, action or conduct, accepts any act, action to borrowing done or made
prior to the date of authorization to commence its transactions or endorses such act or
action, that transaction shall be binding on the company and the other contracting
party; and the person carrying out such act to action shall be released from the
personal liability to be borne pursuant to Sub-section(2).
(4) Notwithstanding anything contained elsewhere in this Section, the consensus
agreement of a private company shall govern any contracts made prior to the
incorporation of such company.
d) The rule of doctrine of indoor management is, however, subject to certain exceptions. In
other words, relief on the ground of ‗indoor management‘ can't be claimed by an outsider
dealing with the company in the following circumstances:
1. Knowledge of Irregularity: -
The first and the most obvious restriction is that the rule has no application where the
party affected by an irregularity had actual notice of it. Knowledge of an irregularity
may arise from the fact that the person contracting was himself a party to the inside
procedure. Similarly, in Howard v. Patent Ivory Manufacturing Co.[10], where the
directors could not defend the issue of debentures to themselves because they should
have known that the extent to which they were lending money to the company
required the assent of the general meeting which they had not obtained.
2. Suspicion of Irregularity: -
The protection of the ―Turquand Rule‖ is also not available where the circumstances
surrounding the contract are suspicious and therefore invite inquiry. Suspicion should
arise, for example, from the fact that an officer is purporting to act in matter, which is
apparently outside the scope of his authority.
3. Forgery: -
Forgery may in circumstances exclude the ‗Turquand Rule‘. The only clear
illustration is found in the Ruben v Great Fingall Consolidates [15]; here in this
case the plaintiff was the transferee of a share certificate issued under the seal of the
defendant‘s company. The company‘s secretary, who had affixed the seal of the
company and forged the signature of the two directors, issued the certificate.
4. Representation through Articles: -
The exception deals with the most controversial and highly confusing aspect of the
―Turquand Rule‖. Articles of association generally contain what is called ‗power of
delegation‘.
The effect of a ―delegation clause‖ is ―that a person who contracts with an individual
director of a company, knowing that the board has power to delegate its authority to
such an individual, may assume that the power of delegation has been exercised.‖
5. Acts outside apparent authority: -
Lastly, if he act as an officer of a company is one which would ordinarily be beyond
the power of such an officer, the plaintiff cannot claim the protection of the
―Turquand rule‖ simply because under the articles power to do the act could have
been delegated to him. In such a case the plaintiff cannot sue the company unless the
power has, in fact, been delegated to the officer with whom he dealt. A clear
illustration is Anand Behari Lal v Dinshaw[18] here the plaintiff accepted a transfer
of a company‘s property from its accountant. Since such a transaction is apparently
beyond the scope of an accountant‘s authority‘ it was void. Not even a ‗delegation
clause‘ in the articles could have validated it, unless he was, in fact, authorized.

e) As per the section 7 of the Companies Act, 2063.


(1) Any company incorporated under this Act shall be an autonomous and corporate
body with perpetual succession.
(2) Subject to this Act, company like an individual, acquire, hold, sell, dispose of or
otherwise deal with, any movable or immovable property.
(3) A company may sue and be also sued by its own name.
(4) A company may, like an individual, enter into a contract and exercise the rights and
perform the obligations as referred to in the contract.
Hence as per the above mentioned provision of the companies Act, Reliance
Hydropower can sue against the supplier on its own.
2. Answer the following questions: (3×5=15)
a) ABC Bank Limited (proposed) has filed an application to Nepal Rastra Bank
(NRB) for the prior approval of incorporation of the bank. NRB has refused
to grant approval citing that name of the proposed bank or financial
institution is identical with the name of another bank in existence. Give your
view on the contention of the NRB referring relevant provision of the Banks
and Financial Institutions Act (BAFIA), 2073.
b) What are the objectives of the Nepal Rastra Bank as per the Nepal Rastra
Bank Act, 2058?
c) Miss Dakshina has been elected as a Board of Director of Garima Bank Ltd.
You, an employee of the bank has to prepare a document including the
matters to be submitted by a board of director after her appointment. List out
the matters you have to collect from her for the disclosure to be made by the
director as per the Banks and Financial Institutions Act (BAFIA), 2073.
Answer:
a) As per section 7 (1)(a) of the Banks and Financial Institution Act, 2073; The Rastra Bank
may refuse to grant prior approval for the incorporation of a bank or financial institution,
if the name or the operation of the proposed bank or financial institution is not in the
interest of the public at large, religion or community, etc.
In the given case the name of the proposed bank is identical with the name of the another
bank in existence which may not be in the interest of public at large. Hence the refusal of
the NRB to grant approval in the above mentioned case is in line with the section 7(1)(ka)
of the Banking and Financial Institution Act, 2073.

b) As per Section 4(1) of Nepal Rastra Bank Act, 2058, the objectives of the Nepal Rastra
Bank shall be as follows:-
(a) To formulate necessary monetary and foreign exchange policies in order to maintain
the stability in the price and balance of payment for financial stability and sustainable
development of economy, and manage it;
(b) To promote entire banking and financial system of the Nepal and to enhance its public
credibility by maintaining stability in banking and financial sector;
(c) To develop a secure, healthy and efficient system of payment;

Also as per the section 4(2), the Bank shall, without any prejudice to the objectives
referred to in Sub-section (1), extend co-operation in the implementation of the economic
policies of Government of Nepal.

c) According to Section 24 of the BAFIA, 2073 has provided the matters to be disclosed after
assuming the office by every director. As per the section, every director shall disclose in
writing to the bank or financial institution the following matters:
1. Full name, address, academic qualification and experience of director.
2. If he or she is a director, office bearer or employee of any other institution, the
details of such post and responsibility thereof.
3. If he or she or any of his or her family members has entered into or going to enter
into any kind of contract with the concerned bank or financial institution, details
thereof.
4. If he or she has any kind of interest in the appointment of the chief executive,
managing director, secretary, auditor and general manager, details thereof.
5. Particulars of such shares or debentures in the concerned bank or financial institution
or in its holding or subsidiary company as subscribed by him or her or by his or her
family.
6. If he or she is a director of any company, details thereof.
7. If any member of his or her family is working as an officer of the bank or financial
institution, details thereof.
8. Power of Attorney given to the Rastra Bank to or caused to enquiry regarding the
director's financial and professional background or to or caused to exchange of such
notice or information.
9. Self-declaration of his eligibility to be a director as per the Act.
10. Such other details prescribed by the Rastra Bank as required to be disclosed by the
director to the Board.

3. Answer the following questions: (2×5=10)


a) Mr. Pradip Baral, one of the directors of XYZ Bank Limited passes some
secret information regarding the securities of the Bank to his brother. On the
basis of that secret information his brother bought huge number of shares of
XYZ Bank. Few months later, share price went up and Mr. Pradip‘s brother
earned huge amount of money from the transaction of those shares. Did Mr.
Pradip commit any crime under the Securities Act, 2063? If yes, what crime
did he commit?
b) State the grounds under which the Insurance Board may cancel the registration
of an insurance company (insurer) under the Insurance Act, 2049.
Answer:
a) Yes, Mr. Pradip Baral committed a crime against the Securities Act, 2063. His activities
are known as insider trading and it is punishable crime under Securities Act, 2063.
Section 91 of the Securities Act, 2063 states that in case any person conducts or causes
others to deal in securities on the basis of any insider information or notice that are
unpublished or communicates any information or notice known to such a person except in
the course of the discharge of his or her duties in manner likely to affect the price of
securities such a person shall be deemed to have been committed an insider trading in
securities.
b) Section 13 of the Insurance Act, 2049 provides the grounds for the cancellation of
registration of insurer by the Insurance Board.
As per section 13 of Insurance Act, the Insurance Board may cancel the registration of an
insurer by providing a written notice with effect from the date prescribed in the same
notice in the following circumstances:
a) If the insurance business is not started within six months from the date of obtaining the
certificate.
b) If it is felt that the liability of the insurer exceeds its assets within Nepal.
c) If the insurer could not fulfill the liability pursuant to the decision within three months
from the date of final decision of the court in the case filed under the insurance policy
issued within Nepal.
d) If the head office of the insurance business of any foreign insurer is situated outside
Nepal and in case it felt that Nepalese insurer has not obtained equal facilities there
which are enjoyed by the foreign insurer pursuant to the prevailing law of such
company.
e) If the insurer does not open its office inside Nepal.
f) If the insurer does not perform the functions to be performed or has performed any
functions which is not to be performed pursuant to this Act or the rule made under this
Act.

Before cancelling the registration of an insurer pursuant to sub-section 1) above, the


Insurance Board shall provide a reasonable time-limit to submit clarification to the
concerned insurer, stating the reason for cancelling its registration.
If the concerned insurer does not submit its clarification within the time period
mentioned in sub-section 2) above or clarification submitted by it is found not to be
satisfactory, the Insurance Board shall cancel the registration of such insurer pursuant
to sub-section 1) above, and shall publish a notice in two major newspapers to be
published in Nepal for the public information in general.
4. Answer the following questions: (2×5=10)
a) Discuss the functions, duties and power of Industrial Promotion Board
specified under the Industrial Enterprises Act, 2073.
b) Mentioned the eligibility criteria for entitlement of the bonus by the employee
as per the Bonus Act, 2030.

Answer:
a) The Functions, Duties and Power of the Board are listed in the section 19(1) of Industrial
Enterprise Act, 2073. Major functions, duties and power of the Board are as follows:-
(a) To make policy decision regarding industrial promotion, safeguarding and promoting
of investment and industrialization.
(b) To make necessary cooperation in implementing policies, laws and regulations
pertaining to the industry.
(c) To make regular evaluation of the policy related, legal, institutional and procedural
structure for industrialization of the whole country and recommend to the
Government of Nepal for necessary amendment in those areas.
(d) To make necessary policy decision on foreign investment and technology transfer
based upon the related existing laws.
(e) To make policy decision for the prevention of the industrial pollution.
(f) To carry out evaluation and review of development of industrial sector of the country
and make recommendation to Government of Nepal to take necessary steps.
(g) To address the issues raise while providing service, facilities and concession to the
industry as per this act and other related acts.
(h) To resolve the complain made by the industrialist by giving directives to the
concerned body.
(i) To make recommendation to Government of Nepal for the change of level,
classification and nature of any industries.
(j) To recommend the Government of Nepal to carry out necessary study, research and
survey for the promotion and development Industrial and foreign investment.

b) As per section 6 of the Bonus Act, 2030;


(1) An employee who has worked at least half period in a fiscal year, shall be entitled to
obtain bonus underthis Act. Provided that, no employee shall be entitled to obtain Bonus
who hasworked casually or in a shift basis.
(2) For the purpose of Sub-section (1), the following periods shall alsobe computed as a
period where an employee has worked.

(a) A period kept on reserve under any contract or under Section 11 of the Labour Act,
2048.
(b) A period under which an employee is on any leave with salary.
(c) A period of disablement caused by accident arising in course of business of the
enterprise.

5. List out the objectives of the ICAN and state the formation of council
including the election of president and vice-president pursuant to the Nepal
Chartered Accountants Act, 2053. (4+3+3=10)
Answer:
Objectives of ICAN:
ICAN has an important role to make accounting profession as reliable, authentic and standard
for its accuracy and enhance economic and social responsibility as a cause for economic
development of the nation. As per Section 5 of the NCA Act, 2053, the objectives of the
Institute shall be as follows:-
1. To play the role of a regulatory body to encourage the members to carry on accounting
profession being within the extent of the code of conduct in order to consolidate and
develop accounting profession as a cause for economic development of the nation.
2. To enhance social recognition and faith in accounting profession by raising awareness of
the general public towards the importance of accounting profession and the economic and
social responsibility of professional accountants.
3. To develop, protect and promote the accounting profession by enabling professional
accountants understand their responsibility towards the importance of accounting
profession and accountancy.
4. To develop mechanism of registration, evaluation and examination of accounting
professionals in consonance with international norms and practices so as to make the
accounting profession respectable and reliable.
Formation of the Council:
Pursuant to Section 7 (3), the Council shall consist of the following Council members:-
 Ten persons elected by and amongst Chartered Accountant members -
Member
 Four persons elected by and amongst Registered Auditors -
Member
 Three persons nominated by Government of Nepal, upon the recommendation of the
Auditor General, from amongst the persons well experienced in the field of accounting
profession. – Member
Election of President and a Vice-President:
Sub-section (4) provides the election of the President and a Vice-President. It reads that
the Council members shall elect a President and a Vice-President from the Fellow
Chartered Accountants (FCA) Council members referred to in clause (a) of sub-
section (3). The term of office of the chairperson and the vice- chairperson shall be
one year; and upon the expiry of their term of office, they may be elected for one
more term.

6. Answer the following: (5×4=20)


a) What are the essential elements of valid contract?
b) As per Securities Act, 2063 what matters need to be mentioned in prospectus
and in what conditions issuance of prospectus is not necessary?
c) What are the Functions, duties and powers of Accounting Standards Board
under the Nepal Chartered Accountants Act, 2053?
d) Mentions the major functions of World Trade Organization (WTO).
e) What special programs may be operated by the Government of Nepal relating
to the social welfare as per Social Welfare Act, 2049?

Answer:
a) The essential elements of valid contract are as follows
i) Proper offer and acceptance
There must be an agreement between two parties to create a contract. The agreement
involves a valid offer by one party and valid acceptance of the offer by the other party.
ii) Legal relationship
The parties of a contract must have to intend that it creates legal relation and has legal
consequences.
iii) Free consent
Free consent of the contracting parties is a must to create a contract. Consent may not
be free if it is obtained on the ground of coercion, undue influence, fraud,
misrepresentation and mistake of law.
iv) Meeting of minds
Meeting of minds (consensus ad idem) is necessary for a valid contract. The parties are
said to be of meeting of mind when they give their consent on the subject matter of the
contract in same sense and at the same time.
v) Competent parties
The parties to the agreement must be competent to contract. If incompetent parties are
in contract, it is not valid. eg. Minor
vi) Lawful objectives
The objective of a contract must be lawful. Those objectives are not lawful if they are
illegal, immoral and opposed to public policy.
vii) Lawful consideration
The agreement is enforceable only when contracting parties are giving and getting
something in return. The consideration must be lawful, which may be in past, present
or future.
viii) Agreement not declared void
The agreement must not be expressly declared void by any existing law.
ix) Certainty of meaning
Agreement must be certain and not vague or indefinite or words of latent ambiguity.
x) Possibility of performance
The objective or terms of contract must be possible to perform. Performance is the
very nature of contract, so impossible contract are not valid.
xi) Legal formalities
A contract may be made expressly but some contracts are recognized after fulfilling
certain formalities.

b) As per Section 32 of Securities Act, 2063 every prospectus shall contain such general
matters as required to be set down in the prospectus, capital and other information of
the issuer, main functions to be done by the issuer, information pertaining to legal
action, economic condition, general administration, management of the issuer,
information relating to the expert preparing the prospectus and the economic
statements contained in the prospectus and such other matters as may be prescribed.
In case of securities issued by the Nepal Rastra Bank, securities issued against the full
guarantee of the Government of Nepal, securities proposed to be sold to up to fifty
persons at a time, securities issued to own workers or employees, and securities
permitted by the SEBON as to issue and sell without issuing a prospectus.
c) Pursuant to Section 15a of the Nepal Chartered Accountant Act 2053, Government of
Nepal shall form a Accounting Standard Board in order to govern and regulate
financial reporting and accounting profession under the chairmanship of a person
nominated by Government of Nepal from amongst the fellow chartered accountants.
The Functions, duties and powers of Accounting Standards Board to be formed
pursuant to Section 15a are given under Section 15b of the Nepal Chartered
Accountants Act, 2053 as follows:
i) To develop Accounting Standards, on the basis of relevant International
Accounting Standards, in order to govern and regulate financial reporting and
accounting profession;
ii) To evolve appropriate process of development of accounting standards and publish
materials relating to accounting standards;
iii) To redraft, improvise and revise standards;
iv) To interpret the standards;
v) To undertake other related tasks related to accounting standards.

d) The major functions of WTO are discussed as below:


1) Administering WTO agreements:
The WTO agreements cover goods, services and intellectual property. They
include individual countries' commitments to lower customs tariffs and other
trade barriers and to open and keep open services markets. It has different
mechanism like General Council which works on behalf of ministerial
conference. It meets to Dispute Settlement Body and Trade Policy Review Body
to oversee procedures for settling disputes and to analyze members' trade
policies. There are Goods Council, Services Council and TRIPS Council with
various committees to works on related sectors. The ministerial conference can
take decisions on all matters under any of the multilateral trade agreements.

2) Forum for trade negotiation:


It provides forum for trade negotiation. For this purpose, its different mechanism
activate to works for their responsible sectors. Issues on trade related aspects
could be submitted through committees and councils for negotiations.

3) Handling trade disputes:


It has dispute settlement mechanism like Dispute Settlement panels and Dispute
Settlement Body (General Council in another guise). It is under the General
Council and finally ministerial conference. The Dispute Settlement Body has the
sole authority to establish ―panels‖ of experts to consider the case, and to accept
or reject the panels‘ findings or the results of an appeal. It monitors the
implementation of the rulings and recommendations, and has the power to
authorize retaliation when a country does not comply with a ruling.
4) Monitoring national trade policies:
It monitors national trade policies through General Council Meetings as Trade
Policy Review Body. Finally, the general council submits the report before
conference.
5) Technical assistance and training for developing countries:
It provides technical assistance and training as it think fit. It has technical
assistance missions that works in this field.
6) Cooperation with other international organizations:
It cooperates with and assist to IMF and IBRD for establishing coherence in
universal economic policy determination. The WTO maintains extensive
institutional relations with many of its sister organizations, participates as
observer in their work and has established several partnerships to help improve
the trading opportunities and capacities of developing and least-developed
countries. Examples of such partnerships are the Enhanced Integrated Framework
(EIF), the Standards and Trade Development Facility (STDF) and the Aid for
Trade Initiative. WTO cooperation with other international organizations
continues to evolve and is more than ever a function of the need for increased
global coordination and better governance.

e) Pursuant to Section 3 of the Social Welfare Act, the Government of Nepal, by means
of different activities relating to the social welfare work, to support the overall
development of the country may operate the social welfare Program through the
concerned Ministry and Social organizations and institutions.
Similarly, as per Section 4, the Government of Nepal may operate special Programs,
relating to the social welfare activity and social service, in the following matters:
(a) To serve interest and render welfare to the children, old age, helpless or disabled
people.
(b) To foster participation in development and to promote and protect the welfare,
rights and interest of the women.
(c) To rehabilitate and help to lead a life of dignity to the victims of social mischief's
and also to juvenile delinquency, drug addicts and similar people involved in other
kind of addictions.
(d) To help to lead a life with dignity to the jobless, poor and illiterate people.
(e) To manage religious places and the activities of the trust Guthi institutions.
(f) To take effective management and actions for the welfare of the backward
communities and classes.
7. Write short notes on the following: (2×5=10)
a) Doctrine of unjust enrichment
b) Formation of Insurance Board
Answer:
a) A general equitable principle that no person should be allowed to profit at another's
expense without making restitution for the reasonable value of any property, services,
or other benefits that have been unfairly received and retained.
Although the unjust enrichment doctrine is sometimes referred to as a quasi-
contractual remedy, unjust enrichment is not based on an express contract. Instead,
litigants normally resort to the remedy of unjust enrichment when they have not
written or verbal contract to support their claim for relief. In such instances litigant
ask a court to find a contractual relationship that is implied in law, a fictitious
relationship created by courts to do justice in a particular case.
Its objective is to prevent a man from retaining the money of or some benefit derived
from another, which it is against conscience that he should keep.
The doctrine is applied in the satisfaction of the following conditions:
1. The plaintiff must have provided the defendant with something of value while
expecting compensation in return
2. The defendant must have acknowledged, accepted, and benefited from whatever
the plaintiff provided.
3. The plaintiff must show that it would be inequitable or unconscionable for the
defendant to enjoy the benefit of the plaintiff's actions without paying for it.
A court will closely examine the facts of each case before awarding this remedy and
will deny claims for unjust enrichment that frustrate public policy or violate the law.

b)
Section 3 of Insurance Act, 2049 has provided the provision of formation of the
Insurance Board. It is formed to systematize, regularize, develop and regulate the
Insurance Business.
The Board pursuant to Sub-section (1) shall consists of the following Members:
(a) A person nominated or designated by the Nepal Government – Chairperson
(b) Representative, Ministry of Law, Justice and Parliamentary Affairs – Member
(c) Representative, Ministry of Finance – Member
(d) A person nominated by the Nepal Government from among the persons having
the special knowledge in the Insurance Business – Member
(e) A person nominated by the Nepal Government from among the Insured –
Member
An employee designated by the Board shall perform the duty as a Secretary of the
Board.
The Nepal Government may make alteration of the Members of the Board by
publishing a notification in the Nepal Gazette, if it deems necessary.
If it is deemed necessary, the Board may invite any national or foreign experts in the
meeting of the Board as an observer.
The tenure of the nominated Members of the Board shall be four years. They may be
re-nominated up to twice after the expiry of their tenure.
The Head office of the Board shall be located in Kathmandu.
Financial Management
Maximum Marks - 100
Total No. of Questions – 7 Total No. of Printed Pages – 14
Time Allowed – 3 Hours
Marks
Attempt all questions.
Working notes should form part of the answer. Make assumptions wherever necessary.
1.
a) Ganesh Enterprises needs someone to supply it with 150,000 cartons of
machine screws per year to support its manufacturing needs over the next five
years, and you‘ve decided to bid on the contract. It will cost you Rs. 780,000
to install the equipment necessary to start production; you‘ll depreciate this
cost straight-line to zero over the project‘s life. You estimate that in five years
this equipment can be salvaged for Rs. 50,000. Your fixed production costs
will be Rs. 240,000 per year, and your variable production costs will be Rs.
8.50 per carton. You also need an initial investment in net working capital of
Rs. 75,000. Your tax rate is 35 percent and you require a 16 percent return on
your investment.
Required: 10
Calculate unit bid price you should submit.

b) Consider the situation of a company, BIKE WASH, which must decide


whether to replace an existing machine. BIKE WASH currently pays no
taxes. The replacement machine costs Rs. 9,000 now and requires
maintenance of Rs. 1,000 at the end of every year for eight years. At the end
of eight years, the machine would be sold for Rs. 2,000 after taxes. The
existing machine requires increasing amounts of maintenance each year, and
its salvage value falls each year as below:
Year Maintenance (Rs.) After tax Salvage (Rs.)
Present 0 3,000
1 1,000 2,500
2 2,000 1,500
3 3,000 1,000
4 4,000 0
The existing machine can be sold for Rs. 3,000 now after taxes. If it is sold
one year from now, the resale price will be Rs. 2,500 after taxes, and Rs.
1,000 must be spent on maintenance during the year to keep it running.
Assume that this maintenance fee is paid at the end of the year. The machine
will last for four more years before it falls apart with zero salvage value at the
end of year 4. BIKE WASH faces an opportunity cost of capital of 15 percent.
Required: 10
Determine when BIKE WASH should replace the machine.
Answer:
a)
i) Initial Outlay
Cost of the Machine 780,000.00
W/C Infusion 75,000.00
855,000.00

ii) Annual Depreciation and Tax Shield


Depreciable Value 780,000.00
Life 5 Yrs
Annual Depreciation 156,000.00
Tax Shield on Annual Depreciation [Depreciation ×
Tax Rate] 54,600.00

iii) Terminal Value


Net proceed from the Sale of Machine 32,500.00
[50,000-(50,000-0)×35%]
Release of Working Capital 75,000.00
Total Terminal Value 107,500.00

iv) Total Cash Cost Per Annum - Post Tax


Annual Variable Cost (150000×Rs.8.50) 1,275,000.00
Annual Fixed Cost 240,000.00
Total annual Cash Cost 1,515,000.00
Tax @ 35% 530,250.00
Annual Post Tax Cash Cost 984,750.00

v) Suppose SP Per Cartoon X


Annual Revenue from Sale of 150,000 Cartoon 150,000 X
Post Tax Revenue from Sale 97,500 X

vi) Net Present Value Table


Particulars Time Cash Flow PVF @ 16% Total PV
Initial Outlay 0 (855,000.00) 1.0000 (855,000.00)
Tax Shield on Annual Depreciation 1-5 54,600.00 3.2743 178,777.00
Terminal Cash Flow 5 107,500.00 0.4761 51,181.00
Post Tax Cash Cost 1-5 (984,750.00) 3.2743 (3,224,367.00)
Post Tax Revenue 1-5 97,500 X 3.2743 319,244.00 X
Net Present Value 0

Since the discount rate 16% incorporates the profit target as well, the required NPV of the Project is
Zero.

Now,
319,244.00X+51181.00+178,777.00=855,000+3,224,367.00
319,244.00X= 3,849,409.00
X= 12.06

Therefore the Bid Price per Cartoon is Rs. 12.06

b)
Our decision will be based on the comparison of the annual cost of the replacement
machine with the annual cost of the old machine.

i) Equivalent Annual Cost of New Machine


The present value of the cost of the new replacement machine is as follows;
Present Value = 9,000 + 1,000 × [PVIFA, 8 Years, 15%] – 2,000× [PVIF, 8th Year, 15%]
= 9,000 + 1,000×4.4873 – 2,000×0.3269
= Rs. 12,833.5
[1]
The Equivalent Annual Cost of new replacement machine equals:
= Present Value / 8 Year annuity factor at 15%
= 12,833.5/4.4873
= Rs. 2,860
This calculation implies that buying a replacement machine is financially equivalent to renting
this machine for Rs. 2,860 per year.

ii) Cost of Old Machine


If BIKE WASH keeps the old machine for one year, the firm must pay maintenance costs of Rs.
1,000 a year from now. BIKE WASH will receive Rs. 2,500 at the end of year if the old machine
is kept for one year but would receive Rs. 3,000 today if the old machine were sold immediately.
This reduction in sales proceeds is clearly a cost as well. Further we normally express cash flows
in terms of present value; the analysis to come is easier if we express the cash flow in terms of its
future value one year from now.

The Value for end of year 1, 2 3 and 4 will be as below;


Particulars Year-1 Year-2 Year-3 Year-4
Beginning Salvage Value [A] 3,000 2,500 1,500 1,000
Maintenance cost expressed in 1,000/1.15 2,000/1.15 3,000/1.15 4,000/1.15
term of beginning value [B] =870 =1,739 =2,609 =3,478
End Salvage value expressed in (2,500)/1.15 (1,500)/1.15 (1,000)/1.15 -
term of beginning value [C] =(2,174) =(1,304) =(870)
Net Present Value D= [A+B-C] 1,696 2,935 3,239 4,478
Future Value [D×1.15] 1,950 3,375 3,725 5,150
[4
iii) Making the Comparison
If BIKE WASH replaces the machine immediately, we can view the annual expense as Rs. 2,860
beginning at the end of the year. This annual expense occurs forever if it replaces the new
machine every eight years.
If BIKE WASH replaces the old machine in one year, its expense from using the old machine for
that year can be viewed as Rs. 1,950 payable at the end of the year.
Therefore BIKE WASH should not replace the machine now. However, if we look into the cost
of operating old machine from 2nd year and onward, the cost is always higher than the equivalent
annual cost of new machine.
Therefore, the old machine should be replaced after operating 1st Year.

2.
a) X and Y are two fast growing companies in the engineering industry. They
are close competitors, and their asset composition, capital structure, and
profitability records have been very similar for several years. The primary
difference between the companies, from a financial management
prospective, is their dividend policy. Company X tries to maintain a non-
decreasing dividend per share, while company Y maintains a constant
dividend payment ratio. Their earnings per share (EPS), dividend per share
(DPS), and average share price are as follows:

Company X (Rs.) Company Y (Rs.)


Year EPS DPS Avg. Price EPS DPS Avg. Price
1 9.30 2.00 87.50 9.50 1.90 70.00
2 7.40 2.00 67.50 7.00 1.40 45.00
3 10.50 2.00 90.00 10.25 2.10 57.50
4 12.75 2.25 110.00 12.25 2.45 100.00
5 20.00 2.50 167.00 20.25 4.05 167.50
6 16.00 2.50 170.00 17.00 3.40 160.00
7 19.00 2.50 182.00 20.00 4.00 160.00

Required: (5+3+2=10)
i) Determine the dividend payment ratio and price earnings ratio for both
companies for all the years.
ii) Determine the average DP ratio and PE ratio for both the companies
over the period 1 through 7 years.
iii) The management of company Y is puzzled as to why their share prices
are lower than those of company X, in spite of the fact that profitability
of company Y is slightly better (particularly of past three years). As a
financial consultant, how would you explain the situation?
b) A company has a total investment of Rs. 4,000,000 in assets and 40,000
outstanding ordinary shares of Rs. 100 per share (par value). It earns at a
rate of 15 percent on its investment, and has a consistent policy of retaining
50 percent of the earnings. The appropriate discount rate of the firm is 10
percent.
Required: (3+2=5)
i) Determine the price of its share using Gordon‘s model.
ii) What shall happen to the price of the shares if the company has a payout
of 20 percent and 60 percent respectively?
Answer:
i) Calculation of DP ratio and PE ratio of Company X
Avr.
Year EPS(Rs.) DPS(Rs.) DP ratio Price(Rs.) PE ratio
1 9.30 2.00 21.50% 87.50 9.41
2 7.40 2.00 27% 67.50 9.12
3 10.50 2.00 19.05% 90.00 8.57
4 12.75 2.25 17.64% 110.00 8.63
5 20.00 2.50 12.50% 167.50 8.38
6 16.00 2.50 15.63% 170.00 10.63
7 19.00 2.50 13.16% 182.50 9.61
94.95 15.75 875

Calculation of DP ratio and PE ratio of Company Y


Avr.
Year EPS(Rs.) DPS(Rs.) DP ratio Price(Rs.) PE ratio
1 9.50 1.90 0.20 70.00 7.37
2 7.00 1.40 0.20 45.00 6.43
3 10.25 2.10 0.20 57.50 5.61
4 12.25 2.45 0.20 100.00 8.16
5 20.25 4.05 0.20 167.50 8.27
6 17.00 3.40 0.20 160.00 9.41
7 20.00 4.00 0.20 160.00 8.00
96.50 19.30 760
ii) The average DP ratio for company X
= 15.75/ 94.95×100
=16.59 %
The average DP ratio for company Y
= 19.30/ 96.50×100
=20 %
The average PE ratio for company X
= 875/94.95
= 9.22
The average PE ratio for company Y
= 760/96.50
= 7.88
iii) Company X is following the stable dividend policy, whereas company Y is
following the stable dividend payment ratio policy. In the latter policy, sporadic
dividend payment occurs, which makes its owners very uncertain about the returns
they can expect from their investment in the firm and, therefore, generally depress
the share prices. It is probably for this reason that company X‘s share price is better
than that of Y (during the last three years).
b)
(i) Price of Share using Gordon‘s model:

The share valuation model of Gordon is as follows:


P0 = DIV1 = (1 – b)EPS1 = (1 – b)rA , where
k–g k – br k – br
A denotes investment per share, which is Rs. 100 in the present case.
When the payout is 50 per cent, the price of share will be:
P0 = (1 – 0.5) 0.15 x 100 = 0.5 x 15 = 7.5/0.025 = Rs. 300
0.10 – (0.15 x 0.5) 0.10 – 0.075

(ii) Price of Share at Payout of 20 and 60 percent:


At Payout of 20 per cent

Ke =10%
b×r = 12%
Since br ˃ Ke as per Gordon's model price could not be computed.

At Payout of 60 per cent:

P0 = (1 – 0.4) 0.15 x 100 = 0.6 x 15 = 9/0.04 = Rs. 225


0.10 – (0.15 x 0.4) 0.10 – 0.06

3.
a) A Company is preparing a cash flow forecast for the three-month period
from January to the end of March. The following sales volumes have been
forecasted:
December January February March April
Sales (units) 1,200 1,250 1,300 1,400 1,500
Additional Information:
 The selling price per unit is Rs. 800 and a selling price increase of 5%
will occur in February. Sales are all on one month‘s credit.
 Production of goods for sale takes place one month before sales.
 Each unit produced requires two units of raw materials, costing Rs. 200
per unit. No raw materials inventory is held. Raw material purchases are
on one months‘ credit.
 Variable overheads and wages equal to Rs. 100 per unit are incurred
during production, and paid in the month of production.
 The opening cash balance at 1st January is expected to be Rs. 40,000.
 A long-term loan of Rs. 300,000 will be received at the beginning of
March.
 A machine costing Rs. 400,000 will be purchased for cash in March.
Required: (6+2=8)
i) Calculate the cash balance at the end of each month in the three-month
period.
ii) Calculate the forecast current ratio at the end of the three-month period.

b) Future Kidd Corporation presently gives credit terms of 'net 30 days'. It has
Rs. 60 Million in credit sales and its average collection period is 45 days.
To stimulate sales, the company may give credit terms of 'net 60 days'. If it
does instigate these terms, sales are expected to be increased by 15%. After
the change, the average collection period is expected to be 75 days with no
difference in payment habits between old and new customers.
Variable cost is Re. 0.80 for every Re 1.00 of sales and the company's
before tax required rate of return on investment in receivables is 20%.
Required:
Should the company extend its credit period? (Assume a 360-days year) 7
Answer:
a) (i)
Monthly Sales
Particulars December January February March April

Sales (units) 1,200 1,250 1,300 1,400 1,500

Selling price (Rs./unit) 800 800 840 840

Sales (Rs.000) 960 1,000 1,092 1,176

Purchases:
December January February March April

Production units 1,250 1,300 1,400 1,500


Raw materials (units) 2,500 2,600 2,800 3,000
(2×Production unit)
Raw materials (Rs. 000) 500 520 560 600
(@Rs.200/unit)
Variable Cost:
December January February March April

Production (units) 1,250 1,300 1,400 1,500


Variable costs (Rs.000) 125 130 140 150
(@Rs. 100/unit)

Monthly cash balances:

Particulars January (Rs. 000) February (Rs. 000) March (Rs. 000)
Cash collection from 960 1,000 1,092
Receivables
Loan 300
Total receipts 960 1,000 1,392
Payments:
Raw materials 500 520 560
Variable costs 130 140 150
Machine 400
Total Payments 630 660 1,110
Net cash flow 330 340 282
Opening balance 40 370 710
Closing balance 370 710 992

(ii) Calculation of current ratio:


Inventory at the end of the three-month period:
This will be the finished goods for April sales of 1,500 units, which can be assumed to
be valued at the cost of production of Rs.400 per unit for materials and Rs.100 per unit
for variable overheads and wages. The value of the inventory is therefore
1,500 x 500 = Rs. 750,000.
Trade receivables at the end of the three-month period:
These will be March sales of Rs. 1,176,000.
Cash balance at the end of the three-month period:
This was forecast to be Rs. 992,000.
Trade payables at the end of the three-month period:
This will be the cash owed for March raw materials of Rs. 600,000.
Forecast current ratio
Assuming that current liabilities consist of trade payables alone:
Current ratio = (750,000 + 1,176,000 + 992,000)/600,000 = 4·9 times

b) Evaluation of decision to extend credit period:


Existing Receivable Turnover= 360/45 =8 times
New Receivable Turnover = 360/75 = 4.8 times
New Sales = Rs. 60m×1.15 = Rs. 69 m
Additional Sales = Rs. 69m –Rs. 60m = Rs. 9 m
Contribution on Estimated additional Sales = 90,00,000×20/100=Rs. 18,00,000
Addl. Receivables associated with increased sales = 90,00,000/4.8 = Rs.
18,75,000
Investment in additional receivables associated with new sales = 18,75,000×80/100
= Rs.15,00,000
Receivables on present sales, on existing terms = 600,00,000/8 =75,00,000
Receivables of present sales, on new terms = 600,00,000/4.8 = 125,00,000
Investment in additional receivables associated with original sales
= 125,00,000-75,00,000 = 50,00,000
Total Investment in additional Receivables = 1500,000+50,00,000=65,00,000
Or
= 15,00,000+50,00,000*80%
= 15,00,000+40,00,000
= 55,00,000*20%
= 11,000
Required Return (before tax) on Addl. Investments = 65,00,000×20/100=
Rs.13,00,000

Analysis: The return on increased sales is more than the interest cost on additional
investments in debtors balances. Hence, it is recommended to extend credit period.
4.
a) X Co. has made plans for the next year. It is estimated that the company will
employ total assets of Rs. 8,00,000; 50 percent of the assets being financed
by borrowed capital at an interest cost of 8 percent per year. The direct costs
for the year are estimated at Rs. 4,80,000 and all other operating expenses
are estimated at Rs. 80,000. The goods will be sold to customer at 150
percent of the direct costs. Tax rate is 25 percent.
Required: Calculate: 8
i) net profit margin
ii) return on assets
iii) assets turnover
iv) return on owners' equity

b) Omni Corporation has target capital structure of 60% equity and 40% debt.
The schedule of financing cost for Omni is shown below:
Amount of New After tax Cost of Amount of New Cost of Equity
Debt (Rs. Million) Debt Equity (Rs. Million)
0-99 4.2 % 0-199 6.5%
100-199 4.6% 200-399 8.0%
200-299 5% 400-599 9.5%

Required: 7
Calculate the Break Point for Omni Corporation and also calculate WACC
for alternate level of financing in those break points.
Answer:
a) Calculation of Net profit
Particulars Amount ( Rs.)
Sales (150% of Rs. 4,80,000) 7,20,000
Direct Costs 4,80,000
Gross profit 2,40,000
Operating Expenses 80,000
Profit before interest and tax 1,60,000
Interest charges (8% of Rs.4,00,000) 32,000
Profit before tax 1,28,000
Taxes @ 25% 32,000
Net profit after tax 96,000
(i) Net profit Margin = Profit after tax/Sales=96,000/7,20,000= 13.33%
(ii) Return on Assets = (EBIT -Tax paid)/Assets= (1,60,000-32,000)/8,00,000=16% Or
EBIT/Total Assets = 20% Or PAT/Total Assets = 12%
(iii) Assets Turnover= Sales/Assets=7,20,000/8,00,000=0.9 times
(iv) Return on Equity=Net profit after tax/Equity=96,000/4,00,000=24%

b) Omni will have a break point each time a component cost of capital changes, for a total of
four break points;
Break Point debt>100mn = Rs. 100 million/0.4 = Rs. 250 Million
Break Point debt>200mn = Rs. 200 million/0.4 = Rs. 500 Million
Break Point equity>200mn = Rs. 200 million/0.6 = Rs. 333 Million
Break Point equity>400mn = Rs. 400 million/0.6 = Rs. 667 Million
Omni Corporation‘s WACC for the different break points.
(i) [3
WACC for Alternative level of Financing at break points
Capital (Rs. Million) Equity Cost of Debt (40%) Cost of WACC
(60%) Equity Debt
250 [ 250- 332] 150 6.5% 100 4.6% 5.74%
333 [ 333-499] 200 8.0% 133 4.6% 6.64%
500 [500-666] 300 8.0% 200 5.0% 6.80%
667 [ 667 and Above] 400 9.5% 267 5.0% 7.70%
(ii) [4
[7
5.
a) Mathura Corporation has two different bonds currently outstanding. Bond M
has a face value of Rs. 20,000 and matures in 20 years. The bond makes no
payments for the first six years, then pays Rs. 1,200 every six months over
the subsequent eight years, and finally pays Rs. 1,500 every six months over
the last six years. Bond N also has a face value of Rs. 20,000 and a maturity
of 20 years; it makes no coupon payments over the life of the bond. The
required return on both of these bonds is 10 percent compounded
semiannually.
Required: 5
What is the current price of Bond M and Bond N?
You can use the following statistical figures:
PVIFA5%, 12 Years =8.8633 PVIFA5%, 28 Years =14.8981
PVIFA5%, 40 Years =17.1591 PVIFA5%, 40th Years=0.1420

b) The net sales of A Ltd. is Rs. 30 crores. Earnings before interest and tax of
the company as a percentage of net sales are 12%. The capital employed
comprises Rs. 10 crores of Equity Shares, Rs. 2 crores of 13% Cumulative
Preference Share and 15% Debentures of Rs. 6 crores. Income tax rate is
40%.
Required: 5
Calculate the return on equity for the company and show segment
decomposition of ROE due to the presence of Preference Share Capital and
Debentures.

c) Mr. X, an investor, is seeking the price to pay for the security whose
standard deviation is 5%. The correlation coefficient for the security with
the market is 0.75 and the market standard deviation is 4%. The return from
the risk-free securities is 6% and from the market portfolio is 11%. Mr. X
knows that only by calculating the required rate of return, he can determine
the price to pay for the security.
Required: 5
What is the required rate of return on the security?
Answer:
a) Bond M is redeemable bond and the value of redeemable bond is discounted
present value of Interest and Principal amount over the life of the bond.
Therefore the value of Bond M may be calculated as below:
Value of Bond M = PV of Interest for First 6 Years + PV of Interest for next 8 Years + PV of
Interest for last 6 Years + PV of Redemption Value of Principal
= 0+ 1,200×[PVIFA 5%, 28 Years –PVIFA 5%, 12 Years] + 1,500×[PVIFA 5%, 40 Years –
PVIFA 5%, 28 Years] + 20,000×[PVIF 5%, 40th Year]
= 0 + 1,200×[14.8981-8.8633]+ 1,500×[17.1591-14.8981]+20000×0.1420
= 7,241.76 + 3,391.50 + 2,840
= Rs. 13,473.26
Bond N is Zero Coupon Bond and the value of Zero Coupon bond is discounted present value of
Principal amount redeemed over the life of the bond. Therefore the value of Bond N may be
calculated as below:
Value of Bond N = PV of Redemption Value of Principal
= 20,000 × [PVIF5%, 40Years]
= 20,000 × 0.1420
= Rs. 2,840
b)
Calculation of Return on Equity [ROE]
[Figures in Rs. Crore]
Sales 30
Earnings Before Interest and Tax [12% of Sales] 3.6
Less: Interest [15% on NRs 6 Crore] 0.9
Earning Before Tax 2.7
Less: Tax@ 40% 1.08
Earning After Tax 1.62
Less: Preference Dividend 0.26
Earning for Equity Holders 1.36
Return on Equity ( 1.36/30) 13.60%
[0.5
Capital Employed = Equity Share Capital + Preference Share Capital + Debt
= 10+2+6
= Rs.18 Crore
Post Tax Return on Investment = [EBIT×(1-Tax)/Capital Employed]
= [3.6×(1-0.4)/18]
= 12%
Segment Decomposition of ROE may be analyzed as below;
ROE = Post Tax ROI + [Post Tax ROI- Cost of Preference Share]×[Preference Share/Equity] + [Post
Tax ROI- Post Tax Cost of Debt]×[Debt/Equity]
= 12% + [12%-13%]×[2/10]+[12%-9%]×[6/10]
=12% -0.20% + 1.8%%
=13.60%
The negative 0.2% and Positive 1.8% is the segment of ROE caused by presence of Preference Share
Capital and Debenture in the Capital Structure.
[1.5
Post Tax Cost of Debt = Coupon Rate×[1-Tax Rate]
= 15%×[1-0.40]
=9%

c) Standard deviation of the security = 5%


Correlation coefficient of portfolio with market = 0.75%
Market standard deviation = 4%
Risk-free rate of return = 6%
Expected return on market portfolio = 11%

The market sensitivity index i.e. the beta factor can be calculated as follows:

Standard deviation of the security 0.05


β= ------------------------------------------------------× CORsm = -------------×.75= 0.9375
Standard deviation of Market 0.04

Now, the expected return on the investment can be ascertained with the help of CAPM
equation as follows:
Rs = Irf +(Rm-Irf) β
= 6 + (11-6)×0.9375
= 10.69%

6. Write short note/ answer on: (4×2.5=10)


a) Effect of leverage on Capital Turnover and Working Capital Ratio
b) Credit Rating
c) Sensitivity Analysis in capital budgeting
d) Agency cost of equity and debt
Answer:
a) Effect of leverage on capital turnover and working capital ratio
An increase in sales improves the net profit ratio, raising the Return on Investment (R.O.I)
to a higher level. This however, is not possible in all situations; a rise in capital turnover is
to be supported by adequate capital base. Thus, as capital turnover ratio increases, working
capital ratio deteriorate, thus, management cannot increase its capital turnover ratio beyond
a certain limit. The main reasons for a fall in ratios showing the working capital position
due to increase in turnover ratios is that as the activity increases without a corresponding
rise in working capital, the working capital position becomes tight. As the sales increases,
both current assets and current liabilities also increase but not in proportion to current ratio.
If current ratio and acid test ratio are high, it is apparent that the capital turnover ratio can
be increased without any problem. However, it may be very risky to increase capital
turnover ratio when, the working capital position is not satisfactory.

b) Credit Rating
Credit rating essentially reflects the probability of timely repayment of principal and
interest by a borrower company. It indicates the risk involved in a debt instrument as well
its qualities. Higher the credit rating, greater is the probability that the borrower will make
timely payment of principal and interest and vice-versa.
It has assumed an important place in the modern and developed financial markets. It is a
boon to the companies as well as investors. It facilitates the company in raising funds in the
capital market and helps the investor to select their risk-return trade-off. By indicating
credit-worthiness of a borrower, it helps the investor in arriving at a correct and rational
decision about making investments. Credit rating system plays a vital role in investor
protection. Fair and good credit ratings motivate the public to invest their savings.
As a fee-based financial advisory service, credit rating is obviously extremely useful to the
investors, the corporate (borrowers) and banks and financial institutions. To the investors, it
is an indicator expressing the underlying credit quality of a security to be floated for in the
market. The investor is fully informed about the company as any effect of changes in
business/economic conditions on the company is evaluated and published regularly by the
rating agencies. The Corporate borrowers can raise funds at a cheaper rate with good rating.
It minimizes the role of the 'name recognition' and less known companies can also
approach the market on the basis of their rating.
c) Sensitivity Analysis
The net present value or Internal Rate of Return of a project is determined by analyzing the
after tax cash flows arrived at by combining forecasts of various variables like Sales
volume, unit selling price, unit variable cost, fixed cost etc. It is difficult to arrive at an
accurate and unbiased forecast of each variable. It can't be certain about the outcome of any
of these variables. The reliability of the NPV or IRR of the project will depend on the
reliability of the forecasts of variables underlying the estimates of net cash flows. To
determine the reliability of the project's NPV or IRR, we can work out how much
difference it makes if any of these forecasts go wrong. We can change each of the forecasts,
one at a time, to at least three values: Pessimistic, expected and optimistic. The NPV of a
project is recalculated under these different assumptions. The method of recalculating NPV
or IRR by changing each forecast is called Sensitivity Analysis.

Sensitivity Analysis is a way of analyzing change in the project's NPV or IRR for a given
change in one of the variables. It indicates how sensitive a project's NPV or IRR is to
changes in particular variables. It basically examines the sensitivity of the variables
underlying the computation of NPV or IRR rather than attempting to quantify risk. It can be
applied to any variable which is an input for the after tax cash flows. It can be conducted
with regard to volume, price, costs etc.

d) Agency cost of equity and debt


Agency cost refers to the cost incurred by a firm because of the problems associated with
the different interests of management and shareholder and the information asymmetry that
exists between the principal (shareholders) and the agent (management).
 Agency Cost of Equity
The agency cost of equity arises because of the difference in interests between the
shareholders and the management. As long as the management‘s interests diverge from
that of the shareholders, the shareholders will have to bear this cost. Management may
be tempted to take suboptimal decisions that may not work towards maximizing the
value for the firm. Any measures implemented to oversee and prevent this will have a
cost associated with it. So, the agency costs will include both, the cost due to the
suboptimal decision, and the cost incurred in monitoring the management to prevent
them from taking these decisions.
 Agency Cost of Debt
The agency cost of debt arises because of different interests of shareholders and debt-
holders. Assume that the management is in favor of the shareholders. If so, the
management can in many ways transfer the wealth to the shareholders and leaving debt-
holders empty handed. Anticipating such activities, the debt-holders will take various
preventive measures to disallow management from doing so. The debt holders may do
so in the form of higher interest rates to protect themselves from the losses. Alternatively
they may impose restrictive covenants.

7. Distinguish between: (4×2.5=10)


a) Spin Off and Carve Out
b) Factoring and Bills Discounting
c) Bank Overdraft and Clean Overdraft
d) Euro Bond and Foreign Bond
Answer:
a) Spin off and Carve out
A spin-off and carve-out are different methods that a company can use to divest certain
assets, a division or a subsidiary. While the choice of a specific method by the parent
company depends on a number of factors, the ultimate objective is to increase shareholder
value.
In a spin-off, the parent company distributes shares of the subsidiary that is being spun-off
to its existing shareholders on a pro rata basis, in the form of a special dividend. The parent
company typically receives no cash consideration for the spin-off. Existing shareholders
benefit by now holding shares of two separate companies after the spin-off instead of one.
The spin-off is a distinct entity from the parent company and has its own management. The
parent company may spin off 100% of the shares in its subsidiary, or it may spin off
controlling interest to its shareholders and hold a minority interest in the subsidiary.
In a carve-out, the parent company sells some or all of the shares in its subsidiary to the
public through an initial public offering (IPO). Unlike a spin-off, the parent company
generally receives a cash inflow through a carve-out. Since shares are sold to the public, a
carve-out also establishes a net set of shareholders in the subsidiary. A carve-out often
precedes the full spin-off of the subsidiary to the parent company's shareholders.

b) Factoring and Bills Discounting


The differences between Factoring and Bills discounting are as below:
 Factoring is called as "Invoice Factoring" where as Bills discounting is known as
"Invoice Discounting"
 In Factoring, the parties are known as client, factor and debtor where as in Bills
discounting, they are known as drawer, drawee and payee
 Factoring is a sort of management of book debts whereas bills discounting is a sort
of borrowing from commercial banks
 For factoring there is no specific law whereas in case of bills discounting the
negotiable instruments law is applicable
c) Bank Overdraft and Clean Overdraft
Bank Overdraft
Bank overdraft refers to an arrangement whereby the bank allows the customers to
overdraw from the current deposit account within a specified limit. The overdraft facility
is granted against the securities of assets or personal security as in case of cash credit.
Interest is charged only on the amount actually withdrawn (i.e. debit balance) for the
actual period of use (i.e., for the period the debit balance in current deposit account
remains outstanding). The cost of raising finance by this method is the interest charged
by the bank.

Clean Overdraft
Bank may entertain clean advances from those customers, which are financially, sound
and reputed for their integrity. The banks in this case rely upon the personal security of
the borrower. Banks are responsible for ensuring customer‘s credit worthiness before
providing them with clean overdraft as there is no assets securing the amount of
advance. The banks normally take guarantee from the persons whom they believe to be
credit worthy.

d) Euro Bond and Foreign Bond


A Eurobond is denominated in a currency other than the home currency of the country or
market in which it is issued. These bonds are frequently grouped together by the
currency in which they are denominated, such as eurodollar or euroyen bonds. Issuance
is usually handled by an international syndicate of financial institutions on behalf of the
borrower, one of which may underwrite the bond, thus guaranteeing purchase of the
entire issue. Eurobonds are issued outside the restrictions that apply to domestic
offerings. The earliest eurobonds were physically delivered to investors. As of 2016,
they are issued electronically through a range of services, including the Depository Trust
Company (DTC) in the United States and the Certificate less Registry for Electronic
Share Transfer (CREST) in the United Kingdom. Eurobonds are usually issued in bearer
form, which makes it easier for investors to avoid regulations and taxes.
Foreign bonds, unlike Eurobonds, are issued in a single country and are usually
denominated in that country‘s currency. Often, the country in which these bonds are
issued will draw distinctions between them and bonds issued by domestic issuers—
including different tax laws, restrictions on the amount issued, and tougher disclosure
rules. Foreign bonds often are nicknamed for the country where they are issued Yankee
bonds (United States), Samurai bonds (Japan), Rembrandt bonds (the Netherlands),
Bulldog bonds (Britain). Partly because of tougher regulations and disclosure
requirements, the foreign bond market hasn‘t grown in past years with the vigor of the
Eurobond market.
Cost and Management Accounting
Maximum Marks - 100
Total No. of Questions: 6 Total No. of Printed Pages - 14
Time Allowed - 3 Hours
Marks
All questions are compulsory. Working notes should form part of the answer.
Make assumptions wherever necessary.
1. The Management Team of Exe Ltd. is considering the possibility of undertaking a
single production process which jointly produces four products in standard
proportions. The output from each 10 kg. batch of raw material input in the
process, together with net realizable value per kg. of output immediately after the
split-off point, is as under:
Material Output per 10 kg. input Net realizable value per kg. of output (Rs.)
A 4 8
B 3 4
C 2 10
D 1 2
The costs of processing each 10 kg. input batch are Rs. 12 and the cost of the raw
material input is Rs. 4 per kg.
For each of the four material jointly produced there is the possibility of further
processing before sale. The further processing will entail manual operation and
mechanical processing as well as incurring some costs directly attributable to each
product. Details of the resources used in, and cost incurred by, the further
processing as well as the final price per kg. as under:
Material Machine hours Labour hours Other direct cost Sales price
A 2 1 4 17
B 6 1 2 13
C 4 5 3 36
D 2 2 2 9
―Other direct costs‖ are variable costs but exclude the cost of labour, also a
variable cost, at Rs. 3 per labour hour. Apart from ―other direct costs‖ and labour
costs , all other costs of this further processing are fixed and are expected amount
to Rs. 3,40,000 per annum.
Exe Ltd. has the opportunity to process 1,00,000 kg. of the basic raw material per
year and machine capacity is capable of fully processing this amount.
The Managing Director feels that all products which are subjected to further
processing must be treated as joint products and all products to be sold
immediately after the split-off point without further processing are to be treated as
by-products of the original process. The net costs of the joint process are allocated
to the joint products in the proportion to the contribution of each product line,
after considering the marginal costs after the split-off point and the sales revenues.
However, the Managing Director is uncertain whether the Rs. 3,40,000 fixed
production costs of the further processing should be allocated to products in
accordance with machine or labour hours.
Required: (6+14=20)
a) Specify which of the jointly produced materials should be subject to further
processing if the joint process is carried out.
b) Produce a product profitability report for the joint products, utilizing the
Managing Director‘s approach to the determination of joint and by-products,
for each of the methods of allocating fixed production overhead he has
mentioned. You may assume all the production will be sold.

Answers:
a) Comparative statement of contribution per kg from further processing and net
realizable value per kg at the split-off point
Materials
A B D
(Rs.) (Rs.) C (Rs.) (Rs.)
Labour cost 3 3 15 6
Other direct cost 4 2 3 2
Marginal cost 7 5 18 8
Sale price 17 13 36 9
Contribution 10 8 18 1
Net realizable value per kg at the split-off point 8 4 10 2
Advantage (Disadvantage) of further processing 2 4 8 -1
The above statement shows that the further processing of materials A, B and C is beneficial.
Hence, they should be processed further. However, material D is giving loss as a result of further
processing. Hence, it should be sold out art spilt- off point.
b) Joint Products profitability Report
i. Statement of profitability when fixed production cost of further processing are
apportioned to product according to machine hours
Products
A B C Total
Output in kg 40,000 30,000 20,000
Rs. Rs. Rs. Rs.
Sales 6,80,000 3,90,000 7,20,000 17,90,000
Costs:
Joint costs (Working Notes 1 & 2) 2,00,000 1,20,000 1,80,000 5,00,000
Labour costs 1,20,000 90,000 3,00,000 5,10,000
Other direct costs 1,60,000 60,000 60,000 2,80,000
Fixed overheads (Working Note 3) 80,000 1,80,000 80,000 3,40,000
Total costs 5,60,000 4,50,000 6,20,000 16,30,000
Profit(loss) 1,20,000 -60,000 1,00,000 1,60,000
ii. .Statement of profitability when fixed production cost of further processing are
apportioned according to labour hours
Products
A B C Total
Rs. Rs. Rs. Rs.
Sales 6,80,000 3,90,000 7,20,000 17,90,000
Costs:
Joint costs (Working Notes 1 & 2) 2,00,000 1,20,000 1,80,000 5,00,000
Labour costs 1,20,000 90,000 3,00,000 5,10,000
Other direct costs 1,60,000 60,000 60,000 2,80,000
Fixed overheads (Working Note 3) 80,000 60,000 2,00,000 3,40,000
Total costs 5,60,000 3,30,000 7,40,000 16,30,000
Profit(loss) 1,20,000 60,000 -20,000 1,60,000
Working Notes:
1) Computation of Joint Costs: Rs.
Raw Material Input (1,00,000 * Rs.4) 4,00,000
Processing Costs (1,00,000 *Rs. 12/10) 1,20,000
5,20,000
Less: Net realizable value of by-product D (1/10 * 1,00,000 *Rs.2) 20,000
Joint costs 5,00,000
2) Apportionment of Joint Costs in proportion to the contribution of each product
Total Allocated Joint
Product Output Contribution per kg Total Contribution Costs
kg Rs. Rs. Rs.
A 40,000 10 4,00,000 2,00,000
B 30,000 8 2,40,000 1,20,000
C 20,000 18 3,60,000 1,80,000
10,00,000 5,00,000
3) Computation of overhead recovery rates on machine / labour hour basis
Total Output Machine Hours Labour hours
Product Per kg Total Per kg Total
A 40,000 2 80,000 1 40,000
B 30,000 6 1,80,000 1 30,000
C 20,000 4 80,000 5 1,00,000
3,40,000 1,70,000
Overhead Recovery Rates:
Per machine hour: Rs, 3,40,000/3,40,000 = Re. 1 per hour
Per labour hour: Rs. 3,40,000/1,70,000 = Rs, 2 per hour
2.
a) The Nepal Sofa Company (NS Co.) makes sofas. It has recently received a
request from a customer to provide a one-off order of sofas, in excess of
normal budgeted production. The order would need to be completed within
two weeks. The following cost estimate has already been prepared:
Additional Rs.
Information
Direct materials: (Notes)
Fabric 200 m² at Rs. 17 per m² 1 3,400
Wood 50 m2 at Rs. 8.20 per m² 2 410
Direct labour:
Skilled 200 hours at Rs. 16 per hour 3 3,200
Semi-skilled 300 hours at Rs. 12 per hour 4 3,600
Factory overheads 500 hours at Rs. 3 per hour 5 1,500
–––––
Total production cost 12,110
Administration overheads at 10% of total production cost 6 1,211
–––––
Total cost 13,321
–––––
Additional Information (Notes):
a. The fabric is regularly used by NS Co. There are currently 300 m² in
inventory, which cost Rs. 17 per m². The current purchase price of the
fabric is Rs. 17·50 per m².
b. This type of wood is regularly used by NS Co. and usually costs Rs. 8·20
per m². However, the company‘s current supplier‘s earliest delivery time
for the wood is in three weeks‘ time. An alternative supplier could deliver
immediately but they would charge Rs. 8·50 per m². NS Co. already has
500 m² in inventory but 480 m² of this is needed to complete other existing
orders in the next two weeks. The remaining 20 m² is not going to be
needed until four weeks‘ time.
c. The skilled labour force is employed under permanent contracts of
employment under which they must be paid for 40 hours‘ per week‘s
labour, even if their time is idle due to absence of orders. Their rate of pay
is Rs. 16 per hour, although any overtime is paid at time and a half. In the
next two weeks, there is spare capacity of 150 labour hours.
d. There is no spare capacity for semi-skilled workers. They are currently
paid Rs. 12 per hour or time and a half for overtime. However, a local
agency can provide additional semi-skilled workers for Rs. 14 per hour.
e. The Rs. 3 absorption rate is NS Co.‘s standard factory overhead absorption
rate; Rs. 1·50 per hour reflects the cost of the factory supervisor‘s salary
and the other Rs. 1·50 per hour reflects general factory costs. The
supervisor is paid an annual salary and is also paid Rs. 15 per hour for any
overtime he works. He will need to work 20 hours‘ overtime if this order is
accepted.
f. This is an apportionment of the general administration overheads incurred
by NS Co.
Required: (4+6=10)
i) Prepare, on a relevant cost basis, the lowest cost estimate which could be
used as the basis for the quotation.
ii) Explain briefly your reasons for including or excluding each of the costs in
your estimate.
b) The standard material cost for a mix of one tonne of final product is based on
the following:
Material Usage (kg.) Price per kg. (Rs.)
A 250 12
B 450 15
C 600 20
During the month of July, 2017, 12 tonnes of final product were produced
from the following:
Material Usage (tonnes) Cost (Rs.)
A 3.50 45,500
B 6.10 85,400
C 6.50 1,43,000
You are required to calculate the material variances and verify them. 10

Answers:
a)
Nepal Sofa Company
Cost Estimate for the quotation
Direct materials: Note Rs.
Fabric 200 m² at Rs. 17·50 per m² 1 3,500
Wood 20 m2 at Rs. 8·20 per m2 2 164
30 m2 at Rs. 8·50 per m2 2 255
Direct labour:
Skilled 50 hours at Rs. 24 per hour 3 1,200
Semi-skilled 300 hours at Rs. 14 per hour 4 4,200
Factory overheads 20 hours at Rs. 15 per hour 5 300
Administration overheads 6 –
––––––
Total cost 9,619
––––––

Reasons for the costs as included above:


1 Since the material is in regular use by NS Co, it is replacement cost which is
the relevant cost for the contract.
2 30 m will have to be ordered from the alternative supplier for immediate
delivery but the remaining 20 m can be used from inventory and replaced by
an order from the usual supplier at a cost of Rs. 8·20 per m.
3 There is no cost for the first 150 hours of labour because there is spare
capacity. The remaining 50 hours will be paid at time and a half, which is Rs.
16 x 1·5, i.e. Rs. 24 per hour.
4 NS Co will choose to use the agency workers, who will cost Rs. 14 per hour,
since this is cheaper than paying existing semi-skilled workers at Rs. 18 per
hour (Rs.12 x 1·5) to work overtime.
5 None of the general factory costs are incremental, so they have all been
excluded. However, the supervisor‘s overtime pay is incremental, so has been
included. The supervisor‘s normal salary, on the other hand, has been
excluded because it is not incremental.
6 These are general overheads and are not incremental, so no value should be
included for them.

b)
Working Notes
1. Total standard cost (TSC)= (SQ x SP)
Rs.
Material A: 250 x12 = 3000
Material B: 450 x 15 = 6750
Material C: 600 x 20 = 12000
TSC for one tonne of final output 21750
TSC for actual output of 12 tonnes = 21750 x 12= Rs.261000
2. Total Actual Cost (TAC) and Actual Price (AP):

Material AQ (kg) Cost (Rs.) AP=(Cost/AQ)


(tonnes x1000 kg) (per kg Rs.)

A 3500 45500 13
B 6100 85400 14
C 6500 143000 22

TAC 16100 273900

Rs.
3. AQ x SP : Material A: 3500 x 12 = 42000
Material B: 6100 x 15 = 91500
Material C: 6500 x 20 = 130000
= 263500
4. Revised Standard Quantity (RSQ) = Total Actual Quantity divide into standard mix ratio.
RSQ : for A= 16100 kg x 250/1300 = 3096 kg.
: for B= 16100 kg x 450/ 1300= 5573 kg.
: for C= 16100 kg x 600/ 1300= 7431 kg.

5. (RSQ x SP) : Material A = 3096 x 12 = 37152


: Material B = 5573 x 15 = 83595
: Material C = 7431 x 20 = 148620
269367
6. Standard yield (SY) by using actual Quantity one tonne SY from 250+450+600=1300 kg
SY from Actual Quantity = 16100/1300=12.3846 tonnes.

Calculation of Material Variances:

1. MCV = TSC –TAC or (SQ x SP) - (AQ x AP)


= Rs.261000 – Rs.273900 = Rs.12900 (A)
2. MPV = AQ (SP-AP) or (AQ x SP)-(AQ x AP)
=Rs.263500 – Rs.273900 = Rs.10400 (A)
3. MUV = SP (SQ - AQ) or (SP x SQ) - (SP x AQ)
= Rs.261000 – Rs.263500 = Rs.2500 (A)
4. Material Mix Variance (MMV)
= SP (RSQ-AQ) or (SP x RSQ) - (SP x AQ)
= Rs.269367 – Rs.263500 = Rs.5867 (F)
5. Material Sub Usage Variance (MSUV)(Yield)
= SP (SQ – RSQ) or (SP x SQ) – (SP x RSQ)
= Rs.261000 – Rs.269367 =Rs.8367 (A)

Verification:
(i) MCV = MPV + MUV
Rs.12900 (A) = Rs.10400 (A) + Rs.2500 (A)
OR
Rs.12900 (A) = Rs.12900 (A)
(ii) MUV = MMV + MSUV
Rs.2500 (A) = Rs.5867 (F) + Rs.8367 (A)
OR
Rs.2500 (A) = Rs.2500 (A)
3.
a) Following information have been extracted from the cost records of XYZ Pvt.
Ltd:
Rs.
Stores:
Opening balance 54,000
Purchases 2,88,000
Transfer from WIP 1,44,000
Issue to WIP 2,88,000
Issue for repairs 36,000
Deficiency found in stock 10,800

Work-in-progress:
Opening balance 1,08,000
Direct wages applied 1,08,000
Overheads charged 4,32,000
Closing balance 72,000

Finished Production:
Entire production is sold at a profit of 15% on cost at WIP
WIP
Wages paid 1,26,000
Overheads incurred 4,50,000
Draw the Stores Ledger Control Account, Work-in-Progress Control Account,
Overheads Control Account and Costing Profit and Loss Account. 8
b) Compute the machine hour rate from the following data: 8
(Rs.)
Cost of machine 1,00,000
Installation Charge 10,000
Estimated scrap value after the expiry of its life (15 yrs.) 5,000
Rent and rates for the shop per month 200
General lighting for the shop per month 300
Insurance premium for the machine per annum 960
Repair and maintenance expenses per annum 1,000
Power consumption - 10 units per hour
Rates of power per 100 units 20
Estimated working hours per annum - 2,200. This includes setting-up
time of 200 hours
Shop supervisor's salary per month 600
The machine occupies ¼ of the total area of the shop. The supervisor is
expected to devote 1/5 of his time for supervising the machine.
c) What is the limitation of Break-even chart? 4
Answers:
a)
Stores Ledger Control A/c

Particulars Rs. Particulars Rs.


To Balance b/d 54,000 By Work in Process A/c 2,88,000
To General Ledger 2,88,000 By Overhead Control A/c 36,000

Adjustment A/c By Overhead Control A/c 10,800*


To Work in Process 1,44,000 (Deficiency)
A/c By Balance c/d 1,51,200

4,86,000 4,86,000
*Deficiency assumed as normal (alternatively can be treated as abnormal loss)
Work in Progress Control A/c
Particulars Rs. Particulars Rs.
To Balance b/d 1,08,000 By Stores Ledger Control 1,44,000
To Stores Ledger Control A/c 2,88,000 a/c Costing P/L A/c
By 7,20,000
To Wages Control A/c 1,08,000 (Balancing figures being
To Overheads Control a/c 4,32,000 of finished goods)
Cost
By Balance c/d 72,000
9,36,000 9,36,000
Overheads Control A/c
Particulars Rs. Particulars Rs.
To Stores Ledger Control A/c 36,000B By
ByBy
Work in Process A/C 4,32,000
To Stores Ledger Control A/c 10,800 By Balance c/d 82,800
To Wages Control A/c 18,000 (Under absorption)
(Rs.1,26,000- Rs.1,08,000)
To Gen. Ledger Adjust. A/c 4,50,000
5,14,800 5,14,800
Costing Profit & Loss A/c
Particulars Rs. Particulars Rs.
To Work in progress 7,20,000 By Gen. Ledger Adjust. A/c 8,28,000
To Gen. Ledger Adjust. 1,08,000 (Sales) (Rs. 7,20,000 × 115%)
(Profit)
A/c
8,28,000 8,28,000
Note: Overhead control A/C under absorption can be transferred to costing PL A/C.
b)
Computation of Machine Hour Rate
Particulars: Rs. Rs.
Standing charges:
Rent and rates (200 *12) * ¼ 600
General lighting (300*12)*1/4 900
Insurance Premium 960
Shop supervisor's salary (600*12) *1/5 1440
Depn (1,10,000 - 5,000)/15 7000
10,900
Hourly rate for standing charges ( Rs.10900/ 2,000hrs) 5.45
Machine Expenses:
Repairs and Maintenance (1,000/2,000) 0.50
Power -10 units per hour @ Rs.0.20 per unit 2.00
Machine - hour rate 7.95
Note: Setting- up time has been presumed as non- productive time and hence
productive time is only 2,000 Hours.

c)
Break even analysis is fundamentally a static analysis as it assumes almost everything
constant (e.g., constant total fixed costs, variable cost per unit, selling price,
productivity, sales mix in case of multi products etc.) The limitations which make the
assumptions to be unrealistic are given below:
1. All costs cannot be separated into fixed and variable components with
accuracy.
2. Fixed costs may change because of change in management policy or after
a range of activity.
3. Variable cost per unit may change because of operation of law of
increasing returns or decreasing returns.
4. Selling price may change because of increase or decrease in output, market
demand & supply, competition etc.
5. In case of multiple products, the sales mix need not necessarily be
constant.
6. In case of multiple products, separate break even points are to be
calculated. This poses a problem of apportionment of fixed costs to each
product.
7. Entire production need not necessarily be sold in practice.
8. When a number of products are produced separate break-even chart will
have to be calculated. This poses a problem of apportionment of fixed
expenses to each product.
9. Break-even charts ignore the capital employed in business which is one of
the important guiding factors in the determination of profitability.

4.
a) XYZ Company produces three products A, B and C. Selling price of product
A Rs. 100, Product B Rs. 80 and Product C Rs. 50.Variable cost per unit of A,
B and C product are Rs. 50, Rs. 40 and Rs. 20 respectively. The product and
sale mixed of A, B and C product are 20%, 30% and 50% respectively. The
total fixed costs are Rs. 14,80,000.
Considering the above information, you are required to find out overall break-
even quantity and product wise break-up of such quantity. 5
b) Juntara Construction Ltd. undertook a contract for Rs. 5,00,000 on 1st July,
2015. On 30th June 2016 when the accounts were closed, the following details
about the contract were gathered:
Particulars Rs.
Materials purchased 1,00,000
Wages paid 45,000
General expenses 10,000
Plant Purchased 50,000
Materials on hand 30-6-2016 25,000
Wages accrued 30-6-2016 5,000
Work certified 2,00,000
Cash received 1,50,000
Depreciation of plant 5,000
Work uncertified 15,000
The above contract contained escalation clauses which read as follows:
―In the event of prices of materials and rates of wages increase by more than
5% the contract price would be increased accordingly by 25% of the rise in the
cost of materials and wages beyond 5% in each case‖.
It was found that since the date of signing the agreement the prices of
materials and wage rates increased by 25%. The value of the work certified
does not take into account the effect of the above clause.
Prepare the contract account. 7
c) Explain the methods of valuation of work-in-process. 3
Answers:
a)
Product A B C
Selling price per Unit 100 80 50
Variable cost per Unit 50 40 20
Contribution per Unit 50 40 30
Manufacture and sold unit in % 20 30 50
Composite contribution
= (50×0.20+40×0.30+30×0.5)
=10+12+15
= Rs 37 per unit
Overall Break even quantity= Fixed cost
Contribution
=14,80,000
37
= 40,000 Units
The product wise break-up of this quantity is as follows:
A 40,000 20% 8,000 Units
B 40,000 30% 12,000 Units
C 40,000 50% 20,000 Units

b) In the books of Juntara construction Ltd.


Contract Account
Dr. for the period 1st July 2015 to 30th June 2016
Cr.
Particulars Amount Particulars Amount
To, Material purchased A/c 1,00,000 By, Material on hand 25,000
To, Wages A/c 50,000 By, Work certified 2,00,000
To, General Expenses A/c 10,000 By, Work uncertified 15,000 2,15,000
To, Depreciation on plant 5,000 By, Escalation claim 5,000
To, Notional Profit c/d 80,000

2,45,000 2,45,000
To, P & L A/c 20,000 By, Notional Profit b/d 80,000
To, Reserve c/d 60,000
80,000 80,000
Working Notes:
1. Contract Escalation
Cost of material & wages incurred = Rs. (1,00,000 + 45,000 + 5,000 – 25,000 )
= Rs.1,25,000
Cost of material & wages before increase in prices = Rs. (1,25,000 x 100/125 )
= Rs.1,00,000
Increase in contract price = 25/100 [Rs.1,25,000 – (Rs.1,00,000 x 105/100)]
= Rs.5,000
2. Percentage of completion of Contract
Work Certified = 2,00,000 = 40%
× 100
Contract Price 5,00,000

3. Profit to be transferred to P&L Account


1/3 x Notional Profit x Cash Received/Work Certified
=1/3 x Rs.80,000 x Rs.1,50,000/Rs.2,00,000
=Rs. 20,000
c) The valuation of work-in-process can be made in the following three ways, depending
upon the assumptions made regarding the flow of costs.
First-in-first out (FIFO) method
Last-in-first out (LIFO) method
Average cost method
A brief account of the procedure followed for the valuation of work-in-process under
the above three methods is as follows;
FIFO method:
According to this method the units first entering the process are completed first. Thus
the units completed during a period would consist partly of the units which were
incomplete at the beginning of the period and partly of the units introduced during the
period. The cost of completed units is affected by the value of the opening inventory,
which is based on the cost of the previous period. The closing inventory of work-in-
process is valued at its current cost.
LIFO method:
According to this method units last entering the process are to be completed first. The
completed units will be shown at their current cost and the closing-work in process
will continue to appear at the cost of the opening inventory of work-in-progress along
with current cost of work in progress if any.
Average cost method:
According to this method opening inventory of work-in-process and its costs are
merged with the production and cost of the current period, respectively. An average
cost per unit is determined by dividing the total cost by the total equivalent units, to
ascertain the value of the units completed and units in process
5.
a) In a manufacturing concern bonus to workers is paid on a slab rate based on
cost savings towards labour and overheads. The following are the slab rates:
Up to 10% saving - 5% of the earning
Up to 15% saving - 9% of the earning
Up to 20% saving - 13% of the earning
Up to 30% saving - 21% of the earning
Up to 40% saving - 28% of the earning
Above 40% saving - 32% of the earning
The wage rate per hour of workers - P, Q, R and S are respectively Rs. 10, Rs.
11, Rs. 12 and Rs. 14. Overheads are recovered on direct wages at the rate of
200%. Standard cost under wages and overhead per unit of production is fixed
at Rs. 300. The workers have completed one unit each in 8, 7, 5½ and 5 hours
respectively.
Calculate in respect of each worker: (3+2+2=7)
i) Amount of bonus earned
ii) Total earnings
iii) Total earnings per hour
b) Outline the types of budget commonly used. 4
c) What is the practical application of differential cost analysis? 4
Answers:
a)
Statement showing computation of amount of bonus, total earnings and earnings per hour
Rs.
Particulars P Q R S
1. Standard cost of wages and overheads 300 300 300 300
2. Time taken in hours 8 7 5.5 5
3. Rate per hour 10 11 12 14
4. Actual wages (2 x 3) 80 77 66 70
5. Overheads (200% of wages) 160 154 132 140
6. Actual cost of Labour & Overhead (4 + 5) 240 231 198 210
7. Savings (1 – 6) 60 69 102 90
8. % of savings (7/1 x 100) 20% 23% 34% 30%
9. % of bonus applicable 13% 21% 28% 21%
10. a) Amount of bonus earned (4 x 9) 10.40 16.17 18.48 14.70
11. b) Total earnings (4 + 10) 90.40 93.17 84.48 84.70
12. c) Total earnings per hour (11/2) 11.30 13.31 15.36 16.94

b) The Institute of Cost and Management Accountants (CIMA) defined budget as 'A
quantitative expression of a plan for a defined period of time. It may include planned
sales volumes and revenues, resource quantities, costs and expenses, assets, liabilities
and cash flows.' The types of budget commonly used are:
Master Budget
A master budget is an aggregate of a company's individual budgets designed to
present a complete picture of its financial activity and health. The master budget
combines factors like sales, operating expenses, assets, and income streams to allow
companies to establish goals and evaluate their overall performance, as well as that of
individual cost centers within the organization
Operating Budget
An operating budget is a forecast and analysis of projected income and expenses over
the course of a specified time period. To create an accurate picture, operating budgets
must account for factors such as sales, production, labor costs, materials costs,
overhead, manufacturing costs, and administrative expenses. Operating budgets are
generally created on a weekly, monthly, or yearly basis.
Cash Flow Budget
A cash flow budget is a means of projecting how and when cash comes in and flows
out of a business within a specified time period. It can be useful in helping a company
determine whether it's managing its cash wisely. Cash flow budgets consider factors
such as accounts payable and accounts receivable to assess whether a company has
ample cash on hand to continue operating, the extent to which it is using its cash
productively, and its likelihood of generating cash in the near future
Financial Budget
A financial budget presents a company's strategy for managing its assets, cash flow,
income, and expenses. A financial budget is used to establish a picture of a company's
financial health and present a comprehensive overview of its spending relative to
revenues from core operations.
c) Differential costing is a technique where mainly differential costs are considered
relevant. Differential cost is the difference in total costs between two acceptable
alternative courses of action. Differential cost analysis is usually made to facilitate
managerial decisions of following kind:
(a) Determination of the most profitable levels of production and price
(b) Acceptance of special orders – offer at a lower price or offering a quotation at
lower selling price in order to increase the capacity.
(c) Sell a product as it is or after further processing
(d) Determination of right price at which materials may be purchased
(e) Decisions regarding alternative capital investment and plant replacement
(f) Decisions such as changing the product mix, method of production, make or buy,
adding new product, etc.
6. Write short notes: (4×2.5=10)
a) Continuous costing
b) Valuation of material receipt
c) Classification of overhead by nature
d) Stock control account
Answers:
a) A category of costing methods applicable to the repetitive production processes with
continuous operations in which the products and services are identical and cannot be
segregated. It enables the management to know what it is costing to do the jobs in
hand and helps it to take corrective action in time to check wastages and
losses. Continuous costing involves very often the use of estimates of expenditure
instead of actual.
As will be seen later, actual expenditure on a job regarding materials and labour can
always be known easily but actual information about other expenses may not be
available for some-time. Hence an estimate of such expenses is necessary.

b) The invoice of material purchased from the market some time contain such items such
as trade discount, quantity discount, freight, duty, insurance, cost of packing, sale tax,
excise duty, cash discount etc. Under such a situation the general principal is that all
the cost incurred up to the point of procuring and storing materials should constitute
the cost of material purchase .The amount of trade discount. The amount of trade
discounts, quantity discounts and VAT are deducted from the invoice of materials
purchased. The transport charges, sale tax, insurance, cost of packing, customs and
excise duty should be included in the invoice cost of materials. The cash discount is
considered as financial gain, so it is kept outside the domain of material cost. In case
the containers are returnable, their resale value should also be taken in the invoice
price of material to correctly ascertain the cost of material purchased. The cost of
material purchased so determined may be used for the entry of material in the Store
Ledger.
c)
i) Fixed or constant: These are expenses that are not affected by any variation in the
volume of activity, e.g., managerial remuneration, rent, that part of depreciation
which is dependent purely on efflux of time, etc. Fixed or constant expenses
remain the same from from one period to another except when they are
deliberately changed, e.g., on increments being granted to staff or additional staff
being engaged.
ii) Variable: Expensed that change in proportion to the change in the volume of
activity; when output goes up by 10% the variable expenses also go up by 10%.
Correspondingly, on a decline of the output, these expenses also decline
proportionately e.g., power consumed; consumable stores; repairs and
maintenance and depreciation (on account of wear and tear) are dependent on the
use of assets.
Variable expenses are generally constant per unit of output or activity. Suppose
variable expenses amount to Rs. 10,000 for a production of 2,000 units i.e; Rs. 5
per unit. When output goes up to 2,200 units, with an increase of 10%, the
variable expenses amount to Rs. 11,000 i.e., 10,000 plus 10%, however, the cost
per unit will be the same as before.
iii) Semi variable: The expenses that either (a) do not change when there is a small
change in the level of activity but change whenever there is a slightly big change
and the change are in small steps; or (b) change in the same direction as change in
the level of activity but not in the same proportion. An expenses for example, may
not change if output goes up or come down by 5 % but may change by 3% when
there is an increase in production between 5% and 10%. Similarly, another item of
expenses may change by 1% for every 2% change in activity. Examples of such
expenses are: delivery van expenses, telephone charges, depreciation as a whole.
d) The account is prepared for each of the following cost items.
I. Raw Material: This account has opening stock and purchases on debit side and
material issues on credit side.
II.WIP: This account is debited with opening stock and factory overhead and credited
with cost of goods finished .The closing stock, if any, will be carried forward to the
next year.
III. Finished stock: This account is known as the finished goods account also. It is
debited with goods finished and credited with the cost of sales.
The above stock accounts are usually used in place of the stock and purchase account
which is maintained in the financial book.
Business Communication
Maximum Marks – 100
Total No. of Questions - 8 Total No. of Printed Pages -7
Time Allowed – 3 Hours
Marks
All questions are compulsory.
Section -'A'
1. Read the following case carefully and answer the questions given below: (4×5=20)
Effective Communication as a Motivator
One common complaint employees voice about supervisors is inconsistent
messages – meaning one supervisor tells them one thing and another tells them
something different. Imagine you are the supervisor/manager for each of the
employees described below. As you read their case, give consideration to how you
might help communicate with the employee to remedy the conflict.

Hari is a 27-year old who is a foodservice manager at a casual dining restaurant.


He is responsible for supervising and managing all employees in the back of the
house. Employees working in the back of the house range in age from 16 years old
to 55 years old. In addition, the employees come from diverse cultural and ethnic
backgrounds. For many, English is not their primary language.

Hari tries his best to keep up with food safety issues in the kitchen but he admits
it‘s not easy. Employees receive ―on the job training‖ about food safety basics.
But with high turnover of employees, training is often rushed and some new
employees are put right into the job without training if it is a busy day. Eventually,
most employees get some kind of food safety training. The owners of the
restaurant are supportive of Hari in his food safety efforts because they know if a
food safety outbreak were ever linked to their restaurant; it would likely put them
out of business. Still, the owners note there are additional costs for training and
making sure food is handled safely.

One day Hari comes to work and is rather upset even before he steps into the
restaurant. Things haven‘t been going well at home and he was lucky to rummage
through some of the dirty laundry and find a relatively clean outfit to wear for
work. He admits he needs a haircut and a good hand scrubbing, especially after
working on his car last evening. When he walks into the kitchen he notices several
trays of uncooked meat sitting out in the kitchen area. It appears these have been
sitting at room temperature for quite some time. Hari is frustrated and doesn‘t
know what to do. He feels like he is beating his head against a brick wall when it
comes to getting employees to practice food safety.

Hari has taken many efforts to get employees to be safe in how they handle food.
He has huge signs posted all over the kitchen with these words: KEEP HOT
FOOD HOT AND COLD FOOD COLD and WASH YOUR HANDS ALWAYS
AND OFTEN. All employees are given a thermometer when they start so that
they can temp food. Hand sinks, soap, and paper towels are available for
employees so that they are encouraged to wash their hands frequently.
Questions:
a) What are the communication challenges and barriers Hari faces?
b) What solutions might Hari consider in addressing each of these challenges and
barriers?
c) What Standard Operating Procedures (SOPs) would be helpful for Hari to
implement and enforce?
d) What are some ways Hari might use effective communication as a motivator
for employees to follow safe food handling practices?
Answer:
a) Communication challenges abound at any workplace. Hari has some common
challenges in his operation.
 Language barriers: not all employees speak English as their first language making
verbal communication a challenge at times.
 Generational (age) barriers: having employees in various age categories can pose a
unique set of challenges. While the younger generation is used to texting and using
shortened messaging, their vocabulary may not be consistent with that of older
employees. Work values and attitudes may also affect communication between
younger and older employees.
 Cultural and ethnic barriers: Cultural differences in food safety practices may be a
challenge for Hari to overcome.
 Non verbal challenges: Hari‘s body language (appearance) is telling others he does
not care about personal appearance and cleanliness.
 Emotional barriers: Emotional barriers can interfere with effective communication.
Hari comes into work after a rough start at home. These negative emotions are
affecting how he communicates with the employees.

b) Hari might consider the following solutions to the identified challenges and barriers:
 Language barriers: Several potential solutions might be addressed here including
posting signs in employees‘ primary language. Putting signage with visuals, not just
words. Hari might learn some simple words in the employees‘ primary language to
help show interest in the employees.
 Generational (age) barriers: Currently there are 4 generations in the workforce and
each potentially has a different preferred method of communication. While the
younger generation might prefer to receive text messages as their preferred way of
communication, older employees may not find this method of communication
acceptable. Consider your employee‘s preferences and be willing to communicate a
message in a few different ways.
 Cultural and ethnic barriers: Hari may need to identify cultural beliefs and work to
understand the ethnic barriers related to food safety. For instance, two employees
come from the same country and they have made comments that controlling
temperature in their country is not a priority; food can be at room temperature for long
periods of time and nothing ever happened.
 Non verbal challenges: Hari‘s appearance is a nonverbal cue to employees. His
appearance is important as he is a role-model to the employees. His actions and
behaviors should be consistent with what he is expecting of them. For example,
because he is expecting the employees to follow proper handwashing procedure, he
should also use proper handwashing procedures.
 Emotional barriers: Emotional barriers can interfere with effective communication. It
will be important for Hari to get his emotions ―in check‖ prior to starting work.
Having self-awareness and potentially seeking outside assistance may be possible
solutions here.

c) Standard Operating Procedures (SOPs) serve as the written documentation of best


practices and serves as the framework for organizational policy and structure. SOPs
identify the who, what, why, when, and how of foodservice practices for employees.
Part of the supervisor‘s role is to assure SOPs are in place, communicated to employees,
and followed. For this case, two helpful SOPs would be:
 New Employee Orientation SOP: Due to time constraints of ―busy days‖ and high
turnover, Hari currently finds orienting new employees to be a challenge. Although it
may take time at the beginning, the pay off may be great and save time in the long
run.
 Employee Health and Personal Hygiene SOP: Although SOPs are generally intended
for employees, it is important to note that as role models, supervisors/managers
should generally adhere to operational SOPs. Hari‘s own lack of adherence to
personal hygiene standards indirectly tells employees that appearance and hygiene are
not important. His own personal hygiene and appearance should serve as an example
to employees – clean, unwrinkled clothing, clean hands, free from grease and dirt with
neatly trimmed hair are a must for him to be a motivator for his employees to have
good appearance and hygiene. At present, his non verbal communication is telling
them personal hygiene and appearance is not important.

d) All supervisors, including Hari, have many ways to use effective communication in
motivating employees. A few suggestions are listed below:
 Provide sincere and encouraging words when employees follow safe food handling
behaviors. Use a communication method appropriate for an employee, so supervisor
must know a bit about the employee. For a high schooler, maybe it‘s a quick ―thank
you‖ text message or an older employee it might be a hand written thank you note.
 Serve as a role model through verbal and non verbal communication. It‘s said that
―actions‖ speak louder than words, so Hari can, through his actions, convey a
message to employees. For example: wear a clean uniform.
 Nonverbal (non-word) communication is the transmission of information in addition
to words in a communication to an audience or receiver of the communication content.

2. What are the advantages and disadvantages of working in teams, describe the
characteristics of effective teams. (6+4=10)
Answer:
A team is a unit of two or more people who share a mission and the responsibility for
working to achieve a common goal. Problem-solving teams and task forces assemble to
resolve specific issues and then disband when their goals have been accomplished. Such
teams are often cross-functional, pulling together people from a variety of departments
who have different areas of expertise and responsibility. The diversity of opinions and
experiences can lead to better decisions, but competing interests can lead to tensions that
highlight the need for effective communication. Committees are formal teams that usually
have a long-life span and can become a permanent part of the organizational structure.
Committees typically deal with regularly recurring tasks, such as an executive committee
that meets monthly to plan strategies and review results.

The teamwork interactions among the employees represent one of the most essential
elements of interpersonal communication. Collaboration —working together to meet
complex challenges—has become a core job responsibility of the professionals. It‘s a
virtual guarantee that everyone will be expected to collaborate in at least some of his/her
work activities. Communication skills will pay off handsomely in these interactions,
because the productivity and quality of collaborative efforts depend heavily on the
communication skills of the professionals involved.

Advantages of teams

When teams are successful, they can improve productivity, creativity, employee
involvement, and even job security. Teams are often at the core of participative
management, the effort to involve employees in the company‘s decision making. A
successful team can provide a number of advantages such as:

Increased information and knowledge: By pooling the experience of several


individuals, a team has access to more information in the decision-making process.

Increased diversity of views: Team members can bring a variety of perspectives to the
decision-making process—as long as these diverse viewpoints are guided by a shared
goal.

Increased acceptance of a solution: Those who participate in making a decision are


more likely to support it and encourage others to accept it.

Higher performance levels: Working in teams can unleash new levels of creativity and
energy in workers who share a sense of purpose and mutual accountability. Effective
teams can be better than top-performing individuals at solving complex problems.

Disadvantages of teams
Although teamwork has many advantages, it also has a number of potential
disadvantages. At the worst, working in teams can be a frustrating waste of time. Teams
need to be aware of and work to counter the following potential disadvantages:
Groupthink: Like other social structures, business teams can generate tremendous
pressures to conform with accepted norms of behaviour. Groupthink occurs when peer
pressures cause individual team members to withhold contrary or unpopular opinions.
The result can be decisions that are worse than the choices the team members might have
made individually.
Hidden agendas: Some team members may have a hidden agenda —private,
counterproductive motives, such as a desire to take control of the group, to undermine
someone else on the team, or to pursue a business goal that runs counter to the team‘s
mission.
Cost: Aligning schedules, arranging meetings, and coordinating individual parts of a
project can eat up a lot of time and money.

Characteristics of effective teams


The most effective teams have a clear objective and shared sense of purpose, have a
strong sense of trust, communicate openly and honestly, reach decisions by consensus,
think creatively, and know how to resolve conflict. Teams that have these attributes can
focus their time and energy on their work, without being disrupted by destructive conflict.
In contrast, teams that lack one or more of these attributes can get bogged down in
conflict or waste time and resources pursuing unclear goals. Two of the most common
reasons cited for unsuccessful teamwork are a lack of trust and poor communication. A
lack of trust can result from team members being suspicious of one another‘s motives or
ability to contribute. Communication breakdowns are most likely to occur when teams
operate across cultures, countries, or time zones. Thus, the group dynamics is necessary
among the team members. The interactions and processes that take place among the
members of a team are called group dynamics. Productive teams tend to develop clear
norms, informal standards of conduct that members share and that guide member
behavior. Group dynamics are influenced by several factors: the roles that team members
assume, the current phase of team development, the team‘s success in resolving conflict,
and the team‘s success in overcoming resistance.

3.
a) Write an e-mail responding to an announcement for a vacancy. 5
b) What are the supplementary parts of a report? Enumerate chronologically and
explain each of them in brief. 5
Answer:
a)

To:

Subject: An application for the post of IT officer

Sir,

I saw your online announcement for the post of IT officer, and I would like to apply for
the same. I have gone through your job descriptions, requirements and conditions. I have
come to realize that my academic profile and professional experiences will most
essentially match with your job requirements.

I have attached my CV along with this application. I am looking forward to your quick
response.
Regards
…………….

b) Supplementary parts follow the text of the report and provide information for readers
who seek more detailed discussion. Supplements are more common in long reports than
in short ones, and they typically include appendixes, a bibliography, and an index.
Appendixes: An appendix contains materials related to the report but not included in the
text because they are too long or perhaps not relevant to everyone in the audience. The
content of report appendixes varies widely, including any sample questionnaires and
cover letters, sample forms, computer printouts, statistical formulas, financial statements
and spreadsheets, copies of important documents, and multipage illustrations that would
break up the flow of text. An appendix is usually identified with a letter and a short,
descriptive title. All appendixes should be mentioned at appropriate places in the text
and listed in the table of contents.
Bibliography: To fulfil your ethical and legal obligation to credit other people for their
work and to assist readers who want to research your topic further, include a
bibliography, a list of the secondary sources you consulted when preparing your report.
This can also be called ―References‖ if it includes works consulted but not mentioned in
the report. In addition to providing a bibliography, some authors prefer to cite references
in the report text. Acknowledging your sources in the body of your report demonstrates
that you have thoroughly researched your topic. Furthermore, mentioning the names of
well-known or important authorities on the subject helps build credibility for your
message. Such source references should be handled as smoothly as possible.
Index:
An index is an alphabetical list of names and subjects mentioned in a report, along with
the pages on which they occur. If the readers need to access specific points of
information in a lengthy report, consider including an index that lists all key topics,
product names, markets, or important persons— whatever is relevant to the subject
matter. As with the table of contents, accuracy in an index is critical.

4. Write short notes on: (5×2=10)


a) Ethical communication
b) Stages of job interview
c) Barriers to effective listening
d) Web Conferencing
e) A follow-up letter

Answer:
a) Ethical communication
Ethics are the accepted principles of conduct that govern behavior within a society.
Ethical behaviour is a companywide concern, but because communication efforts are
the public face of a company, they are subjected to particularly rigorous scrutiny from
regulators, legislators, investors, consumer groups, environmental groups, labor
organizations, and anyone else affected by business activities. Ethical communication
includes all relevant information, is true in every sense, and is not deceptive in any
way. In contrast, unethical communication can distort the truth or manipulate
audiences in a variety of ways.

b) Stages of job interview


Job interview, an important component of employment communication, displays
distinctive features from other types of interviews. It is usually carried out in three
different stages: warming-up stage, question-answer stage and summing-up stage.
The warming-up stage refers to the stage that exists before the main conversation
between the interviewer and interviewee. In this stage both interviewer and
interviewee attempt to be prepared to lead the interview to a successful
communication. The interviewee is prepared with cheerful appearance and positive
thoughts and expectations. The interviewer attempts to make some kind of attachment
with the interviewee with the help of gestures, welcome note, etc. Similarly, in the
question-answer stage, the content based interaction between the interviewee and the
interviewer takes place. This is the largest stage of job interview. The final stage is
known as summing-up stage in which the interviewer signals pre-closing of the
conversation. And finally, the interview gets wrapped up with conventional thank-you
note and farewell exponents.

c) Barriers to Effective Listening


Barriers and noise can interfere with the communication process. There are various
barriers to effective listening and some of the prominent barriers are:
i) Physical barriers: We cannot listen if we cannot hear what is being said. Physical
barriers include hearing disabilities, poor acoustics, and noisy surroundings.
ii) Psychological barriers: Everyone brings to the communication process a different
set of cultural, ethical, and personal values. Each of us has an idea of what is right
and what is important. If other ideas run counter to our preconceived thoughts, we
tend to ―tune out‖ the speaker and thus fail to hear.
iii) Language problems: Unfamiliar words can destroy the communication process
because they lack meaning for the receiver. Similarly emotionally charged words
also cause miscommunication.
iv) Nonverbal distractions: Many of us find it hard to listen if a speaker is different
from what we view as normal. Unusual clothing, speech mannerisms, body
twitches, or a radical hairstyle can cause enough distraction to prevent us from
hearing what the speaker has to say.
v) Thought speed: Because we can process thoughts more than three times faster
than speakers can say them, we can become bored and allow our minds to wander.

d) Web Conferencing
Participants can take part in ―real life‖ meetings from the comfort of their offices.
Web conferencing is similar to videoconferencing but usually without the
transmission of pictures of the participants. They use their computers to access an
online virtual meeting room where they can present PowerPoint slides or share
spreadsheets or Word documents, just as they might do in a face-to-face meeting.
They can even demonstrate products and make changes in real time during a
meeting. Software such as WebEx and Microsoft Live Meeting makes Web
conferencing easy and effective.
e) A follow-up letter
In several situations, the follow-up letter is considered to be an important mode of
communication. It can be written after a business meeting, job interview, business
contract etc. In order to continue communicating with the recipient, you need to
write this type of a letter.

In the early stage of a business relationship, this type of a letter is written. It is


important to adopt the formal letter writing style. It is important to proofread your
letter in order to avoid spelling and grammatical errors.

Here are the two important reasons of sending a follow-up letter:


First is, it can offer an opportunity to maintain an additional contact after a
networking meeting, career, etc.

Second is, it can allow you to reconnect to the person with whom you are not in
touch for a while for the job search.
The follow-up letters are different from the thank you letters. The main purpose of
the letter is to show your thankfulness to someone for meeting you, approving
your project etc. It gives you the chance to re-introduce yourself.
Marketing
Maximum Marks –100
Total No. of Questions - 8 Total No. of Printed Pages -2
Time Allowed –3Hours
Marks
All questions are compulsory.
Section -'B'
5. Read the following case carefully and answer the questions given below: (45=20)
Shrestha tailoring center is a reputed brand in Kathmandu Valley for shirting
suiting and accessories. Thirty-two years ago, Shrestha tailoring center was
established by Babu Kaji Shrestha in Bagbazar. It is mainly famous for suits,
available both in readymade and through custom tailoring. It also offers casuals
and other wears for ladies and kids. ―We cater to the clothing of newborns to old
ones,‖ claims Shrestha. Apart from its main showroom in Bagbazar, it has
branches in several urban locations of Kathmandu such as Chabahil, and
Tripureshor.
The company‘s clients are mainly middle-class people. Though they have many
old clients, they also have increasing new customers. Apart from word of mouth,
it is also using advertisements from electronic media. Frequently, it offers free tie
for those who orders suit to attract customers. ―Last year, we offered free
Bangkok air tickets to lucky customers. We also give discount to customers
buying through debit and credit cards,‖ Shrestha said. It has also sponsored
several fashion shows, beauty pageants and programs in FM stations.
Price charged by the Shrestha tailoring center is slightly higher than other
competitors because of its location, quality clothing and brand image. To meet the
competition, they had offered suit at Rs. 4999 as a Dashain offer to attract price
sensitive customer. But this scheme did not work well. It became unsuccessful to
persuade economy minded customers. From the lesson learnt form the
unsuccessful Dashain scheme they are planning to modify the existing promotion
strategy. They are also planning to go beyond the Kathmandu Valley through
franchise and online business.
a) What is the main issue of the case?
b) Analyze the product strategy of Shrestha tailoring center.
c) Based on above case, identify the marketing concept implemented by Shrestha
tailoring center.
d) Are four P‘s sufficient for Shrestha tailoring center? Give your opinion.
Answer:
a) As per the above case, the main issue is implementation of inappropriate psychological
pricing as a promotional scheme. In the above case, reputed Shrestha tailoring center has
offered odd price i.e. Rs. 4999 to attract economy minded customers. Odd pricing targeted to
price sensitive customer is not matched with the brand personality of the tailoring center.
Thus, it failed to attract economy minded customers. It seems that tailoring center became
unsuccessful to attract new customer and to retain old customer because of inappropriate
pricing which is not in line with company image. Similarly, quality trap may be the reason as
customers perceive high price high quality and low price low quality. This type of appeal also
creates confusion among the quality conscious existing customer.
b) Shrestha tailoring center offers suits and accessories. It offers readymade suits and custom-
made suits. It also offers causals and other wears for ladies and kids. All products offered by
Shrestha tailoring center are closely related. They are closely related in terms of ultimate
consumption, in terms of distribution, in terms of promotion, in terms of target market. If any
company offers more than one product items which are closely related, that is popularly
known as product line strategy. Thus, based on product variety of company, we can say that it
is product line strategy.

c) In the given case, the company has used advertisement through electronic media. It has also
offered free goods i.e. tie for purchasing suits. It seems clear that they have also used sales
promotion tools like lucky draw i.e. free Bangkok air tickets. Several fashion shows, beauty
pageants and programs in FM stations have also been sponsored by Shrestha tailoring center.
It means they are using aggressive selling techniques to increase sales. The case is silent
about low price and easy availability or product quality. Similarly, the case is silent about
customer satisfaction and social responsibility.

Thus, we can conclude that based on given case, they are implementing selling concept of
marketing. As selling concept of marketing holds that customers will not buy their product
unless and until they are well informed. And aggressing selling techniques should be used to
increase sales and earn profit. It ignores customer satisfaction and social responsibility.

d) Marketing mix is popularly known as four P‘s which includes product, place, price and
promotion. Four P‘s are sufficient for physical goods but not sufficient for service products.
In today‘s context, it is very difficult to get pure physical goods or pure services. Features of
service are intangible, inseparable, perishable, indivisible and variable. Tailoring is a service
business. It offers hybrid product which includes both physical goods as well as service.
Thus, for customer satisfaction and organization goal achievement, Shrestha tailoring center
must not rely on four P‘s. They must address additional three P‘s i.e. people, process and
physical evidence while designing and implementing their marketing mix.

6. What is Promotion? What are the components of promotion? Explain the


objectives of promotion. (4+2+4=10)
Answer:
Promotion is the one of the major element of marketing mix which provides various valuable
information to consumers about product, its price, availability, utilities and benefits. Promotion
consists of various activities that facilitates exchanges with target customers through persuasive
communication which stimulates the demand of the product. Promotion is also known as
marketing communication.
According to Philip Kotler, ―Promotion includes all the activities the company undertakes to
communicate and promote its products to the target market.‖
According to Dictionary of Marketing, ―Promotion is the element in an organization‘s marketing
mix that serves to inform, persuade, and remind the market of a product and/ or organization
selling in the hope of influencing the recipients‘ feelings, beliefs, or behavior.‖
Promotion includes various types of marketing communication related activities to inform,
persuade, remind and reinforce the target market about the company‘s offerings which influence
the customer‘s feelings, belief or behavior of the customers.
Components of Promotion
There are mainly four components of promotion which are as follows:
• Advertising
• Personal selling
• Public relation and publicity
• Sales Promotion

Objectives of Promotion
The major objectives of promotion are informing, persuading, reminding and reinforcing or
reassuring the customer about the product.
1) Informing:
The main objective of promotion is to inform the market about product, price, availability,
utilities and benefits. It helps to develop awareness about the product It also provides
alternatives to the customers for purchase decision.
2) Persuading:
Promotion persuades customers to make the purchase decision in favor of the promoted
brand. Promotion is persuasive communication. It stimulates product demand through
appealing ads, incentives and benefits. Promotion influence buyer behavior. Promotion also
persuades middlemen to carry such product.
3) Reminding:
Customer normally has very short-lived memory. It is impossible to customers to remember
all advertised brand. So, marketer reminds customers about the product by using various
promotional tools. The marketer assumes that customer may forget unless they are constantly
reminded.
4) Reinforcing:
Promotion is equally important in post purchase stage of the buying process. Consumer may
feel anxiety after their purchase decision. Marketer often provides reinforcing message to
increase customer‘s satisfaction level. Repeated reinforcement also leads to brand loyalty. By
the reassurance to customer, marketer tries to reduce cognitive dissonance and build brand
and corporate image.

7.
a) What is distribution? Also point out the components of physical distribution. (3+2=5)
b) What is a package and packaging? Explain briefly the essentials of a good
package. (2+3=5)
Answer:
a) Distribution includes all the various activities the company undertakes to make the product
accessible and available to target customers. Distribution or Place is one of the important
elements of marketing mix. It includes the marketing channel and physical distribution.
Marketing channel makes the product available to the customers. Physical distribution makes
the product accessible to the channel members and customers. Distribution is also called the
―the other half of marketing.‖ It fulfills the gap between the producer and consumer.
Distribution channel is a set of interdependent organizations involved in the process of making
a product or service available for use or consumption by the consumer or business user.
Distribution channel may be direct channel, indirect channel or mixed channel. Physical
distribution is related with physical movements of goods from point of production to point of
consumption. It is also called market logistics.
Components of physical distribution are mentioned as follows:
i. Transportation
ii. Warehousing
iii. Inventory management
iv. Order processing
v. Material handling

b) Generally, packaging is an act of designing or producing the package for a product. In other
words, packaging can be defined as the process of designing the container for a product.
Packaging includes all activities required for designing and producing the container or
wrapper for a product.
While, a package means wrapper or cover or container in which a product is enclosed. We
see that some products are enclosed in a paper box some are enclosed in a tube, some are
enclosed into the canes, some are enclosed into a plastic bag, some are enclosed into a wood
box, and some are enclosed into a metal container, and so forth. All these are the types of
package. Use of a particular package largely depends upon the nature of product, demand of
the buyer and the particular country's legal requirements.

Essentials of a Good Package


A good package should pose the following characteristics:
i. It should be economical i.e., it should involve least cost to manufacture, to fill, and to
remove or to handle, so that it does not adequately affect the sales price of the product.
ii. It should be functional i.e., it should perform all those functions what it is supposed to do.
For example, is a package is used to minimize the cost of goods, it should do so. If a
package is used to attract the potential customers, it should do so. If a package is used to
protect from physical problems such as air, water, dust, etc., it should do so.
iii. In should be communicative or it should have promotional value i.e., it should
disseminate all information to the customers and other parties concerned regarding brand,
use and features of products, place and date of manufacture, name of manufacturer, etc.
iv. It should be attractive i.e., a package should be quite attractive since one of the main
objectives of packaging is to attract customers. In order to make a package attractive,
attractive and match-able color combination should be used, it should have good design, it
should use, as far as possible, attractive pictures or symbols or graphic presentation. A
right combination of these attributes will make a package more attractive, which will have
positive impact on company's sales volume and profitability.
v. It should be eco-friendly i.e., it should be environmentally friendly. A package should be
produced by such materials which do not affect environment or which do not create
environmental pollution. The green revolution has suggested the green marketers to use
environmentally friendly package. The actors of green revolution have suggested using
such package in the product, which not only must not contaminate the environment but
should protect it and even liquidate existing environmental damages. Many countries
including EU member countries, USA, Australia, and many more countries have their own
green legislations, which have implemented such conditions strictly. They can ban
importing those products from any country of the world, which do not meet their green
requirements.
vi. It should be durable i.e., a package should lengthen the life of the product so than the
marketers can store goods for a long period and consumers can use the product after a long
time from the date of manufacture.
vii. It should have commercial value i.e.; a package should be able to create adequate demand
for goods to maximize the sales quantity and earn adequate income.
8. Briefly explain the following: (5×2=10)
a) Market Segmentation
b) Pricing
c) Marketing Environment
d) Shopping Product
e) Meaning of E-marketing

Answer:
a) Market Segmentation
The process of defining and subdividing a large homogenous market into clearly
identifiable segments having similar needs, wants, or demand characteristics. Its objective
is to design a marketing mix that precisely matches the expectations of customers in the
targeted segment.
b) Pricing
Pricing refers to an act of determining the exchange value between what buyers get
and what the sellers receives. It is a process of setting prices for various products and
services. In short, pricing is the act of determining exchange value of a product or
service.

c) Marketing Environment
Marketing environment comprises all the actors and forces outside marketing that affect
marketing management‘s ability to develop and maintain successful transactions with its
target customers. Marketing is environment specific and change in marketing
environment creates opportunities and challenges in marketing. E.g. economic
environment, social environment, technological environment etc.

d) Shopping Product
Shopping products are less frequently purchased consumer products and services that
customers compare carefully on suitability, quality, price, and style. When buying
shopping products and services, consumers spend much time and effort in gathering
information and making comparisons. Shopping products marketers usually distribute
their products through fewer outlets but provide deeper sales support to help customers in
their comparison efforts. Clothing, furniture, motorbike are the examples of shopping
product.

e) Meaning of E-marketing
E-marketing or electronic marketing refers to the application of marketing principles and
the techniques through electronic media or internet. It is the process of marketing a brand
using the electronic devices that can be used to connect marketers and their customers via
the worldwide wave with the aim of attracting new customers retaining current business
and developing its brand identity. It is useful to exploring domestic as well as
international market through internet.
Income tax & Vat
Maximum Marks - 100
Total No. of Questions - 8 Total No. of Printed Pages -6
Time Allowed - 3 Hours
Marks
Attempt all questions. Working note should form part of the answer.
1. I & M Pvt. Ltd. Kathmandu, a manufacturing company, deals in production and sales of
garments. Based upon the following information, you are requested to ascertain taxable
income and tax liability for Income Year 2073/74. 20

Particulars Amount (Rs.)


Export Sales 3,00,00,000
Misc. Income 25,00,000
Dividend Income 10,00,000
Opening stock 75,00,000
Raw material import 70,00,000
Freight for raw material 9,00,000
Custom duty paid for raw material 15,00,000
Custom agent fee 1,50,000
VAT paid for import 15,00,000
Wages for production 30,00,000
Overhead cost 20,00,000
Administrative expenses 15,00,000
Selling and distribution expenses 10,00,000
Interest paid to Financial Institution 3,00,000
Penalties paid to Metropolitan Office 2,00,000
Donation 2,00,000
Depreciation 12,00,000

Additional information:

a) Sales include VAT refund of Rs. 10,00,000 paid at custom point for import of raw
material. Miscellaneous income Rs. 20,00,000 is the incentive given by the Nepal
government for export of previous year and is received during this year through Nepal
Rastra Bank. Remaining Miscellaneous income Rs. 5,00,000 has been generated from
the bank deposit.

b) Dividend income has been received from Joint Venture.

c) The opening stock and closing stock has been recorded 1,000 pieces and 1,200 pieces
respectively. The opening stock includes factory fixed overhead Rs. 10 per piece and
repair and maintenance Rs. 15 per piece. During the year, the company produced
2,000 pieces of garments and the overhead cost includes equal rate of previous years
fixed overhead and repair and maintenance cost.

d) Administrative expenses, includes travelling cost Rs. 1,00,000 of employees during


the import of raw material, entertainment expenses Rs. 50,000 provided to the
Custom's employee and custom agent during the import.

e) The company has the policy for employee bonus; annually it makes 10 % provision
from the profit as per the Bonus Act. During this year, it has paid Rs. 2,00,000 and
booked under the administrative expenses.
f) Selling and distribution expenses were paid to the cargo agent for export, invoices
amounting Rs. 2,00,000 from the cargo agent were received, and the remaining
amount pertaining to the transportation of the goods from the factory to the depot
point as per details given by the agent.

g) Interest expense has been charged against the loan taken to purchase the factory
machinery costing Rs. 20,00,000. The machinery was purchased on 1st Ashadh, 2074
and has been used from 29th Shrawan, 2074.

h) The donation includes Rs. 1,00,000 directly paid to the flood victims; the remaining
amount was deposited into Reconstruction Fund established by Nepal government for
earthquake victims.

i) Opening balance (WDV) of depreciable assets as on 2073.04.01 as per books of


account are as follows:
a. Land Rs. 20,00,000
b. Building Rs. 50,00,000
c. Cars Rs. 26,00,000
d. Plant & Machinery Rs. 80,00,000
The company disposed a Santro car having written down value of Rs. 10,00,000 for Rs.
8,00,000 during the year.

Answers:
Particulars Sec. Note Amount
Inclusions:
Sales 7 (2) 1 29,000,000
Miscellaneous Income 7 (2)
Export Incentive 2.1 -
Interest income directly related to 2.2 500,000
business
Dividend Income 7 (3) & 92 3 -
Total Inclusions (A) 29,500,000

Deductions
Interest Expense 14 4 -
Cost of Trading Stock 15 5 13,293,000
Depreciation 19 6 2,413,560
Repair & Improvement Cost 16 7 30,000
General Deductions
Administrative expenses 13 8 1,250,000
Selling & Distribution Expense 13 9 200,000
Provision for Employee Bonus 13 10 531,818
Penalties paid to Metropolitan Office 21 11 -
Total Deductions (B) 17,718,378
Assessable Income from Business 11,781,622
(A-B)

Calculation of Taxable Income


Assessable Income from Business (as 7 11,781,622
above)
Total Assessable Income 11,781,622
Less: Allowable Reductions
Donation to Exempt Entities 12 12 -
Contribution to National 12B 12 100,000
Reconstruction Fund of GON

Taxable Income 11,681,622

Calculation of Tax Liability


Taxable Income 5 11,681,622
Tax Rate 13 15%

Tax Liability 1,752,243


(Alternative Tax Computation)
Taxable Income from Export @15%
Taxable Income from Other @25%

Notes & Working Notes:


1. Calculation of Sales as per Tax
Particulars Amount (Rs.)
Export Sales (given) 30,000,000
Less: VAT refund included in Sales (Note 1.1) (1,000,000)
Sales u/s 7 (2) (b) 29,000,000

1.1. VAT is refundable duty and does not form part of either cost of sales when paid or
income when refunded.

2. Miscellaneous Income
2.1. Export Incentive:
As per Sec. 22 of Income Tax Act, 2058; company must follow accrual basis of accounting. As such, the
export incentive in relation to accrual of previous income year is part of income of previous income year
and not of this Income Year. Thus, the amount does not form part of business income of IY 2073/74 as
per Sec. 22.
(Alternatively, if students assume that the right to receive the export incentive was established during
current Income Year and hence, treated as accrued during the Income Year, the amount may be included
in current year's income with appropriate note regarding the same)

2.2. Interest income, though return of investment in nature, but is related to business objective of the company
and hence included in income as per Sec. 7 (2) (g) of the Income Tax Act.

3. The joint venture distributing dividend is assumed to be resident. As such, dividend income received from
resident joint company (Joint venture is a company as per definition) is a final withholding payment as
per Sec. 92 of the Act. Though it is investment income directly related to business activity of the person
and seems to be included u/s 7 (2) (g) but since it is final withholding, it must not be included in income
as per Sec. 7 (3) of the Act.
(Alternatively if student assume that the Joint venture is a nonresident joint venture, the amount may be
included in income)

4. As per Sec. 14 (1), if interest is accrued on loan borrowed for the purchase/creation of asset during the
income year, such interest expense shall be claimed only when the related asset is used during the income
year. Since, the asset is not used during the year; the interest expense is not deductible.
5. Calculation of Cost of Trading Stock
Particulars Amount
Opening Stock 7,500,000
Less: Adjustment for Repair & Maintenance Expense (15,000)
(15*1000)
Add: Cost of Production
Import of Raw Material 7,000,000
Freight for Raw Material 900,000
Custom duty paid for raw material 1,500,000
Custom agent fee 150,000
Wages 3,000,000
Overhead Expense (assumed to be production overhead) 2,000,000
Less: Adjustment for Repair & Maintenance Expense in cost of
(30,000)
production
(2000*15)
Less: Value of Closing Stock (8,712,000)
(Assumed FIFO)
(Cost of Production divide by 2000 multiplied by 1200)
Cost of Trading Stock 13,293,000

5.1. Refundable duties do not form part of cost of inventories. As such, VAT paid on import is not part of cost
of trading stock, and hence, not included.

6. Calculation of Depreciation Expense


Particulars Pool A Pool C Pool D
I. Depreciation Rate 6.67% 26.67% 20.00%
II. Opening Depreciation Base
2,600,000 8,000,000
5,000,000
III. Absorbed Additions
IV. Disposals 800,000
V. Depreciation Base
5,000,000 1,800,000 8,000,000
VI. Depreciation Expenses
333,500 480,060 1,600,000
VII. Total Depreciation Expense 2,413,560

7. Calculation of Eligible Repair & Improvement Cost


Particulars Pool A Pool C Pool D
I. Depreciation Base
5,000,000 1,800,000 8,000,000
II. 7%of Depreciation Base
350,000 1,26,000 560,000
III. Actual Repair & Improvement Cost
- 0 30,000
IV. Eligible (Lower of ―II‖ or ―III‖)
- - 30,000
Alternatively, full marks shall be granted for repair calculation only for
specific pool and not for all pools.

8. Eligible Administrative Expenses


Particulars Amount
Given 1,500,000
Less: Entertainment Expense Note 8.1 (50,000)
Less: Bonus Paid for previous year
erroneously charged as expense Note 8.2 (200,000)
1,250,000

8.1. Since the amount is paid to custom employee and custom agent without proper receipt of service and
proper justification for such payment, the amount is a cost not for business purpose, thus, is not a
deductible expense u/s 13 (c) and Sec. 21 (1) (f).

8.2. Company must follow accrual basis of accounting, as such, the bonus expense accrued for last year and
paid during the year included as part of administrative expense is not deductible expense u/s 13. (It is an
accounting error and must be rectified, which decrease administrative expense for accounting purpose as
well)

9. Eligible Selling & Distribution Expenses


Particulars Amount
Given 1,000,000
Less: Ineligible due to absence of
supporting (800,000)
Given 200,000

9.1. The expense without supporting is ineligible as the purpose of expense cannot be justified due to lack of
evidence justifying the purpose.

10. Calculation of Bonus Expenses


Accounting profit 58,50,000
Bonus Expense for the Year 531,818 Based on New Circular of IRD
(Profit divided by 1.1 &
multiplied by 0.1)

11. Penalty is not a deductible expense as it is paid as a result of infringement of law u/s 21 (1) (b).

12. Since direct payment to flood victim is not a payment of donation to exempt organization, thus the
amount is not deductible. The contribution to National Reconstruction Fund of GON is eligible for
deduction u/s 12B.

13. Calculation of Effective Tax Rate


Since it is a special industry, the applicable tax rate as per Schedule 1, Section 2 (3) is 20%. As it is
export oriented industry, it is entitled to exemption of 25% on tax rate as per Sec. 11 (3E) and as such, the
effective tax rate is 15%.

2. Mr. Tukaram Bidari joined Star General Stores, a trading firm on 1st Shrawan 2070. The
following information is received in relation to his employment income for the Income
Year 2073-74:
a) Pay scale Rs. 22,000 - 1,500 - 40,000.

b) He was paid with festival allowances equal to one months‘ salary. Apart from the
allowance, the firm provided 1 set TV as a gift to the best performing staff. The
market value of the TV is Rs. 55,000.
c) The firm pays Mr. Bidari Rs. 15,000 p.m. as house rent allowance. Out of the
allowance, he pays only Rs. 12,000 p.m. as house rent and saves Rs. 3,000 p.m.

d) He is entitled to entertainment allowance of Rs. 500 p.m. and annual medical


allowance equal to 2% of salary.

e) The firm paid salary of Rs. 6,000 to the cook of Mr. Bidari in his residence. He
compensated Rs. 2,000 to the firm on this account.

f) Appointed in a senior post, he attended 12 meetings and was paid Rs. 2,000 per
meeting as meeting allowance. He also received allowance Rs. 2,000 p.m. for writing
minutes of Human Resource Committee of the firm.

g) He was provided meal in the office in an equal term to all employees of the firm. The
expenses borne by the firm for this facility was Rs. 36,000 for entire year i.e. 3,000
p.m.

h) He spent Rs. 20,000 for promotion of firm by hosting dinner to customers. Cost on
this account was reimbursed by the firm to Mr. Bidari.

i) The firm paid Rs. 1,000 p.m. as tuition fees for each of two children of Mr. Bidari.
This payment was made directly to school rather than to Mr. Bidari.

j) During the year, he was paid Rs. 10,000 on account of leave encashment. Further, at
the end of the year, his accumulated leave pay was valued at Rs. 50,000 which was
payable on retirement from the firm.

k) Mr. Bidari received a sum of Rs. 30,000 from his previous employer. The payment
was made on account of target commission on sales achieved by him when he was in
employment with previous employee.

l) Mr. Bidari paid life insurance premium as follows:


 Rs. 15,000 for his policy of Rs. 5,00,000
 Rs. 10,000 for policy of Rs. 2,50,000 of his son

m) Mr. Bidari met an accident during the financial year and received Rs. 5,00,000 as
compensation from insurance company, out of which he incurred Rs. 1,00,000 for
medical treatment. Mr. Bidari had contributed only Rs. 1,50,000 as insurance
premium to insurance company so far.

You are required to calculate income from employment and net tax liability of Mr. Bidari
for income earned during FY 2073/74. 10

Answers:
Particulars Note Amount (Rs.)
Basic Salary
318,000
(22,000 + 1500 ×3 ) × 12
Festival Allowance 26,500
Gift 55,000
House Rent Allowance 180,000
Entertainment Allowance 6,000
Medical Allowance (2% of Salary) 6,360
Cook Facility 4000
Meeting Allowance 1 -
Allowance for minutes 24,000
Notes:
Meal Facility 2 - 1. M
Reimbursement of Business Promotion Exp 3 - eeting
Tuition Fees Paid 24,000 allowa
Provident Fund Contribution by employer nce is
Leave Encashment 4 10,000 final
Accrued Leave Provision 5 withho
Commission from previous employer 6 30,000 lding.
Compensation against Accident 7 0 2. A
Assessable Income from Employment 6,83,860 s per
Sec. 8
(3),
Assessable Income
Meals
Less:
or
a. Contribuion to approved retirement fund least of i, ii and
lunche
iii below:
s
i) Actual contribution (Rs. 63,600)
provid
ii) Rs. 300,000
ed to
iii) 1/3 of Assessable income (1/3 for Rs. 7,59,660)
all
Taxable Income 6,83,860
employ
Allowable Deduction under Schedule 1:
8 15,000 ees
Life Insurance Premium Lower of i & ii
under
i) Rs. 20,000
equal
ii) Actual Premium (Rs. 15,000)
terms
Balance Taxable Income 6,68,860
at
First ( 1 % Social Security tax up to first Rs. 3,50,000) 3,500 workpl
Next (15 % up to next Rs. 100,000) 15,000 ace are
Remaining Rs. 2,18,860 @ 25% for (up to next Rs. not
54,715
2,000,000)
include
35% above Rs. 2,500,000 -
d.
Total Tax Liability 73,125 3. R
eimbursement of expenses serving proper business purpose of employer does not form part of
employment income (as per Sec. 8 (3)).
4. Leave encashment paid during the tenure of employment forms part of employment income as it is
not a retirement payment.
5. Natural person must follow cash basis of accounting when calculating his/her income from
employment (as per Sec. 22), as such, accrued leave provision is not included in income.
6. Employment includes past, present and prospective employment, and computation of employment
income must be done under cash basis of accounting. As such, commission received in cash during
the year from past employer is included as part of employment income.
7. As per proviso clause (a) of Sec. 31, Compensation received against physical injury of a natural
person shall not be included in income.
Medical tax credit is not allowed on medical expenditure incurred for treatment of physical injury
using compensation received as per Sec. 31.
8. Payment of investment insurance premium for the insurance of his son is not deductible under Sec. 1
of Schedule 1.
3.
a) Oriental Pvt. Ltd is dealing with buying, constructing and selling of land and building.
It has the following transactions for one of the projects during the year 2073/74.

Land purchase for the project Rs. 50 million


Land purchase for office Rs. 10 million
Housing construction cost (10 houses) Rs. 150 million
Building construction cost for office Rs. 10 million
Land in Stock Rs. 20 million
Housing in Stock Rs. 60 million
Sales of housing (8 houses) Rs. 300 million

Based on the above information, answer the following questions: 5

i) Calculate gain and loss in this transaction,

ii) Whether the company should pay capital gain tax or not?

iii) If the company should pay capital gain tax, how much and where to pay?

iv) If the company does not require to pay the capital gain tax, then what and where
to pay tax on the gain of this transaction?

b) Mr. Ram has been holding shares of various listed companies of Nepal from primary
& secondary markets. Total cost of the shares is Rs. 2 million. He left for USA for a
year on Aswin 30, 2070 but he returned back on Kartik 15, 2072. The market value of
the shares was Rs. 3.5 million at the time he became non-resident.

He sold all the shares for Rs. 3.6 million on Kartik end, 2073. He has received his
payment from share broker after deducting broker commission Rs. 20,000. Advance
tax is deducted by NEPSE as per Section 95Ka (2). Compute gain and loss under
Section 40 (3), gain and loss calculation on Kartik end 2073 by NEPSE and tax
withholding amount in these situations. 5
Answers:
a)
a. Calculation of Gain and loss
The purchase of land and building for official purpose is not considered for calculation of gain
and loss, because land is business assets and building is depreciable asset. Trading stock is taken
into account to calculate the gain and loss:

Incomings:
Sales of housing (8) Rs. 300 Million
Less: Outgoings:
Cost of sales of housing Rs. 90 Million (Rs. 150 M-Rs.60 M closing stock)
Cost of land Rs. 30 Million (Rs. 50 M- Rs.20 M closing stock)
Gain Rs. 180 Million
b. As per section 95ka (6), capital gain tax @ 10 % on capital gain shall be applicable on the
disposal of land and building of any person other than natural person. So, the company should pay
the capital gain tax.
Alternatively, if the student may refer to IRD Circular (without referring it) and conclude that
since it is a trading stock, such advance tax collection is not required.
c. The company should pay the capital gain tax amounting of Rs. 18 Million (10 % of Rs. 180
Million) at the time of registration to the Land Revenue Office.
Alternatively, if student refer to the non applicability of advance tax collection on trading stock
(land and building of real estate company, the amount of capital gain is Zero.

d. Company is not relieved from taxation on its business income (i.e. income from real estate
dealing) even when the company is not required to pay advance tax at the time of registration of
land and building at Land Revenue Office. As such, the company is required to make calculation
of tax liability for the Income Year under Self Assessment System and pay such self assessed tax
along with submission of income return. The payment shall be made in installments and any
residual unpaid balance of tax liability not covered by installment tax shall be paid as and when
the return is filed.

b)

As per Section 40(3) of Income Tax Act, assets and liabilities shall be considered to have disposed
immediately before that person becomes a non-resident person, except for the land or building
situated in Nepal. As per the provision, gain and loss on the shares shall be calculated as follows:

Computation of gain/loss at the time of becoming nonresident

Incomings
Market Value at the time of becoming nonresident (Sec. 41 (1) (a) Rs. 3.5 million
Incomings Rs. 3.5 million

Amount paid at inception Rs. 2.0 million


Amount paid during holding period Rs. 0.00
Amount paid on disposal Rs. 0.00
Outgoings Rs. 2 million

Gain on deemed disposal Rs. 1.5 million

Tax rate applicable 5%


TDS amount 75,000

As the Nepalese Tax system follows Self Assessment process, a natural person becoming nonresident
shall file jeopardy self assessment income return as per Sec. 100 (1) at the time of becoming
nonresident and clear all applicable taxes until such period.

When he sold all the shares, the person shall disclose his Outgoings for the shares as Rs. 3.5 Million
to NEPSE and require NEPSE to collect advance tax as follows as per Sec. 95A:

Incomings from Shares Rs. 3,600,000


Less: Outgoings
U/s 41 (1) (b) Rs. 3,500,000
Broker Commission at the time of actual disposal Rs. 20,000
Gain (Incomings – Outgoings) Rs. 80,000
Tax rate applicable 5 percent
Advance tax collected by NEPSE from shareholder Rs. 4,000
(Advance Tax Collection Rate is 5% in both the case since the person is resident at the time of deemed
disposal and actual disposal)
4.
a) Comfort Airways Pvt. Ltd. has been established in FY 2068/69 with total paid up
capital of Rs. 1.5 billion. Currently, as the domestic airliner, it has been operating all
flights by 4 Beach Crafts to different tourist destinations. These Beach Crafts are
acquired directly from the manufacturer on Operating Lease.
Of late, the management of the company is very much certain to earn additional profit
Rs. 7.5 million at the end of FY 2073/74. To achieve this target, the company plans to
acquire one more 77 seater ATR air craft on finance lease for 15 years period to
expand its wings to India. This is possible subsequent to the intake of additional paid
up capital of Rs. 75 million for which all the shareholders are ready to invest.

As a tax consultant, the management wants to know any tax implication to the
company by adding the aircraft. 5

b) Mention whether the return filing is required or not on the following circumstances: 5

i) Mr. X, a natural person having employment income of Rs. 4,00,000 in 2073/74,


has worked under a foreign consultancy firm, i.e. non-resident for Nepalese tax
purpose. Assume no deduction of retirement fund, insurance, remote area
allowances etc.

ii) Mr. X with Mrs. X, a resident natural person having employment income of Rs.
2.1 million (Rs.1 million of Mr. X and Rs. 1.1 million of Mrs. X) in the year
2073/74. Assume no deduction of retirement fund, insurance, remote area
allowances etc.

iii) S Enterprises, Kathmandu registered in Small and Cottage Industries Office is


dealing with VAT exempt goods. It has annual sales Rs. 15,00,000 in 2072/73 and
Rs. 25,00,000 in 2073/74. It has assessable incomes Rs. 1,50,000 and Rs. 1,00,000
in 2072/73 and 2073/74 respectively.

iv) V Enterprises Kathmandu registered in Small and Cottage Industries Office is


dealing with VAT exempted and taxable goods. It has annual sales Rs. 5,00,000 in
2072/73 and Rs. 30,00,000 in 2073/74. It has assessable income Rs. 1,00,000 and
Rs. 5,00,000 in 2072/73 and 2073/74 respectively.

v) Mrs. Chaulagai, a single natural person has income from various sources like
dividend income Rs. 2,00,000, interest income Rs. 3,00,000 from bank, meeting
allowances Rs. 20,000 and capital gain from sale of shares Rs. 3,00,000.

c) Quantify the value of perquisite as per the provision of Income Tax Act, 2058
provided by PQR Ltd. to the employee and others as under: 5

i) The company pays Rs. 35,000/month for a furnished accommodation facility to


the Executive Officer (Mr. Lama). The value of the furnitures and equipments is
Rs. 1,50,000. Mr. Lama‘s basic salary and grade was Rs. 1,00,000 p.m. during the
financial year 2073/74.

ii) The company pays Rs. 15,000/month for a flat provided to Program Officer, Mr.
Tamang. Mr. Tamang‘s basic salary and grade was Rs. 50,000 per month during
the financial year 2073/74.
iii) The company pays Rs. 1,00,000/month for a flat provided to Mr. Thapa, a board
director. The value of the furniture and equipment provided by the company in the
rented flat was Rs. 5,00,000.

iv) Free four wheeler facility is provided to the Executive Officer (Mr. Lama). The
company meets the cost of fuel and maintenance charge of the vehicle provided to
Mr. Lama. The cost of fuel and maintenance charges paid by the company was
Rs. 2,50,000 and the value of four wheeler was Rs. 75,00,000. Mr. Lama‘s basic
salary and grade was Rs. 100,000 per month during the financial year 2073/74.

v) The company meets the cost of fuel and maintenance charge of vehicle provided
to Mr. Thapa, the board director. The cost of fuel and maintenance charges paid
by the company was Rs. 3,00,000 and the value of four wheeler was Rs.
1,00,00,000.

d) Oppo International Pvt. Ltd., Kathmandu is a dealer of Oppo Electronics. The Oppo
Electronics are manufactured in China by Oppo International. The company imported
the electronics goods worth of US $ 50,000 on 23 Baishakh, 2074, the payment was
done on 23 Shrawan, 2074. The exchange rate of the transaction was as follows:

Baishakh 23: Rs. 104 per US $


Closing of Ashadh: Rs. 107 per US $
Shrawan 23: Rs. 106 per US $

You are required to compute the amount for inclusions or deductions for the tax
assessment of this transaction mentioning the relevant provisions of Income Tax Act,
2058. 5
Answers:
a)

As per the provisions of Section 11 (3 ta), the Airline company operating international flights with an
investment more than Rs. 2 billion are inter alia privileged with 100% tax holiday for the period of 5
years from the date of operation, and thereafter privileged with 50% rebate on tax for the period of 3
years.

However, the privileges as aforesaid are also available to the existing Airline Operators provided they
enhance their existing installed capacity by at least by 25%, and increase their investment to at
least Rs. 2 billion. Such privileges commences immediately on the fulfillment of the two conditions
as aforesaid. In such case, the tax holiday and rebate shall be provided to such income that is
generated from enhanced capacity.

In the case of our problem, Comfort Airways Pvt. Ltd. with the increased installed capacity of
operations by two more international sectors, and total investment of Rs 1.5075 billion will not have
privilege to enjoy tax holidays as well as tax rebate as aforesaid.
b)

i) A person is not required to file return in any of the following conditions unless required by IRD in
writing or through public circular or the total income (including taxable or not except interest of
final withholding in nature and meeting allowance) exceeds 40 Lakhs:
 Under Section 97 along with Section 4(3), resident natural person having salary income from
resident employer with no claim of donation, more than medical tax credit and retirement
contribution, need not to file income tax return, or
 a person having no income tax liability, or
 a natural person having taxable income from operation of vehicle as specified in Sec. 1 (13) of
Schedule 1, or
 person deriving exclusive income from final withholding payment

In this case, since any of the above condition is not satisfied, Mr. X has to file income tax return of
his salary income for the income year.

ii) A person is not required to file income return as explained in answer (i).
Since both the spouses meet the conditions as prescribed by Sec. 4 (3) of the Income Tax Act,
2058 and individual income does not exceed Rs. 40 Lakhs, each of the spouses is not required to
file income return (as it is obvious that a person does not elect to file couple when both the spouses
are earning member since we need to presume that each taxpayer is rational and uses benefits
provided by tax law)
In case a student presumes application of Sec. 50 by spouses and filing jointly (which is absurd in
case of a rational taxpayer), any of the conditions in Sec. 97 (1) will not be satisfied, requiring
them to file income return.

iii) Since, there is tax liability in case of S Enterprises for both the Income Years (whether electing
Sec. 4 (4) or making payment of taxes u/s 4 (4Ka)), and any of the other three conditions as
specified in Sec. 97 (1) (part (i) of this answer) is not satisfied, S. Enterprises require to file tax
return.

iv) Since, there is tax liability in case of V Enterprises for both the Income Years (whether electing
Sec. 4 (4) or making payment of taxes u/s 4 (4Ka)), and any of the other three conditions as
specified in Sec. 97 (1) (part (i) of this answer) is not satisfied, V Enterprises require to file tax
return.

v) Section 97 (1) (kha) states that if a person is related to the section 3(c) the person is not required to
file the income tax return for the income year. Section 3(c) is the provision for the final
withholding payments. As per these provisions, Mrs. Chaulagai is not required to file for the
income of dividend, interest, meeting allowances.

But since she has capital gain income, she is required to file income return.

c) Under Section 27: Quantification of the value of perquisite provided by PQR Ltd. to employee and
others as per the provision of IT Act.
i) Rs. 24,000
2% of salary in case of salaried person i.e. (2% of {12 x (Rs. 1,00,000)}
ii) Rs. 12,000
2% of salary in case of salaried person i.e. (2% of {12 x (Rs50,000)}
iii) Rs. 300,000
25% of rent paid or imputed rent for own house for other person i.e. (25% of (12 x Rs. 100,000)
iv) Rs. 6,000
0.5% of salary in case of salaried person i.e. (0.5% of {12 x (Rs. 1,00,000)}
v) Rs. 100,000
1% p.a. of market value of vehicle per annum i.e. (1% of x Rs. 1,00,00,000)

d)

Conversion to Rupees under section 28, Exchange differences under section 24 (4) and claim for cost
of sales under Section 15 are related for these transactions.

The company should maintain its books of accounting in accrual basis under section 22 of the Act.
Further, the section 24 (4) has mentioned the accounting for exchange difference. The provision is as
follows:

In calculating a person's income earned from a business or investment on the accrual basis, where the
person includes any payment to which the person is entitled or deducts any payment that the person is
obliged to make and a difference in the payment received or made by the person occurs as a result,
inter alia, of a change in exchange rate, the amount of difference shall be adjusted at the time when
payment is received or made.

So, the company should account in Nepalese currency and calculate foreign exchange gain or loss.

Computation of inclusions or deductions


Based on the above provision, the company can include purchase amount of Rs. 52, 00,000 (US $
50,000 *104) in income year 2073/74 while calculating the cost of sales under 15. The exchange loss
of Rs. 100,000 shall be claimed deduction under 13 in income year 2074/75.
(Alternatively since exchange loss is related to loss on disposal of liability, the students may also refer
the same in answer.)

5. Write short notes (4×2.5=10)


a) Valuation of Closing Stock
b) Associated Person
c) Tax Payer‘s Rights
d) Self-assessment

Answers:
a) Valuation of closing stock of a business for income year is done at a lower of the following:
i) Cost of closing stock that remains at the end of year or
ii) Market price of the closing stock at the end of year;

Where identification of each items stock is not possible Income Tax Act permist first in first out
(FIFO)
OR weighted average method for valuation of closing stock

The cost of closing stock (trading stock) shall not include repair and improvement cost and
depreciation of depreciable asset.

b) As per section (ka na) of the Income Tax Act, 2058, Associated person means
a. a person or
b. more than one person or
c. a group of such persons who acts as per intention of each other,
And includes –
a. Natural person and relative of such natural person, or any person and partner of such person
b. Foreign Permanent Establishment and a person having ownership in such establishment
c. Any entity either directly or indirectly through
(a) other person or
(b) related entity or
(c) other person or entities related with such related entity,
Have control over or are entitled to have benefit of 50% or more than 50% of income, capital or
voting power of any entity.
But below persons shall not be treated as associated person
a. Employee,
b. Any other person as specified by the Department
c) As per the Section 74(2) of Income Tax Act 2058,following rights are available to the tax payers:

i) Right to get respectful behavior


ii) Right to receive any information related to tax as per the prevailing laws
iii) Right to get an opportunity of submitting a proof in one‘s own favor as regards tax matter
iv) Right to appoint lawyers or auditors for defense, and
v) Rights to secrecy in respect of tax matters and to keep it inviolable.

d) As per the provision (Sec.96) of Income Tax Act 2058, the tax payer has an obligation to submit the
tax return on time.
Actual Self Assessment:
Under (Sec.99), the person submits a tax return including the information regarding the total tax
payable during the year, and the tax due for payment on the date of submitting the return; it is believe
that the income tax assessment is complete
In general terms, the filling of an annual return is the self-assessment made by the tax payer that is
treated as assessment unless the Jeopardy Assessment (u/s100) & Amended Assessment (u/s 101)
prevail.
Deemed Self Assessment:
Even in the case of a person who fails to file the annual tax return, the income tax for the year is
deemed as assessed on due date of Asoj-end under the following basis:

i) The total tax liability of the tax payer during the year is equal to the amount of withholding tax
deducted by withholding agents on its payments, and the amount of advance tax paid by it.
The deemed tax assessment shows that there is no more tax payable for the year by such tax payer.

6. Namaste Steel Private Limited, a VAT registered Company, has witnessed stiff downfall
in demand of its products, and hence its operation has been shut down since the past 3
months. The company is considering for closure of the business.

Below is the detail of its assets and liabilities, and VAT receivable payable:
a. As per VAT return filed for the previous VAT period, it has VAT credit of Rs. 1
million.
b. Below is the detail of its stock and capital goods and other assets and liabilities as on
the immediately previous VAT period:

i. Finished Goods Inventory (VAT applicable) – Rs. 5 million (Market Value Rs. 4
million)
ii. Raw Material (VAT applicable) – Rs. 1 million (Market Value Rs. 1.5 million)
iii. Damaged Goods (VAT applicable) – Rs. 5,00,000 (Market Value Rs.
1,00,000)
iv. Plant and Machinery (VAT applicable) – Rs. 9 million (Market Value Rs. 4
million)
v. Vehicle – 4 Wheeler (VAT applicable) – Rs. 3 million (Market Value Rs. 4
million)
vi. Vehicle – 2 Wheeler (VAT applicable) – Rs. 1,00,000 (Market Value Rs.
1,00,000)
vii. Land – Rs. 20 million (Market Value Rs. 50 million)
viii. Sundry Debtors – Rs. 6 million (Rs. 3 million is bad debts)
ix. Sundry Creditors – Rs. 4 million (all amount payable)

Suggest the management of the company regarding: 10


a) The provisions in the Value Added Tax Act for cancellation of registration of VAT,
and
b) Amount of VAT payable or refundable to the company.

Answers:

As per Sec. 11 (1), the conditions for Cancellation of Registration are:


a. In case of an organized institution: In case of closure, sale or transfer of the organized institution or the
cessation of the existence of the institution due to any reason
b. In case of ownership of Individual: In the event of death of such individual
c. In case of a partnership firm: In case of dissolution of partnership or the death of a partner
d. In case a registered person leaves conducting taxable transaction
e. In case of a taxpayer who submits Zero Return continuously for a period of one year or does not submit
VAT Return at all
f. In case where the total taxable turnover of a person does not exceed Rs. 5 Million (2 million in case of
service business or mixed business of goods and services) in preceding consecutive 12 months period
g. In case of registration in error

As such, as per Section 11 (1) (a) as the company is considering closure, the VAT registration may be
cancelled when a closure notice is filed with concerned tax officer. For Namaste Steel Private Ltd. to go
for immediate cancellation of registration, it may do so if its turnover of preceding consecutive 12 months
is Rs. 50 Lakhs or less. Unless any of these two conditions is satisfied, it cannot make an application for
cancellation of registration.

For this, following procedure must be applied:


a. VAT registered entity should file application in a prescribed form stating reason for cancellation of
VAT registration for cancellation of Vat registration within 3 months of the date of occurrence of
events mentioned in section 11(1) of Value Added Tax Act, 2058. (Rule 12(1) of Value Added Tax
Rule, 2053)
b. Such entity should file VAT return till date along with VAT amount payable if any. (Rule 12(1) of
Value Added Tax Rule, 2053)
c. Such entity should pay VAT on closing stock of goods (including capital goods) in which VAT credit
had been taken earlier presuming the same has been disposed off at the market value. (section 11(4) of
Value Added Tax Act, 2052)
d. Tax officer shall carry out tax assessment, and decide for whether the VAT registration should be
cancelled or not within 3 months of date of such application. (section 11(3) of Value Added Tax Act,
2052)
e. However, VAT registered person shall file VAT return for next 3 months or date of decision of VAT
cancellation whichever is earlier. (Rule 12(3) of Value Added Tax Rule, 2053)
f. However, such person cannot be relieved of any liability which may be incurred due to something
which has been done which should not have been done or something which has not been done which
should have been done during the period when such person was registered under VAT simply due to
the reason that VAT registration of such person has been cancelled.

Hence, following the above provisions of the law, Namaste Steel Private Limited can cancel its‘ VAT
registration.

Computation of VAT Payable/Refundable to the Company


Particulars Market Value VAT @13%
VAT on closing stock assuming credit already taken
Finished Goods Inventory 4 million 0.52 million
Raw Material 1.5 million 0.195 million
Damaged Goods 0.1 million 0.013 million
Plant and Machinery 4 million 0.52 million
Vehicle – 4 Wheeler 40 Lakh 0.52 million
Vehicle – 2 Wheeler 0.1 million 0.013 million
Land VAT not applicable 0
Sundry Debtors VAT not applicable 0
Sundry Creditors VAT not applicable 0
Total VAT Payable 1.781 million
Opening VAT Receivable 1.0 million
Net VAT Payable 0.781 million

7.
a) Silver Imports and Exports, Kathmandu has the following transactions of garments
and ginger:

Particulars Jestha, 2074 Ashadh, 2074 Shrawan 2074 (Rs..)


(Rs.) (Rs.)
Garments Export 3,00,000 6,00,000 10,00,000
Garments Local Sales 7,00,000 6,00,000 5,00,000
Ginger Export 1,00,000 3,00,000 4,00,000
Ginger Local Sales 4,00,000 5,00,000 6,00,000
Garment Purchase 8,00,000 7,00,000 10,00,000
Ginger Purchase 4,00,000 7,00,000 9,00,000

The company has no previous debit/credit balance in its VAT return. The Company
needs your advice to get refund of the VAT amount for these months. 5

b) Horizon Enterprise, Kathmandu has wholesale transactions of clothes and rice from
two business houses with same PAN. It has the following transactions for 2073
Bhadra to Kartik:
Months Clothes Sales Rs. Rice Sales Rs.
Bhadra 2073 1,00,000 3,00,000
Aswin 2073 5,00,000 5,00,000
Kartik 2073 5,00,000 6,00,000

The enterprise recorded its clothes sales Rs. 31 lakh and rice sales Rs. 20 lakh from
Bhadra 2073 to Shrawan 2074. Then, it has following sales in 2074 Bhadra to Kartik:

Months Clothes Sales Rs. Rice Sales Rs.


Bhadra 2074 10,00,000 3,00,000
Aswin 2074 16,00,000 5,00,000
Kartik 2074 10,00,000 6,00,000

Tax officer has issued a notice to register the business in VAT on 2074/05/10 to the
enterprise. The tax officer has calculated interest, additional fees, and penalties as per
VAT Act. Justify whether the enterprise is required to register in VAT or not. If it is
required to register, then when it has to do so and from when it has to collect tax? 5
Answers:

a)

Section 24 of VAT Act, 2052 has provisions for VAT refund. As per the provisions, when tax
becomes receivable for the month, excess amount may be adjusted with any outstanding amount, if
there is still receivable amount, remaining amount may be adjusted with the payable amount for the
next month. If it is continuous for 6 months, then registered person can apply for a lump sum refund.
For the export business for the fiscal year 2073/74 and 2074/75, any registered person whose export
sales for a month are 40 % or more of his/her total sales for that month, S/he shall be entitled to a
refund of the remaining excess after adjusting any outstanding amount.

Calculation of local sales and Export percentage

Months Local sales Rs. Export Rs. Total Rs.


Jestha 2074
Garments 700,000 3,00,000 10,00,000
Ginger 4,00,000 1,00,000 5,00,000
Total 11,00,000 4,00,000 15,00,000
Percentage 73.33 26.67 100
Ashad 2074
Garments 6,00,000 6,00,000 12,00,000
Ginger 5,00,000 3,00,000 8,00,000
Total 11,00,000 9,00,000 20,00,000
Percentage 55 45 100
Sharawan 2074
Garments 5,00,000 10,00,000 15,00,000
Ginger 6,00,000 4,00,000 10,00,000
Total 11,00,000 14,00,000 25,00,000
Percentage 44 56 100

Calculation of output tax and input tax credit


As per schedule 1 of VAT Act, ginger is vat exempted items, only garment is VAT attractive item
products. Export has 0 rate vat. So, VAT output is applicable only local sales garments and input is
applicable only for the purchase of garments.

Months Local sales VAT Purchase Rs. VAT Paid Rs. VAT
Rs. collected Receivable
Rs. (Rs.)
Jestha 2074 700,000 91,000 8,00,000 1,04,000 13,000
Ashad 2074 6,00,000 78,000 7,00,000 91,000 13,000
Sharawan 5,00,000 65,000 10,00,000 1,30,000 65,000
2074

As per the above mentioned provisions and export percentage, the silver export and import can make
adjustment of Rs. 13,000 for the month of Jestha, 2073 continuously next 6 months, if it is not
adjusted, then after it can claim for refund. It has less than the 40 % export during the month.

For the month of Ashadh and Shrawan, it has export more than 40 percent, so it is entitled for refund
of those receivable amounts during the filing of VAT Return.

b)

As per Rule 6 and Rule 7, there is exemption from registration to small vendors having 50 lakh or less
taxable transaction of goods during the previous twelve months. If the turnover of transaction carried
on by any person exceeds 50 lakh, the person carrying on such transaction has to make an application
in the format referred to in Schedule-1 to the concerned Tax Officer to get the transaction registered
within 30 days of the date of such excess.
Determination of Turnover Threshold- Rice is nontaxable sales, as such; the turnover of rice does not
have any role in determination of threshold for registration
Total Taxable Sales in last 12
Month
months (in lakhs)
12 Months to Shrawan 31
2074
12 months to Bhadra 2074 31-1+10= 40
12 months to Ashwin 2074 31-1-5+10+16= 51
12 Months to Kartik 2074 31-1-5-5+10+16+10=61

The Enterprise has tax exempted and taxable transactions, the taxable transaction exceeds 50 lakhs
(31 lakh-1 lakh-5 lakh + 10 lakh +16 lakh) on Aswin, 2074, the actual date is not given in the
questions, and only 1 lakhs is exceeded than prescribed limit.

The notice issued by tax officer is not compliance with VAT Act and Rules, total transactions of Rs.
51 lakh (31+20) from Bhadra 2073 to Sharawan 2074 is including VAT exempted goods.

The Enterprise is required to submit application within 30 days from Aswin, 2074; it can collect tax
after registration on VAT.

8. Write short note: (4×2.5=10)


a) Circumstances beyond Control
b) Purchase of Under Invoiced Goods
c) Taxable Value for Imported Goods
d) Tax Period
Answers:
a) Circumstances beyond control as per Value Added Tax Act, 2052
VAT should be deposited within due date. If a tax payer fails to deposit VAT within due date, then tax
payer is subject to fine at the prescribed rate. However, tax payer can apply for waiver of such fine to
the Department, and Department may waive such fine if the department is convinced that the tax
payer has not deposited the tax due to circumstances beyond control. Rule 35 of Value Added Tax
Rules 2053 specifies below cases as a situation beyond control:-
1. In case the person required to pay tax becomes disabled due to falling ill; up to seven days of the
date of his recovery.
2. In case the person required to pay tax is to observe obsequies; up to seven days of the end of the
obsequies,
3. In case a woman required to pay tax delivers a child; up to thirty five days of the date of delivery,
4. In case the person required to pay tax dies or becomes insane or disappears and his heir or
guardian submits an application within thirty five days of the date of such incident; up to seven
days of receipt of such application,
5. In circumstances when the person required to pay tax has not been able to come to the Tax Office
because of the closure of a road due to floods, landslides or similar other reasons; up to seven
days of opening of the road,
6. In circumstances when he cannot come to the tax office due to total haltage of transport; up to the
next day of the end of such haltage.

b) Purchase of Under Invoiced Goods as per Value Added Tax Act, 2052
Sale of goods at a price lower than fair market price of such goods at the time of sale of such goods is
known as sale of goods at under invoice. VAT is levied on a transaction value which is a fair market
value. Hence, if any tax payer shows sale of goods at a price lower than fair market price, then there is
avoidance of VAT, and loss of revenue. To protect such activity, VAT Act, 2052 has provided
specific provisions under section 23ga of VAT Act 2052 which provides special rights to Tax officer
to sale or purchase such under invoiced goods. As per the provisions of section 23ga,

1. Notwithstanding anything mentioned in any other laws in force, if any person is found to have
shown transaction by under invoicing by selling goods at a price less than the fair market price,
then Tax Officer can stop the further sale of such goods which are in stock, and may sale or order
for sale such goods at the under invoiced price.

2. If the person refuses to sale goods while purchasing or causing to purchase by the department or
Inland Revenue Office as per sub/rule (1), the department or the office may take in possession of
the goods and shall pay the amount, calculated on the basis of under invoice value, to the person
when the person comes to receive the payment.

3. The goods purchased or caused to be purchased pursuant to Sub-sections (1) and (2) may be sold
or caused to be sold at such price and in accordance with such procedures as may be specified by
the Director General.

c) Taxable value for imported goods: According to Sec. 12(5) and Rule 48; in case of goods, respective
custom office collects VAT based on taxable value. Section 12(6), provides the revaluation of
purchase price at market price in case of significant under-invoicing found or reasons to be believed.
Taxable value for the purpose of import shall be computed as follow:
Value paid to vendor or value as determined for custom duty
 Custom duty and custom service charge
 Excise or countervailing duty
 Other duties
 Demurrage or other penalties
 Insurance and transportation
d) Generally, it means period for which VAT assessment is carried out. Tax period for VAT is inter-related
to the related VAT Return.
Tax Period means a period prescribed by the Act or Rules for calculation of net VAT payable or
receivable.
A registered person has to adopt a month as per Nepali Calendar as tax period. The tax starts from day1 st
of a month and ends at the end day of the same month.
As per the provisions (Rule 26) of VAT Rule, if deemed fit by the tax officer, different tax periods may
have to be adopted / accepted by certain specified tax payers.
i) Tax Period of 2 months:
Under such circumstances, the tax period may be Shrawan and Bhadra, Ashwin and Kartik, Marga
and Poush, Magh and Falgun, Chaitra and Baisakh, and Jestha and Ashad. This tax period is generally
adopted by hotel and tourism enterprises if specifically applied for the same.
ii) Tax Period of 4 months:
Tax period of 4 months is adopted by tax payers specifically, if paper or electronic media or brick
producer applies so.
iii) Different tax period:
In case the registered person maintains its accounts in the approved computer systems, on application
by the tax payer, the tax officer can allow different tax period to be adopted provided the tax officer
deemed fit.
iv) Tax period for 1st time of registration:
In such cases, tax period starts right from time of its registration to the end of the month.
- End-
Suggested Answers/CAP-II/December/2009/Group-I

Advanced Accounting
Attempt all questions. Working notes should form part of the answer.

1.

Balance Sheet of Himal Ltd.


Rs. Rs.
Share Capital 20,00,000 Fixed Assets 15,00,000
General Reserve 15,00,000 Investment 2,50,000
Current Liabilities 15,00,000 Current Assets 32,50,000
50,00,000 50,00,000

Balance Sheet of Hill Ltd.


Rs. Rs.
Share Capital 10,00,000 Fixed Assets 3,00,000
General Reserve 5,00,000 Goodwill 1,00,000
Current Liabilities 2,00,000 Current Assets 14,00,000
Proposed dividend 1,00,000
18,00,000 18,00,000
Himal Ltd. absorbed Hill Ltd. on following terms & conditions:
a) Hill Ltd declares a dividend of 10% before absorption for the payment of which it is to retain
sufficient amount of cash.
b) The net worth of Hill Ltd. is valued at Rs. 14,50,000.
c) The purchase consideration is satisfied by the allotment of fully paid shares of Rs. 100 each in
Himal Ltd.
Following additional informations are also to be taken in to consideration:
 Himal Ltd. holds 2,500 shares of Hill Ltd. at a cost of Rs 2,00,000.
 The stock of Hill Ltd. includes items valued at Rs. 50,000 from Himal Ltd. (cost Rs. 37,500)
 The creditors of Hill Ltd include Rs. 15,000 due to Himal Ltd.

Show ledger account in the books of Hill Ltd. to give effect to the above and Balance Sheet of Himal
Ltd. after completion of absorption. 20

Answer
1)
Books of Hill Ltd.
Realisation Account

Particulars Amount Rs. Particulars Amount Rs.


To Good will 1,00,000 By Current Liabilities 2,00,000
To Fixed Assets 3,00,000 By Himal Ltd. 10,87,500
To Current Assets 13,00,000 By Share Capital 2,50,000
By Equity Shareholders a/c ( 1,62,500
17,00,000 loss) 17,00,000

1
Suggested Answers/CAP-II/December/2009/Group-I

Equity Shareholders (outside) Account


Particulars Amount Rs. Particulars Amount Rs.
To Realisation 2,50,000 By Share Capital 10,00,000
To Realisation Loss 1,62,500 By General reserve 5,00,000
To Shares in Himal Ltd 10,87,500
15,00,000 15,00,000

Balance Sheet Of Himal Ltd.


( After completion of absorption)

Capital & Liabilities Amount Rs. Assets Amount Rs.


Share Capital 30,87,500 Fixed Assets 18,00,000
30875 shares @ Rs. 100 fully Himal Ltd. 15,00,000
paid Hill Ltd. 3,00,000

Reserve & Surplus 16,25,000 Investment 50,000


General reserve 15,25,000
Capital reserve 1,00,000 Current Assets 45,47,500

Current Liabilities 16,85,000


63,97,500 63,97,500

Workings:
Adjusted Balance Sheet of Hill Ltd.
Capital & Liabilities Amount Rs Assets Amount Rs.
Share Capital 10,00,000 Fixed assets 3,00,000
General Reserve 5,00,000 Goodwill 1,00,000
Current Liabilities 2,00,000 Current assets 13,00,000*
17,00,000 17,00,000

Hill Ltd retains Rs. 1,00,000 in cash for dividend ( 10%) ( 14,00,000-1,00,000)

1. Total Purchase consideration ( based on net worth of Hill Ltd.) is Rs. 14,50,000.

2. Himal Ltd. holds 2500 shares in Hill Ltd. The percentage of holding is 25%

3. The net purchase consideration to pay Rs. 14,50,000 * ¾ = 10,87,500

4. Calculation Of Current assets


Current assets Of Himal Ltd. 32,50,000
Add Dividend 25,000
32,75,000
Less inter company amount 15,000
32,60,000
Current Assets Of Hill Ltd. 13,00,000
Less unrealized profit 12,500
( Rs. 50,000-37,500)
12,87,500

2
Suggested Answers/CAP-II/December/2009/Group-I

Total Current Assets 45,47,500

5. Calculation of Current Liabilities


Himal Ltd. 15,00,000
Hill Ltd. ( 2,00,000-15,000) 1,85,000
16,85,000

6. Calculation Of Capital Reserve


Assets taken over from Hill Ltd:
Fixed assets 3,00,000
Current assets ( 13,00,000-12,500) 12,87,500
15,87,500
Liabilities:
Investment in Hill Ltd. 2,00,000
Current Liabilities 2,00,000
Purchase consideration 10,87,500 14,87,500
Capital Reserve 1,00,000

2.
a) Ram, Shaym and Hari were trading in partnership sharing profits and losses equally. The
accounts of the firm are closed on 31st Ashad every year.

The partnership deed provided that:


On the death of a partner the goodwill was to be valued at 3 years’ purchase of the average
profit of past 3 years upto the date of death after deducting interest @ 10% on capital employed
and a fair remuneration of each partner. The profits are assumed to be earned evenly through
out the year.

Following were the profits for different years before charging interest on capital employed:

Year Profit (Rs)


2062/63 67,200
2063/64 75,600
2064/65 72,000
2065/66 62,400

On 30th Poush 2065 Ram died due to an accident and it was agreed on his death to adjust
goodwill in the capital accounts without showing any amount of goodwill in the Balance Sheet.

It was agreed for the purpose of valuation of goodwill that the fair remuneration for work done
by each partner would be Rs.15,000 per annum and that the capital employed is Rs.
1,50,000.Shyam and Hari were to continue the partnership, sharing profit and losses in a ratio of
2:1 after the death of Ram.
Compute the value of the goodwill and show adjustment entries in the books of the firm. 10

b)

3
Suggested Answers/CAP-II/December/2009/Group-I

i) A acquired on 1st January, 2009 a machine under a Hire-Purchase agreement which


provides for 5 half-yearly installments of Rs. 6,000 each, the first installment being due on
1st July, 2009. Assuming that the applicable rate of interest is 10 percent per annum,
calculate the cash value of the machine. 5
ii) Briefly describe the disclosure requirements for "Deferred Tax Assets" and "Deferred Tax
Liabilities" as per relevant Nepal Accounting Standard. 5

Answer

a) Computation of the value of goodwill:


(i) Average Profit for the year ending 30th Poush; before death of Ram:
Year ending 30th Poush, 2063: Rs. Rs.
½ of 2062/63 profits 33,600
½ of 2063/64 profits 37,800 71,400

Year ending 30th Poush, 2064:


½ of 2063/64 profits 37,800
½ of 2064/65 profits 36,000 73,800

Year ending 30th Poush, 2065:


½ of 2064/65 profits 36,000
½ of 2065/66 profit 31,200 67,200
Total profit for 3 years 2,12,400

Average Profit 70,800

(ii) Profit after adjustment of remuneration and interest on capital :


Rs
Average Profit 70800
Less:Remuneration(15000*3) 45000
25800
Less:Interest on Capital 15000
10800

(iii) Goodwill at 3 three years’ purchase =10,800*3=32,400

Adjustment entries for Goodwill


Journal
Dr. (Rs.) Cr. (Rs.)
Goodwill Account 32,400
To Capital Accounts Dr.
Ram 10,800
Shyam 10,800
Hari 10,800
(Goodwill, valued @ Rs. 32,400 adjusted in the
capital accounts of partners on the death of Mr.
Ram in the old profit sharing ratio)

Shyam’s Capital Account 21,600

4
Suggested Answers/CAP-II/December/2009/Group-I

Hari’s Capital Account 10,800


To Goodwill Account Dr. 32,400
(Goodwill written off between continuing partners Dr.
in the new profits sharing ratio)

Answer
b)
i) Statement showing cash value of the machine acquired on hire-purchase basis
Installment Interest @ 5% half yearly Principal Amount (in each
Amount (10% p.a.) = 5/105 = installment)
1/21) (in each
installment)
Rs. Rs. Rs.
5th Instalment 6,000 286 5,714
Less: Interest – 286
5,714
Add: 4th Instalment 6,000
11,714 558 5,442
Less: Interest 558 (11,156–5,714)
11,156
Add: 3rd instalment 6,000
17,156 817 5,183
Less: Interest 817 (16,339–11,156)
16,339
Add: 2nd instalment 6,000
22,339 1,063 4,937
Less: Interest 1,063 (21,276–16,339)
21,276
Add: 1st instalment 6,000
27,276 1,299 4,701
Less: Interest 1,299 (25,977–21,276)
25,977 4,023 25,977

The cash purchase price of machinery is Rs. 25,977.

ii)
 An enterprise should offset deferred tax assets and deferred tax liabilities if:
 the enterprise has a legally enforceable right to set off assets against liabilities
representing current tax, and
 the deferred tax assets and the deferred tax liabilities relate to taxes on income levied by
the same governing taxation laws.
 Deferred tax assets and liabilities should be distinguished from assets and liabilities
representing current tax for the period. Deferred tax assets and liabilities should be disclosed
under a separate heading in the balance sheet of the enterprise, separately from current
assets and current liabilities.

5
Suggested Answers/CAP-II/December/2009/Group-I

 The break-up of deferred tax assets and deferred tax liabilities into major components of the
respective balances should be disclosed in the notes to accounts.
 The nature of the evidence supporting the recognition of deferred tax assets should be
disclosed, if an enterprise has unabsorbed depreciation or carry forward of losses under tax
laws.

3.
a) (3+7=10)
i) Explain the rationale of linking Risk Weighted Assets with Capital of a bank or financial
institution.

ii) Following are the extraction of the Financial Statements of a Financial Institution for the
Year ending 2008/09:

Rs. In lakhs
Cash in Hand 300
Cash at Bank 500
Loan and Advances
Gross- 10,000
Accumulated Provisions 200 9,800
Fixed assets 150
Other Assets 200

Following further information is available:

Risk Weightage given for above assets is 0%,20%,100%,100% and 100% respectively. Other
assets include Rs.130 Lakhs for expenditures not yet written off.
The financial institution has managed its total Capital upto 11% of the total Risk Weighted
Assets.

Find out the amount of Reserve and Surplus if the paid up capital is Rs. 900 Lakhs.

b) (7+3=10)
i) Ramesh runs a stationery shop but does not maintain proper books of account. Following
details are given:
Purchase made during the year Rs. 85,000
Closing stock Rs. 15,000
Opening cash, debtors & fixed assets Rs. 1,15,000
Opening creditors & capital Rs. 1,29,000

The past record of business shows that the rate of gross profit is 20% on sales.

Calculate sales made during the year.

ii) Explain about corporate assets.

Answer
a)

6
Suggested Answers/CAP-II/December/2009/Group-I

i) It is an inbuilt mechanism by which total risk exposure of financial institutions is made directly
proportional to its capital. If total risk exposure is increased or in other words if the institution
intends to expose itself to more risky portfolios, it has to increase its capital. Ultimately, it plays as a
cushion for protecting depositors’ interest. Owners/Shareholders should have adequate capital so
that they can continue managing public deposits in the form of different assets.

ii) Finding out the Reserve and Surplus of the Financial Institution

Calculation of Risk Weighted Assets


Amount Rs. In Lacs
Assets Amount Risk Weight (in %) RWA Amount
Cash in Hand 300 0 0
Cash at Bank 500 20 100
Loan And Advances 10000 100 10000
Fixed Assets 150 100 150
*Other Assets 70 100 70
Total 11020 10320

*Expenses not written off is excluded from RWA as it is to be deducted from its core capital

As the Institution has just maintained its capital of 11 %, Total Capital including Reserve and Surplus comes
to- 11% of 10320,i.e. Rs 1135.2 Lacs

However, there are fictitious assets not written off to the extent of Rs 130 Lacs which is already deducted
while arriving at above total capital.
Hence, total capital comes to Rs (1135.2+130)=Rs. 1265.20 Lacs
Total Capital Comprises of Paid up Capital and Reserve and Surplus and paid up capital is Rs 900 Lacs.

So, total Reserve and Surplus comes to Rs.(1265.20-900)= Rs 365.20 Lacs

Answer
b)
i) The problem is silent on opening stock also. However, this can be easily derived from the relevant
given items of opening statement of affairs. Since the given Rs. 1,29,000 opening creditors and
capital exceeded by Rs. 14,000 over the specified opening cash, debtors and fixed assets, this excess
is taken to represent the opening stock. Now, the sales can be reckoned by deriving cost of sales and
adding there to the gross profit. But note that the expressed 20% gross profit is on sales which are to
be derived. Hence, this 20% on sales be converted in to on cost which would be 25% ( 20*100/80)

Trading Account

To cost of goods sold: By Sales 1,05,000


Opening stock 14,000 ( Balancing figure)
Add: Purchase 85,000
99,000
Less : closing stock: 15,000 84,000

7
Suggested Answers/CAP-II/December/2009/Group-I

Gross profit ( 25% on cost) 21,000


1,05,000 1,05,000

ii) Corporate assets are the assets other than goodwill that contribute to the future cash flow of
both cash generating unit under review and other cash generating units.

4.
a) Spice Ltd. has three departments I, J and K. The following information is provided for the year
ended 31st Ashadh, 2065:

I J K
Rs. Rs. Rs.
Opening stock 5,000 8,000 19,000
Opening reserve for unrealized profit ― 2,000 3,000
Materials consumed 16,000 20,000 ―
Direct labour 9,000 10,000 ―
Closing stock 5,000 20,000 5,000
Sales ― ― 80,000
Area occupied (sq. mtr.) 2,500 1,500 1,000
No. of employees 30 20 10

Stocks of each department are valued at cost to the department concerned. Stocks of I are
transferred to J at cost plus 20% and stocks of J are transferred to K at a gross profit of 20% on
sales. Other common expenses are salaries and staff welfare Rs. 18,000 and rent Rs. 6,000.

Prepare Departmental Trading, Profit and Loss Account for the year ending 31st Ashadh, 2065. 10

b) Explain the major considerations, from an accountants view point, at the time of conversion of
partnerships to a limited company. 5

Answer

a)
Departmental Trading and Profit and Loss Account
for the year ended 31st Ashadh, 2065
I J K Total I J K Total
Rs. Rs. Rs. Rs. Rs. Rs. Rs. Rs.
To Opening By Sales 80,000 80,000
stock 5,000 8,000 19,000 32,000
To Material By Inter-dept.
consumed 16,000 20,000 36,000 transfer
To Direct labour 9,000 10,000 19,000 Closing 30,000 60,000 90,000
By stock
5,000 20,000 5,000 30,000
To Inter-dept. 30,000 60,000 90,000
transfer
To Gross profit

8
Suggested Answers/CAP-II/December/2009/Group-I

5,000 12,000 6,000 23,000 ______ ______ ______ _______


35,000 80,000 85,000 2,00,000 35,000 80,000 85,000 2,00,000
To Salaries and 9,000 6,000 3,000 18,000 By Gross 5,000 12,000 6,000 23,000
staff welfare By profit b/d 7,000 7,000
Net loss
To Rent 3,000 1,800 1,200 6,000
To Net profit ______ 4,200 1,800 6,000 _____ _____ _____ _____
12,000 12,000 6,000 30,000 12,000 12,000 6,000 30,000
To Net loss (I) 7,000 By Stock 5,000
To Stock reserve
reserve (J+K) b/d (J + K)
(Refer W.N.) 3,000 By Net profit 6,000
(J + K)
To Balance 1,000 _____
transferred
to Profit and
loss account
11,000 11,000

Working Note:
Calculation of unrealized profit on closing stock
Stock reserve of J department Rs.

Cost 30,000

Transfer from I department 30,000

60,000

Stock of J department 20,000

Rs.30,000
Proportion of stock of I department = Rs. 20,000 = Rs.10,000
Rs.60,000
20
Stock reserve =Rs.10,000  = Rs.1667 (approx.)
120
Stock reserve of K department Rs.
Stock transferred from J department 5,000

Less: Profit (stock reserve) 5,000  20% 1,000

Cost to J department 4,000

Rs.30,000
Proportion of stock of I department =Rs. 4,000   Rs.2,000
Rs.60,000
20
Stock reserve  Rs.2,000   Rs.333 (approx.)
120
Total stock reserve = Rs.1,000 + Rs.333 = Rs.1,333

9
Suggested Answers/CAP-II/December/2009/Group-I

Answer

b) While converting partnership firms into a company, first consideration should be given to ascertain
position of the firm up to the cut off point. It includes ascertaining past losses by preparing a
Reconstruction Account. The amount of loss so ascertained is ultimately written off by adjusting the
same to the debit of the old partner’s capital accounts. In case creditors agree to join, then whole or
part of the accounts standing to their credit is transferred to their capital account as agreed. If fresh
capital or loan is raised, the same is to be credited to respective parties capital or loan accounts.
Other considerations may be of apportionment of shares among partners. Partners may agree to
continue, in the proposed company, in the same profit sharing ratio as in partnership or it may be
different. If net value placed on the business is more than the capital contributed by the partners the
same should be credited to respective partner’s capital accounts in the old profit sharing ratio.
Similarly, due consideration should be given if different priority is given for repayment of capital
amongst partners or new shareholders.

5.
a) Distinguish between adjusting and non-adjusting events under relevant Nepal
Accounting Standard. 5

b) Following Balance Sheet pertains to Rising Nepal Ltd. as on 31st Ashad 2065:
Rising Nepal Ltd.
Balance Sheet as on 31st Ashad 2065

Liabilities Current Year(Rs)


Share Capital 13,00,000
Reserve and Surplus 4,50,000
Total 17,50,000

Assets Current Year(Rs)


Fixed Assets-Gross 11,00,000
Less: Accumulated Depreciation 2,50,000
Net Block 8,50,000
Current Assets
Cash in Hand 40,000
Cash at Bank 3,25,000
Sundry Debtors 6,60,000
Advance to Supplier 2,50,000
Stock in Hand 50,000
Sub-total 13,25,000
Less:Current Liabilities
Sundry Creditors 3,00,000
Advance from Customers 1,25,000
Sub-total 4,25,000
Net Current Assets 9,00,000
Total 17,50,000

10
Suggested Answers/CAP-II/December/2009/Group-I

Following matters were not considered while preparing above Balance Sheet:
Advance to Supplier includes Raw material advance amounting to Rs. 1,75,000 given to a Limited
company in India. The company was liquidated on December 2007 (Poush 2064).Liquidator made
public notice for any claims against the company during the course of liquidation. Rising Nepal
Limited was aware of the process only on Shrawan 2065. The court declared the company
bankrupt on Shrawan. There was no possibility of recovering the advance in any circumstances.
Similarly, Cheques issued by customers amounting to Rs. 60,000 was received by the company on
2nd Shrawan 2065. Further, there was a fire in one of its plants on first week of Bhadra causing
significant damage to the plant and assets thereon. The book value of the plant along with all
the fixed assets there was Rs. 3,50,000. However, no estimation of loss is finalized yet.

The Financial Statements of the company were approved on 15th Bhadra 2065.

Prepare the Balance Sheet after considering the provisions of applicable Nepal Accounting
Standard. 10

Answer
a) Adjusting events are the events after the balance sheet date which requires adjustment in the
financial statements even if the outcome is known after the balance sheet date. Where as non
adjusting events are those events which qualifies to be events after the balance sheet date under
NAS 05 but do not qualify to be recognized in the financial statements. Adjusting events are
quantified and recognized in the financial statements where as non-adjusting events may or may not
be quantified. When the condition relating such events were in existence at the balance sheet date
and the events only confirmed the outcome before the approval of the financial statements, such
events qualify to be adjusting events and where such conditions were not in existence then the same
should be regarded as non-adjusting events.

b)
Rising Nepal Ltd
Balance Sheet as on Ashad end 2065

Liabilities Current Year(NRs)


Share Capital 1,300,000
Reserve and Surplus 275,000
Total 1,575,000
Assets Current Year(NRs)
Fixed Assets-Gross 1,100,000
Less: Accumulated Depreciation 250,000
Net Block 850,000
Current Assets
Cash in Hand 40,000
Cash at Bank 325,000
Sundry Debtors 660,000
Advance to Supplier 75,000
Stock in Hand 50,000

11
Suggested Answers/CAP-II/December/2009/Group-I

Sub-total 1,150,000
Less:Current Liabilities
Sundry Creditors 300,000
Advance from Customers 125,000
Sub-total 425,000
Net Current Assets 725,000
Total 1,575,000

Disclosure of Plant and Machinery destroyed by fire

A material damage was made by fire to plant and machinery with book value of Rs.3, 50,000.The process of
estimation of the loss is being carried out for the same. The amount of loss and the recovery of the same by
way of insurance claim can be ascertained only upon completion of the process of assessment of the
damage thereon.

Note: Neither the adjustment nor the disclosure is required in case of cheques received in the ordinary
course of the business.

6. Write short notes on (ANY FOUR): (4x2.5=10)


a) Accounting treatment of contingency gains
b) Government Accounting System in Nepal
c) Prediction of insolvency on the basis of ratios
d) Related party
e) Reinsurance and its usefulness to insurance companies

Answer
a) Contingency gains are not recognized in financial statement since their recognition may result in the
recognition of revenue which may never be realized. However, when the realization of a gain is
virtually certain, then such a gain is not a contingency and accounting for the same is appropriate.

b) Government Accounting System in Nepal is generally on Cash Basis. It has set chart of accounts
under which revenue and expenditure are accounted for. It follows double entry system; however,
do not follow the mercantile system of accounting. Government accounting system broadly classifies
expenditure into administrative and development expenses. Accounting system followed by the
government differentiates Capital expenditures and revenue expenditure in its subsidiary records.
Office of the Financial Comptroller General specifies the chart of accounts under which all the
government revenue and expenditure are to be accounted for.

c) The relevance of the ratios in predicting insolvency can be elaborated with the help of the following
illustrative ratios as below:
Working capital to total assets indicates the liquidity position of the firm. If the ratio is too low it
indicates inability of the firm to carry on its day to day activities. If it is negative, the firm will not
have fund for its day to operations. If such situation continues, the firm may be forced to suspend its
operations and it may result insolvency in the long run.
Similarly, ratio of sales to total assets indicates the utilization of its assets to generate sales which
ultimately generates surplus for the firm. If it is too low, it indicates that the firm is keeping idle
assets which in long run may result to insolvency.

12
Suggested Answers/CAP-II/December/2009/Group-I

Another example can be given of retained earnings to total assets. Retained earnings are cushion for
firm’s health. So if it is too thin it may indicate that firm has very low leverage and is posed to
insolvency earlier.

d) Parties are considered to be related if at any time during reporting period one party has ability to
control over other party or exercise significant influence over the other party in making financial and
or operating decision.

e) Reinsurance means insurance affected by an insurance company in order to cover itself against a
large risk. Suppose a building is insured with company A for Rs.50 lakhs. Company A, in order to
reduce its risk, can get the same building insured with company B. The contract between company A
and company B will be reinsurance. Reinsurance can be for part of the amount or for some of the
risks. The reinsurance company will pay a commission on the premium received. In the above
example, company A will pay commission to company B. For company B the commission will be the
“commission on reinsurance accepted” and for company A it will be “commission on reinsurance
ceded”.

The system of reinsurance is very useful to insurance companies as it helps to pass the risk of one
insurer to other insurer. It also helps to comply the requirement of the regulator.

13
Suggested Answers/CAP-II/December/2009/Group-I

Audit & Assurance


Attempt all the questions.

1. Give your opinions with reason on the following cases: (54=20)

a) An audit report states “subject to all the notes on the accounts, we report that the balance sheet
and the profit and loss account give a true and fair view……”

b) A sum of Rs.10,00,000 is received from an Insurance company in respect of a claim for loss of
goods in transit costing Rs.8,00,000. The amount is credited to the Purchases Account.

c) An auditor of a limited company did not verify the investment and he inserted a note in the
balance sheet "Investment not verified". The shareholders approved and adopted the accounts
at the annual general meeting. Subsequently, it transpired that investments were
misappropriated and the company suffered a loss.

d) BMG Ltd. is a manufacturing company produces durable consumer goods with an annual
turnover of Rs. 100 crores. The company receives orders from its commission agents all over the
country, but goods are dispatched directly to the customers. The documents including transport
bills are sent through the bank for collection. At the end of the 6th year, it is found that
documents covering the dispatch of goods worth Rs. 10 crores were still lying with the banks not
cleared by the customers even though the normal collection period of 15 days from the date of
dispatch has expired. Should revenue be recognised in the above case?

e) In view of the qualified report of the statutory auditor, the Board of Directors has decided to
amend the accounts and resubmit the same to the statutory auditor for report before the
accounts are placed for adoption at the annual general meeting.

Answer:
a) The report is not proper. It is not clear whether the auditor is really making a
qualification. Many of the notes on the accounts of explanatory nature can not in any
case be a subject matter of the auditor’s qualification. The auditor should have made
the qualification with reference to specific notes about which he has reservations, and
should have also quantified the effect of the qualifications.

b) All items of income and expense which are recognized in a period should be included in
the determination of net profit or loss for the period. The claim for loss of goods in
transit is arising out of ordinary activities of the enterprise as a part of its normal course
of business. However, the cost of goods lost in transit is only Rs.8,00,000 while the
insurance money received is Rs.10,00,000. Purchases Account need not be credited
since it would distort the purchases done during the year and as also the gross profit.
Therefore, entire amount of Rs.10 lacs needs to be taken to profit and loss account
under an appropriate head. This is an income arising from an ordinary activity of the
enterprise but having regard to amount involved and exceptional nature, a separate
disclosure is to be made in the profit and loss account. Such disclosure would enable
the users to understand the performance of an enterprise for the period.

14
Suggested Answers/CAP-II/December/2009/Group-I

c) In case of audit of a limited company, an auditor has to comply with the statutory duties
as prescribed under Companies Act. Verification of investments is an important function
of an auditor since, it is an important asset shown in the balance sheet. The auditor
cannot be expected to give a report on the truth and fairness of the financial
statements of the company without verifying its investment. If he specifically mentions
in his audit report, the fact that, he did not verify the investments, he would not be
relieved from his statutory duties. Such statutory duties can never be curbed, though
they may be extended.

d) According to NAS - 07 on Revenue, revenue from the sale of goods shall be recognized
when
 the seller of goods has transferred to the buyer the significant risks and rewards of
ownership of the goods;
 the seller retains neither continuing managerial involvement to the degree usually
associated with ownership nor effective control over the goods sold;
 the amount of revenue can be measured reliably.
 It is probable that the economic benefits associated with the transaction will flow to the
entity; and
 The costs incurred or to be incurred in respect of the transaction can be measured reliably.

Since the transport bills were sent through the bank for collection, it may be said that the seller
entity has retained effective control over the ownership of goods. Further since the documents
were not cleared by the customer even after the expiry of the normal period of collection, there
is an uncertainty in the realization of sale proceeds. Apparently, the amount also appears to be
quite material being 10% of total turnover. Hence, revenue should not be recognised in this
case.

e) The Board of Directors is competent to amend the accounts and re -submit the same to
the statutory auditor for his report before the accounts are placed at the annual general
meeting. The auditor should examine the accounts again after amendment to satisfy
about the appropriateness or otherwise of the amendments. He should in particular see
that the points covered by his qualifications have been properly rectified. The report
that may be issued by the statutory auditor on such amended accounts will be in
substitution of the report issued on accounts before amendment. In other words, a
fresh audit report has to be issued on the amended accounts and all copies of the first
report and originally prepared accounts have to be withdrawn. In case it is not possible
to withdraw all copies of the orig inal accounts and the original report of the auditor, an
adequate disclosure of the fact of the revision of accounts approved by the Board and
reported on by the auditor should be made in the amended accounts. In case the
auditor is satisfied about such disclosure, he may not refer to this matter in his report.
However, in case he feels that the disclosure is not adequately comprehensive or no
disclosure has been made in the amended accounts, it will be the duty of the auditor to
refer to the fact in his revised report.

15
Suggested Answers/CAP-II/December/2009/Group-I

2. Answer the following: (35=15)


a) What are the assertions with which an auditor is concerned with while obtaining audit evidence
from substantive procedures?
b) Mention briefly the conditions or events, which increase the risk of fraud or error leading to
material misstatement in financial statements.
c) Explain the advantages of "Audit Working Papers".

Answer:
a) An auditor is concerned with following assertions:-
i) Existence: That an asset or liability exists at a given date.
ii) Rights and obligations: That an asset is a right of the concern and a liability is an
obligation at a given date.
iii) Occurrence: That a transaction or event which took place pertains to the entity
during the relevant period.
iv) Completeness: That there are no unrecorded assets, liabilities or transaction.
v) Valuation: That an asset or liability is recorded at an appropriate carrying value.
vi) Measurement: That a transaction is recorded in the proper amount and revenue or
expense is allocated to the proper period.
vii) Presentation and disclosure: That an item is disclosed classified and described in
accordance with recognised accounting policies and practices and relevant statutory
requirements.

b) In planning and performing his examination, the auditor should take into consideration
the risk of material misstatements of the financial information caused by fraud or error.
Weaknesses in the design of the internal control system and non-compliance with
identified control procedures amongst other conditions or events which increase the risk
of fraud or error are:
i) Weaknesses in the design of internal control system and non-compliance with the laid
down control procedures, e.g., a single person is responsible for the receipt of all
information and marking it to the relevant sections or two persons are responsible for
receipt of information but the same is not followed in actual practice, etc.
ii) Doubts about the integrity or competence of the management, e.g., domination by one
person, high turnover rate of employees, frequent change of legal counsels or auditors,
significant and prolonged understaffing of the accounts department, etc.
iii) Unusual pressures within the entity, for example, industry is doing well but the company
is not performing all right, heavy dependence on a single line of product, inadequate
working capital, entity needs raising share prices to support the market price in the wake
of public offer, etc.
iv) Unusual transactions such as transactions with related parties, excessive payment for
certain services to lawyers, etc.
v) Problems in obtaining sufficient and appropriate audit evidence, e.g., inadequate
documentation, significant differences between the figures as per the accounting
records and confirmation received from third parties, etc.
c) Audit working papers constitute the basic records for the auditor in respect of the audit
carried out by him. They constitute the link between the auditor's report and clients'
record.

16
Suggested Answers/CAP-II/December/2009/Group-I

These include retention of permanent record in the nature of a document to show the
actual audit work executed the nature of the work, the extent of the work and important
points, facts, dates and decisions having bearing on the audit of the accounts audited.
The working papers, if properly maintained, can be used as defense in case of need. The
audit working papers are found very useful in the following aspects as they:
i) aid in the planning and performance of the audit;
ii) aid in the supervision and review of the audit work;
iii) provide evidence of the audit work performed to support the auditor's opinion; and
iv) act as an evidence in the Court of law when a charge of negligence is brought against
the auditor.

3. Give your comments on the following: (35=15)

a) In a medium size trading organisation the accountant was given additional responsibility of
making recoveries from the debtors. On one occasion, when an insurance claim of Rs. 25,000
was received, he credited the same to the account of a debtor and misappropriated the cash
which he had recovered from the said debtor. Pinpoint weaknesses in the internal control system
which led to this situation.

b) Gear Ltd. is engaged in manufacturing and supply of gear boxes to Kathmandu Automobile Ltd.
As per terms of supply, full price of the goods are not released by Kathmandu Automobile Ltd.
but 10% thereof is retained and paid after one year, if there is satisfactory performance of the
parts supplied. Gear Ltd. accounts for only 90% of the invoice value as sale at the time of supply
and balance 10% is accounted as sale in the year of receipt of payment.

c) Inventories of a car manufacturing company include the value of items required for the
manufacture of a model which was removed from the production line five years back, at cost
price.

Answer:
a) Following two essential features of internal control are relevant here :
i) Breaking the chain of the work in a manner so that no single person can handle a
transaction from the beginning to the end and
ii) Segregation of accounting and custodial functions.
Weakness in internal control system in the instant case:
 The accountant is receiving cash and also passing the entries in the books. The
accountant should not have been allowed to effect recoveries.
 It also appears that system for issuing receipts for amount received - whether
cash or cheque is also lacking.
 In a small and to some extent medium size organization, the supervision of the
owner offsets the deficiencies in internal control system. But i n this case, it
appears, that supervision and personal control is also lacking.
Thus, in the given case, the main weakness of the system is that it is ignoring the
basic requirements of a good internal control system.
b) According to NAS 7 on Revenue, revenue from sale of goods should be recognized when
the seller has transferred to the buyer, the property in the goods for a price or when the
seller has transferred all significant risk and rewards and the seller repairs no effective

17
Suggested Answers/CAP-II/December/2009/Group-I

control over goods and no significant uncertainty exists regarding the amount of
consideration and its collectability.
In the given case the goods as well as the risk and ownership has been transferred by Gear
Ltd., to Kathmandu Automobile Ltd., on the basis of invoice and delivery of material.
In the instant case, therefore, Gear Ltd., should recognize sale at full 100% of the invoice
value in spite of the fact that 10% payment will be released after one year. However,
depending upon the past experience regarding collectability of 10% amount, they can make
a provision for the amount that is not likely to be realized.
Hence, the treatment given by the company is not correct and if they do not correct it, the
auditor should qualify his report.

c) Inventory valuation: NAS 4 on “Inventories” provides that the cost of inventories may
not be recoverable if those inventories are damaged, have become wholly or partially
obsolete, or if their selling prices have declined.
Accordingly, the auditor should examine whether appropriate allowance has been made
for the defective, damaged, obsolete and slow-moving inventories in determining the
net realizable value.
In this case, items required for the manufacture of a model which has been withdrawn
from the production line five years ago are included in the stock at cost price resulting
in overstatement of inventory and profit. As it appears from the facts given that the
net realizable value of these items is likely to much lower than the cost at which these
are being shown in the books of account.
Accordingly, it becomes necessary to write down the inventory to ‘net realizable value’
if the items of inventories become wholly or partially obsolete. Under the
circumstance, the auditor should qualify the report appropriately.

4. Answer the following: (35=15)


a) What is an "Audit Evidence"?
b) "Physical presence of the auditor at the time of year end verification of stocks is though not
always possible, it is recommended that he should at least be present as an observer." Signify the
importance of this statement and list out the important aspects which the auditor should look
into to ensure an effective physical verification programme.
c) Indicate briefly the purposes for which analytical procedures are applied by the Auditor.

Answer:
a) Audit Evidence: Audit evidence refers to any information, verbal or written, obtained by
the auditor on which he bases his opinion on financial statements.
The audit evidence may be of varied nature and can assume various forms. For example,
a signature on the voucher of a designated official, the payee’s receipt, etc. Even the
information obtained by the auditor by discussing with the officials of the compan y also
constitutes audit evidence.
Generally audit evidence depending on its source may be classified as internal evidence
or external evidence. Internal evidence is one that has been created within the client’s
organization and without its ever going to outside party. Examples are duplicate sales
invoices, employee’s time reports, etc. External evidence on the other hand is the
evidence that originates outside the client’s organization; for example, purchase invoice,

18
Suggested Answers/CAP-II/December/2009/Group-I

bank statement, debit notes and credit notes coming from parties, quotations,
confirmations, etc.
Sometimes in certain transactions, external evidence is obtained, directly by the auditor,
e.g., certificates as regards bank balance, confirmation of balances of debtors and
creditors, etc.
The auditor also obtains evidence by performing various analytical procedures. The
auditor should evaluate whether he has obtained sufficient appropriate audit evidence
before he draws his conclusions there from.
The reliability of audit evidence depends on its source – internal or external, and on its
nature-visual, documentary or oral. The auditor may gain increased assurance when
audit evidence obtained from different sources or of different nature is consistent. In
these circumstances, he may obtain a cumulative degree of assurance higher than that
which he attaches to the individual items of evidence by themselves.
Conversely, when audit evidence obtained from one source is inconsistent with that
obtained from another, further procedures may have to be performed to resolve the
inconsistency. Audit evidence should, in totality enable the auditor to form an opinion
on the financial information.

b) Physical verification of inventories is the responsibility of the management of the entity.


However, where the inventories are material and the auditor is placing reliance upon the
physical count by the management, it may be appropriate for the auditor to attend the
stock-taking. The extent of auditor's attendance at stock-taking would depend upon his
assessment of the efficacy of relevant internal control procedures, and the results of his
examination of the stock records maintained by the entity and of the analytical review
procedures. The procedures concerning the auditor's attendance at stock -taking depend
upon the method of stock-taking followed by the entity. There are two principal
methods of stock-taking: periodic stock-taking and continuous stock-taking. Under the
first method, physical verification of inventories is carried out at a single point of time,
usually at the year end. Under the second method, physical verification is carried out
throughout the year, with different items of inventory being physically verified at
different points of time. However, the verification programme is normally so designe d
that each material item is physically verified at least once in a year and more often in
appropriate cases. The continuous stock-taking method is effective when a perpetual
inventory system of record-keeping is also in existence. Some entities use continuous
stock-taking methods for certain stocks and carry out a full count of other stocks at a
selected date. The auditor is expected to examine the adequacy of the methods and
procedures of physical verification followed by the entity.

Before commencement of the verification, the management should issue appropriate


instructions to stock-taking personnel. Such instructions should cover all phases of physical
verification and preferably be in writing. It would be useful if the instructions are formulated by
the entity in consultation with the auditor. The auditor should examine these instructions to
assess their efficacy. Where the auditor is present at the time of stock-taking, he should observe
the procedure of physical verification adopted by the stock-taking personnel to ensure that the
instructions issued in this behalf are being actually followed. The auditor should also perform
test-counts to satisfy himself about the effectiveness of the procedures.

19
Suggested Answers/CAP-II/December/2009/Group-I

c) Analytical procedures are used for the following purposes:

i) to assist the auditor in planning the nature, timing and extent of other audit procedures;
ii) as substantive procedures when theirs use can be more effective or efficient than tests of
details in reducing detection risk for specific financial statement assertions, and
iii) as an overall review of the financial statements in the final review stage of the audit.

Thus, analytical procedures may be used extensively while conducting an audit to establish
authenticity of information contained in the financial statements. In some cases, analytical
procedures can be more effective than tests of details in reducing detection risk for specific
financial statement assertions. This is particularly true in case volume of information is
homogeneous and quite large. For instance, in case of salaries and wages the volume is generally
quite large and only internal evidence is available. In such a case, performing analytical
procedures would provide substantive audit evidence and reduce the extent of vouching to a
large extent.

5. Comment on the following: (35=15)

a) Mr. A, a Chartered Accountant gave 50% of the audit fees received by him to the person, who
was not a Chartered Accountant, under the classification of office allowance and such an
arrangement continued for a number of years.

b) Mr. Baidya, who is a practicing Chartered Accountant, accepted his appointment as an auditor at
a lower fee than the audit fee charged by the previous auditor. Some of the practicing
accountants claimed that Mr. Baidya had obtained the audit by undercutting.

c) There is an affirmative relationship between materiality and the level of audit risk.

Answer:
a) Mr. A, Chartered Accountant had shared his profits with a non CA and, therefore, is guilty of
professional misconduct under section 34(2) of ICAN Act. It is not the nomenclature to a
transaction that is material but it is the substance of the transaction, which has to be looked
into.

b) As per Section 10.2 of Code of Ethics issued by ICAN, a professional accountant in public practice
should charge fees taking into account:
i) The skill and knowledge required for the type of services involved
ii) The level of training and experience necessary for the person necessarily engaged in
performing the services
iii) The time necessarily occupied by each person engaged in performing the services
iv) The degree of responsibility that performing those service entails.
As per Section 10.4 of Code of Ethics issued by ICAN, a professional accountant in public practice
should not make a representation that specific professional services in current or future periods
will be performed for either a stated fee, estimated fee or fee range if it is likely at the time of

20
Suggested Answers/CAP-II/December/2009/Group-I

the representation that such fees will be substantially increased and the prospective client is not
advised of that likelihood.

As per Section 10.6 of Code of Ethics issued by ICAN, it is not improper for a professional accountant
in public practice to charge a client a lower fee than has previously been charged to similar services,
provided the fee has been calculated in accordance with Section 10.2 and 10.4 of Code of Ethics
issued by ICAN. In the given case, it would not amount to the undercutting if the fees are based on
the quantum of work, incidental and pocket expenses and other terms of appointment and hence
the claim of the other accountants may not be held tenable.

c) No. As per para 10 of NAS 320 (Audit Materiality) , There is an inverse relationship between
materiality and the level of audit risk, that is, the higher the materiality level, the lower the audit
risk and vice versa. The auditor takes the inverse relationship between materiality and audit risk
into account when determining the nature, timing and extent of audit procedures. For example,
if, after planning for specific audit procedures, the auditor determines that the acceptable
materiality level is lower, audit risk is increased. The auditor would compensate for this by
either:
i) Reducing the assessed risk of material misstatement, where this is possible, and
supporting the reduced level by carrying out extended or additional tests of control; or
ii) Reducing detection risk by modifying the nature, timing and extent of planned
substantive procedures.

6. Write short notes on the following (Any TWO): (25=10)


a) Surprise Checks
b) Modified Auditor's Report
c) Flow Chart

Answer:
a) Surprise checks are mainly intended to ascertain whether the system of internal control is
operating effectively and whether the accounting and other records are prepared concurrently
and kept up-to-date. It has often been found that manipulations and frauds are facilitated under
a system of book-keeping which does not give proper emphasis to the need to keep the books
up-to-date. Errors in book-keeping are often indicative of weaknesses in internal control which
may be taken advantage of in order to perpetrate frauds or manipulations.

The element of surprise in an audit can be both with regard to the time of the audit, that is the
selection of the date at which the auditor visits the clients' office to carry out the audit and the
selection of the items which are subjected to audit.

Surprise checks are a useful method of determining whether or not such errors exist and where
they exist, of bringing the matter promptly to the attention of the management so that
corrective action is taken immediately. Consequently, surprise visits by the auditor can exercise
a good moral check on the client's staff.

b) An Auditor's report is considered to be modified if either an emphasis of matter paragraph(s)


added to the report or if the opinion is other than unqualified. It includes matters that do not

21
Suggested Answers/CAP-II/December/2009/Group-I

affect the auditor's opinion by emphasis of matter or matters that do affect the auditor's opinion
by issuing qualified opinion, disclaimer of opinion or adverse opinion.

Uniformity in the form and content of each type of modified report will enhance the user's
understanding of such reports.

In certain circumstance, an auditor's report may be modified by adding an emphasis of matter


paragraph to highlight a matter affecting the financial statements which is included in a note to
the financial statements that more extensively discusses the matter. The addition of such an
emphasis of matter paragraph does not affect the auditor's opinion. The paragraph would
preferably be included preceding the opinion paragraph and would ordinarily refer to the fact
that the auditor's opinion is not qualified in this respect.

An auditor may not be able to express an unqualified opinion when either of the following
circumstances exists and, in the auditor's judgment, the effect of the matter is or may be
material to the financial statements:

 there is a limitation on the scope of the auditor's work; or


 there is a disagreement with management regarding the acceptability of the accounting
policies selected, the method of their application or the adequacy of financial statement
disclosures.

The circumstances described in (a) could lead to a qualified opinion or a disclaimer of opinion.
The circumstances described in (b) could lead to a qualified opinion or an adverse opinion.

c) It is a graphic presentation of each part of the company's system of internal control. A flow chart
is considered to be the most concise way of recording the auditor's review of the system. It
minimizes the amount of narrative explanation and thereby achieves a consideration or
presentation not possible in any other form. It gives bird's eye view of the system and the flow
of transactions and integration and in documentation, flaws in the system can be easily spotted
and improvements can be suggested.

It is also necessary for the auditor to study the significant features of the business carried on by
the concern; the nature of its activities and various channels of goods and materials as well as
cash, both inward and outward; and also a comprehensive study of the entire process of
manufacturing, trading and administration. The flow chart facilitates auditor to understand and
evaluate the internal controls in the correct perspective.

7. Explain the difference between (Any TWO): (2 5=10)


a) Clean Audit Report and Qualified Audit Report.
b) Continuous and Final Audit
c) Internal Control and Internal Check

Answer:

22
Suggested Answers/CAP-II/December/2009/Group-I

a) Clean Audit Report and Qualified Audit Report: A clean report which is otherwise known as
unconditional opinion is issued by the auditor when he does not have any reservation with
regard to the matters contained in the financial statements. In such a case, the audit report
may state that the financial statements give a true and fair view of the state of affairs and
profit and loss account for the period. Under the following circumstances an auditor is
justified in issuing a clean report:
i) the financial information has been prepared using acceptable ac counting policies,
which have been consistently applied;
ii) the financial information complies with relevant regulations and statutory
requirements; and
iii) there is adequate disclosure of all material matters relevant to the proper
presentation of the financial information, subject to statutory requirements,
where applicable.
Qualified audit report, on the other hand, is one which does not give a clear cut about the
truth and fairness of the financial statements but makes certain reservations.
The gravity of such reservations will vary depending upon the circumstances. In majority of
cases, items which are the subject matter of qualification are not so material as to affect the
truth and fairness of the whole accounts but merely creates uncertainty about a particular
item. In such cases, it is possible for the auditors to report that in their opinion but subject
to specific qualifications mentioned, the accounts present a true and fair view.
Thus, an auditor may give his particular objection or reservation in the audit report and
state "subject to the above, we report that balance sheet shows a true and fair view……..".
The auditor must clearly express the nature of qualification in the report. The auditor
should also give reasons for qualification and all qualifications should be contained in the
auditor's report.
The words "subject to" are essential to state any qualification. It is also necessary that the
auditors should quantify, wherever possible the effect of these qualifications on the
financial statements in clear and unambiguous manner if the same is material and state
aggregate impact of qualifications.
Thus, it is clear from the above that in case of a clean report, the auditor has no reservation
in respect of various matters contained in the financial statements but a qualified report
may involve certain matters involving difference of opinion between the auditor and the
management.

b) Final Audit is commonly understood to be an audit which does not begin until the books
have closed at the end of the accounting period and thereafter is carried on continuously
until completed. Whether an audit ought to be conducted continuously after the close of the
financial year should be decided on a consideration of the size of the business and the
extent of detailed checking required.
Continuous Audit is one in which the auditor's staff is engaged continuously in checking the
accounts of the client the whole year round or when for this purpose the staff attends at
intervals, fixed or otherwise, during the currency of the financial period. Strictly speaking, when
auditor's staff attends the audit work at fixed intervals it may be strictly called interim audit. This
is when an audit is conducted up to a particular date within the accounting period. The auditor
may attend to audit the figures for a month or for a quarter, as the work may require. It would
differ distinctly from the final audit in the extent of the work carried out; verification of assets,

23
Suggested Answers/CAP-II/December/2009/Group-I

for example would be left until the final audit. In case of a continuous audit, the work is
conducted throughout the course of the financial year but is not taken to a specific accounting
period, as is an interim audit. It might be that during the course of the continuous work interim
figures are being audited, but the significant factor here is that the auditor will be engaged
continuously on the audit throughout the financial period. Staff may be in residence throughout
the period or may come and go at irregular intervals, but most of the time, the audit staff is
present at the location. Thus, in case of continuous audit, the audit staff is present at the client's
premises almost during the entire accounting period.

c) Internal Control comprises whole system of controls, financial or otherwise establish ed by


the management for the conduct of business. It includes internal check and internal audit
besides other controls. But internal check refers to a system of allocating duties among the
staff in such a manner that every person records a different aspect of a transaction. It is
narrower in scope.

A system of internal control strives to achieve objectives such as adherence to policies and
procedures laid down by management, safeguarding of assets, prevention and detection of
frauds and errors, accuracy and completeness of records and timely preparation of reliable
financial information. On the other hand, internal check is designed to prevent frauds and errors
and fixing responsibility and safeguarding the assets. It is a part of Internal Control system.

Internal Control system is reviewed occasionally by the management in the light of changes
within the organization, in the economic environment and suggestions of external and internal
auditor. But Internal Check once introduced in the organization is generally stable for a certain
period and, hence, less flexible as compared to the internal control system.

24
Suggested Answers/CAP-II/December/2009/Group-I

Corporate & other Laws


All questions are compulsory.

Part: 'A'
1. Answer the following questions:

a) ABC Ltd. having total of 10,000 fully paid equity shares of Rs. 100 each called the general
meeting of its members to pass a special resolution on 22 October 2009. The notice for
the general meeting was posted on 5 October 2009. One member holding 400 shares
wishes to challenge the resolution passed at the meeting on the ground that the notice
was not valid as per provisions of Section 171 of the Companies Act, 1956 as 96 percent
of the shareholders had provided their consent to the meeting only after 22 October
2009.

i) State the provisions of Section 171 of the Companies Act, 1956 regarding length of
notice for calling a general meeting? 4
ii) What would be your advice in the above circumstances? 5

b) Explain the principle of "Lifting the corporate veil". 6

Answer:
a)
i) Section 171 of the Companies Act, 1956 provides the following provisions regarding
the length of notice.
(1) A general meeting of a company may be called by giving not less than twenty-
one days' notice in writing.
(2) A general meeting may be called after giving shorter notice than that specified in
sub-section (1), if consent is accorded thereto –
(i) in the case of an annual general meeting, by all the members entitled to
vote thereat ; and
(ii) in the case of any other meeting, by members of the company (a) holding, if
the company has a share capital, not less than 95 percent of such part of the
paid up share capital of the company as gives a right to vote at the meeting,
or (b) having, if the company has no share capital, not less than 95 per cent
of the total voting power exercisable at the meeting:
Provided that where any members of a company are entitled to vote only
on some resolution or resolutions to be moved at a meeting and not on the
others, those members shall be taken into account for the purposes of this
sub-section in respect of the former resolution or resolutions and not in
respect of the latter.

ii) The question does not specify whether the meeting is an AGM or any other General
Meeting hence separate advice shall have to be given for both the circumstances.

In case of an AGM, as provided in clause (i) of sub-section (2) of section 171 of the
Companies Act 1956 requires consent of all the members entitled to vote thereat

25
Suggested Answers/CAP-II/December/2009/Group-I

whereas in the mentioned case only 96 percent of the shareholders had provided
their consent which is not adequate to comply with the requirement of consent of
100 percent of its members entitled to vote, hence the resolution passed at the
meeting is challengeable.

In case of a General Meeting other than an AGM,


Even though consent of shareholders for a shorter notice for conducting a meeting
at which a special resolution is passed, is not obtained prior to the meeting, consent
obtained thereafter would validate the resolution as it has been decided in various
English decisions under the English Companies Act, the provisions of which are
similar to the provisions of clause (ii) of sub-section (2) of section171. The only
Indian case, Self Help Private Industrial Estate (P) Ltd. In re (1972) 42 Comp. Cas 6
(Mad.) is in favour of the view that a post consent validates a special resolution
passed without proper notice.

Additional related case:


Where majority of members of the company holding more than 95 percent of such
part of the paid-up share capital which give them a right to vote at a meeting, had
given their consent, subsequent to the meeting, to a shorter notice and had ratified
and accepted the special resolutions passed at the meeting and the company had
stated on affidavit that not a single objection was received from any of the other
members, it was held that in view of the subsequent consent obtained by the
company from its members who formed a majority and held more than 95 percent
of the paid up share capital which gave them a right to vote, the resolution must be
deemed to be valid [Parikh Engg. & Body Building Co. Ltd. In re (1975) 45 Comp.Cas
157 (Pat)
Considering the interpretation in the above mentioned cases read together with
provisions of clause (ii) of sub-section (2) of section 171 mentioned above it will not
be wise to challenge the resolution as the court has not allowed the contention in
similar cases hence the advice has to be in the negative.

b)
A company is a legal person distinct from its members and as a result members or
shareholders cannot be liable for the acts of the company. This principle has been
accepted in famous case Salomon vs Salomon and Co. (1897). However, when promoters
in their ingenuity, use this veil of corporate personality as a cloak for fraud or improper
conduct, it becomes necessary for courts to break through or lift the corporate veil and
look at the persons behind the company who are the real beneficiaries of the corporate
function. In other words 'lifting the corporate veil' means disregarding the corporate entity
and paying regard, instead, to the realities behind the legal façade. Where the courts
ignore the company and concern themselves directly with the members or managers, the
corporate veil may be said to have been lifted.
2.
a) What do you understand by the term 'Inchoate Instrument'? Examine the validity of such
instruments under the provisions of the Negotiable Instruments Act, 1881. 5
b) Who is an 'Expert'? When is an expert not liable for the mis-statement in the prospectus
of a public company as per the provisions of the Companies Act, 1956? 5

26
Suggested Answers/CAP-II/December/2009/Group-I

Answer
a)
Section 20: Inchoate Stamped Instruments. Where one person signs and delivers to
another a paper stamped in accordance with law relating to negotiable instruments then
in force in India and either wholly blank or having written thereon an incomplete
negotiable instrument, he thereby gives prima facie authority to the holder thereof to
make or complete, as the case may be, upon it a negotiable instrument, for any amount
specified therein and not exceeding the amount covered by the stamp. The person so
signing shall be liable upon such instrument in the capacity in which he signed the same,
to any holder in due course for such amount: provided that no person other than a
holder in due course shall recover from the person delivering the instrument anything in
excess of the amount intended to be paid by him there under.

The principle of this rule (namely that a person who gives another possession to his
signature on a blank stamped paper, prima facie authorizes the latter as his agent to fill
it up and give to the world the instrument as accepted by him) is one of estoppels. By
such signature he binds himself as drawer, maker, acceptor or endorser. His signature on
the blank paper purports to be an authority to the holder to fill up the blank, and
complete the paper as a negotiable instrument. Till this filling in and completion, the
instrument is not a valid negotiable instrument, and no action is maintainable on it.
Further, as a condition of liability, the signer as a maker, drawer, endorser or acceptor
must deliver the instrument to another. In the absence of delivery, the signer is not
liable. Furthermore, the paper so signed and delivered must be stamped in accordance
with the law prevalent at the time of signing and on delivering otherwise the signer is
not stopped from showing that the instrument was filled without his authority.
b)
Section 59(2): The expression "expert" includes an engineer, a valuer, an accountant
and any other person whose profession gives authority to a statement made by him in
the prospectus.

The report of an expert cannot be included in a prospectus if he is in any way connected


with the formation or promotion or management of the company.

When an expert is not liable [Section 62(3)] : An expert who would be liable by reason of
having given his consent under Section 58 to the issue of the prospectus containing a
statement made by him shall not be so liable if he can prove:
(a) that, having given his consent under section 58 to the issue of the prospectus, he
withdrew it in writing before delivery of a copy of the prospectus for registration;
(b) that, after delivery of a copy of the prospectus for registration and before allotment
thereunder, he, on becoming aware of the untrue statement, withdrew his consent
in writing and gave reasonable public notice of the withdrawal and of the reason
therefore; or

27
Suggested Answers/CAP-II/December/2009/Group-I

(c) that he was competent to make the statement and that he had reasonable ground to
believe, and did up to the time of the allotment of the shares or debentures, believe,
that the statement was true.

Part: 'B'
3.
a) Mr. Nabraj Khania has purchased 49 percent shares of XYZ Pvt. Ltd on October 22, 2009
from Mr. Dixit soon after commencement of business on October 1, 2009. On November
1, 2009 the company went into liquidation. The creditors of the company had provided
services worth Rs. 5 million prior to incorporation of the Company on the basis of a
consensus agreement of the promoters. The consensus agreement has not been ratified
by the company after incorporation although Mr. Khania was a witness to the consensus
agreement. Now the creditors claim that Mr. Khania is also liable along with other
promoters. However, Mr. Khania claims that the remaining promoters along with Mr.
Dixit are liable since he was not a party to the consensus agreement.
Advise Mr. Khania in the light of the relevant provisions of the Companies Act, 2063 on
the above issue and the consequences. 9
b) Mr. Smith, citizen of England and a renowned industrialist, desires to invest in an
industrial enterprise in Nepal. He asked you about the facilities and concession that a
foreign investor can get under existing Nepalese Laws. Advise him on the basis of the
provisions of the Foreign Investment and Technology Transfer Act, 2049.
6
Answer:
a)
The above issue has to be observed in the light of Section 17 regarding Pre- incorporation
contract and Section 145 (3) regarding consensus agreement of the Companies Act 2063
which are as follows:

Section 17 - Pre-incorporation contract:


(1) A contract made prior to the incorporation of a company shall be a proposed
contract only, and such contract shall not be binding on the company.
(2) If, prior to the incorporation of a company, any person carries on any
transaction or borrows moneys on behalf of the company, such person shall be
personally liable for any contract related with the transaction so carried on,
subject to subsection (3).
(3) If, within the time mentioned in any transaction or within the reasonable time
after the incorporation of a company, the company, through its act, action or
conduct, accepts any act, action or borrowing done or made prior to the date of
authorization to commence its transactions or endorses such act or action, that
transaction shall be binding on the company and the other contracting party;
and the person carrying out such act or action shall be released from the
personal liability to be borne pursuant to sub-section (2).
(4) Notwithstanding anything contained elsewhere in this Section, the consensus
agreement of a private company shall govern any contracts made prior to the
incorporation of such company.

Section 145 (3):

28
Suggested Answers/CAP-II/December/2009/Group-I

The shareholders who, after the conclusion of a consensus agreement, have obtained
the shares as follows shall be deemed to have consented to the agreement and become
party thereof:
(a) Where the shares have been obtained by way of donation or gift;
(b) Where the shares have been obtained in any other manner, with the knowledge of
the existence of such agreement at the time of obtaining the shares.

Considering the provisions of Section 17 and Section 145 (3) mentioned above Mr.
Khania would be liable along with other promoters instead of Mr. Dixit. Mr Khania is
deemed to have knowledge of existence of such agreement as per Section 145 (3) (b) as
he was witness to the consensus agreement when Mr. Dixit had entered alongwith other
promoters hence his liability shall prevail.
b)
Section 5 of Foreign Investment and Technology Transfer Act, 2049 states about the
facilities and concessions to foreign investors. According to that provision, foreign
investors can get these facilities and concessions:
i) A foreign investor shall be levied tax only at a rate of 15 percent on the income
received for the interest on foreign loan, royalty, technical and management
service fees.
ii) A foreign investor making investment in foreign currency shall be entitled to
repatriate the following amount outside Nepal : a) the amount received by the sale of
the share of foreign investment as a whole or any part thereof, (b) the amount
received as profit or dividend in lieu of the foreign investment ,(c) the amount
received as the payment of the principal of, and interest on , any foreign loan.
iii) A foreign investor shall be entitled to repatriate outside Nepal the amount
received under an agreement for the transfer of technology in such currency as set
forth in the concerned agreement.

4.
a) Explain about the Holding and Subsidiary Company as per the Companies Act, 2063. 5
b) What is Security Exchange? Point out the functions, duties and powers of Security
Exchange pursuant to the Securities Act, 2063. 5
Answer
a)
A Company, which controls another company is called a holding Company. According
to section 2(d) of Nepalese companies Act, 2063 'Holding company means a
company having control over subsidiary company '. From this definition it is clear
that a company which controls any company by holding the management of board of
directors or holding more than half of shares of company is a holding company.
Similarly, section 2(e) of Nepalese companies Act states that 'subsidiary company
means a company controlled by a holding company ' . It means principal company
either holding control over the composition of he Board of Directors or taking
majority of its shares such company can control subsidiary company .
b)
Pursuant to section 2(cha) of the Securities Act, 2063 of Nepal "Securities" means
shares, stocks, bonds, debenture stocks or certificate of Collective Investment
Schemes issues by corporate bodies or any such security(ies) issued by the
government or by the corporate bodies under government guarantees or any other
securities which are prescribed by the Securities Exchange Board of Nepal or

29
Suggested Answers/CAP-II/December/2009/Group-I

negotiable or exchangeable, or and also any other negotiable instruments which


could be transacted or negotiated through Security exchange.
"Security Exchange" is a market-place or facility for the organized purchase and sale of
securities. Hence, a group of people who organize themselves to create a market place
is security exchange or stock exchange.
The functions, duties and powers of Security Exchange which are conferred by section
45 of Security Act, 2063 are as follows:
i) to operate or cause to operate security transactions in a transparent and fair manner
and on a regular basis;
ii) to operate or cause to operate security exchange giving important consideration to
the interests of general investors;
iii) to comply or cause to comply the Securities Act and the rules or by-rules made under
this Act, and to monitor and supervise or to cause monitoring and supervision of the
compliance of it;
iv) to arrange for an adequate and facilitative market place, competent staff and
appropriate internal security arrangement for the smooth running of security
exchange business;
v) to devise by-rules required for security transactions, listing of securities and
conferment of membership under the consent of the Securities Board; and
vi) to perform or cause to perform any other acts as are necessary.

5.
a) XYZ Inc. a company incorporated in the USA and involved in liquor business wants to set
up a liquor factory in Nepal for export as well as domestic sale and in the process intends
to obtain permission for its foreign investment:
i) State the provisions as provided in the Schedule to the Foreign Investment and
Technology Transfer Act, 2049 with respect to restriction on Foreign Investments. 3
ii) Advice whether it will be possible to attract the foreign investment. 2
b) What do you mean by Insider Trading of securities? What are the punishments to be
imposed on such person who engaged in insider trading under Securities Act, 2063? 5
Answer
a)
(i) Industries in which Foreign Investments shall not be permitted as per FITTA 2049:
Clause (a)
Cottage Industries, Individual Service enterprises (such as haircutting, beauty
parlors, tailoring and driving training), Arms and Ammunition Industries, Bullets and
Gunpowder Radioactive Materials Industries Sale and Purchase of Houses and Lands
(except construction industries) Motion Picture Industries (state and national
languages), Security printing and Currency & Coinage

Clause (b)
Retail Business, travel agency, trekking agency, water rafting, Pony Trekking, Horse-
Riding, Cigarette, Bidi and Liquor (except those exporting 90 percent of their
products), Internal Courier Service, Atomic
Energy, Tourist Lodges, Poultry Farming, Fish Farming, Bee-Keeping, Consultancy
Service Business such as Management, Accounting, Engineering and Legal Service

30
Suggested Answers/CAP-II/December/2009/Group-I

business, Beauty Parlors, Cereal beating, Grinding and Oil extracting on rental, Local
Catering Service, Rural Tourism.

(ii) Schedule to the FITTA 2049 restricts foreign investment in the liquor industry
except those exporting 90 percent of their products hence if XYZ Inc. intends to
export 90 percent of its products it may apply for foreign investment permission
as per FITTA 2049 else it is not advisable.
b)
Section 91 of Securities Act, 2063 states that 'in case any person conducts or causes
others to conduct transactions in securities on the basis of any inside information or
report which has not been made public and which can have an impact on the value of
the securities, or supplies any such information or report possessed by him to any
other person, except in the course of discharging his duties, he shall be deemed to
have carried out an insider trading of securities.' Moreover, the punishment to be
imposed on the person who is engaged in such business is mentioned in section 101
of Securities Act. According to that provision ' any insider who deals in securities in
contravention of section 91 is proved to have done so, he shall be punished with a
fine equal to the amount involved in the offense, or with imprisonment for a term not
exceeding one year, or with both .

6.
a) Enumerate the activities prohibited for being carried out by licensed institutions by the
Banks and Financial Institutions Act, 2063. 9
b) Mr. Shyam is an employee of ABC Ltd. A dispute arises between Mr. Shyam and Mr.
Ram Bahadur, General Manager of the Company on the issue of payment of bonus. You
are asked to suggest the procedure of settlement of the dispute under the existing laws. 6

Answer
a)
Section 48 - Activities prohibited for being carried out by licensed institutions are as
follows:
(1) No licensed institution shall carry out, or cause to be carried out, the following
activities:
(a) Purchasing or selling goods for commercial purpose, or purchasing any immovable
property except when it is required for its own use;
Provided that this clause shall not be deemed to prevent the licensed institutions of
Classes “B” and “D” from purchasing, selling, distributing and managing lands and
buildings in connection with carrying on their business transactions subject to Section
47.
(b) Advancing credit against the security of its own shares;
(c) Allowing credit or facility to any promoters, directors, persons who have subscribed
one per cent or more of its shares, chief executive or any family member of such persons
or to any firms, companies or institutions which are entitled to nominate or appoint
directors;
(d) Allowing any type of credit or facility to any promoters, directors, persons who have
subscribed one per cent or more of its shares, chief executive or any family member of
such persons or managing agent or any firms, companies or institutions which are

31
Suggested Answers/CAP-II/December/2009/Group-I

entitled to nominate or appoint directors or any firms, companies or institutions in


which the
institution has a financial interest;
(e) Allowing credit or facility in an amount exceeding such percentage of its capital fund
as may be prescribed by the Rastra Bank to a single customer, company and companies
or partnership firms of a single group;
Explanation: The expression "companies or partnership firms of a single group" means a
group of such companies or partnership firms as may be prescribed by the Rastra Bank
from time to time.
(f) Allowing any type of credit to any person, firm, company or institution against the
guarantee given by the promoters, directors or chief executive;
(g) Investment in the securities of those licensed institutions which have obtained
permission from the Rastra Bank to carry on the financial transactions of Classes “A”, “B”
and “C”;
(h) Investment of an amount exceeding the limit prescribed by the Rastra Bank in the
share capital of any other institution;
(i) Indulging with other licensed institutions to mutually create any type of monopoly or
any other type of controlled practice in the financial transactions;
(j) Doing any kind of act which is capable of creating an artificial obstruction in the
competitive environment of the financial sector, with the intention of deriving undue
advantage;
(k) Doing such other acts prohibited from being done by a licensed institution as may be
prescribed by the Rastra Bank.

(2) Notwithstanding anything contained in clauses (c) and (d) of subsection (1) or
elsewhere in this Act, nothing shall be deemed to bar the lending activities against loan-
bonds or fixed deposit receipts issued by the Government of Nepal or the Rastra Bank or
the providing of credit facility or advance as per the provisions made under the
employee facilities scheme to any promoter, director, chief executive or shareholder
holding more than one percent of its shares in cases if such chief executive or
shareholder is an employee at such licensed institution.

(3) No licensed institution shall, for the purpose of rescuing any person, from legal
action, who deposits funds to an account earned in an illegal manner, provide any
kind of assistance to such person to conceal, convert, withdraw or transfer such
funds or to assist such person to conceal or lie about the origin or source thereof or
provide any kind of assistance for the purpose. If any licensed institution receives
information about the commission of any such act or suspects that any such act has
been committed, it shall forthwith provide particulars thereof to the Rastra Bank or
to any other authority responsible for controlling money laundering.
b)
Section 16 of Bonus Act 2030 contains the following provision regarding the settlement
of any disputes occurring between employee and management. The provisions of
section 16 are as follows:
i) In case of any dispute arises between the employee and the manager on the issue
of bonus to be paid under this Act, labour office shall summon both sides and
resolves the dispute through dialogue .

32
Suggested Answers/CAP-II/December/2009/Group-I

ii) In case the dispute fails to be resolved through the dialogue, the labour office
may direct the concerned establishment and the employees to submit necessary
documents as well as accounts and records. The office may take a decision on the
basis of these documents.
iii) Any side which is dissatisfied with the decision taken by the labour office may
appeal to the laour court with in 35 days from the date of receipt of a notice
thereof . The decision taken by the laour court shall be final.
7.
a) State the provisions regarding removal of members name from the register maintained
by ICAN as provided in Rule 47 of the Nepal Chartered Accountants Regulations, 2061. 5
b) What do you understand by the term Corporate Body substantially owned by Nepal
Government? Who will audit the accounts of such body? 5
Answer
a)
Rule 47 of Nepal Chartered Accountants Regulations, 2061:
Removal of a member's name from the register:
(1) The council can remove a member's name from the register if any of the following
situations exist:
(a) If the member's membership or his/her certificate of practice or certificate of
membership is decided to be cancelled as a penalization in accordance with
section (b) or (d) of sub-article (5) of article 14 of the act,
(b) If not eligible to be registered according to article 18 of the act,
(c) If failed to pay the fee according to section (b) of sub-article (1) of article 22 of
the act,
(d) If the certificate of membership or certificate of practice of the member is
cancelled according to article 23 of the act, or
(e) If the membership is cancelled according to sub-rule (3) of rule 46.

(2) Removal of the member's name from the register according to sub-rule (1) shall
be effective from the date decided by the council.

(3) The member shall be notified, publicly as well as at his address provided by him
to ICAN, while removing a member's name from the register according to sub-rule
(2).

(4) The Certificate of Practice of the member if he /she has obtained shall be
assumed cancelled routinely after removal of a member's name from the register.
b)
Section 2(e) of Audit Act , 2048 defines the terminology 'corporate body substantially
owned by Nepal government . According to this provision corporate body
substantially owned by Nepal government means a corporate body whose more than
50 percent shares or assets are owned by Nepal government. So far as the audit of the
account of such body is concerned, it is mentioned in the section 7 of the Act.
According to it, the audit of such body shall be done in accordance with the existing
laws relating to such body. For that purpose, the auditor general shall be consulted
while appointing an auditor for auditing of such body. A copy of the report of audit
shall be delivered to auditor general by such body.
8. Answer the followings questions:

33
Suggested Answers/CAP-II/December/2009/Group-I

a) What are the liabilities of a surety as per the provision of Nepalese Contract Act, 2056? 5
b) Distinguish between a Promissory Note and a Bill of Exchange. 5
c) Write short notes on any two of the followings in the point of view of insurance
contract. (2×2.5=5)
i) Subrogation
ii) Excess Clause
iii) Endorsement.

Answer
a)
The liabilities of a surety as provided in Section 16 of the Nepalese Contract Act 2056 are
as follows:
Section 16: Surety's Liability
Except when otherwise provided for in the contract, the surety's liability shall be as
follows:
i) Liability of the surety shall emerge from the very time when the person who has
to meet the liability fails to meet it.
ii) Liability of the surety shall be similar to that of the person who has to repay the
loan or fulfill any obligation. The surety shall remain responsible until the person
becomes free from the liability of repaying the loan or fulfilling the obligation.
iii) The creditor may have the surety met the liability immediately after the default
by the person who is under obligation to repay the loan or fulfill the obligation.
iv) In case both security and guarantee have been provided in consideration of any
loan or liability, the surety will have no liability to the extent covered by the
security so provided.
v) The liability of a surety shall not terminate simply because the person who has
to repay a loan or fulfill a liability becomes free from the liability because of
activation of any law.

Explanation: For the purpose of this Act, the term "creditor" means a person who
has supplied a loan. The term includes a person who may obtain any benefit from or
have any work done by the person who has to repay the loan or fulfill the liability.
b)
The distinctive features between a Promissory note and a bill of exchange are tabulated
below:
Promissory Note Bill of Exchange
1. It contains a promise to pay It contains an order to pay
2. The liability of the maker The liability of the drawer of
of a note is primary and a bill is secondary and
absolute. conditional. He would be
liable if the drawee, after
accepting the bill fails to pay
the money due upon it
provided notice of dishonour
is given to the drawer within
the prescribed time

34
Suggested Answers/CAP-II/December/2009/Group-I

3. It is presented for payment If a bill is payable some time


without any previous after sight, it is required to be
acceptance by maker. accepted either by the drawee
himself or by some one else
on his behalf, before it can be
presented for payment.
4. The maker of a promissory The maker or drawer of an
note stands in immediate accepted bill stands in
relationship with the payee immediate relationship with
and is primarily liable to the the acceptor and the payee.
payee or the holder
5. It cannot be made payable In case of bill, the drawer and
to the maker himself, that is payee or the drawee and the
the maker and the payee payee may be the same
cannot be the same person. person.
6. In case of a promissory not In case of a bill of exchange
there are only two parties, there are three parties, viz.,
viz., the maker (debtor) and drawer, drawee and payee,
the payee (creditor) and any two of these three
capacities can be filled by one
and the same person.
7. A promissory note cannot The bills can be drawn in
be drawn in sets. sets.
8. A promissory note can A bill of exchange too cannot
never be conditional. be drawn conditionally, but it
can be accepted conditionally
with the consent of the
holder.

Answer
c)
i) Subrogation
Subrogation is a principal of insurance in which the insurer first indemnifies the
insured and accordingly steps in the shoes of the insured and in accordance
therewith it shall have the right to recover the claims of insured. It can also file
the law suit in the court of competent jurisdiction in order to recover the claim
of the insured.

ii) Excess Clause


It is a clause in the contract of insurance binding the insured to bear certain part
of the loss it has incurred. The basic notion of this concept is to put certain
responsibility on the part of the insured to exert certain limit of his or her care to
the subject matter of insurance. This clause restricts the insured from being
carefree on the subject matter of insurance.

iii) Endorsement
In an insurance contract, if any additional risk is required to be added, such is
added by way of endorsement and a separate premium shall be charged upon
such endorsement. Once added, the endorsement takes precedence over the

35
Suggested Answers/CAP-II/December/2009/Group-I

original provisions of the policy. For example, under a homeowner's policy an


inflation guard endorsements is used so that property damages limits are
increased automatically to reflect an increase in the cost of construction in the
community.

36
Suggested Answers/CAP-II/December/2009/Group-I

The Institute of Chartered Accountants of Nepal


Suggested Answers of Cost & Management Accounting

Chartered Accountancy Profession II Examination – December 2009

Marks

All questions are compulsory. Working notes should form part of the answer.

Make assumptions wherever necessary.

1. A company manufactures and sells three models of a product. The selling price and cost data are
collected and presented to you for analysis here in below:

Description Model X Model Y Model Z

Unit Selling Price (Rs.) 800 1,200 2,000

Unit Variable Costs:

Direct Materials (Rs.) 160 240 400

Direct Labor (Rs.) 160 320 480

Overhead (Rs.) 80 160 240

Selling (Rs.) 160 160 160

Product Percentage of Total Sales 10% 50% 40%

The following information is also relevant:

 The company incurs advertising cost Rs.8 million, fixed administrative cost Rs.8 million, and fixed
manufacturing cost Rs.16 million in addition.
 The company has the capacity of producing 100,000 units of all models and is currently utilizing its
80% capacity.

You are required to calculate/answer the following: (3+3+4+5+5=20)

a) The profit from the given data.


b) The total BEP sales from the given data.
c) The company is considering increasing the advertising budget by Rs. 8 million to increase the
total unit sales to full capacity. The product mix would remain same. Is the campaign desirable?

d) The company is considering of providing additional sales commission to sales force of each
product at the rate of 2% to increase the total unit sales to 90% of its capacity. The product mix
would remain same. Is the commission desirable?

37
Suggested Answers/CAP-II/December/2009/Group-I

e) The company is considering altering the production process by installing new machine which will
reduce the direct material, direct labor and variable overhead to 75% of their current level and
will increase the fixed manufacturing cost by Rs.16 million. What is the minimum level of total
sales (in units) for which this change would be desirable?

Answer

a. Profit (Rs.) = (Combined Unit CM*Total Sales)-Total Fixed Costs


= (472*80000)-32000000
= 5,760,000
b. Total BEP Sales (Rs.) = (Total Fixed Costs/ Combined PV Ratio)
= (32000000/31.892%)
= 100,338,642.92
say, 100,338,643
c. Desirability of Advertising Campaign
Additional Advertising Cost (Rs.) = Rs. 8,000,000
Total Fixed Costs after additional Advertising Cost = Rs. 40,000,000
Total Sales Unit after Advertising Campaign (Full
Capacity) = 100,000
Profit (Rs.) = (Combined Unit CM*Total Sales)-Total Fixed Costs
= (472*100000)-40000000
= 7,200,000
Since the advertising campaign causes initial profit to increase, the campaign is desirable.

d. Desirability of Sales Commission


Model X Model Y Model Z
Existing Contribution Margin/ Unit (Rs.) 240 320 720
Less: Additional Sales Commission @2% 16 24 40
New CM/ Unit (Rs.) 224 296 680
Sales Mix 10% 50% 40%
New Combined CM/ Unit (Rs.)= (224*10%+296*50%+680*40%)
= 442.40
New Total Sales Unit (90% of Capacity) 90000
Profit (Rs.) = (New Combined Unit CM*New Total Sales)-Total Fixed Costs
= (442.4*90000)-32000000
= 7,816,000
Since additional sales commission causes initial profit to increase, the commission is
desirable.

e. Minimum Level of Sales for which the alteration of production process as indicated is
desirable
Model X Model Y Model Z
Existing Contribution Margin/ Unit (Rs.) 240 320 720
Add: 25% of Unit VC excluding selling 100 180 280
New CM/ Unit (Rs.) 340 500 1000
Sales Mix 10% 50% 40%

38
Suggested Answers/CAP-II/December/2009/Group-I

New Combined CM/ Unit (Rs.)= (340*10%+500*50%+1000*40%)


= 684
New Total Fixed Cost after additional fixed manufacturing cost of Rs. 16 million
= 48,000,000
The required minimum level of sales
=(New Total Fixed Cost+Initial Profit Level)/ New Combined Unit CM
=(48000000+5760000)/684
= 78596 Units
Below the calculated level of minimum required sales, the change is not desirable.
Working Notes:

S. No. Description Model X Model Y Model Z Total


A. Computation of Combined Contribution Margin Per Unit

I. Unit Selling Price (Rs.) 800 1200 2000


Unit Variable Costs:
Direct Materials (Rs.) 160 240 400
Direct Labor (Rs.) 160 320 480
Overhead (Rs.) 80 160 240
Selling (Rs.) 160 160 160
II. Total Variable Cost/ Unit
(Rs.) 560 880 1280
III. Contribution Margin/ Unit
(I-II) (Rs.) 240 320 720
IV. P/V Ratio(III/I*100) 30.00% 26.67% 36.00%
V. Product % of Total Sales 10.00% 50.00% 40.00%
V. Combined CM/Unit (Rs.) (240*10%+320*50%+720*40%) =472
B. Computation of Combined P/V Ratio

VI. Sales Units (V*80,000) 8,000 40,000 32,000 80,000

Contribution Margin
(III*VI and 472 * 80,000
VII. in the case of Total)
1,920,000 12,800,000 23,040,000 37,760,000

VIII. Sales Value (I*VI) 6,400,000 48,000,000 64,000,000 118,400,000

P/V Ratio (VII/VIII * 31.892%


IX. 100) 30.00% 26.67% 36.00%
X. Total Sales Units = 80% of 100,000 = 80,000

XI. Total Fixed Costs (Rs.):


Advertising 8,000,000
Administrative 8,000,000

39
Suggested Answers/CAP-II/December/2009/Group-I

Manufacturing 16,000,000
Grand Total: 32,000,000

2.
a) Kathmandu Limited having a capacity of 6 lakhs units has prepared the following cost sheet:
Per unit
Direct Materials Rs. 2.50
Direct Wages Rs. 1.00
Factory Overheads Rs. 2.00 (50% fixed)
Selling and Administration Overheads Rs. 1.50 (one-third variable)
Selling Price Rs. 9.00
During the year 2008, the sales volume achieved by the company was 5 lakh units. The
company has launched an expansion programme, the details of which are as under:
i) The capacity will be increased to 10 lakhs units.
ii) The additional fixed overheads will amount to Rs. 4 lakhs up to 8 lakhs units and will
increase by Rs. 2 lakhs more beyond 8 lakhs units.
iii) The cost of investment on expansion is Rs. 8 lakhs which is proposed to be financed
through bank borrowings carrying interest at 15% per annum.
iv) The average depreciation rate on the new investment is 10% based on straight line
method.
After the expansion is put through, the company has two alternatives for operating the expanded
plant as under:

i) Sales can be increased up to 8 lakhs units by spending Rs. 1 lakhs on special


advertisement campaign to explore new market, or
ii) Sales can be increased to 10 lakhs units subject to the following:
a. By an overall price reduction of Rs. 1 per unit on all the units sold.
b. By increasing the variable selling and administration expenses by 5%.
c. The direct material costs would go down by 1% due to discount on bulk buying.
You are required to:
(7+3=10
)

i) Construct a flexible budget at the level of 5 lakhs, 8 lakhs and 10 lakhs units of
production and advise which level of output should be chosen for operation.
ii) Calculate the break-even point both before and after expansion.

b) In a factory, a machine is considered to work for 208 hours in a month. It includes maintenance time of
8 hours and set up time of 20 hours.

The expenses data relating to the machine are as under:

40
Suggested Answers/CAP-II/December/2009/Group-I

 Cost of machine is Rs. 500,000. Life 10 years. Estimated scrap value at the end of life is Rs.
20,000.

Rs.
 Repairs and maintenance per annum 60,480
 Consumable stores per annum 47,520
 Rent of building per annum
(The machine under reference occupies 1/6 of the area) 72,000
 Supervisor's salary per month (Common to three machines) 6,000
 Wages of operator per month per machine 2,500
 General lighting charges per month allocated to the machine 1,000
 Power 25 units per hour at Rs. 2 per unit

Power is required for productive purposes only. Set up time, though productive, does not require
power. The Supervisor and Operator are permanent. Repairs and maintenance and consumable stores
vary with the running of the machine.

You are required to calculate a two-tier machine hour rate for (i) set up time, and (ii) running time.
10

Answer (a)
a) Flexible budget:
Rs. in lakh
Output levels (units) 5 lakhs 8 lakhs 10 lakhs
Sales 45.00 72.00 80.00
Direct Materials (@ 2.50 per unit, but at level of 12.50 20.00 24.75
10 lakhs discount of 1% is to be allowed)
Direct wages 5.00 8.00 10.00
Factory Overheads 5.00 8.00 10.00
Selling and Administration Overheads (at the 2.50 4.00 5.25
level of 10 lakh variable Selling and Administra-
tion Overhead Increases by 5%)
Total Variable Costs 25.00 40.00 50.00

Contribution 20.00 32.00 30.00

Fixed Costs:
Fixed factory overhead 6.00 6.00 6.00
Selling and Adm. Expenses 6.00 6.00 6.00
Additional fixed cost due to expansion - 4.00 6.00
Interest @ 15% on 8 lakh - 1.20 1.20
Depreciation @ 10% on 8 lakhs - 0.80 0.80
Special advertisement - 1.00 -

41
Suggested Answers/CAP-II/December/2009/Group-I

Total Fixed Costs 12.00 19.00 20.00


Profit 8.00 13.00 10.00

Therefore, activity to be chosen is 8 lakhs units since it provides the highest profit of Rs. 13
lakh.

b Break-even points:
Output levels (units) 5 lakhs 8 lakhs 10 lakhs
P/V Ratio (Contribution Margin per Unit/Price 4/9 4/9 3/8
per Unit)
Break-even points (Rs. in lakh) 27.00 42.75 53.33
Working [BEP = Fixed Cost/PV Ratio] 12 X 9/4 19 X 9/4 20 X 8/3
BE) ( in lakh of units) 3 lakh 4.75 lakh 6.67 lakh
Working [BEP = Fixed Cost/CM per Unit] 12/4 19/4 20/3

Answer (b)

Working Notes:
(i) Effective hours for standing charges (208 hours – 8 hours) 200
(ii) Effective hours for variable costs (208 hours – 28 hours) 180

1. Standing charges per hour


Per Month Rs. Per Hour Rs.
Supervisor's salary (Rs.6,000/3 machines) 2,000
General Lighting 1,000
Rent (72,000/6) / 12 1,000
Total standing charges 4,000
Standing charges per hour (4,000/2,00) 20.00

2. Machine expenses per hour


Depreciation (Rs.500,000 – Rs. 20,000/

(10 years x 12 months) 4,000 (Rs. 4,000/200 hrs) 20.00


Repair and maintenance (Rs. 60,480/12 months) 5,040 (Rs. 5,040/180 hrs) 28.00
Consumable stores (Rs.47,520/12 months) 3,960 (Rs.3,960/180 hrs) 22.00
Power ( 25 units x Rs.2 x 180 hrs) 9,000 (Rs. 9,000/180 hrs) 50.00
Wages 2,500 (Rs.2,500/200 hrs) 12.50
Total machine expenses 24,500 132.50

42
Suggested Answers/CAP-II/December/2009/Group-I

Computation of two-tier Machine Hour Rate:


Setup time rate Running time rate

Per machine hour per machine hour


Rs. Rs.

Standing charges (Refer to working note 1) 20.00 20.00


Machine expenses (Refer to working Note 2)
Depreciation 20.00 20.00
Repair and maintenance - 28.00
Consumable stores - 22.00
Power - 50.00
Machine hour rate of overheads 40.00 140.00
Wages 12.50 12.50
Comprehensive machine hour rate 52.50 152.50

3.

a) Raw materials 'X' costing Rs. 100 per kilogram and 'Y' costing Rs.60 per kilogram are mixed in
equal proportion for making product 'A'. The loss of materials in processing works out to
25% of the output. The production expenses are allocated at 50% of direct material cost.

The end product is priced with a margin of % over the total cost. Material Y is not easily
available and substitute raw material 'Z' has been found for 'Y' costing Rs. 50 per kilogram. It
is required to keep the proportion of this substitute material in the mixture as low as
possible and at the same time maintain the selling price of the end product at the existing
level and ensure the same quantum of profit as at present.

You are required to compute what should be the ratio of mix of the raw materials X and Z. 7

b) A company has pressure to increase its production by 25% to meet the increased demand.
To increase the productivity of its 10 workers, the company has given assurance to them of
average 20% increase in their pay through introduction of some incentive scheme; either
Halsey Scheme with 50% bonus or Rowan Scheme.
The company observes the following figures for this month after the assurance:
Guaranteed Hourly Wage Rate Rs. 25
Time Allowed for producing a unit 2 Hours

(Calculated on the basis of previous performance)

43
Suggested Answers/CAP-II/December/2009/Group-I

Number of working hours per day 8 Hours


Number of working days in this month 25 Days
Actual production during this month 1,250 Units

You are required to calculate: (4+4=8)

i) Effective rate of earnings per hour under Halsey Scheme and Rowan Scheme.
ii) Savings to the company in terms of direct labour cost per unit under each
scheme.

Answer a)
i. Output of Product A (assumed) 1.00
Process loss 25% of output 0.25

Input of raw material X and Y 1.25

X and Y are mixed in equal proportion for making product A, therefore 0.625 kg of X and 0.625 kg. of Y will
be mixed to produce on kg of product A.

ii. Statement of Cost and Profit for producing one kg. of A:


Rs.
Raw material X (0.625 x Rs.100) 62.50

Raw material Y (0.625 x Rs.60) 37.50

Material cost 100.00

Add Production Exp. At 50% of material cost 50.00

Total cost 150.00

Add Profit at 33.33% over total cost 50.00

Selling price 200.00

iii. Ratio of mix of raw material X and Z:


Minimum quantity of Z is to be mixed and maintain the same selling price and same amount of
profit. For this purpose, total material cost in one kg of product A should be the same i.e. Rs. 100
only. Cost of Z is Rs.50 per kg. Suppose quantity of Z raw material is Z.

Z x Rs. 50 + (1.25 – Z)kg x Rs. 100 = Rs. 100


50Z + 125 – 100Z = 100
50Z – 100Z =100 – 125
-50Z = -25
Z = 0.5
X will be 1.25 -0.5 kg i.e. 0.75 kg
Ratio will be 3:2 between X and Z (i.e. 0.75 kg and 5 kg). It can be proved as below:

44
Suggested Answers/CAP-II/December/2009/Group-I

Statement of Cost
Rs.
Raw material X 0.75 kg at Rs. 100 75
Raw material Z 0.50 kg at Rs. 50 25
Total material cost 100
Production expenses 50% 50
Total cost 150
Profit 33.33% of total cost 50
Selling price 200

Answer b)
i. Calculation of Effective Hourly Rate of Earning

Under Halsey Scheme

= (Total Time Wages of 10 workers+Total Bonus)/ Total Actual hours worked

= (50000+6250)/2000 Rs. 28.125

Under Rowan Scheme

= (Total Time Wages of 10 workers+Total Bonus)/ Total Actual hours worked

=(50000+10000)/2000 = Rs. 30

ii. Calculation of Saving in terms of direct labour cost per unit

Direct Labour Cost per unit

Under Time Wage= 2 hours * Rs.25 = Rs. 50

Under Halsey=(50000+6250)/1250 = Rs. 45

Under Rowan=(50000+10000)/1250 = Rs. 48

Saving Under Halsey Scheme=50-45 = Rs. 5

Saving Under Rowan Scheme=50-48 = Rs. 2

Working Notes:

A. Total time wages of 10 workers per month

=Working days in month*Working hours per day*Hourly wage rate*No. of workers

45
Suggested Answers/CAP-II/December/2009/Group-I

=25*8*25*10 = Rs. 50000

Time saved per month


B) Time allowed per piece per worker 2 hrs
Actual Unit produced by 10 workers in the month 1250 Units
Total Time Allowed to produce 1250 units (1250*2) 2500 hrs
Actual Time Taken (25 days*8 hrs*10 workers) 2000 hrs
Time saved per month (2500-2000) 500 hrs

C.
Total Bonus under Halsey Scheme

= (50% of time saved)*Hourly wage rate

=500*0.50*25 = Rs. 6250


Bonus Percentage= 12.50%

D.
Total Bonus under Rowan Scheme

= (Time Saved/ Time Allowed)* Time Wages

= (500/2500)*50000 = Rs. 10000


Bonus Percentage= 20.00%

4.

a) Kathmandu Chemical Company Limited carries on production operation in two processes.


Material first passed through process I, where a compound is produced. A loss in weight takes
place at the start of processing. The following data which can be assumed to be representative,
relates to the month just ended:
Quantities: Kg.
Material input 200,000
Opening work-in-process (half processed) 40,000
Work completed 160,000
Closing work-in-process (two third completed) 30,000
Costs: Rs.
Material input 75,000
Processing cost 102,000
Opening work-in-process:

46
Suggested Answers/CAP-II/December/2009/Group-I

Materials 20,000
Processing costs 12,000

Normal process loss in quantity may be assumed to be 20% of material input. Any quantity of
the compound can be sold for Rs. 1.60 per kg. Alternatively, it can be transferred to process II
for further processing and packing to be sold as supercomp for Rs. 2.00 per kg. Further
materials are added in process II which yield 2 kgs of supercomp for every kg of the process I
compound used.
Of the 160,000 kgs per month of work completed in process I, 40,000 kgs are sold as
compound and 120,000 kgs passed through process II for sale as supercomp. Process II has
facilities to handle up to 160,000 kgs of compound per month, if required. The monthly cost
incurred in process II (other than the cost of the compound) are:

120,000 kgs of compound inputs 160,000 kgs of compound inputs


Rs. Rs.
Materials 120,000 160,000
Processing cost 120,000 140,000
You are required to: (6+4=10)
i) Determine using the average cost method, the cost per kg of compound in process I and
the value of both completed and closing work in process for the month just ended.
ii) Advise whether it is worthwhile to process 120,000 kgs of compound further?
b) Explain the advantages and disadvantages of integral accounting. 5

Answer a)
i. Process I
Statement of Equivalent Production

Equivalent units
Input Output
Material Conversion cost

Units (kg) Particulars Units (kg) % %


Particulars Unit Unit
Completion Completion

Opening 40,000 Normal loss 40,000 - -


WIP

New 200,000 Units Intro- 160,000 160,000 100% 160,000 100%


material duced and
introduced completed

Abnormal 10,000 10,000 100% 10,000 100%


loss

Closing WIP 30,000 30,000 100% 20,000 2/3rd

47
Suggested Answers/CAP-II/December/2009/Group-I

240,000 240,000 200,000 190,000

Process I
Statement of Cost for Each Element

Elements of Cost of Cost in Equivalent Cost per Kg.


Total Cost
Cost Opening WIP Process units (kg) (Rs.)

Material Rs. 20,000 Rs. 75,000 Rs. 95,000 200,000 0.475

Conversion 12,000 102,000 114,000 190,000 0.600


Cost

32,000 177,000 209,000 1.075

Statement of apportionment of cost

Units completed Elements Equivalent Cost per Cost Total


units (kg) unit (Rs.) cost

Work completed Material 160,000 0.475 76,000

Conversion cost 160,000 0.600 96,000 172,000

Value of WIP completed Material 30,000 0.475 14,250

Conversion 20,000 0.600 12,000 26,250

Note: The work of abnormal loss can also be found out like “work completed” but it has not been asked in
the question.

ii. There can be two approaches to this problem:

Approach I – Total Cost and Revenue approach

Statement showing comparative data to decide whether 120,000 kg of compound


should be processed further

Alternative I Alternative II
Sell immediately after process I Process further
Sales 120,000 X Rs. 192,000 Sales 240,000 X Rs. 2 480,000

48
Suggested Answers/CAP-II/December/2009/Group-I

Rs. 1.60
Less: Cost from 129,000
process I 120,000 129,000 Cost from process I 120,000
X Rs. 1.075 X Rs. 1.075

Material in process II 120,000


Processing cost in 120,000 369,000
process II
63,000 111,000

Conclusion: The company should take the decision to process further. It will increase the profit
by Rs. 48,000 (111,000 – 63,000).

Approach II – Incremental Cost and Incremental Revenue

Incremental revenue:
Sales 120,000 X Rs. 2 Rs. 240,000
120,000 X (2 – 1.60) Rs. 48,000 Rs. 288,000

Incremental cost:
Material in process II Rs. 120,000
Processing costs in process II Rs. 120,000 Rs. 240,000
Rs. 48,000
Conclusion: Since there is an incremental profit of Rs. 48,000 (288,000 – 240,000) due to the
decision to process, the company should opt for the processing of the compound.

Answer b)
Integral accounting implies a system of accounting where both cost and financial accounts are maintained in
one set of books. The system is designed in a way so as to provide full information required for costing as
well as financial purposes. In other words, the system provides information for ascertaining the cost of each
product, job or process besides making possible ascertainment of marginal cost, variances, abnormal losses
and gains, etc. It also makes possible fur the management to have accounts to see that the business
maintains full control over its assets and liabilities and prepares the profit and loss account and the balance
sheet as per requirement of law.

Advantages:
The main advantages which accrue on account of adoption of this system are as follows:
i. Duplication of works and keeping unnecessary accounts such as cost ledger control account
and general ledger adjustment account, purchase account, as well as stores ledger control
account is eliminated. This saves both time and expenditure.
ii. The system ensures inclusion of all legitimate expenditure in cost accounts and serves as an
automatic check on the correctness of the cost data. Correct and reliable cost data creates
confidence in the management.

iii. The need for reconciling the profits as shown by cost and financial books does not at all arise
as there will be one figure for profit or loss.
iv. The system ensures better coordination between cost and financial staff. The fact that cost
accounting and financial accounting are complementary to each other is better appreciated.

49
Suggested Answers/CAP-II/December/2009/Group-I

v. The system is particularly helpful under mechanized system of accounting.

Disadvantages:
The system suffers from the following disadvantages:

i. The system causes delay in providing information. Since the system is expected to serve both
the costing and the financial requirement, it gets complicated.
ii. The need for preparing a reconciliation statement may still be felt since generally 100%
integration is not possible.

iii. The system does not suit large concerns which require detailed cost and financial information
on a continuing basis.

5.

a) National Limited is engaged in the manufacture of two product A and B. Product A uses one
unit of component P and two units of component Q. Product B uses two units of component P
and one unit of Q and two units of component R. Component R which is assembled in the
factory uses one unit of component Q. Components P and Q are purchased from the market.

The firm has prepared the following forecast of sales and inventory for the next year.
Products
A B

Sales Units 8,000 15,000


Inventories:
At the end of the year Units 1,000 2,000
At the beginning of the year Units 3,000 5,000

The production of both the products and the assembling of the component R will be spread out
uniformly throughout the year.

The firm at present orders its inventory of components P and Q in quantities equivalent to 3 months
consumption. The firm has been advised that savings in the provisioning of components can arise by
changing over to the ordering system based on economic ordering quantities. The firm has compiled
the following data relating to two components.

P Q

Component usage per annum 30,000 48,000


Price per unit Rs. 2.00 0.80
Order placing costs per order Rs. 15.00 5.00
Carrying costs per annum (%) 20 20

50
Suggested Answers/CAP-II/December/2009/Group-I

You are required to : (4+2+4=10)

(i) Prepare a budget of production and requirements of components for the next year.

(ii) Find the economic order quantity.


(iii) Based on the economic order quantity calculated in (ii) above, calculate the savings
arising from switching over to the new ordering system both in terms of cost and
reduction in working capital.
b) Briefly explain the objectives of introducing uniform costing. 5

Answer 5 (a)

i. Statement showing production budget of products A and B.


In units
Product A Product B
Closing inventory 1,000 2,000
Add: Sales 8,000 15,000
9,000 17,000
Less: Opening inventory 3,000 5,000
Production 6,000 12,000
Budget requirement of components P, Q and R

P Q R
For A 1 unit of P X 6,000 6,000
2 units of Q X 6,000 12,000
For B 2 units of P X 12,000 24,000
1 unit of Q X 12,000 12,000
2 units of R X 12,000 24,000
For R 1 unit of Q X 24,000 24,000
30,000 48,000 24,000
______________________________________________________
ii. EOQ = √2 X Annual Consumption X Buying Cost Per Order/(Cost per unit X Storage and carrying cost rate)

________________________
Component P = √2 X 30,000 X 15 / (2 X 20%) = 1,500 units

Component Q = √2 X 48,000 X 15 / (0.80 X 20%) = 3,000 units

iii. Existing system


Component P Component Q

a. Present order quantity (equivalent to 3 30,000 X 1 / 4 = 48,000 X 1 / 4 =


months consumption 7,500 units 12,000 units

b. Average stock (a / 2) 3,750 units 6,000 units

51
Suggested Answers/CAP-II/December/2009/Group-I

c. Investment in inventory (b X Rs. 2 for P & Re Rs. 7,500 Rs. 4,800


0.80 for Q

d. Total investment Rs. 12,300

e. Carry cost (20% of 12,300) Rs. 2,460

f’. Number of order = 30,000 / 7,500 = =48,000/12,000


4 =4

g. Ordering cost = No. of order X ordering cost = 4 X 15 = 60 = 4 X 15 = 60


per order

h. Total cost (e + g) Rs. 2,580

After Switching Over to the New Ordering System

Component P Component Q

a. EOQ 1,500 units 3,000 units

b. Average stock (a / 2) 750 units 1,500 units

c’. Investment in inventory (b X Rs. 2 for P & Re Rs. 1,500 Rs. 1,200
0.80 for Q

d’. Total investment Rs. 2,700

e’. Carrying cost 20% of Rs. 2,700 Rs. 540

f’. Number of order = 30,000/ 1,500 =48,000 / 3,000


=20 = 16

g. Ordering cost = No. of order X ordering cost Rs. 300 Rs. 240
per order

h. Total cost (e + g) 1,080

Saving in cost = 2,580 – 1,080 = Rs. 1,500


Reduction in working capital = 12,300 – 2,700 = Rs. 9,600

Answer 5 (b)
The techniques of uniform costing may be introduced with one or more of the following objectives:

52
Suggested Answers/CAP-II/December/2009/Group-I

i. To avoid competition: It eliminates cut-throat competition by fixing common prices on the


basis of uniform costing procedures. It thus also aims at bringing stability in prices of the
products.
ii. Cost comparison: It enables different firms to compare the costs because the costs are based
on same principle. Thus their profitability can also be compared.
iii. Measurement of efficiency: Comparison of costs and profitability helps in measurement of
efficiency. Uniform costing enables the member participants to use this system as yardstick of
their achievements and performances.
iv. Reliable prices: The confidence is reposed in the public whether prices fixed are based on
sound and uniform costing principles. This will result in better and cordial relations between
members adopting this system and their customers.

v. Cost control: One of the objectives of uniform costing is an effective control over costs. This
facilitates location of unprofitable ventures. Uneconomies and inefficiencies are revealed at
every stage. The uniform cost serves as the standard cost and helps in controlling the off-
standard performances.
vi. Better exchange of information: Members having technical knowledge provide the benefit of
their experience to others. Free exchange of information leads to reduction in costs and
improvement in the quality of the product.

OR

a) One of your clients, a Jobbing Engineer, has until now based his quotations on cost of materials
and labour plus a percentage to cover overheads and profit. He has now accepted your advice
to relate costs to departments (viz., X,Y and Z) through which the work passes. In pursuance of
this policy to charge overheads to jobs on the basis of departmental direct labour hours, he has
compiled the following cost data from records for the year ending 31 Ashad, 2066.
Expenses Rs. Suggested base for apportionment to
departments
Consumable stores 500 Direct labour hours
Rent, rates and insurance 3,000 Floor area
Depreciation 2,500 Plant value
Indirect labour 4,000 Direct labour hours
Repairs and renewals 1,000 Technical estimate, i.e., 3:3:4
respectively
Labour amenities 1,500 No. of employees
Employee Insurance 400 No. of employees
Works Manager's Salary 3,600 Equally

53
Suggested Answers/CAP-II/December/2009/Group-I

General Administration 6,000 Ratio of quotations in


previous years, i.e., 5:4:3
Sundry expenses 400 12:7:1
Departments X Y Z
Direct labour hours 5,000 3,000 2,000
Wages rate per hour (Re.) 0.60 0.50 0.40
No. of employees 20 20 10
Floor area (in sq. ft.) 3,000 2,000 1,000
Value of plant (Rs.) 15,000 9,000 6,000

You are required to: (6+4=10)


i) Calculate the hourly overhead rates to be charged for work in each
department;

ii) Prepare a quotation for a job to which the following data relates:

Material Rs. 300


Direct labour:
X 20 hours
Y 15 hours
Z 10 hours

Profit is 20% of selling price.

b) Briefly explain the terms: Sunk cost and Conversion cost. 5

Answer 5 a) OR

(a) Statement showing Overhead Distribution and Hourly Overhead Rate

Expenses Ratio Total Dep. X Dep. Y Dep. Z


Consumable stores 5:3:2 Rs. 500 Rs. 250 Rs. 150 Rs. 100
Rent, rates and insurance 3:2:1 3000 1500 1000 500
Depreciation 5:3:2 2500 1250 750 500
Indirect labour 5:3:2 4000 2000 1200 800
Repairs and renewals 3:3:4 1000 300 300 400
Labour amenities 2:2:1 1500 600 600 300
Employee Insurance 2:2:1 400 160 160 80
Works Manager Salary Equally 3600 1200 1200 1200
General Admn. 5:4:3 6000 2500 2000 1500
Sundry Expenses 12:7:1 400 240 140 20

54
Suggested Answers/CAP-II/December/2009/Group-I

Total 22900 10000 7500 5400


Labour Hours 5000 3000 2000
Labour Rate per hour Rs.2.00 Rs.2.50 Rs.2.70

(b) Statement showing quotation for the Job

Material Rs. 300.00

Labour: Dep. X 20 hrs X Re. 0.60 Rs. 12.00

Y 15 hrs X Re. 0.50 7.50

Z 10 hrs. X Re. 0.40 4.00 23.50

Overhead: Dep. X 20 hrs X Re. 2.00 40.00 .

Y 15 hrs X Re. 2.50 37.50

Z 10 hrs. X Re. 2.70 27.00 104.50

Total Cost 428.00


Profit 20% of S.P. or 25% of Cost 107.00
Selling price 535.00

b)

Sunk costs are historical costs which are incurrent in the past. These types of costs are not relevant in
connection with the decision making process. For instance, while considering the replacement of equipment,
the book value of the old equipment is irrelevant as the amount is a sunk cost which is to be written off at
the time of replacement of the equipment. These are different from incremental or differential costs in the
sense that there is no effect on such costs due to increase or decrease in the activity level.

Conversion cost is the production cost excluding the cost of direct material of producing partly finished or
fully finished products. This type of cost, however, includes the cost resulting from variations in the direct
materials in weight or volume. Thus, conversion cost of a finished good or work-in-progress covers the cost
of direct labour and manufacturing overhead.

6. Answer any THREE of the following: (35=15)


a) Highlight the differences between Cost Accounting and Management Accounting.

55
Suggested Answers/CAP-II/December/2009/Group-I

b) Briefly discuss the requirements for the establishment of a scheme for inter-firm
comparison.
c) Briefly describe the critical areas where cost reduction methods can be employed.
d) Briefly explain the scope of cost audit.

Answer

a) Difference between Cost Accounting and Management Accounting:

Traditionally, cost accounting was synonymously taken as the ways and means to accumulate and assign
historical costs to units of product and departments for the purpose of inventory valuation and income
determination. Cost accounting these days refers to the gathering and providing of information for decision
needs of all types.
Management accounting includes all those accounting services which help the management in the
formulation of policy, fixation of plans and control of their execution and measurement of performance. It is
primarily concerned with the supply of information which is useful to the management in decision making
for the efficient running of the business and thus contributing toward the maximization of profits.
The principal differences in these two accounting are briefly described below:
i. Cost accounting deals primarily with the techniques of product costing where as management
accounting applies accounting data and helps in management planning and controlling function.
ii. Management accounting maintains the pool of overall data (both cost and financial) for providing
necessary information to management.
iii. Cost accounting data are used by management accounting in decision making, budgeting, and
controlling purpose.
iv. Cost accounting serves the purpose of internal reporting to managers for use in planning and
controlling routine operations and to make non routine decisions in formatting major plans and
policies. It also provides help in the process of external reporting to shareholders, government and
other outside parties.
Management accounting is more concerned with the management planning and control and is directly
focuses on internal reporting.
On the other hand, cost accounting is concerned with both the internal and external reporting. Thus,
cost accounting serves the purpose of internal reporting as is done by management accounting and
external reporting to the extent that its product costing functions satisfies the requirement of external
reporting.

b) Requirement for the Establishment of Inter-firm Comparison:

For the establishment of a scheme of inter-firm comparison, following requirements have to be met:

i) Nature and extent of information to be collected from the participating firms: It all will depend upon
the needs of the management, comparative importance of information and efficiency of the central
organization responsible for the collection of data. It is difficult to give a standard list of data which
needs to be collected that would be appropriate for all the industries. A representative list of
information which could be applicable for all the industries are given below:

56
Suggested Answers/CAP-II/December/2009/Group-I

 Information regarding costs and cost structure,


 Labour efficiency and labour utilization
 Efficiency of machines and machine utilization
 Raw material consumption
 Wastage
 Return on capital employed
 Liquidity
 Creditors and debtors
 Methods of production, and so on.

ii) Organization responsible for the collection, coordination and presentation of information: A central
organizational set up should be established to look after every aspects of the inter-firm comparison.
This unit should be made responsible to carry out research and organizing seminar and workshop on
inter-firm comparison. This organization is also entrusted with the responsibility of compiling
comparative statements and information and providing information for the benefit of different
industries.
iii) Method of collection and presentation of information: In a scheme of inter-firm comparison,
information is provided by the participating firms to the central organization overseeing the
implementation of the scheme at regular intervals. This organizes generally standardize the required
information and circulates standard form to supply the information.

Information may also be collected on the spot by field workers but this will be expensive. Since compilation
of information involves time and expenditure of the participating firms, the frequency of collection and
nature of information to be furnished at each time interval is decided before hand. Information is usually
collected once a year because many of the figures to be supplied are obtained from the profit and loss
account and balance sheet of the respective firms.

After the central organization receives the information, it is first properly sorted out to generate relevant
data in the form of a consolidated report. Once such report is produced, this is issued to the participating
firms.

Secret information are only supplied to the participating firms and not to any other unauthorized person.
Usually, the firms supply ratios and not the absolute figures to the central organization. In order to provide
further safeguard, each type of ratio is provided a code number. Similarly, each firm is known by a number
and the identity of the form is not revealed in the reports.

The design of the presentation of figures to participating member firms is done in such a way that it shows
variances from the norms. In this connection, variances may be of two types. The first type of variances are
those which arise due to basic differences in the nature of the particular firms whereas the second type are
those which results due to efficiency or inefficiency in the performance level. The participating firms need to
investigate and explain only the second type of variances.

c) Cost reduction method may be applied in the following areas:


i. Product design: Cost reduction begins with the improvements in the design of the
product. Product design is the first step in the manufacture of a product and the impact of
cost reduction effected at this stage is felt throughout the manufacturing life of the
product. An investigation into the possibilities of cost reduction should be made, both
when introducing new designs and when making improvements in the existing designs.
ii. Factory organization and production method: All efforts should be constantly made to
reduce the costs by the adoption of new methods of organization and new production
methods.

57
Suggested Answers/CAP-II/December/2009/Group-I

iii. Factory layout: A cost reduction program should make a study of the factory layout to
determine whether there is any scope of cost reduction by elimination of wastage of time,
unnecessary efforts and loss of money due to useless movements and travel of work-in-
progress.

iv. Administration: There is ample scope of cost reduction in this area because cost
reduction is a top management problem. Office should be reorganized if there is scope for
improvement in the efficiency of persons engaged in the office. Use of unnecessary forms
should be avoided to save the cost of stationery and labor cost involved for compiling
them. Efforts should make to reduce the expenses on telephone, lighting and travelling
but not at the cost of efficiency.

v. Marketing: The various activities which can be brought under the cost reduction
program include market research, advertisement, packing, warehouse, distribution, after
sales service, etc.
vi. Finance: With the increasing difficulty in procuring finance, management should
eliminate useless investment. To be able to do so, it must critically examine the amount of
working capital and fixed capital needed and the financial conveniences of reducing
them. Wasteful use of capital is as bad as inadequate capital. Over and under
capitalization are both danger signals; what is needed is fare capitalization. Capital should
be procured at economical cost and it should be economically used so as to give the
maximum return. Fixed assets and inventories which cannot be economically used should
be sold; the money realize from their sale should be reinvested in more profitable
channels.

d) There are two important aspects which come within the purview of cost audit. They are:
i. Propriety audit: It is an audit concerned with such action and plans of management
which have a bearing on the finance and expenditure of the company. The cost auditor has
not only to see that an item of expenditure is properly sanctioned and supported by
vouchers but also is justifiable on grounds of propriety. He has therefore to report:
 Whether or not the planned expenditure could give optimum results.
 Whether or not the size or channels of investment was/ were designed to produce the
best results.
 Whether the return from investment in certain channels could be bettered by some
alternative plan of action.
ii. Efficiency audit: It is an audit concerned with the appraisal of performance to determine not
only that the expenditure has been incurred according to the plan but also to see that the
results have been obtained as planned. It starts with examination of plan (such as financial
or other functional budgets) and extends to the comparison of actual performance with the
budgeted performance and finding out reasons for variances. It thus ensures that:
 Every rupee invested in capital or in other fields gives that optimum return, and
 Investment in different spheres of the business has been so balanced that it gives
maximum results.
The cost auditor thus combines in himself the role of the consultant and financial advisor. He helps
the chief executive of the organization in working out a sound overall judgment on the financial

58
Suggested Answers/CAP-II/December/2009/Group-I

plans and performances of the company by coordinating the results of actions of the various
departments.

59
The Institute of Chartered Accountants of Nepal
Suggested Answers of Financial Management

Chartered Accountancy Profession II Examination – December 2009

Marks

Attempt all questions. Working notes should form part of the answer.

1. A team of entrepreneurs are considering starting a new telecom company with GSM
technology in western part of Nepal. The team carried out a study in this respect which
shows that:

 The telecom regulatory body shall grant the license to the company for a seven
year period for a lump sum fee of Rs.210 million.
 The proposed company should carry out the project by awarding a turnkey
contract of supply, delivery, installation, and commissioning of required
equipment of total 400,000 line capacities for a total sum of US$ 4.5 million
(exchange rate assumed to be 1 US$ equivalent to NRs.80). Half of the contract
price will be paid in advance and the remaining half will be paid when the
equipment has been installed and tested to the satisfaction of the company.
 The equipment installation and testing process will be completed by the end of
the first year. And, the equipments will be out of date and obsolete by end of the
license period.
 Site survey should be carried out by the proposed company itself for installation
of the equipment, which will incur a cost of Rs.1,500,000 during the first year.
 The estimated line distribution and revenue details for the project are as below:

Particulars Year 1 Year 2 Year 3 Year 4 Year 5 and


onwards

New Line Distributed 0 100,000 140,000 160,000 0

Average Revenue Per


Line Per Month (Rs.)
0 230 210 190 170

 The 50% of lines distributed each year earn revenue for 8 months in the year of
distribution on an average and the remaining 50% lines earn revenue only for 3
months in the year of distribution on an average.
 The depreciation is to be provided on straight line basis starting from the year of
operation and is assumed to be same for the purpose of taxation.
 Corporate income tax will be charged @ 25% of taxable income. If there is loss in
any year, it will be allowed to be set off against profit of coming years for
taxation purpose.
 The project will require working capital of Rs.50 million during the license
period.
 The royalty payable to the government is @ 6% p.a. on gross operating revenue.
Suggested Answers of Financial Management
CAP II Examination – December 2009

 The annual cost of service operation and maintenance is 25% of gross revenue.
 The annual administrative cost which will be incurred starting from year 1 is
assumed to be Rs.20 million. It will increase by Rs.20 million in year 2, by further
Rs.30 million in year 3 over year 2, by further Rs.30 million in year 4 over year 3,
and will remain constant thereafter.
 The proposed company’s required rate of return will be 15%.

From the above particulars, you are required to: (6+6+8=20)

a) Prepare income statement,


b) Show calculations of capital cash outlay and cash flows of each year.
c) State, with reason and calculation of NPV and IRR, whether you would
recommend that the project be undertaken.

Answer
a)

Income Statement Amount in Rupees

Particulars Year 1 Year 2 Year 3 Year 4 Year 5 Year 6 Year 7

Annual Operating
Revenue - 126,500,000 413,700,000 714,400,000 816,000,000 816,000,000 816,000,000

Less:

Royalty - 7,590,000 24,822,000 42,864,000 48,960,000 48,960,000 48,960,000

Operation &
Maintenance
Cost - 31,625,000 103,425,000 178,600,000 204,000,000 204,000,000 204,000,000

Administrative
Cost 20,000,000 40,000,000 70,000,000 100,000,000 100,000,000 100,000,000 100,000,000

Amortization of
License Fee 30,000,000 30,000,000 30,000,000 30,000,000 30,000,000 30,000,000 30,000,000

Depreciation - 60,250,000 60,250,000 60,250,000 60,250,000 60,250,000 60,250,000

Total cost 50,000,000 169,465,000 288,497,000 411,714,000 443,210,000 443,210,000 443,210,000

Profit (Loss) (50,000,000) (42,965,000) 125,203,000 302,686,000 372,790,000 372,790,000 372,790,000

Set off of - - (92,965,000) - - - -


previous year

The Institute of Chartered Accountants of Nepal

2 of 119
Suggested Answers of Financial Management
CAP II Examination – December 2009

losses

Profit (Loss)
before tax (50,000,000) (42,965,000) 32,238,000 302,686,000 372,790,000 372,790,000 372,790,000

Taxation - - 8,059,500 75,671,500 93,197,500 93,197,500 93,197,500

Profit (Loss) after


tax (50,000,000) (42,965,000) 117,143,500 227,014,500 279,592,500 279,592,500 279,592,500

Loss to be carried
forward (50,000,000) (92,965,000)

Answer
b)_
Capital Outlay Year 1 (Rs.)

License Fee (210,000,000)

Cost of the Equipment (360,000,000)

Survey cost (1,500,000)

Working Capital tied up (50,000,000)

Total (621,500,000)

The Institute of Chartered Accountants of Nepal

3 of 119
Suggested Answers of Financial Management
CAP II Examination – December 2009

Calculation of CFAT Amount in Rupees

Particulars Year 1 Year 2 Year 3 Year 4 Year 5 Year 6 Year 7

PBT (50,000,000) (42,965,000) 125,203,000 302,686,000 372,790,000 372,790,000 372,790,000

Tax Paid - - 8,059,500 75,671,500 93,197,500 93,197,500 93,197,500

PAT (50,000,000) (42,965,000) 117,143,500 227,014,500 279,592,500 279,592,500 279,592,500

Add back:

Amortization
of License Fee 30,000,000 30,000,000 30,000,000 30,000,000 30,000,000 30,000,000 30,000,000

Depreciation - 60,250,000 60,250,000 60,250,000 60,250,000 60,250,000 60,250,000

(20,000,000) 47,285,000 207,393,500 317,264,500 369,842,500 369,842,500 369,842,500

Capital outlay (621,500,000)

Working
Capital
released 50,000,000

Total CFAT (641,500,000) 47,285,000 207,393,500 317,264,500 369,842,500 369,842,500 419,842,500

Answer
c)
Calculation of NPV at Lower Rate

Year CFAT (Rs.) PVIF at 15% PV (Rs.)

2 47,285,000 0.8696 41,117,391

3 207,393,500 0.7561 156,810,225

The Institute of Chartered Accountants of Nepal

4 of 119
Suggested Answers of Financial Management
CAP II Examination – December 2009

4 317,264,500 0.6575 208,606,559

5 369,842,500 0.5718 211,458,650

6 369,842,500 0.4972 183,877,087

7 419,842,500 0.4323 181,509,499

983,379,410

1 (641,500,000) 1 (641,500,000)

NPV 341,875,410

Calculation of NPV at Higher Rate

Year CFAT (Rs.) PVIF at 30% PV (Rs.)

2 47,285,000 0.7692 36,373,077

3 207,393,500 0.5917 122,714,734

4 317,264,500 0.4552 144,408,056

5 369,842,500 0.3501 129,492,140

6 369,842,500 0.2693 99,609,338

7 419,842,500 0.2072 86,981,378

619,578,723

1 (641,500,000) 1 (641,500,000)

NPV (21,921,277)

IRR= Lr +(NPV at Lr/ ∆PV of Inflow)* ∆r

0.15+{341,875,410 /(983,379,410-
= 619,578,723)}*(0.30-0.15)

= 0.2909

IRR= 29.09%

The Institute of Chartered Accountants of Nepal

5 of 119
Suggested Answers of Financial Management
CAP II Examination – December 2009

Since the NPV at required rate of return of 15% is positive and the IRR is greater than the required
rate of return, the project should be undertaken.

Working Notes:

(1) Calculation of Annual Operating


Revenue
Particulars Year 1 Year 2 Year 3 Year 4 Year 5 Year 6 Year 7

Line Distributed 0 100000 140000 160000 0

Lines in operation 0 100000 240000 400000 400000 400000 400000

Lines earning revenue 0 0 100000 240000 400000 400000 400000


for 12 months

Lines earning revenue 0 50000 70000 80000 0 0 0


for 8 months

Lines earning revenue 0 50000 70000 80000 0 0 0


for 3 months

Average Revenue Per 0 230 210 190 170 170 170


Line Per Month (Rs.)

Annual Operating
Revenue (Rs.’000) - 126,500 413,700 714,400 816,000 816,000 816,000

(2) Calculation of Depreciation

Particulars Year 1 Year 2 Year 3 Year 4 Year 5 Year 6 Year 7

Cost of Equipment
(Rs.’000) 360,000

(4500*80)

Survey cost (Rs.’000) 1,500

Total Cost (Rs.’000) 361,500

Amount of depreciation
(Rs.’000) - 60,250 60,250 60,250 60,250 60,250 60,250

The Institute of Chartered Accountants of Nepal

6 of 119
Suggested Answers of Financial Management
CAP II Examination – December 2009

2. A steel manufacturing company is planning to expand its assets by 50 percent. All financing
for this expansion will come from external sources. The expansion will generate additional
sales of Rs. 6 million with a return of 20 percent on sales before interest and taxes.

The finance department of the company has submitted the following plan for the
consideration of the Board of Directors.

Plan 1: Issue of 12.5% debentures.


Plan 2: Issue of 12.5% debentures for half the required amount and balance in equity shares
to be issued at 20 percent premium
Plan 3: Issue equity shares at 20 percent premium.

The Balance Sheet and Income Statement of the company as on the last day of Ashadh is as
given below.

Balance Sheet of the company as on Ashadh 2066

Liabilities Amount Assets Amount

Equity Capital (Rs. 100 per share) Rs. 8,000,000 Total Assets Rs.24,000,000

10% Debentures 6,000,000

Retained Earnings 4,000,000

Current Liabilities 6,000,000 __________

24,000,000 24,000,000

Income Statement for the year ending on Ashadh 2066

Sales Rs. 38,000,000

Operating Costs 32,000,000

Earning Before Interest and Taxes (EBIT) 6,000,000

Interest 600,000

Earning Before Tax (EBT) 5,400,000

Taxes 1,890,000

Earning After Tax (EAT) 3,510,000

The Institute of Chartered Accountants of Nepal

7 of 119
Suggested Answers of Financial Management
CAP II Examination – December 2009

Earning Per Share (EPS) 43.875

Based on the above data and information, you are required to calculate: (9+6=15)

a) Indifference points between (i) Plan 1 and 2, (ii) Plan 1 and 3, and (iii) Plan 2 and 3.

b) Expected market price of the shares in each of the situations on the assumption that the
price earnings ratio is expected to remain unchanged at 12 if plan 3 is adopted, but is
likely to drop to 9 if either plan 1 or 2 is used to finance the expansion.

Answer

Preliminary Computations:

Number of Equity Share to be issued under Plan 3 = Rs. 12,000,000/Rs. 120 = 100,000

Number of Equity Share to be issued under Plan 2 = Rs. 6,000,000/Rs. 120 = 50,000

12.5% Debentures to be issued under Plan 1 = 50% of Rs. 24,000,000 = Rs. 12,000,000.

12.5% Debentures to be issued under Plan 2 = 50% of Rs. 24,000,000 X 0.5 = Rs. 6,000,000.

(a) Indifference Point among different Finance Plans:

(i) Between Plans 1 and 2

[ (X – I1 – I2) ((1 – t) ] / N1 = [ (X – I1 – I2 ) ((1 – t) ] / N2

Where,
X = Earnings before interest and taxes (EBIT) at the indifferent point
N1 = Number of equity shares outstanding if only equity shares are issued.
N2 = Number of equity shares outstanding if both debentures and equity shares are issued.
I= Amount of interest on debentures
t= Corporate income tax rate

Substituting the values, we get:

[(X – 600,000 –1,500,000) 0.65] = [(X –600,000 – 750,000) 0.65]

80,000 130,000

Or, 13 (X – 2,100,000) = 8 (X – 1,350,000)

Or, 13 X – 27,300,000 = 8 X – 10,800,000

The Institute of Chartered Accountants of Nepal

8 of 119
Suggested Answers of Financial Management
CAP II Examination – December 2009

Or 5 X = 27,300,000 – 10,800,000 = 16,500,000, Therefore X = Rs. 3,300,000

(ii) Between Plans 1 and 3

[(X – 2,100,000) 0.65] = [(X – 600,000) 0.65]

80,000 180,000

Or, 18 (X – 2,100,000) = 8 (X – 600,000)

Or, 18 X – 37,800,000 = 8 X – 4,800,000

Or, 10 X = 33,000,000, Therefore X = Rs. 3,300,000

(iii) Between Plan 2 and 3

[(X –1,350,000) 0.65] = [(X – 600,000) 0.65]

130,000 180,000

Or, 18 (X-1,350,000) = 13 (X – 600,000)

Or, 18 X – 24,300,000 = 13 X – 7,800,000

Or, 5 X = 16,500,000

Therefore X = Rs. 3,300,000.

Answer

(b) Determination of Market Price per Share under Various Alternative Plans:
__________________________________________________________________________

Particulars Plan 1 Plan 2 Plan 3

__________________________________________________________________________________
_________________________________________________________________________

EBIT* Rs. 7,200,000 Rs. 7,200,000 Rs. 7,200,000


Less: Interest 2,100,000 1,350,000 600,000
Earning before Taxes 5,100,000 5,850,000 6,600,000
Less: Taxes (@ 35%) 1,785,000 2,047,500 2,310,000
The Institute of Chartered Accountants of Nepal

9 of 119
Suggested Answers of Financial Management
CAP II Examination – December 2009

EAT 3,315,000 3,802,500 4,290,000


Number of Equity Shares 80,000 130,000 180,000
EPS (EAT/No. of Shares) 41.4375 29.25 23.8333
P/E Ratio 9 9 12
Expected Market Price per Share 372.94 263.25 286.00

______________________________________________________________________________

* Existing EBIT Rs. 6,000,000 + EBIT on Additional Sales of Rs. 6,000,000 (0.20 X 6,000,000) = Rs.
7,200,000

3.

a) A risky portfolio has an expected market return of 14%. What should be the proportion
of investment in risky and risk free investment of a portfolio to secure 20% expected
portfolio return, where government securities are earning 5%? 5

b) Jessica wishes to know the expected return on her following portfolio, when the risk-
free rate is 7% and the return on market is expected to be 20%. 5

Security Percentage of Portfolio Beta factor of Security

R 15 0.2

S 10 1.2

T 5 1.8

U 30 0.9

V 25 0.2

W 15 0.8

c) Explain the concept of an efficiency frontier. 5

Answer

a) Expected market return on risky portfolio, E (rm) = 14%

Risk free rate of return (rf) = 5%

The Institute of Chartered Accountants of Nepal

10 of 119
Suggested Answers of Financial Management
CAP II Examination – December 2009

Expected return on total portfolio, E (rp) = 20%

Now,

E (rp) = Wm * E (rm) + (1- Wm) * rf

20% = Wm * 14% + (1- Wm) * 5%

Wm = 15/9= 1.667

Wf = 1-1.667 = -0.667

Therefore, the proportion of risky investment in the portfolio is 166.67%, and

the proportion of risk free investment is –66.67%.

b) Portfolio’s beta (β) = 0.15*0.2+0.1*1.2+0.05*1.8+0.3*0.9+0.25*0.2+0.15*0.8

= 0.68

Now,

Expected return on the portfolio of Jessica would be

Rp = Rf + β (Rm – Rf)

= 7% + 0.68 (20% – 7%)

= 15.84%

Therefore, Jessica’s expected rate of return on portfolio is 15.84% .

c) The efficiency frontier traces out the set of available portfolio combinations consistent with risk
aversion, i.e. all portfolios which maximize expected returns for a given risk or minimize risk
for a given return. The aim of any rational risk-averting investor is to locate on the boundary,
although precisely where will depend on the extent of his or her risk-aversion.

4. The summarized accounts of New Ideas Ltd. are as follows:

Balance Sheet as at Ashadh end

Year 2 (Rs. 000) Year 1 (Rs. 000)

Fixed assets (net) 6,401 2,519

The Institute of Chartered Accountants of Nepal

11 of 119
Suggested Answers of Financial Management
CAP II Examination – December 2009

Current assets

Stock 25,426 20,231

Debtors 21,856 20,264

Balance at bank __2,917 __6,094

Total 56,600 49,108

Ordinary shares of Rs.100 5,000 5,000

Revenue reserves 14,763 12,263

Deferred taxation 5,433 3,267

10% Debenture loans 10,000 10,000

Current liabilities

Trade creditors 18,762 16,431

Taxation 1,642 1,247

Dividends __1,000 ___900

Total 56,600 49,108

Results for the year ended Ashadh

Year 2 (Rs. 000) Year 1 (Rs. 000)

Sales 264,626 220,393

Trading profit 9,380 8,362

Interest payable 1,000 1,000

Taxation 4,380 3,642

Dividend 1,500 1,400

The following additional information is provided:

 The ordinary shares are quoted at Rs.240.


 The company requires Rs.16 million for an investment project and is considering
one of the following:
o The issue of Rs.16 million 10% debentures.

The Institute of Chartered Accountants of Nepal

12 of 119
Suggested Answers of Financial Management
CAP II Examination – December 2009

o A right issue at par.

You are required to: (6+4+5=15)

a) Calculate for both years (i) two ratios particularly significant to creditors, (ii) two
ratios particularly significant to management, and (iii) two ratios particularly
significant to shareholders. And, express a brief comment on the trend of ratios.
b) Calculate the immediate effect of the two schemes of fund raising on the gearing
of the company and comment.
c) Calculate the effect of the two schemes on the earnings per share, on the
assumption that the Year 2 profits from the existing assets will be maintained and
that Rs.16 million net investments will produce profits of Rs.3.5 million before
tax and interest. The rate of tax can be assumed at 25%.

Answer

a) Year 2 Year 1

(i) Ratios significant to creditors

 Current ratio = Current assets/ Current liabilities

Year 2 (50199/21404) 2.35:1

Year 1 (46589/18578) 2.51:1

 Liquidity ratio = (Current assets-stock)/ Current liabilities

Year 2 (24773/21404) 1.16:1

Year 1 (26358/18578) 1.42:1

(ii) Ratios significant to management

 Activity ratio = Pre-tax profit/ Sales

(pre-tax profit= trading profit- interest)

Year 2 (8380/264626) 3.17%

Year 1 (7362/220393) 3.34%

 Profitability ratio = Pre-tax profit/ Net assets

(where, net assets equals to total assets less current liabilities)

Year 2 (8380/35196) 23.8%

The Institute of Chartered Accountants of Nepal

13 of 119
Suggested Answers of Financial Management
CAP II Examination – December 2009

Year 1 (7362/30530) 24.1%

(iii) Ratios significant to shareholders

 Return on capital employed= PAT/ Shareholders’ Fund

Year 2 (4000/19763) 20.2%

Year 1 (3720/17263) 21.6%

 Dividend cover ratio = PAT/ Dividend

Year 2 (4000/1500) 2.7:1

Year 1 (3720/1400) 2.7:1

Comments on trends of ratios:

In spite of an increase in sales of 20% and an increase in pre-tax profits of 13.8%, the ratios
mentioned show a marginally unfavourable trend between Year 1 and Year 2.

b) Effect of fund-raising schemes on gearing

Gearing = (Loan capital + Preference shares)/ Shareholders’ Equity

Currently, Gearing = 10000/ 19763 = 50.6%

o The issue of Rs.16 million 10% debentures.


Gearing = 26000/ 19763 = 131.6%

o A rights issue at par.


Gearing = 10000/ 35763 = 28%
Comment:

The first scheme will lead to high level of gearing.

c) Effect of fund-raising schemes on EPS

EPS = PAT/ No. of shares outstanding

Current EPS = Rs.4,000,000/ 50000 = Rs.80

o The issue of Rs.16 million 10% debentures.


PAT = Trading profit - interest – tax
= (9380+3500) – (1000+1600) – {4380+25% * (3500 – 1600)}
= 12880 – 2600 – 4855
The Institute of Chartered Accountants of Nepal

14 of 119
Suggested Answers of Financial Management
CAP II Examination – December 2009

= Rs.5425 thousand
EPS = Rs.5425000/ 50000
= Rs.108.5

o A rights issue at par.


PAT = Trading profit - interest – tax
= (9380+3500) – 1000 – (4380+25% * 3500)

= 12880 – 1000 – 5255

= Rs.6625 thousand

EPS = Rs.6625000/(50000+160000)

= Rs.6625000/210000

= Rs.31.55

5.

a) Guheswari Paints Ltd. having its production facilities and head office located in
Biratnagar currently follows a centralized collection system. Most of its customers are
located in the cities of terai region and Kathmandu Valley as well as in the remote hilly
districts of Nepal.

The remittances mailed by customers take on an average 9 days to reach to central


location. Before depositing the remittances in the bank account, the company loses 2
days in processing them. The daily average collection of the company is Rs. 750,000.
The company is thinking of establishing a lock-box system. It is expected that such a
system will reduce the mailing time by two day and processing time by one day.
Based on the above facts and information, you are required to: (3+2+3=8)
i) Determine the reduction in cash balance expected to result from the adoption of
the lock-box system.
ii) Determine the opportunity cost of the present centralized collection system if the
interest rate is assumed to be 15 per cent.
iii) Advise the company whether it should establish lock-box system if its annual cost is
Rs. 250,000.

b) A company has a total investment of Rs. 4,000,000 in assets and 40,000 outstanding
ordinary shares at Rs. 100 per share (par value). It earns at a rate of 15 percent on its
investment, and has a consistent policy of retaining 50 percent of the earnings. If the
appropriate discount rate of the firm is 10 percent: (3+4=7)
i) Determine the price of its share using Gordon’s model.
The Institute of Chartered Accountants of Nepal

15 of 119
Suggested Answers of Financial Management
CAP II Examination – December 2009

ii) What shall happen to the price of the shares if the company has a payout of 20 per
cent and 60 per cent respectively?
Answer
a)
(a) The total time saved by the company by establishing the lock-box system is 3 days.

Reduction in cash balance = Time saved x daily average collection


= 3 x 750,000 = Rs. 2,250,000

(b) Opportunity cost = 15% x Rs. 2,250,000 = Rs. 337,500.


The lock-box system should be established because the opportunity cost of the present system
(Rs. 337,500) is higher than the cost of the lock-box system (Rs. 250,000) by Rs. 87,500.

b)
(a) Price of Share using Gordon’s model:

The share valuation model of Gordon is as follows:

P0 = DIV1 = (1 – b)EPS1 = (1 – b)rA , where

k–g k – br k – br

A denotes investment per share, which is Rs. 100 in the present case.

When the payout is 50 per cent, the price of share will be:

P0 = (1 – 0.5) 0.15 x 100 = 0.5 x 15 = 7.5/0.025 = Rs. 300

0.10 – (0.15 x 0.5) 0.10 – 0.075

(b) Price of Share at Payout of 20 and 60 percent:

(i) Payout of 20 per cent

P0 = (1 – 0.8) 0.15 x 100 = 0.2 x 15 = 3/-0.02 = Rs. - 150

0.10 – (0.15 x 0.8) 0.10 – 0.12

(ii) Payout of 60 per cent:

P0 = (1 – 0.4) 0.15 x 100 = 0.6 x 15 = 9/0.04 = Rs. 225

0.10 – (0.15 x 0.4) 0.10 – 0.06

6. Write short notes on: (42.5=10)


The Institute of Chartered Accountants of Nepal

16 of 119
Suggested Answers of Financial Management
CAP II Examination – December 2009

a) Financial intermediaries
b) Marginal cost of capital
c) Du pont equation
d) Stochastic model of cash management

Answer

a) Financial intermediaries:

Financial intermediaries are the various financial institutions, such as pension funds,
insurance companies, banks, building societies, unit trusts and specialist investment
institutions. Their role is to accept deposits from personal and corporate savers to lend to
customers via the capital and money markets. They perform vital economic service of:

(i) Re-packaging finance: collecting small amounts of finance and re-packaging into larger
bundles for specific lending requirements (e.g. banks).

(ii) Risk-reduction: investing sums, on behalf of individuals and companies, into large,
well-diversified investment portfolios (e.g. pension funds and unit trusts).

(iii) Liquidity transformation: bringing together short-term lenders and long-term


borrowers (e.g. building societies).

(iv) Cost reduction and advice: minimizing transaction costs and providing low-cost
services to lenders and borrowers.

b) Marginal cost of capital:


The marginal cost of capital is defined as the cost of the last rupee of new capital that
the firm raises and the marginal cost rises as more and more capital is raised during a
given period. A firm’s marginal cost of capital, also known as weighted marginal cost
of capital, reflects the fact that as the volume of total new financing increases, the
costs of the various types of financing will increase, raising the firm’s cost of capital.
Breaking points, which are found by dividing the amount of funds available from a
given financing source by its capital structure weight, represent the level of total new
financing at which the cost of one of the financing components rises, causing an
upward shift in the weighted marginal cost of capital. The weighted average cost of
capital WACC is associated with its next rupee of total new financing. The WMCC
schedule relates the WACC to each level of total new financing. It is important for
capital expenditure decisions.

c) Du pont Equation:
A Du pont shows the return on equity is affected by assets turnover, profit margin and
leverage. The Du Pont Chart was developed by Du Pont managers for evaluating
performance and analyzing ways of improving performance.
The Institute of Chartered Accountants of Nepal

17 of 119
Suggested Answers of Financial Management
CAP II Examination – December 2009

The profit margin times the total assets turnover is called the Du Pont equation. This
equation gives the rate of return on assets (ROA).

ROA = Net Profit margin* Total assets turnover


= Net Income/sales *Sales/Total assets

The ROA times the equity multiplier (total assets divided by common equity) yields
the return on equity(ROE). This equation is referred to as the Extended Du Pont
Equation.
ROE= Net Income/Total Assets * Total Assets/Common Equity

d) Stochastic model of cash management:


This model assumes that cash balances randomly fluctuate between an upper limit and
a lower limit. The value of a lower limit is set by the management, the value of return
point and upper limit have been derived by Miller and Orr with a view to maintaining
the total ordering and holding costs.

Return Point (Z)= 3√3bs2/4i +Lower Limit(LL)


Upper Point = 3z-2LL

Where,
b= fixed cost per order
i= daily interest rate
s2 = variance of net the daily cash flows.

The average cash balance is obtained as follows:

ACB = 4z-LL/3

When the cash balance reaches the upper limit an amount equal to the upper limit
minus the return point is converted to marketable securities.

When the cash balance falls to lower limit, the amount converted from marketable
securities to cash is the amount represented by the return point minus lower limit.

7. Distinguish between:
a) Nominal interest rate and effective interest rate 3
b) Share split and reverse split 3
c) Wealth maximization and profit maximization 4

Answer

a) The interest rate that is specified on an annual basis in a loan agreement or security (say, in
the case of a bond) is known as the nominal interest rate. Thus, the nominal interest rate is

The Institute of Chartered Accountants of Nepal

18 of 119
Suggested Answers of Financial Management
CAP II Examination – December 2009

the simple interest rate (without compounding) which is stated in the face of a security or a
loan agreement.

Usually, there are provisions of compounding of the interest payable on a loan or security.
The compounding may be done monthly, quarterly or semi-annually. When compounding of
the interest payable is done more than once a year, the actual annualized interest would be
higher than the nominal interest rate and it is called the effective interest rate.

The general formula for calculating effective interest rate (EIR) can be written in the
following form:

EIR = [ 1 + i/m] n x m – 1, where

i denotes the annual nominal rate of interest,

n denotes the number of year and

m denotes the number of compounding per year.

In the case of annual compounding, m = 1, in quarterly compounding m = 4, and in the case


of monthly compounding, m = 12. The formula given above can be modified to accomplish
the multi-period compounding for any number of years.

(b)

Share Split: The process of reducing the paid up value of a share and increasing the number
of outstanding shares is called the share split. In the stock split, the balance in the equity
share capital account remains the same unlike in the case of bonus share in which the
balances of reserves and surpluses account decreases due to a transfer to the paid-up
capital.

The earnings per share and market price per share will fall proportionately following the split
of the share. The total value of the holdings of a shareholder remains the same with a share
split.

There are a number of reasons considering which the share split is carried out. Some these
are:

 To make the trading in share attractive

With the reduction in paid up value per share, the market price per share comes down
so that it becomes accessible to small investors as well. This helps to make the shares of
such company more marketable and liquid.

 To signal the possibility of higher profits in the future

Often the share split are used by the top management of the company to communicate
to investors that the company is expected to earn higher level of profits in future.

The Institute of Chartered Accountants of Nepal

19 of 119
Suggested Answers of Financial Management
CAP II Examination – December 2009

Expected high profits in future forces the company management to employ share split so
as to make the shares of the company fall inside the popular trading range.

 To give higher dividends to shareholders

In general, there have not been cases of reduction of cash dividends proportionately
after share split. Often, it is found that the total dividend of the shareholders is found to
have increased after such split. It is needless to say that this will have a positive impact
on the market price of the shares of the company.

Reverse Split is employed under the situation of falling price of a company’s share. The
number of outstanding shares is reduced following the reverse split. The paid up value per
share is increased thus bringing down the number of outstanding shares.

The reverse split is generally an indication of financial difficulty of a firm and is primarily
intended to increase the market price per share by making the per share price beyond the
reach of common investors. The reverse split is sometimes used to stop the market price of
the share below a certain limit.

C) Wealth maximization and profit maximization

The differences between the wealth maximization and profit maximization are as
follows:
Wealth maximization Profit maximization
It is shareholder oriented It is owner oriented
It is represented by highest market It is represented by large amount of
value of the stock EPS
More emphasis on long term More emphasis on short term
objectives objectives
It considers the time value of the Does not consider the time vale of
money money
It considers the risk element Preference is not given to the risk
element
It is qualitative It is quantitative
It is concerned with the welfare of It is more concern with owners.
investors, creditors, government,
society.

The Institute of Chartered Accountants of Nepal

20 of 119
Suggested Answers of Financial Management
CAP II Examination – December 2009

The Institute of Chartered Accountants of Nepal

21 of 119
The Institute of Chartered Accountants of Nepal
Suggested Answers of Business Communication & Marketing

Chartered Accountancy Profession II Examination – December 2009

Marks

All Questions are compulsory.

Section -'A'

9. The process of communication does not always run smooth. It often gets adversely disturbed
by unwanted things, happenings, interferences, interruptions, etc. resulting in
communication failure between the sender and the receiver. Such elements are called
communication barriers - (a) physical and (b) mental.
Assume that you are the Chief Editor of a local English Daily Newspaper. Your job involves a
lot of communicating with people- your own reporters and workers, people of other
organizations, and the conscious people. The job is not easy because it is often affected by
miscommunication caused due to the communication barriers. Now, mention, with
examples, five effective ways of overcoming them. 10
Answer 1:
The Chief Editor of the Local English Daily is conscious of the fact that communication often
gets disturbed or adversely affected by unwanted things, happenings, interferences,
interruptions, etc. resulting in communication failure between the sender and the receiver.
Such elements are called communication barriers - (a) physical and (b) mental. He has
identified the following as communication barriers. A number of factors can be alleged as
causes of communication barrier. They are called inappropriate filters

Major communication barriers include: a. noise, b. lack of planning, c. semantic variations,


d. cultural barriers, e. wrong assumptions, f. emotion, g. day dreaming, h. information
overload, i. socio-psychological obstructions, j. poor attention. k. poor retention. l. pre-
occupied brain, m. communication leakage n. hearing weakness o. cross -cultural
communication barriers, and p. inadequacy of communication tools and materials

Apart from the above, there are some others known as organizational barriers.
Suggested Answers of Business Communication and Marketing
CAP II Examination – December 2009

(a) Long chain of command. With the long chain of command, distance between the upper
and lower levels of organizational people increases, resulting in the distortions,
disturbances and distractions in various directions of communication.

(b) Congested room partition, narrow passages between rooms, inconvenient disposition of
furniture, gap and distance between staff members cause organizational
communication.

c) Use of inappropriate and unusable medium hindering face-to-face communication and


easy delivery of messages is another barrier.

The above barriers are classed into such types as i. Physical and mechanical barrier, ii.
Organizational barrier, iii. Socio-psychological barrier, iv. Psychological

Ways of overcoming communication barriers:

Barriers though obstruct the flow of communication, are not problems that defy solutions.
The following are some of the ways and means of overcoming communication barriers. .

(a) Physical barriers caused by physical disturbances can be overcome through temporary
avoidance of communication. Stop talking till the noise continues, or waits till you reach
your destination.

(b) Understanding the nature of socio-cultural barriers before communicating,

(c) Fear to face communication is a barrier, which can be overcome by practicing speaking
before the mirror. Assassinated U.S. President Abraham Lincoln did this.
(d) Selecting appropriate channel for communicating ideas and feelings.

(e) Emphatic listening. Delivered message should be received in a proper way through
proper analysis and interpretation of the facts.

(f) Listening should be unbiased.

The Institute of Chartered Accountants of Nepal

23 of 119
Suggested Answers of Business Communication and Marketing
CAP II Examination – December 2009

(g) Avoiding complex feelings. The listener should think, “I may be wrong and the speaker
may be right.”
(h) Creating proper communication environment.

(i) Matters to be communicated should be planned.

In addition to the above:

(j) Organizational barriers, which include i. long chain of command, ii. Poor layout and iii.
inappropriate medium, can be overcome by shortening scalar chain and appropriate
layout

(k) Socio-psychological barriers which occur due to emotion, poor attention and poor
retention, wrong assumptions and evaluation of things, status consciousness,
perceptual distortion and attitudes and behavior could be minimized through openness
of mind, free and frank expression of views.

Physical barriers, which include noise – traffic and street noise, and any other kind of noise
and noise caused by technical problems, can be controlled by stopping communication for a
while and by correcting the technical problems

10. In the light of the fact that today’s workplace is increasingly getting diversified in terms of
culture, language, ethnicity, religion, etc. Discuss both the benefits and the challenges
resulting from such a diverse working environment. Also, suggest some concrete ways in
which such challenges can be sorted out in order to create a more congenial working milieu. 10
Answer 2)
Workplace diversity refers to varieties of differences between people in an organization. That
sounds quite simplistic, but diversity encompasses race, gender, ethnic group, age, personality,
cognitive style, tenure, organizational function, education, background and more.

Diversity not only involves how people perceive themselves, but how they perceive
others as well. Those perceptions affect their interactions. For a wide assortment of
employees to function effectively as an organization, human resource professionals need
to deal effectively with issues such as communication, adaptability and change. Diversity
will increase significantly in the coming years. Successful organizations recognize the
need for immediate action and are ready and willing to spend resources on managing
diversity in the workplace now.

Benefits of workplace diversity:

An organization’s success and competitiveness depends upon its ability to embrace


diversity and realize the benefits. When organizations actively assess their handling of

The Institute of Chartered Accountants of Nepal

24 of 119
Suggested Answers of Business Communication and Marketing
CAP II Examination – December 2009

workplace diversity issues, develop and implement diversity plans, multiple benefits are
reported such as:

 Increased adaptability:

Organizations employing a diverse workforce can supply a greater variety of solutions to


problems in service, sourcing, and allocation of resources. Employees from diverse
backgrounds bring individual talents and experiences in suggesting ideas that are flexible
in adapting to fluctuating markets and customer demands.

 Broader service range:

A diverse collection of skills and experiences (e.g. languages, cultural understanding)


allows a company to provide service to customers on a global basis.

 Variety of viewpoints:

A diverse workforce that feels comfortable communicating varying points of view


provides a larger pool of ideas and experiences. The organization can draw from that pool
to meet business strategy needs and the needs of customers more effectively.

 More effective execution:


Companies that encourage diversity in the workplace inspire all of their employees to perform
to their highest ability. Company-wide strategies can then be executed; resulting in higher
productivity, profit, and return on investment.

Challenges of diversity in the workplace:

Taking full advantage of the benefits of diversity in the workplace is not without its
challenges. Some of those challenges are:

The Institute of Chartered Accountants of Nepal

25 of 119
Suggested Answers of Business Communication and Marketing
CAP II Examination – December 2009

 Communication:

Perceptual, cultural and language barriers need to be overcome for diversity programs to
succeed. Ineffective communication of key objectives results in confusion, lack of
teamwork, and low morale.

 Resistance to change:

There are always employees who will refuse to accept the fact that the social and cultural
makeup of their workplace is changing. The “we’ve always done it this way” mentality
silences new ideas and inhibits progress.

Successful management of diversity in the workplace:

Diversity training alone is not sufficient for your organization’s diversity management
plan. A strategy must be created and implemented to create a culture of diversity that
permeates every department and function of the organization. The following points can
be recommended for the successful management of diversity:

Assessment of diversity in the workplace: Top companies make assessing and evaluating
their diversity process an integral part of their management system. A customizable
employee satisfaction survey can accomplish this assessment for your company
efficiently and conveniently. It can help your management team determine which
challenges and obstacles to diversity are present in your workplace and which policies
need to be added or eliminated. Reassessment can then determine the success of you
diversity in the workplace plan implementation.

Development of diversity in the workplace plan: Choosing a survey provider that


provides comprehensive reporting is a key decision. That report will be the beginning
structure of your diversity in the workplace plan. The plan must be comprehensive,
attainable and measurable. An organization must decide what changes need to be made
and a timeline for that change to be attained.

Implementation of diversity in the workplace plan: The personal commitment of


executive and managerial teams is a must. Leaders and managers within organizations
must incorporate diversity policies into every aspect of the organization’s function and
purpose. Attitudes toward diversity originate at the top and filter downward. Management
cooperation and participation is required to create a culture conducive to the success of
your organization’s plan. On top of this, we can ward off change resistance with inclusion
by involving every employee possible in formulating and executing diversity initiatives
in the workplace. We can also foster an attitude of openness in our organization by
encouraging employees to express their ideas and opinions so as to attribute a sense of
equal value to all. Promoting diversity in the leadership positions and making use of the
diversity training will also be helpful.

The Institute of Chartered Accountants of Nepal

26 of 119
Suggested Answers of Business Communication and Marketing
CAP II Examination – December 2009

Due to globalization and free market policies almost all around the globe, our workforces
have become increasingly diverse. Organizational success and competitiveness will
depend on the ability to manage diversity in the workplace effectively

11. What is a proposal? A Nepali non-governmental organization working in the field of poverty
alleviation of rural peasants has lately appointed you as the Director to head the
organization’s Resources Department. Your tasks primarily include writing proposals to
international funding organizations in order to carry out the activities launched by the
organization. Prepare an outline of such a funding proposal and briefly discuss each of the
parts. 10
Answer

A proposal is a written persuasive document asking for financial assistance to implement


a project. It is not just a shopping list of things we want. It must rather justify each item
in the list of things we want, so that a donor agency can decide if it wants to provide
some or all of those things. We must know and be able to communicate exactly what we
want to do with these things, and that is why we should design a project to carry out what
we want to achieve.

It is important to carefully formulate and design our project. It is equally important to


write a proposal which will attract the necessary funding. Proposal writing is a skill
which requires some knowledge and practice. Its main job is to inform and to convince. I
should clearly indicate how and when the project will end and become self-supporting.
Proposals should be neat and tidy and it should not contain any extraneous information.

Outline of a proposal:

i) Title Page: This is a single page i.e. the front cover of the proposal. It should
include date, project title, location of the project, name of the organization and
any necessary single line information.
ii) Background: this section is expected to answer why your project is needed. It
is here where we give a description of the situation and focus on the factors
which prompted the formulation of our proposed project. The first thing the
background does is to identify the problem. That is to say, it must name the
problem and locate it. It also indicates the target groups or beneficiaries.
Everything in this section should be justification to approve the project and
the requested funding assistance.
iii) Objectives: The objective of the project should be to solve the problem
described in the background. They should therefore relate to the previous
chapter by stating what is the solution to the above stated problems. We need
to develop a set of general goals and specific objectives.

The Institute of Chartered Accountants of Nepal

27 of 119
Suggested Answers of Business Communication and Marketing
CAP II Examination – December 2009

iv) Beneficiaries: In this chapter we describe the beneficiaries or target groups in


some detail. We may also add indirect or secondary beneficiaries. This can be
an expansion of the topic mentioned in our background section.
v) Targets and activities: This chapter identifies the inputs in our project, which
means the resources such as cash, personnel and actions we will put into our
project.
vi) Schedule: In this section we describe in sequence the activities we plan in
order to achieve our objectives. If we can be so specific as to give dates, even
if approximate, so far the best.
vii) Profile of the organization: this section describes the organization and
management structure needed to carry out the activities described above.
Diagrams can be very useful in this. We also describe in brief the goals and
activities of the organization. The mention of the organization’s members and
human resources will also prove to be an asset.
viii) Costs and benefits: The chapter that deals with the costs and benefits is not the
same as a line by line budget with numbers indicating amounts of money. It
should be analytical and narrative relating to the previous chapters. It should
also discuss those budget lines that mat need explanation.
ix) Monitoring: The monitoring should be done an overseeing committee
comprising the representatives of the locals. Organization and the donor
agency.
x) Reporting: In any agency-funded project, accounting and accountability are
very important. Our reporting should evaluate the progress of the project
while it is underway as it will help us and our donors to see the
accomplishments and the choices available for future action. Reports should
be prepared and submitted optimally every month. The final report include the
same topics as the monthly reports including the lessons learnt and a section
indicating the impact of the project on the target community and the
surrounding areas.
xi) Detailed budget: Each line on our detailed budget should have the total costs
for one budget category. The lines should be grouped into similar kinds of
costs. If possible who should distinguish non-expendable items from the
expendable ones.
xii) Abstract: This is the part on which a potential donor will read and make that
vital preliminary decision: whether or not to seriously consider assisting. This
should not be written until all the above sections are written. We should think
of it as a concise summary and conclusion.

12. What are the techniques used to prepare a resume for computer scanning? 10
Answer
A computer scannable or online resume contains hypertext links to work samples of a
portfolio of additional information. It can be viewed whenever it is convenient for an
employer, and it can be seen by many individuals in an organization without circulating

The Institute of Chartered Accountants of Nepal

28 of 119
Suggested Answers of Business Communication and Marketing
CAP II Examination – December 2009

a paper copy. Although a scannable resume may look similar to traditional print resume,
the scannable resume needs some special care and techniques in its design:

• Avoid punctuation as much as possible

• Avoid vertical and horizontal lines, graphics, and boxes

• Do not fold or staple

• Use only those abbreviations that are familiar with your field of study

• No piece of punctuation should ever touch a word (Punctuation can affect the
employer’s ability to find matching terms in a keyword search).

• Use fonts like Times New Roman, New Century, Courier, Helvetica, and sometimes
Palatino

• Font size should be between 10-12

• Avoid fancy font styles such as italics, underline, and shadows.

• Boldface and CAPITAL letters are acceptable provided that the letters do not touch
each other

• Provide white space between words

• Avoid condensing the spaces between letters and lines

• Use nouns and noun phrases instead of active verbs

• Incorporating words that are commonly used within the industry and indicate your
personal work skills increase the chances of having your resume read.

13. Write short notes on the followings (Any Two): (2×5=10)


a) Conflict resolution
b) Secondary and elementary data in data collection
c) Long report and short report
Answer
a)
In many cases, conflict in the workplace just seems to be a fact of life. Furthermore, the
head of an organization has to come across such situations where different people with
different goals and needs come into conflict. If not managed, the conflict may result in
intense personal hostility that will ultimately have repercussions on the organization’s

The Institute of Chartered Accountants of Nepal

29 of 119
Suggested Answers of Business Communication and Marketing
CAP II Examination – December 2009

output. On the other hand, the successful management of conflict will result in better
output.

Hence, the fact that conflict exists, is not necessarily a bad thing. As long as it is resolved
effectively, it can lead to personal and professional growth. The conflict should be
understood in terms of the dynamics of progress. In many cases, effective conflict resolution
skills can make the difference between positive and negative outcomes. If conflict is
managed successfully, many problems that it has brought to the surface can be resolved, on
top of other benefits that we might not at first expect:

 Increased understanding: The discussion needed to resolve conflict expands people's


awareness of the situation, giving them an insight into how they can achieve their own
goals without undermining those of other people.
 Renewed team spirit: When conflict is resolved effectively, team members can develop
stronger mutual respect and a renewed faith in their ability to work together.
 Improved self-knowledge: Conflict pushes individuals to examine their goals in close
details, helping them understand the things that are most important to them,
sharpening their focus and enhancing their effectiveness.

Nonetheless, if conflict is not handled effectively, the results can be damaging.


Conflicting goals can quickly turn into personal dislike. Teamwork breaks down.
Talent is wasted as people disengage from their work. And it's easy to end up in a
vicious downward spiral of negativity and recrimination. If we want to keep our
organization working effectively, we need to stop this downward spiral as soon as we
can.

b) The first one refers to the data already made available in the form of books, previous
records made during research and investigation.
The second one refers to fresh and new data collected during research and investigation.

c) Long reports: The length of the report depends upon the contents. Thicker the contents,
longer the report.

a. Long reports are written after detailed investigation and analysis of issues with
necessary activities towards ascertaining reliable facts. They include TOR, the
guidelines provided by the authority.
b. Action taken: involves fact-finding activities on which the report will be based.
c. Findings: Including findings ascertained after investigation.
d. Analysis: Impersonal, objective analysis of facts.
e. Conclusion: Drawing conclusion after presentation of factual contents
f. Recommendations: Recommending suggestive measures for improvement.

The Institute of Chartered Accountants of Nepal

30 of 119
Suggested Answers of Business Communication and Marketing
CAP II Examination – December 2009

Short reports: They differ from long ones in many reports. .

a. As the term implies, short reports are short in length.


b. They consume less time then the long reports
c. They may be written at the leisure time.
d. They may not need highly skilled manpower
e. They are less content-loaded

The Institute of Chartered Accountants of Nepal

31 of 119
Suggested Answers of Business Communication and Marketing
CAP II Examination – December 2009

Section -'B'

6. Read the following case carefully and answer the questions that follows:

Sony, a multinational corporation based in Tokyo, Japan, stepped into India in early 1995.
Within a couple of months in the same year, it set up its manufacturing unit. The company
makes it presence felt in India through Sony Entertain Television, Sony Music and Sony India
(digital products).

Sony India is facing some major problems, one of which is the emergence of a gray market
(where the products sold through distribution channels that are not authorized by the
manufacturer) for its brands. Severe competition and many brands are quite common. At
present, some distribution and market research related issues exists with Sony India while it
is targeting its sales around 2,000 crores by the end of 2008. Sony likes to develop its
business units in a few cities of USA, Europe and Asia. Since it is a multinational company-
manufacturing multi- product, it has a challenging task to enter new market, increase
market share and sales volume as well as face competition.

Questions:

a) What major problems do you see in the case relating to market potentialities of Sony
India? 5
b) What suggestions would you like to give to Sony India to make its business successful? 5
Answer

Sony India, though a reputed company may have to face numerous problems while entering
to a new market. A few major problems seen in the above case are:

- Emergence of a gray market (Where the company's products sold through


distribution channels that are not authorized by the manufactures.)

- Selection of research agency: in house or external agency?

- Ambitious target: Sales, market share.

- Severe competition: many brands.

The Institute of Chartered Accountants of Nepal

32 of 119
Suggested Answers of Business Communication and Marketing
CAP II Examination – December 2009

- Distribution related problems: unauthorized dealings.

- Market research related issues.

- Marketing mix and segmentation related problems: product, place, price, and
promotion related issues to compete and achieve goals.

- Entry to many cities of major global markets at a time.

- Other external environmental forces can be the major problems in the potential
market of Sony India.

b) The following suggestions have been made to Sony India on the basis of analysis of the
given case:

- Selection of appropriate research agency, better to select external one.

- Conduct SWOT analysis for market decisions.

- Monitor and control over authorized dealers & intermediaries.

- Launch effective promotional campaigns.

- Establish up dated & reliable Marketing Information Systems.

- Collect sufficient market information & crates better understanding with the
new markets.

Social responsibilities reflecting activities to be given more priority, etc.

7.
a) Differentiate between the production concept and product concept of marketing. 5
b) What is marketing environment? Explain how economic environment affects the
marketing activities. 5
Answer
a) Production concept: an assumption, that consumers will favor those products that are
widely available at affordable price. This concept leads production-oriented firms to
emphasize on achievements of high production efficiency, low costs and mass

The Institute of Chartered Accountants of Nepal

33 of 119
Suggested Answers of Business Communication and Marketing
CAP II Examination – December 2009

distribution. Product concept: an assumption, that consumers will favor those products
that offer the most quality, performance, or innovative features/attributes. It clarifies
that marketing firms should produce superior products as against competing products
and improve them overtime.
b) Marketing operates in a changing environment. It is shaped by both internal and
external environmental forces. Organizations have to manage their activities within the
given environment to achieve their goals. It consists of different forces that influence
the company’s ability to play with the target market. Threats and opportunities in the
external whereas internal environmental forces provide strengths and weaknesses.
Economic environment forces affect the overall marketing program and systems.
Economic forces are: Economic health, national income, monthly and fiscal policy per
capita income, employment opportunity, inflation, resources, competition, loan
availability capital market, interest rate, and institutional infrastructure etc. These above
factors are the elements of a person’s purchasing power. Marketers have to study
activities, reform policies, implement tactics/policies/strategy in action for the benefit of
own company
8.
a) State and explain the process of new product development. 5
b) How pricing policy should be set for a product or service for the first time? 5
Answer
a) This is eight step process. This process consists of:

i) Idea generation
New product development starts with an idea. New product ideas come from a
variety of sources i.e; customers, scientists, competitors, employees, channel
members, investors, marketing consultants, universities, business journals,
seminars etc. Various techniques have been developed and used for idea
generation i.e; attribute listing, forced relationship, morphological analysis,
need/problem identification, brain storming etc.
ii) Idea screening
The purpose of screening is to drop poor ideas as early as possible. The total
generated ideas are caterogorised into three groups i.e;
Promosing ideas – carried on for further testing
Marginal ideas – stored for future use
Reject ideas – dropped altogether
iii) Concept development and testing
A concept development and testing is the method of trying to guage buyer
interest before actual product has been developed.
iv) Marketing strategy development
After testing, the new product manager must develop a preliminary marketing
strategy plan for inducing the new product into the market.
v) Business analysis

The Institute of Chartered Accountants of Nepal

34 of 119
Suggested Answers of Business Communication and Marketing
CAP II Examination – December 2009

This stage is essential in the total process of new product development because
several vital decisions regarding the projects are expanded into business
proposal.
vi) Product development and testing
Having completed the business analysis, it moves to R & D or engineering to be
developed into a physical product. In this step, the idea-on-paper is converted
into a physical product and product testing may be conducted through an
expert panel, central location testing and in-home test.
vii) Market testing
After management is satisfied with functional and psychological performance,
the product is ready to be dressed up with a brand name, packaging and put to
a market testing.
viii) Commercialisation
Full-scale production and marketing programs are planned and the product is
launched in the target market. Upto this point in the development process,
management has generally complete control over the product.

b) Pricing policies are guidelines for management of price and pricing from a long-term
perspective. In order to set and formulate pricing policy for a product or service for the
first time, the price setter should select the pricing objectives, determine demand,
estimate costs, analyse competitor’s costs, prices and offers, select a pricing method
and select the final price.
Pricing policies are developed to meet pricing objectives either in-terms of profit or
sales or status-quo orientation and to guide the price setter and/or the marketer to take
a particular course of action in handling the price. The pricing objective reflects the
overall goal of the organization. It describes what an organization wants to through
pricing. Pricing policies are also developed to meet market demand, manufacturing and
marketing cost and competition and set prices by selecting a general pricing method.
Pricing policies can developed and set for a product or service for the first time to make
more effective and competent price in the market.
Best alternatives can be developed from these following four categories.
 Flexible pricing policies
 Geographical pricing policies
 Channel member’s incentive policies
 Product mix pricing policies
9.
a) What is physical distribution? Explain its components. 5
b) What is market information? Briefly explain the marketing research process. 5

Answer

The Institute of Chartered Accountants of Nepal

35 of 119
Suggested Answers of Business Communication and Marketing
CAP II Examination – December 2009

a) Physical distributions are those activities involved with the storage and movement
of products from the producer to consumer. It is the management of the physical
flow of products and the establishment and operation of efficient flow systems. It
is also called logistics management which aims to provide better consumer
services and faster delivery at reasonable costs.

It helps to minimize the distribution costs.


There are five major components of physical distribution in distribution channel
system:
1) Transportation:
In physical distribution, transportation is major tool for moving the product
from point of origin to the point of consumption. Transportation may include
truck, train, aero plane, ships, human muscles, animal power, electric lines,
pipelines etc.
2) Inventory management:
Inventory management concerned with developing the system of maintaining
and adequate level of product ascertained to meet the consumer demand.
Inventory management is required to control inventory cost such as order
processing, carrying cost etc. For the inventory management economical
measures are the most and managers should be constituted on re-order point
and economic order quantity.
3) Warehousing:

Raw materials and finished product are keep safely in different warehouse.
Goods will be delivered to buyers and materials will be delivered to
manufacturing process.
4) Materials handling:
One of the important components of physical distribution is materials handling.
Materials handling includes all those transportation or movement activities of
material work in process, supplies from suppliers to the producer and the final
products from the producer to consumers or users.
5) Order processing:
Order processing is also an important component of physical distribution. It is
concerned with the firm's ability to deliver the products ordered by the
customers. Order processing includes all those activities involved in collection of
orders from customers; checking them and transmitting sales order information.
b) Information relating to the market- opportunities\threats, environmental changes, all
interrelated and interacting parts and the procedure to generate, analyze, decisions are
to be treated as market information. Many information, different sources, everywhere-
it is the life blood of marketing. Reliable, timely and adequate information is very
important in marketing.

The Institute of Chartered Accountants of Nepal

36 of 119
Suggested Answers of Business Communication and Marketing
CAP II Examination – December 2009

Marketing research is a process to collect information in solving a specific problem.


Helps to solve business problems, provides alternates, guides in making decisions. A
systematic way of solving the problems. A large amount of money spent by big company’s to
achieve goals through research findings. Marketing research process: (Steps)

1) Define the problem


Problem shouldn’t be defined either too broadly or too narrowly. Answering
these questions define problem appropriately i.e;

What is to be researched?

Why is it to be researched?

2) State objectives
In this step, the research objectives must be clearly stated. It should be specific
and precise.

3) Develop research plan


This step of marketing research calls for developing the most efficient plan for
gathering the needed information. The choice of research plan depends on the
depth and extent of data required, the costs and benefits of the research, the
urgency of the work and the time available for completing it. This research plan
involves decisions on the data sources, research approach, research
instruments, sampling plan and contact methods.

4) Collect information
Collecting information may prove expensive, time and effort consuming and
prone to errors whether these are to be collected through primary or secondary
sources.

5) Analyze the information


This step in the marketing research process is to extract findings from the
collected data. The researcher tabulates the data and develops frequency
distributions. The researcher will apply various advanced statistical

10. Briefly explain any five of the followings: (5×2=10)

The Institute of Chartered Accountants of Nepal

37 of 119
Suggested Answers of Business Communication and Marketing
CAP II Examination – December 2009

a) Marketing mix
b) Niche market
c) Pull and push strategies
d) B2B vs. B2C buying behavior
e) Service product
f) Relationship marketing
g) Advertising media

Answer
a) Marketing-mix:
Marketing mix is a combination of product, price, place and promotion which are
used to purchase the firms objectives in the target market. It consists of
everything the firm can do to influence the demand for its product. 4Ps constitute
the components of the marketing mix. Each component has its own mix such as
product-mix, price mix, place mix and promotion mix.
b) Niche market:
A niche market is a more narrowly defined group typically a small market whose
needs are not well served. Niche marketers clearly understand their customer’s
needs so well that the customers willingly pay a premium. The niche marketing
gains certain economies through specialization. The niche market has size, profit
and growth potential. The niche is not likely to attract other competitors.
c) Push & Pull strategy:
Pull strategy
Under pull strategy, promotion is directed at end users or ultimate consumers. The
main purpose of using this strategy is to motivate consumers for asking the
channel members for the product. This strategy largely depends upon
 mass communication
 the objective of pull buyers through channels by creating end-users
demand.
 Popular brand, extensive distribution.
Push strategy
Under push strategy, promotion is directed towards the middlemen. This strategy
can be successfully implemented in case
 the manufacturer has supplied higher quality products with unique selling
points(USP)
 the product are high priced
 adequate incentives to middlemen and salesmen are offered.
d) B2B vs B2C buying behavior
B2B buying behavior is the study of the decision making process by which formal
organizations establish the need for purchased products and services and identify,
evaluate and choose among alternative brands and suppliers. Whereas B2C
buying behavior is the study of all psychological, social and physical behavior of
all potential consumers as they become aware of, evaluate, purchase, consume
and tell others about products and services.

The Institute of Chartered Accountants of Nepal

38 of 119
Suggested Answers of Business Communication and Marketing
CAP II Examination – December 2009

e) Service product includes all: transportation, public utilities, insurance, legal, real-
estate, educational institutions, hospitals etc. It includes organizations that
produce and sell such diverse services as rental housing, recreation and
entertainment, repairs, health care, personal care and
business services etc.Today service product are highly demand and accompanying
with the terrible products are also to be considered
f) Relationship marketing is developing a mutually beneficial relationship with
selected customers. The seller attempts to develop a deeper, longer-lasting
relationship built on trust with key customers. Sellers have to show ability,
knowledge, honesty and behave in a reliable manner to show customer
orientation. Relation with existing customers rather finding new one.
g) Advertising is one of the most commonly used promotion tools by business
houses, non-government organization, service institutions, etc. It is a paid form of
non-personal presentation and promotion of goods\services, ideas by an identified
sponsor. It is done for mass communication to inform, persuade, remind,
reinforce and achieve organization goals. Ad media are the vehicles through
which message passed to scattered target market. Most used Ad media are:
• Print-Newspaper, Journal, Magazine, Book, directory, brochure, Mail, Internet, etc.

• Visual, Poster, display, exhibits, billboard, Sky writing etc.

• Audio-Radio- Audiotape, telephone, etc.

• Audiovisual-Tv, Film, Videotapes, etc

The Institute of Chartered Accountants of Nepal

39 of 119
The Institute of Chartered Accountants of Nepal
Suggested Answers of Income Tax and VAT

Chartered Accountancy Profession II Examination – December 2009

Marks

Attempt all questions. Working note should form part of the answer

Indian Income Tax Act

14. Answer the following questions with reference to Indian Income Tax Act.

a) What do you mean by Gross Total Income? 4


b) What are the factors that determine the residential status of a Hindu Undivided
Family? 3
c) Mr. Wilson, a British national, comes to India for the first time during 2004/05.
During the Financial Years 2004/05, 2005/06, 2006/07, 2007/08 and 2008/09, he
was in India for 55 days, 60 days, 80 days, 160 days and 70 days respectively.
Determine his residential status for the Assessment Year 2009/10. 3
Answer
a) According to section 2(45) “Total Income” means the total amount of income referred to
in section 5, computed in the manner laid down in the Income Tax Act. According to Sec.
80B of the Income Tax Act, “Gross Total Income” means the total income computed in
accordance with the provisions of the Income Tax Act before making any deduction under
chapter VIA.

Income Chargeable to income tax shall be classified under five heads of income for the
purpose of computation of taxable amount subject to certain exemptions and deductions.
Section 14 stipulates that the computation shall be under the five heads of income:

a) Salaries (Sec. 15 to 17)


b) Income from house property (sec.22 to 27)
c) Profits and gains of business or profession (sec. 28 to 44DA)
d) Capital gains (sec. 45 to 55A)
e) Income from other sources (sec.56 to 59)
Suggested Answers of Income Tax and VAT
CAP II Examination – December 2009

The aggregate of taxable income under all the five heads of income, together with clubbed
income/deemed income and after giving effect to set off of current year and brought
forward losses shall be known as “Gross Total Income”.

b) Resident: A Hindu Undivided family is said to be resident in India if control and


management of its affairs is wholly or partly situated in India.

Non-Resident: The Hindu Undivided family is non-resident in India if control and


management of its affairs is wholly situated outside India.

Resident & Not Ordinary Resident: If the control and management of the affairs of the HUF
is wholly or partly situated in India and if the manager of the family satisfies any of the
following conditions, the HUF shall be considered as “Not Ordinarily Resident”

a) He has been a non-resident in India in 9 out of the 10 preceding previous years; OR


b) He has been in India for a period of not exceeding 729 days during the 7
preceding previous years.
Resident and Ordinarily Resident: If the control and management of the affairs of the HUF
is wholly or partly situated in India and if the manager of the family fails to satisfy both the
above conditions, the HUF shall be considered as “Ordinarily Resident”.

c) Mr. Wilson is said to be resident in India in any previous year if he fulfills any one of the
following two basic conditions:

i) He is in India in that previous year for periods of 182 days or more.

ii) He is in India for periods amounting in all to 60 days or more during the previous
year and 365 days or more during the 4 years preceding those previous years.

Based on above, Mr. Wilson did not stay for 182 days in 2008/09. He failed to fulfill the
first condition as mentioned above. Having stayed 70 days during 2008/09, he has
stayed only for 355 days during preceding four Financial Years. Also, he failed to fulfill
the second basic condition. Therefore, his residential status for the assessment year
2009/10 is that of non-resident.

15.
a) Mr. Ram Singh is a salaried employee having taxable salary income amounting to Rs.
1,50,000 during the fiscal year 2066/67. Calculate the tax liability of Mr. Ram Singh
(opting as couple) as per the provisions of Income Tax Act, 2058 for the fiscal year

The Institute of Chartered Accountants of Nepal

41 of 119
Suggested Answers of Income Tax and VAT
CAP II Examination – December 2009

2066/67. Will your answer be different if his earning was from proprietorship
business not from salary? 5
b) Smart Co. Ltd. had contributed Rs. 3,50,000 to a non-approved Provident Fund on
behalf of the employees and has claimed it as expenditure. The Tax Office proposes
to disallow the claim and add to the income. The company argues that since it has
already spent the money, it is a business expenditure spent for the operation of
normal business and it should be fully allowed under Section 13. What should be the
correct action under the provisions of Income Tax Act, 2058? 5
c) Mr. Anup pays an advance amounting to Rs. 10 lakhs in cash to Mr. Binod, who
required it urgently to clear his goods from customs. The same was returned by Mr.
Binod in cash after six months. Discuss the tax consequence of the above
transaction quoting the relevant provisions of Income Tax Act, 2058. 5
d) What is the Tax Avoidance Scheme? Mention the power available to Inland Revenue
Department under General Anti Avoidance Rule for determining tax liabilities 5

Answer
a) 1% tax was levied on the income upto Rs. 2,00,000 for couple. Therefore, the tax
liability of Mr. Ram Singh in his salary income will be Rs. 1,500 (1% of Rs. 1,50,000).

But this 1% tax rate will not be applicable to a person who is doing business by
registering a sole proprietorship firm. Therefore, there will be no tax liability to Mr.
Ram Singh if his income was from proprietorship business in the fiscal year 2066/67.

b) Any contribution to either approved or non-approved provident fund is treated as


business expenditure in case the company has made the payment to each employee
or deposited the amount in individual employees account with the provident fund.
The company should not have any right to receive back the amount, but each
employee should get an enforceable right to receive the amount in future with
benefits attached with the deposits. In case the company has not considered the
individual amount of the contribution at the time of assessing the income of the
employees for the purpose of tax deduction at source under section 87, even then,
the contribution shall be treated as allowable expenses but the company is liable to
pay the amount of tax calculated after including the amount of contribution to each
employees taxable income. In case the company has deposited the amount in lump
sum in an account run by the company, the amount can not be said as contribution
and so not allowed for deduction.

According to Section 8, any type of payment to employee by an employer, under


agreed terms of employment or as a rules or practice of the entity, shall be treated

The Institute of Chartered Accountants of Nepal

42 of 119
Suggested Answers of Income Tax and VAT
CAP II Examination – December 2009

as income from employment of the employee. That is why, it is treated as


expenditure of the entity because of the real interpretation of section 13 of the Act.
The expenses are incurred during the year, incurred by the company as its per rules
and that payment is made in the production of income from the business.

c) Cash payment of Rs. 10 lakhs as advance does not attract the provisions of Section
21, as it is not a payment of expenses. But Mr. Binod has utilized the amount free of
cost for six months. Though Mr. Binod has paid no amount as interest there on, but
utilized the fund for his business and so according to section 27 of Income Tax Act,
2058, a difference between the actual rate of interest and market rate of interest
should be charged as deemed income from business of Mr. Binod. In this case, the
actual interest is zero and so the amount calculated for six months at the prevailing
market rate shall be included in taxable income of Mr. Binod.

d) Tax Avoidance Scheme means any arrangement, one of the main purposes of which
is the avoidance or reduction of tax liability.
Under the provisions of Income Tax Act, the tax authorities, under General Anti
Avoidance Rule, are empowered to prevent tax payers from indulging any
arrangement under Tax Avoidance Scheme. Accordingly, for the purpose of
deferring tax liability under the Income Tax Act [sec 35], the department may:

# re- characterize an arrangement as part of an arrangement that is entered into or


carried out as part of a tax avoidance scheme;

# disregard an arrangement or part of an arrangement that does not have


substantial economic effect;

# re- characterize an arrangement the form of which does not reflect its substance.

16. If Mr. Nayak Shakya who is going to retire from 1st Shrawan 2067, submits the following
details with respect to his employment for the Income Year 2066/67.
a) Net Salary received Rs. 4,57,000
TDS Paid Rs. 37,000

b) Contribution by employer to retirement fund (approved) Rs. 1,00,000


c) His Contribution to approved retirement fund Rs. 1,50,000
d) Compensation received as per Employee Rule Rs. 70,000

The Institute of Chartered Accountants of Nepal

43 of 119
Suggested Answers of Income Tax and VAT
CAP II Examination – December 2009

e) Investment Insurance Premium paid by Employer on his behalf, Rs. 19,000


insured sum of Rs. 2,00,000
f) Encashment of un-availed leave upto Chaitra 18, 2058 Rs. 50,000, after Chaitra
18, 2058 Rs. 1,70,000.
g) Emergency Medical Treatment paid by Employer Rs. 10,000. But he didn’t claim
Medical Tax Credit.
h) Reimbursement of Tour Expenses during Official visit to Hongkong Rs. 2,40,000.
i) Gratuity Received (lumpsum payment)
Upto 18 Chaitra, 2058 Rs. 4,00,000
After 18 Chaitra, 2058 Rs. 7,00,000 (approved)

j) Provident Fund Received upto Chaitra 18, 2058 (Lumpsum Payment) Rs.
4,00,000 (unapproved)
After Chaitra 18, 2058 Rs. 6,00,000 (approved)
Mr. Nayak also conducts a small trading business M/S Nayak Concerns of his own.
Information relating to which are as follows:

Particulars Rs. Particulars Rs.

To Opening Stock 80,000 By Sales 5,00,000

To Purchase 3,90,000 By Closing Stock 1,20,000

To Gross Profit 1,50,000

Total 6,20,000 Total 6,20,000

To Personal Drawings 60,000 By Gross Profit 1,50,000

To Office Expenses 18,000 By Office Furniture Sales 35,000

To Printer Purchased 7,000 By Bad Debt Recovered 1,00,000

To Net Profit 2,50,000 By Dividend 50,000

Ms. Nayak works in a private institution & earns a total taxable salary of Rs.
2,00,000. The couple has opted to be assessed separately for Income Tax Purpose.

Additional Information:

a) M/S Nayak Concern has the following WDV of Fixed Assets as on 01.04.2066.
Furniture Rs. 1,00,000

The Institute of Chartered Accountants of Nepal

44 of 119
Suggested Answers of Income Tax and VAT
CAP II Examination – December 2009

Computer Rs. 45,000

Motorcycle Rs. 80,000

One Spare Table is sold on Rs. 35,000

b) Above Office Expenses includes Rs. 2,000 for the purpose of rice cooker gifted to
a customer on his wedding party.
c) Printer was purchased on 15th Poush, 2066.
d) Bad debt Recovered Rs. 1,00,000 was not allowed as expenses in F/Y 2064/65.
e) Dividend Rs. 50,000 is from NIC Bank Ltd.

He also provides following further information not related to his employment, and
business.

a) Gain Rs. 1,00,000 on Sale of shares.


b) Donation to a political Party Rs. 50,000.
c) Interest of Rs. 20,000 from personal deposit account of NATURAL BANK LTD.
Required: 20
i) Assessable Income from Employment
ii) Assessable Income from Business
iii) Total Assessable Income
iv) Tax Liability.

Answer
i) Calculation of assessable income from employment of Mr. Nayak Shakya for the
Income Year 2066/67.

Particulars Rs. Rs.

Salary 457000.00

TDS paid by Employer 37000.00

Contribution to Provident Fund by Employer 100000.00

Investment Insurance Premium paid by 19000.00


Employer

Encashment of un-availed leave (after Chaitra 170000.00

The Institute of Chartered Accountants of Nepal

45 of 119
Suggested Answers of Income Tax and VAT
CAP II Examination – December 2009

Particulars Rs. Rs.

18, 2058)

Emergency Medical Treatment borne by 10000.00


Employer (Note-2)

Compensation received 70000.00

Assessable Income from Employment 863000.00

ii) Calculation of assessable income from business of Mr. Nayak Shakya for the Income
Year 2066/67.

Particulars Rs. Rs.

Profit or Gain 500000.00

Sales 500000.00

Deduction of Expenses 411250.00

Less: Cost of goods sold (Section 15)

Opening Stock 80000.00

Add: Purchase 390000.00

Less: Closing Stock 120000.00 350000.00

Less: Office Expenses (Rs. 18000 – Rs. 2000)


[Note:6] (Section 13)
16000.00

Less: Depreciation (Note:7] (Section 19) 45,250.00

Assessable Income from Business 88750.00

iii) Statement of Total Assessable & Taxable Income.

Particulars Rs.

The Institute of Chartered Accountants of Nepal

46 of 119
Suggested Answers of Income Tax and VAT
CAP II Examination – December 2009

Assessable Income from Employment (a) 863000.00

Assessable Income from Business (b) 88750.00

Assessable Income from Investment 100000.00

(Gain on disposal of non-business Chargeable Asset)

Total Assessable Income 1051750.00

Less: Allowable Reductions

Contribution to retirement fund

Actual (100000+150000) 250000

or, maximum limit 300000

or, one-third of assessable income (Rs. 1051750x1/3) 350583

250000

Least of above three 250000.00

Donation: Lower of:

Max Limit 100000.00

5% of Adjusted Taxable Income (1051750-250000) X 5% 40087.50 40087.50.

Actual 50000.00

Total Taxable Income including Capital Gain 761662.50

iv) Computation of Tax Liability


Income Tax:

Rs.

First Rs. 19000 @ 0% 0.00

Next Rs. 160000 @ 1 % 1600.00

The Institute of Chartered Accountants of Nepal

47 of 119
Suggested Answers of Income Tax and VAT
CAP II Examination – December 2009

Next Rs. 100000 @ 15% 15000.00

Balance Rs. 382662.50 @ 25% (excluding capital 95665.63


& gain)

Capital Gain Rs. 100000 @ 10%


10000.00

122265.63

Less: Medical Tax Credit (not claimed) Nil

Advance TDS 37000 37000.00

Net Tax Payable 85265.63

Working Notes:

1) Income Tax Act came into effect from 19th Chaitra, 2058. As per Rule 20(6)
all amounts exempt from tax under ITA, 2031 which accrued prior to the Act
coming into force will remain exempt from the tax even if payment is made
after 18th Chaitra, 2058. Accordingly, encashment of un-availed leave,
gratuity received and provident fund (all amounts accrued before 19th
Chaitra, 2058) are exempt from tax.

2) Petty Expenses borne by employer on behalf of employee exceeding Rs. 500


will be included in the Income of Employees.

3) Reimbursement of Tour Expenses incurred in connection with business


purpose will not be included in employment income.

4) In case retirement payment is in the form of a lump sum amount: Rs.


500000 or 50% of such payment whichever is higher is exempt for tax
(section 65) (1) (kha). As per section 88 (amended) in respect of payment
from approved retirement fund TDS @ 5% is to be made on the sum
calculated as per section 65. (i.e. after deduction of Rs. 5,00,000 or 50% of
such payment whichever is higher).

As per section 92, this will be final withholding tax.

Rs.

Gratuity 700000

The Institute of Chartered Accountants of Nepal

48 of 119
Suggested Answers of Income Tax and VAT
CAP II Examination – December 2009

Provident Fund 600000

Total (A) 1300000

50% of 1300000 650000

Maximum Amount 500000

Tax free amount (which ever is higher (B) 650000

Taxable Amount (A-B) 650000

Final TDS @ Rs. 650000 @ 5% 32500

As the retirement payment is made in a lump sum, the taxable amount of


Rs. 650000 is taxed @ 5% final TDS.

5) Interest from personal Bank Account is subject to 5% TDS, which is final,


TDS = Rs. 20000 x 5%

= Rs. 1000

6) Gift to the customer is not allowed as deduction as per Income Tax Act, 2058.

7) Calculation of depreciation on Fixed Assets as per Schedule-2 of Income Tax Act, 2058.

Particulars Rate of Opening Addition Sales Depreciable Depreciation


Depreciation WDV as on Amount
01.04.66

Block B

Furniture 25% 100000.00 - 35000.00 65000.00 16250.00

Computer 25% 45000.00 - - 45000.00 11250.00

Printer 25% - 7000.00 7000.00 1750.00

Block D

Motorcycle 20% 80000.00 - - 80000.00 16000.00

45250.00

The Institute of Chartered Accountants of Nepal

49 of 119
Suggested Answers of Income Tax and VAT
CAP II Examination – December 2009

8) Bad debt not allowed as expenses in F/Y 2064/65, was taxed in F/Y 2064/65.
Hence, not taxable in F/Y 2066/67.

9) Dividend for company is final withholding tax. Hence, not included in Income
Tax Calculation.

10) It is assumed that donation has been made to political Party registered with
Election Commission.

17.
a) Calculate the amount of TDS to be deducted by a payer of the following payments
as per the provisions of the Income Tax Act, 2058. Also state whether the payments
are final withholding payments or not? 5

i) M/S Lama Construction Pvt. Ltd. has received an invoice for Rs. 3,39,000
including VAT from M/S Jurist Company Pvt. Ltd. for legal advisory services
rendered during fiscal year 2065/66.
ii) Amount paid by an approved retirement fund Rs. 15,00,000. The accumulated
principal and the interest thereon as on Chaitra 19, 2058 for the same amount
was Rs. 4,00,000
iii) Amount paid by an unapproved retirement fund Rs. 15,00,000. The
accumulated principal and the interest thereon as on Chaitra 19, 2058 for the
same amount was Rs. 4,00,000.
Total amount deposited by the account holder after Chaitra 19, 2058 to the
retirement fund Rs. 9,00,000.
iv) Binod paid Rs. 50,000 as interest to Ram for loan taken from him. Binod has
taken the loan for his personal purpose.
v) A college is paying Rs. 20,000 to Mr. Ramesh for setting various question papers.
Mr. Ramesh is an employee of the college and getting regular remuneration.

b) State with reason whether the following statements are True or False. 5
i) As per the definition of Income Tax Act, 2058 “every entity is a person but every
person is not an entity”.
ii) A non-resident has to pay income tax on income accrued in Nepal only.
iii) Dividend of a co-operative society is exempt from tax.
iv) An individual has to follow accrual basis of accounting while calculating income
under the head investment.
v) Meeting allowance shall not be included in the taxable income of an individual.

Answer

The Institute of Chartered Accountants of Nepal

50 of 119
Suggested Answers of Income Tax and VAT
CAP II Examination – December 2009

a) Calculate the amount of TDS to be deducted by a payer of the following payments as


per the provisions of the Income Tax Act, 2058. Also sate whether the payments are
final withholding payments of not?

i. M/S Lama Construction Pvt. Ltd. has received an invoice for Rs. 339,000
including VAT from M/S Jurist Company Pvt. Ltd. for legal advisory services
rendered during fiscal year 2065/66.

Total invoice amount excluding VAT is Rs. 3,00,000 and TDS shall be Rs. 4,500
(1.5% of Rs. 3,00,000). This is not the final withholding payment.

ii. Amount paid by an approved retirement fund Rs. 15,00,000. The accumulated
principal and the interest thereon as on Chaitra 19, 2058 for the same amount
was Rs. 4,00,0000.

Total amount of Retirement Fund 15,00,000

Less: Balance as on Chaitra 19, 2058 4,00,000

Total Taxable amount of Retirement Fund payment 11,00,000

Exemption (50% of Rs. 11,00,000) 5,50,000 Net


Taxable Amount 5,50,000

TDS shall be deducted @ 5% Rs. 27,500 and this is final withholding payment.

iii. Amount paid by an unapproved retirement fund Rs. 15,00,000. The


accumulated principal and the interest thereon as on Chaitra 19, 2058 for the
same amount was Rs. 4,00,000.
Total amount deposited by the account holder after Chaitra 19, 2058 to the
retirement fund Rs. 9,00,000.

Total amount of fund after Chaitra 19, 2058 11,00,000

Less: Amount deposited by account holder 9,00,000

Taxable amount of unapproved retirement fund 2,00,000

The Institute of Chartered Accountants of Nepal

51 of 119
Suggested Answers of Income Tax and VAT
CAP II Examination – December 2009

TDS to be deducted @ 5% of Rs. 2,00,000 i.e. Rs. 10,000 and it is final


withholding payment.

iv. Binod paid Rs. 50,000 as interest to Ram for loan taken from him. Binod has
taken the loan for his personal purpose.

TDS deduction is not required by Binod in this payment as the payee is


individual. But this amount will be added in the total income of Mr. Ram.

v. A college is paying Rs. 20,000 to Mr. Ramesh for setting various question papers.
Mr. Ramesh is an employee of the college and getting regular remuneration.

TDS deduction is not required by the college in this payment as per the
provision of section 88(4)(ka1). But this amount will be added in the total
income of Mr. Ramesh.

b) State with reason whether the following statements are True or False.

i. As per the definition of Income Tax Act, 2058 “every entity is a person but
every person is not an entity”.
True.

The definition of a person includes the natural person and entity but the
definition of the entity does not include the natural person. Entity includes
only an artificial person not the natural person.

ii. A non-resident has to pay income tax on income accrued in Nepal only.
True.

The income having the source outside Nepal is not taxable in the hands of
non-resident in Nepal. Therefore, a non-resident has to pay income tax on
income accrued in Nepal only.

The Institute of Chartered Accountants of Nepal

52 of 119
Suggested Answers of Income Tax and VAT
CAP II Examination – December 2009

iii. Dividend of a co-operative society is exempt from tax.


False.

As per section 11(2), dividend distributed by agriculture based and rural co-
operative society is only exempt from tax. But dividend distributed by other
co-operative societies is not exempt from tax.

Alternatively, it can also be treated as Partially True.

iv. An individual has to follow accrual basis of accounting while calculating


income under the head investment.
False.

As per section 22, cash basis of accounting has to be followed while


calculating income from investment.

v. Meeting allowance shall not be included in the taxable income of an


individual.
True.

As per section 92, meeting allowance is a final withholding payment


therefore it shall not be included in the taxable income of an individual.

18.
a) Binod & Company, a proprietor engaged in the business of selling battery operated
tempo, its chassis and battery. Its annual turnover of the business was estimated of
Rs. 10 million. Binod & Company not registered with VAT. The advisor of the
company suggested to get registered with VAT or ready to pay penalty as per VAT
Act. Assuming you are the advisor of Binod & Company, what would be your advice
to the company?
Would your answer be different if Binod & Company is engaged in the business of
solar energy equipments and operate under an umbrella of Alternative Resources
Promotion Centre? 5

b) Mr. Ram, a furniture supplier produced some woods from a community forest
through auction and has not paid any Royalty or VAT on the deal. IRD Officer
The Institute of Chartered Accountants of Nepal

53 of 119
Suggested Answers of Income Tax and VAT
CAP II Examination – December 2009

assessed the transaction and ordered to Mr. Ram to pay VAT on the auctioned price.
Mr. Ram disputed the order and argued that since there was no Royalty amount it
can not be required to pay VAT. Mr. Ram appoints you as a professional advisor.
Please advice Mr. Ram on the applicability of VAT and whether he is required to pay
the amount on the agreed price 5

c) From the following information, compute the amount of VAT. 5

Total cost of goods declared by importer Rs. 2,50,000

Revised cost fixed by Custom Officers

(based on prevailing International Price) Rs. 3,00,000

Freight Rs. 10,000

Insurance Rs. 2,000

Import Duty @ 10 %

d) A manufacturer purchases the taxable raw materials such as iron and steel goods.
He manufactures iron and steel goods including metal office furniture and metal
fencing. He also makes agriculture implements such as hoes and rakes. These
supplies are exempt from VAT. Details of his purchases and sales are given below:

Purchase and Overheads

Iron and Steel goods Rs. 1,00,000

Woods Rs. 40,000

Overheads (telephone and electricity) Rs. 10,000

Overheads (stationery and furniture) Rs. 25,000

Machine Parts Rs. 35,000

Total purchase excluding VAT Rs. 2,10,000

Sales

Agriculture Implements Rs. 1,00,000

Steel Appliances Rs. 1,00,000

The Institute of Chartered Accountants of Nepal

54 of 119
Suggested Answers of Income Tax and VAT
CAP II Examination – December 2009

Office Furniture and Fencing Rs. 3,00,000

Total sales excluding VAT Rs. 5,00,000

You are required to find the ratio between taxable and non-taxable sales and calculate
the value of Input Tax Credit. 5

Answer

a) Section 10 of VAT Act prescribes the registration requirement of VAT. If a person deals
with the VAT applicable items and his annual turnover is more than 2 million, then he
must be registered with VAT compulsorily. A person can register himself voluntarily with
VAT even though his VAT Applicable annual turnover is less than 2 million.

A person dealing with the goods and services exclusively covered in Sehedule I as
exempted goods, then he should not registered with VAT (Section 10(3).

In the given case, Binod and Company is engaged in the business of selling battery
operated tempo, its chassis and battery and its estimated annual turnover is Rs. 10
million. In battery operated tempo and chassis, VAT is not applicable as these items are
covered in Schedule I and only the battery is the VAT applicable item. If the turnover of
battery is 2 million or more then only Binod & Company is required to get registered
with VAT. The situation of compulsory registration as suggested by the advisor will arise
only when the turnover of battery will exceed Rs. 2 million.

If Binod & Company was engaged in the business of solar energy equipments and
operates under an umbrella of Alternative Resources Promotion Centre, it is not
required to get registered with VAT as this item is covered by schedule I of VAT Act.

b) Section 12ka of VAT Act deals about the transaction price of woods in which VAT is
applicable. At the time of auction of woods from national forest, VAT will be
recovered on the royalty amount or auction amount whichever is higher.

The Institute of Chartered Accountants of Nepal

55 of 119
Suggested Answers of Income Tax and VAT
CAP II Examination – December 2009

In case of woods were sold from community forest or from private forest, even
though royalty will not be levied, VAT will be collected as if it was sold from national
forest.

In the given case, the action taken by IRD Officer is correct and Mr. Ram has to pay
VAT on the auction price of the woods purchased from the community forest.

c) Computation of Price of imported goods for VAT purpose:

Cost of goods for custom purpose Rs. 3,00,000

Freight Rs. 10,000

Insurance Rs. 2,000

Total Rs. 3,12,000

Import duty (10%) Rs. 31,200

Total price for VAT purpose Rs.3,43,200


VAT @ 13% = Rs. 44,616

d) Total VAT paid @ 13% = 13% on Rs. 2,10,000 = Rs. 27,300

Now, ratio of taxable sale (Rs. 4,00,000) to total sales (Rs. 5,00,000) is;

4,00,000 = 4

5,00,000 5

He can claim 4/5th of the VAT paid on purchase. Hence, value of Input Tax credit
= 4/5th of Rs. 27,300 = Rs. 21,840.

19. Explain the provisions of the Value Added Tax Act relating to the following: (4×5=20)
a) VAT credit on the damaged goods.
b) VAT credit facility in case of used goods.
The Institute of Chartered Accountants of Nepal

56 of 119
Suggested Answers of Income Tax and VAT
CAP II Examination – December 2009

c) Tax Assessment by Tax Officer.


d) Provisions relating to refund of VAT and refund of VAT to tourists.

Answer
a) Section 16 Kha of VAT Act describes the conditions to take VAT credit on the goods
damaged by fire, burglary, accident etc. Rule 39 Ka of VAT Regulation, 2053 prescribes
the procedures to get the VAT credit on such damaged goods. An application shall be
filed with Inland Revenue Office within 30 days of fire, accident etc. explaining the proof
of loss due to the sale in lower price or write off those goods due to loss. On the receipt
of application, Inland Revenue Department will form a committee to examine the fact
and the finding of the committee proves the loss then VAT credit is allowed on such
goods. If the total amount of VAT credit is upto Rs. 1,00,000 then the concerned Inland
Revenue Officer can allow the credit and for amount of VAT credit above 1,00,000, the
concerned Inland Revenue Office shall recommend for allowing the VAT credit to
Department.

On the receipt of recommendation, IRD can decide to allow the VAT credit on the excess
amount of VAT paid above the VAT collected on sales in lesser value such goods. In case
of insured goods, the concerned Inland Revenue Office can decide to allow the VAT
credit after deducting the amount of insurance claim received.

Answer to Question No. 6 (b)

Rule 33 of VAT Regulation, 2053 prescribes the method of determining the tax in case of
used goods. Tax is assessed only on the saving between the selling price and the
purchase price of such used goods. The seller shall maintain the following permanent
record at the time of purchase or sale of such goods:

i. Purchase related
- Date of purchase
- Full description of goods
- Purchase price excluding VAT
- Rate of tax
- Amount of tax
- Total amount paid

ii. Sales related


- Date of sale
- Sale price excluding VAT

The Institute of Chartered Accountants of Nepal

57 of 119
Suggested Answers of Income Tax and VAT
CAP II Examination – December 2009

- Difference between purchase price and selling price


- Rate of tax
- Amount of tax
- Total amount received

In case, purchase price of used good is more than Rs. 10,000, then separate record
(purchase and sales) for each goods shall be maintained. If the record maintained is
found to be unsatisfactory, then tax officer can assess the tax on the total sale price of
the goods sold by such tax payer and order to deposit the tax with the next tax return.

Section 17 (5) prescribes the about the facility of VAT credit on the transaction of used
goods.

In case, used goods were purchased from the unregistered person and used for personal
purposes also though it was purchased from registered person, then no VAT credit shall
be allowed on such purchases (Rule 44).

Answer to Question No. 6 (c)

Section 20 of the Value Added Tax Act 2052 describes the circumstances where the tax
officer may assess tax at his own.

As per section 20(1), the tax officer can assess tax at his own in the following
circumstances:

i. Tax return not filed within the prescribed time.


ii. Incomplete or erroneous tax return filed.
iii. Submitted the false statement.
iv. Tax officer has reasons to believe that the amount of tax has been shown lesser or
incorrect.
v. Tax officer has reasons to believe that supply has been made under invoiced.
vi. When the supply of under invoiced goods has been made to group companies.
vii. Person having the obligation for the registration does the transaction without
registration.
viii. Invoice not issued at the time of sale.
ix. Tax was collected by an unregistered person.
The Institute of Chartered Accountants of Nepal

58 of 119
Suggested Answers of Income Tax and VAT
CAP II Examination – December 2009

As per section 20(2), the tax officer has to assess tax taking one or more of the following basis.

i. Proof of the transaction.


ii. Tax Scrutiny Report submitted by the concerned tax officer relating to the transaction.
iii. Payment of tax made by other persons on the transaction of similar nature.
Tax assessment under this section shall be made within four years from the date of
submission of the tax return. If the tax is not assessed within above four years period, the tax
return filed by the assessee shall be considered as final. But, if the tax has been evaded by the
act of fraud submitting false accounts, invoices or other documents, IRD can assess the tax at
any time. While assessing the tax under this section, the tax payer shall be given a period of 15
days to submit the clarification/explanation to the tax officer.

Answer to Question No. 6 (d)

Refund of Tax

Section 25 of VAT Act 2052 prescribes the conditions of refunding tax.

As per Section 25(1), the amount of tax will be refunded upon the receipt of application to
refund the tax within the period of 3 years from the date of transaction in the following
circumstances:

i. If Government of Nepal, Foreign Ministry has given diplomatic status to any diplomat
working in any regional, international mission or organization who is given tax
exemption facility, then tax exemption shall be provided to such diplomat upto the limit
of the tax paid on the consumption by him.
ii. The amount of tax paid by the international organization which is given tax exemption
facility by the Ministry of Finance, Government of Nepal.
iii. Tax paid by the projects conducted in Nepal under bilateral or multilateral agreement,
which was given tax exemption facility by the ministry of Finance, Government of Nepal.
iv. Tax recovered by mistake.
Not withstanding anything contained in Section 25(1), tax shall not be refunded if the amount of
one time purchase is less than Rs. 1,500 by the diplomatic missions and diplomat (Section 25
(1Ka).

Tax shall be refunded to the ultimate payer of tax in case tax was collected by mistake.

The Institute of Chartered Accountants of Nepal

59 of 119
Suggested Answers of Income Tax and VAT
CAP II Examination – December 2009

Tax Refund to Tourist

Section 25 Ka prescribes the conditions to refund the tax paid in Nepal by the tourists. The
following conditions shall be met to refund the tax to the tourists:

- The tourist shall return from Nepal by air route,


- The amount of purchase shall be more than Rs. 15,000,
- The tourist has paid the applicable tax on such purchases,
- 3% of the refund amount shall be deducted as service charge, and
- The procedures prescribed by the department have been followed to claim tax refund.

The Institute of Chartered Accountants of Nepal

60 of 119
The Institute of Chartered Accountants of Nepal
Suggested Answers of Advanced Accounting

CAP II Examination – December 2010


Marks

Attempt all questions. Working notes should form part of the answer.

1. A & B are in partnership, sharing profits & losses in the ratio of two third and one third
respectively. The books are kept on single entry system; and their statement of affairs
dated 31st Ashadh 2066 showed their position as follows:

Statement of Affairs of A & B as at 31st Ashadh 2066 Amount in Rs.


Fixed Assets:
Building 60,000
Machinery 20,000
Furniture & Fixtures 5,000 85,000

Working Capital:
Current Assets:
Debtors 60,000
Stock 40,000
Cash & Bank 5,000
Bills Receivables 15,000 120,000

Current Liabilities:
Loan 10,000
Creditors 30,000
Bills Payable 25,000 65,000 55,000

Totals 140,000
Represented By:

Capital:
A 100,000
B 40,000 140,000
Additional information are as follows:
On 32nd Ashadh 2067, the books disclosed following information:
Debtors Rs.80, 000 Creditors on open accounts Rs. 85,000
Cash Rs.8, 000 Creditors for Loan Rs.16,000

The stock was valued at Rs. 42,000 and the bills receivable amounted Rs. 14,000. An
examination of cash book showed that during the year, A had drawn on account of profit
Rs. 15,000 and B Rs. 6,000. A had, in addition, withdrawn Rs.20,000 from his capital
account on 30th Poush 2066.
The partners agreed to reduce the existing valuation of Machinery by 5 % and the
Furniture & Fixtures by 10% by way of depreciation. They also agreed to charge 5 % by
way of interest on capital.
Required:
a) Prepare statement of profit, dividing the balance between A and B for the year ended
32nd Ashadh 2067; and
b) Prepare statement of affairs showing the position as at 32nd Ashadh 2067. (10+10=20)
Suggested Answers of Advanced Accounting
CAP II Examination – December2010
Answer No.1
Statement of Profit & Loss of A and B for the year ended 31stAshad 2067

Figures in Rs.
Combined Closing Capital ( WN 1) 128,000
Add: Combined drawing during the year ( A Rs 3,5000 + B Rs 6000) 41,000
169,000
Less: Combined opening Capital ( Rs 100,000 + Rs 40,000) 140,000
Profit before adjustments 29,000
Less: Adjustments;
Depreciation on Machinery 1000
Depreciation on Furniture & Fixtures 500 1,500
Net Profit 27,500
Less : Interest on capital ( A Rs 4,500 + B Rs 2,000) 6,500
Divisible Profit 21,000
A‘s Share Two third 14,000
B‘s Share One third 7,000

Statement of Affairs as at 31stAshad 2067


Fixed Assets:
Building 60,000
Machinery 20,000
Less: Depreciation 1,000 19,000
Furniture& Fixtures 5,000
Less: Depreciation 500 4,500 83,500
Working Capital:
Current Assets:
Debtors 80,000
Stock 42,000
Cash & bank 8,000
Bills Receivables 14,000 144,000
Current Liabilities:
Loan 16,000
Creditors 85,000 101,000 43,000
Total 126,500
Represented By:
Capital
A ( WN 2) 83,500
B ( WN 2) 43,000 126,500

Working Note 1:
Ascertainment of Combined Closing Capital (before adjustments)

Fixed Assets: Rs. Rs.


Building 60,000
Machinery 20,000
Furniture& Fixtures 5,000 85,000
Working Capital:
Current Assets:
Debtors 80,000
Stock 42,000
Cash & bank 8,000
The Institute of Chartered Accountants of Nepal
2 of 97
Suggested Answers of Advanced Accounting
CAP II Examination – December2010
Bills Receivables 14,000 144,000
Current Liabilities:
Creditors for Loan 16,000
Creditors on open account 85,000 101,000 43,000
Total 128,000
Represented By:
Capital ( Balance figure) 128,000

Working Note: 2
Partners‘ Capital A/C
Particulars A (Rs.) B (Rs.) Particulars A (Rs.) B (Rs.)
To Bank (Capital 20,000 - By Balance b/d 100,000 40,000
drawn) By Interest on 4,500 2,000
To Bank (Drawings) 15,000 6,000 Capital
To Bank c/d 83,500 43,000 By Share on Profit 14,000 7,000
118,500 49,000 118,500 49,000
Working Note: 3
Interest on capital:
A: On Rs.100,000 @ 5% for ½ years = Rs. 2,500 and on Rs 80,000 @5% for ½ years= Rs
2,000
B: On Rs. 40,000 @ 5% for 1 years = Rs 2,000

2.
a) Textile Ltd. prepares separate Departmental Profit and Loss Accounts. The nature of
their operation required frequent supply of articles/ services from one department to
another. The company consisted of three departments- A, B and C. It had been decided
that A department will charge, for services supplied to other departments, the cost
thereof plus 10% thereon. Likewise B department will charge the other departments
cost plus 20% thereof in respect of supplies made to them. The C department‘s supplies
to the other departments will be charged at the prevailing rates applicable to outsiders.
The accounts for the year ended on Ashadh 32, 2067 has been closed without taking
into account the interdepartmental transactions.
Following are the departmental transactions:
Rs.
i) Cost of A Department‘s service extended to :B 8,400
C 4,500
ii) Cost of supplies made by B Department to: A 29,800
C 5,400
iii) Value of supplies made by C Department to: A 400
B 5,600

In addition, the following are the charges to be made for interchange of staffs from one
department to another for temporary period during the year:
B‘s staff shifted to A Department 4,400
A‘s staff shifted to B Department 1,100
Required:
Show the net variation in the departmental Profit and Loss Accounts as a result of
above adjustments. 10

The Institute of Chartered Accountants of Nepal


3 of 97
Suggested Answers of Advanced Accounting
CAP II Examination – December2010
b) The trial balance of Duplex Ltd. as at year ended on 31st March, 2010 shows the
following items:
Dr. (Rs.) Cr. (Rs.)
Advance payment of income-tax 220,000 -
Provision for income tax for the year ended 31.3.2009 - 120,000
The following further information are given :
i) Advance payment of income tax includes Rs. 140,000 for the year 2008/09.
ii) Actual tax liability for the year 2008/09 amounts to Rs. 152,000 and no effect for
the same has so far been given in accounts.
iii) Provision for income tax has to be made for the year 2009/10 for Rs. 160,000.
Required:
(a) Prepare provision for income-tax account,
(b) Prepare advance payment of income tax account,
(c) Prepare liabilities for taxation account and
(d) Show how the relevant items will appear in the profit and loss account and
balance sheet of the Company. (2+2+2+4=10)

Answer No.2
a)
Profit & Loss (Adjustment) Account

Particulars A Rs. B Rs. C Rs. Particulars A Rs. B Rs. C Rs.


To Services from A 9,240 4,950 By Services from A to B 9,240
To Supplies from B 35,760 6,480 By Services from A to C 4,950
To Supplies from C 400 5,600 By Services from B to A 35,760
To Charge in respect of
staff 4,400 1,100 By Services from B to C 6,480
To Increase in Dept.
Profit 30,700 By Services from C to A 400
(or Decrease in Dept.
Loss) By Services from C to B 5,600
By Recovery in respect
of Staff 1,100 4,400
By Decrease in Dept.
Profit
(or Increase in Dept.
Loss) 25,270 5,430

40,560 46,640 11,430 40,560 46,640 11,430

Notes:
10% has been added to cost of A dept. services to find out transfer price for B and C. 20% has been
added to costs of supplies of B dept. to find out transfer price for A and C dept.

The Institute of Chartered Accountants of Nepal


4 of 97
Suggested Answers of Advanced Accounting
CAP II Examination – December2010
Working Note:
Statement showing transfer price

From Dept. To Dept. Cost/ Value (Rs.) Transfer Price (Rs.)


A B 8,400 9,260
A C 4,500 4,950
B A 29,800 35,760
B C 5,400 6,480
C A 400 400
C B 5,600 5,600

b)

Duplex Ltd.
(a) Provision for Income Tax (2008/09) Account
Dr. Cr.
Rs. Rs.
31.3.10 To Advance Payment of 1.4.09 By Balance b/d 120,000
Income-tax A/c 140,000 31.3.10 By Profit and Loss A/c 32,000
31.3.10 To Liability for (152,000-120,000)
Taxation A/c 12,000
152,000 152,000

Provision for Income-tax (2009/10) Account


Rs. Rs.
31.3.10 To Balance c/d 160,000 31.3.10 By Profit and Loss A/c 160,000
160,000 160,000
(b) Advance Payment of Income Tax Account
Rs. Rs.
31.3.10 To Balance b/d 220,000 31.3.09 By Provision for Income tax
(2008/09) A/c 140,000
By Balance c/d 80,000
220,000 220,000
(c) Liability for Taxation Account
Rs. Rs.
31.3.10 To Balance c/d 12,000 31.3.10 By Provision for
Income tax A/c 12,000
12,000 12,000
Profit and Loss Account
for the year ended 31st March, 2010 (Extracts)
Rs. Rs.
Profit before Taxation -----
Less :Taxation for the year 160,000
Taxation adjustment of previous year 32,000 192,000
Net Profit -----
Balance Sheet of Duplex Ltd.
As at 31st March, 2010 (Extracts)
Liabilities Rs. Assets Rs.
Current Liabilities and Provisions Current Assets, Loans and Advances
The Institute of Chartered Accountants of Nepal
5 of 97
Suggested Answers of Advanced Accounting
CAP II Examination – December2010
A. Current Liabilities B. Loans and Advances
Liability for Taxation (2008/09) 12,000 Advance payment of
B. Provisions Income-tax 80,000
Provision for Income-tax 160,000

3.
a)
i) What is Debt Service Coverage Ratio? 2
ii) You are given the following particulars:
Current Ratio 2
Working Capital Rs. 400,000
Capital Block to Current Asset 3:2
Fixed Assets to Turnover 1:3
Sales Cash /Credit 1:2
Creditors Velocity 2 Months
Stock Velocity 2 Months
Debtors Velocity 3 Months
Net Profit 10% of Turnover
Reserve 2.5% of Turnover
Debentures/Shares Capital 1:2
Gross Profit 25%
Required:
Prepare the balance sheet on the basis of above information. 8
b)
i) The paid-up capital of Tele Ltd. amounted to Rs.250,000 consisting of 25,000
equity shares of Rs.10 each.

Due to losses incurred by the company continuously, the directors of the company
prepared a scheme for reconstruction which was duly approved by the court. The
terms of reconstruction were as under:
In lieu of their present holdings, the shareholders are to receive:
Fully Paid Equity Shares equal to 2/5th of their holding.
5% Preference Shares fully paid-up to the extent of 20% of the above new
equity shares.
3,000 6% Second Debentures of Rs.10 each.
An issue of 2,500 5% First Debentures of Rs.10 each was made and fully
subscribed in cash.
The assets were reduced as follows:
Goodwill from Rs.150,000 to Rs.75,000.
Machinery from Rs. 50,000 to Rs.37,500.
Leasehold Premises from Rs.75,000 to Rs. 62,500.
Required:
Show the journal entries to give effect to the above scheme of reconstruction. 7
ii) Define the term ‗Indemnity Period‘ and‘ Standard Turnover‘ in relation to
calculation of insurance claim. 3

Answer No.3
The Institute of Chartered Accountants of Nepal
6 of 97
Suggested Answers of Advanced Accounting
CAP II Examination – December2010
a.i) Debt Servicing means timely payment of loan installments and interest. Normally the
borrower should be able to service debts out of profit. The profit means the amount of
profit available for debt servicing and is represented by Net Profit after tax plus
Depreciation and other non cash charges plus Interest on Debt. A huge coverage ratio
indicates the strength of the business to service its debt properly.
It is calculated as
NPAT + Depn & non cash changes +I
Loan installments + Interest
ii)
Balance Sheet

Capital & Liabilities Rs. Assets Rs.


Share Capital 600,000 Fixed Assets 800,000
Net Profit (P/L A/c) 240,000 Stock 300,000
Reserves 60,000 Debtors 400,000
Debenture 300,000 Other Current Assets 100,000
Sundry Creditors 300,000
Other Current Liabilities 100,000
1,600,000 1,600,000

Working Notes:

Current Assets (CA)


1. Current Ratio =
Current Liabilities (CL)

Or, 2 = CA/ CL
Or, CA = 2 CL
Again,
WC = CA- CL
Or, 400,000= 2CL- CL
Or, CL = Rs. 400,000
& CA = 2* Rs.400,000
CA = Rs.800,000

2. Capital Block: Current Assets = 3:2


If Current Assets = 8,00,000
Capital Block = 12,00,000

3. Debenture: Share Capital = 1:2


Debentures = Share Capital/2

4. Capital Block = Share Capital + Profit + Reserve + Debenture


Let Share Capital be X
Let Sales be Y
Net Profit = 10% of Y = 0.1Y
Reserve = 2.5% of Y = 0.025Y
Capital Block = X+0.1Y+0.025Y+0.5X
or, 1.5X+0.125Y=12,00,000-------------------(I)

5. Fixed Assets : Turnover= 1:3


The Institute of Chartered Accountants of Nepal
7 of 97
Suggested Answers of Advanced Accounting
CAP II Examination – December2010
Turnover
or, Fixed Assets =
3

Y
Fixed Assets =
3
We know, Capital funds - fixed assets = working capital and
Capital funds = Capital Block
or, 1.5X+0.125Y- Y/3 = 4,00,000
or, 4.5X -0.625Y= 12,00,000 ------------------(ii)
Solving the equation (i) and (ii) we get
X= Share Capital = 6,00,000
Y= Sales = 24,00,000

6 Y 2,400,000
a. Fixed Assets = .= .= 800,000
3 3
1
b. Cash Sales = X 2,400,000 .= 800,000
3
2
c. Credit Sales = X 2,400,000 .= 1,600,000
3
10
d. Net Profit = X 2,400,000 .= 240,000
100
2.5
e. Reserve = X 2,400,000 .= 60,000
100
25
f. Gross Profit = X 2,400,000 .= 600,000
100
Share Capital
g. Debentures = .= 300,000
2
Cost of Goods Sold = 2,400,000- 600,000 = 1,800,000

Average Stock
7. Stock Velocity = x 12 = 2
Cost of goods sold
therefore, Closing Stock = 300,000
In the absence of information here it is assumed that average stock = closing stock

Average Debtors
8. Debtor Velocity = x 12 = 3
Credit Sales
therefore, Closing debtors = 400,000
In the absence of information here it is assumed that average debtors = closing debtors

Other current Assets = 800,000-400,000 -300,000 = 100,000

Average Creditors
9. Creditors Velocity= x 12 = 2
Credit Purchases
therefore, Closing Creditors = 300,000
In the absence of information regarding cash and credit purchase and opening and closing
stock, cost of goods sold is considered.
Other current liabilities = 400,000-300,000 = 100,000

The Institute of Chartered Accountants of Nepal


8 of 97
Suggested Answers of Advanced Accounting
CAP II Examination – December2010
b.i) (3+2+3 marks
In the books of Tele Ltd.
Journal Entries
Rs. Rs.
Share Capital A/c (Old) Dr. 250,000
To Equity Share Capital A/c 100,000
(2/5 of Rs. 2,50,000)
To 5% Pref. Shares Capital A/c 20,000
(20/100xRs.1,00,000)
To 6% Second Debentures A/C 30,000
To Capital Reduction A/c 100,000

(Being conversion of 25,000 Equity Shares and balance being transferred to


Capital Reduction A/c in accordance with the Scheme of internal
reconstruction as per Special Resolution dated ----- as confirmed by the
Court Order dated---)

Bank A/c Dr. 25,000


To 5% First Debenture A/c 25,000
( Being issue of Rs.25,000 5% First Debentures for cash as per scheme of
internal reconstruction)

Capital Reduction A/c Dr. 100,000


To Goodwill A/c 75,000
To Plant & Machinery A/c 12,500
To Leasehold Premises A/c 12,500
(Sundry Assets written down as per scheme of internal reconstruction)

ii) Indemnity Period


The period beginning with the occurrence of the Damage and ending not later than 12
months thereafter during which the results of the business shall be affected in
consequence of the damage.

Standard Turnover
The turnover during the period in the twelve months immediately before the date of
damage which corresponds with the indemnity period to which such adjustments shall
be made as may be necessary to provide for the trend of the business and for variations
in or special circumstances affecting the business either before or after the damage.

4.
a) State how the Premium Income, Claim Expense and Commission Expense are
computed in case of insurance company. 5

b) Following facts have been extracted from the record of Adarsha Bank Ltd. in respect of
the year ending Ashadh 32, 2067:
On 1-4-2066 Bills for collection were Rs.700,000. During 2066/067 bill received for
collection amounted to Rs. 6,450,000, bills collected were Rs. 4,700,000 and bills
dishonoured and returned were Rs.550,500.
Required:
The Institute of Chartered Accountants of Nepal
9 of 97
Suggested Answers of Advanced Accounting
CAP II Examination – December2010
Prepare Bills for Collection (Assets) A/C and Bills for Collection (Liability) A/C. 5

c) Following is the cash flow abstract of Alpha Ltd. for the year ended 31st March, 2008:
Inflows Rs. Outflows Rs.
Opening balance: Payment to creditors 90,000
Cash 10,000 Salaries and wages 25,000
Bank 70,000 Payment of overheads 15,000
Share capital 500,000 Fixed assets acquired 400,000
Collection from Debtors 350,000 Debentures redeemed 50,000
Sale of fixed assets 70,000 Bank loan repaid 250,000
Taxation 55,000
Dividends 100,000
Closing balance:
Cash 5,000
Bank 10,000
1,000,000 1,000,000
Required:
Prepare Cash Flow Statement for the year ended 31st March, 2008 in accordance with
Nepal Accounting Standard. 5

Answer No.4
Premium Income:
Premium income should be recognized on cash basis. If the date of premium payment is later
than cash received, the income should be recognized on later date.
Premium on re-insurance accepted may be recognized on accrual basis.
If premium amount is received for more than one year, the amount relating to first year should
be recognized as income and the remaining amount should be treated as deposit. The amount
of deposit should be proportionately treated as income in later years. But, in case of one time
premium, the total amount should be recognized as income. The amount of premium income to
be credited to revenue account for a year may be computed as:

Premium Received on risks undertaken during the year


Direct Insurance
Yearly premium xxxxx
Renewal premium xxxxx
One-time premium xxxxx xxxxx
Re-insurance accepted xxxxx
xxxxx
Less: Premium on re-insurance ceded xxxxx
Premium Income xxxxx

Note:
The above computational and revenue recognition procedure is same for both cases, i. e. for
life insurance business and for non-life insurance business.

Claim Expense:
A claim occurs when a policy falls due for payment. In the case of life insurance business, it will arise
either on death or maturity of policy that is on the expiry of the specified term of years. In the case of
general insurance business, a claim arises only when the loss occurs or the liability arises. The amount
of claim to be charged to revenue account may be worked out as under:
Life Insurance
Claims settled during the year (including legal fees, survey, consultancy charges etc.)
Death claim xxxxx
Maturity claim xxxxx

The Institute of Chartered Accountants of Nepal


10 of 97
Suggested Answers of Advanced Accounting
CAP II Examination – December2010
Partial maturity claim xxxxx
Surrender claim xxxxx
Other claim xxxxx
Total claim paid xxxxx
Adjustment : Share of re-insurance in total claim xxxxx
Net claim expenses xxxxx

Non-life insurance
Claims settled during the year (including legal fees, survey, consultancy charges etc.)
Claim of current year xxxxx
Claim prior to one f.y. xxxxx
Claim prior to two f.y. xxxxx
Claim prior to three f.y. xxxxx
Claim prior to four or more f.y. xxxxx
Total claim paid xxxxx
Adjustment : Share of re-insurance in total claim xxxxx
Net claim expenses xxxxx

Commission Expenses: In insurance business received through agents, the agency commission
expenses should be booked in the year in which the related premium income is
recognized.Commission expense to be charged to revenue account is computed as follows:
Commission on first year premium xxxxx
Commission on Renewal premium xxxxx
Commission on One-time premium xxxxx
Commission expenses xxxxx

b)
Bills for Collection (Assets) A/c
Date Particulars Rs. Date Particulars Rs.
01/04/2066 To balance b/d 700,000 2066/067 By Bills for collection
2066/067 To Bills for collection (Liabilities) A/c 4,700,000
( Liabilities) A/c 6,450,000 By Bills for collection
(Liabilities) A/c 550,500
32/3/2067 By Balance c/d 1,899,500

7,150,000 7,150,000

Bills for Collection (Liabilities) A/c


Date Particulars Rs. Date Particulars Rs.
2066/067 To Bills for collection 01/04/2066 By Balance b/d 700,000
( Assets) A/c 4,700,000 2066/067 By Bills for collection
To Bills for collection (Assets ) A/c 6,450,000
( Assets) A/c 550,500
32/3/2067 To Balance c/d 1,899,500

7,150,000 7,150,000

C)
Cash Flow Statement
for the year ended 31.3.2008
The Institute of Chartered Accountants of Nepal
11 of 97
Suggested Answers of Advanced Accounting
CAP II Examination – December2010
Rs. Rs.
Cash flow from operating activities
Cash received from customers 3,50,000
Cash paid to suppliers (90,000)
Cash paid to employees (salaries and wages) (25,000)
Other cash payments (overheads) (15,000)
Cash generated from operations 2,20,000
Income tax paid (55,000)
Net cash from operating activities 1,65,000
Cash flow from investing activities
Payment for purchase of fixed assets (4,00,000)
Proceeds from sale of fixed assets 70,000
Net cash used in investment activities (3,30,000)
Cash flow from financing activities
Proceeds from issue of share capital 5,00,000
Bank loan repaid (2,50,000)
Debentures redeemed (50,000)
Dividends paid (1,00,000)
Net cash from financing activities 1,00,000
Net decrease in cash and cash equivalents (65,000)
Cash and cash equivalents at the beginning of the year 80,000
Cash and cash equivalents at the end of the year 15,000

5.
a) X, a construction contractor, undertakes the construction of an industrial complex. He
has separate proposals raised for each unit to be constructed in the industrial complex.
Since each unit was subject to separate negotiation, he was able to identify the costs
and revenues attributable to each unit. Should X treat construction of each unit as a
separate construction contract? 5
b) X Ltd. received a grant of Rs. 2 crores from the Government for the purpose of
installation of special machinery during F/Y 2062/063. The cost of Machinery was Rs.
20 crores and it had a useful life of 9 years. During F/Y 2066/067, the grant has become
refundable due to non-fulfillment of certain conditions attached to it. Assuming the
entire grant was deducted from the cost of machinery in the year of acquisition, state
with reasons, the accounting treatment to be followed in the year 2066/067. 5
c) Indicate any three areas in respect of which different accounting policies may be
adopted by different enterprises. Also indicate the requirements with regard to
disclosure of accounting policies as per the relevant NAS. 5

Answer No.5
a) The provision of Nepal Accounting Standard 13 on ―Construction Contracts‖ Para 8
and 9 explains the situations where accounting segmentation and combination of
construction contracts can be applied. As per para 8 of the standard, when separate
The Institute of Chartered Accountants of Nepal
12 of 97
Suggested Answers of Advanced Accounting
CAP II Examination – December2010
proposals have been submitted, and separate negotiations have been concluded, costs
and revenues identified separately, the contract for construction of a number of assets
shall be treated as separate construction contracts. Therefore, X has to treat construction
of each unit as a separate construction contract.
b) As per para 32 of NAS 10 ―Accounting for Government Grants‖, the amount
refundable in respect of a government grant related to a specific fixed assets is recorded
by increasing the book value of the asset. Depreciation on the revised book value
provided prospectively over the residual useful life of the asset. In the given case book
value of machinery will be increased by Rs.2 crores in the year 2066/67. The
computation for the depreciation on machinery can be given as :
Cost of Machinery Rs.20 crores
Less: Grant Received Rs. 2 crores
Cost of Machinery Rs.18 crores
Useful life of Machinery 9 Years
Depreciation per year as per straight line method
(assuming residual value to be zero) Rs.18 crores/9 = Rs. 2 crores.
Total depreciation for 4 years (2062/063 to 2065/066) Rs.8 crores.
Book Value in year 2065/066 Rs.(18 – 8) Crores Rs.10 crores.
Add: Grant refunded Rs. 2 crores
Revised Book Value Rs. 12 crores
Remaining useful life 5 Years
Revised Annual Depreciation (Rs. 12 Crores/5) Rs. 2.4 crores.

Thus, book value of machinery will be Rs.12 crores in the year2066/067 and the
depreciation amounting Rs. 2.4 crores will be charged on machinery. Annual
depreciation of Rs.2.4 crores will be charged in the next four years.
c) Areas in which different accounting policies may be adopted: The following are three
areas in which different accounting policies may be adopted by different enterprises:
(i) Methods of depreciation, depletion and amortization.
(ii) Valuation of inventories.
(iii) Valuation of Fixed Assets.
(The above three areas are not exhaustive. There are other areas also)
Disclosure requirements of accounting policies :The disclosure requirements as
prescribed in Accounting Standard 1 (NAS 1) ‗Disclosure of Accounting Policies‘ are
as follows :
(i) All significant accounting policies adopted in the preparation and presentation of
financial statements should be disclosed at one place and they should form part of the
financial statements.
(ii) Any change in the accounting policies which has a material effect in the current
period should be disclosed along with the amount, to the extent ascertainable, by which
any item in the financial statement is affected. Where such amount is not ascertainable,
wholly or in part, the fact should be indicated. However, if a change in accounting
policies is reasonably expected to have a material effect in later periods, the fact of such
change should be appropriately disclosed in the period in which the change is made.
(iii) If the fundamental accounting assumptions viz .Going concern, Consistency and
Accrual are followed in the preparation of financial statements, specific disclosure is
The Institute of Chartered Accountants of Nepal
13 of 97
Suggested Answers of Advanced Accounting
CAP II Examination – December2010
not required. If a fundamental accounting assumption is not followed, the fact should be
disclosed.

6. Write short notes on: (5×2=10)

a) Sale and lease back transaction


b) Price earnings ratio
c) Concepts of capital accounting in partnership business
d) Conditions to be satisfied to capitalize the borrowing costs
e) Firm underwriting
Answer No.6
a) Sale and Lease Back Transaction
A sale and leaseback transaction involves the sale of an asset by the vendor and the leasing of
the same asset back to the vendor. The lease payments and the sale price are usually
interdependent as they are negotiated as a package. The accounting treatment of a sale and
leaseback transaction depends upon the type of lease involved.

b) Price earnings ratio


Price earnings ratio indicates sensitivity of market price of a share with its earnings per share.
Generally, low P/E ratio indicates the possibility of share price increase in future as far as the
effect of earning is considered and vice versa. It is calculated as price per share divided by
earnings per share. The price per share (numerator) is the market price of a single share of the
stock. The Earnings per share (denominator) is the Net Income of the company for the most
recent 12 month period, divided by number of shares outstanding.

c) Concept of Capital.
There are two methods of accounting- Fixed Capital Method and Fluctuating Capital Method.
In fixed capital method, generally initial capital contributions by the partners are credited to
partners‘ capital accounts and al subsequent transactions and events are dealt with through
current accounts. On the contrary, under the fluctuating capital method, no current account is
maintained. All such transactions and events are passed through capital accounts.

d) Capitalization of borrowing costs as part of the cost of a qualifying asset should commence
only when all the following conditions are satisfied:
1. The expenditure is being incurred for the Acquisition, construction or production of a
qualifying asset;
2. Borrowing costs are being incurred; and
3. Activities that are necessary to prepare the asset for its intended use or sale, (including
any technical or administrative work prior to the commencement of physical
construction but excluding such activities during which no production or development
take place) are in progress.
e) ‗Firm‘ underwriting‘ signifies a definite commitment to take up a specified number of
shares irrespective of the number of shares subscribed for by the public. In such a case,
unless it has been otherwise agreed, the underwriter‘s liability is determined without taking

The Institute of Chartered Accountants of Nepal


14 of 97
Suggested Answers of Advanced Accounting
CAP II Examination – December2010
into account the number of shares taken up ‗firm‘ by him, i.e. the underwriter is obliged to
take up :
1. the number of shares he has applied for ‗firm‘; and
2. the number of shares he is obliged to take up on the basis of the underwriting agreement.

The Institute of Chartered Accountants of Nepal


15 of 97
The Institute of Chartered Accountants of Nepal
Suggested Answers of Audit and Assurance

CAP II Examination – December 2010

Marks

Attempt all the questions.

1. G
ive your opinions with reason on the following cases: (45=20)

a) An auditor of New Nepal Ltd. audited and signed the accompanying financial statement
of the company as of July 15, 2010 (Corresponding to 31st Ashad, 2067). Date of the
signature of the audit report was August 16, 2010.

b) Shareholder of ABC Ltd. appointed Goddar & Co. as statutory auditor for last financial
year. During the audit, auditor found that previous auditor had issued qualified audit
report in respect of assets which was overstated by Rs.15 Million and a director‘s capital
account was credited by that amount. After series of discussion, current auditor reached
in to conclusion that he was appointed for the current financial year and he is not
responsible for the misstatement in previous year which is already reported by previous
auditor.

c) STU Bank Ltd. appointed you as due diligence auditor to find reason of loss in
international Wool Ltd. The company imports woolen threads and manufactures and
exports woolen sweaters to Wall Mart. Profit margin in such business after input vat
refund is 26%. International Wool Ltd. imported woolen raw materials of Rs. 200 million
and sales was 220 million in F/Y 2009/10. Cost of raw materials is inclusive of all
material input. Opening and closing stock remained the same for the year. How would
you plan your audit work and what area would you focus?

d) XYZ Hotels Ltd. has incurred loss and one of its illiterate director wants your assistance
to find out exactly what happened in the company. He suspects that the company has paid
excessive interest on its borrowing. He has found average of 12 month end borrowing
balance was Rs. 120 million and prevailing interest rate was 10% for the year. Interest
paid for 12 month was Rs. 12 million. Suggest to the director on this matter how he can
verify this matter.
Answer No.1
a)
Audit of financial statement is normally carried out for a full financial year. Financial
year as per the prevelling law is first of Shrawan to the end of Ashad month as per Nepali
calendar. Date of the audit report should be after the date of financial year end and after
completation of audit functions. However, in above mentioned case, date of financial
statements is July 15,2010,which is 1 day short of actual financial year end i.e. July
16,2010( corresponding to 32 ashad 2067).This may happen in following situations:
a) An assurance assignment was for audit of financial statements of that date.
b) Company ended its operation from that date.
c) Company is merged to another company from that date.
d) Company went in to liquidation from that date.
e) Auditor signed wrong dated financial statement.
Therefore the auditor should check out the relevant date of financial statements under
audit before signing the report.
Suggested Answers of Audit and Assurance
CAP II Examination – December2010

b) As per NSA 510 Initial Engagements- Opening Balances, for initial audit engagements,
the auditor should obtain sufficient appropriate audit evidence that (a)the opening balance
do not contain misstatements that materially affect the current period‘s financial
statements; (b) the prior period‘s closing balances have been correctly brought forward to
the current period or, when appropriate, have been restated. If the effect of the
misstatement is not properly accounted for and adequately disclosed, the auditor should
express a qualified opinion or an adverse opinion, as appropriate. Since the misstatement
is carried forward during the current year, it has affected current year position as well and
accordingly the, auditor‘s contention is not valid. Goddar &Co. should qualify his report
quantifying the misstatements unless management is ready to correct the statements.

c) Based on the information furnished, audit plan of international wool Ltd. is to obtain or
update the knowledge of the business including consideration of the entity‘s organization,
accounting systems, operating charecteristics and the nature of its assets, liabilities,
revenues and expenses. After initial assessment and knowledge about the nature of
business, areas for special attention can be identified. In this case following areas need
special attention:
1) Whether proper accounting policy is adopted or not.
2) Whether VAT input/VAT refund is excluded from the cost or not
3) Whether process loss is normal and comparable to the industry or not.
4) Whether pilferage and theft of raw material is properly controlled or not.
5) Whether raw material cost and sales value was properly accounted or not.
6) Whether other items of revenue or expenses pertaining to business are properly
recorded in the book of accounts or not.
d) Average interest cost on the average borrowing by XYZ Hotels Ltd. is matching with
prevailing interest rate. However, detailed verification of borrowing and interest expenses
booked need to be carried out as following:
1) Examine the date, rate and amount of borrowing with reference to borrowing
documents.
2) Verify booking date of the balances in individual accounts and total of borrowing
balance.
3) Examine whether there is a procedure for obtaining confirmation of balance
periodically.
4) Check calculations of interest and examine whether interest expenses has been
accurately provided for with reference to the duration of borrowing.
5) Trace the amount of borrowing in to bank account and interest and loan repayment
from bank account.
6) Examine whether borrowing is properly authorized and whether internal control
procedures have been followed.
7) Examine that other items of liability is not booked as borrowing.

2. A
nswer the following:

a) What are the basic elements of an assurance engagement? 7


b) An statutory auditor of a commercial Bank is requested to report about frauds. The
management suspects that in two loan files there could be fraud. The auditor has already
found a case of misuse of authority relating to a loan amount of Rs. 2500000 resulting
into loss to the Bank to the extent of that amount. However, the auditor was unable to
find fraud in any other loan transactions. Suggest the auditor about his course of action. 8

Answer No.2
The Institute of Chartered Accountants of Nepal
2 of 97
Suggested Answers of Audit and Assurance
CAP II Examination – December2010

a)
i) Three party relationship: Assurance engagements involve three separate parties: a
practitioner, a responsible party and intended users.
ii) An appropriate subject matter: Subject matter information may be the reorganization,
measurement, presentation and disclosers represented in financial statements, key
indicators, special documents, assertion about effectiveness or statement of compliance
depending on the nature of assurance engagements.
iii) Suitable criteria: Criteria can be formal or informal depending up on the nature of
assurance engagement. It may be Nepal Standards on Auditing or an established internal
control framework or individual control objectives specifically designed for the
engagement or applicable law, regulation or contract.
iv) Sufficient appropriate evidence: The practitioner plans and performs an assurance
engagement with an attitude of professional skepticism to obtain sufficient appropriate
evidence about whether the subject matter information is free of material misstatement.
The practitioner considers materiality, assurance engagement risk, and the quantity and
quality of available evidence when planning and performing the engagement.
v) Assurance report: The practitioner provides a written report containing a conclusion that
conveys the assurance obtained about the subject matter information.

b) The primary responsibly for the prevention and detection of fraud and error rests with
both those charged with the governance and the management of an entity. The auditor
should obtain evidence that management acknowledges its responsibility for the fair
presentation of the financial statements in accordance with the relevant financial
reporting framework, and has approved the financial statements.
Objective of an audit of financial statements is to enable an auditor to express an opinion
on such financial statements and not the detection and prevention of fraud and error.
Audit involves exercise of judgement. Also, the nature of the audit evidence enables the
auditor to draw only reasonable conclusions therefrom. Because of the inherent
limitation of an audit, there is an unavoidable risk that some material misstatements may
remain undiscovered. While in many situations, the discovery is not the main objective of
audit nor is the auditor‘s program of work specifically designed for such discovery. The
audit cannot, therefore be relied upon to ensure the discovery of all frauds or errors but
where the auditor has such indication, he should extend his procedures to confirm or
dispel his suspicions.

3. G
ive your comments on the following: (35=15)

a) Kathmandu Boarding School was established in year 2010. It recently constructed


swimming pool of 2020 meter size behind its main building. Due to Vastusastra
problem, the swimming pool was reconstructed toward 5 meter west side of the building.
Relocation of the pool incurred additional cost of 20%. Suggest, how this cost be booked
in account.

b) Management of MNO production House argues that the auditor should accept the figure
of stock verification conducted by internal audit. The management is willing to certify
this figure. The engagement partner, however, insists that stock verification should be
carried out again by management in presence of his assistants on 25% of locations.

c) PR Sharma & Co. appointed you as an assistant auditor for audit of a business
organization. You were asked to prepare internal control questionnaire for audit of
payments of the organization.
The Institute of Chartered Accountants of Nepal
3 of 97
Suggested Answers of Audit and Assurance
CAP II Examination – December2010

The Institute of Chartered Accountants of Nepal


4 of 97
Suggested Answers of Audit and Assurance
CAP II Examination – December2010

Answer No 3
a) According to NAS 6 Property, Plant and Equipment, the expenditure for relocation of
swimming pool should be capitalized. However, the carrying amount of those parts that
are replaced should be derecognized in accordance with the de-recognition provisions.

b) The auditor should obtain sufficient and appropriate audit evidence from compliance and
substantive procedures to form his opinion. Accordingly, the auditor can demand the
physical verification be done in the presence of his assistants to obtain a level assurance
of the inventory system in place. However, auditor should also consider the reliance that
may be placed in the report of internal audit in accordance with NSA 610 considering the
work of Internal Auditing. Where such reliance may be placed in the work of an internal
audit, the physical verification is not necessary.

c) Internal control questionnaire for audit of payment is as given below:

1) Are the duties related to payment segregated and rotated periodically?


2) Are the financial powers properly laid down and reasonable?
3) Have the financial powers of various authorities been intimated to various
departments?
4) Whether proper and adequate documents are maintained for each payment?
5) Is the arithmetical accuracy, price, quality, timing and other terms and conditions ( of
the contract) is ensured before approving a payment?
6) Are payments made through account payee cheques except in exceptional cases?
7) Are bills marked as ‗paid‘ once the payment has been made? Are there adequate
controls to ensure that payment is not made twice against the same bill?
8) Is the person/party, to whom payment is made, properly identified?
9) Is issue of cheque properly controlled? Are all cheques pre numbred? Is each cheque
duly accounted for?
10) Is there a system of sending monthly statement of accounts to suppliers and other
parties?
11) Are there adequate controls for safe custody of cheque books?

4. A
nswer the following: (35=15)

a) Ramila & Associates is a proprietorship firm of chartered accountant. Ramila, the


proprietor of the firm, was the board member of Lalima Bank Ltd till 32 Asadh, 2067.
During the AGM of Bank for the financial year 2066/67, she has been appointed as
statutory auditor for the financial year 2067/68. Should she accept the said appointment?

b) What is accounting estimates? Provide eight examples of accounting estimates.


c) What is the importance of working papers to the auditor? List out the documents to be
kept in permanent audit files.

Answer No.4
a) As per section 8.4 of Code of Ethics issued by ICAN, ―when professional accountants in
public practice are or were, within the period under current review or immediately
preceding an assignment: a) a member of the board, an officer or employee of a
company; or b) a partner of or in the employment of, a member of the board or an officer
or employee of a company. They would be regarded as having an interest which could
detract from independence when reporting on that company‖.
The Institute of Chartered Accountants of Nepal
5 of 97
Suggested Answers of Audit and Assurance
CAP II Examination – December2010

Accordingly as per above Para, CA Ramila was board member of the bank immediately
preceding an assignment and the period is not less two years, hence she should not accept
this assignment as statutory auditor for the financial year 2067/68.

b) Accounting estimate means an approximation of the amount of an item in the absence of


a precise means of measurement. On this ICAN has issued NSA 540. The examples are:
i. Allowances to reduce inventory and accounts receivable to their estimated
realizable value.
ii. Provisions to allocate the cost of fixed assets over their estimated useful life.
iii. Accrued Revenue.
iv. Provision for taxation.
v. Provision for a loss from lawsuit.
vi. Insurer`s liability for outstanding claim.
vii. Losses on construction contracts in progress.
viii. Amortization of certain items like goodwill and deferred revenue expenditure.

c) Working papers are the property of the auditor. In view of the importance of working
papers to the auditor, ICAN has issued NSA 230/Documentation. The importance of
working papers are: i.) to assist in the planning and performance of the audit, ii.) to assist
in the supervision and review of the audit work and iii.) to provide evidence of the audit
work performed to support the auditor`s opinion.
Working paper files are generally divided in to two types as per nature of documents;
namely permanent working paper files (permanent audit files) and current working paper
files (current audit files). The contents of permanent audit files are:
i. information concerning the legal and organizational structure of the client. In the
case of a company, this includes the Memorandum and Articles of Association. In
case of a statutory corporation, this includes the Act and Regulations, under which
the corporation functions,
ii. extracts or copies of important legal documents, agreements and minutes relevant
to the audit,
iii. a record of the study and evaluation of the internal controls related to the
accounting system. This might be in the form of narrative descriptions,
questionnaires or flow charts, or some combination thereof,
iv. copies of audited financial statements of previous years,
v. analysis of significant ratios and trends,
vi. copies of management letters issued by the auditor, if any,
vii. record of communication with the retiring auditor, if any before acceptance of the
appointment as auditor,
viii. notes regarding significant accounting policies and
ix. significant audit observations of earlier years.

5. A
nswer the following: (35=15)

a) Define the concept of internal control and explain its inherent limitations.
b) List out the circumstances, where auditing is done through the computer.
c) Briefly explain the concept of Materiality.
Answer No.5

(a) The concept of internal control may be defined as the plan of organization and all the
methods and procedures adopted by the management of an entity to assist in achieving
The Institute of Chartered Accountants of Nepal
6 of 97
Suggested Answers of Audit and Assurance
CAP II Examination – December2010

management‘s objectives of ensuring the orderly and efficient conduct of its business,
including adherence to management policies, the safeguarding of assets, prevention and
detection of fraud and error, the accuracy and completeness of the accounting records,
and the timely preparation of reliable financial information. The system of internal
control extends beyond those matters which relate directly to the functions of the
accounting system and comprises control environment and control procedures. Internal
control is an essential prerequisite for efficient and effective management of any
organization. It is thus, a primary responsibility of every management to establish and
maintain an adequate system of internal control appropriate to the size and nature of the
business of the entity.
An internal control system can provide only reasonable, not absolute, assurance that the
management‘s objectives in establishing the system are achieved. This is because there
are some inherent limitations of internal control. These limitations are mentioned
hereunder:
(a) Controls have to be cost effective. Hence, some control mechanisms may not have
been implemented merely because they are not cost-effective.
(b) Most control tools are directed at transactions of a usual nature. Therefore,
transactions of unusual nature might have been escaped from such controls.
(c) The human error potentiality prevails everywhere, even in the control systems.
(d) Any system of control has its limitations in preventing frauds through collusion
between two or more persons.
(e) Controls may not change with the pace of changes in conditions.
(f) Management itself may manipulate transactions or accounting estimates.
(g) A member of management may himself override the control system.

b) The auditor can use the computer to test the logic and controls prevalent within the system
and the records generated by the system. Depending upon the complexity of the
application system being audited, the approach may be different. It may be fairly simple or
may require extensive technical competence on the part of the auditor. Circumstances that
may cause to conduct audit through the computer are as follows:
(a) The application system processes large volume of input and produces large volumes
of output that make extensive direct examination of the validity of input and output
difficult.
(b) Significant parts of the internal control system are embodied in the computer system.
(c) The logic of the system is complex and there are large portions that facilitate use of
the system or efficient processing.
(d) Because of cost-benefit considerations, there are substantial gaps in the visible audit
trail.

c) Materiality is an important consideration for an auditor to evaluate whether the financial


statements reflect a true and fair view or not. The auditor should consider materiality and its
relationship with audit risk when conducting an audit. The Nepal Accounting Standards Board's
"Framework for the Preparation and Presentation of Financial Statements" defines Materiality in
the following terms: "Information is material if its omission or misstatement could influence the
economic decisions of users taken on the basis of the financial statements. Materiality depends
on the size of the item or error judged in the particular circumstances of its omission or
misstatement. Thus, materiality provides a threshold or cut-off point rather than being a primary
qualitative characteristic which information must have if it is to be useful."
In designing the audit plan, the auditor establishes an acceptable materiality level so as to detect
quantitatively material misstatements. However, both the amount (quantity) and nature (quality)
of misstatements need to be considered. Examples of qualitative misstatements would be the
The Institute of Chartered Accountants of Nepal
7 of 97
Suggested Answers of Audit and Assurance
CAP II Examination – December2010

inadequate or improper description of an accounting policy when it is likely that a user of the
financial statements would be misled by the description, and failure to disclose the breach of
regulatory requirements when it is likely that the consequent imposition of regulatory restrictions
will significantly impair operating capability.
The auditor should consider the possibility of misstatements of relatively small amounts that,
cumulatively, could have a material effect on the financial statements. For example, an error in a
month end procedure could be an indication of a potential material misstatement if that error is
repeated each month.
The auditor should also consider materiality at both the overall financial statement level and in
relation to individual account balances, classes of transactions and disclosures. Materiality may
be influenced by considerations such as legal and regulatory requirements and considerations
relating to individual financial statement account balances and relationships. This process may
result in different materiality levels depending on the aspect of the financial statements being
considered.
While determining the nature, timing and extent of audit procedures; and evaluating the effect of
misstatements, the auditor should consider the materiality. When planning the audit, the auditor
should consider what would make the financial statements materially misstated. The auditor's
assessment of materiality, related to specific account balances and classes of transactions, helps
the auditor decide such questions as what items to examine and whether to use sampling and
analytical procedures. This enables the auditor to select audit procedures that, in combination,
can be expected to reduce audit risk to an acceptably low level. There is an inverse relationship
between materiality and the level of audit risk, that is, the higher the materiality level, the lower
the audit risk and vice versa.
The auditor in addition to exercising professional judgment should consider any legislation or
regulation which may impact that assessment while assessing materiality in the case of the public
sector. In the public sector, materiality is also based on the ―context and nature‖ of an item and
includes, for example, sensitivity as well as value. Sensitivity covers a variety of matters such as
compliance with authorities, legislative concern or public interest.

6. W
rite short notes on the following (Any Two): (25=10)

a) Permanent Audit File.


b) Impairment of assets.
c) Cut off Procedure.
d) Performance Audit.
Answer No.6
(a) In a recurring audit, the file of working papers that are relevant to more than one audit
engagement or core documents are often kept separately in a file known as permanent
audit file. Permanent audit file is updated regularly with information of continuing
importance to succeeding audit. As per NSA 230-Audit Documentation a permanent
audit file normally includes:
i. Information concerning the legal and organizational structure of the client. In the
case of a company, this includes the memorandum and articles of association. In
the case of a statutory corporation, this includes the Act and Regulations under
which the corporation operates.
ii. Extracts or copies of important legal documents, agreements and minutes relevant
to the audit.

The Institute of Chartered Accountants of Nepal


8 of 97
Suggested Answers of Audit and Assurance
CAP II Examination – December2010

iii. A record of the study and evaluation of the internal controls related to the
accounting system.
iv. Copies of audited financial statements of previous years.
v. Analysis of significant ratios and trends.
vi. Copies of management letters issued by the auditor if any.
vii. Record of communication with the retiring auditor, if any before acceptance of
the appointment as a auditor and
viii. Notes regarding significant accounting policies.

b) A
n asset is impaired when the carrying amount of the asset exceeds its recoverable amount.
The difference between the carrying amount of an asset and recoverable amount is
termed as impairment loss. Besides charging annual depreciation on assets by the reason
of normal wear and tear, afflux ion of time and obsolescence to reinstate the correct value
of the assets considering the future cash flows that the asset can generate, impairment
loss needs to be provided. Some indications that an asset might have been impaired are
mentioned as follows:
(a) S
ignificant changes with an adverse effect on the entity have taken place during the
period, or will take place in the near future, in the technological, market,
economic or legal environment in which the entity operates or in the market to
which an asset is dedicated.
(b) T
he carrying amount of the net assets of the entity is more than its market
capitalization.
(c) E
vidence is available of obsolescence or physical damage of an asset.
(d)
Evidence is available from internal reporting that indicates that the economic
performance of an asset is, or will be, worse than expected.
If any of the above indications is present, an entity is required to make a formal estimate
of recoverable amount and impairment loss need to be provided.

c) It refers to the procedure adopted by the management to ensure that transactions of one
period are separated from those at the commencement of the next accounting period. The
cut off r is very significant so as to ensure that revenue and expenditures of one year do
not get recorded in the following year since it will distort the true and fair view of the
accounts. These procedures are applied to ensure that:
(a) The proper procedure has been followed for adjusting the inventory to take into
account movements to and from inventory, which have taken place between the
stock taking date and balance sheet date where stock has been taken on a date
other than the balance sheet date.
(b) Goods sold have been excluded from the inventories and credit has been taken for
the sales in the case of credit sales.
(c) Goods purchased have been in the inventories and that the liabilities have been
provided for in case of credit purchase.

d) Performance audit refers to the evaluation of the economy, efficiency and effectiveness
of selected activities of the client. In this situation, economy is concerned with
The Institute of Chartered Accountants of Nepal
9 of 97
Suggested Answers of Audit and Assurance
CAP II Examination – December2010

minimizing the cost of resources acquired or used, having regard to appropriate quality.
Efficiency refers to the relationship between the output of goods/ services or other results
and the resources used to produce them. Here, the auditor examines how far maximum
output is attained for a given input.
Similarly, effectiveness refers to the relationship between the intended results and the
actual results of the activity. Thus, the auditor examines how successfully the outputs in
the form of goods/ services or other results achieve policy objectives, operational goals
and other expected effects.
In practice, therefore, while carrying out the performance audit, the auditor looks at these
different aspects together particularly when considering the closely linked aspects of
economy and efficiency.

7. J
ustify with reason, whether following statement is true or false. (25=10)

a) G
enerally, purpose of misstatement in a financial statement of a medium sized sole
proprietorship business in Nepal is to benefit proprietor himself.
b) Audit plan is substitute for audit program.
Answer No.7
a) True: Typical medium sized business in Nepal, tend to mispresents their financial
statements to benefit their proprietor by way of lowering or reducing tax liability by
understanding income or over standing the expenses. Typical medium sized business-
man tends to lower payment of tax or do not want to pay any tax. Some time such
business man may mistake his financial statements to justify his loan application to a
bank and financial institutions. In other situations such financial statements may be
misstated to mislead the various stakeholders or users of such financial statements so that
benefits is derived by proprietor of such business.

b) The statement is false: Audit plan is initial step of audit. Whereas, audit program is
setting up of procedures that are needed to implement the audit plan. Overall audit plan is
for expected scope ans conduct of audit. On the other hand audit program shows nature,
time and extend of audit procedures.
In planning an audit, auditor should consider facteo such as complexity of the audit, the
environment in which the clint operates, his previous experience with the clint and
knowledge of clint‘s business. The audit program serves as a set of instructions to the
assistants involved in the audit and a means to control the proper execution of the work.
Therefore, audit program supplement the audit plat for execution and it does not stand out
as substitute.

The Institute of Chartered Accountants of Nepal


10 of 97
The Institute of Chartered Accountants of Nepal
Suggested Answers of Corporate and Other Laws

CAP II Examination – December 2010


Marks
All questions are compulsory.

Part: "A"

1.
a) ABC telecom Ltd., a debt free listed company, having a fully paid up equity share capital of
Rs. 15.00 billion, Cash and Bank Balance of Rs. 20.00 billion and Free reserves of Rs. 20.00
billion wants to improve the profitability of the company through reduction of capital.
Advice the company regarding the power of the company to purchase it own securities in the
light of Section 77A. (1) and (2) of the Companies Act, 1956? 8

b) XYZ Entertainment Company Limited, which commenced business in the June 2007 wants
to bid for the recently introduced Kochi Team of the Indian Premier League and for the
purpose has approached Ms. Deepika Padukone, an actress to be the Brand Ambassador for
the team for a three year period. Ms. Padukone has proposed a two tier remuneration for the
assignment namely Rs. 200.00 million in cash per year for a three year period plus a sweat
equity of at least ten percent of XYZ Entertainment Company Limited. Suggest whether the
above arrangement would be legally acceptable in the light of the provisions of Section 79A
as inserted by the Companies (Amendment) Act, 1999 to the Companies Act, 1956. 7

c) A draws a bill on B for Rs.500 payable to the order of A. B accepts the bill, but
subsequently dishonors it by non-payment. A sues B on the bill. B proves that it was
accepted for values as to Rs.400 and as an accommodation to the plaintiff as to the residue.
Do you think A can recover the whole amount from B? Discuss with the relevant provision
of Negotiable Instrument Act, 1881. 5
d) Company registered with limited liability should insert the word ‗limited‘ in its name suffix.
Can it be revoked in any condition? 5

Answer No.1
a) The provision section 77 A (1) & (2) of the Companies Act, 1956 in respect of power of
company to purchase its own securities are as follows:
1) Notwithstanding anything contained in this Act, but subject to the provisions of sub-
section (2) of this section and section 77B, a company may purchase its own shares or other
specified securities (hereinafter referred to as "buy-back") out of-
(i) its free reserves; or
(ii) the securities premium account; or
(iii) the proceeds of any shares or other specified securities:
Provided that no buy-back of any kind of shares or other specified securities shall be made
out of the proceeds of an earlier issue of the same kind of shares or same kind of other
specified securities.

(2) No company shall purchase its own shares or other specified securities under sub-section
(1) unless-
(a) the buy-back is authorized by its articles;
Suggested Answers of Corporate and Other Laws
CAP II Examination – December2010

(b) a special resolution has been passed in general meeting of the company authorizing
the buy-back:
Provided that nothing in this clause shall apply in and case where-
(A) the buy-back is or less than ten percent of the total paid-up equity capital and free
reserves of the company; and
(B) such buy-back has been authorized by the Board by means of a resolution passed at its
meeting:
Provided further that no offer of buy-back shall be made within a period of three hundred
and sixty-five days reckoned from the date of the preceding offer of buy-back, if any;
Explanation: For the purpose of this clause, the expression "offer of buy-back" means the
offer of such buy-back made in pursuance of the resolution of the Board referred in the first
proviso;
(c) the buy-back is or less than twenty five percent of the total paid-up capital and
free reserves of the company:
Provided that the buy-back of equity shares in any financial year shall not exceed
twenty-five percent of its total paid-up equity capital in that financial year;
(d) the ratio of the debt owed by the company is not more than twice the capital and
free reserves after such buy-back:
Provided that the Central Government may prescribe a higher ratio of the debt than
the specified under this clause for a class or classes of companies;
Explanation- For the purposes of this clause, the expression "debt" includes all
amounts of unsecured and secured debts;
(e) all the shares or other specified securities for buy-back are fully paid –up;
(f) the buy-back of the shares or other specified securities listed on any recognized
stock exchange is in accordance with the regulations made by the Securities and
Exchange Board of India in this behalf;
(g) the buy-back in respect of shares or other specified securities other than those
specified in clause (f) is in accordance with the guidelines as may be prescribed.

Considering the above mentioned provisions of the Companies Act the buy-back can be
arranged up to 25 percent of the paid up capital and free reserves of the company complying
the conditions as specified in the above mentioned section.
b) Section 79A Issue of sweat equity shares
(1) Notwithstanding anything contained in section 79, a company may issue sweat equity shares of
a class of shares already issued if the following conditions are fulfilled, namely:-
(a) the issue of sweat equity shares is authorized by a special resolution passed by the company in
the general meeting;
(b) the resolution specifies the number of shares, current market price, consideration, if any, and the
class or classes of directors or employees to whom such equity shares are to be issued;
(c) not less than one year has, at the date of the issue, elapsed since the date on which the company
was entitled to commence business;

The Institute of Chartered Accountants of Nepal


12 of 97
Suggested Answers of Corporate and Other Laws
CAP II Examination – December2010

(d) the sweat equity shares of a company whose equity shares are listed on a recognized stock
exchange are issued in accordance with the regulations made by the Securities and Exchange Board
of India in this behalf:
Provided that in the case of a company whose equity shares are not listed on any recognized stock
exchange, the sweat equity shares are issued in accordance with the guidelines as may be
prescribed.
Explanation I.- For the purpose of this sub-section, the expression "a company" means the company
incorporated, formed and registered under this Act and includes its subsidiary company
incorporated in a country outside India.
Explanation II.- For the purpose of this Act, the expression "sweat equity shares" means equity
shares issued by the company to employees or directors at a discount or for consideration other than
cash for providing know-how or making available rights in the nature of intellectual property rights
or value additions, by whatever name called.
(2) All the limitations, restrictions and provisions relating to equity shares shall be applicable to
such sweat equity shares under sub-section (1)

Ms. Padukone is neither employee nor director of the company. The company cannot provide Sweat
Equity Shares to Ms. Padukone, however for other cases, it can be negotiated with her.
c) Negotiable Instrument Act, 1881
Section 44.Partial absence or failure of money consideration-
When the consideration for which a person signed a promissory note, bill of exchange or cheque
consisted of money, and was originally absent in part or has subsequently failed in part, the sum
which the holder standing in immediate relation with such signer is entitled to receive from him is
proportionally reduced.
Explanation—The drawer of bill of exchange stands in immediate relation with the acceptor. The
maker of the promissory note, bill of exchange or cheque stands in immediate relation with the
payee, and the endorser with his endorsee. Other signers may by agreement stand in immediate
relation with a holder.
In the light of above provision A can only recover Rs.400.

d) According to provision of Sec. 25 of the Company Act, 1956 the suffix of ‗limited‘ may be
revoked by the central government in following conditions:
i. It should be a limited company.
ii. The objectives should be any of promoting commerce, art, science, religious, charity,
and any other useful objects.
iii. It intends to apply its profit if any, or other Income in promoting its objects, and to
prohibit the payment of any dividend to its members
Then Central Government grant a licence to revoke the suffix ‗limited‘

The Institute of Chartered Accountants of Nepal


13 of 97
Suggested Answers of Corporate and Other Laws
CAP II Examination – December2010

Part: "B"

2.
a) Mr. Kushal an Accountant at Zeta Ltd. is being alleged for insider trading. Mr. Kushal
contends that the matter which is being alleged has been already published outside Nepal.
Enumerate the provisions on insider trading and information or notice deemed to have been
made public as per the Securities Act 2063. 6
b) Standard Chartered Bank Nepal has lent Rs. 8.00 crores against mortgaged collateral
securities worth Rs. 12.00 crores to M/s XYZ Ltd. Liquidation proceedings has commenced
against the company pursuant to the Companies Act, 2063. The Bank intends to auction the
property and recover its dues directly. However the other creditors of the company are
demanding that the collateral security should also be brought under the liquidation process
but the Bank does not want to go through the process. Advice the Bank regarding its rights
vis-à-vis the provisions of the Companies Act, 2063. 5
c) What demands or claims cannot be submitted before General Manager under Labor Act,
2048?

Answer No.2
a) Section 91 and Section 93 of Securities Act, 2063 stipulate the provisions on Insider trading and
information or notice deemed to have been made Public respectively.
Section 91
(1) If any person deals in securities or causes any other person to deal in securities on the basis of
any insider information or notice that are unpublished or communicates any information or notice
known to such a person in the course of the discharge of his or her duties in manner likely to affect
the price of securities such a person shall be deemed to have been committed an insider trading in
securities.
Explanation: For the purposes of this sub-section, "insider information or notice" means any such
specific kind of information or notice not published by a body corporate issuing any securities as
may be capable of affecting the price of such securities if such information or notice is disclosed.

(2) Notwithstanding anything contained in sub-section (1), any transactions already carried on shall
not be deemed to be affected at all merely by the reason that an insider trading has been committed.

Section 93
On any of the following conditions, any information or notice shall be deemed to have been made
public:
(i) If any matter has been published with intent to inform the investors and their business advisers in
accordance with the bye-laws of a stock exchange,
(ii) If there is a provision made under the law that the general public can see an information or a
notice contained in any records,
(iii) If there is a provision that any person desiring to deal in securities is escorted to the business
room of a stock exchange so that such a person can know such an information or a notice,
(iv) If there is a provision that any person desiring to obtain or see such information or notice or get
a copy thereof can obtain and see the same,
(v) If it has been communicated to any specific class, out of the general public,

The Institute of Chartered Accountants of Nepal


14 of 97
Suggested Answers of Corporate and Other Laws
CAP II Examination – December2010

(vi) If there is a provision that information can be obtained only upon payment of fees or if such
information or a notice has been published outside the State of Nepal.

Under sub clause (Vi) above, if such information or a notice has been published outside the State of
Nepal it shall be deemed to have been made public. So, the contention of Mr Kushal as the matter
which is being alleged has been already published outside Nepal is valid and he shall not be
penalized on the alleged matter.

b) Section 135 secures the rights of secured creditors as follows:


Section 135 - Right of secured creditors not to be affected:
The commencement of liquidation proceedings of a company pursuant to this Chapter shall not be
deemed to prejudice in any manner the right of the secured creditors who have lent moneys against
the security of any property of the company to enforce or otherwise deal with such secured property
under the prevailing law.

Considering the above mentioned provisions of the Companies Act 2063 the Bank can move ahead
with the auction process and recover the dues in compliance with other prevailing laws.

c) According to section 75 of the labour Act 2048, the following demand or claim shall not be
allowed to submit –
(i) Which is contrary to the Constitution of Nepal;
(ii) Which would affect other's interest due to being based on un-testified or baseless allegation;
(iii) Matter which is prejudicial to the personal conduct of any worker or employee;
(iv) Matters unrelated to the Enterprise; and
(v) Where a period of two years has not elapsed since the date of last collective agreement.
4

3.
a) State the provisions of Audit Committee as provided in the Companies Act, 2063. 5
b) Enumerate the circumstances on which a chairperson and a member may be removed from
office under the Securities Act, 2063. 5

Answer No.3
a) The following provisions are made in Companies Act, 2063 with regard to Audit committee
(Section 164)
(1) A listed capital with paid up capital of thirty million rupees or more or a company which is
fully or partly owned by the Government of Nepal shall form an audit committee under the
chairmanship of a director who is not involved in the day-to-day operations of the company and
consisting of at least three members.
(2) Any person who is a close relative of the chief executive of a company shall not be eligible to
be a member of the audit committee formed pursuant to sub-section (1).
(3) At least one member of the audit committee shall be an experienced person having obtained
professional certificate on accounting or a person having gained experience in accounting and
financial field after having obtained at least bachelor‘s degree in accounts, commerce, management,
finance or economics.

The Institute of Chartered Accountants of Nepal


15 of 97
Suggested Answers of Corporate and Other Laws
CAP II Examination – December2010

(4) The report of board of directors required to be prepared by a company shall set out a short
description of the activities of the audit committee, working policies adopted by the board of
directors to implement the suggestions, if any, given by the audit committee, the allowances or
facilities, if any, received by the members of the audit committee and the names of the members of
audit committee.
(5) The audit committee may, for inquiring into any matter, notify the managing director of the
company, chief executive of the company or other director, auditor, internal auditor and accounts
chief involved in the day-to-day operations of the company to attend its meeting; and it shall be
their duty to be present in the meeting of that committee if they are so notified.
(6) The board of directors shall implement the suggestions given by the audit committee in respect
of the accounts and financial management of the company; and where any suggestion cannot be
implemented, the board of directors shall also mention the reasons for the same in its report.
(7) Any company shall arrange for such means and resources as may be adequate for the fulfillment
of responsibilities of the audit committee; and the audit committee may fix its internal rules of
procedures on its own.
(8) The chairman of the audit committee shall be present in the annual general meeting of the
company.
(9) The audit committee shall meet as per necessity.
b) Section 12 of the Securities Act, 2063 lays down the following circumstance on which a chair
person and a member may be removed from office.
i. Where there occurs a circumstance for removal of chairperson or the member referred to in section
(ii) the Government of Nepal shall remove the chairperson & member as the case may be,
Provided that prior to making such removal, the Government of Nepal shall not deprive the
concerned person of a reasonable opportunity to defend himself/herself
ii. The chairperson and the member, as the case may be, shall be removed from his or her office in
any of the following circumstance:
a) If he or she is disqualified to be a chairperson and a member, as the case may be, pursuant to
section 11
b) If he or she commits any act contrary to interest of investors in securities or any act that may
cause loss or damage to the development of capital market
c) If he or she suffer from lack of competence to implement, or cause to be implemented such
functions required to be performed by the Board to attain the objectives of the Board pursuant to
this Act or the rules framed under this Act,
d) If he or she has been held disqualified to carry on any occupation or business by the reason
of misconduct and his or her certificate has been revoked or he or she has thus been restricted to
carry on a business.
e) If he or she remains absent from three consecutive meetings of the Board without giving a
notice.

4.
a) What are the provisions regarding quorum required for a general meeting as per the
Companies Act, 2063? 5
b) Koirala Ltd. has not sent any reports to Registrar of Companies for a long period, including
change of registered office address. Registrar of Companies issued public notice in newspaper
intending to cancellation of name from Company Roster, but the company could not answer it
The Institute of Chartered Accountants of Nepal
16 of 97
Suggested Answers of Corporate and Other Laws
CAP II Examination – December2010

because the notice published date was called for a "Nepal Bandha" and newspaper were not reached
to the company. After three years of cancellation, director of company got knowledge that their
company was cancelled before three years. What shall be the impacts of transaction done on those
periods? Can it be revive again? Explain with reference to Companies Act, 2063. 5
Answer No 4
a) Quorum for a general meeting as per Companies Act 2063 (Section 73)
(1) A quorum for the general meeting of a private company shall be as specified in the articles of
association of such company.
(2) Unless the articles of association of a public company provides for a larger number for the
quorum, no proceedings of the meeting of the public company shall be conducted unless at least
three shareholders of the total shareholders, representing more than fifty per cent of the total
number of allotted shares of that company, are present either in person or by proxy.
(3) Where a meeting cannot be held because of quorum as referred to in sub-section (2), and the
meeting is called for the second time by giving a notice of at least seven days, nothing shall prevent
the holding of such a meeting if at least three shareholders, representing twenty five per cent of the
total number of allotted shares of the company, are present either in person or by proxy.
(4) Notwithstanding anything contained elsewhere in this Section, in the case of a company
incorporated under the proviso to sub-section (2) of Section 3 or a company incorporated under sub-
section(1) of Section 173, the presence of three shareholders as mentioned in sub-section (2) or (3)
shall not be mandatory.

b)
Companies Act 2063: Power of Office to cancel registration (Section 136):

(1) The Office may cancel the registration of a company in the following circumstance:

(a) If the promoter of the company makes an application, showing a reason for the failure to
commence the business of the company, and accompanied by the prescribed fees, for the
cancellation of the registration of the company:

(b) If the company is in default in submitting to the Office the returns as referred to in Section 80 or
fails pay the fine as referred to in Section 81 for three consecutive financial years; or

(c) If based on the proofs received in the course of administration of the company, the Office has a
reasonable ground to believe that the company is not carrying on its business or the company is not
in operation.

(2) If it is required to cancel the registration of any company pursuant to Sub-section (1), the Office
shall, prior to the cancellation of registration, give a notice, accompanied by the reason for such
registration, to the concerned company.

(3) while sending a notice to a company pursuant to Sub-section (2), the notice shall be sent to the
company at its registered office or to any officer of such company if the address of the registered
office of such company is not registered with the Office or if the office of the company is not
located at the address registered and to the address of every Promoter as mentioned in the
memorandum of association of the company if even the address of such officer is not available to
the Office or is not known.

The Institute of Chartered Accountants of Nepal


17 of 97
Suggested Answers of Corporate and Other Laws
CAP II Examination – December2010

(4) A notice given pursuant to Sub-section (2) shall also be published in a national daily newspaper,
as per necessity.

(5) If the company fails to make an application, specifying the reasons that the registration of the
company should not be canceled, within two months from the date of receipt by the company of a
notice pursuant to Sub-section (2) or, despite the making of such application, the reasons specified
are not found reasonable, the registration of such company may be canceled.

(6) If the registration of a company is canceled pursuant to Subsection (5), information thereof shall
be given to the concerned directors and shall also be published in a national daily newspaper.

According to Sec. 137 of Company Act, 2063, deregistration of company by Registrar of


Companies can be challenged within 5 years from date of decision of deregistration in one case
only. In case Registrar of Company issued a notice, having reply period of 2 months intending to
deregister and the company, if not replied or replied but Registrar is unsatisfied with registration,
Registrar itself can deregister the company. In the given case of Koirala Ltd., ROC has sent the
notice, but registered office has shifted from there where it was registered. The public notices were
issued but director were unknown of it and the ROC decided to deregistration. The transactions
were done without noticed of deregistration for three years.
In those cases, any member of or creditor of company may apply for court to revive the company
within 5 years. The company can be revived upon the decision of court in this favour. If court
decided so, all the transactions were deemed as done by a normal company and valid for law. The
legal status of company shall be deemed as perpetual within the period of deregistration as normal
company.

5.
a) Mr. Dilli Ram Bajgain has been elected as a director in Lena Bank Ltd., state the provisions
on disclosure by directors as provided in the Banks and Financial Institutions Act, 2063. 5
b) What are the matters as prescribed by Audit Act, 2048 those can be audited in view of
propriety by the Auditor General? 5
c) Enumerate the classification of Industries as per the Industrial Enterprises Act, 2049. 5
Answer No.5
a) Mr. Dilli Ram has to make a disclosure under Section 22 of the BAFIA within seven days
after assuming the office of director as follows:

Section 22 - Disclosure by directors:


(1) Every director shall, no later than seven days after assuming the office of director, disclose in
writing to the bank or financial institution the following matters:
(a) If he or she or any of his or her family members has entered into or going to enter into any kind
of contract with the concerned bank or financial institution, details thereof;
(b) If he or she has any kind of interest in the appointment of the chief executive, managing
director, secretary, auditor and general manager, details thereof;
(c) Particulars of such shares or debentures in the concerned bank or financial institution or in its
holding or subsidiary company as subscribed by him or her or by his or her family;
(d) If he or she is a director of any company, details thereof;
The Institute of Chartered Accountants of Nepal
18 of 97
Suggested Answers of Corporate and Other Laws
CAP II Examination – December2010

(e) If any member of his or her family is working as an officer of the bank or financial institution,
details thereof;
(f) Such other details prescribed by the Rastra Bank as required to be disclosed by the director to the
Board.
(2) In making disclosure pursuant to sub-section (1), a copy of the written agreement, if any,
concluded between the director or his or her family member shall be submitted, and failing such
agreement, substantial and necessary matters concerning the transaction or financial interest or
involvement shall be set out.
(3) The information provided by a director pursuant to sub-section (1) shall be forwarded to the
Rastra Bank within seven days; and upon receipt of such information, the Rastra Bank shall record
the same in a separate register maintained for this purpose.

b) According to Sec. 5 of Audit Act, 2048, following are the matters to be audited in view of
Propriety by the Auditor General:
(a) On the propriety of any expenditure and its authorization, if in the opinion of the Auditor General
such expenditure is a reckless one or is an abuse of national property, whether movable or
immovable, despite that the expenditure confirms to the authorization, and
(b) On the propriety of all authorizations issued in respect of any grant of national property whether
movable or immovable, or underwriting of any revenue, or any contract, license or permits relating
to mining, forest, water resources, etc. and any other act of abandoning movable or immovable
assets of the nation.

c) Under Industrial Enterprises Act, 2049 Industries are classified as follows:

Sec 3:
(i) Manufacturing Industries: Industries which produce goods by utilizing or processing raw
materials, semi-processed materials, by products or waste products or any other goods.
(ii) Energy Based Industries: Industries generating energy from water resources, wind, solar, coal,
natural oil, gas, bio-gas or any other sources.
(iii) Agro and Forest Based Industries: Business mainly based on agriculture or forest products such
as integrated sericulture and silk production, horticulture and fruit processing, animal husbandry,
dairy industry, poultry farming, fishery, tea gardening and processing, coffee farming and
processing, horticulture and herb processing, vegetable seed farming, mushroom, vegetable farming
or vegetable processing, tissue culture, green house, beekeeping, honey production, rubber farming,
floriculture and production and forestry related business such as lease-hold forests, agro-forestry
etc.
(iv) Mineral Industries: Mineral excavation or processing thereof.
(v) Tourism Industries: Tourist lodging, motel, hotel, restaurant, resort, travel agency, skiing,
gliding, water rafting, cable car complex, pony trekking, trekking, hot air ballooning, parasailing,
golf course, polo, horse riding etc.
(vi) Service Industries: Workshop, printing press, consultancy service, ginning and bailing business,
cinematography, construction business, public transportation business, photography, hospital,
nursing home, educational and training institution, laboratory, air services, cold storage etc.
(vii) Construction Industries: Road, bridge, ropeway, railway, trolley bus, tunnel, tunnel, flying
bridge and industrial, commercial and residential complex construction and operation.

The Institute of Chartered Accountants of Nepal


19 of 97
Suggested Answers of Corporate and Other Laws
CAP II Examination – December2010

Sec 4. Cottage Industries: The traditional industries utilizing specific skill or local raw materials
and resources, and labour intensive and related with national tradition, art and culture as mentioned
in Annex 1 shall be named as cottage industries.

Sec 5. Small Industries: Industries with a fixed asset of up to an amount of thirty million rupees
shall be named as small industries.

Sec 6. Medium Industries: Industries with a fixed asset between thirty million rupees and one
hundred million rupees shall be named as medium industries.

Sec 7. Large Industries: Industries with a fixed asset of more than one hundred million rupees shall
be named as large industries.

6.
a) State the provisions on punishment as provided in Section 41 of the Nepal Chartered
Accountants Act, 2053. 5
b) When can Nepal Rastra Bank refuse to issue license to carry on financial transaction?
Discuss with the provision of BAFIA, 2063. 5

Answer No.6

a) Provisions regarding punishment as provided in the Nepal Chartered Act 2053 are as follows:

(1) A person, who carries out auditing without obtaining a Certificate of Practice,

pursuant to this Act, shall be liable of punishment with a penalty of maximum two

thousand rupees or with an imprisonment for a maximum period of three months or with

both.

(2) A person, who in contravention of Section 6 uses the name or the seal of the

Institute or exercises any type of authority bestowed to the Institute, shall be punished

with a penalty of one thousand rupees maximum on first conviction, and on any

subsequent conviction thereafter, a maximum penalty of five thousand rupees or

imprisonment for a maximum period of six months or both.

Provided that this sub-section shall not apply to the organizations or university

established under their own legislation or the units within the Institute.

The Institute of Chartered Accountants of Nepal


20 of 97
Suggested Answers of Corporate and Other Laws
CAP II Examination – December2010

(3) A person, who has not obtained a Certificate of Practice and is proved to have

signed any document in capacity of the member holding Certificate of Practice, shall be

liable to punishment with a penalty up to two thousand rupees or imprisonment for a

period of up to three months or both.

(4) A member, who commits any act contrary to the provisions of this Act or

Regulations framed under this Act other than the provisions of this section, shall be

suspended for a maximum period of five years and shall be liable of punishment with a

maximum penalty of two thousand rupees or imprisonment for a maximum period of

three months or both.

(5) A complainant who lodges a complaint, without any reasonable cause to make

complaint and it is proved that the complaint was made with an intention to harass a

member, shall be liable to punishment with fine up to one thousand rupees.

(6) The complain cases, except those to be heard under Section 14, lodged in the

Council against any member, pursuant to Section 35, shall be instituted in the concerned

Appellate Court.

b)
Power to refuse to issue license to carry on financial transactions under Section 32 of
BAFIA
(1) Notwithstanding anything contained in Section 30, the Rastra Bank may, in any of the
following circumstances, refuse to issue a license to any bank or financial institution to carry on
the financial transactions:

(a) If, in view of the existing condition and potentiality of the banking or financial sector, it does
not appear appropriate to grant a license to additional bank or financial institution to carry on
the financial transactions;
(b) If, in the light of the situation mentioned in Clause (a), for the protection of the interests of
depositors, it does not appear just and appropriate to issue a license to carry on the financial
transactions;
(c) If it does not appear that the details or requirements referred to in Sections 29 and 30 have
been completed.
The Institute of Chartered Accountants of Nepal
21 of 97
Suggested Answers of Corporate and Other Laws
CAP II Examination – December2010

(2) If there exists a situation where the license to carry on the financial transactions cannot be
issued to any bank or financial institution pursuant to Sub-section (1), the Rastra Bank shall give
a notice thereof, accompanied by the reason for the same, to the concerned bank or financial
institution within one hundred twenty days from the date of application. If the Rastra Bank has
requested for any additional details within that period, such notice shall be given within ninety
days from the date of receipt of such details.

7. Write short notes on the following: (3×5=15)


a) Substituted Agent
b) Bailment
c) Liability of guarantor in case of Negotiable Instrument Act, 2034

Answer No.7
a) Substituted agents are not sub agents. They are agents of the principal. Where the principal
appoints an agent and if the agent identifies another person to carry out the acts ordered by
principal, than the second person is not to be treated as a sub agent but only as an agent of the
original principal.

For example 'A" directs 'B' his solicitor to sell his property by auction and 'B' appoints 'C' an
auctioneer. In this regard, 'C' is an agent of 'A' and not a sub agent.

While selecting a "substituted agent" the agent is bound to exercise same amount of diligence as a
man of ordinary prudence and if he does so he will not be responsible for acts or negligence of the
substituted agent.

For example 'X' consigns goods to 'Y' a merchant for sale. 'Y' in due course employs and auctioneer
in goods to sell goods of 'X' and also allows him to receive the proceeds of sale. The auctioneer
becomes insolvent afterwards without handing over the proceeds. Here 'Y' will not be responsible to
'X' as he has discharged his duties as a man of ordinary prudence and diligence.

b) Bailment etymologically means 'handing over' or 'change of possession'. Bailment is an act


whereby goods are delivered by one person to another for some purpose, on a contract, that the
goods shall, when the purpose is accomplished be returned or otherwise disposed of according to
the directions of the person delivering them. The person who delivers the goods is the bailor and the
person to whom the goods are delivered is the bailee.
For example where 'X' delivers his car for repair to 'Y', 'X' is the bailor and 'Y' is the bailee.
The essential characteristics of bailment are:
i) Bailment is based upon a contract. Sometimes it could be implied by law as it
happens in the case of finder of lost goods.
ii) Bailment is only for moveable goods and never for immovable goods or money.
iii) In bailment possession of goods changes. Possession change can happen by physical
deliver or by any action which has the effect of placing the goods in the possession
of bailee.
iv) In bailment bailor continues to be the owner of goods as there is no change in
ownership.
v) Bailee is obliged to return the goods physically to the balor. The bailee cannot
deliver some other gods even of higher value.
The Institute of Chartered Accountants of Nepal
22 of 97
Suggested Answers of Corporate and Other Laws
CAP II Examination – December2010

c) In case the payer fails to pay the sum as indicated in the negotiable instruments like bill of
exchange or promissory notes, and then the reimbursement of payable can be done directly from
affairs of the guarantor. According to Sec. 23 of Negotiable Instrument Act, 2034, the guarantor
cannot claim that the payment firstly required to pay from the estate of main creditor.

The Institute of Chartered Accountants of Nepal


23 of 97
The Institute of Chartered Accountants of Nepal
Suggested Answers of Cost and Management Accounting

CAP II Examination – December 2010

Marks
All questions are compulsory. Working notes should form part of the answer.
Make assumptions wherever necessary.

1. Electronic Equipments Limited manufactures CD players. The management accountant of


the company has prepared the following provisional operating statement for a period:
Rs. Rs.
Sales (30,000 CD players) 375,000
Less: Other Costs (32,000 CD players)
Direct materials 128,000
Direct labour 96,000
Production overhead (64% variable and 36% fixed) 50,000
S & D overhead (75% variable with sales and 25% fixed) 20,000 294,000
Net profit for the year (prior to stock adjustment) 81,000

The following additional information was also available:


a. Fixed production overhead contained Rs. 2,500 of depreciation relating to plant &
equipment which is surplus to current requirements.
b. Over the period, 32,000 units were manufactured.
c. There was no opening stock for current period. In the preparation of the provisional
operating statement, no account has been taken of the closing stock at the end of current
period. For the purposes of internal management accounting, stocks of finished goods are
valued at variable manufacturing cost only.
The management is currently beginning to prepare the budget for the next period. There are
several factors to be considered:
(i) To make better use of the surplus plant & equipment, the company‘s technical manager
has suggested commencing manufacture of disk drives and modems for personal
computers. Disk drives will absorb Rs. 1,500 of the depreciation charge with modems
allocated the remaining Rs. 1,000.
(ii) The manufacturing and sales managers have estimated that the surplus plant &
equipment will be sufficient to produce 5,000 disk drives and 10,000 modems; and
these quantities can be sold in the current market. However, the sales manager has
insisted that, by the end of the period, the company should be carrying minimum buffer
stocks of 10% of annual production for both disk drives and modems.
(iii) Stock levels of CD players should be unchanged, but production levels are expected to
remain at the same as in the current period. The sales manager believes that the
demand for CD players will continue to grow.
(iv) The purchasing manager has forecasted that the direct materials costs of CD players
will increase by Re.1 per unit. All other variable costs of CD players will remain at the
unit cost levels incurred in the current period.
(v) For the new products, the following estimates have been made and assembled at the
projected manufacturing volume levels:
Suggested Answers of Cost and Management Accounting
CAP II Examination – December2010
Disk Drives (Rs.) Modems (Rs.)
Total variable costs:
Direct materials 15,000 10,000
Direct labour 10,000 25,000
Factory overhead 2,500 15,000
Additional fixed overhead, excluding depreciation:
Factory overhead 8,000 13,500
Selling & distribution overhead 2,250 6,750
(vi) The production manager has projected a 60% increase in the fixed element of selling &
distribution overheads (for CD players only) in the budget period. Variable selling and
distribution overheads will be incurred at 2% of sales value for both disk drives and
modems.
(vii) The market will sustain the following prices for the products for the next period:
CD Players: Rs. 15.00 /unit, Disk Drives: Rs. 12.00 /unit, and
Modems Rs. 10.00 /unit
(viii) The management accountant has estimated that the investment in fixed assets and
working capital for next period in the three product lines will be as follows:
CD Players: Rs. 325,000; Disk Drives: Rs. 80,000; and Modems: Rs. 60,000.
You are required to: (7+10+3=20)
a) Compute the break-even number of CD players for the current period.
b) Prepare a budgeted Profit and Loss Account for the next period incorporating the impact
of the introduction of the new products and identifying product profitability.
c) Compute the return on investment for each product for the next period.

Solution to Question No.1

a. Computation of the break-even number of CD players for the current period


Particulars Rs.
Sales 375,000
Less: Variable Costs (Notes)
Direct materials (100% variable) (a) 120,000
Direct labour (100% variable) (a) 90,000
Production overhead (64% variable) (b) 30,000
S & D overhead (75% variable) (c) 15,000 255,000
Contribution margin 120,000
Less: Fixed Costs
Production overhead (36% fixed) (b) 18,000
S & D overhead (25% fixed) (c) 5,000 23,000
Net profit for the year 97,000

Breakeven point
Contribution margin per unit = Rs.120,000/ 30,000 units = Rs.4
BEP= Fixed Costs/ CM per unit= Rs.23,000/ Rs.4 = 5,750 units
Notes:

The Institute of Chartered Accountants of Nepal


25 of 97
Suggested Answers of Cost and Management Accounting
CAP II Examination – December2010
(a) Direct materials & Direct labour are both given as costs of production of 32,000
units. Therefore, they require both to be scaled downwards to 30,000 units sales
for a variable costing format profit & loss account.
(b) Total factory overhead of Rs.50,000 splits into 64% variable (Rs.32,000) and
36% fixed (Rs.18,000). The variable element relates to the manufacture of 32,000
units. Again, it has to be scaled down for 30,000 units.
(c) Total selling & distribution overhead of Rs.20,000 splits into 75% variable
(Rs.15,000) and 25% fixed (Rs.5,000). The variable element is already in line
with sales and hence, there is no requirement to adjust.

b. Preparation of budgeted Profit and Loss Account for the next period
incorporating the impact of the introduction of the new products and
identifying product profitability
CD Disk
Players Drives Modems Notes
Sales Volumes (unit) 32,000 4,500 9,000
Selling Price per unit (Rs.) 15 12 10
Sales Revenue (Rs.) 480,000 54,000 90,000 (a)
Variable costs (Rs.):
Direct materials 160,000 13,500 9,000
(b & c)
Direct labour 96,000 9,000 22,500 (b
& c)
Variable production overhead 32,000 2,250 13,500 (b
& c)
Variable selling & distribution 16,000 1,080 1,800 (d)
304,000 25,830 46,800

Fixed costs (Rs.):


Factory overhead 18,000 8,000 13,500
Depreciation adjustment -2,500 1,500 1,000 (e)
Selling & distribution (+60%) 8,000 2,250 6,750
Total costs (Rs.) 327,500 37,580 68,050
Net Profit (Rs.) 152,500 16,420 21,950

Notes:
(a) The quantity of CD players produced is identical to 2000 – namely 32 000
units. Production and stocks have not changed, and therefore, the number of
units sold is identical to the units produced – namely 32 000 units.
(b) The variable costs of direct materials, direct labour and factory overhead are
increased in the budget in proportion to increase in volume of CD players
(32 000/30 000).
(c) The variable costs for disk drives and modems have been scaled down to the
actual sales levels from the production cost data provided.
(d) Variable selling & distribution costs are computed as per the additional notes.
(e) Depreciation adjustment represents a re-allocation of the depreciation on the
surplus equipment to the new product lines.

c. Computation of the Return on Investment for each product for the next
period.

The Institute of Chartered Accountants of Nepal


26 of 97
Suggested Answers of Cost and Management Accounting
CAP II Examination – December2010
CD Disk Modems
Players Drives
ROI= Net Profit/ Investment= 152,500 16,420 21,950
325,000 80,000 60,000
46.9% 20.5% 36.6%

2.
a) A factory has three production departments P1, P2 and P3 and two service departments S1
and S2. Budgeted overheads for the next fiscal year have been allocated/apportioned by
the cost department among the five departments. The secondary distribution of service
department overheads is pending and the following details are given to you:
Department Overheads apportioned/allocated Estimated level of activity
P1 Rs. 48,000 5,000 machine hours
P2 Rs. 112,000 12,000 machine hours
P3 Rs. 52,000 6,000 machine hours
Apportionment of service departments costs
S1 Rs. 16,000 P1 (20%), P2 (40%) P3 (20%), S2 (20%)
S2 Rs. 24,000 P1 (10%), P2 (60%), P3 (20%), S1 (10%)
You are required to calculate the overhead rate per machine hour of each production
department after completing the distribution of service department costs. 8

b) What is maximum and minimum stock level? Explain how these are calculated. 6

c) Two workmen BED and DEB, produce the same product using the same material. Their
normal wage rate is also the same. BED is paid bonus according to the Rowan system,
while DEB is paid bonus according to the Halsey System. The time allowed to make the
product is 100 hours. BED takes 60 hours while DEB takes 80 hours to complete the
product. The factory overhead rate is Rs.10 per man-hour actually worked. The factory
cost for the product for BED is Rs. 7,280 and for DEB it is Rs. 7,600.
You are required to find the normal rate of wages of BED and DEB. 6

Solution to Question No. 2 (a)

It is given in the question that the secondary distribution of service department overhead is
pending. The same is thus attempted by the use of simultaneous equation method.
Let the total overheads of department S1 = x and
Let total overheads of department S2 = y
Based on the given information and applying simultaneous equation, we get:
x = 16,000 + 0.1 y (i)
y = 24,000 + 0.2 x (ii)
Multiplying equation (ii) by 5, we get:
5 y = 120,000 + x, Or x = 5y – 120,000
By deducting equation (i) from equation (iii), we get:

The Institute of Chartered Accountants of Nepal


27 of 97
Suggested Answers of Cost and Management Accounting
CAP II Examination – December2010
y = 27,755
x = 18,775
With these figures, the secondary distribution of service departments' overhead would be as
given in the following table.

Production Departments P1 (Rs.) P2 (Rs.) P3 (Rs.) Total (Rs.)

Direct Allocation: 48,000 112,000 52,000 212,000


Department S1 (80% of Rs. 18,775) 3,755 7,510 3,755 15,020
Department S2 (90% of Rs. 27,755) 2,776 16,653 5,551 24,980

Total: 54,531 136,163 61,306 252,000

Budgeted capacity (machine hours) 5,000 12,000 6,000

Overhead rate per machine hour Rs. 10.91 Rs. 11.35 Rs. 10.22

Solution to Question No. 2 (b)


Maximum level of stock indicates the maximum quantity of an item of material which should be
held in stock at any time. The stock in hand is regulated in such a way that normally, it does not
exceed this level.
While fixing the maximum level, following factors needs to be considered:
i) Maximum requirement of the material for production purpose,
ii) Rate of consumption and time lag between the date of order and receipt of material (lead
time),
iii) Nature and properties of the material, e.g. this level is kept low for materials which are
liable to quick deterioration.
iv) Cost of storage and insurance,
v) Economy in prices: Maximum levels for items which are available at discount in bulk
purchase are generally high.
vi) Financial considerations: Available of funds and price of the items.

Minimum level of stock indicates the lowest quantity of an item of material which must be
maintained in hand at all times so that there is no stoppage of production due to the
unavailability of material.
While fixing the minimum level, following factors are considered:
i) Nature of the item: No minimum level is necessary for special materials purchased against
the specific orders of a customer.
ii) Rate of consumption of the material.
iii) Lead time.

Calculation of Maximum and Minimum Stock Level:


Maximum and minimum level is calculated in the following manner:
Maximum Level = Reorder level + Reordering Quantity – Minimum consumption during the
period required to obtain delivery
The Institute of Chartered Accountants of Nepal
28 of 97
Suggested Answers of Cost and Management Accounting
CAP II Examination – December2010
Minimum Level = Reorder level – (Normal usage per period x Average Delivery Time)

Solution to Question No. 2 (c)


Let x be the cost of material and y be the normal rate of wage per hour.
Factory cost of workman BED:
Material cost Rs. x
Wages 60 y
Bonus under Rowan System: Time saved x Hours Worked x Rate per Hour
Time allowed
Overhead (60 x 10) = Rs. 600
Factory cost = x + 60 y + 24 y + Rs. 600 = Rs. 7,280, Or x + 84 y = Rs. 6,680 … (i)

Factory cost of workman DEB:


Material Rs. x
Wages 80 y
Bonus under Halsey Premium Plan = (Hours Saved x 50)/ 100 x Rate per Hour
= 20 x ½ y = 10 y
Overhead (80 x 10) = Rs. 800
Factory cost = x + 80y + 10 y + Rs. 800 = 7,600, Or x + 90 y = Rs. 6,800 … … (ii)
Deducting equations (i) from (ii), 6 y = 120, Or, y = 120/6 = 20
The normal rate of wages is therefore Rs. 20 per Hour.

3.
a) A farm incurred Rs. 65,000 of production cost in a joint process to grow a crop with two
joint products, A and B. The following are data related to the operations:
Joint Tons of Sales Price per Per Ton Separate Per Ton Separate Per Ton
Products Production Ton at Split-off costs if sold at costs if processed Final Sales
(Rs.) Split-off (Rs.) further (Rs.) Price (Rs.)
A 45 950 50 236 1,450
B 20 1200 110 200 1,600

You are required to allocate the joint process cost to A and B using: (2+3+3=8)
i) Sales value at split-off.
ii) Net realizable value at split-off.
iii) Approximated net realizable after split-off.

b) In a factory, works overheads are absorbed at 60% of labour cost and office overheads
are 20 % of works cost.
You are required to prepare the following if total expenditure consists of material Rs.
200,000; wages Rs. 150,000; factory expenses Rs. 100,000 and office expenses is Rs.
85,000. 10% of the output is in stock at the end and sales are Rs. 520,000. (3.5+3.5+1=8)
i) Cost sheet,

The Institute of Chartered Accountants of Nepal


29 of 97
Suggested Answers of Cost and Management Accounting
CAP II Examination – December2010
ii) Trading and Profit and Loss Account, and
iii) Reconciliation Statement

c) The manufacturing cost of a work order is Rs. 1,000. 8% of the production against that
order is spoiled and the rejection is estimated to have a realizable value of Rs. 20 only.
The normal rate of spoilage is 2%.
You are required to record this in the costing journal. 4

Solution to Question No 3 (a)


(i) Allocation of the joint process cost using the sales value at split-off

Joint Tons of Sales Sales Ratio of Joint Allocated


Products Production Price per Value Allocation Cost Joint
Ton at (Rs.) (Rs.) Cost
Split-off (Rs.)
(Rs.)
A 45 950 42,750 64% 65,000 41,600
B 20 1200 24,000 36% 65,000 23,400
Total 65 66,750 65,000

Working note 1:
Ratio of Allocation = Sales Value of Each Product/ Total Sales Value
A= 42750/66750 =0.64 =64%
B= 24000/66750 =0.36 =36%

(ii) Allocation of the joint process cost using the net realizable value at split-off

Joint Tons of NRP per Net Ratio of Joint Allocated


Products Production Ton at Realizable Allocation Cost Joint
Split-off Value (Rs.) Cost
(Rs.) (Rs.) (Rs.)
A 45 900 40,500 65% 65,000 42,250
B 20 1,090 21,800 35% 65,000 22,750
Total 65 62,300 65,000

Working note 2:
NRP at split-off = SP at split-off – sales cost at split off
A= Rs.950-Rs.50=Rs.900
B= Rs.1,200-Rs.110= Rs.1,090

Ratio of Allocation = NRV of Each Product/ Total NRV


A= 40500/62300 =0.65 =65%
B= 21800/62300 =0.35 =35%

The Institute of Chartered Accountants of Nepal


30 of 97
Suggested Answers of Cost and Management Accounting
CAP II Examination – December2010
(iii) Allocation of the joint process cost using the approximated net realizable value
after split-off

Joint Tons of Approx. Approx. Ratio of Joint Allocated


Products Production NRP per Net Allocation Cost Joint
Ton after Realizable (Rs.) Cost
Split-off Value (Rs.)
(Rs.) (Rs.)
A 45 1,164 52,380 67% 65,000 43,550
B 20 1,290 25,800 33% 65,000 21,450
Total 65 78,180 65,000

Working note 3:
NRP at split-off = Final SP – sales cost at split off – further processing cost
A= Rs.1,450-Rs.50-Rs.236=Rs.1,164
B= Rs.1,600-Rs.110-Rs.200= Rs.1,290

Ratio of Allocation = Approx. NRV of Each Product/ Total NRV


A= 52380/78180 =0.67 =67%
B= 25800/78180 =0.33 =33%

Solution to Question No 3 (b)


i) Cost Sheet

Particulars Amount
Rs.
Material 2,00,000
Wages 1,50,000
Prime Cost 3,50,000
Factory Overhead (60% of Rs. 1,50,000) 90,000
Works Cost 4,40,000
Office Overheads (20% of works cost) 88,000
Cost of Production 5,28,000

Cost of goods sold Rs.


4,75,200

Profit 44,800
Sales 5,20,000
Profit as per accounts= Rs. 44,800

ii) Trading and Profit and Loss Account


Dr. Cr.
Particulars Amount Particulars Amount
Rs. Rs.
To Material 2,00,000 By Sales 5,20,000
The Institute of Chartered Accountants of Nepal
31 of 97
Suggested Answers of Cost and Management Accounting
CAP II Examination – December2010
To Wages 1,50,000 By Closing Stock

To Gross profit c/d 2,22,800 52,800


5,72,800 5,72,800
To Factory Expenses 1,00,000 By Gross Profit b/d 2,22,800
To Office Expenses 85,000
To Net Profit c/d 37,800
2,22,800 2,22,800

iii) Reconciliation Statement


Rs.
Profit as per cost accounts 44,800
Add: Overcharged in Cost accounts: Office overheads 3,000

47,800
Less: Undercharged in Cost accounts: Factory Overhead 10,000
Profit as per financial records 37,800

Solution to Question No 3 (c)


Actual loss is Rs.60, i.e. Rs. 80 less Rs. 20 recoverable as materials. Of this net loss, Rs. 15 is
normal and Rs. 45 is the abnormal loss to be debited to the Costing profit and Loss account. The
accounting entries necessary for recording the above facts would be:
Rs. Rs.
Materials Control Account Dr. 20
Overhead Control account Dr. 15
Costing Profit and Loss Account Dr. 45

To Work-in Progress Control Account 80

In the case of defectives, being inherent in the manufacturing process, the rectification cost may
be charged to the specific jobs in which they have arisen. In case defectives cannot be identified
with jobs, the cost of rectification may be treated as factory overheads. Abnormal defectives
should be written off to the costing Profit and Loss Account.

4.
a) Sharma Engineering Company undertakes long-term contracts which involve the
fabrications of pre-stressed concrete blocks and the erection of the same on consumer‘s
site.

The following information is supplied regarding the contract which is incomplete on 32


Ashadh 2067:
Rs.
Cost incurred:
Fabrication costs to date:
Direct materials 280,000
Direct labour 90,000
Overheads 75,000
445,000
Erection cost to date 15,000
The Institute of Chartered Accountants of Nepal
32 of 97
Suggested Answers of Cost and Management Accounting
CAP II Examination – December2010
Total: 460,000
Contract price 819,000
Cash received on account 600,000
Technical estimate of work completed to date:
Fabrication 80%
Direct labour and overheads 75%
Erection 25%
You are required to prepare a statement for submission to the management indicating
therein the: (4.5+4.5=9)
i) Estimated profit on the completion of the contract, and
ii) Estimated profit to-date on the contract.

b) In a certain factory, Type A and Type B machines have been designed to produce the
same product but Type A is less automatic than type B and requires somewhat more
labour to operate. Pertinent costs are as follows.
Type A Type B
Set up cost Rs. 400 Rs. 600
Variable cost per unit 4.90 4.40
You are required to suggest which type of machine should be used to process various
sized orders and verify your answer by calculating total costs for the chosen levels of
production. 6

c) Briefly describe the methods of constructing flexible budget. 5

Solution to Question No. 4 (a)

Statement showing the Estimated Profit to date and on Completion of Contract


Cost to Date
% % Further Total
Cost Elements Completion (Amount in Completion Cost Rs. Cost Rs.
Rs.)
Fabrication Cost:
Direct materials 80 280,000 20 70,000 350,000
Direct labour 75 90,000 25 30,000 120,000
Overheads 75 75,000 25 25,000 100,000
Sub-total: Fabrication Cost 445,000 125,000 570,000
Erection cost 25 15,000 75 45,000 60,000
Total Rs. 460,000 170,000 630,000
Estimated profit 138,000 51,000 189,000
(See Working Note 1) 598,000 221,000 819,000

Working Notes:
1. Estimated profit to date has been calculated as follows:
Profit on the whole contract x Costs incurred so far = 189,000 x 460,000 = Rs. 138,000
Total contract costs 630,000

The Institute of Chartered Accountants of Nepal


33 of 97
Suggested Answers of Cost and Management Accounting
CAP II Examination – December2010
The amount of profit to date can also alternatively be calculated on the following basis:
Estimated profit on the whole contract x Cash received/Contract price
= 189,000 x 600,000 = Rs. 138,462.
819,000
It has been presumed that further costs will be incurred on the same pattern as they have been
incurred until now. There are no chances of increase in costs due to inflation or any other reason.

Solution to Question No. 4 (b)


Difference in set up (fixed) cost = Rs. 600 – Rs. 400 = Rs. 200
Difference in variable cost per unit = Rs. 4.90 – Rs. 4.40 = Re 0.50
Break Even Point = Difference in set-up cost = Rs. 200/Re 0.50 = 400 units
Difference in variable cost
Type A Type B
Set-up (fixed) costs Rs. 400 Rs. 600
Variable costs Rs. 1,960 Rs. 1,760
Rs. 2,360 Rs. 2,360
Hence, machine A should be used for less than 400 units as its set-up cost is lower. Similarly,
Machine B should be used for order of more than 400 units as its variable cost per unit is lower,
which will offset the higher set-up costs.

These points are made clear by verification of total costs at production levels of 399 units and
401 units, as follows:
_________________________________________________________________________________________________________________________________________________

Particulars 399 units 401 units


Type A Type B Type A Type B
Set-up costs Rs. 400.00 Rs. 600.00 Rs. 400.00 Rs. 600.00
Variable costs 1,955.10 1,755.60 1,964.90 1,764.40
2,355.10 2,355.60 2,364.90 2,364.40

Answer to Question No. 4 (c)


Methods of Constructing Flexible Budget
Following methods are generally followed in preparing a flexible budget.
(i) Segregating the items of cost into fixed, variable and semi-variable components and
presenting the figures for different levels in a tabular form
At the start of preparing flexible budget, the unit in terms of which different levels of activities
are to be expressed is first selected. It is necessary to set the budget cost allowance for the
budget centres.
(ii) Express the budget cost allowance under the heading fixed, variable and semi-variable.
Fixed cost remains the same for all levels of operations. The fixed cost per unit will change
depending upon the actual level of activity. Variable cost per unit remains the same. Semi-

The Institute of Chartered Accountants of Nepal


34 of 97
Suggested Answers of Cost and Management Accounting
CAP II Examination – December2010
variable cost is segregated into fixed and variable component and is then shown under the
respective categories.
(iii) One budget for normal production
One budget is prepared for normal level of activity by making estimates of cost at that level.
Each type of fixed and variable cost is then indicated as a ratio or a rate per unit of output. The
rate per unit of output may be expressed in terms of units, labour hours or machine hours.
(iv) Flexible Budget for Other Level of Activities
Flexible budget for other level of activities is then determined by applying the rate per unit of
output to different output levels for which the flexible budgets are desired.

Method of graphic presentation: Flexible budget is also prepared by graphic method. Under this
method, an estimate is made of the fixed and variable expenses at various levels of activity. The
figures are then plotted on a graph to get the curves for these levels. The budget cost allowance
for a particular levels of activities can be found through this method.

OR
4.
a) A company is considering three alternative proposals for conveyance facilities for its
sales executive who has to do considerable travelling, approximately 20,000 kilometers
every year. The proposals are as follows:
Proposal I: Purchase and maintain its own fleet of cars. The average cost of a car is Rs.
1,500,000.
Proposal II: Allow the executive to use his own car and reimburse expenses at the rate
of Rs. 16.00 per kilometer and also bear insurance costs.
Proposal III: Hire car from an agency at Rs. 200,000 per year per car. The company will
have to bear costs of petrol, taxes and tyres.
The following further particulars are also available:
Petrol: Rs. 6 per km. Repairs and maintenance: Rs. 2.00 per km.
Tyre: Rs. 1.20 per km. Insurance: Rs. 12,000 per car per annum
Taxes: Rs. 8,000 per car Life of the car: 5 years with annual mileage of 20,000
kms. per annum
Resale value of the car: Rs. 300,000 at the end of the 5th year.
You are required to work out the relative costs of the three proposals and rank them. 8

b) Moon Paints Ltd. has an annual demand from a single customer for 50,000 liters of a
paint product. The total demand can be made up of a range of colour will be produced in
a continuous production run after which a set-up of the machinery will be required to
accommodate the colour change. The total output of each colour will be stored and then
delivered to the customer as a single load immediately before production of the next
colour commences.
The set-up costs are Rs. 100 per set-up. This service is supplied by an outside company
as required.
The holding costs are incurred on rented storage space which costs Rs. 50 per sq. meter
per annum. Each square meter can hold 250 liters suitably stacked.
The Institute of Chartered Accountants of Nepal
35 of 97
Suggested Answers of Cost and Management Accounting
CAP II Examination – December2010
You are required to: (5.5+1.5=7)
i) Calculate the total cost per year where batches may range from 4,000 to 10,000 liters
in multiples of 2,000 liters and choose the production batch size which will minimize
total cost.
ii) Use the economic batch size formula to calculate the batch size which will minimize
total cost.

c) Briefly discuss the main purpose of inter-firm comparison. 5

Solution to Question No. 4 (a) OR


Alternative Proposals _
I. Use of Company's Car II. Use of Own Car III. Use of Hired Car
Rs. Per annum Rs. per km. Rs. per km. Rs. Per km.
Reimbursement (A) 16.00 10.00*
Fixed Cost (B)
Insurance 12,000 0.60** 
Taxes 8,000 
Depreciation 240,000 
Total: 260,000
Fixed cost per km. (Rs. 260,000/20,000 km.)13.00 0.40
Running and Maintenance Cost
Per car per km.
Petrol 6.00  6.00
Repairs and Maintenance 2.00  
Tyre 1.20  1.20
Total cost per km. (A + B + C) 22.20 16.60 17.60
Cost for 20,000 kms. 444,000 332,000 352,000
Ranking of Alternative Proposals: III I II

Decision: Alternative II is the best alternative among the available three alternatives.

Solution to Question No. 4 (b) OR


Production batch size which minimizes the total cost
___________________________________________________________________________________________________________________________________________________

Production Set-up costs Holding costs Total Costs


Batch size (lit.) per annum (Rs.) per annum (Rs.) per annum (Rs.)
___________________________________________________________________________________________________________________________________________________

4,000 1,250 400 1,650


6,000 833 600 1,433
8,000 625 800 1,425
10,000 500 1,000 1,500
____________________________________________________________________________________________________________________________________________________

Working Notes:

The Institute of Chartered Accountants of Nepal


36 of 97
Suggested Answers of Cost and Management Accounting
CAP II Examination – December2010
For a production batch size of 6,000 liters:
1. Number of set-up per year = 50,000/6,000 = 8.33
Hence annual set-up cost per year = 8.33 x Rs. 100 = Rs. 833
2. Average quantity in stock = 6,000/2 = 3,000 liters
This assumes a constant rate of production. At the start of a batch, stock is zero. At the end, the
stock equals the batch size. Hence, on an average, 50% of the batch is on stock at any point of
time.
Holding cost = 3,000 liters x Rs. 50/250 = Rs. 600.

The above table clearly reveals that the total cost is minimum at Rs. 1,425 when the production
batch size is 8,000 liters.

(i) Batch Size which minimizes total cost as per Economic Batch Size Formula

Economic production batch size = = = 7,071.

Solution to Question No. 4 (c) OR


The main purpose of inter-firm comparison is to motivate the management to improve the
efficiency by showing the present level of achievements and possible weakness areas. Such a
comparison can be instrumental in overcoming following types of problems/weaknesses facing a
business entity.
i) Profit adequacy:
Profit is the principal factor to motivate any commercial venture or organization. The relation of
profit to capital employed is the general norm employed to assess the efficiency or return of
commercial firms. If the return on capital employed is less than that of other efficient firms
within the industry, it is an indicator to show that some factors are not operating efficiently
within the firm in question. These can be isolated by means of the various ratios computed for
the firm and other competitors. Suitable corrective and follow up actions is then initiated to
improve the profitability situation of the concerned firm.
ii) Efficiency in selling:
The operating profit to total sales and to capital employed are vital ratios to indicate the profit-
earning capacity of a firm. The first ratio indicates the total margin earned by the sales expressed
as a percentage. On the other hand, the sales to capital employed indicates how much is sold per
rupee invested. A comparison of these ratios with efficient firms and subsequent analysis of the
reasons could throw out areas where the firm needs improvement for improving the efficiency.
iii) Production efficiency:
In order that the firm earns reasonable return, it is necessary that the production departments
produce required volume of output at reasonable costs. For this purpose, the factory cost of sales
is broken down into direct material, direct wages and production overhead costs. A comparison
of these figures with other firms of the industry may point out the sources of inefficiencies. For
instances, production efficiency of a firm as compared to efficient firms within an industry may
have been affected by lower labour untilization or lower labour utilization.

5. Distinguish between: (42.5=10)


a) Cost control and cost reduction

The Institute of Chartered Accountants of Nepal


37 of 97
Suggested Answers of Cost and Management Accounting
CAP II Examination – December2010
b) Fixed and flexible budgeting
c) Cost centre and cost unit
d) Absorption costing and marginal costing

Answer to Question No. 5

a) Cost control and cost reduction:


Cost control is one of the primary objectives for the establishment of a cost accounting system
in an organization. It is defined as the regulation of the costs of operating an undertaking by
executive action. Cost control is exercised through a number of techniques such as standard
costing, budgetary control, inventory control, quality control and performance evaluation,
analysis and reporting.

Cost reduction may be termed as maximizing profits by reducing cost through economies and
savings in cost of manufacture, administration selling and distribution.
It is defined as the real and permanent reduction in the units costs of goods manufactures or
services provided without impairing their suitability for the use intended.
According to above definition, reduction in costs should be real and permanent. Thus, reductions
due to windfalls, changes in government policy such as reduction in taxes or duties do not fall
under the purview of cost reduction.

b) Fixed and flexible budgeting:


A fixed budget is one which is designed for a specific planned level of output and is not adjusted
to the level of activity attained at the time of comparison between the budgeted and actual costs.
Thus, fixed budgets are established only for a small period of time when the actual output is not
anticipated to differ much from the budgeted output. Although not adjusted to the actual volume
attained, a fixed budget is liable to revision in case actual operations differ significantly from
those planned.
A fixed budget is ineffective as a tool for cost control. It is because the difference can not be
explained while comparing the actual cost with a fixed budget.

Flexible budget is a budget which is designed to change based on the fluctuations in output or
turnover. Thus, the flexible budget provides budgeted costs for any level of activity actually
attained.
Flexible budgets may also be used for adjusting budgets to suit current conditions which may
arise due to seasonal variations or changes in the length of the working period.
Flexible budget is useful for the purpose of control since it takes into consideration the changes
in the actual circumstances than previously anticipated.

c) Cost centre and cost unit


Cost centre is defined as a location, person or item or group of equipment for which cost may be
ascertained and used for the purpose of cost control.
In a manufacturing entity, the cost centres generally follow the pattern or layout of the
departments or sections of the factory. As a result, the cost centres are either production cost
centres or service cost centres.

The Institute of Chartered Accountants of Nepal


38 of 97
Suggested Answers of Cost and Management Accounting
CAP II Examination – December2010
The number of cost centres and the size of each vary from one organization to another. These
depend on the expenditure involved and the requirements of the management for the purpose of
cost control. Keeping the number of cost centres high will be expensive while having very few
cost centres may defeat the very purpose of cost control.
Cost unit is a device for the purpose of breaking up or separting costs into small sub-divisions
attributable to products or services. It is the unit of quantity of product, service, or time in
relation to which costs may be determined or expressed.
Thus, cost may be ascertained per tonne of steel, per tonne-kilometre of a transport service or
cost per machine hour.

d) Absorption costing (and marginal costing*)


Absorption costing does not recognize the difference between fixed costs and variable costs.
The statements prepared under this costing method explain in depth the past profits, past losses
and past costs but do not help in predicting the future results.
Marginal costing is the costing system in which variable costs are charged to the cost units and
fixed cost of the period are written off in full against the aggregate contribution.
The difference between absorption costing and marginal costing is summarized as follows:

Absorption costing Marginal costing


1. Both fixed cost and variable costs are 1. Only variable cost is considered for these
considered for product costing and inventory purposes.
valuation.
2. Fixed cost is charged to the production. 2. Fixed overhead is treated as period cost and
profitability of products is judged in terms of
P/V ratio.
3. Net profit of each product is derived after 3. Data is presented to highlight the total
deducting fixed overheads. contribution and contribution of individual
products.
4. Due to the impact of fixed overheads, unit 4. Unit cost of production is not affected by the
cost of production is affected due to the difference in the level of opening and closing
difference in the level of opening and closing stock.;
stock.
* This was missing in the question set.

6. Answer any FOUR of the following questions: (42.5=10)


a) Mention the main advantage of cost plus contracts.
b) Briefly explain your understanding of the term 'rolling budget'.
c) Explain briefly the conditions when supplementary rates are used.
d) Explain the reasons why some companies normally prepare the sales budget first among
all functional budgets while the other companies start with the labour or other budget
first in the budgetary planning process.
e) What are the advantages of cost audit to the management? Explain in brief.

The Institute of Chartered Accountants of Nepal


39 of 97
Suggested Answers of Cost and Management Accounting
CAP II Examination – December2010
Answer to Question No. 6
a) Main advantages of cost plus contracts are:
 Contractor is protected from risk of fluctuation in market price of material, labour and
services.
 Contractee can insure a fair price of the market.
 It is useful specially when the work to be done is not definitely fixed at the time of
making the estimate.
 Contractee can ensure himself about ‗the cost of the contract‘ as he is empowered to
examine the books and documents of the contractor to ascertain the veracity of the
cost of the contract.

b) Rolling budget
It is the budget continuously updated by adding a further period, say a month, quarter or year
and deducting the earliest period. This type of budget is beneficial where future costs and
activities can not be forecast on a reliable manner.
In the preparation of rolling budget, the budgeting is a continuous process. As the month, quarter
or year passes, forecast for that period is dropped and a forecast for a further month, quarter or
year is added in such a way that there is always a forecast of a fixed period say, a year or 2-year,
or 3-year is available.
The preparation of a rolling budget is always a costly affair. However, the use of such a budget
always reduces the operational variances.

c) The conditions when supplementary rates are used


When the amount of under absorbed and over absorbed overhead is significant or large, because
of differences due to wrong estimation, then the cost of product needs to be adjusted by using
supplementary rates (under and over absorption/actual overhead) to avoid misleading
impression.

d) Reasons for preparing the sales budget or labour or other budget first among all
functional budgets
The budgetary planning process usually starts with sales budget because a company is usually
restricted from making and selling more of its products. Under this assumption, sales demand is
the principal budget factor, in which it restricts the performance or level of activity of a
company.
The other limiting factors could be machine capacity, distribution and selling resources, the
availability of key raw materials or labour. However, if the principal budget factor is the
availability of labour, the first functional budget to prepare is the labour budget, in which a
company needs to consider how the limited labour hours are assigned to the optimal mix of
products to maximise its profit.

e) Advantages of Cost Audit to the Management


i) Cost audit assists in the detection of errors and frauds.
ii) Cost audited data is more reliable for the preparation of accurate cost reports and returns
for presentation to the parties interested. The inventory valuation certified by the cost
auditor is considered correct and reliable.

The Institute of Chartered Accountants of Nepal


40 of 97
Suggested Answers of Cost and Management Accounting
CAP II Examination – December2010
iii) Cost audit contributes towards the improved cost accounting methods and thus ensures
better internal control.
iv) The disclosures made in the cost audit reports create cost consciousness in the
management.
v) The cost auditor points out avoidable wasteful routine and procedures and recommends for
the introduction of an efficient cost routine. Thus it will help to reduce expenditure in cost
accounts and at the same time, ensures promptness in its preparation.
vi) Cost audit aids the management to initiate action for economic and efficient usage of
labour, material and other resources. This will lead to higher productivity and better
utilization of resources.
vii) Audited cost data is useful for the purpose of inter-firm comparison.
viii) Cost audit may be useful in identifying the symptoms of sickness in an enterprise. Suitable
remedial measures could be initiated in such situations.

The Institute of Chartered Accountants of Nepal


41 of 97
The Institute of Chartered Accountants of Nepal
Suggested Answers of Financial Management

CAP II Examination – December 2010


Marks

Attempt all questions. Working notes should form part of the answer.

1. The following are the financial statements of PQR Ltd. for 2066/67.

Balance Sheet of PQR Ltd. as on Ashadh end 2066/67

Liabilities Amount (Rs.) Assets Amount (Rs.)


Equity Share Capital 210,000 Cash 105,000
Reserves 420,000 Debtors 525,000
Preference Share Capital 420,000 Stock 735,000
Long-term Debts 1,260,000 Fixed Assets (Net) 1,575,000
Creditors 420,000 Goodwill 210,000
Bills Payable 210,000
Outstanding Expenses 60,000
Provision for Tax 150,000
3,150,000 3,150,000

Income Statement of PQR Ltd.


for the year ending Ashadh, 2067
Rs. Rs.
Sales
Cash 420,000
Credit 1,680,000 2,100,000
Less: Expenses
Cost of Goods Sold 1,260,000
Selling, Administration and General Expenses 210,000
Depreciation 147,000
Interest on Long-term Debt 63,000 1,680,000
Profit Before Taxes 420,000
Taxes 210,000
Profit After Taxes 210,000
Less: Preference Dividend 25,500
Net Profit for Ordinary Shareholders 184,500
Add: Reserve at 1 Shrawan 2066 273,000
Profit Available to Ordinary Shareholders 457,500
Less: Dividend Paid to Equity Shareholders 37,500
Reserve at Ashadh end 2067 420,000

The ratios for the previous two years relating to the company and the industry
ratios are given below:

2064/065 2065/066 Industry


Current Ratio 2.54 2.10 2.30
Acid-test Ratio 1.10 0.96 1.20
Debtors Turnover 6.00 4.80 7.00
Stock Turnover 3.80 3.05 3.85
Long-term Debt to Total Capital 37% 42% 34%
Gross Profit Margin 38% 41% 40%
Suggested Answers of Financial Management
CAP II Examination – December2010

Net Profit Margin 18% 16% 15%


Return on Equity 24% 29% 19%
Return on Total Assets 7% 6.8% 8%
Tangible Assets Turnover 0.80 0.70 1.00
Interest Coverage 10 9 10

Based on the above financial statement and ratios of the company and the industry provided
above, you are required to:
a) Calculate the same ratios as provided above for 2066/067,
b) Evaluate the company‘s financial position of the company on the basis of these ratios and
past ratios of the company and the industry,
c) Using relevant ratios, indicate what decision would be taken in the following situations:
i) PQR Ltd. wants to buy materials of Rs. 210,000 on a three months credit from a
domestic supplier company.
ii) PQR Ltd. wants to issue 15% debentures of Rs. 600,000 with a 10 year maturity
period. (11+5+4)

Answer No.1
Solution to Question No. 1

(a) The ratios for 2066/067 for PQR Ltd. are computed as follows:
__________________________________________________________________________
Computation Ratio for 2066/067
___________________________________________________________________________________________________________________________________________________________

1. Current Ratio 1,365,000/840,000 1.63


2. Acid-test Ratio 630,000/840,000 0.75
3. Debtors Turnover 1,680,000/525,000 3.20
4. Stock Turnover 1,260,000/735,000 1.71
5. Long-term Debt to Total Capital 1,260,000/.2,100,000 60%
6. Gross Profit Margin 840,000/2,100,000 40%
7. Net Profit Margin 210,000/2,100,000 10%
8. Return on Equity 184,800/420,000* 44%
9. Return on Total Assets (420,000 + 63,000) (1 – 0.5)/2,940,000* 8.2%
10. Tangible Assets Turnover 2,100,000/2,940,000* 0.71
11. Interest Coverage 483,000/63,000 7.67
___________________________________________________________________________________________________________________________________________________________
_

* Intangible assets of Rs. 210,000 excluded.

(b) Based on the ratios computed above, evaluation of the company’s position is
presented below:

The Institute of Chartered Accountants of Nepal


2 of 97
Suggested Answers of Financial Management
CAP II Examination – December2010

i. The liquidity position of the firm is falling which is evident from the Ratios 1 to 4
computed above.
ii. The gross profit margin is constant and matches with the industry average, but the net
profit margin ratio is declining. The two ratios together imply that the company‘s
selling and administrative expenses, depreciation and interest charges are on the rise.
iii. The decline in the net margin is partly due to rapid increase in debt (Ratio 5). This
increase also explains why the return on equity (Ratio 8) has been rising while the
return on assets is declining (Ratio 9).
iv. The decline in the net margin and the return on assets can also be attributed to the
decline in assets turnover (Ratio 10).
v. The impact of the increase in debt and overall decline in profitability are also shown by
reduction in the interest coverage (Ratio 11).

(c) Decision under Different Situations:


(i) The supplier would be more concerned with the liquidity of current assets of the
company. Therefore, Ratios 1 to 4 are more relevant to the supplier. In view of the
deteriorating liquidity position and the lengthy terms of payment, the credit may not be
granted to the company.
(ii) The company may find difficulty in selling the debentures. Already, it has a high
leverage ratio. If the debentures are issued its leverage ratio will increase to 68.89 per
cent (Rs. 1,860,000/Rs. 2,700,000) and the interest coverage ratio at the same level of
earning will decline to 4.06. In addition, the liquidity and the profitability of the
company are also declining. Therefore, it is not proper time to issue the debentures.

2.
a) Applying Capital Asset Pricing Model answer the following, the market portfolio has
following characteristics and other information are provided below:

Standard Deviation of security j 20%


Standard Deviation of market portfolio 15%
Expected Return of market portfolio 13%
Correlation between possible returns for security j and the market portfolio 0.80
Risk Free Rate 7%
Required: (3+3+3+1=10)
i) What is the expected return of security j?
ii) What would happen to the required return if the standard deviation of security j is
30?
iii) What would happen if the correlation coefficient is 0.70?
iv) What is the functional relationship between the required return for a security and
market risk?
b) R
ock Star Company Ltd. is attempting to establish the current assets policy. Fixed Assets are
Rs. 600,000 and the company plans to maintain a 50 percent debt to Total assets ratio. The
interest rate is 10 percent on all debt. As a financial consultant, the Rock Star Company
The Institute of Chartered Accountants of Nepal
3 of 97
Suggested Answers of Financial Management
CAP II Examination – December2010

seeks your advice on three alternatives of current asset policies: 40 %, 50%, 60% of
projected sales. The company expects to earn 15 percent before interest and taxes on sales of
Rs. 3 Million. Tax Rate applicable for the company is 40 percent. Provide your advice to the
company by showing Return on Equity under each alternative.

Answer No.2
a)
Given,
Standard Deviation of security j (σ j) = 20%
Standard Deviation of market Portfolio (σ m) = 15%
Expected Return of market portfolio E(R m) = 13%
Correlation between the security and market(r jm) = 0.80
Risk Free Rate(R f) = 7%
a.
Calculation of Beta
βj = r jm * σ j * σ m / σ m2
= 0.80* 20 * 15 / (15) 2
= 1.067
Required Rate of Return E(R j) = R f + [E(R m) - (R f)] βj
= 7% + [13% - 7%]1.067
= 13.40%

b.
Calculation of Beta
βj = r jm * σ j * σ m / σ m2
= 0.80* 30 * 15 / (15) 2
= 1.60
Required Rate of Return E(R j) = R f + [E(R m) - (R f)] βj
= 7% + [13% - 7%]1.6
= 16.6%
Hence the required return would increase.
c.
βj = r jm * σ j * σ m / σ m2
= 0.70* 20 * 15 / (15) 2
= 0.9333
Required Rate of Return E(R j) = R f + [E(R m) - (R f)] βj
= 7% + [13% - 7%] 0.9333
= 12.60%
Hence the required return would decrease.
d. The relationship is linear throughout and is called Security Market Line. The important point
to stress is that in market equilibrium, an expected return relationship with the market portfolio
is implied for all securities.

b)
Given
Fixed Assets = Rs. 600,000
Debt to assets ratio = 50%
Interest on debt = 10%
EBIT = 15% of sales
Sales = Rs. 3 Million
Tax Rate (T) = 40%
The Institute of Chartered Accountants of Nepal
4 of 97
Suggested Answers of Financial Management
CAP II Examination – December2010

Current Assets alternatives = 40 % of sales, 50% of sales, 60% of sales


Return on Equity (ROE) (%) = Required

The Institute of Chartered Accountants of Nepal


5 of 97
Suggested Answers of Financial Management
CAP II Examination – December2010

Rock Star Company's Balance Sheet under three alternatives


Alternatives 40% of sales 50% of sales 60% of sales
Current Assets 1,200,000 1,500,000 1,800,000
Fixed Assets 600,000 600,000 600,000
Total Assets 1,800,000 2,100,000 2,400,000
Debt 900,000 1,050,000 1,200,000
Equity 900,000 1,050,000 1,200,000
Total Liabilities and 1,800,000 2,100,000 2,400,000
Equity

Computation of Interest
Total Debt (Rs.) 900,000 1,050,000 1,200,000
Total Interest( 10% of 90,000 105,000 120,000
total debt)

Rock Star Company's Income Statement under three alternatives


Alternatives 40% of sales 50% of sales 60% of sales
Sales 3,000,000 3,000,000 3,000,000
EBIT( 15%) 450,000 450,000 450,000
Less: Interest( 10%) 90,000 105,000 120,000
EBT 360,000 345,000 330,000
Tax( 40%) 144,000 138,000 132,000
EAT 216,000 207,000 198,000
ROE(%) 24 19.7 16.5

3.
a) Y
ou are evaluating a proposal to invest in two companies whose past ten years of returns are as
shown below:
Percent Returns during the Year
Company
1 2 3 4 5 6 7 8 9 10

ABC 37 24 -7 6 18 32 -5 21 18 6

DEF 32 29 -12 1 15 30 0 18 27 10

In respect of the above companies, you are required to: (4+2.5+1.5+2=10)


i) Calculate the standard deviation of each company‘s returns,

The Institute of Chartered Accountants of Nepal


6 of 97
Suggested Answers of Financial Management
CAP II Examination – December2010

ii) Calculate the correlation coefficient of the two company‘s returns,


iii) Determine the standard deviation of your portfolio and the average yearly return,
assuming that you had placed 50% of your money in each company‘s share, and
iv) Determine the percentage investment in each which would have resulted in the lowest
risk.
b) Weather Coats Paint Ltd. has fixed operating costs of Rs. 36 million a year. Variable
operating costs are 180 per half liter of paint produced, and the average selling price is Rs.
200 per half liter.
You are required to answer the following questions with computations to support each one of
your answer. (2.5 + 1.5 + 1 = 5)
i) What is the annual operating break-even point in half liters (QBE) and in rupees of sales
(SBE)?
ii) What would be the effect on the operating break-even point (QBE) of a simultaneous decline
to Rs. 170 per half liter in the variable operating costs and an increment of 20 per cent in the
fixed cost?
iii) Compute the degree of operating leverage (DOL) at the current sales level of 2 million half
liters.
Answer No.3
a)
(i) Standard Deviation of Returns
Average Returns of the companies are:
R ABC = (37 + 24 – 7 + 6 + 18 + 32 – 5 + 21 + 18 + 6)/10 = 15%
R DEF = (32 + 29 – 12 + 1 + 15 + 30 + 0 + 18 + 27 + 10)/10 = 15%
Standard Deviation of ABC and DEF companies are computed as shown below.
σ ABC = √ (37 – 15)2 + (24 – 15) 2 + (– 7 – 15) 2 + (6 – 15) 2 + (18 – 15) 2 + (32 – 15) 2 + (–5–15)
2
+ (21 – 15) 2 + ( 18 – 15) 2 + (6 – 15) 2
√ 10
= √ (22)2 + (9) 2 + (– 22) 2 + ( – 9) 2 + (3) 2 + (17) 2 + ( – 20) 2 + (6) 2 + ( 3) 2 + ( – 9) 2
√ 10
= √ 484 + 81 + 484 + 81 + 9 + 289 + 400 + 36 + 9 + 81 = √ 1954 = 13.98
√10 √10
σ DEF = √ (32 – 15)2 + (29 – 15) 2 + (– 12 – 15) 2 + (1–15) 2 + (15–15) 2 + (30–15) 2 +(0 – 15) 2 +
(18 – 15) 2 + ( 27 – 15) 2 + (10 – 15) 2
√ 10
= √ (17)2 + (14) 2 + (– 27) 2 + ( – 14) 2 + (0) 2 + (15) 2 + (– 15) 2 + ( 3) 2 + ( 12) 2 + (– 5) 2
√ 10
= √ 289 + 196 + 729 + 196 + 0 + 225 + 225 + 9 + 144 + 25 = √ 2038 = 14.28
√10 √10

(ii) Correlation Coefficient of Returns

Cov. ABC DEF = √ (37 – 15) (32 – 15) + (24 – 15) (29 – 15) + (– 7 – 15) (– 12 – 15) + (6 – 15)
(1 – 15) + (18 – 15) (15 – 15) + (32 – 15) (30 – 15) + ( – 5 – 15)( 0 – 15) +
The Institute of Chartered Accountants of Nepal
7 of 97
Suggested Answers of Financial Management
CAP II Examination – December2010

(21 – 15) (18 – 15) + ( 18 – 15) ( 27 – 15) + (6 – 15) (10 – 15)


10
= (37 – 15) (32 – 15) + (24 – 15) (29 – 15) + (– 7 – 15) (– 12 – 15) + (6 – 15) (1 – 15) +
(18 – 15) (15 – 15) + (32 – 15) (30 – 15) + ( – 5 – 15)( 0 – 15) + (21 – 15) (18 – 15)
+ ( 18 – 15) ( 27 – 15) + (6 – 15) (10 – 15)
10
= (22) (17) + (9) (14) + (– 22) (– 27) + (– 9) (–14) + (3) (0) + (17) (15) + (– 20) ( –15) +
(6) (3) + ( 3) ( 12) + ( – 9) (– 5)
10
= 374 + 126 + 594 + 126 + 0 + 255 + 300 + 18 + 36 + 45 = 1874/10 = 187.4
10
Correlation Coefficient = Cov. ABC DEF = 187.4 = 0.93
σ ABC σ DEF 13.98 x 14.28

(iii) Standard Deviation of the Portfolio and Average Yearly Return,


Assuming 50% Investment in Both the Company’s Share
σ P = √ (WABC2 σ ABC2 + W DEF2 σ DEF2 + 2 WABC W DEF Correlation Coeff. σ ABC σ DEF)
= √ [(0.5)2 (13.98)2 + (0.5)2 (14.28)2 + 2 (0.5) (0.5) (13.98) (14.28) (0.93)]
= √ (48.86 + 50.97 + 92.82) = √ 192.65 = 13.88

E (RP) = 0.5 (15) + 0.5 (15) = 15%

(iv) Percentage Investment in each which


would have resulted in the Lowest Risk
Using the minimum variance equation and let W stand for ABC,
WABC = σ 22 – σ1 σ2 σ1,2 = (14.28) 2 – (13.98) (14.28) (0.93)
σ 12 + σ22 – σ1 σ2 σ1, 2 (13.98)
2 2
+ (14.28) – 2 (13.98) (14.28) (0.93)
= 203.981 – 185.65 = 18.33 = 0.6537, Or 65.37
195.44 + 203.91 – 371.31 28.04

WDEF = 1 – 0.6537 = 0.3463, Or 34.63%

b)

(i) Computation of QBE and SBE


QBE = Rs. 36 Million = Rs. 36 Million = 1,800,000 half liters
Rs. 200 – Rs. 180 Rs. 20

SBE = Rs. 36 Million = Rs. 36 Million = Rs. 36 Million = Rs. 360,000,000 in annual
sales
1 – (Rs. 180/Rs. 200) 1 – 0.90 0.10

(ii) Effect of a Decline to Rs. 160 per half liter in the variable operating costs and an
Increase of 25 per cent in the Fixed Cost on QBE
QBE = Rs. 36 Million x 1.25 = Rs. 45 Million = 1,500,000 half liters
The Institute of Chartered Accountants of Nepal
8 of 97
Suggested Answers of Financial Management
CAP II Examination – December2010

Rs. 200 – Rs. 170 Rs. 30

(iii) Degree of Operating Leverage (DOL) at the current sales level of 2 Million half
liters
DOL 2 million units = 2 Million = 2 Million = 10
(2 – 1.8) Million 0.2 Million

4.
a) Following data related to Universal Manufacturers Ltd. is made available to you.
Particulars Year 1 Year 2
Stocks:
Raw Materials Rs. 300,000 Rs. 405,000
Work-in-process 210,000 270,000
Finished Goods 315,000 360,000
Purchase of Raw Materials 1,440,000 2,025,000
Cost of Goods Sold 2,100,000 2,700,000
Sales 2,400,000 3,000,000
Debtors 480,000 750,000
Creditors 240,000 270,000

You are required to compute the duration of the operating cycle for each of the two years and
comment on the increase/decrease. (Assume 360 days per year for the purpose of computations.)

b) Fast Growing Ltd. has outstanding a Rs. 1000 face value bond with a 12% coupon rate
and 3 years remaining until final maturity. Interest payments are made semi-annually.
You are required to answer the following questions with appropriate supporting computations:(2.5+1.5=4)
i) What value should you place on this bond if your nominal annual required rate of return is 10 per
cent; and
ii) Assuming a bond similar to the one described above except that is a zero-coupon, pure discount
bond, what value should you place on this bond if your nominal annual required rate of return is
16 per cent. (Assume a semiannual compounding.)
c) An investor has made investment in the equity share of Pacific Chemicals Ltd. The
capitalization rate of the company is 20 per cent and the current dividend is 25 per share.
You are required to calculate the value of the company‘s equity share if the company is slowly
sinking with an annual decline rate of 10% in the dividend.
3
Answer no.4
a)

Determination of Operating Cycle:

Particulars Year 1 Year 2

The Institute of Chartered Accountants of Nepal


9 of 97
Suggested Answers of Financial Management
CAP II Examination – December2010

(i) Raw Materials Holding Period:


360 days x Stock of Raw Materials 360 x Rs. 300,000 = 75 360 x Rs. 405,000 = 72
Cost of Raw Materials Consumed* Rs. 1,440,000 Rs. 2,025,000
(* Assumed to be equivalent to purchases)
(ii) Less: Creditors Payment Period:
360 days x Creditors 360 x Rs. 240,000 = (60) 360 x Rs. 270,000 =
(48)
Purchases Rs. 1,440,000 Rs. 2,025,000
(iii) Work-in-process Holding Period:
360 days x Stock of WIP . 360 x Rs. 210,000 = 36 360 x Rs. 270,000 = 36
Cost of Goods Manufactured Rs. 2,100,000 Rs. 2,700,000
(iv) Finished Goods Holding Period:
360 days x Stock of Finished Goods 360 x Rs. 315,000 = 54 360 x Rs. 36,000 = 48
Cost of Goods Sold Rs. 2,100,000 Rs. 2,700,000
(v) Debtors Collection Period:
360 days x Debtors 360 x Rs. 480,000 = 72 360 x Rs. 750,000 = 90
Credit Sales** Rs. 2,400,000 Rs. 3,000,000
(** Assumed to be equal to total sales)
Duration of Operating Cycle [Sum of (i) to (v)] 177 198

Comment on the Increase/Decrease:


The duration of the operating cycle has increased by 21 days in Year 2 as compared to Year 1. It
will necessitate more working capital in Year 2. This increase has been primarily caused by an
increase in debtors‘ collection period and decrease in creditors‘ payment period as shown in the
following table.
____________________________________________________________________________________________________________________

Increase in Debtors‘ Collection Period: 18 days


Decrease in Creditors‘ Payment Period: 12
Less: Decrease in Raw Material Holding Period: (3)
Less: Decrease in Finished Goods Holding Period: (6)
Net Increase in Operating Cycle: 21
____________________________________________________________________________________________________________________
_

b)

(i) Value of Bond when kd = 10%

We have, value of a bond (V) = I/2 (PVIFA kd, 2n) + MV (PVIF kd, 2n), where
 kd is the investor‘s required rate of return
 n is the number of years and 2n is the number of semi-annual periods until maturity.
 I/2 is the periodic interest payment
 MV is the maturity value of the bond
The Institute of Chartered Accountants of Nepal
10 of 97
Suggested Answers of Financial Management
CAP II Examination – December2010

Substituting the given values in the above formula, we get:


V = (Rs. 120/ 2) (PVIFA 0.05, 6) + Rs. 1000 (PVIF 0.05, 6)
= Rs. 60 (5.076) + Rs. 1,000 (0.746)
= Rs. 304.56 + Rs. 746 = Rs. 1,050.56.

(ii) Value of Zero Coupon Bond when kd = 16%

The value of this type of bond is found out simply by discounting the maturity value of the bond
to the present. Thus,
V = Rs. 1,000 (PVIF 0.08, 6) = Rs. 1,000 (0.630) = Rs. 630

c)
The value of the company‘s equity share is given by the following formula:
Ve = D1/(k – g), where D1 is the dividend in the year 1, k is the capitalization rate and g is the
growth rate in dividend.

The value of equity share in the given condition is derived as follows:


Ve = Rs. 25 (1 – 0.10)/[(0.20 – (– 0.10)] = Rs. 25 x 0.90/0.30 = Rs. 22.50/0.30 = Rs. 75

5.
a) As an investment manager, you are provided with the following information:

Investment in Initial Dividend Market Price Beta


Price (Rs.) at the year (Risk
(Rs.) end (Rs.) Factor)
Equity Share of ABC Cement Ltd. 250 20 400 0.8
Equity Share of BCD Sugar Ltd. 350 20 600 0.7
Equity Share of DEF Distillery Ltd. 450 20 1,050 0.5
Government of Nepal Bonds 1,000 140 1,005 0.99
Risk-free return may be taken at 15%.
You are required to calculate: (3+4+2=9)
i) Expected rate of return on market portfolio,
ii) Expected rate of return of individual portfolio using Capital Asset Pricing Model (CAPM),
iii) Average return of portfolio.

b) The bonds of Express Ltd. are currently selling at Rs. 130. They have 9 percent coupon
rate of interest and Rs. 100 par value. The interest is paid annually and the bonds have 20 years
to maturity.
You are required to: (4+2=6)
i) Compute the Yield to Maturity (YTM) of the bond.
ii) Explain the difference between YTM and coupon rate of interest of the bond.
Answer No.5
a)

Investment
Investment Dividend (Rs.) Capital Gain(Rs.)
The Institute of Chartered Accountants of Nepal
11 of 97
Suggested Answers of Financial Management
CAP II Examination – December2010

Amount (Rs.)
Equity Share of ABC Cement Ltd. 250 20 150
Equity Share of BCD Sugar Ltd. 350 20 250
Equity Share of DEF Distillery Ltd. 450 20 600
Government of Nepal Bonds 1,000 140 5
Total: 2,050 200 1,005

(i) Expected Rate of Return on Market Portfolio :

= Dividend Earned + Capital Appreciation x 100


Initial Investment
= 200 + 1,005 x 100 = 1,205/2,050 x 100 = 58.78%
2,050

(ii) Expected rate of return of individual portfolio using Capital Asset Pricing Model
(CAPM)
Now, we can calculate the expected rate of return on individual portfolio by applying
CAPM.
E (Ri) = Rf + βi (Rm – Rf)
ABC Cement Ltd = 15 + 0.8 (58.78 – 15) = 15 + 0.8 x 43.78 = 50.02%
BCD Sugar Ltd = 15 + 0.7 (58.78 – 15) = 15 + 0.7 x 43.78 = 45.65%
DEF Distillery Ltd. = 15 + 0.5 (58.78 – 15) = 15 + 0.5 x 43.78 = 36.89%
Government of Nepal Bonds = 15 + 0.99 (58.78 – 15) = 15 + 0.99 x 43.78 = 58.34%

(iii) Average Return of the Portfolio:


= (50.02 + 45.65 + 36.89 + 58.34)/4 = 190.90/4 = 47.73%

Alternatively, the Average Return could also be found out on the basis of average of beta
factors of all securities in the portfolio in the following manner.
Average of Betas = (0.8 + 0.7 + 0.5 + 0.99)/4 = 2.99/4 = 0.7475

Average Return = 15 + 0.7475 (58.78 – 15) = 15 + 0.7475 x 43.78 = 15 + 32.73 = 47.73%

b)
i) We have,
B = I x (PVIFAkd n) + M x (PVIF kd n)
Where,
B = Value of the Bond
I = Annual Interest Paid
The Institute of Chartered Accountants of Nepal
12 of 97
Suggested Answers of Financial Management
CAP II Examination – December2010

n = Number of Years to Maturity


M = Par/Maturity Value
kd = Required Return on the Bond

Information given in the problem are:


B = Rs. 140
I (Annual Interest Paid) = Rs. 100 X 0.09 = Rs. 9
M (Par/Maturity Value) = Rs. 100
n = 20
kd = 9

Let us try a lower rate of 7 per cent in the formula:


B = I x (PVIFAkd, n) + M x (PVIF kd, n) = Rs. 9 x (PVIFA 7, 20 ) + Rs. 100 x (PVIF 7, 20)
= (Rs. 9 x 10.594) + Rs. 100 x 0.258) = Rs, 95.35 + Rs. 25.80 = Rs. 121.15.
Since Rs. 121.15 < Rs. 135, let us try still a lower rate of 6 per cent.
B = I x (PVIFAkd n) + M x (PVIF kd n) = Rs. 9 x (PVIFA 6, 20 ) + Rs. 100 x (PVIF 6, 20)
= (Rs. 9 x 11.470) + Rs. 100 x 0.312) = Rs, 103.23 + Rs. 31.20 = Rs. 134.43.

By interpolation, YTM = 6% + (134.43 – 130) % = 6% + (4.43/13.28) % = (6 + 0.33) %


(134.43– 121.15)
= 6.33% approximately.

11) The YTM (6.33%) is below the coupon interest rate (9%) of the bond since its
market value (Rs. 130) is above its par value (Rs. 100).

ii) Explanation Ragarding the difference between YTM and Coupon Rate

Yield to maturity (YTM) is the expected rate of return on a bond if bought at its current market
price and held to maturity. It is also called the bond‘s internal rate of return (IRR).
The underlying feature of bond price is that YTM < coupon rate when a bond sells at a premium
and vice versa. Similarly, YTM = coupon rate when a bond sells at par.
In the present case, the bond is selling at a premium of Rs. 30 as compared to the par value of
Rs. 100. This is the reason for the YTM (6.33%) being lower than the coupon interest rate of
9%.

6. Write short notes on: (4x2.5=10)


a) Direct and indirect costs associated with financial distress
b) Revolving credit agreement
c) Perpetuity Rate of Return
d) Leveraged Buyout
Answer No.6
a) Financial distress arises when a firm is not able to meet its obligations towards the payment of
interest and principal to the debt providers which can lead to bankruptcy. Direct costs of
financial distress include the costs of insolvency.
Following are the other direct costs of financial distress:

The Institute of Chartered Accountants of Nepal


13 of 97
Suggested Answers of Financial Management
CAP II Examination – December2010

i. Long period taken in the bankruptcy cases may cause deterioration of the conditions of the
company‘s assets.
ii. Liquidation of the assets may be delayed due to conflicting interests of creditors and other
stakeholders.
iii. When the assets are sold under distress prices, they may fetch a price that is significantly
lower than their current values.
iv. Legal and administrative costs related to the bankruptcy proceedings are generally quite
high.

b) A revolving credit agreement is a formal commitment by a bank to lend up to a certain


amount of money to a company over a specified period of time. This type of debt is for a short
term, usually up to 3 months. The company may, however, renew or borrow additional amount
up to the limit of agreement throughout the duration of the commitment. Although commitment
can be obtained for a shorter period as well, most revolving credit commitments are provided for
3 years.
The interest term of the revolving credit agreements are similar to but slightly higher (usually
between 0.25 to 0.50 per cent higher) than the rate at which a firm can borrow on a short term
basis under a line of credit. The banks generally charge commitment fee usually around 0.5 per
cent per annum on the difference between the amount borrowed and the maximum limit amount.
This type of borrowing arrangement is very useful at times when the firm is not certain about its
fund requirements. The borrowing company will have flexibility in the access to funds at the
time of uncertainty and can make more definite credit arrangement when the situation of
uncertainty is resolved.

c) Perpetuity Rate of Return - Perpetuity Rate of Return (PRR) is a conversion of Profitability


Index (PI) into a perpetuity percentage rate of return of a project. In other words, it is PI of a
project expressed in terms of the Perpetuity rate of return. So it is just the product of PI and
required rate of return from the project under consideration. It gives the financial manager the
instrument for the comparison among projects in percentage terms. This method can be used to
rank the projects of equal lives and risks. But it is not useful to rank the mutually exclusive
projects of unequal lives and different risks.
d) Leveraged Buyout – It is an ownership transfer consummated primarily with debt. Sometimes
it is also called as asset- based financing, the debt is secured by the assets of the enterprise
involved. While some leveraged buyouts involve the acquisition of an entire company, many
involve the purchase of a division of a company or some other subunit. Frequently the sale is to
the management of the division being sold, the company having decided that the division no
longer fits its strategic objectives. Another distinct feature is that leveraged buyouts are cash
purchases, as opposed to stock purchases. Finally the business unit involved invariably becomes
a privately held as opposed to a publicly held company.

7. Distinguish between: (4x2.5=10)


a) Systematic Risk and Unsystematic Risk
b) Investment Decision and Financing Decision
c) Bonus share and stock-split
d) Operating leverage and financial leverage
Answer No.7
a) Systematic Risk and Unsystematic Risk. – Systematic risk is the variability of a security's
return with that of the overall stock market. Risks of inflation, Interest Rate Risk are example of
The Institute of Chartered Accountants of Nepal
14 of 97
Suggested Answers of Financial Management
CAP II Examination – December2010

this kind of risk. This type of risk affects all firms in the economy and a particular firm cannot
avoid it. This is also known as Unavoidable Risk.
Unsystematic risk is the amount of a stock's variance unexplained by overall market movements.
It can be diversified away; hence it is known as Avoidable Risk. A strike may affect only one
company; a new competitor may begin to produce essentially the same product; a technological
breakthrough can make an existing product obsolete. However, by diversification this kind of
risk can be reduced and even eliminated if diversification is efficient.

b) Investment Decision and Financing Decision - Investment decision refers to the capital
expenditure decision. Companies lock up a large amount of funds for future benefits as a result
of the investment decision. It involves risk and it changes the business risk complexion of a
company. In addition, it brings about changes in the composition of assets and determines the
total amount of assets held by a company. This has two important aspects: the evaluation of the
profitability of the project and measurement of cut off rate of return.
After determining the total financial requirement, a financial manager should decide when,
where from and how to raise funds to meet the financial requirement to implement the
investment decision. Funds can be raised from different sources like Short term or Long term.
Funds may be raised by issuing different instruments of debt such as bonds, debenture,
commercial papers and issuing common or preference shares. Cost of the fund varies according
to the sources. A financial manager should make appropriate mix up of funds raised from
different sources in order to minimize the overall cost of capital and maximize the value of the
firm.

c) A bonus share is simply the payment of additional ordinary shares to the existing
shareholders. It is only a bookkeeping shift from the reserve and surplus account to the ordinary
share capital account of the company. A shareholder‘s proportional ownership in the firms
remains unchanged following the bonus issue.
Bonus shares are sometimes employed to conserve cash. Instead of increasing cash dividend as
earnings rise, a company may desire to retain a greater proportion of its earnings and declare the
issue of bonus share.
Stock split is an increase in the number of shares outstanding by a proportional reduction in the
face value of the share. The main purpose behind the stock split is to place the company‘s share
in a more popular trading range thus attracting more buyers.
In the issue of bonus share, face value of the share remains unaffected. On the other hand, a
share split causes the face value to come below the previous value.
Unless there is an increase in the earnings of the company, bonus issue will have the effect of
bring down the earnings per share. Accordingly, it will be difficult for the company to maintain
the earlier dividend per share. Similarly, it will be difficult for a company to maintain the same
cash dividend per share before and after a stock split.

d) Operating leverage occurs when there is fixed operating cost associated with the production
of goods and services. Fixed operating costs are incurred with an assumption that sales volume
will produce revenues more than sufficient to cover all fixed and variable operating costs.
Fixed operating costs do not vary with the change in the volume. On the other hand, variable
operating costs vary directly with the level of output. Therefore, if volume is to change, it is the
effect of fixed operating costs which causes the profit of a firm to change.

The Institute of Chartered Accountants of Nepal


15 of 97
Suggested Answers of Financial Management
CAP II Examination – December2010

The effect of presence of fixed operating costs (or operating leverage) is that a change in the
volume of sales will bring about more than proportional change in operating profit (or loss) of
the company.
Financing leverage is due to the use of fixed financing costs by the firm. It is employed with a
view to increase the return to ordinary shareholders. Favourable leverage occurs when the firm
used funds obtained at a fixed cost to earn more than the fixed cost of financing paid by it. If any
profit is left after paying the fixed financing costs, it belongs to the ordinary shareholders.
There is no choice for the management on the operating fixed costs. For example, a heavy
industry requires huge investment resulting in a large operating cost in the form of depreciation.
This cannot be avoided. On the other hand, financial leverage is always a choice item. Firms
need not have financing through long-term debt or preference share. They have the option to
finance their operations and capital expenditures from internal sources and through the issue of
equity shares.

The Institute of Chartered Accountants of Nepal


16 of 97
The Institute of Chartered Accountants of Nepal
Suggested Answers of Business Communication and Marketing

CAP II Examination – December 2010

Marks
All Questions are compulsory.

Section -'A': Business Communication


1. Re
ad the following case on Pillsbury carefully and respond the questions based on it. (4 2.5=10)
In the early 1990s, few formal channels of communication were in place at Pillsbury. When new
management arrived and the company was restructured, all that changed. ―We started a business-
focused employee newspaper,‖ said Lou de Ocejo, human resources vice president. ―We started
having CEO luncheons every month, and communication meetings with all functions at least once a
quarter. We tried to do a lot of things to tell people what this new age at Pillsbury was all about.‖
Most difficult, though was getting employees to talk back. Management wanted to know what was
going right and what was going wrong. It wanted product ideas and suggestions for improving
procedures. But employees, in most organizations, are cautious, despite pep talks from managers
about how their doors are always open.
Then Pillsbury installed a unique feedback tool, a hotline called ―In Touch.‖ This toll-free, third-
party voice mail service allows employees to leave anonymous messages about any subject at any
time. At first, employees were cynical, but over time they began to respond. More than 2,300
messages have now been recorded. Employees reported such problems as faulty work systems and
ineffective supervisors. But they also provided many new product ideas and cost-saving
suggestions, such as recommendations for pizza toppings and ways to recycle surplus paper.
―Getting this feedback wasn‘t fun the first time out, and sometimes it still isn‘t,‖ reported Lou de
Ocejo, ―but the system does just what we need it to do.‖
Issues:
a) Why are informal channels of communication dangerous in times of organizational upheaval?
b) What obstacles may interrupt the downward, upward, and horizontal flow of information in
organizations undergoing restructuring, such as Pillsbury?
c) How could Pillsbury‘s ―In Touch‖ voice mail system break down common barriers to
communication?
d) Why do some companies continue feedback system when it isn‘t always fun, and sometimes it‘s
even offensive?
Answer to Q. no. 1
a) Informal channels of communication are asystematic, and they mostly create rumors which are
often false. At the time of organizational upheaval such false rumors may obstruct the
organizational establishment and hinder the work. Since it is not clearly known to staff and other
related personnel what will happen the next, such communication channels create confusion and
insecurity in them.
b) Organizational restructuring also suggests restructuring of attitude, feelings and emotions, a factor
that is hard to change. For instance, when Pillsbury decided to lunch an open communication tool
and environment, it was not just about external transformation. It entailed the rewiring of inner
world of relations and emotions. It is difficult for top executives to smother their egos and be ready
to listen to their subordinates and hear complains and suggestions against them. The subordinate
may not be that open as expected because they may fear the indirect reprimanding. So far as
Suggested Answers of Business Communication and Marketing
CAP II Examination – December2010

horizontal peers are concerned, their ego and sense of competition may come in between marring
the ideal set for the reconstruction.
c) Pillsbury‘s ―InTouch‖ voicemail system proved to be comfortable to all the employees since they
could drop their messages and suggestions anonymously. The messages and criticisms which the
employees can not put personally against their managers were also posted with ease. In this way,
Pillsbury‘s voicemail system could break the common communication barrier.
d) Though sometimes such voicemail feedback system can be harmful and often offensive, it often
helps the people in management to know if there is any problem or malfunctioning in the company.
Besides, some of the feedbacks really become constructive. Therefore, the companies continue with
the feedback even though it‘s not always fun.

2. W
hat is a resume? Assuming that the UNDP Office in Nepal is looking for an Administrative
Assistant to be placed in its Head Office in Kathmandu, prepare a complete persuasive resume.
(2+8=10)
Answer to Q. no. 2
Resume or CV is a document that describes the applicant‘s qualifications including his/her
profile, goals, skills, academic degree, training and professional experience so that the prospective
employers can see an overall preview of how he/she can contribute to their company in capacity as
the post applied. All these details in a resume are presented in a standard and recognized order; they
cannot be put down in a random fashion. Down below is a sample of a resume:

Sample Resume for Administrative Assistant

Prem Basnet
Address: Kathmandu Metropolitan City-10, New Baneshwor, Kathmandu
Phone: 977-01-4456783 Cell: 977-01-9841345217
Email: mail2prembasnet@yahoo.com
Sex: Male, Date of Birth: September 23, 1980

Summary:
Experienced administrative assistant with ten years of experience providing support to executives
and school administrators. Expert in all office technology and practices. Excellent interpersonal
skills. Works with accuracy and efficiency. Earned Associates Degree in Business Administration
from Tribhuvan University.

Skills:
 Filing
 10-key calculator
 Office machines
 Communications
 Correspondence
 Client relations
 Mail delivery
 Highly organized

The Institute of Chartered Accountants of Nepal


18 of 97
Suggested Answers of Business Communication and Marketing
CAP II Examination – December2010

 MS Word & MS Excel


 Procurement
 Bookkeeping

Experience:
Kathmandu University, Kathmandu 2007 - till date
Administrative Assistant
Perform a broad range of administrative and clerical duties to the Foreign Language Department.
Input student grades data into online database, help in departmental scholarship award decisions,
and provide administrative support student TAs.
 Designed templates to automate production of award letters and other common
correspondence, reducing generation time by 55%.
 Performed accurate and rapid data entry to ensure timely grade postings.
 Created Excel Spreadsheets to summarize grade distribution for university administrators.
 Handled large phone system for professors and students.
 Developed thorough guidelines for operating office equipment to reduce repair calls by
10%.

Lord Buddha Academy, Manipal University, Kathmandu 2005-2007

Office Assistant

Performed various clerical and administrative duties for Lord Buddha Academy for Biotechnology
and Interior Design Departments, including scheduling meetings and appointments, department
publicity, typing, filing, emailing and answering phones.

 Helped with grading and the posting of grades.


 Organized departmental meeting schedules to eliminate conflicts and ensure attendance.
 Arranged meetings with students and faculty. Organized student contact information and
messages for professors.

Crimson International College, Kathmandu 2003-2005

Clerk

Reported directly to principal, provided key clerical support for 30 teachers and 400 students.
Worked directly with parents, students, suppliers and the media.

 Handled large phone system.


 Organized principal‘s mail and incoming calls to reduce interruptions.
 Achieved 20% discount by ordering college supplies in bulk quarterly.
 Wrote and edited monthly newsletters to parents.
 Created and distributed reports to assist principal with college scheduling: calendars, lunch
verification, student progress report, supplies, inventory and mailing databases.

Education and Training

The Institute of Chartered Accountants of Nepal


19 of 97
Suggested Answers of Business Communication and Marketing
CAP II Examination – December2010

Associate of Arts Degree (Business Administration), 2004

Shankardev Campus, Tribhuvan University, Kathmandu, Nepal

Additional Courses

Microsoft Office for Administrative Assistant

Basic Bookkeeping

FAFSA Processing

Customer Service Training

3. Pr
epare a full-fledged meeting minute of the AGM of any national financial institution in capacity as
the institution‘s Member Secretary. 10
Answer to Q. no. 3

Sample Meeting Minutes of AGM

October 30, 2010

Date: Monday 29, 2010

Meeting Start Time: 10:00 am

Meeting Finish Time: 1:00 pm

Chairperson: Rajendra Shahi

Present: Sabita Nakarmi (Deputy Chairperson), Sajan Sakya (Director), Drona Acharya (Managing
Director), Krishna Belbase (Director), Sharmila Panthi (Director)

In Attendance: Shreedhar Gautam (Member Secretary and CEO), 25% shareholders of the
institution‘s shares and ten observers.

Location: Rastriya Shabhagriha, Exhibition Road, Kathmandu

1. Apologies: Nil
2. Chairperson’s Address:
The Chairperson opened the meeting at 10.00 am as scheduled and noted that as more than five
shareholders (representing greater than 50% of the institution‘s issued shares) were in attendance, a
quorum existed and the meeting was formally declared open.

The Institute of Chartered Accountants of Nepal


20 of 97
Suggested Answers of Business Communication and Marketing
CAP II Examination – December2010

The Chairperson indicated that the meeting was convened under Notice of Meeting distributed
on 26 September 2010 which was greater than 28 days from today‘s date and therefore the meeting
was appropriately constituted.
The Chairperson indicated that the Minutes of the last General Meeting of the institution held on
October 25 2009 were available to the meeting.
Proxy details were read to the meeting and distributed as per the attached schedule 26
September 2010.

3. The Chairperson invited the Managing Director to address the meeting on current matters
relating to the institution’s business.

4. Financial Statements and Reports

4.1 The meeting received and considered the financial statements and reports for the year ending 30
July 2010. These statements were included in the annual report to all shareholders and are tabled
here.

4.2 It was noted that there was no statutory requirement for these statements to be adopted, passed
or resolved at this Annual General Meeting and gave the shareholders the opportunity to ask
questions.

4.3 A number of questions raised by shareholders were responded to by the Chairperson and the
Managing Director.

5. Remuneration Report (Resolution 1)

5.1 The Chairperson proposed that the Remuneration Report as set out in the Annual Report for the
financial year ended 30 July 2010 be adopted. The Chairperson indicated that this resolution
required an advisory vote only from shareholders and does not serve to bind the institution or its
Directors.

5.2 The Chairperson invited questions in connection with the Remuneration Report. Questions
raised by shareholders were responded to by the Chairperson.

5.3 Resolution was seconded by Suman Pokhrel.

5.4 Resolution was passed on a show of hands.

6. Re-election of Sajan Sakya (Resolution 2)

6.1 The Chairperson proposed that Mr. Sajan Sakya , a Director retiring by rotation in accordance
with clause 24 of the institution‘s constitution, and being eligible, be re-elected as a director of the
institution.

6.2 Resolution was seconded by Sakuntala Sapkota.

6.3 Resolution was passed on a show of hands.


The Institute of Chartered Accountants of Nepal
21 of 97
Suggested Answers of Business Communication and Marketing
CAP II Examination – December2010

7. Re-election of Mrs. Sharmila Panthi (Resolution 3)

7.1 The Chairperson proposed that Mrs. Sharmila Panthi, being a Director appointed during the year
and being eligible, be re-elected as a Director of the institution.

7.2 Resolution was seconded by Pramod Thakuri.

7.3 Resolution was passed on a unanimous show of hands.

The Chairperson thanked the staff and Directors for the efforts during the past successful year.
The Chairperson also thanked our legal team, Hari Sapkota and Chidananda Yadav for their
continued support along with shareholders for their support and attendance at the meeting.

There being no further business the meeting closed at 1.00 pm.

4. Di
scuss listening process and its barriers. 10
Answer to Q. no. 4
Listening takes place in four stages—perception, interpretation, evaluation, and action. Barriers,
however, can obstruct the listening process. These barriers may be physical or mental.
Perception: The listening process begins when you hear sounds and concentrate on them. There
may be numerous kinds of sounds around us, but until we tuned to them these sounds just go
unnoticed. The conscious act of listening begins when we focus on the sounds around us and select
those we choose to hear. We tune in when we sense that the message is important, when we are
interested in the topic, or when we are in the mood to listen. Perception is reduced by impaired
hearing, noisy surrounding, inattention, and pseudolistening. Pseudolistening occurs when listeners
―fake‖ it. They look as if they are listening, but their minds are wandering far off.
Interpretation: Once we have focused our attention on a sound or message, we begin to interpret,
or decode, it. Interpretation of a message is colored by our cultural, educational and social frames of
reference. The meanings we attach to the speaker‘s words are filtered through oer expectations and
total life experiences. Thus, our interpretation of the speaker‘s meaning may be quite different from
what the speaker intended because our frame of reference is different.
Evaluation: After interpreting the meaning of a message, you analyze its merit and draw
conclusions. To do this, we attempt to separate fact from opinion. Good listeners try to be objective,
and they avoid prejudging the message. The appearance and mannerism of the speaker can also
affect a listener‘s evaluation of a message. A juror, for example, might jump to the conclusion that
an accused man is guilty because of his fierce expression or his substandard English. Thus, to
evaluate a message accurately and objectively, we should consider all the information, be aware of
our own biases, and avoid jumping to hasty conclusions.
Action: Responding to a message may involve storing the message in memory for future use,
reacting with a physical response (a frown, a smile, a laugh), or supplying feedback to the speaker.
Listener feedback is essential because it helps clarify the message so that it can be decoded
accurately. Feedback also helps the speaker to find out whether the message is getting through
clearly. In one-to-one conversation, of course, no clear distinction exists between the roles of
listener and speaker—we give or receive feedback as our role alternates.

The Institute of Chartered Accountants of Nepal


22 of 97
Suggested Answers of Business Communication and Marketing
CAP II Examination – December2010

5. W
rite short notes on the followings (Any Four): (4×2.5=10)
c) Letter of authorization
d) Upward communication
e) Career objectives
f) Internal unethical dealing and external unethical dealing
g) Use of graphics in non-verbal communication

Answer to Q. no. 5
a) Unless the report is self initiated, someone has asked you to produce a report. Hence, your opening
words in the transmittal document refer to that request and it is called the letter of authorization.
Include the date. Remember that much time may have passed since the initial request and that even
the requester may have forgotten the details. Some report writers include in the appendix the actual
report request as supplementary information.
b) Upward communication, also known as bottom-up communication, is the one in which
communication takes place from bottom to top level. Here, juniors communicate as senders to the
seniors as receivers. Opposite to the downward communication, it is the democratic form of
communication in which lower ranking employees can have communication access the high ranking
staff.
c) Opinion is divided on the effect of including a career objective on a resume. Recruiters think such
statements indicate that a candidate has made a commitment to a career. Moreover, career
objectives make the recruiter‘s life easier by quickly classifying the resume. But such declarations
can also disqualify a candidate if the stated objective doesn‘t match a company‘s job description.
d) Internal and external unethical dealings: Some organizations violate ethical morals to stand and
show themselves as good achievers or to make progress overnight without making any great efforts.
In doing this, established ethical norms and values are violated and unethical things are done.
The unethical practices within the organization is called internal unethical dealing and the other in
which one organization violates ethical norms and values in its dealing with external organizations
is called unethical external dealing.

e) Graphics are presentation in lines, boxes, circles, figures, and sketches used to explain and
clarify things which words may or may not do. As a part of sign language, it is abundantly used in
non-verbal communication, where words can be of little or no use.

The Institute of Chartered Accountants of Nepal


23 of 97
Suggested Answers of Business Communication and Marketing
CAP II Examination – December2010

Section -'B': Marketing

6. Read the following case carefully and answer the question that follows:

Close-up, the youth brand, was the first gel toothpaste launched in India in 1975 and has remained
the category leader ever since. The brand arose out of a universal need for confidence in a social
situation, starting with fresh breath. In 2002, Close-Up was re-launched in two variants – Close-Up
Tingly Red and Close-Up Eucalyptus Waves. Both products with their superior formulation provide
superior cleaning, enhanced flavor and a freshness that extends from nose to throat. The
communication for Close-Up focuses on the base line ‗for the most kissable fresh breath‘ which
encourages young people to go out and get a life.

The new flavors are in line with HLL‘s (Hindustan Lever Limited) endeavor continuously innovates
and offer new advanced products to the consumers. These flavors have been especially designed to
win over competition consumers.

Recently, Close-Up Whitening, the tooth whitening variant, was introduced with a unique self-
check device called ―shade card‖. This helps consumers identify how ‗yellow‘ their teeth really are
and also to track the improvement in teeth color as they use the paste. The toothpaste helps teeth get
whiter in four weeks of regular use.
Questions:

a) O
n the basis of above given information explain the product attributes of Close-up and marketing
strategy of HHL 5

b) Id
entify the target market for Close-Up and provide some suggestions to increase its market share. 5

Answer to Q. No. 6

a)
So far product attributes of close-up is concerned in a long history it has gone through changes,
modification looking at competition. Superior cleaning, safety of teeth, beauty, flavor, freshness
from nose to throats, tooth whitening variant, self check device, kissable breath, etc. are the major
attributes found in close-up tooth paste targeted mainly to young people.

Hindustan Lever Limited (HLL) has to follow product line expansion and product positioning
strategy as most suitable marketing strategy to satisfy consumers as well as face competition.

b)
Target market of HLL seems to be kissable fresh breath to teenagers. Now product line expansion
as well as whitening teeth to attract new groups to product line. As stated earlier favorable product

The Institute of Chartered Accountants of Nepal


24 of 97
Suggested Answers of Business Communication and Marketing
CAP II Examination – December2010

attributes of close-up has made it capable to attract large number of consumers.

HLL has followed good marketing strategy leading towards success. It has to continue to study
market as well as customers reactions, regular monitoring of the market and maintain product
quality, whitening teeth and kissable breath can be the appropriate strategy to increase its market
share.

7.
a) Explain the importance of marketing to consumers. 5

b)
What are the ingredients of the marketing concept? Describe in short. 5

Answer to Q. No. 7

a)
i. Delivering the standard of living
Marketing delivers the standard of living to consumers by offering products and services they need.
If the firm doesn‘t perform marketing properly, consumers will be dissatisfied with the quality of
products and product performance. That‘s why, the marketing firm offers its product in accordance
with glad consumer who dictates.

ii. Providing various information


Marketing provides consumer with various information about the product such as product attributes,
product performance, price, benefits that they get after buying, product availability in right quantity
at right place method of use etc. As a result, they become more selective and efficient in buying
products of their need.

iii. Improving quality of life


Marketing plays its role in improving consumer‘s quality of life by encouraging producers to
produce safer and hygienic products. Manufacturers produce their products considering public
service messages. That is why, many promising firms have started to write on the packages ―sugar
free food, cholesterol free oils and foods, antiseptic cream, friendly instruments of family planning
etc.

iv. Enhancing beliefs


By providing qualitative and affordable products and services to consumers through various
convenient distributing centers and facilitating after sale services, the firm enhances consumer
beliefs on products and services.

v. Promoting life style


Marketing promotes consumer‘s choice relating to the quality, size, model and design of products
that they use are advanced. The advanced family prefers foreign goods where as the average people
rely on the products of value satisfaction. Today‘s sophisticated, luxurious, advanced products make
people‘s life more comfortable.
The Institute of Chartered Accountants of Nepal
25 of 97
Suggested Answers of Business Communication and Marketing
CAP II Examination – December2010

b)
i. Target market Selection
Infact, everyone has differed needs and wants. Hence, marketers moves toward identifying
segments that are highly affected by demographic, phychographic and behavioral need differences.
Thus, marketers do best when they select their target markets carefully and prepare tailored
marketing progam.

ii. Customer needs evaluation


All manufacturing activities of the organization should be focused on determining and satisfying
target customers needs and wants. These needs are defined from the customer point of view i.e.
stated needs, real needs, unstated needs, delight needs and secret needs.

iii. Choosing integrated marketing programmes


In modern marketing environment, delivers of customer value and satisfaction is complex task.
When all the company‘s departments work together to serve the customer‘s interests, the result is
integrated marketing. Integrated marketing takes place on three levels i.e.
First, Coordination with in marketing department
Second, Coordination with in entire organization.
Third, Coordination with concerned external institutions (the key parties)

iv. Achieving marketing objectives


The ultimate purpose of the marketing concept is to achieve marketing objectives i.e. profit, sales
volume, market share survive and growth through customer satisfaction.

8.
a) What is market segmentation? List the factors that are required for effective market
segmentation. 5

b) Describe the main price discounts popularly used by marketers in Nepal. 5

Answer to Q. No. 8
a)
Market segmentation is a process of dividing the total market for a product or service into several
smaller groups, such that the members of each group are similar with respect to the factors that
influence demand .By dividing the total market to distinct smaller groups helps to identify real need
and characteristics of buyers. Mass marketing, product variety marketing, individual marketing and
target marketing are some of the common approaches found in dealing with the market.

Following factors are to be considered for marking market segmentation effective:

 Heterogeneous needs and characteristics


 The needs and characteristics are measurable
 The market is accessible to the marketer, and
The Institute of Chartered Accountants of Nepal
26 of 97
Suggested Answers of Business Communication and Marketing
CAP II Examination – December2010

 It is large enough to be profitable.

b)
Discounts are the rates or percentages of the deduction granted by the seller to the customer for
making purchase of goods. In other words, discount means to put a reduced value on the product.
While allowances are also discounts allowed for the task or work performed accordingly.
There are several forms of discounts and allowances;
i. Quantity discount: Quantity discounts are the deductions from a seller‘s
price list intended to encourage customers to buy in larges amounts. Therefore, the customers
buying larger quantity get high discount than the customers buying small quantity.
ii. Trade discount: Trade discounts are the deductions from manufacturer‘s
price list offered to buyers (i.e., resellers including wholesalers and retailers) in payment for
marketing functions the buyers will perform. Therefore, the manufacturer provides trade discounts
to the wholesalers and retailers for buying and selling his products to the final consumers.
iii. Cash discount: Cash discounts are deductions granted by the seller to the
buyers for paying their bills within a specified time, or for cash sales. Therefore, the customers
willing to make cash payment upon the purchase of goods from the seller get better or attractive
discount compared to the customers willing to purchase goods on credit.
iv. Price-off: Price-off discounts are deductions granted by a manufacturer to
the customers temporarily to attract their attention towards the product. It is printed in the package
and the discounts are granted for a specified time may for one week, two weeks, or month. But it
may not be for more than three months.
v. Seasonal discount: Seasonal discounts are deductions granted by the traders
like wholesalers and retailers to the buyers to reduce their old stock of goods. This type of discount
is provided to the seasonal goods such as readymade clothes, wool products, etc.

9.
a) What are the messages that the product life-cycle communicates to the marketers? 5
b) Discuss main features of organizational buying behavior. 5
Answer to Q. No. 9
a)
Actually, the product life cycle is an attempt to recognize distinct stages in the sales and profit
history of the product. To say that a product has a life cycle is to communicate the following four
things:
 Products have a limited life after which the product may be dead if
appropriate strategies are not adopted accordingly.
 Product sales pass through distinct stages, each posing different challenges to
the seller.
 Product profits rise and fall at different stages of the product life cycle.
 Products require different marketing, financial, manufacturing, purchasing,
and personnel strategies at different stages of their life cycle.

b)
The nature of business market demand has some distinct characteristics;
The Institute of Chartered Accountants of Nepal
27 of 97
Suggested Answers of Business Communication and Marketing
CAP II Examination – December2010

i. Demand is derived from the demand for the consumer products in which that
business product is used;
ii. Demand for business products is inelastic i.e., there is a very little changes in
the demand in response to its price;
iii. Demand is widely fluctuating i.e., business users take more inventories or
less inventories when they expect drastic changes that may take place in prices of the products or in
national economy;
iv. Buyers are well informed about what they are buying than are ultimate
consumers.
v. Organizational buying is usually regulated by law. Every organization may
have ites own purchase policy and rule regarding the purchase of necessary goods and services from
the seller or suppliers. No organization can buy on random basis as does by the individual cosumer.
Organizations have to follow its purchase policy and rules.
vi. Organizational buyers usually buy goods in lot sizes as compared to the
individual consumer.

10. Briefly explain any five of the followings: (5×2=10)

a) Warehousing
b) Marketing research.
c) Application of marketing mix in business.
d) E-commerce
e) Push and pull strategy.
f) Sales promotion and its methods

Answer to Q. No. 10
a)

The word storage means holding the stock of goods for a longer period as the goods are not
immediately in demand, Warehousing involves more than storage. Warehouse perform many
functions of wholesaler, retailer and producer. Products held in inventory are physically stored in a
warehouse. Warehousing embraces a set of functions such as assembling, dividing and storing
products and preparing them for re-shipping. Warehousing vests the products with time and place
utility and in the case of some commodities. ‗form utility as well‘

b)
Marketing research is a systematic and objective search for and analysis of information relevant to
the identification and solution of marketing problems. Marketing research is generally conducted to
solve a particular problem of the firm. It obtains unique information for special decision-making
needs. It is considered as apart of the MIS because it can develop large amounts of marketing
information.
The main function or objective of marketing research is to provide most relevant or most pertinent
information to the managers for marketing decision-making. However, an ideal marketing research
must need to include at least four qualities or terms:
 It is a systematic search

The Institute of Chartered Accountants of Nepal


28 of 97
Suggested Answers of Business Communication and Marketing
CAP II Examination – December2010

 It is objective search
 It is an informative search
 It is a problem-focused study
 It is a decisive search

c)
Marketing mix is applied in business basically to match the firm‘s product/services to the target
customers or target market. In other words, marketing mix is applied in business to design and offer
a right product with right price to right customers at right place at right time or right market
situation communicating through right promotion tools.

d)
E-commerce relates with selling products and services electronically i.e. through internet all
marketing efforts to inform, promote, and sale of products. It is a part of e-business which is done
through online marketing. Consumers place order for the product and services and sell them through
internet. The online systems of marketing is becoming popular as direct marketing channel. Internet
as global web of computer network, facilitated by different user friendly user friendly software to
connect business to consumers in general. E-mail has also contributed in making communication
link between the company and consumers.

e)
Push and pull strategy is one of the factor affecting promotion mix of a company. A good
promotion blend is very important for successful marketing. The promotion mix is also affected by
whether companies adopt push or pull strategy.

In push strategy the promotion is directed at the marketing intermediaries (sales force). The
manufacturer persuades wholesales to carry the product, while wholesalers convince retailers to sell
the product to consumers and retailers pushes the products to consumers by convincing at the time
of sale and displays.
But under pull strategy the producer build consumer awareness through mass advertising. Retailers
place orders to wholesalers and wholesalers to manufacturer to meet customer demand. Consumer
promotions are stressed as demand comes from down to upward direction.

f)
Sales promotion involves marketing activities, other than advertising, publicity, or personal selling,
that stimulate consumer purchases and dealer effectiveness. The forms of sales promotion include
trade shows, demonstrations, coupons, rebates, samples, contests, and many additional activities
that bridge personal selling and advertising.
Sales promotion tools are generally short term measures designed to stimulate quicker and greater
purchase of products or services by consumers or traders. The use of sales promotion has been
significantly increasing in the modern business. It is especially due to increase in competition is the
market. Therefore, sales promotion is also called as ―aggressive selling‖.
Sales promotion tools are directed resellers or channel members, and sales force of the company.
Therefore, those are three methods of sales promotion namely:
i. Consumer promotion,
ii. Trade or Dealer promotion, and
iii. Sales force promotion.
The Institute of Chartered Accountants of Nepal
29 of 97
Suggested Answers of Business Communication and Marketing
CAP II Examination – December2010

For each target group, usually separate sales promotion tools are offered.

The Institute of Chartered Accountants of Nepal


30 of 97
The Institute of Chartered Accountants of Nepal
Suggested Answers of Income Tax and VAT

CAP II Examination – December 2010


Marks

Attempt all questions. Working note should form part of the answer.
4. Answer the following with reference to the Indian Income Tax Act, 1961.
a) Which are the incomes exempted under section 11? 3
b) How would you determine the residential status of a company? Can a company be not
ordinarily resident in India? 3
c) Which period will be treated as previous year for Income Tax purposes for the assessment
year 2009-10 in the following cases: 4
i) Amit starts a new business on 1-11-2008 and prepared final accounts on 6-6-2009.
ii) Mohan joined service in a company on 1-1-2009 at Rs. 16,000 per month. His next
increment in salary will be on 1-1-2010. Prior to this he was unemployed.
iii) Sunil Sharma is a registered doctor and keeps his Income and Expenditure Account on
calendar year basis.
iv) Ashish bought a house on 1-8-2008 and let it out at Rs. 80,000 per month.

Answer No.1
a) The following incomes of a religious or charitable trust or institution are not included in the
total income:

i. Income from property held under trust wholly for charitable or religious purposes.
ii. Income from property held under trust which is applied in part only for charitable or
religious purposes.
iii. Income from property held under trust which is applied for charitable purposes outside
India.
iv. Voluntary contributions forming part of corpus.

The above incomes of a religious or charitable trust will be exempted provided it complied the other
relevant provisions of Income Tax Act, 1961.

b) As per section 6(3), a company is said to be a resident in India in any previous year if:
i. It is an Indian company, or
ii. During the relevant previous year, the control and management of its affairs is situated
wholly in India.
Excepting individual and HUF, all other persons are classified either as resident or non-resident.
They are not to be further classified as ordinary resident or as not ordinary resident. Therefore, a
company cannot be not ordinarily resident in India.

c) The following will be the related previous year for income tax purposes for the assessment year
2009-10:
i. 1-11-2008 to 31-3-2009
ii. 1-1-2009 to 31-3-2009
iii. 1-4-2008 to 31-3-2009
iv. 1-8-2008 to 31-3-2009
Suggested Answers of Income Tax and VAT
CAP II Examination – December2010

5.
a) Mr. A, a resident natural person, is a musician deriving income from concerts performed
from various countries outside Nepal. During Financial Year 2066/67, he performed
concerts in India. Details of his assessable income and tax paid in the country as aforesaid
where the concerts were given are:
Income Tax
India Rs. 1,000,000 Rs. 300,000

Also, he earned Rs. 5,00,000 in Nepal during the financial year 2066/67.
Assuming that he has chosen to be couple, find his tax liability on his total income and
amount for foreign tax credit available to him under section 71(1) for the income year
2066/67. 10
b) Mr. A has been working in the capacity of senior manager for XYZ Co. Ltd. The company
provided him a Volkswagon car for official as well as personal use. The current market price
of the car is Rs. 57,00,000. Details of Mr. A‘s remuneration during the Income year 2066/67
are as follows:
Salary Rs. 1,50,000 per month
Grade Rs. 75,000 per annum
Allowance Rs. 7,000 per month
Other Allowances Rs. 9,000 per month
Quantify the amount for the vehicle facility provided for computing Mr. A‘s taxable income
from remuneration for the F/Y 2066/67.
Will the answer be different in case XYZ Co. Ltd. hired Mr. A for four months in the
capacity of consultant for a fee of Rs. 7,00,000 in total? 5
c) Mr. Ram & Mr. Shyam jointly owned a house property costing Rs. 5,00,00,000. At the time
of acquisition of the said house property, Mr. Ram had invested Rs. 1 crore and rest of the
amount invested by Mr. Shyam. This house property has been sold out for Rs. 6,40,00,000,
and incurred Rs. 7,00,000 on account of brokerage commission plus other incidental
charges.
You are required to appropriate between Mr. Ram & Mr. Shyam- as per sec 30 of
Income Tax Act 2058- for the net income that originated from the jointly owned investment.
5
Answer No.2
a) Income earned
Nepal Rs. 5,00,000
India Rs. 10,00,000
Total assessable income Rs. 15,00,000
Less admissible deductions _____-______
Total taxable income Rs. 15,00,000

Total tax liability (working note) Rs. 3,15,000


Less Allowable foreign tax credit:
On Indian Income (working note) Rs. (2,10,000)
Net tax liability Rs. 1,05,000

The Institute of Chartered Accountants of Nepal


32 of 97
Suggested Answers of Income Tax and VAT
CAP II Examination – December2010

Working note:
Computation of average tax rate for foreign tax credit

Assessable income earned in Nepal Rs. 5,00,000


Assessable income earned in India Rs. 10,00,000
Total assessable income Rs. 15,00,000
Less: deductions ______-_____
Total taxable income Rs. 15,00,000

Tax liability computation: (India)


Up to Rs. 2,00,000 Rs. Nil
Next Rs. 1,00,000 @ 15% Rs. 15,000
Bal. Rs. 12,00,000 @ 25% Rs. 3,00,000
Total Rs. 3,15,000

Therefore, average tax rate= 3,15,000/15,00,000 * 100 = 21%


a) Assessable income earned in India = Rs. 10,00,000
b) Tax paid in India = Rs. 3,00,000
c) Tax to be paid as per average tax rate = Rs. 10,00,000 * 21% = Rs. 2,10,000
d) Excess tax paid to be carried forward for next year (b-c) = Rs. 90,000 [10]

Note: Due to different interpretations, in case any student charges 1% or Rs. 2,00,000, that
should also be treated as right answer.

b) As per the provisions of Income Tax Act 2058 and rules made there under, cost free vehicle facility
provided to be used for official and personal use by employer to employee, or worker or other
persons getting remuneration on monthly basis, has to be quantified in terms of value to be included
in his total remuneration for tax purpose. In such cases, value of such free of cost vehicle facility has
to be quantified at the rate of 0.5% of total salary and grade. Therefore, amount to be included in his
taxable income has been quantified as:

Salary (Rs. 1,50,000 *12) Rs. 18,00,000


Grade Rs. 75,000
Total salary with grade Rs. 18,75,000

Amount to be quantified = Rs. 18,75,000 * 0.5/100 = Rs. 9,375

Therefore, Rs. 9,375 has to be included in his remuneration on account of cost free vehicle provided
to be used for official as well as personal uses.

In case Mr. A is employed in the capacity of consultant then values for such cost free vehicle is to be
quantified for the purpose of including in his taxable business income. Such value to be quantified is
1% of the current market price of such vehicle or car.

Car will be quantified as:


1% of Rs. 57,00,000 = Rs. 57,000/3 = Rs. 19,000

Total taxable business income of Mr. A is


Consulting fees Rs. 7,00,000
The Institute of Chartered Accountants of Nepal
33 of 97
Suggested Answers of Income Tax and VAT
CAP II Examination – December2010

Cost free vehicle/car Rs. 19,000


Total Rs. 7,19,000

c) For the purpose of calculating a person‘s income from an investment which is jointly owned with
another person, amounts to be included and deducted in that calculations should be apportioned
among the joint in proportion to their respective interests in the investment. [sec 30]
Mr. Ram Mr. Shyam
Investment Rs. 1 crore Rs. 4 crore
Investment ratio is 1:4

Computation of net income:


Sales (Incoming) Rs. 6.40 crore
Less: brokerage commission Rs. 0.07 crore
Balance Rs. 6.33 crore
Total initial investment (Outgoing) Rs. 5.00 crore
Net Income Rs. 1.33 crore

Net income of Rs. 1.33 crore to be appropriated based on contributory ratio of 1:4 between Mr. Ram
& Mr. Shyam are as follows:

Mr. Ram Mr. Shyam Total


Net Income Rs. 26,60,000 Rs. 1,06,40,000 Rs. 1,33,00,000

6.
a) From the following Income Statement of M/S ABC Pvt. Ltd., prepare the taxable income for
the income year 2066/67. 10
Rs. In ‗000
Sales 8,000
Cost of sales 5,000
Gross profit 3,000
Administrative expenses 500
Donation 100
Interest expenses 300
Pollution control expenses 1,500
Research & Development expenses 1,000
Depreciation 50
Balance (450)
Other Income 50
Net Profit/ (loss) (400)
b) What is the provision under sec 12(ka) of Income Tax Act 2058 as regards expenses
incurred towards ancient, religious and cultural heritage conservation and sports
development activities? 5
c) Write in brief the provisions of Income Tax Act in case of a married resident couple elect to
be treated as a couple. 5

Answer No 3
a) Computation of taxable income of M/S ABC Pvt. Ltd. for the income year 2066/67.
Rs. in ‗000
Income

The Institute of Chartered Accountants of Nepal


34 of 97
Suggested Answers of Income Tax and VAT
CAP II Examination – December2010

Sales 8,000
Other income 50__
Total Income (A) 8,050
Less: admissible deductions
Cost of sales 5,000
Administrative expenses 500
Depreciation 50
Interest Expenses 300
5,850 5,850
Adjusted Taxable income 2,200
Research & Development expenses
50% of ATI Rs. 2200= Rs. 1,100
Actual expense Rs. 1,000 1,000

Pollution control expenses


50% of ATI Rs. 2200=Rs. 1,100
Actual expense Rs. 1,500 1,100

Donation
Actual expenses Rs. 100
5% of ATI Rs. 110
Maximum Rs. 100 100

Taxable income 0

Note 1: The unabsorbed pollution control cost Rs. 400 could be capitalized to Block D assets.

Note 2: It is supposed that donation is not given to tax exempted entity.

Note: In case any student supposes that the donation is not given to tax exempted entity, the answer
should also be treated right. [10]

b) Under the provisions of Income Tax Act, with the prior approval from the Inland Revenue
Department, a company may claim to have the taxable income from a income year reduced by
expenses incurred towards the conservation of Ancient, Religious and Cultural heritage in Nepal or
construction of the physical infrastructure of public interest under sports development in Nepal.
However, the deduction as aforesaid is limited to the lesser of Rs. 10 crore or 10% of assessable
income. [5]
c) In the case of a married resident couples, each spouse is taxed separately. However, married resident
couples may choose to be treated as a couple for a particular income year. In such cases, their
income will be clubbed and added together, and the basic exemption threshold for couple applies. If
a couple elects to be taxed as couple, each spouse is jointly and severely liable together with the
other spouse that the tax for the couple is paid.

If the income of both spouses is calculated separately, each spouse can claim the basic threshold
applicable for individuals

7.
a) Mr. Ram Kumar has opened a bank account with Grameen Biaks Bank in Parbat with a
deposit of Rs. 50,000 with interest at the rate of 8 percent per annum. During the year, the

The Institute of Chartered Accountants of Nepal


35 of 97
Suggested Answers of Income Tax and VAT
CAP II Examination – December2010

bank has to pay Mr. Ram Kumar Rs. 4000 as Bank Interest. The bank seeks for your advice
regarding the applicability of withholding tax?
If for same deposit of Rs. 50,000 with the interest rate of 24%, the bank has to pay Rs.
12,000 for that year, then will your answer differ? 5
b) A tax payer had applied for an administrative review with Director General of IRD as per
section 115 of the Act. But now he is not satisfied and got aggrieved by the decision of IRD
as per section 115. As a tax consultant, you have to advise him on the remedy available by
making appeal to revenue tribunal with the brief procedure for making it. 5
Answer No.4
a) As per section 88(4)(cha) of the act, in case of deposits made in rural community based
micro finance, grameen bikas bank, postal savings bank, cooperative as mentioned in section
11(2) of the act, interest is accrued upto Rs.10,000 per annnum, then withholding tax shall
not be applicable.
In the first case, annual interest from Grameen Bikas Bank is Rs. 4000 which is within
thelimit of Rs. 10,000, so withholding tax is not applicable. But in the second case, it
exceeds the limit of Rs.10,000, hence withholding tax is applicable.
b) As per section 116 of the act, a tax payer who is aggrieved by a decision of IRD on an
application filed as per section 115 or the tax payer has supposed that the application is
rejected due to expiry of a period of 60 days, may appeal to Revenue Tribunal in accordance
with Revenue Tribunal Act 2031. The tax payer who appeals with the revenue tribunal
should file a copy of the notice of the appeal with IRD within 15 days of doing so. As per
sec. 116(3) of the act, the enforcement of a decision taken by a tax officer shall not be
treated as stayed unless the tribunal stays or otherwise affects the enforcement of a decision
of the tax officer until the appeal is settled.
The procedures for making an appeal are given in Revenue Tribunal Rules. An applicant
should deposit an amount equal to a sum of 50% of the tax amount assessed by the tax
officer and total amount of fines imposed by him. An appeal to a revenue tribunal should be
filed within 35 days of the receipt of assessment order by the tax payer or the taxpayer
receives the decision of IRD or treats the application rejected by IRD.
The documents of appeal should include the following information:
 Names and addresses of the plaintiff and defendant.
 If the appeal is against any decision, the full details of the case.
 A statement that the appeal is filed within the prescribed time and the fees required is
enclosed with the appeal.
 Reasons of dissatisfaction, sections and rules in support of your appeal and demands of the
plaintiff.
 Any case history with regard to similar application.
 Any other course of action of the tribunal required by the plaintiff with regard to the appeal.

8.
Value Added Tax Act
a) State with reason whether the following statements are True or False. 10
The Institute of Chartered Accountants of Nepal
36 of 97
Suggested Answers of Income Tax and VAT
CAP II Examination – December2010

i) A person shall apply for the registration of VAT in the format prescribed in Schedule
– 3 of VAT Regulation, 2053.
ii) A shopkeeper can issue abbreviated tax invoice for the transaction upto Rs.10,000.
iii) A registered person shall preserve all the VAT records upto 6 years.
iv) A separate record for purchase and sale shall be maintained for the used goods which
have purchase price more than Rs. 20,000.
v) Government organizations shall take VAT bill purchasing goods and services above
Rs. 5,000.
vi) The rate of VAT for the import of goods and services will be same as VAT
applicable for the purchase of goods and services in Nepal.
vii) In the case of purchase and sale of land and building, VAT is not applicable.
viii) In the case of import of goods from USA, the place of transaction is USA.
ix) A foreign tourist can take refund of VAT on the purchase of goods amounting to Rs.
15,000 or more.
x) VAT will not applicable in cable car service.
b) Fat Limited has following transactions during the month of Baisakh, 2067, find the amount
of VAT credit available for the month of Jestha:
Amount (Rs.)
Opening Credit available 12,000
Purchases net of VAT 10,00,000
Salary for the month 1,50,000
Electricity expenses 10,000
Telephone expenses with VAT 13,560
Fuel expenses with VAT 28,250
Purchase of car with VAT 11,30,000
Purchase of office supplies net of VAT 1,50,000
Total sales for the month 15,00,000
Additional information:
All the sales of the company were VAT applicable. Out of office supplies Rs. 20,000
purchased by taking abbreviated tax invoice. In purchases, Rs. 375,000 is import of raw
material and customs office has valued these goods Rs. 450,000. Fuel expenses consists
expenses for petrol used for office vehicle. 6
c) A dealer manufactured goods worth Rs. 10 million which were exempt under schedule I of
VAT Act out of the goods purchased partly VAT exempt and partly goods liable to VAT.
VAT exempt goods amounting to Rs. 7 million were used for the production of goods which
were exported. The Dealer claims that all the exports were zero rated and he should get full
credit for VAT on exempted goods. Consider his claim and to set off the VAT paid by him
on the inputs. 4
Answer No 5
a) State with reason whether the following statements are True or False.

a) False. Application for VAT registration shall be filed in the format prescribed in
Schedule – 1 of VAT Regulation, 2053.

b) False. Abbreviated tax invoice can be issued by the shopkeeper for the transaction upto
Rs. 5,000.
The Institute of Chartered Accountants of Nepal
37 of 97
Suggested Answers of Income Tax and VAT
CAP II Examination – December2010

c) True. VAT records shall be preserved by registered person for 6 years (Rule 23(7) of
VAT Regulation, 2053)

d) False. Separate record shall be maintained for used goods costing Rs. 10,000 or more.

e) True. As per Rule 56 of VAT Regulation, Government organizations shall take VAT bill
for the purchases more than Rs. 5,000.
f) True. Same rate of VAT shall be applicable to import and local purchases.
g) True. Land and building is included in Schedule – I of VAT ACT.

h) False. In case of goods import in Nepal, customs entry point will be the place of
transaction.

i) True. As per section 25Ka of VAT Act, tourists can take refund of VAT on the purchase
of goods amounting to Rs. 15,000 or more.

j) False. VAT is applicable in Cable car service as it is excluded in Schedule – I of VAT


Act.
b)
Calculation of VAT Credit Available for Jestha, 2067
Amount (Rs.)
VAT collected in Sales 195,000
VAT Credit on Input:
VAT paid on purchases 139,750
VAT paid on telephone expenses 1,560
VAT credit available on car 52,000
VAT paid on office Supplies 16,900 210,210
Net VAT Credit for the month 15,210
Add: Opening credit available 12,000
Opening VAT credit available for the month of Jestha 27,210

Note:
1. VAT paid on purchases:
Local purchase (1,000,000-3,75,000)*13% 81,250
Import (450,000*13%) 58,500
Total VAT paid on purchases 139,750

2. Credit available on Car purchase


Total VAT paid 130,000
40% credit available 52,000

3. Credit available on VAT paid on Office Supplies


Total Office supplies Net of VAT 150,000
Less: Purchesed by taking Abbreviated Tax invoice 20,000

The Institute of Chartered Accountants of Nepal


38 of 97
Suggested Answers of Income Tax and VAT
CAP II Examination – December2010

Credit available on office supplies purchase 130,000


VAT Credit on office supplies (13% of Rs. 130,000) Rs. 16,900

4. VAT credit cannot be claimed in the purchases of petrol.

c)
No Vat is collectible on the sale of the goods whether those goods are sold locally or exported if
those goods are exempt under schedule I. Since the goods are listed in schedule I, according to
section 5(3) of VAT Act, the exports of those goods are not entitled to zero tax facility under
schedule II.

As per Section 5(3), no deduction or refund is applicable with regard to VAT paid on goods used
for VAT exempt transaction. Hence, the dealer can not claim any input VAT paid on the purchases
against his export sale also in this case.

9.
a) Gi
nger Cellular Ltd., a cellular mobile Phone Set producing domestic enterprise, is engaged in
producing and selling of cellular mobile phone sets. As a tax expert, Ginger Ltd. seeks your
advice on the refund of VAT paid on purchase of its raw materials. Give your opinion on this.
5
b) Ms. Amita is a proprietor of a VAT registered firm. For the month of Shrawan 2067, she has
collected VAT of Rs. 200,000 which she has to deposit within 25th Bhadra. But on Bhadra
10,2067, she has delivered a baby boy due to that she could not deposit the VAT amount within
Bhadra 25. The tax officer wants to levy additional fee of 10% per annum on her. She has
submitted an application to Director General IRD on Bhadra 26, 2067 for waiver of additional
fee levied by the tax officer as the non payment of VAT was due to the circumstances beyond
her control. Can DG waive that additional fee levy imposed by the tax officer? Give your
opinion. 5
c) What are the records to be maintained by registered dealer dealing in used or second hand
materials? 5
d) Discuss the provision of Debit and Credit Note in VAT Rules. 5
Answer No 6
a) As per clause 20 of Schedule 1 of the VAT Act, sixty percent of the VAT paid on purchases of
mobile sets by importer or 60% of the VAT paid on raw materials for producing mobile sets,
shall be refunded by the Inland Revenue Department on producing evidence that the mobile sets
are sold. The procedure of the refund shall be as prescribed by Inland Revenue Department.
In the case given, Ginger cellular is a cellular mobile set producing company, so 60% of VAT
paid it on purchase of raw materials shall be refunded by IRD if it can produce the evidence that
the produced mobiles are sold.
b) As per rule 35 of the VAT Rules, In case a woman required to pay tax delivers a child; up to
thirty five days of the date of delivery is considered as circumstances beyond the control.
Further, as per Rule 36 of VAT Rules, For the remission of the additional charges pursuant to sub-
section (4) of Section 19 of the Act, an application shall be submitted to the Director General within
thirty days of the expiry of time-limit prescribed for payment of tax. In case an application is not
submitted within the time-limit referred, the waiver of additional charges shall not be granted.

The Institute of Chartered Accountants of Nepal


39 of 97
Suggested Answers of Income Tax and VAT
CAP II Examination – December2010

In this case, she has made an application within the limit mentioned in Rule 36. So applying the
provision of section 19(4) of the Act which states " If a taxpayer applies to the Director General
for the exemption of the additional charges levied due to non payment of VAT within time
stating the reason that the failure to make a timely payment was caused by extraordinary
circumstances beyond the taxpayer's control, the Director General may, if he finds the reason
reasonable, exempt such charges.
The Director General after the necessary verification of the matter can waive the additional fee
levied.

c) As per Rule 33 of the VAT Rules, a registered person who is dealing in used or secondhand
goods has to maintain the purchase register and sales register containing the following
information:
Related to Purchases:
 Date of Purchase
 Particulars giving full information of the goods
 Buying price excluding tax
 Rate of tax
 Amount of tax
 Total amount paid.
Related to Sales:
 Date of sale
 Selling price excluding tax
 Difference between the buying price and the selling price
 Rate of tax
 Amount of tax
 Total amount received
In case the buying price of every item of used goods exceeds Rs. 10,000, separate records of
buying or selling shall be maintained.
Failure to maintain the above records satisfactorily may attract the tax officer imposing VAT on
the total selling price of the goods sold by such taxpayer and the tax officer may issue a written
order requiring him to pay such tax along with the next tax return.

d) When a person issues an invoice for supply of goods or services, but after the invoice is issued it
is required to change the value as mentioned in the invoice due to any reason, the person has to
issue debit note or credit note for such changes in value of the goods.
Rule 20 of VAT Rules has not prescribed any format for debit and credit note but the debit or
credit note must include the following information:
 Serial Number of the debit or credit note
 Date of issue
 Name, address and registration number of supplier
 Recipient's name, address and registration number if he is a registered person.
 Number and date of the tax invoice connected with the transaction.
 particulars of the goods or services and reason of issuing the credit or debit note
 Amount debited or credited.
 Tax Amount debited or credited.

The Institute of Chartered Accountants of Nepal


40 of 97
Suggested Answers of Income Tax and VAT
CAP II Examination – December2010

The tax payer has to keep a copy of such debit or credit note received or issued. The taxpayer
has to maintain a register for recording monthly details of debit note or credit note received or
issued during the month.

The Institute of Chartered Accountants of Nepal


41 of 97
The Institute of Chartered Accountants of Nepal
Suggested Answers of Advanced Accounting

CAP II Examination- December 2012

1. A, B and C carried on business in partnership, sharing profits and losses in the ratio of 1:2:3. They decided
to form a private limited company, AB (P) Ltd. and C is not interested to take over the shares in AB (P) Ltd. The
authorized share capital of the company is Rs. 1,200,000 divided into 12,000 ordinary shares of Rs. 100 each.
The company was incorporated and took over goodwill as valued and certain assets of the partnership firm on 31-03-
2012. The Balance Sheet of the partnership firm on that date was as follows:
______________________________________________________________
Liabilities Rs. Assets Rs.
Capital Accounts: PPE:
A 100,000 Machinery 120,000
B 200,000 Land 174,000
C 300,000 Motorcycles 30,000
Current Accounts: Furniture & fittings 11,000
A 39,420 Current Assets:
B 60,580 Stock 235,000
A‟s Loan A/c 28,000 Debtors 43,000
Add: Interest accrued 2,000 30,000 Cash in hand 87,000
Current Liability: C‟s overdrawn 100,000
Creditors 70,000
Total 800,000 Total 800,000
C, who retired was presented by the other partners (A and B) with one motorcycle valued in the books of the firm Rs.
9,000. The remaining motorcycles were sold in the open market for Rs. 13,000. C also received certain furniture for
which he was charged Rs. 2,000. The debtors which were all considered good, were taken over by C for Rs. 40,000.
A and B were charged in their profit sharing ratio for the book value of motorcycle presented by them to C.
It was agreed that C who is not willing to take the shares in AB (P) Ltd. was discharged first by providing necessary
cash. A and B should bring cash, if necessary.
AB (P) Ltd. took over the remaining furniture and fittings at a price of Rs. 13,000, the machinery for Rs. 125,000, the
stock at an agreed value of Rs. 200,000 and the land at its book value. The value of the goodwill of the partnership
firm was agreed at Rs. 88,000. The creditors of the firm were settled by the firm for Rs. 70,000. A‟s loan account
together with interest accrued was transferred to his capital account.
The purchase consideration was discharged by the company by the issue of equal number of fully paid up equity
shares at par to A and B.
Prepare realization account, capital account of the partners and cash account. Also draw the balance sheet of AB (P)
Ltd. 20
Answer:
Dr. Realization Account Cr.
Particulars (Rs.) Particulars (Rs.)-
_____________________________________________________________________________
To Machinery 1,20,000 By Creditors 70,000
To Land 1,74,000 By AB (P) Ltd.-Purchasse
Consideration 6,00,000
(Refer Working Note)
To Motorcycles 30,000 By A‟s Capital A/c 3,000
To Furniture & fittings 11,000 By B‟s Capital A/c 6,000
To Stock 2 ,35,000 By C‟s Capital A/c 42,000
(2000 + 40,000)
To Debtors 43,000 By Cash (Sale of Motorcycle) 13,000
To Cash (payment of creditors) 70,000
To Profit transferred to:
A‟s Capital A/c 8,500
B‟s Capital A/c 17,000
C‟s Capital A/c 25,500

Total 7,34,000 Total 7,34,000

Dr. Partners‟ Capital Accounts Cr.


_____________________________________________________________________________________
A B C A B C
Suggested Answers – Advanced Accounting
CAP II Examination – December 2012
To Current A/c - - 100,000 By Balance b/d 100,000 200,000
3,00,000
To Realisation A/c 3,000 6,000 42,000 By Current A/c 39,420 60,580 -
(Assets taken over)

To Equity Shares in
AB (P) Ltd. 300,000 300,000 - By A‟s Loan 30,000 - -
To Cash - - 183.500 By Realisation A/c 8,500 17,000 25,500
(Realization Profit)
By Cash A/c 125,080 28,420 -

Total 303,000 306,000 3,25,500 Total 303,000 306,000 325,500

Dr. Cash Account Cr.


Particulars (Rs.) Particulars (Rs.)-
_________________________________________________________________________________
To Balance b/d 87,000 By Realisation A/c 70,000
To Realisation A/c 13,000 By C‟s Capital A/c 183,500
To A‟s Capital A/c 125,080
To B‟s Capital A/c 28,420
Total 253,500 Total 2,53,500

Balance Sheet of AB (P) Ltd.


_____________________________________________________________________________
Liabilities Rs. Assets Rs.
Authorized Share Capital PPE:
12,000 Equity Shares of Rs. 100
each 1,200,000 Goodwill 88,000
Issued, Subscribed & Paid up: Land 174,000
6,000 Equity Shares of Rs. 100 Machinery 125,000
each fully paid up (shares were Furniture &Fixture 13,000
issued for consideration otherwise Current Assets:
than for cash) Stock 200,000
600,000
Total 600,000 Total 600,000

Working Note:

Calculation of Purchase Consideration

Assets taken over by AB (P) Ltd. (Rs.)


Machinery 125,000
Furniture & fittings 13,000
Land 174,000
Stock 200,000
Goodwill 88,000
Purchase Consideration 600,000

Purchase consideration is discharged by the issue of equal number of equity shares of Rs. 100 each (3,000 shares) at
par to A & B.

2.
a) ABC Fertilizers presents the following balance sheets as on 31-03-2069 and 32-03-2068. You are
required to prepare cash flow statement: 10

(Rs in '000)
Balance Sheet 31-03-2069 32-03-2068
Equity Share Capital 8,500 7,000
General Reserve 3,800 4,000
Profit & Loss Account - 250
Share Premium Account 1,500 750
Share Holders Fund 13,800 12,000

The Institute of Chartered Accountants of Nepal


2 of 62
Suggested Answers – Advanced Accounting
CAP II Examination – December 2012
Secured Loans 4,800 5,000
Unsecured Loans 5,350 4,000
Loan Funds 10,150 9,000
Total Sources 23,950 21,000
PPE
Gross Block 22,400 21,000
Less: Accumulated Depreciation 3,450 3,200
Net Block 18,950 17,800
Capital Work In Progress 1,860 -
Investments 1,650 2,320
Current Asset, Loans & Advances
Inventories 2,510 2,600
Debtors 1,090 1,200
Cash & Bank Balance 120 280
Loans 1,700 200
Advance Tax - 500
(A) 5,420 4,780
Less Creditors 1,050 1,200
Outstanding Expense 30 -
Tax Provision - 500
Proposed Dividend 3,400 2,800
(B) 4,480 4,500
Net Current Asset 940 280
Miscellaneous Expenditure 550 600
Total Applications 23,950 21,000

Other Information:
i) PPE costing Rs. 400,000 (accumulated depreciation Rs. 300,000) were sold for Rs. 150,000.
ii) Actual tax liability for fiscal year 2067-068 was Rs. 500,000.
iii) Loan represents loan given to other companies.
iv) Interest on loan funds for fiscal year 2068-069 was at Rs. 1,421,000 and interest & dividend income were
Rs. 402,000.
v) Investments costing Rs. 2,000,000 were sold for 2,500,000.

b) Prischal sells goods on hire-purchase at cost plus 25%, prepare hire purchase trading account from
the information given below: 10
Particulars Rs.
Stock with Customers on Hire -Purchase (Opening) 162,000
Stock in hand at shop (Opening) 324,000
Instalments Overdue (Opening) 150,000
Purchases during the year 1,080,000
Goods Repossessed (Instalments not due Rs. 36,000) 9,000
Stock at shop excluding repossessed goods (Closing) 360,000
Cash received during the year 1,035,000
Instalments Overdue (Closing) 162,000
The Vendor Spent Rs. 2,000 on goods repossessed and then sold it for Rs. 15,000.

Answer:
a)
CASH FLOW STATEMENT
(Rs in ‘000)
Cash Flow from Operation Activities Rs.
Change in General Reserve -200
Change in Profit & Loss Account -250
Proposed Dividend 3400
Profit Before Tax 2950
Add: Depreciation 550
Add: Miscellaneous Exp 50
Less: Profit on Sale of PPE -50
Less: Profit on Sale of Investments -500 50
Fund Flow from Operations 3000
The Institute of Chartered Accountants of Nepal
3 of 62
Suggested Answers – Advanced Accounting
CAP II Examination – December 2012
Add: Interest Paid 1421
Less: Interest & Dividend Received -402
Add/(Less): Changes in Working Capital
Inventories 90
Debtors 110
Creditors -150
Outstanding Expense 30 80
Cash Flow from Operation Activities (Before Tax) 4099
Less: Advance Tax for 2068-2069 0
Cash Flow from Operation Activities (After Tax) A 4099
Cash Flow from Financing Activities
Issue of Shares 2250
Repayment of Secured Loans -200
Raising of Unsecured Loans 1350
Interest Payment -1421
Dividend Payment -2800
(B) -821
Cash Flow from Investment Activities
Purchase of PPE -1800
Sale of PPE 150
Capital WIP -1860
PPE(NET) -3510
Purchase of Investments -1330
Sale of Investments 2500
Investments (Net) 1170
Loans -1500
Interest & Dividend Income 402
C -3438
Increase/ Decrease in Cash & Bank Balance (A+B+C) -160
Add: Opening Cash & Bank Balance 280
Closing Cash & Bank Balance 120

b. Hire Purchase Trading Account

Particulars Rs. Particulars Rs.

To Opening Balance By Cash Received (Instalments) 1,035,000


Hire Purchase Debtors 150,000 By Stock Reserve (Opening ) W.N. 2 32,400
Hire Purchase Stock (Instalments 162,000 By Goods sold on Hire Purchase (Loading)
Overdue) (W.N.1) 261,000
By Cash Received (On sale of Re-possessed
goods) 15,000
To Goods sold on hire Purchase
(W.N.1) 1,305,000 By Closing Balance
Hire Purchase Stock (Inst. Overdue) 384,000
To Cash (Expense) 2,000 (W.N.4)

To Stock reserve (W.N.5) 76,800 Hire Purchase Debtors 162,000


To Profit & Loss Account 193,600
1,889,400 1,889,400

Working Notes
1.
Memorandum Stock at Shop Account
Particulars Rs Particulars Rs
To Balance B/d 324,000 By Goods Sold on Hire 1,044,000
Purchase Account

To Purchases(at Cost) 1,080,000 By Balance C/d 360,000

1,404,000 1,404,000

The Institute of Chartered Accountants of Nepal


4 of 62
Suggested Answers – Advanced Accounting
CAP II Examination – December 2012

Goods Sold on Hire Purchase Account at Invoice Price (1044000 x 125%) 1,305,000
Loading Rs 1305000-1044000 261,000

2.
Opening Reserve Account
162,000x25/125=32,400

3.
Hire Purchase Debtor Account
Particulars Rs Particulars Rs
To Balance B/d 150,000 By Cash Received 1,035,000
To Goods Sold on Hire Purchase By Hire Purchase Stock Account (Bal
1,305,000 .Fig) 258,000

By Balance c/d 162,000


1,455,000 1,455,000

4.
Hire Purchase Stock Account
Particulars Rs Particulars Rs
To Balance B/d 162,000 By Goods Repossessed 36,000
To Hire Purchase Debtors 258,000 By Balance C/d (Bal Fig) 384,000

420,000 420,000

5.
Closing Stock Reserve Account
384,000x25/125=76,800

3.
a) Gajani Limited operates a number of retail outlets to which goods are invoiced at wholesale price which is
cost plus 25%. These outlets sell the goods at the retail price which is wholesale price plus 20%.

Following is the information regarding one of the outlets for the year ended 21-3-2012:

Particulars Rs.
Stock at the outlet 1-4-2011 30,000
Goods invoiced to the outlet during the year 324,000
Gross profit made by the outlet 60,000
Goods lost by fire ?
Expenses of the outlet for the year 20,000
Stock at the outlet 31-3-2012 36,000

You are required to prepare the following accounts in the books of Gajani Limited for the year ended 31-3-2012: 10

i) Outlet Stock Account


ii) Outlet Profit & Loss Account
iii) Stock Reserve Account

b)
i) Mr. Gee started work on 1st January 2011 on a contract for Rs. 500,000. On 31st March 2011 when he
prepared his final accounts, the following information relating to the contract was extracted from his books of
account:
Rs.
Materials issued from stores and sent to site 160,000

The Institute of Chartered Accountants of Nepal


5 of 62
Suggested Answers – Advanced Accounting
CAP II Examination – December 2012
Wages Paid 101,200
Wages outstanding on 31st March 2011 37,520
New machine purchased and sent to site on 1st January 2011 148,000
Direct charges paid 7,500
Direct charges outstanding on 31st March 2011 600
Establishment charges apportioned to contract 6,400

On 31st March 2011 materials lying unused at site were valued at Rs. 21,620. Machines were depreciated @ 20% per
annum. Value of work certified by 31st March 2011 was Rs. 350,000 while the cost of work done but not yet certified
as on that date was Rs. 18,000. On the basis of architect‟s certificate Mr. Gee had received a total sum of Rs. 280,000
from the contractee till 31st March 2011.
You are requested to prepare contract account in Mr. Gee‟s ledger with appropriate working notes. 7

ii) X Ltd. takes over Y Ltd. on Shrawan 01, 2069 and discharges consideration for the business as
follows:
a. Issued 42,000 fully paid equity shares of Rs. 10 each at par to the equity share holders of Y Ltd.
b. Issued fully paid up 15% preference shares of Rs. 100 each to discharge the preference shareholders
(Rs. 170,000) of Y Ltd. at a premium of 10%.
c. It is agreed that the debentures of Y Ltd. (Rs. 50,000) will be converted into equal number and
amount of 13% debentures of X Ltd.
Calculate the purchase consideration of X Ltd. 3

Answer:
a)
Dr. Outlet Stock Account Cr.
Particulars (Rs.) Particulars (Rs.)-
_________________________________________________________________________________
To Balance b/d 30,000 By Sales (Working Note 1) 360,000
To Goods sent to outlet 324,000 By Goods Lost by fire 18,000
To Gross Profit c/d 60,000 By Balance c/d 36,000
Total 414,000 Total 414,000

Dr. Outlet Profit & Loss Account Cr.


Particulars (Rs.) Particulars (Rs.)-
_________________________________________________________________________________
To Expenses 20,000 By Gross Profit b/d 60,000
To Goods Lost by fire 18,000
(Working Note 2)
To Profit Transferred 22,000
Total 60,000 Total 60,000

Dr. Stock Reserve Account Cr.


Particulars (Rs.) Particulars (Rs.)-
_________________________________________________________________________________
To Profit & Loss A/c – Transfer 6,000 By Balance b/d 6,000
To Balance c/d 7,200 By Head Office- Profit & Loss A/c
(Working Note 3)
(Stock Reserve Required) 7,200
Total 13,200 Total 13,200

Working Notes:

(1) Wholesale Price 100+25 = 125


Retail Price 125+20% = 150
Gross Profit at the outlet
Wholesale Price – Retail Price (150-125) = 25
Retail Sales Value = 60,000 x 150/25 = 3,60,000

(2) Goods lost by fire


= Opening Stock + Goods Sent + Gross Profit – Sales – Closing Stock
The Institute of Chartered Accountants of Nepal
6 of 62
Suggested Answers – Advanced Accounting
CAP II Examination – December 2012
= 30,000 + 3,24,000 + 60,000 – 3,60,000 – 36,000
= Rs. 18,000

(3) Stock Reserve


On Opening Stock = 30,000 x 25/125 = Rs. 6,000

On Closing Stock = 36,000 x 25/125 = Rs. 7,200

b) i)
In Gee’s Ledger
Contract Account
Particulars Rs. Particulars Rs.
To Materials sent to site 160,000 By Unused materials at site as on 31st
To Wages paid 101,200 March 2011 21,620
Add: Outstanding wages 37,520 138,720 By Machines (written down value as on
To Machines (Cost) 148,000 31st March 2011) 140,600
To Direct Charges 7,500 By Work in progress:
Add: Outstanding DC 600 8,100 Certified 350,000
To Establishment Charges 6,400 Uncertified 18,000 368,000
To Profit & Loss A/c 36,800
To Balance c/d 32,200
530,220 530,220
To Unused Materials at site 21,620
To Machines 140,600
To Work in progress:
Certified 350,000
Uncertified 18,000
368,000
Less: Balance b/d 32,200 335,800

Working Notes:
i) Calculation of written down value of machines on 31 st March 2011:
Cost of Machine on 1st January 2011 148,000
Less: Depreciation on NPR 148,000 @20% p.a. for 3 months
= NPR 148,000 x 20/100 x 3/12 7,400
140,600
ii) Calculation of amount to be transferred from Contract Account to Profit & Loss Account:
Total surplus in Contract Account as on 31st March 2011
= NPR 21,620 + NPR 140,600 + NPR 368,000 – NPR 160,000 – NPR 138,720 – NPR 148,000 – NPR 8,100 – NPR
6,400 = NPR 69,000.

Profit to be credited to Profit & Loss Account


= Notional Profit 

= NPR 69,000 x 2/3 x 280,000/350,000 = NPR 36,800

b) ii)
Particulars Rs.
Equity Shares(42000*10) 420,000
Preference Share Capital 170,000
Add: Premium On redemption 17,000 187,000
Purchase Consideration 607,000

Note: There is no relevance of (c)

4.
a) A company issues 1000, 14% of Rs. 1,000 each at a premium of 20%. 60 % of the issue was underwritten
by Mr. Akash Chettri & Co. for a commission @ 1.50% of the issue price of debentures underwritten.
The Institute of Chartered Accountants of Nepal
7 of 62
Suggested Answers – Advanced Accounting
CAP II Examination – December 2012
Applications were received for 800 debentures which were accepted and payments were received in full. Give the
journal entries. 5

b) Following are the extracts from the record of Alpha Bank Ltd. in respect of the year ending Ashadh 31,
2069:

On Shrawan 01, 2068; bills for collection were Rs. 400,000. During 2068-069 bill received for collection amounted
to Rs. 6,050,000, bills collected were Rs. 4,500,000 and bills dishonored and returned were Rs. 300,000.
Prepare bills for collection (Assets) account and bills for collection (Liability) account. 5
c) GP Ltd. received a grant of Rs. 40 lakhs from the Government for the purpose of special machinery during
fiscal year 2064-065. The cost of machinery was Rs. 400 lakhs and had a useful life of 10 years. During fiscal
year 2068-069, the grant has become refundable due to non-fulfillment of certain conditions attached to it. Assuming
the entire grant was deducted from the cost of machinery in the year of acquisition; state with reason, the accounting
treatment to be followed in the fiscal year 2068-069. 5

Answer:
a)
The Journal Entries are as follows.

S.N. Particulars Debit Credit


1 Bank Account 960,000
To 14% Debentures 800,000
To Premium on Issue of Debentures 160,000
Allotment of 800 debentures @Rs 1200 as per Directors Resolution of ……………
2 Akash Chettri & Co 144,000
To 14% Debentures 120,000
To Premium on Issue of Debentures 24,000

Allotment of 120 debentures to the Underwriters in pursuance of their agreement dated………..


3 Underwriting Commission Account 10,800
To Akash Chettri & Co 10,800
Commission @1.5% on 600 Debentures at the issue price of Rs 1200
4 Bank 133,200
To Akash Chettri & Co 133,200
Amount Received form the underwriter from settlement

Note: Since Akash Chettri & Co underwrite 60% of the issue , the company must itself be treated as an
underwriter of the remaining 40% . In the absence of any information, the applications for 800 debentures must be
deemed to have marked 60% in favour of underwriters i.e. 480 debentures. The underwriters are, therefore, liable
to take up 120 debentures (600-480).

b)
Bills for Collection (Assets) A/c
Date Particulars Rs. Date Particulars Rs.
1/4/2068 To balance b/d 400,000 2068/069 By Bills for collection
2068/069 To Bills for collection (Liabilities) A/c 4,500,000
( Liabilities) A/c 6,050,000 By Bills for collection
(Liabilities) A/c 300,000
31/3/2069 By Balance c/d 1,650,000
6,450,000 6,450,000

Bills for Collection (Liabilities) A/c


Date Particulars Rs. Date Particulars Rs.
2068/069 To Bills for collection 1/4/2068 By Balance b/d 400,000

The Institute of Chartered Accountants of Nepal


8 of 62
Suggested Answers – Advanced Accounting
CAP II Examination – December 2012
( Assets) A/c 4,500,000 2068/069 By Bills for collection
To Bills for collection (Assets ) A/c 6,050,000
( Assets) A/c 300,000
31/3/2069 To Balance c/d 1,650,000
6,450,000 6,450,000

c) As per para 32 of NAS 10 Accounting for Government Grants, in case of refund of government grant related
to a specific fixed asset shall be recorded by increasing the carrying amount of the asset. The cumulative additional
depreciation that would have been recognized to date as an expense in the absence of the grant shall be recognized
immediately as an expense.

The computation of depreciation and expense can be given as:


Cost of Machinery Rs. 400 lacs
Less: Grant Received Rs. 40 lacs
Cost of Machinery Rs. 360 lacs
Useful life of Machinery 10 years
Depreciation per year as per straight line method (Assuming residual value to be zero Rs. 360 lacs/10 = Rs. 36 lacs
Total depreciation for 4 years (2064/65 to 2067/68) = 36×4 =Rs. 144 lacs

Cost of machinery without government grants Rs. 400 lacs


Useful life of Machinery 10 years
Depreciation that would have been recognized in absence of grant Rs. 400/10 = 40 Lacs
Cumulative total depreciation that would have been recognized in absence of grant=40×4 =160 lacs
Expenses to be recognized in FY 2068/069 = 160 lacs – 144 lacs=16 lacs

Depreciation to be charge for another 6 years = (400 – 160)/6


Amount of depreciation as an expense for 2068/069 = 40 + 16 = 56 lacs
Depreciation for remaining 5 years = 40 lacs per year

5.
a) While preparing its final accounts for the year ended 31 st March, 2012 Sky Limited created a provision for
bad and doubtful debts are 2% on trade debtors. A few weeks later the company found that payments from some of
the major debtors were not forthcoming. Consequently the company decided to increase the provision by 10% on the
debtors as on 31st March, 2012 as the accounts were still open awaiting approval of the Board of Directors. Is this to
be considered as an extraordinary item or prior period item? The company wants to treat the expenditure as deferred
revenue expenditure. Give your comments for the financial year ending on 31-03-2012 in the context of relevant
NAS. 5

b) A company obtained term loan during the year ended 31 st March, 2012 to an extent of Rs. 650 lakhs for
modernization and development of its factory. Building worth Rs. 120 lakhs were completed and plant and machinery
worth Rs. 350 lakhs were installed by 31st March, 2012. A sum of Rs. 70 lakhs has been advanced for assets the
installation of which is expected in the following year. Rs. 110 lakhs has been utilized for working capital
requirements. Interest paid on the loan of Rs. 650 lakhs during the fiscal year 2011-012 amounted to Rs. 58.50 lakhs.
How should the interest amount be treated in the account of the company? Give your comments for the financial year
ending on 31-03-2012 in the context of relevant NAS. 5
c) A pharma company spent Rs. 135 lakhs during the accounting year ended 31 st March, 2012 on a research
project to develop a drug to treat “AIDS”. Experts are of the view that it may take four years to establish whether the
drug will be effective or not and even if found effective it may take two to three more years to produce the medicine,
which can be marketed. The company wants to treat the expenditure as deferred revenue expenditure. Give your
comments for the fiscal year ending on 31-03-2012 in the context of relevant NAS. 5

Answer:
a)
As per NAS 2, revision of an estimate does not bring the resulting amount within the definition either of prior item or
of an extraordinary item.

In the given case, Sky Limited created a provision for bad and doubtful debts at 2 % on trade debtors while preparing
its final accounts for the year ended 31st March, 2012. Subsequently, the company decided to increase the provision
by 10%. As per NAS 2, this change in estimate is neither a prior period item nor an extraordinary item.

The Institute of Chartered Accountants of Nepal


9 of 62
Suggested Answers – Advanced Accounting
CAP II Examination – December 2012
However, as per NAS 2, a change in accounting estimate, which has a material effect in the current period, should be
disclosed and quantified. Any change in an accounting estimate, which is expected to have a material effect in later
periods should also be disclosed.

The expenditure of revenue nature may be treated as deferred revenue expenditure if its benefit is likely to be derived
over a number of years and alternatively, if there is a huge loss due to unforeseen circumstances. However, increase
in provision after the balance sheet date in the given case being an accounting estimate cannot be treated as deferred
revenue expenditure.

b)

According to NAS 8 (“Borrowing Cost”), interest on borrowed funds, which is directly related to the acquisition,
construction or production of qualifying asset should be capitalized. As factory building, Plant and Machinery are
qualifying asses as per NAS 8 interest paid on the loan being borrowing cost should be capitalized and included in the
gross book value of these assets. The interest pertaining to the money spent on the working capital should be charged
off to the Profit and Loss Account. In the given case, the interest amount of Rs. 58.50 lakhs shall be treated as
follows:

Proportion Rs. In lakhs


To be added to the cost of building 120  58.50 10.80
650
To be added to the cost of Plant and Machinery 350  58.50 31.5
650
To be added to the cost of work in progress 70  58.50 6.3
650

To be added to the cost of Profit and Loss A/c 110  58.50 9.9
650

Total 58.50

c)
As per NAS 27 on intangible assets, the research cost be expenses as and when incurred, in other words the cost of
research cannot be capitalized. The intangible asset arising from research should not be recorded as an asset and
therefore the research cost of internal project shall be treated as an expense in financial statement.

The development expenses, cost of internal project also to be expenses as incurred unless they meet asset recognition
criteria, before recognizing these costs as assets the following points should be demonstrated:

 Technical feasibility of the product


 Availability of product for use or sale
 Identification of cost incurred
 Probability of external market or
 The realistic expectation that there will be sufficient revenues to cover cost.

In the given case, the above conditions not having been fulfilled (Nothing is stated about future revenue or benefits),
the sum of Rs. 135 lakhs should be charged to the profit and loss account as an expense in the accounting year ended
31st March, 2012.

6. Write short notes on any four of the following: (4×2.5=10)


a) Mention any four areas in which different accounting policies may be adopted by different enterprises.
b) Contingency reserve
c) Non-performing assets
d) Conditions to be satisfied to capitalize the borrowing costs
e) Materiality and prudence

Answer:
a)
Major areas in which different accounting policies may be adopted by different enterprise includes:

 Methods of depreciation, depletion and amortization, e.g., WDV method, SLM method
The Institute of Chartered Accountants of Nepal
10 of 62
Suggested Answers – Advanced Accounting
CAP II Examination – December 2012
 Treatment of expenditure during construction, e.g. capitalization, written off, deferment
 Conversion or translation of foreign currency items, e.g. average rate , TT buying rate
 Valuation of inventories, e.g., FIFO, Weighted average method
 Treatment of goodwill, e.g., capitalization method, super profit method
 Valuation of Investments, e.g. lower of cost and fair value

b)
The Contingency reserves are sum set aside to cover anticipated future liabilities or reduction in assets value. This
reserve is required when the company believes the value of its assets likely decrease or it has incurred liabilities and it
is able to reasonable estimate the amounts loss. Contingency reserves are net up by deducting the appropriate sum
from income. Contingencies include:
 Potentially uncollectable money owed to company.
 Potential obligation under product warranties or related to products defects judgment for pending threaten
litigation.
 Likely loss due to fire and other hazards.
The Contingency reserve must be disclosed in financial statement when required and may be utilized for the
following purposes:
a. Expenses or loss of profits arising out of accidents, strikes or circumstances which the management could
not have prevented.
b. Expenses on replacement or removal of plant or works (other than normal maintenance or renewals).
c. Statutory obligation for payment of any compensation, if there is no special provision for such
compensation.

c)
While preparing financial statements of a bank, it is necessary to identify non-performing assets mostly based on
statutory/regulatory norms. An asset becomes non-performing when installment of matured principal and or income
from it is not received by the bank for a certain period. Income from non-performing assets can only be accounted for
as and when it is actually received. Nepal Rastra Bank has issued directives for the classification of loans and
advances. Necessary provision should be made for non-performing assets classifying as sub-standard, doubtful or loss
assets as the case may be as per the rate prescribed by Nepal Rastra Bank.

d)
The following conditions should be satisfied for capitalization of borrowing costs:
a) Those borrowing costs, which are directly attributable to the acquisition, construction or production of
qualifying asset, are eligible for capitalization. Directly attributable costs are those costs that would have been
avoided if the expenditure on the qualifying asset had not been made.
b) Qualifying assets will give future economic benefit to the enterprise and the cost can be measured reliably.

e)
Materiality:
Information is material if its omission or misstatement could influence the economic decisions of users taken on the
basis of the financial statements. It depends on the size of the item or error judged in the particular circumstances of
its omission or misstatement. Often separate line item or sub-item is decided bases on materiality. National level law
may specify materiality limit for separate disclosure of an item.

Prudence:
Prudence is the inclusion of a degree of caution in the exercise of the judgments needed in the making the estimates
required under conditions of uncertainty, such that assets or income are not overstated and liabilities or expenses are
not understated. The exercise of prudence does not allow, for example, the creation of hidden reserves or excessive
provisions.

The Institute of Chartered Accountants of Nepal


11 of 62
The Institute of Chartered Accountants of Nepal
Suggested Answers of Audit and Assurance

CAP II Examination- December 2012

1. As an auditor, give your opinions with reasons on the following cases: (45=20)

a) Your client has computed the deferred tax assets or liabilities on: i. Fixed assets as per financial base Rs.
500,000, as per tax base Rs. 600,000; ii. Other assets as per financial base 300,000 as per tax base Rs. 200,000 and other
liabilities as per financial base Rs. 100,000 as per tax base Rs. 50,000. Consider applicable tax rate 5% and opening
balance of deferred tax asset is Rs.5,000As a Auditor how would be check the accounting treatment made on deferred
tax by your client (show your computation as well)?

b) Puran (A practicing Chartered Accountant) has been appointed as statutory auditor of ABC Development Bank
Limited for the fiscal year 2068/69. Puran has been holding certain paid up share capital of the bank since the year 2060.
Is the appointment of Puran is valid?

c) New auditor of XYZ Limited signed the financial statements for the year 2068/69 without considering the
previous year auditor's report and financial figures.

d) The Balance sheet of ABC Ltd. includes inventory amounting to Rs. 3 crores out of total assets of 20 crores. The
inventories were valued at cost. The market price of the inventories was Rs. 2.5 crores. The company has disclosed this
fact in the notes to accounts.

Answer:
a) Nepal Accounting Standard 9 (NAS) defined on deferred tax assets and liabilities. The NAS suggested for
computing deferred tax assets or liabilities on temporary differences on tax base and financial base and booking
corresponding income or expenses. The computation of deferred tax has been as follows:

Particulars Financial Tax Base (B) Deferred (Assets) Applicable Deferred Tax (Assets)
Base (A) /Liabilities C= (A-B) /Liabilities @ 5% D= C*5%
1.Fixed Assets 500,000 600,000 (100,000) Yes (5,000)
2.Other Assets 300,000 200,000 100,000 Yes 5,000
3. Liabilities 100,000 50,000 (50,000) Yes (2,500)
Net Asset (2,500)
Previous year Asset Balance (5,000)
Decrease in Asset (Charged to 900,000 850,000 (50,000) (2,500)
PL)

Accordingly closing balance of deferred tax should be Rs. 2,500 and deferred tax asset should be credited by Rs. 2,500
by charging corresponding amount to profit & loss account as deferred tax expenses.

b) Section 112 of the Companies Act, 2063 stated the disqualifications of auditor: As per Act None of the certain persons
or the firms or companies in which such persons are partners shall be qualified for appointment as auditor and shall,
despite appointment as auditor, continue to hold office.

Provision regarding appointing shareholder as auditor as per Companies Act stated that: persons or the firms or
companies who is substantial shareholder of the company or a shareholder holding one percent or more of the paid up
capital of the company or his close relative cannot be appointed as auditor.

In the instance case the % of holding paid up capital is not mentioned, if the Puran himself or firm or company he is
partner or shareholder or his close relative do not hold one percent or more than one percent of the paid up capital of
ABC Development Bank Limited only than Puran is qualified to be appointed as auditor.

c) NSA 24 Para 9 has defined the treatment of the previous figures and auditors' report. As per the Para 9, the auditor
should obtain sufficient appropriate audit evidence that the corresponding figures meet the requirement of the relevant
financial reporting framework. The extent of audit procedures performed on the corresponding figures is significantly
less than for the audit of the current period figures and is ordinarily limited to ensuring that the corresponding figures
have been correctly reported and are appropriately classified. This involves the auditor assessing whether:
i) accounting policies used for the corresponding figures are consistent with those of the current period or whether
appropriate adjustments and/or disclosures have been made; and
ii) corresponding figures agree with the amounts and other disclosures presented in the prior period or whether
appropriate adjustments and/or disclosures have been made.
Suggested Answers – Audit and Assurance
CAP II Examination – December 2012
In Para 10 when the financial statements of the prior period have been audited by another auditor, the incoming auditor
assesses whether the corresponding figures meet the conditions specified in paragraph 9 above and also follows the
guidance in NSA 21, “Initial Engagements-Opening Balances.”
In Para 11 when the financial statements of the prior period were not audited, the incoming auditor nonetheless assesses
whether the corresponding figures meet the conditions specified in paragraph 9 above and also follows the guidance in
NSA 21.
In Para 12 if the auditor becomes aware of a possible material misstatement in the corresponding figures when
performing the current period audit, the auditor performs such additional procedures as are appropriate in the
circumstances.
When the comparatives are presented as corresponding figures, the auditor should issue an auditor‟s report in which the
comparatives are not specifically identified because the audit opinion is on the current period financial statements as a
whole, including the corresponding figures.
When the auditor‟s report on the prior period, as previously issued, included a qualified opinion, disclaimer of opinion,
or adverse opinion and the matter which gave rise to the modification is: (a) unresolved, and results in a modification of
the auditor‟s report regarding the current period figures, the auditor‟s report should also be modified regarding the
corresponding figures; or (b) unresolved, but does not result in a modification of the auditor‟s report regarding the
current period figures, the auditor‟s report should be modified regarding the corresponding figures.

d) As per Nepal Accounting Standard 4, Inventories should be measured at lower of cost or net realizable value. In the
present case the cost price of the inventories is 3 crores and net realizable value is 2.5 crores and hence the inventories
should be presented at 2.5 crores in the balance sheet. However, the company has presented the inventories at 3 crores
and disclosed in the notes to accounts that the inventories have been presented at cost although its net realizable value is
lower than the cost. Mere disclosure of this fact in the notes however does not result into compliance with the accounting
standard. Hence as an auditor I will qualify my audit report because inventory in the present case represents material
item of the assets of the company and it has been materially misstated in the balance sheet.

2. Answer the following:


a) You have taken up the assignment for Due Diligence Review of the financial statement as of 31 Asadh 2069 of Das
Dhunga Bank Ltd for facilitating the merger process with Lucky Finance Co. Ltd. Write down any seven points you have
to pay due consideration on carrying out the review on assets side (covering each asset category) of the financial
statement. 7

b) As a quality control manager of the audit engagement team of D Limited, how can you ensure that your team has
performed the audit works as per the standard maintained by your audit firm along with the other mandatory compliance
of various national and international standards in auditing and accounting? Points out the review procedures to ensure the
quality of the audit work? 8

Answer:
a) Upon taking up the assignment for due diligence review of the Das Dhunga Bank Ltd; i have to pay due
following due consideration on review assets side of the financial statement:
i. Loan & Advances: The due consideration to be paid on loan and advances is whether the adequate loan loss
provision has been made in line with Nepal Rastra Banks Directive applicable to the bank.
ii. Investment: The due consideration to be paid on investment is that whether the investment has been made in
line with Nepal Rastra Banks Directive in eligible portfolio. If investment is doubtful of recovery to see whether
necessary provision has been set aside or otherwise. Further in applicable cases have to see whether required
reserves/funds against the investment been set aside.
iii. Fixed Assets: The due consideration to be paid on fixed assets is that whether all the fixed assets appearing in
financial statement legally owned by the entity, physically available and are in working condition. The major fixed assets
should be revalued with the help of expert for the purposes of DDA.
iv. Advance Tax: The due consideration to be paid on Advance Tax is to see whether the amount shown as
advance tax are really an existed asset representing the cash deposits made and advance tax made by parties and are not
time barred for claiming for assessment of tax purposes.
v. Non-Banking Assets (NBA): To see whether the NBA are on the possession and ownership of the bank. See
whether disposal (sales) process of NBA has not been suffered any hindrances like NBA is insufficient for covering
dues recoverable, NBA is subject to lien of others , NBA is under legal cases. Further should be ensured that NBA is
adequately provisioned as per NRB Directives.
vi. Advances: To see the recoverability of advances. If advances are doubtful of recoverability, ensure that
whether adequate provision against possible loss has been made.
vii. Deposits : To see the existence of various deposits . If deposits are doubtful of refund and are not backed
up with sufficient documents, ensure that whether adequate provision against possible loss has been made.

b) The quality control manager inform the responsibilities of the engagement team and also give some of the things to
be ensured before starting the works as follows;
 understanding the nature of the entity's business
 Possible risk related issues
 Problems that may arise; and
The Institute of Chartered Accountants of Nepal
2 of 62
Suggested Answers – Audit and Assurance
CAP II Examination – December 2012

 The detailed approach to the performance of the engagement.


The engagement team's responsibilities includes maintaining an objective state of mind and an appropriate level of
professional skepticism, and performing the work delegated to them in accordance with the ethical principle of due care.

The quality control manager before review the files, will supervise the engagement team and do the following works:
 Tracking the progress of the audit engagement;
 Considering the capabilities and competence of individual members of the engagement team, whether they have
sufficient time to carry out their work, whether they understand their instructions, and whether the work is being carried
out in accordance with the planned approach to the audit engagement.
 Addressing significant issues arising during the audit engagement, considering their significance and modifying the
planned approach appropriately.
 Identifying matters for consultation or consideration by more experienced engagement team member during the
audit engagement.

The quality control manager will review the work performed by their team members and consider the following while
reviewing the works and working papers:
 The work has been performed in accordance with professional standards and regulatory and legal requirements;
 Significant matters have been raised for further consideration;
 Appropriate consultations have taken place and the resulting conclusions have been documented and implemented.
 There is a need to revise the nature, timing and extent of work performed;
 The work performed supports the conclusions reached and is appropriately documents;
 The evidence obtained is sufficient and appropriate to support the auditor's report;
 The objectives of the engagement procedures have been achieved.

The engagement partner conducts timely reviews at appropriate stages during the engagement. This allows significant
matters to be resolved on a timely basis to the engagement partner‟s satisfaction before the auditor‟s report is issued. The
reviews cover critical areas of judgment, especially those relating to difficult or contentious matters identified during the
course of the engagement, significant risks, and other areas the engagement partner considers important. The engagement
partner need not review all audit documentation. However, the partner documents the extent and timing of the reviews.
Issues arising from the reviews are resolved to the satisfaction of the engagement partner.

3. Giv
e your comments on the following: (35=15)

a) Sahara Garment Private Limited is one of the leading garments industries in Nepal. The company has exported
50,000 readymade shirts to Turkey for equivalent NPR 10 millions and booked NPR 10 millions as income in their
account, however, the government of Turkey for the time being has suspended the equivalent NPR 5 millions amount to
be remitted to Sahara Garment P Ltd on the ground that the quota system as determined by the government of Turkey is
below than the goods as supplied by Sahara Garment from Nepal

b) Mr. Lakhan, Statutory Auditor of Radha Krishna Pvt. Ltd wants to verify cash on hand as on 31st Asadh, 2069. The
Management informs Mr. Lakhan that it is not possible to cooperate, as cashier has been out of station. Advise Mr.
Lakhan on how to deal with the situation.

c) A proprietary audit firm of Chartered Accountant has accepted the engagement to audit the accounts of a private
school with annual turnover of Rs 10 lakhs at the audit fee of Rs 8,500. The firm anticipates that the audit will consume
estimated time of 2 man days of the Chartered Accountant.

Answer:
a) NAS 07 Nepal accounting standards on revenue Para 18 has clearly defined the case of remit suspense. As per this
Para revenue is recognized only when it is probable that the economic benefits associated with the transaction will flow
to the entity. In some cases, this may not be probable until the consideration is received or until an uncertainty is
removed. For example, it may be uncertain that a foreign governmental authority will grant permission to remit the
consideration from a sale in a foreign country. When the permission is granted, the uncertainty is removed and revenue is
recognized. However, when an uncertainty arises about collectability of an amount already included in revenue, the
uncollectable amount or the amount in respect of which recovery has ceased to be probable is recognized as an expense,
rather than as an adjustment of the amount of revenue originally recognized.
Sahara Garment P Ltd has to adjust the balance suspended amount from the revenue which was recognized previously.
Otherwise the auditor should make qualification on the revenue reorganization.

b) The scope of audit may be limited for varied reasons, (i) the entity may impose restriction on scope of audit, (ii) the
limitation may be imposed by circumstances. When the audit is carried out under and as per statute, the auditor should
not accept the assignment when his duties are curtailed by agreement, unless required by any Law.

The Institute of Chartered Accountants of Nepal


3 of 62
Suggested Answers – Audit and Assurance
CAP II Examination – December 2012
When audit is carried out in accordance with the entity‟s terms voluntarily, the auditor may indicate his scope in his
audit report.
Sometimes, the circumstances may impose restrictions on audit scope. For example, if the auditor is appointed after the
year end, he may not be able to participate in inventory checking. Or sometimes, the records required may not be
available so that the auditor may not be able to check details in the manner he liked. Such limitations in scope may
warrant an auditor to express disclaimer of opinion or qualified opinion in his audit report depending upon the
circumstances.
The non co-operation of Radha Krishna Pvt. Limited will amount to limitation on scope of auditors.

c) As per directive issued by the council, a CA member holding COP shall charge the audit fee to his audit client and
the fee shall not be less than Rs 10,000. If the audit client is a financial institution regulated by Nepal Rastra Bank or
insurance company regulated by the Insurance Board, the audit fee shall not be less than Rs 40,000. However, above
minimum fee does not apply where the audit client is a Government Primary School/Community School with annual
turnover of less than Rs 5 lakhs and where the audit client is an organization with annual turnover of less than Rs 2
lakhs. In the present case the CA member has charged the audit fee of Rs 8,500 to a private school with annual turnover
of Rs 10 lakhs which is not covered by the exception for charging lower fee and hence the auditor should have charged at
least Rs 10,000 as audit fee. Hence the member seems to have not followed the directive of the council and accordingly
may be subject to disciplinary action.

4. Answer the following: (35=15)

a) How will you verify the retirement gratuity to employees?


b) An audit is not a guarantee that the financial statements are free from material misstatement, because absolute
assurance is not attainable. What are the factors which hinders the auditor to provide absolute assurance?
c) As statutory Auditor of XYZ Pvt. Ltd. you requested your client for sending letter for balance confirmations from
certain debtors, the client argue that since the said balances with debtors are under dispute and the matter is pending in
the court it is not necessary to ask balance confirmation.

Answer:
a) Verification of retirement gratuity
 Examine the basis on which payable to employees is worked out. The liability for gratuity may either be worked on
actuarial rules or agreement or the presumption that all employees retire on the balance sheet date.
 Verify computation of liability of gratuity on the aggregate basis.
 Check the amount of gratuity paid to employees who retired during the year with reference to number of years of
services rendered by them.
 See that the annual premium has been charged to profit and loss account in cash the concern has taken a policy from
the insurance companies.
 Ensure that the basis of computing gratuity is valid.
 Ensure that the accounting treatment is in accordance with NSA 24 Accounting and Reporting by Retirement
Benefits Plans'.

b) Factors hindering the auditor to provide absolute assurance: Auditor provides reasonable assurance on the
financial statements and is not in the position to provide absolute assurance due to the following factors:
i) Use of testing/sampling rather than 100% checking,
ii) The inherent limitations of internal control (for example, the possibility of management override or collusion),
iii) The fact that most audit evidence is persuasive rather than conclusive and
iv) The work undertaken by the auditor to form an audit opinion is permeated by judgment, in particular
regarding: (a) The gathering of audit evidence, for example, in deciding the nature, timing and extent of audit
procedures; and (b) The drawing of conclusions based on the audit evidence gathered, for example, assessing the
reasonableness of the estimates made by management in preparing the financial statements.

c) NSA 505, “External Confirmations”, establishes standards on the debtor‟s use of external confirmation as a means
of obtaining audit evidence. It requires that the auditor should employ external confirmation procedures in consultation
with the management. The auditor may come across certain situations in which the management may request him not to
seek external confirmation from certain parties because of dispute with the debtors, etc. The management, for example,
might make such a request on the grounds that due to a dispute with the particular debtor, the request for confirmation
might aggravate the sensitive negotiations between the entity and the debtor. In such cases, when an auditor agrees to
management‟s request not to seek external confirmation regarding a particular debtor, the auditor should consider
validity of grounds for such a request and assess management‟s integrity and obtain evidence to support the same. The
auditor should also ask the management to submit its request in a written form, dealing therein the reasons for such a
request. The auditor agrees to management‟s request not to seek external confirmation regarding a particular matter, the
auditor should document the reasons for accepting to the management‟s request and should apply alternative procedures
to obtain sufficient appropriate evidence regarding that matter. While considering the validity of request, in case the
auditor reaches at a conclusion that the same was not valid, he may appropriately modify the report.

5. Comment on the following situations/statements. (35=15)


The Institute of Chartered Accountants of Nepal
4 of 62
Suggested Answers – Audit and Assurance
CAP II Examination – December 2012

a) Though the audit for the year 2068/69 was completed, but not signed the financial statements of X Limited by the
board of directors. In the mean time, the board has decided to stop the operation of Z Limited, one of the major
subsidiary companies of X Limited with effect from Marg 1, 2069.

b) What is the responsibility of the professional accountant " When he knows a material error or omission in a tax
return of a prior year (with which the professional accountant may or may not have been associated), or of the failure to
file a required tax return?

c) Ramesh Sharma was Chief Finance Officer of A Ltd for the period 1 Srawan 2064 to 15 Ashad, 2069. He joined
XYZ Chartered Accountant firm from 1 Srawan 2069 as Senior Audit Manager. A Ltd. has approached the firm on 15
Shrawan 2069 for the audit of accounts for the year 2068/69 with Ramesh Sharma as Engagement Manager of the audit
team. Please provide your opinion on what the audit firm should do?

Answer:
a) NAS 05 section 21 has defined the non – adjusting events after the balance sheet date, but it should be disclosed in
the balance sheet. The sections this events as follows:
If non-adjusting events after the balance sheet date are material, nondisclosure could influence the economic decisions of
users taken on the basis of the financial statements. Accordingly, an entity shall disclose the for each material category of
non-adjusting event after the balance sheet date:
(a) the nature of the event; and
(b) an estimate of its financial effect, or a statement that such an estimate cannot be made.

Section 22 has defined the examples of non adjusting events after the balance sheet date are as follows:
(a) a major business combination after the balance sheet date (NAS 21 Business Combinations requires specific
disclosures in such cases) or disposing of a major subsidiary;
(b) announcing a plan to discontinue an operation;
(c) major purchases of assets, classification of assets as held for sale, other disposal of assets, or expropriation of major
assets by government;
(d) the destruction of a major production plant by a fire after the balance sheet date;
(e) announcing, or commencing the implementation of, a major restructuring;
(f) major ordinary share transactions and potential ordinary share transactions after the balance sheet date (NAS 26
Earnings Per Share requires an entity to disclose a description of such transactions, other than when such transactions
involve capitalization or bonus issues, share splits or reverse share splits all of which are required to be adjusted under
NAS 26);
(g) abnormally large changes after the balance sheet date in asset prices or foreign exchange rates;
(h) changes in tax rates or tax laws enacted or announced after the balance sheet date that have a significant effect on
current and deferred tax assets and liabilities (see NAS 09 Income Taxes);
(i) entering into significant commitments or contingent liabilities, for example, by issuing significant guarantees; and
(j) commencing major litigation arising solely out of events that occurred after the balance sheet date.
In the financial statements of X Limited, a proper disclosure is required regarding the closure of the subsidiary company
Z Limited and its effects though the financial figures will not be changed in the balance sheet.

b) As per Section 5.8 of the Code of Ethics of the institute of the chartered accountants of Nepal in such case
professional accountant is responsible for:

i. Promptly advise the client or employer of the error or omission and recommend that disclosure be made to the
revenue authorities. The professional accountant is not obligated to inform the revenue authorities.

ii. If the client or the employer does not correct the error the professional accountant:

(a) should inform the client or the employer that it is not possible to act for them in connection with the return or other
related information submitted to the authorities; and

(b) should consider whether continued association with the client or employer in any capacity is consistent with
professional responsibilities.

iii. If the professional accountant concludes that a professional relationship with the client or employer can be
continued, all reasonable steps should be taken to ensure that the error is not repeated in subsequent tax returns.

c) Ramesh Sharma was the Chief Finance Officer of A Ltd. for almost entire year 2068/69. He has left A Ltd just a
month before the audit firm was approached by A Ltd for audit of 2068/69 and proposes his name to be engaged as
engagement manager in the audit team. This situation may result into self-interest, self review and familiarity threat to
the firm. Hence the firm should request A Ltd. that the firm cannot assign Ramesh as an audit team member. If A Ltd
does not accept the audit team without Ramesh as senior member in the audit team from the firm, the audit firm should
not accept this engagement.
The Institute of Chartered Accountants of Nepal
5 of 62
Suggested Answers – Audit and Assurance
CAP II Examination – December 2012

6. Write short notes on the following: (42.5=10)


a) Legal liabilities of auditor
b) Internal Audit
c) Engagement Letter
d) "During payments vouching, the auditor does not merely check proof that money has been paid away".

Answer:
a) Legal Liabilities of auditor:
Auditors must always perform their work complying the basic principles of audit, which are, integrity, objectivity,
independence, confidentiality, professional competency, due care and technical standards.

If the same is not complied with, then they can be held liable on account of:
i) Non compliance with Nepal Standards on Auditing; and
ii) Failure to protect the interest of stakeholders relying upon the audited financial statements;

Accordingly, the auditors‟ liability falls under three categories:


i) To their clients (company itself);
ii) To third parties in case of negligence; and
iii) Civil and criminal liabilities.

b) Internal Audit: Internal audit is an independent, objective assurance and consulting activity designed to add value
and improve an organization's operations. It helps an organization accomplish its objectives by bringing a systematic,
disciplined approach to evaluate and improve the effectiveness of risk management, control, and governance processes.

c) Engagement Letter: It is letter issued by the auditor to the auditee which includes written terms of engagement.
This letter basically sets out the responsibilities of the auditor and the auditee and includes other information such as
audit fee and its payment terms, out of pocket expenses etc. This letter helps to resolve the confusion which may arise
during or subsequent to audit.

d) Vouching is a substantive audit procedure which aims at verifying the genuineness and validity of a transaction
contained in the accounting records. It involves examination of documentary evidence to support the genuineness of
transaction.
Thus the object of vouching the payments of a business is not merely to ascertain that money has been paid away; but the
auditor aims to obtain reasonable assurance in respect of following assertions in regard to transactions recorded in the
books of account that :
i) a transaction is recorded in the proper account and revenue or expense is properly allocated to the accounting period;
ii) a transaction pertains to entity and took place during the relevant period;
iii) all transactions which have actually occurred have been recorded;
iv) all transactions were properly authorized; and
v) transactions have been classified and disclosed in accordance with recognizedaccounting policies and practices.
Thus, it is through vouching that the auditor comes to know the genuineness of transactions recorded in the client‟s
books of account wherefrom the financial statements are drawn up.
Thus, the auditor‟s basic duty is to examine the accounts, not merely to see its arithmetical accuracy but also to see its
substantial accuracy and then to make a report thereon.

7. Distinguish between (25=10)


a) Statutory & System Audit
b) Peer review, hot review and cold review

Answer:
a) The major differences between statutory and system auditing can be described as follows:
(i) Purpose: The statutory auditing is basically concerned with the opinion that whether the historical information
recorded is correct or not, whereas the system auditing emphasizes on effectiveness and efficiency of operations for
future performance.
(ii) Area: Statutory audits are restricted to the matters directly affecting the appropriateness of the presented financial
statements whereas the system audit covers all the activities that are related to efficiency and effectiveness of operations
directed towards accomplishment of objectives of organization.
(iii) Reporting: The statutory audit report is sent to all stock holders, bankers and other persons having interest in the
organization. However, the system audit report is primarily for the management.
(iv) End task: The statutory audit has reporting the findings to the persons getting the repost as its end objectives,
however, the system audit is not limited to the reporting only, but includes suggestions for improvements also.
(v) Auditor: In Statutory Audit generally opinion on the financial statement has to be expressed by COP holder of
ICAN, though system audit could be done in-house or by outsourcing where it is not necessary for carrying out by COP
holder.

b) Peer Review
The Institute of Chartered Accountants of Nepal
6 of 62
Suggested Answers – Audit and Assurance
CAP II Examination – December 2012
This is a critical independent review of one public accounting firm practices by another public accounting firm. It is a
review of the firm‟s accounting and auditing practices. It is intended that the review be done by practitioners upon fellow
practitioners.
Such an external review offers a more objective evaluation of the quality of performance than could be done by self
review.
Peer review study the adequacy of the firms established quality control policies and tests to determine the extent of the
firms compliance to these policies.
Suggestions for improvement to the system are outlined in a letter of comments issued by peer reviewers to the reviewed
firm. If a firm fails to take appropriate corrective action, various actions may be imposed e.g. suspension from
membership.
In carrying out the review it is limited to:-
 • Professional aspects of the practice.
 • The overall total quality control policies.
 • Professional aspect of the firms accounting and auditing practices like maintenance of working papers and work
products such as report to the financial statements.

Hot Review
Is an independent review of an audit by a suitably qualified firm of auditors before the issue of the audit report? The
review includes evaluation of the engagement, working papers, reports and the firm compliance with established quality
control policies and professional standards.
Hot review helps the reviewed firm to make necessary changes on their report or have confidence that what they are
reporting was well conducted and no liability can result from their report.
Hot reviewers assess how the firm accepted their engagement i.e. whether they followed professional ethics and whether
they were qualified to act as auditors for the entity. Secondly they review the firms audit plans and audit programs to
consider how duties were assigned delegated and directed. They address the critical areas by evaluating the working
papers and how the audit evidence was gathered i.e. audit tests and procedures were well performed according to the
established standards.
Finally the reviewers will evaluate the report and make conclusion in writing as to whether all the matters raised have
been satisfactorily satisfied/resolved by the firm. They will also recommend whether further tests are necessary to come
up with comprehensive reports.

Cold Review
It is the involvement of an independent accounting firm or a partner to provide assurance that all the firms‟ in house
quality control policies have been complied with as well as provide a second opinion that the audit was performed in
accordance with generally accepted audit standards.
Cold review is an objective examination of an audit assignment after its completion referred to as post audit review.
Cold review assess the policies and procedures the firm used to conduct its audit. This involves audit planning and
ascertaining that the audit team followed the audit plans together with audit programs. It also ascertains that work was
well documented through working papers and judgment or conclusion made.
Cold review report to staff adherence to ISAS and review work done by each staff and see that it is according to the
policies of the firm.

The Institute of Chartered Accountants of Nepal


7 of 62
The Institute of Chartered Accountants of Nepal
Suggested Answers of Corporate and Other Laws

CAP II Examination- December 2012

Part "A"
1. Answer the following questions:
a) M/s. Simon Ltd. was wound up by the court. The official liquidator invited claims from its creditors which stood as
under:
Income Tax dues 11 lakhs
Sales Tax dues 5 lakhs
Dues of workers 25 lakhs
Unsecured loans Payable to directors 25 lakhs
Trade creditors who supplied raw material 15 lakhs
Secured creditors being the banker of the Company 75 lakhs
Total Rs. 156 lakhs

Official liquidator could realize Rs. 80 lakhs by sale of assets and realization made from the Company debtors which is not
sufficient to pay to all the creditors. Decide the order of priority for payment to creditors explaining the relevant provisions
of the Companies Act, 1956. 8

b) A, signs as maker a promissory note and gives it to B who delivers it to C as a gift. C, with a view of cancelling the
note makes two pieces and throws it to a container. D, who was in service of cleaning roads, finds the pieces of note and
fixes them as there was no sign of tear. He passes the note to M who has received in good faith for consideration. Can M
recover the amount stated therein from A? Decide with reasons. 6

c) Mr. Hari Singh was appointed as a managing director of the plaintiff company - Reliance Motors India Co. on the
condition that he shall not at any time while he shall hold the office of a managing director or afterwards, solicit or entice
away the customers of the company. Shortly afterwards, his employment was terminated under an agreement and he
opened a business in the name of a separate company Golden Gate Automobiles Co. and began solicit the customers of the
Reliance Motors India Co. When the case filed by the Reliance Motors India Co., he argued that the business is of the
Golden Gate Automobiles Co. Advise Reliance Motors India Co. whether their claim is maintainable against Hari Singh?
Decide with relevant provisions and principles enshrined under the Companies Act, 1956. 8

d) Explain as to why the combination of 'not negotiable' with 'Account Payee' crossing is considered the safest form of
crossing a cheque. 3

Answer:
a)
Where the assets of the company available for payment of general creditors are insufficient to meet their claims, certain
unsecured debts are paid before the debenture holders. Sec 530 says there shall be paid in priority to all other debts.
1. Rates and taxes having become due to central or state govt.
2. All wages or salary at an employee
3. Compensation payable under the workmen's compensation in respect of death or disablement of any officer or employee
4. All sum due to any employee from a provident, pension, gratuity
5. The expenses of any investigation held under S. 235 or S.237
6. All accrued holyday remuneration payable to any employee
7. All amount due in respect of contributions payable during 12 months
Under Section 529, in the winding up of an insolvent company, the rules shall prevail and be observed with regard to (a)
debts provable; (b) the valuation of annuities and future and continent liabilities; and (c) the respective rights of secured
and unsecured creditors; as are in force for the time being under the law of insolvency with respect to the estates of persons
adjudged insolvent provided that the security of every secured creditor shall be deemed to be subject to a pari passu charge
in favour of the workmen to the extent of the workmen's portion therein and where a secured creditor instead of
relinquishing his security his security and proving his debt, opts to realize his security.
(a) the liquidator shall be entitled to represent the workmen and enforce such charge ;
(b) any amount realised by the liquidator by way of enforcement of such charge shall be applied
rateably for the discharge of workmen's dues ; and (c) so much of the debt due to such secured creditor
as could not be realised by him by virtue of the foregoing provisions of this proviso or the amount of
the workmen's portion in his security, whichever is less, shall rank pari passu with the workmen's dues
for the purposes of section 529A.
Suggested Answers – Corporate and Other Laws
CAP II Examination – December 2012

All persons who in any such case would be entitled to prove for and receive dividends out of the assets
of the company, may come in under the winding up, and make such claims against the company as
they respectively are entitled to make by virtue of this section:
Provided that if a secured creditor instead of relinquishing his security and proving for his debt
proceeds to realise his security, he shall be liable to pay his portion of the expenses incurred by the
liquidator (including a provisional liquidator, if any) for the preservation of the security before its
realisation by the secured creditor.
Section 529A has given the power of the overriding preferential payments. Under Section 529A of the Companies Act,
1956, (i) workmen's dues, and (ii) debts due to secured creditor shall be paid in priority of all debts and shall be paid in full,
unless the assets are insufficient to meet them, in which case they shall abate in equal proportions.

Income tax dues and sale tax dues are preferential creditors under section 530 of the Act, and subject
to the provisions of section 529A, the same may be paid in priority to the claims of unsecured
creditors.

In the present case, the available funds are only to the extent of Rs. 8m which will be distributed
amongst the secured creditors and workmen in proportion to their dues, as follows:
Workmen (1/4th of Rs. 8m) Rs. 2m
Secured creditors (3/4th of Rs. 8m) Rs. 6m

As such, the dues of preferential creditors (namely, Income tax and sales tax dues) and unsecured
creditors (unsecured loan and trade debtors) cannot be paid any amount.
a) A person who receives negotiable instrument for consideration, before its maturity and in good faith without having
notice of defective title is known as the holder in due course. Certain defenses which can set up as against a holder cannot
set up as against the holder in due course. The holder in due course can get good title in the instrument though he has
received it from the transferor whose tile in the instrument is defective.

In this case, M is the holder in due course as he had received the promissory note without having knowledge of its
cancellation as it could not be found whether it was tore or not. Further, as M had received the Note for consideration, no
claim can be made against him as the instrument was theft and delivered to C by way of gift. Therefore, A cannot refuse to
pay the amount stated therein to M. He can recover the amount as mentioned in the note.

b) The separate legal existence of a company cannot be used as a means to achieve some illegal or fraudulent purposes.
The courts will refuse to uphold the separate existence of the company where it is formed to defeat or circumvent law, to
defraud creditors or to avoid legal obligations. This principle was laid down by the court in the case of Gilford Motor Co
vs. Horne (1930)1Ch 935.

In this case the company Golden Gate Automobiles Co. incorporated by Mr. Hari Singh is a mere cloak or sham for the
purpose of enabling him to commit a breach of his covenant against solicitation. The defendant company was just a
channel used by Mr. Hari Singh for the purpose of enabling him to obtain the advantage of the customers of the plaintiff
company, and the defendant company ought to be restrained as well as the defendant Hari Singh. Therefore, their claim is
maintainable against Hari Singh.

c) The addition of the words 'not negotiable' in a crossed cheque does not restrict the transferability of the instrument, but
the cheque is deprived of its special feature of negotiability. The general rule about the negotiability is that the holder in
due course of a bill or promissory note or cheque takes the instrument free from any defect which might be existing in the
title of the transferor. According to Section 130 of the Negotiable Instruments Act, 1881 a person who takes a cheque
bearing 'not negotiable' marking shall not have, and shall not be capable of giving, a better title to the cheque which the
person from whom he took it, had. A bank therefore should be extra careful in paying such cheques. The payment should
be made only after he is satisfied that the person demanding payment is the person entitled to receive it.

Account payee crossing directs the collecting banker to collect it for the payee only and warns that if the amount is
collected for some else, he may be held liable for damages.

In view of the advantages explained above, the combination of 'not negotiable' and 'A/C payee' crossing can be considered
the safest form of crossing.
The Institute of Chartered Accountants of Nepal
9 of 62
Suggested Answers – Corporate and Other Laws
CAP II Examination – December 2012

Part "B"
2. Answer the following questions:
a) D Ltd. was incorporated in accordance with the Company Act, 2063 (2006). It has 100 shareholders. The Articles of
Association (AOA) provides the quorum for the proceedings of the meeting of the company at least 10 shareholders of the
total shareholders representing more than 60 percent of the total numbers of allotted shares of that company present either
in person or by proxy. Further, it also provides that the matters relating to increase the authorized capital of the company
should be presented in general meeting of the company as a special resolution. It has got its license to commence its
business in February 1, 2011. As the company law and its rules, company intended to increase its authorized capital.
However, annual general meeting of the company could not hold except first annual general meeting even the pressure of
the shareholders. It is said to be that the shareholders of the company are the owner of the company. So, in relation to the
Companies Act, 2063 (2006) solve following problems with the help of the facts or the legal provisions of the Companies
Act. (5+3=8)
i) How can the shareholders compel to the company to hold annual general meeting? Explain the legal provisions.

ii)The Company called annual general meeting due to the pressure of the company's shareholders and passed resolution to
increase authorized capital having presence 6 in numbers that represents only 40% shareholders of the total numbers of
allotted share of that company. Is the resolution passed is valid or any else?

b) What do you understand by breach of contract? Explain briefly about the remedy available for the breach of contract.
7

Answer:
a) Shareholders of the Company have certain rights and liability only to the maximum value of shares which may have
subscribe or undertaken to subscribe.

D Ltd. could not hold the annual general meeting. In this connection, section 76 of the Companies
Act, 2063 (2007) has provided about the procedure of the calling annual general meeting. This section
provides alternative way if any public company could not manage to hold annual general meeting.

1) Directions of Company Registrar Office:


Every public company shall hold its annual general meeting every year within six months after the
expiry of its financial year. If such annual general meeting could not be held within six months after
the expiry of its financial year, the company registrar office may give direction to call the annual
general meeting of such company.

2) Making a petition by any shareholder:


Even the company fails to call the annual general meeting within three months after the receipt of the direction as referred
to above any shareholder may make a petition for, setting out the matter, to the court. Where such petition is made, the
court may either cause to hold the annual general meeting or issue any other appropriate order.
According to the above legal provisions, if any company couldn't hold annual general meeting within
time mentioned above the company registrar office may give direction to call the general meeting.
Any shareholder may make a petition setting out the matter to the court if the company unable to hold
the annual general meeting within three months from the date of receipts of the direction.

Here, the second question is testing the validity of increasing authorized capital. This resolution is
passed by the shareholders meetings with the presence of 6 members that represents only 40%
shareholders of the total numbers of allotted share of that company. This quorum is less than the
provision of the companies' AOA. In another side, section 73(2) of the Companies Act, 2063 (2007)
has given right to state required quorum of large number. Unless the article of association of a public
company provides for a larger number for the quorum, the proceeding of the meeting of the public
company shall be conducted unless at least three shareholders of the total shareholders, representing
more than fifty percent of the total number of allotted share of that company are present either in
person or by proxy." However, company is facing an extraordinary situation that is the resolution of
increasing authorized capital could not pass. So, the provision provided in the section 76(4) is the way-
The Institute of Chartered Accountants of Nepal
10 of 62
Suggested Answers – Corporate and Other Laws
CAP II Examination – December 2012

out of solving the extraordinary situation. It has provided circumstantial quorum despite the provision
of AoA or in the section 73(2). The shareholders present in the general meeting called pursuant to the
order of the court under Sub-section (3) shall be deemed to be a quorum. Hence, it is clear that the
resolution increasing authorized capital with fewer quorums is valid.
However such exemption has not been provided in case of meeting called as per the direction of the
Company Registrar Office.
b) Section 82 of the Contract Act, 2056 has defined the condition of the breach of contract. As per the section:
(1) In case any party to a contract does not meet liability under the contract, or gives a notice to the other party that he/she
will not perform the work to be performed under the contract, or in case his/her action or conduct shows that he/she is
incapable of performing the work under the contract, he shall be deemed to have breached the contract. (2) In case a party
has broken the contract under Sub-section (1), or in case his/her action or conduct shows that he/she has not basically
compiled with the contract, the other party shall not be compelled to perform the contract, and may cancel the contract by
furnishing a notice thereof to the other party.

In case of breach of contract, the aggrieved party has becomes entitled to get the following legal remedies;
 Suit for Rescission of cancellation of contract- Rescission means setting aside or cancelling the contract. In other
words, the term “Rescission” may be defined as the cancellation of the contract. When a contract is broken by one party,
the other party may treat the breach as discharge and refused to perform his part of the contract, i.e. putting an end to the
contract. On the rescission of the contract, the aggrieved party is discharged from all the obligations under the contract.
 Suit for Damages; The Damages are a monetary compensation allowed to the injured party for the loss or injury
suffered by him as a result of the breach of contract. In the event of breach of contract, the other party earns certain rights
including the right to claim damages or loss arising there from. The object of awarding is to put the aggrieved party in the
same position, has the contract been performed. The term „damages‟ is used to mean compensation in money as a
substitute for the promised performance. The fundamental principle underlying damages is not punishment but
compensation. Damages are to be awarded for losses which naturally arose from the breach of contract. The law of contract
does not seek to punish the guilty. The guilty party is liable to pay damages to the aggrieved party. The court will compel
the party in breach to make good the loss by paying to the other party.
 Suit for Quantum Meruit; The Latine phrase “Quantum Meruit” means payment in proportion to the amount of “work
done” or “reasonable value of work done”. A person can under circumstances, claim payment for work done of goods
supplied without any contract and in cases where the original contract has terminated by breach of contract by one party or
has become void for some reasons. This is known as “Doctrine of Quantum Meriut”. This doctrine is applies where there is
no express promise to definite remuneration to a person.
 Suit for specific performance; Specific performance means the actual carrying out of the contract as agreed. Where
damages are not an adequate remedy for breach of the contract, the court may direct the party in breach to carryout his
promise according to the terms of the contract. This is called “specific performance “of the contract. Specific performance
of the contract cannot be claimed as a matter of right .following are the circumstances , the claim for specific performance
is not applicable( Section 86(2) ) of the Nepalese contract Act, 2056;
i) Where compensation for the breach is adequate.
ii)Where the court cannot supervise the execution from time to time.
iii) Where the contract is of personal nature s(personal skill, efficiency, or Knowledge).
iv) Where a specific performance is not applicable due to the change of circumstances,
v) Where the breacher party itself has claimed for a specific performance against the injured party.
 Suit for injunction; Injunction means „ to stop doing something”. Injunction is also one of the remedies for an injured
party . Injunction is a court order that restraints the breacher party from doing wrong or continuing the wrongful act,
complained. Such order is usually granted by the court, to enforce negative stipulations in the case where the damage can
not be an adequate relief for the injury party. Such remedy is appropriate where there is an anticipatory breach of contract.
It is a kind of preventive relief for the aggrieved party.

3. Answer the following questions: (2×5=10)


a) Discuss how a disciplinary action is taken against the members and what punishment can be imposed to them by
Council.
b) Discuss the legal provisions for settlement of disputes of bonus under Bonus Act, 2030.

Answer:
a) Under section 14 (1) of the Act, if a member has acted as against the provisions of the Act or rules or code of conduct
on the application of anyone or such is known to the ICAN, to investigate upon the allegation and to recommend for the
necessary action to the council a Disciplinary committee shall be formed by council. Similarly under section 14 (5) of the
Act has provided on the necessary investigation conducted by the Disciplinary committee if a member is found guilty the

The Institute of Chartered Accountants of Nepal


11 of 62
Suggested Answers – Corporate and Other Laws
CAP II Examination – December 2012

committee with its decision may recommend the council for the necessary action. On the recommendation of the
committee the council may punish the accused members as follows:-
(a) Reprimanding;
(b) Removing from the membership for a period not exceeding Five years;
(c) Prohibiting from carrying on the accountancy for any specific period;
(d) Canceling the professional certificate or membership.
Under section 14 (1) it makes mandatory that a reasonable opportunity must be given of being heard before imposing the
punishment and the member punished if not satisfied with the decision of the council may apply to the court of Appeal to
challenge the decision of the council.

b) The Bonus Act, 2030 assures the bonus to the employees out of the profits earned by a factory or enterprise. The Act
also has made legal provisions for the settlement of disputes which may rise between the employee and management.
Section 16 (1) provides that if any dispute arises between employee and management with respect to the bonus, the Labour
Office shall resolve such dispute by negotiations having invited both the parties. Similarly, if the dispute could not be
resolved by negotiation the Labour Office shall ask to the concern enterprise and employees to produce necessary
documents and statements of accounts and shall give a decision on the basis of such documents and statements. When the
decision is given by the Labour Office, the party who is dissatisfied with the decision may appeal to the Labour Court
within 35 day of receipt of such notice and the decision made by the Labour Court shall be final for the matter.

4. Answer the following questions: (2×5=10)


a) When does a public limited company converse into a private limited company? Explain the legal provisions.
b) What are the terms to be abided by any company incorporated under the companies Act, 2063 in addition to those set
forth in the Companies Act, 2063, Memorandum of Association or Articles of Association?

Answer:
a) Section, 14 of the Companies Act, 2063 under different clauses has stated the provisions for the conversion of a public
company into a private company. Sub-section 1 of section 14 reads thus:
In the following circumstance, a public company shall be converted into a private company under this section:
-If the number of shareholders of the public company becomes less than seven,
-If the public company fails to maintain its paid-up capital under section 11 or the paid-up capital as
referred to in section 11 is not maintained because of reduction in capital pursuant to section 57.
Provided, however, that this provision shall not apply to the company as referred to in sub-section (2)
of section 11.
In the event of occurrence of a circumstance as referred to in sub-section (1), the concerned public company shall make
necessary amendments to its memorandum and articles of association and convert it into a private company within six
months. After making the necessary amendments in the memorandum and articles of association, the company, within 30
days after the making of such amendment, shall make an application to the Office of the Company Registrar with the
prescribed fees for being converted into a private company. The Office on the receipt of an application shall mention in the
company register the contents of conversion of such company into a private company and give a company conversion
certificate within sixty days.

b) A company incorporated under this Act shall abide by the following terms, in addition to those set forth in this Act,
memorandum of association or articles of association: Section 10
(a)The company shall carry on all of its activities and transactions by its name.
(b) A private company shall add the words “private limited‟‟ to its name as the last words and a public company shall add
the word “limited” to its name as the last word. However, this provision shall not apply to a company not distributing
profit.
(c) A private company shall not sell its shares and debentures publicly.
(d) A private company shall not pledge, or otherwise transfer title to, its securities to any person other than its shareholder
without fulfilling the procedures contained in the memorandum of consensus agreement,
(e) A company shall not open a partnership or private firm.
(f) Except as otherwise provided in this Act, a company not distributing profits shall not distribute dividends among its
members or pay, directly or indirectly, any amount to a member or his/her close relative.

5. Answer the following questions: (3×5=15)


a) What are the acts prohibited and punishment prescribed in Insurance Act, 2049?
b) Define capital fund under BAFIA, 2063.
c) Highlight the incentives for the foreign investors assured by the Foreign Investment and Technology Transfer Act,
2049
The Institute of Chartered Accountants of Nepal
12 of 62
Suggested Answers – Corporate and Other Laws
CAP II Examination – December 2012

Answer:
a) Insurance Act, 2049 has provided in its section 36 about the acts prohibited or punishment thereto as follows:

i) Violation of Law:
 If any insurer or the Director of the Insurer, employee or Surveyor, Broker or Insurance Agent knowingly violates this
act or the rules made under this act or order or directives;
 If such authority does not perform any function to be performed or does any act not to be done
 The Insurance Board may punish to such Insurer or the Director, employee or *Surveyor, Broker or Insurance Agent
with a fine ranging from three thousand rupees to ten thousand rupees. If such offense has been made frequently, he will be
fined at the rate of extra five hundred rupees for each subsequent offense.

ii) Non-Compliance of Procedures:


 If any Insurer or Insurance Agent or Broker has made any Insurance Business without following the procedures to be
followed pursuant to this Act it shall be deemed to be the offence.
 The Security Board may fine such authority up to ten thousand rupees.

iii) Improper activities or malafide intention:


 The accounts, records, register, details, information or any other documents should be maintained, prepared, formed or
submitted pursuant to this Act or the Rules made under this Act. If the job has not done in time by knowingly or with
malafide intention he/she should be liable for his mal performance.
 Should not maintained or submitted the false details or documents by any-body;
 If there is beyond performance he/she is liable of punishment with a fine up to thirty thousand rupees or imprisonment up
to two years or with both.

b) Section 2 (r ) of the BAFIA Act, 2063 has defined the terms Capital Fund. As per this section “capital fund”
means the total of the primary and supplementary capital of a bank or financial institution, and this term includes any other
fund of the institution as prescribed as such by the Nepal Rastra Bank from time to time.

Section 42 of the Act has explained the capital fund for the licensed institution.
(1) Every licensed institution shall maintain a capital fund in the ratio prescribed by the Rastra Bank on the basis of its total
assets or total risk-weighted assets.
(2) If any bank or financial institution fails to maintain the capital fund as referred to in Sub-section (1), the Board shall
give information thereof to the Rastra Bank within thirty five days. The information so given shall also be accompanied by,
inter alia, the reasons for the failure to maintain the capital fund and the plan or program prepared by the Board to increase
the capital fund and restore it to its previous condition.
(3) On receipt of the information referred to in Sub-section (2), if the Rastra Bank considers the plan or program submitted
by the Board to be reasonable, it may give a directive to the concerned bank or financial institution to implement such plan
or program; and if any amendment or alteration is to be made in the proposed plan or program, it may give a directive,
accompanied by the reasons for such amendment or alteration, to the concerned bank or financial institution to amend or
alter such plan or program and implement the same.
(4) If any bank or financial institution fails to meet the obligation referred to in this Section or if it does not appear that it
can do so immediately, the Rastra Bank may take action referred to in Section 37 against such bank or financial institution.
(5) Notwithstanding anything contained elsewhere in this Section, if it is found that the concerned bank or financial
institution has failed to maintain its capital fund as provided for in Sub-section (1) by the reason of the failure of the Board
of Directors to comply with the directives given by the Rastra Bank from time to time or by the reason of malice
recklessness or mala fide intention of the director or chief executive, the Rastra Bank may punish such director or chief
executive with a fine equal to the amount involved; and if such act of the director or chief executive is found to have
caused any loss or damage as a result of the failure to maintain the capital fund, the Rastra Bank, the concerned bank or
financial institution or any shareholder holding up to five per cent of the paid-up capital of such bank or financial
institution may either individually or collectively file a petition with the court against such director or chief executive for
the recovery of that amount, in accordance with laws in force.

c) Section 2(1) of the Foreign Investment and Technology Transfer Act, 2049 has given the definition of the term
foreign investment. It defines- the term foreign investment means the following investment made by any foreign investor in
any industry of Nepal:
i) Investment in share capital.
The Institute of Chartered Accountants of Nepal
13 of 62
Suggested Answers – Corporate and Other Laws
CAP II Examination – December 2012

ii) Re-investment of income received from the investment in share capital.


iii) Investment as a loan or loan facilities.
There are numbers of facilities given by the Act to attract the foreign investment in Nepal. Pursuant to section 5 of the Act
the following facilities are provided to the foreign investors:
i) Facilities to take away the foreign currencies from Nepal:
The foreign investors can take away the following amount in the foreign currencies from Nepal:
1.Amount received from sale of whole or some part of share.
2.Amount received as profit or dividend of foreign investment.
3.Amount received as interest of loan or the payment of interest.
4.Amount received under the technology transfer agreement.

ii) Visa facilities: Under section 6 of the Act there are three types of visa issued to the foreign
investors. They are- Non-tourist visa, Business visa and Residential visa.
Foreign Investment and Technology Transfer Act 2049 (FITTA 2049) has different provisions on visa
facilities. As per FITTA 2049, a foreign national visiting Nepal in connection with undertaking any
study or carrying out any research with the objective of making investment in Nepal shall be provided
a non tourist visa for up to six months.
A foreign investor or dependent family or authorized representative of a foreign investor and department family of such
authorized representative shall for the purpose of stay in Nepal be provided a business visa until the foreign investment is
retained.
A foreign investor who, at a time, makes investment in an amount no less than one hundred thousand United States dollar
or in convertible foreign currency equivalent thereto, and his/her dependent family shall be granted a residential visa until
such investment is retained.
6. Answer the following questions: (2×5=10)
a) What is insider trading? Who can obtain inside information? Is there any provisions regarding the punishment about
the conviction of offense of insider trading?
b) Professional Solution Ltd announced 5th annual general meeting of the company in Kantipur daily published on 23rd
Katrik 2069.There is no mention any date, time duration, place and agenda of the meeting. Company published a notice for
presence of shareholders in annual general meeting only. Is this meeting valid under Company Act, 2063? Is there any
provision regarding the process of calling annual general meeting of public company? Explain.

Answer:
a) Section 91 of Securities Act, 2063 has defined insider trading as follows:
(1) If any person deals in securities or causes any other person to deal in securities on the basis of any
insider information or notice that are unpublished or communicates any information or notice known
to such a person in the course of the discharge of his or her duties in manner likely to affect the price
of securities such a person shall be deemed to have been committed an insider trading in securities.
(2) Notwithstanding anything contained in Sub-section (1), any transactions already carried on shall
not be deemed to be affected at all merely by the reason that an insider trading has been committed.
Section 92 of the Act has defined the person who is likely to be involved in insider trading: The persons shall be deemed to
be those who have access to the insider information or notice not published by any body corporate:

(a) A director, employee or a person, who can obtain any information or a notice in the capacity of a shareholder of that
body corporate,

(b) A person who can obtain any information or a notice in the capacity of a professional service provider to that body
corporate,

(c) A person who can obtain any information or a notice having a direct or indirect contact with the person or source as
specified in Clauses (a) and (b)

Punishment: A person who commits an insider trading as referred to in Section 91 section 91 shall, upon being convicted
of the offense of insider trading, be liable to the punishment with a fine equal to the amount in controversy or with
imprisonment for a term not exceeding one year or with both punishments.

The Institute of Chartered Accountants of Nepal


14 of 62
Suggested Answers – Corporate and Other Laws
CAP II Examination – December 2012

b) Professional solution ltd company announced a general meeting by publishing a notice in Kantipur Daily newspaper.
In this notice there is no any information regarding the business of the meeting. According to companies Act, 2063, section
67 (2) A public company shall a notice specifying the place, date, and agenda of meeting to every shareholder at the
address supplied by that shareholder to the company, in advance of at least 21 days to hold the annual general meeting A
notice thereof shall also be published at least twice in a national newspaper. Section 67 (3) No decision shall be taken in
any general meeting on any matter which has not been notified in advance pursuant to sub-section (2) except in the
following circumstances;
 Except as otherwise provided in the other sections of this Act, if the shareholders representing sixty seven percent of the
total shares of the company who rare entitled to vote at the general meeting attend in person or by proxy and vote in favour
of taking decision on any matters.
 If the matter was already notified for being transacted in any general meeting which has been adjourned.
Company Act, 2063 make compulsion to call annual general meeting before 21 days with mentioning the information
about the date, time, place and agenda for the meeting that must be send to every shareholders of the company. This is
mandatory for the company management. This is not optional. Here the professional solution limited does not fulfill the
criteria for calling the annual general meeting of the company according to companies Act, 2063. So the validity of the
meeting is not there and that meeting should not be accepted by the company Registrar office.

7. Write short notes on the followings: (3×5=15)


a) Objectives of Nepal Rastra Bank
b) i) Promissory Note ii) Bills of Exchange
c) Propriety Audit

Answer:
a) Objectives of Nepal Rastra Bank
Section 4 of Nepal Rastra Bank Act, 2058 has defined the objectives of the bank as follows:
(a) To formulate necessary monetary and foreign exchange policies in order to maintain the stability of
price and balance of payment for sustainable development of economy, and manage it;
(b) To promote stability and liquidity required in banking and financial sector;
(c) To develop a secure, healthy and efficient system of payment;
(d) To regulate, inspect, supervise and monitor the banking and financial system; and
(e) To promote entire banking and financial system of the Nepal and to enhance its public credibility.

The Bank shall, without any prejudice to the objectives referred to above, extend co-operation in the
implementation of the economic policies of Government of Nepal.

b) i) Promissory Note ii) Bills of Exchange


Section 2(f) of the Negotiable Instrument Act, 2034(1997) has defined the term as "Promissory
Note" means an instrument in writing except or Bank note containing an unconditional undertaking,
signed by the maker, to pay a certain sum of money to, or to the order of, a certain person or to the
bearer of the instrument. Provided that, this word shall not include the document prepared pursuant to
the Chapter of Transaction of the Muluki Ain (General Code). In such way, NRB Act, 2058 and
BAFIA, 2063 also defined this term in Section 2u and 2kk respectively.
This instrument should satisfy the elements of writing, express promise, unconditional, money
only, must be authorized with signature by the maker and certainty of the parties whether certain
person or bearer. In this document, parties are certain and should indicate in the face of the instrument.
Generally, two parties i.e. make- the person who makes the note and is known as the maker, and
payee- to whom the promise is made, are involved in the promissory note.
Therefore, promissory note, is, a written instrument prepared by the maker for the fulfillment of
own promise to pay certain sum of money to the certain person or bearer without condition.

ii)
Section 2(g) of the Negotiable Instruments Act, 2034(1997) has defined the term as "Bill of
Exchange" means an instrument in writing containing an unconditional order, signed by the maker,
directing a certain person to pay a certain sum of money to, or to the order of a certain person or to the

The Institute of Chartered Accountants of Nepal


15 of 62
Suggested Answers – Corporate and Other Laws
CAP II Examination – December 2012

bearer of the instrument in a certain date or after certain period of time or at the demand. In such way,
NRB Act, 2058 and BAFIA, 2063 also defined this term in Section 2t and 2jj respectively.
This instrument should also satisfy the certain elements i.e. writing, imperative order, certainty in
money, its amounts, date, period of time and parties, unconditional and must be authorized with
signature by the maker. Generally, there are three parties: like drawer or maker-who makes the BoE,
drawee-person to whom it is addressed and payee. Sometimes drawer and payee are same.
According to the above mentioned facts, Bills of Exchange are also a written instrument. It
contains a direction or order of a drawer to a certain person to pay a certain sum of money to the
person as specified in the instrument or to bearer.

c) Propriety Audit
The Auditor General of Nepal is considered five principles while conducting the audit of the government owned
corporations. One of these five principles is propriety audit.

Section 5 of the Audit Act, 2048 has defined matters to be audited in view of the propriety audit by the Office of the
Auditor General of Nepal.
(1) The Auditor General shall audit following matters considering the propriety thereof-

(a) On the propriety of any expenditure and its authorization, if in the opinion of the Auditor General such expenditure is a
reckless one or is an abuse of national property, whether movable or immovable, despite that the expenditure confirms to
the authorization, and

(b) On the propriety of all authorizations issued in respect of any grant of national property whether movable or
immovable, fixed or current, or underwriting of any revenue, or any contract, license or permits relating to mining, forest,
water resources, etc. and any other act of abandoning movable or immovable, assets of the nation.

(2) The Auditor General may not include in the report minor items of discrepancy and other items deemed as insignificant
in view of their property which were observed during the audit of income and expenditure.

The Institute of Chartered Accountants of Nepal


16 of 62
The Institute of Chartered Accountants of Nepal
Suggested Answers of Financial Management

CAP II Examination- December 2012

Working notes should form part of the answer. Make assumptions wherever necessary.

1. Following are the condensed Balance Sheets of Omega Ltd. for two years and the Statement of Profit and Loss
for one year:
Balance Sheet as at end of Ashadh
(Figures in Rs. „000)
2069 2068
Equity Share Capital 150 110
10% Redeemable Preference Shares 10 40
Capital Redemption Reserve 10 -
General Reserve 15 10
Profit and Loss Account balance 30 20
8% Debenture with convertible option 20 40
Other Term Loans 15 30
Total 250 250
Fixed Assets less Depreciation 130 100
Long Term Investments 40 50
Working Capital 80 100
Total 250 250
Statement of Profit and Loss
for the year ended 31st Ashadh, 2069
(Figures in Rs. „000)
Sales 600
Less: Cost of Sales (400)
200
Less:
Establishment Charges 30
Selling and Distribution Expenses 60
Loss on Sale of Equipment 15
Interest Expenses 5 (110)
90
Add:
Interest Income 4
Foreign Exchange Gain 10
Dividend Income 2
Damages received for loss of reputation 14 30
120
Less: Depreciation (50)
70
Taxes (30)
40
Dividends (15)
Net profit carried to Balance Sheet 25
Chief accountant of Omega Ltd. informed that ledgers relating to debtors, creditors and stock for both the years
were seized by the income tax authorities for the purpose of investigation and the same would not be available for
at least three months. However, he is able to furnish the following data:
(Figures in Rs. „000)
Ashadh end 2069 Ashadh end 2068
Dividend receivable 2 4
Interest receivable 3 2
Cash on hand and with bank 7 10
Investment maturing within two months 3 2
15 18
Interest payable 4 5
Taxes payable 6 3
10 8
Current ratio 1.5 1.4
Acid test ratio 1.1 0.8
It is also gathered that debenture-holders owning 50% of the debentures outstanding as on Ashadh end 2068
exercised the option for conversion into equity shares during the financial year ending on Ashadh 2069 and the
same was put through. Besides, an equipment was sold for Rs. 25,000 during the financial year 2068/69.
Suggested Answers – Financial Management
CAP II Examination – December 2012
Required:
Prepare a cash flow statement for the financial year 2068/69 under direct method. 20

Answer:
Cash Flow Statement
(Figures in Rs. ‘000)
A. Cash Flows from Operating Activities:
Cash Receipts from customers (WN 2) 621
Cash paid to suppliers and employees (WN 3) (406)
Operating expenses Paid (WN 4) (90)
Cash generated from operations 125
Income-tax paid (WN 5) (27)
Cash flow before extraordinary item 98
Add: Extraordinary items:
Foreign Exchange Gain 10
Damages for loss of reputation 14
Net cash from operating activities 122

B. Cash flows from Investing Activities:


Purchases of fixed assets (WN 6) (120)
Proceeds from sale of equipment 25
Proceeds from sale of investment (50 – 40) 10
Interest received (WN 7) 3
Dividend received (WN 7) 4
Net cash used in investing activities (78)

C. Cash flows from financing Activities:


Proceeds from issue of equity share capital (WN 8) 20
Redemption of Preference Share Capital (30)
Repayment of term loan (15)
Interest paid (WN 9) (6)
Dividend paid (15)
Net cash used in financing activities (46)
Net decrease in cash and cash equivalents (2)
Cash and cash equivalents at the beginning of the period (10 + 2) 12
Cash and cash equivalents at the end of the period 10

Working Notes: (figures in Rs.'000)


1. Determining the value of Current Assets and Current Liabilities 2068 2069
Current Ratio 1.40 1.50
Working Capital 100 80
Current Liabilities (100÷0.4) & (80÷0.5) 250 160
Current Assets (250+100) & (160+80) 350 240
Acid Test Ratio 0.8 1.10
Current Liabilities (as above) 250 160
Therefore, Quick Assets (QA) (CL×ATR) 200 176
Stock (CA – QA) 150 64
Other Current Assets (as given ) 18 15
(Dividend + Interest + Cash Equivalents)
Therefore, Debtors 182 161
(QA – Other Current Assets)
Therefore, Creditors 242 150
(CL – Interest payable – Taxes payable)
2. Cash Receipts from Customers
Sales (on accrual basis) 600
Add: Opening debtors (WN 1) 182 782
Less: Closing debtors (WN 1) - 161
Cash Received from Customers 621
3. Cash paid to suppliers
Cost of sales 400
Add: Opening creditors (WN 1) 242
Closing stock (WN 1) 64 706
Less: Closing creditors (WN 1) 150
Opening stock (WN 1) 150 - 300
406

The Institute of Chartered Accountants of Nepal


2 of 62
Suggested Answers – Financial Management
CAP II Examination – December 2012
4. Operating expenses Paid
Establishment Charges 30
Selling and Distribution expenses 60 90

5. Tax paid during the year


Tax payable in the beginning 3
Add: Provision for tax 30 33
Less: Tax payable at the end: -6
Tax paid during the year 27

6. Purchase of Fixed Assets


Balance at the end 130
Add: Depreciation for current year 50
Assets sold (Book Value ) (25 + 15) 40 220
Balance at the beginning 100
Purchase of Fixed Assets during the year: 120

7. Interest and Dividend received during the year


Interest Dividend
Opening Balance 2 4
Add: Accrued Income (Current Year) 4 2
6 6
Less: Closing Balance -3 -2
Received During the Year 3 4
8. Issue of Equity Share Capital for Cash
Capital at the end 150
Less: Capital issued to debenture-holders - 20 130
(Conversion- 50% of 40)
Less: Opening Capital - 110
Capital issued for Cash 20
9. Interest Paid during the year
Balance in the beginning 5
Add: Accrued during the year 5 10
Less: Balance at the end -4
Paid during the year 6

10. Cash and Cash Equivalents 2068 2069


Cash and Bank 10 7
Investments 2 3
12 10
11. Profit and Loss Account
Opening Balance 20
Profit for Current Year 25 45
Less: Transfer to General Reserve (15-10) 5
Transfer to Capital Redemption Reserve (10-0) 10 - 15
Closing Balance 30

12. The preference share capital in the beginning and at the end was Rs. 40 and Rs. 10 thousand respectively.
This redemption is backed by the issue of Equity Share Capital of Rs. 20 thousand and transfer of Profit and
Loss Account balance of Rs. 10 thousand to Capital Redemption Reserve Account.

2.
a) Albertine Ltd. has an investment opportunity available which will involve a capital outlay in each of the
next 2 years and which will produce benefits during the following 3 years. A summary of the financial implications
of this investment is given below:
Year Cash Flow (Rs. „000) Year Cash Flow (Rs. „000)
1 (1,000) 4 1,300
2 (1,000) 5 3,100
3 100
Albertine Ltd. currently has 100,000 shares in issue. The dividend just paid was Rs. 15 per share. In the absence of
the above investment, dividends are expected at this level for the next 3 years, but will then demonstrate perpetual
growth of 10 per cent per annum. The company is currently all equity financed and the required rate of return of the
equity investor is estimated to be 18 per cent.
The company has a long established policy of not using any debt finance and, because of the current depressed
state of the stock market, could not, in the near future, issue new equity. The only possible way of financing the
investment is, therefore, to reduce the dividend payments in the next 2 years. Cash received from the new
The Institute of Chartered Accountants of Nepal
3 of 62
Suggested Answers – Financial Management
CAP II Examination – December 2012
investment will all be distributed in the form of dividend. Growth in dividends at the rate of 10% will also be
maintained because of other operations.
Required: (3+5=8)
i) Calculate the current price of share of Albertine Ltd. when investment proposal is not accepted.
ii)Calculate the share price after the investment has been accepted using dividend valuation model, assuming that
the market knows of the dividend changes that will result from the investment.

b) As a part of the strategy to increase sales and profits, the sales manager of a fast moving consumer goods
(FMCG) company proposes to sell goods to a group of new customers with 10% risk of non-payment. This group
would require one and a half month credit and is likely to increase sales by Rs. 100,000 per annum. Production and
selling expenses amount to 80% of sales and the income-tax rate is 50%. The company‟s minimum required rate of
return (after tax) is 25%.
Required: (3.5+3.5=7)
i) Comment on the acceptance of the sales manager‟s proposal.
ii) Find the degree of risk of non-payment that the company should be willing to assume if the required rate of
return (after tax) were 30%.

Answer:
a)
i) Current price of Share (when investment proposal is not accepted)
The current market price of the share is the present value of expected future dividends discounted at the required
rate of return, i.e. 18%.Since the company is expected to pay a dividend of Rs. 15 for the next 3 years and
thereafter, the dividend will grow at the rate of 10%. The present market price with these parameters is ascertained
as below:
Dividend per year = Rs. 15
PVAF (at 18%, 3 years) = 2.174
Therefore, PV of dividends = Rs. 15 x 2.174 = Rs. 32.61
Price of share at the end of year 3 (P 3) = D4 = Rs. 15 (1+0.10) = Rs. 206.25
(with perpetual growth of 10%) ke – g 0.18 – 0.10
Present value of this amount at 18% for 3rd year = Rs. 206.25 x PVF(18%, 3)
= Rs. 206.25 X0.609 = Rs. 125.61
Present market price = Rs. 125.61 + Rs 32.61 = Rs. 158.22

ii) Current price of Share (when the investment proposal is accepted)


In first and second year the investment required of Rs.1,000,000 is financed by reducing the old dividend rate of
Rs.15 per share for 100,000 number of shares. And, thereafter all the cash flows from new investment is distributed
as additional dividend.
The present value of dividend under this situation will be as follows:
Year Old Dividend (Rs.) Change in Net Dividend PVIF @ PV
Dividend (Rs.) (Rs.) 18% (Rs.)
1 15 -10 5 0.847 4.24
2 15 -10 5 0.718 3.59
3 15 1 16 0.609 9.74
4 (15+10% of 15) =16.5 13 29.5 0.516 15.22
5 (16.5+10% of 16.5) = 18.15 31 49.15 0.437 21.48
Total 54.27
Price of share at the end of year 5 (P 5) = D6 = Rs. 18.15 (1 + 0.10) = Rs. 250 (with
perpetual growth of 10%) ke – g 0.18 – 0.10
Present value of this amount at 18% for 5th year = Rs. 250 XPVF(18%, 5)
= Rs. 250 X 0.437 = Rs. 109.25
Therefore, the market price under this situation = Rs. 109.25 + Rs. 54.27 = Rs. 163.52

(b)
i) Evaluation of Sales Manager's proposal Rs.
Additional sales from new customers per annum 100,000
Less: Risk of non-payment @ 10% 10,000
Net Turnover 90,000
Production and selling expenses (80% of sales) 80,000
Profit before tax 10,000
Income tax @ 50% 5,000
Profit after Tax 5,000
Average Investment in Debtors:
The credit period being 1 ½ months, there will be turnover of debtors of 8 times considering 12 month‟s year.
Thus, average debtors will be Rs. 100,000 / 8 = Rs. 12,500
Cost of goods sold being 80% of sales, the average investment in debtors would be 80% of Rs. 12,500, i.e. Rs.
10,000.
The Institute of Chartered Accountants of Nepal
4 of 62
Suggested Answers – Financial Management
CAP II Examination – December 2012
Thus, the rate of return (being PAT of Rs. 5,000) =
Rs. 5,000/ 10,000 x 100/10,000 = 50%
Since the company‟s minimum rate of return is 25%, the sales manager‟s proposal should be accepted.
Alternatively,
Cost of investment in debtor = 25% of Rs.10,000 = Rs.2,500
Increase in PAT = Rs. 5,000
Since, increase in PAT is greater than the additional cost of investment in debtor; the Sales Manager's proposal is
acceptable.

ii) Acceptable degree of risk of non-payment with the required rate of return (after tax) of 30%:
Particulars Rs.
Average investment in debtors 10,000
Required profit after tax @ 30% 3,000
Profit before tax (Grossed up by 50%) 6,000
Production and selling expenses 80,000
Required sales to achieved desired return 86,000
Additional sales 100,000
Therefore, acceptable degree of risk of non-payment 14,000
Acceptable degree of risk (in %) 14%

3.
a) South China Corporation is evaluating on investment projects for investment in new machinery to produce a
recently-developed product. The cost of the machinery, which is payable immediately, is Rs. 1.5 million, and the
scrap value of the machinery at the end of four years is expected to be Rs. 100,000. Capital allowances (tax-
allowable depreciation) can be claimed on this investment on a 25% reducing balance basis. Information on results
from the investment has been forecast to be as follows:
Year 1 2 3 4
Sales volume (units/year) 50,000 95,000 140,000 75,000
Selling price (Rs./unit) 25 24 23 23
Variable cost (Rs./unit) 10 11 12 12·50
Fixed costs (Rs./year) 105,000 115,000 125,000 125,000
This information must be adjusted to allow for selling price inflation of 4% per year and variable cost inflation of
2.5% per year. Fixed costs, which are wholly attributable to the project, have already been adjusted for inflation.
South China Corporation pays profit tax of 30% per year on one year in arrears.
South China Corporation has a nominal before-tax weighted average cost of capital of 12% and a nominal after-tax
weighted average cost of capital of 7%.
Required:
Calculate the net present value of the project and comment on whether this project is financially acceptable to
South China Corporation. 12

b) Consider the following information:


Stock A, Beta >1.
Stock B, Beta =1
Stock C, Beta <1.
Required: (2+1=3)
i) Explain the relation between the above stocks and their market.
ii) Explain, if you are a risk averter, in which stock would you invest.

Answer:
(a) Calculation of net present value (NPV)
As nominal after-tax cash flows are to be discounted, the nominal after-tax weighted average cost of capital of 7%
must be used.
Calculation of Net Present Value (Rs.)
Particular Year 1 2 3 4 5
Sales revenue WN 1 1,300,000 2,466,200 3,621,800 2,018,250
Variable costs WN 2 (512,500) (1,098,200) (1,808,800) (1,035,000)
Contribution 787,500 1,368,000 1,813,000 983,250
Fixed costs (105,000) (115,000) (125,000) (125,000)
Taxable cash flow 682,500 1,253,000 1,688,000 858,250
Tax liabilities (204,750) (375,900) (506,400) (257,475)
CA tax benefits WN 3 112,500 84,375 63,281 159,844
After-tax cash flow 682,500 1,160,750 1,396,475 415,131 (97,631)
Scrap value 100,000
Net cash flow 682,500 1,160,750 1,396,475 515,131 (97,631)
The Institute of Chartered Accountants of Nepal
5 of 62
Suggested Answers – Financial Management
CAP II Examination – December 2012
DF at 7% 0.9346 0.8734 0.8163 0.7629 0.7130
Present values 637,865 1,013,800 1,139,943 392,993 (69,611)

Present Value of Cash inflows = Rs.3,114,990


Cost of Machine = Rs.1,500,000
Net Present Value (NPV) = Rs.1,614,990

The project has a positive NPV of Rs. 1,614,990, so it is financially acceptable to South China Co. However, as this
is a recently-developed product, it may be appropriate to use a project-specific discount rate that reflects the risk of
the new product launch.

Working Note 1: Calculation of inflation adjusted Sales Revenue


Year 1 2 3 4
Price Inflation 100% 104% 108.16% 112.486% 116.985%
Selling price (Rs./unit) 25·00 24·00 23·00 23·00
Inflated selling price (Rs./unit) 26·00 25·96 25·87 26·91
Sales volume (units/year) 50,000 95,000 140,000 75,000
Sales revenue (Rs./year) 1,300,000 2,466,200 3,621,800 2,018,250

Working Note 2: Calculation of inflation adjusted Variable Cost


Year 1 2 3 4
Price Inflation 100% 102.5% 105.063% 107.69% 110.382%
Variable cost (Rs./unit) 10·00 11·00 12·00 12·50
Inflated variable cost (Rs./Unit) 10·25 11·56 12·92 13·80
Sales volume (units/year) 50,000 95,000 140,000 75,000
Variable costs (Rs./year) 512,500 1,098,200 1,808,800 1,035,000

Working Note 3: Calculation of capital allowance tax-benefits


Year 1 2 3 4
Capital Assets (Depreciation base) (Rs.) 1,500,000 1,125,000 843,750 532,812*
Depreciation Rate 0.25 0.25 0.25
Depreciation Amount (Rs.) 375,000 281,250 210,938 532,812
Tax Rate 0.30 0.30 0.30 0.30
Capital allowance tax benefits (Can be adjusted in 112,500 84,375 63,281 159,844
the year of relevant tax payable)
*Note: Capital base for 4th year = 843,750-210,938-100,000 = 532,812

(b)

i) The beta factor is a measure of a stock‟s volatility in terms of market risk. Therefore,

Stock A:
Where Beta> 1, the shares are described as aggressive; they outperform the market. This means they give a bigger
return than the market when the market return is positive and a bigger loss than the market, when the market return
is negative.

Stock B:
Where Beta= 1, the shares are described as neutral; their returns are in line with the average return of the stock
market.

Stock C:
Where Beta< 1, the shares are described as defensive; they are less risky than the market generally.

ii)A risk averter would generally invest in those stocks which are less risky than that of market. Therefore, Stock C
would be the choice for risk averters.

4.
a) PQ Limited currently has annual sales of Rs. 500,000 and an average collection period of 30 days. It is
considering a more liberal credit policy. If the credit period is extended, the company expects sales and bad debt
losses to increase in the following manner:
Credit Policy Increase in credit period Increase in sales (Rs.) Bad debts % of total Sales
A 10 days 25,000 1.2
B 15 days 35,000 1.5
C 30 days 40,000 1.8

The Institute of Chartered Accountants of Nepal


6 of 62
Suggested Answers – Financial Management
CAP II Examination – December 2012
D 42 days 50,000 2.2

The selling price per unit is Rs. 2. Average cost per unit at the current level of operation is Rs. 1.50 and variable
cost per unit is Rs. 1.20. The current bad debt loss is 1% of the total sales and the required rate of return on
investments is 20%. Ignore taxes and assume 360 days in a year.
Required:
Recommend the credit policy to be adopted. 8

b) KLS Limited has a total capitalization of Rs. 1,000,000 and it normally earns Rs. 100,000 before interest and
taxes. The finance manager of the company wants to take the decision regarding the capital structure. After a study
of the capital market, he gathers the following data:
Equity Capitalization Rate
Amount of Debt (Rs.) Interest rate (%) at given level of debt (%)
0 - 10.00
100,000 4.0 10.50
200,000 4.0 11.00
300,000 4.5 11.60
400,000 5.0 12.40
500,000 5.5 13.50
600,000 6.0 16.00
700,000 8.0 20.00

Assume that corporate taxes do not exist, and the firm always maintains its capital structure at book values.
Required: (4+3=7)
i) What amount of debt should be employed by the firm if the traditional approach is held valid?
ii) If the Modigliani-Miller approach is followed, what should be the equity capitalization rate?

Answer:
(a) The firm will maximize the shareholders value if it extends its period by additional 30 days (since expected
return is higher than required return). In fact, it can further relax credit period until its expected return becomes 20%
or net gain becomes zero.
Particulars Increase in credit period
Existing 10 days 15 days 30 days 42 days
A Credit period (days) 30 40 45 60 72
B Annual Sales (Rs.) 5,00,000 5,25,000 5,35,000 5,40,000 5,50,000
C Level of receivables 41,667 58,333 66,875 90,000 1,10,000
(at sales value) (AXB)/360 (Rs.)
D Incremental investment in receivables - 16,667 25,208 48,333 68,333
(C-41,667) (Rs.)
E Required incremental profit at 20% - 3,333 5,042 9,667 13,667
(0.20XD) (Rs.)
F Incremental contribution on additional sales - 10,000 14,000 16,000 20,000
@40% (2-1.2)/2 (Rs.)
G Bad debt losses (BX %bad debts) (Rs.) 5,000 6,300 8,025 9,720 12,100
H Incremental Bad debt losses - 1,300 3,025 4,720 7,100
(G-5,000) (Rs.)
I Incremental expected profit - 8,700 10,975 11,280 12,900
(F-H) (Rs.)
J Net Gain (I-E) (Rs.) - 5,367 5,933 1,613 (767)

Alternatively,
The investment in receivables can be calculated at cost. At current level of sales, the firm's average unit cost is Rs.
1.50. Since variable cost per unit is 1.20, we can find the fixed cost as follows:

Fixed Cost = Total Cost-Variable Cost


= (Rs.5,00,000) x 1.50/Rs. 2 - (Rs.5,00,000) x Rs.1.20/Rs.2
= Rs. 375,000 - Rs. 3,00,000
= Rs. 75,000

Thus, the total cost for different level of sales (assuming unit price and fixed cost do not change):
Sales (Rs.) Variable Cost (Rs.) Fixed Cost (Rs.) Total Cost (Rs.)
5,25,000 5,25,000 x 1.20/2 = 315,000 75,000 3,90,000
5,35,000 5,35,000 x 1.20/2 = 321,000 75,000 3,96,000
5,40,000 5,40,000 x 1.20/2 = 324,000 75,000 3,99,000
5,50,000 5,50,000 x 1.20/2 = 330,000 75,000 4,05,000
The Institute of Chartered Accountants of Nepal
7 of 62
Suggested Answers – Financial Management
CAP II Examination – December 2012

Investment in account receivables will be:


Investment in Receivables (Rs.) Changes in Investment (Rs.)
(3,75,000) x 30/360 = 31,250 -
(3,90,000) x 40/360 = 43,333 12,083
(3,96,000) x 45/360 = 49,500 18,250
(3,99,000) x 60/360 = 66,500 35,250
(4,05,000) x 72/360 = 81,000 49,750

The net gain from the credit policy can be re-calculated using incremental investment in accounts receivables at
cost. It would be higher now.
Particulars Increase in credit period
Existing 10 days 15 days 30 days 42 days
A Credit period (days) 30 40 45 60 72
B Incremental investment in receivables (Rs.) - 12,083 18,250 35,250 49,750
C Cost of investment at 20% (Rs.) - 2,417 3,650 7,050 9,950
D Incremental Bad debt losses (Rs.) - 1,300 3,025 4,720 7,100
E Incremental contribution on additional sales - 10,000 14,000 16,000 20,000
@40% (2-1.2)/2 (Rs.)
F Net Gain (E-D-C) (Rs.) - 6,283 7,325 4,230 2,950
In this case, the credit policy can be extended up to 42 days.

(b)
(i) As per the traditional approach, optimum capital structure exists when the weighted average cost of capital is
minimum. The weighted average cost of capital calculations at book value weights are as follows:

ke (1) We (2) kd (3) Wd (4) keWe (5) kdWd (6) k0 (7)=(5)+(6)


0.100 1.0 - - 0.1000 - 0.1000
0.105 0.9 0.040 0.1 0.0945 0.0040 0.0985
0.110 0.8 0.040 0.2 0.0880 0.0080 0.0960
0.116 0.7 0.045 0.3 0.0812 0.0135 0.0947
0.124 0.6 0.050 0.4 0.0744 0.0200 0.0944
0.135 0.5 0.055 0.5 0.0675 0.0275 0.0950
0.160 0.4 0.060 0.6 0.0640 0.0360 0.1000
0.200 0.3 0,080 0.7 0.0600 0.0560 0.1160

The company should employ debt of Rs.4,00,000 as the weighted average cost of capital is minimum at this level of
debt.

(ii) According to M-M approach, the cost of capital is a constant, and the cost of equity increases linearly with debt. The
equilibrium cost of capital is assumed to be equal to pure equity capitalization rate, which is 10% in the present
problem. The equity capitalization rate is given by the formula:
ke = k0+ (k0-kd) x Debt/ Equity

The equity capitalization rates would be:


Debt (Rs.) kd k0 (k0-kd) Debt/Equity ke ke Percentage
0 - 0.10 (0.10-0.000) - 0.1000 10.00
1,00,000 0.040 0.10 (0.10-0.040) 1,00,000/9,00,000 0.1067 10.67
2,00,000 0.040 0.10 (0.10-0.040) 2,00,000/8,00,000 0.1150 11.50
3,00,000 0.045 0.10 (0.10-0.045) 3,00,000/7,00,000 0.1236 12.36
4,00,000 0.050 0.10 (0.10-0.050) 4,00,000/6,00,000 0.1333 13.33
5,00,000 0.055 0.10 (0.10-0.055) 5,00,000/5,00,000 0.1450 14.50
6,00,000 0.060 0.10 (0.10-0.060) 6,00,000/4,00,000 0.1600 16.00
7,00,000 0.080 0.10 (0.10-0.080) 7,00,000/3,00,000 0.1467 14.67

5.
a) Mohan has just own a lottery and has three award options to choose from:
1. To receive a lump sum payment today of Rs. 61 million, or
2. To receive 10 annual end of year payment of Rs. 9.5 million, or
3. To receive 30 annual end of year payment of Rs. 5.5 million.
He expects to earn 8% annual return on his investment.
Required:
Recommend the best option for him. 5

The Institute of Chartered Accountants of Nepal


8 of 62
Suggested Answers – Financial Management
CAP II Examination – December 2012
b) PQR Limited deals in mines and geological survey. It is considering the following investment proposals for
mining:
Project Cash Flows
Year 0 Year 1 Year 2 Year 3
A -10,000 +10,000 Nil Nil
B -10,000 +7,500 +7,500 Nil
C -10,000 +2,000 +4,000 +12,000
D -10,000 +10,000 +3,000 +3,000
Assume Discount Rate of 10 per cent.
Required: (8+2=10)
i) Rank the projects according to each of the following methods:
Internal Rate of Return (IRR)
Net Present Value (NPV)
ii) Assuming the projects are independent, which one should be accepted? If the projects are mutually exclusive,
which project is the best?

Answer:
a) Calculation of present value of each option
1. Lump sum payment of Rs. 61 million; PV = Rs. 61 million
2. 10 annual end of year payment of Rs. 9.5 million;
PV= PMT x (PVIFA 8%, 10 years)
= Rs. 9.5 Million x 6.7101
= Rs. 63.75 Million
3. 30 annual end of year payment of Rs. 5.5 million;
PV= PMT x (PVIFA 8%, 30 years)
= Rs. 5.5 million x 11.2578
= Rs. 61.92 million
Since, option 2 provides highest present value, he should choose to receive 10 year annuity plan.

b) i) Cash Flows of Projects


Year A B C D
0 (10,000) (10,000) (10,000) (10,000)
1 10,000 7,500 2,000 10,000
2 - 7,500 4,000 3,000
3 - - 12,000 3,000

Discounted Cash Flows of Projects:


Year DF at 10% A B C D
0 1 (10,000) (10,000) (10,000) (10,000)
1 0.909 9,090 6,818 1,818 9,090
2 0.826 - 6,195 3,304 2,478
3 0.751 - - 9,012 2,253
NPV (910) 3,013 4,134 3,821
Rank IV III I II

IRR calculations:

Project A: The net cash proceeds in year 1 is just equal to the initial investment, therefore, IRR = 0%

For Project B, C & D Trying for NPV at DF of 40%


Year DF at 40% DCF of B DCF of C DCF of D
0 1 (10,000) (10,000) (10,000)
1 0.714 5,355 1,428 7,140
2 0.510 3,825 2,040 1,530
3 0.364 - 4,368 1,092
NPV (820) (2,164) (238)

IRR (B) = LR + × (HR – LR)

=10 + (40-10)

=10+23.58
= 33.58% (Rank =II)

The Institute of Chartered Accountants of Nepal


9 of 62
Suggested Answers – Financial Management
CAP II Examination – December 2012
IRR (C) =10 + (40-10)

=29.69% (Rank= III)

IRR (D) =10 + (40-10)

=35.83% (Rank=I)

ii) Between, the two time-adjusted (Discounted cash flow) investment criteria, NPV and IRR, NPV gives the
consistent results. If the projects are independent, either IRR or NPV method can be used since the same set of
projects will be accepted by any of the methods. In the present case, except Project A all the three projects should be
accepted if the discount rate is 10%. Under the assumption of 10% discount rate and mutually exclusive projects,
rankings according to IRR and NPV conflicts (except for Project A). If we follow the IRR rule, Project D should be
accepted. But the NPV rule says that Project C is the best.
Since the NPV rule gives consistent results in conformity with the wealth maximization principle, we would
therefore accept Project C following the NPV rule.

6. Write short notes on: (4×2.5=10)


a) Annuities and annuities due
b) Information asymmetry
c) Over the counter (OTC) market
d) The risk-return trade off

Answer:
a) Annuities and Annuities Due:
The term annuity refers to any terminating stream of fixed payments over a specified period of time. This usage is
most commonly seen in discussions of finance, usually in connection with the valuation of the stream of
payments, taking into account time value of money concepts, such as interest rate and future value.
Examples of annuities are regular deposits to a savings account, monthly home mortgage payments, and monthly
insurance payments. Annuities are classified by the frequency of payment dates. The payments (deposits) may be
made weekly, monthly, quarterly, yearly, or at any other interval of time.
An annuity-due is an annuity whose payments are made at the beginning of each period. Deposits in savings, rent
or lease payments, and insurance premiums are examples of annuities due.

b) Information Asymmetry
Information asymmetry deals with the study of decisions in transactions where one party has more or better
information than the other. This creates an imbalance of power in transactions which can sometimes cause the
transactions to go awry, a kind of market failure in the worst case. Examples of this problem are adverse selection,
moral hazard, and information monopoly. Most commonly, information asymmetries are studied in the context of
principal-agent problems. Information asymmetry causes misinforming and is essential in every communication
process.
Information asymmetry models assume that at least one party to a transaction has relevant information whereas the
other(s) do not. Some asymmetric information models can also be used in situations where at least one party can
enforce, or effectively retaliate for breaches of, certain parts of an agreement whereas the other(s) cannot.

c) Over the counter (OTC) market


Over-the-counter market functions as part of the secondary market for stocks and bonds not listed on a stock
exchange. The market is composed of brokers and dealers who stand ready to buy and sell securities at quoted
prices. Most corporate bonds and a growing number of stocks are traded over-the-counter instead of being traded on
an organized exchange.
The OTC market these days has become highly mechanized with market participants linked together by a
telecommunication network. Unlike the organized exchange, the participants of a OTC market do not come together
in a single place. A network is maintained and price quotations are made instantaneously. In the past, most
companies preferred to list their shares on major exchanges as a matter of prestige and necessity. This has undergone
change due to the rapid development in the field of electronics.

d) The Risk- Return Trade Off


This principle steps that potential return rises with an increase in risk. Low levels of uncertainty (low-risk) are
associated with low potential returns, whereas high levels of uncertainty (high-risk) are associated with high
potential returns. According to the risk-return tradeoff, invested money can render higher profits only if it is subject
to the possibility of being lost.
Because of the risk-return trade off, you must be aware of your personal risk tolerance when choosing
investments for your portfolio. Taking on some risk is the price of achieving returns; therefore, if you want to make
money, you can't cut out all risk. The goal instead is to find an appropriate balance - one that generates some profit,
but still allows you to sleep at night.
The Institute of Chartered Accountants of Nepal
10 of 62
Suggested Answers – Financial Management
CAP II Examination – December 2012

7. Distinguish between: (4×2.5=10)


a) Capital planning and Capital rationing
b) Treasury bills and Certificate of deposit
c) Permanent working capital and Variable working capital
d) Capital Assets Pricing Model and Arbitrage Pricing Model

Answer:
a) Capital Planning Vs. Capital Rationing
A proper plan for a company's capital expenditures is called Capital Planning. Capital expenditures are payments
made over a period of more than one year. They are used to acquire assets or improve the useful life of existing
assets; an example of a capital expenditure is the funding to construct a factory. Making a capital budget must
account for the potential profitability of the plans involved. Calculating the net present value or the internal rate of
return are two methods for determining a capital budget.
The act of placing restrictions on the amount of new investments or projects undertaken by a company is called
Capital Rationing. This is accomplished by imposing a higher cost of capital for investment consideration or by
setting a ceiling on the specific sections of the budget. Companies may want to implement capital rationing in
situations where past returns of investment were lower than expected.

b) Treasury bills and Certificate of deposits


Treasury bills are sold by the government on a discount basis. As a result, the investor does not get actual payment
of interest on the Treasury bills. The return to the investor is in the form of difference between the purchase price
and face (or par) value of the bill.
These bills are issued without the investor‟s name upon them, i.e. in the bearer form. This feature of the treasury
bills makes them easily transferable from one investor to another. The secondary market also exists for these bills
making them highly liquid. It is also considered risk-free since it has the backing and guarantee of the government.
Certificate of deposits (CDs) are marketable receipts for funds that have been deposited in a bank/financial
institution for a specified period of time. The funds thus deposited earn a fixed rate of interest. The denomination
and the maturities is agreed upon as per the needs and wishes of the investor.
Since the CDs are not sold at discount, the investor receives the amount deposited plus the interest earned thereon. A
secondary market also exists for the CDs. These may be issued in the registered or bearer form. The second types of
CDs are most common and popular due to their easy transferability and liquidity.

c) Permanent working capital Vs. Variable working capital


Permanent working capital is the minimum amount of gross working capital which is always maintained in spite of
the increase or decrease in the sales during the year. It comprises of the minimum cash balance, minimum inventory
level etc. If a firm does not face a seasonal cycle then it will have only a permanent working capital requirement.
Variable working capital is the amount of gross working capital that exceeds the amount of permanent working
capital at any time during the year. It is also important for the finance manager to decide sources for financing
seasonal current assets. The variable working capital is the result of the periodic fluctuations of the gross working
capital. For example, wheat mill may have higher inventories at the time of harvesting wheat. It causes the increase
in gross working capital.

d) Capital Assets Pricing Model Vs. Arbitrage Pricing Model


The capital asset pricing model (CAPM) states that the return on a stock depends on whether the stock's price
follows prices in the market as a whole. CAPM is useful because it is a statistical representation of past risk. Even
though past performance is no guarantee for future success there is a higher probability that a consistent past
performer will continue to do well over a new untested entry in the market.
Arbitrage pricing model (APM) holds that the expected return of a financial asset is largely based on its "beta".
Beta is the measure of the relationship between company related factors which influence financial performance and
the overall market in which the latter competes. Typically a company which has a beta of one will reflect the
market whereas a beta score of 0.75 means that a company will move up or down to the extent of 75 per cent of the
corresponding market movement.

The Institute of Chartered Accountants of Nepal


11 of 62
The Institute of Chartered Accountants of Nepal
Suggested Answers of Cost and Management Accounting

CAP II Examination- December 2012

1.
a) Rajat Chemicals Ltd., a company within the chemical industry, mixes powdered ingredients in two different
processes to produce one product. The output of Process 1 becomes the input of Process 2 and the output of Process 2 is
transferred to the packing department.
Following are the information pertaining to the week ended 11 th June 2012;
Process 1
Input:
Material A: 6,000 kilograms at Rs. 1 per kilogram
Material B: 4,000 kilograms at Rs. 2 per kilogram
Mixing Labour: 430 hours at Rs. 4 per hour
Normal Loss: 5% of weight input, disposed off at 32 paisa per kilogram
Output: 9,200 kilograms
There is no work-in-process at the beginning or end of the week.

Process 2
Input:
Material C: 6,600 kilograms at Rs. 2.50 per kilogram
Material D: 4,200 kilograms at Rs. 1.50 per kilogram
Flavoring Essence: Rs. 600
Mixing Labour: 370 hours at Rs. 4 per hour
Normal Waste: 5% of weight input with no disposal value
Output: 18,000 kilograms.
There is no work-in-process at the beginning of the week but 1,000 kilograms are in process at the end of the week and
estimated to be only 50% complete so far as labour and overhead were concerned.

Overhead of Rs. 6,400 incurred by the two processes is to be absorbed on the basis of mixing labour hours.

Required: (9+4+3=16)
i) Prepare Process 1 and 2 Account.
ii) Prepare Abnormal Loss Account and Packing Department Account.
iii) Calculate equivalent units produced.

b) Explain Economic Ordering Quantity. 4

Answer:
a)
i.
Dr. Process 1 Cr.
Per Kg. Per Kg.
Particulars Kg Rs. Rs. Particulars Kg Rs. Rs.

To Material A 6,000 1.00 6,000 By Normal Loss 500 0.32 160


By Abnormal
To Material B 4,000 2.00 8,000 Loss (W.N. 2) 300 2.00 600
To Mixing Labor (430
hours @ Rs.4) 1,720 By Transfer to
To Overhead(W.N. 1) 3,440 Process 2 9,200 2 18,400
Total 10,000 19,160 Total 10,000 19,160
Dr. Process 2 Cr.
Per Kg. Per Kg.
Particulars Kg Rs. Rs. Particulars Kg Rs. Rs.
To Process 1 9,200 2.00 18,400 By Normal Loss 1,000 - -
To Material C 6,600 2.50 16,500
To Material D 4,200 1.50 6,300 By Work-in-
To Flavoring Essence 600 progress (W.N.3) 1,000 - 2,320
To Mixing Labour (370
hours @ Rs.4) 1,480 By Packing
To Overhead (W.N.1) 2,960 Department 18,000 2.44 43,920
Total 20,000 46,240 Total 20,000 46,240
Suggested Answers – Cost and Management Accounting
CAP II Examination – December 2012
ii.
Abnormal Loss Account
Dr. Cr.
Per Kg. Per Kg.
Particulars Kg Rs. Rs. Particulars Kg Rs. Rs.
By Normal Loss 300 0.32 96

To Process 1 300 2.00 600 By Balance to P/L A/C 504


Total 300 600 Total 300 600

Packing Department Account


Dr. Cr.
Per Kg. Per Kg.
Particulars Kg Rs. Rs. Particulars Kg Rs. Rs.
To Process 2 18,000 2.44 43,920 By Balance C/D 18,000 2.44 43,920
Total 18,000 43,920 Total 18,000 43,920

iii. Statement of equivalent unit

Equivalent Units
Particulars Output Unit Material Labour Overhead
Completed 18,000 18,000 18,000 18,000
WIP (100% Material, 50%
Labour and Overhead) 1,000 1,000 500 500
Labour and Overhead - - - -
Normal Waste 1,000 - - -
Total 20,000 19,000 18,500 18,500

Working Notes:
1. Total overhead expenses : Rs. 6,400
Total labour hours in Process 1 and 2 = 800
Overhead absorption rate = Rs. 6,400/800 hours = Rs. 8 per labour hour
Overhead under Process 1 = 430 × Rs. 8 = Rs. 3,440
Overhead under Process 2 = 370 × Rs. 8 = Rs. 2,960

2. Cost of 9,500 Kg. of output is = ( Rs.19,160 – Rs. 160) i.e., Rs. 19,000
Hence cost per kg. of output is Re. 2.00

3. Cost per Equivalent Unit and WIP


Material = Rs. 41,800 / 19,000 = Rs. 2.20
Labour = Rs. 1,480 / 18,500 = Rs. 0.08
Overhead = Rs. 2,960 / 18,500 = Rs. 0.16

W.I.P.
Material = 1,000 × Rs. 2.20 = Rs. 2,200
Labour = 500 × 0.08 P = Rs. 40
Overhead = 500 × 0.16 P = Rs. 80
Rs. 2,320

b) Economic Order Quantity (EOQ): Purchase department in manufacturing concerns is usually faced with the problem
of deciding the „quantity of various items‟ which they should purchase. If purchases of material are made in bulk then
inventory carrying cost will be high. On the other hand if order size is small each time, then the ordering cost will be high.
In order to minimise ordering and carrying costs it is necessary to determine the order quantity which minimises these two
costs. The size of the order for which both ordering and carrying cost are minimum is known as economic order quantity.

Calculation of EOQ made as follows:


EOQ = √2AO/C
Where;
A= Annual requirement
O= Ordering cost per order
C= Carrying cost per unit per annum

At EOQ level total cost of material including ordering and holding cost will be minimal.

The Institute of Chartered Accountants of Nepal


13 of 62
Suggested Answers – Cost and Management Accounting
CAP II Examination – December 2012
2.
a) The Acme shelving Co. Ltd. manufactures shelving brackets in batches of 300. During May, Batch No. 23 was
machined at a rate of 15 per hour. Sixty of the brackets failed to pass inspection, but of these, 40 were thought to be
rectifiable. The remaining 20 were scrapped, and the scrap value was credited to the batch cost account. Rectification
work took nine hours. The following details are available for Batch No. 23:
Rs.
Raw materials per bracket 160
Scrap value per bracket 86
Machinists' hourly rate 420
Machine hour overhead rate (running time only) 360
Setting up of machine:
Normal machining 2,100
Rectification 1,800
Required: (4+6+2=12)
i) Calculate the cost of Batch No. 23 in total and per unit, if all units pass inspection.
ii) Calculate the actual cost of Batch No. 23 in total and per unit, after crediting the recovery value of the scrapped
components, and including the rectification costs.
iii) Calculate the loss incurred because of defective work.

b) In a manufacturing concern 20 workmen work in a group. The concern follows a group incentive bonus system
whereby each workman belonging to the group is paid a bonus on the excess output over the hourly production standard
of 250 pieces, in addition to his normal wages at hourly rate. The excess of production over the standard is expressed as a
percentage and two-thirds of this percentage is considered to be the share of the workman and is applied on the notional
hourly rate of Rs. 60 (considered only for the purpose of computation of bonus). The output data for a week are stated
below:

Days Man hours worked Output in pieces


Monday 160 48,000
Tuesday 172 53,000
Wednesday 164 40,000
Thursday 168 52,000
Friday 160 46,000
Saturday 160 42,000
Total 984 281,000

Required: (5+3=8)
i) Workout the amount of bonus for the week and the average rate at which each workman is to be paid.
ii) Compute the total wages including bonus payable to Ram who worked for 48 hours at an hourly rate of Rs. 25
and to Shyam who worked for 52 hours at an hourly rate of Rs. 30.

Answer:
a)
i. Calculation of the cost of Batch No. 23 in total and per unit, if all units pass inspection:
Batch No. 23
Particulars Rs.
Raw materials (300 X Rs.160) 48,000
Direct Labour:
Machinists' Cost (300/ 15 X Rs.420) 8,400
Setting up of machine:
Normal machining 2,100
Overhead (300/ 15 X Rs.360) 7,200
Total Cost 65,700
Per unit cost = Rs. 65,700/ 300 = Rs.219

ii. Calculation of the actual cost of Batch No. 23 in total and per unit, after crediting the recovery value of the scrapped
components, and including the rectification costs
Batch No. 23
Particulars Rs. Rs.
Raw materials (300 X Rs.160) 48,000
Less: Recovery value of scrap (20 X Rs.86) 1,720
46,280
Direct Labour:
Normal- Machinists' Cost (300/ 15 X Rs.420) 8,400
Rectification (9 hours X Rs.420) 3,780
12,180

The Institute of Chartered Accountants of Nepal


14 of 62
Suggested Answers – Cost and Management Accounting
CAP II Examination – December 2012
Setting up of machine:
Normal machining 2,100
Rectification 1,800
3,900
Overhead:
Normal (300/ 15 X Rs.360) 7,200
Rectification (9 hours X Rs.360) 3,240
10,440
Total Cost 72,800
Per unit cost = Rs. 72,800/ 280 = Rs.260

iii. Calculation of the loss incurred because of defective work


Batch No. 23
Particulars Rs.
Loss because of additional costs (Rs.72,800- Rs.65,700) 7,100
Loss because of faulty products (Rs.219 X 20 units) 4,380
Total loss incurred because of defective work 11,480

b)
(i)Computation of group incentive bonus
Man hours Standard output Actual output Excess % of excess 2/3% of excess Bonus (Rs.)
Days
worked (pcs) (W.N.1) (pcs) output (pcs) output (W.N.2) output for bonus (W.N. 3)
Monday 160 40,000 48,000 8,000 20.00 13.33 1,279.68
Tuesday 172 43,000 53,000 10,000 23.26 15.51 1,600.63
Wednesday 164 41,000 40,000 - - - -
Thursday 168 42,000 52,000 10,000 23.81 15.87 1,599.70
Friday 160 40,000 46,000 6,000 15.00 10.00 960.00
Saturday 160 40,000 42,000 2,000 5.00 3.33 319.68
984 5,759.69

Share of each individual workman: Rs. 5,759.69 / 984 = Rs. 5.85 per hour worked.

(ii)
Computation of wages to individual workmen
Ram Shyam
(a) Hours worked 48 52
(b) Hourly rate of payment Rs. 25 30
(c) Total wages at hourly rate (a) × (b) Rs. 1,200.00 1,560.00
(d) Incentive bonus (a) × Rs. 5.85 Rs. 280.80 304.20
(e) Total wages payable (c) + (d) Rs. 1,480.80 1,864.20

Working notes:
(1) Standard output for Monday (160 × 250) = 40,000 pieces and so on for other days.

(2) % of excess output for Monday (8,000 / 40,000 × 100) = 20% and so on for other days.

(3) Bonus for Monday (160 × 60 × 13.33%) = Rs. 1,279.68 and so on for other days.

3.
a) ABC Ltd. manufactures a single product and absorbs the production overheads at a pre-determined rate of Rs. 10 per
machine hour.
At the end of the financial year 2011-12, it has been found that actual production overheads incurred were Rs. 600,000. It
included Rs. 45,000 on account of written off obsolete stores and Rs. 30,000 being the wages paid for the strike period.
The production and sales data for the year 2011-12 is as under:
Production:
Finished goods 20,000 units
Work-in-progress (50% complete in all respect) 8,000 units
Sales:
Finished goods 18,000 units.
The actual machine hours worked during the period were 48,000 hours. It has been found that one-third of the under-
absorption of production overheads was due to lack of production planning and the rest was attributable to normal
increase in costs.

Required: (3+4+3=10)

The Institute of Chartered Accountants of Nepal


15 of 62
Suggested Answers – Cost and Management Accounting
CAP II Examination – December 2012
i) Calculate the amount of under-absorption of production overheads during the year 2011-12 and;
ii) Show the accounting treatment of under-absorption of production overheads
iii) Calculate the apportionment of under-absorption overheads over WIP, finished goods and cost of sales.

b) Deluxe Ltd. undertook a contract for Rs. 500,000 as on 1st July 2011. On 30th June 2012, when the accounts were
closed, the following details about the contract were gathered.
Rs.
Materials purchased 100,000
Wages paid 45,000
General expenses 10,000
Plant purchased 50,000
Materials on hand on 30th June 2012 25,000
Wages accrued on 30th June 2012 5,000
Work certified 200,000
Cash received 150,000
Work uncertified 15,000
Depreciation of plant 5,000

The above contract contained an escalation clause which read as follows.


In the event of materials and rates of wages increase by more than 5% the contract price would be increased accordingly
by 25% of the rise in the cost of materials and wages beyond 5% in each case.
It was found that since the date of signing the agreement, the prices of materials and wage rates increased by 25%. The
value of work certified does not take into account the effects of the above clause.
Prepare Contract Account. 10

Answer:
a) (i) Amount of under-absorption of production overheads during the year 2011-12:

Particulars Rs. Rs.


Total production overheads actually incurred 600,000
Less: Obsolete stores written off 45,000
Wages paid for strike period 30,000 75,000
Net production overheads actually incurred 525,000
Production overheads actually absorbed by 48,000 machine hours @ Rs. 10 480,000
Amount of under-absorption of production overheads 45,000

(ii) Accounting treatment of under absorption of production overheads:


It is given in the question that one third of the under absorbed overheads were due to lack of production planning and the
rest were attributable to normal increase in costs. So accounting treatment of under absorbed overheads will be as follows:
1. Under absorbed overheads due to lack of production planning being abnormal should be debited to the Profit &
Loss Account. Therefore, the amount to be debited to Profit & Loss Account is Rs. 45,000 × 1 / 3 = Rs. 15,000.
2. Under absorbed overheads attributable to normal increase in costs should be distributed over work-in-progress,
finished goods and cost of sales by using supplementary rate. The amount to be so distributed is Rs. 45,000 × 2 / 3 = Rs.
30,000.
(iii) Similarly as per question 20,000 units were completely finished and 8,000 units were 50% complete, apportionment
of unabsorbed overheads over work-in-progress, finished goods and cost of sales will be as follows:
Particulars Equivalent completed units Supplementary rate Rs.
Work-in-progress 4,000 1.25 5,000
Finished goods 2,000 (20,000-8000) 1.25 2,500
Cost of sales 18,000 1.25 22,500
24,000 30,000
Working notes:
Rs. 30,000
Supplementary rate per unit   Rs. 1.25
24,000
b)
Contract A/c for the Year Ended 30th June 2012
Dr. Cr.
Particulars Rs. Particulars Rs.
To Materials 1,00,000 By Work-in-progress A/c:
To Wages paid 45, 000 Work certified 2,00, 000
To Wages outstanding 5, 000 Work uncertified 15, 000
To General expenses 10, 000 Materials on hand 25, 000
To Depreciation of Plant 5, 000 Contract escalation * 5, 000
To Balance c/d – notional profit 80, 000
The Institute of Chartered Accountants of Nepal
16 of 62
Suggested Answers – Cost and Management Accounting
CAP II Examination – December 2012
Total 2,45, 000 Total 2,45, 000

To Profit and Loss A/c # 20, 000 By Balance b/d 80, 000
To Transfer to Reserve 60, 000
Total 80, 000 Total 80, 000

* Escalation:
Materials /wages increased by 25%
[a] Increase in material price [Rs.100000 – Rs.25000] 25/125 = Rs.15,000
[b] Increase in wages Rs.50,000 x 25/125 = Rs.10,000
Total Increase = [a] + [b] = Rs.25,000

This increase is 5% of the contract price, hence escalation clause apply. Escalation is 25% of the rise in the cost of
materials and wages beyond 5% in each case.

 Escalation = (25%-5%)25000=5,000

# Amount of profit to be credited to Profit and Loss A/c: As the contract is less than 50% complete, the following formula
will have to be used for computing the amount of profit to be credited to the Profit and Loss A/c:-

= 1/3 ×(Cash Received/Work Certified)  Notional Profit


= 1/3 × (Rs.1, 50, 000/2,00,000)  Rs.80,000
= Rs.20,000

4.
a) Metalica Trading Ltd. makes and sells a single product. The company‟s trading results for the year 2010 are as
follows :
Rs. ‟000
Sales 3,000
Direct materials 900
Direct labour 600
Overheads 900
Total cost 2,400
Profit 600

For the year 2011, the following are expected:


i) Reduction in the selling price by 10%
ii) Increasing in the quantity sold by 50%
iii) Inflation of direct material cost by 8%
iv) Price inflation in variable overhead by 6%
v) Reduction of fixed overhead expenses by 25%.

It is also known that;


i) In 2009, overhead expenditure totaled to Rs. 800,000.
ii) Total overhead cost inflation for 2010 has been 5% more than in 2009.
iii) Production and sales volumes have been 25% higher in 2010 than in 2009.

Required: (5+3+3=11)
i) Prepare a statement showing the estimated trading results for 2011.
ii) Calculate the break-even point for 2010 and 2011.
iii) Comment on the BEP and profits of the 2010 and 2011.

b) The following information is the extracted from the financial accounts of a manufacturing company for the last
financial year:
Rs.'000
Raw material consumed 5,000
Direct wages 3,000
Works overhead 1,600
Office overhead 700
Selling overhead 960
Bad debts 120
Legal charges 10
Interest received 120
Sales (120,000 units @ Rs.100) 12,000

The Institute of Chartered Accountants of Nepal


17 of 62
Suggested Answers – Cost and Management Accounting
CAP II Examination – December 2012
Closing inventory of WIP 240
Closing inventory of Finished goods (4,000 units) 320

The following information is extracted from the cost accounts for the same financial year:
Raw material consumed- Rs. 5,600,000
Recovery of works overhead- @ 20% on prime cost
Recovery of office overhead- @ Rs. 6 per unit of output
Recovery of selling overhead- @ Rs. 8 per unit sold.

Required: (6+3=9)
i) Prepare financial profit and loss account and Cost sheet for the financial year.
ii) Reconcile the difference in profit under the two sets of accounts.

Answer:
a) (i)
Statement showing trading results

Particulars 2010 2011


A. Sales: 3,000 4,050 (3,000 × 150% × 90%)
B. Less: Variable Costs: Direct material 900 1,458 (900 × 150% × 108%)
Direct labour 600 900 (600 × 150%)
Variable overhead (W.N.1) 300 477 (300 × 150% × 106%)
Total variable cost 1,800 2,835
C. Contribution [A – B] 1,200 1,215
D. Less: Fixed overheads 600 450 (600 × 0.75)
E. Profit [C – D] 600 765

(ii)
P/V Ratio = Contribution /Sales *100 = 1200/3000*100 1215/4050*100
= 40% = 30%
BEP = Fixed Cost / P/V Ratio = 600/40% = 450/30%
= Rs.1500 = Rs. 1500
(iii)
Particulars 2010 2011 % change
BEP 1500 1500 No Change
Fixed overheads 600 450 150/650*100 = 25%
P/V Ratio 40% 30% 10%/40%*100= 25%
Profit 600 765 -165/600*100=- 27.5%

Both fixed cost and P/V ratio have declined by 25% equally. So, BEP sales remain the same. The contribution is only Rs.
1,215 in 2011 though quantity is increased by 50%. This is due to increase in production cost and decrease in selling
price. This is more than made up by decrease in fixed cost so that overall profit has increased by 27.5%.

Working notes:
1. Calculation of variable overheads and fixed overheads
Total overheads for same production in 2010 = 800 × 105% = 840
Variable overheads for 2010 = (900-840)/ (125-100) *125 = 300
Fixed overheads for 2010 = 900 – 300 = 600

b) Solution:
i.
Financial Profit & Loss Account
Particulars Rs.'000 Particulars Rs.'000
To Raw material consumed 5,000 By Sales 12,000
To Direct wages 3,000 By Closing inventory:
To Works overhead 1,600 WIP 240
To Office overhead 700 Finished goods 320
To Selling overhead 960 By Interest received 120
To Bad debts 120
To Legal charges 10
To Net profit 1,290
12,680 12,680

Cost Sheet
For the financial year
The Institute of Chartered Accountants of Nepal
18 of 62
Suggested Answers – Cost and Management Accounting
CAP II Examination – December 2012
Particulars Rs.'000 Rs.'000
Raw material consumed 5,600
Direct wages 3,000
Prime Cost 8,600
Works overhead (20% on Prime cost) 1,720
10320
Less: Closing WIP (240)
Works Cost 10,080
Office overhead (Rs.6 *124,000) 744
Cost of production 10,824
Less: Closing Finished goods (10,824*4000/124000) (349)
Cost of goods sold 10,475
Selling overhead (Rs.8 * 120,000) 960
Cost of sales 11,435
Net profit 565
Sales 12,000
Note: Unit produced = Sold unit + closing inventory

ii.
Reconciliation Statement
Particulars Rs.'000 Rs.'000
Profit as per financial accounts 1,290
Add:
Closing inventory of finished goods overvalued in cost accounts 29
Bad debts not charged in cost accounts 120
Legal charges not charged in cost accounts 10 159
1449
Less:
Raw material overcharged in cost accounts 600
Works overhead over absorbed in cost accounts 120
Office overhead over absorbed in cost accounts 44
Interest received not included in cost accounts 120 884
Profit as per cost accounts 565

5. Distinguish between: (4×2.5 =10)


a) Efficiency Audit and Propriety Audit
b) Cost Control and Cost Reduction
c) Controllable Cost and Uncontrollable Cost
d) Running Charges and Maintenance Charges

Answer:
a) Cost audit, apart from having all the normal ingredients of audit, i.e. vouching, verification etc., has within its
domain elements of efficiency audit and propriety audit. Efficiency audit is directed towards the measurement of whether
corporate plans have been effectively executed. It is concerned with the utilisation of the resources in economical and
most remunerative manner to achieve the objectives of the concerns. It comprises of studying the plans of the
organization, comparing actual performance with plans and investigating the reasons for variances to take remedial action.
For example, the effective utilization of capital in an organization can be gauged by determining the return on capital
employed.
On the other hand propriety audit is concerned with the executive actions and plans bearing on the finances and
expenditure of the company. The cost auditor has to judge:
1. Whether the planned expenditure is designed to give optimum results;
2. Whether the size and channels of expenditure were designed to produce the best results; and
3. Whether the return from expenditure on capital as well as current operations could be bettered by some other
alternative plan of action.

b) Cost control is operated through setting standards of targets and comparing actual performance therewith with a
view to identifying deviations from standards or norms and taking corrective action in order to ensure that future
performance conforms to standards or norms. It is a preventive action.

Cost reduction, on the other hand, is a continuous process of critical cost examination, analysis and challenge of
standards. It is a corrective action. Each aspect of the business viz. products, process, procedures, methods, organization,
personnel, etc is critically examined and reviewed with a view to improving the efficiency and effectiveness and reducing
costs. Even in an organization where efficient cost control is in operation, there is always room for cost reduction. In cost
reduction there is a permanent reduction in per unit cost where such permanency is lacking in cost control.
The Institute of Chartered Accountants of Nepal
19 of 62
Suggested Answers – Cost and Management Accounting
CAP II Examination – December 2012

Cost control lacks the dynamic approach which planned cost reduction demands. In a cost reduction plan, standards which
are the basis of cost control are constantly challenged for improvement.

Budgetary control and standard costing are essential tools and techniques of cost control. There are several distinct tools
and techniques of cost reduction such as value engineering, work study, standardization, simplification, variety reduction,
quality measurement and research, operations research, market research, job evaluation, merit awards, incentives,
improvement in design, mechanization, automation etc.

c) Controllable cost and uncontrollable cost


Controllable cost Uncontrollable cost

i. Controllable cost are the costs which can be influenced i. Uncontrollable cost are the costs which can not be
by action influenced by action
ii.
ii. Controllable cost are influenced by specified member ofiii. Uncontrollable cost are not influenced by specified
an undertaking member of an undertaking
iii. iv.
iv. Can be influenced by the action of executive of each v. Can not be influenced by the action of executive of each
responsibility centers responsibility but could avoid with initial decision
vi.
v. vii. Can not be minimized for cost control purpose
Can be minimized for cost control purpose viii.
vi. ix. Generally Fixed cost are in the nature of uncontrollable
vii. Generally variable cost and semi variable cost are in the cost
nature of controllable cost

d) Running charges and maintenance charges both are the parts of total costs in operating costing. These are used to
prepare a cost statement in an industry where services are rendered and goods are not produced.

Running charges are variable costs relating to rendering of services, where as maintenance charges are generally in the
nature of semi-variable or semi-fixed costs. The items to be included in running charges and maintenance charges depend
on the nature of the operation. For example, cost of fuel in transport sector, cost of food items in restaurant are running
charges; besides the cost of repairs and spares in transport sector and the cost of lighting, consumable stores etc. in hotels
are maintenance charges. To find out per unit cost of services rendered, the costs are required to be segregated into
variable and fixed costs. Therefore, the maintenance charges require to be segregated into variable and fixed costs

6. Answer the following questions: (4×2.5=10)

a) Briefly explain the circumstances that warrant the need for preparation of Flexible budget.
b) Discuss ABC analysis as a technique of inventory control.
c) Explain the role of uniform costing in inter-firm comparison.
d) What do you mean by cost driver? Explain with example.

Answer:
a) A 'Flexible Budget' is defined as 'a budget which, by recognizing the difference between fixed, semi-variable and
variable costs is designed to change in relation to the level of activity attained'. In a fixed budgetary control, budgets are
prepared for one level of activity whereas in a flexible budgetary control system, a series of budgets are prepared for a
number of alternative production levels or volumes. Flexible budgets represent the amount of expenses that is reasonably
necessary to achieve each level of output specified. In other words, the allowances given under flexible budgetary control
system serve as standards of what costs should be at each level of output.

The need for the preparation of the flexible budgets arises in the following circumstances:
(i) Seasonal fluctuations in sales and/or production, for example in soft drink industry;
(ii) A company which keeps on introducing new products or makes changes in the design of its products frequently;
(iii) Industries engaged in make-to- order business like ship building;
(iv) An industry which is influenced by changes in fashion; and
(v) General changes in sales

b) It is a system of inventory control. It exercises discriminating control over different items of stores classified on
the basis of investment involved. Usually they are divided into three categories according to their importance, namely,
their value and frequency of replenishment during a period.

„A‟ category of items consists of only a small percentage i.e. about 10% of total items handles by the stores but require
heavy investment about 70% of inventory value, because of their high price or heavy requirement or both.
The Institute of Chartered Accountants of Nepal
20 of 62
Suggested Answers – Cost and Management Accounting
CAP II Examination – December 2012
„B‟ category of items are relatively less important – 20% of the total items of material handled by stores and % of
investment required is about 20% of total investment in inventories.
„C‟ category – 70% of total items handled and 10% of value.

For „A‟ category items, stocks levels and EOQ are used and effective monitoring is done.
For „B‟ category same tools as in „A‟ category are applied.
For „C‟ category of items, there is no need of exercising constant control. Orders for items in this group may be placed
after 6 months or once in a year, after ascertaining consumption requirement.

c) When several undertakings in an industry, group and association use same costing principles and practices, it is
known as uniform costing. The application of uniform costing in an industry provides the means whereby relevant
information can be obtained and it helps in comparing the production efficiency of two firms at the time of merger.

Inter-firm comparison is the technique of evaluating the performance efficiencies, costs and profits of firms in an industry.
The application of uniform costing greatly facilitates inter-firm comparison by providing information relating to costs,
profits, prices, efficiency etc. Inter-firm comparison will not be possible if uniform costing is not in existence.

d) A cost driver is a variable, such as the level of activity or volume, which causally affects costs over a given time
span. That is, there is a cause-and-effect relationship between a change in the level of activity or volume and a change in
the level of total costs. Cost drivers signify factors, forces or events that determine the costs of activities. Costs drivers are
the links and they can link a pool of costs in an activity center to the product. For example, if product design costs change
with the number of parts in a product, the number of parts is a cost driver of product design costs. Similarly, miles driven
are often a cost driver of distribution costs.

The cost driver of a variable cost is the level of activity or volume whose change causes proportionate changes in the
variable cost. For example, the number of vehicles assembled is the cost driver of the total cost of steering wheels. If setup
workers are paid an hourly wage, the number of setup hours is the cost driver of total (variable) setup costs.

Costs that are fixed in the short run have no cost driver in the short run but may have a cost driver in the long run.
Consider the cost of testing quality of paints in a paint manufacturing company. These costs consists of testing department
equipment and staff costs that are difficult to change and, hence, are fixed in the short run with respect to changes in the
volume of production. In this case, volume of production is not a cost driver of testing costs in the short run. In the long
run, however, the paint manufacturing company will increase or decrease the testing department's equipment and staff to
the levels needed to support future production volumes. In the long run, volume of production is a cost driver of quality
testing costs.

The Institute of Chartered Accountants of Nepal


21 of 62
The Institute of Chartered Accountants of Nepal
Suggested Answers of Business Communication and Marketing

CAP II Examination- December 2012

Section -'A'

1. Read the following case carefully and answer the questions given below: 10
Imagine that you are the Secretary of the CA Association and the latter is planning to hire three Researchers to conduct some
research studies. You have been given the responsibility to conduct the final interview of the ten short-listed candidates.
While interviewing one of the candidates, the door was not closed properly and your mobile phone rang with a loud ring
tone. While making an attempt to turn it off you forgot the question you were about to ask. Also, as the door was open, you
could not hear properly what the candidate was speaking, as the talks among other candidates waiting in the lobby was too
loud. After the interview, the President of the organization called you for a meeting to discuss about the ten short-listed
candidates you interviewed to select the final three.

a) As an interviewer, you should have either switched off your mobile phone or kept it in silent mode. If you were the
candidate giving the interview, what things should you have done while being interviewed for employment?
b) Noise such as the mobile ring and talks among other candidates are some of the common barriers to communication.
What other barriers could affect the interpersonal communication between you and the candidate?
c) What do you understand by information and analytical report? In your meeting with the President, what information
will include in the report?

Answer:
a) Don‟t talk too much during the interview.
-Listen carefully to what the interviewer is asking and do not interrupt the interviewer in the middle of the conversation.
-Dress appropriately and be careful with the body language (sitting postures, use of hands during communication).
-Be confident – gather necessary information about the organization before interview

b) Barriers to the communication between interviewer and interviewee:


-The use of jargon, over-complicated or unfamiliar terms.
-Differences in perception and viewpoint.
-Language differences and the difficulty in understanding unfamiliar accents.
-Expectations and prejudices which may lead to false assumptions or stereotyping. People often want to hear what they
expect to hear rather than what is actually said and jump to incorrect conclusions.
-Cultural differences. lying between and among the interlocutors.
c)
-Informational reports: The reports, which contain functional information with regard to a specific business activity, data,
facts, feedback, and other types of information, are called informational reports.
-Analytical reports: The reports which contain interpretations, analysis and comments of the report writer are called analytical
report. These offer both information and analysis, and they can also include recommendations.
-Information to be included:
-Background of the interview, number of applicants, details of all short-listed candidates for final interview, their strengths
and weaknesses, the points each of them have scored, suggestions on who should be selected and why.

2. What are the most significant barriers to interpersonal communication? Use examples to elaborate your points
wherever possible. 10

Answer:
You can improve your chances of communicating successfully by learning to recognize barriers that are known to disrupt the
process. The most significant barriers for individuals are bypassing, differing frames of reference, lack of language skill, and
distractions.
Bypassing. One of the major barriers to disrupt communication involves words. Each of us attaches a little bundle of
meanings to every word, and these meanings are not always similar. Bypassing happens when people miss each other with
their meanings. Let‟s say your boss asks you to “help” with a large customer mailing. When you arrive to do your share, you
learn that you are expected to do the whole mailing yourself. You and your boss attached different meanings to the word help
. Bypassing can lead to major miscommunication because people assume that meanings are contained in words. Actually,
meanings are in people. For communication to be successful, the receiver and sender must attach the same symbolic
meanings to their words. A recent study revealed a high likelihood of miscommunication when people use common but
vague words such as probably, always, never, usually, often, soon, and right away. What do the words really mean?
Differing Frames of Reference. Another barrier to disrupt communication is your frame of reference . Everything you see
and feel in the world is translated through your individual frame of reference. Your unique frame is formed by a combination
of your experiences, education, culture, expectations, personality, and other elements. As a result, you bring your own biases
Suggested Answers – Business Communication and Marketing
CAP II Examination – December 2012
and expectations to any communication situation. Because your frame of reference is different from everyone else‟s, you will
never see things exactly as others do. American managers eager to reach an agreement with a Chinese parts supplier, for
example, were disappointed with the slow negotiations process. The Chinese managers, on the other hand, were pleased that
so much time had been taken to build personal relationships with the American managers. Wise business communicators
strive to prevent miscommunication by being alert to both their own frames of reference and those of others.
Lack of Language Skill. No matter how extraordinary the idea, it won‟t be understood or fully appreciated unless the
communicators involved have good language skills. Each individual needs an adequate vocabulary, a command of basic
punctuation and grammar, and skill in written and oral expression. Moreover, poor listening skills can prevent us from
hearing oral messages clearly and thus responding properly.
Distractions. Other barriers include emotional interference, physical distractions, and digital interruptions. Shaping an
intelligent message is difficult when one is feeling joy, fear, resentment, hostility, sadness, or some other strong emotion. To
reduce the influence of emotions on communication, both senders and receivers should focus on the content of the message
and try to remain objective. Physical distractions such as faulty acoustics, noisy surroundings, or a poor cell phone
connection can disrupt oral communication. Similarly, sloppy appearance, poor printing, careless formatting, and
typographical or spelling errors can disrupt written messages. What‟s more, technology doesn‟t seem to be helping.
Knowledge workers are increasingly distracted by multitasking, digital and information overload, conflicting demands, and
being constantly available digitally. Clear communication requires focusing on what is important and shutting out
interruptions.

3. Conflicts within an organization need to be settled as early as possible. Suggest five specific ways of conflict
resolution within a business organization. 10
Answer:
Conflicts are very obvious features of organizational management. These are the result of various aspects such as
misunderstanding, egos, etc. If the conflicts are not managed properly they can harm the organization very badly. However, if
the conflicts are carefully managed they are more advantageous rather than harmful. They have positive results too. So,
instead of lingering them they need to be settled as early as possible. For this, very careful and appropriate strategies that fit
the given context are required to be adopted. Some of the specific ways of conflict resolution are:

 Proaction: Addressing minor conflicts before they become serious.


 Communication: Bringing the conflicting parties to a negotiating table to discuss the issues.
 Research: Identify the factual reasons of the conflict.
 Flexibility: Create a flexible environment for free and open discussion.
 Fair Play: Encourage everyone to work out a fair solution to the problem.

4. In the international scene, knowing and identifying cultural differences can make a big difference between
successful business negotiations and humiliating rejections. Elaborate. What are some of the effective international business
communication guidelines? 10

Answer:
International business communication plays a major role in this, as knowing how to communicate through their point of view
is a great advantage. Each country has their own way of saying things, what may not be offending to you may be offending to
them. International business communication is all about knowing the important thing that lies behind peoples‟ words in the
international arena. International business communication is communicating across cultures and the first thing you must
always put in mind is the basic understanding that one size doesn‟t fit all. Always putting in mind that the cultural practices
or habits you have does not mean that everyone else around the globe does.
The most important and worrying variables in the international scene of business are the cultural differences that most
international business executives fail to comprehend resulting in most international business slip-ups. The failure to identify
and adapt to these differences through effective international business communication can mean the difference between a
closed deal and a faceless failure. Assuming cultures affect the way people communicate with each other and language above
all else, is the core of culture.
Culture is the different ways in which people behave, act, or think towards themselves, families, and other people in the
society. It includes customs connected with social, political, family, and religious values. Different cultures have deviating
values for time and its use, for personal space, and for family values and chats. These differences must be identified and
recognized to bridge the gaps between cultures and for real communication to take place.
Learning effective international business communication is communicating across cultures, and it takes great sensitivity and
awareness by studying cultures and becoming more perceptive and adaptable in communication efforts.
Some of the effective international business communication guidelines that may help you are:
Listening with an open mind
Communicating across cultures is all about putting aside personal feelings and listening deeply to the person talking. Develop
your listening skills and learn how to hold your tongue. Knowing your own culture and the others culture as well will give

The Institute of Chartered Accountants of Nepal


23 of 62
Suggested Answers – Business Communication and Marketing
CAP II Examination – December 2012
you a better chance of bridging the differences. Keeping an open mind and sensitivity to these differences is a key factor for
better communication.
Non-verbal and Verbal behavior
Every language had their own linguistic preferences, and these preferences will give clues about those particular cultures
behavior, manners, and thinking. The English use modesty and understatement, and are sometimes purposely unclear to avoid
confrontation and very polite. Eloquence, expressiveness, and flowery of words are most common among Italians and
Spaniards, Germans on the other hand are very logical in manner. Knowing what people mean when they say yes is very
important; it may mean yes, I understand but not yes, I agree.
Gestures
Gestures involves sensitivity and awareness, as gestures that are not offending to you might be very offending to the other, as
in the case of beckoning someone to come over, some would hold out their hand with the palm up and then move the fingers
gesturing the person to come. In Asian culture however, this gesture is considered rude. To gesture someone to come in
Asian culture should be done with the palm facing

5. Briefly explain any FOUR of the following: (4×2.5=10)


a) Conducting an effective presentation
b) A resume
c) Advantages of conflict in team
d) Analyzing data for reports
e) Merits and demerits of grapevine form of communication

Answer:
a) Conducting an Effective Presentation.
Before conducting a presentation, you should always prepare a backup plan. Make an extra copy of your work for backup and
print your speaking notes. Use colour highlighters to mark the key words on the notes. This will help you keep your place as
you speak.
If you have a complex presentation with different multimedia, practise ahead of time. Produce extra copies of the handout
materials. Check out the room ahead of time, and make sure it has everything you need. When presenting your work, use the
following guidelines:
i) Be sure to introduce yourself and any co-presenters to your audience.
ii) Speak slowly and clearly!
iii) Familiarise the audience with a background explanation of your study.
iv) Proceed in a logical manner, telling what you did step by step.
v) Point to graphs, charts, etc., when you refer to them.
vi) Give credit to those whom you have contacted and to those who have helped you.
vii) When you finish speaking, ask the audience if there are any questions they would like to ask.
viii) Make sure that the questions have been addressed and thank the audience for any suggestions.
ix) Thank the audience for attending.

b) Resume
Resume is also known as bio-data. It is the formal arrangement of an individual‟s personal details. It is prepared in a given
pattern. Generally, it includes personal details, academic qualification, work experience, training, awards, etc. The resume
should be as impressive and persuasive as possible so that the employer could be convinced to offer a job to the presenter.
The resume is found in various formats. However, the purpose and the contents are almost similar. It includes systematically
personal details and its main purpose is to convince the employer for job.

c) Advantages of conflict in team


Conflict is a normal part of every workplace and every team. Although the word alone is enough to make our heart begin to
thump, conflict is not always negative. When managed properly, conflict can improve decision making, clarify values,
increase group cohesiveness, stimulate creativity, decrease tensions, and undermine dissatisfaction. Unresolved conflict,
however, can destroy productivity and seriously reduce morale.

d) Analyzing data for reports


Analysis and interpretation of the data is an important phase of preparing report and thesis from a study, or a survey, or a
research. When the data are collected from various sources by using various primary and secondary tools, these data are to be
processed and analyzed. Through the analysis and interpretation of data we can prepare the report with convincing findings
and recommendations. The data analysis procedures also vary significantly according to the nature of the study and report.
Generally, the data can be analyzed by using different statistical tools, scales and figures. We can use percentage, mean,
median, standard deviation and other statistical tools and test scores for the analysis of data. The tables, diagrams and figures
are obviously used. From such analysis comparison, correlation can be explicitly shown.

e) Merits and demerits of the grapevine form of communication:


The Institute of Chartered Accountants of Nepal
24 of 62
Suggested Answers – Business Communication and Marketing
CAP II Examination – December 2012
Merits
- Speedy transmission: It transmits information very speedily.
- Feedback value: The managers or top bosses of an organisation get the feedback regarding their policies, decisions
memos etc. Feedback reaches much faster.
- Support to other channels: It is a supplementary or parallel channel of communication
- Psychological satisfaction: It gives immense psychological satisfaction to the workers and strengthens their solidartiy,
demerits
- It is less credible. It cannot always be taken seriously.
- It does not always carry the complete information.
- If often distorts the picture or often misinforms.

Section -'B'
6. Read the following case carefully and answer the questions given below:
Wheel, a product of Uniliver, is a detergent that caters to the needs of the mass market. It was first launched by Hindustan
Lever in 1987 in India. Wheel Green is the single largest selling detergent brand in Nepal by value. Wheel is packed with
powerful Lather that cleans even the toughest stains on collars and cuffs with case. It cleans effectively with lesser effort,
making washing light and easy. Moreover, Wheel does contain a high percentage of soda.
Wheel also has a premium variant called „Active Wheel‟. It gives consumers the value of three benefits in one. It not only
cleans effectively with less effort but also keeps the colored clothes looking bright and has a fresh fragrance. Active Wheel
gives consumers “quality clean and care” at an affordable price. Involvement of lesser number of middlemen, limited
distribution outlets and weak promotion base has adversely affected its desired level of sales and profit.
In short, Wheel can be summed up as –“Mehenat Kam, dhulai No. 1”. Wheel is, therefore, the smart housewife‟s choice.
Questions:
a) Make a short analysis of the case and describe product attributes of Wheel. 5
b) Give suggestions to Uniliver Ltd. for improving the performance of Active Wheel. 5

Answer
a) Wheel, a product of unilever is found very popular in Nepal. Many people liked wheel for their laundry needs as it
has many good qualities. It is treated as less harmful to hands and comparatively of greater quality and economical. It has
good prospect in Nepalese market. Clear with less efforts, product compositions and good results are its notable features due
to which it is preferred by many people.
Major attributes of wheel detergent are: cleans efficiently with lesser effort, does not burn hands or harm clothes, many
benefits in one-makes clothes bright, fresh fragrance, affordable price, etc.
b) The above attributes have made wheel a popular detergent units less effort and good results. In other words
(mehanat kam dhulai No.1) wheel has several good features leading to be highly – demanded product. Washes clothes clean
and customer care at a reasonable price. Unilever has to increase number of middlemen, improve its promotional base by
using new advertising, publicity, sales promotion strategy so that it can expand more market to improve the performance of
Active Wheel.

7.
a) Define marketing. Explain the importance of marketing to the society. 5
b) Explain the fundamental principles of societal marketing. 5

Answer
a) A few definition of marketing are as under:
Stantion, Etzen and walker have difined:
“Marketing is a total sysyem of business activities designed to plan, price, promote and distribute want –satisfying products
to target markets to active organizational objectives”
Philip Katter:
“Marketing is a societal process by which individuals and groups obtain what they need and want through creating, offering
and freely exchanging products and services of value and others.”
From the above definitions we can define marketing as a societal process and all activities to create exchange relationships
for customer need fulfillment and achieve organizational goals in the given environment. So all the activities directed in the
flow of good and services from producer to consumer, come under marketing.

b) The societal marketing concept assumes that along with the consumers' wants and interests, social interests too
should be considered so that long-run social welfare could be maintained. The underlying premises of the societal marketing
concept are:
i. society's long-run interests;
ii. Consumers will increasingly favor an organization which shows a concern with meeting their wants, long-run
interests, and society's long run interests;
The ultimate goal of marketing should be to achieve organizational through social goods welfare and well being.
The Institute of Chartered Accountants of Nepal
25 of 62
Suggested Answers – Business Communication and Marketing
CAP II Examination – December 2012

8.
a) What makes the market segmentation meaningful and effective? Explain. 5
b) What do you understand by Marketing Information System (MIS)? Explain marketing research as a major part of MIS.
5

Answer
Market segmentation is the process of dividing large, heterogeneous (dissimilar) markets into smaller, homogeneous (similar)
sub-markets. In other words, market segmentation is the process by which an organization attempts to match a total
marketing program to the unique needs of one or more customer groups in the marketplace.
The basic objective of segmenting a market involves improving competency, sales volume, and profitability of the
organization. To achieve this objective market segmentation must be effective and meaningful. In order to make market
segmentation effective and meaningful the following conditions or must be fulfilled or followed.
 There should be similarities in customer characteristics
in Each Segment
 Customer Differences should be Measurable or Identifiable
 Segments should be Accessible or Reachable
 Segments should be large Enough
 Segments should be sufficiently Stable
a) Marketing information system (MIS) is an on-going process consisting of people and equipments, to generate, sort,
analyze, evaluate and distribute needed accurate information in time for use in marketing decision marketing.
Organizations need to design and use an effective MIS to gather information and manage its continued flow to support in
achieving marketing objectives. It consists of input →processing →output →feedback components. Use of information
technology especially the computer and internet play an important role in MIS.
Marketing research system is an important component of MIS. Marketing research is a systematic and objective study of
problems pertaining to the marketing of goods and services. Marketing research is done to provide information to
management so that it can identify to react to marketing opportunities and problems. It also provides the required, reliable
and timely information for making marketing decisions to managers. Most large companies have their own marketing
research department. So marketing research specifics the information required, collecting information, analyzing the results
and communicating the findings. As such marketing research is very valuable to MIS.

9.
a) Point out the new product development process and explain test marketing. 5
b) What is physical distribution? Explain the contributions of physical distribution system in marketing. (2+3=5)

Answer
a) Stages in new product development are as fellows:
- Idea generation
- Idea screening
- Concept dev. & testing
- Marketing strategy and business analysis
- Product development
- Test marketing and
- Commercialization.
Test marketing:
Product is manufactured in small volume for test marketing. With a view to see consumer response on product it is placed on
sale in a test market. It is send with brand name, packaging and all other procedures followed at a limited scale.
Test market helps to correct/ improve the marketing mix. Is a product's test market is ok, its goes to final stage of
commercialization. At this stage trial, first repeat, adoption and purchase frequency are monitored.
b) Physical distribution is concerned with the management of physical flow of goods from the points of suppliers to
the points of purchasers.
The main objective of physical distribution is getting the right products safely to the right places at the right time at the least
possible cost.
The main contributions of physical distribution system are as follows:
a. It provides customer service by managing transportation, warehousing whenever and wherever required;
b. It distributes goods more safely at right time by selecting the proper means of transportation and physical handling.
c. It minimizes the total cost by selecting the right channel, transportation system and proper materials handling
technique; and
d. It supplies goods to the right target market by delivering the desired goods to target customers.
e. It fulfills customers‟ needs and provides them satisfaction because the actual movement and delivery of goods is
possible only through the physical distribution system.

The Institute of Chartered Accountants of Nepal


26 of 62
Suggested Answers – Business Communication and Marketing
CAP II Examination – December 2012
10. Briefly explain the following: (5×2=10)
a) E-marketing
b) Marketing environment in Nepal
c) Stages of consumer buying process
d) Mark-up pricing method
e) Branding

Answer
a) E-marketing
E-marketing describes a wide variety of electronic services used in marketing. Purchase of goods and selling are both done
through electronic media under E-marketing. Direct markets use media to make direct offers to potential buyers. Besides
other electronic data interchange through the use of fax, e-mail, use of ATM and the use of internet and online services are
gaining popularity. The internet today functions as an information source, an entertainment source, a communication
channel, a transaction and distribution channel. Any one having internet service cam send e-mail, exchange views, shop for
produces, access news, and exchange business information. So internet provides marketing opportunities which come under
E-marketing.

b) Nepal is a small landlocked Himalayan country situated at the heart of Asia. It is situated between India in the west,
south and east and the Tibet Autonomous Region of China in the north. Nepal has a multi-party system.
The major environmental forces that have impact on Nepalese market and trade pattern are economic environment,
demographic environment, political-legal environment, technological environment, natural environment, and competitive
environment.
Nepal is an agrarian country. Contribution of agriculture sector to the country‟s real GDP and employment is high. Farming
is still highly labour-intensive.
Recently, marketing opportunities for services sectors like computer software, insurance, banking, education, hospitals and
nursing homes have been growing rapidly.
The demographic character of Nepal shows that the market structure of the country is diverse in nature.
Nepal is a labour-abundant country. Labour is comparatively cheap in Nepal. Industrial development has not taken proper
direction in Nepal, so the use of appropriate technology is still shy. Rural farming is still characterized by traditional local
technology.
New innovations are lacking in Nepal. Almost all the technologies transferred into Nepal are “me too” type of technology
i.e., “imitating” one. Recently, information technology has been gaining momentum in the country in commercial sector.
For the last several years, Nepal has been badly affected by political unrest in the country, which has multiplier effects on the
country‟s economic and social development.

c) Consumer behavior is concerned with the study of consumers‟ activities and actions relating to the selection,
purchase and consumption of goods.
Though there is no hard and fast rule regarding the compulsion to follow up a rigid buying decision process, a rational
consumer usually follows.
buying process of five steps:
 Need recognition
 Information search
 Evaluation of alternatives
 Purchase decision
 Post purchase evaluation

d) Mark-up pricing
Price is the value paid on what is exchanged and measured in monetary units. It is the act of determining the exchange value
between the purchasing power and utility or satisfaction derived though the purchase of products.
Companies are having different objectives, policies and methods of setting prices their products. Out of many commonly
used methods of pricing is cost oriented pricing. Mark-up pricing method is to add a standard mark-up to the product cost.
It per unit cost is Rs.80 and 25 percent mark up then manufacture will charge Rs 100 for the product.

e) Branding
Branding is the art and cornerstone of marketing. A brand is a name, term, sign, symbol or design or a combination of them,
intended to identify the goods or services of one seller or group of sellers and to differentiate them from those of competitors.
So, a brand identifies the producer or seller. A brand can be better positioned by associating its name with a desirable benefit.
Companies promote their brand for different purposes depending upon what they want to prefer quality, performance, safety,
adventure, emotion, etc. So, a brand is essentially a marketer‟s promise to deliver a specific set of features, benefits and
services to the buyers. Brands are legally protected and superior to trade marks. A good brand name requires all such
qualities. Brand names differ with the company. Today, branding is such a strong force that hardly anything goes unbranded.

The Institute of Chartered Accountants of Nepal


27 of 62
The Institute of Chartered Accountants of Nepal
Suggested Answers of Income Tax and VAT

CAP II Examination- December 2012

1. Answer the following with reference to the Indian Income Tax Act, 1961:

a) Is there any income not chargeable to tax in case of Electoral Trust? Are there any conditions to be satisfied? Explain. 2

b) Mr. Phil comes to India for the first time on September 1, 2011. On September 15, 2011, he joins a company on monthly
salary of Rs. 90,000, as a part-time consultant (duty hours: 6.30 pm to 9.30 pm). Prior to September 15, 2011, Mr. Phil does
not have any source of income. On October 9, 2011, he starts a trading business in computer hardware after obtaining
approval of his employer. For the previous year March 31, 2012, he has the following income:
Salary from the part-time employment: Rs. 585,000; income from the business of trading in computer hardware in India: Rs.
1 million; and foreign income from the same business: $ 50,000. Find out the residential status of Mr. Phil for the assessment
year 2012-13. 6

c) Mr. Jury, a foreign citizen, is residing in India since 2004. While completing his assessment for the assessment year 2010-
11, on February 14, 2011, the Assessing Officer comes to know that Mr. Jury will leave India on April 12, 2012 with no
intention of returning. Is there any provision wherein Assessing Officer can make assessment in assessment year 2010-11? 2

Answer:
a) Voluntary contribution received by an electoral trust is treated as income under section 2(24)(iia). By virtue of section 13B,
donation received by an electoral trust will not be chargeable to tax if the following conditions are satisfied;

i. The electoral trust is approved by the Central Board of Direct Taxes in accordance with the scheme made by the Central
Government.
ii. The electoral trust will have to distribute to political parties 95 percent of the aggregate donations received by it during the
previous year along with the surplus, if any, brought forward from any earlier years. For this purpose, a political party means
a political party registered under section 29A of the Representation of the People Act, 1951.

iii. The electoral trust functions in accordance with the rules made in this regard by the Central Government.

b) As per section 6(1), an individual is said to be resident in India in any previous year, if he satisfies at least one of the
following conditions (basic conditions)-
i. He is in India in the previous year for a period of 182 days or more.
ii. He is in India for a period of 60 days or more during the previous year and 365 days or more during 4 years immediately
preceeding previous year.
Further, as per section 6 (6), a resident individual is treated as “resident and ordinarily resident” in India if he satisfies the
following two conditions (additional conditions)-
i. He has been resident in India in at least 2 out of 10 previous years immediately preceeding the relevant previous year
ii. He has been in India for a period of 730 days or more during 7 years immediately preceeding the relevant previous year.

For the assessment year 2012-13, Mr. Phil has the following sources of income in India:
Sources of Income Previous Year Number of days when Mr. Phil
was in India
Salary Income September 15, 2011 to March 31, 2012 199 days
Business Income October 9, 2012 to March 31, 2012 175 days

For the first source on income, Mr. Phil becomes resident in India by satisfying one of the basic conditions. As he comes to
India for the first time in 2011, he is unable to satisfy any of the additional conditions. Thus, he is a resident but not ordinary
resident in India for the first previous year.
For the second source of income, Mr. Phil is a non-resident, as he satisfies none of the basic conditions.
It may be noted that he is non-resident in India for the business income and resident but not ordinarily resident for the salary
income. In view of section 6(5), if a person is resident in India for one of the sources of income, he will be deemed to be
resident in India for all other sources of income in the same assessment year. In respect of the assessment year 2012-13, Mr.
Phil will, therefore, be regarded as resident but not ordinarily resident for all sources of income.
Suggested Answers – Income Tax and VAT
CAP II Examination – December 2012
c) Section 174 prescribes that if it appears to the Assessing Officer that an individual may leave India during the current
assessment year or shortly thereafter with no intention of returning to India, the total income of such individual up to the
probable date of his departure from India shall be chargeable to tax in that assessment year.

Mr. Jury has intention of leaving India only on April 12, 2012 which is not shortly after then the current assessment year as
prescribed by section 174. Hence, in assessment year 2010-11, the assessing officer can make assessment of previous year
2009-10. The assessing officer can assess the income of assessment year 2011-12/2012-13 (For the period April 01, 2012-
April 12, 2012) in assessment year 2011-12 as per section 174.

2.
a) Anubhav & Co. has taken on rent the premises from Buddhi Shree & Co for the operation of its business which pays
monthly rent of Rs. 100,000. Anubhav & Co. is deducting withholding tax at the rate of 10% amounting to Rs. 10,000 per
month and making payment amounting to Rs. 90,000 per month to Buddhi Shree & Co. While submitting Income Tax
Return, tax liability of Buddhi Shree & Co, to be payable during Income Year 2068/69 came to Rs. 200,000. Answer the
following question mentioning the relevant provision of Income Tax Act, 2058.
i) What is the amount of tax to be deposited?
ii) Will your answer be different if tax liability of Buddhi Shree & Co comes to Rs. 100,000 instead of Rs. 200,000?
iii) What is the implication if Buddhi Shree & Co had not adjusted the amount of withholding tax paid through Anubhav &
Co. while submitting income tax return? 8

b) Mr. „Z‟ a Canadian citizen is deputed by the Canadian government to work on a Canadian aided project and he stays in
Nepal for 94 days. He receives his salary in Canada and he is provided a free accommodation and a daily allowance of Rs.
2,000 to meet his boarding and other expenses.

i) Examine the liability to tax under Income Tax Act, 2058.


ii) Will your answer be different if Mr. „Z‟ comes to Nepal under the services of a private contractor instead of the Canadian
government in the above case? 4

c) The status of property, plant and equipments and repair and improvement Expenditure XYZ & Co. during Income Year
2068/69 is as follows;
Block of Assets Repair and Improvement Expenditure (Rs.) Depreciation basis (Rs.)
Building Block 2,000,000 100,000,000
Computer Block 100,000 800,000
Automobile Block 1,600,000 16,000,000
Total 3,700,000 116,800,000
Answer the followings mentioning the relevant provisions of Income Tax Act, 2058;

i) Can repair and improvement expenditure of Rs. 37, 00,000 be claimed under Income Tax Act, 2058. If not, compute the
amount of repair and improvement expenditure that can be claimed by XYZ & Co. during Income Year 2068/69.
ii) What is the implication of the amount of repair and improvement expenditure, if any, which cannot be claimed during
Income Year 2068/69? 8

Answer:
a) Section 93 (2) and (3) of Income Tax Act, 2058 prescribes the followings;
1) In case tax has been deducted from payments other than those not subject to final tax deduction, the person whose tax has
been deducted shall be deemed to have paid tax as follows:
(a) The tax amount deducted from payments under Section 87, 88 or 89.
b) The tax amount under Sub-Section (3) of Section 90, or the tax amount which is to be deemed to have been deducted
from payments, deposited, if any, at the Department by the advance tax deducting person or the person whose tax is
deducted.

2) The person whose tax is deducted under Sub-Section (2) may make a claim for its adjustment only with the amount of tax
payable by him in the income year in which the payment has been made.
Accordingly,
i. The amount of tax liability to be deposited by Buddhi Shree & Co, while submitting income tax return is Rs.80,000 after
adjusting the withholding tax deducted and paid through Anubhav & Co (Rs. 10,000*12 as TDS on rent).
ii. In this case, Buddhi Shree & Co can make a claim for the carry forward in next year or claim for the refund under section
113 of the Act, for the amount which is in excess of amount paid as withholding tax (i.e. Rs. 20,000).

The Institute of Chartered Accountants of Nepal


29 of 62
Suggested Answers – Income Tax and VAT
CAP II Examination – December 2012
iii. If Buddhi Shree & Co had not adjusted the amount of withholding tax paid through Anubhav & Co. during Income Year
2068/69 it cannot claim for the adjustment in forthcoming year. It can claim for the excess tax paid as refund under section
113 of the Act.

b)
i. The income derived by a natural person from the employment of public service of foreign country is exempted under
section 10 of Income Tax Act, 2058. Following conditions shall have to be satisfied in order to get the exemption;
a. Such natural person is resident or non-resident of Nepal because of employment only; and
b. Such payments shall be made from the public fund of foreign county.
Hence Mr. „Z‟ satisfies the above conditions, his salary and allowances are exempt from income tax in Nepal.
The place of receiving the salary of allowances is immaterial for the tax purpose.

ii. If Mr. „Z‟ comes to Nepal under the services of a private contractor instead of the Canadian government, the salary
received in Canada and the allowances received in Nepal combined together shall be taxable in Nepal.
The private contractor (employer) has to deduct withholding taxes from his salary as per section 87. In case the contractor
does not withhold the tax at source and does not deposit the same to revenue department, the contractor and Mr. „Z‟ are
responsible to pay the tax jointly or severally. As his total stay in Nepal is of 94 days only, he is non-resident in Nepal for the
Income Year and he has to pay tax at the rate of 25% on the taxable salary without allowing for initial deductions.

c) Section 16 of Income Tax Act, 2058 prescribes the following provisions;


1) Expenses incurred during and Income Year on repair and improvement of an asset owned and used depreciable asset in
order to generate income, is allowed to be deducted from taxable income.

2) Expenses on repair and improvement regarding a block of owned and used assets during an Income Year in excess of 7%
of the depreciable basis of the respective block at the end of Income Year, cannot be deducted during the Income Year.
3) The portion of the expenses disallowed during the year is allowed to be capitalized to the carrying amount of the
respective block of the assets,
Accordingly,
i. The entire amount of repair and improvement of Rs. 37,00,000 cannot be claimed. The amount of repair and improvement
that can be claimed under section 16 of Income Tax Act, 2058 can be computed as below;
Block of Assets Dep basis (Rs.) 7% of Dep basis (Rs.) Actual Repair and Improv Exp (Rs.) Claimable Expenses (Rs.)
Building Block 100,000,000 7,000,000 2,000,000 2,000,000
Computer Block 800,000 56,000 100,000 56,000
Automobile Block 16,000,000 1,120,000 1,600,000 1,120,000
Total 116,800,000 8,176,000 3,700,000 3,176,000

ii. The following is the implication of the amount which is in excess of the limit prescribed under section 16(2).
Block of Assets Claimable Expenses (Rs.) Amount in excess of limit (Rs.) Implications of the amount in excess of the limit
Building Block 20,00,000 - As the entire expense is claimable, no implication
Computer Block 56,000 44,000 The excess amount shall be included in the
opening depreciation basis of the following IY.
Automobile Block 11,20,00,0 4,80,000 The excess amount shall be included in the
opening depreciation basis of the following IY.
Total 31,76,00,000 524,000

3.
a) Mr. Ram had purchased a Land on 2063.10.01 for Rs.1 Crore. On 2066.05.06, he divorced his wife Mrs. Seeta. As part of
his divorce settlement, he transferred the land to his ex- wife Seeta on 2066.05.10. At the time of transfer, the market value of
the land was Rs. 1 Crore 40 Lacs. During the transfer, Mrs. Seeta incurred Rs. 10,000 towards registration and other legal
charges. Mr. Ram had informed the details of the transfer of land to ex-wife and elected the application of section 43 in
writing to Inland Revenue Office. Mrs. Seeta has sold the land on 2066.10.05 for Rs. 1 Crore 50 Lacs. By mentioning the
relevant provision of section 43, calculate the gain or loss on disposal of the land in the hands of both Mr. Ram and Mrs.
Seeta. If they do not elect the application of this section, how the incomings and outgoings are calculated? 7

b) Mr. Z , retired person from Nepal Government on Ist Sharwan, 2068 after 30 years of Service. After retirement, he joined
Kathmandu Bank at Kathmandu on the same day, .i.e. Sharwan, 2068 and remained in service till Ashad end 2069.
Information about his income are as follows:
S.N. Particulars Nepal government Kathmandu bank
1. Salary Rs. 3,00,000 Per month
2. House rent allowance Rs. 20,000 Per month
3. Other allowance Rs. 10,000 Per month
The Institute of Chartered Accountants of Nepal
30 of 62
Suggested Answers – Income Tax and VAT
CAP II Examination – December 2012
4. PF contribution Additional 10 % of basic salary & grade
5. Pension received Rs. 144,000 during the year ----------------

Other information:
 He has been provided a vehicle for personal as well as official use by the Kathmandu bank.
 Both employers have deducted 10 % of salary for provident fund and deposited 20 % to the approved retirement fund.
 He had received all retirement payment in Ashad 2068.
 He has not been provided Dashai allowances during the period.
 His wife does not have any income in this Income year and they declared they are couple.

Calculate the assessable income, taxable income and tax amount of Mr.Z for the income year. 6

c) Mr. Ram had purchased a Land situated at Bhaktapur on 2062.05.01 for Rs. 50 lacs.During the extension of Arniko
Highway, Nepal Government took possession of the land with a notice of compulsory acquisition on 2066.05.07. As
compensation, Nepal Government has given a sum of Rs. 80 Lacs to Mr. Ram. Mr. Ram purchased another land
approximately 2 Kms. away on 2067.04.05 for Rs. 60 Lacs. Mr. Ram has requested in writing to Inland Revenue Office for
the application of section 46 of the act. Compute the gain or loss on disposal of land in the hands of Mr. Ram by mentioning
the relevant provisions of the Act. Will the amount of gain or loss differ if he has not elected for application of this section? 7

Answer:
a) As per section 43 of the Act, in case a property is transferred due to a divorce settlement or to one's husband, wife, ex-
husband or ex-wife and election has been done for the application of this section, then:
- The net outgoings incurred by the transferor will be treated as if they have been received from the transfer and
- The same amount will be treated as net outgoings for the transferee.
Since election has been done for the application of section 43, gain or loss is calculated as follows:

In the hands of Mr. Ram


Incomings Rs. 1 Crore.
Less: Outgoings Rs. 1 Crore
Gain or Loss Nil
The transferor experiences neither a gain nor loss from the disposal of the property.

In the hands of Mrs. Seeta


Incomings Rs.1,50,00,000
Less:
Outgoings:
Cost of the transferred asset1,00,00,000
Registration and legal charge 10,000 Rs.1,00,10,000
Gain from disposal of land Rs. 49,90,000
If they do not elect the application of this section, then the market value of the property or the actual outgoings, whichever is
higher are treated as incomings for the transferor and outgoings for the transferee.

Thus, if they do not elect sec 43 to be applicable in this case the gain shall be calculated as under:
In the hands of Mr. Ram
Incomings Rs. 14,000,000
Less: Outgoings Rs. 10,000,000
Gain or Loss Rs. 4,000,000
The transferor experiences Rs. 40 lakhs as gain from the disposal of the property.
In the hands of Mrs. Seeta
Incomings Rs.15,000,000
Less:
Outgoings:
Deemed Cost of the transferred asset14,000,000
Registration and legal charge 10,000 Rs.14,010,000
Gain from disposal of land Rs. 990,000

b) Calculation of Assessable Income, taxable income and tax

Particulars Rs.
Salary 3,600,000

The Institute of Chartered Accountants of Nepal


31 of 62
Suggested Answers – Income Tax and VAT
CAP II Examination – December 2012
House rent Allowance 240,000
Other allowance 120,000
Addition on Provident fund 360,000
Vehicle facilities (0.5 % of Salary) 18,000
Pension 144,000
Assessable Income 4,482,000
Less: Contribution to Provident fund 300,000
Whichever is lower
Actual contribution Rs. 720,000
One third of Assessable income Rs. 1,494,000
Upper limit as per section 63 Rs. 300,000

Taxable Income 4,182,000

Less Exemption on pension (25% of basic exemption limit) 50,000

Taxable Income after exemption 4,132,000

Tax calculation

Slab Tax Rate Tax Rs.


200,000 1% 2,000
100,000 15% 15,000
38,32,000 25% 9,580,000
Additional tax (see note) 168,200
Total annual tax for Mr. Z 1,143,200

Note:
Taxable income 4,182,000
Less: 2,500,000
Taxable income over
than Rs. 2,500,000 1,682,000
40% of tax @25% 168,200

c) Section 46 of the Act envisages the circumstances when a person has to dispose of an asset or liability due to orders issued
by government or due to changes in legal decisions or when a person has to dispose of an asset or a liability due to forceful
conditions. The conditions applicable for availing this section are as under:
a. There must be a case of involuntary disposal of an asset or a liability.
b. The asset or liability is exchanged for another same kind of asset or liability within one year of the disposal.
c. The person requests in writing for application of this section for the disposal with replacement.

Section 46 further states that the total incomings for the transferred assets shall be the sum of the following amounts:
- The net outgoings for the assets up to the disposal of the asset and
- In case the amount received from involuntary acquisition exceeds the amount paid or payable for the replaced asset, the
excess of the amount.
In the given case,
1. Compensation received from Government Rs.8,000,000
2. Cost of the Replaced asset Rs. 6,000,000
3. Excess of 1-2 Rs. 2,000,000
4. Net Outgoings for the asset before disposal Rs. 5,000,000
5. Total Incomings (3+4) Rs. 7,000,000
6. Total Outgoings Rs. 5,000,000
7. Gain from Disposal Rs. 2,000,000

If he has not selected for application of this section, then the gain is calculated as under:
Incomings Rs. 8,000,000
Less:
Outgoings Rs. 5,000,000
Gain from disposal Rs. 3,000,000

The Institute of Chartered Accountants of Nepal


32 of 62
Suggested Answers – Income Tax and VAT
CAP II Examination – December 2012
4. ABC Ltd. has entered into a construction contract with PQR Ltd. with a term of 3 years. The contract price and the total
estimated cost are Rs.60 Billion and Rs. 40 Billion respectively in Year1 of the contract, cost amounting to Rs. 10 Billion
was incurred and up to year 2 Rs.25 Billion has been expensed. Calculate the income of ABC Ltd. to be included in the
concerned head of income with respect to long term contract in Year 1 and Year 2 by also showing the calculation of
Cumulative Inclusions and Cumulative Deductions.
Briefly explain on Long Term Contract, Excluded Contract and Deferred Return. (5+5=10)

Answer:
Year 1
Percentage of completion of the contract = 10 Billion/40 Billion =25%.
Cumulative inclusion is derived by multiplying total estimative cumulative inclusions up to the completion by percentage of
contract completed.
Rs. 60 Billion * 25% = Rs. 15 Billion.
Cumulative Deductions is derived by multiplying total estimated deductions up to completion by percentage of contract
completed.
Rs. 40 Billion * 25% = Rs. 10 Billion.
Amount to be included as income in Year 1 = Rs. 15 Billion – Rs. 10 Billion
= Rs. 5 Billion.
Year 2
Percentage of completion of contract = 25/ 40 = 62.5%.
Cumulative inclusions = Rs.60 Billion * 62.5% = Rs. 37.5 Billion
Cumulative Deductions = Rs. 40 Billion * 62.5% = Rs. 25 Billion
Amount to be included in year 2 as income = Rs. 37.5 – Rs.25-Rs.5 = Rs. 7.5 Billion.

Long Term Contract – As per section 26(1) of the act, a long term contract is a contract for production, installation,
construction or the services related to them, which runs for more than twelve months. To establish a long term contract under
this section, there should be a deferred return as a condition of the contract and the contract should not be an excluded
contract.
Excluded Contract – As per Rule 11 of the Income Tax Rules, Excluded contract is any contract created by reason of an
interest in an entity or by obtaining a membership in a retirement fund or any contract of investment insurance. Excluded
contract is not taken as a long term contract.
Deferred Return - As per Rule 10 of the Income Tax Rules, a contract shall be called as deferred return contract if any party
to a contract does not declare the information related to the estimated profit and estimated loss for the period of every six
months starting from the commencement of the contract as required by IRD.

5.
a) EFG Ltd. has given a contract to Garibdas construction, a party not registered in VAT, for construction of a Shopping
Complex built exclusively for commercial purposes. The cost of the shopping complex is Rs.55 lacs. By mentioning the
relevant provision of the act and rules, comment on the taxability of the transaction. 4

b) Moonlight Traders is engaged in the business of selling Television sets on installment basis. As per the policy and
contract agreement, forty percent of the Price of the Television Rs. 20,000, i.e. Rs. 8,000 has to be paid as upfront payment in
cash and thereafter monthly installment of Rs.1000 along with interest 1% per month has to be paid at the end of the month.
Mr. Ram Binod has purchased a television set by paying Rs. 8000 in cash on 2066.09.07. Installment of Magh and Falgun
has been paid on Falgun 05, 2066. By mentioning the relevant provision, determine the time of supply for Magh and Falgun
Month installments. 4

c) M/s Kiran Distributors is engaged in the wholesale business of various types of noodles. The firm is registered in VAT.
The firm has taken a loan of Rs. 10 lacs from M/s Saraswati Bank for purchasing a delivery van. Bank has paid the total
amount for the purchase of Van but has kept the ownership of van in the name of the bank till the loan is fully repaid by
Kiran Distributors. Kiran has to pay Rs. 25,000 per month towards the principal and interest to the Bank as part of their loan
agreement. Rs. 130,000 was paid as VAT during the purchase of van. By mentioning the relevant provision of the Act, can
Kiran Distributors claim the input tax credit? 4

d) ABC Ltd, a foreign party got a contract from an organization in Nepal. Before the start of the work, they have registered
themselves with VAT. As a tax professional, they enquire you on the provision for payment of VAT as per the VAT Act.
Advice ABC Ltd. 4

e) What is Debit / Credit Note? 4

Answer:

The Institute of Chartered Accountants of Nepal


33 of 62
Suggested Answers – Income Tax and VAT
CAP II Examination – December 2012
a) As per section 8(3) of the act, any person registered or not in Nepal engaged in constructing commercial buildings,
apartments, shopping malls or similar having value more than Rs.5 Million need to pay VAT on construction cost, if not paid
to registered person.
Further , Rule 6(kha) of the VAT Rules states that any person who, for commercial purposes, is constructing Building,
Apartment, Shopping Complex or similar as directed by the IRD amounting more than Rs.5 Million has to get it constructed
only from VAT registered person

In the referred case, since EFG Ltd. got the construction of a shopping complex for commercial purposes from non VAT
registered party, hence as per section 8(3) of the act, EFG Ltd. has to deposit the VAT on transaction value i.e. 13% of Rs.55
Lacs = Rs. 7.15 Lacs to the Inland Revenue. With regard to taxability of Garibdas Construction, since the amount of
transaction has exceeded minimum slab, it has to get registered itself in VAT.

b) As per section 6(3)(kha) of the VAT Act, in the case of contractual provision for paying the value of goods or services
partially in more than one day on an installment basis, earliest of due date as per the contract or actual payment date shall be
considered as Time of Supply.
In the month of Magh 2066, due date is Magh end but actual payment date is Falgun 05, hence in this case, time of supply is
Magh End.
For the month of Falgun 2066, due date is Falgun end but actual payment date is Falgun 05, hence in this case, time of supply
is Falgun 05.

c) As per section 17(5ka) of the act, if a tax payer has purchased or imported capital goods under a loan agreement within a
financial lease, then input tax paid can be claimed by the concerned tax payer.

In finance lease, all the benefits and risks are to be borne by the lessee, only legal passing of the title happens after the
stipulated date mentioned in the agreement. In this case, although the legal title still vests with the bank, but the firm can take
the input tax credit as per section 17(5ka).

d) Section 19 of the VAT Act states about the payment of value added tax as following:
1) A taxpayer shall have to pay the tax for each month within twenty five days of the close of the month.
2) If a taxpayer does not pay the tax within the time limit specified under (1), an extra charge of ten percent per annum shall
be imposed on the tax due.
3) If a taxpayer applies to the Director General for the exemption of the additional charges provided by (2) stating the reason
that the failure to make a timely payment was caused by extraordinary circumstances beyond the taxpayer's control, the
Director General may, if he finds the reason reasonable, exempt such charges.
4) The charges pursuant to (2) and and the interest pursuant to Section 26, shall be charged from the date on which the tax
first became due.
5) If a tax officer makes tax assessment under section 20 and finds that the amount of tax to be collected from the tax payer
in a tax period is less than the amount he is entitled for refund, then extra charge and interest shall not collected in that tax
period.
6) Tax can be paid through the Good for Payment Cheque issued by the Bank. In such a case, the day tax office receives the
good for payment cheque, it is deemed that the tax is collected.
7) Interest shall not be collected on Interest, Extra charge and penalties.

e) As per Rule 20 of the VAT Rules, a debit note/credit note is a document issued by a registered person owing to a change
in the value of the goods or services supplied by him previously.
The debit or credit note must include the following contents:
(a) Serial Number,
(b) Date of issue,
(c) Name, address and registration number of the supplier,
(d) Recipient's name, address, and registration number if he is a registered person,
(e) Number and date of the tax invoice connected with the transaction,
(f) Particulars of the goods or services and reason of credit or debit,
(g) Amount credited or debited,
(h) Tax amount credited or debited.
A registered taxpayer shall maintain a monthly record of credit or debit notes referred to in sub-rule (1)

6. Write on the following:


a) A lawyer practicing in Nepal has provided consultancy services to the Legal Firm situated in United Kingdom. The legal
firm situated in United Kingdom has used such services for the release of persons in USA. Where is place of supply of
services as per Value Added Tax Act/Rules? 5
b) There are some cases where Value Added Tax (VAT) paid on purchase is not allowed- no credit, even if output of
registered person is VAT attractive. Mention those cases as prescribed under Value Added Tax/Rules. 5
The Institute of Chartered Accountants of Nepal
34 of 62
Suggested Answers – Income Tax and VAT
CAP II Examination – December 2012
c) State the provisions on Value Added Tax (VAT) refund under Value Added Tax Act, 2052. 5
d) What are the records that are to be maintained by the tax payer as per rule 23 of VAT rules, 2053? 5

Answer:
a) Section 6 of Value Added Tax Act, 2052 deals with time and place of supply of goods or services.
1. For the purpose of assessment and collection of tax under this Act, the determination of the fact whether the supply of any
goods or services has taken place within or outside Nepal shall be as prescribed.
Further, as per rule 16 of Value Added Rules, 2053, the place of supply of a service shall be the place where the benefit of
that service is received.
In the case given in question, the lawyer in Nepal has provided service to Legal Firm in United Kingdom, the Legal Firm of
United Kingdom in turn has used such services and the payment of such services has been made from United Kingdom to
Nepal, and hence the place of supply is United Kingdom.

b) For the purpose of section 17 of Value Added Tax Act, 2052 following are the goods or services wherein VAT paid on
purchase is not allowed-no credit, as per rule 41 of Value Added Rules, 2053.
i. Consumption of drinkable items (soft drinks, water juice or similar)
ii. Liquor items (beer, wine, whiskey or similar)
iii. Petrol for vehicles used for human transport
iv. Expenses on Entertainment

c) Following are the provisions related to Value Added Tax (VAT) refund under Value Added Tax Act, 2052;
1. According to Section 24(3), if a registered person has continues credit of VAT for six months, credit is eligible for VAT
refund. In case any registered person has export during the month at least 40% of total sales in that month, the credit is
eligible for VAT refund as per section 24(4).
2. According to section 25, VAT paid by following person or paid for following event may be refunded, upon request for
refund within 3 years from the date of transaction on which the claim for refund is based;
i. Diplomat, privileged on a reciprocal basis from Ministry of Foreign Affairs, person engaged in Regional or International
Organization or missions having diplomatic privileges. This refund shall not be allowed for diplomats for purchase of goods
or services at a time for less than Rs. 1,500 as per section 15 (1ka).
ii. Institution or VAT paid such institution on which Ministry of Finance, has granted the privileges of tax exemption
iii. Tax exemption project by Ministry of Finance under bilateral and multilateral agreement
iv. Any tax collected by mistake.

3. Foreign tourist visiting in Nepal, if purchase and take away from Nepal via air transport shall get refund VAT on those
assets, if the cost paid is higher than Rs. 15,000. A service charge of 3% of refund is charged on refund (section 25 ka).

d) As per rule 23 of VAT rules, a registered person shall for the purpose of the Act and these Rules maintain records of the
following information, documents and details:-
 Information as per Schedule -7.
 Records relating to trade, accounts, cash receipts and payments.
 Tax invoices and abbreviated tax invoices issued.
 Tax invoices and abbreviated tax invoices received.
 All documents relating to his imports and exports,
 All debit and credit notes.
 Books of purchases and sales as per Schedules 8 and 9.

The Institute of Chartered Accountants of Nepal


35 of 62
The Institute of Chartered Accountants of Nepal

Suggested Answers

CAP II
December 2016

P.T.O.
(2)

Advance Accounting

P.T.O.
(3)

Suggested Answer
Roll No……………. Maximum Marks - 100
Total No. of Questions - 6 Total No. of Printed Pages - 17
Time Allowed - 3 Hours
Marks
Attempt all questions. Working notes should form part of the answer.
1. Following are the summarized Balance Sheet of Company Rishi Ltd. and Muni
Ltd., as at Ashad 31, 2073.
Liabilities Rishi Muni Assets Rishi Muni
Ltd.(Rs.) Ltd.(Rs.) Ltd.(Rs.) Ltd.(Rs.)
Share Capital Goodwill 20,000 -
Equity Shares of
2,000,000 1,500,000 Other Fixed Assets 2,400,000 1,150,000
NRs. 100 Each
10% Preference
Shares of NRs 700,000 400,000 Trade Receivables 625,000 615,000
100 Each
General Reserve 240,000 170,000 Inventory 412,000 680,000
Profit & Loss
- 15,000 Cash at Bank 38,000 155,000
Account
Own Debenture
12% Debenture of
600,000 200,000 (Nominal Value of 192,000 -
NRs. 100 Each
NRs. 200,000/-)
Discount on Issue
Trade Payables 560,000 315,000 2,000
of Debentures
Profit & Loss
411,000 -
Account
Total 4,100,000 2,600,000 Total 4,100,000 2,600,000

On Shrawan 01, 2073, Rishi Ltd. adopted the following scheme of reconstruction:
a) Each equity share shall be sub-divided into 10 equity share of Rs. 10 each
fully paid up. 50% of the equity share capital would be surrendered to the
company.
b) Preference dividend are in arrear for 3 years. Preference shareholders agreed
to waive 80% of the dividend claim and accept payment for the balance.
c) Own debenture of Rs. 80,000/- (Nominal Value) were sold at NRs. 98 cum
interest and remaining own debentures were cancelled.
d) Debentures holders of Rs. 300,000/- agreed to accept one machinery of book
value of Rs. 320,000/- in full settlement.
e) Trade payables, Trade receivables and Inventory were valued at Rs. 500000/-
Rs. 600000/- and Rs. 400000/- respectively.
f) The company paid Rs. 20,000/- as penalty to avoid capital commitments of
Rs. 400,000/-
On Shrawan 02, 2073, a scheme of absorption was adopted. Rishi Ltd. would take
over Muni Ltd. The purchase consideration was fixed as below:

P.T.O.
(4)
a) Equity shareholders of Muni Ltd. will be given 50 equity shares of Rs. 10 each
fully paid up, in exchange for every 5 shared held in Muni Ltd.
b) Issue of preference shares of NRs. 10 each in the ratio of 4 preference shares
of Rishi Ltd. for every 5 preference shares held in Muni Ltd.
c) Issue of 12% debentures of Rs. 100 each of Rishi Ltd. for every 12%
debentures in Muni Ltd.
Pass necessary Journal Entries in the books of Rishi Ltd., and draw a resultant
Balance Sheet as at Shrawan 02, 2073. 20
(Make suitable assumptions required, if any.)
Answer:
Journal Entries in the Books of Rishi Ltd.
Debit Credit
S.N. Particulars
(Amount NRs.) (Amount NRs.)
On Shrawan 01,2073
Equity share capital A/c (Rs.100) Dr. 2,000,000
To, Equity share capital A/c (Rs.10)
1
(Being sub-division of one share of NRs. 100 each into 10 2,000,000
shares of NRs. 10 each.)
Equity share capital A/c (Rs.10) Dr. 1,000,000
2 To, Capital reduction A/c
1,000,000
(Being reduction of capital made by 50%)
Capital reduction A/c Dr. 42,000
To, Bank A/c
3
(Being payment in cash of 20% of arrears of 3 years 42,000
preference dividend.)
Bank A/c Dr. 78,400
To, Own debenture A/c 76,800
[(192,000/200,000)*80,000/-]
4
To, Capital reduction A/c
(Being profit on sale of own debenture transfer to capital 1,600
reduction A/c)
12% Debenture A/c Dr. 120,000
To, Own debenture A/c 115,200
[(192,000/200,000)*120,000/-]
5 4,800
To, Capital reduction A/c
(Being profit on cancellation of own debenture transferred to
Capital reduction A/c)
12% Debenture A/c Dr. 300,000
Capital reduction A/c Dr. 20,000
6 To Machinery A/c
320,000
(Being Machinery of NRs. 320,000/- taken up by debenture
holders for NRs. 300,000/- )

Trade Payables A/c Dr. 60,000


7 To, Capital reduction A/c
60,000
(Being trade payable revalued)

P.T.O.
(5)

Capital reduction A/c Dr. 1,004,400 25,000


To, Trade Receivables A/c 12,000
To, Inventory A/c 20,000
To, Goodwill A/c 2,000
To, Discount on debentures A/c 411,000
8 To, Profit & Loss A/c 20,000
To, Bank A/c 514,400
To, Capital reserve A/c
(Being asset revalued, losses written off and penalty paid off
through capital reduction account and the balance of capital
reduction transferred to capital reserve account)
15,32,000
Business Purchase A/c Dr. (WN-1)
9 To, Liquidators of Muni Ltd. A/c
15,32,000
(Being Purchase consideration payable to Muni Ltd.)
Fixed Asset A/c Dr. 1,150,000
Inventory A/c Dr. 680,000
Trade receivables A/c Dr. 615,000
Cash at Bank A/c Dr. 155,000
315,000
To, Trade payables A/c
200,000
To, 12% Debentures A/c of Muni Ltd.
10 15,000
To, Profit & Loss A/c
170,000
To, General reserve A/c
368,000
To, Capital reserve A/c (WN-2)
15,32,000
To, Business Purchase A/c
(Being takeover of all assets & liabilities of Muni Ltd. by
Rishi Ltd.)
Liquidators of Muni Ltd. A/c Dr. 15,32,000
To, Equity share capital A/c 1,500,000
11
To, 10 % preference share capital A/c 32,000
(Being purchase consideration paid)
12% Debenture of Muni Ltd. A/c Dr.
To, 12% Debenture A/c 200,000
12
(Being Rishi Ltd. Issued their 12% Debenture against 12% 200,000
debenture of Muni Ltd.)

Balance Sheet as on Shrawan 02, 2073.


S.N. Particulars Amount (NPR)
I Equity & Liabilities
a) Shareholders Fund
Share Capital 32,32,000
Reserve & surplus 13,07,400
b) Long term liabilities
Long term borrowings 380,000
c) Current liabilities
Trade Payables 815,000
5,734,400
II Assets
a) Fixed Assets
Tangible assets 3,230,000
P.T.O.
(6)
b) Current Assets
Inventories 1,080,000
Trade Receivables 1,215,000
Cash & cash equivalents 209,400
5,734,400

Working Notes (WN)


S.N. Particulars Amount
1 Share Capital
Equity share capital 2,000,000
Less : Surrender 50% of equity capital (1,000,000)
Add: Equity share issued to Muni Ltd. 1,500,000 2,500,000

10% Preference share capital 700,000


Add: Preference share capital issued to Muni Ltd. 32,000 7,32,000
32,32,000
2 Reserve & Surplus
Profit & loss A/c 15,000
General reserve (240,000+170,000) 410,000
Capital reserve (514,400 +3,68,000) 8,82,400 13,07,400
3 Long term borrowings
12% Debentures 600,000
Less: Settled in consideration of Machinery (300,000)
Less: Cancelled debentures (120,000)
Add: 12 % Debentures issued to Muni Ltd. 200,000 380,000
4 Trade Payables
Rishi Ltd. Payables 560,000
Less: Reduction due to revaluation (60,000)
Add: Trade payables of Muni Ltd. 315,000 815,000
5 Tangible Assets
Balance of other fixed assets 2,400,000
Less: Machinery taken up (320,000)
Add: Other fixed assets of Muni Ltd. 1,150,000 3,230,000
6 Inventories
Inventories of Rishi Ltd. 412,000
Less: Reduction due to revaluation (12,000)
Add: Inventories of Muni Ltd. 680,000 1,080,000
7 Trade receivables
Trade receivables of Rishi Ltd. 625,000
Less: Reduction due to revaluation (25,000)
Add: Trade receivables of Muni Ltd. 615,000 1,215,000
8 Cash & cash equivalents
Cash & cash equivalents of Rishi Ltd. 38,000
Less: Payment of Arrears Dividend (42,000)
Add: Profit on sale of own debenture 78,400
P.T.O.
(7)
Less: Penalty paid (20,000)
Add: Cash & cash equivalent of Muni Ltd. 155,000 209,400
Total
9 Purchase consideration computation
Equity share capital [(15000*50/5)*NRs. 10] 1,500,000
10% Preference share capital [(4000*4/5)*NRs. 10] 32,000
Total purchase consideration 15,32,000

10 Capital Reserve
Share capital of Muni Ltd. (Equity + Preference) 1,900,000
Less: Share capital issued by Rishi Ltd. (15,32,000)
Capital reserve 3,68,000

2.
a) The summarized Balance Sheet of Jackson Ltd. as on 31st December 2014
and 2015 are as follows:
Amounts in Rs.
Liabilities 2014 2015 Assets 2014 2015
Share Capital 100,000 100,000 Building 46,800 45,000
General Reserve 38,400 42,000 Plant & Machinery 38,280 42,030
Creditors 9750 6380 Goodwill 13,000 13,000
Tax Provision 19,000 21,000 Investment 10,000 11,250
Prov. for doubtful debt 1,000 1,200 Stock 30,000 28,000
Debtors 22,070 22,300
Cash 8,000 9,000
Total 168,150 170,580 Total 168,150 170,580

After taking the following information into account, prepare a cash flow
statement for the year ended on 31st December 2015. 10
i) Profit for year 2014/2015 was Rs. 8,600 against this had been charged
depreciation Rs. 3,050 and increase in provision for doubtful debt
Rs.200/-.
ii) Income Tax Rs. 18,000 was paid during the year charged against the
provision and in addition Rs. 20,000 was charged against profit and
carried to the provision.
iii) An interim dividend of Rs. 5,000 was paid in January 2015.
iv) Additional Plant was purchased in September 2014 for Rs. 5,000
v) Investments (cost Rs. 5,000) were sold for Rs. 4,800 in 2015 and on 1st
March 2015 another investment was made for Rs. 6,250.

b) Gimmick Stores Limited, which runs a boutique makes up its accounts


annually on 31st Ashad. For the year ended 31st Ashad 2073, the Profit and
Loss Account was summarized as follows:
Rs. Rs.
Sales 2,160,000
Less: Cost of Sales (Net after discount received) 938,400
Wages 480,000 1,418,400
P.T.O.
(8)
Gross Profit 741,600
Less: Fixed Charges 541,600
Net Profit 200,000

On 1st Chaitra 2072, a fire occurred as a result of which no trading was


possible till 1st Shrawan 2073. On that date, half of the shop was reopened
for business; the other half reopened on 1st Ashwin 2073. The Consequential
Loss Policy covers gross profit and workroom wages, and accountancy
charges at Rs. 5,000 or 2% of the amount of the claim (before such charges)
whichever is higher. The insurer agreed that:
i) Turnover increases by 15% per annum
ii) Workroom wages increase by 10% per annum
iii) The period of three months from 1st Chaitra to 31st Jestha accounts for
one half of the annual turnover.
iv) Fixed charges and wages occur evenly only throughout the year.
v) Discount received equal 1% of turnover.
You are required to compute the Consequential loss Claim for Gimmick
Stores Limited. 10
Answer:

a) Cash Flow Statement for the year ended on 31st December 2015.

Particular Rs. Rs.


1. Cash Flow from Operating Activities
Net profit before tax 28,800
Adjustments:
Depreciation 3,050
Increase in provision for doubtful debt 200
Decrease in stock 2,000
Decrease in creditor (3,370)
Increase in Debtor (230)
Income Tax Paid (18,000)
Net Cash from Operating Activities 12,450
2. Cash Flow from Investing Activities
Investment Purchased (6,250)
Sale of Investment 4,800
Plant Purchased (5,000)
Net Cash Flow from Investing Activities (6,450)
3. Cash Flow from Financing Activities
Payment of Interim Dividend (5,000)
Net increase in cash equivalent 1,000
Add: Opening Cash Balance 8,000
Closing Cash Balance 9,000

Adjusted Profit and Loss Account


Particulars Rs. Particulars Rs.
To Provision for Tax 20,000 By Profit 28,800
To General Reserve 3,600
To Loss on Sale of Investment 200
To Interim Dividend 5,000

28,800 28,800
Provision for Tax Account
P.T.O.
(9)
Particulars Rs. Particulars Rs.
To Bank (Tax Paid) 18,000 By Balance b/d 19,000
To Balance c/d 21,000 By P & L A/C (Provision) 20,000

39,000 39,000

2(b)

1st Alternative: IF IT IS ASSUMED THAT THE GIVEN P & L A/c IS OF ASHADH END 2073
Computation of Consequential Loss Amount in NPR
(i) Ratio of Gross Profit:
Sales 21,60,000
Less: Cost of Sales 9,38,400
Add: Discount (1% of turnover) 21,600 9,60,000
Wages 4,80,000 14,40,000
Gross Profit 7,20,000

Thus ratio of Gross Profit 720,000/21,60,000 or 33.33%

(ii) Loss of Turnover Rs. Rs.


st st
(a) 1 Chaitra 2072 to 31 Ashad 2073
Turnover for 8 months (normal) 21,60,000
Monthly turnover (normal) (21,60,000X1/8) 2,70,000
If total turnover for the year is „A‟ then
A = 21,60,000+2,70,000+1/2xA (given by the question)
Therefore, A=annual turnover = 48,60,000
So, turnover for Chaitra to Jestha = 24,30,000 (half of annual turnover)

Total turnover (Chaitra to Ashadh)(24,30,000+2,70,000) 27,00,000

(b) 1st Shrawan 2073 to 31st Bhadra 2073


Monthly turnover (normal) 2,70,000
Add: 15% increase 40,500
3,10,500
Turnover applicable for 2 months for half the shop
1/2X3,10,500X2 3,10,500
Total Loss of Turnover 30,10,500

(iii) Loss of Profit: Rs.


33.33% of Rs. 30,10,500 10,03,500
Discount @1% on turnover (1% of Rs. 30,10,500) 30,105
Workroom wages for Chaitra to Ashad 1,60,000
(One third of Rs.4,80,000)
Workroom wages for Shrawan and Bhadra 44,000
½ X 2/12 X (4,80,000+10%)
12,37,605
Add: Accountancy charges @ 2% of above 24,752
Total Value of Claim to be lodged 12,62,357

Working Note
1. The given turnover is of only 8 months because after chaitra the business is completely closed till
Ashad 2073.
P.T.O.
(10)
2. Half of the turnover is for the period of 9 months other than Chaitra, Baisakh and Jestha. Hence for
one month, it will be 1/18 of the total.
Note-
1. Due to lack of information, Gross profit ratio is computed from 8 months
transaction.
2. Same Gross profit ratio (33.33%) has been assumed for the
computation of loss of profit for the period 1st Chaitra 2072 to 31st
Ashad 2073 and 1st Shrawan 2073 to 31st Bhadra 2073.
2(b)
2 nd Alternative:
IF IT IS ASSUMED THAT THE GIVEN P & L A/c IS OF ASHADH END 2072 INSTEAD OF ASHAD END 2073
Computation of Consequential Loss Amount in NPR
(i) Ratio of Gross Profit:
Sales 21,60,000
Less: Cost of Sales 9,38,400
Add: Discount (1% of turnover) 21,600 9,60,000
Wages 4,80,000 14,40,000
Gross Profit 7,20,000

Thus ratio of Gross Profit 720,000/21,60,000 or 33.33%

(ii) Loss of Turnover Rs. Rs.


(a) 1st Chaitra 2072 to 31st Ashad 2073
Annual Turnover 21,60,000
Add: 15% increase 3,24,000
24,84,000
1/2 upto 31st Jestha 2073 12,42,000
1/18 for one month upto 31st Ashad 2073 1,38,000
(Refer Working Note No.1)
Total 13,80,000
(b) 1st Shrawan 2073 to 31st Bhadra 2073
Annual turnover as above 24,84,000
Add: 15% increase 3,72,600
28,56,600
Turnover applicable for 2 months for half the shop
2/36X28,56,800 1,58,700
Total Loss of Turnover 15,38,700
(iii) Loss of Profit: Rs.
33.33% of Rs. 15,38,700 5,12,900
Discount @1% on turnover (1% of Rs.15,38,700) 15,387
Workroom wages for Chaitra to Ashad 1,76,000
(One third of Rs.4,80,000 plus 10%)
Workroom wages for Shrawan and Bhadra 48,800
½ X 2/12 X (5,28,000+10%)
7,53,087
Add: Accountancy charges @ 2% of above 15,062

Total Value of Claim to be lodged 7,68,149


Working Note
Half of the turnover is for the period of 9 months other than Chaitra, Baisakh and Jestha.
Hence for one month, it will be 1/18 of the total.

P.T.O.
(11)

Note-
3. Due to lack of information, Gross profit ratio is computed from 8 months
transaction.
4. Same Gross profit ratio (33.33%) has been assumed for the
computation of loss of profit for the period 1st Chaitra 2072 to 31st
Ashad 2073 and 1st Shrawan 2073 to 31st Bhadra 2073.
3.
a) Mahabir Digital Library Society showed the following position on Ashad 31,
2073.
Balance sheet as on Ashad 31, 2073
Liabilities Amount(Rs.) Assets Amount(Rs.)
Capital fund 793,000 Electrical fittings 150,000
Expense payable 7,000 Furniture 50,000
Digital Book license 400,000
Investment in securities 150,000
Cash at bank 25,000
Cash in hand 25,000
800,000 800,000

Receipts and Payment Accounts for the year ended Ashad 31, 2073 is given below:
Receipts Amount(Rs.) Payments Amount(Rs.)
To, Balance b/d By, Electrical charges 7,200
Cash at bank 25,000 By, Postage & stationary 5,000
Cash in hand 25,000 By, Telephone charges 5,000
By, Digital book License 60,000
To, Entrance fee 30,000 By, O/s expense paid 7,000
To, Membership subscription 200,000 By, Rent 88,000
To, Sale proceeds of old By, Investment in securities
1,500 40,000
paper (Magh 01, 2072)
To, Hire of lecture hall 20,000 By, Salaries 66,000
To, Interest on securities 8,000 By, Balance c/d
Cash at bank 20,000
Cash in hand 11,300
309,500 309,500

You are required to prepare Income and Expenditure Account for the year
ended Ashad 31, 2073 and Balance sheet as on Ashad 31, 2073 after making
following adjustments: 10
i) Membership subscription included Rs. 10,000 received in advance.
ii) Provide for Outstanding rent Rs. 4,000 and Salaries Rs. 3,000
iii) Digital book license to be amortized at 10% including additions, electrical
fitting and furniture are to be depreciated at 10%.
iv) 75% of entrance fees to be capitalized.
v) Interest on securities is to be calculated @ 5% p.a. including for purchase
made during the year.

b) Working capital of R.S. Trading Ltd. is Rs. 1,35,000 and current ratio is 2.5,
Liquid ratio is 1.5 and the proprietary ratio 0.75, bank overdraft is Rs.
P.T.O.
(12)
30,000. There are no long term loans and fictions assets. Reserve and surplus
amount to Rs. 90,000 and gearing ratio (Equity capital/preference capital) is
2.
Required: From the above ascertain current assets, current liabilities, net
block, proprietary fund, quick liabilities, quick assets, stock and preference
and equity capital. 5

3(a)
IF IT IS ASSUMED THAT THE GIVEN BALACE SHEET IS OF ASHADH END 2072 INSTEAD OF ASHAD END 2073

Answer:
Mahabir Digital Library Society
Income and expenditure account for the year ended Ashad 31, 2073
Dr. Cr.
Expenditure Amount Incomes Amount
To, Electric charges 7,200 By, Entrance fees 7,500
To, Postage & stationary 5,000 25% of 30000
To, Telephone charges 5,000
By Membership
To, Rent 88,000 200,000
subscription
Less: Received in
Add: Outstanding 4,000 92,000 (10,000) 190,000
Advance
To, Salaries 66,000
By, Sale proceeds of
Add: Outstanding 3,000 69,000 1,500
paper
By, Hire of lecture hall 20,000
To, Amortization & By, Interest on securities
8,000
Depreciation (WN-1) (WN-2)
Electrical fittings 15,000 Add: Receivable 500 8,500
Furniture 5,000
By, Deficit -excess of
Books 46,000 66,000 16,700
expenditure over Income
244,200 244,200

Mahabir Digital Library Society


Balance sheet as on Ashad 31, 2073
Liabilities Amount Assets Amount

Capital fund 793,000 Electrical fittings 150,000


Add: Entrance fees
22,500 Less: Depreciation (15,000) 135,000
capitalization
Less: Excess of expenses
(16,700) 798,800 Furniture 50,000
over income
Less: Depreciation (5,000) 45,000
Outstanding Expense Digital Books 460,000
Rent 4,000 Less: Amortization (46,000) 414,000
Salaries 3,000 7,000
Investment in
190,000
Securities
Membership subscription in advance 10,000 Accrued interest 500 190,500

Cash at bank 20,000

P.T.O.
(13)
- Cash in hand 11,300
815,800 815,800
Working Notes
1 Depreciation & Amortization 2 Interest on securities
Electrical fittings @10% 15,000 Interest @5% p.a. on 150000 full year 7,500
Furniture @10% 5,000 Interest @5% p.a. on 40000 half year 1,000
Digital Books @10% 46,000 Total 8,500
Total 66,000 Less: Received (8,000)
Receivable 500

Altenate -2

IF IT IS ASSUMED THAT THE GIVEN BALACE SHEET IS OF ASHAD END 2073

3(a)
Income and Expenditure Account for the year ended 31st Ashadh 2073
Expenditure Rs. Income Rs.
To Electrical Charges 7,200 BY Entrance Fees (25% of 30,000) 7,500
To Postage and Telegram 5,000 By Membership Subscription (200,000 - 190,000
10,000)
To Telephone Charges 5,000 By Sale proceeds of old paper 1,500
To Amortization of Digital License 44,444 By Hire of Lecture Hall 20,000
Book (400,000 * 10/90)
To Rent (88,000 + 4,000) 92,000 By Interest on Securities (110,000 *5% + 6,500
40,000 *5% *1/2)
To Salaries (66,000 + 3,000) 69,000
To Depreciation on Electrical fittings 16,667
(150,000 *10/90)
To Depreciation on Furniture (50,000 * 5,556 By Deficit 19,367
10/90)
294,867 294,867

Balance Sheet as on 31.3.2073


Liabilities Rs. Assets Rs.
Capital Fund 793,000 Electrical Fittings 150,000
Expenses payable 7,000 Furniture 50,000
Digital Book License 400,000
Investment in Securitas 150,000
Cash 25,000
Bank 25,000
800,000 800,000

b) Current liabilities and current Assets


C.R. =CA =2.5
CL
Or, CA – 2.5CL =0 ………I
Or, CA – CL = Rs. 135,000 ……II
Subtracting Eq. II for Eq. I
1.5 CL = Rs. 135000

CL = Rs. 135000 = Rs. 90,000


1.5
CA = Rs. Rs. 90,000 x 2.5 = Rs. 225,000

P.T.O.
(14)
ii. Quick liabilities, Quick Assets and stock
Quick Liabilities = Current liabilities – Bank overdraft
= Rs. 90,000 – Rs. 30,000 = Rs. 60,000
Liquid Ratio = Quick Assets = 1.5
Quick Liabilities
= Quick Assets =1.5
Rs. 60000
Or, Quick Assets = Rs. 90,000
Stock = Current Assets – Quick Assets
= Rs. 225,000 – Rs. 90,000
= Rs. 135000
iii. Net Fixed Assets and Proprietary Fund
Proprietary Ratio = Proprietary Fund
Total Assets
= Proprietary Fund =0.75
Net Fixed Assets + Current Assets
Proprietary Fund = 0.75 Net Fixed Assets + 168,750
i.e. since there is no long term loan,
Proprietary = Net Fixed Assets + Working Capital
= Net Fixed Assets + 168,750
So, Net Fixed Assets + 135,000 = 0.75 Net Block + 168,750
Or, 0.25 Net Fixed Assets = 33,750
Net Fixed Assets = 135,000
Proprietary Fund = Rs. 135,000 x .75 + Rs. 168,750
= Rs. 270,000
Iv. Preference and Equity Capital
Preference and Equity Capital = Proprietary Fund – Reserve & Surplus
= Rs. 270,000 – 90,000
= Rs. 180,000
Equity Capital = 2 Preference Capital
Or, Equity Capital – 2 Preference Capital = 0 ………..I
Equity Capital + Preference Capital = 180,000 ……….II
Subtracting Eq. I from Eq. II
3 Preference Capital = Rs. 180,000
Preference Capital = 180000
3
= Rs. 60,000
Equity Capital = Rs. 60,000 x 2
= Rs. 120,000

4.
a) Mr. Amit started working on a contract on 1st Baisakh 2071 for Rs.5,00,000.
On 31st Ashad 2071, when he prepared his final accounts, the following
information relating to the contractor was extracted from his books of
accounts:
Particulars Rs.
Material issued from stores and sent to site 1,60,000
Wages paid 1,01,200
Wages outstanding on 31-03-2071 37,520
New machines purchased and sent to the site on 1-1-2071 1,48,000
Direct charges paid 7,500
Direct charges outstanding on 31-03-2071 600
Establishment charges apportioned to contract 6,400

P.T.O.
(15)
On 31 Ashad 2071 materials lying unused at the site were valued at Rs.
21,620. Machines were depreciated at 20% per annum. Value of work
certified by 31st Ashad 2071 was Rs. 3,50,000 while the cost of work done
but yet not certified as on that date was Rs. 18,000. On the basis of
architect‟s certificate, Mr. Amit had received a total sum of Rs. 2,80,000
from the contractee till 31st Ashadh 2071.
Prepare contract account and relevant portion of the balance sheet in the
books of Mr. Amit. 10

b) From the following particulars, calculate: 5


i) Agreed value of two plants taken back by the hire vendor.
ii) Book value of plant left with the hire purchaser.
iii) Profit or loss to hire purchaser on two plants taken back by the hire vendor.
iv) Profit or loss on plants repossessed, when sold by the hire vendor.
Particulars:
a. X purchased three plants from Y on hire purchase basis, the cash
price of each plant being Rs. 100,000.
b. The hire purchaser charged depreciation @ 20% on diminishing
balance method.
c. Two plants were seized by on hire vendor when second installment
was not paid at the end of the second year. The hire vendor valued
the two plants at cash price less 30% depreciation annually charged
at diminishing balance method.
d. The hire vendor spent Rs. 40,000 on overhauling of the plants and
then sold them for a total sum of Rs 160,000.
Answer:
a)
Mr. Amit’s Ledger
Contract Account
Date Particulars Amount Date Particulars Amount
To Materials sent to site 1,60,000 By Unused materials at site 21,620
To Wages 1,01,200 By Machines A/c (WDV as 1,40,600
Add: Outstanding 37,520 1,38,720 on 31st Ashad 2071)
To Machines (Cost) 1,48,000 By Work in Progess:
To Direct Charges 7,500 Certified 3,50,000
Add: Outstanding 600 8,100 Uncertified 18,000 3,68,000
To Establishment charges 6,400
To Profit and Loss A/c 36,800
(Transfer of Profit)
To Balance c/d 32,200
5,30,220 5,30,220
To Materials at site 21,620
To Machines 1,40,600
(Refer Working Note No.1)
By Work in Progess:
Certified 3,50,000
Uncertified 18,000
3,68,000
Less: Balance b/d 32,200 3,35,800

P.T.O.
(16)

Balance Sheet as on 31st Ashad 2071 (Relevant Portion only)


Capital and Liabilities Rs. Assets Rs.
Outstanding expesnses Machines 1,48,000
Wages 37,520 Less: Depreciation 7,400 1,40,600
Direct Charges 600 38,120 Work in Progress:
Capital Account Certified 3,50,000
Profit on Contract Uncertified 18,000
(Refer Working Note No.2) 36,800 3,68,000
Less: Profit in Reserve 32,200
Less: Amount Received 2,80,000 55,800
Unused materials at site 21,620
Working Note
1. Calculation of Written Down Value of Machines on 31st Ashad 2071
Cost of Machine on 1st Baisakh 2071 1,48,000
Less: Depreciation on Rs.1,48,000 for 3 months @ 20% per annum
=1,48,000X20/100X3/12 7,400
Net Written Down Value of Machine 1,40,600

2. Calculation of amount to be transferred from Contract A/c to Profit and Loss A/c
Total surplus in Contract Account as on 31st Ashad 2071:
=Rs.21,620+Rs.1,40,600+Rs.3,68,000-Rs.1,60,000-Rs.1,38,720-Rs.1,48,000-Rs.8,100-
Rs.6,400
=Rs.69,000
Thus Profit to be credited to Profit and Loss Account:
=Rs. 69,000X2/3X2,80,000/3,50,000
=Rs.36,800

b)
Amount Rs.
(i) Agreed value of the plant taken back by the hire vendor:
Price of two plants Rs. 100,000 X 2 = 200,000
Less: Depreciation for the first year @30% - 60,000
140,000
Less: Depreciation for second year @ 30% on Rs. 140,000 - 42,000
Agreed value of the plant taken back by the hire vendor 98,000

(ii) Book value of the plant left with the hire vendor:
Price of one plant Rs. 100,000 X 1 = 100,000
Less: Depreciation for the first year @20% - 20,000
80,000
Less: Depreciation for second year @ 20% on Rs. 80,000 - 16,000
Agreed value of the plant taken back by the hire vendor 64,000

(iii) Profit or Loss to Hire Purchaser:


Book value of one plant as calculated in (ii) 64,000
Book value of two plants taken back = Rs. 64,000 X2 = 128,000

Agreed Value of two plants (i) 98,000


Hence Loss = Rs. 128,000- Rs. 98,000= Rs. 30,000.

(iv) Profit on Resale:


Sales proceeds of plants reprocessed 160,000
P.T.O.
(17)
Less: Value at which plants were taken back= Rs 98,000
Overhauling Charges Rs 40,000 -138,000
Profit on resale 22,000

5.
a) Shree Ganesh Ltd. is a manufacturing company produces durable consumer
goods with an annual turnover of Rs. 100 crores. The company receives
orders from its commission agents all over the country, but goods are
dispatched directly to the customers. The documents including transport bills
are sent through the bank for collection. At the end of the 6th year, it is found
that documents covering the dispatch of goods worth Rs. 10 crores were still
lying with the banks not cleared by the customers even though the normal
collection period of 15 days from the date of dispatch has expired. Should
revenue be recognized in the above case? 5
b) In a production process, normal waste is 5% of input. 5,000 MT of input
were put in process resulting in wastage of 300 MT. Cost per MT of input is
Rs. 1,000. The entire quantity of waste is on stock at the year end. State with
reference to Accounting Standard, how will you value the inventories in this
case? 5
c) The following particulars are available in respect of the business of Lucky
Ltd.:
i) Profits earned for the years:
2012/2013 Rs. 500,000
2013/2014 Rs. 600,000
2014/2015 Rs. 550,000
ii) Normal rate of return = 10%
iii) Capital Employed = Rs. 3,000,000
iv) Present value of an annuity of one rupee for 5 years at 10% discount
rate = Rs. 3.80
v) The profits included non- recurring profits on an average basis Rs.
30,000 a year.
You are required to calculate the value of goodwill of the company: 5
(a) As per five years purchase of super profits:
(b) As per capitalization of super profits method: and
(c) As per annuity method.

Answer:
a) According to NAS - 18 on Revenue, revenue from the sale of goods shall be recognized
when
 the seller of goods has transferred to the buyer the significant risks and rewards of
ownership of the goods;
 the seller retains neither continuing managerial involvement to the degree usually
associated with ownership nor effective control over the goods sold;
 the amount of revenue can be measured reliably.
 It is probable that the economic benefits associated with the transaction will flow to
the entity; and

P.T.O.
(18)
 The costs incurred or to be incurred in respect of the transaction can be measured
reliably.

Since the transport bills were sent through the bank for collection, it may be said that the
seller entity has retained effective control over the ownership of goods. Further since the
documents were not cleared by the customer even after the expiry of the normal period
of collection, there is an uncertainty in the realization of sale proceeds. Hence, revenue
should not be recognized in this case.

b) As per of NAS- 2,” Inventories”, abnormal amount of wasted materials, labour and other
production costs are excluded from cost of inventories and such costs are recognized as
expenses in the period in which they are incurred.
In this case, normal waste is 250 MT and abnormal waste is 50 MT. The cost of 250 MT
will be included in determining the cost of inventories (finished goods) at the year end.
The cost of abnormal waste (50MT x 1,052.63 = Rs 52,632) will be charged to the profit
and loss statement.

Cost per MT (Normal Quantity of 4,750 MT) = 50,00,000 / 4,750 = Rs 1,052.63


Total value of inventory = 4,7000 x Rs 1,052.63 = Rs 49,47,368.

c) Average profits for the last three years


= (Rs. 500,000+ Rs. 600,000+ Rs. 550,000)/3= Rs. 550,000
Less: Non –recurring profits= Rs. 30,000
Recurring Profits= Rs. 520,000
Less: Normal Profits = 10% of Rs. 3,000,000 (capital employed) Rs. 300,000
Super Profit Rs. 220,000

(i) Goodwill at five year's purchase of super profits = Rs. 220,000 X 5= 1,100,000.
(ii) Goodwill as per capitalization of super profits @ 10%= (Rs. 220,000X100)/10= Rs.
2,200,000.
(iii) Goodwill as per annuity method = Rs. 220,000 X Rs. 3.80 = Rs. 836,000.

6. Write short notes on: (5×3=15)


a) Conditions to be satisfied to capitalize the borrowing costs
b) Deferred tax Liabilities
c) General reserve fund
d) Recognition of contingent assets and liabilities
e) Window dressing
Answer:
a) Conditions to be satisfied to capitalize the borrowing costs
Capitalization of borrowing costs as part of the cost of a qualifying asset should commence
only when all the following conditions are satisfied:
 The expenditure is being incurred for the Acquisition, construction or production of a
qualifying asset;
 Borrowing costs are being incurred; and
 Activities that are necessary to prepare the asset for its intended use or sale, (including
any technical or administrative work prior to the commencement of physical construction
but excluding such activities during which no production or development take place) are
in progress.
P.T.O.
(19)

b) Deferred Tax Liabilities


A deferred tax liability is an account on a company's balance sheet that is a result of
temporary differences between the company's accounting and tax carrying values, the
anticipated and enacted income tax rate, and estimated taxes payable for the current year.

According to NAS, A deferred tax liability shall be recognized for all taxable temporary
differences, unless the deferred tax liability arises from the initial recognition of an asset
or liability in a transaction which:
 is not a business combination; and
 at the time of the transaction, affects neither accounting profit nor taxable profit(tax
loss).

c) General Reserve Fund


Section 44 of Bank and Financial Institution Act 2073, states that every Bank and Financial
Institution must maintain a general reserve fund and transfer at least 20 percent of the annual
net profit to such fund every year until the amount of such fund reaches an amount double the
paid-up capital. The amount credited to the general reserve fund of a licensed institution may
not be invested or transferred to any other head without the prior approval of the Nepal Rastra
Bank.

d) Recognition of Contingent Liabilities:


An entity shall not recognize a contingent liability.
1. A contingent liability is disclosed, as required by paragraph 86, unless the possibility
of an outflow of resources embodying economic benefits is remote.
2. Where an entity is jointly and severally liable for an obligation, the part of the
obligation that is expected to be met by other parties is treated as a contingent liability.
The entity recognises a provision for the part of the obligation for which an outflow of
resources embodying economic benefits is probable, except in the extremely rare
circumstances where no reliable estimate can be made.
3. Contingent liabilities may develop in a way not initially expected. Therefore, they are
assessed continually to determine whether an outflow of resources embodying
economic benefits has become probable. If it becomes probable that an outflow of
future economic benefits will be required for an item previously dealt with as a
contingent liability, a provision is recognised in the financial statements of the period
in which the change in probability occurs (except in the extremely rare circumstances
where no reliable estimate can be made).
Recognition of Contingent Assets
An entity shall not recognize a contingent asset.
1. Contingent assets usually arise from unplanned or other unexpected events that give
rise to the possibility of an inflow of economic benefits to the entity. An example is a
claim that an entity is pursuing through legal processes, where the outcome is uncertain.
2. Contingent assets are not recognised in financial statements since this may result in the
recognition of income that may never be realised. However, when the realisation of
income is virtually certain, then the related asset is not a contingent asset and its
recognition is appropriate.
3. A contingent asset is disclosed, as required by paragraph 89, where an inflow of
economic benefits is probable.

P.T.O.
(20)
4. Contingent assets are assessed continually to ensure that developments are
appropriately reflected in the financial statements. If it has become virtually certain that
an inflow of economic benefits will arise, the asset and the related income are
recognised in the financial statements of the period in which the change occurs. If an
inflow of economic benefits has become probable, an entity discloses the contingent
asset (see paragraph 89).

e) Window Dressing
Window dressing is actions taken to improve the appearance of a company's financial
statements. Window dressing is particularly common when a business has a large number
of shareholders, so that management can give the appearance of a well-run company to
investors who probably do not have much day-to-day contact with the business. It may
also be used when a company wants to impress a lender in order to qualify for a loan. If a
business is closely held, the owners are usually better informed about company results, so
there is no reason for anyone to apply window dressing to the financial statements.

Examples of window dressing are:

 Cash. Postpone paying suppliers, so that the period-end cash balance appears higher than
it should be.
 Accounts receivable. Record an unusually low bad debt expense, so that the accounts
receivable (and therefore the current ratio) figure looks better than is really the case.
 Fixed assets. Sell off those fixed assets with large amounts of accumulated depreciation
associated with them, so the net book value of the remaining assets appears to indicate a
relatively new cluster of assets.
 Revenue. Offer customers an early shipment discount, thereby accelerating revenues
from a future period into the current period.
 Depreciation. Switch from accelerated to straight-line depreciation in order to reduce the
amount of depreciation charged to expense in the current period. The mid-month
convention can also be used to further delay expense recognition.
 Expenses. Withhold supplier expenses, so that they are recorded in a later period.

P.T.O.
(21)

Audit & Assurance

P.T.O.
(22)

Suggested
Roll No……………. Maximum Marks - 100

Total No. of Questions- 7 Total No. of Printed Pages- 11

Time Allowed - 3 Hours


Marks
Attempt all questions.

1. As an auditor, give your opinion with explanations on the following cases: (45=20)
a) CA. M Khatri is Chairman of Dotel Consulting Ltd. AGM of KC Textile Ltd. is
planning to appoint Dotel Consulting Ltd. as an auditor for financial year 2073/74.

b) X Ltd. owns five motors that it uses in its business as property, plant, and equipment.
The entity intends to carry three motors under the cost model and the remaining two
under the revaluation model.

c) A Ltd. company has been valuing its closing inventories under the weighted-average
cost method. In 2072/73, management decided to change its accounting policy
relating to the valuation of inventories to first-in, first-out (FIFO) method since it
was considered to more accurately reflect the usage and flow of inventories in the
economic cycle. Is the change in accounting policy justified?

d) M/s Merita Impex is engaged in the business of manufacturing and trading of


musical instruments. A sum of Rs. 5 lakhs, received from an insurance company as
an insurance claim for loss of goods in transit costing Rs. 4 lakhs, is credited to the
purchase account.

Answer:

a) Section 112 of Companies Act, 2063 has explained the disqualification of auditor as
follows:

(a) A director, advisor appointed with entitlement to regular remuneration or cash


benefit, a person or employee or worker involved in the management of the company
or a partner of any of them or an employee of any of such partners or a close relative of
a director or partner, out of them, or an employee of such relative;

(b) A debtor who has borrowed moneys from the company in any manner, or a person
who has failed to pay any dues payable to the company within the time limit and is in
such arrears or close relative of such person;

(c) A person who has been sentenced to punishment for an offense pertaining to audit
and a period of Three years has not elapsed thereafter;

(d) A person who has been declared insolvent;

(e) A substantial shareholder of the company or a shareholder holding one percent or


more of the paid up capital of the company or his/her close relative;

P.T.O.
(23)
(f) A person who has been sentenced to punishment for an offense of corruption, fraud
or a criminal offense involving moral turpitude and a period of five years has not
elapsed thereafter;
(g) A person referred to in Sub-section (3) of Section 111;
(h) In the case of a public company, any person who works, whether full time or part
time, for any governmental body or anybody owned fully or partly by the Government
of Nepal or any other company or a partner of such person or a person who is working
as an employee of such partner or a person who is authorized to sign any documents or
reports to be prepared by the management of the company;
(i) A company or corporate body with limited liability;
(j) A person having interest in any transaction with the company or his/her close
relative or a director, officer or substantial shareholder of another company having any
interest in any transaction with the company.

The auditor shall, prior to his/her appointment ,give information in writing to the
company that he/she is not disqualified pursuant to Subsection(1).
Where any auditor becomes disqualified to audit the accounts of a company or there
arises a situation where he/she becomes disqualified for appointment or can no longer
continue to act as an auditor of the company, he/she shall immediately stop performing
audit which is required to be performed or is being performed by him/her and give
information thereof to the company in writing.

The audit performed by an auditor who has been appointed in contravention of this
Section shall be invalid.
Thus, even though M Khatri is a Chartered Accountant, company cannot be appointed as
auditor of KC Textile Limited.

b) NAS 16 permits an entity to choose between either the cost model or the revaluation model.
It does not allow an entity to apply two different models for the same class of property, plant,
and equipment. Therefore, the company is not allowed to carry three motors under the cost
model while carrying two under the revaluation model, since the standard categorically
prohibits such an accounting treatment.

c) As per NAS 8, an entity shall change an accounting policy only if the change
• Is required by a NFRS; or
• Results in the financial statements providing reliable and more relevant information about
the effects of transactions, other events, or conditions on the entity‟s financial position,
financial performance, or cash flows.
• Reflect the economic substance of transactions, other events and conditions, and not merely
the legal form;
• are neutral, i.e. free from bias;
• are prudent and are complete in all material respect.

In this case, the company has changed its accounting policy in order to accurately reflect the
usage and flow of inventories in the economic cycle. The change in accounting policy is
justified.

d) All items of income and expense which are recognized in a period should be included in the
determination of net profit or loss for the period. The claim for loss of goods in transit is
arising out of ordinary activities of the Impex as a part of its normal course of business.

P.T.O.
(24)
Since the company received Rs. 5 lakhs against goods lost in transit costing Rs. 4 lakhs, there
is profit of Rs. 1 lakh from insurance claim which should be separately recognized in P/L.

Further cost of goods sold should be reduced by Rs. 4 lakhs (i.e. goods lost in transit) so that
gross profit figure is not affected as sales revenue did not arise for these goods.

2. Give your comments on the following cases: (45=20)


a) CA. Oli has been appointed as Tax auditor for the year 2072/73 by ABC Private
Limited. He finds that the company has wrongly claimed excess carry-forward loss
of Rs.1 crore in the previous year‟s (2071/72) tax return in which he was not
associated at all. This has no effect in the tax return for the year 2072/73.
b) CA. Ram was appointed as auditor of Alpha Traders Ltd. for 2071/72 for a fee of
Rs.300,000. The turnover for that year was Rs.3 billion, 90 % of the revenue was
from sale of electronic products of “Samsung” for which the company is authorized
distributor. In the early month of 2072/73, the authorized distributorship of
“Samsung” was terminated. Alpha Traders has requested CA Ram to accept the audit
of 2072/73 for a fee of Rs.100,000 as there would be negligible transactions as
compared to previous year and the time to conduct that audit shall be only 1/3 as
compared to previous year audits.
c) LMN Ltd. deals in electronic goods and has 4 warehouses at different locations in
Nepal out of which2 warehouses at Birgunj Custom borders. The major stocks are
generally supplied from warehouse at Birgunj. LMN Ltd. appointed M/s OPQ & Co.
to conduct its audit for the financial year 2072/73. Due to earthquake and subsequent
strike for 3-4 months, the warehouse at Birgunj was inaccessible and the auditors
were not able to conduct physical verification.
d) CA. Renu was working as General Manager of I Ltd. till financial year 2072/73.
With effect from the 1stShrawan 2073 she resigned the I Ltd. and started working as
practicing Chartered Accountant from the name of "Renu& Associates". The Annual
General Meeting of I Ltd. appointed her as statutory auditor for the financial year
2073/74.
Answer:

a) Section 5.8 of Code of Ethics deals with tax practice by professional accountants. It states that
when a professional accountant learns of a material error or omission in a tax return for previous
year (with which the professional accountant may or may not have been associated), the client
should be advised promptly to disclose the fact to tax authority. However, CA Oli is not obliged
to inform the tax authorities.

CA Oli should inform the client that it is not possible to act for them in connection with that
return or other related information submitted to the authorities. Since, the prior error does not
have any effect in tax return for the year in which he is associated, he may continue professional
relation with client but all reasonable steps should be taken to ensure that the error is not
repeated in subsequent returns.

b) As per Section 10.6 of Code of Ethics, It is not improper for a professional accountant in public
practice to charge a client a lower fee than has previously been charged for similar services,
provided the fee has been calculated in accordance with the factors referred to in paragraphs
10.2 through 10.4. It means professional accountant in public practice shall not quote lower
P.T.O.
(25)
fees than the previously paid fees without justifiable reasons of reduction in the volume of
transactions or time involvement. In the given case above, it is seen that the company has lost
major distributorship securing 90% of its revenue and hence the work volume and time
involvement is expected to decrease drastically. Hence, CA. Ram may accept the lower fees as
compared to previous year on those justifiable grounds. However, it should be taken care that
the quality of work shall not be impaired and that due care will be applied to comply with all
professional standards and quality control procedures.

c) In some cases, attendance at physical inventory counting may be impracticable. This may be
due to factors such as the nature and location of the inventory, for example, where inventory is
held in a location that may pose threats to the safety of the auditor. The matter of general
inconvenience to the auditor, however, is not sufficient to support a decision by the auditor that
attendance is impracticable. Further, as explained in NSA 200 “Overall Objectives of the
Independent Auditor and the Conduct of an Audit in Accordance with Nepal Standards on
Auditing”, the matter of difficulty, time, or cost involved is not in itself a valid basis for the
auditor to omit an audit procedure for which there is no alternative or to be satisfied with audit
evidence that is less than persuasive. Further, where attendance is impracticable, alternative
audit procedures, for example, inspection of documentation of the subsequent sale of specific
inventory items acquired or purchased prior to the physical inventory counting, may provide
sufficient appropriate audit evidence about the existence and condition of inventory.

In some cases, though, it may not be possible to obtain sufficient appropriate audit evidence
regarding the existence and condition of inventory by performing alternative audit procedures.
In such cases, NSA 705 on Modifications to the Opinion in the Independent Auditor‟s Report,
requires the auditor to modify the opinion in the auditor's report as a result of the scope
limitation.

d) As per Para 8.4 of the code of ethics of the Institute of the Chartered Accountants of Nepal ;
when professional accountants in public practice are or were, within the period under current
review or immediately preceding an assignment: (a) a member of the board, an officer or
employee of a company; or (b) a partner of, or in the employment of, a member of the board or
an officer or employee of a company; they would be regarded as having an interest which could
detract from independence when reporting on that company.
It is common practice to prohibit professional accountants in public practice in such situations
being appointed as auditors of the companies concerned. It is also clearly desirable that they
should not accept from such companies other assignments on which an opinion is required.

In the light of aforesaid provision of ICAN Code of Ethics, CA. Renu should not accept the
appointment for statutory auditor for the FY 2073/74.

3. Answer the following: (35=15)


a) NSA 315 Identifying and Assessing the Risks of Material Misstatement through
Understanding the Entity and Its Environment describes the five components of an
entity‟s internal control. Briefly explain the five components of an entity‟s internal
control.
b) Describe substantive procedures you should perform in audit to confirm the
completeness and accuracy of payroll expense.
c) Define audit risk and the components of audit risk.

Answers:

P.T.O.
(26)
a) NSA 315 Identifying and Assessing the Risks of Material Misstatement through
Understanding the Entity and Its Environment considers the components of an entity‟s
internal control. It identifies the following components:
Control environment
The control environment includes the governance and management functions and the
attitudes, awareness, and actions of those charged with governance and management
concerning the entity‟s internal control and its importance in the entity. The control
environment sets the tone of an organization, influencing the control consciousness of its
people.
The control environment has many elements such as communication and enforcement of
integrity and ethical values, commitment to competence, participation of those charged with
governance, management‟s philosophy and operating style, organizational structure,
assignment of authority and responsibility and human resource policies and practices.
Entity’s risk assessment process
For financial reporting purposes, the entity‟s risk assessment process includes how
management identifies business risks relevant to the preparation of financial statements in
accordance with the entity‟s applicable financial reporting framework. It estimates their
significance, assesses the likelihood of their occurrence, and decides upon actions to
respond to and manage them and the results thereof.
Information system, including the related business processes, relevant to financial
reporting, and communication
The information system relevant to financial reporting objectives, which includes the
accounting system, consists of the procedures and records designed and established to
initiate, record, process, and report entity transactions (as well as events and conditions) and
to maintain accountability for the related assets, liabilities, and equity.
Control activities relevant to the audit
Control activities are the policies and procedures which help ensure that management
directives are carried out. Control activities, whether within information technology or
manual systems, have various objectives and are applied at variousorganisational and
functional levels.
Monitoring of controls
Monitoring of controls is a process to assess the effectiveness of internal control
performance over time. It involves assessing the effectiveness of controls on a timely basis
and taking necessary remedial actions. Management accomplishes the monitoring of
controls through ongoing activities, separate evaluations, or a combination of the two.
Ongoing monitoring activities are often built into the normal recurring activities of an entity
and include regular management and supervisory activities.
b) Payroll substantive procedures
 Agree the total wages and salaries expense per the payroll system to the trial balance,
investigate any differences.
 Cast a sample of payroll records to confirm completeness and accuracy of the payroll
expense.
 For a sample of employees, recalculate the gross and net pay and agree to the payroll
records to confirm accuracy.
 Re-perform the calculation of statutory deductions to confirm whether correct
deductions for this year have been made in the payroll.

P.T.O.
(27)
 Compare the total payroll expense to the prior year and investigate any significant
differences.
 Review monthly payroll charges, compare this to the prior month and budgets and
discuss with management for any significant variances.
 Perform a proof in total of total wages and salaries, incorporating joiners and leavers
and the annual pay increase.
 Compare this to the actual wages and salaries in the financial statements and
investigate any significant differences.
 Select a sample of joiners and leavers, agree their start/leaving date to supporting
documentation, recalculate that their first/last pay packet was accurately calculated and
recorded.
 Agree the total net as pay per the payroll records to the bank transfer listing of
payments and to the cashbook.
 Agree the individual wages and salaries as per the payroll to the personnel records for
a sample.
 Select a sample of weekly overtime sheets and trace to overtime payment in payroll
records to confirm completeness of overtime paid.
c) Audit risk and its components
Audit risk is the risk that the auditor expresses an inappropriate audit opinion when the
financial statements are materially misstated. Audit risk is a function of two main
components, being the risk of material misstatement and detection risk. Risk of material
misstatement is made up of a further two components, inherent risk and control risk.
Inherent risk is the susceptibility of an assertion about a class of transaction, account balance
or disclosure to a misstatement which could be material, either individually or when
aggregated with other misstatements, before consideration of any related controls.
Control risk is the risk that a misstatement which could occur in an assertion about a class of
transaction, account balance or disclosure and which could be material, either individually or
when aggregated with other misstatements, will not be prevented, or detected and corrected,
on a timely basis by the entity‟s internal control.
Detection risk is the risk that the procedures performed by the auditor to reduce audit risk to
an acceptably low level will not detect a misstatement which exists and which could be
material, either individually or when aggregated with other misstatements. Detection risk is
affected by sampling and non-sampling risk.
4. Answer the following: (35=15)
a) What are the basic needs to be met to achieve objectives of accounting profession as
per code of ethics?
b) Write briefly on the composition and function of Disciplinary Committee under
ICAN Act, 2053.
c) What due care should a professional accountant in public practice adopt while
handling client‟s monies?

Answer:

a) The code of ethics recognizes that the objectives of the accountancy profession are to work to
the highest standards of professionalism, to attain the highest levels of performance and
generally to meet the public interest requirement set out above. These objectives require four
basis needs to be met:
P.T.O.
(28)

Credibility: In the whole of society, there is need for credibility in information and information
systems

Professionalism: There is need for individuals who can be clearly identified by clients,
employers and other interested parties as professional persons in the accountancy field.

Quality of Services: There is a need for assurance that all services obtained from a professional
accountant are carried out to the highest standards of performance

Confidence: Users of the services of professional accountants should be able to feel


confident that there exists a framework of professional ethics which govern the
provision of those services.

b) As per Section 14 of ICAN Act, a disciplinary committee comprising of following members


shall be constituted:

a. FCA member designated by council from amongst elected CA council member-


Chairman
b. three persons nominated by the council from amongst the council members- Member
c. Two persons nominated by the council amongst the members-Member
d. One person nominated by the Auditor General-Member
The main function of the disciplinary committee is to recommend the council to take necessary
actions after investigations upon complaints lodged against any action contrary to the ICAN Act
or regulations, by-laws or code of conduct framed under the ICAN Act rendered by any member
, or the institute receives any information of such kind.
The chairman or members are not allowed to attend any meeting that hears complaint against
the Chairman or member of disciplinary committee. The Disciplinary committee has the
authority, similar to a Judicial Court, in respect of summoning concerned person and
investigating evidences and witnesses.
The disciplinary committee recommends to the council, along with its opinion and finding, for
necessary action against a member.

c) As per ICAN‟s Code of Ethics, a professional accountant in public practice should:

a. not hold client‟s money if there is reason to believe that they were obtained form, or
are to be used for, illegal activities.
b. keep such client‟s monies separately from personal or firms‟s monies
c. use only for the intended purpose and always be ready to account for those monies to
any persons entitled
d. maintain separate bank account for such monies and any monies received should be
deposited without delay to credit of a client account.
e. monies should only be drawn from the client account on instruction form the client
f. fees may be drawn from client‟s monies only after intimating the amount of fees and
client has agreed to such withdrawal.
g. provide a statement of account to client at least once a year
h. if the monies remain for a significant time, place such monies in interest bearing
account with concurrence from such client. All the interest earned should be credited
to the client account.

P.T.O.
(29)

5. Answer the following: (25=10)


a) Discuss about the procedure mentioned in Companies Act, 2063 regarding
appointment of auditors.
b) Discuss about provision relating to removal of appointed auditor as per Companies
Act, 2063.
Answer:
a) As per Section 110 of Companies Act 2063, every company shall appoint an auditor under to
have its accounts audited. In cases where any company has a branch office outside Nepal, the
auditor so appointed may also audit the accounts of that branch office except as otherwise
provided in the prevailing law of the country where such branch office is situated. Similarly,
as persection111, the auditor of a company shall be appointed, from amongst the auditors
licensed to carry out audit under the prevailing law, by the general meeting subject to
provisions contained in chapter 18 of this act, in the case of a public company, and, in
accordance with the provision as contained in the memorandum of association, articles of
association or consensus agreement, any failing such provision, by the general meeting, in the
case of a private company; and his/her name shall be forwarded to the Office within fifteen
days from the date of such appointment.
Provided, however, that the board of directors may appoint the auditor prior to the holding of
the first annual general meeting.
The auditor appointed shall hold office only until the next annual general meeting. No auditor
or his/her partner or ex-partner or employee or ex-employee shall be appointed as auditor for
more than three consecutive terms to perform the audit of a public company.
Provided, however, that this restriction shall not apply to any partner who ended partnership
or any employee who left the service of such audit or three years before.
As per section 113, Where the annual general meeting of a company fails to appoint an
auditor for any reason or where the annual general meeting itself cannot be held or where the
auditor appointed pursuant to this Act ceases to continue his/her office for any reason, the
Office may, at the request of the board of directors of the company, appoint another auditor.

b) As per section 119 of Companies act 2063, no auditor appointed pursuant to provisions laid
down in this act shall be removed pending the completion of audit of accounts of any
financial year for which he/she was appointed as the auditor. Notwithstanding anything
contained in this section, if any auditor breaches the code of conduct of auditors or does any
act against the interest of the company which has appointed him as the auditor or commits
any act contrary to the prevailing law, such auditor may be removed through the same
process whereby he/she was appointed as auditor, by giving prior information to the ICAN,
and with the approval of the regulatory authority, if any authorized by the prevailing law for
the regulation of business of the company concerned , and failing such authority, with the
approval of the Office.
While removing an auditor, the auditor shall be provided with a reasonable opportunity to
defend him/herself.

6. Write short notes on the following: (42.5=10)


a) Benefits of implementing IFRS or equivalent national standards
b) Government audit in Nepal
c) Concept of materiality
P.T.O.
(30)
d) Written representation

Answers:
a) Benefits of implementing IFRS or equivalent national standards
There are many benefits of implementing IFRS or its national equivalent financial reporting
standards which can be broadly divided into 3 main parts – Economy, Investors and the
Industry.
Benefits to the Economy - As the market expands globally, the need for a global standard is
also increasing. Implementation of IFRS or equivalent standards facilitates the maintenance
of orderly and efficient capital markets and also helps to increase the capital formation and
thereby economic growth.
Benefits to the Investors - Investors who are willing to invest abroad want information
which is more relevant, reliable, timely and comparable across various jurisdictions.
Financial statements prepared using a common set of accounting standards help investors
better understand, at a little cost, the investment opportunities as opposed to financial
statements prepared using a different set of national accounting standards.
Benefits to the Industry - the Industry would be able to raise capital from foreign markets at
a lower cost if it can create confidence in the minds of the foreign investor that their
financial statements comply with globally accepted accounting standards.
b) Government Audit in Nepal
Government audit is as old as history of states. The concept and scope of government audit
has developed in tune with political, social and economic development of the countries.
Government audit aims to promote good governance through an independent, efficient and
effective audit services. To carry out audit in an independent manner, some independent
institutions are set up through constitutional or legal provisions. Such institution is
empowered to carry out audits of all receipts, expenditure and other matters as specified in
mandate from various aspects such as regularity, economy, efficiency, and effectiveness,
and propriety.
In Nepal, government audit is performed by an independent constitutional body, i.e. Office
of Auditor General (OAG). The Constitution of Nepal (2072) provides power to the auditor
General, who is appointed on the recommendation of the Constitutional Council, of
conducting audit of all public sector entities with due regards to regularity, economy,
efficiency, effectiveness and proprietary.
As per the Audit Act, 2048, the Auditor General may conduct final audit of the financial
activities and other activities relating thereto of the offices, bodies or organization under its
jurisdiction, either in detail or sporadically or a random basis and present the facts obtained
there from make critical comments thereon and submit its reports.
The Auditor General submits his/her annual report to the president and which is caused by
the president to be tabled at parliament for discussion through prime minister.
c) Concept of Materiality
The concept of materiality is applied by the auditor both in planning and performing the
audit, and in evaluating the effect of identified misstatements on the audit and of
uncorrected misstatements, if any, on the financial statements.
P.T.O.
(31)

The „Framework for the Preparation and Presentation of Financial Statements‟ states that
information is material if its omission or misstatement could influence the economic
decisions of users taken on the basis of the financial statements. Materiality depends on the
size of the item or error judged in the particular circumstances of its omission or
misstatement. Thus, materiality provides a threshold or cut-off point rather than being a
primary qualitative characteristic which information must have if it is to be useful.
The decision to judge the materiality of the item whether in the aggregation of items,
presentation or classification of items shall depend upon the judgment of preparers of the
account on the circumstances of the particular case.

d) Written Representation
This refers to a written statement by management provided to the auditor to confirm certain
matters or to support other audit evidence. Written representations in this context do not
include financial statements, the assertions therein, or supporting books and records. Written
representations are necessary information that the auditor requires in connection with the
audit of the entity‟s financial statements. Accordingly, similar to responses to inquiries,
written representations are audit evidence. Although written representations provide
necessary audit evidence, they do not provide sufficient appropriate audit evidence on their
own about any of the matters with which they deal. Furthermore, the fact that management
has provided reliable written representations does not affect the nature or extent of other
audit evidence that the auditor obtains about the fulfillment of management‟s
responsibilities, or about specific assertions.
The auditor shall request written representations from management with appropriate
responsibilities for the financial statements and knowledge of the matters concerned.
Written Representations about management‟s responsibilities include:
 Preparation of the Financial Statements;
 Information Provided and Completeness of Transactions; and
 Other responsibility

7. Distinguish between: (25=10)


a) Audit plan and audit programme
b) Reasonable assurance and limited assurance

Answers:
a) Audit Plan and Audit Program
The Distinction between Audit Plan and Audit Program are outlined as follows:
Audit plan is described as developing a general strategy and a detailed approach for the
expected nature, timing and extent of the audit. The auditor plans to perform the audit in
efficient and timely manner where as audit program is the step and guidance and works as a
tool for performing/implementing audit at the execution level. The distinctions of those two
are:
 Audit plan is prepared before preparing audit program.
 Audit plan is border scope than audit program.
P.T.O.
(32)
 Audit plan assists acquiring knowledge of clients business and concentrating on
risk areas which will help for preparing effective audit program.
 Audit plan is generally prepared by senior auditors and program may be prepared
by juniors based on plan and duly approved by seniors.
 Audit plan focuses on broader area where as programme breaks them into small
area or in form of audit questions, checklists or time frames etc.

b) Reasonable Assurance and Limited Assurance


Reasonable Assurance is a concept relating to the accumulation of the audit evidence
necessary for the auditor to conclude that there are no material misstatements in the
financial statements taken as a whole. An audit in accordance with Nepal Standards on
Auditing is designed to provide reasonable assurance that the financial statements taken as a
whole are free from material misstatement. Audit cannot give an absolute assurance due to
certain inherent limitations that affect the ability of detect material misstatement which may
arise generally due to use of testing, limitation of accounting and control system and mostly
due to the fact that audit evidence is persuasive rather than conclusive.
The objective of a reasonable assurance engagement is a reduction in assurance engagement
risk to an acceptably low level in the circumstances of the engagement as the basis for a
positive form of expression of the practitioner‟s conclusion. In other words, the practitioner
expresses conclusion in the positive form.

The objective of a limited assurance engagement is a reduction in assurance engagement risk


to a level that is acceptable in the circumstances of the engagement, but where that risk is
greater than for a reasonable assurance engagement, as the basis for a negative form of
expression of the practitioner‟s conclusion. In other words, the practitioner expresses
conclusion in the negative form.

P.T.O.
(33)

Corporate & Other Laws

P.T.O.
(34)

Suggested Answer
Marks
Attempt all questions.
1. Answer the following questions:
a) Sri Nagar Construction Company couldn't pay loan amount Rs. 50 Lakh
that is taken from a bank. Bank may proceed to reimburse its investment
with interest and other amount as agreed and at any time in accordance
with its rules. Mr. Sanjay Sharma, being a shareholder of that company,
holding Rs. 20 Lakh amount share, asks you on the matter about the
liability of such loan. Give your opinion to him about the extent of liability
of such loan of a shareholder referring the concept and provision under the
Companies Act, 2063. 5
b) The board of directors of Nepal Cell Pvt. Ltd. passed a resolution to
purchase 4G mobile network from Vodafone Inc. for $50,000,000.
Accordingly, a deed of contract drafted and duly signed. Afterwards, some
of the shareholders of the Nepal Cell file a case against the validity of
contract as four out of seven directors found disqualified. Answer the
following questions:
i) What are the disqualifications of board of directors? 4
ii) Is the contract valid? 1
c) Total paid up capital of M/S Siddhartha Movers Pvt. Ltd. is Rs. 100,000.
CA Pradhalad Pradhan a chartered accountant holds 900 shares of Rs. 100
per shares. The annual general meeting of the company appoints CA
Pradhalad Pradhan as auditor of the company for fiscal year 2072/73.
Answer the following question with relevant provisions of the Companies
Act, 2063.
i) Is the appointment of CA Pradhalad Pradhan valid as per provision of
the Companies Act, 2063? 3
ii) If his appointment is invalid by the above reason or any other reason,
what steps must be taken by him? 2
d) A resolution presented to the meeting of the Board of Directors of Ace
Finance Ltd. to appoint Mr. Raman Pandey as an independent Board of
Director where the directors are divided regarding his qualification as he
was convicted for six months jail punishment in a case of assault. Give
your advice to the company that:
i) What persons shall not be eligible to be appointed as an independent director? 3
ii) Is Mr. Pandey eligible as he was convicted for six months in jail? 2
e) Sagarmatha Ltd. a public limited company is making profit for the last two
years, the Board of Directors (BOD) of the Company wants to issue
premium shares. The BOD consults you regarding the pre-conditions of
and the procedures for issuing premium shares. Suggest the BOD of the
Company about the pre-conditions and the procedures for the issue of
premium shares under the Companies Act, 2063, 5

P.T.O.
(35)
Answer:
a) A company, being a legal person, is the owner of its assets and is liable to its all
debts. Company is a separate legal entity. It has independent personality.
Companies Act, 2063, in its section 8, has made possible to establish only the
company having limited liability. It says that the liability of a shareholder of a
company incorporated under this Act, in respect of its transaction, shall be
limited only to the maximum value of shares which he has subscribed or
undertake to subscribe.
In given case it should be examined that whether the loan is within the authority
of the company as mentioned in MOA and AOA of the Company.
In case of loan is within the authority of company. For above referred case, it is
clear that the first liability of the debt is of the company. Shareholders of a
company have secondary liability to the extent of the investment he/she made.
There is no direct right and interest of shareholders to the property of the
company. Due to such reason, members of a company are not compelled to bear
debt liability. Member's liability is confined only on to their shares to which
they have paid or they have to pay. Shareholders are eligible to take benefit in
accordance with the investment in their shares and are in a position to bear
losses only losing their investment not further more.
However in case the loan is not within the authority of company or ultra virus,
the protection of limited liability in not applicable in the case of lifting the
corporate veil. Accordingly, the directors or shareholders of company are made
personally liable for their work carried out by using the veil or mask of
company to evade tax or commit fraud. Likewise, the veil of company is lifted
out in need of identifying the real controller of company whether he is alien
enemy.

b)
i) Section 89(1) of the Companies Act, 2063 has listed the various circumstances
where a person becomes disqualified to be a director. It states that any of the
following persons shall not be eligible to be appointed to the office of director:
a. Person who is below Twenty one years of age, in the case of a public
company;
b. Person who is of unsound mind or is insane;
c. Person who is a declared insolvent and a period of five years has not
lapsed;
d. Person who is convicted of an offense of corruption or of an offense
involving moral turpitude. Provided, that in the case of a private company,
a period of three years has not lapsed from the date of such sentence,
e. Person who is convicted of an offense of theft, fraud, forgery or
embezzlement or misuse of goods or funds entrust to him/her, in an
authorized manner, and sentenced in respect thereof, a period of three year
has not elapsed from the expiry of the sentence; etc.
f. Who has personal interest of any kind in the business or any contract or
transaction of the concerned company;

P.T.O.
(36)
g. Who is already a director, substantial shareholder, employee, auditor or
adviser of another company having similar objectives or has personal
interest of any kind in such company;
Provided, however, that such person of a private company may become a
director of another private company having similar objectives.

ii) Further, Section 106 of the Companies Act, 2063 provided that if it is
afterwards discovered that any provision under this Act has not been complied
with in respect of the appointment of any director, acts already done by such
director him /her before the discovery of such fact shall not be rendered invalid
by that fact. Therefore, the resolution passed by the directors though
disqualified, is valid and contract with Vodafone Ltd. valid and binding.

c)
i) Section 112 of the Companies Act, 2063 makes provision regarding the
disqualification of auditor.
Section 112 (1) states that none of the following persons or the firms or
companies in which such persons are partners shall be qualified for
appointments as an auditor.This section mentions various grounds for
disqualification, among them a substantial shareholder of the company or a
shareholder holding one percent or more than the paid-up capital of the
company or his close relative is one ground for disqualification to become an
auditor.
In the given case CA Pradhalad Pradhan a chartered accountant holds 90
percent shares of M/S Siddhartha Movers Pvt Ltd, so as per the section 112 (1)
of the Companies Act, 2063 CA Pradhan is disqualified to be appointed as the
auditor of M/S Siddhartha Movers Pvt Ltd.
Appointment of CA Pradhalad Pradhan as auditor of the company for fiscal
year 2072/73 by annual general meeting is not valid.
ii) Section 112 (3) of the Company Act, 2063 makes provision regarding steps
must be taken by auditor appointed, whose appointment is invalid as per Act.

Section 112 (3) Where any auditor becomes disqualified to audit the accounts
of a company or there arises a situation where he becomes disqualified for
appointment or can no longer continue to act as an auditor of the company, he
shall immediately stop performing audit which is required to be performed or is
being performed by him and give information thereof to the company in
writing.
So in the given case CA Pradhalad Pradhan shall immediately stop performing
audit which is required to be performed or is being performed by him and give
information thereof to the company in writing

d)
i) Pursuant to Sub-section (2) of Section 86 of the Companies Act, 2063, every
public company shall have a board of directors consisting of a minimum of
three and a maximum of eleven directors. And in forming the board of directors
at least one independent director, in the case of the number of directors not
exceeding seven, and at least two independent directors, in the case of the
number of directors exceeding seven, shall be appointed from amongst the
persons who have the knowledge as prescribed in the articles of association of

P.T.O.
(37)
the company and gained knowledge and experience in the subject related to the
business of the company concerned.
Pursuant to Sub-section (2) of Section 89 of the Companies Act, 2063, the
following persons shall not be eligible to be appointed to the office of
independent director:
i. A person as referred to in Sub-section (1) of section 89.
ii. A shareholder of the concerned company.
iii. Who has not obtained at least bachelor degree in a subject that is related to
the business to be carried on by the concerned company and gained at least
ten years of experience in the related field or in the company management
affairs or who has not obtained at least bachelor degree in finance,
economics, management, accounts, statistics, commerce, trade or law and
gained at least ten years of experience in the related field.
iv. An officer, auditor or employee of the concerned company or a period of
three years has not lapsed after his/her retirement from any such office.
v. The close relative of the office of the concerned company;
vi. An auditor of the concerned company or his/her partner.

ii) Pursuant to Sub-section 1 of Section 89 of the Companies Act, 2063, A person


who is convicted of an offense of corruption or of an offense involving moral
turpitude will not be eligible to be appointed as director in public company.
Similarly, a person convicted of an offense of theft, fraud, forgery or
embezzlement or misuse of goods or funds entrust to him/her, in an authorized
manner, and sentenced in respect thereof, a period of three year has not elapsed
from the expiry of the sentence he cannot be appointed as an independent
director of a public limited company. In the above case, Mr. Pandey was
convicted in an offense of assault which does not involve moral turpitude and
does not come within the categories as mentioned in the Act. Therefore, he can
be appointed as an independent director of the company.

e) Section 2(la) of the Companies Act, 2063 defines the term “premium share” as
a share issued by a company as to sell it for a value in excess of its face value.
Section 29 of the Companies Act, 2063 provides the pre conditions of and
procedures to issue premium shares, which are as follows:
(1) any company fulfilling the following conditions may, with the prior
approval of the Company Registrar‟s Office (Office), issue shares at a
premium:
a) the company has been making profits and distributing dividends for three
consecutive years,
b) the company's net worth exceeds its total liabilities,
c) the company's general meeting has decided to issue shares at a premium

As Sagarmatha Ltd has been making profit for the last two years only, it doesn't
meet the pre condition mentioned above under section 29(1)(a) of the
Companies Act, 2063.Therefore, Sagarmatha Ltd cannot issue premium shares.

2. Answer the following questions:


a) Nepal Rastra Bank Act, 2058 has provided certain special facilities and
privileges to Nepal Rastra Bank itself. What are these privileges and
facilities? Explain. 5

P.T.O.
(38)
b) Local Development Bank Limited has appointed two public directors in its
annual general meeting held on Baishak 2, 2072. However, these two
directors did not provide their particulars of disclosures as required by the
Bank and Financial Institutions Act, 2063 till the end of Baishak 2072
saying that they were unaware with the disclosure parts. Is it really
required to disclose the particulars of directors as required by the Act? If
so what are the matters to be disclosed by each director to the bank?
Explain referring the provisions of thisAct. 5
c) Who is chief executive of a bank and financial institution? Mention the
functions, duties and powers of his as provided in the Bank and Financial
Institution Act, 2063? 5
Answer:

a) Section 8 of Nepal Rastra Bank Act, 2058 has provided the following privileges
and facilities to Nepal Rastra Bank which are as follows:
Notwithstanding anything contained in the prevailing laws, Nepal Rastra Bank
(NRB) shall be entitled to the following privileges and facilities:
(a) Exemption from all types of taxes, fees and charges on the income, capital
transactions, houses, land, assets etc.;
(b) No requirement for the payment of registration fee for registration of the
deeds of loan or refinance to be given by NRB;
(c) No requirement of revenue stamps on any of the documents relating to NRB;
(d) There would be no tax, fee, charge, duty on the export and import of bank
notes, coins, gold, silver and the paper, metal, chemicals, and other materials
to be used for printing bank notes and minting coins.

b) Yes, it is mandatory to disclose the particulars by the newly appointed Directors


of the Local Development Bank Ltd. under Section 22 of the Banks and
Financial Institutions Act, 2063 as it has defined the requirement of disclosure as
follows:
(1) Every director shall, no later than seven days after assuming the office of
director, disclose in writing to the bank or financial institution the following
matters:
(a) If he or she or any of his or her family members has entered into or going to
enter into any kind of contract with the concerned bank or financial
institution, details thereof;
(b) If he or she has any kind of interest in the appointment of the chief executive,
managing director, secretary, auditor and general manager, details thereof;
(c) Particulars of such shares or debentures in the concerned bank or financial
institution or in its holding or subsidiary company as subscribed by him or
her or by his or her family;
(d) If he or she is a director of any company, details thereof;
(e) If any member of his or her family is working as an officer of the bank or
financial institution, details thereof;
(f) Such other details prescribed by Rastra Bank as required to be disclosed by
the director to the Board.
(2) In making disclosure pursuant to Sub-section (1) above, a copy of the written
agreement, if any, concluded between the directors or his or her family
P.T.O.
(39)
member shall be submitted, and failing such agreement, substantial and
necessary matters concerning the transaction or financial interest or
involvement shall be set out.

c) The chief executive of a bank or financial institution is the chief administrator


thereof. He/she is appointed by the Board of the bank or financial institution
subject to the BAFIA, 2063 and the memorandum of association and articles of
association. The qualifications to be required for the chief executive is provided
in this act that is at least bachelor's degree in any subject and have gained at least
five years of experience of the office of director or of the executive level in
banking, financial or corporate sector. The tenure of office of the chief executive
is maximum of four years and be re-appointed. The remuneration and other
conditions of service of the chief executive is as prescribed by the Board. He/ she
should responsible to the Board for all of his work.

The functions, duties and powers of chief executive are as follows as given in the
section 27 of the Bank and Financial Institution Act, 2063:
(a) To implement the decisions of the Board and supervise and control the
activities and transactions of the bank or financial institution, subject to
this Act and the memorandum of association and articles of association;
(b) To prepare annual budgets and action plans of the bank or financial
institution and present them before the Board for approval;
(c) To manage necessary human resources, subject to the personnel bye-laws
of the bank or financial institution;
(d) To implement, or cause to be implemented, the decisions of the Board and
the General Meeting, and the directives of the Rastra Bank;
(e) To present on time all such particulars, documents, decisions, etc. as are
required to be submitted by the bank or financial institution to the Rastra
Bank or any other body, subject to this Act and the memorandum of
association and articles of association;
(f) To perform such other functions as may be prescribed in relation to the
operation of the bank or financial institution.

3. Answer the following questions: (2×5=10)


a) What are the liabilities of the concerned persons for the matter referred to
in the prospectus as per the Securities Act, 2063?
b) Discuss briefly the fundamental principles of insurance.

Answer:
a) The liabilities of the concerned persons for the matters referred to in the
prospectus are laid down under Section 33 of the Securities Act, 2063, which are
as follows:
1) The concerned body corporate and the director signing a prospectus and the
expert preparing such a prospectus shall be personally and collectively liable
for the truth of the details and documents underpinning the information set
down in the prospectus submitted to Securities Board of Nepal (Board) for the
purpose of registering securities with the Board and obtaining permission to
issue such securities.
2) Where any person who subscribes for any securities on the faith of the matters
set down in the prospectus subsequently sustains any loss or damage by the

P.T.O.
(40)
reason that the matters set down in the prospectus have been set down with
mala fide intention or untrue or false statements have been included therein
knowingly, the body, director or experts preparing the prospectus shall be
liable to pay compensation for such loss or damage. Provided that no director
shall be liable to pay such compensation if he or she proves that he or she has
resigned prior to making a decision on the matters set down in the prospectus
with ulterior motive or knowingly or that he or she did not know that the
prospectus was untrue.
3) Where any investor sustains any loss or damage by the reason that the
prospectus, information, statements or returns submitted by a body corporate
to the Board, such an investor may make a petition to the concerned District
Court for compensation within thirty five days from the date of knowledge
within one year after the making of investment.

b) Following are the fundamental principles of insurance.


i. Principles of utmost good faith
Both the parties i.e. the insured and the insurer should have good faith
towards each other. Insurer must provide the insured complete, correct and
clear information of subject matter and terms of condition of the insurance
contract.
ii. Principles of insurable interest
The insured must have insurable interest in the subject matter of insurance.
The owner of the party is said to have insurable interest as long as he is the
owner of it.
iii. Principles of indemnity
Indemnity means a guarantee or assurance to put the insured in the same
position in which he was immediately prior to the happening of the uncertain
event. The insurer undertakes to make good the loss. Insurer agrees to
compensate the insured for the actual loss suffered.
iv. Principles of contribution
Under this principle the insured can claim the compensation only to the
extent of actual loss either from any one insurer or all the insurers.
v. Principles of Subrogation.
As per this principle after the insured is compensated for the loss due to
damage to property insured, then the right of ownership of such property
passes on to the insurer.
vi. Principles of loss minimization
Under this principle it is the duty of the insured to take all possible steps to
minimize the loss to the insured property on the happening of uncertain
event.
vii. Principles of nearest cause
The loss of insured property can be caused by more than one cause in
succession to another. The property may be insured against some causes and
not against all causes. In such an instance, the proximity cause or nearest
cause of loss is to be found out. If the proximate cause is the one which is
insured against, the insurance company is bound to pay the compensation and
vice versa.

4. Answer the following questions: (2×5=10)


a) The workers of a company objected the decision of manager regarding
the bonus to be distributed to them. The workers seek your advice about
P.T.O.
(41)
the action against the company. Advise them about the legal provisions
for the settlement of disputes of bonus under the Bonus Act, 2030.
b) The Yak Hotel Ltd. had deducted remuneration of her workers in respect
of the period of suspension. The decision of the hotel is challenged in the
labour court. Could you advocate on behalf of the hotel as the decision
was made in respecting the existing labour law. Submit your plea about
the circumstances that the remuneration of the workers can be deducted
under the Labour Act, 2048.

Answer:
a) The Bonus Act, 2030 has assured the bonus to the employees out of the profits
earned by an enterprise. The Act also has made legal provisions for the settlement
of disputes which may arise between the employees and management.
Section 16 (1) provides that if any dispute arises between employees and
management with respect to the bonus, the Labour Office shall resolve such
dispute by negotiations having invited both the parties.
Similarly, if the dispute could not be resolved by negotiation, the Labour Office
shall ask to the concern enterprise and employees to produce necessary documents
and statements of accounts and shall give a decision on the basis of such
documents and statements. When the decision is given by the Labour Office, the
party who is dissatisfied with the decision may appeal to the Labour Court within
35 day of receipt of such notice and the decision made by the Labour Court shall
be final for the matter.
Therefore, the workers of the company shall have to follow the procedures as
mentioned above to settle the dispute regarding the payment of bonus.

b) The Yak Hotel Ltd. may not deduct the remuneration of workers except the
following circumstances as provided in section 24 of labour Act, 2048:
(a) In case it is required to realize any fine.
(b) In case it is required to deduct against absence;
(c) In case it is required to deduct against loss or damage of cash or kind of the
Enterprise caused intentionally or negligently;
(d) In case it is required to deduct in respect of providing prescribed facilities;
(e) In case it is required to deduct in respect of advance or over payment of
remuneration;
(f) In case it is required to deduct in respect of the period of suspension;
(g) In case it is required to deduct under the order of government office or court;
(h) In case it is required to deduct as per the notification of Government of Nepal
published in the Nepal Gazette; or
(i) In case it is required to deduct in respect of income tax or any other tax levied
under prevailing laws.
(2) The limit of amount to be deducted pursuant to Sub-section (1) the method of
deduction, the period of deduction and other related matters shall be as
prescribed.
This company has deducted the remuneration of workers due to the reason of period
of suspension. Therefore, the action taken by the hotel is valid and following the
existing labour law.

P.T.O.
(42)
5. How disciplinary action is taken against a member of ICAN and what
punishment can be imposed against him/her by the Council of ICAN? Explain
it in the light of Nepal Chartered Accountants Act, 2053. 10

Answer:
Under section 14 (1) of Nepal Chartered Accountants Act, 2053, A Disciplinary
Committee, shall be constituted to recommend the Council to take
necessary actions after investigation upon complains lodged against any action,
contrary to Nepal Chartered Accountants Act 1997 or Regulations or code of conduct
framed under the Act, rendered by any member, or the Institute receives any
information of such kind.

Section 14 (3) provides that The procedures of the meeting of the Disciplinary
Committee and the term of office of the Chairman and members of the
Disciplinary Committee shall be as prescribed.

As per section 14 (4) The Disciplinary Committee shall have the authority, similar to
a judicial court, in respect of summoning concerned person and
investigating evidences and witnesses.

Similarly as per section 14 (5) of the Act, The Disciplinary Committee shall
recommend to the Council, along with its opinion and finding, for necessary action
against a member, if found guilty, and the Council may, considering such a
recommendation, impose any of the following punishment according to the
degree of offense,
1. Reprimanding
2. Removing from the membership for a period up to five years.
3. Prohibiting from carrying on the accounting profession for any
particular period.
4. Cancellation of the Certificate of Practice or membership.

Section 14 (7) of the Act provides that, Before imposing a punishment referred to in
sub-section (5), the Council shall provide reasonable opportunity to the concerned
members to submit their clarification.

Section 14 (8) of the Act provides that, The concerned member may, if he is not
satisfied with the decision referred to in sub-section (5), file an appeal in the
Appellate Court.

6. Answer the following questions: (5×4=20)


a) Impossibility of performance is, as a rule, not an excuse for non
performance of a contract. Explain.
b) Discuss the presumptions as to Negotiable Instruments under Negotiable
Instrument Act, 2034.
c) What are the functions, duties and powers of the Social Welfare Council
as provided in the Social Welfare Act, 2049?

P.T.O.
(43)
d) Sagarmatha Enterprise, an industry established in the year 2064 being
operated in loss till date. Mr. Anil shah, promoter of the industry thinks
that if the industry is declared sick industry and more facilities would be
obtained and the industry would be exempted tax on the raw materials to
be imported from China. Answer the following questions on the basis of
relevant laws of Nepal.
i) Can such industry be declared as sick industry?
ii) Whether the contention of Mr. Anil Shah regarding exemption of tax
is tenable?
e) List out the major functions of World Trade Organization (WTO).

Answer:
a) There is a general rule 'what is impossible, does not create legal obligation'. A
contract is terminated due to subsequent impossibility.
A contract is not always terminated on the ground of a subsequent impossibility.
There is a general rule of contract that says 'He that, agreed to do, that must to do
it or pay damages for not doing it.' A person is not free from his legal obligations
which he has undertaken. Section 79 (3) specifies the circumstances of
exceptions to the termination by impossibility. There is no excuse for a promisor
in the following circumstances:
i. Difficulty to perform: A contract is not terminated due to the fact that the
performance is unexpectedly difficult, expensive, burdensome or less
profitable than anticipated.
ii. Commercial impossibility: commercial impossibility like the one caused
by a greater cost or a lesser profit is never excused.
iii. Impossibility due to the default of a third party: It is not an excusable
cause to terminate the contract on the default of third party.
iv. Strikes, lockouts and disturbances: Except otherwise in mentioned in the
conditions of a contract, Strikes, lockouts or civil disturbances do not
constitute an adequate cause to excuse the party who does not perform his
liability.
v. In case it becomes necessary to pay additional taxes, fees or other
revenues.
vi. Partial Impossibility: Where a contract is made for several objectives,
failure of one of them does not terminate the whole contract. Other
possible objectives must be attained by the promisor.

b) Section 103 of Negotiable Instrument Act, 2034 states the presumptions as to


Negotiable Instruments.
Until the contrary is proved, the following presumptions shall be made:
a) That every negotiable instrument was made or drawn for consideration, and that
every such instrument, when it has been accepted, endorsed, negotiated or
transferred, was, accepted, endorsed, negotiated or transferred for consideration in
accordance with the prevailing law.
b) That every negotiable instrument bearing a date was made or drawn on such date,
c) That every transfer of a negotiable instrument was made before its maturity,
d) That every accepted bill of exchange was accepted within a reasonable time after its
date and before its maturity.
P.T.O.
(44)
e) That the endorsements appearing upon a negotiable instrument were made in the
order in which they appear thereon.
f) That the Holder of a negotiable instrument is a Holder in due course.
Provided that, the burden of proving that the Holder is a Holder in due course
lies upon him / her in the following condition.
i) When the negotiable instrument has been obtained from its lawful owner or
from any person in lawful custody thereof by means of an offence or fraud.
ii) Where the negotiable instrument has been obtained from the maker, drawer
or acceptor thereof by means of an offence or fraud or unlawful
consideration.

c) The functions, duties and powers of the Social Welfare Council shall be as
follows as provided in the section 9 of Social Welfare Act, 2049:
(a) To run or cause to run the social welfare activities smoothly and effectively, to
extend help to the social organizations and institutions and to develop co-
ordinations among them and to supervise, follow up and carry out evaluations
of their activities.
(b) To extend or cause to extend help and support to establish social organizations
and institutions, their development, strengthening and extensions.
(c) To work or cause to work as co-coordinator between Government of Nepal and
social organizations and institutions.
(d) To provide consultancies to Government of Nepal in order to formulate
policies and programmes directly related to social welfare activities and other
social services.
(e) To establish and conduct or cause to establish and conduct a fund, for the
social welfare activities.
(f) To work or cause to work as a center for dissemination of information and
documentation to the affiliated service oriented organizations and institutions
with Council.
(g) To conduct or cause to conduct trainings, studies and research programmes in
the areas with social welfare.
(h) To carry out or cause to carry out the physical supervisions of the properties of
those social institutions and organizations affiliated with the Council.
(i) To carry out or cause to carry out the necessary functions to implement the
objectives of this Act.
(j) To make or cause to make contract or agreement with the local, foreign or
international organizations and foreign countries.
(k) To collect grant from the national and international agency and to manage the
received grant.

d) Section 25A of the Industrial Enterprises Act, 2049 mentions the following provisions
regarding the sick industry. According to it 1) if any industry is being operated in loss
for a consecutive period of five years and its production level is twenty percent or less
than twenty percent of the total production capacity, the Government of Nepal may, if it
deems necessary, declare such industry a sick industry by notification published in the
Nepal Gazette.
No duty, fee and tax of any kind shall be levied on the machinery imported by any
industry as referred to in sub section (1) above for the extension and diversification of
such industry .
On the basis of above provision of law the answer can be mentioned as follows:

P.T.O.
(45)
i) No, Sagarmatha Enterprises cannot be declared as sick industry because we don't
know the production capacity and current production level of it. As Sagarmatha
Enterprise has been running in loss for more than 5 years, however, without having
proper knowledge of its production capacity and current production this industry
can't be declared as sick industry.
ii) Similarly, the answer again is 'no', because exemption shall be obtained only in case
of import of machinery but not in import of raw materials pursuant to section 25 A
(2) of this Act.

e) Functions of WTO
The major functions of the World Trade Organization (WTO) mentioned in its
Charter are as follows:
1. Administer and implement the trade agreements
2. Act as a forum for multilateral trade negotiations
3. Seek to resolve trade disputes
4. Oversee national trade policies
5. Cooperate with other international institutions involved in global economic
policy making
6. Maintain trade related database. Members are required to notify in detail various
trade measures and statistics
7. Act as watchdog of international trade, constantly examining the trade regimes of
individual members
8. Act as a management consultant for world trade
9. Provide technical assistance and training for developing countries

7. Write short notes on the followings: (2×5=10)


a) Formation of the Nepal Securities Board.
b) Powers of Labour Officer.
Answer:
a) Pursuant to section 3 (1) Securities Act 2063, there shall be established a Board by
the name of Securities Board of Nepal in order to regulate and manage the activities
of the securities markets and persons involved in securities business by regulating
the issue, purchase, sale and exchange of securities in order to develop capital
market and protect the interests of investors in securities.
Pursuant to section 3 (2) the Board shall consist of the following members:-
a. A person appointed by the Government of Nepal -Chairperson
b. Joint Secretary, Ministry of Finance -Member
c. Joint Secretary, Ministry of Law, Justice and
Parliamentary Affairs -Member
d. Representative, Nepal Rastra Bank -Member
e. Representative, Institute of Chartered
Accountants of Nepal -Member
f. Representative, Federation of Nepalese
Chambers of Commerce and Industries -Member

P.T.O.
(46)
g. A person nominated by the Government of Nepal
from amongst the experts who have obtained at least
master degree in economics, management, finance,
commerce or law from recognized university and
gained at least seven years of experience in stock
exchange, management, capital market development,
finance and economic sector.
-Member

b) Pursuant to section 64 of the Labour Act, 2048, the Government of Nepal by


publishing a notice in the Nepal Gazette may appoint one or more Labour Officers,
as per necessity or designate any other officer to perform the functions of a Labour
Officer for one region/sector.
Pursuant to Section 65 of the Labour Act, 2048, the Labour Officer shall have
the following powers –
(a) To enter into the premises of the Enterprise as per necessity;
(b) To examine the documents and registers of the Enterprise relating to workers
and employees;
(c) To function or advise as per necessity for improving labour relations;
(d) To attempt for solving disputes arising between workers or employees and the
Proprietor;
(e) To implement welfare provisions, if it does not exit and where if exist, to
supervise whether or not it is operated properly;
(f) To supervise the implementation of minimum remuneration prescribed by
Government of Nepal;
(g) To record statement, of anybody to fulfill the objectives of this Act, as per
necessity;
(h) To perform tasks of the Factory Inspector in his/her absence, except technical
tasks; and
(i) To perform other tasks as per the directives of Government of Nepal and
Department of Labour.
(j) Other powers, functions and duties shall be as prescribed.

P.T.O.
(47)

Financial Management

P.T.O.
(48)

Suggested
Roll No……………. Maximum Marks - 100
Total No. of Questions – 7 Total No. of Printed Pages –13
Time Allowed – 3 Hours
Marks
Attempt all questions.
Working notes should form part of the answer. Make assumptions wherever necessary.
1. The current assets and current liabilities of ABC Co. at the end of March 2016
are as follows:
Description Amount in (Rs.„000)
Inventory 5,700
Trade receivables 6,575
Total current assets 12,275
Trade payables 2,137
Overdraft 4,682
Total current liabilities 6,819
Net current assets 5,456
For the year to end on March 2016, ABC Co. had domestic and foreign sales of
Rs. 4 crores, all on credit, while cost of sales was Rs. 2 crore 60 lakhs. Trade
payables are related to both domestic and foreign suppliers.
For the year to end on March 2017, ABC Co. has forecasted that credit sales will
remain at Rs. 4 crores while cost of sales will fall to 60% of sales. The company
expects current assets to consist of inventory and trade receivables, and current
liabilities to consist of trade payables and the company‟s overdraft.
ABC Co. also plans to achieve the following target working capital ratio for the
year to the end on March 2017:
Inventory days 60 days
Trade receivables days 75 days
Trade payables days 55 days
Current ratio 1.4 times
Assume 365 days in a year.
Required: (6+6+8=20)
a) Calculate the working capital cycle (cash collection cycle) of ABC Co. at the
end of March 2016 and discuss whether a working capital cycle should be
positive or negative.
b) Calculate the target quick ratio and the target ratio of sales to net working
capital of ABC Co. at the end of March 2017.
c) Analyse and compare the current asset and current liability positions for
March 2016 and March 2017, and discuss how the working capital financing
policy of ABC Co. would have changed.
Answer
a) Calculation of working capital cycle at the end of March 2016
P.T.O.
(49)
Inventory days = 365 x (5,700/26,000) = 80 days
Trade receivables days = 365 x (6,575/40,000) = 60 days
Trade payables days = 365 x (2,137/26,000) = 30 days
Working capital cycle = 80 + 60 – 30 = 110 days
The working cycle of ABC Co is positive and the company pays its trade suppliers 110 days
(on average) before it receives cash from its customers. This represents a financing need as far
as ABC Co is concerned, which could be funded from a short-term or long-term source.
If the working capital cycle had been negative, ABC Co would have been receiving cash from
its customers before it needed to pay its trade suppliers. A company which does not give
credit to its customers can have a negative working capital cycle.
Even if companies might generally prefer to be paid by customers before they have to pay
their suppliers, the question of whether the working capital cycle should be positive or
negative implies that companies are able to make such a choice, but this is not usually the
case. This is because the length of the working capital cycle depends on its elements, which
are inventory days, trade receivables days and trade payables, and these elements usually
depend on the nature of the business undertaken by a company and the way that business is
conducted by its competitors. The length of the working capital cycle is usually therefore
similar between companies in the same business sector, but can differ between business
sectors.
b) Target quick ratio and sales to net working capital ratio at the end of March 2017:
Cost of sales = 40,000,000 x 0·6 = Rs. 24,000,000
Inventory using target inventory days = 24,000,000 x 60/365 = Rs. 3,945,206
Trade receivables using target trade receivables days = 40,000,000 x 75/365
= Rs. 8,219,178
Current assets = 3,945,206 + 8,219,178 = Rs. 12,164,384
If the target current ratio is 1·4 times, current liabilities = 12,164,384/1·4 = Rs. 8,688,846
The target quick ratio (acid test ratio) = 8,219,178/8,688,846 = 0·95 times
Net current assets (wc) at the end of March 2017 = 12,164,384 – 8,688,846 = Rs. 3,475,538
Target sales/net working capital ratio = 40,000,000/3,475,538 = 11.51 times
c) The current liabilities at the end of March 2017, calculated in part (b), can be divided into
trade payables and the forecast overdraft balance.
Trade payables using target trade payables days = 24,000,000 x 55/365 = Rs. 3,616,438.
The overdraft (balancing figure) = 8,688,846 – 3,616,438 = Rs. 5,072,408
Comparing current assets and current liabilities:
Description March 2016 March 2017
(Rs. „000) (Rs. „000)
Inventory 5,700 3,945
Trade receivables 6,575 8,219
Total Current Assets 12,275 12,164
Trade payables 2,137 3,616
Overdraft 4,682 5,072
Total Current Liabilities 6,819 8,688
Net current assets 5,456 3,476
The overdraft as a percentage of current liabilities will fall from 69% (4,682/6,819) to 58%
(5,072/8,688). Even though the overdraft is expected to increase by 8.3%, current liabilities
P.T.O.
(50)
are expected to increase by 27.4% (8,688/6,819). Most of this increase is expected to be
carried by trade payables, which will rise by 69.2% (3,616/2,137), with trade payables days
increasing from 30 days to 55 days.
At the end of March 2016, current liabilities were 56% of current assets (100 × 6,819/12,275),
suggesting that 44% of current assets were financed from a long-term source. At the end of
March 2017, current liabilities are expected to be 71% of current assets (100 × 8,688/12,164),
suggesting that 29% of current assets are financed from a long-term source. This increasing
reliance on short-term finance implies an aggressive change in the working capital financing
policy of ABC Co.
2.
a) Simon Ltd. is considering five capital projects for the years 2017 and 2018.
The company is financed by equity and its cost of capital is 12%. The
expected cash flows of the projects are as follows:

(Figures in Rs.' 000)


Projects 2017 2018 2019 2020
A -70 35 35 20
B -40 -30 45 55
C -50 -60 70 80
D - -90 55 65
E -60 20 40 50
All projects are divisible i.e. size of the investment can be reduced, if
necessary, in relation to the availability of funds. None of the projects can be
delayed or undertaken more than once.
Required: 10
Which projects, Simon Ltd. should undertake if the capital funds available
for the investment are limited to Rs. 110,000 in 2017 and with no limitation
in subsequent years? Use NPV, PI and discounted PBP techniques for your
analysis.
b) The balance sheet of ABC company is given below: (Rs. in
lakh)
Liabilities Amount Assets Amount
Equity share capital 250 Fixed assets 400
General reserve 280 Investment 50
P&L A/c current year 30 Stock 460
Secured loans-long term 300 Debtors 460
Secured loans-short term 360 Cash in hand 10
Creditors 150 Miscellaneous
Other liabilities 30 expenditure (not written
off) 20
1400 1400
Additional information:
i) From the profit and loss account Rs. 90 lakhs was transferred to general
reserve during the year.
ii) Interest cost amounted to Rs. 120 lakhs.
iii) Income tax is levied at the rate of 40%
Required: Calculate 5

P.T.O.
(51)
(i) Debt equity ratio (ii) Current ratio (iii) Interest coverage ratio.

Answer
a) i)Computation of NPV
Rs. In “000”
Year PVF Project A Project B Project C Project D Project E
2017 1 -70 -70 -40 -40 -50 -50 0 0 -60 -60
2018 0.89 35 31.15 -30 -26.7 -60 -53.4 -90 -80.1 20 17.8
2019 0.8 35 28 45 36 70 56 55 44 40 32

2020 0.71 20 14.2 55 39.05 80 56.8 65 46.15 50 35.5


NPV 3.35 8.35 9.4 10.05 25.3

ii ) Calculation of Profitability Index


PV of Cash Inflows
P I = ----------------------------------
PV of Cash Outflows
31.15+28+14.20 73.35
Project A= ------------------------- = ---------- =1.048
70 70
36+39.05 75.05
Project B= ----------------- = ---------- = 1.125
40+26.7 66.70
56+56.8 112.8
Project C= ----------------- = ---------- = 1.091
50+53.4 103.4
44+46.15 90.15
Project D= ----------------- = ---------- = 1.125
80.1 80.1
17.8+32+35.5 85.30
Project E= ----------------- = ---------- = 1.422
60 60
iii ) Calculation of discounted payback:
Rs. In “000”
Project A Project B Project C Project D Project E
PV of Cumm. PV of Cumm PV of Cumm. PV of Cumm. PV of Cumm.
Year Time CFs CFs CFs . CFs CFs CFs CFs CFs CFs CFs
2017 0 -70 -70 -40 -40 -50 -50 0 0 -60 -60
31.1
2018 1 5 -38.85 -26.7 -66.7 -53.4 -103.4 -80.1 -80.1 17.8 -42.2
2019 2 28 -10.85 36 -30.7 56 -47.4 44 -36.1 32 -10.2
39.0
2020 3 14.2 3.35 5 8.35 56.8 9.4 46.15 10.05 35.5 25.3
Un-recouped Outflow
Discounted payback = Years already recouped + ----------------------------------------
PV of next year

P.T.O.
(52)
A B C D E
=2+10850/14200 =2+30700/39050 =2+47400/56800 =2+36100/46150 =2+10200/35500
2.76 Years 2.79 Years 2.83 Years 2.78 Years 2.29 Years
Since, there is limitation of fund only in 2017, selection of the project should be made on the basis of
NPVI (Net Present Value Index)
NPV
P I = ----------------------------------
Initial Outflows
3.35
Project A= ------------------------- = 0.05
70
8.35
Project B= ----------------- = 0.21
40
9.40
Project C= ----------------- = 0.19
50
25.30
Project E= ----------------- = 0.42
60

As per the information given, the capital funds are restricted to Rs. 110,000 in 2017. The Project D
does not require any outflow during 2017 but it is acceptable on the basis of NPV methods, therefore,
can be implemented in 2018. For other projects the capital funds of Rs. 110,000 can be distributed on
the basis of their ranking of NPVI. Therefore, the firm can adopt Project E ( Rs. 60,000), Project B
(Rs. 40,000) and Part of Project C ( Rs. 10,000).

b)

Current Assets Rs. 930


Current Ratio= ---------------------------- = ----------------------------= 1.72 times
Current Liabilities Rs. 540

Debt Rs. 300


Debt Equity Ratio= -------------- = -------------- = 0.56 timrs
Equity Rs. 540

EBIT Rs. 320


Interest Coverage Ratio = -------------- = -------------- = 2.67 times
Interest Rs. 120

Working Notes:
Current Assets=Stock +Debtors+ Cash = 460+460+10= 930
Current Liabilities= Short term loan+ Trade Credit + Other Liabilities= 360+150+30= 540
Debt= Term loan=300
Equity=Capital+ Reserve+ P/L-Mis. Expenses=250+280+30-20=540
EBIT=P&L A/C+ GR transfer +Interest + tax
=30+90+120+(30+90)×40/60=240+80=320
3.
a) Company P and Q are identical in all respects including risk factors expect
for debt/equity. Company P has issued 10% debentures of Rs. 18 lakhs while

P.T.O.
(53)
company Q is unlevered. Both the companies earn 20% before interest and
taxes on their total assets of Rs. 30 lakhs.
Assume a tax rate of 50% and capitalization rate of 15% for all-equity
company.
Required: (4+4=8)
Compute the value of P and Q using:
i) Net income approach, and
ii) Net operating income approach.
b) The following information related to XL Ltd. for the year ended 31st March,
2015 are available to you:
Particulars Amount (Rs.)
Equity Share Capital (Rs. 100 each) 2,500,000
11% Bonds (Rs.1,000 each) 1,850,000
Sales 4,200,000
Fixed cost (excluding interest) 348,000
Other Information:
Financial Leverage 1.39
Profit-Volume Ratio 25.55 %
Income Tax Rate 35%
Required: Calculate 7
(i) Operating Leverage, (ii) Combined Leverage, and (iii) Earning per share.
Answer
a)
(i) Valuation Under Net Income Approach
Particulars P Q
(Amount in Rs.) (Amount in Rs.)
EBIT (20% of Rs.3,000,000) 600,000 600,000
Less: Interest (10% of Rs. 1,800,000) (180,000) -
EBT 420,000 600,000
Less: Tax @ 50% (210,000) 300,000
EAT (available to equity holders) 210,000 300,000
Value of Equity (Capitalised @ 15%) 1,400,000 2,000,000
(EAT/Capitalisation rate)
Add: value of Debt 1,800,000 -
Total value of the Company 3,200,000 2,000,000
(ii) Valuation under Net Operating Income Approach (Here, EBIT=NOI)
Particulars P Q
(Amount in Rs.) (Amount in Rs.)
Capitalisation of Earning @15% 2,000,000 2,000,000
[Rs.6,00,000 x (1-0.5)/0.15]
Less: Value of Debt (900,000) -
((Rs.18,00,000 x (1-0.5))
Value of Equity 1,100,000 2,000,000
Add: Total Value of Debt 1,800,000 -
Total value of the Company 2,900,000 2,000,000
b)
Calculation of Contribution Margin
Profit – Volume Ratio = Contribution / Sales

P.T.O.
(54)
0.2555 = Contribution / 4,200,000
Contribution = Rs. 1,073,100
(i) Operating Leverage = Contribution / (Contribution-Fixed Cost)
= 1,073,100 / (1,073,100 – 348,000)
= 1.48
(ii) Combined Leverage = Operating Leverage × Financial Leverage
= 1.48 × 1.39 = 2.06
(iii) Earnings Per Share
Number of Equity Shares = Rs. 2,500,000/100 = 25,000
Earnings Before Tax (EBT) = Contribution – Fixed Cost – Interest
= 1,073,100 – 348,000 – 203,500
= Rs. 521,600
Profit after Tax = EBT – Tax = 521,600 – 182,560 = Rs. 339,040
EPS = Rs. 339,040/25,000 = Rs.13.56
4.
a) Siddhartha Mutual Fund has the following assets under it on the close of
business as on:
Company No. of Shares 1st Baishakh 2073 2nd Baishakh 2073
market price per market price per
share (Rs.) share (Rs.)
HIDCL 20,000 435 445
Fewa Bikash Bank 30,000 322 360
Garima Bikash Bank 20,000 461 483
Malika Bikash Bank 60,000 525 515
Total number of units 600,000
Required: (3+5=8)
i) Calculate total Net Assets Value (NAV) and per unit NAV of the fund as
on 1st Baishakh 2073.
ii) Assume one Mr. A, submits a cheque of Rs. 3,000,000 to the Mutual
Fund on 1st Baishakh 2073 and the Fund manager of the company
purchases 8,000 shares of Fewa Bikas Bank and the balance amount is
held in bank. In such a case, what would be the position of the fund as on
1st Baishakh 2073? Also find new NAV in total and per unit of the fund
as on 2nd Baishakh, 2073.
b) A new customer has approached a firm to establish new business. The
customer requires 1.5 months of credit. If the proposal is accepted, the sales
of the firm will go up by Rs. 240,000 per annum. The new customer is being
considered as a member of 10% risk of non-payment group.
The cost of sales amounts to 80% of sales. The tax rate is 30% and the
desired rate of return is 40% after tax.
Required: (2+5=7)
i) What is the opportunity cost of the proposal?
ii) Should the proposal be accepted? Why?
Answer
a)
i) NAV of fund as on 1st Baishakh 2073
= (20,000×435)+(30,000×322)+20,000×461)+(60,000×525)
= 8,700,000+9,660,000+9,220,000+31,500,000
= Rs. 59,080,000
P.T.O.
(55)

NAV per unit =59,080,000


600,000
= Rs. 98.47

ii) The revised position of fund as on 1st Baishakh 2073 shall be as follows:
Company No. of shares 1st Baishakh 2073 market Total (Rs.)
price per share(Rs.)
HIDCL 20,000 435 8,700,000
Fewa Bikash Bank 38,000 322 12,236,000
Garima Bikash Bank 20,000 461 9,220,000
Malika Bikash Bank 60,000 525 31,500,000
Cash 424,000
62,080,000
3,000,000
Number of units of fund = 600,000+ 98.47

=630,466 units

On 2ndBaishakh, 2073, the NAV of fund will be as follows:


Company No. of shares 2nd Baishakh 2073 market Total (Rs.)
price per share(Rs.)
HIDCL 20,000 445 8,900,000
Fewa Bikash Bank 38,000 360 13,680,000
Garima Bikash Bank 20,000 483 9,660,000
Malika Bikash Bank 60,000 515 30,900,000
Cash 424,000
NAV as on 2nd Baishakh, 2073 63,564,000

NAV/unit =Rs. 63,564,000


630,466

= Rs. 100.82

b) i) Opportunity cost of the proposal is the opportunity cost of investment on


receivable which is
= (cost of sales/Receivable Turnover)×ROR
=[(240,000×0.80)/(12/1.5)]×40%
=[192,000/8]×40% = Rs. 9,600

ii) Evaluation of Credit Proposal


Description Amount (Rs.)
A Calculation of profit on additional sales
Increase in sales 240,000
Less: Cost of Sales (192,000)
EBIT 48,000
Less: Risk of non-payment (10% of Rs. 24,000
240,000)
Profit Before Tax 24,000
Tax @30% 7,200
Profit After Tax (PAT) 16,800
B Opportunity cost of Investment on 9,600
P.T.O.
(56)
Receivable
C Net Benefit (A-B) 7,200
Decision: Since estimated PAT on additional sales (i.e.Rs.16,800) is more than
the opportunity cost of investment on receivable (i.e. Rs.9,600), the firm should
accept the offer.
5.
a) M/s Atlantic Company Ltd., with a turnover of Rs. 4.80 crores, is expecting
growth of 25% for coming year. Average credit period is 90 days. The past
experience shows that bad debt losses are 1.75% on sales. The company's
annual administering cost for collecting receivables is Rs. 600,000. It has
decided to take factoring services of a factor on terms that the factor will buy
receivables by charging 2% commission and 20% risk with recourse. The
factor will pay advance on receivables to the firm at 16% interest rate p.a.
after withholding 10 % as reserve.
Required: 5
Calculate the annualised effective cost of factoring to the firm (assume 360
days in a year).
b) The following figures are collected from the annual report of XYZ Ltd.:
Net profit Rs. 30 lakhs
Outstanding 12% preference shares Rs. 100 lakhs
Number of equity shares 3 lakhs
Return on investment 20%
Cost of capital 16%
Required: 5
What should be the approximate dividend pay-out ratio so as to keep the
share price at Rs. 42 by using Walter Model?
c) SE Ltd., having an equity capital of Rs. 1 billion (face value of Rs. 100 each)
earned a net profit of Rs. 15 crores during the financial year 2072/73 and is
planning to make new investments of Rs. 60 crores during the current
financial year 2073/74. In line with the company's dividend policy it has a
plan of declaring dividend of Rs. 10 per share at the end of current financial
year. The firm's opportunity cost of capital is 15% and is currently traded at
Rs. 100 per share.
Required: (2+2+1=5)
Considering the Modigliani and Miller's Dividend Irrelevance Theorem,
what is the price of the share at the end of the current financial year if:
i) the dividend is not declared,
ii) the dividend is declared,
iii) How many new shares must be issued to finance its investments
assuming that dividend is declared?
Answer
a) Calculation of Net amount financed (Rs. in Lakh)
Expected credit sales for next year = 480 x 125% 600
Debtors on the basis of 90 days = (600 x 90/360) 150
(-) Factor reserve 10% -15
(-) Commission charged upfront @ 2% of 150 -3
(-) Interest charged upfront (16% of (150-15-3)×90/360) -5.28
Net amount financed by factor 126.72
P.T.O.
(57)

Net periodic cost of factoring for 90 days (Rs. in Lakh)


Factoring commission 3
(+) Interest 5.28
(-) Bad Debts saved (20% of 1.75% of 600 x 90/360) -0.525
(-) Administration Expense Saved (6 x 90/360) -1.5
Net cost of factoring 6.255
Therefore, % cost of factoring for 90 days = (6.255/126.72) x100 = 4.94%
Annualized effective cost of factoring = 4.94% x 360/90 = 19.74%

Assumptions:
1. All sales are credit sales.
2. Factor will bear 20% risk of bad debts
b)
Rs. in lakhs
Net Profit 30
Less: Preference dividend (12% of Rs. 100 laks) 12
Earnings for equity shareholders 18
Therefore, earning per share =1,800,000/300,000 = Rs. 6.00
Let, the dividend pay-out ratio be x and so the share price will be:
P=D/Ke +(r(E-D)/Ke)/Ke (as per Walter Model)
Here D = 6x; E = Rs. 6; r = 0.20 and Ke = 0.16 and P = Rs. 42
Hence Rs. 42 = 6x/0.16+0.2(66x)/(0.16*0.16)
or Rs. 42 = 37.50x + 46.875 (1 –x)
or 9.375x = 4.875
x = 0.52
So, the required dividend payout ratio will be = 52%
c)
i. Price of the share at the end of the current financial year if the dividend is not paid is Rs.115.
P1 = P0 (1 + k) - DIV1
= 100 × (1+0.15) - 0
= 115
ii. Price of the share at the end of the current financial year if the dividend is paid is Rs.105.
P1 = P0 (1 + k) - DIV1
= 100 × (1+0.15) - 10
= 105
Where
P1 = Price at the end of the financial year
P0 = Current Price
k = Opportunity Cost of Capital
DIV1 = Dividend at the end of the financial year
iii. The value of share issued for investment is Rs.105 per share (FV and share
premium). Therefore, no. of shares to be issued:
mP1 = I - (X - nDIV1)
105m = 600,000,000 - (150,000,000 – 10,000,000 × 10)
105m = 550,000,000
m = 550,000,000/105 = 5,238,095 shares
Where
m = Required No. of New Shares
n = Existing No. of Shares
P.T.O.
(58)
I = Investment Amount
X = Net Profit Attributable to Shareholders
DIV1 = Dividend at the end of the financial year
6. Write short notes on: (4×2.5=10)
a) Reasons for changes in working capital
b) Clean overdraft
c) Virtual banking and its advantages
d) Dematerialization of securities
Answer
a) The changes in the level of working capital occur for the following 3 basic reasons:
i. Changes in Sales and operating expenses: The first factor causing a change in the
working capital requirement is a change in the sales and operating expenses. The
changes in this factor may be due to three reasons: First, there may be a long-run
trend of change. For instance, the price of a raw material, say oil, may constantly
rise, necessitating the holding of a large inventory. The second trends would
mainly affect the need for permanent current assets. In the second place, cyclical
changes in the economy leading to ups and downs in business activity influence
the level of working capital, both permanent and temporary. The third source of
change is seasonality in sales activity. Seasonality-peaks and troughs-can be said
to be the main source of variation in the level of temporary working capital.
The change in sales and operating expenses may be either in the form of an
increase or decrease. An increase in the volume of sales is bound to be
accompanied by higher levels of cash, inventory and receivables. The decline in
sales has exactly the opposite effect a decline in the need for working capital. A
change in the operating expenses rise or fall has a similar effect on the levels of
working capital.
ii. Policy Changes: The second major cause of changes in the level of working
capital is because of policy changes initiated by the management. There is a wide
choice in the matter of current assets policy.
iii. Technological changes: Finally, technological changes can cause significant
changes in the level of working capital. If a new process emerges as a result of
technological developments, which shortens the operating cycle, it reduces the
need for working capital and vice versa.

b) A clean overdraft refers to an advance by way of overdraft facility, but not backed
by any tangible securities. Hence request for clean advances are entertained only
from parties which are financially sound and reputed for their integrity. Bank has to
rely upon the personal security of the borrowers. Some factories to be considered by
the banks before granting clean OD's are (a) past operations of the party (b) turnover
in the accounts (c) satisfactory dealings for considerable period (d) reputation in the
market.
As a safeguard, banks take guarantee from other persons who are credit worthy
before granting this facility. A clean advance is generally granted for a short period
and must not be continued for long.

c) Virtual Banking refers to the provision of banking and related services through the
use of information and communication technologies (ICT) without direct resources
to the bank by the customers. The advantages of virtual banking services are as
follows:
 Lower cost of handling transaction
P.T.O.
(59)
 The increased speed of response to the customer requirements
 Lower cost of operating branch network along with reduced staff costs
 Leads to cost and service efficiency
d) Dematerialization is the process of converting physical shares into electronic format.
An investor who wants to dematerialize his physical shares need to open a demat
account with Depository participant. Investor surrenders his physical shares and in
turn gets electronic shares in his demat account. Depository Participant (DP) is the
market intermediary through which investors can avail the depository services.
Depository Participant provides financial services and includes organizations like
banks, brokers, custodians and financial institutions.
Dealing in Demat is beneficial for investors, brokers and companies alike. It reduces
the risk of holding shares in physical format from investor's perspective. It's
beneficial for brokers as it reduces the risk of delayed settlement and enhances profit
because of increased participation.

7. Distinguish between: (4×2.5=10)


a) Venture capital and Private equity
b) Systematic risk and Unsystematic risk
c) Trading securities and Available-for-sale securities
d) Higher financial leverage is better than higher operating leverage. Comment.
Answer
a) Venture capital is a risk capital and is regarded as a subset of private equity. It is
generally raised to specifically finance startups and small and medium sized private
companies with strong growth potential whereas the private equity also has component
of development capital and buyout capital that is targeted to mature businesses that are
already established.
Development capital refers to the pool of capital raised to finance for expansion of
businesses while buy-out capital refers to pool of fund raised through investors and
management team, usually together with borrowed money, to buy a business from its
current owners.
Effectively, therefore the private equity consists of venture capital, development
capital and in certain cases buyout capital (when buyouts are not adequately financed
by banks).
The private equity is generally held in private companies or such public companies
that are not traded in stock exchanges.
b) Systematic Risk : Systematic risk refers to the variability of return on stocks or
portfolio associated with changes in return on the market as a whole. It arises due to
risk factors that affect the overall market such as changes in the nations‟ economy, tax
reform by the Government or a change in the world energy situation. These are risks
that affect securities overall and, consequently, cannot be diversified away. This is the
risk which is common to an entire class of assets or liabilities. The value of
investments may decline over a given time period simply because of economic
changes or other events that impact large portions of the market. Asset allocation and
diversification can protect against systematic risk because different portions of the
market tend to underperform at different times. This is also called market risk.
Unsystematic Risk Unsystematic risk refers to risk unique to a particular company or
industry. It can be avoided through diversification. This is the risk of price change due
to the unique circumstances of a specific security as opposed to the overall market.
This risk can be virtually eliminated from a portfolio through diversification.
c) Trading securities are those investment securities purchased with the intent to take
advantage of short-term price changes. These are classified as current assets.

P.T.O.
(60)
Besides, available-for-sale securities are those investment securities that are purchased
as a store of wealth for the reasons like safety, to hold excess cash temporarily, or just
to earn a normal long-term return. The purchase of available-for-sale securities is then
a more passive investment than an investment in trading securities; the intent is to earn
a normal return, not to make a quick return by guessing which way the market is
going. Available-for-sale securities can be classified as current assets or long-term
assets, depending on management's intent for holding them.
d) Operating leverage indicates the proportion of fixed operating charges. Higher
operating leverage indicates higher quantum of fixed operating charges. If a business
firm has a lot of fixed costs as compared to variable costs, then the firm is said to have
high operating leverage.
The financial leverage indicates the proportion of fixed financial charges, in the form
of interest cost. Higher financial leverage indicates higher quantum of fixed financial
charges.
The company can differ or somewhat convince the financial institution and banks, to
accept the delay in payment, which cannot be possible in the case of provider of
operating activities. Hence we can say that higher financial leverage is better than
higher operating leverage.

P.T.O.
(61)

Cost & Management Accounting

P.T.O.
(62)

Suggested
Roll No……………. Maximum Marks - 100
Total No. of Questions: 6 Total No. of Printed Pages - 14
Time Allowed - 3 Hours
Marks
All questions are compulsory. Working notes should form part of the answer.
Make assumptions wherever necessary.
1. Trilochan Industries is manufacturing several consumer durables which have good
demand in the market. The firm has been established only very recently and
currently it is in the stage of introduction. It has ambitious plans to expand
production after earning a name in the market. However, the company is having
problems to get adequate power supply. Moreover most of its labourers are casual
workers and labour–absenteeism is also affecting production. In view of these
unstable conditions the firm has adopted the practice of preparing quarterly
flexible budgets. For the quarter ending 31st December, 2015 flexible budgets for
three possible levels of production were prepared as follows. The company wanted
to achieve 90% capacity utilization as its products had good demand.
Flexible Budget (Rs. in Lakhs)
Capacity utilization 60% 80% 90%
Budgeted sales 50.00 66.00 75.00
Budgeted costs:
Direct materials 12.00 16.00 18.00
Direct labour 15.00 20.00 22.50
Production overheads 11.80 14.00 15.10
Administration overheads 2.00 2.00 2.00
Selling overheads 7.80 9.80 10.20
Soon after the decision to attain 90% capacity utilization, available power was
reduced by the Nepal Electricity Authority and the reduced supply was sufficient
to meet 50% capacity production. The position has been immediately reviewed and
the firm is considering the following possible options to meet the situation:
a) Stop production for the quarter. As regular employees are only few, lay off
compensation payable will be only Rs. 1.20 lakhs. Further, Fixed Overheads
can be reduced by as much as 60%.
b) Continue production at 50% level. Estimated sales income at this level will be
Rs. 40 lakhs.
c) A private agency in the area has offered surplus captive power available with
it. With this additional power supply production can be maintained at 90%
level. However. The overall variable production overhead will increase by
40%.
d) Sub-contract the balance 40% which cannot be made by the firm to two small
industrial units in the area, which have the necessary facilities, equally at a cost
of Rs. 15 lakhs each.
Required: (12+8=20)
i) Calculate the profit/loss at 50% and 90% capacity utilization.
ii) Evaluate each of the above options and recommend the best plan.
Answer

P.T.O.
(63)

Working Notes:
1. Variable Cost
Direct Materials: At 90% capacity = Rs. 18.00 Lakhs
At 50% capacity = Rs. 18.00 Lakhs/0.9 x 0.5 = Rs 10.00 Lakhs

Direct Labour : At 90% capacity = Rs. 22.50 Lakhs


At 50% capacity = Rs. 22.50 Lakhs/0.9 x 0.5 = Rs 12.50 Lakhs

2. Fixed cost
Administration Overheads = Rs. 2.00 Lakhs

3. Segregation of semi-variable Production Overheads into variable and fixed components. (i)
Variable Component = Change in cost / Change in capacity
= (Rs. 15.10 lakhs – Rs. 11.80 lakhs) /(90% - 60% )
= Rs. 3.30 lakhs /30%
= Rs. 0.11 lakhs for each 1% capacity

Variable production overhead


At 50% capacity = Rs. 0.11 lakhs x 50 = Rs. 5.5 lakhs
At 90% capacity = Rs. 0.11 lakhs x 90 = Rs 9.90 lakhs

(ii) Fixed production overhead = Rs. 11.80 lakhs – (Rs. 0.11 lakhs x 60) = Rs. 5.20 lakhs

4. Segregation of semi-variable Selling Overheads into variable and fixed components.


(i) Variable Component = Change in cost / Change in capacity
= (Rs. 10.20 lakhs – Rs. 7.80 lakhs) /(90% - 60% )
= Rs. 2.40 lakhs /30%
= Rs. 0.08 lakhs for each 1% capacity

Variable selling overhead


At 50% capacity = Rs. 0.08 lakhs x 50 = Rs. 4.00 lakhs
At 90% capacity = Rs. 0.08 lakhs x 90 = Rs. 7.20 lakhs

(ii) Fixed selling overhead = Rs. 7.80 lakhs – (Rs. 0.08 lakhs x 60) = Rs. 3.00 lakhs

(i) Flexible Budget ( Rs. in lakh)


Capacity
50% 90%
Sales (A) 40.00 75.00
Direct material 10.00 18.00
Direct labour 12.50 22.50
Variable overheads
- Production 5.50 9.90
- Selling 4.00 7.20
Total Variable Cost 32.00 57.6
Fixed overheads
- Production 5.20 5.20
- Administration 2.00 2.00
- Selling 3.00 3.00
Total Fixed Cost 10.20 10.20
P.T.O.
(64)
Total cost (B) 42.20 67.80
Net profit / (loss) (A) – (B) (2.20) 7.20

(ii)
(a) Loss to be incurred if stoppage of operations (Rs. Lakhs)
Lay off compensation 1.20
Fixed overheads (Rs. 10.20 lakhs x 40/100) 4.08
Loss if operations are closed 5.28
(b) Loss if continue production at 50% level
Loss would be Rs. 2.20 lakhs (Calculation given above)

(c) Profitability if production is at 90% capacity (Rs. Lakhs)


Profit (as calculated above) 7.20
Less: Additional cost due to purchase of Power from Private agency
(Rs. 9.90 x 40/100) 3.96
Net profit 3.24

(d) Profitability of operation at 50% capacity and sub-contracting the balance 40%
(Rs. Lakhs)
Total cost - at 50% capacity 42.20
Sub-contract charges (Rs. 15.00 lakhs x 2) - for balance 40% capacity 30.00
Variable selling overhead (Rs. 7.20 lakhs – Rs. 4 lakhs) – for 40% capacity 3.20
Total cost 75.40
Loss (balancing figure) 0.40
Sales 75.00

Recommendations: From analysis of above alternative C is most profitable with which


the company can earn a profit of Rs. 3.24 lakhs. Hence, operation at 90% capacity
with the purchase of power from private agency is the suggested mode of action.
2.
a) A company manufactures a product from a raw material, which is purchased at
Rs. 54 per kg. The company incurs a handling cost of Rs. 350 plus freight of
Rs. 400 per order. The incremental carrying cost of inventory of raw material is
Re. 0.50 per kg per month. In addition, the cost of working capital finance on
the investment in inventory of raw material is Rs. 8 per kg per annum. The
annual production of the product is 94,500 units and 2 units are obtained from
one kg of raw material.
Required: (4+1+5=10)
i) Calculate the economic order quantity of raw materials.
ii) Advise, how frequently orders should be placed for procurement.
iii) If the company proposes to rationalize placement of orders on quarterly
basis, what percentage of discount in the price of raw materials should be
negotiated?
b) The standard set for a chemical mixture of a firm is as under;
Material Standard mix % Standard price per Kg. (Rs.)
A 40 20
B 60 30
The standard loss in production is 10%. During a period, the actual
consumption and price paid for a good output of 182 kg are as under:

Material Quantity in Kg. Actual price per Kg. (Rs.)


A 90 18
P.T.O.
(65)
B 110 34
Calculate the variance. 10

Answer
a)

(i) EOQ = 2AO


√ C
A = Annual consumption = 94,500units x 1kg. = 47,250 kgs.
2 units

O = Cost of placing order = Handling cost + Freight = Rs. 350 +Rs. 400 = Rs. 750

C = Carrying cost per unit per annum


Carrying cost (Re. 0.50 × 12) =Rs. 6
Finance charges on investment in inventory = Rs.8
Rs.14

EOQ = 2 x 47,250 x 750 = 2,250 kgs.


√ 14

(ii) Number of orders = 47,250 kgs./ 2,250 kgs. = 21 orders


Frequency in placing orders = 365 days / 21 orders = 17 days

(iii) If company places orders on quarterly basis, percentage of discount in price of raw material
to be negotiated :

Cost under EOQ :


Ordering cost 21 orders × Rs. 750 Rs.15,750
Carrying cost 2,250 kgs. × ½ × Rs. 14 Rs.15,750
Total cost Rs.31,500

Cost under Ordering on Quarterly Basis :


Ordering cost 4 orders × Rs. 750 Rs. 3,000.00
Carrying cost (47,250 kgs./4 orders )× ½ × Rs. 14 Rs. 82,687.50
Total cost Rs. 85,687.50

Incremental cost if orders are placed on quarterly basis = Rs.85,687.50 – Rs.31,500.00


= Rs. 54,187.50

Reduction in purchase price to be negotiated = Rs. 54,187.50/47,250 kgs.


= Rs. 1.15 per kg.

Percentage of discount to be negotiated = Rs. 1.15 × 100


Rs.54
= 2.13%
b)
Take the good output of 182 kgs. The standard quantity of material required for 182 kg of output is
=182/90×100=202.22kgs.
Statement showing the standard and actual costs and standard cost of actual mix
Qty Kg. Rate Amt Rs. Qty Rate Amt Rs Qty Rate Amt
Rs Kg. Rs kg. Rs. Rs.
A(40% of 80.89 20 1,617.80 90 18 1,620 90 20 1800
P.T.O.
(66)
202.22 kg)
B(60%of 121.33 30 3,639.90 110 34 3,740 110 30 3,300
202.22 kg)
Total input 202.22 26 5,257.70 200 26.80 5,360 200 25.50 5,100

(-)Loss 20.22 -- 18 --
Total output 182.00 28.89 5,257.70 182 29.45 5,360 -- --

Standard yield in actual input is 90 % of 200 kg i.e 180 kg.


Variances:
i. Price Variance=Actual qty.(Std price-actual price)
=Rs 5,100-Rs 5,360=Rs 260(A)
ii. Mix Variance=Total actual qty.of input (Std .cost per unit of Std.mix-Std.cost per unit of actual
mix)
= 200 Kgs (Rs.26-Rs.25.50) = Rs.100 (F)
iii. Total usage variance=Std. price (Std qty-Actual qty.)
=Standard cost-Standard cost of actual quantity
=Rs 5,257.70-Rs 5,100=Rs157.70(F)
iv. Yield variance=Standard price of yield .(Actual yield-Std yield)
=Rs 28.89 (182-180)=Rs 57.70(F)
v. Total variance=Std cost-Actual cost=Rs 5257.70-Rs 5,360
=Rs 102.30(A)

3.
a) A textile company purchases cotton from the farmers and produces shirtings as
final product. Cotton is processed into two departments namely weaving
department and Dying Department. The following are the cost Details for the
two departments for the month of January, 2016.
Weaving Dying
Department. Department
Capacity 7200 Hours 3000 Hours
(Rs.) (Rs.)
Direct Labour 1,72,800 72,000
Material Consumed 1,80,000 64,000
Depreciation 30,000 10,000
Overhead apportioned 15,000 3,200
Power Consumption per hour @ Rs. 96 32
3.20 per unit
During the month both departments worked at 80% of their capacity and out of
these 400 hours were expected to be lost due to unavoidable reasons. The
normal processing time to process 100meter of raw product is 3.5 hours and 2
hours in weaving department and dying department respectively.
At the end of the month 1,00,000 meter of completed shirting were produced
and 50,000 meter of the shirting were in incomplete condition on which
processing in dying department is needed. There was no stock at the beginning
of the month. No power is consumed during idle time.
Required: 8
i) Machine hour rate for the two departments.
ii) Cost of 1,00,000 meter of completed shirtings.
iii) Cost of abnormal idle time to be charged to costing profit and loss account.
b) The financial books of a company reveal the following data for the year ended
31st March, 2016:
P.T.O.
(67)

Amount(Rs.)
Opening Stock
Finished goods 625 units 53,125
Work in progress 46,000
01.04.2015 to 31.03.2016
Raw materials consumed 8,40,000
Direct Labour 6,10,000
Factory Overheads 4,22,000
Administrative Overheads 1,98,000
Dividend Paid 1,22,000
Bad Debts 18,000
Selling and Distribution Overheads 72,000
Interest Received 38,000
Rent Received 46,000
Sales 12,615 units 22,80,000
Closing Stock
Finished Goods 415 units 45,650
Work-In-Progress 41,200
The cost records provided as under:
 Factory overheads are absorbed at 70% of direct wages.
 Administrative overheads are recovered at 15% of factory cost.
 Selling and distribution overheads are charged at Rs. 3 per unit sold.
 Opening stock of finished goods is valued at Rs. 120 per unit.
 The company values work-in-progress at factory cost for both financial and
cost profit reporting.
Required: 8
i) Prepare statements for the year ended 31st march, 2016 show
 The profit as per financial records
 The profit as per costing records
ii) Present a statement reconciling the profit as per costing records with the
profit as per financial records.
c) What is a Key Factor? How decisions are made when one of the production
resources is a Key Factor? 4
Answer
a)
1. Computation of machine hour rate

Weaving Department Dying Department


Depreciation 30,000 10,000
Overhead Apportioned 15,000 3,200
45,000 13,200
A
Normal Production Hrs (7,200*80%)-400=5360 Hrs (3000*80%)-400=2000hrs
B
Rate per Hour (A/B) 8.39 6.60
C
Power consumption cost per 96.00 32.00
hour
Machine Hour Rate 104.39 38.60

P.T.O.
(68)
2. Cost of 1,00,000 meter of completed shirtings
Materials - Weaving Dept. = 180,000/150,000 * 100,000 = 120,000
Dying Dept. = 64,000
Labour -Weaving Dept. = 172,800/150,000 * 100,000 = 115,200
Dying Dept. = 72,000
Overhead - Weaving Dept.=100,000/100*3.5*104.39 = 365,365
Dying Dept. 100,000/100*2*38.60 = 77,200
Total Cost 813,765

3. Cost of abnormal idle time to be charged to the costing profit and loss account
Weaving Department Dying
Department
Total working Hrs at 80% capacity 5760 2400
Less: Normal Idle Time 400 400
Normal Production Hour A 5360 2000
Hours for production:
Weaving Department ( 150,000/100) mtr
* 3.5 hrs B 5250

Dying Department ( 1000mtr*2hrs) 2000


Abnormal Idle Time C 110 Nil
Abnormal Idle Time Cost 110 hrs * Rs 8.39 = Rs
922.90
b)
1. Statement of profit as per financial records
( for the year ended March 31st , 2016)
Amount(Rs) Amount(Rs)
To opening stock of finished 53,125 By Sales 22,80,000
goods
To work in progress 46,000 By closing stock 45,650
of finished goods
To raw materials consumed 8,40,000 By work-in- 41,200
progress
To Direct Labour 6,10,000 By rent received 46,000
To factory overheads 4,22,000 By interest 38,000
received
To Admin overheads 1,98,000
To Selling and Distribution 72,000
overheads
To Dividend paid 1,22,000
To Bad Debts 18,000
To Profit 69,725
24,50,850 24,50,850

Statement of profit as per costing records


(for the yended March 31st, 2016)
Amount(Rs)
Sales Revenue(A) 22,80,000
(12,615 units)

Cost Of Sales:
Opening Stock 75,000
(625 units * Rs. 120)
Add: Cost of production of 12,405 units (W.N 1) 21,64,070
P.T.O.
(69)
Less: Closing Stock 72,397
(Rs.21,64,070*415units)/12,405 units
Production cost of goods sold(12,615 units) 21,66,673
Selling & Distribution Overheads 37,845
(12,615 units * Rs. 3)
Cost of Sales: (B) 22,04,518
Profit (A)-(B) 75,482

2. Statement of Reconcilation
(Reconciling the profit as per costing records with the profit as per financial records)
(Rs.) (Rs.)
Profit as per Cost Accounts 75,482
Add: Administration Overheads over absorbed 84,270
(Rs.2,82,270-Rs.1,98,000)
Opening Stock Overvalued 21,875
(RS. 75,000- Rs 53,125)
Interest Received 38,000
Rent Received 46,000
Factory Overheads over recovered 5,000
(Rs 4,27,000- Rs 4,22,000)
2,70,627
Less: Selling and Distribution Overheads Under Recovered 34,155
( Rs 72,000- Rs 37,845)
Closing Stock Overvalued( RS 72,397-Rs 45,650) 26,747
Dividend 1,22,000
Bad Debts 18,000 (2,00,902)
Profit as per financial accounts 69,725
Working Note:1 Rs.
Raw materials consumed 840,000
Direct labour 610,000
Prime cost 1,450,000

Factory overheads
70% of direct wages 427,000
1,877,000
Add: opening WIP 46,000
Less; Closing WIP (41,200)
Factory cost 1,881,800
Administrative overheads
15% of factory cost 282,270
Cost of production 2,164,070
c)
The marginal costing technique provides that the product with highest contribution per unit is
preferred. This inference holds true so long as it is possible to sell as much as it can produce.
But sometimes an organization can sell all it produces but production is limited due to scarcity
of raw material, labour, electricity, plant capacity or capital.
These are called key factors or limiting factors. A key factor or limiting factor puts a limit on
production and profit of the firm. In such situation, management has to take a decision whose
production is to be increased, decreased or stopped. In such cases, selection of the product is
done on the basis of contribution per unit of scarce factor of production. The key factor or
scarce factor should be utilized in such a manner that contribution per unit of scarce resource
is the maximum.
P.T.O.
(70)
Mathematically,

Profitability = Contribution
Key Factor

For example, if raw material is the limiting factor, the profitability of each product is
determined by contribution per Kg of raw material. If machine capacity is a limiting factor
then contribution per machine hour is calculated. It electricity is the limiting factor, then
contribution per unit of electricity of each product is calculated.
In case of Key factor situation, procedure of decision making is as under:
• Identify key factor
• Compute Total Contribution
• Compute Contribution per unit of Key Factor
• Rank the products based on Contribution per unit of key factor
• Allocate the key resources on basis of ranks.
4.
a) Arnav Automobiles distribute its goods to a regional dealer using a single
lorry. The dealer‟s premises are 20km away by roads. The lorry has a capacity
of 10 tones and makes the journey twice a day 90% loaded on the outward
journeys and 10% on return journeys. The following information is available
for a four weekly period during the year 2015:
Petrol Consumption 6 Km. per litre
Petrol Cost Rs. 64 per litre
Lubricants Rs. 125 per week
Driver‟s Salary Rs. 2,000 per week
Repairs Rs. 1,250 per week
Garage Rent Rs. 1,30,000 per annum
Cost of lorry ( excluding tyres) Rs. 9,80,000
Life of lorry 80,000 km
Insurance Rs. 13,650 per annum
Cost of Tyres Rs. 28,000
Estimated value of lorry at end of its life Rs. 1,80,000
Vehicle Licence Cost Rs.5,200 per annum
Other Overhead Costs Rs.45,500 per annum
Life of Tyres 28,000 Kilometers
The lorry operates six days on aweek

Required: 5
i) A statement to show the cost of operating the vehicle for the four weekly
period.
ii) Calculate the vehicle cost per Km and per tone Km.
b) Maxim Ltd. manufacture a product “N-joy”. In the month of August 2016,
14,000 units of the product “N-joy” were sold, the details are as under:
(Rs.)
Sales Revenue 2,52,000
Direct Material 1,12,000
Direct Labour 49,000
Variable Overheads 35,000
Fixed Overheads 28,000

P.T.O.
(71)
A forecast for the month of September 2016 has been carried out by the
General Manager of Maxim Ltd. As per the forecast, price of direct material
and variable overhead will be increased by 10% and 5% respectively.
Required: 5
i) Number of units to be sold to maintain the same quantum of profit that
made in August 2016.
ii) Margin of safety in the month of August 2016 and September 2016.
c) The XYZ Company owns and operates a chain of 50 stores. Budgeted data for
the stores no. 25 and store no. 50 are as follows:
Stores no. 25 Store no. 50
Annual sales Rs. 4,50,000 Rs. 5,00,000
Annual cost of sale and other operating cost Rs. 3,82,000 Rs. 4,25,000
Annual building ownership cost not included above) Rs. 20,000 Rs. 40,000
The company lease the building to a large flower shop for Rs. 4000 and Rs.
5000 the stores no. 25 and store no. 50 per month respectively. Decide whether
to continue operations of these stores or lease out. 5
Answer
a. Statement of total cost of operating the vehicle for the four weekly period
Particulars Rs. Rs.
A Running costs:
Petrol Cost ( W.N 1) 20,480
Lubricant ( Rs. 125 * 4 weeks) 500
Depreciation( 19,200

Depreciation(Tyre) 1920 42,100


(Rs.28,000/28,000km)*1920km.
B Standing Charges
Drivers salary ( Rs. 2000 * 4 weeks) 8,000
Garage rent ( 1,30,000/52*4) 10,000
Insurance ( Rs. 13,650/52 *4) 1,050
Vehicle Liscence ( Rs. 5,200/52 * 4) 400
Other Overhead Cost ( 45,500/52*4) 3500 22,950
C Maintenance Cost

Repairs ( Rs 1,250 * 4 weeks) 5,000 5,000


Total Cost( A+B+C) 70,050

b. Vehicle Cost per kilometer = = Rs 36.48


Cost per tone-km .
Outward Journey: 20km * 9 tonne* 24 days* 2 trip = 8,640
Inward Journey : 20 km * 1 tonne * 24 days* 2 trip = 960
Total Tonne-km 9,600

Cost per tone-km = = Rs 7.30

Working Note
1. Distance travelled = 20km * 2 ways*4 weeks* 6 days* 2 trips = 1,920km
Cost of petrol = = Rs 20,480

P.T.O.
(72)
b)
Calculation of Profit made in the month of August 2016 by selling 14000 units.
Amount per unit (Rs.) Amount (Rs.)
Sales Revenue 18.00 2,52,000
Less: Variable Costs:
- Direct Material 8.00 1,12,000
- Direct Labour 3.50 49,000
- Variable Overhead 2.50 35,000
Contribution 4.00 56,000
Less: Fixed Overhead 2.00 28,000
Profit 2.00 28,000

(i) To maintain the same amount of profit i.e. Rs. 28,000 in September 2016 also, the
company needs to maintain a contribution of Rs. 56,000.
Let, number of units to be sold in September 2016 is „x‟, then the contribution will be
Rs. 18x – [(Rs. 8*1.10) + Rs. 3.5 + (Rs. 2.5*1.05)] x = Rs. 56,000
Rs. 18x – ( Rs. 8.8 + Rs. 3.5 + Rs. 2.625) x = Rs. 56,000

Or, x =

= 18,211.38 units or 18,212 units.

(ii) Margin of Safety


August 2016 September 2016
Profit Rs. 28,000 Rs. 28,000
P/V Ratio

Rs. 1,26,000 Rs.1,63,902.44

Margin of safety

c)

Particulars Continue Operation Lease out


Store no 25 Store no 50 Store no 25 Store no 50
1,Sale Revenue/Lease 4,50,000 5,0,0000 48,000 60,000
rent
Annual cost of sale and 3,82,000 4,25,000 - -
Other operating cost
Ownership cost 20,000 40,000 20,000 40,000
2.Total cost 4,02,000 4,65,000 20,000 40,000
3. Profit/saving(1-2) 48,000 35,000 28,000 20,000
Decision
The above analysis shows that store no 25 and store no 50 continue.
5.
a) Rex Ltd. commenced a contract on1-7-2015 .The total contract price was Rs.
5,00,000 but Rex Limited accepted the same for Rs. 4,50,000. It was decided to
estimate total profit and take to the credit profit & loss Account that proportion
of estimated profit on cash basis which the work completed bore to the total

P.T.O.
(73)
contract. Actual expenditure till 31-12-2015 and estimated expenditure in 2016
are given below:
Expenses Actual till 31-12-2015(Rs) Estimated for 2016(Rs)
Materials 75,000 1,30,000
Labor 55,000 60,000
Plane Purchase(original cost) 40,000 -
Miscellaneous Expenses 20,000 35,500
Plant returned to Store at 10,000 25,000
Original Cost on 31.12.2015) (on 30.9.2016)
Material At site 5,000 -
Work Certified 2,00,000 Full
Work Un Certified 7,500 Nil
Cash Received 1,80,000 Full

The plant is subject to annual depreciation @20% of original cost .The contract
is likely to complete on 30.9.2016.
It is the policy of the company to charge depreciation on time basis.

You are require to prepare the contract account for the year ended 31.12.2015. 7
b) What are the ways for the treatment of defective work? 4
c) Write short notes on Zero-Base Budgeting (ZBB). 4
Answer
a)
Rex Limited
Contract Account for the year ending 31-12-2015
Particular Amount Particular Amount
Rs. Rs.
To Material 75,000 By plant returned to 9,000
store
To Labor 55,000 By plant at site 27,000
To plant 40,000 By material at site 5,000
To misc. Expenses 20,000 By Work in progress
Work Certified 2,00,000
Work Un Certified 7,500
To P&L A/c 26,400
To working progress 32,100
Reserve
2,48,500 2,48,500
Working notes
1. Memorandum Contract Account

1-7-2015to 30-9-2016
Particular Amount Particular Amount
Rs. Rs.
To Material(75,000+1,30,000) By plant return to 27,750
2,05,000 stores(9,000+18,750)
To Labor (55,000+60,0000 1,15,000 By plant at site(5,000- 3,750
1,250)
To plant 40,000 By contractee‟s account 4,50,000
To misc. Expenses (20,000+35,500) 55,500
To Estimated P&L A/c 66,000

P.T.O.
(74)
4,81,500 4,81,500

2.Profit to be transfer to Profit &Loss A/C on the contract ending on 31st December 2015
=Estimated profit× Cash Received× Work Certified
Work certified Total contract Price
=Rs 66,000× Rs1,80,000× Rs,2,00,000
Rs2,00,000 Rs4,50,000
=Rs 26,400
3.(1) Plant return to store on 31-12-2015
Original cost Rs 10,000
Less : Depreciation @20% for six month Rs 1000
Rs 9,000
(2) Plant at site on 31-12-2015
=Original cost of plant-Plant return-Depreciation
= 40,000-10000-3000
=Rs.27,000
(3) ) Plant return to store on 30-9-2016
Original cost Rs 25,000
Less Depreciation @20% for six month Rs 6,250
Rs 18,750
(4)Plant at site on 30-9-2016#
Original cost Rs 5,000
Less: Depreciation for Fifteens months Rs 1,250
Rs 3,750
# Alternatively, it could have also been assumed that plant costing Rs. 5,000 (i.e. 40,000-
35,000) was exhausted at the contract site itself during 2016 and it was a normal loss.
Consequently the figure of the profit etc. would have changed accordingly.

b) The possible ways for the treatment of defective work are as below:
i) Defectives that are considered inherent in the process and are identified as normal
can be recovered by using the following methods:
a. Charged to good products-The loss is absorbed by good units .This method is used
when seconds or first are normal:
b. Charged to general overheads-When the defectives caused in one department are
reflected only on further processing, the rework costs are charged to general
overheads;
c. Charged to Department overheads-If the department responsible for defectives can
be identified then the rectification costs should be charged to that department.
d. Charged to Costing Profit and loss Account-If defectives are abnormal and are due
to causes beyond the control of organization ,the rework cost should be charged to
Costing Profit and Loss Accounts
ii) Where defectives are easily identifiable with specific jobs, the work costs are
debited to the job.

c) Zero Base Budgeting is a method of budgeting starting from scratch or zero level.
Proposals for the coming period should be based on merit and not related to past
performance.
Budgets prepared by conventional methods are the incremental type of budget based
on actual performance in the past periods. In the zero base budget, the results of the
past year is not accepted as a basis, since the past may conceal inefficiencies.
Zero Base Budget is mainly prepared by taking the following steps.
(i) Identification of decision units
(ii) Preparation of decision packages.
(iii) Ranking of decision packages using cost benefit analysis.

P.T.O.
(75)
(iv) Allotment of available funds according to the priority determined by ranking
each decision package is a self contained module explaining the need for a
certain activity, its costs, its benefits consequences if the packages is not
accepted etc. The ranking of package based on cost benefit analysis by the
difficult levels of management starring from the bottom upward ensures
allotment of funds to relatively more important and essential activities.
6. Write short notes on: (4×2.5=10)
a) Cost accounting system
b) Abnormal gain
c) Margin of safety
d) Value analysis
Answer
a) Cost accounting system.
A cost accounting system is a system that accumulates costs, assigns them to cost objects, that
is products, jobs, process, etc. and reports cost information. In addition to this, a proper cost
accounting system assist management in the planning and control of business operation, in
analyzing product profitability, and in accomplishing business objectives through optimum
utilization of available resources.
b) Abnormal gain
More output over the expected or normal output realized is called an abnormal gain.
Abnormal gain arises because of an abnormal effective in the use of raw material or efficiency
in performance so it is known as abnormal effective. Abnormal gain reduces the normal loss
quantity so it comes in the form of profit to the industry.
The process account under which abnormal gain arise is debited with the abnormal gain. The
cost of abnormal gain is computed on the basis of normal production.
Method of determining the value of abnormal gain:
Value of abnormal gain = (Normal cost of normal output/Normal output) Abnormal gain units
c) Margin of safety
The margin of safety represents the difference between the sales at the break-even point and
the total sales. It can be expressed as a percentage as well as in value. The size of the margin
of safety shows the strength of the business. If the margin of safety is small, it may indicate
that the firm has large fixed expenses and is more vulnerable to small changes to sales.
d) Value analysis.
Value analysis is the process of systematic analysis and evaluation of various techniques and
functions with a view to improve organizational performance. It aims at reducing and
controlling the cost of a product from the point of view of its value by analyzing the value
currently received. Value analysis basically centers around determining the essential
characteristics of a product that the customer requires and determining the most economical
method of producing it by balancing cost with the utility of the product.

P.T.O.
(76)

Business Communication

P.T.O.
(77)

Suggested Answer
Roll No……………. Maximum Marks –100
Total No. of Questions - 8 Total No. of Printed Pages -5
Time Allowed –3 Hours
Marks
All questions are compulsory.
Section -'A'

1. Read the following case carefully and answer the questions given below: (4×5=20)
Jack Simpson, newly appointed human resources director for Geridan Contracting
Corporation (GCC), had had an unusual morning. First on his agenda was an exit
interview with Maria Johnson, the company president‟s executive secretary.
Johnson has simply informed Simpson she was quitting, giving no reason. Judging
from her performance reviews over the last few years, Simpson believed her to be
competent, enthusiastic, and dedicated employee. Even though Simpson had little
knowledge of her workload, he could see no obvious reason for the resignation. He
had set up this exit interview hoping to find out why she had quit.
Next on Simpson‟s agenda was an interview with Ryan Ross, the president of
GCC, who wanted to talk to Simpson before Simpson began interviewing later in
the day for the secretary‟s replacement. Simpson had never conducted an interview
for a president‟s secretary before, but he had planned on getting a good idea of
what to look for during his talk with Johnson. He believed Ross would also advise
him on what he expected from the secretary‟s replacement.
However when Simpson and the resigning executive secretary, Johnson, sat down
in a quiet conference room at 8:30 am, the HR director‟s ear began to burn.
Johnson explained that for the last six months she was being sexually harassed by
Ross and that she was considering suing GCC (and Ross in particular).
Simpson needs to know more about this accusation in case it does develop into a
more serious situation. Also, the more facts he has, the better prepared he will be to
discuss the situation with the president. What interview strategy should Simpson
use with the secretary?
Questions:
a) What are the problems in the above case?
b) Make a list of five probable questions Director Simpson would ask to Johnson.
c) If you were Johnson, how would you handle this situation?
d) What are the guidelines for taking an effective interview? Write in brief.
Answer:
a) Jack Simpson is new in his role as HR Director. On the very first day, he has to handle a
very difficult task, i.e. conduct an exit interview for Maria Johnson who is the secretary of
the company president. He does not know how to take exit interviews. He has never done
this before.
Next problem is that Maria blames the president of sexual harassment and Simpson does
not know how to handle such sensitive cases. He has to do justice to both – the victim and
the company.

Jack Simpson is also very much confused for he does not know how to discuss this issue
with the president (or the criminal?) who harassed his own secretary.

P.T.O.
(78)
b) Jack Simpson should try to know the situation as objectively as possible. He may ask her
the questions like (Examinees can write any five of them, or similar to them):
a. How long do you know Ryan Ross?
b. What kind of person is he as a president, and as a human being?
c. If your charges against him are true, why didn‟t you complain before?
d. Now you want to leave this organization, is this the only option that you have now?
e. How would you like the president punished?
f. Would you file a case against him?
g. What kind of help, in this regard, do you want from this company?
h. If the culprit were punished, would you like to continue your job?

c) If I were Maria Johnson, I would not have tolerated the director at all. In the very
beginning I would collect all the evidences against him and file the case. I would not
worry about my job for I believe on my capacity and talent. I wonder why Maria tolerated
him for about six months.
I believe that justice comes sooner or later, and we should all the time be ready to fight
against all sorts of discriminations and social ills. Sexual harassment is a heinous crime
and the culprit must be thrown behind the bar.

d) Interviewing is one of the critical elements in collecting information. Here are some
guidelines for conducting an effective interview.
a. Plan in advance
b. Control the environment
c. Use appropriate questions
d. Maintain a relaxed atmosphere
e. Encourage the candidate to ask questions
f. Conclude the session with appreciation or gratitude

2. Some say that there is no difference between ethics and morals. What are your
views? Explain the importance of ethical behavior at the workplace. (5+5=10)
Answer:
There is a fine distinction between ethics and morals which may be enumerated as follows:
The word „Moral‟ is defined as relating to principles of right and wrong. The root word for
moral is Latin word „mos‟ meaning custom while the root word for ethics is the Greek word
„ethos‟ meaning character. Custom and character however provide two different standards
for defining what is wrong and what is right. Character is a personal attribute while custom
is defined by a group overtime and people have character while societies have custom.

Morals are accepted from an authority i.e. culture, religion etc. while ethics are accepted
because they follow from personally accepted principles.

Morals work on a smaller scale than ethics, more reliably but by addressing human needs
for belonging and emulation, while ethics has much wider scope.
An organisation is first and foremost a human society. If an employer does not take steps to
create a work environment where the employees have a clear, common understanding of
what is right and wrong and feel free to discuss and ask questions about ethical issues and
report violations, significant problems could arise, including:
- increased risk of employees making unethical decisions;
- increased tendency of employees to report violations to out side regulatory authorities
(whistle blowing) because they lack an adequate internal forum;
- inability to recruit and retain top executives;
P.T.O.
(79)
- diminished reputation in the industry and the community; and
- significant legal exposure and loss of competitive advantage in the market place.

3.
a) Present precisely the stepwise process of organizational communication, and
then analyze the process of communication involved in the following stretch of
business discourse. 5

Mr. Mike: Excuse me. I've to have it exchanged.


Salesperson: Well, would you please show what it is?
Mr. Mike: That I had this morning… I came this afternoon too,
but she didn't listen to any complain.
Salesperson: Oh, you should have brought the warranty card,
and requested the manager in person.

b) As a country director of a multinational business company, you have organized


a press conference in order to disseminate new services launched by your
company. You have noticed that the business reporters and other participants
were not satisfied with the presentation of the marketing director of your
company. Now, write briefly what feedback you would give to him. 5
Answer:
a) Communication is an exchange of ideas and information between people and it takes place
in different modes and contexts. Organizational goals are unlikely to be attained unless
there's no effective communication. Organizational communication involves the following
stepwise process:

Step 1: Sender has an idea, and he or she encodes that idea into message through linguistic
codes or symbols.
Step 2: Message travels through the channel.
Step 3: Receiver decodes the message.
Step 4: Receiver gives feedback/ response to the sender

In the given exchange, Mr. Mike has an idea that is concerned with exchanging the thing
that he had bought. This idea is abstract. He encodes the idea as a message into an utterance
or a sentence 'I've to have it exchanged'. Then, this message travels through the face to face
channel as a request to the salesperson. The salesperson tries to decode or make meaning of
that message. He cannot get appropriate meaning of the word 'it', and requests as response or
feedback to Mr. Mike about 'it'. Again, Mr. Mike encodes what he wants to say; then,
message travels to the salesperson, and the salesperson decodes it properly. He gives
appropriate response or feedback to Mr. Mike. This is the usual process of communication.

b) The marketing director would be provided with the following points of feedback ranging
from different perspectives of business communication such as ethics of business
communication, group interactions and listening skills, cross-cultural communication, and
so on.
 Press conference and press release are different forms of public information from the
advertisements. You as a marketing director should not have compared the quality of our

P.T.O.
(80)
service with that of other particular company. Explicit interpretation of our new services
would be enough.
 Another important point to consider is that business organizations need to bear the
responsibility for the welfare of the social issues, and you ignored this aspect. It's not
ethical in business to falsely present the data in an inconsistent manner. The audience
easily found out the false claim that you had made about the previous year's sales.
 Today's buzz word in business communication is 'multicultural sensitivity'. You seemed
totally unconcerned to this issue during your presentation. You were overemphasizing the
role of the Internet in the hilly region of the country among the children of the high class
family.
 During your presentation you looked rather nervous and unplanned. You were not trying
to listen to the complains of the audience, but were intending to impose your own
thoughts.

4. Briefly explain any FOUR of the following: (4×2.5=10)


a) The analytical report
b) Role of a chairperson in business meeting
c) Difference between team and group
d) Interpersonal communication
e) Buffers
Answer:
a) The analytical report
The analytical report examines a problem or issue and recommends an action. Some
analytical reports function as proposals that identify or define problems and argue for
specific ways of resolving them. Other analytical reports are feasibility studies that examine
proposed solutions and determine their practicality.
Experts write analytical reports, often in teams. The reports generally require that these
experts use their professional skills to define an issue, making the study large enough in
scope to identify all the factors that bear on the problem and using a standard professional
methodology, know or learn how such issues have been resolved in other cases, inside and
outside their own company; and accommodate constraints like cost, time, company policy,
union contracts, local and federal law.
b) Role of a chairperson in business meeting
In a more formal meeting, the chairperson will outline the purpose of the meeting and
remind members why they are there. In such a meeting there is little need to refer to this
procedure, as this is implicit in the established etiquette, namely:
 The chair controls the meeting.
 All remarks are addressed through the chair.
 Members do not interrupt each other.
 Members aim to reach a consensus.
 A vote is taken if consensus is not reached.
 The majority wins the vote.
 All members accept the majority decision.

c) Difference between group and team


The words 'group' and 'team' are, for the most part, interchangeable - at least most people
use them that way. But there are clear differences between groups and teams. For example,
we have a football team, not a football group - or we have a special interest group, not a
special interest team. While the differences are subtle, they are indeed different, and we
need to understand what those differences are.

P.T.O.
(81)
The main difference is that a team's strength or focus depends on the commonality of their
purpose and how the individuals are connected to one another. On the other hand, a group
can come from having a large number of people or a cohesive willingness to carry out a
focused action - political reform, for example.
While these differences might be subtle, we have to understand that a group is a number of
individuals forming a unit for a reason or cause, and a team is a collection of accomplished
people coming together for a common goal that needs completion. The subtleness of these
differences is more pronounced when we take these words a step further and look at a work
group and work team.

d) Interpersonal communication
Interpersonal communication is the sending and receiving of messages between two or more
people. Everyone communicates in life and business; however, various barriers can exist
that distort the message. Individuals must be able to identify the barriers and find ways to
eliminate them in a way that improves the communication flow. Learning a few techniques
or using proper preparation can help individuals tailor their communication in a way that
provides focus and improves the message.

Effective interpersonal communication is an ongoing process. As new messages come up or


audiences change, senders must tailor their messages to ensure effective communication.

e) Buffers
Buffers are the tools for creating indirect but courteous relationship with the people in
written transactions such as bad news letters, follow up letters, and other correspondences
which might create resistance in the readers‟ mind. A buffer is useful to establish common
ground with the reader. But, a poorly written buffer can mislead and sometimes insult the
reader. Good buffers balance different elements of communication, and give good
impression to the readers. Agreement, appreciation, cooperation, politeness, fairness, etc. are
some of the types of buffers which are important in most of the business correspondences.

P.T.O.
(82)

Marketing

P.T.O.
(83)

Suggested
Roll No……………. Maximum Marks – 50
Total No. of Questions - 4 Total No. of Printed Pages -6
Time Allowed – 3 Hours
Marks
All questions are compulsory.
Section -'B'

5. Read the following case carefully and answer the questions given below: (45=20)
Considering the fact that men also want flawless skin and are as conscious about it as
women are, Emami Limited launched Fair and Handsome crème – a perfect fairness
cream exclusively for men. It was introduced in the Nepali market simultaneously with
its launch in India in 2005 with the vision to groom men.
Fair and handsome comes in number of ranges – Fair Cream, Face Wash, Oil Control
Cream, Oil Control Face Wash, Fair and Handsome Moisturizing Lotions, Sun
Protection Lotions and Daily Moisturizing Lotions among others. “The texture of
man‟s skin is different from that of a woman, and Fair and Handsome will perfectly
suit all skin types of men. The product is an outcome of a lot of research,” says Madhu
Bashyal, Brand Manager at Emami Limited.
The products were developed by Indian and International Skin care experts using
scientifically tested and clinically proven combinations of natural vitamins, anti-
irritants and sunscreen filters that protect the skin from harmful UVA and UVB rays of
the sun. To maintain the quality of the products, the company carries out strict and
regular quality check and periodic market research to update it.
The brand uses one of the most widely tested and demonstrated skin lightening active
technologies, vitamin B3 that safely works from within to lighten the skin and the dark
spots. The advanced formulation has been designed with a combination of special
„micro absorbers‟ that absorb away the excessive oil and sweat, leaves skin fresh,
sweat and oil free, looking fair and fresh for long time with SPF 15+ protecting from
harmful UV rays of sun that will damage skin and prevents from darkening.
For the promotion of the brand, they are advertising through all media channels
possible. Likewise, they are also into ATL and BTL activities for promotional
campaigns. “The brand holds 80 per cent market share. We plan to be a category
leader and aim to also convert non-users. We spread awareness about our product
through ATL activities,” says Basyal.
Stating that visibility in the market is of utmost importance and branding is an integral
part of promotion, Basyal says, “The Company invests around Rs seven million
annually for branding and promotion and our annual turnover is approximately Rs. 100
million.” Similarly, it is associated with different events through sponsorship and
participation in exhibitions in the city and in regional level exhibitions besides regular
print and electronic advertisement.
Talking about competition, Basyal says, “We are competing with Garnier and Nivea in
the men‟s skincare segment.” The brand, Fair and Handsome, is believed to be the
largest brand in the men‟s fairness segment and second largest in the overall fairness
category in Nepal and in India.
Questions:
a) Give your opinion about the Fair and Handsome Crème.
b) What makes the Fair and Handsome Crème best?
c) Explain the promotion strategies applied in marketing the product.
d) Can this product attract new customers? Why?

P.T.O.
(84)
Answer
a) Fair and handsome crème is the product of Emami Limited. This product is targeted exclusively for
men. There are varieties of fair and handsome crèmes; including fair cream, face wash, oil control
cream, oil control face wash, fair and handsome moisturizing lotions, sun protection lotions and daily
moisturizing lotions. Fair and handsome crème is an outcome of a lot of research works so will
perfectly suit all skin types of men.
b) Not only fair and handsome crème, actually, all products of Emami Limited were developed by
Indian and International Skin care experts using scientifically tested and clinically proven
combinations of natural vitamins, anti-irritants and sunscreen filters that protect the skin from harmful
UVA and UVB rays. To maintain the quality of the products, the company carries out strict and
regular quality check and periodic market research to update it. Therefore, the fair and handsome
crème can be regarded as the best crème.
c) In order to improve the brand equity of the product, the company has invested around 7 million
rupees annually where as the annual turnover is approximately Rs 100 million. Company also invests
in different events through sponsorship and participation in the city and in regional level exhibitions
besides regular print and electronic advertisement.
d) Considering the facts that the product was scientifically tested and clinically proven along with the
regular quality control, we can believe that the product can attract new customers in the product line
group. The aggressive promotion strategy of the company is the additional benefit to this effort. The
company authority says that the company is competing with Garnier and Nivea in the market
segment. The company authority is believed that the brand may be the largest brand in the men‟s
fairness segment.

6. What is marketing? Describe the new marketing concept, societal marketing


concept and holistic marketing concept in your own words. (3+7=10)

Answer:

Most people think that marketing is only about the advertising and selling of goods and services.
Advertising and selling, however, are just two of the many marketing activities. In other words,
selling and advertising are only the tip of the marketing iceberg.
Today, marketing must be understood not in the old sense of making a sale- “telling and
selling”- but in the new sense of satisfying customer needs. If a marketer does good job of
understanding customer needs; develops products that provide superior value; and prices, distributes
and promotes them effectively, these products will sell very easily. Thus selling and advertising are
only part of a marketing mix.
In general, marketing activities are all those associated with identifying the particular wants
and needs of a target market of customers, and then going about satisfying those customers better
than the competitors. This involves doing marketing research on customers, analyzing their needs and
then making strategic decisions about product design, pricing, promotion, and distribution. Thus,
marketing deals with identifying and meeting human and social needs.
According to Philip Kotler & Kevin Lane Keller, “Marketing is a societal process by which
individuals and groups obtain what they need and want through creating, offering, and freely
exchanging products and services of value with others.”
According to Stanton, Etzel & Walker, “Marketing is a total system of business activities
designed to plan, price, promote and distribute want satisfying products to target markets in order to
achieve organizational objectives.”
From the above definitions what we can say is customers‟ wants must be recognized and
satisfied. Entire system of business activities should be customer oriented. Marketing should start
P.T.O.
(85)
with an idea about a want-satisfying product and should not end until the customers‟ wants are
completely satisfied, which may be some time after the exchange is made.
Thus, in conclusion, marketing is meeting needs profitably.
Various concepts of marketing are explained as follows:
The Marketing Concept
The marketing concept holds that achieving organizational goals depends on determining the
needs and wants of target markets and delivering the desired satisfactions more effectively and
efficiency than competitors do. The marketing concept starts with a well-defined market, focuses on
customer needs, coordinates all marketing activities affecting customers, and makes profit by creating
long term customer relationship based on customer value and satisfaction.
Thus, under marketing concept, customer focus and the value are the paths to sales and profits. This
concept is based on “we make what we can sell.”
The Societal Marketing Concept
The societal marketing concept holds that the organization should determine the needs, wants, and
interests of target customers. It should then deliver superior value to costumer in a way that maintains
or improves the customer‟s and the society‟s well-being. The societal marketing concept questions
whether the pure marketing concept is adequate in an age of environmental problems, resource
shortages, rapid population growth, worldwide economic problems, and neglected social services. It
asks if the firm that senses, serves, and satisfies individual short term wants is always doing what‟s
best for consumers and society in the long run. According to the societal marketing concept, the pure
marketing concept overlooks possible conflicts between consumer short term wants and customer
long term welfare. Societal marketing concept calls on marketer to balance among the three
considerations: consumer, organization and society.

The Holistic Marketing Concept


Holistic marketing concept is a concept that tries to identify all the needs and expectations of
the target customers and tries to fulfill them all for their satisfaction. Because this concept assumes
that the customers may not be satisfied unless all their needs and expectations are fulfilled by the
marketing firm. So a marketer must be able to take all possible efforts to keep the customers satisfied.
The holistic marketing concept is based on the development, design, and implementation of
marketing programs, processes, and activities that recognizes their breath and interdependencies.
Holistic Marketing recognizes that “everything matters” with marketing – that a broad, integrated
perspective is often necessary. Four components of holistic marketing are relationship marketing,
integrated marketing, internal marketing and social responsibility marketing.
Holistic marketing is thus an approach to marketing that attempts to recognize and reconcile
the scope and complexities of marketing activities.

7.
a) Explain the meaning and components of marketing information system in brief. (2+3=5)
b) Differentiate between skimming pricing and penetration pricing strategies. 5

Answer:

a) Marketing information system is a set of procedures and methods for the regular, planned collection,
analysis and presentation of information for use in marketing decisions. It is an ongoing, organized
procedure to generate, analyze, disseminate, store and retrieve information for use in making
marketing decisions.
According to Philip Kotler, “A marketing information system (MKIS) consists of people, equipment
and procedures to gather, sort, analysis, evaluate, and distribute needed, timely and accurate
information to marketing decision makers.”

P.T.O.
(86)
Thus, marketing information system is the organized set of procedures, information handling
routines and reporting techniques designed to provide the information required for making marketing
decisions.”
Internal record system, marketing intelligence, marketing research and marketing decision support
system are the four major components of marketing information system which are explained as
follows:
i. Internal Record System
Internal record system collects various types of scattered data which are located inside the company.
The heart of the internal record system is the order-to-payment cycle.
Marketing managers rely on data from internal reports about orders, sales, prices, costs, inventory
levels, receivables, payables and so on. Today many companies organize information in databases-
customer databases, product databases, salesperson databases and so forth- and then combine data
from the different databases. Companies store these data and make them easily accessible to decision
makers to better plan, target, and track marketing programs.
ii. Marketing Intelligence System
As we know the internal records system supplies results data, but the marketing intelligence system
supplies happenings data.
Marketing intelligence is systematic collection and analysis of publicly available information
about competitors and developments in the marketing environment. It is a set of procedures and
sources managers use to obtain everyday information about developments in the marketing
environment.
Marketing managers collect marketing intelligence by reading books, newspapers, and trade
publications, talking to customers, suppliers, and distributors, checking internet sources and meeting
with other company managers.
iii. Marketing Decision Support System
Marketing decision support system (MDSS) is a coordinated collection of data, systems, tools, and
techniques with supporting software and hardware by which an organization gathers and interprets
information from business and the environment and turns it into a basis for marketing action. It is a
procedure that allows a manager to interact with data and methods of analysis to gather, analyze, and
interpret information.
A marketing decision support system may include statistical tools, mathematical tools as well as
financial and managerial tool and models.
iv. Marketing Research
Marketing research is a systematic design, collection, analysis and reporting of data and findings that
are relevant to a specific marketing situation facing by the company.
A company can obtain marketing research in a number of ways. Most large companies have
their own marketing research departments. Some small companies can hire the services of a
marketing research firm or conduct research in creative and affordable ways. They can engage
students or professors to design and carry out projects, they can use the Internet, and they can visit
their competitors.

b) Price skimming strategy: Price skimming is an over-charging strategy. It involves charging a


very high price during the introduction stage of the product life cycle. This strategy is suitable
only for innovative product that can't be compared with other existing products. Consumers are
not aware of its real value. There is a large size of market pioneers and early adaptors. Price
skimming is possible, when there is no competition in the market and entry barriers for new
competitors are also high.

P.T.O.
(87)
Penetration pricing strategy: Penetration pricing strategy involves introducing a new product at a
far lower price than the competitive price. This is an under changing strategy. This strategy is
adopted by an organization for an imitative product who has many brand substitute in the markets.
When consumers are aware of the real value the imitative product they can be persuaded to buy
only under a very low price. Marketers adopted this strategy to achieve rapid market expansion
during the introduction stage of product life cycle.

8. Briefly explain the following: (5×2=10)


a) Market
b) Warehousing
c) Advertising
d) Meaning of public relations
e) Intangible product

Answer:
a) Market
The market consists of all potential customers having a need or want who possess ability and
willingness to engage in exchange to satisfy that need or want. In other words, there should be:
 People with needs or wants to satisfy
 Money to spend
 Willingness to spend through exchange

Markets can be: Consumer, Business, Institutional, and Global.

b) Warehousing
Warehouse is a place where goods are stored for certain period for the distribution of goods to the
target market whenever and wherever required. While warehousing is a specialized system and
process of storing surplus goods (keeping buffer stock) for use or consumption in future because
goods may not have demand immediately after the goods are manufactured.

c) Advertising
Advertising is non-personal presentation and communication of product, service or organization
through paid media with identified sponsor. It is one of the element of marketing communication.
Various indoor , outdoor, direct or display media are used for advertising. It is mass
communication and message is controlled by advertisers. It is salesmanship in print.

d) Meaning of public relations


Public relation is a management tool designed to favorably influence attribute toward an
organization, its products and its policies. It is a social philosophy of marketing expressed in
policies and practices the relationship between marketing funds and publics can be maintained by
using external media and internal media. The basic tools that can be used in public relation
includes annual reports press conference, press release and so on.

e) Intangible product
P.T.O.
(88)
Intangible products are the goods whose ownership cannot be transformed from one person to
another. Such product cannot be touched seen, stored or owned. Such product can only be
realized. Before services are purchased, customers cannot handle examine and tried out. They also
cannot be measured like physical items.

P.T.O.
The Institute of Chartered Accountants of Nepal
Suggested Answers of Accounting

CAP II Examination – June 2009


Marks

Attempt all questions. Working notes should form part of the answer.

Question No. 1
‗A‘ and ‗B‘ are in partnership sharing profits and losses equally. They
keep their books by single entry system. The following balances are
available from their books as on 31.3.2006 and 31.3.2007:
31.3.2006 31.3.2007
NRs. NRs.
Building 1,50,000 1,50,000
Equipments 2,40,000 2,72,000
Furniture 25,000 25,000
Debtors ? 1,00,000
Creditors 65,000 ?
Stock ? 70,000
Bank loan 45,000 35,000
Cash 60,000 ?

The transactions during the year ended 31.3.2007 were the following:
NRs.
Collection from debtors 3,80,000
Payment to creditors 2,50,000
Cash purchases 65,000
Expenses paid 40,000
Drawings by ‗A‘ 30,000

 On 1.4.2006 an equipment of book value NRs.20,000 was sold for NRs.15,000.


On 1.10.2006, some equipments were purchased.
 Cash sales amounted to 10% of sales.
 Credit sales amounted to NRs.4,50,000.
 Credit purchases were 80% of total purchases.
 The firm sells goods at cost plus 25%.
 Discount allowed NRs.5,500 during the year.
 Discount earned NRs.4,800 during the year.
 Outstanding expenses NRs.3,000 as on 31.3.2007.
 Capital of ‗A‘ as on 31.3.2006 was NRs.15,000 more than the capital
of ‗B‘, equipments and furniture to be depreciated at 10% p.a. and
building @ 2% p.a.

You are required to prepare: 20


a) Trading and Profit and Loss account for the year ended 31.3.2007 and
Suggested Answers of Accounting
CAP II Examination – June 2009

b) The Balance Sheet as on that date.


Answer:
a) Trading and Profit and Loss A/c
for the year ended 31.3.2007
Debit Credit
Particulars NRs. Particulars NRs.
To Opening stock (W.N.3) 145,000 By Sales- Cash (W.N.1) 50,000
To Purchases-Cash 65,000 Credit 4,50,000 500,000
Credit (W.N.2) 2,60,000 325,000 By Closing stock 70,000
To Gross profit c/d 100,000
Total 570,000 Total 570,000
To Loss on sale of equip By Gross profit b/d 100,000
(20,000-15,000) 5,000
To Depreciation By Discount received 4,800
Building 3,000
Furniture 2,500
Equipment (W.N.4) 24,600 30,100
To Expenses paid 40,000
Add : Outstanding exp. 3,000 43,000
To Discount allowed 5,500
To Net profit transferred to:
A‘s capital A/c 10,600
B‘s capital A/c 10,600 21,200
Total 104,800 Total 104,800

b) Balance Sheet as on 31-3-2007


Equity and Liabilities NRs. Assets NRs.
A‘s capital (W.N.7) 280,250 Building 1,50,000
Less: Drawings (30,000) Less: Depreciation 3,000 147,000
Add: Net profit 10,600 260,850 Equipments 2,72,000
B‘s capital (W.N.7) 265,250 Less: Depreciation 24,600 247,400
Add: Net profit 10,600 275,850 Furniture 25,000
Sundry creditors (W.N.5) 70,200 Less: Depreciation 2,500 22,500
Bank loan 35,000 Debtors 100,000
Outstanding expenses 3,000 Stock 70,000
Cash balance (W.N.8) 58,000
Total 644,900 Total 644,900

Working Notes:
W.N. 1: Calculation of total sales and cost of goods sold
Cash sales = 10% of total sales
Credit sales = 90% of total sales = NRs.450,000
450,000
Total sales = 100  5,00,000
90
Cash sales = 10% of 500,000 = NRs.50,000

The Institute of Chartered Accountants of Nepal


2 of 109
Suggested Answers of Accounting
CAP II Examination – June 2009

W.N. 2: Calculation of total purchases and credit purchases


Cash purchases = NRs.65,000
Credit purchases = 80% of total purchases
Cash purchases = 20% of total purchases
65,000
Total purchases = 100  Rs .3,25,000
20
Credit purchases = 325,000 – 65,000 = NRs.260,000

W.N. 3: Calculation of opening stock

Debit Stock Account Credit


Particulars NRs. Particulars NRs.
To Balance b/d (Bal. Fig.) 145,000 By Cost of goods sold
500,000
 100 400,000
125
To Total purchases (W.N.2) 325,000 By Balance c/d 70,000
Total 470,000 Total 470,000

W.N. 4: Purchase of equipment & depreciation on equipments

Debit Equipment Account Credit


Particulars NRs. Particulars NRs.
To Balance b/d 240,000 By Cash -equipment sold 15,000
To Cash-purchase (Bal. Fig.) 52,000 By P/L Account ( Loss on sale) 5,000
By Balance c/d 272,000
Total 292,000 Total 292,000

Depreciation on equipment:
Particulars NRs.
@ 10% p.a. on NRs.2,20,000 (i.e. NRs.240,000 – NRs.20,000) = 22,000
@ 10% p.a. on NRs.52,000 for 6 months (i.e. during the year) = 2,600
Total 24,600

W.N. 5: Calculation of closing balance of creditors

Debit Creditors Account Credit


Particulars NRs. Particulars NRs.
To Cash 250,000 By Balance b/d 65,000
To Discount received 4,800 By Credit purchases (W.N.2) 260,000
To Balance c/d (Bal. Fig.) 70,200
Total 325,000 Total 325,000

The Institute of Chartered Accountants of Nepal


3 of 109
Suggested Answers of Accounting
CAP II Examination – June 2009

W.N. 6: Calculation of opening balance of debtors

Debit Debtors Account Credit


Particulars NRs. Particulars NRs.
To Balance b/d (Bal. Fig.) 35,500 By Cash 380,000
To Sales (Credit) 450,000 By Discount allowed 5,500
By Balance c/d 100,000
Total 485,500 Total 485,500

W.N. 7: Calculation of capital accounts of A & B as on 31.3.2006

Balance Sheet as on 31.3.2006


Equity & Liabilities NRs. Assets NRs.
Combined Capital Accounts of
A & B (Bal. Fig.) 545,500 Building 150,000
Creditors 65,000 Equipments 240,000
Bank Loan 45,000 Furniture 25,000
Debtors (W.N.6) 35,500
Stock (W.N.3) 145,000
Cash balance 60,000
Total 655,500 Total 655,500

Particulars NRs.
Combined Capitals of A & B 545,500
Less: Difference in capitals of A and B 15,000
Total 530,500

530,500
A‘s Capital as on 31.3.2006 =  265,250  15,000  NRs.280,250
2
530,500
B‘s Capital as on 31.3.2006 =  NRs.265,250
2

W.N. 8:

Debit Cash Account Credit


Particulars NRs. Particulars NRs.
To Balance b/d 60,000 By Creditors 250,000
To Debtors 380,000 By Purchases 65,000
To Equipment (sales) 15,000 By Expenses 40,000
To Cash sales (W.N.1) 50,000 By A‘s drawings 30,000
By Bank loan paid (45,000-35,000) 10,000
By Equipment purchased (W.N.4) 52,000
By Balance c/d (Bal. Fig.) 58,000
Total 505,000 Total 505,000

The Institute of Chartered Accountants of Nepal


4 of 109
Suggested Answers of Accounting
CAP II Examination – June 2009

Question No. 2
Carlin Co. has head office at New York (USA) and branch at Kathmandu, Nepal.
Nepal branch is an integral foreign operation of Carlin & Co. Nepal branch furnishes
you with its trial balance as on 31st March, 2006 and the additional information given
there after:
Debit Credit
Particulars NRs NRs
Stock on 1st April,2005 300,000.00 -
Purchases and Sales 800,000.00 1,200,000.00
Sundry Debtors and Creditors 400,000.00 300,000.00
Bills of exchange 120,000.00 240,000.00
Wags and Salaries 560,000.00 -
Rent, rates and taxes 360,000.00 -
Sundry charges 160,000.00 -
Computers 240,000.00 -
Bank balance 420,000.00 -
New York office a/c - 1,620,000.00
Total 3,360,000.00 3,360,000.00

Additional Information
a) Computers were acquired from a remittance of US $6,000 received from New
York head office and paid to the supplier. Depreciate computers at 60% for the
year.
b) Unsold stock of Nepal branch was worth NRs. 420,000 on 31st March ,2006.
© The rates of exchange may be taken as follows:
# on 1.4.2005 @ Rs 40 per US$
# on 31.3.2006 @ Rs42 per US $
# Average exchange rate for the year @ Rs 41 per US $
#conversion in $ shall be made up to two decimal accuracy

You are asked to prepare in US dollars the revenue statement for the year ended 31st
March, 2006 and the Balance Sheet as on that date of Nepal branch as would appear in
the books of New York head office of Carlin & Co 16.

You are informed that Nepal branch account showed a debit balance of US $ 39,609.18
on 31.3.2006 in New York books and there were no items pending reconciliation.

Answer
CARLIN & CO. Ltd.
Trading and Profit & Loss Account
Of Kathmandu, Nepal Branch
For the year ended 31st March, 2006
Debit Credit
Particulars US$ Particulars US$
To Opening Stock 7,500.00 By Sales 29,268.29

The Institute of Chartered Accountants of Nepal


5 of 109
Suggested Answers of Accounting
CAP II Examination – June 2009

To Purchase 19,512.20 By Closing Stock 10,000.00


To Wages and salaries 13,658.54 By Gross Loss c/d 1,402.45
Total 40,670.74 Total 40,670.74
To Gross Loss b/d 1,402.45 By Net Loss 17,685.38
To Rent ,rates and taxes 8,780.49
To Sundry charges 3,902.44
To Dep on computers 3,600.00
(US$ 6,000*0.6)
Total 17,685.38 Total 17,685.38

CARLIN & CO. Ltd.


Kathmandu, Nepal Branch
Balance Sheet as on 31st March 2006
Equity & Liabilities US$ Assets US$
New York Office A/c 39,609.18 Computers 6,000.00
Less:
Less: Net Loss 17,685.38 21,923.80 Depreciation 3,600.00 2,400.00
Sundry creditors 7,142.86 Closing Stock 10,000.00
Bills payable 5,714.29 Sundry debtors 9,523.81
Bank balance 10,000.00
Bills receivable 2,857.14
Total 34,780.95 Total 34,780.95

Working Note
CARLIN & CO. Ltd.
Kathmandu, Nepal Branch
Trial Balance in (US$) as on 31st March, 2006
Conversion Debit Credit
Particulars NRs per US $ US$ US$
Stock on 1st April,2005 40 7,500.00 -
Purchases and Sales 41 19,512.20 29,268.29
Sundry Debtors and Creditors 42 9,523.81 7,142.86
Bills of exchange 42 2,857.14 5,714.29
Wags and Salaries 41 13,658.54 -
Rent, Rates and taxes 41 8,780.49 -
Sundry charges 41 3,902.44 -
Computers 6,000.00 -
Bank balance 42 10,000.00 -
New York office a/c - - 39,609.18
Total 81,734.62 81,734.62

Question No. 3
Elite Limited recently made a public issue in respect of which the following
information is available:
a) No. of partly convertible debentures issued 200,000; face value and issue price
NRs.100 per debenture.

The Institute of Chartered Accountants of Nepal


6 of 109
Suggested Answers of Accounting
CAP II Examination – June 2009

b) Convertible portion per debenture 60%, date of conversion on expiry of 6 months


from the date of closing of issue.
c) Date of closure of subscription lists 1.5.2007, date of allotment 1.6.2007, rate of
interest on debenture 15% payable from the date of allotment, value of equity
share for the purpose of conversion NRs. 60 (Face Value NRs. 10).
d) Underwriting Commission 2%.
e) No. of debentures applied for 150,000.
f) Interest payable on debentures half-yearly on 30th September and 31st March.
Write relevant journal entries for all transactions arising out of the above during the
year ended 31st March, 2008 (including cash and bank entries). 15

Answer
Journal Entries
In the books of Elite Ltd
NRs. NRs.
Date Particulars Dr. Cr.

Journal No. 1
1.5.2007 Bank A/c Dr. 15,000,000
To Debenture Application A/c 15,000,000
(Application money received on 150,000 debentures @ NRs.
100 each)

Journal No. 2
1.6.2007 Debenture Application A/c Dr. 15,000,000
Underwriters A/c Dr. 5,000,000
To 15% Debentures A/c 20,000,000
(Allotment of 150,000 debentures to applicants and 50,000
debentures to underwriters)

Journal No. 3
1.6.2007 Underwriting Commission Dr. 400,000
To Underwriters A/c 400,000
(Commission payable to underwriters @ 2% on NRs.
20,000,000)

Journal No. 4
1.6.2007 Bank A/c Dr. 4,600,000
To Underwriters A/c 4,600,000
(Amount received from underwriters in settlement of account)

Journal No. 5
30.9.2007 Debenture Interest A/c Dr. 1,000,000
To Bank A/c 1,000,000

The Institute of Chartered Accountants of Nepal


7 of 109
Suggested Answers of Accounting
CAP II Examination – June 2009

(Interest paid on debentures for 4 months @ 15% on NRs.


20,000,000)

Journal No. 6
30.10.2007 15% Debentures A/c Dr. 12,000,000
To Equity Share Capital A/c 2,000,000
To Securities Premium A/c 10,000,0000
(Conversion of 60% of debentures into shares of NRs. 60 each
with a face value of NRs. 10)

Journal No. 7
31.3.2008 Debenture Interest A/c Dr. 750,000
To Bank A/c 750,000
(Interest paid on debentures for the half year)

Working Note:
Calculation of Debenture Interest for the half year ended 31st March, 2008
On NRs. 8,000,000 for 6 months @ 15% = NRs. 600,000
On NRs. 12,000,000 for 1 months @ 15% = NRs. 150,000
NRs. 750,000

Question No. 4
Following is the Balance Sheet of Mr. Pawan as at 31st March, 2005. He has filed a
petition in the court for being declared as insolvent:
Equity & Liabilities NRs. Assets NRs.
Capital 18,000 Goodwill 5,000
Bank loan (secured by first Machinery 20,000
charge on building) 80,000 Building 115,000
Loan from Finance Co., 30,000 Investment in shares 5,000
(secured by second charge Furniture 7,000
on building) Stock 9,000
Sundry creditors 59,000 Debtors:
Sales tax payable 8,000 Good 14,000
Loan from wife 5,000 Doubtful 8,000
Bad 2,000 24,000
Cash and bank 15,000
200,000 200,000
Mr. Pawan estimated that except the following, all tangible assets are realizable:
i) Machinery costing NRs. 5,000 included in the Balance Sheet has no
value.
ii) Debtors (unrealizable) NRs. 7,600.
iii) Non-moving stock NRs. 3,000.
iv) Useless furniture NRs. 4,000.
v) Investment has no value.
Further Information:
i) Building expected to realize NRs. 120,000.

The Institute of Chartered Accountants of Nepal


8 of 109
Suggested Answers of Accounting
CAP II Examination – June 2009

ii) Loan was given by his wife from her personal sources.
iii) A bill discounted for NRs. 10,000 is likely to be dishonored.
iv) One creditor forgoes his claim for NRs. 4,000.
v) Mr. Pawan started his business on 1.4.2001. His household expenses
up-to 31.3.2005 is NRs. 48,000. His private Life Insurance Policy
matured for NRs. 30,000 on 31.3.2005.

He made profit of NRs. 40,000 up-to 31.3.2003.


He incurred loss of NRs. 50,000 from 1.4.2003 to 31.3.2005.
Also, he suffered speculation loss of NRs. 10,000 in the year ended
31.3.2005.

Based on the above information, prepare Statement of Affairs of Mr.


Pawan as on 31.3.2005 and Deficiency Account. 16

Answer
Statement of Affairs of Mr. Pawan as on 31st March, 2005
Dr. Cr.
Gross Liabilities Expected to Assets Book Estimated to
Liabilities (as stated and estimated by Rank (as stated and estimated Value produce
debtor) by debtor)
NRs. NRs. NRs. NRs. NRs.
74,000 Unsecured creditors as Properties as per List
per List A 70,000 E
80,000 Creditors fully secured as Cash and Bank 15,000 15,000
per List B Stock 9,000 6,000
80,000
Less: Estimated value of Machinery 20,000 15,000
securities Furniture 7,000 3,000
1,20,000
Surplus 40,000 Goodwill 5,000 
Less: Amount carried to Investment in shares 5,000 
List C 30,000 Book debts as per
Balance thereof to contra  List F Good 14,000 14,000
10,000
30,000 Creditors partly secured Doubtful 8,000 2,400
as per List C Bad 2,000 
Less: Surplus carried 30,000 85,000 55,400
from
List B 30,000 Surplus from
Balance   securities in
the hands of fully
8,000 Preferential creditors secured creditors
payable in full as per List D (as per contra) 10,000
65,400
Sales tax payable 8,000 Less: Preferential
Less: Deducted as per creditors as per 8,000

The Institute of Chartered Accountants of Nepal


9 of 109
Suggested Answers of Accounting
CAP II Examination – June 2009

contra 8,000  contra 57,400


Add: Deficiency as
explained in List H
12,600
1,92,000 70,000 70,000

Deficiency Account (List H)


NRs. NRs. NRs.
Excess of assets over Net loss arising from business
liabilities as on 01.04.2001 56,000 since 01.04.2003 50,000
(Balancing figure) Bad debts as per list F 7,600
Net profit arising from Drawings for household expenses 48,000
business up-to 31.03.2003 40,000 Loss on realization of :
Profit on realization of: Furniture 4,000
Building 5,000 Machinery 5,000
Life policy matured 30,000 Stock 3,000
Creditors forgone claims 4,000 Goodwill 5,000
Deficiency as per Investment 5,000 22,000
Statement of Affairs 12,600 Loss on dishonor of bill 10,000
_______ Speculation losses 10,000
1,47,600 1,47,600

Working Note:
1. Detail of unsecured creditors:
Sundry creditors 59,000
Loan from wife 5,000
Liability for discounted bill dishonored 10,000
Total unsecured creditors 74,000

Question No. 5 'a'


From following balances on 31/12/07, Prepare necessary Revenue Account for marine
insurance business of an Insurance company. 8

i) Premium Direct Business Re-insurance


Received 9,200,000.00 1,440,000.00
Receivable 1st January 372,000.00 54,000.00
Receivable 31st December 524,000.00 51,000.00

Paid 920,000.00
Payable 1st January
- 56,250.00
st
Payable31 December - 93,000.00

ii) Claims
Paid 4,700,000.00 600,000.00
Payable 1st January 250,000.00 57,000.00

The Institute of Chartered Accountants of Nepal


10 of 109
Suggested Answers of Accounting
CAP II Examination – June 2009

Payable31st December 312,000.00 66,000.00

Received 255,000.00
Receivable 1st Janaury 24,000.00
Receivable 31st December 34,500.00

iii) Commission
On insurance accepted 440,000.00 38,000.00
On re insurance ceded 39,000.00

iv) Other expenses & Income


Salaries NRs 480,000, Rent Rates & Taxes NRs. 58,000 , Postage & printing &
stationery NRs. 86,000, Income tax paid NRs 660,000. , Interest, Dividend & rent
received (net) NRs. 280,000. Tax deduced at source NRs. 56,000.. Legal expenses
(including NRs. 36,000. insurance settlement of claim) NRs 80,000.
v) Balance of Fund on 1st Jan 08 was NRs. 7,690,000 including additional reserve of
NRs. 690,000 and on 31st December NRs.9,832,250.00. Additional reserve is to be
maintained @ 5 % of net premium in the year.

Answer:
Insurance Co. Ltd.
Marine revenue Account
For year ended 31/12/07
Particulars NRs NRs Particulars NRs NRs
Claim less reinsurance paid Fund bal at the
during the year (W.N. 2) 5,081,000 beginning 7,000,000
Add: Outstanding at the
end of year. (W.N. 4) 343,500 Additional reserve 690,000 7,690,000
Premium less Re Ins.
5,424,500 Received (W.N. 1) 10,789,000
Less Outstanding in Less Re insurance
beginning of year (W.N. 5) ( 283,000) 5,141,500 Premium Paid (W.N. 3) (956,750) 9,832,250
Commission Interest/ Div/ rent 336,000

On direct business 440,000 Less TDS (56,000) 280,000


On Re insurance accepted 38,000 478,000
Expenses on management (W.N. 6) 668,000 Reinsurance Comm 39,000
Income Tax 660,000
Profit & Loss Account 569,887
Bal of fund at end of year 9,832,250
Additional reserve 491,613 10,323,863
Total 17,841,250 Total 17,841,250

Working Notes:

W.N. 1: Premium Less Reinsurance Received W.N. 2 Claim Less Reinsurance Paid:

The Institute of Chartered Accountants of Nepal


11 of 109
Suggested Answers of Accounting
CAP II Examination – June 2009

Particulars NRs. Particulars NRs.


Premium Received:  Direct 4,700,000.00
 Direct 9,200,000.00  Re insurance 600,000.00
 Re insurance 1,440,000.00 Legal expenses 36,000.00
Add: Premium Receivable: Less Re insurance rece (255,000.00)
 Direct 524,000.00 Total 5,081,000.00
 Re insurance 51,000.00
Less due on 1st Jan. W.N. 3 Reinsurance Premium Paid:
 Direct (372,000.00) Particulars NRs.
 Re insurance (54,000.00) Paid 920,000
Total 10,789,000.00 Additional due 93,000.00
Less pay. on 1st Jan (56,250.00)
W.N. 4 Claims outstanding 31/12/07: Total 956,750.00
Particulars NRs.
Direct 312,000.00 W.N. 5 Claims outstanding 01/01/07
Re insurance 66,000.00 Particulars NRs.
Due from reinsurance (34,500.00) Direct 250,000.00
Total 343,500.00 Re insurance 57,000.00
Due from reinsurance (24,000.00)
Total 283,000.00

W.N. 6 Management Expenses


Particulars NRs.
Salaries 480,000
Rent Rates Taxes 58,000
Postage and Stationery 86,000
Legal Expenses 44,000
Total 668,000

Question No. 5 'b'


What are the differences between Cash Flow Statement and Fund Flow Statement?

Answer
The differences between cash flow statement and fund flow statement are as per
below:
i) Cash flow statement deals with the change in cash position between two points of
time. Fund flow statement deals with the changes in working capital position.
ii) Cash flow statement contains opening as well as closing balances of cash and cash
equivalents. The fund flow statement does not contain any such opening and
closing balance.
iii) Cash flow statement records only inflow and outflow of cash. Fund flow
statement records sources and application of funds.

The Institute of Chartered Accountants of Nepal


12 of 109
Suggested Answers of Accounting
CAP II Examination – June 2009

iv) Fund flow statement can be prepared from the cash flow statement under indirect
method. However, a cash flow statement cannot be prepared from fund flow
statement.
v) A statement of changes in working capital is usually prepared alongwith fund flow
statement. No such statement is prepared along with the cash flow statement.

Question No. 6 (5×4=20)


a) What are the information that are extracted from the well designed accounting
system in Agricultural Farm?
Answer
A well designed accounting system can be used for extracting the following
information:
i) Crop-wise performance and overall performance of the agricultural enterprise.
ii) Comprehensive information regarding yield, revenue, input and cost of the
enterprise.
iii) Financial state of affairs i.e. assets and liabilities of the farm at a particular
point of time.
iv) Profit/Loss of the farm during a year.
v) Data base for other decisions, namely (a) acquire assets or hire services for
ploughing, irrigation, weeding, threshing etc.; (b) replacement of draught
animal, machinery and farming technique; (c) selection of crop-mix; (d)
choosing farm size; (e) farm diversification, for example adding crop and non -
farming activities like processing of agricultural products; (f) divestment
decisions whether to discontinue agricultural operations.
vi) Supporting data to the lenders including banks and co-operative societies to
assess farm‘s financial requirements as well as debt servicing ability.
vii) Reliable data for farm management survey.
viii) Reliable data for assessment of agricultural income tax.

b) Briefly describe the disclosure requirements of related party transactions as per


Nepal Accounting Standard 17.
Answer
NAS 17 requires disclosure of related party transactions and outstanding balances
in the separate financial statements of a parent, venture or investor company or
entity. The following information shall be disclosed in the financial statements as
per this Accounting Standard.
i) Relationship between parents and subsidiaries shall be disclosed irrespective
of whether there have been transactions between those related parties.
ii) An entity shall disclose key management personnel compensation.
iii) If there have been transactions between related parties, an entity shall disclose
the nature of the related party relationship as well as information about the
transactions and outstanding balances necessary for an understanding of the
potential effect of the relationship on the financial statements.

The Institute of Chartered Accountants of Nepal


13 of 109
Suggested Answers of Accounting
CAP II Examination – June 2009

iv) Items of the similar nature may be disclosed in aggregate except when separate
disclosure is necessary for an understanding of the effects of related party
transactions on the financial statements of the entity.

c) What are the Final Accounts to be prepared under the ‗Double Accounts System‘
in case of electricity companies?
Answer
Double Accounts System is the name given to the system of processing the final
accounts of certain public utility undertakings e.g. electricity companies and
railway companies. The double accounts system is not a special method of
keeping accounts rather a special method of presenting accounts which are kept
under the normal double entry system.
The final accounts prepared under the double accounts system normally consists of :
i) Revenue Accounts
ii) Net Revenue Accounts
iii) Capital Account (Receipt & Expenditure on Capital Account)
iv) General Balance Sheet

d) What are the advantages of maintaining subsidiary books by a


trading/manufacturing organization?
Answer
Advantages of maintaining subsidiary books by a trading/manufacturing
organization are:
i) Division of work: In place of one journal, there are many subsidiary books.
The accounting work can be divided amongst a number of people.
ii) Specialization and efficiency: As a person is handling only one type of work,
he acquires full knowledge and becomes efficient in handling the work.
Accounting work is done efficiently.
iii) Saving of time: Various accounting processes can be undertaken
simultaneously because of the use of a number of books. This results in
quicker completion of work.
iv) Availability of information: Since a separate register is kept for each class of
transactions, the information relating to each class of transaction is available at
one place.
v) Additional information for VAT, sales tax, excise etc., can also be compiled
from the appropriate columns in the purchases and sales registers.
vi) Facility in checking: When the trial balance does not agree, the location of
errors is facilitated by the existence of separate books. Similarly audit of the
various books of prime entry can be conducted simultaneously by a team of
audit staff.

The Institute of Chartered Accountants of Nepal


14 of 109
Suggested Answers of Accounting
CAP II Examination – June 2009

e) Write short note on Sale and Lease Back Transactions as per Accounting Standard
Answer
Sale and leaseback transactions: As per AS 15 on ‗Leases‘, a sale and leaseback
transaction involves the sale of an asset by the vendor and the leasing of the asset
back to the vendor. The lease payments and the sale price are usually
interdependent, as they are negotiated as a package. The accounting treatment of
a sale and lease back transaction depends upon the type of lease involved.
If a sale and leaseback transaction results in a finance lease, any excess or
deficiency of sale proceeds over the carrying amount should be deferred and
amortized over the lease term in proportion to the depreciation of the leased asset.
If sale and leaseback transaction results in a operating lease, and it is clear that the
transaction is established at fair value, any profit or loss should be recognized
immediately. If the sale price is below fair value any profit or loss should be
recognized immediately except that, if the loss is compensated by future lease
payments at below market price, it should be deferred and amortized in proportion
to the lease payments over the period for which the asset is expected to be used.
If the sale price is above fair value, the excess over fair value should be deferred
and amortized over the period for which the asset is expected to be used.

The Institute of Chartered Accountants of Nepal


15 of 109
The Institute of Chartered Accountants of Nepal
Suggested Answers of Audit and Assurance

CAP II Examination – June 2009

All questions are compulsory.

Question No. 1
Comment and give your views as auditor with reasons on each of the
following case: (4×4=16)

a) The management tells you that the work in process is not valued
since it is difficult to ascertain the same in view of the multiple
processes involved and in any case the value of opening and closing
work in process would be more or less the same.

Answer
According to NAS-4 on "Inventories", it includes among others
those assets which are in the process of production for sale in the
ordinary course of business apart from finished goods and those
materials or supplies to be consumed in the production process or in
the rendering of services. It is, thus, necessary for a company to
ensure that each and every component of inventory is valued
properly. The argument advanced by the company that it is difficult
to ascertain the same in view of the multiple processes involved is
not acceptable. In general, the audit procedures regarding work-in-
process are similar to those used for raw materials and finished
goods. The auditor has to carefully assess the stage of completion of
the work-in-process for assessing the appropriateness of its
valuation.

The argument that the opening and closing work-in-process would


be more or less the same is also not justified because the omission of
those would lead to distortion of true and fair view. Further, costs
incurred for raw materials and the overheads would normally
different and would give rise to different value of opening and
closing stock. In view of the above, the auditor shall have to qualify
the Assurance Report in case work-in-process is not valued and
shown in the financial statements.

b) Mr. Bhandari & Associates, Chartered Accountants was appointed as


an auditor of the company. Subsequently, Mr. Sharma & Associates,
Chartered Accountants was offered appointment as an auditor of the
company replacing Mr. Bhandari & Associates, Chartered
Accountants. Mr. Sharma & Associates, Chartered Accountants for
whom this is an initial audit engagement accepts the appointment
and starts the audit without communicating with anyone.
Suggested Answers of Accounting
CAP II Examination – June 2009

Answer
Mr. Sharma & Associates has started the audit in violation with NSA
300 "Planning the Audit". According to NSA 300 "Planning an
Audit", the auditor should communicate with previous auditor where
there has been a change of auditors, in relevance to ethical
requirements.
And again since it is initial audit, the auditor may need to expand the
planning activities because the auditor does not ordinarily have
previous experience with the entity that is considered when planning
recurring engagements. The auditor should consider the following in
developing overall audit strategy and plan.
 Unless prohibited by the law, arrangement to be made with
previous auditor to review the previous auditor's working papers;
 Any major issues discussed with management in connection with
the initial selection as auditors.
 Other procedures required by the firm's system of quality control
for initial audit engagements

c) The entire liability for interest on deferred payment terms is treated


as part of capital cost of the asset as such liability was incurred at the
time of acquisition of the asset itself.

Answer
According to NAS - 8 on "Borrowing Costs", under allowed
alternative treatment, borrowing costs that are directly attributable to
the acquisition, construction or production of a qualifying asset shall
be capitalised as part of the costs of that asset. The interest on
deferred payment terms are borrowing costs directly attributable to
the acquisition of the asset and can be treated as part of the capital
cost. However, under NAS - 8, the capitalisation of borrowing cost
shall cease when substantially all the activities necessary to prepare
the qualifying asset for its intended use are complete. Hence, the
interest on deferred payment related to the period after the assets are
ready to put to use shall not be capitalised.

d) Fire Ltd. purchased equipment for its power plant from Urja Ltd.
during the year 2006-07 at a cost of NRs.100 lacs. Out of this they
paid only 90% and balance 10% was to be paid after one year on
satisfactory performance of the equipment. During the Financial year
2007-08, Urja Ltd. waived off the balance 10% amount which was
credited to Profit and Loss account by Fire Ltd. as discount received.

Answer
According to NAS-6 on Property, Plant and Equipment, the cost of
an asset may undergo changes subsequent to its acquisition on

The Institute of Chartered Accountants of Nepal


17 of 109
Suggested Answers of Accounting
CAP II Examination – June 2009

account of exchange fluctuation, price adjustment, change in duty


or similar factors. Such change in price /cost needs to be adjusted
with the cost of the asset.
In the give case, Fire Ltd., initially accounted for 100% amount i.e.,
NRs.100 lacs as cost of property plant and equipment although they
paid only NRs.90 lacs and kept NRs.10 lacs as payable to the credit
of Urja Ltd. Now since the supplier has waived off the balance
amount of NRs.10 lacs, this should be treated as change in price and
needs to be adjusted with the cost of asset.
Therefore, the treatment given by Fire Ltd., in crediting NRs.10.
Lacs as discount to Profit & Loss Account is completely wrong and
needs to be corrected. It will have effect on depreciation also and
needs adjustment.
The auditor should report the matter if suitable changes are not made
in the accounts

Question No. 2
What are the special steps involved in conducting the audit of an
Educational Institution? 14

Answer
The special steps involved in the audit of an educational institution are
the following:
a) Examine the Trust Deed or Regulations in the case of school or
college and note all the provisions affecting accounts. In the case of
a university, refer to the Act of Legislature and the Regulations
framed there under.
b) Read through the minutes of the meetings of the Managing
Committee or Governing Body, noting resolutions affecting accounts
to see that these have been duly complied with, specially the
decisions as regards the operation of bank accounts and sanctioning
of expenditure.
c) Check names entered in the Students‘ Fee Register for each month or
term, with the respective class registers, showing names of students
on rolls and test amount of fees charged; and verify that there
operates a system of internal check which ensures that demands
against the students are properly raised.
d) Check fees received by comparing counterfoils of receipts granted
with entries in the cash book and tracing the collections in the Fee
Register to confirm that the revenue from this source has been duly
accounted for.
e) Total up the various columns of the Fees Register for each month or
term to ascertain that fees paid in advance have been carried forward
and the arrears that are irrecoverable have been written off under the
sanction of an appropriate authority.

The Institute of Chartered Accountants of Nepal


18 of 109
Suggested Answers of Accounting
CAP II Examination – June 2009

f) Check admission fees with admission slips signed by the head of the
institution and confirm that the amount had been credited to a
Capital Fund, unless the Managing Committee has taken a decision
to the contrary.
g) See that free studentship and concessions have been granted by a
person authorized to do so, having regard to the prescribed Rules.
h) Confirm that fines for late payment or absence, etc., have either been
collected or remitted under proper authority.
i) Confirm that hostel dues were recovered before students‘ accounts
were closed and their deposits of caution money refunded.
j) Verify rental income from landed property with the rent rolls, etc.
k) Vouch income from endowments and legacies, as well as interest
and dividends from investment; also inspect the securities in respect
of investments held.
l) Verify any Government or local authority grant with the relevant
papers of grant. If any expense has been disallowed for purposes of
grant, ascertain the reasons and compliance thereof.
m) Report any old heavy arrears on account of fees, dormitory rents, etc,
to the Managing Committee.
n) Confirm that caution money and other deposits paid by students on
admission have been shown as liability in the balance sheet and not
transferred to revenue.
o) See that the investments representing endowment funds for prizes
are kept separate and any income in excess of the prizes has been
accumulated and invested along with the corpus.
p) Verify that the Provident Fund money of the staff has been invested
in appropriate securities.
q) Vouch donations, if any, with the list published with the annual
report. If some donations were meant for any specific purpose, see
that the money was utilised for the purpose.
r) Vouch all capital expenditure in the usual way and verify the same
with the sanction for the Committee as contained in the minute book.
s) Vouch in the usual manner all establishment expenses and enquire
into any unduly heavy expenditure under any head.
t) See that increase in the salaries of the staff have been sanctioned and
minuted by the Committee.
u) Ascertain that the system ordering inspection on receipt and issue of
provisions, foodstuffs, clothing and other equipment is efficient and
all bills are duly authorized and passed before payment.
v) Verify the inventories of furniture, stationery, clothing, provision
and all equipment, etc. These should be checked by reference to
Stock Register and values applied to various items should be test
checked.
w) Confirm that the refund of taxes deducted from the income from
investment (interest on securities, etc.) has been claimed and

The Institute of Chartered Accountants of Nepal


19 of 109
Suggested Answers of Accounting
CAP II Examination – June 2009

recovered since the institutions are generally exempted from the


payment of income-tax.
x) Verify the annual statements of accounts and while doing so see that
separate statements of account have been prepared as regards Poor
Boys Fund, Games Fund, Hostel and Provident Fund of Staff, etc.

Question No. 3 'a'


Briefly discuss the basis of disclosure of accounting policies according
to NAS-1 on ―Presentation of Financial Statements". 4

Answer
According to NAS-1, an entity shall disclose in the summary of
significant policies:
 measurement basis used for preparing the financial statements;
 the other accounting policy used that are relevant to an
understanding of the financial statements.
In addition to the specific accounting policies used in the financial
statements, it is important for the users to be aware of the measurement
basis used (historical cost, current cost, realizable value, fair value or
present value) because they form the basis on which the financial
statements are prepared. When more than one measurement basis is used
in the financial statements, for an example, when certain non-current
assets are revalued, it is sufficient to provide an indication of the
categories of assets and liabilities to which each measurement basis is
applied.

In deciding whether a particular policy shall be disclosed, management


considers whether disclosure would assist users in understanding the
way in which transactions and events are reflected in the reported
financial performance and financial position. The accounting policies
that an entity might consider presenting include revenue recognition,
recognition and depreciation/ amortization of tangible and intangible
assets, capitalization of borrowing costs and other expenditure, financial
instruments and investments, leases, research and development costs,
inventories, taxes, provisions, employee benefit costs, foreign currency
transaction, definition of cash and cash equivalent and government
grants.

Question No. 3 'b'


What does qualified opinion mean? What are the situations in which
the auditor should qualify the assurance report? 4

Answer
A qualified opinion is issued when the auditor concludes that he
cannot issue an unqualified opinion but that the effect of any
disagreement, uncertainty or limitations on scope is not so material as

The Institute of Chartered Accountants of Nepal


20 of 109
Suggested Answers of Accounting
CAP II Examination – June 2009

to require an adverse or a disclaimer of an opinion. It is given in


respect of a part of the information reflected in the financial statements
and that the auditor is not in agreement with that part. Moreover, the
part with reference to which he is not in agreement does not materially
affect the view portrayed by the financial statements. Thus, the need
for a qualified opinion arises where the auditor is satisfied with the
truth and fairness of the financial statements; yet because of certain
transactions he is not fully satisfied so as to issue a clean or
unqualified report.
The auditor should qualify their Assurance Reports in case -

 accounting policies have not been properly disclosed, or


 accounts have not been prepared on accrual basis
 the fundamental accounting assumption of going concern has not
been followed and this fact has not been disclosed in the financial
statements, or
 proper disclosures regarding changes in the accounting polices have
not been made.

Question No. 3 'c'


What are the general considerations in framing a system of internal
check? 7

Answer
The following are the general considerations in framing a system of internal check:
 No single person should have an independent control over any
important aspect of the business. All dealings and acts of every
employee should, in the ordinary course, come under the review of
another.
 The duties of members of the staff should be changed from time to
time without any previous notice so that the same officer or
subordinate does not, without a break, perform the same function for
a considerable length of time.
 Every member of the staff should be encouraged to go on leave at
least once in a year. Experience has shown that frauds successfully
concealed by employees are often unearthed when they are on leave.
 Persons having physical custody of assets must not be permitted to
have access to the books of account.
 There should exist an accounting control in respect of each important
class of assets; in addition, these should be periodically inspected so
as to establish their physical condition.
 To prevent loss or misappropriation of cash, mechanical devices,
such as the automatic cash register, should be employed.
 A majority of business concerns now-a-days work according to some
kind of budgetary control. It enables them to review from time to

The Institute of Chartered Accountants of Nepal


21 of 109
Suggested Answers of Accounting
CAP II Examination – June 2009

time the progress of their trading activities. Such business houses


should have a separate staff for the collection of statistical figures
which later on should be checked with corresponding figures from
the financial books. If wide discrepancies are observed, these should
be reconciled.
 For stock-taking, at the close of the year, trading activities should, if
possible, be suspended. The task of stock-taking, and evaluation
should be done by staff belonging to several sections of the
organisation. It may prove dangerous to depend exclusively on the
stock section staff for these tasks, since they may be tempted to
under or over-state the stock.
 The financial and administrative powers should be distributed very
judiciously among different officers and the manner in which these
are actually exercised should be reviewed periodically.
 Procedures should be laid down for periodical verification and
testing of different sections of accounting records to ensure that they
are accurate.
Accounting procedures should be reviewed periodically, for,
even well-designed and carefully installed procedures, in course
of time, cease to be effective.
Question No. 4
How will you vouch and/or verify the following? (5×3=15)

a) Personal expenses of directors met by the company

Answer
The personal expenses of the director are the facilities of his
accommodation, transportation, communication, entertainment etc.
Such expenses that are of personal nature should be verified as per
below:
 Check the articles of association, service contract, minutes of
general meeting, etc., to ensure that the payment complies with
the provision stated therein.
 Enquire to ensure that personal expenses are not camouflaged in
any other revenue items as contemplated under the Companies
Act.
 Ascertain compliance with disclosure and limitation prescribed
by the Companies Act.
 Check documentary evidences in support of the expenses
reimbursed.
 Ensure the compliance with the provisions set forth by the
Income Tax Act 2058 to incorporate such payments in his/her
income from employment and withhold the tax.

b) Preliminary expenses

The Institute of Chartered Accountants of Nepal


22 of 109
Suggested Answers of Accounting
CAP II Examination – June 2009

Answer
It is the expenditure incurred incidental to the creation, formation
and floating of a company. It consists of stamp duties, registration
fees, legal costs, consultant's fees, expenses of printing of
memorandum and articles, etc. The following should be checked:
 Check Board‘s minute book containing the resolution approving
the expenses claimed by promoters as having been spent in
formation of the company.
 Examine supporting papers and vouchers, contracts, agreements,
etc. to support the promoters‘ claims. Also check bills and
receipts issued by the printer of the memorandum and articles of
association, share certificates, etc.
 Check receipt for the registration fee paid for registration of the
company.
 Verify rates of stamp required to be affixed on the memorandum
and articles of association.
 Ascertain Boards‘ minute book for the decision to write off the
preliminary expenses over a period. The quantum thereof which
has not yet been written off for these expenses should be carried
forward in the balance sheet under the head miscellaneous
expenditure (to the extent not written off or adjusted) over a
period of years.
 Check that no expenses other than those constituting preliminary
expenses are booked under this head, e.g. underwriting
commission and brokerage paid.
 Ensure that the whole expenses are charged to the income
statement because of not being eligible for recognition as
Intangible Assets under NAS 27.

c) Patents

Answer
Patents are normally treated as intangible assets subject to
recognition criteria under NAS 27. The same shall be verified as
per below:
 Obtain the schedule containing particulars of the patents owned
by the client as on the balance sheet date. The particulars should
contain the dates of registration of the patents with the related
authorities and the dates in respect of the last renewal.
 See that the total of the values of the patent rights shown in each
list agree with the values shown in the respective ledger
accounts.
 Examine the cost of patent rights. In case of outright purchase of
patent rights, the purchase consideration, legal fees and
registration charges should be included in cost. When they are

The Institute of Chartered Accountants of Nepal


23 of 109
Suggested Answers of Accounting
CAP II Examination – June 2009

developed within the organization, all costs incurred on their


development including legal and registration expenses for
registration of the patent should constitute the cost. Capitalized
value should be amortized over the life of the patent.
 See that the renewal fees in respect of the patent rights have been
paid and the same has been treated as a revenue charge.
 Examine the valuation of the patent rights. It should be seen that
the patent rights have been valued at cost less depreciation
attributable to the expired legal life of the patent rights.
However, if it is found that the patent rights have already lost
substantial part of their commercial value, it would be proper to
value it at their residual commercial value, when it is less than
the book value for their unexpired legal life. In case the product
covered by the patent rights does not have any sale value then
patents should be shown at nil valuation notwithstanding any
residual life. Reference to compliance with the provisions of
NAS 27 Intangible Assets should be verified.

d) Advances given to suppliers

Answer
The amount paid in advance for the supplies of goods or services is
termed as advances to supplier. The same shall be verified as per
below:
 Obtain a schedule of advances given to the suppliers and enquire
the management regarding the purpose of such advances.
 Conduct an aging analysis of the advances to confirm the
balances are regular and moving.
 Where the balances are outstanding since long, ensure that the
management has initiated reasonable steps for its recovery or
settlement.
 Where the balances are doubtful of recovery ensure adequate
provision for loss is set apart in the accounts.
 Where the balances are confirmed of being bad, ensure that the
same are written off from the accounts.
 Cases were noted where advances were shown as deposits in
balance sheet. Hence, ensure booking of advances in proper
accounting heads.
 Carry out external confirmation as per the need to further assure
that the balances stated in the accounts are correct.

e) Forfeiture of Shares

Answer
The shares are forfeited where the applicant fails to pay the sum
called on the allotment. The same shall be verified as per below:

The Institute of Chartered Accountants of Nepal


24 of 109
Suggested Answers of Accounting
CAP II Examination – June 2009

 ascertain that the Articles authorise the Board of Directors to


forfeit shares and that the power has been exercised by the Board
in the best interest of the company;
 verify the amount of call or instalment of calls which was
outstanding in respect of each of the share forfeited.,
 ascertain that the procedure in the Articles has been followed
viz., the notice given to the defaulting shareholders warning them
that in the event of non-payment, by a specified date, of the
amount of call already made on the shares standing in their
names, together with interest, if any, the shares shall be forfeited,
see that the proper resolutions of Directors, first as regards
issuance of notice and afterwards in respect of forfeiture of
shares; and
 verify the entries recorded in the books of account consequent
upon forfeiture of shares to confirm that the premium, if any,
received on the issue of shares has not been transferred to the
Forfeited Shares Account.

Question No. 5 'a'


Mr. A, a practicing chartered accountant receives commission from
Mr. X, another practicing chartered accountant NRs. 50,000 being
50% of the audit fee for the referral of statutory audit of Xerox
Company Limited, a listed company. State your views. 5

Answer
According to Section 10 of Code of Ethics, 2060 issued by Institute of
Chartered Accountants of Nepal, the payment or receipt of
commission by a professional accountant in public practice could
impair objectivity and independence. A professional accountant in
public practice should not therefore pay a commission to obtain a
client nor should a commission be accepted for referral of a client to a
third party. The payment and receipt of commission are permitted only
for such engagements for which independence is not required and the
professional account in practice should nonetheless disclose the facts
to the client. In the above case since the above assignment requires
independence, Mr. A and Mr. X both are not complying with the
ethical requirements under Code of Ethics issued by ICAN. The
disciplinary action can be taken by the Council on fulfilling the
petition requirements and after recommendation of Disciplinary
Committee.

Question No. 5 'b'


Mr. Nimesh, a partner of TMS & Associates, a practicing Chartered
Accountant in its business letter head prints its previous association as
member Secretary to one of the reputed political party of the country.
State your views. 5

The Institute of Chartered Accountants of Nepal


25 of 109
Suggested Answers of Accounting
CAP II Examination – June 2009

Answer
According to preface directives issued by ICAN related to Section 7 and
14 of the Code of Ethics of ICAN, 2060, a professional accountant in
practice, should not print or write its positions not related to the
accounting profession in the name plate, identity card, visiting card,
business card and business stationeries used for the purpose of its
profession. Hence, Mr. Nimesh has violated ethical requirements under
Code of Ethics issued by ICAN and hence attracts the disciplinary
actions by Council under the recommendation of Disciplinary
Committee.

Question No. 6 'a'


Write short notes on the following: (3×5=15)

a) Analytical review

Answer
AAS 5 on Audit Evidence defines analytical review as those tests of
details which consists of studying significant ratios and trends and
investigating unusual fluctuation and items. Thus, analytical reviews
are substantive audit procedure with the help of which auditor can
perform tests of details in more efficient and effective manner.
Therefore, analytical reviews are nothing best analytical review
procedures which have been considered at length in AAS-14 on
―Analytical Procedures‖. According to AAS-14, analytical procedures
include the consideration of comparisons of the entity‘s financial
information with, for example, comparable information for prior
periods or anticipated results of the entity, such as budgets or
forecasts. Consideration of relationships among elements of financial
information that would be expected to conform to a predictable pattern
based on the entity‘s experience, such as gross margin percentages,
between financial information and relevant non-financial information,
such as payroll costs to number of employees also constitute analytical
review procedures.
Analytical review procedures are used for the following purposes:
 to assist the auditor in planning the nature, timing and extent of other
audit procedures;
 as substantive procedures when their use can be more effective or
efficient than tests of details in reducing detection risk for specific
financial statement assertions; and
 as an overall review of the financial statements in the final review
stage of the audit.

The Institute of Chartered Accountants of Nepal


26 of 109
Suggested Answers of Accounting
CAP II Examination – June 2009

The extent of reliance that the auditor places on the results of analytical
review procedures depends on materiality of the items involved,
assessment of inherent and control risks, etc.

b) Audit trail in a computerized accounting environment

Answer
Audit trail or audit log is a chronological sequence of audit records,
each of which contains evidence directly pertaining to and resulting
from the execution of a business process or system function. In simple
accounting words, it refers to documentation of detailed transactions
supporting summary ledger entries. This documentation may be on
paper or electronic records.
An audit trail refers to a situation where it is possible to relate ‗one-to-
one‘ basis, the original input along with the final output. The work of
an auditor would be hardly affected if ―Audit Trail‖ is maintained i.e.
if it were still possible to relate, on a ‗one-to-one‘ basis, the original
input with the final output. A simplified representation of the
documentation in a manually created audit trail. The particular credit
notes may be located by the auditor at any time he may wish to
examine them, even months after the balance sheet date. He also has
the means, should he so wish, of directly verifying the accuracy of the
totals and sub-totals that feature in the control listing, by reference to
individual credit notes. He can, of course, check all detailed
calculations, casts and postings in the accounting records, at any time.
In first and early second-generation computer systems, such a
complete and trail was generally available, no doubt, to management‘s
own healthy skepticism of what the new machine could be relied upon
to achieve – an attitude obviously shared by the auditor. In such a
system
 The output itself is as complete and as detailed as in any manual
system.
 The trail, from beginning to end, is complete, so that all documents
may be identified by located for purposes of vouching, totaling and
cross-referencing.

Any form of audit checking is possible, including depth testing in either


direction. In case audit trail is missing, the auditor employs Computer
Assisted Techniques (CAATs) to ensure the validity of accounting data.

c) Process of judgment formation by Auditor

Answer
After the audit, the opinion that the auditor expresses is the result of
exercise of judgment on facts, evidence and circumstances which he

The Institute of Chartered Accountants of Nepal


27 of 109
Suggested Answers of Accounting
CAP II Examination – June 2009

comes across in the course of audit. The judgment is formed on the


following basis:-
 Identification of the assertions to be examined.
 Evaluation of the assertion as to relative importance.
 Collection of the information or evidence about the assertions to
enable him to give an informed opinion.
 Evaluation of evidence as valid or invalid, pertinent or not
pertinent, sufficient or insufficient.
 Formulation of judgment as to the fairness of the assertions
under consideration.

The Institute of Chartered Accountants of Nepal


28 of 109
Suggested Answers of Accounting
CAP II Examination – June 2009

Question No. 7
Distinguish between following: (3×5=15)

a) Permanent Audit File and Current Audit File


Answer
In a recurring audit, the working papers have to be retained for future
reference as well. In such cases they are classified as permanent audit
file and current audit file.

Permanent audit file is updated regularly with information of


continuing importance to succeeding audit whereas working paper
which contain information relating primarily to the audit of single
period is kept in current audit file.
A permanent audit file normally includes:
 Information concerning the legal and organizational structure of the
entity. In the case of a company, this includes the Memorandum and
Articles of Association. In the case of a statutory corporation, this
includes the Act and Regulations under which the corporation
functions.
 Extracts or copies of important legal documents, agreements and
minutes relevant to the audit.
 A record of the study and the evaluation of the internal controls
related to the accounting system.
 Copies of audited financial statements for previous years.
 Analysis of significant ratios and trends.

The current file normally includes:


 Correspondence relating to acceptance of annual reappointment.
 Extracts of important matters in the minutes of Board Meeting and
General Meetings as relevant to audit.
 Evidence of the planning process of the audit and audit programme.
 Analysis of transactions and balances.
 A record of nature, timing and extent of auditing procedures
performed, and the results of such procedures.

b) Control Risk and Detection Risk


Answer
Control risk is the risk that misstatement that could occur in an account
balance or class of transactions and that could be material, individually
or when aggregated with mis-statements in other balances or classes,
will not be prevented or detected on a timely basis by the system of
internal control There will always be some control risk because of the
intrinsic limitation of any system of internal control To assess control
risk, the auditor should consider the adequacy of control design, as

The Institute of Chartered Accountants of Nepal


29 of 109
Suggested Answers of Accounting
CAP II Examination – June 2009

well as test adherence to control procedures. In the absence of such an


assessment, the auditor should assume that control risk is high.
Detection risk is the risk that an auditor's procedures will not detect a
misstatement that exists in an account balance or class of transactions
that could be material, individually or when aggregated with
misstatements in other balances or classes. The level of detection risk
relates directly to the auditor's procedures. Some detection risk would
always be present even if an auditor were to examine 100 percent of
the account balance or class of transaction because, for example, the
auditor may select an inappropriate audit procedure, misapply an
appropriate audit procedure or misinterpret the audit results.

c) Audit Technique and Audit Procedure


Answer
For collection and accumulation of audit evidence, certain methods
and means are available and these are known as audit techniques. The
two terms, techniques and producers are often used interchangeably; in
fact, however, a distinction does exist. Procedure may comprise a
number of techniques and represents the broad frame of the manner of
handling the audit work; techniques stand for the methods employed
for carrying out the procedure. Vouching is a substantive audit
procedure which involves audit techniques like casting, cross-casting,
checking of posting etc. verification of assets and liabilities is a
substantive audit procedures which involves application of audit
techniques like physical examination, confirmation from third parties
etc.

The Institute of Chartered Accountants of Nepal


30 of 109
The Institute of Chartered Accountants of Nepal
Suggested Answers of Corporate & Other Laws

CAP II Examination – June 2009

All questions are compulsory


Part: 'A'
Question No. 1
Answer the following questions:

a) Walter Limited has its subsidiary company Kluber Limited, which is formed to carry
out some of the objectives of Walter Limited. Walter Limited suspends one of its
several businesses, by passing a resolution at the company's extraordinary general
meeting, with effect from 1 January 2009. The business so suspended continues to be
suspended until March 2009. On 1 April 2009, a group of shareholders of Walter
Limited filed a petition in the court for winding up of the company on the ground of
suspension of business by the company.

Referring to the provisions of the companies Act, 1956 decide:


9
i) Whether the shareholders' contention shall be tenable?
ii) What would be your answer in case Walter Limited suspends all of its businesses?
iii) Can shareholders of Kluber Limited file a petition in the court for winding up on the
ground that the holding company Walter Limited has suspended its entire businesses,
though Kluber Limited has not suspended its business?

Answer
Under Section 433(c), if a company suspends whole of its business for a whole year or
more, a petition for its winding up may be made. However, suspension of one of its
several businesses is not covered under this clause.
i) Thus Shareholder's contention shall not be tenable.
ii) The suspension of business has passed only 3 months, so can not be a ground for
winding up under the said section.
Where of the several business units of a company the business of only one unit was
closed with a proposal to dispose it off and to use the proceeds in the exploitation of other
objects, the court did not agree that it was a ground for winding up. The court said that
even if the business in all the units of the company was suspended, it would still be open
to the court to examine whether it would be possible for the company to resume its
business.(Paramjit Lal Bhadwar vs. Prem Spg and Wvg Mills) it was held in 1986)

iii) Suspension of entire business by the holding company can not be a ground for
winding up of subsidiaries. Each company is a separate legal entity.

Where a company's subsidiaries were functioning, winding up was not ordered through the
company itself had ceased to function.(Eastern Telegraph Co ltd Re(1947)
Suggested Answers of Accounting
CAP II Examination – June 2009

b) “The Memorandum of Association (MOA) is the fundamental law or a charter


defining the objects and limiting the power of Company‖. Explain. 6

Answer
The MOA of a company is its charter which contains the fundamental conditions upon
which alone the company can be incorporated. It sets forth the objects of the company‘s
formation and the utmost possible scope of its operation beyond which its action cannot
go. Thus MOA is said to be a charter of the company. It defines as well as confines the
powers of the company. If anything is done beyond these powers that shall be ultra vires
of the company and so void.
The MOA serves two-fold purposes. It enables shareholders/creditors and all those who
deal with the company to know what their powers are and what are the range of activities.
Thus, the intending shareholders can find out the field in, or the purpose for which his
money is going to be used by the company and what risk he is taking by making the
investment. Also, any one dealing with the company, say a supplier of goods will know
whether the transaction he intends to make with the company is within the objects of the
company and not ultra vires its objects.

Question No. 2 'a'


How 'Noting' and 'Protesting' a bill or note are discharged under the Negotiable
Instruments Act, 1881? Explain. 5

Answer
Noting - Noting is a convenient mode of authenticating the fact that a bill or note has
been dishonored. When a note or a bill has been dishonored by non-acceptance or non-
payment, the holder causes such dishonor to be noted by a Notary Public. Noting is a
minute recorded by a notary public on the dishonored instruments. When an instrument,
say a bill of exchange, is to be noted for dishonor, it is taken to Notary Public who
presents it once again for acceptance or payment, as the case i.e., a minute is prepared
containing the date of dishonor, reason for such dishonor, etc., which is attached to the
instrument; and the facts are noted on the instruments.

Protest- When an instrument is dishonored, the holder may cause the fact not only to be
noted, but also to be certified by a Notary Public that the bill has been dishonored. Such a
certificate is referred to as a protest. The process of protest is called protesting.

If the creditor or an acceptor of a bill is shaken by insolvency or otherwise before the date
of maturity of the bill, the holder may cause such a fact also to be noted and certified.
Such a certificate is called a protest for better security. The contents of a protest are given
in Section 101 of the Negotiable Instruments Act, 1881.

Neither noting nor protesting is compulsory in the case of inland bills. But under Section
104 of the above Act every foreign bill of exchange must be protested for dishonor when
such a protest is required by the law of the country where the bill was drawn. The
advantage of both noting and protesting is that this constitutes prima facie good evidence

The Institute of Chartered Accountants of Nepal


32 of 109
Suggested Answers of Accounting
CAP II Examination – June 2009

in the Court of the fact that the instrument has been dishonored. It is necessary to note
that under Section 119 of the above Act, the Court is bound to recognize a protest. But it
may or may not recognize noting. To make good this lacuna, Section 104 A has been
introduced. It clarifies the position that any bill or document which has been noted can be
protested any time thereafter for taking legal action against the parties. Thus, where a
document has been noted within the time required by law, legal proceeding cannot be
vitiated on account of protest not having been made.
Notice of Protest: When a promissory note or bill of exchange is required by law to be
protested, noticed of such protest in lieu of notice of dishonour must be given in the same
manner as notice of dishonour (Section 102).
Question No. 2 'b'
A company cannot become a party to a contract before it has come into existence.
Explain with reference to the provision of Companies Act 1956. 5

Answer
A company being an artificial person can enter into a contract. The promoters
may enter into contracts with third parties on behalf of the proposed company
before obtaining the certificate of incorporation or after obtaining the certificate
of incorporation but before obtaining the certificate to commence business. Thus,
in the case of a public company following are the three situations when contracts
may be entered into:
i) contracts before incorporation;
ii) contracts after incorporation but before obtaining the certificate to commence
business; and
iii) contracts after obtaining the certificate to commence business.

However, in the case of a private company, it is not required to obtain the


certificate to commence business; There are only two situations, i.e. contracts
before incorporation and ii) contracts after incorporation.

The contracts which are entered into by promoters for the intended company before
registration of the company are known as pre-incorporation or preliminary contracts.

Before the enforcement of Specific Relief Act 1963 a pre-incorporation contract never
binds a company since a legal person cannot contract before his or its existence and a
company before incorporation has not legal existence. Contracts prior to incorporation
were void and also could not be ratified. After the enforcement of Specific Relief Act ,
1963, Section 15 (h) and 19 (e) of the Act provides that when promoter of a company
have, before its incorporation entered into a contract for the purposes of the company and
such contract is warranted by the terms of its, incorporation, the contract may be
specifically enforced by or against the company. It is however, necessary that the
company in such case must have accepted the contract after its incorporation and
communicated such acceptance to the other party to contract. Contracts like preparation
and printing of the MOA/AOA, renting premises, hiring secretarial staff are envisaged
under the Act.

The Institute of Chartered Accountants of Nepal


33 of 109
Suggested Answers of Accounting
CAP II Examination – June 2009

Under English Companies Act specifically provides that such contracts take effect as
contract entered into personally by the persons who make them. Even the knowledge or
ignorance of those persons of the persons that the company has not yet been incorporated
is immaterial.

Part: 'B'

Question No. 3 'a'


The directors of Hydrangea Ltd. recently incorporated company selling garden furniture,
wishes to increase the authorized share capital of the company to Rs. 2 crore and to
change the name of the company to Motormowers Ltd. Referring to the provisions of the
Companies Act, 2063 decide: 8
i) The directors seek your advice on the statutory requirements for calling the meeting
and resolution required to give effect to the above proposal.
ii) The first annual general meeting of Hydrangea Ltd. is due to be held in September
2009 and the directors seek your advice on how the votes of members and proxies
should be taken and counted at the meeting.

Answer
Calling of the meeting
It must call an extraordinary general meeting (EGM) in order that both resolution
might be passed in that meeting.

Notice of the meeting


Notice of the EGM must be sent to every members of the company who is entitled
to attend and vote at the meeting. It should also be sent to the directors and
auditors. The notice should:
a) Give adequate information concerning the date, time and place of the meeting.
b) Describe the proposed special business as such,
c) Give sufficient details of the proposed business at the meeting to enable recipient of
the notice to understand what it is proposed to be done at the meeting.

Length of notice
The length of notice requires is 15 days because of the special resolution.

Increasing the authorized share capital


Under S.83 (a) authority to increase the authorized share capital of a company needs to be
given by a special resolution of the company in general meeting.

Changing the company‘s name


Authority to change the name of a company requires a special of the company in general
which means a 75% majority of votes cast is needed to pass it.

Voting Right:

The Institute of Chartered Accountants of Nepal


34 of 109
Suggested Answers of Accounting
CAP II Examination – June 2009

The rights of members to vote and the number of votes to which they are entitled will be
determined by the company‘s articles.

Proxies
Every member entitled to attend and vote at a meeting may instead appoint a proxy to
attend and vote for him. The proxy need not be a member. An appointment of a proxy is
revocable by the member. It will be deemed to have been revoked where the member
himself attends the meeting in person and vote.

Question No. 3 'b'


Total Paid up capital of Siddhababa Cargo Pvt. Ltd. is Rs 100,000. Mr. Jhamaklal a
Chartered Accountant holds 900 shares at the rate of Rs.100 per share. The general
meeting of the company appoints Mr. Jhamaklal as auditor of the company for fiscal year
2065/066. Answer the following question with relevant provision of Companies Act,
2063. 7
i) Is the appointment of Mr. Jhamaklal valid as per provision of the companies Act?
ii) If his appointment is invalid by the above reason or any other reason, what steps must
be taken by him.
iii) If his appointment is declared invalid and he has audited the books of accounts, what
would be the validity of the audit report signed by him?

Answer
i) Under Section 112(e) a shareholder holding one percent or more of the paid up capital
of the company or his close relative shall be disqualified for appointment as auditor,
and shall not continue to hold office, if appointed as auditor.
Under Section 112(2) the auditor shall, prior to his appointment, give information in
writing to the company that he is not disqualified pursuant to sub-section(1)
As he holds less than one percent of the paid up capital of the company and Mr.
Jhamaklal's his appointment is valid.

ii) Under Section 112(3) where any auditor becomes disqualified to audit the accounts of
a company or there arises a situation where he becomes disqualified for appointment
or can no longer continue to act as an auditor of the company, he shall immediately
stop performing audit which is required to be performed or is being performed by him
and give information thereof to the company in writing.

iii) Under Section 112(4) the audit performed by an auditor who has been
appointed in contravention of this section shall be invalid

Question No. 3 'c'


Distinguish between compulsory liquidation and voluntary liquidation of companies. 5

The Institute of Chartered Accountants of Nepal


35 of 109
Suggested Answers of Accounting
CAP II Examination – June 2009

Answer
The differences between compulsory and voluntary liquidation are associated with
timing, the role of the official receiver, stay of legal proceedings and the dismissal of
employees. The major differences in legal consequences between a compulsory and a
voluntary liquidation are as follows:
i) Timing: Voluntary winding up commences on the day when the resolution to wind up
is passed. It is not retrospective. Compulsory winding up, once agreed to by the court,
commences on the day the petition was presented.

ii) Receiver: The official receiver plays no role in a voluntary winding up. The members
or creditors select and appoint the liquidator and he is not an officer of the court.

iii) Legal proceedings: There is no automatic stay of legal proceedings against the
company nor are previous dispositions or seizure of its assets void in a voluntary
winding up. However the liquidator has a general right to apply to the court to make
any order which the court can make in a compulsory liquidation. He would do so, for
instance, to prevent any creditor obtaining an unfair advantage over the other
creditors.

iv) Management and staff: In any liquidation the liquidator replaces the directors in the
management of the company. However, the employees are not automatically
dismissed by commencement of voluntary liquidation. Insolvent liquidation may
amount to repudiation of their employment contracts.

Question No. 4 'a'


A group of Nepalese persons want to establish a Development Bank in Nepal for which
they need your advice. Advise how and under which Act they should proceed for the
formation of bank. 8

Answer
Establishment of a Development Bank in Nepal
i) The persons desirous of establishing a Development bank to conduct banking and
financial transactions under the Bank and Financial Institutions Act, 2006 may do so
by registering it as a public limited company according to section 3 and 4 of the Act
and under the provision of Companies Act, 2006. The Company Registrar's Office is
empowered to register a company as a Development Bank under section 3, 4 and 5 of
the Companies Act.

ii) Pursuant to section 4 of the Bank & Financial Institution Act, for the purpose of
establishing a Development bank as above, the concerned persons must, before
applying for the registration of the Bank according to the Companies Act, apply to the
Rastra Bank for prior approval along with the following documents and the fee
prescribed by the Rastra Bank:
 Memorandum of Association of the proposed bank.
 Articles of Association of the proposed bank.
 Feasibility study report of the proposed bank.

The Institute of Chartered Accountants of Nepal


36 of 109
Suggested Answers of Accounting
CAP II Examination – June 2009

 Personal particulars of the promoters in the form prescribed by the


Rastra Bank.
 A certified copy of the agreement signed, if any, among the
promoters before the establishment of the bank in relation to the
establishment of the bank.
 Evidence of clearance of tax by the promoters up to the fiscal year
immediately preceding the date of application under this section.
 Other particular and documents prescribed by the Rastra Bank in
relation to the establishment of a bank.

iii) On receipt of an application for prior approval, the Rastra Bank shall, if it finds it
appropriate to grant its approval following the examination of the documents that
have been submitted, grant its approval to establish the concerned bank within 120
days from the date of receipt of the application by prescribing or without prescribing
any conditions. In case such approval is to be denied, the applicants shall be notified
accordingly by explicitly mentioning the reasons there for.

Question No. 4 'b'


What do you understand by securities business? Explain the principles that are to be
followed by the securities business person under Securities Act, 2063. 7

Answer
―Securities‖ means any shares, stocks, bonds, debentures, debenture stock or collective
investment scheme certificates issued by a body corporate or treasury bonds, saving
bonds or bonds issued by the Government of Nepal or by a body corporate against the
guarantee of the Government of Nepal. According to Section 2(j) of Securities Act 2063.
―Securities business‖ means transactions in securities to be carried on by any company or
body licensed to carry on the securities business under the Securities Act. Securities
business person means a company or body licensed under Securities Act 2063 to carry on
securities business.

Under Section 76 of the Securities Act 2063 securities business person should, in
carrying on the business, observe the following business principles:
a) To maintain the operation of securities business fair and of high standards.
b) To carry on the securities business with proper skills, care and hard working.
c) To keep on the higher standard of stock exchanges,
d) To obtain information from customers as to their objective to make
investment and provide services accordingly.
e) To provide such information and advice as may be required for customers to
make decision on investment in securities.
f) To avoid conflicts of own interests with the interests of customers and, in the
event of the existence of such situation, to disclose that matter to customers
and carry on the securities business having regard to the interest of customers.
g) To make such provisions as may be necessary to fulfill commitments made in
relation to the securities business.

The Institute of Chartered Accountants of Nepal


37 of 109
Suggested Answers of Accounting
CAP II Examination – June 2009

h) To properly maintain records relating to the securities business.


i) To provide for necessary training to employees in order to prepare skilled
human resources for the operation of the securities business, and
j) To observe such other principle as prescribed in relation to the operation of
the securities business.

Question No. 5 'a'


How the dispute has to be settled between a foreign investor and a Nepalese investor of
an Industrial Company, if any dispute arises, under the Foreign Investment and
Technology Transfer Act, 1992? 5

Answer
Settlement of such a dispute has to be made according to following provisions of section
7 of the Foreign Investment and Technology Transfer Act, 1992:
i) If any dispute arises between a foreign investor and national investor, the concerned
parties shall be required to settle the dispute by mutual consultations in the presence
of the Department of Industries.
ii) If the dispute could not be settled in the manner as above, it shall be settled by
arbitration in accordance with the prevailing Arbitration rules of the United Nations
Commission on International Trade Law (UNCITRAL).
iii) The arbitration shall be held in Kathmandu. The laws of Nepal shall be applicable in
the arbitration.
iv) Notwithstanding anything contained as above, the dispute arising in regard to the
foreign investment may be settled as mentioned in the foreign investment agreement.

Question No. 5 'b'


Under what grounds the Insurance Board may cancel the registration of the Insurer?
Explain in light of relevant provisions of Insurance Act, 1991. 5

Answer
On the following grounds the Insurance Board may by giving written notice to the Insurer
and after giving an opportunity for representation, cancel the registration of the Insurer
from the register from the date as notified pursuant to section 13 of the Insurance Act,
1991:
i) If the insurance business has not been commenced within six months from the date of
obtaining the insurance business license.
ii) If it is suspected that the liability of the Insurer is more than his/her property in Nepal.
iii) If the Insurer has failed to discharge his/her liability within three months of the
Court's final decision on the case arisen out of Insurance Policy issued in Nepal.
iv) If it is realized that Nepalese Insurer engaged in insurance business in a foreign
country did not obtain treatment as equal to a foreign insurer engaged in insurance
business in Nepal having its head office in that foreign country which conferred it
higher treatment than to the Nepalese one.
v) In case the Insurer did not establish its office in Nepal.
vi) If the Insurer did not comply with other requirements of this Act and rules made
under it.

The Institute of Chartered Accountants of Nepal


38 of 109
Suggested Answers of Accounting
CAP II Examination – June 2009

Question No. 6 'a'


Point out the composition and main functions of One-window Committee as provided
under the Industrial Enterprise Act, 1992. 5
Answer
Constitution of the One-Window Committee has been made under section 17 of
the Industrial Enterprises Act, 1992, as follows:
i) The Government of Nepal shall, for the purpose of making available the facilities and
concessions to be enjoyed by any industry under this Act in time from a single place,
constitute a One-Window Committee consisting of the following:
 The Director-General, Department of Industries - Co-ordinator
 The Director-General. Department of Customs - Member
 The Director-General, Department of Excise - Member
 The Director-General, Inland Revenue Department - Member
 The Director-General, Sales Tax Department - Member
 The Director-General, Department of Commerce - Member
 The Chief Controller, Nepal Rastra Bank - Member
 Representative, FNCCI - Member
 One expert in the field of industry and commerce as designated by the
Government or one representative from the Federation -Member

The Government may, by notification published in the Nepal Gazette, make alterations or
changes in the members of the Committee.
The Committee may, if it deems necessary, invite any national or foreign expert or
consultant to participate in the meetings of the Committee as an observer.
The procedures relating to the meetings of the Committee shall be as determined by the
Committee.

ii) Functions, Duties and Powers of the Committee as provided by section 18 of the
Industrial Enterprise Act, 1992 are as follows:
 To make necessary decisions for making available the facilities and
concessions to be enjoyed by any industry under this Act,
 To perform such functions as may be delegated by the Industrial
Promotion Board under its functions, duties and powers,
 Other functions, duties and powers of the Committee shall be as
prescribed.

Any decision made by the Committee shall be required to be implemented by the


concerned body.

Question No. 6 'b'


What do you understand the term bonus? When is an employee of an industrial
establishment entitled to get bonus under Bonus Act, 2030. 5

The Institute of Chartered Accountants of Nepal


39 of 109
Suggested Answers of Accounting
CAP II Examination – June 2009

Answer
Distribution of profit or sharing of profit means distribution of certain share of profit
between the employer and employee. However the sharing of profit is not wage of an
employee or labor rather it is part of the profit earned by the employer. The principle
behind the distribution of profit is to reduce the economic gap between the employer and
employee i.e. to reduce the gap between haves and have not. It helps to maintain
industrial peace and motivates employees for the promotion of industrial productivity.
Indirectly it approaches towards the promotion of the concept of workers‘ participation in
management. It helps to promote capital labor relationship. Thus bonus distribution plays
a great importance in the field of capital labor relationship and for maintaining industrial
peace. It is said that bonus is neither reward nor a wage, but it is a return or gain made
out of the labor capital relationship.
Bonus Act 2030 aims for distribution of bonus to the workers and employees of an
industrial establishment. According to Section 5 of the Act 10% of the net profit of the
year is to be distributed as bonus to the employees of the establishment. Bonus
distribution amount is subject to the deduction of tax according to the income tax Act and
the sum to be allotted for the fund for the construction of the quarter facilities to the
employees in the establishment under Labor Act 2048.
According to Section 6 of the Bonus Act 2030 every one who has worked for half of the
total working days of the financial year is entitled to get bonus. However, casual workers
are not entitled to get bonus. Under Section 7 of the Act, Bonus is to be paid in
proportion to the salary or wages earned by the employee during that financial year.

Question No. 7 'a'


How is the audit of a fully government owned body corporate is conducted under the
Audit Act, 1991? Explain in brief. 5
Answer
Pursuant to section 6 of the Audit Act, 1991 the audit of the fully government-owned
body corporate shall be done by the Auditor General. In case the auditor General thinks
to appoint others in that case he may appoint any other registered Auditor as his assistant,
in which case a Nepalese auditor shall be given preference in such appointment. The
auditor so appointed shall have to work under the direction, supervision and control of
the Auditor-General. The function, duties, powers and liability of the auditor so
appointed and procedures he/she has to follow for auditing and the reporting of such
auditing shall be as prescribed by the Auditor General. The Auditor-General shall also fix
the remuneration to be obtained by such auditor based on the institutional factors and the
financial position of such body corporate.

Question No. 7 'b'


Enumerate the duties and responsibilities of the member performing accounting
profession under the provisions of Nepal Chartered Accountants Act, 1997. 5

Answer

The Institute of Chartered Accountants of Nepal


40 of 109
Suggested Answers of Accounting
CAP II Examination – June 2009

Every members performing auditing and accounting profession shall have to follow the
followings pursuant to Rule 64 of Nepal Chartered Accountants Rules, 2004:
i) Shall Behave with the stance of impartiality and self-determination;
ii) Shall follow the code of conduct brought by the council;
iii) Shall keep the records according to standards of Nepal auditing and Nepal
accounts;
iv) A member holding Certificate of Practice according to section 31 of the Act shall
explain subjects specified in existing laws with other matters essential; and
v) Shall follow and execute directions issued by the Council from time to time.

Question No. 8 'a'


Write short notes on the following: 5
i) Appointment of Liquidator

Answer
Under Section 127 of Companies Act 2063, the company (in Liquidation) shall
appoint a liquidator to conduct the liquidation process and shall fix his
remuneration. Such a liquidator shall be licensed as insolvency practitioner
under the law in force on insolvency. The company shall submit a copy of the
appointment of the liquidator to the registrar within seven days following such
an appointment. After the liquidator is appointed the directors and official of the
company shall be free from their office and the liquidator shall exercise all the
power that is exercised by such a director and official in relation to management
of the company. The service of the employees of the company shall, ipso facto,
be terminated once the liquidator takes charge for the operation and
management of the company.

ii) Insider Trading in Securities

Answer
Under Section 91 of Security Act 2063, if any person deals in securities or causes any other
person to deal in securities on the basis of any insider information or notice that are
unpublished or communicates any information or notice known to such a person in the
course of the discharge of his or her duties in manner likely to affect the price of
securities such a person shall be deemed to have been committed an insider trading in
securities.
For the purposes of this Act, the following persons shall be deemed to be those
who have access to the insider information or notice not published by any body
corporate:
 A director, employee or a person, who can obtain any information or a notice in the
capacity of a shareholder of that body corporate,
 A person who can obtain any information or a notice in the capacity of a professional
service provider to that body corporate,
 A person who can obtain any information or a notice having a direct or indirect
contact with the person or source as specified in both the cases above.

The Institute of Chartered Accountants of Nepal


41 of 109
Suggested Answers of Accounting
CAP II Examination – June 2009

Question No. 8 'b'


Distinguish between the following: 5

i) Foreign Investment Vs. Technology Transfer


Answer
Foreign Investment Technology Transfer
It means the following investment made by a It means any transfer of technology to be made
foreign investor in any industry u/s Sec.2(b): under an agreement between an industry and a
 Investment in share; foreign investor on the following matters u/s
 Reinvestment of the earning derived from Sec.2(c) :
the investment in share; and  Use of any technological right, specialization,
 Investment made in the form of loan or loan formula, process, patent or technical knowledge
facilities of foreign origin;
 Use of any trademark of foreign ownership; and
Acquiring any foreign technical, consultant,
management and marketing service.
No permission shall be granted for making Permission may be granted for transfer of
foreign investment in the industries set forth in technology in such industries. (Sec 3(4)
the annex.(Sec 3(4) (Eg. Cottage Industries,
Security printings etc.

ii) Negotiability Vs. Assignability


Answer
Negotiability Assignability
When a negotiable instrument is transferred to Transfer of rights or property is called assignment.
any person with a view to constituting that
person the holder, the instrument is deemed to
have been negotiated.
Right of transferee may rise higher than those The assignee has only those rights which the assignor
of transferor. possessed and assignee is subject to all defects.
Negotiation requires mere delivery of a bearer Assignment requires a written document signed by the
instrument and endorsement and delivery of an transferor.
order instrument to effectuate a transfer.
No notice is necessary in a transfer by Notice of assignment of debt must be given by the
negotiation. assignor to the debtor
Consideration is always presumed. The transferee must prove consideration for the transfer.

The Institute of Chartered Accountants of Nepal


42 of 109
The Institute of Chartered Accountants of Nepal
Suggested Answers of Cost Accounting

CAP II Examination – June 2009

All questions are compulsory. Working notes should form part of the answer.
Make assumptions wherever necessary.

Question No. 1 'a'


From the following information relating to a hotel, calculate the room rent to be
charged per day to give a profit of 25% on cost excluding interest. 10
i) Salary of staff NRs. 80,000 p.a.
ii) Wages of the room attendant: NRs. 2 per day
There is a room attendant for each room. He is paid wages only when the
room is occupied.
iii) Lighting, heating and power:
 The normal lighting expenses for a room for the whole month are NRs.
50, when occupied.
 Power is used only in winter and the charges are NRs. 20 for a room, for
the whole month when occupied.
iv) Repairs to building NRs. 10,000 p.a.
v) Linen etc.: NRs. 4,800 p.a.
vi) Sundries: NRs. 6,600 p.a.
vii) Interior decoration and furnishing: NRs. 10,000 p.a.
viii) Depreciation @ 5% is to be charged on buildings costing NRs. 400,000 and
10% on equipments.
ix) Interest to be charged @ 5% on investment in buildings and equipments
amounting to NRs. 500,000.
x) There are 100 rooms in the hotel, 80 percent of the rooms are generally
occupied in summer and 30 percent in winter. The period of summer and
winter may be considered to be of 6 months in each case. A month may be
assumed of 30 days.

Answer
The room rent should be charged at NRs.15 per day to give a profit of 25% on
cost excluding interest. The same is calculated as per below

Operating cost statement showing room rent per day


Particulars Per annum
(A) Total cost NRs.
i. Staff salaries 80,000
ii. Room attendant's wages (W.N. 2) 39,600
iii. Lighting, heating and power (W.N. 3) 36,600
iv. Repair to buildings 10,000
v. Linen etc. 4,800
vi. Sundries 6,600
Suggested Answers of Accounting
CAP II Examination – June 2009

vii. Interior decoration and furnishing 10,000


viii. Deprecation: -
Building @ 5% 20,000
Other equipments@10% 10,000 30,000
(i.e., 10% of 5,00,000-4,00,000)
ix. Interests on Investments 5% on NRs. 5,00,000 25,000
Total 242,600
(B) Profit@25% on cost excluding interest, i.e., 25% on NRs.2,17,600
54,400
(C). Total rent to be charged for all rooms 2,97,000
(D) Room days (W.N. 1) 19,800
(E) Room rent per day (C/D) 15

Working notes:

W.N. 1 : Room days:


Particulars Days
Summer: 100 rooms* 6 months* 30 days*80/100 14,400
Winter: 100 rooms*6 months*30 days* 30/100 5,400
Total 19,800

W.N.2: Room attendant's wages:


Particulars NRs.
Summer: NRs. 2*100 rooms*80%*30 days* 6 months 28,800
Winter: NRs. 2* 100 rooms*30%*30 days*6months 10,800
Total 39,600

W.N.3: Lighting, heating and power:


Particulars NRs.
Lighting:
Summer: NRs. 50*6 months*100 rooms*80% 24,000
Winter: NRs. 50*6months*100 rooms*30% 9,000
Power:
Winter: NRs. 20*6 months*100 rooms*30% 3,600
Total 36,600

Question No. 1 'b'


Brock Construction Ltd. commenced a contract on November 1, 2007. The total
contract was for NRs. 3,937,500. It was decided to estimate the total profit on
the contract and to take to the credit of Profit and Loss A/c that proportion of
estimated profit on cash basis, which work completed bore to the total contract.
Actual expenditure for the period November 1, 2007 to October 31, 2008 and
estimated expenditure for November 1, 2008 to March 31, 2009 are given
below:

The Institute of Chartered Accountants of Nepal


44 of 109
Suggested Answers of Accounting
CAP II Examination – June 2009

Nov1, 2007 to Oct 31, Nov 1, 2008 to Mar 31, 2009


2008 (Actual) NRs. (Estimated) NRs.
Material issued 675,000 1,237,500
Labor: Paid 450,000 562,500
Prepaid 25,000 -
Outstanding - 2,500
Plant purchased 375,000 -
Expenses: Paid 200,000 350,000
Outstanding 50,000 25,000
Plant returns to store 75,000 300,000
(historical cost) (on March 31, 2008) (on March 31, 2009)
Work certified 2,000,000 Full
Work uncertified 75,000
Cash received 1,750,000
Material at site 75,000 37,500

The plant is subject to annual depreciation @ 33% on written down value


method. The contract is likely to be completed on March 31, 2009.

Required:
Prepare the contract A/c. Determine the profit on the contract for the year
November 2007 to October 2008 on prudent basis, which has to be credited to
Profit and Loss A/c.
10
Answer
Brock Construction Ltd
Contract A/C
For the Period from November 1, 2007 to Oct 31, 2008
Debit Credit
Particulars NRs. Particulars NRs.
To Materials issued 6,75,000 By plant return 75,000
To Labor paid 450,000 Less: Dep (1/3) 10,417 64,583
Less: Prepaid 25,000 425,000 By WIP Certified 2,000,000
To Plant purchased 375,000 Uncertified 75,000 2,075,000
To Expenses paid 200,000 By plant at site 300,000
+ Outstanding 50,000 250,000 Less: Dep (1/3) 100,000 200,000
To Notional profit (NP) c/d 689,583 By materials at site 75,000
Total 2,414,583 Total 2,414,583
To P/L A/C (W.N. 1) 148,580 By NP b/d 689,583
To work –in progress 541,003
(Profit in reserve) 689,583 689,583

Working Note: 1

PL Account = Estimated Profit * (Cash Received/Work Certified) * (Work Certified/


Contract Price)

The Institute of Chartered Accountants of Nepal


45 of 109
Suggested Answers of Accounting
CAP II Examination – June 2009

= NRs. 334,30 00,000 / 3,937,500)


= NRs. 148,580

Where estimated profit is derived as per below,

The Institute of Chartered Accountants of Nepal


46 of 109
Suggested Answers of Accounting
CAP II Examination – June 2009

Brock Construction Ltd


Contract A/C
For the Period from 1 November, 2007 to March 31, 2009
Debit Credit
Particulars NRs Particulars NRs
To Materials issued 1,912,500 By plant returned to 64,583
(675,000 + 1,237,500) store on 31/03/08
To Labor paid and 1,015,000 By plant returned to 172,222
outstanding stores on 31/3/09:
(425,000 + 587,500 + Cost 300,000
2,500) Less: Dep.(1/3) 100,000
Dep.5 months 27,778
To Plant purchased 375,000 By Contractee a/c 3,937,500
To expenses 575,000 By Material at site 37,500
(250,000 + 325,000)
To estimated profit 334,305
Total 4,211,805 Total 4,211,805

Question No. 2 'a'


At the beginning of October 2007, Quality Brush Company had in stock 10,000
brushes value at NRs. 10 each. Further purchases were made during the month
as follows:
7th October 4,000 Brushes @ NRs.12.50
14th October 6,000 Brushes @ NRs.15.00
24th October 8,000 Brushes @ NRs.16.50
Issue to shop floor were as follows:
16th October 16,000 Brushes
28th October 10,000 Brushes
You are required: 8
i) to prepare a store ledger card for the month of October on the assumption
that materials were issued on the First-in- First-out principle; and
ii) to state the value of closing stock at the end of October if issue are priced by
the weighted average method.
Answer:
i. Quality Brush Company
Stores Ledger Account (FIFO Method)
Receipts Issues Balance
Date Qty Rate Value Qty Rate Value Qty Rate Value
Oct. Units NRs. NRs. Units NRs. NRs. Units NRs. NRs.
1 10,000 10.00 100,000
7 4,000 12.50 50,000 - - - 10,000 10.00 100,000
4,000 12.50 50,000
14 6,000 15.00 90,000 - - - 10,000 10.00 100,000
4,000 12.50 50,000
6,000 15.00 90,000
16 10,000 10.00 100,000 4,000 15.00 60,000

The Institute of Chartered Accountants of Nepal


47 of 109
Suggested Answers of Accounting
CAP II Examination – June 2009

4,000 12.50 50,000


2,000 15.00 30,000
24 8,000 16.50 132,000 - - - 4,000 15.00 60,000
8,000 16.50 132,000
28 4,000 15.00 60,000 2,000 16.50 33,000
6,000 16.50 99,000
31 2,000 16.50 33,000

ii) The value of closing stock at the end of October if issues are priced by the weighted
average method is NRs.30,000. The same is calculated as per below:

Rate on the 14th October = (10,000x10)+(4,000x12.50)+(6,000x15)


10,000+4000+6000
= NRs.12.
th
Issue on 16 = 16,000x NRs.12
= NRs.192,000

Balance on 16th = 4,000x NRs12


= NRs. 48,000

Rate on 24th Oct. = (4,000x12)+(8,000x16.50)


4,000+8,000
= NRs.15

Hence, the value of stock = 2,000x NRs.15


= NRs. 30,000.

Question No. 2 'b'


During first week of April 2009 the workman Mr. Kalyan manufactured 300
articles. He receives wage for a guaranteed 48 hours week at the rate of NRs. 4
per hour. The estimated time to produce one article is 10 minutes and under
incentive scheme the time allowed is increased by 20%. Calculate his gross
wages and rate per hour according to: 7
i) Piece work with a guaranteed weekly wage,
ii) Rowan premium bonus, and
iii) Halsey premium bonus: 50 % to workman.

Answer
The Gross wages due to Mr. Kalyan under various schemes is as per below:

(i) Piece-work with a guaranteed weekly wage NRs.


NRs
Wages for 60 hours, i.e. standard time @ NRs. 4 per hour
(Being more than the guaranteed wage of NRs. 192 for 48 hours)
240.00

The Institute of Chartered Accountants of Nepal


48 of 109
Suggested Answers of Accounting
CAP II Examination – June 2009

Rate per hour worked: NRs. 240÷48 5.00

(ii) Rowan Premium Bonus Plan


Wages for 48 hours @ NRs.4 per hour 192.00
Bonus = Wages for time worked X (Time saved ÷ Standard time)
= 192 x (12÷60) = 38.40
230.40
Rate per hour worked: NRs.230.40 ÷ 48 = 4.80

(iii) Halsey Premium Bonus Plan


Wages for 48 hours 192.00
Bonus: 50% of the wages for time saved 24.00
216.00
Rate per hour worked NRs. 216÷ 48 = 4.50

Working notes:
Estimated time for one article 10 minutes
Add: 20% increase under incentives scheme 2 minutes
Total for one articles 12 minutes
Total for 300 articles = (300*12 min) 3,600 minutes = 60
hours
Time taken = 48
hours
Time saved = 12
hours

Question No. 3 'a'


RST Ltd. has two production departments: Machining and Finishing. There are
three service departments: Human Resource (HR), Maintenance and Design.
The budgeted costs in these service departments are as follows:
HR Maintenance Design
NRs. NRs. NRs.
Variable 100,000 160,000 100,000
Fixed 400,000 300,000 600,000
500,000 460,000 700,000

The usage of these Service Departments‘ output during the year just completed
is as follows:
Provision of Service Output (in hours of service)
Providers of Service
Users of Service HR Maintenance Design
HR - - -
Maintenance 500 - -
Design 500 500 -
Machining 4,000 3,500 4,500
Finishing 5,000 4,000 1,500

The Institute of Chartered Accountants of Nepal


49 of 109
Suggested Answers of Accounting
CAP II Examination – June 2009

Total 10,000 8,000 6,000

Required: 10

i) Use the direct method to re-apportion RST Ltd.‘s service department cost to
its production departments.
ii) State the proper sequence to be used in re-apportioning the firm‘s service
department cost by step-down method.
iii) Use the step-down method to reapportion the firm‘s service department cost.

The Institute of Chartered Accountants of Nepal


50 of 109
Suggested Answers of Accounting
CAP II Examination – June 2009

Answer:
(i) Apportionment of Service Department Overheads amongst production departments
using Direct Method:
Production Dept. Service Dept
Particulars Machining Finishing HR Maint Design
NRs. NRs. NRs. NRs. NRs.
Budgeted Overheads 500,000 460,000 700,000
Apport of design at ratio(4,500 : 1,500) 525,000 175,000 (700,000)
Apport of Maint at ratio (3,500 : 4,000) 214,667 245,333 (460,000)
Apport of ratio HR (4,000 : 5,000) 222,222 277,778 (500,000)
Total 961,889 698,111 Nil Nil Nil

(ii)The proper sequence for apportionment of service department overheads is First HR,
Second Maintenance, and Third Design. The sequence has been laid down based on
service provided.

(iii) Statement of Apportionment of Service Department Overheads


(Using Step-Down Method)
Production Depts. Service Depts.
Particulars Machining Finishing HR Maint Design
NRs. NRs. NRs. NRs. NRs.
Budgeted Overhead 500,000 460,000 700,000
Apport of HR at ratio (4 : 5 :0.5 : 0.5) 200,000 250,000 (500,000) 25,000 25,000
Apport of Maint at ratio (7 : 8 :1) 212,188 242,500 - (485,000) 30,312
Apport of Design at ratio (3 : 1) 566,484 188,828 - - (7,55,312)
Total 978,672 681,328 Nil Nil Nil

Question No. 3 'b'


Annapurna Cement Factory is located in Dharan. The required limestone for the
factory is brought from two different mines A and B. The distance from mine A
and cement factory is 10 Km while distance from mine B to the factory is 15
Km. A fleet of lorries of 5 tonne carrying capacity is used for the transport of
limestone. The regular data shows that the average speed of lorries in both
mines are 30 Km. per hour. The lorry takes 10 minutes to unload the limestone
at the factory. At mine A loading time is 30 minutes and at mine B loading time
is 20 minutes.
You are given that the depreciation, driver wages, insurance and taxes is NRs 9
per hour operated. Fuel, oil, tyres and maintenance cost is NRs 1.20 per Km.
Draw up a report showing the cost per tonne- kilometer of carrying mineral
from each mine. 10

Answer
Statement showing the cost per Tonne- kilometer of
Carrying mineral from each mine
Mine A Mine B
Particulars NRs. NRs. Ref:

The Institute of Chartered Accountants of Nepal


51 of 109
Suggested Answers of Accounting
CAP II Examination – June 2009

Fixed cost per trip:


(Driver‘s wages, depreciation, Insurance and taxes)
A: 1 hour 20minutes @ NRs 9 per hour 12.00
B: 1 hour 30minutes @NRs 9 per hour 13.50 (W.N.1)
Running and maintenance cost:
(Fuel, oil, tyres, repairs and maintenance)
A: 20 kms NRs1.20 per km. 24.00
B: 30 kms NRs 1.20 per km 36.00
Total cost per trip 36.00 49.50
Cost per Tonne-km 0.72 0.66 (W.N.2)

Working notes:

W.N. 1 Mine A Mine B


Total operated time taken per trip
Running time to & fro 40 minutes 60 minutes
Unloading time 10 minutes 10 minutes
Loading time 30 minutes 20 minutes
Total operated time 80 minutes 90 minutes
1 hours 20 minutes 1 hour 30 minutes
W.N. 2
Effective tones-kms 50 75
(5 tonnes X 10 kms.) (5tonnes X 15
Kms.)

Total cost per trip NRs.36 NRs.49.50

Cost per tonne-km 0.72 0.66


(NRs 36/50tonnes-Kms) (NRs.49.50/75
tonnes-Kms)

Question No. 4
Perfect Piston Ltd. produces 60,000 pistons per annum for its parent company
Perfect Motor Ltd. The pistons are sold to Perfect Motors at NRs. 200 per unit.
The variable cost per piston is NRs. 180. The annual fixed cost of Perfect Piston
Ltd is NRs. 15 lakhs and it is currently operating at 60% capacity.

The company desires to respond to an export enquiry for 30,000 pistons of the
type of it is currently manufacturing. The company's aim is to improve capacity
utilization and avoid loss.

You have to take note of the following benefits that will occur in the export
transactions while determining the F.O.B. price to be quoted.
a) Export incentive by way of cash assistance at 10% of F.O.B. value of
exports.

The Institute of Chartered Accountants of Nepal


52 of 109
Suggested Answers of Accounting
CAP II Examination – June 2009

b) Reimbursement of excise duty on manufacturing inputs by way of 5%


drawback of duty on F.O.B. value of exports.
c) Entitlement of import license to the extent of 10% on F.O.B. value of
exports. The import license can either be sold at a premium of 100% or it
can be utilized to import certain critical auto components that will yield a
30% profit on cost.

Recommend the bare minimum price that the company should quote, in order to breakeven,
assuming: 15
i) It sells the import license in the market.
ii) It imports components against the license and sells them for profit.

Answer
i) Determination of bare minimum price to break-even when import license is sold in
the market
Particulars NRs.
Variable cost per piston 180
Add: Amount per piston towards recovering the present loss
(NRs.300,000 / 30,000 piston) 10
Cost per piston 190
Less: Realization through export benefits:
Cash assistance: 10% on FOB
Drawback of duty: 5% on FOB
Premium on license: 10% on FOB
25% on FOB
i.e. 20% of the cost per piston of NRs. 190 38
Bare minimum FOB price to be quoted 152

ii) Determination of bare minimum price to break-even when import license is used to
import auto components and sell them for profit
Particulars NRs.
Cost per piston, as above 190
Less: export benefits:
Cash assistance: 10% on FOB
Drawback of duty: 5% on FOB
Profit on sale of import: 3% on FOB
18% on FOB
i.e. 15.25% of the cost per piston: NRs. 190 x15.25% 28.98
Bare minimum FOB price to be quoted 161.02

Working note:
Perfect Piston Ltd.'s present operating results:
Contribution per piston = Selling price – Variable cost
= NRs. 200 –NRs. 180
= NRs. 20
Hence,

The Institute of Chartered Accountants of Nepal


53 of 109
Suggested Answers of Accounting
CAP II Examination – June 2009

Total contribution (60,000 piston x NRs. 20) = NRs. 12 lakhs


Less: Annual fixed costs = NRs. 15 lakhs
Loss = NRs. 3 lakhs

Therefore the increase in capacity utilization and the resultant export sales should be
enable to recover the above loss to arrive at a breakeven point.

OR
Question No. 4
The following particulars are available from the records of a manufacturing company.
For 2,000 Units For 5,000 Units
Direct Material NRs. 10,000 NRs. 25,000
Direct Labor 20,000 50,000
Direct Expenses 8,000 20,000
Depreciation 5,000 5,000
Establishment of Expenses 8,000 8,000
The difference in maintenance costs between 2,000 and 5,000 units is NRs.
6,000. The total cost of maintenance for 5,000 units is NRs. 18,000. Similarly,
the difference in selling and distribution expenses is NRs. 5 per unit and total
cost at 2,000 units is NRs. 16,000. Unit selling price amounted to NRs. 33.75.
Required: (4+3+8=15)
i) Segregate the mixed cost into variable and fixed by using high and low point
method.
ii) Find out the total cost for 4,000 and 6,000 units.
iii) Calculate the P/V Ratio, Break Even Point in units, Margin of safety in
Rupees for sale of 4,900 units and target profit at a sales volume of NRs.
162,000.

Answer
i) Maintenance costs and selling and distribution expenses are the items of mixed cost
that require segregation in this case. Accordingly,
Segregation of Maintenance casts:

Unit Variable cost = Δ Cost = 6,000 = NRs.. 2


Δ Output 3,000

Fixed cost = Total cost – Variable cost


18,000 – (5,000× 2)
= NRs. 8,000

Segregation of Selling and Distribution Expenses:

Unit Variable Rate = NRs. 5 (Given)


Fixed Cost = Total Cost – Variable Cost
= 16,000 – (5 × 2,000)
= NRs. 6,000

The Institute of Chartered Accountants of Nepal


54 of 109
Suggested Answers of Accounting
CAP II Examination – June 2009

ii) Calculation of variable cost per unit (total) and fixed cost (total)
(Figures in NRs.)
Particulars VCPU Fixed Cost
Direct Material 5 -
Direct Labor 10 -
Direct Other Expenses 4 -
Depreciation - 5,000
Office and Adm. Expenses 8,000
Maintenance Costs 2 8,000
Selling and Distribution Expenses 5 6,000
Total 26 27,000

Based on the above,


Total cost for 4,000 units = 27,000 + (26×4,000) = NRs..131,000
Total cost for 6,000 units = 27,000 + (26×6,000) = NRs..183,000

iii) P/V Ratio = Unit S.P. – Unit V.C.


Unit S.P.
= 33.75-26
33.75
= 0.2296 or 22.96%

B.E.P in units = Fixed Cost


Unit S.P – unit V.C.
= 27,000
33.75-26
= 27,000
7.75
= 3,484 unit

Margin of safety in rupees = Unit selling price × margin of safety units


(for sales of 4,900 units)
= 33.75 × (4,900-3484)
= NRs. 47,790

Target Profit for sales of NRs.162,000 = (sales × P/V ratio) – Fixed cost
= (162,000 × .2296) – 27,000
= NRs. 10,195

Question No. 5
The following are the details of the Budget and the actual cost in a factory for
six months from January to June 2008. From the figures given below you are
required to prepare the production cost budget for the period from January to
June 2009:

The Institute of Chartered Accountants of Nepal


55 of 109
Suggested Answers of Accounting
CAP II Examination – June 2009

January to June 2008


Particulars Budgeted Actual
Production (units) 20,000 18,000
Material cost NRs.4,000,000 NRs.3,990,000
( 2,000 MT @ NRs.2,000) ( 1,900 MT @ NRs.2,100)
Labor cost NRs.800,000 NRs.799,920
(@ NRs.20 per hour) (@ NRs.22 per hour)
Variable overheads NRs.240,000 NRs.216,000
Fixed overheads NRs.400,000 NRs.420,000

In the first half of 2009, production is budgeted for 25,000 units. Material cost per ton will
increase from last year's actual by NRs.100 but is proposed to maintain the consumption
efficiency of 2008 as budgeted.

Labor efficiency will be lower by another 1% and labor rates will be NRs.22 per hour.

Variable and Fixed overheads will go by 20% over 2008 actual.


You are required to prepare the production cost budget for the period January – June 2009
furnishing all the workings. 10

Answer:
Production Cost Budget
(For the 6 months ending 30th June, 2008)
Production 25,000 units
Total cost (NRs.) Unit cost (NRs.)
Material cost (Note 1) 55,00,000 220.00
Labor cost (Note 2) 11,22,000 44.88
Variable overheads ( Note 3) 3,60,000 14.40
Fixed overheads ( Note 4) 5,04,000 20.16
Total 74,86,000 299.44

Working Notes:
(i) Material cost:
Consumption per unit 2,000/20,000 = 0.10 MT
Consumption for 25,000 units = 2,500 MT
Cost of 2,500 MT @ NRs.2,200 per MT = NRs.55,00,000

(ii) Labor cost:


2008: total budgeted labor hours 800,000/20 = 40,000 hrs.
Labor hour budget for each unit = 40,000/20,000 =2
hrs.
Actual time paid for 799,920/22 = 36,360 hrs.
Less: Standard labor hours for 18,000 units = 36,000 hrs
Extra time taken = 360 hrs. or 1% of standard
labor hours.

2009: Time required for 25,000 units = 50,000 hrs


Add: 2% for lower efficiency = 1,000 hrs
= 51,000 hrs.

The Institute of Chartered Accountants of Nepal


56 of 109
Suggested Answers of Accounting
CAP II Examination – June 2009

51,000 hrs.@ NRs.22 per hour = NRs.11,22,000

(iii)Variable Overhead:
Rate per unit in 2008 = 240,000/20,000 =
NRs.12
Cost for 25,000 @ 2008 rates = NRs.300,000
Add: 20 % = NRs. 60,000
NRs.360,000
(iv) Fixed overhead:
Actual in 2008 = NRs. 420,000
Add:20% = NRs. 84,000
NRs. 504,000

OR
Question No. 5
A factory currently producing 10,000 units working at a 50% capacity level
desires to estimate the profit if operated at 60% and 80% capacity also. The
details of present operation per unit reveal the following:
Selling Price NRs. 200
Material 100
Labor 30
Factory overheads (40% Fixed) 30
Administration Overhead (50% Variable) 20

At 60% working, raw material cost increase by 2% and selling price falls by
2%.

At 80% capacity, raw materials cost increase by 5% and selling price falls by
5%.

Required:
Prepare Flexible budget for all the three capacity level of operation.
10
Answer
Flexible Budget
Capacity in % 50 60 80
Production Units 10,000 12,000 16,000
(Figures in NRs.)
Particulars Rate Total (in'000) Rate Total (in'000) Rate Total (in'000)
A. Sales Revenue 200 2,000 196 2,352 190 3,040
B. Variable cost:
 Materials 100 1,000 102 1,224 105 1,680
 Labor 30 300 30 360 30 480
 Factory Ovh 18 180 18 216 18 288
 Admin Ovh 10 100 10 120 10 160
Sub Total 1,580 1,920 2,608
C. Fixed cost
 Factory 120 120 120

The Institute of Chartered Accountants of Nepal


57 of 109
Suggested Answers of Accounting
CAP II Examination – June 2009

 Administrative 100 100 100


Sub Total 220 220 220
D. Total Cost (B+C) 1,800 2,140 2,828
Net Income (A-D) 200 212 212

Question No. 6
Briefly explain ANY TWO of the following: (2×5=10)

a) Importance of Cost Accounting to business concerns.

Answer
Management of business concerns expects from Cost Accounting detailed
cost information in respect of its operations to equip their executives with
relevant information required for planning, scheduling, controlling and
decision making. To be more specific, management expects from cost
accounting – information and reports to help them in the discharge of the
following functions:
1. Control of material cost: Cost of material usually constitute a substantial
portion of the total cost of product. Therefore, it is necessary to control it
as far as possible. Such control may be exercised (i) by ensuring un-
interrupted supply of material and spares for production; (ii) by
avoiding excessive locking up of funds/capital in stocks of materials and
stores; (iii) also by the use of techniques like value analysis,
standardization etc. to control material cost.

2. Control of labor cost: It can be controlled if workers complete their work


within the standard time limit. Reduction of labor turnover and idle time
to help us to control labor cost.

3. Control of overheads: Overhead consists of indirect expenses which are


incurred in the factory, office and sales department; they are part of
production and sales cost. Such expenses may be controlled by keeping
a strict check over them.

4. Measuring efficiency: For measuring efficiency the cost accounting


department should provide information about standard and actual
performance of the concerned activity.

5. Budgeting: Now a day detailed estimates in terms of quantities and


amounts are drawn up before the start of each activity. This is done to
ensure that a practicable course of action can be chalked out and the
actual performance corresponding with the estimated or budgeted
performance. The preparation of the budget is the function of costing
department.

The Institute of Chartered Accountants of Nepal


58 of 109
Suggested Answers of Accounting
CAP II Examination – June 2009

6. Price determination: Cost account should provide information which


enables the management to fix remunerative selling prices for various
items of products and services in different circumstances.

7. Curtailment of loss during the off seasons: Cost Accounting can also
provide information which may enable reduction of overhead by
utilizing idle capacity during the off season or by lengthening the
season.

8. Expansion: Cost Account may provide estimates of production of


various levels on the basis of which the management may be able to
formulate its approach to expansion.

9. Arriving at decisions: Most of the decisions in a business undertaking


involve correct statements of the likely effect on profits. Cost Accounts
are vital help in this respect. In fact, without proper cost accounting,
decision would be like taking a jump in dark, such as when production
of a product is stopped.

b) Reasons for disagreement of profits as per Cost Accounting and Financial


Accounting.

Answer
Reasons for disagreement of profits as per Cost Accounting and Financial
Accounting are because of the following:

Items included in the financial accounts but not in Cost Accounts


 Appropriation of profits, Transfer to General Reserve, Dividend paid/ received;
 Income tax and other corporate charges;
 Interest Incomes, Matters of pure finance;
 Interest received on bank deposits/investments; amd
 Written off expenditures , Transfer fees , Profit / losses on capital assets
Items included in Cost Accounting
 Opportunity cost of building owned.
 Interest on capital employed in production
 Salary of proprietor.

c) Difference between Cost control and Cost reduction.

Answer
Cost control is operated through setting standards of targets and comparing
actual performance therewith, with a view to identify deviations from

The Institute of Chartered Accountants of Nepal


59 of 109
Suggested Answers of Accounting
CAP II Examination – June 2009

standards or norms and taking corrective action in order to ensure that future
performance confirms to standards or norms.
Cost reduction, on the other hand, is a continuous process of critical cost
examination, analysis and challenge of standards. Each aspect of the
business viz, products, process, procedures, methods, organization,
personnel, etc. is critically examined and reviewed with a view of improving
the efficiency and effectiveness and reducing the costs. Even in an
organization where efficient cost control is in operation, there is always
room for cost reduction.
Cost control lacks the dynamic approach with planned cost reduction
demands. In cost reduction plan, standards which are the basis of control are
constantly challenged for improvement.
Budgetary control and standard costing are essential tools and techniques of
cost control. There are several distinctive tools and techniques of cost
reduction such as value engineering work study, standardization,
simplification, variety reduction, quality measurement and research,
operations research, market research, job evaluation, merit awards,
incentives, improvement in design, mechanization, automation, etc.

Question No. 7
Answer ANY TWO of the following: (2×5=10)

a) Advantages of Cost Audit to Government and Management;

Answer:
Benefit to Government:
 Cost audit facilitates the determination of cost claims submitted to the
Government under the cost plus contracts.
 Price fixation body utilizes the Cost Audit report for fixing the fair selling
price of the commodities.
 Cost Audit may help for granting protection to various industries by giving
them incentives exports, duty drawback etc.
 Audited cost accounts enable government to fix ceiling prices of
commodities to prevent undue profiteering.
 Cost Audit of weak, inefficient or mismanaged units in the industry enable
government to take important decisions, viz., and merger of sick units with
other company.

Benefits to Management:
 Cost Audit helps in detection of errors, defalcations and frauds. thus, the
staff will have to be more careful in their day to day work.

The Institute of Chartered Accountants of Nepal


60 of 109
Suggested Answers of Accounting
CAP II Examination – June 2009

 The management can rely on cost data and cost statements prepared after
getting their cost account audited.
 It helps the management to improve upon the cost accounting methods. It
also facilitates cost control.
 Cost audit help the management in inter firm comparison.
 It enables management to compare actual with estimates and take remedial
measures for adverse variances.

b) Advantages of inter-firm comparison

Answer
 Such a comparison gives an overall view of the industry as a whole to its
members the present position of industry, progress made during the past
and future of the industry.
 It helps a concern in knowing its strengths or weaknesses in relation to
others so that remedial measures may be taken.
 It ensures an unbiased specialized reporting on particular problems of
the concern.
 It develops cost consciousness among members of industry.
 It helps Government in effecting price regulation.
 It helps to improve the quality of products manufactured and to reduce
the cost of production. It is thus, advantageous to the industry as well as
to the society.

c) Limitations of uniform costing.

Answer
 Sometimes it is not possible to adopt uniform standards, methods, and
procedures of costing in different firms due to differing circumstances in
which they operate.

 Many firms do not wish to share cost information and other data with
their competitors.

 Small firms believe that uniform costing is beneficial only in case of


large firms.

 It induces monopolistic trend in the business, due to which prices may


be increased artificially and supplies withheld.

 High initial installation cost:- Small firms in an industry think that the
uniform costing system is meant only for big and medium sized firms
esits installation involves high initial cost which they cannot afford.

The Institute of Chartered Accountants of Nepal


61 of 109
Suggested Answers of Accounting
CAP II Examination – June 2009

Moreover smaller firms may not derive sufficient benefit as compared to


the cost of installation and operation of the system.

 Costing inflexibility:- Since the size and types of firms and the
circumstances in which they operate differ, it is not possible to lay down
standard methods and procedures of costing. The requirement for
uniform casting is inflexible which renders the adaptation of uniform
costing difficult in such firms.

 Non-disclosure of data:- Disclosure of cost and other data are an


essential criterion of uniform costing system. Many efficient firms are
unwilling to disclose their business secrets to the trade association as
they think these will be misused by their competitors in the same
industry. Such non-disclosure of data weakens the uniform costing
system and reduces its utility.

 Unrealistic price fixation:- Uniform costing may lead to monopolistic


tendencies and price may be artificially raised by curtailing the supplies.

 Degree of control:- Uniform costing eliminates divergences in costs up-


to a certain degree beyond which it makes comparisons of costs more
difficult. An attempt to attain complete uniformity in all directions
produces absurd results.

The Institute of Chartered Accountants of Nepal


62 of 109
The Institute of Chartered Accountants of Nepal
Suggested Answers of Financial Management

CAP II Examination – June 2009

Marks
All questions are compulsory

Question No. 1
A textile company is considering two mutually exclusive proposals, proposal I
and II. Under Proposal I, Equipment X will be purchased whereas proposal II
involves the purchase of Equipment Y. Following relevant information is
available in respect of the two proposals:
--------------------------------------------------------------------------------------------
-----------
Proposal I Proposal
II
-------------------------------------------------------------------------------------------------------
Cost Price of Equipment NRs. 1.50 million NRs. 2.50 million
Useful Life of the Equipment 5 years 5 years
Salvage Value of
Equipment at the end of its Useful Life Nil Nil
Additional Working Capital
Required due to Equipment Purchase NRs. 0.50 million NRs. 0.70 million
Sales Revenue Generated
per annum in each of the 5 years NRs. 1.60 million NRs. 2.50 million
Cash Expenses required per annum
in each of the 5 years NRs. 0.35 million NRs. 0.65 million
--------------------------------------------------------------------------------------------
-----------
Both the equipments are subject to written down value method of
depreciation at the rate of 25 percent. You should assume the following:
i) The company does not have any other assets in the block of 25
percent depreciation rate other than the proposed equipment,
ii) Cost of capital of the company is 15 percent,
iii) The company is subject to 30 percent tax rate,
iv) Capital profit is chargeable to tax and capital loss is deductible from
tax at the corporate rate of tax.

On the basis of above information, you are required to: (7+7+2+4=20)


a) Compute the net present value (NPV) of Proposal I,
b) Compute NPV of Proposal II,
c) Advise the company regarding the choice of equipment,
d) Suggest the company on the condition that Equipment X will have
salvage value equal to the book value of the equipment at the end of the
useful life.
Suggested Answers of Accounting
CAP II Examination – June 2009

The Institute of Chartered Accountants of Nepal


64 of 109
Suggested Answers of Accounting
CAP II Examination – June 2009

Answer:
a. Statement of Net Present Value of Proposal I
Particulars NRs. Working Notes
Present Value of Inflows
Sales Revenue 2,933,175 1
Tax Saving on Depreciation 248,069 2
Tax Saving on Capital Loss 53,092 3
Release of Working Capital 248,950 4
A. Sub Total 3,483,286
B. Present Value of Cash Outflows 2,000,000 5
Net Present Value (A-B) 1,483,286

b. Statement of Net Present Value of Proposal II


Particulars NRs. Working Notes
Present Value of Inflows
Sales Revenue 4,341,099 6
Tax Saving on Depreciation 413,447 7
Tax Saving on Capital Loss 88,487 8
Release of Working Capital 348,026 9
A. Sub Total 5,191,059
B. Present Value of Cash Outflows 3,200,000 10
Net Present Value (A-B) 1,991,059

c. Advice regarding the Choice of Equipment


Proposal II involving the purchase of equipment Y is recommended in view of its
higher NVP to the extent of NRs. 507,773 as compared to Proposal I. (NRs.
1,991,059 – NRs. 1,483,286).

d. Suggestion upon change in condition (Working Note 11)


NPV of Proposal I is improved from the previous level of NRs. 1,483,286 to some
extent as a result of the new assumption regarding the salvage value. However, it still
falls short of the NPV of Proposal II by NRs. 383,890 (NRs. 1,991,059 – NRs.
1,607,169). It is therefore recommended that the company should make investment in
Equipment Y rather than in Equipment X.

Working Notes:
W.N. 1
Present Value of Sales Revenue: NRs.
Sales Revenue Generated: 1,600,000
Less: Cash Expenses: 350,000
Cash profit before tax: 1,250,000
Less: Taxes @ 30 per cent: 375,000
Cash Flow after Taxes: 875,000
PV Factor of Annuity for 5 Years: 3.3522

The Institute of Chartered Accountants of Nepal


65 of 109
Suggested Answers of Accounting
CAP II Examination – June 2009

PV of Sales Revenue: 2,933,175

W.N. 2
PV of Tax Saving due to Depreciation:
Year Depreciation Tax Savings PV Factor PV of Tax Savings
(1) (2) (3) = (2) X 0.30 (4) (5) = (3) X (4)
1 NRs. 375,000 NRs. 112,500 0.86957 NRs. 97,827
2 281,250 84,375 0.75614 63,799
3 210,938 63,282 0.65752 41,609
4 158,203 47,461 0.57175 27,136
5 118,652 35,596 0.49718 17,698
1,144,043 NRs. 248,069

W.N. 3
PV of Tax Savings on Capital Loss: NRs. 53,092
(NRs. 1,500,000 – 1,144,043) X 0.30 X 0.49718

W.N. 4
Release of Working Capital: NRs. 248,950
(NRs. 500,000 X 0.49718)
W.N. 5
Particulars NRs.
Cost price of Equipment: 1,500,000
Additional Working Capital: 500,000
PV of outflows 2,000,000

W.N. 6

Present Value of Sales Revenue:


Sales Revenue Generated: 2,500,000
Less: Cash Expenses: 650,000
Cash profit before tax: 1,850,000
Less: Taxes @ 30 per cent: 555,000
Cash Flow after Taxes: 1.295,000
PV Factor of Annuity for 5 Years: 3.3522
PV of Sales Revenue: 4,341,099

W.N. 7

PV of Tax Saving due to Depreciation:


Year Depreciation Tax Savings PV Factor PV of Tax Savings
(1) (2) (3) = (2) X 0.30 (4) (5) = (3) X (4)
1 NRs. 625,000 NRs. 187,500 0.86957 NRs. 163,044
2 468,750 140,625 0.75614 106,332
3 351,563 105,469 0.65752 69,348
4 263,672 79,102 0.57175 45,227

The Institute of Chartered Accountants of Nepal


66 of 109
Suggested Answers of Accounting
CAP II Examination – June 2009

5 197,754 59,326 0.49718 29,496


1,906,739 NRs. 413,447
W.N. 8
PV of Tax Savings on short term Capital Loss: NRs. 88,487
(NRs. 2,500,000 – 1,906,739) X 0.30 X 0.49718

W.N. 9
Release of Working Capital: NRs. 348,026
(NRs. 700,000 X 0.49718)
W.N. 10
Particulars NRs
Cost price of Equipment: 2,500,000
Additional Working Capital: 700,000
PV of Outflows: 3,200,000

W.N. 11
In case Equipment X will have salvage value equal to book value at the end of the useful
life, the computation done earlier in the case of NPV of Proposal I will require
modification for:
a. Cash inflow due to salvage value, and
b. Non-occurrence of short term capital loss computed earlier.

The salvage value of the Equipment X = Purchase Cost – Accumulated


Depreciation
= 1,500,000 – 1,444,043
= NRs. 355,957.

Total Present Value previously computed: NRs. 3,483,286


Less: PV of Tax Saving on Short-term Capital Loss
since there will not be any loss now: (-) 53,092
Add: PV of Salvage Value (355,957 X 0.49718) 176,975
NRs.
3,607,169
Less: Cash Outflows: NRs. 2,000,000
Revised NPV of Proposal I: NRs. 1,607,169

Question No. 2 'a'


Saleways Ltd. purchased a retail store in Kathmandu Valley and commenced
the business on Shrawan 1, 2064. Following information is also available
regarding the operations of the retail store:
Capital introduced on Shrawan 1 NRs. 2,350,000
Drawings during the year 250,000
Working Capital (current assets less current liabilities) 1,150,000
Depreciation of fixed assets during the year, based on
a rate of 20 per cent per annum on cost 150,000
Ratio of annual sales to "year-end values of

The Institute of Chartered Accountants of Nepal


67 of 109
Suggested Answers of Accounting
CAP II Examination – June 2009

fixed assets plus working capital" 2:1


Ratio of current assets to current liabilities at the year-end 2:1
Ratio of liquid assets (cash plus debtors) to
current liabilities on year-end 5:4
Debtors at the year-end as per cent of annual sales 12
General expenses (excluding depreciation)
as per cent of annual sales 20

The current assets consist of stocks (which is unchanged throughout the year),
debtors and cash. Stocks were turned over four times during the year. The
current liabilities consisted only of creditors.

After considering all the information provided above, you are required to
prepare the following in as much details as possible:
(6+6=12)
i) Trading and profit and loss account for the year ended 32 Ashadh 2065, and
ii) Balance sheet as at Ashadh end 2065.
Answer:
i) Trading and P/L Account of Saleways Ltd.
for the year ended on 32 Ashadh 2065
Debit Credit
Particulars NRs Particulars NRs
To Cost of Sales 3,450,000 By Sales 3,500,000
To Gross Profit c/d 50,000
Total 3,500,000 Total 3,500,000

To General Expenses 700,000 By Gross Profit b/f 50,000


To Depreciation 150,000 By Net Loss c/d 800,000
Total 850,000 Total 850,000

Balance Sheet of Saleways Ltd.


as at 32 Ashadh 2065
Equity and liabilities NRs Assets NRs
Opening Capital = 2,350,000 1,750,000 Property Plant & Equip 600,000
Add: Capital Intro= 450,000 Less: Depreciation
Less: Net Loss = (800,000) (750,000 -150,000)
Less: Drawings = (250,000)
Creditors 1,150,000 Current Assets
 Cash and Cash Equivalent 1,017,500
 Debtors 420,000
 Inventories 862,500
Total 2,900,000 Total 2,900,000

Working Notes:

The Institute of Chartered Accountants of Nepal


68 of 109
Suggested Answers of Accounting
CAP II Examination – June 2009

W.N. 1: Special Note


Only one item of current liabilities is Creditors as stated in the question. So the balancing
source of fund of NRs. 450,000 might be treated as capital introduction during the year or
long term loans. Here in solution it is shown as new capital introduction. But students
may alternatively show the balancing figure as long term loans.

W.N. 2: Determination of Current Assets and Current Liabilities


CA – CL = NRs. 1,150,000
0.5 CA – CL = 0
Subtracting equation (2) from equation (1), we get:
0.5 CA = NRs. 1,150,000, Or CA = NRs. 2,300,000
CL = NRs. 1,150,000 since there are no other current liabilities.

W.N. 3: Determination of Property, Plant & Equipment (PPE)


Depreciation @ 20 per cent = NRs. 150,000
Cost of PPE = NRs. 150,000 X 100/20 = NRs. 750,000
PPE (after Depreciation) = NRs. (750,000 – 150,000) = NRs. 600,000

W.N. 4: Determination of Sales


Sales / (Fixed Assets + Working Capital) = 2
Or, Sales / NRs. (600,000 + 1,150,000) =2
.*.Sales = NRs. 3,500,000

W.N. 5: Determination of Liquid Assets


Liquid Ratio = Liquid Assets Or, 1.25 = Liquid Assets
Current Liabilities 1,150,000
Or, Liquid Asset = NRs. 1,437,500, which consists of cash and debtors.
(a) Debtors are 12% of annual sales = NRs. 3,500,000 X 0.12 = NRs. 420,000.
(b) Cash = NRs. 1,437,500 – NRs. 420,000 = NRs. 1,017,500.

W.N. 6: Determination of Stock


Stock = Current Assets – Liquid Assets
= NRs. 2,300,000 – NRs. 1,437,500
= NRs. 862,500

W.N. 7: Determination of Cost of Sales


Stock Turnover Ratio = Cost of Sales/Average Stock
Or, 4 = Cost of Sales/NRs. 862,500,
.*.Cost of Sales = NRs. 3,450,000

Question No. 2 'b'


Assume the total cost of a college education will be NRs.300,000 when your
child enters college in 18th year from now. You presently have Rs.24,240 to
invest. What annual rate of interest must you earn on your investment to cover
the cost of your child‘s college education? 3

The Institute of Chartered Accountants of Nepal


69 of 109
Suggested Answers of Accounting
CAP II Examination – June 2009

Answer
The annual rate of interest that I must earn on my investment to cover the cost of my child's
college can be derived as per the below formula:
Future Value = Present Value (1+Interest Rate) No of years
Or NRs.300,000 = NRs.24,240 (1 + Interest Rate)18
.*.Interest Rate = 15%.

Question No. 3 'a'


The Mike Company‘s financing plans for next year include the sale of long-
term bonds with a 10 percent coupon. The company believes it can sell the
bonds at a price that will provide a yield to maturity of 12 percent. If the
marginal tax rate is 34 percent, what is Mike‘s after tax cost of debt? 3

Answer
Mike's after tax cost of debt can be derived as per the below formula:
After tax cost of debt = Yield to Maturity ( 1- Tax Rate)
= 12% (1- 0.34)
= 7.92%
Question No. 3 'b'
DEF Ltd. plans to expand assets by 60 percent. To finance the expansion, it has
a choice between a straight 9 percent debt issue and equity issue. Its current
balance sheet and income statement are as shown below:
Balance Sheet of DEF Ltd. as on 32 Ashadh
------------------------------------------------------------------------------------------
-----------
5% Debt NRs. 1,000,000 Total Assets 5,000,000
Equity Shares
(NRs. 100 per share) 2,500,000
Retained Earnings 1,500,000 ________ 5,000,000

Income Statement for the year ended on 32 Ashadh


------------------------------------------------------------------------------------------
-----------
Sales NRs.
15,000,000
Total cost (excluding interest) 13,450,000
Earning before Interest and Taxes (EBIT) 1,550,000
Less: Interest on Debt 50,000
Earning before Tax (EBT) 1,500,000
Less: Taxes 450,000
Net Income 1,050,000

If the company finances the proposed expansion with debt, the rate on the
incremental debt will be 9 percent and the price/earning ratio of the equity
shares will be 10. If expansion is financed by equity, the new shares can be sold

The Institute of Chartered Accountants of Nepal


70 of 109
Suggested Answers of Accounting
CAP II Examination – June 2009

at NRs. 300 and the price earnings ratio of all the outstanding equity shares will
remain 12.

Considering all the information given above, you are required to do the
following: (8+2+2=12)

i) Assuming that net income before interest on debt and taxes (EBIT) is 10
percent on sales, calculate EPS at assumed sales of NRs. 10 million, NRs.
20 million and NRs. 25 million under the alternative mode of financing the
expansion program (assume no fixed costs).
ii) Using the price/earnings ratio indicated, calculate the market value of equity
share for each sales level for both the debt and equity methods of financing.
iii) If the firm follows the policy of seeking to maximize the price of its shares,
which form of financing should be employed?

Answer:
(i) & (ii)
Computation of EPS and MPS under alternative financing plans
(Figures in NRs.
‗000)
-------------------------------------------------------------------------------------------------------
-----
Sales Level
_______________
NRs. 10 million NRs. 20 million NRs. 25
million
Debt Equity Debt Equity Debt
Equity
EBIT NRs. 1,000 Rs 1,000 NRs. 2,000 NRs. 2,000 NRs. 2,500
NRs. 2,500
Less: Interest *320 50 320 50 320
50
EBT 680 950 1,680 1,950 2,180
2,450
Less: Taxes (0.30) 204 285 504 585 654
735
EAT 476 665 1,176 1,365 1,526
1,715

Now
1. EAT 476,000 665,0000 1,176,000 1,365,000 1,526,000
1,715,000
2. No. of Equity Shares: 25,000 35,000 25,000 35,000 25,000
35,000

The Institute of Chartered Accountants of Nepal


71 of 109
Suggested Answers of Accounting
CAP II Examination – June 2009

3. EPS (1 / 2) 19.04 19 47.04 39 61.04


49
4. P/E Ratio 10 12 10 12 10
12
5. Market Price of Share: 190.4 228 470.4 468 610
588
-------------------------------------------------------------------------------------------------------
-----

(iii)The answer will depend on the expected level of sales. If the sales level is at
NRs. 10 million, equity form of financing should be employed as it will
have better MPS. At the sales level of NRs. 20 million and NRs. 25 million,
MPS under debt form of financing is to be preferred for higher MPS as
compared to equity mode of financing.

Working Notes:
 In debt financing, the number of equity shares outstanding = 2,500,0000 / 100 =
25,000
 In the case of equity financing, the total number of outstanding shares will be as
computed below:
Additional equity to be raised = 0.60 X 5,000,000 = NRs. 3,000,000
Price per share at which shares can be sold: NRs. 300
Additional Number of shares = NRs. 3,000,000/NRs. 300 = 10,000
Existing Shares = 25,000
Total Number of Shares = 25,000 + 10,000 = 35,000

Question No. 4 'a'


The earnings per share of a company is NRs. 40 and the rate of capitalization is
15 percent. The company has an option of adopting (i) 50 and (ii) 75 per cent
dividend pay out ratio.
You are required to compute the market price of the company‘s quoted shares
as per Walter‘s model if it can earn a return of (i) 20 and (ii) 15 percent on its
retained earnings. 4

Answer
Price of share if return (r) = 0.20
(i) D/P Ratio @ 50% (ii) D/P Ratio @ 75%
Price = 20 + 0.20/0.15 (40 - 20) Price = 30 + 0.20/0.15 (40 – 30)
0.15 0.15
= 20 + 26.66 = 30 +13.33
0.15 0.15
= NRs. 311.11 = NRs. 288.88

Price of share if return (r) = 0.15

The Institute of Chartered Accountants of Nepal


72 of 109
Suggested Answers of Accounting
CAP II Examination – June 2009

(i) D/P Ratio 50% (ii) D/P Ratio 75%


Price = 20 + 0.15/0.15 (40 - 20) Price = 30 + 0.15/0.15 (40 – 30)
0.15 0.15
= 20 + 20 = 30 + 10
0.15 0.15
= NRs. 266.66 = NRs. 266.66

Question No. 4 'b'


A company deals with consumer durables, having an annual turnover of NRs.
65 million, 80 percent of which are credit sales effected through a large number
of dealers. The balance sales are made through company‘s show room on cash
basis. The normal period of credit allowed by the company for sales effected
through dealers is 30 days.
The company wants to expand its business substantially and there is good
prospect as well. However, the marketing manager finds that the dealers have
difficult in holding more stocks due to financial problems. He therefore
proposes a change in the credit policy as follows:
(NRs. in Million)
---------------------------------------------------------------------------------------
------------
Proposal Credit period Anticipated Credit
Sales
---------------------------------------------------------------------------------------------------
A 45 days 65
B 60 days 75
--------------------------------------------------------------------------------------------------
-

- ---------

The products yield an average contribution of 20 per cent on sales.

Fixed costs amount to NRs. 3,200,000 per annum. The company expects a pre-
tax return of 25 percent on capital employed. The finance manager, after a
review of the proposal, has recommended increasing the provision for bad debts
from the current 0.5 percent to 1 percent for proposal A and 2 percent to
proposal B.

Based on the above facts and information, you are required to: (9+1=10)
i) evaluate the merits of the new proposals vis-à-vis their impact on the
profitability of the company, and
ii) recommend the best policy which the company should choose.

Answer
a)
Evaluation of Proposed Credit Policies

The Institute of Chartered Accountants of Nepal


73 of 109
Suggested Answers of Accounting
CAP II Examination – June 2009

-------------------------------------------------------------------------------------------------------
-----
Particulars Present Proposal A Proposal
B
------------------------------------------------------------------------------------------------------------
----
Sales Revenue: NRs. 52,000,000 NRs. 65,000,000 NRs.
75,000,000
Less: Variable Costs (0.80) 41,600,000 52,000,000
60,000,000
Less: Fixed Costs 3,200,000 3,200,000
3,200,000
Less: Bad Debts 260,000 650,000
1,500,000
Less: Cost of Investment in Debtors* 560,000 1,725,000
2,633,333
Profit: 6,380,000 7,425,000
7,666,667
------------------------------------------------------------------------------------------------------------
----
Recommendation: Proposal B generating maximum expected profit of NRs. 7,666,667
is the best among the three proposal and therefore recommended.

Working Notes:
Cost of Investment in Debtors
-------------------------------------------------------------------------------------------------------
-----
Particulars Present Proposal A Proposal
B
------------------------------------------------------------------------------------------------------------
----
(i) Total investment in Debtors: NRs. 44,800,000 NRs. 55,200,000 NRs.
63,200,000
(Total VC + Total FC)
(ii) Debtors Turnover Ratio 12 8 6
(360 days/Average Collection Period) (360/30) (360/15)
(360/60)
(iii) Average Investment in Debtors (i/ii) 3,733,333 6,900,000
10,533,333
(iv) Cost of Average Investment in Debtors: 933,333 1,725,000
2,633,333
(v X 0.25)

Question No. 4 'c'


An investor holds the following portfolio:

The Institute of Chartered Accountants of Nepal


74 of 109
Suggested Answers of Accounting
CAP II Examination – June 2009

---------------------------------------------------------------------------------------
------------
Share Beta Investment
(Rs.)
---------------------------------------------------------------------------------------
------------
Sigma 0.8 2,500,000
Beta 1.2 3,500,000
Alpha 1.5 4,000,000

---------------------------------------------------------------------------------------
------------
You are required to answer the following questions: (3+3=6)
i) What is the portfolio beta of the investor?
ii) What is the expected rate of return on the investor‘s portfolio, if the risk-free
rate is 8 percent and the expected return on market portfolio is 18 percent?

Answer
i. The portfolio beta of the investor is 1.22. (W.N. 1)
ii. The expected rate of return on the investor‘s portfolio is 20.2 percent. (W.N.
2)

Working Notes
W.N.1
For given beta, the required rate of return is obtained using the following
formula:

Share Investment (NRs.) Weight Beta Weight * Beta


Sigma 2,500,000 0.25 0.8 0.20
Beta 3,500,000 0.35 1.2 0.42
Alpha 4,000,000 0.40 1.5 0.60
Σ Weight * Beta = Portfolio Beta 1.22

W.N.2
We have Expected Return on Investor's Portfolio E (rp) = Rf + Bp (Rm – Rf)
Where
Rf = Risk Free Return
Bp = Portfolio Beta
Rm = Reurn on Market Portfolio

Hence,
E (rp) = 8 % + 1.22 (18 % - 8%)
= 20.2 %

The Institute of Chartered Accountants of Nepal


75 of 109
Suggested Answers of Accounting
CAP II Examination – June 2009

OR

Question No. 4 'a'


Excess Ltd. currently pays a dividend of NRs. 40 per share and this dividend is
expected to grow at a 15 per cent annual rate for 3 years, then at a 10 per cent
rate for the next 3 years, after which it is expected to grow at a 5 percent rate
forever.
What value would place on the stock if an 18 percent rate of return were
required? 6

Answer
The present value of stock is NRs. 452.99. The workings are as per below:

Statement showing the Value of the Share


-----------------------------------------------------------------------------------------------------
PV Factor
End of Dividend @ 18% PV of Dividend
-----------------------------------------------------------------------------------------------------
----
Year 1 40 (1.15) = 46.00 0.84746 38.98
2
Year 2 40 1.15) = 52.90 0.71818 37.99
Year 3 40 (1.15)3 = 60.84 0.60863 37.03
Year 4 60.84 (1.10) = 66.92 0.51579 34.52
Year 5 60.84 (1.10)2 = 73.62 0.43711 32.18
3
Year 6 60.84 (1.10) = 80.98 0.37043 30.00
210.70
-----------------------------------------------------------------------------------------------------
----

Year 7 dividend = NRs. 80.98 X 1.05


= NRs. 85.03

Market Value at the end of Year 6 = NRs. 85.029 / (0.18 – 0.05)


= NRs. 85.03/0.13
= NRs. 654.08

Present Value of Market Value at the end of Year 6 = 0.37043 X 654.08


= NRs. 242.29

Hence, Value of the Share = NRs. 210.70 + NRs. 242.29


= NRs. 452.99

Question No. 4 'b'

The Institute of Chartered Accountants of Nepal


76 of 109
Suggested Answers of Accounting
CAP II Examination – June 2009

A manufacturing company sells goods at a gross profit of 25 percent. The


company is following a practice to include depreciation as part of the cost of
production on a consistent basis.

You have also been provided with the following details in respect of the
costs/expenses of the company for the 12-month period ending Ashadh 32,
current year:
- Sales at 3 month‘s credit NRs. 5,400,000
- Materials consumed (Supplier‘s credit is for 3 months) : NRs. 1,350,000
- Total Wages is NRs. 1,080,000. There is a system of payment of wages twice a month.
The second wage payment of a month takes place on the last day of the month.
- Manufacturing expenses outstanding at the end of the year (cash expenses are paid one
month in arrear): NRs. 120,000
- Total administrative expenses (paid as above): NRs. 360,000
- Sales promotion expenses (paid quarterly in advance) : NRs. 180,000

On the basis of the above information, you are required to determine the
working capital needs of the company on a cash cost basis on the following
assumptions: 10
i) A safety margin of 15 percent is to be maintained.
ii) Cash is held to the extent of 60 percent of current liabilities.
iii) There will be no work-in-progress.
iv) Tax is to be ignored.
v) Finished Goods are to be valued at manufacturing costs.
vi) Stocks of raw materials and finished goods are kept at one month‘s
requirement.

Answer
The company needs NRs. 1,575,500 as the working capital on a cash cost basis.
The workings of the calculation is given as per below :

Working Notes

W.N. 1
Since the cash to be held is given to be 50 per cent of current liabilities, we
would first compute the current liabilities and then cash in hand component of
the current assets.

Statement of Computation of Working Capital of a FMCG Company


(A) Current Assets:
i) Raw Materials (1,350,000/12) NRs. 112,500
ii) Finished Goods (3,870,000/12) 322,500
iii) Debtors (4,410,000 X 3/12) 1,102,500
iv) Sales Promotion Expenses (180,000 X 3/12) 45,000

The Institute of Chartered Accountants of Nepal


77 of 109
Suggested Answers of Accounting
CAP II Examination – June 2009

v) Cash in Hand (532,500 X 0.60) 319,500


Total Current Assets: 1,902,500

(B) Current Liabilities:


i) Creditors (1,350,000 X 3/12) 337,500
ii) Manufacturing Expenses 120,000
iii) Administrative Expenses (360,000/12) 30,000
iv) Wages (1,080,000 X 1/24) 45,000
Total Current Liabilities: 532,500

(C) Net Working Capital: (A – B) 1,370,000


Add: Safety Margin: (1,370,050 X 0.15) 205,500
Working Capital Required on Cash Cost basis: NRs. 1,575,500

W.N. 2: Other Computations

Sales: NRs. 5,400,000


Less: Gross Profit Margin: NRs. 1,350,000
(5,400,000 X 0.25) ___________
Total Manufacturing Costs: 4,050,000
Less: Cost of materials consumed: 1,350,000
Less: Wages 1,080,000 2,430,000
 Manufacturing Expenses other than material and wages 1,620,000

 Cash Manufacturing Expenses: (NRs. 120,000 X 12) 1,440,000


 Depreciation (1,620,000 – 1,440,000) 180,000
 Cash Manufacturing Cost (4,050,000 – 180,000) 3,870,000
 Cash Cost of Sales (3,870,000 + 360,000 +180,000) 4,410,000

Question No. 4 'c'


The following information is available in respect of the rate of return on
investment (r), capitalization rate (ke) and earnings per share (E) of Excel Ltd.
r = 12 per cent
E = NRs. 90
Determine the value of the company‘s share under different situations as
described below based on Gordon‘s Model. 4
__________________________________________________________
______
Situation D/P Ratio (1 – b) Retention Ratio (b) ke (%)
__________________________________________________________
______
I 10 90 20
II 20 80 20

The Institute of Chartered Accountants of Nepal


78 of 109
Suggested Answers of Accounting
CAP II Examination – June 2009

III 40 60 15
IV 60 40 15

Answer
The value of the share of the company under different situations based on
Gordon‘s Model is as per below:
Situation Value (NRs.)
I 97.83
II 173.08
III 461.54
IV 529.41
Working Notes:
According to Gordon‘s model, we have:
P = E (1 – b)_ where
(ke – br)
P = Price of a share,
E = Earnings per share,
b = Retention ratio or percentage of earnings retained,
1 – b = D/P ratio which is the percentage of earnings distributed in the form of
dividend,
ke = Capitalization rate or cost of capital
br = g = Growth rate = rate of return on investment of an all-equity firm.

Hence applying the above formula on the various situations, we have the value
of share as per below:

(a) D/P Ratio: 10%, Retention: 90%, r = 12%, br (g) = 0.9 X 0.12 = 0.108
P= 90 (1 – 0.90)_ = 90 X 0.1 = 9/.092 = 97.83
(0.20 – 0.108) 0.092
(b) D/P Ratio : 20%, Retention: 80%, r = 12%, br (g) = 0.8 X 0.12 = 0.0.096
P= 90 (1 – 0.80)_ = 90 X 0.2 = 18/0.104 = 173.08
(0.20 – 0.096) 0.104
(c) D/P Ratio : 40%, Retention: 60%, r = 12%, br (g) = 0.6 X 0.12 = 0.0.072
P= 90 (1 – 0.60)_ = 90 X 0.4 = 36/0.078 = 461.54
(0.15 – 0.072) 0.078
(d) D/P Ratio : 60%, Retention: 40%, r = 12%, br (g) = 0.4 X 0.12 = 0.0.048
P= 90 (1 – 0.40)_ = 90 X 0.6 = 54/0.102 = 529.41
(0.15 – 0.048) 0.102

Question No. 5
You are the financial analyst of a FMCG company based in Hetauda. The
company wishes to raise additional finance of NRs. 30 million to meet its
investment plans. It has NRs. 6.3 million in the form of retained earnings
available for investment purpose.
Following further details are also available:

The Institute of Chartered Accountants of Nepal


79 of 109
Suggested Answers of Accounting
CAP II Examination – June 2009

a) Debt-equity mix to be maintained at 30:70.


b) Cost of debt: Up to NRs. 6 million, 12 percent (before tax); Beyond NRs. 6
million, 15 per cent (before tax)
c) Earnings per share: NRs. 80
d) Dividend pay out, 50 percent of earnings
e) Expected growth rate of dividends, 20 percent
f) Current market price per share, NRs. 660
g) Tax rate, 30 percent
On the basis of information given above, you are required to determine/compute the
following:
(4×2.5=10)
i) Pattern of raising the additional finance, assuming the company intends to
maintain the existing debt equity mix,
ii) Post tax average cost of additional debt,
iii) Cost of retained earnings and cost of equity,
iv) Overall weighted average after tax cost of additional finance.

Answer:
i) Pattern of raising additional finance

Debt = 0.30 X NRs. 30 million = NRs. 9 million


Equity Funds: 0.70 X NRs. 30 million = NRs. 21 million

Statement of Debt- Equity Position


Retained Earnings NRs. 6,300,000
Equity Share Capital (Additional) NRs. 14,700,000 NRs.
21,000,000
Debt Funds (NRs. 9 million)
12% Debt NRs. 6,000,000
15% Debt NRs. 3,000,000 NRs.
9,000,000
NRs.
30,000,000

ii) Post tax average cost of additional debt


kd = Total Interest (1 – t)/9,000,000
= (NRs. 720,000 + NRs. 450,000) (1 – 0.30)/9,000,000
= 1,170,000 X 0.70/9,000,000
= 9.10 %

iii) Cost of retained earnings and cost of equity


a. Cost of equity (ke) = (D/P)+ g
= NRs. 80 (0.5) + 0.2
NRs. 660

The Institute of Chartered Accountants of Nepal


80 of 109
Suggested Answers of Accounting
CAP II Examination – June 2009

= 26.06%

b. Cost of retained earning (kr) = Ke = 26.06 %

iv) Overall weighted average after tax cost of additional finance


ko = (ke*0.70) +(kd*0.3)
= (0.2606*0.70)+ (0.091*0.3)
= 0.1824+0.0273
= 0.2097
= 20.97%

Question No. 6 'a'


Indicate whether the following statements are ‗True or False‘ and also support
your answer with reasons: (4×2.5=10)
i) The cash held with the purpose of future risk and contingencies is referred
to as transaction motive.

Answer
False, because the cash held with the purpose of future risk and
contingencies is referred to as precautionary motive. Transaction motive
refers to cash held in order to carry on the day to day operations of the
business.

ii) A certain firm has a credit term of 3/15, net 45 while selling goods. Here the
term ‗15‘ means discount period.

Answer
True, because if the firm pays within 15 days, it will be entitled to receive 3
percent discount.

iii) Aging schedule of receivables is the maximum limit on the amount the
company will permit the firm to owe at any one point of time.

Answer
False, because aging schedule of receivables is the tabular classification of
amounts remaining to be collected. The maximum limit on the amount the
company will permit the firm to owe at any one point of time is referred to
as a line of credit.

iv) The advantage of long-term debt from company's viewpoint is that they do
not participate in superior profit and do not participate in the control of the
firm.

The Institute of Chartered Accountants of Nepal


81 of 109
Suggested Answers of Accounting
CAP II Examination – June 2009

Answer
True because long-term debt holders do not participate in superior profit and
do not participate in the control of the firm. Common stockholders,
however, participate in superior profit and control of the firm.

Question No. 6 'b'


Write short notes on ANY FOUR: (4×2.5=10)
i) Financial distress
Answer
Financial distress refers to financial failure. Failure can be defined in several
ways, and some failures do not necessarily result in the collapse and
dissolution of a firm. Failure in an economic sense could mean that a firm is
losing money – its revenues do not cover its costs. It could also mean that its
earnings rate is less than its cost of capital. A related definition would be
that the present value of cash flows of the firm is less than its obligations. In
still another sense, failure occurs when the firm‘s actual cash flows are short
of its expected cash flows – its projections have not been met. Or failure
could refer to insolvency. The recent literature relates financial distress to
insolvency, distinguishing between a flow basis and a stock basis.

ii) Horizontal merger and Vertical merger


Answer
Horizontal Merger: A horizontal merger is a merger between two companies
in the same line of business or a merger between two competitors. Suppose,
for example, Pepsi were to buy Coca-Cola. This would be a horizontal
merger. Horizontal mergers may negatively affect the competitive situation
in an industry. Therefore, they frequently run afoul of regulatory officials. A
horizontal merger often increases the degree of concentration in an industry.

Vertical Merger: A vertical merger acquires another firm that is ‗upstream‘


or ‗downstream‘. It occurs when a supplier buys a reseller, or vice versa.
The key point is that the two companies have a buyer-seller relationship.
Suppose that a jewelry retailer purchased a company that manufactures
jewelry. This would be a vertical merger. Or, suppose that a pharmaceutical
company acquired a drugstore chain.

iii) Cost of the perpetual debt


Answer
The cost of perpetual debt is the rate of return which the lenders expect from
such debt. The coupon rate of interest or market yield on debt approximately
represents the cost of debt.

The Institute of Chartered Accountants of Nepal


82 of 109
Suggested Answers of Accounting
CAP II Examination – June 2009

Another aspect which needs mention regarding the cost of perpetual debt is
that effective cost of debt should be computed as after-tax cost not before
tax. The coupon or nominal rate written in the face of the debt document is
the before cost of debt. Therefore, adjustment in the coupon rate is essential
to obtain the after tax cost of perpetual debt.
In order to compute the cost of perpetual debt, following types of data is
required:
i) Net cash proceeds/inflows from the particular type of debt,
ii) Net cash outflows in terms of amount of periodic interest
payment and repayment of principal in installments or lump sum
on maturity.

Since the interest paid on debt by a firm is tax deductible, it is essential that
the effect cost of the debt is obtained after considering the tax savings
generated as a result of interest payment.
Another important aspect which also needs consideration is that the bonds
and debentures can be issued at (i) par, (ii) discount and (iii) premium. The
coupon rate of interest needs adjustment to ascertain the actual cost of debt.
Following two formulas are employed to determine the cost of perpetual
debt:
ki = I
SV
kd = I (1 – t), where
SV
ki = Before tax cost of debt,
kd = Tax-adjusted cost of debt,
I = Annual interest payment
SV = Sale proceeds of the bond/debenture
t = Tax Rate

iv) Safety Stock


Answer
Safety stock may be defined as the minimum additional inventory which
serves as a safety margin or cushion at times when the actual lead time
and/or usage rate are more than previously anticipated.
In the inventory management techniques, we compute economic order
quantity and the reorder point on the assumption that every thing is certain.
In these models, we assume that there will be a constant or fixed usage of
inventory and that the inventory will be replenished as and when needed.
These assumptions do not, however, hold well in actual situations where
uncertainties creep up every now and then due to a number of reasons. For
instance, the demand for the goods produced by the firm may increase
unexpectedly which require materials in excess quantity than previously
planned. Similarly, the receipt of inventory from the markets may be

The Institute of Chartered Accountants of Nepal


83 of 109
Suggested Answers of Accounting
CAP II Examination – June 2009

delayed due man made and natural disaster such as strikes, floods,
transportation and other problems.
When the demand for inventory increases or delivery of inventory is
hindered by whatever reasons, the firm will face a situation of shortage of
inventory or stock-out. Safety stock serves the firm better to tackle such
situations or eventualities

v) Yield to call (YTC) and Yield to Maturity (YTM)


Answer
The Yield to maturity (YTM) or redemption yield of a bond or debentures,
is the internal rate of return (IRR, overall interest rate) earned by an investor
who buys the bond or debenture today at the market price, assuming that the
bond will be held until maturity, and that all coupon and principal payments
will be made on schedule. Yield to maturity is actually an estimation of
future return, as the rate at which coupon payments can be reinvested at
when received is unknown. It enables investors to compare the merits of
different financial instruments.
The Yield to call (YTC) is one of the variants of YTM. It is the rate of
return if held up to call. When a bond or debenture is callable (can be
repurchased by the issuer before the maturity), the market looks also to the
Yield to call, which is the same calculation of the YTM, but assumes that
the bond will be called, so the cash flow is shortened.

The Institute of Chartered Accountants of Nepal


84 of 109
The Institute of Chartered Accountants of Nepal
Suggested Answers of Business Communication and Marketing

CAP II Examination – June 2009

Marks
All questions are compulsory.

SECTION 'A'

Question No. 1
You recently complained to your boss that you were unhappy with a piece of equipment
that you use (printer, computer, copier, fax, or the like). After some thought; the boss
decided you were right and told you to go shopping. Compare at least three different
manufacturers‘ models and recommend one. Since the company will be purchasing ten or
more units and since several managers must approve the purchase, write a careful report
documenting your findings. Establish at least five criteria for comparing the models.
Submit a memo report to your boss. (5+5=10)
Answer ………… 2009
To, …………………

Subject: Quality Survey of Office Computing Equipment

Dear Sir/ Madam

I take pleasure in submitting the report on 'Quality Survey of Office Computing'


Equipment as per your advice dated ………..2009.

I have studied the poor performance quality and tangible drawbacks of some of the
existing computing equipments of our office - computer sets, copiers, printers, fax
machine, etc. and studied the comparative advantages, qualities and benefits of each
with the new equipments readily available in the market.
The various brands available in the market that matches our desired technical
specifications are:
1. Cannon
2. Hewlett Packard
3. Mita
4. Lenovo
Though all the brands are more or less equally good and worth buying I, on the basis of
my observation and personal experiences, and also on the basis of the taste of our
managers of various departments, would like to recommend Canon over all other brands.
I have focused on the following five reasons for the justification of my choice.
1. Lower acquisition cost;
2. Minimum operating cost;
Suggested Answers of Accounting
CAP II Examination – June 2009

3. User friendly;
4. Easy repair and maintenance; and
5. Extended warranty and after sales service.
I believe my report will facilitate the management in acquiring the best equipment
available in the market.

I am available for further clarification if required.

Sincerely,

………………………
(Manager)

Draft notes on the survey report


1. Reviewed the status of office computing equipments - printer, computer, copier, fax
etc.
2. Noted down deficiency, drawbacks of the available equipments- in terms of printing
quality, speed, paper support and delivery systems, their look and convenience for
use.
3. Established the need for new system requirements for office effectiveness.
4. Surveyed the different models and brands available in the market that matches the
specifications.
5. Formed a clear comparative view of different brands.

Recommendations:
1. Replace the existing equipments with the new ones to enhance the operating
efficiency and effectiveness of each department.
2. Use the existing equipment in the training section for secondary use.
3. Accommodate new equipments.
4. Timely servicing and maintenance of the all the equipments.

Question No. 2
Define a proposal. In the light of the fact that a sound proposal largely depends on the
effective presentation of the content, pinpoint the major tenets that help you better
organize the content in the proposal along with a brief explanation for each point. (2+8=10)
Answer
A proposal is a persuasive presentation for consideration of something demanded or
offered. Very much like a sales message, it is also a selling document of a job to be done
at the satisfaction of an individual or work organization. It is, in order words, a bid in
writing, for sale of products or acceptance of services appealing for grants and donations
to do this.
A proposal being a selling document depends upon the quality of persuasiveness which
relies on the quality of presentation. A good proposal is written on AIDA (Attention,
Interest, Desire and Action) formula. Except in the case of a proposal seeking someone‘s
or some work organization‘s .voluntary support or cooperation, e.g. a student seeking

The Institute of Chartered Accountants of Nepal


86 of 109
Suggested Answers of Accounting
CAP II Examination – June 2009

support and financial assistance for carrying out his research work, the writer should be
fully familiar with the recipient‘s problems and service requirements. To this end, the
proposal writer should be able to convince the receiver that the former is able to
cooperate with him and solve his problems.
Proposals are of mainly two types – a. invited and b. uninvited. The same is also
categorized as a. research proposal and b. business proposal. A business proposal has
necessarily to be an invited proposal which needs to review what the proposal requires.
Some written invitations also give a suggested plan for the proposal writing. It is called
TOR (terms of reference).
.
a. The writer‘s purpose and the reader‘s need.
A well-begun proposal begins with a statement that states the writer‘s purpose and the
reader‘s requirements. In case of an invited proposal, the statement should also link up
with what has been said in the invitation. But in the case of an uninvited proposal, its
beginning has to have an additional part that must gain the reader‘s attention. An invited
proposal is very much like sales messages. Its intended readers are not likely to read
them. Therefore, the beginning must address the reader‘s reluctance to read. One
effective way is to summarize the major points of the proposals with an emphasis on the
benefits. Then our clear statement of the purpose and problems may be the most
important strength of the proposal.
b. A review background of information of the proposal also promotes an understanding of
the problem. It will enable the reader to get to know the past records and history of the
problem trying to establish your point that you have been working in the same line. At
the same time, the writer will have to point out the benefits. It will, in turn, help the
readers to have a better perspective of the proposal
© A detailed picture of work procedure: As a selling document, a business proposal gives
a complete picture of the work procedures followed during the project work. It includes
answers to Wh- questions – what? why? where? when? how?, etc. It contains the
following:

i. Background Information. No proposal begins the work procedure without giving


glimpse of the background information, which will help readers understand much about
what the project is about and about the need to initiate the project the project. Example
for this: Concern Nepal- a child exploitation-related non-governmental organization
invites proposals from interested and qualified management consultants/ researchers to
study the situation of rag-picking children in the Metropolitan City of Kathmandu.
The background information of the proposal will be thus:
Background: The increasing population of the abandoned and homeless street children
poses a serious problem of children at risk. With the exception of those who resort to
begging, many of them are seen struggling hard for survival. The historical background
will reflect on the need for undertaking a project on the proposed subject.
Description of the Plan: The heart of the proposal is the description of what the writer
wants to do. This should include the list of things and the manpower requirements
presented in a serial order what the contents include depends upon the type of project.

The Institute of Chartered Accountants of Nepal


87 of 109
Suggested Answers of Accounting
CAP II Examination – June 2009

Particulars: This refers to the particular details such as the work schedules to be done in
different phases, work schedules, time allocation for each phase of work, cost,
equipments and supplies needed.
Evidence of ability to deliver: The proposal writer should convince that the proposal is
able to undertake and complete the assigned task with the time stipulated within the
project period. To this end, it should also refer to its professional expertise and skills.
Benefits of the proposal: The proposal should articulate the benefits of the project- not
only for the proposal seeking organization, but also for the public at large. For example, a
proposal presented by a reputed management consulting firm to Plan International –
Nepal to conduct an investigation study on the birth registration situation in Far Western
Nepal will benefit not only the proposal seeking organization but also residents of the
backward rural communities.
Concluding remarks: The proposal should end with the concluding remarks. One way is
to present a summary review of the highlights. Another way is to offer additional
information that might be needed. Yet, another way would be to urge action on the
proposal.

Question No. 3
Describe briefly what employers look for during an employment interview. Explain the
three stages of a successful employment interview. (5+5=10)
Answer
An interview is a planned interaction between two or more people under a given situation
created for selecting the most capable and competent candidate for a job. An interview is
one of the most formal channels of communication which idealizes the interpersonal
relationship through the active interaction between the interviewer and the interviewee.
An employment interview is a crucial stage for an employer because he has to select the
most appropriate candidate out of a number of candidates to achieve his organizational
goal. Usually, the employer looks for two things during an employment interview. First,
the interviewers will try to know if the applicant is well-qualified for the position even
though they have already reviewed the applicant‘s resume while short-listing the job
applications. They want to see how well his skills match their organizational
requirements. To this end, some interviewers use pre-employment tests to help them
make a decision about the quality of the interviewee.
Second, the interviewers will try to know whether or not the selected applicant will be
compatible with the other people in the organization.
Stages of interviews: Most interviews have three stages: a. the warm-up stage b. the
question-answer stage and c. the concluding stage. The first is the most important stage
because the impression left by the interviewee in the mind of the interviewer determines
the lot of the interviewee.
Apart from this, the first stage involves some important considerations for the
interviewee. They include:

The Institute of Chartered Accountants of Nepal


88 of 109
Suggested Answers of Accounting
CAP II Examination – June 2009

a. Dress-up impression. This does not mean wearing highly fashionable and costly
clothes. The interviewing board expects to see the interviewee in simple but neat and
clean dresses.
b. Punctuality. An interviewee is advised to reach the office some 15-20 minutes before
the appointed time.
c. Avoiding chewing gum or paan (beetle leaf), smoking, taking coffee, etc. because this
seem unruly and uncultured.
d. Being mindful of the body language. The interviewee should be smart enough to
understand the non-verbal cues provided by the interviewer. They tell a lot about the
interviewer‘s expectations.
e. Moving with self-confidence. Every interviewee should think he can perform better in
the interview
g. Coming well-prepared with necessary documents. All the papers which could be
possibly asked for in the interview should be taken.
h. Controlling speaking. Should not speak more than what is expected and
i. Overcoming fears. Job interviews, in most case, are frightening and challenging.
Interviewees do not like to be judged, sometimes humiliated by odd questions posed by
the interviewers. But the fear can be overcome by knowing how it (the interview) works.
and how it should be dealt with.
The second stage, the most important and time-consuming, is the stage of activities for
both the interviewers and interviewee. Also recognized as the stage of discussion., it
occupies nearly 85 percent of the interview weight age Of the many things to be taken
into consideration, listening carefully for the interviewee and watching the interviewee‘s
non-verbal which helps the interviewer achieve his/her goals are crucial.
The third is the stage of performance evaluation. It is in this stage that the interviewer
would have assessed the worth of the candidate.
The last stage is the stage of complimentary close, in which the interviewer, consciously
or unconsciously leaves some clues about the satisfaction or dissatisfaction about the
interview. A very cautions and careful interviewee may discover it. The interview closes
with departure formalities, such as ‗Wish you had a successful interview! See you again!
Wish you a successful life!

Question No. 4

Define non-verbal communication. Discuss the ways to improve non-verbal and oral
communication in multicultural environments. (3+7=10)

Answer
Communication takes place in various ways for a variety of purposes. Usually,
communication takes verbally- orally and in writing. But this is not an end in itself,
because in certain circumstances, communication without the use of words and scripts
plays a paramount role and does what verbal communication cannot. Non-verbal
communication is the interpersonal process of sending and receiving information both

The Institute of Chartered Accountants of Nepal


89 of 109
Suggested Answers of Accounting
CAP II Examination – June 2009

intentionally and unintentionally, without using written or spoken language. The non-
verbal signals include facial expression, gesture and posture. Here, communication takes
place through use of body language, i.e. gestures, signs, symbols. This process of
communication is called non-verbal communication, because it does not use verbs of
action.
Reliability of non-verbal communication: Non-verbal communication is more reliable
than verbal communication. In simple words, vocal commitments, may tell a lie. Fake
words and sheer lies used by those involved in cheating, deceitful, fraudulent business
and criminal activities, do not at all correspond with their facial expressions and bodily
postures. Those who fail to understand the meaning of a deceitful smile are easily
exploited by the former.
Differences between Verbal and Non-Verbal Communication
To speak of some of the differences, words of verbal communication are cerebral (brainy)
Non-verbal communication are visceral (physical), i.e. we feel, but do not think about
what something means. Second, words are specific and self-edited. Contrarily, body
language is not edited, because posture, gestures and movements are unconscious.
Body language. Body language which constitutes the first content of non-verbal
communication deals with bodily parts and the roles they play in the process of
communication. Roles played by the movement of head, face, eyes, limb discussed in
body language. The meanings conveyed by them also culture dependent. Meanings
conveyed by gesture language are subject to territorial distances. As such, we have native
as well as foreign gestures. Lots of unhappy and tragic incidents occur because of the
differences in the meaning of gestures of two localities or two different countries.
Role of signs and symbols. Signs and symbols and signals also play a significant role in
non-verbal communications. Most people are familiar with the ―No: Smoking‖ ―Stop‖,
and ―Wait‖ ―Go‖ signals. Handicapped people and the dumb and deaf people can benefit
much from non-verbal cues.
Kinesics (body language), Pronemics, Haptics (touch) language, Paralanguage (vocal
characteristics), Pitch variations, , Change in the voice for effect, Speaking speed,
Silence, Pause, etc. are well-elaborated in the process of understanding Non-verbal
Communication.
In view of the fact that non-verbal communication is more reliable than verbal
communication, it is indispensable for an effective manager to understand non-verbal
communication. A manager, however qualified and competent, may be deceived by less
qualified and less competent staff, if he/or she is unfamiliar with the non-verbal
communication.

Question No. 5
Write short notes on ANY TWO (2×5=10)
a) Use of you-viewpoint language in business
Answer
In writing business message, the you-viewpoint language helps in the motion o
goodwill of the organization among its clients and customers. As the "you-viewpoint"

The Institute of Chartered Accountants of Nepal


90 of 109
Suggested Answers of Accounting
CAP II Examination – June 2009

language emphasizes the reader‘s interest and concerns rather than the writer‘s. This
technique is very effective to do motivate, influence and win favor of the people. I t
emphasizes you, your and yours and de-emphasizes we, our, and ours. The you-
viewpoint language is an attitude that places the reader in the center of things. It
involves being friendly and treating people the way they like to be treated. For
instance, the statement. "I am happy to report….‖ Place the writer at the center, were
―you will be happy to know…‖ places the reader at the center. The You-viewpoint
language, therefore, provides the reader a kind of psychological boost and confidence.
Apart from the above, we viewpoint different from you viewpoint. Sources self-
centered and speaker-focused. Writing in you viewpoint language is custom- focused.

b) Elements of good talking


Answer
Talking is an oral expression of knowledge, viewpoints, and emotions through
words. In other words, talking is an oral presentation of facts, ideas and knowledge.
Due to the lack of effective presentation, even a well-prepared report or paper will
fail to achieve its goals lags much behind the effectiveness of a good talk.
Furthermore, in the modern competitive business world, the one who can better sell
his/her words is the one who succeeds.
Business organizations also look for those employees who posses dynamism and
have good talking skills. The elements of tood talking are as follows:
i) Voice Quality. Voice quality refers to the vocal sounds one hears when another
speaks. It also includes the speed and volume of sound. Voice should be
pleasing and melodious, not harsh and high-pitched
ii) Style: Style refers to the parts of voice quality (pitch, speed and volume). That
is to say, style refers to a set of voice behaviors that give uniqueness to a
person‘s personality.
iii) Word choice: refers to the choice of appropriate words for language
effectiveness.

c) Pros and cons of emails

Answer
Email is one of the fastest means of communication today. The rapid growth of email has
been the most exciting business communication development in years. Email has
emerged as a mainstream form of communication in a short period of time. It has become
widely used in both small and large organizations. The advantages that email has over
other forms of communication are as follows:
 Email eliminates the problem of trying to conduct busy people who are not
always available for telephone calls.
 Email is timesaving as well as economical.
 Email can speed up the process of making business decisions, because it
permits rapid exchanges from all involved in the decisions.
 Email is cheap.

The Institute of Chartered Accountants of Nepal


91 of 109
Suggested Answers of Accounting
CAP II Examination – June 2009

 It provides a written record.


The disadvantages of emails are as follows:
 Email is not confidential. Email messages have to go through intermediate
computers before reaching their destination and it meansthat it is relatively
easy for others to intercept and read messages.
 Email doesn't communicate the sender's emotions well. Voice, intonations,
facial expressions, and body movements are not a part of email. Due to the
lack of such opportunities, communication through emails may not be as
much effective as the face to face communication.
 Email may be ignored or delayed.

SECTION 'B'

Question No. 6
Tap water is Coca-Cola‘s biggest competitor. Pepsi Co. and Coke have recognized
bottled water as a threat to their growth in the beverage industry. As consumers have
become more health conscious, the soft drink industry has lost ground to bottled water.
The rising standard of living worldwide allows ―people to pay a premium for processed
tap water‖.

In 1999, the sale of bottled water in the U.S. was up 35% vs. 0.5% for soft drinks. Pepsi
Co‘s Aquafina bottled water is ranked as the fifth largest U.S. bottled water brand.
Coke‘s Dasani bottled water is the eighth U.S. bottled water brand.

Thus far, bottled water is a small part of Pepsi Co. or Cokes‘ revenues. Pepsi Co
differentiates Aquafina as purified water essential to the body. Coke differentiates Dasani
by adding minerals. For both companies their bottled water products are key elements in
their product offering.

Unlike the soft drink market where the two companies dominate the industry, the bottled
water market has well established market leaders with global recognition. ―Danone,
which owns Evian, and Nestle, which sells Perrier‖ are global giants and Poland Springs
is the U.S. bottled water leader. The advantage that Pepsi Co. and Coke have is ―their
extensive distribution network‖. This allows them to place their product.
Questions: (2×5=10)
a) Given the worldwide recognition of Coke and Pepsi, why do you believe these
companies did not apply these brand names to their bottled waters?
b) Why would companies like Coke and Pepsi Co. add a product line that potentially
reduces sales of their existing product lines?

Answer
a)
Customer
While developing bottled water, Coke and PepsiCo might have directly interacted with
their customers to know about the features that they would prefer in order to fulfill their

The Institute of Chartered Accountants of Nepal


92 of 109
Suggested Answers of Accounting
CAP II Examination – June 2009

needs. It might have been due to the fact that majority of the customers suggested them to
do what they ultimately did.

Image
If the new product-with the borrowed brand name-doesn't resonate in the marketplace,
the core or parent brand could be irreparably damaged.

Market Trends
PepsiCo and Coke enhance carbonated drinks in the mind of the people. As consumers
have become more health conscious, the soft drinks companies have lost ground to
bottled water. If PepsiCo and Coke apply these brand names to their bottled waters,
consumers might have the same impact of Coke. In order to differentiate water from
carbonated drinks, the companies might have used the brands "Aquafina" and "Dasani".

b) Customer preferences and environmental forces necessitate new product


development. Customers' preferences for soft drinks have changed. As carbonated drinks
loose their fizz, bottled water sales have flown due to the rising standard of living of the
people. In 1999. the sale of the bottled water in U.S was up 35% versus 0.5% for soft
drinks. Competitors like Poland Springs, Danone and Nestle are snatching the
opportunities. In order to counter these challenges, PepsiCo and Coke have followed
product line strategies by adding bottled water in their product lie with new brand names.
The advantage of Coke and PepsiCo have is their extensive distribution network. This
allows them to place their products easily. Now the result is booming. Although bottled
water is only a small part of PepsiCo or Coke revenues, PepsiCo's Aquafina is ranked as
the 5th largest bottled water brand. Coke Dasani bottled water is the 8th largest bottled
water brand. This, Coke's and PepsiCo's added product line would not potentially
cannibalize their existing product lines. They are a part of their companies' product lines
strategies only.

Question No. 7 'a'


Point out the macro (external) environment of marketing. 5

Answer
Forces in the external environment are non-controllable by management and marketing.
However, they can influence them and be influenced by them. Macro environment is
located outside the organization. It consists of political-legal, economic, socio-cultural
and technological (PEST) forces
i. Political and legal forces:
At now, Nepalese market is highly affected by political and legal forces. They strongly
affect marketing decisions. Government policies, government agencies and laws are the
political and legal forces affecting marketing environment.

ii. Economic forces


Economic environment plays a significant role in their marketing system. High economic
growth assures higher level of employment and income. High purchasing power and
willingness to spend lead to marketing boom. Natural resources, income distribution,

The Institute of Chartered Accountants of Nepal


93 of 109
Suggested Answers of Accounting
CAP II Examination – June 2009

economic health, inflation, fiscal policies, competition, regional economic groups are the
economic forces of marketing environment.

iii. Socio-cultural forces


The socio-cultural environment is made up of institutions and other forces that affect a
society‘s basic values, perceptions, preferences and behaviors. Demography, life style,
social forces, cultural forces the marketing environment.

iv. Technological forces:


Modern marketing has been shaped by technology. No technologies offer a main source
of economic growth. Level of technology, pace of technological change, research and
development budget are the technological forces of macro environment.

Question No. 7 'b'


Explain the requirements for effective market segmentation. 5

Answer
Market segmentation is a philosophy of directing products and services at specific target
group. Segmentation or subdivision of the market is based upon the modern marketing
concept. Obviously, it is a customer oriented philosophy. The requirements of effective
market segmentation are measurable, divisible, accessible, sustained and actionable.

i. Measurable
Customer size, customer needs, customer purchasing power and their characteristic
should be measurable for effective market segmentation, otherwise segmentation is
impossible.

ii. Divisible
The segments are conceptually distinguishable and respond differently to different
marketing mix and programs. In other words there must be same basis for dividing
homogeneous group.

iii. Substantial
Segment should be large and profitable enough to source. Segmentation should contribute
to objective achievement of organization.

iv. Accessible
Segmentation has no value if firm cannot find middleman to distribute the product and
services in target market or particular segment.

v. Actionable
The firm should be able to formulate and implement marketing program to serve the
segment.

The Institute of Chartered Accountants of Nepal


94 of 109
Suggested Answers of Accounting
CAP II Examination – June 2009

Question No. 8 'a'


What are the components of marketing information system? 5

Answer
a) Marketing information system is a continuous process that provides a constant flow
of information about such things as prices, advertising, expenditures, sales and
distribution expenses. It provides framework and distribution expenses. It provides
framework for the day to day managing and structuring of information regularly
gathered from sources both inside and outside the firm.
The components of market information system are:
i) Internal reports
Internal report system is a sub system of marketing information system. It is also
called Internal accounting system. It is also called Internal accounting system.
Internal marketing information is secured through accounting system. Data on sales,
inventories, costs, cash folds, account receivables and account payables etc are
generated within the firm.

ii) Marketing Intelligence system


Marketing intelligence system is an organized procedure to collect regularly from
diverse source of the information of potential usefulness for the making of marketing
decisions.

iii) Decisions support system


Decision support system (DSS)is that by means of which marketing data and
marketing issues can be analyzed by logical and questionnaire techniques.

iv) Marketing Research


Marketing research is the objective and systematic collection, recording analyses,
interpretation and reporting of information about existing or potential markets,
marketing strategies and tactics which enables management to assist decision making.

Question No. 8 'a'


Describe the stages of consumer buying process. 5

Answer
Consumer behavior is the process where by individual decide what, when , where, how
and from who to purchase goods and services. Consumer buying process is the process of
decision making to purchase function. It is composed of number of stages:
i) Need recognition:
The first stage of the buying process in which a consumer recognize the need.. A need
can be activated through internal and external stimuli.
Information search:
After need recognition, the consumer tries to solve it and gathers the sources and
information about the products. The consumer can obtain information from any several
sources. These include internal sources and external sources.

The Institute of Chartered Accountants of Nepal


95 of 109
Suggested Answers of Accounting
CAP II Examination – June 2009

ii) Evaluation of alternatives:


After searching the information consumer identifies the different alternative and
evaluates them. The evaluation stage represents the stage of mental trial of the product.

iii) Purchase decision


Purchase is a consumer commitment for a product. It is the terminal stage in the buying
decision that completes a transition. A consumer makes the purchase decision after
evaluating different alternatives.

iv) Post purchase behavior:


Post purchase behavior is the stage of the buyer decision process in which consumers
take further action of purchase based on the satisfaction or dissatisfaction.

Question No. 9 'a'


What is packaging? Explain its objectives. (2+3=5)

Answer:
a) Packaging
Packaging means the subdivision of packing function. Packing means wrapping of gods
or products before they are stored or transported or delivered to a consumer. Packaging
involves ass the activities of designing and producing a container or a wrapper of the
product. Their wrapper or container is called a package packaging provides information
about the quality of the product and methods to use to the buyers. Packaging establishes
public image for the branded products and helps in distinguishing the products of a
manufacture from that of another manufacturer. The utility of packaging is protection,
identification and convenience of selling.
According to Philip Kotler, "Packaging includes all activities of designing and producing
the container or wrapper for a product."
Objectives of Packaging:
Packaging has the following objectives:

i) Protection-
It aims to protect the product from damage. It preserves the quality of the product.
Damages to the product can occur from climatic conditions, mishandling, spilling and
evaporation. Perishable products need sound packaging to prolong their lives.

ii) Storage
Packaging facilitatesd provide storage of products till they are used. It fulfills the storage
needs of the customers as well as middlemen's.

iii) Information
Packaging provides avariety of information about the product. It identifies the product
and its manufacturers. It informs about product ingredients, directions for use, price,
quality, date of manufacturer and expiry and safety precautions for use.

The Institute of Chartered Accountants of Nepal


96 of 109
Suggested Answers of Accounting
CAP II Examination – June 2009

iv) Positioning
Packaging is used for positioning of products. Differentiated packaged facilitate product
differentiation. Consumers perceive product differences based on packaging.

v) Promotion
The package can act as a colorful salesman or effective advertisement at the point of sales
which make the buyer impulsive to buy the product. This facilitates self service
marketing.

Question No. 9 'b'


What is relationship marketing? How can relationship be retained with the customers?
Explain. (2+3=5)

Answer
Relationship marketing is building long term mutually satisfying relations with customers
in order to earn and retain their long term loyalty. The customer is regarded as partner in
creating value. All steps to know customers and to deliver high customer value and
satisfaction are taken.
According to Philip Kotler, "Relationship Marketing is the process of creating,
maintaining and enhancing strong value-laden relationship with high customers
andstakeholders." In other word, it is a long term partnership between marketer and
customer. It aims at customers retentions.
A marketer can retain the relationship with customer in the following ways:
i) Quality Assurance:
Marketers should promise and deliver high quality products at fair prices to customers.
Quality consistence and conformance standards should be ensured.

ii) Economic Benefits:


Strong economic ties are built with customers like:
1) Frequency Marketing Program
Key customers who buy frequently are given attractive discounts and rewards.
2) Club Membership Programs
Club membership is given to customers. Attractive discounts and other benefits are
given.

iii) Social Benefits:


1) Organizing customer get-togethers to meet and enjoy each other.
2) Individualizing and personalizing customer relationship. This provides social
recognition to the customer.

iv) Technical Benefits:


Organizations develop technical ties with their key customers. This help the marketers to
better manage their marketing efforts through:
1) Electronic Data Interchange.
2) Supplying Computer linkages to customer including software programs.

The Institute of Chartered Accountants of Nepal


97 of 109
Suggested Answers of Accounting
CAP II Examination – June 2009

Question No. 10
Briefly explain ANY FIVE of the following:
(5×2=10)
a) Industrial and Consumer market
Answer
Industrial market is the reasons for buying products are business use, resell or to make
other products. They consist of industries, business, resellers etc.
Consumer market is the reasons for buying products are own personal or household use.
They consist of ultimate customers.

b) Advertising
Answer
Advertising as a means of mass communication has mass selling possible. It is the best
known mass communication channel. Marketers and services throughout the country and
or in other nations are fully aware of necessity and importance of advertising promotes
the sale of goods, services, images and ideas through information and persuasion.

c) Retailer
Answer
Retailers operate on a small scale and in the local markets, selling directly to the
consumers a wide, variety of goods to satisfy numerous and changing wants of
consumers.

d) E-commerce
Answer
Traditional multinationals will in time be killed by e-commerce in this hi-tech. The e-
commerce delivery of goods, of services, of repairs , spare parts and maintenance, will
require a different mindset, a different top management and, in the end , different
definition of performance is measured will change. E-commerce is the way of doing
business through the help of computer network.

e) Pricing Policies
Answer
Pricing policies guide marketing decisions. They can be:
1. Single price policy consisting of a list price.
2. Flexible price policy consisting of price discrimination, discount, allowances.
3. Geographic price policy consisting of free on board, zone, base point, uniform
delivery prices.
4. Product mix policy based on product line, features, bundling, ancillary product.

f) Three level channel structure for consumer goods


Answer
Three levels of channel structure for consume goods consists of:
Producer—Sole Agent—Wholesaler—Retailer—Consumer
It is the longest channel and is costly. Global companies use this channel.

The Institute of Chartered Accountants of Nepal


98 of 109
The Institute of Chartered Accountants of Nepal
Suggested Answers of Income Tax & VAT

CAP II Examination – June 2009

Marks
All questions are compulsory. Working notes should form part of the answer.
Make suitable assumptions wherever necessary.
Question No. 1'a'
What do you mean by assessable income? Describe the provisions of the Income Tax
Act, 2058 in this regard. 5
Answer
As per section 6 of the Income Tax Act, the following incomes earned by a person
through any business, employment or investment in an income year shall be regarded as
assessable income, subject to this Act:
i) Income earned by a resident person in that year through business, employment or
investment, irrespective of where its source is located, and
ii) Income earned by a non-resident person in that year through business, employment or
investment having its source in Nepal.
Provided that no income which is exempt from tax under Section 11 or 64 or both
sections shall be included in the assessable income.

Question No. 1'b'


M/s Mechi Cigarette Industries Ltd. (producer of cigarettes) furnished the following
particulars to you pertaining to the income year 2064-2065.
 Opening balance (WDV) of depreciable assets as on 2064-4-1.
Particulars NNRs.
Building 10,00,000
Car 6,00,000
Computers 70,000
Office Equipment 1,20,000
Plant & Machinery 8,00,000
Tools 30,000
Repair & Improvement cost capitalized ( block D) 10,000
 The company has purchased a plant & machinery as on 2065-3-15 for NRs.6,00,000.
The company has also purchased a mini bus as on 2064-6-25 for NRs.3,50,000.
 During the year one old computer having written down value of NRs. 12,500 is sold
for NRs. 7,500. One printer having written down value of NRs. 3,500 became
unusable and the company recovered nothing from it.
 Repair & improvement expenses of the company during the year are:
Particulars NNRs.
Building 90,000
Office Equipment 10,000
Suggested Answers of Accounting
CAP II Examination – June 2009

Car 75,000
Plant & Machinery 80,000
 During the year the company has incurred NRs. 5,00,000 on research and
development. However, only NRs. 3,75,000 is allowable deduction for research and
development cost for the income year 2064-2065.

Required:
i) Classify the assets as per schedule 2 of Income Tax Act, 2058. 5
ii) Amount of depreciation for the income year 2064-65 as per schedule 2 of income Tax
Act 2058. 5
iii) Amount of opening depreciable basis for the Income Year 2065-66. 5

Answer
i) Classification of Assets as per Schedule 2 of Income Tax Act, 2058
Block A Block B Block C Block D
Computer, Off Plant & Mach
Particulars Building Car, Minibus
Equipment Tools
Depreciation Rate 5% 25% 20% 15%
Opening Balance 10,00,000 1,90,000 6,00,000 8,30,000
Repair & Improvement cost - - - 10,000
Add: Absorbed addition (W. N.1) - - 3,50,000 2,00,000
Less: Disposal - (7,500) - -
Depreciation base at the end of the year 10,00,000 1,82,500 9,50,000 10,40,000
Less: Depreciation 50,000 45,625 1,90,000 1,56,000
Written Down Value 9,50,000 1,36,875 7,60,000 8,84,000
Add: Repair & Improv. cost (W. N.2) 20,000 - 8,500 7,200
Add: R & D cost (W. N.3) - - - 1,25,000
Add: Unabsorbed addition (W. N.1) - - - 4,00,000
Opening Dep base for the IY 065/066 9,70,000 1,36,875 7,68,500 14,16,200

Working Notes:

W.N. 1 Absorbed & Unabsorbed additions


Particulars Date of Purchase Block C Block D
Plant & Machinery 2065/3/15 - 6,00,000
Mini Bus 2064/6/25 3,50,000 -
Total additions - 3,50,000 6,00,000
Absorbed portion of additions - 3/3 1/3
Amount of absorbed addition - 3,50,000 2,00,000
Amount of unabsorbed addition - - 4,00,000

The Institute of Chartered Accountants of Nepal


100 of 109
Suggested Answers of Accounting
CAP II Examination – June 2009

W.N. 2 Repair and Improvement cost


Block of (b) Allowed Limit (c) Actual Unabsorbed
Dep base (a)
Assets 7% of (a) Expenses Portion (c-b)
A 10,00,000 70,000 90,000 20,000
B 1,82,500 12,775 10,000 -
C 9,50,000 66,500 75,000 8,500
D 10,40,000 72,800 80,000 7,200
W.N. 3 Research and development cost
Particulars NRs.
Actual expenses of Research & Development 5,00,000
Less: Allowable deduction 3,75,000
Balance to be capitalized under Block D 1,25,000

Question No. 2
a) Indicate the correct answer from the alternative. 10

i.) TDS return should be submitted to the department under Sec.90(1) of the Act from the
end of the every month within:
(a) 7 days (b) 15 days (c) 30 days

Answer
(b) 15 days

ii) Tax deducted should be deposited to the account of the government from the end of
every month within:
(a) 7 days (b) 15 days (c) 30 days

Answer
(b) 15 days

iii) Interest on deficiency of installments of advance tax payable against installments paid
will be calculated:
(a) daily basis, (b) month to month basis, (c) per annum basis

Answer
(b) month to month basis

iv) Interest chargeable on deficiency of installments of advance tax payable against


installments paid will be at the rate of:
(a) 5%, (b) 10%, (c) 15%

Answer

The Institute of Chartered Accountants of Nepal


101 of 109
Suggested Answers of Accounting
CAP II Examination – June 2009

(b) 10%

v) Income tax return under Sec. 99 is to be filed for the year 2065/66 before:
(a) 31st Ashwin 2066 (b) 30th Paush 2066, (c) 31st Chaitra 2066

Answer
(a) 31st Ashwin 2066

vi) Estimate of tax to be submitted by a person liable to pay tax in installment is to be


submitted every year:
a) Up to end of Paush, (b) 15th Paush, (c) 31st Chaitra

Answer
a) Up to end of Paush

vii) The certificate of tax deducted at source should be provided by the tax deductor to the
deductee within 15 days from:
(a) Date of deduction, (b) The end of the month in which deducted, (c) From the end
of the financial year.

Answer
(b) The end of the month in which deducted

viii) Tax to be deducted from payment to a person carrying on business of letting out
buildings on rent:
a) 10%, (b) 15%, (c) 0%

Answer
a) 10%,

ix) Tax to be deducted from the bill of service provider registered under VAT against
VAT bill:
a) 5%, (b) 15%, (c) 1.5%

Answer
(c) 1.5%

x) If the total tax to be paid for the year is less than that, no installment tax need be paid:
a) NRs.10,000 (b)NRs.5,000 (c)NRs.2,000

Answer
(c)NRs.2,000

The Institute of Chartered Accountants of Nepal


102 of 109
Suggested Answers of Accounting
CAP II Examination – June 2009

Question No. 2 'b'


State the entities other than those registered under the Companies Act that are treated as
companies under the Income Tax Act. 5

Answer
U/s 2 (ana) of the Income Tax Act 2058; "Company" means a company established
according to the Company Law in force and for the purpose of tax, the following bodies
also shall be treated like companies. :
a) a corporate body established under the law in force,
b) any unincorporated association, committee, institution, society or group of persons
other than a partnership or a proprietary firm (whether or not registered) or a trust,
c) a partnership firm whether registered or not registered under the law in force, having
20 or more partners, retirement fund, co-operative society, unit trust or a joint
venture.
d) Foreign Company
e) Any other foreign institution notified by the Director General.

Question No. 3 'a'


What are the various actions that can be resorted by the tax office, if a tax payer does not
pay the tax in time as prescribed in the Income Tax Act, 2058? 5

Answer
If the tax payer does not pay the tax in time, then the tax authorities can resort to the
following actions:
a) Under Section 104 of the Income Tax Act 2058, if the tax payer does not pay the tax
within the stipulated time, then Government of Nepal (GON) will have a lien and
claim over the property of the tax payer, and collect the amount by auctioning such
property under Sec. 105.

b) The Department can stop the person from going out of the country by informing the
concerned department U/s 106.

c) The concerned officers of any entity who is in default may be made personally liable
for the tax payable by the entity U/s 107.

d) If any amount is due from any person to the person who is in default of payment of
tax, if any person holds any assets on behalf of the person in default then that person
may be asked by the Department to pay the amount or handover the asset direct to the
department instead of to the tax payer to the extent of the tax due U/s 108 and 109.

e) The tax payer in default may be fined from NRs. 5,000 up to NRs. 30,000 or
imprisoned from one month to six months, and both U/s 123.

Question No. 3 'b'


What are the incomes exempted from tax under the Income Tax Act 2058?

The Institute of Chartered Accountants of Nepal


103 of 109
Suggested Answers of Accounting
CAP II Examination – June 2009

Answer
As per section 11 of the Income Tax Act, 2058, the following incomes are exempted
from tax:
a) Income from an agriculture business: An agricultural income derived from sources in
Nepal during an income-year by a person, other than the income from an agriculture
business derived by a registered firm, or company, or partnership, or a corporate
body, or through the land above the land holding ceiling as prescribed in the Land
Act, 2021, is exempt from income tax.
b) For this section and purpose, the agriculture business means the business of
producing crops from public or private land, or deriving rent from a tenant using land.
c) Similarly, incomes derived by cooperative societies through following activities are
also exempted from tax:
Incomes derived by cooperative societies and community based saving and credit
cooperatives and cooperative associations registered under Cooperative Act, 2048
(1991), from business based on agriculture and forest products such as sericulture and
silk production, horticulture and fruit processing, animal husbandry, diary industries,
poultry farming, fishery, tea gardening and processing, coffee farming and
processing, horticulture and herb processing, vegetable seeds farming, bee-keeping,
honey production, rubber farming, floriculture and production and forestry related
business such as lease-hold forestry, agro-forestry, cold storage established for the
storage of vegetables and business of agricultural seeds, insecticide, fertilizer and
agricultural tools (other than machine operated).
d) For the purpose, rural community means those rural areas other than metro, sub metro
and municipalities and Village Development Committees attached with such urban
area; where the lacks basic infrastructure.
e) Dividends distributed by Cooperatives: Dividends distributed by above mentioned
cooperatives on income earned from above said activities.

Question No. 4
What are the exempt amounts? Describe the provisions of the Income Tax Act in this
regard. 10

Answer
As per section 10 of the Income Tax Act, following are the exempt amounts:
a) Tax exempt amounts received by a person enjoying tax exemption facilities as
provided for in a bilateral or multilateral treaty signed between GON and a foreign
country or an international organization or institution.
b) Amounts received by a natural person in consideration of his employment in the
government service of a foreign country.
Provided that,
i) The person must have become a resident or non-resident person simply because of
his employment, and
ii) Such amounts must have been paid from the state funds of that country.
c) Amounts received by a non-Nepali natural person mentioned in Clause (b), or by his
immediate family member, from the state funds of a foreign country.

The Institute of Chartered Accountants of Nepal


104 of 109
Suggested Answers of Accounting
CAP II Examination – June 2009

d) Amounts received by a non-Nepali citizen appointed in the service of GON on the


condition that he need not pay tax.
e) Allowances paid by GON to widows, elders, and disabled people.
f) Amounts received as gifts or under wills, or as inheritance from heirless persons, or
scholarships, other than those that are to be included in calculating income under
Section 7, 8, or 9.
g) Amounts received for following considerations by an exempt organization:
i) Donations and gifts,
ii) Other contributions directly related to the functions of an exempt organization
under Clause (s) of Section 2 which are made to such organization with or without
expecting any returns,
iii) Amounts earned by the Nepal Rastra Bank according to its objectives,
h) Pension received by a Nepali citizen retired from the army or police service of a
foreign country provided the amounts are payable from the public fund of that
country.
i) Any type of income of Government of Nepal

Question No. 5 'a'


Mr. Deshmukh, an importer imported certain goods at NRs. 50,000. No VAT was paid on
its import. The goods passed to the final consumer through a retailer. Both middlemen
incurred NRs. 1,000 each for administration expenses. Both middlemen charged 15%
profit margin on selling price.

Required:
i) Cost price to the final consumer 5
ii) VAT payable to the government at each stage 3

Answer:
i. Cost price to final consumer = NRs. 81,094.45

ii. VAT charged at Importer‘s level NRs. 7,800.00


VAT charged at Retailer‘s level NRs. 1,529.45
Total VAT payable by consumer NRs. 9,329.45

Working notes:
1. Importer to retailer:
Let the selling price be NRs. x (importer)
Selling price (excluding VAT) = cost price + added value (i.e., cost +profit)
Or, x=NRs.50,000 + {NRs. 1000 + (15/100 X x)
Or, x = 50,000+1000+0.15x
Or, x -0.15x = 51,000
Or, 0.85x = 51,000
Or, x = 51,000/0.85 = NRs. 60,000
Selling price of importer = NRs. x = NRs. 60,000
Added value = S.P.-C.P = NRs. 60,000 – NRs. 50,000
= NRs. 10,000 Or

The Institute of Chartered Accountants of Nepal


105 of 109
Suggested Answers of Accounting
CAP II Examination – June 2009

(1000 +15% X 60,000) = NRs. 10,000

2. Retailer to consumer:
Now, let the selling price of retailer be y.
Selling price (excluding VAT) = C.P. (excluding VAT) + Added Value
Or, y = NRs. 60,000 + (1,000 +15/100 X y)
Or, y = NRs. 60,000 + 1,000 +0.15y
Or, y-0.15y = NRs. 61,000
Y = 61,000/0.15 = NRs. 71,765 (Approx)
Selling price of retailer = y = NRs. 71,765
Value added by retailer = Administration expenses + profit
= NRs. 1,000 + (15% X 71,765)
= NRs. 1,000 + NRs. 10765 = NRs. 11,765
Or value added = S.P.-C.P
NRs. 71,765-61,000 = 11,765

3. Computation of VAT:
Cost price selling price Selling price VAT payable to
Added VAT
Activities/Channel excluding excluding including Government in
value 13%
VAT VAT VAT each level
Imported goods 50,000.00 - - - - -
Importer to retailer 50,000.00 10,000.00 60,000.00 7,800.00 67,800.00 7,800.00
Retailer to consumer 60,000.00 11,765.00 71,765.00 9,329.45 81,094.45 1,529.45

Question No. 5 'b'


Mr. Shrestha, a small business proprietor not required to be registered to VAT, got
voluntarily registered on 1.4.2063. After two years, he now feels that he should not have
gone for registration as it is creating unnecessary burden on his part.
He doesn't know whether he can go for cancellation or not and the procedure he is
required to follow. He approaches you for the advice. Suggest him. 7

Answer
Under Section 11 (1ka) of VAT Act 2052, small enterprises that has been voluntarily
registered to VAT, can not cancel the registration before the expiry of one year from the
date of such registration. However, Mr. Shrestha‘s Business was registered two years
before; he can apply for cancellation if he wishes to do so.

Under Rule 12 of VAT Rule 2053, for cancellation of registration, he has to submit an
application to the concerned tax office, setting out the condition for cancellation of
registration. This has to be done within 30 days of the date on which the condition for
cancellation of registration occurred or the tax officer gets satisfied that the registration of
a registered person is to be cancelled as per the provision of Act.

In this case, Mr. Shrestha can submit the application for cancellation at anytime after one
year from the date of registration and he should also submit the tax return and VAT
amount if any payable up to the period of return submitted.

The Institute of Chartered Accountants of Nepal


106 of 109
Suggested Answers of Accounting
CAP II Examination – June 2009

The remaining stock and capital goods on which tax credit has been availed earlier shall
deemed to be supplied at the market rate and VAT on the same shall also be collected by
the department. After submission of the required documents and VAT amount, a notice
of cancellation will be given by the officer to the concerned registered person.

Question No. 6
Answer the following:
a) Healthy Bottlers Pvt. Ltd. is a manufacturer of glass bottle packaged beverages. The
Company supplies packaged bottles of beverage, billing the selling price of the
beverage. In addition to that, it gets certain amount from the dealers as deposit for the
glass bottles supplied to them. When the dealer returns the empty bottles, the amount
of deposit is refunded. A tax officer, during his inspection of the company, found that
bottles worth NRs. 200,000 were not in physical stock. He made tax assessment under
section 20 of the VAT Act and treated the shortage of bottles as sold by the company.
The company had proved that the bottles were in stock with various dealers in due
course of return. However, the tax officer did not accept the contention and charged
tax and penalty on the deemed sales of the bottles. Critically examine the contention
of the tax officer. 5

Answer
Healthy Bottlers Pvt. Ltd. is selling beverages only. When beverage is consumed by
consumer, the empty bottle is returned to the producer. Thus, the supply of empty
bottles on deposit does not constitute transaction as per VAT Act.

Section 2 (g) of the Act, defined the term supply which means act of selling,
exchanging and delivering any goods or services, or the act of granting permission
thereto or of contract thereof for a consideration. In this case the supply of bottle is
not for consideration and so it is not treated as supply for transaction purpose.

It was held in Pepsi Cola Nepal Pvt. Ltd. vs. VAT Department, the Revenue Tribunal
has interpreted that the bottles would be treated as sold in case of breakage or lost by
the dealer and when the producer charging for such breakage or shortage to the
dealer. The Tribunal has pointed out Section 12 (8) of the Act, according to which
deposit amount could be treated as sales when it is adjusted against consideration
receivable for supply of goods or services.

That is why; the shortage of bottles at factory premises cannot be treated as sold in
case it is proved that the bottles ate lying with the dealers.

b) Mr. Shyam, a businessman, submits his VAT return for the month of Falgun
according to which total sales for the month was NRs. 1,000,000 and purchase was
NRs. 600,000. Out of the total sales, NRs. 650,000 was export to Tibet (Non L/C). He

The Institute of Chartered Accountants of Nepal


107 of 109
Suggested Answers of Accounting
CAP II Examination – June 2009

had no previous debit/credit balance in his VAT return. He claims the refund of the
excess tax paid on purchase on 25th of Chaitra.
State the time and amount of claim he is entitled to. Also mention the documents (if
any) he requires to claim the refund. 5

Answer
In the given situation, out of total sales of NRs. 1,000,000, NRs. 650,000 was
exported to Tibet, which is taxable at the rate of 0%. So total taxable sales for the
month are NRs. 3.5 lacs (NRs. 10 lacs – NRs. 6.5 lacs) and taxable purchase is NRs.
6.5 lacs.
Particulars NRs.
VAT payable of sales (350,000 x 13%) 45,500
VAT claimed on purchase (600,000 x 13%) 78,000
Excess/ (deficit) 32,500

Thus, excess refund claimable is NRs. 32,500. Now, u/s 24 (4) of the VAT Act
2052,where the total sales of the registered person includes export sales of 50% or
more then they are entitled to get refund of tax immediately. Hence, in the given case
since the export sale of Mr. Shyam is 65% (650,000 / 1,000,000) of the total sales
volume, he is entitled to claim a refund of NRs. 32,500 on 25th of Chaitra onwards.
For the purpose of claim, he has to submit the following documents to claim the
refund, in case of export to Tibet.
i) Copy of agreement between buyer & seller.
ii) Copy of Pragyapan Patra of export.
iii) Copy of Pragyapan Patra of import.
iv) Copy of Payment Certificate.

c) Explain 'no-tax credit' and 'partial tax credit' under the VAT Act. 5
Answer
No-tax credit
Where VAT paid or payable on expenses or purchases is not allowed for tax credit,
then it is called no-tax credit. Normally, tax paid on expenses or purchases used for
production of tax exempted goods are not allowed for tax credit. Further, Rule 41(1)
of the VAT Rule 2053 has specifically set a provision of no-tax credit on the
following items:
i) Beverages
ii) Alcohol or alcohol mixed beverages
iii) Light petroleum/fuel for vehicles, and
iv) Entertainment expenses

Partial tax credit


Where tax credit for VAT paid on expenses or purchases are allowed for credit
partially but not fully are called partial tax credit. According to Rule 41 (2) tax
charged on automobiles purchased as fixed asset is allowed for credit 40% of the total
tax charged. Here automobile means any motor vehicle with three or more wheels
used for carriage of passengers.

The Institute of Chartered Accountants of Nepal


108 of 109
Suggested Answers of Accounting
CAP II Examination – June 2009

Question No. 7 'a'


List occasions when purchase and sales books are required to be certified by the VAT
authorities. 3

Answer
The following are the various conditions when purchase and sales book should be
certified.

1) When applied by the taxpayer for certification of purchase and sales book. As per
section 16 (3) all tax payers are required to keep purchase and sales book certified by
Tax Officer to maintain accounts.
2) In the period of tax audit or check.
3) At the time of inspection.

Question No. 7 'b'


How the place of supply of goods and services is determined under the VAT laws? 7

Answer
As per Rule 15 and 16 of VAT Regulations, place of supply of goods and services is
determined as per below:
1) Incase of movable goods, where the goods are sold or handed over.
2) Incase of immovable object, which can not be moved to other place, where the object
is placed.
3) Incase of imported goods, custom point where the goods is imported in Nepal.
4) Incase of self use by producer or seller, place of producer or seller.
5) Incase of service, where the benefit of service is received.

The Institute of Chartered Accountants of Nepal


109 of 109
The Institute of Chartered Accountants of Nepal
Suggested Answers of Advanced Accounting

CAP II Examination June 2010

1. The following list of account balances relates to Virat as at 31st Ashadh, 2067:

Rs'000 Rs'000
Sales revenue 473,300
Purchases 310,500
Operating expenses 18,400
Loan stock interest 5,000
Dividends paid 15,500
Leasehold building at cost (note (ii)) 200,000
Plant and equipment at cost (note (ii)) 124,800
Deferred development expenditure (note (iii)) 75,000
Joint venture (note (iv)) 62,000
Depreciation at 1st Shrawan, 2066 – leasehold 56,000
– plant and equipment 48,800
– development expenditure 15,000
Trade receivables 49,200
st
Inventory – 1 Shrawan, 2066 27,500
Bank 12,100
Trade payables 82,200
Ordinary shares of Rs 100 each 100,000
10% Convertible Loan stock – issued 2065 100,000
Deferred tax at 1st Shrawan, 2066 (note (v)) 11,400
Profit and loss reserve at 1st Shrawan, 2066 13,300
9,00,000 9,00,000

The following notes are relevant:

(i) The cost of the inventory at 31st Ashadh, 2067 was Rs.37.7 million (excluding joint venture
inventory – see note (iv)).

(ii) Non-current assets:


On 1st Shrawan, 2066, Virat‘s leasehold building was revalued at Rs.270 million by an
independent surveyor. The lease was for a 25-year period when Virat acquired it. The directors
wish to incorporate the revalued amount in Virat‘s financial statements. The revaluation reserve
will be deemed to be realised in line with the remaining life of the lease.

Plant is depreciated at 20% per annum on the reducing balance basis.

All depreciation is charged to cost of sales.

(iii) The deferred development expenditure relates to a new product. The project was successfully
completed on 1st Shrawan, 2065 and sales of the new product commenced on that date. The
development costs are being depreciated on a straight-line basis over the expected product life of
five years. Early in the current year, a review of the sales figures for the new product showed that
they were disappointing. In view of this, Virat has estimated that the present value of the expected
net future cash flows from sales of the new product is Rs.30 million; however, Virat has been
approached by a rival company with an offer of Rs.40 million for the rights to the product. At this
stage, Virat intends to continue to market and sell the product.
Suggested Answers of Advanced Accounting
CAP II Examination – June 2010

(iv) On 1st Shrawan, 2066, Virat entered into a joint venture with two other companies. Each venturer
contributes its own assets and pays its own expenses. The agreement stipulates that the joint
venture will be terminated on 31st Ashadh, 2070. Virat is entitled to 30% of the joint venture‘s total
revenues. The joint venture is not a separate entity. Details of Virat‘s joint venture transactions are:
Rs'000
Plant and equipment at cost 70,000
Share of joint venture sales revenues (30% of total sales revenues) (18,000)
Related cost of sales excluding depreciation 8,000
Inventory 31st Ashadh, 2067 2,500
Accounts receivable 31st Ashadh, 2067 3,500
Accounts payable 31st Ashadh, 2067 (4,000)
Net balance included in the above list of balances 62,000
Plant should be depreciated on a straight-line basis. It is not expected to have any residual value at
the end of the joint venture.
(v) The directors have estimated the required provision for income tax for the year to 31st Ashadh,
2067 is Rs.15 million. The deferred tax provision at 31st Ashadh, 2067 is to be adjusted to reflect
the tax base of the company‘s assets being Rs.70 million less than their carrying values. Rs.28·8
million of this Rs. 70 million is attributable to the revaluation of the leasehold. Virat‘s rate of
income tax is 25%.
(vi) The directors have proposed a final ordinary dividend of Rs.24 per share. Virat discloses proposed
dividends as part of shareholders‘ funds.
Required:

Prepare for Virat, in accordance with International Accounting Standards as far as the information
permits:
a) the Income Statement;
b) the Statement of Changes in Equity for the year to 31st Ashadh, 2067; and
c) a Balance Sheet as at 31st Ashadh, 2067.
Notes to the financial statements are not required. (7+4+9=20)
Answer:
a) Virat – Income Statement - for the year to 31st Ashadh, 2067

Rs '000
Sales revenue (473,300 + 18,000 (w (i)) 491,300
Cost of sales (w (ii)) (366,000)
Gross profit 125,300
Operating expenses (18,400)
Impairment of non-current asset (60,000 – 40,000) (w (iv)) (20,000)
Profit on the ordinary activities before interest 86,900
Loan interest (5,000 + 5,000) (10,000)
Profit before tax 76,900
Taxation (w (iii)) (13,900)
Profit for the period 63,000
_______

b) Virat – Statement of Changes in Equity – for the Year to 31st Ashadh, 2067

Ordinary Revaluation Accumulated Total


Shares reserve profits
The Institute of Chartered Accountants of Nepal
2 of 85
Suggested Answers of Advanced Accounting
CAP II Examination – June 2010

Rs '000 Rs '000 Rs '000 Rs '000

Balance at 1st Shrawan, 2066 100,000 nil 13,300 113,300


Surplus on revaluation of leasehold (w (iv)) - 126,000 - 126,000
Transfer to deferred tax (w (iii)) - (7,200) - (7,200)
Transfer to realised profits re leasehold
(126,000 – 7,200 deferred tax)/18 years) - (6,600) 6,600 -
Net profit for the period (63,000 – 24,000) - - 39,000 39,000
Dividends paid - - (15,500) (15,500)
Amount attributable to proposed dividends
(1000×Rs.24) - - 24,000 24,000

Balance at 31st Ashadh, 2067 100,000 112,200 67,400 279,600

c) Virat – Balance Sheet as at 31st Ahsadh 2067

Non-current assets Rs '000 Rs '000


Development expenditure (w (iv)) 30,000
Property, plant and equipment (w (iv)) 368,300
_______
398,300
Current Assets
Inventory (37,700 + 2,500) 40,200
Trade receivables (49,200 + 3,500) 52,700
Bank 12,100
105,000
Total assets 503,300
_______
Equity and liabilities
Ordinary shares Rs. 100 each 100,000
Reserves:
Accumulated profits (see (b) above) 67,400
Revaluation reserve (see (b) above) 112,200 179,600
______ _______
279,600
Non-current liabilities
10% Convertible loan stock 100,000
Deferred tax (70,000 × 25%) 17,500 117,500
______
Current liabilities (w (v)) 106,200
Total equity and liabilities 503,300

Working Notes
(i) IAS 31 ―Financial Reporting of Interests in Joint Ventures‖ requires each venturer in jointly
controlled operations to account for the assets that it controls and the liabilities that it incurs in its
balance sheet and the expenses that it incurs and the revenues that it receives in its income statement.
As this is not a separate entity the above should be included in the appropriate ―line‖ items in the
financial statements e.g. Virat‘s share of the sales of the joint venture should be added to its own sales
figure. A similar procedure should be applied to all other elements of the joint venture.

(ii) Cost of sales: Rs.‘000


Opening inventory 27,500
Purchases 310,500
Joint venture cost of sales 8,000
The Institute of Chartered Accountants of Nepal
3 of 85
Suggested Answers of Advanced Accounting
CAP II Examination – June 2010

Depreciation (w (iv)) – leasehold 15,000


– Plant (15,200 + 17,500) 32,700
– Development expenditure 10,000
Closing inventory (37,700)
_______
366,000
_______

(iii) Taxation Rs. ‗000


Current tax provision for year 15,000
Deferred tax (see below) (1,100)
_______
13,900
_______
The difference between the tax base of the assets and their carrying value of Rs.70 million would
require a balance sheet provision for deferred tax of Rs.17·5 million (at 25%). Of this Rs.7·2
million (Rs.28·8 million × 25%) is taken to the revaluation reserve leaving a balance of Rs.10·3
million. The opening provision was Rs.11·4 million, thus Rs.1·1 million (Rs.11·4 million –
Rs.10·3 million) is released to the income statement.

(iv) Non-current assets/depreciation/revaluation:

Development expenditure:

The capitalised development expenditure has suffered impairment due to falling sales of the related
product. Therefore, it should be written down to its recoverable amount, which is the higher of its
value in use (Rs.30 million) or its realisable value (Rs.40 million). Thus it should be written down
to Rs.40 million.

Impairment loss on development expenditure Rs.'000

Carrying amount (Rs.70 – 15) million 60,000


Less : Higher of realizable value and value in use (i,e. realizable value) 40,000
20,000

Depn. on development expenditure

Amortisation for the current year will be based on the revised carrying value of Rs.40 million and
its remaining life of 4 years.
Depn. on development expenditure = Rs 40/4 = Rs 10 million

a) Leasehold Building Rs. '000


Cost 200,000
Depreciation at 1 Shrawan 2066 (of 7yrs@8milion each year) (56,000)
Net book value prior to revaluation 144,000
(difference on revalued amount and net book value)
Valuation 1 Shrawan 2066 270,000
Revaluation surplus 126,000

Depreciation year to 31 Ashadh 2067 (270,000/18 (i.e. 25 – 7) years) (15,000)

Net book value at 31 Asad 2067=Rs. (270-15) million= 255,000

The Institute of Chartered Accountants of Nepal


4 of 85
Suggested Answers of Advanced Accounting
CAP II Examination – June 2010

_______
b) Plant: Rs. '000
Cost – non joint venture 124,800
– joint venture 70,000
_______
194,800
Depreciation for year
– non joint venture (124,800 – 48,800) × 20% 15,200
– joint venture (70,000/4 years) 17,500
_______
32,700
Net book value (194,800 – (48,800 + 32,700)) 113,300
_______

Net book value of property, plant and equipment (b+c)


(255,000 + 113,300) 368,300
_______

(v) Current liabilities Rs. '000


Per trial balance 82,200
Joint venture 4,000
Accrued loan interest ((10% × 100,000) – 5,000) 5,000
Taxation 15,000
_______
106,200

2.
a) As a bad luck in the very beginning of new year, a fire occurred on 1st Baisakh, 2066 in the
premises of Kanchanjungha Ltd. and business was partially affected up to 30th Bhadra, 2066. The
Company was insured under a loss of profits policy for Rs. 150,000 with a six months period of
indemnity. Following further information are available:
Rs.
st th
Actual turnover from 1 Baisakh, 2066 to 30 Bhadra 2066 100,000
Turnover from 1st Baisakh, 2065 to 30th Bhadra, 2065 200,000
st st
Turnover from 1 Baisakh, to 31 Chaitra, 2065 450,000
Net profit for last financial year 86,000
Insured standing charges for last financial year 43,000
Turnover for the last financial year 430,000

It has been agreed that an adjustment of 20% be made in respect of the upward trend in turnover.
Required:
Computation of the amount of claim under the loss of profit policy. 10

b) On 31st Ashadh, 2065, A Ltd's Balance Sheet showed 10,000, 12 % debentures of Rs. 100 each
outstanding. Interest on debenture is payable on 30th Poush and 31st Ashadh every year. On 1st
Mangsir, 2065, the company purchased 500 of its own debentures as investment at Rs. 97 ex-
interest.
Required:
Journal entries supposing:
i) The company cancels all its own debentures on 1st Ashadh, 2066.
ii) The company resells all its own debentures at Rs. 105 cum- interest on 1st Ashadh, 2066.
(7+3=10)

The Institute of Chartered Accountants of Nepal


5 of 85
Suggested Answers of Advanced Accounting
CAP II Examination – June 2010

Answer:
a) Computation of the amount of claim for the loss of profit
Reduction in turnover Rs.
Turnover from 1st Baisakh, 2065 to 30th Bhadra, 2065 200,000
Add: 20% expected increase 40,000
240,000
Less: Actual Turnover from 2066 to 30th Bhadra, 2066 100,000
Short Sales (Reduction in turnover) 140,000
Gross profit on reduction in turnover @ 30% on Rs.140000 42,000
(see working note 1 for gross profit)
Application of Average Clause:
Amount of Policy x Reduction in GP
GP on Annual Turnover
38,888.88
=(150000/162000) x 42000

Amount of claim under the policy = Rs. 38,888.88

Working notes
(i) Rate of Gross profit for last financial Year:
Gross profit: Rs.
Net Profit 86,000
Add: Insured Standing Charge 43,000
129,000
Turnover for the last financial year 430,000
Rate of Gross Profit = (129,000 / 430,000 ) x 100%

(ii) Annual Turnover and Gross Profit there on:


Turnover from 1st Baisakh 2065 to 31st Chaitra 2065 450,000
Add: 20% expected increase 90,000
540,000
Gross profit on Rs. 5,40,000 @ 30% 162,000

b) Journal Entries
Cr. Dr.

Rs. Rs.
2065 Own debenture Account 48,500
Mansir 1 Debentures Interest Account 2,000
To bank 50,500
(Purchase of 500 of own Debentures of Rs. 100 each
at Rs. 97 ex- interest. Amount of interest paid being
Rs. 12/100 * 4/12 * 50 =Rs. 2,000)
Poush 30 Debenture Interest Account 58,000
2065 To bank 57,000
The Institute of Chartered Accountants of Nepal
6 of 85
Suggested Answers of Advanced Accounting
CAP II Examination – June 2010

To Interest on Own debenture Account 1,000


(Payment of half yearly interest on 9500 debenture
and credit of 2 months interest)
2066 12% Debenture a/c 50,000
Ashadh 1 To Own Debenture A/C 48,500
To Profit on cancelation of own Debenture A/C 1,500
(Cancellation of 500 own debenture purchased earlier
at 97 ex- interest)
2066 Profit on cancelation of own Debenture A/C 1,500
Ashadh 1 To capital Reserve A/c 1,500
(Transfer of profit on cancelation of own debenture
A/c to capital reserve a/C)
2066 Debenture Interest A/C 2,500
Ashadh 1 To interest on own debenture A/c 2,500
(Interest accrued on 500 own debenture from the last
date of payment of interest ( 30th Poush 2065) to the
date of cancelation of debenture ( 1st Ashad 2066)

Ashadh Debenture Interest A/C 57,000


31, 2066 To bank A/c 57,000
(Payment of half yearly interest on 9500 debenture
in cash)
Profit & Loss A/c 119,500
To Debenture Interest A/c 119,500
(Transfer of debenture interest a/c/to p. & L. A/C)
Interest on Own debenture Account 2,500
To Profit & Loss A/c 3,500
(Transfer of interest on debenture a/c/to p. & L. A/C)
2nd option
2066 Bank 52,500
Ashad 1st To Own debenture a/c 50,000
To Interest on own debenture a/c 2,500
(Resale of 500 own debenture at rs. 105 cum-
interest, ex- interest price being credited to own
debenture a/c/and 5 months interest to interest on
own debenture a/c)
Own Debenture a/c 1,500
To profit on resale of Own debenture a/c 1,500
(Profit on resale of own debenture being excess of ex
interest sale price over ex interest purchase
pricetransfered from own debenture a/c to profit on
resale of Own debenture a/c)
Ashad Interest on Own debenture A/C 3,500
31,2066 Profit on resale of Own debenture a/c 1,500
To Profit & Loss A/C 4,000
(Transfer of interest on own debenture and profit on
resale Own Debenture a/c to P/L a/c)

3.
The Institute of Chartered Accountants of Nepal
7 of 85
Suggested Answers of Advanced Accounting
CAP II Examination – June 2010

a)
i) Butwal Electricity Ltd. decides to replace one of its old plant with a modern one with large
capacity. The plant, when installed in 2050 B.S., cost the company Rs. 2,400,000, the
components of material, labour and overhead being in the ratio of 5:3:2. It is ascertained that
the cost of material and labour have gone up by 40% and 80% respectively. The proportion of
overhead to total cost is expected to remain the same as before.

The cost of the new plant as per improved design is Rs. 6,000,000 and, in addition, material
recovered from the old plant of a value of Rs. 240,000 has been used in the construction of the
new plant. The old plant was scrapped and sold for Rs. 750,000.

The accounts of the company are maintained under Double Account System.

Required:
Indicate how much would be capitalized, and the amount that would be charged to revenue by
preparing relevant ledger accounts. 7

ii) Discuss about the treatment of capital losses under Double Account System of electricity
companies. 3

b)
i) Pashupati asked you to prepare his Balance Sheet from the particulars furnished hereunder:
Gross Profit margin: 10%
Stock velocity: 12
Capital turnover ratio: 2
Fixed assets turnover ratio: 5
Debt collection period: 1 month
Creditor‘s payment period: 73 days
Gross profit: Rs. 100,000
Excess of closing stock over opening stock: Rs. 30,000
Make suitable assumptions wherever necessary. 7

ii) Explain the concept of Capital Maintenance. 3

Answer:
a) i)
Plant Account
Rs. Rs.
To balance B/F 2,400,000 By Balance C/d 4,920,000
To Bank A/C 2,280,000
( cost of new plant- capitalized)
To replacement A/c ( old parts) 240,000
4,920,000 4,920,000

Replacement Account
Rs. Rs.
To Bank a/c 3,720,000 By Bank a/c 750,000
(current cost of replacement) ( sale of scrap)
By Plant A/C
(old material used) 240,000
The Institute of Chartered Accountants of Nepal
8 of 85
Suggested Answers of Advanced Accounting
CAP II Examination – June 2010

By revenue A/C
( transfer) 2,730,000
3,720,000 3,720,000
Working notes

Cost to be incurred for replacement of present plant:

Cost of existing plant increase Current cost


Rs. % Rs
Materials 1,200,000 40 1,680,000
Labour 720,000 80 1,296,000
2,976,000
Overheads ( ¼ of above or 1/5 of total) 744,000
Current replacement cost 3,720,000
Current replacement cost 3,720,000
Total cash cost (a) 6,000,000
Amount capitalized excluding old material used 2,280,000

ii) Treatment of Capital Losses under double account system of electricity companies.
If the assets are abandoned or their normal life is exhausted it is not necessary to write off losses in
capital accounts. As a result assets which are not in use or which are of no value to the entity are
carried on the capital accounts at their original cost.
For practical reason and to account for the losses in Profit and Loss Account, suitable alternatives can
made be and adopted by charging the amount against profits and creating certain provisions.

b) i) Balance Sheet of Mr. Pashupati

Liabilities Rs. Assets Rs.


Capital 500,000 Fixed Assets 200,000
Creditors 1,86,000 Stock 90,000
Debtors 83,333
*Bank balance 312,667
686,000 686,000

*taken to be balancing figure


Working Note
(i) Gross profit= Rs. 100,000
Gross profit margin= 10%
Hence, Sales= Rs. 100,000 x 100/10= Rs. 1,000,000
Cost of goods sold = Sales – Gross profit
= Rs.( 1000,000-100,000)
= Rs. 900,000
Purchase = Cost of goods sold + Increase in stock
= Rs. ( 900,000+30,000) = Rs. 930,000

Average stock= Cost of goods sold/stock velocity


= Rs. 900,000/12=75000

The Institute of Chartered Accountants of Nepal


9 of 85
Suggested Answers of Advanced Accounting
CAP II Examination – June 2010

(ii) Capital:
Capital turnover ratio = 2
Sales/Capital=2
Hence, Capital= Rs. 1,000,000/2 = Rs. 500,000

(iii)Creditors:
Creditors payment period = 73 days
Hence, Creditors= Purchases x 73/365 =Rs. 930,000 x 1/5 = Rs. 186,000

(iv) Fixed assets:


Fixed assets turnover ratio = 5
Hence, Fixed Assets = Sales/5= Rs. 1,000,000/5= Rs. 200,000

(v) Closing stock:


Closing stock is Rs. 30,000 more than opening stock; it is Rs. 15000 more than average stock.
Hence, closing stock= Average stock + Rs. 15000
= Rs. 75,000+Rs.15,000= Rs. 90,000

(vi) Debtors:
Debt collection period = 1 month
Hence, debtors= Sales x 1/12 = Rs. 1,000,000 x 1/12 = Rs. 83,333.33
It is assumed that there is no change in capital during the period.

Note: Alternate solutions are possible.

ii) Capital Maintenance


Broadly, there are two major concept of capital maintenance:
First one is the concept of physical capital maintenance where profit means increase in operating
or production capacity of an enterprise at the end of a period compared to that at the beginning of
the period.
In second concept, profit is only earned if the monetary value of the net assets at the end of a
period exceeds the monetary value at the beginning of the period.
Both the concepts exclude any contributions from and distributions to the owners during the
period.
Underlying principle of capital maintenance is that profit should be only recognized, after ensuring
that capital is intact as compared to the beginning of the period and its not eroded as compared to
the starting point of the period under consideration.

4.
a) Sima, Rima and Hima are running a Beauty Parlor in the form of a partnership firm. They share
profits and losses in the proportion of 1:4:5. Sima received an offer for foreign employment, and
retires from the partnership. However, existing partners are not able to pay her share of capital at
the time of retirement. Adjusted capital at the time of retirement of Sima was as follows:

Sima: Rs. 100,000


Rima: Rs. 250,000
Hima: Rs. 300,000

The Institute of Chartered Accountants of Nepal


10 of 85
Suggested Answers of Advanced Accounting
CAP II Examination – June 2010

Remaining two partners continued to carry on the business of partnership and they are in a position
to settle Sima‘s share of capital after 6 months. During the period the Parlor made a net profit of
Rs. 36,000. Sima borrowed loan of Rs. 120,000 from a bank at an interest rate of 10% at the time
of going for foreign employment.

There was a dispute among the partners regarding sharing of profits after separation of Sima.
However, existing partners agreed either to pay bank interest on borrowed fund or to share profits
according to Capital share at the time of separation.

Required:
Suggest Sima of the option that is more beneficial for her. 5

b) Following figures represent expected production of a machinery taken on 5 years operating lease:

Years Lease Amount(Rs) Production(MT)


1 15,000 13,000
2 35,000 21,000
3 50,000 34,000
4 47,000 32,500
5 44,500 30,000
191,500 130,500

Required:
Pass necessary journal entries in the books of lessee for initial two years. 5

c) Briefly explain about the preferential creditors and their order of payment in the case of a company
under liquidation. 5

Answer:
a) As Sima is already retired, she is entitled a profit share in proportion to her capital investment and
previous profit sharing ratio is irrelevant in this situation.

Total profit for 6 month‘s period = Rs 36,000

Sima‘s Share = 100,000/(100,000+250,000+300,000) X 36,000= Rs.5,538.46

Interest on unpaid Capital=100,000 X10%X6/12=Rs.5,000

Since share of profit will be more beneficial to Sima, she has to opt for the option of sharing
profits in the proportion of capital at the time of separation.

b) Lease rental to be recognized as expense in proportionate to units of production.

Here, Total lease rental for 5 years= Rs.191,500


Total Production for 5 years=130,500

Hence, Lease rental to be charged in 1st year=Rs. 13,000/130,500 X 191,500= Rs.19,077


Lease rental to be charged in 2nd year=Rs. 21,000/130,500 X 191,500=Rs.30,816

Journal Entries in the books of lessee for first and second year:
The Institute of Chartered Accountants of Nepal
11 of 85
Suggested Answers of Advanced Accounting
CAP II Examination – June 2010

First Year
a) Lease Rent Dr 15,000
To Bank Cr. 15,000
(Being Lease rent for the first year of lease paid)

b) Lease Rent Dr 4,077


To Lease Rent Adjustment Ac Cr 4,077
(Being adjustment of lease rent for the year)

c) P&L Ac Dr 19,077
To lease Rent Cr 19,077
(Being recognition of lease rent as expense for the year)

Second Year

a) Lease Rent Dr. 35,000


To Bank Cr. 35,000

(Being Payment of Lease rent for the year)

b) Lease Rent Adjustment Dr. 4,184


P& L Ac Dr . 30,816
To Lease Rent Cr. 35,000
(Being recognition of lease rent as expense for the year)

c) Preferential creditors in case of liquidation of a company are those creditors who get priority in
payment over others though they are unsecured creditors by virtue of provisions of insolvency
(Damasahi) Act 2063.

According to insolvency Act, expenses relating to liquidation, wages and remuneration payable
and gratuity, provident fund, leave encashment gets priority over other creditors. The amount
available with liquidator from the disposal of assets of the company needs to pay in the following
priority:
a) Payment of liquidation expenses and liquidator‘s remuneration.
b) Payment of wages and remuneration to workers and employees outstanding at the time of start
of liquidation process or at the time of order of restructure of the company, however, directors
engaged in the capacity of employee are not entitled to get payment under this priority.
c) Payment of accumulated sick leave, home leave, gratuity and provident fund to workers and
employee outstanding at the time order of liquidation or at the time of restructure of the
company.
d) Payment to other creditors whose claims are accepted by liquidator.

Furthermore, The Insurance Act, 2049 and Bank and financial institution Act, 2063 have separate
priority order in respect of payment in case of liquidation of company engaged in the business of
Insurance and Banking respectively and provision of Insolvency Act regarding priority in payment
would not apply to above mentioned companies.

The Institute of Chartered Accountants of Nepal


12 of 85
Suggested Answers of Advanced Accounting
CAP II Examination – June 2010

5.
a) NAS 18 "Impairment of Assets" requires an entity to write down the value of its assets, or group of
assets, whenever the recoverable amount of an asset is less than its carrying value.
Required:
Define ―recoverable amount‖ and state why an asset should be written down to this value if it is
below its carrying value. 5

b) Brindawan Ltd. has an item of plant that is carried in the balance sheet at a revalued amount of Rs.
16,200,000. The plant manufactures a product which, until recently, was the only product of its
type in the market place. A competitor is now manufacturing a similar product, and Brindawan
Ltd.‘s market share has consequently fallen. Brindawan Ltd. has re-assessed the expected cash
flows, to be generated from using the plant over the remaining four years of its life, to be as
follows:
Rs.
2067/68 7,000,000
2068/69 4,000,000
2069/70 3,500,000
2070/71 2,000,000

Brindawan Ltd. has been offered Rs. 15,000,000 for the plant by an overseas company.
Brindawan Ltd. would be responsible for any shipping and conversion costs that are estimated to
be Rs. 1,000,000.
The depreciated historic cost of the asset is Rs 15,200,000.
Brindawan Ltd.‘s cost of capital is 8%. The present value of Re 1 receivable at the end of each
year, based on discount rates of 8% can be taken as:

End of year 1 0·93


2 0·86
3 0·80
4 0·74
Required:

Calculate the impairment loss in respect of the plant and state how the loss should be accounted for
in Brindawan Ltd.‘s financial statements for the year ended 31st Ashadh, 2067. 6

c) NAS 9 ―Income Taxes‖ uses the concept of temporary differences. Temporary differences are the
difference between the carrying value of an asset and its tax base. The standard distinguishes
between ―taxable temporary differences‖ and ―deductible temporary differences‖.
Required:
Explain the distinction between taxable and deductible temporary differences. 4
Answer:
a)
NAS 18 ―Impairment of Assets‖ defines recoverable amount as the higher of an asset‘s fair value less
costs to sell and its value in use.

It further defines the fair value less costs to sell as the amount obtainable from the sale of an asset (in
an arm‘s length transaction between knowledgeable, willing parties) less costs of disposal.

The Institute of Chartered Accountants of Nepal


13 of 85
Suggested Answers of Advanced Accounting
CAP II Examination – June 2010

Also, it defines the value in use as the present value of the future cash flows expected to be derived
from an asset (or cash-generating unit).

The recoverable amount of an asset represents the amount of economic benefits that the asset will
generate for an entity. If the carrying value of an asset exceeds its recoverable amount this means that
the asset will not generate sufficient economic benefits to meet its carrying value, the asset should
therefore be written down to the value that is recoverable by either continuing to use the asset or by
selling it.

b) Calculation of impairment loss


Fair value less costs to sell
Rs
Selling price 15,000,000
Shipping
_ and conversion costs (1,000,000)
Net proceeds 1,4000,000
_______

Value in use
Year Cash flow DF Rs
2067/68 70,00000 0.93 6,510,000
2068/69 40,00000 0.86 3,440,000
2069/70 35,00000 0.80 2,800,000
2070/71 20,00000 0.74 1,480,000
14,230,000

Impairment Loss ( Rs) = Carrying Amount – Value in Use


= 16,200,000 – 14,230,000 = Rs. 1,970,000
_______

The recoverable amount of the asset is the greater amount, i.e. its value in use of Rs 1,4230,000. The
asset is carried at a revalued amount in the balance sheet of Rs 16,200,000, this means that an
impairment loss of Rs 1,970,000 has occurred and must be written off the carrying value of the asset.

As the asset is carried at a revalued amount and the depreciated historic cost of the asset is Rs
15,200,000 there will be a revaluation surplus of Rs 970,000 (Rs.15,2500,000 – Rs.14,230,000) in
respect of the asset, this surplus would initially be utilised for the write down.

The remaining impairment loss of Rs. 1,000,000 will be charged to the profit and loss account for the
period.
c) Distinction between taxable and deductible temporary differences

Temporary differences are differences between the carrying amount of an asset or liability in the
balance sheet and its tax base. The tax base of an asset or liability is the amount attributed to that asset
or liability for tax purposes.

Temporary differences may be either taxable temporary differences or deductible temporary


differences. Taxable temporary differences are temporary differences that will result in taxable
amounts in determining taxable profit (tax loss) of future periods when the carrying amount of the
The Institute of Chartered Accountants of Nepal
14 of 85
Suggested Answers of Advanced Accounting
CAP II Examination – June 2010

asset or liability is recovered or settled. For instance if a tax authority taxes interest income on a cash
basis then any income accrued at the balance sheet date will not be included in the current tax charge
but will be taxed in the future when the cash is received.

Deductible temporary differences are the temporary differences that will result in amounts that are
deductible in determining taxable profit (tax loss) of future periods when the carrying amount of the
asset or liability is recovered or settled. For instance, if a tax authority taxes interest expense on a cash
basis then any expense accrued at the balance sheet date will not be included in the current tax charge
but will be deductible in the future when it is paid.

6. Write Short notes on (ANY FOUR): (4×2.5=10)


a) Super profits in partnership firms
b) Contract revenue
c) Floating Policy
d) Events after Balance Sheet date
e) Consequences of insolvency of a partnership firms

Answer:

a) Super profits in partnership firms


Among various basis of determining the sharing and paying for joining or sacrificing the profits of a
firm one basis is super profit method. Under this method, it is assumed that any partner can get normal
profits by joining any average firm in the market. However, focus should be given on super profits of
the firm. Such super profit implies that the profit can be earned by a firm over and above all ordinary
firms in the industry/market. This is the excess amount of profit earned by a firm over the past years
and expected to continue the same in the future.
If and only if the average profit of the firm is more than the normal profit in the market there comes
into existence the super profit or goodwill.
b) Contract revenue
Contract revenue can be:
A fixed price or cost plus basis prices
Compensation for variations in contract work
Claims
Incentives and
Other possible inflows by way of sale of residue assets otherwise not fit for use at the end of
contract period.
c) Floating Policy
It is some sense which resembles blanket policy of insurance. It is a policy which covers several types
of goods for one lump sum premium. The goods may be lying at different locations and may be
exposed to different level of risk.
The premium, normally charged under this policy, should nearly resemble the sum of premium
charged under specific policy.
The benefit of this policy is that insured need not take many specific policies for goods lying in
different locations and for different types thereon.

d) Events after balance sheet date


All financial events up to the balance sheet date should be taken in to consideration in preparation of
financial statement or for an accounting period. Certain important events may occur after balance
sheet date, the knowledge of which is important for making an assessment of performance and
The Institute of Chartered Accountants of Nepal
15 of 85
Suggested Answers of Advanced Accounting
CAP II Examination – June 2010

making projection for the future. Nepal Accounting Standard defines the events after balance sheet
data as:

Events after the balance sheet date are those events, favorable and unfavorable, that occur between
the balance sheet date and the date when the financial statements are authorized for issue. There are
two types of events.

e) Consequences of insolvency of a partner


Following consequences arise when a partner is insolvent:
-The partner ceases to be a partner
-The firm is dissolved unless there is a contrary contract not to do so.
-The firm and existing partner can not be held liable after the date of declaration of insolvency.
Similarly, the estate of insolvent partner can not be made liable for any subsequent act of the firm or
partners thereon.

The Institute of Chartered Accountants of Nepal


16 of 85
The Institute of Chartered Accountants of Nepal
Suggested Answers of Audit and Assurance

CAP II Examination June 2010

1. G
ive your opinions with reason on the following cases: (45=20)
a) M/s Joshi & Joshi is a practicing Chartered Accountants firm. One of the senior partners of the
firm is also associated with import/export trade and he is the sole proprietor of that business.
Mr. Joshi has also taken huge amount of loan from various banks and he is found visiting more
to government offices than in his own firm or to his clients. Comment on given case with
reference to ‗Code of Ethics‘.
b) Metro Developers Pvt. Ltd is Bharatpur based Real Estate Company. It has invested heavily on
the apartments and commercial complexes on and around Narayangarh. Recently, audit of the
transaction for rental receipts was hastily completed by taking monthly average figure and
multiplying it by 12. You are required to suggest appropriate way of conducting of audit for the
above said transaction of rental receipts.

c) Association for Backward Communities (ABC) is one of the NGOs working in the Midwest
part of the country. There seems to be confusion regarding total receipt of the NGO as what
could be the probable revenue sources and how they should be vouched. The chief of the NGO
is mostly on the foreign tour and the accountant of the NGO simply says ‗boss knows
everything and he is just clerk‘. What documents you will insist to check to deal with the
situation?

d) Torstar‘s Heaven is star level hotel situated at Nagarkot. During the initial discussion it is
noticed that the hotel management is aware of pilferage issues, however due attention is not
paid to address the situation. You are required to express your statement underlying the
importance of internal control on the pilferage issues and the probable consequences of it, if
not addressed properly.

Answers:
1.a.) A professional accountant in public practice should not concurrently engage in any business,
occupation or activity which impairs or might impair integrity, objectivity or independence, or the
good reputation of the profession and therefore would be incompatible with the rendering of
professional services. The simultaneous engagement in other business, occupation or activity
unrelated to professional services, which have the effect of not allowing the professional
accountant in public practice properly to conduct a professional practice in accordance with the
fundamental ethical principles of the accountancy profession should be regarded as inconsistent
with the practice of public accountancy.

b.) To vouch for rental receipts, various documents need to be seen. To start with, copies of bills
issued to tenants should be test checked by reference to copies of tenancy agreements and bills of
charges paid by the landlord on behalf of the tenants e.g., house tax, water tax etc. The amounts
collected from tenants on account of rent should be checked by reference to receipts issued by
them. The entries in the Rental Register in respect of rents accrued afterwards should be verified.
The register should also be scrutinized for finding out the rent amount, which have not been
recovered and are considered bad or irrecoverable, for deciding whether these should be written off
or provision against the same should be made.

c.) The receipt of income of NGO may be checked on the following lines:
i) Contribution and grants for projects and programmes: Check agreements with donors
and grants letters to ensure that funds received have been accounted for. Check that all
Suggested Answers of Audit and Assurance
CAP II Examination – June 2010

foreign contributions receipts are deposited into bank accounts and proper disclosure is
made.
ii) Receipts from fund raising programmes: Verify in detail the internal control system and
ascertain who are the persons responsible for collection of funds and mode of receipt.
iii) Interest and dividends: Check the interest and dividends received and receivable with
investments held during the year.
iv) Miscellaneous receipts: Check sale of scrap, rental receipts etc, if any.

d.) Pilfering is one of the greatest problems in any hotel and the importance of internal control
cannot be over stressed. It is the responsibility of the management to introduce controls, which will
minimize the leakage as far as possible. If the internal control in the hotel is weak, then a very
serious problem exists for the auditor. Preparing accounts at regular interval for each of the cost/
revenue centers and investigating the deviation if any could be helpful in this regard. If necessary,
scope of audit tests will have to be increased and in the event of material margin discrepancy
being unexplained, the auditor may consider qualifying his report.

2. A
nswer the following:
a) T
he auditor may encounter circumstances that, individually or in combination, indicate the
possibility that the financial statements may contain a material misstatement resulting from
fraud or error. List down some of those (at least 8) circumstances that indicate a possible
misstatement. 8
b) S
o far as the auditor is concerned, the examination and evaluation of the internal control system
is an indispensable part of the overall audit programme. What are the key areas that auditor
will enable to know after reviewing internal controls. 7

Answers:
2.a.)
Some of the circumstances that, individually or in combination, indicate the possibility that the
financial statements may contain the material misstatement:
i) Unrealistic time deadlines for audit completion imposed by management.
ii) Reluctance by management to engage in frank communication with appropriate third
parties, such as regulators or bankers.
iii) Limitation of audit scope imposed by management.
iv) Identification of important matters not previously disclosed by management.
v) Significant difficult-to-audit figures in the accounts.
vi) Aggressive application of accounting principles.
vii) Conflicting or unsatisfactory evidence provided by management or employees.
viii) Unusual documentary evidence such as handwritten alterations to documentations
ix) Information provided unwillingly or after unreasonable delay.
x) Seriously incomplete or inadequate accounting records.

b.) The review of internal controls will enable the auditor to know:
i) Whether errors and frauds are likely to be located in the ordinary course of operations
of the business.
ii) Whether an adequate internal control system is in use and operating as planned by the
management.
iii) Whether an effective internal auditing department is operating.
The Institute of Chartered Accountants of Nepal
2 of 85
Suggested Answers of Audit and Assurance
CAP II Examination – June 2010

iv) Whether the controls adequately safeguards the assets.


v) How far and the how adequately the management is discharging its function in so far as
correct recording of transactions is concerned.
vi) How reliable the reports, records and the certificates to the management can be.
vii) The extent and the depth of the examination that he needs to carry out in the different
areas of auditing.
viii) What would be appropriate audit technique and audit procedure in the given
circumstances.
ix) What are the areas where control is weak and where it is excessive.
x) Whether some worthwhile suggestions can be given to improve the control system.

3. G
ive your comments on the following: (35=15)
a) G
iant Ltd. is newly established cement industry and yet to start commercial production. Various
payments, large and small, are being regularly made. You are required to prepare internal
control questionnaire regarding ‗procedure for authorization‘ for payments.
b) During the course of audit, M/s Guess and Guess Associates, an audit firm, has checked 20
percent of the postings from Cashbook to General Ledger. Some serious questions could be
raised about the representative nature of the sample and about the validity of sample results.
What could those serious concerns be?
c) Forward Looking Co Ltd is dealing with various transactions that could be well termed as
contingent liability. As an auditor, suggest some of the procedures for verifying contingent
liabilities?

Answers:
3.a)
The internal control questionnaire for ‗procedure for authorization‘ may be drawn as below:
i) Does the enterprise have a formal document showing the various authorities, which can
sanction payments along with the nature of the relevant payments and the limit on sanctioning
powers?
ii) Does the system provide for authorization for certain payments by the top
management or the governing body?
iii) Are the financial powers of various sanctioning/disbursing authorities reasonable?
iv) Have the financial powers of various authorities been intimated to various
departments of the enterprise?
v) Is there a system of post facto authorization of payments in exceptional cases where
prior sanction could not be obtained?

b. Some of the serious concerns about the representative nature of the sample and about the
validity of sample results could be:
i) Why select, say 20 percent of the posting? Why can it not be 15 or 25 percent? Why check
only 2 months?
ii) Can we just state the percentage size of the sample without reference to the total number of
transactions? Are we taking the same amount of risk if we check 2 out of a total of 10
transactions or 200 out of 2000 transactions?
iii) How can we draw conclusions about the transactions of the whole year by merely checking the
transactions of a few specified months, especially as the level of activity may vary from month
to month?
The Institute of Chartered Accountants of Nepal
3 of 85
Suggested Answers of Audit and Assurance
CAP II Examination – June 2010

iv) By not stating the manner in which the 20 percent of the transactions have to be selected,
would not the audit assistants consciously or unconsciously select only those transactions,
which are simpler and easier to audit?

c.
The auditor may carryout the following procedures for verifying the contingent liabilities:
i) Review the minutes of the meetings of board of directors/ committees.
ii) Review the relevant contracts, agreements and arrangements.
iii) Review the list of pending legal cases, correspondence relating to taxes, duties etc.
iv) Review the records maintained by the entity regarding contingent liabilities.
v) Make enquiries of and hold discussion with the entity‘s management.
vi) Obtain representations from the management.

4. A
nswer the following: (35=15)
a) W
hat are the assertions with which an auditor is concerned with while obtaining audit evidence
from substantive procedures?
b) Mention briefly the conditions or events, which increase the risk of fraud or error leading to
material misstatement in Financial Statements.
c) What is agreed upon procedures?

Answers:
(a) An auditor is concerned with following assertions:-
(i) Existence: That an asset or liability exists at a given date.
(ii) Rights and obligations: That an asset is a right of the concern and a liability is an obligation at
a given date.
(iii)Occurrence: That a transaction or event which took place pertains to the entity during the
relevant period.
(iv) Completeness: That there are no unrecorded assets, liabilities or transaction.
(v) Valuation:That an asset or liability is recorded at an appropriate carrying value.
(vi) Measurement: That a transaction is recorded in the proper amount and revenue or expense is
allocated to the proper period.
(vii)Presentation and disclosure: That an item is disclosed classified and described in accordance
with recognised accounting policies and practices and relevant statutory requirements.

(b) In planning and performing his examination, the auditor should take into consideration the
risk of material misstatements of the financial information caused by fraud or error.
Weaknesses in the design of the internal control system and non-compliance with
identified control procedures amongst other conditions or events which increase the risk of
fraud or error are:
(i) Weaknesses in the design of internal control system and non-compliance with the laid
down control procedures, e.g., a single person is responsible for the receipt of all d ak and
marking it to the relevant sections or two persons are responsible for receipt of dak but the
same is not followed in actual practice, etc.
(ii) Doubts about the integrity or competence of the management, e.g., domination by one
person, high turnover rate of employees, frequent change of legal counsels or auditors,
significant and prolonged understaffing of the accounts department, etc.
The Institute of Chartered Accountants of Nepal
4 of 85
Suggested Answers of Audit and Assurance
CAP II Examination – June 2010

(iii)Unusual pressures within the entity, for example, industry is doing well but the company is
not performing all right, heavy dependence on a single line of product, inadequate working
capital, entity needs raising share prices to support the market price in the wake of public
offer, etc.
(iv)Unusual transactions such as transactions with related parties, excessive payment for
certain services to lawyers, etc.
(v) Problems in obtaining sufficient and appropriate audit evidence, e.g., inadequate
documentation, significant differences between the figures as per the accounting records
and confirmation received from third parties, etc.

(c)
According to the NSA 4400, ―Engagement Agreed upon Procedures Regarding Financial
Information‖ the objective of an agreed-upon procedures engagement is for the auditor to carry out
procedures of an audit nature to which the auditor and the entity and any appropriate third parties
have agreed and to report on factual findings.
As the auditor simply provides a report of the factual findings of agreed-upon procedures, no
assurance is expressed by him in his report. Instead, users of the report assess for themselves the
procedures and the findings reported by the auditor and draw their own conclusions from the work
done by the auditor.
The report is restricted to those parties that have agreed to the procedures to be performed since
others, unaware of the reasons for the procedures, may misinterpret the results. However, it is
possible in certain circumstances that the report of the engagement may not be restricted only to
those parties that have agreed to the procedures to be performed, but made available to a wider
range of entities or individuals, e.g., in case of Government organisations.
The auditor should conduct an agreed-upon procedure engagement in accordance with this NSA
and the terms of the engagement. The auditor should ensure with representatives of the entity and,
ordinarily, other specified parties who will receive copies of the report of factual findings, that
there is a clear understanding regarding the agreed procedures and the conditions of the agreement.

Matters to be agreed include the following:


- Nature of the engagement including the fact that the procedures performed will not constitute
an audit or a review and that accordingly no assurance will be expressed.
- Stated purpose for the engagement.
- Identification of the financial information to which the agreed-upon procedures will be applied.
- Nature, timing and extent of the specific procedures to be applied.
- Limitations on distribution of the report of factual findings. When such limitation would be in
conflict with the legal requirements, if any, the auditor would not accept the engagement.

In certain circumstances, for example, when the procedures have been agreed to between the
regulator, industry representatives and representatives of the accounting profession, the auditor
may not be able to discuss the procedures with all the parties who will receive the report. In such
cases, the auditor may consider, for example, discussing the procedures to be applied with
appropriate representatives of the parties involved, reviewing relevant correspondence from such
parties.

Matters that would be included in the engagement letter include:


- A listing of the procedures to be performed as agreed-upon between the parties.
- A statement that the distribution of the report of factual findings would be restricted to the
specified parties who have agreed to the procedures to be performed.

The Institute of Chartered Accountants of Nepal


5 of 85
Suggested Answers of Audit and Assurance
CAP II Examination – June 2010

5. C
omment on the following: (35=15)

a) W
hat is clean audit report? Explain how it is different from qualified report?

b) H
ow the work of an expert should be evaluated before accepting the same as Audit
evidence?

c) C
are Limited purchased machinery on 1.4.2065 from a foreign country at a price of $ 200
thousands upon terms of credit that the price should be settled within six months from the date
of purchase. The company capitalised the asset and created liability for the capital goods
converting the foreign currency liability to Nepalese Rupees at a rate of exchange prevailing as
on 1.4.2065. When the company settled the liability on 30 Poush 2065, it had to incur an
additional amount of Rs. 5,00,000 due to change in foreign exchange rate on the date of
settlement. It added this additional amount of exchange variation in the capital cost of the asset
and charged depreciation upon the enhanced amount of asset value from 1 Magh 2065. Give
your opinion.

Answers:
a) A clean audit report is a report issued by an auditor in case he does not have any reservation in
respect of matters contained in the financial statements. In such a case, the audit report may
state that the financial statements give a true and fair view of the state of affairs and of profit
and loss account during the period. A clean report may be without any modifications or with
modifications which are just for matter of emphasis. Under the following circumstances an
auditor is justified in issuing a clean report:
(i) the financial information has been prepared using acceptable accounting policies, which have
been consistently applied;
(ii) the financial information complies with relevant regulations and statutory requirements; and
(iii) there is adequate disclosure of all material matters relevant to the proper presentation of the
financial information, subject to statutory requirements, where applicable.
Qualified audit report, on the other hand, is one when auditor does not give a clean chit about the
truthfulness and fairness of the financial statements but makes certain reservations. A qualified
report is a modified report as to the auditor‘s opinion. A qualified report is issued when there is
limitation on the scope of audit or disagreement with management, regarding the acceptability of
accounting policies selected or the method of application or the adequacy of financial statement
disclosure.
The auditor uses the word to indicate his qualification or reservation by placing the word ―subject
to‖ or ―except to‖. The qualifications should indicate impact on profits and account balances and
should be specific, clear and self explanatory. The auditor should also give reasons for
qualification. In case of companies, there is also a legal requirement of the CompanyAct which
provides that where the auditor answers any of the statutory affirmations in negative or with
qualification, his report shall state the reasons for such answer.
Thus, it is clear from the above that in case of a clean report, the auditor has no reservation in
respect of various matters contained in the financial statements but a qualified report may involve
certain matters involving difference of opinion between the auditor and the management.

b) Using the work of an expert: As per the NSA 620, when the auditor intends to use the work
of an expert he should evaluate the following before accepting the same as audit evidence:
(i) Professional qualification of the expert;
The Institute of Chartered Accountants of Nepal
6 of 85
Suggested Answers of Audit and Assurance
CAP II Examination – June 2010

(ii) Experience and reputation of expert in related field;


(iii)Independence and objectivity of the expert;
(iv) The objectives and scope of the expert‘s work;
(v) Expert‘s relationship with the client, if any;
(vi) The source data used;
(vii)Assumptions and method used;
(viii) The results of the expert‘s work in the light of auditor‘s overall knowledge of the business
and of the result of his audit procedures.

c) Effects of Changes in Foreign Exchange rates: According to NAS 11-Para 24, the foreign
currency transactions should be initially recognized at the exchange rate prevailing on the date
of transaction. Accordingly, the asset and liability should be accounted at exchange rate
prevailing on the date of purchase. The monetary items should be reported at the exchange rate
prevailing on the close of the accounting period. The liability for capital goods purchased is a
monetary item.
If during the accounting period, if a monetary liability is settled at a rate different from the rate
at which it was initially recognized the exchange difference should be charged to P&L account
in the year of settlement.
According to NAS 11, hence, it is necessary to write off Rs, 500 thousands being exchange
differences at the date of settlement. It cannot be added to the cost of the capital. Hence, the
company is wrong in capitalizing foreign exchange differences between the amounts of initial
recognition and settlement and computing depreciation on the wrongly capitalized portion of
the asset. This needs correction by the company. Else, the auditor may qualify his report upon
relevant considerations.

6. W
rite short notes on the following (ANY TWO): (25=10)

a) Change in Accounting Policies


b) Management representation as an audit evidence
c) External Confirmation Process to obtain audit evidence at the assertion level
Answers:
a) Change in accounting policies:
- The consistency is also an accounting assumption. Therefore the accounting policies should
consistently be applied and followed from years to years.
- Change in accounting policy is permitted only if such change is to bring accounts in line with
accounting standards, provisions of law or for better presentation of financial statements.
- When change in accounting policies or method is effected, the fact of such change and its
impact on accounts must be disclosed.
- If change is made in the accounting policies which has no material effect on the financial
statements for the current period but which is reasonably expected to have a material effect in
later periods, the fact of such change should be appropriately disclosed in the period in which
the change is adopted.

b) The management representation as an audit evidence:


- During the course of an audit, management makes many representations to the auditor, either
unsolicited or in response to some specific enquiries.
- The auditor also should obtain representation from management, where considered appropriate
and necessary.

The Institute of Chartered Accountants of Nepal


7 of 85
Suggested Answers of Audit and Assurance
CAP II Examination – June 2010

- The management representation is taken to corroborate audit evidence, but representations by


management can not be a substitute for other audit evidences that the auditor could reasonably
expect to be reasonably available.
- In certain cases, where knowledge of facts is confined to management, a representation by
management may be the only audit evidence, which can reasonably be expected to be
available.
- If a management representation is contradicted by an available other audit evidence, the auditor
should examine the circumstances and, when necessary, reconsider the reliability of other
representations made by management.

c) External Confirmation Process:


Para 30 of NSA 505: When confirming confirmation procedures, the auditor should maintain
control over the process of selecting those to whom a request will be sent, the preparation and
sending of confirmation requests and the responses to those requests. Control is maintained
over communications between the intended recipients and the auditor to minimise the
possibility that the results of the confirmation process will be biased because of the
interception and alteration of confirmation requests or responses. The auditor ensures that it is
the auditor who sends out the confirmation requests, that the requests are properly addressed,
and that it is requested that all replies are sent directly to the auditor. The auditor considers
whether replies have come from the purported senders.

7. H
ow will you vouch and verify the followings (ANY TWO): (25=10)
a) Provision for income tax
b) Bank Borrowings
c) Premium paid for insurance of a Motor car.

Answers:
a) Provision for income tax
(i) Obtain the computation of income prepared by the auditee and verify whether it is as per the
Income-tax Act, 1961 and Rules made there under.
(ii) Review adjustments, expenses, disallowed special rebates, etc. with particular reference to the
last available completed assessment.
(iii) Examine relevant records and documents pertaining to advance tax, self assessment tax and
other demands.
(iv) Compute tax payable as per the latest applicable rates in the Finance Act.
(v) Ensure that overall provisions on the date of the balance sheet is adequate having regard to
current year provision, advance tax paid, assessment orders, etc.

b) Bank Borrowings
Borrowings from the banks may be either in the form of overdraft limit or fixed loans.
In each case, borrowings should be verified as follows:
1. Reconcile the balances in the overdraft or loan account with that shown in the pass books and
confirm the last mentioned balance by obtaining balance confirmation certificate from the
bank.
2. Obtain certificate from the bank showing particulars of securities deposited with the bank as
security for the loans or of the charge created on an asset / assets of the concern and confirm
the same has been correctly disclosed and duly registered with the Registrar of Companies and
recorded in the Register of Charges.

The Institute of Chartered Accountants of Nepal


8 of 85
Suggested Answers of Audit and Assurance
CAP II Examination – June 2010

3. Verify the authority under which the loan or overdraft has been raised. In case of the company,
only the Board of Directors is authorised to raise a loan or borrow from a bank.
4. In case of company, confirm restraint of Company Act regarding the maximum amount of loan
that the company can raise has not been contravened.
5. Ascertain the purpose for which the loan has been raised and the manner in which it has been
utilized and that this has not prejudicially affected the concern.

c) Premium paid on Insurance of a Motor Car


(i) Check insurance cover note issued by Insurance Company. Verify car no., period of Insurance
etc.
(ii) See that ―No claim Bonus‖ is given, where entitled, by the Insurance Company.
(iii) Ensure that proper adjustment is made for pre-paid insurance premium.

The Institute of Chartered Accountants of Nepal


9 of 85
The Institute of Chartered Accountants of Nepal
Suggested Answers of Corporate Law

CAP II Examination June 2010

Part: 'A'
1.
a) Answer the following questions with reference to Companies Act, 1956.
i) What is the procedure to be followed, when a Board meeting is adjourned for want of
quorum? 5
ii) What do you understand by passing of a resolution by circulation? 5
b) EZEE Company Ltd. is in the process of winding up and the liquidator could only recover
Rs. 1 million from sale of securities of a secured creditor of the company. The total amount
of workmen's dues is Rs. 1 million and the amount of debts due from the company to its
secured creditors is Rs. 3 million. Both the workmen and the secured creditors are claiming
preferential rights on the amount recovered. Provide your advice stating the relevant
provisions of the Companies Act, 1956? 5
c) What is the meaning of ‗Maturity of negotiable instrument‘? When such instruments
become mature under the Negotiable Instrument Act, 1881. 5
d) State the effects of an irregular allotment as provided in the Companies Act, 1956? 5

Answer

a)
i) Section 288 of the Companies Act, 1956 provides that if a Board Meeting could not be held for
want of quorum then, unless the articles otherwise provide, the meeting shall be automatically
stand adjourned till the same day in the next week at the same time and place, or if that day is a
public holiday, till the next succeeding day which is not a public holiday, at the same time and
place. It is also provided that section 285 (i.e frequency of Board meetings) shall not be deemed
to have been contravened merely by reason of the fact that a meeting of the Board which had
been called in compliance with the terms of that provision, could not be held for want of
quorum.

ii) Section 289 of the Companies Act, 1956 provides for passing a resolution by circulation. No
such resolution shall be deemed to have been duly passed by the Board or Committee thereof of
a Company, unless the following conditions are complied with, viz:
a) the draft resolution, together with supporting papers has been circulated, to all the directors,
or members of the Committee of the quorum for a Board meeting.
b) to all directors of the Board or member of the Committee who are not in India, at their usual
address in India, and
c) the same has been approved by such of the directors, then in India, or by a majority of them,
who are entitled to vote on the resolution.

b) Section 529A. Overriding preferential payments.


(1)Notwithstanding anything contained in any other provision of this Act or any other law for the
time being in force, in the winding up of a company-
(a) workmen's dues; and
(b) debts due to secured creditors to the extent such debts rank under clause
Suggested Answers of Corporate Law
CAP II Examination – June 2010

(c) of the proviso to sub-section (1) of section 529 pari passu with such dues,
shall be paid in priority to all other debts.
(2) The debts payable under clause (a) and clause (b) of sub-section (1) shall be paid in full unless
the assets are insufficient to meet them, in which case they shall abate in equal proportions.
Considering the above mentioned provisions, the aggregate of the amount of workmen's dues
and of the amounts of debts due to secured creditors is Rs. 4.00 lac. The workmen's portion of
the security is, therefore, one-fourth of the value of the security, that is Rs. 25,000 only and the
balance Rs. 75,000 belongs to the secured creditors.

c) Maturity means the date on which payment of an negotiable instrument falls due. A
cheque is become mature immediately after its execution . However, section 22 of Negotiable
Instrument Act,1881 states that every promissory note or bill of exchange expressed to be payable
on a specified day, or at a certain period after date or after sight or at a certain period after the
happening of an event which is certain to happen is at maturity on the third day after the day on
which it is expressed to be payable

d) Effects of an irregular allotment as provided by Section 71 of the Companies Act 1956 are
as follows:
Section 71
(1) An allotment made by a company to an applicant in contravention of the provisions of
section 69 or 70 shall be voidable at the instance of the applicant –
(a) within two months after the holding of the statutory meeting of the company, and not later,
or
(b) in any case where the company is not required to hold a statutory meeting or where the
allotment is made after the holding of the statutory meeting, within two months after the date of
the allotment, and not later.
(2) The allotment shall be voidable as aforesaid, notwithstanding that the company is in course
of being wound up.
(3) If any director of a company knowingly contravenes, or willfully authorizes or permits the
contravention of, any of the provisions of section 69 or 70 with respect to allotment, he shall be
liable to compensate the company and the allottee respectively for any loss, damages or costs
which the company or the allottee may have sustained or incurred thereby:
Provided that proceedings to recover any such loss, damages or costs shall not be commenced
after the expiration of two years from the date of allotment.

Part: 'B'
2.
a) G. Upadhaya & Co. the auditor of XYZ Company in course of its audit comes through mass
irregularities in the accounts of the company but due to certain reasons it is unable to provide an
adverse opinion. The auditors feel that the shareholders need to know about the matter hence
requested the Board of the Company to convene an Extraordinary General Meeting (EGM). The
Board refuses to convene the meeting on the ground that the auditor can present its observations in
the audit report which shall be placed at the AGM. Provide your opinion whether the audit firm can
convene an EGM, clearly stating the circumstances for conducting an EGM as provided in the
Companies Act, 2063. 9

The Institute of Chartered Accountants of Nepal


11 of 85
Suggested Answers of Corporate Law
CAP II Examination – June 2010

b) Mr. Jack, an American National desirous to make investment in water rafting business in
Kaligandaki River. He comes before you to take a consultancy regarding registration of such
business and visa for carrying out research with the objective of making investment. What do you
suggest? 6

Answer
a) The circumstances in which and EGM can be convened as specified in the Companies Act 2063
are as follows:
Extra-ordinary General Meeting (Section 82):
(1) The board of directors of a company may convene an extra-ordinary general meeting if it deems
necessary.
(2) If, in the course of examining the accounts of a company, it is deemed necessary to call an
extraordinary general meeting for any reason, the auditor may request the board of directors to
call such meeting; and if the board of directors fails to call the meeting accordingly, the auditor
may make an application, setting out the matter, to the Office; and if an application is so made,
the Office may call the extra-ordinary general meeting of the company.
(3) If the shareholders holding at least ten per cent shares of the paid-up capital of a company or at
least twenty five per cent shareholders of the total number of shareholders make an application,
setting out the reasons thereof, to the registered office of the company for calling an
extraordinary general meeting, the board of directors shall call the extra-ordinary general
meeting of the company.
(4) If the board of directors does not call the extraordinary general meeting within thirty days from
the date on which an application is made pursuant to sub-section (3), the concerned
shareholders may make a petition to the office setting out the matter; and if such petition is
made, the Office may cause to call such meeting.
(5) If the Office deems necessary to call an extraordinary general meeting in view of the findings of
any inspection or investigation or for any other reason, it may itself call or cause the board of
directors to call such meeting.
Considering the provisions of Section 82 (2) as mentioned above the auditor can approach the
Office of the Company Registrar for convening the EGM to report the irregularities to the
shareholders.

b) Under section 3 of the Foreign Investment and Technology Transfer Act, 2049.
Permission to be Obtained:
(1) Permission of the Department shall be required to be obtained for foreign investment or
technology transfer.
(2) A person desiring to avail the foreign investment or technology transfer shall be required
to make an application to the Department in the prescribed form along with the prescribed
particulars for obtaining permission in that behalf.
(3) If an application is made pursuant to Sub-section (2) the Department shall, in the case of
an industry with fixed assists up to five hundred million rupees, itself, and in the case of an
industry with fixed assets in excess thereof, in accordance with the decision of the Board,
grant permission within thirty days from the date of application. The Department shall
communicate the decision made in regard to such permission to the applicant.
(4) Notwithstanding anything contained in Sub-section (1) and (2) no permission shall be
granted for making foreign investment in the industries set forth in the Annex.

The Institute of Chartered Accountants of Nepal


12 of 85
Suggested Answers of Corporate Law
CAP II Examination – June 2010

Under Annex of the Act, Water Rafting Industries shall not be granted permission for making
foreign investment.
Under section 6(1) A foreign national visiting Nepal in connection with undertaking any study
or carrying out any research with the objective of making investment in Nepal shall be provided
a non tourist visa for up to six months.
But Mr. Jack is desirous to make investment which is prohibited under the act. He shall not be
allowed visa for carrying out such research.

3.
a) Enumerate the circumstances under which the Securities Board of Nepal may revoke a license
issued under the Securities Act, 2063. 5
b) Who are directors? Discuss the provision of Companies Act, 2063 relating to voting on election
of director. 5

Answer
a) Revocation of License by the Nepal Securities Board (Section 89):
(1) The Board may, on any of the following circumstances, revoke a license obtained by any
stock exchange or securities business (the licensee) to operate the stock exchange or
securities business under this Act:-
(i) If the licensee stops operating the stock exchange or securities business,
(ii) If the licensee operates the stock exchange or securities business in contrary to the interest
of investors,
(iii) If the licensee violates the terms set forth in the license,
(iv) If the licensee violates any provision of this Act or the rules and bye-laws framed under this
Act,
(v) If the licensee fails to comply with any order or direction issued by the Board,
(vi) If the licensee becomes insolvent being unable to repay its dues to its creditors,
(vii) If the company or body having obtained a license to operate the stock exchange or
securities business is wound up,
(viii) If the licensee having obtained a license to operate the stock exchange or securities
business makes an application for the revocation of the license,
(ix) If the securities business having removed the agent appointed by it does not appoint another
agent in lieu of such an agent,
(x) If the licensee fails to submit such financial and transaction related statements as required to
be submitted to the Board,
(xi) If the licensee fails to pay such fees as required to be paid pursuant to Section 50 to the
Board within the specified time limit.

(2) No revocation of a license made pursuant to subsection (1) shall have any effect on any
liabilities arising from any activities done by such a stock exchange or securities business
person prior to the cancellation of license.

b) Though a company is legal entity in the eye of the law, it cannot, act in its own person. It
must act through human agency. Persons through whom it acts and does its business would be its
agents. Such persons are usually called directors. They are appointed by shareholders. Under

The Institute of Chartered Accountants of Nepal


13 of 85
Suggested Answers of Corporate Law
CAP II Examination – June 2010

Section 2(y) of Companies Act, 2063 ―Director‖ means any director of a company and this term
includes any alternate director.
Section 72 of the Companies Act, 2063 (Provision on voting in election of director):
(1) Except as otherwise provided in the articles of association, on a poll in election of directors,
every shareholder shall be entitled to cast such number of votes as may be set after multiplying the
number of shares held by him/her by the number of directors to be appointed; and the director who
casts such votes may cast all his/her votes for a single candidate or may cast votes in a manner that
his/her votes are divided for more than one candidate as indicated by him.

(2) A corporate body entitled to appoint a director pursuant to this Act or articles of association may
appoint directors in proportion to its share holding and in such a case, it shall but be entitled to cast
vote in the election.
Provided, however, that a corporate body which is not able to appoint even a single director in
proportion to the number of shares and the total number of directors or which fails to appoint a
director in exercise of the power conferred by this Sub-section may, like other shareholder, take part
in the election of directors representing shareholders, cast vote or file candidacy up to the number of
directors that can be elected in proportion to the shares held by that body in such election.

4.
a) What do you understand by the term Government office? What are the matters to be audited in
view of propriety? Explain with reference to Audit Act, 2048. 5
b) The board of directors of M/s XYZ Ltd. having net worth of Rs. 125 million (paid up capital Rs.
100 million plus free reserves Rs. 25 million) intend to provide a loan of Rs. 100 million to BC
Ltd. Mr. Y one of the shareholders of XYZ Ltd., upon coming to know that M/s BC Ltd. is not
financially stable, had reported to the board of M/s XYZ Ltd. however they took no attention to
his complaint. Provide your opinion to Mr. Y regarding the validity of transaction between
companies as per the provisions of the Companies Act, 2063. 5
Answer
a) The term "Government Office" means all government office and courts, including the Supreme
Court, parliament, Commission for investigation of Abuse of Authority, Auditor General,
Public Service Commission and offices of other Constitutional bodies and Nepal Army and
Nepal Police.

The Auditor General shall audit following maters considering the propriety
thereof:
i) On the propriety of any expenditure and authorizations, if the opinion of the Auditor
General such expenditure is a reckless one or is an abuse of national property whether
movable or immovable, despite that the expenditure confirms to the authorization and
ii) On the propriety of all authorizations issued in respect of any grant of national property
whether movable or immovable, fixed or current or underwriting of any revenue, or any
contract, license or permits relating to mining, forest, water resources etc and any other act
of abandoning movable or immovable assets of the nation.
iii) The Auditor General may not include in the report minor items of discrepancy and other
items deemed as insignificant in view of their property which were observed during the
audit of income and expenditure.
The Institute of Chartered Accountants of Nepal
14 of 85
Suggested Answers of Corporate Law
CAP II Examination – June 2010

b)Restriction on transaction between companies (Section 176 of Companies Act, 2063):


(1) No company shall, whether directly or indirectly, lend money to another company in excess of
an amount that is sixty per cent of its paid-up capital and free reserves or an amount to be set by
hundred per cent of its free reserves, whichever is higher, or give guarantee for a loan borrowed
by another company or make investment in the securities of another company in excess of the
said amount.
(2) Provided, however, that this provision shall not apply to a company carrying on banking or
financial transaction, insurance company, company with main object to buy and sell securities,
private company which has not borrowed any loan from a bank or financial institution, company
with object to provide infrastructure facility, and investment made by the holding company in its
fully owned subsidiary company, moneys lent by such holding company to such subsidiary and
guarantee given by such holding company for a loan borrowed by such subsidiary and
investment made in the right shares issued under this Act.
(3) Subject to sub-section (1), a company shall maintain the details, as prescribed, on the moneys
lent by it to another company, investment made by it in such company or guarantee given by it
for a loan borrowed by such other company.
Considering the above mentioned provisions of Section 176 Mr. Y can lodge a complaint to the
Office of the Company Registrar regarding violations of the Act and upon making an inquiry
into the matter the Office can give necessary directive under Section 178 to the company,
directors, officer or employees to refrain from doing acts in contravention of the Companies
Act.

5.
a) State the Objectives, Functions, Duties and Powers of the Nepal Rastra Bank as provided in the
Nepal Rastra Bank Act, 2058. 9
b) Sagarmatha Enterprises, an industry established in the year 2060 being operated in loss till date.
Mr. Luckless, promoter of the industry thinks that if the industry is declared sick industry and
more facilities would be obtained and the industry would be exempted tax on the raw material
to be imported from India. 6
i) Can such industry be declared as sick industry?
ii) Who has the right to declare the industry as sick industry?
iii) Whether the contention of Mr. Luckless regarding exemption of tax is tenable?

Answer
a)
Objectives of the Bank (Section 4):
(1) The objectives of the Bank shall be as follows:-
(a) To formulate necessary monetary and foreign exchange policies in order to maintain the stability
of price and balance of payment for sustainable development of economy, and manage it;
(b) To promote stability and liquidity required in banking and financial sector;
(c) To develop a secure, healthy and efficient system of payment;
(d) To regulate, inspect, supervise and monitor the banking and financial system; and
(e) To promote entire banking and financial system of Nepal and to enhance its public credibility.

The Institute of Chartered Accountants of Nepal


15 of 85
Suggested Answers of Corporate Law
CAP II Examination – June 2010

(2) The Bank shall, without any prejudice to the objectives referred to in subsection (1), extend co-
operation in the implementation of the economic policies of the Government of Nepal.

Functions, Duties and Powers of the Bank (Section 5):


(1) In order to achieve the objectives referred to in section 4, the functions, duties and powers of the
Bank shall be as follows:
(a) To issue bank notes and coins;
(b) To formulate necessary monetary policies in order to maintain price stability and to implement
or cause to implement them;
(c) To formulate foreign exchange policies and to implement or cause to implement them;
(d) To determine the system of foreign exchange rate;
(e) To manage and operate foreign exchange reserve;
(f) To issue license to commercial banks and financial institutions to carry on banking and financial
business and to regulate, inspect, supervise and monitor such transactions;
(g) To act as a banker, advisor and financial agent of Government of Nepal;
(h) To act as the banker of commercial banks and financial institutions and to function as the lender
of the last resort;
(i) To establish and promote the system of payment, clearing and settlement and to regulate these
activities; and
(j) To implement or cause to implement any other necessary functions which the Bank has to carry
out in order to achieve the objectives of the Bank under this Act;

(2) While exercising the powers conferred by this Act or any other prevailing law, the Bank shall
have the power to carry out other functions and take actions, which are incidental thereto.

(3) No one shall violate powers conferred on the Bank under this Act.

b) Section 25a of the Industrial Enterprise Act, 2049 (Provisions relating to Sick Industries):
(1) If any industry is being operated in loss for a consecutive period of five years and its production
level is twenty percent or less than twenty percent of the total production capacity, Government of
Nepal may, if it deems necessary, declare it a sick industry by notification published in the Nepal
Gazette.
(2) No duty, fee and tax of any kind shall be levied on the machinery imported by any industry as
referred to in Sub-section (1) for the extension and diversification of such industry.
On the basis of above provision the answer can be given as follows.
i) No, as per above provision If any industry is being operated in loss for a consecutive period of
five years and its production level is twenty percent or less than twenty percent of the total
production capacity can only be declared as sick industry.
ii) Government
iii) No, because exemption shall be obtained only in case of import of machinery.

6.
a) How the council of the Institute of Chartered Accountants of Nepal (ICAN) takes a decision for
the report received from Disciplinary Committee? 5
b) Under what circumstances, the registration certificate of insurers cannot be renewed. Explain in
light of relevant provisions of Insurance Act, 2049. 5

The Institute of Chartered Accountants of Nepal


16 of 85
Suggested Answers of Corporate Law
CAP II Examination – June 2010

Answer
a) Rule 84 of the Nepal Chartered Accountants Rules, 2061 (Report to be submitted):
(1) The disciplinary committee shall have to submit a report in the council on the proceedings of
the investigation carried on the complaint filed against a member or an accounting firm in
accordance with rule 81 with its recommendations.

2) After being reported according to sub-rule (1), the council shall make a decision on the
complaint following the procedures written below:
i) The council shall inform the complainer and order to replete the complaint if recommended to
so by the disciplinary committee.
ii) The council sending the alleged member or accounting firm a copy of the report shall, once
again grant an opportunity according to sub section (7) of section 14 of the Act to defend of his
own or by appointing a legal advisor if the disciplinary committee in its report has
recommended an action. After receiving a reply from the alleged member or accounting firm in
this connection the council can direct the disciplinary committee for re-reporting performing
more investigation on the subject if it feels essential.
iii) After the completion of the procedure according to sub rule (ii) if a member or an accounting
firm is found guilty the council shall order punishment in accordance with sub-section (5) of the
section 14 of the Act.

b)
(1) Notwithstanding anything contained in section 11 of the Insurance Act, 2049, the Board shall
not renew the Certificate of Registration of an insurer in any of the following circumstances:
i) In case it does not submit the balance sheet according to section 23.
ii) In case it does not submit the statement of income according to section 24.
iii) In case it does not submit the Auditor's report according to section 25.
iv) In case it does not submit the Actuary's report according to section 26.
v) In case it does not pay the insurance service fee according to section 40.
vi) In case it has been prohibited from engaging in the insurance business according to section 12A.

(2) In case the certificate of registration of an insurer cannot be renewed because of any of the
circumstances mentioned in the above, the Board shall notify the insurer accordingly within 15 days
from the date of emergence of such circumstances.

(3) In case the insurer submits an application to the Board within 15 days from the date of receipt of
a notice under section (2) by explicitly mentioning appropriate reason for its inability to fulfill the
obligations to be fulfilled by it under section 23, 24, 25, 26 and 40, the Board may it considers the
reasons to be appropriate, provide an additional time limit of not more than one month to fulfill
those obligations.

7. Write short notes on the followings: (3×5=15)


a) "Standard Form of Contract"
b) Welfare Fund under Labour Rules, 2049
c) Privileges of a "holder in due course"

The Institute of Chartered Accountants of Nepal


17 of 85
Suggested Answers of Corporate Law
CAP II Examination – June 2010

Answer
a) Where the term and condition of agreement has already been fixed in a standard form and the
other party of the agreement has to accept those term and condition such types of contract is
called 'Standard Form of Contract'. In the modern age some persons, institutions or
establishments such as the Railway, insurance companies, Bank, Manufacturers of various
goods, etc. may have to enter into a very large numbers of contracts with thousands of persons,
they cannot possibly negotiate individually with the persons with whom the contracts are to be
made. Contracts with pre-drafted matters are generally prepared by one party, which the other
has to agree to it or leave it.

b) The provisions relating to welfare funds under Labour Rules are as follows:

i) The establishment must establish a welfare fund under section 37 of the Labour Act, 2048 in
order to carry out activities for the benefit and welfare of workers or employees.
ii) Seventy percent of the balance left after distributing bonus from the amount allocated for the
purpose under section 5 of the Bonus Act, 2030 shall be carried to the fund established under
above (i).
iii) The amount credited to the Fund established under sub-Rule (i) shall be deposited in an
account opened in any commercial bank.
iv)The welfare fund shall be operated in the manner prescribed by the Labour Relations
Committee under its direct control and direction. The accounts of the Fund shall be operated
through the joint signatures of the two members of the Labour Relations Committee
designated by itself.

c) The privileges of a holder in due course are as follows:


i) A person signing and delivering to another a stamped but otherwise inchoate instrument is
debarred from asserting, as against a holder in due course, that the instrument has not been
filled in accordance with the authority given by him, the stamp being sufficient to cover the
amount.
ii)In case a bill of exchange is drawn payable to the drawer's order in a fictitious name and is
endorsed by the same hand as the drawer's signature, it is not permissible for acceptor to
allege as against the holder in due course that such name is fictitious.
iii) In case a bill or note is negotiated to a holder in due course, the other parties to the bill or
note cannot avoid liability on the ground that the delivery of the instrument was conditional
or for a special purpose only.
iv)The person liable in a negotiable instrument cannot set up against the holder in due course
the defenses that the instrument had been lost or obtained from the former by means of an
offence or fraud or for an unlawful consideration.
No maker of a promissory note, and no drawer of a bill or cheque and no acceptor of a bill
for the honour of the drawer shall, in a suit thereon by a holder in due course be permitted to
deny the validity of the instrument as originally made or drawn.
No maker of a promissory note and no acceptor of a bill payable to order shall, in a suit thereon
by a holder in due course, be permitted to deny the payee's capacity, at the rate of the note or
bill, to endorse the same. In short, a holder in due course gets good title to the bill.

The Institute of Chartered Accountants of Nepal


18 of 85
The Institute of Chartered Accountants of Nepal
Suggested Answers of Cost Accounting

CAP II Examination June 2010

1. HB Ltd. is a small manufacturing company which produces a range of three special cashmere
sweaters under the brand-names of Alpine, Border and Island.
For the month of Shrawan, the management accountant has prepared the following forecast
of trading results:
Alpine Border Island Total
Rs. Rs. Rs. Rs.
Sales 1,000,000 960,000 320,000 2,280,000
Variable costs
Direct materials 350,000 300,000 100,000 750,000
Direct labour 50,000 80,000 30,000 160,000
Works overhead 200,000 180,000 110,000 490,000
Fixed overhead 300,000 270,000 100,000 670,000
(Apportioned)
Net profit/ (Loss) 100,000 130,000 (20,000) 210,000
The following information is also available:
 A
lthough there are other materials which are required for the manufacture of sweaters, the
major material is cashmere, purchased directly from China. Unfortunately, due to delays
in shipment from China, production for the month of Shrawan will be limited by the
availability of cashmere supplies. The purchasing manager believes that he will be able to
increase the current stockholding of 61,000 kg by a delivery of 31,000 kg in mid-
Shrawan. There are no substitute suppliers in the short term.
 T
he production manager advises that the different products absorb different quantities of
cashmere: Alpine- 8 kg per unit, Border- 4 kg per unit and Island- 1 kg per unit.
 T
he sales manager confirms that the current unit prices of the products are: Alpine- Rs.100,
Border- Rs.120 and Island- Rs.80. He is convinced that sales of the Border product could
be substantially increased beyond the current forecast, although he is unable to quantify
the effect. Extra advertising of Rs. 80,000 would be required to achieve this increase,
together with a 10% reduction in the price of the Alpine product. The current forecast of
trading results does not include the view.
 T
he managing director has reviewed the forecast for Shrawan and believes that results can
be improved immediately through stopping the manufacture of the Island product.
Neither the sales manager nor the production manager agrees with his view, but they are
not certain why they disagree.
 F
ixed overhead is apportioned over the product lines on the basis of an allocation by space
occupied on the factory floor.

The managing director is confused by the different proposals being put forward and he
seeks your services as the company‘s external financial consultant to assist him in
drawing up a sensible plan of action.

You are required to do the following: (3+9+5+3=20)


Suggested Answers of Cost Accounting
CAP II Examination – June 2010

a) Assuming that the full sales forecast for Shrawan can be achieved, assess the impact of
the managing director‘s proposal to drop the entire Island range and advise on its
desirability.
b) In view of the shortage of cashmere, assist the company by preparing an optimal
production plan and a revised forecast of trading results for Shrawan.
c) In light of (b) above, assess the viability and effects of the sales manager‘s plans to
increase sales of the Border range through extra advertising expenditure.
d) What other points of commercial interest would you wish to draw to the attention of the
managing director?

Solution to the Question No. 1

(a) The managing director believes that the act of dropping the Island range of products will
increase profits by Rs.20,000; since it seems that the loss forecast would be avoided.
Unfortunately, it is necessary to appraise him (as tactfully as possible) that his thinking is
being unduly influenced by the arbitrary allocation of Rs.100,000 of fixed overheads to
the Island range. If the Island range was stopped, then Rs.100,000 of fixed overhead
would not be saved, but merely reallocated to the other two lines.
The correct approach is to assess the contribution to fixed overheads arising from the
Island product, as follows:

Statement of contribution of Island


Particulars Rs. Rs.
Sales 320,000
Less: Variable Costs
Direct materials 100,000
Direct labour 30,000
Works overhead 110,000 240,000
Contribution 80,000

In fact, therefore, if this proposal had been adopted, the forecast trading profit of
Rs.210,000 would have fallen to Rs.130,000 (i.e. Rs.210,000-Rs.80,000) due to above
lost contribution. Hence, this proposal should be soundly rejected.

(b) Since the shortage of cashmere constitutes a limiting factor on Shrawan‘s output, it is
necessary to deploy the available cashmere over the three products in a manner which
will optimize overall profit. In the first instance, it is necessary to identify the ranking of
the products, in terms of the contribution they generate from use of per kg of cashmere:

The Institute of Chartered Accountants of Nepal


20 of 85
Suggested Answers of Cost Accounting
CAP II Examination – June 2010

Statement of contribution per kg of cashmere used


Particulars Alpine Border Island
Rs. Rs. Rs. Rs. Rs. Rs.
Sales 1,000,000 960,000 320,000
Less: Variable Costs
Direct materials 350,000 300,000 100,000
Direct labour 50,000 80,000 30,000
Works overhead 200,000 600,000 180,000 560,000 110,000 240,000
Contribution 400,000 400,000 80,000
Unit Selling Price (Rs.) 100 120 80
Therefore, no. of units 10,000 8,000 4,000
Contribution per unit (Rs.) 40 50 20
Cashmere Used per unit (kg) 8 4 1
Contribution per kg of 5 12.50 20
cashmere (Rs.)
Ranking III II I

The ranking exercise indicates that the Island product (contrary to the managing director‘
belief) is the best product in the current situation of limited supplies of cashmere.
As a result, the optimal production plan is:

Statement showing Revised optimal production plan for Shrawan


Product as Maximum Cashmere Cashmere Remaining Revised Contribution Total
per ranking forcasted required to to be used quantity of optimal per unit (Rs.)
sales unit meet (kgs) from cashmere production (Rs.)
maximum 92000 kgs (kgs) and sales
forcasted available unit
sales unit
(kgs)
Island 4000 4000 4000 88000 4000 20 80000
Border 8000 32000 32000 56000 8000 50 400000
Alpine 10000 80000 56000 0 7000 40 280000
Total revised contribution 760000
Total Fixed costs 670000
Total revised profit 90000

Note: As the third-ranked product, Alpine received only the residual balance of the
available cashmere, i.e.56,000 kgs. This means that only 7,000 units can be
manufactured, leaving an unsatisfied demand for 3000 units in Shrawan.

(c) Assessment of the effect of Sales Manager's Plan


The sales manager is not clear about the extent to which the sales of Border could be increased.
In any case, production/sales of Border can be increased at the expense of Alpine which has the
lowest contribution per kg. of cashmere of Rs. 5 keeping the sales units of Island intact at the
level of revised production plan as found out under (b) above. As per the conditions given, the
sales unit of Border will have to be accompanied by a 10% reduction in the price of Alpine, the
contribution per unit of Alpine will be reduced to Rs. 30 per unit.

The Institute of Chartered Accountants of Nepal


21 of 85
Suggested Answers of Cost Accounting
CAP II Examination – June 2010

Considering the additional advertisement cost of Rs. 80,000 to increase the sales of Border, a
total contribution of at least Rs. 840,000 (Contribution as per revised plan of Rs. 760,000 +
Advertisement cost of Rs. 80,000) is needed to make the sales manager's plan financially
justifiable.
Let x be the increase in the sales units of Border from the units of revised optimum level derived
in (b). Based on this, the total contribution generated by the respective products is computed in
the table given below.
Cashmere Contribution
Product Sales Units
Required (kg.) Per Unit (Rs.) Total (Rs.)
Island 4,000 4,000 20 80,000
Border 8,000 + x 32,000 + 4x 50 400,000 + 50 x
Alpine 7,000 – ½ x** 56,000 – 4 x* 30 210,000 – 15 x
Total: 43,000 + ½ x 92,000 690,000 + 35 x

In order to ensure that the sales manager's plan is financially viable, the contribution derived in
the above table should be greater than Rs. 840,000.
Therefore, Rs. 690,000 + 35 x > 840,000, Or, 35 x > 840,000 – 690,000
Or, x > 150,000/35 Or, x > 4,285.71, say, 4,286 units.

Thus, the sales units of Border should be 12,286 (8,000 + 4,286) or 53.575% more than sales
units derived under option (b) for the sales manager's plan to yield at least the same level of
profit as obtained under that option. Any increase in the sales units of Border in excess of that
level by reducing the sales units of Alpine will make the sales manager's proposal financially
better as compared to the revised optimum production/sales plan.

(d) Other possible points of commercial interest are:


 What can be done to increase the sales of the Island range since it is the best performing
product?
 Is it possible to increase the selling prices of all the products as a means to restrict demand
to attainable levels?
 Is there any impact on sales caused by the restricted availability of any one range of
product?
 Would any customers be prepared to wait for deliveries until cashmere is in greater supply?

2.
a) Jagger Products Ltd. manufactures toy dolls in a moulding process. The management
accountant has been taken ill and you have been requested to assist in identifying the
usual monthly adjustments to measure output and cost of output.
Opening work-in-progress amounted to 8000 dolls, with costs attached of materials Rs.
398,400 and labour/conversion of Rs. 384,000. Materials are added at the beginning of
the moulding process. Opening work-in-progress is 50% complete in terms of
labour/conversion costs.
During the month, a further 40000 dolls have been input to the process, incurring material
costs of Rs. 2,400,000 and labour/conversion costs of Rs. 3,993,600. Completed dolls

The Institute of Chartered Accountants of Nepal


22 of 85
Suggested Answers of Cost Accounting
CAP II Examination – June 2010

totaled 42000 in the month, leaving 6000 in work-in-progress, 60% complete in terms of
labour/conversion costs.
The company operates a FIFO approach to accounting for stock movements.
You are required to compute/prepare: (4+2+2+2=10)
i) Equivalent unit statement and cost per equivalent unit;
ii) Costs attached to closing work-in-progress;
iii) Costs attached to units started and completed;
iv) Costs attached to completion of opening work-in-progress.

b) Component CT is made entirely in a machine shop. Material cost is Rs. 15 per


component. Each component requires 4 minutes to produce and the machine operator is
paid Rs. 90 per hour. Machine hour rate is Rs. 360 per hour.
It takes 2 hours for the operator to set up the machine before the production of
components can take place.
You are required to prepare cost sheet showing the setting up costs and production costs,
both in total for the batch and per component, assuming a batch size of 500 components. 5

c) Nepal Travels runs 20 buses in various routes of Kathmandu and Dhulikhel which are 25
kilometer apart. Seating capacity of each bus is 40 passengers. The expenses for the
month of Chaitra 2066 were as follows:
Salaries of the drivers and conductors : Rs. 2,60,000
Salaries of mechanical staff : Rs. 30,000
Diesel oil and lubricants : Rs. 4,80,000
Taxes and Insurance : Rs. 28,000
Repairs and Maintenance : Rs. 1,04,000
Depreciation : Rs. 4,18,000
Seating capacity utilized was 60%. All the buses ran 25 days of the month. Each bus
made four round-trips.
You are required to find:
Cost per passenger kilometer and cost per round trip per passenger. 5

Solution to the Question No. 2 a)

The Institute of Chartered Accountants of Nepal


23 of 85
Suggested Answers of Cost Accounting
CAP II Examination – June 2010

Equivalent unit statement


Particulars Materials Labour/ Remarks
Conversion
Units Units
Opening work-in-progress 0 4000 (8000 units 50% complete in
terms of labour/ conversion)
Units started and completed 34000 34000 (40000 units less closing WIP of
6000 units on FIFO basis)
Closing work-in-progress 6000 3600 (6000 units 60% complete in
terms of labour/ conversion)
Equivalent units 40000 41600
Rs. Rs. Total (Rs.)
Costs to be accounted for 2,400,000 3,993,600 6,393,600
incurred in the month
Costs per equivalent unit 60 96 156

(ii)
Costs attached to closing work-in-progress
Particulars of costs Equivalent Rate per Total costs (Rs.)
Unit Unit (Rs.)
Materials 6000 60 360,000
Labour/ conversion 3600 96 345,600
Total 705,600

(iii)
Costs attached to unit started and completed
Particulars of costs Equivalent Rate per Total costs (Rs.)
Unit Unit (Rs.)
Total costs 34000 156 5,304,000

(iv)
Costs attached to completion of opening work-in-progress
Particulars of costs Equivalent Rate per Total costs (Rs.)
Unit Unit (Rs.)
Materials (already complete) 0
Labour/ conversion 4000 96 384,000
Total 384,000
Note:
In completing opening work-in-progress, costs incurred last month and brought forward
(Materials Rs.398,400 and Labour/ conversion Rs.384,000) will also be transferred to finished
goods.

Solution to the Question No. 2 b)


A. Setting up cost: Total Batch Cost per
Cost Component
Wages (2 hours @ Rs. 90) Rs. 180

The Institute of Chartered Accountants of Nepal


24 of 85
Suggested Answers of Cost Accounting
CAP II Examination – June 2010

Machine Expenses (2 hours @ Rs. 360 per hour) 720


Total Setting up cost: 900 1.80
B. Production cost
Material (500 X Rs. 15) 7,500
Wages (500 X 4 minutes X Rs. 90/60) 3,000
Machine Expenses (500 X 4 minutes X 360/60) 12000
Total Production Cost 22,500 45.00
Total Setting up and Production Cost 23,400 46.80

Solution to the Question No. 2 c)

Calculation of Cost per Passenger Km. & Cost per round trip
Since the two destinations are 25 kms. apart, a round trip will cover 50 kms.
Four round trips are made for 25 days
Total running in a month = 50 kms. X 4 trips X 25 days X 20 buses = 100,000 Kms
Seating Capacity utilized = 60% or 24 passengers
Therefore, passenger kilometer run = 24 X 100,000 = 2,400,000
Total cost per month = Rs. (260,000 + 30,000 + 480,000 + 28,000 + 104,000 +
418,000)
= Rs. 1,320,000
Cost per passenger kilometer = Rs. 1,320,000/Rs. 2,400,000 = Rs. 0.55
Cost per round trip per passenger = 50 X Rs. 0.55 = Rs. 27.50

3.
a) S
V Limited which has established its product K furnishes the following forecast of sales in
units for 2009.
I Quarter II Quarter III Quarter IV Quarter
Units 12,000 15,000 13,500 9,000
The opening stock on 1.1.2009 is expected to be 10,000 units and the company proposes to
maintain a closing stock of 4,500 units on 31.12.2009. The rejection in the process of
manufacture of product K is 12% and the production will be spread out uniformly throughout the
year.
Two raw materials C and D are used for the manufacture of product K. At present the company
orders inventory of the two raw materials in quantities equivalent to 13 weeks‘ consumption. The
management of the company has been advised that considerable economies in provisioning of
raw materials can be effected by changing over to the ordering system based on economic order
quantities. The Materials Manager has complied the following data:
C D
Raw material quantity required per unit of output of K (kg) 2.4 4.2
Raw material usage rate/week (kg) 2,300 4,000
The Institute of Chartered Accountants of Nepal
25 of 85
Suggested Answers of Cost Accounting
CAP II Examination – June 2010

Price per kg (Rs.) 2.00 4.00


Lead time to obtain deliveries (weeks) 5 3
Order costs per order (Rs.) 10.00 10.00
Carrying Costs 20% 25%
You are required to: (4+4+4+3=15)
i) Prepare a Production and Raw Material Requirement Budget for the year 2009.
ii) Calculate the Economic Order Quantity for raw materials C & D using Tabular
Method assuming order sizes of 1,200, 1,800, 2,400 and 3,000.
iii) The company feels that a safety stock should be built up to cover a lead time of 8
weeks and 5 weeks respectively for C and D and increase in the usage of raw
materials up to 3,000 kg and 5,000 kg respectively for C and D per week. Calculate
the ordering level to meet the above requirement.
iv) Based on the ordering level calculate at (iii) above, find the saving arising from
switching over to the new ordering system.

b) What is integral accounting? Briefly describe its main features. 5

Solution to the Question No. 3 a)


i (a) Annual Production budget (quantitative)
Units
Total annual sales (12,000 +15,000 + 13,000 + 9,000) 49,500
Add: desired closing Stock 4,500
54,000
Less: Expected Opening Stock 10,000
Production of good units 44,000
Rejection 12%
Units to be produced 44,000 x (100/88) 50,000

(b) Material Budget (Quantitative)


C D
Quantity per unit 2.4 kg 4.2 kg
Material requirement per annum (kg) 120,000 210,000

ii) Computation of Economic Order Quantity (EOQ)


EOQ for C
Lot Size No of Average Ordering Inventory Total
Orders Inventory Costs Carrying Costs
Rs. Cost Rs.
Rs.
1,200 100 600 1,000 240 1,240
1,800 67 900 670 360 1,030
2,400 50 1,200 500 480 980
3,000 40 1,500 400 600 1,000
EOQ for D
1,200 175 600 1,750 600 2,350
1,800 117 900 1,170 900 2,070
2,400 88 1,200 880 1,200 2,080

The Institute of Chartered Accountants of Nepal


26 of 85
Suggested Answers of Cost Accounting
CAP II Examination – June 2010

3,000 70 1,500 700 1,500 2,200


Thus, EOQ for Material C is 2,400 units while for Material D it is 1,800 units.

iii) Computation of Revised Ordering Level


C D
Kg kg
Normal Ordering Level = Lead Time x Inventory Usage (2,300 x 5) (4,000 x 3)
= 11,500 = 12,000
Usage over increased lead time
C : 2,300 x 3 6,900
D : 4,000 x 2 8,000
Usage increase per week
C : (3,000 – 2,300) x 8 5,600
D : (5,000 – 4,000) x 5 5,000
Revised ordering level 24,000 25,000

iv) Computation of Savings due to change in system


Rs. Rs.
(a) Present : No. of orders p.a. 4 4
Average Inventory (once in 13 weeks) (120,000/4 x ½) (210,000/4 x ½)
= 15,000 = 26,250
Ordering cost 40 40
Carrying cost (15,000 x 2 x 20%) (26,250 x 4 x 25%)
= 6,000 = 26,250
Total cost 6,040 26,290
(b) Proposed : Total cost (See (ii) 980 2,070
above)
(c) Saving : (a) – (b) 5,060 24,220

Solution to the Question No. 3 b)


Under integral accounting, financial and costing transactions are recorded in one self contained
ledger called integral ledger. When integral accounting is used, there will be no need for
reconciliation of costing and financial results.
Features of Integral Accounting
The principal features of integral accounting are as follows:
i) R
ecords are maintained to undertake complete analysis of cost and sales.
ii) C
omplete details regarding all receipts and payments are kept.
iii) R
ecords to show all the details of assets and liabilities are kept. This system does not use a
notional account to represent all impersonal accounts.
In non-integral system, a cost control account or general ledger adjustment account is used in
cost ledger. In integral accounting system, this adjustment account is not used. Instead, detailed
accounts for assets and liabilities are maintained.
The Institute of Chartered Accountants of Nepal
27 of 85
Suggested Answers of Cost Accounting
CAP II Examination – June 2010

In integral system, all accounts necessary for the classification of cost is used. The use of
following accounts replaces the general ledger adjustment account as used in non-integral
accounting system:
(i) B
ank account
(ii) D
ebtors account
(iii) C
reditors account
(iv) P
rovision for depreciation account
(v) D
iscount account
(vi) F
ixed assets account
(vii) Share Capital account, etc.

4.
a) A
company is producing three types of products, A,B,C. The sales territory of the company is
divided into three areas X,Y and Z. The estimated sales for the year are as under:
Territories
X Y Z
Product Rs. Rs. Rs.
A 50,000 20,000
B 30,000 - 80,000
C - 70,000 40,000

Budgeted advertising cost is as under:


Territories
X Y Z Total
Rs. Rs. Rs.
Local cost 3,200 4,500 4,200 11,900
General - - - 5,800
You are required to find the advertising cost per cent on sales for each product and territory
showing how you will present the statement to management. 10

b) Calculate the earnings of workers A and B from the following particulars for a month and
allocate the labour cost to each job X, Y and Z: 10
A B
(i) Basic Wages (Rs.) 100 160
(ii) Dearness Allowance 50% 50%
(iii) Contribution to Provident Fund (on basic wages) 8% 8%
(iv) Contribution to Employees‘ Life Insurance (on basic wages) 2% 2%
(v) Overtime Hours 10 -

The Institute of Chartered Accountants of Nepal


28 of 85
Suggested Answers of Cost Accounting
CAP II Examination – June 2010

The Normal working hours for the month are 200. Overtime is paid at double the total of normal
wages and dearness allowance. Employer‘s contribution to life Insurance and Provident Fund are
made at the rates equal to employees‘ contributions. The two workers were employed on jobs X,
Y and Z in the following proportions:

Jobs
X Y Z
Workers A 40% 30% 30%
Worker B 50% 20% 30%
Overtime was done on job Y.

Solution to the Question No. 4 a)


General advertising cost of Rs. 5,800 is allocated to territories on the basis of sales value, as
follows:
Sales Value General advertising cost

Territory X: Product A 50,000 1,000


Product B 30,000 80,000 600 1,600
Territory Y: Product A 20,000 400
Product C 70,000 90,000 1,400 1,800
Territory Z: Product B 80,000 1,600
Product C 40,000 120,000 800 2,400
Total : 5,800
Local costs allocated to territories are apportioned to products on the basis of sales value:
Territory X: Product A 50,000 2,000
Product B 30,000 80,000 1,200 3,200
Territory Y: Product A 20,000 1,000
Product C 70,000 90,000 3,500 4,500
Territory Z: Product B 80,000 2,800
Product C 40,000 120,000 1,400 4,200
Total: 11,900

Presentation of advertisement cost to the management will be made in the following statement:
Territories
X Y Z Total % on Sales
Product A 3,000 1,400 - 4,400 6.28
Product B 1,800 - 4,400 6,200 5.64
Product C - 4,900 2,200 7,100 6.45
Total 4,800 6,300 6,600 17,700 6.10
% on Sales 6.00 7.00 5.50 6.10

The Institute of Chartered Accountants of Nepal


29 of 85
Suggested Answers of Cost Accounting
CAP II Examination – June 2010

Solution to the Question No. 4 b)

Statement Showing Earnings of Workers A and B


Workers: A B
Rs. Rs.
Basic Wages 100 160
Dearness Allowance (50% of Basic Wages) 50 80
Overtime Wages (Refer to Working Note 1) 15 -
Gross Wages earned 165 240
Less: - Provident Fund – 8% of Basic wages and ELI – 10 16
2% of Basic wages
Net Wages paid 155 224
Statement of Labour Cost: Rs. Rs.
Gross Wages (excluding overtime) 150 240
Employer‘s Contribution to P.F. and E.L.I. 10 16
Ordinary wages 160 256
Labour Rate per hour 0.80 1.28
(Rs. 160/200) (Rs. 256/200)

Statement showing allocation of Wages to Jobs


Jobs
Total Wages: X Y Z
Rs. Rs. Rs. Rs.
Worker A:
Ordinary Wages: (4 : 3 :3) 160 64 48 48
Overtime 15 - 15 -
Workers B:
Ordinary Wages: (5: 2 : 3) 256 128 51.20 76.8
431 192 114.2 124.8

Working Notes:
Normal Wages are considered as basic wages
Overtime = 2 x (Basic wage + D.A.)
200
= 2 × (Rs. 150/200) × 10 hours = Rs. 15/-.

5. D
istinguish between: (4×2.5=10)
a) Product costs and period costs
b) Cost control and cost reduction
c) Shift Premium and Overtime Premium
d) Treatment of spoilage and defectives

Solution to the Question No. 5 a)

The Institute of Chartered Accountants of Nepal


30 of 85
Suggested Answers of Cost Accounting
CAP II Examination – June 2010

Product costs are costs which can be identified with goods produced for resale. They vary with
production because these costs are directly affected by the production volume. If there is no
production, these costs will not be incurred. Raw materials and direct labour costs are the
examples of product costs.
Period costs are costs which are matched against the revenue of the current period. These costs
vary with the passage of time and not with the volume of production. Therefore, even if there is
no production, these costs have to be incurred. Rent, insurance, salary etc are the examples of
period costs.

Solution to the Question No. 5 b)


Cost control process involves setting standards, ascertaining actual performance, comparing
actual with standards, investigating variances, and taking corrective actions finally resulting to
cost reduction. Cost control aims at achieving standards where as cost reduction aims at
improving the standards. Cost reduction assumes existence of concealed potential savings in the
standards. It is dynamic and always looks for measures of reducing cost.
Cost control is preventive. It attempts to optimize costs before they are incurred. Cost reduction
is corrective and attempts to find scope of reducing incurred costs under controlled conditions.
Cost control requires existence of standards and management guidelines; but cost reduction is
applicable to every activity of the business. It does not require standard and provides guidelines
to management.

Solution to the Question No. 5 c)


Shift premium/differential refers to the payment of higher hourly rates for working in less
desirable shifts of job, such as evening or night shift. It is charged to factory overhead control
rather than work-in-process, and spread over all units produced because they are not caused by
specific units. For example, if day time rate per hour is Rs.50 and night time rate is Rs.60 per
hour, the shift premium per hour is Rs. 10 only, which is charged to factory overhead.
Overtime work is caused due to random scheduling of jobs, requirement of a specific job and
poor workmanship or negligence. It may be paid at the regular rate or higher rate as required. If
higher rate is paid for the overtime hours, the differential is termed as overtime premium. If it is
caused by random scheduling of jobs, it is treated as shift premium; if it is caused by requirement
of specific job, it is charged to that specific job; and if it is caused by poor workmanship or
negligence, it is charged to profit and loss account.

Solution to the Question No. 5 d)


Spoilage occurs when materials are so damaged in manufacturing operations that they are taken
out of the process and disposed off without further work.
Normal spoilage is included in cost either by charging the loss to the production order or
charging it to production overhead. The cost of abnormal spoilage is charged to costing profit
and loss account.
Defectives represent unit of output which fail to comply with a set quality standard. These can be
subsequently rectified, sold as 'seconds' or disposed off as scrap.

The Institute of Chartered Accountants of Nepal


31 of 85
Suggested Answers of Cost Accounting
CAP II Examination – June 2010

Normal defectives can be recovered; charged to good production: charged to general overhead;
or charged to department. If defectives are abnormal and are due to causes beyond the control of
organization, they should be charged to profit and loss account.

6. A
nswer the following questions: (5×2=10)
a) State the appropriate ‗cost unit‘ for the following industries:
i) Steel
ii) Bricks making
iii) Sugar
iv) Power
b) What are the assumptions involved in the definition of cost reduction?
c) Briefly explain the benefits of cost audit to the management.
d) Describe what do you understand by engineered costs?
e) What items are generally included in good uniform costing manual?

Solution to the Question No. 6 a)


Industry Cost Unit
i) Steel : Per ton
ii) Bricks making : Per 1000 bricks
iii) Sugar : Per quintal or 100 kg.
iv) Power : Kilo-watt hour

Solution to the Question No. 6 b)


The threefold assumption involved in the definition of cost reduction may be
summarized as under:
(i) There is a saving in unit cost.
(ii) Such saving is of permanent nature.
(iii) The utility and quality of goods and services remain unaffected, if not
improved.

Solution to the Question No. 6 c)


Benefits of Cost Audit to the management
i) Cost audit assists in the detection of errors and frauds. Continuous cost audit prevents
manipulation and frauds.
ii) Cost data becomes more reliable. Inventory valuation certified by the cost auditor ensures
the correctness and integrity.
iii) Cost audit facilitates improved cost accounting methods and better internal control.

The Institute of Chartered Accountants of Nepal


32 of 85
Suggested Answers of Cost Accounting
CAP II Examination – June 2010

iv) Disclosures made in the cost audit reports create cost consciousness in the management.
v) Cost audit helps to improve the effectiveness of cost control and cost presentation. This is
achieved when the cost auditor points out avoidable wasteful routines and procedures and
recommends the introduction of cost efficient routines and procedures.
vi) Cost audit assists the management to take action for economic and efficient use of labour
material and other resources. This will contribute towards achieving higher productivity
and higher utilization of capacity.
vii) Cost audited data could be helpful in comparing inter-firm performance.
viii) Cost audit could be instrumental in identifying the symptoms of sickness in a unit. This
will help the management to initiate timely remedial actions to prevent the sickness.

Solution to the Question No. 6 d)


Engineered costs are built into the product or output and an organization cannot avoid incurring
them. For example, direct material, direct labor and directly related overhead are designed into
the product and simply must be incurred if output is to be achieved.

Engineered costs should be contrasted with discretionary costs, the level of which is determined
solely by management. For example, the level of R & D, marketing and selling, and maintenance
expenditures are determined by management and are not necessarily related to the level of
productive output.

Solution to the Question No. 6 e)


Uniform costing manual includes essential information and instructions to implement accounting
procedures.
(i) Introduction: It includes objects and scope of the planning.
(ii) Accounting procedure and planning includes rules, and general principle to be followed.
(iii) Cost accounting planning includes methods of costing, relation between cost and financial
accounts and methods of integration.

The Institute of Chartered Accountants of Nepal


33 of 85
The Institute of Chartered Accountants of Nepal
Suggested Answers of Financial Management

CAP II Examination June 2010

1. Growmore Ltd. is considering two projects, A and B, to undertake. The projects are
mutually exclusive and the firm can choose any one these two. There is a controversy at the
top management level of Growmore regarding the capital budgeting technique to be
employed as the basis for selection of the investment projects.
The finance director is of the view that the project with higher net present value (NPV)
should be chosen whereas the managing director strongly feels that the one with higher
internal rate of return (IRR) should be undertaken especially when the mutually exclusive
projects have the same initial outlay and length of life.
The company anticipates a cost of capital of 10% and the net after tax cash flow of the
projects (in ‗000 rupees) are as given below:

Projects _
Year A B
0 (-) 800 (-) 800
1 140 872
2 320 40
3 360 40
4 300 16
5 80 12

You are required to: (6+3+5+6=20)


a) Calculate the NPV and IRR of each project;
b) Recommend, with reasons, which project should be undertaken (if either);
c) Explain the inconsistency in ranking of the two projects in the light of the remarks of the
directors; and
d) Identify the cost of capital at which your recommendation made in part (b) would be
reversed.
Following discount factors may be adopted:
Discount Year
Factor 0 1 2 3 4 5
At 10% 1.0000 0.9091 0.8264 0.7513 0.6830 0.6209
At 15% 1.0000 0.8696 0.7561 0.6575 0.5718 0.4972
At 20% 1.0000 0.8333 0.6944 0.5787 0.4823 0.4019
Suggested Answers of Financial Management
CAP II Examination – June 2010

Answer

(a) Computation of NPV and IRR:


Project A:

Discount Factors Cash Flow NPV at


Year 10% 15% 20%
10% 15% 20%

0 1.0000 1.0000 1.0000 – 800 – 800.00 – 800.00 – 800.00


1 0.9091 0.8696 0.8333 140 27.27 121.74 116.66
2 0.8264 0.7561 0.6944 320 264.45 241.95 222.21
3 0.7513 0.6575 0.5787 360 270.47 236.70 208.33
4 0.6830 0.5718 0.4823 300 204.90 171.54 144.69
5 0.6209 0.4972 0.4010 80 49.67 39.78 32.15
Total: 116.76 11.71 -75.96

Project B:

Discount Factors Cash Flow NPV at


Year 10% 15% 20%
10% 15% 20%

0 1.0000 1.0000 1.0000 – 800 – 800.00 – 800.00 – 800.00


1 0.9091 0.8696 0.8333 872 792.74 758.29 726.63
2 0.8264 0.7561 0.6944 40 33.06 30.24 27.76
3 0.7513 0.6575 0.5787 40 30.05 26.30 23.15
4 0.6830 0.5718 0.4823 16 10.93 9.15 7.72
5 0.6209 0.4972 0.4010 12 7.45 5.97 4.82
Total: 74.23 29.95 -9.82

From the above table, at 10% discount rate:

Formula: IRR=

NPV of Project A = Rs. 116,760


NPV of Project B = Rs. 74,230
Using interpolation method, IRR of individual projects are computed as follows:
Project A = 15% + [11.71/ 11.71 – (-75.96)] X 5%
= 15% + [11.71/ (11.71+ 75.96)] X 5%
= 15% + (11.71/ 87.67) X 5% = 15% + 0.67% = 15.67%
The Institute of Chartered Accountants of Nepal
2 of 85
Suggested Answers of Financial Management
CAP II Examination – June 2010

Project B = 15% + [(29.95/(29.95 + 9.82) X 5%]


= 15% + [(29.95/(39.77 + 9.82) X 5%]
= 15% + 29.95/ 39.77 X 5% = 15% + 3.77 % = 18.77%

Note: Using the rate of 10% and 20% rate for discounting, IRR derived for project A and B will
be 16.06% and 18.83% respectively.

(b) Recommendation on the Selection of Project:


Under NPV technique, project A has higher NPV (Rs. 116,760) as compared to project B which
has a NPV of Rs. 74,230. On the contrary, IRR of project B (18.77%) is much higher than that
of Project A which has an IRR of 15.67%.
The projects are mutually exclusive and conflicting rankings have occurred. In this situation,
NPV method will indicate the correct rankings due to certain limitation of IRR method as
explained under point (c). It is therefore recommended that project A should be selected for
implementation since it yields the higher NPV at a discount rate of 10%.
(c) Reasons for Inconsistency in the ranking of two Projects:
Such an inconsistency in the rankings generally occurs when the cash inflow in the project with
lower NPV is heavily loaded in the earlier years. That is exactly what has happened in the case
of Project B in the present case. Exactly, 88% of the cash inflow occurred in the first year in this
project whose NPV is lower as computed under point (a) above.
The superiority of NPV technique over the IRR method in such instances can be explained in
terms of the following factors:
(i) Percentage Returns: IRR expresses the results in percentage rather than in absolute or
monetary terms. Comparison of percentage can be misleading. For instance, an investment
of Rs. 500,000 that generates a return of 15 per cent is better than an investment of Rs.
200,000 which yields a return of 30 per cent. If the two projects are mutually exclusive, the
first investment will yield Rs. 75,000 but the second will only contribute Rs. 60,000
towards the profit pool of the firm. Therefore, if the objective is to maximize the
shareholders wealth, NPV is the correct measure.
(ii) Reinvestment assumptions: When NPV method is adopted, the implicit assumption is that
the cash flows generated from an investment will be reinvested at the cost of capital.
However, the IRR method assumes that all the proceeds from a project can be reinvested
to earn a return equal to the IRR of the original project. The underlying assumption of
NPV method is therefore more realistic as compared to the assumption made in IRR
method.

(d) Cost of Capital at which the Recommendation would be Reversed:


The cost of capital at which Project A would be preferred to Project B can be ascertained by
calculating the IRR on incremental investment, A – B.

Year 0 Year 1 Year 2 Year 3 Year 4 Year 5 Total


Particulars
(Rs. in ‘000)
Project A : Cash Flow – 800 140 320 360 300 80
Project B : Cash Flow – 800 872 40 40 16 12
Project (A – B) 0 –732 280 320 284 68

The Institute of Chartered Accountants of Nepal


3 of 85
Suggested Answers of Financial Management
CAP II Examination – June 2010

Discount Factor 10% 1.0000 0.9091 0.8264 0.7513 0.683 0.6209


NPV @ 10% 0 –665.46 231.39 240.42 193.97 42.22 42.54
Discount Factor 15% 1.0000 0.8696 0.7561 0.6575 0.5718 0.4972
NPV @ 15% 0 –636.55 211.71 210.40 162.39 33.81 –18.54

Using interpolation method, IRR on incremental investment


= [10% + 42.54/(42.54 + 18.24) X 5%] = 10% + 42.54/ 60.78 X 5%
= 10% + 3.5% = 13.5%
Thus, the IRR on incremental investment (A – B) is 13.5 per cent. This implies that the decision
recommended in (b) above would be reversed if the cost of capital were in excess of 13.5 per
cent assuming that of the projects has a positive NPV.

2. a) Describe Walter's approach to dividend policy along with his formulation. 5


b) X
YZ Ltd. is foreseeing a growth rate of 12% per annum in the next 2 years. The growth
rate is likely to fall to 10% for the third year and fourth year. After that the growth rate is
expected to stabilise at 8% per annum. If the last dividend paid was Rs. 1.50 per share
and the investors' required rate of return is 16%, find out the intrinsic value per share of
Z Ltd. as of date. You may use the following table: 10

Years 0 1 2 3 4 5
Discounting factor at 16% 1 0.86 0.74 0.64 0.55 0.48

Answer

a) The formula given by Prof. James. E. Walter shows how the dividend policy can be used to
maximise the wealth position of the equity holders. He argues that in the long run, the share
prices reflect only the present value of the expected dividends. Retentions influence share prices
only through their effect on further dividends. The relationship between dividend and share price
can be shown on the basis of the following formula:

Vc = D + Ra/Rc (E - D)
Rc
where,
Vc = Market value of the ordinary shares of the company
Ra = Return on internal retention i.e. the rate company earns on retained profits.
Rc = Capitalisation rate
E = Earnings per share
D = Dividend per share

Professor Walter emphasizes two factors which influence the market price of a share. The first is
the dividend per share and the second is the relationship between internal return on retained
earnings and the market expectation from that company as reflected in the capitalisation rate. In
other words, if the internal return on retained earnings is higher than the market capitalisation
rate, the value of the ordinary shares would be high even if the dividends are low. However, if
The Institute of Chartered Accountants of Nepal
4 of 85
Suggested Answers of Financial Management
CAP II Examination – June 2010

the internal return within the business is lower than what the market expects, the value of the
share would be low. In such a case, the share holders would prefer that a higher dividend is
declared so that they can utilise the funds in more profitable opportunities elsewhere.

b) Present value of dividend stream for first 2 years:


Rs. 1.50 (1.12) x 0.86 + 1.50 (1.12)2 x 0.74
Rs. 1.68 x 0.86 + 1.88 x 0.74
Rs. 1.45 + 1.39 = 2.84 (A)

Present value of dividend stream for next 2 years:


Rs. 1.88 (1.1) x 0.64 + 1.88 (1.1)2 x 0.55
Rs. 2.07 x 0.64 + 2.28 x 0.55
Rs. 1.33 + 1.25 = 2.58 (B)

Market value of equity share at the end of 4th year computed by using the constant dividend
growth model would be:
P4 = D5
Ks - gn
Where D5 is dividend in the fifth year, gn is the growth rate and Ks is required rate of return.
Now, D5 = D4 (1 + gn)
D5 = Rs. 2.28 ( 1 + 0.08)
= Rs. 2.46
P4 = Rs. 2.46
0.16 - 0.08
= Rs. 30.75

Present market value of P4 = 30.75 x 0.55 = Rs. 16.91 (C)


Hence the intrinsic value per share of Z Ltd. would be
A + B + C i.e. Rs. 2.84 + 2.58 + 16.91 = Rs. 22.33

3 M
/s X. Ltd. has three divisions, each of approximately the same size. Its Finance Department
has estimated the rates of return for different states of nature as given. 15
State Probability Rm Rate of Rate of Rate of
Return of Return of Return of
Division 1 Division 2 Division 3
Great 0.25 0.35 0.40 0.6 0.20
Good 0.25 0.20 0.36 0.3 0.12
Average 0.25 0.13 0.24 0.16 0.08
Horrible 0.25 -0.08 0 -0.26 -0. 02

a) If the risk free rate is 9 %, what rate of return does the market require for each division?
b) What is the beta of the entire company?
c) If the company has 30 percent of its funds provided by riskless debt and the remainder
by equity what is the equity beta for the company?
d) Which of the divisions should be kept? Which should be spun off?
The Institute of Chartered Accountants of Nepal
5 of 85
Suggested Answers of Financial Management
CAP II Examination – June 2010

e) What will the company's beta be if the actions in part (d) are undertaken?

Answer to Question 3
Calculation of the expected return of market, division1, division 2, division 3.

State Prob. Rm R1 R2 R3 pjRm pjR1 PjR2 PjR3


Great 0.25 0.35 0.40 0.6 0.20 0.0875 0.10 0.15 0.05
Good 0.25 0.20 0.36 0.3 0.12 0.05 0.09 0.075 0.03
Average 0.25 0.13 0.24 0.16 0.08 0.0325 0.06 0.04 0.02
Horrible 0.25 -0.08 0 -0.26 -0. 02 -0.02 0 -0.065 -0.005
Total 0.15 0.25 0.20 0.095

Expected Return from market E(Rm) = ∑ pjRm= 0.15 =15%


Expected Return from division 1 E(R1) = ∑ pjR1= 0.25 =25%
Expected Return from division 2 E(R2) = ∑ pjR2= 0.20 =20%
Expected Return from division 3 E(R3) = ∑ pjR3= 0.095 =9.5%
Calculation of the market variance ( cov1m, cov2m,cov3m)

State [Rm-E(Rm)]2 *Pj [R1-E(R1)][Rm- [R2-E(R2)][Rm- [R3-E(R3)][Rm-


E(Rm)] * Pj E(Rm)] * Pj E(Rm)] * Pj
Great 0.01 0.0075 0.02 0.00525
Good 0.000625 0.001375 0.00125 0.0003125
Average 0.0001 0.00005 0.0002 0.000075
Horrible 0.0132 0.014375 0.02645 0.0066125
Total 0.02395 0.023300 0.0495 0.01225

Market Variance σm2 = ∑[Rm-E(Rm)]2 * Pj = 0.02395


Covariance between division 1 and market cov1m = ∑ [R1-E(R1)][Rm-E(Rm)] * Pj =0.023300
Covariance between division 2 and market cov2m = ∑[R2-E(R2)][Rm-E(Rm)] * Pj = 0.0495
Covariance between division 3 and market cov3m = ∑[R3-E(R3)][Rm-E(Rm)] * Pj = 0.01225
Calculation of Betas
Beta of division 1 β1 = cov1m/ σm2= 0.023300/0.02395 = 0.97287
Beta of division 2 β 2= cov2m/ σm2= 0.0495/0.02395 = 2.067
Beta of division 3 β3 = cov3m/ σm2= 0.01225/0.02395 = 0.5115

a. Required Rate of return for each division


E(Rj) = Rf + [E(Rm) – Rf] βj
E(Rj) = 9% + [15% – 9%] βj

The Institute of Chartered Accountants of Nepal


6 of 85
Suggested Answers of Financial Management
CAP II Examination – June 2010

E(R1) = 9% + [15% – 9%] 0.97287 = 14.8%


E(R2) = 9% + [15% – 9%] 2.067 = 21.4%
E(R3) = 9% + [15% – 9%] 0.5115 = 12.1%

b. The Beta of the entire company


Average beta (β) = 0.97287 + 2.067 + 0.5115/3 = 1.18614

c. The equity beta for the company


Proportion of riskless debt (Wd) = 0.3
Beta of the riskless debt (βd) =0
Proportion of equity( Ws) = 0.70
We have,
β = βdWd + βsWs
1.18614 = 0*3.3 + 0.7 * βs
βs = 1.18614/0.7 = 1.6945

d. Division 1 should be kept. Division 2 and division 3 spun off.

e. If the action in part(d) taken, the company's beta will be equal to division 1. So the
company's beta will be 0.97287.

4.
a) Zee Limited, manufacturer of Colour TV sets, are considering the liaberalisation of
existing credit terms to three of their large customers A, B and C. The credit period and
likely quantity of TV sets that will be lifted by the customers are as follows:
(Quantity lifted) (No. of TV Sets)
Credit Period (Days) A B C
0 1,000 1,000 -
30 1,000 1,500 -
60 1,000 2,000 1,000
90 1,000 2,500 1,500
The selling price per TV set is Rs. 9,000. The expected contribution is 20% of the selling
price. The cost of carrying debtors averages 20% per annum.

You are required to determine the credit period to be allowed to each customer.
(Assume 360 days in a year for calculation purposes). 8

b) Explain briefly two basic principles of effective portfolio management. 7

Answer
a) In case of customer A; there is no increase in sales as far as the credit period is
concerned. Apparently A enjoys good liquidity and the demand for TV sets in his area is
limited. Hence there is no point in allowing credit to 'A'. In case of customers 'B' and 'C'
The Institute of Chartered Accountants of Nepal
7 of 85
Suggested Answers of Financial Management
CAP II Examination – June 2010

the credit period can be determined by trading off between profitability of additional
sales and the cost of carrying additional debtors as a result of relaxation of credit period.

Profitability of additional sales on account of relaxing credit period


Customer 'B Customer 'C'
Credit period 0 30 60 90 0 30 60 90

(days) 1000 1,500 2,000 2,500 - - 1,000 1,500

Rs. lakhs Rs. lakhs


Sales 90 135 180 225 - - 90 135
Contribution at 20% 18 27 36 45 - - 18 27
Incremental
Contribution (A) - 9 9 9 - - 18 9

Debtors:
Credit period X sales - 11.25 30 56.25 - - 15 33.75
360

Incremental Debtors - 11.25 18.75 26.25 - - 15 18.75


Cost of incremental
debtors at 80% - 9 15 21 - - 12 15
Cost of carrying incremental
debtors at 20% (B) - 1.8 3 4.2 - - 2.4 3

Excess incremental contribution


over cost of carrying incremental
debtors (A - B) 7.2 6 4.8 - - 15.6 6

It is seen from the above computations that incremental contribution exceeds incremental cost of
carrying additional debtors at each credit period. Therefore, credit period to B and C should be
90 days.

b) The two basic principles of effective portfolio management are:


i. Effective investment planning for the investment in securities by considering the
following factors.
a. Fiscal, financial and monetary policies of the Government and Central Bank.
b. Industrial and economic environment and its impact on industry prospects in terms
of prospective technological changes, competition in the market, capacity
utilisation with industry and demand prospects etc.
ii. Constant review of investment: Portfolio managers are required to review their
investment in securities on a continuous basis to indentify more profitable avenues for
selling and purchasing their investment. For this purpose, they will have to carry out
the following analysis:
a. Assessment of quality of management of the companies in which investment has
already been made or is proposed to be made.

The Institute of Chartered Accountants of Nepal


8 of 85
Suggested Answers of Financial Management
CAP II Examination – June 2010

b. Financial and trend analysis of companies' balance sheets / profit and loss accounts
to identify sound companies with optimum capital structure and better performance
and to disinvest the holding of those companies whose performance is found to be
slackening.
c. The analysis of securities market and its trend is to be done on a continuous basis.

5. a) The clients of an accounting firm wherein you are employed are concerned about the fall
in dividend from a company whose shares they hold as investment. The abridged profit
and loss account and balance sheet of the company for two years are given as follows:
Abridged P & L A/C (year ended Ashadh 31) (Rs. in lakh)

Particulars Current year Previous year


Income:
Sales and other income 19,200 15,500
Expenditure:
Operating and other expenses 15,600 11,900
Depreciation 700 650
Interest 1,850 1,750
18,150 14,300
Profit for the year 1,050 1,200
Taxes 500 200
Profit after taxes 550 1,000
Proposed dividend 200 400

Abridged Balance Sheet as on Ashadh 31 (Rs. in lakh)


Particulars Current year Previous year
Sources of funds:
Share capital (of Rs. 10 each) 4,200 2,600
Reserves and surplus 7,550 1,200
Convertible portion of
12.5% Debentures -- 500
Loan funds:
Secured loans (16%) 10,100 8,700
Unsecured loans (15%) 1,000 3,300
Total 22,850 16,300

Application of funds:
Fixed Assets:
Cost 14,800 11,200
Less: Depreciation 2,700 2,000
12,100 9,20
Advances on capital A/C
& work in progress 1,000 200
13,100 9,400

Current Assets, Loans and Advances


Inventories 8,600 7,100
Sundry debtors 1,400 550
The Institute of Chartered Accountants of Nepal
9 of 85
Suggested Answers of Financial Management
CAP II Examination – June 2010

Cash and bank balances 850 680


Loans and advances 3,000 1,600
13,850 9,930
Less: Current liabilities 4,100 3,030
9,750 6,900
Total 22,850 16,300

Compute the following: interest cover, return on net worth, earnings per
share, dividend cover. 10

b.)What is stock repurchase and why company repurchases its own stock. 5

Answer:
(a)
Abridged P & L A/C (year ended Ashadh 31) (Rs. in lakh)

Particulars Current year Previous year

Sales and other income 19,200 15,500


Less: Operating and other expenses 15,600 11,900
Depreciation 700 650
Earnings before interest and taxes (EBIT) 2,900 2,950
Less: Interest 1,850 1,750
1,050 1,200
Earnings before taxes 1,050 1,200
Less: Taxes 500 200
Earnings after taxes (EAT) 550 1,000
Proposed dividend (Dp) 200 400
Interest coverage ratio (EBIT/Interest) 1.57 1.69
Return on Net worth (EAT/Net worth)* 0.047 0.263
Earnings per share (EAT/no. of shares)** 1.31 3.85
Dividend cover (EAT/ Dp) 2.75 2.50

* Net worth: previous year = Rs. 3,800 (Rs. 2,600 + 1,200); current year = Rs. 11,750 (Rs.
4,200 + 7,550)
**No. of shares: previous year = 260 lakh; current year = 420 lakh.

(b) Stock repurchase is a method, in which a firm buys back shares of its own stock, thereby
decreasing shares outstanding, increasing EPS and often increasing the price of the stock. Stock
repurchases are an alternative to dividends for transmitting cash to stockholders.
Stock repurchased by the issuing firm is called Treasury Stock.
Stock price for repurchase or the equilibrium price is calculated from the following equation.
Repurchase price = S*Pc/S-n
Where,
S=Total number of shares outstanding
Pc= Current market price per share
n= Number of shares to be repurchased

The Institute of Chartered Accountants of Nepal


10 of 85
Suggested Answers of Financial Management
CAP II Examination – June 2010

Alternatively,
Repurchase price= Market price before stock repurchase/1-stock repurchase in fraction.
Reason for purchasing its own stock - If a firm has excess cash, it may repurchase its own
stock leaving fewer shares outstanding and increasing the earnings per share. Stock repurchase
may be alternative to paying cash dividends. The benefits to the shareholder are the same under
a cash dividend policy and stock repurchase. Firms also repurchase their stock if the stock price
is low. The market often sees the stock repurchases as a signal to future prosperity. Stock
repurchases may be used for employee stock options. Stock repurchases reduces the possibility
of being taken over by another firm. Stock repurchases can be made in open market or on tender
offer or on negotiation basis.

Question 6: Write short notes on: (42.5=10)


a) Working Capital Cycle
b) Tools of Financial forecasting
c) Tax consideration influencing the dividend policy of the firm
d) Bridge Finance
Answers
(a) Working Capital Cycle
Every business undertaking requires funds for two purposes - investment in fixed assets and
investment in current assets. Funds required to invest in stock, debtors and other current assets
keep on changing shape and volume. For example, a company has some cash in the beginning.
This cash may be paid to the suppliers of raw materials, to meet labour costs and other
overheads. These three combined would generate work-in-progress which will be converted into
finished goods on the completion of the production process. On sale, these finished goods get
converted into debtors and, when debtors pay, the firm will again have cash. The cash will again
be used for financing raw materials, work-in-progress etc. Thus there is a complete cycle when
cash gets converted into raw materials, work-in-progress, finished goods, debtors and finally
again cash. This time period is known as the working capital cycle of the firm.

(b) Tools of Financial forecasting


i. Days' sales method is a traditional method under which an attempt is made to calculate the
number of days sales and tie it up with the balance sheet items. As different components of
the balance sheet are forecasted in terms of days' dale, this method measures the resources
that are to be financed.

ii. Percentage of sales method is another tool of financial forecasting in which the balance sheet
items are expressed as percentage of sales. This will clearly (to some extent) show the
financial needs caused by increase in sales.
iii. In simple regression method, with sales forecast as starting point and based on past
relationship between sales and assets items, it is possible to construct a line of best fit or the
regression line for them. This method is used for long term forecasting.
iv. In multiple regression method, it is assumed that sales are a function of several variables,
while in simple regression method only one variable is considered.

(c) Tax consideration influencing the dividend policy of the firm

The Institute of Chartered Accountants of Nepal


11 of 85
Suggested Answers of Financial Management
CAP II Examination – June 2010

The firm's dividend policy is directed by the provisions of income-tax law. If a firm has a large
number of owners, in high tax bracket, its dividend policy may be to have higher retention. As
against this if the majority of shareholders are in lower tax bracket requiring regular income the
firm may resort to higher dividend payout, because they need current income and the greater
certainty associated with receiving the dividend now, instead of the less certain prospect of
capital gains later.

(d) Bridge Finance

Bridge Finance refers, normally, to loans taken by a business, usually from commercial banks
for a short period, pending disbursement of term loans by financial institutions. Normally, it
takes time for the financial institution to finalise procedures of creation of security, tie-up
participation with other institutions etc., even though a positive appraisal of the project has been
made. However, once the loans are approved in principle, firms, in order not to lose further time
in starting their projects, arrange for bridge finance. Such temporary loan is normally repaid out
of the proceeds of the principal term loans. Generally the rate of interest on bridge finance is 1%
or 2% higher than on normal term loans.

7. Distinguish between: (42.5=10)


a) Operating Leverage and Financial Leverage.
b) Floatation cost and Transaction Costs
c) Retention policy and Pay Out policy
d) Investing Activities and Financing Activities

Answers to Question 7
(a) Operating Leverage and Financial Leverage.
Operating leverage results from the existence in the firm's income stream. The operating
leverage may be defined as the firm's ability to use fixed operating costs to magnify the effects
of changes in sales on its earnings before interest and taxes. Operating leverage occurs any time
a firm has fixed costs that must be met regardless of volume.

Financial Leverage results from the presence of fixed financial charges in the firm's income
stream. Financial Leverage is concerned with the effects of changes in EBIT on the earnings
available to equity holders. It is defined as the ability of a firm to use fixed financial charges to
magnify the effects of changes in EBIT on the earnings per share.

(b) Floatation cost and Transaction Costs


Floatation cost refers to the cost involved in raising capital from the market, for instance,
underwriting, commission, brokerage and other expenses. The presence of floatation costs
affects the balancing nature of internal (retained earnings) and external (dividend payments)
financing. The introduction of floatation costs implies that the net proceeds from the sale of new
shares would be less than the face value of the shares, depending upon their size.

The Institute of Chartered Accountants of Nepal


12 of 85
Suggested Answers of Financial Management
CAP II Examination – June 2010

Transaction costs refer to costs associated with the sale of securities by the shareholder
investors. In the Modigliani Miller Hypothesis, it is assumed that if dividends are not paid (or
earnings are retained), the investors desirous of current income to meet consumption need can
sell a part of their holdings without incurring any cost, like brokerage and so on. This is
obviously an unrealistic assumption. Since the sale of securities involves cost, to get current
income equivalent to the dividend, if paid, the investors would have to sell securities in excess
of income that they will receive.

(c) Retention policy and Pay Out policy


The firms resort to different dividend policies according to the situations. The firms deciding to
retain the internal accruals and deciding not to pay dividends are called retention policy.
Whereas the firms deciding to pay the dividends are called pay out policy.

The Higher retention policy will lead to lower pay out policy and vice-versa.

(d) Investing Activities and Financing Activities


The investing activities relate to the acquisition and disposal of long-term assets and other
investments not included in cash-equivalents. Their separate disclosure in Cash flow statement is
important as they represent the extent to which expenditures have been made for resources
intended to generate future income and cash flows.
The financing activities report the changes in the size and composition of the share/owner's
capital and debt of the enterprise. Their separate disclosure is useful in predicting claims on
future cash flow by providers of funds (both capital and borrowings) to the enterprise.

The Institute of Chartered Accountants of Nepal


13 of 85
The Institute of Chartered Accountants of Nepal
Suggested Answers of Business Communication and Marketing

CAP II Examination June 2010

Section –―Business Communication‖

8. As per the vacancy announcement appeared in an English daily by the Bank of Asia, you had submitted
your covering letter and a resume for the post of an Accountant. However, you have not heard from them
till date. Now, write a sample follow up letter to the bank. 10
Answer:
Kathmandu-14,
Kalimati
11 April 2010

Dear Sir/Madam,

I submitted a letter of application and a resume earlier this month for the position of an accountant
advertised in The Himalayan Times dated 12 January 2010. Till date, I have not heard from your office. I
would, therefore, like to confirm receipt of my application and reiterate my interest in the job.

I am very interested in working at your bank and I believe that my experience and skills would be an ideal
match for this position.

If necessary, I would be glad to resend my application materials or to provide with any further information
you might need regarding my candidacy. I can be reached at 977-01-4765897 or mail2dinesh@gmail.com.
I look forward to hearing from you.

Thanking you,

Yours sincerely,

Dinesh Wagle

9. What are the barriers to effective interpersonal communication? Suggest some of the ways to overcome
them (5+5=10)
Answer:

Communication plays a major role in employer-employee relationships on farms. It also affects the
relationships among family members on the management team. Although effective communication does
not guarantee success of a farm business, its absence usually assures problems. A communication problem
may soon become a crisis or it may linger on for years.

Problems with any one of the components of the communication model can become a barrier to effective
communication. Here are some of the barriers to effective interpersonal communication. The barriers can
be treated with some means of solution. Each of these barriers are discussed below with their solutions.
Suggested Answers of BC and Marketing
CAP II Examination – June 2010
1. Muddled messages
Muddled messages are a barrier to communication because the sender leaves the receiver unclear about the
intent of the sender. Muddled messages have many causes. The sender may be confused in his or her
thinking. The message may be little more than a vague idea. The problem may be semantic, e.g., note this
muddled newspaper ad: "Dog for sale. Will eat anything. Especially likes children. Call 888-3599 for more
information."
Feedback from the receiver is the best way for a sender to be sure that the message is clear rather than
muddled. Clarifying muddled messages is the responsibility of the sender. The sender hoping the receiver
will figure out the message does little to remove this barrier to communication.

2. Stereotyping

Stereotyping causes us to typify a person, a group, an event or a thing on oversimplified conceptions,


beliefs, or opinions. Thus, basketball players can be stereotyped as tall, green equipment as better than red
equipment, football linemen as dumb, Ford as better than Chevrolet, Vikings as handsome, and people
raised on dairy farms as interested in animals. Stereotyping can substitute for thinking, analysis and open
mindedness to a new situation.
Stereotyping is a barrier to communication when it causes people to act as if they already know the
message that is coming from the sender or worse, as if no message is necessary because "everybody
already knows." Both senders and listeners should continuously look for and address thinking, conclusions
and actions based on stereotypes.

10. Wrong channel

"Good morning." An oral channel for this message is highly appropriate. Writing "GOOD MORNING!"
on a chalkboard, in the machine shed is less effective than a warm oral greeting. On the other hand, a
detailed request to a contractor for construction of a farrowing house should be in writing, i.e., non-oral. A
long conversation between a pork producer and a contractor about the farrowing house construction, with
neither taking notes, surely will result in confusion and misunderstanding. These simple examples illustrate
how the wrong channel can be barrier to communication. Variation of channels helps the receiver
understand the nature and importance of a message.
Simple rules for selection of a channel cause more problems than they solve. In choice of a channel, the
sender needs to be sensitive to such things as the complexity of the message (good morning versus a
construction contract); the consequences of a misunderstanding (medication for a sick animal versus a
guess about tomorrow's weather); knowledge, skills and abilities of the receiver (a new employee versus a
partner in the business); and immediacy of action to be taken from the message (instructions for this
morning's work versus a plan of work for 1994).

4. Language

Words are not reality. Words as the sender understands them are combined with the perceptions of those
words by the receiver. Language represents only part of the whole. We fill in the rest with perceptions.
Trying to understand a foreign language easily demonstrates words not being reality. Being "foreign" is not
limited to the language of another country. It can be the language of another farm. The Gerken house may
be where the Browns now live. The green goose may be a trailer painted red long after it was given the
name green goose. A brassy day may say much about temperature and little about color.
Each new employee needs to be taught the language of the farm. unless the farm's language is learned, it
can be as much a barrier to communication as a foreign language.
The Institute of Chartered Accountants of Nepal
15 of 85
Suggested Answers of BC and Marketing
CAP II Examination – June 2010
5. Lack of feedback

Feedback is the mirror of communication. Feedback mirrors what the sender has sent. Feedback is the
receiver sending back to the sender the message as perceived. Without feedback, communication is one-
way.
Feedback happens in a variety of ways. Asking a person to repeat what has been said, e.g., repeat
instructions, is a very direct way of getting feedback. Feedback may be as subtle as a stare, a puzzled look,
a nod, or failure to ask any questions after complicated instructions have been given. Both sender and
receiver can play an active role in using feedback to make communication truly two-way.
Feedback should be helpful rather than hurtful. Prompt feedback is more effective than the feedback saved
up until the "right" moment. Feedback should deal in specifics rather than generalities. Approach feedback
is a problem in perception rather than a problem of discovering the facts.

6. Poor listening skills

Listening is difficult. A typical speaker says about 125 words per minute. The typical listener can receive
400-600 words per minute. Thus, about 75 percent of listening time is free time. The free time often
sidetracks the listener. The solution is to be an active rather than passive listener.
One important listening skill is to be prepared to listen. Tune out thoughts about other people and other
problems. Search for meaning in what the person is saying. A mental outline or summary of key thoughts
can be very helpful. Avoid interrupting the speaker. "Shut up" is a useful listening guideline. "Shut up
some more" is a useful extension of this guideline. Withhold evaluation and judgment until the other
person has finished with the message. A listener's premature frown, shaking of the head, or bored look can
easily convince the other person there is no reason to elaborate or try again to communicate his or her
excellent idea.
Providing feedback is the most important active listening skill. Ask questions. Nod in agreement. Look the
person straight in the eye. Lean forward. Be an animated listener. Focus on what the other person is saying.
Repeat key points.

7. Interruptions

A farm is a lively place. Few days are routine. Long periods of calm and quiet rarely interrupt the usual
hectic pace. In this environment, conversations, meetings, instructions and even casual talk about last
night's game are likely to be interrupted. The interruptions may be due to something more pressing,
rudeness, lack of privacy for discussion, a drop-in visitor, an emergency, or even the curiosity of someone
else wanting to know what two other people are saying.
Regardless of the cause, interruptions are a barrier to communication. In the extreme, there is a reluctance
of employees and family members even to attempt discussion with a manager because of the near certainty
that the conversation will be interrupted. Less extreme but serious is the problem of incomplete instructions
because someone came by with a pressing question.

3. Need for communication across cultures: As business and very many things concerned with human
development are getting globalized, communicating across cultures has become indispensable for people
intending to achieve development goals. Besides, cross-cultural aspects of human communication have
greatly impacted national and international trade and commerce.
Justify the above statements with concrete examples. 10

The Institute of Chartered Accountants of Nepal


16 of 85
Suggested Answers of BC and Marketing
CAP II Examination – June 2010
Need for communication across cultures: In the remote past, when people lived in a closed society,
communication was understood in a narrow sense. Things that were once considered taboos are talked
about freely now. With things changing with the times, it has been strongly felt that no country can survive
without linkage, communication and association with other nations. Korean Peninsula, for example, split
into two countries after World War I- the South and the North – one making the best of its resources-
human, material and technical for material prosperity and global dignity and the other depleting its
resources for gaining military strength. In spite of their long standing enemity, the latter during the time of
food crisis sought assistance (supply of food grains) from the former.

To survive in a world of cross-cultural communication, it is necessary to understand and interpret culture in


a broader dimension. Culture is a way of life through which people understand and interpret the world in a
broad dimension. As cultural diversity is a matter of complexities, culture is wrongly assumed to be the
cause of misconception. This problem can be overcome by trying to understand cultural differences and
differences in the ways of life of people of different parts of the world. Cultural differences make people of
different places behave differently – in eating, in sitting, in walking, in speaking, and in looking into things
and interpreting them in their own ways. These differences might look complicated for people of different
people but in an age of cross-cultural communication, people have to compromise in the process of
globalization of communication. This requires us to consider the following so as to promote cross-cultural
communication.

 People should be culturally intelligent to understand other people‘s ways of life.


 Be mindful of the differences of others‘ cultures
 Be aware of various factors of human relationship between cultural differences.
 Respect others‘ cultural norms and values
 Understand the impact of cultural differences on people‘s non-verbal communication.
 Understand others‘ attitudes and behavior in relation to time, place,etc.

Though the world of business communication is also affected by cross-cultural traits the cultural gap
bridges in money mattes. It is rightly said that when there is the question of money, everyone is of the
same culture.
To conclude, with things changing much faster in a situation of globalization and sophistication, many
cultures have lost their values. Old traditions and customs are dying out. In the process of modernization,
bigger cultures are dominating the smaller ones. But cross-cultural traits will survive against all odds.

11. Identify the characteristics of successful teams (or groups) and explain them briefly. 10
Answer:

The use of teams has been called the ―solution‖ to many ills in the current work place. Someone even
observed that as an acronym TEAM means ―Together, Everyone Achieves More.‖ Yet, many teams do not
work well together. In fact, some teams can actually increase frustration, lower productivity, and create
employee dissatisfaction. Experts who have studied team workings and decisions have discovered that
effective teams share some or all of the following characteristics.

Small Size, Diverse Makeup. For most functions the best teams range from 2 to 25 members, although 4 or
5 is optimum for many projects. Larger groups have trouble interacting constructively, much less agreeing
on actions. For the most creative decisions, teams generally have male and female members who differ in
age, social background, training, and experience. Members should bring complementary skills to a team.

The Institute of Chartered Accountants of Nepal


17 of 85
Suggested Answers of BC and Marketing
CAP II Examination – June 2010
Paul Fireman, founder of sports shoe manufacturer Reebok, wisely remarked, "If you put five centers on
the basketball court, you‘re going to lose the game. You need, we all need, people of different
backgrounds. Diverse teams can produce innovative solutions with broader applications than homogeneous
teams can".

Agreement on Purpose. An effective team begins with a purpose. Xerox scientists who invented personal
computing developed their team purpose after the chairman of Xerox called for an ―architecture of
information.‖ A team at Sealed Air Corporation developed its purpose when management instructed it to
cut waste and reduce downtime. Working from a general purpose to specific goals typically requires a huge
investment of time and effort. Meaningful discussions, however, motivate team members to ―buy into‖ the
project.

Agreement on Procedures. The best teams develop procedures to guide them. They set up intermediate
goals with deadlines. They assign roles and tasks, requiring all members to contribute equivalent amounts
of team work. They decide how they will reach decisions using one of the strategies shown in the
accompanying Career Coach box. Procedures are continually evaluated to ensure movement toward
attainment of the team‘s goals.

Ability to Confront Conflict. Poorly functioning teams avoid conflict, preferring sulking, gossip, or
backstabbing. A better plan is to acknowledge conflict and address the root of the problem openly.
Although it may feel emotionally risky, direct confrontation saves time and enhances team commitment in
the long run. To be constructive, however, confrontation must be task oriented, not person oriented. An
open airing of differences, in which all team members have a chance to speak their minds, should center on
strengths and weaknesses of the different positions and ideas—not on personalities. After hearing all sides,
team members must negotiate a fair settlement, no matter how long it takes. The best decisions are based
on consensus: all members agree.

Use of Good Communication Techniques. The best teams exchange information and contribute ideas freely
in an informal environment. Team members speak clearly and concisely, avoiding generalities. They
encourage feedback. Listeners become actively involved, read body language, and ask clarifying questions
before responding. Tactful, constructive disagreement is encouraged. Although a team‘s task is taken
seriously, successful teams are able to inject humor into their interactions.

Ability to Collaborate Rather than Compete. Effective team members are genuinely interested in achieving
team goals instead of receiving individual recognition. They contribute ideas and feedback unselfishly.
They monitor team progress, including what‘s going right, what‘s going wrong, and what to do about it.
They celebrate individual and team accomplishments.

Shared Leadership. Effective teams often have no formal leader. Instead, leadership rotates to those with
the appropriate expertise as the team evolves and moves from one phase to another. Many teams operate
under a democratic approach. This approach can achieve buy-in to team decisions, boost morale, and create
fewer hurt feelings and less resentment. But in times of crisis, a strong team member may need to step up
as leader.

12. Write short notes on the followings (Any Two): (2×5=10)


a) Organizing information in reports
b) Effective presentation of information
c) Difference between formal and informal communication

The Institute of Chartered Accountants of Nepal


18 of 85
Suggested Answers of BC and Marketing
CAP II Examination – June 2010
Answers:
a) Organizing information in reports

A report may be defined as formal and factual statement describing a state of affairs on what happened and
how it happened. In other words, a report is a detailed description of a problem or a situation, findings of
investigation with conclusions and recommendations. A report is produced after a study or a research work
- be it or large or small scale - has been carried out. A report should be well formed to become cohesive
and coherent. This is to say, writing a report entails organizing information in a systematic way. It includes:

i) an orderly presentation of facts


ii) close observation of facts with an investigative approach
iii) collection of information supported by evidences
iv) verifying reliability of information
v) drawing on conclusion,
vi) suggesting recommendations

b) Effective presentation of information

Every piece of communication dessiminates some kind of information – formal or informal, complete or
incomplete. The way and style of presentation of information varies with the receiver - individual, group or
public. Matter (contents) and manner (way and style) count much in an effective presentation. The
presentation of information needs to consider the following points:
i) the contents of the message (information)
ii) clear objective of presentation of the message
iii) the identification of the recipient of the information- the audience
iv) choice of appropriate channel of communication for the presentation
v) the relationship between the presenter and the audience, the level of understanding of the two, geo-
physical conditions in which the information is presented
vi) language and style of presentation: using short, simple and understandable language using simple
and impressive style
vii) application of appropriate presentation skills.

c) Difference between formal and informal communication

Formal communication, also called mainline operational communication takes place in a transparent
manner. It is hierarchical and structured and functions under certain organizational rules and regulations.
Much of the organizational communication is formal.
Official correspondence, policy plans, manuals, employee bulletins, newsletters, directive, memos, letters,
reports, proposals, etc. are examples of formal communication.

Informal communication is defined as the channel of communication which does not follow any prescribed
codes of conduct or the rules and regulations. To simplify, an informal communication could be a
discussion or simple talk between two different organizational heads, two senior managers of an
organization. Such talks and communication cannot affect the existing rules and regulations.
Both forms of communication have their advantages as well as disadvantages.

The Institute of Chartered Accountants of Nepal


19 of 85
Suggested Answers of BC and Marketing
CAP II Examination – June 2010
Section –“MARKETING”

6. Read the following case carefully and answer the questions that follows: (4×2.5=10)
Bimala Giri, migrated from Dhading District, has enough knowledge about cooking varieties of
pastries, Momo and Chowmin. She survives with two small children. So she decided to open a small
pastry and tea shop in the periphery of Global College and KCMIT, Baneswor, hoping that a large
number of students may come to the shop for varieties of pastry, coffee and tea. She bought the
necessary equipments for kitchen and shop with a cash outlay of Rs. 50,000. She managed for
furniture and furnishings. She decided to name her shop as ―Hazur Ko Khaja Ghar‖. She maintained
neat and clean in the shop to satisfy the consumers. Beside it, her behaviour with the consumers was
also impressive.

The business has been operating for the last ten months. Both sales volume and value increased
satisfactorily. The major consumers are college students, who drop in for varieties of pastry, tea and
coffee. Occasionally, professors come by with their friends for pastry and a cup of tea. With the passes
of time, a large number of pedestrians also started coming to the shop for tea, coffee and pastries.
Local households and shop-keepers also come to purchase pastries over the counter. Consumers are
satisfied with her services and treatment. But many students, pedestrians and shopkeepers frequently
asked for keeping Momo and Chowmin dishes in the shop. During the vacation period, Bimala noticed
considerable drop in the sales. Though she has knowledge about cooking Momo and Chowmin, she
needs to make additional investment of about Rs. 25,000 for managing Momo and Chowmin in the
shop, for which she has to take loan from others. This money is necessary not only to purchase
necessary equipments and furniture but also needs to hire more space in the rented house.

Actually, Bimala hesitates to keep additional items like Momo and Chowmin dishes, because of
limited managerial and financial capacity. She was, to some extent, satisfied with the existing business
flow and profit margin. But if the demanded items could be added, she is confident that she would be
able to earn more money. On the other, if she avoids keeping the demanded items and insists only in
the current items, it may become difficult for her to retain the current customers and expand volume
and value of business in future.

Bimala is now in a big problem, because she does not know about the modern marketing concept and
marketing techniques; so she is searching for persons who can provide her an expert advice in
marketing field. As a marketing student, you are asked to give answers to the following questions for
the improvement of her business.

Questions:
a) Identify the target market of Bimala‘s business with reason.
b) Who are the customers of Bimala‘s shop and why?
c) What are the various needs of the customers?
d) Should Bimala keep the additional dishes in her shop as per demand of the students, pedestrians,
and shop-keepers? Or, continue the existing tea, coffee, and pastry items only? Why? Give answer
with justifiable reasons from marketing point of view.

Answers:
(Given below are the hints to the above questions. Students are expected to elaborate the hints to
arrive at a full solution)

The Institute of Chartered Accountants of Nepal


20 of 85
Suggested Answers of BC and Marketing
CAP II Examination – June 2010
a) Students of Global College and KCMIT are the target market, because Bimala started her business
focusing on them.
b) Students, pedestrians, Professors, shopkeepers, local households, offices of the location are the
customers of Bimala‘s business, because they drop to the shop for the purchase and consumption of
tea, coffee, and pastries.
c) The various needs of the customers are tea, coffee, and varieties of pastries.
d) As a marketer I should suggest Bimala to add the demanded items – momo and chowmin. This is
because; marketing is a customer-oriented philosophy of business. She should address the demand of
the customers and should try to satisfy them by offering as per their demand.

7.
a) Summarize the economic and demographic environments in Nepal. 5
b) Point out the components of marketing information system and explain briefly the Marketing
Decision Support System. (2+3=5)

Answers:
a) The following Para explains the economic and demographic environments in Nepal-
Economic Environment:
Nepal is an agrarian country. Contribution of agriculture sector to the country‘s real GDP and
employment is high. Farming is still highly labour-intensive.
Recently, marketing opportunities for services sectors like computer software, insurance, banking,
education, hospitals and nursing homes have been fostering rapidly with bright future. Economically,
Nepal hopes from the SAFTA & WTO regimes.

Demographic Environment:
According to Population Census 2001, Nepal has 23.2 million people and is increasing at the rate of
2.05% annually. Both male and female occupy equal status in size. About 90% of the total population
lives in rural area. Though the Nepalese market is characterized by rural market, the marketing
activities are heavily concentrated in urban area due to the difficult topography of the country.
The demographic character of Nepal shows that the market structure of the country is diverse in
nature.

b) The components of marketing information system includes –


i. Internal Records System;
ii. Marketing Intelligence System;
iii. Marketing Information Processing System; and
iv. Marketing Information Analysis System or Marketing Decision Support System (MDSS)/

The following Para explains about MDSS:


Analytical system is a set of statistical or mathematical tools and decision models that help the
marketing managers in analyzing data gathered or supplied through the sub-systems of the MIS and
making rational decision. Analytical system helps marketers to make decisions on the problems by
telling the truth about the situations. In other words, analytical systems allow a firm to ask, "Why did
that happen?" and "What will happen if……..?" Mathematical and/or statistical tools and decision
models are used to solve several marketing problems such as media selection problems to evaluate
new product acceptance o weigh sales and pricing alternatives, and so forth. Therefore, analytical
systems are also called as "marketing decision support systems". Ultimately, the MDSS serves as a
―Data Bank‖.
The Institute of Chartered Accountants of Nepal
21 of 85
Suggested Answers of BC and Marketing
CAP II Examination – June 2010

8.
a) Give the meaning of a product and explain the feature of industrial products. (2+3=5)
b) How is marketing mix controllable and tactical in nature? 5

Answers:
a) A product is both what a seller has to sell and buyer has to buy as a most important part of
marketing mix of a company. Products are the physical goods, services, events, persons, places, ideas,
information, organizations, etc. A product is anything that can be offered to a market to satisfy a need
or want. Tangible or intangible, durable or nondurable, consumer or industrial type and features
differentiate the products.
Product is the first and most important element of the marketing mix. A good product mix, brands
packaging and labeling all are very important. Products can be classified in several ways. In terms of
use products can be broadly classified in two groups namely: Consumer goods and industrial goods.
As demanded by the question only the features of industrial products of industrial products are
mentioned here:
Products that are destined to be sold primarily to use in producing other goods or rendering services or
operating a business institution are industrial products. Material and parts, capital items or suppliers
and business services related products comes under this category.
Features of industrial products are:
- Large scale purchase but fewer buyers
- Systematic and routine purchase
- Role of professional purchase procedure is important
- Customer supplier long fear relations
- Marketing is geographically concentrated
- Some are expensive, capital expenditures
- Durability, brand, direct purchase differs to product types
The above points are to be briefly explained by the students.

b) Controllable
Marketing mix is a set of product, price, place and promotion, which are used to pursue the firm‘s
marketing objectives in the target market. Marketing tools, techniques, strategies, activities, etc. are
decided by the management. So, these are controllable in spite of being flexible in nature as per the
flexibility that occurs in the marketing environment. Marketing mix consists of four components that
can separately be named as product mix, price mix, place mix and promotion mix.
Product mix Price mix Place mix Promotion mix
 Product attributes  Discounts,  Channels  Advertising
 Product variety with allowance  Channel  Sales
quality  Pricing structures promotion
 Product lines and objectives  Channel  Personal
components  Pricing policies dynamics selling
 Product mix and  Pricing methods  Channel conflict  Publicity
innovation  Pricing management  Public
 Product branding, strategies  Physical relations
packaging, labeling  Pricing terms distribution and  strategies
 Service product and conditions logistics
strategies

The Institute of Chartered Accountants of Nepal


22 of 85
Suggested Answers of BC and Marketing
CAP II Examination – June 2010
The four Ps of marketing mix are tactical in nature of their uses. That means, the firm needs to use its
own techniques while implementing them for objective achievement. The decided mix is used as per
the situation of competition being tricky.
o Normal
o Intensive (national, global)
o Throat-cut

9.
a) State and explain the methods of cost-based pricing. 5
b) Draw channel levels structure for consumer market and explain them in that structure. 5

Answer:
a) Methods o f cost-based pricing
This pricing method includes mark-up pricing, target return pricing and BEP pricing.

Mark up pricing method


Mark-up price is that selling price which any manufacturer/marketer decides by expecting a certain
percentage of on sales of each item. It does so to recover costs and expected profit as soon as possible.
o MUP = Unit cost / (1 – desired profit percent)

Target return pricing method


When the firm determines price that would yield its target rate of return on investment, it is known as
target return pricing. This can be determined by
o TRP = Unit cost +(Desired profit % X Investment)/Assumed total sales

Breakeven pricing method


According to this method price in determined at that point where revenue equals total cost. This point
is called Break Even Point or BEP. The BEP can be computed in two ways i.e. In terms of units and in
terms of money value.
o BEP in units= Total fixed cost / (TRP – Variable cost per unit)
o BEP in Rs. = BEP in units X TRP

b) Channel levels structure for consumer market

Manufacturer

Agent
Retailer Whole-
saler
Whole-saler

Retailer Retailer

Consumer

The Institute of Chartered Accountants of Nepal


23 of 85
Suggested Answers of BC and Marketing
CAP II Examination – June 2010

The major channel participants or components of channel structure of consumer market are
producers/manufacturers, agents, wholesalers and retailers. There are four channel levels structure
alternatives for the distribution of goods to consumer markets, ranging from a zero level to three
levels. Under zero level structure, the manufacturer may distribute the merchandise directly to
consumers by passing all marketing intermediaries. Under one level structure, the manufacturer may
use retailers to reach consumers. In two levels structure, the product may reach consumers through
wholesalers and retailers. Similarly, in three levels structure, the manufacturer may use the agents to
contact and execute the sales transaction to different categories of buyers.

10. Briefly explain any five of the followings (5×2=10)


a) Promotion objectives
b) Post purchase behavior
c) Psychological discount pricing strategy
d) Types of labels
e) High involvement purchase.
f) E-marketing
g) Importance of physical distribution system.

Answers:
a) Promotion objectives:
Promotion seeks to stimulate dormant needs of target markets. Promotion performs the major role of
communicating to the target markets. Promotional efforts within a firm include advertising, personal
selling, sales promotion, publicity and public relations. The major objectives of promotion are to
inform, persuade, remind and reassure the market about the organization and its products.
 Informing: The primary task of promotion is to inform the buyers about the product/brand, it‘s
price, availability, utilities and benefits.
 Persuading: Promotion seeks to persuade buyers to make at least one purchase decision in favor of
organization‘s product.
 Reminding: Most promotions seek to constantly remind the buyers about the brand and company
names.
 Reassuring: Promotion also plays the role of reassuring the buyers‘ on the quality and benefits of
the product. It helps in reducing buyers‘ anxiety resulting from cognitive dissonance.

b) Post purchase behavior:


A consumer has to pass through six stages to complete the buying process. Post purchase behavior is
also one major stage of the buying process. After having purchased the product, it may be possible that
the buyer may experience some restlessness in his mind. The marketer‘s job does not end when the
product is brought. After purchasing the product, the consumer will be satisfied or dissatisfied and will
engage in post purchase behavior of interest to the marketer. In this stage, consumer will compare their
expectations before purchasing the product and the product‘s perceived performance. If the product
falls short of expectations, the consumer is disappointed, if it meets expectations, the consumer is
satisfied, if it exceeds expectations, and the consumer is delighted.

c) Psychological discount pricing strategy:


Psychological discount pricing encourage purchases on emotional responses. A superficial discounting
price is charged by showing heavy discount for consumers. Making buyers feel that they are getting
high priced things at very low price. In fact, they are deceived.
The Institute of Chartered Accountants of Nepal
24 of 85
Suggested Answers of BC and Marketing
CAP II Examination – June 2010

d) Types of labels: There are mainly three types of labels i.e.


Brand label: It is the simplest form of label in which only the brand name appears on it. For example,
Nike, Levi‘s, brand names are either stamped or tagged to the product.
Descriptive label: This type of label provides information on the quality or grade of the product along
with the brand name. They are mostly used in perishable and semi-perishable products such as fruits
Grade label: This is more elaborate than brand and grade labels. It contains full description of the
products and their ingredients including their uses, operation methods, precautions etc.

e) High Involvement Purchase:


The complexity of the buying process is affected by whether the product to be bought is of high
involvement or low involvement. In high involvement purchase, the consumer is prepared to invest
more time, resources and efforts. In this situation, the consumer lacks full information about the
product, the amount of money involved is large, the product has high social importance and the
product is perceived to give long-run benefits. Normally, the purchase of a car or a house is high
involvement.

f) E-Marketing: E-marketing is the newest channels for direct marketing based upon electronic
devices. E-business describes a wide variety of electronic platforms, such as the sending of purpose
orders to suppliers via electronic data interchange or extranets, the use of fax & emails to conduct
transactions, the use of ATM and smart Cards to facilitate payment and the use of Interest and online
services all these can be termed as business in a ― market space‖.
The internet today functions as an information source, an entertainment source, a communication
channel, a transaction channel, and every a distribution channel. The way telemarketing has become a
major direct marketing tool. Under direct marketing as an interactive method, especially electronic
marketing is becoming very popular and is showing explosive growth.

g) Importance of physical distribution system:


The importance of physical distribution system – Physical distribution refers to the management of
physical movement of goods from production point to consumption points for final consumption of
goods and services. The contribution of physical distribution include: -
i. It performs several distribution-related services like arrangement of transportation, storing and
warehousing, order processing, materials handling, etc; so it helps maximizing services to the
customers to supply right product at right time;
ii. It helps distributing goods more safely by adopting appropriate materials handling techniques;
iii. It attempts to use appropriate channel of distribution so it helps minimizes the distribution time and
cost; and
iv. It helps supplying necessary goods to the right market place.

The Institute of Chartered Accountants of Nepal


25 of 85
The Institute of Chartered Accountants of Nepal
Suggested Answers of Income Tax and Value Added Tax

CAP II Examination June 2010

13. Answer the following questions with reference to Indian Income Tax Act, 1961.
a) Define the ‗Income‘. Distinguish between ‗Gross Total Income‘ and ‗Total Income‘ 3
b) Discuss the income which are exempt in the case of a political party. Are there any
conditions for claiming such exemptions? 4
c) A Ltd. is an Indian company. It carries business in Mumbai and London. The entire control
and management of A Ltd. is situated outside India. 80% of the total income of the company
is from the business in London. What is the residential status of A Ltd.? 3

Answers:
a) Generally speaking, the word ‗Income‘ covers receipts in the shape of money or money‘s
worth which arise with certain regularity or expected regularity from a definite source. As per
Section 2 (24), income includes the profits and gains, dividend, perquisites etc. which shall be
included while calculating income under the following five heads:
i. Salaries,
ii. Income from house property,
iii. Profits and Gains from Business or Profession.
iv. Capital Gains,
v. Income from other sources.

The aggregate of the incomes computed under the above five heads, after applying the clubbing
provisions and making adjustments of set off and carry forward of losses, is known as Gross
Total Income.

The total income of an assessee is computed by deducting from the gross total income, all
deductions permissible under chapter VIA of the Income Tax Act, i.e. deductions under sections
80C to 80U.

b) Section 13A of the Income Tax Act prescribes the conditions for the exemption of incomes of a
political party. The following incomes derived by a political party are not included in computing
its total income:
i. Income which is chargeable under the head ‗Income from house property‘.
ii. Income which is chargeable under the head ‗Income from other sources‘.
iii. Any income by way of voluntary contribution from any person.
iv. Any income by way of capital gains.

The exemption of the above income shall be available only when the following conditions are
satisfied:
i. The political party keeps and maintains such books of accounts and other documents as will
enable the Assessing Officer to properly deduce its income therefrom.
ii. Where the voluntary contributions from a person exceed Rs. 20,000, it keeps and maintains
a record of such contribution and the name and address of the person who has made such
contribution.
iii. The accounts of the political party are audited by a Chartered Accountant.
Suggested Answers of IT and VAT
CAP II Examination – June 2010

iv. The treasurer of such political party or any person authorized by the political party in this
behalf must submit a report under section 29C(3) of the Representation of People Act, 1951
for the relevant financial year.

Political party for the purpose of this section means a political party registered under section 29a
of the Representation of People Act, 1951.

c) As per section 6(3), a company is said to be a resident in India in any previous year if:
i. It is and Indian company, or
ii. During the relevant previous year, the control and management of its affairs is situated in
India.

Since A Ltd. is an Indian Company, in is resident in India.

14.
a) Mr. Clinton is working in an embassy representing USA and come to Nepal on 1st Ashwin,
2065. His salary and other emoluments are paid by the country he represents. He declared
the following sources of income during the year 2065/66.
i) Salary of Rs. 2,80,000 per month.
ii) Dearness allowance Rs. 1,50,000 per month.
iii) Foreign allowance Rs. 80,000 per month.
iv) He runs a handicraft business in Nepal and earned a net profit of Rs.
1,75,000 in the income year 2065/66.
v) He was a member of Royal Golf Club of Nepal. He won a prize of Rs.
1,50,000 from the game.
However he claimed the following expenses:
i) Renewal of membership and subscription Rs. 50,000
ii) Golf Expenses Rs. 32,000
iii) Donation to a school of remote district Rs. 50,000
(School is the organization recognized as tax exempted by IRD)
iv) Tax deducted at source in Nepal Rs. 2,000

You are required to find Mr. Clinton‘s residential status and taxable income for the income
year 2065/66. 10

b) KMC Pvt. Ltd. had these depreciated balances in ‗Block B‘ of assets as on Ashadh 31,
2066:
Amount (Rs.)
Furniture and Fixtures 50,000
Computers 25,000
Other office Equipments 10,000
Total of Block B 85,000

The company on Shrawan 10, 2066 disposed of the computer for the following
considerations:
The Institute of Chartered Accountants of Nepal
27 of 85
Suggested Answers of IT and VAT
CAP II Examination – June 2010

Condition I: Sales consideration received was Rs. 100,000.


Condition II: Sales consideration received was Rs. 30,000.
Calculate the taxable gain from the disposal of Computer and balances in the Block for the
purpose of depreciation for the year 2066-67 in each of the above conditions. 5

c) Mr. Ramseh has sold a building with one ropani land at a consideration of Rs. 15,000,000 on
Chaitra 31, 2066. This building was purchased by him on Falgun 20, 2061 at a consideration of
Rs. 5,000,000 and also incurred an expenditure of Rs. 1,000,000 for extension of building.
Calculate the amount of tax to be deducted at the time of the sale of land.
Will your answer be different if Mr. Ramesh had purchased that building on Falgun 20, 2055?
5
Answers:
a)
i. Residential status of Mr. Clinton:
Mr. Clinton has lived in Nepal from 1st Ashwin 2065 to last of Ashadh, 2066 i.e 300 days
approximately (10 months x 30 days) which is more than 183 days, thus he is resident for the
income year 2065-66.

ii. Income from Employment:


Remuneration received by a foreign diplomat (representative of foreign countries) is non taxable
in Nepal but other than remuneration income earned in Nepal are taxable.

iii. Computation of Taxable income of Mr. Clinton:

Computation of Taxable income of Mr. Clinton for the Income Year 2065-66

Particulars (Rs.) (Rs.)


Income from Business:
Income from handicraft business 175,000
Assessable income from Business 175,000
Less: Allowable Deduction:
Donation to a school 8,750
Taxable Income 166,250

Taxable income of Mr. Clinton for the Income Year 2065-66 is Rs. 166,250.

Working Notes:
1. Calculation of donation allowable for deduction
Lower of the following amount:
i. Actual donation Rs. 50,000.
ii. 5% of adjustable taxable income (i.e. Rs. 175,000) Rs. 8,750.
iii. Maximum amount Rs. 100,000.
Therefore, donation allowable for deduction is Rs. 8,750.

2. It is assumed that total number of days in a month is 30.


3. TDS @ 25% will be deducted in the prize income and it will be the final withholding.

The Institute of Chartered Accountants of Nepal


28 of 85
Suggested Answers of IT and VAT
CAP II Examination – June 2010

Note: 2 marks each for Residential status and income from employment and 6 marks for
working notes and solution.

b) Calculation of Taxable Income under Condition I:

Sale proceeds 100,000


Less: Depreciated balance of the Block B 85,000
Taxable gain from the sale of computer 15,000

Balances in the Block for the purpose of depreciation for the year 2066-67:
In this case the value of assets in Block B after the disposal is treated as nil as the sales proceed is
more than the depreciated value of the Block.

Note: It is assumed that there is no absorbed addition during the year.

Calculation of Taxable Income under Condition II:

Sale proceeds 30,000


Less: Depreciated balance of the Block B 85,000
Taxable gain from the sale of computer Nil

Since the sale proceed of the Block B is lower than the depreciable balance, the taxable balance
from the disposal is nil during the year.

Balance in the Block B for the purpose of depreciation for the year 2066-67:
Balance of Block B at the beginning of the year Rs. 85,000
Less: Sale proceeds of Computer Rs. 30,000
Balance of Block for the year 2066-67 Rs. 55,000

Note: It is assumed that there is no absorbed addition during the year.

c) Calculation of Tax to be deducted at the time of Sale of Building (Section 89 Kha)

Sale proceed 15,000,000


Less: Purchase price 5,000,000
Building Extension expenses 1,000,000 6,000,000
Taxable profit 9,000,000

Tax to be deducted @ 5% of Rs. 9,000,000 Rs. 450,000

As the building was in the ownership of Mr. Ramesh for more than 5 years therefore 5% tax to be
deducted by Malpot Office at the time of sale (section 89 Kha).

If that building was purchased by Mr. Ramesh on Falgun 20, 2055 than tax shall not be deducted in
this transaction as the building was in the ownership of Mr. Ramesh for more than 10 years. In this

The Institute of Chartered Accountants of Nepal


29 of 85
Suggested Answers of IT and VAT
CAP II Examination – June 2010

case, this building will not be the non-business chargeable asset as per the definition of section 2
(da).
[ Answer should be elaborated with all the provisions. ]

15.
a) What do you mean by Income Splitting? Mention the right available to Inland Revenue
Department to prevent any reduction in tax payable as a result of the splitting of income. 5
b) From the following Income Statement of an exempt controlled resident entity, compute the
taxable income for the financial year 2066/67. 5

Income
Sales Rs. 7,50,000
Interest Income Rs. 5,000
Total Income Rs. 7,55,000

Expenditure
Cost of sales Rs. 5,00,000
Administrative Expenses Rs. 1,20,000
Interest expenses to controlling (exempt) entity Rs. 75,000
Depreciation expenses Rs. 20,000
Rs. 7,15,000
Net Profit Rs. 40,000

c) Mr. X held 500 shares of ABC Ltd., a Listed Company, of Rs. 100 each. He acquired those
shares through a stockbroker by paying Rs. 700 each, and incurred Rs. 2,800 towards broker
commission. He sold all those shares through the broker for Rs. 3,500 each with the
brokerage commission of Rs. 8,750. Assuming that Mr. X is a non-resident, compute the
gain amount and withholding tax amount. 5
d) What are the conditions that enable Inland Revenue Officer to levy interest on failure to pay
tax on due date and to comply with Act under sec. 119 and 119(ka) respectively? 5
Answers:
a) Income splitting is also a kind of Transfer Pricing. Generally income splitting technique is
indulged where the "Progressive Rates of Taxation" is in existence. Income splitting helps to
cause tax reduction where a tax payer attempts to split income with another tax payer.
- Where a taxpayer attempts to split income with another tax payer and this income
splitting is likely to cause a tax reduction, the Inland Revenue Department (IRD) is
given the right to adjust amounts to be included or deducted in calculating the income of
each persons. The duty of IRD is to prevent any reduction in tax payable as a result of
splitting of income. Sec [34(1)].

The Institute of Chartered Accountants of Nepal


30 of 85
Suggested Answers of IT and VAT
CAP II Examination – June 2010

- If a taxpayer attempts income splitting by a way of transfer of following with an intent to


reduce the total tax payable by the person either directly or indirectly:
- amounts to be derived or costs to be incurred OR
- an amount received by the transferee of an asset that is derived from the asset; or an amount
paid in owning the asset.
In deciding as aforesaid whether a person is seeking to split income, the IRD has power to consider
the market value of any payment made for transfer.

b) Computation of Taxable Income of Exempt Controlled Resident Entity for the financial year
2066/67.

Income
Sales Rs. 7,50,000
Interest Income Rs. 5,000
Total Income Rs. 755,000

Expenditure
Cost of sales Rs. 5,00,000
Administrative Expenses Rs. 1,20,000
Interest Expenses (see working note) Rs. 60,000
Depreciation Expenses Rs. 20,000 Rs. 7,00,000
Taxable Income Rs. 55,000

Working note:
Computation of allowable Interest Expenses under sec. [14(b)]
Interest Income Rs. 5,000
Profit exclusive of Interest Income/Expenses
Net Profit Rs. 40,000
Less Interest Income Rs. 5,000
Rs. 35,000
Plus Interest Expenditure Rs. 75,000
Total Profit Rs. 1,10,000
Total allowable Interest
50% of the above profit Rs. 55,000
Total allowable Interest Expenses Rs. 60,000

Excess of allowable interest expenses over actual interest expenses to controlling entity to be
carried forward to the subsequent year (Rs. 75,000- Rs. 60,000)= Rs. 15,000

c) Computation of gain on the sales of shares of ABC Ltd.


Sales (500 shares of Rs. 3,500 each) (a) Rs. 17,50,000
Less: Outgoings
500 shares at Rs. 700 each Rs. 3.50,000
Brokerage commission:
Acquisition of shares Rs. 2,800
The Institute of Chartered Accountants of Nepal
31 of 85
Suggested Answers of IT and VAT
CAP II Examination – June 2010

Sales Rs. 8,750


Total outgoing (b) Rs. 3,61,550
Gain on sale of shares (a-b) Rs. 13,88,450

Amount of withholding tax by Stock Exchange Ltd. on total gain amount is:
15 % on Rs. 13,88,450 = Rs. 2,08,267
Note: Withholding Tax @ 15 % on gain is applicable to the non resident
person.
d) In case no tax is paid on due date, the tax payer is liable to pay interest for each month and
part of a month at the standard interest rate (10% p.a) on the tax amount outstanding.
For the purpose of calculating interest payable as above, any time extension granted (u/s 98)
will be ignored.
Also, a withholding agent may not recover from a withholdee interest payable by the agent in
respect of a failure to deposit at Inland Revenue Office, the withheld amount on behalf of
withholdee (u/s 90(4)).
Except as stated otherwise, any person who fails to comply with any provisions of the
Income Tax Act 2058 and Rules made there under will be liable to a interest of not less than
Rs. 5000 and not more than Rs. 30,000 [u/s 119(ka)]

16. Answer the following questions quoting the relevant provisions of Income Tax
Act, 2058.
a) Explain briefly the tax provisions relating to loan loss provisions available to
the banks and financial institutions. 5
b) What are the matters that should be included in the assessment notice issued
by Inland Revenue Department? 5
Answers:
a) Section 59 of the Income Tax Act, 2058 deals about the special provisions applicable to the
Banking Business. A person running a banking business along with other businesses, it has to
treat the banking business as being run by a separate person. It means the accounts, activities,
transactions etc. related to the banking business should be kept separate from the same of other
business.

A banking business is allowed a deduction of a loss of loan provided by it according to the


regulation of Nepal Rastra Bank, subject to a maximum ceiling of 5% of the outstanding loan as
on the last day of the income year.

Actual expenses for a loan loss, if incurred, should be deducted from the provision outstanding
on that date. If deducted as an expense, it is not allowed for tax purpose. In case the amount of
loan loss provision is capitalized or utilized for distribution, or for payment of dividend in any
income year, up to that amount, it should be included in the taxable income of the income year.

Section 24 (3) has given an authority to IRD to accept the accounting procedures followed by
banking businesses as per the instructions given by NRB.
The Institute of Chartered Accountants of Nepal
32 of 85
Suggested Answers of IT and VAT
CAP II Examination – June 2010

b) As per the section 102 of Income Tax Act, 2058, IRO has to give notice of the assessment
under section 100(2) or section 101 to the assessee which shall include the following
information:
a. The tax payable by the tax payer for the year of assessment and the tax due to him.
b. The method of calculation of the tax liability.
c. The reason for making the amended assessment by the IRO.
d. The period within which the tax due is payable and
e. Where, when and how to appeal against the order if the taxpayer is not satisfied with
the amended assessment.

A period of 15 days should be given to the tax payer to explain and produce evidence against the
IRO‘s contention.

17.
a) ABC Export House has a credit of Rs. 200,000 in the first month representing 30% of its
total sales during that month. However, it has a credit of Rs. 600,000 representing 60% of its
total sales in the following month.
How does ABC Export House claim refund if it does not always have exports over 50% of
total sales? 5

b) A company dealing with computer parts sold the goods worth Rs. 25 lacs in the month of
Baisakh 2067. It has imported the computer parts on which input VAT has been paid as per
the cost fixed/determined by Custom Offices on CIF basis with details below:
Cost declared by importer Rs. 200,000
Revised cost fixed by Custom Officer Rs. 250,000
(based on prevailing international price)
Transportation Rs. 25,000
Insurance Rs. 5,000
Import Duty @ 6%
Besides, the credit balance up to Chaitra 2066 is Rs. 20,500. In addition, it has omitted to
adjust input tax credit on the purchase of Rs. 30,000 pertaining to the month of Chaitra
2066.
Compute the Input and Output VAT. 10

c) What are the types of accounts and records to be kept by a taxpayer under VAT Act, 2052?
What are their contents? 5

Answers:
a) In the case of given question, the exporter will not be entitled to a refund as the sales
reported in first month did not amount to 50% of its total sales. The credit of Rs. 2,00,000
will be carried forward in the following month for refund.
Since its credit amount represents 60% of the total sales in the following month, ABC
Export House is entitled to claim the full refund of Rs. 6 lac plus Rs. 2 lac from the
preceding months. [U/s 39(5) of VAT Rule]

The Institute of Chartered Accountants of Nepal


33 of 85
Suggested Answers of IT and VAT
CAP II Examination – June 2010

b) Computation of Net VAT Payable.


Credit balance up to Chaitra '066 Rs. 20,500
Input Tax (working note) Rs. 38,584
Adjustment for omitted Input tax credit Rs. 3,900
Total Input Tax to be credited (A) Rs. 62,984
Total sales in Baisakh '067 Rs. 25,00,000
Output VAT on sales of Rs. 25 lac (B) Rs. 3,25,000
Tax payable (Net output VAT) (A-B) Rs. 2,62,016
Working note:
Computation of Input VAT amount on import of computer parts.
Cost of computers Rs. 2,50,000
Transportations Rs. 25,000
Insurance Rs, 5,000
Total Rs. 2,80,000
Import duty @ 6% Rs. 16,800
Total cost for VAT purpose Rs. 2,96,800
Input VAT amount = 13% on Rs. 2,96,800
= Rs. 38,584

c) A taxpayer must keep the following accounts and records:


1. a purchase book
2. a sales book, and
3. a VAT account.
Purchase and sales books include:
1. the invoice number
2. the invoice date
3. the supplier's name and PAN/VAT in the purchase book
4. the customer's name and PAN/VAT in the sales book
5. the taxable value, and
6. the amount of VAT.
Businesses which sell both taxable and exempt goods will need to complete additional columns of
information to separate exempt sales and the purchases related to them.

18. Answer the following questions with reference to the Value Added Tax Act,
2052. (4×5=20)
a) What are the conditions for compulsory registration on VAT? Explain.
b) What are the conditions for refund of VAT to the foreign tourist? Answer
citing the provisions of the Act.
c) Mention the provisions regarding collection of tax by custom authorities.
d) Elaborate the provisions relating the market value and its determination.
The Institute of Chartered Accountants of Nepal
34 of 85
Suggested Answers of IT and VAT
CAP II Examination – June 2010

Answers:
a) As per the Section 10 (2) of the Value Added Tax Act, if any person who is dealing in the
product which is taxable as per the Act, or any person doing business with in metropolitan,
sub-metropolitan, municipality or such other area specified by the Department in hardware,
sanitary, furniture, fixture, furnishing, automobiles, motor parts, electronics, marble and color
lab operation shall apply for registration with in 30 days of starting the business.

Above provision requires person who deals in the taxable goods and does business of a
particular nature in a particular area shall have to compulsorily get registered. There is
exception to this mandatory provision in Sec 9 which provides that up to a specified threshold
of taxable transaction by a small scale business, the registration requirement is waived. Such
threshold is 20 lakh currently. Such small scale business, however can get registered
voluntarily.

b) Section 25 Ka provides for the refund of VAT to the foreign tourists. Following are the
conditions:

1. The tourist should have purchased at least 15 thousand worth of goods which are
taxable,
2. The tourist should be returning from the air route.
3. Three percent of such refund shall be deducted as service charge.

c) Section 28 of the Act has prescribed the authorities of the custom officer as per the Act to
collect the tax. Following are the provisions:

1) Unless otherwise prescribed by the Ministry of Finance, Government of Nepal, Custom


Officer will collect the tax on imports as per the Value Added Tax Act.

1 Ka) When goods produced or prepared in Nepal, after completing the customs procedures
for export or after reaching the foreign land, rejected by the party or due to any other
reason such goods are re-imported to the country with the aim to export again within 3
months, then value added tax payable at the time of returning the goods may be allowed to
be paid as deposit and when the goods is exported such deposit can be refunded. Custom
office should be providing the details to the Inland Revenue Department.

2) In case of goods being imported the custom officer can exercise the authority from this and
the prevailing Customs Act for collection of tax.

d) Section 13 reading along with the Rule 22 will clarify the provision relating the market
value and its determination.

The market value of goods or services shall be determined as the consideration in money
which the supply of these goods or services would generally be agreed on if the transaction
were made under similar circumstances between persons who are unrelated and the supply
freely offered.
The Institute of Chartered Accountants of Nepal
35 of 85
Suggested Answers of IT and VAT
CAP II Examination – June 2010

While determining the market value as per above provision, the tax officer shall determine
the market value by studying the transactions and value of other vendors registered in regard
to the transaction of the same nature.

In cases where the market value of any goods or services cannot be determined as set forth as
prescribed in the Act, the Director General shall determine the value on the basis also of the
information received in that regard by him from the registered persons of the same nature.

The Institute of Chartered Accountants of Nepal


36 of 85
Advanced Accounting
CAP-II, June 2011

Maximum Marks - 100

Total No. of Questions - 6

Time Allowed - 3 Hours


Marks

Attempt all questions. Working notes should form part of the answer.

1. Sudeep and Mayur had been carrying business independently. They agree to
amalgamate and form a new company Sumeru Ltd. with an authorized share
capital of Rs. 2,00,000 divided in to 40,000 equity shares of Rs. 5 each..
On 31st Ashad 2066, the respective balance Sheet of Sudeep and Mayur were as
follows:

Description Sudeep Rs Mayur Rs.


Fixed Assets 3,17,500 1,82,500
Current assets 1,63,500 83,875
4,81,000 2,66,375
Less: Current Liabilities 2,98,500 90,125
Representing Capital 1,82,500 1,76,250

Additional information:
a) Revalued figures of Fixed assets were Sudeep Rs 3,55,000 and Mayur Rs.
1,95,000 and revalued figure of current Assets were, Sudeep Rs. 1,49,750 and
Mayur Rs. 78,875.
b) The debtors of Sudeep includes Rs. 21,675 to be received from Mayur.
c) The purchase consideration is satisfied by issue of the following shares and
debentures:
d) 30,000 equity shares of Sumeru Ltd to Sudeep and Mayur in the proportion of
the profitability of their respective business based on the average net profit
during the last three years which were as follows:

Sudeep Rs Mayur Rs
2064 Profit 2,24,788 1,36,950
2065 Loss / Profit ( 1,250) 1,71,050
2066 Profit 1,88,962 1,79,500

e) 15% Debenture in Sumeru Ltd. at par to provide an income equivalent to 8 %


return on capital employed in their respective business as on 31st Ashad 2066
after revaluation of assets.

You are required to:


i) Compute the amount of Debentures and shares to be issued to Sudeep and
Mayur
ii) A Balance Sheet of Sumeru Ltd. showing the position immediately after
amalgamation.

Answer No.1

NMG P.T.O.
(2)

Sudeep Rs Mayur Rs
(1) Average Profit:
Rs (2,24,788-1,250+1,88,962)/3 1,37,500
Rs ( 1,36,950+1,71,050+1,79,500)/3 1,62,500
(2) Equity Shares issued:
(a) Ratio of distribution 1375 1625
(b) No. of shares 13,750 16,250
(c) Amount:
13,750@ Rs. 5 68,750
16,250 @ Rs. 5 81,250

(3) Capital employed (after revaluation of assets)


Fixed assets 3,55,000 1,95,000
Current Assets 1,49,750 78,875
5,04,750 2,73,875
Less: Current Liabilities 2,98,500 90,125
2,06,250 1,83,750

(4)Debenture Issued
8% debenture on capital employed 16,500 14,700
15% Debenture to be issued to provide equivalent income
Sudeep Rs 16,500*100/15 1,10,000
Mayur Rs 14,700*100/15 98,000

Working No.1 Calculation of Purchase Consideration:


Description Sudeep Rs Mayor Rs Total Rs.

Equity shares issued 68,750 81,250 1,50,000


15% Debenture issued 1,10,000 98,000 2,08,000
1,78,750 1,79,250 3,58,000

WN 2: Calculation of Capital Reserve

(a) Net Assets taken over:


Fixed Assets 3,55,000 1,95,000 5,50,000
Current assets 1,28,075 78,875* 2,06,950
4,83,075 2,73,875 7,56,950
Less: Current Liabilities 2,98,500 68,450** 3,66,950
184,575 2,05,425 3,90,000

(b) Purchase Consideration 1,78,750 1,79,250 3,58,000

(c ) Capital Reserve (a-b) 5,825 26,175 32,000

* Rs 1,49,750- Rs 21,675 ** Rs 90,125- Rs 21,675


(3)

Balance Sheet of Sudeep Ltd as on 31st Ashad 2066

Rs.
Fixed Assets 5,50,000
Current assets, Loans & Advances Rs. 2,06,950
Less; Current Liabilities Rs 3,66,950 ( 1,60,000)
3,90,000

Represented by:
Share Capital** 1,50,000
Reserve & Surplus 32,000
15% Debenture 2,08,000 3,90,000

**
Share Capital:
Authorised share capital 40,000 shares of Rs 5 2,00,000
Issued and subscribed capital:
30,000 equity shares of Rs 5 each 1,50,000
( all the above shares have been allotted as fully
Paid up without payments being received in cash.)

2.
a) Media Pvt. Ltd. obtained advertisement rights for One Day World Cup
Cricket Tournament to be held in March/April, 2011 for Rs. 50,00,000.
By 31st January, 2011 they have paid Rs. 30,00,000 to secure these
advertisement rights. The balance Rs. 20,00,000 was paid in February, 2011.
By 31st January, 2011 they procured advertisement for 70% of the available
time for Rs. 70,00,000. The advertisers paid 60% of the amount by that date.
The balance 40% was received in February, 2011.
Advertisements for the balance 30% time were procured in February, 2011 for
Rs. 30,00,000. The advertisers paid the full amount while booking the
advertisement.
75% of the advertisement time is expected to be available in March, 2011 and
the balance 25% in April, 2011.
You are asked to:
i) Pass journal entries in relation to the above.
ii) Show in columnar from as to how the items will appear in the monthly
financial statements for January, February, March and April 2011. Give
reasons for your treatment. (6+4=10)

b) Ramesh and Suraj decided to work in partnership in the following scheme,


agreeing to share profit as under:
Ranesh 3/4th share
Suraj 1/4th share

They guaranteed the subscription at par of 10,00,000 shares of Rs.10 each in


Sheetal ltd and to pay all expenses up to allotment in consideration of Sheetal
(4)

Ltd. furthermore 50,000 Share of Rs. 10 each fully paid (other than 10,00,000
Share issued to public) was issued to them. Ramesh introduced cash in to the
business to meet the following expenses:

Registration fees Rs. 48,500


Advertising charges Rs 28,500
Printing charges of Memorandum of Association and
Articles of Association and Prospectus etc. Rs. 23,000

Suraj introduced cash to meet following expenses:


Rent Rs. 37,000
Legal fees Rs. 13,000

Application fell short of the 10,00,000 shares by 30,000 shares. Suraj


introduced further cash on joint account for the said 30,000 shares. This
amount was utilized to subscribe the said 30,000 shares and paid to the
company.

The guarantee having been fulfilled, Sheetal Ltd. Handed over the Ramesh
and Suraj 50,000 shares.
The partnership sold all shares. Suraj received the sale proceed of 20,000
shares amounting to Rs. 1,80,000 and Ramesh of the remaining 60,000 shares
amounting Rs. 5,00,000.

Give necessary accounts in the books of both parties. 10

Answer No.2
a)
In the books of Media Pvt. Ltd.
Journal Entries
Dr. Cr.
Rs. in lakhs Rs. in lakhs
2011
Jan Advance for advertisement rights (purchase) A/c Dr. 30.00
To Bank A/c 30.00
(Being advance paid for obtaining advertisement rights)
Bank A/c Dr. 42.00
To Advance for advertisement time (sale) A/c 42.00
(Being advance received from advertisers
amounting to 60% of Rs. 70 lakhs for booking
70% advertisement time)
Feb Advance for advertisement rights (purchase) A/c Dr. 20.00
To Bank A/c 20.00
(Being balance advance i.e., Rs. 50 lakhs less
Rs. 30 lakhs paid)
Bank A/c Dr. 28.00
(5)

To Advance for advertisement time (sale) A/c 28.00


(Being balance advance i.e., Rs. 70 lakhs less
Rs. 42 lakhs received from advertisers)
Bank A/c Dr. 30.00
To Advance for advertisement time (sale) A/c 30.00
(Being advance received from advertisers
in respect of booking of balance 30% time)
March Advertisement rights (purchase) A/c Dr. 37.50
To Advance for advertisement rights (purchase) A/c 37.50
(Being cost of advertisement rights used in March
i.e., 75% of Rs. 50 lakhs, adjusted against advance
paid)
Advance for advertisement time (sale) A/c Dr. 75.00
To Advertisement time (sale) A/c 75.00
(Being sale price of advertisement time in March i.e.,
75% of Rs. 100 lakhs adjusted, against advance
received from advertisers)
Profit and Loss A/c Dr. 37.50
To Advertisement rights (purchase) A/c 37.50
(Being cost of advertisement rights debited to Profit
and Loss Account in March)
Advertisement time (sale) A/c Dr. 75.00
To Profit and Loss A/c 75.00
(Being revenue recognized in Profit and Loss
Account in March)
April Advertisement rights (purchase) A/c Dr. 12.50
To Advance for advertisement rights (purchase) A/c 12.50
(Being cost of advertisement rights used in April, i.e.,
25% of Rs. 50 lakhs, adjusted against
advance paid)
Advance for advertisement time (sale) A/c Dr. 25.00
To Advertisement time (sale) A/c 25.00
(Being sale price of advertisement time availed in
April i.e., 25% of Rs. 100 lakhs, adjusted against
advance received from advertisers)
Profit and Loss A/c Dr. 12.50
To Advertisement rights (purchase) A/c 12.50
(6)

(Being cost of advertisement rights used in Apri,


debited to Profit and Loss Account in April)
Advertisement time (sale) A/c Dr. 25.00
To Profit and Loss Account 25.00
(Being revenue recognized in April)

(ii) Monthly financial statements


(1) Revenue statement (Rs. in lakhs)
Jan Feb March April
Rs. Rs. Rs. Rs.
Sale of advertisement time – – 75.00 25.00
Less: Purchase of advertisement rights – – 37.50 12.50
Net profit – – 37.50 12.50
(2) Balance sheet as at 31.1.2011 28.2.2011 31.3.2011 30.4.2011
Sources of funds:
Net profit – – 37.50 50.00
Application of funds:
Current assets, loans and advances:
Advance for advertisement rights 30.00 50.00 12.50 –
Bank Balance 12.00 50.00 50.00 50.00
42.00 100.00 62.50 50.00
Less: Current liabilities
Advance for advertisement time
(received from advertisers) 42.00 100.00 25.00 –
Net current assets – – 37.50 50.00
As per NAS 7 on Revenue Recognition, under proportionate completion method, revenue from service
transactions is recognized proportionately by reference to the performance of each act where
performance consists of the execution of more than one act.
Therefore, income from advertisement is recognized in March, 2011 (75%) and April, 2011 (25%) in
the proportion of availability of the advertisement time. (6+4=10 marks)

b)
Memorandum Joint Venture Account

To Ramesh: Rs. Rs. By Ramesh; Rs


Registration charges 48,500 Sale proceeds of
Advertising 28,500 60,000 shares 5,00,000
Printing 23,000 1,00,000
To Suraj: By Suraj:
Rent 37,000 Sale proceeds of
Legal fees 13,000 50,000 20,000 shares 1,80,000
To Suraj: subscription of 3,00,000
Unapplied for shares
To Ramesh (3/4th profit) 1,72,500
th
To Suraj ( 1/4 profit) 57,500
Total 6,80,000 Total 6,80,000
Student should be careful to make no entry for the shares received from the company as consideration
for the guarantee.

In the book of Ramesh, the account with Suraj will be as follows:


(7)

Suraj in Joint Venture Account


To Bank: Rs. By Bank: Rs.
Registration charges 48,500 Sale proceed of 60,000
Advertising 28,500 shares 5,00,000
Printing 23,000
To Profit & Loss Account
2/3 ths share of profit 1,72,500
To Bank 2,27,500
Total 5,00,000 Total 5,00,000

In the book of Suraj, the account with Ramesh will be as follows:


Ramesh in Joint VentureAccount
To Bank: Rs. By Bank: Rs.
Rent 37,000 Sale proceed of 20,000
Legal 13,000 shares 1,80,000

To Bank: Subscription of By Bank: Settlement 2,27,500


Unapplied for shares 3,00,000
To Profit & Loss Account:
1/4th share of profit 57,500
Total 4,07,500 Total 4,07,500

3.
a)
i) Find out the income to be recognized in the case of Good Faith Bank
Limited for the year ended 31st Asadh, 2067 as per the directives issued by
NRB.

(Rs. in Lakhs)

Performing Assets Non-performing Assets


Interest Realized in Interest Realized in
accrued cash accrued cash
Term Loans 540 400 450 20
Consumer Loan 250 205 360 16
2
ii) The following is an extract from the Trial Balance of Golden Bank Ltd. as
at 31st March, 2011:
Rebate on bills discounted as on 1-4-2010 341,295 (Cr.)
Discount received 850,780 (Cr.)
Analysis of the bills discounted reveals as follows:
Amount (Rs.) Due date
14,00,000 June 1,2011
43,60,000 June 8,2011
(8)

28,20,000 June 21,2011


40,60,000 July 1,2011
30,00,000 July 5,2011
You are required to find out the amount of discount to be credited to Profit
and Loss account for the year ending 31st March, 2011 and pass Journal
Entries. The rate of discount may be taken at 10% per annum. (4+4=8)

b)
i) What is entity concept? 2
ii) Following are data of Robinson
 Market Value of Stock as on Shrawan 01, 2066: Rs. 300,000
(Market Price being cost plus 20% thereof).
 Purchases for the year: Rs. 1,450,000
(Out of which goods worth Rs.150,000 was delivered on Shrawan 08,
2067)
 Sales for the year: Rs. 1,480,000.
 Goods costing Rs. 20,000 was taken by the proprietor for personal use.
 Goods costing Rs. 10,000 was used for firm‟s office stationery.
 Normally, the firm sells goods on cost plus 40%

Determine the value of stock to be taken for Balance Sheet purpose of


Robinson as on Ashadh end, 2067 8

Answer No.3
a) (i)Calculation of interest income of Good Faith Bank Ltd. to be recognized for the year ended
31st Asadh, 2067.
Term Loans Rs. in Lakhs
Interest realized in cash regarding performing assets 400
Interest realized in cash regarding non-performing assets 20 420
Consumer Loan
Interest realized in cash regarding performing assets 205
Interest realized in cash regarding non-performing assets 16 221
Total Interest to be recognized 641
(2 Marks)
(ii)
The amount of rebate on bills discounted as on 31st March, 2011 the period which has not
been expired upto that day will be calculated as follows:
Discount on Rs.14,00,000 for 62 days @ 10% 23780
Discount on Rs.43,60,000 for 69 days @ 10% 82,420
Discount on Rs.28,20,000 for 82 days @ 10% 63,355
Discount on Rs.40,60,000 for 92 days @ 10% 102,335
Discount on Rs.30,00,000 for 96 days @ 10% 78,905
Total 350,795
The amount of discount to be credited to the profit and loss account will be:
(9)

Rs.
Transfer from rebate on bills discounted as on 31.03.2010 341,295
Add: Discount received during the year 850,780
1,192,075
Less: Rebate on bills discounted as on 31.03.2011 (as above) 350,795
841,280
Journal Entries
Rs. Rs.
Rebate on bills discounted A/c Dr. 341,295
To Discount on bills A/c 341,295
(Transfer of unexpired discount on 31.03.2010)
Discount on bills A/c Dr. 350,795
To Rebate on bills discounted 350,795
(Unexpired discount on 31.03.2011 taken into account)
Discount on Bills A/c Dr. 841,280
To P & L A/c 841,280
(Discount earned in the year, transferred to P&L A/c)
i. Business consists of person and resources. Person representing the business is separate and
distinct from the business enterprises. Accounting system deals with the economic activities
of the business not of owner. In Entity Concept, preparation of Balance Sheet of the business
does not consider the personal assets and liability of the owner of the business.

ii.
Robinson
Statement showing Value of Stock
as on Ashadh end, 2067
Opening Stock ( as on Shrawan 01, 2066) 250,000
Add:
Purchases during the Year 1,450,000
Less: Goods in Transit (150,000)
Less: Goods taken by proprietor for personal use (20,000)
Less: Goods used for office's stationery (10,000) 1,270,000
Less:
Cost of Goods Sold:
Sales for the Year 1,480,000
Less: Profit on Sales (422,857)
1,057,143
Stock in Hand (as on Ashadh end, 2067) 462,857
Add:
Goods in Transit 150,000 150,000
Value of Stock to be shown in Balance Sheet (as on
Ashadh end, 2067) 612,857

Working Notes:
(10)

1. Calculation of Opening Stock:


Market Value of Stock: 300,000
Profit Margin (on Cost) 20%
Cost Price of Opening Stock: =300,000 = 250,000
(100+20)
2. Calculation of Profit on
Sales:
Sales 1,480,000
Profit Margin (on Cost) 40%
Profit Amount =1,480,000 X 40 = 422,857 1,057,143
(100+40)

4.
a) On Baishakh 01 2063, Rama & Rama took delivery of a car from Sita & Sita
on hire purchase system, Rs. 105,000 being paid on delivery and the balance
in six annual installments of Rs. 60,000 each payable at the end of each year.
The Cash Price of the car is Rs. 3,60,000. Calculate the amount of interest for
each year. 5

b) Following is the information of two business organizations belonging to same


industry for 2065/066;
Particulars Birat Industry Kirat industry
Rs Rs

Opening balance 25,000 15,000


Cash flow from operating activities 8,500 24,000
Cash from investing activities;
Sale of fixed assets 35,000 6,000
Cash from financing activities:
Amount borrowed 40,000 5,000
Closing Balance 1,08,500 50,000
Birat claims that it‟s cash generating activity is better than that of Kirat. Do
you agree ? Comment on the basis of cash generating ability of both
organization 5

c) Palpa Power company decided to replace some parts of its plant by an


improved plant. The plant to be replaced was built in 2065 for Rs.70,00,000. It
is estimated that it would cost Rs.130,00,000 to build a new plant of the same
size and capacity. The cost of the new plant as per the improved design was
Rs.2,10,00,000 and in addition, material belonging to the old plant valued at
Rs.7,60,000 was used in the construction of the new plant. The balance of the
plant was sold for Rs.6, 00,000.Compute the amount to be written off to
revenue and the amount to be capitalized. Also prepare Plant account and
Replacement account. 5
Answer No.4
a) Amount Weight
2063 Amount outstanding for interest after Down Payment 360,000 6/21
2064 Amount outstanding for interest after First Installment 300,000 5/21
(11)

Amount outstanding for interest after Second


2065 Installment 240,000 4/21
2066 Amount outstanding for interest after Third Installment 180,000 3/21
Amount outstanding for interest after Fourth
2067 Installment 120,000 2/21
2068 Amount outstanding for interest after Fifth Installment 60,000 1/21

Total Interest for all the six years is Rs. 105,000 (360000+105000-105000) which should be
divided in the ratio of 6:5:4:3:2:1 for all these years.
Hence, Interest for each year:
2063 : 105,000 X 6/21 = 30,000
2064 : 105,000 X 5/21 = 25,000
2065 : 105,000 X 4/21 = 20,000
2066 : 105,000 X 3/21 = 15,000
2067 : 105,000 X 2/21 = 10,000
2068: 105,000 X 1/21 = 5,000

b) Kirat organization has a better capacity to generate cash. Claim of Birat organization is not
appropriate. The evaluation is based on the quantum of cash flow from operating activities
rather than on investing or financing activities. Birat organization is generating funds from
either sale of goods or out of borrowing and quantum of funds so generated is not an
appropriate indicator for evaluation.

C)

(a) (i) Calculation of amount chargeable to revenue Rs. Rs.


Estimated current cost of replacing old plant 130,00,000
Less: Realization from sale of remaining of the old plant 6,00,000
Value of materials belonging to the old Plant
used in the construction of new plant 7,60,000 13,60,000
Total 116,40,000
(ii) Calculation of amount to be capitalized
Cost of building new plant (cash) 2,10,00,000
Add: Value of materials belonging to the old
plant used in the construction of the new plant 7,60,000 217,60,000
Less: Estimated current cost of replacing old plant 130,00,000
Total 87,60,000
(iii) Plant Account
Rs. Rs.
To Balance b/d 70,00,000 By Balanced c/d 157,60,000
To Cost of construction:
Cash
(210,00,000-130,00,000) 80,00,000
Cost of old material used 760,000 _________
157,60,000 157,60,000
(iv) Replacement Account
Rs. Rs.
To Bank account (portion to By Bank account 6,00,000
be written off out of the By Plant account 760,000
replacement cost) 130,00,000 By Revenue account 116,40,000
(12)

130,00,000 130,00,000

5.
a) The company deals in three products, A, B & C, which are neither similar nor
interchangeable. At the time of closing of its account for the year 2066/067,
the Historical and Net Realizable Value of the items of closing stock are
determined as follows:

Items Historical Cost Net Realizable Value


( Rs. in Lakhs) ( Rs. in Lakhs)
A 40 28
B 32 32
C 16 24

Compute the value of closing stock? 5


b) What are the conditions that have to be satisfied for recognition of revenue
from sale of goods? 5
c) A Ltd. received a grant of Rs 2 crores from the Government for purchase of a
special purpose machinery during 2062-063. The cost of machinery was Rs 38
crores and had a useful life of 9 years. During the current year 2065-066 the
grant has become refundable due to non fulfillment of certain conditions
attached to it. Assuming that the entire amount of grant was deducted from the
cost of machinery in the year of acquisition, state with reasons, the accounting
treatment to be followed in the rear 2065-066 5

Answer No.5
a) As per para 9 of NAS 4 on Inventories, inventory should be valued at the lower of cost and net
realizable value. Inventories should be written down to net realizable value on an item-by-item
basis in the given case:
Historical Net Realizable Valuation of Closing
Items Cost Value Stock
(Rs. in Lakhs) (Rs. in Lakhs) (Rs. in Lakhs)
A 40.00 28.00 28.00
B 32.00 32.00 32.00
C 16.00 24.00 16.00
88.00 84.00 76.00

Hence, closing stock will be valued at Rs. 76 lakhs

b) As per NAS 7, Revenue from Sale of goods shall be recognized when all the following
conditions have been satisfied:
i. The entity has transferred to the buyer the significant risks and rewards of ownership
of goods;
ii. The entity retains neither continuing managerial involvement to the degree usually
associated with ownership nor effective control over the goods sold;
iii. The amount of revenue can be measured reliably;
iv. It is probable that the economic benefits associated with the transaction will flow to
the entity; and
(13)

v. The cost incurred or to be incurred in respect of the transaction can be measured


reliably.

c) Value of asset (Without grant) = Rs.38 crore


Depreciation (Without Grant) = Rs.38 crore/9years
= 4.22 crore per year
As per NAS, the additional depreciation needs to be charge of expenses. Hence,
Already charge Depreciation
Book value (before refund of grant) = Rs 20 crore
(assuming SLM method and no residual value, annual depreciation will be Rs.(38-2)/9 = Rs. 4 crore,
depreciation for 4 years will be Rs.16 crore
Additional Depreciation for 4 years = (4.22×4)-(4×4)
(change as expenses for current year = 16.88-16=0.88 crore
Revises book Value = 38-16.88 =21.12 crore, Depreciation for next 5 year =4.22

6. Write short notes on:

a) Share and Stock. 4


b) Escalation Clause in Contracts 3
c) Accounting estimate. 3
Answer No.6
a) Share and Stock
In today‟s financial markets, the distinction between stocks and shares has been
somewhat blurred. Generally, these words are used interchangeably to refer to the pieces
of paper that denote ownership in a particular company, called stock certificates.
However, the difference between the two words comes from the context in which they
are used. For example, “stock” is a general term used to describe the ownership
certificates of any company, in general, and “shares” refers” to a the ownership
certificates of a particular company. So, if investors say they own stocks, they are
generally referring to their overall ownership in one or more companies. Technically, if
someone says that own shares- the question then becomes – shares in what company?
Bottom line, stocks and shares are the same thing. The minor distinction between stocks
and shares is usually overlooked, and it has more to do with syntax than financial or legal
accuracy.

b) Escalation Clause in Contracts:


This is a clause provided in the contracts to cover up any changes in the price of the
contract due to changes in the price of raw materials and labour or change in utilization of
factor of production. The object of this clause is to safeguard the interest on both sides
against unfavourable change in prices. The basis, on which the factor prices are based, is
laid down in the contract. For e.g.: in a contract with transport undertaking, it may be
provided that the price per ton km will increase or decrease for each rise or fall in price of
diesel by 10% of the prevailing prices.

c) Accounting estimate.
As a result of the uncertainties in business activities, many financial statement items
cannot be measured with precision but can only be estimated. These are called accounting
(14)

estimates. Therefore, the management makes various estimates and assumptions of


assets, liabilities, incomes and expenses as on the date of preparation of financial
statements. This process of estimation involves judgements based on the latest
information available.
Examples of estimation in some fields are:
(i) Estimation of useful life of depreciable assets.
(ii) Estimation of provision to be made for bad and doubtful debts.
The Institute of Chartered Accountants of Nepal
Suggested Answers of Auditing and Assurance
CAP II Examination – June 2011

1. Give your opinions with reasons on the following cases: (45=20)

a) Terai Bank Limited provided a long term loan amounting to Rs. 250 Million to Relax Hotel Pvt. Limited for hotel
operation. The hotel is unable to repay installment (principal & interest) as per repayment schedule. Accordingly you are
appointed by the Bank for special review of the income of the hotel to ensure whether income procedures are duly complied
with and the amount has been fully accounted for. As a special reviewer how would you plan your review work?

b) A Donor has appointed you as an auditor for ensuring financial management capacity of the NGO before funding
made to the NGO to rely on the fund operating capability of the NGO. What would be your special focus area for such
assignment?

c) The Board of Directors meeting of M/s Dikpal Commercial Bank Limited, proposed dividend @ 50 % for the fiscal
year 2066/067. The Annual General Meeting approved the dividend @ 70% instead of the proposed dividend by the Board
of Directors. Examine the decision of the Annual General Meeting to approve dividend in excess of the proposed dividend
by the Board of Directors and whether it is commensurate with the company Act, 2063.

d) Paudel Provident Fund (PPF) is a approved retirement fund, which manages retirement benefits of employees of
group of companies relating to "Poudel Group". PPF has entered into an agreement with a commercial bank for maintenance
and operation of the retirements accounts relating to its beneficiaries. Under the agreement, PPF is required to open a
deposit account in the bank and maintain the entire fund only with the bank. During the audit of PPF, auditor finds that bank
has credited its account with excess interest by 4,32,000. All the interest income in the account is distributed on pro-rata
basis to the individual employees through accounts maintained in the bank itself.

Answer No.1
a) An efficient and effective review can only be performed if this has been thoroughly and properly planned. The planning
stage of the review should be used to establish an overall strategy for the review.
Adequate planning will ensure that appropriate attention is given to crucial areas of the review and that potential problems
are identified on a timely basis. At the planning stage the engagement partner should assign the necessary staffs who
possess the skills and ability required in order to ensure the review is carried out efficiently and in accordance with the
Nepal Standards on Auditing.
Planning Activities
At the planning stage the overall audit strategy is developed. The audit strategy sets the scope, timing and direction of the
review. At this stage the reviewer will develop the detailed review plan which will help identify problem areas and
important review areas.
Once the strategy has been established, then the reviewer is able to develop the more detailed review plan to address the
matters identified in the overall strategy.
Contents of the Plan
The review plan should document the nature, timing and extent of the procedures to be adopted which should be sufficient
and effective enough to be able to assess the risks associated with review engagements.
The detailed plan should also contain a description of the nature, timing and extent of planned further audit procedures at the
assertion level for each material class of transactions, account balances and disclosures.
Finally, the review plan should also contain details of other procedures to be adopted so that the review can be carried out
effectively.
Typical contents of the detailed plan are:
• Nature of the business and what it does
• Risk and problematic areas (both business risk and financial statement risk)
• Details of any complexities associated with the assignment
• Specific accounting and auditing standards relevant to the assignment
• Budgets
• Planned review procedures
• Details of sampling techniques
• Key personnel employed at the clients
For the purpose of this review of the income of the hotel and purpose of assignment an audit program with detailed
procedures should also be developed to conduct the review work efficiently and effectively. Audit procedures inter alia
includes amongst the following:
i. Whether rack rates has been maintained for different types of rooms (Suit/Deluxe/Superior etc), banquette rates are
formalized with recipe and other rates are formalized & approved by competent authority.
ii. Whether there are formal discount rates. If discount rate is in excess of formal rate, whether this is approved by the
competent authority.

NMG P.T.O.
(16)

iii. Whether rate agreements has been entered with regular official & individual parties and duly signed of both parties. If
yes agreed rates have been applied or otherwise to ensure how the rate is formalized in such cases.
iv. Whether complimentary service provided to guests are within the power of approving authority.
v. Whether the billing are made on timely basis and accounted accordingly on departure of guest with evidence of
acceptance by guests. If bill is to be forwarded to customer through mail, is it forwarded on timely basis?
vi. Whether there is clear cut credit policy and credit facility provided only to eligible guest backing with deposition of
advance.
vii. Whether revenue as per income of auditor`s report, accounting system and VAT return has been matched. In case of
difference, is proper reconciliation has been made.
viii. Whether rates have been reviewed as per formal policy of hotel and approved by the competent authority.
ix. Whether record of revenue charged has been reflected in the concerned ledger folios of the guest.
x. Whether accounting system (Software/Manual) for revenue is commensurable with volume of transactions of the
hotel.
xi. Check the closing procedures of the hotel during physical presence on sample night audits.
xii. Make the surprise check on different point of sales and observed if any evidences of concealing the income (by not
issuing bill, by providing huge discounts, by undervaluing the bill etc.)
xiii. Check the recovery rate from debtors and utilization of cash.

b) For ensuring financial management capacity of the NGO my special focus area would be:
i. To ensure whether adequate internal control system for accounting administration has been existed within the
organization.
ii. To ensure the required experiences & academic qualification of accounting staff.
iii. To ensure the volume of donors fund previously operated by the NGO.
iv. To ensure the adequacy of policies etc (like accounting manual, chart of account, financial & administrative
regulation, software are in place & adequate).
v. To see the annual audit report, internal audit report & other types of audit report for ensuring serious and risky areas
if any to pay due attention.
vi. To ensure the basic accounting concept of non-accounting staff for handling the advances.
vii. To see overall account of the NGO, whether there is any diversion of project fund to institutional fund through any
unfair means.
viii. To see the composition of program cost & office running cost for ensuring whether office running cost mainly salary
is excessively high than program cost.
ix. To the composition of common cost (cost shared by different donors) in past period to ensure that whether same cost
has been borne by different donors in books of accounts.
x. To ensure whether periodical reconciliation (bank/cash/fund etc) system are in place.
xi. To ensure whether budgetary control system are in place.
xii. To ensure whether there is excessive turnover of the accounting staff.
xiii. To ensure whether proper & adequate delegation of authority has been made and properly exercised in practice.

c) The Annual General Meeting is authorized body to approve dividend under section 77 (6) of the Company Act,
2063. The proviso to the said section has however, restricted the Annual General Meeting to approve the dividend in
excess of the dividend proposed by the Board of Directors. However, the dividend at lower rate can be approved. Thus,
the decision of Annual General Meeting of Ms. Shangrila Commercial Bank Ltd. to approve dividend in excess of the
dividend proposed by the Board of Directors is void and Annual General Meeting can only approve the dividend at the
lower rate or at the rate proposed by the Board of Directors.

d) As per NSA 200 "Objective and General Principles Governing an Audit of Financial Statements", The objective
of an audit of financial statements is to enable the auditor to express an opinion whether the financial statements are
prepared, in all material respects, in accordance with an identified financial reporting framework.
Once the financial statements are misstated, auditor is required to report on the matter by quantifying its impact on the
financial statements. Therefore, auditor should look in to the matter by verifying the fact about how PPF has treated the
excess credit by the bank in its account. Even if bank cannot recover from PPF's employee, and the PPF has not given any
effect to this item in its account, auditor should ask management to disclose the fact in notes to the account of financial
statements and of require, should modify the audit report accordingly.

2. Answer the following:


a) What are the auditor‟s responsibilities for detection of frauds and errors? 10
b) Briefly explain the concept of Materiality. 5

Answer No. 2
a) It has been explicitly mentioned in the NSA 240 that the primary responsibility for the prevention and detection
of fraud rests with both those charged with governance of the entity and with management. The respective responsibilities
of those charged with governance and of management may vary by entity to entity. It is important that management, with
(17)

the oversight of those charged with governance, place a strong emphasis on fraud prevention, which may reduce
opportunities for fraud to take place, and fraud deterrence, which could persuade individuals not to commit fraud because
of the likelihood of detection and punishment. It is the responsibility of those charged with governance of the entity to
ensure, through oversight of management, that the entity establishes and maintains internal control to provide reasonable
assurance with regard to reliability of financial reporting, effectiveness and efficiency of operations and compliance with
applicable laws and regulations. This responsibility includes establishing and maintaining controls pertaining to the
entity‟s objective of preparing financial statements that give a true and fair view (or are presented fairly in all material
respects) in accordance with the applicable financial reporting framework and managing risks that may give rise to
material misstatements in those financial statements. Such controls reduce but do not eliminate the risks of misstatement.
An auditor conducting an audit in accordance with NSAs obtains reasonable assurance that the financial statements taken
as a whole are free from material misstatement, whether caused by fraud or error. An auditor cannot obtain absolute
assurance that material misstatements in the financial statements will be detected because of such factors as the use of
judgment, the use of testing, the inherent limitations of internal control and the fact that much of the audit evidence
available to the auditor is persuasive rather than conclusive in nature.
The primary objective of an auditor is to express an opinion on the financial statements. However, the auditor while
conducting the audit is required to consider the risk of material misstatements in the financial statements resulting from
fraud or error. An audit conducted in accordance with the auditing standards generally accepted in Nepal is designed to
provide reasonable assurance that the financial statements taken as a whole are free from material misstatement, whether
caused by fraud or error. The fact that an audit is carried out may act as a deterrent, but the auditor is not and cannot be
held responsible for the prevention of fraud and error.
The auditor's opinion on the financial statements is based on the concept of obtaining reasonable assurance; hence, in
an audit, the auditor does not guarantee that material misstatements, whether from fraud or error, will be detected.

b) Materiality is an important consideration for an auditor to evaluate whether the financial statements reflect a true
and fair or not. The auditor should consider materiality and its relationship with audit risk when conducting an audit. The
Nepal Accounting Standards Board's "Framework for the Preparation and Presentation of Financial Statements" defines
Materiality in the following terms: "Information is material if its omission or misstatement could influence the economic
decisions of users taken on the basis of the financial statements. Materiality depends on the size of the item or error
judged in the particular circumstances of its omission or misstatement. Thus, materiality provides a threshold or cut-off
point rather than being a primary qualitative characteristic which information must have if it is to be useful."
In designing the audit plan, the auditor establishes an acceptable materiality level so as to detect quantitatively material
misstatements. However, both the amount (quantity) and nature (quality) of misstatements need to be considered.
Examples of qualitative misstatements would be the inadequate or improper description of an accounting policy when it is
likely that a user of the financial statements would be misled by the description, and failure to disclose the breach of
regulatory requirements when it is likely that the consequent imposition of regulatory restrictions will significantly impair
operating capability.
The auditor should consider the possibility of misstatements of relatively small amounts that, cumulatively, could have a
material effect on the financial statements. For example, an error in a month end procedure could be an indication of a
potential material misstatement if that error is repeated each month.
The auditor should also consider materiality at both the overall financial statement level and in relation to individual
account balances, classes of transactions and disclosures. Materiality may be influenced by considerations such as legal
and regulatory requirements and considerations relating to individual financial statement account balances and
relationships. This process may result in different materiality levels depending on the aspect of the financial statements
being considered.
While determining the nature, timing and extent of audit procedures; and evaluating the effect of misstatements, the
auditor should consider the materiality. When planning the audit, the auditor should consider what would make the
financial statements materially misstated. The auditor's assessment of materiality, related to specific account balances and
classes of transactions, helps the auditor decide such questions as what items to examine and whether to use sampling and
analytical procedures. This enables the auditor to select audit procedures that, in combination, can be expected to reduce
audit risk to an acceptably low level. There is an inverse relationship between materiality and the level of audit risk, that
is, the higher the materiality level, the lower the audit risk and vice versa.
The auditor in addition to exercising professional judgment should consider any legislation or regulation which may
impact that assessment while assessing materiality in the case of the public sector. In the public sector, materiality is also
based on the “context and nature” of an item and includes, for example, sensitivity as well as value. Sensitivity covers a
variety of matters such as compliance with authorities, legislative concern or public interest.

3. Give your comments on the following: (35=15)


a) You have attended physical verification of Supreme Garments Limited and noted no discrepancies between the
physical quantity and the Store Register for the year ending 15 July, 2010. The management provides a statement of value of
stocks but refuses to provide with the cost sheet and calculations regarding how the value has been arrived. The company
discloses that the value of stocks is as “certified by the management” in the Financial Statement. The management is ready
to disclose the same in the Management Representation letter.
(18)

b) The Critical Pollution Extinction Company Limited procured a pollution controlling machine for which the
government has 50% rebate in customs duty upon the precondition that the machine should be used for at least 5 years.
During the course of audit you found that the company has credited 50% rebate provided to income for the year by
disclosing the same in the Notes to Accounts in the Financial Statement. What is your opinion as regards the accounting
treatment by the company?

c) Mr. Yadav, a Chartered Accountant in practice enters into an agreement with Mr. Puspa Adhikari, an individual
who has passed one group of CA final 2 years back. The agreement provides that Mr. Puspa Adhikari shall work in all
professional assignments of Mr. Yadav and shall receive 15 percent of fee received from such assignments as remuneration.
Explain whether the agreement is in order as per Code of Ethics or any other relevant reference? If remuneration is based on
cash flow, does it violate the provisions of the Code of Ethics?

Answer No.3
a) As per NSA-500 “Audit Evidence”, Audit evidence regarding an assertion of existence of inventory will not
compensate for failure to obtain assertion regarding valuation. The auditor should satisfy himself that the valuation of
inventories is in accordance with the Nepal Accounting Standard -02 “Inventories”. The auditor should examine the
methods of applying the basis of inventory valuation which include examination of stock sheets, costing records and
treatment of overhead expenses as a part of cost of inventories. In the given case, it will be construed as limitation on the
scope of auditors. Accordingly, the auditor will have to issue a qualified opinion.
Certification by Management of Supreme Garments Ltd. cannot be taken as conclusive evidence and it cannot relieve the
duty of auditors when there are other procedures and means of gathering audit evidence. Just because management had
owned responsibility for the correctness of its valuation of Stock/inventory, the auditor cannot shirk his responsibility.
This is negligence on his part if he relies on the management representation without assessing the corroborative available
evidences.

b) As mentioned in Para 8 of NAS 10 Accounting for Government Grants and Disclosure of Government
Assistance, a government grant is not recognized until there is reasonable assurance that the entity will comply with the
conditions attaching to it, and that the grant will be received. Receipt of a grant does not of itself provide conclusive
evidence that the conditions attaching to the grant have been or will be fulfilled. Government grants shall be recognized as
income over the periods necessary to match them with the related costs which they are intended to compensate, on a
systematic basis. They shall not be credited directly to shareholders‟ interests. It is fundamental to the income approach
that government grants be recognized as income on a systematic and rational basis over the periods necessary to match
them with the related costs. Income recognition of government grants on a receipts basis is not in accordance with the
accrual accounting assumption. Government grants related to assets, including non- monetary grants at fair value, shall be
presented in the balance sheet either by setting up the grant as deferred income or by deducting the grant in arriving at the
carrying amount of the asset. Hence the NAS has provided two acceptable alternative methods of presentation in financial
statements of grants related to assets. According to the first method, the grant amount is accounted for as deferred income
which is recognized as income on a systematic and rational basis over the useful life of the asset. Under the other
alternative method, government grant is deducted in arriving at the carrying cost of the asset and the grant is recognized as
income over the depreciable asset by way of a reduced depreciation charge.
Hence, as mentioned here above, the treatment to credit whole of the credit rebate amount in the year of purchase of the
machine by Critical Pollution Extinction Company is not appropriate since it is bound by the precondition that the
machine should be used for at least 5 years. The 50% rebate received on customs duty should be credited to income for at
least over a period of 5 years.

c) As per section 34(3) of Nepal Chartered Accountants Act, 1997, one shall not share the auditing fees or
remuneration or distribute as profit with any person other than a member of the institute and shall not pay any
commission, brokerage, etc. out of the professional fees earned to any person or member.
In the above case also, Mr. Yadav a Chartered Accountant in practice enter into agreement to share auditing fees.
Accordingly, Mr. Yadav would be held guilty of professional misconduct since he agreed to share the fees to his
employee.
Even if remuneration were based on cash flow, it would tantamount to be based on fees received on cash and accordingly
Mr. Yadav would be held guilty of professional misconduct.

4. Answer the following: (35=15)


a) Babarmahal Trading Ltd (BTL) is a wholesaler of Chinese goods and all accounting information is stored on BTL‟s
computer. You are the audit senior in charge of the audit of the receivables balance. Explain the audit procedures that
should be carried out on the receivables balance at BTL.
b) Proper audit is not possible without adequate knowledge of client‟s business. Explain it.
c) Define the concept of internal control and explain its inherent limitations.

Answer No.4
(19)

a) Procedures for verification of receivable balance of BTL are prescribed below:


 Cast the receivables ledger to ensure it agrees with the total on the receivables control account.
 Compare the balance on each receivable account with its credit limit to ensure this has not been exceeded.
 Review the balances in the receivables ledger to ensure no balance exceeds total sales to that customer.
 Calculate receivables days for each month end to monitor control of receivables over the year.
 Stratify receivables balances to show all material items and select appropriate sample for testing.
 Produce an aged receivables analysis to assist with the identification of irrecoverable receivables.
 Obtaining the external confirmations as per NSA 505 and evaluating whether the results of the external confirmation
process together with the results from any other audit procedures performed, provide sufficient appropriate audit
evidence regarding the assertion being audited. In conducting this evaluation the auditor considers the guidance
provided by NSA 330 and NSA 530, “Audit Sampling and Other Selective Procedures.”

b) Obtaining an understanding of the entity and its environment is an essential aspect of performing an audit in
accordance with Nepal Standard on Auditing . In particular, that understanding establishes a frame of reference within
which the auditor plans the audit and exercises professional judgment about assessing risks of material misstatement of the
financial statements and responding to those risks throughout the audit, for example when:
 Establishing materiality and evaluating whether the judgment about materiality remains appropriate as the audit
progresses;
 Considering the appropriateness of the selection and application of accounting policies, and the adequacy of financial
statement disclosures;
 Identifying areas where special audit consideration may be necessary, for example, related party transactions, the
appropriateness of management‟s use of the going concern assumption, or considering the business purpose of
transactions;
 Developing expectations for use when performing analytical procedures;
 Designing and performing further audit procedures to reduce audit risk to an acceptably low level; and
 Evaluating the sufficiency and appropriateness of audit evidence obtained, such as the appropriateness of assumptions
and of management‟s oral and written representations.
The auditor uses professional judgment to determine the extent of the understanding required of the entity and its
environment, including its internal control. The auditor‟s primary consideration is whether the understanding that has been
obtained is sufficient to assess the risks of material misstatement of the financial statements and to design and perform
further audit procedures. The depth of the overall understanding that is required by the auditor in performing the audit is less
than that possessed by management in managing the entity.

c) The concept of internal control may be defined as the plan of organization and all the methods and procedures
adopted by the management of an entity to assist in achieving management‟s objectives of ensuring the orderly and efficient
conduct of its business, including adherence to management policies, the safeguarding of assets, prevention and detection of
fraud and error, the accuracy and completeness of the accounting records, and the timely preparation of reliable financial
information. The system of internal control extends beyond those matters which relate directly to the functions of the
accounting system and comprises control environment and control procedures. Internal control is an essential prerequisite
for efficient and effective management of any organization. It is thus, a primary responsibility of every management to
establish and maintain an adequate system of internal control appropriate to the size and nature of the business of the entity.
An internal control system can provide only reasonable, not absolute, assurance that the management‟s objectives in
establishing the system are achieved. This is because there are some inherent limitations of internal control. These
limitations are mentioned hereunder:
 Controls have to be cost effective. Hence, some control mechanisms may not have been implemented merely because
they are not cost-effective.
 Most control tools are directed at transactions of a usual nature. Therefore, transactions of unusual nature might have
been escaped from such controls.
 The human error potentiality prevails everywhere, even in the control systems.
 Any system of control has its limitations in preventing frauds through collusion between two or more persons.
 Controls may not change with the pace of changes in conditions.
 Management itself may manipulate transactions or accounting estimates.
 A member of management may himself override the control system.

5. Answer the following: (35=15)

a) Nepal Standards on Auditing relating to "Management Representation" provides guidance on the use of management
representation as audit evidence. Enumerate five items that could be included in a management representation letter.
b) Briefly explain five methods of sampling.
c) What are the principal contents of audit engagement letter?

Answer No. 5
(20)

a) Following matters form part of the management representation letter as per NSA 580 „Management
Representations‟:
 It is management's responsibility for the fair presentation of the financial statements in accordance with Nepal
Accounting Standards or relevant practices.
 No irregularities involving management or employees that could have a material effect on the financial statements
 All books of account and supporting documentation have been made available to the auditors
 Information and disclosures with reference to related parties is complete
 Financial statements are free from material misstatements including omissions
 No non-compliance with any statute or regulatory authority requirements.
 No plans that will materially alter the carrying value or classification of assets or liabilities in the financial statements
 No events, unless already disclosed, after the end of the reporting period that needs disclosure in the financial
statements.

b) Methods of sampling is presented below:


Haphazard Sampling
Haphazard sampling is a technique adopted by the auditor where the sample does not follow a structured technique.
Haphazard sampling is not appropriate when using statistical sampling and the auditor should always ensure that haphazard
sampling is not „doctored‟ in such a way that it deliberately avoids sampling items which, for example, are difficult to
locate. All items in the population should stand a chance of being sampled.
Stratified Sampling
This is a technique where the auditor will split items in a sample into their various strata‟s. For example, in a payroll sample
the auditor might split the sample between full-time males, full-time females, part-time males and part-time females and
work out the percentage of the strata in the population (the population being the total amount that makes up a figure). For
example if 30% of the population are full-time males, 40% full-time females, 20% part-time males and 10% part-time
females, then the sample will consist of 30% full-time males, 40% full-time females etc.
Systematic Sampling
Often referred to as „interval‟ sampling this is where the auditor will take the number of sampling units in the population
and divide this into the sample size to give a sampling interval. For example in a sales invoice sample where the sampling
interval is 20, then the auditor will determine a starting point for sampling and sample every 20th sales invoice thereafter.
Block Sampling
Block sampling is a technique where the auditor applies procedures to such items that all occur in the same block of time or
sequence. For example testing amounts received from customers in the month of September. Alternatively, a „block‟ of
remittance advices received in September would be tested in their entirety. It is to be noted that block sampling should be
used with caution because valid references cannot be made beyond the period or block examined. Where the auditor does
use block sampling, then many blocks should be selected to help minimise sampling risk.
Judgement
Auditors can use their judgement in selecting items for sampling. There are three basic issues which determine which items
are selected:
 The value of items
 The relative risk (items prone to error should be given special attention)
 The representativeness (the sample should be representative of the population)

c) The form and content of audit engagement letters may vary for each client, but they would generally include
reference to:
 The objective of the audit of financial statements;
 Management‟s responsibility for the financial statements;
 The scope of the audit, including reference to applicable legislation, regulations, or pronouncements of professional
bodies to which the auditor adheres;
 The form of any reports or other communication of results of the engagement;
 The fact that because of the test nature and other inherent limitations of an audit, together with the inherent limitations
of internal control, there is an unavoidable risk that even some material misstatement may remain undiscovered; and
 Unrestricted access to whatever records, documentation and other information requested in connection with the audit.
 Management‟s responsibility for establishing and maintaining effective internal control.

6. Write short notes on the following: (25=10)
a) Disadvantages of the use of an audit program.
b) Flow Chart

Answer No. 6
a) The possible disadvantages of the use of an audit program are:
i. The work may become mechanical and particular parts of the programme may be carried out without any
understanding of the object of such parts in the whole audit scheme.
(21)

ii. The programme often tends to become rigid and inflexible following set grooves; the business may change in its
operation of conduct, but the old programme may still be carried on. Changes in staff or internal control may render
precaution necessary at points different from those originally decided upon.
iii. Inefficient assistants may take shelter behind the programme i.e. defend deficiencies in their work on the ground that no
instruction in the matter is contained therein.
iv. A hard & fast audit program may kill the initiative of efficient and enterprising assistants.
v. Preparation of audit program without preliminary depth assessment of clients business and without considering scope of
work to be carried out by auditor will not provide objectivity of the audit work.

c) Flow Chart is a graphical presentation of all the processes within an organization. It gives bird`s eye view of the system
and the flow of transactions that is of particular interest for the auditors. It facilitates the auditors in accumulation of the
information necessary for the proper review and evaluation of internal controls implemented within an organization. It
is considered to be the most concise way of recording the auditor`s review of the system. It minimizes the amount of
narrative explanation and thereby achieves a consideration or presentation not possible in any other form.

7. Justify with reason, whether following statement is true or false. (25=10)


a) Reducing assurance engagement risk to zero is very rare.
b) Balance confirmation from debtors/creditors can only be obtained at the end of the financial year.

Answer No. 7
a) The statement is true. Reducing assurance engagement risk to zero is very rare because of the following reasons:
 The use of selective testing;
 The inherent limitations of internal control;
 The fact that much of the evidence available to practitioner is persuasive rather than conclusive; and
 The use of judgment in gathering and evaluating evidence and framing conclusions based on that evidence.

b) The statement is false. Direct confirmation of balances from debtors/creditors in respect of balances standing in
their accounts at end of the financial year end is perhaps the best method of ascertaining whether the balances are genuine
accurately stated and undisputed where internal control system is relatively weak. The conformation date, methods of
obtaining confirmation etc. are to be determined by the auditor. Debtors/creditors may be requested to confirm balance
either as at the date of balance sheet or any other selected date which reasonably close to the date of balance sheet. The
date should be settled by the auditor in consultation with the entity. Therefore, it is not necessary that balances of
debtors/creditors should be obtained and verified at the end of financial year.
The Institute of Chartered Accountants of Nepal
Suggested Answers of Corporate and Other Laws
CAP II Examination – June 2011

1.
a) The Registrar of Companies issued a certificate of incorporation on 8th January, 2011. However, by mistake, the
certificate was dated "5th January 2011" and the allotment of shares was made on 7 th January 2011. Can the allotment be
declared void on the ground that it was made before the company was incorporated? Explain with reference to the
provisions of the Companies Act, 1956? 5

b) There are various legal relations between companies based on shareholdings. Based on relevant provisions of
Companies Act, 1956 in reference, state the relations in the following cases of:
i) Gaumati Company Ltd. incorporated in 1950 have equal equity and preference shares in its share capital. Both
shareholders are equal voting rights in the general meeting of the company. Fedi Lal Company Ltd. is holding 100% of
preference share and 20% of equity shares. Does this relation create holding subsidiary relationships? 5

ii) Binayak Company Ltd. is subsidiary of Amar Company Ltd., Chadani Company Ltd. is subsidiary of Binayak
Company Ltd. Damodaran Company Ltd. is Subsidiary of Chadani Company Ltd. What is the relation of Amar Company
Ltd. and Damodaran Company Ltd? 5

c) CG Company Ltd. had issued a cheque for Rs. 30.00 lacs to Mr. X in full discharge of the debt owed by the company to
Mr. X. The cheque was presented to the Bank for payment however it was dishonored for insufficiency of funds. Advice
Mr. X regarding the provisions of the Negotiable Instruments Act, 1881 for taking a legal recourse. 5

d) Enumerate the circumstances in which a company may be wound up by the Tribunal as per provisions of the
Companies Act, 1956? 5

Answer No 1
a) The allotment of shares is valid. Section 35 of the Companies Act, 1956 provides that a certificate of incorporation
issued by the Registrar in respect of any association shall be conclusive evidence of the fact that all the requirements of the
Act have been complied with respect of registration and matters precedent and incidental thereto, and that the association is
a company authorized to be registered and duly registered under the Act.

The Certificate of Incorporation is conclusive evidence as to all administrative acts relating to incorporation and as to the
date of incorporation. In Jubilee Cotton Mills v. Lewis (1924) A.C.958, the Registrar issued on 8 th January a certificate
dated 5th January. It was held that an allotment of shares made on 7 th January could not be declared void on the ground that
it was made before the company was incorporated. The certificate is not, however, conclusive evidence of that all the
objects of the company, as set out in the memorandum, are legal. (Bowen v. Secular Society (1917) A.C. 406, 432)

b) (i) Companies established before enactment of Indian Companies Act, 1956 are called existing companies for the
purpose of Companies Act, 1956. According to provision of Sec. 4(b)(i), existing company having provision that preference
share has voting right in general meeting as equity shares, holding-subsidiary relation to be counted based on combined
voting powers. In this case, Fedi Lal Company Ltd. is holding 100% of preference share and 20% of equity shares. In the
absence of further information, assuming preference shares and equity shares are in equal weight, Fedi Lal Company Ltd
has 60% voting rights in general meeting, so it is holding company for Gaumati Company Ltd.
(ii) Subsidiary of Subsidiary company is subsidiary for the first holding company. According to provision of Sec. 4(c) and
illustration thereto of Companies Act, 1956, Amar Company Ltd. and other relation is as follows:
Amar Company Ltd. Holding of all companies
Subsidiary
Binayak Company Ltd. Holding of Chadani and Damodaran companies
Subsidiary
Chadani Company Ltd. Holding of Damodaran companies
Subsidiary
Damodaran Company Ltd.

c) According to the Section 138 of the Negotiable Instruments Act, 1881,where any cheque drawn by a person on an
account maintained by him with a banker for payment of any amount of money to another person from out of that account
for the discharge, in whole or in part, of any debt or other liability, is returned by the bank unpaid either because of the
amount of money standing to the credit of that account is insufficient to honour the cheque or that it exceeds the amount
arranged to be paid from that account by an agreement made with that bank, such person shall be deemed to have committed
an offence and shall, without prejudice. to any other provision of this Act, be punished with imprisonment for a term which
may extend to two year, or with fine which may extend to twice the amount of the cheque, or with both:

NMG P.T.O.
(23)

Provided that nothing contained in this section shall apply unless-


(a) the cheque has been, presented to the bank within a period of six months from the date on which it is drawn or within the
period of its validity, whichever is earlier;
(b) the payee or the holder in due course of the cheque as the case may be, makes a demand for the payment of the said
amount of money by giving a notice, in writing, to the drawer of the cheque, within 30 days of the receipt of information by
him from the bank regarding the return of the cheque as unpaid; and
(c) the drawer of such cheque fails to make the payment of the said amount of money to the payee or, as the case may be, to
the holder in due course of the cheque, within fifteen days of the receipt of the said notice.
Explanation.-For the purposes of this section, "debt or other liability" means a legally enforceable debt or other
liability.

As per provisions of Section 138 of the Negotiable Instruments Act mentioned above dishonour of a cheque for non
availability of funds is an offence for which Mr. X can sue the company subject to compliance of the conditions mentioned
therein. Hence Mr. X should comply with the conditions mentioned in the proviso to Section 138 and if the cheque is
dishonoured even thereafter then he can initiate legal remedies.

d) A company may be wound up by the Tribunal under the following circumstances ( Section 433)
(a) if the company has, by special resolution, resolved that the company be wound up by the tribunal;
(b) if default is made in delivering the statutory report to the Registrar or in holding the statutory meeting;
(c) if the company does not commence its business within a year from its incorporation, or suspends its business for a
whole year;
(d) if the number of members is reduced, in the case of a public company, below seven, and in the case of a private
company, below two;
(e) if the company is unable to pay its debts;
(f) if the tribunal is of the opinion that it is just and equitable that the company should be wound up;
(g) if the company has made a default in filing with the registrar its balance sheet and profit and loss account or annual
return for any five consecutive financials years;
(h) if the company has acted against the interests of the sovereignty and integrity of India, the security of the State, friendly
relations with foreign States, public order, decency or morality;
(i) if the Tribunal is of the opinion that the company should be wound up under the circumstances specified in section 424
G;
Provided that the Tribunal shall make an order for winding up of a company under clause (h) on application made by the
Central Government or a State Government.

Part : "B"

2.
a) M/s Duggar Enterprises has entered into a contract for carriage of food items from Nepalgunj to Kathmandu, with
United Transport Ltd. This arrangement has been continuing for the the last three years. During Kartik 2067 due to Tihar
festival, demand for fresh fruit emerges in Kathmandu as a result the unit located at Neplagunj sends one truckload of fresh
fruit worth Rs. 10.00 lacs in the truck meant for supplying foodgrains using the same contract. The driver of the truck while
being midway goes and visits his sister, living in the vicinity of Butwal, on Bhai Tika day. The act of the driver delays the
delivery of goods by two days as a result the whole truckload of fruits gets rotten and damaged? Examine the liability of the
transporter as per the Contract Act, 2056? 8

b) What conduct of worker or employee shall be deemed as misconduct under Labour Act, 2048? Discuss briefly. 7

Answer No.2

a) The liability of a transporter as limited by the Contract Act 2056 is elaborated as follows:

Section 68. Liability of transporter to be limited:


(1) Except when the owner of goods or his/her agent has clearly declared at the time of concluding the contract that the
goods to be transported are worth more than Ten Thousand Rupees or except when otherwise provided for in the contract,
the amount of compensation to be paid by a transporter for any loss or damage to goods transported by him/her shall not
exceed Ten Thousand Rupees.

(2) Notwithstanding anything contained in Sub-section (1), the transporter shall not be held liable for any loss or
damage to gold, sliver, diamond jewels or goods made thereof, precious stone, negotiable instruments, securities, documents
registered by offices, certificates issued by educational and other institutes, coins, bank notes, postal stamps, fish, meat,
fresh fruits and vegetables, insecticide, inflammable materials, petroleum products, precious art pieces, idols, curio goods or
fragile goods, wildlife and pets, handicrafts, arms and ammunition, explosives, radio, television, computer and similar other
goods and their spare parts, machinery and goods specified in prevailing law as those which have to be declared by the
(24)

owner before their transportation, except when the owner or his agent has clearly declared them at the time of signing the
contract or at the time of handing them over for transportation to the transporter.

(3) For the purpose of bearing the risk involved in the transportation of goods mentioned in Sub-section (2), the
transporter might have them insured against the risk through their owner or his/her agent or by him/herself by collecting a
separate fee for the purpose, or take other necessary arrangement to avoid the risk.

Considering the above mentioned provisions the transporter may not be liable for compensation as per Section
68(2) mentioned above.

b) As per section 51 of the Labour Act, 2048, for the purpose of section 50, the following conduct of
the worker or employee shall be deemed as misconduct:
(a) In case of any bodily harm or injury or fetters or detains to the Proprietor, Manager or Employee of the Enterprise with
or without use of arms or injury or causes any violence or destruction or assault within the Enterprise in connection with the
labour dispute or on any other matter;
(b) In case creates or causes to create any stir within the Enterprise with an intention or affecting the production process or
service works of the Enterprise, or prevents the supply of food and water, or connection of telephone and electricity, or
obstructs the entry into or movement within the Enterprise;
(c) In case steals the property of enterprise;
(C1) If commits embezzlement in the transactions of the Enterprise,
(C2) If absents in the Enterprise more than a consecutive period of thirty days without notices.
(d) In case accepts or offers brides;
(e) In case imprisoned on being convicted on a criminal offence Involving moral turpitude.
(f) In case participates or compels any other person to participate
in any authorized strike or in a strike which is declared illegal;
(g) In case strikes without fulfilling the legal requirements or intentionally slow down the work against interests of the
Enterprise;
(h) In case intentionally destroys any property of the Enterprise, or causes damage thereon or takes and uses it outside the
Enterprise or gives its use to unauthorized person without permission of the competent person;
(i) In case intentionally violates intentionally the orders or directives issued under this Act or the Rules made hereunder
frequently, or the Bylaws made by the Enterprise, or misbehaves with the customers of the Enterprise;
(j) In case remains absent from the work frequently without obtaining permission or comes late after the regular time;
(k) If it is certified by a doctor that one has come to the duty after consuming or has consumed alcoholic substances during
the working time.
(l) In case performs any activity with a motive of causing damage to secrecy relating to special technology of the Enterprise,

(m) In case abuses any items which has been kept for the interest, Health and safety of the workers or employees or causes
damage to them intentionally;

3.
a) What are the liabilities for the matters referred to in the prospectus as per the Securities Act, 2063? 5

b) What are the circumstances where attendance or voting of shareholders in General meeting is restricted under
Companies Act, 2063? Explain briefly. 5

Answer No.3
a) The liabilities for the matters referred to in the prospectus as laid out in Section 33 of the Securities Act, 2063 are as
follows:

(1) The concerned body corporate and the director signing a prospectus and the expert preparing such a prospectus shall be
severally and collectively liable for the truth of the details and documents underpinning the information set down in the
prospectus submitted to the Board for the purpose of registering securities with the Board and obtaining permission to issue
such securities.
(2) Where any person who subscribes for any securities on the faith of the matters set down in the prospectus subsequently
sustains any loss or damage by the reason that the matters set down in the prospectus have been set down with mala fide
intention or untrue or false statements have been included therein knowingly, the body, director or experts preparing the
prospectus shall be liable to pay compensation for such loss or damage.
Provided that no director shall be liable to pay such compensation if he or she proves that he or she has resigned prior to
making a decision on the matters set down in the prospectus with ulterior motive or knowingly or that he or she did not
know that the prospectus was untrue.
(25)

(3) Where any investor sustains any loss or damage by the reason that the prospectus, information, statements or returns
submitted by a body corporate to the Board, such an investor may make a petition to the concerned District Court for
compensation within thirty five days from the date of knowledge within one year after the making of investment.

b) The companies Act, 2063, provides the following cases where attendance or voting in meeting is restricted (Section 70):
(1) No person shall be entitled to attend and vote in any general meeting, in the capacity of a shareholder, either in person or
by proxy on any discussion to be held in respect of any terms and conditions entered into or to be entered into between
him/herself and the company.
(2) No director or his/her partner or his/her proxy shall be entitled to vote on any discussion to be held at any General
Meeting in respect of the responsibility for any act done or omitted to be done or done wrongfully by him/her or in respect
of his/her own appointment, dismissal, transfer or confirmation, with respect to the provision of, or reduction or increment
in remuneration, allowance or bonus or in respect of any agreement, contract or arrangement regarding his employment or
anything in which his interest or concern is involved.
(3) Any shareholder who has not paid calls on the shares shall not be entitled to attend and vote in the general meeting.
(4) Where any shareholder appoints a director of the concerned company as his proxy pursuant to Sub-section(2) of Section
71, such director shall not be entitled to vote in the general meeting ,as a proxy of any one , on any matter in which his/her
interest or concern is involved or on the matter of his/her appointment.
(5) Notwithstanding anything contained in this Act or the prevailing law, where a bank or financial institution incorporated
under the prevailing law, which institutes legal action against a shareholder, who has borrowed a loan from such bank or
financial institution against the pledge or security of the shares held by him, for his default in repaying the loan, writes to the
concerned company to prevent him from exercising voting right in respect of shares, then the company shall prevent such
shareholder from exercising voting right in respect of the shares held by him/her for a period until he/she repays the loan.

4.
a) What are the procedures for settlement of disputes as per the Foreign Investment & Technology Transfer Act,
2049? 5

b) Write the procedure for amending the Memorandum of Association of a Limited liability company as per
Companies Act, 2063. 5

Answer No 4
a) The procedure for settlement of disputes as provided in Section 7 of the Foreign Investment &
Technology Transfer Act, 2049 are as follows:
1) If any dispute arises between a foreign investor, national investor or the concerned industry, the concerned parties shall
be required to settle the dispute by mutual consultations in the presence of the department.
2) If the dispute cannot be settled in the manner referred to in sub-section (1) above, it shall be settled by arbitration in
accordance with the prevailing arbitration rules of the United Nations Commission on International Trade Law
(UNCITRAL).
3) The arbitration shall be held in Kathmandu. The laws of Nepal shall be applicable in the arbitration.
4) Notwithstanding anything contained in sub-section (1), (2) and (3) above, disputes arising in regards to foreign
investments made in the industries with investments as prescribed may be settled as mentioned in the foreign investment
agreement.

b) According to the provision of Sec. 21 of Company Act, 2063, procedure for amending the
Memorandum of Association is as follows:
 Company at its general meeting with special resolution may change the clauses of MOA.
 The name of the company can be changed upon special resolution and if pre-authorised by the Registrar. Pre-authority
can be taken upon payment of prescribed fees.
 The resolution, so passed shall be laid to the notice of the Registrar within 30 days of date of resolution and the
Registrar shall record it within 7 days of notice so received.
 In case any aggrieved shareholder (holding not less than 5%of the paid up capital) applied to the court, the resolution
shall be suspended till date of court‟s final verdict.

5.
a) Enumerate the disqualification to become a director as listed in the Banks and Financial Institutions Act, 2063?
9

b) State the provisions with respect to contingent contracts as provided in the Contract Act, 2056? 6

Answer No.5
(26)

a) Section 18 of the Banks and Financial Institutions Act, 2063, provides the following provisions with regard
Disqualification of directors:
(1) Any of the following persons shall not be eligible to be elected or nominated to the office of, or to continue to hold the
office of, a director:
(a) Who is below 21 years of age;
(b) Who is of unsound mind or is insane;
(c) Who is a declared insolvent;
(d) Who has been blacklisted in connection with any transaction with any bank or financial institution, and a period of at
least three years has not lapsed after his or her name has been removed from that list;
(e) Who is a director of any bank or financial institution or any financial institution carrying on the transactions relating to
any kind of deposits or an employee serving in such institution or a serving auditor or advisor of any other bank or financial
institution;
(f) Who is a partner in any kind of contract agreement with the concerned bank or financial institution;
(g) Who has acquired membership of the Stock Exchange to act as a securities dealer;
(h) Who has not subscribed the minimum number of shares required to be subscribed to be eligible for appointment as a
director pursuant to the articles of association of the bank or financial institution;
(i) Who is a serving employee of the Government of Nepal, Rastra Bank or bank or financial institution;
Provided that this provision shall not be applicable to the nomination of any person made by the Government of Nepal as a
director of any bank or financial institution of which shares have been purchased by it, or to the nomination of any person
made by any bank or financial institution as a director of any other bank or financial institution of which shares have been
purchased by it;
(j) In the case of a person who has been relieved of or retired from the service of the Rastra Bank after serving in the post of
Governor, Deputy-Governor or Special Class employee, at least one year from the date of such relief or retirement from the
service has not elapsed;
(k) Who, having a liability to pay tax pursuant to the laws in force, has failed to pay the same;
(l) Who is convicted by a court of an offence involving moral turpitude and sentenced in respect thereof, and a period of five
years has not elapsed from the expiry of the sentence;
(n) Who is convicted by a court of an offence of corruption or cheating;
(o) A period of five years has not lapsed after the date on which the Rastra Bank has taken action against him or her for
failure to observe a direction of the Rastra Bank or the date of suspension or dissolution after taking management by the
Rastra Bank under its control.
(2) No person shall continue to hold the office of a director in any of the following circumstances:
(a) If he or she suffers from any disqualification for appointment to the post of director as mentioned in sub- section (1);
(b) If the general meeting passes a resolution to remove him or her from the office of director;
(c) If the resignation tendered by the director is accepted;
(d) If he or she is held by a court to have done any act involving dishonesty or ulterior motive in the activities of the bank or
financial institution;
(e) If he or she does any act prohibited by this Act from being done by a director.

b) The Provisions with regard to Contingent Contracts under Section 12 of the Contract Act, 2056, are as follows:

(1) In case a contract has been concluded for performing or not performing any work if any event happens in the future, the
contract shall not create any liability until such event happens.
(2) In case a contract has been concluded subject to the condition that it shall be deemed to have been concluded in case any
person performs any specified work in the future, no liability shall be deemed to have emerged from that contract if such
person does anything in such a manner as not to perform that work or in such a manner that the work cannot be performed.
(3) In case a contract has been concluded to perform or not to perform any work if any uncertain event does not happen in
future, liability under that contract shall emerge only after the happening of that event becomes impossible.
(4) In case a contract has been concluded with a provision to perform or not to perform any work if any event happens
within a specified period in the future, the contract shall be deemed to have become invalid after the happening of that event
becomes impossible within the specified period or after the expiry of that period.
(5) In case a contract has been concluded with a provision to perform or not to perform any work if any event does not
happen within a specified period in the future, liability under such contract shall emerge if that event does not happen within
that period or if it becomes certain that the event will not happen within that period.

6.
a) What types of establishment require to pay bonus? Can the punishment of establishment for any offence under Bonus
Act, 1974 be inflicted on the General Manager? 5

b) State the composition of council of Institute of Chartered Accountants‟ of Nepal and their tenure of office. 5

Answer No.6
(27)

a) Establishment which makes Profits is eligible to Pay Bonus (Section 5). The other provisions are as
follows
1. Every establishment which makes a profit shall allocate an amount equal to ten percent of the net profit made during a
fiscal year for payment of bonus to its employees.
2. The following amount also shall be debited from the net income Tax Act while determining the net profit made during
any fiscal year by an establishment under Sub-Section (1).
a. Amount set aside for providing quarters to employees under Sub-Section (1) or Section 41 of the Labor
Act 2048.
b. Deleted …….
c. Amount of bonus paid in excess under Sub-Section (3) of Section 11.
3. Notwithstanding anything contained in Sub-Section (1), the percentage and other matters relating to bonus payable by
an establishment owned by Government shall be as decided by Government.

Responsibility of General Manager (Section 23)


In case any establishment is to be punished for any offense punishable under this act, such punishment shall be inflicted on
the person who was functioning in the capacity of its general Manager at the time when such offense was committed.
Provided that a General Manager who is able to prove that such offense was committed without his knowledge or consent
shall not be liable to punishment under this section.

b) According to Sec. 7 of Institute of Chartered Accountants of Nepal Act, 2053, the composition of Council shall be as
follows:
(a) Ten persons elected by and amongst Chartered Accountant members -Member
(b) Four persons elected by and amongst Registered Auditors -Member
(c) Three persons nominated by Government of Nepal, upon the recommendation of the Auditor General, from amongst
the persons well experienced in the field of accounting profession. -Member
The Council members shall elect a President and a Vice-President from the Fellow Chartered Accountants (F.C.A) Council
members.
The term of office of the President and the Vice-President shall be of one year and upon expiry of the term of office, they
shall be eligible to be elected for one more term.
The term of office of the Council members shall be of three years and upon expiry of the term of office, they shall be
eligible to be re-elected or re-nominated.

7. Write short notes on: (3×5=15)

a) Methods of Audit under Audit Act, 2048


b) Payment of Insurance claims after the cancellation of the Insurer
c) Negotiation

Answer No 7
a) Under Section 3 of Audit Act 2048.
1. The auditor general may conduct final audit of the financial activities and other activities relating thereto of the offices,
bodies or organizations under its jurisdiction, either in detail or sporadically or in samples, prescribe scope, methodology
and timing of audit and present the facts obtained there from make critical comments thereon and submit its reports.
2. The auditor general may, if it deems necessary in course of audit, exercise the following powers

a) To check at any time the status of the program and project being operated under the grants obtained by Nepal
Government and examine documents relating to accounts;
b) To require contractors of government contracts to produce relevant documents or other evidence relating to the contract,
which are supposed to be in his possession;
c) To hire services of any expert on the task of audit and, if necessary, engage him under contract with reasonable
remuneration.

b) According to Sec. 16 of Insurance Act, 2052 regarding Payment of Insurance Claims After the
Cancellation of the Insurer, the Insurer, dissolved by the cause of the cancellation of its registration
pursuant to Section 13, shall refund the amount received by it for Insurance to the person, organization
or the Board, within the period and method specified by the Board. It shall refund the principal amount
along with bonus as specified by the Board in the case of Life Insurance and it shall refund the
principal amount as specified by the Board on a proportional basis in the case of Non-Life Insurance.
(28)

c) Negotiation means the act of transfer of Negotiable Instruments to any person so as to constitute
that person the holder thereof. When the instrument is transferred either by delivery or by endorsement
and delivery so as to constitute the transferee as a holder thereof, the instrument is said to be negotiated.
An instrument can be negotiated in following ways-
i) Delivery: The function of making, acceptance or endorsement of a Negotiable Instrument shall be regarded to be
completed only after its delivery.
ii) Negotiation by Delivery: A Negotiable instrument payable to bearer is negotiable by delivery thereof. If the Negotiable
Instrument is payable to the person written on the instrument or to one ordered by him/her, such instrument should be
endorsed by the holder.
iii) Effect of Endorsement: The right shall be transferred after the endorsement of a Negotiable Instrument followed by
delivery. The holder may give such instrument to any other by endorsing it or receive payment by himself.
The Institute of Chartered Accountants of Nepal
Suggested Answers of Cost and Management Accounting
CAP II Examination – June 2011

1) Alpha Limited has prepared the following budget estimates for the coming year:
Product A Product B
Sales (in units) 6,000 16,000
Rs./Unit Rs./Unit
Selling Price 400 640
Direct Materials 120 220
Direct Wages @ Rs. 10 per hour 80 120
Variable Overheads 40 60
Fixed Overhead 80 120
Total Cost 320 520
Profit 80 120
After finalization of the above budget estimates, it is observed that 1/3rd of the production capacity are still idle. In order to
improve the performance, the following proposals are under consideration:

a) Product A will be discontinued and the capacity so released will be used for product B. The selling price of product B
will, however, have to be reduced by Rs. 20 per unit in order to increase the volume of sales.
b) Product B will be discontinued and the capacity so released will be diverted to the production of product C. The
particulars relating to per unit of product C are as under:
Selling Price Rs. 520 Direct Wages Rs. 100
Direct Materials Rs. 150 Variable Overheads Rs. 50
c) The idle capacity will be utilised for meeting an export demand for product D. The particulars relating to per unit of
product D are as under:
Selling Price Rs. 720 Direct Wages Rs. 200
Direct Materials Rs. 400 Variable Overheads Rs. 100
d) The idle capacity will be hired out by fixing a price in such a way that the same rate of profit per direct labour hour as
obtained in the budget estimates is achieved.

Required: (4+15+1=20)
i) Prepare a statement showing the profitability of the products A & B as envisaged in the budget estimates.
ii) Evaluate each of the above four proposals separately showing the profitability under each proposal.
iii) Recommend most profitable proposal for Alpha Limited.

Answer:
Alpha Ltd.
(i) Budgeted Profitability Statement
Product A Product B
Sales units 6,000 16,000
Rs./Unit Rs./Unit
Sales price per unit 400 640
Variable cost per unit:
Direct Material 120 220
Direct Wages 80 120
Variable Overheads 40 60
240 400
Contribution per unit 160 240
Total Contribution A = Rs. 160 × 6,000 Rs. 960,000
B = Rs. 240 × 16,000 Rs. 3,840,000
Rs. 4,800,000
Less: Fixed Cost Rs. 2,400,000
Profit Rs. 2,400,000

(ii) Evaluation of proposals


Proposal (a): Discontinue Product A and use the capacity for Product B

Spare capacity because of discontinuing Product A (8 × 6000) 48,000 hours


Additional units of Product B produced = 48,000/12 4,000 units
Total production of Product B = (16,000 + 4,000) units 20,000 units

NMG P.T.O.
(30)

Statement of profitability
Sales unit 20,000
Rs./Unit
Selling Price per unit Rs. (640 – 20) 620
Variable cost: Direct Materials 220
Direct Wages 120
Variable Overheads 60
400
Contribution per unit 220
Total Contribution (Rs. 220 × 20,000) Rs. 4,400,000
Less: Fixed Cost Rs. 2,400,000
Profit Rs. 2,000,000

Proposal (b): Discontinue Product B and use the capacity for new Product C

Spare capacity because of discontinuing Product B (12 × 16,000) 192,000 hrs


Units of Product C produced = 192,000/(100/10) 19,200 units

Statement of Profitability
Product A Product C
Sales units 6,000 19,200
Rs./Unit Rs./Unit
Sales price per unit 400 520
Variable cost per unit:
Direct Material 120 150
Direct Wages 80 100
Variable Overheads 40 50
240 300
Contribution per unit 160 220
Total Contribution A = Rs. 160 × 6,000 Rs. 960,000
C = Rs. 220 × 19,200 Rs. 4,224,000
Rs. 5,184,000
Less: Fixed Cost Rs. 2,400,000
Profit Rs. 2,784,000

Proposal (c): Use idle capacity for export demand of Product D

Idle production capacity (labour hours) 120,000 hrs


Units of Product D produced = 120,000/(200/10) 6,000 units

Statement of Profitability
Product A Product B Product D
Sales units 6,000 16,000 6,000
Rs./Unit Rs./Unit Rs./Unit
Sales price per unit 400 640 720
Variable cost per unit:
Direct Material 120 220 400
Direct Wages 80 120 200
Variable Overheads 40 60 100
240 400 700
Contribution per unit 160 240 20
Total Contribution A = Rs. 160 × 6,000 Rs. 960,000
B = Rs. 240 × 16,000 Rs. 3,840,000
D = Rs. 20 × 6,000 Rs. 120,000
Rs. 4,920,000
Less: Fixed Cost Rs. 2,400,000
Profit Rs. 2,520,000

Proposal (d): Hire out idle capacity


Statement of Profitability
Rs.
(31)

Profit as per Budget Estimates (I) 2,400,000


Budgeted Direct Labour Hours 240,000
Profit per Direct Labour Hour 10
Idle Capacity (Direct Labour Hour) 120,000
Additional profit by hiring out the idle capacity (II) 1,200,000
Total Profit [(I) + (II)] 3,600,000

(iii) Recommendation:
Since proposal (d) gives highest profit i.e. Rs. 3,600,000 compared to the all other proposals, it is recommended to go for
Proposal (d).

Working Notes:
(i) Idle Production Capacity (Labour Hour)
Product A Product B Total
Direct Labour Hour per unit 80/10 = 8 120/10 = 12
Number of units 6,000 16,000
Direct Labour Hours utilised 48,000 192,000 240,000
The above utilised hours is only 2/3rd. Hence Total Capacity
hours =240000/ 2/3rd 360,000
Idle Capacity 120,000

(ii) Fixed Cost


Rs.
Product A = Rs. 80 × 6,000 480,000
Product B = Rs. 120 × 16,000 1,920,000
Total 2,400,000

2)
a) Your company operates for 300 days a year on average. It is facing severe problem of electric power cut in its day to
day operation. The electricity supply is not available for nearly 4 hours per day in average during total working hours of 7
hours per day for whole the year. This situation is expected to prevail for nearly five more years. To cope with this situation,
you are considering the alternative sources of power generation, i.e. 140 KVA Generator Set and you desire to know the
cost per unit of electricity generated.

The following estimations are available:


i) Number of units to be generated per month is 10,000.
ii) Cost of Gen Set with installation charges is Rs.5 million. It is to be fully depreciated within 5 years, after that period, it
can be disposed off for Rs.250,000.
iii) Regular cleaning and repair and maintenance cost per month is Rs.20,000.
iv) The Gen Set will consume 22 Ltrs. of diesel per hour. The cost of diesel per liter is Rs.65. Other fuel charge is Rs.65
per hour.
v) Two staffs are directly involved in operation and maintenance of the Gen Set, who are paid salary at the rate of
Rs.10,000 per month each.
vi) Share of administrative charges is Rs.10,000 per month.
5

b) A Company produces two joint products P and Q in 70 : 30 ratio from basic raw materials in department A. The input
output ratio of department A is 100 : 85. Product P can be sold at the split of stage or can be processed further at department
B and sold as product AR. The input output ratio is 100 : 90 of department B. The department B is created to process
product A only and to make it product AR.
The selling prices per kg. are as under:
Product P Rs. 85
Product Q Rs. 290
Product AR Rs. 115
The production will be taken up in the next month.
Raw Materials 8,00,000 kgs.
Purchase price Rs. 80 per kg.

Monthly expenses:
Dept. A Dept. B
Rs. in lakhs Rs. in lakhs
(32)

Direct Materials 35.00 5.00


Direct Labour 30.00 9.00
Variable overheads 45.00 18.00
Fixed overheads _40.00 _32.00
Total 150.00 64.00

Selling Expenses:
Rs. in lakhs
Product P 24.60
Product Q 21.60
Product AR 16.80

Required: (4+4=8)
i) Prepare a statement showing the apportionment of joint costs.
ii) State whether it is advisable to produce product AR or not.

c) A telecom company in Nepal has total GSM prepaid active subscriber base of around 5 million. Its Average Revenue
Per User (ARPU) is Rs.238.50 per month. Assuming that its variable cost and profit are 35% and 40% respectively of total
revenue in this segment of operation, you are required to calculate the number of subscriber to be added to justify the 20%
reduction in average call tariff which is currently Rs.1.50 per minute. Also assume that the proposed reduction in tariff will
increase call duration by 20%. 7

Answer:

a) Calculation of the cost per unit of electricity generated


Units generated: 10000/ month
SN Particulars Cost per month Cost per unit Working Remarks
(Rs.) (Rs.)
a. Cost of diesel 143,000 14.30 (22 Ltr. x 4 hrs. x 300/ 12 days x Rs.65)
b. Other fuel charges 6,500 0.65 (Rs. 65x 4 hrs. x 300/ 12 days)
c. Repair & maintenance 20,000 2.00
d. Staff salary 20,000 2.00 (Rs.10000 x 2)
e. Depreciation charge 79,167 7.92 (Rs.5,000,000-250000/5/12)
f. Shared administrative charge 10,000 1.00
Total 278,667 27.87

b) Input in Dept. „A‟ 800,000 kgs.


Yield 85%
Therefore output = 85% of 8,00,000 = 6,80,000 kgs.
Ratio of output for P and Q = 70 : 30
Product of P = 70% of 6,80,000 = 4,76,000 kgs.
Product of Q = 30% of 6,80,000 = 2,04,000 kgs.

(i) Statement showing apportionment of Joint cost


P Q Total
Product Kgs. 4,76,000 2,04,000
Selling price per kg. Rs. 85.00 290.00
Rs. Lakhs Rs. Lakhs Rs. Lakhs
Sales 404.60 591.60 996.20
Less: Selling expenses __24.60 __21.60 __46.20
Net Sales _380 __570 _950
Ratio 40% 60% 100%

Rs. In lakhs
Raw materials (8,00,000 kgs. ×Rs. 80) 640
Process cost of department „A‟ __150
__790
Apportionment of Joint Cost
(In the ratio of Net Sales i.e. P : Q, 40% : 60%)
joint cost of „P‟ = Rs. 316 lakhs
Joint cost of „Q‟ = Rs. 474 lakhs
(33)

(ii) Profitability statement of further processing of product ‘P’ and converting it into product ‘AR’

Output = 90% of 4,76,000 kgs. = 4,28,400 kgs. Rs. in lakhs


Joint costs (As above) 316.00
Cost of department B 64.00
Selling expenses _16.80

396.80
Sales value (Rs. 115×4,28,400) 492.66
Profit (492.66-396.80) __95.86

If „P‟ is not processed profitability is as under:


Rs. in lakhs
Sales 380.00
Less: joint expenses 316.00
Profit __64.00

Further processing of product „P‟ and converting it into product „AR‟ is beneficial to the company because the profit is
increased by Rs. 31.86 lakhs (95.86-64.00).

c)
Computation Table to find out additional number of subscriber to justify tariff reduction
SN Particulars Total Per User Working Remarks
a. No. of Active Prepaid GSM Subscribers 5,000,000
b. ARPU/ month (Rs.) 238.50
c. Total Revenue/ month (Rs.) 1,192,500,000 (a x b)
d. Variable Expenses/ month (Rs.) 417,375,000 35% of Total Revenue
e. Profit/ month (Rs.) 477,000,000 40% of Total Revenue
f. Fixed Costs/ month (Rs.) 298,125,000 (c-d-e)
g. Contribution/ month (Rs.) 775,125,000 155.03
h. Current call tariff/ minute (Rs.) 1.50
i. Average call duration per user/ month 159 (b/h)
j. New call tariff/ minute (Rs.) 1.2 20% less than current
k. New avg. call duration/ user/ month 191 20% more than current
l. New ARPU/ month (Rs.) 229 (j x k)
m. New contribution/ user/ month (Rs.) 149 65% of New ARPU(100-35)
n. Required profit/ month (Rs.) 477,000,000
o. Required no. of total active subscribers = (298125000+477000000)/ 149 (Fixed Cost+Reqd. Profit)/ CMPU
= 5,208,333
p. No. of additional subscribers required to
justify the 20% tariff reduction 208,333 (o-a)

3)
a) Mega Company has just completed its first year of operations. The unit costs on a normal costing basis are as under:
Rs.
Direct material 4 kg @ Rs.4 16.00
Direct labour 3 hrs @ Rs.18 54.00
Variable overhead 3 hrs @ Rs.4 12.00
Fixed overhead 3 hrs @ Rs.6 _18.00
100.00
Selling and administrative costs:
Variable Rs.20 per unit
Fixed Rs.7,60,000

During the year the company has the following activity:


Units produced 24,000
Units sold 21,500
Unit selling price Rs.168
Direct labour hours worked 72,000
(34)

Actual fixed overhead was Rs.48,000 less than the budgeted fixed overhead. Budgeted variable overhead was
Rs.20,000 less than the actual variable overhead. The company used an expected actual activity level of 72,000 direct
labour hours to compute the predetermine overhead rates.

Required: (4+6+2=12)
i) Calculate under or over absorption of overhead.
ii) Compute the unit cost and total income under:
(a) Absorption costing
(b) Marginal costing
iii) Reconcile the difference between the total income under absorption and marginal costing.

b) A company is engaged in manufacturing two products A and B. Product A uses one unit of component X and two units
of component Y. product B uses two units of component X and one unit of component Y and two units of component Z.
component Z which is assembled in the factory uses one unit of component Y.
Components X and Y are purchased from the market. The company has prepared the following forecast of sales and
inventory for the next year:
Product A B
(units) (units)
Sales 80,000 1,50,000
Stock at the end of the year 10,000 20,000
Stock at the beginning of the year 30,000 50,000

The production of both the products and the assembling of the component Z will be spread out uniformly
throughout the year. The company at present orders its inventory of X and Y in quantities equivalent to 3 months
production. The company has complied the following data related with the two components:
X Y
Price per unit (Rs.) 20 8
Order placing cost per order (Rs.) 1,500 1,500
Carrying cost per annum 20% 20%

Prepare production budget, component budget and calculate EOQ of components X & Y. 8

Answer:
a)
i) Under or over absorption of overhead: Rs.
Budgeted Fixed Overhead 72,000 Hrs. × Rs.6 4,32,000
Less: Actual Overhead less than Budgeted Fixed Overhead __ 48,000
Actual Fixed Overhead 3,84,000
Budgeted Variable Overhead (72,000 Hrs. × Rs.4) 2,88,000
Add: Actual Overhead higher than Budgeted 20,000
Actual Variable Overhead 3,08,000

Fixed & Variable Overhead applied 72,000 Hrs × Rs.10 7,20,000


Actual Overhead (3,84,000 + 3,08,000) 6,92,000
Over Absorption __28,000

ii) Computation of Unit Cost & Total Income


Unit Cost Absorption Costing Marginal Costing
(Rs.) (Rs.)
Direct Material 16.00 16.00
Direct Labour 54.00 54.00
Variable Overhead 12.00 12.00
Fixed Overhead 18.00 ____-
Unit Cost 100.00 82.00
Income Statements
Absorption Costing
Sales (21500 × Rs.168) 36,12,000
Less: Cost of goods sold (21500 × 100) 21,50,000
Less: Over Absorption ___28,000 __21,22,000
14,90,000
Less: Selling & Distribution Expenses (2,500×20=760,000) 11,90,000
Profit 3,00,000
(35)

Marginal Costing
Sales 36,12,000
Less: Cost of goods sold (21500×82) 17,63,000
Add: Under Absorption 20,000 17,83,000
18,29,000
Less: Selling & Distribution Expenses (Variable only) 4,30,000
Contribution 13,99,000
Less: Fixed Factory and Selling & Distribution
Overhead (3,84,000 + 7,60,000) 11,44,000
Profit 2,55,000

(iii) Reconciliation of Profit


Difference in Profit: = Rs.3,00,000 – 2,55,000
= Rs.45,000

Due to Fixed Factory Overhead being included in Closing Stock in Absorption Costing but not in Marginal Costing such
difference arises. Therefore, difference in Profit:
= Fixed Overhead Rate (Production – Sale)
= 18× (24,000 – 21,500)
= 45,000

b) Production Budget
Product “A” Product “B”
Sales 80,000 1,50,000
Add: Closing stock 10,000 20,000
Less: Opening stock 30,000 50,000
Production Budget 60,000 1,20,000
Budget of Component
Component X Y Z
Product A: Production 60,000 units 60,000 1,20,000
Product B: Production 1,20,000 units 2,40,000 1,20,000 2,40,000
Component Z: 2,40,000 units 2,40,000
Total 3,00,000 4,80,000 2,40,000

Optimal order quantity of components X & Y


Component X Y
Order placing costs Rs. 1,500 1,500
Price of the component Rs. 20 8
Carrying cost @ 20% Rs. 4 1.60

X Y
(2  3,00,000  1,500) (2  4,80,000 1,500)
EOQ =
4 1.60
= 15,000 = 30,000

4)
a) At the end of first year on 31st March, 2011 in the books of ABC Constructions Ltd. the Bridge Contract Account stands
debited with the cost of material issued, labour, overheads expended and plant issued and it stands credited with material at
site Rs. 25,000; material returned Rs. 15,000 and plant at site Rs. 476,000 after charging depreciation at 15 percent. The
material issued, labour, overheads and plant issued debited to the contract account, are in the ratio of 5: 4: 2: 4. The
contractee's architect had certified 75 percent of the contract as completed at the end of the year and 90 percent of the
certified work value had been received in cash Rs. 1,620,000. The accounts department informs that 2/3 of the profit on cash
basis credited to Profit and Loss account on the contract is Rs. 213,600

You are required to prepare the Bridge Contract Account showing the cost of work done but uncertified. 8

b) The existing incentive system of Beta Ltd. is as under:


Normal working week : 5 days of 8 hours each plus 3 late shifts of
3 hours each.
Rate of payment : Day work: Rs. 160 per hour
(36)

: Late shift: Rs. 225 per hour


Average output per operator for 49 hours
week i.e. including 3 late shifts : 120 articles.

In order to increase output and eliminate overtime, it was decided to switch on to a system of payment by results. The
following information is obtained:
Time-rate (as usual) : Rs. 160 per hour
Basic time allowed for 15 articles : 5 hours
Piece-work rate : Add 20% to basic piece-rate
Premium Bonus : Add 50% to time.

Required:
Prepare a statement showing hours worked, weekly earnings, number of articles produced and labour cost per article for one
operator under the following systems:
i) Existing time-rate
ii) Straight piece-work
iii) Rowan system
iv) Halsey premium system
Assume that 135 articles are produced in a 40 hour week under straight piece work, Rowan premium system, and Halsey
premium system above and worker earns half the time saved under Halsey premium system. 8

c) X Ltd. is reviewing its stock policy, and has the following alternatives available for the
evaluation of stock:
i) Purchase stock twice in a month, 400 units.
ii) Purchase monthly, 800 units
iii) Purchase every three months, 2,400 units
iv) Purchase every six month, 4,800 units
v) Purchase annually, 9,600 units
It is ascertained that the purchase price per unit is Rs. 40 for deliveries up to 2,000 units. A 5% discount is offered by the
supplier on the whole order where deliveries are 2,001 to 4,000 units and 10% reduction on the total order for deliveries in
excess of 4,000 units. Each purchase order incurs administration costs of Rs. 250. Interest on capital and other storage costs
are Rs. 12.50 per unit of average stock quantity held.
Calculate the optimum order size. 4

Answer:
a) Bridge Contract Account for the year ended 31 st March, 2011
Particulars Rs. Particulars Rs.
To Material issued 700,000 By Material at site 25,000
To Labour 560,000 By Material returned 15,000
To Overheads 280,000 By Plant at site 476,000
To Plant issued 560,000 By Work-in-Progress A/c:
To Notional Profit [W.N. 2] 356,000 Work certified 1,800,000
Work uncertified 140,000* 1,940,000
2,456,000 2,456,000
To P/L A/c 213,600 By Notional Profit b/d 356,000
To Reserve A/c 142,400
356,000 356,000
* Balancing figure.

Working Notes:
1. Calculation of the amount of Material issued, Labour, Overheads and Plant issued:
(i) Cost of Plant issued = Rs. 476,000 × 100/85 = Rs. 560,000. [Depreciation @ 15%]

(ii) The ratio of Material issued, Labour, Overheads and Plant issued had been given 5 : 4 : 2 :
4. So, the element wise amount would be:
Material issued Rs. 560,000 × 5/4 = Rs. 700,000
Labour Rs. 560,000 × 4/4 = Rs. 560,000
Overheads Rs. 560,000 × 2/4 = Rs. 280,000
2. Calculation of amount of Notional Profit:
Profit taken to P/L A/c = Notional Profit × 2/3 × Cash Received/Work Certified.
Rs. 213,600 = Notional Profit × 2/3 × 90/100.
Notional Profit = Rs. 213,600 × 3/2 × 100/90
= Rs. 356,000.
(37)

3. Value of Work certified: Cash received being 90% of Work certified.


Cash Received × 100/90
= Rs. 1,620,000 × 100/90 = Rs. 1,800,000

b) Table showing Labour cost per article:

Method of payment Hours Weekly Number of Labour cost


Worked earnings articles per article
Rs. produced Rs.

Existing time rate 49 8,425.00 120 70.21


Straight piece rate system 40 8,640.00 135 64.00
Rowan Premium system 40 9,007.41 135 66.72
Halsey Premium system 40 8,600.00 135 63.70
Working notes:

Existing time rate Rs.


Weekly wages 40 hours @ Rs. 160 per hour 6,400
9 hours @ Rs. 225 per hour __2,025
8,425
Piece Rate system:
Basic time: 5 hours for 15 articles (120/40hour=3hr/Article)
Cost of 15 articles at hourly rate of Rs. 160/hr. 800
Add 20% __160
960

Rate per article = Rs. 960/15 = Rs. 64


Earning for the week = 135 articles ×Rs. 64 = Rs. 8,640

Rowan Premium system:


Basic Time : 5 hours for 15 articles
Add : 50% to time
7.5 hours for 15 articles
Or 30 minutes per article
Time allowed for 135 articles = 67.5 hours
Actual time taken for 135 articles = 40 hours

 TA  HW 
Earnings = (HW ×RH) +   HW  RH 
 TA 
 67.5  40 
= (40 hrs. ×Rs. 160) +   40  Rs .160 
 67.5 
= Rs. 9,007.41

Halsey Premium System:


50
Earnings = HW ×RH + (TA –HW) ×RH
100
= 40× Rs. 160+ ½ (67.5 -40) ×Rs. 160
= Rs. 8,600

c) The purchase cost is not constant per unit. It is therefore, not possible to use the EOQ formula. For optimum order
size statement of cost is prepared as follows:
Annual Purchase Cost Storage Admin Total Cost
Order size No. of Order
Rs. Rate/unit Cost (Rs) Cost (Rs) (Rs)
9,600 1 345,600 36 60,000 250 405,850
4,800 2 345,600 36 30,000 500 376,100
2,400 4 364,800 38 15,000 1,000 380,800
800 12 384,000 40 5,000 3,000 392,000
400 24 384,000 40 2,500 6,000 392,500
The optimum order size is 4,800 units which cost Rs. 376,100 in total.
(38)

Working Notes:
(i) Order size up to 2,000 units, Purchase price is Rs. 40
Order size 2,001 – 4,000 units, Purchase price is Rs. 40 – 5% Discount = Rs. 38.
Order size greater than 4,000 units, Purchase price is Rs. 40 – 10% Discount = Rs. 36.
(ii) Storage cost: Rs. 12.50 per unit of average stock
For order size 400 units, Average stock 200 units × Rs. 12.50 = Rs. 2,500 and so on.

5) Distinguish between: (4×2.5 =10)


a) Opportunity cost and Imputed cost
b) Cost control and Cost reduction
c) Waste and Scrap.
d) Job costing and Process costing

Answer
a) Opportunity cost refers to the value of sacrifice made or benefit of opportunity forgone in accepting an alternative
course of action. For example, a firm financing its expansion plan by withdrawing money from its bank deposits. In such a
case the loss of interest on the bank deposit is the opportunity cost for carrying out the expansion plan. It is the cost of any
activity measured in terms of the best alternative forgone. It is the sacrifice related to the second best choice available to
someone who has picked among several mutually exclusive choices. The notion of opportunity cost plays a crucial part in
ensuring that scarce resources are used efficiently. Thus opportunity costs are not restricted to monetary or financial costs:
the real cost of output forgone, lost time, pleasure or any other benefit that provides utility should also be considered
opportunity costs.
Imputed cost is a cost that is incurred by virtue of using an asset instead of investing it or undertaking an alternative
course of action. An imputed cost is an invisible cost that is not incurred directly, as opposed to an explicit cost, which is
incurred directly. It is also known as notional cost. Imputed cost is a hypothetical cost and it does not appear in financial
records. But it is relevant for decision making. Interest on capital is a common type of imputed cost. Financial accountancy
accords recognition to interest on capital only if it is actually paid or constitutes a legal liability. If desirability of a project is
being evaluated, failure to consider interest cost may result in an erroneous decision. For all practical purpose there is no
difference in opportunity costs and imputed costs.

b) Cost Control and Cost Reduction: Cost control is operated through setting standards of targets and comparing actual
performance therewith, with a view to identify deviations from standards or norms and taking corrective action in order to
ensure that future performance conforms to standards or norms. Cost reduction is a continuous process of critical cost
examination, analysis and challenge of standards. Each aspect of business viz., products, process, procedures, methods,
organization, personnel, etc. is critically examined and reviewed with a view of improving efficiency and effectiveness and
reducing the costs. Cost control is a method whereby costs are managed and monitored to ensure they do not grow beyond
the projected amount.
Cost control lacks the dynamic approach which planned cost reduction demands. In cost reduction, standards which are the
basis of control are constantly challenged for improvement. Cost reduction is a method whereby costs are reduced from the
projected amount.

c) Waste and Scrap


Waste represents the portion of basic raw materials lost in production process having no recoverable value. Waste may be
visible – remnants of basic raw materials or invisible e.g. disappearance of basic raw materials through evaporation, smoke
etc. Shrinkage of material due to natural causes may also be a form of a material wastage. In the production process, waste
of material does occur, such waste has no recoverable scheme, hence there is no chance of recovery from it.
Scrap is the incidental material residue coming out of certain types of manufacturing processes, usually of small amount and
low value, recoverable without further processing. Scrap is visible and do have recoverable value. Material defective goods
may be sold as scrap.

d) Job costing relates to a costing system where each unit or batch of output of products or services is unique. This creates
the need for the cost of each unit or batch to be calculated separately. Direct costs and factory overheads are allocated to
individual units of production and the finished goods stock consists of stock of unlike units. In contrast, a process costing
system relates to the situation where masses of identical units or batches are produced thus making is unnecessary to assign
costs to individual units or batches of output. Instead, the average cost per unit or batch of output is calculated by dividing
the total costs assigned to a product or services of the period by the number of units or batches of output for that period.
Direct costs and factory overhead costs are allocated to processes. When units are completed, they are transferred to finished
goods stock at average unit cost. Therefore, the finished goods stock consists of stock of like units valued at average unit
cost of production.
(39)

The main difference between the two is the cost object used for cost accumulation. Job costing is when for example a
tradesman comes to give you a quote for how much he is willing to do the job/repair that you want to be done, whereas
process costing are what a business has to spend in order to keep functioning, overheads etc.

6) Write short notes on: (4×2.5 =10)


a) Functions of management accounting system
b) Limitations of uniform costing
c) Information required for preparation of cost audit
d) Volume-based and non-volume-based cost drivers

Answer:
a) Management accounting systems provide information to assist managers in their planning and control activities.
Management accounting activities include collecting, classifying, processing, analyzing, and reporting information to
managers. It is so designed that its information help decision making within the firm. Its scope include information on sales
backlogs, unit quantities, prices, demands on capacity resources, and extensive performance measures based on physical or
non financial measures. The challenge is to develop management accounting practices that support the basic managerial
tasks of organizing, planning, and controlling operations to achieve excellence throughout the organization.

b) The following are the limitations of uniform costing:


(i) Sometimes it is not possible to adopt uniform standards, methods and procedures of costing in different firms due to
differing circumstances in which they operate. Hence, the adoption of uniform costing becomes difficult in such firms.
(ii) Disclosure of cost information and other data is an essential requirement of a uniform costing system. Many firms do
not wish to share such information with their competitors in the same industry.
(iii) Small firms in an industry believe that uniform costing system is only meant for big and medium size firms, because
they can't afford it.
(iv) It induces monopolistic trend in the business, due to which prices may be increased artificially and supplies withheld.

c) Preparation for cost audit of an organization requires the knowledge and gathering of following information and
data:
 Cost accounting system used in the organization,
 Production methods and manufacturing processes,
 Raw materials and components used in production,
 Cost records and documents,
 Cost accounting rules or cost accounting manual used in the organization,
 Important information mentioned about the costing requirement in Memorandum and Articles of Association,
etc.

d) Cost drivers are the factors, forces or events that influence and determine the cost of a particular activity and help
in assigning the cost to production unit. Direct costs are traced to cost and they themselves are cost drivers. When indirect
costs are assigned or allocated to product or services the factors, such as number of production run, number of machine set
up, number of purchase order etc. are used which are known as cost drivers. Cost drivers are unique physical aspects of the
production process which can be viewed as causing the cost pools to be incurred.

Volume-based cost drivers assume that a product‟s consumption of overhead resources is directly related or highly
correlated to production volume or unit. Some examples of volume-based cost drivers are units of output, direct labour
hours, machine hours, etc. There are some activities performed which are not directly related to volume of production such
as production run, machine set up, purchase order etc. The cost of these activities are allocated to the product on the basis of
non-volume-based cost drivers, such as number of production run, number of machine set up, number of purchase order etc.
Both costs derivers are meant for cost control. Volume-based cost drivers facilitate to control cost directly in proportion to
production output. Non volume-based cost drivers facilitate to control cost through parameters not directly linked to
production volume.
The Institute of Chartered Accountants of Nepal
Suggested Answers of Financial Management
CAP II Examination – June 2011

1. A plastic manufacturer has under consideration the proposal of production of high quality plastic glasses. The necessary
equipment to manufacture the glasses would cost Rs. 80,000. Investment allowance rate on purchases of equipment is
20%. The production equipment would last 5 years with no salvage value. The glasses can be sold at Rs. 3 each.
Regardless of the level of production, the manufacturer will incur cash costs of Rs. 25,000 each year, if the project is
undertaken. The overhead costs allocated to this new line would be Rs. 5,000. The variable cost is estimated at Rs. 2.0
per glass. The manufacturer estimates it will sell about 75,000 glasses per year; the straight line method of depreciation
will be used; the applicable tax rate is 55%.
a) Calculate the cash outflows of the project. 5
b) Determine the project‟s total present value at 0, 10, 20, 30 and 40 percent discount rate. 5
c) Present the net present value profile for the proposal. 3
d) Explore the relationship between Pay Back Reciprocal and IRR? 5
e) What is the basic assumption behind terminal Value Approach? 2

You can take the help of following PV table:


Year 10% 20% 30% 40%
1 0.909 0.833 0.769 0.714
2 0.826 0.694 0.592 0.510
3 0.751 0.579 0.455 0.364
4 0.683 0.482 0.350 0.260
5 0.621 0.402 0.269 0.186

Answer No. 1
(i) Cash outflows:
Particulars Rs
Cost of new equipment purchases 80,000
Less: Investment Tax Credit (Rs16,000) x 55% 8,800
Net cash outflow 71,200
Cash inflows:
Particulars Rs
Sales Revenue 225,000
Less Costs:
Variable Costs 150,000
Additional Fixed Cost 25,000
Additional Depreciation 16,000
Earning Before Taxes 34,000
Less Taxes 18,700
Earning After Taxes 15,300
Add Depreciation 16,000
Cash Flow After Tax (t = 1 – 5) 31,300
(Note: Costs allocated from other departments will not be considered as they do not involve any corresponding
incremental cash outflows)
(ii) PV at different rates of discount:
Rate of discount (%) PV factor Time (Years) CFAT (Rs.) Total PV (Rs.)
0 5.000 1-5 31,300 156,500
10 3.791 1-5 31,300 118,658
20 2.991 1-5 31,300 93,618
30 2.436 1-5 31,300 76,247
40 2.035 1-5 31,300 63,696
(iii)
Net present value profile for the project:
Rate of discount (%) NPV (Rs.)
0 85,300
10 47,458
20 22,418
30 5,046
40 (7,505)

NMG P.T.O.
(41)

(iv) The reciprocal of the pay back is a good approximation of the IRR. The pay back period reciprocal can be applied to
both annuity and mixed streams of cash flows. In case of annuity, pay back period of the proposed investment project is
determined and factor closest to the pay back period in the year row is looked into. In case of mixed stream cash flows,
average annual cash inflow is first calculated and approximated IRR is determined with the help of „fake‟ pay back period.
(v) Basic assumption behind the Terminal Value approach is that each cash inflow is re-invested in another asset at a certain
rate of return from the moment it is received until the termination of the project.
2. Following book value capital structure is available in respect of PQR Ltd.
(Rs. in million)
__________________________________________________________________________________
_______
Equity Capital (in shares of Rs. 100 each, fully paid-up at par) 150
11% Preference Capital (in shares of Rs. 100 each, fully paid-up at par) 10
Retained Earnings 200
13.5% Debentures (of Rs. 100 each) 100
15% Term Loan 125
__________________________________________________________________________________
_______
The next expected dividend per share on equity shares is Rs. 36 and the dividend per share is expected to grow at the rate of
7%. The market price per share is Rs. 400.
Preference stock, redeemable after 10 years, is currently selling at Rs. 75 per share. Debentures, redeemable after 6 years,
are selling at Rs. 80 per debenture. The income tax rate for the company is 25%.

You are required to: (8 +7=15)


a) Calculate the weighted average cost of capital using market value proportion., and
b) Determine the weighted marginal cost of capital for the company, if it raises Rs. 100 million next year, given the
following information:
 The amount will be raised by equity and debt in equal proportions.
 the company expects to retain Rs. 15 million earnings next year.
 the additional issue of equity shares will result in the net price per share being fixed at Rs. 320.
 the debt capital raised by way of term loan will cost 15% for the first Rs. 25 million and 16% for the next Rs. 25
million.

Answer No. 2
Working Notes:
(1) Cost of Equity Capital (Ke) and Cost of Retained Earnings (Kr)
Ke = D1/P0 + g = 36/400 + 0.07 = 0.09 + 0.07 =0.16 or 16%

(2) Cost of Preference Share Capital (Kp)


Kp = D + (Rv – Sv) / N = 11 + (100 – 75) / 10 = 11 + 2.5 = 0.1543 or 15.43%
(Rv + Sv) /2 (100 + 75) / 2 87.5
(3) Cost of Debentures (Kd)
Kd = I + (Rv – Sv) / N (1 – t) = 13.5 + (100 – 80) / 6 (1 – 0.25) = (13.5 + 3.33) 0.75
(Rv + Sv) /2 (100 + 80) / 2 90
= 0.14025, say, or 14.03%

Alternatively
Cost of Debenture (Kd)
Kd = I(1 - t) + (Rv – Sv) / N =
(Rv + Sv) /2
Kd =13.5 (1 –0.25) +(100-80)/6
(100 + 80) /2
= 10.125+3.333
90
= 13.458
90
= 0.1495 or 14.95%
Note: Students may also use either of one formula to calculate the cost of debenture and same can be used to calculate
WACC. In the solution first alternative is used.

(4) Cost of Term Loan (Kt)


Kt = I (1 – t)
0.15 (1 – 0.25) = 0.15 x 0.75 = 0.1125 or 11.25%
On first Rs. 25 million Term Loan = 0.15 (1 – 0.25) = 0.1125 or 11.25%
(42)

On the next Rs. 25 million Term Loan = 0.16 (1 – 0.25) = 0.12 or 12%
(5) Cost of Fresh Equity Shares (Ke)
Ke = D1/P0 + g = 36/320 + 0.07 = 0.1825 or 18.25
(i) Calculation of Weighted Average Cost of Capital (WACC) using market value proportion:
____________________________________________________________________________________________
____
Source of Finance Market Value Weight Cost of Weighted cost
(Rs. Millions) Capital of Capital %
____________________________________________________________________________________________
_______
Equity Capital 600.00 0.739 0.1600 0.11824
(1.5 million shares x Rs. 400
11% Preference Capital
(1 lakh shares x Rs. 75) 7.50 0.009 0.1543 0,00139
13.5% Debentures 80.00 0.098 0.1403 0.01375
(1 million debentures x Rs, 80)
15% Term Loan 125.00 0.154 0.1125 0.01733
812.50 WACC: 0.15071
____________________________________________________________________________________________
_______
Therefore, WACC = 15.07%
Note: Retained earnings are not considered for calculating WACC since it does not have any market value
separately. The market value of equity shares reflects the value of retained earnings as well.
* Alternatively 0.1495 or 14.95% can be used to calculate WACC.

(ii) Calculation of WACC of PQR Ltd. when it raises Rs. 100 million next year:
____________________________________________________________________________________________
____
Source of Finance Amount Weight Cost of Weighted cost
(Rs. Millions) of Capital of Capital %
____________________________________________________________________________________________
_______
Retained Earnings 15 0.15 0.1600 0.02400
Debt 15 0.15 0.1125 0.01688
Equity Shares 10 0.10 0.1825 0.01825
Debt 10 0.10 0.1125 0.01125
Equity Shares 25 0.25 0.1825 0.04563
Debt 25 0.25 0.1200 0.03000
100 0.14601
Therefore, WACC of raising Rs.100 million next year = 14.60%

3.
a) Progressive Manufacturers Ltd. has sales of Rs. 250 million of which 80 per cent is on credit basis. The present credit
terms of the company are “2/15, net 45”. At present, the average collection period is 30 days. The proportion of sales on
which customers currently take discount is 0.50.
The firm is considering relaxing its discount terms to “3/15, net 45”. Such a relaxation is expected to increase current
credit sales by Rs. 10 million, reduce the average collection period to 27 days and increase the proportion of discount
sales to 0.60. The average selling price of he company‟s product is Rs. 1,000 per unit and variable cost per unit works
out to be Rs. 800. The company is subject to a tax rate of 25 per cent and it‟s, before tax rate of borrowings for working
capital is 12 per cent.
Should the firm change its credit terms to “3/15, net 45”? Support your answers by calculating the expected change in
net profit (assume 360 days in a year) 8
(* Note: In the question the information is found missing "by Rs. 10 million, reduce the average collection period to 27
days and increase the proportion of discount sales to 0.60.")

b) Describe the term “beta co-efficient” as used in the portfolio theory. Explain what does the value of beta of 1, less than
1 and more than 1 signify. (5+2=7)

Answer No. 3
a)
Total Sales = Rs. 250 million
Credit Sales = Rs. 250 x 0.80 = Rs. 200 million
(a) Present Credit Policy:
(43)

Present credit terms are „2/15, net 45‟


Present average collection period = 30 days
Proportion of sales on which customers currently take discount is 0.5, or 50%.
(b) Basic revenue and cost structure applicable to both the policies:
Selling price per unit: Rs. 1,000
Variable cost per unit: Rs. 800
Contribution per unit: Rs. 200
P / V Ratio = 200/1,000 x 100 = 20%
Contribution from increased sales = Rs. 1,000,000 x 20% = Rs. 200,000
(c) Relaxed Credit Policy:
Reduction in average collection period to 27 days
Increase in proportion of discount sales to 0.60, or 60%
Change in the investment of Receivables
= [Rs. 200 million x 27 – 30 ] + [Rs. 10 million x 80/100 x 27/360]
360
= (-) Rs. 1.667 million + Rs. 0.60 = (-) Rs. 1.0667 million
Therefore, the reduction in receivables investment is Rs. 1.0667 million.
Saving in cost from reduction in receivables investment
= 1.0667 x 0.12 (1 – 0.25) = 0.096 million
Increase in discount cost
= [(Rs. 200 + 10) x 60/100 x 3/100] – (200 x 50/100 x 2/100)
= 3.78 – 2 = Rs. 1.78 million
Statement showing Profitability of Relaxing Credit Policy
(Rs. in million)
Contribution from Increased Sales 2.000
Cost savings from Reduction in the Receivables Investment 0.096
2.096
Less: Incremental Discount Cost 1.780
Incremental Profit 0.316
The firm can increase its profits by Rs. 0.316 million by relaxing the credit policy. Therefore, it is suggested to change the
credit terms to „3/15, net 45‟ from the present “2/15, net 45”.

b) Under capital asset pricing model (CAPM), the risk of an individual security can be estimated. The market related risk,
which is also called „systematic risk‟ is unavoidable even by diversification of the portfolio. The systematic risk of an
individual security is measured in terms of its sensitivity to market movements which is referred to as security‟s beta.
Beta coefficient is a measure of the volatility of stock price in relation to movement in stock index of the market. Thus, beta
is the index of systematic risk. The beta factor of the market as a whole is 1.0. A beta of 1.0 of individual security indicates
the average level of risk as compared to the market.
Mathematically, the beta coefficient of a security is the security‟s covariance with the market portfolio divided by the
variance of the market portfolio. Symbolically,
βi = Cov im. = σi σm Cor im, where
Varm σm2
βi = Beta of an individual security
Cov im. = Covariance of returns of individual security with the market portfolio
Varm = Variance of returns of market portfolio (σ m2)
Cor im = Correlation coefficient between the return s of individual security and the market portfolio
σi = Standard deviation of returns of individual security
σm = Standard deviation of returns of market portfolio
The degree of volatility can be expressed as follows:
 If beta is 1, then it has the same level of risk profile as the market as a whole.
 If the beta is less than 1, it is not as sensitive to systematic or market risk as the average investment.
 If beta is more than 1, it is more sensitive to the market risk than the average investment.

4.
a) Based on the credit rating of the bonds, an investor has decided to apply the following discount rate for valuing the
bonds.
Credit rating Discount rate
AAA 364-day Treasury-bill rate + 3% spread
AA AAA + 2% spread
A AAA + 3% spread
(44)

The investor is considering investing in an AA rated, Rs. 1,000 face value bond currently selling at Rs. 1,010. The bond has
five years to maturity and the coupon rate on the bond is 15% per annum payable annually. The next interest payment is due
one year from today and the bond is redeemable at par. (Assume 364-day Treasury bill rate to be 9%)
You are required to calculate: (5+3=8)
i) Intrinsic value of the bond for the investor. Should the investor invest in the bond?
ii) Current yield (CY) and the yield to maturity (YTM) of the bond.

b) The capital structure of Stable Ltd. is extracted below:


(Rs. in Million)
Equity capital: 100 thousand shares of Rs.100 each 10.0
Reserve and surplus 12.0
12% preference shares: 55,000 shares of Rs. 100 each fully paid up 5.5
14% debentures of Rs. 1,000 each; 3,000 numbers 3.0
Long-term loan from financial institution at 12% per annum 2.0
32.5
The company is also availing a bank overdraft of Rs. 2 million carrying interest at 15% per annum. The company is now
drawing up its profit plan for the next year. It wants to pay dividend to equity shareholders at 15% and keep the total
dividend payout (equity as well as preference shareholders) at 60%.
Assuming that the tax rate applicable to the company is 25%., what level of earnings (EBIT) should the company try to
achieve to meet its plan?
7
Answer No. 4
a) AA rated face value of bond = Rs. 1,000
Current selling price = Rs 1,010
Maturity period of bond = 5 years
Coupon rate of the bond = 15% per annum payable annually
Bond redeemable at par at the end of 5th year.
Net interest payment is due on year from today.
Discount rate for AA rated bond = 9% + 3% + 2% = 14%
Calculation of Present Value of Cash Inflow from Bond
(Rs.
___________________________________________________________________________________________________
_________________________________________________________
Year-end Cash Inflow PV Factor at 14% Present Values
___________________________________________________________________________________________________
________
1 150 0.8772 131.58
2 150 0.7695 115.43
3 150 0.6750 101.25
4 150 0.5921 88.82
5 1,150 0.5194 597.31
Present value of total Cash Inflow: 1,034.40

Thus, the intrinsic value of bond is Rs. 1,034.40. Since the intrinsic value of bond (Rs. 1,034.40) is more than its current
market value (Rs. 1,010), it is suggested to purchase the bond.

Current yield = Annual Bond Interest x 100 = 150 x 100 = 14.85%


Market price 1,010
Yield to Maturity (YTM)
P = Rs. 150 x PVIFA @ 15% for 4 years + Rs. 1,150 x PVIF at 15% for 5 th year
= (150 x 2.855) + (1,150 x 0.4972 = 428 + 571.78 = 1,000.03
Present value at 14% = Rs. 1,034.40
Present value at 15% = 1,000.03
By interpolation, YTM = 14 + 1,034.40 – 1,010 x 1 = 14% + (24.40 / 34.37) = 14.71%
1,034.40 – 1,000.03
b)
Let „x‟ be the EBIT to meet the company‟s commitments.
Interest Payable Yearly (Rs. in Million)
On debentures @ 14% on Rs. 3 million 0.42
On long term loan of Rs. 2 million @ 12% 0.24
On bank overdraft of Rs. 2 million @ 15% 0.30
0.96
Profit before tax (PBT) = EBIT – 0.96 = x – 0.96
(45)

Tax at 25% = (x – 0.96) / 4


Profit after tax (PAT) = 3 (x – 0.96) / 4

Total dividend payable (Rs. in Million)


On preference capital of Rs. 5.5 million @ 12% 0.66
On equity capital of Rs. 10 million 1.50
2.16

Total dividend payout is limited to 60% of PAT and is also equal to Rs. 2.16.
Therefore, 3 (x – 0.0.96) /4 x 60 / 100 = 2.16
Or 3(X-0.96)/4=216/60
Or X-0.96=3.6x4/3
Or, x – 0.96 = 4.80
Or, x = 4.80 + 0.96 = 5.76
Hence, earnings before interest and tax should be Rs. 5.76 million.

5.
a) The following information pertains to a company:
Net profit (Rs. in „000) 60,000
12% preference shares capital (Rs. in „000) 20,000
Number of equity shares outstanding (in „000) 60
Return on investment 20%
Equity capitalization rate 16%
You are required to: (4.5+1.5=6)
i) Compute the dividend payout ratio so as to keep the share price at Rs. 412.50 by using Walter model, and
ii) Ascertain (giving reasons) the optimum payout ratio if return on investment is 16% and equity
capitalization rate is 18%.
a) The beta co-efficient of security X is 1.6. The risk free rate of return is 12% and the required rate of return is 18% on
the market portfolio. If the dividend expected during the coming year is Rs. 25 and the growth rate of dividend and
earnings is 8%, at what price should the security X can be sold based on the capital asset pricing model. 5
b) An investor saw an opportunity to invest in a new security with excellent growth potential. He wants to invest more
than he had, which was only Rs. 100,000. He sold another security short with an expected rate of return of 15%. The
total amount he sold was Rs. 400,000, and the total amount he invested in the growth security, which had an expected
rate of return of 30%, was Rs. 500,000. Assuming no margin requirements, what is the investor‟s expected rate of
return? 4

Answer No. 5
a) Calculation of Earnings per Share (EPS) (Rs. in „000)
Net Profit 60,000
Less: Preference Dividend (20,000 X 12 / 100) 2,400
Net Profit after Preference Dividend 57,600
Earnings per Share in Rs. [Rs 57,600,000/60,000] 960

(i) Calculation of Dividend Payout Ratio:


Let dividend payout ratio be „x‟. The formula for share price under Walter model is::
P = D + r / Ke (E – D), where
Ke
P = Market price per share (Rs. 412.50 Given)
E = Earnings per share (Rs. 960 derived above)
D = Dividend per share (Rs. 960 x Given)
r = return on investment (0.20 given)
Ke = Cost of equity (0.16 Given)
Substituting the values, we get:
412.50 = 960 x + 0.20/0.16 (960 – 960 x)
0.16
Or, 412.50 = 60 x + 1.25 (60 – 60 x)
0.16
Or, 412.50 = 60 x + 75 - 75 x
0.16
Or, 66 = 1,200 – 240x
Or, x = 4.725, or 472.5%
Thus, the required dividend payout ratio is 472.5%.
(ii) Optimum Payout Ratio when Return on Investment (16%) is less than Equity Capitalization Rate (18%)
(46)

According to Walter model, when return on investment is less than the cost of capital, the value of the share is highest when
dividend payout is maximum. It is evident that when r/Ke is less than 1, higher dividend will maximize the value per share.
Therefore, the dividend payout should be 100% in this case.
b) Expected rate of return is calculated as follows by applying CAPM formula:
E (Ri) = Rf + Bi (Rm – Rf)
= 12% + 1.6 (18% - 12%) = 12% + 9.6% = 21.6%.
Price of security X is calculated with the use of dividend growth model formula as follows:
Re = D1 / P0 + g, where
D1 = Expected dividend during the coming year
Re = Expected rate of return on security X
g = Growth rate of dividend
P0 = Price of security X
Substituting the values, we get:
0.216 = 25/ P0 + 0.08,
Or, 0.216 = 2.50 + 0.08 P0
P0
Or, 0.216 P0 = 25 + 0.08 P0
Or, 0.216 P0 – 0.08 P0 = 25,
Or, 0.136 P0 = 25
Or, P0 = 25 / 0.136 = Rs. 183.82.
The price at which the security X should be sold as per CAPM is Rs. 183.82.
c)
Computing the portfolio weights for each security is done with the formula:
Investment in A (sold short)
Total equity investment
From the given problem, we find:
WA = - Rs, 400,000/ Rs. 100,000 = - 4.0
WB = Rs. 500,000 / Rs. 100,000 = 5.0
Rp = (- 4 x 0.15) + (5 x 0.30) = - 0.60 + 1.50 = 0.90, or 90%.
Thus, the expected rate of return on this portfolio is 90%.

6. Write short notes on: (4×2.5=10)


a) Factoring services
b) Commercial paper
c) Perpetuities
d) Inflation Premium

Answer No. 6
a) Factoring services - Factoring is a unique financial innovation. It is both a financial as well as a management
support to a client. It is a method of converting a non productive, inactive asset like receivable into a productive asset like
cash by selling receivables to a company that specializes in their collection and administration. Factoring is a business
involving a continuing legal relationship between a financial institution (factor) and a business concern (client) selling goods
or providing services to trade customers whereby the factor purchases the client's account receivable and in relation thereto,
controls the credit, extended to customers and administers the sales ledger. Basically three kinds of services fall into this:
sales ledger administration and credit management, credit collection and protection against default and bad debt losses,
financial accommodation against the assigned book debts.
b) Commercial Paper - It is an important money market instrument in advanced countries like USA to raise short term
funds. It is a form of unsecured promissory note issued by firms to raise short term funds. The commercial paper market in
the USA is a blue-chip market where financially sound and highest rated companies are able to issue commercial papers.
The buyers of commercial paper include banks, insurance companies, unit trusts and firms with surplus funds to invest for a
short period with minimum risk. Given this objective of the investors in the commercial paper market, there would be
demand for commercial papers of highly creditworthy companies.
c) Perpetuities can be defined as a stream of equal payments expected to continue for ever. Most annuities call for
payments to be made over some finite period of time, for example, Rs1000 per year for five years. However, some annuities
go for indefinitely, or perpetually, and these are called perpetuities. The present value of perpetuities is found as below:
PV (Perpetuities) = Payment/ Interest Rate
Most preferred stocks entitle their owners to regular, fixed dividend payments lasting forever. These are one of the examples
of „perpetuities‟.
d) „Inflation Premium‟ is the premium for expected inflation that investors add to the real risk free rate of return.
Inflation has a major impact on interest rates because it erodes the purchasing power and lowers the real rate of return on
investment. Investors are well aware of all this, so when they lend money, they build in an inflation premium equal to the
average inflation rate expected over the life of the security. Therefore, if the real risk free rate is 4 percent and if inflation is
expected to be 5% (and hence inflation premium=5%) during the next year, then the quoted rate of interest would be 9%.
(47)

7. Distinguish between: (4×2.5=10)


a) Asset Beta Vs. Equity Beta
b) Discrete Probability Distribution and Continuous Probability Distribution
c) Proxy fight and Takeover
d) Business Risk and Financial Risk
Answer No. 7
a) Assets of a leveraged firm are financed by debt and equity. Therefore, the assets beta should be the weighted
average of the equity beta and the debt beta. For an unlevered (all equity) firm, the asset beta and the equity beta would be
the same. Debt is less risky than equity. Hence the beta of debt will be lower than the equity beta. In case of the risk free
debt, beta will be zero. For a levered firm, the proportion of equity will be less than 1. Therefore, the beta of asset will be
less than the beta of equity.
There is also a linear relationship between the equity beta and the financial leverage. As the financial
leverage increases, the equity beta also increases. The equity beta is equal to the asset beta if debt is zero.
b) Discrete Probability Distribution and Continuous Probability Distribution
In case of „Discrete Probability Distribution‟ the number of possible outcome is limited or finite. Suppose, if we assume that
there will be only three states of economy; recession, normal or boom, this will be the example of discrete probability
distribution.
In other hand, if we assume that there will be unlimited or infinite number of possible outcomes that will be the
case of continuous probability distribution. With continuous distribution, it is more appropriate to ask what the probability is
of obtaining at least some specified rate of return than to ask what the probability is of exactly that rate.
c) Management always solicits stockholders‟ proxies and usually gets them. However if earnings are poor and
stockholders are dissatisfied, an outside group might solicit the proxies in an effort to overthrow management and take
control of the business. This is known as proxy fight.
Takeover is an action whereby a person or group succeeds in ousting a firm‟s management and taking control of
the company. In recent years there are cases, where attempts have been made by one corporation to take over another by
purchasing a majority of the outstanding stock.
d) Business Risk is defined as the uncertainty inherent in projections of future Return on Assets (ROA), or of Returns
on Equity (ROE) if the firm uses no debt. Business risk is the single most important determinant of capital structure.
Business risk varies from one industry to another and also among firms in given industry.
Financial risk is the additional risk placed on the common stockholders as a result of using financial leverage,
which results when a firm uses fixed income securities (debt and preferred stock) to raise capital. Thus, it is the portion of
stockholders‟ risk, over and above basic business risk, resulting from the manner in which the firm is financed.
The Institute of Chartered Accountants of Nepal
Suggested Answers of Business Communication and Marketing
CAP II Examination – June 2011

Section -'A' : Business Communication

1. Read the following case carefully and answer the questions given below:
The CEO of Giga Bank Ltd., with its head office based in Kathmandu has asked you, a Senior Recruitment Officer of the
bank to write a reply to Mrs. Lata Dawadi, a permanent and short-listed candidate of the bank, in response to her complaint
that a fresh candidate was selected ignoring her merit, work efficiency and experience. Reply in an encouraging and
convincing language including the following, 10
a) That the Selection Committee was aware of the complainant‟s qualifications, work efficiency and
experience,
b) That she could not be selected only for some technical reasons- the selected outside candidate has
long years of association with two or three banking organizations.
c) That the bank assures Mrs. Lata Dawadi that her appointment to a higher post will be considered in
case of any new opening in the future.
Answer No.1
Mrs. Lata Dawadi
Administrative Officer
Giga Bank Ltd.
Kathmandu

Regarding your complaint that an outside fresh candidate was selected to the post of the bank‟s senior administrative officer
and that you were left out as a disqualified candidate, we would like to persuade you that the bank management is well
aware of your qualifications, competence and work experience. But the interview board considered it necessary to pick up
the candidate on ground that the latter had the long years of association with two or three banking institutions. The
testimonials that one has submitted bear witness to the fact one had the wide experience in the related field. So, the selection
committee has made a wide decision to priotize the wider experience and higher qualification. So, we hope that you would
take it as a reasonable ground for making such a decision. Yet, we assure that the bank will take your experience and
qualification into account, in the further advertisement. In case of the new opening in future, bank assures to appoint you to
a higher post.

Wishing you a successful administrative career,

Senior Recruitment Officer

2. Analyze how information is transmitted through nonverbal messages and discuss how to improve nonverbal
communication skills. 10
Answer No.2
Nonverbal messages are sent by our eyes, face, and body. For example, sustained eye contact indicates
trust or admiration; brief eye contact may signify fear or stress. Expressions on a communicator‟s face
can supplement or entirely replace verbal messages. Posture can indicate status, confidence, shyness, or
submissiveness. Gestures also semd nonverbal messages, many of which are culture dependent.
Moreover, how a communicator uses time, space, and territory sends messages that require no words.
The amount of space we need for social interaction can be another means of sending messages
nonverbally.
Communicators may improve their nonverbal communication skills by maintaining eye contact, looking alert, eliminating
physical barriers that separate them from their listeners, and improving their comprehension of nonverbal signals. Besides,
following specific activities might prove valuable in improving nonverbal skill.
 Self-monitor your actions and tones. When you start to notice a habits or oddities that occur while
you speak, write them down. This will help correct yourself when it comes down to it.
 Be aware of how people see you. We all see ourselves one way, but others can see us in a different
manner. Notice how people react to you while you speak.
 Ask friends to help you determine where you're going wrong. They will know if you are sending
any conflicting signals. If you feel comfortable, ask your colleagues as well. They see another side
of you that your friends normally don't.

NMG P.T.O.
(49)

 Videotape yourself while you speak. Make sure you are able to see at least your upper half. Notice
any gestures, facial expressions, or tones that can conflict with the message you are giving with
your words.
 Adapt with the roles you have in your life. You act differently as a student and as a mother, for
example. Be sure that your words, tones, actions, and gestures correlate with the environment and
role that you are in at that time.

3. Self-dependent students, who are quite concerned with job-search, make a lot of efforts towards job seeking to
support their studies. During their job-search efforts, they perform various steps. Mention five major steps which most of
the students follow in their efforts towards job search. 10

Answer No.3
Self-dependent students, who do learning by earning, need to do a lot of job searching during their career development
period. Many of such students need to get some kind of job to support their education. As getting employment is not an easy
task, they will have to spend quite a considerable amount of time during job search. They do this by building a network of
contacts with a number of related personalities and organizational heads at many places. The job search follows a process
which takes the searcher to a wider circle of friends facing similar problems and those who have already gone a few steps
ahead of them in getting the job. The most probable people they meet include: university professors, business executives,
businessmen and industrialists. They also need to approach community organizations, NGOs and INGOs and make contacts
with them through internships The job search process involves some of the following steps

Identifying and Preparing for Appropriate jobs During job search; students have to consider both internal and external
factors to get the right job. During this time, greater efforts are made to get the job that matches with the applicant‟s
education, skills and experiences. Here, the job searcher‟s specific qualities, distinct personal qualifications, work
experience and special qualitiescommunication skills matter much for the potential employer. Most schools have career
counseling centers that provide career counseling on job during the study period of the student. Besides, classified
advertisements published in the local papers also help the students in getting appropriate jobs.

Preparing for Employment: The stage of job search is followed by the second step in which the student prepares
himself/herself in preparing application documents. Here, the student makes personal visits, does online contacts and faxes
or emails his/ her resume to various organizations. He may also write a persuasive letter for an interview for employment
with follow-ups.

After finding one, he or she writes an application to select a group of companies, decides on whether to send a message
alone with a resume or with a resume and a reference sheet.

Placing of Resume in Job Search: A job application followed by a resume knocks at the door of the employer, who sets the
applicant to a challenging task of an interview.

Understanding & Writing an Effective Resume: A resume (also called data sheet and curriculum vitae) is the formal
arrangement of a writer‟s personal inventory. It lists facts in some orderly way. Normally, one applies for jobs by using
resume and letters. Writing complete resume require meeting certain requirements: contents, presentations, format and style
with appropriate style.

Resume Writing Process: One‟s job search activities begin with writing a good resume. The success of a resume in getting a
job depends upon three things: (a) Planning, (b) Start writing and (c) Completing the writing.

Planning involves: i. analyzing the situation in which the purpose of your writing the resume should be clear, ii. gathering
information about what the work organization intends to seek in its new job hire, iii. selecting the right medium and iv.
organizing the information

The Writing Stage: Here, start writing your resume in properly worded sentence format. Write things in a professional tone
to capture the audience attention.

Completing the writing: This involves evaluating the contents, editing and rewriting and producing the message to the
concerned as per instructions of the employer

4. What importance does ethical communication have in business? Discuss some of the ways of maintaining ethical
standards in business. 10
(50)

Answer No.4
As a business communicator, you should understand basic ethical principles so that you can make
logical decisions when faced with dilemmas in the workplace. Professionals in any field must deal with
moral dilemmas on the job. However, just being a moral person and having sound personal ethics may
not be sufficient to handle the ethical issues that you may face in the workplace. On the job you will
face many dilemmas, and you will want to react ethically.
Taking ethics into consideration can be painful in the short term. But in the long term ethical behaviour makes sense and
pays off. Dealing honestly with colleagues and customers develops trust and builds stronger relationships. Many businesses
today recognize that ethical practices make good business sense. Ethical companies endure less litigation, less resentment,
and less government regulation. The following guidelines can help you set specific ethical goals and maintain a high ethical
standard.
Abiding by the Law. Know the laws in your field and follow them. Particularly important for business communicators are
issues of copyright law. Don't assume that Internet items are in the "public domain" and free to be used. Internet items are
also covered by copyright laws.
Telling the Truth. Ethical business communicators do not intentionally make statements that are untrue or deceptive. We
become aware of dishonesty in business when violators break laws, notably in advertising, packaging, and marketing. Half
truths, exaggerations, and deceptions constitute unethical communication. But conflicting loyalties in the workplace
sometimes blur the line between right and wrong.
Labeling Opinions. Sensitive communicators know the difference between facts and opinions. Facts are verifiable and often
are quantifiable; opinions are beliefs held with confidence but without substantiation. Stating opinions as if they were facts
is unethical.
Being Objective. Ethical business communicators recognize their own biases and strive to keep them from distorting a
message. Honest reporting means presenting the whole picture and relating all facts fairly.
Using Inclusive Language. Strive to use language that includes rather than excludes. Do not use expressions that
discriminate against individuals or groups on the basis of their gender, ethnicity, disability, or age. Language is
discriminatory when it stereotypes, insults, or excludes people.
Giving Credit. Ethical communicators give credit for ideas by (1) referring to originators names within the text, (2) using
quotation marks, and (3) documenting sources with endnotes, footnotes, or internal references. In school or on the job,
stealing ideas or words from others is unethical.

5. Briefly explain any four of the followings: (4×2.5=10)


a) Use of you-viewpoint language in business
b) Ethnocentrism
c) Good news letter
d) High-context vs. low-context cultures
e) Managing publicity communication

Answer No.5
a) In writing business messages, the use of the you-viewpoint language helps in the promotion of
goodwill of the organization among its clients and customers. As the you-viewpoint language
emphasizes the reader‟s interests and concerns rather than the writer‟s. This technique is very effective
to motivate, influence and win favor of the people. It emphasizes you and your and de-emphasizes we
and our. The you-viewpoint language is an attitude that places the reader in the center of things. It
involves being friendly and treating people the way they like to be treated. For instance, the statement
“I am happy to report…” places the writer at the center, whereas “you will be happy to know….”
places the reader at the center. The you-viewpoint language, therefore, provides the reader a kind of
psychological boost and confidence.

b) The belief in the superiority of one‟s own race is known as ethnocentrism, a natural attitude
inherent in all cultures. It is the view that uncritically presupposes that one‟s own culture is the criterion
against which all other cultures must be judged. It is almost always used in a negative sense to describe
attitudes that refuse to recognize the validity of values that differ from their own. It is difficult to avoid
some measure of ethnocentrism as many cultural values are considered to be universal values or truths.

c) A good-news letter also known as an indirect letter conveys a message of good news
(promotion, transfer to a better place) written by a senior member to the junior one of an organization.
(51)

Such a letter is tuned with the expression of feelings of congratulations. Unlike in a bad-news latter,
the happy aspect of the message is expressed without any delay

d) Members of high-context cultures, particularly Asians, are more collectivist. They emphasize
membership in organizations, groups, and teams; they typically resist independence because it fosters
competition and confrontation instead of consensus. In contrast, members of low-context cultures,
particularly Americans, tend to value individualism. They believe that initiative and self assertion result
in personal achievement. They believe in individual action and personal responsibility, and they desire
a large degree of freedom in their personal lives.

e) Managing Publicity Communication has already been argued that communication is a


two-way process in which the sender and the receiver both play prominent roles. But in Publicity
Communication, the sender apparently plays the significant role to persuade the public at large
(receiver) to understand what has been said. Here the receiver plays a receptive role trying to
understand the contents of the message released by the sender. Like in regular communication,
the public assimilates the useful contents. Such a communication, in which the receiver does not
play a direct role, also generates feedback This is what happens in a PRESS RELEASE
(52)

Section -'B' : Marketing

6. Read the following case carefully and answer the questions that follow:

McDonald having 14,000 branches in USA is a multinational company operating as a fast food chain in the world. It has
more than 32,000 outlets in 121 countries around the globe. It serves about 46 million customers every day. It has recently
offered 50 thousand additional jobs in one day. In its bid to expand its chain across the globe, it entered India in the late
1990s. It set the target to serve 200 million prosperous middle-class people in India and set up 150 outlets there by 2008.

McDonald as the giant fast food chain is popular in the world for hamburgers made mostly of beef. It is the world‟s biggest
user of beef. Beef is prohibited to Hindus, the majority population in India. Likewise, pork is restricted to Muslims, the
second biggest community in India. In addition, there is a sizable population of vegetarians that also include staunch
followers of Jainism.

Well aware of this fact, McDonald offered new product line to Indian market. Maharaja Mac was made of mutton, and
McAloo Tikki Burger was made of potato. McDonald carefully segregated its product into vegetarian and non vegetarian
categories.

When McDonald had just begun positioning itself into the Indian market, it had to face a law suit back in USA. This led to
street protests and pressures from consumers and political parties to close down its operations in India. It hit the image of
McDonald badly.

Questions:
a) What is the marketing mix of McDonald? 5
b) If McDonald is planning to enter Nepalese market, what cultural factors should it consider? 5

Answer No.6

a) As stated in the Case, McDonald‟s is a popular big multinational Company operating in many countries, expanding its
chain to new markets. One of the famous items- hamburgers is being sold in different combination for vegetarians and non
vegetarians and to Hindus, Buddhists, Muslims, Christians etc. Well aware of cultural differences, market segmentation and
product mix is done differently. E.g. Maharaja Mac (of mutton), McAloo Tikki Burger (of vegetables), use of beef-mutton-
pork (differently to different religions).

Generally product is sold at higher standard, expensive retail outlets. Varitiy of products to different segments and central
and local promotions are the other elements of its marketing mix.

b)
The following cultural factors need to be considered for feasibility of McDonald‟s in Nepalese market:
 First of all conduct marketing research +environmental analysis.
 Most cultural issues of Indian market also apply to Nepalese market.
 Emerging eating out habit in urban societies of Nepal.
 Use of red meat consumption is decreasing.
 Brahmins reject buff whereas most of the ethnic groups prefer buff for its low price and taste.
 Changing life styles of urban people.
 Religious neutrality exits in Nepal but majority people are Hindu. Hindus say no to beef whereas Muslim hate pork. On
the other hand Jains are vegetarians.
 Use of public media to convince the people about its product line and product mix.
 Slight modification in marketing mix as well as distribution system for nepalese market.
 Be well aware of street protests, pressure of consumer and political parties.

7.
a) Differentiate between the selling concept and marketing concept. 5
b) How does environment influences industrial development in Nepal? 5

Answer No 7
a) The differences between the selling concept and the marketing concept are as follows
i. The selling concept is a management philosophy based on the premises that consumers generally do not buy products
unless the organization itself puts aggressive selling efforts to create awareness and interest of the consumer in its products
and makes them inclined positively to buy these products. Whereas the marketing concept is a management philosophy that
focuses on the needs and wants of the customers of the target markets and the organization needs to find products and
(53)

services that will prove to be a useful solution to solve the requirement of such needs and wants to deliver the satisfaction
of the customers.

ii. The selling concept helps in getting business. While the marketing concept regards customers as the pivotal point in the
entire marketing plan. The customer‟s needs and wants are supreme. The customer is the boss.

iii. The selling concept focuses on the needs of the sellers, whereas the marketing on concept focuses the needs of the buyer.

iv. The selling concept starts with the existing products and services and tries to fit into the needs of the customers whereas
the marketing concept starts with the proper understanding of target customer‟s needs and wants and then developing
products which would help customers to use and derive satisfaction.

b) Environment changes bring uncertainty in the industrial development. The environmental forces can be political,
economic, social and technological.
(i) Political changes in Nepal have the following impact on Nepalese industrial sector:
 Increasing labor agitation and strikes in industries;
 Close down of industries;
.
(ii) Economic changes have the following impact on Nepalese industrial sector:
 Rise in inflation rate and rise in price.
 Increase in imports and decrease in exports.
 Capital flight due to unsuitable investment climate.

(iii) Social Changes have the following impact on Nepalese industrial sector:
 Increase or decrease in size of population which affect size of market.
 Migration of population.
 Changes in attitudes, values and beliefs.

(iv) Technological changes have the following impact on Nepalese industrial sector:
 New technology replaces old technology
 Technology transfer increases
 New materials and processes become possible.

8.
a) Write a short note on market segmentation and explain its benefits to marketers. 5
b) Explain the significance of marketing information system. 5

Answer No.8
a) Market segmentation is a process of dividing the total market into several smaller groups, such that the members of each
group are similar with respect to the factors that influence demand .By dividing the total market in distinct smaller groups
helps to identify needs and characteristics of buyers. Mass marketing, product variety marketing, individual marketing and
target marketing are some of the common approaches found in dealing with the market.
Benefits of market Segmentation:
To achieve effective market segmentation requires measurability, accessibility, profitability and heterogeneous needs and
characteristics in the market. A proper marketing mix and programs through appropriate segmentation strategy provides the
following benefits:
- Identification of profitable markets
- Market specialization
- Effective use of available resources
- Efficient monitoring of market changes, etc.

b) Successful marketting planning and formulation of strategy requires supporting information and relevant data. No
marketing company can function effectively without appropriate and timely information on each and every aspect of
marketing like the consumer, the market, the competition and the environment. The decision making process becomes easier
with relevant information.
Marketing managers need a continuous flow of information to respond to the environmental changes and to shape and
reshape the marketing mix in order to earn long term business. Marketing department has to organize and distribute a
continuous flow of information to its marketing managers by studying their information needs and designing marketing
information system to meet these needs.
Marketing information system is an organized way of continually gathering and analyzing data to provide relevant
information needed to prepare marketing plan and program. In the words of Philip Kotler, “a marketing information system
(54)

consists of people, equipment and procedures to gather, sort, analyze, evaluate and distribute needed, timely and accurate
information to marketing decision makers.” The significance of marketing information system is:
i. It helps marketing planning by making available and reliable information about the external
environment and the internal realities of the company.
ii. It helps to take advantage of marketing opportunities and to counter marketing threats.
iii. It helps early spotting of changing trends and provides marketing intelligence to the firm.
iv. It facilitates the development of action programs for achieving goals.
v. It helps the firm to adjust its products and services to the needs and tastes of target customers.
vi. It helps the firm to control its marketing activities.

9.
a) Describe briefly the steps of new product development process. 5
b) Describe Distribution structure for consumer goods. 5

Answer No.9
a) New product development is a time consuming and painful process. It steps in this process are:
i. Idea generation: New product development starts with an idea. New product ideas come from a variety of sources i.e.
customers, scientists, competitors, employees, channel members, investors, marketing consultants, universities, business
journals, seminars etc. Various techniques have been developed and used for idea generation i.e. attributes listing, forced
relationship, morphological analysis, need/problem identification, and brainstorming.

ii. Idea screening: The purpose of screening is to drop poor ideas as early as possible. The total ideas are categorized into
three groups i.e. promosing ideas, marginal ideas and reject ideas.

iii. Concept development and testing: It is the method of trying to guage buyer interest before actual product has been
developed. A product idea can be turned into several concepts. In fact, consumers buy product concepts. Concept testing
entails presenting consumers with an elaborated version of the concept. Intercept interview technique is mainly used for
concept testing. Generally, this type of interview focuses on buying intent, frequency of purchase of the product, uniqueness
of the concept, the price, and the problem solving attributes of the product.

iv. Marketing Strategy development: After testing, the product manager must develop a marketing strategy for the new
product. Such a marketing strategy plan includes the target market size and structure, possible positioning for the product,
preliminary estimate of sales, market share, the product‟s price, distribution, promotion and marketing budget, long run
sales and profit goals and marketing mix program overtime.

v. Business analysis: This step is essential in the total process of new product development because several vital decisions
regarding the product are expanded into business proposal. These decisions should be financial as well as marketing point of
view. Management should analyze various activities in this step i.e. identify product features, estimate market demand,
competition and product‟s profitability, establish a program to develop the market and assign responsibility for study of the
feasibility.

vi. Product development and testing: In this step, the idea-on-paper is converted into a physical product. Having completed
the business analysis, it moves to R&D or engineering to be developed into a physical product. Product testing may be
conducted through an expert panel, central location testing, in-home test etc.

vii. Market testing: After management is satisfied with functional performance, the product is given a brand name,
packaging and put to market test. The new product is introduced in selected market to learn how large the market is and how
consumers and dealers react to handling, using and repurchasing the product.
viii. Commercialization: Full-scale production and marketing programs are planned and the product is launched in the target
market.

b) Distribution structure for consumer goods should be as follows:


Four channel structures are widely used in the marketing of consumer goods. The commonly used channels for consumer
goods are as follows:
(55)

Channel Structure for Consumer Goods.

Manufacturer of Consumers Goods

1 2 3 4

Agent

Wholesaler Wholesaler

Retailer Retailer Retailer

Consumer

1. Manufacturer - Consumer Channel


This is a direct channel which involves no middleman. Therefore, this channel is also known as Zero-level channel of
distribution, Here, the manufacturer sells goods and services directly to the consumers through indoor sales process or
outdoor sales process.

2. Manufacturer-Retailer-Consumer Channel
In this channel structure, one-level of middlemen is used i.e., retailers, Manufacturer sells to the retailers and retailers
sell to the ultimate consumers. Here, retailers function as middleman.

3. Manufacturer-Wholesaler-Retailer-Consumer Channel
In this channel structure, two-levels of middlemen are used. They are wholesalers and retailers. In this system,
manufacturer sells goods to the wholesalers, wholesalers sell to the retailers, and retailers sell to the ultimate consumers.

4. Manufacturer-Agent-Wholesaler-Retailer-Consumers Channel
In this channel structure, three levels of channel middlemen are used to reach the ultimate consumers. They are agent,
wholesaler and retailer. Here, agent collects goods from the manufacturer and distributes to the wholesaler, wholesaler
sells them to the retailers, and the retailers sell to the consumers.
The second, third, and fourth channel of distribution are indirect channels of distribution since independent
middlemen are used to reach the final consumers. In direct channel system, the manufacturer exercises maximum level
of distribution control. While in indirect channel system, the manufacturer looses controlling power over distribution
functions.

10. Briefly explain the following: (5×2=10)

a) Telemarketing.
b) Elements of marketing communication.
c) Test marketing.
d) Resolution of channel conflicts.
e) Promotion mix.

Answer No.10
a) Telemarketing
It is marketing done by using telephone and call centers to attract customers. It sells to existing customers and provides
service by taking order and answering questions. It is becoming popular as business houses carryout telesales, telecoverage,
teleprospecting, customer services and technical support. Customers get improved satisfaction from telemarketing as a direct
marketing tool. Telemarketer has reduced the need of personal selling significantly.

b) Elements of marketing communication


The elements of marketing communication are as follows:
Source: The communication process begins with a source who is generally the sender of the message. The source is
generally the manufacturer, marketing channel members who initiate and intend to send a communication.
Encoding: It is the process of designing message by putting thoughts and ideas in symbolic language.
Message Channel: It is generally called media. This is the path through which the message moves from one party to another.
Media may be audio, video or print.
(56)

Decoding: Decoding is the process by which the receiver assigns meaning to symbol transmitted by the sender.
Receiver: The receiver is a person, a group or organization to which the communication is directed.
Feedback: It provides information to redesign communication to meet changing needs of marketing.

c)Test Marketing:
It is related with testing commercial viability of new product in the context of a market segment on a limited scale.
Marketing is done to have a reasonable estimate of sales and market share to make necessary changes in the marketing mix.
The product is sold in selected areas with the help of local level promotion in few shops. Test marketing is periodically
evaluated in terms of trial rate, advertising effectiveness, and repeat purchase rate.

d) Resolution of channel conflicts


Channel members usually specialize in particular marketing functions. Such as, manufacturers focus on production and
national promotion, wholesalers perform distribution task, and retailers specialize in distribution and promotion at the
consumer level. Such specialization creates conflicts among channel members. As a result, conflict resolution among
channel members becomes essential.
In practice, there are four methods of resolving channel conflicts:
 Resolution through Mitigation
 Resolution through Conciliation
 Resolution through Arbitration
 Resolution through Regulation.

 Conflict Resolution through Mitigation


Mitigation is a method of resolving disputes through mutual understanding between the parties without using any mediator.
It is possible only when both the parties honestly realize that the good faith between them must be restored for their long-
term relationship. In this case, both the parties actively and mutually take actions to restore their relationship without asking
the help of any third party. This method of resolving conflict is regarded as the best method.

 Conflict Resolution through Conciliation


Conciliation agreement between parties to resolve disputes by asking a third party to mediate differences voluntarily. The
third party does hot have any power to force the conflicting parties. If the agreement is reached, a conciliation statement
based on the signed agreement is recorded.

 Conflict Resolution through Arbitration


Under this method, the conflicting parties gather together with an arbitrator. They disclose all their opinions and then make
an agreement to continue the business with full understanding and confidence. This method permits them to continue their
business without damaging the goodwill of either of the parties. Timely resolution of the channel conflict is another positive
point of this method of resolution.

 Conflict Resolution through Regulation


Under this method, conflicts are resolved through law. All the conflicting parties take the help of law and go to the court for
the resolution of their disputes. However, this method of conflict resolution is an expensive, time consuming, and harmful to
all conflicting parties; It may damage the goodwill of both the parties. This method of conflict resolution is not popular and
practical. This method is adopted only when the channel conflict between the parties is difficult to settle through arbitration.

e) Promotional mix:
Promotion is the fourth element in the marketing mix, which is generally divided into five different tools. They are:
 Personal selling: It consists of executing sales through salespersons.
 Advertising: Any paid form of non-personal presentation to promote products and services by
identified sponsor.
 Sales promotion: Short term incentives that encourage people to buy a product/service.
 Publicity: An unpaid form of non-personal presentation of news and information about products or
services to prospective customers.
 Direct marketing: Non-personal presentation to the prospective customers to which they can
respond directly and quickly.
The Institute of Chartered Accountants of Nepal
Suggested Answers of Income Tax and VAT
CAP II Examination – June 2011

8. Answer the following with reference to the Indian Income Tax Act, 1961.
a) What are the bases for the dividend income to be includible in the total income of the assessee?
2
b) Mr. Rama Swami is an Indian citizen. He left India for the first time on May 10, 2004. During the previous year
2008/09, he came to India once on June for a period of 10 days. Determine his residential status for the assessment year
2009/10 on the assumption that financial year is previous year. Does it make any difference if Mr. Rama Swami is not
an Indian citizen?. 3
c) Income earned during a previous year is charged to tax in the financial year next following it.” Write in brief if there are
any exceptions to this normal rule.
5

Answer No.1
a) Dividend income is includible in the total income of the assessee on the following basis:
Nature of dividend Year of chargeability
1. Final dividend The previous year in which it is declared in the AGM.
2. Interim dividend The previous year in which it is unconditionally made available.
3. Deemed dividend The previous year in which it is paid or distributed.

b) Since Mr. Rama Swami comes to India only for 10 days in the previous year 2008/09, he does not satisfy any of the
basic conditions mentioned in sec 6(1). Therefore, he is non-resident in India for the assessment year 2009/10.
Even if he is not a citizen of India, he will remain a non-resident for the assessment year 2009/10.

c) The exceptions to the said rule are as follows:


i. In the case of shipping business of a non-resident, the ship is allowed to leave the port only if the tax due thereon has been
paid or arrangement has been made for payment thereof.(Sec.172)

ii. If a person is leaving India, the total income of such individual for the period from the expiry of the previous year for that
assessment year up to the date of his probable departure is chargeable to tax in that assessment year. (Sec.174)
iii. In the case of a person likely to transfer property to avoid tax, the total income of such person for the period from the
expiry of the previous year for that assessment year to the date when the Assessing Officer starts preceding is chargeable to
tax in that assessment year. (Sec.175)
iv. Where any business is discontinued in any year, any sum received after the discontinuance is deemed to be the income of
the recipient and charged to tax accordingly in the year of receipt. (Sec.176).

9.
a) Mr. Yubaraj Thapa, a bank officer, retired from his job in the year 2065/66. He received Rs. 4,50,000 as retirement
payment. The payment was made through an approved retirement fund. From the Shrawan 1, 2066 he is entitled to get a
pension of Rs. 21,000 per month. He also entitled to get one month's pension as dashain bonus. Mr. Yubaraj received
interest of Rs. 50,000 from a fixed deposit from Rastriya Banijya Bank. He has also received a dividend of Rs. 95,000 from
Nabil Bank. He started a consultancy business in the month of Shrawan 2066 and earned net profit of Rs. 25,000 in the year
2066/67.
His wife Mrs. Lolita is a school teacher in a government school and getting Rs. 10,000 monthly salary. She is also operating
a boutique since last five years and the net profit and the annual sales for the year 2066/67 are Rs. 1,50,000 and Rs.
12,45,000 respectively. Mrs. Lolita owns a house located at Putalisadak and the rental income of the house is Rs. 50,000 per
month. The house was rented to a commercial bank. She also does the assignment of question paper setting and evaluation
of answers and she received Rs. 1,05,000 during the year 2066/67.
Mr. Yubaraj and his wife have not selected the option as a couple for tax returns purpose. Calculate the tax liability of Mr.
Yubaraj and Mrs. Lolita and also state their responsibility to file the income tax return for the year 2065/66 and 2066/67. 12

b) Smart Pvt. Ltd. purchased a piece of land on Ashwin 25, 2064 for Rs. 10,00,000 and incurred an expenditure of Rs.
1,50,000 on registration and brokerage. It constructed a building on the land costing Rs. 25,00,000. The building was ready
for use on Shrawan 25, 2065. The depreciated value in Block A was Rs. 10,00,000 at the end of year 2064/65 excluding the
cost of the building under construction. During the year 2065/66 the company capitalized Rs. 60,000 as repair and
improvement cost in Block A.
On Shrawan 1, 2066, the company sold the land and newly constructed building for Rs. 40,00,000. The market value of land
on that date was Rs. 15,00,000 and that of the building was Rs. 25,00,000. You are required to calculate taxable gain for the
disposal of land and building for the year 2066/67.
5

NMG P.T.O.
(58)

c) In the occasion of Deepawali 2067, Himal Group organized a nationwide song competition and Mr. Ghanashyam
wins a first prize of Rs. 5,00,000. The organizer of the program wants to deduct TDS @ 25% on the prize amount and pay
the balance to him.

d) Mr. Ghanashyam was of the view that prize is related to the work of art and TDS is not applicable on this payment.
As a tax consultant, you are required to advice to Mr. Ghanashyam whether TDS is applicable in this payment or not also
mention the relevant provisions of Income Tax Act 2058.
3

Answer No.2
a) Calculation of Tax Liability for the year 2065/66
Mr. Yubaraj was the bank officer in the year 2065/66 his employer must have deducted TDS from his salary and
deposited to the revenue. The payment from approved retirement fund Rs. 4,50,000 is exempt (as it is less than
5,00,000) from tax as per the provisions of section 65.

There is no information regarding the total income of Mrs. Lolita for the year 2065/66, therefore we could not calculate
her tax liability for this year.

Calculation of Tax Liability for the year 2066/67

Calculation of Tax Liability of Mr. Yubaraj


Pension income (21000*12) 2,52,000
Dashain Bonus 21,000
Net Profit from consultancy business 25,000
Total assessable income 2,98,000

Tax Liability:
Upto Rs. 1,60,000 @1% 1,600
Additional exemption for pension 25% of Rs. 160000
or actual pension whichever lower = Rs. 40,000 Nil
Balance Rs. 98,000 @ 15% 14,700
Total Tax Liability 16,300

Calculation of Tax Liability of Mrs. Lolita


Salary Income (10,000*12) 1,20,000
Dashain Bonus 10,000
Income from boutique 1,50,000
Income from question paper setting and evaluation 1,05,000
Total assessable income 3,85,000

Tax Liability:
Upto Rs. 1,60,000 @1% 1,600
First Rs. 1,00,000 @ 15% 15,000
Balance Rs. 1,25,000 @ 25% 31,250
Total Tax Liability 47,850

Responsibility to file income tax return:

Mr. Yubaraj is required to file income tax return as follows:


i. He is not required to file income tax return for the year 2065/66 as he has only salary income and his employer must have
deducted TDS from his salary.
ii. He should file income tax return for the year 2066/67 as he earned income from pension and consultancy business.

Mrs. Lolita is required to file income tax return as follows:


i. She is required to file income tax return for the year 2065/66 as she has boutique business in that year.

ii. She is also required file income tax return in the year 2066/67 as she has salary income as well as business income.

Notes:
i. Bank interest and dividend are final withholding income of Mr. Yubaraj therefore these incomes are not included in his
income.
ii. House rent income of Mrs. Lolita is also a final withholding income.
iii. It is assumed that Mrs. Lolita will get dashain bonus equal to one month‟s salary.
(59)

iv. TDS shall not be deducted while making payment for the setting of question papers and evaluation of answers but this
income shall be included while calculating the taxable income of the recipient.

b. Calculation of taxable gain or loss from the disposal of land:

Selling price of land 15,00,000


Cost of land:
Purchase price 10,00,000
Registration and brokerage 1,50,000
Total cost of land 11,50,000
Gain on disposal of land 3,50,000

Calculation of taxable gain or loss from the disposal of building:

Depreciated balance at the end of the year 2064/65 10,00,000


Absorbed addition for the year 2065/66 (cost of building) 25,00,000
Depreciable basis at the end of 2065/66 35,00,000
Less: Depreciation for the year 2065/66 @5% 1,75,000
Depreciated balance 33,25,000
Add: Repairs capitalized 60,000
Depreciated balance at the end of 2065/66 33,85,000
Less: Selling price of building 25,00,000
Depreciable basis of building 8,85,000

Note 1:
Taxable gain from the disposal of land is Rs. 3,50,000 and there is no taxable gain from the disposal of building. The sales
value of building shall be deducted from the total amount of block and the remaining amount Rs. 8,85,000 will be the
depreciable balance of Block A.

c. As per section 88ka, TDS @ 25% shall be deducted in any windfall gain. Therefore, the organizer has to deduct TDS @
25% while making this payment to Mr. Ghanashyam.
The contention of Mr. Ghanashyam is not correct in this case as this amount has not exempted by Nepal Government from
windfall gain tax.

10.
a. As a tax consultant, you have been enquired of the implication on income tax on the following transactions: (2+1+1+1=5)
i) Kanyam Tea Estate Ltd. is a co-operative society established under the Co-operative Society Act 2048. In that society,
40 farmers of Kanyam region are involved in tea gardening and processing. In the financial year 2067/68, Kanyam Tea
Estate has earned Rs. 1 Crore. State with reason on the taxability of such income earned. Further, what will be the impact on
income tax if dividends are declared by that society?
ii) Income of the Securities Board of Nepal.
iii) Foreign investors have earned Rs. 50,00,000 as foreign technology and management service fee from the industry
established in the special economic zone
iv) Income earned by the Entities listed in the securities market and engaged in the business of production, Tourism sector.

b. As a tax consultant, you have been enquired by the clients on the depreciation facility for the purpose of income tax on
following cases: (2×2.5=5)
i) Assets, required for power generation for its industry, capitalized by the production oriented industries.
ii) If a person who wants to issue the tax invoices using fiscal printer and cash machine.

c. What do you understand by the expenses of Domestic or Personal nature? 5


d. Define Income, Windfall gain, Assessable Income and Taxable Income. 5

Answer No.3
a)
i. As per section 11(2) of the Act, Income of a co-operative society, having incorporated under the Cooperative Society Act,
2048 earned from any or more activities specified including tea gardening and processing is exempted from income tax.
Hence Rs. 1 crore is exempted from income tax.
Further as per the same section, dividend distributed by such cooperatives is also exempted from tax and hence no need to
deduct TDS while making payment to its members.
(60)

ii. As per section 10(Chha) of the Act, income of Securities Board of Nepal earned as per its objectives is exempted from
income tax.
iii. As per section 11(3Ka)(gha) of the Act, in case of income earned by a foreign investors towards foreign technology and
management service fee and royalty income from the industries established in Special Economic Zones, 50% exemption in
the applicable tax is available.
iv. As per section 11(3Chha) of the Act, entities engaged in the business of Production, Tourism, Hydroelectricity
generation, distribution and transmission etc. and listed in the securities market get 10% exemption in applicable income
tax.
b)
i. As per Sec 3(3) of schedule 2 of Income Tax Act 2058, Production oriented industries if capitalize the assets required for
power generation for its industries can claim 50% of total capitalized amount as depreciation in the year of capitalization.

ii. As per Sec 3(4) of schedule 2 of Income Tax Act 2058, if a person wants to issue the tax invoice by using fiscal printer
and cash machine, then total amount spent for such fiscal printer and cash machine can be claimed as depreciation in the
year of purchase.

c) Expenses of domestic nature or personal nature are not allowed for deduction for the purpose of income tax. The
clarification clause of section 21 explains the nature of domestic or personal nature as follows:
- Interest incurred on amount borrowed to the extent to which it is used for personal purpose.

- Expenses of very personal nature incurred for an individual in providing residence, meals, refreshment, entertainment or
other leisure activities.
- Expenses incurred by an individual on conveyance from residence to office and office to residence.

- Expenses incurred on an individual for clothing which is also suitable to wear outside the work

- Expenses incurred on education and training. But the expenses incurred on such training directly relating to the
business, yet not leading to a degree or diploma, are allowed for deduction.

- Any expenses incurred to make a payment to a natural person or the expenses incurred for a third person, except in and
to the extent of the following conditions:
 The payment is included in calculating the income of the individual- such as house rent, driver facility, gardener,
servant, telephone in residence provided to an employee. If the expenses are included in the taxable income of the
individual, the expenses are allowed for deduction to the person.
 The individual makes a return payment of an equal market value to the person as a consideration for the payment.

 Small amount incurred in this respect for which keeping an individual account is impracticable, for tea, stationery,
awards, emergency medical facility or any other expenses as provided by IRD up to Rs. 500 at a time.

d) As per section 2(ja) of the Act, income is the total amount received from business, employment, investment or windfall
gain. Income denotes the total receipts, which are included in calculation of taxable income (sec. 7, 8 and 9).

As per section 2(ja1) of the act, windfall gains includes income from lottery, gift, prize, tip, win-rings and other similar
casual incomes. As per section 6 of the act, assessable income is the sum of income from business, employment, investment.
In case of a resident person, it includes all income from any country for the concerned income year and in the case of non
resident person, it includes the income having source in Nepal. As per proviso to section 6 of the act, assessable income
does not include any concession under section 11 and income of an Approved Retirement Fund under section 64.
As per section 5 of the act, taxable income is determined by deducting the amount of donation, subject to limitation under
section 12 and 12ka and contribution to approved retirement fund subject to limitation under section 63 from the assessable
income.

4.
a. XYZ Co. Ltd purchased an Accounting Software for Rs. 50,00,000 with the life span of 10 yrs and 5 months. It was
installed on 1st Shrawan 2065. Also, the Company purchased Inventory Management Software for Rs. 31,50,000 with the
working life of 10 yrs and 6 months. This was put in use in the month of Baisakh 2066.
Compute the allowable depreciation for Tax purpose for Financial Year 2065/066. 5

b. Details of annual estimated tax and withholding tax of A & Co. are as follows:
Estimated Tax
Poush end Rs. 10,00,000
Chaitra end Rs. 12,00,000 (Re-estimated)
(61)

Ashad end Rs. 12,00,000 (Re-estimated)

Withholding Tax details:


Upto Poush end Rs. 20,000
Magh to Chaitra end Rs. 10,000
Chaitra to Ashad end Rs. 15,000

Compute the total Advance Tax to be paid for each installment. 5

Answer No.4
a) These intangible assets fall under Block “E”. As per section 3 of schedule 2 of Income Tax Act, the depreciation of
intangible assets shall be charged as follows:
Cost of the software/ useful life= Amount of depreciation per year (straight line Basis).
In case the year of useful life is in fraction of a year, the nearest half year shall be considered.

According to section 1(2)(kha) of schedule 2, each intangible assets should be disclosed separately.

Accounting software:
Cost Rs. 5,000,000 and the useful life is 10 years 5 months = 10.5 years.
As the software was installed on 1st Shrawan 2065, for the financial year 2065-66, depreciation shall be available for the
whole year.
Thus, depreciation for 2065-66 shall be:
Rs. 5,000,000 /10.5 = Rs. 4,76,190.48

Inventory Management software:


Cost Rs. 3,150,000 and the useful life is 10 years 6 months = 10.50 years.
As the software was installed on Baisakh 2066, as per section 5 of the schedule, depreciation shall be available on 1/3 rd of
the amount for financial year 2065-66.

Thus, the amount of depreciation shall be:


Rs. 3,150,000/10.5/3 = Rs. 100,000.

Thus, the total depreciation for 2065-66 shall be: Rs. 4,76,190.48 + Rs. 100,000= Rs. 5,76,190.48.

b) 1st Installment (Poush end)


= 40% of estimated tax – withholding tax – Advance tax
= 40% of Rs. 10,00,000 – Rs. 20,000 – 0
= Rs. 3,80,000

2nd Installment (Chaitra end)


= 70% of Rs. 12,00,000 – Rs. 30,000 – Rs. 3,80,000
= Rs. 4,30,000

3rd Installment (Ashad end)


= 100% of Rs. 12,00,000 – Rs. 45,000 – Rs. 8,10,000
= Rs. 3,45,000

Total Advance Tax paid:


Withholding Tax Rs. 45,000
1st Installment Rs. 3,80,000
2nd Installment Rs. 4,30,000
3rd Installment Rs. 3,45,000
Total Rs. 12,00,000
5.
a. Sharma & Sharma Co. has been a self employed VAT registered trader since 2064 B.S, and is in the process of
completing the VAT return for the month ended 31 Baisakh 2068. The following information is relevant to the completion
of the VAT return;
i) Sales invoices totaling Rs. 44,00,000 (excluding VAT) were issued to VAT registered customers in respect of VAT
sales. The firm offers a 5% discount for prompt payment.
ii) Sales invoices totaling Rs. 16,92,000 were issued to customers that were not registered for VAT. Of this figure, Rs.
51,700 was in respect of zero-rated VAT sales.
iii) During the month of Baisakh 068, goods amounted to Rs. 11,20,000 were purchased. Of this figure, Rs. 80,000 was
used for Mr. Sharma‟s private purpose.
(62)

iv) On 15 Baisakh 068, a Toyota Hilux vehicle was purchased for Rs. 24,00,000. The cost is inclusive of VAT.
v) During the month ended 31 Baisakh 068, Rs. 40,000 was spent on mobile telephone calls, of which 30% relates to
private calls.
vi) On 17 Baisakh 068, an office equipment was purchased for Rs. 6,00,000. The purchase was partly financed by a bank
loan of Rs. 5,00,000.
Unless stated otherwise, all of the figures are exclusive of VAT.
Calculate the amount of VAT payable by Sharma & Sharma Co. for the month ended 31 Baisakh 2068.
10

b. XYZ Pvt. Ltd. imports/purchases the taxable raw materials such as Iron Ingots and Steel Plates to manufacture them into
different furniture items as well as agriculture tools.
Details of total imports excluding Input VAT are as follows:
Cost of Iron Ingots declared by XYZ Pvt. Ltd. Rs. 3,00,000
Revised cost fixed by Customs Officer Rs. 3,50,000
Freight Rs. 50,000
Insurance Rs. 5,000
Import Duty @ 6%
In addition, it has also purchased following raw materials:
Steel plates (exclusive of VAT) Rs. 1,00,000
Woods (exclusive of VAT) Rs. 35,000

The products manufactured out of the above raw materials are sold out with the details as given below:
Agriculture tools (VAT exempt items) Rs. 2,50,000
Furniture Rs. 3,00,000
Find the ratio between taxable and non taxable sales and calculate the amount of Input VAT that XYZ Pvt. Ltd. is entitled to
claim under sec 17 (3) of VAT Act. 10

Answer No 5
a) Statement of VAT Return for the month ended 31 Baisakh 2068:
Sales (Rs. 44,00,000 * 13%) Rs. 5,72,000
Sales (Rs. 16,92,000 – Rs. 51,700 = Rs. 16,40,300* 13%) Rs. 2,13,239
Sales (Rs. 51,700 * 0%) Rs. 0______
Total output VAT (A) Rs. 7,85,239

Goods Purchased (Rs. 11,20,000 - Rs. 80,000) * 13% Rs. 1,35,200


Vehicle (Rs. 24,00,000 * 40% * 13/113) Rs. 1,10,442
Mobile expenses (Rs. 40,000 * 70%) * 13% Rs. 3,640
Office Equipment Purchase (Rs. 6,00,000 * 13%) Rs. 78,000_
Total Input VAT (B) Rs. 3,27,282

Net VAT payable (A-B) Rs. 4,57,957


b.
Computation of Input VAT amount on import:
Cost of Iron Ingots Rs. 3,50,000
Freight Rs. 50,000
Insurance Rs. 5,000
Total Rs. 4,05,000
Import Duty Rs. 24,300
Total cost for VAT purpose Rs. 4,29,300

Input VAT on import = 13% of Rs. 4,29,300


= Rs. 55,809
Computation of Input VAT amount on local purchase:
Steel plates Rs. 1,00,000
Woods Rs. 35,000
Total cost Rs. 1,35,000

Input VAT = 13% of Rs. 1,35,000


= Rs. 17,550
Total input VAT:
Input VAT on Import = Rs. 55,809
Input VAT on local purchase = Rs. 17,550
Total = Rs. 73,359
(63)

Computation of sales:
Taxable sales (Furniture) Rs. 3,00,000
VAT Exempted sales (Agriculture tools) Rs. 2,50,000
Total sales Rs. 5,50,000

Ratio between taxable and non-taxable sales is 54.546:45.454 say 55:45.


Therefore, XYZ Pvt. Ltd. is entitled to claim only for 55% or Rs. 40,347.45 out of total Input VAT of Rs. 73,359.

Note: Total amount of VAT paid on raw materials could be setoff against the sales of finished products if the finished
product is taxable item. In the question, it is not given that the raw materials purchased are used only for taxable item or
non-taxable item. Thus, the VAT paid on raw materials is proportionately on the basis of sales of taxable and non-taxable
sales.

6.
a. How is the taxable value for second hand or used goods computed? Similarly how the taxable value for wood of national
forest, private and community forest is computed? 5
b. Describe the procedure of Administrative Review in Value Added Tax. 5
c. What are the penalties mentioned in section 29 of the act on the following infringements: (5×1=5)
i) Registration as mentioned in section 10(1) and (2) related infringement.
ii) An unregistered person issuing an invoice or documents showing collection of tax.
iii) On obstruction in inspection by a tax officer.
iv) On infringement of the VAT act and the rules.
v) To erase and edit the data in software of approved computer Billing System.

d. M/s XYZ Drinkers Ltd. has sold a kind of drink to non VAT registered party for Rs.10,000. As per the retail price
published by the same company under the direction of IRD, the retail price is Rs. 12,000. The company has collected VAT
from the party on the amount of Rs.10,000 as it says it has given trade discount to the party and the trade discount can be
deducted to arrive at the transaction value. But the assessing officer insists to collect VAT on the published price. As an
expert, give your opinion on this. 5

Answer No.6
a) As per Rule 33 of VAT Rules, taxable value for dealers in secondhand or used materials is the difference of Sales
amount and Purchase Amount (including VAT). In case the purchase price of every item of used goods exceeds Rs.10,000,
separate records of buying or selling shall be maintained. As per rule 19, in case of registered person dealing in second hand
or used goods is not required to issue tax invoice if the selling price is less than the purchase price and the cost of the
material is less than Rs.10,000. It means if used goods sold in loss (purchase price + additional cost+ VAT paid on purchase
– selling amount without VAT), then the taxable amount is nil and tax invoice need not to be issued. In case a registered
person is observed by Tax Officer that it has not maintained the prescribed records satisfactorily, Tax Officer may impose
VAT on the total selling price of the goods sold by such tax payer, and the tax officer may issue a written order requiring
him to pay such tax along with the next tax return.

As per section 12ka, in case the wood from a national forest is being sold, tax shall be levied on the amount on which
royalty is being calculated or the amount of the auction, whichever is higher. The amount considered for such calculation
shall be on the basis of the earlier of these happenings: auction of wood of the national forest, issue of delivery order or
issue of an order to cut the wood. Similarly, wood of a private area, private forest and community forest, in case it is sold for
business, although royalty is not charged on such sales, for tax purpose, it is determined on the basis given for the wood of
national forest.

b) As per section 31(ka) of the Act, in case a taxpayer is not satisfied with any decision on tax assessment made by a tax
officer, it can file an application to IRD for an administrative review, within thirty days of the receipt of the notice of the
decision.
In case the taxpayer is not able to submit an application within the specified days, it can apply to IRD for extension of the
time, specifying the valid reasons for the delay, within seven days from the expiry of the time. IRD may extend the time for
a period not exceeding 30 days from the expiry of the time.
In case IRD, on scrutiny of the application and the documents submitted by the taxpayer, thinks that the application is
allowable, after noting the fact in a file, it may order a reassessment by the same tax officer or by any other tax officer.

IRD, up to possible, has to decide on the application within 60 days of the date of application received.

In case IRD fails to decide on the application within sixty days, the taxpayer may appeal to the Revenue Tribunal supposing
that IRD has given its verdict against the taxpayer. [0.5]
A person who applies for departmental review has to deposit 100% of the undisputed amount and 1/3 of the disputed
amount before the application is submitted.
(64)

c)
i. Non registration within the time period mentioned attracts penalty Rs.10,000 for each tax period.

ii. If an unregistered person issues an invoices or documents showing collection of tax, then 100% of the tax collected shall
be the applicable penalty.
iii. If obstruction is caused in inspection by a tax officer, then Rs.5,000 for each time shall be applicable as penalty.

iv. On infringement of the VAT Act and the Rules, then Rs.1000 for each time shall be the applicable penalty.

v. If the tax payer has erased and edit the data in software of approved billing system, then Rs. 5 Lac will be the applicable
penalty.
d) As general rules, the taxable value of an item is net of discount allowed in invoice. But, as per section 14(7) of VAT
Act, in case a registered person sales a notified item to a non-registered person, then the taxable value shall be the retail
published price. If discount is to be given that should be given after charging VAT.
Here notified items means the items for which IRD has notified that a producer of the items is required to publish its retail
price. Hence the contention of the assessing officer is correct.
CAP-II, Advanced Accounting, June 2012
Suggested Answer
Roll No……………. Maximum Marks - 100

Total No. of Questions - 6 Total No. of Printed Pages - 4

Time Allowed - 3 Hours


Marks
Attempt all questions. Working notes should form part of the answer.

1. From the following details relating to the accounts of XYZ Ltd. Prepare Cash
flow statement for the year ended 16.07.2011. 20
16.07.2011 16.07.2010
Liabilities Rs. Rs.
Share Capital 5,00,000 4,00,000
Reserve 2,00,000 1,80,000
Profit & Loss Account 1,50,000 1,30,000
Debentures 2,00,000 2,50,000
Provision for Income Tax 1,40,000 1,50,000
Sundry Creditors 1,70,000 1,90,000
Proposed Dividend 1,40,000 1,30,000
15,00,000 14,30,000
Assets
Goodwill 1,90,000 2,00,000
Plant & Machinery 5,29,250 3,98,000
Debenture Discount 15,000 18,000
Prepaid Expenses 15,750 14,000
Investments 1,60,000 2,00,000
Sundry Debtors 2,90,000 3,40,000
Stock 1,80,000 1,50,000
Cash and Bank 1,20,000 1,10,000
15,00,000 14,30,000
a) 15% Depreciation has been charged in the accounts on Plant & Machinery.
b) Old machine costing Rs. 1,00,000 (WDV Rs. 40,000) have been sold for
Rs. 70,000.
c) A machine costing Rs. 20,000 (WDV Rs. 6,000) has been disregarded.
d) Rs. 20,000 profit has been earned by sale of investments.
e) Debentures have been redeemed at 5% premium.
f) Rs. 45,000 income tax has been paid and adjusted against Provision for
Income Tax Account.

ZYA P.T.O.
(2)

Answer No. 1
Cash flow statement of XYZ Ltd. for the year ended as on 16.07.2011
Particulars Rs. Rs.
Cash Flows from Operating Activities:
Increase in reserves 20,000
Increase in P/L A/c 20,000
Add: Goodwill written off 10,000
Plant & machinery disregarded 6,000
Depreciation on plant & machinery 93,397
Discount on debentures written off 3,000
Premium on debentures redeemed 2,500
Provision for income tax 35,000
Proposed dividend 140,000
Decrease in sundry debtors 50,000 339,897
379,897
Less: Profit on sale of plant & machinery 30,000
Profit on sale of investments 20,000
Increase in prepaid expenses 1,750
Increase in stock 30,000
Decrease in sundry creditors 20,000
Income tax paid 45,000 (146,750)
Net Cash from Operating Activities 233,147
Cash Flows from Investing Activities:
Sale of plant & machinery 70,000
Sale of investment 60,000
Purchase of plant & machinery (270,647)
Net cash used in Investing Activities (140,647)
Cash Flows from Financing Activities:
Issue of share capital 100,000
Redemption of debentures with premium (52,500)
Dividend paid (130,000)
Net cash used in Financing Activities (82,500)
Net increase in Cash & Bank Balances 10,000
Cash & Bank Balances at beginning 110,000
Cash & Bank Balances at end 120,000

ZYA P.T.O.
(3)

Working Notes:
1.
Plant & Machinery Account
Particulars Rs. Particulars Rs.
To Balance b/d 398,000 By Bank 70,000
To Profit & Loss Account 30,000 By Profit & Loss Account 6,000
To Bank 270,647 By Depreciation 93,397
By Balance c/d 529,250
698,647 698,647

2.
Provision for Tax Account
Particulars Rs. Particulars Rs.
To Bank 45,000 By Balance b/d 150,000
To Balance c/d 140,000 By Profit & Loss Account 35,000
185,000 185,000

3.
Investment Account
Particulars Rs. Particulars Rs.
To Balance b/d 200,000 By Bank 60,000
To Profit & Loss Account 20,000 By Balance c/d 160,000
220,000 220,000

4. Addition to plant & machinery during the year and depreciation:


Particulars Rs. Rs.
Closing balance (WDV) of total plant & machinery 529,250
Less: Closing balance (WDV) of opening plant & machinery:
Opening WDV of plant & machinery 398,000
Less: WDV of machinery sold 40,000
Less: WDV of machinery disregarded 6,000
Balance of opening machinery (WDV) 352,000
Less: Depreciation @ 15% thereof 52,800 299,200
Closing balance (WDV) of machinery purchased during the year 230,050
Original cost of machinery purchased during the year (Rs. 230,050 / 0.85) 270,647
Depreciation @ 15% thereof 40,597
Total depreciation for the year [Rs. 52,800 + Rs. 40,597] 93,397

ZYA P.T.O.
(4)

2.
a) The following transactions by Mr. Nishchal took place during the year ended
31st March, 2012:
1st April Purchased Rs. 12,00,000 8 % bonds @ Rs. 80.5 cum-interest.
Interest is payable on 1st May.
12th April Purchased Rs. 1,00,000 equity shares of Rs. 10 each in X Ltd.
for Rs. 40,00,000.
1st May Received half year‘s interest on 8% bonds.
15th May X Ltd. made a bonus issue of three equity shares for every two
held. Mr. Nishchal sold 1,25,000 bonus shares @ Rs. 20 each.
1st October Sold Rs. 3,00,000 8% bonds @ Rs. 81 ex-interest.
1st November Received half year‘s bond interest.
1st December Received 18% dividend on equity shares in X Ltd.
Prepare the relevant investment accounts in the books of Mr. Nishchal for the
year ended in 31st March, 2012. 10
b) The premises of ABC Limited were partially destroyed by fire on 1st March,
2011 and as a result, the business was practically disorganized upto 31st
August, 2011. The company is insured under a loss of profits policy for Rs.
1,65,000 having an indemnity period of 6 months.
From the following information, prepare a claim under the policy. 10
Rs.
Actual turnover during the period of dislocation (1-3-2011 to 31-8-
2011) 80,000
Turnover for the corresponding period (dislocation) in the 12 months
immediately before the fire (1-3-2010 to 31-8-2010) 2,40,000
Turnover for the 12 months immediately preceding the fire (1-3-2010
to 28-2-2011) 6,00,000
Net profit for the last financial year 90,000
Insured standing charges for the last financial year 60,000
Uninsured standing charges 5,000
Turnover for the last financial year 5,00,000
Due to substantial increase in trade, before and up to the time of the fire, it
was agreed that an adjustment of 10% should be made in respect of the
upward trend in turnover. The company incurred additional expenses
amounting to Rs. 9,300 immediately after the fire and but for this expenditure,
the turnover during the period of dislocation would have been only Rs.
55,000. There was also a saving during the indemnity period of Rs. 2,700 in
insured standing charges as a result of the fire.

ZYA P.T.O.
(5)

Answer No. 2(a)


In the books of Mr. Nischal
8% Bond Account
Nominal Income Cost Nominal Income Cost
Date Particulars Date Particulars
Value [Rs] [Rs] [Rs] Value [Rs] [Rs] [Rs]
1/4/11 To Bank [1] 1,200,000 40,000 926,000 1/5/11 By Bank [2] - 48,000 -
31/3/12 To P/L A/c: 1/10/11 By Bank [3] 300,000 10,000 243,000
Int. income - 84,000 - 1/11/11 By Bank [4] - 36,000 -
Profit on sale - - 11,500 31/3/12 By Bal. c/d 900,000 30,000 694,500
1,200,000 124,000 937,500 1,200,000 124,000 937,500

Equity Shares in X Ltd.


Nominal Income Cost Nominal Income Cost
Date Particulars Date Particulars
Value [Rs] [Rs] [Rs] Value [Rs] [Rs] [Rs]
12/4/11 To Bank 1,000,000 - 4,000,000 15/5/11 By Bank 1,250,000 - 2,500,000
15/5/11 To Bonus [7] 1,500,000 - - 1/12/11 By Bank [8] - 225,000 -
31/3/12 To P/L A/c: 31/3/12 By Bal. c/d 1,250,000 - 2,000,000
Dividend - 225,000 -
Profit on sale - - 500,000
2,500,000 225,000 4,500,000 2,500,000 225,000 4,500,000

Working Notes:
1. Cost & interest income of 8% Bond:
Rs.
Cum interest purchase cost [Rs. 1,200,000 / 100 × 80.50] 966,000
Less: Interest for 5 months [Rs. 1,200,000 × 8% × 5 / 12] 40,000
Ex-interest purchase cost 926,000

2. Half yearly interest [Rs. 1,200,000 × 8% × 6 / 12] = Rs. 48,000

3. Sale value of Bond [Rs. 300,000 / 100 × 81] = Rs. 243,000.


Interest for 5 months [Rs. 300,000 × 8% × 5 / 12] = Rs. 10,000
Profit on sale of Bond [Rs. 243,000 – (Rs. 926,000 / 1,200,000 × 300,000)] = Rs. 11,500

4. Half yearly interest [Rs. 900,000 × 8% × 6 / 12] = Rs. 36,000

5. Accrued interest on Bond for 5 months as on 31.03.2012 [Rs. 900,000 × 8% × 5 / 12] = Rs. 30,000

6. Closing balance of 8% Bond as on 31.03.2012 [Rs. 926,000 / 1,200,000 × 900,000] = Rs. 694,500

7. Bonus issue of equity shares of X Ltd. [100,000 / 2 × 3] = 150,000 shares of Rs.10 each.

8. Dividend on equity shares [Rs. 1,000,000 + Rs. 1,500,000 – Rs. 1,250,000] × 18% = Rs. 225,000

ZYA P.T.O.
(6)

9. Profit on sale of equity shares:


Rs.
Sales value [125,000 × Rs. 20] 2,500,000
Less: Cost [Rs. 4,000,000 / 250,000 × 125,000] 2,000,000
Profit 500,000

10. Closing balance of equity shares of X Ltd. [Rs. 4,000,000 / 250,000 × 125,000] = Rs. 2,000,000

Answer No. 2(b)

Computation of insurance claim for Loss of Profit:


Rs.
1. Rate of Gross Profit:
Net profit for the last financial year 90,000
Add: Insured standing charges 60,000
150,000
Turnover for the last financial year 500,000
Rate of gross profit [Rs. 150,000 / Rs. 500,000 × 100] = 30%
2. Short sales:
Standard turnover 240,000
Add: 10% increasing trend 24,000
264,000
Less: Actual turnover for the period of dislocation 80,000
184,000
3. Annual adjusted turnover:
Annual turnover [1.3.2010 to 28.2.2011] 600,000
Add: 10% increasing trend [See Note] 60,000
660,000
4. Additional expenses:
(a) Actual expenses 9,300
(b) Gross profit on sales generated by additional expenses [WN 1] 7,500
(c) [30% of Rs. 660,000 / (30% of Rs. 660,000 + Rs. 5,000)] × Rs. 9,300 9,071
Allowable additional expenses – Least of the above three figures 7,500
5. Loss of profit:
Loss of profit on short sales [Rs 184,000 × 30%] 55,200
Add: Allowable expenses 7,500
62,700
Less: Savings in insured standing charges 2,700
Loss of profit 60,000

ZYA P.T.O.
(7)

6. Claim to be lodged on application of average clause:


165,000 
 Rs .   60,000
198,000 
 Rs . 50,000
Note:
It is assumed that trend adjustment is required on total amount of annual turnover. However, part of
the annual turnover represents trend adjusted figure. Alternatively, the students may ignore trend
and take simply annual turnover. The claim would be Rs. 55,000 which is more than the claim
computed in (6) above. So the Insurance Company would insist on trend adjusted on annual
turnover.

Working Note:

1. Gross profit on sales generated by additional expenses:


Rs.
Actual turnover during the period of dislocation 80,000
Expected turnover if no additional expenses were incurred 55,000
Sales attributable to additional expenses 25,000
Gross profit @ 30% 7,500

3.
a)
i) Nepal Insurance Co. Ltd. received Rs. 5,90,000 as premium on new
policies and Rs. 1,20,000 as renewal premium. The company received Rs.
90,000 towards reinsurance accepted and paid Rs. 70,000 towards
reinsurance ceded. How much will be credited to Revenue Account
towards premium? 2
ii) The books of Mr. Z showed the following information:
1.1.2011 31.12.2011
Rs. Rs.
Bank balance - 50,000
Debtors - 87,500
Creditors - 46,000
Stock 50,000 62,500
Fixed assets 7,500 9,000

The following are the details of the bank transactions:


Rs.
Receipt from customers 3,40,000
Payments to creditors 2,80,000
Capital brought in 5,000
Sale of fixed assets 1,750
Expenses paid 49,250
Drawings 25,000
Purchase of fixed assets 5,000
ZYA P.T.O.
(8)

Other information:
(i) Cost of goods sold Rs. 2,60,000
(ii) Gross profit 25% on cost of goods sold
(iii)Book value of assets sold Rs. 2,500

Prepare Trading, Profit and Loss account for the year ended 31.12.2011
and Balance Sheet as at that date. 8
b) Ram, Bharat, Laxman and Shatrughan are partners in a firm sharing profits
and losses in the ratio of 4:1:2:3. The following is their Balance Sheet as at
16th July, 2011:
Liabilities Rs. Assets Rs.
Sundry Creditors 3,00,000 Sundry Debtors 3,50,000
Capita Accounts: Less: Doubtful Debts 50,000
Ram 7,00,000 3,00,000
Shatrughan 3,00,000 10,00,000
Cash in hand 1,40,000
Stocks 2,00,000
Other Assets 3,10,000
Capital Accounts :
Bharat 2,00,000
Laxman 1,50,000 3,50,000
13,00,000 13,00,000

On 16th July 2011, the firm is dissolved and the following points are agreed
upon:
i) Ram is to take over sundry debtors at 80% of book value.
ii) Shatrughan is to take over the stocks at 95% of the value and
iii) Laxman is to discharge sundry creditors.
Other assets realize Rs. 3,00,000 and the expenses of realization come to
Rs. 30,000.
Bharat is found insolvent and Rs. 21,900 is realized from his estate.
Prepare Realization Account and Capital Accounts of the partners. The loss
arising out of capital deficiency may be distributed following the decision in
Garner vs. Murray. 10

ZYA P.T.O.
(9)

Answer No. 3[a](i):


Computation of premium to be credited to 'Revenue Account'
Rs.
Premium received in respect of new policies 590,000
Add: Renewal premium received 120,000
Add: Premium received on reinsurance accepted 90,000
Total premium received 800,000
Less: Premium paid on reinsurance ceded 70,000
Premium to be credited to Revenue Account 730,000

Answer No 3[a](ii):
In the books of Mr. Z
Trading and Profit and Loss Account
For the year ended 31st December, 2011
Particulars Rs. Particulars Rs.
To Opening stock 50,000 By Sales 325,000
To Purchases 272,500 By Closing stock 62,500
To Gross profit 65,000
387,500 387,500
To Expenses 49,250 By Gross profit b/d 65,000
To Loss on sale of asset 750
To Depreciation 1,000
To Net profit 14,000
65,000 65,000

Balance Sheet
As on 31st December, 2011
Liabilities Rs. Assets Rs.
Capital: Fixed assets 9,000
Opening balance 169,000 Stock 62,500
Add: Net profit 14,000 Debtors 87,500
Add: Additional capital 5,000 Bank balances 50,000
188,000
Less: Drawings 25,000 163,000
Creditors 46,000
209,000 209,000

ZYA P.T.O.
(10)

Working Notes:
1. Opening Balance Sheet:
Balance Sheet as on 1st January, 2011
Liabilities Rs. Assets Rs.
Capital – Bal. figure 169,000 Fixed assets 7,500
Creditors 53,500 Debtors 102,500
Stock 50,000
Bank balance 62,500
222,500 222,500

2. Opening bank balances:


Bank Account
Particulars Rs. Particulars Rs.
To Balance b/d – Bal. figure 62,500 By Creditors 280,000
To Debtors 340,000 By Expenses 49,250
To Capital bought in 5,000 By Drawings 25,000
To Fixed assets - Sales 1,750 By Fixed assets – Purchases 5,000
By Balance c/d 50,000
409,250 409,250

3. Opening debtors:
Debtors account
Particulars Rs. Particulars Rs.
To Balance b/d – Bal. figure 102,500 By Bank 340,000
To Sales (WN 8) 325,000 By Balance c/d 87,500
427,500 427,500

4. Opening creditors:
Creditors account
Particulars Rs. Particulars Rs.
To Bank 280,000 By Balance b/d – Bal. figure 53,500
To Balance c/d 46,000 By Purchases (WN 7) 272,500
326,000 326,000

5. Depreciation:
Fixed Assets account
Particulars Rs. Particulars Rs.
To Balance b/d 7,500 By Bank (Sale) 1,750
To Bank 5,000 By P/Loss A/c - loss on sale 750
By Depreciation – Bal. figure 1,000
By Balance c/d 9,000
12,500 12,500

ZYA P.T.O.
(11)

6. Gross Profit = Rs. 260,000 × 25% = Rs. 65,000.

7. Cost of goods sold = Opening stock + Purchases – Closing stock


Rs. 260,000 = Rs. 50,000 + Purchases - Rs. 62,500
Purchases = Rs. 272,500.

8. Sales = Cost of goods sold + gross profit


= Rs. 260,000 + Rs. 65,000
= Rs. 325,000.
Note:
The question was printed as if there were no opening bank balance, opening debtors and opening
creditors. If students are aware of this anomaly in the question, then answer to this question would be
as follows:

In the books of Mr. Z


Trading and Profit and Loss Account
For the year ended 31st December, 2011
Particulars Rs. Particulars Rs.
To Opening stock 50,000 By Sales 325,000
To Purchases 272,500 By Closing stock 62,500
To Gross profit c/d 65,000
387,500 387,500
To Expenses 49,250 By Gross profit b/d 65,000
To Loss on sale of asset 750
To Depreciation 1,000
To Net profit c/d 14,000
65,000 65,000

Balance Sheet
As on 31st December, 2011
Liabilities Rs. Assets Rs.
Capital: Fixed assets 9,000
Opening balance 57,500 Stock 62,500
Add: Net profit 14,000 Debtors 87,500
Add: Additional capital 5,000 Bank balances 50,000
56,500 Advance to suppliers 53,500
Less: Drawings 25,000 51,500
Advance from customers 102,500
Loan 62,500
Creditors 46,000
262,500 262,500

ZYA P.T.O.
(12)

Working Notes:
1. Opening balance sheet:
Balance Sheet as on 1st January, 2011
Liabilities Rs. Assets Rs.
Capital – Bal. figure 57,500 Fixed assets 7,500
Stock 50,000
57,500 57,500

2. Bank balances / Loan:


Bank Account
Particulars Rs. Particulars Rs.
To Debtors 340,000 By Creditors 280,000
To Capital bought in 5,000 By Expenses 49,250
To Fixed assets – Sale 1,750 By Drawings 25,000
To Loan – Bal. figure 62,500 By Fixed assets – Purchases 5,000
By Balance c/d 50,000
409,250 409,250

3. Advances from customers:


Debtors account
Particulars Rs. Particulars Rs.
To Sales (WN 8) 325,000 By Bank 340,000
To Advances – Bal. figure 102,500 By Balance c/d 87,500
427,500 427,500

4. Advances to suppliers:
Creditors account
Particulars Rs. Particulars Rs.
To Bank 280,000 By Purchases (WN 7) 272,500
To Balance c/d 46,000 By Advances – Bal. figure 53,500
326,000 326,000

5. Depreciation:
Fixed Assets account
Particulars Rs. Particulars Rs.
To Balance b/d 7,500 By Bank – Sale 1,750
To Bank 5,000 By P/Loss A/c - loss on sale 750
By Depreciation – Bal. figure 1,000
By Balance c/d 9,000
12,500 12,500

ZYA P.T.O.
(13)

6. Gross Profit = [Rs. 260,000 × 25% = Rs. 65,000.

7. Cost of goods sold = Opening stock + Purchases – Closing stock


Rs. 260,000 = Rs. 50,000 + Purchases - Rs. 62,500
Purchases = Rs. 272,500.

8. Sales = Cost of goods sold + gross profit


= Rs. 260,000 + Rs. 65,000
= Rs. 325,000.

Answer No 3(b)
Realization Account
Particulars Rs. Particulars Rs.
To Sundry debtors 350,000 By Sundry creditors 300,000
To Stock 200,000 By Provision for doubtful debts 50,000
To Other assets 310,000 By Ram's capital – Debtors 280,000
To Laxman's capital – Creditors 300,000 By Shatrughan's capital – Stock 190,000
To Cash – Expenses 30,000 By Cash – Other assets 300,000
By Ram's capital 28,000
By Bharat's capital 7,000
By Laxman's capital 14,000
By Shatrughan's capital 21,000
1,190,000 1,190,000

Ram's Capital Accounts


Particulars Rs. Particulars Rs.
To Realization A/c – Debtors 280,000 By Balance b/d 700,000
To Realization A/c – Loss 28,000
To Bharat's capital 129,570
To Cash 262,430
392,000 392,000

Bharat's Capital Accounts


Particulars Rs. Particulars Rs.
To Balance b/d 200,000 By Cash 21,900
To Realization A/c – Loss 7,000 By Ram's capital 129,570
By Shatrughan's capital 55,530
207,000 207,000

ZYA P.T.O.
(14)

Laxman's Capital Accounts


Particulars Rs. Particulars Rs.
To Balance b/d 150,000 By Realization A/c – Creditors 300,000
To Realization A/c – Loss 14,000
To Cash 136,000
300,000 300,000

Shatrughan's Capital Account


Particulars Rs. Particulars Rs.
To Realization A/c – Stocks 190,000 By Balance b/d 300,000
To Realization A/c – Loss 21,000
To Bharat's capital 55,530
To Cash 33,470
300,000 300,000

Note:
Bharat‘s deficiency will be borne by Ram and Shatrughan in the ratio of 7 : 3 i.e. on opening capitals
of Rs. 7,00,000 and Rs. 3,00,000. Laxman will not bear any portion of the loss since at the time of
dissolution he had a debit balance in his capital account.

4.
a) Rajesh Brothers sells their goods to their approved customers on ―Sale or
Return‖ basis treating all such transactions as actual sales at the time of
dispatch. They sent on 15th December goods costing Rs. 10,000 to Rama
Stores at 20% profit on sale and passed the goods through Sales Day Book.
How would you adjust the transaction on 31st December, if Rama Store‘s
consent is pending? 5
b) The following was the expenditure on a contract for Rs. 6,00,000 commenced
in February, 2011:
Materials Rs. 1,20,000; Wages Rs. 1,64,400; Plant Rs. 20,000; Business
Charges Rs. 8,600.
Cash received on account by 31st December, 2011 amounted to Rs. 2,40,000
being 80% of work certified; the value of materials on hand at 31st December,
2011 was Rs. 10,000.
Prepare the Contract Account for 2011 showing the profit to be credited to
year‘s Profit and Loss Account. Plant is to be depreciated at 10%. 5
c) Ms. Suman acquired on 1st January, 2010 a washing machine under a Hire –
Purchase agreement which provides for 5 half yearly installments of Rs. 6,000
each, the first installment being due on 1st July, 2010. Assuming that the
applicable rate of interest is 10 per cent per annum, calculate the cash value of
the machine. 5

ZYA P.T.O.
(15)

Answer No. 4(a):


Rs.
Cost of goods sent to customers 10,000
Add: 20% Profit on sales i.e. 25% on cost. 2,500
Selling price of goods sent to customer 12,500

As stated in the question, Rajesh Brothers sells their goods to approved customers on "Sale or
Return" basis treating all such transactions as actual sales at the time of dispatch. So following
journal entry must have been passed at the time the goods were sent to Rama Stores:
Dr Cr
Date Particulars
Rs. Rs.
15th December Rama Stores Dr 12,500
To Sales A/c 12,500

Since Rama Store's consent has not been received till 31st December, following adjustment entry
needs to be passed on 31st December:
Dr Cr
Date Particulars
Rs. Rs.
31st December (a) For cancellation of sale:
Sales A/c Dr 12,500
To Rama Stores 12,500
(Being reversal of sales for goods lying with
Rama Stores earlier treated as sales)
(b) For recording stock with customer at cost:
Stock with customers Dr 10,000
To Trading A/c 10,000
(Being goods with Rama Stores recorded as
stock with customer)

Answer No. 4(b):


Contract Account
Particulars Rs. Particulars Rs.
To Materials 120,000 By Plant in hand 20,000
To Wages 164,400 Less: Dep. 2,000 18,000
To Plant 20,000 By Materials in hand 10,000
To Business charges 8,600 By WIP [Rs. 240,000 / 0.8] 300,000
To Notional Profit c/d 15,000
328,000 328,000
To Profit & Loss A/c 8,000 By Notional Profit b/d 15,000
To WIP Reserve 7,000
15,000 15,000

Working note – Profit to be taken to Profit & Loss Account:


ZYA P.T.O.
(16)

2 80
Rs . 15,000    Rs . 8,000
3 100

Answer No. 4(c):

Statement showing cash value of the machine acquired on hire – purchase basis
Instalment Amount Outstanding Interest Principal Outstanding
Particulars
Rs. Rs. Rs. Rs.
5th Instalment 6,000 6,000 286 5,714
4th Instalment 6,000 11,714 558 11,156
3rd Instalment 6,000 17,156 817 16,339
2nd Instalment 6,000 22,339 1064 21,275
1st Instalment 6,000 27,275 1,299 25,976

Therefore, cash value of the machine is Rs. 25,976

Note:
Interest rate is 10% p.a. so half yearly is 5% and installment will be Rs. 105, principal amount will
be [100 / 105*100] & interest will be [100 / 105 *5]

ZYA P.T.O.
(17)

5.
a) ABC Limited closed its accounting year on 30.06.2011 and the accounts for
that period were considered and approved by the board of directors on 20th
August, 2011. The company was engaged in laying pipeline for an oil
company, deep beneath the earth. While doing the boring work on 01.09.2011
it had met a rocky surface for which it was estimated that there would be extra
cost to the tune of Rs. 100 lakhs. Further, the court had given its verdict
against the company for the liability of Rs. 50 lakhs on 02.09.2011 shown as
contingent liabilities on accounts. You are required to state with reasons, how
it would be dealt with in the financial statements. 5
b) Everest Ltd. incurred Rs. 20,00,000 as fixed production overhead per year. It
normally produces 1,00,000 units in a year. In 2010-11 however its production
has been only 40,000 units. At the year-end 16th July 2011, the closing stock
was 10,000 units. The cost of unit is below:
Material = Rs. 500 per unit
Labour = Rs. 250 per unit
Fixed Production overhead = Rs. 20,00,000 per annum
Fixed Administration = Rs. 10,00,000 per annum
Calculate the value of closing stock. 5
c) Define Borrowing cost & qualifying assets as per NAS 8? When borrowing
cost is capitalized? 5

Answer No. 5

a) Para 3 of NAS-5, on ―Events occurring after the Balance Sheet Date‖ defines ‗events
occurring after the balance sheet date‘ are those events, favorable and unfavorable, that
occur between the balance sheet date and the date when the financial statements are
authorized for issue. Two types of events can be identified:
1. Those that provide evidence of conditions that existed at the balance sheet date
(adjusting events after the balance sheet date); and
2. Those that are indicative of conditions that arose after the balance sheet date (non-
adjusting events after the balance sheet date).

In the first case the incidence, which was expected to push cost became evident after the
date of approval of the accounts. So it is only indicative of conditions that arose after the
balance sheet date, which is non-adjusting event after the balance sheet date. However,
this may be mentioned in the Director‘s Report.

In the second case the incidence, the settlement after the balance sheet date of a court
case that confirms that the entity had a present obligation on the balance sheet date. So it
provides evidence of conditions that existed at the balance sheet date, adjusting events
after the balance sheet date. Hence, new provision for the amount of Rs. 50 lakhs should
be adjusted against the profit & loss account.

b) In accordance with NAS 4 ―Inventories‖, the costs of conversion include a systematic


allocation of fixed and variable production overheads that are incurred in converting
materials into finished goods. The allocation of fixed production overheads for the
purpose of their inclusion in the cost of conversion is based on the normal capacity of the
production facilities.
3
ZYA P.T.O.
(18)

Thus, cost per unit of finished goods can be computed as follows:

Particulars Rs./unit
Material cost 500
Labor cost 250
Fixed Production Overhead [Rs. 2,000,000 / 100,000 units] 20
Total Cost of conversion 770

Thus, the value of 10,000 units of finished goods on stock at the year end will be Rs. 7,700,000
(10,000 units X Rs. 770 per unit).

c) Borrowing costs are interest and other cost incurred by an entity in connections with
borrowing of funds.
Qualifying asset is an asset that necessarily takes a substantial period of time to get ready
for its intended use or sale.
Borrowing cost is capitalized if
I. Expenditure for the asset are being incurred
II. Borrowing costs are being incurred ;and
III. Activities that are necessary to prepare the assets for its intended use or sale are in
progress.

6. Write short notes on: (4×2.5=10)


a) Matching Concept
b) Importance of Ratios analysis
c) Component of Financial Statement as per NAS
d) Rate of Loan Loss Provision for Performing & Non Performing Loan as per
NRB Directives

Answer No. 6
a) Matching concept requires periodic revenue should be matched with that period‘s
expense. While reporting financial performance of the entity only that portion of expense
should be recognized as expense which is attributable for earning revenue and remaining
portion of the expenses should be carried forward for the next accounting period so as to
derive actual performance of the entity during that period. That is the reason why prepaid
expenses are not recognized in periodic performance though outflow is incurred during
that particular period. Provisioning of the expense follow same rule. Depreciation is
charged on the ground of same underlying principle.

b) The following are the important managerial use of ratio analysis:


1. Helpful in assessing operating efficiency of the business.
2. Helpful in measuring financial solvency.
3. Helpful in future forecasting.
4. Helpful in decision making.
5. Helpful as corrective measures.
6. Helpful in comparing inter firm performance.
7. Helpful in cost control.

ZYA P.T.O.
(19)

c) A complete set of financial statement includes the following components:


1. A Balance Sheet: Statement showing financial position of entity as on date.
2. An income statement: Account showing performance of the entity during the period.
3. A Statement of changes in equity showing either:
i) All changes in equity, or
ii) Changes in equity other than those arising from transactions with equity holders
acting in their capacity as equity holder
4. A cash flow statement: Summary of cash inflow and outflow from operating, financing and
investing activities.
5. Notes, comprising a summary of significant accounting policies and other explanatory notes.

d) For the loans and bills purchase classified according to NRB Directives, the following
loan loss provision shall be maintained based on the remaining amount of principal:
Assets Rate of Provision
Pass / Good 1%
Sub-standard 25 %
Doubtful 50 %
Loss loan/the loan extended to blacklisted
persons, firms, company or corporate body 100%

ZYA P.T.O.
CAP-II, Audit & Assurance, June 2012
Suggested Answer
Roll No……………. Maximum Marks - 100

Total No. of Questions- 7 Total No. of Printed Pages- 2

Time Allowed - 3 Hours


Marks
Attempt all the questions.

1. As an auditor, give your opinions with reasons on the following cases: (45=20)
a) A firm of a father and a son is receiving Rs. 2 lakhs towards job work done for
XYZ Ltd. during the year ended on 32.03.2068. The total job work charges paid by
XYZ Ltd. during the year are over Rs. 50 lakhs. The father is a Managing Director
of XYZ Ltd. having substantial holding. The Managing Director told the auditor
that since he is not involved in the activities of the firm and since the amount paid
to it is insignificant; there is no need to disclose the transaction. He further
contended that such a payment made in the last year was not disclosed. Is
Managing Director right in his approach?
b) Shree Ltd. has 2 divisions X and Y. The finished products of division X are
transferred to division Y where further processing is carried out before sale to
customers. To achieve transparency and accountability between the divisions,
division X raises an invoice on division Y at cost plus normal margins. At the year
end the unrealized profits on inter-division stocks are eliminated. However, the
transfers are recorded at the invoice value as sales and purchases in the respective
divisions for the purpose of preparing the Profit & Loss Account. Suitable
disclosures, for this are given in the ‗Notes to Accounts‘.
c) Surya, a Chartered Accountant was engaged by Terai Company Pvt. Ltd. for
auditing their accounts. He sent his letter of engagement to the Board of Directors,
which was accepted by the company. In the course of audit of the company, the
auditor was unable to obtain appropriate sufficient audit evidence regarding
inventories. The client requested for a change in the terms of engagement.
d) During the course of audit of M/s Grow Company Limited, you, as an auditor
found huge difference between the control accounts and subsidiary records. The
chief finance controller informed that this is common due to huge volume of
business conducted by the company during the year. How would you deal in this
situation?

Answer:
a)
Nepal Accounting Standard 16 and IAS 24,―Related Party Disclosures‖ applies to the facts of the case.
IAS 24 requires disclosure of party relationship and transactions between a reporting enterprise and its
related parties. The parties are considered to be related if at any time during the reporting period, one
party has the ability to control the other party or exercise significant influence over the other party in
making decisions. As per the explanation given in IAS 24, significant influence is said to exist in case
the investing party has 20% or more voting power in the enterprise. In the instant case, the managing
director of XYZ Ltd. is a partner in the firm with his son, which has been paid Rs. 2 lakhs as job work
charges. The managing director is having a substantial holding in the firm. The case is covered by NAS
16. The approach of the managing director is not tenable under the law and accordingly all disclosure
requirements have to be complied. Since there is related party transaction the contention of managing

ZYA P.T.O.
(21)

Director is not correct and the auditor should advise him to make proper disclosure as required by NAS
and if the management refuses, the auditor shall express a qualified report.

b)

As per the definition of the term ―Revenue‖ in NAS 07, revenue is the gross inflow of cash,
receivables or other consideration arising in the course of the ordinary activities of an enterprise
from the sale of goods, from the rendering of services, and from the use by others of enterprise
resources yielding interest, royalties and dividends. Revenue is measured by the charges made
to customers or clients for goods supplied and services rendered to them and by the charges and
rewards arising from the use of resources by them.

The definition clearly implies that the transfers within the enterprise cannot be considered as
fulfilling the definition of the term ―revenue‖. Thus, the recognition of inter-divisional transfers
as sales is an inappropriate accounting treatment and is inconsistent with NAS 07. Further, in
case of inter-divisional transfers, risks and rewards remain within the enterprise and also there is
no consideration from the point of view of the enterprise as a whole. Thus, the recognition
criteria for revenue recognition are also not fulfilled in respect of inter-divisional transfers. In
the instant case, therefore, Shree Ltd cannot recognize inter-division transfers from X to Y as
sales and the same will have to be eliminated during finalization. If management do not agree to
do so, the auditor shall qualify his report.

c)
Nepal Standard on Auditing (NSA-210) stated about the ―Terms of Audit Engagements‖.
Auditor should consider following matters:

i. An auditor who is required to change the engagement which requires lower level of
assurance before the completion of engagement should consider the appropriateness of
doing so.
ii. But when the terms of engagement are changed, both the auditor and the clientshould
agree on the new terms.
iii. However, the auditor should not agree to a change in terms where there is no
reasonable justification for doing so.

In the instant case, the auditor was unable to obtain sufficient evidence regardinginventories.
The client requested him for a change in the terms of the agreement to avoid qualified/adverse
opinion. Hence there is no reasonable justification for change in the terms of engagement.
Thus the auditor should not agree for change in the terms of engagement letter.

d)
The finding of huge difference between the control accounts and subsidiary records of M/s Grow
Company Limited indicates that there may be material misstatements requiring detailed
examination of the books and records so as to ascertain the reason for the difference. The
contention of the chief financial controller cannot be acceptable simply because the company has
conducted huge volume of business during the year. Such finding indicates recording of business
transactions is not being done properly or the accounting system in the company fails to capture
all transactions in time. The auditor would rather check whether it is a recurring phenomenon or
not. He would also verify why such reconciliation could not be done at a subsequent date.
Considering all these facts, it may indicate the possibility of some kind of material misstatements
in the financial statements. As per NSA 240, ―The Auditor‘s Responsibility to Consider Fraud
and Error in an audit of Financial Statements‖ the auditor should perform procedures to
ZYA P.T.O.
(22)

determine whether the financial statements are materially misstated when the auditor encounters
the situation that there is material misstatement in the financial statements. The auditor is
required to report appropriately if he came across any material information involving fraud or
gross irregularity.

2. Answer the following:


a) While compiling the financial statements of a concern, you observed that the input
information supplied by the concern is incomplete, incorrect and few of the
Accounting Standards have not been followed. Describe, in brief, the procedure
you will follow in the given circumstances. 8
b) Describe the policies to be considered by an auditor regarding quality control as
prescribed in quality control standard. 7

Answer:
a)
According to AAS 31(SA 4410) ―Engagements to Compile Financial Information‖, an
accountant would normally have to rely upon the management for information to compile the
financial statements in a compilation engagement. If in the course of compilation of financial
statements, it is observed that the information supplied by the entity is incorrect, incomplete or
otherwise unsatisfactory, the accountant should perform following procedures:

(i) Make any enquiries of management to assess the reliability and completeness of the
information provided;

(ii) Assess internal controls prevailing in the entity; and

(iii) Verify any matters or explanations.

The accountant may also request the management to provide additional information. This may
be asked in the form of management representation letter. If the management refuses to provide
additional information, the accountant should withdraw from the engagement, informing the
entity of the reasons for such withdrawal.

If one or more accounting standards are not complied with, the same should be brought to the
notice of the management and if the same is not rectified by the management, the accountant
should include the same in notes to the accounts and the compilation report to the management.

b) The policies to be considered by an auditor regarding quality control as prescribed in quality control
standard (NSQC) are given below:
- Professional requirements as mentioned in Code of Ethics,
Personnel in the firm are to adhere to the principles of independence, integrity, objectivity,
confidentiality and personal behaviours.
- Skills and competence,
Staffing should be arranged to maintain the technical and professional competence
required to enable them to fulfil their responsibilities with due care.
- Assignment,

ZYA P.T.O.
(23)

Audit work to be assigned to personnel who have the degree of technical training and
proficiency required in the circumstances.
- Delegation,
There is to sufficient direction, supervision and review of work at all levels to provide
reasonable assurance that the work performed meets appropriate standards of quality.
- Consultation,
Whenever necessary, consultation within or outside the firm is to occur with those who
have appropriate expertise.
- Acceptance and Retention of Clients,
An evaluation of prospective clients and review, on an ongoing basis, of existing clients is
to be conducted. In making a decision to accept or retain a client, the firm's independence
and ability to serve the client properly are to be considered.
- Monitoring,
The continued adequacy and operational effectiveness of quality control policies and
procedures is to be monitored.

3. Give your comments on the following: (35=15)


a) Howard Ltd., as part of overall cost cutting measure announced voluntary
retirement scheme (VRS) to its employees, to reduce the employee strength.
During the year ended 32.03.2068 the company paid a compensation of Rs.10
million to those who availed the scheme. The chief accountant has reflected this
payment as part of regular salaries and wages paid by the company.
b) Reena Ltd. received Rs. 50 lakhs as grant from the Nepal Government towards the
part cost of a specific machinery. The company credited the above sum of Rs. 50
lakhs as income in its profit & loss account for the year.

c) Himalayan Co. Ltd. reappointed Ram and Ramita as their joint auditors in the
Annual General Meeting. The AGM authorized the Board for fill up the vacancy
on their own in the event of both or either of auditors declined to accept the
assignment. The Board passed a resolution to appoint Prabhat if any of the auditors
declined to accept the assignment. Later on Ramita declined to accept the
assignment and Board of Directors appointed Prabhat in place of Ramita as per its
resolution.
Answer
a)
NAS 1, ―Presentation of Financial Statements‖ clearly states that when the items of income
and expenses are material, their nature and amount shall be disclosed separately. Such a
disclosure shall assist in understanding the financial performance achieved and in assessing
future results. In the instant case the payment made to the employees on account of VRS as
an overall cost cutting measure would fall under the domain of material item. Accordingly it
is eligible to be shown separately in the income statement of Howard Ltd., so that the effect
of it on the operating results of the Company during the previous year can be perceived.
Therefore, clubbing of Rs. 10 million with the regular salaries and wages of the company by
the Chief Accountant is not appropriate.

b) NAS 10: Accounting for government grants recognized two methods of presentation of
grants related to specific fixed assets in financial statements as acceptable alternatives:
ZYA P.T.O.
(24)

(i) Under first alternative, the grant is shown in the Balance Sheet as a deduction from the
gross value of a machinery. The grant is recognized in P& L A/c over the useful life of a
depreciable asset by way of a reduced depreciation charges.

(ii) Under second alternative, it can be treated as deferred income which should be
recognized in P & L A/c over useful life of asset in proportion in which depreciation on
machinery will be charged. The deferred income pending its apportionment to P & L A/c
should be disclosed in Balance Sheet with a suitable description e.g. Deferred Government
Grants.

In the given case, Reena Ltd. received Rs.50 lakhs as grant towards part cost of specific
machinery. The company has credited the said sum as income in its Profit and Loss account
which is incorrect as per the above provisions.

c)
In the present case Ramita is one of the joint auditors who was appointed in Annual General
Meeting, but declined to accept the appointment. The Board of Directors as per their
resolution, appointed Prabhat as a joint auditor in her place. In this case, the vacancy created
by Ramita is neither caused by resignation of Ramita nor is it a casual vacancy because
Ramita‘s appointment had not become effective. Hence, appointment of Prabhat as joint
auditor by the Board is not valid. Prabhat can only be appointed as joint auditor by
shareholders in the General Meeting.

4. Answer the following: (35=15)


a) "The auditors should communicate audit matters of governance interest arising
from the audit of financial statements with those charged with the governance of
an entity". Briefly state the five major matters to be included in such
Communication.
b) Discuss ―Audit through the computer‖.
c) Treatment of foreign currency monetary items on balance sheet date.

Answer:

a) NSA 260 deals with communications of audit matters with those charged with governance.
The following are the audit matters of governance interest which are to be communicated.

(i) The general approach and overall scope of audit including expected limitations.
(ii) The selection of or change in significant accounting policies and practices that have
a material effect on the entity‘s financial statements.
(iii) The potential effect on the financial statements of any significant risks and exposures.
(iv) Adjustment to financial statements arising out of audit which have a significant effecton
the financial statement.
(v) Material uncertainties that may cast significant doubt on the entity‘s ability to continue
as a going concern.
(vi) Disagreement with management on matters which could have significant impact to the
financial statements and to audit report.
(vii) Expected modifications to the audit report.
(viii) Others matters like material weakness in internal control measures, questions on
management integrity and fraud involving management.
(ix) Other matters agreed in terms of audit engagement.

ZYA P.T.O.
(25)

b)
Computerization of accounts does not affect the basic objectives of auditing. However, the
auditor would need to modify his audit procedures, approach and technical capabilities so as
to be able to form an opinion on the accounts processed in a computerized environment.
Audit through the Computer is the audit, with the help of computer technology in the audit
of accounts processed in a computerized environment.Audit through the computer
requires that the auditor submits data to the computer for processing. The results are then
analyzed for the processing reliability and accuracy of the computer program. Technical
and other developments that necessitated this approach include the following;
i. On line data entry
ii. Elimination or reduction of print outs
iii. Real time files up dating

The auditor can use the computer to test:


i. The logic and controls existing within the system, and
ii. The records produced by the system

Depending upon the complexity of the application system being audited, the approach may
be fairly simple or require extensive technical competence on the part of the auditor.
There are several circumstances where auditing through the computer must be used:
a. The application system processes large volumes of input and produces large
volumes of output that make extensive direct examination of the validity of input
and output difficult.
b. Significant parts of the internal control system are embodied in the computer
system. For example, in on line banking system a computer programme may
batch transactions for individual tellers to provide control totals for reconciliation
at the end of the day‘s processing.
c. The logic of the system is complex and there are large portions that facilitate use
of the system for efficient processing.
d. There are substantial gaps in the visible audit trail.

The primary advantage of this approach is that the auditor has increased power to effectively
test the computer system. The range and capability of tests that can be performed increases
and the auditor acquires greater confidence that data processing is correct. By examining the
system‘s processing, the auditor also can assess the system‘s ability to cope with
environment change.
The primary disadvantages of the approach are generally high costs and the need for
extensive technical expertise when systems are complex. However, these disadvantages are
really not that important if auditing through the computer is the only viable method of
carrying out the audit.
Auditing through computer may be conducted through test data, computer programme, etc.
c)
Foreign Currency Monetary Items and its Treatment on Balance Sheet date: As per NAS- 11
on ―Accounting for the Effects of Changes in Foreign Exchange Rates‖ monetary items are
money held and assets and liabilities to be received or paid in fixed or determinable amounts
of money, e.g., cash, receivable, payables, etc. Regarding foreign currency transactions,
NAS 11 requires that while reporting effects of changes in exchange rates subsequent to
initial recognition, at each balance sheet date, monetary items denominated in a foreign
currency, (e.g., foreign currency notes, balances in bank accounts denominated in a foreign
currency, and receivables, payables and loans denominated in a foreign currency) should be
ZYA P.T.O.
(26)

reported using the closing rate prevailing on balance sheet date. However in certain
circumstances the closing rate may not reflect, with reasonable accuracy, the amount in
reporting currency that is likely to be realized from or required to be disbursed to because
the rate is unrealistic. In such circumstances, the relevant monetary item should be reported
in the reporting currency at the amount which is likely to be realized from or required to be
disbursed to at the balance sheet date.

5. Comment on the following situations/statements. (35=15)


a) On final audit of Hattiban Limited, the physical verification of fixed assets was
conducted. However the auditor was not able to confirm the existence of valuable
items and costly equipments. Though, the auditor received a certificate from the
management to prove its existence and value and accepted the same blindly
without applying any further procedures.
b) A firm or a member of the audit team accepts gifts or hospitality of which the
value is not trivial and inconsequential. (Give your answer on the basis of the
Provision of IFAC Code of Ethics adopted by ICAN.)

c) ABC Commercial Bank provided loan to an audit team member under terms and
conditions other than normal. (Give your answer on the basis of the Provision of
IFAC Code of Ethics adopted by ICAN.)

Answer:
a) The physical verification of fixed assets is the primary responsibility of the management.
The auditor, however, is required to examine the verification programme. Further, he must
satisfy himself about the existence, ownership, procession and valuation of fixed assets. It
appears from the facts of the case that the auditor has not been able to verify either existence or
valuation of significant fixed assets despite conducting physical verification audit procedure
himself. Ultimately, he accepted the certificate from the management without performing
further procedures. As per NSA 580, ―Written Representations‖, representation by management
cannot be a substitute for other audit evidence that the auditor could reasonably expect to be
available. Thus, a representation by management as to the existence of valuables and machinery
is no substitute for adopting normal audit procedures regarding verification of valuable and
important machinery. If the auditor is unable to obtain sufficient appropriate audit evidence that
he believes will be available, this will constitute a limitation on the scope of his examination
even if he has obtained a representation from management on the matter.

b) There will be self-interest and familiarity threat to compliance with the fundamental principles
which the auditors are required to comply. The threat would be so significant that no safeguards
could reduce it to an acceptable level. The firm or the member of the audit team shall not accept
such gifts or hospitality.

c) There will be self-interest threat and the significance of the threat would be so significant that
no safeguard could reduce it to an acceptable level. So such loan shall not be accepted.

6. Write short notes on the following (Any Four): (42.5=10)


a) Contingent assets
b) Disclaimer of opinion
c) Three important characteristics of an effective system of computer audit
programme
d) Analytical procedures in planning an audit

ZYA P.T.O.
(27)

e) Familiarity threat

Answer:
a) The contingent assets are those which may arise on the happening of an uncertain event. As a
general practice, contingent assets are not recorded in the balance sheet because that would imply taking
credit for revenue which has not accrued. But it is logical as the contingent liabilities are shown in the
balance sheet the contingent assets should also be shown. The Companies Act does not require disclosure
of contingent assets in the balance sheet. However, if contingent asset have a significant value, it may be
advisable to disclose such assets in a note to the balance sheet.
As regards valuation of contingent assets, it may be noted that ordinarily no valuation would be required.
However, if such assets were disclosed by way of a note, a proper valuation based on the related contract
would be made. Where full realization of such assets is doubtful even on the face of contingency
occurring, it would be safer to value the assets on a realisable basis.

b)
Depending upon the circumstances, an auditor expresses an opinion on the financial statement.
A disclaimer of opinion should be expressed when the possible effect of a limitation on scope is
so material and pervasive that the auditor has not been able to obtain sufficient appropriate audit
evidence and accordingly is unable to express an opinion on the financial statements.

c)
Important characteristics of an effective system of computer audit program:

(i) Should be simple to use and eliminate the need to remember countless details normally
required in writing or revising computer programs.
(ii) Should be easily understandable even by those with little computer expertise and easyto
use.
(iii) Should be capable of being used with different configuration of computers.
(iv)The package has to include adequate support at the time of installation, provideadequate
training to the staff and to provide documentation. There should be a provision for
future revision of the program.
(v) The package should have statistical sampling capability.
(vi) The system has to be acceptable to all users in terms of easy execution and compatible
with the existing system.
(vii) The program has to be capable of processing different types of applications.
(viii) The program should have strong report writing function including the ability to
preparemultiple reports in a single program run and to generate flexible output report
formats.

d)
In the planning stage, analytical procedures assist the auditor in understanding the client‘s
business and in identifying areas of potential risk by indicating aspects of and developments
in the entity‘s business of which he was previously unaware. This information will assist the
auditor in determining the nature, timing and extent of his other audit procedures.

Analytical procedures in planning the audit use both financial data and non-financial
information, such as number of employees, the square feet of selling space, volume of goods
produced and similar information.
ZYA P.T.O.
(28)

e) Familiarity threat:
The circumstances in which professional accountants operate may create specific threats to
compliance with the fundamental principles which the auditors are required to comply.
Threats may be created by a broad range of relationship and circumstances. One of such
threat may be Familiarity threat. It is a threat that due to a long or close relationship with a
client or employer, a professional accountant will be too sympathetic to their interests or too
accepting of their work. Some of the circumstances that create familiarity threat include:
 A member of the audit team having a close or immediate family member who is a
director or officer of the client,
 Senior personal having a long association with the audit client etc.

7. Distinguish between (Any Two): (25=10)


a) Internal check and Internal control
b) Test checking and statistical sampling
c) Evidence Vs. validity of transaction
Answer:
a)Internal check is a check on day-to-day transactions which operate continuously as part of the
routine system whereby the work of one person is proved independently or is complementary
to the work of another, the object being the prevention or early detection of errors or fraud.
Internal check is a part of the overall internal control system and operates as built-in device
as far as the staff organization and job allocation aspects of the control system are concerned.
The internal control is a plan of an organization and all the methods and procedures adopted
by the management of an entity to assist in achieving management's objective of ensuring, as
far as practicable, the orderly and efficient conduct of its business, including adherence to
management policies, the safeguarding of assets, prevention and detection of fraud and error,
the accuracy and completeness of the accounting records and the timely preparation of
reliable financial information.
b)
During audit, entries, involving large amounts or relating to material accounts are seen
exhaustively and they are picked up for verification at random from the remainder according
to certain plan is known as test checking. The only quality that this technique can claim lies
in its keenness to cover larger amounts and material accounts. Even if errors, frauds etc.
remain undetected in the part not checked, they are not likely to be too big as to upset the
truth and fairness of the financial statement.
On the other hand, Statistical sampling is a method of audit testing which is more scientific
based entirely on the auditors own judgment because it involves use of mathematical laws of
probability in determining the appropriate sample size in varying circumstances. Statistical
sampling has reasonably wide application where a population to be tested consists of a large
number of similar items and more in the case of transactions involving compliance testing,
debtors confirmation, payroll checking, vouching of invoices and petty cash vouchers.

C) Entries in the books are usually made on the basis of some kind of documentary evidence. It
generally exists in a variety of forms e.g. payees receipts, suppliers invoices, statements of
account of parties, minutes of Board of Directors or shareholders, contract documents etc.
These all form documentary evidence for transactions. On the other hand, it is also the
function of audit to establish that payments have been made validly to the person who is
shown to be recipients e.g. salary is paid to partners according to provision contained in the
partnership deed, director's fee is paid according to the minute of shareholders meeting,

ZYA P.T.O.
(29)

suppliers are paid according to their invoices etc. It is termed as the validation of the
transactions.

ZYA P.T.O.
CAP-II, Corporate & Other Laws, June 2012
Suggested Answer

Roll No……………. Maximum Marks - 100

Total No. of Questions - 7 Total No. of Printed Pages -2

Time Allowed - 3 Hours


Marks
Attempt all questions.
Part "A"
1.
a) Who is chairperson of a general meeting? A General meeting was called for
changing the company's memorandum for facilitating an amalgamation. A
fairly large number of members wanted to oppose the move. The meeting
became overcrowded. The venue proved too small. The members were seated
in the adjoining rooms and a sound system was installed to enable them to
hear and to participate. That system failed. Then there was a bewildering
confusion and in that state of things the chairperson arranged for an
alternative venue and resolved to adjourn the meeting to meet again in the
evening at the new venue. There was opposition to this move. The meeting in
the evening at the new venue was held. Decide the validity of the meeting
with reference to the provisions of Companies Act, 1956and legality of
adjournment of meeting referring to relevant case law.
8
b)
i) Mr. Mohamad,a shareholder of Company H, holds 5% of shares.
Company calls 40% of par value of shares. Mr. Mohamad is leaving
India for 10 years and wants to pay uncalled 60% of the par value of
shares. Can it be paid to Company H? Explain with reference to the
provisions of Companies Act, 1956. 3
ii) Write down the mode of winding up of a company referring the relevant
section of Companies Act, 1956. 3
c) ‗A‘ signs, as maker, a blank stamped paper and gives it to ‗B‘, and authorises
him to fill it as a note for Rs. 500, to secure an advance which ‗C‘ is to make
to ‗B‘. ‗B‘ fraudulently fills it up as a note for Rs.2,000, payable to ‗C‘, who
has in good faith advanced Rs. 2,000. Decide, with reasons referring the
provisions of Negotiable Instrument Act, 1881, whether ‗C‘ is entitled to
recover the amount, and if so, up to what extent? 6
d) What is the doctrine of lifting of "Corporate Veil"? What are the causes of
lifting corporate veil? Illustrate your answer with court cases. 5
Answer

a) "Chairperson" is the person who has been designated or elected to preside over
and conduct the proceedings of a meeting. He is usually a member of the body
over which he is to preside. For the proper conduct of business at a meeting, a
chairperson is necessary. His/her appointment is usually regulated by the articles
of association. But if there is nothing in the articles the members personally
present at the meeting shall elect one of themselves to be the chairman.

ZYA P.T.O.
(31)

Section 175(1) of the Companies Act, 1956 provides that, unless the articles of
the company otherwise provide, the members personally present at the meeting
shall elect one of themselves to be the chairman thereof on a show of hands.
Section 175(2) of the Companies Act, 1956 provides that, if a poll is demanded
on the election of the chairman, it shall be taken forthwith in accordance with the
provisions of this Act, the chairman elected on a show of hands exercising all the
powers of the chairman under the said provisions. If some other person is elected
chairman as a result of the poll, he shall be chairman for the rest of the meeting.
The above fact of the case is similar to a British case - Bying Vs. Rondan Life
Assn Limited( 1989) where the meeting in the evening at the new venue was
held to be invalid and the chairman's adjournment of the morning meeting
without taking a sense of the meeting was held to be not lawful. The court
recognized that it is not necessary that all the members should be present in one
room but that if the members are seated in different rooms here should be an
audio visual link between all the rooms. In this case the system having failed, the
gathering could not be constituted into a valid meeting except for rudimentary
purpose of adjourning itself. Under the articles of the company the chairperson
could adjourn a meeting only with the consent of the meeting. But even so the
court was of the view that a residuary power of adjournment would survive
because circumstances may develop as they did in this case, where it is
impossible to ascertain the view of the majority. Finally the court came to the
conclusion that the chairman's power of adjournment was not fairly exercised.
He failed to take in to account the relevant factors such as member attempts
adjournment their objections to the temporary adjournment and that there was no
such urgency. The court said that for testing the reasonableness of the conduct of
a chairman, the same test is applicable which is generally applied in all cases of
judicial review namely whether the chairman reached a conclusion which no
reasonable chairman could have reached having regarded to the purpose of his
power to adjourn.

b) (i) According to Sec. 92 of the Companies Act, 1956 uncalled share amount can
be paid to the company in the following conditions:
(1) A company may, if so authorised by its articles accept from any member the
whole or part of the amount remaining unpaid on any shares held by him,
although no part of that amount has been called up.
(2)The member shall not however be entitled, where the company is one limited
shares, to any voting rights in respect of the moneys so paid by him until the
same would, but for such payment, become presently payable.
Based upon the above provisions, Mr. Mohamad, a shareholder of Company
H, can pay the uncalled amount.

(ii) According to Sec. 425 of the Companies Act , 1956 a Company can
wounded up in the following three modes:
 By the court
 Voluntarily and
 Subject to supervision of the court

ZYA P.T.O.
(32)

c) Section 20 of the Negotiable Instruments Act, 1881 provides that when one
person signs and delivers to another a paper stamped in accordance with the law
relating to negotiable instrument then in force in India and either wholly blank
or having written thereon an incomplete negotiable instrument, he thereby gives
prima facie authority to the holder thereof to make or complete, as the case may
be, upon it a negotiable instrument, for any amount specified therein and not
exceeding the amount covered by the stamps. The person so signing shall be
liable upon such instrument, in the capacity in which he signed the same to any
holder in due course for such amount. A person other than holder in due course
is not authorised to recover anything in excess of the amount intends by him to
be paid thereunder.

The doctrine contained in section 20 is that a person who gives another


possession of his signature on blank stamped paper prima facie authorises the
latter as his agent to fill it up and give to the world the instrument as accepted by
him. The doctrine is one of estoppel. In the given problem ‗A‘ is estopped from
setting up B‘s fraud, and ‗C‘ is entitled to recover Rs. 2,000/- from ‗A‘ because
‗C‘ has obtained it as a holder in due course. This liability does not stand of a
person other than the holder in due course. ‗C‘ as a holder in due course is
entitled to enforce payment of the full amount even though the authority has been
exceeded but it is necessary that the sum ought not to exceed the amount covered
by the stamp. [Lloyds Bank vs. Cooke (1907) KB 794]

d) Company is a structure comprised by its machinery like Board of Director,


promoter, executive personnel and so on. When it is incorporated it bears with
its independent or corporate personality. What this means is that the company
has life of its own, can own property, can sue and be sued in its own name, has
perpetual life and existence to name a few of the benefits of incorporation. This
rule was laid down by the House of Lords in Salomon v. Salomon & Co1, in
1897, in which it was held that even if one individual held almost all the shares
and debentures in a company, and if the remaining shares were held on trust for
him, the company is not to be regarded as a mere shadow of that individual.

This doctrine is recognized by the law court when it is incorporated, operated


and used to attain legal objectives. However, court are not recognized being the
independent existence and personality when company is incorporated or operated
and used to attain illegal objectives, to defraud promoters or to do against
national or social interest. At this connection, in the sense of justice, company's
corporate veil is lifted to find out reality of the company and to render justice for
the aggrieved parties. Who are in the company? What are their functions and
duties? Who has indulged in misconduct, illegal activities, corruption and in
doing against the interest of investors, shareholders are examined having set
aside the independent personality of the company. Persons indulge in said
activity is made personally liable. Such rule or doctrine is said to be the doctrine
of lifting the corporate veil.
The rule in Salomon lies at the heart of corporate personality. However, there are
situations in which the courts look beyond that personality to the members or
directors of the company: in doing so they are said to lift or pierce the corporate
veil. There is no single basis on which the veil may be lifted, rather the cases fall
into several loose categories, which are examined below.
ZYA P.T.O.
(33)

i) Fraud:

The courts have been more that prepared to pierce the corporate veil when it fells
that fraud is or could be perpetrated behind the veil. The courts will not allow the
Solomon principal to be used as an engine of fraud. The two classic cases of the
fraud exception are Gilford motor company ltd v. Horne and Jones v. Lipman . In
order to defeat this he incorporated a limited company in his wife's name and
solicited the customers of the company. in this case it was clear that the main
purpose of incorporating the new company was to perpetrate fraud. Thus the
court of appeal regarded it as a mere sham to cloak his wrongdoings

ii) Group Enterprise:

Sometimes in the case of group of enterprises, the Solomon principal may not be
adhered to and the court may lift the veil in order to look at the economic
realities of the group itself. In the case of D.H.N.food products Ltd. V. Tower
Hamlets it has been said that the courts may disregard Solomon's case whenever
it is just and equitable to do so. In the above-mentioned case the court of appeal
thought that the present case where it was one suitable for lifting the corporate
veil. Here the three subsidiary companies were treated as a part of the same
economic entity or group and were entitled to compensation.

iii) Agency:

A company having power to act as an agent may do so as an agent for its parent
company or indeed for all or any of the individual members if it or they authorize
it to do so. In cases where the agency agreement holds good and the parties
concerned have expressly agreed to such a agreement them the corporate veil
shall be lifted and the principal shall be liable for an acts of the agent.

iv) Trust:

The courts may pierce the corporate veil to look at the characteristics of the
shareholders. In the case of Abbey and Planning the court lifted the corporate
veil. In this case a school was run life a company but the shares were held by
trustees on educational charitable trusts. They pierced the veil in order to look
into the terms on which the trustee held the shares.

v) Tort:

Usually the English courts have not lifted the veil on the ground of tort it is a
phenomenon not witnessed in most common law jurisdictions apart from
Canada.

vi) Enemy character:

In times of war the court is prepared to lift the corporate veil and determine the
nature of shareholding as it did in the Daimler Co. Ltd v. Continental Tyres
and Rubber Co. case where Germen shareholders held the shares of an English
company during the time of world war 1.
ZYA P.T.O.
(34)

vii) Tax:

Corporate Veil is lifted or pierced when a company is incorporated, operated or


used to evade tax or defraud. In Income Tax Commissioner V. Sri Meenakshi
Mills case it has hold that: The income tax authorities are entitled to pierce the
veil of corporate entity and to look at the reality of the transactions. It is true that
from the juristic point of view the company is a legal personality entirely distinct
from its members and the company is capable of enjoying right and being
subjected to duties, which are not the same as those enjoyed or borne by its
members. But in exceptional cases the court is entitled to lift the veil of corporate
entity and to pay regard to the economic realities behind the legal facade.

Part"B"
2.
a) Mrs. Merritt has sold one Ropani land to Mr John in the price of Rs.8 Lakh
that is less in two times than market price. Mr. John had threatened to injure
body and reputation of Mrs. Merritt before such transaction. What kind of
such contract is? Whether this contract is valid or not? What are the remedies
available to Mrs. Merritt provided in the Contract Act, 2056? Explain briefly. 8
b) A loan of Rs. 15,00,000 has been sanctioned by Laxmi Bank Ltd. to
Loktrantra Resort Pvt. Ltd. having authorized capital of Rs. 1,00,00,000, and
paid up capital of Rs. 10,00,000. The board of directors of Loktrantra Resort
Pvt. Ltd. resolved to obtain loan from the Bank. The chairperson of
Loktrantra Resort Pvt. Ltd. before obtaining the sanctioned loan wants to
know with you that whether the loan which the company is going to borrow
as stated above is as per Companies Act, 2063. Advise the chairperson as per
the provision of Companies Act, 2063 about its legality or formality or
consequences if any. 7
Answer

a) According to the above question Mrs Merritt has no will or consent to sell
own property as proposed by the Mr John. However, She has sold and
transferred the property in the price she is offered. It means the transaction is
made through against the consent and free will of Mrs Merritt.

In connection with the above facts in issue, it should be mention the


provision of section 14. It provides about voidable contract and sub-section 1
state as the following contracts may be made void by the aggrieved party if it
is concluded through coercion. Coercion has defined in its explanation as a
person shall be deemed to have indulged in coercion if he/she, with the
objective of compelling any person, to accept any contract against his/her
will, withholds or threatens to withhold property belonging to him/her, or
threatens to defame him/her, or takes or threatens to take any other action in
contravention of prevailing law.

Section 89 ( 2) (b) of the Act provides Limitations in the case of a voidable


contract, as defined in the section 14(1)(a), within a year from the date when

ZYA P.T.O.
(35)

the party that can have the contract declared void learns about the reason for
having the contract declared to be void.

In case a contract is made void under the law, the cash or goods received by
one party should be refund after adjusting the accounts until the term of the
contract expires from the amount paid in cash or in kind. In case any service
or benefit other than cash or goods has been provided, the beneficiary must
pay a reasonable amount to the other party in consideration thereof.

In the connection with the above matters, it is clear that threatening to injure
body and reputation is the terms and element of the coercion. Mr. John has
compelled to Mrs. Merritt to accept the selling transaction. Any contract
involving coercion is called voidable contract and may be made void by the
aggrieved party Mrs. Merritt on the reference of above provision. Mrs.
Merritt has to file a case against such transaction within a year she learns as
coercion involved. If the activity of Mr. John is proved as coercion then the
contract is void. Mrs. Merritt may regain the property and she has to refund
the amount if she has already received from Mr. John.
b) Under section 105(1)(b) of the Companies Act, the board of directors of a
public company, or of a private company receiving loans from any bank or
financial institution, shall not, except with a special resolution being adopted
by the general meeting of shareholders, do or cause to be done the following
act:

borrowing moneys, where the moneys to be borrowed will exceed the


aggregate of the paid up capital of the company and its free reserves, apart
from any loans and faculties with a term of less than six months obtained by
it from a bank or financial institution in the ordinary course of business
transaction;

As per above provision the loan to be borrowed exceeds the paid up capital.
The question is silent about the free reserves. It the money to be borrowed
exceeds special resolution in the general meeting should be passed. Thus the
loan which is going to be borrowed by the Loktrantra Resort Pvt. Ltd is not
void in the sense that it has not passed special resolution.

Consequences
Under section 180 of the Act where any act or action required to be done or
taken under this Act has not been done or taken or any act or action
prohibited has been done or taken by any company or in respect of such
company, such act or action shall be void.

Under section 160(y) of the Act director or officer who acts contrary to
Section 105 shall be punished with a fine from twenty thousand rupees to
fifty thousand rupees or with imprisonment for a term not exceeding two
years or with both punishments.
3.

(2×5=10)

ZYA P.T.O.
(36)

a) A' borrows moneys from 'B" on behalf of the company pursuant to the
contract made between A and B prior to incorporation of the company. At
this situation, what is the status of such contract? Whether such contract is
mandatory to the company or not? Who is responsible to that transaction and
borrowing? How does 'A' release from personal liability? Discuss briefly
referring the provisions of Companies Act, 2063.
b) Explain the power of the court under the Securities Act, 2063 to impose
punishment on an offence relating to securities.

Answer

a) This questions' concern is pre-incorporation contract. It should be assessed


whether pre-incorporation contract is mandatory to the company or not with
the reference of the Companies Act, 2063. Section 17 of the Companies Act,
2063, has made provision on pre-incorporation contract as follows:
i) A contract made prior to the incorporation of a company shall be a
proposed contract only.
ii) such contract shall not be binding on the company.
iii) Any person, prior to the incorporation of the company, carries on any
transaction or borrows money on behalf of the company, such person
shall be personally liable for any contract related with the transaction
so carried on.
iv) If, within the time mentioned in any transactions or within the
reasonable time after the incorporation of a company, the company,
through its act, action or conduct, accepts any act, action or conduct, to
borrowing done or made prior to the date of authorization to
commence its transactions or endorses such act or action, that
transaction shall be binding on the company and the other contracting
party.
v) The person carrying out such act or action shall be released from the
personal liability to be borne pursuant to the above provision.
vi) For the private company the consensus agreement of a private
company shall govern any contracts made prior to the incorporation of
such company.

A contract made prior to the incorporation of a company is called pre -


incorporation contract. According to above provision of the Act such contract
is remain only proposed contract. Company could not bear responsibility
only because the contract is concluded for and on behalf of company. For
this, A is personally liable. However, company is liable when it, within the
time mentioned in the contract and within the reasonable time with the act,
actions or conduct accepts such transactions or borrowing. A can escape or
get rid from the liability of B.

In the context of private company, the consensus agreement of a private


company shall govern any contracts made prior to the incorporation of such
company pursuant to Sub-section (4) of section 17 of this Act.

b) Under section 101 Securities Act (1) A person who commits an insider
trading as referred to in Section 91 shall, upon being convicted of the offense
ZYA P.T.O.
(37)

of insider trading, be liable to the punishment with a fine equal to the amount
in controversy or with imprisonment for a term not exceeding one year or
with both punishments.
(2) A person who commits any act referred to in Sections 94(False trading),
95 (Fluctuation in price) and 96 (To affect stock exchange) shall be liable to
the punishment with a fine of fifty thousand rupees to one hundred thousand
rupees or with imprisonment for a term not exceeding one year or with both
punishments, and where anyone has suffered any loss or damage from such
an act, such loss or damage has also to be recovered.
(3) A person who commits any act referred to in Sections 97 (To supply
misleading statements:, 98 (Fraudulent transaction), 99 (Prohibition on
transaction of securities by fraud or misrepresentation) and 100 (Destroy or
concealment of documents, statements or records) shall be liable to the
punishment with a fine of one hundred thousand rupees to three hundred
thousand rupees or with imprisonment for a term not exceeding two years or
with both punishments, and where anyone has suffered any loss or damage
from such transactions, such loss or damage shall also be recovered.

Securities Act empowers the court to impose punishment on above offense.


However the word court has not been mentioned on the above section but it
has been mentioned the power of securities board in some other offenses. It
should be deemed the power of the court to impose punishment other than the
matters expressly mentioned as the power of securities board.

4.
a) Kathmandu Bank Ltd. had come in breakeven point in last Fiscal Year and
going to earn profit more than Rs. 20 Crore this Year 2068/069. This Bank is
bringing a proposal of distributing bonus in cash and share for the coming
AGM. Mr. Sushil K.C got such unpublished information through one of the
director of the Bank and purchased securities of such Bank on the basis of
insider information or notice that are unpublished. Answer the following
questions on the basis of Securities Act, 2063. (2+1+2=5)
i) What kind of offense of the director and Mr. Sushil K.C committed and
why? Explain.
ii) Do you think that the purchasing securities by Mr Sushil KC deemed to
be affected at all or not merely by the reason that they have committed an
insider trading?
iii) Who are the persons likely to be involved in insider trading?
b) State the procedure for changing the Company name as per Companies Act,
2063. 5

Answer
a)
i. According to Section 91of the Securities Act, 2063 if any person deals in
securities or causes any other person to deal in securities on the basis of any
insider information or notice that are unpublished or communicates any
information or notice known to such a person in the course of the discharge
ZYA P.T.O.
(38)

of his or her duties in manner likely to affect the price of securities such a
person shall be deemed to have been committed an insider trading in
securities. On the above case, it is same to the provision provided in the
section 91. So, the director and Sushil KC have committed the offence of
insider trading.

ii. Yes, actually, director and Mr. Sushil KC have committed offence of insider
trading. However, pursuant to section 91(2), the purchasing of securities shall
not be deemed to be affected at all merely by the reason an insider trading
has been committed.

iii. The following persons shall be deemed to be those who have access to the
insider information or notice not published by any body corporate:

a. A director, employee or a person, who can obtain any information or a


notice in the capacity of a shareholder of that body corporate,

b. A person who can obtain any information or a notice in the capacity


of a professional service provider to that body corporate,

c. A person who can obtain any information or a notice having a direct or


indirect contact with the person or source as specified in Clauses (a) and
(b).

b) According to Sec. 21(3) of Company Act, 2063, if any existing company


wants to change its name, the new name to be passed from its shareholders‘
meeting under special resolution mechanism. Company need to apply to the
Registrar of the Companies for the new purposed name with fee prescribed
for the time being. If ROC approves the purposed name, new name shall be
allowed for. Until the ROC approves the new name, company shall operate
its business under old name.
5. (3×5=15)
a) What are the conditions to declare any commercial bank or financial
institution problematic under the Nepal Rastra Bank Act, 2058? State briefly.
b) What is Cottage Industry? What industries are covered by Cottage Industry?
Discuss referring the Industrial Enterprises Act, 2049.
c) What are the conditions regarding eligibility of bonus and restrictions to
obtain thereof enshrined in the Bonus Act, 2030?
Answer

a) The following conditions are prevailing in any commercial bank or financial


institution Bank shall declare problematic as per the section of 86.b of Nepal
Rastra Bank Act, 2058.

ZYA P.T.O.
(39)

(a) Any action which is against the interest of the depositors, shareholders,
creditors, or general public,
(b) Not fulfillment of any financial liabilities or not having probability to do
that or not payment of due amount,
(c) In case of insolvency or going to fall under insolvency or facing material
financial difficulties,
(d) In case of discredit or breach of this Act, prevailing law related to bank
and financial institution, other prevailing law, terms of license or
regulation, directives or order of bank,
(e) In case the license obtained on the basis of submitting false, fraudulent,
wrong document or data,
(f) Unable to maintain the capital fund as per the NRB Act, prevailing law
related to bank and financial institution and directives issued by the bank
at time to time,
(g) Initiation of the process of liquidation or insolvency of any commercial
bank or financial institution under the prevailing law,
(h) In case of undue delay in the process of voluntary liquidation,
(i) While foreign commercial bank or financial institution is in insolvent or
liquidator is appointed for the liquidation or the license of such
commercial bank or financial institution is terminated under the
provision of the law of respective country or transaction is banned either
full or partial or in case of operation of banking transaction being
involved with such commercial bank or financial institution, or
(j) If the bank is convinced that commercial bank or financial institution is
unable to pay it‘s due or can make negative effect in its liability or
duties, which it has to perform.

b) Under section 4 Industrial Enterprises Act 2049, the traditional industries


utilizing specific skill or local raw materials and resources, and labour
intensive and related with national tradition, art and culture as mentioned in
Annex I shall be named as cottage industries.

Annex-1
(Relating to Section 4)
COTTAGE INDUSTRIES
With the exception of cigarette, bidi, cigar, chewing tobacco, khaini industries
and industries producing other goods of a similar nature utilizing tobacco as
the basic raw material, alcohol and beer producing industries,* Handloom,
Pedalloom, Semiautomatic loom, Warping, Dyeing and printing, Tailoring
(Other than Readymade Garments), Knitting, Handknitted Woolen Mat and
Blanket (Radi, Pakhi), Woolen Carpet, Pashmina, Wollen Garments,
Carpentry, Wooden Artistic product, Cane and Bamboo Works, Natural Fiber
Products, Hand Made paper and Goods made up thereof, Gold, Philigiree
Products including Silver, Brass, Copper Precious and Semi-Precious Stones,
Ornaments, Sculptures and Pottery, Honey, chyuri, Cardamom Processing,
Clay or Ceramic Pottery, Leather Cutting and Tanning, Rural Tanning and
Leather Goods producing Works, Jute, Sabai Grass, Babio, Choya, Cotton
Thread Products, Artistic Products made up of Bones and Horns, Stone
Carving, Ceramic Fine Arts, Pauwa, Boutique, Incense Stick (Dhup), Dolls
and Toys industries and cottage industries with the fixed asset of up to two
hundred thousand rupees.
Notes:-
ZYA P.T.O.
(40)

1. Unless otherwise mentioned specifically, machines of the above mentioned


industries should not employ through electric motors of diesel or petrol or
crude oil engine more than a total of five kilowatt. Power looms shall not be
included under Cottage Industries.
2. Permission shall be required for the establishment of mechanized woolen
spinning and mechanized woolen carpet manufacturing.
c) Bonus Act, 2030 has mentioned in its section 6 and 8 about the suggestion of
above question.

As per the Section 6 of the Bonus Act, 2030, an employee who has worked for
a half period to be worked in a fiscal year, shall be entitled to obtain bonus
under to this Act. However, no employee shall be entitled to obtain Bonus
who has worked casually or in a shift basis.

The working period of an employee is also play vital role to determine


whether he/she obtain bonus or not. The following periods shall also be
computed as a period where an employee has worked.

i)A period kept on reserve under any contract or under Section 11 of the
Labour Act, 2048 (1991).
ii)A period under which an employee is on any leave with salary.
iii)A period of disablement caused by accident arising in course of business of
the enterprise.

Restrictions to obtain bonus as per the Bonus Act, 2030 is as follows:


Section 8 of this Act has state restrictions to obtain bonus. 8. Notwithstanding
anything contained in Section 6, an employee shall not be entitled to obtain
bonus under this Act, if he/she is punished or dismissed from service for
committing any act as follows:

Provided that, this Section shall not be deemed to be prejudiced to obtain in


the case of the bonus for a period before committing such a punishable act.

i) Theft of the property of the enterprise or any damage to such


property.
ii) Illegal strike or abetment to other for such strike,
iii) Riots or breaching of discipline.

6. (2×5=10)
a) Who are the parties of the Negotiable Instrument? What liability is given to
such party as mentioned in the Negotiable Instrument Act, 2034?
b) What are the circumstances wherein the Councilor's office remains vacant?
What is the procedure for fulfillment of vacancy as provided in the Nepal
Chartered Accountants Act,2053?

Answer
Parties of the Negotiable Instrument are as follows:
i)Agent

ZYA P.T.O.
(41)

ii)Heir
iii)Drawer
iv)Bank
v)Maker of promissory note and Acceptor of bill of exchange
vi)Endorser
vii)Parties to holder in due course
viii)Maker or drawer of Negotiable Instrument
ix) Prior party
x) Surety
Liability of the parties of the Negotiable Instrument as mentioned above is stated as
follows:
i) Liability of an Agent: If an agent who signs his/her name to a Negotiable
Instrument without indicating thereon that he/she signs as an agent, or he
does not intend thereby to incur personal responsibility, is liable personally to
such Negotiable Instrument.
ii) Liability of an Heir: If a person who signs his/her name to a Negotiable
Instrument as an heir of a deceased person is fully liable personally thereon
unless he/she expressly limits his liability to the extent of the assets received
by him as such.

iii) Liability of Drawer: In case the Drawer has been provided due notice of
dishonour of the bill of exchange by the acceptor or Drawee, it shall be the
duty of the Drawer to Compensate the Holder.

iv) Liability of the Bank Giving Payment of the Cheque: The Bank having
sufficient funds of the Drawer in the account, properly applicable to the
payment of the Cheque must pay the Cheque, and, in default of such payment
must compensate the Drawer or Holder in due Course for any loss or damage
caused by such default pursuant to this Act.

v) Liability of Maker of Promisory Note and Acceptor of bill of exchange:


The maker of a Promissory Note or the acceptor of bill of exchange is bound
to pay the amount thereof at Maturity when the Negotiable Instrument is duly
presented for the payment. In default of such payment, the maker of the
Promissory Note or the acceptor of the bill of exchange is bound to
compensate any party payable for any loss or damage sustained by him and
caused by such default.

vi) Liability of Endorser: Any person who indorses and delivers a Negotiable
Instrument expressly limiting or excluding his/her own liability, shall be
responsible to every subsequent Holder except otherwise a contract is made
thereto.

vii) Liability of the Parties to Holder in Due Course: Every prior party to a
Negotiable Instrument is liable thereon to a Holder in due Course until the
instrument is duly satisfied.]

viii) Liability of maker or drawer to be Equivalent to Principal Debtor: The


maker or Drawer of the Negotiable Instrument until acceptance, and the
acceptor are in the absence of a contract to the contrary respectively liable
thereon as principal debtors, and the other parties thereto are liable thereon as
sureties for the maker, Drawer or acceptor, as the case may be.

ZYA P.T.O.
(42)

ix) Prior party to be Equivalent to Principal Debtor in Respect of each


Subsequent Party: As between the parties so liable as sureties, each prior
party is, in the absence of a contract to the contrary, liable thereon as a
principal debtor in respect of each subsequent party.

x) Liability of Surety: Notwithstanding anything contained in the prevailing


laws, the surety of Bill of Exchange cannot take stand that only in case the
realization could not be possible from the property of the principal debtor,
shall be realized from his/her property, it can be directly realized from the
person being a surety
b) In the following circumstances, the Councilor's office shall be deemed to
remain vacant if
i. he ceased to be member of the Institute provided that this provision
shall not be applicable to the nominated council member,
ii. the council accepts his resignation,
iii. he, without giving a notice with reason ,absconds himself from three
consecutive meeting of the council,
iv. his term of office expires ,
v. his non compliance with the code of conduct referred to in section 34 is
provided,
vi. he dies
Procedure for fulfillment of vacancy
The council shall, if any seat of any council member elected amongst
Chartered Accountant members or Registered Auditors members turns
vacant due to death or resignation or disqualification to continue as a
member of the institute pursuant to the other provisions of this Act,
designate any member as council member for the remaining term of office,
provided the remaining period of such vacated office is of less than a year;
and if such term is more than a year, the vacancy shall be filled through
election.
A seat, falling vacant owing to death or resignation of any council member
nominated by government upon the recommendation of Auditor General,
shall be fulfilled, for the remaining term of office, as per the procedure in set
forth in the section 7 of the Nepal Chartered Accountants Act 2053.

7 Write short notes on the followings:


(3×5=15)
a) Impact of liquidation of an insurance company on the insures
b) Bill in Sets
c) Validity of agreement between shareholders
Answer
a) According to Sec. 16 of Insurance Act, 2052 regarding Payment of Insurance
Claims After the Cancellation of the Insurer, the Insurer, dissolved by the
cause of the cancellation of its registration pursuant to Section 13, shall
refund the amount received by it for Insurance to the person, organization or
the Board, within the period and method specified by the Board. It shall
refund the principal amount along with bonus as specified by the Board in the

ZYA P.T.O.
(43)

case of Life Insurance and it shall refund the principal amount as specified by
the Board on a proportional basis in the case of Non-Life Insurance.

b) Generally foreign bills are drawn in a set of two, three or four in order to
avoid delay which may be caused if a single bill sent for acceptance is lost or
miscarried in transit. A bill is therefore drawn in parts. Each part is called a
'via' and is sent by a separate mail so that at least one part of them safely
reaches the drawee for his acceptance. When the latter receives any of these
parts and accepts it, the remaining ones are treated as cancelled. All the parts
together constitute one set and the whole set constitutes one bill; each part
bears reference to the other parts.

Under Section 98 of Negotiable Instruments Act 2034Bills of Exchange may


be drawn in parts, each part being numbered and containing a provision that
it shall continue only so long as the others remain unpaid. All the parts
together make a set; but the whole set constitutes only one bill, and is
extinguished when one of the parts, if a separate bill, would be extinguished.

c) A company is an organized body comprising promoter, shareholder, Board of


Director, personnel and so on. They should maintain management, operation
of the company and the use of voting right conferred to them. Regarding such
functions they may enter into an agreement.

Like this, shareholders may conclude an agreement in respect of the


management, operation of the company and the use of voting right conferred
to them. Such agreement shall be binding on them pursuant to section 187 of
Companies Act, 2063. However, if any provision of such agreement is
prejudicial to the interest of the company or its minority shareholders, such
provision shall ipso facto be invalid to that extent.
The agreement entered between shareholders should submit two copies to the
company within 15 days after the agreement entered into. And such
agreement should submit one copy in the company registrar office.

ZYA P.T.O.
CAP-II, Cost & Management Accounting, June 2012
Suggested Answer

Roll No……………. Maximum Marks - 100


Total No. of Questions: 6 Total No. of Printed Pages - 5
Time Allowed - 3 Hours

Marks
All questions are compulsory. Working notes should form part of the
answer.
Make assumptions wherever necessary.

1. Exclusive Cable Ltd. manufactures plastic pipes (Normal) from a material


which gets completed in two processes; Fabrication and Finishing. The
details of material used and expenses for production of the Normal Product
for the month of Chaitra 2068 were as below:
Fabrication Process:
15,000 kgs of material were used during the month and out of the same,
13,500 kgs were fabricated and transferred to Finishing Process and 1,500
kgs remained as work- in- progress. The closing work-in-progress was 1/3rd
completed in respect of labour and overheads. There was no work-in-
progress at the beginning of the month. The cost of material used was
Rs.1,800,000 and cost of labour and overheads incurred in the process was
Rs. 414,000.
Finishing Process:
There were 900 kgs of product in work-in-progress at the beginning of the
month. The opening work-in-progress was 1/3rd completed in respect of
labour and overheads. The cost of work-in-progress was Rs. 141,000. At the
end of the month there were 600 kgs of product as work-in-progress. The
closing work-in-progress was 25% complete in respect of labour and
overheads. The company incurred Rs. 273,000 for labour and overheads in
this process during the month.
The finished product (Normal) produced after completion of Finishing
Process is sold at Rs. 200 per kg.
Seeing the demand for durable products to be used in buildings, the
management of the company is considering production of more durable and
better plastic pipes (Advanced) by further treatment of the finished product
(Normal) of Finishing Process. This Advanced product could be sold at Rs.
235 per kg in the market. The treatment plant installation costs Rs. 8,000,000
and the cost for treating the quantity produced by Finishing Process in the
month is estimated to be Rs. 345,000. For the implementation of this plan,
the management desires a minimum return on investment of 25% per annum.

Required: (8+4+2+6=20)
a) Prepare equivalent unit statement for both processes and show the cost of
closing stock and completed units.
b) Prepare process accounts.
c) Prepare profitability statement of the Normal Product.

ZYA P.T.O.
(45)

d) Determine whether the implementation of the management's plan to


produce Advanced Product is acceptable.

Answer No. 1
a)
Equivalent Unit Statement for Fabrication Process
Particulars Material Completion Labour & Completion
% Overheads %
Opening WIP (kgs) (work done in this period) - - - -
Completed Unit (kgs) (Input - Closing Stock) 13,500 100 13,500 100
Closing WIP (kgs) (work done in this period) 1,500 100 500 33.33
Total Equivalent Units (kgs) (work done in this period) 15,000 14,000
Cost incurred (Rs.) 1,800,000 414,000
Cost per equivalent unit (Rs.) 120 29.57

Cost of Closing Stock of WIP Rs.


Material Cost (1500 kgs x Rs.120) 180,000
Labour cost (500 kgs x Rs.29.57) 14,785
Total 194,785

Cost of completed stock (transferred to next process) Rs.


Cost of Opening stock of WIP -
Material Cost of input 1,800,000
Labour cost incurred 414,000
Less: cost of closing stock of WIP 194,785
Total 2,019,215

Equivalent Unit Statement for Finishing Process


Particulars Material Completion Labour & Completion
% Overheads %
Opening WIP (kgs) (work done in this period) - - 600 66.67
Completed Unit (kgs) (Input - Closing Stock) 12,900 100 12,900 100
Closing WIP (kgs) (work done in this period) 600 100 150 25
Total Equivalent Units (kgs) (work done in this period) 13,500 13,650
Cost incurred (Rs.) 2,019,215 273,000
Cost per equivalent unit (Rs.) 149.57 20

Cost of Closing Stock of WIP Rs.


Material Cost (600 kgs x 149.57) 89,742
Labour Cost (150 kgs x 20) 3,000
Total 92,742

Cost of completed stock Rs.


Cost of Opening stock of WIP 141,000
Material Cost of input 2,019,215
Labour cost incurred 273,000
Less: cost of closing stock of WIP 92,742
Total 2,340,473

ZYA P.T.O.
(46)

Assumption: FIFO method of inventory issue for production process.

b)
Fabrication Process A/c
Particulars Unit (Kg) Amount (Rs.) Particulars Unit (Kg) Amount (Rs.)
To materials a/c 15,000 1,800,000 By Finishing Process a/c 13,500 2,019,215
To labour & overheads a/c 414,000 By closing stock (WIP) a/c 1,500 194,785
15,000 2,214,000 15,000 2,214,000

Finishing Process A/c


Particulars Unit (Kg) Amount (Rs.) Particulars Unit (Kg) Amount (Rs.)
To opening stock (WIP) a/c 900 141,000 By Finished stock a/c 13,800 2,340,473
To Fabrication process a/c 13,500 2,019,215 By closing stock (WIP) a/c 600 92,742
To labour & overheads a/c 273,000
14,400 2,433,215 14,400 2,433,215

c)
Profitability Statement of Normal Product
Particulars Amount (Rs.)
Sales (13800 kgs @ Rs.200 per kg.) 2,760,000
Less: cost of production (from finishing process) 2,340,473
Profit for the month 419,527

d)
Statement of evaluation of management's plan of further treatment of Normal Product
Particulars Amount (Rs.) Amount (Rs.)
Sales after further treatment (13800 kgs @ Rs.235/kg) 3,243,000
Less: Cost of Production
Cost of Normal Product from Finishing Process 2,340,473
Further treatment cost incurred 345,000 2,685,473
Profit per month 557,527
Less: Profit per month without further treatment 419,527
Additional profit per month due to further treatment 138,000

Return on Investment (%) 20.70% per annum


(138000 x 12 x 100/ 8,000,000)

Recommendation:
Considering the management's desired rate of return on investment of 25%,
the plan of further treatment is not acceptable becasue it gives only 20.70%
of return on investment.

2.
a) Cello Pen Company established a separate ball pen unit in year 2010 with
normal capacity of 240,000 units per year. The company is selling ball
pen for Rs. 35 per unit. Following data and information are available for
the year 2011:
Opening stock 52,000 unit
ZYA P.T.O.
(47)

Unit produced 234,000 unit


Units sold 240,000 unit
Direct material cost Rs.6 per unit
Direct Labour cost Rs.6 per unit
Variable manufacturing overhead Rs.3 per unit
Fixed manufacturing overhead Rs.1,200,000
Variable Administrative overhead Rs. 0.80 per unit
Fixed Administrative overhead Rs.120,000
Variable selling and administrative overhead Rs.2 per unit
Fixed selling and administrative overhead Rs.240,000
Required: (8+2+3+2=15)
i) Prepare profitability statement under marginal costing and absorption
costing method.
ii) Prepare reconciliation statement of profitability under marginal
costing and absorption costing method.
iii) Suggest the level of sales volume and sales unit where company will
make neither profit nor loss from the ball pen unit.
iv) How many units are to be sold to earn a profit of 25% on cost?

b) One of your friends established a vehicle-repairing workshop and was


worried about the increasing cost of operation and decreasing margin. He
was from engineering background so have little knowledge of
accounting. He wishes to appoint a cost auditor for review of his existing
system but he is confused about the purpose of cost audit. Describe the
purpose of cost audit and suggest your friend. 5
Answer No. 2
a)
i) Statement of Profitability under Marginal costing
Particulars Rs.
A. Total Sales (240,000 units × Rs. 35 p. u.) 84,00,000

Less: Variable Cost


Direct Material(234,000 units × Rs. 6 p. u.) 14,04,000
Direct Labour(234,000 units × Rs. 6 p. u.) 14,04,000
Variable manufacturing cost (234,000 units × Rs. 3 p. u.) 702,000
Add : Opening Stock ( 52,000 × Rs. 15) 7,80,000
Less: Closing Stock (46,000× Rs. 15) (6,90,000)
Variable cost of goods sold 36,00,000
Add : Variable Administrative Overhead ( 0.8×2,40,000) 1,92,000
Variable selling and distribution overhead (2×240000) 4,80,000
B. Total Variable Cost 42,72,000
C. Contribution 41,28,000
D. Less: Fixed Cost:
Manufacturing (12,00,000)
Administrative (1,20,000)
Selling and Distribution (2,40,000)
Profit under marginal costing 25,68,000

Statement of Profitability under absorption costing

ZYA P.T.O.
(48)

Particulars Rs.
A.Total Sales 84,00,000

Less: Manufacturing cost of Goods Sold


Direct Material 14,04,000
Direct Labour 14,04,000
Variable manufacturing cost 702,000
Fixed Production Overhead (12,00,000/2,40,000×234,000) 11,70,000
B. Manufacturing cost of Goods Produced (46,80,000)
Add : Opening Stock ( 52,000 units × Rs. 20) (W.N. 1) 10,40,000
Less: Closing Stock ( 46,000 × Rs. 20) (9,20,000)
Adjusted cost of Goods Sold 48,00,000
Add: Under-absorption of overhead ( 5×6,000) 30,000
a. Cost of goods Sold 48,30,000
D. Gross Profit (A-C) 35,70,000
Less: Variable Administrative Overhead ( 0.8×2,40,000) (192,000)
Fixed Administrative Overhead (120,000)
Variable selling and distribution overhead (2×240000) (4,80,000)
Fixed Selling and distribution overhead (2,40,000)
Profit under absorption costing 25,38,000
W.N. 1:
Total Manufacturing cost of goods produced Rs. 46,80,000
No. of units manufactured 234,000
Cost per unit Rs. 20

ii) Reconciliation of profit under Marginal costing and absorption costing


Particulars Rs.
Profit under marginal costing 25,68,000

Add: Fixed Manufacturing overhead in closing stock ( 46,000×5) 230,000


Less: Fixed Manufacturing overhead in Opening stock ( 52,000×5) (2,60,000)

Profit under absorption costing 25,38,000

iii) Calculation of break-even-point


P/V ratio = contribution/sales ×100
= 41,28,000/84,00,000×100 =49.142%

Contribution per unit = Sales price per unit – Variable cost per unit
=Rs.35-Rs.17.80
= Rs.17.20
Break-even point =Fixed cost/ P/V ratio
= 15,60,000/49.142%
= Rs.31,74,474
Break-even-point ( units) = Fixed cost/contribution per unit
=15,60,000/17.20
= 90,698 unit

ZYA P.T.O.
(49)

iv) Sales unit to earn 25% profit on cost; i.e. 20% on sales
Sales = (Fixed cost + Desired Profit)/ P/V Ratio
Let sales be X
Than X = (15,60,000 +20% of X)/49.142%
Or, 49.142% X = 15,60,000 +0.2X
X = Rs.1,560,000/0.29142
= Rs. 5,353,099

Sales in unit = 5,353,099/35 = 152,946 units

b) The purpose of cost audit is to examine whether the methods used for
ascertaining cost and other decisions are being properly implemented and
whether cost accounting plan has been adhered to or not.

Mainly we could classify purpose of cost auditing as; i) Protective and ii)
constructive

Protective purpose of cost audit: Cost audit aims at examining that there is no
undue wastage or loss and costing system brings out the correct and realistic cost
of production or processing.

Constructive purpose of cost audit: cost audit can provide information useful in
regulating production; choosing economical methods of operation, reducing
operating cost and reformulating plans etc. on the basis of findings during the
course of cost audit.

So I will suggest my friend to hire cost auditor having knowledge of vehicle


repairing workshop, who can help him using protective and constructive module
of cost audit

3.
a) A company uses an old method of machining a part manufactured for
sale. The estimate of operating details for the year 2011 – 12 are as under:
Number of parts to be manufactured and sold 30,000
Raw materials required per part 10 kg @ Rs. 2 per kg
Average wage rate per worker Rs. 40 per day of 8 hours
Average labour efficiency 60%
Standard time required for manufacturing one part 2 hours
Overhead rate Rs. 10 per clock hour
Material handling expenses 2% of the value of raw materials
The company has a suggestion box scheme and an award equivalent to
three months saving in labour cost is passed on to the employee whose
suggestion is accepted. In response to this scheme suggestion has been
received from an employee to use a special Jig in the manufacture of
aforesaid part. The cost of Jig which has life of one year is Rs. 31,500
and the use of the Jig will reduce the standard time by 20 minutes and
labour efficiency will increase by 20%.
Required: (5+5=10)
i) Compute the amount of award payable to the employee who has
given the suggestion.

ZYA P.T.O.
(50)

ii) Prepare a statement showing the annual cost of production before and
after the implementation of the suggestion to use the Jig and indicate
the annual savings.
b) Prepare a budget for year 2012 for direct labour costs and overhead
expenses of a production department at the activity levels of 80%, 90%
and 100%, using the information listed below: 10
i) Direct labour hourly rate is expected to be Rs.3.75
ii) 100% activity represents 60,000 direct labour hours.
iii) Variable cost:
Indirect labour Rs.0.75 per direct labour hour
Consumable supplies Rs.0.375 per direct labour hour
Canteen and other expenses 6% of direct and indirect labour cost
iv) Semi-variable costs are expected to relate to the direct labour hours in
the same manner as for the last five years
Years Direct labour hrs. Semi variable costs (Rs.)
2007 64,000 20,800
2008 59,000 19,800
2009 53,000 18,600
2010 49,000 17,800
2011 40,000 16,000
v) Fixed overhead per labour hour at 100% activity are:
Overheads Rs.
Depreciation 0.30
Maintenance 0.20
Insurance 0.10
Fee and Taxes 0.25
Management salaries 0.40
vi) Inflation is to be ignored.
Answer No. 3
a) (i) Computation of amount of award payable to employee:
Total labour cost under existing condition:
Standard time required for one part 2 hours or 120 minutes
Average efficiency 60%
Direct labour hours required for one part 120 × 100 / 60 = 200 minutes
Time required for 30,000 components 30,000 × 200 / 60 = 100,000 hours
Labour cost @ Rs. 5 per hour [A] Rs. 500,000
Total labour cost under suggested condition:
Standard time required for one part (120 – 20) = 100 minutes
Average efficiency of labour 80%
Direct labour hours required for one part 100 × 100 / 80 = 125 minutes
Time required for 30,000 components 30,000 × 125 / 60 = 62,500 hours
Labour cost @ Rs. 5 per hour [B] Rs. 312,500
Labour cost saved in a year [A] – [B] Rs. 187,500
Award equivalent to 3 months cost saving Rs. 187,500×(3/12) = 46,875
ZYA P.T.O.
(51)

(ii) Annual cost of production to the company under both the condition and savings made:
Before After
Particulars
Rs. Rs.
Raw materials [30,000 × 10 × Rs. 2] 600,000 600,000
Direct wages @ Rs. 5 per hour
For 100,000 hours 500,000 -
For 62,500 hours - 312,500
Overhead absorbed @ Rs. 10 per hour 1,000,000 625,000
Material handling charges @ 2% 12,000 12,000
Cost of Jigs - 31,500
Total costs 2,112,000 1,581,000
Saving in costs [Rs. 2,112,000 – Rs. 1,581,000] 531,000
Less: Award to employee 46,875
Net cost saving 484,125

b)
Flexible Budget for the year 2012
( Rs. in ‘000)
Particulars 80% level 90% level 100% level
48,000 hrs 54,000 hrs 60,000 hrs

Direct Labour 180.00 202.5 225.00


Other variable costs:
Indirect labour 36.00 40.50 45.00
Consumable supplies 18.00 20.25 22.50
Canteen etc. 12.96 14.58 16.20
Total Variable Cost 246.96 277.83 308.70
Semi Variable cost (W.N.1) 17.60 18.80 20.00
Fixed cost
Depreciation (60×0.3) 18.00 18.00 18.00
Maintenance (60×0.2) 12.00 12.00 12.00
Insurance (60×0.1) 6.00 6.00 6.00
Fee and Taxes (60×0.25) 15.00 15.00 15.00
Management salaries ( 60×0.4) 24.00 24.00 24.00
Budgeted cost 339.56 371.63 403.70

Working Note 1:

Segregation of semi variable cost using high/low method:


Rs.
Total cost of 64,000 hours 20,800
Total cost of 40,000 hours 16,000
Variable cost of 24,000 hours 4,800

ZYA P.T.O.
(52)

Variable cost per hour ( 4,800/24,000) 0.20

Total cost of 64,000 hours 20,800


Variable cost of 64,000 hours × 0.2 12,800
Fixed costs 8,000

Semi-variable costs are calculated as follows:


60,000 hours (60,000×0.20)+8,000 = 20,000
54,000 hours (54,000×0.20)+8,000 = 18,800
48,000 hours (48,000×0.20)+8,000 = 17,600

4.
a) A company prepared the following budgeted income statement for next
financial year:
Particulars Amount Amount
(Rs.) (Rs.)
Sales (52,000 units @ Rs.850 each) 44,200,000
Cost of goods sold:
Opening stock (2,000 units @ Rs.650 each) 1,300,000
Purchases (52,000 units @ Rs.700 each) 36,400,000
Closing stock (2,000 units @ Rs.700 each) (1,400,000) 36,300,000
Gross profit 7,900,000
Expenditures:
Purchasing cost- variable (@Rs.30,000 per order) 240,000
Purchasing cost- fixed 840,000
Transportation cost (@Rs.55,000 per order) 440,000
(charged by the supplier of goods)
Stock insurance cost (5,500 units @ Rs.40 each) 220,000
(based on average stockholding)
Fixed warehouse costs 2,300,000 4,040,000
Net profit before tax 3,860,000

At present, sales occur evenly throughout the year and a buffer stock of
2,000 units is maintained. Recently, the company has contracted with
supplier to buy 52,000 units of goods, the payment of which shall be
made in equal monthly installment throughout the year irrespective of the
order size. As per the contract, transportation costs are to be paid at the
beginning of the year.
The supplier has offered Rs.10,000 discount in transportation cost per
order if the company increase the order size from 6,500 units to a
minimum of 10,000 units per order.
The cost of capital of the company is 20% per annum.
Required: (5+7=12)
i) Assuming that the buffer stock level of 2,000 units is maintained;
calculate the optimal order size for the company.
ii) Show the improvement in net profit before tax with the
implementation of the optimal order size.

ZYA P.T.O.
(53)

b) From the following information relating to a hotel, calculate the room


rent to be charged to give a profit of 25% on cost excluding interest. 8
i) Salary to staffs – Rs. 1,022,000 per annum.
ii) Wages of the room attendant – Rs. 40 per day. There is a room
attendant for each room. He is paid wages only when the room is
occupied.
iii) Lighting and Heating power per month:
(a) Lighting expenses for each room are Rs. 1,000 when occupied.
(b) Heating power is used only in winter and the charges are Rs. 400
for a room when occupied.
iv) Repairs to buildings – Rs. 100,000 per annum.
v) License – Rs. 48,000 per annum.
vi) Sundries – Rs. 66,000 per annum.
vii) Interior decoration and furnishing – Rs. 100,000 per annum.
viii) Depreciation @ 5% is to be charged on building costing Rs.
4,000,000 and @ 10% on equipment.
ix) Interest to be charged @ 20% on investment on buildings and
equipment amounting to Rs. 5,000,000.
x) There are 100 rooms in the hotel. 80% of the rooms are generally
occupied in summer and 30% in winter. The period of summer and
winter may be considered to be of 6 months in each case and a
month may be assumed of 30 days.
Answer No. 4
a)
i) Calculation of Optimal Order Size
To calculate optimal order size, we have to find out EOQ
EOQ =√(2 x A x O/ C)
Where,
EOQ= Economic Order Quantity
A = Annual Usage
O = Ordering Cost
C = Carrying Cost

ZYA P.T.O.
(54)

Now,
EOQ = √ (2 x 52,000 x 85,000 / 40) = 14,866 units

At this EOQ level, the company will be eligible to get concession of Rs.10,000 on
transportation cost from supplier. Therefore, the ordering cost should be taken as
Rs. 75,000 for calculating EOQ. Hence, new EOQ is

EOQ = √ (2 x 52,000 x 75,000 / 40) = 13,964 units

The buffer stock level of 2,000 units are covered in above 13,964 units of order size,
therefore, the optimal order size for the company is 13,964 units per order.

ii)

ZYA P.T.O.
(55)

Statement showing improvement in net profit before tax


with implementation of optimal order size
Particulars Amount (Rs.) Amount (Rs.)
Sales (52,000 units @ Rs.850 each) 44,200,000
Less Cost of goods sold:
Opening stock (2,000 units @ Rs.650 each) 1,300,000
Purchases (52,000 units @ Rs.700 each) 36,400,000
Closing stock (2,000 units @ Rs.700 each) (1,400,000) 36,300,000
Gross profit 7,900,000
Less Expenditures:
Purchasing cost- variable (WN 1) 120,000
Purchasing cost- fixed 840,000
Transportation cost (WN 2) 180,000
Stock insurance cost (WN 3) 359,280
Fixed warehouse costs 2,300,000 3,799,280
Revised net profit before tax 4,100,720
Original net profit before tax 3,860,000
Improvement in net profit before tax 240,720

WN 1) Calculation of variable purchasing cost


Annual requirement (units) 52,000
Order size (units) 13,964
No. of orders (Annual requirement/ order size) 4
Variable purchasing cost per order (Rs.) 30,000
Total variable purchasing cost (Rs.) 120,000

WN 2) Calculation of transportation cost


No. of orders 4
Transportation cost per order (Rs.) 45,000
Total transportation cost (Rs.) 180,000

WN 3) Calculation of stock insurance cost


Buffer stock level (units) 2,000
Average stock (13964/2) (units) 6,982
Total average stockholding 8,982
Insurance cost per unit (Rs.) 40
Total stock insurance cost (Rs.) 359,280
b)

Operating Cost Statement showing Room Rent per Day


(Room days 19,800)
Particulars Rs. Rs.
Staff salaries 1,022,000
Room attendant's wages [Working note 2] 792,000
Lighting and heating power [Working note 3] 732,000
Repairs to buildings 100,000
Licence etc. 48,000
Sundries 66,000
Interior decoration and furnishing 100,000
ZYA P.T.O.
(56)

Depreciation on:
Buildings (5% of Rs. 4,000,000) 200,000
Equipments [10% of Rs. (5,000,000 – 4,000,000)] 100,000 300,000
Total cost before charging interest 3,160,000
Interest on investment @ 20% 1,000,000
Total cost 4,160,000
Profit @ 25% on Rs. 3,160,000 790,000
Total rent to be charged for all rooms 4,950,000
Room – days [Working note 1] 19,800
Room rent per day 250

Working notes:
1. Calculation of Room Days:
Summer: (100 × 80 / 100 × 6 × 30) 14,400
Winter: (100 × 30 / 100 × 6 × 30) 5,400
Total 19,800
2. Calculation of Room Attendant's Wages:
Rs.
Summer: (Rs. 40 × 100 × 80 / 100 × 6 × 30) 576,000
Winter: (Rs. 40 × 100 × 30 / 100 × 6 × 30) 216,000
Total 792,000
3. Calculation of charges of Lighting and Heating Power:
Rs.
Lighting: Summer (Rs. 1,000 × 100 × 80 / 100 × 6) 480,000
Winter (Rs. 1,000 × 100 × 30 / 100 × 6) 180,000
Heating Power: Winter (Rs. 400 × 100 × 30 / 100 × 6) 72,000
Total 732,000
5. Distinguish between: (4×2.5 =10)
a) Integrated accounting system and Uniform costing
b) Bin cards and Store ledger
c) Job evaluation and Merit rating
d) Absorption costing and Variable costing
Answer No. 5
a) Integrated accounting system is the one which contains both financial and cost
accounts in a single book-keeping system. Thus, it discards the concept of
requirement of separate profit and loss account for financial and costing
purpose. For implementation of integrated accounting system, factors such as
degree of integration of records, preparation of control accounts for various
elements of costs, details of cost data to be provided to cost accounting
department, and need for creation of suspense accounts should be considered.
Uniform costing is defined as the use by several undertakings of the same
costing principles and/ or practices. It is not a separate method of cost
accounting; it only points to a situation where a number of business firms are

ZYA P.T.O.
(57)

applying similar costing principles and practices. The designing and


applications of uniform costing require that a uniform cost manual containing
instructions, clarifications, rules and guidelines about cost determination, cost
analysis and cost control, should be developed and circulated among the
undertakings deciding to use uniform costing.

b)
Bin cards Store ledger
Bin Cards maintained by Store Keeper Store ledger is maintained by cost accounting
department
It is the store recording documents It is an accounting record
It contains information as records to quantities; It contains both quantitative and value
receipt, issue and balance information of receipt, issue and balance
Entries in bin cards are made when transaction Entries are made only after the transaction has
take place; i.e. entities made and movement of taken place
goods are made.
Bin Card records each transaction Store ledger records the same information in a
summarized form
Inter Departmental transfer of materials does Inter Departmental transfer of materials appear
not appear here.

c) The term job evaluation may be defined as the process of analysis and
assessment of jobs to ascertain reliably their relative worth and to provide
management with a reasonably sound basis for determining the basic internal
wage and salary structure for the various job positions. In other words, job
evaluation provides a rationale for differential wages and salaries for different
group of employees and ensures that these differentials are consistent and
equitable.
On the other hand merit rating is a systematic evaluation of the personality and
performance of each employee by his supervisor or some other qualified person.
The main distinctions between the two are as follows:
1. Job evaluation is the assessment of the relative worth of jobs within a
company and merit rating is the assessment of the relative worth of the man
behind a job. In other words, merit rating rates employees on their jobs while
job evaluation rates the jobs.
2. Job evaluation and its accomplishment are means to set up a rational wage
and salary structure whereas merit rating provides a scientific basis for
determining fair wages for each worker based on his ability and
performance.
3. Job evaluation simplifies wage administration by bringing uniformity in
wage rates. On the other hand merit rating is used to determine fair rate of
pay for different workers on the basis of their performance.

d) Absorption costing treats the costs of all manufacturing components (direct


material, direct labour, variable overhead and fixed overhead) as inventoriable

ZYA P.T.O.
(58)

or product costs. Costs incurred in the non-manufacturing areas are considered


as period costs.
In contrast, variable costing is a cost accumulation method that includes only
variable production costs (direct material, direct labour, and variable
overhead) as inventoriable or product costs. Fixed manufacturing overhead is
treated as period costs.
Two basic differences can be seen between absorption costing and variable
costing. The first difference is the way fixed overhead (FOH) is treated for
product costing purposes. Under absorption costing, FOH is considered a
product cost because it advocates that products cannot be made without the
capacity provided by fixed manufacturing costs; under variable costing, it is
considered a period cost because it advocates that fixed manufacturing costs
would be incurred whether or not production occurs. The second difference is
in the presentation of costs on the income statement. Absorption costing
classifies expenses by function, whereas variable costing categorizes expenses
first by behavior and then may further classify them by fuction.

6.
a) You joined a manufacturing company with very good profitability record.
Staff and owners were happy with the profit margin. Do you think there
may be still some room for cost reduction to increase profit margin?
Justify your answer. 2.5
b) Write short notes on the following: (3×2.5=7.5)
i) Pareto Inventory Analysis
ii) Capacity Cost
iii) Control Ratios
Answer No. 6
a) Even though industry is making good profit margin and running at good
profit; there should be cost reduction attainable in almost areas of business
activities. There is perhaps no situation which cannot be improved for cost
reduction. We can review new layout, product design, production methods,
material used, labour efficiency, machines used, office layout, innovation in
marketing, packaging, warehousing, handling, purchasing, use of
administrative facilities and even utilization of financial resources.

Cost may be additionally incurred because of:


i. Lack of information about raw materials, processes, products and
components etc.
ii. Lack of utilization of ideas generated from performance and economic
analysis
iii. Hones but wrong beliefs that certain things are impossible for
achievement
iv. Temporary circumstances like features developed under pressure or
modifications made to meet certain circumstances.
v. Habits and attitudes of confining to one conventional method

ZYA P.T.O.
(59)

Considering all factors we can review all of the cost factors and analyze the
probability of cost reduction in all areas at a same time or one by one considering
the size of operation and cost involved.

After making such detailed study, cost reduction areas could be identify in one or
more areas of operation.

b)
i) Unit cost commonly affects the degree of control that should be maintained
over an inventory item. As unit cost increases, internal controls (such as
inventory access) are typically tightened and perpetual inventory system is
more often used. Recognition of cost-benefit relationships may result in a
Pareto Inventory Analysis, which separates inventory into three groups based
on annual cost-to-volume usage.
Items having the highest value are referred to as A items; C items represent
the lowest value; and all other items are categorized as B items. Once this
categorization is done, management can determine the best inventory control
method for items in each category.

ii) Capacity cost is an alternative term used for fixed cost. It represents cost of
providing facilities of a system for a particular period. Capacity cost can be
classified further into standby cost and enabling cost. Standby cost is that
cost which continues to be incurred even if operations or facilities are
shutdown temporarily. Examples are depreciation, property taxes,
management salaries etc. Enabling cost is the cost which can be avoided by a
temporary shutdown, but it must be incurred if production is resumed.

iii) Control Ratios


Control ratios are the tools used to monitor and control performance of the
organization. This is determined by comparing the planned values (budgets) with
the actual values as they occur/achieved during a period. Under budgetary
control system certain ratios are used to determine the effective use of the
resources. Such ratios are used by the management to know whether the
deviations of the actual performance from the budgeted performance are
favourable or not. There are three major control ratios, activity ratio, capacity
ratio and efficiency ratio.

1. Activity Ratio

Activity Ratio: It is a measure of the level of activity attained over a period of


time. It is obtained by expressing the number of standard hours equivalent to the
work produced as a percentage of the budgeted hours. Higher the ratio better is
the performance. Mathematically it is expressed as follows:

Activity Ratio = Standard hours of actual production x 100


Budgeted hours

Level of activity is arrived by comparing the actual production with the


anticipated production as shown in the budget.

ZYA P.T.O.
(60)

Calendar Ratio: Actual activity gets affected if the budgeted number of days
could not be worked. Hence, calendar ratio is calculated to control the number of
days actually available for work. This ratio indicates whether all budgeted
working days have been actually available for working in the budgeted period.

Calendar Ratio = Actual working days x 100


Budgeted working days
2. Capacity Ratio

Capacity Usage Ratio: It indicates the relationship between the budgeted


number of working hours and the maximum possible number of working hours
in a budgeted period.

Capacity Usage Ratio = Budgeted number of working hours x 100


Maximum number of working hours

Standard Capacity Employed Ratio: It indicates the extent to which facilities


were actually utilized during the budgeted period. Higher ratio indicate that
higher capacity were used in actual compare to budget.

Standard Capacity Employed Ratio = Actual hours worked x 100


Budgeted hours
3. Efficiency Ratio: This indicates the efficiency attained in production during the
budgeted period. It is calculated as follows:

Efficiency Ratio = Standard hours of actual production x 100


Actual hours worked

ZYA P.T.O.
HYI
CAP-II, Financial Management, June 2012
Suggested Answer

Roll No……………. Maximum Marks - 100


Total No. of Questions – 7 Total No. of Printed Pages – 3
Time Allowed – 3 Hours
Marks
Attempt all questions.

Working notes should form part of the answer. Make assumptions wherever necessary.

1. D Co. needs to increase production capacity to meet increasing demand for an


existing product, ‗Q‘, which is used in food processing. A new machine, with a
useful life of four years and a maximum output of 600,000 kgs. of Q per year,
could be bought for Rs. 800,000, payable immediately. The scrap value of the
machine after four years would be Rs. 30,000. Forecast demand of Q over the
next four years is as follows:

Year 1 2 3 4
Demand (kg.) 1.4 million 1.5 million 1.6 million 1.7 million

Existing production capacity for Q is limited to one million kilograms per year,
and the new machine would only be used for catering demand additional to this.
The current selling price of Q is Rs. 8.00 per kilogram and the variable cost of
materials is Rs. 5.00 per kilogram. Other variable costs of production are Rs. 1.90
per kilogram. Fixed costs of production associated with the new machine would
be Rs. 240,000 in the first year of production, increasing by Rs. 20,000 per year in
each subsequent year of operation. D Co. pays tax one year in arrears at an annual
rate of 30% and can claim capital allowances (tax-allowable depreciation) on a
25% reducing balance basis. The balancing allowance is claimed in the final year
of operation of new machine.
D Co. uses its after-tax weighted average cost of capital when appraising
investment projects. It has a cost of equity of 11% and a before-tax cost of debt of
8.6%. The long-term finance of the company, on a market-value basis, consists of
80% equity and 20% debt.

Required:
a) Calculate the net present value of buying the new machine and advise on the
acceptability of the proposed purchase. 12
b) Calculate the internal rate of return of buying the new machine and advise on
the acceptability of the proposed purchase. 4
c) What are the limitations of the investment appraisal made in (a) and (b)
above? Explain how they can be addressed. 4

Answer No. 1

ZYA P.T.O.
(62)

a) Net present value evaluation of investment

After-tax weighted average cost of capital (%) = (11 x 0·8) + (8·6 x (1 – 0·3) x 0·2)
= 8.8 + 1.20 =10% (approx.)
(Figure in Rs.)
Year 1 2 3 4 5

Contribution 440,000 550,000 660,000 660,000 -


Fixed costs (240,000) (260,000) (280,000) (300,000) -
––––– ––––– ––––– –––––
Taxable cash flow 200,000 290,000 380,000 360,000 -
Taxation@30% - (60,000) (87,000) (114,000) (108,000)
CA tax benefits - 60,000 45,000 33,750 92,250
Scrap value 30,000
––––– ––––– ––––– –––––
After-tax cash flows 200,000 290,000 338,000 309,750 (15,750)
DF at 10% 0·909 0·826 0·751 0·683 0·621
––––– ––––– ––––– ––––– –––––
Present values 181,800 239,540 253,838 211,559 (9,781)
––––– ––––– ––––– ––––– –––––
Rs.
Present value of benefits 876,956
Initial investment (800,000)

Net present value 76,956

Advice:
The net present value is positive and so the investment is financially acceptable.

Working Note 1: Calculation of Annual contribution


Year 1 2 3 4
Excess demand (kg/yr) 400,000 500,000 600,000 700,000
New machine output (kg/yr) 400,000 500,000 600,000 600,000*
Contribution (Rs./kg)
Rs. (8 – 5 – 1.9) 1·1 1·1 1·1 1·1
–––––––– –––––––– –––––––– ––––––––
Contribution (Rs./yr) 440,000 550,000 660,000 660,000
–––––––– –––––––– –––––––– ––––––––
*Maximum output from new machine.
Working Note 2: Capital allowance (CA) tax benefits

Year Capital allowance (Rs.) Tax benefit (Rs.)


1 200,000 (800,000 x 0·25) 60,000 (0·3 x 200,000)
2 150,000 (600,000 x 0·25) 45,000 (0·3 x 150,000)
3 112,500 (450,000 x 0·25) 33,750 (0·3 x 112,500)
––––––––
462,500
30,000 (scrap value)
––––––––
492,500
4 307,500 (by difference) 92,250 (0·3 x 307,500)
––––––––
ZYA P.T.O.
(63)

800,000
––––––––

b) Internal rate of return evaluation of investment


(Using trial discount factor at 20%)

Year 1 2 3 4 5
Rs. Rs. Rs. Rs. Rs.
After-tax cash flows
(as in (a) above 200,000 290,000 338,000 309,750 (15,750)
Discount factor
at 20% 0·833 0·694 0·579 0·482 0·402
––––– ––––– ––––– ––––– –––––
Present values 166,600 201,260 195,702 149,300 (6,332)
––––– ––––– ––––– ––––– –––––

Rs.
Present value of benefits 706,530
Initial investment (800,000)

Net present value (93,470)

Internal rate of return = 10 + [((20 – 10) x 76,956)/(76,956 + 93,470)] = 10 + 4·52= 14·52%

Advice:
The investment is financially acceptable since the internal rate of return is greater than the
cost of capital used for investment appraisal purposes.

c) The limitations of the above investment appraisal are:


i) Maximum output constraint of the proposed investment, which is insufficient
to address the increasing demand in year four and onwards.
ii) Uncertainty about the required additional investment to address the additional
demand in year 4 and onwards.
iii) Constant selling price and variable cost but increasing fixed cost and demand.
They can be addressed by:
i) Assumption of constant selling price and variable cost of the product.
ii) Assumption of linear increase in fixed production cost and demand of the
product.
iii) Including necessary cost of additional investment required in year 4 and
onward.

2.
a) Phel Typewriter Ltd. and Gillis Typewriter Ltd. are identical in all respect
except for capital structure. Phel has 50 percent debt and 50 percent equity
financing whereas Gillis has 20 percent debt and 80 percent equity financing,
in market value terms. The borrowing rate for both companies is 13 percent in
a no-tax world and capital markets are assumed to be perfect. The earnings of
both companies are not expected to grow, and all earnings are paid out to
shareholders in the form of dividends.
ZYA P.T.O.
(64)

Required:

i) If you own 2 percent of common stock of Phel, what would be your rupee
return if the company has net operating income of Rs. 360,000 and the
overall capitalization rate of the company is 18 percent? What is the
implied equity capitalization rate? 4
ii) Gillis has same net operating income and overall capitalization rate as
Phel. What is the implied equity capitalization rate for Gillis? Why does it
differ from Phel? 4

b) Assume that RCT Limited has owner's equity of Rs. 100,000. The ratios for
the firm are as follows: 7
Current Debt to total debt 0.40
Total Debt to owner's equity 0.60
Fixed Assets to owner's equity 0.60
Total assets turnover 2 times
Inventory turnover 8 times

Required:
Complete the following balance sheet:
Liabilities and Capital Amount Assets Amount
(Rs.) (Rs.)
Current Debt ........... Cash ...........
Long Term Debt ........... Inventory ...........
Total Debt ............ Total Current Assets ...........
Owners Equity ........... Fixed Assets ...........
Total Capital and Liabilities .......... Total Assets ...........

Answer No. 2
a)
i) Phel Typewriter Ltd.
Net Operating Income(Rs.) 360,000
Overall Capitalization rate 0.18
Total Value of the firm (Rs. 360,000/0.18) 2,000,000
Market Value of debt (50%) (Rs.) 1,000,000
Market Value of stock (50%) (Rs.) 1,000,000
Interest on debts (13% of Rs. 1,000,000) 130,000
Earnings available to common shareholders 230,000
(NOI – Interest)
Common Stock holding = 2%
Therefore,
Rupee Return = 2% of earning available to shareholders
= 2% × 230,000
= Rs. 4,600.
Implied equity capitalization rate = Earnings available to common shareholders/ Market value of
stock
= 230,000/1,000,000
= 23%

ZYA P.T.O.
(65)

ii) Gillis Typewriter Ltd.


Net Operating Income (Rs.) 360,000
Overall Capitalization rate 0.18
Total Value of the firm(Rs. 360,000/0.18) 2,000,000
Market Value of debt (20%) (Rs.) 400,000
Market Value of stock (80%) (Rs.) 1,600,000
Interest on debts (13% of Rs. 400,000) 52,000
Earnings available to common shareholders(Rs.) 308,000
(NOI – Interest)

Implied equity capitalization rate = Earnings available to common shareholders/ Market value of
stock
= 308,000/1,600,000
= 19.25%

Gillis has a lower capitalization rate than Phel because Gillis uses less debt in its capital
structure. As the equity capitalization rate is linear function of the debt-to-equity ratio when
we use the net operating income approach, the decline in the equity capitalization rate exactly
offsets the disadvantage of not employing so much in the way of cheaper debt funds.

b)
i. Total Debt = 0.60 X Owner's equity = 0.60X100,000 = Rs. 60,000
ii. Fixed Assets = 0.60XOwner's equity = 0.60X100,000= Rs. 60,000
iii. Total Capital = Total Debt + Owner's Equity = Rs. 60,000 + 100,000= Rs. 160,000
iv. Total assets consisting of current assets and fixed assets must be equal to Rs. 160,000 (Assets=
Liabilities+ Owner's equity). Fixed Assets are Rs. 60,000, therefore, current assets should be Rs.
160,000 - 60,000 = Rs. 100,000
v. Sales are found as follows:
Assets turnover=Sales/Assets= 2 = Sales/Rs.160,000=2
Thus, Sales= 160,000X2= Rs. 320,000
vi. Inventories are found as follows:
Inventory turnover = Sales/Inventories=Rs. 320,000/Inventories=8
Thus, Inventories=320,000/8 = Rs. 40,000
vii. Cash = Current assets - Inventories = Rs. 100,000-40,000= Rs.60,000
viii. Current Debts =0.40XTotal debts= 0.40XRs.60,000= Rs. 24,000
ix. Long term debts = Total Debts-Current Debts = Rs.60,000-Rs. 24,000= Rs.36,000
x. With all the above information, the balance sheets of RCT Ltd. would be as under:

Liabilities and Capital Amount (Rs.) Assets Amount (Rs.)


Current Debt 24,000 Cash 60,000
Long term debt 36,000 Inventory 40,000
Total Debt 60,000 Total Current Assets 100,000
Owners Equity 100,000 Fixed Assets 60,000
Total Capital and Liabilities 160,000 Total Assets 160,000

3.
a) Following is the capital structure of ABC Ltd. on Ashadh end, 2066:
Rs.
Equity Share Capital (of Rs. 100 each) 1,000,000
Share Premium 1,500,000

ZYA P.T.O.
(66)

Reserves and Surplus 500,000


3,000,000
On 1 Shrawan 2066, the company made a bonus issue of 2 shares for every 5
shares held.
Mr. P had purchased 100 shares of ABC Ltd. on 1 Shrawan 2062 at the
market price of Rs. 300. He sold all the shares on Ashadh end, 2068 at the
market price of Rs. 450 per share (cum-dividend). He had to pay tax @ of 5%
on his dividend income and 10% on capital gains.

Required:
If the company pays a regular dividend @ 10%, find out whether the investor
P was able to earn his required rate of return of 10% on his investments during
the period, ignoring brokerage and other transaction related costs. (Present
value factors @ 10% for 1 – 6 years are: 0.91, 0.83, 0.75, 0.68, 0.62 and 0.56
respectively). 8
b) Kathmandu Wool House is a manufacturer and exporter of woolen garments
to most of the European countries and to United States of America. The
business of the company is expanding day by day and in the previous financial
year, the company has registered a 25% growth in export business.
The company is in the process of considering a new investment project. It is
an all equity financed company with 500,000 equity shares of face value of
Rs. 100 per share. The current market price of this share is Rs. 250 ex-
dividend. Annual earnings are Rs. 50 per share, and in the absence of new
investment, this will remain constant in perpetuity. All earnings are distributed
at present. A new investment is available which will cost Rs. 17,500,000 in
one year‘s time and will produce annual cash inflows thereafter of Rs.
5,000,000.
Required:
Analyze the effect of the new project on dividend payments and the share
price of the company. 7
Answer No. 3
a) Year Mr. P‘s holding in ABC Ltd. Cost(Rs.) PV
(Rs.)
2062-63 to 2065-66 100 shares@ Rs. 300 30,000
10,000
2066-67 to 2067-68 100 shares + 100×(2/5)=140 shares 30,000
14,000

Calculation of Present Value of Dividend Income


(Figures in Rs.)
Date Dividend @ 10% Dividend after Tax PV Factor PV
Ashadh end 2063 1,000 950 0.91 864.50
Ashadh end 2064 1,000 950 0.83 788.50
Ashadh end 2065 1,000 950 0.75 712.50
Ashadh end 2066 1,000 950 0.68 646.00

ZYA P.T.O.
(67)

Ashadh end 2067 1,400 1,330 0.62 824.60


3,836.10

Calculation of Present Value of Sale Proceeds: Rs.


Selling Price of 140 shares @ Rs. 450 per share on Ashadh end 2068: 63,000
Less: Total Cost of Purchase of Shares (100 X Rs. 300) : 30,000
Capital Gain : 33,000
Capital Gain Tax @ 10% : 3,300
Net Gain to the Investor : 29,700
Total Cash Inflow (29,700 + 30,000) : 59,700

This amount is received on Ashadh end 2068, i.e. after 6 years from the date of purchase.
Present Value Factor @ 10% for 6 years = 0.56
Present value of Rs. 59,700 (59,700 X 0.56) = Rs. 33,432.00
Total Present Value of Inflows (33,432 + 3,836.10) = Rs. 37,268.10
Less: Cash Outflow (Cost of Purchase) = (Rs. 30,000.00)
Net Present Value = Rs. 7,268.10
Since the net present value (gain) of the Mr. P in shares of ABC Ltd. is positive, discounting
at the rate of 10%, the investor P is getting a return on his investment at a rate in excess of
10%.

b) Current Market Price of share(P) = Rs. 250


Annual Earning (D) = Rs. 50
Cost of Equity (In case of perpetuity) (Ke) = ×100

Therefore, Cost of Equity (ke)


Ke = D / P X 100 = 50/250 X 100 = 20%
Earnings per share = Rs. 50
Total earnings = 500,000 X Rs. 50 = Rs. 25,000,000
New Project Cost = Rs. 17,500,000
Since the project is financed out of internal accruals (equity financed company), the amount
available for dividend at the end of 1st year is = 1st year earnings – project cost = Rs.
25,000,000 – Rs. 17,500,000 = Rs. 7,500,000
Dividend per share in 1st year:
= Rs. 7,500,000 /500,000 shares = Rs. 15 per share
Dividend per share in 2nd year, which will remain constant in perpetuity.
Rs. 25,000,000 + Rs. 5,000,000 = Rs. 60 per share
500,000 shares
Since all earnings are distributed, EPS and DPS will remain the same.
ZYA P.T.O.
(68)

The present value of new share price after the new project is taken up:
P = ( Rs. 15.00 ) + ( Rs. 60 X 1 ) = 12.50 + 250 = Rs. 262.50
1.20 0.20 1.20

It is seen that due to the investment in new project, dividend payments in 1st year will decrease but
from 2nd year onward it will increase and the share price of the company will increase.

4.
a) Prove that the Beta of the market portfolio is equal to 1. 4

b) "Nature of Business, Operating Leverage, and Financial Leverage are three


fundamental factors that determine Beta". Comment on this statement? 4

c) Consider the following information relating to two stocks, A and B :


Year Return on A (%) Return on B (%)
2009 10 12
2010 16 18

Required:
Determine: 7
i) The expected return on a portfolio, containing A and B in the proportion
of 40% and 60% respectively.
ii) The Standard deviation of return from each of the two stocks.
iii) The covariance of returns from the two stocks.
iv) Correlation coefficient between the returns of the two stocks.
v) The risk of a portfolio, containing A and B in the proportion of 40% and
60% respectively.
Answer No. 4
a) Beta is a relative measure of market risk. It is the ratio of the covariance of the rate of return to
the variance of the market portfolio return.

βj = Covjm / σm2
=1
It suggests that the stock is equally risky as market risk. Hence,

βj = βm
We know,
βj = Covjm / σm2
Substituting the values βj = βm,
βm = Covmm / σm2
= ρmm * σm * σm / σm2
= ρmm
=1

ZYA P.T.O.
(69)

The correlation of the rate of return on the market portfolio with itself must be positive and
perfect. Hence, it is proved that the Beta of the market portfolio is equal to 1.

b) The Beta is the ratio of covariance between returns on market and a security to variance
of the market returns. The variance and covariance and hence Beta depends on three
fundamental factors – the nature of business, the operating leverage and the financial
leverage.

Nature of Business – All economies go through business cycles. Firms behave differently
within a business cycle. The earnings of some companies fluctuate more with the
business cycles. Their earnings grow during the growth phase of the business cycle and
decline during the contraction phase. When we regress a company's earnings with the
aggregate earnings of all companies in the economy, we would obtain a sensitivity index,
which we can call the company's accounting beta. The real or the market beta is based on
share market returns rather than earnings. The accounting betas are significantly
correlated with the market betas. This implies that if a firm's earnings are more sensitive
to business conditions, it is likely to have higher beta.

Operating Leverage – It refers to the use of fixed costs. The degree of operating leverage
is defined as the change in a company's earnings before interest and tax, due to change in
sales. Since variable costs change in direct proportion of sales and fixed costs remain
constant, the variability in EBIT, when sales change is caused by fixed costs. Higher the
fixed cost, higher the variability in EBIT for a given change in sales. Other things
remaining the same, companies with higher operating leverage (because of higher fixed
costs) are more risky. Operating leverage intensifies the effect of cyclicality on a
company's earnings. As a consequence, companies with higher degrees of operating
leverage have high beta.

Financial Leverage – It refers to a debt in a firm's capital structure. Firms with debt in the
capital structure are called levered firms. The interest payments on debt are fixed
irrespective of the firm's earnings. Hence, interest charges are fixed costs of debt
financing. The fixed financial costs result in financial leverage and cause profit after tax
to vary with changes in EBIT. Since, financial leverage increases the firm's financial risk,
it will increase the equity beta of the firm.

c) (i) Expected return of the portfolio A and B, containing 40% A and 60% B
Expected Return {E (A)} = (10 + 16) / 2 = 13%
Expected Return {E (B)} = (12 + 18) / 2 = 15%
Expected Return on portfolio {Rp} = 0.4(13) + 0.6(15)
=5.2% + 9% = 14.2%

(ii) Standard deviation of return from each stock


Stock A:
Variance = 0.5 (10 – 13)² + 0.5 (16 – 13) ² = 9%
Standard deviation = √9 = 3%

Stock B:
Variance = 0.5 (12 – 15) ² + 0.5 (18 – 15) ² = 9%
Standard deviation = √9 = 3%

ZYA P.T.O.
(70)

(iii) Covariance of return from stocks A and B


CovAB = 0.5 (10 – 13) (12 – 15) + 0.5 (16 – 13) (18 – 15) = 9%

(iv) Correlation of coefficient between the returns of the two stocks.

rAB = CovAB/ σA σB = 9 / 3*3 = 1

(v) Portfolio Risk, which contains 40% A and 60% B.

σp =
X2Aσ2A + X2Bσ2B + 2XAXB(σA σBrAB)

=
(0.4)2 (3)2 + (0.6)2 (3)2 + 2 (0.4) (0.6) (3) (3) (1)

= 1.44 + 3.24 + 4.32

= 3%

5.
a) A company wishes to find out its weighted marginal cost of capital (WMCC)
based on target capital structure proportions. The company presented the
following data to you to assist the company in determining its WMCC.

Source Proportion Range Cost


Equity share capital 50% Up to Rs. 300,000 13.00%
Rs. 300,000 – Rs. 750,000 13.30%
Rs. 750,000 and above 15.50%
Preference shares 10% Up to Rs. 100,000 9.33%
Rs. 100,000 and above 10.60%
Long term debt 40% Up to Rs. 400,000 5.68%
Rs. 400,000 – Rs. 800,000 6.50%
Rs. 800,000 and above 7.10%

Required:
Determine the WMCC for the company. 8
a) The following details of XYZ Ltd. for the year ended on Ashadh end, 2068
are given below:
Operating leverage : 1.4
Combined leverage : 2.8
Fixed cost (excluding interest) : Rs. 204 thousand
Sales : Rs. 3,000 thousand
12% Debentures of Rs. 100 each : Rs. 2,125 thousand
ZYA P.T.O.
(71)

Equity shares capital of Rs. 100 each : Rs. 1,700 thousand


Income-tax rate : 30 per cent
Required:
Calculate the P/V ratio and Earnings per share (EPS). (2.5+1.5=4)
b) A 12-payment annuity of Rs. 10,000 will begin 8 years hence, i.e. the first
payment occurs at the end of 8 years. What is the present value of this
annuity, if the discount rate is 14 per cent? 3

Answer No. 5
a)
(i) Determination of Breaking point of different sources:

Source Proportion Range Breaking Points(Rs.)


Equity share capital 0.50 Up to Rs. 300,000 300,000/0.5= 600,000
300,000 – 750,000 750,000/0.5= 1,500,000
750,000 and above -
Preference shares 0.10 Up to Rs. 100,000 100,000/0.10 = 1,000,000
100,000 and above -
Long term debt 0.40 Up to Rs. 400,000 400,000/0.4 = 1,000,000
400,000 – 800,000 800,000/0.40 = 2,000,000
800,000 and above -

We now calculate below the WMCC for different ranges of new financing.

Range Source Proportion C/C % WMCC %


Up to Rs.600,000 Equity shares 0.50 13.00 6.50
Preference shares 0.10 9.33 0.93
Long term debt 0.40 5.68 2.27
WMCC 9.70
Rs. 600,000 – 1,000,000 Equity shares 0.50 13.30 6.65
Preference shares 0.10 9.33 0.93
Long term debt 0.40 5.68 2.27
WMCC 9.85
Rs. 1,000,000 – 1,500,000 Equity shares 0.50 13.30 6.65
Preference shares 0.10 10.60 1.06
Long term debt 0.40 6.50 2.60
WMCC 10.31
Rs. 1,500,000 – 2,000,000 Equity shares 0.50 15.50 7.75
Preference shares 0.10 10.60 1.06
Long term debt 0.40 6.50 2.60
WMCC 11.41

ZYA P.T.O.
(72)

Rs. 2,000,000 – and above Equity shares 0.50 15.50 7.75


Preference shares 0.10 10.60 1.06
Long term debt 0.40 7.10 2.84
WMCC 11.65
b)
(i) Calculation of P/V Ratio:
P/ V Ratio = Contribution / Sales X 100
Operating Leverage = C / (C – F) X 100
1.4 =C / C -204,000
1.4 (C – 204,000) = C
1.4 C – 285,600 = C
0.4 C = 285,600
Therefore, C = 285,000/0.4 = Rs. 714,000
P/V ratio = 714,000 /3,000,000 X 100 = 23.8%
(ii) Calculation of EPS:
EBT = Contribution – Fixed Cost – Interest = 714,000 – 204,000 – 255,000 = Rs.
255,000
(Interest =Rs. 2,125,000 × 12% =Rs. 255,000
EAT = EBT – Tax = 255,000 – 76,500 = Rs. 178,500
(Tax = Rs. 255,000 × 30% = Rs. 76,500)
EPS = EAT / No. of Equity Shares = 178,500/17,000 = Rs. 10.50.

c) In the first step, we determine the value of this annuity a year before the first payment
begins, i.e. 7 years from now. This is equal to:
Rs. 10,000 (PVIFA 14%, 12 years) = Rs. 10,000 (5.660) = Rs. 56,600

In the second step, the present value of the amount obtained in step 1 is found out as
follows:
Rs. 56,600 (PVIF 14%, 7 years) = Rs. 56,600 (0.400) = Rs. 22,640 (Approx.)

Alternatively,
Annuity amount (Rs.) = 10,000
No. of payment = 12
Discounting Rate = 14%
PVIFA at 14% for years 8-19 = 2.2621
Therefore,
PV = Rs.10,000 x 2.2640
= Rs.22,640 (Approx.)

ZYA P.T.O.
(73)

6. Write short notes on: (4×2.5=10)


a) Intrinsic value of an asset
b) Financial distress
c) Valuation of compulsorily convertible debenture
d) Line of credit with a bank
Answer No. 6
a) The intrinsic value of an asset is equal to the present value of incremental future cash
inflows likely to accrue due to the acquisition of the asset, discounted at the appropriate
required rate of return. It represents the maximum price the buyer would be willing to pay
for such an asset. The principal of valuation based on the discounted cash flow approach
is used in capital budgeting decisions. In the case of business intended to be purchased,
its valuation is equivalent to the present value of incremental future cash inflows after
taxes, likely to accrue to the acquiring firm, discounted at the relevant risk adjusted
discount rate, as applicable to the acquired business. The intrinsic value indicates the
maximum price at which the business can be acquired.

b) Financial distress is a term used to indicate a condition when promises to creditors of a


company are broken or honored with difficulty.
• Sometimes financial distress can lead to bankruptcy.
• Financial distress is usually associated with some costs to the company; these are
known as costs of financial distress.
• This is a situation where a firm‘s operating cash flows are not sufficient to satisfy
current obligations and the firm is forced to take corrective action.
• Financial distress may lead a firm to default on a contract, and it may involve
financial restructuring between the firm, its creditors, and its equity investors.

c) Valuation of Compulsorily Convertible Debenture:


The debenture-holders of a Compulsorily Convertible Debenture (CCD) receives interest
at a specified rate for a pre-determined period after which a part or full value of the CCD
is converted into specific number of equity shares. The cashflows resulting in the case of
valuation of CCD are;
- Periodic interest receivable from the company.
- Expected market price of the share received on conversion.
- Redemption amount, if any.
The value of a CCD is then found out by using the following formula:
n
B0 (CCD) = ∑ Ii + mPt + RV
i
i=1 (1 + kd) (1+ke)t (1+kd)n
where, B0 (CCD) = Value of a CCD
I = Interest amount receivable per year
ke = Required rate of return on equity component
m = Number of shares received on conversion
Pt = Share price at the time of conversion
RV = Redemption value, if any

ZYA P.T.O.
(74)

n = Life of the debentures


kd = Rate of discount of debt.
In the case of partially convertible debentures, the annual interest before conversion and after
conversion would be different whereas in the case of fully convertible debentures, there will
not be any RV.
d) Line of Credit with a Bank:
A line of credit with a bank is an informal arrangement between a bank and its customers
specifying the maximum amount of credit which the bank will permit the firm to borrow at
any point of time. Normally, credit lines are established for a one-year period. The line of
credit is renewed by the bank once it receives the latest annual report and reviews the
progress of the borrower. The amount of the line of credit depends on the bank‘s assessment
of the creditworthiness and the credit needs of the borrower.
A line of credit may be adjusted upwards or downwards at the time of renewal based on the
changes in these conditions. The cash budget of a firm is an indicator of the borrower‘s short
term credit needs. The firm usually seeks a line of credit amount slightly in excess of
maximum or peak borrowing needs during the forthcoming year to give a margin of safety.
The bank may impose a ‗clean up‘ provision in a line of credit arrangement. Accordingly, the
borrower would be required to clean up the bank debt for a specified period of time during
the year. Such a cleanup period is usually one to two months.
It is, however, to be noted that a line of credit does not constitute a legal commitment on the
part of the bank to extend credit. The borrower is informed of the line of credit through a
letter indicating that the bank is willing to extend credit up to a certain amount. If the credit
worthiness of the borrower gets deteriorated during the year, the bank might not want to
extend credit or reduce the amount of credit already intimated.

7. Distinguish between: (4×2.5=10)


a) Risk aversion Vs. Risk diversification
b) Investment bankers Vs. Mortgage bankers
c) Horizontal merger Vs. Vertical merger
d) Operating breakeven point Vs. Financial breakeven point
Answer No. 7
a) Risk aversion is the reluctance of a person to accept a bargain with an uncertain payoff
rather than another bargain with a more certain, but possibly lower, expected payoff. For
example, a risk-averse investor might choose to put his or her money into a bank account
with a low but guaranteed interest rate, rather than into a stock that may have high
returns, but also has a chance of becoming worthless. An investor is said to be risk averse
if he prefers less risk to more risk, all else being equal.
Risk Diversification refers to minimization of risk which an investor may choose by
investing in various types of securities. An investor may not want to concentrate his
investment in a single risky security, as a result of which he may choose to invest in
various other securities to minimize his level of risk and harmonize his returns.

b) Investment bankers are middlemen who are involved in the sale of stocks and bonds.
When a company decides to raise funds, an investment bank comes up with the proposal
to buy the issue at wholesale and then go to sell the same to investors at retail.

ZYA P.T.O.
(75)

Being in the business of matching users of funds with suppliers, investment bankers can
sell issues more efficiently than the issuing company. For the services provided, they
receive fee as the difference between the amounts received from the sale of securities to
the public and the amount paid to the companies.
Mortgage bankers are involved in acquiring and placing mortgages. The mortgages to
the mortgage bankers come through the individuals, businesses and builders and real
estate agents. These bankers do not hold mortgages in their own portfolios for a long
time. They usually service these for the ultimate investors. They receive fees from the
ultimate investor for the services provided to them.

c) Horizontal merger takes place when two or more corporate firms dealing in similar lines
of activity combine together. Elimination or reduction in competition, putting an end to
price cutting, economies of scale in production, research and development, marketing and
management are often cited motives underlying such mergers.

Vertical merger occurs when a firm acquires firms upstream from it and/ or firms
downstream from it. In the case of an upstream merger, it extends to the suppliers of raw
materials, and to those firms that sell eventually to the consumer in the event of a
downstream merger. Thus the combination involves two or more stages of production or
distribution that are usually separate. Lower buying cost of materials, lower distribution
costs, assured supplies and market, increasing or creating barriers to entry for potential
competitors or placing them at a cost disadvantage are the chief gains accruing from such
mergers.

d) The operating breakeven point is defined as the units of output at which total revenues are
equal to total operating costs (fixed costs plus variable costs). The operating breakeven
point is calculated as follows:
Operating BEP = Fixed Cost/ Contribution Margin.

Financial breakeven point is the situation where EBIT equals to financing cost. In this
analysis, the firms needs to just cover all of its financing costs and produce earnings per
share equal to zero. Financial Breakeven analysis can be used to help determine the
impact of the firm's financing mix on the earnings available to common stockholders.

ZYA P.T.O.
CAP-II, BC, June 2012
Suggested Answer
Maximum Marks – 100
Total No. of Questions - 10
Time Allowed – 3 Hours
Marks
All questions are compulsory.

Section -'A'

1. Read the following case carefully and answer the questions given below:

Staying tuned in to a customer‘s concerns is the number one focus of proposal


writers at Hewlett-Packard. A proposal today, especially in a global environment,
involves more than merely supplying a product. Mary Piecewicz, at Hewlett-
Packard, says, ―It‘s more like a marriage between two companies. Before we can
write a proposal, we need to be detectives and learn as much as possible about
the potential partner.‖ HP begins searching for information. From the customer‘s
annual report, news release, industry reports, and other sources, HP detectives
search for specifics about its products, strategies, and company culture. What hot
buttons is the company likely to respond to?
In organizing proposals, HP always responds to the customer‘s outline. If the
customer doesn‘t specify a plan, proposals are arranged as follows: Section 1
includes the executive summary, which is the most important part of the
proposal. It spotlights the hot buttons and the proposal criteria set forth in the
request for proposal (RFP). Section 2 covers specifications and technical
descriptions. Section 3 lists costs, terms, and conditions. Section 4 presents
supplemental literature, including generic resume and staff qualifications.
Authorization to proceed is usually discussed after receipt of the proposal. In this
way customers are able to negotiate the formal agreement with the sales account
manager.

Because of its importance, the executive summary gets special attention. It may
open with a brief history of HP, but its primary focus is on the customer‘s needs.
―We address every customer issue and specify our 'differentiators',‖ explains
Piecewicz. ―What makes HP stand out from our competitors? The executive
summary is really the selling tool in our proposals, and we spend the most time
on it.‖

Issues for Critical Thinking: 10


a) Why is it important to become a ―detective‖ before beginning to write a
proposal?
b) If Hewlett-Packard were preparing a proposal for a company, Fortune 500,
what kinds of information and where should the proposal team investigate?
Where could it find such information?
c) What is the most important part of an HP proposal, and what should it
include?
Answer No. 1
a) Writing a proposal demands a considerable ―detective‖ work. It is because a proposal
writer needs to collect considerable amount of information from the customer. Unless
the writer makes sure that s/he has understood the customer‘s problem well, the proposal
ZYA P.T.O.
(77)

cannot be persuasive, and hence it is not accepted. Since no customer wants to reveal its
internal information to the public, it is inaccessible to the proposal writer too. Therefore,
a report writer must research all the significant areas to learn all it can about the
customers.

b) HP team needs to collect information from such areas as a customer‘s corporate culture,
degree of environmental consciousness, current business strategies, and how Fortune
500 serves its customers. (examinees need to explain these points in brief)

c) The most important part of an HP proposal, as its member Piecewicz explains, is its
executive summary. Opening with a brief history of HP, it will mainly focus on the
customer‘s need. Besides, it should include proposal highlights that comprise summary
of objectives to be met and procedures to be followed. In fact, the executive summary
should speak for the complete proposal, it should be able to stand alone. Budget figures
are frequently omitted because proposal abstracts may receive wide distribution.

2. Discuss in brief the concept of ‗diversity in the workplace‘? How are the
problems caused by workplace diversity settled? 10
Answer No. 2
The term ‗diversity in the workplace‘ refers to the situation where workers may vary
because of many variables such as gender, culture, nationality, religion, ability, ideology,
etc. In the cross-cultural and global context of the world, the work places have been
characterized with diversity. The work place diversity may invite a number of problems
related to the characteristics of the workers with different races, cultures and genders.
Misunderstanding may take place because of diverse situations. The success of the
organization may have a lot of challenges when conflict rises among the diverse workers.
The lack of cooperation and emotional support may also result from work place diversity. In
such condition where there is work place diversity, it can be managed quite trickily and
wisely. The increasing diversity of workplace should be understood as an organizational
asset with multiple opportunities and potentials. The problems caused by the diversity can
be settled with efforts such as the following:
 Conduct a diversity audit in organizations to find the most effective program for
your needs.
 Design increasing meetings, useful for all.
 Build cohesive multi-cultural work teams.
 Create a corporate culture that can accommodate diversity to maximize the potential
of your workforce

3. How can you as a participant involve actively and productively in a meeting? 10

Answer No. 3
Meetings are opportunities for you to showcase your abilities and boost your career. To get the most
out of the meetings you attend, try these techniques:
 Arrive early. You show respect and look well organized by arriving a little early.
 Come prepared. Bring the agenda and any distributed materials. Study the topics and be ready
with questions, comments, and good ideas.
 Have a positive attitude. Use positive body language; speak energetically.
ZYA P.T.O.
(78)

 Contribute respectfully. Wait for your turn to speak; raise your hand to be recognized.
 Wait for others to finish. Show respect and good manners by not interrupting.
 Keep your voice calm and pleasant, yet energetic. Avoid showing anger as this focuses
attention on your behavior rather than on your ideas.
 Give credit to others. Gain allies and enhance your credibility by recognizing others in front of
peers and superiors.
 Put the cell phone and laptop away. Focus your attention on the meeting, not on answering e-
mails or working on your computer.
 Help summarize. Assist the meeting leader by reviewing points you have noted.
 Express your views in the meeting. Build trust by not holding postmeeting ―sidebars‖ that
involve criticism and judgments.
 Follow up. Send the signal that you are efficient and caring by completing the actions assigned
to you.

4. Describe a situation in which you exhibited outstanding listening behavior and


capabilities. How did they influence the speaker‘s subsequent communication
attitude and behavior? 10

Answer No. 4
Listening may be defined as the ability to understand and respond effectively to the speaker
(sender) during oral (face-to-face) communication. Though listening and hearing sound alike, they
are different in organizational communication context. While listening perceives the contents of
what is said, hearing just perceives sound without paying any attention to what is said. The latter
does not involve any response or feedback.
Listening is one of the most fundamental skills of human communication activities that idealize
interpersonal human relations through sharing and exchange of information. Listening involves
sensing. Listening is always a participatory approach to effective communication. Most of the
evidences show that the receptive side of listening is more crucial and causes more problems than
transmitting messages for others to understand them. Poor listening is the major cause of
miscommunication. Improving listening ability is the major challenge for better communication
Listening is the most important of all communication skills. The listening process takes place in
four stages: perception, interpretation, evaluation and action. Listening, the most basic quality of
human communication, is an act of sharing and exchange of ideas, experiences and opinions. The
best way to understand people is to listen to them properly. So listening remains to be the most
prominent and structured procedure of communication that broadly consists of methodological
procedures like sensing, interpreting, evaluating, remember and responding. Ninety percent of the
organizational problems are caused due to misleading and misrepresentation resulting from
listening problems. The same piece of message conveyed by the speaker may be subjected to both
hearing and listening. Sometimes, a piece of message is just heard, if it is considered less important.
It also depends upon how the listener reacts to it. Other things remaining the same, a good manager
is required to be a good listener in order to be a good speaker.
Listening involves filtering – a process of decoding the message to get the meaning through mental
verification. It depends upon our knowledge, beliefs, biases and experiences. To elaborate, listening
involves:

Sensing: As a member of a management organization, Mr. Y, a typically good listener gains


adequate amount of listening skills and feels confident that his other communication skills depend.
In the process of developing listening skills, he first considers sensing things by perception.

ZYA P.T.O.
(79)

Interpretation: He focuses on interpretation as the deliberate process to have explanations,


illustrations and establish proper understanding. This is followed by a process of finding logic and
evidence. Every good listener is required to have proper analytical skills to prepare proper logic and
evidence, classification and sub-classification of ideas and argumentative habits.

Remembering: He considers remembering as the process of internationalizing information before it


is communicated to the listeners. In short, it is the quality of retaining in mind what has been
received (listened to). Various factors determine our capacity to remember things. Some people
have photographic memory and can reproduce past information.

Responding: Responding is the final stage of human communication process in which the listeners
reproduce replies or respond to the speaker on the specific state of affairs, i.e. queries made by the
speaker. The responses reflect the listeners‘ reactions to various aspects of the message. They could
be positive and negative and confused understanding.

Listening as a process of communication could be best understood as a set of the following ten
commandments.

The Ten Commandments of Listening:


1. Develop positive thinking attitude towards speakers.
2. Question and counter question for the appropriate compressions.
3. Raise the speaker in status of role model.
4. Listen to concepts, key ideas and facts.
5. Listen attentively and patiently.
6. Put the speaker at ease .
7. Be patient.
8. Judge content, not the appearances or habits of the speaker.
9. Keep an open mind when confronted with temperamental influences.
10. Apply the standard of mind filter.

5. Briefly explain any FOUR of the following: (4×2.5=10)


a) Communication process
b) Active listening techniques
c) Benefits and challenges of workforce diversity
d) Techniques of closing the interview
e) Informational reports
Answer No. 5
a) Communication process
 The sender encodes (selects) words or symbols to express an idea. The message is sent verbally
over a channel (such as an e-mail, letter, or telephone call) or is expressed nonverbally, perhaps
with gestures or body language. ―Noise―—such as loud sounds, misspelled words, or other
distractions—may interfere with the transmission. The receiver decodes (interprets) the message
and attempts to make sense of it. The receiver responds with feedback, informing the sender of
the effectiveness of the message. The objective of communication is the transmission of
meaning so that a receiver understands a message as intended by the sender.
b) Active Listening Techniques

ZYA P.T.O.
(80)

 Experts say that we listen at only 25 percent efficiency. While listening to superiors on the job,
take selective notes, avoid interrupting, ask pertinent questions, and paraphrase what you hear.
When listening to colleagues and teammates, listen critically to recognize facts and listen
discriminately to identify main ideas and to understand logical arguments. When listening to
customers, defer judgment, pay attention to content rather than form, listen completely, control
emotions, give affirming statements, and invite additional comments. Keys to building powerful
listening skills include controlling external and internal distractions, becoming actively
involved, separating facts from opinions, identifying important facts, refraining from
interrupting, asking clarifying questions, paraphrasing, taking advantage of lag time, taking
notes to ensure retention, and being aware of gender differences.
c) Benefits and challenges of workforce diversity
 Having a diverse workforce can benefit consumers, work teams, and business organizations.
However, diversity can also cause divisiveness among identity groups. Business communicators
should be aware of and sensitive to differences in the communication techniques of men and
women. To promote harmony and communication in diverse workplaces, many organizations
develop diversity training programs. You must understand and accept the value of differences.
Don‘t expect conformity, and create zero tolerance for bias and prejudice. Learn about your
cultural self, make fewer assumptions, and seek common ground when disagreements arise.
d) Techniques of closing the interview
 Once the interview nears conclusion, start thinking about how to end on a positive note. It is
easy to become flustered after a challenging interview, so be sure to practice questions that you
plan to ask. Also, focus on how to leave a lasting positive impression.
 Asking your own questions: Be prepared with meaningful, thoughtful questions to help you
determine whether this job is right for you.
 Ending the interview positively: Summarize your strongest qualifications, show your
enthusiasm for the job, and thank the interviewer. Ask for the interviewer‘s business card.
Shake hands, and acknowledge anyone else on the way out.

e) Informational reports
 Periodic, trip, convention, progress, and investigative reports are examples of typical
informational reports. Such reports include an introduction that may preview the report purpose
and supply background data if necessary. The body of the report is generally divided into three
to five segments that may be organized by time, component, importance, criteria, or convention.
The body should include clear headings and may use an informal, conversational style unless
the audience expects a more formal tone. The summary or conclusion reviews the main points
and discusses the action that will follow. The conclusion may offer a final thought, express
appreciation, or signal a willingness to provide further information. Like all professional
business documents, a clear, concise, well-written report cements the writer‘s credibility with
the audience.

CAP-II, Marketing, June 2012


Suggested Answer
ZYA P.T.O.
(81)

Section -'B'
6. Read the following case carefully and answer the questions given below:
Dell computers sold personal computers directly to end users while most other
computer manufacturers sold their products through distributors and resellers.
Selling direct first started through telephone then via internet. Dell‘s direct
distribution enabled customers, especially corporate customers, to express what
features they wanted in computers. Dell then quickly assembled the components
and shipped them to the target customers. Dell became very popular among
computer buyers since they can get computers exactly in the shape, size and
functions they desired. Moreover, the price-value offer of Dell was unbeatable.
Customers got computers at almost the same price as other brands with their
own specializations. In addition, the waiting time for computers was cut down
to minimum. This strategy gave the company a competitive edge in the
computer market. In order to maximize the customer services and minimize
distribution costs, the company worked very closely with local suppliers of
spare parts and customer service providers. When the business expanded, the
company was forced to add local distributors in the channels. The costs were cut
down by keeping 15 days inventory of parts and components while the major
competitors were keeping inventory for 65 days.
Questions:
a) Analyse Dell‘s distribution strategy to get the competitive edge. 5
b) Can Dell‘s distribution strategy be applied in marketing of other consumer
durables like refrigerators and TV sets? Give reasons for your answer. 5
Answer No. 6

a) Dell‘s distribution strategy considers:

 Market profile: size, competitors, stage of product life cycle


 Customer segments: groups of prospects with similar wants & needs
 Competitive analysis: strengths, weaknesses, opportunities and threats in the landscape
 Value proposition: the type of value to the delivered to the market
 Positioning strategy: position of offering to win mindshare

Dell has experienced astounding growth over the last several years. Dell‘s competitive edge
comes from

(1) Just in Time purchasing of components


(2) Lower inventory costs (avoiding carrying and inventory costs)
(3) Low distribution channel related costs and markups.

b) Obviously, this distribution strategy can be applied in case of marketing of other consumer
durables like refrigerators and TV sets. In Today‘s consumer durables market, competitive
advantage is gained by offering superior user experiences to consumers. In order to provide
quick and timely delivery to their dealers, these consumer durables can also gain competitive
edge. Todays‘ market is hypercompetitive market that‘s why to remain long time in the
business, differentiation strategy is required. Todays‘ customers are value maximizers. They try
to obtain more value from limited resourcrs.

7.

ZYA P.T.O.
(82)

a) Define the term ‗marketing‘ and explain the fundamental principles of a


customer-oriented philosophy of marketing. (2+3=5)
b) Write a note on usefulness of relationship marketing. 5
Answer No. 7
a) Generally, by marketing we mean the selling and buying of goods and services. The seller sales
goods and services and, in return, he receives payment from the buyer in the form of money or
something else. But, actually, the concept of marketing is much broader than what the general
people think.
Some scholars define the term ‗marketing‘ as business activities, some define as business
processes, and some define as social process. However, it needs to be defined in simpler and
meaningful way as follow:
Marketing is a process of doing business by understanding the real needs and problems of
target customers, fulfilling their needs and solving their problems through integrating suitable
marketing tools, and expecting profits through customer satisfaction.
Actually, marketing is a customer-oriented philosophy of business. A customer-oriented
philosophy of business is a target market-oriented, integrated marketing, and profit-oriented
philosophy of business. A marketer following the customer-oriented philosophy of business
needs to follow the following four fundamental principles –
i. First of all, identify or select the target market.
ii. Secondly, identify and understand the real needs and problems of the target market.
iii. Thirdly, address such needs and problems by integrating and adopting the suitable
marketing tools, including right product, right price, right place of distribution, right
promotion tools, better service delivery process, better relations, and better physical
environment in the office premises.
iv. Last, but not the least, try to earn desired profit from the business after the customers are
satisfied with offers.

b) Usefulness of relationship marketing


Relationship marketing has the aim of building long-term relations with key parties-customers,
suppliers, distributors. In return, company earns profit. It is concerned with building an effective
network of relationships with key stakeholders. Sellers offer quality products and services for
greater satisfaction of their customers at a fair price in proper time.
So, relationship marketing is useful due to the following facts.
- Cuts down transaction cost and time.
- Building of effective marketing network.
- Mutually profitable business relationship.
- Competition between marketing networks.
- Increase in company profit.

8.
a) What is buyer behavior? How does individual buyer behavior affect buying
decision for a particular product? Explain. (2+3=5)

b) Highlight the importance of advertising to the marketers. 5


Answer No. 8
a) A buyer is anyone who might buy a given product. A buyer may be either an individual person
or an organization that have an interest in the product and the means to acquire it. Therefore, a

ZYA P.T.O.
(83)

buyer includes both individual and organization, which is potentially willing and able to buy
products offered by the marketer.
Various scholars have defined the term ‗buyer behavior‘ in different ways. However, by
buyer behavior refers to actions and activities of the people engaged in selecting, purchasing,
and using products and services to satisfy needs. Such activities involve mental and emotional
processes in addition to physical actions.
There are several factors affecting the individual‘s buying decision: psychological factor is
among them. Psychology is a state of one's mind which requires interpretive or insight
knowledge about the consumer.
While analyzing psychology of a consumer, a marketer must be able to analyze as to why a
person behaves as he does; what motivates him to behave in a particular manner; and what role,
attitude, personality and his own image play in influencing behavior:
Psychology of a person consists of several factors, which must be carefully analyzed by the
marketer. These factors include:
 Motivation
 Perception
 Learning
 Beliefs and attitudes
 Personality

b) Advertising is any paid form of non-personal presentation and promotion of ideas, goods, or
service by an identified sponsor. It is a cost effective way to disseminate message to scattered
large masses. Advertising can be used for giving information, persuading consumers, reminding
and reassuring them to create demand.
Advertising today has got a prominent role in the promotion program of all firms/companies.
Media agencies are playing active role to meet advertising objectives of the companies. A lot of
people are involved in designing attractive advertising messages focusing on mission, message,
media and measurement elements known as the five Ms of advertising. Newspaper, journals,
posters, radio, TV, video are a few ad vehicles popularly used by the organizations.
The following points indicate the importance of advertising:
 Communication of information/message.
 Creation and extension of demand.
 Introduce new product.
 Educate, persuade and remind customers.
 Influence buying decision.
 Save time and money etc.
 Achieve organizational goals.
 Image building.
 Selling on mass scale.
 Help middle men
 Protection from competition etc.

9.
ZYA P.T.O.
(84)

a) What is a new product? Explain the process of new product development. (2+3=5)
b) Describe the environmental forces that affect the company‘s ability to serve
its customers. 5
Answer No. 9
a) By the term "new product", the general people think that it should be an innovative product,
which is developed and introduced in the marker for the first time. However, in marketing, a
new product has a wider meaning. For marketers a new product may be either an innovative
product, an imitation product, or a modified product.
A new product development process usually calls for the following six steps.
 Generation of new-product ideas: They are targeted at satisfaction of unmet needs of
customers.
 Screening and evaluation of product ideas: unfeasible ideas are eliminated.
 Concept development and testing
 Business analysis: It is done to evaluate product‘s business attractiveness.
 Product Development: Product concept is developed into physical product.
 Test marketing: Product is put on sale in a test market.
 Commercialization: Full scale production and marketing programmes are prepared.

b) The marketing environment is made up of a microenvironment and macroenvironment. The


company‘s microenvironment consists of actors close to the company that combine to form the
company‘s value delivery system or which affect its ability to serve its customers. This includes
the suppliers, marketing intermediaries, customers, competitors and public. The forces in
microenvironment directly influence whether and how a marketing manager can perform
marketing activities. It may affect a marketing manager‘s decisions and actions through their
influence on consumer‘s reactions toward the firm‘s marketing mix. The macroenvironment
consists of six larger societal forces that affect marketing. These include demographic,
economic, natural, technological, political-legal and socio-cultural forces. These forces shape
opportunities and pose threats to the company.

10. Briefly explain the following: (5×2=10)


a) Marketing research process
b) Marketing mix
c) Features of packaging
d) Physical distribution
e) Personal selling
Answer No. 10
a) Marketing research is the systematic design, collection, analysis and reporting of data and
findings relevant to a specific marketing situation facing the company. In many large
companies a separate marketing research department is found. In order to solve various
organizational problems marketing research is very helpful. Its popularity and use is widely
accepted. The marketing research process involves the following steps:

- Define the problem and research objectives.


- Develop the research plan.
- Collect the information.
- Analysis the information.
ZYA P.T.O.
(85)

- Report the findings.


- Make the decision related to marketing problem.

b) Marketing mix is the combination of the four major components that comprise a company's
marketing program. Marketing mix is the core of marketing program.
Kotler defines marketing mix as "a set of marketing tools that the firm uses to pursue its marketing
objectives in the target market".
Similarly, Mc Carthy defines marketing mix as "the controllable variables, which the company puts
together to satisfy its target market".
Similarly, W. J. Stanton defines the term “marketing mix” as “the term used to describe the
combination of the four inputs which constitute the core of a company’s marketing system; the
product, the price structure, the promotional activities, and the distribution system”.
Altogether, seven elements or components of marketing mix have been identified, called as 7P‘s,
including four major components (product, price, place and promotion) and three supporting
components (process, people and physical evidence) for service products..

c) Packaging is one of the important parts of product. Most products have to be packed. It is
concerned with all the activities of designing and producing the container for a product. Well-
designed packages can create convenience and promotional value as such many marketers treat
packaging as the fifth P in the marketing mix.
The following features are found in a packaging system:
A good package must be convenient, attractive, communicative, economic and environment
friendly. In general packaging is done with a view to handle, carry, store, identify and promote
the product. Brand image results from good packaging system.

d) Physical distribution as a major part of market logistics constitutes about 25 percent of


product's cost. Today customers want frequent deliveries, short order cycle times, direct
delivery, careful handling of products in a coordinated manner with the help of IT. In a
competitive market physical distribution as a part of distribution system has ultimate goal to
meet customers' requirements is an effective and profitable manner.

There are many components of physical distributions like order processing, warehousing,
inventory management, material handling, transportation. They all are concerned with the
proper flow of goods and services from the production location to the point of consumption.
Marketing channel needs to the integrated with channel system to achieve the distribution
goals.
e) It is a process of informing customers and persuading them to purchase product
through personal communication. They are sales personnel who show
salesmanship to deal and handle customers. Sales force occupies a significant
role in marketing and promotion. Managing the sales force has become a
challenging task to big companies. Personal selling is personal communication
to inform and persuade target customers. Sales people are in direct contact with
the buyers so that they can overcome problems as well as convince and satisfy
the market. Sales people are also capable to attract new buyers and help to
introduce new products through indoor or outdoor sales process. Though
expensive and difficult, it is two way communication & less wasteful method of
promotion.

ZYA P.T.O.
CAP-II, Income Tax & VAT, June 2012

Roll No……………. Maximum Marks - 100

Total No. of Questions - 6 Total No. of Printed Pages -5

Time Allowed - 3 Hours


Marks
Attempt all questions. Working note should form part of the answer.

1. Answer the following with reference to the Indian Income Tax Act, 1961:
a) What do you mean by ―Previous Year‖? When income of previous year is not
taxable in the immediately following assessment year? (3+2=5)

b) Mr. Ravichandran, an Indian citizen, who is appointed as senior taxation


officer by the Government of Uganda, leaves India, for the first time, on
September 26, 2010 for joining his duties in Uganda. During the previous
year 2011-2012, he comes to India for 176 days. Determine residential status
of Mr. Ravichandran for the assessment years 2011-2012 and 2012-2013. 3

c) Whether a trust/institution claiming exemption under section 11 can carry out


business activities or not? Explain in brief. 2
2.
a) Rolls Airways is conducting the airplane services between Kathmandu and
Saudi Arabia. Rolls Airways has a branch office at Kathmandu for the
purpose of operation of its services. The income of Rolls Airways for the
month of Shrawan, 2067 while conducting the airplane services and other
operations are as below:

SN Particulars Amount (Rs.)


1 Amount received from the passengers departed from Kathmandu 10,000,000
2 Amount received from the transportation of the goods having first 5,000,000
point entry at Kathmandu
3 Income from the travels from Bhutan and Lhasa having transit point 5,000,000
at Kathmandu
4 Amount received from the rentals of the goods carried from 5,000,000
Kathmandu which were transported from Bhutan
5 Amount received from the services of restaurant operated for the 2,500,000
facility of passengers at Kathmandu Airport
6 Amount received from the services of Ground Handling for other 2,500,000
airlines‘ operation
Total Income 30,000,000

The followings are the details of expenditures of Rolls Airways:

ZYA P.T.O.
(87)

SN Particulars Amount (Rs)


7 Expenditures from Air tickets 5,000,000
8 Expenditures against the accommodation and fooding for crew 5,000,000
members
9 Expenditures against ground handling services 3,000,000
10 Expenditure for the operation of restaurant 2,000,000
11 Expenditure for the transportation of passengers from Bhutan, Lhasa 5,000,000
12 Expenditure for the re-transportation of the goods transported from 5,000,000
Bhutan
Total Expenditure 25,000,000

Note: Rolls Airways has managed the ground handling against 100 flights of
its own and 100 flights of other airlines during Fiscal Year 2067/68.
Explain about the inclusion of above incomes and allowances of expenditures
of a non-resident person, for the purpose of section 70 of Income Tax Act,
2058 stating the relevant provisions. 10

b) Mr. Anil has held 500 shares, at the rate of Rs. 100 per share (par value), of
Standard Chartered Bank Nepal Ltd (listed on Nepal Stock Exchange Ltd).
He has requested to the stock broker to sell those shares held by him. Mr.
Anil has purchased those shares at the rate of Rs. 1,000 per share. He has
paid Rs. 4,000 as brokerage commission at the time of purchase of shares. He
sold those shares at the rate of Rs. 7,000 per share and paid the brokerage
commission of Rs. 17,500 to the broker on sale.
i) Compute the gain on shares disposal.
ii) Is there any implication of withholding taxes? If yes, calculate the
payment to be made to Mr. Anil stating relevant provisions of the Income
Tax Act, 2058.
iii) What will be the implication if the above mentioned shares were held and
sold by an entity instead of Mr. Anil? 6
c) N estimated Rs. 1,000,000 for the Fiscal Year 2067/68 as estimated tax and
paid Rs. 400,000, Rs. 300,000 and Rs. 300,000 at Poush, Chaitra and Ashadh
respectively. Actual tax assessed under section 99 found Rs. 1,300,000
settled on Kartik end with filing income tax return. Compute interest under
section 118 and 119. 4

3.
a)
Ram and Krishna had contributed Rs. 20 lakhs & Rs 30 lakhs respectively
and deposited the amount into a common bank account to be used to
ZYA P.T.O.
(88)

purchase a plot of land to resale the same later on. After 6 months of
depositing the money into the bank account, they purchased a plot of land for
Rs. 45 lakhs and got the land registered in their joint name (i.e. in the name
of both Ram & Krishna jointly). Expenses on registration and commission at
the time of purchase of land amounted to Rs. 500,000. They later on sold that
plot of land at Rs. 86 lakhs and they have paid Rs. 6 lakhs as sales
commission and other incidental expenses related with sales. The bank has
given interest @18% PA on such bank deposit.
Calculate the amount to be incorporated as ―income‖ in the hand of Ram and
Krishna respectively from above mentioned transaction. How much would be
the interest income and how it is treated in the hand of Ram & Krishna? 5
b) What are the entities considered as tax exempted entities? Whether Sections
87, 88, 88ka and 89 of Income Tax Act with regard to TDS are attracted to
such entities? Explain. 5
c) Mention, with reasons, the amount of tax that need to be deducted at
source/accounted for in the books in following cases: (5×2=10)
i) M Ltd. has entered into an agreement with PC Pvt. Ltd. to conduct an
awareness program on maternal health in 5 districts. The total amount
paid by M Ltd. against such agreement was Rs. 40,000. In addition to
such amount, M Ltd. also reimbursed Rs. 20,000 as expenses on air –
ticket, lodging and fooding, daily expenses etc of staffs of PC Pvt. Ltd. to
PC Pvt. Ltd. as per the terms of the agreement. PC Pvt. Ltd. did not give
VAT invoice.

ii) M/S ABC Company Limited has 60 shareholders. The company has
distributed iPhone worth Rs. 38,000 to each shareholder free of cost in
FY 2068/69.

iii) Mahesh is constructing a building for his personal residence. He has


appointed Ramesh, an engineer, to take care of the overall construction
activities at monthly remuneration of Rs. 30,000.

iv) MNO Company Pvt. Ltd. has taken loan Rs. 10 lakhs from Hamro Bank
Limited @ 12% PA. MNO Company Pvt. Ltd. has paid loan service
charge of Rs. 25,000 and interest Rs. 90,000 during the year to the Hamro
Bank Limited.

v) Mr. Ram is invited to deliver lecture on corporate finance by KBC


College. KBC College has paid Rs. 10,000 to Mr. Ram for delivering
lectures, Rs. 3,000 for setting up model questions and Rs. 5,000 for
checking the answer-sheets.

4.
a) Calculate the applicable tax rate on ABC & Co. based on following
information; 5

ZYA P.T.O.
(89)

 ABC & Co. is registered as a special industry as provided in section 11 of


Income Tax Act, 2058.
 The industry is situated in an underdeveloped area with an employment
of 350 people.
 Mr. Z has a source of income in Nepal and also in more than one foreign
country. During the Fiscal Year 2067/68, income and tax paid in each
foreign country is given below;
Name of the Country Income (Rs.) Tax Paid (Rs.)
USA 200,000 60,000
Australia 150,000 30,000
UAE 100,000 5,000
Nepal 250,000 -
He is a resident natural person and selected for the couple as tax payers
during the year. Calculate his tax liability during the Fiscal Year 2067/68. 5

5.
a) Satellite Telecom Pvt. Ltd., a telecom operator company in Nepal, seeks your
advice whether it can take credit of the VAT paid in the following
transactions. 5
i) VAT paid Rs. 18,000 for the purchase of petrol to be used for vehicle
operation.
ii) VAT paid at Birgunj Customs Office Rs. 1,000,000 while importing the
telecom equipments from China.
iii) The company deposited VAT amount Rs. 175,000 in Inland Revenue
Office on the service charges payable to satellite bandwidth provider and
want to take VAT credit of this whole amount.
iv) The company purchases beverage in the occasion of annual day and paid
VAT Rs. 10,000 on this bill.
v) VAT paid Rs. 13,000 for the purchase of petrol to be used for the
operation of generators as BTS sites.
b) Mercantile Limited is providing service in the field of information
technology and its sales in the month of Chaitra 2067 was Rs. 5 million. Out
of total sales Rs. 3 million was export. The company has got some service
from Star Inc. registered and working in USA. Cost paid for the services was
Rs. 2 million and the company paid Rs. 260,000 in IRO as reverse charging
payment. Total purchases for the month of Chaitra were Rs. 1 million and it
has also incurred the following expenditures:
- Telephone charges Rs. 10,000
- Petrol for vehicle Rs. 5,000
- Printing and stationery Rs. 5,000

The company has taken VAT bill for all the above purchases. You are
required to calculate the actual amount of VAT payable by Mercantile
Limited for the month of Chaitra 2067. 5
c) In the course of VAT refund audit of M/S Wipro Ltd., an export-oriented
undertaking, the tax officer issued notice under section 20 of the Value
Added Tax Act, 2052 and Rule 29 of the Value Added Tax Regulation, 2053
ZYA P.T.O.
(90)

disallowing certain input tax credit on the ground that it is unrelated to the
business. What are the remedies available to the company? Advice. 5
d) Cosmos Pvt. Ltd. having registered office in Jawalakhel, Lalitpur transferred
some goods costing Rs. 1,000,000 to its branch in Bangladesh. In the same
month, branch in Bangladesh transferred some goods costing Rs. 500,000 to
its head office in Nepal. Gross Profit margin of head office is 20% and
branch is 25%. The profit of branch is consolidated at the end of each
financial year. Find the VAT impact on the above transactions. 5
6.
a) State the place of supply as per Value Added Tax Act, 2052/Value Added
Rules, 2053, in case of: 5
i) Sale of movable goods
ii) Immovable goods
iii) Imported goods
iv) Self-consumption
v) Service

b) Explain the term ―Abbreviated Tax Invoice‖ as per Value Added Tax Act,
2052. 5

c) Mention the penalties prescribed under Value Added Tax Act, 2052 on the
following infringements: 5
i) Failure to issue a tax invoice and in case of transportation of goods out of
the area specified by Inland Revenue Department, worth more than Rs.
10,000 without accompanied with a tax invoice.
ii) Registration as mentioned in section 10(1) and (2) related infringement.
iii) Failure to display the tax board as per VAT Act/Rules.
iv) An unregistered person issuing an invoice or documents showing
collection of tax.
v) Tax officer finding that a taxpayer has reduced tax liability by making
infringement of any provisions of this act or rules there-under.
d) Mr. Cristiano Messi, a foreign tourist, visited Nepal for a month and is now
returning to his home country Japan. He has purchased many goods in Nepal
and paid VAT on such purchase. He seeks your advice on refund of VAT on
paid by him on purchase and expenses/costs associated with such refund. 5

OUC
CAP-II, Income Tax & VAT, June 2012
Suggested Answer

Answer No.1

ZYA P.T.O.
(91)

a) Income earned in a year is taxable in the next year. The year in which income is
earned is known as previous year. From the assessment year 1989-1990
onwards, all assesses are required to follow financial year (i.e. April 1 to March
31) as the previous year. This uniform previous year has to be followed for all
sources of income. In case of a newly set up business/profession, the first
previous year commences on the date of setting up of the business/profession and
ends on the immediately following March 31.

The rule that the income of the previous year is assessable as the income of the
immediately following assessment year has certain exceptions. These are:
i. Income of non-resident from shipping.
ii. Income of persons leaving India either permanently or for a long period of
time.
iii. Income of bodies formed for short duration.
iv. Income of a person trying to alienate his assets with a view to avoiding
payment of tax;
v. Income of a discontinued business.

In these cases, income of a previous year may be taxed as the income of the
assessment year immediately preceding the normal assessment year.

b) During the previous year 2010-11, Mr. Ravichandran is in India for 179 days and
during four previous years preceding the previous year 2010-11, he was in India
for more than 365 days. He will be non-resident for the assessment year 2011-12.
An Indian citizen, leaving India for the purpose of employment will be treated as
resident in India only if he has been in India in that year for at least 182 days.

During the previous year 2011-12 (for the assessment year 2012-13), Mr.
Ravichandran comes to India for 176 days. He will be treated as non-resident in
India for the assessment year 2012-13. An Indian citizen who comes to on a visit
to India during the previous year, will be treated as resident only is he has been
in India in the year for at least 182 days.

c) The trust and institution can carry out business activities if the business activities
are incidental to the attainment of its objective and separate books are maintained
without losing complete exemption from income-tax.

Answer No.2

a) Section 70 of Income Tax Act, 2058 prescribes the following provisions regarding
taxation on the income of a non-resident person in Nepal from transport or
transmission business.
ZYA P.T.O.
(92)

The gross receipts from the following activities of a non-resident person are treated
as the taxable income of the person from the activities.
A person engaged in Nepal in any road, air, water or a chartered transport services,
other than transmission in Nepal, for any of the following services;
i. The carriage of passengers who embark from within a territory of Nepal.
ii. The services of mail, livestock, or other movable tangible assets that are
embarked from the territory of Nepal.

Expenditure incurred in conducting such activities are neither allowed to be deducted


from the taxable income from the above sources nor allowed to be deducted from
any other sources of income of the person.

The tax rate applicable shall be as prescribed under section 2(7) of annexure 1 of
Income Tax Act (which is 5% of gross receipts during the year).

No tax credits shall be allowed to the person to reduce the tax payable by the person
under this section.

Accordingly, following treatment shall be done for the purpose of section 70 in case
of Rolls Airways.
1. For the purpose of section 70, the income of Rolls Airways for the month
of Shrawan, 2067 as mentioned in point no. 1 and 2 of question shall be Rs.
15,000,000. Tax shall be 5% of Rs. 15,000,000=Rs. 750,000 as per section
2(7) of annexure 1 of Income Tax Act.

2. The amount mentioned in point no. 3 and 4 in the question shall not be
taxed as per Income Tax Act, 2058 being sources of payment not in Nepal.

3. The amount mentioned in point no. 5 and 6 in the question are not the
incomes to be included for the purpose of section 70. These incomes shall
be taxed like the entity having general business income.

4. The expenditures incurred in connection with generation of income as


mentioned in point no 5 and 6 in the question are eligible for deduction.

5. The expenditures as mentioned in point no 7, 8 and 9 in the question are not


eligible for deduction for the purpose of computation of income as per
section 70 even though they are incurred in connection with the generation
of income from ground handling services by its own.

6. The expenditures as mentioned in point no 11 and 12 in the question are not


eligible for deduction for the purpose of computation of income under
section 70 as these are incurred in connection with generation of income,
ZYA P.T.O.
(93)

the taxation of which are not required to be paid under Income Tax Act,
2058.

7. Half of the ground handling expenditures amounting to Rs. 1,500,000 and


expenditure against restaurant operation amounting to Rs. 2,000,000 (total
Rs. 3,500,000) are allowable expenditure for deduction as these are
incurred in connection with generation of income.

8. Corporate tax at the rate of 25 percent shall be paid on the amount as


mentioned in point no 5 and 6 in the question amounting to Rs 5,000,000
less expenditure of Rs. 3,500,000 (assuming no other eligible expenditure
to be claimed).

b) Section 95 ka of Income Tax Act deals with the gain on disposal of interest on
resident entity.

If case any person disposes its interest in a company listed with SEBON for publicly
trading of its securities, NEPSE has to deduct advance tax from the amount payable
to it, at the prescribed rate on the gain on disposal of the security as computed under
section 37 of the Act.
The prescribed rate is 5% for a natural person and 10% for others.

Accordingly,
i.
Computation of gain on disposal of shares:

Particulars Amount
(Rs)
Sale value (500* Rs. 7,000) 3,500,000
Outgoings:
Share value (500*Rs. 1,000) 500,000
Add: Share Brokerage Commission
On Purchase 4,000
On Disposal 17,500 (521,500)
Gain on disposal of shares 2,978,500

ii.
As per section 95ka, NEPSE has to withhold advance tax at the rate of 5% of
the gain which is Rs 148,925 through respective broker and make the net
payment Rs. 3,333,575 to Mr. Anil as computed below;

Particulars Amount
(Rs)
ZYA P.T.O.
(94)

Sale value (500* Rs. 7,000) 3,500,000


Less:
Brokerage Commission on Disposal of shares (17,500)
Withholding Taxes (148,925)
Net Payment 3,333,575

iii. In case shares were held and sold by an entity instead of Mr. Anil, withholding
tax shall be deducted at the rate of 10% on the amount of gain as calculated
above.

c) Computation of Interest under Section 118 for the Fiscal Year 2067/68
Amount
(Rs)

Liability u/s 99 1,300,000


90% 1,170,000
Installment
Installmen Amount Amount Shortfall/Exces Cumulativ Interest @
Due on t due due Paid s e 15%

Push-I 40% 468,000 400,000 68,000 68,000 2,550


Chaitra-
II 30% 351,000 300,000 51,000 119,000 4,463
Ashadh-
III 30% 351,000 300,000 51,000 170,000 6,375

1,000,00
Total 1,170,000 0 170,000 13,388
Interest under section 118 13,388

Computation of Interest under Section 119 for the Fiscal Year 2067/68
Interest to be paid for deficit since Kartik for all due amounts at the rate of 15% according to
section 119. So the interest under section 119 is Rs. 3,750 [(1,300,000-
1,000,000)*1/12*15%].

Answer No.3
a)
Amount
Particulars Remarks
Ram Krishna

ZYA P.T.O.
(95)

Expenses
Cost of plot of
4,500,000
Land
Other expenses 500,000
Apportioned on the basis of
Total Cost of Land 5,000,000 2,000,000 3,000,000
2:3

Sales Income
Sales Price 8,600,000
less: Expenses 600,000
Apportioned on the basis of
Net Sales Income 8,000,000 3,200,000 4,800,000
2:3

Income 1,200,000 1,800,000 in the ratio of 2:3

Apart from abovementioned income, Ram & Krishna have received interest income
of Rs. 500,000 from the bank as interest for 6 months. This interest income shall also
be shared between Ram & Krishna in the ratio of 2:3 as follows:

Ram : Rs. 180,000 Krishna : Rs. 270,000

However, the bank will deducted TDS from such interest payment and such
withholding shall be final withholding. Hence, Ram & Krishna need not incorporate
the interest income in their annual income return.

b) The definition of Tax Exempted Organization is given in section 2 (dh) of the


Income Tax Act, 2058 as follows:

a. An organization registered in Inland Revenue Department as tax exempted


organization and established with either of these two objectives:
- Established without having a profit motive, and have social, religious,
educational, or charitable objectives.
- An amateur sporting association formed for the purpose of promoting social
or sporting amenities without having an aim to distribute the income among
its members.

b. A political party registered with the election commission.


c. A village development committee, a municipality, and a district development
committee.

But, these organizations can get exemption only those donation amounts and donated
assets received as per the objective of the organization.

Applicability of Section 87, 88, 88Ka, 89 :


All these sections are fully applicable to the Tax Exempted Organizations and
therefore these organizations have to deduct and deposit Tax on payments.

ZYA P.T.O.
(96)

As per the Circular of IRD (Chh No. 235) dated 2064/1/20, these organizations must
take PAN and shall deduct TDS on all the payments in which TDS is applicable.

c) TDS
i. 15% on Rs. 60,000 (Rs. 40,000 + Rs. 20,000) = Rs. 9,000
ii. U/S 53, the cost of iPhone Rs. 38,000 shall be considered as deemed dividend
and TDS @5% need to be calculated. Further Rs. 38,000 (cost of iPhone) shall
be considered as net of tax i.e. 95%. Hence the total TDS that need to be
accounted for shall be :

No of
60
Shareholders
Cost of
40,000
iPhone
Total
deemed 2,400,000
dividend
TDS @5% 120,000

iii. TDS deduction is not required. U/S 88 (4) (Ka), being the payer is a natural
person. (2)
iv. Interest payable to the bank is not subject to TDS U/s 88(4)(kha), but the bank
charge is subject to TDS, but in practice TDS is not being deducted.
v. KBC College needs to deduct tax @15% on Rs. 10,000. On 3000 & on 5000
TDS deduction is not required.

Answer No.4
a) As the company is a special industry, the applicable tax rate is 20% as mentioned
under section 2 of annexure 1 of Income Tax Act, 2058.

Section 11(3) deals with provisions on special industry and the industry situated in
underdeveloped area.

As per the information given in the question, the industry has 350 employees during
the year. It is supposed that all the employees or more than 300 employees were
Nepali citizens and the company has engaged these 300 Nepali citizens throughout
the year. Under the circumstances the tax rate applicable will be 20%*90%=18%.

The industry is situated in an under-developed area, and it is supposed that the


industry have not completed its tenth year of commencement of its business during
the year. Under the circumstances the tax rate applicable will be 20%*30%=6%.

The company has a choice to select any one of the two concessions given and
naturally the company selects the rate of 6%.

b) Calculation of the tax liability of Mr. Z for the Fiscal Year 2067/68
Total assessable income from all the sources
ZYA P.T.O.
(97)

Particulars Amount (Rs)


Net Income from Nepal 250,000
Net Income from USA 200,000
Net Income from Australia 150,000
Net Income from UAE 100,000
Total Income 700,000
Tax Calculation
First Rs. 200,000 0
Next Rs. 100,000 15,000
Remaining Rs. 400,000 100,000
Total Tax 115,000
Average Tax Rate (115,000/700,000*100) = 16.43%
Tax credit for the year shall be available for
Country Income Tax Paid Tax Tax credit Unabsorbed
(Rs) (Rs) calculated at available for tax credit to
average rate the year (Rs) be carried
(Rs) forward
(Rs)
USA 200,000 60,000 32,860 32,860 27,140
Australia 150,000 30,000 24,645 24,645 5,355
UAE 100,000 5,000 16,430 5,000 0
Total 62,505 32,495

Tax payable during the year comes to Rs. 115,000 – 62,505 = Rs. 52,495.

Note: It is supposed that all the incomes are from business and he has opted to be a couple.

Answer No.5
a)
i. The Company cannot claim any VAT credit on the purchase of petrol as per
Rule 41.

ii. The company can claim Rs. 10,00,000 as VAT credit which was paid at Birgunj
Customs Office.

iii. It is the case of reverse charging payment and the company can claim VAT
Credit of Rs. 1,75,000 in this case.

iv. As per Rule 41, VAT credit cannot be claimed on the purchases of beverages.
Therefore, VAT credit is not allowed in this case.

v. The Company can claim VAT Credit of Rs. 13,000 in this case as the petrol was
used for the operation of generator at BTS sites.

b) Calculation of local sales:

Total Sales 50,00,000


Less: Export sale 30,00,000
ZYA P.T.O.
(98)

Local sales 20,00,000

Calculation of VAT payable for the month of Chaitra, 2067

Particulars Amount (Rs.)


VAT payable on local sales 2,60,000
VAT credit on purchases:
VAT deposited in IRO 2,60,000
Local purchases 1,30,000
Telephone charges 1,300
Printing and stationery 650 3,91,950

Net VAT Credit for the month of Chaitra 2067 1,31,950

Note:
1. No VAT is applicable on export sale.
2. Mercantile Limited can take a VAT credit on the reverse charging payment of VAT.
3. As per rule 41 of VAT Regulation, VAT credit cannot be taken on the purchase of petrol
used for vehicles.

c) Notice issued under Section 20 and Rule 29

Ms. Wipro Ltd., has to collect the grounds taken by the Tax officer for disallowing
the expenses, and think over it whether these are acceptable to it or not. In the case
of tax payer thinks that the grounds taken by the tax officer are not valid and the
decision is not acceptable to it, it can file an application to IRD for administrative
review over the decision under section 31ka of the Act.

The application should be submitted within 30 days of the final assessment order
received by it. In case the company is unable to file the application within the period,
it can apply for extension of the time for further 30 days. But the application should
be submitted within 7 days of the expiry of the time.

Before submitting the application for administrative review, it has to deposit a sum
of 100% of the undisputed amount and one third of the disputed amount.

The burden of proof that any expenditure is unrelated to business is on to the tax
officer under section 20(3) of Value Added Tax Act, 2052.

In case the decision of the IRD is also not in favor of the company, it can appeal
against the decision in Revenue Tribunal adopting a procedure and within the time
limit as prescribed by Revenue Tribunal Act.

d) For the purpose of VAT, the transactions of head office and branches within Nepal
are settled by way of stock transfer, therefore there will not be any VAT impact.

ZYA P.T.O.
(99)

In the case of stock transferred to foreign branch or vice versa, it is taken as VAT
attractive transaction and valuation is to be done at market price as per section 12 of
Value Added Tax Act, 2052.

In the given case, the following will be the VAT impact:

a. Stock transferred to Bangladesh branch is VAT attractive and VAT has to be collected at
0% (being export) on the market price of Rs. 12,00,000. So, output will be Rs. 12,00,000
and output tax will be nil.

b. Stock transferred from Bangladesh branch is import for head office. VAT needs to be
paid at customs office at 13% on market price of Rs. 6,25,000 plus customs duties, if
any. VAT paid at customs office will be allowed as input tax credit.

Answer No.6
a) Following are the place of supply for any goods under rule 15 or services under rule
16 of Value Added Rules, 2053 for any transaction:
i. Sale of Movable goods: In the case of movable goods transferred by sale, the
place where such goods were sold or transferred.

ii. Immovable goods: In the case of any immovable goods whose location cannot
be transferred even if their ownership is changed, the place where such goods are
located.
iii. Imported goods: In the case of imported goods, the custom point in Nepal
through which such goods are imported.

iv. Self-consumption: In case any producer or vendor supplies the goods to itself
for self consumption, the place where the producer or vendor of such goods
resides.
v. Service: The place of supply of a service shall be the place where the benefit of
that service is received.

b) Rule 18 of Value Added Tax Rules, 2053 prescribes for the provision of
―Abbreviated Tax Invoice‖.

A retailer, with prior permission from the tax office, can issue an abbreviated tax
invoice, the format of which is given in Annexure 6 of the rules. An abbreviated tax
invoice shall not be issued for more than NPR 5,000.

A registered person, in case it is a retailer, it may apply to the respective Inland


Revenue Office for permission to issue abbreviated tax invoice. Any registered
person cannot issue abbreviated tax invoice without such permission.

Abbreviated tax invoice is issued including the amount of VAT. Means the amount
of VAT is not shown separately. Moreover, the seller is not required to give details
about the sold goods as these are necessarily to be mentioned in the tax invoice

ZYA P.T.O.
(100)

issued as per annexure 5.

The selling price mentioned in an abbreviated tax invoice is including of VAT and
so the selling price and the amount of VAT shall be calculated by using the
following formula:
Sales Amount = Selling Price including VAT × 100
100+rate of Tax
VAT Amount = Selling Price including VAT× Rate of VAT
100+rate of Tax

As per the rule, in case the buyer requires tax invoice as per Annexure 5, the supplier
has to issue the tax invoice as required by the buyer, instead of issuing abbreviated
tax invoice to him.

c) The detailed provisions related to penalties are mentioned in section 29 of Value


Added Tax Act, 2062.

i. In case a taxpayer fails to issue the tax invoice and in case of transportation of
goods out of the area specified by Inland Revenue Department, worth more than
Rs. 10,000 without accompanied with a tax invoice, tax payer is liable for
penalty of Rs. 5,000 for each time as per section 29(1)(ga)

ii. In case a taxpayer fails to make the registration as prescribed under section10(1)
and (2), he shall be liable for penalty of Rs. 10,000 for each tax period as per
section 29(1)(ka)

iii. In case a taxpayer fails to display the tax board as per VAT act/rules, he shall be
liable for penalty of Rs. 2,000 per week and Rs. 1,000 for displayed at another
place than prescribed as per section 29(10)(kha1)

iv. In case an unregistered person issuing an invoice or documents showing


collection of tax, he shall be liable for penalty of 100% of tax payable as per
section 29(1)(gha)
As per section 15(2), the tax officer should assess the tax and collect the amount from
the taxpayer in addition to the fine as mentioned above.

v. In case a tax officer finds that a taxpayer has reduced tax liability by making
infringement of any provisions of this act or rules there-under, the tax officer
may charge, following a procedure set by Inland Revenue Department, a penalty
up to 25% of the tax payable [section 29(ka1)]

Section 29(2) is also applicable in this case and the tax officer may charge a fine
of 100% of the tax evaded amount.

ZYA P.T.O.
(101)

d) U/S 25 ka of VAT Act 2052, a foreign tourist who has paid VAT on purchase can
claim for refund of such VAT amount. However, for such refund , following
conditions must have been fulfilled-
i. He must have purchased taxable goods of more than Rs. 15,000.
ii. He must be returning via air. Refund facility is not available for
foreign tourist returning by road.
iii. The purchased goods must have been taken out of country by the
foreign tourist.

Mr. Cristiano Messi can claim for refund for VAT purchased by fulfilling
abovementioned conditions U/s 25 Ka. Service charge at the rate of 3% on amount
refunded shall be deducted by the department at the time of refund.

ZYA P.T.O.
Advanced Accounting
Suggested Answer
CAP II, June 2013
Roll No……………. Maximum Marks - 100

Total No. of Questions - 6 Total No. of Printed Pages - 4

Time Allowed - 3 Hours


Marks
Attempt all questions. Working notes should form part of the answer.

1. Radha, Kishan, Hari and Shyam were partners sharing Profit & Losses in the ratio of
3:3:2:2. Following was the Balance Sheet as on 31st Ashadh, 2069:
Liabilities Rs. Rs. Assets Rs. Rs.
Trade Creditors 23,250 Trade Debtors 24,000
Less: Provision for
Kamal‟s Loan 15,000 Bad Debts (750) 23,250
Capital Accounts: Inventories 15,000
Radha 30,000 Cash at Bank 3,000
Kishan 22,500 52,500 Furniture & Fixtures 6,000
Trade Marks 10,500
Capital Accounts:
Hari 24,000
Shyam 9,000 33,000
90,750 90,750
st
On 31 Ashadh, 2069, the partnership was dissolved.
The assets realized were as follows:
Trade Debtors Rs. 16,500; Inventories Rs. 12,000; Furniture & Fixtures Rs. 1500;
Trade Mark Rs. 6,000; Trade Creditors were settled at Rs. 23,000; also there was a
joint life insurance policy for Rs. 45,000; this was surrendered for Rs. 4,500.
Expenses of realisation amounted to Rs. 750. Hari was insolvent, but Rs. 5,500 was
recovered from his estate.
Show the following accounts in the books of the partnership firm: 20
a) Realisation Account
b) Cash Account
c) Partners‟ Capital Account

ZYA P.T.O.
(2)

Answer:
a)
Realisation Account As on Ashadh 2069
Particulars Amount Amount Particulars Amount Amount
By Provision for Doubtful
To Sundry Assets: Debts 750
Furniture & Fixtures 6000 By Trade Creditors 23250
Trade Marks 10500 By Cash:
Trade Debtors 24000 Furniture & Fixtures 1500
Inventories 15000 55500 Trade Marks 6000
To Cash Trade Debtors 16500
(Payment to
Creditors) 23000 Inventories 12000
To Cash Surrender Value of Policy 4500 40500
(Realization Expense) 750 By Partners Capital Account
(Loss on Realization)
Radha 4425
Kishan 4425
Hari 2950
Shyam 2950 14750

79250 79250
b)
Cash Account
Particulars Amount Amount Particulars Amount Amount
To Balance B/d 3,000 By Realization Account
To Realisation Account 40,500 By Trade Creditors 23,000
(Assets Realized) Realisation Expenses 750 23,750
To Partners Capital By Kamal's Loan 15,000
Radha 4,425 To Partners Capital:
Kishan 4,425 Final Payment
Shyam 2,950 11,800 Radha's Account 17,743
To Hari Capital 5,500 Kishan's Account 13,307 31,050
To Shyam Capital 9,000

69,800 69,800

ZYA P.T.O.
c) Partners Capital Account
(3)
Particulars Radha Kishan Hari Shyam Particulars Radha Kishan Hari Shyam
To balance By balance
B/d - - 24,000 9,000 B/d 30,000 22,500 - -
To
Realization
Account
(Loss) 4,425 4,425 2,950 2,950 By Cash 4,425 4,425 - 2,950
To Hari
Capital 12,257 9,193 - - By Cash - - 5,500 9,000
(Loss of
Capital By Radha
Written off) Capital - - 12,257 -
By Kishan
Capital - - 9,193 -
To Cash A/C 17,743 13,307 - -
34,425 26,925 26,950 11,950 34,425 26,925 26,950 11,950

Working Notes:
1. There was a debit balance of Rs 9,000 in the Shyam‟s Capital and Shyam is a Solvent
Partner; hence, he must bring cash for balance capital
2. Hari is insolvent therefore he is not able to bring cash. The deficiency in his account is borne
by Radha & Kishan in the ratio 4:3 (Capital Ratio) as per Garner & Murray.
Deficiency in Hari Capital =Rs 24000+Rs. 2950-Rs. 5500=Rs. 21450
Borne By Radha=4/7*21450=12257
Borne by Kishan=3/7*21450=9193
Mr. Kamal Loan is paid off in Cash

2.
a) The following information is given to you:
Balance Sheet of Yan Ltd. as on 31st March, 2012
Liabilities Amount Assets Amount
Rs. Rs.
Share Capital: Land & Building 100,000
2,000 14% preference shares of Plant & Machinery 250,000
Rs. 100 each, fully paid up 200,000 Patents 40,000
1,000 Equity shares of Rs. 100 Stock at Cost 55,000
each, Rs. 75 paid up 75,000 Sundry Debtors 110,000
3,000 Equity shares of Rs. 100 Cash at Bank 75,500
each, Rs. 60 paid up 180,000 Profit & Loss Account 83,500
14% Debentures having floating
charge on all assets 100,000
Interest Outstanding 14,000
Creditors 145,000
Total 714,000 Total 714,000
The company went into liquidation on the above date.
Preference dividends were in arrear for two years. The arrears are payable
automatically on liquidation. Creditors include a loan of Rs. 50,000 on the
mortgage of Land & Building. The assets were realized as follows:
Rs.
ZYA P.T.O.
(4)

Land & Building 120,000


Plant & Machinery 200,000
Patents 30,000
Stock 60,000
Sundry Debtors 80,000

The expense of liquidation amounted to Rs. 10,900. The liquidator is entitled to a


commission of 3% on all assets realized except cash and a commission of 2% on
the amounts distributed among unsecured creditors. Preferential creditors amount
to Rs. 15,000. Assume the payment was made on September 30, 2012. Prepare the
Liquidator's Statement of Accounts. 10

b) Digital traders started business on 1st Shrawan 2068 selling one model of digital
cameras on hire purchase. During the year to 31st Ashadh 2069, it purchased
2,000 cameras at a uniform price of Rs. 9,000 and sold 1,900 cameras at a total
selling price under hire purchase agreements of Rs. 15,000 per camera, payable by
an initial deposit of Rs. 4,500 and 10 quarterly installments of Rs. 1,050.
The following trial balance is extracted from the firm's books as at 31st Ashadh
2069:

Particulars Rs. Rs.


Capital 13,602,600
Drawings 4,000,000
Fixed assets 950,000
Purchases 18,000,000
Cash collected from customers 12,540,000
Rent, rates and insurance 500,000
Salaries 2,700,000
General expenses 510,000
Balance at bank 742,600
Accounts payable 1,260,000
27,402,600 27,402,600

The personal accounts of customers are memorandum records (i.e. they are not
part of the double entry system).

The firm prepares its financial statements on the basis of taking credit for profit
(including interest) in proportion to cash collected from customers.

Ignore depreciation of fixed assets, and (6+4=10)


i) Prepare hire purchase income statement for the year ending 31st Ashadh 2069.
ii) Prepare the balance sheet as at 31st Ashadh 2069.

ZYA P.T.O.
(5)

Answer:
a) Liquidator's Statement of Account as at 30th September 2012
Receipts Estimate Value Payments Amount
d Value Realized Paid
Assets realized: Liquidator's remuneration:
Cash 75,500 75,500 3% on NPR 490,000 = 14,700
Debtors 110,000 80,000 2% on NPR 95,000= 1,900 16,600
Stock 55,000 60,000 Expenses of liquidation 10,900
Plant & Machinery 250,000 200,000 Debentures having a floating
Patents 40,000 30,000 charge 100,000
Land & Building: Interest o/s 14,000
Surplus from securities : Interest up to 30th
(100000-50,000) = 50,000 September 2012 (6month) 7,000 121,000
Creditors:
(120000-50000)= 70,000 Preferential 15,000
Other 80,000 95,000
Preferential shareholders 200,000
Arrears of preference dividend 56,000
Equity shareholders:
NPR 15 on 1,000 shares 15,000
NPR 0.25 on 4,000 shares 1,000 16,000
515,500 515,500

b)
Digital Traders
Income Statement for the year ended 31st Ashadh 2069
Particulars Rs. Rs.
Sales (1900 x Rs.15,000) 28,500,000
Less: Cost of sales
Purchases (2000 x Rs.9,000) 18,000,000
Less closing stock (100 x Rs.9,000) (900,000) (17,100,000)
Provision for unrealized profit & interest (WN 1) (6,384,000)
Gross Profit 5,016,000
Less: Other expenses
Rent, rates and insurance 500,000
Salaries 2,700,000
General expenses 510,000 (3,710,000)
Net profit 1,306,000

Digital Traders
Balance Sheet as at 31st Ashadh 2069
Capital & Liabilities Rs. Rs. Rs.
Capital 13,602,600
Add: Net profit for the year 1,306,000
Less: Drawing (4,000,000)
Total 10,908,600
ZYA P.T.O.
(6)

Assets
Fixed Assets 950,000
Current Assets:
Closing stock 900,000
HP accounts receivable 15,960,000
Less: Provision for unrealized profit & interest (6,384,000) 9,576,000
Balance at bank 742,600
11,218,600
Less: Current Liabilities
Accounts payable (1,260,000)
9,958,600
Total 10,908,600

WN 1) Calculation of amount of unrealized profit


Rs.
a) Sales amount 28,500,000
Less: Cash collected from customers 12,540,000
Cash yet to be collected 15,960,000
b) Profit & interest per unit of camera = Rs. 15,000 –Rs. 9,000 = Rs.6,000

c) Unrealized Profit and interest = (15,960,000/28,500,000) x Rs. 6,000 x 1900


= Rs. 6,384,000

3.
a) Following are the extracts of financial statements of a commercial bank:

Particulars Rs. '000


Total risk weighted exposure 62,704,174
Paid up equity share capital 3,012,924
Share Premium 11,849
Proposed bonus equity shares 753,231
Statutory general reserves 1,532,353
Retained earning 19,635
Debenture redemption reserve 630,364
Investment in equity of institutions with financial interests 101,500
Subordinated term debt 1,050,000
General loan loss provision 414,741
Exchange equalization reserve 50,633
Investment adjustment reserve 38,920

Required: (2+1+1+4+2=10)
i) Amount of core capital (Tier I)
ii) Amount of supplementary capital (Tier II)
iii) Amount of total capital fund
iv) Capital adequacy ratios with regard to:
(a) Core capital
(b) Total capital fund
v) Explain whether the bank meets standard capital adequacy ratios requirement.

ZYA P.T.O.
(7)

b) The following information is extracted from the annual report of a limited


company for a year end:
Issued and paid up equity shares of Rs. 100 each Rs. 12,000,000
Net working capital Rs. 6,060,000
Current ratio 1.75 : 1
Liquidity ratio 1.25 : 1
Net fixed assets to shareholders' equity 60%
Gross profit margin 20%
Net profit to share capital 16%
Inventory turnover 6.575 times
Average age of outstanding debtors 2 months
Make appropriate assumptions and prepare an Income Statement and Balance
Sheet for the company. 10
Answer:
a) Solution:
i) Calculation of amount of Core Capital (Tier I)
Particulars Rs.'000
Paid up Equity Share Capital 3,012,924
Share Premium 11,849
Proposed Bonus Equity Shares 753,231
Statutory General Reserves 1,532,353
Retained Earning 19,635
Debenture Redemption Reserve 630,364
less: Investment in equity of institutions with financial interests (101,500)
Total Core Capital 5,858,856

ii) Calculation of amount of Supplementary Capital (Tier II)


Particulars Rs.'000
Subordinated Term Debt 1,050,000
General loan loss provision 414,741
Exchange Equalization Reserve 50,633
Investment Adjustment Reserve 38,920
Total Supplementary Capital 1,554,294

iii) Amount of total capital fund (Rs.'000) = Core Capital + Supplementary Capital
= 5,858,856 + 1,554,294
= 7,413,150

iv) Capital Adequacy Ratios


Core Capital to Total Risk weighted exposure = Core Capital/ Total RWE
= 5,858,856/ 62,704,174
= 0.0934
= 9.34%

Total capital fund to Total Risk weighted exposure = Total capital fund/ Total RWE
= 7,413,150/ 62,704,174
= 0.1182
= 11.82%

ZYA P.T.O.
(8)

v) As per NRB requirement, the core capital to total RWE should be at least 6% and total
capital fund to total RWE should be at least 10%. From the above calculation, it is
evident that the capital adequacy ratios of the bank meet the standard requirement.

b)
Income Statement
Particulars Rs. Rs.
Sales (WN 4) 26,563,000
Less: Cost of goods sold (WN 4) (21,250,400)
Gross profit 5,312,600
Other expenses (balancing figure) (3,392,600)
Net profit (WN 7) 1,920,000

Balance Sheet
Rs. Rs.
Share capital: 12,000,000
120,000 equity shares of Rs.100 each fully paid up
Reserve and Surplus: 3,150,000
Retained earnings (WN 9)
Total 15,150,000
Net fixed assets (WN 8) 9,090,000
Current assets:
Inventory (WN 3) 4,040,000
Debtors (WN 5) 4,427,167
Cash and bank balance (WN 6) 5,672,833
14,140,000
Less: Current liabilities (WN 1) (8,080,000) 6,060,000
Total 15,150,000

Working Notes:
1) Calculation of current assets and current liabilities
Net working capital = Current assets – Current liabilities
or, CA – CL = Rs.6,060,000………………….. (i)
Besides,
Current ratio = 1.75 : 1
or, CA/ CL = 1.75/1
or, CA = 1.75CL
or, CA – 1.75CL = 0 …………………………(ii)

Subtracting eq. (ii) from eq. (i)


0.75CL = Rs.6,060,000
or, CL = Rs.6,060,000/ 0.75
or, CL = Rs.8,080,000
and,
CA = 1.75 x Rs.8,080,000
= Rs.14,140,000

2) Calculation of liquid assets


Liquid assets/ CL = 1.25/ 1
or, Liquid assets/ Rs.8,080,000 = 1.25/ 1
ZYA P.T.O.
(9)

or, Liquid assets = 1.25 x Rs.8,080,000


or, Liquid assets = Rs.10,100,000

3) Calculation of inventory
Since liquid assets exclude inventory from total current assets,
Inventory = Current assets – Liquid assets
= Rs.14,140,000 - Rs.10,100,000
= Rs.4,040,000

4) Calculation of cost of goods sold, gross profit and sales


Inventory turnover = 6.575 times
or, Sales/ Inventory = 6.575
or, Sales = 6.575 x Rs.4,040,000
Sales = Rs,26,563,000

Gross profit = 20% x Rs,26,563,000 = Rs.5,312,600


Cost of goods sold = Rs,26,563,000 - Rs.5,312,600 = Rs.21,250,400

5) Calculation of Debtors
Assumed that all sales were on credit.

Debtors = Sales x Average collection period/ Month in a year


= Rs,26,563,000 x 2/ 12
= Rs.4,427,167

6) Calculation of cash and bank balance


Assumed that liquid assets consisted of debtors and cash and bank balance only.

Cash and bank balance = Liquid assets – Debtors


= Rs.10,100,000 - Rs.4,427,167
= Rs.5,672,833

7) Calculation of Net profit

Net profit = 16% of Share capital


= 0.16 x Rs.12,000,000
= Rs.1,920,000

8) Calculation of Net fixed assets and shareholders' equity


Shareholders equity = Net fixed assets + Net working capital
If Net fixed assets are 60% (given) of shareholders' equity, Net working capital is 40% of
the same.
Therefore,
Net fixed assets = Net working capital x 60/40
= Rs.6,060,000 x 60/40
= Rs.9,090,000

Shareholders' equity = Net fixed assets/ 0.60


= Rs.9,090,000/ 0.60
= Rs.15,150,000

9) Calculation of Reserve and surplus

ZYA P.T.O.
(10)

Shareholders' equity = Share capital + Reserve and surplus


Reserve and surplus = Rs.15,150,000 – Rs.12,000,000
= Rs.3,150,000
4.
a) A fire broke out in the godown of a business house on Shrawan 08, 2068. Goods
costing Rs. 203,000 in a small sub-godown remain unaffected by fire. The goods
retrieved in a damaged condition from the main godown were valued at
Rs. 197,000. The following particulars were available from the books of account:

Stock on the last balance sheet date at 30.03.2068 was Rs. 1,572,000
Purchases for the period from Shrawan 01 to Shrawan 07 were Rs. 3,710,000
Sales during the same period amounted to Rs. 5,260,000
The average gross profit margin was 30% on sales.
The business house has a fire insurance policy for Rs. 1,000,000 in respect of its
entire stock. Assist the accountant of the business house in computing the amount
of claim of loss by fire. 7

b) Goverdhan Ltd. has equity capital of Rs. 2,000,000 consisting of fully paid equity
shares of Rs. 10 each. The net profit for the year ended 31.03.2069 was
Rs. 3,000,000. It has also issued 18,000, 10% convertible debentures of Rs. 50
each. Each debenture is convertible into 5 equity shares. The tax rate applicable is
30%.
Compute the diluted earnings. 3

c) A Ltd. Purchased fixed assets costing Rs. 850 Lakhs on 1.1.2069. This was
financed by foreign currency loan (U.S. Dollars) payable in three equal
instalments. Exchange rates were $1=Rs. 85 and Rs. 88 as on 1.1.2069 and
31.3.2069. First instalment was paid on 31.3.2069. You are required to state, how
these financial transactions would be accounted for? 5
Answer:
a)

Calculation of Amount of Claim Amount Amount


Value of Stock on 8th Shrawan 16,00,000
(Refer W.N.)
Less:
Value of Stock remaining unaffected by fire. 2,03,000
Agreed value of damaged goods 1,97,000 4,00,000
Loss of Stock 12,00,000

Applying Average Clause:


Amount of Claim=Amount of Policy x Loss of Stock
Stock on the date of Fire
=Rs 10,00,000x12,00,000
Rs16,00,000
=Rs 750,000

ZYA P.T.O.
(11)

Working Note:
Memorandum Trading Account for the Period from 1St Shrawan 2068 to 8Th Shrawan2068
Particulars Amount Particulars Amount
To Opening Stock 15,72,000 By Sales 52,60,000
To Purchases 37,10,000 By Closing Stock (Bal. Fig) 16,00,000

To Gross Profit (30% of Sales) 15,78,000

68,60,000 68,60,000
b) Solution:
Particulars Amount
Interest on Debentures @10% for the year 90,000
(18,000 debentures x Rs50x10%)
Less Tax on Interest @ 30% -27,000
63,000
Add: Net Profit for the year ended 31.3.2069 30,00,000
Diluted Earning 30,63,000

c) As per NAS-11: The Effects to Changes in Foreign Exchange Rates, exchange differences
arising on the settlement of monetary items or on reporting an enterprise monetary items at
rates different from those at which they were initially recorded during the period, or reported
in previous financial Statements, should be recognised as income or expense in the period
they arise. Thus exchange differences arising on repayment of liabilities incurred for the
purpose of acquiring fixed assets are recognised as income or expense.

Calculation of Exchange Difference:

Foreign Exchange Loan = Rs 850 Lacs / Rs 85 = $10 Lacs


Exchange Difference = $ 10 Lacs X (88-85) = Rs 30 Lacs

Loss due to exchange difference amounting to Rs 30 Lacs should be charged to Profit & Loss
Account for the year ended 31.03.2069.

5.
a) On 25th Poush 2068, ABC Advertising P. Ltd. obtained advertising rights for
ACC-Football Cup to be held in Baishakh and Jestha 2069 for Rs. 520 Lakhs.
They furnish the following information:
i) The company obtained the advertisements for 70% available time for Rs. 700
Lakhs by 25th Falgun 2068.
ii) For the balance time they got bookings in 20th Chaitra 2068 for Rs. 240 Lakhs.
iii) All the advertisers paid the full amount at the time of booking the
advertisements.
iv) 40% of the advertisements appeared before the public in Baishakh and balance
60% appeared in the month of Jestha.

ZYA P.T.O.
(12)

Calculate the amount of profit/loss to be recognised for the month Baishakh and
Jestha, 2069 as per NAS-7. 5
b) Discuss accounting treatment of repayment of Government Grants as per NAS 10. 5
c) In preparing financial statements of Lotus Limited for the year ended 31.3.2069,
you come across the following information. State with reason, how you deal with
them in the financial statements?
The company invested Rs. 50 Lakhs in 28.04.2069 in the acquisition of another
company doing similar business, the negotiation for which started. 5

Answer:
a) As per NAS-7, Revenue Recognition, in a transaction involving the rendering of services,
performance should be measured either under the completed service contract method or
under the proportionate completion method, which ever relates the revenue to the work
accomplished. Further as per NAS, it states that revenue from advertising should be
recognised when the service is completed. The Service as regards advertisement is deemed to
be completed when the related advertisement appears before the public.
In the given problem 40% advertisements appeared in the month of Baisakh 2069 and
Balance in the month of Jestha 2069.

Total Profit Computed as follows:

Particulars In Lacs
Advertisement for 70% of Available time obtained by
Falgun,2068 700
Advertisement for 30% of available time obtained in by
Chaitra2068 240
Total 940
Less:
Cost of Advertisement Rights -520
Profit 420
The Profit amounting to Rs 420 Lacs should be apportioned in the ratio of 40:60 for the
months of Baisakh & Jestha 2069.
Hence the company should recognise Rs 168 Lacs (Rs 420*40%) & rest Rs 252 Lacs in the
month of Baisakh & Jestha 2069respectively.

b) As per para 32 NAS 10, government grant that becomes repayable shall be accounted for as a
revision to an accounting estimate. Repayment of a grant related to income shall be applied
first against any unamortized deferred credit set up in respect of the grant. To the extent that
the repayment exceeds any such deferred credit, or where no deferred credit exists, the
repayment shall be recognized immediately as an expense. Repayment of a grant related to
an asset shall be recorded by increasing the carrying amount of the asset or reducing the
deferred income balance by the amount repayable. The cumulative additional depreciation
that would have been recognized to date as an expense in the absence of the grant shall be
recognized immediately as an expense.

ZYA P.T.O.
(13)

c) The Investments of Rs 50 Lacs in Ashadh 2069 for acquisition of another company is under
negotiation stage and is not finalized yet. On the other hand, it is also not affecting the
figures stated in the Financial Statements of 2068/69. Hence the details regarding such
negotiation and investment planning of Rs 50 Lacs are indicative of conditions that arose
after the balance sheet date (non-adjusting events after the balance sheet date). Therefore as
per para 10 of NAS 5 lotus limited shall not adjust the amount recognized in its financial
statement.
Since, such event constitute material, lotus limited shall disclose in the financial statement:
i) The nature of event ; and
ii) An estimate of its financial effect
6. Write short notes on:
a) Accounting treatments leading to formation of secret reserve 3
b) Need for revaluation of partnership assets 3
c) Mark-up and margin 2
d) Contingent asset 2
Answer:
a) Following accounting treatment may result in formation of secret reserves within an
organization:
i) Writing off excessive depreciation;
ii) Charging capital expenditures to profit & loss account;
iii) Undervaluation of closing stocks;
iv) Suppression of sales;
v) Showing a contingent liability as an actual liability;
vi) Showing an assets as contingent assets;
vii) Crediting revenue receipt to an assets account

b) The partnership assets need to be revalued mainly in the following situation:


 Admission of a new partner
 Retirement or death of a partner
 Change in profit or loss sharing ratio
 Sale or dissolution of partnership

c) When gross profit is shown as a fraction or percentage of the cost price, it is termed as mark-
up. Besides, when gross profit is shown as a fraction or percentage of the selling price, it is
termed as margin. Both the terms refer to the same gross profit, but expressed as percentage
of different figures.

d) NAS 12 on " Provisions, contingent liabilities and contingent assets" defines a contingent
asset as a possible asset that arises from past events and whose existence will be confirmed
only by the occurrence or non-occurrence of one or more uncertain future events not wholly
within the control of the entity; e.g. a claim that the entity is pursuing through legal
processes, where the outcome is uncertain.

ZYA P.T.O.
(14)

Contingent asset is not recognized in financial statements since this may result in the
recognition of income that may never be realized. It is disclosed where an inflow of
economic benefit is probable.

ZYA P.T.O.
F-Suggested Answer
Audit & Assurance
CAP II, June 2013

Roll No……………. Maximum Marks - 100

Total No. of Questions- 7 Total No. of Printed Pages- 2

Time Allowed - 3 Hours


Marks
Attempt all the questions.

1. As an auditor, give your opinions with reasons on the following cases: (45=20)
a) During the course of audit of a limited company, the auditor detected that the
managing director had committed a fraud involving a loss to the company. And
the managing director has also fully compensated the loss as committed by him
to the company immediately after the detection of fraud.
b) On the basis of approval accounting policy, Bee Limited has revalued its
property and charged Rs. 5 crores revaluation loss in profit and loss account in
2067/68 whereas transferred Rs. 10 crores the revaluation gain to revaluation
reserve in 2068/69.
c) A company wants to adjust the bank balance on the balance sheet date by
reversing the entry for a cheque issued in the normal course of business and
cancelled after the year end but before the finalization of accounts. The cheque
was returned on the ground that the signature differs.
d) Richak & Associates, a chartered accountant firm, was appointed as an auditor
of a company on 15 Aswin 2069 for the year 2068/69 where Ram Lal, one of
the partners of the audit firm, was holding 5% shares of the company since
2065. But Ram Lal sold all his shares of the company on 30 Aswin 2069. It was
further noted that the audit report was signed by another partner of the firm on
15 Falgun 2069 only.
Answer:
a) The detection of a fraud committed by the Managing Director involving a loss to the
company is a serious matter irrespective of the fact that the Managing Director has fully
compensated the company before the end of the fiscal year. In this context, NSA-240 on
“the Auditor responsibility to consider fraud and error in an audit of financial statements”
enumerates responsibilities of the auditor in case fraud and error to exist while
conducting the audit. It requires, first of all that auditor should consider the implications
of the circumstances on the true and fair view which the financial statements ought to
convey and secondly, where the significant fraud has occurred, that auditor should
consider the necessary for a disclosure of fraud in the financial statements and if adequate
disclosure is not made, the necessity for a suitable disclosure in his report.

NSA 240 further defined the duties of the auditor to communicate these matters to the
appropriate level of management on a timely basis, and consider the need to report such
matters to those charged with governance in such case.

When the auditor has obtained evidence that fraud exists or may exist, it is important that the
matter be brought to the attention of an appropriate level of management. This is so even if
RCP P.T.O.
(2)
the matter might be considered inconsequential (for example, a minor defalcation by an
employee at a low level in the entity‟s organization). The determination of which level of
management is the appropriate one is also affected in these circumstances by the likelihood
of collusion or the involvement of a member of management. If the auditor has determined
that the misstatement is, or may be, the result of fraud, and either has determined that the
effect could be material to the financial statements or has been unable to evaluate whether the
effect is material, the auditor: (a) discusses the matter and the approach to further
investigation with an appropriate level of management that is at least one level above those
involved, and with management at the highest level; and (b) if appropriate, suggests that
management consult with legal counsel.

b) NAS 6 paras 39, 40 and 41 are related with the treatment of revaluation of fixed assets. As per
this para, if an asset‟s carrying amount is increased as a result of a revaluation, the increase shall
be credited directly to equity under the heading of revaluation surplus. However, it reverses a
revaluation decrease of the same asset previously recognized in profit or loss.

If an asset‟s carrying amount is decreased as a result of a revaluation, the decrease shall be


recognized in profit or loss. However, the decrease shall be debited directly to equity under
heading of revaluation surplus to the extent of any credit balance existing in the revaluation
surplus in respect of that asset.

The revaluation surplus included in equity in respect of an item of property, plant and equipment
may be transferred directly to retained earnings when the asset is derecognized. This may involve
transferring the whole of the surplus when the asset is retired or disposed of. However, some of
the surplus may be transferred as the asset is used by an entity. In such a case, the amount of the
surplus transferred would be the difference between depreciation based on the revalued carrying
amount of the asset and depreciation based on the asset‟s original cost. Transfers from revaluation
surplus to retained earnings are not made through profit or loss.

Hence in a given case, the revaluation profit of Rs 5 crores is to be credited to profit or


loss account of the year 2068/69, the amount equivalent to revaluation loss which was
charged to profit or loss account in earlier years and Rs 5 crores is to be credited to equity
directly (i.e. revaluation reserve). So, the accounting treatment on revaluation of property,
plant and equipment provided by the company for the year 2068/69 is not in accordance
with the requirement of NAS.

c) According to the NAS-05 “Events After the Balance Sheet Date”, assets and liabilities
should be adjusted for significant events occurring after the balance sheet date that
provide additional evidence to assess estimation of amounts relating to conditions
existing at the balance sheet date. Since the phenomenon of difference in signature
existed on the balance sheet due (though known afterwards); the reversal of the entry
can be made as on the balance sheet date if the amount is material.

d) A person or a firm in which such person has substantial shareholding of the company or a
shareholder holding one percent or more of the paid up capital of the company or his/her
close relative is not qualified for appointment as an auditor as per section 112(1) of the
Companies Act. So, in the present case since Ram Lal, a partner of Richak & Associates,
holds 5% shares of the company, Richak & Associates is disqualified from appointment
as an auditor as on 15 Asoj 2069. But Ram Lal disposes all his shares after appointment
of his firm as an auditor and audit report is signed after his disposal of shares. Despite
this fact as per section 112(4) of the Companies Act since the appointment of auditor is in
contravention of the provisions of the Companies Act, the audit cannot be considered as
valid.
2. Answer the following:

RCP
(3)
a) As an auditor of a small business, how can you assure the internal control
system of that company? 8
b) How would you safeguard your client against the payment for fictitious
purchases? 7
Answer:
a) Internal control is a set of internally generated policies and procedures adopted by the
management of an enterprise which is pre requisitive for an organization‟s efficient and
effective performance. It is thus a primary responsibility of every management to create
and maintained adequate system of internal control appropriate to the size and nature of
the business entity.
The system of internal control as the process designed, implemented and maintained by
those charged with governance, management and other personnel to provide reasonable
assurance about the achievement of an entity‟s objectives with regard to reliability of
financial reporting, effectiveness & efficiency of operations, safeguarding of assets and
compliance with applicable laws & regulations.

The auditor needs to obtain the same degree of assurance in order to given an unqualified
opinion on the financial statements of both small & large entities. However many
controls which would be relevant to large entities are not practical in small business. For
example, in a small business, accounting procedure may be performed by a few persons.
Small business is characterized by the lower number of employees, minimum investment
of capital, small capacity of production, difficulty to segregate the owner and the
management and there is less use of technology. It means the business is producing the
product (if it is a manufacturing industry) manually using very less mechanical tools and
traditional tools.

Like, another characteristic is the simplicity and local products designed to cover up the
local environment. It is highly labour intensive and less machine oriented in nature. Also
the business is dependent with the skill labour instead of installing high value of machine.

Those persons may have both operating and custodial responsibilities and segregation of
function may be missing or severely limited.

Inadequate segregation of duties, may in some cases, is offset by supervisory controls


exercised by the owner. The supervisory function by the owner becomes possible because
of the fact that he has direct personal knowledge of the business and involvement in the
business transactions.
In circumstances where segregation of duties is limited and the evidence of supervisory
controls is lacking the evidence necessary to support the auditors opinion on the financial
information may have to be obtained largely through substantive procedure.

b) The auditor has to be very cautious while verifying the purchases that no payments have
been made for the fictitious purchases. For this purpose, he may have to take the
following actions:
i) He should examine first the internal control system in connection with purchases and
satisfy himself with regard to its effectiveness.
ii) He should ensure that before passing the invoices for payment, they are checked with
the original order, with goods received book and the stock records.
iii) He should inspect the invoices and see that the authorities responsible for passing them
for payment have duly checked them and initialed.
iv) He should test check the invoices to see that dates given in the invoices are for the
period concerned and they have been addressed in the name of the client.

RCP
(4)
v) He should also compare a number of invoices with the records in the goods received
book and stock records.
vi) He can make physical verification of the goods purchased, if a part of it is still in the
stock.
vii) He should also compare the supplier‟s statement with the supplier‟s account.
viii) Postings in the various suppliers‟ accounts should also be checked and compared with
the statement received from them.

3. Give your comments on the following: (35=15)


a) The chief executive officer of a client company has returned your draft
representation letter stating that the directors fail to see why such a letter is
necessary and declaiming to issue the letter.
b) The management of Shri Ram Pvt. Ltd. argued that auditor has not carried out the
work properly citing reason that certain fraud and error were revealed after
issuing audit report, whereas audit report was silent on such fraud & error.
c) Mr. Raj, a fellow member of the Institute of Chartered Accountants of Nepal,
working as manager of Rahul & Associates, a chartered accountant firm, signed
the audit report of Om Ltd. on behalf of Rahul & Associates.
Answer:
a) NSA 580 has defined the management representations while conducting the audit. The
auditor should obtain written representations from management on matters material to the
financial statements when other sufficient appropriate audit evidence cannot reasonably be
expected to exist. The possibility of misunderstandings between the auditor and management
is reduced when oral representations are confirmed by management in writing. Matters which
might be included in a letter from management or in a confirmatory letter to management are
contained in the example of a management representation letter in the Appendix to this NSA.
8. Written representations requested from management may be limited to matters that are
considered either individually or collectively material to the financial statements. Regarding
certain items it may be necessary to inform management of the auditor‟s understanding of
materiality.

If management refuses to provide a representation that the auditor considers necessary, this
constitutes a scope limitation and the auditor should express a qualified opinion or a
disclaimer of opinion. In such circumstances, the auditor would evaluate any reliance placed
on other representations made by management during the course of the audit and consider if
the other implications of the refusal may have any additional effect on the auditor‟s report.

b) Nepal Standards on Auditing (NSA) 240 on „The Auditor's Responsibility to Consider


Fraud and Error in an Audit of Financial Statements‟ deals the matter on auditor
responsibility. NSA 01 has defined the objective of an audit of financial statements and
as per this standard, the auditor should express an opinion whether the financial
statements are prepared, in all material respects, in accordance with an identified
financial reporting framework or relevant practices. An audit conducted in accordance
with NSAs or relevant practices is designed to provide reasonable assurance that the
financial statements taken as a whole are free from material misstatement, whether
caused by fraud or error. The fact that an audit is carried out may act as a deterrent, but
the auditor is not and cannot be held responsible for detection of fraud and error.

The management connotation on charge to auditor will be tenable only if the auditor fails
to carry out his duty under following parameters:

i) Using professional skepticism for understanding the risk of material misstatements


resulting from the errors and frauds in financial statements.

RCP
(5)
ii) Audit preliminaries for gaining knowledge about any susceptibility of misstatements
arising from fraud; Audit planning with audit team, audit enquiry of management;
discussion with charge of corporate governance; identification of fraud risk
factors etc.
iii) Audit Response when fraud risk factors are identified to be present-Considering
modifying substantive procedures to reduce detection risk.
iv) Circumstances indicative of presence of misstatement resulting from fraud,
identification of misstatement.
v) Determining the effect of misstatement in financial statements, tagging it with fraud,
documentation, communication and reporting.
vi) Awareness of auditor‟s responsibility in addition with the responsibility of that charge
of governance.
c) Signature on Audit Report: Section 116 of the Companies Act, 2063 requires that only a
person appointed as the auditor of the company or where a firm is so appointed, the
member who has been authorized by a decision of the partners of such institution, may
sign the auditor‟s report or sign or authenticate any other document of the company
required by law to be signed or authenticated by the auditor. Therefore, Mr. Raj, a fellow
member of the Institute and a manager of M/s Ram & Associates., Chartered
Accountants, cannot sign on behalf of the firm in view of the specific requirements of the
Companies Act, 2063.

4. Answer the following: (35=15)


a) Under what circumstances an independent auditor can issue comfort letter to
his/her client?
b) District Education Office has appointed you as an auditor for some community
schools of Humla District. What special points do you consider while doing
audit of such schools? Explain.
c) Write down ten areas where different accounting policies are applied on
accounting the financial transactions based on the nature of entities.
Answer:
a) A letter from independent auditor including in a preliminary prospectus stating that,
while a full audit has not undertaken, the auditor has done a review sufficient to assure
that financial statement information in the preliminary prospectus is correctly prepared to
the best of the auditor knowledge. The auditor in effort states that, had a full audit been
done, they are comfortable that the audited financial statements would not be materially
different from the ones presented in the preliminary prospectus.
Before or during a new issue, a statement by an auditor stating that, while a full audit has
not been done, a review of the issue's prospectus has revealed nothing inaccurate or
misleading. The comfort letter also states that the auditor is confident that a full audit
would not uncover anything unusual that would negatively affect the issue.
In other word, a letter is given to organizations or persons of interest by external auditors
regarding statutory audits, statements and reports used in a prospectus. The comfort letter
will be attached to the preliminary statements as assurance that it will not be materially
different from the final version.

b) Audit of Education Institutions i.e. Community Schools of Humla District:

The special steps involved in their audit are the following:

RCP
(6)
 Examine the Education Act and Education Rules in these cases as these are the
Community Schools which are under the control of the District Education Office and
note all the provisions affecting accounts. .
 Read through the minutes of the meeting of the School Management Committee, noting
resolutions affecting account to see that these have been duly complied with, specially the
decisions as regards the operation of bank account and sanctioning of expenditure.
 Check names entered in the Students Fee Register for each month or term, with the
respective Class Registers, showing names of student on rolls and test amount of fees and
extra charged; and verify that there operates a system of internal check which ensures that
demands against the student are properly raised.
 Check fees received by comparing counterfoils or receipts granted with entries in the
Cash Book and tracing the collections in the Fee Register to confirm that the revenue
from this source has been duly accounted for.
 Total up the various columns of the Fees Register for each month or term to ascertain that
fees paid in advance have been carried forward and that the arrears that are irrecoverable
have been written off under the sanction of an appropriate authority.
 Check admissions fees with admissions slips signed by the head of the institution and
confirm that the amount has been credited to a Capital Fund, unless the School
Management Committee has taken a decision to the contrary.
 See the free studentship and concessions have been granted by a person authorized to do
so, having regard to the Rules prepared by the Management Committee.
 Confirm that fines for late payment or absence, etc. have been either collected or remitted
under proper authority.
 Confirm that hostel dues were recovered before student's accounts were closed and their
deposits of caution money refunded.
 Verify rental income from landed property with the rent rolls, etc.
 Verify any government or local authority grant with the memo of grant. If any expense
has been disallowed for purposes for grant, ascertain the reasons thereof.
 Report any old heavy arrears on account of fees, dormitory rents, etc. to the Management
Committee.
 Confirm that caution moneys and other deposits paid by students on admission, have
been shown as liability in the balance sheet and not transferred to revenue, unless they are
not refundable.
 See that the investments representing endowment funds for prizes are kept separate and
any income in excess of the prizes has been accumulated and inappropriate securities.
 Check the distribution of scholarship to the students and ensure the same with the receipts
of the students.
 Check the distribution sheet of textbooks, payroll of the PCF teachers with the number of
students.

c) Various Nepal Accounting Standards (NAS) are issued by the Accounting Standard
Board of Nepal which are mandatory in nature and few are recommendatory as well. For
prudent accounting practices; application of NAS should be made on accounting of
financial transactions. Some areas where accounting policies suggested by NAS are:

i) Method of depreciation, depletion and amortization-Straight Line Method, Written


Down Value method.
ii) Accounting Concept: Historical cost convention is to be defined.
iii) Fluctuation of foreign currency and its treatment should be defined.
iv) Valuation of inventories – FIFO, LIFO, weighted average etc.
v) Treatment of goodwill – write off, retain.
vi) Valuation of investment –at cost, market or net realizable value etc.
vii) Treatment of retirement benefits-Actuarial, funded through trust, insurance policy etc.
viii) Recognition of Revenue either on mercantile or accrual basis is defined.
ix) Revaluation of fixed assets and writeoff of fixed assets having certain value say Rs.
1,000 or less.
RCP
(7)
x) Treatment of contingent liabilities.

5. Comment on the following situations/statements. (35=15)


a) Siddhi & Co., a chartered accountant firm is appointed a fund manager of an
INGO situated in Nepal. The INGO has then transferred money in the current
account of Siddhi & Co. for the operation of project activities. Opine on the act
of Siddhi & Co. for:
i) transferring certain money to saving bank account and
ii) charging/ drawing fee due from the client.
b) RB Associates, a firm of chartered accountants has two partners, R and B. The
firm is already holding audit of 130 companies including audit of 90 NGOs. The
firm is offered the audit of Labour Welfare Association.
c) CA. Khari Adhikari heads the internal audit department of Tee limited. The
external audit firm decided to rely on the internal auditor‟s work relevant for
external audit because of the professional competency of the chief of internal
audit of Tee Limited. Later it was found that the external auditor had conducted
audit procedures himself instead of relying on internal auditor‟s work; however
claimed that he should not be made responsible for inappropriate audit opinion
because he had relied on the work of qualified internal auditor.
Answer:

a) Section 12 of the Code of Ethics of the Institute of the Chartered Accountants of Nepal
permits keeping client‟s money for using on legal activities on behalf of client and which
is also permitted by the ToR and the agreement with the client.
i) The act of transferring client‟s money in saving bank account for reasonable
time is valid only when it seems likely that the client‟s monies remain on
current account for a significant period of time and Siddhi & Co. have taken
prior concurrence of the client.
ii) Siddhi & Co., can charge/draw fees due from a client‟s monies provided that
client, after being notified of the amount of such fees, has agreed to such
withdrawal.

b) As per the notification issued by the ICAN :

i) In case of partnership firms of auditor, the ceiling on audit is hundred entities per
partner of the firm.
ii) The maximum number of audits of the public company registered under Company Act
is 15 and number of audits of other entities is 85 per partner.
iii) The firm can taken maximum of two hundred audits (100 × 2), but the number of
Audits of NGOs should not exceed 90. There are two partners – R and B.
iv) The firm is holding 90 audits of NGOs at present.
v) The firm can, therefore, accept audit of Labour Welfare Association assuming that
Labour Welfare Association is registered under the Association Registration Act, 2034.
c) The role of internal auditing is determined by management, and its objectives differ from
those of the external auditor who is appointed to report independently on the financial
statements. The internal audit function‟s objectives vary according to management‟s
requirements. The external auditor‟s primary concern is whether the financial statements are
free from material misstatements. Nevertheless some of the means of achieving their
respective objectives are often similar and thus certain aspects of internal auditing may be
useful in determining the nature, timing and extent of external audit procedures. Internal
auditing is part of the entity. Irrespective of the degree of autonomy and objectivity of
RCP
(8)
internal auditing, it cannot achieve the same degree of independence as required of the
external auditor when expressing an opinion on the financial statements. The external auditor
has sole responsibility for the audit opinion expressed, and that responsibility is not reduced
by any use made of internal auditing. All judgments relating to the audit of the financial
statements are those of the external auditor.

The external auditor has sole responsibility for the audit opinion expressed, and that
responsibility is not reduced by any use made of internal auditing. All judgments relating
to the audit of the financial statements are those of the external auditor. Hence the
external audit firm in the given situation cannot be relieved of its duty to express opinion
on the financial statements by relying on the work of internal auditor.

6. Write short notes on the following: (42.5=10)


a) Detection risk
b) Audit strategy
c) Features of government audit
d) Tolerable error
Answer:
a) Detection risk: Detection risk is the risk that the auditor will not detect a misstatement
that exists in an assertion that could be material, either individually or when aggregated
with other misstatements. Detection risk is a function of the effectiveness of an audit
procedure and of its application by the auditor. Detection risk cannot be reduced to zero
because the auditor usually does not examine all of a class of transactions, account
balance, or disclosure and because of other factors. Such other factors include the
possibility that an auditor might select an inappropriate audit procedure, misapply an
appropriate audit procedure, or misinterpret the audit results. These other factors
ordinarily can be addressed through adequate planning, proper assignment of personnel to
the engagement team, the application of professional skepticism, and supervision and
review of the audit work performed.

b) Audit Strategy
Audit planning is the process of gathering information and design audit strategies. The
main output of audit planning is a tailored audit approach supported by appropriate
administrative arrangements.

Audit strategy is concerned with designing optimized audit approaches that seeks to
achieve the necessary audit assurance at the lowest cost within the constraints of the
information available. Audit procedures should be relevant to the important assertions,
and as cost effective as possible to perform.

Audit strategy generally involves the following steps:


i) Obtaining knowledge of clients business,
ii) Performing analytical procedures at initial stage,
iii) Evaluating inherent risks,
iv) Evaluating internal control system for strategy purpose and
v) Formulating the strategy.

The auditor should also develop the strategy by considering the results of gathering or
updating information about the client, and making preliminary judgment about
materiality, inherent risk and control effectiveness.

RCP
(9)
The initial assessment of the quality and complexity of the client‟s system will affect the
amount of the information the auditor needs to gather. Sometimes, on a new engagement,
the appropriate strategy may be oblivious from a limited amount of investigation work.

c) Features of Government Audit


Citizen of the country are more concern on issues concerning public accountability, including the
misuse of public funds, evaluation of the effectiveness and outcomes of government programs,
information disclosure based on the “right-to-know,” and requests for disclosure of government
financial data through financial statements. To meet these challenges, each government has one
Supreme Audit Institution that is responsible for the public auditing whereas Financial
Comptroller General is responsible to maintain public accounting.

Supreme Audit Institution in Nepal is called the office of the Auditor General (OAGN) who
conduct public audit of the government budget and expenditure. Regarding the aim of auditing,
public audits are carried out as comprehensive audits that include financial auditing and
performance auditing or value-for-money (VFM) auditing.

The aim of audit of government auditing has some audit areas that are completely different from
those concerning the auditing of corporate accounts. In government auditing, the scope of
financial auditing includes audit areas other than the audit of accounts, and performance auditing
includes the evaluation of economy, efficiency, and effectiveness. OAGN also performs the audit
of regularity and propriety.

Government auditing was initially conducted as compliance audits, which means accurate
account auditing or financial auditing. Subsequently, government organizations were required to
provide effective public services by efficient management in the performance of their trusteeship
obligations, which in turn necessitated performance auditing.

As a feature of performance auditing, the Auditing Standards of the International Organization of


Supreme Audit Institutions (INTOSAI) point out that “performance audits should not concentrate
solely on criticism of the past but should be constructive.

d) Tolerable error: Tolerable error is the maximum error that that the auditor would be
willing to accept and still concludes that the result from the sample has achieved the audit
objective. Tolerable error is considered during the planning stage and for substantive
procedures related to the auditor judgment about materiality. The smaller the tolerable
error, the greater the sample size will need‟s be.

In test of control, the tolerable error is the maximum rate of deviation from a prescribed
control by control procedure that at the auditor would be willing to accept, based on the
preliminary assessment of control risk. In substantive procedure, tolerable error is the
maximum monetary error in an account balance or a class of transactions that the auditor
would be willing accept, so that when the result of all audit procedure are reasonable
assurance, that the financial statement are not materially mis-stated.

7. Distinguish between : (25=10)


a) Computerized and manual accounting system
b) Test checking and routine checking
Answer:
a) Distinction between computerized and Manual Accounting System:
i) Faster and efficient in processing of information in computerized system and no such
faster and efficient in processing of information in manual system
ii) Automatic generation of accounting documents like invoices, cheques and statement of
account which manual system cannot produce.

RCP
(10)
iii) With the larger reductions in the cost of hardware and software and availability of user-
friendly accounting software package, it is relatively cheaper like maintaining a manual
accounting system;
iv) More timely information can be produced than manual system
v) No more manual processing of the data- all automatically posted to the various
ledgers/accounts and many types of useful reports can be automatically generated for
management to make decisions where as such reports cannot generated on manual system
vi) Power failure, computer viruses and hackers are the inherent problems of using
computerized systems, such risk not remain in manual system
vii) Once data been input into the system, automatically the output are obtained hence the
data being input needs to be validated for accuracy and completeness, we should not
forget concept of GIGO (Garbage In(Input) Garbage out ( Output) where validation in
manual system can be checked on inception
viii) Accounting system not properly set up to meet the requirement of the business due to
badly programmed or inappropriate software or hardware or personnel problems can
caused more havoc , where manual system does not have such problem.
ix) Danger of computer fraud if proper level of control and security whether internal and
external are not properly been instituted, where manual system does not have such
problem.
b) Test checking and routine checking can be distinguished on the basis of following points:
i) Concept: Test checking involves selecting a few transactions on the basis of auditor‟s
judgment and examining them. But routine checking involves checking of books and
records.
ii) Object: The main object of test checking is to form an opinion on the financial statements
on the basis of examination of selected sample. While the main object of routine
checking is ensuring arithmetical accuracy of the entries in the original books and ledgers
and posting to correct ledgers accounts.
iii) Relationship: Routine checks may be performed on the basis of test checking.

RCP
SUGGESTED
CAP-II, Corporate and Other Laws – June 2013
Roll No……………. Maximum Marks - 100
Total No. of Questions - 7 Total No. of Printed Pages -2
Time Allowed - 3 Hours
Marks
Attempt all questions.
Part "A"
1.
a) Mr. F, an assessee, was a wealthy man earning huge income by way of dividend
and interest. He formed three private companies and agreed with each to hold a
block of investment as an agent for them. The dividend and interest income
received by the companies was handed back to Mr. F as a pretended loan. This
way, Mr. F divided his income into three parts in a bid to reduce his tax
liability.Decide, for what purpose the three companies were established? Whether
the legal personality of all the three companies may be disregarded? 5
b) Mr. B, a member of XYZ Co. Ltd, holding some shares in his own name on which
final call money has not been paid, is denied voting right by the company at its
general meeting on the ground that the Articles of Association do not permit a
member to vote if he has not paid the calls on the shares held by him. Examine the
validity of company's arguments with reference to the provisions of the
Companies Act,1956. 5
c) Explain the doctrine of „Indoor Management‟ in brief.
The Secretary of a Company issued a share certificate to „A‟ under the
Company‟s seal with his own signature and the signature of a Director forged by
him. „A‟ borrowed money from „B‟ on the strength of this certificate. „B‟ wanted
to realise the security and requested the company to register him as a holder of the
shares. Explain whether „B‟ will succeed in getting the share registered in his
name? 5
d) What do you mean by 'ambiguous instruments' and 'inchoate stamped instruments'
under Negotiable Instruments Act, 1881? 5
e) How does a director resign from the board of a private limited company if the
board fails to accept his resignation? Furnish your answer highlighting the
provisionof the Companies Act, 1956. 5

Answer:

a) The House of Lords of England in Salomon Vs Salomon & Co. Ltd. laid down that a
company is a person distinct and separate from its members, and therefore, has an
independent separate legal existence from its members who have constituted the
company. But under certain circumstances the corporate veil may be lifted by the
courts. It means looking behind the corporate façade and disregarding the corporate
entity. Where a company is incorporated and formed by certain persons only for the
purpose of evading taxes, by taking shelter of the corporate nature, the courts have
discretion to disregard the corporate entity in the matter of tax evasion.
(1) The problem asked in the question is based upon the aforesaid principle. The three
companies were formed by the assessee purely and simply as a means of avoiding tax

FOZ P.T.O.
(12)
and the companies were nothing more than the assessee himself. Therefore the whole
idea of Mr. F was simply to split his income into three parts with a view to evade tax.

(2) The legal personality of the three private companies may be disregarded because
the companies were formed only to avoid tax liability and the company was nothing
more than the assessee himself. It did no business, but was created simply as a legal
entity to ostensibly receive the dividend and interest and to handover those over to the
assesse as pretended loans. The same was upheld in Re Sir DinshawManeckji Petit
AIR 1927 Bom.371 and Juggilal vs. Commissioner of Income Tax AIR (1969) SC
(932)

b) Section 87 of the Companies Act, 1956 governs the voting rights of the members.
Every holder of an equity share has the right to vote, by virtue of his shares in the
capital, on every resolution placed before the company. And his voting right on a poll
shall be in proportion to his share of the paid up equity capital of the company.
Member's right to vote may be exercised by him personally or through a proxy.

A company may, by making provision in its articles, restrain a shareholder from


exercising his voting rights in respect of any shares registered in his name on which
any call or other amount due has not been paid or on which the company has a lien or
exercised a lien. In the given case since Mr. B , a member of XYZ Co. Ltd. has not
paid the final call money and furthermore it has been explicitly mentioned in the
articles, therefore the company's argument is valid in the light of the provision of the
Companies Act, 1956.

c) The doctrine of Indoor Management is discussed in the Royal British Bank vs.
Turquand(1956) 6E&B 327. In this case the directors of Royal British Bank also gave
a bond toTurquand. The Article empowered the directors to issue such bonds under
the authority of a proper resolution. In fact no such resolution was passed.
Notwithstanding that, it was held that Turquand could sue on the bonds on the ground
that he was entitled to assume that the resolution had been duly passed. Thus the
persons dealing with the company are entitled to assume that the acts of the directors
or the officers of the company are validly performed, if they are within the scope of
their apparent authority. But this doctrine is not applicable where the person dealing
with the company has notice of irregularity or where the person dealing with the
company is put upon on inquiry or when an instrument purporting to be enacted on
behalf of the company is a forgery.

In the instant problem the doctrine of Indoor Management can apply only in case of
irregularities which might otherwise affect the transaction, but it cannot apply to
forgery which must be regarded as nullity. Hence „B‟ will not succeed in getting the
share registered in his name as this is decided by House of Lord of England in Ruben
v Great Fingall Consolidated [1906] AC 439.

d) Section 17 of Negotiable Instruments Act, 1881, defines the term ambiguous


instrumentsto those instruments which read as 'where an instrument may be construed
either as a promissory note or bill of exchange, the holder may at his election treat it
as either and the instrument shall be thenceforward treated accordingly.'

Similarly, section 20 mentions about inchoate stamped instruments that means 'where
one person signs and delivers to another a paper stamped in accordance with the law
FOZ P.T.O.
(13)
relating to negotiable instruments then in force inIndia, and either wholly blank or
having written thereon an incomplete negotiable instrument, he thereby gives prima
facie authority to the holder thereof to make or complete, as the case may be, upon it
a negotiable instrument, for any amount specified therein and not exceeding the
amount covered by the stamp. The person so signing shall be liable upon such
instrument, in the capacity in which he signed the same, to any holder in due course
for such amount; provided that no person other than a holder in due course shall
recover from the person delivering the instrument anything in excess of the amount
intended by him to be paid thereunder.
e) There are no provisions in the Companies Act, 1956, relating to the resignation of his
office by a director of a company. If there is any provision in the articles of the
company giving the right to a direction to resign at any time, the resignation will take
affect without any need for its acceptance by the Board. Where a director is elected or
has contracted to act for a fix period, his resignation before the expiration of the
period may make him liable for damages, unless the articles permit such resignation.
In the absence of a provision in respect of resignation under the Act or under the
articles of the company, the resignation tendered by a director unequivocally in
writing shall take effect from the time when such resignation is tendered. It is
advisable that the resignation should be in writing and also indicate the time when it
is to take effect. The Directors cannot refuse the resignation of a co-director unless
such a provision is there in the Articles of Association of Company.

Part"B"
2.
a) M & N concluded a contract to supply and purchase 50,000 Lt. petrol as provided
in the contract. M is compelled to be a signatory party and reluctantly make a sign
to the agreement. They are bound to perform respective contractual liability,
however, M is intended to quit from liability because of strike and market price.
Advise him how he can quit or discharge from the contractual liability? Elucidate
your answer. 8
b) Mr.X , a Nepalese citizens, possessing one year experience of operator, applied
for a job of "Press machine operator" in a press company. However, company
denied him to provide job as per his application, alleging he has lack of
efficiency. He knew that Non-Nepalese citizen recruited at that post. His pleading
is that the recruitment is against the legal provisions enshrined in the Labour Act,
2048. Draw your solution on following questions on the basis of mentioned
circumstances and provisions of the Act. 7
i) Whether Non-Nepalese citizen is prohibited to be engaged at works or not?
ii) If it is yes, then, how the recruitment of Non-Nepalese citizen on the job of
press machine operator is possible?
iii) Whether the Act intended for an engagement to Non-Nepalese citizen on that
post for long term. 7

Answer:

a) Solution:

FOZ P.T.O.
(14)
Contract is an agreement concluded between and among parties thereto. They have
right to perform or not to perform the contract if they are upon the consent. Mr. M has
following options to quit from the contract.Section 73, of the Contract Act 2056 has
prescribed following circumstances in which contracts need not be performed:
a. In case one party to the contract absolves the other party fromfulfilling the
obligations according to the contract;
b. In case a voidable contract is made void by the partyconcerned;
c. In case one cannot execute the contract due to its violation bythe other party;
d. In case it becomes unnecessary to perform the work mentionedin the contract
under any provision of this Act;
e. In case it becomes unnecessary to comply with the contractunder Section 79
Section 79, sub-sections 1 prescribed that in case it becomes impossible to execute a
contractas a result of fundamental change in the situation prevailing at the time
ofsigning of the contract, the work under the contract need not be performed. Sub-
section 2 stated that without prejudice to the generality of Sub-section
(1),fundamental change shall be deemed to have come in the situationprevailing at the
time of signing of the contract in any of the followingcircumstances:
a. In case the contract becomes illegal and it cannot beexecuted;
b. In case it becomes impossible to execute the contract due toemergence of such
situations as war, floods landslides, fire,earthquakes, and volcanic eruptions,
which are beyond thecontrol of human beings;
c. In case anything essential for executing the contract isdestroyed or damaged,
or no longer exists, or cannot beobtained;
d. In case the contract has been signed with a provision toprovide services on the
basis of efficiency, skill or talent, andthe person providing such service dies or
loses his/her senseor becomes incapable of performing the contract because
ofphysical or mental disability.
However, sub-section 3 prescribed that notwithstanding anything contained in Sub-
section (2) fundamental changes shall not be deemed to have come in the
situationprevailing at the time of signing the contract in any of the
followingcircumstances:
a. In case it becomes difficult to perform the contract;
b. In case profit margin is low or loss is expected;
c. In case any party to a contract is dependent upon any thirdparty who is not a
party to the contract for performing thecontract, if the third party commits a
mistake or becomesunfit;
d. In the event of strikes and lockouts;
e. In case it becomes necessary to pay additional tax, fee orother revenue;
f. In case the contract has been signed with several objectivesand only some of
them cannot be fulfilled.

Hence A cannot quite the contract because of strike and market price. Nevertheless,
following are the possibilities to him:

FOZ P.T.O.
(15)
Relief or exemption from the liability: Sec 73 (A)
Contractual liability should perform by the parties. If one party wishes to quit
from the liability he should have taken the consent of other party. Here, if N gives
relief or exemption from the liability of the contract, M need not perform the
contract as they agreed upon.
In case of Voidable Contract: Sec 14, 73(B)
Voidable contract either be made void through the court furnishing evidence that
it is coercion, undue influence, misrepresentation and fraud, or perform with
acceptance thereto. So, it gives an option to think about the contract. Here, it is
said that M is compelled to be a signatory party of the contract. In other word, he
is compelled to sign in the contract by means of coercion. If he actually wishes to
quit from the contract he has to file a case to the court submitting evidence of
coercion. If it is proved that N has made coercion to M making him a party to the
contract then he need not perform the contractual liability.
b) Solution
i.
Section 3 of Labor Act, 2048 has directed to Proprietors to classify the job of the
workersand employees of the Enterprise according to the nature of production
process, service or functions of the Enterprise. Yes. Non-Nepalese citizen is
prohibited to be engaged at work in any of the posts classified pursuant to Section 3
of this Act. It assures the right of labour of Nepalese citizen.
ii.
However, an enterprise may engage Non-Nepalese citizens mentioned on the
following:
a) Submission of an application:
An enterprise should make advertisement for the skilled technical post in
national level public newspapers and journal. The manager should submit
an application to the Department of Labour along with evidence of such
fact if Nepalese citizen would not be available for that post.
b) Approval from Department of Labour:
If Department, making an inquiry on an application, found such fact that
Nepalese citizen would not be available for that post, on the
recommendation of the Labour Office, may grant approval to engage Non-
Nepalese citizen at work years for a maximum period of upto five years
not exceeding two years at a time and, in the specialized kind of skilled
technical post, for a period upto seven years.
c) Arrangements of Nepalese citizen to be made skilled and Non-Nepalese
citizen to be replaced.
iii.
This Act has intended not to engage the Non-Nepalese citizen as long termotherthan
maximum of 7 years. Therefore, The Manager should make arrangements for making
Nepalese citizens skilled and for replacing the Non-Nepalese citizens gradually by
them.
3. (2×5=10)
FOZ P.T.O.
(16)
a) Z company is incorporated under Companies Act, 2063. It has transacted duly and
with compliance of concerned law, rules and regulation. It has ability to pay its
debts also. However, appointment of CEO of the company drew attention and
conflicts among directors. Finally, they concluded to liquidate the company. At
this situation, Board of Directors has consulted you about how and at what
circumstances the company may be liquidated and what is the status of company's
directors, authorities and employees?
b) Enumerate the circumstances on which a chairperson and a member of the
Securities Board may be removed from office under the Securities Act, 2063.

Answer:
a) Solution:
Simply, the term 'liquidation' is a process of ending the existence of a company. It
brings about an end to the life of company. It is process or affair or act including the
closing a company down and collecting its assets, distribution of its liabilities etc. So,
it is an act of liquidator that he/she takes control of the company, collects its assets,
pays its debts and finally distributes any surplus among the members in accordance
with their rights.
Z Company is able to pay its debts and not mentioned about the situation of
insolvency. Hence section 126 prescribed provision where the directors must
consider the following before taking to liquidation:
 Directors need to be sure if there exists no pending of insolvency application
or no situation of insolvency proceeding under the prevailing law on
insolvency;
 If the directors of the company, have, after due inquiry, made a declaration in
writing that the company is able to pay its debts and other liabilities in full and
that the debts and liabilities to be paid on behalf of such company can be paid
up or can be fully settled in any other process within one year from the date of
the adoption of the resolution to liquidate the company;
 If the written declaration of mentioned above made by the directors pursuant
was made at the time of discussions on that matter in the general meeting.
Company may liquidate either by adopting a special resolution in the general meeting
or subject to the provisions of mentioned in the memorandum of association, articles
of association or consensus agreement. It is the duty of the company to submit
adopted special resolution and written statement of Directors within 7 days of
resolution adoption. Company may appoint a liquidator to a licensed practitioner after
adopting such special resolution to conduct liquidation proceedings of company.
Further, office should be given information no later than seven days after the date of
such appointment. After the appointment of such liquidator the directors and officers
of the company shall relieved of their office and the liquidator shall exercise all such
powers with respect to the operation and management of company as may be
exercisable by the directors and officers of the company. The service of employees of
a company shall, ipso facto, be terminated after the liquidator commences the
operation and management of the company. However, the liquidator may retain or
appoint necessary employees for his/her support and assistance.

FOZ P.T.O.
(17)
b) Section 12 of Securities Act,2063 mentions different circumstances where Nepal
Government may remove chairperson and a member. These circumstances are as
follows:
a. If one is disqualified to be a Chairperson and a member, as the case may be,
pursuant to Section 11,
b. If one commits any act contrary to the interest of investors in securities or any
act that may cause loss or damage to the development of capital market,
c. If one suffers from lack of competence to implement, or cause to be
implemented, such functions required to be performed by the Board to attain
the objectives of the Board pursuant to this Act or the Rules framed under this
Act,
d. If one has been held disqualified to carry on any occupation or business by the
reason of misconduct and his or her certificate has been revoked or he or she
has thus been restricted to carry on a business,
e. If one remains absent from three consecutive meetings of the Board without
giving a notice

4.
a) Investment Ltd., incorporated under prevailing company and security law, having
objectives to act as a Sales Manager as well as operating Collective Investment
Scheme mentioning in the Memorandum of Association (MOA), has no fulfilled
any other official or legal formalities to operate collective investment scheme.
Advise the company about following question.
i) What are the pre-conditions and procedure to operate Collective Investment
Scheme?
ii) Who and how such scheme is operated?
iii) How the Collective Investment Scheme may be revoked?
5
b) What are the conditions stated in Bonus Act 2030 to receive and restriction to
receive the Bonus. 5
Answer:
a) Solution:

i. Pre-conditions and procedure to operate collective investment scheme:


a. Statutory Requirement of obtaining permission: Sec. 71
It should be obtain permission from the Board constituted in accordance with the
Securities Act, 2063 to operate a collective investment scheme.

b. Approval of Board:
A scheme manager should submit an application to the Board with details and
fees as may be prescribed in order to register such scheme with the Board and
obtain permission to operate the same. After an inquiring on the application if it is

FOZ P.T.O.
(18)
considered appropriate to grant permission, Board shall give permission to
operate it within 90 days from the date of receipt of such an application. Board
may demand of notice, description, information or documents form concerned
scheme manager while it appears necessary. Board may specify necessary terms
and conditions in relation to the operation of the collective investment scheme and
the issue of the certificate on participation.
ii. Scheme Operation:
The scheme manager may operate the collective investment scheme of one or
several types and by one or several names as prescribed with a view to operate a
collective investment scheme. The scheme manager shall, prior to operating any
scheme make an agreement with the depository as prescribed.
The Securities Board may revoke the permission in accordance with the Section
74 of the Securities Act, 2063 on following circumstances:
a) Failure to fulfill the necessary terms and conditions of operation in a
satisfactory manner,
b) It is not appropriate to continue such a scheme upon considering the
interest of participants,
c) Violation of this Act or the Rules or Bye-laws framed under this Act or
provision of any false details to the Board in respect of a scheme by the
scheme manager and the depository.

b) Section 6 of Bonus Act 2030 has made following provision regarding the eligibility
of receive for Bonus:

(1) An employee who has worked for the half period to be worked in a fiscal
year, shall be entitled to obtain bonus under to this Act. Provided that, no
employee shall be entitled to obtain Bonus who has worked casually or in
a shift basis.

(2) For the purpose of Sub-section (1), the following periods shall also be
computed as a period where an employee has worked.
(a) A period kept on reserve under any contract or under Section 11 of the
Labour Act, 2048 (1991).
(b) A period under which an employee is on any leave with salary.
(c) A period of disablement caused by accident arising in course of business
of the enterprise.
However under section 8 of the Act has made restriction to obtain Bonus as under :

Notwithstanding anything contained in Section 6, an employee shall not be entitled to


obtain bonus under this Act, if he/she is punished or dismissed from service for
committing any act as follows:

(a) theft of the property of the enterprise or any damage to such property.
(b) Illegal strike or abetment to other for such strike,
(c) Riots or breaching of discipline.

FOZ P.T.O.
(19)
Provided that, this Section shall not be deemed to be prejudiced to obtain in the case
of the bonus for a period before committing such a punishable act.

5. (2×5=10)
a) How adispute between foreign investors and concerned industry can be solved
under Foreign Investment and Technology Transfer Act, 2049?
b) What are the functions of Nepal Rastra Bank?
Answer:
a) Section 7 of Foreign Investment and Technology Transfer Act,1992 has stated the
way of solving disputes between foreign investors and a concerned industry.
According to it if any dispute arises between a foreign investor or the concerned
industry, the concerned parties shall be required to settle the dispute by mutual
consultations in the presence of Department of Industry. If the dispute could not be
settled through mutual consultation , it shall be settled by an arbitration in accordance
with the prevailing Arbitration Rules of the UNCITRAL. Moreover, the place of
arbitration shall be held in Kathmandu and the Nepalese laws shall be applicable in
the arbitration.

b) To achieve the objectives referred to in Section 4 of Nepal Rastra Bank Act 2058,
Section 5 of the Nepal RastraBank Act, 2058has prescribed the functions, duties and
powers of Nepal Rastra Bank as follows:

i) To issue bank notes and coins;


ii) To formulate necessary monetary policies in order to maintain price stability and
to implement or cause to implement them;
iii) To formulate foreign exchange policies and to implement or cause to implement
them;
iv) To determine the system of foreign exchange rate;
v) To manage and operate foreign exchange reserve;
vi) To issue license to commercial banks and financial institutions to carry on
banking and financial business and to regulate, inspect, supervise and monitor
such transactions;
vii) To act as a banker, advisor and financial agent of Government of Nepal;
viii) To act as the banker of commercial banks and financial institutions and to
function as the lender of the last resort;
ix) To establish and promote the system of payment, clearing and settlement and to
regulate these activities; and
x) To implement or cause to implement any other necessary functions which the
Bank has to carry out in order to achieve the objectives of the Bank under this
Act;

While exercising the powers conferred by this Act or any other prevailing law, the
Bank shall have power to carry out other functions and take actions, which are
incidental thereto. No one shall violate powers conferred on the Bank under this Act.

6. (3×5=15)
FOZ P.T.O.
(20)
a) Mr Rajesh Sharma FCA has charged a fee for representing his client M/S
Annapurna Fabrics Pvt.Ltd in Tax settlement commission on the expected relief
to his client as a result of the settlement commission. Is this arrangement is in
accordance with the Nepal Chartered Accountant Act,2053?
b) As perthe Negotiable Instruments Act, 2034,what is negotiation? Justify the
liability of a bank on the following;
i) Where cheque of doubtful validity?
ii) Where customer has died?
iii) Where customer has become insolvent?
iv) Where customer has become an unsound mind?
c) Who is an executive director? What are the functions, duties & powers of an
executive director mentioned in the ICAN Act?
Answer:
a) In the given question, Mr Rajesh Sharma FCA has entered in to an agreement with
M/S Annapurna Fabrics Pvt Ltd representing him in Tax settlement commission with
charging fee on the basis of expected relief to the client as a result of the settlement
commission. Section 34 of the Chartered Accountant Act 2052 has prescribed the
provision regarding conduct to be observedby the member. Member having obtained
professional certificate shall fully observe this Act or the Rules framed under this Act.

As per subsection 10 of section 34 of the Act, no member having obtained the


professional certificate shall base the remuneration to which he or she is entitled for
his or her work on a percentage of profits or on any other uncertain result and, or any
other.
Hence, the arrangement made by Mr. Rajesh Sharma FCA is not in accordance with
the Nepal chartered Accountant Act 1997. Mr Sharma shall be liable for the
professional misconduct.

b) The transfer of an instrument by one party to another so as to constitute the transferee


a holder is called "negotiation". Section 2d of the Negotiable Instruments Act,
2034has defined that an instrument payable to bearer can be negotiated by simple
delivery. Negotiation is done by delivery (Sec 28) and by indorsement. Every party of
the Negotiable Instrument can indorse or negotiate such instrument except it is
restricted by the Act.

The bank having sufficient funds of the Drawer in the account, properly applicable to
the payment of the Cheque must pay the Cheque, and, in default of such payment
must compensate the Drawer or Holder in due Course for any loss or damage caused
by such default pursuant to this Act.
Liability Where Cheque of doubtful validity:
The validity of a cheque may be in doubt where it is not properly drawn. It may, for
example, not be dated, or the amount in words and figures is different, or its
indorsements are not proper or regular or there is some other confusion about the
cheque. The mandate in the cheque is not clear. However, bank should not make
payments on such doubtful cheque.

FOZ P.T.O.
(21)
Liability where customer has died:
The authority of a banker to pay cheques is terminated when he receives notice of the
customer's death. Payments made before the banker has notice of death are justified.
Bank should not pay or justify his/her death for the payment of cheque.
Liability if customer becomes insolvent:
The banker's authority is also terminated when he has received notice of the
customer's insolvency or of presentation of an insolvency petition against him.
Liability where customer has become an unsound mind:
A person of unsound mind is not competent to contract. A cheque, being a contract of
payment, is suspended during the period of the customer's insanity provided that the
banker has notice of the fact.
c) An executive Director is an authority appointed (in accordnace with the section 38 of
ICAN Act) for the operation of the administrative business of the Institute for a
period of four years and if it desire for reappoint. He/Shemust have experience in
accounting profession as the Executive Director. In the absence of the Executive
Director, council may designate any officer level employee of the Institute to act as
the Executive Director. He has to act as chief executive of the institute and should
accountable to the Council. He/She has to discharge respective duties under the
control and supervision of the Council.

This Act has provided the Functions, Duties and powers of the Executive Director in
its respective section 39 as follows:
a) To act as chief executive of the Institute being accountable to the Council,
b) To carry out day-to-day administrative business of the Institute;
c) To submit annual budget of the Institute to the Council;
d) To maintain and cause to be maintained the books of accounts of the
Institute,
e) To keep in custody of the Members' Register and Register of the Members
holding Certificate of Practice and keep it up-to-date.

7. Write short notes on the followings: (3×5=15)


a) Fundamental principles of insurance
b) Sick Industries
c) Voluntary liquidation of companies

Answer:
a) Solution:
i) Principle of Uberrimaefidei (Utmost Good Faith):
Principle of Uberrimaefidei (a Latin phrase), or in simple english words, the
Principle of Utmost Good Faith, is a very basic and first primary principle of
insurance. According to this principle, the insurance contract must be signed by
both parties (i.e insurer and insured) in an absolute good faith or belief or trust. The
principle of Uberrimaefidei applies to all types of insurance contracts.
ii) Principle of Insurable Interest:
FOZ P.T.O.
(22)
The principle of insurable interest states that the person getting insured must have
insurable interest in the object of insurance. A person has an insurable interest when
the physical existence of the insured object gives him some gain but its non-
existence will give him a loss. In simple words, the insured person must suffer some
financial loss by the damage of the insured object.
iii) Principle of Indemnity:
Indemnity means security, protection and compensation given against damage, loss
or injury. According to the principle of indemnity, an insurance contract is signed
only for getting protection against unpredicted financial losses arising due to future
uncertainties. Insurance contract is not made for making profit else its sole purpose
is to give compensation in case of any damage or loss.
iv) Principle of Contribution:
Principle of Contribution is a corollary of the principle of indemnity. It applies to
all contracts of indemnity, if the insured has taken out more than one policy on the
same subject matter. According to this principle, the insured can claim the
compensation only to the extent of actual loss either from all insurers or from any
one insurer. If one insurer pays full compensation then that insurer can claim
proportionate claim from the other insurers.
v) Principle of Subrogation:
Subrogation means substituting one creditor for another. Principle of Subrogation is
an extension and another corollary of the principle of indemnity. It also applies to
all contracts of indemnity. According to the principle of subrogation, when the
insured is compensated for the losses due to damage to his insured property, then
the ownership right of such property shifts to the insurer.
vi) Principle of Loss Minimization:
According to the Principle of Loss Minimization, insured must always try his level
best to minimize the loss of his insured property, in case of uncertain events like a
fire outbreak or blast, etc. The insured must take all possible measures and
necessary steps to control and reduce the losses in such a scenario. The insured
must not neglect and behave irresponsibly during such events just because the
property is insured. Hence it is a responsibility of the insured to protect his insured
property and avoid further losses.
vii) Principle of CausaProxima (Nearest Cause).
Principle of CausaProxima (a Latin phrase), or in simple english words, the
Principle of Proximate (i.e Nearest) Cause, means when a loss is caused by more
than one causes, the proximate or the nearest or the closest cause should be taken
into consideration to decide the liability of the insurer.
The principle states that to find out whether the insurer is liable for the loss or not,
the proximate (closest) and not the remote (farest) must be looked into.
b) Section 25A of Industrial Enterprises Act,2049 states the provision related to sick
industry as follows:
1. If any industry is being operated in loss for a consecutive period of five years
and its production level is twenty percent or less than twenty percent of the

FOZ P.T.O.
(23)
total production capacity, Government of Nepal may, if it deems necessary,
declare it a sick industry by notification published in the Nepal Gazette.
2. No duty, fee and tax of any kind shall be levied on the machinery imported by
any industry as referred to in Sub-section (1) for the extension and
diversification of such industry.

c) As per section 126 of the Companies Act 2063 voluntary liquidation of the company
may be under the following circumstance:

1. If the company is able to pay its debts or other liabilities in Full.

2. If there exists no situation where an application for the review of insolvency of


the company is pending under the prevailing law on insolvency or where the
company would be in any manner subject to an insolvency proceeding under the
prevailing law on insolvency.

3. If the directors of the company, have, after due inquiry, made a declaration in
writing that the company is able to pay its debts and other liabilities in full and
that the debts and liabilities to be paid on behalf of such company can bepaid up
or can be fully settled in any other process within one year from the date of the
adoption of the resolution to liquidate the company.

4. If the written declaration made by the directors above was presented in the
general meeting called to discuss the matter of liquidation of the company or
such declaration was made at the time of discussions on that matter in the
general meeting.

Expect in case where a company has become insolvent in accordance with the
prevailing law on insolvency, the shareholders of the company may liquidate the
company either by adopting a special resolution in the general meeting or
memorandum of association, articles of association or consensus agreement.A copy
of the special resolution adopted with respect to the liquidation of a company under
the section 126 and a written declaration of directors shall be submitted to the
Office in later than seven days after the date adoption of the resolution.

FOZ P.T.O.
CAP-II, Cost & Management Accounting, June 2013

Roll No……………. Maximum Marks - 100


Total No. of Questions: 6 Total No. of Printed Pages - 4
Time Allowed - 3 Hours
Marks
All questions are compulsory. Working notes should form part of the answer.
Make assumptions wherever necessary.

1. Bridgewater Tyre Company‟s budgeted unit sales for the year 2013 were:
Bike tyres 60,000
Bus tyres 12,500

The budgeted selling price for Bus tyres was Rs. 15,000 per tyre and for Bike tyres
was Rs. 4,500 per tyre. The beginning finished goods inventories were expected to be
2,500 Bus tyres and 6,000 Bike tyres, for a total cost of Rs. 2,00,25,500, with desired
ending inventories at 2,000 and 5,000, respectively, with a total cost of Rs.
1,63,23,900. There was no anticipated beginning or ending work-in-process inventory
for either type of tyres. The standard material quantities for each type of tyre were as
follows:
Bus Bike
Rubber 35 Kgs 15 Kgs
Steel Belts 4.5 Kgs 2.0 Kgs
The purchase prices of rubber and steel were Rs. 150 and Rs. 100 per Kg,
respectively. The desired ending inventories for rubber and steel were 60,000 and
6,000 Kgs, respectively. The estimated beginning inventories for rubber and steel
were 75,000 and 7,500 Kgs respectively.

The direct labor hours required for each type of tyre were as follows:
Molding Department Finishing Department
Bus tyre 0.20 0.10
Bike tyre 0.10 0.05

The direct labor rate for each department is as follows:


Molding Department Rs. 650 per hour
Finishing Department Rs. 750 per hour

Budgeted factory overhead costs for 2013 were as follows:


Particulars Rs.
Indirect Material 85,28,000
Indirect Labour 79,40,000
Depreciation of Building and Equipment 49,16,000
Power and Light 63,00,000
Total 2,76,84,000

Required: (3+3+5+3+6=20)
Prepare each of the following budgets for the year ended 2013:
a) Sales budget
b) Production budget
c) Direct material budget
d) Direct labor budget
e) Cost of goods sold budget.

Answer No. 1
CAY P.T.O.
(25)
a)
Bridgewater Tyre Company
Sales Budget
For the year ended December 31, 2013

Product Unit Sales Unit Selling Price Total Sales


Volume Rs. Rs.
Bike Tyres 60,000 4,500 27,00,00,000
Bus Tyres 12,500 15,000 18,75,00,000
Total 72,500 45,75,00,000

b)
Production Budget
For the year ended December 31, 2013
Units
Bike tyres Bus tyres
Sales (from sales budget) 60,000 12,500
Add: Desired ending inventory, Dec. 31 5,000 2,000
Total 65,000 14,500
Less estimated beginning inventory, Jan. 1 6,000 2,500
Total production 59,000 12,000

c)
Direct Materials Budget
For the year ended December 31, 2013
Direct Materials Total
Rubber (Kgs.) Steel Belts (Kgs.)
Quantities required for production:
Bike tyres:
59,000 × 15 Kgs. 8,85,000
59,000 × 2.0 Kgs. 1,18,000
Bus tyres:
12,000 × 35 Kgs. 4,20,000
12,000 × 4.5 Kgs. 54,000
Add: Desired ending inventory, Dec. 31 60,000 6,000
Total 13,65,000 1,78,000
Less: Estimated beginning inventory, Jan. 1 (75,000) (7,500)
Total quantity to be purchased 12,90,000 1,70,500
Unit price Rs. 150 Rs. 100

Total direct materials purchased Rs. 19,35,00,000 Rs. 1,70,50,000 Rs. 21,05,50,000

d)
Direct Labor Budget
for the year ended December 31, 2013
Department Total
Molding Finishing
Hours required for production:
Bike tyres:
59,000 × .10 5,900
59,000 × .05 2,950
Bus tyres:
12,000 × .20 2,400
12,000 × .10 1,200
Total 8,300 4,150
CAY P.T.O.
(26)

Hourly rate Rs. 650 Rs. 750


Total direct labor cost Rs. 53,95,000 Rs. 31,12,500 Rs. 85,07,500

e)
Cost of Goods Sold Budget
for the year ended December 31, 2013
Rs.
Direct materials inventory Jan. 1(W. N. 1) 1,20,00,000
Direct materials purchases 21,05,50,000
Total direct materials available 22,25,50,000
Less: Direct materials inventory, Dec. 31 (W. N. 1) 96,00,000
Cost of direct materials used 21,29,50,000
Direct labor 85,07,500
Factory overhead 2,76,84,000
Cost of goods manufactured 24,91,41,500
Add: Finished goods inventory, Jan.1 2,00,25,500
Cost of goods available for sale 26,91,67,000
Less: Finished goods inventory, Dec. 31 1,63,23,900
Cost of goods sold 25,28,43,100

Working notes

W.N.: Direct material inventory (beginning)

Rubber 75,000 Kgs. × 150 Rs. 1,12,50,000


Steel belts 7,500 Kgs. × 100 7,50,000
Rs. 1,20,00,000

W.N.2 Direct material inventory (ending)


Rubber 60,000 Kgs. × 150 Rs. 90,00,000
Steel belts 6,000 Kgs. × 100 6,00,000
Rs. 96,00,000

2.
a) A manufacturing company has pre-determined overhead recovery rates at 200%
of the direct wages for works expense, 10% of works cost as management
expenses and 20% on cost of production towards selling and distribution
expenses. At the year-end it is found that works overhead stand under- absorbed
to the extent of 20% of direct wages, management expenses show under-
recovery of 10% of the absorbed amount and selling and distribution expenses
recovery resulted in over absorption of 30% of the absorbed amount.

Direct cost and selling price of the job X, Y and Z is given below.
Job X Job Y Job Z
Direct materials (Rs.) 50 40 30
Direct wages (Rs.) 30 25 20
Selling price (Rs.) 200 160 120

Find the profit or loss on the respective selling price both on the pre-
determined cost and on the basis of full absorption of overheads. 10

b) PQ Limited plans to start a lodging house at a tourist center with a capacity of


32 single occupancy rooms. Cost per day has been estimated as under:

CAY P.T.O.
(27)
Cost per day per room
(Rs.)
When occupied:
(i) Electricity and utilities 4
(ii) Linen, laundry and sanitary supplies 9
When unoccupied:
(iii) Dusting, sweeping and cleaning 2
15
Over and above these costs, the following expenses represent the estimate of
fixed charges per annum (365 days)
Staff expenses Rs. 3,20,000
Other office expenses` Rs. 64,000
Taxes, insurance, maintenance and depreciation Rs. 42,320

PQ Limited defines 100% occupancy to mean all the 32 rooms to fetch revenue
for all 365 days.

You are required to answer the following, using a planning period of one year:
(4+4+2=10)
(a) What would be the tariff per day per room in order to reach break-even at an
occupancy level of 50%?
(b) What would be the profits, if the occupancy level reaches (i) 60% (ii) 70%
and (iii) 80% respectively?
(c) What would be the profits, if the tariff per day is reduced by 10% from
the answer in (a) above and the occupancy level is 100%?

Answer No. 2
a)
Statement of Cost of Production and Profit or Loss
Under Pre-determined Cost Basis
(Amount in Rs.)
Job X Job Y Job Z
Direct materials 50.00 40.00 30.00
Direct wages 30.00 25.00 20.00
Prime cost 80.00 65.00 50.00
Works expenses [200% of direct wages] 60.00 50.00 40.00
Works cost 140.00 115.00 90.00
Management expenses [10% of works cost] 14.00 11.50 9.00
Cost of production 154.00 126.50 99.00
Selling & distribution expenses [20% of cost of production] 30.80 25.30 19.80
Total costs 184.80 151.80 118.80
Profit (Balancing figure) 15.20 8.20 1.20
Selling price 200.00 160.00 120.00

Statement of Cost of Production and Profit or Loss


Under Full Absorption of Overheads Basis
(Amount in Rs.)
Job X Job Y Job Z
Direct materials 50.00 40.00 30.00
Direct wages 30.00 25.00 20.00
CAY P.T.O.
(28)
Prime cost 80.00 65.00 50.00
Works expenses [220% of direct wages] 66.00 55.00 44.00
Works cost 146.00 120.00 94.00
Management expenses [Working note 1] 15.40 12.65 9.90
Cost of production 161.40 132.65 103.90
Selling & distribution expenses [Working note 2] 21.56 17.71 13.86
Total costs 182.96 150.36 117.76
Profit (Balancing figure) 17.04 9.64 2.24
Selling price 200.00 160.00 120.00

Working notes:
1. Management expenses:
Job X Job Y Job Z
Rs. Rs. Rs.
Amount on pre-determined basis 14.00 11.50 9.00
Add: 10% for under absorption 1.40 1.15 0.90
Actual expenses 15.40 12.65 9.90
2. Selling & distribution expenses:
Job X Job Y Job Z
Rs. Rs. Rs.
Amount on pre-determined basis 30.80 25.30 19.80
Less: 30% for over absorption 9.24 7.59 5.94
Actual expenses 21.56 17.71 13.86

b)
(a) Tariff to break-even at 50% occupancy level
Fixed cost Rs. 4,26,320
Expenses when unoccupied Rs. 2 x 11,680 23,360
Expenses when occupied Rs. 13 x 5,840 75,920
5,25,600
Tariff per day to break-even: Rs. 5,25,600 /5,840 = Rs. 90

(b) Profit at various occupancy level

Contribution margin = (Rs. 90 – Rs. 13)= Rs. 77


Profit= (Man-days occupied – BEP man-days) x Contribution margin
Therefore,
Profit at 60% occupancy level =(7,008-5,840) x Rs. 77 = Rs. 89,936
Profit at 70% occupancy level =(8,176-5,840) x Rs. 77 = Rs. 179,872
Profit at 80% occupancy level =(9,344-5,840) x Rs. 77 = Rs. 269,808

(c) Contribution margin at reduced tariff = (0.90 x 90-13) = Rs. 68


Profit at 100% occupancy level = Contribution- Fixed Costs
= 11680 x Rs. 68- Rs, 426,320
= Rs. 367,920

Working Note:
100% occupancy = 32 x 365 days =11,680 room-days
50% occupancy = 0.5 x 11680 = 5840 room-days
60% occupancy = 0.6 x 11680 = 7008 room days
70% occupancy = 0.7 x 11680 = 8176 room-days
80% occupancy =0.8 x 11680 = 9344 room-days

CAY P.T.O.
(29)
3.
a) From the following information for the month of October 2012, prepare Process
III cost account:

Opening WIP in Process III : 1,800 units at Rs. 27,000


Transfer from Process II : 47,700 units at Rs. 536,625
Transferred to warehouse : 43,200 units
Closing WIP of Process III : 4,500 units
Units scrapped : 1,800 units
Direct material added in Process III : Rs. 177,840
Direct Wages : Rs. 87,840
Production Overheads : Rs. 43,920

Degree of Completion:
Opening Stock Closing Stock Scrap
Material 80% 70% 100%
Labour 60% 50% 70%
Overheads 60% 50% 70%

The normal loss in the process was 5% of the production and scrap was sold at
Rs. 6.75 per unit. 15

b) Shyam Enterprises operating an integral system of accounting. The following


transactions incurred for the year end 2012.

Transaction Amount (Rs.)


Raw material Purchased (40% in cash) 10, 00,000
Material issued to production 6, 00,000
Wages paid (50% Direct) 2, 00,000
Wages charged to production 1, 20,000
Factory Overhead paid 1, 20,000
Factory Overhead charged to Production 110,000
Selling and distribution overhead paid 30,000
Finished goods finalized at cost 6, 50,000
Sales (70% in credit) 11, 00,000
Closing stock of finished goods -
Payment received from Customer 3, 00,000
Paid to supplier 5, 00,000

You are required to pass journal Entries in the books of Shyam Enterprises
under integrated system of accounting for the period ended 2012. 5

Answer No. 3
a)
Statement of Equivalent Production
(Process III)
Equivalent production
Input Output Material Material Labour and
Received from added in overheads

CAY P.T.O.
(30)
Process II Process III
Details Quantity Quantity Quantity % Quantity % Quantity %
Units units units units units
Op WIP 1800 Work on Op. WIP 1,,800 - - 360 20 720 40
Process II 47700 Introduced and 41400 41,400 100 41,400 100 41,400 100
Transfer completed during the
month (Bal. fig.)
Normal Loss (5% of 2,250 - - - - - -
45,000 units)
Closing WIP 4,500 4,500 100 3,150 70 2,250 50
49,950 45,900 44,910 44,370
Abnormal gain -450 -450 100 -450 100 -450 100
Total 49,520 49,500 45,450 44,460 43,920

Working Note
(i) Production units = Opening units + Units transferred from Process II – Closing Units
= 1800 units + 47,700 units -4500 units
= 45000 units
(ii) Abnormal gain(units) = Production-Normal Loss-Transfer to warehouse
= 45,000-(5% of 45,000)-43,200
= 450
Statement of Cost
Cost Rs. Equivalent units Cost per equivalent unit Rs.
Material Received from Process II 536,625
Less: Scrap value of normal loss 15,187.50
(2,250 units x Rs. 6.75)
521,437.50 45,450 11.4728
Material added in Process III 177,840 44,460 4.0000
Labour 87,840 43,920 2.0000
Overheads 43,920 43,920 1.0000
8,31,037.50 18.4728

Statement of Apportionment of Process Cost


Rs.
Opening WIP Material From Process II 27,000
Completed opening WIP - Material added in Process III 360 units x Rs. 4=Rs. 1440
1,800 units
Wages 720 units x Rs. 2= Rs. 1440
Overheads 720 units x Rs. 1= Rs. 720 3,600
Introduced and completed 41400 units x Rs. 18.4728 7,64,773
41,400 units
Total cost of 43,200 finished 7,95,373
goods units
Closing WIP -4,500 units Material from Process II 4,500 units x Rs. 11.4728 51,628

CAY P.T.O.
(31)
Material in process III 3,150 units x Rs. 4 12,600
Wages 2,250 units x Rs.2 4,500
Overheads 2,250 units x Re.1 2,250
70,978
Abnormal gain -450 units 450 units x Rs. 18.4728 8,313

Process III A/c


Units Rs. Units Rs.
To Balance b/d 1800 27,000 By Normal Loss 2250 15,187
To Process II a/c 47700 5,36,625 By Finished goods stock 43200 7,95,373
To Direct Material 1,77,840
To Direct Wages 87,840
To Production overheads 43,920 By Closing WIP 4500 70,978
To Abnormal gain 450 8,313
49,950 881,538 49,950 881,538

b) Journal Entries in the books of Shyam Enterprises under integrated system of


accounting for the period ended 2012.

Store Ledger Control A/C Rs.10,00,000


To Sundry Creditors A/C Rs.6,00,000
To Cash/Bank A/C Rs.400,000
(Material purchased)
.............

Work-in-Progress Control A/C Rs.6,00,000


To Store Ledger Control A/C Rs.6,00,000
( Material issued to production)
.............

Wages Control A/C Rs.200,000


To Cash/Bank A/C Rs.200,000
(Wages Paid)
.............

Work-in-Progress Control A/C Rs.1,20,000


To Wages Control A/C Rs.1,20,000
(Wages charged to production)
.............

Factory Overhead Control A/C Rs.120,000


To Cash/Bank A/C Rs.120,000
( Factory overhead paid)
.............

Work –in Progress Control A/C Rs.110,000


To Factory Overhead Control A/C Rs.110,000
(Factory overhead charged to production)
.............

CAY P.T.O.
(32)

Selling and Distribution Overhead Control A/C Rs.30,000


To cash/Bank A/C Rs.30,000
(Selling/distribution overhead paid)
.............

Finished Stock Ledger Control A/C Rs.650,000


To Work-in-progress Control A/C Rs.650,000
(Cost of finished goods transferred from work in progress)
.............

Cost of Sales A/C Rs.6,80,000


To Finished Stock Ledger Control A/C Rs.650,000
To Selling and Distribution control A/C Rs.30,000
.............

Sundry Debtors Account Rs.770,000


Cash/Bank A/C Rs.330,000
To Sales Control Account Rs.11,00,000
(Finished stock sold)
.............

Cash/Bank Account Rs.300,000


To Sundry Debtors A/C Rs.300,000
( Amount received from customer)
.............

Sundry Creditors A/C Rs.500,000


To Cash/Bank A/C Rs.500,000
( Payment made to creditors)
.............

4.
a) The cost structure of an article the selling price of which is Rs. 45,000 is as
follows:
Direct Material 50%
Direct Labour 20%
Overheads 30%
An increase of 15% in the cost of direct materials and of 25% in the cost of direct
labour is anticipated. These increased costs in relation to the present selling price
would cause a 25% decrease in the amount of present profit per article.
Required: (5+3=8)

i) Prepare a statement of profit per article at present, and


ii) Calculate the revised selling price to produce the same percentage of profit
to sales as before.

b) Both direct and indirect labor of a department in a factory is entitled to production


bonus in accordance with a Group Incentive Scheme, the outlines of which are as
follows:
i) For any production in excess of the standard rate fixed at 10,000 tons per
month (of 25 days) a general incentive of Rs. 10 per ton is paid in aggregate.
The total amount payable to each separate group is determined on the basis of
an assumed percentage of such excess production being contributed by it,
CAY P.T.O.
(33)
namely @ 70% by direct labor, @ 10% by inspection staff, @ 12% by
maintenance staff and @ 8% by supervisory staff.
ii) If the excess production is more than 20% above the standard, direct labor
also gets a special bonus @ Rs. 5 per ton for all production in excess of 120%
of standard.
iii) Inspection staff is penalized @ Rs. 20 per ton for rejection by customer in
excess of 10% of production
iv) Maintenance staff is penalized @ Rs. 20 per hour of breakdown.
From the following particulars for a month work out the production bonus earned
by each group:

(i) Actual working days : 20


(ii) Production : 11,000 tons
(iii) Rejection by customers : 200 tons
(iv) Machine breakdown : 40 hours 8
c) Discuss briefly the principles to be followed while taking credit for profit on
incomplete contracts. 4

Answer No. 4
a)
Let the total cost of the article be "X".
Now,
Present condition Revised condition
Direct Material 0.5x 0.575x
Direct Labour 0.2x 0.250x
Overheads 0.3x 0.300x
Total 1.0x 1.125x
Selling Price Rs. 45,000 Rs. 45,000
Profit (Rs. 45,000 – x) (Rs. 45,000 – 1.125x)

From the above exercise, following equation can be made:


(Rs. 45,000 – x) – (Rs. 45,000 – 1.125x) = 25% of (Rs. 45,000 – x)
Or, -x + 1.125x = Rs. 11,250 – 0.25x
Or, 0.375x = Rs. 11,250
Or, x = Rs. 30,000.

Statement of profit per article under present condition


Rs.
Direct Material Rs. 30,000 × 0.5 15,000
Direct Labour Rs. 30,000 × 0.2 6,000
Overheads Rs. 30,000 × 0.3 9,000
Total Cost 30,000
Profit (balancing figure) 15,000
Selling Price 45,000
Percentage of profit to cost [Rs. 15,000 / Rs. 30,000 × 100] 50%
Percentage of profit to selling price [Rs. 15,000 / Rs. 45,000 × 100] 33.33%

Statement of profit per article under revised condition


Rs.
Direct Material Rs. 30,000 × 0.575 17,250
Direct Labour Rs. 30,000 × 0.250 7,500
Overheads Rs. 30,000 × 0.300 9,000
Total Cost 33,750
Profit (50% of cost or 33.33% of selling price) 16,875
Selling Price 50,625
CAY P.T.O.
(34)

b)
(i) No. of working days during month : 20
(ii) Standard production for 20 days @ 10,000 tons per month of 25 days
= 10,000 x 20/25 = 8,000 tons
(iii)Actual production during the month = 11,000 tons
(iv) Excess production during the month = 11,000 – 8,000 = 3,000 ton
(v) Excess production above 20% of standard : 3,000 – 20% of 8,000
= 3,000 – 1,600 = 1,400 tons

Statement showing Bonus earned by each category of staff:


Category General Incentive @ Rs. Special Incentive Penalty Bonus
10 per ton @ Rs. 5 per ton earned
% Tons Amount Tons Amount Rs Rs
(a) Direct labor 70 2,100 21,000 1,400 7,000 - 28,000
(b) Inspection Staff 10 300 3,000 - - -* 3,000
(c) Maintenance staff 12 360 3,600 - - 800** 2,800
(d) Supervisory staff 8 240 2,400 - - - 2,400
Total 100 3,000 30,000 1,400 7,000 800 36,200
* Penalty for rejection: Not applicable (Actual rejection is less than allowed level)
** Penalty for machine breakdown for 40 hours @ Rs. 20 per hour.

c)
Under Contract Accounting it may be noticed that certain contracts are completed, while others
are still in progress at the end of a financial year. These incomplete contracts may require a few
more years for their completion. The figures of profit made (the excess of credit over the debit
items in a contract) on completed contracts can be safely taken to the credit of Profit and Loss
Account, but this practice is not being followed in the case of incomplete contracts.

In the case of incomplete contracts the entire profit is not being credited to Profit and Loss
Account because some provision is to be made for meeting contingencies and unforeseen losses.
There are no hard and fast rules regarding the calculation of figure of profit to be taken to the
credit of profit and loss account. However, the following principles may be followed:–

i. Profit should be considered in respect of work certified and uncertified work should be
valued at cost.

ii. If the amount of work certified is less than 1/4th of the contract price, no profit should be
taken to Profit and Loss Account. The entire amount in such contracts should be kept as
reserve for meeting out contingencies.

iii. If the amount of work certified is 1/4th or more but less than 1/2 of the contract price,
then 1/3rd of the profit disclosed as reduced by the percentage of cash received from the
contractee should be taken to the Profit and Loss Account. The balance should be
allowed to remain as a reserve.
iv. If the amount of work certified is 1/2 or more of the contract price, then 2/3rd of the
profit disclosed as reduced by the percentage of cash received from the contractee, should
be taken to the Profit and Loss Account. The balance should be treated as reserve.
v. If the contract is near completion, the total cost of completing the contract may be
estimated if possible. By deducting the total estimated cost from the contract price, the
estimated total profit of the contract should be calculated. The proportion of total
estimated profit on cash basis, which the work certified bears to the total contract price
should be credited to profit and loss account.
vi. The entire loss, if any, should be transferred to the Profit and Loss Account.

CAY P.T.O.
(35)
5. Distinguish between: (4×2.5=10)
a) Cost Centre and Cost Unit
b) Job Costing and Batch Costing
c) Forecast and Budget
d) Opportunity Cost and Sunk Cost

Answer No. 5
a) Cost Centre and Cost Unit
Cost centre is defined as a location, person or an item of equipment (or group of these) for
which cost may be ascertained and used for the purpose of cost control. Cost centres are of
two types, viz. Personal and Impersonal Cost Centre. A personal cost centre consists of a
person or group of persons and an impersonal cost centre consists of a location or an item of
equipment (or group of these).

Cost unit is a unit of product, service or time (or combination of these) in relation to which
costs may be ascertained or expressed. We may, for instance, determine the cost per tonne of
steel, per tonne kilometre of a transport service or cost per machine hour. Sometimes, a
single order or contract constitutes a cost unit. A batch which consists of a group of identical
items and maintains its identity through one or more stages of production may also be
considered as a cost unit.

b) Job Costing and Batch Costing

Job Costing :The system of job costing is used where production is not highly repetitive and
in addition consists of distinct jobs so that the material and labor costs can be identified by
order number. This method of costing is very common in commercial foundries and drop
forging shops and in plants making specialized industrial equipment. In all these cases, an
account is opened for each job and all appropriate expenditure is charged thereto.

Batch Costing: This method is employed where orders or jobs are arranged in different
batches after taking into account the convenience of producing articles. The unit of cost is a
batch or a group of identical products instead of a single job order or contract. This method is
particularly suitable for general engineering factories which produce components in
convenient economic batches and pharmaceutical industries.

c) Difference between Forecast and Budget


Forecast Budget
i. Forecast is merely an estimate of what is i. Budget shows the policy and programme
likely to happen. It is a statement of to be followed in a period under planned
probable events which are likely to happen conditions.
under anticipated conditions during a ii. A budget is a tool of control since it
specified period of time. represents actions which can be shaped
ii. Forecasts, being statements of future according to will so that it can be suited
events, do not connote any sense of control. to the conditions which may or may not
iii. Forecasting is a preliminary step for happen.
budgeting. It ends with the forecast of iii. It begins when forecasting ends.
likely events. Forecasts are converted into budget.
iv. Forecasts are wider in scope and it can be iv. Budgets have limited scope. It can be
made in those spheres, also where budgets made of phenomenon capable of being
cannot interfere. expressed quantitatively.

d) Opportunity Cost and Sunk Cost

CAY P.T.O.
(36)
Opportunity cost refers to the value of sacrifice made or benefit of opportunity forgone in
accepting an alternative course of action. For example, a firm financing its expansion project
by withdrawing money from its bank deposits. In such a case the loss of interest on the bank
deposit is the opportunity cost for carrying out the expansion project.

Historical costs incurred in the past are known as sunk costs. They play no role in decision
making in the current period. For example, in the case of a decision relating to the
replacement of a machine, the written down value of the existing machine is a sunk cost and
therefore not considered.

6. Write short notes on: (4×2.5=10)


a) Budgetary control
b) Circumstances under which cost audit is ordered
c) Advantages of ABC analysis
d) Objectives of Uniform Costing

Answer No. 6
a) Budgetary control is a methodical control of an organization‟s operations through
establishments of standards and targets regarding income and expenditure and a continuous
monitoring and adjustment of performance against them. Budgetary control is the
establishment of budgets relating the responsibilities of executives to the requirements of a
policy, and the continuous comparison of actual with budgeted results, either to secure by
individual action the objectives of that policy or to provide a firm basis of its revision.

The objectives of budgetary control are:


 Definition of Goals: Portraying with precision, the overall aims of the business and
determining targets of performance for each section or department of the business.
 Defining Responsibilities: Laying down the responsibilities of each individual so that
everyone knows what is expected of him and how he will be judged.
 Basis for Performance Evaluation: Providing basis for the comparison of actual
performance with the predetermined targets and investigation of deviation, if any, of
actual performance and expenses from the budgeted figures. It helps to take timely
corrective measures.
 Optimum use of Resources: Ensuring the best use of all available resources to maximize
profit or production, subject to the limiting factors.
 Co-ordination: Coordinating various activities of the business and centralizing control,
and also facilitating for the management to decentralize responsibility and delegate
authority.
 Planned action: Engendering a spirit of careful forethought, assessment of what is
possible and an attempt at it. It leads to dynamism without recklessness. It also helps to
draw up long range plans with a fair measure of accuracy.
 Basis for policy: Providing a basis for revision of current and future policies.

Disadvantages / Limitation
 Estimates: Budgets may or may not be true, as they are based on estimates. The
assumptions about future events may or may not actually happen.
 Rigidity: Budgets are considered as rigid document. Too much emphasis on budgets may
affect day-to-day operations and ignores the dynamic state of organizational functioning.
 False Sense of Security: Mere budgeting cannot lead to profitability. Budgets cannot
beexecuted automatically. It may create a false sense of security that everything has been
taken care of in the budgets.
 Lack of co-ordination: Staff co-operation is usually not available during budgetary
control exercise.

CAY P.T.O.
(37)
 Time and Cost: The introduction and implementation of the budgetary control system
may be expensive.

b) Discuss the circumstances under which a Cost Audit is ordered along with the purpose of
Cost Audit.

Circumstances under which Cost Audit is ordered


i. Price Fixation – The need for fixation of retention price in case of materials of
national importance like steel, cement etc., may cause a necessity for cost audit.
ii. Cost variation within an industry: Cost audit may be necessary to find reasons for
such differences.
iii. Inefficient Management – Where a factory is run inefficiently and uneconomically,
institution of cost audit may be necessary.
iv. Tax assessment – Where a duty or tax is levied on products based on cost of
production, the levying authorities may ask for cost audit to determine the correct cost
of production.
v. Trade disputes: cost audit may be useful in settlement of trade disputes about claim
for higher wages, incentives etc.

Purposes of Cost Audit


i. Protective purpose: To examine that there is no undue wastage or losses and the
costing system brings out the correct cost of production or processing.
ii. Constructive purpose – Cost Audit provides information to management useful in
regulating production, choosing economical methods of operation and reducing
operations cost.

c) Advantages of ABC analysis


Followings are the advantages of ABC analysis system:
(i) It ensures that, without there being any danger of interruption of production for want of
materials or stores, minimum investment will be made in inventories of stocks of
materials or stocks to be carried.
(ii) The cost of placing orders, receiving goods and maintaining stocks is minimized
especially if the system is coupled with the determination of proper economic order
quantities.
(iii)Management time is saved since attention need be paid only to some of the items rather
than all the items as would be the case if the ABC system was not in operation.
(iv) With the introduction of the ABC system much of the work connected with purchases can
be systematized on a routine basis to be handled by subordinate staff.

d) Objectives of Uniform Costing


(i) To facilitate the comparison of costs and performance of different units in the same
industry; it provides objective basis.
(ii) To eliminate unhealthy competition among the different units of an industry.
(iii)To improve production capacity level and labour efficiency by comparing the production
costs of different units with each other.
(iv) To provide relevant cost information or data to the government for fixing and regulating
prices of the products.
(v) To bring standardization and uniformity in the operation of participating units.
(vi) To reduce production, administration, selling & distribution costs, and to exercise control
on fixed costs.

CAY P.T.O.
CAP-II, Financial Management, June 2013
Suggested Answer

Roll No……………. Maximum Marks - 100


Total No. of Questions – 7 Total No. of Printed Pages – 4
Time Allowed – 3 Hours
Marks
Attempt all questions.

Working notes should form part of the answer. Make assumptions wherever necessary.

1. BRT Co. has developed a new confectionery line that can be sold for Rs. 5 per box
and that is expected to have continuing popularity for many years. The finance
manager of the company proposed that the investment in the new product should be
evaluated over a four year time horizon, even though sales would continue after the
fourth year on the ground that cash flows after fourth years are too uncertain to be
included in the evaluation.
The average variable and fixed costs will depend on sales volume as follows:
less than 1 1–1·9 2–2·9 3–3·9
Sales volume (boxes)
million million million million
Variable cost (Rs. per box) 2·80 3·00 3·00 3·05
Total fixed costs (Rs. in million) 1 1·8 2·8 3·8
Forecast sales volumes are as follows:
Year 1 2 3 4
0·7 1·6 2·1 3·0
Sales Volume (boxes)
million million million million
The production equipment for the new confectionery line would cost Rs. 2 million
and an additional initial investment of Rs. 750,000 would be needed for working
capital. Capital allowances (tax-allowable depreciation) on a 25% reducing balance
basis could be claimed on the cost of equipment. Profit tax of 25% per year will be
payable one year in arrear. A balancing allowance would be claimed in the fourth
year of operation. BRT Co. uses a nominal after-tax cost of capital of 12% to appraise
new investment projects.
Required: (12+8=20)
a) Assuming that production only lasts for four years, calculate the net present value
of investing in the new product line using a nominal terms approach and advise
on its financial acceptability (work to the nearest Rs. 1,000).
b) Comment briefly on the proposal to use a four-year time horizon to evaluate the
project, and calculate and discuss a value that could be placed on the after-tax
cash flows arising after fourth year of operation, using a perpetuity approach.
Assume, for this part of the question only, that before-tax cash flows and profit
tax are constant from year five onwards, profit taxes are payable in the same year
and that capital allowances and working capital can be ignored.
Answer No. 1
a) Net present value evaluation of new confectionery investment (Rs. „000)

Year 1 2 3 4
Sales (WN1) 3,500 8,000 10,500 15,000

HYI P.T.O.
(2)
Variable cost(WN2) (1,960) (4,800) (6,300) (9,150)
1,540 3,200 4,200 5,850
Fixed costs (WN3) (1,000) (1,800) (2,800) (3,800)
Taxable cash flow 540 1,400 1,400 2,050
Less: Tax@ 25% - (135) (350) (350)
Add: CA tax benefit(WN4) - 125 94 70
540 1,390 1,144 1,770
Add: Working capital released - - - 750
540 1,390 1,144 2,520
PVIF@12% 0.893 0.797 0.712 0.636
Present values 482 1,108 815 1,603
" (Rs. „0000)
Sum of present values 4,008
Less: cost of Equipment [At year
(2,000) (2,000×1.00)
0]= =(2,000)
WC tied up [At year 0] = (750×1.00) =(750)
Net present value 1,258

Advice: The proposed investment in the new product is financially acceptable, as the NPV is
positive.
Working Notes:
1)Calculation of sales
Year 1 2 3 4
Sales volume (boxes) 2,100,000
700,000 1,600,000 3,000,000
selling price (/box) (Rs.) 5 5 5 5
Sales (/yr) Rs. „000 3,500 8,000 10,500 15,000

2)calculation of variable cost


Year 1 2 3 4
Sales volume (boxes) 2,100,000
700,000 1,600,000 3,000,000
Variable cost per box (Rs.) 3.00 3.00 3·05
2.80
Variable cost (/yr) Rs. „000 1,960 4,800 6,300 9,150

3)Calculation of fixed costs


Year 1 2 3 4
Sales volume (boxes) 700,000 1,600,000 2,100,000 3,000,000
Fixed costs (Rs. „000) 1,000 1,800 2,800 3,800

4)Calculation of Deprecation & tax benefits


Year 1 2 3 4
Capital allowance(Depn) 500,000 375,000 281,250 843,750
Tax benefit (25%) 125,000 93,750 70,312.50 210,937.50
Tax benefit (Rs. „000) 125 94 70 211

b) The proposal to use a four-year time horizon


The finance manager believes that cash flows are too uncertain after four years to be included in
the net present value calculation, even though sales will continue beyond four years. While it is
true that uncertainty increases with project life, cutting off the analysis after four years will
underestimate the value of the investment to the extent that cash flows after the cut-off point are

HYI P.T.O.
(3)
ignored. Furthermore, since the new confectionery line is expected to be popular, cash flows
after year four could be substantial, increasing the extent of the under valuation. Artificially
terminating the evaluation after four years has accelerated the recovery of working capital and
has also led to a large balancing allowance. These increased cash flows, which arise in years
four and five respectively, will overestimate the value of the investment.

The value of cash flows after the fourth year of operation:


The approach here should be to calculate the present value of the expected future cash flows
beyond year four. If the before-tax cash flows are assumed to be constant and if the one-year
delay in tax liabilities is ignored, the year four present value of future cash flows beyond year
four can be estimated using a perpetuity approach. The year four present value of cash flows
from year five onwards will be: 2,050 x (1 – 0·25)/0·12 = 12,813 (Rs. „000)

The year zero present value of these cash flows = 12,813x 0·636 = 8,149 (Rs. „000)
Although these calculations ignore the capital allowance tax benefits (which will decrease each
year) and the incremental investment in working capital (which will increase slightly each year),
the present value of cash flows after year four is still substantial.
2.
a) Answer the following, supporting the same with proper reasoning: (3×2.5=7.5)
i) Whether the present value decreases at a linear rate, at an increasing rate, or at
a decreasing rate with the discount rate and why?
ii) Which ratio would a rich investor interested in investing in equity shares most
likely consult while considering the financing of seasonal inventory?
iii) The credit policy of Firm “A” is – A high percentage of bad-debt loss but
normal receivable turnover and credit rejection rate. What is the effect of this
policy on sales and profit?
b) Ciron Limited has the following capital structure:
9% Debentures Rs. 275,000
11% Preference Shares Rs. 225,000
Equity Shares (face value Rs. 10 per share) Rs. 500,000
Rs. 1,000,000
Additional information:
i) Rs. 100 per debenture redeemable at par have 2% floatation cost and 10 years
of maturity. The market price per debenture is Rs. 105.
ii) Rs. 100 per preference share redeemable at par has 3% floatation cost and 10
years of maturity. The market price per preference share is Rs. 106.
iii) Equity share has Rs. 4 floatation cost and market price per share of Rs. 24.
The next year expected dividend is Rs. 2 per share with annual growth of 5%.
The firm has a practice of paying all earnings in the form of dividends.
iv) Corporate income-tax rate is 35%.
Required: 7.5
Calculate Weighted Average Cost of Capital (WACC) using market value
weights.
Answer No. 2
a)
i) The present value decreases at a decreasing rate with discount rate. As the discount rate
increases, the discount factor goes on decreading. It is because the denominator of the
present value equation increases at an increasing rate with multiple of increase in period
„n‟.

HYI P.T.O.
(4)
ii) While considering the financing of seasonal inventory, the rich equity investor would be
consulting the profitability ratios and ratios that provide information about risk relating to
the investment because he is mostly cautious of balancing the risk-return trade off..
iii) The effect of this policy is that the sales remain unaffected while profits decrease. This
policy indicates that the firm has poor collection policy. Accounts that are collectable are
being written off too soon. Therefore, the turnover is being maintained at the expense of
increased bad debt losses.
b)
Cost of Equity (Ke)

[where p0=Market price – floatation cost)

=15%

Cost of Debt (Kd)

( ) ( )
[ ]
( )

( ) ( )
( )

=6.11%

Cost of Preference Shares (Kp )

( )
[ ]
( )

( )
( )

=11.47%

Calculation of WACC using Market Value Weights


Source of Capital Market Weights to Specific Total Cost
Value (Rs.) Total Capital Cost
Debenture ( Rs.105 per debenture) 2,88,750 0.1672 0.0611 0.0102
Preference shares ( Rs.106 per preference 2,38,500 0.1381 0.1147 0.0158
shares)
Equity Shares ( Rs.24 per share) 12,00,000 0.6947 0.1500 0.1042
17,27,250 1 0.1302
WACC = 13.02%

3.
a) Vikas Engineering Ltd., currently has 100,000 outstanding shares selling at
Rs. 100 each. The firm has net profit of Rs. 1,000,000 and wants to make new

HYI P.T.O.
(5)
investments of Rs. 2,000,000 during the period. The firm is also thinking of
declaring a dividend of Rs. 5 per share at the end of the current fiscal year. The
firm‟s opportunity cost of capital is 10 percent.
Required: (6+2=8)
i) What will be the price of the share at the end of the year if a dividend is not
declared, and if a dividend is declared? What will be the impact on
shareholders‟ wealth?
ii) How many new shares must be issued when dividend is declared?
b) P has an expected return of 22 percent and standard deviation of 40 percent. Q has
an expected return of 24 percent and standard deviation of 38 percent. P has a
beta of 0.86 and that of Q is 1.24. The correlation between the returns of P and Q
is 0.72. The standard deviation of the market return is 20 percent.
Required: (2+2+2+1=7)
i) Is investing in Q better than investing in P?
ii) If you invest 30 percent in Q and 70 percent in P, what is your expected rate
of return and the portfolio standard deviation?
iii) What is the market portfolio‟s expected rate of return and how much is the
risk-free rate?
iv) What is the beta of portfolio if P‟s weight is 70 percent and Q's 30 percent?

Answer No. 3
a)
i) The price of the share at the end of the current fiscal year is determined as follows:
P0 =
P1= P0 (1+K) – DIV1
The value of P, when dividend is not paid, is:
P1 = Rs 100 (1.10) - 0 = Rs 110
When dividend is paid it is:
P1 = Rs 100 (1.10) - Rs 5 = Rs 105.
It can be observed that whether dividend is paid or not the wealth of shareholders remains
the same. When the dividend is not paid the shareholder will get Rs 110 by way of the
price per share at the end of the current fiscal year. On the other hand, when dividend is
paid, the shareholder will realize Rs 105 by way of the price per share at the end of the
current fiscal year plus Rs 5 as dividend.

ii) The number of new shares to be issued by the company to finance its investments is
determined as follows:

mP1 = I –(E-n× DIV1)


m×105=[2,000,000 – {1,000,000 – (100,000×50}]
105m = 2,000,000 – (1,000,000 -500,000)
105m = 1,500,000
m= 1,500,000/105 =14,286 shares.

Where,
m= No. of new shares to be issued
I= Investment
N=No. of existing shares
DIV1= Dividend per share
E=earning

b)

HYI P.T.O.
(6)
i) P has lower return and higher standard deviation than Q. Therefore, investing in Q will
give more return with less volatility. However, investing in both will yield diversification
advantage.

(ii) Expected rate of return (rp) = 22


Portfolio standard deviation ( )
=√
=√
=√
=√
=0.37
=37%

iii) The risk- free rate will be the same for P and Q. Their rates of return are given as follows:
rp, 22=rf +(rm-rf) 0.86
rq, 24=rf+(rm-rf) 1.24
rp-rq , -2= (rm-rf) (-0.38)
rm-rf ,-2/-0.38=5.26%
rp , 22=rf+(5.26) 0.86
rf = 17.5%
rq = 24=rf+(5.26) 1.24
rf = 17.5%
rm – 17.5 =5.26
rm = 22.76%
Market portfolio expected return (rm) = 22.76%
Risk-free rate (rf) = 17.5%
iv) pq = p p+ q q =0.86

4.
a) Nepal Gas Company franchise “NP Gas” stations in east and west sites of Nepal.
All payments by franchises for gas product, which average Rs. 420,000 a day are
by cheque. Presently, the overall time between mailing of the cheque and the time
the company has collected or available funds at its bank is six days.
Required: (1+3+3=7)
i) How much money is tied up in this interval of time?
ii) To reduce this delay, the company is considering daily pickups of cheques
from the stations. In all, three cars would be needed and three additional
people would be hired. This daily pick up would cost Rs. 93,000 on an annual
basis and it would reduce the overall delay by two days. Currently, the
opportunity cost of funds is 9% that being the interest rate on marketable
securities. Should the company inaugurate the prick up plan? Why?
iii) Rather than mailing cheques to its bank, the company could deliver them by
messenger service. This would reduce the overall delay by one day and would
cost Rs. 10,300 annually. Should the company undertake this plan? Why?
b) Kathmandu Medical Hospital is planning to introduce a new CT scan machine
which costs Rs. 16 million. Expected annual revenue of the machine is projected
to be Rs. 18 million. Variable cost is 60% of sales and fixed costs are Rs. 2
million. The firm is planning to finance the fund requirement by bank loan of Rs.
5 million @ 12%, by issue of debenture of Rs. 5 million @ 8% and remaining by
equity shares which will be issued at Rs. 10 (par) per share. The taxation rate
applicable to the firm is 25%.
Required: (5+3=8)
HYI P.T.O.
(7)
i) Calculate operating leverage, financial leverage and combined leverage.
ii) Briefly explain the inter-linkage between leverage, profit and risk.

Answer No. 4
a)
Average daily payment by cheque= Rs. 420,000
Required time for collection of fund= 6 days

i) Cash tied up= Rs. 420,000×6 days= Rs. 2,520,000


ii) Calculation of annual net cost/benefit for pick up plan:
Opportunity cost savings = Rs. 420,000 × 2 days × 9% = Rs. 75,600
Less: Annual cost of pick up plan = (Rs. 93,000)
Net annual loss = (Rs. 17,400)
Since this option brings loss, the company should not start pick up plan.

iii) Calculation of annual net cost/benefit by employing messenger:


Opportunity cost savings (Rs. 420,000 × 1 day × 9% = Rs. 37,800
Less: Annual cost of messenger service = (Rs. 10,300)
Annual Net savings =Rs. 27,500
Since this option brings net savings to the company, this is a viable option for
the company.
b)
i) Calculation of leverages

Int Rate Rs.


Cost of Project 16,000,000
Annual Sales 18,000,000
Variable Cost 10,800,000
Contribution 7,200,000
Fixed Cost 2,000,000

Earnings before interest and taxes 5,200,000


interest 5,000,000 12% 600,000
5,000,000 8% 400,000
Earning before tax 4,200,000
Tax 25% 1,050,000
Earning after tax 3,150,000
Operating Leverage (Cont/EBIT) 1.38
Financial Leverage (EBIT/EBT) 1.24
Combined Leverage (DFL X DOL) 1.71

ii) Interlinkage between leverage, return and risk


Leverage is a position when capital is funded largely by external sources like debt, bank
loan. The higher the livered firm, higher the profit. This is because, if the firm is funded
by debt sources, it is a cheaper source of finance as debt interest are tax deductible. But is
the firm is unlevered, i.e. funded by equity, profit will be lower as dividend to
shareholders is not tax deductible from income tax purpose. However, if the firm is too
levered, there is a risk of solvency because debt funds are repayable. The firm will be in
risk of fund at the times when debts are matured. Hence, an optimal mix of debt and
equity is required to maintain adequate profitability with solvency.
5.
a) Beta Company is contemplating conversion of 500, 14% convertible bonds of Rs.
1,000 each. Market price of the bond is Rs. 1,080. Bond indenture provides that
HYI P.T.O.
(8)
one bond will be exchanged for 10 shares. Price–earnings ratio before redemption
is 20:1 and anticipated price-earnings ratio after redemption is 25:1. Number of
shares outstanding prior to redemption are 10,000. EBIT amounts to Rs. 200,000.
The company is in the 35% tax bracket. Should the company convert bond into
shares? Support your analytical comments with required calculations. 4
b) A company offers a fixed deposit scheme whereby Rs. 10,000 matures to
Rs. 12,625 after 2 years, on a half-yearly compounding basis. If the company
wishes to amend the scheme by compounding interest every quarter, what will be
the revised maturity value? 3
c) Following are the ratios of the business of Ganesh Traders Ltd., dealing in the
machineries, for the year ended 31st Ashadh, 2069:
Average Collection Period 3 months
Stock Turnover 1.5 times
Average Payment Period 2 months
Gross Profit Ratio 25%
Opening Receivables Rs. 600,000
Gross Profit for the year ended 31st Ashadh, 2069 amounted to Rs. 800,000.
Closing stock of the year is Rs. 20,000 above the opening stock.
Closing bills receivable amounted to Rs. 50,000 and bills payable to Rs. 20,000.
Required: calculate (1+3+2+2=8)
i) Sales
ii) Sundry Debtors
iii) Closing Stock
iv) Sundry Creditors
Answer No. 5
a)

Particulars Pre- redemption Post-redemption


EBIT (Rs.) 2,00,000 2,00,000
Interest @14% (Rs.) 70,000 Nil
Taxable Income (Rs.) 1,30,000 2,00,000
Less: Tax @ 35 percent (Rs.) 45,500 70,000
Net Income after Tax (Rs.) 84,500 1,30,000
Outstanding Shares ( Nos) 10,000 15,000
EPS (Rs.) 8.45 8.66
P/E Ratio 20:1 25:1
Market Price per share (Rs.) 169 216.50
(i.e. price-earnings ratio times×EPS)

Comment:
This is two-in-one benefit scheme. The company should convert the bond into shares because
both shareholders and debenture holders stand to gain. The post-redemption market price of
the equity shares would be Rs.216.50 than the pre-redemption market price of Rs.169.
Moreover the debenture holder/ bondholders would receive Rs.1, 690 in stock (i.e. 169×10
shares, in place of receiving cash Rs.1, 080 only)

b)
Computation of Revised Maturity Value
Principal = Rs. 10,000
Amount = Rs. 12,625

10,000= *( +
)
HYI P.T.O.
(9)
Pn = A × (PVFn, i)
10,000 = 12,625 (PVF4, i)
0.7921 = (PVF4, i)
According to the Table on Present Value Factor (PVF4,i) of a lump sum of Re. 1, a PVF of
0.7921 for half year at interest (i) = 6 percent. Therefore, the annual interest rate is 2 ×0.06 = 12
percent.
i = 6% for half year
i = 12% for full year.
Therefore, Rate of Interest = 12% per annum

Revised Maturity Value ( )

( )

( )
=10,000×1.267 [considering (CVF 8,3)=1.267]
Revised Maturity Value = 12,670.
c)
i) Calculation of Total Sales
Gross Profit Ratio =* +

25 =* +

Net Sales = Rs.32,00,000

ii) Calculation of Sundry Debtors


Average Collection Period = 3 months
Average Collection period = No of months in year/ Debtors Turnover Ratio (DTR)
3 month =12 Months/Debtors Turnover ratio (DTR)
Debtors Turnover Ratio (DTR) = 12 Months/3 Months= 4 times
Debtors Turnover Ratio (DTR) = Net Credit Sales/ Average Accounts Receivables
4 = 32, 00,000/Average Accounts Receivable

Average Accounts Receivable= Rs.32,00,000/4 =Rs.8,00,000


(Opening Receivables + Closing Receivables)/2 = Rs.8, 00,000
(Opening Receivables + Closing Receivables) = Rs.8, 00,000×2
(6,00,000 + Closing Receivables) = Rs.16, 00,000
Closing Receivables = Rs.16, 00,000-Rs.6, 00,000
=Rs.10, 00,000
Sundry Debtors = Closing Receivables- Bills Receivables
=Rs.10, 00,000-50,000
=Rs.9, 50,000
iii) Calculation of Closing Stock
Stock Turnover Ratio (STR) = 1.5 times
STR =Cost of Goods sold/ Average stock
Cost of goods sold = Sales- Gross Profit

HYI P.T.O.
(10)
1.5 =24, 00,000/ Average Stock
Average Stock =24, 00,000/1.5
=Rs.16, 00,000
Average Stock = (Opening Stock +Closing Stock)/2
Rs.16, 00,000 = (Opening Stock + Closing Stock)/2
Opening Stock + Closing Stock =Rs.16, 00,000×2
Closing Stock is higher than opening stock by Rs.20, 000
Then opening Stock = (Rs.32, 00,000-Rs.20, 000)/2
Opening Stock = 31, 80,000/2=Rs.15, 90,000
Hence Closing Stock =Rs.15, 90,000+ Rs.20, 000
=Rs.16, 10,000
iv) Calculation of Sundry Creditors
Credit Purchase = Cost of goods sold+ Closing Stock- Opening Stock
=Rs.24, 00,000+Rs.16, 10,000-Rs.15, 90,000= Rs.24, 20,000
Credit Turnover Ratio = 12 Months/ 2 months= 6 months
Credit Turnover Ratio (CTR) = Net Credit Purchase /Average Payables
Average Payables = 24, 20,000/6= Rs.4, 03,333
Creditors = Accounts Payable- Bills Payable
=Rs.403, 333-Rs.20, 000
=Rs.383, 333

6. Write short notes on: (4×2.5=10)


a) Security market line
b) Gearing ratio
c) Bridge finance
d) Project under capital rationing
Answer No. 6
a) Security market line
Security market line (Beta function) is simply an index of Systematic Risk which
cannot be reduced by Portfolio Diversification. The slope of the SML indicates the
change in excess return of the stock over the change in excess return on the market
portfolio. The Beta of the portfolio is simply a weighted average of the individual
stock Betas of the portfolio. It shows the sensitivity of return on the stock to change
in return on market portfolio.
Results of Beta Function:
 If the Beta=1.0, this implies that the excess return for the stock varies
proportionally with the excess return for the market portfolio.

 If the Beta>1.0, this implies that the excess return for the stock varies more than
proportionally with the excess return for the market portfolio. (Aggressive)

 If the Beta<1.0, this implies that the excess return for the stock varies less than
proportionally with the excess return for the market portfolio. (Defensive).

b) Gearing ratio
A general term describing a financial ratio that compares some form of owner's equity (or
capital) to borrowed funds. Gearing is a measure of financial leverage, demonstrating the
degree to which a firm's activities are funded by owner's funds versus creditor's funds.
The higher a company's degree of leverage, the more the company is considered risky. As
for most ratios, an acceptable level is determined by its comparison to ratios
of companies in the same industry. The best known examples of gearing ratios include
the debt-to-equity ratio (total debt / total equity), times interest earned (EBIT / total
interest), equity ratio (equity / assets), and debt ratio (total debt / total assets).

HYI P.T.O.
(11)
A company with high gearing (high leverage) is more vulnerable to downturns in the
business cycle because the company must continue to service its debt regardless of how
bad sales are. A greater proportion of equity provides a cushion and is seen as a measure
of financial strength.

c) Bridge Finance
Bridge Finance refers, normally to loans taken by a business, usually from commercial
banks for a short period, pending disbursement of terms loan by financial institutions.
Normally, it takes time for the financial institution to finalize procedures of creation of
security, tie-up participation with other institution etc., even though a positive appraisal
of project has been made. However, once the loans are approved in principal, firms, in
order not to lose further time in starting their projects, arrange for bridge finance. Such
temporary loan is normally repaid out of the proceeds of the principal term loans.
Generally the rate of interest on bridge finance is 1% or 2% higher than on normal term
loans.

d) Project under Capital Rationing


The capital rationing situation refers to the choice of investment proposals under financial
constraints in terms of given size of capital expenditure budget. The objective to select
the combination of projects would be the maximization of total NPV. The project
selection under capital rationing involves two stages
(i) Identification of the acceptable projects
(ii) Selection of the combination of projects.

The acceptability of projects can be based either on profitability index or IRR. The
method of selecting investment projects under capital rationing situation will depend
upon whether the projects are indivisible or divisible. In case the project is to be
accepted/rejected in its entirety, it is called an individual project; a divisible project, on
the other hand, can be accepted/ rejected in part.
7. Distinguish between: (4×2.5=10)
a) Return on Equity and Return on Capital Employed
b) Business Risk and Financial Risk
c) Global Depository Receipts and Euro Convertible Bonds
d) Over Capitalization and Under Capitalization
Answer No. 7
a) Return on equity and Return on capital employed
Return on equity (ROE) measures the rate of return on the ownership interest (shareholders'
equity) of the common stock owners. It measures a firm's efficiency at generating profits
from every unit of shareholders' equity (also known as net assets or assets minus liabilities).
ROE shows how well a company uses investment funds to generate earnings growth. ROEs
between 15% and 20% are generally considered good. Return on equity reveals how much
profit a company earned in comparison to the total amount of shareholder equity found on
the balance sheet. If you think back to lesson three, you will remember that shareholder
equity is equal to total assets minus total liabilities. It's what the shareholders "own".
Shareholder equity is a creation of accounting that represents the assets created by the
retained earnings of the business and the paid-in capital of the owners.
Return on capital employed (ROCE) is an accounting ratio used in finance, valuation, and
accounting. Return on capital employed (ROCE) is a measure of the returns that a business
is achieving from the capital employed, usually expressed in percentage terms. Capital
employed equals a company's Equity plus Non-current liabilities (or Total Assets − Current
Liabilities), in other words all the long-term funds used by the company. ROCE indicates
the efficiency and profitability of a company's capital investments. ROCE should always be
higher than the rate at which the company borrows otherwise any increase in borrowing will

HYI P.T.O.
(12)
reduce shareholders' earnings, and vice versa; a good ROCE is one that is greater than the
rate at which the company borrows. It can be calculated as follows:

b) Business Risk and Financial Risk

Business risk refers to the risk associated with the firm‟s operations. It is an unavoidable risk
because of the environment in which the firm has to operate and the business risk is
represented by the variability of earnings before interest and tax (EBIT). The variability in
turn is influenced by revenues and expenses. Revenues and expenses are affected by demand
of firm‟s products, variations in prices and proportion of fixed cost in total cost.
Whereas, financial risk refers to the additional risk placed on firm‟s shareholders as a result
of debt use in financing. Companies that issue more debt instruments would have higher
financial risk than companies financed mostly by equity. Financial risk can be measured by
ratios such as firm‟s financial leverage multiplier, total debt to assets ratio etc.

Business risk is the relative dispersion in the firm‟s expected earnings before interest and
taxes. Whereas, financial risk is the additional variability in the earnings available to the
firm‟s common stockholders and additional chance of insolvency borne by the common
stockholders caused by the use of financial leverage.
c) Global Depository Receipts and Euro Convertible Bonds
Global Depository Receipts (GDR) is a negotiable certificate denominated in US dollars
which represents a Non-US company‟s publically traded local currency equity shares. GDR
are created when the local currency shares of an Indian company are delivered to
Depository‟s local custodian Bank against which the Depository bank issues depository
receipts in US dollars. The GDR may be traded freely in the overseas market like any other
dollar-expressed security either on a foreign stock exchange or in the over-the-counter
market or among qualified institutional buyers.
Whereas, Euro Convertible bonds are quasi-debt securities (unsecured) which can be
converted into depository receipts or local shares. ECBs offer the investor an option to
convert the bond into equity at a fixed price after the minimum lock-in period. The price of
equity shares at the time of conversion will have a premium element. The bonds carry a fixed
rate of interest. These are bearer securities and generally the issue of such bonds may carry
two options viz. call option and put option. In the case of ECBs, the payment of interest and
the redemption of the bonds will be made by the issuer company in US dollars. ECBs issues
are listed at London or Luxemburg stock exchanges.

d) Over capitalization and Under capitalization

Overcapitalization is a situation in which actual profits of a company are not sufficient


enough to pay interest on debentures, on loans and pay dividends on shares over a period of
time. This situation arises when the company raises more capital than required. A part of
capital always remains idle. With a result, the rate of return shows a declining trend. The
causes can be High promotion cost , Purchase of assets at higher prices, company‟s floatation
n boom period, Inadequate provision for depreciation, liberal dividend policy and over-
estimation of earnings.

HYI P.T.O.
(13)
Under Capitalization is the situation where exceptionally high profits are earned as compared
to other firms in the industry. An undercapitalized company situation arises when the
estimated earnings are very low as compared to actual profits. This gives rise to additional
funds, additional profits, high goodwill, high earnings and thus the return on capital shows an
increasing trend. The causes can be Low promotion costs, Purchase of assets at deflated
rates, conservative dividend policy, Floatation of company in depression stage, High
efficiency of directors, adequate provision of depreciation and large secret reserves are
maintained.

HYI P.T.O.
CAP-II, Business Communication, June 2013(Suggested Answer)

Roll No……………. Maximum Marks – 50


Total No. of Questions - 5
Time Allowed – 1 and 1/2 Hours
Marks
All questions are compulsory.
Section -'A'
1. Read the following case carefully and answer the questions given below: (3+3+4=10)
When Indira Sharma joined PepsiCo as chief strategist, she convinced management to
sell off its fast-food holdings in Taco Bell, Pizza Hut and KFC. She was intent on
moving the company “from snack food to health food, from caffeine colas to fruit
juices, and from shareholder value to sustainable enterprise.” Her goal was to do good
for business as well as doing good for people and the planet. PepsiCo, under her
leadership, would favor wind and solar power, avoid fossil fuels, fight against
obesity, and promote diversity.
To achieve these ambitious goals, Sharma instituted “Performance with Purpose,” a
portion of which is explained in the following three components:
 Human sustainability refers to PepsiCo‟s efforts to nourish consumers with a
range of products from treats to healthful eats.
 Environmental sustainability reinforces PepsiCo‟s commitment to protect natural
resources and operate in a way that minimizes the company‟s environmental
footprint.
 Talent sustainability focuses on developing the company‟s employees by creating
a diverse and inclusive culture, and evolving PepsiCo as an attractive destination
for the world‟s best people.
Convincing PepsiCo managers and employees to accept her “Performance with
Purpose” plan was a monumental task that would involve significant changes in
products and procedures within the enterprise. In any organization, change is
accepted slowly and often reluctantly. Encouraging managers, employees and
investors to buy changes in company philosophy is often difficult.
Issues for Critical Thinking:
a) In communicating the changes in philosophy and practice within PepsiCo, should
Indira Sharma and her management team use formal or informal channels of
communication? Give examples of each.
b) In the midst of organizational changes at PepsiCo, what kinds of messages do you
think will be traveling downward, upward, and horizontally?
c) What kinds of barriers block the flow of communication in any organization
undergoing change? (3+3+4=10)
Answer:
a) Certainly, Ms Shrama and her management team should use both formal and informal
channels of communication. In formal channels, they may organize the formal meetings with
stakeholders, send written correspondences within and outside the organizations, advertise
the new program through public medias, disseminate the message through press conferences,
etc. In informal channels, they may visit the target groups and talk personally, organize
parties and gatherings, use telephone calls, etc.

XDB P.T.O.
(15)
b) Memos like, asking responses of the staff about going to the new practices by upper level
management, asking the staff to perform certain tasks regarding these new practices, and
assigning them the responsibilities are some of the examples of downward communication.
Staff sending responses to the upper level management, asking for clarification of some
aspects in which they are confused, sending their reactions and feelings to the new changes
through memos and other channels are the examples of upward communication. Likewise,
interdepartmental discussions about the new changes, meetings of the officers of same level,
interdepartmental coordination of certain tasks are the examples of horizontal communication
which will be practiced during the organizational changes at PepsiCo.
c) An organization that is undergoing changes may suffer from different types of
communication barriers. Suspicion to new policy and fear of uncertainty may hinder the
effective flow of communication in such organization. In most strategic organizational
change, at least some employees will be asked to assume different responsibilities or focus
on different aspects of their knowledge or skill. The greater the change a person is asked to
make, the more pervasive that person‟s fear will be. There will be fear of change. More
important, however, there will be fear of failure in the new role. Involving employees as soon
as possible in the change effort, letting them create as much of the change as is possible and
practical is key to a successful change effort.
Besides, the receiver may not give proper attention and show no interest to the proposal of
some new member and may think the new proposal irrelevant for the organization that is
running smoothly. Hence, the receiver may bypass the message. Sometimes, people may
have different perceptions because of which they may not come to the common ground of
sharing.

2. Show your acquaintance with the concept of „globalization of culture‟. Discuss the
role of cross-cultural communication in the globalized context of the world cultures.
Also suggest some useful strategies to promote cross-cultural communication in the
organizational level. 10
Answer:
In the context of the rapidly changing world, culture is viewed, interpreted and understood in
the vast global context. Cross cultural campaigns are conducted in every sector such as
business, education, communication, media, and so on. Culture is being globalized in many
ways. Basically media is responsible for globalization. People learn the ways of living and
working from others in today‟s post modern world. Globalization of the cultures involves
three primary things:
 Understanding culture in broader perspective,
 Promoting cross-cultural relationship between /among people of diverse cultures working
in diverse work situation, each respecting each other‟s cultural norms and \ values, and
 Changing traditional cultural perceptions incompatible with the changing needs of today.

Cross cultural communication should be promoted in the globalized context of the world
cultures. Work places are largely diverse. In order to reduce the misunderstandings caused by
work place diversity, cross cultural communication networks need to be established. For the
promotion of the cross cultural communication, the following strategies may be adopted:
 Allow different languages for official work. Cultures of even minorities need to
be acknowledged.
 Conduct seminars, workshops, etc. to familiarize people with each other.
 No single language and working style may be a standard one; every language, and
working style is equally important.
 Design increasing meetings, useful for all cultures.
XDB
(16)
 Build cohesive multi-cultural work teams.
 Create a corporate culture that can accommodate diversity to maximize the potential of
your workforce

3. Imagine that you work for XYZ Company that produces computer chips. Though
there is high competition in the market, the company had run its business smoothly
for last few years. From the last quarter of the year, the sales dropped by 14% and the
CEO asked you and your team to make a thorough market study and find out the
causes of the low sales rate. After a month‟s study you came to the conclusion that
lack of effective communication between sales associates and buyers is responsible
for the problem. Now, write a report to your CEO Rajani Sharma in memo format
stating problem and proposed solutions. Your report may contain the sections like,
overview, problem statement, proposed solutions, advantages/disadvantages of the
solutions, and conclusion. 10
Answer:
 TO: Rajani Sharma, CEO
FROM: Indra Prasad Joshi
DATE: October 23, 2012
RE: Report to Reduce Communication Problems and Increase Sales
Overview
Last quarter, our sales dropped 14%. I believe a communication breakdown between sales
associates and buyers has contributed to the decline.
In our company, most sales associates do not know their buyers. The sales associates I spoke
with felt intimidated by and did not communicate with the buyers. The buyers, on the other
hand, aren‟t receiving the information they need to purchase the merchandise that the
customers want.
In this report, I‟ll provide more information on the problem and offer a solution that I believe
will help improve communication and increase sales.
Statement of the Problem
Simply stated, the problem is a lack of communication between the buyers and sales
associates. The sales associates, who work directly with the customers, know what shoppers
are looking for and what they request. Now that the company is emphasizing customer
satisfaction, it is more important than ever to have the merchandise that customers want.
Proposed Solution
To ensure that our customers receive the merchandise they want, we need to set up a regular
exchange of information between the buyers and the sales associates. This can be
accomplished by having regular meetings and by conversations over the phone.
We could arrange meetings, which would require each department‟s buyers to go to three
stores monthly. Since there are 22 stores and two buyers per department, one buyer would
visit each department about once every quarter. The meetings could be held in either the
department‟s office or in the store training room.
Between meetings, I recommend that sales associates call their buyers directly at least
monthly.
Advantages and Disadvantages of Adopting the Solution
To save time and costs, I considered communication solely by mail or telephone, but I
rejected these options because they lack timeliness, direct feedback, and face-to-face
interaction. Without meeting and brainstorming, the quality of the discussions is diminished.
XDB
(17)
Sales associates are used to working directly with customers, and they feel more comfortable
meeting face-to-face.
Communicating by mail would be less expensive than having meetings, but it creates a time
delay in any interaction. This delay could frustrate both the associates and the buyers, which
could lead to reluctance in writing and a return to the original problem.
Communication only during the meetings between buyers and sales associates would not be
effective simply because of infrequent interaction (every 3–4 months).
Other benefits of improving communication between the buyers and the sales associates
follow:
• Improved customer relations because we‟ll be responding to their merchandise requests
• A mutually beneficial working relationship between buyers and sales associates
• Increased sales because we‟ll be stocking merchandise that customers want
An added benefit could result when the buyers actually visit the departments of the sales
associates and see the floor plans. With their merchandising experience, buyers might
suggest alternative ways of arranging the products on the floor to increase customer traffic
and encourage sales.
With direct interaction between buyers and sales associates leading to more frequent
communication, we should notice a better and more productive working relationship.
Conclusion
Our sales have dropped 14% in the last quarter, and I believe that lack of communication
between sales associates and buyers has contributed to the drop-off. To resolve the lack of
communication, I recommend establishing regular meetings and telephone conversations
between buyers and sales associates as described in this report.
If communication between sales associates and buyers takes place, buyers should begin to
understand the needs of customers through their sales associates. From this, customer
satisfaction will increase as well as sales.
4. Following a specific total quality management (TQM) strategy will help your team
succeed. What strategies would you adopt for such successful collaboration? 10
Answer:
 Following the TQM strategy could lead to a successful collaboration:
- Develop your team
- Choose a team leader
o Assign duties
o Create schedules (project milestones)
o Encourage group participation
- Determine your team goal
- Identify the problem
- Analyze the problem‟s causes
- Determine potential improvements
- Verify suggested solutions
- Write the text
5. Briefly explain any FOUR of the followings: (4×2.5=10)
a) Preparing and practicing for job interview
b) Business ethics
c) Non-verbal communication skills
d) Internal and external functions of communication
e) Combating stage fright during business presentations
XDB
(18)

Answer:
a) After you have learned about the target organization, study the job description or job listing.
Knowing as much as you and about the position enables you to practice your best response
strategies prior to the interview. The most successful job candidates rehearse success stories
and practice answers to typical questions. As part of their preparation before the interview,
they decide what to wear, and they gather the items they plan to take with them.

b) The term „business ethics‟ refers to the communication and dealing tendencies based on
truth, honesty and fairness. Business dealings should always be based on business ethics.
Successful business dealing is oriented to truth, and fair activities. Sometimes we also find
business organization and people who are commonly violating the norms and ethics of
business. If such violence occurs inside the organization, it is called internal unethical
dealings. If the violence of business ethics occurs in the dealings with outsiders such as
costumers, suppliers, etc. it is known as external unethical dealings. In this way, unethical
dealings which take place with the outsiders are called external unethical dealings. For
example, plagiarisms, presenting false statement, etc. are the general instances of external
unethical dealings. Business dealings that avoid both internal and external unethical features
are assumed to have business ethics.

c) Non verbal communication skills: These include the aspects such as body language, gestures,
facial expressions, eye contact, etc., which also become a part of the communicating process;
as well as the written and typed modes of communications. It avoids the use of words.
Verbal communication is normally used for communicating information about events
external to the speakers, non-verbal codes are used to establish and maintain interpersonal
relationships. It is considered more polite or nicer to communicate attitudes towards others
non-verbally rather than verbally, for instance in order to avoid embarrassing situations.

d) Internal communication includes exchanging ideas and messages with superiors, coworkers,
and subordinates. Some of the functions of internal communication are to issue and clarify
procedures and policies, inform management of progress, develop new products and services,
persuade employees or management to make changes or improvements, coordinate activities,
and evaluate and reward employees.

e) Nearly everyone experiences some degree of stage fright when speaking before a group.
Being afraid is quite natural and results from actual physiological changes occurring in your
body. Faced with a frightening situation, your body responds with the fight-or-flight
response. Following the techniques suggested by communication experts, like preparing
thoroughly and rehearsing, taking deep breath before speaking, diverting the attention from
oneself to the subject matter, and many more will help you combat the stage fright.

XDB
SUGGESTED ANSWER
CAP – II – MARKETING – JUNE, 2013

Roll No……………. Maximum Marks – 50


Total No. of Questions - 5 Total No. of Printed Pages - 1
Time Allowed – 1.5 Hours
Marks
Section -'B'
6. Read the following case carefully and answer the questions given below:
The Bata Shoe Organization (BSO) was founded by Tomas Bata on 24 August 1894
in Czech Republic (former Czechoslovakia). Over the years, Bata has established
itself globally and operates more than 4600 retail stores in over 50 countries including
Nepal. Bata is estimated to serve one million customers each day and employs more
than 40000 people. It has sold more than 15 billion pairs of shoes – more than the
number of people who have walked the earth during this time. If all the shoes sold by
Bata are laid end to end, they would extend to 12 million kilometers. The Bata pricing
in global market has nothing to do with psychological pricing as is widely believed.
The company has given emphasis on continuous product quality improvement.
According to the Marketing Director of BSO, “Quality is the central of all marketing
and thus Bata shoes deserves premium price in global market in comparison to
competing brands.”
a) Comment on the distribution strategy used by the BSO. 5
b) On the basis of above case, explain the marketing philosophy used by BSO. 5
Answer:
a) Comment on the distribution strategy used by the BSO.
Ans: Distribution strategy used by Bata is intensive distribution. In intensive distribution
strategy, marketers use maximum number of retailers to make the product easily accessible
and available in the target market. In the given case it is mentioned that Bata has established
itself globally and operates more than 4600 retail stores in over 50 countries including Nepal.
Operation of more than 4600 retail stores in over 50 countries means in an average there are
at least 92 retail stores in each country which clearly indicates use of intensive distribution
strategy.
b) On the basis of above case, explain the marketing philosophy used by the
organization.
Ans: On the basis of given case, marketing philosophy used by the organization is product
concept. This concept holds that customers are ready to pay reasonable price for quality
product. The main emphasis is given to quality improvement. In this case the focus of all
marketing is quality. High quality with premium price and high profit margin in the global
market indicates that the organization‟s marketing activities are guided by the product
concept philosophy of marketing.

7.
a) What is marketing? Briefly explain the importance of marketing to our dynamic
society of today. (2+3=5)
b) What is marketing information system? Explain the role of decision support
system in a firm‟s marketing information system. (2+3=5)
Answer:
a) Marketing is a total system of business activities designed to plan, price, promote and
distribute want satisfying product to target market to achieve organizational objectives. It
includes all exchange related activities which direct the flow of goods and services from
ORK P.T.O.
(20)
producer to consumer. Today marketing is fast growing. It is dynamic, competitive and
consumer oriented. Marketing provides value and benefits to customers, firms and the
society. A large number of people have got employment in this sector. Besides customers
supremacy, marketers fulfill social responsibilities. Economically marketing is very
important for the country.
Importance of marketing to our dynamic society can be seen from the following
points:
-creation and delivery of utility and rise in standard of living
-employment generation
-optimum utilization of resources
-revenue generation
-economic development
-poverty reduction
-globalization through marketing networks etc.
b) Basically, MIS is a structured interacting complex of persons, machines, and procedures
designed to generate an orderly flow of information collected from both internal and
external sources for use as the basis for decision making in specified areas of marketing
management.
 The MIS is a carefully developed master plan for information flow.
 There must be coordination among departments and individuals, including top
management, marketing research, personnel, finance, system analysts, programmers,
and computer experts.
 Advance in computer technology, data copying, storage, and retrieval have been
instrumental in the growth of sophisticated MIS.
MIS consists of basically the following components –
i. Internal Records System;
ii. Marketing Intelligence System;
iii. Information Processing System;
iv. Decision Support System; and
v. Marketing Research.
The role of Decision Support System(DDS) is very important in marketing information
system because it provides “ready-to-use” information to the marketing decision-makers. Its
significant role in marketing information system can be viewed in the following ways -
 Decision Support System is a set of statistical and mathematical tools and decision
models that help the marketing managers in analyzing data gathered through the sub-
systems of MIS for making rational decisions.
 Analytical system helps marketers to make decisions. In other words, analytical systems
allow a firm to ask, "Why did that happen?" and "What will happen if……..?"
Mathematical and/or statistical tools and decision models are used to solve several
marketing problems such as media selection problems to evaluate new product
acceptance, weighing sales and pricing alternatives, and so forth.
 It maintains a “Data Bank” which can supply required ready-to-use data to the decision-
makers whenever required.
8.
a) What is target market? Explain the evaluation criteria to select the target market
for business purpose. (2+3=5)
b) What is packaging? Explain the features of good packaging. (2+3=5)

ORK
(21)
Answer:
a) Target market consists of specified or identified market segments where the marketer wants
to enter for obtaining expected benefits. In other words, target markets are those groups of
people whom the firm wants to serve or wants to design product or services to meet their
needs and wants.
Selection of appropriate target market is essential for every marketing firm. This task can
be done by adopting the following evaluation criteria:
 Predict demand for each segment of a product market,
 Analyze competitive position of the company in each segment.
 Predict segment size and profit size.
 Analyze and assess the potential corporate goals and objectives in the market
segment.
b) Packaging is one of the very important parts of product development process. Most
physical products have to be packed. It is concerned with all the activities of designing and
producing the container or wrapper for a product. Properly designed packages help to
promote sale, convenience to consumers and protection to the product. It also helps to
create brand image and facilitate self service system in retail stores. The following are the
features of good packaging:
-convenient
-attractive
-economical
-protective
-communicative
-environment friendly
9.
a) Identify the components of physical distribution system and explain the
importance of transportation decision in marketing system. (2+3=5)
b) What is pricing? Explain how price is determined under-going rate pricing
method? (2+3=5)
Answer:
a) Physical distribution is concerned with the management of physical flow of goods from the
points of suppliers to the points of purchasers. The main objective of physical distribution
is getting the right products safely to the right places at the right time at the least possible
cost. More specifically, the objectives of physical distribution are:
a. To provide customer service;
b. To distribute goods more safely at right time;
c. to minimize the total cost; and
d. To supply goods to the right target market.
Physical distribution system consists of a set of interrelated functions with the following
elements:
i. Order Processing
ii. Inventory Management & Control
iii. Warehousing
iv. Materials handling and
v. Transportation
ORK
(22)
vi. Channel information management.
Transportation plays a significant role in marketing system in the following ways:
i. It assures regular mobility of factors of production from one place to another. It makes
continuous production possible;
ii. It facilitates distribution of goods within a short period of time i.e., it ensures quick
delivery of goods;
iii. It makes shopping of goods convenient throughout the country and the world as well;
iv. It can create adequate demand for the company' products;
v. It makes possible to link many geographically separated markets;
vi. It ensures regular supply of goods even during the period of shortages, and hence, helps
to stabilize market prices;
vii. It is the main basis of economic and social development;
viii. It discourages monopoly and promotes healthy competition by bringing together the
products of various manufacturers in the market;
ix. It ensures regular supply of the required factors of production to the industries and final
products to the market, and hence, leads to specialization in production sector;
x. It promotes both agricultural and industrial development in the country;
xi. It develops sound political relations among the countries by developing social and
cultural linkages between the countries.
b) Price is the value placed on what is exchanged. It is measured in monetary terns. It is the
act of determining the exchange value between the purchasing power and satisfaction
derived through the purchase and consumption of products. Firms are having different
objectives, policies and methods of setting prices. So price is one of the important elements
of the marketing mix that produces revenue, the other element produce costs. In most case
price is set by negotiation between buyers and sellers. Organizations have to handle prices
very carefully.
In going rate pricing, the company bases its price mainly on competitors prices. It is
competition oriented, so the company may fix same, more or less price than that of
competitors. It is useful mainly for homogeneous products. The smaller organizations
follow the leader. This method is popular as it is easy and useful where costs are difficult to
measure or competitive response is uncertain.
10. Briefly explain the following: (5×2=10)
a) E-commerce
b) Sales force promotion tools
c) Labeling
d) Display advertising
e) Relationship marketing
Answer:
a) Electronic commerce, commonly known as e-commerce, is a type of marketing where buying
and selling of product or service is conducted over electronic systems such as the Internet
and other computer networks. Electronic commerce draws on technologies such as mobile
commerce, electronic funds transfer, supply chain management, Internet marketing, online
transaction processing, electronic data interchange (EDI), inventory management systems,
and automated data collection systems. Modern electronic commerce typically uses

ORK
(23)
the World Wide Web although it may encompass a wider range of technologies such as e-
mail, mobile devices, social media, and telephones as well.
Electronic commerce also consists of the exchange of data to facilitate the financing and
payment aspects of business transactions.
b) Sales Force Promotion Tools are important for sales promotion. Sales force indicates the
sales staff appointed by the manufacturers to sell manufactured products to the resellers and
customers through personal selling. They establish direct contact with the customers,
motivate them and sell the manufactured products.
Effective sales through sales force are possible when they are motivated. Dissatisfied sales
force cannot motivate the customers. Therefore, manufacturer offer sales promotion tools
directed at the sales force. Some of the sales promotion tools directed at the sales force are as
follows:
i. Sales Contest
ii. Bonus and Commission
iii. Promotional Kits
iv. Gift Items
v. Trade Shows and Conventions
c) Label is an important feature of a product. Every physical product must carry a label, which
may be a simple tag attached to the product or an elaborately designed graphic that is part of
the package. Labels perform several functions. First, the label identifies the product or brand.
The label might also grade the product, the way vegetables are graded A, B, and C. The label
might describe the product: who made it, where it was made, when it was made, what it
contains, how it is to be used, and how to use it safely. Finally, the label might promote the
product through attractive graphics.
d) Display advertising is a Media of advertising. Under this media, goods are displayed
beautifully to assist the salesmen. It can attract the attention of the prospects and arouse their
interests. In this type of advertising, goods and the like are properly displayed in shops,
windows, fairs and exhibitions, walls, sky, electric signs etc.
Handicraft products, fashion products, and other most commonly used products are generally
displayed in the windows to attract the tourists. Some marketers arrange their store with
lighting and beautiful colors. These days' electric signs are getting popular. Electric signs can
be large permanent sign installed at the main centers of the city. They are built of steel
beams, metal sheets, plastics using bright and flashlights, animation movies, neon signs.
During the nighttime they look beautiful and attractive. Name and shape of the products are
attractively displayed in larger shape to attract the attention of the viewers.
e) Relationship marketing has the aim of building long-term relations with key parties such as,
suppliers, distributors. In return, company earns profit. It is concerned with building an
effective network of relationships with key stakeholders. Sellers offer quality products and
services for greater satisfaction of their customer at a fair price in proper time.
The relevance of relationship marketing is clear from the following facts:
- Cuts down transaction cost and time.
- Builds effective marketing network.
- Develops mutually profitable business relationship.
- Increases company profit, etc.
This new philosophy is getting popularity in the field of marketing.
ORK
CAP-II, Income Tax & VAT, June 2013
Suggested Answer
Roll No……………. Maximum Marks - 100
Total No. of Questions - 6 Total No. of Printed Pages -3
Time Allowed - 3 Hours
Marks
Attempt all questions. Working note should form part of the answer.

1. Answer the following with reference to the Indian Income Tax Act, 1961:
a) What do you mean by "assessment" and "assessment year"? A Sri Lanka based ship
company carried the goods on first day of February, 2011 from Sri Lanka to a port in
India. The ship was departed for Sri Lanka on 15th February, 2011. As per
agreement, freight of the carriage shall be paid after 2 months of goods delivered at
the port. Determine the time of taxable on income of freight and assessment year
with reference to the provisions if: 6
i) The Ship was chartered by the Indian.
ii) The Ship was chartered by the transporter of Sri Lanka and amount
was also paid in Sri Lanka.
b) "Income which are not included while computing total income if it satisfies certain
conditions, in case of Political Party ". Comment. 4

Answer a) In general context, the word "assessment" means computation of tax and procedure for
imposing tax liability. As per section 2(8) of Indian Income Tax Act, 1961 the word
"assessment" is defined to include reassessment and "assessment year" means the period of
twelve months commencing on the 1st day of April every year as per section 2 (8) & (9) of
the act.

Indian Income Tax, section 172 is applicable for the purpose of the levy and recovery of tax
in the case of any ship, belonging to or chartered by a non-resident, which carries
passengers, livestock, mail or goods shipped at a port in India. When these conditions are
satisfied, 7.5 per cent of the amount paid or payable on account of such carriage to the
owner or the charterer or to any person on his behalf, whether that amount is paid or payable
in or out of India, shall be deemed to be income accruing in India to the owner or charterer
on account of such carriage. Before the departure from any port in India of any such ship,
the master of the ship shall prepare and furnish a return of the full amount paid or payable.
On receipt of the return, the Assessing Officer shall assess the income and determine the
sum payable as tax thereon before the expiry of nine months from the end of the financial
year in which the return is furnished. In this case, the income of previous year is not taxable
in the immediately following assessment year but it is taxable in the same year in which
freight is paid or payable.

The time of taxable and assessment year

i) The Ship was chartered by the Indian, the time of taxable before 15th February, 2011. The
ownership was with the non-resident. In this case, previous year and assessment year were
irrelevant but tax officer could assess before December end 2011.

AWN
P.T.O.
(25)

ii) The Ship was chartered by the transporter of Sri Lankan and amount was also paid in Sri
Lanka. Both parties were non-resident, so the time of taxable before 15th February, 2011.
Previous year and assessment year were irrelevant but tax officer could assess before
December end 2011.

b) As per section 13A of Indian Income Tax Act, 1961, the following incomes derived by a
political party are not included while computing its total income;
i. Income which is chargeable under the head „Income from house property‟, 'Capital
gains' and 'Income from other sources' and
ii. Any income by way of voluntary contributions.

The exemption of the above income shall be available only when the following conditions
are satisfied:
i. the political party keeps and maintains such books of accounts and other documents as
will enable the Assessing Officer to properly deduce its income there from;
ii. where the voluntary contribution from a person exceeds Rs. 20,000, it keeps and
maintains a record of such contribution and the name and address of the person who has
made such contribution;
iii. the accounts of the political party are audited by a Chartered Accountant or other
qualified accountant; and
iv. the treasurer of such political party ( or any authorized person) shall in each financial
year prepare a report in respect of contribution received by the political party in excess
of Rs. 20,000 from any person/company in that year and submit it (before due date of
submission of return of income) to the Election Commission.

2.
a) Green Development Bank Ltd. has appointed Mr. Hira Paudel as CEO for the
effective operation of the bank. As per the TOR with the CEO, annual salary and
allowances Rs. 5,000,000 (net of tax) shall be paid to the CEO by the bank. All tax
related to the employment of the CEO shall be borne by the company and paid to
Inland Revenue Office. Mr. Hira is to be assessed as couple and he claimed Rs.
20,000 of insurance premium. Should the bank deduct the tax amount from his
salary? If yes, then mention the relevant provisions of the Act; compute the tax
liability of Mr. Hira and deductible salary and allowances for this payment on the tax
assessment of the bank. 5
b) DCM Pvt. Ltd. is registered as a special industry in 2063. It is a food production
industry and located in underdeveloped area. The company has not submitted its
estimated tax return and income tax return for the year 2067/68 and 2068/69. It has
also not paid any installments. Mention the relevant provision on the payment of tax
in installment and calculate the fees and interest under Section 117, 118 and 119 of
the Income Tax Act, 2058 till Chaitra end 2069 on the basis of following information
for the year 2067/68. 5

The management provides the following information:

AWN
P.T.O.
(26)

 Total sales Rs. 50,000,000


 Cost of sales Rs. 35,000,000
 Total allowable administrative expenses pertaining to this year is Rs. 5,000,000

c) Explain the followings with reasons; on tax/withholding taxes and implication


thereon, with reference to the Income Tax Act, 2058. (5×2=10)
i) X & Co., Chartered Accountants, has entered into a contract for tax consultancy
services to Z Ltd. for Rs. 200,000 per month. X & Co. raised VAT invoice for the
month of Shrawan, 2068 amounting to Rs. 226,000 (Rs. 200,000+13%VAT =
Rs. 226,000).
ii) Mr. Hari, an individual person, owned the furnished building situated at Kalanki,
Kathmandu. He has given on lease the furnished building to a development bank
for Rs. 40,000 per month from Shrawan 2068. Mr. Hari is not conducting any
business and has rent income only.
iii) Mr. Krishna is an economist. He published an article on "Current Status of
Economy" in Kantipur daily and received Rs. 6,000. He also teaches, occasionally,
"Economics" in an educational institution for which he received Rs. 10,000 for a
class in Income Year 2068/69.
iv) Mrs. Sila has opened the account in Paschimanchal Rural Development Bank,
Parbat and deposited Rs. 60,000. The interest rate is 8% per annum and Mrs. Sila
earned interest income of Rs. 4,800 during Income Year 2068/69.
v) A & Company Ltd. decided to declare and distribute the cash dividend at the rate
of 10% amounting to Rs. 50,000 and bonus shares at the rate of 20% amounting to
Rs. 100,000 out of the profit of Income Year 2067/68, from its annual general
meeting dated Mangsir 28, 2068.

Answer a) The bank is liable to pay withholding tax from his salary & allowance. As per section
87 of Income Tax Act, each resident employer shall deduct withholding tax at the rate
referred to in Schedule-1 while making payment having the source in Nepal. In this
case, since Mr. Hira is getting salary & allowance net of tax, the bank has to do
backward calculation of gross salary.

Computation of tax on income of Mr. Hira

Salary & allowance Rs. 5,000,000


Less Insurance premium Rs. 20,000
Remaining taxable amount Rs. 4,980,000

1 % on 200,000 Rs. 2,000


15 % on Rs. 100,000 Rs. 15,000
25 % on Rs. 2,200,000 Rs. 550,000
35% on Rs. 2,480,000 Rs. 868,000
Total tax Rs. 1,435,000

Total tax amount Rs.1, 435,000 is payable if the total salary is Rs. 5,000,000. But
in this case Rs. 5,000,000 is net of tax payment. Hence, the amount calculated by
performing backward calculation of gross salary becomes taxable amount. To
AWN
P.T.O.
(27)

calculate gross salary including this tax amount, this tax amount is grossed up
considering the maximum effective tax rate for the person. In this case, the
maximum effective tax rate (including additional tax of 40% on highest rate of
25%) 35 %. So, net of tax amount is Rs. 1,435,000 (65 %) and total tax liability on
this payment is Rs.2, 207,692 (1,435,000/65%).

Total salary and allowance paid to him Rs. 5,000,000


Tax liability Rs. 2,207,692
Allowable salary & expenses Rs. 7207,692

b) Income tax act, 2058 section 94 (1) states that a person who has or will have assessable
income in any income year from any business or investment has to pay tax in three
installments as follows:

Date on which payment has to be made Amount to be paid

By the end of Poush out of 40% of the estimated tax amount of


due and payable tax
By the end of Chaitra out of 70% of the estimated tax amount of
due and payable tax
By the end of Ashad out of 100% of the estimated tax amount of
due and payable tax

Calculation of Total Tax liability


Applicable tax rate is 20 % for special industry as per schedule 1 of income tax act and 30
% of 20 % for the special industry of underdeveloped area.

Taxable income (WN 1) Rs. 10,000,000


Tax rate applicable 6%
Tax assessment under section 99 for the year Rs. 600,000
Fees under section 117 (1) (ka) Rs. 2,000
Fees under section 117 (1) (ga) (WN 2) Rs. 75,000
Interest under section 118 (1) (WN 2) Rs. 42,525
Interest under section 119 from Kartik 2068 to Chaitra 2069 Rs. 135,000

Total tax payable at the end of Chaitra, 2069 is Rs. 854,525.00

Working note 1:
Total sales Rs. 50,000,000
Less cost of sales Cost of sales Rs. 35,000,000
Less allowable administrative expenses Rs. 5,000,000
Taxable Income Rs. 10,000,000

Working Note 2: Calculation of Fees and interest


 Under section 117 (1)(ga): Total assessable income as per the section is Rs. Rs.
50,000,000, fees 0.1 % annually of Rs. 50,000,000 for 18 months (from Kartik 2068 to
Chaitra 2069) is Rs. 75,000 or Rs. 100 per month whichever is higher.

 Under section 118: total tax Rs. 600,000, interest shall be charged on Rs. 540,000 (90
% ) of the tax amount as follows:

AWN
P.T.O.
(28)

Installments To be paid Period for interest calculation Interest


Rs. amount Rs.
Poush end 216,000 Magh 2067 to Ashoj end 2068 24,300
Chaitra end 162,000 Baishakha 2068 to Ashoj end 2068 12,150
Ashad end 162,000 Ashad 2068 to Ashoj end 2068 6,075
Total 540,000 42,525

c)

i. As per section 88(1)(4) of Income Tax Act, 2058, any payment made to the service
provider registered under VAT shall be subject to withholding tax at the rate of 1.5%.
Hence, X & Co. is service provider (tax consultancy service) registered under VAT, the
payment of invoice for the month of Shrawan 2068 is subject to withholding tax at the
rate of 1.5% and X & Co. is entitled to receive Rs. 223,000 {(Rs. 200,000 less 1.5%
withholding tax) + VAT Rs. 26,000}.

ii. As per section 92(1)(kha) of Income Tax Act, 2058, any payment to an individual
person having source in Nepal against leasing of land or building and associated fittings
and fixtures for other than conducting business, is subjected to final withholding. Since
the rent income of Mr. Hari is subject to final withholding, it is not required to be
included in his income. The payment of Rs. 40,000 per month is subject to final
withholding taxes at the rate of 10% i.e. Rs. 4,000 and Mr. Hari will get Rs. 36,000 per
month.
iii. As per section 88(4)(Ka1) of Income Tax Act, 2058, any payment made for the
publication of articles in newspaper is not subject to withholding tax. Hence, Kantipur
daily will make the full payment and is not liable to deduct withholding tax on payment
of Rs. 6,000. But Mr. Krishna has to include the amount received as income in his
return of income.
As per section 92(1)(ja) payment made for occasional teaching is subject to deduction
of final withholding taxes. Hence, amount received by Mr. Krishana against occasional
teaching is subject to final withholding tax at the rate of 15% i. e. 1,500 and he will
receive Rs. 8,500. The amount received by Mr. Krishana is not included in his income.

iv. As per section 88(4)(cha) of Income Tax Act, 2058, payment of interest on deposit, by
a Rural Development Bank based on rural community, to the extent of Rs. 10,000 is not
subject to withholding tax. Hence, the bank is not liable to deduct withholding tax on
payment of interest of Rs. 4,800.
v. As per section 88(2)(ka) of Income Tax Act, 2058, payment of dividend from a source
in Nepal, by a resident person is subject to withholding tax at the rate of 5%. Further, as
per section 53, distribution from an entity includes capitalization of profit and dividend
tax is attracted on such distribution as per section 54.

Hence, amount of Rs. 1,50,000 is subjected to dividend tax at the rate of 5% i. e. Rs.
7,500.

AWN
P.T.O.
(29)

3.
a) State the decisions under which a revision petition can be filled before the Inland
Revenue Department under the Income Tax Act, 2058. 5

b) List out the payments not included in the income from employment under the
Income Tax Act, 2058. 5

c) Explain on the taxability and the implication thereon, of the following transactions
as per the Income Tax Act, 2058. (6+4=10 )

i) A Den airline registered in Denmark, having contact office in Nepal and is


operating its airlines business. During Income Year 2068/69, it has sold the
tickets in Nepal as follows:
1. Sale of tickets from the passengers departing from Nepal - Rs. 50 crores.
2. Sale of tickets in Nepal, for the passengers departing from country other
than Nepal – Rs. 10 crores.
ii) Singtel Ltd. is a company registered in Singapore. The company, with its
objective to transmit information and storing data, has a communication hub in
Nepal (without any office in Nepal). Through such system, the companies in
Europe and America are storing data and transmitting information. Singtel has
received USD 1 million for such services.
Answer a) In case a taxpayer is not satisfied with any decision of IRO, it has to file an
application, as its first step, to IRD for an administrative review. According toSec
114, an application against the following decisions should be moved to IRD for an
administrative review:
i. Advance ruling issued under Sec. 76 by IRD;
ii. Decision or order to withholding agent under Section 90(8)
iii. Reassessment of estimation of advance payment by a taxpayer made by a Tax
Officer under Section 95(7);
iv. Decision by a Tax Officer to require a taxpayer to file return of tax under Section
96(5) or 97;
v. Decision by a Tax Officer with regard to an extension of time for filling returns
under Sec.98;
vi. Jeopardy assessment under Section 100, an amended assessment under Section
101, an assessment of expenses incurred on auction sales under Section 105(5),
or fees and interest imposed under Sections 122;
vii. Notification by the IRO of an amount to be set aside by a receiver under Section
108(2);
viii. Order by a tax office to a debtor of the taxpayer to pay the amount due to the tax
office instead of to the taxpayer under Section 109(1);
ix. Order by a Tax Officer to a person to pay tax on behalf of a non-resident person
under Section110(1);
x. Decision of IRO on an application by a taxpayer for a refund of a tax under
Section 113(5); and

AWN
P.T.O.
(30)

xi. Decision of IRD on an application by a taxpayer for extension of time within


which to file an objection under Section 115(3).

b) Following are the payments, which are not included while computing income from
employment:
i. Any amount received by an employee for which exemption is given under Section 10 of
the Act and any amount received which is subject to final withholding of tax.
ii. Work-time meals or refreshments provided by the employer in equal terms for all the
employees at working place or uniform applicable to working place only.
iii. Any reimbursement of expenses incurred by the employee:
 That serves the purpose of the business of the employer; or
 That would otherwise be deductible in calculating the individual‟s income from
the business or investment.
 Reimbursement of outstation cost-travelling or daily allowance
iv. Any prescribed small amounts, which are too small and thus unreasonable or
administratively impracticable to make accounting for them. The amount prescribed by
the rule is Rs. 500 at a time. The expenses prescribed by the rule include tea expenses,
stationery expenses, prizes, gifts, emergency medical facility, or other such payments as
specified by IRD.

c) i)
For the provision of section 70 of Income Tax Act, 2058, the 'non-resident' means a resident
entity that is a part of a group of associated entities the main office is situated outside Nepal
(explanation to section 70).

The gross receipts from the following activity of non-resident person are treated as the
taxable income of the person from the activities:
 A person engaged in Nepal in any, air, water or chartered transport services, other than
transmission in Nepal, for the carriage or passengers who embark from within the
territory of Nepal [section 70(1)].

Hence, the gross receipts of Den Airline from the sale of tickets in Nepal departing the
passengers from Nepal amounting to Rs. 50 crores and for the passengers departing from
country other than Nepal – Rs. 10 crores are taxable at the rates of 5% and 2% respectively
as per schedule 1 Section 2 subsection 7

No tax credits shall be allowed to the person to reduce the tax payable by the person under
this section. Further, expenditures incurred in conducting such activities are neither allowed
to be deducted from the taxable income from the above sources nor allowed to be deducted
from any other source of income of the person.
ii.
The gross receipts from the following activity of non-resident person are treated as the
taxable income of the person from the activity:
 A person engaged in the transmission of messages by cable, radio, optical fiber, or
satellite through an apparatus or equipment established in Nepal, irrespective of the
place of origin of messages [section 70(2)].

AWN
P.T.O.
(31)

Hence, the gross receipts of Singtel from the communication hub (apparatus) established in
Nepal amounting to USD 1 million is taxable at the rates of 5 % as per schedule 1 Section 2
sub Section 7 irrespective of the place of origin of messages.

No tax credits shall be allowed to the person to reduce the tax payable by the person under
this section. Further, expenditures incurred in conducting such activities are neither allowed
to be deducted from the taxable income from the above sources nor allowed to be deducted
from any other source of income of the person.

4. Write short note of the followings with reference to Income Tax Act, 2058. (5×2=10)
a) Tax
b) Debt Claim
c) Exempt Organization
d) Underlying Ownership
e) Payment
Answer a) As per sec 2 (Dha), "Tax" means income tax imposed under the Income Tax Act and
includes following payments:-
i. Expenses incurred in the process of creating charge and performing auction of the
property of tax Debtors by the department as mentioned in section 104 (8) (a);
ii. Amount payable by a withholding agent Withholding agent or withholdee under section
90, or amount payable by an installment payer under section 94, and on assessment
under sections 99, 100,and 101; and amount payable by person who required to deposit
tax under section 95 ka
iii. Amount payable to the Department in respect of a tax liability of a third party under
section 107(2), 108(3) or (4), 109(1), and 110(1);
iv. Amount payable by way of interest and fees under Chapter 22; and
v. Amount payable by way of fines in order of the department as per section 129.

b) As per Sec 2(Tha) of Income Tax Act, Debt claim means a right of one person to receive
a payment from another person and includes a right to repayment of an amount paid by
one person to another person as well as deposits in banks and other financial institutions,
accounts receivable, notes, bills of exchange, bonds, and rights under annuities, finance
leases and installment sales.

c) As per sec 2(Dhna), "Exempt organization" means the following entities:


i. Following entities registered with the Inland Revenue Department as an exempt
organization:
a. a social, religious, educational, or a charitable organization of a public
character established without having a profit motive,
b. an amateur sporting association formed for the purpose of promoting
social or sporting facilities not involving the acquisition of gain,

ii. a political party registered with the Election Commission,


iii. a village development committee, municipality or district development
committee,

Provided that, in cases where any person has derived any benefit from the property of
that organization and the monies obtained from that organization except in making
payment for the property or the service provided by any person to that organization or in

AWN
P.T.O.
(32)

discharging functions in consonance with the objective of the organization entitled to


exemption, tax exemption shall not be granted.

d) As per Sec 2 (Ra) of Income Tax Act, "Underlying ownership" means following
ownership:-
i. in relation to an entity, an ownership created on basis of an interest held in the
entity directly or indirectly through one or more interposed entities by an
individual or by an entity in which no individual has an interest; or
ii. in relation to an asset owned by an entity, an ownership of the asset that is
determined on basis of proportion to the ownership held by the persons having
underlying ownership of the entity.

e) As per Sec 2(Ha), "Payment" means:


i. the transfer by one person of money or an asset to another person or the transfer
by another person of a liability to the one person;
ii. the creation by one person of an asset that on creation is owned by another
person or the decrease by one person of a liability owed by another person;
iii. the provision by one person of services to another person; and the use, or
availability for use, of an asset owned by one person to another person.

5.
a) The Z Pvt. Ltd. has booked following expenses of imported electronic goods in the
ledger. 8
Cost of material as per invoice Rs. 200,000
Bank charges of L/C Rs. 15,000
Insurance Rs. 5,000
Calcutta port clearing expense Rs. 20,000
Freight up to custom point Rs. 14,000
Fright from custom point to Kathmandu Rs. 7,000
Other tax paid at custom including local development tax Rs. 2,500
Nepal custom expenses including 1 % custom duty Rs. 30,000
Discount receivable Rs. 12,000
Further information:
Discount amount has not been deducted in invoices. The above cost is the
gross purchase.
i) Explain the taxable value in case of imported goods with reference to the
provisions of the Value Added Tax Act, 2052.
ii) Compute the Value Added Tax payable at custom point as per above
information.
iii) Assume that the company has 20 % gross profit. What would be its sales price
and VAT amount?
b) Gramin Krishi Sewa Pvt. Ltd. sales vegetables and fruits. During the month of
Baishakh 2070 following sales and purchase made. Compute the net Value Added
Tax payable under the Value Added Tax Act, 2052. 8
Items Purchase (Rs.) Sales (Rs.)
Green Tea 50,000 57,500
Barley 40,000 48,000
Garlic 30,000 37,500

AWN
P.T.O.
(33)

Black Tea 80,000 100,000


Bhakti Fish 150,000 175,000
Millet 45,000 49,500
Paneer 70,000 78,400
Curd 60,000 64,375
Telephone Bill 5,000
Stationery Purchase 20,000
c) Merchantile Pvt. Ltd. is a company providing telecommunication services in Nepal. It
has purchased a consultancy service from a company in US for setting up and
expanding its operation in Karnali zone of Nepal. The company has paid USD 500,000
against such services in the month of Jestha 2069. Merchantile Pvt. Ltd. is of the view
that since payment is made to foreign company there is no implication of Value Added
Tax. State your view referring relevant provision of the Value Added Tax Act, 2052. 4
Answer a)
i) As per section 12(5) of VAT act, 2052, the taxable value for any imported goods
shall be its customs value including transportation, insurance, freight, commissions
of agents and other persons, plus customs duties, countervailing duties plus any other
taxes if levied on imports, except the value added tax. As per the provision, all
expenditures upto the custom point is related to determine the taxable value of
imported goods.

ii) Calculation of taxable value and VAT payable at custom point:

Cost of material as per invoice Rs. 200,000


Insurance Rs. 5,000
Calcutta port clearing expense Rs. 20,000
Freight up to custom point Rs. 14000
Total cost upto custom point Rs. 239,000
Other tax paid at custom Rs. 2,500
1 % custom duty (1 % of Rs. 239,000.00) Rs. 23,90
Total taxable value at custom point Rs. 243,890
VAT payable at custom point Rs. 31,705.70

Note:
 Bank charge is processing cost, so it is not included in the cost at custom point.
 Fright from custom point to Kathmandu is not included at custom point.
 Custom duty is included in the taxable value but not included other custom expenses.
Other expenses are incurred in Nepal
 Discount receivable is not deducted because the company has not received at the
time of purchase.

iii) Calculation of sales price and vat amount

Cost of material as per invoice Rs. 200,000


Bank charges Rs. 15,000
Insurance Rs. 5,000
Calcutta port clearing expense Rs. 20,000
Freight up to custom point Rs. 14000
Fright from custom point to Kathmandu Rs. 7,000

AWN
P.T.O.
(34)

Other tax paid at custom Rs. 2,500


Nepal custom expenses Rs. 30,000
Discount receivable (Rs. 12,000)
Cost of the goods Rs. 281,500
Gross profit (25 % of Rs.281,500) Rs. 70,375
Sales price Rs. 351,875
VAT receivable on sales price Rs. 45,744
b) Here, output is mixed, vat exempted and vat attractive both. First, we have to segregate
the items in two parts:

Items Purchase (Rs.) Sales (Rs.)

Vat attractive
i. Garlic 30,000 37,500
ii. Black tea 80,000 1,00,000
iii. Bhakti Fish 1,50,000 1,75,000
iv. Paneer 70,000 78,400
Total 3,30,000 3,90,900

Vat exempted
i. Green Tea 50,000 57,500
ii. Barley 40,000 48,000
iii. Millet 45,000 49,500
iv. Curd 60,000 64,375
Total 1,95,000 2,19,375
Grand Total 5,25,000 6,10,275

Vat attractive ratio= Vat attractive sales/Total Sales


= 3,90,900/6,10,275x100
= 64.05%
Input vat credit on Telephone bill = Rs. 5,000 X 13% X 64%
=Rs. 416

Input vat credit on Stationery Bill = Rs. 20,000 X 13% X 64%


=Rs.1,664
Purchase of Garlic,Black tea, Bhakti fish and Paneer are Vat attracted items. Full
vat is allowed for input tax credit.
Output tax is Rs. 50,817 ( 3,90,900 X 13%)
Input tax credit is Rs.44,980 ( 3,30,000 X 13% + 416 + 1,664)
Net Vat payable = Rs. 5,837
c) According to section 8(2) of Value Added Tax Act, 2052, any person (registered or not) in
Nepal, receiving service from a person registered outside Nepal, has to assess and collect
tax as per Value Added Tax Act and rules made thereon. This is called Reverse Charging.

Further, the amount paid as value added tax (VAT) on procurement of such services is
allowed to take VAT credit under section 17(5kha).

Hence, the view of Merchantile Pvt Ltd is not acceptable. It has to assess and collect VAT
on the amount paid (i.e. USD 500,000) against the consultancy services in the month of
AWN
P.T.O.
(35)

Jestha, 2069. VAT is charged at the rate of 13% on the amount paid as consultancy
services.

The amount paid as VAT on such consultancy service is allowed to take VAT credit while
submitting the VAT return.

6.
a) What are the records to be maintained by a registered person dealing in used or second
hand materials? How the tax is assessed in such case? Answer with reference to the
Value Added Tax Rules, 2053. 5
b) Explain about the “Tax Periods” under the Value Added Tax Act/Rules. 5
c) Describe the circumstances beyond the control under the Value Added Tax Act, 2052. 10

Answer a) As per rule 33 of Value Added Tax Rules, 2053, following are the provisions regarding
records to be maintained, for a registered person dealing in used or second had
goods.
1) A registered person who is dealing in used or secondhand goods has to maintain
purchase register and sales register containing the following particulars:
Relating to purchases:
i. Date of purchase
ii. Particulars giving full information of the goods
iii. Buying price excluding tax
iv. Rate of tax
v. Amount of tax
vi. Total amount paid

Relating to Sales:
i. Date of sale
ii. Selling price excluding tax
iii. Difference between the buying price and selling price
iv. Rate of tax
v. Amount of tax
vi. Total amount received.

(2) In case the buying price of every item of used goods exceeds Rs. 10,000, separate records
of buying or selling shall be maintained.

(3) In case a registered person is found not to have satisfactorily maintained the records as
prescribed above, tax officer may impose VAT on the total selling price of the goods sold
by such taxpayer, and the tax officer may issue a written order requiring him to pay such tax
along with the next tax return.

(4) In case of used or second hand goods, tax shall be assessed on the amount which is
difference between the selling price and buying price. Buying price means price including
taxes.

AWN
P.T.O.
(36)

b) Tax Period means a period prescribed by the Act or Rules for calculation of net VAT
payable or receivable.

Generally, a registered person has to adopt a month as per Nepali Calendar as tax period.
The tax starts from day 1st of a month and ends at end day of the same month.

As per Rule 26 of VAT Rule, 2053, the different tax periods could be adopted by certain
specific taxpayers.

i. Tax Period of 2 months:


Tax period of 2 months is allowed to a taxpayer having taxable transaction during
previous 12 months more than NPR 2 million but not more than NPR 10 million.
The tax period shall be, Shrawan and Bhadra, Ashwin and Kartik, Marg and Paush,
Magh and Falgun, Chaitra and Baisakh and Jestha and Ashad.
If hotel and tourism enterprises opt, the department may be allowed a tax period of 2
months.
ii. Tax period of 4 months:
Tax period of 4 months is allowed to a taxpayer who has taken the registration
voluntarily and having taxable transaction during previous 12 months less than NPR
2 million.
The tax period shall be Shrawan to Kartik, Marg to Falgun, and Chaitra to Ashad.
iii. Different tax period:
In case a registered person who maintains its accounts adopting computer systems, it
may apply to a tax officer for allowing it to adopt different tax period. In case the tax
officer finds it proper, it may allow the taxpayer to adopt different tax period.
iv. Tax period for first time of registration:
In case of a person has obtained registration at a middle of a month, the tax period
shall be started from the date on which the registration is obtained to the end of the
month.

c) As per Rule 35 of Vat Rule 2053, the following circumstances shall be deemed to be
circumstances beyond control for the purpose of sub-section (4) of Section 19 of the
Act:
(i) In case the person required to pay tax becomes disabled due to falling ill; up
to seven days of the date of his recovery.
(ii) In case the person required to pay tax is to observe obsequies; up to seven
days of the end of the obsequies,
(iii) In case a woman required to pay tax delivers a child; up to thirty five days of
the date of delivery,
(iv) In case the person required to pay tax dies or becomes insane or disappears
and his heir or guardian submits an application within thirty five days of the
date of such incident; up to seven days of receipt of such application,
(v) In circumstances when the person required to pay tax has not been able to
come to the IRO because of the closure of a road due to floods, landslides or
similar other reasons; up to seven days of opening of the road,
(vi) In circumstances when he cannot come to the IRO due to total haltage of
transport; up to the next day of the end of such haltage.

AWN
P.T.O.
(37)

vii. In case where the natural calamities like fire earthquake, arises; up to thirty days
from the date when such calamities occur.

In case an additional time limit shall be required to be requested due to circumstances


beyond control referred to point (ii), (iii), (iv), (v) & (vii) above ; the recommendation of
the concerned Village Development Committee or Municipality shall be submitted.

While requesting for an additional time-limit due to the circumstance referred to point
no. (vi) , the recommendation of the Village Development Committee or Municipality
concerned with the place where the haltage of means of transport has taken place, shall
be submitted.

AWN
P.T.O.
Advanced Accounting

Suggested
Roll No……………. Maximum Marks - 100
Total No. of Questions - 6 Total No. of Printed Pages - 5
Time Allowed - 3 Hours
Marks
Attempt all questions. Working notes should form part of the answer.
1. A, B and C are partners of M/s Sun Trading sharing profits and losses in the ratio of
3:3:2. Their balances as on July 16, 2015 is as under:
Particulars Amount (Rs.) Amount (Rs.)
Fixed Assets:
Cost 360,000
Less: Depreciation 216,000 144,000
Investment:Cost (Market Value Rs. 250,000) 120,000
Working Capital 400,000
Total 664,000
Financed by:
Loans from D 180,000
E 120,000 300,000
Reserves 264,000
Capitals B 240,000
C 120,000
360,000
Less: A 260,000 100,000
Total 664,000

On that day, A retired from business. B and C decided to admit D as a partner and E,
who is a minor, to benefits of partnership. B, C, D and E are to share profits in the
ratio of 3:3:2:2. Losses, if any, are to be borne by B, C and D in the ratio of 3:3:2.
For the purpose of above retirement-cum admission, it is decided that:
a) Goodwill of the firm is valued at Rs. 250,000. However, no account is to appear
for goodwill. Treatment of goodwill in relation to minor is to be deferred to the
date when he opts to be a partner on attaining majority, when the value of
goodwill will be deemed, for this purpose, to remain unchanged at Rs. 250,000.
b) A takes over: (i) Fixed Assets (cost Rs. 125,000; WDV Rs.55,000) for
Rs.100,000; and (ii) Investments (cost Rs. 15,000) at their market value of Rs.
12,500.
c) The continuing partners take over the remaining assets at their book values,
except investments which are taken over at their market values.
d) Loans are to be transferred to capital accounts.
e) The amount due to/from A is to be settled by Cheque immediately.
You are required to prepare: (a) the Revaluation / Adjustment Account; (b) the
Capital Accounts; and, (c) the Balance Sheet of the firm after giving effect to all the
above points of agreement. 20
Answer
a) Revaluation Account:

JZU P.T.O.
(2)
Particulars Amount Particulars Amount
To Investment A/c 2,500 By Fixed Assets A/c 45,000
To Partners Capital A/c By Investment A/c 132,500
A 65,625
B 65,625
C 43,750
177,500 177,500
b) Partners Capital Account:

Particulars A B C D E Particulars A B C D E
To Balance By Balance
B/d 260,000 c/d 240,000 120,000
To A's Capital 31,250 62,500 By C's Capital 31,250
To Fixed By D's
Assets 100,000 Capital 62,500
To
Investment 12,500 By Reserves 99,000 99,000 66,000
To Balance By
c/d - 404,625 198,500 117,500 120,000 Revaluation 65,625 65,625 43,750

By Loan A/c - - - 180,000 120,000


By Bank A/c 114,125

372,500 404,625 229,750 180,000 120,000 372,500 404,625 229,750 180,000 120,000
c) Balance Sheet of the new firm as on 16.07.2015.

Liabilities Amount Assets Amount


Capital Accounts Fixed Assets (Cost Less 89,000
Dep)
B 404,625 Investment (Market Value) 237,500
C 198,500 Working Capital 514,125
D 117,500
E 120,000
840,625 840,625

Working Notes:
1. Adjustment of Goodwill:

Goodwill Value: Rs. 250,000


Particulars A B C D
A. Credited in the old Profit Sharing ratio (Rs.) 93,750 93,750 62,500
B. Debited in the new profit sharing ratio (Rs.) - 93,750 93,750 62,500
Required Adjustment (A-B) 93,750 - (31,250) (62,500)

Note: E’s Account cannot be debited for writing – off goodwill since he is a minor.
2. Provision for Depreciation Account:

Particulars Amount Particulars Amount


To Assets Account 70,000.00 By Balance b/d 216,000.00
To Balance c/d 146,000.00
216,000.00 216,000.00
3. Fixed Assets Account:

Particulars Amount Particulars Amount


JZU P.T.O.
(3)
To Balance b/d 360,000.00 By Prov for Depreciation 70,000.00
To Revaluation A/c 45,000.00 By A's Capital A/c 100,000.00
By Balance c/d 235,000.00
405,000.00 405,000.00
4. Investment Account:

Particulars Amount Particulars Amount


To Balance B/d 120,000.00 By A's Capital 12,500.00
To Revaluation A/c 132,500.00 By Revaluation A/c 2,500.00
By Balance c/d 237,500.00
252,500.00 252,500.00

5. Ascertainment of Working Capital:


Opening working Capital: Rs. 400,000
Add: Cheque by A: Rs. 114,125
Closing Working Capital: Rs. 514,125

2.
a) A Commercial Bank has the following capital funds and assets. Segregate the
capital funds into core capital (Tier I) and supplementary capital (Tier II). Find
out the risk weighted asset and capital fund ratio.
Particulars (Rs. in crores)
Equity share capital 500.00
Statutory reserve 270.00
Capital reserve (of which Rs. 16 crores were due to
revaluation of assets and the balance due to sale of capital 78.00
asset)
Assets:
Cash balance with Nepal Rastra Bank (NRB) 10.00
Balance with other banks 18.00
Other investments 36.00
Loans and advances:
(i) Guaranteed by the Government 16.50
(ii) Others 5,675.00
Premises, furniture and fixtures 78.00
Off-Balance Sheet items:
(i) Advance Payment Guarantee 800.00
(ii) Contingent Liability in respect of Income Tax 4,800.00
10
st
b) On 31 Ashadh 2072 Varun Limited with an issued and subscribed capital of
Rs.700,000 divided into 70,000 fully paid equity shares of Rs.10 each had net
tangible assets of Rs.950,000 and Goodwill amounting to Rs.50,000. On this date
Paul Limited took over Varun Limited‟s business for Rs.1,300,000 payable as to
Rs.100,000 cash and Rs.1,200,000 in 100,000 equity shares of Rs.10 each valued
at Rs.12 per share. However it was decided that the shares so allotted should be
recorded by both the companies in the books at par value only.
On the date of purchase of business, Varun Limited‟s debtors included Rs.20,000
due from Paul Limited for goods sold to it at the profit of 25% on cost. Paul
Limited had sold only 1/4th of these goods by the date of absorption, the
remaining goods lying unsold in stock.
JZU P.T.O.
(4)
You are required to pass necessary journal entries in the books of both the
companies. 10
Answer
a)
(i) Rs in crores Rs in crores
Core Capital- Tier I
Equity share capital 500
Statutory reserve 270
Capital reserve (arising of sale of assets) (78-16) 62
832
Supplementary Capital – Tier II
(Capital Reserve (arising out of revaluation of 16 0.32
assets)
As per NRB Directive- Capital Reserve relates
with revaluation reserve of assets, upto 2% of
total Supplementary Capital only allowed for the
computation)
Total Capital Fund 832.32

Rs in crores % of weight Rs in crores


(ii) Risk Weighted Assets
Funded Risk Assets
Cash balance with NRB 10 0 0
Balance with other banks 18 20 3.60
Other investments 36 100 36
Loans and advances:
i) Guaranteed by the government 16.5 0 0
ii) Others 5,675 100 5,675
Premises, furniture and fixtures 78 100 78
5,792.60
Rs in crores Credit
conversion
factor
Off-Balance Sheet items:
Advance Payment Guarantees 800 100 800
Contingent Liability in respect of
Income Tax 4,800 100 4,800
11,392.60

Capital Fund Ratio:


Capital fund x100
Risk weighted assets
(832.32/11,392.60) x 100 =7.31%

b) Working Note:
Summarized Balance Sheet of Varun Limited as on 31stAshad 2072:

Share capital Goodwill 50,000


Issued and Subscribed Net Sundry Tangible Assets 9,50,000
70,000 equity shares of Rs.10
Each fully paid up 7,00,000
Reserves 3,00,000

JZU P.T.O.
(5)
The consideration on the basis of par value of shares would be:
Rs.
Cash 1,00,000
1,00,000 Equity shares of Rs.10 each 10,00,000
11,00,000

In the books of VarunLimited


Journal Entries

Date Particulars Dr Cr
Amount Amount
2072 Realization A/c Dr. 10,00,000
31/3 To Sundry Net Tangible Assets 9,50,000
To Goodwill 50,000
(Being transfer of goodwill and other sundry
tangible assets to realization account.)
31/3 Paul Limited A/c Dr. 11,00,000
To Realisation A/c 11,00,000
(Being amount calculated on the basis of par
value of shares due to Paul Limited for
business sold to it.)
31/3 Bank. A/c Dr. 1,00,000
Equity Shares in Paul Limited Dr. 10,00,000
To Paul Limited A/c 11,00,000
(Being receipt of cash and equity shares in
Paul Limited at [ar from Paul Limited in
discharge of the purchase consideration for
the business sold to it.)
31/3 Equity Share Capital A/c Dr. 7,00,000
Reserves A/c Dr. 3,00,000
Realization A/c Dr. 1,00,000
To Equity Shareholders A/c 11,00,000
(Transfer of equity share capital account,
Reserves and profit on realization to the
Equity Shareholders A/c.)
31/3 Equity Shareholders A/c Dr. 11,00,000
To Bank A/c 1,00,000
To Equity Shares in Paul Ltd. A/c 10,00,000
(Being amount received from the insurance
company less commission.)

In the books of Paul Limited


Journal Entries

Date Particulars Dr Cr
Amount Amount
2072 Business Purchase A/c Dr. 11,00,000
31/3 To Liquidator of Varun Limited A/c 11,00,000
(Being amount calculated on the basis of par
value of shares payable to liquidator of
Varun Limited for the business purchased.)
31/3 Goodwill A/c Dr. 1,50,000
Net Sundry Assets A/c Dr. 9,50,000
To Realization A/c 11,00,000
JZU P.T.O.
(6)
(Incorporation of the net sundry tangible
assets taken over and the amount paid for
goodwill..)
31/3 Liquidator of Varun Limited A/c Dr. 11,00,000
To Bank A/c 1,00,000
To Equity Share Capital Account 10,00,000
(Payment of Cash and allotment of 1,00,000
equity shares of Rs.10 each (recorded at par)
to the liquidators of Varun Limited in
discharge of the consideration.)
31/3 Sundry Creditors Dr. 20,000
To Sundry Debtors A/c 20,000
(Elimination of mutual owing.)
31/3 Goodwill A/c Dr. 3,000
To Stock A/c 3,000
(Being elimination of unrealized profit in
respect of 3/4th of goods purchased earlier
from Varun Limited and remaining unsold on
the date of absorption.)
3.
a) D Ltd., which operates a wholesale business, had a fire on premises on
September 17, 2014, which destroyed most of the premises, although stock to the
value of Rs. 3,960 was salvaged. The company has an insurance policy (with
suitable average clauses) covering Stocks for Rs. 600,000, Building for Rs.
800,000, and Loss of Profits including standing charges for Rs. 250,000 with a
six months period of indemnity.
The company‟s last Profit & Loss Account, for the year ended 16th July, 2014,
showed the following position:
DR CR
Particulars Amount (Rs.) Particulars Amount (Rs.)
To Opening Stock 412,500 By Sales 2,000,000
To Purchases 1,812,500 By Stock 525,000
To Insured Standing Charges 167,500 By Interest 5,000
To Other Expenses 80,000
To Net Profit for the Year 57,500
2,530,000 2,530,000

The company‟s records show that the sales for period up to September 17, 2014
had been the same as for the corresponding months in the previous year at Rs.
100,000, payments made to trade creditors in October-November were Rs
106,680 and at the end of that month the balances owing to trade creditors had
increased by Rs. 3,320. The company‟s business was disrupted until the mid of
December 2015, during which period turnover fell by Rs. 180,000 compared
with the same period in the previous year. It was agreed that three quarters of the
value of the building had been lost and that at the time of the fire it had been
worth Rs. 1,000,000.
Ascertain the amount of various claims to be lodged with the insurers. 10
b) M/s Manakamna Limited has three department stores A, B & C. From the
particulars given below the values of stock as on 31st December 2015 and the
departmental trading result for the year ended 31st December 2015. 5
JZU P.T.O.
(7)
(a)
A B C
(Rs.) (Rs.) (Rs.)
Stock as on 1st January 2015 24,000 36,000 12,000
Purchases 146,000 124,000 48,000
Actual sales 172,500 159,400 74,600
G P on normal selling prices 20% 25% 33.33%

(b) During the year certain items were sold at a discount and these discounts
were reflected in the values of sales shown above. The items sold at
discount were:
Particulars A B C
(Rs.) (Rs.) (Rs.)
Sales at normal price 10,000 3,000 1,000
Sales at actual prices 7,500 2,400 600

Answer
a)
(i) Loss of Stock

Trading Account
(for the period ended September 17, 2014)
Dr Cr
Particulars Amount Particulars Amount
To Opening Stock 525,000 By Sales 100,000
To Purchases 110,000 By Closing Stock
To Gross Profit @ 15% Sales Rs.
100,000 15,000 (Balancing Fig.) 550,000

650,000 650,000
Estimated Stock at the time of fire Rs.550,000
Less: Salvaged Stock Rs. 3,960
Loss of Stock: Rs. 546,040

Working Notes:
1. Gross Profit shown by Last Year‟s Accounts is Rs 300,000 against Rs. 2,000,000 Sales i.e.
15% of Sales. In absence, of any information; the same GP % is assumed for current year
also.
2. Purchases = Payment to Creditors + Increase in Creditors Balance = 106,680 + 3,320 = 110,000.
(ii) Building
Loss: Three quarters of Rs. 1,000,000 : Rs. 750,000
The building was under-insured, as the insurance cover was for Rs. 800,000 only; Hence,
Average Clause is applicable. Insured Covered = 800,000 / 1,000,000 X 100% = 4/5 0or 80%.
Therefore, the amount to be claimed= 4/5 of 750,000 =Rs. 600,000

(iii) Loss of Profit:


Indemnity Period: September 17 to December 15 i.e. 3 Months

JZU P.T.O.
(8)
Profit Ratio for the year ending July 16, 2014
Net Profit (Less Interest) + Insured Standing Charges * 100
Sales
= (57,500-5,000) + 167,500 * 100
2000000
= 11%.
As Short Sales = Rs. 180,000,
Claim for Loss of Profit = 11% of 180,000 = Rs. 19,800.

b) Departmental Trading Account


For the year ending 31st December 2015
A B C A B C
Rs. Rs. Rs. Rs. Rs. Rs.
To Opening Stock 24,000 36,000 12,000 By Sales 172,500 159,400 74,600
To Purchases 146,000 124,000 48,000 By Closing Stock 30,000 40,000 10,000
To Gross Profit 32,500 39,400 24,600

202,500 199,400 84,600 202,500 199,400 84,600

Working Note:
(i) Ascertainment of cost of sales A B C
Rs. Rs. Rs.
Sales at actual price 172,500 159,400 74,600
Less: Discounted sales price (7,500) (2,400) (600)
165,000 157,000 74,000
Add: Sales at normal selling price 10,000 3,000 1,000
Total Sales at normal price 175,000 160,000 75,000

Profit on normal prices 20% 25% 33.33%

4.
a) X Limited sells products on hire purchase terms, the price being cost plus 33
1/3%. From the following particulars for the year ended 31stAshadh 2071,
prepare the necessary accounts on stock-debtors system to reveal the profit
earned: 10
2070
1st Shrawan Stock out on hire at Hire purchase price 400,000
Stock in hand at the shop 50,000
Installments due (Customers still paying) 30,000
2071
31stAshadh Stock out on hire at Hire Purchase price 460,000
Stock in hand at the shop 70,000
Installments due (Customers still paying) 50,000
Cash received during the year 800,000
b) Butwal Hydro Power Company Ltd. came up with public issue of 3000,000
equity shares of Rs. 10 each at Rs. 15 per share. A, B and C took underwriting of
the issue in 3: 2: 1 ratio.
Applications were received for 2,700,000 shares.
The marked applications were received as under:
JZU P.T.O.
(9)

A 800,000 shares
B 700,000 shares
C 600,000 shares
Commission payable to underwriters is at 5% on the face value of shares.
i) Compute the liability of each underwriter as regards the number of shares to
be taken up.
ii) Pass journal entries in the books of Butwal Hydro Power Company Ltd. to
record the transactions relating to underwriters. 5
Answer
Hire Purchase Debtors Account
Date Particulars Amount Date Particulars Amount
2070 2070
01/04 To balance b/d 30,000 01/04 By Cash 8,00,000
01/04 To Hire Purchase stock to By Balance c/d 50,000
to account –total number of 21/03
31/03/ installments which become 8.20,000 2071
2071 due (balancing figure 1)

8,50,000 8,50,000

Hire Purchase Stock Account


Date Particulars Amount Date Particulars Amount
2070 2070
01/04 To balance b/d 4,00,000 01/04 By Hire Purchase Debtors A/c 8,20,000
01/04 To goods sold on hire to (1)
to purchase (balancing figure 2) 8,80,000 31/03
31/03/ 2071
2071 31/03 4,60,000
By Balance c/d
12,80,000 12,80,000

Hire Purchase Adjustment Account


Date Particulars Amount Date Particulars Amount
2071 2070
31/03 To Hire purchase Stock 1,15,000 01/04 By Hire Purchase Stock 1,00,000
Reserve A/c Reserve A/c (opening balance)
2071
31/03 To Profit and Loss A/c 2,05,000 31/03 By Goods sold on hire 2,20,000
(transfer of profit) purchase A/c (Loading 25% of
Rs.8,80,000)

3,20,000 3,20,000

b)
Computation of liability of underwriters in respect of shares
(In shares)
Particulars A B C
Gross liability (Total Issue-Promoters etc) in agreed 15,00,000 10,00,000 5,00,000
ratio of 3: 2: 1
Less: Unmarked applications (Subscribed shares- (3,00,000) (2,00,000) (1,00,000)
marked shares) in 3: 2: 1
JZU P.T.O.
(10)
Marked shares as per agreed ratio 12,00,000 8,00,000 4,00,000
Less: Marked applications actually received (8,00,000) (7,00,000) (6,00,000)
Shortfall / surplus in marked shares 4,00,000 1,00,000 (2,00,000)
Surplus of C distributed to A & B in 3:2 ratio (1,20,000) (80,000) 2,00,000
Net liability for underwriting shares 2,80,000 20,000 Nil

Journal Entries in the books of Butwal Hydro Power Company Ltd.


Particulars Rs Rs
A's Account Dr 42,00,000
B's Account Dr 3,00,000
To Share Capital Account 30,00,000
To Securities Premium Account 15,00,000
(Being the shares to be taken up by the
underwriters)
Underwriting Commission Account Dr. 15,00,000
To A's Account 7,50,000
To B's Account 5,00,000
To C's Account 2,50,000
(Being the underwriting commission due to the
underwriters)
Bank Account Dr. 34,50,000
To A's Account 34,50,000
(Being the amount received from underwriter A
for the shares taken up by him after adjustment of
his commission)
B's Account Dr. 2,00,000
To Bank Account 2,00,000
(Being the amount paid to underwriter B after
adjustment of the shares taken by him against
underwriting commission due to him)
C's Account Dr. 2,50,000
To Bank Account 2,50,000
(Being the underwriting commission paid to C)
Note: C had sold in excess of the underwriting obligation and hence he will not be
required to purchase any shares but will get commission for underwriting.
5.
a) M/s. Zipee Traders sold a car to a customer on installment sales basis.
The sales price of the car is Rs. 6,000,000.
The customer was required to pay following:
Shrawan Down Payment (At the time of
01, 2072 delivery) 3,000,000
Shrawan (includes Interest
01, 2073 At the end of Year 1 1,860,000 of Rs. 360,000)
Shrawan (includes Interest
01, 2074 At the end of Year 2 1,680,000 of Rs. 180,000)
Total Amount 6,540,000
M/s. Zipee Traders booked the entire Rs. 6,540,000 as sales on Shrawan 01, 2072.
With reference to the applicable NAS, explain whether the accounting treatment
of M/s. Zipee Traders is justifiable or not. 5

JZU P.T.O.
(11)
b) Discuss the meaning of Chart of Accounts in Computerized Accounting
Environment and explain how you can improve the exiting chart of accounts so
that it matches the organisations‟ requirement. 5
c) State with reasons, how the following events would be dealt with in the financial
statements of National Trading Ltd. for the year ended 31stAshadh, 2072:
(i) An agreement to sell a land for Rs. 30 lakh to another company was entered
into on 1stAshadh, 2072. The value of land is shown at Rs. 20 lakh in the Balance
sheet as on 31stAshadh, 2071. However, the sale Deed was registered on
15thShrawan, 2072.
(ii) The negotiation with another company for acquisition of its business was
started on 2ndMagh, 2071. National Trading Ltd. invested Rs. 40 lakh on
12thAshoj, 2072. 5
Answer
a) As per NAS 07, 'Nepal Accounting Standard on Revenue: „Revenue attributable to the
sales price, exclusive of interest, is recognized at the date of sale. The sale price is the
present value of the consideration, determined by discounting the installments receivable
at the imputed rate of interest. The interest element is recognized as revenue as it is
earned, using the effective interest method.‟
In the given case:
The sale price of the car is Rs. 6,000,000
The interest portion is Rs. 360,000 in Year 1.
The interest portion is Rs. 180,000 in Year 2.

Since Rs. 6,000,000 is only attributable to the sales price, exclusive of interest, is
recognized at the date of sale; M/s. Zipee Traders treatment of booking the entire amount
Rs. 6,540,000 as sales on Shrawan 01, 2072 is WRONG.

In the given case; Rs. 6,000,000 should only booked as Sales Revenue on Shrawan 01,
2072. Rs. 360,000 should be booked as interest income for F/Y 2072/73 and Rs. 180,000
should be booked as interest income for F/Y 2073/74.
b) Chart of Accounts in Computerized environment:
The chart of accounts is a listing of all accounts used in the general ledger of an organization. The
chart is used by the accounting software to aggregate information into an entity's financial
statements.

The chart is usually sorted in order by account number, to ease the task of locating specific
accounts. The accounts are usually numeric, but can also be alphabetic or alphanumeric.

Accounts are usually listed in order of their appearance in the financial statements, starting with the
balance sheet and continuing with the income statement. Thus, the chart of accounts begins with
cash, proceeds through liabilities and shareholders' equity, and then continues with accounts for
revenues and then expenses. Many organizations structure their chart of accounts so that expense
information is separately compiled by department; thus, the sales department, engineering
department, and accounting department all have the same set of expense accounts.

Typical accounts found in the chart of accounts are:

JZU P.T.O.
(12)
Assets: Liabilities: Stockholders' Equity: Revenue:

 Cash  Accounts Payable  Common Stock  Revenue


 Marketable Securities Accrued Liabilities  Retained Earnings  Sales returns and
 Accounts Receivable Taxes Payable allowances (contra
 Prepaid Expenses  Wages Payable Expenses: account)
 Inventory  Notes Payable
 Fixed Assets  Cost of Goods Sold
 Accumulated  Advertising Expense
Depreciation (contra  Bank Fees
account)  Depreciation Expense
 Other Assets  Payroll Tax Expense
 Rent Expense
 Supplies Expense
 Utilities Expense
 Wages Expense
 Other Expenses

The following points can improve the chart of accounts concept for a company:

 Consistency. It is of some importance to initially create a chart of accounts that is unlikely to


change for several years, so that you can compare the results in the same account over a multi-
year period. If you start with a small number of accounts and then gradually expand the number
of accounts over time, it becomes increasingly difficult to obtain comparable financial
information for more than the past year.
 Lock down. Do not allow subsidiaries to change the standard chart of accounts without a very good reason,
since having many versions in use makes it more difficult to consolidate the results of the business.
 Size reduction. Periodically review the account list to see if any accounts contain relatively immaterial
amounts. If so, and if this information is not needed for special reports, shut down these accounts and roll
the stored information into a larger account. Doing this periodically keeps the number of accounts down to
a manageable level.

c) (i) According to NAS 10 ''Event after the reporting period'', assets and liabilities should be
adjusted for events occurring after the balance sheet date that provide additional evidence to
assist the estimation of amounts relating to conditions existing at the balance sheet date. In the
given case, sale of immovable property was carried out before the closure of the books of
accounts. This is clearly an event occurring after the balance sheet date but agreement to sell
was effected on 1stAshadh, 2072 i.e. before the balance sheet date. Registration of the sale deed
on 15thShrawan, 2072, simply provides additional information relating to the conditions
existing at the balance sheet date. Therefore, adjustment to assets for sale of land is necessary
in the financial statements of National Trading Ltd. for the year ended 31stAshadh, 2072.
(ii) According to NAS 10 ''Event after the reporting period'' as those significant events, both
favorable and unfavorable, those occur between the balance sheet date and the date on which
the financial statements are approved by the Board of Directors in the case of a company.
Accordingly, the acquisition of another company is an event occurring after the balance sheet
date. However, no adjustment to assets and liabilities is required as the event does not after the
determination and the condition of the amount stated in the financial statements for the year
ended 31stAshadh, 2072.
Applying provision of the standard which clearly state that/disclosure should be made in
the report of the approving authority of those events occurring after the balance sheet date that
JZU P.T.O.
(13)
represent material changes and commitments affecting the financial position of the enterprise,
the investment of Rs. 40 lakhs in Ashoj, 2072 in the acquisition of another company should be
disclosed in the report of the Board of Directors to enable users of financial statements to make
proper evaluations and decisions.
6. Write short notes on: (5×3=15)
a) Presentation of „Government Grant related to Assets‟ in Financial Statement.
b) Reserve for Outstanding Insurance Claim Payables
c) Watch list in Loan loss provisioning
d) „Current Ratio‟ vs. „Quick Ratio‟.
e) Fundamental Accounting Assumptions
Answer
a) Government grants related to assets, including non-monetary grants at fair value, shall be
presented in the balance sheet either by setting up the grant as deferred income or by
deducting the grant in arriving at the carrying amount of the asset.
One method sets up the grant as deferred income which is recognized as income on a
systematic and rational basis over the useful life of the asset.
The other method deducts the grant in arriving at the carrying amount of the asset. The grant
is recognized as income over the life of a depreciable asset by way of a reduced depreciation
charge.
b) As Per Rule15 of the Insurance Regulation 2049, every insurer shall provide an amount
of one hundred fifteen percent (115%) of the remaining amounts of the payment against
the claim made by the Insurer before the expiry of each fiscal year. Such amount shall be
recognized as income in next year.

c) With effect from 2071/12/18, Nepal Rastra Bank has formulated a new category of loan for
provisioning purposes. As per the Central Bank‟s Rule, all loans are required to be classified into 5
different categories including Watch List whereby 5% of the total loan is required to be kept as
provisioning though the provision can be reversed when the loan becomes performing later. As per
the circular issued by Central Bank, the loans having the following characteristics are to be
classified as Watch List loans:
1. If interest and principal repayments are outstanding for more than a month.
2. Short term/Working Capital Loans that are not renewed on time and are renewed on
temporary basis.
3. Loans and advances to customers/ group of customers who have been categorized as non
performing by other banks and financial institutions.
4. Firms/Companies/Organizations having negative net worth or negative operating cash
inflows though interest and principal are served on regular basis.
d) Current Ratio:
The current ratio is a financial ratio that investors and analysts use to examine the liquidity of
a company and its ability to pay short-term liabilities (debt and payables) with its short-term
assets (cash, inventory, receivables).
Current Ratio = Current Assets / Current Liabilities.
Quick Ratio:
The quick ratio, on the other hand, is a liquidity indicator that filters the current ratio by
measuring the amount of the most liquid current assets there are to cover current liabilities
(you can think of the “quick” part as meaning assets that can be liquidated fast). The quick
JZU P.T.O.
(14)
ratio, also called the “acid-test ratio,” is calculated by adding cash & equivalents, marketable
investments and accounts receivables, and dividing that sum by current liabilities.
Quick Ratio = (Current Assets – Inventories) / Current Liabilities.

The main difference between the current ratio and the quick ratio is that the latter offers a
more conservative view of the company‟s ability to meets its short-term liabilities with its
short-term assets because it does not include inventory and other current assets that are more
difficult to liquidate (i.e., turn into cash). By excluding inventory (and other less liquid
assets) the quick ratio focuses on the company‟s more liquid assets.

e) Accounting has its own assumption to make it relevant and for the purpose of facility. The basic
assumptions underlying accounts are as follows:
1. Going concern: The enterprise is normally viewed as a going concern, i.e. as continuing
operations for the foreseeable future. It is assumed that the enterprise has neither the intention
nor the necessity of liquidation.If an enterprise is not a going concern -Valuation of its assets
and liabilities on historical cost becomes irrelevant and as a consequence its profit/loss may not
give reliable information.
2. Consistency: It is assumed that accounting policies are consistent from one period to another.
This adds the virtue of comparability to accounting data. It comparability is lost, the relevance
of accounting data for users‟ judgment and decision making is gone.
3. Accrual : Revenues and costs are accrued, that is, recognized as they are earned or incurred
(and not as money is received or paid) and recorded in the financial statements of the periods to
which they relate. This assumption is the core of accrual accounting system.
Disclosure requirements If the fundamental accounting assumption, viz. going Concern,
Consistency, and Accrual are followed in financial statements, specific disclosure is not
required. If a fundamental accounting assumption is not followed, the fact should be disclosed.

JZU P.T.O.
Audit and Assurance
Suggested Answers
Roll No……………. Maximum Marks - 100

Total No. of Questions- 7 Total No. of Pages- 10

Time Allowed - 3 Hours


Marks
Attempt all questions.

1. As an auditor, give your opinion with explanations on the following cases: (45=20)
a) M/s DC Limited signed an agreement with workers for increase in wages with
retrospective effect. The outflow on account of arrears was for 2011-12 Rs. 10.00
lakhs, for 2012-13 Rs. 12.00 lakhs and for 2013-14 Rs. 12.00 lakhs. This amount
is payable in September, 2014. The accountant wants to charge Rs. 22.00 lakhs as
prior period charges in financial statement for 2014-15.
b) X Ltd. sold theapartment to M Ltd. for Rs. 60 lakhs on 30.09.2014 and gave
possession of the property to M Ltd. However, Malpotdocumentation and legal
formalities are pending. Due to this, the company has not recorded the sale and
has shown the amount received as an advance. The book value of the building is
Rs. 25 lakhs as on March 31, 2014.
c) Manu Manufacturing Limited acquired an asset which has been declared by
municipality as not meeting the requirements of environment laws which have
been recently enacted. The asset has to be destroyed as per the law. The asset is
carried in the Balance Sheet at the year end at Rs. 6,00,000. The estimated cost of
destroying the asset is Rs. 70,000. The accountant wishes to charge off net
balance of Rs.5,30,000 in next 5 years.
d) KP is Chartered Accountant Member of the Institute of Chartered Accountants of
Nepal. He could not pay off the loan taken from one of the Commercial Bank.
The Bank blacklisted and published the name in national daily newspaper. The
Bank wrote a letter to ICAN for action.
Answer
a) The term prior period item refers only to income or expenses which arise in the current
period as a result of errors or omission in the preparation of the financial statements of
one or more prior periods. The term does not include other adjustments necessitated by
circumstances, which though related to prior periods are determined in the current period.
The full amount of wage arrears paid to workers will be treated as an expense of current
year and it will be charged to profit and loss account as current expenses and not as prior
period expenses. It may be mentioned that additional wages is an expense arising from
the ordinary activities of the company.

Although abnormal in amount, such an expense does not qualify as an extraordinary item.
However, as per NAS, when items of income and expense within profit or loss from
ordinary activities are of such size, nature or incidence that their disclosure is relevant to
explain the performance of the enterprise for the period, the nature and amount of such
items should be disclosed separately.

b) Principles of prudence, substance over form and materiality should be looked into, to
ensure true and fair consideration in a transaction. In the given case, the economic reality

XES P.T.O.
(2)
and substance of the transaction is that the rights and beneficial interest in the property
has been transferred although legal title has not been transferred. Hence, X Ltd. should
record the sale and recognize the profit of Rs. 35 lakhs in its financial statements for the
year ended 31st March, 2015; value of building should be removed from the balance
sheet.

Therefore the treatment given by the company is not correct.

c) As per NAS on Impairment of Assets, impairment loss is the amount by which the
carrying amount of an asset exceeds its recoverable amount, where, recoverable amount
is the higher of an asset‟s net selling price and its value in use·. In the given case,
recoverable amount will be nil [higher of value in use (nil) and net selling price is also
nil. Thus impairment loss will be the sum of Rs. 670,000 including the cost of disposable
of the fixed assets.

Therefore, asset is to be fully impaired and impairment loss of Rs. 6,70,000 has to be
recognized as an expense immediately in the statement of income as per NAS.

d) As per section 22 of Nepal Chartered Accountants Act, 1997 on provision of removal of


names and re-instatement, the Council may issue an order to remove the name of any
member from the Membership Register on any of the following circumstances:-

 If he is convicted by a court in a criminal offense involving moral turpitude and


punished therefor,
 If he fails to pay any fees required to be paid to the Institute,
 If he fails to abide by the professional conduct referred to in this Act and the Rules
framed under this Act,
 If he becomes unsound-minded, or
 If he is dead.

Based on the above provision, mere on the ground of loan defaulter, ICAN cannot initiate
action.
2. Give your comments on the following cases: (45=20)
a) The auditor of a company is unable to obtain audit evidence relating to business
promotion expenditures of Rs. 1 lakh. The company has earned net profit of Rs. 1
billion and has net asset base of Rs. 10 billion. The management explains that the
expenditure is genuine although the said invoices are misplaced. However, auditor
requests the management either not to charge the said promotional expenditure to
profit or loss statement or he will qualify his audit report. The auditor does not
have any issue raising question on the faithful presentation and preparation of the
financial statements.
b) XYZ has trade receivable balance of Rs. 1 crore as on year end date. The cut off
procedure indicates that cheque of Rs 25 lakhs received in the last week of the
year was not entered in the books and it was entered in cheques in hand register.
In the history of the company the cheques are never lost or misappropriated and
the cheques are deposited within the first week of receipt. These cheques were
also deposited and trade receivable balances were accordingly reduced in the next
year.

KHF P.T.O.
(3)
c) Quantity of an inventory item as per records was 4 lakhs units as on the year-end
date. Auditor was present during the inventory count at the year-end date and he
noticed that only 350,000 units were there during count. Management explains
that there might be counting error because the record was correct and reconciled
with opening stock, purchases and sales.
d) The audit report and the general purpose financial statements of XYZ Pvt.
Limited for the year 2070/71 were signed by an auditor. The audited financial
statements were submitted to tax authority along with the income tax return and to
a Bank also for loan processing. It was noted that the profit of the company was
Rs 1 crore in the financial statement submitted to tax authority and Rs 10 crores in
the financial statements submitted to Bank in the same financial year which was
audited by the same auditor. Please comment whether auditor shall be held
responsible for professional misconduct.
Answer

a) As per NSA 705,"Modification to the opinion in the independent Auditor's Report', the
auditor shall express a qualified opinion when:
i) The auditor, having obtained sufficient appropriate audit evidence, concludes that
misstatements, individually or in the aggregate, are material, but not pervasive, to
the financial statements; or
ii) The auditor is unable to obtain sufficient appropriate audit evidence on which to
base the opinion, but the auditor concludes that the possible effects on the financial
statements of undetected misstatements, if any, could be material but not pervasive.
Rs 1 lakh expenses for a company which earns net profit of Rs 1 billion and having net
assets base of Rs 10 billion seems to be immaterial/insignificant because omission or
misstatement of expenditure by Rs 1 lakh in this case is unlikely to affect the decision
of the users due to this omission/misstatement. Since the auditor does not have any
other issue on faithful preparation and presentation of the financial statements,
qualifying audit opinion for immaterial impact does not seem to be appropriate. The
auditor should however communicate the finding through management letter with the
recommendation to strengthen the system of proper maintenance and retention of
supporting evidence.
b) Internal control procedure of the company seems to be appropriate to record all cheques
which have not been deposited on the same date and keep it properly till deposited.
However, at the year end all balances should also reflect the true and fair view. In the
given case since no entries are made for cheques in hand, cash balance is understated
and trade receivable balances are overstated by Rs 25 lakhs. So, the company should
deduct the trade receivable balance in the current year itself thereby presenting cheques
in hand under cash and cash equivalents. In the next year when cheques are deposited,
cash in hand balance will be reduced and bank balance should be increased.
c) Certain audit evidences are more reliable than the other audit evidences. Stock quantity
as recorded in the stock ledger was reconciled with opening stock, purchases and sales.
But on physical verification, stock quantity was found to be short. Since the auditor
himself has observed that stock quantity as per physical verification is lower than the
quantity as per record, the evidence as per self-verification of the auditor should be
considered as more reliable than the recorded information. The auditor should consider
whether to conclude based on the evidence he/she has in hand or go further corrective
evidence.

KHF P.T.O.
(4)

d) In the given case it seems that either:


Sub section (11) of Section 34 of Nepal Chartered Accountants Act has clearly
explained that no member shall knowingly or recklessly mention any false matter in
any notice, explanation or statement required to be given to any office or department of
Government of Nepal or any organization. In a given case the auditor has signed two
balance sheets of different figures of the same organization of the same financial years
to different party's purpose on the request or pressure of the client. This is a serious
misconduct by the auditor. Hence disciplinary committee will take immediate action
against him.

The auditor has signed both financial statements of the same company for the same
year in which profits are different. It clearly indicates professional misconduct on the
part of the auditor and hence he shall be liable for disciplinary action.
3. Answer the following: (35=15)
a) Knowledge of client business is important for effective and efficient conduct of
the audit. Please explain the various sources from which the auditor can obtain
such knowledge.
b) Audit evidences collected from different sources and of different nature are not
equally reliable. Please explain.
c) It is not necessary to sign audit engagement letter every year in case of
recurring/ongoing audits. Please explain the statement as per provisions of NSA
210.
Answer
a) Sources for obtaining Knowledge of the client business: The various sources from
which the auditor can obtain knowledge of the client business are:

i) General economic factors and industry conditions affecting the client‟s business.
ii) Important characteristics of the client, its business, its financial performance and
its reporting requirements including changes since the date of the prior audit.
iii) The general level of competence of the management.
iv) The clients annual report to shareholders.
v) Minutes of meetings of shareholders, board of directors and important
committees.
vi) Internal financial management reports for current and previous periods, including
budgets.
vii) The previous year audit working papers and other relevant files.
viii) Firm personnel responsible for non-audit services to the client who may be able
to provide information on matters that may affect audit.
ix) Discussions with the client.
x) The client‟s policy and procedures manual.
xi) Relevant publications of ICAN and other professional bodies, industry
publications, trade journals, magazines, newspapers etc.
xii) Consideration of state of economy and its effect on the client‟s business and
xiii) Visits to the client‟s premises and plant facilities.

b) Reliability of audit evidences: The reliability of audit evidence is influenced by its


source and by its nature and is dependent on the individual circumstances under which it

KHF P.T.O.
(5)
is obtained. Generalizations about the reliability of various kinds of audit evidence can
be made; however, such generalizations are subject to important exceptions. (1)
While recognizing that exceptions may exist, the following generalizations about the
reliability of audit evidence may be useful:
o Audit evidence is more reliable when it is obtained from independent sources
outside the entity.
o Audit evidence that is generated internally is more reliable when the related
controls imposed by the entity are effective.
o Audit evidence obtained directly by the auditor (for example, observation of the
application of a control) is more reliable than audit evidence obtained indirectly or
by inference (for example, inquiry about the application of a control).
o Audit evidence is more reliable when it exists in documentary form, whether paper,
electronic, or other medium (for example, a contemporaneously written record of a
meeting is more reliable than a subsequent oral representation of the matters
discussed).
o Audit evidence provided by original documents is more reliable than audit evidence
provided by photocopies or facsimiles.

c) Audit Engagement Letter in Recurring Audit: As per NSA 210, on recurring audits,
the auditor shall assess whether circumstances require the terms of the audit engagement
to be revised and whether there is a need to remind the entity of the existing terms of the
audit engagement. The auditor may decide not to send a new audit engagement letter or
other written agreement each period.

However, the following factors may make it appropriate to revise the terms of the audit
engagement or to remind the entity of existing terms:
• Any indication that the entity misunderstands the objective and scope of the audit.
• Any revised or special terms of the audit engagement.
• A recent change of senior management.
• A significant change in ownership.
• A significant change in nature or size of the entity‟s business.
• A change in legal or regulatory requirements.
• A change in the financial reporting framework adopted in the preparation of the
financial statements.
• A change in other reporting requirements.
4. Answer the following: (35=15)
a) What is the requirement of Code of Ethics regarding use and disclosure of
confidential information of client acquired by a professional accountant as a result
of his service to the client?
b) What are the fundamental principles of the Code of Ethics issued by ICAN?
c) A Chartered accountant who is a member of ICAN is a full time employee of
ABCD & Associates (a CA firm registered with ICAN). During the audit of an
INGO from his firm (ABCD), he was requested by the client to audit a partner
organization of INGO in his personal capacity. Shall he accept the appointment of
auditor of Partner Organization of INGO?
Answer
a) Use and Disclosure of Confidential Information of Client: Code of Ethics requires
that a professional accountant shall not use confidential information acquired as a result
of his/her service to the client to his/her personal advantage or the advantage of third
parties. Similarly, a professional accountant shall not disclose confidential information

KHF P.T.O.
(6)
acquired as a result of his/her service to the client without proper and specific authority
or unless there is a legal or professional right or duty to disclose such information.
b) Fundamental Principles of Code of Ethics: There are the following fundamental
principles:

i. Integrity – to be straightforward and honest in all professional and business


relationships

ii. Objectivity– to not allow bias, conflict of interest or undue influence of others to
override professional or business judgments.

iii. Professional Competence and Due Care – to maintain professional knowledge


and skill at the level required to ensure that a client or employer receives
competent professional service.

iv. Confidentiality– neither discloses any confidential information to third parties


without proper and specific authority nor use the information for the personal
advantage of the professional accountant or third parties.

v. Professional Behavior – to comply with relevant laws and regulations and avoid
any action that discredits the profession.

c) Audit by a CA member in employment: As per decision of the council of ICAN, a


CA member of ICAN who is in full time employment is not entitled to hold active
Certificate of Practice. In other words, a CA member can either be in full time
employment or in practice (also popularly known as one man one profession). So, in
the give case the CA member should not accept the audit assignment in his personal
name.
5. Answer the following: (25=10)
a) Write down the functions, duties and powers of audit committee Company Act,
2063.
b) Write down the disqualifications of the auditor as per Company Act, 2063.

Answer
a) Section 165 of the company act 2063 has prescribed the functions, duties and powers of
audit committee: The functions, duties and powers of the audit committee formed
pursuant to subsection (1) of Section 164 shall be as follows:
(a) To review the accounts and financial statements of the company and ascertain
the truth of the facts mentioned in such statements;
(b) To review the internal financial control system and the risk management system
of the company;
(c) To supervise and review the internal auditing activity or the company;
(d) To recommend the names of potential auditors for the appointment of the
auditorof the company, fix the remuneration and terms and conditions of
appointment of the auditor and present the same in the general meeting for the
ratification thereof;
(e) To review and supervise as to whether the auditor of the company has observed
such conduct, standards and directives determined by the competent body

KHF P.T.O.
(7)
pursuantto theprevailing law as required to be observed in the course of doing
auditing work;
(f) Based on the conduct, standard and directives determined by the competent
body pursuant to the prevailing law, to formulate the polices required to be
observed by the company in respect of the appointment and selection of the
auditor;
(g) To prepare the accounts related policy of the company and enforce, or cause to
be enforced, the same;
(h) Where any regulatory body has provided for the long term audit report to be set
out in the audit report of the company, to comply with the terms required
preparing such report;
(i) To perform such other terms as prescribed by the board of directors in respect of
the accounts, financial management and audit of the company.
b) Section 112 (1) of the company act 2063 stated the disqualification of auditor.
Accordingly none of the following persons or the firms or companies in which such
persons are partners shall be qualified for appointment as auditor and shall, despite
appointment as auditor, continue to hold office:
(a) A director, advisor appointed with entitlement to regular remuneration or cash
benefit, a person or employee or worker involved in the management of the
company or a partner of any of them or an employee of any of such partners or a
close relative of a director or partner, out of them, or an employee of such
relative;
(b) A debtor who has borrowed moneys from the company in any manner, or a
person who has failed to pay any dues payable to the company within the time
limit and is in such arrears or close relative of such person;
(c) A person who has been sentenced to punishment for an offense pertaining to
audit and a period of five years has not elapsed thereafter;
(d) A person who has been declared insolvent;
(e) A substantial shareholder of the company or a shareholder holding one percent
or more of the paid up capital of the company or his close relative;
(f) A person who has been sentenced to punishment for an offense of corruption,
fraud or a criminal offense involving moral turpitude and a period of five years
has not elapsed thereafter;
(g) A person referred to in Sub-section (3) of Section 111;
(h) In the case of a public company, any person who works, whether full time or
part time, for any governmental body or anybody owned fully or partly by the
Government of Nepal or any other company or a partner of such person or a
person who is working as an employee of such partner or a person who is
authorized to sign any documents or reports to be prepared by the management
of the company;
(i) A company or corporate body with limited liability;
(j) A person having interest in any transaction with the company or his/her close
relative or a director, officer or substantial shareholder of another company
having any interest in any transaction with the company.

6. Write short notes on the following: (42.5=10)

a) Audit risk at the financial statement level


b) Competence and objectivity of the expert engaged by auditor
c) Competence of the audit engagement team
d) Financial indicators of going concern issue

KHF P.T.O.
(8)
Answer
a) Audit risk is considered at the financial statement level during the audit planning
process. At this time, the auditor should undertake an overall audit risk assessment
based on his knowledge of the client‟s business, industry, management, control
environment and operations. Such an assessment provides preliminary information
about the general approach to the engagement, the auditor‟s staffing needs and the
framework within which materiality and audit risk assessments can be made at the
individual account balance or class of transactions level. As part of this overall risk
assessment, the auditor should consider whether there is potential for pervasive
problems, for example, liquidity or going concern problems.
b) i. When planning to use the work of an expert, the auditor should assess the
professional competence of the expert which will involve considering the expert‟s:
a. Professional certification or licensing by, or membership in, an appropriate
professional body;
b. Experience and reputation in the field in which the auditor is seeking audit evidence.
ii. The auditor should assess the objectivity of the expert. The risk that an expert‟s
objectivity will be impaired increases when the expert is:
a. Employed by the entity; or
b. Related in some other manner to the entity, for example, by being financially
dependent upon or having an investment in the entity. If the auditor is concerned
regarding the competence or objectivity of the expert, the auditor needs to discuss any
reservations with management and consider whether sufficient appropriate audit
evidence can be obtained concerning the work of an expert.
The auditor may need to undertake additional audit procedures or seek audit evidence
from another expert.

c) The appropriate capabilities and competence expected of the engagement team as a


whole include the following:
 An understanding of, and practical experience with, audit engagements of a similar
nature and complexity through appropriate training and participation.
 An understanding of professional standards and regulatory and legal requirements.
 Appropriate technical knowledge, including knowledge of relevant information
technology.
 Knowledge of relevant industries in which the client operates.
 Ability to apply professional judgment.
 An understanding of the firm‟s quality control policies and procedures.

d) Financial Indicators
 Net liability or net current liability position.
 Fixed‐term borrowings approaching maturity without realistic prospects of renewal
or repayment; or excessive reliance on short‐term borrowings to finance long‐term
assets.
 Indications of withdrawal of financial support by debtors and other creditors.
 Negative operating cash flows indicated by historical or prospective financial
statements.
 Adverse key financial ratios.
 Substantial operating losses or significant deterioration in the value of assets used
to generate cash flows.
 Arrears or discontinuance of dividends.
 Inability to pay creditors on due dates.

KHF P.T.O.
(9)
 Inability to comply with the terms of loan agreements.
 Change from credit to cash‐on‐delivery transactions with suppliers.
 Inability to obtain financing for essential new product development or other
essential investments.

7. Distinguish between: (25=10)


a) Hot review & Cold Review in audit
b) Accounting & Auditing
Answer
a) Hot file review or hot review is conducted usually conducted during the audit and/or audit
work is completed but before the auditor‟s report is issued. This in nature is a detailed review
that is conducted with an aim to find out if there s any weakness in application of audit
procedures or if the results have been misinterpreted. Hot reviews are usually carried out
usually by the senior the audit team or someone with the same authority who is not connected
with the engagement. Such reviews mostly include meetings with audit team personnel and
their individual work so that both work and the skills of members are improved by pointing out
discrepancies and providing recommendations.

The purpose of a hot review is to identify any key areas that need to be addressed prior to
signing the report. The categories for review which may be undertaken can be described as
follows:
i. Comfort reviews
ii. High risk reviews
iii. Training reviews
iv. Independence reviews
v. NSQC reviews

To summarize, hot review is conducted during the audit work is conducted but before the
auditor‟s report is issued with a prime objective to ensure compliance with relevant auditing
standards and achieving engagement‟s objectives

Cold file review or cold review is an objective evaluation on the date of auditor‟s report and is
performed by the auditor i.e. partner himself when all the audit work has been concluded and
the required sufficient appropriate audit evidence has been obtained and conclusions drawn and
reported. This review usually takes place when the auditor‟s report is signed off. The purpose
of this review is to ensure compliance with relevant auditing standards and to analyze
weaknesses in the way whole audit work is conducted and how it can be improved for next
similar assignments by updating firm‟s quality control standards, training the staff etc.
Normally the cold file review would aim to:
 Identify whether the disclosure requirements had been properly met - incorrect disclosures
are the largest subject of complaints to the Institute.
 Identify whether the Auditing Standards and Regulations have been properly complied
with - each audit would be "scored" using a comprehensive file review checklist.
 Assess the effectiveness of any independent manager review and the partner review,
looking for any points that should have been picked up by a manager but had not been, and
likewise with the partner.
To summarize, cold review is conducted with a view to check for the weaknesses in the firm‟s
quality control procedures and system, proficiency of audit team members and how they can be
improved to make later audit assignment more effective and efficient.

KHF P.T.O.
(10)

b) Distinction between Accounting and Auditing:


Point of
difference Accounting Auditing
1. Meaning It is recording of all the day to It is the critical examination of
day transactions in the books of the transactions recorded in the
accounts leading to preparation books of accounts.
of financial statements.
2. Nature It is concerned with finalization
It is concerned with establishment
of accounts. of reliability of financial
statements.
3. Objects Accounting commences when Auditing begins when accounting
book keeping ends. ends.
4. It involves various financial It depends upon the agreement or
Commencement statements. It involves upon the provisions of law.
maintenance of books of
accounts.
5. Scope It does not go beyond books of It goes beyond books of accounts.
accounts.

KHF P.T.O.
Corporate and other Laws
Suggested Answer
Roll No……………. Maximum Marks - 100

Total No. of Questions - 7 Total No. of Pages -12


Time Allowed - 3 Hours
Marks
Attempt all questions.
1. Answer the following questions:
a) Wild Fibers Pvt. Ltd. has intended to issue right shares but there was no
unanimity within the Boards of Directors. Hence, the Directors who were opposed
to issue right shares asked you for your opinion regarding the benefits of issuing
of right shares. How can you give your opinion regarding the benefits of right
shares? (1+4=5)
b) Mr. XYZ was appointed as a director of a company in an Annual General
Meeting. He took over the office and carried out his functions as director.
Subsequently, it was found that there were some irregularities in the appointment
and hence the appointment was declared invalid. Would the duties performed by
Mr. XYZ, while in office as director, be binding upon the company?Also mention
when a company be bound by the decision or action taken by its director under
Companies Act, 2063? (2+3=5)
c) Some of the shareholders of Transnational Works Pvt. Ltd. objected the resolution
relating to issue bonus share forwarded by the Board of Directors stating that the
resolution should be presented as a special resolution. Advise the directors on the
following issues.
i) What matters should be presented as a special resolution?
ii) Whether the resolution to issue bonus share is special resolution.
d) Who is the debenture trustee? What are the matters to be included in the
agreement to be concluded between debenture trustee and company under
Companies Act, 2063? (1+4=5)
e) Mount Everest Ltd. has decided to appoint independent director(s) to the
company. Advise the company on the following legal issues as to the appointment
of independent director(s) pursuant to the Companies Act, 2063. (2+3=5)
i) What is the legal provision of appointment of independent director(s)?
ii) Who are the persons not eligible to be appointed as an independent director?
Answer:
a) Right Shares are those types of shares which are issued to existing shareholders only. The
right shares help the company to raise its share capital without increasing the number of
shareholders and the existing shareholders can get opportunity to increase their shares in
the company.
Benefits of Issuing Right Shares:
1. More control on existing shareholders
Because right shares are issued to existing shareholder, so there is no risk of losing of
control of existing shareholders. Existing shareholders‟ share will increase in company
YOP P.T.O.
(12)
and they can take decision without any compromise with the principles of company. It
is very helpful to achieve the missions of company.
2. No loss to existing shareholder
By issuing shares to existing shareholders, value of share will increase due to stability
in controlling power of company. So, there will not be any loss to existing
shareholders with right shares.
3. No cost for issuing shares to public
Company has not to give any invitation to public, so advertising cost and other new
issue cost will decrease with right shares.
4. Capital formation
Company can get capital at any time without any delay because company can easily
issue shares to existing shareholders just sending right shares offer notice. Further, it is
also way to increase the goodwill and reputation of company in industry.
5. More scientific
Distribution technique of right shares issue is more scientific. Not all shares will get by
single shareholders but it will be in the proportion of existing shares which is in the
hand of old shareholders at this time.
b) Section 106 of the Company Act, 2063 provides for the validity of acts already done by a
director whose appointment comes to be invalid. According to it if it is afterwards
discovered that any provision under this Act has not been complied with in respect of the
appointment of any director, the acts already done by such director before the discovery
of such fact does not become invalid by that fact.
Moreover, Section 104 prescribes the provision regarding company to be bound by the
act done by a director. It states that any act done or action taken by or document signed
by at least one director authorized by a company or any person authorized to act for the
company shall be valid and binding for the company.
Where any person does any transaction with a company in good faith, such
transaction shall be binding for the company; and nothing contained in memorandum of
association, articles of association of the company or in any resolution adopted by the
general meeting or in any agreement concluded between the company and its shareholder
shall be deemed to have made any limitation in or restriction on the authority of the
director or the authorized person to do such transaction.
Provided, however, that if any officer does any act or transaction mentioned
above in excess of his/her authority, such officer shall be personally liable for such act or
transaction unless such authority is ratified by the general meeting pursuant to this Act;
and the company may also recover from him/her the loss or damage, if any caused to the
company from such act or transaction.
c)
i) Regarding the special resolution, Section 83 of the Companies Act, 2063 mentions that
the following matters shall be presented as special resolutions in the general meeting
of a company:
1. Increasing the authorized capital of the company,
2. Decreasing or altering the share capital of the company,
3. Altering the name or main objectives of the company,
4. Amalgamating one company into another company,
5. Issuing bonus share,
6. Buying back of own shares by the company,

YOP
(13)
7. Selling shares at a discount,
8. Converting a private company into a public company or vice versa,
Such other matter in respect of which the company is required by this Act or the articles
of association to adopt a special resolution.
ii) As mentioned above, issuing a bonus share should be forwarded as a special
resolution. Therefore, BoD has to present a special resolution to issue bonus share.
d) Section 2 (t) of the Companies Act 2063 defines Debenture-Trusteeas a body corporate
undertaking the responsibility for the protection of interests of debenture-holders at the
time of issuance of debentures by a company.
Section 36 of the Companies Act 2063 states the matters to be included in the agreement
to be concluded between debenture trustee and company, which are as follows:

(a)That the debenture trustee is entitled to carry out, or cause to be carried out, valuation of
the company's assets, project analysis or management analysis,
(b) The period of repayment of the principal and interest of debentures subscribed by the
debenture-holder, interest rate, mode of repayment of the principal and interest, and matters
of conversion of debentures into shares, if there is such provision,
(c)Matters relating to a provision made on the rights of other creditors over the assets of the
company and liabilities that may arise there from in the future.
(d) A provision that, in the event of violation or non-fulfillment of the terms mentioned in the
agreement or for any other reasonable reason, if it is required to take the control of
financial transactions of the security as referred to in the agreement, the debenture trustee
may take in his possession the assets or property that he has taken as the security or
guarantee or hold the security or guarantee or hold the security or guarantee with himself or
sell the same by auction or in any other appropriate manner,
(e)Procedures for payment by the company of the service charges and other direct expenses of
the debenture trustee,
(f) That the debenture trustee shall not be liable to any loss or damage caused to the company
or the debenture-holder from any act done by the trustee in the capacity,
(g) That, in the event of occurrence of any circumstance necessitating the liquidation of the
company, the debenture trustee is entitled to take such legal action as may be taken on
behalf of the debenture-holder and exercise the powers of the debenture-holder,
(h) Other necessary matters on the protection of interest of the debenture-holder.
e)
i) Section 86 of the companies Act 2063 provides provision regarding the
appointment of the Directors of the company.
Pursuant to section 86 (2) of the companies Act 2063 every public company shall have a
Board of Directors consisting of a minimum of three and maximum of eleven directors.
As per section 86(3) of Act, in forming the board of directors pursuant to sub-section (2)
above, at least one independent director , in the case of the number of directors not
exceeding seven, and at least two independent director , in the case of the number of
directors exceeding seven, shall be appointed from amongst the persons who have the
knowledge as prescribed in the articles of association of the company and gained
knowledge and experience in the subject related to the business of the company
concerned.
This legal provision should be considered by the Mount Everest Ltd. for the appointment
of the independent director(s) depending upon the numbers of directors it appoints.

YOP
(14)
ii) In pursuance to section 89(2) of the Companies Act, 2063 any of the following
persons shall not be eligible to appointed to the office of independent director.
a. Who is a person disqualified to be appointed to the office of a director pursuant to
sub-section(1) of section 89 of the Companies Act, 2063;
b. Who is the shareholder of the concerned company?
c. Who has not obtained at least bachelor degree in a subject that is related to the
business to be carried on by the concerned company and gained at least ten years of
experience in the related field or in the company management affairs or who has not
obtained bachelor degree in finance, economics, management, accounts, statistics,
commerce, trade or law and gained at least ten years of experience in the related
field.;
d. Who is an officer, auditor or employee of concerned company or a period of three
years has not lapsed after his/her retirement from any such office;
e. Who is close relative of the office of the concerned company;
f. Who is an auditor of the concerned company or his/her partner?

2. Answer the following questions:


a) When Nepal Rastra Bank may cancel the license obtained by a licensed institution
to carry on the financial transactions pursuant to BAFIA, 2063? Discuss.
b) What are the objectives of Nepal Rastra Bank?
c) Under what circumstances Bank & Financial Institutions are restricted from
distributing dividends?
Answer:
a) As a regulating body, Nepal Rastra Bank shall have full authority to regulate and
systematize the functions and activities of all licensed institutions. It is the bank of banks.
Section 35 (2) of the Bank and Financial Institutions Act, 2063 has stated that the Rastra
Bank may cancel the license obtained by a licensed institution to carry on the financial
transactions pursuant to this Act in any of the following circumstances.
1. If the concerned licensed bank or financial institution requests for the cancellation
of its license.
2. If it fails to carry on the financial transactions within six months from the date of
receipt of the license.
3. If it stops carrying on the financial transactions since more than one month ago
continuously.
4. If it carries on the financial transactions in such a manner as to be contrary to the
rights and interests of depositors.
5. If it violates the Nepal Rastra Bank Act and its rules.
6. If it violates the conditions prescribed by the Rastra Bank.
7. If it fails to comply with the orders or directives issued by the Rastra Bank.
8. If it becomes insolvent.
9. If the bank or financial institution is found to have obtained the license by submitting
false details.
10. If the licensed institution is merged with another bank or financial institution.

YOP
(15)
b) As per the section 4 of Nepal Rastra Bank Act 2058 following are the objectives of Nepal
Rastra Bank.
 To formulate necessary monetary and foreign exchange policies in order to maintain
the stability of price and balance of payment for sustainable development of economy
and manage it;
 To promote stability and liquidity required in banking and financial sector;
 To develop a secure, healthy and efficient system of payments;
 To regulate, inspect, supervise and monitor the banking and financial system and
 To promote entire banking and financial system of the Nepal and to enhance its
public credibility and
 To extend co-operation in the implementation of the economic policies of the
Government of Nepal.

c) Section 46 of Bank and Financial Institution Act 2063, states the circumstances under
which Bank & Financial Institutions are restricted from distributing dividends.
As per the section 46 of the Act, Bank & Financial Institutions are restricted from
distributing dividends till the following conditions are fulfilled.
 All preliminary expenses are recovered.
 Cumulative losses up to previous year is recovered.
 Capital fund is maintained.
 Risk bearing fund is adequately maintained.
 General Reserve as per section 44 of the Act is provided.
 Share capital for general public is issued and fully subscribed

3. Answer the following questions:


a) State the various penalties imposed on insurance offences.
b) Mr. Lekhapal wants to know the provision regarding the fund of Securities Board
of Nepal under Securities Act, 2063.Advise him.
Answer:
a) The punishment to be imposed to any insurer or the Director of the insurer, employee or
surveyor, broker or insurance agent who have knowingly violated the provisions of
Insurance Act and rules made under the Act or order or directives is mentioned in Section
36 of the Insurance Act, 2049. The Insurance Board,in such cases, may punish to such
insurer or the Director of the insurer, employee or surveyor, broker or insurance agent
with a fine ranging from Rs.3000 to NRs. 10000. If such offense has been made
frequently, he will be fined at the rate of extra NRs. 500 for each subsequent offense.
Similarly, sub-section (2) provides that if any insurer or insurance agent or broker has
made any insurance business without following the procedures to be followed pursuant to
this Act, the Board may fine him up to NRs. 10000. Further, if the accounts, records,
register, details, information or any other documents has not maintained, prepared or
submitted in time or made falsely, he may be punished with a fine up to NRs. 30000 or
imprisonment up to two years or with both.
b) Section 22 of the Securities Act, 2063 prescribes the provision regarding fund of
Securities Board.
1) The Board shall have a separate fund of its own.
YOP
(16)
2) The following amount shall be credited to such fund.
a. Amounts received from the Government of Nepal
b. Amounts obtained as grants, assistance or loans from any national, foreign or
international organizations, institutions or bodies.
Provided that prior approval of the Government of Nepal shall be obtained in
receiving such grants, assistance or loans.
c. Amounts received by way of the license fees.
d. Amounts received by way of the fees for registration of securities.
e. Amounts received by way of fees, charges, dues for transactions in securities.
f. Amounts received from any other sources.

3) All amounts to be obtained to the fund of Board as above shall be credited to an account
to be opened with any commercial bank within Nepal.
4) All the expenditure to be made on behalf of Board shall be chargeable on such fund.
5) In making expenditure pursuant to above, such expenditure shall be made subject to the
budget approved by the Board for the incomes and expenditures to be made in each fiscal
year.
6) The operation of the fund of the Board shall be as prescribed.
4. Answer the following questions:
a) In relation to the Labour Act, 2048, how this act prohibit in the engagement of
non-Nepalese citizens at work? Briefly present your answer with legal provision
and relevant case law.
b) The workers of a company objected the decision of manager regarding the bonus
to be distributed to them. The workers seek your advice about the action against
the company. Advise them about the legal provisions for the settlement of
disputes under the Bonus Act, 2030.
Answer:
a) After the decision in Balkrishna Neupane Vs. Parliamentary Secretariat, Nepal (Nepal
KanoonPatrika 2050, Volume-8, Decision No. 4768, P-450) Labour Act has been
amended with inserting restriction to engage Non-Nepalese citizen at work in Nepal.
Formerly, its Section 4.1 has mentioned as "Nepali citizen shall be given priority in the
recruitment in such manner". Supreme Court of Nepal had held that, this provision has
adverse impact in the right to freedom conferred to the Nepali citizens by the Article
12.2.E and declared it as void. After the decision of the court, Government has amended
the law byin sorting the restrictive provision to the Non-Nepalese Citizen at work Section
4A that is given as follows:

(1) Non-Nepalese citizens shall not be permitted to be engaged at work in any of the
posts classified pursuant to Section 3.
(2) Notwithstanding anything contained in Sub-section (1), if a Nepalese citizen could
not be available for any skilled technical post even after publishing an advertisement
in national level public newspapers and journals, the Manager may submit an
application to the Department of Labour along with the evidence of such fact for the
approval to appoint a non-Nepalese citizen.
(3) If it is found, in conduction of an inquiry upon the submission of any application
pursuant to Sub-section (2), that a Nepalese citizen would not be available for the
skilled technical post mentioned in the application, the Department of Labour may,
YOP
(17)
on the recommendation of the Labour Office, grant approval to engage a non-
Nepalese citizen at work years for a maximum period of upto five years not
exceeding two years at a time and, in the specialized kind of skilled technical post,
for a period upto seven years.
(4) The Manager, who engages non-Nepalese citizens at work pursuant to Sub-section
(3), shall have to make arrangements for making the Nepalese citizens skilled and
for replacing the non-Nepalese citizens gradually by them.

b) The Bonus Act, 2030 assures the bonus to the employees out of the profits earned by an
enterprise. The Act also has made legal provisions for the settlement of disputes which
may rise between the employees and management. Section 16 (1) provides that if any
dispute arises between employees and management with respect to the bonus, the Labour
Office shall resolve such dispute by negotiations having invited both the parties.
Similarly, if the dispute could not be resolved by negotiation the Labour Office shall ask
to the concern enterprise and employees to produce necessary documents and statements
of accounts and shall give a decision on the basis of such documents and statements.
When the decision is given by the Labour Office, the party who is dissatisfied with the
decision may appeal to the Labour Court within 35 day of receipt of such notice and the
decision made by the Labour Court shall be final for the matter.
Therefore, the workers of the company shall have to follow the procedures as mentioned
above to settle the dispute regarding the payment of bonus.

5. Suppose you are a Chartered Accountant practicing in Nepal, Mr. Alberto, an auditor from the
USA, is interested to know the punishment provision under Nepal Chartered Accountants Act,
2053. Explain those disciplinary actions and punishment provision that ICAN can impose
against auditor under Nepal Chartered Accountants Act, 2053?

Answer:
Section 14 and 35 of Nepal Chartered Accountants Act, 2053 has given power to the council
of Institute of Chartered Accountants of Nepal to take different disciplinary action against an
auditor.
Section 35 ofthis Act prescribe that a person or individual may lodge a complaint to the
ICAN against the auditor (holding certificate of practice) for not upholding the conduct
mentioned or for violation of this Act or Regulation framed under this Act. The Secretary
shall, if he finds convincing information that proves any member or member holding
certificate of practice is not observing the conduct, submit the proposal along with related
facts to the council of the ICAN for further action against such member.
There shall be a Disciplinary Committee constituted by the ICAN council under section 14 of
the Act, to inquire into a compliant and recommend the council for necessary action in cases
where anyone lodges a compliant in the council that any member has done any act or action
contrary to this Act or Rules or Code of Conduct framed under this Act, or where the Council
receives such information.
The Disciplinary Committee shall make recommendation, along with its opinion and finding,
to the Council for taking necessary action against a member found guilty from its
investigation. The council may in view of such recommendation, impose any of the
following penalties on the concerned member, according to the gravity of the offence:
a. Reprimanding

YOP
(18)
b. Removing from membership for a period not exceeding Five years.
c. Prohibiting from carrying on the accountancy profession for any specific period,
d. Cancellation of the certificate of practice or membership.

Moreover, there is a provision of removal of names and re-instatement of membership in


section 22 of the Act.

1. The Council may issue an order to remove the name of the auditor from the membership
register for not abiding by the professional conduct referred to in this Act and Rules
framed under 'this Act.
2. If his/her name has been removed from membership as above, he/ she can make an
application, accompanied by a reasonable ground, to again obtain membership, and the
council may decide to grant membership, by re-registering his / her name, upon receipt
of the fees as prescribed.

6. Answer the following questions:


a) What are the special program relating to social welfare, as per Social Welfare Act,
2049?
b) What are the voidable contracts under Contract Act, 2056?
c) Can a bank or financial institution buy back its share? Discuss with relevant legal
provisions.
d) Mention the punishment provision to be imposed while violating the Act, Rules or
directives issued under Insurance Act,2049.
e) Mr.A, a Malaysian, endorsing it in the front, drew a Bill of Exchange (BoE) of
MR. 50,000 to Nepalese person, Mr.B mentioning it is payable in Nepal only.
Illustrate your answer in the following query.
i) Which law governs in the matters of dishonour?
ii) What is the validity of the BoE?
iii) In which condition law relating to negotiable instruments of foreign country is
presumed to the same as that of Nepalese law?
Answer:
a) Section 4 of Social Welfare Act 2049, describes regarding Special program relating to
Social Welfare.
Government of Nepal may operate special program relating to the social welfare activities
and social service in the following matters
1. To serve interest and render welfare to the children, aged, helpless or disabled people.
2. To foster participation in development and to promote and protect the welfare, rights
and interest of the women.
3. To rehabilitate and help to lead a life of dignity to the victims of social mischief and
also to juvenile delinquency, drug addicts and similar type of people involved in other
kind of addictions.
4. To help to lead a life with dignity to the jobless, poor and illiterate people.
5. To manage religious places and the activities of the trust Guthi institutions.

YOP
(19)
6. To take effective management and actions for the welfare of the backward communities
and classes.

b) Voidable contracts are such which are defective in the process of making it but not really
void due to any defects in the essence of it.These contracts can be made void or may be
made enforceable at the interest of an aggrieved party.
Section 14 of Nepal Contract Act has the following provisions regarding voidable contracts;
i) Contracts concluded through coercion
A person shall be deemed to have indulged in coercion if he, with the objective of
compelling any person to accept any contract against his will, withholds or threatens
to withhold his property or issues a threat to his life, body or prestige, or takes or
threatens to take any other section i contravention of current law.
ii) Contracts concluded through undue influence

"Undue influence" means influence exercised by a person upon another person who is
under his influence or is amenable to his wishes with the intention of deriving some
undue advantage for personal benefit or to fulfil selfish interests.
Without prejudice to the generality of above, the following categories of persons shall
be deemed to be under influence of any person or amenable to is wishes:
(a) Persons living under his guardianship, protection or custody.
(b) Persons who cannot take care of their interests temporarily or permanently by
reason of old age, sickness or physical or mental weakness.
(c) Persons who can be subjected to economic pressure or pressure based on posts.
iii) Contracts by fraud
A party to the contract or is agent to believe, or takes any action which leads the
latter to believe, that a particular matter is true, although he knows full well that it
is false, or suppresses any information in his possession, or indulges in any other
fraudulent act punishable under current law, with the intention of deceiving the
opposite part or his agent.
iv)Contracts concluded through deceit
The following actions shall be taken as deceit:
(a) Submission of false particulars on any matter without a reasonable basis for
doing so.
(b) Misleading any party so as to aggrieve him.
(c) Causing any mistake or error in respect to the particulars of the contract.

c) Section 10 (1) of the Bank and Financial Institutions Act, 2063 has imposed prohibition
on buy back of shares by the bank or financial institutions. But Sub-section 2 of section
10 has mentioned the grounds on which the banks or financial institutions can buy back
of their shares. In the following conditions a bank or financial institution may, with the
approval of the Nepal Rastra Bank, buy back its shares out of its free reserves available
for being distributed as dividends not exceeding the percentage prescribed by the Nepal
Rastra Bank.
1. If the shares issued are fully paid up.
2. If the shares are already listed in the Securities Board.
3. If the buyback of shares is authorised by the articles of association.
4. If a special resolution has been adopted for the buyback of shares.
5. If the ratio of debt is not more than twice the capital and general reserve fund after
such buyback.
YOP
(20)
6. If the value of share bought back is not more than 20% of the total paid up capital
general reserve fund.
7. If the buyback of share is not contravention to the directives issued by the Nepal
Rastra Bank.
d) Section 36 of the Insurance Act prescribes the provision regarding the Punishment of
concerned responsible persons as follows:
1. If any Insurer or the Director of the Insurer, employee or Surveyor, Broker or
Insurance Agent knowingly violates this Act or the Rules made under this Act or order
or directives or does not perform any function to be performed or does any act to be
done, the Board may punish to such Insurer or the Director, employee or Surveyor,
Broker or Insurance Agent with a fine ranging from three thousand rupees to ten
thousand rupees. If such offense has been made frequently, he will be fined at the rate
of extra five hundred rupees for each subsequent offense.
2. If any Insurer or Insurance Agent or Broker has made any Insurance Business within
following the procedures to be followed pursuant to this Act, the Board may fine him
up to ten thousand rupees.
3. If the accounts, records, register, details, information or any other documents to be
maintained, prepared, formed or submitted pursuant to this Act or the Rules made
under this Act has not maintained or submitted in time by knowingly or with malafied
intention or has maintained or submitted the false details or documents by any-body,
he may be punished with a fine up to thirty thousand rupees or imprisonment up to two
years or with both.
Any person or corporate body dissatisfied with the decision made by the Board pursuant
to this Act, may make an appeal in the concerned Appellate Court within thirty five days
from the date of such decision under Section 37 of the Act.
e)
i) Governing law in the matters of dishonor and its condition: Sec 100
The Negotiable Instrument Act, 2034, in its section 100, provided about the matter of
dishonor. Where a Negotiable Instrument is made payable in a different country from
that in which it is made or endorsed, the law of the country where it is made payable
determines what constitutes dishonour and what notice of dishonour is sufficient.
ii) Validity of Bill of Exchange (BoE) : Section 101
If a Negotiable Instrument is made drawn, accepted or endorsed outside Nepal, but in
accordance with Nepalese Law, the circumstance that any agreement evidenced by such
instrument is invalid according to the law of the country wherein it was entered into
does not invalidate any subsequent acceptance or endorsement made thereon within
Nepal.
iii) Presumption of foreign law relating to Negotiable Instruments: Sec 102
The law of any foreign country regarding Negotiable Instrument shall be presumed to be
the same as that of Nepalese Law, unless and until the contrary is proved.

7. Write short notes on the following: (2×5=10)


a) Distinguish between Share and Stock
b) Fundamental principles of World Trade Organization (WTO).

YOP
(21)
Answer:
a)
S.No. Share Stock
1. Shares in physical form bear distinct Stocks are the consolidated value of
numbers. share capital.
2. Shares may or may not be fully paid-up. Stock is always fully paid-up.
3. Shares have a nominal value. Stock does not have any nominal
value.
4. All shares are of equal denomination. Denomination of stocks varies.
5. It is not possible to transfer shares into Stock is divisible into any amount
fraction. required. Thus, it is possible to
transfer even into fractions.
6. Shares come into existence before the stock Stock comes into existence after
and it is issued initially. conversion of shares into stock and
on conversion of shares into stock,
the provision of the Act governing
the shares shall cease to apply to the
share capital as it is converted into
stock.

b) The fundamental principles as embodied in the WTO Agreement in respect of


multilateral trading system are derived from the GATT. Trade without
discrimination is one of the basic principles included in the multilateral
agreements on trade in goods in GATS, and in the TRIPs Agreements. It includes
the following principles:
1. The Most-Favoured-Nations Treatment: It is the pillar of the system from the inception of the
GATT in 1947. Members of the WTO have entered into similar commitments, under the
GATT 1994 for trade in goods in relation to treatment of services, under the TRIPs
agreement in regard to the protection of intellectual property.
2. Fair competition and Non discrimination: Principles of non-discrimination prohibit
discrimination among WTO members while providing facilities and services i.e. information,
dispute settlement , trade assistance, negotiation facilities. It includes two major components
viz. national treatment and MFN treatment.
3. Transparency: Provisions on notification requirements and the Trade Policy Review
mechanism are set out in the WTO Agreement and its Annexes, with the objective of
guaranteeing the fullest transparency in trade policies of its members in goods, services and
the protection of intellectual property rights.
4. Predictable and growing access to markets: Predictable and growing access to markets for
goods and services is an essential principle of the WTO. This principle is fulfilled through
various provisions so as to guarantee security, predictability and continued liberalization of
trade.
5. Trade in goods: In case of goods, a basic GATT postulate is that tariffs should normally be
the only instrument used to protect domestic industry.
6. Trade in services: GATS establishes a multilateral framework of principles and rules for
trade in services with a view to the expansion of such trade under conditions of transparency

YOP
(22)
and progressive liberalization, and a means to promote the economic growth of all countries
and the development of developing countries.

YOP
Financial Management
Suggested
Roll No……………. Maximum Marks - 100

Total No. of Questions – 7 Total No. of Printed Pages –5

Time Allowed – 3 Hours


Marks
Attempt all questions.
Working notes should form part of the answer. Make assumptions wherever necessary.
1. Mr. X, an Agricultural Engineering graduate, is planning to set up a private limited
commercial agriculture firm with registered office in Kathmandu Metropolitan City.
During the initial days of the proposed company, it will focus on cultivating and
harvesting a variety of premium apples and sell them in the ever increasing
domestic market.
In order to set up his project, he has already reached agreements with the local
government authority and the local communities through a cooperative society of
one of the hilly districts suitable for farming. As such, the agreement has guaranteed
minimum 25 years of lease of land property having a total area of 800 ropanis with
a rent of Rs. 2.88 crore per annum. The rent will be paid on an annual basis with
increment of 30% at the interval of every 5 years and the plantation properties
(agricultural estate and the project office) will be transferred to the cooperative
society at the end of 25th year. The rent accrual is agreed from the commercial
operation date (COD), which tentatively is proposed from 1st of Shrawan, 2073. No
incentive or residual value will be received by the proposed company at the end of
project i.e. on Ashadh end 2098.
In order to operate the project successfully, you have been hired as his finance
consultant and want you to assess the information properly. From the available
research data adjusted with current inflation figures, you have been able to gather
the following information in respect of annual operating cost and capital cost:
Operating costs (Amount in Rs.)
Particulars of operating expenses Estimated annual cost Annual
growth (in
every 5 years)
Orchard Activities (Pruning and Training, Thinning,
Fertilizers, General Farm Labor, Irrigation and Electricity)
Cost of activities starting from Year 1 1,88,10,000 15%
Cost of activities starting from Year 2 14,85,000 15%
Harvesting Activities (Picking Labor, Other Labor and Hauling of Apples)
Cost of activities starting from Year 3 40,19,000 15%
Administration Expenses (except rent)
Cost of activities starting from Year 3 54,00,000 15%
Capital cost
Particulars of Capital Expenses Cost
Land Development and Ancillary Cost 5,00,00,000
Plantation Cost 12,00,00,000
Other capital investments (viz., quarters and project office 2,00,00,000
set up in leased land)
Further, you have been informed that he expects to harvest 1,500 metric tons of
apples during the 3rd year after COD, then 200 metric tons increase every year until
SFB P.T.O.
(2)
8th year after COD and steady production from 9th year onwards with no further
growth or decline during the entire project period. In addition, he expects to fetch at
least Rs.60 per kg during the first harvesting season, i.e. on 3rd year after COD and
then 15% growth after every three years adjusted with inflation till 10th year
(consider price rounding in nearest rupee).
Consider:
 tax rate as 20%;
 no tax outflows during the accumulated loss period;
 useful life of all initial investments as equivalent to the project period (lease
term) and depreciation method as SLM basis
 All assets accounted as leasehold asset and categorised under Class "E", hence
no deferred tax and temporary differences; and
 cost of capital as 10% (3 decimal places) as given below
Year 0 1 2 3 4 5 6 7 8 9 10
PV Factor @ 10% 1.000 0.909 0.826 0.751 0.683 0.621 0.565 0.514 0.467 0.425 0.386

Required: (14+6=20)
a) Based on the NPV & IRR from the aforementioned information, would you
suggest him to take up this project based on the 10 years cash flow, at least from
financial perspective? Consider carrying amount of initial capital investment at
the end of 10th year as its terminal value for your computation. (Calculate in
nearest thousand).
b) What three factors other than the financial should an investor look into before
taking up the project? Your answer should present factors in connection to the
above apple farming case study.
Answer
a)
Calculation of Initial Investments, Annual Depreciation and Terminal Value
Particulars Rs. In 000
Initial Investment
Land Development and Ancillary Cost 50000
Plantation Costs 120000
Other Capital Investments 20000
Total Initial Investments 190000
Useful Life of Initial Investments 25 years
Annual Depreciation 7600
Total Depreciation on 10 years 76000
Terminal Value after 10 years 114000

Year 0 1 2 3 4 5 6 7 8 9 10
Production in Tons 0 0 1500 1700 1900 2100 2300 2500 2500 2500
Price per Kg 60 60 60 69 69 69 79 79

Rs. In 000
Sales 0 0 90000 102000 114000 144900 158700 172500 197500 197500

18810 18810 18810 18810 18810 21632 21632 21632 21632 21632
Orchard Activities
1485 1485 1485 1485 1485 1708 1708 1708 1708
Harvesting
Activities 4019 4019 4019 4019 4019 4622 4622 4622
Administration
Expenses (except
rent) 5400 5400 5400 5400 5400 6210 6210 6210

SFB P.T.O.
(3)

Total Costs Except


Rent and
Depreciation 18810 20295 29714 29714 29714 32536 32759 34172 34172 34172
Rent 28800 28800 28800 28800 28800 37440 37440 37440 37440 37440
Depreciation 7600 7600 7600 7600 7600 7600 7600 7600 7600 7600
Total Costs 55210 56695 66114 66114 66114 77576 77799 79212 79212 79212

Profit Before Tax -55210 -56695 23886 35886 47886 67324 80901 93288 118288 118288

Accumulated
Profit -55210 -111905 -88019 -52133 -4247 63077 143978 237266 355554 473842

Tax 0 0 0 0 0 12615 16180 18658 23658 23658

Profit After Tax -55210 -56695 23886 35886 47886 54709 64721 74630 94630 94630
Add: Depreciation 7600 7600 7600 7600 7600 7600 7600 7600 7600 7600
Cash Profit -47610 -49095 31486 43486 55486 62309 72321 82230 102230 102230
Initial Investments -190000
Terminal Value 114000
Net Cash Flow -190000 -47610 -49095 31486 43486 55486 62309 72321 82230 102230 216230
PV Factor @ 10% 1.000 0.909 0.826 0.751 0.683 0.621 0.565 0.514 0.467 0.425 0.386
Discounted Cash
Flow -190000 -43277 -40552 23646 29701 34457 35205 37173 38401 43448 83465
Net Present Value 51667

At discount rate of 10%, the project has a positive NPV of Rs.51,667K hence a much higher discount rate
(15%) is considered to compute IRR.

Net Cash Flow -190000 -47610 -49095 31486 43486 55486 62309 72321 82230 102230 216230
PV Factor @ 15% 1.000 0.870 0.757 0.658 0.572 0.497 0.432 0.376 0.327 0.284 0.247
Discounted Cash
Flow -190000 -41421 -37165 20718 24874 27577 26917 27193 26889 29033 53409
Net Present Value -31976

Thus IRR shall be


NPV at Low Rate
= + Low × (High Rate - Low
Rate NPV at Low Rate - NPV at High Rate)
Rate
= 10%+ (51,667 / ( 51,667 - ( -31,976 ))) × 5%
= 13.09%

From the above calculation of NPV & IRR, the project can be undertaken.
b) The financial assessment of the project is indeed very important to ascertain its viability.
Nevertheless, the investors should also look into other factors such as political, social and
technological in order to determine whether these factors complement to its progression.
In addition, there is also a growing trend of examining the factors like environmental and legal /
regulatory that could influence the feasibility of the project. As far as assessment is concerned, all
these components are indispensable and interrelated.
The political factors have significant bearing in the investment of the project. These include law and
order situation, stability of the government and its policies, availability and development of adequate
infrastructures, tax regime and subsidies to the investors, transportation facilities, adequate law for
free flow of goods and services within the region etc. Any disorder in the factors mentioned above
would discourage the investors to put in their hard earned money as there will be severe uncertainty
on economic rewards they expect from the project.

SFB P.T.O.
(4)
Besides political, social factors also have important bearing on the feasibility assessment. For
instance, the proposed project is planned to be implemented in one of the hilly regions and it is
important for Mr. X to understand the behaviors, attitudes, education, population, age group and
employment status of the local community. If the district lacks youth force, the project may find
difficulty in getting workers and labors and consequently the cost of production may shoot up
making the project less attractive.
In addition to the two factors mentioned above, the technology bestows several advantages to the
investors of the present world. The process automation and the more use of technology eventually
contribute in reducing the cost of operations thereby assisting the project to achieve the goal of long
term sustainability. Again, the research and development plays a crucial role in developing and fine
tuning the product which would lead innovating to a unique product. Like any other product
researches, the apple fruit research cannot be ignored at all.
2.
a) The Samriddhi Ltd. propose to raise its turnover from Rs. 6,00,000 to Rs.
8,40,000 next year and to Rs. 9,60,000 in the succeeding year. It is expected that
the purchases will go up from Rs. 1,80,000 to Rs. 2,40,000 and then to Rs.
2,70,000 in the next two years.
A steady profit of 10% on turnover is estimated over the years; and the materials,
labour and factory overheads are expected uniformly to be 30%, 20% and 30%
respectively of the total cost of goods sold.
At the end of each year the raw materials stock would amount to two months'
consumption, work-in-progress to one month's factory cost and finished goods to
half a month's total cost. There is a two months' credit period allowed to
customers and received from suppliers.
The company has a policy of carrying cost equivalent to one month's requirement
for payment of labour and other overhead cost. Ignoring prepayments and accrued
charges as they normally offset each other, please work out an estimate of
working capital requirement for all the three years separately. State assumptions,
if any. 10
b) A firm has a total sales of Rs. 200 lakhs of which 80% is on credit. It is offering
credit terms of 2/40, net 120. Of the total, 50% of customers avail discounts and
the balance pay in 120 days. Past experience indicates that bad debt losses are
around 1% of credit sales. The firm spends about Rs. 2,40,000 per annum to
administer its credit sales. These are avoidable as a factor is prepared to buy the
firm‟s receivables. He will charge 2% commission. He will pay advance against
receivables to the firm at an interest rate of 18 % after withholding 10% as
reserve. 5
i) What is the effective cost of factoring? Consider year as 360 days.
ii) If bank finance for working capital is available at 14% interest, should the
firm avail of factoring service?
Answer
a) Workings:

SFB P.T.O.
(5)
Particulars Year 1 Year 2 Year 3
Estimated sales 600,000 840,000 960,000
Less: Profit (@10% on sales) 60,000 84,000 96,000
Cost of goods sold (a) 540,000 756,000 864,000

Factory costs
Material cost (30% of total cost) 162,000 226,800 259,200
Labour cost (20% of total cost) 108,000 151,200 172,800
Overhead cost (30% of total cost) 162,000 226,800 259,200
(b) 432,000 604,800 691,200

Administration, selling and distribution expenses (a-b) 108,000 151,200 172,800

Purchase estimation 180,000 240,000 270,000


Estimation of Working Capital requirement
Particulars Year 1 Year 2 Year 3

Current Assets:
Raw material stock 2 months raw material cost 27,000 37,800 43,200

Work in Progress 1 month factory cost 36,000 50,400 57,600

Finished goods 1/2 month of cost of goods sold 22,500 31,500 36,000

Sundry debtors 2 months of credit sales 100,000 140,000 160,000


1 month of labour cost, factory
overhead, administration selling
Cash and distribution expenses 31,500 44,100 50,400
a 217,000 303,800 347,200

Current Liabilities:
Sundry Creditors 2 months of purchase estimation 30,000 40,000 45,000
b 30,000 40,000 45,000

Estimated Working Capital (a-b) 187,000 263,800 302,200

b) Total sales = Rs. 200 Lakhs


Credit Sales (80%) = Rs. 160 lakhs
Receivable for 40 days = Rs. 80 lakhs
Receivable for 120 days = Rs. 80 lakhs
Average collection period [(40x0.5) + 120x0.5)] = 80 days
Average level of receivables (Rs. 1,60,00,000x80/360)=Rs. 35,55,556
Factoring Commission= Rs. 35,55,556 x 2% = Rs.71,111
Factoring Reserve = Rs. 35,55,556 x 10% = Rs.3,55,556
Amount available for advance = Rs. 35,55,556 – (3,55,556+71,111)=Rs.31,28,889
Factor will deduct his interest at 18%,
So, interest = Rs.31,28,889 x 18% x 80/360= Rs. 1,25,156
Advance to be paid = Rs.31,28,889 – 1,25,156=Rs.30,03,733
(i) Calculation of effective cost of factoring
Annual cost of factoring to the firm:
Factoring Commission (Rs.71,111 x 360/80) = Rs. 3,20,000
Interest Charges (Rs.1,25,156 x 360/80) = Rs. 5,63,200
Sub Total Rs. 8,83,200
Firm‟s saving on using Factoring services
Cost of credit administration saved Rs. 2,40,000
Bad debts (Rs.1,60,00,000 x 1%) avoided Rs. 1,60,000
Sub Total Rs. 4,00,000
Net Cost to the firm = Rs,8,83,200 – Rs. 4,00,00 = Rs. 4,83,200
SFB P.T.O.
(6)
Effective cost of factoring = Rs.4,83,200 / Rs. 30,03,733 x 100 = 16.09 %

(ii) If bank finance for working capital is available at 14%, the firm should not avail factoring
services as 14% is lower than effective cost of factoring service i.e. 16.09%
3.
a) Following are the financial details of two farmers Mr. M and Mr. N, who belongs
to the same area at the end of financial year 2071-72:
Income Statement
Rs. in Lakhs
M N
Gross Revenue (Value from farm 125 80
production)
Cost of Production 64 56
Gross Farm Income 61 24
Operating Expense attributable to
agriculture 24 12
Interest Expense on Agricultural Loans 4 1
Net Farm Income from Operations 33 11
Tax - -
Net Farm Income 33 11
Balance Sheet
Rs. in Lakhs
M N
Assets
Biological Asset (harvested paddy) 49 22
Non Current Asset 75 40
Current Asset 9 3
Total Assets 133 65
Previous Year Total Assets 97 48

Equity and Liabilities


Equity 63 37
Non Current Liability (full debt) 14 8
Current Liability (80% renewable debt) 56 20
Total Equity and Liabilities 133 65
The increase in total assets of Mr. M owed to increase in non current asset (80%
of total assets increment), whereas increase in total assets of Mr. N owed to
increase in biological asset (75% of total asset increment). Furthermore, the
current asset of Mr. M and Mr. N grew by Rs.2 lakhs and Rs.1 lakh respectively.
Required (2+2+4=8)
i) Calculate profit margin ratio, asset turnover and debt-equity ratio.
ii) Calculate return on equity using Du Pont formula.
iii) Analyse the ratios from calculation parts (i and ii) with reference to the
information given above and make apt suggestions. You may consider
suitable assumptions / calculations and subsidies on interest rates and
insurance for your justification.

b) The Board of Z Ltd. is seeking an improvement plan on the company‟s working


capital management. Currently, the Company has total turnover (credit sales) of
Rs. 23,09,00,000 per year and has a policy of providing 30 days credit to its
customers. Per facts furnished to the Board by the management, the average
SFB P.T.O.
(7)
collection period is more than the prescribed limit and it is estimated that around
1% of the turnover turn into bad that are not recoverable. The company has
around Rs. 5,07,00,000 in accounts receivables and has a cost of short-term
finance at 8% per annum.
The CFO has invited for a proposal from a factoring company, D Ltd., to manage
the receivables/sales account of the company on a with-recourse basis. Per
proposal submitted to the company, the D Ltd. has committed to use its expertise
to reduce average trade receivables days to 27 days, cutting bad debts by 60% and
reducing administration costs by Rs. 15,00,000 per year.
In addition, D Ltd. shall also advance Z Ltd. 70% of the value of invoices raised
at an interest rate of 9% per year. The D Ltd. shall charge an annual fee of 1% of
credit sales.
Assume 365 days for a year.
Required: 7
Is the factor‟s offer financially viable to Z Ltd?
Answer
a)
i. Calculation of Profit Margin Ratio, Asset Turnover and Debt-Equity Ratio
Mr. M Mr. N
Net Farm Income (Rs. in Lakh) A 33 11
Gross Revenue (Value from farm production) (Rs. in Lakh) B 125 80
Profit Margin Ratio A/B 26.40% 13.75%

Gross Revenue (Value from farm production) (Rs. in Lakh) C 125 80


Average Farm Assets (Rs. in Lakh) D 115 56.5
Asset Turnover C/D 1.09 1.42

Debt (Full Debt + Renewable Debt) E 59 24


Equity F 63 37
Debt - Equity Ratio E/F 0.93 0.65
ii. Calculation of Return on Equity using Du Pont formula
Mr. M Mr. N

Profit Margin Ratio G 26.40% 13.75%

Asset Turnover H 1.09 1.42

Calculation of Equity Multiplier


Average Farm Assets (Rs. in Lakh) I 115 56.5
Equity J 63 37
Equity Multiplier K(I/J) 1.83 1.53

Return on Equity GHK 52.66% 29.87%


iii. The profit margin ratio of Mr. M is very strong (26.40%) vis-à-vis Mr. N who has almost of
half at 13.75%; however the asset turnover of Mr. M is lower than the Mr. N (1.09 versus
1.42). The equity multiplier and debt equity ratio of the Mr. M is higher than the Mr. N
indicating that Mr. M has higher financial leverage ratios than Mr. N.
As per given information, Mr. M has acquired non-current assets of Rs.28.8 lakhs (Rs.36 lakhs
 80%), whereas Mr. N has acquired non-current assets of mere Rs.3.25 lakhs (Rs.17 lakhs 
25% - Rs.1 lakh). This resembles that Mr. M is expanding its business the consequences of
which may be evident in the years to come when he uses his infrastructure at the optimum.
SFB P.T.O.
(8)
On the contrary, the increment of biological asset of Mr. N is Rs.12.75 lakhs (Rs.17 lakhs 
75%) as against Rs.5.20 lakhs (Rs.36 lakhs  20% - Rs.2 lakhs) of Mr. M. The inventory
turnover ratio of Mr. M and Mr. N is 1.31 and 2.09 times respectively which coupled with asset
turnover ratio reflects that Mr. N had better performance during the year and was more efficient
in terms of infrastructures.
The debt equity ratio of both farmers is below 1 and therefore can be increased ideally to the
extent of 2:1 in order to improve their profitability ratios giving due consideration to the
subsidies available to the farmers on interest rates (as low as 5% on agricultural credit) and
crop insurance (50% waiver).
Considering the asset and inventory turnovers, which seem quite low assuming that there is no
seasonal storing at the end of the financial year, both need to improve their production
efficiency or price levels. In addition, they may reduce biological assets through regular
disposal thereby reducing financing and inventory handling cost and also focusing to integrate
the paddy farming with animal husbandry, dairy farming, beekeeping, vegetable farming etc.
Working Note:
1. Calculation of Inventory Turnover Ratio
Cost of production 64 56
Biological Asset 49 22
Inventory Turnover 1.31 2.55

b) The offer of factor will be financially viable only if there is net benefit from the arrangement.

Current accounts receivables 5,07,00,000


Revised accounts receivables 23,09,00,000 27/365 1,70,80,274
Reduction in receivables 3,36,19,726

Reduction in financing costs of


receivables @ 8% 3,36,19,726  8% 26,89,578
Saving in bad debts 23,09,00,000  1%  60% 13,85,400
Reduction in administration costs (given) 15,00,000
Total Savings 55,74,978

Factor's annual fee 23,09,00,000  1% 23,09,000


Increase in financing cost on 23,09,00,000  70%  (9% -
advance 8%) 16,16,300
Total Cost 39,25,300

Net Benefit from the arrangement 16,49,678

The factor‟s offer is financially viable to Z Ltd. since it saves Rs.16,49,678 annually from the
arrangement.
4.
a) The Capital structure of the company is as below:
Particulars Amount in (Rs.)
Equity share capital of Rs.10 each 8,00,000
8% preference share capital of Rs.10 each 6,25,000
10% Debenture of Rs.100 each 4,00,000
Total 18,25,000
Additional information:
Profit after tax (tax rate 30%) Rs. 1,82,000
Operating expenses (including depreciation Rs. 90,000) being 1.50 times of EBIT.
Assume all operating expenses excluding depreciation are variable.
SFB P.T.O.
(9)
Equity dividend paid 15%
Market price per equity share Rs.20
Required: (2+2+2+2=8)
i) Operating and financial leverage
ii) Cover for the preference and equity share of dividends
iii) The earning yield and price earnings ratio
iv) The net fund flow
b) A company requires Rs. 25,00,000 for a new plant. This plant is expected to yield
earnings before interest and taxes of Rs. 5,00,000. While deciding about the
financial plan, the company considers the objective of maximizing earning per
share. It has three alternatives to finance the project as below:
i) Raise debt of Rs. 2,50,000 and balance by issuing equity shares,
ii) Raise debt of Rs. 10,00,000 and balance by issuing equity shares
iii) Raise debt of Rs. 15,00,000 and balance by issuing equity shares
The company‟s share is currently selling at Rs. 150, but is expected to decline to
Rs. 125 in case the funds are borrowed in excess of Rs. 10,00,000. Assume that
company can raise cash from issue of equity at these market prices.
The funds can be borrowed at the rate of 10% up to Rs. 2,50,000, at 15% over Rs.
2,50,000 and up to Rs. 10,00,000 and at 20% over Rs. 10,00,000. The tax arte
applicable to the company is 50%.
Required: 7
Which form of financing should the company choose?
Answer
a) Working Notes:
Net profit after tax Rs. 1,82,000
Tax @30% 78,000
Earning Before tax 2,60,000
Interest on Debenture@10% 40,000
EBIT 3,00,000
Operating Expenses 1.5 times 4,50,000
Sales 7,50,000
(i) Operating Leverage= Contribution/EBIT=(7,50,000-3,60,000)/3,00,000=1.30 times
Financial Leverage = EBIT
EBIT-Int- PD
1-Tax
= 3,00,000
3,00,000-40,000- 5,000
1-0.30

= 1.59 times
(ii) Preference Dividend Cover = PAT/Preference share dividend
= Rs. 1,82,000/(6,25,000 x 8%) = 3.64 times
Equity dividend cover = (PAT – Preference share dividend)/equity dividend
=(Rs.1,82,000 – 50,000)/ (8,00,000 x15%)=1.10 times
(iii) Earning yield = (EPS/ Market price) x 100 = [(1,32,000/80,000)/20] = 8.25%
Price Earning ratio = Market price/EPS = 20/1.65=12.12 times
(iv) Net Fund flow = Net profit after tax + depreciation – total dividend
= 1,82,000 + 90,000 – (50,000 + 1,20,000)
= 2,72,000 – 1,70,000
= Rs. 1,02,000

SFB P.T.O.
(10)
b) Calculation of Earning Per Share for three alternatives to finance the project
Alternatives
I II III
Raise debt of Raise debt of Raise debt of
Particulars
Rs.2,50,000 and Rs.10,00,000 and Rs.15,00,000 and
equity of equity of equity of
Rs.22,50,000 Rs.15,00,000 Rs.10,00,000
EBIT Expected 5,00,000 5,00,000 5,00,000
Less: Interest on 25,000 1,37,500 2,37,500
Debt at rates given
in question
Earnings Before Tax 4,75,000 3,62,500 2,62,500
Less: tax at 50% 2,37,500 1,81,250 1,31,250
Earnings After Tax 2,37,500 1,81,250 1,31,250
Number of shares in 15,000 10,000 8,000
each alternative
(Equity/Market Price
per share)
Earning Per Share 15.833 18.125 16.406
The company should raise Rs.10,00,000 from debt and Rs.15,00,000 by issuing equity shares as
this alternative gives highest EPS.
5.
a) Determine the market value of equity shares of the company from the following
information as per Walter`s Model:
Earnings of the company Rs. 5,00,000
Dividend paid Rs. 3,00,000
Number of the shares outstanding 1,00,000
Price-earning ratio 8
Rate of return on investment 15%
Are you satisfied with the current dividend policy of the firm? If not what should
be the optimal dividend payout ratio? 5
b) The following data relating to two securities, A and B.
A B
Expected return 22% 17%
Beta factor 1.5 0.7
Assume Risk Free Interest (Rf)=10% and Return Market (Rm)=18%. Find out
whether the securities A and B are correctly priced? 5
c) Calculate the value of equity share from the followings: 5
Particulars Amount in Rs.
Equity Share Capital (Rs. 20 each) 50,00,000
Reserve and Surplus 5,00,000
15% Secured Loans 25,00,000
12.5% Unsecured Loans 10,00,000
Fixed Assets 30,00,000
Investments 5,00,000
Operating Profit 25,00,000
Tax Rate 50%
P/E Ratio 12.5

SFB P.T.O.
(11)
Answer
a) Price earning ratio= Market Price/EPS
8= Market Price/5
So, Market Price=Rs. 40
EPS= 500,000/100,000=Rs.5
DPS=300,000/100,000=Rs. 3
Dividend payout ratio=DPS/EPS*100=3/5*100=60%
Walter’s Formula
As the P/E ratio is given 8, and the cost of capital (Ke) is also defined as the reciprocal of P/E ratio,
therefore the Ke may be taken as 1/8=.125 i.e.12.5%
Since, this is a growth firm having rate of return (15%) more than cost of capital(12.5%), therefore,
the company will maximize its market price if it retains its 100% of its profits. The current market
price of Rs. 40 ( based on P/E Ratio can be increased by reducing the payout ratio. If the company
opts for 100% retention (i.e. 0% payout), the market price of the share as per Walter‟s formula
should be as follows;
P=D/Ke+((r/Ke)(E-D))/Ke
P=0/.125+((.15/.125)(5-0))/.125
= Rs. 48
So, the firm can increase the market price of the share up to Rs. 48 by increasing the retention ratio
to 100% or in other words, the optimal dividend payout for the firm is 0.

b) Calculation of return under CAPM


Company A=Rf+B(Rm-Rf)
=10+1.5(18-10)
=22%
Company B=Rf+B(Rm-Rf)
=10+0.7(18-10)
=15.6%
Security E(R) Expected Return Return under CAPM Position
A 22% 22% Correctly Priced
B 17% 15.6% Under Priced

The return from security A exactly equal to the calculated return under CAPM hence it is correctly
priced securities.
The return from security B is better than the return under CAPM. It indicates a favorable position
.i.e. the security is currently traded at underpriced position.
c) In the given situation, the value of the share can be ascertained on the basis of earnings of the firm
and the price-earnings multiple as follows:
Value = EPS* P/E Ratio
The P/E Ratio is given and the EPS may be ascertained as follows:
Amount in Rs.
Operating Profit i.e. EBIT 2,500,000
Less: Interest on 15% secured loans 375,000
Interest on 12.5% Unsecured loans 125,000
Profit before Tax(PBT) 2,000,000
Tax@50% 1,000,000
Profit after Tax 1,000,000
Number of Equity Shares(Rs. 5000000/20) 250,000

Therefore, EPS(Rs.1000000/250000) 4.00


P/E Ratio(given) 12.5
Therefore,
Value of equity shares= EPS*P/E Ratio

SFB P.T.O.
(12)
=4*12.5= Rs. 50
6. Write Short Notes on: (4×2.5=10)
a) Debt Securitisation
b) NEPSE Index
c) Risk and Uncertainty
d) Basic functions of financial management
Answer
a) Debt securitisation is a method of recycling of funds. It is especially beneficial to financial
intermediary to support the lending volumes. Assets generating steady cash flows are packaged
together and against this assets pool, market securities can be issued, e.g. housing finance, auto
loans and credit cards receivables. The process of securitization is generally without recourse i.e.
investors bear the credit risk and issuer is under an obligation to pay to investors only if the cash
flows are received by him from the collateral. The benefits to the originator are that assets are
shifted off the balance sheet, thus the originator recourse to off-balance sheet funding.
b) The Nepal Stock Exchange is a value weighted index of all shares listed at the Nepal Stock
Exchange and calculated on a daily basis (for the days market remain open) at the closing price.
The calculation of the NEPSE index is based on the concept of the market capitalization which is
the sum of the market capitalization of all the company listed in the Nepal Stock Exchange. If the
ratio of current period market capitalization to the base period market capitalization is multiplied
by the multiplier 100, we get NEPSE index. This method of index calculation is called value
weighted method.
Total Market Capitalization of all the Companies Listed
NEPSE Index= -------------------------------------------------------------------------------* 100
Total Base Year`s Market Capitalization

However in reality, the number of the listed companies keeps on changing, and the number of the
outstanding shares also keeps on changing as the company issues right shares or bonus shares or
common shares at the time of capital needs. The actual practice to adjust the base period is as
follows:
Adjusted Base Period =(New Market Capitalization including new listing/New Market
Capitalizations excluding new listing )* Base Year's Market Capitalization.
c) In common parlance, the terms Risk and Uncertainty have synonymous meaning. However, they
differ from each other. Risk may be defined as the chance of future loss that can be foreseen. In
other word, in case of risk an estimate can be made about the degree of happening of the loss. This
is usually done by assigning to the probabilities of risk on the basis of past data and the probable
trends.
Uncertainty may be defined as “the unforeseen chance for future loss or damages.” In case of
uncertainty, since the firm cannot anticipate the future loss, and hence it cannot directly deal with it
in its planning process, as is possible in the case of risk. For example, a firm cannot foresee the loss
which may be due to destruction of its plant in account of earthquake.
d) The basic functions of the financial management are as discussed below:
i) Procurement of funds – identifying the source of financing considering the risk, costs and
control.
ii) Effective utilization of funds – funds are used for beneficial ventures and not kept idle.
iii) Generation of reports for decision makers
iv) Preparation of financial reports
v) Ensure proper financial accountability
7. Distinguish between: (4×2.5=10)
a) Preference share and Debenture

SFB P.T.O.
(13)
b) Horizontal analysis Vs. Vertical analysis
c) Financial lease and Operating lease
d) Euro convertible bonds Vs. Euro convertible zero bonds.
Answer
a) Preference share is a special kind of share whose holders enjoy priority both as regard to the payment
of a fixed amount of dividend and also towards repayment of capital in case of winding up of a
company. Preference shares are a hybrid form of financing with some characteristics of equity shares
and some attributes of debt capital.
A debenture is a type of loan which can be raised from the public. It carries fixed percentage of
interest. Debentures are instruments for raising long term capital with a period of maturity. Some
debenture can be convertible to equity.
b) Horizontal analysis: this technique is also known as comparative analysis. It is conducted by setting
consecutive balance sheet, income statement or cash flow side by side and reviewing changes in
individual categories on year to year or multi year basis. The most important item revealed by
comparative financial statement analysis is trend. A comparison of statements over several years
reveals direction speed & extents of a trends. The horizontal financial statement is done by restating
amount of each item or group of items as a percentage.
Vertical analysis: Vertical/Cross sectional/Common size statements came from the problems in
comparing the financial statements of firms that differ in size. The vertical analysis represents the
relationship of different items of financial statements which some common items by expressing each
item as a percentage of common item. In common size income statements, each item is stated as
percentage of net sales. The percentage of different items are computed by dividing the absolute item
by the common base (i.e. the balance sheet total or net sales as the case may be), and multiplying by
hundred.
c) In Financial lease, the risk and reward incident to ownership are passed on to the lessee. The lessor
only retains the legal ownership of the assets. The lessor does not bear the cost of repairs,
maintenance or operations. The lease is usually full payout.
In operating lease, the lessee is only provided the use of the assets for a certain time. Risk incident to
ownership belongs to the lessor. Usually, the lessor bears the cost of repairs, maintenance and
operations. The lease is usually non-payout.
d) Euro convertible bond is a Euro bond, a debt instrument which gives the bond holders an option to
convert them into a pre determined number of equity shares of the company. Usually the price of the
equity shares at the time of conversion will have a call option (where the issuer company has the
option of calling/buying the bonds for redemption prior to the maturity date) or a put option (which
gives the holder the option to put/sell his bonds to the issuer company at a predetermined date &
price).
Euro convertible zero bonds are structured as convertible bond. No interest is payable on the
bonds. But conversion of bonds takes place on maturity at a predetermined price. Usually there is a
five years maturity period and they are treated as deferred equity issue.

SFB P.T.O.
Cost & Management Accounting
Suggested
Roll No……………. Maximum Marks - 100
Total No. of Questions: 6 Total No. of Printed Pages - 5
Time Allowed - 3 Hours
Marks
All questions are compulsory. Working notes should form part of the answer.
Make assumptions wherever necessary.
1. PQR Limited produces two joint products P and Q together with a by-product R, from
a single main process (process 1). Product P is sold at the point of separation for Rs.5
per kg., whereas product Q is sold for Rs.7 per kg. after further processing into
product Q2. By- product R is sold without further processing for Rs.1.75 per kg.
Process 1 is closely monitored by a team of chemists, who planned the output per
1000 kg of input materials to be as follows:
Product P 500 kg.
Product Q 350 kg.
Product R 100 kg.
Toxic waste 50 kg.
The toxic waste is disposed of at a cost of Rs.1.50 per kg, and arises at the end of
processing.Process 2, which is used for further processing of product Q into product
Q2, has the following cost structure:
Fixed costs Rs.6,000 per week
Variable costs Rs.1.50 per kg. processed
The following actual data relate to the first week of accounting period 10:
Process 1
Opening work in process Nil
Materials input 10,000 kg.costing Rs.15,000
Direct labour Rs.10,000
Variable overhead Rs.4,000
Fixed overhead Rs.6,000
Outputs:
Product P 4,800 kg.
Product Q 3,600 kg.
Product R 1,000 kg.
Toxic waste 600 kg.
Closing work in progress Nil
Process 2
Opening work in process Nil
Input of product Q 3,600 kg.
Output of product Q2 3,300 kg.
Closing work in progress 300 kg, 50% converted
Conversion costs were incurred in accordance with the planned cost structure.
Required: (7+7+6=20)
a) Prepare the main process account for the first week of period 10 using the final
sales value method to attribute pre-separation costs to joint products.
b) Prepare the toxic waste accounts and process 2 accounts for the first week of
period 10.
YDT P.T.O.
(15)
c) Advise the management of PQR Limited whether or not, on purely financial
grounds, it should continue to process product Q into product Q2:
i) if product Q could be sold at the point of separation for Rs.4.30 per kg; and
ii) if 60% of the weekly fixed costs of process 2 were avoided by not processing
product Q further.
Answer
a) Normal loss (toxic waste) = 50 kg per ,000 kg of input (i.e. 5%)
Actual input = 10,000 kg
Abnormal loss = Actual toxic waste (600) less normal loss (500) = 100 kg
By-product R net revenues of Rs.1750 are credited to the joint (main) process account and
normal and abnormal losses are valued at the average cost per unit of output:
Net cost of production (35,750-1,750)/ Expected output of the joint products (8,500 kg) = Rs.4

The cost of the output of the joint products is Rs.33,600 (8,400 kg ×Rs.4) and this is to be
allocated to the individual products on the basis of final sales value (i.e. 4,800 kg ×Rs.5 =
Rs.24,000 for P and 3600 kg ×Rs.7 = Rs.25 200 for Q):
P = Rs.24 000/Rs.49 200 ×Rs.33 600 = Rs.16 390
Q = Rs.25 200/Rs,49 200 ×Rs.33 600 = Rs.17 210
The main process account is as follows:
Main process account
Kg Rs. Kg Rs.
Materials 10,000 15,000 P Finished 4,800 16,390
goods
Direct Labor 10,000 Q Process 2 3,600 17,210
Variable Overhead 4,000 By-Product R 1,000 1,750
Fixed Overhead 6,000 Normal 500
Toxic Waste
Toxic Waste 750 Abnormal 100 400
disposal A/c Toxic Waste
10,000 35,750 10,000 35,750
b)
Normal Waste A/c
Rs. Rs.
Bank 750 Main Process 750
Account
750 750

Abnormal toxic waste account


Rs. Rs.
Main Process 400 Profit and Loss 550
Account Account
To bank ( 100 ×1.50) 150
550 550

Process 2 account
Kg Rs. Kg Rs.
Main Process Q 3,600 17,210 Finished 3,300 26,465
Goods Q (
Note 2)
Fixed Cost 6,000 Closing 300 1,920
Work in
Progress
YDT P.T.O.
(16)
( Note 2)
Variable Cost 5,175
( Note 1)
3,600 28,385 3,600 28,385
Notes:
Note1. 3300 + (50% * 300) * Rs.1.50 = Rs.5175
Note2.
Rs. Completed WIP Total Cost Per
Units Equivalent Equivalent Unit
Units Units
Previous Process Cost 17,210 3,300 300 3,600 Rs.4.78
Conversion Cost 11,175 3,300 150 3,450 Rs.3.24
Rs.8.02
Therefore,
Rs.
Completed Units (3,300 × Rs.8.02) 26,465
WIP (300×Rs.4.78 + 150×Rs.3.24) 1,920
Total 28,385

(c)
Incremental sales revenue per kg from further processing (Rs.7 – Rs.4.30) Rs.2.70
Incremental (variable) cost per kg of further processing Rs.1.50
Incremental contribution per kg from further processing Rs.1.20
At an output of 3600 kg the incremental contribution is Rs. 4320
Avoidable fixed costs Rs.3600
Net benefit Rs.720
Since the net benefit increased by Rs. 720 under further processing, it should be further processed.

2.
a) The annual demand for an item of raw material is 4,000 units and the purchase
price is expected to be Rs. 90 per unit. The incremental cost of processing an
order is Rs. 135 and the cost of storage is estimated to be Rs. 12 per unit. (2+4+4=10)
i) What is the optimal order quantity and total relevant cost of this order
quantity?
ii) At the commencement of the year a supplier offers 4,000 units at a price of
Rs. 86.The materials will be delivered immediately and placed in the stores
and the incremental cost of placing the order is zero. Should the order be
accepted?
iii) At the time of annual cost audit it was revealed that Rs. 135 as estimated to be
the incremental cost of processing an order is incorrect and should have been
Rs. 80. All other estimates are correct. What is the difference in cost on
account of this error?
b) The following standards have been set to manufacture a product:
Rs.
Direct material
2 units of A @ Rs.4 per unit 8.00
3 units of B @ Rs.3 per unit 9.00
15 units of C @ Re. 1 per unit 15.00
32.00
Direct labour 3 hrs. @ Rs.8 per hour 24.00
YDT P.T.O.
(17)
Total standard prime cost 56.00
The company manufactured and sold 6,000 units of the product during the year.
Direct material costs were as follows:
12,500 units of A at Rs.4.40 per unit
18,000 units of B at Rs.2.80 per unit
88,500 units of C atRs.1.20 per unit
The company worked 17,500 direct labour hours during the year. For 2,500 of
these hours the company paid at Rs.12 per hour while for the remaining the
wages were paid at standard rate.
Calculate materials price variances and usage variances and labour rate and
efficiency variances. 8
c) State the objectives of standard costing technique? 2
Answer
a)
i) Optimum order quantity (EOQ) = 2AO
√ C
Where
A = Total Annual requirements of raw materials in units
O = Ordering cost per order
C = Raw materials carrying cost per unit per annum

= 2× 4,000 units× Rs.135


√ Rs. 12

= 300 units
Total relevant cost when order quantity is 300 units
= Ordering cost + Carrying cost
= No. of orders × ordering cost per order + ½ Order size × carrying cost per unit per annum
= 4,000 units ×Rs. 135 + ½ × 300 units × Rs. 12
300 units
= Rs. 1,800 + Rs. 1,800
= Rs. 3,600

ii) Comparision of cost at Special offer and EOQ


Order Total units Price per Total cost of Ordering Carrying Total Cost
Size purchased unit purchase Cost Cost (Rs.)
(units) (Rs.) (Rs.) (Rs.) (Rs.)
Special 4,000 4,000 86 3,44,000 Nil 24,000 3,68,000
Offer
EOQ 300 4,000 90 3,60,000 1,800 1,800 3,63,600
Difference in cost 4,400

Since the special offer of Rs. 86 per unit on the initial purchase of 4,000 units imposes an
additional cost of Rs.4,400, the offer should not be accepted.
iii)
Revised optimal order quantity = 2× 4,000 units × Rs.80
√ Rs. 12
= 231 units

Revised relevant cost when order quantity is 231 units


= 4,000 units ×Rs. 80 + ½ × 231 units × Rs. 12
231 units
= Rs. 1,385.28 + Rs. 1,386
YDT P.T.O.
(18)
= Rs. 2,771.28
Difference in the relevant cost on account of wrong estimation of ordering cost
= Rs. 3600 – Rs. 2,771.28
= Rs. 828.72
b)
For material cost variances
Actual cost of material used
A 12,500 units x Rs. 4.40 = Rs.55,000
B 18,000 units x Rs. 2.80 = Rs.50,400
C 88,500 units x Rs. 1.20 = Rs 1,06,200
Rs. 2,11,600
Standard cost of material used
A 12,500 units x Rs. 4.00 = Rs. 50,000
B 18,000 units x Rs. 3.00 = Rs.54,000
C 88,500 units x Rs. 1.00 = Rs. 88,500
Rs. 1,92,500
Standard material cost of production 6,000 units x Rs. 32 = Rs. 1,92,000
Variances:
Material price variance = Actual cost of material used – Standard cost of material used
= Rs. 2,11,600 – Rs. 1,92,500
= Rs. 19,100 (A)
Material usage variance = Standard cost of material – Standard material cost of production = Rs.
1,92,500 – Rs. 1,92,000
= Rs. 500 (A)
For labour cost variance
Actual wages paid to workers
2,500 hrs.xRs. 12 = Rs.30,000
15,000 hrs.xRs. 8 = Rs. 1,20,000
Rs. 1,50,000
Payment involved, if workers had been paid at standard rate = 17,500 hrs. xRs. 8
= Rs. 1,40,000
Standard labour cost of output achieved = 6,000 units x Rs 24 = Rs. 1,44,000
Variances:
Labour rate variance = Rs.1,50,000 – Rs.1,40,000 = Rs.10,000 (A)
Labour efficiency variance = Rs. 1,40,000 – Rs. 1,44,000 = Rs. 4,000 (F)
c)
The objectives of standard costing technique are as follows:
(i) To provide a formal basis for assessing performance and efficiency.
(ii) To control costs by establishing standards and analysis of variances.
(iii) To enable the principle of „Management by exception‟ to be practiced at the operational
level.
(iv) To assist in setting budgets.
(v) The standard costs are readily available substitutes for actual average unit costs and can
be used for stock and work-in-progress valuations, profit planning and decision making
and as a basis of pricing where „cost-plus‟ systems are used.
(vi) To assist in assigning responsibility for non-standard performance in order to correct
deficiencies or to capitalize on benefits.
(vii) To motivate staff and management.
(viii)To provide a basis for estimating.
(ix) To provide guidance on possible ways of improving performance.

3.
a) Calcutta Company Ltd. manufactures and sells four types ofproducts under the
YDT P.T.O.
(19)
brand name ACE,UTILITY,LUXURY and SUPREME.The sales mix in value
comprises:
Brand Percentage
ACE 33-1/3%
UTILITY 41-2/3%
LUXURY 16-2/3%
SUPREME 8-1/3%
100%
The total budgeted sales(100%) are Rs. 6,00,000 per month .The operating cost are:
ACE60% of the selling priceLUXURY 80 % of the selling price.
UTILITY 68%of the selling priceSUPREME 40 % of the selling price.
The fixed costs are Rs.1,59,000 per month. Calculate the break even point for the
products on an overall basis.
It has been proposed to change the sales mix as follows, the total sales per month
remaining Rs. 600,000:
Brand Percentage
ACE 25%
UTILITY 40%
LUXURY 30%
SUPREME 5%
100%
Assuming that this proposal is implemented, calculate the new break- even point. 8
b) The boilerhouse is one of the service departments of a company. Steam is raised
and then transferred to production departments and other service departments as
required.
The basic monthly budget figures for 2016 are as follows:
Boiler operating hours: 480
Steam raised: 80,00,000 kg.
Costs:
Fuel (V) Rs.19,200
Chemicals (V) Rs. 960
Wages(F) Rs. 2,400
Sundry overheads(F) Rs. 3,000
The actual figures for February 2016 are as follows:
Boiler operating hours: 432
Steam raised: 67,50,000kg.
Costs:
Fuel (V) Rs. 18,000
Chemicals (V) Rs. 990
Wages(F) Rs. 2,200
Sundry overhead (F) Rs. 3,000
It is expected that the price of chemicals for all output will fall by 2% where the
boiler operates in excess of 480 hours per month. Sundry fixed(F) costs are
expected to fall by Rs. 200 where the boiler is operated for less than 425 hours
and to increase from the normal level by Rs. 250 where the boiler is operated for
more than 480 hours.
Variable(V) costs vary in proportion to boiler hours.
Required: (4+4=8)

YDT P.T.O.
(20)
i) Prepare a budget summary which shows the cost of the boilerhouse in total
and per „000 kg steam for boiler operating levels of 400, 432, 480,and 540
hours
ii) Prepare a control statement which compares budget with actual cost of the
boilerhouse for February where a flexible budgeting system is in operation.
Comment on the variances in the statement.
c) What are the ways for the treatment of defective work? 4
Answer
a)
Computation of the overall break even point
Particulars ACE UTILITY LUXURY SUPREME TOTAL
Sales Mix 33-1/3% 41-2/3% 16-2/3% 8-1/3% 100%
Rs Rs Rs Rs Rs
Sales 200,000 2,50,000 1,00,000 50,000 6,00,000
Less:VariableCosts 1,20,000 1,70,000 80,000 20,000 3,90,000
Contribution 80,000 80,000 20,000 30,000 2,10,000
Composite P/V Ratio =Total Contribution/Total Sales×100
=Rs 2,10,000/Rs 6,00,000×100=35%
Break -even Point(sales value)=Total Fixed Costs/Composite P/V Ratio
=Rs 1,59,000/35%=Rs4,54,286

COMPUTATION OF NEW BREAK EVEN POINT(after change in sales mix)


Particulars ACE UTILITY LUXURY SUPREME TOTAL
Revised Sales Mix 25% 40 % 30% 5% 100%
Rs.RsRsRsRs
Sales Mix 1,50,000 2,40,000 1,80,000 30,000 6,00,000
Less: Variable Costs 90,000 1,63,200 1,44,000 12,000 4,09,200
Revised Contribution 60,000 76,800 36,000 18,000 1,90,800

New Composite P/V ratio =Rs 1,90,800/Rs 6,00,000×100 =31.8%


New Break -even Point (Sales value) =Rs 1,59,000/31.8% = Rs 5,00,000
b)
(a) Flexible Budget Summary
Boiler operating hours: 400 432 480 540
Steam raised(„000kg) 6667 7200 8000 9000
Costs
Fuel Rs. 16,000 Rs. 17,280 Rs. 19,200 Rs. 21,600
Chemicals(v) 800 864 960 1,058.4
Wages(F) 2400 2400 2400 2,400.0
Sundry overheads 2800 3000 3000 3,250.0
Total 22,000 23,544 25,560 28,308.4
Cost/‟000 kg 3.30 3.27 3.195 3.145
(b) Control statement for the month
Budget Actual Variances
Boiler operating hours 432 432
Steam raised („000kg) 7,200 6,750 450 (Adverse)
Costs: Rs Rs Rs.
Fuel(V) 17,280 18,000 720 (Adverse)
Chemical(V) 864 990 126 (Adverse)
Wages(F) 2,400 2,200 200 (Favourable)
Sundry overhead(F) 3,000 3,000 nil nil
YDT P.T.O.
(21)
_____ _____ ____
23,544 24,190 646 (Adverse)
c)
The possible ways for the treatment of defective work are as below:
i) Defectives that are considered inherent in the process and are identified as normal can be
recovered by using the following methods:
a. Charged to good products-The loss is absorbed by good units .This method is used when
seconds or first are normal:
b. Charged to general overheads-When the defectives caused in one department are reflected
only on further processing,the rework costs are charged to general overheads;
c.Charged to Department overheads-If the department responsible for defectives can be
identified then the rectification costs should be charged to that department.
d.Charged to Costing Profit and loss Account-If defectives are abnormal and are due to causes
beyond the control of organization, the rework cost should be charged to Costing Profit and
Loss Accounts.
ii) Where defectives are easily identifiable with specific jobs, the work costs are debited to the
job.
4.
a) A manufacturing company disclosed a net loss of Rs. 3,47,000 as per their Cost
Accounts for the year ended March 31,2016. The financial accounts however
disclosed a net loss of Rs. 5,10,000 for the same period. The following
information was revealed as a result of scrutiny of the figures of both the sets of
accounts.
(Rs.)
Factory overheads under-absorbed 40,000
Administration overheads over-absorbed 60,000
Depreciation charged in financial accounts 3,25,000
Depreciation charged in cost accounts 2,75,000
Interest on investments not included in cost accounts 96,000
Income-tax provided 54,000
Interest on loan funds in financial accounts 2,45,000
Transfer fees (credit in financial books) 24,000
Stores adjustment (credit in financial books) 14,000
Dividend received 32,000
Prepare a memorandum reconciliation account. 5

b) From the following particulars make out a monthly cost sheet or production
account of ABC manufacturing company showing cost per ton and percentage of
cost used in each item of output:
January 2015:
Tons Rate Per ton (Rs.) Amount (Rs.)
Coal used 5,000 12.50
Coke produced 3,500 25.00
Tar Produced 210 50.00
Sulphate of ammonia produced 49 150.00
Benzolproduced 48 65.00
Raw materials used 8,750

YDT P.T.O.
(22)
Wages paid 3,585
Repairs and renewals 2,815
General charges 4,050
5
c) The following information is related to the Himalayan Coffee House located at
City Centre, Kathmandu:
No. of coffee cups sold in January Average cost for making a cup of
2015 coffee
1,000 5.50
1,200 5.00
You have just been appointed as an accountant of the Himalayan Coffee House.
The Coffee House sold 1500 cups of coffee in February, 2015 at the average
selling price of Rs.5.00 per cup. One of the board member said that there is a loss
in February month. Do you agree? 5
Answer
a) Memorandum Reconciliation Account
Dr, Cr.
( Rs.) (Rs.)
To net loss as per Costing books 3,47,000 By Administration overheads over 60,000
recovered in cost accounts
To Factory overheads under absorbed 40,000 By Interest on investment not 96,000
in Cost Accounts included in cost accounts
To Depreciation under charged in Cost 50,000 By Transfer fees in financial books 24,000
Accounts
To income-Tax not provided in cost 54,000 By Store adjustment (Credit in 14,000
accounts financial books)
To Interest on loan funds in financial 2,45,000 By Dividend received in financial 32,000
accounts books
By Net loss as per financial books 5,10,000
7,36,000 7,36,000

b)

1. Calculation of Joint Cost:


Particulars Amount
Coal used 62,500
(5,000x12.5)
Raw Material used 8,750
Wages Paid 3,585
Repair & Renewals 2,815
General Charges 4,050
81,700

2. Statement showing Segregation of Joint Cost


(Sales Value Method)
Joint Product Selling Price/ton Qty (Ton) Sales Value Joint Cost
(a) (b) (a x b)
Coke 25 3,500 87,500 65,905.3
Tar 50 210 10,500 7,908.7
Sulphate of Amonia 150 49 7,350 5,536
Benzol 65 48 3,120 2,350
Total 1,08,470 81,700
YDT P.T.O.
(23)
3. Statement showing Cost Sheet:
Product Amt (a) Qty (b) Cost/ton(a/b) Percentage of Cost
Coke 65,905.3 3,500 18.63 80.67%
Tar 7,908.7 210 37.66 9.68%
Sulphate of Amonia 5,536 49 112.98 6.78%
Benzol 2,350 48 48.96 2.88%
81,700 100%
Note:
Alternative Physical Output Method can be used for segregation of joint cost.

c) Total Cost at 1,000 cups = No. of cups x Average Cost


= 1,000 x 5.50 = Rs. 5,500
Total Cost at 1,200 cups = No. of cups x Average Cost
= 1,200 x 5.00 = Rs. 6,000
Variable Cost per unit = Difference in Cost
Difference in Units
= 6,000-5,500/ 1,200-1,000
=500/200 = Rs. 2.50 per unit
Fixed Cost = Total Cost- Variable cost per unit x No. of Cups
= 6,000 - ( 2.50x 1,200) = 6,000-3,000 = Rs. 3,000
Total Cost at 1,500 coffee cups = Fixed Cost + Variable cost
= 3,000+(1500 x 2.5)
= Rs. 6,750
Sales Value of 1,500 coffee cups = 1500 x 5 = Rs 7,500
Profit on sales of 1500 cups of Coffee = 7,500- 6,750 = Rs. 750
Conclusion: I do not agree with the board member since there is a profit of Rs. 750.
5.
a) In an engineering company, the factory overheads are recovered on a fixed
percentage basis on direct wages and the administration overheads are absorbed
on a fixed percentage basis on factory cost.
The company has furnished the following data relating to two jobs undertaken by
it in a period:
Job 101 (Rs.) Job 102 (Rs.)
Direct materials 54,000 37,500
Direct wages 42,000 30,000
Selling price 1,66,650 1,28,250
Profit percentage on total cost 10% 20%
Required: 7
i) Computation of percentage recovery rates of factory overheads and
administrative overheads.
ii) Calculation of the amount of factory overheads, administrative overheads and
profit for each of the two jobs.

b) What role does a management accountant have in cost control and cost reduction? 4
c) Fixed costs are irrelevant for decision-making. What are the exceptions? 4
Answer
a)

YDT P.T.O.
(24)
i. Let factory overhead recovery rate, as percentage of direct wages be F and administrative
overheads recovery rate, as percentage of factory cost be A.
Factory Cost of Jobs:
Job 101 = Rs. 96,000 + Rs. 42,000F
Job 102 = Rs. 67,500 + Rs. 30,000F

Total Cost of Production of Jobs:


Job 101 = (Rs. 96,000 + Rs. 42,000F) + (Rs. 96,000 + Rs. 42,000F)A
= Rs. 1,51,500…..(i)
Job 102 = (Rs. 67,500 + Rs. 30,000F) + (Rs. 67,500 + Rs. 30,000 F) A
= Rs. 1,06,875…..(ii)
(refer to Working Note)
42,000 F + 96,000 A + 42,000 FA = 55,500…………………….. (iii) x 5
30,000 F + 67,500 A + 30,000 FA = 39375………………………(iv) x 7

On solving above relations


2,10,000 F + 4,80,000 A + 2,10,000 FA = 2,77,500
2,10,000 F + 4,72,500 A + 2,10,000 FA = 2,75,625
7,500 A = 1,875
A = 0.25
Putting the value of „A‟ in equation no. (iv) above to get the value of „F‟
30,000 F + 67,500 * 0.25 + 30,000 * 0.25 F = 39,375
37,500 F = 39,375 – 16,875
Or, F = 0.60
Hence percentage recovery rates of factory overheads and administrative overheads are
60% and 25% respectively.
Working Note:
Job 101 Job 102
Total cost of production Rs. 1,51,500 1,06,875
Selling price ( Rs. 1,66,650/110%) ( Rs. 1,28,250/120%)
(100% + Percentage of profit

ii. Statements of jobs, showing amount of factory overheads, administrative overheads and
profit
Job 101 (Rs.) Job 102 (Rs.)
Direct Materials 54,000 37,500
Direct Wages 42,000 30,000
Prime Cost 96,000 67,500
Factory Overheads (60% of Direct Wages*) 25,200 18,000
Factory Cost 1,21,200 85,500
Administrative Overheads (25% of Factory Cost*) 30,300 21,375
Total Cost 1,51,500 1,06,857
Profit (difference figure) 15,150 21,375
Selling Price 1,66,650 1,28,250
*As calculated in requirement (i) above.

b) Management Accountants role in cost control and cost reduction is perhaps central to his role
as amember of the management team. Indeed, for effective cost control, it may be necessary
to spend more onthe items which will reduce waste and scrap, improve quality, increase
productivity or conserve energy. Inany large organization the points at which costs are
incurred are usually numerous and relatively fewline managers have the mechanism of
collating and analyzing all the costs they incur, with a view toimplementing cost control
measures. The Management Accountant is uniquely placed in this respect andit usually falls
on him to play a catalytic role in getting the management team to work together to
YDT P.T.O.
(25)
achievespecific cost control objectives. It is also upto the Management Accountant to
channelize the cost control and cost reduction efforts intoareas which will give the greater
results. Without this direction, cost control and cost reduction can toooften degenerate into
symbolic actions like reusing envelopes or downgrading the class of air travel,which
generally have little impact on the overall cost structure but can substantially harm morale
andmotivation. It is important for the Management Accountant to guide the company‟s cost
control and costreduction programme into productive lines and not let it degenerate into a
morale damaging axing ofpetty expenditure.
c) Fixed costs are unrelated to output and are generally irrelevant for decision-making purpose.
However, inthe following circumstances, Fixed Costs become relevant for decision-making.

(i) When fixed costs are specifically incurred for any contract.
(ii) When fixed costs are incremental in nature.
(iii) When the fixed portion of semi-variable cost increases due to change in level of activity
consequentto acceptance of a contract.
(iv) When fixed costs are avoidable or discretionary.
(v) When fixed costs are such that one cost is incurred in lieu of another (the
difference in costs willbe relevant for decision-making).
6. Write short notes on: (4×2.5=10)
a) How flexible budget can help in management decision making?
b) Enumerate the essential pre-requisites of integrated accounting system.
c) Explain „Cost Centre‟ and „Cost Unit‟.
d) Discuss how you would treat the shortage in stock taking.
Answer
a) Flexible budget is a budget which, by recognizing the difference in behavior between fixed
and variablecosts in relation to fluctuations in output, turnover, or other variable factors, etc.
is designed to change inrelation to the level of activity actually attained.A flexible budget is
one that takes account of a range of possible volumes. It is sometimes referred to asa multi-
volume budget. The range of possible outputs may be known as the relevant range. Flexing
abudget takes place when the original budget is deliberately amended to take account
ofchange in activitylevels.Flexible budget enable an organization to predict its performance
and income levels at a given range ofsales levels and activity levels. It can be seen the impact
of changes in sales and production levels onrevenue, expenses and ultimatelyincome. It
enables more accurate assessment of managerial and organizational performance. So,
Flexible budget is an important aid to management to decision making.

b) The essential pre-requisites for integrated accounts include the following steps.
(i) The management's decision about the extent of integration of the two sets of books, some
concerns find it useful to integrate upto the stage of primary cost or factory cost, while
others prefer full integration of the entire accounting records.
(ii) A suitable coding system must be made available so as to serve the accounting purposes
of financial and cost accounts.
(iii)An agreed routine, with regard to the treatment of provision for accruals, prepaid
expenses, other adjustment necessary for preparation of interim accounts.
(iv) Perfect coordination should exist between the staff responsible for the financial and cost
aspects of the accounts and an efficient processing of accounting documents should be
ensured.

c) CIMA defines Cost Centre as “a production or service, function, activity or item of


equipment whose costs may be attributed to cost units. A cost centre is the smallest
organisational sub-unit for which separate cost allocation is attempted”. A cost centre is an
individual activity or group of similar activities for which costs are accumulated. For
example in production departments, a machine or group of machines within a department or
YDT P.T.O.
(26)
a work group is considered as cost centre. Any part of an enterprise to which costs can be
charged is called as „cost centre‟.
A cost centre can be:
(i) Geographical i.e. an area such as production department, stores, sales area.
(ii) An item of equipment e.g. a lathe, forklift, truck or delivery vehicle.
(iii) A person e.g. a sales person.

CIMA defines Cost Unit as “a quantitative unit of product or service in relation to which
costs are ascertained”. A „cost unit‟ is a unit of product or unit of service to which costs are
ascertained by means of allocation, apportionment and absorption. It is a unit of quantity of
product, service or time or a combination of these in relation to which costs are expressed or
ascertained. For example, specific job, contract, unit of product like fabrication job, road
construction contract, an automobile truck, a table, 1000 bricks etc. The cost units which pass
through the cost centre, the direct and indirect costs of the cost centre are charged to the units
of production by means of an absorption rate. The unit of output in relation to which cost
incurred by a cost centre is expressed is called „cost unit‟. Cost units can be developed for all
kinds of organizations, whether manufacturing, commercial or public utility services.
d) At the time of stock taking generally discrepancies are found between physical stock shown
in the bin card and store ledger. These discrepancies are in the form of shortages or losses.
The causes for these discrepancies may be classified as unavoidable or avoidable.
Losses arising from unavoidable causes should be taken care of by setting up a standard
percentage of loss based on the study of the past data. The issue price may be inflated to
cover the standard loss percentage. Alternatively, issues may be made at the purchase price
but the cost of the loss or shortage may be treated as overheads.
Actual losses should be compared with the standard and excess losses should be analyzed to
see whether they are due to normal or abnormal reasons. If they are attributable to normal
causes, an additional charge to overheads should be made on the basis of the value of
material consumed. If they arise from abnormal causes, they should be charged to the costing
profit and loss account.
Avoidable losses are generally treated as abnormal losses. These losses should be debited to
the costing profit and loss account. Losses and surpluses arising from errors in
documentation, posting etc. should be corrected through adjustment entries.

YDT P.T.O.
Business Communication
Suggested Answer
Roll No……………. Maximum Marks – 100
Total No. of Questions - 10 Total No. of Printed Pages -2
Time Allowed – 3 Hours
Marks
All questions are compulsory.

Section -'A'

1. Read the following case carefully and answer the questions given below: (4×5=20)
Dr. Aditya Sharma, a consultant business researcher primarily works in the
organizational management sector, and has conducted varieties of research including
case study, action research, business surveys, and so on about varieties of
organizational issues such as auditing, business administration, conflict management,
marketing, advertising, international relationships, etc. Dr. Sharma is asked by one of
the multinational companies based on Kathmandu, Nepal to conduct a comprehensive
study and submit the report about the business problems seen in the changing context
of Nepal. Nepal's Regional Office of the company has investigated from the
preliminary study that the company is bearing 12% loss every year for the recent
three years. The new executive director of the company has decided to conduct an
intensive study about the potential causes and problems of the company's loss, and
has appointed Dr. Sharma as the chief researcher who is expected to lead a task force.

Mr. Sharma has recently started the preliminary activities and procedures of the
research. The research is problem based, and immediate solutions are expected. The
TOR has mentioned that the report must include practice related and policy related
recommendations. The research team of Mr. Sharma has organized a meeting, and
conceptualized the design and procedures of the research. They have prepared the
time framework, and have planned about the tools and procedures for data collection.
Questions:
a) Which type of report is Mr. Sharma asked to prepare: analytical or investigative?
What are the major components of this type of report? Describe each in brief.
b) What would be the highly efficient tools for gathering information for this report?
Discuss any two of them in brief.
c) How can data be analyzed and interpreted?
d) Write statement of problem and objectives of Mr. Sharma's report.
Answer:
a) Since Mr. Sharma is asked to carry out a critical analysis of a particular business situation with
an aim at investigating the major problems and recommending the effective solutions, this
report is investigative in nature. Unlike informative reports, this type of report is more focused
to a specific problem of the organization. The major components of investigative report
include:
 Introduction
 Statement of problem
 Objectives
 Methodology (sampling, data collection tools and procedures, …)
 Analysis and interpretation of data
JMB P.T.O.
(28)
 Findings, Conclusion and Recommendations
 Bibliography

b) The major tools for data collection for the research report of Mr. Sharma may include:
 interview
 questionnaire
 observation
 focused group discussion
 interactions with clients and retailers
Among them, the highly effective tools for gathering information can be „interview‟ and
„focused group discussion‟ which are explained below in brief:
Interview: it is a face to face interaction of the researcher with the sampled subjects such as
clients, retailers, dealers, and other stakeholders. Interview can be of two types: structured and
unstructured. Structured interview involves systematic planning about the questions to be
asked to the subjects. For this report, the questions can be structured within the framework or
the contents such as popularity of the brand, marketing strategies, prices, consumer
satisfaction, delivery process, dealership model, and so on. The unstructured interviews are
open in nature. The questions are not set before the interview. Interaction is often context
dependent.
Focused group discussion: this technique is really worth talking for the particular case of the
given report. The representatives of the stakeholders such as staff, clients, retailers, dealers, etc.
can be put together in a meeting or a discussion session. Their opinions, reflections and
reactions can be recorded as the data for detecting the problems.

c) The data collected by different tools can be analyzed by using different strategies and
procedures. The qualitative information is gathered and analyzed by categorizing it into
different themes, titles and options. The common tendencies of the data are elaborated, and
verified with the previously defined theories and facts. The quantitative data are analysed in
terms of different statistical tools such as mean, standard deviation, correlation coefficient, and
so on. The data are presented in the tables, charts, graphs, etc. The analysis is done in line with
the objectives set for the research. Report is produced on the basis of the findings attained by
analysis and interpretation of the data.

d) Statement of problem : The present report is prepared focusing basically on the problems that
are implicitly and explicitly associated with the business strategies adopted by the regional
office of Anuradha International Company (Kathmandu Branch). The research emerges out of
the basic problem related to the decreasing financial growth of the company. The causes of
consistent loss seen in the recent annual reports are the primary issues that this report deals
with. The latest audit report shows that the company is bearing gross loss of 12%. The research
question is: what is the factor that affects the financial status of the company?
Objectives
The report has the following specific objectives:
 to study and analyze managerial efficiency of the company
 to find out the marketing strategies of the company
 to analyze consumer responses of the products
 to explore the problems related to management, marketing, sales and public dealing
 to recommend potential solutions of the identified problems.

JMB
(29)

2. Prepare a persuasive cover letter in response to the advertisement given below: 10


Assistant Product Manager: We are currently seeking an assistant product manager
for our fast-paced organization. S/he must possess as sound knowledge of marketing
related activities such as making a marketing plan; executing, monitoring, &
evaluating it. S/he should be very good at computer use and must have very good
interpersonal communication skills. Please send your application to Ms. Courtney L.
Donahue, director of Del Rio Enterprises.

Answer:
Satdobato, Lalitpur
May 23, 2016

Ms. Courtney L. Donahue


Director, Human Resources
Del Rio Enterprises
Denver, CO 82511

Dear Ms. Donahue:

Your advertisement for an assistant product manager, appearing May 22 in Section C of the Denver
Post, immediately caught my attention because my education and training closely parallel your
needs. According to your advertisement, the job includes “assisting in the coordination of a wide
range of marketing programs as well as analyzing sales results and tracking marketing budgets.” A
recent internship at Ventana Corporation introduced me to similar tasks. Assisting the marketing
manager enabled me to analyze the promotion, budget, and overall sales success of two products
Ventana was evaluating. My ten-page report examined the nature of the current market, the
products‟ life cycles, and their sales/profit return. In addition to this research, I helped formulate a
product merchandising plan and answered consumers‟ questions at a local trade show.

Intensive course work in marketing and management, as well as proficiency in computer


spreadsheets and databases, has given me the kind of marketing and computer training that Del Rio
probably demands in a product manager. Moreover, my recent retail sales experience and
participation in campus organizations have helped me develop the kind of customer service and
interpersonal skills necessary for an effective product manager.

After you have examined the enclosed résumé for details of my qualifications, I would be happy to
answer questions. Please call me at 9851133333 to arrange an interview at your convenience so that
we may discuss how my marketing experience, computer training, and interpersonal skills could
contribute to Del Rio Enterprises.

Sincerely,

Kendra A. Hawkins

3.
a) Effective listening is one of the important determinants for the successful business
interactions. Listening becomes more complex in many contexts because of many
JMB
(30)
different barriers to it. Explain in brief how barriers to effective listening can be
overcome, and success in business interactions can be enhanced. 5
b) State the contents of a persuasive resume, and write the resume head and
objective statement of your resume. 5
Answer:
a) Listening is a very common receptive skill which often becomes far more complex than most
people think. Complexity in effective and successful listening is mostly caused by many
different factors such as content, channel, etc. of communication. Listening becomes complex
because of various barriers to it, too. Good listeners always look for ways to overcome potential
barriers throughout the listening process.
Barriers to effective listening may vary from physical reception to listener's mental conditions.
The physical barriers such as room acoustics, phone rings and other noises, background music
etc. can be serious when the listener has to extract particular information or content from
listening. Similarly, lack of attention, interruption, schema, prejudgment etc. can be the examples
of mental barriers to effective listening. Some of the strategies for overcoming the barriers to
effective listening can be:
 try to minimize the sources of physical barriers such as noises and poor acoustics
 avoid selective listening and focus on the speaker
 keep an open mind and avoid prejudgment
 try to interpret the message from the context of the speaker
 do not interrupt the speaker
 write, record or try to capture the information
 try to associate, categorize and visualize the information
 pay proper attention

b) The contents of a persuasive resume:


 Resume head (Name, address,..)
 Objective Statement
 Academic details
 Professional details
 Experience record
 Special skills & involvement
 Referees
 Signature & date

Suman Bista
Kathmandu 13, Kalanki
9841xxxxxx
discover.xxxxxx@gmail.com

Objective statement:
To serve the nation and the organization where I work with consistent dedication and hard
work, and to enhance the business environment of my organization, focusing on business
ethics and standard.

4. Briefly explain any FOUR of the following: (4×2.5=10)


a) Disadvantages of grapevine in an organization
JMB
(31)
b) Ethics in business communication
c) Parts of a formal report
d) Group dynamics
e) Follow up letter

Answer:
a) A grapevine is more like a rumor. It doesn't come straight from the horse‟s mouth and is usually not
fact. These rumors can cause distrust or mutiny within an organization. It is important for managers
of a marketing team to address these issues to all personnel before the rumors get out of hand.
Monthly mandatory meetings with employees can help quench this bomb fuse. It is important to
create constant communication between the hierarchy reinforced with internal newsletters to keep
information factual.

b) Any company that aims to be socially and ethically responsible must make a priority of ethical
communication both inside the company and in its interactions with the public. In theory, many
consumers prefer to do business with companies they believe are ethical which gives those ethical
businesses an advantage in the market. Ethical issues of business communication are one such issue.
Some of the vital characteristics of ethical communication are discussed below.

While communicating to the audience, conveying the desired message to them in a significant
manner is of primary importance. For instance, the employees in a company can be asked to
increase their efficiency in a demanding manner whereas managers and executives will feel
offended if the same tone is used on them.
Maintaining the same wavelength with the audience is very important for a communicator to ensure
the audiences feel at home. Experienced communicators immediately build a relationship based on
trust with the audience as soon as they start speaking.

c) These are three major parts of a formal report:


1. Prefatory parts
• Title fly
• Title page
• Letter of authorization
• Letter of transmittal/preface/ foreword
• Table of content
• Executive summary
2. Report Proper
• Introduction
• Report findings
• Summary, recommendation
3. Appended parts
• Bibliography
• Appendix
• Index

d) Strength of team work is always appreciated in the sector of business and organizational
management. The role of each member in the team or group is important in order to accomplish

JMB
(32)
group tasks and get success in business. To accomplish the business goals successfully, team
members constantly contact with one another. The interactions and processes that take place
between the members of a team are called group dynamics. Some teams are more effective than
others simply because the dynamics of the group facilitate member input and the resolution of
differences. Group dynamics are affected by several factors: the role that team members assume,
the current phase of team development, the team's success in resolving conflict, and its success in
overcoming resistance. Group dynamics promote understanding and cooperation among the
members.

e) Follow-up letters are different than thank you letters. The purpose of thank you letters is to thank
someone for meeting with you and to reaffirm your interest in the position or industry at hand.
Thank you letters are, in turn, sent immediately. A follow-up letter is sent several weeks after
sending a thank you letter in order to touch base once again with your contact and to reintroduce
yourself. A follow-up letter provides you with the opportunity to restate your interest and to
check for new job leads or information. Another type of follow-up letter may be sent after
submitting your resume and cover letter. If after several weeks, you have not heard from the
employer, you should send a follow-up letter to reemphasize your interest in the company and
the position. Follow-up letters can be sent by post or email. You should make your letter as
formal or as informal as your relationship with the contact dictates.

Follow- up letter shows:


 That you appreciated your initial meeting
 Where you stand in your current job search
 Anything relevant that has occurred between the initial meeting and now
 What type of help you now need

JMB
Marketing
Suggested Answer
Roll No……………. Maximum Marks – 100
Total No. of Questions - 4 Total No. of Printed Pages -2
Time Allowed – 3 Hours
Marks
All questions are compulsory.

Section -'B'

5. Read the following case carefully and answer the questions given below:
Kanchhi Balami and her husband migrated to Kathmandu from Nuwakot few years
ago. Her husband Kumar Balami works as a driver in a private firm. She is 25 years
old and has one son Krishna and one daughter Bimala. Both the children are enrolled
in a government school.
Kanchhi has been working as a cook in a private house while working as a domestic
service worker in a private house; so she has adequate knowledge in cooking. During
the period, she had obtained from her house-owner the „pastry making‟ skill, where
she works. So pastries baked by her are delicious. Once she expressed her interest
with her husband, instead of working at a private house, to open a pastry shop near by
the SANN and CAB College to put her talent in practice, because sometime she felt
lonely and bored staying home alone, especially at day time, and at the same time,
their income was not sufficient to meet their children‟s expenses. Her husband
understood her problem and interest, so he gave her wife permission to do according
to her interest and also assured to support her in this effort. So she was impressed by
the support of her husband and decided to open a small pastry shop near by these
colleges. She rented a small shop near by the College, hoping that a large number of
students may come to the shop for varieties of pastry, coffee and tea. She bought the
necessary equipments for kitchen and shop with a cash outlay of Rs.50, 000. She
managed for furniture and furnishings. Now, Kanchhi decided to name her shop as
“Bhatbhateni Bakery Shop". Kanchhi maintained neat and clean in the shop to satisfy
the customers. Beside it, her treatment with the customers was also impressive.
The business has been operating for the last ten months. Sales are slowly increasing.
The major customers are college students, who drop in for a pastry and coffee.
Occasionally, professors come by with their friends for pastry and cup of tea.
Shopkeepers and offices in the nearby area are also her regular clients who send
orders for shop delivery. With the passes of time, a large number of travellers also
come to the shop for tea, coffee and pastries. Local households also come to purchase
pastries over the counter. Customers are satisfied with her services and treatment. But
many customers frequently asked for Momo and Chow-Min dishes. During the
vacation period, Kanchhi has noticed considerable drop in the sales. Though she has
knowledge about cooking Momo and Chow-Min, she needs to make additional
investment of about Rs. 50, 000 for managing Momo and Chow-Min in the shop, for
which she has to take loan from financial institution or somebody else. This money is
necessary not only to purchase necessary equipments and furniture but also needs to
hire more space in the rented house. She is confident that, if additional dishes could
be managed in the shop, the loan would be repaid within six months and then
afterwards she would be able to earn handsome profit from the business. She
discussed with her husband on this issue. Her husband was impressed by her proposal

JMB P.T.O.
(34)
and permitted to do accordingly. However, she wants to change neither the existing
business name nor the location.
Actually, Kanchhi Balami hesitates to keep additional items like Momo and Chow-
Min dishes, because of limited managerial capacity and limited financial capacity.
She was satisfied with the existing business flow and profit margin. But if the
demanded items could be added, she is confident that she will be able to earn more
money. On the other, if she avoids keeping the demanded items and insists only in the
current items, it becomes difficult for her to expand volume and value of business in
future.
Kanchhi Balami is now in a big problem because she does not know about the
modern marketing concept and marketing techniques; so she is searching for persons,
who can provide her an expert advice in this connection. You as a marketing student,
she puts the following questions and issues on you for providing her reasonable
solutions, which can promote her business in the days to come.
Questions:
a) Why did Kanchi Balami decide to open a pastry shop near by the SANN and
CAB colleges? 5
b) State the target market and customers of Kanchi Balami. 5
c) What factors compelled Kanchi Balami to think for adding Momo and Chowmin
dishes? 5
d) Explain the reasons, whether Kanchi Balami should add these two dishes or not. 5

Answer1(a)
The main intention of Kanchhi Balami to open the pastry shop was to help her husband
financially, because the income of single person was not adequate to feed and educated her
children properly.
However, the main objective of opening the pastry shop near by the SAAN and CAB colleges
was that the colleges have many students who may come to her pastry shop and drink tea &
coffee and eat pastry items.
Answer1(b)
She decided to open her pastry shop in that area thinking that Kanchhi Balami may have a large
target market and can earn adequate income from students. Her customers include college
professors, travelers, nearby shop-keepers, local house-owners, and students as well.

Answer1(c)
When the classes were off during the vacation period, Kanchhi Balami noticed considerable drop
in sales. Similarly, many customers demanded for momo and chowmin items. Due these
situations, Kanchhi Balami was compelled to think for keeping the most demanded additional
dishes in her shop.

Answer1(d)
Since she does not have good knowledge about marketing, she decided to take the suggestions of
marketing students. If she really wants to be a marketer, she should add the two dishes as
demanded by most of the customers; because marketing is a customer-oriented philosophy of
business, so she as a marketer should go ahead as per the demand of customers. The
sustainability of her business is dependent on how she perceives the customers‟ sentiment and
work accordingly.

JMB
(35)
6. What is marketing? Explain the selling concept and new marketing concept. (4+6=10)
Answer2
Most people think that marketing is only about the advertising and selling of goods and services.
Advertising and selling, however, are just two of the many marketing activities. In other words,
selling and advertising are only the tip of the marketing iceberg. Although they are important,
they are only two of many marketing functions and are often not the most important ones.
Today, marketing must be understood not in the old sense of making a sale- “telling and
selling”- but in the new sense of satisfying customer needs. If a marketer does good job of
understanding customer needs; develops products that provide superior value; and prices,
distributes and promotes them effectively, these products will sell very easily. Thus selling and
advertising are only part of a marketing mix.
In general, marketing activities are all those associated with identifying the particular
wants and needs of a target market of customers, and then going about satisfying those customers
better than the competitors. This involves doing marketing research on customers, analyzing
their needs and then making strategic decisions about product design, pricing, promotion, and
distribution. Thus, marketing deals with identifying and meeting human and social needs.
According to Philip Kotler & Kevin Lane Keller, “Marketing is a societal process by
which individuals and groups obtain what they need and want through creating, offering, and
freely exchanging products and services of value with others.”
According to American Marketing Association, “Marketing is the process of
planning and executing the conception, pricing, promotion and distribution of ideas, goods and
services to create exchanges that satisfy individual and organizational objectives.
According to Stanton, Etzel & Walker, “Marketing is a total system of business activities
designed to plan, price promote and distribute want satisfying products to target markets in order
to achieve organizational objectives.”
From the above definitions what we can say is customers‟ wants must be recognized and
satisfied. Entire system of business activities should be customer oriented. Marketing should start
with an idea about a want-satisfying product and should not end until the customers‟ wants are
completely satisfied, which may be some time after the exchange is made.
Thus, in conclusion, marketing is meeting needs profitably.
The selling Concept:
Selling concept holds that consumers and businesses, if left alone, will ordinarily not buy enough
of the organization‟s products unless it undertakes a large-scale selling and promotion effort.
Therefore, the organization must undertake an aggressive selling and promotion effort. The
selling concept is practiced most aggressively with unsought products, products that buyers
normally do not think buying, such as insurance and encyclopedias. Most firms practice the
selling concept when they have overcapacity. Their aim is to sell what they make rather than
make what the market wants. Such marketing carries high risks. It focuses on creating sales
transactions rather than on building profitable relationships with customers. It assumes that
consumers who are coaxed into buying the product will like it. Or if they don‟t like it, they will
possibly forget their disappointment buy it again later. These are usually poor assumptions to
make about buyers. Most studies show that dissatisfied customers do not buy it again. Worse yet,
whereas the average satisfied customer tells three others about good experiences, the average
dissatisfied customer tells ten others about his or her bad experiences.

JMB
(36)
The New Marketing Concept
The new marketing concept holds that achieving organizational goals depends on determining
the needs and wants of target markets and delivering the desired satisfactions more effectively
and efficiency than competitors do. The marketing concept starts with a well-defined market,
focuses on customer needs, coordinates all marketing activities affecting customers, and makes
profit by creating long term customer relationship based on customer value and satisfaction.
Thus, under marketing concept, customer focus and the value are the paths to sales and profits.
This concept is based on “we make what we can sell.”

7.

a) Explain the meaning and objectives of promotion. (2+3=5)


b) What is a new product for a marketing firm? Explain briefly the reasons for
product failure in the market. (2+3=5)

Answer3(a)
Promotion is one of the major element of marketing mix which provides various valuable
information to consumers about product, its price, availability, utilities and benefits. Promotion
consists of various activities that facilitates exchanges with target customers through persuasive
communication which stimulates the demand of the product.
According to Philip Kotler, “Promotion includes all the activities the company undertakes to
communicate and promote its products to the target market.”
Promotion includes various types of activities such as advertising, sales promotion, personal
selling, publicity and public relation to inform, persuade, remind and reinforce the target market
about the company‟s offerings. These activities influence the customer‟s feelings, belief or
behavior for buying.
The major objectives of promotion are informing, persuading, reminding and reinforcing or
reassuring the customer about the product.
1. Informing:
The main objective of promotion is to inform the market about product, price, availability,
utilities and benefits. It helps to develop awareness about the product It also provides alternatives
to the customers for purchase decision.
2. Persuading:
Promotion persuades customers to make the purchase decision in favor of the promoted brand.
Promotion is persuasive communication. It stimulates product demand through appealing ads,
incentives and benefits. Promotion influence buyer behavior. Promotion also persuades
middlemen to carry such product.
3. Reminding:
Customer normally have very short-lived memory. It is impossible to customers to remember all
advertised brand. So, marketer reminds customers about the product by using various
promotional tools. The marketer assumes that customer may forget unless they are constantly
reminded.
4. Reinforcing:
Promotion is equally important in post purchase stage of the buying process. Consumer may feel
anxiety after their purchase decision. Marketer often provides reinforcing message to increase
customer‟s satisfaction level. Repeated reinforcement also leads to brand loyalty. By the
JMB
(37)
reassurance to customer, marketer tries to reduce cognitive dissonance and build brand and
corporate image.

Answer3(b)
A product is anything, tangible or intangible, that can satisfy the expected need of the target
customers. People generally think that a new product should be completely a new and innovative
product which is new for the customers. But for a marketer, a product may be the following
products –
i. Imitating product – It is a new product copied by an organization from an original firm;
ii. Improved product – It is an existing product of the firm which is modified with a view to
increase sale of expand market;
iii. Innovative product – It is completely a new product, which is developed through the
research and development activities;

Each year, certain products become failure and are dissuaded from the market. The main reasons
for product failure in the market may include –
i. Lack of differentiation with the existing competitive product;
ii. Due to poor positioning strategy of the marketer;
iii. Poor value of the product compared to their prices;
iv. Due to small size of the market or fragmented markets;
v. Due to the increased rate of product imitation in the market by the competitors;
vi. Due to ineffective and poor distribution system;
vii. Due to high level of unhealthy competition In the market;
viii. Due to lack of adaptation ability of the new product to the changed needs of the
markets, etc.
8. Briefly explain the following: (5×2=10)
a) Market entry pricing strategy
b) Niche marketing
c) Product line
d) Importance of understanding consumer behavior
e) Marketing mix

Answer4 (a)
(a) Market-entry pricing decision is a pricing strategy that a marketer follows while
entering to the market for the first time. Under this circumstance, there may be two
alternative pricing strategies – market penetration pricing and market skimming
pricing.
i. Market penetration pricing strategy: Market penetration pricing involves setting
low price for the new product in the initial period with a view to penetrate the
mass market immediately and thus obtain a large sales volume and larger market
share. After the market for the product is established, price will gradually be
increased.
ii. Market-skimming pricing strategy: Market-skimming pricing involves setting
high price for a new product in the initial stage assuming that the customers will
pay high price for the new product. But with an increase in competition, the initial
price will be lowered to meet competition and the changed demand.
JMB
(38)
Answer 4(b)
A niche is more narrowly defined group seeking a distinctive mix of benefits. Marketers usually
identified niches by dividing a segment into sub-segments. For example, the segment of heavy
smokers includes two niches: those who are trying to stop smoking and those who don‟t care.
In an attractive niche, customers have a distinct set of needs; they will pay a premium to the firm
that best satisfies their needs: the niche is not likely to attract other competitors; the nicher gains
certain economies through specialization and the niche has size, profit and growth potential.
Answer4 (c)
Product line is a group of closely related products. They perform similar functions, are sold to
the same customer groups, are marketed through the same channels and fall within given price
ranges. Product line strategy includes product line length, product line modernization and line
featuring strategies. For example, if any bakery shop offers verities of bakery product that is
called product line strategy.

Answer4 (d)
Buyer behavior refers to the activities and actions expressed by the buyer or potential buyer
while selecting, buying and using the product. Understanding buyer behavior is a difficult task
for the marketer because it involves, to a great extent, the psychological phenomenon. which is
difficult to understand. Though it is difficult to understand, the marketers need to understand the
buyer behavior properly to formulate a suitable marketing program. The reasons behind the need
of understanding buyer behavior are –
i. Different persons and communities have different needs because they live in different
environment.
ii. Their needs go on changing and may demand different goods and services in different
situations;
iii. different persons may demand different goods and services at the same time;
iv. Some needs are fulfilled while some are not.
v. Degree of need fulfillment, by and large, depends upon the marketing environment
that exists in the market;

Answer 4(e)
The term marketing mix was coined by Professor Neil Borden. Marketing mix is the set of
marketing tools that the firm uses to pursue its marketing objectives in the target market.
Marketing mix is a blend of product, price, place and promotion tools. Thus, it is popularly
known as four Ps‟ of marketing which was popularized by Professor McCarthy. Nowadays, four
P‟s has expended to seven Ps‟ by adding people process and physical evidence especially for
service marketing.

JMB
JMB P.T.O.
CHARTERED ACCOUNTANCY PROFESSIONAL CAP-II

SUGGESTED ANSWER
June 2017

The Institute of Chartered Accountants of Nepal

TRP P.T.O.
(2)

Advanced Accounting
(3)

Suggested
Roll No……………. Maximum Marks - 100
Total No. of Questions - 6 Total No. of Printed Pages - 14
Time Allowed - 3 Hours
Marks
Attempt all questions. Working notes should form part of the answer.
1. The following are the ledger balances of Sri Babubhaj for the year ended 31st March
2015.
(Rs.)
Capital Account 2,00,000
Drawing Account 15,250
Sundry Creditors 56,562
Loans taken on Mortgage 20,000
Sundry Debtors 17,800
Goodwill 10,000
Interest on Mortgage Loan 400
Cash in Hand 364
Bad Debts Provision (Cr.) 450
Plant & Machinery 22,500
Sales less Returns 1,50,210
Cash at Bank 6,756
Electric Power 3,250
Discount Paid 1,216
Dividends Received 3,250
Bills Payables 11,575
Investment reserve as at 1st April, 2014(Cr.) 1,250
Purchase less Returns 1,65,265
Salaries 15,200
Rent and Rates 3,756
Electricity Charges 2,120
Insurances 1,000
Advertisement 17,256
Stock on 1st April, 2014
Raw Materials 51,265
Finished Goods 12,625
Factory Land & Buildings 21,200
Bad Debts 450
Carriage Inwards 5,210
Wages 18,560
Investments at Market Price as on 1st April, 2014 36,150
Bills Receivables 16,263
Outstanding Expenses 2,100
General Expenses 4,065
Prepaid Insurances 200
Discount 2715
a) Factory buildings were constructed on land purchased for Rs. 10,000 during the
year but this was wrongly posted to purchase account.
b) Provide depreciation at 10% on plant and 5% on buildings.
c) Interest has to be provided on mortgage loan at 6% per annum keeping in view
the fact that a sum of Rs. 5,000 was repaid on 30th June, 2014.
(4)
d) The market value of the investments as at 31st march, 2015 was Rs. 37,000. The
investment reserve account represents the difference between the cost and market
value.
e) Electricity charges were paid in advance to the extent of Rs. 155.
f) Some investments were sold during the year for Rs. 1,260 realising a profit of Rs.
125 when compared to their market value on 1st April, 2014. The sale proceeds
were, however, credited to the sales account.
g) Provision for bad debts should be maintained at 10%.
h) Out of the bills receivables, one bill for Rs. 1,200 matured for the payment in the
last week of March, 2015. However, the bankers informed that they could not
collect the said bill on 31st March, 2015. The information was not recorded in the
books.
i) The closing stock was as follows:
Raw materials Rs. 85,263
Finished goods Rs. 64,987
You are required to prepare the Manufacturing, Trading, and Profit and Loss
account and the Balance Sheet. 20
Answer

SHRI BABUBHAJ
Manufacturing, Trading, & Profit and Loss Account
For the Year Ending 31st March, 2015
Rs. Rs. Rs.
To Material Consumed: By Trading A/C( Cost of
Goods produced during the
Year 1,51,597

Opening Stock 51,265


Purchase less return 1,55,265
2,06,530
Less; Closing Stock 85,263 1,21,267
To Wages 18,560
To Carriage Inwards 5,210
To Electric Power 3,250
To Depreciation on
Plant & Machinery 2,250
Factory Building 1,060 3,310
1,51,597 1,51,597

To Opening Stock By Sales 1,48,950


Finished Goods 12,625 By Closing Stock of
Finished goods 64,987
To Cost of Goods Produced 1,51,597
To Gross Profit c/d 49,715
2,13,937 2,13,937

To Salaries 15,200 By Gross profit b/d 49,715


To General Expenses 4,065 By Discount 2,715
To Rent and Rates 3,756 By Dividend Received 3,250
To Electricity 1,965 By Profit on Sale of
Investment 125
(5)
To Advertising 17,256
To Insurances 1,000
To Interest on Mortgage Loan 1,275
To Provision for bad Debts 1900
Add: Bad Debts 450
2,350
Less: Existing Provision 450 1,900
To Discount Allowed 1,216
To Net Profit transferred to Capital A/C 8,172
55,805 55,805

Balance Sheet of Shri Babubhaj as on 31st March, 2015

Capital & Liabilities Assets

Rs. Rs. Rs. Rs.

Capital 2,00,000 Fixed Assets


Add: Profit 8,172 Goodwill 10,000
2,08,172 Factory Land &
Less: Drawing 15,250 1,92,922 Building 21,200
Addition during
The year 10000
Investment Reserve 3,235 31,200
Loan on Mortgage 20,000 Less: Depreciation1,060 30,140
Add: Interest accrued 875 20,875
Sundry Creditors 56,562 Plant & Machinery 22,500
Bills Payables 11,575 Less: Depreciation 2,250 20,250
Expenses Outstanding 2,100 Investment at Market Value 37,000
Suspense Account 9 Current Assets
Stock in Trade(assumed at cost)
Raw Materials 85,263
Finished Goods 64,987 150,250
Sundry Debtors 19,000
Less: Provision for
Bad debts 1,900 17,100
Bills Receivables 15,063
Cash In Hand 364
Cash at Bank 6,756
Prepaid Expenses 355
2,87,278 2,87,278

Notes:

(i) Rs. 10,000 has been deducted from purchases and added to Land & Buildings.
(ii) Rs. 1200 has been deducted from Bills Receivables and added to the sundry debtors.
(iii) Rs. 1260 has been deducted from Sales, being sale proceeds of investments the cost Rs. 1135
has been from investments.
(iv) The cost of investments is Rs. 35,015 i.e. Rs. 36,150 less Rs. 1135. Since these have to be
shown at Rs. 37,000, Rs. 1985(i.e. Rs. 37000-Rs. 35015) has been added to investment Reserve
which now becomes Rs. 3235. Strictly, speaking the cost, of investments sold being less than
the market value, the profit should be more than Rs. 125.
However, the cost of each investment would be different hence this point has been ignored.

(v) Interest on Mortgage Loan:


(6)
Interest on Rs. 25,000 @ 6% for 3 months 375
Interest on Rs. 20,000@6% for 9 months 900
1275
Less: Paid 400
Outstanding 875
2.
a) The summarized Balance Sheet of Krishna Ltd. As on 31st Ashad, 2073 was as
follows:
Amount Amount
Liabilities Assets (Rs.)
(Rs.)
Equity Shares of Rs. 10 fully 30,00,000 Goodwill 5,00,000
paid Profit Reserves
Export 8,50,000 Tangible Fixed Assets 30,00,000
General Reserves 50,000 Stock 10,40,000
Profit and Loss Account 5,50,000 Debtors 1,80,000
9% Debentures 5,00,000 Cash & Bank 2,80,000
Trade Creditors 1,00,000 Preliminary Expenses 50,000
50,50,000 50,50,000
st
Radha Ltd. agreed to absorb the business of Krishna Ltd. with effect from 1
Shrawan, 2073.

i) The purchase consideration settled by Radha Ltd. as agreed:


(i) 4,50,000 equity shares of Rs. 10 each issued by Radha Ltd. by valuing
its share @ Rs. 15 per share.
(ii) Cash payment equivalent to Rs. 2.50 for every share in Krishna Ltd.
ii) The issue of such an amount of fully paid 8% debentures in Radha Ltd. at
96% as is sufficient to discharge 9% debentures in Krishna Ltd. at a
premium of 20%.
iii) Radha Ltd. will take over the tangible fixed assets at 100% more than the
book value, stock at Rs. 7,10,000 and debtors at their face value subject to a
provision of 5% for doubtful debts.
iv) The actual cost of liquidation of Krishna Ltd. was Rs. 75,000. Liquidation
cost of Krishna Ltd. is to be reimbursed by Radha Ltd. to the extent of Rs.
50,000.
v) Statutory reserves are to be maintained for 1 more year.
You are required to: 10
(i) Close the books of Krishna Ltd. by preparing realisation account, Radha
Ltd. Account, shareholders account and debenture account.
(ii) Pass journal entries in the books of Radha Ltd. regarding acquisition of
business
b) Fantastic Home Ltd. commenced business on 1st January, 2016. The business is to
sell Radio Recorders and Televisions both in hire -purchase and cash basis. The
information is given below:-
Radio Recorder Television Set
Cost Price (Rs.) 6,000 18,000
Sale Price (Cash) (Rs.) 7,800 19,500
Down payment for H.P. (Rs.) 600 3,000
Monthly instalments for H. P. (Rs.) 360 660
No. of instalments 20 25
(7)
The company purchased goods costing Rs. 24,00,000 in all and made cash sales
totaling Rs. 15,00,000. Stock on hand was valued at Rs. 3,90,000. Hire Purchase
transactions were as follows:

Number Instalments Instalments-due


Sold Collected (customer’s Paying)
Radio Recorders 30 300 15
Television Sets 25 250 10
4 Radio Recorders and 3 Television sets on which only 10 monthly instalments
were collected were repossessed and were valued at Rs. 45,000. This is not
included in the stock mentioned above.
Prepare hire purchase trading account to show the profit or loss made by the hire
vendor company. 10
Answer
a)
(i) Purchase consideration computation Rs.
Cash payment for(3,00,000xRs.2.5) 7,50,000
Equity Shares(4,50,000xRs.15) 67,50,000
75,00,000
In the books of Krishna Ltd.
Realisation Account
Rs. Rs.
To Goodwill 5,00,000 By 9% Debentures 5,00,000
To Tangible Fixed 30,00,000 By Creditors 1,00,000
To Assets
Stock 10,40,000 By By Radha Ltd. 75,00,000
To Debtors 1,80,000 (Purchase
To Cash & Bank A/c 2,55,000 consideration)
(2,80,000-25,000)
To Cash & Bank A/c 25,000
(Realization
To expenses)
Profit on realization
Transfer to 31,00,000
shareholders
81,00,000 81,00,000
Equity Shareholders A/c
Rs. Rs.
To Preliminary expenses 50,000 By Equity Share Capital 30,00,000
To Equity Shares in Radha Ltd. 67,50,000 By Export Profit 8,50,000
To Cash & Bank A/c Reserves
7,50,000 By General Reserves 50,000
By P&LA/c 5,50,000
By Realization A/c 31,00,000
75,50,000 75,50,000
9%DebenturesAccount
Rs. Rs.
To Realization A/c 5,00,000 B Balance b/d 5,00,000
y
Radha Ltd.
Rs. Rs.
To Realization A/c 75,00,000 By Share Capital 67,50,000
7,50,000
(8)
75,00,000 By Bank A/c 75,00,000

(ii) Journal Entries in the books of Radha Ltd.


Rs. Rs.
1 Business Purchase A/c Dr. 75,00,000
To Liquidator of Krishna Ltd 75,00,000
(Being business of Krishna Ltd. taken over)
2 Tangible Fixed Assets Dr. 60,00,000
Stock Dr. 7,10,000
Debtors Dr. 1,80,000
Cash & Bank A/c Dr. 2,55,000
Goodwill A/c (Bal.fig.) Dr. 10,64,000
To Provision for doubtful debts 9,000
To Liability for 9% Debentures 6,00,000
To Creditors 1,00,000
To Business Purchase account (Being assets 75,00,000
and liabilities taken over)
3 Amalgamation Adjustment A/c Dr. 8,50,000
To Export Profit Reserves 8,50,000
(Being statutory Reserves taken over)

4 Goodwill Dr. 50,000


To Bank A/c 50,000
(Liquidation expenses reimbursed))
5 Dr. 75,00,000
Liquidator of Krishna Ltd.
45,00,000
To Equity Share Capital
22,50,000
To Securities Premium
7,50,000
To Bank A/c
(Being purchase consideration discharged)
6 Liability for 9% Debentures ( 5,00,000x 120/100) Dr. 6,00,000
Discount on issue of debentures 25,000
To 8% Debentures (6,00,000x100/96) 6,25,000
(Being liability of debenture holders’ discharged)

b)
Hire Purchase Trading Account
Rs. Rs.

To Goods Sold on H.P. (1) 7,21,500 By Cash 3,66,000


By Goods repossessed 45,000
To Stock reserve on instalments By Instalments due 12,000
not due (4) 36,600 By Goods sold an H.P. Loading 91,500

By Instalments not yet


Profit and Loss Account (Profit) 55,800 due-stock with customers c/d (3) 29,9,400
813,900 813,900
(9)
Working Notes:
1. Cost Price:
Radio Recorders (30 × 6,000) 1,80,000 30 × 7,800 2,34,000
Television sets (25 × 18000) 4,50,000 25 × 19,500 4,87,500
Total 6,30,000 72,1500
Loading (Margin) = 7,21,500 – 6,30,000 = Rs. 91,500

2. Cash Collected: Radio Reorders Television sets


Rs. Rs
Down Payment 30 × 600 = 18,000 25 × 3000 = 75,000
Instalments 300 × 360 = 1,08,000 250 × 660 = 1,65,000
1,26,000 = 2,40,000
Total 1,26,000 + 240,000 = 3,66,000

3. Instalments not due:

Radio Recorders:
Total Instalments on 26 sets = 26 × 20 520
Less: Instalments collected and due (315 – 40) 275
Not yet due 245
Amount 245 × 360 = Rs. 88,200

Television Sets
Total Instalments on 22 sets = 22 × 25 550
Less: Collected and due (260-30) 230
Not yet due 320
Amount 320 × 660 = Rs. 2,11,200

Total Installments not yet due = Rs. 88,200 + Rs. 2,11,200 = Rs. 2,99,400

4. Stock Reserve
Radio Recorders Television Sets

Total Amount due H.P.P. 7,800 19,500


Cost 6,000 18,000
1,800 1,500

Reserve Required 1,800/7,800 X 88,200 1,500/19,500X 2,11,200


Rs. 20,354 Rs. 16246
Total = Rs. 2,0354 + Rs. 16,246 Rs. 36,600

3.
a) Mr. Bhandari commenced business as a cloth merchant on 1st Shrawan, 2068 with
a capital Rs. 20,000. On the same day, he purchased furniture and fittings for Rs.
10,000 in cash.
Other information:
Particulars Amount (Rs.)
Sales (inclusive of cash sales Rs. 47,000) 1,29,000
Purchases( inclusive of cash purchases Rs. 14,000) 1,01,000
Mr. Bhandari’s drawings (50% cloth) 13,200
(10)
Salaries paid to staffs 12,300
Bad debts written off 500
Business expenses paid 11,700
Mr. Bhandari took cloth worth Rs. 700 from the shop for private use and paid
Rs.500 to his son, but he omitted to record these transactions in books. On 31st
Ashad, 2069 his sundry debtors were Rs. 5, 200 and sundry creditors were Rs.
13,600. Stock in hand on 31st Ashad, 2069 was Rs. 18,550.
From the above transactions obtained from his books kept on single entry, you are
asked to prepare financial statements for the year ended 31st Ashad, 2069. 10
b) A fire engulfed in the premises of a business of M/s K on the morning of 1 st July
2016. The building, equipment and stock were destroyed and the salvage recorded
the following:
Building Rs. 4,000, Equipment Rs. 2,500 and Stock Rs. 20,000. The following
other information was obtained from the records saved for the period from 1st
January 2016 to 30th June 2016.
Particulars Amount (Rs.)
Sales 11,10,000
Purchases 9,37,500
Cartage Inward 17,500
Wages 7,500
Stock in hand 31st December 2015 1,50,000
Building (Value on 31st December 2015) 3,75,000
Equipment (Value on 31st December 2015) 75,000
Depreciation provision till 31st December 2015 on:
Building 1,25,000
Equipment 22,500
No depreciation has been provided since 31st December 2015. The latest rate of
depreciation is 5% p.a on building and 15% p.a on equipment by straight line
method. Normally business makes a profit of 25% on sales. You are required to
prepare the statement of claim for submission to the insurance company. 5
Answer
a)

Trading and Profit & Loss Account


For the year ended 31.03.2069
Dr Cr
Particulars NRs Particulars NRs.
To Purchase 101,000 By Sales 1,29,000
Less: Drawings (7,300) 93,700 By Closing Stock 18,550
(6,600+700)
To Gross Profit 53,850
Total 147,550 Total 1,47,550
To Salaries to staff 12,300 By Gross Profit 53,850
To Bad Debts written off 500
To Business expenses 11,700
To Net Profit 29,350
Total 53,850 53,850

Statement of Financial Position


As on 31.03.2069
Capital 20,000 Furniture and Fittings 10,000
Add: Net Profit 29,350 Sundry Debtors 5,200
(11)
Less: Drawings (14,400) 34,950 Cash (W.N-3) 14,800
Sundry Creditors- 13,600 Closing Stock 18,550
48,550 48,550

Working Notes
Sundry Debtors Account
Dr Cr
Particulars Amount Particulars Amount
To Credit Sales (129,000-47,000) 82,000 By Cash 76,300
By Bad Debts 500
By Balance c/d 5,200
82,000 82,000

Sundry Creditors Account


Dr Cr
Particulars Amount Particulars Amount
To Cash 73,400 By Credit Purchase 87,000
(101,000-14,000)
To Balance c/d 13,600
87,000 87,000

Cash Account
Dr Cr
Particulars Amount Particulars Amount
To Capital 20,000 By Furniture & Fittings 10,000
To Cash Sales 47,000 By Purchases 14,000
To Sundry Debtors (W.N-1) 76,300 By Sundry Creditors(W.N-2) 73,400
By Salaries to staff 12,300
By Drawings (6,600+500) 7,100
By Business Expenses 11,700
By Balance c/d 14,800
1,43,300 143,300

b)

Items Cost Rs. Depreciation Rs. Salvage Rs. Claim Rs.


Stock (w.n 2) 2,80,000 20,000 2,60,000
Building 3,75,000 1,25,000 + 9,375 4,000 2,36,625
Equipment 75,000 22,500 + 5,625 2,500 44,375
Claim Amount 5,41,000

Working Note:
1. Memorandum Trading Account for the period from 1.1.2016 to 30.6.2016
Particulars Rs. Particulars Rs.
To Opening Stock 1.1.2016 1,50,000 By Sales 11,10,000
To Purchases 9,37,500 By Closing Stock 2,80,000
To Cartage Inward 17,500 ( Balancing figure)
To Wages 7,500
To Gross Profit (25% of 2,77,500
11,10,000)
13,90,000 13,90,000

2. Stock Destroyed Account


(12)
Particulars Rs. Particulars Rs.
To Trading Account 2,80,000 By Stock Salvage Account 20,000
By Balance c/d (For Claim) 2,60,000
280,000 280,000
4.
a) Orchid Ltd. undertook a contract for Rs. 500,000 on 1st Magh 2069, On 30th
Paush 2070, when the accounts were closed, the following details about the
contract were gathered:
Rs.
Materials purchased 1,00,000
Wages paid 45,000
General expenses 10,000
Plant purchased 50,000
Material on hand 30.9.2070 25,000
Wages accrued 30.9.2070 5,000
Work certified 2,00,000
Cash received 1,50,000
Work uncertified 15,000
Depreciation on plant 5,000
The above contract contained an escalation clause which read as follows:
“In the event of price of materials and rates of wages increased by more than 5%
the contract price will be increased accordingly by 25% of the rise in the cost of
materials and wages beyond 5% in each case.”
It was found that since the date of signing the agreement, the price of materials
and wages rates increased by 25%. The value of the work certified does not take
into account the effect of the above clause.
Prepare the contract account. 12
b) A contractor has entered into a contract to construct a building. The costs on the
contract worked out to Rs. 10,50,000.
What would be his profit or loss on the contract? 3
i) If the contract is for a fixed price of Rs. 10,00,000
ii) For a price of cost plus 10%.
Answer
a)
Contract Account
Rs. Rs.
To Materials 1,00,000 By Work -in –
To 50,000 progress: 2,00,000
Wages(45,000+5000) 10,000 Work certified 15,000
To General Expenses 5,000 Work uncertified 5,000
To Depreciation on 20,000 Contract escalation
Plant 60,000 (Working note 1) 25000
To P&L A/c(W.N.2) 2,45,000 By Materials in hand 2,45,000
To Balance taken to
WIP
Working Notes:
1. Escalation charges:
(a) Materials
Effect of increase in price of materials
Total Increase Upto 5% Rs. Beyond
75,000×25/125 75,000×5/125
(13)
=15,000 =3000 =12000

(b) Wages
Effect of increase in Wage rate
50000×25/125 50000×5/125
=10000 =2000 =8000

(c) Total Increase (a) +(b)=25000 =5000 =20000


(d) Increase in Contract Price 20,000×25/100 = Rs. 5000
Computation of profit transferred to Profit & Loss Account.
Since more than 1/4th but less than ½ of the contract has been competed 1/3 of the profit earned as
reduced on cash basis has been transferred to Profit & Loss account.
80,000×1/3×150000/200000= Rs. 20,000.
b)
i) In this case as the revenue is Rs. 10,00,000 and the cost are Rs. 10,50,00 the loss on
contract is Rs. 50,000.
ii) In this case, the contractor would receive
Rs. 10,50,000 plus 10% of Rs. 10,50,000 Rs. 11,55,000
Less: Cost incurred 10,50,000
Profit on the contract Rs. 1,05,000
5.
a) Sagun Ltd. took a factory premises on lease on 01.04.2073 for Rs. 1, 00,000 per
month. The lease is operating lease. During Ashad, 2074, Sagun Ltd. relocates its
operation to a new factory building. The lease of the old factory premises
continues to live upto 31.12.2076. The lease cannot be cancelled and cannot be
sub-let to another user. The auditor insists that lease rent of balance 33 months
upto 31.12.2076 should be provided in the accounts for the year ending
31.03.2074. Sagun Ltd. seeks your advice. 5
b) M/s. Laghu Udyog Limited has been charging depreciation on an item of plant and
machinery on straight line basis. The machine as purchased on 1-4-2070 at Rs.
3,25,000. It is expected to have a total useful life of 5 years from the date of
purchase and residual value of Rs. 25,000. Calculate the book value of the
machine as on 1-4-2072 and the total depreciation charged till 31-3-2072 under
SLM. The company wants to change the method of depreciation and charge
depreciation @ 20% on WDV from 2072-73. Is it valid to change the method of
depreciation? Explain the treatment required to be done in the books of accounts in
the context of Accounting Standards.
Ascertain the amount of depreciation to be charged for 2072-73 and the net book
value of the machine as on 31-3-2073 after giving effect of the above change. 5
c) Alpha Ltd. has entered into a sale contract of Rs. 7 crores with Gamma Ltd.
during 2072-73 financial years. The profit on this transaction is Rs. 1 crore. The
delivery of goods to take place during the first month of 2073-74 financial years.
In case of failure of Alpha Ltd. to deliver within the schedule, a compensation of
Rs. 2 crores is to be paid to Gamma Ltd. Alpha Ltd. planned to manufacture the
goods during the last month of 2072-73 financial year. As on balance sheet date
(31.3.2073), the goods were not manufactured and it was unlikely that Alpha Ltd.
will be in a position to meet the contractual obligation. You are required to advise
Alpha Ltd. on requirement of provision for contingency in the financial
statements for the year ended 31st Ashad, 2073, in line with provisions of
Accounting Standards? 5
(14)
Answer
a) In accordance with the provisions of NAS 37 ‘Provisions, Contingent Liabilities and
Contingent Assets’, if an enterprise has a contract that is onerous, the present
obligation under the contract should be recognized and measured as a provision. An onerous
contract is a contract in which the unavoidable cost of meeting the obligations under the
contract exceed the economic benefit expected to be received under it.

In the given case, the operating lease contract has become onerous as the economic benefit of
lease contract for next 33 months up to 31.12.2076 will be nil. However, the lessee, Sagun
Ltd., has to pay lease rent of Rs.3, 300,000 (i.e. Rs.100, 000 p.m. for next 33 months).
Therefore, provision on account of Rs.3, 300,000 is to be provided in the accounts for the
year ending 31.03.2074.
Hence auditor’s contention to provide for the lease rent of balance 33 months upto
31.12.2076 in the accounts for the year ending 31.03.2074 is correct.
b)
As per NAS 16 ‘ Property, Plant & Equipment’, the depreciation method applied to an asset shall be
reviewed at least at each financial year end and, if there has been a significant change in the expected
pattern of consumption of the future economic benefits embodied in the asset, the method shall be
changed to reflect the changed pattern. Such a change shall be accounted for as a change in an
accounting estimate in accordance with NAS 08.

As per NAS 08 ‘Accounting Policies, Changes in Accounting Estimates & Errors’, changes in
accounting estimates shall be adjusted prospectively that means the effect of a change in an accounting
estimate shall be included in the determination of net profit or loss in:
(a) The period of the change, if the change affects the period only; or
(b) The period of the change and future periods, if the change affects both.

In the given case, the company can change the method of depreciation from year 2072-73 if the
conditions set aside in above paragraph have been fulfilled.

Depreciation for year 2072-73 and net book value of Machine as on 31.3.73 Rs.
after effect of the change
Book value of Machinery as on 01.04.2072 2,05,000
Current year depreciation as per new method (WDV) (2,05,000 X 20%) 41,000
Net Book value as on 31.03.2073 (2,05,000–41,000) 1,64,000
Working Note:
Book Value of Machinery and Depreciation under SLM as on 01-04-2072
Rs.
Cost of Machine purchased on 01.04.2070 3,25,000
Less: Residual Value 25,000
Depreciable amount 3,00,000
Useful life of Machine 5 Years
Depreciation for 2 Years (Rs.3,00,000x2/5) 1,20,000
Book value as on 01.04.2072 2,05,000
c)
NAS 37 “Provisions, Contingent Liabilities and Contingent Assets” provides that when an
enterprise has a present obligation, as a result of past events, that probably requires an outflow of
resources and a reliable estimate can be made of the amount of obligation, a provision should be
recognized.
(15)
Alpha Ltd. has the obligation to deliver the goods within the scheduled time as per the contract. It
is probable that Alpha Ltd. will fail to deliver the goods within the schedule and it is also possible
to estimate the amount of compensation. Therefore, Alpha Ltd. should provide for the contingency
amounting Rs. 2 crores as per NAS 37.
6. Write short notes on: (5×3=15)
a) Employee stock option plan and its importance
b) Adjusting entries and correcting entries
c) Advantages of customized accounting packages
d) Challenges of farm accounting in Nepal
e) Situations which normally leads a lease being classified as finance lease
Answer

a) It is a plan under which the enterprise grants employee stock options. Employee stock option
is a contract that gives the employees of the enterprise the right, but not the obligation, for a
specified period of time to purchase or subscribe the shares of the company at a fixed or
determinable price.
The importance of these plans is as follows:
1) Employee stock option plans encourage employees to have higher participation in the
company.
2) Stock options provide an opportunity to employees to contribute in the growth of the
company.
3) Stock option creates long term wealth in the hands of the employees.
4) They are important means to attract, retain and motivate the best available talent for the
company.
5) It creates a sense of ownership between the company and its employees.
b)
Generally, adjusting entries are required every accounting period so that a company's
financial statements reflect the accrual method of accounting.
It is typical for the adjusting entries to be dated as of the last day of the accounting period
and to include an income statement account and a balance sheet account.

Adjusting entries are necessary to:


I. accrue expenses and losses and the related liabilities.
II. accrue revenues and gains and the related assets.
III. defer expenses and the related assets.
IV. defer revenues and the related liabilities.
V. record depreciation expense or bad debts expense and the change in the related contra asset
account.

A correcting entry is needed only if an error is discovered in an account. Correcting entries


can involve any combination of income statement and balance sheet accounts.
Correcting entries are recorded if:
I. an erroneous amount was used in a previously posted entry.
II. an entry was recorded in the wrong account.
c)
Following are the advantages of the customized accounting packages:
1. The input screens can be tailor made to match the input documents for ease of data entry.
2. The reports can be as per the specification of the organization. Many additional MIS reports
can be included in the list of reports.
(16)
3. Bar-codes canners can be used as input devices suitable for the specific need so fan
individual organization.
4. The system can suitably match with the organizational structure of the company.
d)
There are many challenges of farm accounting in Nepal. Some of the challenges are as follows:

1) Agriculture sector in Nepal is unorganized. It is dominated by small farmers and most of these
small famers hold only few acres of land. The average size of holding of land is very small for
commercial farming.
2) Most of the small farmers are illiterate and very poor. Hence, they are neither aware about the
necessity of accounting nor they can afford the expenses of employing someone to maintain the
account.
3) Even big farmers who own considerable land do not keep any accounting to ascertain the profit
from the operations or to take any economic decisions.
4) Even the state does not impose the strong provision of maintaining the farm accounting.

e)

Whether a lease is a finance lease or an operating lease depends on the substance of the transaction
rather than the form of the contract. Finance lease is a lease, which transfers substantially all the risks
and rewards incidental to ownership of an asset to the lessee by the lessor but not the legal ownership.
As per NAS 17, situation that individually or in combination would normally lead to a lease being
classified as a finance lease are:

1) The lessee will get the ownership of leased asset at the end of the lease term.
2) The lessee has an option to purchase the leased asset at the end of the lease term at price, which is
lower than its expected fair value at the date on which option will be exercised.
3) The lease term covers the major part of the economic life of asset even if title is not transferred.
4) At the beginning of lease term, present value of minimum lease rental amount covers at least
substantially all the fair value of leased asset.
5) The leased assets are of such a specialized nature that only the lessee can use them without major
modifications.
(17)

Audit and Assurance


(18)
Suggested

Roll No……………. Maximum Marks - 100

Total No. of Questions- 7 Total No. of Printed Pages- 2

Time Allowed - 3 Hours


Marks
Attempt all questions.

1. As an auditor, give your opinion with explanations on the following cases: (45=20)

a) CA Aarjit Dongol has been appointed as statutory auditor of MNO Limited for the fiscal
year 2072/73. He has been holding certain paid up share capital of the company since the
year 2070. Is the appointment of CA Dongol valid?
b) Nice Guy Ltd. sells goods with a cost of Rs. 100,000 to Start-up Co. for Rs. 140,000 and
a credit period of six months. Nice Guy’s normal cash price would have been Rs. 125,000
with a credit period of one month or with a Rs. 5,000 discount for cash on delivery. The
company wishes to book the revenue as Rs. 140,000.
c) AA Ltd. provided Rs 64 lakhs for Inventory obsolescence in 2072/73. In the subsequent
year, it was determined that 50% of such inventory was usable. The Board of Directors
wants to adjust the same through prior period adjustment.
d) A firm of a father and a son is receiving Rs. 2 lakhs towards job work done for XYZ Ltd.
during the year ended on 32.03.2072. The total job work charges paid by XYZ Ltd.
during the year are over Rs. 50 lakhs. The father is a Managing Director of XYZ Ltd.
having substantial holding. The Managing Director told the auditor that since he is not
involved in the activities of the firm and since the amount paid to it is insignificant; there
is no need to disclose the transaction. He further contended that such a payment made in
the last year was not disclosed. Is Managing Director right in his approach?

Answer:

a) Section 112 of the Companies Act, 2063 states the disqualifications of an auditor. As per the
Act, none of the certain persons or the firms or companies in which such persons are partners
shall be qualified for appointment as auditor and shall, despite appointment as auditor, continue
to hold office.
Provision regarding appointing shareholder as auditor as per Section 112 Sub Section c of
Company Act, 2063 states that: persons or the firms or companies who is substantial
shareholder of the company or a shareholder holding one percent or more of the paid up capital
of the company or his close relative cannot be appointed as auditor.
In the given case, the percentage of holding of paid up capital is not mentioned. If CA Dongol
himself or firm or company in which he is partner or shareholder, or his close relative do not
hold one percent or more than one percent of the paid up capital of MNO Limited only then he
is qualified to be appointed as auditor.

b) Effectively, Nice Guy Ltd is financing Start-up Co. for a period of six months. The normal price
would have been Rs. 125,000 as the goods sold with a credit period of six months. Therefore,
cash discount should not be applicable for the goods sold on credit settlement. Sales revenue
should be accounted for Rs. 125000 being the fair value of consideration receivable. The
difference between the nominal amount of Rs. 140,000 and the normal price of goods Rs.
(19)
125000 i.e. Rs. 15,000 would be recognized as interest income over the period of finance of six
months.

As per NAS 18, Revenue is the gross inflow of economic benefits during the period arising in
the course of the ordinary activities of an entity when those inflows result in increases in equity,
other than increases relating to contributions from equity participants. In a given case it has been
shown after deducting in the purchase which is not as per this standard and it should be
disclosed in gross as per this NAS.

c) As per NAS 10 on "Events after the Reporting Date ", prior period items are income or expenses
which arise in the current period as a result of errors or omissions in the preparation of the
financial statements of one or more prior periods. The write-back of provision made in respect
of inventories in the earlier year does not constitute prior period adjustment since it neither
constitutes error nor omission but it merely involves making estimates based on prevailing
circumstances when financial statements were being prepared. It is a mere estimate process
involving judgment based on the latest information available.
An estimate may have to be revised if changes occur regarding the circumstances on which the
estimate was based, or as a result of new information, more experience or subsequent
developments. The revision of the estimate, by its nature, does not bring the adjustment within
the definitions of an extraordinary item or a prior period item.
In this case, AA Ltd. provided Rs 64 lakhs for inventory obsolescence in 2072-73. In the
subsequent year due to change in circumstances, it was determined that 50% of such inventory
was usable. Revision of such an estimate does not bring the resulting amount of Rs.32 lakhs
within the definition either of a prior period item or of an extraordinary item. The amount,
however, involved is material and requires separate disclosure to understand the financial
position and performance of an enterprise. Accordingly, adjustment in the value of the inventory
through prior period item would not be appropriate.

d) Nepal Accounting Standard 24,“Related Party Disclosures” applies to the facts of the case.
NAS 24 requires disclosure of party relationship and transactions between a reporting enterprise
and its related parties. The parties are considered to be related if at any time during the reporting
period, one party has the ability to control the other party or exercise significant influence over
the other party in making decisions.
In the given case, the managing director of XYZ Ltd. is a partner in the firm with his son, which
has been paid Rs. 2 lakhs as job work charges. The managing director is having a substantial
holding in the firm. The case is covered by NAS 24. The approach of the managing director is
not tenable under the standard and accordingly all disclosure requirements have to be complied.
Since there is related party transaction the contention of managing Director is not correct and the
auditor should advise him to make proper disclosure as required by NAS and if the management
refuses, the auditor shall express a qualified report.

2. Give your comments on the following cases: (45=20)


a) The Joint Merger Committee of Alpha Bank and Beta Bank has appointed CA Ramarjun
as Due Diligence Auditor of both banks for the merger process. The appointment letter
has following clause regarding fees: “Fees amounting to Rs.300, 000 for each bank
(exclusive of VAT) shall be paid on submission of DDA report. Additional payment of
Rs.150,000 (inclusive of VAT) shall be made, if the merger is successful.”
b) CA Chandan was appointed auditor of Delta Telecom Ltd. He presented audit plan where
in ten teams comprising 4 personnel in each team will be visiting 40 branches (out of 50)
across the country for a total of around 4 months. The finance controller of Delta Telecom
offered SIM cards for all the team members engaged in the audit with one year free 4G
data and voice services and unlimited one year free 10 MB internet to Auditor’s Office as
logistic support so that communication/emails with all the members are uninterrupted
(20)
during the course of audit. It is expected that CA Chandan will be appointed as auditor for
next two years as well. The general one time price of SIM cards and installation charges
for internet however shall have to be paid for. The Finance Controller requests for
replacing some of the branches with others for audit.
c) Verisk Limited has employed an Independent Actuary (Dansk & Co.) for actuarial
valuation of its employees’ long term liabilities for the year end 31st Ashadh 2073. This
was the first year of actuarial valuation. CA Mangesh was statutory auditor for the
company. He was satisfied with the appropriateness and reasonableness of the
assumptions and methods used along with financial data reflected in the financial
statement and decided to issue an unmodified report. He also added following in his
report for more clarity on the subject “The employees’ long term liabilities is based on
actuarial valuation by Dansk & C0.”
d) .Sagar International Ltd. has acquired a solar power system on 01.04.2072 for Rs. 100
lakhs. On 02.04.2072, it applied to Alternative Energy Promotion Board of Nepal for a
50% subsidy. The subsidy application was finally approved on 01.06.2073. While
finalizing the accounts, the company has accounted the subsidy as adjusting event after
reporting period. The board of directors has not yet authorized the financials for issue.
Answer: 2(a)

Section 240 “Fees and other types of remuneration” of Code of Ethics of ICAN , ( Part B) states that
Professional services should not be offered or rendered to a client under an arrangement whereby no
fee will be charged unless a specified finding or result is obtained or when the fee is otherwise
contingent upon the findings or results of such services. Fees charged on a percentage or similar
basis should be regarded as contingent.

In the given instance; though the first part of the clause of fee is straight forward and will be received
on submission of the DDA report, the second part of the fee is contingent upon the successful
merger. The second part of the fee will not be there if the merger is unsuccessful. It is understood
that the success of merger shall be significantly influenced by the DDA Report. This may also cause
self-interest threat on the objectivity of the assignment. Given the facts, CA Ramarjun should not
accept the assignment based on the given terms of fee.

Answer:2(b)
Section 260 “Gifts & Hospitality” of Code of Ethics ICAN, Part B states that acceptance of goods
and services from a client may create a threat to self-interest threat or familiarity threat to objectivity.
Goods and services should not be accepted except on business terms no more favorable than those
generally available to others. The existence and significance of any threat will depend on the nature,
value, and intent of the offer.

In the given case, the purchase of SIM cards at regular price seem no more favorable that to others
and the facility seemed normal in terms of logistic support for smooth conduction of the audit works.
However, the offer for free 4G data charges, voice service and internet for one year which is
extended beyond the normal course (4 months) of audit execution period. Therefore, it may not be
concluded that the offer is in normal course without the specific intent to influence the decision
making. Acceptance of the offer may pose a threat on independence and objectivity and hence should
not be accepted.

Answer:2(c)
NSA 620 “Using the work of an expert” requires that “when issuing an unmodified auditor’s report,
the auditor should not refer to the work of an expert”. Such reference might be misunderstood to be a
qualification of the auditor’s opinion or diversion of responsibility, neither of which is intended.
(21)
In the given context, the reporting by CA Mangesh is more inclined to “emphasis of matter” under
modified reports (matters that do not affect the auditor’s opinion). Hence, CA Mangesh should not
refer the actuarial valuation in his audit report. However, he must ensure that adequate disclosures
about the actuarial valuation and the related figures have been reflected by the Management in the
Financial Statement.

Answer:2(d)
NAS 10 on “events after reporting period” requires the value of assets and liabilities to be adjusted
for events occurring after the balance sheet date which occur upto the date of approval of accounts
by the Board of Directors ( authorized for issue) if they provide evidence of the conditions existing at
the end of the reporting period. Since, in this case books of account have not been approved, grant of
subsidy will be considered as an adjusting event. Hence, the accounts should be adjusted for the
subsidy in 2072-73.

Hence, the subsidy should be either credited to the cost of the system or alternatively may be treated
as deferred income to be written off over the useful life in proportion in which depreciation is written
off.

3. Answer the following: (35=15)


a) NSA 300 Planning an Audit of Financial Statements provides guidance to assist auditors
in planning an audit. Explain the benefits of audit planning.
b) NSA 210 Agreeing the Terms of Audit Engagements requires auditors to agree the terms
of an engagement with those charged with governance and formalize these in an
engagement letter. Identify and explain the factors which would indicate that an
engagement letter for an existing audit client should be revised.
c) Explain about the factors that influence the reliability of Audit Evidence.

Answer: 3(a)
Benefits of audit planning
Audit planning is addressed by NSA 300 Planning an Audit of Financial Statements. It states that
adequate planning benefits the audit of financial statements in several ways:
 Helping the auditor to devote appropriate attention to important areas of the audit.
 Helping the auditor to identify and resolve potential problems on a timely basis.
 Helping the auditor to properly organize and manage the audit engagement so that it is
performed in an effective and efficient manner.
 Assisting in the selection of engagement team members with appropriate levels of capabilities
and competence to respond to anticipated risks and the proper assignment of work to them.
 Facilitating the direction and supervision of engagement team members and the review of their
work.
 Assisting, where applicable, in coordination of work done by experts.

Answer: 3(b)
Engagement letters for recurring/existing clients should be revised if any of the following factors
are present:
(22)
 Any indication that the entity misunderstands the objective and scope of the audit, as this
misunderstanding would need to be clarified.
 Any revised or special terms of the audit engagement, as these would require inclusion in the
engagement letter.
 A recent change of senior management or significant change in ownership. The letter is
signed by a director on behalf of those charged with governance; if there have been
significant changes in management they need to be made aware of what the audit engagement
letter includes.
 A significant change in nature or size of the entity’s business. The approach taken by the
auditor may need to change to reflect the change in the entity and this should be clarified in
the engagement letter.
 A change in legal or regulatory requirements. The engagement letter is a contract; hence if
legal or regulatory changes occur, then the contract could be out of date.
 A change in the financial reporting framework adopted in the preparation of the financial
statements. The engagement letter clarifies the role of auditors and those charged with
governance, it identifies the reporting framework of the financial statements and if these
changes, then the letter requires updating.
 A change in other reporting requirements. Other reporting requirements may be stipulated in
the engagement letter; hence if these change, the letter should be updated.

Answer: 3(c)
The reliability of information to be used as audit evidence is influenced by its source, nature, and
the circumstances under which it is obtained, including the controls over its preparation and
maintenance. Even when information to be used as audit evidence is obtained from sources
external to the entity, circumstances may exist that could affect its reliability. For example,
information obtained from an independent external source may not be reliable if the expert may
lack objectivity. While recognizing that exceptions may exist, the following generalizations about
the reliability of audit evidence may be useful:
(i) Independent Source - The reliability of audit evidence is increased when it is obtained
from independent sources outside the entity.
(ii) Effective internal control system - The reliability of audit evidence that is generated
internally is increased when the related controls, including its preparation and maintenance,
imposed by the entity are effective.
(iii) Method of obtaining - Audit evidence obtained directly by the auditor (for example,
observation of the application of a control) is more reliable than audit evidence obtained indirectly
or by inference (for example, inquiry about the application of a control).
(iv) Form of Audit Document - Audit evidence in documentary form, whether paper,
electronic, or other medium, is more reliable than evidence obtained orally (for example, a
contemporaneously written record of a meeting is more reliable than a subsequent oral
representation of the matters discussed).
(v) Type of Documents - Audit evidence provided by original documents is more reliable
than audit evidence provided by photocopies or facsimiles, or documents that have been filmed,
digitized or otherwise transformed into electronic form, the reliability of which may depend on the
controls over their preparation and maintenance.

4. Answer/Comment on the following: (35=15)


(23)
a) Rabin & Associates; Chartered Accountants has been operating a separate bank account
for keeping client`s money in course of availing fund manager service to the client. The
interest earned on such account during financial year 2072/73 amounting Rs. 100,000 has
been transferred to firm`s bank account and booked as miscellaneous income with
corresponding miscellaneous income & expenditure in client`s bank account.
b) When professional accountant performs services in a country other than Nepal and
differences on specific matters exist between ethical requirements of the two countries,
write down the three provisions to be applied to professional accountant on such situation.
c) You are the auditor of Peace Nepal Ltd. for FY 2072/73. Peace Nepal Ltd. is seeking your
assistance for preparation of accounting records for F.Y. 2072/73.
Answer Q. No.4 (a): :

As per Section 270 “Custody of Client Assets” of Code of Ethics of the Institute of Chartered
Accountants of Nepal, it has been clearly stated that: “All interest earned on clients` monies should be
credited to the client`s account".

The accounting entries made by Rabin & Associates; chartered accountants for interest income in
client`s account is initially correct. But subsequent transfer of interest income to firm's account by
booking as miscellaneous expense in client`s account is not correct.

In the light of the provision contained in Section 270 of Code of Ethics , a professional accountant in
public practice entrusted with money (or other assets) belonging to others shall therefore;
a) Keep such assets separately from personal or firm assets;
b) Use such assets only for the intended purpose;
c) Be ready to present accounting of those assets and any income generated from such assets at all
time to the client; and
d) Comply all the relevant laws and regulations applicable for the custody of client\s assets.
Hence, Rabin & Associates has breached the code of ethics as prescribed by ICAN and is under the
disciplinary action for non-compliance of ICAN code of ethics.
Answer Q. No. 4 (b) :

As per Section 6.3 of the Code of Ethics of the institute of the chartered accountants of Nepal
provisions applied to professional in such situation are:

(i) When the ethical requirements of the country in which the services are being performed are less
strict than the ICAN Code of Ethics, then the ICAN Code of Ethics should be applied.
(ii) When the ethical requirements of the country in which services are being performed are stricter
than the ICAN Code of Ethics, then the ethical requirements in the country where services are
being performed should be applied.
(iii).When the ethical requirements of Nepal are mandatory for services performed outside that country
and are stricter than set out in (i) and (ii) above, then the ethical requirements of Nepal should
be applied. (In the case of cross border advertising and solicitation it should be dealt as per
provision under section 14.)

Answer Q. No. 4 (c):

As per section 8.5 of the code of ethics of ICAN following requirements should be considered by the
independent auditor before assisting client for preparing accounting records:

(i) Should not have any relationship or combination of relationships with the client or any conflict of
interest which would impair integrity or independence.
(ii) The client should accept the responsibility for the statements.
(24)
(iii)Should not assume the role of employee or of management conducting the operations of an
enterprise.
(iv) Staff assigned to the preparation of accounting records ideally should not participate in the
examination of such records. The fact that the professional accountant in public practice has
processed or maintained certain records does not eliminate the need to make sufficient audit tests.

In the light of the provision contained in Section 8.5 of Code of Ethics of the Institute of Chartered
Accountants of Nepal ; I would like to the ensure the compliance of aforesaid provision before
accepting such request of my client.

5. Answer the following: (25=10)


a) Explain the procedures for appointment of auditor of corporate bodies wholly owned by
Government of Nepal.
b) Define the term "Performance Audit".
Answers Q. No.5 (a):

Section 6 of Audit Act 2048 (1991 AD), deals with the provision for appointing Auditor of Corporate
Bodies Wholly Owned by Government of Nepal. The related provisions are:

 As per Section 6 (1); notwithstanding anything contained in the existing laws, the audit of the
corporate bodies wholly owned by Government of Nepal shall be audited by the Auditor General
pursuant to this Act.

 As per section 6 (2); If the Auditor General is constrained by time and resources to audit the
corporate bodies wholly owned by Government of Nepal pursuant to Sub-section (1) he/she may
appoint license holder auditors under the prevailing laws an assistant. While appointing auditor
as such, he/she shall give priority to the Nepali citizen.

 As per section 6 (3); The auditor appointed pursuant to Sub-section (2) shall act under the direction,
supervision and control of the Auditor General.
 As per section 6 (4); The powers, functions, duties and responsibilities of the auditors appointed
pursuant to Sub-section (2) and the procedures to be followed by them in course audit and provisions
relating to their report shall be as prescribed by the Auditor General.
 As per section 6 (5); The remuneration to be paid by the concerned organization to the auditors
appointed pursuant to Sub-section (2) shall be fixed by the Auditor General keeping in view the
volume of financial transactions, status of accounts, number of branches and sub-branches, work
load and work progress of the concerned organization.

Answers Q. No.5 (b):

According to INTOSAI’s Auditing Standards, performance audit is an independent examination of the


efficiency and effectiveness of government undertakings, programs or organizations, with due regard to
economy, and the aim of leading to improvements.

Performance audit is based on decisions made or goals established by the legislature, and it may be
carried out throughout the whole public sector.

Performance audit is concerned with the audit of economy, efficiency and effectiveness (the three E’s).
A performance audit assignment may include all/one or a combination of two aspects of “3Es”.
(25)
“Economy” means the acquisition of the appropriate quality and quantity of human, financial, physical
and information resources at the appropriate times and at the lower cost possible. It is minimising the
cost of resources used for an activity (input), having due regard to appropriate quality.
“Efficiency” means the use of human, financial, physical and information resources such that the
output is maximized for any given set of resource inputs, or input is minimized for any given
quantity and quality of output.
“Effectiveness” means the achievement of the objectives or other intended effects of activities. It
addresses whether policy objectives or goals have been met and whether this can be attributed to the
policy pursued.
The concept of performance auditing emerged in response to:
 increasing demand for information on efficiency and economy in managing resources and the
effectiveness with which objectives are met;
 need to determine whether :
 the operations of audit entities were conducted in a way that ensures the best possible use of
resources or considering the 3Es;
 officials in the public sector have met their accountability obligations;
 reporting on performance is credible and adequate.

Regularity and propriety related issues which impact performance may be considered in the conduct of
performance audits.

6. Write short notes on the following: (42.5=10)


a) Uses of Negative External Confirmation Requests
b) Use of Assertion in an Audit of Financial Statements
c) Key Audit Matters
d) Powers to issue Directives by the Auditor General
Answer:
a) Uses of Negative External Confirmation Requests
A negative external confirmation request asks the respondent to reply only in the event of
disagreement with the information provided in the request. However, when no response has been
received to a negative confirmation request, the auditor remains aware that there will be no explicit
evidence that intended third parties have received the confirmation requests and verified that the
information contained therein is correct. Accordingly, the use of negative confirmation requests
ordinarily provides less reliable evidence than the use of positive confirmation requests, and the
auditor considers performing other substantive procedures to supplement the use of negative
confirmations.

Negative confirmation requests may be used to reduce audit risk to an acceptable level when: (a) the
assessed level of inherent and control risk is low; (b) a large number of small balances is involved; (c)
a substantial number of errors is not expected; and (d) the auditor has no reason to believe that
respondents will disregard these requests.

b) Use of Assertion in an Audit of Financial Statements


Assertions are the implicit or explicit claims and representations made by the management
responsible for the preparation of financial statements regarding the appropriateness of the
various elements of financial statements and disclosures.
Auditor may use those assertions for audit examination during audit of Financial Statements. In
preparing financial statements, management is making implicit or explicit claims (i.e. assertions)
regarding the recognition, measurement and presentation
(26)
of assets, liabilities, equity, income, expenses and disclosures in accordance with the applicable
financial reporting framework (e.g. NAS/NFRS).
Following Assertions for class of transaction, account balance and disclosures can be used.
 Occurrence,
 Completeness,
 Accuracy cut-off,
 Classification and understandability
 Existence,
 Rights and Obligations
 Valuation and allocation

c) Key Audit Matters


Those matters that, in auditor's professional judgment, are of most significant in the audit of
financial statements of the current period are referred as key audit matters. Such key audit matters
are selected from matters communicated with those charged with governance. The auditor is
required to communicate such matters to those charged with governance and include in audit
report in the Key Audit Matters Section.
The purpose of communicating key audit matters is to enhance the communicative value of
auditor's report by providing greater transparency about the audit that was performed.
Communicating key audit matters provides additional information to intended users of the
financial statements to assist them in understanding those matters which are of most significance
in the audit of the financial statements. Communicating key audit matters is not a substitute for
disclosure in the financial statements or the substitute for the auditor expressing a modified
opinion. In determining key audit matters, auditor may take into account the areas of higher
assessed risks and areas requiring significant management judgment.

d) Powers to issue Directives by the Auditor General


The Auditor General may, subject to the Constitution of Nepal and the prevailing laws,
issue directives to the concerned Government Offices, and Corporate Bodies wholly or
substantially owned by Government of Nepal, from time to time to make proper
arrangements on matters of accounts and to maintain regularity therein. It shall be the
duty of the concerned offices or organizations to abide by such directives.
7. Distinguish between: (25=10)
a) Computerized and manual accounting system
b) Auditing around computer and Audit through computer

Answer:
a) Distinction between computerized and Manual Accounting System:
i. Faster and efficient in processing of information in computerized system and no such faster
and efficient in processing of information in manual system
ii. Automatic generation of accounting documents like invoices, cheques and statement of
account which manual system cannot produce.
iii. With the larger reductions in the cost of hardware and software and availability of user-
friendly accounting software package, it is relatively cheaper like maintaining a manual
accounting system;
iv. More timely information can be produced than manual system
(27)
v. No more manual processing of the data- all automatically posted to the various
ledgers/accounts and many types of useful reports can be automatically generated for
management to make decisions where as such reports cannot generated on manual system
vi. Power failure, computer viruses and hackers are the inherent problems of using computerized
systems, such risk not remain in manual system
vii. Once data been input into the system, automatically the output are obtained hence the data
being input needs to be validated for accuracy and completeness, we should not forget concept
of GIGO (Garbage In(Input) Garbage out ( Output) where validation in manual system can be
checked on inception
viii. Accounting system not properly set up to meet the requirement of the business due to badly
programmed or inappropriate software or hardware or personnel problems can caused more
havoc, where manual system does not have such problem.
ix. Danger of computer fraud if proper level of control and security whether internal and external
b)
i. Audit Around the Computer: Audit around the computer involves forming of an audit opinion
wherein the existence of computer is not taken into account. Rather the principle of
conventional audit like examination of internal controls and substantive testing is done. The
auditor views the computer as a black box, as the application system processing is not
examined directly. The main advantage of auditing around the computer is its simplicity.
Audit around the computer is applicable in the following situations:
ii. The system is simple and uses generalized software that is well tested and widely used.
iii. Processing mainly consists of sorting the input data and updating the master file in sequence.
iv. Audit trail is clear. Detailed reports are prepared at key processing points within the system.
v. Control over input transactions can be maintained through normal methods, i.e. separation of
duties, and management supervision.

Generalized software packages, like payroll and provident fund package, accounts receivable and
payable package, etc. are available, developed by software vendors. Though, the auditor may
decide not to go in details of the processing aspects, if there are well tested widely used packages
provided by a reputed vendor. However, he has to ensure that there are adequate controls to
prevent unauthorized modifications of the package. However, it may be noted that all such
generalized packages do not make the system amenable to audit. Some software packages provide
generalized functions that still must be selected and combined to achieve the required application
system. In such a case, instead of simply examining the systems input and output, the auditor must
check the system in depth to satisfy him about such system. The main disadvantages of the system
of auditing around the computer are:

a) It is not beneficial for complex systems of large scale in very large multi-unit, multi locational
companies, having various inter unit transactions. It can be used only in case of small
organizations having simple operations.
b) It is difficult for the auditor to assess the degradation in the system in case of change in
environment, and whether the system can cope with a changed environment.
(ii) Auditing through the Computer: This approach involves actual use of computer for
processing the information by auditor. The circumstances, where auditing through the computer is
done are as follows:
(i) The organization has developed either in house or through a reputed vendor, a software
package suitable to its requirement, because of inability of a generalized package to cater to the
complex nature of transactions.
(ii) The system processes very large volumes of output. This makes examination of validity of
input and output difficult.
(iii) The major part of the internal control system in the organization is in the computer system
itself, as the majority of the records is processed through the computer. Examples are system in
bank, insurance companies, online booking in case of Railway, etc.
(iv)The logic of the system is quite complex, and there is virtually no visible audit trail. The
auditor has to use the computer to test the logic and controls existing within the system.
(28)
The auditor has to use the computer system itself for verification, for which he has to be
sufficiently computer literate, and should have adequate technical knowledge and expertise. The
auditor can through the computer, increase his performance, and can rely on the data processing by
carrying out the required tests and applying his skill
(29)

Corporate Law
(30)
Suggested Answers

Marks
Attempt all questions.
1. Answer the following questions: (5×5=25)
a) The directors of Orbit Fibers Pvt. Ltd. borrowed a sum of money from NCC Bank. The company’s
articles provided that the directors might borrow loan as may from time to time be authorized by a
resolution passed at a general meeting of the company. When the bank sued the company to
recover debt, the shareholders claimed that there had been no such resolution passed authorizing
the loan, hence, the loan is ultra-vires and not binding to the company. Can the bank recover the
debt amount against the company? Explain.
b) Mr. Sharma, 20 Years old, has been appointed as a director of XYZ Ltd. by its promoters. Ms.
Shanti, a shareholder, opposed on such appointment. Whether Mr. Sharma’s appointment as a
director is legal? What kinds of mechanisms of appointing a director are there in Companies Act,
2063? Explain.
c) Running Bulls Pvt. Ltd. is an emerging investment company having sound track record in profit
earning established by a group of new investors five years ago. Now they have intended to convert
this private limited company to public limited company but have no idea about the relevant legal
provisions; therefore, they appointed you as an expert on this matter. Advise them on the basis of
the relevant provision of the Companies Act, 2063.
d) Board of Director of XYZ Insurance Company is in the opinion to increase their capital with the
medium of Further Public Offering (FPO) in premium price. Advise the BOD of the insurance
company regarding the pre-conditions of and the procedures for issuing FPO at premium price
citing the relevant provision of the Companies Act, 2063.
e) Promoters of a prospective company Bhatbhateni Ltd. applied to the Company Registrar Office for
its registration. Few days after filing the application, the Company Registrar office refused its
registration. Now in this context, state the circumstances under which the Company Registrar
Office may refuse registration of prospective company Bhatbhateni Ltd.?

Answers:
a) The doctrine of indoor management has been developed to rescue the third parties or to enable
them to be secured from the internal irregularities committed by the directors which they should
manage. Similarly, it saves the third parties from the rigour of the strict implication of the doctrine
of constructive notice. This implies that the outsiders are not bound to inquire whether the act of
the directors which is related to internal management had been properly and regularly performed.
It was ruled in the case of Royal British Bank v. Turquand:(1856) 6 E. & B. 327 that where the
directors exceed their powers or infringe the restrictions imposed upon them, the company may be
bound; for an outsider dealing with the company is required to see that the transaction on the face
of it is regular and consistent with the articles.
In this case, the company is bound by the loan. Once it was found that the directors could borrow
subject to a resolution, the Bank had the right to infer that the necessary resolution must have been
passed. When third party has dealt with the company through the board of directors which is not
ultra-vires to the Articles and Memorandum of the company, the company cannot avoid liability to
pay the loan to the bank on the ground that such resolution never been passed at the general
meeting of the company.

b) XYZ Company is a public company. The director, Mr. Sharma, appointed by promoters, is of 20
years old. Section 89 (1) (a) of the Companies Act has provided the various grounds for
disqualification among them one is below Twenty One years old. Obviously, he is under age of
(31)
requirements age to become director of a public company. Therefore, Mr. Sharma is disqualified
for the post of director of such company. On the other hand, the opposing logic and complaint of
Ms. Shanti is valid and Mr. Sharma could not hold the office of a director in accordance with the
section 89(3)(a) of the Companies Act.

A director of a company can be appointed following the procedures mention in the companies Act
and through the mechanism as provided.

a. Appointment by Promoters:
Basically, the primary authority of appointment of director is rest on the General Meeting
(GM). However, Promoters can also appoint director in the preliminary phase of any
company. Promoters can exercise the appointing authority before the first general meeting is
held. These directors are called first directors or can be called interim period director of a
company. They are remained in the post of director till other directors are appointed by the
GM. It mentioned in the first proviso of section 87(1).

b. Appointment by Board of Directors (BoD) :


Such kind of appointment is made for the interim or remaining period. BoD may appoint the
directors in the vacant post only. Second Proviso of Section 87 (1) has provided such
mechanism and procedure. If the office of any director appointed by the annual general
meeting is vacated for any reason, the board of directors shall appoint another director in that
vacancy.

c. Appointment by General Meeting (GM):


It is already said that the primary authority of appointment of director is rest upon the GM.
Other mechanisms are exercised such power in the extraordinary situation. Section 87(1) has
provided such authority to GM. In a private company directors are appointed as provided in its
AoA (Sec 86).

d. Appointment by Corporate Body :


A corporate body can also appoint the directors. On the basis of amount of shares it holds,
can appoint directors and an alternative director as well as to attend the meeting and exercise
voting right in his absence on the proportional basis of the total number of directors and
numbers of shares hold.

c) As per the Section 13 (1) of the Companies Act, 2063; a private company shall be converted into a
public company if the general meeting of the private company, by adopting a special resolution, decides
to convert that company into a public company, Provided, however, that no private company shall be
capable of being converted into a public company unless and until it fulfills the requirements to be
fulfilled under Companies Act 2063 for being a public company.
1. Number of shareholder should be minimum of 7.
2. The paid-up capital shall be a minimum of ten million rupees.
So, in the above case of the Running Bulls Pvt. Ltd, it can be changed to public company by adopting
special resolution in the general meeting and also fulfilling all other criteria required for registration of
the Public Limited Company as per the Companies Act, 2063.

d) As per section 29 (1) of the Companies Act, 2063 any company fulfilling the following conditions
may, with the prior approval of the Office, issue shares at a premium:
(a) The company has been making profits and distributing dividends for three consecutive years,
(b) The company’s net worth exceeds its total liabilities,
(c) The company’s general meeting has decided to issue shares at a premium.
(32)

Further as per the sub section (2) of the section 29, where the shares are sold at a premium
pursuant to Sub-section (1), a sum in excess of the face value, out of the proceeds thereof, shall be
deposited in a premium account to be opened to that effect.

e) Section 6 of the Companies Act, 2063 provides the circumstances under which the Company
Registrar's Office may refuse to register the prospective company Bhatbhateni Ltd.
Pursuant to Section 6 of the Companies Act, 2063 the Company Registrar's Office may refuse to
register the prospective company Bhatbhateni Ltd in any of the following circumstances,
a. If the name of the proposed company is identical with the name by which a company in
existence has been previously registered or so resembles the name of that company as it might
cause misleading.
b. If the name or objective of the proposed company is contrary to the prevailing law or appears
to be improper or undesirable in view of public interest, morality, decency, etiquette etc or
reflects criminal motive.
c. If the name of the proposed company is identical with the name of which registration has been
cancelled pursuant to this Act or that of a company which has been insolvent under the
prevailing law or so resembles such name as it might cause misleading and a period of five
years has not expired after such cancellation of registration or insolvency.
d. If the requirements for the incorporation of a company under this Act are not fulfilled.

If Company Registrar's Office refuses to register the prospective company Bhatbhateni Ltd in any
of the circumstances referred to in section 6(1) of Companies Act, 2063, it shall give a notice there
of, accompanied by the reason there for, to the applicant Bhatbhateni Ltd, no later than 15 days
after the date of application made for the incorporation of company pursuant to section 4 of
Companies Act, 2063.

2. Answer the following questions: (3×5=15)


a) What are the functions not to be carried out by the Nepal Rastra Bank under the Nepal Rastra
Bank Act, 2058?
b) Kantipur Bank limited has adopted special resolution for buy back of its 10 percent share out of
the money borrowed by issuing the 10% Debenture. But the bank is in doubt regarding the
decision made whether it is in contradiction with the relevant provision of Banking and
Financial Institution Act, 2063 and seek your opinion on this matter. Suggest the bank about
the relevant provision of the Banking and Financial Institution Act, 2063 regarding decision of
the bank to buy back its share.
c) Grand Finance Co. Ltd. had stopped its business continuously for more than 1 month. Further,
it was complained against the Finance Co. for the violation of law and banking rules, therefore,
NRB formed an inquiry committee to recommend whether the license of the Finance Co.
withdrawn. State the legal provisions of BAFIA, 2063 regarding the cancellation of the license
obtained by a licensed institution to carry on the financial transactions by the NRB

Answers:
a) As per Section 7 of Nepal Rastra Bank Act, 2058, except otherwise provided for in
this Act, the Bank shall not carry out the following functions:
(a) Providing any loan, accepting any type of deposit or making any type of financial gift;
(b) Purchasing shares of any commercial bank, financial institution, public corporation or a
company or acquiring any type of proprietary right in any financial, commercial, agricultural,
industrial or other institution;
(c) Carrying out any type of trade; and
(33)
(d) Acquiring right over movable and immovable property by way of purchase, lease or in any
manner whatsoever.
Provided that the Bank may acquire such property as required for carrying out its function or for
achieving its objectives.

Notwithstanding anything contained in Sub-section (1), the Bank may carry out the following
functions: -
(a) To provide loan to and invest in the shares of the institutions which carry out the functions
helpful in carrying out the function of the Bank or in attaining its objectives, not exceeding
ten percent of the total capital of such institutions.
(b) To provide loan to its own employees.

b) As per section 10 (1) of the Banking and Financial Institution Act, 2063; no bank or financial
institution shall purchase its own shares (buy-back) or lend moneys against security of its own
shares.
But as per sub section (2), notwithstanding anything contained in Sub-section (1), in following
circumstances, a bank or financial institution may, with the approval of the Nepal Rastra Bank,
buy back its shares out of its free reserves available for being distributed as dividends not
exceeding the percentage prescribed by the Rastra Bank.
1. If all the shares issued by the Bank / FI is fully subscribed
2. If shares issued by the Bank / FI is listed in Securities Board.
3. If Article of Association of Bank / FI has provision of buyback of own share.
4. If special resolution on buy back of own share is passed in General Meeting of Bank / FI.
5. If the loan payable by the bank is not more than double of share capital after buy back and
general reserve.
6. If amount of such buyback is not more than 20 % share capital and general reserve.
7. If such buyback is not against the directives of Nepal Rastra Bank regarding buyback of own
share.

So, the special resolution adopted by the Kantipur Bank for buy back of its share out of the money
borrowed by issuing debenture is in contradiction to the provision of section 10 (2) of the Banking
and Financial Institution Act 2063 which cannot be executed.

c) As a regulatory body, the Nepal Rastra Bank shall have full authority to regulate and systematize
the functions and activities of all licensed institutions. It is the bank of banks. Section 35 (2) of the
Bank and Financial Institutions Act, 2063 has stated that the Rastra Bank may cancel the license
obtained by a licensed institution to carry on the financial transactions pursuant to this Act in any
of the following circumstances.
(1) If the concerned licensed bank or financial institution requests for the cancellation of its
license.
(2) If it fails to carry on the financial transactions within six months from the date of receipt of
the license.
(3) If it stops carrying on the financial transactions since more than one month ago
continuously.
(4) If it carries on the financial transactions in such a manner as to be contrary to the rights and
interests of depositors.
(5) If it violates the conditions prescribed by the Rastra Bank.
(6) If it fails to comply with the orders or directives issued by the Rastra Bank.
(7) If it becomes insolvent.
(8) If the bank or financial institution is found to have obtained the license by submitting false
details.
(9) If the licensed institution is merged with another bank or financial institution.
(34)

As the Grand Finance Co. Ltd. has stopped its financial transaction for more than one month
continuously, the NRB can take action to cancel the license obtained by it to carry on the financial
transaction.

3. Answer the following questions: (2×5=10)


a) State the circumstances on which the chairperson of Securities Board of Nepal may
be removed from office by the Government of Nepal pursuant to the Securities Act,
2053.
b) Mention the provision of Revolving Fund as per Securities Act, 2063.
Answers:
a) Section 12 of the Securities Act, 2053 provides the circumstances on which a chairperson of
Securities Board of Nepal may be removed from office.
Section 12 (2) states that the Chairperson, as the case may be, shall be removed from the office in
any of the following circumstances:
a. If one is disqualified to be a Chairperson, as the case may be, pursuant to section 11 of
this Act,
i. One who is an office bearer of a political party.
ii. A person involved in securities business.
iii. One who is adjudicated as an insolvent.
iv. One who is insane.
v. One who has been convicted by the court of an offence involving moral
turpitude.
b. If one commits any act contrary to the interest of investors in securities or any act that
may cause loss or damage to the development of capital markets.
c. If one suffers from lack of competence to implement, or cause to implement, such
functions required to be performed by the Securities Board to attain its objectives
pursuant to this Act or the Rules framed under this Act.
d. If one has been held disqualified to carry any occupation or business by the reason of
misconduct and his or her certificate has been revoked or he or she has thus been
restricted to carry on business.
e. If one remains absent from three consecutive meetings of the Board without giving
notice to it.
Section 12(1) states that, where there occurs a circumstances for removal of the chairperson
of the Board as above the Government of Nepal shall remove the Chairperson, as the case
may be.
Provided that prior to making such removal, the Government of Nepal, shall not deprive the
concerned person of a reasonable opportunity to defend him/herself.

b) As per section 23 of the Securities Act, 2063 regarding Revolving Fund:


(1)The Board may establish a revolving fund to manage its source of income and such amounts as
specified by the Board shall be credited to that fund each year.
(2)The amounts of the revolving fund may be held in securities issued by the Government of Nepal
or in such a periodic account as may be prescribed by the Board.
(3)Generally, no moneys held in the revolving fund, other than income earned out of the moneys in
that fund, shall be spent.
(4)Provisions relating to the operation of the revolving fund shall be as prescribed.

4. Answer the following questions: (2×5=10)


(35)
a) 33.33% of the total employees are under disputes and want to start collective
disputes and go for the strike. Is this possible? How strike may be initiated? Give
your answer by referring Labor Act, 2048.
b) Describe when and how bonus is to be transferred to welfare fund under bonus Act,
2030.

Answers:
a) As per section 73 of the Labor Act, 2048; prescribe the procedure for collective dispute,
(1) The claim shall have to be presented in writing to the concerned Proprietors signed by at least
fifty one percent of the workers.
(2) Upon receipt of the claim the Proprietor shall hold bilateral discussion with the Representatives
of workers .
(3) If the dispute could not be solved as per (2) above, the dispute shall be solved within fifteen
days by holding bilateral discussion in the presence of Labour Office.
(4) If the dispute could not be solved through the bilateral discussion the dispute may be referred to
a mediator and decide the dispute within fifteen days.
(5) If the mediator or the Committee does not make a decision within the time-limit or Nepal
Government does not give decision on appeal within sixty days from the date of filing such appeal,
the workers or employees may strike.

Here in the given case, only 33.33% worker or employees are under disputes so this is not regarded
as collective disputes and can’t go for the strike.

b) Section 13 of Bonus Act 2030 states that seventy percent of the residuary amount after
distribution of bonus from the allocated amount for the bonus pursuant to clause 5 of Bonus Act
shall be deposited with the Welfare Fund established in accordance with section 37 of the Labor
Act, 2048 and remaining thirty percent shall be deposited with the National Level Welfare Fund,
established by Government of Nepal for the interest of the employees of the enterprises. The
operation of the welfare funds as provided shall be in participation of employees as prescribed.

5. Answer the following questions: (2×5=10)


a) State the different types of membership issued by Nepal Chartered Accountant
council. Also, mention the qualifications of a person to hold such membership
pursuant to the Nepal Chartered Accountants Act, 2053.
b) What are the punishment provisions in the Nepal Chartered Accountants Act, 2053.
?
Answers:
a) Pursuant to section 16 (1) of the Nepal chartered Accountants Act 2053, there are two types of
membership which are issued by the Nepal chartered accountants council, chartered accountant
member and registered auditor member.
Pursuant to section 18 of the Nepal chartered Accountants Act 2053, the following persons are
disqualified to be a member of the ICAN
1) Persons not possessing qualification under sub section 2 & 3 of the section 16 of the Nepal
chartered accountants Act, 2053
2) Persons not attaining the age of 21 years of old
(36)
3) Insolvent person
4) Person punished by a competent court on the charge of committing crime of moral turpitude.
5) Person of unsound mind

b) Punishment provisions as provided in the section 41 of Nepal chartered Accountants Act 2053 are
as follows:
(1) A person, who carries out audit without obtaining a Certificate of Practice, pursuant to this Act,
shall be liable of punishment with a penalty of maximum two thousand rupees or with an
imprisonment for a maximum period of three months or with both.
(2) A person, who in contravention of Section 6 uses the name or the seal of the Institute or exercises
any type of authority bestowed to the Institute, shall be punished with a penalty of one thousand rupees
maximum on first conviction, and on any subsequent conviction thereafter, a maximum penalty of
five thousand rupees or imprisonment for a maximum period of six months or both.
Provided that this sub-section shall not apply to the organizations or university established under their
own legislation or the units within the Institute.
(3) A person, who has not obtained a Certificate of Practice and is proved to have signed any
document in capacity of the member holding Certificate of Practice, shall be liable to punishment with
a penalty up to two thousand rupees or imprisonment for a period of up to three months or both.
(4) A member, who commits any act contrary to the provisions of this Act or Regulations framed
under this Act other than the provisions of this section, shall be suspended for a maximum period of
five years and shall be liable of punishment with a maximum penalty of two thousand rupees or
imprisonment for a maximum period of three months or both.
(5) A complainant who lodges a complaint, without any reasonable cause to make complaint and it is
proved that the complaint was made with an intention to harass a member, shall be liable to
punishment with fine up to one thousand rupees.
(6) The complaint cases, except those to be heard under Section 14, lodged in the Council against any
member, pursuant to Section 35, shall be instituted in the concerned High Court.

6) Answer the following: (5×4=20)


a) Mention the classifications of Industries under section 3 of The Industrial
Enterprises Act, 2049?
b) Mr. A, a foreign nationals, has wished to establish a social organization for
undertaking social service. He has asked you about permission and affiliation with
the Social Welfare Council. Can a foreign national works within the Government
of Nepal by establishing non-governmental organization? If it so, provide him the
affiliation process with the council.
c) Mr. Anil gives a cheque to Mr. Bhola with "not negotiable" crossing. Mr.
Chandra steals the cheque from Mr. Bhola and delivers to Mr. Durga for
consideration. Durga takes the cheque without knowledge that cheque has
been stolen cheque. Suggest Mr. Durga referring Negotiable Instrument Act,
2034.
d) Mention various types of offers in regard to contract law.
e) Explain the provisions relating to ceiling on bonus payment to be made to the
employees as per the Bonus Act, 2030.
Answers:
a) Classifications of Industries under section 3 of The Industrial Enterprises Act 2049 are as
follows:
(37)
i) Manufacturing Industries
Which produces goods by utilizing or processing raw materials, semi processed materials,
by products or waste products or any other goods.
ii) Energy based Industries
Industries generating energy from water resources, wind, solar, coal, natural oil, gas, bio
gas or any other sources.
iii) Agro based Industries
Business mainly based on agriculture or forest products such as integrated sericulture and
silk production, horticulture and fruit processing, animal husbandry, dairy industry, poultry
farming, fishery, tea gardening and processing, vegetable seed farming, mushroom,
vegetable farming or vegetable processing, tissue culture, green house, bee keeping honey
production, rubber farming, floriculture and production and forestry related business such
as lease hold forest, agro forestry
iv) Mineral Industries
Mineral excavation or processing thereof.
v) Tourism Industries
Tourist lodging, Motel, Hotel, Restaurant, resort, Travel agency, skiing, gliding, water
rafting, cable car complex, pony trekking, Hot air ballooning, Para sailing, golf course,
Polo, Horse riding etc.
vi) Service Industries
Workshop, Printing press, Consultancy services, ginning and bailing business,
Cinematography, construction business, Public transportation business, Photography,
Hospital, Nursing home, Educational and training institution, Laboratory, Air services,
Cold storage etc.
vii) Construction Industries
Road, Bridge, Ropeway, Railway, Trolley bus, Tunnel, Flying bridge, and Industrial,
commercial or Residential complex construction and operation.

b) Permission and agreement is provided in section 12 Social Welfare Act 2049


are as follows:
1. Any foreign non-governmental organization if desires to work within the Nepal, before
starting the work shall submit an application to the Council for permission.
2. The council, after receiving and application pursuant to sub-section (1) may give
permission deciding within three months.
3. The permitted foreign non-governmental organization, pursuant to sub-section (2) before
operating the work within Nepal shall have to reach in an agreement with the Council.

Affiliation with the Council is provided in section 13.

(1) Social organizations and institutions willing to keep affiliation with the Council
shall have to submit an application as prescribed form.
(2) The organizations and institutions applying pursuant to sub-section (1) shall submit
and mention its Constitutions, names of executive committee members, their
occupations and addresses and the office where the organization or institution has
been registered and the date of the registration along with the application.
(3) After receiving the application pursuant to sub-section (1) if it deems to be affiliated
such institutions or organization with the Council, the Council shall issue the
certificate as prescribed form taking the fees as prescribed.
(4) The organization or institutions affiliated with the Council may keep out its
affiliation as prescribed.
(38)
In the given case, Mr. A can establish a non-governmental organization undertaking social
welfare work after taking permission and affiliation from the Social Welfare Council
fulfilling the terms and conditions of section 12 and 13 and other related provision.

c) "Not negotiable" crossing does not make negotiable instrument not transferable. Any instrument
with "not negotiable" crossing means that it is negotiable until its title is good.
Section 89 of Negotiable Instrument Act 2034 make provision regarding Cheque bearing "not
negotiable". A person taking a cheque crossed generally or specially bearing in either case the
words "not negotiable" shall not have, and shall not be capable of giving a better title to the cheque
than that which the person from whom he took it had.
There is general rule that holder in due course obtain better title than the transferor. However in
case of negotiable instrument having "Not negotiable" crossing does not obtain better title than the
transferor.
Principle of "Nemo Dat Quod Non Habet" which means "No one can transfer title better than he
himself had" is applicable in case of negotiable instrument having "Not negotiable" crossing.
So in the given case Mr. Durga is holder in due course, but as the negotiable instrument is having
"Not negotiable" crossing he cannot get better title than Mr. Chandra. As Mr. Chandra has no title
over the instrument being stolen by him and accordingly Mr. Durga also does not have title over
the instrument.
Person taking the Negotiable Instrument having "Not negotiable" crossing should inquire and
assure the title of the transferor over Negotiable Instrument.

d) Various types of offer under contract Act are as follows:


i) Expressed and Implied Offer:
Expressed offer is made by words spoken and written by the parties.

Implied offer is one which inferred from the conduct of a person or the circumstances of the
particular case.

ii) Specific and General Offer :


The word 'specific' says to someone or something expressly. An offer which is forwarded to
a particular person or that particular group or persons is known as specific offer. Person or
group of person are identified in this offer. The identified person or group of persons only
make an acceptance for the purpose of generating contract.

Other hand, an offer addressed to the world at large/public in general is known as a general
offer. Such an offer can be accepted by any person by fulfilling the terms of the offer with
notice of the offer. Contract is made to a person who come forward and make an
acceptance into the terms. In this offer receiver of offer is not identified so it is called
general offer.

iii) Counter Offer:

An offer presenting with an amendment or altering by a person to which original offer is


made is called counter offer. It can also called reverse offer. It amends or alters to first or
original offer from so called promiser. While counter offer is made it could not be called as
an acceptance.
iv) Cross offer/Identical Offer:
When two offers, similar in all respects, are made by one party to the other, in ignorance of
each other's offer, they are cross offers. Such offers are also called as identical offers. Such
offer constitutes a contract while one offer is made an acceptance.
(39)

v) Standing Offer :
There is a series of continuous offer is called standing offer. It means such sort of offer
stands for the specific period and promiser should provide or perform series of goods or
actions. A standing offer is in the nature of a tender where a person invites tender to another
to supply certain goods at a certain rate upto a fixed period. It is an open or continuing
offer. Such kind of offer does not become a binding contract as soon as it is accepted.

e) Section 7 of Bonus Act 2030, prescribe the ceiling on the distribution of


bonus.
(1) The management shall assess the percentage of bonus amount to be obtained by an
Employee in a Fiscal year. Whatever the percentage assessed by the management, the bonus to be
obtained by an employee shall not exceed the following amounts:
a) An amount equivalent to the salary or wage of Six months, to an employee, who obtains upto
Five Thousand Rupees as salary or wage.
b) An amount equivalent to the salary or wage of Four months to an employee, who obtains Five
thousand One Rupees to Fifteen Thousand Rupees as salary or wage.
c) An amount equivalent to the wage or salary of Three months to an employee who obtains more
than Fifteen Thousand rupees as salary or wage.

(2) The minimum bonus amount to be obtained under Clauses (b) and (c) of Sub-section (3) of this
section above shall not be less than the maximum bonus amount to be obtained under Clauses (a)
and (b) respectively as above.

7) Write short notes on the following: (2×5=10)


a) Certificate of Practice (COP) under Nepal Chartered Accountants Act, 2053
b) Pre-incorporation contracts.
Answers:
a) Section 2(j) of Nepal Chartered Accountants Act, 2053 defines the Certificate
of Practice (COP)
"Certificate of Practice (COP)" means the certificate issued pursuant to section 28, to render
accounting profession.
Section 28 of Nepal Chartered Accountants Act, 2053 states that
"Certificate of Practice (COP)"
(1)Member willing to carry out audit profession shall make an application, in a prescribed
format, for Certificate of Practice, along with the prescribed fees, to the Institute.
(2)The Council, prescribed that the applying member as per sub-section (1), has fulfilled all
conditions prescribed by the Council, shall provide a Certificate of Practice, in a prescribed
format, to such member.
(3)The Council shall ensure that the members observe or shall cause to observe conditions
prescribed for members holding Certificate of Practice may prescribe Code of Conduct for such
member.

b) Pre-incorporation contract:
Contracts entered into on behalf and for the company by the supposed or future directors
or any other persons with third persons before its incorporation is called pre-incorporation
contracts. A general rule is that pre-incorporation contracts are void and not binding to the
company.
Section 17 of Companies Act 2063 had made provision related to Pre-incorporation contract.
(40)
(1) A contract made prior to the incorporation of a company shall be a proposed
contract only, and such contract shall not be binding on the company.
(2) If, prior to the incorporation of a company, any person carries on any transaction or
borrowed moneys on behalf of the company, such person shall be personally liable for any
contract related with the transaction so carried on, subject to (3) below.
(3) If, within the time mentioned in any transaction or within the reasonable time after
the incorporation of a company, the company through its act, action or conduct, accepts
any act, action or borrowing done or made prior to the date of authorization to commence
its transactions or endorses such act or action, that transaction shall be binding on the
company and the other contracting party; and the person carrying out such act or action
shall be released from the personal liability to be borne pursuant to sub-section (2).
(4) Notwithstanding anything contained elsewhere in this Section, the consensus
agreement of a private company shall govern any contracts made prior to the incorporation
of such company.
(41)

Financial Management
(42)
Suggested
Roll No……………. Maximum Marks - 100
Total No. of Questions – 7 Total No. of Printed Pages –4
Time Allowed – 3 Hours
Marks
Attempt all questions.
Working notes should form part of the answer. Make assumptions wherever necessary.
1. ABC Ltd. is evaluating two investment projects independently with different
investment modality. Information for these projects are as follows:
Investment Analysis 1:
This is an investment in new machinery to produce a recently-developed product. The
cost of the machinery, which is payable immediately, is Rs. 1,500,000, and the scrap
value of the machinery at the end of four years, is expected to be Rs. 100,000.
Depreciation for tax purpose can be claimed on this machinery on a 25% reducing
balance basis. Information on future returns from the investment has been forecasted
to be as follows:
Year 1 2 3 4
Sales volume (units/year) 50,000 95,000 140,000 75,000
Selling price (Rs./unit) 25.00 24.00 23.00 23.00
Variable cost (Rs./unit) 10.00 11.00 12.00 12.50
Fixed costs (Rs./year) 105,000 115,000 125,000 125,000
This information must be adjusted to allow for selling price inflation of 4% per year
and variable cost inflation of 2.5% per year. Fixed costs, which are wholly
attributable to the project, have already been adjusted for inflation. ABC Ltd. pays
profit tax of 30% per year one year in arrears.
Investment Analysis 2:
ABC Ltd. plans to replace an existing machine and must choose between two
machines. Machine 1 has an initial cost of Rs. 200,000 and will have a scrap value of
Rs. 25,000 after four years. Machine 2 has an initial cost of Rs. 225,000 and will have
a scrap value of Rs. 50,000 after three years. Annual maintenance costs of the two
machines are as follows:
Year 1 2 3 4
Machine 1 (Rs./year) 25,000 29,000 32,000 35,000
Machine 2 (Rs./year) 15,000 20,000 25,000 -
Where relevant, all information relating to investment analysis 2 has already been
adjusted to include expected future inflation.
Taxation and depreciation allowances must be ignored in relation to Machine 1 and
Machine 2.
Other information:
ABC Ltd. has a nominal before-tax weighted average cost of capital of 12% and a
nominal after-tax weighted average cost of capital of 7%.
Required: (10+6+4=20)
a) Calculate the net present value of investment analysis 1 and comment on whether
this project is financially acceptable to ABC Ltd.
b) Calculate the equivalent annual costs of Machine 1 and Machine 2 in investment
analysis 2, and discuss which machine should be purchased.
c) Critically discuss the use of sensitivity analysis and probability analysis as ways
of including risk in the investment appraisal process, referring in your answer to
the relative effectiveness of each method.
(43)
Answer
(a) Calculation of net present value (NPV) of Investment Analysis 1:
As nominal after-tax cash flows are to be discounted, the nominal after-tax weighted average cost of
capital of 7% must be used.
(Rs.’000)
Year 1 2 3 4 5
Sales revenue (WN1) 1,300 2,466 3,622 2,018
Variable costs (WN2) (513) (1,098) (1,809) (1,035)
Contribution 787 1,368 1,813 983
Fixed costs (105) (115) (125) (125)
Taxable cash flow 682 1,253 1,688 858
Tax liabilities @30% (205) (376) (506) (257)
Depreciation tax benefits (WN3) 113 84 63 160
After-tax cash flow 682 1,161 1,396 415 (97)
Scrap value - - - 100 -
Net cash flow 682 1,161 1,396 515 (97)
PVIF at 7% 0·935 0·873 0·816 0·763 0·713
Present values 638 1,014 1,139 393 (69)

Rs'.000
Present value of cash inflows 3,115
Cost of machine (1,500)
NPV 1,615

Investment 1 has a positive NPV of Rs.1,615,000, so it is financially acceptable to ABC Ltd.


Workings:
1. Sales revenue
Year 1 2 3 4
Selling price (Rs./unit) 25.00 24.00 23.00 23.00
Inflated selling price (Rs./unit)[sp×(1+i)n] 26.00 25.96 25.87 26.91
Sales volume (units/year) 50,000 95,000 140,000 75,000
Sales revenue (Rs./year) 1,300,000 2,466,200 3,621,800 2,018,250
2. Variable cost
Year 1 2 3 4
Variable cost (Rs./unit) 10.00 11.00 12.00 12.50
Inflated variable cost (Rs./unit) [vc×(1+i)n] 10.25 11.56 12.92 13.80
Sales volume (units/year) 50,000 95,000 140,000 75,000
Variable costs (Rs./year) 512,500 1,098,200 1,808,800 1,035,000
3. Depreciation tax benefits
Year Capital allowance Tax benefit Year benefit received
1 1,500,000 x 0.25 = Rs.375,000 375,000 x 0.3 = Rs.112,500 2
2 1,125,000 x 0.25 = Rs.281,250 281,250 x 0.3 = Rs.84,375 3
3 843,750 x 0·25 = Rs.210,938 210,938 x 0.3 = Rs.63,281 4
4 Rs.532,812* 532,812 x 0.3 = Rs.159,844 5

*843,750 – 210,938 – 100,000 = Rs.532,812


Alternative calculation of net cash flow is acceptable.
(b) Calculation of equivalent annual cost for machine 1:
Since taxation and capital allowances are to be ignored, and where relevant all information relating
to project 2 has already been adjusted to include future inflation, the correct discount rate to use
here is the nominal before-tax weighted average cost of capital of 12%.
Year 0 1 2 3 4
Maintenance costs (Rs.) --- (25,000) (29,000) (32,000) (35,000)
(44)
Investment and scrap (Rs.) (200,000) - - - 25,000
Net cash flow (Rs.) (200,000) (25,000) (29,000) (32,000) (10,000)
Discount at 12% 1.000 0.893 0.797 0.712 0.636
Present values (200,000) (22,325) (23,113) (22,784) (6,360)
Total present value of cash flows Rs.274,582
Cumulative present value factor 3.038
Equivalent annual cost = 274,582/3.038
= Rs.90,382
Calculation of equivalent annual cost for machine 2:
Year 0 1 2 3
Maintenance costs (Rs.) - (15,000) (20,000) (25,000)
Investment and scrap (Rs.) (225,000) - - 50,000
Net cash flow (Rs.) (225,000) (15,000) (20,000) 25,000
Discount at 12% 1.000 0.893 0.797 0.712
Present values (225,000) (13,395) (15,940) 17,800

Total present value of cash flows Rs.2,36,535


Cumulative present value factor 2·402
Equivalent annual cost = 2,36,535/2·402
= Rs.98,474
The machine with the lowest equivalent annual cost should be purchased and calculation shows
this to be Machine 1.
(c) Within the context of investment appraisal, risk relates to the variability of returns and so it can be
quantified, for example by forecasting the probabilities related to future cash flows. From this point
of view, risk can be differentiated from uncertainty, which cannot be quantified. Uncertainty can
be said to increase with project life, while risk increases with the variability of returns.
It is commonly said that risk can be included in the investment appraisal process by using
sensitivity analysis, which determines the effect on project net present value of a change in
individual project variables. The analysis highlights the project variable to which the project net
present value is most sensitive in relative terms. However, since sensitivity analysis changes only
one variable at a time, it ignores interrelationships between project variables.
While sensitivity analysis can indicate the key or critical variable, it does not indicate the
likelihood of a change in the future value of this variable, i.e. sensitivity analysis does not indicate
the probability of a change in the future value of the key or critical variable. For this reason, given
the earlier comments on risk and uncertainty, it can be said that sensitivity analysis is not a method
of including risk in the investment appraisal process.
Probability analysis, as its name implies, attaches probabilities to the expected future cash flows of
an investment project and uses these to calculate the expected net present value (ENPV). The
ENPV is the average NPV that would be expected to occur if an investment project could be
repeated a large number of times. The ENPV can also be seen as the mean or expected value of an
NPV probability distribution. Given the earlier discussion of risk and uncertainty, it is clear that
probability analysis is a way of including a consideration of risk in the investment appraisal
process. It is certainly a more effective way of considering the risk of investment projects than
sensitivity analysis.
A weakness of probability analysis, however, lies in the difficulty of estimating the probabilities
that are to be attached to expected future cash flows. While these probabilities can be based on
expert judgement and previous experience of similar investment projects, there remains an element
of subjectivity which cannot be escaped.
2.
a) A public company is commencing a new project for manufacturing of a plastic
component. The following cost information has been ascertained for annual
production of 12,000 units at full capacity:
(45)
Particulars Cost per unit (Rs.)
Materials 40
Direct labour and variable expenses 20
Fixed manufacturing expenses 6
Depreciation 10
Fixed administrative expenses 4
80
Selling price per unit is expected to be Rs. 96 and selling expenses Rs. 5 per unit,
80% of which is variable.
In the first two years of operations, production and sales are expected to be as
follows:
Year Production (No. of units) Sales (No. of units)
1 6,000 5,000
2 9,000 8,500
Following additional information is available:
Stock of materials : 2.25 months’ average consumption
Work in process : Nil
Debtors : 1 month’s average sales
Cash Balance : Rs. 10,000
Creditors for supply of materials : 1 month’s average purchase during the year
Creditors for expenses : 1 month’s average of all expenses during the year
Required: 10
Prepare projected profit and loss account and statement of working capital
requirement for the two years.
b) Nepal Ltd. provides you the following information:
Gross profit ratio 40%, Net profit (after tax) ratio 12%, Operating profit ratio
30%, 15% Debt-Equity ratio 2:1, Tax rate 50%, Shareholder’s fund Rs. 400,000.
Required: 5
Calculate (i) Gross profit (ii) Operating expenses (iii) Interest coverage ratio, (iv)
Return on capital employed, (v) Return on shareholders’ funds.
Answer
a)
i) Projected Profit and Loss Account

Particulars Year 1 Year 2


Production (Units) 6,000 9,000
Sales (Units) 5,000 8,500
Rs. Rs.
Sales Revenue (a) (sales units × Rs. 96) 480,000 816,000

Cost of Production:
Materials (production unit× Rs.40) 240,000 360,000
Direct labour and variable expenses (@ Rs. 20 p.u.) 120,000 180,000
Fixed Manufacturing cost ( 12000 units × Rs. 6) 72,000 72,000
Depreciation ( 12000 units × Rs. 10) 120,000 120,000
Fixed administration expenses (12000 units × Rs. 4) 48,000 48,000
Cost of Production: 600,000 780,000

Add: Opening stock of finished goods ( Yr 1 nil ; Yr-


2 1000 units) (@ Rs. 600,000/6,000 units) 0 100,000
600,000 880,000
(46)

Less: Closing stock of finished goods ( Yr 1 - 1000 *


units, Yr 2 -1500 units) (@production cost per unit) 100,000 130,000
Cost of goods sold 500,000 750,000
Add: Selling expenses
- Variable @ Rs. 4 p.u. 20,000 34,000
-Fixed ( 12000 units @ Re. 1) 12,000 12,000
Cost of sales (b ) 532,000 796,000

Profit / ( Loss) (a- b) -52,000 20,000

*Alternative
Average cost method can also used
8,80,000 ×1,500
9,000+1,000
=1,32,000

ii) Calculation of Creditors for Supply of Materials


Particulars Year 1 (Rs.) Year 2 (Rs.)
Materials consumed 240,000 360,000
Add: Closing stock (2.25 month consumption) 45,000 67,500
285,000 427,500
Less: Opening Stock - 45,000
Purchases 285,000 382,500
Average purchase per month (Creditors) 23,750 31,875
iii) Calculation of creditors for Expenses
Particulars Year 1 (Rs.) Year 2 (Rs.)
Total direct labour, manufacturing,
administration and selling expenses for the
year 272,000 346,000

Average per month 22,667 28,833

iv ) Computation of Projected Statement of Working Capital Requirement


Particulars Year 1 (Rs.) Year 2 (Rs.)
Current Assets:
Stock of materials (2.25months' average
consumption) 45,000 67,500
Stock of Finished goods 100,000 130,000
Debtors (one month average sales) 40,000 68,000
Cash 10,000 10,000
Total (a) 195,000 275,500

Current Liabilities:
Creditors for supply of materials 23,750 31,875
Creditors for expenses 22,667 28,833
Total (b) 46,417 60,708

Estimated Working Capital (a- b) 148,583 214,792


b)
(47)
15% Debt = Rs. 400,000 × 2 = Rs. 800,000
Interest on Debt = 15% × 800000 = Rs. 120,000

Assume the sales be x;


Sales X
Less: Cost of Goods Sold 0.6x
Gross Profit 0.4x
Less: operating expenses 0.1x
Operating profit @ 30% 0.3x
Less: Interest on long term debt 120,000
Profit before tax 0.3x-Rs 120,000
Less: Tax 50%(0.3x-120000)
Profit after tax 0.15x-60,000
Net profit ratio = Profit after tax/Net Sales x 100%
12% = (0.15x-60,000)/x
0.15x-0.12x = 60,000
x = 60,000/0.03
Sales = Rs. 20,00,000
Gross Profit = 40% of Rs 20,00,000
= Rs 800,000
Operating Expenses = 10% of Rs 20,00,000 = Rs 200,000
Interest coverage ratio = Profit before interest and tax/Interest on long term debt
=(0.3×2,000,000)/120,000 = 5 times
Return on Capital Employed=Profit before interest and tax/Capital employed x 100%
=30% of 20,00,000 X 100 = 50%
12,00,000
Return on Shareholders fund= Profit after interest and tax/Shareholders fund X
100%
= Rs. 240,000/400,000 x 100 = 60%
Note:
Profit after interest and Tax = 50%[0.3x20,00,000-120,000)
=50%[600,000-120,000]
= 240,000
3.
a) Consider the following information of a manufacturing company:
Installed capacity 4,000 units
Actual production and sales 75% of the capacity
Selling price Rs. 30 per unit
Variable cost Rs. 15 per unit
Fixed cost:
Under situation I: Rs. 15,000
Under situation II: Rs. 20,000
Capital structure:
Particulars Financial Plan
A (Rs.) B (Rs.)
Equity 10,000 15,000
20% Debt 10,000 5,000
20,000 20,000
Required: 8
Calculate the operating leverage, financial leverage and combined leverage under
situation I and II and financial plan A and B.
(48)
b) RP Ltd. has annual sales of Rs. 400 crores. It sells 80 percent of its products on a
60-day credit. Its average collection period is 80 days. The company’s bad debts,
based on the past experience, could be estimated as 0.9 percent of credit sales.
The company’s annual cost of administering credit sales is 0.46875%. It is
possible to avoid Rs. 4,000,000 of these costs if credit administration is
transferred by the company to a factor. The factor will charge 1.75 percent non-
recourse commission for his services. He can extend advance against receivables
to the company at an interest rate 16.5 percent after withholding 10 percent as
reserve.
Required: 7
Should the company hire services of the factors? Assume 15% Required Rate of
Return and 360 days in a year.
Answer
a)
Operating Leverage:
Particulars Situation-I (Rs.) Situation-II (Rs.)
Sales (3,000 units@Rs.30 per unit) 90,000 90,000
Less: Variable Costs @ Rs.15 per (45,000) (45,000)
unit
Contribution 45,000 45,000
Less: Fixed Cost 15,000 20,000
EBIT 30,000 25,000
Operating Leverage 45,000/30,000 45,000/25,000
(Contribution/EBIT)
=1.5 =1.8
Financial Leverage:
Situation I
Plan A (Rs.) Plan B (Rs.)
EBIT 30,000 30,000
Less: Interest on Debt (2,000) (1,000)
EBT 28,000 29,000
Financial Leverage 30,000/28,000 30,000/29,000
(EBIT/EBT)
= 1.07 = 1.03
Situation II
EBIT 25,000 25,000
Less: Interest on Debt (2,000) (1,000)
EBT 23,000 24,000
Financial Leverage 25,000/23,000 25,000/24,000
(EBIT/EBT)
= 1.09 = 1.04
Combined Leverage:
Plan A (Rs.) Plan B (Rs.)
Situation I 1.5 x 1.07 = 1.61 1.5 x 1.03 = 1.55
Situation II 1.8 x 1.09 = 1.96 1.8 x 1.04 = 1.87
b) Calculation of Factoring Commission, Interest charged and Advance Granted
Particulars Amount (Rs
in Crores)
Average Level of Receivable ( 320 X 80 )/360 71.11
Less: Factoring Commission (1.75% X 71.11) 1.24
Less: Factoring Reserve (10% of 71.11) 7.11
Eligible amount of advance 62.76
Less: Interest charged on advance (62.76 X 16.5%X80/360) 2.30
(49)
Actual amount received from factor 60.46
STATEMENT SHOWING THE EVALUATION OF FACTORING ARRANGEMENT
Particulars Amount (Rs in Crores)
A. Annual Benefits of Factoring to the firm:
Credit administration cost avoided 0.40
Bad Debt avoided (0.9% of 320 crores) 2.88
Total 3.28
B. Annual Cost of Factoring to the firm
Factoring Commission (1.24 X 360)/80 5.60
Interest Charged on advance granted (2.3X360/80) 10.35
Total 15.95
C. Net annual cost of Factoring to the firm: 12.67
Rate of Effective cost = Net Annual Cost/Actual 20.96%
advance granted = 12.67/60.46
Or
12.67 ×100% =20.19%
*
62.76
If computed before interest it is acceptable.

Recommendation: The company should continue with its in house credit


management since its Rate of Effective Cost is less than the cost of other
sources of financing (i.e. 15%)

Working Notes:
i. Credit Sales = 400 crores x 80% = 320 crores

4.
a) The equity beta of Fence Co. is 0.9 and the company has issued 10 million
ordinary shares. The market value of each ordinary share is Rs. 7.50. The
company is also financed by 7% bonds with a nominal value of Rs. 100 per bond,
which will be redeemed in seven years’ time at nominal value. The bonds have a
total nominal value of Rs. 14 million. Interest on the bonds has just been paid and
the current market value of each bond is Rs. 107.14.
Fence Co. plans to invest in a project which is different to its existing business
operations and has identified a company, Hex Co. as the project. The equity beta
of Hex Co. is 1.2 and the company has an equity market value of Rs. 54 million.
The market value of the debt of Hex Co. is Rs. 12 million.
The risk-free rate of return is 4% per year and the average return on the stock
market is 11% per year. Both companies pay corporation tax at a rate of 20% per
year.
Required: (4+4=8)
i) Calculate the current weighted average cost of capital of Fence Co.
ii) Calculate a cost of equity which could be used in appraising the new project.
b) Bhardhwaj Trader Ltd. is a growing supplier of office materials. Analysts project
the following free cash flow during the next 3 years of operation of the company,
after which the free cash flow is expected to grow at a constant rate of 7%.
Year 1 2 3
Free cash flow (Rs. in millions) (20) 30 40
The firm's weighted average cost of capital is 13%.
(50)
Required: (3+2+2=7)
rd
i) What is the terminal value of free cash flows after 3 year?
ii) What is the value of the firm today?
iii) If the company has Rs. 100 million in debt and 10 million ordinary shares
outstanding, what is the price per share?
Answer
a) i) Calculation of WACC
WACC = (Ke ×We) + (Kd × Wd )
The current cost of equity can be calculated using the capital asset pricing model.

Equity or market risk premium = market rate of return – risk free rate
=11% – 4 %
= 7%
Cost of equity (Ke) = 4% + (0·9 x 7%)
= 4% + 6·3%
= 10·3%
After-tax cost of debt using trial and error:
After-tax interest payment = 100 x 0·07 x (1 – 0·2) = Rs.5·60 per bond.

Year Cash flow Rs. 5% discount PV (Rs.) 4% discount PV (Rs.)


0 market value (107·14) 1·000 (107·14) 1·000 (107·14)
1–7 interest 5·60 5·786 32·40 6·002 33·61
7 redemption 100·00 0·711 71·10 0·760 76·00
* (3·64) 2·47
After-tax cost of debt = 4 + ((5 – 4) x 2·47)/(2·47 + 3·64)
= 4 + 0·4
= 4·4%
* OR
Kd = A.I(1-t) + CO – CI
n______
CO + CI
2
= 7(1-0.2) + 100 – 107.14
7______
100+ 107.14
2

= 5.6 – 1.02
103.57
= 4.42 %

Market value of equity = 10,000,000 x 7·50


= Rs.75,000,000
Market value of Fence Co debt = 14,000,000 x 107·14/100
= Rs.15,000,000
Total market value of company = 75,000,000 + 15,000,000
= Rs.90,000,000
WACC = ((10·3 x 75/90) + (4·4 x 15/90)
= 8.58 % +0.73 %
= 9.31 %
(51)
(ii) Since the investment project is different to business operations, its business risk is different to
that of existing operations. A cost of equity for appraising it can, therefore, be found using the
capital asset pricing model.
Ungearing Hex company equity beta
Asset beta = 1·2 x 54/(54 + (12 x 0·8))
= 1·2 x 54/63·6
= 1·019
Regearing asset beta
Market value of debt = Rs.15m (calculated in part (a))
Regeared asset beta = 1·019 x (75 + (15 x 0·8))/75
= 1·019 x 87/75
= 1·182

Using the CAPM


Equity or market risk premium = 11% – 4%
= 7%
Cost of equity = 4 + (1·182 x 7)
= 4 + 8·3
= 12·3%

b)
i) Terminal Value of Free Cash Flows after 3rd year:
Free Cash Flow of 3rd year (1+g)
=
(WACC-g)
40(1+0.07)
=
0.13-0.07

= Rs. 713.33 Million


ii) Calculation of Value of the Firm Today
Year FCF/Terminal Value (Rs. in millions) PVIF @ 13% PV (Rs. in
millions)
1 (20) 0.8850 (17.70)
2 30 0.7831 23.493
3 40 0.6931 27.724
3 713.33 0.6931 494.41
Value of the Firm Today 527.927
iii) Calculation of Price Per Share
Value of Common
Equity = Value of Firm Today - Value of Debt

= 527.927 Million - 100 Million

= Rs. 427.927 Million

Value of Equity
Price Per Share =
No. of Equity Share

Rs. 427.927 Million


=
10 Million

= Rs. 42.7927
(52)
5.
a) TL Ltd. provides you with following figures:
Particulars Rs.
Profit before interest and tax 260,000
Less: Interest on debentures @ 12% 60,000
Profit before tax 200,000
Less: Income tax @ 50% 100,000
Profit after tax 100,000
Number of equity shares (of Rs. 10 each) 40,000
EPS (Earning per share) (Rs.) 2.50
Market price per share (Rs.) 25
PE Ratio 10
The company has undistributed reserves of Rs. 600,000. The company needs Rs.
200,000 for expansion. This amount will earn at the same rate as funds already
employed. A debt equity ratio Debt/ (Debt+ Equity) of more than 35% will push
the P/E Ratio of the company down to 8 and raise the interest rate on additional
amount borrowed to 14%.
Required: 5
i) What will be the price of the share, if the additional fund are raised as debt?
ii) What will be the price of the share, if the amount is raised by issuing equity
shares?
b) Mr. X wants to get her daughter admitted into a medical college after 15 years
from now. He will require total Rs. 2,500,000 to get admission into the college.
For this, he has identified a fund, which pays interest at 9% p.a.
Required: 5
Determine the amount to be invested:
i) If Mr. X decides to make annual payment into the fund at the end of each year.
ii) If Mr. X decides to invest a lump sum in the fund at the end of the year.
iii) If Mr. X decides to make annual payment into the fund at the beginning of
each year.
[FVIF/CVF(15, 0.09) = 3.642, FVIFA/CVFA(15, 0.09) = 29.361]
c) The following data relat to two securities, A and B:
A B
Expected Return 22% 17%
Beta Factor 1.5 0.7
Risk Free Interest rate is 10% and Return on Market is 18%.
Required: 5
Find out whether the securities A and B are correctly priced?
Answer
a)
Computation of existing capital and return on capital employed: -
Rs.
Equity share Capital 40,000 x 10 4,00,000
12% debentures 60 000 /12% 5,00,000
Undistributed Reserves 6,00,000
Existing Capital 15,00,000

Return on Capital employed 2,60,000/ 15,00,000 x 100%


= 17.33%
(53)
Calculation of Debt Equity Ratio
( Amount in Rs.)
Plan I (Debt Plan) Plan II (Equity Plan)
Existing Equity (Capital + 10,00,000 10,00,000
Reserve)
Additional equity - 2,00,000
Total equity (A) 10,00,000 12,00,000
Existing Debt 5,00,000 5,00,000
Additional Debt 2,00,000 -
Total Debt (B) 7,00,000 5,00,000
DE Ratio = Debt/( Debt + equity) 7 00 000/1,7000,000 500,000/(1,700,000 x
x 100 100
= 41.18% = 29.41%
Applicable P/E Ratio 8 10
Computation of probable market price of share after expansion:-
(i) ( ii ) (Rs.)
Plan I (Debt) Plan II
(Equity)
EBIT (17,00,000 x 17.33%) 2,94,610 2,94,610
2. Interest (Existing + Additional) 88,000 60,000
3. PBT (1-2) 2,06,610 2,34,610
4. Tax @ 50% 1,03,305 1,17,305
5. PAT (3-4) 1,03,305 1,17,305
6. Preference Dividend - -
7. Equity Earnings (5-6) 1,03,305 1,17,305
8. No. of equity shares (Existing + 40,000 48,000
Additional)
9. EPS 2.58 2.44
10. P/E Ratio 8 10
11. Market Price [= EPS × P/E Ratio] 20.64 24.40
b)
(i) To get Rs. 25,00,000 after 15 years from now, Mr. X needs to deposit an amount at the end of
each year, which gets accumulated @ 9% p.a. for 15 years to become an amount to Rs. 25,00,000.
Future Value = Annual payment x FVIFAn, I
Annual payment = future value /FVIFAn, I = 25,00,000/29.361 = Rs.85,146.96 p.a.
(ii) To get Rs. 25,00,000 after 14, one years from now. Mr. X needs to deposit a lump sum payment
to the fund which gets accumulated @ 9% p.a. for 14 years to become an amount to Rs. 25,00,000.
Future Value = Amount x P.V Single sum factor 9% for 14 years.
Amount = 25,00,000 × 0.2992= Rs. 748,000
(iii) To get Rs. 25,00,000 after 15 years from now, Mr. X needs to deposit an amount at the
beginning of each year which gets accumulated @ 9% p.a. for 15 years to become an amount to
Rs. 25,00,000.
Future Value = Annual payment x (FVIFAn, i) x (1+i)
Annual Payment = 25,00,000/ (29.361 x 1.09) = Rs. 78, 117.68 p.a.
c)
Calculation of return under CAPM
(54)
Company A=Rf+B(Rm-Rf)
=10+1.5(18-10)
=22%
Company B=Rf+B(Rm-Rf)
=10+0.7(18-10)
=15.6%
Security E(R) Expected Return Return under CAPM Position
A 22% 22% Correctly Priced
B 17% 15.6% Under Priced

The return from security A exactly equal to the calculated return under CAPM hence it is correctly
priced securities.
The return from security B is better than the return under CAPM. It indicates a favorable position;
i.e. the security is currently traded at underpriced position.
6. Write short note/ answer on: (4×2.5=10)
a) Venture capital financing methods
b) Conflict in project choice using PI and NPV criterion
c) Application supported by blocked account (ASBA)
d) ‘Loan syndication is one of the project finance services.’ Discuss.
Answer
a)
Venture Capital financing refers to financing of high risk ventures promoted by new qualified
entrepreneurs who require funds to give shape to their ideas. Here, a financier (called venture
capitalist) invest in the equity or debt of an entrepreneur (promoter) undertaking who has a potentially
successful business idea but does not have desired track record or financing backing. Generally,
venture capital funding is associated with heavy initial investment business like energy conservation,
quality up gradation or with sunrise sector like information technology.

Methods of venture capital financing

i) Equity financing: The investor’s contribution does not exceed 49% of the total equity capital of
the undertaking. Hence, the effective control and ownership remains with the entrepreneur.
ii) Conditional Loan: A conditional loan is repayable in the form of royalty after the venture is able
to generate sales. No interest is paid on such loans. Sometimes, the entrepreneur has a choice of
paying high rate of interest instead of royalty on sales once the activity becomes commercially
sound.
iii) Income note: It is a hybrid type of finance, which combines the features of both conventional
loan and conditional loan. The entrepreneur has to pay both interest and royalty on sales but at
substantially low rates.
iv) Participating debentures: Interest on such debentures is payable at three different rates based on
the phase of operations as under.
b)
The conflict in project choice using PI and NPV criterion arises in case of mutual exclusive
projects of unequal investment size having different net present values because NPV gives ranking
on the basis of absolute amount whereas PI gives ranking on the basis of ratio. In such a case,
mutual exclusive project having highest NPV should be selected since it would increase the firm’s
wealth if the project is accepted which is consistent with the wealth maximization objective of the
financial management.
c)
Application Supported by Blocked Amount (ASBA) refers to an application mechanism for
subscribing to initial public offers (IPO) or to Further public offering (FPO). The system which
ensures that the applicant's money remains in his/her bank account till the shares are allotted. The
applicant‘s bank account will only be debited post allotment of the shares. The amount debited
(55)
depends on the shares/MF units allotted to the applicant and the remaining amount, if any, is freed
for use. SEBON has introduced ASBA system from Magh 2073. However, the option, though
available, is not mandatory for the investors. They can continue to make applications through the
existing facility of applying with cash/cheque. Prior to ASBA while applying in an IPO or FPO,
the entire amount had to be paid up front with the applications. In case there was no allotment or
part-allotment, the amount would get refunded to the applicant after 45-75 days. This was a lost
opportunity as the funds did not earn any interest during the period either. However, if the
application is made via ASBA, the funds stay blocked in the investor‘s account, but continue to
earn the interest. Further, operational hassle of collecting and or depositing refund slip can be
avoided.
d)
Loan syndication involves obtaining commitment for term loans from the financial institutions and
banks to finance the project. Basically it refers to the services rendered by merchant bankers in
arranging and procuring credit from financial institutions, banks and other lending and investment
organisation or financing the client project cost or working capital requirements.
Loan syndication is in fact a tie up of term loans from the different financial institutions. The
process of loan syndication involves various formalities such as:
i. Preparation of project details,
ii. Preparation of loan application,
iii. Selection of financial institutions for loan syndication,
iv. Issue of sanction letter of intent from the financial institutions,
v. Compliance of terms and conditions for availing of the loan,
vi. Documentation, and
vii. Disbursement of the loan.
7. Distinguish between: (4×2.5=10)
a) Growth firm and Declining firm for relevance of dividend
b) Permanent and Temporary working capital
c) IPO and FPO
d) Risk and Uncertainty
Answer
a)
According to the relevance theory of dividend, dividends are relevant and the amount of dividend affects
the value of the firm. Walter, Gorden and others propounded that dividend decisions are relevant in
influencing the value of the firm.

Growth Firm

In growth firms internal rate of return is greater than the normal rate (r>k). Therefore, r/k factor will be
greater than 1. Such firms must reinvest retained earnings since existing alternative investments offer a
lower return than the firm is able to secure. Each rupee of retained earnings will have a weighting in
Walter`s formula than a comparable rupee of dividend.

Thus, large the firm retains, higher the value of the firm. Optimum dividend payout ratio for such a firm
will be zero.

Declining Firm

Firms which earn on their investments less than the minimum rate required by investments are designated
as declining firms. The management of such firms would like to distribute its earnings to the stockholders
so that they may either spend it or invest elsewhere to earn higher return than earned by the declining
firms. Under such a situation each rupee of retained earnings will receive lower weight than dividends and
market value of the firm will tend to be maximum when it does not retain earnings at all.
b)
Permanent Working Capital
(56)
It refers to a certain minimum level of current assets, which is essential for the firm to carry on
its business irrespective of the level of operations. This is the irreducible minimum amount
necessary for maintaining the circulation of the current assets. This minimum level of investment
in current assets is permanently locked up in business and is, therefore, referred to as permanent
or fixed or hardcore working capital. It is permanent in the same way as investment in firm’s
fixed assets is. This amount of working capital should be financed with long term funds.
Temporary Working Capital.
It refers to the amount of working capital over and above the fixed minimum amount of working
capital, which is required to meet seasonal and other temporary requirements. It may keep on
fluctuating from period to period depend upon the several factors. It is also called fluctuating or
variable or seasonal working capital.
c)
An initial public offering (IPO) is the first time that the stock of a company is offered to a large
public. IPOs are often issued by smaller, younger companies seeking capital to expand, but they
can also be done by large privately owned companies looking to become publicly traded. A
further public offer (FPO) is an issuing of shares to investors by a public company that is already
listed on an exchange. An FPO is essentially a stock issue of supplementary shares made by a
company that is already publicly listed and has gone through the IPO process.
The main differences are as follows:
 A company makes an IPO for raising money and an FPO for adding to the initial public
offerings or for managing capital structure.
 IPO is the first sale whereas the FPO is the second sale for expanding businesses.
 IPOs are risky investments as an individual investor cannot predict what will happen to
the initial trading in the coming days. In the case of FPOs, the risk is lower as an investor
already has an idea about the investment and future growth of the company.
 IPOs are more profitable than FPOs.
d)
In common parlance, the terms "Risk" and "Uncertainty" have synonymous meaning. However, they
differ from each other:
Risk may be defined as “the chance of future loss that can be foreseen”. In other words, in case of
risk an estimate can be made about the degree of happening of the loss. This is usually done by
assigning probabilities to the risk on the basis of past data and the probable trends.
Whereas uncertainty may be defined as “the unforeseen chance for future loss or damages.” In
case of uncertainty since the firm cannot anticipate the future loss and hence it cannot directly
deal with it in its planning process, as is possible in the case of risk
(57)

Cost and Management Accounting


(58)
Suggested
Roll No……………. Maximum Marks - 100
Total No. of Questions: 6 Total No. of Printed Pages - 16
Time Allowed - 3 Hours
Marks
All questions are compulsory. Working notes should form part of the answer.
Make assumptions wherever necessary.
1. In a chemical manufacturing company, three products A, B and C emerge at a single
split off stage in department P. Product A is further processed in department Q, product
B in department R and product C in department S. There is no loss in further Processing
of any of the three products. The cost data for a month are as under:

Cost of raw materials introduced in department P Rs. 12,68,800


Departments Direct Wages (Rs )
P 3,84,000
Q 96,000
R 64,000
S 36,000

Factory overheads of Rs. 4,64,000 are to be apportioned to the departments on direct


wage basis.
During the month under reference, the company sold all three products after processing
them further as under:

Products A B C
Output sold (kg.) 44,000 40,000 20,000
Selling Price per kg. (Rs.) 32 24 16

There is no opening or closing stocks. If these products were sold at the split off stage,
that is, without further processing, the selling prices would have been Rs. 20, Rs. 22 and
Rs. 10 each per kg. respectively for A, B and C.
Required: (6+9+3+2=20)
a) Prepare a statement showing the apportionment of joint costs to joint products.
b) Present a statement showing product-wise and total profit for the month under
reference as per the company’s current processing policy.
c) What processing decision should have been taken to improve the profitability of the
company?
d) Calculate the product-wise and total profit arising from your recommendation in (c)
above.
Answer:
a) Statement showing the apportionment of joint costs to joint products

Products
A B C Total
Output sold Kg.: (I) 44,000 40,000 20,000
Selling price per kg. at split 20 22 10
(59)
off (Rs. ): (II)
Sales value at split off
8,80,000 8,80,000 2,00,000 19,60,000
(Rs.): (I) x (II)
Joint costs (costs incurred
8,80,000 8,80,000 2,00,000 19,60,000
in department
P (Rs.)
(apportioned on the basis of
sales value at the
point of split off) i.e.
(22:22:5) (Working Note
1)

b) Statement showing product-wise and total profit for the month under reference
(as per the company’s current processing policy)

Products
A B C Total
Output (kg.) : (a) 44,000 40,000 20,000
Selling price per kg. after
32 24 16
further processing (Rs. ): (b)
Sales value after further 14,08,000 9,60,000 3,20,000 26,88,000
processing (Rs. ).:(c) = {(a) x
(b)}
Joint costs (Rs. ): (d) 8,80,000 8,80,000 2,00,000 19,60,000
Further processing costs (Rs.):
(e)
(Working Note 2) 1,72,800 1,15,200 64,800 3,52,800
Total costs (Rs.): (f) = [(d) +
10,52,800 9,95,200 2,64,800 23,12,800
(e)}
Profit/ (Loss) (Rs.): [(c))– (f)} 3,55,200 (35,200) 55,200 3,75,200

Alternatively:

Incremental sales revenue


5,28,000 80,000 1,20,000
(Rs. )
(44,000 units (40,000 units (20,000 units
x Rs. 12) x Rs. 2) x Rs. 6)
Less: Further processing
costs (Rs. )
[Refer to Working Note 2 (ii)] 1,72,800 1,15,200 64,800
Incremental net profit / (loss) 3,55,200 (35,200) 55,200

c) Processing decision to improve the profitability of the company.


44,000 units of product A and 20,000 units of product C should be further processed
because the incremental sales revenue generated after further processing is more
than the further processing costs incurred. 40,000 units of product B should be sold
at the point of-split off because the incremental revenue generated after further
processing is less than the further processing costs.

d) The product wise and total profit arising from the recommendation in (c) above is
as follows:
(60)

Product A B C Total
Profit (Rs. ) 3,55,200 - 55,200 4,10,400

Working Notes:

1. Statement of department-wise costs

P Q R S
(Rs.) (Rs.) (Rs.) (Rs.)
Raw materials 12,68,800
Wages 3,84,000 96,000 64,000 36,000
Overheads 3,07,200 76,800 51,200 28,800
(Apportioned on basis of the
departmental direct wages i.e.
96:24:16:9)
Total Cost 19,60,000 1,72,800 1,15,200 64,800

2. Joint costs and further processing costs


(i) Costs incurred in the department P are joint costs of products A, B and C
and are equal to Rs. 19,60,000.
(ii) Costs incurred in the departments Q, R and S are further processing costs of
products A, B and C respectively. Further processing costs of products A, B
and C thus are Rs. 1,72,800; Rs. 1,15,200 and Rs. 64,800 respectively.

2.
a) Adarsh Mahila Udhyog which has been regularly producing and marketing a very
popular washing powder named ‘Ujyalo’ intends to present its budget for the fourth
quarter of 2073/74.
The following information are made available for this purpose.

1) It expects to sell 50,000 bags of Ujyalo during the fourth quarter of 2073/74 at
the selling price of Rs.90 per bag.
2) Each bag of Ujyalo requires 2.5 kgs. of raw-material ‘A’ and 7.5 kgs. of raw-
material ‘B’
3) Stock levels are planned as follows:
Beginning of quarter End of quarter
Finished bags of ‘Ujyalo’ (Nos.) 15,000 11,000
Raw-material A (Kgs.) 32,000 26,000
Raw-material B (Kgs.) 57,000 47,000
Empty bags (Nos.) 37,000 28,000

4) Raw-materials A costs Rs.12 per kg., B costs Rs.2 per kg. and empty bag costs
Rs. 8 each.
5) It requires 9 minutes of direct labour time to produce and fill one bag of
‘Ujyalo’. Labour cost is Rs. 50 per hour.
6) Variable manufacturing costs are Rs. 4.50 per bag and fixed manufacturing
costs Rs. 3,00,000 per quarter.
7) Variable selling and administration expenses are 5% of sales and fixed
administration and selling expenses are Rs. 2,50,000 per quarter.

Required: (1+4+2+3=10)
(61)

i) Prepare a production budget for the fourth quarter.


ii) Prepare a raw-materials purchase budget for A, B and empty bags for the fourth
quarter in terms of quantity as well as rupees.
iii) Compute the budgeted variable cost to produce one bag of ‘Ujyalo’.
iv) Prepare a statement of budgeted net income for the fourth quarter in terms of
both per unit and total cost data.

b) Pathibhara Paints Ltd. produces product ‘Royal Play’ a special effect paint which
passes through two processes before it is completed and transferred to finished stock.
The following data relates to Chaitra 2073.
Particulars Process-I Process-II Finished Stock
Rs. Rs. Rs.
Opening Stock 1,50,000 1,80,000 4,50,000
Direct Materials 3,00,000 3,15,000 -
Direct Wages 2,24,000 2,25,000 -
Factory Overheads 2,10,000 90,000 -
Closing Stock 74,000 90,000 2,25,000
Inter process profit included
in Opening Stock NIL 30,000 1,65,000
Output of Process-I is transferred to Process-II at 25 percent profit on the transfer
price, whereas output of process-II is transferred to finished stock at 25 percent on
total cost. Stocks in processes are valued at prime cost. Finished stock is valued at
the price at which it is received from process-II. Sales for the month is Rs.
28,00,000.

Required: (2+4+4=10)
i) Process-I account.
ii) Process-II account.
iii) Finished stock account.

2 (a). Solution

(i) Production Budget of ‘Ujyalo’ for the fourth quarter


Bags (Nos.)
Budgeted sales 50,000
Add: Desired closing stock 11,000
Total requirements 61,000
Less: Opening stock 15,000
Required production 46,000

(ii) Raw-materials purchase budget in quantity as well as in rupees for 46,000 bags of
‘Ujyalo’
A B Empty Bags
Kgs. Kgs. Nos.
Production requirement per
Bag of ‘Ujyalo’ 2.5 7.5 1
Requirement for 46,000 bags 1,15,000 3,45,000 46,000
Add: desired closing stock 26,000 47,000 28,000
Total requirements 1,41,000 3,92,000 74,000
Less: Opening stock 32,000 57,000 37,000
Quantity to be purchased 1,09,000 3,35,000 37,000
Cost per Kg./Bag Rs. 12 Rs. 2 Rs. 8
(62)
Cost of purchase (Rs.) 13,08,000 6,70,000 2,96,000

(iii) Computation of budgeted variable cost of production of one bag of ‘Ujyalo’.


Rs. Rs.
Raw-material A: (2.5 Kg. × Rs. 12) 30.00
Raw-material B: (7.5 Kg. × Rs. 2) 15.00 45.00
Empty Bag 8.00
Direct Labour (Rs. 50 × 9/60) 7.50
Variable manufacturing overhead 4.50
Variable cost of production per bag 65.00

(iv) Statement of budgeted net income for the fourth quarter


Per Bag Total
Rs. Rs.
Sale value (50,000 bags) 90.00 45,00,000
Less: variable costs
Production cost 65.00 32,50,000
Selling & administration expenses
(5% of Sale price) 4.50 2,25,000
69.50 34,75,000
Budgeted contribution 20.50 10,25,000
Less: Fixed costs
Manufacturing Rs.300,000
Admn & selling Rs.250,000 5,50,000
Budgeted net income 4,75,000

2 (b). Solution

Process-I account
Particulars Total (Rs.) Cost (Rs.) Profit Particulars Total (Rs.) Cost Profit
(Rs.) (Rs.) (Rs.)
To - By
Opening transfer to
Balance 1,50,000 1,50,000 Process-II 10,80,000 8,10,000 2,70,000
To Direct -
Materials 3,00,000 3,00,000
To Direct -
Wages 2,24,000 2,24,000
6,74,000 6,74,000 -
Less: -
Closing
Stock 74,000 74,000
Prime Cost 6,00,000 6,00,000 -
To Factory -
Overheads 2,10,000 2,10,000
Total cost 8,10,000 8,10,000 -
Profit 25%
on transfer
price
(33.33% on
cost) 2,70,000 - 2,70,000
10,80,000 8,10,000 2,70,000 10,80,000 8,10,000 2,70,000
(63)
Process-II account
Particulars Total (Rs.) Cost (Rs.) Profit Particulars Total (Rs.) Cost (Rs.) Profit
(Rs.) (Rs.)
To By
Opening transfer to
Balance 1,80,000 1,50,000 30,000 Finished
Stock 22,50,000 15,15,000 7,35,000
To Direct -
Materials 3,15,000 3,15,000
To Direct -
Wages 2,25,000 2,25,000
To
Transfer
from
Process-I 10,80,000 8,10,000 2,70,000
18,00,000 15,00,000 3,00,000
Less:
Closing
Stock 90,000 75,000 15,000
Prime Cost 17,10,000 14,25,000 2,85000
To Factory
Overheads 90,000 90,000 -
Total cost 18,00,000 15,15,000 2,85,000
Profit 20%
on transfer
price (25%
on cost) 4,50,000 - 4,50,000
22,50,000 15,15,000 7,35,000 22,50,000 15,15,000 7,35,000
Working note:
Profit element in closing stock- Process-II = Rs.(3,00,000/18,00,000) × Rs. 90,000
= Rs. 15,000

Finished stock account


Particulars Total (Rs.) Cost (Rs.) Profit Particulars Total (Rs.) Cost (Rs.) Profit
(Rs.) (Rs.)
To By Sales 28,00,000 16,50,000 11,50,000
Opening
Balance 4,50,000 2,85,000 1,65,000
To
Transfer
from
Process-II 22,50,000 15,15,000 7,35,000
27,00,000 18,00,000 9,00,000
Less:
Closing
Stock 2,25,000 1,50,000 75,000
Total cost 24,75,000 16,50,000 8,25,000
Profit
(Balancing
figure) 3,25,000 - 3,25,000
28,00,000 16,50,000 11,50,000 28,00,000 16,50,000 11,50,000
Working note:
Profit element in closing stock-Finished stock = Rs. (9,00,000/27,00,000) × Rs. 2,25,000
= Rs. 75,000
(64)

3.
a) XYZ Ltd. produces washing detergent powder "Diyalo" in a batch of 10 Kgs.
Standard material inputs required for 10 Kg. of "Diyalo" are as follows:

Material Quantity (in Kg.) Rate per Kg. (Rs.)


A 5 110
B 3 320
C 3 460
During the month of June 2016, actual production was 5,000 Kgs. of "Diyalo" for
which the actual quantities of material used for a batch and the prices paid thereof
are as under:

Material Quantity (in kgs.) Rate per kg. (Rs.)


A 6 115
B 2.5 330
C 2 405
You are required to calculate the following variances based on the above given
information for the month of June 2016. 10
i) Material Cost Variance
ii) Material Price Variance
iii) Material Usage Variance
iv) Material Mix Variance
v) Material Yield Variance
b) Deluxe Limited undertook a contract for Rs. 5,00,000 on 1st July 2016. On 30th June
2017, when the accounts were closed, the following details about the contract were
gathered:
Rs. Rs.
Wages accrued
Materials purchased 1,00,000 30.6.2017 5,000
Wage paid 45,000 Work Certified 2,00,000
General expenses 10,000 Cash received 1,50,000
Plant purchased 50,000 Work Uncertified 15,000
Materials on hand
30.6.2017 25,000 Depreciation on plant 5,000

The above contract contained an escalation clause which reads as follows:

In the event of prices of materials and rates of wages increase by more than 5%, the
contract price will be increased accordingly by 25% of the rise in the cost of
materials and wages beyond 5% in each case.

It was found that since the date of signing the agreement the prices of materials and
wages rates increased by 25%. The value of the work certified does not take into
account the effect of above clause.

Prepare the contract account. 8


c) Describe briefly on method of Accounting of administrative overhead. 2
(65)

Answer:

a)
SQ – Standard Quantity of Materials for Actual Output:
A 5 kgs. × 5,000 kgs = 2,500 kgs.
10 kgs.
B 3 kgs × 5,000 kgs = 1,500 kgs.
10 kgs
C 3 kgs × 5,000 kgs = 1,500 kgs.
10 kgs

AQ – Actual Quantity of Materials used for Actual Output:


A 6 kgs. × 5,000 kgs = 3,000 kgs.
10 kgs.
B 2.5 kgs × 5,000 kgs = 1,250 kgs.
10 kgs
C 2 kgs × 5,000 kgs = 1,000 kgs.
10 kgs

RSQ – Revised Standrad Quantity of Materials:


A 5 kgs. × 5,250 kgs = 2,386 kgs.
11 kgs.
B 3 kgs × 5,250 kgs = 1,432 kgs.
11 kgs
C 3 kgs × 5,250 kgs = 1,432 kgs.
11 kgs

Material SQ × SP AQ × SP AQ × AP RSQ × SP
A 2,500 kgs × Rs. 110 3,000 kgs ×Rs. 110 3,000 kgs ×Rs. 115 2,386 kgs ×Rs. 110
= Rs. 2,75,000 = Rs. 3,30,000 = Rs. 3,45,000 = Rs. 2,62,460
B 1,500 kgs × Rs. 320 1,250 kgs × Rs. 320 1,250 kgs × Rs. 330 1,432 kgs × Rs. 320
= Rs. 4,80,000 = Rs. 4,00,000 = Rs. 4,12,500 = Rs. 4,58,240
C 1,500 kgs × Rs. 460 1,000 kgs × Rs. 460 1,000 kgs × Rs. 405 1,432 kgs × Rs. 460
= Rs. 6,90,000 = Rs. 4,60,000 = Rs. 4,05,000 = Rs. 6,58,720
Total Rs. 14,45,000 Rs. 11,90,000 Rs. 11,62,500 Rs. 13,79,420

(i) Material Cost Variance = Std. Cost – Actual Cost


(SQ × SP) – (AQ × AP)
A = Rs. 2,75,000 – Rs. 3,45,000 = Rs. 70,000 (A)
B = Rs. 4,80,000 – Rs. 4,12,500 = Rs. 67,500 (F)
C = Rs. 6,90,000 – Rs. 4,05,000 = Rs. 2,85,000 (F)
Rs. 2,82,500 (F)
(ii) Material Price Variance = Actual Qty. (Std. Price – Actual Price)
(AQ × SP) – (AQ × AP)
A = Rs. 3,30,000 – Rs. 3,45,000 = Rs. 15,000 (A)
B = Rs. 4,00,000 – Rs. 4,12,500 = Rs. 12,500 (A)
C = Rs. 4,60,000 – Rs. 4,05,000 = Rs. 55,000 (F)
Rs. 27,500 (F)
(iii)Material Usage Variance = Std. Price (Std. Qty. – Actual Qty.)
(SQ × SP) – (AQ × SP)
A = Rs. 2,75,000 – Rs. 3,30,000 = Rs. 55,000 (A)
B = Rs. 4,80,000 – Rs. 4,00,000 = Rs. 80,000 (F)
C = Rs. 6,90,000 – Rs. 4,60,000 = Rs. 2,30,000 (F)
(66)
Rs. 2,55,000 (F)

(iv) Material Mix Variance = Std. Price (Revised Std. Qty. – Actual Qty.)
(RSQ × SP) – (AQ × SP)
A = Rs. 2,62,460 – Rs. 3,30,000 = Rs. 67,540 (A)
B = Rs. 4,58,240 – Rs. 4,00,000 = Rs. 58,240 (F)
C = Rs. 6,58,720 – Rs. 4,60,000 = Rs. 1,98,720 (F)
Rs. 1,89,420 (F)
(v) Material Yield Variance = Std. Price (Std. Qty. – Revised Std. Qty.)
(SQ × SP) – (RSQ × SP)
A = Rs. 2,75,000 – Rs. 2,62,460 = Rs. 12,540 (F)
B = Rs. 4,80,000 – Rs. 4,58,240 = Rs. 21,760 (F)
C = Rs. 6,90,000 – Rs. 6,58,720 = Rs. 31,280 (F)
Rs. 65,580 (F)

b)

In the Books of Deluxe Ltd. For the period 1st July 2016- 30th June 2017.
Contract Account
Rs.
To Materials 1,00,000 By Work-in-progress:
To Wages (45,000 +
5,000) 50,000 Work certified 2,00,000
To General expenses 10,000 Work uncertified 15,000
To Depreciation on By Contract
Plant 5,000 escalation(WN: 1) 5,000
To Profit : P&L (WN:
2) 20,000 By Materials in hand 25,000
WIP
(Reserve) 60,000
2,45,000 2,45,000

Working Notes:

1. Escalation Charges:
(a)Material

Total increase up to 5% beyond

Effect of increase in 75000*(25/125) 75,000*(5/125)


Price of Materials =15,000 =3,000 =12,000
(b)Wage
Effect of increase in 50,000*(25/125) 50,000*( 5/125)
Wage rate =10,000 =2,000 =8,000
Total Increase (a-b) =25,000 =5,000 =20,000
Increase in contract price
(25% of Increase beyond 5%) 20,000*(25/100)= 5000
2. Computation of Profit transferred to Profit & Loss Account:
Since more than 1/4th but less than ½ of the contract has been completed, 1/3 of the profit
earned as reduced on cash basis has been transferred to Profit and Loss Account.
80,000 * 1/3 * 1, 50,000/ 2, 00,000 = Rs. 20,000.

c) Method of Accounting of administrative overhead

There are three distinct methods of accounting of administrative overheads.


(67)
(a)Appropriation of Administrative Overheads between Production and sales Department

(b) Charging to Profit and loss Account

This method is used when the overhead not directly related to production and selling &
distribution, where difficult to determine suitable base.

(c) Treatment of administrative overheads as a separate additional to cost of Production

This method considers administration as a separate function like production and sale.

4.
a) Auto Ltd. manufactures pistons used in car engines. As per the study conducted by
the Auto Manufacturers Association, there will be a demand of 80 million pistons in
the coming year. Auto Ltd. is expected to have a market share of 1.15% of the total
market demand of the pistons in the coming year. It is estimated that it costs Rs.1.50
as inventory holding cost per piston per month and that the set-up cost per run of
piston manufacture is Rs. 3,500.
i) What would be the optimum run size for piston manufacturing?
ii) Assuming that the company has a policy of manufacturing 40,000 pistons per
run, how much extra costs the company would be incurring as compared to the
optimum run suggested in (i) above? 5

b) A manufacturing unit has purchased and installed a new machine of Rs. 12,70,000
to its fleet of 7 existing machines. The new machine has an estimated life of 12
years and is expected to realize Rs. 70,000 as scrap at the end of its working life.
Other relevant data are as follows:
1) Budgeted working hours are 2,592 based on 8 hours per day for 324 days. This
includes 300 hours for plant maintenance and 92 hours for setting up of plant.
2) Estimated cost of maintenance of the machine is Rs. 25,000 (p. a,).
3) The machine requires a special chemical solution, which is replaced at the end
of each week (6 days in a week) at a cost of Rs. 400 each time.
4) Four operators control operation of 8 machines and the average wages per
person amounts to Rs. 420 per week plus 15% fringe benefits.
5) Electricity used by the machine during the production is 16 units per hour at a
cost of Rs. 3 per unit. No current is taken during maintenance and setting up.
6) Departmental and genera1 works overhead allocated to the operation during last
year was Rs. 50,000. During the current year it is estimated to increase 10% of
this amount.
Calculate machine hour rate, if (a) setting up time is unproductive; (b) setting up
time is productive. 5
c) An Executive manager of Nepal Bank Ltd. spends Rs. 10.00 per kilometer on taxi
fares for his office work. He is considering two other alternatives, the purchase of a
new Nano car or a second hand Alto car. The estimated cost figures are as follows:

Items New Nano Car Old Alto Car


Purchase Price Rs. 1,35,000 Rs. 1,60,000
Sale price, after 5 years Rs. 25,000 Rs. 40,000
Repairs and servicing per annum Rs. 12,000 Rs. 18,000
Taxes and insurance per annum Rs. 3,200 Rs. 2,400
Petrol consumption per liter 20 km 15 km
Petrol/Diesel price, per liter Rs. 68 Rs. 42
(68)
He estimates that he has to travel 10,800 km. annually. Which of the three
alternatives will be economical? If his official visit increases and he has to do
18,000 km per annum what should be his decision? 5

Answer:
a)
i) Optimum Run Size:

Optimum run size or Economic Batch Quantity (EBQ) =

Where, D = Annual demand i.e. 1.15% of 8,00,00,000 = 9,20,000 units


S = Set-up cost per run = Rs. 3,500
C = Inventory holding cost per unit per annum
= Rs.1.5 × 12 months = Rs. 18

EBQ =
= 18,915 units
(ii) Calculation of Total Cost of set-up and inventory holding

Batch size No. of set-ups Set-up Cost (Rs.) Inventory holding Total Cost
cost (Rs.) (Rs.)
A 40,000 units 23 80,500 3,60,000 4,40,500
(9,20,000/40,000) (23 × Rs. 3,500) (40,000*Rs.18/2)

B 18,915 units 48.64 1,70,235 1,70,235 3,40,470


(9,20,000/18,915) (48.64 × Rs.3,500) (18,915*Rs.18/2)

Extra Cost (A – B) 100,030

b)

Computation of Machine hour Rate


Per year Per hour Per hour
(unproductive) (productive)
Standing charges
Operators wages
4x 420 x 54 90,720
Add: Fringe Benefits 15% 13,608
1,04,328
Departmental and general
overhead
(50,000+ 5,000) 55,000
Total Standing Charges for 8
1,59,328
machines
Cost per Machine (1,59,328/8) 19,916
Cost per Machine hour
9.05
(19,916/2,200)
(69)
(19,916/2,292) 8.69
Machine hours:
Setting time unproductive
(2,592-300-92)= 2200
Setting time productive (2,592-
300)= 2,292
Machine expenses
Depreciation (12,70,000 - 45.45
70,000)/(12*2,200)
(12,70,000-70,000)/(12*2,292) 43.63

Electricity (l6*3) 48.00


(16*3*2,200)/2,292 46.07
Special chemical solution 9.82 9.42
(400x54)/2,200
(400x54)/2,292
Maitenance (25,000/2,200) 11.36 10.91
(25,000/2,292)
Machine Hour Rate 123.68 118.72

Note: Alternatively depreciation can be placed under standing charge.


c)
Statement showing comparative costs of alternatives modes of conveyance
New Nano Old Alto
Item Car Car Taxi
Rs. Rs. Rs.
Fixed costs per annum:
Depreciation 22,000 24,000
Repairs and servicing 12,000 18,000
Taxes and insurance 3,200 2,400
(A) Total 37,200 44,400
Variable costs per annum:
(B) Petrol Diesel: 10,800km 36,720 30,240
(C) Petrol Diesel: 18,000 km 61,200 50,400
Total Costs:
10,800 km (A+B) 73,920 74,640 1,08,000
18,000 km (A+C) 98,400 94,800 1,80,000

Conclusion. For present official tour of 10,800 km, the total cost for the new Nano car
is the lowest and thus this is the cheapest alternative. But when official tour increases
to 18,000 km per annum the old Alto car will be cheapest.

5.
a) Saubhagya Ltd is operating at 70 percent capacity and presents the following
information:

Break-even point Rs. 200 crores


P/V ratio 40 per cent
Margin of safety Rs.50 crores
(70)
Saubhagya’s management has decided to increase the production to 95 percent
capacity level with the following modifications:

1) The selling price will be reduced by 8 per cent.


2) The variable cost will be reduced by 5 percent on sales.
3) The fixed cost will increase by Rs. 20 crores, including depreciation on
additions, but excluding interest on additional capital.
4) Additional capital of Rs. 50 crores will be needed for capital expenditure and
working capital.

Required: 7
i) Indicate the sales figures, with the working, that will be needed to earn Rs. 10
crores over and above the present profit and also meet 20 % interest on
additional capital.
ii) What will be the revised?
i) P/V Ratio.
ii) Break-even point.
iii) Margin of safety.
b) Outline the Important Factors for the control of labour cost. 4
c) Outline the main objective of uniform costing. 4

Answer:

5 (a).

Working notes:
1. Total sales = Break-even sales + Margin of safety
= Rs. 200 crores + Rs. 50 crores
= Rs. 250 crores.
2. P/V Ratio = 40%
Variable cost = 60% of sales
= Rs. 250 crores x 60%
= Rs. 150 crores
3. Fixed Cost = Break-even sales x P/V Ratio
= Rs. 200 crores x 40%
=Rs. 80 crores
4. Total Cost = Rs. 150 crores + 80 Crores
= Rs. 230 crores
5. Profit = Rs. 250 crores – 230 crores
= Rs. 20 crores

6. Revised P/V Ratio


Let the present selling price is Rs. 100.00
Revised selling price (Rs. 100 – Rs.8) Rs. 92.00
Revised variable cost (Reduced from 60% to 55%)
(Rs. 100 x 0.55) Rs. 55.00
Contribution Rs. 37.00

New P/V Ratio = (Contribution/Sales) x 100


= (Rs. 37.00/Rs. 92.00) x 100
= 40.217%
(71)
A. Revised sales
(Rs. in crores)
Present fixed cost 80
Increase in fixed cost 20
Interest @ 20% on additional capital of Rs. 50 crore 10
Total revised fixed cost 110

Required contribution = Rs. 110 crores + Rs. 30 crores


= Rs. 140 crores

Required Sales = Required contribution/ New P/V Ratio


= Rs. 140 crores/40.217%
= Rs. 348.11 crores

B. (i) Revised P/V Ratio = 40.217% (working note 6)


(ii) Revised BEP = Revised Fixed cost/ New P/V Ratio
= Rs. 110 crores/40.217%
= Rs. 273.516 crores

(iii) Revised Margin of safety = Revised sales – Revised BE sales


= Rs. 348.11 crores – 273.516 crores
= Rs. 74.594 crores

5 (b).

To exercise an effective control over the labour cost, the essential requisite is efficient utilization of
labour and allied factors. The main points which need consideration for controlling labour cost are
the following.

1. Assessment of manpower requirement


2. Control over time keeping and time booking
3. Time and motion study
4. Control over ideal time and overtime
5. Control over labour turnover
6. Wage system and Incentive system
7. Job Evaluation and Merit ratings
8. Labour Productivity

5 (c).
The main objectives of uniform costing are as follows:
 To facilitate the comparisons of cost and performance of different units in the same
industry.
 To eliminate unhealthy competition among the different units of an industry.
 To improve production capacity level and labor efficiency by comparing the production
costs of different units with each other.
 To provide relevant cost information to the government for fixing and regulating prices
of the products.
 To bring standardization and uniformity in the operation of participating units.

6. Distinguish between the following : (4×2.5=10)


a) Job Costing and Process Costing
(72)
b) Just in Time manufacturing and Traditional manufacturing
c) Absorption costing and Variable Costing
d) Job evaluation and Job Rating

a) Distinction between Job Costing and Process Costing

Job Costing Process Costing


1) Production is carried on by specific order. 1) Production is a continuous flow and the
products are homogeneous.
2) Costs are determined by jobs or batches of 2)Costs are compiled on time basis i.e for
products. production for a given accounting period for
each process
3) Various jobs are separate and independent 3) Processes are related to each other
products also lose their individual entity.
4) Unit cost of a job is calculated by dividing 4) The unit cost of a process which is
the total cost by units produced in the lot or computed by dividing the total cost for the
batch. period into the output of the process during
that period is an average cost(after adjusting
opening and closing WIP)for the period.
5) Costs are calculated when a job is 5) Costs are calculated at the end of the
completed. period under each process.
6) There may not be opening or closing WIP 6) Production in process costing is
in an accounting period. continuous and therefore there is normally
WIP at beginning and closing.
7) There is normally no transfer from one job 7) Transfer from one process to another is a
to another .It will be only when there is usual feature.
surplus or excess production.
8) Each product unit is different and therefore 8) Production is standardized and stable the
more managerial attention is needed for control is, therefore easier.
proper control.

b) Distinction between Just in Time and Traditional Manufacturing

Just in Time manufacturing Traditional manufacturing


1. It is a demand pull system. 1. It is a production push system.
2.There is insignificant or zero inventories 2.There is significant inventories
3.There are manufacturing sales (work 3. There is process structure
centers)
4. There is multifunction labor. 4. There is specialized labor.
5. There is total quality management system. 5.There is acceptable quality labor(AQL)
6. It uses simple cost accounting 6. It uses complex cost accounting system.

c) Distinction between absorption and variable Costing

The basic difference between absorption and variable costing relates to the handling of fixed
manufacturing overhead. Under absorption costing, fixed manufacturing overhead is treated as a
period cost and is deducted in full from the current periods revenue. The other differences are:
(73)
1. The value of closing stock under absorption costing will be more as the unit cost under it is more
than the variable cost.
2. The cost of manufacturing under variable costing is less than the absorption costing
3. Variable costing takes contribution margin as the basis for decision making and absorption
costing takes the profit as the basis
4. Absorption of overheads is a major issue under absorption costing and it is a non-issue under
variable costing.
5. Absorption costing is accepted for external reporting and the variable costing is found to be
useful to decision -making by the management, hence internal orientation.
d.) Distinction between Job Evaluation and Merit Rating
Job evaluation. It can be defined as the process of analysis and assessment of jobs to ascertain
reliably their relative worth and to provide management with a reasonably sound basis for
determining the basic internal wage and salary structure for the various job positio ns. In
other words, job evaluation provides a rationale for differential wages and salaries for
different groups of employees and ensures that these differentials are consistent and
equitable.
Merit Rating. It is a systematic evaluation of the personality and performance of each employee
by his supervisor or some other qualified persons.
Thus the main points of distinction between job evaluation and merit rating are as follows:
1. Job evaluation is the assessment of the relative worth of jobs within a company and
merit rating is the assessment of the relative worth of the man behind a job. In other
words job evaluation rate the jobs while merit rating rate employees on their jobs.
2. Job evaluation and its accomplishment are means to set up a rational wage and sa lary
structure whereas merit rating provides scientific basis for determining fair wages for
each worker based on his ability and performance.
3. Job evaluation simplifies wage administration by bringing auniformity in wage rates.
On the other hand merit rating is used to determine fair rate of pay for different workers
on the basis of their performance.
(74)

Business Communication
(75)
Suggested
Roll No……………. Maximum Marks – 100
Total No. of Questions - 8 Total No. of Printed Pages -5
Time Allowed – 3 Hours
Marks
All questions are compulsory.
Section -'A'
1. Read the following case carefully and answer the questions given below: (4×5=20)
Avishek manages the customer relations department of a public utility company. The
department is responsible for replying to customer requests for information and to
customer complaints. There are seven typists in the department who handle the
correspondence to customers dedicated by the staff.

Avishek recently promoted Rachana to office supervisor. Rachana had been a typist in
the department for ten years. She was considered the fastest and most accurate typist
who had ever worked for Avishek. Rachana has the best attendance record in the
department and Avishek considered her his most dependable employee. She was very
well liked by all the typists and they considered her to be a good personal friend.

As a supervisor, Rachana does a good job handing out work assignments, but she does
little else to supervise the typists. She does not like to criticize the typists and does not
enforce office rules. No matter what a typist does, Rachana will not take any
disciplinary action. She makes no attempts to check the work of the typists for
compliance with quality standards or to see that work is completed on time. In fact, she
spends most of her time typing to reduce the work load of other typists.

The human resource department has been receiving an increasing number of complaints
from various staff from different departments about the poor quality of the typing and
about the slow turnaround time of the work. A number of the complaints target towards
Rachana’s poor supervisory skills. Complaints were also received about the typists
under Rachana taking excessively long coffee breaks and spending time on personal
phone calls. The human resource department conveyed the complaints to Avishek who
has been asked to inquire and rectify the problem.

When Avishek talked to the typists, they told him that Rachana frequently invites them
to her house for dinner or to play bridge. It appears to Avishek that the typists all like
Rachana as a friend, but they are becoming concerned about her lack of supervisory
skills. Avishek mentions to Rachana that she should focus her effort on improving the
work of the typists. She replied, “These woman are my friends and I don’t feel right
cracking down on them”.

Questions:
a) Can an individual effectively manage a group and be a close personal friend with
group members?

ANS: An individual's managing a group depends on the organizational context and the personalities of
ones' colleagues. The elasticity of relation depends, also, according to the situation and types of job one
has to perform. It seems that almost every organization is managed by an individual because there is
always a corporate head, who takes charges of and monitors overall office affairs. In this connection an
individual is believed to manage a group. However, the role of subordinates, although marginalized, has
greater importance in maintaining the group bonding. Hence, an individual, no doubt, requires to
(76)
maintain friendly relation with his subordinates, group members and coworkers. Similarly, as far as
close and personal friendship is concerned, an individual should have two "selves" i.e. personal and
official. Acknowledging the office hierarchy and portfolio systems one can still maintain a good relation
but not that friendly. One should very carefully regard the limitations in office affairs to maintain a
conducive office atmosphere.

b) What should Avishek do to ensure that the work of the typists will improve?

ANS: Avishek as a manager must have been vigilant from the beginning with every possible activities of
his staff. Although 'it is better late than never', Avishek requires to find out the root causes of the activities
taking place in the office and he needs to take the decision that gets win-win situation. Avishek should call
Rachana and inquire her about her perception to the coworkers. Rachana must have been too friendly may
be because she has any other "domestic psychological disturbance" as a result she loves to hangout with
the office coworkers. Rachana along with other typists' performances can be improved with asking for as
well as giving advices and recommendation. May be she is frustrated with the low payment system and
that is a type of "systematic soldering" as a protest. Simply promoting to the position doesn’t ensure her
financial stability. Avishek must be responsible to review his managerial skill as well. Besides these,
Avishek may invite an expert and motivate Rachana along with other typist colleagues to motivate and
excel their skill in everyday affairs.
c) Should Rachana have been promoted to office supervisor?

ANS: The manager understands the need of the organization and go for the appropriate recruitment
procedures. Most of the time the manager will have eyes on the staff in the organization if they are capable
of taking the responsibilities and give their best performances. In case the candidate is fit within the
organization a manager doesn’t go for costly and lengthy recruitment process. Avishek must have thought
Rachana a best fit and promoted her. Nonetheless, Avishek as a manager must have reviewed the
performances and should envision the characteristics of the individual he is promoting. In this case
Avishek seems to find Rachana very dependable for the post thus Rachana's promotion doesn’t look
illogical. By the same token, Rachana's past performances aptly qualify her for the position. Despite of
promoting her to the post, Avishek should have shrewd eyes to observe her performance record, internal
operational, external operational and personal communication. In my view promoting Rachana is not a
blunder but the activities after her promotion needed to be monitored carefully.
d) If you were Rachana, what would you do?

ANS: I would definitely talk to Avishek about all the ongoing activities frankly. I would also request
Avishek to check the work on his own rather than listening to others. Other might have reported negatively
out of jealousy over my promotion in the office. Further, if there were some witnesses on the blaming
made, I will review them and take the chance as an opportunity to appropriate myself in the organizational
culture. Moreover, I would express my understanding that the office work is not disturbed because of my
friendly relation with other members in the organization. I would pay close attention, monitor the fellow
workers politely and hold a meeting on how to improve work performances to excel oneself for the career
growth.

2. There are several factors that can contribute to a successful multinational meeting.
Discuss any five of the most essential factors that contribute to the success of
international business meeting. 10
Answer:
(77)
Multinational meetings are core concerns of today’s business communication. Along with the
globalization of market, business organizations have growing tendencies of extending across
the cultural and national boundaries. In this context, international business meetings have
become a major issue for consideration among the practitioners of multinational business. The
most important factor has become communication, i.e. language and its specific style. The
value of international communication is growing. Apart from communication channel and
process, there are several other factors that make meaningful contribution to a successful
international business meeting. Some of them include:
 Topic for discussion: the participants of the international meetings must be update with
what is being talked about. They have to think in advance about exactly what it is that they
want to discuss and decide.
 Participants: the participants of the international meetings need to know who will be the
other participants in the meeting. The basic cultural norms, interests, expertise, positions,
etc. of the other participants will help to make the contributions in the meeting more
relevant, cooperative and successful.
 Means of communication: in the multinational meetings, the choice of code is remarkably
important. The proficiency in the international language like English needs to be fairly
high. Appropriate preparation about relevant vocabulary, business terms, structures,
exponents, utterances, etc. is prerequisite to the multinational meetings.
 Manner of interaction: interaction in the multinational meetings follows the general maxims
of politeness and cooperation. The participants must be polite as well as cooperative in the
discussion. Furthermore, they should not hesitate to interrupt and argue freely in the
discussion. The native speakers of the dominant language are not expected to dominate the
discussion. Listening to others is essential for every participant.
 Use of nonverbal cues: the nonverbal devices such as gestures, body postures, signs, etc.
also play important role in the multinational meetings. Cross-cultural interpretations and
understandings of the nonverbal devices are to be considered properly so that conflicts are
less likely.

3.
a) Define notice and mention what its various parts are. 5
b) You are the president of Student’s Association of Life School. Write a notice
informing the students about an audition for Radio Show. 5
Answer 3(a):
A notice is the most common method of communication, which gives information regarding
an important event that is about to take place. A notice conveys information in a very precise
manner. These are the parts of notice:

1. Name of organization
2. Word Notice
3. Date of issue
4. Subject
5. Purpose
6. Details of schedule
7. Sign, name, designation

Answer 3(b):
(78)
Life School
Notice
April

Audition for Radio Show


Our school will be presenting a popular Radio Show named Children’s World . Talented
students willing to conjure up original items on the theme Adventure can participate.

Date: April ,
Time: PM
Venue: School Auditorium

For further details, please contact the undersigned.


Asmita Roy
######
President, Student’s Association

4. Write differences between/ short notes on: (4×2.5=10)


a) Formal and Informal reports
b) Cross culture communication
c) Resolving Conflict
d) Channel breakdowns

Answer:

a) The differences between formal and informal reports include tone, structure, scope,
content and purpose. Another difference is that formal reports are often used in academic
papers or to provide a lengthy overview of a major change or development within a
business, while informal reports are used for shorter documents, such as memos and
newsletters. Consequently, formal reports are more detailed than informal reports.
Formal reports have a set structure: an introduction, body and conclusion. However,
informal reports can be structured in any way. Formal reports also have a formal tone and
use third-person narrative, while most informal reports use first- or second-person
narrative. Although some informal reports use a professional tone, it is not required in
many cases.
The scope of informal and formal reports is another major difference. Formal reports
analyze a specific topic in great detail. On the other hand, informal reports are often very
brief and only include the most relevant information.
Formal reports also provide evidence to back up the information in the report. Since
informal reports are brief, this is not included in this type of report. Formal reports usually
include citations for the sources used to generate the report. Charts, graphs and statistical
information are also common components of formal reports. This information provides
evidence and summarizes the findings of the report.

b) The increasing diversity in the workplace poses challenges to the workers as well their
managers. Along with globalization of business, workplace diversity in the workplace has
(79)
become a burning issue in the sector of business and management. As solution has a
phenomenon emerged quite lately, i.e. intercultural communication which is also known as
cross-cultural communication. It refers to the way of communication between the people of
different cultures, without posing any kind of threat to any kind of culture. The
international languages and their nativised varieties are commonly used as effective means
of intercultural communication. Basically, the workers of a business house are given
trainings on intercultural norms, values, beliefs and practices. And, they are given
guidelines to communicate each other in line with the perspectives of different cultures.

c) Conflict in team activities can arise for a number of reasons: competition for resources,
disagreement over goals or responsibilities, poor communication, power struggles, or
fundamental differences in values, attitudes, and personalities. Teamwork isn’t necessarily
about happiness and harmony; even teams that have some interpersonal friction can excel
with effective leadership and team players committed to strong results. The following seven
measures can help team members successfully resolve conflict:
i. Proactive behavior: Deal with minor conflict before it becomes major conflict.
ii.Communication: Get those directly involved in a conflict to participate in resolving it.
iii.Openness: Get feelings out in the open before dealing with the main issues.
iv. Research: Seek factual reasons for a problem before seeking solutions.
v. Flexibility: Don’t let anyone lock into a position before considering other solutions.
vi. Fair play: Insist on fair outcomes and don’t let anyone avoid a fair solution by hiding
behind the rules.
vii.Alliance: Get opponents to fight together against an “outside force” instead of against
each other.
d) Sometimes the channel simply breaks down and fails to deliver the message at all. A
colleague you were counting on to deliver a message to your boss might have forgotten to
do so, or a computer server might have crashed and prevented your blog from updating.
Everyone in an organization can help minimize barriers and distractions. In any situation, a
small dose of common sense and courtesy goes a long way. Turn off that mobile phone
before you step into a meeting. Don’t talk across the tops of other people’s cubicles. Be
sensitive to personal differences, too. For instance, some people enjoy working with music
on, but music is a huge distraction for others, so use headphones. Finally, take steps to
insulate yourself from distractions. Don’t let messages interrupt you every minute of the
day. Instead, try to set aside time to attend to messages all at once so that you can focus the
rest of the time.
(80)

Marketing
(81)
Suggested
Roll No……………. Maximum Marks – 50
Total No. of Questions - 4 Total No. of Printed Pages -2
Time Allowed – 3 Hours
Marks
All questions are compulsory.
Section -'B'

5. Read the following case carefully and answer the questions given below: (45=20)
Five years ago, Kathmandu college, was established in Tinkune, Kathmandu by a group of
Kathmandu University MBA graduates to retain the students of elite class family in the
country by offering high quality education by renowned professors. At present more than
five hundred students are studying in various programs such BBS, BBA, and BHM. The
college is affiliated to Tribhuvan University. There are 50 renowned faculty members in
the college which consists of 20 full time and 30 part time faculty members. There are also
15 administrative staffs to run various programs. The college is running in morning and
day shifts.
From the very beginning, the college has been practicing odd pricing while determining the
fee structure for their programs where as their competitors has been practicing even
pricing. For example, annual fee structure of Kathmandu college is: BBS Rs.99,000, BBA
Rs.119,000, and BHM Rs.129,000. There is no provision of discount for anyone. They
charge same fee structure for all students. But their nearest competitor has been offering
various discount offers based on student’s score in 10+2.
Similarly, they have been promoting their college aggressively from the very beginning.
For advertising, they are using radio, TV and daily newspaper. They always participate in
education fair organized by Kathmandu Post. Prospectus and leaflet have also been used
for college promotion. Famous film star, Rajesh Hamal is used as a brand ambassador of
the college to create brand awareness.
The college management has tried their best to provide quality education. It has already
gained popularity for quality education. The result is excellent in comparison to other
competitive colleges.
Last month, college management had conducted a students’ satisfaction survey by
distributing closed ended questionnaires. In the survey, it was found that students are not
satisfied with college management. Students dissatisfaction is related with poor
infrastructure, lack of sports facilities and no welcome and farewell programs. It was found
that students were highly satisfied with faculty members but not satisfied with
administrative staffs.
As you are the student of marketing, the college management is going consult with you to
solve the problem identified during survey.
a) Are they customer oriented marketer? Give your opinion.
b) Comment on the brand name Kathmandu College.
c) Is odd pricing appropriate for the college? Give your argument.
d) What is the main issue of the case? Explain.
Answer
a)
Ans: In my opinion, they are not customer oriented marketers. Customer orientation is one of the
fundamental principles of new marketing concept. To be a customer oriented marketer, they must create
highly satisfy customers by offering high value. Customer satisfaction must be their motto to become a
(82)
customer oriented marketer. In the above case, it has mentioned that as per the students’ satisfaction
survey, students are not satisfied to the college management. It means marketing program designed by the
management is not customer friendly. In the above case, it has also been mentioned that they have been
relied upon aggressive promotion like advertising and event management. By implementing selling
concept of marketing, no marketer can be a customer oriented marketer. In nutshell, it is concluded that
they are not customer oriented marketer.
b)
Ans: In Nepalese context, the word ‘Kathmandu’ is multi-used brand. There are so many marketers who
use Kathmandu as a brand. Marketers use brand name mainly for identification and differentiation. But
multi-used brand can not give specific brand identity to identify and differentiate the brand from brands.
For example, Kathmandu University, Kathmandu Hospital, Kathmandu steel, Bank of Kathmandu,
Kathmandu Engineering college, Kathmandu college of Management etc. Selected brand must be strong,
favorable and unique for specific brand positioning. Thus, brand name Kathmandu college is not
appropriate from marketing perspective. It only creates confusion to customers.
c)
Ans: odd pricing is a type of psychological pricing. It is suitable for price sensitive customers as it gives
psychologically cheaper feeling for economy minded customers. Based on the above case, it is mentioned
that the college management has tried their best to provide quality education and it has already gained
popularity for quality education. So, to reflect quality in fee structure of the Kathmandu college it is not
good to set odd pricing. Odd pricing is appropriate if marketer want to establish their product as a cheaper
product. So, for quality positioning it is better to set even pricing such as Rs. 100,000, Rs. 120,000 and Rs.
130,000 instead of Rs. 99,000, Rs. 119,000 and Rs. 129,000.
d)
Ans: Based on the given case, the main issue of the case is students’ dissatisfaction because of the
inappropriate marketing mix. Their target market is elite class people, and pricing and promotion strategies
they adopted are not appropriate as per the characteristics of target market. Mere quality education and
excellent result is not enough to satisfy their target market. They must design their marketing program as
per the need of target market. Implementation of traditional marketing concept by inappropriate marketing
mix has led to student’s dissatisfaction. Thus, they must be customer oriented marketer and must
formulate appropriate marketing program as per the need and characteristics of target market.
6. Give the meaning of marketing environment and explain the components of micro
environment of marketing. (4+6=10)

Answer

A company’s marketing environment consists of all the factors and forces outside marketing that affect
marketing management’s ability to build and maintain successful relationships with target customers.
Successful companies know the vital importance of constantly watching and adapting to the changing
environment. As we move into the twenty-first century, both the customers and marketers wonder what
the future will bring. The environment continues to change rapidly. More than any other group in the
company, marketer must be trend trackers and opportunity seekers.
According to Professor Philip Kotler, “marketing environment comprises all the actors and forces
outside marketing that affect marketing management’s ability to develop and maintain successful
transactions with its target customers.”
The marketing environment surrounds and impacts upon the organization. There are two key
perspectives on the marketing environment, namely the macro environment and the micro environment.
The major components of the micro environment of marketing are as follows:
i. The company:
In designing marketing plans, marketing management takes other company groups into account –
groups such as top management, finance, R & D, purchasing, operations, and accounting etc. All these
interrelated groups form the internal environment. Together, all of these departments have an impact on
the marketing department’s plans and actions. Under the marketing concept, all of these functions must
“think customer”. They should work in harmony to provide superior customer value and satisfaction.
ii. Suppliers
(83)
Suppliers form an important link in the company’s overall customer value delivery system. They
provide the resources needed by the company to produce its goods and services. Supplier problems can
seriously affect marketing. Marketing managers must watch supply availability – supply shortages or
delays, labor strikes, and other events can cost sales in the short run and damage customer satisfaction
in the long run. Marketing managers also monitor the price trends of their key inputs. Rising supply
costs may force price increases that can harm the company’s sales volume.
iii. Marketing intermediaries
Marketing intermediaries help the company to promote, sell, and distribute its goods to final buyers.
They include resellers, physical distribution firms, marketing services agencies and financial
intermediaries.
Like suppliers, marketing intermediaries form an important component of the company’s overall value
delivery system. In its quest to create satisfying customer relationships, the company must do more than
just optimize its own performance. It must partner effectively with marketing intermediaries to optimize
the performance of the entire system.
iv. Customers
The company needs to study five types of customer markets closely. Consumer markets consists of
individuals and households that buy goods and services for personal consumption. Business markets
buy goods and services for further processing or for use in their production process, whereas reseller
markets buy goods and services to resell at a profit. Government markets are made up of government
agencies that buy goods and services to produce public services or transfer the goods and services to
others who need them. Finally, international markets consist of these buyers in other countries,
including consumers, producers, resellers and governments. Each market type has special
characteristics that call for careful study by the seller.
v. Competitors
The marketing concept states that to be successful, a company must provide greater customer value and
satisfaction than its competitors do. Thus, marketers must do more than simply adapt to the needs of
target customers. They also must gain strategic advantage by positioning their offerings strongly against
competitors’ offerings in the mind of consumers.
Competition can be generic, product, brand, price and non price.
vi. Publics
The company’s marketing environment also includes various publics likefinancial publics, media
publics, local publics, internal publics, government publics, citizen-action publics, general public etc.
7.
a) Explain the criteria for evaluating market segments. 5
b) What is buyer behavior? Explain briefly the role of age in buying decision. (2+3=5)

Answer

a) After completing the segmentation of market that should be evaluated and better market segments
should be identified. While evaluating market segment, the company should analyze the size and
growth of the segments structural attractiveness of the segments and objectives and resources of
the company.
i. Segment size and growth:
While evaluating market segments, at first information about sale, growth rate and expected
profit should be collected and analyzed. The segment size and growth rate should be the best. But
size and the growth rate should be looked comparatively. Some companies like to target those
markets which have prospective of large quantity sale, high growth rate and high share of profit.
But such market segment may not be attractive for all companies. The companies select the best
market segment on the basis of their efficiency, means, and resources.
ii. Segment attractiveness:
While evaluating market segment, the size, growth rate and profit should be analyzed and its
structural attractiveness be considered. According to Michael Porter, The structural components to
affect attractiveness of market segments are competitors, substitute products, Buyer’s power, and
supplier’s power. Etc. should be studied.
(84)
iii. Company’s objectives and resources:
Even if the size, growth, and structural attractiveness is favorable in the market segments,
the company should compulsorily consider its objectives and resources. Some market segments
may attractive but they should be abandoned, if they are not suitable to the company’s long term
objective. Thus, a company should select only those market segments which are suited to the
company’s objectives. Similarly, while analyzing market segments, production facilities employees
and their abilities, financial condition, means and resources also should be considered.
iv. Inter segment relationship:
Firm should pay much attention for segment interrelationships in terms of cost performance
and technology. Existing distribution channel, advertising media and agency can be used or not ?
The firm also give much attention for inter segment coordination which helps to improve the
overall performance of the organization.
v. Ethical Consideration:
Ethical aspects of various segments should also evaluated with due consideration to environmental
concerns. Well being of customers and society should be carefully evaluated.
vi. Government policies:
Government policies, rules and regulations should be carefully considered for evaluating the
market segments. Ignorance of law is of no excuse. Thus, legal provisions should be strictly
complied within evaluating segments.
b) A buyer is anyone who might buy a given product. A buyer may be either an individual person or
an organization that have an interest in the product and the means to acquire it. Cravens, Hills and
Woodruff define ‘buyer behavior’ as ‘the activities and actions of people (and organizations) that purchase and use economic goods and
services, including, the influences on these activities and actions’.

Buyer behavior is affected by a number of factors including economic forces, demographic forces,
psychological forces, behavior-related forces, etc.
Now, let us try to find the answer the question relating how age affects the buying decision of a
buyer while buying a particular product.
On the basis of age, people can be classified into various groups such as children, youths and old-
aged people. Similarly, each age group can also be divided into various groups – including, 0 – 5, 5
– 10, 10 – 15, 15 – 25, 25 – 35, 35 – 45, 45 – 55, 55 – 65, and above 65 years. The taste,
preferences and the choice of each age group differ from one another.
For example, children prefer to buy toys or related items, Youths may be attracted more towards
fashion product, music items, sports items, etc. Similarly, the choice of old-aged people is to some
extent saturated and may like traditional items.
Marketers need to motivate and satisfy different age-groups in different ways, because all of them
cannot be satisfied with similar items, offers, and treatment.
8. Briefly explain the following: (5×2=10)
a) Marketing Information System (MkIS)
b) Green marketing
c) Service product
d) Holistic marketing concept
e) Point out the objectives of distribution
Answer
a) Marketing Information System (MkIS) is a mechanism established for providing decision making data
and information to marketing planner and marketers. It is an ongoing organizational process includes
the activities such as collection of information, sorting, analysis, evaluation and timely dissemination
of information to marketing decision makers.
b) Green marketing refers to the process of selling products and/or services based on their environmental
benefits. Such a product or service may be environmentally friendly in itself or produced in an
environmentally friendly way.
(85)
c) Any activity or benefits that one party can offer to another that is essentially intangible is called
service product. The product is normally inseparable from service provider. Service product and its
price and quality are varied according to service provider, capacity, goodwill, place and time.
Production and consumption of service are done at same time. So, no organization can keep or store
service. Communication, hotel, restaurant, counseling, and insurance are the example of service
product.
d) The holistic concept explains that there are so many categories of customers, including individuals and
organizations or individual consumers and society as well, whose needs and wants are unlimited and
are not identifiable at all times. And on the other hand, there is high level of competition in the market.
All the needs of customers cannot be satisfied by a single marketing effort or limited marketing efforts
like in the above-mentioned marketing concepts; in order to meet all these needs and satisfy all
customers, marketer needs to use all possible marketing tools and effort.
The major tools of this concept are:
i. Integrated Marketing: This concept emphasis to utilize all available resources
effectively and efficiently to satisfy the various needs and problems of the customers.
ii. Relationship Marketing: This concept emphasizes to establish and maintain good
relation with all the social groups so that they perceive the company and its products
positively.
iii. Managing Internal Marketing: This concept emphasizes to make all the internal staffs
customer-oriented so that they can provide better services and treatment to customers
for their satisfaction.
iv. Societal marketing: This concept also emphasizes to focus on social needs and
problems and work for welfare and wellbeing in society.
e) Main objectives of distributions are:
i. Delivery of satisfaction
ii. Standard of living
iii. Value addition
iv. Communication
v. Employment
vi. Efficiency
vii. Financing
(86)

Income Tax and VAT


(87)
Suggested
Roll No……………. Maximum Marks - 100
Total No. of Questions - 8 Total No. of Printed Pages -18
Time Allowed - 3 Hours
Marks
Attempt all questions. Working note should form part of the answer.
6. Citizen Cement Ltd. is a company situated in Biratnagar and listed in Nepal Stock
Exchange and engaged in manufacturing and sale of premium grade cement for Nepal
and Export. Following is the provisional Income Statement of Citizen Cement Ltd., for
the year ended Ashad 31, 2073.
Particulars Amount (Rs.)
Income
Export Sales 60,00,000
Domestic Sales 40,00,000
Dividend Received (Net of Tax) 1,50,000
Rent Income (Related with Business) 50,000
Total Income 1,02,00,000
Expenditure
Cost of Materials Consumed 30,00,000
Manufacturing Expenses 5,00,000
Employee Cost 10,00,000
Selling and Administrative Expenses 15,00,000
Interest and Bank Charges 5,00,000
Exchange Loss 2,50,000
Depreciation 5,00,000
Bad Debt Written Off 70,000
Loss on Sale of Assets 3,00,000
Total Expenditure 76,20,000

Operating Profit 25,80,000


Add: Provision for Doubtful Receivables Written Back 7,20,000
Less: Provision for Bonus 3,00,000
Profit Before Tax 30,00,000
Additional information:
a) Cost of material consumed
For export sales : Rs. 18,00,000
For local sales : Rs. 12,00,000
b) Exchange loss includes Rs. 1,00,000 against revaluation of creditors at the year end
date.
c) Asset detail for income tax purpose is as below –

Building Plant and Computer, Vehicles


Machinery Office
Equipment and
Furniture
Opening 11,00,000 1,00,00,000 15,00,000 29,50,000
Depreciation
Base
Addition upto - - 2,00,000 -
Poush End
Addition Magh to 6,00,000 - 1,50,000 -
(88)
Chaitra end
Addition Baishak - - - 9,00,000
to Ashad End
Sales Proceed - 15,00,000
d) Manufacturing expenses includes repair and maintenance expenses as below:
Building Repair and Maintenance : Rs. 1,50,000
Office Equipment Repair and Maintenance : Rs. 50,000
Vehicle Repair and Maintenance : Rs. 20,000
e) Selling and Distribution Expenses includes Rs. 7,00,000 donation given to Prime
Minister Disaster Relief Fund and Rs. 3,00,000 given for construction of school.
f) Bonus of Rs. 2,00,000 only distributed to the employees till the time of filing of
income tax return. It has been decided by the management not to pay the
undistributed portion.
g) Following expenses are of below nature:
a. Rs. 50,000 included in employee cost for staff welfare is for personal use of
directors.
b. Manufacturing expenses includes Rs. 30,000 for electricity bill of previous
years.
c. Selling and administrative expenses includes Rs. 10,000 for business promotion
which is not related to business.
h) Company employed 1,300 Nepali employees during the whole year out of which 50
employees are foreign employee.
Based on the above information, please advise Citizen Cement Ltd. regarding. (5+15=20)
i) Various tax exemptions available to them as per Income Tax Act, 2058, and
which tax exemptions Citizen Cement Ltd. should opt for?
ii) Compute taxable income and tax liability of the company segregating income on
the basis of cost of material consumed for common cost and income.
Answer
a. Various Tax Exemptions Available to Citizen Cement Limited as per Income Tax Act, 2058
As per section 11 of Income Tax Act, 2058 below are the exemptions available to Citizen Cement Limited being
a Special Industry.
i. Citizen Cement Limited employs more than 1,200 Nepali employees whole year. So, it is entitled
for 20% tax rebate as per section 11(3)(Ka) for employing 1,200 or more Nepali Employees
ii. Citizen Cements Limited has export sales. So, 25% tax rebate for export Income only as per section
11(3)(3Nga).
iii. Citizen Cements Limited is a listed production based Company. Hence, it is entitled to 10% rebate
as per section 11(3)(3chha) .
b. Selection of Tax Exemption
i. As per section 11(5), if more than one tax exemption is available for the same income, then only
one tax exemption should be availed as per choice of the Taxpayer
ii. Hence, Citizen Cement Limited should choose the most beneficial option among the following
exemptions .
Exemption Rebate on Export Income Rebate on
Domestic
Income
20% tax rebate as per section 20% 20%
11(3)(Ka) for employing 1,200 or
more Nepali Employees
25% tax rebate for export Income 25% 0%
only as per section 11(3)(3anga)
10% rebate as per section 10% 10%
11(3)(3chha) for Citizen Cements
Limited is a listed production based
Company
Therefore, Selection of Exemption 25% tax rebate for export 20% tax rebate
(89)
shall be: Income only as per section as per section
11(3)(3anga) 11(3)(Ka) for
employing
1,200 or more
Nepali
Employees

Computation of Taxable Income of Citizen Cement Limited


for the year ended Ashadh 31, 2073
Amount Rs.
Particulars Export Domestic Total Remarks
Income Income
Income From
Business:
Export Sales 60,00,000 - 60,00,000
Domestic Sales - 40,00,000 40,00,000
Dividend - - - Final
Withholding
Tax
Rent Income - 50,000 50,000
Total Income 60,00,000 40,50,000 1,00,50,000
Allowable Expenses :
Cost of Material 18,00,000 12,00,000 30,00,000
Consumed
Manufacturing 1,50,000 1,00,000 2,50,000 Working Note 1
Expenses
Employee Cost 5,70,000 3,80,000 9,50,000 Working Note 2
Selling and 7,14,000 4,76,000 11,90,000 Working Note 3
Administrative
Expenses
Interest and Bank 3,00,000 2,00,000 5,00,000
Charges
Exchange Loss 150,000 100,000 2,50,000 Working Note 4
Bad debt Written Off - Not Allowed
Loss on Sale of Asset - Separate
Treatment
Provision for doubtful - Earlier
receivables written disallowed,
back hence not
considered in
Income
Provision for Bonus 1,20,000 80,000 2,00,000 Working Note 5
Depreciation 19,00,000 12,66,667 31,66,667 Working Note 6
Repair and 1,05,000 70,000 1,75,000 Working Note 7
Maintenance
Total Allowable 58,09,000 38,72,667 96,81,667
Expenses
Net taxable Income 1,91,000 1,77,333 3,68,333
Income Tax @ 20% 38,200 35,467 73,667
being Special
Industry
25% tax rebate for 9,550 - 9,550
export Income only as
per section
11(3)(3anga)
20% tax rebate for - 7,093 7,093
Domestic Income as
per section 11(3)(Ka)
for employing more
(90)
than 1,200 Nepali
Employees
Net Tax Liability 28,650 28,374 57,024

Common Cost and Income Allocation Ratio for Export Income and Local Income
Amount Rs. Ratio for Allocation
Cost of Material Consumed for Export 18,00,000 60% of total Cost or Income
Sales
Cost of Material Consumed for Local 12,00,000 40% of total Cost or Income
Sales
Total 30,00,000

Working Note : 1 Manufacturing Expenses :


Particulars Amount Rs.
Manufacturing Expenses as per Income Statement 5,00,000
Less : Repair and Maintenance Expenses (Separate Treatment)
Building Repair and Maintenance 1,50,000
Office Equipment Repair and Maintenance 50,000
Vehicle Repair and Maintenance 20,000
Less : Prior Period Electricity Expenses not allowed 30,000
Net Allowable Manufacturing Expenses 2,50,000
Manufacturing Expenses for Export Income – 60% of Total Expenses 1,50,000
Manufacturing Expenses for Domestic Income – 40% of Total 1,00,000
Expenses
Working Note : 2 Employee Cost
Particulars Amount
Employee Cost as per Income Statement 10,00,000
Less : Personal nature staff welfare expenses not allowed 50,000
Net Allowable Employee Cost 9,50,000
Employee Cost for Export Income – 60% of Total Expenses 5,70,000
Employee Cost for Domestic Income – 40% of Total Expenses 3,80,000
Working Note : 3 Selling and Administrative Expenses
Particulars Amount
Selling and Administrative Expenses as per Income Statement 15,00,000
Less : Donation given to School not allowed 300,000
Less : Business promotion Expenses not related to business 10,000
Net Allowable Selling and Administrative Expenses 11,90,000
Selling and Administrative Expenses for Export Income – 60% of 7,14,000
Total Expenses
Selling and Administrative Expenses for Domestic Income – 40% of 4,76,000
Total Expenses
Donation given to Prime Minister Disaster Relief Fund allowable.
Working Note : 4 Exchange Loss
Particulars Amount
Exchange Loss as per Income Statement 2,50,000
Allowable Exchange Loss 2,50,000
Exchange Loss for Export Income – 60% of Total Expenses 150,000
Exchange Loss for Domestic Income – 40% of Total Expenses 100,000

Working Note : 5 Provision for Bonus


Particulars Amount
Provision for Bonus as per Income Statement 3,00,000
Less : Undistributed Bonus 1,00,000
Net Allowable Exchange Loss 2,00,000
Bonus for Export Income – 60% of Total Expenses 1,20,000
Bonus for Domestic Income – 40% of Total Expenses 80,000
(91)
Working Note : 6 Depreciation
Particulars Block A Block B Block C Block D Total
Building Computer, Vehicles Plant and
Office Machinery
Equipment
and
Furniture
Opening Depreciation 11,00,000 15,00,000 29,50,000 1,00,00,000 65,50,000
Base
Addition upto Poush – - 2,00,000 - - 2,00,000
100%
Addition from Magh to 4,00,000 1,00,000 - - 5,00,000
Chaitra – 2/3rd
Addition for Baishak to - - 3,00,000 - 3,00,000
Ashad – 1/3rd
Less : Disposal of Asset - - (15,00,000) - (15,00,000)
Depreciation Base for 15,00,000 18,00,000 17,50,000 1,00,00,000 1,50,50,000
income year 2072-73
Depreciation rate 5% 25% 20% 15%
Depreciation Amount 75,000 4,50,000 3,50,000 15,00,000 23,75,000
Additional Depreciation 25,000 1,50,000 1,16,667 5,00,000 8,25,000
for Special Industry
1/3rd of normal
Depreciation
Total Depreciation 1,00,000 6,00,000 4,66,667 20,00,000 31,66,667
Depreciation for Export Income – 60% of Total Expenses 19,00,000
Depreciation Expenses for Domestic Income – 40% of Total Expenses 12,66,667
Working Note : 7 Repair and Maintenance
Particulars Block A Block B Block C Block D Total
Building Computer, Vehicles Plant and
Office Machinery
Equipment
and
Furniture
Depreciation Base for 15,00,000 18,00,000 17,50,000 1,00,00,000 1,50,50,000
income year 2072-73
Allowable Repair and 1,05,000 1,26,000 1,22,500 7,00,000 10,53,500
Maintenance- 7% of
Depreciation Base
Actual Repair and 1,50,000 50,000 20,000 - 2,20,000
Maintenance
Allowable Repair and 1,05,000 50,000 20,000 - 1,75,000
Maintenance
Balance to be Capitalized 45,000 - - 45,000
Next Year
Repair and Maintenance for Export Income – 60% of Total Expenses 1,05,000
Repair and Maintenance Expenses for Domestic Income – 40% of Total 70,000
Expenses

Note:
Answer will be different if Bad Debts Written Off has been shown as an expense.

7. Mohani Sharma was school English teacher in a reputed private school located at remote
area (classified as A class area by GON) between shrawan 2072 to Poush 2072. She
joined an international organization located at Kathmandu immediately after resigning
with effect from 1st Magh 2072.
Mrs. Sharma has received following salary and benefits during the income year 2072-
73:
(92)
A) Salary & benefit received from private school
Particulars Amount (Rs.)
Basic Salary (Gross) 50,000 p.m.
Dashain Allowance 50,000
House Rent Allowance 5,000 p.m.
Remote Area Allowance 10,000 p.m.
Leave Encashment 25,000
B) Salary & benefit received from INGO in Kathmandu
Particulars Amount (Rs.)
Basic Salary (Gross) 1,00,000 p.m.
Advance Salary 1 month
House Rent Allowance 25,000 p.m.
Telephone Allowance 5,000 p.m.
Travelling & Daily Allowance (TADA) 10,000/day
School tuition fees of Mrs. Sharma children directly paid by 4,000 p.m.
INGO.
Other information:
i) Mrs. Sharma managed to deposit in retirement fund approved by IRD operated by
listed commercial bank in Kathmandu. She has deposited 4,50,000 in the retirement
fund within income year 2072-73.
ii) Mrs. Sharma contributed donation of Rs. 3,00,000 to tax exempt organization.
iii) Mrs. Sharma had gone outstation for 25 days to monitor INGO activities. TADA
provided to Mrs. Sharma is only to pay cost of foods & lodging charges. Mrs.
Sharma could only submit bills of Rs. 50,000 incurred by her for food and lodging
charges.
iv) Life insurance premium paid by Mrs. Sharma Rs. 25,000.
v) Mrs. Sharma was sick and incurred medical expenses of Rs. 5,000 to hospital run by
GON.
vi) TDS deducted & deposited by school for Mrs. Sharma salary & benefits withdrawn
by her up to Poush 2072 was Rs. 63,388 and INGO had deposited 1,00,000 within
the end of Ashad 2073.
Required: (6+1+3=10)
(a) Calculate taxable income and tax liability of Mrs. Sharma for the income year
2072-73.
(b) Does Mrs. Sharma require to file income tax return as per the provision of
Income Tax Act, 2058.
(c) Compute amount of TDS need to be deposited as last installment for the salary
& benefit withdrawn by Mrs. Sharma during the income year 2072-73.
(93)
Answer

Computation of Taxable Income of Mrs. Sharma for income year 2072-73


Description Amt. Rs. Note
A. Salary & benefit received from private school:
Basic Salary –(50,000 x 6 months) 300,000
Dashain Allowance 50,000
House Rent Allowance–(5,000 x 6 months) 30,000
Remote area allowance–(10,000 x 6 months) 60,000
Leave encashment 25,000
B. Salary & benefit received from INGO in Ktm:
Basic Salary –(100,000 x 6 months) 600,000
Advance Salary-(1 month basic pay) 100,000
House Rent Allowance–(25,000 x 6 months) 150,000
Telephone Allowance–(5,000 x 6 months) 30,000
Travelling & Daily Allowance (TADA) 200,000 (Rs. 10,000*25-50,000)
Tuition fees of children –(4,000 x 6 months) 24,000

Total assessable income 1,569,000


Less: Deduction:
Contribution to approved retirement fund:
Minimum of : (300,000)
1) Maximum of Rs. 300,000
2) 1/3 of Ass. Income (Rs. 1,569,000/3)
3) Actual contribution Rs. 450,000
Adjusted taxable Income (Rs. 1,569,000-Rs. 300,000) 1,269,000
Donation to tax exempt organization:
Minimum of :
1) Maximum of Rs. 100,000
2) 5% of Adj. taxable income (5% of 1,269,000) (63,450)
3) Actual contribution Rs. 300,000
Remote area allowance- (Rs. 50,000/2)- six months (25,000) Only ½ allowed
Life insurance premium (20,000) Maximum available
Taxable Income 1,160,550

Computation of Tax Liability of Mrs. Sharma for income year 2072-73


(Assumed Couple Status)
Tax Liability
Particulars
Taxable income 1,160,550
Social security Tax 1%- up to 300,000 3,000
15%- tax up to next 100,000 15,000
25%- tax up to next 760,550 190,137
Tax Payable 208,137

Less: Reduction:
1.Medical benefit tax:
Minimum of:
1) 15 % of approved medical exp. (15 % of Rs. 5,000) 750
2) Actual eligible medical exp. + unrecovered medical exp. earlier
3) Rs. 750 per annum
Total Tax Payable 207,388
Less: Withholding Tax 163,388
Net Tax payable 44,000
(94)

Note:
 Marks will be awarded even if tax liability has been computed as per the tax rate of
Income Year 2073-74
 If she elects single natural person option then also answer will be correct and womens tax
credit shall be available to her.
 If student assume leave encashment as retirement payment then also answer will be
correct.

ii. Mrs. Sharma need to file income tax return of income year 2072-73 if she claimed/wants
to claim donation provided to tax exempt organization. However, if she does not claim/does
not want to claim donation from her income tax liability she is not required to file income tax
return for income year 2072-73.

iii. Mrs. Sharma is required to disclose amount of salary & benefits withdrawn & TDS
deducted & deposited by her previous employer (School) during relevant year to
existing/present employer (INGO). The new employer (INGO) will adjust the salary &
benefit withdrawn by her from previous employer and TDS deposited by them during relevant
year while computing assessable income and final tax liability of Mrs. Sharma for income
year 2072-73.
Total tax payable of Mrs. Sharma for the financial year 2072-73 was Rs. 2,08,137, her
previous employer deposited Rs. 63,388 and Rs. 100,000 had been already deposited by
present employer resulting balance of Rs. 44,000 as last installment of tax payable for the
income year 2072 - 73.
Note: Leave encashment is not considered as retirement payment.

8.
a) Mr. Apurva Das has 250 shares of Nabil Bank Ltd. from IPO. Bonus shares has been
distributed 300% (100% before Income Tax Act and remaining thereafter) till now.
He became a non-resident for income year 2072-73. The market value at the time of
introduction of the Act was Rs. 1,800 and at the time of becoming non-resident was
Rs. 2,600. Quoting the related criteria for deemed disposal of shares, advise him if
any tax liability arises on those shares for income year 2072-73. 5
b) Calculate the TDS to be deducted on following cases:– (2.5+2.5=5)
th
i) Mr. Santosh Sharma has bought a house on 20 Asoj 2067 for Rs. 90,00,000
including registration expenses and other expenses. Again, he has purchased a
plot of land on 3rd Chaitra 2070 for Rs. 1,50,00,000 including registration
expenses and other expenses. He sold both of the property on 22nd Mangsir 2073
to Mr. Binod Sharma for Rs. 3,00,00,000 and Rs. 3,50,00,000 respectively.
Stating the relevant provisions of the Income Tax Act, 2058, advise whether
TDS needs to be deducted or not in the said transaction, and if yes, advise who
will deduct the TDS and amount of TDS to be deducted on the above
transaction.
ii) Mr. Shyam Silwal and Everest Holdings Pvt. Ltd. are shareholders of Shree
Holdings Pvt. Ltd. Mr. Shyam Silwal holds 5,000 shares and Everest Holdings
Pvt. Ltd. holds 10,000 shares of Shree Holdings Pvt. Ltd. which is 10% and 20%
shares of total shares of Shree Holdings Pvt. Ltd. respectively.
Mr. Shyam Silwal has purchased said share from Mr. Naresh Rana on 25
Mangsir 2065 for Rs. 22,50,000. He sold the said share on 25 Poush 2073 for Rs.
30,00,000. He has paid interest of Rs. 5,00,000 against loan taken for purchase
of the said share.
(95)
Everest Holdings Pvt. Ltd. is a promoter shareholder of the Shree Holdings Pvt.
Ltd., and has paid Rs. 10,00,000. Everest Holdings Pvt. Ltd. sold the said share
on 25 Poush 2073 for Rs. 60,00,000. Everest Holdings Pvt. Ltd. has purchased
the said share from their own capital.
Stating the relevant provisions of the Income Tax Act, 2058, advise Shree
Holdings Pvt. Ltd. whether TDS needs to be deducted or not in the said
transaction, and if yes, compute TDS amount to be deducted by Shree Holdings
Pvt. Ltd. on the above transaction.
Answer
a)
According to Section 40 (3) (Cha), if a person turns to be a non-resident; his/her assets, except the
land or buildings situated in Nepal, are deemed to have made a disposal.
So, Mr. Das has made a disposal of the shares. The tax liability is calculated as below:

S. Particulars Workings Amount Rs.


No.
1. Incomings 1000 shares *Rs. 2,600 (WN1) 2,600,000.00
2. Outgoings 500 shares * Rs. 1,800.00 900,000.00
(WN2)
3. Gain Incomings – Outgoings 1,700,000.00
4. Deduction Under Section 36 Nil
5. Net Gain 1,700,000.00
6. Tax Rate 5% (WN3) 85,000.00
Workings:
1. Original Shares 250 + 300% bonus shares i.e 750 = 1000 shares. Market value is taken for
calculating incomings.
2. Outgoings for assets of before the introduction of the Income Tax Act is taken as the
market value at the time of the introduction of the Income Tax Act.
3. The tax rate for listed shares is 5% as per Section 1 (4) of schedule 1 of the
Act.
b) i)
1. As per section 95ka(3) of income tax Act, TDS will be collected by Malpot Office on disposal of Land or
Building as below at the time of registration
a. if the ownership of non business chargeable asset being Land or building is for more than 5 years,
then @ 2.5%
b. if the ownership of non business chargeable asset being Land or building is for less than 5 years,
then @ 5%
2. Hence, TDS will be deducted by Malpot Office at the time of registration as below
a. On a sale of House purchased on 20th Asoj 2067 –
i. House has been owned for more than 5 years.
ii. Hence, TDS will be deducted @ 2.5% of
Sales Proceed - Rs 3,00,00,000
Less : Cost - Rs 90,00,000
Income - Rs 2,10,00,000
TDS @ 2.5% - Rs 5,25,000
b. On a sale of Land purchased on 3rd Chaitra 2070 .
i. Land has been owned for more less than 5 years.
ii. Hence, TDS will be deducted @ 5% of
Sales Proceed - Rs 3,50,00,000
Less : Cost - Rs 1,50,00,000
Income - Rs 2,00,00,000
TDS @ 5% - Rs 10,00,000
ii)
1. As per section 95ka(2)(kha) of income tax Act, in the case of share not listed in Nepal Stock Exchange,
TDS will be collected by the Company whose shares has been disposed off on disposal of shares as below
a. if the shares are disposed off by natural person, then @ 10%
b. if the shares are disposed off by any other person other than natural person, then @ 15%
(96)
2. Hence, TDS will be deducted by Shree Holdings Private Limited at the time of disposal of shares as below –
a. On a sale of Shares by Mr. Shyam Silwal
i. TDS will be deducted @ 10% of
Sales Proceed - Rs 30,00,000
Less : Cost - Rs 22,50,000
Less : Interest - Rs 5,00,000
Income - Rs 2,50,000
TDS @ 10% - Rs 25,000
b. On a sale of Shares by Everest Holdings Private Limited
i. TDS will be deducted @ 15% of
Sales Proceed - Rs 60,00,000
Less : Cost - Rs 10,00,000
Income - Rs 50,00,000
TDS @ 15% - Rs 7,50,000

9.
a) Mrs. Aang Futi has stayed in Nepal in the following periods. Find out the residential
status of her for the income years 2072-73 and 2073-74. 5
Period of Stay
2071 Falgun 14 to Chaitra 11
2072 Baishakh 13 to Ashadh 26
2072 Shrawan 3 to Bhadra 25
2072 Marga 8 to Falgun 13
2073 Jeshtha 18 to Bhadra 16
The number of days in the relevant months are as follows:
Year (BS) Month No. of Year (BS) Month No. of
Days Days
2071 Falgun 30 2072 Poush 30
Chaitra 30 Magh 29
2072 Baishakh 31 Falgun 30
Jeshtha 32 Chaitra 30
Ashadh 31 2073 Baishakh 31
Shrawan 32 Jeshtha 32
Bhadra 31 Ashadh 31
Ashwin 30 Shrawan 32
Kartik 30 Bhadra 31
Marga 29
b) Mr. Naresh Yadav has been working as a Chief Legal Advisor of a bank. During the
income year 2072-73, he has received remuneration income of Rs. 50,00,000 from
the bank after deducting TDS of Rs. 22,00,000. During the year he has won a lottery
of Rs. 10,00,000. Company deducted TDS of Rs. 2,50,000 on the said lottery and
paid him Rs. 7,50,000. He has income of Rs. 5,00,000 from his agricultural farm. He
has also received dividend of Rs. 5,00,000 on the shares invested by him in an
insurance company. Insurance Company has deducted TDS of Rs. 25,000 on the
said amount, and paid balance Rs. 4,75,000. He has also received bonus share of Rs.
2,00,000 on the shares invested by him in a Hydro Power Company. Hydro Power
Company has deposited TDS of Rs. 10,000 on the said bonus share. 5
Discussing the relevant provisions of the Income Tax Act, 2058 determine whether
he is required to file Income Tax Return or not, if yes, prepare his Income Tax
Return.
c) Sitaram Poudel has received a notice from Tax officer on 1st Chaitra 2072 for
submission of Income Tax Return for income year 2072-73 by 1st Baishakh 2073.
He has been planning to migrate to Australia. Sitaram Poudel’s contention is he need
(97)
not have to submit tax return since he has income from employment only, and due
date of filing income tax return is also end of Asoj.
Stating the relevant provisions of Income Tax Act, 2058, please advise Sitaram
Poudel on the above matter. 5
d) Calculate the tax liabilities for income year 2072-73 in the following cases. 5
i) Fine Distillery Pvt. Ltd. in Mustang District started its business of producing
apple cider with 11% alcohol by volume since 2069-70. For the first time during
the income year 2072-73 company made taxable profit of Rs. 1,150,080 prior to
the adjustment of previous years assessed loss of Rs. 6,55,000.
ii) Gulf Bank, a Dubai based banking company has an office in Kathmandu for its
wholesale banking service to local banks. During the year it made Rs. 75,00,000
profit from the operation in Nepal of which Rs. 50,00,000 was repatriated to
head office Dubai.
Answer
a)
Test for 2072/73
Step 1: The last day of her stay in the Income Year is 2073 Ashadh End and the previous 365 days
is from 2072 Shrawan 1.
Period of Stay Days Total Days in Cumulative Days
the Period
2072 Shrawan 3 to Bhadra 25 30+25 55 55
2072 Marga 8 to Falgun 13 22+30+29+13 94 149
2073 Jeshtha 18 to Ashadh 15+31 46 195
End
Since, she has stayed 183 days or more i.e 195 days in the consecutive previous 365 days, she is a
resident for Income Year 2072/73 as per Clause (KaNga)(1) (Kha) of Section 2.

Test for 2073/74


Step 1: The last day of her stay is 2073 Bhadra 16 and the previous 365 days is from 2072 Bhadra
17.

Period of Stay Days Total Days in Cumulative


the Period Days
2072 Bhadra 17 to Bhadra 25 9 9 9
2072 Marga 8 to Falgun 13 22+30+29+14 95 104
2073 Jeshtha 18 to Bhadra 16 15+31+32+16 94 198
Since, she has stayed 183 days or more in the consecutive previous 365 days, she is a resident for
Income Year 2073/74 as per Clause (KaNga) (1) (Kha) of Section 2.
Notes:
1. In lack of precise mention in the Act, both the arrival and departure dates are taken as the
period of stay.
2. Had the first test not satisfied for the condition of a residential status, second test could be
carried in the consecutive 365 days looking back from the end date of interrupted previous
period’s stay in that Income Year
3. If students refer provision as per Income Tax Directive (Income Year wise) then also marks
will be awarded.

b)
As per section 97(2) of Income Tax Act, natural person having income of more than Rs 40 lacs per annum
compulsorily needs to file Income Tax return. Hence, Mr. Naresh Yadav has to file Income Tax Return
Income Tax Return of Mr. Naresh Yadav for income year 2072-73
(2 marks each for inclusion and deduction part)
(98)
Particulars Amount
Inclusion
Remuneration Income (gross) 72,00,000
Windfall Gain 10,00,000
Agriculture Income 5,00,000
Dividend from Insurance Company 5,00,000
Bonus Share from Hydro Power Company 2,00,000
Total Income 94,00,000
Less : Deductions
Windfall Gain subject to final withholding 10,00,000
Tax Exempt Income u/s 10 - Agriculture Income 5,00,000
Income Received after deduction of Final With holding 5,00,000
Tax : Dividend from Insurance Company
Income Received after deduction of Final With holding 2,00,000
Tax : Bonus Share from Hydro Power Company
Total Deduction 22,00,000
Net Taxable Income 72,00,000

c)
As per section 96(5) of Income Tax Act, 2058, Tax Office can issue notice in writing to any person for
filing of tax return of any income year or part of any income year even before date of filing of income tax
return as per section 96(1) subject to provisions of section 100 in below cases
1. If the said person has become insolvent, or is under heavy debt, or has been dissolved
2. If the said person is leaving Nepal for uncertain period of time
3. If the said person is going to discontinue the work in Nepal
4. In any other case as deemed justified by Tax office

Hence, considering the above provision of Income tax Act, 2058, it is advised to Sitaram for submission of
Tax Return as asked by the Tax Office since he is planning to migrate to Australia
d)
i) Tax Liabilities of Fine Distillery

Profit before carried loss setoff 1,150,080


Less: unrelieved loss of the previous years 655,000
Taxable Profit 495,080
Tax @ 30% 148,524
Less: 40% tax rebate for producing fruit based wine in
extremely undeveloped area
59,409
Final Tax Liability 89,114

ii) Tax Liabilities of Gulf Bank


Tax on profit from the operation in Nepal @ 30 %
2,250,000
Tax on profit repatriated to Dubai @ 5% 250,000
Total Tax Liability of Gulf Bank 25,00,000
(99)
10.
a) Define the following terms as per Income tax Act, 2058. (2×2.5=5)
i) Company
ii) Natural person
b) What are the possible methods of taking foreign tax credit to avoid double taxation
of resident person? 5
Answer
a)
i) Company:
As per section 2 (da) “Company” means any company incorporated under the company’s law in
force, and for the purpose of tax following institutions shall also be treated as if they were
companies:
a. Any corporate body established under the law in force;
b. Any unincorporated union, board, association or society or except registered or
unregistered sole proprietorship firm and partnership or trust;
c. Any partnership firm consisting of Twenty or more partners whether registered or not under
the law in force, and any retirement fund, cooperatives institution, unit trust, joint venture;
d. Any foreign company;
e. Any foreign institution as specified by Director General.

ii) Natural Person:


“Natural person” as defined by section 2 (wa) as follows:
a. A single natural person;
b. A proprietorship firm 100% owned by a single natural person;
c. A couple opted being as a single taxation unit under section 50;
d. A natural person being widow or widower with dependent opted being as a single taxation
unit under section 50.
b)
The following methods of foreign tax credit is prescribed as per Section 71 of the Income Tax Act,
2058 to avoid double taxation of a resident person:
a) Credit Method: Income from foreign country is taxable to the person in his country of residence
and permitted to set off of tax paid in foreign country (generally to the extent of effective tax rate
of country of residence)
b) Expense Method: Tax paid in foreign country is eligible to claim as expenses while computing
taxable income in his country of resident

11. LUC Group Nepal is an emerging business house established by group of clever
business minds. It carries diversified business activities under same permanent account
number and group motto is halo to hydro. Business activities of the group for the month
of Bhadra 2073 is as follows:
a) Account receivable from a client for contract completed, approved and invoiced
during the month Rs. 36,55,434.
b) Sales of cellular mobile phone during the month Rs. 12,55,660 of which amount Rs.
4,56,360 was sold to non registered customers.
c) Fruits & vegetable sales during the month Rs. 5,34,600 of which Rs. 1,53,600 was
sales of packed garlic in stock since last 3 months.
d) Deep cycled lid acid battery sales Rs. 12,56,400 out of total Rs. 3,25,600 was sold to
Safa Tempo workshop with recommendation from Ministry of Population &
Environments.
e) Industrial machineries for bakery, confectionary & distillery falling under
Harmonized code 84.39 sold during the month Rs. 55,24,300
(100)
f) The group has following purchase/Import during the month exclusive of VAT,
where applicable
 Cement, Iron Nails, Bricks Rs. 21,22,456
 Cellular Mobile & its parts Rs. 16,56,400
 Battery Rs. 7,85,900
 Tower cranes for construction use Rs. 55,60,300
 Fruits & Vegetable Rs. 4,26,300
 Bakery, confectionary & distillery machineries for trading purpose Rs.
60,65,480
 Statutory & internal audit fees Rs. 4,75,000
 Computer for official use Rs.3,25,400
 Stationeries Rs. 5,25,600
 Purchase of car for group president Rs. 40,00,000
g) During the same month internal auditor of the group submitted their report to the
management that reveals:
 Purchase book of VAT for the month of Shrawan 2072 was over casted by Rs.
87,500.
 No credit claim for the VAT amount Rs. 56,980 paid to department of roads for
auction purchase of road roller during the month of Kartik 2072.
 No adjustment for batteries used for office purpose amounting Rs. 45,600 during
the month of Jestha 2073.
Required: (8+2=10)
i) VAT statement for the month of Bhadra 2073 and VAT payable by the group
assuming there was no opening VAT credit.
ii) Discuss about VAT refundable, if any.
Answer
1. Statement of VAT collection by LUC Group during B
2073…………………………….
Particulars Standard Zero Exempt Total
Construction Revenue billed
(3655434/1.13) 3,234,897.35 3,234,897.35
Sales of Cellular mobile phone 1,255,600.00 1,255,600.00
Fruits & Vegetable 153,600.00 381,000.00 534,600.00
Battery Sales 930800 325,600.00 1,256,400.00
Industrial Machineries 5,524,300.00 5,524,300.00
Deemed sale of battery self use
during Jestha 2073 reported by
internal auditor
45,600.00 45,600.00
Total 5,620,497.35 6,230,900.00 11,851,397.35
Tax Rate 13% -

Tax collection 730,664.65 - 730,664.65

Proportionate ration of Taxable & exempt transaction

Total Transaction 11,851,397.35 100%


Taxable Transaction 5,620,497.35 47.42%
Exempt Transaction 6,230,900.00 52.58%

2. Statement of VAT Credit during Bhadra 2073


(101)
Particulars Full Partial No Total
Cement, iron, nail & bricks 2,122,456.00 2,122,456.00
Cellular mobile 1656400 1,656,400.00
Battery 785,900.00 785,900.00
Statutory & internal Audit fees 475,000.00 475,000.00
Computer for office 325,400.00 325,400.00
Stationeries 525,600.00 525,600.00
Car Purchase (4000000*0.4) 1,600,000.00 1,600,000.00

Total 4,564,756.00 2,926,000.00 - 7,490,756.00

VAT paid on Purchase 593,418.28 380,380.00 - 973,798.28

Credit eligible VAT Ratio 100% 47.42%% -

Credit eligible VAT amount 593,418.28 180,376.20 - 773794.48

(Fruits & vegetables, machineries & tower crane are non VAT items listed in Schedule 1)

i. VAT Statement for the Month of Bhadra

VAT Collected on sales (Note 1) 730,664.65


VAT Paid on Purchase (Note 2) 773,794.48
VAT payable/(credit forward) (43129.83)
Opening VAT Credit -
Adjusted VAT credit(Road Roller Purchased from DOR during Kartik 2072)
(56,980.00)
Total VAT credit forwarded for Aswin 2073 (100,109.83)

VAT refundable:

Of the total cellular mobile phones Rs. 799,300.00 was sold to registered buyers. LUC Group Nepal can claim
for refund of 40% VAT paid on purchase/import of Cellular mobile & its accessories that is sold to registered
taxpayers.
Notes:
 Deep cycled lid acid battery is included in both schedule 1 & 2. Students treating
this item as exempt or 0 rated both are correct answer.
 Mistake in casting of VAT Purchase account has no adjustments as the period of
mistake has elapsed one year.
12.
a) ABC Oil & Ghee industry having factory at Lalitpur, Nepal producing Mustard oil,
Vanaspati ghee and other processed cooking oil in Nepal as given below. The sales
of ABC Oil & Ghee industry to VAT registered and unregistered person are in the
ration of 40% and 60% respectively. The ABC Oil & Ghee industry had following
sales transaction during the month of Falgun, 2073:
(Amount in Rs.)
S.N. Particulars Sale
1. Mustard oil- (Domestic mustard seeds) 6,00,000
2. Mustard oil- (Imported mustard seeds) 4,00,000
3. Vanaspati Ghee- (Domestically produced) 4,00,000
4. Vanaspati Ghee- (Imported) 6,00,000
5. Other processed cooking oil 10,00,000
Total 30,00,000
(102)
Whereas XYZ oil industry having factory at Bhaktapur import processed edible
cooking oil in bulk quantities and refill it in small packages before sale. The XYZ
oil industry had following sales transaction during the month of Falgun, 2073:
(Amount in Rs.)
S.N. Particulars Sale
1. Refilled small oil packages (Imported 10,00,000
processed edible cooking oil in bulk
quantities)
Total 30,00,000
Required: 5
Compute the amount of VAT refund available if any to ABC Oil & Ghee industry
and XYZ Oil Industry with reference to the provision of schedule I of VAT Act.
b) Recondition House Baneshwor dealing in used goods made following Purchase &
sales during Kartik 2073:
Item Purchase with VAT (Rs.) Sales without VAT (Rs.)
Sofa 14000 21000
Freeze 32000 43000
Wooden Furniture 45000 38000
Calculate the amount of VAT payable by recondition house. 5
Answer
a)
Any domestic industry producing Mustard oil, domestically produced Vanaspati Ghee and other
processed cooking oil in Nepal and sells to registered person then such domestic industry can have
facility of Vat refund (Square off arrangement) as per schedule-I of Vat Act. Vat collected only to
the extent of 40% from registered person on sale of Mustard oil, domestically produced Vanaspati
Ghee and other processed cooking oil are refundable in accordance with the procedure specified by
Inland Revenue Department. However the facility of such VAT refund under schedule-I is not
available on vat collected on sale of Vanaspati Ghee not produced in Nepal and to those domestic
industries which import only processed edible cooking oil in bulk quantities and refill it in small
packages before sale

Sales to registered person are only 40% of total sales hence sales & VAT collected from registered
person are as follows:
Amount in Rs.
S.N. Particulars Sale to Reg. VAT
Person only (40%) Collected
1. Mustard oil- (Domestic mustard seeds) 240,000 31,200
2. Mustard oil- (Imported mustard seeds) 160,000 20,800
3. Vanaspati Ghee- (Domestically produced) 160,000 20,800
4. Vanaspati Ghee- (Imported) 240,000 31,200
5. Other processed cooking oil 400,000 52,000
Total 1,200,000 156,000

Vat collected only to the extent of 40% from registered person on sale of Mustard oil, domestically
produced Vanaspati Ghee only and other processed cooking oil, hence the ABC Oil & Ghee
industry can have facility of VAT refund under schedule-I of Vat Act are as follows:
Amount in Rs.
S.N. Particulars VAT Collected VAT Refund
from Reg. Person (40%)
1. Mustard oil- (Domestic mustard seeds) 31,200 12,480
2. Mustard oil- (Imported mustard seeds) 20,800 8,320
3. Vanaspati Ghee- (Domestically produced) 20,800 8,320
4. Vanaspati Ghee- (Imported) 31,200
(103)
5. Other processed cooking oil 52,000 10,400
Total 156,000 39,520
ABC Oil & Ghee industry can have facility of VAT refund of Rs. 39,520 as per schedule-I of Vat
Act
However XYZ oil industry import only processed edible cooking oil in bulk quantities and refill it
in small packages before sale, hence it could not have facility of VAT refund under schedule-I of
VAT Act.

b)
Statement of VAT payable by Recondition House Baneshwar……………………3
Purchase Sales Taxable VAT
inclusive exclusive Value Payable
Item of VAT of VAT
Sofa 14,000.00 21,000.00 7,000.00 910.00
Freeze 32,000.00 43,000.00 11,000.00 1,430.00
Wooden Furniture 45,000.00 38,000.00 -
Total 2,340.00

There is negative value addition of Rs. 7000 from wooden furniture. It is required to determine
taxable value of each individual used item and there is no provision for setoff for negative value
addition. Recondition house is not required to issue tax invoice for sale of wooden furniture

13. Answer the followings with reference to VAT Act/Rules (4×2.5=10)


a) State the provisions on input tax credit on VAT paid on the lost goods
b) Advise about the threshold of procurement for goods, service, and construction
contract that a public institution requires to procure mandatorily from VAT
registered party.
c) Can an unregistered person collect VAT?
d) Enumerate the transactions that are VAT attracted.

Answer
a)
In case the loss of assets by fire, theft, accident, accidental damages, terror or riot compels a person to
write off the goods/(assets) or sale it at lower selling rate, the person shall made an application in
writing to respective Inland Revenue Office along with evidence within 30 days of happening of such
incidence.
The tax office shall investigate the matter and finalize the quantum of tax credit to be allowed to claim.
On the basis of such investigation, the tax office may allow the tax payer to claim input tax credit of
vat paid on such assets/ (goods).
In case the assets are insured the tax officer may allow the tax payer to claim input tax credit on such
goods to the extent of compensation paid by the insurance company.

b)
As per Rule No 6Ka of VAT Rules, 2053; a Government body, public institution or registered person
should acquire consultancy service for more than Rs. 1 lakh in a year or providing contract for more than
Rs. 20 lakh only from registered person.
Likewise, Rule 56 mentions that while purchasing or acquiring the goods or services on which tax of
value of five thousand rupees is payable at a time within the State of Nepal by GON or the association,
organization or office owned by GON or constitutional bodies, such goods or services shall be
purchased or acquired only with a registered person.
(104)
c)
As per Section 15 of VAT Act, normally a person who is not registered shall not issue an invoice or
other document showing the collection of tax and shall not collect tax. But Sub-section 3 of Section
mentions that tax should be recovered on sale of taxable goods or services by local body,
international organization or mission situated in Nepal, Government of Nepal or public corporations
dealing in VAT exempt goods.
d)
According to Section 5 (1) of VAT Act, except otherwise provided for in this Act, VAT shall be imposed on the following
transactions:
(Ka) Goods or services supplied into the State of Nepal,
(Kha) Goods or services imported into the State of Nepal,
(Ga) Goods or services exported outside the State of Nepal,
Sub-section 3 stipulates that notwithstanding Subsection (1), no tax shall be levied on the transactions of
goods or services set forth in Schedule 1.
CAP II Paper 1: Advanced Accounting

CHARTERED ACCOUNTANCY PROFESSIONAL II


(CAP-II)

Suggested Answer
June 2018

The Institute of Chartered Accountants of Nepal

The Institute of Chartered Accountants of Nepal 1


Suggested Answer - June 2018

Paper 1: Advanced Accounting

The Institute of Chartered Accountants of Nepal 2


CAP II Paper 1: Advanced Accounting

Maximum Marks - 100


Total No. of Questions - 6 Total No. of Printed Pages -14
Time Allowed - 3 Hours
Marks
Attempt all questions. Working notes should form part of the answer.
1. A and B were carrying on business sharing profits and losses equally.
The firm's Balance Sheet as at 31.03.2074 was:
Liabilities Rs. Assets Rs.
Capital Accounts: Plant 1,60,000
A 1,50,000
B 1,30,000 2,80,000
Sundry Creditors 80,000 Building 48,000
Bank Overdraft 45,000 Debtors 75,000
Stock 70,000
Joint Life Policy 6,000
Profit & Loss A/c 30,000
Drawing Account: 16,000
A 9,000
B 7,000
Total 4,05,000 Total 4,05,000
The operations of the business were carried on till 30.09.2074. A and B both withdrew in
equal amount half the amount of profit made during the current period of six months
after charging depreciation at 10% per annum on plant and after writing off 5% on
building . During the current period of six months, creditors were reduced by Rs. 20,000
and bank overdraft by Rs. 5,000.
The life policy was surrendered for Rs. 6,000 before 30th Poush, 2074. Stock was valued
at Rs. 84,000 and debtors at Rs. 68,000 on 30th Poush, 2074. The other items remained
the same as 31.03.2074.
On 30.09.2074, the firm sold its business to AB Ltd. The value of goodwill was estimated
at Rs. 1,30,000 and the remaining assets were valued on the basis of the balance sheet as
on 30.09.2074.
AB Ltd. paid the purchase consideration in equity shares of Rs. 10 each.
You are required to prepare; 20
a) Balance Sheet of the firm as at 30.09.2074,
b) Realization account,
c) Partners' Capital Accounts showing the final settlement between them.
Answer:
a) Balance Sheet of the firm as at 30.09.2074
Liabilities Rs. Assets Rs.

The Institute of Chartered Accountants of Nepal 3


Suggested Answer - June 2018

Capital Account : Plant :


A's Capital 1,33,800 Opening Balance 1,60,000
B's Capital 1,15,800 Less: Depreciation @ 10% 8,000 1,52,000
Creditors 60,000 Building :
Bank Overdraft 40,000 Opening Balance 48,000
Less: Written-off @ 5% 2,400 45,600
Debtors 68,000
Stock 84,000

Total 3,49,600 Total 3,49,600

b) Realisation Account
Dr.
Cr.
Particular Amount Particular Amount
To Sundry Assets: By Creditors 60,000
Plant 1,52,000 By Bank Overdraft 40,000
Building 45,600
Stock 84,000 By AB Ltd A/c 3,79,600
Debtors 68,000 (working note 2 )
To Profit:
A's Capital A/c 65,000
B's Capital A/c 65,000
------------- ---------------
4,79,600 4,79,600
c) Partner's Capital Accounts
Date Particular A (Rs.) B (Rs.) Date Particular A (Rs.) B (Rs.)
01.04.74 To Profit & 01.04.74 By balance b/d 1,50,000 1,30,000
Loss A/c 15,000 15,000
01.04.74 To Drawing A/c 9,000 7,000 30.09.74 By Profit 15,600 15600
30.09.74 To Drawing A/c 7,800 7,800 (W.N.1)
(W. N.1)
30.09.74 To Balance c/d 1,33,800 1,15,800 ---------- ----------
- --
Total 1,65,600 1,45,600 Total 1,65,000 1,45,600
30.09.74 By Balance b/d 1,33,800 1,15800

30.09.74 To shares in AB 1,98,800 1,80,800 30.09.74 By Realisation 65,000 65,000


Ltd. A/c (Profit)
1,98,800 1,80,000 1,98,800 1,80,000

Working Notes:
(1) Ascertainment of profit for the period of 6 Months ended 30.09.2074
Amount (Rs)
Closing Assets :

The Institute of Chartered Accountants of Nepal 4


CAP II Paper 1: Advanced Accounting

Stock 84,000
Debtors 68,000
Plants less Depreciation 1,52,000
Building Less Written off 45,600
Total 3,49,600
Less: Closing Liabilities :
Creditors 60,000
Bank Overdraft 40,000 1,00,000
Closing Net Assets 2,49,600
Less: Opening adjusted Capitals
A(Rs. 1,50,000 - Rs. 15,000 - Rs. 9,000) 1,26,000
B(Rs. 1,30,000 - Rs. 15,000 - Rs. 7,000) 1,08,000 2,34,000
Profit Net of drawings 15,600
Actual Profit for six months before drawings (half of profit )=15,600x2 31,200
Combined Drawing during six months (half of profit) 15,600
(2) Ascertainment of purchase consideration
Rs.
Closing Net Assets (As above) 2,49,600
Add: Goodwill 1,30,000
Total Purchase consideration 3,79,600

2.
a) The following is the Balance Sheet of a concern on 31st Ashadh, 2073:
Liabilities Rs. Assets Rs.
Capital 10,00,000 Fixed Assets 4,00,000
Creditors (Trade) 1,40,000 Stock 3,00,000
Profit & Loss A\c 60,000 Debtors 1,50,000
Cash & Bank 3,50,000
12,00,000 12,00,000
The management estimates the purchases and sales for the year ended 31st Ashadh,
2074 as under:
Particulars Upto 32.2.2074 (Rs.) Ashadh 2074 (Rs.)
Purchases 14,10,000 1,10,000
Sales 19,20,000 2,00,000
It was decided to invest Rs. 1,00,000 in purchases of fixed assets, which are
depreciated @ 10% on cost.
The time lag for payment to Trade Creditors for purchase and receipt from Sales is
one month. The business earns a gross profit of 30% on turnover. The entire sales and
purchases are taken on credit basis. The expenses against gross profit amount to 10%
of the turnover. The amount of depreciation is not included in these expenses.
Draft a Balance Sheet as of 31st Ashadh, 2074 assuming that creditors are all Trade
Creditors for purchases and debtors for sales and there is no other item of current
assets and liabilities apart from stock and cash and bank balances. 10

The Institute of Chartered Accountants of Nepal 5


Suggested Answer - June 2018

b) From the following Balance Sheets of Mr. Ram, prepare a Cash Flow Statement as per
NAS 3 for the year ended 31.03.2074:

Balance Sheet of Mr. Ram


As on 1.4.2073 As on 31.03.2074
Liabilities:
Ram‘s Capital Account 5,00,000 6,12,000
Sundry creditors 1,60,000 1,76,000
Mrs. Ram‘s loan 1,00,000 -
Long term loan from bank 1,60,000 2,00,000
9,20,000 9,88,000

Assets:
Land 3,00,000 4,40,000
Plant & Machinery 3,20,000 2,20,000
Stock 1,40,000 1,00,000
Debtors 1,20,000 2,00,000
Cash 40,000 28,000
9,20,000 9,88,000
Additional information:
A machine costing Rs. 40,000 (accumulated depreciation there on Rs. 12,000) was
sold for Rs. 20,000. The provision for depreciation on 1.4.2073 was Rs. 1,00,000 and
on 31.03.2074 was Rs. 1,60,000. The net profit for the year ended on 31.03.2074 was
Rs. 1,80,000. 10

Answer:
a)
Projected Balance Sheet of
As on 31st Ashadh, 2074
Liabilities Rs Assets Rs
Capital 10,00,000 Fixed Assets 4,00,000
Profit & Loss Additions 1,00,000
Account as on
1st Shrawan, 60,000 5,00,000
2073
Add: Profit for 3,74,000 4,34,000 Less: Depreciation (50,000) 4,50,000
the year
Creditors 1,10,000 Stock in trade 3,36,000
(Trade)
Sundry Debtors 2,00,000
Cash & Bank Balances 5,58,000
15,44,000 15,44,000
Working Notes:
Projected Trading and Profit and Loss Account

The Institute of Chartered Accountants of Nepal 6


CAP II Paper 1: Advanced Accounting

For the year ended 31st Ashadh, 2074


Particulars Rs Particulars Rs
To Opening Stock 3,00,000 By Sales 21,20,000
To Purchases 15,20,000 By Closing Stock 3,36,000
(balancing figure)
To Gross Profit c\d (30% on 6,36,000
sales)
24,56,000 24,56,000
To Sundry Expenses (10% on 2,12,000 By Gross Profit b\d 6,36,000
sales)
To Depreciation 50,000
To Net Profit 3,74,000
6,36,000 6,36,000
Cash and Bank Account
1st Shrawan, 2073 to 31st Ashadh, 2074
Rs. Rs
.
To Balance b/d 3,50,000 By Sundry Creditors 15,50,000
To Sundry Debtors 20,70,000 (Rs1,40,000+Rs 14,10,000)
(Rs 1,50,000 + Rs By Expenses 2,12,000
19,20,000)
By Fixed Assets 1,00,000
By Balance c/d 5,58,000
24,20,000 24,20,000

b) Cash Flow Statement of Mr. Ram for the year ended 31.03.2074
Rs.
(i)Cash flow from operation activities
Net Profit (given) 1,80,000
Adjustment for Depreciation on Plant & Machinery 72,000
(W.N.2) 8,000 80,000
Loss on sale of Machinery (W.N.1) 2,60,000
Operating profit before working capital changes 40,000
Decrease in Stock (80,000)
Increase in Debtors 16,000 ( 24,000)
Increase in Creditors 2,36,000
Net cash from operating activities
(ii) Cash flow from investing activities: 20,000
Sale of Machinery (1,40,000)
Purchase of Land(4,40,000-3,00,000) (1,20,000)
Net cash used in investing activities
(iii) Cash flow from financing activities: (1,00,000)
Payment of Mrs. Ram‘s Loan (68,000)
Drawings(W.N. 3) 40,000
Loan from bank (1,28,000)

The Institute of Chartered Accountants of Nepal 7


Suggested Answer - June 2018

Net cash used in financing activities (12,000)


Net decrease in cash 40,000
Cash balance as on 1.4.2073 28,000
Cash balance as on 31.03.2074

Working Notes:
1 Plant & Machinery A/c
Rs. Rs.
To Balance b/d 4,20,000 By Bank-sales 20,000
( 3,20,000+1,00,000) By Provision for
depreciation A/c 12,000
By Profit & Loss A/c-loss
on sale(40,000-20,000- 8,000
12,000) 3,80,000
4,20,000 By balance c/d 4,20,000
(2,20,000+1,60,000)
2 Provision for depreciation on Plant & Machinery A/c
Rs. Rs.
To Plant & Machinery A/c 12,000 By Balance b/d 1,00,000
To Balance c/d 1,60,000 By Profit & Loss 72,000
1,72,000 A/c(Bal.fig) 1,72,000

3.Mr. Ram‘s Drawing


Rs.
Opening Capital 5,00,000
Add: Net profit 1,80,000
6,80,000
Less Closing Capital (6,12,000)
Drawings 68,000
3.
a) Pashupati Stores Pvt Ltd with its head office at Kathmandu, invoiced goods to its
branch at Gajuri at 20% less than the list price which is cost plus 100%, with
instruction that cash sales were to be made at invoice price and credit sales at list
price.
From the following particulars available from the branch, prepare Branch Stock
Account, Branch Adjustment A/c, Branch Profit and Loss A/c and Branch Debtors
A/c for the year ending 31st Chaitra, 2074. 10
Particulars Rs. Rs.
Stock on 1st Baishakh, 2074 ( invoice price) 6,000
Debtors on 1st Baishakh, 2074 5,000
Goods received from Head Office (invoice price) 66,000
Sales
Cash 23,000

The Institute of Chartered Accountants of Nepal 8


CAP II Paper 1: Advanced Accounting

Credit 50,000 73,000


Cash received from debtors 42,817
Expenses at branch 8,683
Debtors on 31st Chaitra, 2074 12,183
Stock on 31st Chaitra, 2074 (invoice price) 8,800
Remittance to Head Office 60,000

b) Following balances relating to loan and advances are extracted from records of a ‗A‘
class Commercial Bank.
Amount in lakhs
Categories of Loan Insured Amount Other Amount Total Amount
Pass 3,335 161,528 164,863
Restructured/Rescheduled 775 4,464 5,239
Substandard 138 706 844
Doubtful 217 1,927 2,144
Loss 662 3,539 4,201
Total 5,127 172,164 177,291

While scrutinizing the records of the bank, it was found that the bank has provided a
credit of Rs. 5,560 lakhs in excess of SOL. The bank has categorized this loan as
Pass. Find out the amount of loan loss provisions as per NRB Directives. 5
Answer:
a) In the Books of Pashupati Stores Pvt Ltd:
Dr Branch Stock Account Cr
Particulars Amount Particulars Amount
To Balance b/d 6,000 By Bank ( Cash Sales) 23,000
To Goods sent to Branch 66,000 By Branch Debtor 50,000
To Branch Adjustment( W.N-1) 10,000 By Shortage in stock (W. N-2) 200
By Balance c/d 8,800
Total 82,000 Total 82,000

In the Books of Pashupati Stores Pvt Ltd


Dr Branch Adjustment Account Cr
Particulars Amount Particulars Amount
To Shortage in stock (W.N-2) 75 By Branch Stock(W.N-1) 10,000
To Stock Reserve (W.N-5) 3,300 By Stock Reserve (W.N-3) 2,250
To Gross Profit & Loss 33,625 By Goods sent to Branch (W.N-4) 24,750
Total 37,000 Total 37,000

The Institute of Chartered Accountants of Nepal 9


Suggested Answer - June 2018

In the Books of Pashupati Stores Pvt Ltd


Dr Branch Profit and Loss Account Cr
Particulars Amount Particulars Amount
To Branch Expenses 8,683 By Gross Profit 33,625
To Shortage in sales (cost) 125
To General Profit and Loss 24,817
Total 33,625 Total 33,625

In the Books of Pashupati Stores Pvt Ltd


Dr Branch Debtors Account Cr
Particulars Amount Particulars Amount
To Balance b/d 5,000 By Bank 42,817
To Branch Stock 50,000 By Balance c/d 12,183
Total 55,000 Total 55,000
Working Notes
1) Let the Cost price= X
List price = 100% of cost price
=100 +100= Rs.200
Invoice price = 20% less than List price
=200-(20% of 200)
=Rs.160
Cash sales are made at invoice price i.e.Rs.160 and credit sales are made at list
price i.e.Rs.200
Excess amount charged on credit sales= Rs.200-Rs.160= Rs.40 i.e.
40/200*100%= 20 % of list price.
Good sold on credit= Rs.50, 000
Amount charged in excess of the invoice price = 20% of list price
= 20% of Rs.50, 000
= Rs.10, 000
2) If the cost =Rs.100 then invoice price = Rs.160
Therefore loading on invoice price = 60/160*100%=37.5%
Loading on Shortage =37.5% of Rs.200= Rs.75
= Rs.200-Rs.75= Rs.125
3) Loading of Opening stock
=37.5% of Rs.6, 000=Rs.2, 250
4) Loading on Goods sent to Branch
=37.5% of Rs.66, 000=Rs.24, 750
5) Loading on closing stock
=37.5% of Rs.8, 800=Rs.3, 300
b)
Computation of Loan Provisions Amount
(Amount in Lakhs)

The Institute of Chartered Accountants of Nepal 10


CAP II Paper 1: Advanced Accounting

Categories Insured Rate Provision Other RateProvision Total


of Loan Amount Amount Amount Provision
Amount
Pass 3,335 0.25 8 161,528 1% 1615 1624
Restructured 775 3.13 24 4,464 12.5% 558 582
/Rescheduled
Substandard 138 6.25 9 706 25% 177 185
Doubtful 217 12.5 27 1,927 50% 964 991
Loss 662 25 166 3,539 100% 3539 3705
Total 5,127 234 172,164 6852 7086
Additional 100% Provision for Credit Provided in Excess of SOL (B) =5560
Total Loan Loss Provisions (A+B) = 12646
Working Note:
Provisioning Rate for Insured Loan i.e. = 25% of prescribed rate
1×0.25=0.25%
12.5×0.25=3.125
20×0.25=6.25%
50×0.25=12.5%
100×0.25=25%

4.
a) Retirement Benefit Plan of ABC Bank Ltd. purchased preference shares of XYZ
Bank Ltd. face value of Rs. 1,000,000 at Rs. 1,037,000 on 1st Shrawan, 2069. The
coupon rate of the preference shares is 12% and maturity date of the shares is 31 st
Ashadh, 2074. You are required to prepare the investment account in the books of
retirement benefit plan up to the year ended 31st Ashadh, 2074. The effective interest
rate of the investment is 11%. 10
b) From the following information relating to M/s Genuine General Insurance find out the
revenue profit for the period ended 31.12.2074. 5
in Lakhs
Gross premium (unrealized Rs. 120) 8,000
Claim paid (including cheque issued but not collected Rs. 95) 1,680
Other administrative expenses 1,455
Unexpired risk reserve (opening) 2,450
Unexpired risk reserve (closing) 2,760
Outstanding claim liability (opening) 1,440
Outstanding claim liability (closing) 1,230
As per company's strategy 35% business is covered under reinsurance treaty,
which provides average 6.5% commission.

a)
Retirement Benefit Plan of ABC Bank Ltd.
Investment Account
Date Particulars Amount Date Particulars Amount

The Institute of Chartered Accountants of Nepal 11


Suggested Answer - June 2018

1,04.2069 To Bank A/c 10,37,000 31.03.2070 By Bank A/c 5,930


31.03.2070 By Balance C/d 10,31,070

Total 10,37,000 Total 10,37,000


1,04.2070 To Balance B/d 10,31,070 31.03.2071 By Bank A/c 6,582
31.03.2071 By Balance C/d 10,24,488

Total 10,31,070 Total 10,31,070


1,04.2071 To Balance B/d 10,24,488 31.03.2072 By Bank A/c 7,306
31.03.2072 By Balance C/d 10,17,181

Total 10,24,488 Total 10,24,488


1,04.2072 To Balance B/d 10,17,181 31.03.2073 By Bank A/c 8,110
31.03.2073 By Balance C/d 10,09,071

Total 10,17,181 Total 10,17,181


1,04.2073 To Balance B/d 10,09,071 31.03.2074 By Bank A/c 9,071
31.03.2074 By Bank A/c 10,00,000

Total 10,09,071 Total 10,09,071


Amortization Schedule
Year (A) Beginning (B) Dividend (C) Reported (D) (E) End-of-
of period and maturity dividend Amortization period
amortized cost cash inflow income of Premium amortized
[=(A)×11%] [=(C) – (B)] cost[=(A)+(D)]
2069- 10,37,000 1,20,000 1,14,070 5,930 10,31,070
70
2070- 10,31,070 1,20,000 1,13,418 6,582 10,24,488
71
2071- 10,24,488 1,20,000 1,12,694 7,306 10,17,181
72
2072- 10,17,181 1,20,000 1,11,890 8,110 10,09,071
73
2073- 10,09,071 11,20,000 1,10,929 9,071 10,00,000
74
b)

Statement of calculation of Revenue Profit


For the period ended 31.12.2074
in Lakhs
Gross premium (less unrealized Rs. 120) 7,880.00

The Institute of Chartered Accountants of Nepal 12


CAP II Paper 1: Advanced Accounting

Less: reinsurance portion (35%) (2,758.00)


Reinsurance commission (6.5% of Rs. 2,758) 179.27
Claim paid (1,680.00)
Other administrative expenses (1,455.00)
Unexpired risk reserve (opening) 2,450.00
Unexpired risk reserve (closing) (2,760.00)
Outstanding claim liability (opening) 1,440.00
Outstanding claim liability (closing) (1,230.00)
Revenue Profit for the period 2,066.27

Alternate Solution
Statement of calculation of Revenue Profit
For the period ended 31.12.2074
in Lakhs
Gross premium (less unrealized Rs. 120) 7,880.00
Less: reinsurance portion (35%) (2,758.00)
Reinsurance commission (6.5% of Rs. 2,758) 179.27
Claim paid (1,680.00)
Other administrative expenses (1,455.00)
Unexpired risk reserve (opening) 2,450.00
Unexpired risk reserve (closing) (2,760.00)
Outstanding claim liability (opening) 115% 1,656.00
Outstanding claim liability (closing) 115% (1,414.50)
Revenue Profit for the period 2,097.77

5.
a) The following information are related with Purple Nepal Ltd.
i) Goods of Rs. 60,000 were sold on 20-3-2074 but at the request of the buyer these
were delivered on 10-4-2074.
ii) On 15-3-2074 goods of Rs. 1,50,000 were sent on consignment basis of which
20% of the goods unsold are lying with the consignee as on 31-3-2074.
iii) Rs. 1,20,000 worth of goods were sold on approval basis on 1-12-2073. The
period of approval was 3 months after which they were considered sold. Buyer
sent approval for 75% goods up to 31-1-2074 and no approval or disapproval
received for the remaining goods till 31-3-2074.
iv) Apart from the above, the company has made cash sales of Rs. 7,80,000 (gross).
Trade discount of 5% was allowed on the cash sales.
You are required to advise the accountant of Purple Nepal Ltd. with valid reasons, the
amount to be recognized as revenue in above cases in the context of NAS -18 and
also determine the total revenue to be recognized for the year ending 31-3-2074. 5
b) M/s Dalima Ltd. is in a dispute with the competitor company. The dispute is
regarding the alleged infringement of copyrights. The competitor has filed a suit in
the court seeking damages of Rs. 325 lakhs.

The Institute of Chartered Accountants of Nepal 13


Suggested Answer - June 2018

Directors are of the view that the claim can be successfully resisted by the company.
How the matter be dealt in the financial statements of the company in the light of
NAS 37. Explain in brief giving reasons for your answer. 5
th
c) Gorkha Company Ltd. imported raw materials worth USD 9,000 on 24 Jestha, 2074,
when the exchange rate was Rs.104 per USD. The transaction was recorded in the
books at the above mentioned rate. The payment of the transaction was made on 10th
Shrawan, 2074, when the exchange rate was Rs.108 per USD. At the year end 31st
Ashadh, 2074, the rate of exchange was Rs.109 per USD.
The Account Officer of the company passed an entry on 31st Ashadh, 2074 adjusting
the cost of the raw material consumed for the difference between Rs.108 and Rs.104
per USD. Discuss whether this treatment is justified as per the provision of NAS-21. 5
Answer:
a) As per NAS 18 "Revenue" is a transaction involving the sale of goods, performance
should be regarded as being achieved when the following conditions are fulfilled:
(a) The seller of goods has transferred to the buyer the property in the goods for a
price or all significant risks and rewards of ownership have been transferred to the
buyer and seller retains no effective control of the goods transferred to a degree
usually associated with ownership : and
(b) no significant uncertainty exists regarding the amount of the consideration that will
be derived from the sales of the goods.
In case (i):
The sale is complete but delivery has been postponed at buyer's request. Purple Nepal
Ltd. should recognize the entire sale of Rs. 60,000 for the year ended 31 st Ashadh,
2074.
In case (ii):
20% goods lying unsold with consignee should be treated as closing inventory and
sales should be recognized for Rs. 1,20,000 (80% of Rs. 1.50,000). In case of
consignment sale revenue should not be recognized until the goods are sold to a third
party.
In case (iii):
In case of goods sold on approval basis, revenue should not be recognized until the
goods have been formally accepted by the buyer or the buyer has done an act
adopting the transaction or the time period for rejection has elapsed or where no time
has been fixed, a reasonable time has elapsed. Therefore in case (iii) revenue should
be recognized for the total sales amounting Rs.1,20,000 as the time period for
rejecting the goods had expired.
In case (iv):
Trade discounts given should be deducted in determining revenue. Thus Rs. 39,000
should be deducted from the amount of turnover of Rs.7,80,000 for the purpose of
recognition of revenue. Thus, revenue should be Rs. 7,41,000.
Thus total revenue amounting Rs. 10,41,000 (60,000+1,20,000+1,20,000+7,41,000)
will be recognized for the year ended 31st Ashadh, 2074 in the books of Purple Nepal
Ltd.
b) As per NAS 37 "Provisions, Contingent liabilities and Contingent assets" a provision

The Institute of Chartered Accountants of Nepal 14


CAP II Paper 1: Advanced Accounting

should be recognized when


a) an entity has a present obligation as a result of a past event.
b) it is probable that the outflow of resources embodying economic benefits will be
required to settle the obligation and
c) a reliable estimate can be made of the amount of obligation.
if these conditions are not met, no provision should be recognized.
In the given situation, since the directors of the company are of the opinion that the
claim can be successfully resisted by the company, therefore there will be no outflow
of resources. Hence no provision is required. The company can disclose the same as
contingent liability by way of following note.
Litigation is in the process against the company relating to dispute with the
competitor who alleged that the company has infringed copyrights and seeking
damages of Rs. 325 lakhs. However the director are of the opinion that the claim can
be successfully resisted by the company.

c) As per NAS-21, ―The effects of changes in foreign exchange rates‖,


(i) Initial recognition of a foreign currency transaction is done in the functional
currency, by applying the spot exchange rate between the functional currency and
foreign currency at the date of the transaction.
(ii) At the end of each reporting period, foreign currency monetary items shall be
translated using the closing rate.
(iii)Exchange difference arising on settlement of monetary items or on translating
monetary items at rates different from those at which they were translated on initial
recognition during the period or in previous financial statement shall be recognized
in profit and loss in the period in which they arise.
In the given case, at the date of transaction the raw material purchased and its
creditors will be recorded at USD 9,000×Rs.104= Rs. 936, 000.
At Balance Sheet date such transaction is reported at closing rate of exchange,
hence it will be valued at the closing rate i.e. Rs.109 per USD (USD
9,000×Rs.109=Rs.981, 000)
The difference of exchange rate between the closing date and transaction date is
Rs.5 per USD (i.e. Rs. 109-Rs.104). The difference of Rs.45,000 (USD 9,000×5)
will be shown as an exchange loss in the profit and loss account for the year ended
31st Ashadh, 2074 and will not be adjusted against the cost of raw materials.
At the settlement date, the company would recognize or provide in the profit and
loss account an exchange gain of Rs. 9,000 (i.e. at the rate Rs. 1 per USD, the
difference of exchange rate between the balance sheet date and the date of
settlement, i.e. Rs.109 and Rs.108 per USD).
Hence the accounting treatment adopted by the Account officer is not as per NAS
21.
6. Write short notes on: (5×3=15)
a) Life Insurance Fund
b) PEARLS System of Monitoring the Cooperative.
c) ‗Other Comprehensive Incomes‘ as per NFRS.

The Institute of Chartered Accountants of Nepal 15


Suggested Answer - June 2018

d) When can a company change its accounting policy?


e) Calculation of Profit from the Notional profit.
Answer:
a) Life Insurance Fund
As per section 22 of the Insurance Act, 2049, every Insurer shall maintain a reserve fund
as specified by the Insurance Board for the liability relating to its Insurance Business
within the state of Nepal. Further, section 21 of the Act states that an Insurer shall
maintain separate fund for each category of Insurance Business and amount to be
received from each Insurance Business shall be deposited in the concerned fund. The
fund maintained for one category of Insurance Business shall not be utilized to bear the
liabilities relating to other category of Insurance Business.
The following amount shall be transferred to the Life Insurance Fund:
 An amount not less than the total liability as specified by the Actuary on the basis of the
Insurance Policies published by the Insurer of the Life Insurance Business within the
state of Nepal.
 In case of fiscal year where no Actuary Valuation is done, the whole surplus amount of
Revenue Account; and
 Amount specified by Insurance Board for Solvency Margin from time to time.
The amount kept under Life Insurance Fund shall be distributed as per Actuary
Valuation Report following the directives of the Insurance Board on this regard.
b) PEARLS System of Monitoring the Cooperative
PEARLS is defined as a set of financial ratios to monitor the financial stability of the
credit unions within World Council of Credit Union's developing movement projects.
These ratios provide credit unions, project staff, national federations and regulators
with essential tools for monitoring, planning, standardizing, ranking and facilitating
supervisory control in credit unions. Each letter in the word PEARLS measures the
key areas of credit union operations: Protection, Effective financial structure, Asset
quality, Rate of return and Costs and Liquidity and Sign of growth. PEARLS system
is designed as a management tool that goes beyond the simple identification of
problems. It helps managers find meaningful solutions to serious institutional
deficiencies. Use of the system permits managers quickly and accurately pinpoint
troubled areas, and to make the necessary adjustments before problems become
serious. In essence, PEARLS is an "early warning system" that generates invaluable
management information.
c) ‗Other Comprehensive Incomes‘ as per NFRS
Other comprehensive income comprises items of income and expenses (including
reclassification adjustments) that are not recognized in profit and loss as required or
permitted by other NFRSs.
The components of other comprehensive income include;
1. Changes in revaluation surplus
2. Re-measurements of defined benefit plans
3. Gains and losses arising from translating the financial statements of a foreign operation
4. Gains and losses from investments in equity instruments measured at fair value through
other comprehensive income in accordance NFRS related with financial instruments

The Institute of Chartered Accountants of Nepal 16


CAP II Paper 1: Advanced Accounting

5. The effective portion of gains and losses on hedging instruments in a cash flow hedge
For particular liabilities designed as at fair value through profit or loss, the amount of the
change in the fair value that is attributable to changes in the liability‘s credit risk.
d) A change in accounting policy should be made in the following conditions:
(i) If the change is required by some statute or for compliance with an Accounting
Standard.
(ii) Change would result in more appropriate presentation of the financial statement.
Change in accounting policy may have a material effect on the items of financial
statements. For example, if cost formula used for inventory valuation is changed from
weighted average to FIFO, or if interest is capitalized which was earlier not in practice,
or if proportionate amount of interest is changed to inventory which was earlier not the
practice, all these may increase or decrease the net profit. Unless the effect of such
change in accounting policy is quantified, the financial statements may not help the
users of accounts. Therefore, it is necessary to quantify and disclose the effect of
change on financial statement items like assets, liabilities, profit/loss.

e) Calculation of Profit from the Notional profit


If the contract account shows a loss, such loss must be fully provided. However, when
the contract account discloses a profit, all such profit cannot be deemed as profit since,
in a subsequent accounting period there may be escalation of costs and such other
contingencies. Therefore, only a proportion of the notional profit is deemed to be
profit and balance is credited to work in progress account. There are certain rules of
thumb regarding the portion of profit to be provided for contingencies. They are given
below:-
Completion stage Profits to be provided for contingencies
Below 25% Full
25% to less than 50% Two –Thirds
50% to less than 95% One- Third
95% and above Nil
The profits to be taken, as earned for the accounting period, must further be reduced
on what is called ‗cash basis‘. Cash basis is the proportion of cash received to work
certified. For example, if 40% of contract is completed, Rs. 60,000 is the notional
profit and 80% of work certified is received in cash from the contractee, profit earned
would be calculated as shown below:
Profit earned = Notional profit X 1/3 X80%
= Rs. 60,000 X 1/3 X80%
= Rs. 16,000
In the above example, out of the notional profit of Rs. 60,000, a sum of Rs. 16,000 will
be deemed to be the profit for the accounting period and the balance of Rs. 44,000 will
be carried forward towards contingencies.
Alternate Solution
If the contract account shows a loss, such loss must be fully provided. However, when

The Institute of Chartered Accountants of Nepal 17


Suggested Answer - June 2018

the contract account discloses a profit, all such profit cannot be deemed as profit since,
in a subsequent accounting period there may be escalation of costs and such other
contingencies. Therefore, only a proportion of the notional profit is deemed to be
profit.
As per the provision of NAS 11, profit to be recognized during a particular year will
be calculated as follows:

Total Contract Revenue xxx


Total Estimated Cost xxx
Total Estimated or Notional Profit xxx

Profit to be recognized during a particular year= Total Estimated Profit* Stage of


completion- Profit recognized up to previous year

Where, Stage of completion can be calculated by any of the following method

i. the proportion of contract costs incurred for work performed to date bear to the
estimated total contract costs;
ii. surveys of work performed; or
iii. completion of a physical proportion of the contract work

The Institute of Chartered Accountants of Nepal 18


CAP II Paper 1: Advanced Accounting

Specific Comments on the performance of the students


Batch: - June 2018
Level: - CAP-II
Subject: Advanced Accounting

Question No. 1
Calculation of Partners' capital and depreciation on Building is not done well.
Calculation of profit is also not done by majority of students. Generally, students had
problem in determining purchase consideration and showing profit made during
period. Most of the students randomly attempted the question; however, none could
solve it well. Some brought different way than model answer. Almost all students
confused in calculating 6 months profit. Few were confused on written off and
depreciation of Building, computation of purchase consideration. Students were not
able to calculate profit and drawing.
Question No. 2
(a) Calculation of Debtors and Creditors is wrong in some cases. Most of the students
confused on one month credit policy.
(b) Majority of students are lack of knowledge while preparing CFS. Cash flow
Statement was correctly answered by few students only. Most of the students were
confused on drawing depreciation.
Question No. 3
(a) Loading on stocks not accurate in some cases and also calculation of shortage of
stock is not done by majority of students. Most of the students confused on
calculation profit on debtor amount and shortage/loss of stock.
(b) Knowledge of provisioning is lacking. Few are confused to apply rate on insured
and SOL.
Question No. 4
(a) Majority of students fails to understand the question. Almost nil students
answered Investment account. Students failed to compute the amortizing schedule
Calculation of reinsurance portion is not done right by majority of students.
Question No. 5
(a) Goods on approval basis are not calculated by majority students. Few confused on
delivery concept, consignment, and sales by approval.
(b) Difference between provision and contingent liability is not clear.
(c) Calculation of Ex. Gain/Loss is not done due to lack of concept.
Question No. 6
(a) Majority of students fails to define life insurance fund.
(b) Almost none of the students fails to attempt the question.
(c) Majority of students did not have knowledge about OCI.
(d) Lack of concept.

The Institute of Chartered Accountants of Nepal 19


Suggested Answer - June 2018

Paper 2:

Audit & Assurance

The Institute of Chartered Accountants of Nepal 20


CAP II Paper 2: Audit & Assurance

Maximum Marks - 100

Total No. of Questions- 7 Total No. of Printed Pages- 10

Time Allowed - 3 Hours


Marks
Attempt all questions.
As an auditor, give your opinion with explanations on the following cases: (45=20)
a) AJ Limited has an investment worth NPR 1,000,000 in its financial statements at 31st
Ashadh 2074. Due to the continuing recession, the investment reduced in value to NPR
900,000 by 15th Shrawan 2074.
b) MNS Ltd. (The Company) is engaged in manufacturing business. The book value of
plant & machinery of the company was Rs. 900 million as on Ashadh end 2073
(purchased at Rs. 1,000 million on 1st Shrawan 2072). It provided depreciation on
straight line basis at 10% per annum based on useful life of the plant & machinery.
Imported asset of Rs. 100 million, the component of above plant & machinery was
acquired on 1st Shrawan 2073 that would be obsolete in 2 years. The company wants to
write off this asset over 2 years. Can the company do so?
c) X Ltd. entered into an agreement with Y Ltd. to dispatch goods valuing Rs. one lakh
every month for six months upon receipt of entire payments. Y Ltd. accordingly made
the payment. In third month due to a natural calamity Y Ltd. requested X Ltd. not to
dispatch until further notice.

d) During the financial year 2073/74, Y & R Private Limited, a service providing company
purchased generator of Rs. 2 million for smooth functioning of its office. The accountant
claims that there is no necessity to provide for depreciation in respect of generator as it
was kept standby but not used at all during the financial year.

Answer:
a) NAS 10 Events after the Reporting Period provides guidance whether an entity should
adjust its financial statements or shall disclose for the events after reporting period.
Since reduction in investment value occurred only after the reporting period, it is
indicative of condition that arose after the reporting period which is a non-adjusting event
as per para 3 of NAS 10. An entity shall not adjust the amounts recognised in financial
statement to reflect non-adjusting events after the reporting period. The decline in fair
value does not normally relate to the condition of the investments at the end of the
reporting period, but reflects circumstances that have arisen subsequently.
Therefore, the entity does not update the amounts disclosed for the investments as at the
end of the reporting period, however it may need to give additional disclosure about the

The Institute of Chartered Accountants of Nepal 21


Suggested Answer - June 2018

nature of event and an estimate of its financial effect , or a statement that such an estimate
cannot be made.

b) As per Nepal Accounting Standard 16 (Property, Plant and Equipment), each part of an
item of property, plant and equipment with a cost that is significant in relation to the total
cost of the item shall be depreciated separately.
An entity shall allocate the amount initially recognised in respect of an item of property,
plant and equipment to its significant parts and depreciate separately each such part. To
the extent that an entity depreciates separately some parts of an item of PPE it also can
depreciate separately the remainder of the item. As it appears that imported assets of Rs.
100 million, which is component of plant and machinery, is having independent useful
life.
Therefore, the company can choose to depreciate the significant parts at 10% p.a. and
remainder imported assets over two years.

c) NAS 18 ―Revenue‖ specifies that revenue from sale of goods should be recognized when
following conditions have been fulfilled:
i) The seller of the goods has transferred all significant risks and rewards of ownership
to the buyer.
ii) the seller retain no effective control of the goods sold usually associated with
ownership;
iii) The amount of revenue can be measured reliably.
iv) It is probable that the economic benefits associated with the transaction will flow to
the entity and
v) The cost in respect of the transaction can be measured reliably.

In this case X Ltd had transferred the significant risk and rewards of the property at an
agreed price. As such sale has been fully completed because upon receipt of the entire
payment. X Ltd was required to dispatch goods valuing Rs 100,000 for six month out of
its inventory. However, in the third month, Y Ltd requested to stop dispatch until further
intimation due to a natural calamity. X Ltd had transferred the goods at an agreed price
and all significant risks and rewards. The delivery was to be effected as per the schedules
indicated by Y Ltd. As per NAS 18, Revenue, mere postponement of delivery at buyers
request does not alter the period in which revenue should be recognized. Accordingly X
Ltd should recognize the entire 600,000 as Sales.

d) As per para 55 of NAS 16 "Property, Plant and Equipment", depreciation of an assets


begin when it is available for use, i.e. when it is in the location and condition necessary
for it to be capable of operating in the manner intended by the management.
Depreciation is a measure of the wearing out, consumption or other loss of value of a
depreciable asset arising from use, efflux of time or obsolescence through technology and

The Institute of Chartered Accountants of Nepal 22


CAP II Paper 2: Audit & Assurance

market changes. Thus, depreciation has to be charged even in case of these assets which
are not used at all during the year but by mere efflux of time provided such assets qualify
as depreciable assets.
When the generator was kept ready for use as stand-by, it means it was intended to be
used for the purpose of business. Depreciation in respect of this generator would have
been provided in the accounts for the year ended 31st Ashadh 2074. If there is an
intention to use an asset, though it may not have actually been used, it is a 'constructive'
or 'passive' use and eligible for charging depreciation.

2. Give your comments on the following cases: (45=20)


a) Mr. KC, Partner of CA firm involved in Audit of X Limited was offered with luxury
car by X Limited for his personal use till financial statements is approved in AGM.
b) Your firm has been appointed as the statutory auditor of Super Express Bank Ltd. for
the financial year 2074/75. You, as the engagement partner, are in the process of
drafting audit plan of the said audit. When obtaining an understanding and
performing a preliminary assessment of the internal audit function for drafting your
audit plan, what are the important criteria to be considered.
c) While auditing accounts of a public limited company for the year ended 31st Ashadh
2074, an auditor found out an error in the valuation of inventory, which affects the
financial statement materially.
d) Auditor of Maya Limited was unable to confirm the existence and valuation of
imported inventory lying with the transporter and accepted a certificate from the
management without obtaining audit evidence. The inventory lying with the
transporter is material to the financial statements.
Answer:
a) A professional accountant in public practice or an immediate or close family member may
be offered gifts and hospitality from a client that may create threats to compliance with
the fundamental principles of code of ethics.
When a professional accountant in public practice or an immediate or close family
member is offered gift and hospitality, the situation shall be evaluated. In the instant case,
an offer of using luxury car for personal use may influence the opinion of professional
accountant in public practicee. Self-interest threats to objectivity or confidentiality are
created when gifts and hospitality is made to unduly influence actions or decisions,
encourage illegal or dishonest behavior, or obtain confidential information. Intimidation
threats to objectivity or confidentiality are created if such gifts & hospitality is accepted
by the professional accountant in public practice or an immediate or close family
member.
A professional accountant in public practice shall evaluate the significance of such
threats and apply safeguards when necessary to eliminate the threats or reduce them to an

The Institute of Chartered Accountants of Nepal 23


Suggested Answer - June 2018

acceptable level. Accordingly, when the threats cannot be eliminated or reduced to an


acceptable level through the application of safeguards, such offer shall not be accepted. .

b) As per NSA 610; "Using the work of internal auditors" the statutory auditor should
consider the activities of internal auditors and their effect, if any, on statutory audit
procedures. In the light of aforesaid provision of NSA, the following aspects should be
considered for drafting the audit plan of Super Express Bank Ltd. for the financial year
2074/75:
1. Organizational Status: specific status of internal auditing in the entity and the effect
this has on its ability to be objective. In the ideal situation, internal auditing will
report to the highest level of management and be free of any other operating
responsibility. Any constraints or restrictions placed on internal auditing by
management would need to be carefully considered. In particular, the internal auditors
should be free to communicate fully with the external auditor.

2. Scope of Function: the nature and extent of internal auditing assignments performed.
The external auditor would also need to consider whether management acts on
internal audit recommendations and how this is evidenced.

3. Technical Competence: whether internal auditing is performed by persons having


adequate technical training and proficiency as internal auditors. The external auditor
may, for example, review the policies for hiring and training the internal auditing staff
and their experience and professional qualifications.

4. Due Professional Care: whether internal auditing is properly planned, supervised,


reviewed and documented. The existence of adequate audit manuals, work programs
and working papers would be considered.

c) NSA 450 ―Evaluation of Misstatements identified during the audit‖ deals with the
auditor‘s responsibility to evaluate the effect of identified misstatements on the audit and
of uncorrected misstatements, if any, on the financial statements. The auditor should
consider requesting the management to adjust the financial information or consider
extending his audit procedures. If the management refuses to adjust the financial
information and the results of extended audit procedures do not enable the auditor to
conclude that the aggregate of uncorrected misstatements is not material, the auditor
should express a qualified or adverse opinion, as appropriate.
In the instant case, the auditor has detected the material errors affecting the financial
statements; the auditor should communicate his findings to the management on a timely
basis, consider the implications on true and fair view and think about modifying the
report.

d) As per NSA 580 ―Written Representations‖ auditor may rely on the representation by
the management but he should seek corroborative audit evidence. The management

The Institute of Chartered Accountants of Nepal 24


CAP II Paper 2: Audit & Assurance

representation cannot substitute other evidence that the auditor could reasonably expect
to be available to the auditors.
Also, NSA 501 ―Audit Evidence‖ Specific Consideration for Selected items‖ requires
obtaining sufficient and appropriate evidence regarding the existence and condition of
inventory lying with third party if material to the financial statement.

Supporting evidences can be obtained from inside or outside sources. The audit evidence
for verification of inventory lying with the transporter - say purchase order, invoice,
custom clearance certificate, inspection, confirmation from transporter etc. are available
evidences which auditor should verify.

Just because the management had confirmed the existence and valuation of imported
inventory lying with the transporter the auditor cannot shrink his responsibility. This is
negligence on his part.

3. Answer the following: (35=15)


a) What are the provisions on ‗Timing of Liaison and Coordination‘ amongst internal
audit and external audit in NSA 610?
b) Mr. Shyam was appointed as the auditor of M/s Himalayan Ltd. and intends to apply
the concept of materiality for the financial statements as a whole. Please guide him as
to the factors that may affect the identification of an appropriate benchmark for this
purpose.
c) Explain the concept of True and Fair View.

Answer:
a) When planning to use the work of internal auditing, the external auditor will need to
consider internal auditing‘s tentative plan for the period and discuss it at as early a stage
as possible.
Where the work of internal auditing is to be a factor in determining the nature, timing
and extent of the external auditor‘s procedures, it is desirable to agree in advance the
timing of such work, the extent of audit coverage, test levels and proposed methods of
sample selection, documentation of the work performed and review and reporting
procedures.
Liaison with internal auditing is more effective when meetings are held at appropriate
intervals during the period. The external auditor would need to be advised of and have
access to relevant internal auditing reports and be kept informed of any significant
matter that comes to the internal auditor‘s attention which may affect the work of the
external auditor. Similarly, the external auditor would ordinarily inform the internal
auditor of any significant matters which may affect internal auditing.

The Institute of Chartered Accountants of Nepal 25


Suggested Answer - June 2018

b) Use of benchmark in determining Materiality: NSA 320 Materiality in planning and


performing an audit prescribes the use of benchmarks in determining materiality for the
Financial Statements as a Whole. Accordingly determining materiality involves the
exercise of professional judgment. A percentage is often applied to a chosen benchmark
as a starting point in determining materiality for the financial statements as a whole.
Factors that may affect the identification of an appropriate benchmark include the
following:
i. The elements of the financial statements (for example, assets, liabilities, equity,
revenue, expenses);
ii. Whether there are items on which the attention of the users of the particular entity‘s
financial statements tends to be focused (for example, for the purpose of evaluating
financial performance users may tend to focus on profit, revenue or net assets);
iii. The nature of the entity, where the entity is at in its life cycle, and the industry and
economic environment in which the entity operates;
iv. The entity‘s ownership structure and the way it is financed (for example, if an entity is
financed solely by debt rather than equity, users may put more emphasis on assets,
and claims on them, than on the entity‘s earnings); and
v. The relative volatility of the benchmark.

c) 'True and fair view' is a phrase usually auditors use to express audit opinion on the
financial statements of an entity. It implies that the financial statements are presented
fairly in all materials respect; the position, performance, cash flows and changes in
equity of the entity. The auditor expresses such opinion upon assessment of the internal
control system of the entity and test checking the financial transactions carried out
during the fiscal year. The auditor's act is guided by the provisions set forth in the Nepal
Standards on Auditing together with the Code of Ethics applicable to the professional
accountants.

As per NSA 200 ―Overall Objectives of the Independent Auditor and the Conduct of an
Audit in accordance with NSA‖, the auditor‘s expression of true and fair view is
supposed to be received as only the ―reasonable assurance and not the absolute
assurance‖ of the state of the financial statements. This implies that the users are not
supposed to absolutely rely on auditor‘s judgment for making their financial decisions
relating to the entity. This is because the auditor is not expected to, and cannot, reduce
audit risk to zero and cannot therefore obtain absolute assurance that the financial
statements are free from material misstatement due to fraud or error. This is because there
are inherent limitations of an audit, which result in most of the audit evidence on which
the auditor draws conclusions and bases the auditor‘s opinion being persuasive rather than
conclusive.
Broadly speaking, the financial statements are considered as presenting to true and fair
view if:
 The information contained in them are not materially misstated;

The Institute of Chartered Accountants of Nepal 26


CAP II Paper 2: Audit & Assurance

 There is an appropriate application of Nepal Accounting Standards, with additional


disclosure in the case of companies registered under Companies Act. In the case of
other entities there is an appropriate application of generally accepted accounting
principles as is applicable; and
 They comply with the provisions of applicable laws and regulations of the company.

4. Answer/Comment on the following: (35=15)


a) What is ‗Independence of Mind‘ and ‗Independence in Appearance‘?
b) You are the statutory auditor of PQR Ltd., while carrying out the audit you found
existence of certain threats to objectivity at significant level. What is your duty in
such situation? Give the examples of safeguards you will apply in such situation?
c) Explain the provision relating to 'Conflicts of Interest' with reference to Part B of
Code of Ethics.

Answer:
a) In case of audit engagements, it is in the public interest and, therefore, required by section
290 0f Code of ethics, that members of audit teams, firms and network firms shall be
independent of audit clients.
Independence of Mind
Independence is a state of mind and personal character and an enlightened view of the
professional duties involved. The state of mind that permits the expression of a
conclusion without being affected by influences that compromise professional judgment,
thereby allowing an individual to act with integrity and exercise objectivity and
professional skepticism.
Independence in Appearance
Independence of auditor must not only exist in fact, but should also appear to exist to all
reasonable persons. The avoidance of facts and circumstances that are so significant that a
reasonable and informed third party would be likely to conclude, weighing all the specific
facts and circumstances, that a firm‘s, or a member of the audit team‘s, integrity,
objectivity or professional skepticism has been compromised.

b) As per Section 280 (4) of ICAN Code of Ethics, I should evaluate the significance of such
threats and should apply safeguards when necessary to eliminate them or reduce them to
an acceptable level. The safeguards that should be applied are:
i. Withdrawing from the engagement team;
ii. Supervisory procedures;
iii. Terminating the financial or business relationship giving rise to the threat;
iv. Discussing the issue with higher levels of management within the firm;
v. Discussing the issue with those charged with governance of the client.

The Institute of Chartered Accountants of Nepal 27


Suggested Answer - June 2018

If safeguards cannot eliminate or reduce the threat to an acceptable level, I should decline
or terminate the audit engagement of PQR Ltd.

Section 220 of Code of Ethics contains the provision relating to Conflict of Interest. A
professional accountant in public practice may be faced with a conflict of interest when
performing a professional service. A conflict of interest creates a threat to objectivity and
may create threats to other fundamental principles.

Such threats may be created when:


 The professional accountant provides a professional service related to a particular
matter for two or more clients whose interests with respect to that matter are in
conflict or
 The interests of the professional accountant with respect to a particular matter and the
interests of the client for whom the professional accountant provides a professional
service related to that matter are in conflict.

A professional accountant shall not allow a conflict of interest to compromise


professional or business judgement. The professional accountant in public practice shall
apply safeguards, when necessary, to eliminate the threats to compliance or reduce them
to acceptable level.

5. Answer the following: (25=10)


a) What are the matters to be audited in view of propriety as per Audit Act,
2048 as amended in 2073?
b) Describe the functions and duties of auditors as per Companies Act, 2063.

Answer:
a) Section 5 of the Audit Act 2048 (As amended in 2073) states that ; the Auditor General
shall audit following matters considering the propriety thereof :
(i) On the propriety of any expenditure and its authorization, if in the opinion of the
Auditor General such expenditure is a reckless one or is an abuse of national property,
whether fixed or current, despite that the expenditure conforms to the authorization,
and
(ii) On the propriety of all authorizations issued in respect of any grant of national
property whether fixed or current, or underwriting of any revenue, or any contract,
license or permits relating to mining, forest, water resources, etc. and any other act of
abandoning fixed or current assets of the nation.
Though the Auditor General may not include in his report minor items of discrepancy and
other items deemed by him as insignificant in view of their propriety which were
observed during the audit of income and expenditures.

The Institute of Chartered Accountants of Nepal 28


CAP II Paper 2: Audit & Assurance

b) The function and duties of auditors as per section 115 of the Companies Act,
2063 are as follows:
(1) The auditor shall, addressing the shareholders or the appointing authority, submit to the
company his/her report, certifying the balance sheet, profit and loss account and cash
flow statement based on the books of account, records and accounts audited by
him/her.
(2) The audit report shall be prepared in accordance with the prevailing law or in
consonance with the audit standards prescribed by the competent body; and such
report shall state the matters to be set out under this Act, as per necessity.
(3) The audit report as referred to in Sub-section (2) shall also indicate the following
matters, inter alia:
(i) Whether such information and explanations have been made available as were
required for the completion of audit;
(ii) Whether the books of account as required by this Act have been properly
maintained by the company in a manner to reflect the real affairs of its business;
(iii) Whether the balance sheet, profit and loss account and cash flow statements
received have been prepared in compliance with the accounting standards
prescribed under the prevailing law and whether such statements are in agreement
with the books of account maintained by the company;
(iv) Whether, in the opinion of the auditor based on the explanations and information
made available in the course of auditing, the present balance sheet properly
reflects the financial situation of the company, and the profit and loss account and
cash flow statement for the year ended on the same date properly reflect the profit
and loss, cash flow of the company, respectively;
(v) Whether the board of directors or any representative or any employee has acted
contrary to law or misappropriated any property of the company or caused any
loss or damage to the company or not;
(vi) Whether any accounting fraud has been committed in the company
(vii) Suggestion, if any

6. Write short notes on the following: (42.5=10)


a) Audit Risk
b) Audit Committee
c) Cut Off Procedure
d) Corporate Governance
Answer:
a) Audit Risk
The risk that the auditor expresses an inappropriate audit opinion when the financial
statements are materially misstated. Audit risk is a function of the risks of material
misstatement and detection risk. Risk of material misstatement is a product of inherent

The Institute of Chartered Accountants of Nepal 29


Suggested Answer - June 2018

risk and control risk whereas detection risk is a situation where auditors audit procedures
fail to detect material misstatement. To reduce audit risk at acceptable level, the auditor
should appropriately respond to the assessed risk.

b) Audit Committee
As per Section 164 of Companies Act as amended, a listed company with paid up capital
of thirty million rupees or more or a company which is fully or partly owned by the
Government of Nepal shall form an audit committee under the Chairpersonship of a
director who is not involved in the day-to–day operations of the company and consisting
of at least three members. At least one member of the audit committee shall be an
experienced person having obtained professional certificate in accounting or a person
having gained experience in accounting and financial field after having obtained at least
bachelor’s degree in accounts, commerce, management, finance or economics. The
committee is responsible to review internal control systems of the company.

c) Cut Off Procedure


Cut-off procedures mean procedures employed to ensure the separation of transactions at
the end of one year from those in the commencement of the next year. For the cut off
procedure of inventory, the auditor should satisfy himself by examination and test check
that these procedures adequately ensure that:
 Goods purchased for which property has passed to the client have in fact been
included in inventories and that the liability if any, has been provided for.
 Goods sold have been excluded from the inventories and credit has been taken for
sales.
The auditor may examine a sample of documents evidencing the movement of stocks into
and out of stores, including documents pertaining to period shortly before and shortly
after the cutoff date, and check whether the stocks represented by those documents were
included or excluded, as appropriate, during the stock-taking.
d) Corporate Governance
Corporate Governance is a system by which the business organizations are directed and
controlled. It is a set of processes, customs, policies, laws, and institutions affecting the
way an organization is directed, administered or controlled. The corporate governance
structure specifies the allocation of rights and responsibilities among board, managers,
shareholders, employees, suppliers, customers, government etc. and spells out the rules
and procedures for making decisions on corporate affairs.

Corporate governance is thus 'an internal system encompassing policies, processes and
people, which serves the needs of shareholders and other stakeholders, by directing and
controlling management activities with corporate fairness, transparency, independence,
integrity and accountability. Further, it influences how the objectives of the company are
set and achieved, how risk is monitored and assessed, and how performance is optimized.
It encourages companies to create value (through entrepreneurism, innovation,
development and exploration) and provide accountability and control systems
commensurate with the risks involved

The Institute of Chartered Accountants of Nepal 30


CAP II Paper 2: Audit & Assurance

7. Distinguish between: (25=10)


a) Cost Accounting and Financial Accounting
b) Reserves and Provisions
Answer:
a) Distinction between Cost Accounting and Financial Accounting
Cost Accounting is a close follower of financial accounting. It is not independent of
financial accounting. Though there are common grounds between the two, the important
differences are given below:
(i) Reporting: The major objective of financial accounting is external reporting whereas the
focus of cost accounting has been essentially internal i.e. management.
(ii) Flexibility: Financial accounting is mostly historical or after the event while cost
accounting is much more flexible and open minded and includes in both retrospective
and anticipatory calculations.
(iii)Nature: Financial accounting classifies records, presents and interprets in terms of
money transactions whereas cost accounting classifies, records, presents and interprets
in a significant manner the material, labour and overhead costs involved in
manufacturing and selling each product.
(iv) Financial accounting uses Relevant Accounting Standards while recording, classifying
summarizing and reputing business transactions whereas cost accounting is not bound
to use such Accounting Standards and it can use any technique or practice which
generates useful information.
(v) Time Span: Financial accounting data are developed for a definite period, usually a
year, half year or a quarter, but cost accounting reports and statements can be prepared
whenever needed.
(vi) Accounting Method: Financial Accounting follows the double-entry system for
recording, classifying and summarizing business transactions.
The data under Cost Accounting can be gathered for small or large segments or activities
of an organization and monetary as well as other measures can be used for different
activities in the organization.

b) Reserves and Provisions


(i) Reserve is an appropriation of profit whereas provision is a charge against Profit.
(ii) Reserves are not intended to meet specific liability, contingency or diminution in the
value of assets. Provisions are made to provide for depreciation, renewal or a known
liability or a disputed claim.
(iii) Reserves cannot be created unless there is a profit except revaluation reserve and
capital subsidy. Provisions must be created whether or not there is profit.
(iv) Reserves are generally optional except in certain situations – Capital Redemption
reserve, Debenture Redemption Reserve, etc. Provisions are not optional and have to

The Institute of Chartered Accountants of Nepal 31


Suggested Answer - June 2018

be made as per generally accepted accounting principles.


(v) Reserves are shown on the liability side. Provisions for depreciation and provision for
doubtful debts are shown as deduction from respective assets. Provision for liability is
shown on the liability side.

The Institute of Chartered Accountants of Nepal 32


CAP II Paper 2: Audit & Assurance

Specific Comments on the performance of the students


Batch: - June 2018
Level: - CAP-II
Subject: Audit and Assurance
Question No. 1
Most of the students did not understand the question about non-adjusting event. They are
unaware about NAS PPE(16). Students are aware about the related accounting standard and
provision. However, while answering the question they are not correctly given the
justification. It seems the student has surface knowledge only. Most of the students were not
able to relate Q.no. 1 (c) with revenue recognition. Some students cannot answer in line with
NAS, other irrelevant NASs are referred. 50% answer are irrelevant, could not linked with
NAS - 18
Question No. 2
(c) part is confusing, so mostly internal audit function has been ignored only focused on audit
plan. 2(b) most of the student wrote about audit planning instead of preliminary assessment
of internal audit function. The question is not understood by student properly. 'Self- int.
threat' could not be referred by majorities. Majorities 90%, fail to link the audit plan with
"internal audit function". No student can refer NSA 450.
Question No. 3
Satisfactorily. Instead of writing about timing of liaison and coordination most student write
on using the work of internal Auditor.
Question No. 4
Students found difficulty due to generalization of question.
Question No. 5
Not referred as per audit act, attempted definition of propriety audit. It seems student does
not understand the concept of proprietary audit properly.
Question No. 6
Most of all answered satisfactorily. There is confusion regarding Audit committee for some
students. They wrote the detail of Audit committee of ICAN, instead Audit committee as per
company Act.
Question No. 7
Differences are not specific. Attempted by most of the students and answering properly.
However, some student repeated same point again and again in different language.

The Institute of Chartered Accountants of Nepal 33


Suggested Answer - June 2018

Paper 3:

Corporate & Other Law

The Institute of Chartered Accountants of Nepal 34


CAP II Paper 3: Corporate & Other Law

Roll No……………. Maximum Marks - 100

Total No. of Questions - 7 Total No. of Printed Pages -12


Time Allowed - 3 Hours
Marks
Attempt all questions.
1. Answer the following questions: (5×5=25)
a) During the course of Audit of Nepal Commercial Bank Limited, auditor of the bank
found some irregularities during the course of audit. He would like the shareholders
to know about these irregularities. Therefore, he wants to have the bank‘s extra
ordinary general meeting (EGM) convened and he approaches you for consultation
for convening EGM. Could you explain how the EGM of the bank can be convened
pursuant to the Companies Act, 2063?
b) Mr. Ramesh Mahato, a renowned businessman is planning to carry out insurance
business in Nepal by establishing a private company. Give your suggestion/opinion to
Mr. Mahato on his plan by citing the relevant provision of the Companies Act, 2063.
c) Mr. 'A' is appointed as an auditor of Prime Development Bank Ltd. in this Fiscal
Year. He is willing to continue in this company for F/Y 2076/2077. Mr. B, who had
been a partner of A before 3 years, is also willing to be an auditor in this company. In
spite of this, company is planning to substitute another auditor. In this circumstances,
is the willingness of A and B is justified? If the company has planned to remove Mr.
A then how it can be removed? Justify your answer with reference to the Companies
Act, 2063.
d) Mentions the circumstances where special resolution shall be presented in the general
meeting of a company for decision.
e) A resolution is passed by the general meeting of Alliance Insurance Ltd. to appoint
Rajan as an independent Director. After three months an action is filed against him as
he was convicted for 1 year jail imprisonment in a case of assault, hence, disqualified.
Advise the company regarding :
i) The disqualifications of an independent director?
ii) Is Rajan is disqualified in this respect?
Answer
a) As per the section 82(2) of the Companies Act 2063, if in the course of auditing the
account of the bank, it is deemed necessary to call an extra ordinary general meeting
for discussion of the irregularities at the EGM, the auditor may request the board of
directors to call such meeting; and if the board of directors fails to call the meeting
accordingly, the auditor may make an application, setting out the matter, to the
Office of Company Registrar and if an application is so made, the office may call the
extra-ordinary general meeting of the company.
b) Besides incorporation of a company as a public company certain companies can be
incorporated as a public company only. According to the section 12 of the
companies Act 2063 a company carrying on the business of banking, financial

The Institute of Chartered Accountants of Nepal 35


Suggested Answer - June 2018

transactions, insurance business related transaction, stock exchange business,


provision fund etc. as prescribed. Thus Mr. Ramesh Mahato, if interested to carryout
insurance business in Nepal, it is mandatory to be registered company as public
company only.

c) According to section 110 of the Companies Act, 2063, every company must appoint
an auditor to audit its accounts.
The companies Act, 2063, in section 111, has provided different rules as to
appointment of auditor. An auditor, from the amongst the auditors registered or
licensed under the existing law, can be appointment, in case of a public company, by
the General Meeting and in the case of a private company as provided in its
Memorandum of Association, Articles of Association and consensus agreement or by
the General Meeting in the absence of such arrangements.
The prime authority to appoint an auditor is general meeting. The company registrar
office also performs an appointing authority with reference of Board of Director due
to fail to appoint an auditor by AGM and fails to be held meeting and ceases to hold
the office for any reason.
The auditor appointed above remains in the office until next AGM is held. No auditor
or his/her partner or ex-partner or employee or ex-employee shall be appointed as
auditor for more than three consecutive terms to perform the audit of a public
company.

Both candidates are eligible in the eye of prevailing law. In regard to the willingness
of Mr. A, he can be continued up to 3 consecutive terms. However, the issue is
subject to the approval and consent of the shareholders meetings. Further, Mr. B is
also eligible to be appointed as an auditor of the said company. He has fulfilled
requirement as Mr. A has owned.
Regarding the removal process of an auditor, the companies Act, 2063, it its section
119, has mentioned, that, an auditor should be removed after completion of the audit
accounts of such F/Y. Auditor can be removed after fulfilling following conditions:
 Breaching the code of conduct of auditors or does any act against the interest of
the company which has appointed him as the auditor, or commits ant act contrary
to the prevailing law.
 Auditor may be removed through same process whereby he/she was appointed as
auditor.
 Prior information should be given to Nepal Chartered Accountants Institute.
 Approval should be taken from regulatory authority as provided by the prevailing
law.
 Approval from the office in the time while failing such authority.
 The auditor should be provided with a reasonable opportunity to defend
him/herself.
d) In a general meeting of the Company, it is required to submit matters to be decided
as resolution which may be either ordinary or special resolution. However Section
83 of the Companies Act prescribes the subject matter which requires to be
presented as special resolutions in the general meeting of a company for decision.

The Institute of Chartered Accountants of Nepal 36


CAP II Paper 3: Corporate & Other Law

i) Increasing the authorized capital of the company,


ii) Decreasing or altering share capital of the company,
iii) Altering the name or main objectives of the company,
iv) Amalgamating one company into another company,
v) Issuing bonus share,
vi) Buying back of own shares by the company,
vii) Selling shares at a discount ,
viii) Converting a private company into a public company or vice versa,
ix) Such other matter in respect of which the company is required by this Act or
the articles of association to adopt a special resolution.

e)
i) Pursuant to Sub-section (2) of Section 89 of the Companies Act, 2063, the following
persons shall not be eligible to be appointed to the office of independent director:
1. A person as referred to in Sub-section (1) of section 89.
2. A shareholder of the concerned company.
3. Who has not obtained at least bachelor degree in a subject that is related to the
business to be carried on by the concerned company and gained at least ten
years of experience in the related field or in the company management affairs or
who has not obtained at least bachelor degree in finance, economics,
management, accounts, statistics, commerce, trade or law and gained at least ten
years of experience in the related field.
4. An officer, auditor or employee of the concerned company or a period of three
years has not lapsed after his/her retirement from any such office.
5. The close relative of the office of the concerned company;
6. An auditor of the concerned company or his/her partner.
ii) As per Sub-section 1 of Section 89 of the Companies Act, 2063, A person who is
convicted of an offense of corruption or of an offense involving moral turpitude and
convicted of an offense of theft, fraud, forgery or embezzlement or misuse of goods
or funds entrust to him/her, in an authorized manner, and sentenced in respect thereof,
a period of three year has not elapsed from the expiry of the sentence he cannot be
appointed as an independent director of a public limited company. In the above case,
Mr. Rajan was convicted in an offense of assault which does not involve moral
turpitude and does not come within the categories as mentioned in the Act. Therefore,
he can be appointed as an independent director of the company.

2. Answer the following questions: (3×5=15)


a) Mr. Ramesh Raj a promoter of the Gorkha Bank Ltd approached CEO of the bank
and requested that he wanted to take loan from the bank. CEO of bank refused to
allow the loan stating that there are various acts that cannot be done by Banks &
Financial Institution. Mr. Raj was confused why he could not obtain loan from his
own bank and approached you and asked you, what are the acts prohibited to be done
by Banks & Financial Institution as per the Banks & Financial Institutions Act, 2073.

The Institute of Chartered Accountants of Nepal 37


Suggested Answer - June 2018

b) The board of directors of the ABC Bank Ltd has decided to distribute bonus share
from the amount accumulated in the General Reserve Fund for the fiscal year
2073/74. As an internal auditor of the bank, give your opinion on the decision of the
board of directors referring the relevant provision of the Bank and Financial
Institution Act 2073.
c) Mr. Dhakal is a CEO of a Nepal Commercial Bank Ltd. established and worked for
Five Years in Nepal. As he heard that NRB provides loan and refinance to the bank
like this. You are there in a position of economic advisor. Give suggestion regarding
the loan and refinance provision as prescribed by the NRB Act, 2058.
Answer
a) Section 50 of the Banks& Financial Institutions Act, 2073 prescribes the provision
of the act prohibited to be done by Banks & Financial Institutions. According to it
Banks & Financial Institutions are not allowed to carry out, or cause to be carried out
the following activities:
1.Purchasing or selling goods for commercial purpose or constructing building or
purchasing any immovable property except when it is required for its own use;
2.Advancing credit against the security of its own share.
3.Supplying credit or facility to any directors, person who have subscribed one percent
or more of its share, chief executive or any family members of such persons,
managing agents or any firms, companies or institutions which are entitled or
nominate or appoint directors; or any firm companies or institutions in which the
institution has a financial interest.
4.Supplying credit or facility in an amount exceeding such percentage of capital fund
as may be prescribed by the Nepal Rastra Bank to a single customer, company and
companies or partnership firm of single group.
5.Supplying any type of credit to any person, firm, company or institution against
gurantee given by the promoters, directors, or chief executive.
6.Making investment in the securities of Bank or financial institutions of Classes A, B
and C classified by the Nepal Rastra Bank.
7.Making investment of an amount exceeding the limit prescribed by the Nepal Rastra
Bank in the share capital of any other institutions.
8.Indulging with other licensed institutions to mutually create any type of monopoly or
any other type of controlled practice in the financial transactions.
9.Doing any kind of act which is capable of creating any artificial obstruction in the
competitive environment of the financial sector, with the intention of deriving undue
advantage.
10. Doing such other acts prohibited from being done by a bank or financial institutions
as may be prescribed by the Nepal Rastra Bank.

b) As per Section 44 of the Banks and Financial Institution Act, 2073;

The Institute of Chartered Accountants of Nepal 38


CAP II Paper 3: Corporate & Other Law

(1) A licensed institution must maintain a general reserve fund at least twenty per cent
of the net profits of each year shall be kept on being credited to such fund until the
amount of such fund doubles the paid-up capital and must be continued adding at
least ten percent each year thereafter
(2) The amount credited to the reserve fund of a licensed institution under Sub-section
(1) may not be invested or transferred to any other head without the prior approval of
the Nepal Rastra Bank.
So, the decision taken by the board of directors of the ABC bank is against the
provision of BAFIA which is not executable.

c) NRB is a banker of the bank. It inspects and supervises the mobilization of financial
resources whether it has collected through public or from loan and refinance or its
promoters. As this matter is concerned, NRB may make available loan and refinance
to commercial bank and financial institution under the provision of section 49 of
NRB Act. The provision is as follows:
(1) The Bank may, subject to the terms and conditions prescribed by it, make available
loan and refinance to commercial banks and financial institutions for a maximum
period of one year against the security of the following assets:-
(a) International negotiable instrument referred to in Clause (e) of Sub-section (1)
of Section 66;
(b) The debt bond issued by Government of Nepal payable within Nepal;
(c) The deposits accumulated in the Bank or the gold and precious metals, which
the Bank may transact under this Act;
(d) The bill of exchange or the promissory notes referred to in Sub-section (1) of
Section 48;
(e) Other securities as prescribed.
(2) Notwithstanding anything contained in Sub-section (1) of the Bank may provide any
type of credit to a commercial bank and financial institution for a maximum period
of one year in cases where Government of Nepal has, for the sake of public interest
and welfare, deemed it appropriate to provide loan and has requested the Bank
therefore and Government of Nepal has given a guarantee of securities of prevailing
market rate for such loan or in extraordinary circumstances where the Bank has to
work as a lender of the last resort.
(3) Loan, as provided in accordance with the provision of sub section (1) or (2), shall be
renewed for the period of One year subject to the terms and conditions prescribed by
it.

3. Answer the following questions: (2×5=10)


a) Under what circumstances, the registration certificate of insurers cannot be renewed.
Explain in the light of relevant provisions of the Insurance Act, 2049.
b) Securities Act, 2063 has made provision for Compensation Fund to be established by
a Stock Exchange and should made rule for the operation of such Compensation
Fund. Enumerate the provisions that should be included in such Rule.
Answer

The Institute of Chartered Accountants of Nepal 39


Suggested Answer - June 2018

a) Section 11A of Insurance Act, 2049 mentions few circumstances that can lead to
non-renewal of certificate of registration of an insurer. These circumstances are as
follows:
i) In case it doesn't submit the balance sheet according to section 23
ii) in case it doesn't submit the statement of income according to section 24
iii) In case it doesn't submit auditor's report according to section 25
iv) In case it doesn't submit the actuary's report according to section 26
v) In case it doesn't pay the insurance service fee according to section 40
vi) In case it has been prohibited from engaging in the insurance business according
to section 12A.

In case the certificate of registration of an insurer cannot be renewed because of any


of the circumstances mentioned in the above, the Board shall notify the insurer
accordingly within 15 days from the date of emergence of such circumstances.

b) Section 53 of the Securities Act, 2063 has made provision for Compensation Fund to
be established by a Stock Exchange to protect investor against possible loss or
damage.
As per the section 54 of the Securities Act, 2063 made provisions shall be made in the
Rules in relation to the operation of the compensation fund as follows:
(a) Provisions relating to the deposit of money to the fund,
(b) Maximum amount to be paid as compensation from the fund,
(c) Provisions relating to the accounts and audit of the fund,
(d) Conditions for making claim to obtain amount from the compensation fund and
procedures from making such a claim,
(e) Conditions where any claim cannot be made on the compensation fund,
(f) Procedures for taking action and making decision on payment of money as
claimed from the compensation fund,
(g) Maximum limit of amount payable as compensation to one person,
(h) Other necessary matters in relation to the examination of compensation claims,
(i) Provisions to be made in the event of the revocation of the license of a stock
exchange,
J. Other necessary provisions in relation to compensation.

4. Answer the following questions: (2×5=10)


a) Krishna Tower Ltd. is going to distribute bonus of F/Y 2074/75. In this year some
of the employee has involved in riots and indisciplinery acts. Company is in
dilemma and confusion how bonus is to be distributed to such employee? Suggest in
following query in accordance with the Bonus Act, 2030.
i) What are the eligibility criteria for distributing bonus?
ii) Is there any restriction to have or to distribute bonus?
b) What are the provisions prescribed by the Labour Act, 2074 regarding working
hours and over time work?

The Institute of Chartered Accountants of Nepal 40


CAP II Paper 3: Corporate & Other Law

Answer
a)
i) Bonus Act, 2030, section 6, has mentioned the eligibility criteria of bonus as
follows:
(1) An employee who has worked for the half period to be worked in a fiscal year,
shall be entitled to obtain bonus under to this Act.
Provided that, no employee shall be entitled to obtain Bonus who has worked
casually or in a shift basis.
(2) For the purpose of Sub-section (1), the following periods shall also be computed
as a period where an employee has worked.
(a) A period kept on reserve under any contract or under Section 11 of the
Labour Act, 2048 (1991).
(b) A period under which an employee is on any leave with salary.
(c) A period of disablement caused by accident arising in course of business of
the enterprise.

ii) There are restriction provisions to have or to distribute bonus in the Bonus Act,
2030. Section 8 has prescribed the restriction provision that an employee shall
not be entitled to obtain bonus under this Act, if he/she is punished or dismissed
from service for committing any act as follows:
(a) Theft of the property of the enterprise or any damage to such property.
(b) Illegal strike or abetment to other for such strike,
(c) Riots or breaching of discipline.
Provided that, this Section shall not be deemed to be prejudiced to obtain in the
case of the bonus for a period before committing such a punishable act.

b) Section 28, 29 and 30 of the Labor Act, 2074 prescribed provision of working hours,
and overtime work of the labor.
As per the provision of employees or workers are allowed to work as follows:
i. Employer cannot engage any labor more than 8 hour a day and 48 hours a week.
ii. Labor should be allowed recess of half an hour after 5 hours of continuous work.
iii. If the nature of work is of continuous and cannot be stop, arrangement of recess
should be made in rotation basis.
iv. Recess period will be calculated in working hour.
v. Labor shall not be forced to work over time. However if non completion of work
affect Life, security or health of any person or employer might incur heavy loss,
labor could be engage for overtime work.
vi. Overtime work as above shall not be more than 4 hours a day and 24 hours a
week.
In case of women workers, they can be engaged only from 6 a.m to 6.pm. If they are
to be engaged beyond 6 p.m i.e at night time, it required to take their consent by
arranging necessary safety measures and transportation facilities. In case of minor
workers it is prohibited to engage them from 6 p.m. to 6 a.m and is allowed to work 6
hours a day and 36 hours a week.

The Institute of Chartered Accountants of Nepal 41


Suggested Answer - June 2018

5. Answer the following questions: (2×5=10)


a) There is complaint lodged into the Council against Mr. Dikpak, the member of the
council, alleging that he used to share or distribute the profit the auditing fees or
remuneration with any other person other than a member of the Institute. How and
why this act is regulated by the existing law.
b) Discuss the provisions of the punishment as prescribed by the Nepal Chartered
Accountants Act, 2053?
Answer
a) As mentioned in the question that Mr. Dipak has alleged for the breach of conduct
prescribed for the member of the Council that he used to share or distribute the profit
the auditing fees or remuneration with any other person other than a member of the
Institute as mentioned in the section 34(3) Nepal Chartered Accountants Act 2053. It
is clear that the matter is concerned with the breach of conduct.
Regarding the regulation of such act, the Council is the final authority to punish the
alleged member. However, before punish to such allegation, the recommendation of
Disciplinary Committee have necessary. As provided in the section 14, the
Disciplinary Committee, has to recommend the Council to take necessary actions
after investigation upon complains lodged against any action, contrary to this Act or
Regulations by any member. The Disciplinary Committee have the authority similar
to a judicial court in respect of summoning concerned person and investigating
evidences and witnesses. This Committee shall recommend to the Council, along with
its opinion and finding, for necessary action against a member, if found guilty. The
Council may, considering such a recommendation, impose any of the following
punishment according to the degree of offense:-
(a) Reprimanding,
(b) Removing from the membership for a period up to five years,
(c) Prohibiting from carrying on the accounting profession for any particular period,
(d) Cancellation of the Certificate of Practice or membership.
Any Council member against whom the Disciplinary Committee, after investigating
upon the complaint of his action contrary to the Act or Regulations, Bye-laws or code
of conduct framed under the Act, has decided to recommend the Council to take
necessary action, shall not be allowed to attend and to vote at the Council meeting
where the Council is hearing at such recommendation. Before imposing a punishment
referred in above, the Council shall provide reasonable opportunity to the concerned
members to submit their clarification. The concerned member may, if he is not
satisfied with the decision referred to in above, file an appeal in the Appellate Court.

b) Section 41 of the Nepal Chartered Accountants Act, 2053 has prescribed punishment
as follows:
(1) A person, who carries out audit without obtaining a Certificate of Practice, pursuant
to this Act, shall be liable of punishment with a penalty of maximum two thousand
rupees or with an imprisonment for a maximum period of three months or with both.
(2) A person, who in contravention of Section 6 uses the name or the seal of the Institute
or exercises any type of authority bestowed to the Institute, shall be punished with a

The Institute of Chartered Accountants of Nepal 42


CAP II Paper 3: Corporate & Other Law

penalty of one thousand rupees maximum on first conviction, and on any subsequent
conviction thereafter, a maximum penalty of five thousand rupees or imprisonment
for a maximum period of six months or both.
Provided that this sub-section shall not apply to the organizations or university
established under their own legislation or the units within the Institute.
(3) A person, who has not obtained a Certificate of Practice and is proved to have signed
any document in capacity of the member holding Certificate of Practice, shall be
liable to punishment with a penalty up to two thousand rupees or imprisonment for a
period of up to three months or both.
(4) A member, who commits any act contrary to the provisions of this Act or Regulations
framed under this Act other than the provisions of this section, shall be suspended for
a maximum period of five years and shall be liable of punishment with a maximum
penalty of two thousand rupees or imprisonment for a maximum period of three
months or both.
(5) A complainant who lodges a complaint, without any reasonable cause to make
complaint and it is proved that the complaint was made with an intention to harass a
member, shall be liable to punishment with fine up to one thousand rupees.
(6) The complaint cases, except those to be heard under Section 14, lodged in the Council
against any member, pursuant to Section 35, shall be instituted in the concerned
Appellate Court.

6. Answer the following questions: (5×4=20)


a) When the drawer, acceptor or endorser of a Negotiable Instrument shall be
discharged from his/her liability?
b) State the meaning of Foreign Investment as per the Foreign Investment and
Technology Transfer Act, 2049. Explain repatriation facilities provided to the foreign
investors under the Act.
c) Explain the provisions relating to 'specific performance' of a contract under the
Contract Act, 2056.
d) Explain "Sick Industry" under Industrial Enterprises Act 2073.
e) Write notes on Special Program relating to Social Welfare as mentioned in the Social
Welfare Act, 2049.

Answer
a) As per section 56 of the Negotiable Instrument Act 2034; the drawer, acceptor or
endorser of a Negotiable Instrument shall be discharged from his/her liability in the
following conditions:-
I. If the Holder intentionally cancels the name of the acceptor or endorser with
intent to discharge him/her to such Holder and all parties claiming through such
Holder.
II. If a Holder thereof who otherwise discharges such acceptor or endorser or Drawer
and to all parties deriving title under such Holder after notice of such discharge.

The Institute of Chartered Accountants of Nepal 43


Suggested Answer - June 2018

III. If the Negotiable Instrument is payable to bearer or has beenindorsed in blank and
such Drawer, acceptor or endorser makes Payment in due Course of the amount
due thereon is discharged to all concerned parties thereto related with such
Negotiable Instrument.

b) Section 2(1) of the Foreign Investment and Technology Transfer Act, 2049 has given
the definition of the term foreign investment. It defines- the term foreign investment
means the following investment made by any foreign investor in any industry of
Nepal:
a. Investment in share capital.
b. Re-investment of income received from the investment in share capital.
c. Investment as a loan or loan facilities.
There are numbers of facilities given by the Act to attract the foreign investment in
Nepal. Pursuant to section 5 of the Act the following facilities are provided to the
foreign investors:
The foreign investors can repatriate the following amount in the foreign currencies
from Nepal:
a) Amount received from sale of whole or some part of share.
b) Amount received as profit or dividend of foreign investment.
c) Amount received as interest of loan or the payment of interest.
d) Amount received under the technology transfer agreement.

c) Section 86 of the Contract Act, 2056 provides for ‗Specific Performance‘. Accordingly
in case the cash compensation paid in consideration of the actual loss or damage
suffered by the aggrieved party as a result of breach of contract is not reasonable or
adequate, the aggrieved party may demand the execution of the contract as stipulated
specific performance instead of making a claim for compensation.
Notwithstanding anything contained above, no claims for execution of the contract as
stipulated specific performance shall be heard in any of the following circumstances;
i. In case the amount paid in cash as compensation for breach of contract is
adequate;
ii. In case the court cannot supervise whether or not the work to be performed under
the contract has been actually performed;
iii. In case the contract has been signed for providing services relating to personal
expertise, skill or knowledge;
iv. In case the situation is -such that the contract cannot be executed as stipulated;
v. In case the party violating the contract him/herself demands that the contract be
executed as stipulated.

d) Section 37 of Industrial Enterprises Act 2073, prescribes the provision about the
Sick Industry.
i. if any industry is being operated at least five years and in loss for a consecutive
period of last 3 years due to operate in thirty percent or below of the total
production capacity due to condition out of control and not being due to

The Institute of Chartered Accountants of Nepal 44


CAP II Paper 3: Corporate & Other Law

negligence or weakness of management, following the procedure mentioned


under prescribed guidelines, Government of Nepal may identify as sick industry.
ii. On the basis of contribution made by the industry before being sick industry on
generation of employment, import substitution or earn foreign exchange by export
promotion and can be re-operated if provided fixed concession, facility or rebate,
Nepal Government may take necessary action to restructure, reformation and
management of such industry.

e) As per the section 4 of the Social Welfare Act, 2049; Government of Nepal may
operate special Programs, relating to the social welfare activity and social service,
in the following matters:
(a) To serve interest and render welfare to the children, oldage, helpless or disabled
people.
(b) To foster participation in development and to promote and protect the welfare,
rights and interest of the women.
(c) To rehabilitate and help to lead a life of dignity to the victims of social mischief's
and also to juvenile delinquency, drug addicts and similar people involved in other
kind of addictions.
(d) To help to lead a life with dignity to the jobless, poor and illiterate people.
(e) To manage religious places and the activities of the trust Guthi institutions.
(f) To take effective management and actions for the welfare of the backward
communities and classes.

7. Write short notes on the following: (2×5=10)


a) Formation of the World Trade Organization (WTO) and its objectives.
b) List out the various leave and holidays facilities prescribed by the Labour Act, 2074.
Answer
a) It was officially constituted on January 1, 1995 which took the place of GATT as an
effective formal, organization. Contrary to the temporary nature of GATT, WTO is a
permanent organization which has been established on the basis of Multilateral
Agreements approved by participating countries. The WTO has nearly 164 members
accounting for over 98% of world trade. Around 23 others are negotiating membership.
Decisions are made by the entire membership. This is typically by consensus. The
WTO‘s top level decision-making body is the Ministerial Conference which meets once
in very two years.

The WTO secretariat, based in Geneva, has around 600 staff and is headed by a
Director-General. Its annual budget is roughly 160 million Swiss Francs. It does not
have branch offices outside Geneva. Since decisions are taken by the members
themselves, the secretariat does not have the decision making the role that other
international bureaucracies are given. The secretariat's main duties are to supply
technical support for the various councils and committees and the ministerial
conferences, to provide technical assistance for developing countries, to analyze world
trade and to explain WTO affairs to the public and media.

The Institute of Chartered Accountants of Nepal 45


Suggested Answer - June 2018

The important objectives of WTO are:


1. To improve the standard of living of people in the member countries.
2. To ensure full employment and broad increase in effective demand.
3. To enlarge production and trade of goods.
4. To increase the trade of services.
5. To ensure optimum utilization of world resources.
6. To protect the environment.
7. To accept the concept of sustainable development.

b) Labour Act, 2074 under sections 40 to 48 has provided the leave and holidays facilities
as follows.
 Weekly holiday- 1 day every week
 Public Holiday- 13 days including May Day and additional 1 day to female
employees including International Women's Day.
 Home Leave- 1 day for every 20 worked days.
 Sick Leave- up to 12 days fully paid annually.
 Mourning Leave- 13 days in case of death of persons.
 Leave in lieu- For the laborers put in work on public holiday or weekly off will be
provided accordingly.
 Maternity Leave- 98 days. Fully paid up to 60 days either before or after the
delivery.
 Paternity Leave- 15 days. Fully paid.

The Institute of Chartered Accountants of Nepal 46


CAP II Paper 3: Corporate & Other Law

Specific Comments on the performance of the students


Batch: - June 2018
Level: - CAP-II
Subject: Corporate & Other Laws
Question No. 1
Case laws and practical questions are satisfactorily solved by the students. Most of the
students have not suggested or gave answer as it is required to answer. It shows the little
preparation and not understanding of Company law. Most of the students could not answer of
the question 1(c) could not write general concept of Sections 111 and 119 of Company Act,
2063. Most students found confusion whether moral turpitude tall in offense or not.

Question No. 2
Most of the students have not answered the questions relating to the Banking laws. This
question requires the NRB and BAFIA's legal provision. However, they submit their answer
with their logic. Most of the students have written mode of the distribution of bonus which is
not relevant part of the answer.

Question No. 3
Students have not answered the questions in well manner. They are lack of knowledge of
relevant legal provisions. Students focused on benefit of compensation fund than its
operation process.

Question No. 4
Most of the students have answered this question in well manner. Few students were in
confusion about criteria of bonus distribution.
Question No. 5
Students are confused to Section 34 and 41 of NCA Act, 2053. They have mentioned 31
instead of 41 or vice-versa.
Question No. 6
Answer to 6 (a) and 6 (c) is less than standard as they had no idea of the concerned laws.
Their answers are not well prepared in other cases too. Holder of the Negotiable Instrument –
concept, meaning is not clear in answer. Most of the students have confusion regarding
holder-drawer-acceptor and endorse.

Question No. 7
Students have only common knowledge about the questions. Few students did not answer the
holidays stating the particular days.

The Institute of Chartered Accountants of Nepal 47


Suggested Answer - June 2018

Paper 4:

Financial Management

The Institute of Chartered Accountants of Nepal 48


CAP II Paper 4: Financial Management

Roll No……………. Maximum Marks - 100


Total No. of Questions – 7 Total No. of Printed Pages –15
Time Allowed – 3 Hours
Marks
Attempt all questions.
Working notes should form part of the answer. Make assumptions wherever
necessary.
1. After extensive research and development, Goodweek Tires Ltd., has recently developed a
new tire, the Super Tread, and must decide whether to make the investment necessary to
produce and market it. The tire would be ideal for drivers doing a large amount of wet
weather and off-road driving in addition to normal freeway usage. The research and
development costs so far have totaled about Rs. 10 million. The Super Tread would be put
on the market beginning this year, and Goodweek expects it to stay on the market for a
total of four years. Test marketing costing Rs. 5 million has shown that there is a significant
market for a Super Tread-type tire. Goodweek must initially invest Rs. 120 million in
production equipment to make the Super Tread. This equipment can be sold for Rs. 51
million at the end of four years. Goodweek intends to sell the Super Tread to two distinct
markets:
a. The original equipment manufacturer (OEM) market: The OEM market consists
primarily of the large automobile companies that buy tires for new cars. In the OEM
market, the Super Tread is expected to sell for Rs. 3,600 per tire when introduced. The
variable cost to produce each tire is Rs. 1,800 in first year of production.
b. The replacement market: The replacement market consists of all tires purchased after the
automobile has left the factory. This market allows higher margins; Goodweek expects
to sell the Super Tread for Rs. 5,900 per tire there. Variable costs are the same as in the
OEM market.
Goodweek intends to raise prices of its product in both markets at 5 percent every year as it
expects the same increase in variable costs. In addition, the Super Tread project will incur
Rs. 25 million in marketing and general administration costs during the first year. This cost
is expected to increase at 4 percent in the subsequent years.
Goodweek's corporate tax rate is 25 percent. The company uses a 16 percent discount rate
to evaluate new product decisions. Automotive industry analysts expect automobile
manufacturers to produce 20,000 new cars this year and production to grow at 2.5 percent
per year thereafter. Each new car needs four tires (the spare tires are undersized and are in a
different category). Goodweek expects the Super Tread to capture 11 percent of the OEM
market. Industry analysts estimate that the replacement tire market size will be 140,000
tires this year and that it will grow at 2 percent annually. Goodweek expects the Super Tread
to capture an 8 percent of this market share.
The appropriate depreciation schedule is as per SLM. The immediate initial working capital
requirement is Rs. 11 million. Thereafter, the net working capital requirements will be 15
percent of next year's sales. Except for the initial investment that will occur immediately,
assume all cash flows will occur at year-end.
Required: 20

The Institute of Chartered Accountants of Nepal 49


Suggested Answer - June 2018

Based on net present value analysis, recommend whether investing in the project is worth
taking.
Answer:

The Research & Development Cost and Test Marketing Cost incurred so far are sunk for
decision making.
A. Calculation of Initial Out Lay (Rs.)
Cost of Equipment 120,000,000
Initial W/C Infusion 11,000,000
131,000,000

B. Calculation of Annual Depreciation (Rs.)


Cost of the Machine 120,000,000
Estimated Salvage value 51,000,000
Depreciable Value 69,000,000
Useful Life 4 Yrs
Annual Depreciation 17,250,000

C. Calculation of Sales Volume (Rs.)


1 2 3 4
OEM Market
Total Market Size for the Car 20,000 20,500 21,013 21,538
Total Number of Tires Requirement 80,000 82,000 84,052 86,152
Share of Goodweek (A) (11%) 8,800 9,020 9,246 9,477
Replacement Market
Total Market of Replacement Tires 140,000 142,800 145,656 148,569
Share of Goodweek (B) (8%) 11,200 11,424 11,652 11,886
Total Size of Sales Volume (A+B) 20,000 20,444 20,898 21,363

D. Sales Piece Per Unit and Annual Sales Value (Rs.)


1 2 3 4
OEM Market 3600 3,780 3,969 4,168
Replacement Market 5900 6,195 6,505 6,830
36,697,37
Sales Value OEM Market 31,680,000 34,095,600 4 39,500,136
75,796,26
Sales Value Replacement Market 66,080,000 70,771,680 0 81,181,380
112,493,6
Total Sales Value 97,760,000 10,48,67,280 34 120,681,516

The Institute of Chartered Accountants of Nepal 50


CAP II Paper 4: Financial Management

E. Variable Cost Per Unit and Annual Total Variable Cost (Rs.)
1 2 3 4
Variable Cost for all market 1,800 1,890 1,985 2,085
Total Variable Cost 36,000,000 38,639,160 41,482,530 44,541,855

F. Estimation of Further Working Capital Requirement (Rs.)


Initial 1 2 3 4
Initial W/C
Requirement 11,000,000
W/C need at year end 15,730,092 16,874,045 18,102,227
Additional Infusion 4,730,092 1,143,953 1,228,182
Release of W/C 18,102,227

G. Terminal Cash Flows


Net Proceed from the sale (Rs.)
Cash Salvage Value 51,000,000
Book Salvage Value 51,000,000
Gain/Loss -
Tax on Gain/Loss -
Net proceed from Sale 51,000,000
Release of W/C 18,102,227
Total Terminal Flow 69,102,227
(Rs.)
H. Annual CFAT Y1 Y2 Y3 Y4
Sales 97,760,000 104,867,280 112,493,634 120,681,516
Variable Cost 36,000,000 38,639,160 41,482,530 44,541,855
Marketing & Administration
Cost 25,000,000 26,000,000 27,040,000 28,121,600
Depreciation 17,250,000 17,250,000 17,250,000 17,250,000
EBT 19,510,000 22,978,120 267,21,104 30,768,061
Tax@ 25% 4,877,500 5,744,530 6,680,276 7,692,015
EAT 14,632,500 17,233,590 20,040,828 23,076,046
Add: Depreciation 17,250,000 17,250,000 17,250,000 17,250,000
CFAT 31,882,500 34,483,590 37,290,828 40,326,046
Working Capital Adjustment (4,730,092) (1,143,953) (1,228,182) -
CFAT after WC Adjustment 27,152,408 33,339,637 36,062,646 40,326,046

I. Calculation of NPV (Rs.)

The Institute of Chartered Accountants of Nepal 51


Suggested Answer - June 2018

PVF
Particulars Time Cash Flow @16% Total PV
Initial Outlay 0 (131,000,000) 1.00 (131,000,000)
Recurring Cash Flow 1 27,152,408 0.8621 23,408,091
2 33,339,637 0.7432 24,778,018
3 36,062,646 0.6407 23,105,337
4 40,326,046 0.5523 22,272,075
Terminal Cash Flow 4 69,102,227 0.5523 38,165,160
Net Present Value 728,681

Since the NPV of the Project is positive, it should be implemented.

2.
a) AB & Co. has applied for working capital finance from a commercial bank. The
following is the firm‘s projected profit and loss account:-
Particulars Amount (Rs.)
Sales 2,247,000
Cost of goods sold 1,637,100
Gross profit 609,900
Administrative expenses, 149,800
Selling expenses 139,100 288,900
Profit before tax 321,000
Tax provision 107,000
Profit after tax 214,000
Total cost of goods sold (COGS) is calculated as follows:
Particulars Amount (Rs.)
Material used 898,800
Wages and other manufacturing expenses 668,750
Depreciation 251,450
1,819,000
Less: Stock of finished goods (10% product not yet sold) 181,900
Cost of goods sold 1,637,100
 The figures given above relate only to the goods that have been finished, and not
to work in progress.
 Goods equal to 15 percent of the year‘s production (in terms of physical units)
are in progress on an average requiring full material but only 40 percent of other
expenses.
 The firm has a policy of keeping two months‘ consumption of material in stock.
 All expenses are paid one month in arrears.

The Institute of Chartered Accountants of Nepal 52


CAP II Paper 4: Financial Management

 Suppliers of material grant one and half months‘ credit;


 Sales are 20 percent cash while remaining sold on two months‘ credit.
 70 percent of the income tax has to be paid in advance in quarterly installments.
Required: 10
Prepare an estimate of the working capital requirements of the firm on cash cost
basis. You may add 10 percent to your estimated figure to account for exigencies.
b) The Chitwan Krishi Company is planning to relax its credit policy to motivate
customers to buy on new credit terms. It is expected that the variable cost will remain
75 percent of sales. The incremental sales are expected to be sold on credit. For the
perceived increase in risk of liberalizing the credit terms, the company requires higher
return. The following table shows the projected information:
Credit Policy Required Return on Collection New Sales (Rs.)
investment Period(days)
A 20% 40 300,000
B on
25% 45 400,000
C 32% 55 500,000
D 40% 70 600,000
Required: 5
Which credit policy should the company pursue? Assume 360 days in a year.

Answers:
a)
STATEMENT SHOWING THE REQUIREMENTS OF WORKING
CAPITAL (ON CASH COST BASIS)
Particulars Rs.
A. Current Assets:
Stock of raw material (898,800+1,34,820)×2/12 172,270
Stock of Work in Progress WN II 174,945
Stock of Finished Goods WN III 156,755
Debtors 1,359,756 x 2/12 226,626
Total Current Assets 730,796
B. Current Liabilities
Creditors for Raw Materials 1,205,890 x 1.5/12 150,736
Creditors for Wages 708,875 x 1/12 59,073
Creditors for Office and Adm. Expenses 149,800 x 1/12 12,483
Creditors for Selling & Distribution 139,100 x 1/12 11,592
Expenses
Provision for Taxation 107,000 x 30/100 32,100
Total Current Liabilities 265,984
C. Net Working Capital A-B 464,612
D. Add: Safety Margin 464,612 x 10/100 46,461

The Institute of Chartered Accountants of Nepal 53


Suggested Answer - June 2018

E. Required Working Capital C + D 511,073


Working Notes:
I) Calculation of raw material consumed:
Material used in Finished goods = 898,800
Materials used in WIP = 15% ×898,800 = 134,820
Raw material consumed = 898,800 + 134,820 = 1,033,620
II) CALCULATION OF STOCK OF WORK-IN-PROGRESS
Particulars Rs.
Raw Material (898,800 x 15%) 1,34,820
Wages & Manufacturing Expenses (668,750 x 0.15 x 0.40) 40,125
Total 174,945
III) CALCULATION OF STOCK OF FINISHED GOODS AND COST OF SALES
Particulars Rs.
Direct Material Cost (898,800+134820) 1,033,620
Wages & Manufacturing Expenses (668,750+40,125) 708,875
Gross factory Cost 1,742,495
Less: Closing WIP (As per WN II) 174,945
Cost of Goods Produced 1,567,550
Less: Closing Stock (10% x 1,567,550) (156,755)
Cost of goods sold 1,410,795
Add: Office & Adm. Expenses 149,800
Add : Selling and Distribution Expenses 139,100
Total Cash cost of sales 1,699,695
Total Cash cost of Credit Sales (80% of 1,699,695) 1,359,756
OR: WN III
Valuation of Finished Goods
Raw material = 898,800×10% = 89,880
Wages and manufacturing exp.= 6,68,750×10% = 66,875
1,56,755
WN IV: Computation. Of Debtors
Debtors on sales value = ( 22,47,000×80%)×2/12 = 2,99,600
Component cost of debtors
Raw material (8,98,800 - 89,880)/22,47,000 = 36%
Wages & mgmf (6,68,750 - 66,875)/22,47,000 = 26.78%
Adm and selling exp. (1,49,800 + 1,39,100)/22,47,000 = 12.86%
Total cash cost % of Debtors = 75.6428%
Cash cost of Debtors = 2,99,600×75.6428 = 2,26,626

The Institute of Chartered Accountants of Nepal 54


CAP II Paper 4: Financial Management

IV) CALCULATION OF CREDIT PURCHASES & AMOUNT OF CREDITORS


Particulars Rs.
A. Raw Material Consumed 1,033,620
B. Add: Closing Stock 172,270
C. Opening Stock Nil
D. Purchases (A+B-C) 1,205,890
E. Creditors ( 1,205,890×1.5/12) 150,736

b)
Statement showing the Evaluation of Debtors Management Policies
Particulars Proposed Proposed Proposed Proposed
Policy A Policy B Policy C Policy D

A. Expected Profits
a) Credit Sales 300,000 400,000 500,000 600,000
b) Total Cost
i) Variable Cost 225,000 300,000 375,000 450,000
c) Expected profit (a)-(b) 75,000 100,000 125,000 150,000
B. Opportunity Cost of Investment in 5000 9375 18,333 35,000
Receivables (WN)
Net Benefits (a)-(b) 70,000 90,625 106,667 115,000
Recommendation: Policy D should be adopted since the net benefits under this policy
is higher than those under other policies.
Working Note: Calculation of opportunity cost of Investments in receivables.(Rs.)
Opportunity Cost = Total Cost × Collection Period/360 × Rate of Return/100
Policy A = 225,000×40/360×20% = 5,000
Policy B = 300,000×45/360×25% = 9,375
Policy C = 375,000×55/360×32% = 18,333
Policy D = 450,000×70/360×40% = 35,000

3.
a) Northern California Fruit Company‘s latest earnings are Rs. 2 per share. Earnings per
share are expected to grow at a 20 percent compounded annually for 4 years, at a 12
percent annually for the next 4 years and at 6 percent thereafter. The dividend-pay-out
ratio is expected to be 25 percent for the first 4 years, 40 percent for the next 4 years
and 50 percent thereafter. At the end of year 8, the price -earnings ratio for the
company is expected to be 8.5 times, where year 9‘s expected earnings per share are
used in the denominator.
Required: (4+4=8)
i) If the required rate of return is 14 present, what is the present
market price per share?

The Institute of Chartered Accountants of Nepal 55


Suggested Answer - June 2018

ii) If the present market price per share is Rs. 30, what is the stocks
expected return?
b) Following are the ratios of trading activities of East West Ltd. :-
Average collection Period 3 months Opening trade receivables Rs. 600,000
Inventory turnover ratio 1.5 times Gross profit Rs. 800,000
Average payment period 2 months Opening inventory Rs. 1,590,000
Gross profit ratio 25% Closing bills receivable Rs. 50,000
Closing bills payable Rs. 20,000
All sales are made in credit.
Required: (1+3+3=7)
i) Calculate revenue from operations
ii) Calculate closing trade debtors
iii) Calculate closing inventory
Answers:
a)
Growth Earning(Rs.) DP Ratio Dividend(Rs.)
Year 0 - 2.00 - -
Year 1 20% 2.40 25% 0.60
Year 2 20% 2.88 25% 0.72
Year 3 20% 3.46 25% 0.86
Year 4 20% 4.15 25% 1.04
Year 5 12% 4.65 40% 1.86
Year 6 12% 5.21 40% 2.08
Year 7 12% 5.84 40% 2.34
Year 8 12% 6.54 40% 2.62
Year 9 6% 6.93 50% 3.47

B. Price at the End of Eight Year


Given,
P8/E9 = 8.5 Times
P8 = 8.5 * E9
P8 = 58.90
Therefore, price per share at the end of year 8 ( P8) will be = Rs. 58.90
C. Calculation of Current Market Price Per Share

The Institute of Chartered Accountants of Nepal 56


CAP II Paper 4: Financial Management

2.34 2.62 58.90

Rs 26.72 / Share
ii) Calculation of Stock‘s Expected Return on Market Price of Rs. 30 Per Share
2.34

2.62 58.90

The expected rate of return needs to be calculated using interpolation technique.


Therefore we need to use Hit and Trial Method at different rate.
Try at 10 % of Discount Rate

2.34 2.62 58.90

P0 = Rs. 34.87
As given market price of Rs. 30 lies in between the price per share of Rs. 32.76 and
Rs. 25.25 calculated using the discount rate at 10% and 14% respectively; therefore
value can be interpolated in between 10% and 14%.
Through interpolation
Expected Rate of Return = LR X [HR-LR]
= 10% + 34.87 - 30 X [14%-10%]
34.87 - 26.72
= 10 + 4.87/8.15
= 12.39%
Therefore the expected return at current market Price of NRs 30 is
11.47%.
b) i) Revenue from operations

Gross Profit Ratio = Gross Profit * 100/Revenue from operation


25 = 800,000*100/ Revenue from operation
= Rs. 3,200,000

ii) Trade debtors

Average Colletion Period = 12/Debtor turnover ratio


3 months = 12/Debtor turnover ratio
So, Debtor turnover ratio = 4 times
Debtor turnover ratio = Credit Revenue/ Average trade receivables

The Institute of Chartered Accountants of Nepal 57


Suggested Answer - June 2018

4 = 3200,000/ Average trade Receivables


Average trade receivables = Rs. 800,000
(Opening + Closing Trade receivables)/2 = Rs. 800,000
600,000+Closing Receivables = 1600,000
Closing trade receivables = Rs. 1000,000
Closing trade debtors = Rs. 1000,000 – Bills receivables
= Rs. 950,000

iii) Closing inventory:


Cost of goods sold = Revenue – Gross Profit
= 3200,000-800,000
= Rs. 2,400,000
Inventory turn over ratio = Cost of Goods Sold/Average inventory
1.5 = Rs. 2,400,000/Average Inventory
Average inventory = Rs. 1,600,000
(Opening +Closing inventory)/2 = Rs. 1,600,000
Closing inventory = Rs. 1,610,000
4.
a) A company earns profit of Rs. 300,000 per annum after meeting its interest liability of
Rs. 120,000 on 12% debentures. The numbers of equity shares of Rs. 10 each are
80,000 and retained earnings amount to Rs. 1,200,000. The company proposes to take
up an expansion scheme for which a sum of Rs. 400,000 is required. It is anticipated
that, after expansion, the company will be able to achieve the same return on
investment as at present. The funds required for expansion can be raised either
through debt at the rate of 12% or by issuing equity shares at par. The tax rate is 25%.
Required: (7+1=8)
i) Compute the earnings per share, if:
 The additional funds were raised as debt.
 The additional funds were raised by issue of equity shares.
ii) Advise the company as to which source of finance is preferable.

b) The following is the capital structure of Simons Company Ltd. as on 31st Ashadh
2074:
Rs.
Equity shares (of Rs. 100 each) 1,000,000
10% Preference Shares (of Rs. 100 each) 400,000
12% Debentures 600,000
2,000,000
The market price of the company`s share is Rs. 110 and it is expected that a dividend
of Rs. 10 per share would be declared after 1 year. The dividend growth rate is 6%.
The company is in the 25% tax bracket.
Required: (3+4=7)
i) Compute the weighted average cost of capital.

The Institute of Chartered Accountants of Nepal 58


CAP II Paper 4: Financial Management

ii) Assuming that in order to finance an expansion plan, the company intends to
borrow a fund of Rs. 1 million bearing 14% rate of interest, what will be the
company`s revised weighted average cost of capital? This financing decision is
expected to increase dividend from Rs. 10 to Rs. 12 per share. However, the
market price of equity share is expected to decline from Rs. 110 to Rs. 105 per
share.
Answers:
a)

i)
Capital Employed Before expansion plan Rs.
Equity Shares 800,000
Debentures (Rs. 1,20,000/12)ˣ100 1,000,000
Retained Earnings 1,200,000
Total Capital Employed 3,000,000
Earnings Before the payment of Interest and Tax (EBIT)
Rs.
Profit 300,000
Interest 120,000
EBIT 420,000

Return on Investment (ROI)


ROI = (EBIT×100)/Capital Employed
= Rs. 420,000ˣ100/Rs. 3,000,000
=14%
Earnings before Interest and Tax (EBIT) After Expansion
Capital Employed after expansion = Rs. 3,400,000
(3,000,000+400,000)
Desired EBIT = 3,400,000 ˣ 14/100 = Rs. 476,000
Statement showing EPS under Present and Anticipated Expansion
Scheme
Expansion Scheme
Particulars Present Situation Additional Funds raised as
Debt Equity
EBIT (i) 420,000 476,000 476,000
Interest - Old Debt 120,000 120,000 120,000
Interest - New Debt 0 48,000 0
Total Interest (ii) 120,000 168,000 120,000
EBT (i)-(ii) 300,000 308,000 356,000
Less: Tax @ 25% 75,000 77,000 89,000
EAT 225,000 231,000 267,000
No. of Equity Shares 80,000 80,000 120,000
EPS 2.81 2.89 2.23

The Institute of Chartered Accountants of Nepal 59


Suggested Answer - June 2018

ii) If the company raises additional funds as debt the EPS would be
greater. Hence, it is suggested to raise additional funds in the form of
debt.
b) (i) Computation of the Weighted Average Cost of Capital
Source Weight(W) C/C W*C/C
Equity share 0.5 15.09% 7.55%
10% Preference share 0.2 10% 2.00%
12% Debentures 0.3 9% 2.7%
Weighted Average Cost of Capital 12.25%
ii) Computation of Revised Weighted Average Cost of Capital
Source Weight(W) C/C W*C/C
Equity share 0.333 17.43% 5.81%
10% Preference share 0.133 10% 1.33%
12% Debentures 0.200 9% 1.80%
14% Loan 0.333 10.5% 3.5%
Weighted Average Cost of Capital 12.44%
Working Notes:
1) Cost of equity shares (Ke) at present,
D1
Ke = --------------- + g = 10/110+0.06= 0.1509= 15.09%
P0
2) Revised Cost of Equity shares (Ke),
Ke = 12/105+0.06= 17.43
5.
a) Horizon Enterprises is a manufacturer and exporter of woolen garments to most of the
European countries. Its business is expanding day by day, and in the previous
financial year the company has registered a 25 percent growth in export business.
The company is in the process of considering a new investment project. It is an all
equity financed company with 1,000,000 equity shares of Rs. 50 per share. The
current issue price of this share is Rs. 125 ex-dividend. Annual earnings of Rs. 25 per
share, in the absence of new investments, will remain constant in perpetuity. All
earnings are distributed at present. A new investment is available which will cost Rs.
17,500,000 in one year's time and will produce annual cash inflows of Rs. 5,000,000
thereafter. The new investment is financed through internal accruals.
Required: 5
Analyze the effect of the new investment on dividend payments and the share price.
b) Ms. Smile currently holds two equity shares X and Y in equal proportion with the
following risk and return characteristics:
Return (RX) 24% Return (RY) 19%
σX 28% σY 23%
The returns of these securities have a positive correlation of 0.6.
Required: (3+2=5)

The Institute of Chartered Accountants of Nepal 60


CAP II Paper 4: Financial Management

i) Calculate the portfolio return and risk.


ii) How much should the correlation coefficient be to bring the portfolio risk to her
desired 15% level?
c) An analyst working with McKinsey, Singapore is trying to figure out the cost of capital
for a Hydro Power Project in Nepal for his client based at USA. As the proposed
investment is cross-border, and country risk is significantly high for Nepal in
comparison to USA, he wants to include a country risk premium in his estimate of the
cost of equity for the project. The analyst has compiled the following information for
his analysis;
 Nepali US dollar denominated 10-Year Government Bond Yield = 8.8%
 10 Year US Treasury Bond yield = 5%
 Annualized Standard Deviation of NEPSE= 32%
 Annualized Standard Deviation of Nepali US Dollar denominated 10-Year
Government Bond= 18%
 Beta = 1.25
 Expected Market Return= 10.4%
 Risk Free Rate = 4.2%
Required: 5
Calculate the Country Risk Premium and the Cost of Equity for the Hydro Power
Project in Nepal.
Answers:
a)
Calculation of Cost of Equity (ke)
Ke = D/P ˣ 100
= 25/125 ˣ 100
=20%
Earnings per share = Rs. 25
Total Earnings = 1,000,000 shares @ Rs. 25 each =Rs. 25,000,000
New project cost = Rs. 17,500,000
Dividend per share in 1st year
Since the investment is financed out of internal accruals, the amount available for
dividend at the end of first year is Rs:
=1st Year's earnings - Project cost
=Rs. 25,000,000-Rs. 17,500,000
=Rs. 7,500,000
Since all the earnings are distributed, earnings per share and dividend per share are
same.
Dividend per share in 1st year = Rs. 7,500,000/1,000,000 shares = Rs. 7.50
Dividend per share in 2nd year and will remain constant in perpetuity
=Rs 25,000,000+Rs. 5,000,000
1,000,000 shares
=Rs. 30 per share
The present value of new share price after the new investment is taken up

The Institute of Chartered Accountants of Nepal 61


Suggested Answer - June 2018

P = (Rs. 7.5/1.2) +(Rs 30/0.2 ˣ 1/1.2)


= 6.25+125
= Rs. 131.25
It is evident that the dividend payment and the price of share will increase
after new investment.

b) i) The portfolio return and risk are as under:


Portfolio Return [E(RP)] = RX × ProportionX + RY × ProportionY
= 24% × 50% + 19% × 50%
= 12% + 9.5%
= 21.5%


Portfolio risk [σP] = ơX2× ProportionX2 +
2 2
ơY ×ProportionY + 2 × ơX ×
ProportionX × ơY × ProportionY ×
CorrelationXY

= 282 × 0.502 + 232 × 0.502 + 2 × 28 × 0.50 × 23 ×


0.50 × 0.6 √
= √ 521.45 = 22.84%
ii) If Ms Smile desires the portfolio standard deviation to remain at 15%, then
correlation of equity shares X and Y shall be -0.321 as below:
152 = 282 × 0.502 + 232 × 0.502 + 2 × 28 × 0.50 × 23 × 0.50 × CorXY
225 = 328.25 + 322 CorXY
CorXY = (225 - 328.25)/322
= - 0.321
c)
i. Calculation of Country Risk Premium
Country Risk Premium (CRP) = (Yield on Nepali bond - Yield on US Bond)×(SD of NEPSE/SD
of Nepali bond)
=[0.088-0.05]*[0.32/0.18] = 0.06756 = 6.76%

ii. Cost of Equity


KCE = RF + ß [RM – RF + CRP]
= 0.042+1.25[0.104-0.042+0.06756]
= 20.4%

6. Write short note/ answer on: (4×2.5=10)


a) Assumptions of economic lot size technique
b) Cost and benefits of factoring
c) NEPSE Index
d) Venture capital

The Institute of Chartered Accountants of Nepal 62


CAP II Paper 4: Financial Management

Answers:
a) Following are the assumptions of economic lot size technique:
 Constant Annual requirement of Cash
 Constant rate of demand of cash
 Constant Transaction costs
 Constant holding costs, and
 Zero conversion period
b) The cost of factoring includes:
 Factoring commission
 Interest charged by factor on advance granted
The benefits of factoring includes:
 Saving in costs of in house credit collection department
 Saving in bad debt losses
 Saving in cost of funds invested in receivables due to reduction in average
collection period
 Saving in cash discount allowed (if any)
c) The NEPSE Index is a value weighted index of all shares listed at the Nepal Stock
Exchange and calculated on a daily basis (for the days market remain open) at the
closing price. The calculation of the NEPSE index is based on the concept of the
market capitalization which is the sum of the market capitalization of all the company
listed in the Nepal Stock Exchange. If the ratio of current period market capitalization
to the base period market capitalization is multiplied by the multiplier 100, we get
NEPSE index. This method of index calculation is called value weighted method.
Total Market Capitalization of all the Companies Listed
NEPSE Index= -------------------------------------------------------------------------------* 100
Total Base Year`s Market Capitalization

However in reality, the number of the listed companies keeps on changing, and the
number of the outstanding shares also keeps on changing as the company issues right
shares or bonus shares or common shares at the time of capital needs. The actual
practice to adjust the base period is as follows:
Adjusted Base Period =(New Market Capitalization including new listing/New
Market Capitalizations excluding new listing )* Base Year's Market Capitalization.
d) Venture Capital is long term equity investments in such business which has the
potentiality for significant growth and high return on capital. The main futures of
venture capital investment are:
(i) Providing finance to entrepreneurial talents;
(ii) Providing capital to persons having management skills;
(iii) Expecting a high return in the form of capital gain.
The venture capital schemes are designed to promote technological advancement and
innovation through introduction of new products, process or plants and equipment's.
The activities, which are in general needs venture capital support, are:
 Commercial production of viable new process products
 Technological up gradation including adoption of imported technology
suitable to local conditions.

The Institute of Chartered Accountants of Nepal 63


Suggested Answer - June 2018

 Energy conservation with innovative technology.


 Commercial exploitation of proven technology.
Thus venture capital involves in risky ventures in technology development and long
gestation technology development projects. Venture capital normally enters at the
different stages in the projects viz.
 Early Stage
Seed Capital is provided to entrepreneur for concept formulation or start-up capital
for commercial exploitation of proven product.
 Expansion Financing
Finance is provided either for major expansion of the commercial production or for
working capital support. The firm of investment takes many shapes. The most
common forms are equity purchase, conditional loans, income notes and participation
debentures.
7. Distinguish between: (4×2.5=10)
a) NPV and IRR
b) Horizontal analysis and Vertical analysis
c) Accounting break-even and Financial break-even
d) Digital money and Crypto currencies
Answers:
a) Evaluation of Non-conventional investments-Both are non-equivalent as regards the
acceptance/rejection of non-conventional investments if the projects differ in their (a)
expected lives or (b) estimated cash outflows or (c) timings of cash flows.
Reinvestment Rates-IRR assumes that intermediate cash inflows are reinvested at
IRR while NPV assumes that intermediate cash inflows are reinvested at required rate
of return (i.e. firm‘s cost of capital).
Multiple/Negative Rates-IRR can yield negative rates/multiple rates under certain
circumstances while there is no such possibility under NPV method.
b) Horizontal analysis: This technique is also known as comparative analysis. It is
conducted by setting consecutive balance sheet, income statement of statement of
cash flow side by side and reviewing changes in individual categories on year-to-year
or multi year basis. The most important item revealed by comparative financial
statement analysis is trend. The horizontal financial statements analysis is done by
restating amount of each item or group of items as a percentage.
Vertical analysis: Vertical/ Cross-sectional/ Common size statements came from the
problems in comparing the financial statements of firms that differ in size. The
vertical analysis represents the relationship of different items of financial statements
with some common item by expressing each item as a percentage of the common
item. For example:
 In the balance sheet, the assets as well as the liabilities and equity are each
expressed as a 100% and each item in these categories is expressed as a
percentage of the respective totals.
 In the common size income statement, turnover is expressed as 100% and every
item in the income statement is expressed as a percentage of turn over.
c) Accounting break-even method is the most common form of the analysis done and
one of the easiest. It is calculated as being the number of units that need to be sold in

The Institute of Chartered Accountants of Nepal 64


CAP II Paper 4: Financial Management

order to produce zero profit. More formally, the number of units required can be
calculated as total fixed cost divided by the difference between unit price and variable
cost. The difference between unit price and variable cost can be considered the profit
per unit produced and sold and a business must sell enough units to cover its fixed
costs before it can become profitable.
Accounting BEP
Financial break-even is a similar concept to accounting break-even but uses very
different measurements. It is the level of earnings needed before a firm's earnings per
share is equal to zero. Here, earnings is defined as earnings before interest and taxes,
or gross profit minus cost of sales and operating expenses and earnings per share is
most often defined as being earnings divided by the number of outstanding common
shares.

Financial BEP
d) Digital money exists only in the digital form. It doesn‘t have any physical equivalent
in the real world. Nevertheless, it has all the characteristics of traditional money.
Digital currencies don‘t have geographical or political borders; transactions might be
sent from any place and received at any point in the world. Actually, digital accounts
and digital wallets may be regarded as bank deposits.
Whereas, cryptocurrency is an asset used as a means of exchanging. It is considered
reliable because it‘s based on cryptography. It creates and analyzes the algorithms
and protocols so no information is changed or interrupted during the conversation by
third parties. Cryptocurrencies use Blockchain and a decentralized ledger. It means
that no supervisory authority controls all the actions in the network. This comes at
the expanse of all the users.
Though cryptocurrency is a type of digital currency, there are some fundamental
differences.
Structure: Digital currencies are centralized; there is a group of people and
computers that regulates the state of the transactions in the network.
Cryptocurrencies are decentralized, and the regulations are made by the majority of
the community.
Anonymity: In general, digital currencies require user identification. One needs to
upload a photo and some documents issued by the public authorities. Buying,
investing and any other processes with cryptocurrencies do not need require any of
that. Nevertheless, cryptocurrencies are not fully anonymous. Though the addresses
don‘t contain any confidential information such as name, residential address, etc.,
each transaction is registered; the senders and the receivers are publicly known.
Thus, all the transactions are tracked.
Transparency: Digital currencies are not transparent. One cannot choose the address
of the wallet and see all the money transfers. This information is confidential.

The Institute of Chartered Accountants of Nepal 65


Suggested Answer - June 2018

Cryptocurrencies are transparent. Everyone can see any transactions of any user,
since all the revenue streams are placed in a public chain.
Transaction manipulation: Digital currencies have a central authority that deals with
issues. It can cancel or freeze transactions upon the request of the participant or
authorities or on suspicion of fraud or money-laundering. Cryptocurrencies are
regulated by the community.
Legal aspects: Most countries have some legal framework for digital currencies, but
official status for cryptocurrency is not defined. The establishment of the legal
framework is only in the process.

The Institute of Chartered Accountants of Nepal 66


CAP II Paper 4: Financial Management

Specific Comments on the performance of the students


Batch: - June 2018
Level: - CAP-II
Subject: Financial Management
Question No. 1
Since question is too long students are not able to understand it properly. So, most of the
student are not able to solve it fully even if they try to attempt it. Failed to calculate CFAT.
Lacks of concept on Sunk cost. Failed to calculate working capital movement and its effect
on working capital.

Question No. 2
Most of the students who have attempted to solve part a of the question lacks conceptual
knowledge. Tax payable treatment is also not correct. Part b of the question was solved by
most of the students satisfactorily. Majority of students fails to calculate the working capital
cash cost basis. Some students could not calculate the opportunity cost.

Question No. 3
(a) Sub part of this question not able to solve by the most of student.
(b) Most solve the question but some made mistake in using COGS for calculating Inventory
used sales formula. Most of the students failed to calculate price of the share using P/E
ratio for year 8. Further almost all of them failed to apply IRR technique to arrive at the
required rate of return. Calculation of trade debtors is wrong in majority of cases.

Question No. 4
(a) Student fall to compute the EPS for the present condition and decision made without
considering present EPS. In some case working are missing. Some students fails to
calculate future earnings.
(b) Most solve this question, but some used market value weight. Calculation of weight is
wrong in some cases.

Question No. 5
(a) Not able to solve by most of the student. Calculation of new share price is wrong.
(b) Can solve by the most of the student but some made calculation error. Calculation of
portfolio risk and correlation coefficient is wrong.
(c) Being concept not practice by the student. Majority of students did not understand the
question.

Question No. 6
(a) Most of student writes on the basis of inventory management instead of cash
management.
(b) Solve by most of the student
(c) Not able to write to the point.
(d) Not able to write to the point.
Question No. 7

The Institute of Chartered Accountants of Nepal 67


Suggested Answer - June 2018

Answer not specific.

The Institute of Chartered Accountants of Nepal 68


CAP II Paper 5: Cost & Management Accounting

Paper 5:

Cost & Management Accounting

The Institute of Chartered Accountants of Nepal 69


Suggested Answer - June 2018

Maximum Marks - 100


Total No. of Questions: 6 Total No. of Printed Pages - 15
Time Allowed - 3 Hours

Marks
All questions are compulsory. Working notes should form part of the
answer.
Make assumptions wherever necessary.
1. Pappu Manufacturing Ltd. manufactures two products A and B. The manufacturing
division consists of two production department P1 and P2 and two service departments
S1 and S2. Budgeted overhead rates are used in the production departments to absorb
factory overhead to the products. The rate of department P1 is based on direct machine
hours, while the rate of department P2 is based on direct labour hours. In applying
overheads, the predetermined rates are multiplied by actual hours. For allocating the
service department costs to production departments, the basis adopted is as follows:
(i) Cost of department S1 to department P1 and P2 equal and
(ii) Cost of department S2 to department P1 and P2 in the ratio of 2:1 respectively
Annual profit plan data:
Factory Overheads budgeted for the year:
Departments P1 P2 S1 S2
Amount (Rs.) 27,85,000 22,55,000 7,50,000 5,10,000
Budgeted output of product A and B are 50,000 units and 30,000 units respectively.
Budgeted raw material cost per unit for product A and B are Rs.120 and Rs.150
respectively. Budgeted time required for production per unit are as follows:
Product A Product B
Department P1 1.5 machine hours 1.0 machine hours
Department P2 2 direct labour hours 2.5 direct labour hours
Average wage rates budgeted in Department P2 are: Product A – Rs. 72
per hour and Product B – Rs. 75 per hour.
All materials are used in Department P1 only.
Actual data (for the month of July 2017)
Units actually produced:
Product A 4,000 units
Product B 3,000 units.
Actual direct machine hours worked in Department P1:
On Product A - 6,100 hours, Product B - 4,150 hours.
Actual direct labour hours worked in Department P2:
On Product A - 8,200 hours, Product B - 7,400 hours.
Costs actually incurred:
Product A Product B
Raw Materials 5,10,000 4,80,000

The Institute of Chartered Accountants of Nepal 70


CAP II Paper 5: Cost & Management Accounting

Wages 5,80,000 5,50,000


Factory Overheads:
Departments P1 P2 S1 S2
Amount 2,81,000 2,25,000 72,000 51,000
Required: 20
i) Compute the predetermined overhead rate for each production department.
ii) Prepare a Statement showing Budgeted and Actual costs for the month of July, 2017.
Answer:
i) Computation of predetermined overhead rate for each production department for budgeted
data

Particulars Production Departments Service Departments


P1 (Rs.) P2 (Rs.) S1 (Rs.) S2 (Rs.)
Budgeted overhead for the year 27,85,000 22,55,000 7,50,000 5,10,000
Allocation of Service department 3,75,000 3,75,000 (7,50,000) -
S1's cost to Production Dept. P1
and P2 equally
Allocation of Service department 3,40,000 1,70,000 - (5,10,000)
S2‘s cost to Production Dept.
P1and P2 in the ratio of 2:1
Total 35,00,000 28,00,000 Nil Nil
Budgeted Machine hours in 1,05,000
department P1(working note 1)
Budgeted Direct labour hours in 1,75,000
department P2 (working note 1)
Budgeted Machine/ Direct labour Rs. 33.33 Rs. 16
hour rate

ii) Statement showing Budgeted and Actual Costs for the month of July 2017.

Particulars Budgeted (Rs.) Actual (Rs.)


Raw Materials used in Department P1
A (4,000 units x Rs.120) 4,80,000 5,10,000
B (3,000 units x Rs.150) 4,50,000 4,80,000
Direct Labour Cost on the basis of labour hours worked
in
department P2 5,76,000 5,80,000
A (4,000 x 2 hrs x Rs. 72) 5,62,500 5,50,000
B (3,000 x 2.5 hrs x Rs. 75)
Factory Overheads:
On machine hour basis in Department P1
A (4,000 x1.5 hrs x Rs. 33.33) 1,99,980 2,08,888
B (3,000 x 1hr x Rs. 33.33) (working note 3) 99,990 1,42,112
On Direct labour hour basis in Department P2
A (4,000 x 2 hrs x Rs. 16) 1,28,000 1,46,128

The Institute of Chartered Accountants of Nepal 71


Suggested Answer - June 2018

B (3,000 x 2.5 hrs x Rs. 16) (working note 3) 1,20,000 1,31,872


Total 26,16,470 27,49,000

Working Notes:
Product A Product B Total
1. Budgeted output (in units) 50,000 30,000
Budgeted Machine hours in 75,000 hrs 30,000 hrs 1,05,000
Department P1 (50,000 x 1.5 (30,000 x 1 hrs
hrs) hr
Budgeted Direct labour hour in 1,00,000 hrs 75,000 hrs 1,75,000
Department P2 (50,000 x 2 (30,000 x 2.5 hrs
hrs) hrs)
2. Actual output (units) 4,000 3,000
Actual Machine hours utilized in 6,100 4,150 10,250
Department P1
Actual Direct labour hours utilized in 8,200 7,400 15,600
Department P2
Working Notes: 3
Computation of actual overhead rates for each production department from actual data.
Particulars Production Departments Service Departments
P1 (Rs.) P2 (Rs.) S1 (Rs.) S2 (Rs.)
Actual factory overhead for July 2,81,000 2,25,000 72,000 51,000
2017
Allocation of Service department 36,000 36,000 (72,000) -
S1's
cost to Production Dept. P1 and P2
equally
Allocation of Service department 34,000 17,000 - (51,000)
S2's
cost to Production Dept. P1 and P2
in
the ratio of 2:1
Total 3,51,000 2,78,000 Nil Nil
Actual Machine hours in 10,250
department P1 (Working note 2)
Actual Direct labour hours in 15,600
department P2 (Working note 2)
Machine hour rate Rs. 34.2439
(Rs. 3,51,000 /10,250)
Direct Labour hour rate Rs. 17.8205
(Rs. 2,78,000 / 15,600)
Overhead Absorbed
Product A 2,08,888 1,46,128
(Rs.34.2439 (Rs.17.8205
x 6,100) x 8,200)

The Institute of Chartered Accountants of Nepal 72


CAP II Paper 5: Cost & Management Accounting

Product B 1,42,112 1,31,872


(Rs.34.2439 (Rs.17.8205
x 4,150) x 7,400)
Total 3,51,000 2,78,000

2.
a) The Margos company has just completed operation for the year 2017. The company‘s
Assistant Accountant(who is very inexperienced) prepared the following Profit and
Loss Account for the years activities:
Rs.
Sales 32,00,000
Operating Expenses: Rs.
Insurance 40,000
Gas Electricity and water 1,00,000
Direct Labour Cost 6,00,000
Indirect Labour Cost 1,20,000
Depreciation of Factory Equipment 1,60,000
Raw Materials purchased during the year 12,00,000
Rent 4,00,000
Selling and Admn. Overheads 3,20,000
29,40,000
Net Profit 2,60,000
You have been asked to assist the company in preparing correct Profit & Loss
Account for the year 2017. The following additional information is available:
1) The company is a manufacturing firm that produces a product for sale to outside
customers.
2) 80 percent of the rent paid applies to factory operation and the remainder to
Selling and Administration activities.
3) No raw materials were on hand on 1st January. However, raw material of the value
of Rs 1,50,000 purchased during 2017 were still on hand on 31st December. The
remainder was used in production during the year.
4) 70 percent of the Insurance and 90% of the Gas Electricity and Water paid apply
to factory operations the remainder apply to selling and Administration activities.
5) Work in Progress and finished goods inventories were
1st January 31st
December
Work in progress Rs. 4,20,000 Rs. 4,80,000
Finished goods Rs. 5,40,000 Rs. 4,00,000
Required: 10
i) A statement of cost of goods manufactured in 2017, and
ii) A corrected Profit & loss Account for the year ended 31st December, 2017.

b) Budhanilkantha is a Public School having 25 buses each plying in different directions


for the transport of its school students. In view of large number of students availing of
the bus service, the buses work two shifts daily both in the morning and in the

The Institute of Chartered Accountants of Nepal 73


Suggested Answer - June 2018

afternoon. The buses are garaged in the school. The workload of the students has been
so arranged that in the morning, the first trip picks up senior students and the second
trip plying an hour later picks up junior students. Similarly, in the afternoon, the first
trip takes the junior students and an hour later the second trip takes the senior students
home.
The distance travelled by each bus, one way is 16 kms. The school works 24 days in a
month and remains closed for vacation in Mansir and Kartik. The bus fee, however, is
payable by the students for all the 12 months in a year.
The details of expenses for the year 2073-2074 are as under:
Driver's salary -payable for all the 12 months Rs. 5,000 per month per driver
Cleaner's salary payable for all the 12 months
(one cleaner has been employed for every five Rs. 3,000 per month per cleaner
buses)
Licence Fees, Taxes etc. Rs. 2,300 per bus per annum
Insurance Premium Rs. 15,600 per bus per annum
Repairs and Maintenance Rs. 16,400 per bus per annum
Purchase price of the bus Rs 16,50,000 each
Life of the bus 16 years
Scrap value Rs. 1,50,000
Diesel Cost Rs. 18.50 per litre

Each bus gives an average of 10 kms per litre of diesel. The seating capacity of each bus
is 60 students. The seating capacity is fully occupied during the whole year.
The school follows differential bus fees based on distance travelled as under:
Students picked up and dropped within Bus fee Percentage of students
the range of distance from the school availing this facility
4 kms 25% of Full 15%
8 kms 50% of Full 30%
16 kms Full 55%
Ignore interest. The bus fees has to be based on average cost, you are
required to: 10
i) Prepare a statement showing the expenses of operating a single bus
and the fleet of 25 buses for a year.
ii) Work out average cost per student per month in respect of
(a) Students coming from a distance of upto 4 kms from the school;
(b) Students coming from a distance of upto 8 kms from the school; and
(c) Students coming from a distance of upto 16 kms from the school.
Answer:
a) Statement of Cost of Goods Manufactured in 2017.
Direct Material consumed :Purchased in Rs .12,00,000
Less: Closing Stock 1,50,000 Rs 10,50,000

The Institute of Chartered Accountants of Nepal 74


CAP II Paper 5: Cost & Management Accounting

Direct Labour 6,00,000


Manufacturing Overhead:
Indirect labour 1,20,000
Gas . Electricity and Water(90% of Rs. 1,00,000) 90,000
Insurance(70% of Rs. 40,000) 28,000
Depreciation 1,60,000
Rent(80% of Rs. 4,00,000) 3,20,000 7,18,000
23,68,000
Add: Opening Work –in-progress 4,20,000
27,88,000
Less: Closing Work –in – progress 4,80,000
Cost of Goods manufactured 23,08,000

(b) Corrected Profit & Loss Account


for the year ended December 31, 2017
Rs Rs
Sales 32,00,000
Less: Cost of Sales:
Cost of goods manufacture (as per ‗a‘) 23,08,000
Add: Opening stock of finished goods 5,40,000
28,48,000
Less: Closing stock of finished goods 4,00,000
Cost of goods sold 24,48,000
Add: Selling Adm:
Selling & Admn. Overhead 3,20,000
Rent (20% of Rs. 4,00,000) 80,000
Insurance (30% of Rs. 40,000) 12,000
Gas.Electricity & Wate 10% of Rs. 1,00,000) 10,000 4,22,000
Cost sales 28,70,000
Profit 3,30,000

i) Budhanilkantha Public School


Statement showing the expenses of operating a single bus and the fleet of 25
buses for a year
Particulars Fleet of 25
Per bus per annum buses per
annum
(Rs.) (Rs.)
Running costs : (A)
Diesel 56,832 14,20,800
(Refer to working note 1 )
Repairs & maintenance costs : 16,400 4,10,000
(B)
Fixed charges :
Driver's salary 60,000 15,00,000

The Institute of Chartered Accountants of Nepal 75


Suggested Answer - June 2018

Cleaners salary 7,200 1,80,000


Licence fee, taxes etc. 2,300 57,500
lnsurance 15,600 3,90,000
Depreciation 93,750 23,43,750
Total fixed charges : ( C ) 1,78,850 44,71,250
Total expenses: (A + B + C) 2,52,082 63,02,050

ii) Average cost per student per month in respect of students coming from a
distance of :

a) 4 kms. from the school


{Rs. 2,52,082 /(354 students x 12 months)} Rs. 59.34
(Refer to working note 2)
b) 8 kms from the school
(Rs. 59.34 x 2) Rs. 118.68
c) 16 kms from the school
(Rs. 59.34 x 4) Rs. 237.36

Working notes:
1. Calculation of diesel cost per bus;

No. of trips made by a bus each day : 4


Distance travelled in one trip both :
32 kms
ways
(16 kms x 2 trips)

Distance travelled per day by a bus :


128 krns
(32 kms x 4 shifts)
Distance travelled during a month : 3,072 kms
(128 kms x 24 days)
Distance travelled per year : 30,720 kms
(3,072 kms x 10 months)
No. of litres of diesel required per :
3,072 litres
bus per year
(30,720 kms /10 km)
Cost of diesel per bus per year : Rs. 56,832
(3,072 litres x Rs. 18.50)

2. Calculation of Equivalent Number of full fare paying student:


Distance % No Wt Equivalent
Within 4 km 15 18 25% 4.5
4-8 30 36 50% 18
8-16 55 66 100% 66
Total 120 88.5

The Institute of Chartered Accountants of Nepal 76


CAP II Paper 5: Cost & Management Accounting

Avg Cost/ full fare paying student = 252082


88.5x12
= 237.36
3.
a) Nepa Engineering produces one of the component ―Metal Shaft‖ from a single raw
material in economic lots of 2,000 units. The raw material cost is Rs. 2 per Metal
Shaft. Average annual demand is 20,000 units. The annual holding cost of material is
Re. 0.25 per unit and the minimum stock level is set at 400 units. Direct labour costs
for the component are Rs. 6 per unit, fixed manufacturing overhead is charged at a
rate of Rs. 3 per unit based on normal activity of 20,000 units. The company also
hires a machine on which the components are produced at a rate of Rs 200 per month.
Required: (5+3=8)
i) What will be the total annual manufacturing cost of 20,000 units of Metal Shaft?
ii) Nepa Engineering is considering the possibility of purchasing from a supplier the
components now it makes. The supplier will provide necessary components at a
unit price of Rs. 9. Transportation and storage cost would be negligible. Should the
company purchase the component instead of manufacturing?
b) A job can be executed either through workman M or N. M takes 32 hours to complete
the job while N finishes it in 30 hours The standard time to finish the job is 40 hours.
The hourly wage rate is same for both the workers. In addition workman M is entitled
to receive bonus according to Halsey plan (50%) sharing while N is paid bonus as per
Rowan plan. The works overheads are absorbed on the job at Rs. 7.50 per labour hour
worked. The factory cost of the job comes to Rs. 2,600 irrespective of the workman
engaged.
Find out the hourly wage rate and cost of raw materials input. Also show cost against
each element of cost included in factory cost. 8
c) How will you treat Idle Capacity Cost in Accounting? 4
Answer:
a) Working Note:
1. The cost of placing an order when the raw material is purchased is not given.
Let the cost of placing an order be X
_______________________________________
Economic Batch Quantity (EBQ) = 2 x Annual Consumption x Cost of placing an order
√ Cost of carrying one unit of inventory for one
year

Substituting the available Information


______________
2,000 = 2 x 20,000 x X
√ 0.25
X = 40,00,000/160,000
Or X = Rs. 25

The Institute of Chartered Accountants of Nepal 77


Suggested Answer - June 2018

Cost of placing an order = Rs. 25


2. Average stock level = Minimum stock level + ½ EOQ
= 400 + ½ (2000)
= 1,400 units

i) Calculation of total annual manufacturing cost


Rs.
Material Cost (20,000 x Rs.2) 40,000
Storage cost (1,400 x Re. 0.25) 350
Ordering Cost (20,000/2,000) x Rs. 25 250
Labour cost (20,000 x Rs. 6) 1,20,000
Rental Charges (Rs.200 x 12) 2,400
Fixed Manufacturing Overhead (20,000 x 3) 60,000
Total annual manufacturing cost 223,000

ii) Total annual manufacturing cost (calculated as above) 223,000


Less: Fixed Manufacturing Overhead (Sunk cost) 60,000
Manufacturing Cost 163,000
Purchase cost (20,000 x 9) 180,000
Fixed cost being sunk cost is not relevant for decision making. The company
should not purchase the component. Bying the components will be beneficial
only if there is alternative use of existing capacity and the opportunity cost
exceeds Rs. 17,000 i.e. Rs. 1,80,000 – Rs. 1,63,000.
b)
Working notes:
1. Time saved and wages:
Workmen M N
Standard time (hrs) 40 40
Actual time taken (hrs) 32 30
Time saved (hrs) 08 10
Wages paid @ Rs. x per hr. (Rs.) 32x 30x
2. Bonus Plan:
Halsey Rowan
Time saved (hrs) 8 10
Bonus (Rs.) 4x 7.5x
(8 hrs × Rs x) (10 hrs/40 hrs × 30 hrs × Rs x)
2
3. Total wages:
Workman M: 32x + 4x = Rs 36x
Workman N: 30x + 7.5x = Rs 37.5x
Let Material Cost be y
Statement of factory cost of the job
Workmen M N
Rs. Rs.
Material cost y y

The Institute of Chartered Accountants of Nepal 78


CAP II Paper 5: Cost & Management Accounting

Wages (Refer to working note 3) 36x 37.5x


Works overhead 240 225
Factory cost 2,600 2,600
The above relations can be written as follows:
36x + y + 240 = 2,600 …. (i)
37.5x+ y+ 225 = 2,600 …..(ii)
Subtracting (i) from (ii) we get
1.5x – 15 = 0
or 1.5 x = 15
or x = Rs. 10 per hour
On substituting the value of x in (i) we get y = Rs. 2,000
Hence the wage rate per hour is Rs. 10 and the cost of raw material input is Rs. 2,000 on
the job.
Statement of showing factory cost
Particulars M N
Material 2000 2000
Wages 360 375
Factory OH 240 225
Total 2600 2600
c)
Treatment of Idle capacity cost: Idle capacity costs can be treated in product costing, in the
following ways:
(i) If the idle capacity cost is due to unavoidable reasons such as repairs, maintenance,
changeover of job, etc, a supplementary overhead rate may be used to recover the idle
capacity cost. In this case, the costs are charged to the production capacity utilised.
(ii) If the idle capacity cost is due to avoidable reasons such as faulty planning, power
failure etc., the cost should be charged to profit and loss account.
(iii) If the idle capacity cost is due to seasonal factors, then, the cost should be charged to
the cost of production by inflating overhead rates.

4.
a) Xion Ltd. sold 3,00,000 units of its product at Rs. 40 per unit. Variable costs are Rs.
20 per unit (manufacturing costs of Rs. 14 and selling cost Rs. 6 per unit). Fixed costs
are incurred uniformly throughout the year and amount to Rs. 35,00,000 (including
depreciation of Rs. 15,00,000). There are no beginning or ending inventories.
You are required to calculate: 5
i) Break-even sales level quantity and cash break-even sales level quantity.
ii) P/V ratio.
iii) Number of units that must be sold to earn an income (EBIT) of Rs. 2,50,000.
iv) Sales level required to achieve an after-tax income (PAT) of Rs. 2,50,000.
Assume 40% corporate Income Tax rate.
b) Kalika Limited obtained a contract No. 999 for Rs. 50 lacs. The
following details are available in respect of this contract for the year
ended Ashad 31, 2074:

The Institute of Chartered Accountants of Nepal 79


Suggested Answer - June 2018

Rs.
Materials purchased 1,60,000
Materials issued from stores 5,00,000
Wages and salaries paid 7,00,000
Drawing and maps 60,000
Sundry expenses 15,000
Electricity charges 25,000
Plant hire expenses 60,000
Sub-contract cost 20,000
Materials returned to stores 30,000
Materials returned to suppliers 20,000
The following balances relating to the contract No. 999 for the year
ended on Ashad 31, 2073 and Ashad 31, 2074 are available:
as on 31st Ashad, 2073 as on 31st Ashad, 2074
Work certified 12,00,000 35,00,000
Work uncertified 20,000 40,000
Materials at site 15,000 30,000
Wages outstanding 10,000 20,000
The contractor receives 75% of work certified in cash.
Prepare Contract Account. 5
c) Moti manufacturers – a small scale enterprise, produces a single
product and has adopted a policy to recover the production overheads
of the factory by adopting a single blanket rate based on machine
hours. The annual budgeted production overheads for the year 2016-
17 are Rs. 44,00,000 and budgeted annual machine hours are
2,20,000.
For a period of first six months of the financial year 2016-2017,
following information were extracted from the books:
Actual production overheads Rs. 24,88,200
Amount included in the production overheads:
Paid as per court‘s order Rs. 1,28,000
Expenses of previous year booked in current year Rs. 1,200
Paid to workers for strike period under an award Rs. 44,000
Obsolete stores written off Rs. 6,700
Production and sales data of the concern for the first six months are as under:
Production:
Finished goods 24,000 units
Works-in-progress

The Institute of Chartered Accountants of Nepal 80


CAP II Paper 5: Cost & Management Accounting

(50% complete in every respect) 18,000 units


Sale:
Finished goods 21,600 units
The actual machine hours worked during the period were 1,16,000 hours. It is revealed
from the analysis of information that ¼ of the under/ over absorption was due to
defective production policies and the balance was attributable to increase/decrease in
costs.
Determine the amount of under/over absorption of production overheads for the six
months period of 2016-17 and point out accounting treatment of under/over absorption
of production overheads. 5
Answer:
a)
(i) Break-even Sales Quantity = (Fixed Cost /Contribution per unit)
= 35,00,000 / 20
= 1,75,000 units
Cash break even Sales Quantity = (Cash Fixed Cost/Contribution per unit)
= 20,00,000 /20
= 1,00,000 units
Where cash fixed cost = Fixed cost - Depreciation
= 35,00,000 - 15,00,000
= 20,00,000
(ii) PV Ratio = (Contribution/selling price per unit) x 100
= (20/40)*100
= 50%
(iii) No. of units that must be sold to earn an Income (EBIT) of Rs. 2,50,000
= ( Fixed Cost + Desired EBIT Level)/ Ccontribution per unit
= (35,00,000+2,50,000)/20
= 1,87,500 units
(iv) After Tax Income(PAT) = Rs. 2,50,000
Tax rate = 40%
Desired level of Profit before tax = (2,50,000/60)x100 = Rs. 4,16,667
Estimate Sales Level = (Fixed Cost + Desired Profit)/ PV ratio
= (35,00,000 + 4,16,667) / 50%
= Rs. 78,33,334

b) In the books of Kalika Ltd.


Contract No. 999 Account for the year ended 31st Ashad, 2074
Dr. Cr.
Amount
Particulars Particulars Amount (Rs.)
(Rs.)
By Material
To Work in progress bld: 30,000
returned to store
-Work certified 12,00,000 By Material 20,000

The Institute of Chartered Accountants of Nepal 81


Suggested Answer - June 2018

returned to suppliers
By Stock (Material)
-Work uncertified 20,000 30,000
c/d
15,000 By Work in
To Stock (Materials) bld
progress c/d:
To Material purchased 1,60,000 -Work certified 35,00,000
To Material issued 5,00,000 -Work uncertified 40,000
To Wages paid 7,00,000
Less: Opening 0/s (10,000)
Add: Closing 0/s 20,000 7,10,000
To Drawing and maps* 60,000
To Sundry expenses 15,000
To Electricity charges 25,000
To Plant hire expenses 60,000
To Sub- contract cost 20,000
To Notional profit cld
(balancing figure) 8,35,000
36,20,000 36,20,000
By Notional profit
To Costing P& L Alc (W N -1) 4,17,500 8,35,000
b/d
To WIP Reserve (balancing
4,17,500
figure)
8,35,000 8,35,000

*Assumed that expenses incurred for drawing and maps are used exclusively for this contract
only.
Working Note:
% Completion of contract = Work certified/Contract price×100%
= 35,00,000/50,00,000×100%
= 70%
Profit to be transferred to Costing Profit & Loss Account
=2/3*Notional profit* Cash received/Work certified
= 2/3*8,35,000*75/100
= Rs. 4,17,500
c) Amount of under/ over absorption of production overheads during the
period of first six months of the year 2016-2017:
Amount Amount
(Rs) (Rs)
Total production overheads actually
24,88,200
incurred during the period

Less: Amount paid to worker as per 1,28,000


court order

The Institute of Chartered Accountants of Nepal 82


CAP II Paper 5: Cost & Management Accounting

Expenses of previous year booked in


1,200
the current year

Wages paid for the strike period under


44,000
an award

Obsolete stores written off 6,700 (1,79,900)


23,08,300
Less: Production overheads absorbed
as per machine hour rate (1,16,000 23,20,000
hours × Rs. 20*)
Amount of over absorbed production 11,700
overheads

*Budgeted Machine hour rate (Blanket rate) = 44,00,000/2,20,000 hours


= Rs. 20 per hour

Accounting treatment of over absorbed production overheads:

The over/under absorption of overhead can be treated as


i) Charged to costing P/L A/C
ii) Carry forward to next Accounting Period
iii) Use of supplementary rate
Here the over/under absorbed overhead due to abnormal reason is charged to costing P/L
A/C and the balance under/over absorbed overhead due to normal reason is charged to
i) Cost of sales for units sold
ii) Closing stock for units unsold by calculating supplementary rate.

5.
a) From the following data of A and Co. Ltd. relating to budgeted and actual
performance for the month of March 2017, compute the Direct Material and Direct
Labour Cost variances. 7
Budgeted data for March:
Units to be manufactured 1,50,000
Units of direct material requires ( based on std. Rates) 4,95,000
Planned purchase of Raw Materials ( units) 5,40,000
Average unit cost of direct materials Rs. 8
Direct Labour hours per unit of finished goods 3/4 hr
Direct Labour Cost (total) Rs. 29,92,500
Actual data at the end of March:
Units actually manufactured 1,60,000
Direct Material Cost ( purchase cost based on units actually
issued) Rs. 43,41,900

The Institute of Chartered Accountants of Nepal 83


Suggested Answer - June 2018

Direct Material Cost ( purchase cost based on units actually


purchased) Rs. 45,10,000
Average unit cost of direct materials Rs. 8.20
Total Direct Labour hours for March 1,25,000
Total Direct Labour cost for March Rs. 33,75,000
b) It should be management‘s endeavor to increase inventory turnover but to reduce
labour turnover. Expand and illustrate the idea contained in this statement. 4
c) There is generally a divergence between financial profits and cost profits. Explain this
statement and give reasons for such divergence. 4
Answer:
a)
Direct
Material
Variance:
= Std. Cost of Actual Output - Actual Cost
= 1,60,000*3.3*8-43,41,900
DMCV = 42,24,000-43,41,900 = 1,17,900 (Adverse)
= Actual Qty.*( Std. Rates- Actual Rates)
DMPV = 5,29,500*(8-8.20) =Rs.1,05,900( Adverse)
= Std. Rate* (Std. Qty. for Actual Output- Actual Qty.)
= 8*(1,60,000*3.30-5,29,500)
DMUV =Rs. 12,000 (Adverse)

Direct
Labour
Variances
= Std. Cost for Actual Output- Actual Cost
= 1,60,000*3/4*26.60-33,75,000
DLCV = Rs. 1,83,000(Adverse)
DLRV =Actual time * (Std. Rate-Actual Rate)
=1,25,000*(26.60-27)
=Rs.50,000 (Adverse)
=Std. Rate* (Std. Time for Actual Output- Actual Time)
=26.60*( 1,60,000*3/4-1,25,000)
DLEV =Rs. 1,33,000 (Adverse)

Working Notes:
i) Standard Units of Direct Material required per unit of output:
= 4,95,000/1,50,000 = 3.30Units
ii) Total Actual Quantity of Direct Materials Used:
= 43,41,900/8.20 = 5,29,500 Units
iii) Standard Direct Labour Cost 29,92,500/ (1,50,000*3/4) =
per hour: Rs. 26.60
iv) Actual Direct Labour cost per hour:

The Institute of Chartered Accountants of Nepal 84


CAP II Paper 5: Cost & Management Accounting

=33,75,000/1,25,000 = Rs. 27
OR
Direct Material variance
Standard cost
Actual cost
SQ×SP SP×AQ
AQ×AP
(495000/150000×160000)×8 8×(4341900/8.2)
4224000 4236000
4341900

Material usage variance = 12000(A) Material Price variance =


105900(A)

Material Cost variance = 117900(A)


Where Actual quantity of material = 4341900/8.2 = 529500

Labor Variance
Standard cost
Actual cost SHr×SR SR×AHr
AHr×AR
(3/4×160000)×26.6 26.6×125000
3192000 3325000
3375000

Labor efficiency variance = 133000(A) Labor Rate variance = 50000(A)

Labour cost variance = 183000(A)


Where
Standard Rate hour = Standard Labour cost/Standard Labour Hour
= 2992500
150000×3/4
= Rs. 26.6
b)
Inventory turnover: It is a ratio of the value of materials consumed during a period to the
average value of inventory held during the period. A high inventory turnover indicates fast
movement of stock.
Labour turnover: It is defined as an index denoting change in the labour force for an
organization during a specified period. Labour turnover in excess of normal rate is termed as
high and below it as low turnover.

The Institute of Chartered Accountants of Nepal 85


Suggested Answer - June 2018

Effects of high inventory turnover and low labour turnover: High inventory turnover
reduces the investment of funds in inventory and thus accounts for the effective use of the
concern‘s financial resources. It also accounts for the increase of profitability of a business
concern. As against high labour turnover the low labour turnover is preferred because high
labour turnover causes-decrease in production targets; increase in the chances of break-down
of machines at the shop floor level; increase in the number of accidents; loss of customers
and their brand loyalty due to either non-supply of the finished goods or due to sub-standard
production of finished goods; increase in the cost of selection, recruitment and training;
increase in the material wastage and tools breakage.
All the above listed effects of high labour turnover account for the increase in the cost of
production/ process/ service. This increase in the cost finally accounts for the reduction of
concern‘s profitability. Thus, it is necessary to keep the labour turnover at a low level.
As such, it is correct that management should endeavour to increase inventory turnover and
reduce labour turnover for optimum and best utilization of available resources and reduce the
cost of production and thus increase the profitability of the organization.

c) Financial accounts are concerned with the ascertainment of profit or loss for the whole
operation of the organization or a relatively long period usually a year, without being too
much concerned with cost computation, whereas cost accounts are provided for ascertaining
the profit or loss made by manufacturing or product division/products for cost comparison
and preparation and use of variety of cost statements. As these two sets of accounts are
maintained in different forms or follow different approach, it is quite natural that their results
may also differ. Invariably, the profit and loss revealed by the financial accounts may not
agree with the profit or loss as per cost accounts.
Reasons for Disagreement
Disagreements between financial profits and cost profits may arise due to the following
reasons:
1. Items shown only in financial accounts: There are certain items which are included in
financial accounts but find no place in cost accounts. These may be–
(a) Purely financial charges: e.g., loss on sale of fixed assets, discount on issue of
shares, damages payable, etc.
(b) Purely financial income: e.g. profit on sale of fixed assets, interest received, transfer
fees, etc.
(c) Appropriation of profits: e.g. dividends, income-tax, transfer to reserves, etc.
2.Items included in cost accounts only: There are certain items which are included in cost
accounts but not in financial accounts, e.g. notional interest on capital, notional rent on
premises owned, etc.
3.Over or under absorption of overheads : In cost accounts overheads are charged to
production on pre-determined rates while financial accounts show the actual amount of

The Institute of Chartered Accountants of Nepal 86


CAP II Paper 5: Cost & Management Accounting

overheads. If the overheads charged are not equal to the amount of overheads incurred, the
difference gives rise to over or under absorption causing difference in profits.
4.Different bases of stock valuation: In financial books, stocks are valued at cost or market
price whichever is less. However, in cost accounts stock of materials may be valued on
FIFO, LIFO, average method, etc. and work-in-progress may be valued at prime cost or
works cost, thus, there is difference in profits.
5.Different methods of charging depreciation: The amount of depreciation charged may be
different in two sets of books either because of the different methods of calculating
depreciation or the rates adopted, hence the profits may be different.
6. Abnormal gains and losses: Abnormal gains and losses may completely be excluded from
cost accounts or may be taken to costing profit and loss account. If it is excluded, costing
profit/loss will differ from financial profit/loss and adjustment will be required.

6. Write short notes: (4×2.5=10)


a) Operating Costing
b) Uniform Costing
c) Standard Costing
d) Efficiency Rating
Answer:
a) Operating Costing
It is the method of ascertaining costs of providing or operating a service. This method of
costing is applied by those undertaking which provide service rather than production of
commodities. The emphasis under operating costing is on the ascertainment of cost of
services rather than on the cost of manufacturing a product. This costing method is usually
made use of by transport companies, gas and water works department, electricity supply
companies, canteens, hospitals, theatres, schools etc.

b) Uniform Costing
Uniform costing is not a separate method of cost accounting. The basic idea behind uniform
costing is that the different concerns in an industry should adopt a common method of
costing and apply uniformly the same principles and techniques for better cost comparison
and common good.
The main objectives of uniform costing are as follows:-
a) To facilitate the comparisons of cost and performance of different units in the same
industry; it provides objective basis
b) To eliminate unhealthy competition among the different units of an industry
c) To improve production capacity level and labor efficiency by comparing the production
cost of different units with each other.
d) To bring standardization and uniformity in the operation of participating units.
c) Standard Costing

The Institute of Chartered Accountants of Nepal 87


Suggested Answer - June 2018

Standard costing is defined as the presentation and use of standard costs, their comparison
with actual costs and the analysis of variances to their causes and points of incidence.
Standard costing, thus is a system of costing which can be used in conjuction with any
method of costing, like job costing, process costing etc. Standard costs are pre-determined by
using a careful analysis of production methods, physical conditions and price factors. They
represent achievable targets and help to build up budgets gauge performance and obtain
product costs. The actual costs will vary from month to month or even from day to day.
The basic objective, therefore, of standard costing system is to assists the departmental head
by identifying and describing the variances over which he has control.

d) Efficiency Rating
Efficiency is usually related with performance and may be computed by computing the time
taken with the standard time allotted to perform the given job/task. If the time taken by the
worker on a job is equal or less than the standard time, then he is rated efficient. In case he
takes more time than the standard time he is rated as inefficient. It may be computed as
follows;
Efficiency in Percent=Time allowed as per std*100/ Time taken

The Institute of Chartered Accountants of Nepal 88


CAP II Paper 5: Cost & Management Accounting

Specific Comments on the performance of the students


Batch: - June 2018
Level: - CAP-II
Subject: Cost & Management Accounting
Question No. 1
Majority of students fail to answer this question correctly. Presentation and working
note was not clear. Very few students get pass mark in this question. Only few
students were able to solve the problem. Students mainly face difficulty in calculating
the accrual and budgeted overheads.
Question No. 2
Almost all the students attempted this question. They spent more time on part a. Most
of the students did not able to calculate equivalent no. of full fare paying students.
Concept of cost sheet is not proper in number of student. Difficulty in calculation of
equivalent student and must be focus on format of running charge and standing
charge. Students were struggling to calculate the rate (full fare/ equivalent student).
Average performance is good. Few mistakes are seen in calculating direct and indirect
expenses cost of goods manufacture. Almost students not able to calculate direct cost
and cost per students. Most of the student could not calculate the average cost per
student. Lack of conceptual clarity in most of the students was noticed.
Question No. 3
Almost all the students attempted this question. No one answer correctly for 2nd part
of a. Conceptual knowledge regarding relevant costing is lacking. All most all of the
students are not able to get current ordering cost and storage cost. Negligence on
theory part. Most of the students failed to calculate the effective cost of purchase
without considering the fixed overhead. Not able to calculate ordering cost. Few
students did mistake while calculating wages because of lack of conceptual
knowledge.
Question No. 4
Majority of students answered 'a' and 'b' part correctly. For 'c' part only few students
answered correctly. Students did not properly understand the requirements of part
three. They have provided accounting treatment with entry instead of mentioning or
explaining the accounting treatment. Computed the wrong lab cost in Contract
costing. Students were not able to explain the treatment of over/under absorption of
OHS. Some of the students could not calculate the over absorbed amount.
Question No. 5
Average performance on this part. Students have tried for part two and three. But their
knowledge is poor to explain the given statements. Students were not able to compute
the variances. Student did not perform well because not able to calculate actual
quantity of material and standard rate per hour. Some of the students gave sketchy
answers.
Question No. 6
Few students even don't know the concept and just answer to attempt the question.
Students have little knowledge about efficiency rating. Theory was not up to the
mark.

The Institute of Chartered Accountants of Nepal 89


Suggested Answer - June 2018

Paper 6:

Business Communication & Marketing

The Institute of Chartered Accountants of Nepal 90


CAP II Paper 6: Business Communication & Marketing

Part A Business Communication

The Institute of Chartered Accountants of Nepal 91


Suggested Answer - June 2018

Roll No……………. Maximum Marks – 100


Total No. of Questions - 8 Total No. of Printed Pages -7
Time Allowed – 3 Hours
Marks
All questions are compulsory.
Section -'A'
1. Read the following case carefully and answer the questions given below: (4×5=20)
Dr. Chandani Barma is the CEO at one of the reputed banking companies in Nepal. She
was appointed for the post due to the rich international exposure and experience that she
had attained in the banking sector. Right from her appointment she has realized that the
company is not making desirable progress despite the proactive role of the entire team of
staff and directors. She has recently investigated from her preliminary survey that
succeeding at her workplace has a serious threat due to the subtle conflict among the
promoters of the bank that constitute the Board of Directors. The policies that they‘ve
adopted are not perfectly matching with the contexts of the new-era market. Neither have
they been able to launch the innovative and fascinating programs for their clients and
customers.
During informal communication, they often blame each other of not being ready for
taking risks for the promotion of the bank. But, in the Board meeting, neither of them
spell out clearly about the challenges and changes they desire to have in their company. In
this condition, the CEO, Dr. Barma has had a proposal approved from the BoD to carry
out a comprehensive survey among the shareholders of the company about their desires to
have challenges and changes in the company.
Now, Dr. Barma is required to develop research tools, collect data, analyze data and
prepare an analytical survey report with useful findings and recommendations.
Questions:
a) Dr. Barma is required to prepare an analytical report. What are the basic features of
this type of report? What are its major components?
b) Prepare a set of questionnaire for the survey as one of the tools of data collection.
c) Write two objectives and the statement of problem that Dr. Barma would present in
her report.
d) How is information organized in this report? Illustrate.

Answer:
a) An analytical report is usually a research report. It is also called investigative report. It
is prepared on the basis of the information obtained from respondents of the related
field. It requires basically the research tools such as questionnaires, interview, focused
group discussion, observation report, tests, discourse analysis, etc. Scientific analysis
and possible interpretations of the data are made in this type of report.
The basic components of an analytical report are:

The Institute of Chartered Accountants of Nepal 92


CAP II Paper 6: Business Communication & Marketing

 Introduction
 General background
 Statement of problem
 Objectives
 Methodology
 Analysis and interpretation
 Findings, conclusion and recommendations.

b) Questionnaire for the shareholders

Tick the best option. (SA: strongly agree, A: agree, NA: not agree)
Opinions/attitudes SA A NA
1. I prefer risks and challenges in business; they help to
grow the company.
2. Risks are useful to motivate me to work and
concentrate on duty.
3. I don‘t like to be tied up by business commitments and
relationships. It‘s good to keep on what is with us
conventionally.

4. I‘m ready to allow the BoD to invest excessively on new


technology and globalization of market.

5. I don‘t care whether one failure in business loses


everything. So, our bank must invest as required on the
innovative activities.

6. I‘d like to follow the same pattern of business since it


has to bear less or no risk.

7. …

c) The objectives of the report

 To present the accurate views of the shareholders of the bank towards taking
risks in the growth of business;
 To recommend the BoD about adopting useful strategies for business growth
with certain innovations
Statement of problem
Siddhi Bank Pvt. Limited has been launching a number of financial programs and
activities including saving, loans, e-remit, e-banking, etc. The recent perspectives and
practices of the banking corporate have been rapidly and vastly changing. In the fast
growing era of this sector this bank has not been able to attain relatively the expected
success. In this particular situation, a mini-research among the shareholders became
urgency of time so that it could give useful guidelines to the executives and Board of

The Institute of Chartered Accountants of Nepal 93


Suggested Answer - June 2018

Directors in achieving the corporate goals. This report has been prepared as a result of
this urgency. It has been prepared after a very systematic survey research.

d) The information can be organized systematically by categorizing it into different issue


based themes such as options for investment, desires to update the IT service, desires
to take risk, desire for changes, desires for no-risk, and so on. Then the information is
tabulated to ensure more systematic data for the report. The data can be organized and
analysed using tables, graphs, statistical tools such as percentage, mean, standard
deviation, etc. The irrelevant ideas/options are avoided from analysis. While
organizing the data, the extremely high level desires are arranged in one part, the
average level of remarks in the other, and the low level desires for changes and
challenges are organized in the different sub-section.

2. What are the prefatory parts of a report? Enumerate chronologically and


explain each of them in brief. 10

Answer:
Prefatory parts are front-end materials that provide key preliminary information in the
report so that readers can decide whether and how to read the report. Many of these
parts—such as the table of contents, list of illustrations and executive summary—are
easier to prepare after the text has been completed because they are based on the main
text of the report. Report prefatory parts are enumerated as:
1. Cover: Many companies have standard covers for reports, made of heavy paper and
imprinted with the company‘s name and logo. If a company doesn‘t have such covers,
you can usually find something suitable in a good stationery or office supply store.
Look for cover stock that is attractive, convenient, and appropriate to the subject
matter. Covers are typically labeled with the report title, the writer‘s name (optional),
and the submission date (also optional). Think carefully about the title. You want it to
be concise and compelling while still communicating the essence of the subject
matter.

2. Title Fly and Title Page: The title fly is a single sheet of paper with only the title of
the report on it. It adds a touch of formality, but it isn‘t really necessary, and it
consumes additional paper. The title page includes four blocks of information: (1) the
title of the report; (2) the name, title, and address of the person, group, or organization
that authorized the report (if anyone); (3) the name, title, and address of the person,
group, or organization that prepared the report; and (4) the date on which the report
was submitted. On some title pages, the second block of information is preceded by
the words Prepared for or Submitted to, and the third block of information is preceded
by Prepared by or Submitted by. In some cases, the title page serves as the cover of
the report, especially if the report is relatively short and is intended solely for internal
use.

3. Letter of Authorization and Letter of Acceptance: If you received written


authorization to prepare a report, you might want to include that letter of authorization
(or memo of authorization) in your report. If you wrote a letter of acceptance (or
memo of acceptance) in response to that communication, accepting the assignment

The Institute of Chartered Accountants of Nepal 94


CAP II Paper 6: Business Communication & Marketing

and clarifying any conditions or limitations, you might also include that letter here, in
the report‘s prefatory parts. In general, letters of authorization and acceptance are
included in only the most formal reports. However, consider including one or both if a
significant amount of time has passed since you started the project or if you do not
have a close working relationship with the audience. These pieces help make sure
everyone is clear about the report‘s intent and the approach you took to create it.

4. Letter of Transmittal: The letter of transmittal (or memo of transmittal), a specialized


form of a cover letter that is usually positioned right before the table of contents,
introduces your report to the audience. This piece says what you would say if you
were handing the report directly to the person who authorized it, so the style is often
less formal than the rest of the report. If your readers are likely to be skeptical of or
even hostile to something in your report, the letter of transmittal is a good place to
acknowledge their concerns and explain how the report addresses the issues they care
about. Also, if you need to convey sensitive information to selected audience
members, you can opt to include the letter in just those copies. Depending on the
nature of your report, your letter of transmittal can follow either the direct approach
for routine or positive messages or the indirect approach for negative messages. Open
by introducing the report and summarizing its purpose, with a statement such as ―Here
is the report you asked me to prepare on. . . .‖ The rest of the introduction includes
information about the scope of the report, the methods used to complete the study,
limitations, and any special messages you need to convey. You may also want to
acknowledge help given by others.

5. Table of Content: The table of contents indicates in outline form the coverage,
sequence, and relative importance of the information in the report. The headings used
in the text of the report are the basis for the table of contents. Depending on the length
and complexity of the report, you may need to decide how many levels of headings to
show in the contents; it‘s a trade-off between simplicity and completeness. Contents
that show every level of heading—down to the fourth or fifth level in detailed
reports—identify all the sections but can intimidate readers and blur the focus by
detracting from your most important message points. Where the detailed table of
contents could have dozens or even hundreds of entries, consider including two
tables: a high-level table that shows only major headings, followed by a detailed table
that includes everything. No matter how many levels you include, make sure readers
can easily distinguish between them.

6. List of Illustration: If you have more than a handful of illustrations in your report, or
if you want to call attention to them, include a list of illustrations after the table of
contents. For simplicity‘s sake, some reports refer to all visuals as illustrations or
exhibits.

7. Synopsis or Executive Summary: A synopsis is a brief overview of a report‘s most


important points, designed to give readers a quick preview of the contents. It‘s often
included in long informational reports dealing with technical, professional, or
academic subjects and can also be called an abstract. Because it‘s a concise
representation of an entire report, it may be distributed separately to a wide audience;

The Institute of Chartered Accountants of Nepal 95


Suggested Answer - June 2018

interested readers can then request a copy of the entire report.

3.
a) What is the communication process? Explain each of the processes. 5
b) Construct a resume for the position of Staff Accountant Analyst to
apply in one of the companies in the United States. In the resume
present the candidate's ability to contribute to a new employer. Make
sure that the employer could easily scan through the resume to find
sections of interest. 5
Answer:
a) Communication is ―the transmission of information and meaning from one individual
or group to another.‖ The crucial element in this definition is meaning.
Communication has as its central objective the transmission of meaning. The process
of communication is successful only when the receiver understands an idea as the
sender intended it. The communication process generally involves eight steps:

1. Sender has an idea: The form of the idea may be influenced by the sender‘s
mood, frame of reference, background, culture, and physical makeup, as well as
the context of the situation.
2. Sender encodes the idea in a message: Encoding means converting the idea into
words or gestures that will convey meaning. A major problem in communicating
any message verbally is that words have different meanings for different people.
That‘s why skilled communicators try to choose familiar words with concrete
meanings on which both senders and receivers agree.
3. The sender produces the message in a transmittable medium: with the appropriate
message to express an idea, the sender now needs a communication medium to
present that message to the intended audience. Media for transmitting messages
can be divided into oral, written, visual, and electronic forms.
4. Message travels over a channel: the medium over which the message is
transmitted is the channel. Messages may be sent by computer, telephone, letter,
or memorandum. They may also be sent by means of a report, announcement,
picture, spoken word, fax, or other channel. Because both verbal and nonverbal
messages are carried, senders must choose channels carefully. Anything that
disrupts the transmission of a message in the communication process is called
noise. Channel noise ranges from static that disrupts a telephone conversation to
spelling errors in an e-mail message. Such errors damage the credibility of the
sender.
5. The audience receives the message: if the channel functions properly, the message
reaches its intended audience. However, mere arrival at the destination is no
guarantee that the message will be noticed or understood correctly. Many
messages are either ignored or misinterpreted as noise.
6. Receiver decodes message: the person for whom a message is intended is the
receiver. Translating the message from its symbol form into meaning involves
decoding. Successful communication takes place only when a receiver
understands the meaning intended by the sender. Such success is often hard to

The Institute of Chartered Accountants of Nepal 96


CAP II Paper 6: Business Communication & Marketing

achieve because no two people share the same background. Success is further
limited because barriers and noise may disrupt the process.
7. The audience responds to the message: by crafting messages in ways that show
the benefits of responding, senders can increase the chances that recipients will
respond in positive ways. However, whether a receiver responds as the sender
hopes depends on the receiver (a) remembering the message long enough to act on
it, (b) being able to act on it, and (c) being motivated to respond.
8. Feedback travels to sender: the verbal and nonverbal responses of the receiver
create feedback, a vital part of the entire communication process. Feedback helps
the sender know that the message was received and understood. Senders can
encourage feedback by asking questions such as, ―Am I making myself clear?‖
and, ―Is there anything you don‘t understand?‖ Senders can further improve
feedback by delivering the message at a time when receivers can respond. Senders
should provide only as much information as a receiver can handle. Receivers can
improve the process by paraphrasing the sender‘s message.

b)
Rabindra Sing Yadav
137 Paneku Marga
Siphal, Kathamndu
Home: (0977) 01-4456789
Cell Phone: 9808967765
Email: rabindra_yadav@gmail.com

International Experienced international accountant and financial analyst with proven leadership, planning
negotiating, and intercultural communication skills. Demonstrated ability to improve process
Accounting efficiency and reduce operating costs.
Management
Staff Accountant Analyst
Inter-continental Imports: Kathmandu, Nepal
 Prepare accounting reports for wholesale giftware importer
Experience
 Adult financial transactions with suppliers in 8 countries of
SAARC
06/2010  Serve as project and team leader
to present  Created a computerized model to adjust accountants for
fluctuations in currency exchange rates
 Negotiated joint-venture agreements with major suppliers
in India and China
 Implemented electronic funds transfer for vendor
disbursements, improving cash flow and eliminating
payables clerk position

03/2007 Staff Accountant


to 06/2010 Royal Agricultural Chemicals: Biratnagar, Nepal
 Handled budgeting, billing, and credit-processing
functions
 Audited travel & entertainment expenses for 30-members
Nepali sales force
 Helped launch an online system to automated all
The Institute of Chartered Accountants of Nepal 97
accounting functions, improving reporting accuracy by 65
%
Suggested Answer - June 2018

Education Master of Business Administration with emphasis


08/2006 in International business
George Mason University, Fairfax, Virginia, USA
07/2004
Bachelor of Business Administration, Accounting
University of Texas, Austin, Texas, USA
06/2000
Chartered Accountant
The Institute of Chartered Accountants of Nepal

Special Skill Cultural  Fluent in English, Chinese and Hindi


 Extensive business contacts in South Asia
Technical  Proficient with a wide range of financial software
and systems, including Excel, Access, Microsoft
Dynamics, and SAP Business One

Reference Available on request

4. Write short notes on: (4×2.5=10)


a) Corporate social responsibilities
b) Ethnocentrism versus Cultural relativism
c) Audience-centered approach
d) Channel breakdowns

Answer:
a) Corporate social responsibilities
Corporate social responsibility (CSR) is an essential component of an ethical and
responsible business organization. It is a business movement in which commercial
organizations address the social issues identified through different sources, and run
the programs for social welfare. They invest certain amount of money so that their
business as well as the society where they have to survive can grow together.
Different infrastructural activities, educational programs, public health programs,
sports events etc. are conducted by the business organizations as their responsibility to
the society.

b) Ethnocentrism versus Cultural relativism


Ethnocentrism is the tendency to think of your own culture as the norm and to defend
your culture as the only valid worldview. It is a common phenomenon and it is found
in many cultures.

The Institute of Chartered Accountants of Nepal 98


CAP II Paper 6: Business Communication & Marketing

There is a simple way to explain ethnocentrism and how it works. Imagine a man who
drives his car every day and has been taught to stay on the right side of the street
when he does so. This man then goes to a country where people drive on the left side.
After he has returned home he goes to his friend's house for dinner and then he says,
―It is incredible, all the people drive in the opposite way. They all drive on the wrong
side!‖

Cultural relativism is somehow an opposite concept. We have seen that every culture
has the natural tendency to adopt a particular worldview. A cultural relativist is a
person who believes that every culture has to be understood in his particular context
and that cultures should not be thought as universally valid worldviews but as
particular expressions of a people.

c) Audience-centered approach
An audience-centered approach involves understanding and respecting the members
of audience and making every effort to get the message across in a way that is
meaningful to the audience. This approach is also known as adopting the ―you‖
attitude, in contrast to messages that are about ―me.‖ Learn as much as possible about
the biases, education, age, status, style, and personal and professional concerns of
your receivers. If you‘re addressing people you don‘t know and you‘re unable to find
out more about them, try to project yourself into their position by using common
sense and imagination. This ability to relate to the needs of others is a key part of
emotional intelligence, which is widely considered to be a vital characteristic of
successful managers and leaders. The more one knows about the people you‘re
communicating with, the easier it will be to concentrate on his/his needs—which, in
turn, will make it easier for them to hear your message, understand it, and respond
positively.

d) Channel breakdowns
Sometimes the channel simply breaks down and fails to deliver the message at all. A
colleague you were counting on to deliver a message to your boss might have
forgotten to do so, or a computer server might have crashed and prevented your blog
from updating. Everyone in an organization can help minimize barriers and
distractions. In any situation, a small dose of common sense and courtesy goes a long
way. Turn off that mobile phone before you step into a meeting. Don‘t talk across the
tops of other people‘s cubicles. Be sensitive to personal differences, too. For instance,
some people enjoy working with music on, but music is a huge distraction for others,
so use headphones. Finally, take steps to insulate yourself from distractions. Don‘t let
messages interrupt you every minute of the day. Instead, try to set aside time to attend
to messages all at once so that you can focus the rest of the time.

The Institute of Chartered Accountants of Nepal 99


Suggested Answer - June 2018

Part B : Marketing

The Institute of Chartered Accountants of Nepal 100


CAP II Paper 6: Business Communication & Marketing

Roll No……………. Maximum Marks – 50


Total No. of Questions - 4 Total No. of Printed Pages -6
Time Allowed – 3 Hours
Marks
All questions are compulsory.
Section -'B'

5. Read the following case carefully and answer the questions given below: (45=20)
XYZ supermarket stands out as one of the most valuable brands in Nepalese supermarket
industry in a very short span since its inception. The first store was launched in one of the
finest mall located in Kathmandu valley, covering an area of 11,000 Sq. Ft. It has been
recognized for its innovation in introducing super marketing to the Nepalese market with
a very different and organized approach. XYZ supermarket has all the material including
groceries and vegetables for the people to shop at one place it stands out as being the first
in Nepal to provide the VALUE AND SAVINGS concept to its shoppers. The XYZ
includes the following features.
 Easy shopping: Easy to search items at all pages
 Big save: every item has shopping mall and wholesale mart (B2B). Shopping mall is
for consumer. Wholesale mart is for merchants or bulk or discount shopping
 Easy to set up your store: set up a new store in our shopping mall by suing our
wholesale mart, valuable goods source. You do not have to spend time to find
wholesaler.
 low cost AD: advertise your goods or website at low cost (big size slide head banner
only), visit low cost AD page
 You can sell without fee: why pay so much selling fee? Advertising fee is free as long
as you buy from XYZ wholesale network visit free AD page.
 Easy access: easy access to xyz.com through Google, Bing, Yelp, Face book,
LinkedIn
XYZ department store is planning a major expansion drive in the valley after three years
of successful run in Kathmandu. The retail store is planning to open 25 stores within
2018. XYZ has become one of the well-known retailer stores. "Within the first three
years of its operation, XYZ has evolved as one of the most trusted brands in the Nepali
market. We have been receiving encouraging response from our consumer and this has
prompted us to extend our service areas, "said Antim Ranjit, marketing and branding
manager at XYZ. He added that the company had identified probable places for its
expansion. "XYZ has been known for the availability of quality foods and grocery
product. Therefore, we will maintain our focus on this segment," Ranjit said.
The new store will feature around 25,000 varieties of product in the initial stage. Apart
from this, the store will have a separate compartment for meat from where people can
buy mutton and chicken.
An ATM lounge and food stalls in the 'Thela" concept will be other added attractions.
"We will put on our best effort to make sure that the store becomes successful in catering

The Institute of Chartered Accountants of Nepal 101


Suggested Answer - June 2018

to their needs, ' said Ranjit. After having selected a smaller area for the new stores, the
company said that it was planning to expand in every small nook and corner of the
Kathmandu valley to cater to the demand of people from all economic groups. 'Compared
to our previous two stores, we have chosen a smaller space, "said Ranjit. XYZ is also
planning to open in other locations. "We will have relatively larger stores in these
places," Ranjit said. The company has so far distributed 45,000 privilege cards to its loyal
customers and the figure is increasing daily. Even though XYZ refused to disclose its
investment in the upcoming projects, it is said that it would be spending substantially on
these world class stores.
a) What are the major retail marketing strategies adopted by the XYZ?
b) Explain the retailing processes of XYZ.
c) State and explain the major features of XYZ. Relate them to product mix of retail
marketing.
d) What are the retail marketing mixes that the XYZ should implement to gain
competitive advantage?
Answer:

a) The major retail marketing strategies adopted by the XYZ is multi-channel marketing. It
is a chain store which has operated supermarket, department store, online store, shopping
mall, wholesale mart etc. in various locations of Kathmandu Valley. Similarly,
adoption of different and organized approach for innovation is the another strategy of
XYZ. Again, value and savings concept to its shoppers is the customer oriented strategy
adopted by the company. The consumers were provided by the easy access at one place
for purchase. XYZ facilitates the online shopping facilities through Google, Bing, Yelp,
Face book, LinkedIn. The firms also created the value and adoption of saving concept for
customer. They also provided the bulk buying discount facilities to their regular
customer. Also, the supermarket provided the wide product varieties, about 25000
product varieties.

b) So far, the retailing processes of XYZ is concerned, it has adapted two different
process, i.e. offline store and online store. In offline store, customers directly pay visit
to stores to buy their needs under single roof. Customers can make a choice by
comparing among the varieties of available products. Physical presence of customer in
the store is one of the key features of store retailing. Regarding offline store, they have
operated super market, department store and shopping mall where retail buying process
is quite different because of method of operation of retail business.. Secondly, online
shopping through mart‘s webpage is another retail buying process. Under this retailing
process customers are allowed to search all items available at all pages where they can
save their time by avoiding personal visit at stores. In online shopping, consumers
access to xyz.com through Google, Bing, Yelp, Facebook and LinkedIn.

The Institute of Chartered Accountants of Nepal 102


CAP II Paper 6: Business Communication & Marketing

c) The XYZ has been operating the retail business from different perspectives where
customer can purchase from both personal visit to store for buying and online buying
without visiting actual store. With respect to personal visit the store has provided value
and saving concept to customer by making available all merchandises at one place. The
store has retail and whole mart where customer can save by bulk and discount shopping.
This has also been characterized by easy setting up store at mall for B2B purpose.
Online shopping has been characterized for easy access through different search engine,
easy shopping as easy to search all items at all pages and advertisement facilities or free
ad page on store‘s blog. The store has also equipped with wide product varieties.
The features discussed above can be reflected in terms of product mix. Retailers
provide stores full of products to suit every consumer‘s needs to meet expectations of
varied customers. It offers all the materials including groceries and vegetables, meats.
It also provides ATM lounge and food stalls in Thela concept. The store has also
separate mart for end consumer and business consumer. Besides company‘s webpage is
also equipped with easy shopping as it is easy to search all items.

d) Retail is a service business. Thus, it should include all the seven P‘s while designing
marketing mix to gain competitive advantage. Product, place, price, promotion, people,
process and physical evidence all elements are fundamentally important to retail
management to gain competitive advantage.
In the case, XYZ has operated supermarket, department store, shopping mall, wholesale
mart and online store. First of all, there must be the same retail format and uniformity in
retail operation. If they use different retail format, at that time it is very difficult to
positioning their retail business.
As they are extending their outlets within Kathmandu valley, locations of stores could
be the major decision for its marketing mix. The store should target the area where
proximity of customer is very high. Besides distribution channels should be decided
very carefully, that ease store to manage inventory level smoothly.
Finally, the retailer has already adapted selling through internet, which is being
considered as very important promotional tool. This is the good method of marketing
communication that maintains the link with customers. However, the store should not
rely only on internet as promotional tool.

6. What is product line strategy? Discuss the trading up and trading down
strategies with suitable examples. (4+6=10)

Answer:

If any company offers more than one product items which are closely related, is popularly
known as product line strategy. For example, Nebico Biscuits offers varieties of biscuits
such as Digestive, Marie, Coconut, Glucose, Thin arrowroot, Salty, Creamy biscuits etc.

The Institute of Chartered Accountants of Nepal 103


Suggested Answer - June 2018

Companies make decisions that concern either adding new items in existing product lines,
deleting products from existing product lines, or adding new product lines. Another aspect
relates to upgrading the existing technology either to reduce the product costs or to improve
quality, for stretching (downwards, upwards, or both ways), or line filling. Among them,
trading up and trading down strategies are discussed as follows:
Trading Down or downward Line Stretching: If any company adds new product item in
their product line which is low price low quality in comparison to their existing products, it
is known as trading down or downward line stretching strategy. Companies sometimes
introduce new products with an objective of communicating an image of technical
excellence and high quality and locate at the upper end of the market. Subsequently, the
company might stretch downwards due to competitor‘s attack by introducing a low-end
product in response to competitive attack, or a company may introduce a low-end product
to fill up a vacant slot that may seem attractive to a new competitor. Another possibility is
that market may become more attractive at low-end due to faster growth rate. Downward
stretch sometimes poses risks. Low-end competitors may attack by moving into high-end,
or for a prestige-image company introduction of a low-end model may adversely affect its
product-image.
For example, Tata Nano car offered to price sensitive customer by Tata company is best
example of trading down. Similarly, CG Foods Private limited, producer of popular brand
Waiwai has added Mama instant noodle which is low price low quality in comparison to
Waiwai is the best example of trading down strategy.
Trading Up or Upward Stretching: If any company adds new product item in their product
line which is high price high quality in comparison to their existing products, it is known as
trading up or upward line stretching strategy. In this situation, companies operating at low-
end may opt to enter high- end because of better opportunities as a result of faster market
growth, or the need to create an image of full line company. There may be certain risks
associated with upward line stretching. These may include prospective customers‘
perceptions that the newcomer in the high-end category may not produce high-quality
products, or competitors already well-established in the high-end market may retaliate by
introducing items in the low-end of the market.
For example, Maruti Udyog introduced its medium-priced models such as Maruti Zen,
Maruti Esteem, Wagon R, Alto, and Swift after it had entered the market with its low-end
Maruti 800 and Maruti Omni. Toyota introduced its Lexus luxury car as a standalone
product (with no outward link to Toyota) for just this very reason. It did not want it to be in
any way affected by Toyota‘s no-doubt superb, but mass market image. Similarly, a
Tribhuvan University affiliated college which has been offering BBS is now going to offer
BBA, BHM or BBM with premium fee structure. It is called trading up strategy because
BBA, BHM or BBM is considered as high price high quality program in comparison to
BBS.

7.

The Institute of Chartered Accountants of Nepal 104


CAP II Paper 6: Business Communication & Marketing

a) ―Understanding buyer‘s behavior is a major input for marketing


strategy.‖ Justify this statement. 5
b) Give the meaning of marketing intelligence system and also point out
the sources of marketing intelligence system. 5

Answer:
a) Behavior shown by consumers while they are purchasing, consuming or disposing any
product or services is called buyer‘s behavior. It involves searching for, evaluation of,
purchase and using the product. Behavior also covers the post purchase behavior of
product – i.e. consumer satisfaction or dissatisfaction where it involves disposal of
product.
On other hand, the aim of marketing is to meet and satisfy target customers‘ needs and
wants better than competitors. In this respect, understanding buyer‘s behavior is most
important for every marketer as it is the study of how individuals, groups, and
organizations select, buy, use and dispose of goods, services, ideas, ore experiences to
satisfy their needs and wants. Understanding buyer‘s behavior provides clues for
improving or introducing products or services, setting prices, devising channels,
crafting messages, and developing other marketing activities. Furthermore, marketers
are always looking for emerging trends that suggest new marketing opportunities. And
studying consumer behavior would be the input for marketing strategy for improvement
by understanding issues such as:

 The psychology of how consumers think, feel, reason, and select between different
alternatives (e.g., brands, products, and retailers);
 The psychology of how the consumer is influenced by his or her environment (e.g.,
culture, family, signs, media);
 The behavior of consumers while shopping or making other marketing decisions;
 Limitations in consumer knowledge or information processing abilities influence
decisions and marketing outcome;
 How consumer motivation and decision strategies differ between products that
differ in their level of importance or interest that they entail for the consumer; and
 How marketers can adapt and improve their marketing campaigns and marketing
strategies to more effectively reach the consumer.

b) Marketing intelligence is the systematic collection and analysis of publicly available


information about consumers, competitors, and developments in the marketplace. The
goal of marketing intelligence is to improve strategic decision making by understanding
the consumer environment, assessing and tracking competitors‘ actions, and providing
early warning of opportunities and threats.
There are two sources of marketing intelligence. They are formal and informal.
Salesmen, intermediaries, information centers, experts, private agencies etc. are formal
sources and newspapers, trade journals, magazines, employees etc. are informal

The Institute of Chartered Accountants of Nepal 105


Suggested Answer - June 2018

sources.
1. Formal Sources: Formal sources of marketing intelligence system includes
middlemen, sales force, marketing information Centre, vacancy announcement, private
agencies, stakeholders, and international sources. The formal sources of marketing
intelligence are as follows:
a. Intermediaries: The intermediaries include dealers, distributors, wholesalers,
retailers etc. Middlemen deal with many competitors‘ products at one time.
Therefore, on the one hand, middlemen know about the policy and strategy of new
products, promotion, and pricing etc. of the different companies. And on the other
hand, they have adequate information about the market because of their direct link
with customers. They can be motivated to provide vital market information.
b. Sales-forces (Salesmen): They spot and report new developments in the market-
place. Organizations train and motivate them for marketing intelligence purposes.
Sales forces provide useful information to the business organization.
c. Marketing Information Centre: Organizations can establish a marketing
information centre for marketing intelligence. Then to receive the information
secretly, staff of this centre are sent to the competitors‘ organization. Those staff
will send the information of the competitors about their policy and strategy secretly
and regularly to the centre. They are also called business spy.
d. Vacancy Announcement: It is difficult to receive information openly from
competitors‘ staff. But the competitors‘ staff may also be the important source to get
information. So, the organization announces the vacancy where the competitors‘
staff also applies for the job. These vacancies may be both imaginary and real. In
this way, organization receives the important secret information through the
interview of the applicants.
e. Private Agencies: Private agencies may also be the sources to receive information.
Some of the private agencies collect the secret information regularly and provide
them to the interested organization. Any organization can buy the secret information
from such agencies.
f. Stakeholders: Stakeholder includes customers, government, suppliers, middlemen
and staff. They also provide information about the market. For example, from
customers organization can know about product‘s quality, feature, taste, price,
buying capacity etc. From suppliers, one can know about raw material‘s price,
technology, alternative raw materials and competitors. From government it can
know about product‘s demand, population, inflation, per capita income, etc. From
middlemen one can know about distribution cost, distribution capacity and customer
response, etc. Similarly, through staff organization can know about administrative
problems, ability to implement decision and disability, etc.
g. Specialist/Expert: A business organization may also appoint specialists for
collecting market information. Such specialists are skilled in scanning market

The Institute of Chartered Accountants of Nepal 106


CAP II Paper 6: Business Communication & Marketing

environment.
h. International Sources: Information can be obtained through international
organizations and the articles written by the expert or critics. Similarly, articles
published by WTO, SAFTA, INDP, UNDP, UNIDO, ILO, GATT, World Bank can
also be the sources of information.
2. Informal Sources: The informal sources of marketing intelligence are as follows:

Reading newspapers, magazines, trade journals etc.
 Having interactions with production manager, financial manager, personnel manager,
research and development manager, sales manager, employees etc.
 Having interaction with customers, intermediaries, other concerned persons etc.
8. Briefly explain the following: (5×2=10)
a) Any two features of marketing environment
b) Price discrimination
c) E-commerce
d) Distribution
e) Sales promotion

Answer:
a) Any two features of Marketing environment

Marketing environment consists of surrounding of the firm which affects the marketing
decision including policies and strategies of a firm. Marketing environment is characterized
by following facts.
i) It includes all the factors and forces which influence marketing , sales and consumer
satisfaction.
ii) It is always dynamic in nature.

b) Price discrimination

If any marketer sets different price for different customer for the same product. It is called
price discrimination. It is a flexible pricing policy where different price is determined for
same product on the basis of types of customer, time, quantity, geographical area, season
etc.

c) E-commerce

E-commerce is internet-based marketing. It means that the company or its website offers to
transact or facilitate the selling of products and services online. It has given rise in turn to e-
purchasing and e-marketing.

d) Distribution

The Institute of Chartered Accountants of Nepal 107


Suggested Answer - June 2018

Distribution includes all the various activities the company undertakes to make the product
accessible and available to target customers. It involves the marketing channel and physical
distribution. Marketing channel makes the product available to the customers. Physical
distribution makes the product accessible to the channel members and customers

e) Sales promotion
Sales promotion consists of all promotional activities other than advertising, personal
selling and publicity that stimulate short-term or temporary market demand for products.
The basic purpose is to stimulate on the spot buying by people through short-term
incentives. Incentives such as a samples, price concessions, and prize contests are offered to
customers to encourage buying.

The Institute of Chartered Accountants of Nepal 108


CAP II Paper 6: Business Communication & Marketing

Specific Comments on the performance of the students


Batch: - June 2018
Level: - CAP-II
Subject: Business Communication
Question No. 1
Survey questions follow a format. Most of the student doesn't know this. Students are
not clear on objectives and problem statements.
Question No. 2
Students failed to notice the word 'prefatory parts' of the report. so, 98% students
answered incorrectly.
Question No. 3
Questions are easy and clear. Answers are satisfactory. But very few of them
misunderstood the concept of CV. Instead of writing CV they have written job
application.
Question No. 4
'Channel breakdown' has confused many students. The question is about
communication channel breakdown but many students have understood it as
'organizational breakdown'.
Subject: Marketing
Question No. 5
This part is case analysis, where majority of the candidates did not get the main theme
of the case as well as theory of case, policy matter.
Question No. 6
This is the marketing strategy where majority of the students did not follow the
concept because they did not well-practice in their basic books. Most of the students
have no knowledge about trading up and trading down strategy.
Question No. 7
This part of questions focused on conceptual knowledge where students provided
reasoning while answering the questions. Students have no knowledge about the
understanding buyer's behavior and about marketing intelligence system.
Question No. 8
Here short answers questions were asked and all most all students responded properly.

The Institute of Chartered Accountants of Nepal 109


Suggested Answer - June 2018

Paper 7:

Income tax & VAT

The Institute of Chartered Accountants of Nepal 110


CAP II Paper 7: Income tax & VAT

Maximum Marks - 100


Total No. of Questions - 8 Total No. of Printed Pages -16
Time Allowed - 3 Hours
Marks
Attempt all questions. Working note should form part of the answer.
1. Mustang Cashmere Manufacturing Co. Pvt. Limited is engaged in manufacturing and sale
of high grade cashmere clothes. It provides employment opportunity to 650 Nepalese
people throughout the year. Following are the extracts of the Income Statement of the
company for the year ended on Ashadh 31, 2074.
Particulars Amount Rs.
Income:
Export Sales 6,000,000.00
Domestic Sales 4,000,000.00
Dividend Received (Net of Tax) 150,000.00
Rent Income (Related with Business) 50,000.00
Total Income 10,200,000.00
Expenditure:
Cost of Materials Consumed 3,000,000.00
Manufacturing Expenses 500,000.00
Employee Cost 1,000,000.00
Selling and Administrative Expenses 1,500,000.00
Interest and Bank Charges 500,000.00
Exchange Loss 250,000.00
Loss on Sale of Depreciable Assets 300,000.00
Total Expenditure 7,050,000.00
Operating Profit 3,150,000.00
Less: Provision for Bonus 300,000.00
Profit Before Tax 2,850,000.00
Additional information:
a) Cost of materials is consumed in the ratio of sales.
b) Exchange loss includes Rs 100,000.00 against revaluation of creditors at the year-end.
c) Selling and Administrative Expenses include Rs 70,000.00 donation given to Prime
Minister Disaster Relief Fund and Rs 300,000.00 given for construction of school.
d) Out of total provision for bonus, Rs 200,000.00 was distributed to the employees till
the time of filing income tax return. It has been decided by the management not to
pay the undistributed portion.
e) You are given the following information in regard to some of the expenses:
i) Rs 50,000.00 included in employee cost as staff welfare is personal expenses of
directors.
ii) Manufacturing expenses include Rs 30,000.00 for electricity bill of previous year.

The Institute of Chartered Accountants of Nepal 111


Suggested Answer - June 2018

iii) Selling and Administrative Expenses include Rs 10,000.00 for business


promotion which is not related to business.
Based on the above information, Compute tax liability of the Company.
Answer:
I. Determination of Tax Rates:
As per section 11 of Income Tax Act, 2058 below are the exemptions available to
Manufacturer cum Exporter of cashmere clothes being a Special Industry
i) The Company employs altogether 650 Nepali employees‘ throughout the year. So, it
is entitled to effective tax rate of 90% of applicable tax rate as per section 11(3)(Ka)
for employing 300 or more Nepali Employees
ii) The Company has export sales. So, 25% tax rebate for export Income only as per
section 11(3) (3nga), i.e. effective tax rate is 75% of applicable tax rate
Selection of Tax Exemption
As per section 11(5), if more than one tax exemption is available for the same income,
then only one tax exemption should be availed as per choice of the Taxpayer.
Hence, the Company should choose one of the exemptions mentioned below:
Exemption Effective Tax Rate Effective Tax Rate on
for Export Income Domestic Income
As a result of employing 300 or more 90% of 20% = 18% 90% of 20% = 18%
Nepali Nationals
Concessional rate as a result of export 20% - 20%*25%= 20% -0= 20%
15%
Selection of Exemption (highest rebate 15% 18%
exemption for particular Income)

II. Computation of Taxable Income and Tax Liability of Mustang Cashmere


Manufacturing Co. Pvt. Ltd. for the year ended Ashadh 31, 2074
Particulars Export Domestic Total (Rs.)
Income (Rs.) Income (Rs.)
Income From Business
Export Sales 60,00,000 - 60,00,000
Domestic Sales - 40,00,000 40,00,000
Dividend - - -
Rent Income - 50,000 50,000
Total Income 60,00,000 40,50,000 1,00,50,000
Allowable Expenses:
Interest Expenses (sec. 14) 3,00,000 2,00,000 5,00,000
Cost of Trading Stock (Sec. 15) 20,82,000 13,88,000 34,70,000
Employee Cost (Sec. 13) 5,70,000 3,80,000 9,50,000
Selling and Administrative 6,72,000 4,48,000 11,90,000
Expenses (Sec. 13)
Exchange Loss 90,000 60,000 1,50,000
Loss on Sale of Depreciable - - -
Assets

The Institute of Chartered Accountants of Nepal 112


CAP II Paper 7: Income tax & VAT

Provision for Bonus 1,80,000 1,20,000 300,000


Total Deductions 38,94,000 25,96,000 64,90,000
Assessable income from 21,06,000 14,54,000
Business
Less: Contribution to PM Relief 42,000 28,000 70,000
Fund
Taxable Income 20,64,000 14,26,000
Tax Liability 309,600 256,680 566,280
(@15% for export income and
@18% for domestic income)

Working Notesː
1. Costs of materials consumed have been allocated at the rate 60:40 and in lack of
further information other costs are also allocated in same ratio in the given solution.
2. Cost of Trading Stock
Particulars Amount (Rs.)
Cost of Materials Consumed 30,00,000
Manufacturing Expenses as per Income Statement 5,00,000
Less : Prior Period Electricity Expenses not allowed 30,000
Net Allowable Cost of Trading Stock 34,70,000
Cost of Trading Stock for Export Income – 60% of Total Expenses 20,82,000
Cost of Trading Stock for Domestic Income – 40% of Total Expenses 13,88,000
3. Employee Cost
Particulars Amount(Rs.)
Employee Cost as per Income Statement 10,00,000
Less : Personal expenses of directors not allowed 50,000
Net Allowable Employee Cost 9,50,000
Employee Cost for Export Income – 60% of Total Expenses 5,70,000
Employee Cost for Domestic Income – 40% of Total Expenses 3,80,000
4. Selling and Administrative Expenses
Particulars Amount(Rs)
Selling and Administrative Expenses as per Income Statement 15,00,000
Less : Donation given to School not allowed & PM Relief fund 370,000
Less : Business promotion Expenses not related to business 10,000
Net Allowable Selling and Administrative Expenses 11,20,000
Selling and Administrative Expenses for Export Income – 60% 6,72,000
Selling and Administrative Expenses for Domestic Income – 40% 4,48,000
5. Exchange Loss
Particulars Amount (Rs.)
Exchange Loss as per Income Statement 2,50,000
Less : Exchange Loss on revaluation of creditors 1,00,000
Net Allowable Exchange Loss 1,50,000
Exchange Loss for Export Income – 60% of Total Expenses 90,000

The Institute of Chartered Accountants of Nepal 113


Suggested Answer - June 2018

Exchange Loss for Domestic Income – 40% of Total Expenses 60,000


6. Loss on sale of Assets (1 mark for this note)
As per Income Tax 2058, the realized value of sold assets has to be adjusted in the
depreciation base amount of respective group of assets Schedule 2 Section 2(3)]. It is
assumed that the total realized value has been adjusted accordingly and hence loss on sale
of assets has not been claimed as expenses.

7. Provision for Bonus


As per Income Tax Act, 2058; a company must follow accrual basis of accounting. As
such, bonus expenses incurred during the year is deductible during the year, though the
related distribution is included in income of the employee during the Income Year when
the amount is distributed as a result of application of cash basis of accounting. As per the
latest circular by IRD, any undistributed bonus forms part if income of the person in the
immediately following Income Year (CAP II students are not expected to study circulars
issued by IRD)
Particulars Amount (Rs.)
Provision for Bonus as per Income Statement 3,00,000
Bonus for Export Income – 60% of Total Expenses 1,20,000
Bonus for Domestic Income – 40% of Total Expenses 80,000

8. Depreciation is not deducted in lack of information.

2. Mrs. Kriti is an employee in Nepal Bank Ltd. Besides employment income, she has the
following transactions for Income Year 2073/74. Sort out the income from investment
and tax liability for the year assuming that 0% and 15% tax slabs are already over by
income from employment. 10
S. No. Particulars Amount (Rs)
1 Net gain on disposal of listed shares 100,000.00
2 Purchase of New listed shares 600,000.00
3 Income from Taxi 400,000.00
4 Gain on sale of gold 100,000.00
5 Interest from savings deposit from 'A' class bank 4,000.00
6 Interest from fixed deposit from 'A' class bank 40,000.00
Net gain from disposal of ancestral house (owned and
7 resided since Bikram Sambat 2060) 5,000,000.00
8 Royalty from music video 100,000.00
9 House rent 300,000.00
10 Shares transaction commission expenses 5,000.00
11 Salary to taxi driver and other taxi related expenses 200,000.00
12 Music video expenses 20,000.00
13 House repair and maintenance expenses 20,000.00
10
Answer
S. Particulars Amount(Rs.)

The Institute of Chartered Accountants of Nepal 114


CAP II Paper 7: Income tax & VAT

No.
1 Inclusion 2,00,000.00
1.1 Net gain on disposal of shares 1,00,000.00
1.2 Royalty from music video 1,00,000.00
2 Deductions 20,000.00
2.1 Music Video expenses 20,000.00
3 Assessable income from investment 1,80,000.00

Calculation of Tax liability:

Royalty income 80,000×25% 20,000.00


Income from disposal of listed shares 1,00,000×5% 5,000.00
Tax liability 25,000.00

Working Notes:

1) Purchase of new listed shares forms part of investment income, but will only be
considered while computing net gain from disposal of non business chargeable asset.
2) Income from taxi is taxed at a fixed presumptive tax. So, this is not an investment
income.
3) Gold is personal asset and not an investment asset and is not taxed on gain from it.
4) Interest paid to natural persons from a bank and financial institution is a final
withholding income. So, interests from savings and fixed deposits are not included.
5) House rent income of a natural person not related to business is a final withholding
income, so, not included in income from investment.
6) Shares transaction commission expenses is already adjusted while calculating net gain
on disposal of listed shares, so, no further deduction required.
7) Expenses related to final withholding incomes are not deductible for calculating
assessable income.
8) Disposal of ancestral house owned and resided for 10 or more years is not an
investment activity. So, it is not included in income from investment.
3.
a) Bottlers Nepal Ltd. Kathmandu has paid consulting fees Rs. 30 million in the year
2074/75 to International Consultancy Group, New York (ICGN). As per the
agreement, the consultant should deploy at least 2 employees at the project site
located at Butwal, Province No. 5 throughout the year. Can we define ICGN as a
permanent establishment in Nepal? What about the tax withholding implications in
the following situations: 5
i) If the company is registered in Nepal through its local agent and payment was
made to local agent.
ii) If the company is not registered in Nepal and payment was transferred directly to
their bank maintained in New York.
b) Mr. Ramji Paneru retired in 2075 Baishakh end from 30 years of his service with
Nepal Timber Corporation and received lump sum retirement payments as follows: 5
i) Payment from approved retirement fund Rs. 1,675,852.00.

The Institute of Chartered Accountants of Nepal 115


Suggested Answer - June 2018

ii) Payment from unapproved retirement fund Rs. 675,800.00 to which his
contribution was Rs. 511,256.00. This payment is related only with the period
after Income Tax Act, 2058.
You are required to calculate the tax withholding on these payments along with
reason therefor.
Answers:
a) As per Section 2(Ka Nga) a resident person includes a foreign permanent
establishment of a non-resident person situated in Nepal. Further, section 2(Ka Na)
defines permanent establishment that also includes the place: "one or more than one
place in any country where any person has delivered technical, professional or
consultancy service through an employee or in any other manner for more than ninety
days at one or several times in a period of any twelve months". As per these
definitions, International consultancy group is considered a permanent establishment
in Nepal.
i) If payment was made in Nepal, as since the entity is registered for tax purpose; it
must register for VAT as well. While making payment of service fee to VAT
registered service provider, tax shall be withheld @ 1.5% of payment as per
proviso (gha) of Sec. 88 (1) of the Act. If the permanent establishment is not
registered for VAT purpose, the tax shall be withheld @ 15% as per Sec. 88 (1)
(Alternatively, students may also assume that the payment of consulting fee is not
part of independent consulting contract but part of supervisory activities in
relation to construction contract when Sec. 89 is applicable and TDS @ 1.5%
shall be deducted as per Sec. 89 (1) in both cases when there is VAT registration
or not)
ii) If amount was transferred directly, tax rate shall be 15 % as per section 88(1). As
per Income Tax Act, the PE is required to register in IRD for the transaction
including VAT registration. So, the first option is in compliance of Income Tax
Act.
b) TDS Calculation on retirement Payment to Mr. Ramji Paneru

Approved Unapproved
Particulars Retirement Fund Retirement Fund
Payment 1,675,852 675,800
Less Accrued Prior 2045.02.01 to 18 Chaitra
2058 (13.88/30 Yrs.) 775,361 -
Less 50% Deduction: higher of 500,000 or
50% of (1,675,852-775,361) 500,000 -
Less employees contribution - 511,256
Taxable Amount 400,491 164,544
Rate of Tax 5% 5%

TDS Amount 20,024 8,227.20


4.
a) Shreedhar Kumar is appointed on 2066.04.01 as Accounts Officer of Nepal Telecom
Limited with salary structure of Rs. 25,000.00 - 500.00(6)-1,000.00(12). He has

The Institute of Chartered Accountants of Nepal 116


CAP II Paper 7: Income tax & VAT

received interest free home loan Rs. 1,624,000.00 from the employer. Assume market
rate of interest 9% and make other practical assumptions as necessary. Calculate
monthly withholding tax to be deducted from his salary in Income Year 2074/75. 5
b) AB (Pvt) Ltd has made an agreement with ABC (Pvt) Ltd to arrange the cash flow for
its transactions. Chairman of AB (Pvt) Ltd. is also a representative board member of
ABC (Pvt) Ltd. In Shrawan 1, 2072, loan amounting Rs. 50 million had been
provided by the ABC to the AB. Interest rate mutually agreed was 10 percent. Both
the companies are located at Kathmandu. Tax Officer denied the interest expenses of
Rs. 5 million claimed by AB (Pvt) Ltd. on the ground that this transaction is a related
party's transaction. Answer whether the Tax Officer's assessment is correct.
c) Mr. A and Mr. B contributed Rs. 4 million and Rs. 6 million respectively and
deposited into a common bank account to be used to purchase a plot of land and to
resale the same in the future. After 6 months of depositing the money into the bank
account, they purchased a plot of land for Rs. 9 million and got the land registered in
their joint name. Expenses on registration and commission at the time of purchasing
the land are amounted to Rs. 1 million. Later, they sold the plot of land for Rs. 17.2
million. Also, they paid Rs. 1.2 million as sales commission and other incidental
expenses pertinent to the disposal. The bank has given interest @ 12% per annum on
such deposit.
Compute the amount to be included as income in the hand of Mr. A and Mr. B. 5
d) Mr. Ram Prasad, Nepali resident with single status for taxation, had submitted the
following details of income from business and investment as self-assessment of his
tax liability for the Income Year 2073-74:
Amount (Rs.)
Country Net Assessable Income Tax paid
Nepal 500,000.00 25,000.00
USA 200,000.00 65,000.00
UK 300,000.00 0
Canada 500,000.00 80,000.00
Total 1,500,000.00 170,000.00
Compute any unabsorbed foreign tax credit of Mr. Ram Prasad during
the assessment. 5
Answers:
a) Calculation of monthly withholding tax of Mr. Shreedhar Kumar
(Single Status)
Particulars Amount Remarks
(25000+500*6+1000*2)*
Basic Salary 360,000 12
at the rate of 9% on the
Deemed Income on Interest free loan average loan outstanding
146,160 during the year
Assessable Income 506,160
Taxable Income 506,160
First Rs. 350,000 @1% 3,500

The Institute of Chartered Accountants of Nepal 117


Suggested Answer - June 2018

Next Rs. 100,000@15% 15,000


Balance Rs. 56,160 @25% 14,040
Total Tax Liability for the year 32,540
Monthly Deduction required 2,711.67
Social Security Tax 291.67
Income Tax 2,420

Alternatively,
a. The student may assume couple assessment, in which case, the tax liability and monthly
deduction will be as follows:
First Rs. 400,000 @ 1% - 4,000
Next Rs. 100,000 @ 15%- 15,000
Balance Rs. 6,160 @25%- 1,540
Total tax liability – 20,540
Monthly deduction- 1711.67
SST- 333.33
Income Tax- 1378.33
b. The students may make assumptions of employers‘ contribution to ARF @ 10% of basic
salary and grade and 20% of basic salary and grade deposited in the approved retirement fund
account of the employee. Students further assume that festive allowance equal to one month‘s
basic salary and grade is provided by the employer during the year.

b) Section 2(Ka Na) defines the associated person, to any one or more than one person
or group of persons who act as per the intention of each other, it also includes: any
entity which by itself or jointly with any other person related with it or with an
assisting entity or any other person or entity related with such assisting entity controls
50 % or more of the income, capital or voting right of any entity or derives benefits
therefrom.
If associated person is established as per the definition, then tax officer can, by
issuing a notice in writing, distribute, appropriate or allocate the amounts to be
included or deducted in computing the income between those persons in such a
manner as to reflect taxable income or the payable tax that could be set for them as
per Section 33 of Income Tax Act.
As per these provisions of Income Tax Act, the ground of the tax officer is wrong; the
associated person could not be established due to lacking of holding percentage on
AB Pvt. Ltd. by the ABC Pvt. Ltd. But the tax officer should make assessment of
TDS @ 15 % on interest payment as per Section 88(1). AB Pvt. Ltd should deduct the
TDS of Rs. 750,000 (5 million*.15) on the interest payment.
(Alternatively students can solve the problem as above citing the provisions of Sec. 14
of the Act and demonstrates that the provisions does not restrict the deduction of
interest expenses merely on the basis they are associated, even when the relation of
associated persons are established)

c)
(Rs. in Million)

The Institute of Chartered Accountants of Nepal 118


CAP II Paper 7: Income tax & VAT

Particulars Amount Remarks


Expensesː
MA Total Mr. A Mr. B
Cost of Plot of Land 9
Other Expenses 1
Total cost of Land (A) 10 4 Apportioned
6 on the basis of 2ː3

Sales Income 17.2


Lessː Expenses 1.2
Net sales Income (B) 16 6.4 9.6
Income (B-A) 6 2.4 Apportioned
3.6 on the basis of 2ː3
Apart from aforesaid income, Mr. A and Mr. B have received interest income of
Rs.600,000 (10,000,000×12%×6/12) as the bank interest. This interest income amount
also needs to be shared between Mr. A and Mr. B in the ratio of 2ː3 as followsː
Mr. A ː Rs.240 thousand, Mr. B ː Rs.360 thousand,
The above interest income is subject to deduction of applicable TDS which is final.
d)

Computation of Taxable Income of Mr. Ram Prasad for FY.2073-74


Calculation of Average Tax Rate in Nepal:
Tax liability
First 350,000 0% -
Next 100,000 15% 15,000
Rest 1,050,000 25% 262500

Therefore, Average Tax Rate = Tax liability before foreign tax credit/taxable income
* 100
(277,500/1,500,1000)×100
= 18.5%
Computation of foreign tax credit for the FY 2073-74
Countries Foreign Ass. Tax Avg. rate Maximum Eligible Unaborbed
Income paid (a) of tax in eligible foreign foreign tax
Nepal tax credit (b) credit
minimum of
(a) or (b)
USA 200,000 65,000 37,000 37,000 28,000
UK 300,000 -
55,500 - -
Canada 500,000 80000 18.5% 92,500 80,000 -
5. Write short notes: (4×2.5=10)
a) Deductible expenses from cash payment
b) Interest
c) Amended assessment
d) Permanent Account Number

The Institute of Chartered Accountants of Nepal 119


Suggested Answer - June 2018

Answers:
a) According to Section 21, of Income Tax Act, 2058 any expenditure paid through cash
for more than Rs 50,000 by a taxpayer having annual turnover for more than Rs. 20
lakh cannot be deductible. However, the following payments are deductible even
though, they exceed the cash payment limit of Rs. 50,000.
 Payment to GON, Constitutional bodies, public enterprises and BFI,
 Payment to producer of primary agricultural produces even when the products are
primarily processed,
 Payment of retirement contribution or retirement payment
 Payment made on the day of closure of banking service or on circumstances to be
paid only in cash, or
 Payment made in such area where banking facility is not available
 Amount deposited into the bank account of the receiver.
b) "Interest" means the following payment or profit:
(1) Payment under debt liability except the principal,
(2) Profit made from concession, premium, alteration payment or from similar
payment, and
(3) The amounts referred to in Section 32 receivable as an interest out of the payment
to be made by a person who acquires any property under annuities or installment
sale or of the payment made to any person for the use of any property under a
financial lease.

c) As per Section 101 of Income Tax Act, 2058, Inland Revenue Officer (IRO) can
make an amended assessment of any return filed by any tax payers solely on the
ground that the IRO deemed fit to do such amended assessment based on the IRO‘s
best judgement, and should be done in the manner that is consistent with the intention
of the Act.

In case IRD thinks it proper to do so, the assessment may be done as many times as
the IRD thinks appropriate but within 4 years of:
 In the case of an assessment under section 99, the due date of filling of the return,
or
 In the case of jeopardy assessment, the date on which the notice of assessment is
served to the tax payer under sec.102.
The maximum time limit of 4 years is not applicable in case of fraudulent assessment
identified by IRD, when the assessment shall be completed within one year of the
information of fraud.
Amended assessment is not allowed within 4 years time limit as well when the
matters in relation to the assessment are finalized by revenue tribunal or competent
court unless such courts or tribunal permits the re-assessment.

The Institute of Chartered Accountants of Nepal 120


CAP II Paper 7: Income tax & VAT

Before finalizing the amended assessment, a show cause notice shall be sent to tax
payer for defense by providing a time limit of 15 days.

d) As per Section 78, the following provisions are stipulated about PAN.
a. Subject to this Act, the Department shall issue a person with a number to be
known as a Permanent Account Number for the purposes of identifying the
person.
b. The Department may require a person to show their Permanent Account Number
in any return, statement, or other document used for the purposes of this Act.
c. GON may prescribe situations in which a person is required to show or quote their
Permanent Account Number.
d. Notwithstanding anything contained in subsection 1, person mentioned in
subsection 3 must receive PAN before commencement of business transaction.
e. A person cannot relieve from tax liability by reason of not having PAN.
6. Nepal Surgical House (Pvt) Limited is a principal stockiest for Nepal of Medtronic Inc,
Hong Kong (MIH). The company has policy to sell the items to other distributors with
15% markup at landed cost. Medical and surgical equipment manufactured by MIH are
subject to both VAT attractive and VAT exempt. MIH makes the shipment by billing on
FOB Price and import is done through irrevocable letter of credit. The Company provides
following information for the month of Chaitra 2074:
a) All items are subject to 5% custom duty under HS code 84.
b) Shipment against the LC established in Falgun 2074 having FOB value USD
126,300.00 of which USD 43,500.00 is subject to VAT was released during the
month and entire stock was sold on the same month.
c) Of the total import of taxable goods during the month, items having FOB Price USD
26,400.00 was revalued to USD 33,100.00 by the custom authorities.
d) Terms of Payment 20% during the establishment of LC and 80% on release of
documents.
e) Effective exchange rate per USD was 102.6, 103.8 & 103.45 during the date of LC
establishment, document release and custom clearance respectively.
f) Previous non-VAT stock having landed cost of Rs. 2,526,200.00 was also sold during
the month.
g) The company paid Rs. 146,800.00 inclusive of tax for Birgunj Kathmandu
transportation and insurance of the consignment.
On the basis of above information, prepare VAT statement of Nepal Surgical House (Pvt.)
Limited for the month of Chaitra 2074 considering opening VAT credit of Rs. 73,612.00.
Answers:
Amount (Rs.) Value Added Tax
Particular (Rs.)
Taxable Local Sales 5,491,073.90 713,839.61
Exempt Local Sales 12,978,175.14 -
Taxable Export - -
Exempt Export - -

The Institute of Chartered Accountants of Nepal 121


Suggested Answer - June 2018

Total 18,469,249.04 713,839.61


VAT on Import 708,870.44
VAT on Transportation Cost (Birgunj to Ktm)
(44,743.87*13%) 5,816.70
VAT Dr. for the month 4,969.17
Opening VAT Credit 73,612
Credit balance carried forward for Baisakh
2075 (74,459.53)

Note 1: Calculations of the Sales during the month

Particular Total Taxable Exempt

Previous No VAT stock sold 2,526,200 2,526,200


Sale of the Import Stock During the
month 15,943,049.04 5,491,073.90 10,451,975.14

Total Sales During the Month 18,469,249.04 5,491,073.90 12,978,175.14

VAT Collected on Sales 713,839.61 713,839.61 -

Note 2: Calculations dealers‘ price of the import

Particular Total Taxable Exempt

FOB Price in USD 126,300 43,500 82,800


Equivalent NPR
20% of FOB paid during LC
establishment and effective
exchange rate of NPR 102.6 per
USD 2,591,676 892,620 1,699,056
80% of FOB paid during LC
establishment and effective
exchange rate of NPR 103.8 per
USD 10,487,952 3,612,240 6,875,712

Total FOB Price in NPR 13,079,628 4,504,860 8,574,768

Custom Duty @ 5% 653,981.40 225,243 428,738.40

Transportation & Insurance in Nepal 129,911.50 44,743.87 85,167.64

Landed Cost 13,863,520.90 4,774,846.87 9,088,674.04

15% Markup 2,079,528.14 716,227.03 1,363,301.11

The Institute of Chartered Accountants of Nepal 122


CAP II Paper 7: Income tax & VAT

Sales Price to the distributors 15,943,049.04 5,491,073.90 10,451,975.14

Note 3: Calculations VAT paid on import

Particular Total Taxable Exempt

FOB Price in USD 126,300 43,500 82,800


Additional Revaluation by custom
authorities 6,700 6,700
Revised FOB Price in USD
133,000 50,200 82,800
Applicable Exchange Rate
103.45 103.45 103.45

Total FOB Price in NPR 13,758,850 5,193,190 8,565,660

Custom Duty @ 5% 687,942.50 259,659.50 428,283

Taxable Value 14,446,792.50 5,452,849.50 8,993,943

Value Added Tax Paid in Purchase 708,870.44 708,870.44 -

7.
a) Sussex (Pvt) Ltd, Kathmandu imports goods and directly supplies to
the retailers. From the following information, compute the amount of
VAT on sales.
Total cost of goods declared by importer Rs. 350,000.00
Valuation for cost of goods by Custom Officer Rs. 400,000.00
Freight & Insurance up to custom point Rs. 17,000.00
Freight from custom point to warehouse Rs. 20,000.00
Freight from warehouse to the retailers Rs. 8,000.00
Import Duty @ 10 %, excise duty @ 5% applicable only on import.
Gross profit 10 %. 5
b) Morang Distributers (Pvt.) Ltd is a sole purchaser of VAT attractive
goods from the manufacturer. Per unit cost price of the goods is Rs.
40,000.00. The company distributes the goods to the wholesaler,
wholesaler sells the goods to the retailer. The company and the
wholesaler, each incurs transportation expenses Rs. 500.00 for the
goods and they charge 10 % profit margin to determine the selling
price. The wholesaler provides 5 % discount on selling price to the
retailer. The retailer sells the goods on the same selling price fixed by
the wholesaler. The goods are taxable as per VAT Act, 2052.
Required: 5
i) Actual cost with VAT to the final consumer.

The Institute of Chartered Accountants of Nepal 123


Suggested Answer - June 2018

ii) Total VAT payable to the government.


Ignore the VAT payable by the manufacturer.
Answers:
a)

Taxable value in case of local sales is the cash consideration received or


receivable by the supplier at the time of supply of goods after deducting trade
discounts or trade rebates, if any [Sec. 12 (1) & (2)]
Calculation of sales price and VAT amount
Total cost with duties (As per custom valuation, WN) Rs. 4,81,635.00
Less: Over valuation by custom officer Rs. 50,000.00
Actual Cost at Custom Point Rs.4,31,635.00
Other direct cost
Freight from custom point to warehouse Rs. 20,000.00
Cost of the goods s Rs. 4,51,635.00
Gross profit (4,51,635/.9-4,51,635) Rs. 50,181.67
Sales price Rs. 5,01,816.67
VAT on sales price Rs. 65,236.16
Working note
Calculation of custom valuation and duty of the imported goods
Customs valuation Rs. 4,00,000.00
Freight & Insurance up to custom point Rs. 17,000.00
Transaction value Rs.4,17,000.00
Custom Duty @ 10% Rs. 41,700.00
Value for excise duty Rs. 4,58,700.00
Excise duty @ 5% Rs. 22,935.00
Total cost with duties Rs. 4,81,635.00
b)
i. Actual cost with VAT to final consumer =Selling price + VAT (WN 3)
Rs. 50,556+6,572.28
= Rs.57,128.28
ii. Total VAT payable to government=
Payable by the company (WN1) = Rs. 650
Payable by Wholesalers (WN2) = Rs. 394
Payable by Retailers (WN 3) = Rs. 328.61
Rs. 1,372.61
Working notes:
WN 1: The company's selling price and VAT payable:
Particulars price Rs. VAT Rs.
Cost price 40,000 5,200 input
Transportation cost 500
Actual cost 40,500
Profit Margin 10%
So, selling price 40,500/.90= 45,000 5,850 output
Net Profit and VAT payable 4,500 650

The Institute of Chartered Accountants of Nepal 124


CAP II Paper 7: Income tax & VAT

WN 2: Wholesalers' selling price and VAT payable:


Particulars Price Rs. VAT Rs.
Cost price 45,000 5,850 input
Transportation cost 500
Actual cost 45,500
Profit Margin 10%
Fixed selling price 45,500/.90= 50,556
Less: 5 % discount on Rs. 50,556 2,527.80
Net selling price 48,028.20 6,244 Output
Net Profit and VAT payable 2,528.20 394
WN 3: Retailers to consumers
Fixed selling price 45,500/.90= 50,556 6,572.28 Output
Cost price (Net selling price of W.S) 48,028.20 6,244 Input
Net Profit and VAT payable 2,527.80 328.61
8. Write short notes on the basis of VAT Act, 2052: (4×2.5=10)
a) Timing of goods supplied
b) Person
c) Proportionate credit
d) VAT refund to diplomats
Answers:
a) In case of supply of goods, earliest of following shall be time of supply under
sec. 6(2):
 Date of issuance of invoice, or
 Date of possession of goods/ date of removal of goods by the recipients
from suppliers business place, or
 Date of receiving consideration by the supplier.
b) Person" means any Individual, firm, company, association, institution,
partnership firm, co-operative, joint venture, trust or fund and this term also
includes the government entity, religious organization, charitable institution or
other similar association and their branch or sub-branch thereof engaged with
or without the objective of profit, in Taxable transaction.
c) The following provisions are mentioned in sub-rules (3), (4) and (5) under
rules 40 of VAT rules, 2053.
(3) In case any taxpayer has carried on both the transactions of taxable goods
or services and tax-exempt goods or services such taxpayer may deduct only
the tax paid on purchases or imports directly related to the taxable goods or
services.
(4) In case a taxpayer carrying out the transactions of both taxable and tax
exempt goods or services fails to establish the direct relationship of the
purchased or imported goods with the taxable goods or services, such taxpayer
may deduct the amount of tax paid on his purchases or imports by calculating
the proportion of taxable transaction value out of his total sale value.
(5) While calculating tax pursuant to sub-rule (4), if the tax officer feels that it
cannot be calculated proportionally, he may seek direction from the
Department to calculate it through another alternative method.

The Institute of Chartered Accountants of Nepal 125


Suggested Answer - June 2018

d) According to Section 25(1) (ka) of the VAT Act, the tax amount paid within
Nepal by a diplomat recognized by the Ministry of Foreign Affairs of GON on
the basis of mutual recognition by a foreign nation shall be refunded to the
extent of the amount consumed in Nepal. The tax refund application should be
submitted within 3 years from the date of transaction. The VAT credit is not
provided in any purchase less than Rs. 5,000 at a time.

The Institute of Chartered Accountants of Nepal 126


CAP II Paper 7: Income tax & VAT

Specific Comments on the performance of the students


Batch: - June 2018
Level: - CAP-II
Subject: Income Tax & VAT
Question No. 1
Mostly students solved the question but only few students answered by segregating
expert for local business. Most of the students failed to compute income differently or
export and domestic sales.
Question No. 2
Mostly student unable to which items should be included and which items should be
excluded as income from investment. Most students treat interest income as taxable
although it is FWT.
Question No. 3
Most students treat all the retirement payment as taxable income and does not
segregate the R.B before Chaitra 18, 2058
Question No. 4
Students answered the questions who have concept of PI, Aug tax rate, joint
investment but only few have knowledge about what constitute related party and
implication of transaction between related parties
Question No. 5
Students have solved who had studied the IT Act.
Question No. 6
Not a single student answers this question accurately. Probably, due to custom Act
provision, students confused how to solve it.
Question No. 7
Although this question is basically arithmetical based, students are unable to solve it
properly.
Question No. 8
Students answer the question who had studied the VAT Act.

The Institute of Chartered Accountants of Nepal 127


Suggested Answer - June 2018

The End

The Institute of Chartered Accountants of Nepal 128


CAP-II, Advanced Accounting, June 2014
Suggested Answer
Roll No……………. Maximum Marks - 100
Total No. of Questions - 6 Total No. of Printed Pages - 4
Time Allowed - 3 Hours
Marks
Attempt all questions. Working notes should form part of the answer.
1. The following is the trial balance as on 31st March 2013 of Amar, Bisnhu and Chetan
who had branches at different places and who shared profits & losses in the ratio of
2:1:2 respectively,
Rs. Rs.
Dr. Cr.
st
Capitals, 1 April 2012
Amar 140,000
Bishnu 140,000
Chetan 20,000
Drawings
Amar 22,720
Bishnu 19,920
Chetan 9,160
Patents & Trade Marks 24,000
Sundry Creditors 69,800
Profit & Loss account for the year (before allowing interest @ 10%
on capital, as a charge) 25,000
Sundry Debtors-
Dhankuta 67,500
Pokhara 32,500
Kailali 15,000
Furniture-
Dhankuta 10,000
Pokhara 5,000
Kailali 6,000
Stock-
Dhankuta 46,000
Pokhara 34,000
Kailali 53,000
_____ ________
369,800 369,800
The firm was dissolved on that date. Amar took over Dhankuta Branch, and Bisnu
took over Pokhara Branch; the assets being taken at 10% less than the book values.
Patents and Trade Marks were found valueless. The business at Kailali was sold to a
limited company which allotted 6,000 equity shares of Rs. 10 each credited as fully
paid. To pay the liabilities, Amar and Bisnhu introduced cash in the profit sharing
ratio. The cost of winding up came to Rs. 4,000 for which Amar advanced cash.
Chetan is insolvent and can pay nothing. Bishnu received all the shares.
Prepare necessary accounts to close the books of the firm; partners give effect to the
correct position without having to make up losses in cash. Assume the capitals to be
fluctuating. Amar and Bishnu settles accounts themselves. 20
Answer:
Dr. Profit & Loss Account Cr.
2013 NPR 2013 NPR
March 31 To Loss for the year 25,000 March 31 By Loss transferred to
XTQ P.T.O.
(2)

To Interest on Capitals Amar's Capital 22,000


Amar's Capital 14,000 Bishnu's Capital 11,000
Bishnu's Capital 14,000 Chetan's Capital 22,000 55,000
Chetan's Capital 2,000 30,000

55,000 55,000

Dr. Realization Account Cr.


2013 NPR 2013 NPR
March 31 To Patents & Trade Marks 24,000 March 31 By Amar's Capital-
To Sundry Debtors 115,000 Debtors at Dhankuta 67,500
To Furniture 21,000 Furniture at Dhanluta 10,000
To Stock 133,000 Stock at Dhankuta 46,000
To Cash (Expenses) 4,000 Total 123,500
Less : 10% 12,350 111,150
By Bishnu's Capital
Debtors at Pokhara 32,500
Furniture at Pokhara 5,000
Stock at Pokhara 34,000
Total 71,500
Less : 10% 7,150 64,350

By Shares in New Company Ltd 60,000

By Loss transferred to:


Amar's Capital 24,600
Bishnu's Capital 12,300
Chetan's Capital 24,600 61,500

297,000 297,000

Dr. Sundry Creditors Account Cr.


2013 NPR 2013 NPR
March 31 To Cash 69,800 March 31 By Balance b/fd 69,800

69,800 69,800

Dr. Cash Account Cr.


2013 NPR 2013 NPR
March 31 To Amar's Capital A/C 46,533 March 31 By Realization A/c 4,000
(Expenses) 69,800
To Bishnu's Capital A/C 23,267 By Sundry Creditors
(Cash advance to pay
creditors)
To Amar's Capital A/C 4,000
(Cash paid for expenses)

73,800 73,800

Dr. Shares in New Company Ltd. Cr.


2013 NPR 2013 NPR
March 31 To Realization A/c 60,000 March 31 By Bishnu's Capital A/c 60,000

60,000 60,000

XTQ
(3)

Dr. Capital Account


Cr.
Particulars Amar Bishnu Cheta Particulars Amar Bishnu Chetan
n

To Drawings 22,720 19,920 9,160 By Balance b/d 140,000 140,000 20,000


To Profit & Loss A/c 22,000 11,000 22,000 By Interest on Capital 14,000 14,000 2,000
To Balance c/d 109,280 123,080 - By Balance c/d - - 9,160

154,000 154,000 31,160 154,000 154,000 31,160

To Balance b/d 9,160 By Balance b/d 109,280 123,080


To Realization A/c 111,150 64,350 By Cash 46,533 23,267
(Assets taken over) 24,600 12,300 24,600 By Cash Expenses 4,000
To Realization (loss) By Amar's Capital A/c 15,880
To Chetan's Capital A/c 15,880 17,880 By Bishnu Capital A/c 17,880
To Shares in New Co. 60,000 By Bishnu's Capital A/c 8,183
Ltd. (Cash paid to Amar)
To Amar's Capital ( Cash 8,183
received from Bishnu)
159,813 154,530 33,760 159,813 154,530 33,760

Note: Chetan's loss has been divided between Amar and Bishnu in the ratio of 109,280 : 123,080
respectively.

2.
a) M/s Icon Limited gives you the following information to find out total sales and
total purchases:
Rs.
Debtors as on 1.4.2069 157,500
Creditors as on 1.4.2069 182,250
Bills Receivables received during the year 105,750
Bills Payable issued during the year 119,250
Cash received from customers 351,000
Cash paid to suppliers 387,000
Bad debts recovered 36,000
Bills receivable endorsed to creditors 60,750
Bills receivable dishonoured by customers 11,250
Discount allowed by suppliers 15,750
Discount allowed to customers 20,250
Endorsed Bills Receivables dishonoured 6,750
Sales Return 24,750
Bills Receivable discounted 18,000
Discounted Bills Receivable dishonoured 4,500
Cash sales 379,125
Cash Purchased 445,050
Debtors as on 31.3.2070 184,500
Creditors as on 31.3.2070 213,750
10
th
b) A fire occurred in the premises of M/s Careless Co. on 30 Mangsir 2070. From
the following particulars, relating to the period 1st Shrawan 2070 to 30th Mangsir
2070. You are required to ascertain the amount of claim to be filed with the
insurance company for the loss of stock.
Rs.
st
i) Stock as per Balance Sheet as at 31 Ashadh 2070 99,000
ii) Purchases (including purchase of a machinery
Costing Rs. 30,000) 170,000
XTQ
(4)

iii) Wages (including wages for the installation


of Machinery Rs. 3,000) 50,000
iv) Sales (including goods sold on approval basis amounting
to Rs. 49,500. No confirmation had been received in
respect of two-thirds of such Goods sold
on approval basis. 275,000
v) Sales value of goods drawn by proprietor 15,000
vi) Cost of goods sent to consignee on 15 Mangsir 2070 lying unsold
With them 16,500
vii) Sales value of goods distributed as free samples 1,500
The average rate of gross profit had been 20% in the past. This selling price had
been increased by 20% with effect from 1st Shrawan 2070.
For valuing the stocks for the Balance Sheet as at 31st Ashadh 2070, Rs. 1,000 had
been written off in respect of a slow moving item, the cost of which was Rs.
5,000. A portion of those goods were sold at a loss of Rs. 500 on the original cost
of Rs. 2,500. The remainder of the stock was now estimated to be worth the
original cost.
Subject to the above exceptions, the gross profit had remained at a uniform rate
throughout. The value of goods salvaged was estimated at Rs. 25,000. The
enterprise had taken an insurance policy for Rs. 60,000 which was subject to the
average clause. 10
Answer:
a)

1. Total sales = Cash sales + Credit sales(W.N.1)


= Rs. 3,79,125+ Rs. 5,06,250
= Rs. 8,85,375
2. Purchases = Cash Purchases + Credit Purchases
= Rs.4,45,050 + 6,07,500 (W.N.2)
= Rs. 10,52,550
Working Notes:

1 Debtors Account
Particulars Rs. Particulars Rs.
To Balance B/d 157500 By Bills Receivable 105750
To Bills Receivable
dishonoured 11250 By Cash 351000
To Bills Receivable
dishonoured
(endorsed) 6750 By Discount allowed 20250
To Bills Receivable
dishonoured
(discounted) 4500 By sales return 24750
To Credit sales (bal.fig) 506250 By Balance C/d 184500
686250 686250

2 Creditors Account
Particulars Rs. Particulars Rs.
To Bills Payable 119250 By Balance B/d 182250
By Bill Receivable
To Cash 387000 dishonoured (endorsed) 6750
By Credit Purcahses
To Discount Received 15750 (bal.fig) 607500
XTQ
(5)

To Bills Receivable
endorsed 60750
To Balance C/d 213750 ______
796500 796500

b)
Memorandum Trading Account for the period from 1.4.2070 to 30.8.2070
Particulars Normal Abnormal Particulars Normal Abnormal
Rs. Rs. Rs. Rs.
Opening Stock (W.N.1) 95000 5000 Sales (W.N. 2&1) 240000 2000
Stock lying with
customers on
Purchases 170000 approval (W.N.3) 22000
Less Machinery Goods sent to
purchase 30000 140000 consignee 16500
Goods drawn by
Wages 50000 proprietor 10000
Less: wages for instl.
of machinery 3000 47000 Free samples (W.N. 5) 1000
Gross Profit (33.33% of
sales)
(W.N. 4) 80000 Loss (W.N.) 500
Stock on the date of
_________ _________ fire 72500 2500
362000 5000 362000 5000

Working Notes
(1) Abnormal items: Rs.
Original cost of slow moving items 5000
cost of slow moving item sold 2500
Balance of slow moving items in stock 2500
Original cost of slow moving items sold 2500
Loss incurred on such sale 500
Sale proceeds of slow moving items 2000

(2) Sales as given 275000


Less: goods sold on approval basis 49500
225500
Add: sales confirmed with respect

to goods sold on approval basis


/' 3 x 49,500 16500
Total sales for the period 242000
Less: sale of abnormal items 2000
Normal sales 240000

(3) Goods sold on approval basis 49500


Less: sales confirmed 16500

Goods with customers on approval basis


-sales not being confirmed 33000
Less: profit element 1/3 of Rs. 33,000 11000
Goods on approval at cost 22000

(4) Previous year G.P. margin on sales = 20%


Cost 100-20 = 80
XTQ
(6)

Selling price in the current year 100+20 120


G.P Margin in current year 40/120 x 100 33.33%

(5) Sales value of free samples distributed 1500


Less: Profit on such sale value @ 33.33% 500
Cost of sample distribution 1000

Statement of claim Rs.


Stock on the date of fire
Normal items 72500
Slow moving items 2500
75000
Less Salvage 25000
50000
Claim (applying average clause
Value of Policy/Stock on the fire date x loss of stock
= 60000/75000 x 50000 =Rs. 40000

3.
a) Abhinav Enterprises has a retail branch at Pokhara. Goods are sold to the
customers at Cost plus 100%. The wholesale price is cost plus 80%. Goods are
invoiced to Pokhara at wholesale price. From the following particulars, find out
the profit made at Head office and Pokhara for the year ended 31st Ashadh 2070 :

Head Office Pokhara


Rs. Rs.
st
Stock as on 1 Shrawan 2069 225,000 -
Purchases 2,150,000 -
Goods sent to branch at invoice price 954,000 -
Sales 2,853,000 950,000
st
Stock as on 31 Ashadh 2070 260,000 99,000
Sales at Head Office are made only on wholesale basis and that at Branch only to
consumers. Stock at branch is valued at invoice price. 3
b) Arial Constructions Ltd issues 80,000 equity shares of Rs. 10 each at a premium
of 25%, Rs. 4 per share being payable along with application and the balance
including premium being payable on allotment. Applications total 91,000 shares
out of which applications for 11,000 shares are rejected while all other application
are fully accepted. Allotment money on 200 shares is not received. After due
notices have been served, the Board of directors decided to forfeit the shares.
Later all these shares are issued as fully paid up @ Rs. 9 per share.
Pass necessary journal entries for the above transaction crediting Share Premium
Account only when the amount of the premium has been received. 7
c) BN Bank Limited provides you the following information regarding Loan Loss
provisioning as on 31st Ashadh 2069:
Category Amount Rs.
1. Good 5000,000
2. Rescheduled /Restructured 210,000
3. Substandard 500,000
4. Doubtful 300,000
5. Bad 500,000

XTQ
(7)

During financial year 2069-70 additional loans amounting to Rs. 3,000,000 were
disbursed. The Bad loans amounting to Rs. 200,000 was written off during the
year. Loans amounting to Rs. 150,000 were shifted from Doubtful category to
Bad category. Similarly, Loans amounting to Rs. 500,000 shifted from Good
category to substandard category and Substandard Loans amounting to Rs.
200,000 were rescheduled during the year.
From the above information, you are required to calculate the loan loss provision
and pass necessary journal entries in the books of BN Bank Limited as per the
directive issued by Nepal Rastra Bank and find the percentage of total Non
Performing assets. 8
d) A and B are partners sharing profits and losses in the ratio 5:3. C is admitted as a
partner getting 3/10 of the profit share. If C acquires 4/20 of his share from A and
2/20 from B, what would be the new profit sharing ratio? 2
Answer:
Question 3(a) Solution
In the books of Abhinav Enterprises.
Trading and Profit and Loss Account
Head Pokhara Head Pokhara
Office Office
To opening stock 225,000 - By Sales 28,53,000 9,50,000
To Purchases 21,50,000 - By Goods sent to branch 9,54,000 -
To Goods received - 9,54,000 By Closing stock 2,60,000 99,000
from HO
To Profit and Loss A/c 16,92,000 95,000
(Gross Profit)

40,67,000 10,49,000 40,67,000 10,49,000


To Branch stock 44,000 By Gross Profit c/d 16,92,000 95,000
reserve
(99,000X80/180)
To Net Profit 16,48,000 95,000
(Before expenses)

Question 3(b) Solution


Journal Entries

Bank A/c Dr. 3,64,000


To Equity share application A/c 3,64,000
(Application money received for 91,000 equity shares @Rs. 4
per share)

Equity Share Application A/c Dr. 3,20,000


Equity Share Allotment A/c Dr. 4,80,000
To Equity Share Capital A/c 8,00,000
(Credit to equity share application a/c for application money
@Rs.4 per share and that part of allotment money which is
payable towards the capital i.e @Rs.6 per share on allotment of
80,000 equity shares of Rs.10 each at a premium of 25%.)

Equity share application A/c Dr. 44,000


To Bank A/c 44,000
(Return of application money to application for 11,000 shares
because of rejection of their application.)

XTQ
(8)

Bank A/c Dr. 6,78,300


To Equity Share allotment A/c 4,78,800
To Share premium A/c 1,99,500
(Receipt of allotment money on 79,800 shares @Rs.8.50 per
share, Rs. 6 per share towards Share capital and Rs.2.5 as share
premium.)

Equity Share Capital A/c 2,000


To Equity Share Allotment A/c 1,200
To Forfeited Shares A/c 800
(Forfeiture of 200 equity shares on which application money @
Rs.4 per share has been received for non-payment of allotment
money.)

Bank A/c Dr. 1,800


Forfeited Shares A/c Dr. 700
To Equity Share Capital A/c 2,000
To Share Premium A/c 500
(Being reissue of 200 equity shares as fully paid up @Rs.9 per
share)

Forfeited Shares A/c Dr. 100


To Capital Reserve A/c 100
(Transfer of balance of Forfeited Shares A/c to Capital Reserve
on reissue of all forfeited shares.)

Question 3(c) Solution


(Rs.)
Calculation of Loan Loss Provisioning
Provision
as on
Loan 31st Amount
Amount 31st Ashad Additional 31st Ashad Rate of Total
Category Ashad 2069 2069 Loans Shifting 2070 Provision Provision

1. Good 5,000,000 50,000 3,000,000 (500,000) 7,500,000 0.01 75,000


2. Rescheduled
/Restructured 210,000 26,250 200,000 410,000 0.125 51,250

3. Substandard 500,000 125,000 300,000 800,000 0.25 200,000

4. Doubtful 300,000 150,000 (150,000) 150,000 0.5 75,000

5. Bad 500,000 500,000 (50,000) 450,000 1 450,000

Total 6,510,000 851,250 9,310,000 851,250

Total NPA 1,400,000


NPA/Total
Loans 15.038

Journal Entries
In the books of BN Bank Limited

XTQ
(9)

Loans and Advances Dr.


To Cash A/c 30,00,000
(Being Loans given during the year. Assumed that all the loans 30,00,000
are taken in cash)
Profit and Loss A/c Dr.
Restructured Loans Dr. 2,00,000
Substandard Loans Dr. 2,00,000
To Good Loans 3,00,000
To Doubtful Loans 5,00,000
To Bad loans 1,50,000
(Being changes of in various categories of loans and advances 50,000
during the year.)
Profit and Loss A/c Dr.
To LLP Good Loans 1,25,000
To LLP Restructured Loans 25,000
To LLP Substandard 25,000
(Being changes of in various categories of loans loss 75,000
provisioning during the year.)
LLP Doubtful Loans Dr.
LLP Bad Loans Dr. 75,000
To Profit and Loss A/c 50,000
(Being changes of in various categories of loans and advances 1,25,000
during the year.)
Note:
Though no additional entry is required for the amount of LLP as the amount of LLP last year and this
year is same but individual category wise LLP has changed thus requiring the above journal entries.

Question 3(d) Solution


Calculation of new profit sharing ratio
A: 5/8-4/20 = 25-8/40 =17/40
B: 3/8-2/20 = 15-4/40 =11/40
C: 3/10 or 12/40 Thus new profit sharing ratio would be 17:11:12

4.
a) On 1st Shrawan 2069, Mr. Pradhan has 100,000 equity shares of ABC Bank Ltd.
at a book value of Rs. 150 per share (face value Rs. 100 each). Further
information have been provided to you.
i) On 20th Ashwin 2069, he purchased another 20,000 shares of ABC Bank Ltd
at Rs. 160 per share.
ii) 1st Mangsir 2069, ABC Bank Ltd. issued one equity bonus share for every six
shares held by the shareholders.
iii) 29th Magh, 2069, the directors of ABC Bank Ltd. announced a right issue
which entitles the holders to subscribe three shares for every seven shares at
Rs. 150 per share. Shareholders can transfer their rights in full or in part.
Mr. Pradhan sold '/3rd entitlement to Mr. Sharma for a consideration of Rs. 20 per
share and subscribe the rest on 5th Fagun 2069.
You are required to prepare Investment A/c in the books of Mr. Pradhan for the
year ending 31st Ashadh 2070. 5
b) M/s Biotic Company Limited obtained a loan for Rs. 14 crores on Shrawan 15,
2069 from Nepal Bank Limited, to be utilized as under:
Construction of Factory building Rs. 2.5 crores

XTQ
(10)

Purchase of Plant and Equipment Rs. 2.0 crores


Working Capital Rs'. 1.5 crores
Advance for purchase of trucks Rs. 1.0 crore
In Ashadh 2070, construction of the factory building was completed and Plant and
Equipments, which was ready for its intended use, was installed. Delivery of
trucks was received in the next FY. Total interest of Rs. 9,100,000 was charged
by the bank for the financial year ending 31.3.2070.
Show the treatment of interest under NAS 08 ―Borrowing Cost‖ and also explain
the nature of assets. 5
st
c) A Company purchased fixed assets costing Rs. 50,88 lakhs on 1 Shrawan 2069
and the same was fully financed by foreign currency loan in U.S. Dollars,
repayable in four equal annual installments. Exchange rate at the time of purchase
was 1 US Dollar Rs. 84.80. The first installment was paid on 31 Asadh 2070
when 1 US Dollar 90.80. The entire loss on exchange was included in cost of
goods sold of normal business operations. A Company provides depreciation on
their fixed assets at 20% on WDV basis. Show the correct accounting treatment
with reference to relevant accounting standards. 5
Answer:
a)
In the books of Mr. Pradhan
Investment Account
(Equity share in ABC Bank Ltd.)
No. of Amount No. of Amount
Date Particulars Shares Rs. Date Particulars Shares Rs.
By Bank A/c
(sale of right)
1.4.069 To Balance B/d 100000 15000000 5.11.069 (W.N.3) 400000
By Balance C/d
20.6.069 To Bank A/c 20000 3200000 31.32070 (Bal.Fig) 180000 23800000
To Bonus issue
1.8.069 (W.N.1) 20000 0
To Bank
A/c(Right
shares)
5.11.069 (W.N. 4) 40000 6000000 ______ ________
180000 24200000 180000 24200000

Working Notes
1) Bonus shares = (100000+20000)/6 20000
2) Right shares = (100000+20000+20000)/7 x 3 60000
3) Sale of right = 60000 x 1/3 x Rs. 20 400000
4) Right subscribed = 40000 shares x Rs. 150 6000000

b) .
(a) Treatment of Interest (Borrowing cost) as per NAS 08 ‗Borrowing Cost‘
S.No Particulars Nature Interest to be Interest to be
. capitalized charged to P/L
A/c
Rs. Rs.
(i) Construction of Qualifying 91,00,000 x 25/70
Factory Building Assets =Rs.32,50,000
(Refer Note 1)
(ii) Purchase of Plant and Non Qualifying 91,00,000 x
Equipment Assets 20/70

XTQ
(11)

=Rs.26,00,000
(iii) Working Capital Non Qualifying 91,00,000 x
Assets 15/70
=Rs.19,50,000
(iv) Advance for purchase Non Qualifying 91,00,000 x
of Trucks Assets 10/70
=Rs.13,00,000
Total Rs. 32,50,000 Rs. 58,50,000

Notes:
1. It is assumed that construction of a factory building was completed on 31st Asadh, 2070
2. It is assumed that the Plant and Equipment was ready for its intended use at the time of its
acquisition.
As per NAS 08, assets have been defined as ‗qualifying asset‘ and non-qualifying assets.
(i) Qualifying assets is an asset that necessarily takes a substantial period of time to get ready
for its intended used or sale; whereas
(ii) Non-qualifying asset is an asset which is ready for its intended use or sale at the time of its
acquisition.
The Loan at one place is 14 crore but in detail is 7 crore. Hence, there are 2 options:
i) As given in Answer
ii) 6.5%
c) As per NAS 11 ‗The Effects of changes in Foreign Exchange Rates‘, foreign currency non-
monetary items which are carried in terms of historical cost denominated in a foreign currency
should be reported using the exchange rate at the date of the transaction and exchange
differences arising on the settlement of monetary items or on reporting an enterprise‘s monetary
items at rates different from those at which they were initially recorded during the period, or
reported in previous financial statements, should be recognized as income or as expense in the
period in which they arise. Foreign currency loss will be computed as Rs. 360 lakhs {$60 lakhs
[5088 Lakhs/84.80] x (Rs. 90.80-Rs. 84.80)}. The entire loss on exchange difference of Rs. 360
lakhs should be recognized as an expense for the year ended 31st Asadh 2070 and should not be
included in the cost of goods sold. Depreciation on fixed assets amounting Rs. 1017.60 lakhs
(20% of 5088 lakhs) should be provided for in the financial statements for the year ended 31st
Asadh 2070.

5.
a) An amount of Rs. 3,465,000 was incurred on a contract work upto 31.03.2070.
Certificates have been received to date to the value of Rs. 4,200,000 against
which Rs. 3,780,000 has been received in cash. The cost of work done but not
certified amounting to Rs. 78,750. It is estimated that by spending an additional
amount of Rs. 210,000 (including provision for contingencies) the work can be
completed in all respects in another two months. The agreed contract price of
work is Rs. 4,375,000. Compute a conservative estimate of the profit to be taken
to the Profit and Loss Account as per NAS 13. 5
b) Discuss the treatment of upward and downward revaluation of assets as per NAS
6. 5
c) Krishna Infrastructures Limited (KIL) is engaged in carrying out the construction
activities of hydropower projects and sanitation projects in Joint venture with
other private construction companies. Recently, KIL entered into an agreement
under its leadership and management to lay the pipelines in Hetauda Industrial
XTQ
(12)

Area with Premier constructions Pvt. Ltd., Raktakali Nirman Sewa and Pukar
Nirman and Constructions Limited. KIL wants to close the books of accounts as
on 31st Ashadh 2070. The management of the company seeks your suggestion as
to whether Premier constructions Pvt. Ltd., Raktakali Nirman Sewa and Pukar
Nirman and Constructions Limited are to be disclosed as Related Party in the
financial statements or not. Advice the management taking into account relevant
pronouncements of Nepal Accounting Standard. 5
Answer:
a)
Computation of estimate of profit as per NAS 13
Rs.
Expenditure incurred upto 31.3.2070 3465000
Estimated additional expenses (including provision for
contingency) 210000
Estimated Cost (A) 3675000
Contract Price (B) 4375000
Total estimated profit (B-A) 700000
Percentage of completion (34,65,000/36,75,000) x 100 94.29
Computation of estimate of the profit to be taken to Profit and Loss Account
Total estimated profit x Expenses incurred till 31.3.2070/Total estimated cost
700,000 x 34,65,000/36,75,000 = Rs. 660,000

According to para 22 of NAS 13 ‗Construction Contract‘, when the outcomes of a


construction contract can be estimated reliably, contract revenue and contract
costs associated with the construction contract should be recognized as revenue
and expenses respectively by reference to state of completion of the contract
activity at the reporting date. Thus estimated profit amounting Rs. 700,000 should
be recognized as revenue in the statement of profit and loss.

b) As per NAS 16 if an asset‘s carrying amount is increased as a result of a


revaluation, the increase shall be recognised in other comprehensive income and
accumulated in equity under the heading of revaluation surplus. However, the
increase shall be recognised in profit or loss to the extent that it reverses a
revaluation decrease of the same asset previously recognised in profit or loss.
If an asset‘s carrying amount is decreased as a result of a revaluation, the decrease shall be
recognised in profit or loss. However, the decrease shall be recognised in other comprehensive
income to the extent of any credit balance existing in the revaluation surplus in respect of that
asset. The decrease recognised in other comprehensive income reduces the amount accumulated
in equity under the heading of revaluation surplus.

c) According to Nepal Accounting Standard 16 relating to Related Party, Related


party, a party is related to an entity if:
(a) directly, or indirectly through one or more intermediaries, the party:
(i) controls, is controlled by, or is under common control with, the entity (this includes
parents, subsidiaries and fellow subsidiaries);
(ii) has an interest in the entity that gives it significant influence over the entity; or
(iii) has joint control over the entity;
In this case, KIL entered into an agreement to lay the pipelines in Hetauda Industrial Area with
Premier constructions Pvt. Ltd., Raktakali Nirman Sewa and Pukar Nirman and Constructions
Limited. KIL has the leadership as well as management to control the project and the other 3
entities are just the intermediaries of KIL. In case of any other projects there may be different
types of intermediaries.

XTQ
(13)

However, it should be noted that the intermediaries are not such that these three entities are the
subsidiaries of KIL. In considering each possible related party relationship, attention is directed to
the substance of the relationship and not merely the legal form.
In the context of this Standard, the parties need not necessarily be related only because they share
joint control over a joint venture. Thus, there may not necessarily be related party relationship
between KIL and other three entities.

6. Write short notes on: (4×2.5=10)


a) Qualitative characteristics of the financial statements
b) Recognition of expenses
c) Provisions and other liabilities
d) Initial recognition of finance lease in the financial statement of Lessee.
Answer:
a) The qualitative characteristics are attributes that improve the usefulness of
information provided in financial statements. The framework suggests that the
financial statements should observe and maintain the following four qualitative
characteristics as far as possible within limits of reasonable cost/ benefit.
1. Understandability: The financial statements should present information in a manner as to
be readily understandable by the users with reasonable knowledge of business and
economic activities. It is not right to think that more disclosures are always better. A mass
of irrelevant information creates confusion and can be even more harmful than non-
disclosure. No relevant information can be however withheld on the grounds of
complexity.
2. Relevance: The financial statements should contain relevant information only.
Information, which is likely to influence the economic decisions by the users, is said to be
relevant. Such information may help the users to evaluate past, present or future events or
may help in confirming or correcting past evaluations. The relevance of a piece of
information should be judged by its materiality. A piece of information is said to be
material if its omission or misstatement can influence economic decisions of a user.
3. Reliability: To be useful, the information must be reliable; that is to say, they must be free
from material error and bias. The information provided are not likely to be reliable unless:
(a) Transactions and events reported are faithfully represented.
(b) Transactions and events are reported in terms of their substance and economic reality
not merely on the basis of their legal form. This principle is called the principle of
substance over form'.
(c) The reporting of transactions and events are neutral, i.e. free from bias.
(d) Prudence is exercised in reporting uncertain outcome of transactions or events.
4. Comparability: Comparison of financial statements is one of the most frequently used and
most effective tools of financial analysis. The financial statements should permit both
inter-firm and intra-firm comparison. One essential requirement of comparability is
disclosure of financial effect of change in accounting policies.
5. True and Fair View: Financial statements are required to show a true and fair view of the
performance, financial position and cash flows of an enterprise. The conceptual
framework does not deal directly with this concept of true and fair view, yet the
application of the principal qualitative characteristics and of appropriate accounting
standards normally results in financial statements portraying true and fair view of
information about an enterprise.
b) Recognition of expenses
Expenses are recognized in the income statement when a decrease in future economic
benefit related to a decrease in an asset or an increase of liability has arisen that can be
measured reliably. The recognition of expenses occurs simultaneously with the
recognition of an increase in liabilities or a decrease in assets.

XTQ
(14)

Expenses are recognized in income statement on the basis of a direct association between
the costs incurred and the earning of specific items of income. This process, commonly
referred to as the matching of costs with revenues, involves the simultaneous or combined
recognition of revenue and expenses that result directly and jointly from the same
transactions or other events.
When economic benefits are expected to arise over several accounting periods and the
association with income can only be broadly or indirectly determined, expenses are
recognized in the income statement on the basis of systematic and rational allocation
procedures.
An expense is recognized immediately in the income statement when an expenditure
produces no future economic benefits or when, and to the extent that, future economic
benefit do not qualify, or cease to qualify.

c) Provisions and other liabilities


Provision can be distinguished from other liabilities such as trade payable and accruals because
there is uncertainty about the timing or amount of future expenditure required in settlement. By
contrast:
(a) Trade payables are liabilities to pay for goods or services that have been received or
supplied and have been invoiced or formally agreed with the supplier; and
(b) Accruals are ;liabilities to pay for goods or services that have been received or supplied
but not have been paid, invoiced or formally agreed with the supplier, including amounts due to
employees. Although it is sometimes necessary to estimate the amount or timing of accruals, the
uncertainty is generally much less than for provision.
Accruals are often reported as part of trade and other payables, where as provisions are reported
separately.
d) Initial recognition of finance lease in the financial statement of Lessee.
minimum lease payments, each determined at the inception of the lease. The discount
rate to be used in calculating the present value of the minimum lease payments is the
interest rate implicit in the lease, if practicable to determine; if not, the lessee‘s
incremental borrowing rate shall be used. Any initial direct cost of the lessee are
added to the amount as an asset.

XTQ
(15)

CAP-II, Audit & Assurance, Jun-14


Suggested Answer

Roll No……………. Maximum Marks - 100

Total No. of Questions- 7 Total No. of Printed Pages- 10

Time Allowed - 3 Hours


Marks

Attempt all the questions.

1. As an auditor, give your opinions with reasons on the following cases: (45=20)
a) Satya Limited is company listed in Recognized Stock Exchange of Nepal. During
normal course of operation, a fire broke out on 15th Ashoj, 2070, in which material
worth 50 lakhs which was lying in stock since 1st Ashadh, 2070 was totally destroyed.
The financial statements of the company have not been adopted till the date of fire.
The management of the company argues that since the loss occurred in the year, 2070-
71, no provision for the loss needs to be made in the financial statements for 2069-70.
State whether argument of management is in accordance with applicable provision.
b) Dhaulagiri Ltd. is an associate of Sagarmatha Ltd. How do you ensure whether
Sagarmatha Ltd. has significance influence in Dhaulagiri Ltd. or otherwise?
c) Commission for the Investigation of Abuse of Authority (CIAA) has asked certain
information from XYZ & Co.; Chartered Accountants relating to certain client of the
audit firm for which legal proceeding is in process with CIAA. The XYZ & Co.
refused to provide such information to CIAA. Is the action of XYZ & Co. is tenable?
d) While conducting the audit of X-ray Limited, Mr. Pratap, the statutory auditor found
that the detection risk relating to certain transactions cannot be reduced to acceptable
level. As a Statutory Auditor, how would you deal in the given situation?

Answer:
a)
Event occurring after the balance sheet date: This case requires attention to NSA 560 ―Subsequent
Events‖ and NAS 5 ―Events After Balance Sheet Date‖. As per NAS 5 ―Events After Balance
Sheet Date‖, adjustments to assets and liabilities are required for events occurring after the balance
sheet date that provide additional information materially affecting the determination of the
amounts relating to conditions existing at the balance sheet date or that indicate that the
fundamental accounting assumption of going concern (i.e., the continuance of existence or
substratum of the enterprise) is not appropriate. NAS – 5 also requires disclosure of the non-
adjusting event, in the report of the approving authority.
Further, as per NSA 560 ―Subsequent Events‖, the auditor should assure that all events occurring
subsequent to the date of the financial statements and for which the applicable financial reporting
framework requires adjustment or disclosure have been adjusted or disclosed. The event took place
after the close of the accounting year and does not relate to conditions existing at the balance sheet
date. Thus, it will have no effect on items appearing at the balance sheet date because as per NAS
– 5 ―Contingencies and Events Occurring after Balance Sheet Date‖ have to be adjusted that
provide evidence of conditions existing as at the balance sheet date. However, the auditor has to
ensure that this loss will not materially affect the substratum of the enterprises as per its size,
nature and complexity of operations.
Thus, subject to satisfaction in respect of non-violation of going concern concept, the company has
correctly accounted by not providing provision. However, the auditor is required to ensure the
proper disclosure of abovementioned event.

XTQ
(16)

b)
As per Para 7 of NAS 25 , investment in associates ; if an investor holds, directly or indirectly
(e.g. through subsidiaries), 20 per cent or more of the voting power of the investee, it is presumed
that the investor has significant influence, unless it can be clearly demonstrated that this is not the
case. Conversely, if the investor holds, directly or indirectly (e.g. through subsidiaries), less than
20 per cent of the voting power of the investee, it is presumed that the investor does not have
significant influence, unless such influence can be clearly demonstrated. A substantial or majority
ownership by another investor does not necessarily preclude an investor from having significant
influence.

Further Para 8 of NAS 25 stated that the existence of significant influence by an investor is usually
evidenced in one or more of the following ways: (a) representation on the board of directors or
equivalent governing body of the investee;(b) participation in policy-making processes, including
participation in decisions about dividends or other distributions; (c) material transactions between
the investor and the investee; (d) interchange of managerial personnel; or (e) provision of essential
technical information.

Accordingly in the light of aforesaid provision of NAS 25, for ensuring whether Sagarmatha Ltd.
have significance influence over Dhaulagiri Ltd ; first of all i have to ensure percentage of voting
power of Sagarmatha Ltd. in Dhaulagiri Ltd. or evidencing factor as mentioned in above Para ,
existence of any one evidence is deemed to have significant influence.

c)
Section 4 of the Code of Ethics of the Institute of the Chartered Accountants of Nepal deals with
"Confidentiality" applicable too Professional Accountants in Public Practice. Section 4.1 stated
that Professional accountants have an obligation to respect the confidentiality of information about
a client‘s or employer‘s affairs acquired in the course of professional services. The duty of
confidentiality continues even after the end of the relationship between the professional accountant
and the client or employer. However Section 4.7 exemplified the points which should be
considered in determining whether confidential information may be disclosed: (a) When disclosure
is authorized. When authorization to disclose is given by the client or the employer the interests of
all the parties including those third parties whose interests might be affected should be considered.
(b) When disclosure is required by law. Examples of when a professional accountant is required by
law to disclose confidential information are: (i) to produce documents or to give evidence in the
course of legal proceedings; and (ii) to disclose to the appropriate public authorities infringements
of the law which come to light. (c) When there is a professional duty or right to disclose : (i) to
comply with technical standards and ethics requirements; such disclosure is not contrary to this
section; (ii) to protect the professional interests of a professional accountant in legal proceedings ;
(iii) to comply with the quality (or peer) review of ICAN ; and (iv) to respond to an inquiry or
investigation by ICAN or regulatory body.

In view of the above provision of Code of Ethics of the Institute of the Chartered Accountants of
Nepal ; XYZ & Co. has the professional liability for disclosing the client`s information to CIAA ,
since a professional accountant is required by law to disclose confidential information are to
produce documents or to give evidence in the course of legal proceedings .

Hence the action of XYZ & Co. is not tenable, though the audit firm may consult ICAN before
providing client`s information to CIAA.

d)
The auditor should use professional judgment to assess audit risk and to design audit procedures to
ensure that it is reduced to an acceptably low level.

―Detection risk‖ is the risk that an auditor‘s substantive procedures will not detect a misstatement
that exists in an account balance or class of transactions that could be material. The higher the
assessment of inherent and control risks, the more audit evidence the auditor should obtain from
the performance of substantive procedures. When both inherent and control risks are assessed as
high, the auditor needs to consider whether substantive procedures can provide sufficient
appropriate audit evidence to reduce detection risk, and therefore audit risk, to an acceptably low
XTQ
(17)

level. The auditor should use his professional judgments to assess audit risk and to design audit
procedures to ensure that it is reduced to an acceptably low level.

If it cannot be reduced to an acceptable level, the auditor should express a qualified opinion or a
disclaimer of opinion as may be appropriate.

2. Answer the following: (3×5=15)


a) Veri Ltd. is the Public Ltd. Company. It has not appointed Company Secretary. As an
internal auditor of the Company suggest whether the company has to appoint the
Company Secretary including for the required qualification of the Company
Secretary.
b) Auditor of Elephant Ltd. was unable to confirm the existence and valuation of stock
lying at the remote project site and accepted a certificate from the management
without obtaining other audit evidence.
c) P.G. & Company is the auditors of Professional Systems Company Ltd. The
Managing Director of the Company demands copies of the working papers from the
auditors. Are the auditors bound to oblige the Managing Director?

Answer:
a) Section 185 of the company act deals with the provision for appointment of company
secretary: Section 185 (1) states that public company with the paid –up capital of ten
million rupees or more shall appoint the company secretary a Nepalese citizen who has the
qualification mentioned in Sub-section (2) . Section (2) states that a Nepalese citizen who
has worked in the related field for at least two years after obtaining the professional
certificate of company secretary issued by a native or foreign body authorized to issue the
professional certificate of company secretary pursuant to the prevailing law or who has
worked in the related field or in the field of company management for at least three years
after doing at least bachelor degree in law, management ,commerce or economics may be
appointed to the post of company secretary. Provided, however, that this provision shall not
apply to the company secretary who is incumbent at the time of commencement of this Act
for three years after the date of commencement of this Act.

In the light of above provision of company act, I have to suggest the applicability of
aforesaid provision for Veri Ltd. if its paid up capital is Rs. 10 million or more .

b) As per NSA 580 on ― Management Representation‖ in the course of audit, an auditor comes
across various matters in respect of which he is not able to obtain sufficient appropriate
audit evidence. In such a situation he may rely on the submission by the management but he
should seek corroborative audit evidence from sources inside or outside the entity and
evaluate the representation made by management.

Management representation is not a substitute for other audit evidence. The auditor should
seek and apply normal audit procedure. Mere possession of a certificate does not absolve the
auditor from his liability. He should not seek or accept certificates when subject matter is
such that it is capable of verification from internal and/or external evidences.

In the instant case, the stock site material lying with the remote project site can be easily
verified with purchase order, invoice, bill of entry, custom document, physical verification
report from independent authority, visual means for observing stock etc.

Therefore the auditor in this instant case has not used available evidences. He should not
have rested with the certificate obtained from the management and could have evaluated
other evidences. He may be held liable for negligence and professional misjudgment. But, if
the stock is not material to the financial statement because of low value, the auditor can
accept management representation as evidence if there are no reasons to question the
authenticity of representation made by the management.
XTQ
(18)

c) As per principle of Audit Documentation, the working papers are the property of the auditor,
the auditor may, at his discretion make portion of or extracts from his working papers
available to the client. In the instant case the managing director of the company has
demanded copies of the working papers from the auditor. He has no right to obtain copies of
the working papers from the auditor because they are the property of the auditor. However
the auditor may at his discretion make portions of or extracts from the working paper to the
managing director of Professional Systems Company Ltd.

Conclusion:
The auditor is not bound to oblige the managing director by supplying copies of the audit
working papers.

3. Give your comments on the following: (35=15)


th
a) Firm of Ms. KD was appointed as auditor in 10 AGM of Neupane Limited. She was removed
by Board of Directors when she was out of Nepal on personal visit.
b) Explain briefly the technique of "Internal Control Questionnaire" to facilitate the
accumulation of information necessary for proper evaluation of internal control.
c) As an auditor, how will you verify application and allotment money received on shares issued
for cash?

Answer:
a) Section 119 (1) of the Company Act, 2063 provides that no auditor appointed pursuant to
Companies Act shall be removed pending the completion of audit of accounts of any
financial year for which he/she was appointed as the auditor.

As per Sub-section (2), notwithstanding anything contained in Sub-section (1) , if any


auditor breaches the code of conduct of auditors or does any act against the interest of the
company which has appointed him/her as the auditor or commits any act contrary to the
prevailing law, such auditor may be removed through the same process whereby he/she was
appointed as auditor, by giving prior information to the ICAN, and with the approval of the
regulatory authority, if any authorized by the prevailing law for the regulation of business of
the company concerned , and failing such authority, with the approval of the Office of
Registrar.

While removing an auditor pursuant to Sub-section (2) above, the auditor shall be provided
with a reasonable opportunity to defend him/herself.

Thus, Board of Directors cannot remove if auditor has been appointed through AGM.
Further, reasonable opportunity to defend herself should be provided.

b) Internal Control Questionnaire


Internal control questionnaire is a comprehensive series of questions concerning internal
control. It is the most widely used form for collecting information about the existence,
operation and efficiency of internal control in the organisation.

In the questionnaire, questions are generally so framed that a ‗Yes‘ answer denotes
satisfactory position and a ‗No‘ answer suggests weakness. Provision is made for an
explanation or further details of ‗No‘ answers. In respect of questions not relevant to the
business, ‗Not Applicable‘ reply is given.

XTQ
(19)

The questionnaire is usually issued to the client and the client is requested to get it filled by
the concerned executives and employees. If on a perusal of the answers, inconsistencies or
apparent incongruities are noticed, the matter is further discussed by auditor‘s staff with the
client‘s employees for a clear picture. The concerned auditor then prepares a report of
deficiencies and recommendations for improvement.

An important advantage of the questionnaire approach is that oversight or omission of


significant internal control review procedures is less likely to occur with this method. With a
proper questionnaire, all internal control evaluation can be completed at one time or in
sections. The review can more easily be made on an interim basis. The questionnaire form
also provides an orderly means of disclosing control defects. It is the general practice to
review the internal control system annually and record the review in detail.

c) Verification of application and allotment money received on Shares Issued for


Cash shall be carried out as under:
On Application
Verify the amount received along with the applications for shares in the following manner:
 Check entries in the Application and Allotment Book (or Sheets) with the original
applications;
 Check entries in the Application and the Allotment Book as regards deposits of money,
received with the applications, with those in the Cash Book;
 Vouch amounts refunded to the unsuccessful applicants with copies of Letters of Regret;
 Check the totals columns in the Application and Allotment Book and confirm the journal
entry debiting Share Application Account and crediting Share Capital Account.
On Allotment
 Examine Director‘s Minutes Book to verify approval of allotments.
 Compare copies of letters of allotment with entries in the Application and Allotment
Book.
 Trace entries in the Cash book into the Application and Allotment Book for the
verification of amounts collected on allotment.
 Trace the amount collected on application as well as those on allotment from the
Application and Allotment Book into the Share Register.
 Check totals of amounts payable on allotment and verify the journal entry debiting Share
Allotment Account and crediting Share Capital Account.

4. Answer the following: (35=15)


a) Auditors of a commercial bank was interviewed by the inspection team of Nepal Rastra Bank
and requested the auditors to share their views on the quality of loans of the bank to assist
inspection. Will this situation affect the confidentiality clause in code of ethics?
b) Explain the limitations of Internal Control.
c) What are the factors that are to be considered while designing a confirmation request?

Answer:
a) The principle of confidentiality imposes an obligation on all professional accountants to
maintain confidentiality of information acquired as a result of professional or business
relationship unless there is legal or professional right or duty to disclose. However there are
certain circumstances when confidential information can be disclosed.

As per 4.7 of the Code of Ethics of ICAN provides the circumstances where confidential
information can be disclosed as below

a. When disclosure is authorized.


XTQ
(20)

b. When disclosure is required by law.


c. When there is a professional duty or right to disclose.
i) To comply with technical standards and ethics requirements
ii) To protect the professional interests of a professional accountant in legal
proceedings;
iii) To comply with the quality (or peer) review of ACAN, and
iv) To respond to an inquiry or investigation by ICAN or regulatory body;

In view of the above requirement of 4.7 (c) (iv) auditor can assist the regulator for
inspection.

b) An Internal Control system can provide only reasonable assurance that the management‘s
objectives in establishing the system are achieved. That is, no internal control system can
provide absolute assurance that the control objectives are achieved. This is due to the fact
that any internal control system has certain internal limitations. The limitations may arise
due to:

i. Management‘s usual requirement that the cost of an internal control does not exceed
the expected benefits to be derived.
ii. Most internal controls tend to be directed at routine transactions rather than non-
routine transactions.
iii. The potential for human error due to carelessness, distraction, mistakes of judgment
and the misunderstanding of instructions.
iv. The possibility of circumvention of internal controls through the collusion of a
member of management or an employee with parties outside or inside the entity.
v. The possibility that a person responsible for exercising an internal control could abuse
that responsibility, for example, a member of management overriding an internal
control.
vi. The possibility that procedures may become inadequate due to changes in conditions,
and compliance with procedures may deteriorate. Controls has to be cost-effective.
The inherent limitation of internal control system requires the auditor to perform substantive
procedure to be able to express an opinion.

c) As per NSA -505 ―External Confirmations‖, the design of a confirmation request may
directly affect the confirmation response rate, and the reliability and the nature of the audit
evidence obtained from responses. The following factors should be considered while
designing a confirmation request:-
 The assertions being addressed.
 Specific identified risks of material misstatement, including fraud risks.
 The layout and presentation of the confirmation request.
 Prior experience on the audit or similar engagements.
 Management‘s authorization to the confirming parties to respond to the auditor.
Confirming parties may only be willing to respond to a confirmation request containing
management‘s authorization.
 The ability of the confirming party to provide the requested information

5. Comment on the following situations/statements: (35=15)


a) Auditor of Makalu Ltd. is of the opinion that ―Nepal Standards of Auditing‖ are meant only
for references and it is not mandatory to adhere such Standards.
b) Mr. A, a practicing Chartered Accountant has been found guilty in respect of Professional
Misconduct. So, Mr. A, a Chartered Accountant in practice has been suspended from practice

XTQ
(21)

for a period of 4 months. During the said period, though he did not undertake the audit
assignment since he had surrendered certificate of practice, he had appeared before Income
Tax authorities in his capacity as a Chartered Accountant.
c) Explain Principal Auditor in line with NSA 600. What are the factors to be considered by
auditor to act as Principal Auditor?

Answer:
a) Contention of the auditor is totally wrong and is against the fundamental assumptions and
guidelines governing Nepal Standards on Auditing.

NSA 200 "Objectives and General Principles Governing an Audit of Financial Statements"
stated that the auditor should conduct an audit in accordance with the Nepal Standards on
Auditing. These contain basic principles and essential procedures together with related
guidance in the form of explanatory and other material.

As per ICAN, while discharging their attest function, it will be the duty of the members of
the Institute to ensure that the NSAs are followed. The NSAs will apply whenever an
independent financial audit is carried out to express an opinion thereon.

The member of the Institute must follow the NSAs. The auditors must draw attention to the
material departures from NSAs in their audit report along with the reasons for such
departure. Auditors in their report has to mention that audit was conducted in accordance
with ―Nepal Standards on Auditing‖ in Nepalese context.

Hence the auditor is duty bound to follow the NSAs.

b) Undertaking Tax Representation Work:


A chartered accountant not holding certificate of practice cannot take up any other work
because it would amount to violation of the relevant provisions of the Chartered
Accountants Act, 2053. In case a member is suspended and is not holding Certificate of
Practice, he cannot in any other capacity take up any practice separable from his capacity to
practices as a member of the Institute. This is because once a member becomes a member of
the Institute; he is bound by the provisions of the Chartered Accountants Act, 2053 and its
Regulations. If he appears before the income tax authorities, he is only doing so in his
capacity as a chartered accountant and a member of the Institute. Having bound himself by
the said Act and its Regulations made there under, he cannot then set the Regulations at
naught by contending that even though he continues to be a member and has been punished
by suspension, he would be entitled to practice in some other capacity. Thus, in the instant
case, a chartered accountant would not be allowed to represent before the income tax
authorities for the period he remains suspended. Accordingly, in the present case he is guilty
of professional misconduct. However, if the Chartered Accountant is summoned by Income
Tax Authorities for any work conducted by the person before suspension of COP by ICAN,
the Chartered Accountant will be answerable to the authorities.

c)
As per NSA 600 Principal auditor means the auditor with responsibility for reporting on the
financial statements of an entity when those financial statements include financial
information of one or more components audited by another auditor.

The auditor should consider whether the auditor‘s own participation is sufficient to be able
to act as the principal auditor. For this purpose the principal auditor would consider:

(a) the materiality of the portion of the financial statements which the principal auditor
audits;
XTQ
(22)

(b) the principal auditor‘s degree of knowledge regarding the business of the components;
(c) the risk of material misstatements in the financial statements of thecomponents audited
by the other auditor; and
(d) the performance of additional procedures as set out in this NSA regarding the
components audited by the other auditor resulting in the principal auditor having
significant participation in such audit.

6. Write short notes on the following: (42.5=10)


a) Professional skepticism
b) Use of positive confirmations
c) Use of flow chart in evaluation of internal control
d) Cut off arrangements

Answer:
a) It is a requirement of NSA 200 that, when planning and performing an audit, the auditor
should adopt an attitude of professional skepticism. Professional skepticism is defined by
NSA 200 as an attitude that includes a questioning mind, being alert to conditions which
may indicate possible misstatement due to error or fraud, and a critical assessment of audit
evidence.

This does not mean that the auditors should disbelieve everything they are told, but they
should view what they are told with a skeptical attitude, and consider whether it appears
reasonable and whether it conflicts with any other evidence. In other words, they must not
simply believe everything management tells them.
b) A positive external confirmation request asks the respondent to reply to the auditor in all
cases either by indicating the respondent‘s agreement with the given information, or by
asking the respondent to fill in information. A response to a positive confirmation request is
ordinarily expected to provide reliable audit evidence. There is a risk; however, that a
respondent may reply to the confirmation request without verifying that the information is
correct. The auditor is not ordinarily able to detect whether this has occurred. The auditor
may reduce this risk, however, by using positive confirmation requests that do not state the
amount (or other information) on the confirmation request, but ask the respondent to fill in
the amount or furnish other information. On the other hand, use of this type of ―blank‖
confirmation request may result in lower response rates because additional effort is required
of the respondents.

b) Use of Flow Charts in evaluation of internal control: It is a graphic presentation of each part
of the company‘s system of internal control. A flow chart is considered to be the most
concise way of recording the auditor‘s review of the system. It minimizes the amount of
narrative explanation and thereby achieves a consideration or presentation not possible in
any other form. It gives bird‘s eye view of the system and the flow of transactions and
integration and in documentation, can be easily spotted and improvements can be suggested.
It is also necessary for the auditor to study the significant features of the business carried on
by the concern; the nature of its activities and various channels of goods and materials as
well as cash, both inward and outward; and also a comprehensive study of the entire process
of manufacturing, trading and administration. This will help him to understand and evaluate
the internal controls in the correct perspective.

d) Cut-off arrangements: Accounting is a continuous process because the business never comes

XTQ
(23)

to halt. It is, therefore, necessary that transactions of one period would be separated from
those in the ensuing period so that the results of the working of each period can be correctly
ascertained. The arrangement that is made for this purpose is technically known as ―cut-off
arrangement‖. It essentially forms part of the internal control system of the organization.
Accounts, other than sales, purchase and stock are not usually affected by the continuity of
the business and therefore, this arrangement is generally applied only to sales, purchase and
stock. The auditor satisfies by examination and test-checks that the cut-off procedures are
adequately followed and ensure that:
 Goods purchased, property in which passed on to the client, have in fact been included
in the inventories and that the liability has been provided for in case of credit purchase.
 Goods sold have been excluded from the inventories and credit has been taken for the
sales. If the value of sales is to be received, the concerned party has been debited.
The auditor may examine a sample of documents, evidencing the movement of stock into
and out of stores, including documents pertaining to period shortly before and after the cut-
off date and check whether stocks represented by those documents were included or
excluded as appropriate during stock taking for perfect and correct presentation in the
financial statements.

7. Distinguish between: (25=10)


a) Compliance Procedure and Substantial Procedures
b) Internal Control Questionnaire and Internal Control Evaluation
Answer:
a) Auditor should obtain sufficient and appropriate audit evidences and test them
before framing an opinion about the assertions the financial statements reveal. For
this, the auditor checks evidences through
 Compliance procedure and
 Substantial procedure.

Compliance procedures are tests designed to obtain reasonable assurance that those internal
control on which audit reliance is to be placed are in effect. It seeks to test that
 there exists internal control,
 the existing internal control is effective and
 the internal control is working without break or lacunae during the period under review.

When internal control is found to be to an acceptable level, the accounting entries generated in
such a system is more reliable than in one where the control is weak.
Mere satisfaction about the existence of internal control may not be sufficient for auditors to
express opinion about the assertions the financial data in the form of balances and transactions.

These i.e. transactions and balances need to be tested. This is done by audit procedure called
substantive checking. Substantive procedures are designed to obtain audit evidence as to the
completeness, accuracy and validity of the data produced by the accounting system.

The substantive procedures involve


 checking of transactions and balances and
 analytical review.

The checking of transaction and balances involves vouching of sales, purchases, payments,
receipts and scrutiny of ledgers. The analytical procedure involves critically examining the
accounts in an overall manner and it may entail computation of ratios, trend analysis so as to
dwell in length for examination of unusual or unexplained deviations.

XTQ
(24)

b) Internal Control Questionnaire (ICQ) and Internal Control Evaluation (ICE) : The internal
control questionnaires show the area where weakness occur or likely to occur. They do not
give any idea of the importance of those weaknesses. The Internal Control Evaluation brings
to light importance of those weakness disclosed by ICQ. Main points of distinctions are:

(i) ICQ incorporates a large number of detailed questions but does not attempt
to distinguish their relation in materiality. ICE isolates the main control
objectives within the area of review.

(ii) Weaknesses are highlighted by answer ―Yes‖ on ICE compared with ‗No‘ on
ICQ.

(iii) Answer ‗no‘ in ICQ indicates a weakness real or potential, but its significance
is not revealed. Whereas ICE requires audit personnel to state whether, an apparent
weakness may prove to be material in relation to the accounts as a whole.

(iv) The ‗Control Checklist‘ in ICE is more than a summary of key control
factors, and is no substitute for ICQ.

XTQ
(25)

CAP-II, Corporate & Other Law, Jun-14


Suggested Answer

Total No. of Questions - 7 Maximum Marks - 100

Time Allowed - 3 Hours


Marks
Attempt all questions.

Part: "A"
1. Answer the following questions:
a) The directors of a company borrowed a sum of money from a bank. The
company‘s articles provided that the directors might borrow on bonds such
sums as may from time to time be authorized by a resolution passed at a
general meeting of the company. There was no such resolution passed at a
general meeting of the company authorizing the directors to borrow loan.
When the company failed to repay the amount the bank filed a case. The
shareholders claimed that there had been no such resolution authorizing the
loan and, therefore, it was taken without their authority. Decide whether the
company bound to repay the loan to the bank. 5
b) The memorandum of association of a public company was signed by two
adult persons and by a guardian of the other five members, who were minors
at the time. The guardian made separate signature for each of the minors by
himself. The Registrar, however, registered the company and issued under
his hand a certificate of incorporation. When the minors knew the facts, it
was objected that the memorandum of association had not been signed by
the guardian of all the minors and that the provisions of the Act had not been
complied with. Decide the following issues in reference with the Companies
Act, 1956:
i) What is the legality of the certificate of incorporation? 2.5
ii) Can the minors or their guardian have right to make objection on the
ground that the provisions of the Act had not been complied with? 2.5
c) Who is a holder in due course? State his privileges in reference to the
Negotiable Instrument Act, 1881. (3+5=8)
d) Some of the shareholders were carried on a competing business with the
company's business. The majority of the shares were held by the directors,
who passed a special resolution altering the company's articles and
introducing a power for the directors to require any shareholders who
competed with the company's business to transfer their shares at their full
value to the nominees of the directors. Some of the shareholders who carried
on a competing business with the company's business file a case challenging
the validity of the resolution passed by the directors in majority. Give your
opinion on the legality of the special resolution passed by the majority of the
shareholders. 7

Answer:
1.a) This issue is related to the doctrine of indoor management which is opposed to that of the rule of
constructive notice. Constructive notice seeks to protect the company against the outsider the doctrine
of indoor management operates to protect outsiders against the company. The rule of constructive
notice is confined to the external position of the company and therefore, it follows that there is no
notice as to how the company‘s internal machinery is handled by its officers. If the contract is
consistent with the public documents, the person contracting will not be prejudiced by irregularities that
may beset the indoor working of the company. This implies that the outsiders are not bound to inquire
XTQ
(26)

whether the act of the directors which is related to internal management, had been properly and
regularly performed. Even where the directors exceed their powers or infringe the restrictions imposed
upon them, the company may be bound, for an outsider dealing with the company. This rule was
recognized in the case of Royal British Bank v. Tarquand: (1856) 6 E.&B 327.
Following this rule of Tarquand, it is thus stated that ―if the directors have power and authority to bind
the company, but certain preliminaries are required to be gone through on the part of the company
before that power can be duly exercised, then the person contracting with the directors is not bound to
see that all these preliminaries have been observed. He is entitled to presume that the directors are
acting lawfully in what they do. Thus, where the directors of a company having the power to allot share
only with the consent of the general meeting, allotted them without any such consent, the company was
held liable.
In this case, the company is bound to repay the loan. Once it was found that the directors could borrow
subject to a resolution, the plaintiff had the right to infer that the necessary resolution must have been
passed. When third party has dealt with the company through the board of directors which is not ultra-
vires to the Articles and Memorandum of the company, the company cannot avoid liability to pay the
loan to the bank on the ground that such resolution never been passed at the general meeting of the
company.
1 b) i) This issue is related to the certificate of incorporation. The certificate of incorporation brings the
company into existence as a legal person. Upon its issue the company is born. Section 34(2) of the
companies Act 1956 provides that ―from the date of incorporation such of the subscribers of the
memorandum and other persons as may from time to time be the members of the company, shall be
body corporate,…capable forth with of exercising all the functions of an incorporated company.‖ The
company‘s life commences from the date mentioned in the certificate of incorporation and the date
appearing on it is conclusive even if wrong.
Thus, under Section 35 of the Companies Act 1956 certificate of incorporation is the conclusive
evidence that all the requirements of the Act have been complied with in respect of registration. In
other words, the validity of the certificate cannot be disputed on any grounds whatsoever. Given issue
is illustrated by the decision of the Judicial Committee of the Privy Council in Moosa Ghoolam Ariff
vs. Ebrahim Goolam Ariff: IRL(1913) 40 Cal 1 PC. In this case their lordship observes that ―When
the certificate of incorporation is given nothing is to be inquired into as the regularity of the prior
proceeding. The certificate prevents all recurrence to prior matters essential to registration and that it is
conclusive that all previous requisites have been complied with‖.

ii) Answer to this question has been prescribed by Section 35 of the Companies Act 1956. In other
words the registration of a company cannot be challenged because of the conclusive effect of the
certificate. The basic objective of this provision is to immune from the problems arising from
defectively incorporated companies. In other words, the validity of the certificate cannot be disputed on
any grounds whatsoever.Thus, in Moosa Ghoolam Ariff vs. Ebrahim Goolam Ariff: IRL(1913) 40
Cal 1 PC. Lord Mac Naghten said:

―Their lordships will assume that the conditions of registration prescribed by the Indian Companies Act
were not duly complied with; that there were no seven subscribers to the memorandum and that the
Registrar ought not to have granted the certificate. But the certificate is conclusive for all purposes.‖

Therefore, the minors or their guardian have no right to make objection on the ground that the
provisions of the Act had not been complied with.

1(c)
Meaning of Holder in due course:
As per section 9 of the Negotiable Instrument Act, 1881, the holder in due course is
any person who
(i) for valuable consideration,
(ii) becomes the possessor of a negotiable instrument payable to bearer or the
endorsee or payee thereof,

XTQ
(27)

(iii) before the amount mentioned in the document becomes payable, and
(iv) without having sufficient cause to believe that any defect existed in the
title of the person from whom he derives his title.

The essential qualification of a holder in due course may, therefore, be summed up


as follows:
1. He must be a holder for valuable consideration.
2. He must have become a holder (possessor) before the date of maturity of
the negotiable instrument.
3. He must have become holder of the negotiable instrument in good faith.

A holder in due course must take the negotiable instrument complete and regular on
the face of it.

Privileges of a holder in due course:


1. Instrument purged of all defects:
A holder in due course who gets the instrument in good faith in the course of its
currency is not only himself protected against all defects of title of the person from
whom he has received it, but also serves, as a channel to protect all subsequent 38
holders. A holder in due course can recover the amount of the instrument from all
previous parties although, as a matter of fact, no consideration was paid by some of
the previous parties to instrument or there was a defect of title in the party from
whom he took it. Once an instrument passes through the hands of a holder in due
course, it is purged of all defects. It is like a current coin. Who-so-ever takes it can
recover the amount from all parties previous to such holder.
Examples: A bill of exchange payable to bearer is stolen. The thief delivers it to B, a
holder in due course. B can recover the money of the bill.

2. Rights not affected in case of an inchoate instrument:


Right of a holder in due course to recover money is not at all affected even though
the instrument was originally an inchoate stamped instrument and the transferor
completed the instrument for a sum greater than what was intended by the maker.

3. All prior parties liable:


All prior parties to the instrument (the maker or drawer, acceptor and intervening
endorsers) continue to remain liable to the holder in due course until the instrument
is duty satisfied. The holder in due course can file a suit against the parties liable to
pay, in his own name.

4. Can enforce payment of a fictitious bill:


Where both drawer and payee of a bill are fictitious persons, the acceptor is liable
on the bill to a holder in due course. If the latter can show that the signature of the
supposed drawer and the first endorser are in the same hand, for the bill being
payable to the drawer‘s order the fictitious drawer must endorse the bill before he
can negotiate it.

5. No effect of conditional delivery:


Where negotiable instrument is delivered conditionally or for a special purpose and
is negotiated to a holder in due course, a valid delivery of it is conclusively
presumed and he acquired good title to it.

6. No effect of absence of consideration or presence of an unlawful consideration:

XTQ
(28)

The plea of absence of or unlawful consideration is not available against the holder
in due course. The party responsible will have to make payment.

7. Estoppel against denying original validity of instrument:


The plea of original invalidity of the instrument cannot be put forth, against the
holder in due course by the drawer of a bill of exchange or cheque or by an acceptor
for the honour of the drawer. But where the instrument is void on the face of it e.g.
promissory note made payable to ―bearer‖, even the holder in due course cannot
recover the money.

8. Estoppel against denying capacity of the payee to endorsee:


No maker of promissory note and no acceptor of a bill of exchange payable to order
shall, in a suit therein by a holder in due course, be permitted to resist the claim of
the holder in due course on the plea that the payee had not the capacity to endorse
the instrument on the date of the note as he was a minor or insane or that he had no
legal existence.

9. Estoppel against endorser to deny capacity of parties:


An endorser of the bill by his endorsement guarantees that all previous
endorsements are genuine and that all prior parties had capacity to enter into valid
contracts. Therefore, he on a suit thereon by the subsequent holder cannot deny the
signature or capacity to contract of any prior party to the instrument.
1(d)
The basic principle relating to the administration of the affairs of a company is that
the court will not, in general, intervene at the instance of the shareholders in matters
of internal administration where the majority powers has been exercised in good
faith for the benefit of the company as a whole. It follows that the court will
interfere to protect the minority only where the majority shareholders of a company
propose to benefit themselves at the expense of the minority.
In Foss v. Harbottle: the action was by two shareholders in a company against the
directors charging them concerting and effecting various fraudulent and illegal
transactions whereby the property of the company was misapplied and wasted, and
pleading that the defendant directors might be decreed to make good to the
company the losses.

The action was rejected in respect of those transactions which a majority of the
shareholders had the power to confirm. The court held that the conduct with which
the defendants are charged is an injury not to the plaintiffs exclusively; it is an
injury to the whole corporation. In such cases, the rule is that the corporation should
sue in its own name and in its corporate character. It is not a matter of course for
any individual members of a corporation thus to assume to themselves the right of
suing in the name of the corporation.
In this case, it is very much for the benefit of the company to get rid of the members
who were in competing business; as such members have the unique opportunity of
exploiting the company's business secrets against its very interest.

Therefore, the resolution passed by the majority of the shareholders is valid and not
oppressive to the minority of shareholders as the individual interest may be

XTQ
(29)

sacrificed to the economic exigencies of the company.

Part: "B"
2.
a) The Annual General meeting of Cee Limited has appointed Shreenath, as an
auditor of the company for the year 2069/70. During the course of audit, it
was found that Shreenath is the son-in-law of T. Shivam, who holds 1.02%
paid up capital of the company. Do you think this appointment is valid?
What types of persons or the firms or companies in which such persons are
partners shall be disqualified for appointment as auditor? Explain referring
the provisions of Company Act, 2063. 5

b) Satyam Petro-chemical Ltd. a Nepali Company entered into a contract with


Aditya Suppliers, an agent of a foreign company, Castrol Refineries,
London, for the supply of 10,000 K. L. of crude oil with 65 percent of
petrol. Clause 10 of the contract has clearly stated that the Nepali agent shall
not be held liable for the act or default of principal regarding the supply of
consignments. When the consignment arrived, the Satyam petro-chemicals
discovered that the recovery petrol was much less than 65 percent as
guaranteed by the foreign principal. Satyam Petro-chemicals filed a suit in
Nepal as against the agent. Decide with reasons the following issues:
i) Is there breach of contract? 2.5
ii) Is the suit against the agent maintainable? 2.5
c) Under what circumstances the negotiable instrument deemed to be dishonored? 5
Answer:
a) According to the provisions of Company Act, 2063, Section 112 various
disqualifications to be an auditor of the company. Sub Section (1)(e) has
clarified the substantial shareholder of the company or his relative, both of
them are disqualified to become the auditor of the company. In a given case
T.Shivam is close relative of Mr. Shreenath who is a substantial shareholder
of the company as his shareholding in the company is more than 1%. Under
this section section has further defined other conditions of disqualification
as follows:

(a) A director, advisor appointed with entitlement to regular remuneration or cash


benefit, a person or employee or worker involved in the management of the
company or a partner of any of them or and employee of any of such partners or
a close relative of a director or partner, out of them, or and employee of such
relative;

(b) A debtor who has borrowed moneys from the company in any manner, or a
person who has failed to pay any dues payable to the company within the time
limit and is in such arrears or close relative of such person;
(c) A person who has been sentenced to punishment for an offense pertaining to
audit and a period of Three years has not elapsed thereafter;
(d) A person who has been declared insolvent;
(e) A substantial shareholder of the company or a shareholder holding one percent
or more of the paid up capital of the company or his/her close relative;
(f) A person who has been sentenced to punishment for an offense of corruption,
fraud or a criminal offense involving moral turpitude and a period of five years
has not elapsed there after;
XTQ
(30)

(g) A person referred to in Sub-section (3) of Section 111;


(h) In the case of a public company, any person who works, whether full time or
part time , for any governmental body or anybody owned fully or partly by the
Government of Nepal or any other company or a partner of such person or a
person who is working as an employee of such partner or a person who is
authorized to sign any documents or reports to be prepared by the engagement
of the company;
(i) A company or corporate body with limited liability;
(j) A person having interest in any transaction with the company or his/her close
relative or a director, officer or substantial shareholder of another company
having any interest in any transaction with the company.

b) There is breach of contract, because


i. Pursuant to section 82 (1) of the Contract Act, 2056, if one of the parties of a
contract fails or refuses or becomes unable to fulfill his/her obligation as per the
terms and conditions of the contract between them without lawful excuse it is called
the breach of contract. In this case the contract has been breached as the
consignment was not in the terms of contract agreed as the recovery petrol was
much less than 65 percent as specified in the contract. Where there is breach of
contract, the injured party can file a case before the court for the various remedies
under the Act. One of the important remedies is suit for damages or compensation.
The compensation can be claimed against the party who has breached the contract.

ii. In this case the contract has been breached as there was the default delivery by the
Castrol Refineries, London and case is filed by the Satyam Petro-chemical against
the Nepali agent. The general rule of agency is that for the act of the agent in the
business of agency the principal is responsible towards the third party. But where an
agent acts for a foreign principal, he incurred personal liability. Therefore, under the
exception the agent in Nepal acting as a foreign principal can be held personally
liable for the acting on behalf of or default of the principal. Hence, the suit against
the agent is maintainable.

c) According to S. 51, 64 & 65 of the Negotiable Instruments Act 2034 the Negotiable
Instruments shall be deemed to be dishonoured in the following situation:
In pursuance to Section 51 the Negotiable Instrument shall be deemed to be
dishonoured in the following situation and no presentment is necessary for its
payment:-
(a) If the maker, acceptor or drawee deliberately prevents the presentment of the
Negotiable Instrument, or
(b) If the instrument being payable at the specified place, neither the payer or his
Agent to pay it attends at such place during the business hours on a business day,
or
(c) If the payer closes his/her office during the business hours on a business day, or
(d) If the Negotiable Instrument not being payable at any specified place, the
concerned party cannot be found for the presentment after due search.
Section 64 prescribes the provision Dishonour by Non acceptance as follows:
A bill of exchange shall be deemed be dishonoured by non-acceptance when the
drawee, or one of several drawee not being partners, makes default in acceptance
uon being duly required to accept the bill. If agreed to accept the bill with

XTQ
(31)

conditions or the drawee is incompetent to contract, the bill of exchange will be


deemed to be dishonoured.
Section 65 prescribes the provision Dishonour by Non-payment as follows:
A promissory Note, bill of exchange or Cheque will be deemed to be dishonoured
by non-payment when the maker of the note, accepter of the bill or drawee of the
cheque maked default in payment upon being duly required to pay the same.
3.
a) Explain about provision regarding revolving fund under the Securities Act, 2063.
Also mention the provision regarding the audit of accounts of the Board. (3+2= 5)
b) What is Board‘s Report? State the matters to be included in it. 5
Answer:
a) Securities Act, 2012 has made special provision regarding revolving fund manage
its sources of income. Under Section 23 of Securities Act, 2063 incorporates the
provision regarding revolving fund which are as follows:
i) The Board may establish a revolving fund to manage its source of
income and such amounts as specified by the Securities Board shall
be credited to that fund each year.
ii) The amounts of the revolving fund may be held in securities issued
by the Government of Nepal or in such a periodic account as may be
prescribed by the Board.
iii) Generally, no moneys held in the revolving fund, other than income
earned out of the moneys in that fund, shall be spent.
Section 25 of the Act mentions the provision regarding the
audit of accounts of the Board. It states that the accounts and books
of the Board shall be audited by the Auditor General or any auditor
designated by him. While auditing the accounts and books of the
Board, auditor shall mention the following matters in his/her audit
report.
a) Whether the statement of income and expenditure truly
reflect the incomes and expenditures of the Board in that
fiscal year,
b) Whether the balance sheet of that fiscal year reflects the
true economic condition of the Board.

b) Board Reports means the BoD‘s of every public or private company with the
paid up capital of ten million rupees or more or with an annual turnover of ten
millions rupees or more shall also proper a separate report of the Board of directors‘
Under section 109(4) of the Companies Act, 2063, the directors must prepare a
report for each financial year. This must contain a fair view of the development of
the business of the company and its subsidiaries during the financial year and the
position at the end of it, they must state what amount if any, they recommend
should be paid as dividend, and what amount if any, they proposed to carry to
reserves. In addition, it must give the name of all persons who at any time during
the financial year were director of the company and describe the principal activities
of the company and its subsidiaries and any changes therein during the course of the
year.

The board of directors in addition to the annual financial statements shall prepare a
separate report stating the following matters: -
1. Review of the transaction of the previous year.

XTQ
(32)

2. Impacts, if any, caused on the transactions of the company from national


and international situations.
3. Achievements in the current year as at the date of report and opinions of the
board of directors on matters to be done in the future.
4. Industrial or professional relation of the company.
5. Alterations in the board of directors and the reasons thereof.
6. Amount recommended for payment by way of dividend.
7. Details of forfeiture of shares and the event of buy back of shares, the
reasons and number of shares.
8. Details of total management expenses during the previous financial year.
9. Any other matters required to be set out in the board's report under the
Companies Act or other prevailing laws.
10. Other necessary matters in the opinion of the board of directors.

4.
a) Can a Bank or financial institution buy back of its share? Discuss. 5
b) State the terms of punishment or penalties can be imposed under the
Insurance Act, 2049. 5
Answer:
a) Bank and financial Institution can’t buy back of its shares although the
provisions of this act
Section 10 (1) of the Bank and Financial Institutions Act, 2063 has imposed
prohibition on buy back of shares by the bank or financial institutions. But
Sub-section 2 of section 10 has mentioned the grounds on which the banks
or financial institutions can buy back of their shares. In the following
conditions a bank or financial institution may, with the approval of Nepal
Rastra Bank, buy back its shares out of its free reserves available for being
distributed as dividends not exceeding the percentage prescribed by Nepal
Rastra Bank.
1. If the shares issued are fully paid.
2. If the issued shares are already listed in the Securities Board.
3. If the buyback of shares is authorised by the articles of association of concerned Bank and
financial Institutions.
4. If a special resolution has been adopted for the buyback of shares.
5. If the ratio of debt is not more than twice the capital and general reserve fund
after such buyback.
6. If the value of share bought back is not more than 20% of the total paid up
capital general reserve fund.
7. If the buyback of share is not in contravention to the directives issued by Nepal
Rastra Bank.
b) The Insurance Act, 2049 has special provisions for the punishment to be imposed
to any insurer or the Director of the insurer, employee or surveyor, broker or
insurance agent who has knowingly violated the provisions of Insurance Act and
rules made under the Act or order or directives is mentioned in Section 36 of the
Insurance Act, 2049. The Insurance Board, in such cases, may punish to such
insurer or the Director of the insurer, employee or surveyor, broker or insurance
agent with a fine ranging from Rs. 3,000 to Rs. 10,000. If such offense has been
made frequently, he will be fined at the rate of extra Rs. 500 for each subsequent
offense.
XTQ
(33)

Similarly, sub-section (2) provides that if any insurer or insurance agent or broker
has done any insurance business without following the procedures to be followed
pursuant to this Act, the Board may fine him up to Rs. 10,000. Further, if the
accounts, records, register, details, information or any other documents has not
maintained, prepared or submitted in time or made falsely, he may be punished with
a fine up to Rs. 30,000 or imprisonment up to two years or with both.

5. Answer the following questions:


a) Mr. Scot Wilson, an Irish national, is interested to make investment in the
exploration of petroleum products in Dailekh and other possible areas of the
country. Can Mr. Scot Wilson invest in the exploration of petroleum in
Nepal? If so, what are the procedures? 5

b) Khujeli Tea Estate of Fikkal is producing tea leaves in a small scale since
May 12, 2013, however the estate was producing tea leaves without getting
the permission from the concerned offices of the Government of Nepal. As a
result the government of Nepal is investigating and planning to impose
action against this tea estate. Elaborate the possible action that the
Authorities can take against the tea estate considering the legal provisions of
the Industrial Enterprises Act, 2049. 5

c) What do you understand by the term Government Office? What are the
matters to be audited in view of propriety? Explain with reference to Audit
Act, 2048. (2+3=5)
Answer:

a) The Foreign Investment and Technology Transfer Act, 2049 has given the
opportunities to the foreign national or the companies to invest their money in Nepal.
Mr. Scot Wilson has also the right to invest in Nepal. However, permission from the
Government of Nepal is required before making the investment in Nepal.

Section 3 of The Foreign Investment and Technology Transfer Act, 2049 has provided
the procedures to obtain the permission before transferring foreign currency as
investment in Nepal. As per this Section:
(1) Permission of the Department shall be required to be obtained for foreign
investment or technology transfer.

(2) A person desiring to avail the foreign investment or technology transfer shall
be required to make an application to the Department in the prescribed form along
with the prescribed particulars for obtaining permission in that regard.

(3) If an application is made pursuant to Sub-section (2) the Department shall, in the
case of an industry with fixed assets up to five hundred million rupees, itself, and in
the case of an industry with fixed assets in excess thereof, in accordance with the
decision of the Board, grant permission within thirty days from the date of
application. The Department shall communicate the decision made in regard to such
permission to the applicant.

(4) Notwithstanding anything contained in Sub-section (1) and (2) no permission


shall be granted for making foreign investment in the industries set forth in the
Annex.

XTQ
(34)

b) According to section 9 of the Industrial Enterprises Act 2049, the industries other than
those as set in Annex 2 which may significantly cause adverse effect on the security,
public health and the environment shall not be required permission for establishment,
extension and diversification.

Annex 2 of Industrial Enterprises Act 2049, industries requiring permission are


1. Industries producing Explosive including arms and ammunition and gunpowder,
security printing, bank notes and coin industries.
2. Cigarette, bidi, chewing tobacco, Khaini industries and industries producing other
goods of a similar nature utilizing tobacco as a basic raw material, alcohol and
beer producing industries.

Since Khujeli Tea Estate of Fikkal is producing tea leaves which does not fall under
Annex 2 of The Industrial Enterprises Act 2049 " i.e. industries requiring permission",
permission under Industrial Enterprises Act 2049 is not required.
Thus, any action cannot be taken on the ground that permission under Industrial Enterprises
Act 2049 is not obtained by Khujeli Tea Estate of Fikkal for producing tea leaves.

c)
Government office means all offices of the Government of Nepal and the office
includes the Courts including Supreme Court, Parliament, and Commission for
Investigation of Abuse of Authority, Auditor General, Public Service Commission
and Offices of other Constitutional Bodies and Nepal Army and Nepal Police. This
is defined in Section 2(a) of Audit Act, 2048.

On the basis of Section 5 of the Audit Act, The Auditor General shall audit
following matters considering the propriety there of:

i) On the propriety of any expenditure and authorizations, if the opinion of the


Auditor General such expenditure is a reckless one or is an abuse of national
propriety whether movable or immovable, despite that the expenditure
confirms to the authorization and

ii) On the propriety of all authorizations issued in respect of any grant of


national property whether movable or immovable, fixed or current or
underwriting of any revenue, or any contract, license or permits relating to
mining, forest, water resources etc and any other act of abandoning movable
assets of the nation.

The Auditor General may not include in the report minor items of
discrepancy and other items deemed as insignificant in view of their
propriety which were observed during the audit of income and expenditure.

6. Answer the following questions:


a) Mention the provision regarding the composition of Council of the Institute
of Chartered Accountants of Nepal and their tenure of office under the
Nepal Chartered Accountants Act, 2053. (3+2=5)
b) Nepal Rastra Bank is the regulatory body of all commercial banks and
financial institutions of the country. Mention the regulatory powers of Nepal
Rastra Bank upon commercial banks and financial institutions in order to
control them properly. 5

XTQ
(35)

Answer:
a)
According to Section 7 of Nepal Chartered Accountants Act, 2053 the composition of
Council shall be as follows:

i) Ten persons elected by and amongst chartered accountant members- member


ii) Four persons elected by and amongst registered auditors - member
iii) Three persons nominated by Government of Nepal, upon the recommendation
of the Auditor General of Nepal, from amongst the persons well experienced in
the field of accounting profession - member
The council members shall elect a president and a vice-president from the fellow
chartered accountant council members.

The tenure of office of the president and the vice-president shall be of one year and
upon expiry of the term of office, they shall be eligible to be elected for one more
term.

The tenure of Council Members shall be of three years and upon expiry of the tenure,
they shall be eligible to be re-elected and re-nominated.

b)
Nepal Rastra Bank shall have full powers to regulate the functions and activities of
commercial banks and financial institutions. For the purpose of the regulation, the
Bank may frame rules and bye – laws on the matters which the Bank deems
appropriate and issue necessary order, directives and circular and it shall be the duty of
the concerned commercial bank and financial institution to abide by such Rules, bye –
laws, order, directive and circulars.

Section 79(5) has provided Nepal Rastra Bank the following powers upon the
commercial banks and financial institutions:

(a) To enforce authority and responsibility granted under this Act and any other Act
enacted for licensing, supervising and regulating commercial banks and financial
institutions and to revoke the license of commercial banks and financial institutions
and to take over or to provide in trusteeship the commercial banks or financial
institutions which have been declared insolvent or are on the verge of insolvency;

(b) To investigate or inspect, or supervise or to cause to investigate, inspect or


supervise by any official of the Bank or the person designated by the Bank the books
and accounts, records, documents or register of commercial banks or financial
institutions in order to find whether or not any commercial bank or financial institution
has conducted business and transaction in accordance with the provision made under
this Act or the Rules, Bye-laws framed there under and an order or directive issued
there under;

(c) To issue order to the member of the Board of Directors, official or employee of any
commercial bank or financial institution to provide necessary information about the
bank or institution in cases where it is necessary to inspect and supervise the
transaction of such bank or financial institution.

7.
a) Write short note on misconducts of labour. 5
b) Distinguish between: (25=10)

XTQ
(36)

i) Right Shares and Issue of Shares at Premium


ii) Promissory Notes and Bills of Exchange
Answer:
a)
Labour Act, 2048 has defined the various misconducts of labour. It is defined in
Section 51 of the said Act. As per this Section the following conducts of the
worker or employee shall be deemed as misconduct:

1. Any bodily harm or injury or fetters or detains to the proprietor, manager or


employee with or without use of arm or injury or causes any violence or
destruction or assault within the enterprise.
2. Stealing the property of enterprise.
3. Embezzlement in the transactions of the enterprise.
4. Accepting or offering bribes.
5. Imprisonment in a crime involving moral turpitude.
6. Strikes without fulfilling the legal requirements.
7. Intentionally destroy of any property of the enterprise or cause damage thereon.
8. Absent from the work frequently without obtaining permission or comes late
after the regular time.
9. Consuming alcoholic substances during the working time.
10. To perform any activity with a motive of causing damage to secrecy relating to
special technology of the enterprise.

b)
Rights shares
Section 56 of Company Act, 2063 has made certain provisions where that company
alters its shares. The company can increase its capital by issuing rights shares.

A rights issue is an issue of rights to buy additional securities in a company made to


the company's existing security holders. When the rights are for equity securities,
such as shares, in a public company, it is a way to raise capital under a seasoned
equity offering. Rights issues are sometimes carried out as a shelf offering. With the
issued rights, existing security-holders have the privilege to buy a specified number
of new securities from the firm at a specified price within a specified time. In a
public company, a rights issue is a form of public offering (different from most
other types of public offering, where shares are issued to the general public).

Rights issues may be particularly useful for closed-end companies, which cannot
retain earnings, because they distribute essentially all of their realized income and
capital gains each year; therefore, they raise additional capital through rights
offerings. As equity issues are generally preferable to debt issues from the
company's viewpoint, companies usually opt for a rights issue when they have
problems raising equity capital from the general public and choose to ask their
existing shareholders to buy more shares.

Share issue at premium


Section 29 of Company Act, 2063 has made provision to issue shares at premium.
Each who fulfilled these conditions could apply for issuance of shares at premium.

A term is used when a company issues shares of its stock at price above its par
value. The excess cash, or premium received by the company is place in a shared
premium account and can be used to pay up unissued shares for distribution as
bonus shares; to pay a premium on the redemption of preferred stock ; writing down
company expenses or expenses incurred in the issuance of the shares.
XTQ
(37)

A company issues its shares at a premium when the price at which it sells the shares
is higher than their par value. This is quite common, since the par value is typically
set at a minimal value, such as Rs. 100 per share. The amount of the premium is the
difference between the par value and the selling price.

If shares do not have a par value, then there is no premium. In this case, the entire
amount paid is recorded in the common stock account (if the payment is for
common stock, rather than for some form of preferred stock).

For example, if ABC Company sells a share of common stock to an investor for Rs.
110, and the stock has a par value of Rs. 100, then it has issued the share at a
premium of Rs. 10.

c)
A negotiable instrument is a document guaranteeing the payment of a specific
amount of money, either on demand, or at a set time, with the payer named on the
document. More specifically, it is a document contemplated by or consisting of a
contract, which promises the payment of money without condition, which may be
paid either on demand or at a future date. The term can have different meanings,
depending on what law is being applied and what country it is used in and what
context it is used in. Promissory Note and bill of exchange are also a kind of
negotiable instruments, however there are certain difference between them as
follows:

(1) Parties.
There are three parties to a bill of exchange, namely, the drawer, the drawee and the
payee; while in a promissory note there are only two parties – maker and payee.

(2) Nature of payment.


In a bill of exchange, there is an unconditional order to pay, while in a promissory
note there is an unconditional promise to pay.

(3) Acceptance.
A bill of exchange requires an acceptance of the drawee before it is presented for
payment, while a promissory note does not require any acceptance since it is signed
by the persons who is liable to pay.

(4) Liability.
The liability of the maker of a promissory note is primary and absolute, while the
liability of a drawer of bill of exchange is secondary and conditional. It is only
when the drawee fails to pay that the drawer would be liable as a surety.

(5) Notice of dishonor.


In case of dishonor of bill of exchange either due to non-payment or non-
acceptance, notice must be given to all persons liable to pay. But in the case of a
promissory note, notice of dishonor to the maker is not necessary.

(6) Maker’s position.


The drawer of a bill of exchange stands in immediate relationship with the acceptor
and not the payee. While in the case of a promissory note, the maker stands in
immediate relationship with the payee.

XTQ
(38)

(7) Nature of acceptance.


A promissory note can never be conditional, while a bill of exchange can be
accepted conditionally.

(8) Copies.
A bill of exchange can be drawn in sets, but a promissory note cannot be drawn in
sets.

(9) Payable to bearer.


A promissory note cannot be made payable to a bearer, while a bill of exchange can
be so drawn provided it is not payable to bearer on demand.

(10) Payable to maker.


In a promissory note, the maker cannot pay to himself. While in the case of a bill of
exchange, the drawer and the payee may be one person.

(11) Protest.
Foreign bills must be protested for dishonor when such protest is required by the
law of the place where they are drawn. But no such protest is required in the case of
a promissory note

XTQ
(39)

CAP-II--Suggested—FM—June,2014

Roll No……………. Maximum Marks - 100


Total No. of Questions – 7 Total No. of Printed Pages – 4
Time Allowed – 3 Hours
Marks
Attempt all questions.
Working notes should form part of the answer. Make assumptions wherever necessary.

1. DVD Limited is a manufacturer of DVDs. It is currently producing 50,000 DVDs


annually and all the DVDs are sold on credit basis. Currently, 80% of its working
capital requirement is financed by Nepal Bank at an interest rate of 15% p.a.
Remaining working capital requirement is financed by informal loans at 24% p.a.
Following is the costing and revenue per unit:
Rs.
Cost of raw material 40
Labor charge 15
Overhead cost 30
Cost of production 85
Selling price of DVD 105
Moreover, following working capital parameters are observed:
a) Average raw material in stock: 1 month
b) Work in progress: a half month (Completion Stage: 50%)
c) Average finished goods in stock: 1 month.
d) Credit period allowed for DVD Limited: 1 month.
e) Credit period allowed to debtors: 2 months average time
f) DVD Limited normally pays to workers and overhead only after one month of
work done.
g) Average level of bank and cash balances is Rs. 50,000, in all situations.
DVD Limited is planning of scaling up its operations to the production and sales level
of 100,000 units of DVD per year. It wants to finance its working capital requirement
by another bank because the other bank is ready to finance its 90% of working capital
requirement at 12% p.a. However, minimum loan of Rs. 1.5 million needs to be
taken. If it decides to scale up, raw material, labor and overhead costs are likely to
increase by 10% in incremental output. However, selling price of the DVD will come
to the level of Rs. 102 per unit, if production is increased from existing level, and will
apply to all the output.
Required: (17+3=20)
i) Assess whether DVD Limited‘s plan is considerable or not.
ii) Briefly explain about the sources of working capital finance, and state how firms
develop a short term financing plan.
Answer:
i)
Calculation of Current Income
Rs.
Gross
Particulars Output Rate Revenue

Sales revenue 50,000 105 5,250,000


Cost of production 85
XTQ
(40)

50,000 4,250,000

Gross margin 1,000,000


Less: Interest on working capital
Existing Working Capital
Requirement 1,013,542 (W.N. 1)

Bank Loan Interest (on 80% of WC) 810,834 15% (121,625)


Interest for Informal
Loan(remaining) 202,708 24% (48,650)
(170,275)

Net Margin 829,725

Calculation of income in Scaled Up Production Situation Rs.


Gross
Particulars Output Rate Revenue
Sales Revenue 100,000 102 10,200,000
Cost of Production 100,000 89.25 (average) 8,925,000
Gross margin 1,275,000
Less: interest on working capital:
Total Working Capital Requirement 2,073,438 (W.N. 2)
Bank Loan Interest (90% of WC) 1,866,094 12% 223,931)
Interest for Informal
Loan(remaining) 207,344 24% (49,763)
(273,694)
Net Margin 1,001,306

Since scaling up of production will result more net margin as compared to existing, the scaling up plan
is considerable.
Working Notes:
1. Calculation of Working Capital Requirement (Current Production Level)

Working
Output Per Unit Capital
Particulars Level Cost Time Lag (Rs.)
Current Assets :

Stock of Raw material 50000 40 1 month 166,667


Work in Progress:
Half Month (50%
Raw Material 50000 40 Complete) 41,667
Half Month (50%
Labor Charge 50000 15 Complete) 15,625
Half Month (50%
Overhead cost 50000 30 Complete) 31,250
Stock of Finished
Goods 50000 85 One Month 354,167

Average Debtor 50000 85 Two Months 708,333


Cash and Bank
Balances 50,000
XTQ
(41)

Less: Current
Liabilities

Average Creditors 50000 40 One Month (166,667)


Outstanding labour
charge 50000 15 One Month (62,500)
Outstanding overhead
cost 50000 30 One Month (125,000)
Total working capital requirement 1,013,542

2. Calculation of Working Capital Requirement in Scale Up situation

Per Unit Cost Incremental


Increment (existing + 10% Working
Particulars al Output of existing) Time Lag Capital(Rs.)
Current Assets:
Stock of Raw
material 50000 44 1 month 183,333
Work in Progress:
Half Month
(50% 45,833
Raw Material 50000 44 Complete)
Half Month
(50% 17,188
Labor Charge 50000 16.5 Complete)
Half Month
(50% 34,375
Overhead Cost 50000 33 Complete)
Stock of Finished
Goods 50000 93.5 One Month 389,583

Average Debtor 50000 93.5 Two Months 779,167


Less: Current
Liabilities

Average Creditors 50000 44 One Month (183,333)


Outstanding labour
charge 50000 16.5 One Month (68,750)
Outstanding overhead
cost 50000 33 One Month (137,500)

Incremental WC 1,059,896

Total Working Capital requirement in Scale up situation 2,073,438


(existing plus incremental WC)

ii)
There can be broadly two types of working capital finances. One is internal source and other is
external. Trade credit, credit from employees, credit from suppliers of raw materials etc. are those
sources which are generated within the business. Besides, short term loans from banks, commercial
paper, factoring etc. are other sources of working capital finances.

XTQ
(42)

Designing for the best short-term financial plan inevitably proceeds by trial and error. The financial
manager must explore the consequences of different assumptions about cash requirements, interest
rates, limits on financing from particular sources, and so on. This is because calculation of how
much short term financing is required at what time is very much crucial as there may be various
options of financings with varied cost and other impacts. Firms are increasingly using computerized
financial models to help in this process. The financial manager should remember the key
differences between the various sources of short-term financing—for example, the differences
between bank lines of credit and commercial paper. It should also be remembered that firms often
raise money on the strength of their current assets, especially accounts receivable and inventories.

2.
a) In order to increase sales from their present annual level of Rs. 2, 40,000, Agni
Associates is considering a more liberal credit policy. Currently, the firm has an
average collection period of 30 days. However, it is believed that as collection
Period is lengthened, sales will increase by following amounts-
Credit Policy Increase in Average Collection Period Increase in Sales
A 15 Days Rs. 10,000
B 30 Days Rs. 15,000
C 45 Days Rs. 17,000
D 60 Days Rs. 18,000
The Variable Costs of the Firm‘s product is 60% of Sales Price.
If the Firm has pre-tax opportunity cost of 20%, which credit policy should be
pursued? (Assume a 360-Day year). 7
b) Following are the data on a capital project being evaluated by the Management of
Sagun Ltd.
A Annual Cost Saving Rs. 40,000
B Useful Life 4 years
C Internal Rate of Return 15%
D Profitability Index 1.064
E Net Present Value ?
F Cost of Capital ?
G Cost of Project ?
H Payback ?
I Salvage Value Nil
Required: 5
Find the missing value considering the following table of discount factor only
Discount Factor 15% 14% 13% 12%
1 year 0.869 0.877 0.885 0.893
2 Year 0.756 0.769 0.783 0.797
3 Year 0.658 0.675 0.693 0.712
4 Year 0.572 0.592 0.613 0.636
2.855 2.913 2.974 3.038
c) The data for the three securities A, B and C are as follows:
2)

Securities Likely Return Standard Deviation


A 16% 0.21
B 16% 0.20
C 21% 0.25

XTQ
(43)

Does anyone security dominates another? Which type of investor prefers security
C? 3
Answer:

a) Solution: Evaluation of alternative credit policies


Particulars Present Policy A Policy B Policy C Policy D
1. Sales(given) Rs.2,40,000 Rs.2,50,000 Rs.2,55,000 Rs.2,57,000 Rs.2,58,000
2.Variable cost at 60% of sales Rs.1,44,000 Rs.1,50,000 Rs.1,53,000 Rs.1,54,200 Rs.1,54,800
3. Contribution=(1-2) Rs.96,000 Rs.1,00,000 Rs.1,02,000 Rs.1,02,800 Rs.1,03,200
4. Cost of Debtors Rs.1,44,000 Rs.1,50,000 Rs.1,53,000 Rs.1,54,200 Rs.1,54,800
p.a.=Variable Cost of sales
5. Collection Period (in days) 30 days 45 days 60 days 75days 90 days
6. Average Debtors= Rs.12,000 Rs.18,750 Rs.25,500 Rs.32,125 Rs.38,700
7. Interests on Average Rs.2,400 Rs.3,750 Rs.5,100 Rs.6,425 Rs.7,740
Debtors (20%)
8. Net Benefit =(3-7) Rs.93,600 Rs.96,250 Rs.96,900 Rs.96,375 Rs.95,460

Decision:
The Firm should select Policy B, i.e. 60 Days credit, since maximum benefit is obtained
under that policy.

Note: Alternatively, student may solve the above question under incremental approach.

b) Solution
(i) Calculation of Cost of Project
Let cost of project be x
Cost of Project at IRR 15% is equal to PV of Cash Inflow (Annual Cost Saving)
X =Rs.40, 000×2.855
=Rs.1, 14,200

(ii) Calculation of Pay Back period


Pay Back Period = Cost of Project/ Annual Cash Inflow
=Rs.1,14,200/Rs.40,000
=2.855 years

(iii)Calculation of PV of Cash Inflow


Profitability Index =PV of Cash Inflow/PV of Cash Outflow
PV of Cash Inflow = 1.064 ×Rs.1, 14,200
=1, 21,509

(iv) Calculation of Net Present Value


NPV = PV of Cash Inflow-PV of Cash Outflow
=Rs.1, 21,509-Rs.1, 14,200
=Rs.7, 309

(v) Calculation of Cost of Capital


PV of Cash Inflow = Annual Cash Inflow×PVF for 4 years at Cost of Capital
PVF for 4 years = Rs.1,21,509/Rs.40,000
=3.0378
Cost of copied = 12 %
c)

XTQ
(44)

-In case of security A and B return is equal but security B has comparatively lower
standard deviation i.e. it has lower risk, hence security B dominates security A.
-Investors who prefer high return will prefer security C irrespective of having risk i.e. high standard
deviation. He belongs to risk taking group.
3.
a) The XYZ limited is contemplating a debenture issue on the following terms:
Face value = Rs. 100 per debenture
Term of maturity= 7 years
Coupon rate of Interest:
Years 1-2=8% p.a.
3-4=12% p.a.
5-7=15% p.a.
The Current market rate of interest on similar debenture is 15% p.a. The company
proposes to price the issue so as to yield a (compounded) return of 16% p.a. to the
investor. Determine the issue price. Assume the redemption on debenture at a
premium of 5% (Note: The present value interest factors at 16% p.a. for years 1 to
7 are .862, .743, .641, .552, .476, .410, and .354 respectively). 4
b) A limited, a widely held company is considering a major expansion of its
production facilities and the following alternatives are available:

(Rs in Lakhs)
Alternatives
Share capital 50 20 10
14% Debentures - 20 15
Loan from Financial - 10 25
institution @ 18% p.a.
Expected rate of return before tax @ 25%. Income Tax Rate 50%. The rate of
dividend of the company is not less than 20%. The company at present has low
debt.
Which of the alternatives you would choose? 4
c) The Servex Company has the following capital structure on 30th June 1998:
Rs.
2,00,000 Ordinary shares 40,00,000
10% Preference shares 10,00,000
14% Debenture 30,00,000
80,00,000
The share of the company sells for Rs. 20 per share. It is expected that the
company will pay a dividend of Rs. 2 per share next year which will grow at 7
percent forever. Assume a 50 percent tax rate.
Required: (1+3+3=7)
i) Compute a weighted average cost of capital based on the existing capital
structure.
ii) Compute the new weighted average cost of capital if the company raises an
additional Rs. 2 million debt by issuing 15 percent debentures. This would
result in increasing the expected dividend to Rs. 3 and leave the growth rate
unchanged, but the price of share will fall to Rs. 15 per share.
iii) Compute the cost of capital, if in ii) above growth rate increases to 10 percent.
Answer:
a) The interest payments over the life of the debentures and their present values are given in the
following table:
XTQ
(45)

Year Interest(Rs.) PVF@16% Present Value(Rs.)


1 8 .862 6.896
2 8 .743 5.944
3 12 .641 7.692
4 12 .552 6.624
5 15 .476 7.14
6 15 .410 6.15
7 15 .354 5.31
Total 45.756

The present value of the redemption amount of Rs. 105 (Rs.100+Rs.5)@16% p.a. is Rs.
105*.354=Rs. 37.17
Therefore, the present value of the debenture is Rs. 45.76+Rs. 37.17=Rs. 82.93. The company
should issue the debenture at this value in order to yield a return of 16% to the investors.
b) Solution
Evaluation of Financial Alternatives
Particulars A B C
Return on Rs. 50 Lakhs@25% 12.5 12.5 12.5
Less: interest on debentures - 2.80 2.10
Less: Interest on loan - 1.8 4.5
Taxable Profit 12.5 7.9 5.9
Less: Income Tax 50% 6.25 3.95 2.95
Profit after tax available for shareholders 6.25 3.95 2.95
Rate of return on share capital 12.5% 19.75% 29.5%

From shareholders point of view alternative C is to be chosen as it gives the highest rate of return
on share capital.
c)
i) WACC: Existing capital structure

After-tax Cost Weights Weighted Cost

Ordinary Shares (WN 1) 0.17 0.500 0.0850


10% Preference 0.10 0.125 0.0125
14% Debentures 0.07 0.375 0.0262
WACC 0.1237 or 12.37%

ii) WACC: New capital structure

Amount Rs. After-tax Cost Weights Weighted Cost

Ordinary Shares 4,000,000 0.27 (WN3) 0.40 0.108


10% Preference 1,000,000 0.10 0.10 0.010
14% Debentures 3,000,000 0.07 0.30 0.021
15% Debentures 2,000,000 0.075(WN4) 0.20 0.015
Weighted Average Cost of Capital 0.154 or 15.4%

iii) WACC : changed growth rate

After-tax Cost Weight Weighted Cost


Ordinary Shares 0.30 (WN5) 0.40 0.120
10% preference 0.10 0.10 0.010
14% Debenture 0.07 0.30 0.021
15% Debenture 0.075 0.20 0.015
Weighted Average Cost of Capital 0.166 or 16.6%

XTQ
(46)

(WN1) Cost of ordinary share is: ke= +g = +0.07 = 0.10+0.07 = 0.17


(WN2) After tax cost of debenture = 14%×(1-0.5)=7%
(WN3) Cost of ordinary share is: ke= +g = +0.07 = 0.20+0.07 = 0.27
(WN4) After cost of debenture= 15%×(1-0.5)=7.5%
(WN5) Cost of ordinary share is: ke= = +0.10 = 0.20+0.10 = 0.30
4.
a) Determine the market value of equity shares of the company from the following
information as per Walter`s Model:
Earnings of the company Rs. 5,00,000
Dividend paid Rs. 3,00,000
Number of the shares outstanding 1,00,000
Price-earnings ratio 8
Rate of return on investment 15%
Are you satisfied with the current dividend policy of the firm? If not what should
be the optimal dividend payout ratio? 6
b) Consider two bonds with Rs. 1,000 face value that carry coupon rate of 8%, make
annual coupon payment and exhibit similar risk characteristics. The first bond has
5 years to maturity whereas the second has 10 years to maturity. The appropriate
discount rate for the investment of similar risk securities is 8%.
Required: (2+2=4)
i) Calculate current market price of both the bonds.
ii) If this discount rate rises by 2 %, what will be the respective percentage price
changes of the two bonds, and why?
c) Explain the Miller and Modigliani as irreverence theory of Dividend policy. 5
Answer:
a)
Price earning ratio= Market Price/EPS
8= Market Price/5
So, Market Price=Rs. 40
EPS= 500,000/100,000=Rs.5
DPS=300,000/100,000=Rs. 3
Dividend payout ratio=DPS/EPS*100=3/5*100=60%

Walter’s Formula
As the P/E ratio is given 8, and the cost of capital (Ke) is also defined as the reciprocal of P/E ratio,
therefore the Ke may be taken as 1/8=.125 i.e.12.5%
Since, this is a growth firm having rate of return (15%) more than cost of capital(12.5%), therefore, the
company will maximize its market price if it retains its 100% of its profits. The current market price of
Rs. 40 ( based on P/E Ratio can be increased by reducing the payout ratio. If the company opts for
100% retention (i.e. 0% payout), the market price of the share as per Walter‘s formula should be as
follows;
P=D/Ke+((r/Ke)(E-D))/Ke
P=0/.125+((.15/.125)(5-0))/.125
= Rs. 48
So, the firm can increase the market price of the share up to Rs. 48 by increasing the retention ratio to
100% or in other words, the optimal dividend payout for the firm is 0.
b)
i) Since the required rate of return, i.e. discount rate is equal to the coupon rate of 8%, the current
market price is equal to their face value i.e. Rs. 1000.
XTQ
(47)

First Bond
MP =Int. (PVIFA, 5 yrs) + M(PVIF, 5th yrs)
=80(8%, 5 yrs) + 1000(8%, 5th yrs)
=80×3.9927 + 1000×0.6806
=319.42 + 680.60
= Rs. 1,000.02 = Rs. 1000
2nd Bond
MP=80(8%, 10 yrs) + 1,000(8%, 10th yr)
=80×6.7101 + 1,000×0.4632
=536.81 + 463.20
=Rs. 1,000.01 = Rs. 1,000

ii) When discount rate is increased by 2%:


New discount rate: 8+2= 10%.

Face Value 1000


Coupon 8%
Required rate of return: 10%
First Bond
MP = Int (PVIFA 10%, 5 yrs) + M (PVIF 10%, 5th yr)
=80X 3.7908 +1000X 0.6209
=Rs. 924.16
% change in price
924.16-1000
1000
= - 7.58%

Second Bond
MP = Int (PVIFA 10%, 10 yrs) + M (PVIF 10%, 10th yr)
= 80X 6.1446 +1000*0.3855
=Rs. 877.07
% change in price
= 877.07-1000
1000
= -12.29%

Therefore, change in price of second bond is greater than that of bond first because of its longer
maturity period. It is because the longer the maturity period, the higher the sensitivity of bond price
to the interest rate change.

c) Miller and Modigliani have opined that the price of equity shares of a firm depends solely on its
earnings power and is not influenced by the manner in which its earnings are split dividend and
retained earnings.
They observed `under condition of perfect capital markets, rational investors, absence of tax
discrimination dividend income and capital appreciation given the firm`s investment policy, its
dividend policy may have no influence on the market price of the shares‘. In other words, the price
of the share is not affected by the size of the dividend.
M-M‘s hypothesis of irrelevance is based on the following assumptions:
Perfect capital markets: The firm operates in perfect capital markets where investors behave
rationally, information is freely available to all and transactions and floatation costs do not exists.
Perfect capital also implies that no investor is large enough to affect the market price of a share.
No Taxes: Taxes do not exists ; or there are no differences in the tax rates applicable to capital
gains and dividends. This means that investors value a rupee of dividend as much as a rupee of
capital gain.

XTQ
(48)

Investment policy:The firm has a fixed investment policy.


No Risk:Risk of uncertainty does not exist. That is, investors are able to forcast future prices and
dividends with certainty, and one discount rate is appropriate for all securities and all time periods.
Thus, r=k=k, for all t.
MM provide the following proof in support of their views. According to them, market price of share
in beginning of a year (P) is equal to present value of sum of dividend at the end of the year (D1)
and market price of the share at the end of year (P1).
(D1+P1)
P0= -------------
(1+Ke)
Where Ke=Cost of equity capital. It is also referred as capitalization rate discount. MM conclude
that does not affect the market price of share.
5.
a) Calculate Operating, Financial and Combined Leverage from the following data
under situations I and II of Financial Plans A and B. Installed Capacity is for
4,000 units, whereas actual production and sales are 75% of the capacity only.
Selling Price Rs. 30 per unit and variable cost ratio- 50% Fixed Cost under
Situation I is Rs. 15,000 and under Situation II is Rs. 20, 000. 8

Particulars Financial Plan A Financial Plan B


Equity Rs. 10,000 Rs. 15,000
Debt at 20% interest Rs. 10,000 Rs. 5,000
Total Rs. 20,000 Rs. 20,000
b) Using the following information, complete the Balance Sheet given below
Total Debt to Net worth :- 1:2
Total Assets Turnover :- 2
Gross Profit on Sales :- 30%
Average Collection period (assume 360 days in a year): 40 days
Inventory Turnover Ratio based on cost of goods sold and year end inventory: 3
Acid Test Ratio = 0.75
Liabilities Rs. Assets Rs.
Equity Share Capital 4,00,000 Plant and Machinery and ?
other Fixed Assets
Reserve and Surplus 6,00,000 Current Assets
Current Liabilities -? Inventory ?
- Debtors ?
- Cash ?
Total Total
7
Answer:
a) Solution
I. Computation of Earning before Interest & Tax(EBIT) and Earning before Tax( EBT)
Particulars Financial Plan A Financial Plan B
Situation I Situation II Situation I Situation II
Sales Revenue Rs.90,000 Rs.90,000 Rs.90,000 Rs.90,000
(3,000units × s.30) (3,000units×Rs.30) (3,000units × Rs.30) (3,000units×Rs.30)
Less: Rs. Rs.45,000 Rs.45,000 Rs.45,000 Rs.45,000
Variable cost (Rs.90,000×50 (Rs.90,000×5 (Rs.90,000× (Rs.90,0
00×50%)

XTQ
(49)

%) 0%) 50%)

Contribution Rs.45,000 Rs.45,000 Rs.45,000 Rs.45,000


Less: Fixed Rs.15,000 Rs.20,000 Rs.15,000 Rs.20,000
Cost
EBIT Rs.30,000 Rs.25,000 Rs.30,000 Rs.25,000
Less: Rs.2,000 Rs.2,000 Rs.1,000 Rs.1,000
Interest on (10,000*20%) (10,000*20%) (5,000*20%) (5,000*20%)
Debt
EBT Rs.28,000 Rs.23,000 Rs.29,000 Rs.24,000

Note- Actual production and sales=4,000units× 75%=3,000 units

II. Computation of Operating, Financial and Combined Leverage


Particulars Financial Plan A Financial Plan B
Situation I Situation II Situation I Situation II
Earning before
Rs.28,000 Rs.23,000 Rs.29,000 Rs.24,000
Tax
OL=
Contribution/EBIT

FL=
EBIT/EBT

DCL=DOL ×DFL 1.50×1.07=1.61 1.80×1.09=1.96 1.50×1.03=1.55 1.80×1.04=1.87

b) Solution
Liabilities Rs. Assets Rs.
Equity Share Capital 400,000 Plant and Machinery and 425,000
other Fixed Assets
Reserve and Surplus 600,000 Current Assets
Current Liabilities 500,000 - Inventory 700,000

- Debtors 333,333

- Cash 41,667

Total 15,00,000 Total 15,00,000

XTQ
(50)

Working Notes
1) Net Worth= Equity Share Capital + Reserve and surplus= Rs.400,000+600,000=Rs.10,00,000

2) So, Hence,

3) Total of Balance Sheet ( on Liabilities side)= Rs.15,00,000 ( after updating working Note 2), so
total Assets= Rs.15,00,000

4) Total Assets T/O= So, Turnover ( i.e. sales)

=Rs.15,00,000×2=Rs.30,00,000
5) Cost of Goods Sold= Sales less Gross Profit= Rs.30,00,000- 30% = Rs.21,00,000
6) Debtors=Sales × =Rs.30,00,000×

7) So, Closing Inventory=

8)

So Cash=Rs.41,667
Note: Quick Liabilities= Current Liabilities in this question, since there is no Bank Overdraft in
Balance Sheet format.

6. Write short notes on: (4×2.5=10)


a) Debt Securitization
b) Vertical Merger
c) Venture Capital Financing
d) Dividend Coverage Ratio
Answer:
Solution
a) Debt Securitization:
It is the method of recycling of funds. It is especially beneficial to financial intermediaries to
support the lending volumes. Assets generally steady cash flows are packaged together and against
this assets pool, market securities can be issued, e.g. housing finance, auto loans and credit card
receivables.
Process of Debt Securitization:
1) The origination function- A borrower seeks a loan from a bank and financial institution. The
credit worthiness of borrower is evaluated and contract is entered into with repayment schedule
structured over the life of loan.
2) The pooling function – Similar loans on receivables are clubbed together to create an underlying
pool of assets. The pool is transferred in favors of special purpose vehicle (SPV), which acts as a
trustee for investors.

XTQ
(51)

3) The securitization function- SPV will structure and issued securities on the basis of
assets pool. The securities carry coupon and expected maturity which can be assets-
based/mortgage based.
b) Vertical Merger:
A vertical merger acquires another firm that is `upstream‘ or ‗downstream‘. It occurs when a
supplier buys a reseller, or vice versa. The key point is that the two companies have a buyer-seller
relationship. Suppose that a jewelry retailer purchased a company that manufactures jewelry. This
would be the vertical merger. Or suppose that a pharmaceutical Company acquired a drugstore
chain.
c) Venture Capital financing refers to financing of high risk ventures promoted by new qualified
entrepreneurs who require funds to give shape to their ideas. Here, a financier (called venture
capitalist) invest in the equity or debt of an entrepreneur (promoter) undertaking who has a
potentially successful business idea but does not have desired track record or financing
backing.Generally, venture capital funding is associated with heavy initial investment business like
energy conservation, quality up gradation or with sunrise sector like information technology.
d) Dividend Coverage Ratio

It measures the ability to pay dividend on preference share which carry a stated rate of return. This
ratio is the ratio (expressed as X number of times) of net profits after taxes (EAT) and the amount
of preference dividend. Thus,
Dividend coverage= EAT/ Preference dividend
It can be seen that although preference dividend is fixed obligation, the earning taken into account
are after taxes. This is because, unlike debt on which interest is a charge on the profits of the firm,
the preference dividend is treated as an appropriation of profit. The ratio, like the interest coverage
ratio, reveals the safety margin available to the preference shareholders. As a rule, the higher the
coverage, the better it is from their point of view.
7. Distinguish between: (4×2.5=10)
a) Recourse and Non-Recourse factoring
b) Annuity and Perpetuity
c) Horizontal analysis and Vertical analysis
d) Yield to maturity (YTM) and Yield to call (YTC)
Answer:
a) Recourse and Non-recourse Factoring:
In a recourse or pure factoring, the factor firm is only involved in the work of collection of the
receivables. It does not bear any risk of default by the debtors. Such a risk will have to be invariably
borne by selling the firm. Thus, in case of default by a customer, the selling firm will have to refund
the amount of advanced together with charges as per the agreement which was given by the factor
to the selling firm against the receivables.
In a non-recourse factoring, the firm purchases the receivable of the selling firm by paying the
agreed amount (sales value less commission) to the latter. The payments may be made immediately
or after receiving from the customer buying.
The main feature of non-recourse factoring is that the risk of default by the buyer is borne by the
factor firm and the selling firm receives the sales amount. Thus, this type of factoring will result in
the purchase of receivable by the factor firm.

XTQ
(52)

b) Annuity and Perpetuity


S.No Annuity Perpetuity
1 An annuity is a stream of regular periodic Perpetuity is a stream of payments or type
cash flows (either payments made or of annuity that starts payments on fixed date
received) for a specified period of time and such payments continue forever, i.e.
perpetually. Thus, Perpetuity is a constant
stream of identical cash flows with no end.
2 Future value of Annuity can be computed Perpetuity is a type of annuity which is
using Compounding Technique never-ending, its sum if future value cannot
be calculated
3 Examples Examples
a) Recurring Deposit installments paid to
a) Dividend on Irredeemable Preference
bank.
Share Capital.
b) Life insurance premium per annum
b) Interest on Irredeemable Debt/Bonds.
c) Scholarships paid perpetually from an
endowment Fund, etc.

c) Horizontal analysis and vertical analysis


Horizontal analysis: This technique is also known as comparative analysis. It is conducted by
setting consecutive balance sheet, income statement of statement of cash flow side by side and
reviewing changes in individual categories on year-to-year or multiyear basis. The most important
item revealed by comparative financial statement analysis is trend. The horizontal financial
statements analysis is done by restating amount of each item or group of items as a percentage.

Vertical analysis: Vertical/ Cross-sectional/ Common size statements came from the problems in
comparing the financial statements of firms that differ in size. The vertical analysis represents the
relationship of different items of a financial statement which some common item by expressing
each item as a percentage of the common item.

- In the balance sheet, for example, the assets as well as the liabilities and equity are each
expressed as a 100% and each item in these categories is expressed as a percentage of the
respective totals.
- In the common size income statement, turnover is expressed as 100% and every item in the
income statement is expressed as a percentage of turnover.

d) Yield to Maturity (YTM)andYield to call (YTC)


The Yield to maturity (YTM) or redemption yield of a bond or debentures, is the internal rate of
return (IRR, overall interest rate) earned by an investor who buys the bond or debenture today at the
market price, assuming that the bond will be held until maturity, and that all coupon and principal
payments will be made on schedule. Yield is to maturity is actually a future return, as the rate at
which coupon payments can be reinvested at when received is unknown. It enables investors to
compare the merits of different financial instruments.

XTQ
(53)

The Yield to call (YTC) is one of the variants of YTM. It is the return if held up to call. When bond
or debenture is recallable (can be repurchased by the issuer before the maturity), the market looks
also to the Yield to Call, which is the same calculation of the YTM, but assumes that the bond will
be called, so the cash flow is shortened.

XTQ
(54)

CAP-II, Business Communication, Jun-14


Suggested Answer
Roll No……………. Maximum Marks – 100
Total No. of Questions - 10 Total No. of Printed Pages -2
Time Allowed – 3 Hours
Marks
All questions are compulsory.

Section -'A'

1. Read the following case and analyze it on the basis of the following questions. (3+3.5+3.5=10)
Ishan recently began working at a new job. Although he‘s been in his new job only
five weeks, he likes it a lot, and he‘s told you that he sees a future for himself with
this company. But last week, a problem arose. Along with all other employees, Ishan
was invited to the annual company banquet, at which everyone socializes and awards
are given for outstanding performance. Ishan‘s daughter was in a play the night of
the banquet, so Ishan chose to attend his daughter‘s play rather than the company
event. The invitation to the banquet had stated only, ―Hope to see you there‖ and had
not been RSVP, so Ishan didn‘t mention to anyone that he couldn‘t attend. When he
arrived at work the next Monday morning, however, he discovered the case was
otherwise and had the exchange that follows with his manager. Later, when Ishan
talked with several co-workers who had been around a few years, he discovered that
top management sees the annual banquet as a ―command performance‖ that signifies
company unity and loyalty.
Ishan‘s manager: You skipped the banquet last Saturday. I had really thought you
were committed to our company.
Ishan: My daughter was in a play that night.
Ishan‘s manager: I don‘t care why you didn‘t come. We notice who is really with us
and who isn‘t.
Questions:
a) Explain the misunderstanding that appeared between Ishan and his manager.
b) How might Ishan use the informal network in his organization to learn the
normative practices of the company and the meanings they have to others in the
company?
c) How would you suggest that Ishan repair the damage done by his absence from
the company banquet? What might he say to his manager?
Answer:
a) Since Ishan was new in the company, he couldn‘t understand the invitation as the
compulsory call for attending the banquet. Therefore, he decided to attend his
daughter‘s play which was equally important for him. But for his manager, not
attending the banquet would mean that Ishan was not committed and he was not in
support of the company. This misunderstanding appeared because being a new
employee Ishan did not understand the normative practices of the company. Neither
the manager provided him the clear message, nor did he try to understand the culture
of the company.
b) Ishan would talk to his coworkers informally at tea breaks or at any off hours about
the invitation and what it would mean. He just read the message and understood
what it would generally mean. In that sense, he is not a cautious person. He also
would share his difficulty of attending the banquet to any of his coworkers, or would
ask what he or she could do in such a situation.
c) He can repair the damage by clearly explaining what difficulty he had in attending
the banquet in that evening. He also can talk about the lack of clarity of the message,
and explain that being a new employee he could not understand the normative
XTQ
(55)

practice of the company. Neither any of the coworkers talked to him regarding the
company‘s practices nor the company itself thought to orient him in its practices.

2. Some say that there is no difference between ethics and morals. What are your
views? Explain the importance of ethical behavior at the workplace. (5+5=10)
Answer:
a) There is a fine distinction between ethics and morals which may be enumerated as
follows:

The word ‗Moral‘ is defined as relating to principles of right and wrong. The root word for moral
is Latin word ‗mos‘ meaning custom while the root word for ethics is the Greek word ‗ethos‘
meaning character. Custom and character however provide two different standards for defining
what is wrong and what is right. Character is a personal attribute while custom is defined by a group
overtime and people have character while societies have custom.

Morals are accepted from an authority i.e. culture, religion etc. while ethics are accepted because
they follow from personally accepted principles.

Morals work on a smaller scale than ethics, more reliably but by addressing human needs for
belonging and emulation, while ethics has much wider scope.

An organisation, whether a business or government agency, is first and foremost a human society. If
an employer does not take steps to create a work environment where the employees have a clear,
common understanding of what is right and wrong and feel free to discuss and ask questions about
ethical issues and report violations, significant problems could arise, including:
- increased risk of employees making unethical decisions;
- increased tendency of employees to report violations to out side regulatory authorities (whistle
blowing) because they lack an adequate internal forum;
- inability to recruit and retain top executives;
- diminished reputation in the industry and the community; and
- significant legal exposure and loss of competitive advantage in the market place.

3. How and what will you include while writing for an international audience? 10
Answer:
The following tips below could be used as a guideline while writing for people who have a low level of
English.

Start and finish with courtesy: In many cultures, building a good relationship is more important than any
other part of business communication. So be polite and courteous, especially in openings and closings.

Use short, one-idea sentences: You’re less likely to create a grammatically confusing sentence if it’s short
and has only one idea in it.

Beware of using metaphors: People who speak little English often take metaphors literally and get
confused. Avoid writing ‘The market in your area has a lot of low-hanging fruit’ or ‘We’ve been doing some
blue-sky thinking’.

Prefer one precise word over a few short ones: Prefer one precise word, such as ‘meet’, over two short
words, such as ‘run into’. Those combinations of small words are difficult for international readers to
understand, especially if they take each of the two words literally. ‘The problem should blow over in a
week’ might confuse someone (‘Is it windy?’); whereas ‘The problem should pass in a week’ will work
better.

Avoid double negatives: In some languages, a double negative is a very strong ‘no’. If you write ‘The visit
was not unproductive’, your reader might think the visit was a disaster.
XTQ
(56)

Avoid humor: What our culture finds funny could easily be offensive in another culture. Also, remember
that puns usually mean nothing to someone who is unfamiliar with English.

Use one word to name one thing: Pick one word to name something, and stick with it. For example, if you
say ‘report’, don’t add in ‘paper’, ‘research’, or ‘document’ for elegant variation. You’ll only confuse your
reader.

Write ‘Joanne’ rather than ‘she’ or ‘her’: Use names more than pronouns. Write ‘Please give the report to
Sven’, not ‘Please give it to him’. An international reader could have trouble working out what ‘it’ refers to
and who ‘him’ is.

Write dates in full: Write dates in full to avoid confusion, for example, write 6
February 2011, not 06/02/11 or 02/06/11.

4. How are data analyzed and organized while producing a business report? 10
Answer:
Data are the systematic information collected by the researcher in order to reach a conclusion of the
study. According to the objectives of the study, the tools for collecting data are designed, and the
data are analyzed so as to draw out the appropriate findings. Then a report is written about the
study. Data are analyzed in a number of different ways.
The quantitative data are analyzed by using varieties of statistical tools such as mean, median,
range, standard deviation, co relational coefficient, and so on. Different para-orthographic tools
such as tables, pie charts, bar diagrams, etc. are also used for the analysis and organization of report
data. In the case of qualitative research report, data are more commonly analyzed by description and
interpretation. Examples, ideas, and information are presented in the paragraphs and points. Such
supporting description can also be used in the analysis of quantitative data after the tables, charts, or
figures. Comparison is commonly used in the analysis of quantitative data of a report.
Apart from the technical aspects, there are some attitudinal aspects that a report writer needs to be
aware of. For example, personally biased points should not be incorporated in the analysis of
research data. The interpretation that is made needs to be tested properly, and verified. However,
while analyzing the data, the researcher can put logically critical views on the data which are
collected for a report.
5. Briefly explain any FOUR of the following: (4×2.5=10)
a) Workplace diversity and its challenges
b) Conflict Resolution
c) Encoding and decoding in communication
d) Active Listening Techniques
e) Writing a Successful Proposal
Answer:
a) The reality of the present day work-situation entails that the changing dynamic world has permitted
people with diverse culture, identity and ideology to work together for a grand common goal of
business, trade and industry. Such diversity in the workplace is commonly termed as ‗workplace
diversity‘ in the field of international business communication. The term is also defined in the
reference of the characteristics of cross-cultural communication. Cross cultural campaigns are
conducted in every sector such as business, education, communication, media, and so on. The
increasing diversity of the workplace has posed a challenge to the managers and executive heads.
Messages may be interpreted differently by workers of different gender, racial and cultural
background. Employees may not able to adjust themselves into the diverse cultural situation, and
can be misunderstood. Because of lack of cooperation and support, conflicts between the workers
may also arise. The diverse workplace may lack proper exposure to international communication.
Different companions and programs such as seminars, workshops etc. should be organized to
minimize the challenges of diverse workplace.
XTQ
(57)

b) Conflict arises from differences. It occurs whenever people disagree over their values, motivations,
perceptions, ideas, or desires. Sometimes these differences look trivial, but when a conflict triggers
strong feelings, a deep personal need is at the core of the problem, such as a need to feel safe and
secure, a need to feel respected and valued, or a need for greater closeness and intimacy.

The ability to successfully resolve conflict depends on your ability to:


- Manage stress quickly while remaining alert and calm.
- By staying calm, you can accurately read and interpret verbal and nonverbal
communication.
- Control your emotions and behavior. When you‘re in control of your emotions, you can
communicate your needs without threatening, frightening, or punishing others.
- Pay attention to the feelings being expressed as well as the spoken words of others.
Be aware of and respectful of differences. By avoiding disrespectful words and actions, you can
resolve the problem faster.
c) Business communication involves a number of linguistic and no linguistic elements such as words,
phrases, discourse markers, graphic tools, paralanguage features, instruments and so on which are
used by the participants to give a specific meaning. Nonverbal devices such as gestures and
postures also play remarkably important role in business communication. It is human mind that is
essentially important in making the meaning of what we have used as linguistic or nonlinguistic
device for communication. The mind gives or determines particular meaning of the linguistic or
non-linguistic devices. The sender‘s mind is responsible for encoding the meaning of language
used. With the help of linguistic, socio- cultural, and experiential knowledge, a speaker encodes the
meaning of his or her speech, and intends that the receiver will also decode the meaning in the same
way. To be more specific, encoding is a sender‘s mental process of presenting ideas or information
in oral or written form, using sounds, letters, words, figures or symbols. Decoding is the receiver‘s
mental process or act of assigning the meaning to the words and symbols used by the speakers or
writers. While encoding is concerned with production, decoding is concerned with perception of
discourse meaning in context.
d) Experts say that we listen at only 25 percent efficiency. To make listening more efficient and
effective, some Active Listening Techniques are suggested. While listening to superiors on the job,
take selective notes, avoid interrupting, ask pertinent questions, and paraphrase what you hear.
When listening to colleagues and teammates listen critically to recognize facts and listen
discriminately to identify main ideas and to understand logical arguments. When listening to
customers, defer judgment, pay attention to content rather than form, listen completely, control
emotions, give affirming statements, and invite additional comments. Keys to building powerful
listening skills include controlling external and internal distractions, becoming actively involved,
separating facts from opinions, identifying important facts, refraining from interrupting, asking
clarifying questions, paraphrasing, taking advantage of lag time, taking notes to ensure retention,
and being aware of gender differences.

e) The following guidelines will help you write successful proposals of any type:

 Approach your proposal as a problem-solving activity. Make readers feel that your goal is to
solve a problem for them and that you have the ability to do so.
 Regard your audience as skeptical. Don‘t think readers will automatically accept your
plan as the best way to solve their problem. Expect them to question everything you say.
 Research your proposal thoroughly. You need facts, not generalizations, to persuade your
readers.
 Scout out your competitors. Check your competitors Web sites for information on their
products or services and to get an idea of their costs.
 Prove that your proposal is workable. The bottom-line question from your readers will be
whether your plan will work.

XTQ
(58)

 Be sure your proposal is financially realistic. Another basic question readers will ask is if
your plan is worth the money. Do not submit a proposal that requires excessive funds to
implement.
 Package your proposal attractively. Readers will see your proposal as evidence of the
kind of work you do, so make it look good.

XTQ
(59)

CAP-II, Marketing, Jun-14


Suggested Answer
Roll No……………. Maximum Marks – 50
Total No. of Questions - 5 Total No. of Printed Pages -1
Time Allowed – 1.5 Hours
Marks
All questions are compulsory.

Section -'B'
6. Read the following case carefully and answer the questions given below:
When BMW Car first made a strong competitive push into the U.S. market in the
early 1980s, it positioned the brand as being the only automobile that offered both
luxury and performance. At that time, American luxury cars were seen by many as
lacking performance, and American performance cars were seen as lacking luxury.
By relying on the design of its cars and a well-designed marketing program, BMW
was able to simultaneously achieve (1) a point of difference on performance and a
point of parity on luxury with respect to luxury cars and (2) a point of difference on
luxury and a point of parity on performance with respect to performance cars. The
clever slogan, "The Ultimate Driving Machine", effectively captured the newly
created category-luxury performance cars.

Questions:
a) What were the points-of parity and points-of-difference strategies of BMW for
American market? 5
b) What does the slogan "The Ultimate Driving Machine" mean? 5
Answer:
a)
The points-of-parity strategies of BMW car with American cars include:
 A point of parity on luxury cars and,
 A point of parity on performance with respect to performance cars.
The points-of-difference of BMW car with American cars include:
 A point of difference on performance which the American luxury cars do not
provide, and
 A point of difference on luxury which American performance cars do not
provide.
b) The ultimate driving machine indicates that the BMW car is the best for American
customers and there is no alternative to this car, because, this car provides both luxury as
well as better performance, which the American cars do not provide. The American cars
have only one quality either performance or luxury, they do not provide both of these
qualities.
7.
a) Define marketing. Also explain the societal concept of marketing. (2+3=5)
b) Explain the technological environment of marketing. 5

XTQ
(60)

Answer:
a) In general, marketing activities are associated with identifying the particular wants and
needs of target of customers, and then going about satisfying those customers better than
the competitors. This involves doing marketing research on customers, analyzing their
needs and then making strategic decisions about product design, pricing, promotion, and
distribution. Thus, marketing deals with identifying and meeting human and social needs.
According to Stanton, Etzel & Walker, ―Marketing is a total system of business activities
designed to plan, price promote and distribute want satisfying products to target markets in
order to achieve organizational objectives.‖
From the above definition we can say that customers‘ wants must be recognized and
satisfied. Entire system of business activities should be customer oriented. Marketing
should start with want-satisfying product and should not end until the customers‘ wants
are completely satisfied, which may be some time after the exchange is made.
The societal marketing concept holds that the organization should determine the needs,
wants, and interests of target customers. It should then deliver superior value to costumer
in a way that maintains or improves the customer‘s and the society‘s well-being. The
societal marketing concept questions whether the pure marketing concept is adequate in an
age of environmental problems, resource shortages, rapid population growth, worldwide
economic problems, and neglected social services. It asks if the firm that senses, serves,
and satisfies wants is always doing what‘s best for consumers and society in the long run.
According to the societal marketing concept, the pure marketing concept overlooks
possible conflicts between consumer‘s short term wants and society‘s long term welfare.
Societal marketing concept calls on marketer to balance consumer, organization and
society.

b) Technological environment includes forces that create new technologies, creating new
products and new market opportunities. The technological environment is perhaps the
most dramatic force now shaping our destiny. Technology has released such wonders as
antibiotics, organ transplants, notebook computers and the Internet. It also has released
such horrors as nuclear missiles, chemical weapons and assault rifles. It has also released
such mixed blessings as automobile, television and credit cards. Our attitude toward
technology depends on whether we are more impressed with its wonders or its blunders.
The technological environment changes rapidly. Think of today‘s common products that
were not available 100 years or even 30 years ago, such as Abraham automobiles,
airplanes, radio, electric light, television, automatic dishwashers, air conditioners,
antibiotics, computers, xerography, synthetic detergents, tape recorders, birth control pills,
earth satellites, personal computers and the World Wide Web.
New technologies create new markets and opportunities. However, every new technology
replaces older technology. Cell phone hurt the pager industry, xerography hurt the carbon-
paper business and the compact disks hurt phonograph records. When old industries
fought or ignored new technologies, their business declined. Thus, marketers should watch
the technological environment closely. Companies that do not keep up with technological
change soon will find their products outdated.
Scientists today are researching a wide range of promising new products and
services, ranging from practical solar energy, electric cars, and cancer cures to voice-
controlled computers and genetically engineered food crops. Many companies are adding

XTQ
(61)

marketing people to R&D teams to try to obtain a stronger marketing orientation. The
challenge in each case is not only technical but also commercial.
In developed countries, government agencies investigate and ban potentially
unsafe products. These agencies set safety standards for consumer products and penalizes
companies that fail to meet them. Marketers should be aware of these regulations when
applying new technologies for developing new products.

8.
a) What is marketing segmentation? Explain the basic requirements of meaningful
market segmentation. (2+3=5)
b) What is branding? Show your acquaintance with the terms brand name, brand
mark and trade mark. (2+3=5)
Answer:
a) Market segmentation, is the process of dividing large, heterogeneous markets onto
smaller, homogeneous sub-markets. In other words, market segmentation is the process by
which an organization attempts to match a total marketing program to the needs of one or
more customer groups in the marketplace.
The main objectives of market segmentation are:
 To match the product with the market needs
 To minimize the effects of market competition.
 To assure the sale of product and survive in the market place
 To maximize gains from business
 To assure regular satisfaction of customers.
To achieve these objectives, segmentation must be effective and meaningful. In order to
make market segmentation effective and meaningful, the following conditions must be
fulfilled:
i. There should be differences in customer responses. If the entire customer group
respond in similar way, no segmentation is needed.
ii. Customer differences should be measurable or identifiable so that the marketer can
formulate a suitable marketing program that can address the real needs and
requirements of the target customer.
iii. Segments should be accessible or reachable so that the target customers can benefit
from segmentation effort.
iv. Segments should be Large Enough to create adequate demand for the firm's products.

b) A brand is a name, term, sign, symbol, or design or combination of these, that identifies the
maker or seller of a product or service. It is a name and/or mark intended to identify the
product of one seller and to differentiate the product from competing products. Consumers
view brand as an important part of a product, and branding can add value to a product.
According to Philip Kotler, “Brand is a name, term, sign, symbol or design, or a
combination of these, that is intended to identify the goods and services of one business or
group of businesses and to differentiate them from those of competitors”.
XTQ
(62)

Brand Name

A brand name consists of words, letters, and/or numbers that can be vocalized. For
example, Liril, Lux and Lifebouy are brand name of bathing soaps.

Brand Mark

A brand mark is the part of the brand that appears in the form of a symbol, design, or
distinctive color or lettering. A brand mark is recognized by sight but can not be expressed
when a person pronounces the brand name. Tree used in Dabur products, G>>> used in
Gorakhkali tyre. Are examples of brand marks.

Trade Mark

A trademark is a brand that has been adopted by a seller with legal protection. A trademark
includes not just brand mark but also the brand name. A brand name or brand mark can be
converted into a trademark only when they are legally registered with the concerned
government office.

9.
a) What is physical distribution management? Explain the importance of materials
handling in physical distribution management. (2+3=5)
b) Selling is the tip of marketing iceberg.‖ Comment. 5
Answer:
a) Physical distribution management is concerned with the management of physical flow of
goods from the point of suppliers to the points of purchasers. Stanton defines physical
distribution as 'the activities concerned with the movement of the right amount of the right
products to the right place at the right time'
The main objective of physical distribution is getting the right products safely to the right
place at the right time at the least possible cost.
A physical distribution system consists of a set of interrelated functions which are mentioned
below:
i. Transportation
ii. Warehousing
iii. Inventory Management and Control
iv. Order Processing and
v. Materials handling.
Importance of Materials Handling
Materials handling, also called the physical handling of goods, is an activity that consists
inventory management warehousing, and transportation. The characteristics of a product
determine to a large extent how it will be handled.
A variety of equipment are used to handle wide range of product of various sizes, shapes,
volumes, and weights. The most popular material handling equipments includes trucks,
conveyors, and cranes. Its importance is clear from the following:
i. Proper handling of goods minimizes losses from breakage and spoilage of goods.
ii. It can minimize the cost of distribution.
XTQ
(63)

iii. Goods can be supplied in marketplace as and when needed.


iv. It can minimize the exploitation of natural resources or primary materials through
which goods are manufactured, and thus save ecology of the nation.
b) Many people think of marketing only as selling of goods to end users. And no wonder –
everyday we are bombarded with television commercials, newspaper ads, sales calls,
direct mails etc. Although they are important, they are only a few marketing functions.
Today, marketing must be understood not in the old sense of making a sale by ―telling
and selling‖ but in the new sense of satisfying customer needs and managing profitable
customer relationship. It the marketer does a good job of understanding consumer needs,
develops products that provide superior value, distributes and promotes them effectively,
these products will sell very easily. Selling is only a part of marketing mix and thus we can
say that it is a tip of marketing iceberg. It is believed that, as per the new concept of
marketing, we can lose sales but we cannot lose customers.
Thus, marketing is not only selling, it is a social and managerial process by which
individuals and groups obtain what they need and want through creating and exchanging
products and value with others.
10. Briefly explain the following: (5×2=10)
a) Core Product
b) Perceived-value pricing
c) Internal marketing
d) E-commerce
e) Promotion scenario in Nepal
Answer:
a) Core Product is the service or benefit the customer is really buying. The core product
stands at the centre of the total product. It consists of the core, problem solving benefits
that customer seeks when they buy a product or service. The most basic level is core
product, which addresses the question What is the buyer really buying? A hotel guest is
buying rest and sleep. A girl buying cosmetics is buying beauty. Marketer must see
themselves as benefit providers.

b) Perceived-value pricing is a method of setting price for the product at that level at which
the majority of customers become ready to pay. Under this method of pricing, companies
see the buyer's perception of value as the key to pricing. They attempt to measure the
relative perceived value of their offer and utilize this in setting the price.
In this case, the customer decides „how much he should pay for the product to get that
product in relation to the benefit provided by the product‟'
The process of setting perceived-value price involves the following steps:
i. Identify product features, performance, and company's services,
ii. Decide on the alternative price levels for choice along with the product features,
iii. Conduct market survey to indentify the customers' response on each product
feature, performance, and company's service,
iv. Rank their emphasis on the basis of the value they placed,
XTQ
(64)

v. Identify the price they wish to pay for the company's product or service. This price
is called as perceived-value price.
The key to perceived-value pricing is to make an accurate determination of the
customers' perception of the value of the product.
c) Internal marketing is the task of hiring, training, and motivating able employees who want
to serve customers well. Smart marketers recognize that marketing activities within the
company can be as important as marketing activities directed outside the company. It
makes no sense to promise excellent service before the company‘s staff is ready to provide
it.
Internal marketing must take place on two levels. At one level, the various marketing
functions – sales force, advertising, customer service, product management, marketing
research – must work together. At another level, marketing must be embraced by the other
departments, who must also ―think customer.‖

d) E-commerce is establishing exchange relationships electronically through email, internet


and electronic platforms to satisfy individual needs of customers. It is direct marketing
based on electronic communication.
e) Various forms of promotion are used in Nepal inform the customers about goods and
services. Among several promotion tools, personal selling and advertising are the
traditional tools used by Nepalese traders form the time immemorial.
These days, we have several types of promotion tools in Nepal including print Media,
radio, television, and FM channels.
Print advertising began in 1949 B.S., radio advertising began in 2007 B.S., television
advertising began in 2041 B.S., and advertising through FM channels began in 2051 B.S.
In Nepal, most of the business communities use advertising, sales promotion and
personnel selling tools covering almost all types of products including fast foods, soap,
electronic and electric goods, bank and insurance services, games, readymade garments,
tourist service packages etc.

XTQ
(65)

CAP – II, Cost Management – June, 2014


Suggested Answer
Roll No……………. Maximum Marks - 100
Total No. of Questions: 6 Total No. of Printed Pages - 5
Time Allowed - 3 Hours
Marks
All questions are compulsory. Working notes should form part of the answer.
Make assumptions wherever necessary.
1. Citizen Ltd. manufactures three products X, Y and Z. Standard selling prices and
costs per unit have been established for the year 2071 as follows:
(Amount in Rs.)
X Y Z
Selling price 280 600 1,250
Direct materials 80 150 200
Direct wages 100 200 500
Variable overheads 50 100 250
Direct wages are paid at the rate of Rs. 20 per hour in each case. Fixed overheads
are budgeted at Rs. 250,000 for the coming year. In short run, the company cannot
increase its direct labour strength and as a result, only 35,000 direct labour hours
will be available in the coming year. The company has commitments to produce 500
units of each product. It has been suggested that after meeting the minimum
requirements for X, Y and Z, the balance of available direct labour hours should be
used to produce product Z.
Required: (6+6+3+5=20)
a) Prepare an income statement showing the expected results if the proposal is
adopted.
b) Comment on the statement you have produced in (a) above and prepare an
income statement for any alternative policy which you consider would be more
profitable.
c) Based on your calculations in (b) above, show the company's break-even points
in terms of units and sales value.
d) Show the sales value which is required to produce an after tax return of 10% on
capital employed of Rs. 900,000 assuming tax rate of 50%.
Answer:
a) Statement showing the Income of Citizen Limited for the year 2071
X Y Z
Selling price per unit (Rs.) 280 600 1,250
Variable cost per unit: (Rs.)
Direct materials 80 150 200
Direct wages 100 200 500
Variable overheads 50 100 250
Total variable costs (Rs.) 230 450 950
Contribution margin per unit (Rs.) 50 150 300
Units produced 500 500 1,100
Total contribution (Rs.) 25,000 75,000 330,000

Total contribution (Rs.) 430,000


Less: Fixed costs (Rs.) 250,000

XTQ
(66)

Net Profit (Rs.) 180,000

Working Notes:
Total direct labour hours available 35,000
Labour hours used for 500 units of X [Rs. 100 / Rs. 20 × 500] 2,500
Labour hours used for 500 units of Y [Rs. 200 / Rs. 20 × 500] 5,000
Labour hours used for 500 units of Z [Rs. 500 / Rs. 20 × 500] 12,500 20,000
Remaining direct labour hours 15,000

As given in the question, the remaining 15,000 direct labour hours as calculated above
will be used to produce product Z. Therefore, the units of product Z produced will be
as follows:
Minimum committed units 500
Units produced from remaining labour hours [15,000 / 25] 600
Total units 1,100

b) In the given question the limiting factor is direct labour hours. So, the remaining
15,000 direct labour hours should be used for the product which gives maximum
contribution per direct labour hour. The contribution per direct labour hour is
calculated as follows:
X Y Z
Contribution margin per unit as above (Rs.) 50 150 300
Direct labour hours per unit (Wages / Rs. 20) 5 10 25
Contribution margin per direct labour hour (Rs.) 10 15 12
Ranking III I II

As product Y gives the maximum contribution per direct labour hour, the remaining
15,000 direct labour hours should be utilized to produce product Y. Therefore, the
production of product Y will be as follows:

Minimum committed units 500


Units produced from remaining labour hours [15,000 / 10] 1,500
Total units 2,000

Statement of income of Citizen Limited under alternative policy for the year 2071
X Y Z
Contribution margin per unit as per (a) above (Rs.) 50 150 300
Units produced 500 2,000 500
Total contribution (Rs.) 25,000 300,000 150,000

Total contribution (Rs.) 475,000


Less: Fixed costs (Rs.) 250,000

XTQ
(67)

Net Profit (Rs.) 225,000

c) Citizen Ltd's break-even point in terms of sales units and value:


It is given in the question that the company has to produce 500 units of each product. At
break-even point, the contribution is just sufficient to meet the fixed costs. The contribution
from the minimum committed units of three products is as follows:
X Y Z
Contribution margin per unit as per (a) above (Rs.) 50 150 300
Units produced 500 500 500
Total contribution (Rs.) 25,000 75,000 150,000

Total contribution from minimum committed units is Rs. 250,000 and fixed costs of the
company is also Rs. 250,000. Therefore, the break-even point in terms of sales unit is 500
units of each product. Similarly break-even point in terms of sales value is [(Rs. 280 × 500)
+ (Rs. 600 × 500) + (Rs. 1,250 × 500)] = Rs. 1,065,000

OR,

Can be calculated through overall PV ratio as below

PVR=

=0.234742

Overall BEP sales =

BEP unit product of each product

d) Computation of sales value to produce an after tax return of 10% on capital employed:
Rs.
Capital employed 900,000
Required return after tax @10% 90,000
Tax @ 50% 90,000
Total required return 180,000
As per (c) above, the company has to produce 500 units of each product to break-even. The
remaining 15,000 direct labour hours must be utilized to produce product Y, which gives
highest contribution per labour hour. Therefore, for finding the sales value to produce a

XTQ
(68)

10% return on capital employed, the gross return should be divided by contribution per unit
of product Y as follows:
Desired gross return (Rs.) 180,000
Contribution margin per unit of Y (Rs.) 150
Number of additional units to be produced 1,200
Now, total sales value will be:
Rs.
Product X : 500 units @ Rs. 280 140,000
Product Y : 1,700 units @ Rs. 600 1,020,000
Product Z : 500 units @ Rs. 1,250 625,000
Total 1,785,000
2.
a) Nits Ltd. specializes in seasonal novelty products and is considering the
manufacture of a new range of items to coincide with a major sporting event of
Cricket in the city. The range will initially comprise of 2 products, Flags and
Bunting. To assist with budgeting, Nits Ltd has collected the following projected
information for the month of July 2014:
Projected sales: Quantity Sales revenue per item (Rs.)
Flags 4,000 18
Bunting 2,000 50

Production requirements: Cost per meter Flags Bunting


Material A Rs. 4.00 0.5 m 4m
Material B Rs. 2.00 1.0 m 3m
Finished goods inventory: Flags Bunting
1st July 200 -
31st July 950 1,325

There is no opening or closing work in progress, however due to inefficiencies in


the production process, management expect that 5% of output will not pass
quality control and therefore cannot be sold.
Materials inventory: A B
1st July 6,000 m 20,000 m
31st July 10,200 m 14,000 m
(m denotes the measurement of cloths in meter)
Labour & Overhead:
The standard direct labour required to produce each Flag unit is 30 minutes and a
Bunting unit takes 1 hour to produce. Labour is paid at Rs. 10 per hour. Variable
overheads (which will be incurred evenly over the year) are projected at
Rs. 360,000 per annum and these are to be absorbed into production on the basis
of direct labour hours.
Prepare the following Budget Statements: (2+3+3+2+2 =12)
i) Sales Budget
ii) Production Budget
iii) Material Purchasing Budget
iv) Labour Budget
v) Overhead Absorption Budget

XTQ
(69)

b) Dokomo makes and sells a single product, Product "X". It is currently producing
112,000 units per month, and is operating at 80% of full capacity. Total monthly
costs at the current level of capacity are Rs. 611,000. At 100% capacity, total
monthly costs would be Rs. 695,000. Fixed costs would be the same per month at
all levels of capacity between 80% and 100%. At the normal selling price for
Product X, the contribution/sales ratio is 60%. A new customer has offered to
buy 25,000 units of Product X each month, at 20% below the normal selling
price. Dokomo estimates that for every five units that it sells to this customer, it
will lose one unit of its current monthly sales to other customers.
Required: (2+2+4=8)
i) Calculate the variable cost per unit of Product X and total fixed costs per
month.
ii) Calculate the current normal sales price per unit, and the contribution per unit
at this price.
iii) Calculate the effect on total profit each month of accepting the new
customer‘s offer, and selling 25,000 units per month to this customer.
Recommend whether the customer‘s offer should be accepted.
Answer:
a)
i) Sales Budget:
Particulars Flags Bunting Total
Sales units 4,000 2,000 -
Sales price per unit (Rs.) 18 50 -
Sales value (Rs.) 72,000 100,000 172,000

ii) Production Budget (units):


Particulars Flags Bunting
Sales 4,000 2,000
Add: Closing stock 950 1,325
4,950 3,325
Less: Opening stock 200 -
Net production 4,750 3,325
Normal loss (%) 5 5
Total production (Net production / 95%) 5,000 3,500

iii) Materials Purchasing Budget:


Particulars Materials Total
A (in meters) B (in meters)
Flag (5,000 units) 2,500 5,000 -
Bunting (3,500 units) 14,000 10,500 -
16,500 15,500 -
Add: Closing stock 10,200 14,000 -
26,700 29,500 -
Less: Opening stock 6,000 20,000 -
Purchases 20,700 9,500 -
Price per meter Rs. 4.00 Rs. 2.00 -
XTQ
(70)

Total purchases Rs. 82,800 Rs. 19,000 Rs. 101,800

iv) Labor Budget:


Particulars Flags Bunting Total
Production units 5,000 3,500 -
Labor hour per unit 0.5 1.0 -
Total labor hour 2,500 3,500 -
Rate per hour Rs. 10 Rs. 10 -
Total labor cost Rs. 25,000 Rs. 35,000 Rs. 60,000

v) Variable Overhead Absorption Budget:


Particulars Amt. in Rs.
Variable overheads for the year 360,000
Variable overheads for the month 30,000
Direct labor hour (2,500 + 3,500) 6,000 hour
Overhead rate per hour 5
Overhead absorption:
Flags (2,500 × Rs. 5) 12,500
Bunting (3,500 × Rs. 5) 17,500

XTQ
(71)

b)
i) Units that can be produced per month at 100% capacity level are 112,000 / 0.80 =
140,000 units.
High Low Difference
Units produced 140,000 112,000 28,000
Total costs (Rs.) 695,000 611,000 84,000
Variable cost per unit = Rs. 84,000 / 28,000 units = Rs. 3.00

Computation of Fixed Cost:


112,000 units 140,000 units
Total costs (Rs.) 611,000 695,000
Variable costs @ Rs. 3 per unit 336,000 420,000
Fixed costs per month (Rs.) 275,000 275,000

Note: Calculation of fixed cost for any one activity level is good enough to award
full marks.

ii) Contribution/sales ratio = 60%


Therefore, variable cost/sales ratio = 40%
The normal sales price per unit = Rs. 3 / 0.40 = Rs. 7.50
The contribution per unit at the normal selling price is Rs. 7.50 – Rs. 3.00 = Rs. 4.50

iii) If the customer‘s offer is accepted, the sales price for the 25,000 units will be
Rs. 7.50 × 80% = Rs. 6 per unit.
The contribution per unit for these units will be Rs. 6 – Rs. 3 = Rs. 3
The reduction in monthly sales at the normal price will be 1/5 × 25,000 = 5,000
units.
Therefore,
Increase in contribution from 25,000 units sold (25,000 × Rs. 3) Rs. 75,000
Loss of contribution from fall in other sales (5,000 × Rs. 4.50) Rs. 22,500
Net increase in profit each month Rs. 52,500
By accepting the new customer‘s offer, the profit would increase by Rs. 52,500 each
month. The offer should therefore, be accepted.
3.
a) Super Builder Constructions are engaged in building contracts. One of their
contracts commenced 15 months ago remains unfinished. The following
information relating to the contract has been prepared for the year just ended:
Particulars Rs. ‘000
Contract price 2,750
Value of work certified at the end of the year 2,420
Cost of work not yet certified at the end of year 44
Opening balances:
Cost of work completed 330
Materials on site (physical stock) 11
During the year:
Materials delivered to site 671
Wages 638
Hire of plant 121

XTQ
(72)

Other expenses 99
Closing balance : Materials on site (physical stock) 22
As soon as materials are delivered to the site, they are charged to the contract
account. A record is also kept of materials as they are actually used on the
contract. Periodically a stock check is made and any discrepancy between book
stock and physical stock is transferred to a general ―Contract Material
Discrepancy‖ account. This is absorbed back to each contract, currently at the
rate of 0.5% of materials booked. The stock check at the year-end revealed a
stock shortage of Rs. 5,500.
In addition to direct charges listed above, general overhead are charged to
contracts at 5% of value of work certified. General overhead of Rs. 16,500 had
been absorbed into the cost of work completed at the beginning of the year. It has
been estimated that further costs to compete the contract will be Rs. 242,000.
This estimate included the cost of materials on site at the end of the year just
finished and also a provision for rectification.
Required: (8+2=10)

i) Determine the profitability of the above contract and recommend how much
profit should be taken for the year just ended 2013.
ii) State how your recommendation in above would be affected if the Contract
Price was Rs. 44 lakhs (rather than Rs. 27.50 lakhs) and if no estimate has
been made of costs to completion. Assume retention money = 20%.

b) From the details furnished below you are required to compute a comprehensive
machine hours rate: 10
Original purchase price of the machine Rs. 324,000
(Subject to depreciation at 10% p.a. on original cost)
Normal working hours for the month 200 hours
(The machine works to only 75% of capacity)
Wages of machine man Rs. 125 per day (of 8 hours)
Wages for a helper (machine attendant) Rs. 75 per day (of 8
hours)
Power cost for the month for the time worked Rs. 15,000
Supervision charges apportioned for the machine centre
for the month Rs. 3,000
Electricity & lighting for the month Rs. 7,500
Repairs & maintenance (machine) including consumable
stores per month Rs. 17,500
Insurance of plant & building (apportioned) for the year Rs. 16,250
Other general expenses per annum Rs. 27,500

The workers are paid a fixed dearness allowance of Rs. 1,575 per month.
Production bonus payable to workers in terms of an award is equal to 33.33% of
basic wages and dearness allowance. Add 10% of the basic wages and dearness
allowance against leave wages and holiday with pay to arrive at a comprehensive
labour wage for debit to production.

XTQ
(73)

Answer:
a)
i) Super Builder Constructions
Contract Account for the year ended 2013
Particulars Rs.'000 Particulars Rs.'000
To Work-in-progress b/d 330.00 By Work-in-progress a/c:
To Materials b/d 11.00 Work certified 2,420.00
To Materials issued 671.00 Work uncertified 44.00
To Wages 638.00 By Material discrepancy A/c 5.50
To Plant hire charges 121.00 By Materials on site 22.00
To Other expenses 99.00
To Material discrepancy A/c 3.30
[Rs. (11 + 671 - 22) × 0.5%]
To General overheads 104.50
(Rs. 2,420 × 5% – Rs. 16.50)
To Notional profit c/d 513.70
2,491.50 2,491.50
To Profit and Loss A/c – transfer 490.78 By Notional profit b/d 513.70
To Reserve c/d 22.92
513.70 513.70

Working notes (Amount in '000):


(1) Calculation of estimated profit:
Cost incurred to date = Rs. (2,491.50 – 513.70 – 5.50 – 22.00)
= Rs. 1,950.30
Estimated total costs = Cost to date + Further estimated costs
= Rs. 1,950.30 + Rs. 242.00
= Rs. 2,192.30
Estimated total profit = Contract Price – Estimated total costs
= Rs. 2,750.00 – Rs. 2,192.30
= Rs. 557.70
(2) Profit to be taken to profit & loss A/c:
Percentage of completion = (Work certified / Contract price) × 100
= (Rs. 2,420 / Rs. 2,750) × 100
= 88%
Profit to be recognized = Estimated profit × Work certified / Contract price
= Rs. 557.70 × Rs. 2,420 / Rs. 2,750
= Rs. 490.78

ii) If contract price were Rs. 44 lakhs and no cost estimation has been made:
Percentage of completion = (Rs. 2,420 / Rs. 4,400) ×100 = 55%
Therefore, profit recognition will be based on notional profit
Cash received = Rs. 2,420 × 80% = Rs. 1,936
Profit to be recognized = 2 / 3 × Notional profit × Cash received / Work certified
= 2 / 3 × Rs. 513.70 × Rs. 1,936 / Rs. 2,420
= Rs. 273.97
XTQ
(74)

b) Computation of comprehensive machine hour rate


Per month Per hour
Rs. Rs.
Fixed cost:
Supervision charges 3,000.00
Electricity and lighting 7,500.00
Insurance of plant & building (Rs. 16,250 × 1 / 12) 1,354.17
Other general expenses (Rs. 27,500 × 1 / 12) 2,291.67
Depreciation (Rs. 32,400 × 1 / 12) 2,700.00
16,845.84 112.31
Variable cost:
Repairs & maintenance 17,500.00 116.67
Power 15,000.00 100.00
Wages of machine man 6,737.00 44.91
Wages of helper 4,945.00 32.97
Machine hour rate (comprehensive) 406.86

Effective machine working hours per month = 200 hours × 75% = 150 hrs.
Working note:
Wages for machine man and helper
Machine man Helper
Rs. Rs.
Wages for 200 hours (Rs. 125 × 25) 3,125 -
(Rs. 75 × 25) - 1,875
Dearness allowances 1,575 1,575
Total of wages & dearness allowances 4,700 3,450
Production bonus (1/3 of above) 1,567 1,150
6,267 4,600
Leave wages – 10% of wages & dearness allowances 470 345
Total wages 6,737 4,945
Wages rate per machine hour (total 150 hrs) 44.91 32.97
4.
a) C Ltd. operates a process which produces three joint products. In the period just
ended costs of production totaled Rs. 509,640. Output from the process during
the period was:
Product - W 276,000 kilos
Product - X 334,000 kilos
Product - Y 134,000 kilos
There were no opening stocks of the three products. Products W and X are sold
in this state. Product Y is subjected to further processing. Sales of products W
and X during the period was:
Product - W 255,000 kilos at Rs. 0.945 per kilo
Product - X 312,000 kilos at Rs. 0.890 per kilo
128,000 kilos of product Y were further processed during the period. The
balance of the period production of the three products W, X and Y remained in
stock at the end of the period. The value of closing stock of individual products
is calculated by apportioning costs according to weight of output.
The additional costs in the period of further processing product Y, which is
converted into product Z were:
XTQ
(75)

Direct labour Rs. 10,850


Production overhead Rs. 7,070
96,000 kilos of product Z were produced from the 128,000 kilos of product Y. A
by-product BP is also produced which can be sold for Rs. 0.12 per kilo. 8,000
kilos of BP were produced and sold during the period.
Sales of product Z during the period were 94,000 kilos, with total revenue of
Rs. 100,110. Opening stock of product Z was 8,000 kilos, valued at Rs. 8,640.
The FIFO method is used for pricing transfers of product Z to cost of sales.
Selling and administration costs are charged to all main products when sold, at
10% of revenue.
Required: (12+3=15)
i) Prepare a profit and loss account for the period, identifying separately the
profitability of each of the three main products (W, X and Z).
ii) C Ltd has now received an offer from another company to purchase the total
output of product Y (i.e. before further processing) for Rs. 0.62 per kilo.
Calculate the viability of this alternative.
b) Explain Economic Ordering Quantity. What are the implications of Economic
Order Quantity in proper inventory management? 5

Answer:
a)
i)
Profit and loss account
Product W Product X Product Z Total
Rs. Rs. Rs. Rs.
Opening stock - - 8,640 8,640
Production cost 189,060 228,790 108,750 526,600
Closing stock (14,385) (15,070) (15,010) (44,465)
Cost of sales 174,675 213,720 102,380 490,775
Selling and administration costs 24,098 27,768 10,011 61,877
Total costs 198,773 241 488 112,391 552,652
Sales 240,975 277,680 100,110 618,765
Profit / (loss) 42,202 36,192 (12,281) 66,113

Workings notes:
(1) Joint process cost per kilo of output = (Rs. 509,640 / 744,000) = Rs. 0.685
(2) Production cost for products W, X and Y:
Rs.
Product W (276,000 kg × Rs. 0.685) 189,060
Product X (334,000 kg × Rs. 0.685) 228,790
Product Y (134,000 kg × Rs. 0.685) 91,790
(3) Closing stocks of products W and X:
Rs.
Product W (21,000 kg × Rs. 0.685) 14,385
Product X (22,000 kg × Rs. 0.685) 15,070
(4) Cost per kilo of product Z:
Rs.
Product Y (128,000 kg × Rs. 0.685) 87,680
Further processing costs 17,920
XTQ
(76)

Less: by-product sales (8,000 kg × Rs. 0.12) 960


104,640

Cost per kilo (Rs.104,640 / 96,000) Rs. 1.09


(5) Closing stock relating to product Z:
Rs.
Closing stock of product Z (10,000 kg × Rs. 1.09) 10,900
Add closing stock of input Y (6,000 kg × Rs. 0.685) 4,110
Closing stock relating to product Z 15,010
(6) Production cost relating to final product Z:
Rs.
Product Y (134,000 kg × Rs. 0.685) 91,790
Further processing costs 17,920
Less: by-product sales 960
108,750
(7) Sales revenue of product W and X:
Product W: 255,000 kg @ Rs. 0.945 per kg Rs. 240,975
Product X: 312,000 kg @ Rs. 0.890 per kg Rs. 277,680

ii) The joint costs are common and unavoidable to both alternatives, and are therefore
not relevant for the decision under consideration. Further processing from an input
of 128,000 kg of Y has resulted in an output of 96,000 kg of Z. Thus it requires 1.33
kg of Y to produce 1 kg of Z (128,000 / 96,000)
Rs.
Revenue per kilo for product Z (Rs.100,110 / 94,000) 1.065
Sale proceeds at split-off point (1.33 × Rs. 0.62) 0.823
Incremental revenue per kg from further processing 0.242
Incremental costs of further processing [(Rs. 17,920 – Rs. 960) / 96 000] 0.177
Incremental profit from further processing 0.065

It is assumed that selling and administration costs are fixed and will be unaffected
by which alternative is selected. The company should therefore process Y further
into product Z and not accept the offer from the other company to purchase the
entire output of product Y.

b) Economic Order Quantity (EOQ): Purchase department in manufacturing concerns


is usually faced with the problem of deciding the ‗quantity of various items‘ which
they should purchase. If purchases of material are made in bulk then inventory
carrying cost will be high. On the other hand, if order size is small each time, then
the ordering cost will be high. In order to minimize ordering and carrying costs it is
necessary to determine the order quantity, which minimizes these two costs. The
size of the order for which both ordering and carrying costs are minimum is known
as economic order quantity.
The prime objective of inventory management is to find out and maintain optimum
level of investment in inventory to minimize the total costs associated with it.
Economic Order Quantity is the size of the order for which both ordering and
carrying costs are minimum. Economic Order Quantity forms the very basis of
inventory management. It refers to the size of each purchase order quantity for each
item, which gives the maximum economy in purchase of that raw material or
finished goods or stores materials.
While placing any order for purchase of any item, it must be ensured that the order
quantity is neither too large nor too small. A large order, no doubt, shall also mean
XTQ
(77)

the lower ordering cost but it shall mean a higher and sometimes prohibitive
carrying costs. On the other hand, a small order may reduce the inventory carrying
cost but the ordering costs would increase as the company may have to place a new
order every now and then, besides, it may result in occasional production halts also.

Therefore, a proper balance has to be struck between these two factors and the
Economic Order Quantity shall be fixed at a point, where the aggregate cost of the
two is minimum i.e., the total cost associated with the inventory management is
minimum.
5. Distinguish between: (4×2.5=10)
a) Cost Audit and Statutory Audit
b) Abnormal Loss and Abnormal Gain in Process Costing
c) Job Costing and Contract Costing
d) Cost Control and Cost Reduction

Answer:
a) Cost Audit and Statutory Audit
Cost audit offers valuable assistance to the management in its decision-making
process by examining the reliability of cost accounting data and information. Due
to the assistance provided by cost audit, management is in a position to know
what price is to be fixed for a product, whether the wastages are avoidable
whether to reorganize sales or inventory system to make work more efficient and
effective. Cost audit, apart from having all the normal ingredients of audits i.e.
vouching, verification etc has been within its domain elements of efficiency audit
and proprietary audit.
Statutory audit is the examination of books and records to form an opinion on
whether profit & loss account and balance sheet of a company or business
concern provides a true and fair view of profits (in the relevant financial period)
and financial position on a particular date. Statutory audit is related with only
historical figures and data. It is performed after the expenditures have been
incurred and accounts have been prepared.
b) Abnormal Loss and Abnormal Gain in Process Costing
Abnormal process loss is defined as the loss in excess of the pre-determined loss.
This type of loss may occur due to the carelessness of workers, a bad plant
design or operation etc. The cost of an abnormal process loss unit is equal to the
cost of a good unit. The total cost of abnormal process loss is credited to the
process account from which is arises and it is debited to costing profit and loss
account.
Cost of abnormal loss = {(Total cost - Value of normal loss) ÷ Normal output (in
units)} × Abnormal losses units

Abnormal gain is the reverse of the abnormal loss. Sometimes, loss under a
process is less than the anticipated normal figure. Under such a situation the
difference between actual and expected loss is known as abnormal gain. The
process account under which abnormal gain arises is debited with the abnormal
gain and it is credited to costing profit and loss account. The amount of abnormal
gain is computed based on cost of each good unit.
c) Job costing and contract costing
In job costing, job work is carried in the premises. An order, a unit, lot or batch
of product may be taken as cost unit. Cost is first allocated to cost centers and
XTQ
(78)

then charged to individual jobs. It is a system of costing in which the elements of


cost are accumulated separately for each job or work undertaken by an
organization. The prices of the jobs are fixed based on the nature of costs and
policy of the firm.
In contract costing, contract work is carried on at site. Each contract is cost unit.
Most of the expenses are of direct nature and are directly charged to respective
contract accounts. Only general overheads and head office expenses are
apportioned to individual contracts. The pricing is generally through bidding and
external forces have major influence in fixing the offer price.
d) Cost Control and Cost Reduction
Cost control is operated through setting standards of targets and comparing actual
performance therewith with a view to identifying deviations from standards or
norms and taking corrective action in order to ensure that future performance
conforms to standards or norms. It is a preventive action.
Cost reduction, on the other hand, is a continuous process of critical cost
examination, analysis and challenge of standards. It is a corrective action. Each
aspect of the business viz. products, process, procedures, methods, organization,
personnel, etc is critically examined and reviewed with a view to improving the
efficiency and effectiveness and reducing costs. Even in an organization where
efficient cost control is in operation, there is always room for cost reduction. In
cost reduction there is a permanent reduction in per unit cost where such
permanency is lacking in cost control.
Cost control lacks the dynamic approach, which planned cost reduction demands.
In a cost reduction plan, standards, which are the basis of cost control, are
constantly challenged for improvement.
Budgetary control and standard costing are essential tools and techniques of cost
control. There are several distinct tools and techniques of cost reduction such as
value engineering, work-study, standardization, simplification, variety reduction,
quality measurement and research, operations research, market research, job
evaluation, merit awards, incentives, improvement in design, mechanization,
automation etc.
6. Answer the following questions: (4×2.5=10)
a) What do you mean by perpetual inventory system? State its advantages.
b) Discuss briefly the treatment of overtime in cost accounting.
c) Bonus paid under the Halsey Plan with bonus at 50% for the time saved equals
the bonus paid under the Rowan System. When will this statement hold good?
Your answer should contain the proof.
d) Explain the concept of key factor.

Answer:
a) Perpetual Inventory System means continuous stock taking. CIMA defines
Perpetual Inventory System as ‗the recording as they occur of receipts, issues and
resulting balances of individual items of stock in either quantity or quantity and
value‘.
Under this system, a continuous record of receipt and issue of materials is
maintained by the stores department & the information about the stock of
materials is always available. Entries in the bin card and stores ledger are made
after every receipt and issue and the balance is reconciled on regular basis with
the physical stock.

XTQ
(79)

The main advantages are:


i) Physical stock can be counted and book balances adjusted as and when
desired without waiting for the entire stock taking to be done.
ii) Quick compilation of profit and loss account (for interim period) due to
prompt availability of stock figures.
iii) Discrepancies are easily located and thus corrective action can be promptly
taken to avoid the recurrence.
iv) A systematic review of perpetual inventory reveals the existence of surplus,
dormant, obsolete and slow-moving materials, so that remedial measures may
be taken in time.
v) Fixation of various stock level and checking of actual balances in hand with
these levels assist the storekeeper in maintaining stocks within limits and in
initiating purchase requisitions for correct quantity at proper time.
b) If overtime is restored to at the desire of the customer, then overtime premium
may be charged to the job directly. If overtime is required to cope with general
production program or for meeting urgent orders, the overtime premium should
be treated as overhead cost of the particular department or cost center, which
works overtime.
If overtime is worked in a department due to the fault of another department, the
overtime premium should be charged to the latter department‘s cost.
Overtime worked on account of abnormal conditions such as flood, earthquake
etc. should not be charged to cost but to costing profit and loss account.
c) Bonus under Halsey Plan:

 Time saved ............... i 


50
S tan dard rate 
100
Bonus under Rowan Plan:

 Time saved ................ ii 


Time taken
S tan dard rate 
Time allowed
Now,
50 Time taken
S tan dard rate   Time saved = S tan dard rate   Time saved
100 Time allowed
50 Time taken
Or, 
100 Time allowed
Or, Time taken = 50% of time allowed.
Therefore, when the time taken to perform a job is 50% of the time allowed, the
bonus under Halsey and Rowan Plan is equal.
d) Under marginal costing, profitability is ascertained as aggregate of contribution
from all products sold. With an objective to maximize profit, those products
which yield highest contribution are produced/sold in maximum quantities. It is
generally assumed that there will be no limitation which may create restriction in
increasing quantities of one or more products. But in practice, there may be
number of factors which may create limitations. These may be shortage of
material, labor, plant capacity or sales. These are called key factors or limiting
factors. Hence, this will prevail:
Contribution per unit = contribution/key factor

XTQ
(80)

CAP-II
Income Tax & VAT
Final-Suggested Answer
Roll No……………. Maximum Marks - 100
Total No. of Questions - 6 Total No. of Printed Pages -5
Time Allowed - 3 Hours
Marks
Attempt all questions. Working note should form part of the answer.
8. Answer the following with reference to the Indian Income Tax Act, 1961.
a) Define the term "assesses" under section 2(7) of the Indian Income Tax Act? 2
b) Mr. Frank, a France National, visit India on 12.11.2012 and stays up to 20-12-
2013 before leaving for France. During 2010-11 he stayed in India for 195 days
and in 2009-10 for 95 days. Prior to 2009-10 he never visited India. Determine
his residential status for the assessment year 2014-15. 5
c) Are there any capital gains derived by a charitable trust are not chargeable to tax
under section 11(1A) of Income Tax Act, 1961? Discuss. 3
Answer a)As per section 2(7) of Indian Tax Act, 1961, "assessee" means a person by whom any tax or any
other sum of money is payable under this Act, and includes:
(a) every person in respect of whom any proceeding under this Act has been taken for the
assessment of his income or assessment of fringe benefits or of the income of any other person
in respect of which he is assessable, or of the loss sustained by him or by such other person, or
of the amount of refund due to him or to such other person;
(b) Every person who is deemed to be an assessee under any provision of this Act;
(c) Every person who is deemed to be an assessee in default under any provision
of this Act;
b) An individual is said to be resident in India in any in any previous year, if he satisfies at least:
i) He has been India during the previous year for a period of 182 days or more or
ii) He has been India for a period of 60 days or more during the previous year and 365 days or
more during 4 years immediately preceding the previous year.
Further, a resident individual is said to be "not ordinarily resident" in India in any previous year
if such person is
(a) He has been a non-resident in India in 9 out of the 10 previous years preceding that year, or
(b) He has during the seven previous years preceding the relevant previous year been in India
for a period of 729 days or less.
As per the information, Mr. Frank is in India as follows:
Year Presence in India Remarks
(Days)
2009-10 95
2010-11 195 Resident during this year
2011-12 0
2012-13 140 Nov.12, 2012 to 31 March, 2013
2013-14 264 1stApril, 2013 to 20th
December,2013
Total 694 days

Mr. Frank is in India in the previous year of 264 days. He is resident for the
assessment year 2014-15. From the above presence days in India of Mr. Frank,

XTQ
(81)

he satisfies the conditions of not ordinarily resident as mention above. So, He is


resident but not ordinarily resident in that assessment year.

Note: Alternatively, if the answer is given defining ―resident and ordinary


resident‖ and conclusion drawn.
c) In some situations, capital gain derived by a charitable trust is not chargeable to tax under
section 11 (1A), the situations is :
Where a capital asset, being property held under trust wholly for charitable or religious
purposes, is transferred and the whole or any part of the net consideration is utilised for
acquiring another capital asset to be so held, then, the capital gain arising from the transfer shall
be deemed to have been applied to charitable or religious purposes to the extent specified
hereunder, namely:—
(i) Where the whole of the net consideration is utilised in acquiring the new capital asset, the
whole of such capital gain;
(ii) where only a part of the net consideration is utilised for acquiring the new capital asset, so
much of such capital gain as is equal to the amount, if any, by which the amount so utilised
exceeds the cost of the transferred asset;
9.
a) RamhariRice Mill, a complete manufacturing industry has the following
transaction during the Income Year 2069/70. The Mill is a special industry under
section 11 of the Income Tax Act, 2058. The Mill is the sole proprietorship firm
and the owner of the Mill has not selected the section 50 of the income Tax Act,
2058. You are required to calculate the applicable tax rate and compute the tax
liability based on the following information. 6
Local sales Rs. 11,250,000
Export Rs. 3,750,000
Cost of sales Rs. 9,000,000
Indirect cost Rs. 500,000
b) Mr. Sailesh, a disabled person, is working in Dailekh branch of Nabil Bank Ltd.
In Fiscal Year 2069/70, following are the transactions;
i) Salary and allowances Rs. 50,000 per month; Dashain allowance Rs. 20,000;
Bonus Rs. 40,000.
ii) Bank has managed to contribute to Provident Fund (approved) Rs. 24,000
from employer as well as employee side.
iii) In addition to the remuneration received from Bank, Mr. Sailesh has also
received Rs. 150,000 from Nepal Government in the form of Pension.
iv) He has covered the insurance and paid the insurance premium of Rs. 25,000
during Fiscal Year 2069/70.
v) He has declared the couple under section 50, in Fiscal Year 2069/70.
Compute Assessable Income, Taxable Income, Tax liability of Mr. Sailesh for
the Fiscal Year 2069/70 stating relevant provisions of Income Tax Act, 2058. 10
Note: Applicable tax rate for the purpose of computation to be considered is as
per Finance Act, 2070.
c) Mr. Shyam, a sole shareholder of M/s Ganpati Industries Pvt. Ltd., was worried
about the performance of the Company as it incurred losses to the tune of Rs. 5
crores during last 4 financial years ending the F/Y 2068/69. Mr. Mohan, an
expert acquired 60% stake in the Company on Ashadh 31, 2069. Miracally, the
company has managed to earn Rs. 1.5 crore as profit in F/Y 2069/70. The
Company has submitted the Income Tax Return by assessing a taxable loss of
Rs. 3.5 crore for F/Y 2069/70 under self assessment by adjusting the carry

XTQ
(82)

forward losses of Rs. 5 crore up to F/Y 2068/69 u/s 20 of Income Tax Act, 2058.
The Chief Tax Officer issued an order to pay income tax on Rs. 1.5 crore along
with interest thereon. The management of the Company seeks your advice on the
said order of Inland Revenue Office. 4
Answer a) The provisions have been mentioned in 1(1) of schedule 1 of income tax act to determine the tax
liability for the income year 2069/70 of sole proprietorship firm selected single. Further, section
11(3e) of the act has provisioned that if a manufacturing industry exports goods that it has
manufactured, tax rebate shall be granted by Twenty Five percent on the rate of tax leviable on
income earned therefrom. As per the Provisions, applibale tax rate are:
Calculation of applicable tax rate for local sales and export sales
Amount Tax rate
UptoRs. 1,60,000 0
Upto 2,60,000 15 %
Above Rs. 2,60,000 25 % Normal rate
Above Rs. 25,00,000 40 % additional on normal rate
Local sales
For manufacturing industry 20 % instead of 25 % (As per Sec 1(14) of
Schedule I)
Above Rs. 25,00,000 20 % + 40% of 20%= 28%

Export
Applicable rate 15 % instead of 25 % (As per Sec 1(1) of
Schedule I)
For manufacturing industry 25 % less than applicable rate
= 11.25 %
Above Rs. 25,00,000 11.25 % + 40% of 11.25%= 15.75 %
Calculation of tax liability
Income 7(2) Rs. 1,50,00,000 ( Rs. 1,12,50,000+37,50,000)
Less deduction
Cost of sales Rs. 90,00,000
Indirect cost Rs. 5,00,000
Total allowable expenses Rs. 95,00,000
Total taxable income Rs. 55,00,000
Local taxable income and taxable income from export are as follows. The taxable income has
been divided proportionately based on sales.

Note: Credits and insurance claims are not considered due to lack of the information provided.

Local sales

4,125,000.00 160,000.00 0% -
100,000.00 15% 15,000.00
2,240,000.00 20% 448,000.00
1,625,000.00 28% 455,000.00
Export Sales
1,375,000.00 All Exceeding 25 lakhs
1,375,000.00 15.75% 216,562.50
- -
Total Tax 1,134,562.50

b) Computation of assessable Income, Taxable Income and Tax Liability of Mr. Sailesh for
the fiscal year 2069/070
XTQ
(83)

Particulars Amount (Rs) Working


Reference
Salary (50,000*12) 600,000
Dashain Allowance 20,000
Bonus 40,000
Contribution to Provident Fund (PF) 24,000
Pension 150,000
Assessable income from employment 834,000
Deductible Amount
Deductible amount against contribution to PF (48,000)
Actual Contribution
Employer's and Employee's Contribution 48,000
1/3 rd of Assessable income 278,000
Maximum Limit 300,000 Note
1
Adjusted Taxable Income 786,000
Deduction for Remote Allowance under category C (30,000) Note
2
Deduction under pension (62,500) Note
3
Deduction for Disability (125,000) Note
4
Taxable Income 568,500
Deduction against Insurance (20,000)
Premium paid 25,000
Maximum Limit 20,000 Note
5
Taxable Income (For Tax Calculation) 548,500
Computation of Tax Liability
Up to NPR 250,000 (For Couple) 2,500
NPR 100,000 @ 15% 15,000
Balance @ 25% 49,625
Total 67,125 Note
6
Total Tax Liability 67,125

Working Notes
Note 1
As per rule 21 of Income Tax Rules, 2059 an individual, who has made a contribution to an
approved retirement fund during a Fiscal Year, is entitled to get exemption of the amount
deposited to the fund subject to a maximum limit of 1/3rd of the Assessable Income of him/her
during the year or Rs. 300,000 whichever is lower.
Note 2
As per section 1(5) of Annexure 1 of Income Tax Act, 2058, in case the income is generated
from an activity at a remote area of Nepal an additional exemption limit has been prescribed for
a resident natural person. The amount prescribed under category ―C‖ (Dailekh district falls
under category ―C‖) is Rs. 30,000.
Note 3
As per section 1(9) of Annexure 1 of Income Tax Act, 2058, any natural person who has also an
income from pension, the tax exemption limit as prescribed under subsection (1) (a) or (1)(b),
whichever is applicable, shall increase by 25% of the basic limiti.eRs.62,500 (Rs.

XTQ
(84)

2,50,000*25%). But as per rule 39 of Income Tax Rules, 2059, the additional exemption given
to the pension holders should not exceed the actual amount of pension s/he is receiving.

Note 4
As per section 1(10) of Annexure 1 of Income Tax Act, 2058, any natural person who is
disabled individual or couple, the tax exemption limit as prescribed under subsection (1) (a) or
(1)(b), whichever is applicable, shall increase by 50% of the basic limitRs.1,25,000 (Rs.
2,50,000*50%).
Note 5
As per section 1(12) of Annexure 1 of Income Tax Act, 2058, in case a resident natural person
has taken life insurance on his life, Rs 20,000 or the actual premium paid during the year,
whichever is lower, is available for deduction from the taxable income of the individual.
Note 6
For an individual natural person, who has opted for couple under section 50, the tax applicable
shall be as prescribed under section 1(2) of annexure 1 of Income Tax Act, 2058 and the
minimum exemption shall be Rs. 250,000.

c) As per section 57 (1) & (2) of Income Tax Act, 2058, if the ownership of any entity changes by
50% or more during the last three financial years, the Company is not allowed to carry forward
its accumulated losses of the period prior to such transfer of ownership. In this case, the
ownership of M/s Ganpati Industries Pvt. Ltd. was change by 60% as the shares of the Company
was sold by old management to the new management, therefore, the Company can not adjust
any accumulated losses for the period untillAshad 31, 2069. Thus, the assessment order issued
by the Inland Revenue Office is correct & the Company has to pay tax on the profit of the
Company earned during the F/Y 2069/70.

10.
a) Worthy Jute Industries Limited, a 100% export oriented entity, is engaged in the
manufacturing of jute products. There are 1501 Nepali citizens working in the
company throughout the year. The accountant of the company computed the
taxable income amounting to Rs. 12,550,000 for F/Y 2070/71 and income tax of
Rs. 2,008,000. Being a tax auditor, whether you agree with the tax amount
calculated by the accountant of the Company? Will your answer be different if
there are 1,499 Nepali citizens working throughout the year instead of 1,501? 5
b) The Inland Revenue Department has issued notice to Mr. Rohit Shrestha, a
renowned business tycoon of Nepal; to pay the due tax liability amounting to Rs.
25,566,725 within Baisakh 25, 2071 and the same notice was also published in
the National Level Newspaper. The notice also contains that if Mr. Shrestha does
not pay the tax dues on or before the specified time limit then appropriate action
shall be taken against him. Mr. Shrestha aggressively trying to arrange the
money but he could not be able to pay the due tax on time. In the mean time, Mr.
Shrestha had a prescheduled visit to New Delhi on Baisakh 26, 2071 to attend a
marriage ceremony of one of his friend's daughter and accordingly he went to
Tribhuvan International Airport (TIA) on the schedule time to fly to New Delhi.
At the TIA, the immigration staff stopped him to enter to the aircraft stating that
the Inland Revenue Department has given notice to stop Mr. Rohit Shrestha to
going abroad. In the light of the above fact, please state whether the immigration
staff at TIA has got the right to do so? If not, what remedy is available to Mr.
Shrestha for the unauthorized act of the immigration staff as per the provisions of
Income Tax Act, 2058. 5
c) Singha road Ltd. located at Hetauda is a public infrastructure project. The project
will be completed on Chaitra end 2075 as per estimation, and then it will be

XTQ
(85)

handover to Government of Nepal. The Ltd. has incurred loss continuously from
income year 2061/62 to 2068/69 as follows:
Income Year Loss amount Rs.
2061/62 1.000,000
2062/63 500,000
2063/64 600,000
2064/65 200,000
2065/66 150,000
2066/67 100,000
2067/68 75,000
2068/69 10,000
During income year 2069/70, it has incurred the profit of Rs. 1,500,000 Compute
the taxable income for the income year 2069/70 and carry forward losses for the
income year 2070/71 for set off with references to the provisions of Income Tax
Act, 2058 relevant to this project. 5
d) Fewa Pvt. Ltd. has been established by Nepalese promoters under foreign direct
investment.The eighty percent of the total capital has been hold by non-resident
persons. In addition, The Pvt. Ltd.has borrowed the amount from the foreign
investors. For the year ended Ashadh end, 2070, the profit and loss account of
the company has the following transactions. 5
Amount in Rs.
Sales 5,000,000
Interest income 75,000
Total income 5,075,000

Expenses
Cost of sales 2,000,000
Administrative expenses 1,100,000
Interest expenses 1,500,000
Pollution control expense 300,000

Total Expenses 4,900,000


Net profit 175,000
i) Interest expense Rs. 1,400,000 out of total has been charged on the borrowed
amount from foreign investors. Answer whether all the above expenses can
be claimed under Income Tax Act, 2058?
ii) If these expenses cannot be claimed during the year, what would be the
implication?
Answer a) With reference to the section 11 (3)(ka), if any special industry creates employment for 1200 or
more Nepali citizen throughout the year, than only 80% of normal income tax shall be levied.
Further, under the same section, 25% rebate on applicable tax rate is available for the 11 (3 Nga)
manufacturing industries having profit on export sales. In this case, Worthy Jute Industries
Limited is special industries, so, applicable tax rate is 20% & the income tax of the company
would be as follows:
Particulars Option 1 Option 2
Normal Tax Rate for the company( As per Sec. 2(3a)
of Schedule 1) % 20% 20%
Rebate for export 25% of 20% % 5% -
Applicable Tax Rate % 15% 20%
XTQ
(86)

Taxable income for the year Rs. 12,550,000 12,550,000


Applicable Income Tax Liability Rs. 1,882,500 2,510,000
Concession @ 20% for more than 1200 employment
creation Rs. 502,000
Net Tax Liability for the year Rs. 2,008,000

taxpayer has two option under the provisions of Income Tax Act. As calculated above, as per
option 1, the tax liability would be 1,882,500 and as per option 2, the tax liability shall be Rs.
2,008,000.
A rational tax payer shall choose the lower amount. However as the Act has provided option to
tax payer, it can choose to pay 2,008,000 also. Hence, the amount calculated by the accountant
is not incorrect although it is not desirable.

Answer to the second part of the question:


No, there will not be any change in the tax liability whether there are 1499 Nepali
citizens working throughout the year.
b) As per Section 106 of Income Tax Act, 2058, where a person fails to pay tax on or before
the date the tax is payable, the Department may, by notice in writing served on the
concerned office of Nepal Government, order the office to prevent the person from
leaving Nepal for a period of 72 hours after the expiry of time limit specified in a notice
served for the purpose of paying tax.
In the given case, as Mr. Shrestha was unable to pay the due tax on time, the IRD has
power to issue a notice to the Immigration of Nepal to prevent him from going Abroad
for up to 72 hours. As per the prescheduled plan of Mr. Shrestha for going New Delhi on
Baisakh 26, 2017, the immigration staff has right to stop him from going abroad.
c) Section 20 (1) of income tax act, 2058 has the following provisions for carry forward the losses:
For purposes of computing the income earned by any person from any business or
investment in any income year, such person may deduct the loss as mentioned
below:-
(a) Loss suffered by that person from any other business and not deducted in that
year, and
(b) Loss suffered by that person from any business which was not deducted in the
last seven income years.
Provided that, in the case of the projects which involve building and
operation of public infrastructures to be transferred to the Government of Nepal,
powerhouses construction, generation and transmission of electricity and
petroleum works pursuant to the Nepal Petroleum Act, 2040 any loss not
deducted in the last twelve income years.
As per the provisions, the project has not fulfilled the conditions of BOT. It will be
handover after the completion. So, it can carry forward only 7 years losses.
Calculation of taxable income for the year 2069/70.
Net income during the year Rs. 15,00,000
Less: carry forward losses Rs. 16,35,000
2062/63 5,00,000
2063/64 6,00,000
2064/65 2,00,000
2065/66 1,50,000
2066/67 1,00,000
2067/68 75,000
2068/69 10,000
XTQ
(87)

16,35,000
Taxable loss for the year 2069/70 (1,35,000)
Carry forward losses for the year 2070/71
The taxable loss for the income year 2069/70 is Rs. 1,35,000. The loss of 2063/64 to
2065/66 is 14,50,000 which is fully setoff in FY 2069/70. The loss of Rs. 50,000 of
2066/67 and the loss of 2067/68 and 2068/69 is carried forward to 2070/71, total of Rs.
135,000.

d)

i) All the amount of cost of sales under section 15 and adminitrative expenses under section 13
can be claimed for deductions while computing the taxable income. There is limitations on
interest expenses and pollution control expenses.
Calculation of allowable interest expenses
Net Profit: Rs. 1,75,000
Add carry forward interest expenses Rs. 0
Add interest expenses Rs. 15,00,000
Less interest Income (Rs.75,000)
Adjusted taxable income: Rs.16,00,000
Interest Income Rs. 75,000
50 % of Adjusted taxable income Rs. 800,000
Deductible interest expenses Rs. 8,75,000
Interest expenses under section 14 can be claimed only Rs. 9,75,000 out of total Rs. 15,00,000-
Rs. 8,75,000under 14 (2) and Rs. 1,00,000 under 14(1)

Further, pollution control expenses under section can be deducted fifty percent of
adjusted taxable income.
Calculation of deductible pollution control expenses:
Net Profit: Rs. 1,75,000
Add expenses during the year: Rs. 3,00,000
Adjusted taxable income: Rs.4,75,000
50 %of Adjusted taxable income: Rs. 2,37,500
So, all the expenses of pollution control cannot be allowable for deduction. Pollution control
expenses Rs. 2,37,500 out of Rs. 3,00,000 can be claimed for deductions.

ii)
As per section 14 (3) of the act any interest not allowed to be deducted or not deducted
pursuant to Sub-section (2) may be carried forward or credited in the forthcoming
income year. So, the balance interest of Rs. 5,25,000(Rs. 14,00,000-Rs.8,75,000) will be
carried forward in the coming year and computed interest expenses as per above.
The balance amount Rs. 62,500 of pollution control expenses will be capitalized in block
D of the assets and depreciation will be claimed from the next income year.
11. Write short notes of the following with reference to Income Tax Act, 2058. (5×2=10)
a) Company
b) Lease
c) Investment
d) Interest
e) Receiver
Answer a) Company
XTQ
(88)

As per section 2(m), "Company" means any company incorporated under the companies'
law in force, and for the purpose of tax the following institutions shall also be treated as
if they were companies:
(1) Any corporate body established under the law in force,
(2) Any unincorporated union, board, association or society or sole proprietorship
whether incorporated or not and any group of persons or trust except a partnership,
(3) Any partnership firm consisting of Twenty or more partners whether registered or
not under the law in force, any retirement fund, cooperative institution, unit trust, joint
venture,
(4) Any foreign company;
(5) Any other foreign institution as specified by the Director General.
b) Lease
As per section 2(ab), "Lease" means the provisional right of any person to enjoy or use
any property except movable property belonging to another person, and this term also
includes a license, rent agreement, trenches, royalty agreement or right of a lessee/
tenant.
c) Investment
As per section 2(al),"Investment" means the act of holding one or more properties or
investing such properties, except with the followings:
(1) Holding any property used by the owner thereof in personal use, or
(2) Employment or occupation.
Provided that holding non-business taxable property shall be deemed as investment.
d) Interest
As per section 2(as), "Interest" means the following payment or profit:
(1) Payment under debt liability except the principal,
(2) Profit made from concession, exemption, premium under loan liability, alteration
payment or from similar payment, and
(3) The amounts referred to in Section 32 receivable as an interest out of
the payment to be made by a person who acquires any property under
annuities or installment sale or of the payment made to any person for the
use of any property under a financial lease.
e) Receiver:
As per Section 108 of Income Tax Act, 2058 ―Receiver‖ means any of the following persons:
(1) a liquidator;
(2) a receiver appointed out of court or by a court in respect of an asset or entity;
(3) a mortgagee in possession;
(4) an executor, administrator, or direct heir of a deceased individual's estate; or
(5) any person conducting the affairs of an incapacitated individual.
12.
a) JivanJyoti Gases Industries Pvt. Ltd. is engaged in manufacturing of Oxygen &
Nitrogen Gases. The transactions of the industry for the month of Kartik, 2070 is
as follows:

Sales Details (Excluding VAT)


Nitrogen
Oxygen
Sold to Date Gases
Gases(inRs.)
(inRs.)
Teaching Hospital 2070.07.02 50,000 -

XTQ
(89)

Ram Metal Works 2070.07.05 40,000 25,000


B & B Hospital 2070.07.10 55,000 -
Ghorahi Cement Industries 2070.07.14 125,000 250,000
Sine Hydro Corporation 2070.07.25 150,000 30,000
Total Sales for the month 420,000 305,000

Purchase Details(Excluding VAT)


Amount
Particulars
( inRs.)
Diesel purchase for Generator
Running 90,000
Telephone Chagres 18,000
Purchase of Vehicle for General
Manager 2,250,000

The Industries also paid the consultancy fee amounting to IRS 50,000 to Sigma
Consultancy, New Delhi, India.
VAT receivable as per Ashwin, 2070 VAT Return is NRS 28,000.
You are requested to compute the VAT collected on Sales, VAT Paid on
Purchase and Net VAT payable/receivable for the Kartik end 2070. 10
b) National Mobile Hub without being registered under VAT Act is dealing in retail
of various branded mobile sets at NLIC, Kamalpokhari. During the F/Y 2069-70,
the firm returned its file to the IRO on 2070.06.25 with the following Income
Statement:
Income Statement for F/Y 2069/70
Expenses Amount (Rs.) Income Amount (Rs.)
Opening Stock 175,870 Sales 1,787,654
Purchases 1,576,543 Closing Stock 545,387
Gross Profit C/D 580,628
2,333,041 2,333,041

Gross Profit
Salary 91,000 B/D 580,628
Showroom Rent 108,000
Telephone Charges 18,000
Printing & Stationery 5,075
Local Conveyance 2,400
Miscellaneous Expenses 1,556
Net Profit Before Tax 354,597
580,628 580,628
The Tax Officer made an assessment on 2070.08.15 demanding a sum of Rs.
445,778 as VAT along with interest and penalty and also ordered to register the
firm under VAT. The proprietor of National Mobile Hub was surprised by the
assessment order and rushed to you. He explained you that though his firm not
able to sale the goods above the threshold limit fixed under VAT Act, 2052, the
IRD assessed the VAT. He feels that the VAT should not be applicable on him.
Is the contention of the proprietor of the firm is correct? 5

XTQ
(90)

c) MNP Pvt. Ltd. is a company voluntarily registered to VAT on 1stShrawan, 2067.


Details of his transactions are as follows:
Fiscal Year (FY) 2068/69 Amount (Rs.)
Shrawan- Kartik 550,000
Mangsir- Falgun 500,000
Chaitra- Ashadh 930,000
1,980,000
Fiscal Year 2069/70
Shrawan- Kartik 400,000
Mangsir- Falgun 680,000
Chaitra- Ashadh 590,000
1,670,000
The companyis submitting VAT return on four monthly basis, since heitis
voluntarily registered and the turnover of any F/Y doesn't exceed Rs. 20 lakhs,
its claiming that it need not have to submit its VAT return on monthly basis.
Comment on the company's contention with reference to Value Added Tax
Act/Rules. 5
Answer a)
As per Schedule 1 relating to section 5 (3) of Value Added Tax Act, 2052, Oxygen Gas
used for the Medical Treatment is exempt from VAT. In this case, Oxygen gas sold to
Hospitals will be non taxable.
As per Section 41 of Value Added Tax Rules, 2053 only 40% of VAT Paid on purchase
of Automobiles eligible for claim as input VAT.
As per section 8(2) of Value Added Tax Act, any resident or non-resident person imports
services from unregistered person residing outside Nepal, shall have to collect VAT on
the taxable value of the services. This is known as Reverse Charge system of VAT.
Therefore, in this case, consultancy fee paid to Sigma Consultancy, New Delhi, India
amounting to IRS 50,000 is Vatable& the person paying the same should collect VAT.
Based on the above background information, the computation of VAT is done as under:
i) Vat Collected on Sales
Sold to Taxable Non Taxable
Teaching Hospital - 50,000
Ram Metal Works 65,000 -
B & B Hospital - 55,000
Ghorahi Cement Industries 375,000 -
Sine Hydro Coorporation 180,000 -
Base Amount 620,000 105,000
VAT @ 13% 80,600 -

ii) VAT paid on purchase


Purchase
VAT
Particulars Amount
Diesel purchase for Generator
Running 90,000 11,700
Telephone Chagres 18,000 2,340
Purchase of Vehicle for General (Only 40% is eligible for
Manager 2,250,000 292,500 input VAT)
Consultancy Fee (IRS 50,000) 80,000 10,400
Total VAT Paid on Purchase 3,16,940
Computation of Net VAT Payable/Receivable for the Kartik, 2070

XTQ
(91)

As the company has transaction both taxable & Non-taxable. Therefore, the total VAT paid on
purchase can be claimed only to the extent of proportionate basis of sales of taxable items.
a) Sales Proportionate
Sales
Particulars Amount Proportionate
Taxable Sales 620,000 85.52%
Non Taxable Sales 105,000 14.48%
Total 725,000 100.00%

b) Input VAT
Particulars Amount
Total Vat Paid on Purchase 3,16,940
Less: 60% of VAT on Purchase of Vehicle 1,75,,500
Balance 1,41,440
Less: Non Taxable Portion (14.48%) 20,480
Net Amount for Input VAT 1,20,959

c) Net VAT Payable/Receivable:


Particulars Amount
VAT Collected on Sales 80,600
Eligible Input VAT 1,20,959
Net Receivable for the month 40,359
Opening Receivable 28,000
Net Vat Receivable for the Kartik, 2070 68,359

b) Under Rule 7 (5) of Value Added Tax Rules, 2053, there are certain proxy criteria for
determining the threshhold limit of any taxpayer. Under that provision, if any taxpayer paying
rent & telephone bill above Rs. 100,000in a year then he is mandatorily liable to VAT
registration. In the given case of National Mobile Hub, though the firm is not transaction of
purchases& sales in excess of Rs. 20,00,000, the threshold limit fixed by the act, is still liable to
compulsorily VAT registration as the total rent & telephone charges is above Rs. 100,000 in
F/Y 2069-70. Therefore, the contention of the proprietor is not correct.
Further, the company is trading electronic goods, it should compulsory registered on VAT.
c) As per Value Added Rule, No. 26, if the turnover with last 12 months is within 20 lakhs to
1 crore, VAT return should be filed on every two months on Shrawan, Ashwin, MArg,
Magh, Chaitra and Jesth”
As per Rule 26(3) of the Value Added Tax Rules 2053, taxpayer registered voluntarily as per
section 9 of the act and having taxable transaction of not more than Rs. 2 million during
previous twelve months, shall submit tax return to concerned tax officer within 25th of
Shrawan, Mangsir and Chaitra.
Further as per Rule 7(1), in case any person has reason to presume that his annual transaction
shall exceed Rs. 2 million, he shall submit an application setting out such conditions, to the
concerned tax officer in the format set out in schedule 1 for registration of the transaction.
The turnover of company does not exceed Rs.20 lacs in any fiscal year, but to calculate the limit
of Rs.20 lacs, turnover of past 12 months shall be taken into consideration.

Turnover of company of the past 12 months for the period ended on Falgun 2069, exceeds
Rs.20 lacs as below:
Particulars Amount(Rs.)
Chaitra – Ashad 2069 930,000
XTQ
(92)

Shrawan-Kartik 2069 400,000


Mangsir-Falgun 2069 680,000
2,010,000
Hence, M/s MNP Pvt. Ltd. has to submit monthly VAT return.

13.
a) State with reasons whether the following statements are true or false with
reference to Value Added Tax Act/Rules. (5×2=10)
i) 'No VAT' and 'Zero VAT' have the same meaning as VAT in both the cases
is zero.
ii) In the case where there is provision of a contract for paying partly the value
of goods or services in more than one day on an installment basis, the time of
supply shall be the date of payment.
iii) Roy & Co. is a VAT registered firm engaged in the business of importing
passengers' car and selling them in local market. The firm is claiming full
input tax credit on purchase of such cars.
iv) If a taxpayer fails to submit tax return as per section 18, the penalty imposed
is Rs. 10,000 per month.
v) A separate record for purchase and sale shall be maintained for the used
goods which have purchase price more than Rs. 20,000.
b) What are conditions for compulsory registration under the Value Added Tax Act,
2052 and Rules 2053? 10
Answer a)
i. The statement is false. 'No VAT' and 'Zero VAT' have different meaning.
Transactions of those goods and services which are included in schedule 1 of the VAT Act,
2052 are exempted from tax. This is called no VAT items.
Where a supply of a goods or services is exempt from VAT, the input tax credit is not
allowable.
Out of the goods or services, which are subject to VAT and are transacted as per schedule 2 of
the Act, the rate of VAT shall be charged by zero percentage.
Where a supply of a goods or services is at zero rate, the input tax credit is allowable.
ii.
The statement is false.
As per section 6(3)(kha) of Value Added Tax Act, 2052, in the case where there is provision of
a contract for paying partly the value of goods or services in more than one day on an
installment basis, the time of supply shall be
- at the time of the payment or
- the day on which the payment is to be made according to the terms of contract
whichever occurs earlier.
iii.
The statement is true.
As per rule 41(3) of Value Added Tax Rules, 2053, for a registered person, who carries on a
business of those goods mentioned in sub- rule (1) and (2) of Rule 41 as the principal business,
there shall be no restriction for the tax credit in accordance with the procedures mentioned in
these rules.
Hence, Roy & Co. is allowed to claim full input tax credit on purchase of cars as their principal
business is to import passengers car and sell into local market.
iv.
The statement is false.

XTQ
(93)

As per section 29(1)(ja) of Value Added Tax Act, if a taxpayer fails to submit tax return as per
section 18, 0.05% per day of tax payable or Rs. 1,000 per tax period whichever is higher.
v.
The statement is false.
As per Rule 33(3) of Value Added Tax Rules, 2053, where the purchase price of each item of
used goods exceeds Rs. 10,000, separate records of purchase or sale shall have to be maintained.

b) Section 9 and 10 of the VAT act, 2052 and rules 6 and 7 of the VAT rules state the provisions
for compulsory registration for any person. As per the provisions, any person should register
compulsorily in VAT in the following conditions:
i) The person having more than 2 million taxable transactions of goods during the previous
twelve months as per section 9 and rules 6(1)
ii) The person having more than one million taxable transactions of service or combine taxable
transaction of goods and services during the previous twelve months as per section 9 and
rules 6(1),
iii) The person who wishing to engage in taxable transaction under section 10 (1)
iv) The person's annual transaction, as per estimation may exceed the threshold of Rs. 2
millions as per rules 7(1)
v) The person's transaction exceed the threshold of Rs. 2 millions as per rules 7(2)
vi) The person who has conducted their business in a metropolis, sub-metropolis or a
municipality, and is dealing in prescribed goods and services as per section 10 (2). The
prescribed goods and services are hardware, sanitary, furniture, fixture, furnishing,
automobiles, motor parts, electronics, marble, educational consultancy, discotheque,
health club, catering service, party place business, parking service, dry cleaners using
machinery equipment,restaurant with bar, and colour lab.
vii) At the time of inspection by the tax officer, in case the stock of taxable goods available with
the person exceeds the limit, based on the nature of the goods, prescribed by the department
as per rules 7(5)(a)
viii) In case a person's telephone expenses and house rent exceed Rs. 1 lakh during a year as
per rules 7(5)(b) or
ix) In case the business place is in the area specified by the department for different roads or
different market areas.

XTQ

You might also like